COMPILATION OF ALL 72 SETS OF BIOLOGY SSC CHSL-2016 · OF BIOLOGY SSC CHSL-2016 PREPARED BY : SSC...

54
FOR MORE UPDATES & MATERIALS DO LIKE OUR FACEBOOK PAGE :http://www.facebook.com/sscmentorsofficial COMPILATION OF ALL 72 SETS OF BIOLOGY SSC CHSL-2016 PREPARED BY : SSC MENTORS BIOLOGY SPECIAL

Transcript of COMPILATION OF ALL 72 SETS OF BIOLOGY SSC CHSL-2016 · OF BIOLOGY SSC CHSL-2016 PREPARED BY : SSC...

Page 1: COMPILATION OF ALL 72 SETS OF BIOLOGY SSC CHSL-2016 · OF BIOLOGY SSC CHSL-2016 PREPARED BY : SSC MENTORS BIOLOGY SPECIAL . F A C E B O O K P A G E : h t t p : / / w w w . f a c e

FOR MORE UPDATES amp MATERIALS DO LIKE OUR FACEBOOK PAGE httpwwwfacebookcomsscmentorsofficial

COMPILATION

OF ALL 72 SETS

OF BIOLOGY SSC

CHSL-2016

PREPARED BY SSC MENTORS

BIOLOGY SPECIAL

F A C E B O O K

P A G E h t t p w w w f a c e b o o k c o m s s c m e n t o r s o f f i c i a l P a g e | 1

FOR MORE UPDATES AND MORE MATERIAL DO LIKE OUR FACEBOOK PAGE httpwwwfacebookcomsscmentorsofficial

For more materials you can

visit our store

httpswwwinstamojocomS

SCMentors

Facebook group_for SSC

CPO amp CGL 2018

httpswwwfacebookcompg

sscmentorsofficialgroupsref

=page_internal

official telegram channel

httpstmesscmentorsofficial

Facebook page

httpwwwfacebookcomssc

mentorsofficial

F A C E B O O K

P A G E h t t p w w w f a c e b o o k c o m s s c m e n t o r s o f f i c i a l P a g e | 2

FOR MORE UPDATES AND MORE MATERIAL DO LIKE OUR FACEBOOK PAGE httpwwwfacebookcomsscmentorsofficial

There were 231 questions

asked in SSC CHSL Science

(biology) 2016 Question Paper

Important questions are

explained here also Q1 Diabetes is caused by

मधमहॳह हह८न का कारणहहॴ

7-Jan -2017

Options

1) Excess of insulin

इनसलिनकीबहिता 2) Low production of Insulin

इनसलिनकाकमउतपादन

3) Malfunction of liver

यकतकहॳ काययमगड़बड़ीहह८ना 4) Higher production of bilirubin

बबि बबनकाउचचतरउतपादन

Correct Answer Low production of

Insulin

Diabetes is caused by the immune

system destroying the cells in the

pancreas that make insulin This causes

diabetes by leaving the body without

enough insulin to function normally

Q2 Tectona grandis Linn is the

scientific name of

टहॳकटह८नागहॴननिसलिन mdashmdashndash कावहॴजञाननकनामहहॴ 7-Jan -2017

Options

1)Guavav

अम द

2)Teak

सागह९न

3) Amla

आविा 4) Chiku

चचक

Correct Answer Teak

Q3 Sea-Anemones belongs to the

phylum

सी-एननमोस mdashmdash- परजानतकहॳ अतगयतआतहॳहहॴ 7-Jan -2017 Options

1)Arthropoda

अनरह८पह८ड़ा 2)Cnidaria

ननड़हॳररया 3) Porifera

पह८ररफहॳ रा 4) Mollusca

मह८िसक

Correct Answer Cnidaria

Sea anemones are a group of marine

predatory animals of the order

Actiniaria They are named after the

anemone a terrestrial flowering plant

because of the colourful appearance of

many Sea anemones are classified in the

phylum Cnidaria

Q4 Which of the following is also

known as a Common Water Hyacinth

नननननलिखितमसहॳककसहॳआमजिकनभभीकहाजाताहहॴ

7-Jan -2017

Options

1)Pistia

पपनसटया 2)Opuntia

ओपलिया 3)Aegilops

एनजिह८पस

4) Echhornia

एकह८ननयया Correct Answer Echhornia

Water hyacinth is a free-floating

perennial aquatic plant (or hydrophyte)

native to tropical and sub-tropical South

America

F A C E B O O K

P A G E h t t p w w w f a c e b o o k c o m s s c m e n t o r s o f f i c i a l P a g e | 3

FOR MORE UPDATES AND MORE MATERIAL DO LIKE OUR FACEBOOK PAGE httpwwwfacebookcomsscmentorsofficial

Q5 Which is the largest organ in

human beings

मानवह८मसबसहॳबड़ाअगकह९नसाहह८ताहहॴ

7-Jan -2017

Options

1) Skin

तवचा 2) Large Intestine

बड़ीआत

3) Small Intestine

छह८टीआत

4) Liver

यकत

Correct Answer Skin

The skin is the largest organ of the

body with a total area of about 20

square feetThe largest internal organ is

the liver The longest bone in the human

body is the femur The largest artery is

the aorta and the largest vein is the

inferior vena cava

Q6 Delonix regia Rafin is the scientific

name of

िहॴिोननकसरनजयारकफन (Delonix regia

Rafin)mdashmdash- कावहॴजञाननकनाम 7-Jan -2017

Options

1) Banyan

बरगद

2) Gulmohar

गिमह८हर

3) Tamarind

इमिी 4) Chiku

चचक

Correct Answer Gulmohar

Delonix regia is a species of flowering

plant in the bean family Fabaceae

Q7 Amoeba belongs to the phylum

अमीबा mdashmdashmdash- परजानतकहॳ अतगयतआताहहॴ

7-Jan -2017

Options

1) Protozoa

परह८टह८जआ

2) Annelida

ऐनहॳलििा 3)Porifera

पह८ररफहॳ रा 4) Platyhelminthes

पिहॳटटहहॳनममननिस

Correct Answer Protozoa

Q8 Deficiency of which of the following

causes non-clotting of blood

नननननलिखितमसहॳककसकीकमीकहॳ कारणरकतकािककानहीजमता

8-Jan -2017

Options

1) Vitamin C

पवटालमन C

2) Vitamin K

पवटालमन K

3) Vitamin E

पवटालमन E

4) Vitamin B12

पवटालमन B12

Correct Answer Vitamin K

Vitamin C is found in citrus fruits and

vegetables Scurvy results from a

deficiency of vitamin C in the diet

Vitamin E deficiency associated with

this disease causes problems such as

poor transmission of nerve impulses

muscle weakness and degeneration of

the retina that can cause blindness

Vitamin B12 deficiency may lead to a

reduction in healthy red blood cells

Q9 The process of producing energy in

plants is known as

F A C E B O O K

P A G E h t t p w w w f a c e b o o k c o m s s c m e n t o r s o f f i c i a l P a g e | 4

FOR MORE UPDATES AND MORE MATERIAL DO LIKE OUR FACEBOOK PAGE httpwwwfacebookcomsscmentorsofficial

पहॳड़ह८मउजायउतपननकरनहॳकीपरनतकियाकह८ mdashmdashndash

कहॳ नामजाताहहॴ 8-Jan -2017

Options

1) Absorption

अविह८षण

2) Reduction

अवकरण

3) Photosynthesis

परकािसशरिहॳषण

4) Transpiration

वाषपीकरण

Correct Answer Photosynthesis

Q10 Which Virus causes Chicken Pox

ककसवायरसकहॳ कारणचहॳचकहह८ताहहॴ

8-Jan -2017

Options

1) Rubella Virus

बहॳिावायरस

2) Herpes Zoster Virus

हपपरयसजह८सटरवायरस

3) Rabies

रहॳबीज़

4) Variola Virus

वहॳरीओिावायरस

Correct Answer Herpes Zoster Virus

Rubella also known as German

measles or

three ndashday measles is an infection

caused by the rubella virus

Smallpox is caused by infection with

variola

Virus

Q11 What is the total number of bones

in the human body

मानविरीरमकिककतनीहडडियहह८तीहहॴ

8-Jan -2017

Options

1)206

2)103

3)309

4)412

Correct Answer 206

Q12 Emblica officinalis is the scientific

name of

एननलिकाओफीलसनहॳलिस (Emblica

offcinalis)mdashmdash- कावहॴजञाननकनामहहॴ 8-Jan -2017

Options

1) Peepal

पीपि

2) Mango

आम

3) Amla

आविा 4) Drumstick

सहजन

Correct Answer Amla

Q13 Sponges belongs to the phylum

सपजककसपरजानतकहॳ अतगयतआतहहॴ

8-Jan -2017

Options

1) Protozoa

परह८टह८जआ

2) Annelida

एननलििा 3) Porifera

पह८ररफहॳ रा 4) Cnidaria

ननिहॳररया Correct Answer Porifera

Q14 Which of the following is a

symptom of haemophilia

नननननलिखितमसहॳटहमह८फीलियाकािकषणकह९नसा हहॴ

9-Jan -2017

Options

F A C E B O O K

P A G E h t t p w w w f a c e b o o k c o m s s c m e n t o r s o f f i c i a l P a g e | 5

FOR MORE UPDATES AND MORE MATERIAL DO LIKE OUR FACEBOOK PAGE httpwwwfacebookcomsscmentorsofficial

1) Night Blindness

रतोधी 2) No clotting of Blood

रकतकािककानजमना 3) Rickets

ररकहॳ ट

4) Loss of haemoglobin

टहमह८गिह८बबनकीअमपता Correct Answer No clotting of Blood

Haemophilia also spelled hemophilia is

a mostly inherited genetic disorder that

impairs the bodyrsquos ability to make blood

clots a process needed to stop bleeding

This results in people bleeding longer

after an injury easy bruising and an

increased risk of bleeding inside joints

or the brain

Q15 The process of pollination by birds

is also known as

पकषकषयोदवाराकीजानहॳवािीपरागणकीपरनतकियाकह८mdashndash कहॳ नामसहॳभीजानाजाताहहॴ 9-Jan -2017

Options

1) Hydrophily

हाइडरह८कफिी 2) Entomophily

एनटोमह८कफिी 3) Embryophily

एननियह८कफिी 4) Ornithophily

ओननयिह८कफिी Correct Answer Ornithophily

Q16 Spiders belong to the phylum

मकडड़याककसपरजानतकहॳ अतगयतआतीहहॴ 9-Jan -2017

Options

1) Mollusca

मह८िसका 2) Annelida

एननलििा 3) Cnidaria

ननिहॳररया 4) Arthropoda

अरोपह८ड़ा Correct Answer Arthropoda

Q17 Banana freckle is a plant disease

It is caused by a

कहॳ िहॳकीझाईपह८धह८कीएकबीमारीहहॴ यहएक mdashndash

कहॳ कारणहह८तीहहॴ 9-Jan -2017

Options

1) Virus

वायरस

2) Fungus

कवक

3) Bacteria

बहॴकटीररया 4) Insect

कीटक

Correct Answer Fungus

Banana Freckle is a disease caused by

the fungus Guignardia musae

(telomorph) or Phyllosticta musarum (

anamorph )

Q18 Which of the following Indian

chilly is considered one of the hottest in

the world

नननननलिखितभारतीयलमचचययह८मसहॳकह९नसीसबसहॳकह९नसीपवशवकीसबसहॳतीिीलमचचययह८मसहॳएकमानीजातीहहॴ

9-Jan -2017

Options

1) Bhut Jolokia

भतझह८िककया 2) Bhut Mahabora

भतमहाबह८रा 3) Lal Chitin

F A C E B O O K

P A G E h t t p w w w f a c e b o o k c o m s s c m e n t o r s o f f i c i a l P a g e | 6

FOR MORE UPDATES AND MORE MATERIAL DO LIKE OUR FACEBOOK PAGE httpwwwfacebookcomsscmentorsofficial

िािचीटटन

4) Lal Shamak

िाििामक

Correct Answer Bhut Jolokia

Q19 Brain fever is a disease spread

through which of the following

मनसतषकजवरनमकरह८गनननननलिखितमसहॳककसकहॳ कारणहह८ताहहॴ 9-Jan -2017

Options

1) Flies

मनकियो 2) Mosquito

मचछर

3) Virus

वायरस

4) Cockroach

नतिच हॳ Correct Answer Mosquito

Q20 Mangroves are plants that have

मगरह८ववहॳपहॳिहहॴनजनमहॳहह८ताहहॴ 9-Jan -2017

Options

1) Modified Roots

पातररतजड़हॳ 2) Modified Stems

पातररततनहॳ 3) Respiratory Roots

शरवसनकरनहॳवािीजड़हॳ 4) Respiratory Stems

शरवसनकरनहॳवािीतनहॳ Correct Answer Respiratory Roots

A mangrove is a shrub or small tree that

grows in coastal saline or brackish

water

Q21 Rodentia Sciurus is the scientific

name of

रह८िहॳलियासकीयरस mdashmdash कावयजजञाननकनामहहॴ

9-Jan -2017

Options

1) Rat

चहा 2) Platypus

पिहॳटीपस

3) Squirrel

चगिहरी 4) Beaver

बीवर

Correct Answer Squirrel

Q22 Which of the following is induced

by Oncogene

नननननलिखितमसहॳकह९नओकह८जीनदवारापरहॳररयतहह८ताहहॴ

10-Jan -2017

Options

1) Polio

पह८लियह८ 2) Cancer

क सर

3) Diarrhoea

दसत

4) Dengue

िग Correct Answer Cancer

An oncogene is a gene that has the

potential to cause cancer In tumor

cells they are often mutated andor

expressed at high levels

Q23 Azadirachata indica is the

scientific name of

अजाटदराकटाइडिका mdashmdashवहॴजञाननकनामहहॴ SSC CHSL Science (biology) 2016

Question Paper

10-Jan -2017

Options

1) Neem

नीम

F A C E B O O K

P A G E h t t p w w w f a c e b o o k c o m s s c m e n t o r s o f f i c i a l P a g e | 7

FOR MORE UPDATES AND MORE MATERIAL DO LIKE OUR FACEBOOK PAGE httpwwwfacebookcomsscmentorsofficial

2) Teak

सागह९न

3) Silver Oak

लसमवरओक

4) Tulsi

तिसी Correct Answer Neem

Q24 Octopus belongs to the phylum

ऑकटह८पसककसपरजानतकहॳ अतगयतआताहहॴ 10-

Jan -2017

Options

1) Mollusca

मह८िसका 2) Cnidaria

ननिहॳररया 3) Echinodermata

इकाइनह८ड़हॳमता 4) Chordata

कह८िता Correct Answer Mollusca

Q25 A living part of the organisms

environment is known as

जीवाणकहॳ वातावरणकहॳ जीपवतभागकह८ mdash-

कहतहॳहहॴ 10-Jan -2017

Options

1) Abiotic Factor

अजहॴपवककारक

2) Habitat

आवास

3) Biotic Factor

जहॴपवककारक

4) Nonliving factor

अ-जीपवतकारक

Correct Answer Biotic Factor

Abiotic factors are nonndash living chemical

and physical parts of the environment

that affect living organisms and the

functioning of ecosystems like rain

wind temperature altitude soil

pollution nutrients pH types of soil

and sunlight

Q26 Medulla oblongata is a part of

which of the following

महॳडयिाऑबिॉनगहॳटानननननलिखितमसहॳककसअगकाटहससाहहॴ

10-Jan -2017

Options

1) Heart

हदय

2) Brain

मनसतषक

3) Lungs

फहॳ फड़हॳ 4) Stomach

पहॳट

Correct Answer Brain

The medulla oblongata helps regulate

breathing heart and blood vessel

function digestion sneezing and

swallowing This part of the brain is a

center for respiration and circulation

Sensory and motor neurons (nerve cells)

from the forebrain and midbrain travel

through the medulla

Q27 ___________ is a typically

onecelled reproductive unit capable of

giving rise to a new individual without

sexual fusion

mdashmdash एकआमतह९रपरएककह८लिकीयहॳ परजननममसमकषइकाईहहॴजह८यह९नसियनकहॳ बबनाएकनयीइकाईकह८जनमदहॳतीहहॴ 10-Jan -2017

Options

1) Egg

अिाण

2) Spore

बीजाण

F A C E B O O K

P A G E h t t p w w w f a c e b o o k c o m s s c m e n t o r s o f f i c i a l P a g e | 8

FOR MORE UPDATES AND MORE MATERIAL DO LIKE OUR FACEBOOK PAGE httpwwwfacebookcomsscmentorsofficial

3) Sperm

ििाण

4) Seed

बीज

Correct Answer Spore

Q28 Bacteria was discovered by

बहॴकटीररयाकीिह८जककसकहॳ दवाराकीगयीिी

10-Jan -2017

1) Antonie van Leeuwenhoek

एटह८नीवहॳनलिबहॳनहक

2) Belarus

बहॳिा स

3) Hugo de Vries

हयगह८दीराईस

4)Robert Brown

रॉबटयिाउन

Correct Answer Antonie van

Leeuwenhoek

Q29 Which of the following is

responsible for Vermicomposting

नननननलिखितमसहॳकह९नकलमिादकहॳ लिएनजनमहॳदारहहॴ

10-Jan -2017

Options

1) Fungus

कवक

2) Worms

कलम

3) Bacteria

बहॴकटीररया 4) Birds

पकषी Correct Answer Worms

Vermicompost (or vermi-compost) is the

product of the composting process using

various species of worms usually red

wigglers white worms and other

earthworms to create a heterogeneous

mixture of decomposing vegetable or

food waste bedding materials and

vermicast

Q30 Scurvy (bleeding of gums) is

caused by the deficiency of which

vitamin

सकवी (मसढह८सहॳिनआना) ककसपवटालमनकीकमीकहॳ कारणहह८ताहहॴ

10-Jan-2017

Options

1) Vitamin K

पवटालमन K

2) Vitamin BZ

पवटालमन BZ

3) Vitamin C

पवटालमन C

4) Vitamin A

पवटालमन A

Correct Answer Vitamin C

Q31 Achras sapota is the scientific

name of

एिाससपह८ताइसकावहॴजञाननकनामहहॴ 10-Jan-2017

Options

1) Custard Apple

सीताफि

2) Gulmohar

गिमह८हर

3) Tamarind

इमिी 4) Chiku

चचक

Correct Answer Chiku

Q32 Prawn belongs to the phylum

झीगा mdashmdash- परजानतकहॳ अतगयतआताहहॴ 10-Jan-2017

Options

1) Arthropoda

F A C E B O O K

P A G E h t t p w w w f a c e b o o k c o m s s c m e n t o r s o f f i c i a l P a g e | 9

FOR MORE UPDATES AND MORE MATERIAL DO LIKE OUR FACEBOOK PAGE httpwwwfacebookcomsscmentorsofficial

अरोपह८िा 2) Cnidaria

नीिहॳररया 3) Echinodermata

इकाईनह८िमटा 4) Chordata

कह८िटा Correct Answer Arthropoda

Q33 Pulses are a rich source of which of

the following

दािहॳनननननलिखितमसहॳककसकीपरचरसह८तरहहॴ

11-Jan-2017

Options

1) Carbohydrates

काबोहाइडराईट

2) Proteins

परह८टीनस

3) Minerals

िननज

4) Vitamin A

पवटालमन A

Correct Answer Proteins

Q34 Plant cell wall is made up of

वनसपनतकह८लिकालभनततइससहॳबनीहह८तीहहॴ

11-Jan-2017

Options

1) Cellulose

सहॳमयिह८ज

2) Glucose

गिकह८ज

3) Sucrose

सिह८ज

4) Fructose

फरकटह८ज

Correct Answer Cellulose

Plant cell wall the major carbohydrates

are cellulose hemicellulose and pectin

The cellulose microfibrils are linked via

hemicellulosic tethers to form the

cellulose-hemicellulose network which

is embedded in the pectin matrix

Q35 The study of Fungi is also known

as कवकह८कहॳ अधययनकह८कहाजाताहहॴ

11-Jan-2017

Options

1) Cytology

सायटह८िह८जी 2) Myology

मायह८िह८जी 3) Mycology

मायकह८िह८जी 4) Neurology

नयरह८िह८जी Correct Answer Mycology

Cytology - structure and function of

plant and animal cells

Myology is the study of the muscular

system

Neurology is the branch of medicine

concerned with the study and treatment

of disorders of the nervous system

Q36 The outermost layer of skin is

तवचाकीसबसहॳबाहरीपरतकयाहह८तीहहॴ 11-Jan-

2017

Options

1) Epidermis

इपपिलमयस

2) Dermis

िलमयस

3) Tissues

ऊतक

4) Hypodermis

हायपह८िलमयस

Correct Answer Epidermis

Q37 Which of the following plants have

root nodules

F A C E B O O K

P A G E h t t p w w w f a c e b o o k c o m s s c m e n t o r s o f f i c i a l P a g e | 10

FOR MORE UPDATES AND MORE MATERIAL DO LIKE OUR FACEBOOK PAGE httpwwwfacebookcomsscmentorsofficial

नननननलिखितपह९धह८मसहॳककसकीजड़ह८मगाठहह८तीहहॴ

11-Jan-2017

Options

1) Leguminous plants

िहॳगयलमनसपह९धहॳ 2) Parasitic plants

परजीवीपह९धहॳ 3) Epiphytic Plants

एपीफाइटटकपह९धहॳ 4) Aquatic Plants

जिीयपह९धहॳ Correct Answer Leguminous plants

Q38 Earth-worms belongs to the

phylum

कहॳ चएmdashmdash- परजानतकहॳ अतगयतआतहॳहहॴ 11-Jan-2017

Options

1) Protozoa

परह८टह८जआ

2) Cnidaria

नीिहॳररया 3) Annelida

एनीलििा 4) Mollusca

मह८िसका Correct Answer Annelida

Q39 Ringworm is a disease caused by

ररगवमयनामकबीमारी mdashmdash- कहॳ कारणहह८तीहहॴ 11-Jan-2017

Options

1) Fungi

कवक

2) Bacteria

बहॴकटीररया 3) Virus

वायरस

4) Flies

मनकियाा Correct Answer Fungi

Q40 Mangifera indica is the scientific

name of

मननगफहॳ राइडिकाककसकावहॴजञाननकनामहहॴ 11-

Jan-2017

Options

1) Guava

अम द

2) Mango

आम

3) Amla

आविा 4) Jack fruit

कटहि

Correct Answer Mango

Q41 Crabs belongs to the phylum

कहॳ कड़हॳmdashmdash- परजानतकहॳ अतगयतआतहॳहहॴ 11-Jan-2017

Options

1) Mollusca

मह८िसका 2) Cnidaria

नीिहॳररया 3) Arthropoda

अरोपह८ड़ा 4) Platyhelminthes

पिहॳटटहहॳनममननिस

Correct Answer Arthropoda

Q42 Myopia is a defect of eyes which is

also known as

मायह८पपयाआिोकादह८षहहॴ नजसहॳ mdashmdashndash

भीकहाजाताहहॴ

12-Jan-2017

Options

1) Far Sightedness

F A C E B O O K

P A G E h t t p w w w f a c e b o o k c o m s s c m e n t o r s o f f i c i a l P a g e | 11

FOR MORE UPDATES AND MORE MATERIAL DO LIKE OUR FACEBOOK PAGE httpwwwfacebookcomsscmentorsofficial

दरदनषटदह८ष

2) Near Sightedness

ननकटदनषटदह८ष

3) Astigmatism

एसटीगमहॳटटजम

4) Night Blindness

रतोधी Correct Answer Near Sightedness

Myopia occurs when the eyeball is too

long relative to the focusing power of

the cornea and lens of the eye This

causes light rays to focus at a point in

front of the retina rather than directly

on its surface

Hyperopia Hypermetropia (

Farsightedness )- when light rays

entering the eye focus behind the retina

rather than directly on it The eyeball of

a farsighted person is shorter than

normal

Astigmatism usually is caused by an

irregularly shaped cornea Instead of

the cornea having a symmetrically

round shape (like a baseball) it is

shaped more like an American football

Nyctalopia also called night ndash blindness

is a condition making it difficult or

impossible to see in relatively low light

Q43 Who is known as the father of

Green Revolution

हररतिानतकहॳ जनककहॳ पमककसहॳजानाजाताहहॴ

12-Jan-2017

1) Dr Robert Nucleus

िॉ रॉबटयनयनकियस

2) Dr Ian Wilmut

िॉ इयानपविमट

3) Dr NE Borlaug

िॉ एनईबह८रिॉग

4) Dr JC Bose

िॉ जहॳसीबह८स

Correct Answer Dr NE Borlaug

Q44 Panthera Tigris is the scientific

name of

पिहॳराटटगरीस mdashmdashmdash कावहॴजञाननकनामहहॴ 12-Jan-2017

Options

1) Panther

तदआ

2) Tiger

बाघ

3) Whale

हहॳि

4)Goat

बकरी Correct Answer Tiger

Q45 How many facial bones are there

हमारहॳचहॳहरहॳमककतनीहडडियााहह८तीहहॴ 13-Jan-2017

Options

1)34

2)24

3)14

4)4

Correct Answer 14

Q46 ndash Halophytes are plants that grow

in

हहॴिह८फाईटसवहॳपह९धहॳहह८तीहहॴजह८ mdash- मउगतहॳहहॴ SSC CHSL Science (biology) 2016

Question Paper

13-Jan-2017

Options

1) Fresh Water

ताजापानी 2) Cold Water

ठिापानी 3) Ponds

तािाब

4) Salt Water

िारापानी Correct Answer Salt Water

F A C E B O O K

P A G E h t t p w w w f a c e b o o k c o m s s c m e n t o r s o f f i c i a l P a g e | 12

FOR MORE UPDATES AND MORE MATERIAL DO LIKE OUR FACEBOOK PAGE httpwwwfacebookcomsscmentorsofficial

Q47 Felis Catus is the scientific name of

फहॳ लिसकहॴ टस mdashndash कावहॴजञाननकनामहहॴ 13-Jan-2017

Options

1) Cat

बबमिी 2) Dog

कतता 3) Mouse

चहा 4) Porcupine

साही Correct Answer Cat

Q48 Which of the following induces

nitrogen fixation in soil

नननननलिखितमसहॳकह९नलम ीमनाइटरह८जनननयतनकह८परहॳररतकरताहहॴ

15-Jan-2017

Options

1) Protozoa

परह८टह८जआ

2) Bacteria

बहॴकटीररया 3) Fungi

कवक

4)Algae

िहॴवाि

Correct Answer Bacteria

Bacteria that change nitrogen gas from

the atmosphere into solid nitrogen

usable by plants are called nitrogen-

fixing bacteria These bacteria are

found both in the soil and in symbiotic

relationships with plants

They contain symbiotic bacteria called

rhizobia within nodules in their root

systems producing nitrogen compounds

that help the plant to grow and compete

with other plants When the plant dies

the fixed nitrogen is released making it

available to other plant

Q49 Which of the following is the

largest known cell

नननननलिखितमसहॳकह९नसीसबसहॳबड़ीजञातकह८लिकाहहॴ

SSC CHSL Science (biology) 2016

Question Paper

15-Jan-2017

1) Eukaryotic Cell

यकहॳ ररयह८टटककह८लिका 2) Prokaryotic Cell

परह८कहॳ ररयह८टटककह८लिका 3) Mycoplasma

मायकह८पिासम

4) Ostrich Eggs

ितरमगयकाअिा Correct Answer Ostrich Eggs

Q50 The association of animals in

which both the partners are benefitted

is known as

जानवरोकावहसहयह८गनजसमहॳदह८नोभागीदारिाभापवनतहह८तहॳहहॴ उसहॳ mdashmdashndash कहॳ पमजानाजाताहहॴ SSC CHSL Science (biology) 2016

Question Paper

15-Jan-2017

Options

1) Amensalism

सहजीपवत

2) Commensalism

परजीपवत

3) Colony

कॉिनी 4) Mutualism

अनयह८नयाशरयवाद

Correct Answer Mutualism

Amensalism is any relationship between

organisms of different species in which

F A C E B O O K

P A G E h t t p w w w f a c e b o o k c o m s s c m e n t o r s o f f i c i a l P a g e | 13

FOR MORE UPDATES AND MORE MATERIAL DO LIKE OUR FACEBOOK PAGE httpwwwfacebookcomsscmentorsofficial

one organism is inhibited or destroyed

while the other organism remains

unaffected

Commensalism an association between

two organisms in which one benefits and

the other derives neither benefit nor

harm

Q51 Pneumonia affects which of the

following organs of human body

ननमह८ननयामानविरीरकहॳ नननननलिखितमसहॳककसअगकह८परभापवतकरताहहॴ

15-Jan-2017

Options

1)Kidneys

गद

2)Lungs

फहॳ फड़हॳ 3) Throat

गिहॳ 4) Liver

यकत

Correct Answer Lungs

When the germs that cause pneumonia

reach your lungs the lungsrsquo air sacs

(alveoli) become inflamed and fill up

with fluid This causes the symptoms of

pneumonia such as a cough fever

chills and trouble breathing When you

have pneumonia oxygen may have

trouble reaching your blood

Q52 Mendel is known as

मििकह८ mdashmdash- कहॳ पमजानाजाताहहॴ 15-Jan-2017

Options

1) Father of Physiology

िरीरकियािासतरकहॳ जनक

2) Father of Geology

भगभयिासतरकहॳ जनक

3) Father of Genetics

जहॳनहॳटटकसकहॳ जनक

4) Father of Biology

जीविासतरकहॳ जनक

Correct Answer Father of Genetics

Q53 Which of the following are also

known as Suicidal bag of Cells

ननननलिखितमसहॳककसहॳआतमहतयाकरनहॳवािीकह८लिकाओकाबहॴगकहाजाताहहॴ

15-Jan-2017

Options

1) Lysosomes

िायसोसह८म

2) Lycosome

िायकह८सह८म

3) Nucleus

नालभक

4) Chromosome

िह८मह८सह८म

Correct Answer Lysosomes

Q54 Mesothelioma is a type of cancer

The most common area affected in it is

the lining of the ________

लमज़ह८िहॳिहॳलमयाक सरकाएकपरकारहहॴ इससहॳपरभापवतहह८नहॳवािासबसहॳसामानयकषहॳतर mdash

mdashmdash काअसतरहहॴ 15-Jan-2017

Options

1)Heart

हदय

2)Brain

मनसतषक

3)Stomach

आमािय

4)Lungs

फहॳ फड़हॳ Correct Answer lungs

Asbestos exposure is the main cause of

pleural mesothelioma When asbestos

fibers are breathed in they travel to the

F A C E B O O K

P A G E h t t p w w w f a c e b o o k c o m s s c m e n t o r s o f f i c i a l P a g e | 14

FOR MORE UPDATES AND MORE MATERIAL DO LIKE OUR FACEBOOK PAGE httpwwwfacebookcomsscmentorsofficial

ends of small air passages and reach the

pleura where they can cause

inflammation and

scarring

Q55 Which one of the following is an

insectivorous plant

नननननलिखितमसहॳकह९नसाएकककटाहरीवनसपनतहहॴ

15-Jan-2017

Options

1) Utricularia

यटरीकिहॳररया 2) Sequoia

सहॳकयओइया 3) Nostoc

नॉसटह८क

4) Bryophyta

िायह८फाईटा Correct Answer Utricularia

Q56 ______________ is a

multibranched polysaccharide of

glucose that serves as a form of energy

storage in animals and fungi

mdashmdashगिकह८जकाएकबहिािायकतपह८िीसहॳकहॳ राइिहहॴ जह८जानवरोऔरकवकमउजायभणिारणकहॳ एक पमकाययकरताहहॴ 15-Jan-2017

Options

1) Cellulose

सहॳमयिह८ज

2) Glycogen

गिायकह८जन

3) Pectin

पहॳनकटन

4) Chitin

चीटटन

Correct Answer Glycogen

Q57 The largest gland of the human

body is

mdashmdashmdashमानविरीरकीसबसहॳबड़ीगरिीहहॴ 16-Jan-2017

Options

1) Pancreas

अगयािय

2) Thyroid

िायरॉइि

3) Large Intestine

बड़ीआत

4) Liver

यकत

Correct Answer Liver

Q58 Photosynthesis in plants takes

place in

वनसपनतयोमपरकािसशिहॳषणकीकियाहह८तीहहॴ

16-Jan-2017

Options

1) Stem

तना 2) Leaves

पनततयाा 3) Roots

जड़हॳ 4) Flower

फि

Correct Answer Leaves

During this reaction carbon dioxide

and water are converted into glucose

and oxygen The reaction requires light

energy which is absorbed by a green

substance called

chlorophyll Photosynthesis takes place

in leaf

cells These contain chloroplasts which

are tiny objects containing chlorophyll

F A C E B O O K

P A G E h t t p w w w f a c e b o o k c o m s s c m e n t o r s o f f i c i a l P a g e | 15

FOR MORE UPDATES AND MORE MATERIAL DO LIKE OUR FACEBOOK PAGE httpwwwfacebookcomsscmentorsofficial

Q59 Insects that transmit diseases are

known as

जह८कीड़हॳरह८गसचाररतकरतहॳहहॴ उनह mdashmdash-

कहॳ नामसहॳजानाजाताहहॴ 16-Jan-2017

1)Pathogens

रह८गज़नक

2) Vectors

वहॳकटर

3) Drones

परजीवी 4)Scalars

अटदषट

Correct Answer Vectors

A vector is an organism that does not

cause disease itself but which spreads

infection by conveying pathogens from

one host to another Species of mosquito

for example serve as vectors for the

deadly disease Malaria

Q60 Which is the second largest gland

of Human body

मानविरीरकीदसरीसबसहॳबड़ीगरिीकह९नसीहहॴ

SSC CHSL Science (biology)

2016 Question Paper

16-Jan-2017

Options

1) Liver

यकत

2) Large Intestine

बड़ीआत

3) Thorax

छाती 4) Pancreas

अगनयािय

Correct Answer Pancreas

Q61 Annona squamosa is the scientific

name of

एनह८नासकवामह८सा (Annona squamosa) mdash

mdashmdash कावहॴजञाननकनामहहॴ 16-Jan-2017

Options

1) Custard Apple

सीताफि

2) Papaya

पपीता 3) Babhul

बबि

4) Drumstick

सहजन

Correct Answer Custard Apple

Q62 The disease Beri Beri is caused due

to the deficiency of which of the

following

बहॳरीबहॳरीरह८गनननननलिखितमसहॳककसकीकमीकहॳकारणहह८ताहहॴ

16-Jan-2017

Options

1) Vitamin B2

पवटालमन B2

2) Vitamin B1

पवटालमन B1

3) Vitamin B12

पवटालमन B12

4) Vitamin E

पवटालमन E

Correct Answer Vitamin B1

Beriberi is a disease caused by a vitamin

B-1 deficiency also known as thiamine

deficiency

Q63 Chlorophyll was first isolated and

named by

किह८रह८कफिकह८ mdash-

दवारापहिहॳपिकऔरनालमतककयागया 16-Jan-2017

F A C E B O O K

P A G E h t t p w w w f a c e b o o k c o m s s c m e n t o r s o f f i c i a l P a g e | 16

FOR MORE UPDATES AND MORE MATERIAL DO LIKE OUR FACEBOOK PAGE httpwwwfacebookcomsscmentorsofficial

Options

1) Caventou

कहॳ वहॳत 2) Pelletier

पहॳिहॳटटयर

3) Chlorophyll

किह८रह८कफि

4) Caventou and Pelletier

कहॳ वहॳतऔरपहॳिहॳटटयर

Correct Answer Caventou and Pelletier

Chlorophyll was first isolated and

named by

Joseph Bienaimeacute Caventou and Pierre

Joseph Pelletier in 1817 The presence of

magnesium in chlorophyll was

discovered in 1906 and was the first

time that magnesium had been detected

in living tissue

Q64 Which of the following organisms

does not fit into the Cell Theory

नननननलिखितमसहॳकह९नसाजीवकह८लिकालसदातअन पनहीहहॴ

16-Jan-2017

Options

1) Bacteria

बहॴकटीररया 2) Virus

वायरस

3) Fungi

कवक

4) Plants

पह९धहॳ Correct Answer Virus

The bottom line is that viruses are not

alive and not related to cells in any way

The cell theory states that all living

things are made of cells cells are the

basic units of structure and function of

living things and that all cells come

from other cells Since viruses are not

made of cells and do not use cells in any

of their processes they are not related to

the cell theory

Q65 Which of these is not a

macronutrient for Plants

नननननलिखितमसहॳकह९नसापह९धह८कहॳ लिएमिह८नयटरीएटनहीहहॴ

SSC CHSL Science (biology) 2016

Question Paper

17-Jan-2017

Options

1) Nitrogen

नाइटरह८जन

2) Phosphorus

फासफह८रस

3) Potassium

पह८टालसयम

4) Chlorine

किह८रीन

Correct Answer Chlorine

In relatively large amounts the soil

supplies nitrogen phosphorus

potassium calcium magnesium and

sulfur these are often called the

macronutrients In relatively small

amounts the soil supplies iron

manganese boron molybdenum

copper zinc chlorine and cobalt the

so-called micronutrients

Q66 Name the respiratory organs of

insects

कीटह८मनसतिशरवसनअगनामकानामहहॴ

17-Jan-2017

Options

1) Skin

तवचा 2) Body Surface

िरीरकीसतह

F A C E B O O K

P A G E h t t p w w w f a c e b o o k c o m s s c m e n t o r s o f f i c i a l P a g e | 17

FOR MORE UPDATES AND MORE MATERIAL DO LIKE OUR FACEBOOK PAGE httpwwwfacebookcomsscmentorsofficial

3) Gills

गिफड़हॳ 4) Tracheae

शरावस- निी Correct Answer Tracheae

Air enters the respiratory systems of

insects through a series of external

openings called

spiracles These external openings

which act as muscular valves in some

insects lead to the internal respiratory

system a densely networked array of

tubes called tracheae

Q67 The poisonous gas accidentally

released in Bhopal Gas Tragedy is

भह८पािगहॴसतरासदीमगितीसहॳमकतहईजहरीिीगहॴसिी

17-Jan-2017

1) Methane

मीिहॳन

2) Nitrous Oxide

नाइटरसऑकसाइि

3) Methyl Isocyanate

महॴचििआयसोसायनहॳट

4) Cyanogen

सायनह८जहॳन

Correct Answer Methyl Isocyanate

Q68 What does Trypsin do

टटरनपसनकयाकरताहहॴ

SSC CHSL Science (biology) 2016

Question Paper

17-Jan-2017

Options

1) Breaks down Carbohydrates

काबोहाइडरहॳटकापवघटनकरताहहॴ 2) Synthesizes proteins

परह८टीनकासििहॳषणकरताहहॴ 3) Breaks down fats

वसाकापवघटनकरताहहॴ 4) Breaks down proteins

परह८टीनकापवघटनकरताहहॴ Correct Answer Breaks down proteins

Trypsin is one of the three principal

digestive

proteinases the other two being pepsin

and

chymotrypsin In the digestive process

trypsin acts with the other proteinases

to break down dietary protein molecules

to their component

peptides and amino acids

A protease is any enzyme that performs

proteolysis protein catabolism by

hydrolysis of peptide bonds

Q69 Name the source from which

Aspirin is produced

उससरह८तकानामबताइए

नजससहॳएनसपररनकाउतपादनककयाजाताहहॴ

17-Jan-2017

Options

1) Willow bark

पविह८कीछाि

2) Oak Tree

ओककावकष

3) Acacia

बबि

4) Eucalyptus

नीिचगरी Correct Answer Willow bark

The compound from which the active

ingredient in aspirin was first derived

salicylic acid was found in the bark of a

willow tree in 1763 by Reverend

Edmund Stone of Chipping-Norton

Q70 Cannis Familiaris is the scientific

name of

कहॴ ननसफहॳ लमलियहॳररस mdash- कावहॴजञाननकनामहहॴ

17-Jan-2017

F A C E B O O K

P A G E h t t p w w w f a c e b o o k c o m s s c m e n t o r s o f f i c i a l P a g e | 18

FOR MORE UPDATES AND MORE MATERIAL DO LIKE OUR FACEBOOK PAGE httpwwwfacebookcomsscmentorsofficial

Options

1) Cat

बबमिी 2)Dog

कतता 3) Fox

िह८मड़ी 4) Wolf

भहॳडड़या Correct Answer Dog

Q71 Harmful bacteria in potable water

make the water

पीनहॳकहॳ पानीमनसतिघातकबहॴकटीररयाउसपानीकह८बनातहॳहहॴ 17-Jan-2017

Options

1) unfit to drink

पीनहॳकहॳ लिएअयह८गय

2) smelly

दगयनधयकत

3) Colored

रगीन

4) Turbid

मटमहॴिा Correct Answer unfit to drink

Q72 Musa paradisiaca is the scientific

name of which plant

मसापहॴराडिलसयाकाककसपह९धहॳकावहॴजञाननकनामहहॴ

17-Jan-2017

Options

1) Mango

आम

2) Wheat

गहॳह

3) Corn

भ ा 4) banana

कहॳ िा Correct Answer banana

Q73 Prawns belong to which family

झीगहॳककसपररवारकहॳ हह८तहॳहहॴ 17-Jan-2017

Options

1) Crustaceans

िसटहॳलियन

2)Fish

मछिी 3) Amphibians

अननफबबयस

4) Reptiles

रहॳपटाइमस

Correct Answer Crustaceans

Q74 Name the drug that is yielded from

Cinchona tree and is used to cure

malaria

उसऔषचधकानामबताइएनजसहॳलसगकह८नापहॳड़सहॳपरापतककयाजाताहहॴऔरनजसकाउपयह८गमिहॳररयाकहॳ उपचारमककयाजाताहहॴ 17-Jan-2017

Options

1) Camptothea

कहॴ नटह८चिया 2) Acuminata

एकयलमनहॳटा 3) Quinine

कनहॴन

4) Cinchonia

लसकह८ननया Correct Answer Quinine

Q75 Blood Circulation was discovered

by

रकतपररसचरणकी mdashmdashndash दवारािह८जकीिी 17-Jan-2017

Options

1) Mary Anderson

F A C E B O O K

P A G E h t t p w w w f a c e b o o k c o m s s c m e n t o r s o f f i c i a l P a g e | 19

FOR MORE UPDATES AND MORE MATERIAL DO LIKE OUR FACEBOOK PAGE httpwwwfacebookcomsscmentorsofficial

महॴरीएिरसन

2) Virginia Apgar

वनजयननयाएपगार

3) William Harvey

पवलियमहाव

4) Robert Feulgen

रॉबटयफ़यिजहॳन Correct Answer William Harvey

Q76 Vitamin A is also known as

पवटालमन A कह८ mdashmdash- कहॳ नामसहॳभीजानाजाताहहॴ SSC CHSL Science (biology) 2016

Question Paper

18Jan2017

Options

1) Thiamine

िायलमन

2) Riboflavin

ररबह८फिहॳपवन

3) Retinol

रहॳटटनॉि

4) Calciferol

कहॴ नमसफहॳ रह८ि

Correct Answer Retinol

Q77 Some roots called arise from an

organ other than the radicle

कछजड़हॳनजनह mdashmdashmdash कहाजाताहहॴ वहमिकहॳ अिावाककसीअनयअगसहॳउतपननहह८तीहहॴ 18Jan2017

Options

1) tap roots

मखयजड़

2) stilt roots

ि ाजड़

3) fibrous roots

रहॳिहॳदारजड़

4) adventitious roots

आकनसमकजड़

Correct Answer adventitious roots

Q78 Spiders belong to which class of

animals

मकडड़यापराणीवगीकरणकहॳ ककसवगयमआतीहहॴ 18Jan2017

Options

1) Arachnids

एरहॳकननडस

2) Aves

एपवस

3) Gastropods

गहॴसटरोपह८िस

4) Anthozoa

एिह८जआ

Correct Answer Arachnids

Q79 How many layers does Human

Skin have

मानवतवचामककतनीपरतहॳहह८तीहहॴ

18Jan2017

Options

1) 5

2) 7

3) 11

4) 3

Correct Answer 3

Skin has three layers The epidermis

the outermost layer of skin provides a

waterproof barrier and creates our skin

tone The dermis beneath the

epidermis contains tough connective

tissue hair follicles and sweat glands

The deeper subcutaneous tissue (

hypodermis ) is made of fat and

connective tissue

Q80 Allium Cepa is the scientific name

of

एलियमलसपपा mdashmdashndash कावहॴजञाननकनामहहॴ 18Jan2017

F A C E B O O K

P A G E h t t p w w w f a c e b o o k c o m s s c m e n t o r s o f f i c i a l P a g e | 20

FOR MORE UPDATES AND MORE MATERIAL DO LIKE OUR FACEBOOK PAGE httpwwwfacebookcomsscmentorsofficial

Options

1) Carrot

गाजर

2) Tomato

टमाटर

3) Potato

आि 4) Onion

पयाज़

Correct Answer Onion

Q81 DNA stands for

िीएनएकापणय प mdashmdash- हहॴ 18Jan2017

Options

1) Di Nucleic Acid

िाईनयनकिकएलसि

2) Deoxy Nucleic Acid

िीओकसीनयनकिकएलसि

3) Diribonucleic Acid

िाईराइबह८नयनकिकएलसि

4) Deoxyribonucleic Acid

िीऑकसीराइबह८नयनकिकएलसि

Correct Answer Deoxyribonucleic Acid

Q82 Organisms that generate energy

using light are known as

जह८जीवाणपरकािकाउपयह८गकरउजायउतपननकरतीहहॴ उनह mdashmdash कहॳ पमजानाजाताहहॴ

18Jan2017

Options

1) Chaemolithotrophs

ककमह८लििह८टरह८पस

2) Oligotrophs

ओलिगह८टरह८पस

3) Bacteria

बहॴकटीररया 4)Photoautotrophs

फह८टह८ओटह८टरह८पस

Correct Answer Photoautotrophs

An oligotroph is an organism that can

live in an environment that offers very

low levels of nutrients

Q83 Which drug is used as an

Antidepressant

ककसदवाएकहतािारह८धीकहॳ पमपयोगककयाजाताहहॴ Options

1) Oxybutynin

ओकसीलयटीनन

2)Tramadol

टरहॳमहॳिह८ि

3 ) Sumatriptan

समहॳटरीपटहॳन

4) Bupropion

लयपरह८पपयह८न

Correct Answer Bupropion

लयपरह८पपयह८न

Q84 The orange colour of carrot is

because of

गाजरकानारगीरगनननननलिखितमसहॳककसीएककीवजहसहॳहह८ताहहॴ 18Jan2017

Options

1) it grows in the soil

यहलम ीमउगतीहहॴ 2) Carotene

कहॴ रह८टीन

3) it is not exposed to sunlight

यहसययपरकािकहॳ सपकय मनहीआती 4) the entire plant is oranqe in colour

सनपणयपह९धानारगीरगकाहह८ताहहॴ Correct Answer Carotene

Q85 Snake venom is highly modified

saliva containing

F A C E B O O K

P A G E h t t p w w w f a c e b o o k c o m s s c m e n t o r s o f f i c i a l P a g e | 21

FOR MORE UPDATES AND MORE MATERIAL DO LIKE OUR FACEBOOK PAGE httpwwwfacebookcomsscmentorsofficial

सापकाजहरअततयाचधकसिह८चधतिारहह८तीहहॴनजसमहॳ mdashmdash- हह८ताहहॴ Options

l)Prototoxins

परह८टह८टॉनकसस

2)Neutrotoxins

नयटरोटॉनकसस

3)Zootoxins

जटॉनकसस

4)Electrotoxins

इिहॳकटरह८टॉनकसस

Correct Answer Zootoxins

जटॉनकसस

Q86 Which type of pathogen causes the

water-borne disease Schistosomiasis

ककसपरकारकारह८गज़नकजिजननतरह८गलससटह८सह८लमलससकाकारणबनताहहॴ

18Jan2017

Option

1) Parasitic

परजीवी 2)Protozoan

परह८टह८जआ

3) Bacterial

बहॴकटीररयि

4) Viral

वायरि

Correct Answer Parasitic

Schistosomiasis also known as snail

fever and bilharzia is a disease caused

by parasitic

flatworms called schistosomes

Q87 Prothrombin responsible for

clotting of blood is released by

परह८िह८ननबन

जह८रकतकािककाजमनहॳकहॳ लिएनजनमहॳदारहहॴ mdashndash

कहॳ दवारासतरापवतककयाजाताहहॴ

19Jan2017

Options

1) Small Intestine

छह८टीआत

2) Blood Platelets

रकतपिहॳटिहॳटस

3) Large Intestine

बड़ीआत

4Heart

हदय

Correct Answer Blood Platelets

Q88 Acacia arabica is the scientific

name of

अकहॳ लियाअरहॳबबका mdashmdashndash कावहॴजञाननकनामहहॴ 19-Jan-2017

Options

1) Neem

नीम

2) Teak

सागह९न

3) Babhul

बबि

4) Pomegranate

अनार

Correct Answer Babhul

Q89 Cannis Vulpes is the scientific

name of

कहॴ ननसवनमपस mdashmdash- कावहॴजञाननकनामहहॴ 19-Jan-2017

Options

1) Dog

कतता 2) Wolf

भहॳडड़या 3) Fox

िह८मड़ी 4) Hyena

िाकिबगघा

F A C E B O O K

P A G E h t t p w w w f a c e b o o k c o m s s c m e n t o r s o f f i c i a l P a g e | 22

FOR MORE UPDATES AND MORE MATERIAL DO LIKE OUR FACEBOOK PAGE httpwwwfacebookcomsscmentorsofficial

Correct Answer Fox

Q90 The beetroot is the portion of the

beet plant

चकदरपह९धहॳका mdashmdashndash भागहहॴ 19-Jan-2017

Options

1) tap root

मखयजड़

2) Adventitious

आकनसमक

3) bulb of the stem

तनहॳकाकद

4) Rhizome

परकद

Correct Answer tap root

Q91 What is the basic unit of heredity

आनवलिकताकीबननयादीइकाईकयाहहॴ 19-Jan-2017

Options

1) DNA

िीएनए

2) RNA

आरएनए

3) Chromosome

िह८मह८सह८म

4) Gene

जीन

Correct Answer gene

Genes are the units of heredity and are

the instructions that make up the bodyrsquos

blueprint They code for the proteins

that determine virtually all of a personrsquos

characteristics Most genes come in

pairs and are made of strands of genetic

material called deoxyribonucleic acid

or DNA

Q92 Lungs are the primary organs of

फहॳ फड़हॳmdashndashकहॳ परािलमकअगहहॴ

19-Jan-2017

Options

1) Digestion

पाचन

2) Constipation

कलज

3) Perspiration

पसीना 4)Respiration

शवसन

Correct Answer Respiration

Q93 Sugarcane is a type of

गननाएकपरकारका mdash- हहॴ 20-Jan-2017

Options

1)creeper

िता 2)tree

पहॳड़

3)shrub

झाड़ी 4)grass

घास

Correct Answer grass

Q94 Who is commonly known as ldquothe

Father of Microbiologyrdquo

सामानयत ldquo सकषमजीवपवजञानकहॳ जनक lsquo

कहॳ नामसहॳककसहॳजानाजातहहॴ 20-Jan-2017

Options

1) Robert Hooke

रॉबटयहक

2) Antonie Philips van Leeuwenhoek

एटह८नीकफलिपवानमयएनहह८क

3) Carl Linnaeus

काियिीनाईयस

4) Charles Darwin

चामसयिापवयन

F A C E B O O K

P A G E h t t p w w w f a c e b o o k c o m s s c m e n t o r s o f f i c i a l P a g e | 23

FOR MORE UPDATES AND MORE MATERIAL DO LIKE OUR FACEBOOK PAGE httpwwwfacebookcomsscmentorsofficial

Correct Answer Antonie Philips van

Leeuwenhoek

Q95 For the aquatic organisms the

source of food is

जिीयजीवाणकािाघसरह८तहहॴ 20-Jan-2017

Options

1) Phytoplankton

फायटह८पिहॳकटन

2) Sea Weed

समदरीिहॴवाि

3)Aqua plankton

एकवापिहॳकटन

4) Zooplankton

जपिहॳकटन

Correct Answer Phytoplankton

Q96 Haemoglobin has the highest

affinity with which of the following

हीमह८गिह८बबनकीननननमसहॳककसकहॳ सािउततमसमानताहहॴ

20-Jan-2017

Options

1)SO2

2)CO2

3)CO

4)NO2

Correct Answer CO

It has a greater affinity for hemoglobin

than oxygen does It displaces oxygen

and quickly binds so very little oxygen

is transported through the body cells

Q97 Who developed the theory of

Evolution

उदपवकासकालसदातककसनहॳपवकलसतककया

20-Jan-2017

Options

1) Charles Darwin

चामसयिापवयन

2) Isaac Newton

आयजहॳकनयटन

3) Pranav Mistry

परणवलमसतरी 4) Galileo Galilei

गहॳलिलियह८गहॳिीिी Correct Answer Charles Darwin

Q98 The primary function of RNA is

RNA कापरािलमककाययहह८ताहहॴ 20-Jan-2017

Options

1) Photosynthesis

परकािसशिहॳषण

2) Protein Synthesis

परह८टीनसशिहॳषण

3) Replication

परनतकनतबनाना 4) Translation

अनवादकरना Correct Answer Protein Synthesis

There are two main functions of RNA

It assists DNA by serving as a messenger

to relay the proper genetic information

to countless numbers of ribosomes in

your body The other main function of

RNA is to select the correct amino acid

needed by each ribosome to build new

proteins for your body

Q99 ______is the movement of

molecules across a cell membrane from

a region of their lower concentration to

a region of their higher concertration

उचचसादरताकहॳ कषहॳतरसहॳउसकीकमसादरतावािहॳकषहॳतरकीतरफएककह८लिकाखझमिीकहॳ माधयमसहॳहह८नहॳवािाअणओकहॳ सचिनकह८ mdash- कहतहॳहहॴ Options

1) Diffusion

पवसरण

2) Osmosis

ऑसमह८लसस

F A C E B O O K

P A G E h t t p w w w f a c e b o o k c o m s s c m e n t o r s o f f i c i a l P a g e | 24

FOR MORE UPDATES AND MORE MATERIAL DO LIKE OUR FACEBOOK PAGE httpwwwfacebookcomsscmentorsofficial

3) Active Transport

सकियआवागमन

4) Passive Transport

नननषियआवागमन

Correct Answer Active Transport

Q100 Study of classification of

organisms is known as 20-Jan-2017

जीवाणओकहॳ वगीकरणकहॳ अधययनकह८ mdash-

कहाजाताहहॴ Options

1) Serpentology

सपरहॳटह८िह८जी 2) Virology

वायरह८िह८जी 3) Taxonomy

टहॴकसोनह८मी 4) Physiology

कफनज़यह८िह८जी Correct Answer Taxonomy

Q101 Photosynthesis takes place inside

plant cells in

परकािसशिहॳषणवनसपनतकह८लिकामनसति mdash

mdashmdash महह८ताहहॴ 20-Jan-2017

Options

1) Ribosomes

राइबह८सह८नस

2) Chloroplasts

किह८रह८पिासट

3) Nucleus

नयकलियम

4) Mitochondria

माईटह८कोडडरया Correct Answer Chloroplasts

Q102 ______ is the cell organelle in

which the biochemical processes of

respiration and energy production

occur

mdashmdash- वहकह८लिकाअगहहॴ नजसमहॳशवसनऔरउजायउतपादनकहॳ जहॴसीजहॴवरासायननकपरकियायहह८तीहहॴ 20-Jan-2017

Options

1) Mitochondria

माइटह८कोडडरया 2) Chloroplast

किह८रह८पिासट

3) Ribosomes

राइबह८सह८नस

4) Nucleus

नयकिीयस

Correct Answer Mitochondria

Q103 Which non-flowering spore

bearing plants have roots

ककसफिनिगनहॳवािहॳऔरबीजाणधारकपह९धह८कीजड़हॳहह८तीहहॴ 21-Jan-2017

Options

1) Mosses

मह८सहॳस

2) Angiosperms

एननजयह८सपनसय 3) Ferns

फनसय 4) Gymnosperms

नजननह८सपनसय Correct Answer ferns

Q104 Which of the following is an

excretory organ of cockroach

नननननलिखितमसहॳकह९नसानतिच हॳकाउतसजयनअगहहॴ

21-Jan-2017

Options

F A C E B O O K

P A G E h t t p w w w f a c e b o o k c o m s s c m e n t o r s o f f i c i a l P a g e | 25

FOR MORE UPDATES AND MORE MATERIAL DO LIKE OUR FACEBOOK PAGE httpwwwfacebookcomsscmentorsofficial

1) Malphigian Tubules

मनमफनजयनटयबमस

2) Nephridia

नहॳकफरडिया 3) Coxal Gland

कह८कसिगरचिया 4) Green Gland

गरीनगरचिया Correct Answer Malphigian Tubules

Q105 Evaporation of water takes place

in which part of plants

पानीकहॳ वाषपीकरणकीकियापह९धोकहॳ ककसभागसहॳहह८तीहहॴ 21-Jan-2017

Options

1) Stem

तना 2) Stomata

सटह८मटा 3) Branch

िािाए

4) Fruit

फि

Correct Answer Stomata

Evaporation accounts for the movement

of water to the air from sources such as

the soil canopy interception and

waterbodies Transpiration accounts for

the movement of water within a plant

and the subsequent loss of water as

vapour through stomata in its leaves

Q106 A is the fleshy spore-bearing

fruiting body of a fungus

mdashmdashndashकवककामासि

बीजाणधारणकरनहॳवािाफिनहॳवािाअगहहॴ 21-

Jan-2017

Options

1) aloe vera

एिह८वहॳरा 2) Coral

मगा 3) Cactus

कहॴ कटस

4) Mushroom

ककरमतता Correct Answer mushroom

Q107 Which of the following is a fungal

disease

नननननलिखितमसहॳकह९नसाफफदसहॳहह८नहॳवािाएकरह८ग हहॴ

21-Jan-2017

Options

1) Dermatitis

तवचािह८ध

2) Cholera

हहॴजा 3) Jaundice

पीलिया 4) Indigofera

इननिगह८फहॳ रा Correct Answer Dermatitis

Dermatitis also known as eczema is a

group of diseases that results in

inflammation of the skin These diseases

are characterized by itchiness red skin

and a rash In cases of short duration

there may be small blisters while in

long-term cases the skin may become

thickened

Q108 In which form is glucose stored in

our body

हमारहॳिरीरमगिकह८जकासचयककस पमककयाजाताहहॴ

21-Jan-2017

Options

1) Insulin

F A C E B O O K

P A G E h t t p w w w f a c e b o o k c o m s s c m e n t o r s o f f i c i a l P a g e | 26

FOR MORE UPDATES AND MORE MATERIAL DO LIKE OUR FACEBOOK PAGE httpwwwfacebookcomsscmentorsofficial

इसलिन

2) Glucose

गिकह८ज

3) Glycogen

गिायकह८जहॳन

4) Fat

वसा Correct Answer Glycogen

Excess glucose is stored in the liver as

the large compound called glycogen

Glycogen is a polysaccharide of glucose

but its structure allows it to pack

compactly so more of it can be stored in

cells for later use

Q109 Where do plants synthesize

protein from

पह९धहॳपरह८टीनसशिहॳषणकहासहॳकरतहॳहहॴ

Options

1) Fatty Acids

वसाऐलसि

2) Sugar

िकर

3) Amino Acids

एलमनह८ऐलसि

4) Starch

सटाचय Correct Answer Amino Acids

Q110 Which part of the brain is

responsible for triggering actions like

thinking intelligence memory and

ability to learn

मनसतषककाकह९नसाटहससासह८चनहॳ बनधदमानी याददाशतऔरसीिनहॳकीकषमताजहॴसीकियाओकह८परहॳररतकरताहहॴ 21-Jan-2017

Options

1) Diencephalon

िायएनसहॳफहॳ िह८न

2) Hypothalamus

हयपह८िहॳिहॳमस

3) Cerebrum

सहॳरहॳिम

4) Control

कटरह८ि

Correct Answer Cerebrum

Q111 Which of the following is also

known as the Biochemical Laboratory

of the Human Body

नननननलिखितमसहॳककसहॳमानविरीरकीजहॴवरसायनपरयह८गिािाभीकहाजाताहहॴ 21-Jan-2017

Options

1) Small Intestine

छह८टीआत

2)Brain

मनसतषक

3) Pancreas

अगनयािय

4) Liver

नजगर

Correct Answer Liver

The liver makes bile that will help

emulsify and digest the fats we eat

The liver takes toxic substances and

convert them using enzymes the liver

cells makes into a non toxic form so the

body can dispose of them

The liver also converts fats protein and

carbohydrates into glucose which is the

energy source for our cells to use

The liver takes amino acids and makes

proteins by combining them

Q112 The yellow colour of human urine

is due to

मानवमतरकापीिारग mdashndash कीवजहसहॳहह८ताहहॴ 22-

Jan-2017

Options

1) Bile Salts

F A C E B O O K

P A G E h t t p w w w f a c e b o o k c o m s s c m e n t o r s o f f i c i a l P a g e | 27

FOR MORE UPDATES AND MORE MATERIAL DO LIKE OUR FACEBOOK PAGE httpwwwfacebookcomsscmentorsofficial

पपततनमक

2) Cholesterol

कह८िहॳसटरह८ि

3) Lymph

लिनफ

4) Urochrome

यरह८िह८म

Correct Answer Urochrome

Urobilin or urochrome is the chemical

primarily responsible for the yellow

color of urine

Q113 The wilting of plants takes place

due to

पह९धह८कालिचििहह८नाकी mdashmdash- कीवजहसहॳहह८ताहहॴ 22-Jan-2017

Options

1)Photosynthesis

परकािसशिहॳषण

2) Transpiration

वाषपह८तसजयन

3) Absorption

अविह८षण

4) Respiration

शरवसन

Correct Answer Transpiration

Wilting is the loss of rigidity of non-

woody parts of plants This occurs when

the turgor pressure in non-lignified

plant cells falls towards zero as a result

of diminished water in the cells

Q114 Bovidae Ovis is the scientific name of

बह८पविीओपवस mdashndash कावहॴजञाननकनामहहॴ 22-Jan-2017

Options

1) Goat

बकरी 2) Cow

गाय

3) Buffalo

भहॳस

4) Sheep

भहॳड़

Correct Answer Sheep

Q115 Plants get their energy to produce

food from which of the following

पह८धहॳभह८जनकाननमायणकरनहॳकहॳ लिएनननननलिखितमसहॳककससहॳउजायपरापतकरतहॳहहॴ

22-Jan-2017

Options

1) Photosynthesis

परकािसशिहॳषण

2)Bacteria

बहॴकटीररया 3)Fungi

कवक

4)Sun

सयय Correct Answer Sun

Q116 Which of the following is secreted

by the liver

नननननलिखितमसहॳककसकासरावनजगरसहॳहह८ताहहॴ

22-Jan-2017

Options

1) Glucose

गिकह८ज

2) Iodine

आयह८िीन

3) Cortisol

काटटरयसह८ि

4) Bile

पपतत

Correct Answer Bile

The liver makes bile that will help

emulsify and

digest the fats we eat

F A C E B O O K

P A G E h t t p w w w f a c e b o o k c o m s s c m e n t o r s o f f i c i a l P a g e | 28

FOR MORE UPDATES AND MORE MATERIAL DO LIKE OUR FACEBOOK PAGE httpwwwfacebookcomsscmentorsofficial

Q117 Ferns belong to which division of

plants

फनसयपह९धह८कहॳ ककसभागमआतहॳहहॴ

22-Jan-2017

Options

1) Gymnosperms

नजननह८सपनसय 2) Angiosperms

एनजयह८सपनसय 3) Thallophyta

िहॴिह८फाईटा 4)Pteridophyta

टहॳररिह८फाईटा Correct Answer Pteridophyta

Q118 Who invented Antibiotics

एटीबायह८टटककाअपवषकारककसनहॳककयािा

22-Jan-2017

Options

1) Joseph Lister

जह८सहॳफलिसटर

2) William Harvey

पवलियमहाव

3) Robert Knock

रॉबटयनॉक

4)Alexander Fleming

अिहॳकज़िरफिहॳलमग

Correct Answer Alexander Fleming

Q119 Milbecycin is used in the

eradication of

लममबहॳसायलसनका mdashndash

मउनमिनमपरयह८गककयाजाताहहॴ 22-Jan-2017

Options

1) Agricultural Fungus

कपषकवक

2) Agricultural Pests

कपषकीटक

3) Agricultural Herbs

कपषिाक

4)Agricultural Weeds

कपषननराना Correct Answer Agricultural Pests

Milbemycin oxime is a veterinary drug

from the group of milbemycins used as

a broad spectrum antiparasitic It is

active against worms and mites(insects

Q120 Intestinal bacteria synthesizes

which of the following in the human

body

मानविरीरमआतोकहॳ बहॴकटीररयानननननलिखितमसहॳककसकासशिहॳषणकरतहॳहहॴ 22-Jan-2017

Options

1) Vitamin K

पवटालमन K

2) Proteins

परह८टीन

3) Fats

वसा 4) Vitamin D

पवटालमन D

Correct Answer Vitamin K

Q121 is the study of the physical form

and external structure of plants

mdashmdash-

मपह९धह८काभहॴनतक पऔरबाहरीसरचनाकाआदयाककयाजाताहहॴ 22-Jan-2017

Options

1) Physiology

कफनजयह८िह८जी 2) Anatomy

िरीररचनापवजञान

3) Phytomorphology

फाईटह८मह८फह८िह८जी 4)Cytology

कह८लिकापवजञान

Correct Answer Phytomorphology

F A C E B O O K

P A G E h t t p w w w f a c e b o o k c o m s s c m e n t o r s o f f i c i a l P a g e | 29

FOR MORE UPDATES AND MORE MATERIAL DO LIKE OUR FACEBOOK PAGE httpwwwfacebookcomsscmentorsofficial

Q122 Which of the following is a

structural and functional unit of

kidneys

नननननलिखितमसहॳकह९नसीगदोकीसरचनातमकऔरकाययकरीईकाईहहॴ

22-Jan-2017

Options

1) Renette Cells

रहॳनहॳटकह८लिकाए

2) Flame Cells

फिहॳमकह८लिकाए

3) Nephrites

नहॳफ़राइटस

4)Nephrons

नहॳफरोस

Correct Answer Nephrons

Nephron functional unit of the kidney

the structure that actually produces

urine in the process of removing waste

and excess substances from the blood

There are about 1000000 nephrons in

each human kidney

Q123 Which of the following is the

largest part of the human brain

नननननलिखितमसहॳकह९नसामानवमनसतषककासबसहॳबड़ाटहससाहहॴ

23-Jan-2017

Options

1) Ribs

पसलियाा 2) Cerebrum

सहॳरहॳिम

3) Pons

पोस

4)Thalamus

िहॴिहॳमस

Correct Answer Cerebrum

The cerebrum is the largest part of the

human brain making up about two-

thirds of the brainrsquos mass It has two

hemispheres each of which has four

lobes frontal parietal temporal and

occipital

Q124 The auxiliary buds

सहायककालियाmdashndash 23-Jan-2017

Options

1) grow endogenously from the pericycle

पहॳरीसाईककिसहॳअनतजातयपवकलसतहह८ताहहॴ 2) arise endogenously from the main

growing point

मिवपदसहॳअनतजातयउठताहहॴ 3) is an embryonic shoot located in the

axil of a leaf

एकभरणिटहहॴजह८एकपततीकहॳ अकषपरनसतिहह८ताहहॴ 4)arise exogenously from the epidermis

एपपिलमयससहॳबटहजातयतरीकहॳ सहॳउठताहहॴ Correct Answer is an embryonic shoot

located in the axil of a leaf

Q125 Which of the following is a viral

disease

इनमहॳसहॳकह९सीएकवायरिबीमारीहहॴ

23-Jan-2017

Options

1) Polio

पह८लियह८ 2) Tetanus

धनसतनभ

3) Leprosy

कषठरह८ग

4) Plague

पिहॳग

Correct Answer Polio

A viral disease (or viral infection)

occurs when an organismrsquos body is

invaded by pathogenic viruses and

infectious virus particles (virions) attach

to and enter susceptible cells

F A C E B O O K

P A G E h t t p w w w f a c e b o o k c o m s s c m e n t o r s o f f i c i a l P a g e | 30

FOR MORE UPDATES AND MORE MATERIAL DO LIKE OUR FACEBOOK PAGE httpwwwfacebookcomsscmentorsofficial

Poliomyelitis often called polio or

infantile paralysis is an infectious

disease caused by the poliovirus

Tetanusmdash A serious bacterial infection

that causes painful muscle spasms and

can lead to death

Leprosy also known as Hansenrsquos

disease (HD) is a long-term infection by

the bacterium Mycobacterium leprae or

Mycobacterium lepromatosis

Plague is an infectious disease caused by

the bacterium Yersinia pestis

Symptoms include fever weakness and

headache

Q126 Which organisms can help to

carry out Vermicomposting

कह९नसाजीववमीकनपह८नसटगममददकरताहहॴ

23-Jan-2017

Options

1) Nitrifying Bacteria

नाईटरीफाईगबहॴकटीररया 2) Earthworms

कहॴ चऐ

3) Algae

िहॴवि

4) Fungus

कवक

Correct Answer Earthworms

Q127 Contraction of heart is also

known as

हदयकहॳ सकचनकह८ mdash- भीकहाजाताहहॴ 23-Jan-

2017

Options

1) Systole

लससटह८ि

2) Aristotle

अरसत

3) Diastole

िायसटह८ि

4) Lub

मयब

Correct Answer Systole

Diastole is the part of the cardiac cycle

when the heart refills with blood

following systole (contraction)

Ventricular diastole is the period during

which the ventricles are filling and

relaxing while atrial diastole is the

period during which the atria are

relaxing

Q128 Azadirachta indica is the

botanical name of which of the

following

अजाटदराचताइडिकानननननलिखितमसहॳककसकावानसपनतनामहहॴ

23-Jan-2017

Options

1) Rose plant

गिाबकापह९धा 2) Apple tree

सहॳबकापहॳड़

3) Neem

नीम

4)Mango

आम

Correct Answer Neem

Q129 Which of the following is the

main end product of carbohydrate

digestion

नननननलिखितमसहॳकह९नसाकाबोहाइडरहॳटकहॳ पाचनकापरमिअतउतपादकहह८ताहहॴ 23-Jan-2017

Options

1) Fats

वसा 2) Lipids

लिपपडस

3) Glucose

गिकह८ज

4) Cellulose

F A C E B O O K

P A G E h t t p w w w f a c e b o o k c o m s s c m e n t o r s o f f i c i a l P a g e | 31

FOR MORE UPDATES AND MORE MATERIAL DO LIKE OUR FACEBOOK PAGE httpwwwfacebookcomsscmentorsofficial

सहॳमयिह८ज

Correct Answer Glucose

Intestinal absorption of end products

from digestion of carbohydrates and

proteins in the pig hellip During absorption some sugars (fructose or

galactose) released from the

corresponding sucrose and lactose

respectively during digestion were

partly metabolized into glucose by the

enterocyte

Q130 Which of the following glands is a

source of the enzyme Ptyalin

नननननलिखितगरचियोमसहॳएजाइमटयालिनकासरह८तहहॴ 23-Jan-2017

Options

1) Pancreas

अगरािय

2) Thyroid Gland

िाइराइिगरिी 3) Pituitary Gland

पीयषगरिी 4) Salivary Glands

िारगरचियाा Correct Answer Salivary Glands

Q131 Which of the following is not true

about Pteridophyta

ननननमसहॳकह९नसीबातटहॳररिह८फाईटकहॳ बारहॳमसचनहीहहॴ 23-Jan-2017

Options

1) Dominant phase is saprophytes

परमिचरणसहॳपरह८फाईइटसहह८ताहहॴ 2) Main plant body is diploid

पह९दह८कामखयिरीरदपवगखणतहह८ताहहॴ 3) Seeds are present

बीजमह९जदहह८तहॳहहॴ 4)Flowers are absent

फिअनपनसतिहह८तहॳहहॴ

Correct Answer Seeds are present

Q132 The largest dolphin species is the

orca also called as

िॉिकफनकीसबसहॳबड़ीपरजानतकाकानामआकायहहॴनजसहॳ mdash- भीकहतहॳहहॴ 23-Jan-2017

Options

1) Bottle Nose

बाटिनह८ज

2) Baiji

बहॳजी 3) Killer whale

ककिरहहॳि

4)Tucuxi

टकवसी Correct Answer Killer whale

Q133 The fat digesting enzyme Lipase

is secreted by which of the following

वसाकापाचनकरनहॳवािाएजाइमिाइपहॳजनननननलिखितमसहॳककसकहॳ दवारासतरापवतहह८ताहहॴ

24-Jan-2017

Options

1) Kidneys

गद

2) Pancreas

अगनयािय

3) Large Intestine

बड़ीआत

4)Liver

नजगर

Correct Answer Pancreas

Lipase is an enzyme that splits fats so

the intestines can absorb them Lipase

hydrolyzes fats like triglycerides into

their component fatty acid and glycerol

molecules It is found in the blood

gastric juices pancreatic secretions

intestinal juices and adipose tissues

F A C E B O O K

P A G E h t t p w w w f a c e b o o k c o m s s c m e n t o r s o f f i c i a l P a g e | 32

FOR MORE UPDATES AND MORE MATERIAL DO LIKE OUR FACEBOOK PAGE httpwwwfacebookcomsscmentorsofficial

Q134 The arrangement of leaves on an

axis or stem is called

एकअकषयातनहॳपरपनततयोकीयवसिाकह८कयाकहाजाताहहॴ SSC CHSL Science (biology) 2016

Question Paper

24-Jan-2017

Options

1) Phyllotaxy

फाइिह८टहॴकसी 2) Vernation

वनिन

3) Venation

वहॳनहॳिन

4)Phytotaxy

फाइटह८टहॴकसी Correct Answer Phyllotaxy

In botany phyllotaxis or phyllotaxy is

the arrangement of leaves on a plant

stem (from Ancient Greek phyacutellon

ldquoleafrdquo and taacutexis ldquoarrangementrdquo)

Phyllotactic spirals form a distinctive

class of patterns in nature

Q135 The study of Cells is also known

as

कह८लिकाओकहॳ अधययनकह८ mdashmdashndash

भीकहाजाताहहॴ 24-Jan-2017

Options

1) Cytology

सायटह८िह८जी 2) Physiology

कफनजयह८िह८जी 3) Nucleology

नयककमयह८िह८जी 4)Cellology

सहॳिह८िह८जी Correct Answer Cytology

Q136 Which of the following scientists

is also known as the Father of Biology

नननननलिखितमसहॳककसवहॴजञाननककह८ ldquoजीवपवजञानकहॳ जनकrdquoकहॳ नामसहॳभीजानाजाताहहॴ 24-Jan-2017

Options

1) Herbert Spencer

हबयटयसपसर

2) Aristotle

अरसत 3) Lamarck

िहॳमाकय 4)Darwin

िापवयन

Correct Answer Aristotle

Q137 Which cells give rise to various

organs of the plant and keep the plant

growing

कह९नसीकह८लिकाएपह९धह८कहॳ लभननअगह८कह८जनमदहॳतीहहॴऔरपह९धह८कह८बढ़नहॳममददकरतीहहॴ

24-Jan-2017

Options

1) Permanent

सिायी 2) Dermal

तवचीय

3) Meristematic

मररसटहॳमटटक

4)Mature

परह८ढ़

Correct Answer Meristematic

A meristem is the tissue in most plants

containing undifferentiated cells

(meristematic cells) found in zones of

the plant where growth can take place

Q138 Rodentia Muridae is the scientific

name of

F A C E B O O K

P A G E h t t p w w w f a c e b o o k c o m s s c m e n t o r s o f f i c i a l P a g e | 33

FOR MORE UPDATES AND MORE MATERIAL DO LIKE OUR FACEBOOK PAGE httpwwwfacebookcomsscmentorsofficial

रह८िहॳलियानयररिी mdashmdash- कावहॴजञाननकनामहहॴ 24-

Jan-2017

Options

1) Mouse

चहा 2) Squirrel

चगिहरी 3) Monkey

बदर

4) Lizard

नछपकिी Correct Answer Mouse

Q139 Name the scientist who proposed

the cell theory

कह८लिकालसदातकापरसतावदहॳनहॳवािहॳवहॴजञाननककानामबताइए 24-Jan-2017

Options

1) Schleiden and Schwann

िीमिनऔरशरववान

2) Lamarck

िहॳमाकय 3) Treviranus

टरहॳवायरहॳनस

4)Whittaker and Stanley

हीटकरऔरसटहॳनिहॳ Correct Answer Schleiden and

Schwann

Q140 The flower with the worldrsquos

largest bloom is

दननयाकासबसहॳबड़ाफिखििनहॳवािा mdashmdashndash हहॴ 24-Jan-2017

Options

1) Pando

पािह८ 2) Posidonia

पह८सीिह८ननया 3) Rafflesia arnoldii

ररफिहॳलियाअनोमिी 4)Helianthus annuus

हहॳलिएनिसएनयअस

Correct Answer Rafflesia arnoldii

Rafflesia arnoldii is a species of

flowering plant in the parasitic genus

Rafflesia It is noted for producing the

largest individual flower on earth It has

a very strong and horrible odour of

decaying flesh earning it the nickname

ldquocorpse flower

Q141 Deficiency of which vitamin

causes night blindness

ककसपवटालमनकीकमीकहॳ कारणरतौधीहह८ताहहॴ 24-Jan-2017

Options

1) Vitamin K

पवटालमन K

2) Vitamin C

पवटालमन C

3) Vitamin B1

पवटालमन B1

4)Vitamin A

पवटालमन A

Correct Answer Vitamin A

Q142 Nongreen plants lack which of the

following

गहॴर-

हररतवनसपनतमनननननलिखितमसहॳककसकीकमीहह८तीहहॴ

24-Jan-2017

Options

1) Chlorophyll

किह८रह८कफि

2) Lycophyll

िायकह८कफि

3) Cyanophyll

F A C E B O O K

P A G E h t t p w w w f a c e b o o k c o m s s c m e n t o r s o f f i c i a l P a g e | 34

FOR MORE UPDATES AND MORE MATERIAL DO LIKE OUR FACEBOOK PAGE httpwwwfacebookcomsscmentorsofficial

सायनह८कफि

4)Phototropism

फह८टह८टरोपपजम

Correct Answer Chlorophyll

Q143 Organisms that use light to

prepare food are known as

जह८जीवपरकािकाउपयह८गकरभह८जनतहॴयारकरतहॳहहॴ उनह mdashmdash- कहॳ पमजानजाताहहॴ 24-Jan-2017

Options

1) Autotrophs

सवपह८षी 2) Heterotrophs

पवषमपह८षज

3) Omnivores

सवायहारी 4)Decomposers

पवघटनकरनहॳवािा Correct Answer Autotrophs

autotrophs often make their own food

by using sunlight carbon dioxide and

water to form sugars which they can use

for energy Some examples of

autotrophs include plants algae and

even some bacteria Autotrophs

(producer) are important because they

are a food source for heterotrophs

(consumers)

A heterotroph is an organism that

ingests or absorbs organic carbon

(rather than fix carbon from inorganic

sources such as carbon dioxide) in order

to be able to produce energy and

synthesize compounds to maintain its

life Ninety-five percent or more of all

types of living organisms are

heterotrophic including all animals and

fungi and some bacteria

Q144 Which of the following is a

primary function of haemoglobin

नननननलिखितमसहॳकह९नसाटहमह८गिह८बबनकाएकपरािलमककाययहहॴ

25-Jan-2017

Options

1) Utilization of energy

उजायकाउपयह८गकरना 2) Prevention of anaemia

रकतामपताहह८नहॳसहॳरह८कना 3) Destruction of bacteria

बहॴकटीररयाकापवनािकरना 4) To transport oxygen

ऑकसीजनकावहनकरना Correct Answer To transport oxygen

Q145 Vascular bundles are absent in

सवहनीबिि mdashmdash- मअनपनसतिरहतहॳहहॴ 25-Jan-2017

Options

1) Bryophyta

िायह८फाइटा 2) Pteridophyta

टहॳररिह८फाईटा 3) Gymnosperms

नजननह८सपमय 4) Angiosperms

एननजयह८सपहॳनसय Correct Answer Bryophyta

Q146 Sauria Lacertidae is the scientific

name of

सहॴररयािहॳसरटाईिी mdashmdashndash कावहॴजञाननकनामहहॴ 25-Jan-2017

Options

1) Crocodile

मगरमचछ

2) Hippopotamus

टहपपह८पह८टहॳमस

3) Lizard

नछपकिी 4) House fly

F A C E B O O K

P A G E h t t p w w w f a c e b o o k c o m s s c m e n t o r s o f f i c i a l P a g e | 35

FOR MORE UPDATES AND MORE MATERIAL DO LIKE OUR FACEBOOK PAGE httpwwwfacebookcomsscmentorsofficial

घरहॳिमकिी Correct Answer Lizard

Q147 Which type of pathogen causes

the water-borne disease SARS (Severe

Acute Respiratory Syndrome)

ककसपरकािकारह८गज़नकजिजननतबीमारीसासयकाकारणबनताहहॴ 25-Jan-2017

Options

1) Viral

वायरि

2) Parasitic

परजीवी 3) Protozoan

परह८टह८जअन

4) Bacterial

बहॴकटीररयि

Correct Answer Viral

Q148 Which of the following organs

produces the enzyme lipase

नननननलिखितमसहॳकह९नसाअगिायपहॳजएजाइमउतपननकरताहहॴ 25-Jan-2017

Options

1) Pancreas

अगनयािय

2) Large Intestine

बड़ीआत

3) Liver

नजगर

4) Small Intestine

छह८टीआत

Correct Answer Pancreas

Q149 A is a long internode forming the

basal part or the whole of a peduncle

एक mdashmdash- एकिबाइटरनह८िहहॴ जह८ननचिाटहससायासनपणयिठिबनताहहॴ 25-

Jan-2017

Options

1) Rhizome

परकद

2) Rachis

महॳ दि

3) floral axis

पषपअकष

4) Scape

भगदड़

Correct Answer scape

Q150 ndash Which of the following

organisms are considered to be both

Living and Non-living

नननननलिखितमसहॳकह९नसहॳजीवाणकह८जीपवतऔरअजीपवतमानाजाताहहॴ

25-Jan-2017

Options

1) Bacteria

बहॴकटीररया 2) Fungi

कवक

3) Algae

िहॴवाि

4)Virus

वायरस

Correct Answer Virus

They are considered to be living as they

possess a protein coat as a protective

covering DNA as the genetic material

etc

They are said to be non-living as they

can be crystallised and they survive for

billions of years They can tolerate high

temperatures freezing cold

temperatures ultra-violet radiations etc

Q151 Deficiency of fluorine causes

which of the following

फिह८ररनकीकमीकहॳ कारणनननननलिखितमसहॳकयाहह८ताहहॴ

F A C E B O O K

P A G E h t t p w w w f a c e b o o k c o m s s c m e n t o r s o f f i c i a l P a g e | 36

FOR MORE UPDATES AND MORE MATERIAL DO LIKE OUR FACEBOOK PAGE httpwwwfacebookcomsscmentorsofficial

27-Jan-2017

Options

1) Dental Caries

िटिकहॴ ररज

2) Scurvy

सकवरी 3) Anaemia

रकतामपता 4) Arthritis

गटठया Correct Answer Dental Caries

Q152 In a Punnett Square with the

cross AaBb x AaBb how many Aabb

genotypes would be created

पनहॳटसककायरमिह८स AaBb x AaBb कहॳ साि

ककतनहॳ Aabb जीनह८टाइपबनगहॳ 27-Jan-2017

Options

1) 1

2) 8

3) 2

4) 3

Correct Answer 2

Q153 Which of the following is the

Controlling Center of the Cell

नननननलिखित म सहॳ कह८लिकाका ननयतरण

क दर कह९न हहॴ

27-Jan-2017

Options

1) Nucleus

क दर

2) Plasma

पिाजमा 3) Lysosome

िायसह८सह८म

4) Chromosome

िह८मह८सह८म

Correct Answer Nucleus

The control centre of the cell is the

nucleus in eukaryotic cells The nucleus

contains genetic material in the form of

DNA

Q154 Myopia affects which of the

following organs

मायह८पपयानननननलिखितअगह८मसहॳककसहॳपरभापवतकरताहहॴ

25-Jan-2017

Options

1) Heart

हदय

2) Skin

तवचा 3) Eyes

आािहॳ 4)Mouth

मह

Correct Answer Eyes

Q155 Which of the following bears

flowers

नननननलिखितमसहॳकह९नफिधारणकरताहहॴ

25-Jan-2017

Options

1) Bryophyta

िायह८फाइटा 2) Pteridophyta

टहॳरीिह८फाईटा 3) Gymnosperms

नजननह८सपमय 4)Angiosperms

एननजयह८सपमय Correct Answer Angiosperms

Q156 Oxygenated blood flows out of the

heart through the

ऑकसीजनयकतरकत mdashmdashmdash

कहॳ माधयमसहॳहदयकहॳ बाहरबहताहहॴ 25-Jan-2017

F A C E B O O K

P A G E h t t p w w w f a c e b o o k c o m s s c m e n t o r s o f f i c i a l P a g e | 37

FOR MORE UPDATES AND MORE MATERIAL DO LIKE OUR FACEBOOK PAGE httpwwwfacebookcomsscmentorsofficial

Options

1) Aorta

महाधमनी 2) pulmonary artery

फहॳ फड़हॳकीधमनी 3) vena cava

वहॳनाकावा 4)Atrium

चह९क

Correct Answer aorta

Q157 Blood leaving the liver and

moving towards the

heart has a higher concentration of

नजगरसहॳननकिकरहदयकीतरफजानहॳवािहॳरकतम mdashmdashmdashmdash कीउचचसादरताहह८तीहहॴ 27-Jan-2017

Options

1) Lipids

लिपपडस

2) Urea

यररया 3) Bile Pigments

पपततकहॳ रगकरण

4) Carbon dioxide

काबयनिायऑकसाइि

Correct Answer Bile Pigments

Urea is nitrogen containing substance

which is produced in the liver in order

to deal with excess amino-acids in the

body As urea is produced it leaves the

liver in the blood stream and passes via

the circulatory system to all parts of the

body

Q158 Bulb is a modification of which

part of a plant

बमबएकपह९धहॳकहॳ ककसटहससहॳकाएक पातरणहह८ताहहॴ 27-Jan-2017

Options

1) The root

जड़

2) The stem

तना 3) The radicle

मिाकर

4)The fruit

फि

Correct Answer The stem

Q159 Which of the following carries

blood away from the heart to different

body parts

इनमहॳसहॳकह९नरकतकह८हदयसहॳिरीरकहॳ पवलभननअगह८तकिहॳजातीहहॴ

27-Jan-2017

Options

1) Arteries

धमननया 2) Nerves

तबतरहाए

3) Capillaries

कहॳ लिकाए

4)Veins

नसहॳ Correct Answer Arteries

Q160 The series of processes by which

nitrogen and its compounds are

interconverted in the environment and

in living organisms is called

27-Jan-2017

Options

1)Absorption of Nitrogen

2)Ammonification

3)Nitrogen Fixation

4)Nitrogen Cycle

Correct Answer Nitrogen Cycle

Ammonification or Mineralization is

performed by bacteria to convert

organic nitrogen to ammonia

F A C E B O O K

P A G E h t t p w w w f a c e b o o k c o m s s c m e n t o r s o f f i c i a l P a g e | 38

FOR MORE UPDATES AND MORE MATERIAL DO LIKE OUR FACEBOOK PAGE httpwwwfacebookcomsscmentorsofficial

Nitrification can then occur to convert

the ammonium to nitrite and nitrate

Nitrogen fixation is a process by which

nitrogen in the Earthrsquos atmosphere is

converted into ammonia (NH3) or other

molecules available to living organisms

Q161 BCG vaccine is given to protect

from which of the following

बीसीजीकाटटकानननननलिखितमसहॳककसकहॳ बचावकहॳ लिएटदयाजातहहॴ

27-Jan-2017

Options

1) Jaundice

पीलिया 2) Anaemia

रकतमपता 3) Tuberculosis

कषयरह८ग

4) Polio

पह८लियह८ Correct Answer Tuberculosis

Q162 Parallel venation is found in

समानतरवहॳनहॳिन mdashmdashmdash- मपायाजाताहहॴ 27-Jan-2017

Options

1) plants which are monocots

पह९धहॳजह८एकबीजपतरीहह८तहॳहहॴ 2) plants which have a dicot stem

वहॳपह९धहॳनजनकातनादपवदलियहह८ताहहॴ 3) plants with leaves similar to Tulsi

वहॳपह९धहॳनजनकीपनततयतिसीकीपनततयोकहॳ समानहह८तहॳहहॴ 4)plants with tap roots

टहॳप टवािहॳपह९धहॳ Correct Answer plants which are

monocots

Q163 The hardest part of the body is

िरीरकासबसहॳकठह८रभाग mdashndash हहॴ 27-Jan-2017

Options

1) Bones

हडडिय

2) Tooth Enamel

दातकहॳ इनहॳमि

3) Skull

िह८पड़ी 4) Spinal Cord

महॳ रजज

Correct Answer Tooth Enamel

Q164 Which type of pathogen causes

the waterborne disease E coli Infection

ककसपरकारकारह८गजननकजिजननतरह८गईकह८िाईसिमणकाकारणबनताहहॴ 27-Jan-2017

Options

1) Protozoan

परह८टह८जआ

2) Parasitic

परजीवी 3) Bacterial

बहॴकटीररयि

4)Viral

वायरि

Correct Answer Bacterial

Q165 The amount of blood filtered

together by both the kidneys in a 70 kg

adult male human in a minute is

70 की गरा वािहॳएकवयसकप षमएकलमनटमदह८नोगदकहॳदवाराएकसािचाबनीगयीरकतकीमातरहह८तीहहॴ 29-Jan-2017

Options

1) 1100 ml

1100 लमलि

2) 100 ml

F A C E B O O K

P A G E h t t p w w w f a c e b o o k c o m s s c m e n t o r s o f f i c i a l P a g e | 39

FOR MORE UPDATES AND MORE MATERIAL DO LIKE OUR FACEBOOK PAGE httpwwwfacebookcomsscmentorsofficial

100 लमलि

3) 1500 ml

1500 लमलि

4) 500 ml

500 लमलि

Correct Answer 1100 ml

Q166 Which feature of a plant helps to

distinguish a monocot from a dicot

पह९धहॳकीवहकह९नसीपविहॳषताहहॴजह८एकदपवदलियहॳऔरएकएकदिीयपह९धहॳसहॳभहॳदकरनहॳममददकरतीहहॴ 29-Jan-2017

Options

1) Pollination

परागम

2) Venation

वहॳनहॳिन

3) Vernation

वनिन

4) Aestivation

एसटीवहॳिहॳन

Correct Answer venation

Q167 The Mutation Theory was

proposed by

उतवररवतयनकालसदात mdashmdashndash

कहॳ दवरापरसतापवतककयाजाताहहॴ 29-Jan-2017

Options

1) Charles Lyell

चामसयलियहॳि

2) William Smith

पवलियमनसमि

3) Hugo De Vries

हयगह८िीराईस

4)Harrison Schmitt

हहॳरीसननसमट

Correct Answer Hugo De Vries

Q168 Which type of pathogen causes

the waterborne disease HepatitisA

ककसपरकारकहॳ रह८गजनकजिजननतरह८गहहॳपहॳटाइटटस-A काकारणबनताहहॴ

29-Jan-2017

Options

1) Parasitic

परजीवी 2) Viral

वायरि

3) Protozoan

परह८टह८जआ

4) Bacterial

बहॴकटीररयि

Correct Answer Viral

Q169 In a Punnett Square with the

cross AaBb x Aabb how many AaBb

genotypes would be created

पनहॳटसकवायरमिह८स AaBb x Aabb

कहॳ सािककतनहॳ AaBb जीनह८टाइपबनगहॳ 29-Jan-

2017

Options

1) 4

2) 1

3) 7

4) 6

Correct Answer 4

Q170 Arboreal Ateles is the scientific

name of

अिह८ररयिएटटलिस mdashmdashmdash कावहॴजञाननकनामहहॴ 29-Jan-2017

Options

1) Squirrel

चगिहरी 2) Sparrow

गह८रहॴया 3) Lizard

नछपकिी 4) Spider monkey

F A C E B O O K

P A G E h t t p w w w f a c e b o o k c o m s s c m e n t o r s o f f i c i a l P a g e | 40

FOR MORE UPDATES AND MORE MATERIAL DO LIKE OUR FACEBOOK PAGE httpwwwfacebookcomsscmentorsofficial

मकड़ीबदर

Correct Answer Spider monkey

Q171 Which type of pathogen causes

the waterborne disease Salmonellosis

ककसपरकारकारह८गाणजिजननतबीमारीसािमह८नहॳिह८लसज़काकारकहहॴ

29-Jan-2017

Options

1) Algal

िहॳवालियहॳ 2) Parasitic

परजीवी 3) Bacterial

बहॴकटीररयि

4)Viral

वायरि

Correct Answer Bacterial

An infection with salmonella bacteria

commonly caused by contaminated food

or water

Symptoms include diarrhoea fever

chills and abdominal pain

Q172 is a condition in which there is a

deficiency of red cells or of haemoglobin

in the blood

mdashmdash-

एकनसिनतहहॴनजसमहॳरकतमिािकह८लिकाओकीयाहीमह८गिह८बबनकीकमीहह८तीहहॴ 29-Jan-2017

Options

1) Albinism

एनमबननजम

2) Propyria

परह८पीररया 3) Anaemia

एनीलमया 4)Keloid disorder

कहॳ िह८इिडिसओिर

Correct Answer Anaemia

Q173 Ananas comosus is the scientific

name of

Options

अनानासकह८मह८सस mdashmdashmdashndash

कावहॴजञाननकनामहहॴ 29-Jan-2017

1) Custard Apple

सीताफि

2) Pineapple

पाइनएपपि

3) Bamboo

बास

4)Pomegranate

अनार

Correct Answer Pineapple

Q174 Which organ produces insulin

कह९नसाअगइनसलिनपहॴदाकरताहहॴ 29-Jan-

2017

Options

1) Liver

यकत

2) Thyroid gland

िायराइिगरिी 3) Spleen

पिीहा 4)Pancreas

अगरयिय

Correct Answer Pancreas

Q175 Which of the following disease is

not caused by water pollution

नननननलिखितमसहॳकह९नसारह८गपानीकहॳ परदषणकहॳकारणनहीहह८ता

29-Jan-2017

Options

1) Cholera

हहॴजा 2) Typhoid

F A C E B O O K

P A G E h t t p w w w f a c e b o o k c o m s s c m e n t o r s o f f i c i a l P a g e | 41

FOR MORE UPDATES AND MORE MATERIAL DO LIKE OUR FACEBOOK PAGE httpwwwfacebookcomsscmentorsofficial

टाइफाइि

3) Asthma

दमा 4)Diarrhoea

दसत

Correct Answer Asthma

Q176 Ocimum tenuiflorum is the

scientific name of

ओलिलममटहॳयईफिह८रमइसकावहॴजञाननकनाम mdash

ndash हहॴ 30-Jan-2017

Options

1) Neem

नीम

2) Mango

आम

3) Babul

बबि

4)Tulsi

तिसी Correct Answer Tulsi

Q177 Which gland secretes bile a

digestive fluid

कह९नसीगरिीपपतत एकपाचनतरिपरदािय सरापवतकरतीहहॴ 30-Jan-2017

Options

1) Pancreas

अगनयािय

2) Liver

यकत

3) Thyroid

िायराइि

4) Testes

टहॳनसटस

Correct Answer liver

Q178 In which of the following the

dominant phase is Gametophyte

नननननलिखितमसहॳककसकहॳ परमिचरणयगमकह८दपवधद (Gametophyte)हहॴ 30-Jan-2017

Options

1) Bryophyta

िायह८फाइटा 2) Pteridophyta

टहॳररिह८फाइटा 3) Gymnosperms

नजननह८सपमय 4) Angiosperms

एननजयह८सपमय Correct Answer Bryophyta

Q179 Anaerobic respiration refers to

which of the following

नननननलिखितमसहॳककसहॳअवायवीयशवसनकहाजाताहहॴ

30-Jan-2017

Options

1) Respiration without Oxygen

ऑकसीजनकहॳ बबनाशवसन

2) Respiration with Oxygen

ऑकसीजनकहॳ सािशवसन

3) Respiration without CO2

काबयनिायऑकसाइिकहॳ बबनाशवसन

4) Respiration with CO2

काबयनिायऑकसाइिकहॳ सािशविन

Correct Answer Respiration without

Oxygen

Q180 Which type of pathogen causes

the waterborne disease Cholera

ककसपरकारकारह८गजनकजिजननतरह८गहहॴजाकाकारणबनताहहॴ

30-Jan-2017

Options

1) Algal

िहॴवालियहॳ

F A C E B O O K

P A G E h t t p w w w f a c e b o o k c o m s s c m e n t o r s o f f i c i a l P a g e | 42

FOR MORE UPDATES AND MORE MATERIAL DO LIKE OUR FACEBOOK PAGE httpwwwfacebookcomsscmentorsofficial

2) Bacterial

बहॴकटीररयि

3) Protozoan

परह८टह८जआ

4) Viral

वायरि

Correct Answer Bacterial

Q181 To which class does

Oxyreductases transferases hydrolases

belong

ओकसीररिकटहॳसटरासफरहॳजहॳस

हाइडरह८िहॳसहॳसककसवगयमआतहॳहहॴ 30-Jan-2017

Options

1) Hormones

हारमोस

2) Enzymes

एजाइनस

3) Proteins

परह८टीनस

4) Vitamins

पवटालमनस

Correct Answer Enzymes

Q182 Which of the following is not true

about Gymnosperms

ननननमसहॳकह९नसीबातअनावतबीजीकहॳ बारहॳमसचनहीहहॴ 30-Jan-2017

Options

1) Dominant phase is saprophytes

परमिचरणसहॳपरह८फाइटसहह८ताहहॴ 2) Vascular bundles are absent

सवहनीबििअनपनसितहह८ताहहॴ 3) spores are heterospores

बीजाणहहॳटहॳरह८सपह८रसहह८तहॳहहॴ 4) Flowers are absent

फिअनपनसितहह८तहॳहहॴ

Correct Answer Vascular bundles are

absent

Q183 The name of first mammal clone sheep is

भहॳड़कीपरिमसतनपायीपरनत प (किह८न)

कानामहहॴ 30-Jan-2017

Options

1) Noori

नरी 2) Dolly

िॉिी 3) Louise

िसी 4)Durga

दगाय Correct Answer Dolly

Q184 Which type of pathogen causes

the water-borne disease Typhoid fever

ककसपरकारकारह८गजनकजिजननतरह८गटाइफाइिबिारकाकारणबनताहहॴ 30-Jan-2017

Options

1) Algal

िहॴवािीय

2) Parasitic

परजीवी 3) Protozoan

परह८टह८जनअन

4)Bacterial

बहॴकटीररयि

Correct Answer Bacterial

Q185 In which part of the cell are

proteins made

कह८लिकाकहॳ ककसटहससहॳमपरह८टीनबनायाजाताहहॴ

31-Jan-2017

Options

1) Reticulum

रहॳटटकिम

F A C E B O O K

P A G E h t t p w w w f a c e b o o k c o m s s c m e n t o r s o f f i c i a l P a g e | 43

FOR MORE UPDATES AND MORE MATERIAL DO LIKE OUR FACEBOOK PAGE httpwwwfacebookcomsscmentorsofficial

2) Golgi apparatus

गह८मजीएपहॳरहॳटस

3) Ribosomes

ररबह८सह८नस

4) Lysosome

िायसह८सह८नस

Correct Answer ribosomes

Proteins are produced by stringing

amino acids together in the order

specified by messenger RNA strands

that were transcribed from DNA in the

cell nucleus The process of synthesizing

a protein is called translation and it

occurs on ribosomes in the cytoplasm of

a cell

Q186 Polio is a disease caused by which

of the following

नननननलिखितमसहॳपह८लियह८कीबबमारह८हह८नहॳकाकारणकयाहहॴ

31-Jan-2017

Options

1) Bacteria

बहॴकटीररयि

2) Mosquito

मचछर

3) Virus

वायरस

4) Cockroach

नतिच हॳ Correct Answer Virus

Polio or poliomyelitis is a crippling and

potentially deadly infectious disease It

is caused by the poliovirus

Q187 ndash Hay fever is a sign of which of

the following

हहॳकफवरनननननलिखितमसहॳककसकाएकसकहॳ तहहॴ

31-Jan-2017

Options

1) Old Age

वदावसिा 2) Malnutrition

कपह८सण

3) Allergy

एिनजय 4) Over Work

अतयचधककाययकरना Correct Answer Allergy

Q188 How many chromosomes does a

human cell contain

एकमानवकह८लिकामककतनहॳगणसतरहह८तहॳहहॴ

29-Jan-2017

Options

1) 6

2) 26

3) 46

4) 66

Correct Answer 46

In humans each cell normally contains

23 pairs of chromosomes for a total of

46 Twenty-two of these pairs called

autosomes look the same in both males

and females The 23rd pair the sex

chromosomes differ between males and

females

Q189 Which of the following is not true

about Bryophyta

ननननमसहॳकह९नसीबातिायह८फाइटकहॳ बारहॳमसचनहीहहॴ 31-Jan-2017

Options

1) Dominant phase is gametophytes

परमिचरणगहॳलमतह८फाइटसहह८ताहहॴ 2) Main plant body is haploid

पह९धहॳकामखयिरीरअगखणतहह८ताहहॴ 3) Spores are homospores

बीजाणहह८मह८सफह८रसहह८तहॳहहॴ 4) Flowers are present

फिमह८जदहह८तहॳहहॴ Correct Answer Flowers are present

F A C E B O O K

P A G E h t t p w w w f a c e b o o k c o m s s c m e n t o r s o f f i c i a l P a g e | 44

FOR MORE UPDATES AND MORE MATERIAL DO LIKE OUR FACEBOOK PAGE httpwwwfacebookcomsscmentorsofficial

Q190 Which aquatic animal has

trailing tentacles

ककसजिीयजानवरकहॳ पीछहॳचिनहॳवािहॳटहॳटकिसहह८तहॳहहॴ

31-Jan-2017

Options

1) Sea horse

समदरीघह८िा 2) Corals

मगा 3) Jelly fish

जहॳिीमछिी 4) Star fish

तारामछिी Correct Answer Jelly fish

Jellyfish with its umbrella-shaped bell

and trailing tentacles

Q191 Which type of pathogen causes

the water-borne disease Poliomyelitis

(Polio)

ककसपरकारकारह८गजनकजिजननतरह८गपह८लियह८मायहॳटटस (पह८लियह८) काकारणहहॴ 31-Jan-

2017

Options

1) Parasitic

परजीवी 2) Algal

िहॴवालिय

3) Viral

वायरि

4) Bacterial

बहॴकटीररयि

Correct Answer Viral

Q192 The outer white part of the eye

that protects the inner structures is

आािकाबाहरीसफहॳ दटहससाजह८आतररकसरचनाओकीरकषाकरताहहॴ वह mdashmdashmdash हहॴ 31-Jan-

2017

Options

1) Iris

आयररस

2) Sclera

सकिहॳरा 3) Retina

रहॳटटना 4) Cornea

कह८ननयया Correct Answer Sclera

Q193 Proteins are made up of

परह८टीनकाननमायण mdashndash सहॳहह८ताहहॴ 31-Jan-2017

Options

1) Amino acids

एलमनह८अनि

2) Fatty acids

वसायकतअनि

3) Glucose

गिकह८ज

4)Nucleotides

नयनकियह८टाईिस

Correct Answer Amino acids

Q194 Moringa Oleifera is the scientific

name of

मह८ररगओलिफहॳ रा mdashmdashndash कावहॴजञाननकनामहहॴ 31-Jan-2017

Options

1) Banyan

बरगद

2) Gulmohar

गिमह८हर

3) Amla

आमिा

F A C E B O O K

P A G E h t t p w w w f a c e b o o k c o m s s c m e n t o r s o f f i c i a l P a g e | 45

FOR MORE UPDATES AND MORE MATERIAL DO LIKE OUR FACEBOOK PAGE httpwwwfacebookcomsscmentorsofficial

4) Drumstick

डरमनसटक

Correct Answer Drumstick

Q195 Kidney stones are composed of

गदकीपिरी mdashndash सहॳबनीहह८तीहहॴ 1-Feb-2017

Options

1) Calcium Oxalate

कहॴ नमसयमओकजहॳिहॳट

2) Sodium Chloride

सह८डियमकिह८राइि

3) Magnesium Nitrate

महॳनगनलियमनाइतटरहॳट

4) Calcium Bicarbonate

कहॴ नमियमबायकबोनहॳट

Correct Answer Calcium Oxalate

Q196 ndash Which of the following is not

true about Angiosperms

ननननमसहॳकह९नसीबातआवतबीजीकहॳ बारहॳमसचनहीहहॴ 1-Feb-2017

Options

1) Dominant phase is gametophytes

परमिचरणगहॳलमतह८फाइटहह८ताहहॴ 2) Vascular bundles are present

सवहनीबििमह९जदहह८ताहहॴ 3) Spores are heterospores

बीजाणहहॳटहॳरह८सपह८रसहह८तहॳहहॴ 4) Seeds are covered

बीजढकहॳ हह८तहॳहहॴ Correct Answer Dominant phase is

gametophytes

Q197 All of the following are excretory

(waste) products of animals except

नननननलिखितमसहॳककसएककह८छह८ड़करअनयसभीपराखणयोदवाराउतसनजयतपदाियहहॴ 1-Feb-

2017

Options

1) Uric Acid

यररकएलसि

2) Ammonia

अमह८ननया 3) Carbohydrates

काबोहाइडरहॳट

4) Urea

यररया Correct Answer Carbohydrates

In animals the main excretory products

are carbon dioxide ammonia (in

ammoniotelics) urea (in ureotelics) uric

acid (in uricotelics) guanine (in

Arachnida) and creatine

Q198 RNA is a polymeric molecule

What does RNA stand for

आरएनइएएकबहिकआणहहॴ इसकाकापवय पकयाहहॴ 1-Feb-2017

Options

1) Rado Nuclear Acid

रािह८नयनकियरएलसि

2) Ribo Nucleic Acid

राइबह८नयनकिकएलसि

3) Rhino Nuclear Acid

हाइनह८नयनकियरएलसि

4) Resto Nucleus Acid

रहॳसटह८नयकिीयसएलसि

Correct Answer Ribo Nucleic Acid

Q199 Which organ does detoxification

and produces chemicals needed for

digestion

कह९नसाअगपवषहरणकरताहहॴऔरपाचनकहॳ लिएआवशयकरसायनोकह८पहॴदाकरताहहॴ 1-Feb-

2017

Options

1) Salivary glands

िारगरचिया 2) Pancreas

अगनयािय

F A C E B O O K

P A G E h t t p w w w f a c e b o o k c o m s s c m e n t o r s o f f i c i a l P a g e | 46

FOR MORE UPDATES AND MORE MATERIAL DO LIKE OUR FACEBOOK PAGE httpwwwfacebookcomsscmentorsofficial

3) Thyroid gland

िायराइिगरिी 4) Liver

यकत

Correct Answer Liver

Q200 Psidium guajava is the scientific

name of

लसडियमगआजावा mdashmdash कावहॴजञाननकनामहहॴ 1-

Feb-2017

Options

1) Guava

अम द

2) Mango

आम

3) Bamboo

बास

4) Jack fruit

कटहि

Correct Answer Guava

Q201 Which drug is used as a Blood

Thinner

चधरकह८पतिाकरनहॳकहॳ पमककसदवाकापरयह८गककयाजाताहहॴ

1-Feb-2017

Options

1) Warfarin

वाफर न

2) Tramadol

टरहॳमािह८ि

3) Azithromycin

एनजरह८मायलसन

4) Hydralazine

हाइडरह८िहॳनजन

Correct Answer Warfarin

Q202 Which of the following disease is

caused due to the deficiency of protein

परह८टीनकीकमीकहॳ कारणनननननलिखितमसहॳकह९नसारह८गहह८ताहहॴ 1-Feb-2017

Options

1) Arthritis

गटठया 2) Kwashiorkor

कािीओकय र

3) Goitre

गाइटर

4) Night Blindness

रतह९चध

Correct Answer Kwashiorkor

Q203 A is species of plant that has

adapted to survive in an environment

with little liquid water

mdashmdashndashपह९धहॳकीएकऐसहॳऐसहॳपरजानतहहॴ नजसनहॳकमपानीवािहॳवातावरणमजीपवतरहनहॳकहॳलिएअनकिनहहॴ 1-Feb-2017

Options

1) Xerophyte

म दपवद

2) Hydrophyte

जिीयपादप

3) Mesophyte

समह८दपवद

4) Thallophyte

िहॴिह८फाइटा Correct Answer xerophyte

xerophyte is a species of plant that has

adapted to survive in an environment

with little liquid water such as a desert

or an ice- or snow-covered region in the

Alps or the Arctic

Mesophytes are terrestrial plants which

are adapted to neither a particularly

dry nor particularly wet environment

An example of a mesophytic habitat

would be a rural temperate meadow

F A C E B O O K

P A G E h t t p w w w f a c e b o o k c o m s s c m e n t o r s o f f i c i a l P a g e | 47

FOR MORE UPDATES AND MORE MATERIAL DO LIKE OUR FACEBOOK PAGE httpwwwfacebookcomsscmentorsofficial

which might contain goldenrod clover

oxeye daisy and Rosa multiflora

thallophyte any of a group of plants or

plantlike organisms (such as algae and

fungi) that lack differentiated stems

leaves and roots and that were formerly

classified as a primary division

(Thallophyta) of the plant kingdom

Q204 How many types of teeth are

there in humans

मनषयोमककतनहॳपरकारकहॳ दातहह८तहॳहहॴ

1-Feb-2017

Options

1) 4

2) 5

3) 2

4) 3

Correct Answer 4

teeth -Humans have four types of

teethincisors canines premolars and

molars each with a specific function

The incisors cut the food the canines

tear the food and the molars and

premolars crush the food

Q205 Carica papaya is the scientific name of

कहॴ ररकापपाया mdashmdashndash कावहॴजञाननकनामहहॴ 2-

Feb-2017

Options

1) Peepal

पीपि

2) Papaya

पपीता 3) Tamarind

इमिी 4) Drumstick

ढह८िकाछड़ी Correct Answer Papaya

Q206 Muscles get tired when there is

shortfall of

जब mdashndash कीकमीहह८तीहहॴतबपहॳिीयिकजातीहहॴ 2-Feb-2017

Options

1) Lactic acid

िहॴनकटकएलसि

2) Na+ ions

Na+ आयन

3) ATP

एटीपी 4) Sulphates

समफहॳ टस

Correct Answer ATP

ATP is the energy source muscle fibers

use to make muscles contract

muscle tissuersquos main source of energy

called adenosine triphosphate or ATP

As your muscles use up this energy

source they become tired and fatigued

Oxygen is the key ingredient that helps

create new ATP to replenish the burned

up ATP in your muscles

Q207 Artocarpus integra is the

scientific name of आटह८कापयसइटीगरा mdashmdashmdash कावहॴजञाननकनामहहॴ 2-Feb-2017

Options

1) Guava

अम द

2) Pineapple

अनानास

3) Silver Oak

लसमवरओक

4) Jack fruit

कटहि

Correct Answer Jack fruit

Q208 Which organ stores fat soluble

vitamins

कह९नसाअगवसामघिनिीिपवटालमनह८काभिाराकरताहहॴ

2-Feb-2017

F A C E B O O K

P A G E h t t p w w w f a c e b o o k c o m s s c m e n t o r s o f f i c i a l P a g e | 48

FOR MORE UPDATES AND MORE MATERIAL DO LIKE OUR FACEBOOK PAGE httpwwwfacebookcomsscmentorsofficial

Options

1) Blood

रकत

2) Skin

तवचा 3) Liver

यकत

4) Pancreas

अगनयािय

Correct Answer Liver

Q209 Which disease is caused due to

deficiency of Iodine

आयह८िीनकहॳ कारणकह९नसारह८गहह८ताहहॴ 2-Feb-2017

Options

1) Rickets

ररकहॳ टस

2) Scurvy

सकवी 3) Goitre

गणमािा 4) Growth retardation

पवकासका कना Correct Answer Goitre

rickets A softening and weakening of

bones in children usually due to

inadequate vitamin D

Q210 Grevillea Robusta is the scientific name of

गरहॳपवलियारह८बसटा mdashmdashmdash- कापवजञाननकनामहहॴ 2-Feb-2017

Options

1) Peepal

पीपि

2) Teak

सागह९न

3) Silver Oak

लसमवरओक

4) Jack fruit

कटहि

Correct Answer Silver Oak

Q211 When a Cuttlefish is described as a Molluscs it is at which level of

classification

जबएककटिकफिकह८एकमह८िसकाकहॳ पमवखणयतककयाजाताहहॴतबयहॳवगीकरणकहॳ ककससतरपहॳनसितहहॴ 2-Feb-2017

Options

1) Class

वगय 2) Order

िम

3) Family

पररवार

4) Phylum

सघ

Correct Answer Phylum

Q212 Bambusa dendrocalmus is the

scientific name of बानबसािहॳडराकामस mdashmdashmdash कावहॴजञाननकनामहहॴ 3-Feb-2017

Options

1) Banyan

बरगद

2) Papaya

पपीता 3) Bamboo

बास

4) Pomegranate

अनार

Correct Answer Bamboo

Q213 Acinonyx Jubatus is the scientific name of

एलसनह८ननकसजयबहॳटस mdashmdashmdash

कावहॴजञाननकनामहहॴ 3-Feb-2017

F A C E B O O K

P A G E h t t p w w w f a c e b o o k c o m s s c m e n t o r s o f f i c i a l P a g e | 49

FOR MORE UPDATES AND MORE MATERIAL DO LIKE OUR FACEBOOK PAGE httpwwwfacebookcomsscmentorsofficial

Options

1) Bear

भाि 2) Horse

घह८िा 3) Cheetah

चीता 4) Zebra

जहॳिा Correct Answer Cheetah

Q214 The pale yellow colour of urine is

due to the presence of which pigment

मतरकाफीकापीिारगरगदरयकहॳ उपनसिनतकहॳ कारणहह८ताहहॴ

3-Feb-2017

Options

1) Urochrome

यरह८िह८म

2) Urophyll

यरह८कफि

3) Chlorophyll

किह८रह८कफि

4) Chloroplast

किह८रह८पिासट

Correct Answer Urochrome

Q215 Which of the following constitute

to form a gene

नननननलिखितमसहॳकह९नसीचीज़एकजीनकागठनकरतीहहॴ

3-Feb-2017

Options

1) Polynucleotides

पह८िीनयनकियह८टाईडस

2) Hydrocarbons

हाइडरह८काबोस

3) Lipoproteins

िाईपह८परह८टीनस

4) Lipids

लिपपडस

Correct Answer Polynucleotides

Polynucleotide molecule is a biopolymer

composed of 13 or more nucleotide

monomers covalently bonded in a chain

DNA (deoxyribonucleic acid) and RNA

(ribonucleic acid) are examples of

polynucleotides with distinct biological

function

Q216 Vertebrates belongs to the

phylum

रीढ़कीहडिीवािहॳपराणी mdashmdashmdash

परजानतकहॳ अतगायतआतहॳहहॴ 3-Feb-2017

Options

1) Arthropoda

आरह८पह८ड़ा 2) Annelida

एननलििा 3) Cnidaria

ननिहॳररया 4) Chordata

कह८िटा Correct Answer Chordata

Q217 Punica granatum is the scientific name of

पननकगरहॳनहॳटस mdashmdashmdash कावहॴजञाननकनामहहॴ 3-Feb-2017

Options

1) Custard Apple

सीताफि

2) Gulmohar

गिमह८हर

3) Silver Oak

लसमवरओक

4) Pomegranate

अनार

Correct Answer Pomegranate

F A C E B O O K

P A G E h t t p w w w f a c e b o o k c o m s s c m e n t o r s o f f i c i a l P a g e | 50

FOR MORE UPDATES AND MORE MATERIAL DO LIKE OUR FACEBOOK PAGE httpwwwfacebookcomsscmentorsofficial

Q218 Between a tiger and an monkey

which of the following is different

एकबाघऔरबदरकहॳ बीचनननननलिखितमसहॳकह९नसीबातअिगहहॴ 3-Feb-2017

Options

1) Kingdom

राजय

2) Phylum

जानत

3) Order

िम

4) Class

वगय Correct Answer order

Q219 The artificial heart was invented by

कबतरमहदयका mdashmdashmdash

दवाराअपवषकारककयागयािा 3-Feb-2017

Options

1) Muhammad Yunus

महनमदयनस

2) Linus Yale Jr

िाइनसयहॳिजय

3) Gazi Yasargil

गाजीयासचगयि

4) Paul Winchell

पह९िपवमकि Correct Answer Paul Winchell

Q220 Tamarindus indica is the

scientific name of

टहॳमररनडसइडिका mdashmdash कावहॴजञाननकनामहहॴ 7-

Feb-2017

Options

1) Neem

नीम

2) Pineapple

अनानास

3) Tamarind

इमिी 4)Chiku

चीक

Correct Answer Tamarind

Q221 In eukaryotic cells synthesis of

RNA takes place in the

यकहॳ योटटककह८लिकाओमआरएनएकासशिहॳषण

mdashndash महह८ताहहॴ 7-Feb-2017

Options

1) Mitochondria

माईटह८कोडडरया 2) Centrioles

सटरीयह८मस

3) Ribosomes

ररबह८सह८नस

4) Nucleus

नयनकियस

Correct Answer nucleus

eukaryotic cell -Transcription is the

process of synthesizing ribonucleic acid

(RNA)Synthesis takes place within the

nucleus of eukaryotic cells or in the

cytoplasm of prokaryotes and converts

the genetic code from a gene in

deoxyribonucleic acid ( DNA ) to a

strand of RNA that then directs

proteinsynthesis

Q222 _________is caused by parasites

of the Plasmodium genus

पिाजमह८डियमजातीकहॳ परजीवी mdash- कहॳ कारणहहॴ 7-Feb-2017

Options

1) Dysentery

पहॳचचि

2) Malaria

मिहॳररया 3) Chickenpox

F A C E B O O K

P A G E h t t p w w w f a c e b o o k c o m s s c m e n t o r s o f f i c i a l P a g e | 51

FOR MORE UPDATES AND MORE MATERIAL DO LIKE OUR FACEBOOK PAGE httpwwwfacebookcomsscmentorsofficial

चहॳचक

4) Herpes

हहॳपपयस

Correct Answer Malaria

Q223 Carotene in fruits and vegetables

gives it which color

फिह८औरसनलजयोमनसितकहॳ रह८टीनउनहकह९नसारगपरदानकरताहहॴ 7-Feb-2017

Options

1) Green

हरा 2) Pink

गिाबी 3) Orange

नारगी 4) Blue

नीिा Correct Answer Orange

Q224 Equus Caballus is the scientific

name of

एकवसकहॴ बहॳिस mdashmdashndash कापवजञाननकनामहहॴ 7-Feb-2017

Options

1) Horse

घह८िा 2) Zebra

ज़हॳिा 3) Donkey

गधा 4) Buffalo

भस

Correct Answer Horse

Q225 Elapidae Naja is the scientific name of

एिीपीिीनाजा mdashmdash- कावहॴजञाननकनामहहॴ 8-Feb-2017

Options

1) Cobra

कह८बरा 2) Elephant

हािी 3) Eagle

ग ि

4) Owl

उमि Correct Answer Cobra

Q226 Which disease is caused due to

deficiency of Iron

िह८हकीकमीकहॳ कारणकह९नसारह८गहह८ताहहॴ 8-Feb-

2017

Options

1) Beriberi

बहॳरीबहॳरी 2) Tetany

टहॳटनी 3) Kwashiorkor

कवािीऔरकर

4) Anaemia

रकतामपता Correct Answer Anaemia

Beriberi is a disease caused by a vitamin

B-1 deficiency also known as thiamine

deficiency

Tetany can be the result of an

electrolyte imbalance Most often itrsquos a

dramatically low calcium level also

known as hypocalcemia Tetany can also

be caused by magnesium deficiency or

too little potassium Having too much

acid (acidosis) or too much alkali

(alkalosis) in the body can also result in

tetany

Kwashiorkor also known as

ldquoedematous malnutrition It is a form of

malnutrition caused by a lack of protein

in the diet

Anaemia means that you have fewer red

blood cells than normal or you have less

F A C E B O O K

P A G E h t t p w w w f a c e b o o k c o m s s c m e n t o r s o f f i c i a l P a g e | 52

FOR MORE UPDATES AND MORE MATERIAL DO LIKE OUR FACEBOOK PAGE httpwwwfacebookcomsscmentorsofficial

haemoglobin than normal in each red

blood cell

Q227 is a leaf where the leaflets are

arranged along the middle vein

mdashndashएकपततीहहॴजहापतरकह८कीरचनाक ररयालिराकहॳ आसपासहह८तीहहॴ 8-Feb-2017

Options

1) Pinnately compound leaf

पपनहॳटिीसयकतपतती 2) Palmately compound leaf

पामहॳटिीसयकतपतती 3) Compound leaf

सयकतपतती 4) Simple leaf

साधारणपतती Correct Answer Pinnately compound

leaf

Q228 Haustoria or sucking roots are

found in which of the following

हह८सटह८ररयायाचसनहॳवािीजड़हॳनननननलिखितमसहॳककसमपाईजातीहहॴ 8-Feb-2017

Options

1) Wheat

गहॳह

2) Mango

आम

3) Chestnut

चहॳसटनट

4) Cuscuta

कसकयटा Correct Answer Cuscuta

Haustorial roots -The roots of parasitic

plants which penetrate into the host

tissues to absorb nourishment are

called haustorial roots hellip Also known as suckingor parasitic roots

Q229 Equs Asinus is the scientific name

of

एकवसएलसनस mdashmdashndash कावहॴजञाननकनामहहॴ 8-

Feb-2017

Options

1) Donkey

गधा 2) Cow

गाय

3) Deer

टहरन

4) Kangaroo

कगा

Correct Answer Donkey

Q230 Ficus benghalensis is the scientific name of

फाईकसबहॳनगहॳिहॳलसस mdashndash कापवजञाननकनामहहॴ 8-Feb-2017

Options

1) Banyan

बरगद

2) Pineapple

अनानास

3) Babul

बबि

4) Tulsi

तिसी Correct Answer Banyan

Q231 Equus burchellii is the scientific name of

एकवसबचिी mdashmdash- कापवजञाननकनामहहॴ 8-Feb-2017

Options

1) Horse

घह८िा 2) Zebra

जहॳिा 3) Buffalo

F A C E B O O K

P A G E h t t p w w w f a c e b o o k c o m s s c m e n t o r s o f f i c i a l P a g e | 53

FOR MORE UPDATES AND MORE MATERIAL DO LIKE OUR FACEBOOK PAGE httpwwwfacebookcomsscmentorsofficial

भस

4) Ass

गधा Correct Answer Zebra

Page 2: COMPILATION OF ALL 72 SETS OF BIOLOGY SSC CHSL-2016 · OF BIOLOGY SSC CHSL-2016 PREPARED BY : SSC MENTORS BIOLOGY SPECIAL . F A C E B O O K P A G E : h t t p : / / w w w . f a c e

F A C E B O O K

P A G E h t t p w w w f a c e b o o k c o m s s c m e n t o r s o f f i c i a l P a g e | 1

FOR MORE UPDATES AND MORE MATERIAL DO LIKE OUR FACEBOOK PAGE httpwwwfacebookcomsscmentorsofficial

For more materials you can

visit our store

httpswwwinstamojocomS

SCMentors

Facebook group_for SSC

CPO amp CGL 2018

httpswwwfacebookcompg

sscmentorsofficialgroupsref

=page_internal

official telegram channel

httpstmesscmentorsofficial

Facebook page

httpwwwfacebookcomssc

mentorsofficial

F A C E B O O K

P A G E h t t p w w w f a c e b o o k c o m s s c m e n t o r s o f f i c i a l P a g e | 2

FOR MORE UPDATES AND MORE MATERIAL DO LIKE OUR FACEBOOK PAGE httpwwwfacebookcomsscmentorsofficial

There were 231 questions

asked in SSC CHSL Science

(biology) 2016 Question Paper

Important questions are

explained here also Q1 Diabetes is caused by

मधमहॳह हह८न का कारणहहॴ

7-Jan -2017

Options

1) Excess of insulin

इनसलिनकीबहिता 2) Low production of Insulin

इनसलिनकाकमउतपादन

3) Malfunction of liver

यकतकहॳ काययमगड़बड़ीहह८ना 4) Higher production of bilirubin

बबि बबनकाउचचतरउतपादन

Correct Answer Low production of

Insulin

Diabetes is caused by the immune

system destroying the cells in the

pancreas that make insulin This causes

diabetes by leaving the body without

enough insulin to function normally

Q2 Tectona grandis Linn is the

scientific name of

टहॳकटह८नागहॴननिसलिन mdashmdashndash कावहॴजञाननकनामहहॴ 7-Jan -2017

Options

1)Guavav

अम द

2)Teak

सागह९न

3) Amla

आविा 4) Chiku

चचक

Correct Answer Teak

Q3 Sea-Anemones belongs to the

phylum

सी-एननमोस mdashmdash- परजानतकहॳ अतगयतआतहॳहहॴ 7-Jan -2017 Options

1)Arthropoda

अनरह८पह८ड़ा 2)Cnidaria

ननड़हॳररया 3) Porifera

पह८ररफहॳ रा 4) Mollusca

मह८िसक

Correct Answer Cnidaria

Sea anemones are a group of marine

predatory animals of the order

Actiniaria They are named after the

anemone a terrestrial flowering plant

because of the colourful appearance of

many Sea anemones are classified in the

phylum Cnidaria

Q4 Which of the following is also

known as a Common Water Hyacinth

नननननलिखितमसहॳककसहॳआमजिकनभभीकहाजाताहहॴ

7-Jan -2017

Options

1)Pistia

पपनसटया 2)Opuntia

ओपलिया 3)Aegilops

एनजिह८पस

4) Echhornia

एकह८ननयया Correct Answer Echhornia

Water hyacinth is a free-floating

perennial aquatic plant (or hydrophyte)

native to tropical and sub-tropical South

America

F A C E B O O K

P A G E h t t p w w w f a c e b o o k c o m s s c m e n t o r s o f f i c i a l P a g e | 3

FOR MORE UPDATES AND MORE MATERIAL DO LIKE OUR FACEBOOK PAGE httpwwwfacebookcomsscmentorsofficial

Q5 Which is the largest organ in

human beings

मानवह८मसबसहॳबड़ाअगकह९नसाहह८ताहहॴ

7-Jan -2017

Options

1) Skin

तवचा 2) Large Intestine

बड़ीआत

3) Small Intestine

छह८टीआत

4) Liver

यकत

Correct Answer Skin

The skin is the largest organ of the

body with a total area of about 20

square feetThe largest internal organ is

the liver The longest bone in the human

body is the femur The largest artery is

the aorta and the largest vein is the

inferior vena cava

Q6 Delonix regia Rafin is the scientific

name of

िहॴिोननकसरनजयारकफन (Delonix regia

Rafin)mdashmdash- कावहॴजञाननकनाम 7-Jan -2017

Options

1) Banyan

बरगद

2) Gulmohar

गिमह८हर

3) Tamarind

इमिी 4) Chiku

चचक

Correct Answer Gulmohar

Delonix regia is a species of flowering

plant in the bean family Fabaceae

Q7 Amoeba belongs to the phylum

अमीबा mdashmdashmdash- परजानतकहॳ अतगयतआताहहॴ

7-Jan -2017

Options

1) Protozoa

परह८टह८जआ

2) Annelida

ऐनहॳलििा 3)Porifera

पह८ररफहॳ रा 4) Platyhelminthes

पिहॳटटहहॳनममननिस

Correct Answer Protozoa

Q8 Deficiency of which of the following

causes non-clotting of blood

नननननलिखितमसहॳककसकीकमीकहॳ कारणरकतकािककानहीजमता

8-Jan -2017

Options

1) Vitamin C

पवटालमन C

2) Vitamin K

पवटालमन K

3) Vitamin E

पवटालमन E

4) Vitamin B12

पवटालमन B12

Correct Answer Vitamin K

Vitamin C is found in citrus fruits and

vegetables Scurvy results from a

deficiency of vitamin C in the diet

Vitamin E deficiency associated with

this disease causes problems such as

poor transmission of nerve impulses

muscle weakness and degeneration of

the retina that can cause blindness

Vitamin B12 deficiency may lead to a

reduction in healthy red blood cells

Q9 The process of producing energy in

plants is known as

F A C E B O O K

P A G E h t t p w w w f a c e b o o k c o m s s c m e n t o r s o f f i c i a l P a g e | 4

FOR MORE UPDATES AND MORE MATERIAL DO LIKE OUR FACEBOOK PAGE httpwwwfacebookcomsscmentorsofficial

पहॳड़ह८मउजायउतपननकरनहॳकीपरनतकियाकह८ mdashmdashndash

कहॳ नामजाताहहॴ 8-Jan -2017

Options

1) Absorption

अविह८षण

2) Reduction

अवकरण

3) Photosynthesis

परकािसशरिहॳषण

4) Transpiration

वाषपीकरण

Correct Answer Photosynthesis

Q10 Which Virus causes Chicken Pox

ककसवायरसकहॳ कारणचहॳचकहह८ताहहॴ

8-Jan -2017

Options

1) Rubella Virus

बहॳिावायरस

2) Herpes Zoster Virus

हपपरयसजह८सटरवायरस

3) Rabies

रहॳबीज़

4) Variola Virus

वहॳरीओिावायरस

Correct Answer Herpes Zoster Virus

Rubella also known as German

measles or

three ndashday measles is an infection

caused by the rubella virus

Smallpox is caused by infection with

variola

Virus

Q11 What is the total number of bones

in the human body

मानविरीरमकिककतनीहडडियहह८तीहहॴ

8-Jan -2017

Options

1)206

2)103

3)309

4)412

Correct Answer 206

Q12 Emblica officinalis is the scientific

name of

एननलिकाओफीलसनहॳलिस (Emblica

offcinalis)mdashmdash- कावहॴजञाननकनामहहॴ 8-Jan -2017

Options

1) Peepal

पीपि

2) Mango

आम

3) Amla

आविा 4) Drumstick

सहजन

Correct Answer Amla

Q13 Sponges belongs to the phylum

सपजककसपरजानतकहॳ अतगयतआतहहॴ

8-Jan -2017

Options

1) Protozoa

परह८टह८जआ

2) Annelida

एननलििा 3) Porifera

पह८ररफहॳ रा 4) Cnidaria

ननिहॳररया Correct Answer Porifera

Q14 Which of the following is a

symptom of haemophilia

नननननलिखितमसहॳटहमह८फीलियाकािकषणकह९नसा हहॴ

9-Jan -2017

Options

F A C E B O O K

P A G E h t t p w w w f a c e b o o k c o m s s c m e n t o r s o f f i c i a l P a g e | 5

FOR MORE UPDATES AND MORE MATERIAL DO LIKE OUR FACEBOOK PAGE httpwwwfacebookcomsscmentorsofficial

1) Night Blindness

रतोधी 2) No clotting of Blood

रकतकािककानजमना 3) Rickets

ररकहॳ ट

4) Loss of haemoglobin

टहमह८गिह८बबनकीअमपता Correct Answer No clotting of Blood

Haemophilia also spelled hemophilia is

a mostly inherited genetic disorder that

impairs the bodyrsquos ability to make blood

clots a process needed to stop bleeding

This results in people bleeding longer

after an injury easy bruising and an

increased risk of bleeding inside joints

or the brain

Q15 The process of pollination by birds

is also known as

पकषकषयोदवाराकीजानहॳवािीपरागणकीपरनतकियाकह८mdashndash कहॳ नामसहॳभीजानाजाताहहॴ 9-Jan -2017

Options

1) Hydrophily

हाइडरह८कफिी 2) Entomophily

एनटोमह८कफिी 3) Embryophily

एननियह८कफिी 4) Ornithophily

ओननयिह८कफिी Correct Answer Ornithophily

Q16 Spiders belong to the phylum

मकडड़याककसपरजानतकहॳ अतगयतआतीहहॴ 9-Jan -2017

Options

1) Mollusca

मह८िसका 2) Annelida

एननलििा 3) Cnidaria

ननिहॳररया 4) Arthropoda

अरोपह८ड़ा Correct Answer Arthropoda

Q17 Banana freckle is a plant disease

It is caused by a

कहॳ िहॳकीझाईपह८धह८कीएकबीमारीहहॴ यहएक mdashndash

कहॳ कारणहह८तीहहॴ 9-Jan -2017

Options

1) Virus

वायरस

2) Fungus

कवक

3) Bacteria

बहॴकटीररया 4) Insect

कीटक

Correct Answer Fungus

Banana Freckle is a disease caused by

the fungus Guignardia musae

(telomorph) or Phyllosticta musarum (

anamorph )

Q18 Which of the following Indian

chilly is considered one of the hottest in

the world

नननननलिखितभारतीयलमचचययह८मसहॳकह९नसीसबसहॳकह९नसीपवशवकीसबसहॳतीिीलमचचययह८मसहॳएकमानीजातीहहॴ

9-Jan -2017

Options

1) Bhut Jolokia

भतझह८िककया 2) Bhut Mahabora

भतमहाबह८रा 3) Lal Chitin

F A C E B O O K

P A G E h t t p w w w f a c e b o o k c o m s s c m e n t o r s o f f i c i a l P a g e | 6

FOR MORE UPDATES AND MORE MATERIAL DO LIKE OUR FACEBOOK PAGE httpwwwfacebookcomsscmentorsofficial

िािचीटटन

4) Lal Shamak

िाििामक

Correct Answer Bhut Jolokia

Q19 Brain fever is a disease spread

through which of the following

मनसतषकजवरनमकरह८गनननननलिखितमसहॳककसकहॳ कारणहह८ताहहॴ 9-Jan -2017

Options

1) Flies

मनकियो 2) Mosquito

मचछर

3) Virus

वायरस

4) Cockroach

नतिच हॳ Correct Answer Mosquito

Q20 Mangroves are plants that have

मगरह८ववहॳपहॳिहहॴनजनमहॳहह८ताहहॴ 9-Jan -2017

Options

1) Modified Roots

पातररतजड़हॳ 2) Modified Stems

पातररततनहॳ 3) Respiratory Roots

शरवसनकरनहॳवािीजड़हॳ 4) Respiratory Stems

शरवसनकरनहॳवािीतनहॳ Correct Answer Respiratory Roots

A mangrove is a shrub or small tree that

grows in coastal saline or brackish

water

Q21 Rodentia Sciurus is the scientific

name of

रह८िहॳलियासकीयरस mdashmdash कावयजजञाननकनामहहॴ

9-Jan -2017

Options

1) Rat

चहा 2) Platypus

पिहॳटीपस

3) Squirrel

चगिहरी 4) Beaver

बीवर

Correct Answer Squirrel

Q22 Which of the following is induced

by Oncogene

नननननलिखितमसहॳकह९नओकह८जीनदवारापरहॳररयतहह८ताहहॴ

10-Jan -2017

Options

1) Polio

पह८लियह८ 2) Cancer

क सर

3) Diarrhoea

दसत

4) Dengue

िग Correct Answer Cancer

An oncogene is a gene that has the

potential to cause cancer In tumor

cells they are often mutated andor

expressed at high levels

Q23 Azadirachata indica is the

scientific name of

अजाटदराकटाइडिका mdashmdashवहॴजञाननकनामहहॴ SSC CHSL Science (biology) 2016

Question Paper

10-Jan -2017

Options

1) Neem

नीम

F A C E B O O K

P A G E h t t p w w w f a c e b o o k c o m s s c m e n t o r s o f f i c i a l P a g e | 7

FOR MORE UPDATES AND MORE MATERIAL DO LIKE OUR FACEBOOK PAGE httpwwwfacebookcomsscmentorsofficial

2) Teak

सागह९न

3) Silver Oak

लसमवरओक

4) Tulsi

तिसी Correct Answer Neem

Q24 Octopus belongs to the phylum

ऑकटह८पसककसपरजानतकहॳ अतगयतआताहहॴ 10-

Jan -2017

Options

1) Mollusca

मह८िसका 2) Cnidaria

ननिहॳररया 3) Echinodermata

इकाइनह८ड़हॳमता 4) Chordata

कह८िता Correct Answer Mollusca

Q25 A living part of the organisms

environment is known as

जीवाणकहॳ वातावरणकहॳ जीपवतभागकह८ mdash-

कहतहॳहहॴ 10-Jan -2017

Options

1) Abiotic Factor

अजहॴपवककारक

2) Habitat

आवास

3) Biotic Factor

जहॴपवककारक

4) Nonliving factor

अ-जीपवतकारक

Correct Answer Biotic Factor

Abiotic factors are nonndash living chemical

and physical parts of the environment

that affect living organisms and the

functioning of ecosystems like rain

wind temperature altitude soil

pollution nutrients pH types of soil

and sunlight

Q26 Medulla oblongata is a part of

which of the following

महॳडयिाऑबिॉनगहॳटानननननलिखितमसहॳककसअगकाटहससाहहॴ

10-Jan -2017

Options

1) Heart

हदय

2) Brain

मनसतषक

3) Lungs

फहॳ फड़हॳ 4) Stomach

पहॳट

Correct Answer Brain

The medulla oblongata helps regulate

breathing heart and blood vessel

function digestion sneezing and

swallowing This part of the brain is a

center for respiration and circulation

Sensory and motor neurons (nerve cells)

from the forebrain and midbrain travel

through the medulla

Q27 ___________ is a typically

onecelled reproductive unit capable of

giving rise to a new individual without

sexual fusion

mdashmdash एकआमतह९रपरएककह८लिकीयहॳ परजननममसमकषइकाईहहॴजह८यह९नसियनकहॳ बबनाएकनयीइकाईकह८जनमदहॳतीहहॴ 10-Jan -2017

Options

1) Egg

अिाण

2) Spore

बीजाण

F A C E B O O K

P A G E h t t p w w w f a c e b o o k c o m s s c m e n t o r s o f f i c i a l P a g e | 8

FOR MORE UPDATES AND MORE MATERIAL DO LIKE OUR FACEBOOK PAGE httpwwwfacebookcomsscmentorsofficial

3) Sperm

ििाण

4) Seed

बीज

Correct Answer Spore

Q28 Bacteria was discovered by

बहॴकटीररयाकीिह८जककसकहॳ दवाराकीगयीिी

10-Jan -2017

1) Antonie van Leeuwenhoek

एटह८नीवहॳनलिबहॳनहक

2) Belarus

बहॳिा स

3) Hugo de Vries

हयगह८दीराईस

4)Robert Brown

रॉबटयिाउन

Correct Answer Antonie van

Leeuwenhoek

Q29 Which of the following is

responsible for Vermicomposting

नननननलिखितमसहॳकह९नकलमिादकहॳ लिएनजनमहॳदारहहॴ

10-Jan -2017

Options

1) Fungus

कवक

2) Worms

कलम

3) Bacteria

बहॴकटीररया 4) Birds

पकषी Correct Answer Worms

Vermicompost (or vermi-compost) is the

product of the composting process using

various species of worms usually red

wigglers white worms and other

earthworms to create a heterogeneous

mixture of decomposing vegetable or

food waste bedding materials and

vermicast

Q30 Scurvy (bleeding of gums) is

caused by the deficiency of which

vitamin

सकवी (मसढह८सहॳिनआना) ककसपवटालमनकीकमीकहॳ कारणहह८ताहहॴ

10-Jan-2017

Options

1) Vitamin K

पवटालमन K

2) Vitamin BZ

पवटालमन BZ

3) Vitamin C

पवटालमन C

4) Vitamin A

पवटालमन A

Correct Answer Vitamin C

Q31 Achras sapota is the scientific

name of

एिाससपह८ताइसकावहॴजञाननकनामहहॴ 10-Jan-2017

Options

1) Custard Apple

सीताफि

2) Gulmohar

गिमह८हर

3) Tamarind

इमिी 4) Chiku

चचक

Correct Answer Chiku

Q32 Prawn belongs to the phylum

झीगा mdashmdash- परजानतकहॳ अतगयतआताहहॴ 10-Jan-2017

Options

1) Arthropoda

F A C E B O O K

P A G E h t t p w w w f a c e b o o k c o m s s c m e n t o r s o f f i c i a l P a g e | 9

FOR MORE UPDATES AND MORE MATERIAL DO LIKE OUR FACEBOOK PAGE httpwwwfacebookcomsscmentorsofficial

अरोपह८िा 2) Cnidaria

नीिहॳररया 3) Echinodermata

इकाईनह८िमटा 4) Chordata

कह८िटा Correct Answer Arthropoda

Q33 Pulses are a rich source of which of

the following

दािहॳनननननलिखितमसहॳककसकीपरचरसह८तरहहॴ

11-Jan-2017

Options

1) Carbohydrates

काबोहाइडराईट

2) Proteins

परह८टीनस

3) Minerals

िननज

4) Vitamin A

पवटालमन A

Correct Answer Proteins

Q34 Plant cell wall is made up of

वनसपनतकह८लिकालभनततइससहॳबनीहह८तीहहॴ

11-Jan-2017

Options

1) Cellulose

सहॳमयिह८ज

2) Glucose

गिकह८ज

3) Sucrose

सिह८ज

4) Fructose

फरकटह८ज

Correct Answer Cellulose

Plant cell wall the major carbohydrates

are cellulose hemicellulose and pectin

The cellulose microfibrils are linked via

hemicellulosic tethers to form the

cellulose-hemicellulose network which

is embedded in the pectin matrix

Q35 The study of Fungi is also known

as कवकह८कहॳ अधययनकह८कहाजाताहहॴ

11-Jan-2017

Options

1) Cytology

सायटह८िह८जी 2) Myology

मायह८िह८जी 3) Mycology

मायकह८िह८जी 4) Neurology

नयरह८िह८जी Correct Answer Mycology

Cytology - structure and function of

plant and animal cells

Myology is the study of the muscular

system

Neurology is the branch of medicine

concerned with the study and treatment

of disorders of the nervous system

Q36 The outermost layer of skin is

तवचाकीसबसहॳबाहरीपरतकयाहह८तीहहॴ 11-Jan-

2017

Options

1) Epidermis

इपपिलमयस

2) Dermis

िलमयस

3) Tissues

ऊतक

4) Hypodermis

हायपह८िलमयस

Correct Answer Epidermis

Q37 Which of the following plants have

root nodules

F A C E B O O K

P A G E h t t p w w w f a c e b o o k c o m s s c m e n t o r s o f f i c i a l P a g e | 10

FOR MORE UPDATES AND MORE MATERIAL DO LIKE OUR FACEBOOK PAGE httpwwwfacebookcomsscmentorsofficial

नननननलिखितपह९धह८मसहॳककसकीजड़ह८मगाठहह८तीहहॴ

11-Jan-2017

Options

1) Leguminous plants

िहॳगयलमनसपह९धहॳ 2) Parasitic plants

परजीवीपह९धहॳ 3) Epiphytic Plants

एपीफाइटटकपह९धहॳ 4) Aquatic Plants

जिीयपह९धहॳ Correct Answer Leguminous plants

Q38 Earth-worms belongs to the

phylum

कहॳ चएmdashmdash- परजानतकहॳ अतगयतआतहॳहहॴ 11-Jan-2017

Options

1) Protozoa

परह८टह८जआ

2) Cnidaria

नीिहॳररया 3) Annelida

एनीलििा 4) Mollusca

मह८िसका Correct Answer Annelida

Q39 Ringworm is a disease caused by

ररगवमयनामकबीमारी mdashmdash- कहॳ कारणहह८तीहहॴ 11-Jan-2017

Options

1) Fungi

कवक

2) Bacteria

बहॴकटीररया 3) Virus

वायरस

4) Flies

मनकियाा Correct Answer Fungi

Q40 Mangifera indica is the scientific

name of

मननगफहॳ राइडिकाककसकावहॴजञाननकनामहहॴ 11-

Jan-2017

Options

1) Guava

अम द

2) Mango

आम

3) Amla

आविा 4) Jack fruit

कटहि

Correct Answer Mango

Q41 Crabs belongs to the phylum

कहॳ कड़हॳmdashmdash- परजानतकहॳ अतगयतआतहॳहहॴ 11-Jan-2017

Options

1) Mollusca

मह८िसका 2) Cnidaria

नीिहॳररया 3) Arthropoda

अरोपह८ड़ा 4) Platyhelminthes

पिहॳटटहहॳनममननिस

Correct Answer Arthropoda

Q42 Myopia is a defect of eyes which is

also known as

मायह८पपयाआिोकादह८षहहॴ नजसहॳ mdashmdashndash

भीकहाजाताहहॴ

12-Jan-2017

Options

1) Far Sightedness

F A C E B O O K

P A G E h t t p w w w f a c e b o o k c o m s s c m e n t o r s o f f i c i a l P a g e | 11

FOR MORE UPDATES AND MORE MATERIAL DO LIKE OUR FACEBOOK PAGE httpwwwfacebookcomsscmentorsofficial

दरदनषटदह८ष

2) Near Sightedness

ननकटदनषटदह८ष

3) Astigmatism

एसटीगमहॳटटजम

4) Night Blindness

रतोधी Correct Answer Near Sightedness

Myopia occurs when the eyeball is too

long relative to the focusing power of

the cornea and lens of the eye This

causes light rays to focus at a point in

front of the retina rather than directly

on its surface

Hyperopia Hypermetropia (

Farsightedness )- when light rays

entering the eye focus behind the retina

rather than directly on it The eyeball of

a farsighted person is shorter than

normal

Astigmatism usually is caused by an

irregularly shaped cornea Instead of

the cornea having a symmetrically

round shape (like a baseball) it is

shaped more like an American football

Nyctalopia also called night ndash blindness

is a condition making it difficult or

impossible to see in relatively low light

Q43 Who is known as the father of

Green Revolution

हररतिानतकहॳ जनककहॳ पमककसहॳजानाजाताहहॴ

12-Jan-2017

1) Dr Robert Nucleus

िॉ रॉबटयनयनकियस

2) Dr Ian Wilmut

िॉ इयानपविमट

3) Dr NE Borlaug

िॉ एनईबह८रिॉग

4) Dr JC Bose

िॉ जहॳसीबह८स

Correct Answer Dr NE Borlaug

Q44 Panthera Tigris is the scientific

name of

पिहॳराटटगरीस mdashmdashmdash कावहॴजञाननकनामहहॴ 12-Jan-2017

Options

1) Panther

तदआ

2) Tiger

बाघ

3) Whale

हहॳि

4)Goat

बकरी Correct Answer Tiger

Q45 How many facial bones are there

हमारहॳचहॳहरहॳमककतनीहडडियााहह८तीहहॴ 13-Jan-2017

Options

1)34

2)24

3)14

4)4

Correct Answer 14

Q46 ndash Halophytes are plants that grow

in

हहॴिह८फाईटसवहॳपह९धहॳहह८तीहहॴजह८ mdash- मउगतहॳहहॴ SSC CHSL Science (biology) 2016

Question Paper

13-Jan-2017

Options

1) Fresh Water

ताजापानी 2) Cold Water

ठिापानी 3) Ponds

तािाब

4) Salt Water

िारापानी Correct Answer Salt Water

F A C E B O O K

P A G E h t t p w w w f a c e b o o k c o m s s c m e n t o r s o f f i c i a l P a g e | 12

FOR MORE UPDATES AND MORE MATERIAL DO LIKE OUR FACEBOOK PAGE httpwwwfacebookcomsscmentorsofficial

Q47 Felis Catus is the scientific name of

फहॳ लिसकहॴ टस mdashndash कावहॴजञाननकनामहहॴ 13-Jan-2017

Options

1) Cat

बबमिी 2) Dog

कतता 3) Mouse

चहा 4) Porcupine

साही Correct Answer Cat

Q48 Which of the following induces

nitrogen fixation in soil

नननननलिखितमसहॳकह९नलम ीमनाइटरह८जनननयतनकह८परहॳररतकरताहहॴ

15-Jan-2017

Options

1) Protozoa

परह८टह८जआ

2) Bacteria

बहॴकटीररया 3) Fungi

कवक

4)Algae

िहॴवाि

Correct Answer Bacteria

Bacteria that change nitrogen gas from

the atmosphere into solid nitrogen

usable by plants are called nitrogen-

fixing bacteria These bacteria are

found both in the soil and in symbiotic

relationships with plants

They contain symbiotic bacteria called

rhizobia within nodules in their root

systems producing nitrogen compounds

that help the plant to grow and compete

with other plants When the plant dies

the fixed nitrogen is released making it

available to other plant

Q49 Which of the following is the

largest known cell

नननननलिखितमसहॳकह९नसीसबसहॳबड़ीजञातकह८लिकाहहॴ

SSC CHSL Science (biology) 2016

Question Paper

15-Jan-2017

1) Eukaryotic Cell

यकहॳ ररयह८टटककह८लिका 2) Prokaryotic Cell

परह८कहॳ ररयह८टटककह८लिका 3) Mycoplasma

मायकह८पिासम

4) Ostrich Eggs

ितरमगयकाअिा Correct Answer Ostrich Eggs

Q50 The association of animals in

which both the partners are benefitted

is known as

जानवरोकावहसहयह८गनजसमहॳदह८नोभागीदारिाभापवनतहह८तहॳहहॴ उसहॳ mdashmdashndash कहॳ पमजानाजाताहहॴ SSC CHSL Science (biology) 2016

Question Paper

15-Jan-2017

Options

1) Amensalism

सहजीपवत

2) Commensalism

परजीपवत

3) Colony

कॉिनी 4) Mutualism

अनयह८नयाशरयवाद

Correct Answer Mutualism

Amensalism is any relationship between

organisms of different species in which

F A C E B O O K

P A G E h t t p w w w f a c e b o o k c o m s s c m e n t o r s o f f i c i a l P a g e | 13

FOR MORE UPDATES AND MORE MATERIAL DO LIKE OUR FACEBOOK PAGE httpwwwfacebookcomsscmentorsofficial

one organism is inhibited or destroyed

while the other organism remains

unaffected

Commensalism an association between

two organisms in which one benefits and

the other derives neither benefit nor

harm

Q51 Pneumonia affects which of the

following organs of human body

ननमह८ननयामानविरीरकहॳ नननननलिखितमसहॳककसअगकह८परभापवतकरताहहॴ

15-Jan-2017

Options

1)Kidneys

गद

2)Lungs

फहॳ फड़हॳ 3) Throat

गिहॳ 4) Liver

यकत

Correct Answer Lungs

When the germs that cause pneumonia

reach your lungs the lungsrsquo air sacs

(alveoli) become inflamed and fill up

with fluid This causes the symptoms of

pneumonia such as a cough fever

chills and trouble breathing When you

have pneumonia oxygen may have

trouble reaching your blood

Q52 Mendel is known as

मििकह८ mdashmdash- कहॳ पमजानाजाताहहॴ 15-Jan-2017

Options

1) Father of Physiology

िरीरकियािासतरकहॳ जनक

2) Father of Geology

भगभयिासतरकहॳ जनक

3) Father of Genetics

जहॳनहॳटटकसकहॳ जनक

4) Father of Biology

जीविासतरकहॳ जनक

Correct Answer Father of Genetics

Q53 Which of the following are also

known as Suicidal bag of Cells

ननननलिखितमसहॳककसहॳआतमहतयाकरनहॳवािीकह८लिकाओकाबहॴगकहाजाताहहॴ

15-Jan-2017

Options

1) Lysosomes

िायसोसह८म

2) Lycosome

िायकह८सह८म

3) Nucleus

नालभक

4) Chromosome

िह८मह८सह८म

Correct Answer Lysosomes

Q54 Mesothelioma is a type of cancer

The most common area affected in it is

the lining of the ________

लमज़ह८िहॳिहॳलमयाक सरकाएकपरकारहहॴ इससहॳपरभापवतहह८नहॳवािासबसहॳसामानयकषहॳतर mdash

mdashmdash काअसतरहहॴ 15-Jan-2017

Options

1)Heart

हदय

2)Brain

मनसतषक

3)Stomach

आमािय

4)Lungs

फहॳ फड़हॳ Correct Answer lungs

Asbestos exposure is the main cause of

pleural mesothelioma When asbestos

fibers are breathed in they travel to the

F A C E B O O K

P A G E h t t p w w w f a c e b o o k c o m s s c m e n t o r s o f f i c i a l P a g e | 14

FOR MORE UPDATES AND MORE MATERIAL DO LIKE OUR FACEBOOK PAGE httpwwwfacebookcomsscmentorsofficial

ends of small air passages and reach the

pleura where they can cause

inflammation and

scarring

Q55 Which one of the following is an

insectivorous plant

नननननलिखितमसहॳकह९नसाएकककटाहरीवनसपनतहहॴ

15-Jan-2017

Options

1) Utricularia

यटरीकिहॳररया 2) Sequoia

सहॳकयओइया 3) Nostoc

नॉसटह८क

4) Bryophyta

िायह८फाईटा Correct Answer Utricularia

Q56 ______________ is a

multibranched polysaccharide of

glucose that serves as a form of energy

storage in animals and fungi

mdashmdashगिकह८जकाएकबहिािायकतपह८िीसहॳकहॳ राइिहहॴ जह८जानवरोऔरकवकमउजायभणिारणकहॳ एक पमकाययकरताहहॴ 15-Jan-2017

Options

1) Cellulose

सहॳमयिह८ज

2) Glycogen

गिायकह८जन

3) Pectin

पहॳनकटन

4) Chitin

चीटटन

Correct Answer Glycogen

Q57 The largest gland of the human

body is

mdashmdashmdashमानविरीरकीसबसहॳबड़ीगरिीहहॴ 16-Jan-2017

Options

1) Pancreas

अगयािय

2) Thyroid

िायरॉइि

3) Large Intestine

बड़ीआत

4) Liver

यकत

Correct Answer Liver

Q58 Photosynthesis in plants takes

place in

वनसपनतयोमपरकािसशिहॳषणकीकियाहह८तीहहॴ

16-Jan-2017

Options

1) Stem

तना 2) Leaves

पनततयाा 3) Roots

जड़हॳ 4) Flower

फि

Correct Answer Leaves

During this reaction carbon dioxide

and water are converted into glucose

and oxygen The reaction requires light

energy which is absorbed by a green

substance called

chlorophyll Photosynthesis takes place

in leaf

cells These contain chloroplasts which

are tiny objects containing chlorophyll

F A C E B O O K

P A G E h t t p w w w f a c e b o o k c o m s s c m e n t o r s o f f i c i a l P a g e | 15

FOR MORE UPDATES AND MORE MATERIAL DO LIKE OUR FACEBOOK PAGE httpwwwfacebookcomsscmentorsofficial

Q59 Insects that transmit diseases are

known as

जह८कीड़हॳरह८गसचाररतकरतहॳहहॴ उनह mdashmdash-

कहॳ नामसहॳजानाजाताहहॴ 16-Jan-2017

1)Pathogens

रह८गज़नक

2) Vectors

वहॳकटर

3) Drones

परजीवी 4)Scalars

अटदषट

Correct Answer Vectors

A vector is an organism that does not

cause disease itself but which spreads

infection by conveying pathogens from

one host to another Species of mosquito

for example serve as vectors for the

deadly disease Malaria

Q60 Which is the second largest gland

of Human body

मानविरीरकीदसरीसबसहॳबड़ीगरिीकह९नसीहहॴ

SSC CHSL Science (biology)

2016 Question Paper

16-Jan-2017

Options

1) Liver

यकत

2) Large Intestine

बड़ीआत

3) Thorax

छाती 4) Pancreas

अगनयािय

Correct Answer Pancreas

Q61 Annona squamosa is the scientific

name of

एनह८नासकवामह८सा (Annona squamosa) mdash

mdashmdash कावहॴजञाननकनामहहॴ 16-Jan-2017

Options

1) Custard Apple

सीताफि

2) Papaya

पपीता 3) Babhul

बबि

4) Drumstick

सहजन

Correct Answer Custard Apple

Q62 The disease Beri Beri is caused due

to the deficiency of which of the

following

बहॳरीबहॳरीरह८गनननननलिखितमसहॳककसकीकमीकहॳकारणहह८ताहहॴ

16-Jan-2017

Options

1) Vitamin B2

पवटालमन B2

2) Vitamin B1

पवटालमन B1

3) Vitamin B12

पवटालमन B12

4) Vitamin E

पवटालमन E

Correct Answer Vitamin B1

Beriberi is a disease caused by a vitamin

B-1 deficiency also known as thiamine

deficiency

Q63 Chlorophyll was first isolated and

named by

किह८रह८कफिकह८ mdash-

दवारापहिहॳपिकऔरनालमतककयागया 16-Jan-2017

F A C E B O O K

P A G E h t t p w w w f a c e b o o k c o m s s c m e n t o r s o f f i c i a l P a g e | 16

FOR MORE UPDATES AND MORE MATERIAL DO LIKE OUR FACEBOOK PAGE httpwwwfacebookcomsscmentorsofficial

Options

1) Caventou

कहॳ वहॳत 2) Pelletier

पहॳिहॳटटयर

3) Chlorophyll

किह८रह८कफि

4) Caventou and Pelletier

कहॳ वहॳतऔरपहॳिहॳटटयर

Correct Answer Caventou and Pelletier

Chlorophyll was first isolated and

named by

Joseph Bienaimeacute Caventou and Pierre

Joseph Pelletier in 1817 The presence of

magnesium in chlorophyll was

discovered in 1906 and was the first

time that magnesium had been detected

in living tissue

Q64 Which of the following organisms

does not fit into the Cell Theory

नननननलिखितमसहॳकह९नसाजीवकह८लिकालसदातअन पनहीहहॴ

16-Jan-2017

Options

1) Bacteria

बहॴकटीररया 2) Virus

वायरस

3) Fungi

कवक

4) Plants

पह९धहॳ Correct Answer Virus

The bottom line is that viruses are not

alive and not related to cells in any way

The cell theory states that all living

things are made of cells cells are the

basic units of structure and function of

living things and that all cells come

from other cells Since viruses are not

made of cells and do not use cells in any

of their processes they are not related to

the cell theory

Q65 Which of these is not a

macronutrient for Plants

नननननलिखितमसहॳकह९नसापह९धह८कहॳ लिएमिह८नयटरीएटनहीहहॴ

SSC CHSL Science (biology) 2016

Question Paper

17-Jan-2017

Options

1) Nitrogen

नाइटरह८जन

2) Phosphorus

फासफह८रस

3) Potassium

पह८टालसयम

4) Chlorine

किह८रीन

Correct Answer Chlorine

In relatively large amounts the soil

supplies nitrogen phosphorus

potassium calcium magnesium and

sulfur these are often called the

macronutrients In relatively small

amounts the soil supplies iron

manganese boron molybdenum

copper zinc chlorine and cobalt the

so-called micronutrients

Q66 Name the respiratory organs of

insects

कीटह८मनसतिशरवसनअगनामकानामहहॴ

17-Jan-2017

Options

1) Skin

तवचा 2) Body Surface

िरीरकीसतह

F A C E B O O K

P A G E h t t p w w w f a c e b o o k c o m s s c m e n t o r s o f f i c i a l P a g e | 17

FOR MORE UPDATES AND MORE MATERIAL DO LIKE OUR FACEBOOK PAGE httpwwwfacebookcomsscmentorsofficial

3) Gills

गिफड़हॳ 4) Tracheae

शरावस- निी Correct Answer Tracheae

Air enters the respiratory systems of

insects through a series of external

openings called

spiracles These external openings

which act as muscular valves in some

insects lead to the internal respiratory

system a densely networked array of

tubes called tracheae

Q67 The poisonous gas accidentally

released in Bhopal Gas Tragedy is

भह८पािगहॴसतरासदीमगितीसहॳमकतहईजहरीिीगहॴसिी

17-Jan-2017

1) Methane

मीिहॳन

2) Nitrous Oxide

नाइटरसऑकसाइि

3) Methyl Isocyanate

महॴचििआयसोसायनहॳट

4) Cyanogen

सायनह८जहॳन

Correct Answer Methyl Isocyanate

Q68 What does Trypsin do

टटरनपसनकयाकरताहहॴ

SSC CHSL Science (biology) 2016

Question Paper

17-Jan-2017

Options

1) Breaks down Carbohydrates

काबोहाइडरहॳटकापवघटनकरताहहॴ 2) Synthesizes proteins

परह८टीनकासििहॳषणकरताहहॴ 3) Breaks down fats

वसाकापवघटनकरताहहॴ 4) Breaks down proteins

परह८टीनकापवघटनकरताहहॴ Correct Answer Breaks down proteins

Trypsin is one of the three principal

digestive

proteinases the other two being pepsin

and

chymotrypsin In the digestive process

trypsin acts with the other proteinases

to break down dietary protein molecules

to their component

peptides and amino acids

A protease is any enzyme that performs

proteolysis protein catabolism by

hydrolysis of peptide bonds

Q69 Name the source from which

Aspirin is produced

उससरह८तकानामबताइए

नजससहॳएनसपररनकाउतपादनककयाजाताहहॴ

17-Jan-2017

Options

1) Willow bark

पविह८कीछाि

2) Oak Tree

ओककावकष

3) Acacia

बबि

4) Eucalyptus

नीिचगरी Correct Answer Willow bark

The compound from which the active

ingredient in aspirin was first derived

salicylic acid was found in the bark of a

willow tree in 1763 by Reverend

Edmund Stone of Chipping-Norton

Q70 Cannis Familiaris is the scientific

name of

कहॴ ननसफहॳ लमलियहॳररस mdash- कावहॴजञाननकनामहहॴ

17-Jan-2017

F A C E B O O K

P A G E h t t p w w w f a c e b o o k c o m s s c m e n t o r s o f f i c i a l P a g e | 18

FOR MORE UPDATES AND MORE MATERIAL DO LIKE OUR FACEBOOK PAGE httpwwwfacebookcomsscmentorsofficial

Options

1) Cat

बबमिी 2)Dog

कतता 3) Fox

िह८मड़ी 4) Wolf

भहॳडड़या Correct Answer Dog

Q71 Harmful bacteria in potable water

make the water

पीनहॳकहॳ पानीमनसतिघातकबहॴकटीररयाउसपानीकह८बनातहॳहहॴ 17-Jan-2017

Options

1) unfit to drink

पीनहॳकहॳ लिएअयह८गय

2) smelly

दगयनधयकत

3) Colored

रगीन

4) Turbid

मटमहॴिा Correct Answer unfit to drink

Q72 Musa paradisiaca is the scientific

name of which plant

मसापहॴराडिलसयाकाककसपह९धहॳकावहॴजञाननकनामहहॴ

17-Jan-2017

Options

1) Mango

आम

2) Wheat

गहॳह

3) Corn

भ ा 4) banana

कहॳ िा Correct Answer banana

Q73 Prawns belong to which family

झीगहॳककसपररवारकहॳ हह८तहॳहहॴ 17-Jan-2017

Options

1) Crustaceans

िसटहॳलियन

2)Fish

मछिी 3) Amphibians

अननफबबयस

4) Reptiles

रहॳपटाइमस

Correct Answer Crustaceans

Q74 Name the drug that is yielded from

Cinchona tree and is used to cure

malaria

उसऔषचधकानामबताइएनजसहॳलसगकह८नापहॳड़सहॳपरापतककयाजाताहहॴऔरनजसकाउपयह८गमिहॳररयाकहॳ उपचारमककयाजाताहहॴ 17-Jan-2017

Options

1) Camptothea

कहॴ नटह८चिया 2) Acuminata

एकयलमनहॳटा 3) Quinine

कनहॴन

4) Cinchonia

लसकह८ननया Correct Answer Quinine

Q75 Blood Circulation was discovered

by

रकतपररसचरणकी mdashmdashndash दवारािह८जकीिी 17-Jan-2017

Options

1) Mary Anderson

F A C E B O O K

P A G E h t t p w w w f a c e b o o k c o m s s c m e n t o r s o f f i c i a l P a g e | 19

FOR MORE UPDATES AND MORE MATERIAL DO LIKE OUR FACEBOOK PAGE httpwwwfacebookcomsscmentorsofficial

महॴरीएिरसन

2) Virginia Apgar

वनजयननयाएपगार

3) William Harvey

पवलियमहाव

4) Robert Feulgen

रॉबटयफ़यिजहॳन Correct Answer William Harvey

Q76 Vitamin A is also known as

पवटालमन A कह८ mdashmdash- कहॳ नामसहॳभीजानाजाताहहॴ SSC CHSL Science (biology) 2016

Question Paper

18Jan2017

Options

1) Thiamine

िायलमन

2) Riboflavin

ररबह८फिहॳपवन

3) Retinol

रहॳटटनॉि

4) Calciferol

कहॴ नमसफहॳ रह८ि

Correct Answer Retinol

Q77 Some roots called arise from an

organ other than the radicle

कछजड़हॳनजनह mdashmdashmdash कहाजाताहहॴ वहमिकहॳ अिावाककसीअनयअगसहॳउतपननहह८तीहहॴ 18Jan2017

Options

1) tap roots

मखयजड़

2) stilt roots

ि ाजड़

3) fibrous roots

रहॳिहॳदारजड़

4) adventitious roots

आकनसमकजड़

Correct Answer adventitious roots

Q78 Spiders belong to which class of

animals

मकडड़यापराणीवगीकरणकहॳ ककसवगयमआतीहहॴ 18Jan2017

Options

1) Arachnids

एरहॳकननडस

2) Aves

एपवस

3) Gastropods

गहॴसटरोपह८िस

4) Anthozoa

एिह८जआ

Correct Answer Arachnids

Q79 How many layers does Human

Skin have

मानवतवचामककतनीपरतहॳहह८तीहहॴ

18Jan2017

Options

1) 5

2) 7

3) 11

4) 3

Correct Answer 3

Skin has three layers The epidermis

the outermost layer of skin provides a

waterproof barrier and creates our skin

tone The dermis beneath the

epidermis contains tough connective

tissue hair follicles and sweat glands

The deeper subcutaneous tissue (

hypodermis ) is made of fat and

connective tissue

Q80 Allium Cepa is the scientific name

of

एलियमलसपपा mdashmdashndash कावहॴजञाननकनामहहॴ 18Jan2017

F A C E B O O K

P A G E h t t p w w w f a c e b o o k c o m s s c m e n t o r s o f f i c i a l P a g e | 20

FOR MORE UPDATES AND MORE MATERIAL DO LIKE OUR FACEBOOK PAGE httpwwwfacebookcomsscmentorsofficial

Options

1) Carrot

गाजर

2) Tomato

टमाटर

3) Potato

आि 4) Onion

पयाज़

Correct Answer Onion

Q81 DNA stands for

िीएनएकापणय प mdashmdash- हहॴ 18Jan2017

Options

1) Di Nucleic Acid

िाईनयनकिकएलसि

2) Deoxy Nucleic Acid

िीओकसीनयनकिकएलसि

3) Diribonucleic Acid

िाईराइबह८नयनकिकएलसि

4) Deoxyribonucleic Acid

िीऑकसीराइबह८नयनकिकएलसि

Correct Answer Deoxyribonucleic Acid

Q82 Organisms that generate energy

using light are known as

जह८जीवाणपरकािकाउपयह८गकरउजायउतपननकरतीहहॴ उनह mdashmdash कहॳ पमजानाजाताहहॴ

18Jan2017

Options

1) Chaemolithotrophs

ककमह८लििह८टरह८पस

2) Oligotrophs

ओलिगह८टरह८पस

3) Bacteria

बहॴकटीररया 4)Photoautotrophs

फह८टह८ओटह८टरह८पस

Correct Answer Photoautotrophs

An oligotroph is an organism that can

live in an environment that offers very

low levels of nutrients

Q83 Which drug is used as an

Antidepressant

ककसदवाएकहतािारह८धीकहॳ पमपयोगककयाजाताहहॴ Options

1) Oxybutynin

ओकसीलयटीनन

2)Tramadol

टरहॳमहॳिह८ि

3 ) Sumatriptan

समहॳटरीपटहॳन

4) Bupropion

लयपरह८पपयह८न

Correct Answer Bupropion

लयपरह८पपयह८न

Q84 The orange colour of carrot is

because of

गाजरकानारगीरगनननननलिखितमसहॳककसीएककीवजहसहॳहह८ताहहॴ 18Jan2017

Options

1) it grows in the soil

यहलम ीमउगतीहहॴ 2) Carotene

कहॴ रह८टीन

3) it is not exposed to sunlight

यहसययपरकािकहॳ सपकय मनहीआती 4) the entire plant is oranqe in colour

सनपणयपह९धानारगीरगकाहह८ताहहॴ Correct Answer Carotene

Q85 Snake venom is highly modified

saliva containing

F A C E B O O K

P A G E h t t p w w w f a c e b o o k c o m s s c m e n t o r s o f f i c i a l P a g e | 21

FOR MORE UPDATES AND MORE MATERIAL DO LIKE OUR FACEBOOK PAGE httpwwwfacebookcomsscmentorsofficial

सापकाजहरअततयाचधकसिह८चधतिारहह८तीहहॴनजसमहॳ mdashmdash- हह८ताहहॴ Options

l)Prototoxins

परह८टह८टॉनकसस

2)Neutrotoxins

नयटरोटॉनकसस

3)Zootoxins

जटॉनकसस

4)Electrotoxins

इिहॳकटरह८टॉनकसस

Correct Answer Zootoxins

जटॉनकसस

Q86 Which type of pathogen causes the

water-borne disease Schistosomiasis

ककसपरकारकारह८गज़नकजिजननतरह८गलससटह८सह८लमलससकाकारणबनताहहॴ

18Jan2017

Option

1) Parasitic

परजीवी 2)Protozoan

परह८टह८जआ

3) Bacterial

बहॴकटीररयि

4) Viral

वायरि

Correct Answer Parasitic

Schistosomiasis also known as snail

fever and bilharzia is a disease caused

by parasitic

flatworms called schistosomes

Q87 Prothrombin responsible for

clotting of blood is released by

परह८िह८ननबन

जह८रकतकािककाजमनहॳकहॳ लिएनजनमहॳदारहहॴ mdashndash

कहॳ दवारासतरापवतककयाजाताहहॴ

19Jan2017

Options

1) Small Intestine

छह८टीआत

2) Blood Platelets

रकतपिहॳटिहॳटस

3) Large Intestine

बड़ीआत

4Heart

हदय

Correct Answer Blood Platelets

Q88 Acacia arabica is the scientific

name of

अकहॳ लियाअरहॳबबका mdashmdashndash कावहॴजञाननकनामहहॴ 19-Jan-2017

Options

1) Neem

नीम

2) Teak

सागह९न

3) Babhul

बबि

4) Pomegranate

अनार

Correct Answer Babhul

Q89 Cannis Vulpes is the scientific

name of

कहॴ ननसवनमपस mdashmdash- कावहॴजञाननकनामहहॴ 19-Jan-2017

Options

1) Dog

कतता 2) Wolf

भहॳडड़या 3) Fox

िह८मड़ी 4) Hyena

िाकिबगघा

F A C E B O O K

P A G E h t t p w w w f a c e b o o k c o m s s c m e n t o r s o f f i c i a l P a g e | 22

FOR MORE UPDATES AND MORE MATERIAL DO LIKE OUR FACEBOOK PAGE httpwwwfacebookcomsscmentorsofficial

Correct Answer Fox

Q90 The beetroot is the portion of the

beet plant

चकदरपह९धहॳका mdashmdashndash भागहहॴ 19-Jan-2017

Options

1) tap root

मखयजड़

2) Adventitious

आकनसमक

3) bulb of the stem

तनहॳकाकद

4) Rhizome

परकद

Correct Answer tap root

Q91 What is the basic unit of heredity

आनवलिकताकीबननयादीइकाईकयाहहॴ 19-Jan-2017

Options

1) DNA

िीएनए

2) RNA

आरएनए

3) Chromosome

िह८मह८सह८म

4) Gene

जीन

Correct Answer gene

Genes are the units of heredity and are

the instructions that make up the bodyrsquos

blueprint They code for the proteins

that determine virtually all of a personrsquos

characteristics Most genes come in

pairs and are made of strands of genetic

material called deoxyribonucleic acid

or DNA

Q92 Lungs are the primary organs of

फहॳ फड़हॳmdashndashकहॳ परािलमकअगहहॴ

19-Jan-2017

Options

1) Digestion

पाचन

2) Constipation

कलज

3) Perspiration

पसीना 4)Respiration

शवसन

Correct Answer Respiration

Q93 Sugarcane is a type of

गननाएकपरकारका mdash- हहॴ 20-Jan-2017

Options

1)creeper

िता 2)tree

पहॳड़

3)shrub

झाड़ी 4)grass

घास

Correct Answer grass

Q94 Who is commonly known as ldquothe

Father of Microbiologyrdquo

सामानयत ldquo सकषमजीवपवजञानकहॳ जनक lsquo

कहॳ नामसहॳककसहॳजानाजातहहॴ 20-Jan-2017

Options

1) Robert Hooke

रॉबटयहक

2) Antonie Philips van Leeuwenhoek

एटह८नीकफलिपवानमयएनहह८क

3) Carl Linnaeus

काियिीनाईयस

4) Charles Darwin

चामसयिापवयन

F A C E B O O K

P A G E h t t p w w w f a c e b o o k c o m s s c m e n t o r s o f f i c i a l P a g e | 23

FOR MORE UPDATES AND MORE MATERIAL DO LIKE OUR FACEBOOK PAGE httpwwwfacebookcomsscmentorsofficial

Correct Answer Antonie Philips van

Leeuwenhoek

Q95 For the aquatic organisms the

source of food is

जिीयजीवाणकािाघसरह८तहहॴ 20-Jan-2017

Options

1) Phytoplankton

फायटह८पिहॳकटन

2) Sea Weed

समदरीिहॴवाि

3)Aqua plankton

एकवापिहॳकटन

4) Zooplankton

जपिहॳकटन

Correct Answer Phytoplankton

Q96 Haemoglobin has the highest

affinity with which of the following

हीमह८गिह८बबनकीननननमसहॳककसकहॳ सािउततमसमानताहहॴ

20-Jan-2017

Options

1)SO2

2)CO2

3)CO

4)NO2

Correct Answer CO

It has a greater affinity for hemoglobin

than oxygen does It displaces oxygen

and quickly binds so very little oxygen

is transported through the body cells

Q97 Who developed the theory of

Evolution

उदपवकासकालसदातककसनहॳपवकलसतककया

20-Jan-2017

Options

1) Charles Darwin

चामसयिापवयन

2) Isaac Newton

आयजहॳकनयटन

3) Pranav Mistry

परणवलमसतरी 4) Galileo Galilei

गहॳलिलियह८गहॳिीिी Correct Answer Charles Darwin

Q98 The primary function of RNA is

RNA कापरािलमककाययहह८ताहहॴ 20-Jan-2017

Options

1) Photosynthesis

परकािसशिहॳषण

2) Protein Synthesis

परह८टीनसशिहॳषण

3) Replication

परनतकनतबनाना 4) Translation

अनवादकरना Correct Answer Protein Synthesis

There are two main functions of RNA

It assists DNA by serving as a messenger

to relay the proper genetic information

to countless numbers of ribosomes in

your body The other main function of

RNA is to select the correct amino acid

needed by each ribosome to build new

proteins for your body

Q99 ______is the movement of

molecules across a cell membrane from

a region of their lower concentration to

a region of their higher concertration

उचचसादरताकहॳ कषहॳतरसहॳउसकीकमसादरतावािहॳकषहॳतरकीतरफएककह८लिकाखझमिीकहॳ माधयमसहॳहह८नहॳवािाअणओकहॳ सचिनकह८ mdash- कहतहॳहहॴ Options

1) Diffusion

पवसरण

2) Osmosis

ऑसमह८लसस

F A C E B O O K

P A G E h t t p w w w f a c e b o o k c o m s s c m e n t o r s o f f i c i a l P a g e | 24

FOR MORE UPDATES AND MORE MATERIAL DO LIKE OUR FACEBOOK PAGE httpwwwfacebookcomsscmentorsofficial

3) Active Transport

सकियआवागमन

4) Passive Transport

नननषियआवागमन

Correct Answer Active Transport

Q100 Study of classification of

organisms is known as 20-Jan-2017

जीवाणओकहॳ वगीकरणकहॳ अधययनकह८ mdash-

कहाजाताहहॴ Options

1) Serpentology

सपरहॳटह८िह८जी 2) Virology

वायरह८िह८जी 3) Taxonomy

टहॴकसोनह८मी 4) Physiology

कफनज़यह८िह८जी Correct Answer Taxonomy

Q101 Photosynthesis takes place inside

plant cells in

परकािसशिहॳषणवनसपनतकह८लिकामनसति mdash

mdashmdash महह८ताहहॴ 20-Jan-2017

Options

1) Ribosomes

राइबह८सह८नस

2) Chloroplasts

किह८रह८पिासट

3) Nucleus

नयकलियम

4) Mitochondria

माईटह८कोडडरया Correct Answer Chloroplasts

Q102 ______ is the cell organelle in

which the biochemical processes of

respiration and energy production

occur

mdashmdash- वहकह८लिकाअगहहॴ नजसमहॳशवसनऔरउजायउतपादनकहॳ जहॴसीजहॴवरासायननकपरकियायहह८तीहहॴ 20-Jan-2017

Options

1) Mitochondria

माइटह८कोडडरया 2) Chloroplast

किह८रह८पिासट

3) Ribosomes

राइबह८सह८नस

4) Nucleus

नयकिीयस

Correct Answer Mitochondria

Q103 Which non-flowering spore

bearing plants have roots

ककसफिनिगनहॳवािहॳऔरबीजाणधारकपह९धह८कीजड़हॳहह८तीहहॴ 21-Jan-2017

Options

1) Mosses

मह८सहॳस

2) Angiosperms

एननजयह८सपनसय 3) Ferns

फनसय 4) Gymnosperms

नजननह८सपनसय Correct Answer ferns

Q104 Which of the following is an

excretory organ of cockroach

नननननलिखितमसहॳकह९नसानतिच हॳकाउतसजयनअगहहॴ

21-Jan-2017

Options

F A C E B O O K

P A G E h t t p w w w f a c e b o o k c o m s s c m e n t o r s o f f i c i a l P a g e | 25

FOR MORE UPDATES AND MORE MATERIAL DO LIKE OUR FACEBOOK PAGE httpwwwfacebookcomsscmentorsofficial

1) Malphigian Tubules

मनमफनजयनटयबमस

2) Nephridia

नहॳकफरडिया 3) Coxal Gland

कह८कसिगरचिया 4) Green Gland

गरीनगरचिया Correct Answer Malphigian Tubules

Q105 Evaporation of water takes place

in which part of plants

पानीकहॳ वाषपीकरणकीकियापह९धोकहॳ ककसभागसहॳहह८तीहहॴ 21-Jan-2017

Options

1) Stem

तना 2) Stomata

सटह८मटा 3) Branch

िािाए

4) Fruit

फि

Correct Answer Stomata

Evaporation accounts for the movement

of water to the air from sources such as

the soil canopy interception and

waterbodies Transpiration accounts for

the movement of water within a plant

and the subsequent loss of water as

vapour through stomata in its leaves

Q106 A is the fleshy spore-bearing

fruiting body of a fungus

mdashmdashndashकवककामासि

बीजाणधारणकरनहॳवािाफिनहॳवािाअगहहॴ 21-

Jan-2017

Options

1) aloe vera

एिह८वहॳरा 2) Coral

मगा 3) Cactus

कहॴ कटस

4) Mushroom

ककरमतता Correct Answer mushroom

Q107 Which of the following is a fungal

disease

नननननलिखितमसहॳकह९नसाफफदसहॳहह८नहॳवािाएकरह८ग हहॴ

21-Jan-2017

Options

1) Dermatitis

तवचािह८ध

2) Cholera

हहॴजा 3) Jaundice

पीलिया 4) Indigofera

इननिगह८फहॳ रा Correct Answer Dermatitis

Dermatitis also known as eczema is a

group of diseases that results in

inflammation of the skin These diseases

are characterized by itchiness red skin

and a rash In cases of short duration

there may be small blisters while in

long-term cases the skin may become

thickened

Q108 In which form is glucose stored in

our body

हमारहॳिरीरमगिकह८जकासचयककस पमककयाजाताहहॴ

21-Jan-2017

Options

1) Insulin

F A C E B O O K

P A G E h t t p w w w f a c e b o o k c o m s s c m e n t o r s o f f i c i a l P a g e | 26

FOR MORE UPDATES AND MORE MATERIAL DO LIKE OUR FACEBOOK PAGE httpwwwfacebookcomsscmentorsofficial

इसलिन

2) Glucose

गिकह८ज

3) Glycogen

गिायकह८जहॳन

4) Fat

वसा Correct Answer Glycogen

Excess glucose is stored in the liver as

the large compound called glycogen

Glycogen is a polysaccharide of glucose

but its structure allows it to pack

compactly so more of it can be stored in

cells for later use

Q109 Where do plants synthesize

protein from

पह९धहॳपरह८टीनसशिहॳषणकहासहॳकरतहॳहहॴ

Options

1) Fatty Acids

वसाऐलसि

2) Sugar

िकर

3) Amino Acids

एलमनह८ऐलसि

4) Starch

सटाचय Correct Answer Amino Acids

Q110 Which part of the brain is

responsible for triggering actions like

thinking intelligence memory and

ability to learn

मनसतषककाकह९नसाटहससासह८चनहॳ बनधदमानी याददाशतऔरसीिनहॳकीकषमताजहॴसीकियाओकह८परहॳररतकरताहहॴ 21-Jan-2017

Options

1) Diencephalon

िायएनसहॳफहॳ िह८न

2) Hypothalamus

हयपह८िहॳिहॳमस

3) Cerebrum

सहॳरहॳिम

4) Control

कटरह८ि

Correct Answer Cerebrum

Q111 Which of the following is also

known as the Biochemical Laboratory

of the Human Body

नननननलिखितमसहॳककसहॳमानविरीरकीजहॴवरसायनपरयह८गिािाभीकहाजाताहहॴ 21-Jan-2017

Options

1) Small Intestine

छह८टीआत

2)Brain

मनसतषक

3) Pancreas

अगनयािय

4) Liver

नजगर

Correct Answer Liver

The liver makes bile that will help

emulsify and digest the fats we eat

The liver takes toxic substances and

convert them using enzymes the liver

cells makes into a non toxic form so the

body can dispose of them

The liver also converts fats protein and

carbohydrates into glucose which is the

energy source for our cells to use

The liver takes amino acids and makes

proteins by combining them

Q112 The yellow colour of human urine

is due to

मानवमतरकापीिारग mdashndash कीवजहसहॳहह८ताहहॴ 22-

Jan-2017

Options

1) Bile Salts

F A C E B O O K

P A G E h t t p w w w f a c e b o o k c o m s s c m e n t o r s o f f i c i a l P a g e | 27

FOR MORE UPDATES AND MORE MATERIAL DO LIKE OUR FACEBOOK PAGE httpwwwfacebookcomsscmentorsofficial

पपततनमक

2) Cholesterol

कह८िहॳसटरह८ि

3) Lymph

लिनफ

4) Urochrome

यरह८िह८म

Correct Answer Urochrome

Urobilin or urochrome is the chemical

primarily responsible for the yellow

color of urine

Q113 The wilting of plants takes place

due to

पह९धह८कालिचििहह८नाकी mdashmdash- कीवजहसहॳहह८ताहहॴ 22-Jan-2017

Options

1)Photosynthesis

परकािसशिहॳषण

2) Transpiration

वाषपह८तसजयन

3) Absorption

अविह८षण

4) Respiration

शरवसन

Correct Answer Transpiration

Wilting is the loss of rigidity of non-

woody parts of plants This occurs when

the turgor pressure in non-lignified

plant cells falls towards zero as a result

of diminished water in the cells

Q114 Bovidae Ovis is the scientific name of

बह८पविीओपवस mdashndash कावहॴजञाननकनामहहॴ 22-Jan-2017

Options

1) Goat

बकरी 2) Cow

गाय

3) Buffalo

भहॳस

4) Sheep

भहॳड़

Correct Answer Sheep

Q115 Plants get their energy to produce

food from which of the following

पह८धहॳभह८जनकाननमायणकरनहॳकहॳ लिएनननननलिखितमसहॳककससहॳउजायपरापतकरतहॳहहॴ

22-Jan-2017

Options

1) Photosynthesis

परकािसशिहॳषण

2)Bacteria

बहॴकटीररया 3)Fungi

कवक

4)Sun

सयय Correct Answer Sun

Q116 Which of the following is secreted

by the liver

नननननलिखितमसहॳककसकासरावनजगरसहॳहह८ताहहॴ

22-Jan-2017

Options

1) Glucose

गिकह८ज

2) Iodine

आयह८िीन

3) Cortisol

काटटरयसह८ि

4) Bile

पपतत

Correct Answer Bile

The liver makes bile that will help

emulsify and

digest the fats we eat

F A C E B O O K

P A G E h t t p w w w f a c e b o o k c o m s s c m e n t o r s o f f i c i a l P a g e | 28

FOR MORE UPDATES AND MORE MATERIAL DO LIKE OUR FACEBOOK PAGE httpwwwfacebookcomsscmentorsofficial

Q117 Ferns belong to which division of

plants

फनसयपह९धह८कहॳ ककसभागमआतहॳहहॴ

22-Jan-2017

Options

1) Gymnosperms

नजननह८सपनसय 2) Angiosperms

एनजयह८सपनसय 3) Thallophyta

िहॴिह८फाईटा 4)Pteridophyta

टहॳररिह८फाईटा Correct Answer Pteridophyta

Q118 Who invented Antibiotics

एटीबायह८टटककाअपवषकारककसनहॳककयािा

22-Jan-2017

Options

1) Joseph Lister

जह८सहॳफलिसटर

2) William Harvey

पवलियमहाव

3) Robert Knock

रॉबटयनॉक

4)Alexander Fleming

अिहॳकज़िरफिहॳलमग

Correct Answer Alexander Fleming

Q119 Milbecycin is used in the

eradication of

लममबहॳसायलसनका mdashndash

मउनमिनमपरयह८गककयाजाताहहॴ 22-Jan-2017

Options

1) Agricultural Fungus

कपषकवक

2) Agricultural Pests

कपषकीटक

3) Agricultural Herbs

कपषिाक

4)Agricultural Weeds

कपषननराना Correct Answer Agricultural Pests

Milbemycin oxime is a veterinary drug

from the group of milbemycins used as

a broad spectrum antiparasitic It is

active against worms and mites(insects

Q120 Intestinal bacteria synthesizes

which of the following in the human

body

मानविरीरमआतोकहॳ बहॴकटीररयानननननलिखितमसहॳककसकासशिहॳषणकरतहॳहहॴ 22-Jan-2017

Options

1) Vitamin K

पवटालमन K

2) Proteins

परह८टीन

3) Fats

वसा 4) Vitamin D

पवटालमन D

Correct Answer Vitamin K

Q121 is the study of the physical form

and external structure of plants

mdashmdash-

मपह९धह८काभहॴनतक पऔरबाहरीसरचनाकाआदयाककयाजाताहहॴ 22-Jan-2017

Options

1) Physiology

कफनजयह८िह८जी 2) Anatomy

िरीररचनापवजञान

3) Phytomorphology

फाईटह८मह८फह८िह८जी 4)Cytology

कह८लिकापवजञान

Correct Answer Phytomorphology

F A C E B O O K

P A G E h t t p w w w f a c e b o o k c o m s s c m e n t o r s o f f i c i a l P a g e | 29

FOR MORE UPDATES AND MORE MATERIAL DO LIKE OUR FACEBOOK PAGE httpwwwfacebookcomsscmentorsofficial

Q122 Which of the following is a

structural and functional unit of

kidneys

नननननलिखितमसहॳकह९नसीगदोकीसरचनातमकऔरकाययकरीईकाईहहॴ

22-Jan-2017

Options

1) Renette Cells

रहॳनहॳटकह८लिकाए

2) Flame Cells

फिहॳमकह८लिकाए

3) Nephrites

नहॳफ़राइटस

4)Nephrons

नहॳफरोस

Correct Answer Nephrons

Nephron functional unit of the kidney

the structure that actually produces

urine in the process of removing waste

and excess substances from the blood

There are about 1000000 nephrons in

each human kidney

Q123 Which of the following is the

largest part of the human brain

नननननलिखितमसहॳकह९नसामानवमनसतषककासबसहॳबड़ाटहससाहहॴ

23-Jan-2017

Options

1) Ribs

पसलियाा 2) Cerebrum

सहॳरहॳिम

3) Pons

पोस

4)Thalamus

िहॴिहॳमस

Correct Answer Cerebrum

The cerebrum is the largest part of the

human brain making up about two-

thirds of the brainrsquos mass It has two

hemispheres each of which has four

lobes frontal parietal temporal and

occipital

Q124 The auxiliary buds

सहायककालियाmdashndash 23-Jan-2017

Options

1) grow endogenously from the pericycle

पहॳरीसाईककिसहॳअनतजातयपवकलसतहह८ताहहॴ 2) arise endogenously from the main

growing point

मिवपदसहॳअनतजातयउठताहहॴ 3) is an embryonic shoot located in the

axil of a leaf

एकभरणिटहहॴजह८एकपततीकहॳ अकषपरनसतिहह८ताहहॴ 4)arise exogenously from the epidermis

एपपिलमयससहॳबटहजातयतरीकहॳ सहॳउठताहहॴ Correct Answer is an embryonic shoot

located in the axil of a leaf

Q125 Which of the following is a viral

disease

इनमहॳसहॳकह९सीएकवायरिबीमारीहहॴ

23-Jan-2017

Options

1) Polio

पह८लियह८ 2) Tetanus

धनसतनभ

3) Leprosy

कषठरह८ग

4) Plague

पिहॳग

Correct Answer Polio

A viral disease (or viral infection)

occurs when an organismrsquos body is

invaded by pathogenic viruses and

infectious virus particles (virions) attach

to and enter susceptible cells

F A C E B O O K

P A G E h t t p w w w f a c e b o o k c o m s s c m e n t o r s o f f i c i a l P a g e | 30

FOR MORE UPDATES AND MORE MATERIAL DO LIKE OUR FACEBOOK PAGE httpwwwfacebookcomsscmentorsofficial

Poliomyelitis often called polio or

infantile paralysis is an infectious

disease caused by the poliovirus

Tetanusmdash A serious bacterial infection

that causes painful muscle spasms and

can lead to death

Leprosy also known as Hansenrsquos

disease (HD) is a long-term infection by

the bacterium Mycobacterium leprae or

Mycobacterium lepromatosis

Plague is an infectious disease caused by

the bacterium Yersinia pestis

Symptoms include fever weakness and

headache

Q126 Which organisms can help to

carry out Vermicomposting

कह९नसाजीववमीकनपह८नसटगममददकरताहहॴ

23-Jan-2017

Options

1) Nitrifying Bacteria

नाईटरीफाईगबहॴकटीररया 2) Earthworms

कहॴ चऐ

3) Algae

िहॴवि

4) Fungus

कवक

Correct Answer Earthworms

Q127 Contraction of heart is also

known as

हदयकहॳ सकचनकह८ mdash- भीकहाजाताहहॴ 23-Jan-

2017

Options

1) Systole

लससटह८ि

2) Aristotle

अरसत

3) Diastole

िायसटह८ि

4) Lub

मयब

Correct Answer Systole

Diastole is the part of the cardiac cycle

when the heart refills with blood

following systole (contraction)

Ventricular diastole is the period during

which the ventricles are filling and

relaxing while atrial diastole is the

period during which the atria are

relaxing

Q128 Azadirachta indica is the

botanical name of which of the

following

अजाटदराचताइडिकानननननलिखितमसहॳककसकावानसपनतनामहहॴ

23-Jan-2017

Options

1) Rose plant

गिाबकापह९धा 2) Apple tree

सहॳबकापहॳड़

3) Neem

नीम

4)Mango

आम

Correct Answer Neem

Q129 Which of the following is the

main end product of carbohydrate

digestion

नननननलिखितमसहॳकह९नसाकाबोहाइडरहॳटकहॳ पाचनकापरमिअतउतपादकहह८ताहहॴ 23-Jan-2017

Options

1) Fats

वसा 2) Lipids

लिपपडस

3) Glucose

गिकह८ज

4) Cellulose

F A C E B O O K

P A G E h t t p w w w f a c e b o o k c o m s s c m e n t o r s o f f i c i a l P a g e | 31

FOR MORE UPDATES AND MORE MATERIAL DO LIKE OUR FACEBOOK PAGE httpwwwfacebookcomsscmentorsofficial

सहॳमयिह८ज

Correct Answer Glucose

Intestinal absorption of end products

from digestion of carbohydrates and

proteins in the pig hellip During absorption some sugars (fructose or

galactose) released from the

corresponding sucrose and lactose

respectively during digestion were

partly metabolized into glucose by the

enterocyte

Q130 Which of the following glands is a

source of the enzyme Ptyalin

नननननलिखितगरचियोमसहॳएजाइमटयालिनकासरह८तहहॴ 23-Jan-2017

Options

1) Pancreas

अगरािय

2) Thyroid Gland

िाइराइिगरिी 3) Pituitary Gland

पीयषगरिी 4) Salivary Glands

िारगरचियाा Correct Answer Salivary Glands

Q131 Which of the following is not true

about Pteridophyta

ननननमसहॳकह९नसीबातटहॳररिह८फाईटकहॳ बारहॳमसचनहीहहॴ 23-Jan-2017

Options

1) Dominant phase is saprophytes

परमिचरणसहॳपरह८फाईइटसहह८ताहहॴ 2) Main plant body is diploid

पह९दह८कामखयिरीरदपवगखणतहह८ताहहॴ 3) Seeds are present

बीजमह९जदहह८तहॳहहॴ 4)Flowers are absent

फिअनपनसतिहह८तहॳहहॴ

Correct Answer Seeds are present

Q132 The largest dolphin species is the

orca also called as

िॉिकफनकीसबसहॳबड़ीपरजानतकाकानामआकायहहॴनजसहॳ mdash- भीकहतहॳहहॴ 23-Jan-2017

Options

1) Bottle Nose

बाटिनह८ज

2) Baiji

बहॳजी 3) Killer whale

ककिरहहॳि

4)Tucuxi

टकवसी Correct Answer Killer whale

Q133 The fat digesting enzyme Lipase

is secreted by which of the following

वसाकापाचनकरनहॳवािाएजाइमिाइपहॳजनननननलिखितमसहॳककसकहॳ दवारासतरापवतहह८ताहहॴ

24-Jan-2017

Options

1) Kidneys

गद

2) Pancreas

अगनयािय

3) Large Intestine

बड़ीआत

4)Liver

नजगर

Correct Answer Pancreas

Lipase is an enzyme that splits fats so

the intestines can absorb them Lipase

hydrolyzes fats like triglycerides into

their component fatty acid and glycerol

molecules It is found in the blood

gastric juices pancreatic secretions

intestinal juices and adipose tissues

F A C E B O O K

P A G E h t t p w w w f a c e b o o k c o m s s c m e n t o r s o f f i c i a l P a g e | 32

FOR MORE UPDATES AND MORE MATERIAL DO LIKE OUR FACEBOOK PAGE httpwwwfacebookcomsscmentorsofficial

Q134 The arrangement of leaves on an

axis or stem is called

एकअकषयातनहॳपरपनततयोकीयवसिाकह८कयाकहाजाताहहॴ SSC CHSL Science (biology) 2016

Question Paper

24-Jan-2017

Options

1) Phyllotaxy

फाइिह८टहॴकसी 2) Vernation

वनिन

3) Venation

वहॳनहॳिन

4)Phytotaxy

फाइटह८टहॴकसी Correct Answer Phyllotaxy

In botany phyllotaxis or phyllotaxy is

the arrangement of leaves on a plant

stem (from Ancient Greek phyacutellon

ldquoleafrdquo and taacutexis ldquoarrangementrdquo)

Phyllotactic spirals form a distinctive

class of patterns in nature

Q135 The study of Cells is also known

as

कह८लिकाओकहॳ अधययनकह८ mdashmdashndash

भीकहाजाताहहॴ 24-Jan-2017

Options

1) Cytology

सायटह८िह८जी 2) Physiology

कफनजयह८िह८जी 3) Nucleology

नयककमयह८िह८जी 4)Cellology

सहॳिह८िह८जी Correct Answer Cytology

Q136 Which of the following scientists

is also known as the Father of Biology

नननननलिखितमसहॳककसवहॴजञाननककह८ ldquoजीवपवजञानकहॳ जनकrdquoकहॳ नामसहॳभीजानाजाताहहॴ 24-Jan-2017

Options

1) Herbert Spencer

हबयटयसपसर

2) Aristotle

अरसत 3) Lamarck

िहॳमाकय 4)Darwin

िापवयन

Correct Answer Aristotle

Q137 Which cells give rise to various

organs of the plant and keep the plant

growing

कह९नसीकह८लिकाएपह९धह८कहॳ लभननअगह८कह८जनमदहॳतीहहॴऔरपह९धह८कह८बढ़नहॳममददकरतीहहॴ

24-Jan-2017

Options

1) Permanent

सिायी 2) Dermal

तवचीय

3) Meristematic

मररसटहॳमटटक

4)Mature

परह८ढ़

Correct Answer Meristematic

A meristem is the tissue in most plants

containing undifferentiated cells

(meristematic cells) found in zones of

the plant where growth can take place

Q138 Rodentia Muridae is the scientific

name of

F A C E B O O K

P A G E h t t p w w w f a c e b o o k c o m s s c m e n t o r s o f f i c i a l P a g e | 33

FOR MORE UPDATES AND MORE MATERIAL DO LIKE OUR FACEBOOK PAGE httpwwwfacebookcomsscmentorsofficial

रह८िहॳलियानयररिी mdashmdash- कावहॴजञाननकनामहहॴ 24-

Jan-2017

Options

1) Mouse

चहा 2) Squirrel

चगिहरी 3) Monkey

बदर

4) Lizard

नछपकिी Correct Answer Mouse

Q139 Name the scientist who proposed

the cell theory

कह८लिकालसदातकापरसतावदहॳनहॳवािहॳवहॴजञाननककानामबताइए 24-Jan-2017

Options

1) Schleiden and Schwann

िीमिनऔरशरववान

2) Lamarck

िहॳमाकय 3) Treviranus

टरहॳवायरहॳनस

4)Whittaker and Stanley

हीटकरऔरसटहॳनिहॳ Correct Answer Schleiden and

Schwann

Q140 The flower with the worldrsquos

largest bloom is

दननयाकासबसहॳबड़ाफिखििनहॳवािा mdashmdashndash हहॴ 24-Jan-2017

Options

1) Pando

पािह८ 2) Posidonia

पह८सीिह८ननया 3) Rafflesia arnoldii

ररफिहॳलियाअनोमिी 4)Helianthus annuus

हहॳलिएनिसएनयअस

Correct Answer Rafflesia arnoldii

Rafflesia arnoldii is a species of

flowering plant in the parasitic genus

Rafflesia It is noted for producing the

largest individual flower on earth It has

a very strong and horrible odour of

decaying flesh earning it the nickname

ldquocorpse flower

Q141 Deficiency of which vitamin

causes night blindness

ककसपवटालमनकीकमीकहॳ कारणरतौधीहह८ताहहॴ 24-Jan-2017

Options

1) Vitamin K

पवटालमन K

2) Vitamin C

पवटालमन C

3) Vitamin B1

पवटालमन B1

4)Vitamin A

पवटालमन A

Correct Answer Vitamin A

Q142 Nongreen plants lack which of the

following

गहॴर-

हररतवनसपनतमनननननलिखितमसहॳककसकीकमीहह८तीहहॴ

24-Jan-2017

Options

1) Chlorophyll

किह८रह८कफि

2) Lycophyll

िायकह८कफि

3) Cyanophyll

F A C E B O O K

P A G E h t t p w w w f a c e b o o k c o m s s c m e n t o r s o f f i c i a l P a g e | 34

FOR MORE UPDATES AND MORE MATERIAL DO LIKE OUR FACEBOOK PAGE httpwwwfacebookcomsscmentorsofficial

सायनह८कफि

4)Phototropism

फह८टह८टरोपपजम

Correct Answer Chlorophyll

Q143 Organisms that use light to

prepare food are known as

जह८जीवपरकािकाउपयह८गकरभह८जनतहॴयारकरतहॳहहॴ उनह mdashmdash- कहॳ पमजानजाताहहॴ 24-Jan-2017

Options

1) Autotrophs

सवपह८षी 2) Heterotrophs

पवषमपह८षज

3) Omnivores

सवायहारी 4)Decomposers

पवघटनकरनहॳवािा Correct Answer Autotrophs

autotrophs often make their own food

by using sunlight carbon dioxide and

water to form sugars which they can use

for energy Some examples of

autotrophs include plants algae and

even some bacteria Autotrophs

(producer) are important because they

are a food source for heterotrophs

(consumers)

A heterotroph is an organism that

ingests or absorbs organic carbon

(rather than fix carbon from inorganic

sources such as carbon dioxide) in order

to be able to produce energy and

synthesize compounds to maintain its

life Ninety-five percent or more of all

types of living organisms are

heterotrophic including all animals and

fungi and some bacteria

Q144 Which of the following is a

primary function of haemoglobin

नननननलिखितमसहॳकह९नसाटहमह८गिह८बबनकाएकपरािलमककाययहहॴ

25-Jan-2017

Options

1) Utilization of energy

उजायकाउपयह८गकरना 2) Prevention of anaemia

रकतामपताहह८नहॳसहॳरह८कना 3) Destruction of bacteria

बहॴकटीररयाकापवनािकरना 4) To transport oxygen

ऑकसीजनकावहनकरना Correct Answer To transport oxygen

Q145 Vascular bundles are absent in

सवहनीबिि mdashmdash- मअनपनसतिरहतहॳहहॴ 25-Jan-2017

Options

1) Bryophyta

िायह८फाइटा 2) Pteridophyta

टहॳररिह८फाईटा 3) Gymnosperms

नजननह८सपमय 4) Angiosperms

एननजयह८सपहॳनसय Correct Answer Bryophyta

Q146 Sauria Lacertidae is the scientific

name of

सहॴररयािहॳसरटाईिी mdashmdashndash कावहॴजञाननकनामहहॴ 25-Jan-2017

Options

1) Crocodile

मगरमचछ

2) Hippopotamus

टहपपह८पह८टहॳमस

3) Lizard

नछपकिी 4) House fly

F A C E B O O K

P A G E h t t p w w w f a c e b o o k c o m s s c m e n t o r s o f f i c i a l P a g e | 35

FOR MORE UPDATES AND MORE MATERIAL DO LIKE OUR FACEBOOK PAGE httpwwwfacebookcomsscmentorsofficial

घरहॳिमकिी Correct Answer Lizard

Q147 Which type of pathogen causes

the water-borne disease SARS (Severe

Acute Respiratory Syndrome)

ककसपरकािकारह८गज़नकजिजननतबीमारीसासयकाकारणबनताहहॴ 25-Jan-2017

Options

1) Viral

वायरि

2) Parasitic

परजीवी 3) Protozoan

परह८टह८जअन

4) Bacterial

बहॴकटीररयि

Correct Answer Viral

Q148 Which of the following organs

produces the enzyme lipase

नननननलिखितमसहॳकह९नसाअगिायपहॳजएजाइमउतपननकरताहहॴ 25-Jan-2017

Options

1) Pancreas

अगनयािय

2) Large Intestine

बड़ीआत

3) Liver

नजगर

4) Small Intestine

छह८टीआत

Correct Answer Pancreas

Q149 A is a long internode forming the

basal part or the whole of a peduncle

एक mdashmdash- एकिबाइटरनह८िहहॴ जह८ननचिाटहससायासनपणयिठिबनताहहॴ 25-

Jan-2017

Options

1) Rhizome

परकद

2) Rachis

महॳ दि

3) floral axis

पषपअकष

4) Scape

भगदड़

Correct Answer scape

Q150 ndash Which of the following

organisms are considered to be both

Living and Non-living

नननननलिखितमसहॳकह९नसहॳजीवाणकह८जीपवतऔरअजीपवतमानाजाताहहॴ

25-Jan-2017

Options

1) Bacteria

बहॴकटीररया 2) Fungi

कवक

3) Algae

िहॴवाि

4)Virus

वायरस

Correct Answer Virus

They are considered to be living as they

possess a protein coat as a protective

covering DNA as the genetic material

etc

They are said to be non-living as they

can be crystallised and they survive for

billions of years They can tolerate high

temperatures freezing cold

temperatures ultra-violet radiations etc

Q151 Deficiency of fluorine causes

which of the following

फिह८ररनकीकमीकहॳ कारणनननननलिखितमसहॳकयाहह८ताहहॴ

F A C E B O O K

P A G E h t t p w w w f a c e b o o k c o m s s c m e n t o r s o f f i c i a l P a g e | 36

FOR MORE UPDATES AND MORE MATERIAL DO LIKE OUR FACEBOOK PAGE httpwwwfacebookcomsscmentorsofficial

27-Jan-2017

Options

1) Dental Caries

िटिकहॴ ररज

2) Scurvy

सकवरी 3) Anaemia

रकतामपता 4) Arthritis

गटठया Correct Answer Dental Caries

Q152 In a Punnett Square with the

cross AaBb x AaBb how many Aabb

genotypes would be created

पनहॳटसककायरमिह८स AaBb x AaBb कहॳ साि

ककतनहॳ Aabb जीनह८टाइपबनगहॳ 27-Jan-2017

Options

1) 1

2) 8

3) 2

4) 3

Correct Answer 2

Q153 Which of the following is the

Controlling Center of the Cell

नननननलिखित म सहॳ कह८लिकाका ननयतरण

क दर कह९न हहॴ

27-Jan-2017

Options

1) Nucleus

क दर

2) Plasma

पिाजमा 3) Lysosome

िायसह८सह८म

4) Chromosome

िह८मह८सह८म

Correct Answer Nucleus

The control centre of the cell is the

nucleus in eukaryotic cells The nucleus

contains genetic material in the form of

DNA

Q154 Myopia affects which of the

following organs

मायह८पपयानननननलिखितअगह८मसहॳककसहॳपरभापवतकरताहहॴ

25-Jan-2017

Options

1) Heart

हदय

2) Skin

तवचा 3) Eyes

आािहॳ 4)Mouth

मह

Correct Answer Eyes

Q155 Which of the following bears

flowers

नननननलिखितमसहॳकह९नफिधारणकरताहहॴ

25-Jan-2017

Options

1) Bryophyta

िायह८फाइटा 2) Pteridophyta

टहॳरीिह८फाईटा 3) Gymnosperms

नजननह८सपमय 4)Angiosperms

एननजयह८सपमय Correct Answer Angiosperms

Q156 Oxygenated blood flows out of the

heart through the

ऑकसीजनयकतरकत mdashmdashmdash

कहॳ माधयमसहॳहदयकहॳ बाहरबहताहहॴ 25-Jan-2017

F A C E B O O K

P A G E h t t p w w w f a c e b o o k c o m s s c m e n t o r s o f f i c i a l P a g e | 37

FOR MORE UPDATES AND MORE MATERIAL DO LIKE OUR FACEBOOK PAGE httpwwwfacebookcomsscmentorsofficial

Options

1) Aorta

महाधमनी 2) pulmonary artery

फहॳ फड़हॳकीधमनी 3) vena cava

वहॳनाकावा 4)Atrium

चह९क

Correct Answer aorta

Q157 Blood leaving the liver and

moving towards the

heart has a higher concentration of

नजगरसहॳननकिकरहदयकीतरफजानहॳवािहॳरकतम mdashmdashmdashmdash कीउचचसादरताहह८तीहहॴ 27-Jan-2017

Options

1) Lipids

लिपपडस

2) Urea

यररया 3) Bile Pigments

पपततकहॳ रगकरण

4) Carbon dioxide

काबयनिायऑकसाइि

Correct Answer Bile Pigments

Urea is nitrogen containing substance

which is produced in the liver in order

to deal with excess amino-acids in the

body As urea is produced it leaves the

liver in the blood stream and passes via

the circulatory system to all parts of the

body

Q158 Bulb is a modification of which

part of a plant

बमबएकपह९धहॳकहॳ ककसटहससहॳकाएक पातरणहह८ताहहॴ 27-Jan-2017

Options

1) The root

जड़

2) The stem

तना 3) The radicle

मिाकर

4)The fruit

फि

Correct Answer The stem

Q159 Which of the following carries

blood away from the heart to different

body parts

इनमहॳसहॳकह९नरकतकह८हदयसहॳिरीरकहॳ पवलभननअगह८तकिहॳजातीहहॴ

27-Jan-2017

Options

1) Arteries

धमननया 2) Nerves

तबतरहाए

3) Capillaries

कहॳ लिकाए

4)Veins

नसहॳ Correct Answer Arteries

Q160 The series of processes by which

nitrogen and its compounds are

interconverted in the environment and

in living organisms is called

27-Jan-2017

Options

1)Absorption of Nitrogen

2)Ammonification

3)Nitrogen Fixation

4)Nitrogen Cycle

Correct Answer Nitrogen Cycle

Ammonification or Mineralization is

performed by bacteria to convert

organic nitrogen to ammonia

F A C E B O O K

P A G E h t t p w w w f a c e b o o k c o m s s c m e n t o r s o f f i c i a l P a g e | 38

FOR MORE UPDATES AND MORE MATERIAL DO LIKE OUR FACEBOOK PAGE httpwwwfacebookcomsscmentorsofficial

Nitrification can then occur to convert

the ammonium to nitrite and nitrate

Nitrogen fixation is a process by which

nitrogen in the Earthrsquos atmosphere is

converted into ammonia (NH3) or other

molecules available to living organisms

Q161 BCG vaccine is given to protect

from which of the following

बीसीजीकाटटकानननननलिखितमसहॳककसकहॳ बचावकहॳ लिएटदयाजातहहॴ

27-Jan-2017

Options

1) Jaundice

पीलिया 2) Anaemia

रकतमपता 3) Tuberculosis

कषयरह८ग

4) Polio

पह८लियह८ Correct Answer Tuberculosis

Q162 Parallel venation is found in

समानतरवहॳनहॳिन mdashmdashmdash- मपायाजाताहहॴ 27-Jan-2017

Options

1) plants which are monocots

पह९धहॳजह८एकबीजपतरीहह८तहॳहहॴ 2) plants which have a dicot stem

वहॳपह९धहॳनजनकातनादपवदलियहह८ताहहॴ 3) plants with leaves similar to Tulsi

वहॳपह९धहॳनजनकीपनततयतिसीकीपनततयोकहॳ समानहह८तहॳहहॴ 4)plants with tap roots

टहॳप टवािहॳपह९धहॳ Correct Answer plants which are

monocots

Q163 The hardest part of the body is

िरीरकासबसहॳकठह८रभाग mdashndash हहॴ 27-Jan-2017

Options

1) Bones

हडडिय

2) Tooth Enamel

दातकहॳ इनहॳमि

3) Skull

िह८पड़ी 4) Spinal Cord

महॳ रजज

Correct Answer Tooth Enamel

Q164 Which type of pathogen causes

the waterborne disease E coli Infection

ककसपरकारकारह८गजननकजिजननतरह८गईकह८िाईसिमणकाकारणबनताहहॴ 27-Jan-2017

Options

1) Protozoan

परह८टह८जआ

2) Parasitic

परजीवी 3) Bacterial

बहॴकटीररयि

4)Viral

वायरि

Correct Answer Bacterial

Q165 The amount of blood filtered

together by both the kidneys in a 70 kg

adult male human in a minute is

70 की गरा वािहॳएकवयसकप षमएकलमनटमदह८नोगदकहॳदवाराएकसािचाबनीगयीरकतकीमातरहह८तीहहॴ 29-Jan-2017

Options

1) 1100 ml

1100 लमलि

2) 100 ml

F A C E B O O K

P A G E h t t p w w w f a c e b o o k c o m s s c m e n t o r s o f f i c i a l P a g e | 39

FOR MORE UPDATES AND MORE MATERIAL DO LIKE OUR FACEBOOK PAGE httpwwwfacebookcomsscmentorsofficial

100 लमलि

3) 1500 ml

1500 लमलि

4) 500 ml

500 लमलि

Correct Answer 1100 ml

Q166 Which feature of a plant helps to

distinguish a monocot from a dicot

पह९धहॳकीवहकह९नसीपविहॳषताहहॴजह८एकदपवदलियहॳऔरएकएकदिीयपह९धहॳसहॳभहॳदकरनहॳममददकरतीहहॴ 29-Jan-2017

Options

1) Pollination

परागम

2) Venation

वहॳनहॳिन

3) Vernation

वनिन

4) Aestivation

एसटीवहॳिहॳन

Correct Answer venation

Q167 The Mutation Theory was

proposed by

उतवररवतयनकालसदात mdashmdashndash

कहॳ दवरापरसतापवतककयाजाताहहॴ 29-Jan-2017

Options

1) Charles Lyell

चामसयलियहॳि

2) William Smith

पवलियमनसमि

3) Hugo De Vries

हयगह८िीराईस

4)Harrison Schmitt

हहॳरीसननसमट

Correct Answer Hugo De Vries

Q168 Which type of pathogen causes

the waterborne disease HepatitisA

ककसपरकारकहॳ रह८गजनकजिजननतरह८गहहॳपहॳटाइटटस-A काकारणबनताहहॴ

29-Jan-2017

Options

1) Parasitic

परजीवी 2) Viral

वायरि

3) Protozoan

परह८टह८जआ

4) Bacterial

बहॴकटीररयि

Correct Answer Viral

Q169 In a Punnett Square with the

cross AaBb x Aabb how many AaBb

genotypes would be created

पनहॳटसकवायरमिह८स AaBb x Aabb

कहॳ सािककतनहॳ AaBb जीनह८टाइपबनगहॳ 29-Jan-

2017

Options

1) 4

2) 1

3) 7

4) 6

Correct Answer 4

Q170 Arboreal Ateles is the scientific

name of

अिह८ररयिएटटलिस mdashmdashmdash कावहॴजञाननकनामहहॴ 29-Jan-2017

Options

1) Squirrel

चगिहरी 2) Sparrow

गह८रहॴया 3) Lizard

नछपकिी 4) Spider monkey

F A C E B O O K

P A G E h t t p w w w f a c e b o o k c o m s s c m e n t o r s o f f i c i a l P a g e | 40

FOR MORE UPDATES AND MORE MATERIAL DO LIKE OUR FACEBOOK PAGE httpwwwfacebookcomsscmentorsofficial

मकड़ीबदर

Correct Answer Spider monkey

Q171 Which type of pathogen causes

the waterborne disease Salmonellosis

ककसपरकारकारह८गाणजिजननतबीमारीसािमह८नहॳिह८लसज़काकारकहहॴ

29-Jan-2017

Options

1) Algal

िहॳवालियहॳ 2) Parasitic

परजीवी 3) Bacterial

बहॴकटीररयि

4)Viral

वायरि

Correct Answer Bacterial

An infection with salmonella bacteria

commonly caused by contaminated food

or water

Symptoms include diarrhoea fever

chills and abdominal pain

Q172 is a condition in which there is a

deficiency of red cells or of haemoglobin

in the blood

mdashmdash-

एकनसिनतहहॴनजसमहॳरकतमिािकह८लिकाओकीयाहीमह८गिह८बबनकीकमीहह८तीहहॴ 29-Jan-2017

Options

1) Albinism

एनमबननजम

2) Propyria

परह८पीररया 3) Anaemia

एनीलमया 4)Keloid disorder

कहॳ िह८इिडिसओिर

Correct Answer Anaemia

Q173 Ananas comosus is the scientific

name of

Options

अनानासकह८मह८सस mdashmdashmdashndash

कावहॴजञाननकनामहहॴ 29-Jan-2017

1) Custard Apple

सीताफि

2) Pineapple

पाइनएपपि

3) Bamboo

बास

4)Pomegranate

अनार

Correct Answer Pineapple

Q174 Which organ produces insulin

कह९नसाअगइनसलिनपहॴदाकरताहहॴ 29-Jan-

2017

Options

1) Liver

यकत

2) Thyroid gland

िायराइिगरिी 3) Spleen

पिीहा 4)Pancreas

अगरयिय

Correct Answer Pancreas

Q175 Which of the following disease is

not caused by water pollution

नननननलिखितमसहॳकह९नसारह८गपानीकहॳ परदषणकहॳकारणनहीहह८ता

29-Jan-2017

Options

1) Cholera

हहॴजा 2) Typhoid

F A C E B O O K

P A G E h t t p w w w f a c e b o o k c o m s s c m e n t o r s o f f i c i a l P a g e | 41

FOR MORE UPDATES AND MORE MATERIAL DO LIKE OUR FACEBOOK PAGE httpwwwfacebookcomsscmentorsofficial

टाइफाइि

3) Asthma

दमा 4)Diarrhoea

दसत

Correct Answer Asthma

Q176 Ocimum tenuiflorum is the

scientific name of

ओलिलममटहॳयईफिह८रमइसकावहॴजञाननकनाम mdash

ndash हहॴ 30-Jan-2017

Options

1) Neem

नीम

2) Mango

आम

3) Babul

बबि

4)Tulsi

तिसी Correct Answer Tulsi

Q177 Which gland secretes bile a

digestive fluid

कह९नसीगरिीपपतत एकपाचनतरिपरदािय सरापवतकरतीहहॴ 30-Jan-2017

Options

1) Pancreas

अगनयािय

2) Liver

यकत

3) Thyroid

िायराइि

4) Testes

टहॳनसटस

Correct Answer liver

Q178 In which of the following the

dominant phase is Gametophyte

नननननलिखितमसहॳककसकहॳ परमिचरणयगमकह८दपवधद (Gametophyte)हहॴ 30-Jan-2017

Options

1) Bryophyta

िायह८फाइटा 2) Pteridophyta

टहॳररिह८फाइटा 3) Gymnosperms

नजननह८सपमय 4) Angiosperms

एननजयह८सपमय Correct Answer Bryophyta

Q179 Anaerobic respiration refers to

which of the following

नननननलिखितमसहॳककसहॳअवायवीयशवसनकहाजाताहहॴ

30-Jan-2017

Options

1) Respiration without Oxygen

ऑकसीजनकहॳ बबनाशवसन

2) Respiration with Oxygen

ऑकसीजनकहॳ सािशवसन

3) Respiration without CO2

काबयनिायऑकसाइिकहॳ बबनाशवसन

4) Respiration with CO2

काबयनिायऑकसाइिकहॳ सािशविन

Correct Answer Respiration without

Oxygen

Q180 Which type of pathogen causes

the waterborne disease Cholera

ककसपरकारकारह८गजनकजिजननतरह८गहहॴजाकाकारणबनताहहॴ

30-Jan-2017

Options

1) Algal

िहॴवालियहॳ

F A C E B O O K

P A G E h t t p w w w f a c e b o o k c o m s s c m e n t o r s o f f i c i a l P a g e | 42

FOR MORE UPDATES AND MORE MATERIAL DO LIKE OUR FACEBOOK PAGE httpwwwfacebookcomsscmentorsofficial

2) Bacterial

बहॴकटीररयि

3) Protozoan

परह८टह८जआ

4) Viral

वायरि

Correct Answer Bacterial

Q181 To which class does

Oxyreductases transferases hydrolases

belong

ओकसीररिकटहॳसटरासफरहॳजहॳस

हाइडरह८िहॳसहॳसककसवगयमआतहॳहहॴ 30-Jan-2017

Options

1) Hormones

हारमोस

2) Enzymes

एजाइनस

3) Proteins

परह८टीनस

4) Vitamins

पवटालमनस

Correct Answer Enzymes

Q182 Which of the following is not true

about Gymnosperms

ननननमसहॳकह९नसीबातअनावतबीजीकहॳ बारहॳमसचनहीहहॴ 30-Jan-2017

Options

1) Dominant phase is saprophytes

परमिचरणसहॳपरह८फाइटसहह८ताहहॴ 2) Vascular bundles are absent

सवहनीबििअनपनसितहह८ताहहॴ 3) spores are heterospores

बीजाणहहॳटहॳरह८सपह८रसहह८तहॳहहॴ 4) Flowers are absent

फिअनपनसितहह८तहॳहहॴ

Correct Answer Vascular bundles are

absent

Q183 The name of first mammal clone sheep is

भहॳड़कीपरिमसतनपायीपरनत प (किह८न)

कानामहहॴ 30-Jan-2017

Options

1) Noori

नरी 2) Dolly

िॉिी 3) Louise

िसी 4)Durga

दगाय Correct Answer Dolly

Q184 Which type of pathogen causes

the water-borne disease Typhoid fever

ककसपरकारकारह८गजनकजिजननतरह८गटाइफाइिबिारकाकारणबनताहहॴ 30-Jan-2017

Options

1) Algal

िहॴवािीय

2) Parasitic

परजीवी 3) Protozoan

परह८टह८जनअन

4)Bacterial

बहॴकटीररयि

Correct Answer Bacterial

Q185 In which part of the cell are

proteins made

कह८लिकाकहॳ ककसटहससहॳमपरह८टीनबनायाजाताहहॴ

31-Jan-2017

Options

1) Reticulum

रहॳटटकिम

F A C E B O O K

P A G E h t t p w w w f a c e b o o k c o m s s c m e n t o r s o f f i c i a l P a g e | 43

FOR MORE UPDATES AND MORE MATERIAL DO LIKE OUR FACEBOOK PAGE httpwwwfacebookcomsscmentorsofficial

2) Golgi apparatus

गह८मजीएपहॳरहॳटस

3) Ribosomes

ररबह८सह८नस

4) Lysosome

िायसह८सह८नस

Correct Answer ribosomes

Proteins are produced by stringing

amino acids together in the order

specified by messenger RNA strands

that were transcribed from DNA in the

cell nucleus The process of synthesizing

a protein is called translation and it

occurs on ribosomes in the cytoplasm of

a cell

Q186 Polio is a disease caused by which

of the following

नननननलिखितमसहॳपह८लियह८कीबबमारह८हह८नहॳकाकारणकयाहहॴ

31-Jan-2017

Options

1) Bacteria

बहॴकटीररयि

2) Mosquito

मचछर

3) Virus

वायरस

4) Cockroach

नतिच हॳ Correct Answer Virus

Polio or poliomyelitis is a crippling and

potentially deadly infectious disease It

is caused by the poliovirus

Q187 ndash Hay fever is a sign of which of

the following

हहॳकफवरनननननलिखितमसहॳककसकाएकसकहॳ तहहॴ

31-Jan-2017

Options

1) Old Age

वदावसिा 2) Malnutrition

कपह८सण

3) Allergy

एिनजय 4) Over Work

अतयचधककाययकरना Correct Answer Allergy

Q188 How many chromosomes does a

human cell contain

एकमानवकह८लिकामककतनहॳगणसतरहह८तहॳहहॴ

29-Jan-2017

Options

1) 6

2) 26

3) 46

4) 66

Correct Answer 46

In humans each cell normally contains

23 pairs of chromosomes for a total of

46 Twenty-two of these pairs called

autosomes look the same in both males

and females The 23rd pair the sex

chromosomes differ between males and

females

Q189 Which of the following is not true

about Bryophyta

ननननमसहॳकह९नसीबातिायह८फाइटकहॳ बारहॳमसचनहीहहॴ 31-Jan-2017

Options

1) Dominant phase is gametophytes

परमिचरणगहॳलमतह८फाइटसहह८ताहहॴ 2) Main plant body is haploid

पह९धहॳकामखयिरीरअगखणतहह८ताहहॴ 3) Spores are homospores

बीजाणहह८मह८सफह८रसहह८तहॳहहॴ 4) Flowers are present

फिमह८जदहह८तहॳहहॴ Correct Answer Flowers are present

F A C E B O O K

P A G E h t t p w w w f a c e b o o k c o m s s c m e n t o r s o f f i c i a l P a g e | 44

FOR MORE UPDATES AND MORE MATERIAL DO LIKE OUR FACEBOOK PAGE httpwwwfacebookcomsscmentorsofficial

Q190 Which aquatic animal has

trailing tentacles

ककसजिीयजानवरकहॳ पीछहॳचिनहॳवािहॳटहॳटकिसहह८तहॳहहॴ

31-Jan-2017

Options

1) Sea horse

समदरीघह८िा 2) Corals

मगा 3) Jelly fish

जहॳिीमछिी 4) Star fish

तारामछिी Correct Answer Jelly fish

Jellyfish with its umbrella-shaped bell

and trailing tentacles

Q191 Which type of pathogen causes

the water-borne disease Poliomyelitis

(Polio)

ककसपरकारकारह८गजनकजिजननतरह८गपह८लियह८मायहॳटटस (पह८लियह८) काकारणहहॴ 31-Jan-

2017

Options

1) Parasitic

परजीवी 2) Algal

िहॴवालिय

3) Viral

वायरि

4) Bacterial

बहॴकटीररयि

Correct Answer Viral

Q192 The outer white part of the eye

that protects the inner structures is

आािकाबाहरीसफहॳ दटहससाजह८आतररकसरचनाओकीरकषाकरताहहॴ वह mdashmdashmdash हहॴ 31-Jan-

2017

Options

1) Iris

आयररस

2) Sclera

सकिहॳरा 3) Retina

रहॳटटना 4) Cornea

कह८ननयया Correct Answer Sclera

Q193 Proteins are made up of

परह८टीनकाननमायण mdashndash सहॳहह८ताहहॴ 31-Jan-2017

Options

1) Amino acids

एलमनह८अनि

2) Fatty acids

वसायकतअनि

3) Glucose

गिकह८ज

4)Nucleotides

नयनकियह८टाईिस

Correct Answer Amino acids

Q194 Moringa Oleifera is the scientific

name of

मह८ररगओलिफहॳ रा mdashmdashndash कावहॴजञाननकनामहहॴ 31-Jan-2017

Options

1) Banyan

बरगद

2) Gulmohar

गिमह८हर

3) Amla

आमिा

F A C E B O O K

P A G E h t t p w w w f a c e b o o k c o m s s c m e n t o r s o f f i c i a l P a g e | 45

FOR MORE UPDATES AND MORE MATERIAL DO LIKE OUR FACEBOOK PAGE httpwwwfacebookcomsscmentorsofficial

4) Drumstick

डरमनसटक

Correct Answer Drumstick

Q195 Kidney stones are composed of

गदकीपिरी mdashndash सहॳबनीहह८तीहहॴ 1-Feb-2017

Options

1) Calcium Oxalate

कहॴ नमसयमओकजहॳिहॳट

2) Sodium Chloride

सह८डियमकिह८राइि

3) Magnesium Nitrate

महॳनगनलियमनाइतटरहॳट

4) Calcium Bicarbonate

कहॴ नमियमबायकबोनहॳट

Correct Answer Calcium Oxalate

Q196 ndash Which of the following is not

true about Angiosperms

ननननमसहॳकह९नसीबातआवतबीजीकहॳ बारहॳमसचनहीहहॴ 1-Feb-2017

Options

1) Dominant phase is gametophytes

परमिचरणगहॳलमतह८फाइटहह८ताहहॴ 2) Vascular bundles are present

सवहनीबििमह९जदहह८ताहहॴ 3) Spores are heterospores

बीजाणहहॳटहॳरह८सपह८रसहह८तहॳहहॴ 4) Seeds are covered

बीजढकहॳ हह८तहॳहहॴ Correct Answer Dominant phase is

gametophytes

Q197 All of the following are excretory

(waste) products of animals except

नननननलिखितमसहॳककसएककह८छह८ड़करअनयसभीपराखणयोदवाराउतसनजयतपदाियहहॴ 1-Feb-

2017

Options

1) Uric Acid

यररकएलसि

2) Ammonia

अमह८ननया 3) Carbohydrates

काबोहाइडरहॳट

4) Urea

यररया Correct Answer Carbohydrates

In animals the main excretory products

are carbon dioxide ammonia (in

ammoniotelics) urea (in ureotelics) uric

acid (in uricotelics) guanine (in

Arachnida) and creatine

Q198 RNA is a polymeric molecule

What does RNA stand for

आरएनइएएकबहिकआणहहॴ इसकाकापवय पकयाहहॴ 1-Feb-2017

Options

1) Rado Nuclear Acid

रािह८नयनकियरएलसि

2) Ribo Nucleic Acid

राइबह८नयनकिकएलसि

3) Rhino Nuclear Acid

हाइनह८नयनकियरएलसि

4) Resto Nucleus Acid

रहॳसटह८नयकिीयसएलसि

Correct Answer Ribo Nucleic Acid

Q199 Which organ does detoxification

and produces chemicals needed for

digestion

कह९नसाअगपवषहरणकरताहहॴऔरपाचनकहॳ लिएआवशयकरसायनोकह८पहॴदाकरताहहॴ 1-Feb-

2017

Options

1) Salivary glands

िारगरचिया 2) Pancreas

अगनयािय

F A C E B O O K

P A G E h t t p w w w f a c e b o o k c o m s s c m e n t o r s o f f i c i a l P a g e | 46

FOR MORE UPDATES AND MORE MATERIAL DO LIKE OUR FACEBOOK PAGE httpwwwfacebookcomsscmentorsofficial

3) Thyroid gland

िायराइिगरिी 4) Liver

यकत

Correct Answer Liver

Q200 Psidium guajava is the scientific

name of

लसडियमगआजावा mdashmdash कावहॴजञाननकनामहहॴ 1-

Feb-2017

Options

1) Guava

अम द

2) Mango

आम

3) Bamboo

बास

4) Jack fruit

कटहि

Correct Answer Guava

Q201 Which drug is used as a Blood

Thinner

चधरकह८पतिाकरनहॳकहॳ पमककसदवाकापरयह८गककयाजाताहहॴ

1-Feb-2017

Options

1) Warfarin

वाफर न

2) Tramadol

टरहॳमािह८ि

3) Azithromycin

एनजरह८मायलसन

4) Hydralazine

हाइडरह८िहॳनजन

Correct Answer Warfarin

Q202 Which of the following disease is

caused due to the deficiency of protein

परह८टीनकीकमीकहॳ कारणनननननलिखितमसहॳकह९नसारह८गहह८ताहहॴ 1-Feb-2017

Options

1) Arthritis

गटठया 2) Kwashiorkor

कािीओकय र

3) Goitre

गाइटर

4) Night Blindness

रतह९चध

Correct Answer Kwashiorkor

Q203 A is species of plant that has

adapted to survive in an environment

with little liquid water

mdashmdashndashपह९धहॳकीएकऐसहॳऐसहॳपरजानतहहॴ नजसनहॳकमपानीवािहॳवातावरणमजीपवतरहनहॳकहॳलिएअनकिनहहॴ 1-Feb-2017

Options

1) Xerophyte

म दपवद

2) Hydrophyte

जिीयपादप

3) Mesophyte

समह८दपवद

4) Thallophyte

िहॴिह८फाइटा Correct Answer xerophyte

xerophyte is a species of plant that has

adapted to survive in an environment

with little liquid water such as a desert

or an ice- or snow-covered region in the

Alps or the Arctic

Mesophytes are terrestrial plants which

are adapted to neither a particularly

dry nor particularly wet environment

An example of a mesophytic habitat

would be a rural temperate meadow

F A C E B O O K

P A G E h t t p w w w f a c e b o o k c o m s s c m e n t o r s o f f i c i a l P a g e | 47

FOR MORE UPDATES AND MORE MATERIAL DO LIKE OUR FACEBOOK PAGE httpwwwfacebookcomsscmentorsofficial

which might contain goldenrod clover

oxeye daisy and Rosa multiflora

thallophyte any of a group of plants or

plantlike organisms (such as algae and

fungi) that lack differentiated stems

leaves and roots and that were formerly

classified as a primary division

(Thallophyta) of the plant kingdom

Q204 How many types of teeth are

there in humans

मनषयोमककतनहॳपरकारकहॳ दातहह८तहॳहहॴ

1-Feb-2017

Options

1) 4

2) 5

3) 2

4) 3

Correct Answer 4

teeth -Humans have four types of

teethincisors canines premolars and

molars each with a specific function

The incisors cut the food the canines

tear the food and the molars and

premolars crush the food

Q205 Carica papaya is the scientific name of

कहॴ ररकापपाया mdashmdashndash कावहॴजञाननकनामहहॴ 2-

Feb-2017

Options

1) Peepal

पीपि

2) Papaya

पपीता 3) Tamarind

इमिी 4) Drumstick

ढह८िकाछड़ी Correct Answer Papaya

Q206 Muscles get tired when there is

shortfall of

जब mdashndash कीकमीहह८तीहहॴतबपहॳिीयिकजातीहहॴ 2-Feb-2017

Options

1) Lactic acid

िहॴनकटकएलसि

2) Na+ ions

Na+ आयन

3) ATP

एटीपी 4) Sulphates

समफहॳ टस

Correct Answer ATP

ATP is the energy source muscle fibers

use to make muscles contract

muscle tissuersquos main source of energy

called adenosine triphosphate or ATP

As your muscles use up this energy

source they become tired and fatigued

Oxygen is the key ingredient that helps

create new ATP to replenish the burned

up ATP in your muscles

Q207 Artocarpus integra is the

scientific name of आटह८कापयसइटीगरा mdashmdashmdash कावहॴजञाननकनामहहॴ 2-Feb-2017

Options

1) Guava

अम द

2) Pineapple

अनानास

3) Silver Oak

लसमवरओक

4) Jack fruit

कटहि

Correct Answer Jack fruit

Q208 Which organ stores fat soluble

vitamins

कह९नसाअगवसामघिनिीिपवटालमनह८काभिाराकरताहहॴ

2-Feb-2017

F A C E B O O K

P A G E h t t p w w w f a c e b o o k c o m s s c m e n t o r s o f f i c i a l P a g e | 48

FOR MORE UPDATES AND MORE MATERIAL DO LIKE OUR FACEBOOK PAGE httpwwwfacebookcomsscmentorsofficial

Options

1) Blood

रकत

2) Skin

तवचा 3) Liver

यकत

4) Pancreas

अगनयािय

Correct Answer Liver

Q209 Which disease is caused due to

deficiency of Iodine

आयह८िीनकहॳ कारणकह९नसारह८गहह८ताहहॴ 2-Feb-2017

Options

1) Rickets

ररकहॳ टस

2) Scurvy

सकवी 3) Goitre

गणमािा 4) Growth retardation

पवकासका कना Correct Answer Goitre

rickets A softening and weakening of

bones in children usually due to

inadequate vitamin D

Q210 Grevillea Robusta is the scientific name of

गरहॳपवलियारह८बसटा mdashmdashmdash- कापवजञाननकनामहहॴ 2-Feb-2017

Options

1) Peepal

पीपि

2) Teak

सागह९न

3) Silver Oak

लसमवरओक

4) Jack fruit

कटहि

Correct Answer Silver Oak

Q211 When a Cuttlefish is described as a Molluscs it is at which level of

classification

जबएककटिकफिकह८एकमह८िसकाकहॳ पमवखणयतककयाजाताहहॴतबयहॳवगीकरणकहॳ ककससतरपहॳनसितहहॴ 2-Feb-2017

Options

1) Class

वगय 2) Order

िम

3) Family

पररवार

4) Phylum

सघ

Correct Answer Phylum

Q212 Bambusa dendrocalmus is the

scientific name of बानबसािहॳडराकामस mdashmdashmdash कावहॴजञाननकनामहहॴ 3-Feb-2017

Options

1) Banyan

बरगद

2) Papaya

पपीता 3) Bamboo

बास

4) Pomegranate

अनार

Correct Answer Bamboo

Q213 Acinonyx Jubatus is the scientific name of

एलसनह८ननकसजयबहॳटस mdashmdashmdash

कावहॴजञाननकनामहहॴ 3-Feb-2017

F A C E B O O K

P A G E h t t p w w w f a c e b o o k c o m s s c m e n t o r s o f f i c i a l P a g e | 49

FOR MORE UPDATES AND MORE MATERIAL DO LIKE OUR FACEBOOK PAGE httpwwwfacebookcomsscmentorsofficial

Options

1) Bear

भाि 2) Horse

घह८िा 3) Cheetah

चीता 4) Zebra

जहॳिा Correct Answer Cheetah

Q214 The pale yellow colour of urine is

due to the presence of which pigment

मतरकाफीकापीिारगरगदरयकहॳ उपनसिनतकहॳ कारणहह८ताहहॴ

3-Feb-2017

Options

1) Urochrome

यरह८िह८म

2) Urophyll

यरह८कफि

3) Chlorophyll

किह८रह८कफि

4) Chloroplast

किह८रह८पिासट

Correct Answer Urochrome

Q215 Which of the following constitute

to form a gene

नननननलिखितमसहॳकह९नसीचीज़एकजीनकागठनकरतीहहॴ

3-Feb-2017

Options

1) Polynucleotides

पह८िीनयनकियह८टाईडस

2) Hydrocarbons

हाइडरह८काबोस

3) Lipoproteins

िाईपह८परह८टीनस

4) Lipids

लिपपडस

Correct Answer Polynucleotides

Polynucleotide molecule is a biopolymer

composed of 13 or more nucleotide

monomers covalently bonded in a chain

DNA (deoxyribonucleic acid) and RNA

(ribonucleic acid) are examples of

polynucleotides with distinct biological

function

Q216 Vertebrates belongs to the

phylum

रीढ़कीहडिीवािहॳपराणी mdashmdashmdash

परजानतकहॳ अतगायतआतहॳहहॴ 3-Feb-2017

Options

1) Arthropoda

आरह८पह८ड़ा 2) Annelida

एननलििा 3) Cnidaria

ननिहॳररया 4) Chordata

कह८िटा Correct Answer Chordata

Q217 Punica granatum is the scientific name of

पननकगरहॳनहॳटस mdashmdashmdash कावहॴजञाननकनामहहॴ 3-Feb-2017

Options

1) Custard Apple

सीताफि

2) Gulmohar

गिमह८हर

3) Silver Oak

लसमवरओक

4) Pomegranate

अनार

Correct Answer Pomegranate

F A C E B O O K

P A G E h t t p w w w f a c e b o o k c o m s s c m e n t o r s o f f i c i a l P a g e | 50

FOR MORE UPDATES AND MORE MATERIAL DO LIKE OUR FACEBOOK PAGE httpwwwfacebookcomsscmentorsofficial

Q218 Between a tiger and an monkey

which of the following is different

एकबाघऔरबदरकहॳ बीचनननननलिखितमसहॳकह९नसीबातअिगहहॴ 3-Feb-2017

Options

1) Kingdom

राजय

2) Phylum

जानत

3) Order

िम

4) Class

वगय Correct Answer order

Q219 The artificial heart was invented by

कबतरमहदयका mdashmdashmdash

दवाराअपवषकारककयागयािा 3-Feb-2017

Options

1) Muhammad Yunus

महनमदयनस

2) Linus Yale Jr

िाइनसयहॳिजय

3) Gazi Yasargil

गाजीयासचगयि

4) Paul Winchell

पह९िपवमकि Correct Answer Paul Winchell

Q220 Tamarindus indica is the

scientific name of

टहॳमररनडसइडिका mdashmdash कावहॴजञाननकनामहहॴ 7-

Feb-2017

Options

1) Neem

नीम

2) Pineapple

अनानास

3) Tamarind

इमिी 4)Chiku

चीक

Correct Answer Tamarind

Q221 In eukaryotic cells synthesis of

RNA takes place in the

यकहॳ योटटककह८लिकाओमआरएनएकासशिहॳषण

mdashndash महह८ताहहॴ 7-Feb-2017

Options

1) Mitochondria

माईटह८कोडडरया 2) Centrioles

सटरीयह८मस

3) Ribosomes

ररबह८सह८नस

4) Nucleus

नयनकियस

Correct Answer nucleus

eukaryotic cell -Transcription is the

process of synthesizing ribonucleic acid

(RNA)Synthesis takes place within the

nucleus of eukaryotic cells or in the

cytoplasm of prokaryotes and converts

the genetic code from a gene in

deoxyribonucleic acid ( DNA ) to a

strand of RNA that then directs

proteinsynthesis

Q222 _________is caused by parasites

of the Plasmodium genus

पिाजमह८डियमजातीकहॳ परजीवी mdash- कहॳ कारणहहॴ 7-Feb-2017

Options

1) Dysentery

पहॳचचि

2) Malaria

मिहॳररया 3) Chickenpox

F A C E B O O K

P A G E h t t p w w w f a c e b o o k c o m s s c m e n t o r s o f f i c i a l P a g e | 51

FOR MORE UPDATES AND MORE MATERIAL DO LIKE OUR FACEBOOK PAGE httpwwwfacebookcomsscmentorsofficial

चहॳचक

4) Herpes

हहॳपपयस

Correct Answer Malaria

Q223 Carotene in fruits and vegetables

gives it which color

फिह८औरसनलजयोमनसितकहॳ रह८टीनउनहकह९नसारगपरदानकरताहहॴ 7-Feb-2017

Options

1) Green

हरा 2) Pink

गिाबी 3) Orange

नारगी 4) Blue

नीिा Correct Answer Orange

Q224 Equus Caballus is the scientific

name of

एकवसकहॴ बहॳिस mdashmdashndash कापवजञाननकनामहहॴ 7-Feb-2017

Options

1) Horse

घह८िा 2) Zebra

ज़हॳिा 3) Donkey

गधा 4) Buffalo

भस

Correct Answer Horse

Q225 Elapidae Naja is the scientific name of

एिीपीिीनाजा mdashmdash- कावहॴजञाननकनामहहॴ 8-Feb-2017

Options

1) Cobra

कह८बरा 2) Elephant

हािी 3) Eagle

ग ि

4) Owl

उमि Correct Answer Cobra

Q226 Which disease is caused due to

deficiency of Iron

िह८हकीकमीकहॳ कारणकह९नसारह८गहह८ताहहॴ 8-Feb-

2017

Options

1) Beriberi

बहॳरीबहॳरी 2) Tetany

टहॳटनी 3) Kwashiorkor

कवािीऔरकर

4) Anaemia

रकतामपता Correct Answer Anaemia

Beriberi is a disease caused by a vitamin

B-1 deficiency also known as thiamine

deficiency

Tetany can be the result of an

electrolyte imbalance Most often itrsquos a

dramatically low calcium level also

known as hypocalcemia Tetany can also

be caused by magnesium deficiency or

too little potassium Having too much

acid (acidosis) or too much alkali

(alkalosis) in the body can also result in

tetany

Kwashiorkor also known as

ldquoedematous malnutrition It is a form of

malnutrition caused by a lack of protein

in the diet

Anaemia means that you have fewer red

blood cells than normal or you have less

F A C E B O O K

P A G E h t t p w w w f a c e b o o k c o m s s c m e n t o r s o f f i c i a l P a g e | 52

FOR MORE UPDATES AND MORE MATERIAL DO LIKE OUR FACEBOOK PAGE httpwwwfacebookcomsscmentorsofficial

haemoglobin than normal in each red

blood cell

Q227 is a leaf where the leaflets are

arranged along the middle vein

mdashndashएकपततीहहॴजहापतरकह८कीरचनाक ररयालिराकहॳ आसपासहह८तीहहॴ 8-Feb-2017

Options

1) Pinnately compound leaf

पपनहॳटिीसयकतपतती 2) Palmately compound leaf

पामहॳटिीसयकतपतती 3) Compound leaf

सयकतपतती 4) Simple leaf

साधारणपतती Correct Answer Pinnately compound

leaf

Q228 Haustoria or sucking roots are

found in which of the following

हह८सटह८ररयायाचसनहॳवािीजड़हॳनननननलिखितमसहॳककसमपाईजातीहहॴ 8-Feb-2017

Options

1) Wheat

गहॳह

2) Mango

आम

3) Chestnut

चहॳसटनट

4) Cuscuta

कसकयटा Correct Answer Cuscuta

Haustorial roots -The roots of parasitic

plants which penetrate into the host

tissues to absorb nourishment are

called haustorial roots hellip Also known as suckingor parasitic roots

Q229 Equs Asinus is the scientific name

of

एकवसएलसनस mdashmdashndash कावहॴजञाननकनामहहॴ 8-

Feb-2017

Options

1) Donkey

गधा 2) Cow

गाय

3) Deer

टहरन

4) Kangaroo

कगा

Correct Answer Donkey

Q230 Ficus benghalensis is the scientific name of

फाईकसबहॳनगहॳिहॳलसस mdashndash कापवजञाननकनामहहॴ 8-Feb-2017

Options

1) Banyan

बरगद

2) Pineapple

अनानास

3) Babul

बबि

4) Tulsi

तिसी Correct Answer Banyan

Q231 Equus burchellii is the scientific name of

एकवसबचिी mdashmdash- कापवजञाननकनामहहॴ 8-Feb-2017

Options

1) Horse

घह८िा 2) Zebra

जहॳिा 3) Buffalo

F A C E B O O K

P A G E h t t p w w w f a c e b o o k c o m s s c m e n t o r s o f f i c i a l P a g e | 53

FOR MORE UPDATES AND MORE MATERIAL DO LIKE OUR FACEBOOK PAGE httpwwwfacebookcomsscmentorsofficial

भस

4) Ass

गधा Correct Answer Zebra

Page 3: COMPILATION OF ALL 72 SETS OF BIOLOGY SSC CHSL-2016 · OF BIOLOGY SSC CHSL-2016 PREPARED BY : SSC MENTORS BIOLOGY SPECIAL . F A C E B O O K P A G E : h t t p : / / w w w . f a c e

F A C E B O O K

P A G E h t t p w w w f a c e b o o k c o m s s c m e n t o r s o f f i c i a l P a g e | 2

FOR MORE UPDATES AND MORE MATERIAL DO LIKE OUR FACEBOOK PAGE httpwwwfacebookcomsscmentorsofficial

There were 231 questions

asked in SSC CHSL Science

(biology) 2016 Question Paper

Important questions are

explained here also Q1 Diabetes is caused by

मधमहॳह हह८न का कारणहहॴ

7-Jan -2017

Options

1) Excess of insulin

इनसलिनकीबहिता 2) Low production of Insulin

इनसलिनकाकमउतपादन

3) Malfunction of liver

यकतकहॳ काययमगड़बड़ीहह८ना 4) Higher production of bilirubin

बबि बबनकाउचचतरउतपादन

Correct Answer Low production of

Insulin

Diabetes is caused by the immune

system destroying the cells in the

pancreas that make insulin This causes

diabetes by leaving the body without

enough insulin to function normally

Q2 Tectona grandis Linn is the

scientific name of

टहॳकटह८नागहॴननिसलिन mdashmdashndash कावहॴजञाननकनामहहॴ 7-Jan -2017

Options

1)Guavav

अम द

2)Teak

सागह९न

3) Amla

आविा 4) Chiku

चचक

Correct Answer Teak

Q3 Sea-Anemones belongs to the

phylum

सी-एननमोस mdashmdash- परजानतकहॳ अतगयतआतहॳहहॴ 7-Jan -2017 Options

1)Arthropoda

अनरह८पह८ड़ा 2)Cnidaria

ननड़हॳररया 3) Porifera

पह८ररफहॳ रा 4) Mollusca

मह८िसक

Correct Answer Cnidaria

Sea anemones are a group of marine

predatory animals of the order

Actiniaria They are named after the

anemone a terrestrial flowering plant

because of the colourful appearance of

many Sea anemones are classified in the

phylum Cnidaria

Q4 Which of the following is also

known as a Common Water Hyacinth

नननननलिखितमसहॳककसहॳआमजिकनभभीकहाजाताहहॴ

7-Jan -2017

Options

1)Pistia

पपनसटया 2)Opuntia

ओपलिया 3)Aegilops

एनजिह८पस

4) Echhornia

एकह८ननयया Correct Answer Echhornia

Water hyacinth is a free-floating

perennial aquatic plant (or hydrophyte)

native to tropical and sub-tropical South

America

F A C E B O O K

P A G E h t t p w w w f a c e b o o k c o m s s c m e n t o r s o f f i c i a l P a g e | 3

FOR MORE UPDATES AND MORE MATERIAL DO LIKE OUR FACEBOOK PAGE httpwwwfacebookcomsscmentorsofficial

Q5 Which is the largest organ in

human beings

मानवह८मसबसहॳबड़ाअगकह९नसाहह८ताहहॴ

7-Jan -2017

Options

1) Skin

तवचा 2) Large Intestine

बड़ीआत

3) Small Intestine

छह८टीआत

4) Liver

यकत

Correct Answer Skin

The skin is the largest organ of the

body with a total area of about 20

square feetThe largest internal organ is

the liver The longest bone in the human

body is the femur The largest artery is

the aorta and the largest vein is the

inferior vena cava

Q6 Delonix regia Rafin is the scientific

name of

िहॴिोननकसरनजयारकफन (Delonix regia

Rafin)mdashmdash- कावहॴजञाननकनाम 7-Jan -2017

Options

1) Banyan

बरगद

2) Gulmohar

गिमह८हर

3) Tamarind

इमिी 4) Chiku

चचक

Correct Answer Gulmohar

Delonix regia is a species of flowering

plant in the bean family Fabaceae

Q7 Amoeba belongs to the phylum

अमीबा mdashmdashmdash- परजानतकहॳ अतगयतआताहहॴ

7-Jan -2017

Options

1) Protozoa

परह८टह८जआ

2) Annelida

ऐनहॳलििा 3)Porifera

पह८ररफहॳ रा 4) Platyhelminthes

पिहॳटटहहॳनममननिस

Correct Answer Protozoa

Q8 Deficiency of which of the following

causes non-clotting of blood

नननननलिखितमसहॳककसकीकमीकहॳ कारणरकतकािककानहीजमता

8-Jan -2017

Options

1) Vitamin C

पवटालमन C

2) Vitamin K

पवटालमन K

3) Vitamin E

पवटालमन E

4) Vitamin B12

पवटालमन B12

Correct Answer Vitamin K

Vitamin C is found in citrus fruits and

vegetables Scurvy results from a

deficiency of vitamin C in the diet

Vitamin E deficiency associated with

this disease causes problems such as

poor transmission of nerve impulses

muscle weakness and degeneration of

the retina that can cause blindness

Vitamin B12 deficiency may lead to a

reduction in healthy red blood cells

Q9 The process of producing energy in

plants is known as

F A C E B O O K

P A G E h t t p w w w f a c e b o o k c o m s s c m e n t o r s o f f i c i a l P a g e | 4

FOR MORE UPDATES AND MORE MATERIAL DO LIKE OUR FACEBOOK PAGE httpwwwfacebookcomsscmentorsofficial

पहॳड़ह८मउजायउतपननकरनहॳकीपरनतकियाकह८ mdashmdashndash

कहॳ नामजाताहहॴ 8-Jan -2017

Options

1) Absorption

अविह८षण

2) Reduction

अवकरण

3) Photosynthesis

परकािसशरिहॳषण

4) Transpiration

वाषपीकरण

Correct Answer Photosynthesis

Q10 Which Virus causes Chicken Pox

ककसवायरसकहॳ कारणचहॳचकहह८ताहहॴ

8-Jan -2017

Options

1) Rubella Virus

बहॳिावायरस

2) Herpes Zoster Virus

हपपरयसजह८सटरवायरस

3) Rabies

रहॳबीज़

4) Variola Virus

वहॳरीओिावायरस

Correct Answer Herpes Zoster Virus

Rubella also known as German

measles or

three ndashday measles is an infection

caused by the rubella virus

Smallpox is caused by infection with

variola

Virus

Q11 What is the total number of bones

in the human body

मानविरीरमकिककतनीहडडियहह८तीहहॴ

8-Jan -2017

Options

1)206

2)103

3)309

4)412

Correct Answer 206

Q12 Emblica officinalis is the scientific

name of

एननलिकाओफीलसनहॳलिस (Emblica

offcinalis)mdashmdash- कावहॴजञाननकनामहहॴ 8-Jan -2017

Options

1) Peepal

पीपि

2) Mango

आम

3) Amla

आविा 4) Drumstick

सहजन

Correct Answer Amla

Q13 Sponges belongs to the phylum

सपजककसपरजानतकहॳ अतगयतआतहहॴ

8-Jan -2017

Options

1) Protozoa

परह८टह८जआ

2) Annelida

एननलििा 3) Porifera

पह८ररफहॳ रा 4) Cnidaria

ननिहॳररया Correct Answer Porifera

Q14 Which of the following is a

symptom of haemophilia

नननननलिखितमसहॳटहमह८फीलियाकािकषणकह९नसा हहॴ

9-Jan -2017

Options

F A C E B O O K

P A G E h t t p w w w f a c e b o o k c o m s s c m e n t o r s o f f i c i a l P a g e | 5

FOR MORE UPDATES AND MORE MATERIAL DO LIKE OUR FACEBOOK PAGE httpwwwfacebookcomsscmentorsofficial

1) Night Blindness

रतोधी 2) No clotting of Blood

रकतकािककानजमना 3) Rickets

ररकहॳ ट

4) Loss of haemoglobin

टहमह८गिह८बबनकीअमपता Correct Answer No clotting of Blood

Haemophilia also spelled hemophilia is

a mostly inherited genetic disorder that

impairs the bodyrsquos ability to make blood

clots a process needed to stop bleeding

This results in people bleeding longer

after an injury easy bruising and an

increased risk of bleeding inside joints

or the brain

Q15 The process of pollination by birds

is also known as

पकषकषयोदवाराकीजानहॳवािीपरागणकीपरनतकियाकह८mdashndash कहॳ नामसहॳभीजानाजाताहहॴ 9-Jan -2017

Options

1) Hydrophily

हाइडरह८कफिी 2) Entomophily

एनटोमह८कफिी 3) Embryophily

एननियह८कफिी 4) Ornithophily

ओननयिह८कफिी Correct Answer Ornithophily

Q16 Spiders belong to the phylum

मकडड़याककसपरजानतकहॳ अतगयतआतीहहॴ 9-Jan -2017

Options

1) Mollusca

मह८िसका 2) Annelida

एननलििा 3) Cnidaria

ननिहॳररया 4) Arthropoda

अरोपह८ड़ा Correct Answer Arthropoda

Q17 Banana freckle is a plant disease

It is caused by a

कहॳ िहॳकीझाईपह८धह८कीएकबीमारीहहॴ यहएक mdashndash

कहॳ कारणहह८तीहहॴ 9-Jan -2017

Options

1) Virus

वायरस

2) Fungus

कवक

3) Bacteria

बहॴकटीररया 4) Insect

कीटक

Correct Answer Fungus

Banana Freckle is a disease caused by

the fungus Guignardia musae

(telomorph) or Phyllosticta musarum (

anamorph )

Q18 Which of the following Indian

chilly is considered one of the hottest in

the world

नननननलिखितभारतीयलमचचययह८मसहॳकह९नसीसबसहॳकह९नसीपवशवकीसबसहॳतीिीलमचचययह८मसहॳएकमानीजातीहहॴ

9-Jan -2017

Options

1) Bhut Jolokia

भतझह८िककया 2) Bhut Mahabora

भतमहाबह८रा 3) Lal Chitin

F A C E B O O K

P A G E h t t p w w w f a c e b o o k c o m s s c m e n t o r s o f f i c i a l P a g e | 6

FOR MORE UPDATES AND MORE MATERIAL DO LIKE OUR FACEBOOK PAGE httpwwwfacebookcomsscmentorsofficial

िािचीटटन

4) Lal Shamak

िाििामक

Correct Answer Bhut Jolokia

Q19 Brain fever is a disease spread

through which of the following

मनसतषकजवरनमकरह८गनननननलिखितमसहॳककसकहॳ कारणहह८ताहहॴ 9-Jan -2017

Options

1) Flies

मनकियो 2) Mosquito

मचछर

3) Virus

वायरस

4) Cockroach

नतिच हॳ Correct Answer Mosquito

Q20 Mangroves are plants that have

मगरह८ववहॳपहॳिहहॴनजनमहॳहह८ताहहॴ 9-Jan -2017

Options

1) Modified Roots

पातररतजड़हॳ 2) Modified Stems

पातररततनहॳ 3) Respiratory Roots

शरवसनकरनहॳवािीजड़हॳ 4) Respiratory Stems

शरवसनकरनहॳवािीतनहॳ Correct Answer Respiratory Roots

A mangrove is a shrub or small tree that

grows in coastal saline or brackish

water

Q21 Rodentia Sciurus is the scientific

name of

रह८िहॳलियासकीयरस mdashmdash कावयजजञाननकनामहहॴ

9-Jan -2017

Options

1) Rat

चहा 2) Platypus

पिहॳटीपस

3) Squirrel

चगिहरी 4) Beaver

बीवर

Correct Answer Squirrel

Q22 Which of the following is induced

by Oncogene

नननननलिखितमसहॳकह९नओकह८जीनदवारापरहॳररयतहह८ताहहॴ

10-Jan -2017

Options

1) Polio

पह८लियह८ 2) Cancer

क सर

3) Diarrhoea

दसत

4) Dengue

िग Correct Answer Cancer

An oncogene is a gene that has the

potential to cause cancer In tumor

cells they are often mutated andor

expressed at high levels

Q23 Azadirachata indica is the

scientific name of

अजाटदराकटाइडिका mdashmdashवहॴजञाननकनामहहॴ SSC CHSL Science (biology) 2016

Question Paper

10-Jan -2017

Options

1) Neem

नीम

F A C E B O O K

P A G E h t t p w w w f a c e b o o k c o m s s c m e n t o r s o f f i c i a l P a g e | 7

FOR MORE UPDATES AND MORE MATERIAL DO LIKE OUR FACEBOOK PAGE httpwwwfacebookcomsscmentorsofficial

2) Teak

सागह९न

3) Silver Oak

लसमवरओक

4) Tulsi

तिसी Correct Answer Neem

Q24 Octopus belongs to the phylum

ऑकटह८पसककसपरजानतकहॳ अतगयतआताहहॴ 10-

Jan -2017

Options

1) Mollusca

मह८िसका 2) Cnidaria

ननिहॳररया 3) Echinodermata

इकाइनह८ड़हॳमता 4) Chordata

कह८िता Correct Answer Mollusca

Q25 A living part of the organisms

environment is known as

जीवाणकहॳ वातावरणकहॳ जीपवतभागकह८ mdash-

कहतहॳहहॴ 10-Jan -2017

Options

1) Abiotic Factor

अजहॴपवककारक

2) Habitat

आवास

3) Biotic Factor

जहॴपवककारक

4) Nonliving factor

अ-जीपवतकारक

Correct Answer Biotic Factor

Abiotic factors are nonndash living chemical

and physical parts of the environment

that affect living organisms and the

functioning of ecosystems like rain

wind temperature altitude soil

pollution nutrients pH types of soil

and sunlight

Q26 Medulla oblongata is a part of

which of the following

महॳडयिाऑबिॉनगहॳटानननननलिखितमसहॳककसअगकाटहससाहहॴ

10-Jan -2017

Options

1) Heart

हदय

2) Brain

मनसतषक

3) Lungs

फहॳ फड़हॳ 4) Stomach

पहॳट

Correct Answer Brain

The medulla oblongata helps regulate

breathing heart and blood vessel

function digestion sneezing and

swallowing This part of the brain is a

center for respiration and circulation

Sensory and motor neurons (nerve cells)

from the forebrain and midbrain travel

through the medulla

Q27 ___________ is a typically

onecelled reproductive unit capable of

giving rise to a new individual without

sexual fusion

mdashmdash एकआमतह९रपरएककह८लिकीयहॳ परजननममसमकषइकाईहहॴजह८यह९नसियनकहॳ बबनाएकनयीइकाईकह८जनमदहॳतीहहॴ 10-Jan -2017

Options

1) Egg

अिाण

2) Spore

बीजाण

F A C E B O O K

P A G E h t t p w w w f a c e b o o k c o m s s c m e n t o r s o f f i c i a l P a g e | 8

FOR MORE UPDATES AND MORE MATERIAL DO LIKE OUR FACEBOOK PAGE httpwwwfacebookcomsscmentorsofficial

3) Sperm

ििाण

4) Seed

बीज

Correct Answer Spore

Q28 Bacteria was discovered by

बहॴकटीररयाकीिह८जककसकहॳ दवाराकीगयीिी

10-Jan -2017

1) Antonie van Leeuwenhoek

एटह८नीवहॳनलिबहॳनहक

2) Belarus

बहॳिा स

3) Hugo de Vries

हयगह८दीराईस

4)Robert Brown

रॉबटयिाउन

Correct Answer Antonie van

Leeuwenhoek

Q29 Which of the following is

responsible for Vermicomposting

नननननलिखितमसहॳकह९नकलमिादकहॳ लिएनजनमहॳदारहहॴ

10-Jan -2017

Options

1) Fungus

कवक

2) Worms

कलम

3) Bacteria

बहॴकटीररया 4) Birds

पकषी Correct Answer Worms

Vermicompost (or vermi-compost) is the

product of the composting process using

various species of worms usually red

wigglers white worms and other

earthworms to create a heterogeneous

mixture of decomposing vegetable or

food waste bedding materials and

vermicast

Q30 Scurvy (bleeding of gums) is

caused by the deficiency of which

vitamin

सकवी (मसढह८सहॳिनआना) ककसपवटालमनकीकमीकहॳ कारणहह८ताहहॴ

10-Jan-2017

Options

1) Vitamin K

पवटालमन K

2) Vitamin BZ

पवटालमन BZ

3) Vitamin C

पवटालमन C

4) Vitamin A

पवटालमन A

Correct Answer Vitamin C

Q31 Achras sapota is the scientific

name of

एिाससपह८ताइसकावहॴजञाननकनामहहॴ 10-Jan-2017

Options

1) Custard Apple

सीताफि

2) Gulmohar

गिमह८हर

3) Tamarind

इमिी 4) Chiku

चचक

Correct Answer Chiku

Q32 Prawn belongs to the phylum

झीगा mdashmdash- परजानतकहॳ अतगयतआताहहॴ 10-Jan-2017

Options

1) Arthropoda

F A C E B O O K

P A G E h t t p w w w f a c e b o o k c o m s s c m e n t o r s o f f i c i a l P a g e | 9

FOR MORE UPDATES AND MORE MATERIAL DO LIKE OUR FACEBOOK PAGE httpwwwfacebookcomsscmentorsofficial

अरोपह८िा 2) Cnidaria

नीिहॳररया 3) Echinodermata

इकाईनह८िमटा 4) Chordata

कह८िटा Correct Answer Arthropoda

Q33 Pulses are a rich source of which of

the following

दािहॳनननननलिखितमसहॳककसकीपरचरसह८तरहहॴ

11-Jan-2017

Options

1) Carbohydrates

काबोहाइडराईट

2) Proteins

परह८टीनस

3) Minerals

िननज

4) Vitamin A

पवटालमन A

Correct Answer Proteins

Q34 Plant cell wall is made up of

वनसपनतकह८लिकालभनततइससहॳबनीहह८तीहहॴ

11-Jan-2017

Options

1) Cellulose

सहॳमयिह८ज

2) Glucose

गिकह८ज

3) Sucrose

सिह८ज

4) Fructose

फरकटह८ज

Correct Answer Cellulose

Plant cell wall the major carbohydrates

are cellulose hemicellulose and pectin

The cellulose microfibrils are linked via

hemicellulosic tethers to form the

cellulose-hemicellulose network which

is embedded in the pectin matrix

Q35 The study of Fungi is also known

as कवकह८कहॳ अधययनकह८कहाजाताहहॴ

11-Jan-2017

Options

1) Cytology

सायटह८िह८जी 2) Myology

मायह८िह८जी 3) Mycology

मायकह८िह८जी 4) Neurology

नयरह८िह८जी Correct Answer Mycology

Cytology - structure and function of

plant and animal cells

Myology is the study of the muscular

system

Neurology is the branch of medicine

concerned with the study and treatment

of disorders of the nervous system

Q36 The outermost layer of skin is

तवचाकीसबसहॳबाहरीपरतकयाहह८तीहहॴ 11-Jan-

2017

Options

1) Epidermis

इपपिलमयस

2) Dermis

िलमयस

3) Tissues

ऊतक

4) Hypodermis

हायपह८िलमयस

Correct Answer Epidermis

Q37 Which of the following plants have

root nodules

F A C E B O O K

P A G E h t t p w w w f a c e b o o k c o m s s c m e n t o r s o f f i c i a l P a g e | 10

FOR MORE UPDATES AND MORE MATERIAL DO LIKE OUR FACEBOOK PAGE httpwwwfacebookcomsscmentorsofficial

नननननलिखितपह९धह८मसहॳककसकीजड़ह८मगाठहह८तीहहॴ

11-Jan-2017

Options

1) Leguminous plants

िहॳगयलमनसपह९धहॳ 2) Parasitic plants

परजीवीपह९धहॳ 3) Epiphytic Plants

एपीफाइटटकपह९धहॳ 4) Aquatic Plants

जिीयपह९धहॳ Correct Answer Leguminous plants

Q38 Earth-worms belongs to the

phylum

कहॳ चएmdashmdash- परजानतकहॳ अतगयतआतहॳहहॴ 11-Jan-2017

Options

1) Protozoa

परह८टह८जआ

2) Cnidaria

नीिहॳररया 3) Annelida

एनीलििा 4) Mollusca

मह८िसका Correct Answer Annelida

Q39 Ringworm is a disease caused by

ररगवमयनामकबीमारी mdashmdash- कहॳ कारणहह८तीहहॴ 11-Jan-2017

Options

1) Fungi

कवक

2) Bacteria

बहॴकटीररया 3) Virus

वायरस

4) Flies

मनकियाा Correct Answer Fungi

Q40 Mangifera indica is the scientific

name of

मननगफहॳ राइडिकाककसकावहॴजञाननकनामहहॴ 11-

Jan-2017

Options

1) Guava

अम द

2) Mango

आम

3) Amla

आविा 4) Jack fruit

कटहि

Correct Answer Mango

Q41 Crabs belongs to the phylum

कहॳ कड़हॳmdashmdash- परजानतकहॳ अतगयतआतहॳहहॴ 11-Jan-2017

Options

1) Mollusca

मह८िसका 2) Cnidaria

नीिहॳररया 3) Arthropoda

अरोपह८ड़ा 4) Platyhelminthes

पिहॳटटहहॳनममननिस

Correct Answer Arthropoda

Q42 Myopia is a defect of eyes which is

also known as

मायह८पपयाआिोकादह८षहहॴ नजसहॳ mdashmdashndash

भीकहाजाताहहॴ

12-Jan-2017

Options

1) Far Sightedness

F A C E B O O K

P A G E h t t p w w w f a c e b o o k c o m s s c m e n t o r s o f f i c i a l P a g e | 11

FOR MORE UPDATES AND MORE MATERIAL DO LIKE OUR FACEBOOK PAGE httpwwwfacebookcomsscmentorsofficial

दरदनषटदह८ष

2) Near Sightedness

ननकटदनषटदह८ष

3) Astigmatism

एसटीगमहॳटटजम

4) Night Blindness

रतोधी Correct Answer Near Sightedness

Myopia occurs when the eyeball is too

long relative to the focusing power of

the cornea and lens of the eye This

causes light rays to focus at a point in

front of the retina rather than directly

on its surface

Hyperopia Hypermetropia (

Farsightedness )- when light rays

entering the eye focus behind the retina

rather than directly on it The eyeball of

a farsighted person is shorter than

normal

Astigmatism usually is caused by an

irregularly shaped cornea Instead of

the cornea having a symmetrically

round shape (like a baseball) it is

shaped more like an American football

Nyctalopia also called night ndash blindness

is a condition making it difficult or

impossible to see in relatively low light

Q43 Who is known as the father of

Green Revolution

हररतिानतकहॳ जनककहॳ पमककसहॳजानाजाताहहॴ

12-Jan-2017

1) Dr Robert Nucleus

िॉ रॉबटयनयनकियस

2) Dr Ian Wilmut

िॉ इयानपविमट

3) Dr NE Borlaug

िॉ एनईबह८रिॉग

4) Dr JC Bose

िॉ जहॳसीबह८स

Correct Answer Dr NE Borlaug

Q44 Panthera Tigris is the scientific

name of

पिहॳराटटगरीस mdashmdashmdash कावहॴजञाननकनामहहॴ 12-Jan-2017

Options

1) Panther

तदआ

2) Tiger

बाघ

3) Whale

हहॳि

4)Goat

बकरी Correct Answer Tiger

Q45 How many facial bones are there

हमारहॳचहॳहरहॳमककतनीहडडियााहह८तीहहॴ 13-Jan-2017

Options

1)34

2)24

3)14

4)4

Correct Answer 14

Q46 ndash Halophytes are plants that grow

in

हहॴिह८फाईटसवहॳपह९धहॳहह८तीहहॴजह८ mdash- मउगतहॳहहॴ SSC CHSL Science (biology) 2016

Question Paper

13-Jan-2017

Options

1) Fresh Water

ताजापानी 2) Cold Water

ठिापानी 3) Ponds

तािाब

4) Salt Water

िारापानी Correct Answer Salt Water

F A C E B O O K

P A G E h t t p w w w f a c e b o o k c o m s s c m e n t o r s o f f i c i a l P a g e | 12

FOR MORE UPDATES AND MORE MATERIAL DO LIKE OUR FACEBOOK PAGE httpwwwfacebookcomsscmentorsofficial

Q47 Felis Catus is the scientific name of

फहॳ लिसकहॴ टस mdashndash कावहॴजञाननकनामहहॴ 13-Jan-2017

Options

1) Cat

बबमिी 2) Dog

कतता 3) Mouse

चहा 4) Porcupine

साही Correct Answer Cat

Q48 Which of the following induces

nitrogen fixation in soil

नननननलिखितमसहॳकह९नलम ीमनाइटरह८जनननयतनकह८परहॳररतकरताहहॴ

15-Jan-2017

Options

1) Protozoa

परह८टह८जआ

2) Bacteria

बहॴकटीररया 3) Fungi

कवक

4)Algae

िहॴवाि

Correct Answer Bacteria

Bacteria that change nitrogen gas from

the atmosphere into solid nitrogen

usable by plants are called nitrogen-

fixing bacteria These bacteria are

found both in the soil and in symbiotic

relationships with plants

They contain symbiotic bacteria called

rhizobia within nodules in their root

systems producing nitrogen compounds

that help the plant to grow and compete

with other plants When the plant dies

the fixed nitrogen is released making it

available to other plant

Q49 Which of the following is the

largest known cell

नननननलिखितमसहॳकह९नसीसबसहॳबड़ीजञातकह८लिकाहहॴ

SSC CHSL Science (biology) 2016

Question Paper

15-Jan-2017

1) Eukaryotic Cell

यकहॳ ररयह८टटककह८लिका 2) Prokaryotic Cell

परह८कहॳ ररयह८टटककह८लिका 3) Mycoplasma

मायकह८पिासम

4) Ostrich Eggs

ितरमगयकाअिा Correct Answer Ostrich Eggs

Q50 The association of animals in

which both the partners are benefitted

is known as

जानवरोकावहसहयह८गनजसमहॳदह८नोभागीदारिाभापवनतहह८तहॳहहॴ उसहॳ mdashmdashndash कहॳ पमजानाजाताहहॴ SSC CHSL Science (biology) 2016

Question Paper

15-Jan-2017

Options

1) Amensalism

सहजीपवत

2) Commensalism

परजीपवत

3) Colony

कॉिनी 4) Mutualism

अनयह८नयाशरयवाद

Correct Answer Mutualism

Amensalism is any relationship between

organisms of different species in which

F A C E B O O K

P A G E h t t p w w w f a c e b o o k c o m s s c m e n t o r s o f f i c i a l P a g e | 13

FOR MORE UPDATES AND MORE MATERIAL DO LIKE OUR FACEBOOK PAGE httpwwwfacebookcomsscmentorsofficial

one organism is inhibited or destroyed

while the other organism remains

unaffected

Commensalism an association between

two organisms in which one benefits and

the other derives neither benefit nor

harm

Q51 Pneumonia affects which of the

following organs of human body

ननमह८ननयामानविरीरकहॳ नननननलिखितमसहॳककसअगकह८परभापवतकरताहहॴ

15-Jan-2017

Options

1)Kidneys

गद

2)Lungs

फहॳ फड़हॳ 3) Throat

गिहॳ 4) Liver

यकत

Correct Answer Lungs

When the germs that cause pneumonia

reach your lungs the lungsrsquo air sacs

(alveoli) become inflamed and fill up

with fluid This causes the symptoms of

pneumonia such as a cough fever

chills and trouble breathing When you

have pneumonia oxygen may have

trouble reaching your blood

Q52 Mendel is known as

मििकह८ mdashmdash- कहॳ पमजानाजाताहहॴ 15-Jan-2017

Options

1) Father of Physiology

िरीरकियािासतरकहॳ जनक

2) Father of Geology

भगभयिासतरकहॳ जनक

3) Father of Genetics

जहॳनहॳटटकसकहॳ जनक

4) Father of Biology

जीविासतरकहॳ जनक

Correct Answer Father of Genetics

Q53 Which of the following are also

known as Suicidal bag of Cells

ननननलिखितमसहॳककसहॳआतमहतयाकरनहॳवािीकह८लिकाओकाबहॴगकहाजाताहहॴ

15-Jan-2017

Options

1) Lysosomes

िायसोसह८म

2) Lycosome

िायकह८सह८म

3) Nucleus

नालभक

4) Chromosome

िह८मह८सह८म

Correct Answer Lysosomes

Q54 Mesothelioma is a type of cancer

The most common area affected in it is

the lining of the ________

लमज़ह८िहॳिहॳलमयाक सरकाएकपरकारहहॴ इससहॳपरभापवतहह८नहॳवािासबसहॳसामानयकषहॳतर mdash

mdashmdash काअसतरहहॴ 15-Jan-2017

Options

1)Heart

हदय

2)Brain

मनसतषक

3)Stomach

आमािय

4)Lungs

फहॳ फड़हॳ Correct Answer lungs

Asbestos exposure is the main cause of

pleural mesothelioma When asbestos

fibers are breathed in they travel to the

F A C E B O O K

P A G E h t t p w w w f a c e b o o k c o m s s c m e n t o r s o f f i c i a l P a g e | 14

FOR MORE UPDATES AND MORE MATERIAL DO LIKE OUR FACEBOOK PAGE httpwwwfacebookcomsscmentorsofficial

ends of small air passages and reach the

pleura where they can cause

inflammation and

scarring

Q55 Which one of the following is an

insectivorous plant

नननननलिखितमसहॳकह९नसाएकककटाहरीवनसपनतहहॴ

15-Jan-2017

Options

1) Utricularia

यटरीकिहॳररया 2) Sequoia

सहॳकयओइया 3) Nostoc

नॉसटह८क

4) Bryophyta

िायह८फाईटा Correct Answer Utricularia

Q56 ______________ is a

multibranched polysaccharide of

glucose that serves as a form of energy

storage in animals and fungi

mdashmdashगिकह८जकाएकबहिािायकतपह८िीसहॳकहॳ राइिहहॴ जह८जानवरोऔरकवकमउजायभणिारणकहॳ एक पमकाययकरताहहॴ 15-Jan-2017

Options

1) Cellulose

सहॳमयिह८ज

2) Glycogen

गिायकह८जन

3) Pectin

पहॳनकटन

4) Chitin

चीटटन

Correct Answer Glycogen

Q57 The largest gland of the human

body is

mdashmdashmdashमानविरीरकीसबसहॳबड़ीगरिीहहॴ 16-Jan-2017

Options

1) Pancreas

अगयािय

2) Thyroid

िायरॉइि

3) Large Intestine

बड़ीआत

4) Liver

यकत

Correct Answer Liver

Q58 Photosynthesis in plants takes

place in

वनसपनतयोमपरकािसशिहॳषणकीकियाहह८तीहहॴ

16-Jan-2017

Options

1) Stem

तना 2) Leaves

पनततयाा 3) Roots

जड़हॳ 4) Flower

फि

Correct Answer Leaves

During this reaction carbon dioxide

and water are converted into glucose

and oxygen The reaction requires light

energy which is absorbed by a green

substance called

chlorophyll Photosynthesis takes place

in leaf

cells These contain chloroplasts which

are tiny objects containing chlorophyll

F A C E B O O K

P A G E h t t p w w w f a c e b o o k c o m s s c m e n t o r s o f f i c i a l P a g e | 15

FOR MORE UPDATES AND MORE MATERIAL DO LIKE OUR FACEBOOK PAGE httpwwwfacebookcomsscmentorsofficial

Q59 Insects that transmit diseases are

known as

जह८कीड़हॳरह८गसचाररतकरतहॳहहॴ उनह mdashmdash-

कहॳ नामसहॳजानाजाताहहॴ 16-Jan-2017

1)Pathogens

रह८गज़नक

2) Vectors

वहॳकटर

3) Drones

परजीवी 4)Scalars

अटदषट

Correct Answer Vectors

A vector is an organism that does not

cause disease itself but which spreads

infection by conveying pathogens from

one host to another Species of mosquito

for example serve as vectors for the

deadly disease Malaria

Q60 Which is the second largest gland

of Human body

मानविरीरकीदसरीसबसहॳबड़ीगरिीकह९नसीहहॴ

SSC CHSL Science (biology)

2016 Question Paper

16-Jan-2017

Options

1) Liver

यकत

2) Large Intestine

बड़ीआत

3) Thorax

छाती 4) Pancreas

अगनयािय

Correct Answer Pancreas

Q61 Annona squamosa is the scientific

name of

एनह८नासकवामह८सा (Annona squamosa) mdash

mdashmdash कावहॴजञाननकनामहहॴ 16-Jan-2017

Options

1) Custard Apple

सीताफि

2) Papaya

पपीता 3) Babhul

बबि

4) Drumstick

सहजन

Correct Answer Custard Apple

Q62 The disease Beri Beri is caused due

to the deficiency of which of the

following

बहॳरीबहॳरीरह८गनननननलिखितमसहॳककसकीकमीकहॳकारणहह८ताहहॴ

16-Jan-2017

Options

1) Vitamin B2

पवटालमन B2

2) Vitamin B1

पवटालमन B1

3) Vitamin B12

पवटालमन B12

4) Vitamin E

पवटालमन E

Correct Answer Vitamin B1

Beriberi is a disease caused by a vitamin

B-1 deficiency also known as thiamine

deficiency

Q63 Chlorophyll was first isolated and

named by

किह८रह८कफिकह८ mdash-

दवारापहिहॳपिकऔरनालमतककयागया 16-Jan-2017

F A C E B O O K

P A G E h t t p w w w f a c e b o o k c o m s s c m e n t o r s o f f i c i a l P a g e | 16

FOR MORE UPDATES AND MORE MATERIAL DO LIKE OUR FACEBOOK PAGE httpwwwfacebookcomsscmentorsofficial

Options

1) Caventou

कहॳ वहॳत 2) Pelletier

पहॳिहॳटटयर

3) Chlorophyll

किह८रह८कफि

4) Caventou and Pelletier

कहॳ वहॳतऔरपहॳिहॳटटयर

Correct Answer Caventou and Pelletier

Chlorophyll was first isolated and

named by

Joseph Bienaimeacute Caventou and Pierre

Joseph Pelletier in 1817 The presence of

magnesium in chlorophyll was

discovered in 1906 and was the first

time that magnesium had been detected

in living tissue

Q64 Which of the following organisms

does not fit into the Cell Theory

नननननलिखितमसहॳकह९नसाजीवकह८लिकालसदातअन पनहीहहॴ

16-Jan-2017

Options

1) Bacteria

बहॴकटीररया 2) Virus

वायरस

3) Fungi

कवक

4) Plants

पह९धहॳ Correct Answer Virus

The bottom line is that viruses are not

alive and not related to cells in any way

The cell theory states that all living

things are made of cells cells are the

basic units of structure and function of

living things and that all cells come

from other cells Since viruses are not

made of cells and do not use cells in any

of their processes they are not related to

the cell theory

Q65 Which of these is not a

macronutrient for Plants

नननननलिखितमसहॳकह९नसापह९धह८कहॳ लिएमिह८नयटरीएटनहीहहॴ

SSC CHSL Science (biology) 2016

Question Paper

17-Jan-2017

Options

1) Nitrogen

नाइटरह८जन

2) Phosphorus

फासफह८रस

3) Potassium

पह८टालसयम

4) Chlorine

किह८रीन

Correct Answer Chlorine

In relatively large amounts the soil

supplies nitrogen phosphorus

potassium calcium magnesium and

sulfur these are often called the

macronutrients In relatively small

amounts the soil supplies iron

manganese boron molybdenum

copper zinc chlorine and cobalt the

so-called micronutrients

Q66 Name the respiratory organs of

insects

कीटह८मनसतिशरवसनअगनामकानामहहॴ

17-Jan-2017

Options

1) Skin

तवचा 2) Body Surface

िरीरकीसतह

F A C E B O O K

P A G E h t t p w w w f a c e b o o k c o m s s c m e n t o r s o f f i c i a l P a g e | 17

FOR MORE UPDATES AND MORE MATERIAL DO LIKE OUR FACEBOOK PAGE httpwwwfacebookcomsscmentorsofficial

3) Gills

गिफड़हॳ 4) Tracheae

शरावस- निी Correct Answer Tracheae

Air enters the respiratory systems of

insects through a series of external

openings called

spiracles These external openings

which act as muscular valves in some

insects lead to the internal respiratory

system a densely networked array of

tubes called tracheae

Q67 The poisonous gas accidentally

released in Bhopal Gas Tragedy is

भह८पािगहॴसतरासदीमगितीसहॳमकतहईजहरीिीगहॴसिी

17-Jan-2017

1) Methane

मीिहॳन

2) Nitrous Oxide

नाइटरसऑकसाइि

3) Methyl Isocyanate

महॴचििआयसोसायनहॳट

4) Cyanogen

सायनह८जहॳन

Correct Answer Methyl Isocyanate

Q68 What does Trypsin do

टटरनपसनकयाकरताहहॴ

SSC CHSL Science (biology) 2016

Question Paper

17-Jan-2017

Options

1) Breaks down Carbohydrates

काबोहाइडरहॳटकापवघटनकरताहहॴ 2) Synthesizes proteins

परह८टीनकासििहॳषणकरताहहॴ 3) Breaks down fats

वसाकापवघटनकरताहहॴ 4) Breaks down proteins

परह८टीनकापवघटनकरताहहॴ Correct Answer Breaks down proteins

Trypsin is one of the three principal

digestive

proteinases the other two being pepsin

and

chymotrypsin In the digestive process

trypsin acts with the other proteinases

to break down dietary protein molecules

to their component

peptides and amino acids

A protease is any enzyme that performs

proteolysis protein catabolism by

hydrolysis of peptide bonds

Q69 Name the source from which

Aspirin is produced

उससरह८तकानामबताइए

नजससहॳएनसपररनकाउतपादनककयाजाताहहॴ

17-Jan-2017

Options

1) Willow bark

पविह८कीछाि

2) Oak Tree

ओककावकष

3) Acacia

बबि

4) Eucalyptus

नीिचगरी Correct Answer Willow bark

The compound from which the active

ingredient in aspirin was first derived

salicylic acid was found in the bark of a

willow tree in 1763 by Reverend

Edmund Stone of Chipping-Norton

Q70 Cannis Familiaris is the scientific

name of

कहॴ ननसफहॳ लमलियहॳररस mdash- कावहॴजञाननकनामहहॴ

17-Jan-2017

F A C E B O O K

P A G E h t t p w w w f a c e b o o k c o m s s c m e n t o r s o f f i c i a l P a g e | 18

FOR MORE UPDATES AND MORE MATERIAL DO LIKE OUR FACEBOOK PAGE httpwwwfacebookcomsscmentorsofficial

Options

1) Cat

बबमिी 2)Dog

कतता 3) Fox

िह८मड़ी 4) Wolf

भहॳडड़या Correct Answer Dog

Q71 Harmful bacteria in potable water

make the water

पीनहॳकहॳ पानीमनसतिघातकबहॴकटीररयाउसपानीकह८बनातहॳहहॴ 17-Jan-2017

Options

1) unfit to drink

पीनहॳकहॳ लिएअयह८गय

2) smelly

दगयनधयकत

3) Colored

रगीन

4) Turbid

मटमहॴिा Correct Answer unfit to drink

Q72 Musa paradisiaca is the scientific

name of which plant

मसापहॴराडिलसयाकाककसपह९धहॳकावहॴजञाननकनामहहॴ

17-Jan-2017

Options

1) Mango

आम

2) Wheat

गहॳह

3) Corn

भ ा 4) banana

कहॳ िा Correct Answer banana

Q73 Prawns belong to which family

झीगहॳककसपररवारकहॳ हह८तहॳहहॴ 17-Jan-2017

Options

1) Crustaceans

िसटहॳलियन

2)Fish

मछिी 3) Amphibians

अननफबबयस

4) Reptiles

रहॳपटाइमस

Correct Answer Crustaceans

Q74 Name the drug that is yielded from

Cinchona tree and is used to cure

malaria

उसऔषचधकानामबताइएनजसहॳलसगकह८नापहॳड़सहॳपरापतककयाजाताहहॴऔरनजसकाउपयह८गमिहॳररयाकहॳ उपचारमककयाजाताहहॴ 17-Jan-2017

Options

1) Camptothea

कहॴ नटह८चिया 2) Acuminata

एकयलमनहॳटा 3) Quinine

कनहॴन

4) Cinchonia

लसकह८ननया Correct Answer Quinine

Q75 Blood Circulation was discovered

by

रकतपररसचरणकी mdashmdashndash दवारािह८जकीिी 17-Jan-2017

Options

1) Mary Anderson

F A C E B O O K

P A G E h t t p w w w f a c e b o o k c o m s s c m e n t o r s o f f i c i a l P a g e | 19

FOR MORE UPDATES AND MORE MATERIAL DO LIKE OUR FACEBOOK PAGE httpwwwfacebookcomsscmentorsofficial

महॴरीएिरसन

2) Virginia Apgar

वनजयननयाएपगार

3) William Harvey

पवलियमहाव

4) Robert Feulgen

रॉबटयफ़यिजहॳन Correct Answer William Harvey

Q76 Vitamin A is also known as

पवटालमन A कह८ mdashmdash- कहॳ नामसहॳभीजानाजाताहहॴ SSC CHSL Science (biology) 2016

Question Paper

18Jan2017

Options

1) Thiamine

िायलमन

2) Riboflavin

ररबह८फिहॳपवन

3) Retinol

रहॳटटनॉि

4) Calciferol

कहॴ नमसफहॳ रह८ि

Correct Answer Retinol

Q77 Some roots called arise from an

organ other than the radicle

कछजड़हॳनजनह mdashmdashmdash कहाजाताहहॴ वहमिकहॳ अिावाककसीअनयअगसहॳउतपननहह८तीहहॴ 18Jan2017

Options

1) tap roots

मखयजड़

2) stilt roots

ि ाजड़

3) fibrous roots

रहॳिहॳदारजड़

4) adventitious roots

आकनसमकजड़

Correct Answer adventitious roots

Q78 Spiders belong to which class of

animals

मकडड़यापराणीवगीकरणकहॳ ककसवगयमआतीहहॴ 18Jan2017

Options

1) Arachnids

एरहॳकननडस

2) Aves

एपवस

3) Gastropods

गहॴसटरोपह८िस

4) Anthozoa

एिह८जआ

Correct Answer Arachnids

Q79 How many layers does Human

Skin have

मानवतवचामककतनीपरतहॳहह८तीहहॴ

18Jan2017

Options

1) 5

2) 7

3) 11

4) 3

Correct Answer 3

Skin has three layers The epidermis

the outermost layer of skin provides a

waterproof barrier and creates our skin

tone The dermis beneath the

epidermis contains tough connective

tissue hair follicles and sweat glands

The deeper subcutaneous tissue (

hypodermis ) is made of fat and

connective tissue

Q80 Allium Cepa is the scientific name

of

एलियमलसपपा mdashmdashndash कावहॴजञाननकनामहहॴ 18Jan2017

F A C E B O O K

P A G E h t t p w w w f a c e b o o k c o m s s c m e n t o r s o f f i c i a l P a g e | 20

FOR MORE UPDATES AND MORE MATERIAL DO LIKE OUR FACEBOOK PAGE httpwwwfacebookcomsscmentorsofficial

Options

1) Carrot

गाजर

2) Tomato

टमाटर

3) Potato

आि 4) Onion

पयाज़

Correct Answer Onion

Q81 DNA stands for

िीएनएकापणय प mdashmdash- हहॴ 18Jan2017

Options

1) Di Nucleic Acid

िाईनयनकिकएलसि

2) Deoxy Nucleic Acid

िीओकसीनयनकिकएलसि

3) Diribonucleic Acid

िाईराइबह८नयनकिकएलसि

4) Deoxyribonucleic Acid

िीऑकसीराइबह८नयनकिकएलसि

Correct Answer Deoxyribonucleic Acid

Q82 Organisms that generate energy

using light are known as

जह८जीवाणपरकािकाउपयह८गकरउजायउतपननकरतीहहॴ उनह mdashmdash कहॳ पमजानाजाताहहॴ

18Jan2017

Options

1) Chaemolithotrophs

ककमह८लििह८टरह८पस

2) Oligotrophs

ओलिगह८टरह८पस

3) Bacteria

बहॴकटीररया 4)Photoautotrophs

फह८टह८ओटह८टरह८पस

Correct Answer Photoautotrophs

An oligotroph is an organism that can

live in an environment that offers very

low levels of nutrients

Q83 Which drug is used as an

Antidepressant

ककसदवाएकहतािारह८धीकहॳ पमपयोगककयाजाताहहॴ Options

1) Oxybutynin

ओकसीलयटीनन

2)Tramadol

टरहॳमहॳिह८ि

3 ) Sumatriptan

समहॳटरीपटहॳन

4) Bupropion

लयपरह८पपयह८न

Correct Answer Bupropion

लयपरह८पपयह८न

Q84 The orange colour of carrot is

because of

गाजरकानारगीरगनननननलिखितमसहॳककसीएककीवजहसहॳहह८ताहहॴ 18Jan2017

Options

1) it grows in the soil

यहलम ीमउगतीहहॴ 2) Carotene

कहॴ रह८टीन

3) it is not exposed to sunlight

यहसययपरकािकहॳ सपकय मनहीआती 4) the entire plant is oranqe in colour

सनपणयपह९धानारगीरगकाहह८ताहहॴ Correct Answer Carotene

Q85 Snake venom is highly modified

saliva containing

F A C E B O O K

P A G E h t t p w w w f a c e b o o k c o m s s c m e n t o r s o f f i c i a l P a g e | 21

FOR MORE UPDATES AND MORE MATERIAL DO LIKE OUR FACEBOOK PAGE httpwwwfacebookcomsscmentorsofficial

सापकाजहरअततयाचधकसिह८चधतिारहह८तीहहॴनजसमहॳ mdashmdash- हह८ताहहॴ Options

l)Prototoxins

परह८टह८टॉनकसस

2)Neutrotoxins

नयटरोटॉनकसस

3)Zootoxins

जटॉनकसस

4)Electrotoxins

इिहॳकटरह८टॉनकसस

Correct Answer Zootoxins

जटॉनकसस

Q86 Which type of pathogen causes the

water-borne disease Schistosomiasis

ककसपरकारकारह८गज़नकजिजननतरह८गलससटह८सह८लमलससकाकारणबनताहहॴ

18Jan2017

Option

1) Parasitic

परजीवी 2)Protozoan

परह८टह८जआ

3) Bacterial

बहॴकटीररयि

4) Viral

वायरि

Correct Answer Parasitic

Schistosomiasis also known as snail

fever and bilharzia is a disease caused

by parasitic

flatworms called schistosomes

Q87 Prothrombin responsible for

clotting of blood is released by

परह८िह८ननबन

जह८रकतकािककाजमनहॳकहॳ लिएनजनमहॳदारहहॴ mdashndash

कहॳ दवारासतरापवतककयाजाताहहॴ

19Jan2017

Options

1) Small Intestine

छह८टीआत

2) Blood Platelets

रकतपिहॳटिहॳटस

3) Large Intestine

बड़ीआत

4Heart

हदय

Correct Answer Blood Platelets

Q88 Acacia arabica is the scientific

name of

अकहॳ लियाअरहॳबबका mdashmdashndash कावहॴजञाननकनामहहॴ 19-Jan-2017

Options

1) Neem

नीम

2) Teak

सागह९न

3) Babhul

बबि

4) Pomegranate

अनार

Correct Answer Babhul

Q89 Cannis Vulpes is the scientific

name of

कहॴ ननसवनमपस mdashmdash- कावहॴजञाननकनामहहॴ 19-Jan-2017

Options

1) Dog

कतता 2) Wolf

भहॳडड़या 3) Fox

िह८मड़ी 4) Hyena

िाकिबगघा

F A C E B O O K

P A G E h t t p w w w f a c e b o o k c o m s s c m e n t o r s o f f i c i a l P a g e | 22

FOR MORE UPDATES AND MORE MATERIAL DO LIKE OUR FACEBOOK PAGE httpwwwfacebookcomsscmentorsofficial

Correct Answer Fox

Q90 The beetroot is the portion of the

beet plant

चकदरपह९धहॳका mdashmdashndash भागहहॴ 19-Jan-2017

Options

1) tap root

मखयजड़

2) Adventitious

आकनसमक

3) bulb of the stem

तनहॳकाकद

4) Rhizome

परकद

Correct Answer tap root

Q91 What is the basic unit of heredity

आनवलिकताकीबननयादीइकाईकयाहहॴ 19-Jan-2017

Options

1) DNA

िीएनए

2) RNA

आरएनए

3) Chromosome

िह८मह८सह८म

4) Gene

जीन

Correct Answer gene

Genes are the units of heredity and are

the instructions that make up the bodyrsquos

blueprint They code for the proteins

that determine virtually all of a personrsquos

characteristics Most genes come in

pairs and are made of strands of genetic

material called deoxyribonucleic acid

or DNA

Q92 Lungs are the primary organs of

फहॳ फड़हॳmdashndashकहॳ परािलमकअगहहॴ

19-Jan-2017

Options

1) Digestion

पाचन

2) Constipation

कलज

3) Perspiration

पसीना 4)Respiration

शवसन

Correct Answer Respiration

Q93 Sugarcane is a type of

गननाएकपरकारका mdash- हहॴ 20-Jan-2017

Options

1)creeper

िता 2)tree

पहॳड़

3)shrub

झाड़ी 4)grass

घास

Correct Answer grass

Q94 Who is commonly known as ldquothe

Father of Microbiologyrdquo

सामानयत ldquo सकषमजीवपवजञानकहॳ जनक lsquo

कहॳ नामसहॳककसहॳजानाजातहहॴ 20-Jan-2017

Options

1) Robert Hooke

रॉबटयहक

2) Antonie Philips van Leeuwenhoek

एटह८नीकफलिपवानमयएनहह८क

3) Carl Linnaeus

काियिीनाईयस

4) Charles Darwin

चामसयिापवयन

F A C E B O O K

P A G E h t t p w w w f a c e b o o k c o m s s c m e n t o r s o f f i c i a l P a g e | 23

FOR MORE UPDATES AND MORE MATERIAL DO LIKE OUR FACEBOOK PAGE httpwwwfacebookcomsscmentorsofficial

Correct Answer Antonie Philips van

Leeuwenhoek

Q95 For the aquatic organisms the

source of food is

जिीयजीवाणकािाघसरह८तहहॴ 20-Jan-2017

Options

1) Phytoplankton

फायटह८पिहॳकटन

2) Sea Weed

समदरीिहॴवाि

3)Aqua plankton

एकवापिहॳकटन

4) Zooplankton

जपिहॳकटन

Correct Answer Phytoplankton

Q96 Haemoglobin has the highest

affinity with which of the following

हीमह८गिह८बबनकीननननमसहॳककसकहॳ सािउततमसमानताहहॴ

20-Jan-2017

Options

1)SO2

2)CO2

3)CO

4)NO2

Correct Answer CO

It has a greater affinity for hemoglobin

than oxygen does It displaces oxygen

and quickly binds so very little oxygen

is transported through the body cells

Q97 Who developed the theory of

Evolution

उदपवकासकालसदातककसनहॳपवकलसतककया

20-Jan-2017

Options

1) Charles Darwin

चामसयिापवयन

2) Isaac Newton

आयजहॳकनयटन

3) Pranav Mistry

परणवलमसतरी 4) Galileo Galilei

गहॳलिलियह८गहॳिीिी Correct Answer Charles Darwin

Q98 The primary function of RNA is

RNA कापरािलमककाययहह८ताहहॴ 20-Jan-2017

Options

1) Photosynthesis

परकािसशिहॳषण

2) Protein Synthesis

परह८टीनसशिहॳषण

3) Replication

परनतकनतबनाना 4) Translation

अनवादकरना Correct Answer Protein Synthesis

There are two main functions of RNA

It assists DNA by serving as a messenger

to relay the proper genetic information

to countless numbers of ribosomes in

your body The other main function of

RNA is to select the correct amino acid

needed by each ribosome to build new

proteins for your body

Q99 ______is the movement of

molecules across a cell membrane from

a region of their lower concentration to

a region of their higher concertration

उचचसादरताकहॳ कषहॳतरसहॳउसकीकमसादरतावािहॳकषहॳतरकीतरफएककह८लिकाखझमिीकहॳ माधयमसहॳहह८नहॳवािाअणओकहॳ सचिनकह८ mdash- कहतहॳहहॴ Options

1) Diffusion

पवसरण

2) Osmosis

ऑसमह८लसस

F A C E B O O K

P A G E h t t p w w w f a c e b o o k c o m s s c m e n t o r s o f f i c i a l P a g e | 24

FOR MORE UPDATES AND MORE MATERIAL DO LIKE OUR FACEBOOK PAGE httpwwwfacebookcomsscmentorsofficial

3) Active Transport

सकियआवागमन

4) Passive Transport

नननषियआवागमन

Correct Answer Active Transport

Q100 Study of classification of

organisms is known as 20-Jan-2017

जीवाणओकहॳ वगीकरणकहॳ अधययनकह८ mdash-

कहाजाताहहॴ Options

1) Serpentology

सपरहॳटह८िह८जी 2) Virology

वायरह८िह८जी 3) Taxonomy

टहॴकसोनह८मी 4) Physiology

कफनज़यह८िह८जी Correct Answer Taxonomy

Q101 Photosynthesis takes place inside

plant cells in

परकािसशिहॳषणवनसपनतकह८लिकामनसति mdash

mdashmdash महह८ताहहॴ 20-Jan-2017

Options

1) Ribosomes

राइबह८सह८नस

2) Chloroplasts

किह८रह८पिासट

3) Nucleus

नयकलियम

4) Mitochondria

माईटह८कोडडरया Correct Answer Chloroplasts

Q102 ______ is the cell organelle in

which the biochemical processes of

respiration and energy production

occur

mdashmdash- वहकह८लिकाअगहहॴ नजसमहॳशवसनऔरउजायउतपादनकहॳ जहॴसीजहॴवरासायननकपरकियायहह८तीहहॴ 20-Jan-2017

Options

1) Mitochondria

माइटह८कोडडरया 2) Chloroplast

किह८रह८पिासट

3) Ribosomes

राइबह८सह८नस

4) Nucleus

नयकिीयस

Correct Answer Mitochondria

Q103 Which non-flowering spore

bearing plants have roots

ककसफिनिगनहॳवािहॳऔरबीजाणधारकपह९धह८कीजड़हॳहह८तीहहॴ 21-Jan-2017

Options

1) Mosses

मह८सहॳस

2) Angiosperms

एननजयह८सपनसय 3) Ferns

फनसय 4) Gymnosperms

नजननह८सपनसय Correct Answer ferns

Q104 Which of the following is an

excretory organ of cockroach

नननननलिखितमसहॳकह९नसानतिच हॳकाउतसजयनअगहहॴ

21-Jan-2017

Options

F A C E B O O K

P A G E h t t p w w w f a c e b o o k c o m s s c m e n t o r s o f f i c i a l P a g e | 25

FOR MORE UPDATES AND MORE MATERIAL DO LIKE OUR FACEBOOK PAGE httpwwwfacebookcomsscmentorsofficial

1) Malphigian Tubules

मनमफनजयनटयबमस

2) Nephridia

नहॳकफरडिया 3) Coxal Gland

कह८कसिगरचिया 4) Green Gland

गरीनगरचिया Correct Answer Malphigian Tubules

Q105 Evaporation of water takes place

in which part of plants

पानीकहॳ वाषपीकरणकीकियापह९धोकहॳ ककसभागसहॳहह८तीहहॴ 21-Jan-2017

Options

1) Stem

तना 2) Stomata

सटह८मटा 3) Branch

िािाए

4) Fruit

फि

Correct Answer Stomata

Evaporation accounts for the movement

of water to the air from sources such as

the soil canopy interception and

waterbodies Transpiration accounts for

the movement of water within a plant

and the subsequent loss of water as

vapour through stomata in its leaves

Q106 A is the fleshy spore-bearing

fruiting body of a fungus

mdashmdashndashकवककामासि

बीजाणधारणकरनहॳवािाफिनहॳवािाअगहहॴ 21-

Jan-2017

Options

1) aloe vera

एिह८वहॳरा 2) Coral

मगा 3) Cactus

कहॴ कटस

4) Mushroom

ककरमतता Correct Answer mushroom

Q107 Which of the following is a fungal

disease

नननननलिखितमसहॳकह९नसाफफदसहॳहह८नहॳवािाएकरह८ग हहॴ

21-Jan-2017

Options

1) Dermatitis

तवचािह८ध

2) Cholera

हहॴजा 3) Jaundice

पीलिया 4) Indigofera

इननिगह८फहॳ रा Correct Answer Dermatitis

Dermatitis also known as eczema is a

group of diseases that results in

inflammation of the skin These diseases

are characterized by itchiness red skin

and a rash In cases of short duration

there may be small blisters while in

long-term cases the skin may become

thickened

Q108 In which form is glucose stored in

our body

हमारहॳिरीरमगिकह८जकासचयककस पमककयाजाताहहॴ

21-Jan-2017

Options

1) Insulin

F A C E B O O K

P A G E h t t p w w w f a c e b o o k c o m s s c m e n t o r s o f f i c i a l P a g e | 26

FOR MORE UPDATES AND MORE MATERIAL DO LIKE OUR FACEBOOK PAGE httpwwwfacebookcomsscmentorsofficial

इसलिन

2) Glucose

गिकह८ज

3) Glycogen

गिायकह८जहॳन

4) Fat

वसा Correct Answer Glycogen

Excess glucose is stored in the liver as

the large compound called glycogen

Glycogen is a polysaccharide of glucose

but its structure allows it to pack

compactly so more of it can be stored in

cells for later use

Q109 Where do plants synthesize

protein from

पह९धहॳपरह८टीनसशिहॳषणकहासहॳकरतहॳहहॴ

Options

1) Fatty Acids

वसाऐलसि

2) Sugar

िकर

3) Amino Acids

एलमनह८ऐलसि

4) Starch

सटाचय Correct Answer Amino Acids

Q110 Which part of the brain is

responsible for triggering actions like

thinking intelligence memory and

ability to learn

मनसतषककाकह९नसाटहससासह८चनहॳ बनधदमानी याददाशतऔरसीिनहॳकीकषमताजहॴसीकियाओकह८परहॳररतकरताहहॴ 21-Jan-2017

Options

1) Diencephalon

िायएनसहॳफहॳ िह८न

2) Hypothalamus

हयपह८िहॳिहॳमस

3) Cerebrum

सहॳरहॳिम

4) Control

कटरह८ि

Correct Answer Cerebrum

Q111 Which of the following is also

known as the Biochemical Laboratory

of the Human Body

नननननलिखितमसहॳककसहॳमानविरीरकीजहॴवरसायनपरयह८गिािाभीकहाजाताहहॴ 21-Jan-2017

Options

1) Small Intestine

छह८टीआत

2)Brain

मनसतषक

3) Pancreas

अगनयािय

4) Liver

नजगर

Correct Answer Liver

The liver makes bile that will help

emulsify and digest the fats we eat

The liver takes toxic substances and

convert them using enzymes the liver

cells makes into a non toxic form so the

body can dispose of them

The liver also converts fats protein and

carbohydrates into glucose which is the

energy source for our cells to use

The liver takes amino acids and makes

proteins by combining them

Q112 The yellow colour of human urine

is due to

मानवमतरकापीिारग mdashndash कीवजहसहॳहह८ताहहॴ 22-

Jan-2017

Options

1) Bile Salts

F A C E B O O K

P A G E h t t p w w w f a c e b o o k c o m s s c m e n t o r s o f f i c i a l P a g e | 27

FOR MORE UPDATES AND MORE MATERIAL DO LIKE OUR FACEBOOK PAGE httpwwwfacebookcomsscmentorsofficial

पपततनमक

2) Cholesterol

कह८िहॳसटरह८ि

3) Lymph

लिनफ

4) Urochrome

यरह८िह८म

Correct Answer Urochrome

Urobilin or urochrome is the chemical

primarily responsible for the yellow

color of urine

Q113 The wilting of plants takes place

due to

पह९धह८कालिचििहह८नाकी mdashmdash- कीवजहसहॳहह८ताहहॴ 22-Jan-2017

Options

1)Photosynthesis

परकािसशिहॳषण

2) Transpiration

वाषपह८तसजयन

3) Absorption

अविह८षण

4) Respiration

शरवसन

Correct Answer Transpiration

Wilting is the loss of rigidity of non-

woody parts of plants This occurs when

the turgor pressure in non-lignified

plant cells falls towards zero as a result

of diminished water in the cells

Q114 Bovidae Ovis is the scientific name of

बह८पविीओपवस mdashndash कावहॴजञाननकनामहहॴ 22-Jan-2017

Options

1) Goat

बकरी 2) Cow

गाय

3) Buffalo

भहॳस

4) Sheep

भहॳड़

Correct Answer Sheep

Q115 Plants get their energy to produce

food from which of the following

पह८धहॳभह८जनकाननमायणकरनहॳकहॳ लिएनननननलिखितमसहॳककससहॳउजायपरापतकरतहॳहहॴ

22-Jan-2017

Options

1) Photosynthesis

परकािसशिहॳषण

2)Bacteria

बहॴकटीररया 3)Fungi

कवक

4)Sun

सयय Correct Answer Sun

Q116 Which of the following is secreted

by the liver

नननननलिखितमसहॳककसकासरावनजगरसहॳहह८ताहहॴ

22-Jan-2017

Options

1) Glucose

गिकह८ज

2) Iodine

आयह८िीन

3) Cortisol

काटटरयसह८ि

4) Bile

पपतत

Correct Answer Bile

The liver makes bile that will help

emulsify and

digest the fats we eat

F A C E B O O K

P A G E h t t p w w w f a c e b o o k c o m s s c m e n t o r s o f f i c i a l P a g e | 28

FOR MORE UPDATES AND MORE MATERIAL DO LIKE OUR FACEBOOK PAGE httpwwwfacebookcomsscmentorsofficial

Q117 Ferns belong to which division of

plants

फनसयपह९धह८कहॳ ककसभागमआतहॳहहॴ

22-Jan-2017

Options

1) Gymnosperms

नजननह८सपनसय 2) Angiosperms

एनजयह८सपनसय 3) Thallophyta

िहॴिह८फाईटा 4)Pteridophyta

टहॳररिह८फाईटा Correct Answer Pteridophyta

Q118 Who invented Antibiotics

एटीबायह८टटककाअपवषकारककसनहॳककयािा

22-Jan-2017

Options

1) Joseph Lister

जह८सहॳफलिसटर

2) William Harvey

पवलियमहाव

3) Robert Knock

रॉबटयनॉक

4)Alexander Fleming

अिहॳकज़िरफिहॳलमग

Correct Answer Alexander Fleming

Q119 Milbecycin is used in the

eradication of

लममबहॳसायलसनका mdashndash

मउनमिनमपरयह८गककयाजाताहहॴ 22-Jan-2017

Options

1) Agricultural Fungus

कपषकवक

2) Agricultural Pests

कपषकीटक

3) Agricultural Herbs

कपषिाक

4)Agricultural Weeds

कपषननराना Correct Answer Agricultural Pests

Milbemycin oxime is a veterinary drug

from the group of milbemycins used as

a broad spectrum antiparasitic It is

active against worms and mites(insects

Q120 Intestinal bacteria synthesizes

which of the following in the human

body

मानविरीरमआतोकहॳ बहॴकटीररयानननननलिखितमसहॳककसकासशिहॳषणकरतहॳहहॴ 22-Jan-2017

Options

1) Vitamin K

पवटालमन K

2) Proteins

परह८टीन

3) Fats

वसा 4) Vitamin D

पवटालमन D

Correct Answer Vitamin K

Q121 is the study of the physical form

and external structure of plants

mdashmdash-

मपह९धह८काभहॴनतक पऔरबाहरीसरचनाकाआदयाककयाजाताहहॴ 22-Jan-2017

Options

1) Physiology

कफनजयह८िह८जी 2) Anatomy

िरीररचनापवजञान

3) Phytomorphology

फाईटह८मह८फह८िह८जी 4)Cytology

कह८लिकापवजञान

Correct Answer Phytomorphology

F A C E B O O K

P A G E h t t p w w w f a c e b o o k c o m s s c m e n t o r s o f f i c i a l P a g e | 29

FOR MORE UPDATES AND MORE MATERIAL DO LIKE OUR FACEBOOK PAGE httpwwwfacebookcomsscmentorsofficial

Q122 Which of the following is a

structural and functional unit of

kidneys

नननननलिखितमसहॳकह९नसीगदोकीसरचनातमकऔरकाययकरीईकाईहहॴ

22-Jan-2017

Options

1) Renette Cells

रहॳनहॳटकह८लिकाए

2) Flame Cells

फिहॳमकह८लिकाए

3) Nephrites

नहॳफ़राइटस

4)Nephrons

नहॳफरोस

Correct Answer Nephrons

Nephron functional unit of the kidney

the structure that actually produces

urine in the process of removing waste

and excess substances from the blood

There are about 1000000 nephrons in

each human kidney

Q123 Which of the following is the

largest part of the human brain

नननननलिखितमसहॳकह९नसामानवमनसतषककासबसहॳबड़ाटहससाहहॴ

23-Jan-2017

Options

1) Ribs

पसलियाा 2) Cerebrum

सहॳरहॳिम

3) Pons

पोस

4)Thalamus

िहॴिहॳमस

Correct Answer Cerebrum

The cerebrum is the largest part of the

human brain making up about two-

thirds of the brainrsquos mass It has two

hemispheres each of which has four

lobes frontal parietal temporal and

occipital

Q124 The auxiliary buds

सहायककालियाmdashndash 23-Jan-2017

Options

1) grow endogenously from the pericycle

पहॳरीसाईककिसहॳअनतजातयपवकलसतहह८ताहहॴ 2) arise endogenously from the main

growing point

मिवपदसहॳअनतजातयउठताहहॴ 3) is an embryonic shoot located in the

axil of a leaf

एकभरणिटहहॴजह८एकपततीकहॳ अकषपरनसतिहह८ताहहॴ 4)arise exogenously from the epidermis

एपपिलमयससहॳबटहजातयतरीकहॳ सहॳउठताहहॴ Correct Answer is an embryonic shoot

located in the axil of a leaf

Q125 Which of the following is a viral

disease

इनमहॳसहॳकह९सीएकवायरिबीमारीहहॴ

23-Jan-2017

Options

1) Polio

पह८लियह८ 2) Tetanus

धनसतनभ

3) Leprosy

कषठरह८ग

4) Plague

पिहॳग

Correct Answer Polio

A viral disease (or viral infection)

occurs when an organismrsquos body is

invaded by pathogenic viruses and

infectious virus particles (virions) attach

to and enter susceptible cells

F A C E B O O K

P A G E h t t p w w w f a c e b o o k c o m s s c m e n t o r s o f f i c i a l P a g e | 30

FOR MORE UPDATES AND MORE MATERIAL DO LIKE OUR FACEBOOK PAGE httpwwwfacebookcomsscmentorsofficial

Poliomyelitis often called polio or

infantile paralysis is an infectious

disease caused by the poliovirus

Tetanusmdash A serious bacterial infection

that causes painful muscle spasms and

can lead to death

Leprosy also known as Hansenrsquos

disease (HD) is a long-term infection by

the bacterium Mycobacterium leprae or

Mycobacterium lepromatosis

Plague is an infectious disease caused by

the bacterium Yersinia pestis

Symptoms include fever weakness and

headache

Q126 Which organisms can help to

carry out Vermicomposting

कह९नसाजीववमीकनपह८नसटगममददकरताहहॴ

23-Jan-2017

Options

1) Nitrifying Bacteria

नाईटरीफाईगबहॴकटीररया 2) Earthworms

कहॴ चऐ

3) Algae

िहॴवि

4) Fungus

कवक

Correct Answer Earthworms

Q127 Contraction of heart is also

known as

हदयकहॳ सकचनकह८ mdash- भीकहाजाताहहॴ 23-Jan-

2017

Options

1) Systole

लससटह८ि

2) Aristotle

अरसत

3) Diastole

िायसटह८ि

4) Lub

मयब

Correct Answer Systole

Diastole is the part of the cardiac cycle

when the heart refills with blood

following systole (contraction)

Ventricular diastole is the period during

which the ventricles are filling and

relaxing while atrial diastole is the

period during which the atria are

relaxing

Q128 Azadirachta indica is the

botanical name of which of the

following

अजाटदराचताइडिकानननननलिखितमसहॳककसकावानसपनतनामहहॴ

23-Jan-2017

Options

1) Rose plant

गिाबकापह९धा 2) Apple tree

सहॳबकापहॳड़

3) Neem

नीम

4)Mango

आम

Correct Answer Neem

Q129 Which of the following is the

main end product of carbohydrate

digestion

नननननलिखितमसहॳकह९नसाकाबोहाइडरहॳटकहॳ पाचनकापरमिअतउतपादकहह८ताहहॴ 23-Jan-2017

Options

1) Fats

वसा 2) Lipids

लिपपडस

3) Glucose

गिकह८ज

4) Cellulose

F A C E B O O K

P A G E h t t p w w w f a c e b o o k c o m s s c m e n t o r s o f f i c i a l P a g e | 31

FOR MORE UPDATES AND MORE MATERIAL DO LIKE OUR FACEBOOK PAGE httpwwwfacebookcomsscmentorsofficial

सहॳमयिह८ज

Correct Answer Glucose

Intestinal absorption of end products

from digestion of carbohydrates and

proteins in the pig hellip During absorption some sugars (fructose or

galactose) released from the

corresponding sucrose and lactose

respectively during digestion were

partly metabolized into glucose by the

enterocyte

Q130 Which of the following glands is a

source of the enzyme Ptyalin

नननननलिखितगरचियोमसहॳएजाइमटयालिनकासरह८तहहॴ 23-Jan-2017

Options

1) Pancreas

अगरािय

2) Thyroid Gland

िाइराइिगरिी 3) Pituitary Gland

पीयषगरिी 4) Salivary Glands

िारगरचियाा Correct Answer Salivary Glands

Q131 Which of the following is not true

about Pteridophyta

ननननमसहॳकह९नसीबातटहॳररिह८फाईटकहॳ बारहॳमसचनहीहहॴ 23-Jan-2017

Options

1) Dominant phase is saprophytes

परमिचरणसहॳपरह८फाईइटसहह८ताहहॴ 2) Main plant body is diploid

पह९दह८कामखयिरीरदपवगखणतहह८ताहहॴ 3) Seeds are present

बीजमह९जदहह८तहॳहहॴ 4)Flowers are absent

फिअनपनसतिहह८तहॳहहॴ

Correct Answer Seeds are present

Q132 The largest dolphin species is the

orca also called as

िॉिकफनकीसबसहॳबड़ीपरजानतकाकानामआकायहहॴनजसहॳ mdash- भीकहतहॳहहॴ 23-Jan-2017

Options

1) Bottle Nose

बाटिनह८ज

2) Baiji

बहॳजी 3) Killer whale

ककिरहहॳि

4)Tucuxi

टकवसी Correct Answer Killer whale

Q133 The fat digesting enzyme Lipase

is secreted by which of the following

वसाकापाचनकरनहॳवािाएजाइमिाइपहॳजनननननलिखितमसहॳककसकहॳ दवारासतरापवतहह८ताहहॴ

24-Jan-2017

Options

1) Kidneys

गद

2) Pancreas

अगनयािय

3) Large Intestine

बड़ीआत

4)Liver

नजगर

Correct Answer Pancreas

Lipase is an enzyme that splits fats so

the intestines can absorb them Lipase

hydrolyzes fats like triglycerides into

their component fatty acid and glycerol

molecules It is found in the blood

gastric juices pancreatic secretions

intestinal juices and adipose tissues

F A C E B O O K

P A G E h t t p w w w f a c e b o o k c o m s s c m e n t o r s o f f i c i a l P a g e | 32

FOR MORE UPDATES AND MORE MATERIAL DO LIKE OUR FACEBOOK PAGE httpwwwfacebookcomsscmentorsofficial

Q134 The arrangement of leaves on an

axis or stem is called

एकअकषयातनहॳपरपनततयोकीयवसिाकह८कयाकहाजाताहहॴ SSC CHSL Science (biology) 2016

Question Paper

24-Jan-2017

Options

1) Phyllotaxy

फाइिह८टहॴकसी 2) Vernation

वनिन

3) Venation

वहॳनहॳिन

4)Phytotaxy

फाइटह८टहॴकसी Correct Answer Phyllotaxy

In botany phyllotaxis or phyllotaxy is

the arrangement of leaves on a plant

stem (from Ancient Greek phyacutellon

ldquoleafrdquo and taacutexis ldquoarrangementrdquo)

Phyllotactic spirals form a distinctive

class of patterns in nature

Q135 The study of Cells is also known

as

कह८लिकाओकहॳ अधययनकह८ mdashmdashndash

भीकहाजाताहहॴ 24-Jan-2017

Options

1) Cytology

सायटह८िह८जी 2) Physiology

कफनजयह८िह८जी 3) Nucleology

नयककमयह८िह८जी 4)Cellology

सहॳिह८िह८जी Correct Answer Cytology

Q136 Which of the following scientists

is also known as the Father of Biology

नननननलिखितमसहॳककसवहॴजञाननककह८ ldquoजीवपवजञानकहॳ जनकrdquoकहॳ नामसहॳभीजानाजाताहहॴ 24-Jan-2017

Options

1) Herbert Spencer

हबयटयसपसर

2) Aristotle

अरसत 3) Lamarck

िहॳमाकय 4)Darwin

िापवयन

Correct Answer Aristotle

Q137 Which cells give rise to various

organs of the plant and keep the plant

growing

कह९नसीकह८लिकाएपह९धह८कहॳ लभननअगह८कह८जनमदहॳतीहहॴऔरपह९धह८कह८बढ़नहॳममददकरतीहहॴ

24-Jan-2017

Options

1) Permanent

सिायी 2) Dermal

तवचीय

3) Meristematic

मररसटहॳमटटक

4)Mature

परह८ढ़

Correct Answer Meristematic

A meristem is the tissue in most plants

containing undifferentiated cells

(meristematic cells) found in zones of

the plant where growth can take place

Q138 Rodentia Muridae is the scientific

name of

F A C E B O O K

P A G E h t t p w w w f a c e b o o k c o m s s c m e n t o r s o f f i c i a l P a g e | 33

FOR MORE UPDATES AND MORE MATERIAL DO LIKE OUR FACEBOOK PAGE httpwwwfacebookcomsscmentorsofficial

रह८िहॳलियानयररिी mdashmdash- कावहॴजञाननकनामहहॴ 24-

Jan-2017

Options

1) Mouse

चहा 2) Squirrel

चगिहरी 3) Monkey

बदर

4) Lizard

नछपकिी Correct Answer Mouse

Q139 Name the scientist who proposed

the cell theory

कह८लिकालसदातकापरसतावदहॳनहॳवािहॳवहॴजञाननककानामबताइए 24-Jan-2017

Options

1) Schleiden and Schwann

िीमिनऔरशरववान

2) Lamarck

िहॳमाकय 3) Treviranus

टरहॳवायरहॳनस

4)Whittaker and Stanley

हीटकरऔरसटहॳनिहॳ Correct Answer Schleiden and

Schwann

Q140 The flower with the worldrsquos

largest bloom is

दननयाकासबसहॳबड़ाफिखििनहॳवािा mdashmdashndash हहॴ 24-Jan-2017

Options

1) Pando

पािह८ 2) Posidonia

पह८सीिह८ननया 3) Rafflesia arnoldii

ररफिहॳलियाअनोमिी 4)Helianthus annuus

हहॳलिएनिसएनयअस

Correct Answer Rafflesia arnoldii

Rafflesia arnoldii is a species of

flowering plant in the parasitic genus

Rafflesia It is noted for producing the

largest individual flower on earth It has

a very strong and horrible odour of

decaying flesh earning it the nickname

ldquocorpse flower

Q141 Deficiency of which vitamin

causes night blindness

ककसपवटालमनकीकमीकहॳ कारणरतौधीहह८ताहहॴ 24-Jan-2017

Options

1) Vitamin K

पवटालमन K

2) Vitamin C

पवटालमन C

3) Vitamin B1

पवटालमन B1

4)Vitamin A

पवटालमन A

Correct Answer Vitamin A

Q142 Nongreen plants lack which of the

following

गहॴर-

हररतवनसपनतमनननननलिखितमसहॳककसकीकमीहह८तीहहॴ

24-Jan-2017

Options

1) Chlorophyll

किह८रह८कफि

2) Lycophyll

िायकह८कफि

3) Cyanophyll

F A C E B O O K

P A G E h t t p w w w f a c e b o o k c o m s s c m e n t o r s o f f i c i a l P a g e | 34

FOR MORE UPDATES AND MORE MATERIAL DO LIKE OUR FACEBOOK PAGE httpwwwfacebookcomsscmentorsofficial

सायनह८कफि

4)Phototropism

फह८टह८टरोपपजम

Correct Answer Chlorophyll

Q143 Organisms that use light to

prepare food are known as

जह८जीवपरकािकाउपयह८गकरभह८जनतहॴयारकरतहॳहहॴ उनह mdashmdash- कहॳ पमजानजाताहहॴ 24-Jan-2017

Options

1) Autotrophs

सवपह८षी 2) Heterotrophs

पवषमपह८षज

3) Omnivores

सवायहारी 4)Decomposers

पवघटनकरनहॳवािा Correct Answer Autotrophs

autotrophs often make their own food

by using sunlight carbon dioxide and

water to form sugars which they can use

for energy Some examples of

autotrophs include plants algae and

even some bacteria Autotrophs

(producer) are important because they

are a food source for heterotrophs

(consumers)

A heterotroph is an organism that

ingests or absorbs organic carbon

(rather than fix carbon from inorganic

sources such as carbon dioxide) in order

to be able to produce energy and

synthesize compounds to maintain its

life Ninety-five percent or more of all

types of living organisms are

heterotrophic including all animals and

fungi and some bacteria

Q144 Which of the following is a

primary function of haemoglobin

नननननलिखितमसहॳकह९नसाटहमह८गिह८बबनकाएकपरािलमककाययहहॴ

25-Jan-2017

Options

1) Utilization of energy

उजायकाउपयह८गकरना 2) Prevention of anaemia

रकतामपताहह८नहॳसहॳरह८कना 3) Destruction of bacteria

बहॴकटीररयाकापवनािकरना 4) To transport oxygen

ऑकसीजनकावहनकरना Correct Answer To transport oxygen

Q145 Vascular bundles are absent in

सवहनीबिि mdashmdash- मअनपनसतिरहतहॳहहॴ 25-Jan-2017

Options

1) Bryophyta

िायह८फाइटा 2) Pteridophyta

टहॳररिह८फाईटा 3) Gymnosperms

नजननह८सपमय 4) Angiosperms

एननजयह८सपहॳनसय Correct Answer Bryophyta

Q146 Sauria Lacertidae is the scientific

name of

सहॴररयािहॳसरटाईिी mdashmdashndash कावहॴजञाननकनामहहॴ 25-Jan-2017

Options

1) Crocodile

मगरमचछ

2) Hippopotamus

टहपपह८पह८टहॳमस

3) Lizard

नछपकिी 4) House fly

F A C E B O O K

P A G E h t t p w w w f a c e b o o k c o m s s c m e n t o r s o f f i c i a l P a g e | 35

FOR MORE UPDATES AND MORE MATERIAL DO LIKE OUR FACEBOOK PAGE httpwwwfacebookcomsscmentorsofficial

घरहॳिमकिी Correct Answer Lizard

Q147 Which type of pathogen causes

the water-borne disease SARS (Severe

Acute Respiratory Syndrome)

ककसपरकािकारह८गज़नकजिजननतबीमारीसासयकाकारणबनताहहॴ 25-Jan-2017

Options

1) Viral

वायरि

2) Parasitic

परजीवी 3) Protozoan

परह८टह८जअन

4) Bacterial

बहॴकटीररयि

Correct Answer Viral

Q148 Which of the following organs

produces the enzyme lipase

नननननलिखितमसहॳकह९नसाअगिायपहॳजएजाइमउतपननकरताहहॴ 25-Jan-2017

Options

1) Pancreas

अगनयािय

2) Large Intestine

बड़ीआत

3) Liver

नजगर

4) Small Intestine

छह८टीआत

Correct Answer Pancreas

Q149 A is a long internode forming the

basal part or the whole of a peduncle

एक mdashmdash- एकिबाइटरनह८िहहॴ जह८ननचिाटहससायासनपणयिठिबनताहहॴ 25-

Jan-2017

Options

1) Rhizome

परकद

2) Rachis

महॳ दि

3) floral axis

पषपअकष

4) Scape

भगदड़

Correct Answer scape

Q150 ndash Which of the following

organisms are considered to be both

Living and Non-living

नननननलिखितमसहॳकह९नसहॳजीवाणकह८जीपवतऔरअजीपवतमानाजाताहहॴ

25-Jan-2017

Options

1) Bacteria

बहॴकटीररया 2) Fungi

कवक

3) Algae

िहॴवाि

4)Virus

वायरस

Correct Answer Virus

They are considered to be living as they

possess a protein coat as a protective

covering DNA as the genetic material

etc

They are said to be non-living as they

can be crystallised and they survive for

billions of years They can tolerate high

temperatures freezing cold

temperatures ultra-violet radiations etc

Q151 Deficiency of fluorine causes

which of the following

फिह८ररनकीकमीकहॳ कारणनननननलिखितमसहॳकयाहह८ताहहॴ

F A C E B O O K

P A G E h t t p w w w f a c e b o o k c o m s s c m e n t o r s o f f i c i a l P a g e | 36

FOR MORE UPDATES AND MORE MATERIAL DO LIKE OUR FACEBOOK PAGE httpwwwfacebookcomsscmentorsofficial

27-Jan-2017

Options

1) Dental Caries

िटिकहॴ ररज

2) Scurvy

सकवरी 3) Anaemia

रकतामपता 4) Arthritis

गटठया Correct Answer Dental Caries

Q152 In a Punnett Square with the

cross AaBb x AaBb how many Aabb

genotypes would be created

पनहॳटसककायरमिह८स AaBb x AaBb कहॳ साि

ककतनहॳ Aabb जीनह८टाइपबनगहॳ 27-Jan-2017

Options

1) 1

2) 8

3) 2

4) 3

Correct Answer 2

Q153 Which of the following is the

Controlling Center of the Cell

नननननलिखित म सहॳ कह८लिकाका ननयतरण

क दर कह९न हहॴ

27-Jan-2017

Options

1) Nucleus

क दर

2) Plasma

पिाजमा 3) Lysosome

िायसह८सह८म

4) Chromosome

िह८मह८सह८म

Correct Answer Nucleus

The control centre of the cell is the

nucleus in eukaryotic cells The nucleus

contains genetic material in the form of

DNA

Q154 Myopia affects which of the

following organs

मायह८पपयानननननलिखितअगह८मसहॳककसहॳपरभापवतकरताहहॴ

25-Jan-2017

Options

1) Heart

हदय

2) Skin

तवचा 3) Eyes

आािहॳ 4)Mouth

मह

Correct Answer Eyes

Q155 Which of the following bears

flowers

नननननलिखितमसहॳकह९नफिधारणकरताहहॴ

25-Jan-2017

Options

1) Bryophyta

िायह८फाइटा 2) Pteridophyta

टहॳरीिह८फाईटा 3) Gymnosperms

नजननह८सपमय 4)Angiosperms

एननजयह८सपमय Correct Answer Angiosperms

Q156 Oxygenated blood flows out of the

heart through the

ऑकसीजनयकतरकत mdashmdashmdash

कहॳ माधयमसहॳहदयकहॳ बाहरबहताहहॴ 25-Jan-2017

F A C E B O O K

P A G E h t t p w w w f a c e b o o k c o m s s c m e n t o r s o f f i c i a l P a g e | 37

FOR MORE UPDATES AND MORE MATERIAL DO LIKE OUR FACEBOOK PAGE httpwwwfacebookcomsscmentorsofficial

Options

1) Aorta

महाधमनी 2) pulmonary artery

फहॳ फड़हॳकीधमनी 3) vena cava

वहॳनाकावा 4)Atrium

चह९क

Correct Answer aorta

Q157 Blood leaving the liver and

moving towards the

heart has a higher concentration of

नजगरसहॳननकिकरहदयकीतरफजानहॳवािहॳरकतम mdashmdashmdashmdash कीउचचसादरताहह८तीहहॴ 27-Jan-2017

Options

1) Lipids

लिपपडस

2) Urea

यररया 3) Bile Pigments

पपततकहॳ रगकरण

4) Carbon dioxide

काबयनिायऑकसाइि

Correct Answer Bile Pigments

Urea is nitrogen containing substance

which is produced in the liver in order

to deal with excess amino-acids in the

body As urea is produced it leaves the

liver in the blood stream and passes via

the circulatory system to all parts of the

body

Q158 Bulb is a modification of which

part of a plant

बमबएकपह९धहॳकहॳ ककसटहससहॳकाएक पातरणहह८ताहहॴ 27-Jan-2017

Options

1) The root

जड़

2) The stem

तना 3) The radicle

मिाकर

4)The fruit

फि

Correct Answer The stem

Q159 Which of the following carries

blood away from the heart to different

body parts

इनमहॳसहॳकह९नरकतकह८हदयसहॳिरीरकहॳ पवलभननअगह८तकिहॳजातीहहॴ

27-Jan-2017

Options

1) Arteries

धमननया 2) Nerves

तबतरहाए

3) Capillaries

कहॳ लिकाए

4)Veins

नसहॳ Correct Answer Arteries

Q160 The series of processes by which

nitrogen and its compounds are

interconverted in the environment and

in living organisms is called

27-Jan-2017

Options

1)Absorption of Nitrogen

2)Ammonification

3)Nitrogen Fixation

4)Nitrogen Cycle

Correct Answer Nitrogen Cycle

Ammonification or Mineralization is

performed by bacteria to convert

organic nitrogen to ammonia

F A C E B O O K

P A G E h t t p w w w f a c e b o o k c o m s s c m e n t o r s o f f i c i a l P a g e | 38

FOR MORE UPDATES AND MORE MATERIAL DO LIKE OUR FACEBOOK PAGE httpwwwfacebookcomsscmentorsofficial

Nitrification can then occur to convert

the ammonium to nitrite and nitrate

Nitrogen fixation is a process by which

nitrogen in the Earthrsquos atmosphere is

converted into ammonia (NH3) or other

molecules available to living organisms

Q161 BCG vaccine is given to protect

from which of the following

बीसीजीकाटटकानननननलिखितमसहॳककसकहॳ बचावकहॳ लिएटदयाजातहहॴ

27-Jan-2017

Options

1) Jaundice

पीलिया 2) Anaemia

रकतमपता 3) Tuberculosis

कषयरह८ग

4) Polio

पह८लियह८ Correct Answer Tuberculosis

Q162 Parallel venation is found in

समानतरवहॳनहॳिन mdashmdashmdash- मपायाजाताहहॴ 27-Jan-2017

Options

1) plants which are monocots

पह९धहॳजह८एकबीजपतरीहह८तहॳहहॴ 2) plants which have a dicot stem

वहॳपह९धहॳनजनकातनादपवदलियहह८ताहहॴ 3) plants with leaves similar to Tulsi

वहॳपह९धहॳनजनकीपनततयतिसीकीपनततयोकहॳ समानहह८तहॳहहॴ 4)plants with tap roots

टहॳप टवािहॳपह९धहॳ Correct Answer plants which are

monocots

Q163 The hardest part of the body is

िरीरकासबसहॳकठह८रभाग mdashndash हहॴ 27-Jan-2017

Options

1) Bones

हडडिय

2) Tooth Enamel

दातकहॳ इनहॳमि

3) Skull

िह८पड़ी 4) Spinal Cord

महॳ रजज

Correct Answer Tooth Enamel

Q164 Which type of pathogen causes

the waterborne disease E coli Infection

ककसपरकारकारह८गजननकजिजननतरह८गईकह८िाईसिमणकाकारणबनताहहॴ 27-Jan-2017

Options

1) Protozoan

परह८टह८जआ

2) Parasitic

परजीवी 3) Bacterial

बहॴकटीररयि

4)Viral

वायरि

Correct Answer Bacterial

Q165 The amount of blood filtered

together by both the kidneys in a 70 kg

adult male human in a minute is

70 की गरा वािहॳएकवयसकप षमएकलमनटमदह८नोगदकहॳदवाराएकसािचाबनीगयीरकतकीमातरहह८तीहहॴ 29-Jan-2017

Options

1) 1100 ml

1100 लमलि

2) 100 ml

F A C E B O O K

P A G E h t t p w w w f a c e b o o k c o m s s c m e n t o r s o f f i c i a l P a g e | 39

FOR MORE UPDATES AND MORE MATERIAL DO LIKE OUR FACEBOOK PAGE httpwwwfacebookcomsscmentorsofficial

100 लमलि

3) 1500 ml

1500 लमलि

4) 500 ml

500 लमलि

Correct Answer 1100 ml

Q166 Which feature of a plant helps to

distinguish a monocot from a dicot

पह९धहॳकीवहकह९नसीपविहॳषताहहॴजह८एकदपवदलियहॳऔरएकएकदिीयपह९धहॳसहॳभहॳदकरनहॳममददकरतीहहॴ 29-Jan-2017

Options

1) Pollination

परागम

2) Venation

वहॳनहॳिन

3) Vernation

वनिन

4) Aestivation

एसटीवहॳिहॳन

Correct Answer venation

Q167 The Mutation Theory was

proposed by

उतवररवतयनकालसदात mdashmdashndash

कहॳ दवरापरसतापवतककयाजाताहहॴ 29-Jan-2017

Options

1) Charles Lyell

चामसयलियहॳि

2) William Smith

पवलियमनसमि

3) Hugo De Vries

हयगह८िीराईस

4)Harrison Schmitt

हहॳरीसननसमट

Correct Answer Hugo De Vries

Q168 Which type of pathogen causes

the waterborne disease HepatitisA

ककसपरकारकहॳ रह८गजनकजिजननतरह८गहहॳपहॳटाइटटस-A काकारणबनताहहॴ

29-Jan-2017

Options

1) Parasitic

परजीवी 2) Viral

वायरि

3) Protozoan

परह८टह८जआ

4) Bacterial

बहॴकटीररयि

Correct Answer Viral

Q169 In a Punnett Square with the

cross AaBb x Aabb how many AaBb

genotypes would be created

पनहॳटसकवायरमिह८स AaBb x Aabb

कहॳ सािककतनहॳ AaBb जीनह८टाइपबनगहॳ 29-Jan-

2017

Options

1) 4

2) 1

3) 7

4) 6

Correct Answer 4

Q170 Arboreal Ateles is the scientific

name of

अिह८ररयिएटटलिस mdashmdashmdash कावहॴजञाननकनामहहॴ 29-Jan-2017

Options

1) Squirrel

चगिहरी 2) Sparrow

गह८रहॴया 3) Lizard

नछपकिी 4) Spider monkey

F A C E B O O K

P A G E h t t p w w w f a c e b o o k c o m s s c m e n t o r s o f f i c i a l P a g e | 40

FOR MORE UPDATES AND MORE MATERIAL DO LIKE OUR FACEBOOK PAGE httpwwwfacebookcomsscmentorsofficial

मकड़ीबदर

Correct Answer Spider monkey

Q171 Which type of pathogen causes

the waterborne disease Salmonellosis

ककसपरकारकारह८गाणजिजननतबीमारीसािमह८नहॳिह८लसज़काकारकहहॴ

29-Jan-2017

Options

1) Algal

िहॳवालियहॳ 2) Parasitic

परजीवी 3) Bacterial

बहॴकटीररयि

4)Viral

वायरि

Correct Answer Bacterial

An infection with salmonella bacteria

commonly caused by contaminated food

or water

Symptoms include diarrhoea fever

chills and abdominal pain

Q172 is a condition in which there is a

deficiency of red cells or of haemoglobin

in the blood

mdashmdash-

एकनसिनतहहॴनजसमहॳरकतमिािकह८लिकाओकीयाहीमह८गिह८बबनकीकमीहह८तीहहॴ 29-Jan-2017

Options

1) Albinism

एनमबननजम

2) Propyria

परह८पीररया 3) Anaemia

एनीलमया 4)Keloid disorder

कहॳ िह८इिडिसओिर

Correct Answer Anaemia

Q173 Ananas comosus is the scientific

name of

Options

अनानासकह८मह८सस mdashmdashmdashndash

कावहॴजञाननकनामहहॴ 29-Jan-2017

1) Custard Apple

सीताफि

2) Pineapple

पाइनएपपि

3) Bamboo

बास

4)Pomegranate

अनार

Correct Answer Pineapple

Q174 Which organ produces insulin

कह९नसाअगइनसलिनपहॴदाकरताहहॴ 29-Jan-

2017

Options

1) Liver

यकत

2) Thyroid gland

िायराइिगरिी 3) Spleen

पिीहा 4)Pancreas

अगरयिय

Correct Answer Pancreas

Q175 Which of the following disease is

not caused by water pollution

नननननलिखितमसहॳकह९नसारह८गपानीकहॳ परदषणकहॳकारणनहीहह८ता

29-Jan-2017

Options

1) Cholera

हहॴजा 2) Typhoid

F A C E B O O K

P A G E h t t p w w w f a c e b o o k c o m s s c m e n t o r s o f f i c i a l P a g e | 41

FOR MORE UPDATES AND MORE MATERIAL DO LIKE OUR FACEBOOK PAGE httpwwwfacebookcomsscmentorsofficial

टाइफाइि

3) Asthma

दमा 4)Diarrhoea

दसत

Correct Answer Asthma

Q176 Ocimum tenuiflorum is the

scientific name of

ओलिलममटहॳयईफिह८रमइसकावहॴजञाननकनाम mdash

ndash हहॴ 30-Jan-2017

Options

1) Neem

नीम

2) Mango

आम

3) Babul

बबि

4)Tulsi

तिसी Correct Answer Tulsi

Q177 Which gland secretes bile a

digestive fluid

कह९नसीगरिीपपतत एकपाचनतरिपरदािय सरापवतकरतीहहॴ 30-Jan-2017

Options

1) Pancreas

अगनयािय

2) Liver

यकत

3) Thyroid

िायराइि

4) Testes

टहॳनसटस

Correct Answer liver

Q178 In which of the following the

dominant phase is Gametophyte

नननननलिखितमसहॳककसकहॳ परमिचरणयगमकह८दपवधद (Gametophyte)हहॴ 30-Jan-2017

Options

1) Bryophyta

िायह८फाइटा 2) Pteridophyta

टहॳररिह८फाइटा 3) Gymnosperms

नजननह८सपमय 4) Angiosperms

एननजयह८सपमय Correct Answer Bryophyta

Q179 Anaerobic respiration refers to

which of the following

नननननलिखितमसहॳककसहॳअवायवीयशवसनकहाजाताहहॴ

30-Jan-2017

Options

1) Respiration without Oxygen

ऑकसीजनकहॳ बबनाशवसन

2) Respiration with Oxygen

ऑकसीजनकहॳ सािशवसन

3) Respiration without CO2

काबयनिायऑकसाइिकहॳ बबनाशवसन

4) Respiration with CO2

काबयनिायऑकसाइिकहॳ सािशविन

Correct Answer Respiration without

Oxygen

Q180 Which type of pathogen causes

the waterborne disease Cholera

ककसपरकारकारह८गजनकजिजननतरह८गहहॴजाकाकारणबनताहहॴ

30-Jan-2017

Options

1) Algal

िहॴवालियहॳ

F A C E B O O K

P A G E h t t p w w w f a c e b o o k c o m s s c m e n t o r s o f f i c i a l P a g e | 42

FOR MORE UPDATES AND MORE MATERIAL DO LIKE OUR FACEBOOK PAGE httpwwwfacebookcomsscmentorsofficial

2) Bacterial

बहॴकटीररयि

3) Protozoan

परह८टह८जआ

4) Viral

वायरि

Correct Answer Bacterial

Q181 To which class does

Oxyreductases transferases hydrolases

belong

ओकसीररिकटहॳसटरासफरहॳजहॳस

हाइडरह८िहॳसहॳसककसवगयमआतहॳहहॴ 30-Jan-2017

Options

1) Hormones

हारमोस

2) Enzymes

एजाइनस

3) Proteins

परह८टीनस

4) Vitamins

पवटालमनस

Correct Answer Enzymes

Q182 Which of the following is not true

about Gymnosperms

ननननमसहॳकह९नसीबातअनावतबीजीकहॳ बारहॳमसचनहीहहॴ 30-Jan-2017

Options

1) Dominant phase is saprophytes

परमिचरणसहॳपरह८फाइटसहह८ताहहॴ 2) Vascular bundles are absent

सवहनीबििअनपनसितहह८ताहहॴ 3) spores are heterospores

बीजाणहहॳटहॳरह८सपह८रसहह८तहॳहहॴ 4) Flowers are absent

फिअनपनसितहह८तहॳहहॴ

Correct Answer Vascular bundles are

absent

Q183 The name of first mammal clone sheep is

भहॳड़कीपरिमसतनपायीपरनत प (किह८न)

कानामहहॴ 30-Jan-2017

Options

1) Noori

नरी 2) Dolly

िॉिी 3) Louise

िसी 4)Durga

दगाय Correct Answer Dolly

Q184 Which type of pathogen causes

the water-borne disease Typhoid fever

ककसपरकारकारह८गजनकजिजननतरह८गटाइफाइिबिारकाकारणबनताहहॴ 30-Jan-2017

Options

1) Algal

िहॴवािीय

2) Parasitic

परजीवी 3) Protozoan

परह८टह८जनअन

4)Bacterial

बहॴकटीररयि

Correct Answer Bacterial

Q185 In which part of the cell are

proteins made

कह८लिकाकहॳ ककसटहससहॳमपरह८टीनबनायाजाताहहॴ

31-Jan-2017

Options

1) Reticulum

रहॳटटकिम

F A C E B O O K

P A G E h t t p w w w f a c e b o o k c o m s s c m e n t o r s o f f i c i a l P a g e | 43

FOR MORE UPDATES AND MORE MATERIAL DO LIKE OUR FACEBOOK PAGE httpwwwfacebookcomsscmentorsofficial

2) Golgi apparatus

गह८मजीएपहॳरहॳटस

3) Ribosomes

ररबह८सह८नस

4) Lysosome

िायसह८सह८नस

Correct Answer ribosomes

Proteins are produced by stringing

amino acids together in the order

specified by messenger RNA strands

that were transcribed from DNA in the

cell nucleus The process of synthesizing

a protein is called translation and it

occurs on ribosomes in the cytoplasm of

a cell

Q186 Polio is a disease caused by which

of the following

नननननलिखितमसहॳपह८लियह८कीबबमारह८हह८नहॳकाकारणकयाहहॴ

31-Jan-2017

Options

1) Bacteria

बहॴकटीररयि

2) Mosquito

मचछर

3) Virus

वायरस

4) Cockroach

नतिच हॳ Correct Answer Virus

Polio or poliomyelitis is a crippling and

potentially deadly infectious disease It

is caused by the poliovirus

Q187 ndash Hay fever is a sign of which of

the following

हहॳकफवरनननननलिखितमसहॳककसकाएकसकहॳ तहहॴ

31-Jan-2017

Options

1) Old Age

वदावसिा 2) Malnutrition

कपह८सण

3) Allergy

एिनजय 4) Over Work

अतयचधककाययकरना Correct Answer Allergy

Q188 How many chromosomes does a

human cell contain

एकमानवकह८लिकामककतनहॳगणसतरहह८तहॳहहॴ

29-Jan-2017

Options

1) 6

2) 26

3) 46

4) 66

Correct Answer 46

In humans each cell normally contains

23 pairs of chromosomes for a total of

46 Twenty-two of these pairs called

autosomes look the same in both males

and females The 23rd pair the sex

chromosomes differ between males and

females

Q189 Which of the following is not true

about Bryophyta

ननननमसहॳकह९नसीबातिायह८फाइटकहॳ बारहॳमसचनहीहहॴ 31-Jan-2017

Options

1) Dominant phase is gametophytes

परमिचरणगहॳलमतह८फाइटसहह८ताहहॴ 2) Main plant body is haploid

पह९धहॳकामखयिरीरअगखणतहह८ताहहॴ 3) Spores are homospores

बीजाणहह८मह८सफह८रसहह८तहॳहहॴ 4) Flowers are present

फिमह८जदहह८तहॳहहॴ Correct Answer Flowers are present

F A C E B O O K

P A G E h t t p w w w f a c e b o o k c o m s s c m e n t o r s o f f i c i a l P a g e | 44

FOR MORE UPDATES AND MORE MATERIAL DO LIKE OUR FACEBOOK PAGE httpwwwfacebookcomsscmentorsofficial

Q190 Which aquatic animal has

trailing tentacles

ककसजिीयजानवरकहॳ पीछहॳचिनहॳवािहॳटहॳटकिसहह८तहॳहहॴ

31-Jan-2017

Options

1) Sea horse

समदरीघह८िा 2) Corals

मगा 3) Jelly fish

जहॳिीमछिी 4) Star fish

तारामछिी Correct Answer Jelly fish

Jellyfish with its umbrella-shaped bell

and trailing tentacles

Q191 Which type of pathogen causes

the water-borne disease Poliomyelitis

(Polio)

ककसपरकारकारह८गजनकजिजननतरह८गपह८लियह८मायहॳटटस (पह८लियह८) काकारणहहॴ 31-Jan-

2017

Options

1) Parasitic

परजीवी 2) Algal

िहॴवालिय

3) Viral

वायरि

4) Bacterial

बहॴकटीररयि

Correct Answer Viral

Q192 The outer white part of the eye

that protects the inner structures is

आािकाबाहरीसफहॳ दटहससाजह८आतररकसरचनाओकीरकषाकरताहहॴ वह mdashmdashmdash हहॴ 31-Jan-

2017

Options

1) Iris

आयररस

2) Sclera

सकिहॳरा 3) Retina

रहॳटटना 4) Cornea

कह८ननयया Correct Answer Sclera

Q193 Proteins are made up of

परह८टीनकाननमायण mdashndash सहॳहह८ताहहॴ 31-Jan-2017

Options

1) Amino acids

एलमनह८अनि

2) Fatty acids

वसायकतअनि

3) Glucose

गिकह८ज

4)Nucleotides

नयनकियह८टाईिस

Correct Answer Amino acids

Q194 Moringa Oleifera is the scientific

name of

मह८ररगओलिफहॳ रा mdashmdashndash कावहॴजञाननकनामहहॴ 31-Jan-2017

Options

1) Banyan

बरगद

2) Gulmohar

गिमह८हर

3) Amla

आमिा

F A C E B O O K

P A G E h t t p w w w f a c e b o o k c o m s s c m e n t o r s o f f i c i a l P a g e | 45

FOR MORE UPDATES AND MORE MATERIAL DO LIKE OUR FACEBOOK PAGE httpwwwfacebookcomsscmentorsofficial

4) Drumstick

डरमनसटक

Correct Answer Drumstick

Q195 Kidney stones are composed of

गदकीपिरी mdashndash सहॳबनीहह८तीहहॴ 1-Feb-2017

Options

1) Calcium Oxalate

कहॴ नमसयमओकजहॳिहॳट

2) Sodium Chloride

सह८डियमकिह८राइि

3) Magnesium Nitrate

महॳनगनलियमनाइतटरहॳट

4) Calcium Bicarbonate

कहॴ नमियमबायकबोनहॳट

Correct Answer Calcium Oxalate

Q196 ndash Which of the following is not

true about Angiosperms

ननननमसहॳकह९नसीबातआवतबीजीकहॳ बारहॳमसचनहीहहॴ 1-Feb-2017

Options

1) Dominant phase is gametophytes

परमिचरणगहॳलमतह८फाइटहह८ताहहॴ 2) Vascular bundles are present

सवहनीबििमह९जदहह८ताहहॴ 3) Spores are heterospores

बीजाणहहॳटहॳरह८सपह८रसहह८तहॳहहॴ 4) Seeds are covered

बीजढकहॳ हह८तहॳहहॴ Correct Answer Dominant phase is

gametophytes

Q197 All of the following are excretory

(waste) products of animals except

नननननलिखितमसहॳककसएककह८छह८ड़करअनयसभीपराखणयोदवाराउतसनजयतपदाियहहॴ 1-Feb-

2017

Options

1) Uric Acid

यररकएलसि

2) Ammonia

अमह८ननया 3) Carbohydrates

काबोहाइडरहॳट

4) Urea

यररया Correct Answer Carbohydrates

In animals the main excretory products

are carbon dioxide ammonia (in

ammoniotelics) urea (in ureotelics) uric

acid (in uricotelics) guanine (in

Arachnida) and creatine

Q198 RNA is a polymeric molecule

What does RNA stand for

आरएनइएएकबहिकआणहहॴ इसकाकापवय पकयाहहॴ 1-Feb-2017

Options

1) Rado Nuclear Acid

रािह८नयनकियरएलसि

2) Ribo Nucleic Acid

राइबह८नयनकिकएलसि

3) Rhino Nuclear Acid

हाइनह८नयनकियरएलसि

4) Resto Nucleus Acid

रहॳसटह८नयकिीयसएलसि

Correct Answer Ribo Nucleic Acid

Q199 Which organ does detoxification

and produces chemicals needed for

digestion

कह९नसाअगपवषहरणकरताहहॴऔरपाचनकहॳ लिएआवशयकरसायनोकह८पहॴदाकरताहहॴ 1-Feb-

2017

Options

1) Salivary glands

िारगरचिया 2) Pancreas

अगनयािय

F A C E B O O K

P A G E h t t p w w w f a c e b o o k c o m s s c m e n t o r s o f f i c i a l P a g e | 46

FOR MORE UPDATES AND MORE MATERIAL DO LIKE OUR FACEBOOK PAGE httpwwwfacebookcomsscmentorsofficial

3) Thyroid gland

िायराइिगरिी 4) Liver

यकत

Correct Answer Liver

Q200 Psidium guajava is the scientific

name of

लसडियमगआजावा mdashmdash कावहॴजञाननकनामहहॴ 1-

Feb-2017

Options

1) Guava

अम द

2) Mango

आम

3) Bamboo

बास

4) Jack fruit

कटहि

Correct Answer Guava

Q201 Which drug is used as a Blood

Thinner

चधरकह८पतिाकरनहॳकहॳ पमककसदवाकापरयह८गककयाजाताहहॴ

1-Feb-2017

Options

1) Warfarin

वाफर न

2) Tramadol

टरहॳमािह८ि

3) Azithromycin

एनजरह८मायलसन

4) Hydralazine

हाइडरह८िहॳनजन

Correct Answer Warfarin

Q202 Which of the following disease is

caused due to the deficiency of protein

परह८टीनकीकमीकहॳ कारणनननननलिखितमसहॳकह९नसारह८गहह८ताहहॴ 1-Feb-2017

Options

1) Arthritis

गटठया 2) Kwashiorkor

कािीओकय र

3) Goitre

गाइटर

4) Night Blindness

रतह९चध

Correct Answer Kwashiorkor

Q203 A is species of plant that has

adapted to survive in an environment

with little liquid water

mdashmdashndashपह९धहॳकीएकऐसहॳऐसहॳपरजानतहहॴ नजसनहॳकमपानीवािहॳवातावरणमजीपवतरहनहॳकहॳलिएअनकिनहहॴ 1-Feb-2017

Options

1) Xerophyte

म दपवद

2) Hydrophyte

जिीयपादप

3) Mesophyte

समह८दपवद

4) Thallophyte

िहॴिह८फाइटा Correct Answer xerophyte

xerophyte is a species of plant that has

adapted to survive in an environment

with little liquid water such as a desert

or an ice- or snow-covered region in the

Alps or the Arctic

Mesophytes are terrestrial plants which

are adapted to neither a particularly

dry nor particularly wet environment

An example of a mesophytic habitat

would be a rural temperate meadow

F A C E B O O K

P A G E h t t p w w w f a c e b o o k c o m s s c m e n t o r s o f f i c i a l P a g e | 47

FOR MORE UPDATES AND MORE MATERIAL DO LIKE OUR FACEBOOK PAGE httpwwwfacebookcomsscmentorsofficial

which might contain goldenrod clover

oxeye daisy and Rosa multiflora

thallophyte any of a group of plants or

plantlike organisms (such as algae and

fungi) that lack differentiated stems

leaves and roots and that were formerly

classified as a primary division

(Thallophyta) of the plant kingdom

Q204 How many types of teeth are

there in humans

मनषयोमककतनहॳपरकारकहॳ दातहह८तहॳहहॴ

1-Feb-2017

Options

1) 4

2) 5

3) 2

4) 3

Correct Answer 4

teeth -Humans have four types of

teethincisors canines premolars and

molars each with a specific function

The incisors cut the food the canines

tear the food and the molars and

premolars crush the food

Q205 Carica papaya is the scientific name of

कहॴ ररकापपाया mdashmdashndash कावहॴजञाननकनामहहॴ 2-

Feb-2017

Options

1) Peepal

पीपि

2) Papaya

पपीता 3) Tamarind

इमिी 4) Drumstick

ढह८िकाछड़ी Correct Answer Papaya

Q206 Muscles get tired when there is

shortfall of

जब mdashndash कीकमीहह८तीहहॴतबपहॳिीयिकजातीहहॴ 2-Feb-2017

Options

1) Lactic acid

िहॴनकटकएलसि

2) Na+ ions

Na+ आयन

3) ATP

एटीपी 4) Sulphates

समफहॳ टस

Correct Answer ATP

ATP is the energy source muscle fibers

use to make muscles contract

muscle tissuersquos main source of energy

called adenosine triphosphate or ATP

As your muscles use up this energy

source they become tired and fatigued

Oxygen is the key ingredient that helps

create new ATP to replenish the burned

up ATP in your muscles

Q207 Artocarpus integra is the

scientific name of आटह८कापयसइटीगरा mdashmdashmdash कावहॴजञाननकनामहहॴ 2-Feb-2017

Options

1) Guava

अम द

2) Pineapple

अनानास

3) Silver Oak

लसमवरओक

4) Jack fruit

कटहि

Correct Answer Jack fruit

Q208 Which organ stores fat soluble

vitamins

कह९नसाअगवसामघिनिीिपवटालमनह८काभिाराकरताहहॴ

2-Feb-2017

F A C E B O O K

P A G E h t t p w w w f a c e b o o k c o m s s c m e n t o r s o f f i c i a l P a g e | 48

FOR MORE UPDATES AND MORE MATERIAL DO LIKE OUR FACEBOOK PAGE httpwwwfacebookcomsscmentorsofficial

Options

1) Blood

रकत

2) Skin

तवचा 3) Liver

यकत

4) Pancreas

अगनयािय

Correct Answer Liver

Q209 Which disease is caused due to

deficiency of Iodine

आयह८िीनकहॳ कारणकह९नसारह८गहह८ताहहॴ 2-Feb-2017

Options

1) Rickets

ररकहॳ टस

2) Scurvy

सकवी 3) Goitre

गणमािा 4) Growth retardation

पवकासका कना Correct Answer Goitre

rickets A softening and weakening of

bones in children usually due to

inadequate vitamin D

Q210 Grevillea Robusta is the scientific name of

गरहॳपवलियारह८बसटा mdashmdashmdash- कापवजञाननकनामहहॴ 2-Feb-2017

Options

1) Peepal

पीपि

2) Teak

सागह९न

3) Silver Oak

लसमवरओक

4) Jack fruit

कटहि

Correct Answer Silver Oak

Q211 When a Cuttlefish is described as a Molluscs it is at which level of

classification

जबएककटिकफिकह८एकमह८िसकाकहॳ पमवखणयतककयाजाताहहॴतबयहॳवगीकरणकहॳ ककससतरपहॳनसितहहॴ 2-Feb-2017

Options

1) Class

वगय 2) Order

िम

3) Family

पररवार

4) Phylum

सघ

Correct Answer Phylum

Q212 Bambusa dendrocalmus is the

scientific name of बानबसािहॳडराकामस mdashmdashmdash कावहॴजञाननकनामहहॴ 3-Feb-2017

Options

1) Banyan

बरगद

2) Papaya

पपीता 3) Bamboo

बास

4) Pomegranate

अनार

Correct Answer Bamboo

Q213 Acinonyx Jubatus is the scientific name of

एलसनह८ननकसजयबहॳटस mdashmdashmdash

कावहॴजञाननकनामहहॴ 3-Feb-2017

F A C E B O O K

P A G E h t t p w w w f a c e b o o k c o m s s c m e n t o r s o f f i c i a l P a g e | 49

FOR MORE UPDATES AND MORE MATERIAL DO LIKE OUR FACEBOOK PAGE httpwwwfacebookcomsscmentorsofficial

Options

1) Bear

भाि 2) Horse

घह८िा 3) Cheetah

चीता 4) Zebra

जहॳिा Correct Answer Cheetah

Q214 The pale yellow colour of urine is

due to the presence of which pigment

मतरकाफीकापीिारगरगदरयकहॳ उपनसिनतकहॳ कारणहह८ताहहॴ

3-Feb-2017

Options

1) Urochrome

यरह८िह८म

2) Urophyll

यरह८कफि

3) Chlorophyll

किह८रह८कफि

4) Chloroplast

किह८रह८पिासट

Correct Answer Urochrome

Q215 Which of the following constitute

to form a gene

नननननलिखितमसहॳकह९नसीचीज़एकजीनकागठनकरतीहहॴ

3-Feb-2017

Options

1) Polynucleotides

पह८िीनयनकियह८टाईडस

2) Hydrocarbons

हाइडरह८काबोस

3) Lipoproteins

िाईपह८परह८टीनस

4) Lipids

लिपपडस

Correct Answer Polynucleotides

Polynucleotide molecule is a biopolymer

composed of 13 or more nucleotide

monomers covalently bonded in a chain

DNA (deoxyribonucleic acid) and RNA

(ribonucleic acid) are examples of

polynucleotides with distinct biological

function

Q216 Vertebrates belongs to the

phylum

रीढ़कीहडिीवािहॳपराणी mdashmdashmdash

परजानतकहॳ अतगायतआतहॳहहॴ 3-Feb-2017

Options

1) Arthropoda

आरह८पह८ड़ा 2) Annelida

एननलििा 3) Cnidaria

ननिहॳररया 4) Chordata

कह८िटा Correct Answer Chordata

Q217 Punica granatum is the scientific name of

पननकगरहॳनहॳटस mdashmdashmdash कावहॴजञाननकनामहहॴ 3-Feb-2017

Options

1) Custard Apple

सीताफि

2) Gulmohar

गिमह८हर

3) Silver Oak

लसमवरओक

4) Pomegranate

अनार

Correct Answer Pomegranate

F A C E B O O K

P A G E h t t p w w w f a c e b o o k c o m s s c m e n t o r s o f f i c i a l P a g e | 50

FOR MORE UPDATES AND MORE MATERIAL DO LIKE OUR FACEBOOK PAGE httpwwwfacebookcomsscmentorsofficial

Q218 Between a tiger and an monkey

which of the following is different

एकबाघऔरबदरकहॳ बीचनननननलिखितमसहॳकह९नसीबातअिगहहॴ 3-Feb-2017

Options

1) Kingdom

राजय

2) Phylum

जानत

3) Order

िम

4) Class

वगय Correct Answer order

Q219 The artificial heart was invented by

कबतरमहदयका mdashmdashmdash

दवाराअपवषकारककयागयािा 3-Feb-2017

Options

1) Muhammad Yunus

महनमदयनस

2) Linus Yale Jr

िाइनसयहॳिजय

3) Gazi Yasargil

गाजीयासचगयि

4) Paul Winchell

पह९िपवमकि Correct Answer Paul Winchell

Q220 Tamarindus indica is the

scientific name of

टहॳमररनडसइडिका mdashmdash कावहॴजञाननकनामहहॴ 7-

Feb-2017

Options

1) Neem

नीम

2) Pineapple

अनानास

3) Tamarind

इमिी 4)Chiku

चीक

Correct Answer Tamarind

Q221 In eukaryotic cells synthesis of

RNA takes place in the

यकहॳ योटटककह८लिकाओमआरएनएकासशिहॳषण

mdashndash महह८ताहहॴ 7-Feb-2017

Options

1) Mitochondria

माईटह८कोडडरया 2) Centrioles

सटरीयह८मस

3) Ribosomes

ररबह८सह८नस

4) Nucleus

नयनकियस

Correct Answer nucleus

eukaryotic cell -Transcription is the

process of synthesizing ribonucleic acid

(RNA)Synthesis takes place within the

nucleus of eukaryotic cells or in the

cytoplasm of prokaryotes and converts

the genetic code from a gene in

deoxyribonucleic acid ( DNA ) to a

strand of RNA that then directs

proteinsynthesis

Q222 _________is caused by parasites

of the Plasmodium genus

पिाजमह८डियमजातीकहॳ परजीवी mdash- कहॳ कारणहहॴ 7-Feb-2017

Options

1) Dysentery

पहॳचचि

2) Malaria

मिहॳररया 3) Chickenpox

F A C E B O O K

P A G E h t t p w w w f a c e b o o k c o m s s c m e n t o r s o f f i c i a l P a g e | 51

FOR MORE UPDATES AND MORE MATERIAL DO LIKE OUR FACEBOOK PAGE httpwwwfacebookcomsscmentorsofficial

चहॳचक

4) Herpes

हहॳपपयस

Correct Answer Malaria

Q223 Carotene in fruits and vegetables

gives it which color

फिह८औरसनलजयोमनसितकहॳ रह८टीनउनहकह९नसारगपरदानकरताहहॴ 7-Feb-2017

Options

1) Green

हरा 2) Pink

गिाबी 3) Orange

नारगी 4) Blue

नीिा Correct Answer Orange

Q224 Equus Caballus is the scientific

name of

एकवसकहॴ बहॳिस mdashmdashndash कापवजञाननकनामहहॴ 7-Feb-2017

Options

1) Horse

घह८िा 2) Zebra

ज़हॳिा 3) Donkey

गधा 4) Buffalo

भस

Correct Answer Horse

Q225 Elapidae Naja is the scientific name of

एिीपीिीनाजा mdashmdash- कावहॴजञाननकनामहहॴ 8-Feb-2017

Options

1) Cobra

कह८बरा 2) Elephant

हािी 3) Eagle

ग ि

4) Owl

उमि Correct Answer Cobra

Q226 Which disease is caused due to

deficiency of Iron

िह८हकीकमीकहॳ कारणकह९नसारह८गहह८ताहहॴ 8-Feb-

2017

Options

1) Beriberi

बहॳरीबहॳरी 2) Tetany

टहॳटनी 3) Kwashiorkor

कवािीऔरकर

4) Anaemia

रकतामपता Correct Answer Anaemia

Beriberi is a disease caused by a vitamin

B-1 deficiency also known as thiamine

deficiency

Tetany can be the result of an

electrolyte imbalance Most often itrsquos a

dramatically low calcium level also

known as hypocalcemia Tetany can also

be caused by magnesium deficiency or

too little potassium Having too much

acid (acidosis) or too much alkali

(alkalosis) in the body can also result in

tetany

Kwashiorkor also known as

ldquoedematous malnutrition It is a form of

malnutrition caused by a lack of protein

in the diet

Anaemia means that you have fewer red

blood cells than normal or you have less

F A C E B O O K

P A G E h t t p w w w f a c e b o o k c o m s s c m e n t o r s o f f i c i a l P a g e | 52

FOR MORE UPDATES AND MORE MATERIAL DO LIKE OUR FACEBOOK PAGE httpwwwfacebookcomsscmentorsofficial

haemoglobin than normal in each red

blood cell

Q227 is a leaf where the leaflets are

arranged along the middle vein

mdashndashएकपततीहहॴजहापतरकह८कीरचनाक ररयालिराकहॳ आसपासहह८तीहहॴ 8-Feb-2017

Options

1) Pinnately compound leaf

पपनहॳटिीसयकतपतती 2) Palmately compound leaf

पामहॳटिीसयकतपतती 3) Compound leaf

सयकतपतती 4) Simple leaf

साधारणपतती Correct Answer Pinnately compound

leaf

Q228 Haustoria or sucking roots are

found in which of the following

हह८सटह८ररयायाचसनहॳवािीजड़हॳनननननलिखितमसहॳककसमपाईजातीहहॴ 8-Feb-2017

Options

1) Wheat

गहॳह

2) Mango

आम

3) Chestnut

चहॳसटनट

4) Cuscuta

कसकयटा Correct Answer Cuscuta

Haustorial roots -The roots of parasitic

plants which penetrate into the host

tissues to absorb nourishment are

called haustorial roots hellip Also known as suckingor parasitic roots

Q229 Equs Asinus is the scientific name

of

एकवसएलसनस mdashmdashndash कावहॴजञाननकनामहहॴ 8-

Feb-2017

Options

1) Donkey

गधा 2) Cow

गाय

3) Deer

टहरन

4) Kangaroo

कगा

Correct Answer Donkey

Q230 Ficus benghalensis is the scientific name of

फाईकसबहॳनगहॳिहॳलसस mdashndash कापवजञाननकनामहहॴ 8-Feb-2017

Options

1) Banyan

बरगद

2) Pineapple

अनानास

3) Babul

बबि

4) Tulsi

तिसी Correct Answer Banyan

Q231 Equus burchellii is the scientific name of

एकवसबचिी mdashmdash- कापवजञाननकनामहहॴ 8-Feb-2017

Options

1) Horse

घह८िा 2) Zebra

जहॳिा 3) Buffalo

F A C E B O O K

P A G E h t t p w w w f a c e b o o k c o m s s c m e n t o r s o f f i c i a l P a g e | 53

FOR MORE UPDATES AND MORE MATERIAL DO LIKE OUR FACEBOOK PAGE httpwwwfacebookcomsscmentorsofficial

भस

4) Ass

गधा Correct Answer Zebra

Page 4: COMPILATION OF ALL 72 SETS OF BIOLOGY SSC CHSL-2016 · OF BIOLOGY SSC CHSL-2016 PREPARED BY : SSC MENTORS BIOLOGY SPECIAL . F A C E B O O K P A G E : h t t p : / / w w w . f a c e

F A C E B O O K

P A G E h t t p w w w f a c e b o o k c o m s s c m e n t o r s o f f i c i a l P a g e | 3

FOR MORE UPDATES AND MORE MATERIAL DO LIKE OUR FACEBOOK PAGE httpwwwfacebookcomsscmentorsofficial

Q5 Which is the largest organ in

human beings

मानवह८मसबसहॳबड़ाअगकह९नसाहह८ताहहॴ

7-Jan -2017

Options

1) Skin

तवचा 2) Large Intestine

बड़ीआत

3) Small Intestine

छह८टीआत

4) Liver

यकत

Correct Answer Skin

The skin is the largest organ of the

body with a total area of about 20

square feetThe largest internal organ is

the liver The longest bone in the human

body is the femur The largest artery is

the aorta and the largest vein is the

inferior vena cava

Q6 Delonix regia Rafin is the scientific

name of

िहॴिोननकसरनजयारकफन (Delonix regia

Rafin)mdashmdash- कावहॴजञाननकनाम 7-Jan -2017

Options

1) Banyan

बरगद

2) Gulmohar

गिमह८हर

3) Tamarind

इमिी 4) Chiku

चचक

Correct Answer Gulmohar

Delonix regia is a species of flowering

plant in the bean family Fabaceae

Q7 Amoeba belongs to the phylum

अमीबा mdashmdashmdash- परजानतकहॳ अतगयतआताहहॴ

7-Jan -2017

Options

1) Protozoa

परह८टह८जआ

2) Annelida

ऐनहॳलििा 3)Porifera

पह८ररफहॳ रा 4) Platyhelminthes

पिहॳटटहहॳनममननिस

Correct Answer Protozoa

Q8 Deficiency of which of the following

causes non-clotting of blood

नननननलिखितमसहॳककसकीकमीकहॳ कारणरकतकािककानहीजमता

8-Jan -2017

Options

1) Vitamin C

पवटालमन C

2) Vitamin K

पवटालमन K

3) Vitamin E

पवटालमन E

4) Vitamin B12

पवटालमन B12

Correct Answer Vitamin K

Vitamin C is found in citrus fruits and

vegetables Scurvy results from a

deficiency of vitamin C in the diet

Vitamin E deficiency associated with

this disease causes problems such as

poor transmission of nerve impulses

muscle weakness and degeneration of

the retina that can cause blindness

Vitamin B12 deficiency may lead to a

reduction in healthy red blood cells

Q9 The process of producing energy in

plants is known as

F A C E B O O K

P A G E h t t p w w w f a c e b o o k c o m s s c m e n t o r s o f f i c i a l P a g e | 4

FOR MORE UPDATES AND MORE MATERIAL DO LIKE OUR FACEBOOK PAGE httpwwwfacebookcomsscmentorsofficial

पहॳड़ह८मउजायउतपननकरनहॳकीपरनतकियाकह८ mdashmdashndash

कहॳ नामजाताहहॴ 8-Jan -2017

Options

1) Absorption

अविह८षण

2) Reduction

अवकरण

3) Photosynthesis

परकािसशरिहॳषण

4) Transpiration

वाषपीकरण

Correct Answer Photosynthesis

Q10 Which Virus causes Chicken Pox

ककसवायरसकहॳ कारणचहॳचकहह८ताहहॴ

8-Jan -2017

Options

1) Rubella Virus

बहॳिावायरस

2) Herpes Zoster Virus

हपपरयसजह८सटरवायरस

3) Rabies

रहॳबीज़

4) Variola Virus

वहॳरीओिावायरस

Correct Answer Herpes Zoster Virus

Rubella also known as German

measles or

three ndashday measles is an infection

caused by the rubella virus

Smallpox is caused by infection with

variola

Virus

Q11 What is the total number of bones

in the human body

मानविरीरमकिककतनीहडडियहह८तीहहॴ

8-Jan -2017

Options

1)206

2)103

3)309

4)412

Correct Answer 206

Q12 Emblica officinalis is the scientific

name of

एननलिकाओफीलसनहॳलिस (Emblica

offcinalis)mdashmdash- कावहॴजञाननकनामहहॴ 8-Jan -2017

Options

1) Peepal

पीपि

2) Mango

आम

3) Amla

आविा 4) Drumstick

सहजन

Correct Answer Amla

Q13 Sponges belongs to the phylum

सपजककसपरजानतकहॳ अतगयतआतहहॴ

8-Jan -2017

Options

1) Protozoa

परह८टह८जआ

2) Annelida

एननलििा 3) Porifera

पह८ररफहॳ रा 4) Cnidaria

ननिहॳररया Correct Answer Porifera

Q14 Which of the following is a

symptom of haemophilia

नननननलिखितमसहॳटहमह८फीलियाकािकषणकह९नसा हहॴ

9-Jan -2017

Options

F A C E B O O K

P A G E h t t p w w w f a c e b o o k c o m s s c m e n t o r s o f f i c i a l P a g e | 5

FOR MORE UPDATES AND MORE MATERIAL DO LIKE OUR FACEBOOK PAGE httpwwwfacebookcomsscmentorsofficial

1) Night Blindness

रतोधी 2) No clotting of Blood

रकतकािककानजमना 3) Rickets

ररकहॳ ट

4) Loss of haemoglobin

टहमह८गिह८बबनकीअमपता Correct Answer No clotting of Blood

Haemophilia also spelled hemophilia is

a mostly inherited genetic disorder that

impairs the bodyrsquos ability to make blood

clots a process needed to stop bleeding

This results in people bleeding longer

after an injury easy bruising and an

increased risk of bleeding inside joints

or the brain

Q15 The process of pollination by birds

is also known as

पकषकषयोदवाराकीजानहॳवािीपरागणकीपरनतकियाकह८mdashndash कहॳ नामसहॳभीजानाजाताहहॴ 9-Jan -2017

Options

1) Hydrophily

हाइडरह८कफिी 2) Entomophily

एनटोमह८कफिी 3) Embryophily

एननियह८कफिी 4) Ornithophily

ओननयिह८कफिी Correct Answer Ornithophily

Q16 Spiders belong to the phylum

मकडड़याककसपरजानतकहॳ अतगयतआतीहहॴ 9-Jan -2017

Options

1) Mollusca

मह८िसका 2) Annelida

एननलििा 3) Cnidaria

ननिहॳररया 4) Arthropoda

अरोपह८ड़ा Correct Answer Arthropoda

Q17 Banana freckle is a plant disease

It is caused by a

कहॳ िहॳकीझाईपह८धह८कीएकबीमारीहहॴ यहएक mdashndash

कहॳ कारणहह८तीहहॴ 9-Jan -2017

Options

1) Virus

वायरस

2) Fungus

कवक

3) Bacteria

बहॴकटीररया 4) Insect

कीटक

Correct Answer Fungus

Banana Freckle is a disease caused by

the fungus Guignardia musae

(telomorph) or Phyllosticta musarum (

anamorph )

Q18 Which of the following Indian

chilly is considered one of the hottest in

the world

नननननलिखितभारतीयलमचचययह८मसहॳकह९नसीसबसहॳकह९नसीपवशवकीसबसहॳतीिीलमचचययह८मसहॳएकमानीजातीहहॴ

9-Jan -2017

Options

1) Bhut Jolokia

भतझह८िककया 2) Bhut Mahabora

भतमहाबह८रा 3) Lal Chitin

F A C E B O O K

P A G E h t t p w w w f a c e b o o k c o m s s c m e n t o r s o f f i c i a l P a g e | 6

FOR MORE UPDATES AND MORE MATERIAL DO LIKE OUR FACEBOOK PAGE httpwwwfacebookcomsscmentorsofficial

िािचीटटन

4) Lal Shamak

िाििामक

Correct Answer Bhut Jolokia

Q19 Brain fever is a disease spread

through which of the following

मनसतषकजवरनमकरह८गनननननलिखितमसहॳककसकहॳ कारणहह८ताहहॴ 9-Jan -2017

Options

1) Flies

मनकियो 2) Mosquito

मचछर

3) Virus

वायरस

4) Cockroach

नतिच हॳ Correct Answer Mosquito

Q20 Mangroves are plants that have

मगरह८ववहॳपहॳिहहॴनजनमहॳहह८ताहहॴ 9-Jan -2017

Options

1) Modified Roots

पातररतजड़हॳ 2) Modified Stems

पातररततनहॳ 3) Respiratory Roots

शरवसनकरनहॳवािीजड़हॳ 4) Respiratory Stems

शरवसनकरनहॳवािीतनहॳ Correct Answer Respiratory Roots

A mangrove is a shrub or small tree that

grows in coastal saline or brackish

water

Q21 Rodentia Sciurus is the scientific

name of

रह८िहॳलियासकीयरस mdashmdash कावयजजञाननकनामहहॴ

9-Jan -2017

Options

1) Rat

चहा 2) Platypus

पिहॳटीपस

3) Squirrel

चगिहरी 4) Beaver

बीवर

Correct Answer Squirrel

Q22 Which of the following is induced

by Oncogene

नननननलिखितमसहॳकह९नओकह८जीनदवारापरहॳररयतहह८ताहहॴ

10-Jan -2017

Options

1) Polio

पह८लियह८ 2) Cancer

क सर

3) Diarrhoea

दसत

4) Dengue

िग Correct Answer Cancer

An oncogene is a gene that has the

potential to cause cancer In tumor

cells they are often mutated andor

expressed at high levels

Q23 Azadirachata indica is the

scientific name of

अजाटदराकटाइडिका mdashmdashवहॴजञाननकनामहहॴ SSC CHSL Science (biology) 2016

Question Paper

10-Jan -2017

Options

1) Neem

नीम

F A C E B O O K

P A G E h t t p w w w f a c e b o o k c o m s s c m e n t o r s o f f i c i a l P a g e | 7

FOR MORE UPDATES AND MORE MATERIAL DO LIKE OUR FACEBOOK PAGE httpwwwfacebookcomsscmentorsofficial

2) Teak

सागह९न

3) Silver Oak

लसमवरओक

4) Tulsi

तिसी Correct Answer Neem

Q24 Octopus belongs to the phylum

ऑकटह८पसककसपरजानतकहॳ अतगयतआताहहॴ 10-

Jan -2017

Options

1) Mollusca

मह८िसका 2) Cnidaria

ननिहॳररया 3) Echinodermata

इकाइनह८ड़हॳमता 4) Chordata

कह८िता Correct Answer Mollusca

Q25 A living part of the organisms

environment is known as

जीवाणकहॳ वातावरणकहॳ जीपवतभागकह८ mdash-

कहतहॳहहॴ 10-Jan -2017

Options

1) Abiotic Factor

अजहॴपवककारक

2) Habitat

आवास

3) Biotic Factor

जहॴपवककारक

4) Nonliving factor

अ-जीपवतकारक

Correct Answer Biotic Factor

Abiotic factors are nonndash living chemical

and physical parts of the environment

that affect living organisms and the

functioning of ecosystems like rain

wind temperature altitude soil

pollution nutrients pH types of soil

and sunlight

Q26 Medulla oblongata is a part of

which of the following

महॳडयिाऑबिॉनगहॳटानननननलिखितमसहॳककसअगकाटहससाहहॴ

10-Jan -2017

Options

1) Heart

हदय

2) Brain

मनसतषक

3) Lungs

फहॳ फड़हॳ 4) Stomach

पहॳट

Correct Answer Brain

The medulla oblongata helps regulate

breathing heart and blood vessel

function digestion sneezing and

swallowing This part of the brain is a

center for respiration and circulation

Sensory and motor neurons (nerve cells)

from the forebrain and midbrain travel

through the medulla

Q27 ___________ is a typically

onecelled reproductive unit capable of

giving rise to a new individual without

sexual fusion

mdashmdash एकआमतह९रपरएककह८लिकीयहॳ परजननममसमकषइकाईहहॴजह८यह९नसियनकहॳ बबनाएकनयीइकाईकह८जनमदहॳतीहहॴ 10-Jan -2017

Options

1) Egg

अिाण

2) Spore

बीजाण

F A C E B O O K

P A G E h t t p w w w f a c e b o o k c o m s s c m e n t o r s o f f i c i a l P a g e | 8

FOR MORE UPDATES AND MORE MATERIAL DO LIKE OUR FACEBOOK PAGE httpwwwfacebookcomsscmentorsofficial

3) Sperm

ििाण

4) Seed

बीज

Correct Answer Spore

Q28 Bacteria was discovered by

बहॴकटीररयाकीिह८जककसकहॳ दवाराकीगयीिी

10-Jan -2017

1) Antonie van Leeuwenhoek

एटह८नीवहॳनलिबहॳनहक

2) Belarus

बहॳिा स

3) Hugo de Vries

हयगह८दीराईस

4)Robert Brown

रॉबटयिाउन

Correct Answer Antonie van

Leeuwenhoek

Q29 Which of the following is

responsible for Vermicomposting

नननननलिखितमसहॳकह९नकलमिादकहॳ लिएनजनमहॳदारहहॴ

10-Jan -2017

Options

1) Fungus

कवक

2) Worms

कलम

3) Bacteria

बहॴकटीररया 4) Birds

पकषी Correct Answer Worms

Vermicompost (or vermi-compost) is the

product of the composting process using

various species of worms usually red

wigglers white worms and other

earthworms to create a heterogeneous

mixture of decomposing vegetable or

food waste bedding materials and

vermicast

Q30 Scurvy (bleeding of gums) is

caused by the deficiency of which

vitamin

सकवी (मसढह८सहॳिनआना) ककसपवटालमनकीकमीकहॳ कारणहह८ताहहॴ

10-Jan-2017

Options

1) Vitamin K

पवटालमन K

2) Vitamin BZ

पवटालमन BZ

3) Vitamin C

पवटालमन C

4) Vitamin A

पवटालमन A

Correct Answer Vitamin C

Q31 Achras sapota is the scientific

name of

एिाससपह८ताइसकावहॴजञाननकनामहहॴ 10-Jan-2017

Options

1) Custard Apple

सीताफि

2) Gulmohar

गिमह८हर

3) Tamarind

इमिी 4) Chiku

चचक

Correct Answer Chiku

Q32 Prawn belongs to the phylum

झीगा mdashmdash- परजानतकहॳ अतगयतआताहहॴ 10-Jan-2017

Options

1) Arthropoda

F A C E B O O K

P A G E h t t p w w w f a c e b o o k c o m s s c m e n t o r s o f f i c i a l P a g e | 9

FOR MORE UPDATES AND MORE MATERIAL DO LIKE OUR FACEBOOK PAGE httpwwwfacebookcomsscmentorsofficial

अरोपह८िा 2) Cnidaria

नीिहॳररया 3) Echinodermata

इकाईनह८िमटा 4) Chordata

कह८िटा Correct Answer Arthropoda

Q33 Pulses are a rich source of which of

the following

दािहॳनननननलिखितमसहॳककसकीपरचरसह८तरहहॴ

11-Jan-2017

Options

1) Carbohydrates

काबोहाइडराईट

2) Proteins

परह८टीनस

3) Minerals

िननज

4) Vitamin A

पवटालमन A

Correct Answer Proteins

Q34 Plant cell wall is made up of

वनसपनतकह८लिकालभनततइससहॳबनीहह८तीहहॴ

11-Jan-2017

Options

1) Cellulose

सहॳमयिह८ज

2) Glucose

गिकह८ज

3) Sucrose

सिह८ज

4) Fructose

फरकटह८ज

Correct Answer Cellulose

Plant cell wall the major carbohydrates

are cellulose hemicellulose and pectin

The cellulose microfibrils are linked via

hemicellulosic tethers to form the

cellulose-hemicellulose network which

is embedded in the pectin matrix

Q35 The study of Fungi is also known

as कवकह८कहॳ अधययनकह८कहाजाताहहॴ

11-Jan-2017

Options

1) Cytology

सायटह८िह८जी 2) Myology

मायह८िह८जी 3) Mycology

मायकह८िह८जी 4) Neurology

नयरह८िह८जी Correct Answer Mycology

Cytology - structure and function of

plant and animal cells

Myology is the study of the muscular

system

Neurology is the branch of medicine

concerned with the study and treatment

of disorders of the nervous system

Q36 The outermost layer of skin is

तवचाकीसबसहॳबाहरीपरतकयाहह८तीहहॴ 11-Jan-

2017

Options

1) Epidermis

इपपिलमयस

2) Dermis

िलमयस

3) Tissues

ऊतक

4) Hypodermis

हायपह८िलमयस

Correct Answer Epidermis

Q37 Which of the following plants have

root nodules

F A C E B O O K

P A G E h t t p w w w f a c e b o o k c o m s s c m e n t o r s o f f i c i a l P a g e | 10

FOR MORE UPDATES AND MORE MATERIAL DO LIKE OUR FACEBOOK PAGE httpwwwfacebookcomsscmentorsofficial

नननननलिखितपह९धह८मसहॳककसकीजड़ह८मगाठहह८तीहहॴ

11-Jan-2017

Options

1) Leguminous plants

िहॳगयलमनसपह९धहॳ 2) Parasitic plants

परजीवीपह९धहॳ 3) Epiphytic Plants

एपीफाइटटकपह९धहॳ 4) Aquatic Plants

जिीयपह९धहॳ Correct Answer Leguminous plants

Q38 Earth-worms belongs to the

phylum

कहॳ चएmdashmdash- परजानतकहॳ अतगयतआतहॳहहॴ 11-Jan-2017

Options

1) Protozoa

परह८टह८जआ

2) Cnidaria

नीिहॳररया 3) Annelida

एनीलििा 4) Mollusca

मह८िसका Correct Answer Annelida

Q39 Ringworm is a disease caused by

ररगवमयनामकबीमारी mdashmdash- कहॳ कारणहह८तीहहॴ 11-Jan-2017

Options

1) Fungi

कवक

2) Bacteria

बहॴकटीररया 3) Virus

वायरस

4) Flies

मनकियाा Correct Answer Fungi

Q40 Mangifera indica is the scientific

name of

मननगफहॳ राइडिकाककसकावहॴजञाननकनामहहॴ 11-

Jan-2017

Options

1) Guava

अम द

2) Mango

आम

3) Amla

आविा 4) Jack fruit

कटहि

Correct Answer Mango

Q41 Crabs belongs to the phylum

कहॳ कड़हॳmdashmdash- परजानतकहॳ अतगयतआतहॳहहॴ 11-Jan-2017

Options

1) Mollusca

मह८िसका 2) Cnidaria

नीिहॳररया 3) Arthropoda

अरोपह८ड़ा 4) Platyhelminthes

पिहॳटटहहॳनममननिस

Correct Answer Arthropoda

Q42 Myopia is a defect of eyes which is

also known as

मायह८पपयाआिोकादह८षहहॴ नजसहॳ mdashmdashndash

भीकहाजाताहहॴ

12-Jan-2017

Options

1) Far Sightedness

F A C E B O O K

P A G E h t t p w w w f a c e b o o k c o m s s c m e n t o r s o f f i c i a l P a g e | 11

FOR MORE UPDATES AND MORE MATERIAL DO LIKE OUR FACEBOOK PAGE httpwwwfacebookcomsscmentorsofficial

दरदनषटदह८ष

2) Near Sightedness

ननकटदनषटदह८ष

3) Astigmatism

एसटीगमहॳटटजम

4) Night Blindness

रतोधी Correct Answer Near Sightedness

Myopia occurs when the eyeball is too

long relative to the focusing power of

the cornea and lens of the eye This

causes light rays to focus at a point in

front of the retina rather than directly

on its surface

Hyperopia Hypermetropia (

Farsightedness )- when light rays

entering the eye focus behind the retina

rather than directly on it The eyeball of

a farsighted person is shorter than

normal

Astigmatism usually is caused by an

irregularly shaped cornea Instead of

the cornea having a symmetrically

round shape (like a baseball) it is

shaped more like an American football

Nyctalopia also called night ndash blindness

is a condition making it difficult or

impossible to see in relatively low light

Q43 Who is known as the father of

Green Revolution

हररतिानतकहॳ जनककहॳ पमककसहॳजानाजाताहहॴ

12-Jan-2017

1) Dr Robert Nucleus

िॉ रॉबटयनयनकियस

2) Dr Ian Wilmut

िॉ इयानपविमट

3) Dr NE Borlaug

िॉ एनईबह८रिॉग

4) Dr JC Bose

िॉ जहॳसीबह८स

Correct Answer Dr NE Borlaug

Q44 Panthera Tigris is the scientific

name of

पिहॳराटटगरीस mdashmdashmdash कावहॴजञाननकनामहहॴ 12-Jan-2017

Options

1) Panther

तदआ

2) Tiger

बाघ

3) Whale

हहॳि

4)Goat

बकरी Correct Answer Tiger

Q45 How many facial bones are there

हमारहॳचहॳहरहॳमककतनीहडडियााहह८तीहहॴ 13-Jan-2017

Options

1)34

2)24

3)14

4)4

Correct Answer 14

Q46 ndash Halophytes are plants that grow

in

हहॴिह८फाईटसवहॳपह९धहॳहह८तीहहॴजह८ mdash- मउगतहॳहहॴ SSC CHSL Science (biology) 2016

Question Paper

13-Jan-2017

Options

1) Fresh Water

ताजापानी 2) Cold Water

ठिापानी 3) Ponds

तािाब

4) Salt Water

िारापानी Correct Answer Salt Water

F A C E B O O K

P A G E h t t p w w w f a c e b o o k c o m s s c m e n t o r s o f f i c i a l P a g e | 12

FOR MORE UPDATES AND MORE MATERIAL DO LIKE OUR FACEBOOK PAGE httpwwwfacebookcomsscmentorsofficial

Q47 Felis Catus is the scientific name of

फहॳ लिसकहॴ टस mdashndash कावहॴजञाननकनामहहॴ 13-Jan-2017

Options

1) Cat

बबमिी 2) Dog

कतता 3) Mouse

चहा 4) Porcupine

साही Correct Answer Cat

Q48 Which of the following induces

nitrogen fixation in soil

नननननलिखितमसहॳकह९नलम ीमनाइटरह८जनननयतनकह८परहॳररतकरताहहॴ

15-Jan-2017

Options

1) Protozoa

परह८टह८जआ

2) Bacteria

बहॴकटीररया 3) Fungi

कवक

4)Algae

िहॴवाि

Correct Answer Bacteria

Bacteria that change nitrogen gas from

the atmosphere into solid nitrogen

usable by plants are called nitrogen-

fixing bacteria These bacteria are

found both in the soil and in symbiotic

relationships with plants

They contain symbiotic bacteria called

rhizobia within nodules in their root

systems producing nitrogen compounds

that help the plant to grow and compete

with other plants When the plant dies

the fixed nitrogen is released making it

available to other plant

Q49 Which of the following is the

largest known cell

नननननलिखितमसहॳकह९नसीसबसहॳबड़ीजञातकह८लिकाहहॴ

SSC CHSL Science (biology) 2016

Question Paper

15-Jan-2017

1) Eukaryotic Cell

यकहॳ ररयह८टटककह८लिका 2) Prokaryotic Cell

परह८कहॳ ररयह८टटककह८लिका 3) Mycoplasma

मायकह८पिासम

4) Ostrich Eggs

ितरमगयकाअिा Correct Answer Ostrich Eggs

Q50 The association of animals in

which both the partners are benefitted

is known as

जानवरोकावहसहयह८गनजसमहॳदह८नोभागीदारिाभापवनतहह८तहॳहहॴ उसहॳ mdashmdashndash कहॳ पमजानाजाताहहॴ SSC CHSL Science (biology) 2016

Question Paper

15-Jan-2017

Options

1) Amensalism

सहजीपवत

2) Commensalism

परजीपवत

3) Colony

कॉिनी 4) Mutualism

अनयह८नयाशरयवाद

Correct Answer Mutualism

Amensalism is any relationship between

organisms of different species in which

F A C E B O O K

P A G E h t t p w w w f a c e b o o k c o m s s c m e n t o r s o f f i c i a l P a g e | 13

FOR MORE UPDATES AND MORE MATERIAL DO LIKE OUR FACEBOOK PAGE httpwwwfacebookcomsscmentorsofficial

one organism is inhibited or destroyed

while the other organism remains

unaffected

Commensalism an association between

two organisms in which one benefits and

the other derives neither benefit nor

harm

Q51 Pneumonia affects which of the

following organs of human body

ननमह८ननयामानविरीरकहॳ नननननलिखितमसहॳककसअगकह८परभापवतकरताहहॴ

15-Jan-2017

Options

1)Kidneys

गद

2)Lungs

फहॳ फड़हॳ 3) Throat

गिहॳ 4) Liver

यकत

Correct Answer Lungs

When the germs that cause pneumonia

reach your lungs the lungsrsquo air sacs

(alveoli) become inflamed and fill up

with fluid This causes the symptoms of

pneumonia such as a cough fever

chills and trouble breathing When you

have pneumonia oxygen may have

trouble reaching your blood

Q52 Mendel is known as

मििकह८ mdashmdash- कहॳ पमजानाजाताहहॴ 15-Jan-2017

Options

1) Father of Physiology

िरीरकियािासतरकहॳ जनक

2) Father of Geology

भगभयिासतरकहॳ जनक

3) Father of Genetics

जहॳनहॳटटकसकहॳ जनक

4) Father of Biology

जीविासतरकहॳ जनक

Correct Answer Father of Genetics

Q53 Which of the following are also

known as Suicidal bag of Cells

ननननलिखितमसहॳककसहॳआतमहतयाकरनहॳवािीकह८लिकाओकाबहॴगकहाजाताहहॴ

15-Jan-2017

Options

1) Lysosomes

िायसोसह८म

2) Lycosome

िायकह८सह८म

3) Nucleus

नालभक

4) Chromosome

िह८मह८सह८म

Correct Answer Lysosomes

Q54 Mesothelioma is a type of cancer

The most common area affected in it is

the lining of the ________

लमज़ह८िहॳिहॳलमयाक सरकाएकपरकारहहॴ इससहॳपरभापवतहह८नहॳवािासबसहॳसामानयकषहॳतर mdash

mdashmdash काअसतरहहॴ 15-Jan-2017

Options

1)Heart

हदय

2)Brain

मनसतषक

3)Stomach

आमािय

4)Lungs

फहॳ फड़हॳ Correct Answer lungs

Asbestos exposure is the main cause of

pleural mesothelioma When asbestos

fibers are breathed in they travel to the

F A C E B O O K

P A G E h t t p w w w f a c e b o o k c o m s s c m e n t o r s o f f i c i a l P a g e | 14

FOR MORE UPDATES AND MORE MATERIAL DO LIKE OUR FACEBOOK PAGE httpwwwfacebookcomsscmentorsofficial

ends of small air passages and reach the

pleura where they can cause

inflammation and

scarring

Q55 Which one of the following is an

insectivorous plant

नननननलिखितमसहॳकह९नसाएकककटाहरीवनसपनतहहॴ

15-Jan-2017

Options

1) Utricularia

यटरीकिहॳररया 2) Sequoia

सहॳकयओइया 3) Nostoc

नॉसटह८क

4) Bryophyta

िायह८फाईटा Correct Answer Utricularia

Q56 ______________ is a

multibranched polysaccharide of

glucose that serves as a form of energy

storage in animals and fungi

mdashmdashगिकह८जकाएकबहिािायकतपह८िीसहॳकहॳ राइिहहॴ जह८जानवरोऔरकवकमउजायभणिारणकहॳ एक पमकाययकरताहहॴ 15-Jan-2017

Options

1) Cellulose

सहॳमयिह८ज

2) Glycogen

गिायकह८जन

3) Pectin

पहॳनकटन

4) Chitin

चीटटन

Correct Answer Glycogen

Q57 The largest gland of the human

body is

mdashmdashmdashमानविरीरकीसबसहॳबड़ीगरिीहहॴ 16-Jan-2017

Options

1) Pancreas

अगयािय

2) Thyroid

िायरॉइि

3) Large Intestine

बड़ीआत

4) Liver

यकत

Correct Answer Liver

Q58 Photosynthesis in plants takes

place in

वनसपनतयोमपरकािसशिहॳषणकीकियाहह८तीहहॴ

16-Jan-2017

Options

1) Stem

तना 2) Leaves

पनततयाा 3) Roots

जड़हॳ 4) Flower

फि

Correct Answer Leaves

During this reaction carbon dioxide

and water are converted into glucose

and oxygen The reaction requires light

energy which is absorbed by a green

substance called

chlorophyll Photosynthesis takes place

in leaf

cells These contain chloroplasts which

are tiny objects containing chlorophyll

F A C E B O O K

P A G E h t t p w w w f a c e b o o k c o m s s c m e n t o r s o f f i c i a l P a g e | 15

FOR MORE UPDATES AND MORE MATERIAL DO LIKE OUR FACEBOOK PAGE httpwwwfacebookcomsscmentorsofficial

Q59 Insects that transmit diseases are

known as

जह८कीड़हॳरह८गसचाररतकरतहॳहहॴ उनह mdashmdash-

कहॳ नामसहॳजानाजाताहहॴ 16-Jan-2017

1)Pathogens

रह८गज़नक

2) Vectors

वहॳकटर

3) Drones

परजीवी 4)Scalars

अटदषट

Correct Answer Vectors

A vector is an organism that does not

cause disease itself but which spreads

infection by conveying pathogens from

one host to another Species of mosquito

for example serve as vectors for the

deadly disease Malaria

Q60 Which is the second largest gland

of Human body

मानविरीरकीदसरीसबसहॳबड़ीगरिीकह९नसीहहॴ

SSC CHSL Science (biology)

2016 Question Paper

16-Jan-2017

Options

1) Liver

यकत

2) Large Intestine

बड़ीआत

3) Thorax

छाती 4) Pancreas

अगनयािय

Correct Answer Pancreas

Q61 Annona squamosa is the scientific

name of

एनह८नासकवामह८सा (Annona squamosa) mdash

mdashmdash कावहॴजञाननकनामहहॴ 16-Jan-2017

Options

1) Custard Apple

सीताफि

2) Papaya

पपीता 3) Babhul

बबि

4) Drumstick

सहजन

Correct Answer Custard Apple

Q62 The disease Beri Beri is caused due

to the deficiency of which of the

following

बहॳरीबहॳरीरह८गनननननलिखितमसहॳककसकीकमीकहॳकारणहह८ताहहॴ

16-Jan-2017

Options

1) Vitamin B2

पवटालमन B2

2) Vitamin B1

पवटालमन B1

3) Vitamin B12

पवटालमन B12

4) Vitamin E

पवटालमन E

Correct Answer Vitamin B1

Beriberi is a disease caused by a vitamin

B-1 deficiency also known as thiamine

deficiency

Q63 Chlorophyll was first isolated and

named by

किह८रह८कफिकह८ mdash-

दवारापहिहॳपिकऔरनालमतककयागया 16-Jan-2017

F A C E B O O K

P A G E h t t p w w w f a c e b o o k c o m s s c m e n t o r s o f f i c i a l P a g e | 16

FOR MORE UPDATES AND MORE MATERIAL DO LIKE OUR FACEBOOK PAGE httpwwwfacebookcomsscmentorsofficial

Options

1) Caventou

कहॳ वहॳत 2) Pelletier

पहॳिहॳटटयर

3) Chlorophyll

किह८रह८कफि

4) Caventou and Pelletier

कहॳ वहॳतऔरपहॳिहॳटटयर

Correct Answer Caventou and Pelletier

Chlorophyll was first isolated and

named by

Joseph Bienaimeacute Caventou and Pierre

Joseph Pelletier in 1817 The presence of

magnesium in chlorophyll was

discovered in 1906 and was the first

time that magnesium had been detected

in living tissue

Q64 Which of the following organisms

does not fit into the Cell Theory

नननननलिखितमसहॳकह९नसाजीवकह८लिकालसदातअन पनहीहहॴ

16-Jan-2017

Options

1) Bacteria

बहॴकटीररया 2) Virus

वायरस

3) Fungi

कवक

4) Plants

पह९धहॳ Correct Answer Virus

The bottom line is that viruses are not

alive and not related to cells in any way

The cell theory states that all living

things are made of cells cells are the

basic units of structure and function of

living things and that all cells come

from other cells Since viruses are not

made of cells and do not use cells in any

of their processes they are not related to

the cell theory

Q65 Which of these is not a

macronutrient for Plants

नननननलिखितमसहॳकह९नसापह९धह८कहॳ लिएमिह८नयटरीएटनहीहहॴ

SSC CHSL Science (biology) 2016

Question Paper

17-Jan-2017

Options

1) Nitrogen

नाइटरह८जन

2) Phosphorus

फासफह८रस

3) Potassium

पह८टालसयम

4) Chlorine

किह८रीन

Correct Answer Chlorine

In relatively large amounts the soil

supplies nitrogen phosphorus

potassium calcium magnesium and

sulfur these are often called the

macronutrients In relatively small

amounts the soil supplies iron

manganese boron molybdenum

copper zinc chlorine and cobalt the

so-called micronutrients

Q66 Name the respiratory organs of

insects

कीटह८मनसतिशरवसनअगनामकानामहहॴ

17-Jan-2017

Options

1) Skin

तवचा 2) Body Surface

िरीरकीसतह

F A C E B O O K

P A G E h t t p w w w f a c e b o o k c o m s s c m e n t o r s o f f i c i a l P a g e | 17

FOR MORE UPDATES AND MORE MATERIAL DO LIKE OUR FACEBOOK PAGE httpwwwfacebookcomsscmentorsofficial

3) Gills

गिफड़हॳ 4) Tracheae

शरावस- निी Correct Answer Tracheae

Air enters the respiratory systems of

insects through a series of external

openings called

spiracles These external openings

which act as muscular valves in some

insects lead to the internal respiratory

system a densely networked array of

tubes called tracheae

Q67 The poisonous gas accidentally

released in Bhopal Gas Tragedy is

भह८पािगहॴसतरासदीमगितीसहॳमकतहईजहरीिीगहॴसिी

17-Jan-2017

1) Methane

मीिहॳन

2) Nitrous Oxide

नाइटरसऑकसाइि

3) Methyl Isocyanate

महॴचििआयसोसायनहॳट

4) Cyanogen

सायनह८जहॳन

Correct Answer Methyl Isocyanate

Q68 What does Trypsin do

टटरनपसनकयाकरताहहॴ

SSC CHSL Science (biology) 2016

Question Paper

17-Jan-2017

Options

1) Breaks down Carbohydrates

काबोहाइडरहॳटकापवघटनकरताहहॴ 2) Synthesizes proteins

परह८टीनकासििहॳषणकरताहहॴ 3) Breaks down fats

वसाकापवघटनकरताहहॴ 4) Breaks down proteins

परह८टीनकापवघटनकरताहहॴ Correct Answer Breaks down proteins

Trypsin is one of the three principal

digestive

proteinases the other two being pepsin

and

chymotrypsin In the digestive process

trypsin acts with the other proteinases

to break down dietary protein molecules

to their component

peptides and amino acids

A protease is any enzyme that performs

proteolysis protein catabolism by

hydrolysis of peptide bonds

Q69 Name the source from which

Aspirin is produced

उससरह८तकानामबताइए

नजससहॳएनसपररनकाउतपादनककयाजाताहहॴ

17-Jan-2017

Options

1) Willow bark

पविह८कीछाि

2) Oak Tree

ओककावकष

3) Acacia

बबि

4) Eucalyptus

नीिचगरी Correct Answer Willow bark

The compound from which the active

ingredient in aspirin was first derived

salicylic acid was found in the bark of a

willow tree in 1763 by Reverend

Edmund Stone of Chipping-Norton

Q70 Cannis Familiaris is the scientific

name of

कहॴ ननसफहॳ लमलियहॳररस mdash- कावहॴजञाननकनामहहॴ

17-Jan-2017

F A C E B O O K

P A G E h t t p w w w f a c e b o o k c o m s s c m e n t o r s o f f i c i a l P a g e | 18

FOR MORE UPDATES AND MORE MATERIAL DO LIKE OUR FACEBOOK PAGE httpwwwfacebookcomsscmentorsofficial

Options

1) Cat

बबमिी 2)Dog

कतता 3) Fox

िह८मड़ी 4) Wolf

भहॳडड़या Correct Answer Dog

Q71 Harmful bacteria in potable water

make the water

पीनहॳकहॳ पानीमनसतिघातकबहॴकटीररयाउसपानीकह८बनातहॳहहॴ 17-Jan-2017

Options

1) unfit to drink

पीनहॳकहॳ लिएअयह८गय

2) smelly

दगयनधयकत

3) Colored

रगीन

4) Turbid

मटमहॴिा Correct Answer unfit to drink

Q72 Musa paradisiaca is the scientific

name of which plant

मसापहॴराडिलसयाकाककसपह९धहॳकावहॴजञाननकनामहहॴ

17-Jan-2017

Options

1) Mango

आम

2) Wheat

गहॳह

3) Corn

भ ा 4) banana

कहॳ िा Correct Answer banana

Q73 Prawns belong to which family

झीगहॳककसपररवारकहॳ हह८तहॳहहॴ 17-Jan-2017

Options

1) Crustaceans

िसटहॳलियन

2)Fish

मछिी 3) Amphibians

अननफबबयस

4) Reptiles

रहॳपटाइमस

Correct Answer Crustaceans

Q74 Name the drug that is yielded from

Cinchona tree and is used to cure

malaria

उसऔषचधकानामबताइएनजसहॳलसगकह८नापहॳड़सहॳपरापतककयाजाताहहॴऔरनजसकाउपयह८गमिहॳररयाकहॳ उपचारमककयाजाताहहॴ 17-Jan-2017

Options

1) Camptothea

कहॴ नटह८चिया 2) Acuminata

एकयलमनहॳटा 3) Quinine

कनहॴन

4) Cinchonia

लसकह८ननया Correct Answer Quinine

Q75 Blood Circulation was discovered

by

रकतपररसचरणकी mdashmdashndash दवारािह८जकीिी 17-Jan-2017

Options

1) Mary Anderson

F A C E B O O K

P A G E h t t p w w w f a c e b o o k c o m s s c m e n t o r s o f f i c i a l P a g e | 19

FOR MORE UPDATES AND MORE MATERIAL DO LIKE OUR FACEBOOK PAGE httpwwwfacebookcomsscmentorsofficial

महॴरीएिरसन

2) Virginia Apgar

वनजयननयाएपगार

3) William Harvey

पवलियमहाव

4) Robert Feulgen

रॉबटयफ़यिजहॳन Correct Answer William Harvey

Q76 Vitamin A is also known as

पवटालमन A कह८ mdashmdash- कहॳ नामसहॳभीजानाजाताहहॴ SSC CHSL Science (biology) 2016

Question Paper

18Jan2017

Options

1) Thiamine

िायलमन

2) Riboflavin

ररबह८फिहॳपवन

3) Retinol

रहॳटटनॉि

4) Calciferol

कहॴ नमसफहॳ रह८ि

Correct Answer Retinol

Q77 Some roots called arise from an

organ other than the radicle

कछजड़हॳनजनह mdashmdashmdash कहाजाताहहॴ वहमिकहॳ अिावाककसीअनयअगसहॳउतपननहह८तीहहॴ 18Jan2017

Options

1) tap roots

मखयजड़

2) stilt roots

ि ाजड़

3) fibrous roots

रहॳिहॳदारजड़

4) adventitious roots

आकनसमकजड़

Correct Answer adventitious roots

Q78 Spiders belong to which class of

animals

मकडड़यापराणीवगीकरणकहॳ ककसवगयमआतीहहॴ 18Jan2017

Options

1) Arachnids

एरहॳकननडस

2) Aves

एपवस

3) Gastropods

गहॴसटरोपह८िस

4) Anthozoa

एिह८जआ

Correct Answer Arachnids

Q79 How many layers does Human

Skin have

मानवतवचामककतनीपरतहॳहह८तीहहॴ

18Jan2017

Options

1) 5

2) 7

3) 11

4) 3

Correct Answer 3

Skin has three layers The epidermis

the outermost layer of skin provides a

waterproof barrier and creates our skin

tone The dermis beneath the

epidermis contains tough connective

tissue hair follicles and sweat glands

The deeper subcutaneous tissue (

hypodermis ) is made of fat and

connective tissue

Q80 Allium Cepa is the scientific name

of

एलियमलसपपा mdashmdashndash कावहॴजञाननकनामहहॴ 18Jan2017

F A C E B O O K

P A G E h t t p w w w f a c e b o o k c o m s s c m e n t o r s o f f i c i a l P a g e | 20

FOR MORE UPDATES AND MORE MATERIAL DO LIKE OUR FACEBOOK PAGE httpwwwfacebookcomsscmentorsofficial

Options

1) Carrot

गाजर

2) Tomato

टमाटर

3) Potato

आि 4) Onion

पयाज़

Correct Answer Onion

Q81 DNA stands for

िीएनएकापणय प mdashmdash- हहॴ 18Jan2017

Options

1) Di Nucleic Acid

िाईनयनकिकएलसि

2) Deoxy Nucleic Acid

िीओकसीनयनकिकएलसि

3) Diribonucleic Acid

िाईराइबह८नयनकिकएलसि

4) Deoxyribonucleic Acid

िीऑकसीराइबह८नयनकिकएलसि

Correct Answer Deoxyribonucleic Acid

Q82 Organisms that generate energy

using light are known as

जह८जीवाणपरकािकाउपयह८गकरउजायउतपननकरतीहहॴ उनह mdashmdash कहॳ पमजानाजाताहहॴ

18Jan2017

Options

1) Chaemolithotrophs

ककमह८लििह८टरह८पस

2) Oligotrophs

ओलिगह८टरह८पस

3) Bacteria

बहॴकटीररया 4)Photoautotrophs

फह८टह८ओटह८टरह८पस

Correct Answer Photoautotrophs

An oligotroph is an organism that can

live in an environment that offers very

low levels of nutrients

Q83 Which drug is used as an

Antidepressant

ककसदवाएकहतािारह८धीकहॳ पमपयोगककयाजाताहहॴ Options

1) Oxybutynin

ओकसीलयटीनन

2)Tramadol

टरहॳमहॳिह८ि

3 ) Sumatriptan

समहॳटरीपटहॳन

4) Bupropion

लयपरह८पपयह८न

Correct Answer Bupropion

लयपरह८पपयह८न

Q84 The orange colour of carrot is

because of

गाजरकानारगीरगनननननलिखितमसहॳककसीएककीवजहसहॳहह८ताहहॴ 18Jan2017

Options

1) it grows in the soil

यहलम ीमउगतीहहॴ 2) Carotene

कहॴ रह८टीन

3) it is not exposed to sunlight

यहसययपरकािकहॳ सपकय मनहीआती 4) the entire plant is oranqe in colour

सनपणयपह९धानारगीरगकाहह८ताहहॴ Correct Answer Carotene

Q85 Snake venom is highly modified

saliva containing

F A C E B O O K

P A G E h t t p w w w f a c e b o o k c o m s s c m e n t o r s o f f i c i a l P a g e | 21

FOR MORE UPDATES AND MORE MATERIAL DO LIKE OUR FACEBOOK PAGE httpwwwfacebookcomsscmentorsofficial

सापकाजहरअततयाचधकसिह८चधतिारहह८तीहहॴनजसमहॳ mdashmdash- हह८ताहहॴ Options

l)Prototoxins

परह८टह८टॉनकसस

2)Neutrotoxins

नयटरोटॉनकसस

3)Zootoxins

जटॉनकसस

4)Electrotoxins

इिहॳकटरह८टॉनकसस

Correct Answer Zootoxins

जटॉनकसस

Q86 Which type of pathogen causes the

water-borne disease Schistosomiasis

ककसपरकारकारह८गज़नकजिजननतरह८गलससटह८सह८लमलससकाकारणबनताहहॴ

18Jan2017

Option

1) Parasitic

परजीवी 2)Protozoan

परह८टह८जआ

3) Bacterial

बहॴकटीररयि

4) Viral

वायरि

Correct Answer Parasitic

Schistosomiasis also known as snail

fever and bilharzia is a disease caused

by parasitic

flatworms called schistosomes

Q87 Prothrombin responsible for

clotting of blood is released by

परह८िह८ननबन

जह८रकतकािककाजमनहॳकहॳ लिएनजनमहॳदारहहॴ mdashndash

कहॳ दवारासतरापवतककयाजाताहहॴ

19Jan2017

Options

1) Small Intestine

छह८टीआत

2) Blood Platelets

रकतपिहॳटिहॳटस

3) Large Intestine

बड़ीआत

4Heart

हदय

Correct Answer Blood Platelets

Q88 Acacia arabica is the scientific

name of

अकहॳ लियाअरहॳबबका mdashmdashndash कावहॴजञाननकनामहहॴ 19-Jan-2017

Options

1) Neem

नीम

2) Teak

सागह९न

3) Babhul

बबि

4) Pomegranate

अनार

Correct Answer Babhul

Q89 Cannis Vulpes is the scientific

name of

कहॴ ननसवनमपस mdashmdash- कावहॴजञाननकनामहहॴ 19-Jan-2017

Options

1) Dog

कतता 2) Wolf

भहॳडड़या 3) Fox

िह८मड़ी 4) Hyena

िाकिबगघा

F A C E B O O K

P A G E h t t p w w w f a c e b o o k c o m s s c m e n t o r s o f f i c i a l P a g e | 22

FOR MORE UPDATES AND MORE MATERIAL DO LIKE OUR FACEBOOK PAGE httpwwwfacebookcomsscmentorsofficial

Correct Answer Fox

Q90 The beetroot is the portion of the

beet plant

चकदरपह९धहॳका mdashmdashndash भागहहॴ 19-Jan-2017

Options

1) tap root

मखयजड़

2) Adventitious

आकनसमक

3) bulb of the stem

तनहॳकाकद

4) Rhizome

परकद

Correct Answer tap root

Q91 What is the basic unit of heredity

आनवलिकताकीबननयादीइकाईकयाहहॴ 19-Jan-2017

Options

1) DNA

िीएनए

2) RNA

आरएनए

3) Chromosome

िह८मह८सह८म

4) Gene

जीन

Correct Answer gene

Genes are the units of heredity and are

the instructions that make up the bodyrsquos

blueprint They code for the proteins

that determine virtually all of a personrsquos

characteristics Most genes come in

pairs and are made of strands of genetic

material called deoxyribonucleic acid

or DNA

Q92 Lungs are the primary organs of

फहॳ फड़हॳmdashndashकहॳ परािलमकअगहहॴ

19-Jan-2017

Options

1) Digestion

पाचन

2) Constipation

कलज

3) Perspiration

पसीना 4)Respiration

शवसन

Correct Answer Respiration

Q93 Sugarcane is a type of

गननाएकपरकारका mdash- हहॴ 20-Jan-2017

Options

1)creeper

िता 2)tree

पहॳड़

3)shrub

झाड़ी 4)grass

घास

Correct Answer grass

Q94 Who is commonly known as ldquothe

Father of Microbiologyrdquo

सामानयत ldquo सकषमजीवपवजञानकहॳ जनक lsquo

कहॳ नामसहॳककसहॳजानाजातहहॴ 20-Jan-2017

Options

1) Robert Hooke

रॉबटयहक

2) Antonie Philips van Leeuwenhoek

एटह८नीकफलिपवानमयएनहह८क

3) Carl Linnaeus

काियिीनाईयस

4) Charles Darwin

चामसयिापवयन

F A C E B O O K

P A G E h t t p w w w f a c e b o o k c o m s s c m e n t o r s o f f i c i a l P a g e | 23

FOR MORE UPDATES AND MORE MATERIAL DO LIKE OUR FACEBOOK PAGE httpwwwfacebookcomsscmentorsofficial

Correct Answer Antonie Philips van

Leeuwenhoek

Q95 For the aquatic organisms the

source of food is

जिीयजीवाणकािाघसरह८तहहॴ 20-Jan-2017

Options

1) Phytoplankton

फायटह८पिहॳकटन

2) Sea Weed

समदरीिहॴवाि

3)Aqua plankton

एकवापिहॳकटन

4) Zooplankton

जपिहॳकटन

Correct Answer Phytoplankton

Q96 Haemoglobin has the highest

affinity with which of the following

हीमह८गिह८बबनकीननननमसहॳककसकहॳ सािउततमसमानताहहॴ

20-Jan-2017

Options

1)SO2

2)CO2

3)CO

4)NO2

Correct Answer CO

It has a greater affinity for hemoglobin

than oxygen does It displaces oxygen

and quickly binds so very little oxygen

is transported through the body cells

Q97 Who developed the theory of

Evolution

उदपवकासकालसदातककसनहॳपवकलसतककया

20-Jan-2017

Options

1) Charles Darwin

चामसयिापवयन

2) Isaac Newton

आयजहॳकनयटन

3) Pranav Mistry

परणवलमसतरी 4) Galileo Galilei

गहॳलिलियह८गहॳिीिी Correct Answer Charles Darwin

Q98 The primary function of RNA is

RNA कापरािलमककाययहह८ताहहॴ 20-Jan-2017

Options

1) Photosynthesis

परकािसशिहॳषण

2) Protein Synthesis

परह८टीनसशिहॳषण

3) Replication

परनतकनतबनाना 4) Translation

अनवादकरना Correct Answer Protein Synthesis

There are two main functions of RNA

It assists DNA by serving as a messenger

to relay the proper genetic information

to countless numbers of ribosomes in

your body The other main function of

RNA is to select the correct amino acid

needed by each ribosome to build new

proteins for your body

Q99 ______is the movement of

molecules across a cell membrane from

a region of their lower concentration to

a region of their higher concertration

उचचसादरताकहॳ कषहॳतरसहॳउसकीकमसादरतावािहॳकषहॳतरकीतरफएककह८लिकाखझमिीकहॳ माधयमसहॳहह८नहॳवािाअणओकहॳ सचिनकह८ mdash- कहतहॳहहॴ Options

1) Diffusion

पवसरण

2) Osmosis

ऑसमह८लसस

F A C E B O O K

P A G E h t t p w w w f a c e b o o k c o m s s c m e n t o r s o f f i c i a l P a g e | 24

FOR MORE UPDATES AND MORE MATERIAL DO LIKE OUR FACEBOOK PAGE httpwwwfacebookcomsscmentorsofficial

3) Active Transport

सकियआवागमन

4) Passive Transport

नननषियआवागमन

Correct Answer Active Transport

Q100 Study of classification of

organisms is known as 20-Jan-2017

जीवाणओकहॳ वगीकरणकहॳ अधययनकह८ mdash-

कहाजाताहहॴ Options

1) Serpentology

सपरहॳटह८िह८जी 2) Virology

वायरह८िह८जी 3) Taxonomy

टहॴकसोनह८मी 4) Physiology

कफनज़यह८िह८जी Correct Answer Taxonomy

Q101 Photosynthesis takes place inside

plant cells in

परकािसशिहॳषणवनसपनतकह८लिकामनसति mdash

mdashmdash महह८ताहहॴ 20-Jan-2017

Options

1) Ribosomes

राइबह८सह८नस

2) Chloroplasts

किह८रह८पिासट

3) Nucleus

नयकलियम

4) Mitochondria

माईटह८कोडडरया Correct Answer Chloroplasts

Q102 ______ is the cell organelle in

which the biochemical processes of

respiration and energy production

occur

mdashmdash- वहकह८लिकाअगहहॴ नजसमहॳशवसनऔरउजायउतपादनकहॳ जहॴसीजहॴवरासायननकपरकियायहह८तीहहॴ 20-Jan-2017

Options

1) Mitochondria

माइटह८कोडडरया 2) Chloroplast

किह८रह८पिासट

3) Ribosomes

राइबह८सह८नस

4) Nucleus

नयकिीयस

Correct Answer Mitochondria

Q103 Which non-flowering spore

bearing plants have roots

ककसफिनिगनहॳवािहॳऔरबीजाणधारकपह९धह८कीजड़हॳहह८तीहहॴ 21-Jan-2017

Options

1) Mosses

मह८सहॳस

2) Angiosperms

एननजयह८सपनसय 3) Ferns

फनसय 4) Gymnosperms

नजननह८सपनसय Correct Answer ferns

Q104 Which of the following is an

excretory organ of cockroach

नननननलिखितमसहॳकह९नसानतिच हॳकाउतसजयनअगहहॴ

21-Jan-2017

Options

F A C E B O O K

P A G E h t t p w w w f a c e b o o k c o m s s c m e n t o r s o f f i c i a l P a g e | 25

FOR MORE UPDATES AND MORE MATERIAL DO LIKE OUR FACEBOOK PAGE httpwwwfacebookcomsscmentorsofficial

1) Malphigian Tubules

मनमफनजयनटयबमस

2) Nephridia

नहॳकफरडिया 3) Coxal Gland

कह८कसिगरचिया 4) Green Gland

गरीनगरचिया Correct Answer Malphigian Tubules

Q105 Evaporation of water takes place

in which part of plants

पानीकहॳ वाषपीकरणकीकियापह९धोकहॳ ककसभागसहॳहह८तीहहॴ 21-Jan-2017

Options

1) Stem

तना 2) Stomata

सटह८मटा 3) Branch

िािाए

4) Fruit

फि

Correct Answer Stomata

Evaporation accounts for the movement

of water to the air from sources such as

the soil canopy interception and

waterbodies Transpiration accounts for

the movement of water within a plant

and the subsequent loss of water as

vapour through stomata in its leaves

Q106 A is the fleshy spore-bearing

fruiting body of a fungus

mdashmdashndashकवककामासि

बीजाणधारणकरनहॳवािाफिनहॳवािाअगहहॴ 21-

Jan-2017

Options

1) aloe vera

एिह८वहॳरा 2) Coral

मगा 3) Cactus

कहॴ कटस

4) Mushroom

ककरमतता Correct Answer mushroom

Q107 Which of the following is a fungal

disease

नननननलिखितमसहॳकह९नसाफफदसहॳहह८नहॳवािाएकरह८ग हहॴ

21-Jan-2017

Options

1) Dermatitis

तवचािह८ध

2) Cholera

हहॴजा 3) Jaundice

पीलिया 4) Indigofera

इननिगह८फहॳ रा Correct Answer Dermatitis

Dermatitis also known as eczema is a

group of diseases that results in

inflammation of the skin These diseases

are characterized by itchiness red skin

and a rash In cases of short duration

there may be small blisters while in

long-term cases the skin may become

thickened

Q108 In which form is glucose stored in

our body

हमारहॳिरीरमगिकह८जकासचयककस पमककयाजाताहहॴ

21-Jan-2017

Options

1) Insulin

F A C E B O O K

P A G E h t t p w w w f a c e b o o k c o m s s c m e n t o r s o f f i c i a l P a g e | 26

FOR MORE UPDATES AND MORE MATERIAL DO LIKE OUR FACEBOOK PAGE httpwwwfacebookcomsscmentorsofficial

इसलिन

2) Glucose

गिकह८ज

3) Glycogen

गिायकह८जहॳन

4) Fat

वसा Correct Answer Glycogen

Excess glucose is stored in the liver as

the large compound called glycogen

Glycogen is a polysaccharide of glucose

but its structure allows it to pack

compactly so more of it can be stored in

cells for later use

Q109 Where do plants synthesize

protein from

पह९धहॳपरह८टीनसशिहॳषणकहासहॳकरतहॳहहॴ

Options

1) Fatty Acids

वसाऐलसि

2) Sugar

िकर

3) Amino Acids

एलमनह८ऐलसि

4) Starch

सटाचय Correct Answer Amino Acids

Q110 Which part of the brain is

responsible for triggering actions like

thinking intelligence memory and

ability to learn

मनसतषककाकह९नसाटहससासह८चनहॳ बनधदमानी याददाशतऔरसीिनहॳकीकषमताजहॴसीकियाओकह८परहॳररतकरताहहॴ 21-Jan-2017

Options

1) Diencephalon

िायएनसहॳफहॳ िह८न

2) Hypothalamus

हयपह८िहॳिहॳमस

3) Cerebrum

सहॳरहॳिम

4) Control

कटरह८ि

Correct Answer Cerebrum

Q111 Which of the following is also

known as the Biochemical Laboratory

of the Human Body

नननननलिखितमसहॳककसहॳमानविरीरकीजहॴवरसायनपरयह८गिािाभीकहाजाताहहॴ 21-Jan-2017

Options

1) Small Intestine

छह८टीआत

2)Brain

मनसतषक

3) Pancreas

अगनयािय

4) Liver

नजगर

Correct Answer Liver

The liver makes bile that will help

emulsify and digest the fats we eat

The liver takes toxic substances and

convert them using enzymes the liver

cells makes into a non toxic form so the

body can dispose of them

The liver also converts fats protein and

carbohydrates into glucose which is the

energy source for our cells to use

The liver takes amino acids and makes

proteins by combining them

Q112 The yellow colour of human urine

is due to

मानवमतरकापीिारग mdashndash कीवजहसहॳहह८ताहहॴ 22-

Jan-2017

Options

1) Bile Salts

F A C E B O O K

P A G E h t t p w w w f a c e b o o k c o m s s c m e n t o r s o f f i c i a l P a g e | 27

FOR MORE UPDATES AND MORE MATERIAL DO LIKE OUR FACEBOOK PAGE httpwwwfacebookcomsscmentorsofficial

पपततनमक

2) Cholesterol

कह८िहॳसटरह८ि

3) Lymph

लिनफ

4) Urochrome

यरह८िह८म

Correct Answer Urochrome

Urobilin or urochrome is the chemical

primarily responsible for the yellow

color of urine

Q113 The wilting of plants takes place

due to

पह९धह८कालिचििहह८नाकी mdashmdash- कीवजहसहॳहह८ताहहॴ 22-Jan-2017

Options

1)Photosynthesis

परकािसशिहॳषण

2) Transpiration

वाषपह८तसजयन

3) Absorption

अविह८षण

4) Respiration

शरवसन

Correct Answer Transpiration

Wilting is the loss of rigidity of non-

woody parts of plants This occurs when

the turgor pressure in non-lignified

plant cells falls towards zero as a result

of diminished water in the cells

Q114 Bovidae Ovis is the scientific name of

बह८पविीओपवस mdashndash कावहॴजञाननकनामहहॴ 22-Jan-2017

Options

1) Goat

बकरी 2) Cow

गाय

3) Buffalo

भहॳस

4) Sheep

भहॳड़

Correct Answer Sheep

Q115 Plants get their energy to produce

food from which of the following

पह८धहॳभह८जनकाननमायणकरनहॳकहॳ लिएनननननलिखितमसहॳककससहॳउजायपरापतकरतहॳहहॴ

22-Jan-2017

Options

1) Photosynthesis

परकािसशिहॳषण

2)Bacteria

बहॴकटीररया 3)Fungi

कवक

4)Sun

सयय Correct Answer Sun

Q116 Which of the following is secreted

by the liver

नननननलिखितमसहॳककसकासरावनजगरसहॳहह८ताहहॴ

22-Jan-2017

Options

1) Glucose

गिकह८ज

2) Iodine

आयह८िीन

3) Cortisol

काटटरयसह८ि

4) Bile

पपतत

Correct Answer Bile

The liver makes bile that will help

emulsify and

digest the fats we eat

F A C E B O O K

P A G E h t t p w w w f a c e b o o k c o m s s c m e n t o r s o f f i c i a l P a g e | 28

FOR MORE UPDATES AND MORE MATERIAL DO LIKE OUR FACEBOOK PAGE httpwwwfacebookcomsscmentorsofficial

Q117 Ferns belong to which division of

plants

फनसयपह९धह८कहॳ ककसभागमआतहॳहहॴ

22-Jan-2017

Options

1) Gymnosperms

नजननह८सपनसय 2) Angiosperms

एनजयह८सपनसय 3) Thallophyta

िहॴिह८फाईटा 4)Pteridophyta

टहॳररिह८फाईटा Correct Answer Pteridophyta

Q118 Who invented Antibiotics

एटीबायह८टटककाअपवषकारककसनहॳककयािा

22-Jan-2017

Options

1) Joseph Lister

जह८सहॳफलिसटर

2) William Harvey

पवलियमहाव

3) Robert Knock

रॉबटयनॉक

4)Alexander Fleming

अिहॳकज़िरफिहॳलमग

Correct Answer Alexander Fleming

Q119 Milbecycin is used in the

eradication of

लममबहॳसायलसनका mdashndash

मउनमिनमपरयह८गककयाजाताहहॴ 22-Jan-2017

Options

1) Agricultural Fungus

कपषकवक

2) Agricultural Pests

कपषकीटक

3) Agricultural Herbs

कपषिाक

4)Agricultural Weeds

कपषननराना Correct Answer Agricultural Pests

Milbemycin oxime is a veterinary drug

from the group of milbemycins used as

a broad spectrum antiparasitic It is

active against worms and mites(insects

Q120 Intestinal bacteria synthesizes

which of the following in the human

body

मानविरीरमआतोकहॳ बहॴकटीररयानननननलिखितमसहॳककसकासशिहॳषणकरतहॳहहॴ 22-Jan-2017

Options

1) Vitamin K

पवटालमन K

2) Proteins

परह८टीन

3) Fats

वसा 4) Vitamin D

पवटालमन D

Correct Answer Vitamin K

Q121 is the study of the physical form

and external structure of plants

mdashmdash-

मपह९धह८काभहॴनतक पऔरबाहरीसरचनाकाआदयाककयाजाताहहॴ 22-Jan-2017

Options

1) Physiology

कफनजयह८िह८जी 2) Anatomy

िरीररचनापवजञान

3) Phytomorphology

फाईटह८मह८फह८िह८जी 4)Cytology

कह८लिकापवजञान

Correct Answer Phytomorphology

F A C E B O O K

P A G E h t t p w w w f a c e b o o k c o m s s c m e n t o r s o f f i c i a l P a g e | 29

FOR MORE UPDATES AND MORE MATERIAL DO LIKE OUR FACEBOOK PAGE httpwwwfacebookcomsscmentorsofficial

Q122 Which of the following is a

structural and functional unit of

kidneys

नननननलिखितमसहॳकह९नसीगदोकीसरचनातमकऔरकाययकरीईकाईहहॴ

22-Jan-2017

Options

1) Renette Cells

रहॳनहॳटकह८लिकाए

2) Flame Cells

फिहॳमकह८लिकाए

3) Nephrites

नहॳफ़राइटस

4)Nephrons

नहॳफरोस

Correct Answer Nephrons

Nephron functional unit of the kidney

the structure that actually produces

urine in the process of removing waste

and excess substances from the blood

There are about 1000000 nephrons in

each human kidney

Q123 Which of the following is the

largest part of the human brain

नननननलिखितमसहॳकह९नसामानवमनसतषककासबसहॳबड़ाटहससाहहॴ

23-Jan-2017

Options

1) Ribs

पसलियाा 2) Cerebrum

सहॳरहॳिम

3) Pons

पोस

4)Thalamus

िहॴिहॳमस

Correct Answer Cerebrum

The cerebrum is the largest part of the

human brain making up about two-

thirds of the brainrsquos mass It has two

hemispheres each of which has four

lobes frontal parietal temporal and

occipital

Q124 The auxiliary buds

सहायककालियाmdashndash 23-Jan-2017

Options

1) grow endogenously from the pericycle

पहॳरीसाईककिसहॳअनतजातयपवकलसतहह८ताहहॴ 2) arise endogenously from the main

growing point

मिवपदसहॳअनतजातयउठताहहॴ 3) is an embryonic shoot located in the

axil of a leaf

एकभरणिटहहॴजह८एकपततीकहॳ अकषपरनसतिहह८ताहहॴ 4)arise exogenously from the epidermis

एपपिलमयससहॳबटहजातयतरीकहॳ सहॳउठताहहॴ Correct Answer is an embryonic shoot

located in the axil of a leaf

Q125 Which of the following is a viral

disease

इनमहॳसहॳकह९सीएकवायरिबीमारीहहॴ

23-Jan-2017

Options

1) Polio

पह८लियह८ 2) Tetanus

धनसतनभ

3) Leprosy

कषठरह८ग

4) Plague

पिहॳग

Correct Answer Polio

A viral disease (or viral infection)

occurs when an organismrsquos body is

invaded by pathogenic viruses and

infectious virus particles (virions) attach

to and enter susceptible cells

F A C E B O O K

P A G E h t t p w w w f a c e b o o k c o m s s c m e n t o r s o f f i c i a l P a g e | 30

FOR MORE UPDATES AND MORE MATERIAL DO LIKE OUR FACEBOOK PAGE httpwwwfacebookcomsscmentorsofficial

Poliomyelitis often called polio or

infantile paralysis is an infectious

disease caused by the poliovirus

Tetanusmdash A serious bacterial infection

that causes painful muscle spasms and

can lead to death

Leprosy also known as Hansenrsquos

disease (HD) is a long-term infection by

the bacterium Mycobacterium leprae or

Mycobacterium lepromatosis

Plague is an infectious disease caused by

the bacterium Yersinia pestis

Symptoms include fever weakness and

headache

Q126 Which organisms can help to

carry out Vermicomposting

कह९नसाजीववमीकनपह८नसटगममददकरताहहॴ

23-Jan-2017

Options

1) Nitrifying Bacteria

नाईटरीफाईगबहॴकटीररया 2) Earthworms

कहॴ चऐ

3) Algae

िहॴवि

4) Fungus

कवक

Correct Answer Earthworms

Q127 Contraction of heart is also

known as

हदयकहॳ सकचनकह८ mdash- भीकहाजाताहहॴ 23-Jan-

2017

Options

1) Systole

लससटह८ि

2) Aristotle

अरसत

3) Diastole

िायसटह८ि

4) Lub

मयब

Correct Answer Systole

Diastole is the part of the cardiac cycle

when the heart refills with blood

following systole (contraction)

Ventricular diastole is the period during

which the ventricles are filling and

relaxing while atrial diastole is the

period during which the atria are

relaxing

Q128 Azadirachta indica is the

botanical name of which of the

following

अजाटदराचताइडिकानननननलिखितमसहॳककसकावानसपनतनामहहॴ

23-Jan-2017

Options

1) Rose plant

गिाबकापह९धा 2) Apple tree

सहॳबकापहॳड़

3) Neem

नीम

4)Mango

आम

Correct Answer Neem

Q129 Which of the following is the

main end product of carbohydrate

digestion

नननननलिखितमसहॳकह९नसाकाबोहाइडरहॳटकहॳ पाचनकापरमिअतउतपादकहह८ताहहॴ 23-Jan-2017

Options

1) Fats

वसा 2) Lipids

लिपपडस

3) Glucose

गिकह८ज

4) Cellulose

F A C E B O O K

P A G E h t t p w w w f a c e b o o k c o m s s c m e n t o r s o f f i c i a l P a g e | 31

FOR MORE UPDATES AND MORE MATERIAL DO LIKE OUR FACEBOOK PAGE httpwwwfacebookcomsscmentorsofficial

सहॳमयिह८ज

Correct Answer Glucose

Intestinal absorption of end products

from digestion of carbohydrates and

proteins in the pig hellip During absorption some sugars (fructose or

galactose) released from the

corresponding sucrose and lactose

respectively during digestion were

partly metabolized into glucose by the

enterocyte

Q130 Which of the following glands is a

source of the enzyme Ptyalin

नननननलिखितगरचियोमसहॳएजाइमटयालिनकासरह८तहहॴ 23-Jan-2017

Options

1) Pancreas

अगरािय

2) Thyroid Gland

िाइराइिगरिी 3) Pituitary Gland

पीयषगरिी 4) Salivary Glands

िारगरचियाा Correct Answer Salivary Glands

Q131 Which of the following is not true

about Pteridophyta

ननननमसहॳकह९नसीबातटहॳररिह८फाईटकहॳ बारहॳमसचनहीहहॴ 23-Jan-2017

Options

1) Dominant phase is saprophytes

परमिचरणसहॳपरह८फाईइटसहह८ताहहॴ 2) Main plant body is diploid

पह९दह८कामखयिरीरदपवगखणतहह८ताहहॴ 3) Seeds are present

बीजमह९जदहह८तहॳहहॴ 4)Flowers are absent

फिअनपनसतिहह८तहॳहहॴ

Correct Answer Seeds are present

Q132 The largest dolphin species is the

orca also called as

िॉिकफनकीसबसहॳबड़ीपरजानतकाकानामआकायहहॴनजसहॳ mdash- भीकहतहॳहहॴ 23-Jan-2017

Options

1) Bottle Nose

बाटिनह८ज

2) Baiji

बहॳजी 3) Killer whale

ककिरहहॳि

4)Tucuxi

टकवसी Correct Answer Killer whale

Q133 The fat digesting enzyme Lipase

is secreted by which of the following

वसाकापाचनकरनहॳवािाएजाइमिाइपहॳजनननननलिखितमसहॳककसकहॳ दवारासतरापवतहह८ताहहॴ

24-Jan-2017

Options

1) Kidneys

गद

2) Pancreas

अगनयािय

3) Large Intestine

बड़ीआत

4)Liver

नजगर

Correct Answer Pancreas

Lipase is an enzyme that splits fats so

the intestines can absorb them Lipase

hydrolyzes fats like triglycerides into

their component fatty acid and glycerol

molecules It is found in the blood

gastric juices pancreatic secretions

intestinal juices and adipose tissues

F A C E B O O K

P A G E h t t p w w w f a c e b o o k c o m s s c m e n t o r s o f f i c i a l P a g e | 32

FOR MORE UPDATES AND MORE MATERIAL DO LIKE OUR FACEBOOK PAGE httpwwwfacebookcomsscmentorsofficial

Q134 The arrangement of leaves on an

axis or stem is called

एकअकषयातनहॳपरपनततयोकीयवसिाकह८कयाकहाजाताहहॴ SSC CHSL Science (biology) 2016

Question Paper

24-Jan-2017

Options

1) Phyllotaxy

फाइिह८टहॴकसी 2) Vernation

वनिन

3) Venation

वहॳनहॳिन

4)Phytotaxy

फाइटह८टहॴकसी Correct Answer Phyllotaxy

In botany phyllotaxis or phyllotaxy is

the arrangement of leaves on a plant

stem (from Ancient Greek phyacutellon

ldquoleafrdquo and taacutexis ldquoarrangementrdquo)

Phyllotactic spirals form a distinctive

class of patterns in nature

Q135 The study of Cells is also known

as

कह८लिकाओकहॳ अधययनकह८ mdashmdashndash

भीकहाजाताहहॴ 24-Jan-2017

Options

1) Cytology

सायटह८िह८जी 2) Physiology

कफनजयह८िह८जी 3) Nucleology

नयककमयह८िह८जी 4)Cellology

सहॳिह८िह८जी Correct Answer Cytology

Q136 Which of the following scientists

is also known as the Father of Biology

नननननलिखितमसहॳककसवहॴजञाननककह८ ldquoजीवपवजञानकहॳ जनकrdquoकहॳ नामसहॳभीजानाजाताहहॴ 24-Jan-2017

Options

1) Herbert Spencer

हबयटयसपसर

2) Aristotle

अरसत 3) Lamarck

िहॳमाकय 4)Darwin

िापवयन

Correct Answer Aristotle

Q137 Which cells give rise to various

organs of the plant and keep the plant

growing

कह९नसीकह८लिकाएपह९धह८कहॳ लभननअगह८कह८जनमदहॳतीहहॴऔरपह९धह८कह८बढ़नहॳममददकरतीहहॴ

24-Jan-2017

Options

1) Permanent

सिायी 2) Dermal

तवचीय

3) Meristematic

मररसटहॳमटटक

4)Mature

परह८ढ़

Correct Answer Meristematic

A meristem is the tissue in most plants

containing undifferentiated cells

(meristematic cells) found in zones of

the plant where growth can take place

Q138 Rodentia Muridae is the scientific

name of

F A C E B O O K

P A G E h t t p w w w f a c e b o o k c o m s s c m e n t o r s o f f i c i a l P a g e | 33

FOR MORE UPDATES AND MORE MATERIAL DO LIKE OUR FACEBOOK PAGE httpwwwfacebookcomsscmentorsofficial

रह८िहॳलियानयररिी mdashmdash- कावहॴजञाननकनामहहॴ 24-

Jan-2017

Options

1) Mouse

चहा 2) Squirrel

चगिहरी 3) Monkey

बदर

4) Lizard

नछपकिी Correct Answer Mouse

Q139 Name the scientist who proposed

the cell theory

कह८लिकालसदातकापरसतावदहॳनहॳवािहॳवहॴजञाननककानामबताइए 24-Jan-2017

Options

1) Schleiden and Schwann

िीमिनऔरशरववान

2) Lamarck

िहॳमाकय 3) Treviranus

टरहॳवायरहॳनस

4)Whittaker and Stanley

हीटकरऔरसटहॳनिहॳ Correct Answer Schleiden and

Schwann

Q140 The flower with the worldrsquos

largest bloom is

दननयाकासबसहॳबड़ाफिखििनहॳवािा mdashmdashndash हहॴ 24-Jan-2017

Options

1) Pando

पािह८ 2) Posidonia

पह८सीिह८ननया 3) Rafflesia arnoldii

ररफिहॳलियाअनोमिी 4)Helianthus annuus

हहॳलिएनिसएनयअस

Correct Answer Rafflesia arnoldii

Rafflesia arnoldii is a species of

flowering plant in the parasitic genus

Rafflesia It is noted for producing the

largest individual flower on earth It has

a very strong and horrible odour of

decaying flesh earning it the nickname

ldquocorpse flower

Q141 Deficiency of which vitamin

causes night blindness

ककसपवटालमनकीकमीकहॳ कारणरतौधीहह८ताहहॴ 24-Jan-2017

Options

1) Vitamin K

पवटालमन K

2) Vitamin C

पवटालमन C

3) Vitamin B1

पवटालमन B1

4)Vitamin A

पवटालमन A

Correct Answer Vitamin A

Q142 Nongreen plants lack which of the

following

गहॴर-

हररतवनसपनतमनननननलिखितमसहॳककसकीकमीहह८तीहहॴ

24-Jan-2017

Options

1) Chlorophyll

किह८रह८कफि

2) Lycophyll

िायकह८कफि

3) Cyanophyll

F A C E B O O K

P A G E h t t p w w w f a c e b o o k c o m s s c m e n t o r s o f f i c i a l P a g e | 34

FOR MORE UPDATES AND MORE MATERIAL DO LIKE OUR FACEBOOK PAGE httpwwwfacebookcomsscmentorsofficial

सायनह८कफि

4)Phototropism

फह८टह८टरोपपजम

Correct Answer Chlorophyll

Q143 Organisms that use light to

prepare food are known as

जह८जीवपरकािकाउपयह८गकरभह८जनतहॴयारकरतहॳहहॴ उनह mdashmdash- कहॳ पमजानजाताहहॴ 24-Jan-2017

Options

1) Autotrophs

सवपह८षी 2) Heterotrophs

पवषमपह८षज

3) Omnivores

सवायहारी 4)Decomposers

पवघटनकरनहॳवािा Correct Answer Autotrophs

autotrophs often make their own food

by using sunlight carbon dioxide and

water to form sugars which they can use

for energy Some examples of

autotrophs include plants algae and

even some bacteria Autotrophs

(producer) are important because they

are a food source for heterotrophs

(consumers)

A heterotroph is an organism that

ingests or absorbs organic carbon

(rather than fix carbon from inorganic

sources such as carbon dioxide) in order

to be able to produce energy and

synthesize compounds to maintain its

life Ninety-five percent or more of all

types of living organisms are

heterotrophic including all animals and

fungi and some bacteria

Q144 Which of the following is a

primary function of haemoglobin

नननननलिखितमसहॳकह९नसाटहमह८गिह८बबनकाएकपरािलमककाययहहॴ

25-Jan-2017

Options

1) Utilization of energy

उजायकाउपयह८गकरना 2) Prevention of anaemia

रकतामपताहह८नहॳसहॳरह८कना 3) Destruction of bacteria

बहॴकटीररयाकापवनािकरना 4) To transport oxygen

ऑकसीजनकावहनकरना Correct Answer To transport oxygen

Q145 Vascular bundles are absent in

सवहनीबिि mdashmdash- मअनपनसतिरहतहॳहहॴ 25-Jan-2017

Options

1) Bryophyta

िायह८फाइटा 2) Pteridophyta

टहॳररिह८फाईटा 3) Gymnosperms

नजननह८सपमय 4) Angiosperms

एननजयह८सपहॳनसय Correct Answer Bryophyta

Q146 Sauria Lacertidae is the scientific

name of

सहॴररयािहॳसरटाईिी mdashmdashndash कावहॴजञाननकनामहहॴ 25-Jan-2017

Options

1) Crocodile

मगरमचछ

2) Hippopotamus

टहपपह८पह८टहॳमस

3) Lizard

नछपकिी 4) House fly

F A C E B O O K

P A G E h t t p w w w f a c e b o o k c o m s s c m e n t o r s o f f i c i a l P a g e | 35

FOR MORE UPDATES AND MORE MATERIAL DO LIKE OUR FACEBOOK PAGE httpwwwfacebookcomsscmentorsofficial

घरहॳिमकिी Correct Answer Lizard

Q147 Which type of pathogen causes

the water-borne disease SARS (Severe

Acute Respiratory Syndrome)

ककसपरकािकारह८गज़नकजिजननतबीमारीसासयकाकारणबनताहहॴ 25-Jan-2017

Options

1) Viral

वायरि

2) Parasitic

परजीवी 3) Protozoan

परह८टह८जअन

4) Bacterial

बहॴकटीररयि

Correct Answer Viral

Q148 Which of the following organs

produces the enzyme lipase

नननननलिखितमसहॳकह९नसाअगिायपहॳजएजाइमउतपननकरताहहॴ 25-Jan-2017

Options

1) Pancreas

अगनयािय

2) Large Intestine

बड़ीआत

3) Liver

नजगर

4) Small Intestine

छह८टीआत

Correct Answer Pancreas

Q149 A is a long internode forming the

basal part or the whole of a peduncle

एक mdashmdash- एकिबाइटरनह८िहहॴ जह८ननचिाटहससायासनपणयिठिबनताहहॴ 25-

Jan-2017

Options

1) Rhizome

परकद

2) Rachis

महॳ दि

3) floral axis

पषपअकष

4) Scape

भगदड़

Correct Answer scape

Q150 ndash Which of the following

organisms are considered to be both

Living and Non-living

नननननलिखितमसहॳकह९नसहॳजीवाणकह८जीपवतऔरअजीपवतमानाजाताहहॴ

25-Jan-2017

Options

1) Bacteria

बहॴकटीररया 2) Fungi

कवक

3) Algae

िहॴवाि

4)Virus

वायरस

Correct Answer Virus

They are considered to be living as they

possess a protein coat as a protective

covering DNA as the genetic material

etc

They are said to be non-living as they

can be crystallised and they survive for

billions of years They can tolerate high

temperatures freezing cold

temperatures ultra-violet radiations etc

Q151 Deficiency of fluorine causes

which of the following

फिह८ररनकीकमीकहॳ कारणनननननलिखितमसहॳकयाहह८ताहहॴ

F A C E B O O K

P A G E h t t p w w w f a c e b o o k c o m s s c m e n t o r s o f f i c i a l P a g e | 36

FOR MORE UPDATES AND MORE MATERIAL DO LIKE OUR FACEBOOK PAGE httpwwwfacebookcomsscmentorsofficial

27-Jan-2017

Options

1) Dental Caries

िटिकहॴ ररज

2) Scurvy

सकवरी 3) Anaemia

रकतामपता 4) Arthritis

गटठया Correct Answer Dental Caries

Q152 In a Punnett Square with the

cross AaBb x AaBb how many Aabb

genotypes would be created

पनहॳटसककायरमिह८स AaBb x AaBb कहॳ साि

ककतनहॳ Aabb जीनह८टाइपबनगहॳ 27-Jan-2017

Options

1) 1

2) 8

3) 2

4) 3

Correct Answer 2

Q153 Which of the following is the

Controlling Center of the Cell

नननननलिखित म सहॳ कह८लिकाका ननयतरण

क दर कह९न हहॴ

27-Jan-2017

Options

1) Nucleus

क दर

2) Plasma

पिाजमा 3) Lysosome

िायसह८सह८म

4) Chromosome

िह८मह८सह८म

Correct Answer Nucleus

The control centre of the cell is the

nucleus in eukaryotic cells The nucleus

contains genetic material in the form of

DNA

Q154 Myopia affects which of the

following organs

मायह८पपयानननननलिखितअगह८मसहॳककसहॳपरभापवतकरताहहॴ

25-Jan-2017

Options

1) Heart

हदय

2) Skin

तवचा 3) Eyes

आािहॳ 4)Mouth

मह

Correct Answer Eyes

Q155 Which of the following bears

flowers

नननननलिखितमसहॳकह९नफिधारणकरताहहॴ

25-Jan-2017

Options

1) Bryophyta

िायह८फाइटा 2) Pteridophyta

टहॳरीिह८फाईटा 3) Gymnosperms

नजननह८सपमय 4)Angiosperms

एननजयह८सपमय Correct Answer Angiosperms

Q156 Oxygenated blood flows out of the

heart through the

ऑकसीजनयकतरकत mdashmdashmdash

कहॳ माधयमसहॳहदयकहॳ बाहरबहताहहॴ 25-Jan-2017

F A C E B O O K

P A G E h t t p w w w f a c e b o o k c o m s s c m e n t o r s o f f i c i a l P a g e | 37

FOR MORE UPDATES AND MORE MATERIAL DO LIKE OUR FACEBOOK PAGE httpwwwfacebookcomsscmentorsofficial

Options

1) Aorta

महाधमनी 2) pulmonary artery

फहॳ फड़हॳकीधमनी 3) vena cava

वहॳनाकावा 4)Atrium

चह९क

Correct Answer aorta

Q157 Blood leaving the liver and

moving towards the

heart has a higher concentration of

नजगरसहॳननकिकरहदयकीतरफजानहॳवािहॳरकतम mdashmdashmdashmdash कीउचचसादरताहह८तीहहॴ 27-Jan-2017

Options

1) Lipids

लिपपडस

2) Urea

यररया 3) Bile Pigments

पपततकहॳ रगकरण

4) Carbon dioxide

काबयनिायऑकसाइि

Correct Answer Bile Pigments

Urea is nitrogen containing substance

which is produced in the liver in order

to deal with excess amino-acids in the

body As urea is produced it leaves the

liver in the blood stream and passes via

the circulatory system to all parts of the

body

Q158 Bulb is a modification of which

part of a plant

बमबएकपह९धहॳकहॳ ककसटहससहॳकाएक पातरणहह८ताहहॴ 27-Jan-2017

Options

1) The root

जड़

2) The stem

तना 3) The radicle

मिाकर

4)The fruit

फि

Correct Answer The stem

Q159 Which of the following carries

blood away from the heart to different

body parts

इनमहॳसहॳकह९नरकतकह८हदयसहॳिरीरकहॳ पवलभननअगह८तकिहॳजातीहहॴ

27-Jan-2017

Options

1) Arteries

धमननया 2) Nerves

तबतरहाए

3) Capillaries

कहॳ लिकाए

4)Veins

नसहॳ Correct Answer Arteries

Q160 The series of processes by which

nitrogen and its compounds are

interconverted in the environment and

in living organisms is called

27-Jan-2017

Options

1)Absorption of Nitrogen

2)Ammonification

3)Nitrogen Fixation

4)Nitrogen Cycle

Correct Answer Nitrogen Cycle

Ammonification or Mineralization is

performed by bacteria to convert

organic nitrogen to ammonia

F A C E B O O K

P A G E h t t p w w w f a c e b o o k c o m s s c m e n t o r s o f f i c i a l P a g e | 38

FOR MORE UPDATES AND MORE MATERIAL DO LIKE OUR FACEBOOK PAGE httpwwwfacebookcomsscmentorsofficial

Nitrification can then occur to convert

the ammonium to nitrite and nitrate

Nitrogen fixation is a process by which

nitrogen in the Earthrsquos atmosphere is

converted into ammonia (NH3) or other

molecules available to living organisms

Q161 BCG vaccine is given to protect

from which of the following

बीसीजीकाटटकानननननलिखितमसहॳककसकहॳ बचावकहॳ लिएटदयाजातहहॴ

27-Jan-2017

Options

1) Jaundice

पीलिया 2) Anaemia

रकतमपता 3) Tuberculosis

कषयरह८ग

4) Polio

पह८लियह८ Correct Answer Tuberculosis

Q162 Parallel venation is found in

समानतरवहॳनहॳिन mdashmdashmdash- मपायाजाताहहॴ 27-Jan-2017

Options

1) plants which are monocots

पह९धहॳजह८एकबीजपतरीहह८तहॳहहॴ 2) plants which have a dicot stem

वहॳपह९धहॳनजनकातनादपवदलियहह८ताहहॴ 3) plants with leaves similar to Tulsi

वहॳपह९धहॳनजनकीपनततयतिसीकीपनततयोकहॳ समानहह८तहॳहहॴ 4)plants with tap roots

टहॳप टवािहॳपह९धहॳ Correct Answer plants which are

monocots

Q163 The hardest part of the body is

िरीरकासबसहॳकठह८रभाग mdashndash हहॴ 27-Jan-2017

Options

1) Bones

हडडिय

2) Tooth Enamel

दातकहॳ इनहॳमि

3) Skull

िह८पड़ी 4) Spinal Cord

महॳ रजज

Correct Answer Tooth Enamel

Q164 Which type of pathogen causes

the waterborne disease E coli Infection

ककसपरकारकारह८गजननकजिजननतरह८गईकह८िाईसिमणकाकारणबनताहहॴ 27-Jan-2017

Options

1) Protozoan

परह८टह८जआ

2) Parasitic

परजीवी 3) Bacterial

बहॴकटीररयि

4)Viral

वायरि

Correct Answer Bacterial

Q165 The amount of blood filtered

together by both the kidneys in a 70 kg

adult male human in a minute is

70 की गरा वािहॳएकवयसकप षमएकलमनटमदह८नोगदकहॳदवाराएकसािचाबनीगयीरकतकीमातरहह८तीहहॴ 29-Jan-2017

Options

1) 1100 ml

1100 लमलि

2) 100 ml

F A C E B O O K

P A G E h t t p w w w f a c e b o o k c o m s s c m e n t o r s o f f i c i a l P a g e | 39

FOR MORE UPDATES AND MORE MATERIAL DO LIKE OUR FACEBOOK PAGE httpwwwfacebookcomsscmentorsofficial

100 लमलि

3) 1500 ml

1500 लमलि

4) 500 ml

500 लमलि

Correct Answer 1100 ml

Q166 Which feature of a plant helps to

distinguish a monocot from a dicot

पह९धहॳकीवहकह९नसीपविहॳषताहहॴजह८एकदपवदलियहॳऔरएकएकदिीयपह९धहॳसहॳभहॳदकरनहॳममददकरतीहहॴ 29-Jan-2017

Options

1) Pollination

परागम

2) Venation

वहॳनहॳिन

3) Vernation

वनिन

4) Aestivation

एसटीवहॳिहॳन

Correct Answer venation

Q167 The Mutation Theory was

proposed by

उतवररवतयनकालसदात mdashmdashndash

कहॳ दवरापरसतापवतककयाजाताहहॴ 29-Jan-2017

Options

1) Charles Lyell

चामसयलियहॳि

2) William Smith

पवलियमनसमि

3) Hugo De Vries

हयगह८िीराईस

4)Harrison Schmitt

हहॳरीसननसमट

Correct Answer Hugo De Vries

Q168 Which type of pathogen causes

the waterborne disease HepatitisA

ककसपरकारकहॳ रह८गजनकजिजननतरह८गहहॳपहॳटाइटटस-A काकारणबनताहहॴ

29-Jan-2017

Options

1) Parasitic

परजीवी 2) Viral

वायरि

3) Protozoan

परह८टह८जआ

4) Bacterial

बहॴकटीररयि

Correct Answer Viral

Q169 In a Punnett Square with the

cross AaBb x Aabb how many AaBb

genotypes would be created

पनहॳटसकवायरमिह८स AaBb x Aabb

कहॳ सािककतनहॳ AaBb जीनह८टाइपबनगहॳ 29-Jan-

2017

Options

1) 4

2) 1

3) 7

4) 6

Correct Answer 4

Q170 Arboreal Ateles is the scientific

name of

अिह८ररयिएटटलिस mdashmdashmdash कावहॴजञाननकनामहहॴ 29-Jan-2017

Options

1) Squirrel

चगिहरी 2) Sparrow

गह८रहॴया 3) Lizard

नछपकिी 4) Spider monkey

F A C E B O O K

P A G E h t t p w w w f a c e b o o k c o m s s c m e n t o r s o f f i c i a l P a g e | 40

FOR MORE UPDATES AND MORE MATERIAL DO LIKE OUR FACEBOOK PAGE httpwwwfacebookcomsscmentorsofficial

मकड़ीबदर

Correct Answer Spider monkey

Q171 Which type of pathogen causes

the waterborne disease Salmonellosis

ककसपरकारकारह८गाणजिजननतबीमारीसािमह८नहॳिह८लसज़काकारकहहॴ

29-Jan-2017

Options

1) Algal

िहॳवालियहॳ 2) Parasitic

परजीवी 3) Bacterial

बहॴकटीररयि

4)Viral

वायरि

Correct Answer Bacterial

An infection with salmonella bacteria

commonly caused by contaminated food

or water

Symptoms include diarrhoea fever

chills and abdominal pain

Q172 is a condition in which there is a

deficiency of red cells or of haemoglobin

in the blood

mdashmdash-

एकनसिनतहहॴनजसमहॳरकतमिािकह८लिकाओकीयाहीमह८गिह८बबनकीकमीहह८तीहहॴ 29-Jan-2017

Options

1) Albinism

एनमबननजम

2) Propyria

परह८पीररया 3) Anaemia

एनीलमया 4)Keloid disorder

कहॳ िह८इिडिसओिर

Correct Answer Anaemia

Q173 Ananas comosus is the scientific

name of

Options

अनानासकह८मह८सस mdashmdashmdashndash

कावहॴजञाननकनामहहॴ 29-Jan-2017

1) Custard Apple

सीताफि

2) Pineapple

पाइनएपपि

3) Bamboo

बास

4)Pomegranate

अनार

Correct Answer Pineapple

Q174 Which organ produces insulin

कह९नसाअगइनसलिनपहॴदाकरताहहॴ 29-Jan-

2017

Options

1) Liver

यकत

2) Thyroid gland

िायराइिगरिी 3) Spleen

पिीहा 4)Pancreas

अगरयिय

Correct Answer Pancreas

Q175 Which of the following disease is

not caused by water pollution

नननननलिखितमसहॳकह९नसारह८गपानीकहॳ परदषणकहॳकारणनहीहह८ता

29-Jan-2017

Options

1) Cholera

हहॴजा 2) Typhoid

F A C E B O O K

P A G E h t t p w w w f a c e b o o k c o m s s c m e n t o r s o f f i c i a l P a g e | 41

FOR MORE UPDATES AND MORE MATERIAL DO LIKE OUR FACEBOOK PAGE httpwwwfacebookcomsscmentorsofficial

टाइफाइि

3) Asthma

दमा 4)Diarrhoea

दसत

Correct Answer Asthma

Q176 Ocimum tenuiflorum is the

scientific name of

ओलिलममटहॳयईफिह८रमइसकावहॴजञाननकनाम mdash

ndash हहॴ 30-Jan-2017

Options

1) Neem

नीम

2) Mango

आम

3) Babul

बबि

4)Tulsi

तिसी Correct Answer Tulsi

Q177 Which gland secretes bile a

digestive fluid

कह९नसीगरिीपपतत एकपाचनतरिपरदािय सरापवतकरतीहहॴ 30-Jan-2017

Options

1) Pancreas

अगनयािय

2) Liver

यकत

3) Thyroid

िायराइि

4) Testes

टहॳनसटस

Correct Answer liver

Q178 In which of the following the

dominant phase is Gametophyte

नननननलिखितमसहॳककसकहॳ परमिचरणयगमकह८दपवधद (Gametophyte)हहॴ 30-Jan-2017

Options

1) Bryophyta

िायह८फाइटा 2) Pteridophyta

टहॳररिह८फाइटा 3) Gymnosperms

नजननह८सपमय 4) Angiosperms

एननजयह८सपमय Correct Answer Bryophyta

Q179 Anaerobic respiration refers to

which of the following

नननननलिखितमसहॳककसहॳअवायवीयशवसनकहाजाताहहॴ

30-Jan-2017

Options

1) Respiration without Oxygen

ऑकसीजनकहॳ बबनाशवसन

2) Respiration with Oxygen

ऑकसीजनकहॳ सािशवसन

3) Respiration without CO2

काबयनिायऑकसाइिकहॳ बबनाशवसन

4) Respiration with CO2

काबयनिायऑकसाइिकहॳ सािशविन

Correct Answer Respiration without

Oxygen

Q180 Which type of pathogen causes

the waterborne disease Cholera

ककसपरकारकारह८गजनकजिजननतरह८गहहॴजाकाकारणबनताहहॴ

30-Jan-2017

Options

1) Algal

िहॴवालियहॳ

F A C E B O O K

P A G E h t t p w w w f a c e b o o k c o m s s c m e n t o r s o f f i c i a l P a g e | 42

FOR MORE UPDATES AND MORE MATERIAL DO LIKE OUR FACEBOOK PAGE httpwwwfacebookcomsscmentorsofficial

2) Bacterial

बहॴकटीररयि

3) Protozoan

परह८टह८जआ

4) Viral

वायरि

Correct Answer Bacterial

Q181 To which class does

Oxyreductases transferases hydrolases

belong

ओकसीररिकटहॳसटरासफरहॳजहॳस

हाइडरह८िहॳसहॳसककसवगयमआतहॳहहॴ 30-Jan-2017

Options

1) Hormones

हारमोस

2) Enzymes

एजाइनस

3) Proteins

परह८टीनस

4) Vitamins

पवटालमनस

Correct Answer Enzymes

Q182 Which of the following is not true

about Gymnosperms

ननननमसहॳकह९नसीबातअनावतबीजीकहॳ बारहॳमसचनहीहहॴ 30-Jan-2017

Options

1) Dominant phase is saprophytes

परमिचरणसहॳपरह८फाइटसहह८ताहहॴ 2) Vascular bundles are absent

सवहनीबििअनपनसितहह८ताहहॴ 3) spores are heterospores

बीजाणहहॳटहॳरह८सपह८रसहह८तहॳहहॴ 4) Flowers are absent

फिअनपनसितहह८तहॳहहॴ

Correct Answer Vascular bundles are

absent

Q183 The name of first mammal clone sheep is

भहॳड़कीपरिमसतनपायीपरनत प (किह८न)

कानामहहॴ 30-Jan-2017

Options

1) Noori

नरी 2) Dolly

िॉिी 3) Louise

िसी 4)Durga

दगाय Correct Answer Dolly

Q184 Which type of pathogen causes

the water-borne disease Typhoid fever

ककसपरकारकारह८गजनकजिजननतरह८गटाइफाइिबिारकाकारणबनताहहॴ 30-Jan-2017

Options

1) Algal

िहॴवािीय

2) Parasitic

परजीवी 3) Protozoan

परह८टह८जनअन

4)Bacterial

बहॴकटीररयि

Correct Answer Bacterial

Q185 In which part of the cell are

proteins made

कह८लिकाकहॳ ककसटहससहॳमपरह८टीनबनायाजाताहहॴ

31-Jan-2017

Options

1) Reticulum

रहॳटटकिम

F A C E B O O K

P A G E h t t p w w w f a c e b o o k c o m s s c m e n t o r s o f f i c i a l P a g e | 43

FOR MORE UPDATES AND MORE MATERIAL DO LIKE OUR FACEBOOK PAGE httpwwwfacebookcomsscmentorsofficial

2) Golgi apparatus

गह८मजीएपहॳरहॳटस

3) Ribosomes

ररबह८सह८नस

4) Lysosome

िायसह८सह८नस

Correct Answer ribosomes

Proteins are produced by stringing

amino acids together in the order

specified by messenger RNA strands

that were transcribed from DNA in the

cell nucleus The process of synthesizing

a protein is called translation and it

occurs on ribosomes in the cytoplasm of

a cell

Q186 Polio is a disease caused by which

of the following

नननननलिखितमसहॳपह८लियह८कीबबमारह८हह८नहॳकाकारणकयाहहॴ

31-Jan-2017

Options

1) Bacteria

बहॴकटीररयि

2) Mosquito

मचछर

3) Virus

वायरस

4) Cockroach

नतिच हॳ Correct Answer Virus

Polio or poliomyelitis is a crippling and

potentially deadly infectious disease It

is caused by the poliovirus

Q187 ndash Hay fever is a sign of which of

the following

हहॳकफवरनननननलिखितमसहॳककसकाएकसकहॳ तहहॴ

31-Jan-2017

Options

1) Old Age

वदावसिा 2) Malnutrition

कपह८सण

3) Allergy

एिनजय 4) Over Work

अतयचधककाययकरना Correct Answer Allergy

Q188 How many chromosomes does a

human cell contain

एकमानवकह८लिकामककतनहॳगणसतरहह८तहॳहहॴ

29-Jan-2017

Options

1) 6

2) 26

3) 46

4) 66

Correct Answer 46

In humans each cell normally contains

23 pairs of chromosomes for a total of

46 Twenty-two of these pairs called

autosomes look the same in both males

and females The 23rd pair the sex

chromosomes differ between males and

females

Q189 Which of the following is not true

about Bryophyta

ननननमसहॳकह९नसीबातिायह८फाइटकहॳ बारहॳमसचनहीहहॴ 31-Jan-2017

Options

1) Dominant phase is gametophytes

परमिचरणगहॳलमतह८फाइटसहह८ताहहॴ 2) Main plant body is haploid

पह९धहॳकामखयिरीरअगखणतहह८ताहहॴ 3) Spores are homospores

बीजाणहह८मह८सफह८रसहह८तहॳहहॴ 4) Flowers are present

फिमह८जदहह८तहॳहहॴ Correct Answer Flowers are present

F A C E B O O K

P A G E h t t p w w w f a c e b o o k c o m s s c m e n t o r s o f f i c i a l P a g e | 44

FOR MORE UPDATES AND MORE MATERIAL DO LIKE OUR FACEBOOK PAGE httpwwwfacebookcomsscmentorsofficial

Q190 Which aquatic animal has

trailing tentacles

ककसजिीयजानवरकहॳ पीछहॳचिनहॳवािहॳटहॳटकिसहह८तहॳहहॴ

31-Jan-2017

Options

1) Sea horse

समदरीघह८िा 2) Corals

मगा 3) Jelly fish

जहॳिीमछिी 4) Star fish

तारामछिी Correct Answer Jelly fish

Jellyfish with its umbrella-shaped bell

and trailing tentacles

Q191 Which type of pathogen causes

the water-borne disease Poliomyelitis

(Polio)

ककसपरकारकारह८गजनकजिजननतरह८गपह८लियह८मायहॳटटस (पह८लियह८) काकारणहहॴ 31-Jan-

2017

Options

1) Parasitic

परजीवी 2) Algal

िहॴवालिय

3) Viral

वायरि

4) Bacterial

बहॴकटीररयि

Correct Answer Viral

Q192 The outer white part of the eye

that protects the inner structures is

आािकाबाहरीसफहॳ दटहससाजह८आतररकसरचनाओकीरकषाकरताहहॴ वह mdashmdashmdash हहॴ 31-Jan-

2017

Options

1) Iris

आयररस

2) Sclera

सकिहॳरा 3) Retina

रहॳटटना 4) Cornea

कह८ननयया Correct Answer Sclera

Q193 Proteins are made up of

परह८टीनकाननमायण mdashndash सहॳहह८ताहहॴ 31-Jan-2017

Options

1) Amino acids

एलमनह८अनि

2) Fatty acids

वसायकतअनि

3) Glucose

गिकह८ज

4)Nucleotides

नयनकियह८टाईिस

Correct Answer Amino acids

Q194 Moringa Oleifera is the scientific

name of

मह८ररगओलिफहॳ रा mdashmdashndash कावहॴजञाननकनामहहॴ 31-Jan-2017

Options

1) Banyan

बरगद

2) Gulmohar

गिमह८हर

3) Amla

आमिा

F A C E B O O K

P A G E h t t p w w w f a c e b o o k c o m s s c m e n t o r s o f f i c i a l P a g e | 45

FOR MORE UPDATES AND MORE MATERIAL DO LIKE OUR FACEBOOK PAGE httpwwwfacebookcomsscmentorsofficial

4) Drumstick

डरमनसटक

Correct Answer Drumstick

Q195 Kidney stones are composed of

गदकीपिरी mdashndash सहॳबनीहह८तीहहॴ 1-Feb-2017

Options

1) Calcium Oxalate

कहॴ नमसयमओकजहॳिहॳट

2) Sodium Chloride

सह८डियमकिह८राइि

3) Magnesium Nitrate

महॳनगनलियमनाइतटरहॳट

4) Calcium Bicarbonate

कहॴ नमियमबायकबोनहॳट

Correct Answer Calcium Oxalate

Q196 ndash Which of the following is not

true about Angiosperms

ननननमसहॳकह९नसीबातआवतबीजीकहॳ बारहॳमसचनहीहहॴ 1-Feb-2017

Options

1) Dominant phase is gametophytes

परमिचरणगहॳलमतह८फाइटहह८ताहहॴ 2) Vascular bundles are present

सवहनीबििमह९जदहह८ताहहॴ 3) Spores are heterospores

बीजाणहहॳटहॳरह८सपह८रसहह८तहॳहहॴ 4) Seeds are covered

बीजढकहॳ हह८तहॳहहॴ Correct Answer Dominant phase is

gametophytes

Q197 All of the following are excretory

(waste) products of animals except

नननननलिखितमसहॳककसएककह८छह८ड़करअनयसभीपराखणयोदवाराउतसनजयतपदाियहहॴ 1-Feb-

2017

Options

1) Uric Acid

यररकएलसि

2) Ammonia

अमह८ननया 3) Carbohydrates

काबोहाइडरहॳट

4) Urea

यररया Correct Answer Carbohydrates

In animals the main excretory products

are carbon dioxide ammonia (in

ammoniotelics) urea (in ureotelics) uric

acid (in uricotelics) guanine (in

Arachnida) and creatine

Q198 RNA is a polymeric molecule

What does RNA stand for

आरएनइएएकबहिकआणहहॴ इसकाकापवय पकयाहहॴ 1-Feb-2017

Options

1) Rado Nuclear Acid

रािह८नयनकियरएलसि

2) Ribo Nucleic Acid

राइबह८नयनकिकएलसि

3) Rhino Nuclear Acid

हाइनह८नयनकियरएलसि

4) Resto Nucleus Acid

रहॳसटह८नयकिीयसएलसि

Correct Answer Ribo Nucleic Acid

Q199 Which organ does detoxification

and produces chemicals needed for

digestion

कह९नसाअगपवषहरणकरताहहॴऔरपाचनकहॳ लिएआवशयकरसायनोकह८पहॴदाकरताहहॴ 1-Feb-

2017

Options

1) Salivary glands

िारगरचिया 2) Pancreas

अगनयािय

F A C E B O O K

P A G E h t t p w w w f a c e b o o k c o m s s c m e n t o r s o f f i c i a l P a g e | 46

FOR MORE UPDATES AND MORE MATERIAL DO LIKE OUR FACEBOOK PAGE httpwwwfacebookcomsscmentorsofficial

3) Thyroid gland

िायराइिगरिी 4) Liver

यकत

Correct Answer Liver

Q200 Psidium guajava is the scientific

name of

लसडियमगआजावा mdashmdash कावहॴजञाननकनामहहॴ 1-

Feb-2017

Options

1) Guava

अम द

2) Mango

आम

3) Bamboo

बास

4) Jack fruit

कटहि

Correct Answer Guava

Q201 Which drug is used as a Blood

Thinner

चधरकह८पतिाकरनहॳकहॳ पमककसदवाकापरयह८गककयाजाताहहॴ

1-Feb-2017

Options

1) Warfarin

वाफर न

2) Tramadol

टरहॳमािह८ि

3) Azithromycin

एनजरह८मायलसन

4) Hydralazine

हाइडरह८िहॳनजन

Correct Answer Warfarin

Q202 Which of the following disease is

caused due to the deficiency of protein

परह८टीनकीकमीकहॳ कारणनननननलिखितमसहॳकह९नसारह८गहह८ताहहॴ 1-Feb-2017

Options

1) Arthritis

गटठया 2) Kwashiorkor

कािीओकय र

3) Goitre

गाइटर

4) Night Blindness

रतह९चध

Correct Answer Kwashiorkor

Q203 A is species of plant that has

adapted to survive in an environment

with little liquid water

mdashmdashndashपह९धहॳकीएकऐसहॳऐसहॳपरजानतहहॴ नजसनहॳकमपानीवािहॳवातावरणमजीपवतरहनहॳकहॳलिएअनकिनहहॴ 1-Feb-2017

Options

1) Xerophyte

म दपवद

2) Hydrophyte

जिीयपादप

3) Mesophyte

समह८दपवद

4) Thallophyte

िहॴिह८फाइटा Correct Answer xerophyte

xerophyte is a species of plant that has

adapted to survive in an environment

with little liquid water such as a desert

or an ice- or snow-covered region in the

Alps or the Arctic

Mesophytes are terrestrial plants which

are adapted to neither a particularly

dry nor particularly wet environment

An example of a mesophytic habitat

would be a rural temperate meadow

F A C E B O O K

P A G E h t t p w w w f a c e b o o k c o m s s c m e n t o r s o f f i c i a l P a g e | 47

FOR MORE UPDATES AND MORE MATERIAL DO LIKE OUR FACEBOOK PAGE httpwwwfacebookcomsscmentorsofficial

which might contain goldenrod clover

oxeye daisy and Rosa multiflora

thallophyte any of a group of plants or

plantlike organisms (such as algae and

fungi) that lack differentiated stems

leaves and roots and that were formerly

classified as a primary division

(Thallophyta) of the plant kingdom

Q204 How many types of teeth are

there in humans

मनषयोमककतनहॳपरकारकहॳ दातहह८तहॳहहॴ

1-Feb-2017

Options

1) 4

2) 5

3) 2

4) 3

Correct Answer 4

teeth -Humans have four types of

teethincisors canines premolars and

molars each with a specific function

The incisors cut the food the canines

tear the food and the molars and

premolars crush the food

Q205 Carica papaya is the scientific name of

कहॴ ररकापपाया mdashmdashndash कावहॴजञाननकनामहहॴ 2-

Feb-2017

Options

1) Peepal

पीपि

2) Papaya

पपीता 3) Tamarind

इमिी 4) Drumstick

ढह८िकाछड़ी Correct Answer Papaya

Q206 Muscles get tired when there is

shortfall of

जब mdashndash कीकमीहह८तीहहॴतबपहॳिीयिकजातीहहॴ 2-Feb-2017

Options

1) Lactic acid

िहॴनकटकएलसि

2) Na+ ions

Na+ आयन

3) ATP

एटीपी 4) Sulphates

समफहॳ टस

Correct Answer ATP

ATP is the energy source muscle fibers

use to make muscles contract

muscle tissuersquos main source of energy

called adenosine triphosphate or ATP

As your muscles use up this energy

source they become tired and fatigued

Oxygen is the key ingredient that helps

create new ATP to replenish the burned

up ATP in your muscles

Q207 Artocarpus integra is the

scientific name of आटह८कापयसइटीगरा mdashmdashmdash कावहॴजञाननकनामहहॴ 2-Feb-2017

Options

1) Guava

अम द

2) Pineapple

अनानास

3) Silver Oak

लसमवरओक

4) Jack fruit

कटहि

Correct Answer Jack fruit

Q208 Which organ stores fat soluble

vitamins

कह९नसाअगवसामघिनिीिपवटालमनह८काभिाराकरताहहॴ

2-Feb-2017

F A C E B O O K

P A G E h t t p w w w f a c e b o o k c o m s s c m e n t o r s o f f i c i a l P a g e | 48

FOR MORE UPDATES AND MORE MATERIAL DO LIKE OUR FACEBOOK PAGE httpwwwfacebookcomsscmentorsofficial

Options

1) Blood

रकत

2) Skin

तवचा 3) Liver

यकत

4) Pancreas

अगनयािय

Correct Answer Liver

Q209 Which disease is caused due to

deficiency of Iodine

आयह८िीनकहॳ कारणकह९नसारह८गहह८ताहहॴ 2-Feb-2017

Options

1) Rickets

ररकहॳ टस

2) Scurvy

सकवी 3) Goitre

गणमािा 4) Growth retardation

पवकासका कना Correct Answer Goitre

rickets A softening and weakening of

bones in children usually due to

inadequate vitamin D

Q210 Grevillea Robusta is the scientific name of

गरहॳपवलियारह८बसटा mdashmdashmdash- कापवजञाननकनामहहॴ 2-Feb-2017

Options

1) Peepal

पीपि

2) Teak

सागह९न

3) Silver Oak

लसमवरओक

4) Jack fruit

कटहि

Correct Answer Silver Oak

Q211 When a Cuttlefish is described as a Molluscs it is at which level of

classification

जबएककटिकफिकह८एकमह८िसकाकहॳ पमवखणयतककयाजाताहहॴतबयहॳवगीकरणकहॳ ककससतरपहॳनसितहहॴ 2-Feb-2017

Options

1) Class

वगय 2) Order

िम

3) Family

पररवार

4) Phylum

सघ

Correct Answer Phylum

Q212 Bambusa dendrocalmus is the

scientific name of बानबसािहॳडराकामस mdashmdashmdash कावहॴजञाननकनामहहॴ 3-Feb-2017

Options

1) Banyan

बरगद

2) Papaya

पपीता 3) Bamboo

बास

4) Pomegranate

अनार

Correct Answer Bamboo

Q213 Acinonyx Jubatus is the scientific name of

एलसनह८ननकसजयबहॳटस mdashmdashmdash

कावहॴजञाननकनामहहॴ 3-Feb-2017

F A C E B O O K

P A G E h t t p w w w f a c e b o o k c o m s s c m e n t o r s o f f i c i a l P a g e | 49

FOR MORE UPDATES AND MORE MATERIAL DO LIKE OUR FACEBOOK PAGE httpwwwfacebookcomsscmentorsofficial

Options

1) Bear

भाि 2) Horse

घह८िा 3) Cheetah

चीता 4) Zebra

जहॳिा Correct Answer Cheetah

Q214 The pale yellow colour of urine is

due to the presence of which pigment

मतरकाफीकापीिारगरगदरयकहॳ उपनसिनतकहॳ कारणहह८ताहहॴ

3-Feb-2017

Options

1) Urochrome

यरह८िह८म

2) Urophyll

यरह८कफि

3) Chlorophyll

किह८रह८कफि

4) Chloroplast

किह८रह८पिासट

Correct Answer Urochrome

Q215 Which of the following constitute

to form a gene

नननननलिखितमसहॳकह९नसीचीज़एकजीनकागठनकरतीहहॴ

3-Feb-2017

Options

1) Polynucleotides

पह८िीनयनकियह८टाईडस

2) Hydrocarbons

हाइडरह८काबोस

3) Lipoproteins

िाईपह८परह८टीनस

4) Lipids

लिपपडस

Correct Answer Polynucleotides

Polynucleotide molecule is a biopolymer

composed of 13 or more nucleotide

monomers covalently bonded in a chain

DNA (deoxyribonucleic acid) and RNA

(ribonucleic acid) are examples of

polynucleotides with distinct biological

function

Q216 Vertebrates belongs to the

phylum

रीढ़कीहडिीवािहॳपराणी mdashmdashmdash

परजानतकहॳ अतगायतआतहॳहहॴ 3-Feb-2017

Options

1) Arthropoda

आरह८पह८ड़ा 2) Annelida

एननलििा 3) Cnidaria

ननिहॳररया 4) Chordata

कह८िटा Correct Answer Chordata

Q217 Punica granatum is the scientific name of

पननकगरहॳनहॳटस mdashmdashmdash कावहॴजञाननकनामहहॴ 3-Feb-2017

Options

1) Custard Apple

सीताफि

2) Gulmohar

गिमह८हर

3) Silver Oak

लसमवरओक

4) Pomegranate

अनार

Correct Answer Pomegranate

F A C E B O O K

P A G E h t t p w w w f a c e b o o k c o m s s c m e n t o r s o f f i c i a l P a g e | 50

FOR MORE UPDATES AND MORE MATERIAL DO LIKE OUR FACEBOOK PAGE httpwwwfacebookcomsscmentorsofficial

Q218 Between a tiger and an monkey

which of the following is different

एकबाघऔरबदरकहॳ बीचनननननलिखितमसहॳकह९नसीबातअिगहहॴ 3-Feb-2017

Options

1) Kingdom

राजय

2) Phylum

जानत

3) Order

िम

4) Class

वगय Correct Answer order

Q219 The artificial heart was invented by

कबतरमहदयका mdashmdashmdash

दवाराअपवषकारककयागयािा 3-Feb-2017

Options

1) Muhammad Yunus

महनमदयनस

2) Linus Yale Jr

िाइनसयहॳिजय

3) Gazi Yasargil

गाजीयासचगयि

4) Paul Winchell

पह९िपवमकि Correct Answer Paul Winchell

Q220 Tamarindus indica is the

scientific name of

टहॳमररनडसइडिका mdashmdash कावहॴजञाननकनामहहॴ 7-

Feb-2017

Options

1) Neem

नीम

2) Pineapple

अनानास

3) Tamarind

इमिी 4)Chiku

चीक

Correct Answer Tamarind

Q221 In eukaryotic cells synthesis of

RNA takes place in the

यकहॳ योटटककह८लिकाओमआरएनएकासशिहॳषण

mdashndash महह८ताहहॴ 7-Feb-2017

Options

1) Mitochondria

माईटह८कोडडरया 2) Centrioles

सटरीयह८मस

3) Ribosomes

ररबह८सह८नस

4) Nucleus

नयनकियस

Correct Answer nucleus

eukaryotic cell -Transcription is the

process of synthesizing ribonucleic acid

(RNA)Synthesis takes place within the

nucleus of eukaryotic cells or in the

cytoplasm of prokaryotes and converts

the genetic code from a gene in

deoxyribonucleic acid ( DNA ) to a

strand of RNA that then directs

proteinsynthesis

Q222 _________is caused by parasites

of the Plasmodium genus

पिाजमह८डियमजातीकहॳ परजीवी mdash- कहॳ कारणहहॴ 7-Feb-2017

Options

1) Dysentery

पहॳचचि

2) Malaria

मिहॳररया 3) Chickenpox

F A C E B O O K

P A G E h t t p w w w f a c e b o o k c o m s s c m e n t o r s o f f i c i a l P a g e | 51

FOR MORE UPDATES AND MORE MATERIAL DO LIKE OUR FACEBOOK PAGE httpwwwfacebookcomsscmentorsofficial

चहॳचक

4) Herpes

हहॳपपयस

Correct Answer Malaria

Q223 Carotene in fruits and vegetables

gives it which color

फिह८औरसनलजयोमनसितकहॳ रह८टीनउनहकह९नसारगपरदानकरताहहॴ 7-Feb-2017

Options

1) Green

हरा 2) Pink

गिाबी 3) Orange

नारगी 4) Blue

नीिा Correct Answer Orange

Q224 Equus Caballus is the scientific

name of

एकवसकहॴ बहॳिस mdashmdashndash कापवजञाननकनामहहॴ 7-Feb-2017

Options

1) Horse

घह८िा 2) Zebra

ज़हॳिा 3) Donkey

गधा 4) Buffalo

भस

Correct Answer Horse

Q225 Elapidae Naja is the scientific name of

एिीपीिीनाजा mdashmdash- कावहॴजञाननकनामहहॴ 8-Feb-2017

Options

1) Cobra

कह८बरा 2) Elephant

हािी 3) Eagle

ग ि

4) Owl

उमि Correct Answer Cobra

Q226 Which disease is caused due to

deficiency of Iron

िह८हकीकमीकहॳ कारणकह९नसारह८गहह८ताहहॴ 8-Feb-

2017

Options

1) Beriberi

बहॳरीबहॳरी 2) Tetany

टहॳटनी 3) Kwashiorkor

कवािीऔरकर

4) Anaemia

रकतामपता Correct Answer Anaemia

Beriberi is a disease caused by a vitamin

B-1 deficiency also known as thiamine

deficiency

Tetany can be the result of an

electrolyte imbalance Most often itrsquos a

dramatically low calcium level also

known as hypocalcemia Tetany can also

be caused by magnesium deficiency or

too little potassium Having too much

acid (acidosis) or too much alkali

(alkalosis) in the body can also result in

tetany

Kwashiorkor also known as

ldquoedematous malnutrition It is a form of

malnutrition caused by a lack of protein

in the diet

Anaemia means that you have fewer red

blood cells than normal or you have less

F A C E B O O K

P A G E h t t p w w w f a c e b o o k c o m s s c m e n t o r s o f f i c i a l P a g e | 52

FOR MORE UPDATES AND MORE MATERIAL DO LIKE OUR FACEBOOK PAGE httpwwwfacebookcomsscmentorsofficial

haemoglobin than normal in each red

blood cell

Q227 is a leaf where the leaflets are

arranged along the middle vein

mdashndashएकपततीहहॴजहापतरकह८कीरचनाक ररयालिराकहॳ आसपासहह८तीहहॴ 8-Feb-2017

Options

1) Pinnately compound leaf

पपनहॳटिीसयकतपतती 2) Palmately compound leaf

पामहॳटिीसयकतपतती 3) Compound leaf

सयकतपतती 4) Simple leaf

साधारणपतती Correct Answer Pinnately compound

leaf

Q228 Haustoria or sucking roots are

found in which of the following

हह८सटह८ररयायाचसनहॳवािीजड़हॳनननननलिखितमसहॳककसमपाईजातीहहॴ 8-Feb-2017

Options

1) Wheat

गहॳह

2) Mango

आम

3) Chestnut

चहॳसटनट

4) Cuscuta

कसकयटा Correct Answer Cuscuta

Haustorial roots -The roots of parasitic

plants which penetrate into the host

tissues to absorb nourishment are

called haustorial roots hellip Also known as suckingor parasitic roots

Q229 Equs Asinus is the scientific name

of

एकवसएलसनस mdashmdashndash कावहॴजञाननकनामहहॴ 8-

Feb-2017

Options

1) Donkey

गधा 2) Cow

गाय

3) Deer

टहरन

4) Kangaroo

कगा

Correct Answer Donkey

Q230 Ficus benghalensis is the scientific name of

फाईकसबहॳनगहॳिहॳलसस mdashndash कापवजञाननकनामहहॴ 8-Feb-2017

Options

1) Banyan

बरगद

2) Pineapple

अनानास

3) Babul

बबि

4) Tulsi

तिसी Correct Answer Banyan

Q231 Equus burchellii is the scientific name of

एकवसबचिी mdashmdash- कापवजञाननकनामहहॴ 8-Feb-2017

Options

1) Horse

घह८िा 2) Zebra

जहॳिा 3) Buffalo

F A C E B O O K

P A G E h t t p w w w f a c e b o o k c o m s s c m e n t o r s o f f i c i a l P a g e | 53

FOR MORE UPDATES AND MORE MATERIAL DO LIKE OUR FACEBOOK PAGE httpwwwfacebookcomsscmentorsofficial

भस

4) Ass

गधा Correct Answer Zebra

Page 5: COMPILATION OF ALL 72 SETS OF BIOLOGY SSC CHSL-2016 · OF BIOLOGY SSC CHSL-2016 PREPARED BY : SSC MENTORS BIOLOGY SPECIAL . F A C E B O O K P A G E : h t t p : / / w w w . f a c e

F A C E B O O K

P A G E h t t p w w w f a c e b o o k c o m s s c m e n t o r s o f f i c i a l P a g e | 4

FOR MORE UPDATES AND MORE MATERIAL DO LIKE OUR FACEBOOK PAGE httpwwwfacebookcomsscmentorsofficial

पहॳड़ह८मउजायउतपननकरनहॳकीपरनतकियाकह८ mdashmdashndash

कहॳ नामजाताहहॴ 8-Jan -2017

Options

1) Absorption

अविह८षण

2) Reduction

अवकरण

3) Photosynthesis

परकािसशरिहॳषण

4) Transpiration

वाषपीकरण

Correct Answer Photosynthesis

Q10 Which Virus causes Chicken Pox

ककसवायरसकहॳ कारणचहॳचकहह८ताहहॴ

8-Jan -2017

Options

1) Rubella Virus

बहॳिावायरस

2) Herpes Zoster Virus

हपपरयसजह८सटरवायरस

3) Rabies

रहॳबीज़

4) Variola Virus

वहॳरीओिावायरस

Correct Answer Herpes Zoster Virus

Rubella also known as German

measles or

three ndashday measles is an infection

caused by the rubella virus

Smallpox is caused by infection with

variola

Virus

Q11 What is the total number of bones

in the human body

मानविरीरमकिककतनीहडडियहह८तीहहॴ

8-Jan -2017

Options

1)206

2)103

3)309

4)412

Correct Answer 206

Q12 Emblica officinalis is the scientific

name of

एननलिकाओफीलसनहॳलिस (Emblica

offcinalis)mdashmdash- कावहॴजञाननकनामहहॴ 8-Jan -2017

Options

1) Peepal

पीपि

2) Mango

आम

3) Amla

आविा 4) Drumstick

सहजन

Correct Answer Amla

Q13 Sponges belongs to the phylum

सपजककसपरजानतकहॳ अतगयतआतहहॴ

8-Jan -2017

Options

1) Protozoa

परह८टह८जआ

2) Annelida

एननलििा 3) Porifera

पह८ररफहॳ रा 4) Cnidaria

ननिहॳररया Correct Answer Porifera

Q14 Which of the following is a

symptom of haemophilia

नननननलिखितमसहॳटहमह८फीलियाकािकषणकह९नसा हहॴ

9-Jan -2017

Options

F A C E B O O K

P A G E h t t p w w w f a c e b o o k c o m s s c m e n t o r s o f f i c i a l P a g e | 5

FOR MORE UPDATES AND MORE MATERIAL DO LIKE OUR FACEBOOK PAGE httpwwwfacebookcomsscmentorsofficial

1) Night Blindness

रतोधी 2) No clotting of Blood

रकतकािककानजमना 3) Rickets

ररकहॳ ट

4) Loss of haemoglobin

टहमह८गिह८बबनकीअमपता Correct Answer No clotting of Blood

Haemophilia also spelled hemophilia is

a mostly inherited genetic disorder that

impairs the bodyrsquos ability to make blood

clots a process needed to stop bleeding

This results in people bleeding longer

after an injury easy bruising and an

increased risk of bleeding inside joints

or the brain

Q15 The process of pollination by birds

is also known as

पकषकषयोदवाराकीजानहॳवािीपरागणकीपरनतकियाकह८mdashndash कहॳ नामसहॳभीजानाजाताहहॴ 9-Jan -2017

Options

1) Hydrophily

हाइडरह८कफिी 2) Entomophily

एनटोमह८कफिी 3) Embryophily

एननियह८कफिी 4) Ornithophily

ओननयिह८कफिी Correct Answer Ornithophily

Q16 Spiders belong to the phylum

मकडड़याककसपरजानतकहॳ अतगयतआतीहहॴ 9-Jan -2017

Options

1) Mollusca

मह८िसका 2) Annelida

एननलििा 3) Cnidaria

ननिहॳररया 4) Arthropoda

अरोपह८ड़ा Correct Answer Arthropoda

Q17 Banana freckle is a plant disease

It is caused by a

कहॳ िहॳकीझाईपह८धह८कीएकबीमारीहहॴ यहएक mdashndash

कहॳ कारणहह८तीहहॴ 9-Jan -2017

Options

1) Virus

वायरस

2) Fungus

कवक

3) Bacteria

बहॴकटीररया 4) Insect

कीटक

Correct Answer Fungus

Banana Freckle is a disease caused by

the fungus Guignardia musae

(telomorph) or Phyllosticta musarum (

anamorph )

Q18 Which of the following Indian

chilly is considered one of the hottest in

the world

नननननलिखितभारतीयलमचचययह८मसहॳकह९नसीसबसहॳकह९नसीपवशवकीसबसहॳतीिीलमचचययह८मसहॳएकमानीजातीहहॴ

9-Jan -2017

Options

1) Bhut Jolokia

भतझह८िककया 2) Bhut Mahabora

भतमहाबह८रा 3) Lal Chitin

F A C E B O O K

P A G E h t t p w w w f a c e b o o k c o m s s c m e n t o r s o f f i c i a l P a g e | 6

FOR MORE UPDATES AND MORE MATERIAL DO LIKE OUR FACEBOOK PAGE httpwwwfacebookcomsscmentorsofficial

िािचीटटन

4) Lal Shamak

िाििामक

Correct Answer Bhut Jolokia

Q19 Brain fever is a disease spread

through which of the following

मनसतषकजवरनमकरह८गनननननलिखितमसहॳककसकहॳ कारणहह८ताहहॴ 9-Jan -2017

Options

1) Flies

मनकियो 2) Mosquito

मचछर

3) Virus

वायरस

4) Cockroach

नतिच हॳ Correct Answer Mosquito

Q20 Mangroves are plants that have

मगरह८ववहॳपहॳिहहॴनजनमहॳहह८ताहहॴ 9-Jan -2017

Options

1) Modified Roots

पातररतजड़हॳ 2) Modified Stems

पातररततनहॳ 3) Respiratory Roots

शरवसनकरनहॳवािीजड़हॳ 4) Respiratory Stems

शरवसनकरनहॳवािीतनहॳ Correct Answer Respiratory Roots

A mangrove is a shrub or small tree that

grows in coastal saline or brackish

water

Q21 Rodentia Sciurus is the scientific

name of

रह८िहॳलियासकीयरस mdashmdash कावयजजञाननकनामहहॴ

9-Jan -2017

Options

1) Rat

चहा 2) Platypus

पिहॳटीपस

3) Squirrel

चगिहरी 4) Beaver

बीवर

Correct Answer Squirrel

Q22 Which of the following is induced

by Oncogene

नननननलिखितमसहॳकह९नओकह८जीनदवारापरहॳररयतहह८ताहहॴ

10-Jan -2017

Options

1) Polio

पह८लियह८ 2) Cancer

क सर

3) Diarrhoea

दसत

4) Dengue

िग Correct Answer Cancer

An oncogene is a gene that has the

potential to cause cancer In tumor

cells they are often mutated andor

expressed at high levels

Q23 Azadirachata indica is the

scientific name of

अजाटदराकटाइडिका mdashmdashवहॴजञाननकनामहहॴ SSC CHSL Science (biology) 2016

Question Paper

10-Jan -2017

Options

1) Neem

नीम

F A C E B O O K

P A G E h t t p w w w f a c e b o o k c o m s s c m e n t o r s o f f i c i a l P a g e | 7

FOR MORE UPDATES AND MORE MATERIAL DO LIKE OUR FACEBOOK PAGE httpwwwfacebookcomsscmentorsofficial

2) Teak

सागह९न

3) Silver Oak

लसमवरओक

4) Tulsi

तिसी Correct Answer Neem

Q24 Octopus belongs to the phylum

ऑकटह८पसककसपरजानतकहॳ अतगयतआताहहॴ 10-

Jan -2017

Options

1) Mollusca

मह८िसका 2) Cnidaria

ननिहॳररया 3) Echinodermata

इकाइनह८ड़हॳमता 4) Chordata

कह८िता Correct Answer Mollusca

Q25 A living part of the organisms

environment is known as

जीवाणकहॳ वातावरणकहॳ जीपवतभागकह८ mdash-

कहतहॳहहॴ 10-Jan -2017

Options

1) Abiotic Factor

अजहॴपवककारक

2) Habitat

आवास

3) Biotic Factor

जहॴपवककारक

4) Nonliving factor

अ-जीपवतकारक

Correct Answer Biotic Factor

Abiotic factors are nonndash living chemical

and physical parts of the environment

that affect living organisms and the

functioning of ecosystems like rain

wind temperature altitude soil

pollution nutrients pH types of soil

and sunlight

Q26 Medulla oblongata is a part of

which of the following

महॳडयिाऑबिॉनगहॳटानननननलिखितमसहॳककसअगकाटहससाहहॴ

10-Jan -2017

Options

1) Heart

हदय

2) Brain

मनसतषक

3) Lungs

फहॳ फड़हॳ 4) Stomach

पहॳट

Correct Answer Brain

The medulla oblongata helps regulate

breathing heart and blood vessel

function digestion sneezing and

swallowing This part of the brain is a

center for respiration and circulation

Sensory and motor neurons (nerve cells)

from the forebrain and midbrain travel

through the medulla

Q27 ___________ is a typically

onecelled reproductive unit capable of

giving rise to a new individual without

sexual fusion

mdashmdash एकआमतह९रपरएककह८लिकीयहॳ परजननममसमकषइकाईहहॴजह८यह९नसियनकहॳ बबनाएकनयीइकाईकह८जनमदहॳतीहहॴ 10-Jan -2017

Options

1) Egg

अिाण

2) Spore

बीजाण

F A C E B O O K

P A G E h t t p w w w f a c e b o o k c o m s s c m e n t o r s o f f i c i a l P a g e | 8

FOR MORE UPDATES AND MORE MATERIAL DO LIKE OUR FACEBOOK PAGE httpwwwfacebookcomsscmentorsofficial

3) Sperm

ििाण

4) Seed

बीज

Correct Answer Spore

Q28 Bacteria was discovered by

बहॴकटीररयाकीिह८जककसकहॳ दवाराकीगयीिी

10-Jan -2017

1) Antonie van Leeuwenhoek

एटह८नीवहॳनलिबहॳनहक

2) Belarus

बहॳिा स

3) Hugo de Vries

हयगह८दीराईस

4)Robert Brown

रॉबटयिाउन

Correct Answer Antonie van

Leeuwenhoek

Q29 Which of the following is

responsible for Vermicomposting

नननननलिखितमसहॳकह९नकलमिादकहॳ लिएनजनमहॳदारहहॴ

10-Jan -2017

Options

1) Fungus

कवक

2) Worms

कलम

3) Bacteria

बहॴकटीररया 4) Birds

पकषी Correct Answer Worms

Vermicompost (or vermi-compost) is the

product of the composting process using

various species of worms usually red

wigglers white worms and other

earthworms to create a heterogeneous

mixture of decomposing vegetable or

food waste bedding materials and

vermicast

Q30 Scurvy (bleeding of gums) is

caused by the deficiency of which

vitamin

सकवी (मसढह८सहॳिनआना) ककसपवटालमनकीकमीकहॳ कारणहह८ताहहॴ

10-Jan-2017

Options

1) Vitamin K

पवटालमन K

2) Vitamin BZ

पवटालमन BZ

3) Vitamin C

पवटालमन C

4) Vitamin A

पवटालमन A

Correct Answer Vitamin C

Q31 Achras sapota is the scientific

name of

एिाससपह८ताइसकावहॴजञाननकनामहहॴ 10-Jan-2017

Options

1) Custard Apple

सीताफि

2) Gulmohar

गिमह८हर

3) Tamarind

इमिी 4) Chiku

चचक

Correct Answer Chiku

Q32 Prawn belongs to the phylum

झीगा mdashmdash- परजानतकहॳ अतगयतआताहहॴ 10-Jan-2017

Options

1) Arthropoda

F A C E B O O K

P A G E h t t p w w w f a c e b o o k c o m s s c m e n t o r s o f f i c i a l P a g e | 9

FOR MORE UPDATES AND MORE MATERIAL DO LIKE OUR FACEBOOK PAGE httpwwwfacebookcomsscmentorsofficial

अरोपह८िा 2) Cnidaria

नीिहॳररया 3) Echinodermata

इकाईनह८िमटा 4) Chordata

कह८िटा Correct Answer Arthropoda

Q33 Pulses are a rich source of which of

the following

दािहॳनननननलिखितमसहॳककसकीपरचरसह८तरहहॴ

11-Jan-2017

Options

1) Carbohydrates

काबोहाइडराईट

2) Proteins

परह८टीनस

3) Minerals

िननज

4) Vitamin A

पवटालमन A

Correct Answer Proteins

Q34 Plant cell wall is made up of

वनसपनतकह८लिकालभनततइससहॳबनीहह८तीहहॴ

11-Jan-2017

Options

1) Cellulose

सहॳमयिह८ज

2) Glucose

गिकह८ज

3) Sucrose

सिह८ज

4) Fructose

फरकटह८ज

Correct Answer Cellulose

Plant cell wall the major carbohydrates

are cellulose hemicellulose and pectin

The cellulose microfibrils are linked via

hemicellulosic tethers to form the

cellulose-hemicellulose network which

is embedded in the pectin matrix

Q35 The study of Fungi is also known

as कवकह८कहॳ अधययनकह८कहाजाताहहॴ

11-Jan-2017

Options

1) Cytology

सायटह८िह८जी 2) Myology

मायह८िह८जी 3) Mycology

मायकह८िह८जी 4) Neurology

नयरह८िह८जी Correct Answer Mycology

Cytology - structure and function of

plant and animal cells

Myology is the study of the muscular

system

Neurology is the branch of medicine

concerned with the study and treatment

of disorders of the nervous system

Q36 The outermost layer of skin is

तवचाकीसबसहॳबाहरीपरतकयाहह८तीहहॴ 11-Jan-

2017

Options

1) Epidermis

इपपिलमयस

2) Dermis

िलमयस

3) Tissues

ऊतक

4) Hypodermis

हायपह८िलमयस

Correct Answer Epidermis

Q37 Which of the following plants have

root nodules

F A C E B O O K

P A G E h t t p w w w f a c e b o o k c o m s s c m e n t o r s o f f i c i a l P a g e | 10

FOR MORE UPDATES AND MORE MATERIAL DO LIKE OUR FACEBOOK PAGE httpwwwfacebookcomsscmentorsofficial

नननननलिखितपह९धह८मसहॳककसकीजड़ह८मगाठहह८तीहहॴ

11-Jan-2017

Options

1) Leguminous plants

िहॳगयलमनसपह९धहॳ 2) Parasitic plants

परजीवीपह९धहॳ 3) Epiphytic Plants

एपीफाइटटकपह९धहॳ 4) Aquatic Plants

जिीयपह९धहॳ Correct Answer Leguminous plants

Q38 Earth-worms belongs to the

phylum

कहॳ चएmdashmdash- परजानतकहॳ अतगयतआतहॳहहॴ 11-Jan-2017

Options

1) Protozoa

परह८टह८जआ

2) Cnidaria

नीिहॳररया 3) Annelida

एनीलििा 4) Mollusca

मह८िसका Correct Answer Annelida

Q39 Ringworm is a disease caused by

ररगवमयनामकबीमारी mdashmdash- कहॳ कारणहह८तीहहॴ 11-Jan-2017

Options

1) Fungi

कवक

2) Bacteria

बहॴकटीररया 3) Virus

वायरस

4) Flies

मनकियाा Correct Answer Fungi

Q40 Mangifera indica is the scientific

name of

मननगफहॳ राइडिकाककसकावहॴजञाननकनामहहॴ 11-

Jan-2017

Options

1) Guava

अम द

2) Mango

आम

3) Amla

आविा 4) Jack fruit

कटहि

Correct Answer Mango

Q41 Crabs belongs to the phylum

कहॳ कड़हॳmdashmdash- परजानतकहॳ अतगयतआतहॳहहॴ 11-Jan-2017

Options

1) Mollusca

मह८िसका 2) Cnidaria

नीिहॳररया 3) Arthropoda

अरोपह८ड़ा 4) Platyhelminthes

पिहॳटटहहॳनममननिस

Correct Answer Arthropoda

Q42 Myopia is a defect of eyes which is

also known as

मायह८पपयाआिोकादह८षहहॴ नजसहॳ mdashmdashndash

भीकहाजाताहहॴ

12-Jan-2017

Options

1) Far Sightedness

F A C E B O O K

P A G E h t t p w w w f a c e b o o k c o m s s c m e n t o r s o f f i c i a l P a g e | 11

FOR MORE UPDATES AND MORE MATERIAL DO LIKE OUR FACEBOOK PAGE httpwwwfacebookcomsscmentorsofficial

दरदनषटदह८ष

2) Near Sightedness

ननकटदनषटदह८ष

3) Astigmatism

एसटीगमहॳटटजम

4) Night Blindness

रतोधी Correct Answer Near Sightedness

Myopia occurs when the eyeball is too

long relative to the focusing power of

the cornea and lens of the eye This

causes light rays to focus at a point in

front of the retina rather than directly

on its surface

Hyperopia Hypermetropia (

Farsightedness )- when light rays

entering the eye focus behind the retina

rather than directly on it The eyeball of

a farsighted person is shorter than

normal

Astigmatism usually is caused by an

irregularly shaped cornea Instead of

the cornea having a symmetrically

round shape (like a baseball) it is

shaped more like an American football

Nyctalopia also called night ndash blindness

is a condition making it difficult or

impossible to see in relatively low light

Q43 Who is known as the father of

Green Revolution

हररतिानतकहॳ जनककहॳ पमककसहॳजानाजाताहहॴ

12-Jan-2017

1) Dr Robert Nucleus

िॉ रॉबटयनयनकियस

2) Dr Ian Wilmut

िॉ इयानपविमट

3) Dr NE Borlaug

िॉ एनईबह८रिॉग

4) Dr JC Bose

िॉ जहॳसीबह८स

Correct Answer Dr NE Borlaug

Q44 Panthera Tigris is the scientific

name of

पिहॳराटटगरीस mdashmdashmdash कावहॴजञाननकनामहहॴ 12-Jan-2017

Options

1) Panther

तदआ

2) Tiger

बाघ

3) Whale

हहॳि

4)Goat

बकरी Correct Answer Tiger

Q45 How many facial bones are there

हमारहॳचहॳहरहॳमककतनीहडडियााहह८तीहहॴ 13-Jan-2017

Options

1)34

2)24

3)14

4)4

Correct Answer 14

Q46 ndash Halophytes are plants that grow

in

हहॴिह८फाईटसवहॳपह९धहॳहह८तीहहॴजह८ mdash- मउगतहॳहहॴ SSC CHSL Science (biology) 2016

Question Paper

13-Jan-2017

Options

1) Fresh Water

ताजापानी 2) Cold Water

ठिापानी 3) Ponds

तािाब

4) Salt Water

िारापानी Correct Answer Salt Water

F A C E B O O K

P A G E h t t p w w w f a c e b o o k c o m s s c m e n t o r s o f f i c i a l P a g e | 12

FOR MORE UPDATES AND MORE MATERIAL DO LIKE OUR FACEBOOK PAGE httpwwwfacebookcomsscmentorsofficial

Q47 Felis Catus is the scientific name of

फहॳ लिसकहॴ टस mdashndash कावहॴजञाननकनामहहॴ 13-Jan-2017

Options

1) Cat

बबमिी 2) Dog

कतता 3) Mouse

चहा 4) Porcupine

साही Correct Answer Cat

Q48 Which of the following induces

nitrogen fixation in soil

नननननलिखितमसहॳकह९नलम ीमनाइटरह८जनननयतनकह८परहॳररतकरताहहॴ

15-Jan-2017

Options

1) Protozoa

परह८टह८जआ

2) Bacteria

बहॴकटीररया 3) Fungi

कवक

4)Algae

िहॴवाि

Correct Answer Bacteria

Bacteria that change nitrogen gas from

the atmosphere into solid nitrogen

usable by plants are called nitrogen-

fixing bacteria These bacteria are

found both in the soil and in symbiotic

relationships with plants

They contain symbiotic bacteria called

rhizobia within nodules in their root

systems producing nitrogen compounds

that help the plant to grow and compete

with other plants When the plant dies

the fixed nitrogen is released making it

available to other plant

Q49 Which of the following is the

largest known cell

नननननलिखितमसहॳकह९नसीसबसहॳबड़ीजञातकह८लिकाहहॴ

SSC CHSL Science (biology) 2016

Question Paper

15-Jan-2017

1) Eukaryotic Cell

यकहॳ ररयह८टटककह८लिका 2) Prokaryotic Cell

परह८कहॳ ररयह८टटककह८लिका 3) Mycoplasma

मायकह८पिासम

4) Ostrich Eggs

ितरमगयकाअिा Correct Answer Ostrich Eggs

Q50 The association of animals in

which both the partners are benefitted

is known as

जानवरोकावहसहयह८गनजसमहॳदह८नोभागीदारिाभापवनतहह८तहॳहहॴ उसहॳ mdashmdashndash कहॳ पमजानाजाताहहॴ SSC CHSL Science (biology) 2016

Question Paper

15-Jan-2017

Options

1) Amensalism

सहजीपवत

2) Commensalism

परजीपवत

3) Colony

कॉिनी 4) Mutualism

अनयह८नयाशरयवाद

Correct Answer Mutualism

Amensalism is any relationship between

organisms of different species in which

F A C E B O O K

P A G E h t t p w w w f a c e b o o k c o m s s c m e n t o r s o f f i c i a l P a g e | 13

FOR MORE UPDATES AND MORE MATERIAL DO LIKE OUR FACEBOOK PAGE httpwwwfacebookcomsscmentorsofficial

one organism is inhibited or destroyed

while the other organism remains

unaffected

Commensalism an association between

two organisms in which one benefits and

the other derives neither benefit nor

harm

Q51 Pneumonia affects which of the

following organs of human body

ननमह८ननयामानविरीरकहॳ नननननलिखितमसहॳककसअगकह८परभापवतकरताहहॴ

15-Jan-2017

Options

1)Kidneys

गद

2)Lungs

फहॳ फड़हॳ 3) Throat

गिहॳ 4) Liver

यकत

Correct Answer Lungs

When the germs that cause pneumonia

reach your lungs the lungsrsquo air sacs

(alveoli) become inflamed and fill up

with fluid This causes the symptoms of

pneumonia such as a cough fever

chills and trouble breathing When you

have pneumonia oxygen may have

trouble reaching your blood

Q52 Mendel is known as

मििकह८ mdashmdash- कहॳ पमजानाजाताहहॴ 15-Jan-2017

Options

1) Father of Physiology

िरीरकियािासतरकहॳ जनक

2) Father of Geology

भगभयिासतरकहॳ जनक

3) Father of Genetics

जहॳनहॳटटकसकहॳ जनक

4) Father of Biology

जीविासतरकहॳ जनक

Correct Answer Father of Genetics

Q53 Which of the following are also

known as Suicidal bag of Cells

ननननलिखितमसहॳककसहॳआतमहतयाकरनहॳवािीकह८लिकाओकाबहॴगकहाजाताहहॴ

15-Jan-2017

Options

1) Lysosomes

िायसोसह८म

2) Lycosome

िायकह८सह८म

3) Nucleus

नालभक

4) Chromosome

िह८मह८सह८म

Correct Answer Lysosomes

Q54 Mesothelioma is a type of cancer

The most common area affected in it is

the lining of the ________

लमज़ह८िहॳिहॳलमयाक सरकाएकपरकारहहॴ इससहॳपरभापवतहह८नहॳवािासबसहॳसामानयकषहॳतर mdash

mdashmdash काअसतरहहॴ 15-Jan-2017

Options

1)Heart

हदय

2)Brain

मनसतषक

3)Stomach

आमािय

4)Lungs

फहॳ फड़हॳ Correct Answer lungs

Asbestos exposure is the main cause of

pleural mesothelioma When asbestos

fibers are breathed in they travel to the

F A C E B O O K

P A G E h t t p w w w f a c e b o o k c o m s s c m e n t o r s o f f i c i a l P a g e | 14

FOR MORE UPDATES AND MORE MATERIAL DO LIKE OUR FACEBOOK PAGE httpwwwfacebookcomsscmentorsofficial

ends of small air passages and reach the

pleura where they can cause

inflammation and

scarring

Q55 Which one of the following is an

insectivorous plant

नननननलिखितमसहॳकह९नसाएकककटाहरीवनसपनतहहॴ

15-Jan-2017

Options

1) Utricularia

यटरीकिहॳररया 2) Sequoia

सहॳकयओइया 3) Nostoc

नॉसटह८क

4) Bryophyta

िायह८फाईटा Correct Answer Utricularia

Q56 ______________ is a

multibranched polysaccharide of

glucose that serves as a form of energy

storage in animals and fungi

mdashmdashगिकह८जकाएकबहिािायकतपह८िीसहॳकहॳ राइिहहॴ जह८जानवरोऔरकवकमउजायभणिारणकहॳ एक पमकाययकरताहहॴ 15-Jan-2017

Options

1) Cellulose

सहॳमयिह८ज

2) Glycogen

गिायकह८जन

3) Pectin

पहॳनकटन

4) Chitin

चीटटन

Correct Answer Glycogen

Q57 The largest gland of the human

body is

mdashmdashmdashमानविरीरकीसबसहॳबड़ीगरिीहहॴ 16-Jan-2017

Options

1) Pancreas

अगयािय

2) Thyroid

िायरॉइि

3) Large Intestine

बड़ीआत

4) Liver

यकत

Correct Answer Liver

Q58 Photosynthesis in plants takes

place in

वनसपनतयोमपरकािसशिहॳषणकीकियाहह८तीहहॴ

16-Jan-2017

Options

1) Stem

तना 2) Leaves

पनततयाा 3) Roots

जड़हॳ 4) Flower

फि

Correct Answer Leaves

During this reaction carbon dioxide

and water are converted into glucose

and oxygen The reaction requires light

energy which is absorbed by a green

substance called

chlorophyll Photosynthesis takes place

in leaf

cells These contain chloroplasts which

are tiny objects containing chlorophyll

F A C E B O O K

P A G E h t t p w w w f a c e b o o k c o m s s c m e n t o r s o f f i c i a l P a g e | 15

FOR MORE UPDATES AND MORE MATERIAL DO LIKE OUR FACEBOOK PAGE httpwwwfacebookcomsscmentorsofficial

Q59 Insects that transmit diseases are

known as

जह८कीड़हॳरह८गसचाररतकरतहॳहहॴ उनह mdashmdash-

कहॳ नामसहॳजानाजाताहहॴ 16-Jan-2017

1)Pathogens

रह८गज़नक

2) Vectors

वहॳकटर

3) Drones

परजीवी 4)Scalars

अटदषट

Correct Answer Vectors

A vector is an organism that does not

cause disease itself but which spreads

infection by conveying pathogens from

one host to another Species of mosquito

for example serve as vectors for the

deadly disease Malaria

Q60 Which is the second largest gland

of Human body

मानविरीरकीदसरीसबसहॳबड़ीगरिीकह९नसीहहॴ

SSC CHSL Science (biology)

2016 Question Paper

16-Jan-2017

Options

1) Liver

यकत

2) Large Intestine

बड़ीआत

3) Thorax

छाती 4) Pancreas

अगनयािय

Correct Answer Pancreas

Q61 Annona squamosa is the scientific

name of

एनह८नासकवामह८सा (Annona squamosa) mdash

mdashmdash कावहॴजञाननकनामहहॴ 16-Jan-2017

Options

1) Custard Apple

सीताफि

2) Papaya

पपीता 3) Babhul

बबि

4) Drumstick

सहजन

Correct Answer Custard Apple

Q62 The disease Beri Beri is caused due

to the deficiency of which of the

following

बहॳरीबहॳरीरह८गनननननलिखितमसहॳककसकीकमीकहॳकारणहह८ताहहॴ

16-Jan-2017

Options

1) Vitamin B2

पवटालमन B2

2) Vitamin B1

पवटालमन B1

3) Vitamin B12

पवटालमन B12

4) Vitamin E

पवटालमन E

Correct Answer Vitamin B1

Beriberi is a disease caused by a vitamin

B-1 deficiency also known as thiamine

deficiency

Q63 Chlorophyll was first isolated and

named by

किह८रह८कफिकह८ mdash-

दवारापहिहॳपिकऔरनालमतककयागया 16-Jan-2017

F A C E B O O K

P A G E h t t p w w w f a c e b o o k c o m s s c m e n t o r s o f f i c i a l P a g e | 16

FOR MORE UPDATES AND MORE MATERIAL DO LIKE OUR FACEBOOK PAGE httpwwwfacebookcomsscmentorsofficial

Options

1) Caventou

कहॳ वहॳत 2) Pelletier

पहॳिहॳटटयर

3) Chlorophyll

किह८रह८कफि

4) Caventou and Pelletier

कहॳ वहॳतऔरपहॳिहॳटटयर

Correct Answer Caventou and Pelletier

Chlorophyll was first isolated and

named by

Joseph Bienaimeacute Caventou and Pierre

Joseph Pelletier in 1817 The presence of

magnesium in chlorophyll was

discovered in 1906 and was the first

time that magnesium had been detected

in living tissue

Q64 Which of the following organisms

does not fit into the Cell Theory

नननननलिखितमसहॳकह९नसाजीवकह८लिकालसदातअन पनहीहहॴ

16-Jan-2017

Options

1) Bacteria

बहॴकटीररया 2) Virus

वायरस

3) Fungi

कवक

4) Plants

पह९धहॳ Correct Answer Virus

The bottom line is that viruses are not

alive and not related to cells in any way

The cell theory states that all living

things are made of cells cells are the

basic units of structure and function of

living things and that all cells come

from other cells Since viruses are not

made of cells and do not use cells in any

of their processes they are not related to

the cell theory

Q65 Which of these is not a

macronutrient for Plants

नननननलिखितमसहॳकह९नसापह९धह८कहॳ लिएमिह८नयटरीएटनहीहहॴ

SSC CHSL Science (biology) 2016

Question Paper

17-Jan-2017

Options

1) Nitrogen

नाइटरह८जन

2) Phosphorus

फासफह८रस

3) Potassium

पह८टालसयम

4) Chlorine

किह८रीन

Correct Answer Chlorine

In relatively large amounts the soil

supplies nitrogen phosphorus

potassium calcium magnesium and

sulfur these are often called the

macronutrients In relatively small

amounts the soil supplies iron

manganese boron molybdenum

copper zinc chlorine and cobalt the

so-called micronutrients

Q66 Name the respiratory organs of

insects

कीटह८मनसतिशरवसनअगनामकानामहहॴ

17-Jan-2017

Options

1) Skin

तवचा 2) Body Surface

िरीरकीसतह

F A C E B O O K

P A G E h t t p w w w f a c e b o o k c o m s s c m e n t o r s o f f i c i a l P a g e | 17

FOR MORE UPDATES AND MORE MATERIAL DO LIKE OUR FACEBOOK PAGE httpwwwfacebookcomsscmentorsofficial

3) Gills

गिफड़हॳ 4) Tracheae

शरावस- निी Correct Answer Tracheae

Air enters the respiratory systems of

insects through a series of external

openings called

spiracles These external openings

which act as muscular valves in some

insects lead to the internal respiratory

system a densely networked array of

tubes called tracheae

Q67 The poisonous gas accidentally

released in Bhopal Gas Tragedy is

भह८पािगहॴसतरासदीमगितीसहॳमकतहईजहरीिीगहॴसिी

17-Jan-2017

1) Methane

मीिहॳन

2) Nitrous Oxide

नाइटरसऑकसाइि

3) Methyl Isocyanate

महॴचििआयसोसायनहॳट

4) Cyanogen

सायनह८जहॳन

Correct Answer Methyl Isocyanate

Q68 What does Trypsin do

टटरनपसनकयाकरताहहॴ

SSC CHSL Science (biology) 2016

Question Paper

17-Jan-2017

Options

1) Breaks down Carbohydrates

काबोहाइडरहॳटकापवघटनकरताहहॴ 2) Synthesizes proteins

परह८टीनकासििहॳषणकरताहहॴ 3) Breaks down fats

वसाकापवघटनकरताहहॴ 4) Breaks down proteins

परह८टीनकापवघटनकरताहहॴ Correct Answer Breaks down proteins

Trypsin is one of the three principal

digestive

proteinases the other two being pepsin

and

chymotrypsin In the digestive process

trypsin acts with the other proteinases

to break down dietary protein molecules

to their component

peptides and amino acids

A protease is any enzyme that performs

proteolysis protein catabolism by

hydrolysis of peptide bonds

Q69 Name the source from which

Aspirin is produced

उससरह८तकानामबताइए

नजससहॳएनसपररनकाउतपादनककयाजाताहहॴ

17-Jan-2017

Options

1) Willow bark

पविह८कीछाि

2) Oak Tree

ओककावकष

3) Acacia

बबि

4) Eucalyptus

नीिचगरी Correct Answer Willow bark

The compound from which the active

ingredient in aspirin was first derived

salicylic acid was found in the bark of a

willow tree in 1763 by Reverend

Edmund Stone of Chipping-Norton

Q70 Cannis Familiaris is the scientific

name of

कहॴ ननसफहॳ लमलियहॳररस mdash- कावहॴजञाननकनामहहॴ

17-Jan-2017

F A C E B O O K

P A G E h t t p w w w f a c e b o o k c o m s s c m e n t o r s o f f i c i a l P a g e | 18

FOR MORE UPDATES AND MORE MATERIAL DO LIKE OUR FACEBOOK PAGE httpwwwfacebookcomsscmentorsofficial

Options

1) Cat

बबमिी 2)Dog

कतता 3) Fox

िह८मड़ी 4) Wolf

भहॳडड़या Correct Answer Dog

Q71 Harmful bacteria in potable water

make the water

पीनहॳकहॳ पानीमनसतिघातकबहॴकटीररयाउसपानीकह८बनातहॳहहॴ 17-Jan-2017

Options

1) unfit to drink

पीनहॳकहॳ लिएअयह८गय

2) smelly

दगयनधयकत

3) Colored

रगीन

4) Turbid

मटमहॴिा Correct Answer unfit to drink

Q72 Musa paradisiaca is the scientific

name of which plant

मसापहॴराडिलसयाकाककसपह९धहॳकावहॴजञाननकनामहहॴ

17-Jan-2017

Options

1) Mango

आम

2) Wheat

गहॳह

3) Corn

भ ा 4) banana

कहॳ िा Correct Answer banana

Q73 Prawns belong to which family

झीगहॳककसपररवारकहॳ हह८तहॳहहॴ 17-Jan-2017

Options

1) Crustaceans

िसटहॳलियन

2)Fish

मछिी 3) Amphibians

अननफबबयस

4) Reptiles

रहॳपटाइमस

Correct Answer Crustaceans

Q74 Name the drug that is yielded from

Cinchona tree and is used to cure

malaria

उसऔषचधकानामबताइएनजसहॳलसगकह८नापहॳड़सहॳपरापतककयाजाताहहॴऔरनजसकाउपयह८गमिहॳररयाकहॳ उपचारमककयाजाताहहॴ 17-Jan-2017

Options

1) Camptothea

कहॴ नटह८चिया 2) Acuminata

एकयलमनहॳटा 3) Quinine

कनहॴन

4) Cinchonia

लसकह८ननया Correct Answer Quinine

Q75 Blood Circulation was discovered

by

रकतपररसचरणकी mdashmdashndash दवारािह८जकीिी 17-Jan-2017

Options

1) Mary Anderson

F A C E B O O K

P A G E h t t p w w w f a c e b o o k c o m s s c m e n t o r s o f f i c i a l P a g e | 19

FOR MORE UPDATES AND MORE MATERIAL DO LIKE OUR FACEBOOK PAGE httpwwwfacebookcomsscmentorsofficial

महॴरीएिरसन

2) Virginia Apgar

वनजयननयाएपगार

3) William Harvey

पवलियमहाव

4) Robert Feulgen

रॉबटयफ़यिजहॳन Correct Answer William Harvey

Q76 Vitamin A is also known as

पवटालमन A कह८ mdashmdash- कहॳ नामसहॳभीजानाजाताहहॴ SSC CHSL Science (biology) 2016

Question Paper

18Jan2017

Options

1) Thiamine

िायलमन

2) Riboflavin

ररबह८फिहॳपवन

3) Retinol

रहॳटटनॉि

4) Calciferol

कहॴ नमसफहॳ रह८ि

Correct Answer Retinol

Q77 Some roots called arise from an

organ other than the radicle

कछजड़हॳनजनह mdashmdashmdash कहाजाताहहॴ वहमिकहॳ अिावाककसीअनयअगसहॳउतपननहह८तीहहॴ 18Jan2017

Options

1) tap roots

मखयजड़

2) stilt roots

ि ाजड़

3) fibrous roots

रहॳिहॳदारजड़

4) adventitious roots

आकनसमकजड़

Correct Answer adventitious roots

Q78 Spiders belong to which class of

animals

मकडड़यापराणीवगीकरणकहॳ ककसवगयमआतीहहॴ 18Jan2017

Options

1) Arachnids

एरहॳकननडस

2) Aves

एपवस

3) Gastropods

गहॴसटरोपह८िस

4) Anthozoa

एिह८जआ

Correct Answer Arachnids

Q79 How many layers does Human

Skin have

मानवतवचामककतनीपरतहॳहह८तीहहॴ

18Jan2017

Options

1) 5

2) 7

3) 11

4) 3

Correct Answer 3

Skin has three layers The epidermis

the outermost layer of skin provides a

waterproof barrier and creates our skin

tone The dermis beneath the

epidermis contains tough connective

tissue hair follicles and sweat glands

The deeper subcutaneous tissue (

hypodermis ) is made of fat and

connective tissue

Q80 Allium Cepa is the scientific name

of

एलियमलसपपा mdashmdashndash कावहॴजञाननकनामहहॴ 18Jan2017

F A C E B O O K

P A G E h t t p w w w f a c e b o o k c o m s s c m e n t o r s o f f i c i a l P a g e | 20

FOR MORE UPDATES AND MORE MATERIAL DO LIKE OUR FACEBOOK PAGE httpwwwfacebookcomsscmentorsofficial

Options

1) Carrot

गाजर

2) Tomato

टमाटर

3) Potato

आि 4) Onion

पयाज़

Correct Answer Onion

Q81 DNA stands for

िीएनएकापणय प mdashmdash- हहॴ 18Jan2017

Options

1) Di Nucleic Acid

िाईनयनकिकएलसि

2) Deoxy Nucleic Acid

िीओकसीनयनकिकएलसि

3) Diribonucleic Acid

िाईराइबह८नयनकिकएलसि

4) Deoxyribonucleic Acid

िीऑकसीराइबह८नयनकिकएलसि

Correct Answer Deoxyribonucleic Acid

Q82 Organisms that generate energy

using light are known as

जह८जीवाणपरकािकाउपयह८गकरउजायउतपननकरतीहहॴ उनह mdashmdash कहॳ पमजानाजाताहहॴ

18Jan2017

Options

1) Chaemolithotrophs

ककमह८लििह८टरह८पस

2) Oligotrophs

ओलिगह८टरह८पस

3) Bacteria

बहॴकटीररया 4)Photoautotrophs

फह८टह८ओटह८टरह८पस

Correct Answer Photoautotrophs

An oligotroph is an organism that can

live in an environment that offers very

low levels of nutrients

Q83 Which drug is used as an

Antidepressant

ककसदवाएकहतािारह८धीकहॳ पमपयोगककयाजाताहहॴ Options

1) Oxybutynin

ओकसीलयटीनन

2)Tramadol

टरहॳमहॳिह८ि

3 ) Sumatriptan

समहॳटरीपटहॳन

4) Bupropion

लयपरह८पपयह८न

Correct Answer Bupropion

लयपरह८पपयह८न

Q84 The orange colour of carrot is

because of

गाजरकानारगीरगनननननलिखितमसहॳककसीएककीवजहसहॳहह८ताहहॴ 18Jan2017

Options

1) it grows in the soil

यहलम ीमउगतीहहॴ 2) Carotene

कहॴ रह८टीन

3) it is not exposed to sunlight

यहसययपरकािकहॳ सपकय मनहीआती 4) the entire plant is oranqe in colour

सनपणयपह९धानारगीरगकाहह८ताहहॴ Correct Answer Carotene

Q85 Snake venom is highly modified

saliva containing

F A C E B O O K

P A G E h t t p w w w f a c e b o o k c o m s s c m e n t o r s o f f i c i a l P a g e | 21

FOR MORE UPDATES AND MORE MATERIAL DO LIKE OUR FACEBOOK PAGE httpwwwfacebookcomsscmentorsofficial

सापकाजहरअततयाचधकसिह८चधतिारहह८तीहहॴनजसमहॳ mdashmdash- हह८ताहहॴ Options

l)Prototoxins

परह८टह८टॉनकसस

2)Neutrotoxins

नयटरोटॉनकसस

3)Zootoxins

जटॉनकसस

4)Electrotoxins

इिहॳकटरह८टॉनकसस

Correct Answer Zootoxins

जटॉनकसस

Q86 Which type of pathogen causes the

water-borne disease Schistosomiasis

ककसपरकारकारह८गज़नकजिजननतरह८गलससटह८सह८लमलससकाकारणबनताहहॴ

18Jan2017

Option

1) Parasitic

परजीवी 2)Protozoan

परह८टह८जआ

3) Bacterial

बहॴकटीररयि

4) Viral

वायरि

Correct Answer Parasitic

Schistosomiasis also known as snail

fever and bilharzia is a disease caused

by parasitic

flatworms called schistosomes

Q87 Prothrombin responsible for

clotting of blood is released by

परह८िह८ननबन

जह८रकतकािककाजमनहॳकहॳ लिएनजनमहॳदारहहॴ mdashndash

कहॳ दवारासतरापवतककयाजाताहहॴ

19Jan2017

Options

1) Small Intestine

छह८टीआत

2) Blood Platelets

रकतपिहॳटिहॳटस

3) Large Intestine

बड़ीआत

4Heart

हदय

Correct Answer Blood Platelets

Q88 Acacia arabica is the scientific

name of

अकहॳ लियाअरहॳबबका mdashmdashndash कावहॴजञाननकनामहहॴ 19-Jan-2017

Options

1) Neem

नीम

2) Teak

सागह९न

3) Babhul

बबि

4) Pomegranate

अनार

Correct Answer Babhul

Q89 Cannis Vulpes is the scientific

name of

कहॴ ननसवनमपस mdashmdash- कावहॴजञाननकनामहहॴ 19-Jan-2017

Options

1) Dog

कतता 2) Wolf

भहॳडड़या 3) Fox

िह८मड़ी 4) Hyena

िाकिबगघा

F A C E B O O K

P A G E h t t p w w w f a c e b o o k c o m s s c m e n t o r s o f f i c i a l P a g e | 22

FOR MORE UPDATES AND MORE MATERIAL DO LIKE OUR FACEBOOK PAGE httpwwwfacebookcomsscmentorsofficial

Correct Answer Fox

Q90 The beetroot is the portion of the

beet plant

चकदरपह९धहॳका mdashmdashndash भागहहॴ 19-Jan-2017

Options

1) tap root

मखयजड़

2) Adventitious

आकनसमक

3) bulb of the stem

तनहॳकाकद

4) Rhizome

परकद

Correct Answer tap root

Q91 What is the basic unit of heredity

आनवलिकताकीबननयादीइकाईकयाहहॴ 19-Jan-2017

Options

1) DNA

िीएनए

2) RNA

आरएनए

3) Chromosome

िह८मह८सह८म

4) Gene

जीन

Correct Answer gene

Genes are the units of heredity and are

the instructions that make up the bodyrsquos

blueprint They code for the proteins

that determine virtually all of a personrsquos

characteristics Most genes come in

pairs and are made of strands of genetic

material called deoxyribonucleic acid

or DNA

Q92 Lungs are the primary organs of

फहॳ फड़हॳmdashndashकहॳ परािलमकअगहहॴ

19-Jan-2017

Options

1) Digestion

पाचन

2) Constipation

कलज

3) Perspiration

पसीना 4)Respiration

शवसन

Correct Answer Respiration

Q93 Sugarcane is a type of

गननाएकपरकारका mdash- हहॴ 20-Jan-2017

Options

1)creeper

िता 2)tree

पहॳड़

3)shrub

झाड़ी 4)grass

घास

Correct Answer grass

Q94 Who is commonly known as ldquothe

Father of Microbiologyrdquo

सामानयत ldquo सकषमजीवपवजञानकहॳ जनक lsquo

कहॳ नामसहॳककसहॳजानाजातहहॴ 20-Jan-2017

Options

1) Robert Hooke

रॉबटयहक

2) Antonie Philips van Leeuwenhoek

एटह८नीकफलिपवानमयएनहह८क

3) Carl Linnaeus

काियिीनाईयस

4) Charles Darwin

चामसयिापवयन

F A C E B O O K

P A G E h t t p w w w f a c e b o o k c o m s s c m e n t o r s o f f i c i a l P a g e | 23

FOR MORE UPDATES AND MORE MATERIAL DO LIKE OUR FACEBOOK PAGE httpwwwfacebookcomsscmentorsofficial

Correct Answer Antonie Philips van

Leeuwenhoek

Q95 For the aquatic organisms the

source of food is

जिीयजीवाणकािाघसरह८तहहॴ 20-Jan-2017

Options

1) Phytoplankton

फायटह८पिहॳकटन

2) Sea Weed

समदरीिहॴवाि

3)Aqua plankton

एकवापिहॳकटन

4) Zooplankton

जपिहॳकटन

Correct Answer Phytoplankton

Q96 Haemoglobin has the highest

affinity with which of the following

हीमह८गिह८बबनकीननननमसहॳककसकहॳ सािउततमसमानताहहॴ

20-Jan-2017

Options

1)SO2

2)CO2

3)CO

4)NO2

Correct Answer CO

It has a greater affinity for hemoglobin

than oxygen does It displaces oxygen

and quickly binds so very little oxygen

is transported through the body cells

Q97 Who developed the theory of

Evolution

उदपवकासकालसदातककसनहॳपवकलसतककया

20-Jan-2017

Options

1) Charles Darwin

चामसयिापवयन

2) Isaac Newton

आयजहॳकनयटन

3) Pranav Mistry

परणवलमसतरी 4) Galileo Galilei

गहॳलिलियह८गहॳिीिी Correct Answer Charles Darwin

Q98 The primary function of RNA is

RNA कापरािलमककाययहह८ताहहॴ 20-Jan-2017

Options

1) Photosynthesis

परकािसशिहॳषण

2) Protein Synthesis

परह८टीनसशिहॳषण

3) Replication

परनतकनतबनाना 4) Translation

अनवादकरना Correct Answer Protein Synthesis

There are two main functions of RNA

It assists DNA by serving as a messenger

to relay the proper genetic information

to countless numbers of ribosomes in

your body The other main function of

RNA is to select the correct amino acid

needed by each ribosome to build new

proteins for your body

Q99 ______is the movement of

molecules across a cell membrane from

a region of their lower concentration to

a region of their higher concertration

उचचसादरताकहॳ कषहॳतरसहॳउसकीकमसादरतावािहॳकषहॳतरकीतरफएककह८लिकाखझमिीकहॳ माधयमसहॳहह८नहॳवािाअणओकहॳ सचिनकह८ mdash- कहतहॳहहॴ Options

1) Diffusion

पवसरण

2) Osmosis

ऑसमह८लसस

F A C E B O O K

P A G E h t t p w w w f a c e b o o k c o m s s c m e n t o r s o f f i c i a l P a g e | 24

FOR MORE UPDATES AND MORE MATERIAL DO LIKE OUR FACEBOOK PAGE httpwwwfacebookcomsscmentorsofficial

3) Active Transport

सकियआवागमन

4) Passive Transport

नननषियआवागमन

Correct Answer Active Transport

Q100 Study of classification of

organisms is known as 20-Jan-2017

जीवाणओकहॳ वगीकरणकहॳ अधययनकह८ mdash-

कहाजाताहहॴ Options

1) Serpentology

सपरहॳटह८िह८जी 2) Virology

वायरह८िह८जी 3) Taxonomy

टहॴकसोनह८मी 4) Physiology

कफनज़यह८िह८जी Correct Answer Taxonomy

Q101 Photosynthesis takes place inside

plant cells in

परकािसशिहॳषणवनसपनतकह८लिकामनसति mdash

mdashmdash महह८ताहहॴ 20-Jan-2017

Options

1) Ribosomes

राइबह८सह८नस

2) Chloroplasts

किह८रह८पिासट

3) Nucleus

नयकलियम

4) Mitochondria

माईटह८कोडडरया Correct Answer Chloroplasts

Q102 ______ is the cell organelle in

which the biochemical processes of

respiration and energy production

occur

mdashmdash- वहकह८लिकाअगहहॴ नजसमहॳशवसनऔरउजायउतपादनकहॳ जहॴसीजहॴवरासायननकपरकियायहह८तीहहॴ 20-Jan-2017

Options

1) Mitochondria

माइटह८कोडडरया 2) Chloroplast

किह८रह८पिासट

3) Ribosomes

राइबह८सह८नस

4) Nucleus

नयकिीयस

Correct Answer Mitochondria

Q103 Which non-flowering spore

bearing plants have roots

ककसफिनिगनहॳवािहॳऔरबीजाणधारकपह९धह८कीजड़हॳहह८तीहहॴ 21-Jan-2017

Options

1) Mosses

मह८सहॳस

2) Angiosperms

एननजयह८सपनसय 3) Ferns

फनसय 4) Gymnosperms

नजननह८सपनसय Correct Answer ferns

Q104 Which of the following is an

excretory organ of cockroach

नननननलिखितमसहॳकह९नसानतिच हॳकाउतसजयनअगहहॴ

21-Jan-2017

Options

F A C E B O O K

P A G E h t t p w w w f a c e b o o k c o m s s c m e n t o r s o f f i c i a l P a g e | 25

FOR MORE UPDATES AND MORE MATERIAL DO LIKE OUR FACEBOOK PAGE httpwwwfacebookcomsscmentorsofficial

1) Malphigian Tubules

मनमफनजयनटयबमस

2) Nephridia

नहॳकफरडिया 3) Coxal Gland

कह८कसिगरचिया 4) Green Gland

गरीनगरचिया Correct Answer Malphigian Tubules

Q105 Evaporation of water takes place

in which part of plants

पानीकहॳ वाषपीकरणकीकियापह९धोकहॳ ककसभागसहॳहह८तीहहॴ 21-Jan-2017

Options

1) Stem

तना 2) Stomata

सटह८मटा 3) Branch

िािाए

4) Fruit

फि

Correct Answer Stomata

Evaporation accounts for the movement

of water to the air from sources such as

the soil canopy interception and

waterbodies Transpiration accounts for

the movement of water within a plant

and the subsequent loss of water as

vapour through stomata in its leaves

Q106 A is the fleshy spore-bearing

fruiting body of a fungus

mdashmdashndashकवककामासि

बीजाणधारणकरनहॳवािाफिनहॳवािाअगहहॴ 21-

Jan-2017

Options

1) aloe vera

एिह८वहॳरा 2) Coral

मगा 3) Cactus

कहॴ कटस

4) Mushroom

ककरमतता Correct Answer mushroom

Q107 Which of the following is a fungal

disease

नननननलिखितमसहॳकह९नसाफफदसहॳहह८नहॳवािाएकरह८ग हहॴ

21-Jan-2017

Options

1) Dermatitis

तवचािह८ध

2) Cholera

हहॴजा 3) Jaundice

पीलिया 4) Indigofera

इननिगह८फहॳ रा Correct Answer Dermatitis

Dermatitis also known as eczema is a

group of diseases that results in

inflammation of the skin These diseases

are characterized by itchiness red skin

and a rash In cases of short duration

there may be small blisters while in

long-term cases the skin may become

thickened

Q108 In which form is glucose stored in

our body

हमारहॳिरीरमगिकह८जकासचयककस पमककयाजाताहहॴ

21-Jan-2017

Options

1) Insulin

F A C E B O O K

P A G E h t t p w w w f a c e b o o k c o m s s c m e n t o r s o f f i c i a l P a g e | 26

FOR MORE UPDATES AND MORE MATERIAL DO LIKE OUR FACEBOOK PAGE httpwwwfacebookcomsscmentorsofficial

इसलिन

2) Glucose

गिकह८ज

3) Glycogen

गिायकह८जहॳन

4) Fat

वसा Correct Answer Glycogen

Excess glucose is stored in the liver as

the large compound called glycogen

Glycogen is a polysaccharide of glucose

but its structure allows it to pack

compactly so more of it can be stored in

cells for later use

Q109 Where do plants synthesize

protein from

पह९धहॳपरह८टीनसशिहॳषणकहासहॳकरतहॳहहॴ

Options

1) Fatty Acids

वसाऐलसि

2) Sugar

िकर

3) Amino Acids

एलमनह८ऐलसि

4) Starch

सटाचय Correct Answer Amino Acids

Q110 Which part of the brain is

responsible for triggering actions like

thinking intelligence memory and

ability to learn

मनसतषककाकह९नसाटहससासह८चनहॳ बनधदमानी याददाशतऔरसीिनहॳकीकषमताजहॴसीकियाओकह८परहॳररतकरताहहॴ 21-Jan-2017

Options

1) Diencephalon

िायएनसहॳफहॳ िह८न

2) Hypothalamus

हयपह८िहॳिहॳमस

3) Cerebrum

सहॳरहॳिम

4) Control

कटरह८ि

Correct Answer Cerebrum

Q111 Which of the following is also

known as the Biochemical Laboratory

of the Human Body

नननननलिखितमसहॳककसहॳमानविरीरकीजहॴवरसायनपरयह८गिािाभीकहाजाताहहॴ 21-Jan-2017

Options

1) Small Intestine

छह८टीआत

2)Brain

मनसतषक

3) Pancreas

अगनयािय

4) Liver

नजगर

Correct Answer Liver

The liver makes bile that will help

emulsify and digest the fats we eat

The liver takes toxic substances and

convert them using enzymes the liver

cells makes into a non toxic form so the

body can dispose of them

The liver also converts fats protein and

carbohydrates into glucose which is the

energy source for our cells to use

The liver takes amino acids and makes

proteins by combining them

Q112 The yellow colour of human urine

is due to

मानवमतरकापीिारग mdashndash कीवजहसहॳहह८ताहहॴ 22-

Jan-2017

Options

1) Bile Salts

F A C E B O O K

P A G E h t t p w w w f a c e b o o k c o m s s c m e n t o r s o f f i c i a l P a g e | 27

FOR MORE UPDATES AND MORE MATERIAL DO LIKE OUR FACEBOOK PAGE httpwwwfacebookcomsscmentorsofficial

पपततनमक

2) Cholesterol

कह८िहॳसटरह८ि

3) Lymph

लिनफ

4) Urochrome

यरह८िह८म

Correct Answer Urochrome

Urobilin or urochrome is the chemical

primarily responsible for the yellow

color of urine

Q113 The wilting of plants takes place

due to

पह९धह८कालिचििहह८नाकी mdashmdash- कीवजहसहॳहह८ताहहॴ 22-Jan-2017

Options

1)Photosynthesis

परकािसशिहॳषण

2) Transpiration

वाषपह८तसजयन

3) Absorption

अविह८षण

4) Respiration

शरवसन

Correct Answer Transpiration

Wilting is the loss of rigidity of non-

woody parts of plants This occurs when

the turgor pressure in non-lignified

plant cells falls towards zero as a result

of diminished water in the cells

Q114 Bovidae Ovis is the scientific name of

बह८पविीओपवस mdashndash कावहॴजञाननकनामहहॴ 22-Jan-2017

Options

1) Goat

बकरी 2) Cow

गाय

3) Buffalo

भहॳस

4) Sheep

भहॳड़

Correct Answer Sheep

Q115 Plants get their energy to produce

food from which of the following

पह८धहॳभह८जनकाननमायणकरनहॳकहॳ लिएनननननलिखितमसहॳककससहॳउजायपरापतकरतहॳहहॴ

22-Jan-2017

Options

1) Photosynthesis

परकािसशिहॳषण

2)Bacteria

बहॴकटीररया 3)Fungi

कवक

4)Sun

सयय Correct Answer Sun

Q116 Which of the following is secreted

by the liver

नननननलिखितमसहॳककसकासरावनजगरसहॳहह८ताहहॴ

22-Jan-2017

Options

1) Glucose

गिकह८ज

2) Iodine

आयह८िीन

3) Cortisol

काटटरयसह८ि

4) Bile

पपतत

Correct Answer Bile

The liver makes bile that will help

emulsify and

digest the fats we eat

F A C E B O O K

P A G E h t t p w w w f a c e b o o k c o m s s c m e n t o r s o f f i c i a l P a g e | 28

FOR MORE UPDATES AND MORE MATERIAL DO LIKE OUR FACEBOOK PAGE httpwwwfacebookcomsscmentorsofficial

Q117 Ferns belong to which division of

plants

फनसयपह९धह८कहॳ ककसभागमआतहॳहहॴ

22-Jan-2017

Options

1) Gymnosperms

नजननह८सपनसय 2) Angiosperms

एनजयह८सपनसय 3) Thallophyta

िहॴिह८फाईटा 4)Pteridophyta

टहॳररिह८फाईटा Correct Answer Pteridophyta

Q118 Who invented Antibiotics

एटीबायह८टटककाअपवषकारककसनहॳककयािा

22-Jan-2017

Options

1) Joseph Lister

जह८सहॳफलिसटर

2) William Harvey

पवलियमहाव

3) Robert Knock

रॉबटयनॉक

4)Alexander Fleming

अिहॳकज़िरफिहॳलमग

Correct Answer Alexander Fleming

Q119 Milbecycin is used in the

eradication of

लममबहॳसायलसनका mdashndash

मउनमिनमपरयह८गककयाजाताहहॴ 22-Jan-2017

Options

1) Agricultural Fungus

कपषकवक

2) Agricultural Pests

कपषकीटक

3) Agricultural Herbs

कपषिाक

4)Agricultural Weeds

कपषननराना Correct Answer Agricultural Pests

Milbemycin oxime is a veterinary drug

from the group of milbemycins used as

a broad spectrum antiparasitic It is

active against worms and mites(insects

Q120 Intestinal bacteria synthesizes

which of the following in the human

body

मानविरीरमआतोकहॳ बहॴकटीररयानननननलिखितमसहॳककसकासशिहॳषणकरतहॳहहॴ 22-Jan-2017

Options

1) Vitamin K

पवटालमन K

2) Proteins

परह८टीन

3) Fats

वसा 4) Vitamin D

पवटालमन D

Correct Answer Vitamin K

Q121 is the study of the physical form

and external structure of plants

mdashmdash-

मपह९धह८काभहॴनतक पऔरबाहरीसरचनाकाआदयाककयाजाताहहॴ 22-Jan-2017

Options

1) Physiology

कफनजयह८िह८जी 2) Anatomy

िरीररचनापवजञान

3) Phytomorphology

फाईटह८मह८फह८िह८जी 4)Cytology

कह८लिकापवजञान

Correct Answer Phytomorphology

F A C E B O O K

P A G E h t t p w w w f a c e b o o k c o m s s c m e n t o r s o f f i c i a l P a g e | 29

FOR MORE UPDATES AND MORE MATERIAL DO LIKE OUR FACEBOOK PAGE httpwwwfacebookcomsscmentorsofficial

Q122 Which of the following is a

structural and functional unit of

kidneys

नननननलिखितमसहॳकह९नसीगदोकीसरचनातमकऔरकाययकरीईकाईहहॴ

22-Jan-2017

Options

1) Renette Cells

रहॳनहॳटकह८लिकाए

2) Flame Cells

फिहॳमकह८लिकाए

3) Nephrites

नहॳफ़राइटस

4)Nephrons

नहॳफरोस

Correct Answer Nephrons

Nephron functional unit of the kidney

the structure that actually produces

urine in the process of removing waste

and excess substances from the blood

There are about 1000000 nephrons in

each human kidney

Q123 Which of the following is the

largest part of the human brain

नननननलिखितमसहॳकह९नसामानवमनसतषककासबसहॳबड़ाटहससाहहॴ

23-Jan-2017

Options

1) Ribs

पसलियाा 2) Cerebrum

सहॳरहॳिम

3) Pons

पोस

4)Thalamus

िहॴिहॳमस

Correct Answer Cerebrum

The cerebrum is the largest part of the

human brain making up about two-

thirds of the brainrsquos mass It has two

hemispheres each of which has four

lobes frontal parietal temporal and

occipital

Q124 The auxiliary buds

सहायककालियाmdashndash 23-Jan-2017

Options

1) grow endogenously from the pericycle

पहॳरीसाईककिसहॳअनतजातयपवकलसतहह८ताहहॴ 2) arise endogenously from the main

growing point

मिवपदसहॳअनतजातयउठताहहॴ 3) is an embryonic shoot located in the

axil of a leaf

एकभरणिटहहॴजह८एकपततीकहॳ अकषपरनसतिहह८ताहहॴ 4)arise exogenously from the epidermis

एपपिलमयससहॳबटहजातयतरीकहॳ सहॳउठताहहॴ Correct Answer is an embryonic shoot

located in the axil of a leaf

Q125 Which of the following is a viral

disease

इनमहॳसहॳकह९सीएकवायरिबीमारीहहॴ

23-Jan-2017

Options

1) Polio

पह८लियह८ 2) Tetanus

धनसतनभ

3) Leprosy

कषठरह८ग

4) Plague

पिहॳग

Correct Answer Polio

A viral disease (or viral infection)

occurs when an organismrsquos body is

invaded by pathogenic viruses and

infectious virus particles (virions) attach

to and enter susceptible cells

F A C E B O O K

P A G E h t t p w w w f a c e b o o k c o m s s c m e n t o r s o f f i c i a l P a g e | 30

FOR MORE UPDATES AND MORE MATERIAL DO LIKE OUR FACEBOOK PAGE httpwwwfacebookcomsscmentorsofficial

Poliomyelitis often called polio or

infantile paralysis is an infectious

disease caused by the poliovirus

Tetanusmdash A serious bacterial infection

that causes painful muscle spasms and

can lead to death

Leprosy also known as Hansenrsquos

disease (HD) is a long-term infection by

the bacterium Mycobacterium leprae or

Mycobacterium lepromatosis

Plague is an infectious disease caused by

the bacterium Yersinia pestis

Symptoms include fever weakness and

headache

Q126 Which organisms can help to

carry out Vermicomposting

कह९नसाजीववमीकनपह८नसटगममददकरताहहॴ

23-Jan-2017

Options

1) Nitrifying Bacteria

नाईटरीफाईगबहॴकटीररया 2) Earthworms

कहॴ चऐ

3) Algae

िहॴवि

4) Fungus

कवक

Correct Answer Earthworms

Q127 Contraction of heart is also

known as

हदयकहॳ सकचनकह८ mdash- भीकहाजाताहहॴ 23-Jan-

2017

Options

1) Systole

लससटह८ि

2) Aristotle

अरसत

3) Diastole

िायसटह८ि

4) Lub

मयब

Correct Answer Systole

Diastole is the part of the cardiac cycle

when the heart refills with blood

following systole (contraction)

Ventricular diastole is the period during

which the ventricles are filling and

relaxing while atrial diastole is the

period during which the atria are

relaxing

Q128 Azadirachta indica is the

botanical name of which of the

following

अजाटदराचताइडिकानननननलिखितमसहॳककसकावानसपनतनामहहॴ

23-Jan-2017

Options

1) Rose plant

गिाबकापह९धा 2) Apple tree

सहॳबकापहॳड़

3) Neem

नीम

4)Mango

आम

Correct Answer Neem

Q129 Which of the following is the

main end product of carbohydrate

digestion

नननननलिखितमसहॳकह९नसाकाबोहाइडरहॳटकहॳ पाचनकापरमिअतउतपादकहह८ताहहॴ 23-Jan-2017

Options

1) Fats

वसा 2) Lipids

लिपपडस

3) Glucose

गिकह८ज

4) Cellulose

F A C E B O O K

P A G E h t t p w w w f a c e b o o k c o m s s c m e n t o r s o f f i c i a l P a g e | 31

FOR MORE UPDATES AND MORE MATERIAL DO LIKE OUR FACEBOOK PAGE httpwwwfacebookcomsscmentorsofficial

सहॳमयिह८ज

Correct Answer Glucose

Intestinal absorption of end products

from digestion of carbohydrates and

proteins in the pig hellip During absorption some sugars (fructose or

galactose) released from the

corresponding sucrose and lactose

respectively during digestion were

partly metabolized into glucose by the

enterocyte

Q130 Which of the following glands is a

source of the enzyme Ptyalin

नननननलिखितगरचियोमसहॳएजाइमटयालिनकासरह८तहहॴ 23-Jan-2017

Options

1) Pancreas

अगरािय

2) Thyroid Gland

िाइराइिगरिी 3) Pituitary Gland

पीयषगरिी 4) Salivary Glands

िारगरचियाा Correct Answer Salivary Glands

Q131 Which of the following is not true

about Pteridophyta

ननननमसहॳकह९नसीबातटहॳररिह८फाईटकहॳ बारहॳमसचनहीहहॴ 23-Jan-2017

Options

1) Dominant phase is saprophytes

परमिचरणसहॳपरह८फाईइटसहह८ताहहॴ 2) Main plant body is diploid

पह९दह८कामखयिरीरदपवगखणतहह८ताहहॴ 3) Seeds are present

बीजमह९जदहह८तहॳहहॴ 4)Flowers are absent

फिअनपनसतिहह८तहॳहहॴ

Correct Answer Seeds are present

Q132 The largest dolphin species is the

orca also called as

िॉिकफनकीसबसहॳबड़ीपरजानतकाकानामआकायहहॴनजसहॳ mdash- भीकहतहॳहहॴ 23-Jan-2017

Options

1) Bottle Nose

बाटिनह८ज

2) Baiji

बहॳजी 3) Killer whale

ककिरहहॳि

4)Tucuxi

टकवसी Correct Answer Killer whale

Q133 The fat digesting enzyme Lipase

is secreted by which of the following

वसाकापाचनकरनहॳवािाएजाइमिाइपहॳजनननननलिखितमसहॳककसकहॳ दवारासतरापवतहह८ताहहॴ

24-Jan-2017

Options

1) Kidneys

गद

2) Pancreas

अगनयािय

3) Large Intestine

बड़ीआत

4)Liver

नजगर

Correct Answer Pancreas

Lipase is an enzyme that splits fats so

the intestines can absorb them Lipase

hydrolyzes fats like triglycerides into

their component fatty acid and glycerol

molecules It is found in the blood

gastric juices pancreatic secretions

intestinal juices and adipose tissues

F A C E B O O K

P A G E h t t p w w w f a c e b o o k c o m s s c m e n t o r s o f f i c i a l P a g e | 32

FOR MORE UPDATES AND MORE MATERIAL DO LIKE OUR FACEBOOK PAGE httpwwwfacebookcomsscmentorsofficial

Q134 The arrangement of leaves on an

axis or stem is called

एकअकषयातनहॳपरपनततयोकीयवसिाकह८कयाकहाजाताहहॴ SSC CHSL Science (biology) 2016

Question Paper

24-Jan-2017

Options

1) Phyllotaxy

फाइिह८टहॴकसी 2) Vernation

वनिन

3) Venation

वहॳनहॳिन

4)Phytotaxy

फाइटह८टहॴकसी Correct Answer Phyllotaxy

In botany phyllotaxis or phyllotaxy is

the arrangement of leaves on a plant

stem (from Ancient Greek phyacutellon

ldquoleafrdquo and taacutexis ldquoarrangementrdquo)

Phyllotactic spirals form a distinctive

class of patterns in nature

Q135 The study of Cells is also known

as

कह८लिकाओकहॳ अधययनकह८ mdashmdashndash

भीकहाजाताहहॴ 24-Jan-2017

Options

1) Cytology

सायटह८िह८जी 2) Physiology

कफनजयह८िह८जी 3) Nucleology

नयककमयह८िह८जी 4)Cellology

सहॳिह८िह८जी Correct Answer Cytology

Q136 Which of the following scientists

is also known as the Father of Biology

नननननलिखितमसहॳककसवहॴजञाननककह८ ldquoजीवपवजञानकहॳ जनकrdquoकहॳ नामसहॳभीजानाजाताहहॴ 24-Jan-2017

Options

1) Herbert Spencer

हबयटयसपसर

2) Aristotle

अरसत 3) Lamarck

िहॳमाकय 4)Darwin

िापवयन

Correct Answer Aristotle

Q137 Which cells give rise to various

organs of the plant and keep the plant

growing

कह९नसीकह८लिकाएपह९धह८कहॳ लभननअगह८कह८जनमदहॳतीहहॴऔरपह९धह८कह८बढ़नहॳममददकरतीहहॴ

24-Jan-2017

Options

1) Permanent

सिायी 2) Dermal

तवचीय

3) Meristematic

मररसटहॳमटटक

4)Mature

परह८ढ़

Correct Answer Meristematic

A meristem is the tissue in most plants

containing undifferentiated cells

(meristematic cells) found in zones of

the plant where growth can take place

Q138 Rodentia Muridae is the scientific

name of

F A C E B O O K

P A G E h t t p w w w f a c e b o o k c o m s s c m e n t o r s o f f i c i a l P a g e | 33

FOR MORE UPDATES AND MORE MATERIAL DO LIKE OUR FACEBOOK PAGE httpwwwfacebookcomsscmentorsofficial

रह८िहॳलियानयररिी mdashmdash- कावहॴजञाननकनामहहॴ 24-

Jan-2017

Options

1) Mouse

चहा 2) Squirrel

चगिहरी 3) Monkey

बदर

4) Lizard

नछपकिी Correct Answer Mouse

Q139 Name the scientist who proposed

the cell theory

कह८लिकालसदातकापरसतावदहॳनहॳवािहॳवहॴजञाननककानामबताइए 24-Jan-2017

Options

1) Schleiden and Schwann

िीमिनऔरशरववान

2) Lamarck

िहॳमाकय 3) Treviranus

टरहॳवायरहॳनस

4)Whittaker and Stanley

हीटकरऔरसटहॳनिहॳ Correct Answer Schleiden and

Schwann

Q140 The flower with the worldrsquos

largest bloom is

दननयाकासबसहॳबड़ाफिखििनहॳवािा mdashmdashndash हहॴ 24-Jan-2017

Options

1) Pando

पािह८ 2) Posidonia

पह८सीिह८ननया 3) Rafflesia arnoldii

ररफिहॳलियाअनोमिी 4)Helianthus annuus

हहॳलिएनिसएनयअस

Correct Answer Rafflesia arnoldii

Rafflesia arnoldii is a species of

flowering plant in the parasitic genus

Rafflesia It is noted for producing the

largest individual flower on earth It has

a very strong and horrible odour of

decaying flesh earning it the nickname

ldquocorpse flower

Q141 Deficiency of which vitamin

causes night blindness

ककसपवटालमनकीकमीकहॳ कारणरतौधीहह८ताहहॴ 24-Jan-2017

Options

1) Vitamin K

पवटालमन K

2) Vitamin C

पवटालमन C

3) Vitamin B1

पवटालमन B1

4)Vitamin A

पवटालमन A

Correct Answer Vitamin A

Q142 Nongreen plants lack which of the

following

गहॴर-

हररतवनसपनतमनननननलिखितमसहॳककसकीकमीहह८तीहहॴ

24-Jan-2017

Options

1) Chlorophyll

किह८रह८कफि

2) Lycophyll

िायकह८कफि

3) Cyanophyll

F A C E B O O K

P A G E h t t p w w w f a c e b o o k c o m s s c m e n t o r s o f f i c i a l P a g e | 34

FOR MORE UPDATES AND MORE MATERIAL DO LIKE OUR FACEBOOK PAGE httpwwwfacebookcomsscmentorsofficial

सायनह८कफि

4)Phototropism

फह८टह८टरोपपजम

Correct Answer Chlorophyll

Q143 Organisms that use light to

prepare food are known as

जह८जीवपरकािकाउपयह८गकरभह८जनतहॴयारकरतहॳहहॴ उनह mdashmdash- कहॳ पमजानजाताहहॴ 24-Jan-2017

Options

1) Autotrophs

सवपह८षी 2) Heterotrophs

पवषमपह८षज

3) Omnivores

सवायहारी 4)Decomposers

पवघटनकरनहॳवािा Correct Answer Autotrophs

autotrophs often make their own food

by using sunlight carbon dioxide and

water to form sugars which they can use

for energy Some examples of

autotrophs include plants algae and

even some bacteria Autotrophs

(producer) are important because they

are a food source for heterotrophs

(consumers)

A heterotroph is an organism that

ingests or absorbs organic carbon

(rather than fix carbon from inorganic

sources such as carbon dioxide) in order

to be able to produce energy and

synthesize compounds to maintain its

life Ninety-five percent or more of all

types of living organisms are

heterotrophic including all animals and

fungi and some bacteria

Q144 Which of the following is a

primary function of haemoglobin

नननननलिखितमसहॳकह९नसाटहमह८गिह८बबनकाएकपरािलमककाययहहॴ

25-Jan-2017

Options

1) Utilization of energy

उजायकाउपयह८गकरना 2) Prevention of anaemia

रकतामपताहह८नहॳसहॳरह८कना 3) Destruction of bacteria

बहॴकटीररयाकापवनािकरना 4) To transport oxygen

ऑकसीजनकावहनकरना Correct Answer To transport oxygen

Q145 Vascular bundles are absent in

सवहनीबिि mdashmdash- मअनपनसतिरहतहॳहहॴ 25-Jan-2017

Options

1) Bryophyta

िायह८फाइटा 2) Pteridophyta

टहॳररिह८फाईटा 3) Gymnosperms

नजननह८सपमय 4) Angiosperms

एननजयह८सपहॳनसय Correct Answer Bryophyta

Q146 Sauria Lacertidae is the scientific

name of

सहॴररयािहॳसरटाईिी mdashmdashndash कावहॴजञाननकनामहहॴ 25-Jan-2017

Options

1) Crocodile

मगरमचछ

2) Hippopotamus

टहपपह८पह८टहॳमस

3) Lizard

नछपकिी 4) House fly

F A C E B O O K

P A G E h t t p w w w f a c e b o o k c o m s s c m e n t o r s o f f i c i a l P a g e | 35

FOR MORE UPDATES AND MORE MATERIAL DO LIKE OUR FACEBOOK PAGE httpwwwfacebookcomsscmentorsofficial

घरहॳिमकिी Correct Answer Lizard

Q147 Which type of pathogen causes

the water-borne disease SARS (Severe

Acute Respiratory Syndrome)

ककसपरकािकारह८गज़नकजिजननतबीमारीसासयकाकारणबनताहहॴ 25-Jan-2017

Options

1) Viral

वायरि

2) Parasitic

परजीवी 3) Protozoan

परह८टह८जअन

4) Bacterial

बहॴकटीररयि

Correct Answer Viral

Q148 Which of the following organs

produces the enzyme lipase

नननननलिखितमसहॳकह९नसाअगिायपहॳजएजाइमउतपननकरताहहॴ 25-Jan-2017

Options

1) Pancreas

अगनयािय

2) Large Intestine

बड़ीआत

3) Liver

नजगर

4) Small Intestine

छह८टीआत

Correct Answer Pancreas

Q149 A is a long internode forming the

basal part or the whole of a peduncle

एक mdashmdash- एकिबाइटरनह८िहहॴ जह८ननचिाटहससायासनपणयिठिबनताहहॴ 25-

Jan-2017

Options

1) Rhizome

परकद

2) Rachis

महॳ दि

3) floral axis

पषपअकष

4) Scape

भगदड़

Correct Answer scape

Q150 ndash Which of the following

organisms are considered to be both

Living and Non-living

नननननलिखितमसहॳकह९नसहॳजीवाणकह८जीपवतऔरअजीपवतमानाजाताहहॴ

25-Jan-2017

Options

1) Bacteria

बहॴकटीररया 2) Fungi

कवक

3) Algae

िहॴवाि

4)Virus

वायरस

Correct Answer Virus

They are considered to be living as they

possess a protein coat as a protective

covering DNA as the genetic material

etc

They are said to be non-living as they

can be crystallised and they survive for

billions of years They can tolerate high

temperatures freezing cold

temperatures ultra-violet radiations etc

Q151 Deficiency of fluorine causes

which of the following

फिह८ररनकीकमीकहॳ कारणनननननलिखितमसहॳकयाहह८ताहहॴ

F A C E B O O K

P A G E h t t p w w w f a c e b o o k c o m s s c m e n t o r s o f f i c i a l P a g e | 36

FOR MORE UPDATES AND MORE MATERIAL DO LIKE OUR FACEBOOK PAGE httpwwwfacebookcomsscmentorsofficial

27-Jan-2017

Options

1) Dental Caries

िटिकहॴ ररज

2) Scurvy

सकवरी 3) Anaemia

रकतामपता 4) Arthritis

गटठया Correct Answer Dental Caries

Q152 In a Punnett Square with the

cross AaBb x AaBb how many Aabb

genotypes would be created

पनहॳटसककायरमिह८स AaBb x AaBb कहॳ साि

ककतनहॳ Aabb जीनह८टाइपबनगहॳ 27-Jan-2017

Options

1) 1

2) 8

3) 2

4) 3

Correct Answer 2

Q153 Which of the following is the

Controlling Center of the Cell

नननननलिखित म सहॳ कह८लिकाका ननयतरण

क दर कह९न हहॴ

27-Jan-2017

Options

1) Nucleus

क दर

2) Plasma

पिाजमा 3) Lysosome

िायसह८सह८म

4) Chromosome

िह८मह८सह८म

Correct Answer Nucleus

The control centre of the cell is the

nucleus in eukaryotic cells The nucleus

contains genetic material in the form of

DNA

Q154 Myopia affects which of the

following organs

मायह८पपयानननननलिखितअगह८मसहॳककसहॳपरभापवतकरताहहॴ

25-Jan-2017

Options

1) Heart

हदय

2) Skin

तवचा 3) Eyes

आािहॳ 4)Mouth

मह

Correct Answer Eyes

Q155 Which of the following bears

flowers

नननननलिखितमसहॳकह९नफिधारणकरताहहॴ

25-Jan-2017

Options

1) Bryophyta

िायह८फाइटा 2) Pteridophyta

टहॳरीिह८फाईटा 3) Gymnosperms

नजननह८सपमय 4)Angiosperms

एननजयह८सपमय Correct Answer Angiosperms

Q156 Oxygenated blood flows out of the

heart through the

ऑकसीजनयकतरकत mdashmdashmdash

कहॳ माधयमसहॳहदयकहॳ बाहरबहताहहॴ 25-Jan-2017

F A C E B O O K

P A G E h t t p w w w f a c e b o o k c o m s s c m e n t o r s o f f i c i a l P a g e | 37

FOR MORE UPDATES AND MORE MATERIAL DO LIKE OUR FACEBOOK PAGE httpwwwfacebookcomsscmentorsofficial

Options

1) Aorta

महाधमनी 2) pulmonary artery

फहॳ फड़हॳकीधमनी 3) vena cava

वहॳनाकावा 4)Atrium

चह९क

Correct Answer aorta

Q157 Blood leaving the liver and

moving towards the

heart has a higher concentration of

नजगरसहॳननकिकरहदयकीतरफजानहॳवािहॳरकतम mdashmdashmdashmdash कीउचचसादरताहह८तीहहॴ 27-Jan-2017

Options

1) Lipids

लिपपडस

2) Urea

यररया 3) Bile Pigments

पपततकहॳ रगकरण

4) Carbon dioxide

काबयनिायऑकसाइि

Correct Answer Bile Pigments

Urea is nitrogen containing substance

which is produced in the liver in order

to deal with excess amino-acids in the

body As urea is produced it leaves the

liver in the blood stream and passes via

the circulatory system to all parts of the

body

Q158 Bulb is a modification of which

part of a plant

बमबएकपह९धहॳकहॳ ककसटहससहॳकाएक पातरणहह८ताहहॴ 27-Jan-2017

Options

1) The root

जड़

2) The stem

तना 3) The radicle

मिाकर

4)The fruit

फि

Correct Answer The stem

Q159 Which of the following carries

blood away from the heart to different

body parts

इनमहॳसहॳकह९नरकतकह८हदयसहॳिरीरकहॳ पवलभननअगह८तकिहॳजातीहहॴ

27-Jan-2017

Options

1) Arteries

धमननया 2) Nerves

तबतरहाए

3) Capillaries

कहॳ लिकाए

4)Veins

नसहॳ Correct Answer Arteries

Q160 The series of processes by which

nitrogen and its compounds are

interconverted in the environment and

in living organisms is called

27-Jan-2017

Options

1)Absorption of Nitrogen

2)Ammonification

3)Nitrogen Fixation

4)Nitrogen Cycle

Correct Answer Nitrogen Cycle

Ammonification or Mineralization is

performed by bacteria to convert

organic nitrogen to ammonia

F A C E B O O K

P A G E h t t p w w w f a c e b o o k c o m s s c m e n t o r s o f f i c i a l P a g e | 38

FOR MORE UPDATES AND MORE MATERIAL DO LIKE OUR FACEBOOK PAGE httpwwwfacebookcomsscmentorsofficial

Nitrification can then occur to convert

the ammonium to nitrite and nitrate

Nitrogen fixation is a process by which

nitrogen in the Earthrsquos atmosphere is

converted into ammonia (NH3) or other

molecules available to living organisms

Q161 BCG vaccine is given to protect

from which of the following

बीसीजीकाटटकानननननलिखितमसहॳककसकहॳ बचावकहॳ लिएटदयाजातहहॴ

27-Jan-2017

Options

1) Jaundice

पीलिया 2) Anaemia

रकतमपता 3) Tuberculosis

कषयरह८ग

4) Polio

पह८लियह८ Correct Answer Tuberculosis

Q162 Parallel venation is found in

समानतरवहॳनहॳिन mdashmdashmdash- मपायाजाताहहॴ 27-Jan-2017

Options

1) plants which are monocots

पह९धहॳजह८एकबीजपतरीहह८तहॳहहॴ 2) plants which have a dicot stem

वहॳपह९धहॳनजनकातनादपवदलियहह८ताहहॴ 3) plants with leaves similar to Tulsi

वहॳपह९धहॳनजनकीपनततयतिसीकीपनततयोकहॳ समानहह८तहॳहहॴ 4)plants with tap roots

टहॳप टवािहॳपह९धहॳ Correct Answer plants which are

monocots

Q163 The hardest part of the body is

िरीरकासबसहॳकठह८रभाग mdashndash हहॴ 27-Jan-2017

Options

1) Bones

हडडिय

2) Tooth Enamel

दातकहॳ इनहॳमि

3) Skull

िह८पड़ी 4) Spinal Cord

महॳ रजज

Correct Answer Tooth Enamel

Q164 Which type of pathogen causes

the waterborne disease E coli Infection

ककसपरकारकारह८गजननकजिजननतरह८गईकह८िाईसिमणकाकारणबनताहहॴ 27-Jan-2017

Options

1) Protozoan

परह८टह८जआ

2) Parasitic

परजीवी 3) Bacterial

बहॴकटीररयि

4)Viral

वायरि

Correct Answer Bacterial

Q165 The amount of blood filtered

together by both the kidneys in a 70 kg

adult male human in a minute is

70 की गरा वािहॳएकवयसकप षमएकलमनटमदह८नोगदकहॳदवाराएकसािचाबनीगयीरकतकीमातरहह८तीहहॴ 29-Jan-2017

Options

1) 1100 ml

1100 लमलि

2) 100 ml

F A C E B O O K

P A G E h t t p w w w f a c e b o o k c o m s s c m e n t o r s o f f i c i a l P a g e | 39

FOR MORE UPDATES AND MORE MATERIAL DO LIKE OUR FACEBOOK PAGE httpwwwfacebookcomsscmentorsofficial

100 लमलि

3) 1500 ml

1500 लमलि

4) 500 ml

500 लमलि

Correct Answer 1100 ml

Q166 Which feature of a plant helps to

distinguish a monocot from a dicot

पह९धहॳकीवहकह९नसीपविहॳषताहहॴजह८एकदपवदलियहॳऔरएकएकदिीयपह९धहॳसहॳभहॳदकरनहॳममददकरतीहहॴ 29-Jan-2017

Options

1) Pollination

परागम

2) Venation

वहॳनहॳिन

3) Vernation

वनिन

4) Aestivation

एसटीवहॳिहॳन

Correct Answer venation

Q167 The Mutation Theory was

proposed by

उतवररवतयनकालसदात mdashmdashndash

कहॳ दवरापरसतापवतककयाजाताहहॴ 29-Jan-2017

Options

1) Charles Lyell

चामसयलियहॳि

2) William Smith

पवलियमनसमि

3) Hugo De Vries

हयगह८िीराईस

4)Harrison Schmitt

हहॳरीसननसमट

Correct Answer Hugo De Vries

Q168 Which type of pathogen causes

the waterborne disease HepatitisA

ककसपरकारकहॳ रह८गजनकजिजननतरह८गहहॳपहॳटाइटटस-A काकारणबनताहहॴ

29-Jan-2017

Options

1) Parasitic

परजीवी 2) Viral

वायरि

3) Protozoan

परह८टह८जआ

4) Bacterial

बहॴकटीररयि

Correct Answer Viral

Q169 In a Punnett Square with the

cross AaBb x Aabb how many AaBb

genotypes would be created

पनहॳटसकवायरमिह८स AaBb x Aabb

कहॳ सािककतनहॳ AaBb जीनह८टाइपबनगहॳ 29-Jan-

2017

Options

1) 4

2) 1

3) 7

4) 6

Correct Answer 4

Q170 Arboreal Ateles is the scientific

name of

अिह८ररयिएटटलिस mdashmdashmdash कावहॴजञाननकनामहहॴ 29-Jan-2017

Options

1) Squirrel

चगिहरी 2) Sparrow

गह८रहॴया 3) Lizard

नछपकिी 4) Spider monkey

F A C E B O O K

P A G E h t t p w w w f a c e b o o k c o m s s c m e n t o r s o f f i c i a l P a g e | 40

FOR MORE UPDATES AND MORE MATERIAL DO LIKE OUR FACEBOOK PAGE httpwwwfacebookcomsscmentorsofficial

मकड़ीबदर

Correct Answer Spider monkey

Q171 Which type of pathogen causes

the waterborne disease Salmonellosis

ककसपरकारकारह८गाणजिजननतबीमारीसािमह८नहॳिह८लसज़काकारकहहॴ

29-Jan-2017

Options

1) Algal

िहॳवालियहॳ 2) Parasitic

परजीवी 3) Bacterial

बहॴकटीररयि

4)Viral

वायरि

Correct Answer Bacterial

An infection with salmonella bacteria

commonly caused by contaminated food

or water

Symptoms include diarrhoea fever

chills and abdominal pain

Q172 is a condition in which there is a

deficiency of red cells or of haemoglobin

in the blood

mdashmdash-

एकनसिनतहहॴनजसमहॳरकतमिािकह८लिकाओकीयाहीमह८गिह८बबनकीकमीहह८तीहहॴ 29-Jan-2017

Options

1) Albinism

एनमबननजम

2) Propyria

परह८पीररया 3) Anaemia

एनीलमया 4)Keloid disorder

कहॳ िह८इिडिसओिर

Correct Answer Anaemia

Q173 Ananas comosus is the scientific

name of

Options

अनानासकह८मह८सस mdashmdashmdashndash

कावहॴजञाननकनामहहॴ 29-Jan-2017

1) Custard Apple

सीताफि

2) Pineapple

पाइनएपपि

3) Bamboo

बास

4)Pomegranate

अनार

Correct Answer Pineapple

Q174 Which organ produces insulin

कह९नसाअगइनसलिनपहॴदाकरताहहॴ 29-Jan-

2017

Options

1) Liver

यकत

2) Thyroid gland

िायराइिगरिी 3) Spleen

पिीहा 4)Pancreas

अगरयिय

Correct Answer Pancreas

Q175 Which of the following disease is

not caused by water pollution

नननननलिखितमसहॳकह९नसारह८गपानीकहॳ परदषणकहॳकारणनहीहह८ता

29-Jan-2017

Options

1) Cholera

हहॴजा 2) Typhoid

F A C E B O O K

P A G E h t t p w w w f a c e b o o k c o m s s c m e n t o r s o f f i c i a l P a g e | 41

FOR MORE UPDATES AND MORE MATERIAL DO LIKE OUR FACEBOOK PAGE httpwwwfacebookcomsscmentorsofficial

टाइफाइि

3) Asthma

दमा 4)Diarrhoea

दसत

Correct Answer Asthma

Q176 Ocimum tenuiflorum is the

scientific name of

ओलिलममटहॳयईफिह८रमइसकावहॴजञाननकनाम mdash

ndash हहॴ 30-Jan-2017

Options

1) Neem

नीम

2) Mango

आम

3) Babul

बबि

4)Tulsi

तिसी Correct Answer Tulsi

Q177 Which gland secretes bile a

digestive fluid

कह९नसीगरिीपपतत एकपाचनतरिपरदािय सरापवतकरतीहहॴ 30-Jan-2017

Options

1) Pancreas

अगनयािय

2) Liver

यकत

3) Thyroid

िायराइि

4) Testes

टहॳनसटस

Correct Answer liver

Q178 In which of the following the

dominant phase is Gametophyte

नननननलिखितमसहॳककसकहॳ परमिचरणयगमकह८दपवधद (Gametophyte)हहॴ 30-Jan-2017

Options

1) Bryophyta

िायह८फाइटा 2) Pteridophyta

टहॳररिह८फाइटा 3) Gymnosperms

नजननह८सपमय 4) Angiosperms

एननजयह८सपमय Correct Answer Bryophyta

Q179 Anaerobic respiration refers to

which of the following

नननननलिखितमसहॳककसहॳअवायवीयशवसनकहाजाताहहॴ

30-Jan-2017

Options

1) Respiration without Oxygen

ऑकसीजनकहॳ बबनाशवसन

2) Respiration with Oxygen

ऑकसीजनकहॳ सािशवसन

3) Respiration without CO2

काबयनिायऑकसाइिकहॳ बबनाशवसन

4) Respiration with CO2

काबयनिायऑकसाइिकहॳ सािशविन

Correct Answer Respiration without

Oxygen

Q180 Which type of pathogen causes

the waterborne disease Cholera

ककसपरकारकारह८गजनकजिजननतरह८गहहॴजाकाकारणबनताहहॴ

30-Jan-2017

Options

1) Algal

िहॴवालियहॳ

F A C E B O O K

P A G E h t t p w w w f a c e b o o k c o m s s c m e n t o r s o f f i c i a l P a g e | 42

FOR MORE UPDATES AND MORE MATERIAL DO LIKE OUR FACEBOOK PAGE httpwwwfacebookcomsscmentorsofficial

2) Bacterial

बहॴकटीररयि

3) Protozoan

परह८टह८जआ

4) Viral

वायरि

Correct Answer Bacterial

Q181 To which class does

Oxyreductases transferases hydrolases

belong

ओकसीररिकटहॳसटरासफरहॳजहॳस

हाइडरह८िहॳसहॳसककसवगयमआतहॳहहॴ 30-Jan-2017

Options

1) Hormones

हारमोस

2) Enzymes

एजाइनस

3) Proteins

परह८टीनस

4) Vitamins

पवटालमनस

Correct Answer Enzymes

Q182 Which of the following is not true

about Gymnosperms

ननननमसहॳकह९नसीबातअनावतबीजीकहॳ बारहॳमसचनहीहहॴ 30-Jan-2017

Options

1) Dominant phase is saprophytes

परमिचरणसहॳपरह८फाइटसहह८ताहहॴ 2) Vascular bundles are absent

सवहनीबििअनपनसितहह८ताहहॴ 3) spores are heterospores

बीजाणहहॳटहॳरह८सपह८रसहह८तहॳहहॴ 4) Flowers are absent

फिअनपनसितहह८तहॳहहॴ

Correct Answer Vascular bundles are

absent

Q183 The name of first mammal clone sheep is

भहॳड़कीपरिमसतनपायीपरनत प (किह८न)

कानामहहॴ 30-Jan-2017

Options

1) Noori

नरी 2) Dolly

िॉिी 3) Louise

िसी 4)Durga

दगाय Correct Answer Dolly

Q184 Which type of pathogen causes

the water-borne disease Typhoid fever

ककसपरकारकारह८गजनकजिजननतरह८गटाइफाइिबिारकाकारणबनताहहॴ 30-Jan-2017

Options

1) Algal

िहॴवािीय

2) Parasitic

परजीवी 3) Protozoan

परह८टह८जनअन

4)Bacterial

बहॴकटीररयि

Correct Answer Bacterial

Q185 In which part of the cell are

proteins made

कह८लिकाकहॳ ककसटहससहॳमपरह८टीनबनायाजाताहहॴ

31-Jan-2017

Options

1) Reticulum

रहॳटटकिम

F A C E B O O K

P A G E h t t p w w w f a c e b o o k c o m s s c m e n t o r s o f f i c i a l P a g e | 43

FOR MORE UPDATES AND MORE MATERIAL DO LIKE OUR FACEBOOK PAGE httpwwwfacebookcomsscmentorsofficial

2) Golgi apparatus

गह८मजीएपहॳरहॳटस

3) Ribosomes

ररबह८सह८नस

4) Lysosome

िायसह८सह८नस

Correct Answer ribosomes

Proteins are produced by stringing

amino acids together in the order

specified by messenger RNA strands

that were transcribed from DNA in the

cell nucleus The process of synthesizing

a protein is called translation and it

occurs on ribosomes in the cytoplasm of

a cell

Q186 Polio is a disease caused by which

of the following

नननननलिखितमसहॳपह८लियह८कीबबमारह८हह८नहॳकाकारणकयाहहॴ

31-Jan-2017

Options

1) Bacteria

बहॴकटीररयि

2) Mosquito

मचछर

3) Virus

वायरस

4) Cockroach

नतिच हॳ Correct Answer Virus

Polio or poliomyelitis is a crippling and

potentially deadly infectious disease It

is caused by the poliovirus

Q187 ndash Hay fever is a sign of which of

the following

हहॳकफवरनननननलिखितमसहॳककसकाएकसकहॳ तहहॴ

31-Jan-2017

Options

1) Old Age

वदावसिा 2) Malnutrition

कपह८सण

3) Allergy

एिनजय 4) Over Work

अतयचधककाययकरना Correct Answer Allergy

Q188 How many chromosomes does a

human cell contain

एकमानवकह८लिकामककतनहॳगणसतरहह८तहॳहहॴ

29-Jan-2017

Options

1) 6

2) 26

3) 46

4) 66

Correct Answer 46

In humans each cell normally contains

23 pairs of chromosomes for a total of

46 Twenty-two of these pairs called

autosomes look the same in both males

and females The 23rd pair the sex

chromosomes differ between males and

females

Q189 Which of the following is not true

about Bryophyta

ननननमसहॳकह९नसीबातिायह८फाइटकहॳ बारहॳमसचनहीहहॴ 31-Jan-2017

Options

1) Dominant phase is gametophytes

परमिचरणगहॳलमतह८फाइटसहह८ताहहॴ 2) Main plant body is haploid

पह९धहॳकामखयिरीरअगखणतहह८ताहहॴ 3) Spores are homospores

बीजाणहह८मह८सफह८रसहह८तहॳहहॴ 4) Flowers are present

फिमह८जदहह८तहॳहहॴ Correct Answer Flowers are present

F A C E B O O K

P A G E h t t p w w w f a c e b o o k c o m s s c m e n t o r s o f f i c i a l P a g e | 44

FOR MORE UPDATES AND MORE MATERIAL DO LIKE OUR FACEBOOK PAGE httpwwwfacebookcomsscmentorsofficial

Q190 Which aquatic animal has

trailing tentacles

ककसजिीयजानवरकहॳ पीछहॳचिनहॳवािहॳटहॳटकिसहह८तहॳहहॴ

31-Jan-2017

Options

1) Sea horse

समदरीघह८िा 2) Corals

मगा 3) Jelly fish

जहॳिीमछिी 4) Star fish

तारामछिी Correct Answer Jelly fish

Jellyfish with its umbrella-shaped bell

and trailing tentacles

Q191 Which type of pathogen causes

the water-borne disease Poliomyelitis

(Polio)

ककसपरकारकारह८गजनकजिजननतरह८गपह८लियह८मायहॳटटस (पह८लियह८) काकारणहहॴ 31-Jan-

2017

Options

1) Parasitic

परजीवी 2) Algal

िहॴवालिय

3) Viral

वायरि

4) Bacterial

बहॴकटीररयि

Correct Answer Viral

Q192 The outer white part of the eye

that protects the inner structures is

आािकाबाहरीसफहॳ दटहससाजह८आतररकसरचनाओकीरकषाकरताहहॴ वह mdashmdashmdash हहॴ 31-Jan-

2017

Options

1) Iris

आयररस

2) Sclera

सकिहॳरा 3) Retina

रहॳटटना 4) Cornea

कह८ननयया Correct Answer Sclera

Q193 Proteins are made up of

परह८टीनकाननमायण mdashndash सहॳहह८ताहहॴ 31-Jan-2017

Options

1) Amino acids

एलमनह८अनि

2) Fatty acids

वसायकतअनि

3) Glucose

गिकह८ज

4)Nucleotides

नयनकियह८टाईिस

Correct Answer Amino acids

Q194 Moringa Oleifera is the scientific

name of

मह८ररगओलिफहॳ रा mdashmdashndash कावहॴजञाननकनामहहॴ 31-Jan-2017

Options

1) Banyan

बरगद

2) Gulmohar

गिमह८हर

3) Amla

आमिा

F A C E B O O K

P A G E h t t p w w w f a c e b o o k c o m s s c m e n t o r s o f f i c i a l P a g e | 45

FOR MORE UPDATES AND MORE MATERIAL DO LIKE OUR FACEBOOK PAGE httpwwwfacebookcomsscmentorsofficial

4) Drumstick

डरमनसटक

Correct Answer Drumstick

Q195 Kidney stones are composed of

गदकीपिरी mdashndash सहॳबनीहह८तीहहॴ 1-Feb-2017

Options

1) Calcium Oxalate

कहॴ नमसयमओकजहॳिहॳट

2) Sodium Chloride

सह८डियमकिह८राइि

3) Magnesium Nitrate

महॳनगनलियमनाइतटरहॳट

4) Calcium Bicarbonate

कहॴ नमियमबायकबोनहॳट

Correct Answer Calcium Oxalate

Q196 ndash Which of the following is not

true about Angiosperms

ननननमसहॳकह९नसीबातआवतबीजीकहॳ बारहॳमसचनहीहहॴ 1-Feb-2017

Options

1) Dominant phase is gametophytes

परमिचरणगहॳलमतह८फाइटहह८ताहहॴ 2) Vascular bundles are present

सवहनीबििमह९जदहह८ताहहॴ 3) Spores are heterospores

बीजाणहहॳटहॳरह८सपह८रसहह८तहॳहहॴ 4) Seeds are covered

बीजढकहॳ हह८तहॳहहॴ Correct Answer Dominant phase is

gametophytes

Q197 All of the following are excretory

(waste) products of animals except

नननननलिखितमसहॳककसएककह८छह८ड़करअनयसभीपराखणयोदवाराउतसनजयतपदाियहहॴ 1-Feb-

2017

Options

1) Uric Acid

यररकएलसि

2) Ammonia

अमह८ननया 3) Carbohydrates

काबोहाइडरहॳट

4) Urea

यररया Correct Answer Carbohydrates

In animals the main excretory products

are carbon dioxide ammonia (in

ammoniotelics) urea (in ureotelics) uric

acid (in uricotelics) guanine (in

Arachnida) and creatine

Q198 RNA is a polymeric molecule

What does RNA stand for

आरएनइएएकबहिकआणहहॴ इसकाकापवय पकयाहहॴ 1-Feb-2017

Options

1) Rado Nuclear Acid

रािह८नयनकियरएलसि

2) Ribo Nucleic Acid

राइबह८नयनकिकएलसि

3) Rhino Nuclear Acid

हाइनह८नयनकियरएलसि

4) Resto Nucleus Acid

रहॳसटह८नयकिीयसएलसि

Correct Answer Ribo Nucleic Acid

Q199 Which organ does detoxification

and produces chemicals needed for

digestion

कह९नसाअगपवषहरणकरताहहॴऔरपाचनकहॳ लिएआवशयकरसायनोकह८पहॴदाकरताहहॴ 1-Feb-

2017

Options

1) Salivary glands

िारगरचिया 2) Pancreas

अगनयािय

F A C E B O O K

P A G E h t t p w w w f a c e b o o k c o m s s c m e n t o r s o f f i c i a l P a g e | 46

FOR MORE UPDATES AND MORE MATERIAL DO LIKE OUR FACEBOOK PAGE httpwwwfacebookcomsscmentorsofficial

3) Thyroid gland

िायराइिगरिी 4) Liver

यकत

Correct Answer Liver

Q200 Psidium guajava is the scientific

name of

लसडियमगआजावा mdashmdash कावहॴजञाननकनामहहॴ 1-

Feb-2017

Options

1) Guava

अम द

2) Mango

आम

3) Bamboo

बास

4) Jack fruit

कटहि

Correct Answer Guava

Q201 Which drug is used as a Blood

Thinner

चधरकह८पतिाकरनहॳकहॳ पमककसदवाकापरयह८गककयाजाताहहॴ

1-Feb-2017

Options

1) Warfarin

वाफर न

2) Tramadol

टरहॳमािह८ि

3) Azithromycin

एनजरह८मायलसन

4) Hydralazine

हाइडरह८िहॳनजन

Correct Answer Warfarin

Q202 Which of the following disease is

caused due to the deficiency of protein

परह८टीनकीकमीकहॳ कारणनननननलिखितमसहॳकह९नसारह८गहह८ताहहॴ 1-Feb-2017

Options

1) Arthritis

गटठया 2) Kwashiorkor

कािीओकय र

3) Goitre

गाइटर

4) Night Blindness

रतह९चध

Correct Answer Kwashiorkor

Q203 A is species of plant that has

adapted to survive in an environment

with little liquid water

mdashmdashndashपह९धहॳकीएकऐसहॳऐसहॳपरजानतहहॴ नजसनहॳकमपानीवािहॳवातावरणमजीपवतरहनहॳकहॳलिएअनकिनहहॴ 1-Feb-2017

Options

1) Xerophyte

म दपवद

2) Hydrophyte

जिीयपादप

3) Mesophyte

समह८दपवद

4) Thallophyte

िहॴिह८फाइटा Correct Answer xerophyte

xerophyte is a species of plant that has

adapted to survive in an environment

with little liquid water such as a desert

or an ice- or snow-covered region in the

Alps or the Arctic

Mesophytes are terrestrial plants which

are adapted to neither a particularly

dry nor particularly wet environment

An example of a mesophytic habitat

would be a rural temperate meadow

F A C E B O O K

P A G E h t t p w w w f a c e b o o k c o m s s c m e n t o r s o f f i c i a l P a g e | 47

FOR MORE UPDATES AND MORE MATERIAL DO LIKE OUR FACEBOOK PAGE httpwwwfacebookcomsscmentorsofficial

which might contain goldenrod clover

oxeye daisy and Rosa multiflora

thallophyte any of a group of plants or

plantlike organisms (such as algae and

fungi) that lack differentiated stems

leaves and roots and that were formerly

classified as a primary division

(Thallophyta) of the plant kingdom

Q204 How many types of teeth are

there in humans

मनषयोमककतनहॳपरकारकहॳ दातहह८तहॳहहॴ

1-Feb-2017

Options

1) 4

2) 5

3) 2

4) 3

Correct Answer 4

teeth -Humans have four types of

teethincisors canines premolars and

molars each with a specific function

The incisors cut the food the canines

tear the food and the molars and

premolars crush the food

Q205 Carica papaya is the scientific name of

कहॴ ररकापपाया mdashmdashndash कावहॴजञाननकनामहहॴ 2-

Feb-2017

Options

1) Peepal

पीपि

2) Papaya

पपीता 3) Tamarind

इमिी 4) Drumstick

ढह८िकाछड़ी Correct Answer Papaya

Q206 Muscles get tired when there is

shortfall of

जब mdashndash कीकमीहह८तीहहॴतबपहॳिीयिकजातीहहॴ 2-Feb-2017

Options

1) Lactic acid

िहॴनकटकएलसि

2) Na+ ions

Na+ आयन

3) ATP

एटीपी 4) Sulphates

समफहॳ टस

Correct Answer ATP

ATP is the energy source muscle fibers

use to make muscles contract

muscle tissuersquos main source of energy

called adenosine triphosphate or ATP

As your muscles use up this energy

source they become tired and fatigued

Oxygen is the key ingredient that helps

create new ATP to replenish the burned

up ATP in your muscles

Q207 Artocarpus integra is the

scientific name of आटह८कापयसइटीगरा mdashmdashmdash कावहॴजञाननकनामहहॴ 2-Feb-2017

Options

1) Guava

अम द

2) Pineapple

अनानास

3) Silver Oak

लसमवरओक

4) Jack fruit

कटहि

Correct Answer Jack fruit

Q208 Which organ stores fat soluble

vitamins

कह९नसाअगवसामघिनिीिपवटालमनह८काभिाराकरताहहॴ

2-Feb-2017

F A C E B O O K

P A G E h t t p w w w f a c e b o o k c o m s s c m e n t o r s o f f i c i a l P a g e | 48

FOR MORE UPDATES AND MORE MATERIAL DO LIKE OUR FACEBOOK PAGE httpwwwfacebookcomsscmentorsofficial

Options

1) Blood

रकत

2) Skin

तवचा 3) Liver

यकत

4) Pancreas

अगनयािय

Correct Answer Liver

Q209 Which disease is caused due to

deficiency of Iodine

आयह८िीनकहॳ कारणकह९नसारह८गहह८ताहहॴ 2-Feb-2017

Options

1) Rickets

ररकहॳ टस

2) Scurvy

सकवी 3) Goitre

गणमािा 4) Growth retardation

पवकासका कना Correct Answer Goitre

rickets A softening and weakening of

bones in children usually due to

inadequate vitamin D

Q210 Grevillea Robusta is the scientific name of

गरहॳपवलियारह८बसटा mdashmdashmdash- कापवजञाननकनामहहॴ 2-Feb-2017

Options

1) Peepal

पीपि

2) Teak

सागह९न

3) Silver Oak

लसमवरओक

4) Jack fruit

कटहि

Correct Answer Silver Oak

Q211 When a Cuttlefish is described as a Molluscs it is at which level of

classification

जबएककटिकफिकह८एकमह८िसकाकहॳ पमवखणयतककयाजाताहहॴतबयहॳवगीकरणकहॳ ककससतरपहॳनसितहहॴ 2-Feb-2017

Options

1) Class

वगय 2) Order

िम

3) Family

पररवार

4) Phylum

सघ

Correct Answer Phylum

Q212 Bambusa dendrocalmus is the

scientific name of बानबसािहॳडराकामस mdashmdashmdash कावहॴजञाननकनामहहॴ 3-Feb-2017

Options

1) Banyan

बरगद

2) Papaya

पपीता 3) Bamboo

बास

4) Pomegranate

अनार

Correct Answer Bamboo

Q213 Acinonyx Jubatus is the scientific name of

एलसनह८ननकसजयबहॳटस mdashmdashmdash

कावहॴजञाननकनामहहॴ 3-Feb-2017

F A C E B O O K

P A G E h t t p w w w f a c e b o o k c o m s s c m e n t o r s o f f i c i a l P a g e | 49

FOR MORE UPDATES AND MORE MATERIAL DO LIKE OUR FACEBOOK PAGE httpwwwfacebookcomsscmentorsofficial

Options

1) Bear

भाि 2) Horse

घह८िा 3) Cheetah

चीता 4) Zebra

जहॳिा Correct Answer Cheetah

Q214 The pale yellow colour of urine is

due to the presence of which pigment

मतरकाफीकापीिारगरगदरयकहॳ उपनसिनतकहॳ कारणहह८ताहहॴ

3-Feb-2017

Options

1) Urochrome

यरह८िह८म

2) Urophyll

यरह८कफि

3) Chlorophyll

किह८रह८कफि

4) Chloroplast

किह८रह८पिासट

Correct Answer Urochrome

Q215 Which of the following constitute

to form a gene

नननननलिखितमसहॳकह९नसीचीज़एकजीनकागठनकरतीहहॴ

3-Feb-2017

Options

1) Polynucleotides

पह८िीनयनकियह८टाईडस

2) Hydrocarbons

हाइडरह८काबोस

3) Lipoproteins

िाईपह८परह८टीनस

4) Lipids

लिपपडस

Correct Answer Polynucleotides

Polynucleotide molecule is a biopolymer

composed of 13 or more nucleotide

monomers covalently bonded in a chain

DNA (deoxyribonucleic acid) and RNA

(ribonucleic acid) are examples of

polynucleotides with distinct biological

function

Q216 Vertebrates belongs to the

phylum

रीढ़कीहडिीवािहॳपराणी mdashmdashmdash

परजानतकहॳ अतगायतआतहॳहहॴ 3-Feb-2017

Options

1) Arthropoda

आरह८पह८ड़ा 2) Annelida

एननलििा 3) Cnidaria

ननिहॳररया 4) Chordata

कह८िटा Correct Answer Chordata

Q217 Punica granatum is the scientific name of

पननकगरहॳनहॳटस mdashmdashmdash कावहॴजञाननकनामहहॴ 3-Feb-2017

Options

1) Custard Apple

सीताफि

2) Gulmohar

गिमह८हर

3) Silver Oak

लसमवरओक

4) Pomegranate

अनार

Correct Answer Pomegranate

F A C E B O O K

P A G E h t t p w w w f a c e b o o k c o m s s c m e n t o r s o f f i c i a l P a g e | 50

FOR MORE UPDATES AND MORE MATERIAL DO LIKE OUR FACEBOOK PAGE httpwwwfacebookcomsscmentorsofficial

Q218 Between a tiger and an monkey

which of the following is different

एकबाघऔरबदरकहॳ बीचनननननलिखितमसहॳकह९नसीबातअिगहहॴ 3-Feb-2017

Options

1) Kingdom

राजय

2) Phylum

जानत

3) Order

िम

4) Class

वगय Correct Answer order

Q219 The artificial heart was invented by

कबतरमहदयका mdashmdashmdash

दवाराअपवषकारककयागयािा 3-Feb-2017

Options

1) Muhammad Yunus

महनमदयनस

2) Linus Yale Jr

िाइनसयहॳिजय

3) Gazi Yasargil

गाजीयासचगयि

4) Paul Winchell

पह९िपवमकि Correct Answer Paul Winchell

Q220 Tamarindus indica is the

scientific name of

टहॳमररनडसइडिका mdashmdash कावहॴजञाननकनामहहॴ 7-

Feb-2017

Options

1) Neem

नीम

2) Pineapple

अनानास

3) Tamarind

इमिी 4)Chiku

चीक

Correct Answer Tamarind

Q221 In eukaryotic cells synthesis of

RNA takes place in the

यकहॳ योटटककह८लिकाओमआरएनएकासशिहॳषण

mdashndash महह८ताहहॴ 7-Feb-2017

Options

1) Mitochondria

माईटह८कोडडरया 2) Centrioles

सटरीयह८मस

3) Ribosomes

ररबह८सह८नस

4) Nucleus

नयनकियस

Correct Answer nucleus

eukaryotic cell -Transcription is the

process of synthesizing ribonucleic acid

(RNA)Synthesis takes place within the

nucleus of eukaryotic cells or in the

cytoplasm of prokaryotes and converts

the genetic code from a gene in

deoxyribonucleic acid ( DNA ) to a

strand of RNA that then directs

proteinsynthesis

Q222 _________is caused by parasites

of the Plasmodium genus

पिाजमह८डियमजातीकहॳ परजीवी mdash- कहॳ कारणहहॴ 7-Feb-2017

Options

1) Dysentery

पहॳचचि

2) Malaria

मिहॳररया 3) Chickenpox

F A C E B O O K

P A G E h t t p w w w f a c e b o o k c o m s s c m e n t o r s o f f i c i a l P a g e | 51

FOR MORE UPDATES AND MORE MATERIAL DO LIKE OUR FACEBOOK PAGE httpwwwfacebookcomsscmentorsofficial

चहॳचक

4) Herpes

हहॳपपयस

Correct Answer Malaria

Q223 Carotene in fruits and vegetables

gives it which color

फिह८औरसनलजयोमनसितकहॳ रह८टीनउनहकह९नसारगपरदानकरताहहॴ 7-Feb-2017

Options

1) Green

हरा 2) Pink

गिाबी 3) Orange

नारगी 4) Blue

नीिा Correct Answer Orange

Q224 Equus Caballus is the scientific

name of

एकवसकहॴ बहॳिस mdashmdashndash कापवजञाननकनामहहॴ 7-Feb-2017

Options

1) Horse

घह८िा 2) Zebra

ज़हॳिा 3) Donkey

गधा 4) Buffalo

भस

Correct Answer Horse

Q225 Elapidae Naja is the scientific name of

एिीपीिीनाजा mdashmdash- कावहॴजञाननकनामहहॴ 8-Feb-2017

Options

1) Cobra

कह८बरा 2) Elephant

हािी 3) Eagle

ग ि

4) Owl

उमि Correct Answer Cobra

Q226 Which disease is caused due to

deficiency of Iron

िह८हकीकमीकहॳ कारणकह९नसारह८गहह८ताहहॴ 8-Feb-

2017

Options

1) Beriberi

बहॳरीबहॳरी 2) Tetany

टहॳटनी 3) Kwashiorkor

कवािीऔरकर

4) Anaemia

रकतामपता Correct Answer Anaemia

Beriberi is a disease caused by a vitamin

B-1 deficiency also known as thiamine

deficiency

Tetany can be the result of an

electrolyte imbalance Most often itrsquos a

dramatically low calcium level also

known as hypocalcemia Tetany can also

be caused by magnesium deficiency or

too little potassium Having too much

acid (acidosis) or too much alkali

(alkalosis) in the body can also result in

tetany

Kwashiorkor also known as

ldquoedematous malnutrition It is a form of

malnutrition caused by a lack of protein

in the diet

Anaemia means that you have fewer red

blood cells than normal or you have less

F A C E B O O K

P A G E h t t p w w w f a c e b o o k c o m s s c m e n t o r s o f f i c i a l P a g e | 52

FOR MORE UPDATES AND MORE MATERIAL DO LIKE OUR FACEBOOK PAGE httpwwwfacebookcomsscmentorsofficial

haemoglobin than normal in each red

blood cell

Q227 is a leaf where the leaflets are

arranged along the middle vein

mdashndashएकपततीहहॴजहापतरकह८कीरचनाक ररयालिराकहॳ आसपासहह८तीहहॴ 8-Feb-2017

Options

1) Pinnately compound leaf

पपनहॳटिीसयकतपतती 2) Palmately compound leaf

पामहॳटिीसयकतपतती 3) Compound leaf

सयकतपतती 4) Simple leaf

साधारणपतती Correct Answer Pinnately compound

leaf

Q228 Haustoria or sucking roots are

found in which of the following

हह८सटह८ररयायाचसनहॳवािीजड़हॳनननननलिखितमसहॳककसमपाईजातीहहॴ 8-Feb-2017

Options

1) Wheat

गहॳह

2) Mango

आम

3) Chestnut

चहॳसटनट

4) Cuscuta

कसकयटा Correct Answer Cuscuta

Haustorial roots -The roots of parasitic

plants which penetrate into the host

tissues to absorb nourishment are

called haustorial roots hellip Also known as suckingor parasitic roots

Q229 Equs Asinus is the scientific name

of

एकवसएलसनस mdashmdashndash कावहॴजञाननकनामहहॴ 8-

Feb-2017

Options

1) Donkey

गधा 2) Cow

गाय

3) Deer

टहरन

4) Kangaroo

कगा

Correct Answer Donkey

Q230 Ficus benghalensis is the scientific name of

फाईकसबहॳनगहॳिहॳलसस mdashndash कापवजञाननकनामहहॴ 8-Feb-2017

Options

1) Banyan

बरगद

2) Pineapple

अनानास

3) Babul

बबि

4) Tulsi

तिसी Correct Answer Banyan

Q231 Equus burchellii is the scientific name of

एकवसबचिी mdashmdash- कापवजञाननकनामहहॴ 8-Feb-2017

Options

1) Horse

घह८िा 2) Zebra

जहॳिा 3) Buffalo

F A C E B O O K

P A G E h t t p w w w f a c e b o o k c o m s s c m e n t o r s o f f i c i a l P a g e | 53

FOR MORE UPDATES AND MORE MATERIAL DO LIKE OUR FACEBOOK PAGE httpwwwfacebookcomsscmentorsofficial

भस

4) Ass

गधा Correct Answer Zebra

Page 6: COMPILATION OF ALL 72 SETS OF BIOLOGY SSC CHSL-2016 · OF BIOLOGY SSC CHSL-2016 PREPARED BY : SSC MENTORS BIOLOGY SPECIAL . F A C E B O O K P A G E : h t t p : / / w w w . f a c e

F A C E B O O K

P A G E h t t p w w w f a c e b o o k c o m s s c m e n t o r s o f f i c i a l P a g e | 5

FOR MORE UPDATES AND MORE MATERIAL DO LIKE OUR FACEBOOK PAGE httpwwwfacebookcomsscmentorsofficial

1) Night Blindness

रतोधी 2) No clotting of Blood

रकतकािककानजमना 3) Rickets

ररकहॳ ट

4) Loss of haemoglobin

टहमह८गिह८बबनकीअमपता Correct Answer No clotting of Blood

Haemophilia also spelled hemophilia is

a mostly inherited genetic disorder that

impairs the bodyrsquos ability to make blood

clots a process needed to stop bleeding

This results in people bleeding longer

after an injury easy bruising and an

increased risk of bleeding inside joints

or the brain

Q15 The process of pollination by birds

is also known as

पकषकषयोदवाराकीजानहॳवािीपरागणकीपरनतकियाकह८mdashndash कहॳ नामसहॳभीजानाजाताहहॴ 9-Jan -2017

Options

1) Hydrophily

हाइडरह८कफिी 2) Entomophily

एनटोमह८कफिी 3) Embryophily

एननियह८कफिी 4) Ornithophily

ओननयिह८कफिी Correct Answer Ornithophily

Q16 Spiders belong to the phylum

मकडड़याककसपरजानतकहॳ अतगयतआतीहहॴ 9-Jan -2017

Options

1) Mollusca

मह८िसका 2) Annelida

एननलििा 3) Cnidaria

ननिहॳररया 4) Arthropoda

अरोपह८ड़ा Correct Answer Arthropoda

Q17 Banana freckle is a plant disease

It is caused by a

कहॳ िहॳकीझाईपह८धह८कीएकबीमारीहहॴ यहएक mdashndash

कहॳ कारणहह८तीहहॴ 9-Jan -2017

Options

1) Virus

वायरस

2) Fungus

कवक

3) Bacteria

बहॴकटीररया 4) Insect

कीटक

Correct Answer Fungus

Banana Freckle is a disease caused by

the fungus Guignardia musae

(telomorph) or Phyllosticta musarum (

anamorph )

Q18 Which of the following Indian

chilly is considered one of the hottest in

the world

नननननलिखितभारतीयलमचचययह८मसहॳकह९नसीसबसहॳकह९नसीपवशवकीसबसहॳतीिीलमचचययह८मसहॳएकमानीजातीहहॴ

9-Jan -2017

Options

1) Bhut Jolokia

भतझह८िककया 2) Bhut Mahabora

भतमहाबह८रा 3) Lal Chitin

F A C E B O O K

P A G E h t t p w w w f a c e b o o k c o m s s c m e n t o r s o f f i c i a l P a g e | 6

FOR MORE UPDATES AND MORE MATERIAL DO LIKE OUR FACEBOOK PAGE httpwwwfacebookcomsscmentorsofficial

िािचीटटन

4) Lal Shamak

िाििामक

Correct Answer Bhut Jolokia

Q19 Brain fever is a disease spread

through which of the following

मनसतषकजवरनमकरह८गनननननलिखितमसहॳककसकहॳ कारणहह८ताहहॴ 9-Jan -2017

Options

1) Flies

मनकियो 2) Mosquito

मचछर

3) Virus

वायरस

4) Cockroach

नतिच हॳ Correct Answer Mosquito

Q20 Mangroves are plants that have

मगरह८ववहॳपहॳिहहॴनजनमहॳहह८ताहहॴ 9-Jan -2017

Options

1) Modified Roots

पातररतजड़हॳ 2) Modified Stems

पातररततनहॳ 3) Respiratory Roots

शरवसनकरनहॳवािीजड़हॳ 4) Respiratory Stems

शरवसनकरनहॳवािीतनहॳ Correct Answer Respiratory Roots

A mangrove is a shrub or small tree that

grows in coastal saline or brackish

water

Q21 Rodentia Sciurus is the scientific

name of

रह८िहॳलियासकीयरस mdashmdash कावयजजञाननकनामहहॴ

9-Jan -2017

Options

1) Rat

चहा 2) Platypus

पिहॳटीपस

3) Squirrel

चगिहरी 4) Beaver

बीवर

Correct Answer Squirrel

Q22 Which of the following is induced

by Oncogene

नननननलिखितमसहॳकह९नओकह८जीनदवारापरहॳररयतहह८ताहहॴ

10-Jan -2017

Options

1) Polio

पह८लियह८ 2) Cancer

क सर

3) Diarrhoea

दसत

4) Dengue

िग Correct Answer Cancer

An oncogene is a gene that has the

potential to cause cancer In tumor

cells they are often mutated andor

expressed at high levels

Q23 Azadirachata indica is the

scientific name of

अजाटदराकटाइडिका mdashmdashवहॴजञाननकनामहहॴ SSC CHSL Science (biology) 2016

Question Paper

10-Jan -2017

Options

1) Neem

नीम

F A C E B O O K

P A G E h t t p w w w f a c e b o o k c o m s s c m e n t o r s o f f i c i a l P a g e | 7

FOR MORE UPDATES AND MORE MATERIAL DO LIKE OUR FACEBOOK PAGE httpwwwfacebookcomsscmentorsofficial

2) Teak

सागह९न

3) Silver Oak

लसमवरओक

4) Tulsi

तिसी Correct Answer Neem

Q24 Octopus belongs to the phylum

ऑकटह८पसककसपरजानतकहॳ अतगयतआताहहॴ 10-

Jan -2017

Options

1) Mollusca

मह८िसका 2) Cnidaria

ननिहॳररया 3) Echinodermata

इकाइनह८ड़हॳमता 4) Chordata

कह८िता Correct Answer Mollusca

Q25 A living part of the organisms

environment is known as

जीवाणकहॳ वातावरणकहॳ जीपवतभागकह८ mdash-

कहतहॳहहॴ 10-Jan -2017

Options

1) Abiotic Factor

अजहॴपवककारक

2) Habitat

आवास

3) Biotic Factor

जहॴपवककारक

4) Nonliving factor

अ-जीपवतकारक

Correct Answer Biotic Factor

Abiotic factors are nonndash living chemical

and physical parts of the environment

that affect living organisms and the

functioning of ecosystems like rain

wind temperature altitude soil

pollution nutrients pH types of soil

and sunlight

Q26 Medulla oblongata is a part of

which of the following

महॳडयिाऑबिॉनगहॳटानननननलिखितमसहॳककसअगकाटहससाहहॴ

10-Jan -2017

Options

1) Heart

हदय

2) Brain

मनसतषक

3) Lungs

फहॳ फड़हॳ 4) Stomach

पहॳट

Correct Answer Brain

The medulla oblongata helps regulate

breathing heart and blood vessel

function digestion sneezing and

swallowing This part of the brain is a

center for respiration and circulation

Sensory and motor neurons (nerve cells)

from the forebrain and midbrain travel

through the medulla

Q27 ___________ is a typically

onecelled reproductive unit capable of

giving rise to a new individual without

sexual fusion

mdashmdash एकआमतह९रपरएककह८लिकीयहॳ परजननममसमकषइकाईहहॴजह८यह९नसियनकहॳ बबनाएकनयीइकाईकह८जनमदहॳतीहहॴ 10-Jan -2017

Options

1) Egg

अिाण

2) Spore

बीजाण

F A C E B O O K

P A G E h t t p w w w f a c e b o o k c o m s s c m e n t o r s o f f i c i a l P a g e | 8

FOR MORE UPDATES AND MORE MATERIAL DO LIKE OUR FACEBOOK PAGE httpwwwfacebookcomsscmentorsofficial

3) Sperm

ििाण

4) Seed

बीज

Correct Answer Spore

Q28 Bacteria was discovered by

बहॴकटीररयाकीिह८जककसकहॳ दवाराकीगयीिी

10-Jan -2017

1) Antonie van Leeuwenhoek

एटह८नीवहॳनलिबहॳनहक

2) Belarus

बहॳिा स

3) Hugo de Vries

हयगह८दीराईस

4)Robert Brown

रॉबटयिाउन

Correct Answer Antonie van

Leeuwenhoek

Q29 Which of the following is

responsible for Vermicomposting

नननननलिखितमसहॳकह९नकलमिादकहॳ लिएनजनमहॳदारहहॴ

10-Jan -2017

Options

1) Fungus

कवक

2) Worms

कलम

3) Bacteria

बहॴकटीररया 4) Birds

पकषी Correct Answer Worms

Vermicompost (or vermi-compost) is the

product of the composting process using

various species of worms usually red

wigglers white worms and other

earthworms to create a heterogeneous

mixture of decomposing vegetable or

food waste bedding materials and

vermicast

Q30 Scurvy (bleeding of gums) is

caused by the deficiency of which

vitamin

सकवी (मसढह८सहॳिनआना) ककसपवटालमनकीकमीकहॳ कारणहह८ताहहॴ

10-Jan-2017

Options

1) Vitamin K

पवटालमन K

2) Vitamin BZ

पवटालमन BZ

3) Vitamin C

पवटालमन C

4) Vitamin A

पवटालमन A

Correct Answer Vitamin C

Q31 Achras sapota is the scientific

name of

एिाससपह८ताइसकावहॴजञाननकनामहहॴ 10-Jan-2017

Options

1) Custard Apple

सीताफि

2) Gulmohar

गिमह८हर

3) Tamarind

इमिी 4) Chiku

चचक

Correct Answer Chiku

Q32 Prawn belongs to the phylum

झीगा mdashmdash- परजानतकहॳ अतगयतआताहहॴ 10-Jan-2017

Options

1) Arthropoda

F A C E B O O K

P A G E h t t p w w w f a c e b o o k c o m s s c m e n t o r s o f f i c i a l P a g e | 9

FOR MORE UPDATES AND MORE MATERIAL DO LIKE OUR FACEBOOK PAGE httpwwwfacebookcomsscmentorsofficial

अरोपह८िा 2) Cnidaria

नीिहॳररया 3) Echinodermata

इकाईनह८िमटा 4) Chordata

कह८िटा Correct Answer Arthropoda

Q33 Pulses are a rich source of which of

the following

दािहॳनननननलिखितमसहॳककसकीपरचरसह८तरहहॴ

11-Jan-2017

Options

1) Carbohydrates

काबोहाइडराईट

2) Proteins

परह८टीनस

3) Minerals

िननज

4) Vitamin A

पवटालमन A

Correct Answer Proteins

Q34 Plant cell wall is made up of

वनसपनतकह८लिकालभनततइससहॳबनीहह८तीहहॴ

11-Jan-2017

Options

1) Cellulose

सहॳमयिह८ज

2) Glucose

गिकह८ज

3) Sucrose

सिह८ज

4) Fructose

फरकटह८ज

Correct Answer Cellulose

Plant cell wall the major carbohydrates

are cellulose hemicellulose and pectin

The cellulose microfibrils are linked via

hemicellulosic tethers to form the

cellulose-hemicellulose network which

is embedded in the pectin matrix

Q35 The study of Fungi is also known

as कवकह८कहॳ अधययनकह८कहाजाताहहॴ

11-Jan-2017

Options

1) Cytology

सायटह८िह८जी 2) Myology

मायह८िह८जी 3) Mycology

मायकह८िह८जी 4) Neurology

नयरह८िह८जी Correct Answer Mycology

Cytology - structure and function of

plant and animal cells

Myology is the study of the muscular

system

Neurology is the branch of medicine

concerned with the study and treatment

of disorders of the nervous system

Q36 The outermost layer of skin is

तवचाकीसबसहॳबाहरीपरतकयाहह८तीहहॴ 11-Jan-

2017

Options

1) Epidermis

इपपिलमयस

2) Dermis

िलमयस

3) Tissues

ऊतक

4) Hypodermis

हायपह८िलमयस

Correct Answer Epidermis

Q37 Which of the following plants have

root nodules

F A C E B O O K

P A G E h t t p w w w f a c e b o o k c o m s s c m e n t o r s o f f i c i a l P a g e | 10

FOR MORE UPDATES AND MORE MATERIAL DO LIKE OUR FACEBOOK PAGE httpwwwfacebookcomsscmentorsofficial

नननननलिखितपह९धह८मसहॳककसकीजड़ह८मगाठहह८तीहहॴ

11-Jan-2017

Options

1) Leguminous plants

िहॳगयलमनसपह९धहॳ 2) Parasitic plants

परजीवीपह९धहॳ 3) Epiphytic Plants

एपीफाइटटकपह९धहॳ 4) Aquatic Plants

जिीयपह९धहॳ Correct Answer Leguminous plants

Q38 Earth-worms belongs to the

phylum

कहॳ चएmdashmdash- परजानतकहॳ अतगयतआतहॳहहॴ 11-Jan-2017

Options

1) Protozoa

परह८टह८जआ

2) Cnidaria

नीिहॳररया 3) Annelida

एनीलििा 4) Mollusca

मह८िसका Correct Answer Annelida

Q39 Ringworm is a disease caused by

ररगवमयनामकबीमारी mdashmdash- कहॳ कारणहह८तीहहॴ 11-Jan-2017

Options

1) Fungi

कवक

2) Bacteria

बहॴकटीररया 3) Virus

वायरस

4) Flies

मनकियाा Correct Answer Fungi

Q40 Mangifera indica is the scientific

name of

मननगफहॳ राइडिकाककसकावहॴजञाननकनामहहॴ 11-

Jan-2017

Options

1) Guava

अम द

2) Mango

आम

3) Amla

आविा 4) Jack fruit

कटहि

Correct Answer Mango

Q41 Crabs belongs to the phylum

कहॳ कड़हॳmdashmdash- परजानतकहॳ अतगयतआतहॳहहॴ 11-Jan-2017

Options

1) Mollusca

मह८िसका 2) Cnidaria

नीिहॳररया 3) Arthropoda

अरोपह८ड़ा 4) Platyhelminthes

पिहॳटटहहॳनममननिस

Correct Answer Arthropoda

Q42 Myopia is a defect of eyes which is

also known as

मायह८पपयाआिोकादह८षहहॴ नजसहॳ mdashmdashndash

भीकहाजाताहहॴ

12-Jan-2017

Options

1) Far Sightedness

F A C E B O O K

P A G E h t t p w w w f a c e b o o k c o m s s c m e n t o r s o f f i c i a l P a g e | 11

FOR MORE UPDATES AND MORE MATERIAL DO LIKE OUR FACEBOOK PAGE httpwwwfacebookcomsscmentorsofficial

दरदनषटदह८ष

2) Near Sightedness

ननकटदनषटदह८ष

3) Astigmatism

एसटीगमहॳटटजम

4) Night Blindness

रतोधी Correct Answer Near Sightedness

Myopia occurs when the eyeball is too

long relative to the focusing power of

the cornea and lens of the eye This

causes light rays to focus at a point in

front of the retina rather than directly

on its surface

Hyperopia Hypermetropia (

Farsightedness )- when light rays

entering the eye focus behind the retina

rather than directly on it The eyeball of

a farsighted person is shorter than

normal

Astigmatism usually is caused by an

irregularly shaped cornea Instead of

the cornea having a symmetrically

round shape (like a baseball) it is

shaped more like an American football

Nyctalopia also called night ndash blindness

is a condition making it difficult or

impossible to see in relatively low light

Q43 Who is known as the father of

Green Revolution

हररतिानतकहॳ जनककहॳ पमककसहॳजानाजाताहहॴ

12-Jan-2017

1) Dr Robert Nucleus

िॉ रॉबटयनयनकियस

2) Dr Ian Wilmut

िॉ इयानपविमट

3) Dr NE Borlaug

िॉ एनईबह८रिॉग

4) Dr JC Bose

िॉ जहॳसीबह८स

Correct Answer Dr NE Borlaug

Q44 Panthera Tigris is the scientific

name of

पिहॳराटटगरीस mdashmdashmdash कावहॴजञाननकनामहहॴ 12-Jan-2017

Options

1) Panther

तदआ

2) Tiger

बाघ

3) Whale

हहॳि

4)Goat

बकरी Correct Answer Tiger

Q45 How many facial bones are there

हमारहॳचहॳहरहॳमककतनीहडडियााहह८तीहहॴ 13-Jan-2017

Options

1)34

2)24

3)14

4)4

Correct Answer 14

Q46 ndash Halophytes are plants that grow

in

हहॴिह८फाईटसवहॳपह९धहॳहह८तीहहॴजह८ mdash- मउगतहॳहहॴ SSC CHSL Science (biology) 2016

Question Paper

13-Jan-2017

Options

1) Fresh Water

ताजापानी 2) Cold Water

ठिापानी 3) Ponds

तािाब

4) Salt Water

िारापानी Correct Answer Salt Water

F A C E B O O K

P A G E h t t p w w w f a c e b o o k c o m s s c m e n t o r s o f f i c i a l P a g e | 12

FOR MORE UPDATES AND MORE MATERIAL DO LIKE OUR FACEBOOK PAGE httpwwwfacebookcomsscmentorsofficial

Q47 Felis Catus is the scientific name of

फहॳ लिसकहॴ टस mdashndash कावहॴजञाननकनामहहॴ 13-Jan-2017

Options

1) Cat

बबमिी 2) Dog

कतता 3) Mouse

चहा 4) Porcupine

साही Correct Answer Cat

Q48 Which of the following induces

nitrogen fixation in soil

नननननलिखितमसहॳकह९नलम ीमनाइटरह८जनननयतनकह८परहॳररतकरताहहॴ

15-Jan-2017

Options

1) Protozoa

परह८टह८जआ

2) Bacteria

बहॴकटीररया 3) Fungi

कवक

4)Algae

िहॴवाि

Correct Answer Bacteria

Bacteria that change nitrogen gas from

the atmosphere into solid nitrogen

usable by plants are called nitrogen-

fixing bacteria These bacteria are

found both in the soil and in symbiotic

relationships with plants

They contain symbiotic bacteria called

rhizobia within nodules in their root

systems producing nitrogen compounds

that help the plant to grow and compete

with other plants When the plant dies

the fixed nitrogen is released making it

available to other plant

Q49 Which of the following is the

largest known cell

नननननलिखितमसहॳकह९नसीसबसहॳबड़ीजञातकह८लिकाहहॴ

SSC CHSL Science (biology) 2016

Question Paper

15-Jan-2017

1) Eukaryotic Cell

यकहॳ ररयह८टटककह८लिका 2) Prokaryotic Cell

परह८कहॳ ररयह८टटककह८लिका 3) Mycoplasma

मायकह८पिासम

4) Ostrich Eggs

ितरमगयकाअिा Correct Answer Ostrich Eggs

Q50 The association of animals in

which both the partners are benefitted

is known as

जानवरोकावहसहयह८गनजसमहॳदह८नोभागीदारिाभापवनतहह८तहॳहहॴ उसहॳ mdashmdashndash कहॳ पमजानाजाताहहॴ SSC CHSL Science (biology) 2016

Question Paper

15-Jan-2017

Options

1) Amensalism

सहजीपवत

2) Commensalism

परजीपवत

3) Colony

कॉिनी 4) Mutualism

अनयह८नयाशरयवाद

Correct Answer Mutualism

Amensalism is any relationship between

organisms of different species in which

F A C E B O O K

P A G E h t t p w w w f a c e b o o k c o m s s c m e n t o r s o f f i c i a l P a g e | 13

FOR MORE UPDATES AND MORE MATERIAL DO LIKE OUR FACEBOOK PAGE httpwwwfacebookcomsscmentorsofficial

one organism is inhibited or destroyed

while the other organism remains

unaffected

Commensalism an association between

two organisms in which one benefits and

the other derives neither benefit nor

harm

Q51 Pneumonia affects which of the

following organs of human body

ननमह८ननयामानविरीरकहॳ नननननलिखितमसहॳककसअगकह८परभापवतकरताहहॴ

15-Jan-2017

Options

1)Kidneys

गद

2)Lungs

फहॳ फड़हॳ 3) Throat

गिहॳ 4) Liver

यकत

Correct Answer Lungs

When the germs that cause pneumonia

reach your lungs the lungsrsquo air sacs

(alveoli) become inflamed and fill up

with fluid This causes the symptoms of

pneumonia such as a cough fever

chills and trouble breathing When you

have pneumonia oxygen may have

trouble reaching your blood

Q52 Mendel is known as

मििकह८ mdashmdash- कहॳ पमजानाजाताहहॴ 15-Jan-2017

Options

1) Father of Physiology

िरीरकियािासतरकहॳ जनक

2) Father of Geology

भगभयिासतरकहॳ जनक

3) Father of Genetics

जहॳनहॳटटकसकहॳ जनक

4) Father of Biology

जीविासतरकहॳ जनक

Correct Answer Father of Genetics

Q53 Which of the following are also

known as Suicidal bag of Cells

ननननलिखितमसहॳककसहॳआतमहतयाकरनहॳवािीकह८लिकाओकाबहॴगकहाजाताहहॴ

15-Jan-2017

Options

1) Lysosomes

िायसोसह८म

2) Lycosome

िायकह८सह८म

3) Nucleus

नालभक

4) Chromosome

िह८मह८सह८म

Correct Answer Lysosomes

Q54 Mesothelioma is a type of cancer

The most common area affected in it is

the lining of the ________

लमज़ह८िहॳिहॳलमयाक सरकाएकपरकारहहॴ इससहॳपरभापवतहह८नहॳवािासबसहॳसामानयकषहॳतर mdash

mdashmdash काअसतरहहॴ 15-Jan-2017

Options

1)Heart

हदय

2)Brain

मनसतषक

3)Stomach

आमािय

4)Lungs

फहॳ फड़हॳ Correct Answer lungs

Asbestos exposure is the main cause of

pleural mesothelioma When asbestos

fibers are breathed in they travel to the

F A C E B O O K

P A G E h t t p w w w f a c e b o o k c o m s s c m e n t o r s o f f i c i a l P a g e | 14

FOR MORE UPDATES AND MORE MATERIAL DO LIKE OUR FACEBOOK PAGE httpwwwfacebookcomsscmentorsofficial

ends of small air passages and reach the

pleura where they can cause

inflammation and

scarring

Q55 Which one of the following is an

insectivorous plant

नननननलिखितमसहॳकह९नसाएकककटाहरीवनसपनतहहॴ

15-Jan-2017

Options

1) Utricularia

यटरीकिहॳररया 2) Sequoia

सहॳकयओइया 3) Nostoc

नॉसटह८क

4) Bryophyta

िायह८फाईटा Correct Answer Utricularia

Q56 ______________ is a

multibranched polysaccharide of

glucose that serves as a form of energy

storage in animals and fungi

mdashmdashगिकह८जकाएकबहिािायकतपह८िीसहॳकहॳ राइिहहॴ जह८जानवरोऔरकवकमउजायभणिारणकहॳ एक पमकाययकरताहहॴ 15-Jan-2017

Options

1) Cellulose

सहॳमयिह८ज

2) Glycogen

गिायकह८जन

3) Pectin

पहॳनकटन

4) Chitin

चीटटन

Correct Answer Glycogen

Q57 The largest gland of the human

body is

mdashmdashmdashमानविरीरकीसबसहॳबड़ीगरिीहहॴ 16-Jan-2017

Options

1) Pancreas

अगयािय

2) Thyroid

िायरॉइि

3) Large Intestine

बड़ीआत

4) Liver

यकत

Correct Answer Liver

Q58 Photosynthesis in plants takes

place in

वनसपनतयोमपरकािसशिहॳषणकीकियाहह८तीहहॴ

16-Jan-2017

Options

1) Stem

तना 2) Leaves

पनततयाा 3) Roots

जड़हॳ 4) Flower

फि

Correct Answer Leaves

During this reaction carbon dioxide

and water are converted into glucose

and oxygen The reaction requires light

energy which is absorbed by a green

substance called

chlorophyll Photosynthesis takes place

in leaf

cells These contain chloroplasts which

are tiny objects containing chlorophyll

F A C E B O O K

P A G E h t t p w w w f a c e b o o k c o m s s c m e n t o r s o f f i c i a l P a g e | 15

FOR MORE UPDATES AND MORE MATERIAL DO LIKE OUR FACEBOOK PAGE httpwwwfacebookcomsscmentorsofficial

Q59 Insects that transmit diseases are

known as

जह८कीड़हॳरह८गसचाररतकरतहॳहहॴ उनह mdashmdash-

कहॳ नामसहॳजानाजाताहहॴ 16-Jan-2017

1)Pathogens

रह८गज़नक

2) Vectors

वहॳकटर

3) Drones

परजीवी 4)Scalars

अटदषट

Correct Answer Vectors

A vector is an organism that does not

cause disease itself but which spreads

infection by conveying pathogens from

one host to another Species of mosquito

for example serve as vectors for the

deadly disease Malaria

Q60 Which is the second largest gland

of Human body

मानविरीरकीदसरीसबसहॳबड़ीगरिीकह९नसीहहॴ

SSC CHSL Science (biology)

2016 Question Paper

16-Jan-2017

Options

1) Liver

यकत

2) Large Intestine

बड़ीआत

3) Thorax

छाती 4) Pancreas

अगनयािय

Correct Answer Pancreas

Q61 Annona squamosa is the scientific

name of

एनह८नासकवामह८सा (Annona squamosa) mdash

mdashmdash कावहॴजञाननकनामहहॴ 16-Jan-2017

Options

1) Custard Apple

सीताफि

2) Papaya

पपीता 3) Babhul

बबि

4) Drumstick

सहजन

Correct Answer Custard Apple

Q62 The disease Beri Beri is caused due

to the deficiency of which of the

following

बहॳरीबहॳरीरह८गनननननलिखितमसहॳककसकीकमीकहॳकारणहह८ताहहॴ

16-Jan-2017

Options

1) Vitamin B2

पवटालमन B2

2) Vitamin B1

पवटालमन B1

3) Vitamin B12

पवटालमन B12

4) Vitamin E

पवटालमन E

Correct Answer Vitamin B1

Beriberi is a disease caused by a vitamin

B-1 deficiency also known as thiamine

deficiency

Q63 Chlorophyll was first isolated and

named by

किह८रह८कफिकह८ mdash-

दवारापहिहॳपिकऔरनालमतककयागया 16-Jan-2017

F A C E B O O K

P A G E h t t p w w w f a c e b o o k c o m s s c m e n t o r s o f f i c i a l P a g e | 16

FOR MORE UPDATES AND MORE MATERIAL DO LIKE OUR FACEBOOK PAGE httpwwwfacebookcomsscmentorsofficial

Options

1) Caventou

कहॳ वहॳत 2) Pelletier

पहॳिहॳटटयर

3) Chlorophyll

किह८रह८कफि

4) Caventou and Pelletier

कहॳ वहॳतऔरपहॳिहॳटटयर

Correct Answer Caventou and Pelletier

Chlorophyll was first isolated and

named by

Joseph Bienaimeacute Caventou and Pierre

Joseph Pelletier in 1817 The presence of

magnesium in chlorophyll was

discovered in 1906 and was the first

time that magnesium had been detected

in living tissue

Q64 Which of the following organisms

does not fit into the Cell Theory

नननननलिखितमसहॳकह९नसाजीवकह८लिकालसदातअन पनहीहहॴ

16-Jan-2017

Options

1) Bacteria

बहॴकटीररया 2) Virus

वायरस

3) Fungi

कवक

4) Plants

पह९धहॳ Correct Answer Virus

The bottom line is that viruses are not

alive and not related to cells in any way

The cell theory states that all living

things are made of cells cells are the

basic units of structure and function of

living things and that all cells come

from other cells Since viruses are not

made of cells and do not use cells in any

of their processes they are not related to

the cell theory

Q65 Which of these is not a

macronutrient for Plants

नननननलिखितमसहॳकह९नसापह९धह८कहॳ लिएमिह८नयटरीएटनहीहहॴ

SSC CHSL Science (biology) 2016

Question Paper

17-Jan-2017

Options

1) Nitrogen

नाइटरह८जन

2) Phosphorus

फासफह८रस

3) Potassium

पह८टालसयम

4) Chlorine

किह८रीन

Correct Answer Chlorine

In relatively large amounts the soil

supplies nitrogen phosphorus

potassium calcium magnesium and

sulfur these are often called the

macronutrients In relatively small

amounts the soil supplies iron

manganese boron molybdenum

copper zinc chlorine and cobalt the

so-called micronutrients

Q66 Name the respiratory organs of

insects

कीटह८मनसतिशरवसनअगनामकानामहहॴ

17-Jan-2017

Options

1) Skin

तवचा 2) Body Surface

िरीरकीसतह

F A C E B O O K

P A G E h t t p w w w f a c e b o o k c o m s s c m e n t o r s o f f i c i a l P a g e | 17

FOR MORE UPDATES AND MORE MATERIAL DO LIKE OUR FACEBOOK PAGE httpwwwfacebookcomsscmentorsofficial

3) Gills

गिफड़हॳ 4) Tracheae

शरावस- निी Correct Answer Tracheae

Air enters the respiratory systems of

insects through a series of external

openings called

spiracles These external openings

which act as muscular valves in some

insects lead to the internal respiratory

system a densely networked array of

tubes called tracheae

Q67 The poisonous gas accidentally

released in Bhopal Gas Tragedy is

भह८पािगहॴसतरासदीमगितीसहॳमकतहईजहरीिीगहॴसिी

17-Jan-2017

1) Methane

मीिहॳन

2) Nitrous Oxide

नाइटरसऑकसाइि

3) Methyl Isocyanate

महॴचििआयसोसायनहॳट

4) Cyanogen

सायनह८जहॳन

Correct Answer Methyl Isocyanate

Q68 What does Trypsin do

टटरनपसनकयाकरताहहॴ

SSC CHSL Science (biology) 2016

Question Paper

17-Jan-2017

Options

1) Breaks down Carbohydrates

काबोहाइडरहॳटकापवघटनकरताहहॴ 2) Synthesizes proteins

परह८टीनकासििहॳषणकरताहहॴ 3) Breaks down fats

वसाकापवघटनकरताहहॴ 4) Breaks down proteins

परह८टीनकापवघटनकरताहहॴ Correct Answer Breaks down proteins

Trypsin is one of the three principal

digestive

proteinases the other two being pepsin

and

chymotrypsin In the digestive process

trypsin acts with the other proteinases

to break down dietary protein molecules

to their component

peptides and amino acids

A protease is any enzyme that performs

proteolysis protein catabolism by

hydrolysis of peptide bonds

Q69 Name the source from which

Aspirin is produced

उससरह८तकानामबताइए

नजससहॳएनसपररनकाउतपादनककयाजाताहहॴ

17-Jan-2017

Options

1) Willow bark

पविह८कीछाि

2) Oak Tree

ओककावकष

3) Acacia

बबि

4) Eucalyptus

नीिचगरी Correct Answer Willow bark

The compound from which the active

ingredient in aspirin was first derived

salicylic acid was found in the bark of a

willow tree in 1763 by Reverend

Edmund Stone of Chipping-Norton

Q70 Cannis Familiaris is the scientific

name of

कहॴ ननसफहॳ लमलियहॳररस mdash- कावहॴजञाननकनामहहॴ

17-Jan-2017

F A C E B O O K

P A G E h t t p w w w f a c e b o o k c o m s s c m e n t o r s o f f i c i a l P a g e | 18

FOR MORE UPDATES AND MORE MATERIAL DO LIKE OUR FACEBOOK PAGE httpwwwfacebookcomsscmentorsofficial

Options

1) Cat

बबमिी 2)Dog

कतता 3) Fox

िह८मड़ी 4) Wolf

भहॳडड़या Correct Answer Dog

Q71 Harmful bacteria in potable water

make the water

पीनहॳकहॳ पानीमनसतिघातकबहॴकटीररयाउसपानीकह८बनातहॳहहॴ 17-Jan-2017

Options

1) unfit to drink

पीनहॳकहॳ लिएअयह८गय

2) smelly

दगयनधयकत

3) Colored

रगीन

4) Turbid

मटमहॴिा Correct Answer unfit to drink

Q72 Musa paradisiaca is the scientific

name of which plant

मसापहॴराडिलसयाकाककसपह९धहॳकावहॴजञाननकनामहहॴ

17-Jan-2017

Options

1) Mango

आम

2) Wheat

गहॳह

3) Corn

भ ा 4) banana

कहॳ िा Correct Answer banana

Q73 Prawns belong to which family

झीगहॳककसपररवारकहॳ हह८तहॳहहॴ 17-Jan-2017

Options

1) Crustaceans

िसटहॳलियन

2)Fish

मछिी 3) Amphibians

अननफबबयस

4) Reptiles

रहॳपटाइमस

Correct Answer Crustaceans

Q74 Name the drug that is yielded from

Cinchona tree and is used to cure

malaria

उसऔषचधकानामबताइएनजसहॳलसगकह८नापहॳड़सहॳपरापतककयाजाताहहॴऔरनजसकाउपयह८गमिहॳररयाकहॳ उपचारमककयाजाताहहॴ 17-Jan-2017

Options

1) Camptothea

कहॴ नटह८चिया 2) Acuminata

एकयलमनहॳटा 3) Quinine

कनहॴन

4) Cinchonia

लसकह८ननया Correct Answer Quinine

Q75 Blood Circulation was discovered

by

रकतपररसचरणकी mdashmdashndash दवारािह८जकीिी 17-Jan-2017

Options

1) Mary Anderson

F A C E B O O K

P A G E h t t p w w w f a c e b o o k c o m s s c m e n t o r s o f f i c i a l P a g e | 19

FOR MORE UPDATES AND MORE MATERIAL DO LIKE OUR FACEBOOK PAGE httpwwwfacebookcomsscmentorsofficial

महॴरीएिरसन

2) Virginia Apgar

वनजयननयाएपगार

3) William Harvey

पवलियमहाव

4) Robert Feulgen

रॉबटयफ़यिजहॳन Correct Answer William Harvey

Q76 Vitamin A is also known as

पवटालमन A कह८ mdashmdash- कहॳ नामसहॳभीजानाजाताहहॴ SSC CHSL Science (biology) 2016

Question Paper

18Jan2017

Options

1) Thiamine

िायलमन

2) Riboflavin

ररबह८फिहॳपवन

3) Retinol

रहॳटटनॉि

4) Calciferol

कहॴ नमसफहॳ रह८ि

Correct Answer Retinol

Q77 Some roots called arise from an

organ other than the radicle

कछजड़हॳनजनह mdashmdashmdash कहाजाताहहॴ वहमिकहॳ अिावाककसीअनयअगसहॳउतपननहह८तीहहॴ 18Jan2017

Options

1) tap roots

मखयजड़

2) stilt roots

ि ाजड़

3) fibrous roots

रहॳिहॳदारजड़

4) adventitious roots

आकनसमकजड़

Correct Answer adventitious roots

Q78 Spiders belong to which class of

animals

मकडड़यापराणीवगीकरणकहॳ ककसवगयमआतीहहॴ 18Jan2017

Options

1) Arachnids

एरहॳकननडस

2) Aves

एपवस

3) Gastropods

गहॴसटरोपह८िस

4) Anthozoa

एिह८जआ

Correct Answer Arachnids

Q79 How many layers does Human

Skin have

मानवतवचामककतनीपरतहॳहह८तीहहॴ

18Jan2017

Options

1) 5

2) 7

3) 11

4) 3

Correct Answer 3

Skin has three layers The epidermis

the outermost layer of skin provides a

waterproof barrier and creates our skin

tone The dermis beneath the

epidermis contains tough connective

tissue hair follicles and sweat glands

The deeper subcutaneous tissue (

hypodermis ) is made of fat and

connective tissue

Q80 Allium Cepa is the scientific name

of

एलियमलसपपा mdashmdashndash कावहॴजञाननकनामहहॴ 18Jan2017

F A C E B O O K

P A G E h t t p w w w f a c e b o o k c o m s s c m e n t o r s o f f i c i a l P a g e | 20

FOR MORE UPDATES AND MORE MATERIAL DO LIKE OUR FACEBOOK PAGE httpwwwfacebookcomsscmentorsofficial

Options

1) Carrot

गाजर

2) Tomato

टमाटर

3) Potato

आि 4) Onion

पयाज़

Correct Answer Onion

Q81 DNA stands for

िीएनएकापणय प mdashmdash- हहॴ 18Jan2017

Options

1) Di Nucleic Acid

िाईनयनकिकएलसि

2) Deoxy Nucleic Acid

िीओकसीनयनकिकएलसि

3) Diribonucleic Acid

िाईराइबह८नयनकिकएलसि

4) Deoxyribonucleic Acid

िीऑकसीराइबह८नयनकिकएलसि

Correct Answer Deoxyribonucleic Acid

Q82 Organisms that generate energy

using light are known as

जह८जीवाणपरकािकाउपयह८गकरउजायउतपननकरतीहहॴ उनह mdashmdash कहॳ पमजानाजाताहहॴ

18Jan2017

Options

1) Chaemolithotrophs

ककमह८लििह८टरह८पस

2) Oligotrophs

ओलिगह८टरह८पस

3) Bacteria

बहॴकटीररया 4)Photoautotrophs

फह८टह८ओटह८टरह८पस

Correct Answer Photoautotrophs

An oligotroph is an organism that can

live in an environment that offers very

low levels of nutrients

Q83 Which drug is used as an

Antidepressant

ककसदवाएकहतािारह८धीकहॳ पमपयोगककयाजाताहहॴ Options

1) Oxybutynin

ओकसीलयटीनन

2)Tramadol

टरहॳमहॳिह८ि

3 ) Sumatriptan

समहॳटरीपटहॳन

4) Bupropion

लयपरह८पपयह८न

Correct Answer Bupropion

लयपरह८पपयह८न

Q84 The orange colour of carrot is

because of

गाजरकानारगीरगनननननलिखितमसहॳककसीएककीवजहसहॳहह८ताहहॴ 18Jan2017

Options

1) it grows in the soil

यहलम ीमउगतीहहॴ 2) Carotene

कहॴ रह८टीन

3) it is not exposed to sunlight

यहसययपरकािकहॳ सपकय मनहीआती 4) the entire plant is oranqe in colour

सनपणयपह९धानारगीरगकाहह८ताहहॴ Correct Answer Carotene

Q85 Snake venom is highly modified

saliva containing

F A C E B O O K

P A G E h t t p w w w f a c e b o o k c o m s s c m e n t o r s o f f i c i a l P a g e | 21

FOR MORE UPDATES AND MORE MATERIAL DO LIKE OUR FACEBOOK PAGE httpwwwfacebookcomsscmentorsofficial

सापकाजहरअततयाचधकसिह८चधतिारहह८तीहहॴनजसमहॳ mdashmdash- हह८ताहहॴ Options

l)Prototoxins

परह८टह८टॉनकसस

2)Neutrotoxins

नयटरोटॉनकसस

3)Zootoxins

जटॉनकसस

4)Electrotoxins

इिहॳकटरह८टॉनकसस

Correct Answer Zootoxins

जटॉनकसस

Q86 Which type of pathogen causes the

water-borne disease Schistosomiasis

ककसपरकारकारह८गज़नकजिजननतरह८गलससटह८सह८लमलससकाकारणबनताहहॴ

18Jan2017

Option

1) Parasitic

परजीवी 2)Protozoan

परह८टह८जआ

3) Bacterial

बहॴकटीररयि

4) Viral

वायरि

Correct Answer Parasitic

Schistosomiasis also known as snail

fever and bilharzia is a disease caused

by parasitic

flatworms called schistosomes

Q87 Prothrombin responsible for

clotting of blood is released by

परह८िह८ननबन

जह८रकतकािककाजमनहॳकहॳ लिएनजनमहॳदारहहॴ mdashndash

कहॳ दवारासतरापवतककयाजाताहहॴ

19Jan2017

Options

1) Small Intestine

छह८टीआत

2) Blood Platelets

रकतपिहॳटिहॳटस

3) Large Intestine

बड़ीआत

4Heart

हदय

Correct Answer Blood Platelets

Q88 Acacia arabica is the scientific

name of

अकहॳ लियाअरहॳबबका mdashmdashndash कावहॴजञाननकनामहहॴ 19-Jan-2017

Options

1) Neem

नीम

2) Teak

सागह९न

3) Babhul

बबि

4) Pomegranate

अनार

Correct Answer Babhul

Q89 Cannis Vulpes is the scientific

name of

कहॴ ननसवनमपस mdashmdash- कावहॴजञाननकनामहहॴ 19-Jan-2017

Options

1) Dog

कतता 2) Wolf

भहॳडड़या 3) Fox

िह८मड़ी 4) Hyena

िाकिबगघा

F A C E B O O K

P A G E h t t p w w w f a c e b o o k c o m s s c m e n t o r s o f f i c i a l P a g e | 22

FOR MORE UPDATES AND MORE MATERIAL DO LIKE OUR FACEBOOK PAGE httpwwwfacebookcomsscmentorsofficial

Correct Answer Fox

Q90 The beetroot is the portion of the

beet plant

चकदरपह९धहॳका mdashmdashndash भागहहॴ 19-Jan-2017

Options

1) tap root

मखयजड़

2) Adventitious

आकनसमक

3) bulb of the stem

तनहॳकाकद

4) Rhizome

परकद

Correct Answer tap root

Q91 What is the basic unit of heredity

आनवलिकताकीबननयादीइकाईकयाहहॴ 19-Jan-2017

Options

1) DNA

िीएनए

2) RNA

आरएनए

3) Chromosome

िह८मह८सह८म

4) Gene

जीन

Correct Answer gene

Genes are the units of heredity and are

the instructions that make up the bodyrsquos

blueprint They code for the proteins

that determine virtually all of a personrsquos

characteristics Most genes come in

pairs and are made of strands of genetic

material called deoxyribonucleic acid

or DNA

Q92 Lungs are the primary organs of

फहॳ फड़हॳmdashndashकहॳ परािलमकअगहहॴ

19-Jan-2017

Options

1) Digestion

पाचन

2) Constipation

कलज

3) Perspiration

पसीना 4)Respiration

शवसन

Correct Answer Respiration

Q93 Sugarcane is a type of

गननाएकपरकारका mdash- हहॴ 20-Jan-2017

Options

1)creeper

िता 2)tree

पहॳड़

3)shrub

झाड़ी 4)grass

घास

Correct Answer grass

Q94 Who is commonly known as ldquothe

Father of Microbiologyrdquo

सामानयत ldquo सकषमजीवपवजञानकहॳ जनक lsquo

कहॳ नामसहॳककसहॳजानाजातहहॴ 20-Jan-2017

Options

1) Robert Hooke

रॉबटयहक

2) Antonie Philips van Leeuwenhoek

एटह८नीकफलिपवानमयएनहह८क

3) Carl Linnaeus

काियिीनाईयस

4) Charles Darwin

चामसयिापवयन

F A C E B O O K

P A G E h t t p w w w f a c e b o o k c o m s s c m e n t o r s o f f i c i a l P a g e | 23

FOR MORE UPDATES AND MORE MATERIAL DO LIKE OUR FACEBOOK PAGE httpwwwfacebookcomsscmentorsofficial

Correct Answer Antonie Philips van

Leeuwenhoek

Q95 For the aquatic organisms the

source of food is

जिीयजीवाणकािाघसरह८तहहॴ 20-Jan-2017

Options

1) Phytoplankton

फायटह८पिहॳकटन

2) Sea Weed

समदरीिहॴवाि

3)Aqua plankton

एकवापिहॳकटन

4) Zooplankton

जपिहॳकटन

Correct Answer Phytoplankton

Q96 Haemoglobin has the highest

affinity with which of the following

हीमह८गिह८बबनकीननननमसहॳककसकहॳ सािउततमसमानताहहॴ

20-Jan-2017

Options

1)SO2

2)CO2

3)CO

4)NO2

Correct Answer CO

It has a greater affinity for hemoglobin

than oxygen does It displaces oxygen

and quickly binds so very little oxygen

is transported through the body cells

Q97 Who developed the theory of

Evolution

उदपवकासकालसदातककसनहॳपवकलसतककया

20-Jan-2017

Options

1) Charles Darwin

चामसयिापवयन

2) Isaac Newton

आयजहॳकनयटन

3) Pranav Mistry

परणवलमसतरी 4) Galileo Galilei

गहॳलिलियह८गहॳिीिी Correct Answer Charles Darwin

Q98 The primary function of RNA is

RNA कापरािलमककाययहह८ताहहॴ 20-Jan-2017

Options

1) Photosynthesis

परकािसशिहॳषण

2) Protein Synthesis

परह८टीनसशिहॳषण

3) Replication

परनतकनतबनाना 4) Translation

अनवादकरना Correct Answer Protein Synthesis

There are two main functions of RNA

It assists DNA by serving as a messenger

to relay the proper genetic information

to countless numbers of ribosomes in

your body The other main function of

RNA is to select the correct amino acid

needed by each ribosome to build new

proteins for your body

Q99 ______is the movement of

molecules across a cell membrane from

a region of their lower concentration to

a region of their higher concertration

उचचसादरताकहॳ कषहॳतरसहॳउसकीकमसादरतावािहॳकषहॳतरकीतरफएककह८लिकाखझमिीकहॳ माधयमसहॳहह८नहॳवािाअणओकहॳ सचिनकह८ mdash- कहतहॳहहॴ Options

1) Diffusion

पवसरण

2) Osmosis

ऑसमह८लसस

F A C E B O O K

P A G E h t t p w w w f a c e b o o k c o m s s c m e n t o r s o f f i c i a l P a g e | 24

FOR MORE UPDATES AND MORE MATERIAL DO LIKE OUR FACEBOOK PAGE httpwwwfacebookcomsscmentorsofficial

3) Active Transport

सकियआवागमन

4) Passive Transport

नननषियआवागमन

Correct Answer Active Transport

Q100 Study of classification of

organisms is known as 20-Jan-2017

जीवाणओकहॳ वगीकरणकहॳ अधययनकह८ mdash-

कहाजाताहहॴ Options

1) Serpentology

सपरहॳटह८िह८जी 2) Virology

वायरह८िह८जी 3) Taxonomy

टहॴकसोनह८मी 4) Physiology

कफनज़यह८िह८जी Correct Answer Taxonomy

Q101 Photosynthesis takes place inside

plant cells in

परकािसशिहॳषणवनसपनतकह८लिकामनसति mdash

mdashmdash महह८ताहहॴ 20-Jan-2017

Options

1) Ribosomes

राइबह८सह८नस

2) Chloroplasts

किह८रह८पिासट

3) Nucleus

नयकलियम

4) Mitochondria

माईटह८कोडडरया Correct Answer Chloroplasts

Q102 ______ is the cell organelle in

which the biochemical processes of

respiration and energy production

occur

mdashmdash- वहकह८लिकाअगहहॴ नजसमहॳशवसनऔरउजायउतपादनकहॳ जहॴसीजहॴवरासायननकपरकियायहह८तीहहॴ 20-Jan-2017

Options

1) Mitochondria

माइटह८कोडडरया 2) Chloroplast

किह८रह८पिासट

3) Ribosomes

राइबह८सह८नस

4) Nucleus

नयकिीयस

Correct Answer Mitochondria

Q103 Which non-flowering spore

bearing plants have roots

ककसफिनिगनहॳवािहॳऔरबीजाणधारकपह९धह८कीजड़हॳहह८तीहहॴ 21-Jan-2017

Options

1) Mosses

मह८सहॳस

2) Angiosperms

एननजयह८सपनसय 3) Ferns

फनसय 4) Gymnosperms

नजननह८सपनसय Correct Answer ferns

Q104 Which of the following is an

excretory organ of cockroach

नननननलिखितमसहॳकह९नसानतिच हॳकाउतसजयनअगहहॴ

21-Jan-2017

Options

F A C E B O O K

P A G E h t t p w w w f a c e b o o k c o m s s c m e n t o r s o f f i c i a l P a g e | 25

FOR MORE UPDATES AND MORE MATERIAL DO LIKE OUR FACEBOOK PAGE httpwwwfacebookcomsscmentorsofficial

1) Malphigian Tubules

मनमफनजयनटयबमस

2) Nephridia

नहॳकफरडिया 3) Coxal Gland

कह८कसिगरचिया 4) Green Gland

गरीनगरचिया Correct Answer Malphigian Tubules

Q105 Evaporation of water takes place

in which part of plants

पानीकहॳ वाषपीकरणकीकियापह९धोकहॳ ककसभागसहॳहह८तीहहॴ 21-Jan-2017

Options

1) Stem

तना 2) Stomata

सटह८मटा 3) Branch

िािाए

4) Fruit

फि

Correct Answer Stomata

Evaporation accounts for the movement

of water to the air from sources such as

the soil canopy interception and

waterbodies Transpiration accounts for

the movement of water within a plant

and the subsequent loss of water as

vapour through stomata in its leaves

Q106 A is the fleshy spore-bearing

fruiting body of a fungus

mdashmdashndashकवककामासि

बीजाणधारणकरनहॳवािाफिनहॳवािाअगहहॴ 21-

Jan-2017

Options

1) aloe vera

एिह८वहॳरा 2) Coral

मगा 3) Cactus

कहॴ कटस

4) Mushroom

ककरमतता Correct Answer mushroom

Q107 Which of the following is a fungal

disease

नननननलिखितमसहॳकह९नसाफफदसहॳहह८नहॳवािाएकरह८ग हहॴ

21-Jan-2017

Options

1) Dermatitis

तवचािह८ध

2) Cholera

हहॴजा 3) Jaundice

पीलिया 4) Indigofera

इननिगह८फहॳ रा Correct Answer Dermatitis

Dermatitis also known as eczema is a

group of diseases that results in

inflammation of the skin These diseases

are characterized by itchiness red skin

and a rash In cases of short duration

there may be small blisters while in

long-term cases the skin may become

thickened

Q108 In which form is glucose stored in

our body

हमारहॳिरीरमगिकह८जकासचयककस पमककयाजाताहहॴ

21-Jan-2017

Options

1) Insulin

F A C E B O O K

P A G E h t t p w w w f a c e b o o k c o m s s c m e n t o r s o f f i c i a l P a g e | 26

FOR MORE UPDATES AND MORE MATERIAL DO LIKE OUR FACEBOOK PAGE httpwwwfacebookcomsscmentorsofficial

इसलिन

2) Glucose

गिकह८ज

3) Glycogen

गिायकह८जहॳन

4) Fat

वसा Correct Answer Glycogen

Excess glucose is stored in the liver as

the large compound called glycogen

Glycogen is a polysaccharide of glucose

but its structure allows it to pack

compactly so more of it can be stored in

cells for later use

Q109 Where do plants synthesize

protein from

पह९धहॳपरह८टीनसशिहॳषणकहासहॳकरतहॳहहॴ

Options

1) Fatty Acids

वसाऐलसि

2) Sugar

िकर

3) Amino Acids

एलमनह८ऐलसि

4) Starch

सटाचय Correct Answer Amino Acids

Q110 Which part of the brain is

responsible for triggering actions like

thinking intelligence memory and

ability to learn

मनसतषककाकह९नसाटहससासह८चनहॳ बनधदमानी याददाशतऔरसीिनहॳकीकषमताजहॴसीकियाओकह८परहॳररतकरताहहॴ 21-Jan-2017

Options

1) Diencephalon

िायएनसहॳफहॳ िह८न

2) Hypothalamus

हयपह८िहॳिहॳमस

3) Cerebrum

सहॳरहॳिम

4) Control

कटरह८ि

Correct Answer Cerebrum

Q111 Which of the following is also

known as the Biochemical Laboratory

of the Human Body

नननननलिखितमसहॳककसहॳमानविरीरकीजहॴवरसायनपरयह८गिािाभीकहाजाताहहॴ 21-Jan-2017

Options

1) Small Intestine

छह८टीआत

2)Brain

मनसतषक

3) Pancreas

अगनयािय

4) Liver

नजगर

Correct Answer Liver

The liver makes bile that will help

emulsify and digest the fats we eat

The liver takes toxic substances and

convert them using enzymes the liver

cells makes into a non toxic form so the

body can dispose of them

The liver also converts fats protein and

carbohydrates into glucose which is the

energy source for our cells to use

The liver takes amino acids and makes

proteins by combining them

Q112 The yellow colour of human urine

is due to

मानवमतरकापीिारग mdashndash कीवजहसहॳहह८ताहहॴ 22-

Jan-2017

Options

1) Bile Salts

F A C E B O O K

P A G E h t t p w w w f a c e b o o k c o m s s c m e n t o r s o f f i c i a l P a g e | 27

FOR MORE UPDATES AND MORE MATERIAL DO LIKE OUR FACEBOOK PAGE httpwwwfacebookcomsscmentorsofficial

पपततनमक

2) Cholesterol

कह८िहॳसटरह८ि

3) Lymph

लिनफ

4) Urochrome

यरह८िह८म

Correct Answer Urochrome

Urobilin or urochrome is the chemical

primarily responsible for the yellow

color of urine

Q113 The wilting of plants takes place

due to

पह९धह८कालिचििहह८नाकी mdashmdash- कीवजहसहॳहह८ताहहॴ 22-Jan-2017

Options

1)Photosynthesis

परकािसशिहॳषण

2) Transpiration

वाषपह८तसजयन

3) Absorption

अविह८षण

4) Respiration

शरवसन

Correct Answer Transpiration

Wilting is the loss of rigidity of non-

woody parts of plants This occurs when

the turgor pressure in non-lignified

plant cells falls towards zero as a result

of diminished water in the cells

Q114 Bovidae Ovis is the scientific name of

बह८पविीओपवस mdashndash कावहॴजञाननकनामहहॴ 22-Jan-2017

Options

1) Goat

बकरी 2) Cow

गाय

3) Buffalo

भहॳस

4) Sheep

भहॳड़

Correct Answer Sheep

Q115 Plants get their energy to produce

food from which of the following

पह८धहॳभह८जनकाननमायणकरनहॳकहॳ लिएनननननलिखितमसहॳककससहॳउजायपरापतकरतहॳहहॴ

22-Jan-2017

Options

1) Photosynthesis

परकािसशिहॳषण

2)Bacteria

बहॴकटीररया 3)Fungi

कवक

4)Sun

सयय Correct Answer Sun

Q116 Which of the following is secreted

by the liver

नननननलिखितमसहॳककसकासरावनजगरसहॳहह८ताहहॴ

22-Jan-2017

Options

1) Glucose

गिकह८ज

2) Iodine

आयह८िीन

3) Cortisol

काटटरयसह८ि

4) Bile

पपतत

Correct Answer Bile

The liver makes bile that will help

emulsify and

digest the fats we eat

F A C E B O O K

P A G E h t t p w w w f a c e b o o k c o m s s c m e n t o r s o f f i c i a l P a g e | 28

FOR MORE UPDATES AND MORE MATERIAL DO LIKE OUR FACEBOOK PAGE httpwwwfacebookcomsscmentorsofficial

Q117 Ferns belong to which division of

plants

फनसयपह९धह८कहॳ ककसभागमआतहॳहहॴ

22-Jan-2017

Options

1) Gymnosperms

नजननह८सपनसय 2) Angiosperms

एनजयह८सपनसय 3) Thallophyta

िहॴिह८फाईटा 4)Pteridophyta

टहॳररिह८फाईटा Correct Answer Pteridophyta

Q118 Who invented Antibiotics

एटीबायह८टटककाअपवषकारककसनहॳककयािा

22-Jan-2017

Options

1) Joseph Lister

जह८सहॳफलिसटर

2) William Harvey

पवलियमहाव

3) Robert Knock

रॉबटयनॉक

4)Alexander Fleming

अिहॳकज़िरफिहॳलमग

Correct Answer Alexander Fleming

Q119 Milbecycin is used in the

eradication of

लममबहॳसायलसनका mdashndash

मउनमिनमपरयह८गककयाजाताहहॴ 22-Jan-2017

Options

1) Agricultural Fungus

कपषकवक

2) Agricultural Pests

कपषकीटक

3) Agricultural Herbs

कपषिाक

4)Agricultural Weeds

कपषननराना Correct Answer Agricultural Pests

Milbemycin oxime is a veterinary drug

from the group of milbemycins used as

a broad spectrum antiparasitic It is

active against worms and mites(insects

Q120 Intestinal bacteria synthesizes

which of the following in the human

body

मानविरीरमआतोकहॳ बहॴकटीररयानननननलिखितमसहॳककसकासशिहॳषणकरतहॳहहॴ 22-Jan-2017

Options

1) Vitamin K

पवटालमन K

2) Proteins

परह८टीन

3) Fats

वसा 4) Vitamin D

पवटालमन D

Correct Answer Vitamin K

Q121 is the study of the physical form

and external structure of plants

mdashmdash-

मपह९धह८काभहॴनतक पऔरबाहरीसरचनाकाआदयाककयाजाताहहॴ 22-Jan-2017

Options

1) Physiology

कफनजयह८िह८जी 2) Anatomy

िरीररचनापवजञान

3) Phytomorphology

फाईटह८मह८फह८िह८जी 4)Cytology

कह८लिकापवजञान

Correct Answer Phytomorphology

F A C E B O O K

P A G E h t t p w w w f a c e b o o k c o m s s c m e n t o r s o f f i c i a l P a g e | 29

FOR MORE UPDATES AND MORE MATERIAL DO LIKE OUR FACEBOOK PAGE httpwwwfacebookcomsscmentorsofficial

Q122 Which of the following is a

structural and functional unit of

kidneys

नननननलिखितमसहॳकह९नसीगदोकीसरचनातमकऔरकाययकरीईकाईहहॴ

22-Jan-2017

Options

1) Renette Cells

रहॳनहॳटकह८लिकाए

2) Flame Cells

फिहॳमकह८लिकाए

3) Nephrites

नहॳफ़राइटस

4)Nephrons

नहॳफरोस

Correct Answer Nephrons

Nephron functional unit of the kidney

the structure that actually produces

urine in the process of removing waste

and excess substances from the blood

There are about 1000000 nephrons in

each human kidney

Q123 Which of the following is the

largest part of the human brain

नननननलिखितमसहॳकह९नसामानवमनसतषककासबसहॳबड़ाटहससाहहॴ

23-Jan-2017

Options

1) Ribs

पसलियाा 2) Cerebrum

सहॳरहॳिम

3) Pons

पोस

4)Thalamus

िहॴिहॳमस

Correct Answer Cerebrum

The cerebrum is the largest part of the

human brain making up about two-

thirds of the brainrsquos mass It has two

hemispheres each of which has four

lobes frontal parietal temporal and

occipital

Q124 The auxiliary buds

सहायककालियाmdashndash 23-Jan-2017

Options

1) grow endogenously from the pericycle

पहॳरीसाईककिसहॳअनतजातयपवकलसतहह८ताहहॴ 2) arise endogenously from the main

growing point

मिवपदसहॳअनतजातयउठताहहॴ 3) is an embryonic shoot located in the

axil of a leaf

एकभरणिटहहॴजह८एकपततीकहॳ अकषपरनसतिहह८ताहहॴ 4)arise exogenously from the epidermis

एपपिलमयससहॳबटहजातयतरीकहॳ सहॳउठताहहॴ Correct Answer is an embryonic shoot

located in the axil of a leaf

Q125 Which of the following is a viral

disease

इनमहॳसहॳकह९सीएकवायरिबीमारीहहॴ

23-Jan-2017

Options

1) Polio

पह८लियह८ 2) Tetanus

धनसतनभ

3) Leprosy

कषठरह८ग

4) Plague

पिहॳग

Correct Answer Polio

A viral disease (or viral infection)

occurs when an organismrsquos body is

invaded by pathogenic viruses and

infectious virus particles (virions) attach

to and enter susceptible cells

F A C E B O O K

P A G E h t t p w w w f a c e b o o k c o m s s c m e n t o r s o f f i c i a l P a g e | 30

FOR MORE UPDATES AND MORE MATERIAL DO LIKE OUR FACEBOOK PAGE httpwwwfacebookcomsscmentorsofficial

Poliomyelitis often called polio or

infantile paralysis is an infectious

disease caused by the poliovirus

Tetanusmdash A serious bacterial infection

that causes painful muscle spasms and

can lead to death

Leprosy also known as Hansenrsquos

disease (HD) is a long-term infection by

the bacterium Mycobacterium leprae or

Mycobacterium lepromatosis

Plague is an infectious disease caused by

the bacterium Yersinia pestis

Symptoms include fever weakness and

headache

Q126 Which organisms can help to

carry out Vermicomposting

कह९नसाजीववमीकनपह८नसटगममददकरताहहॴ

23-Jan-2017

Options

1) Nitrifying Bacteria

नाईटरीफाईगबहॴकटीररया 2) Earthworms

कहॴ चऐ

3) Algae

िहॴवि

4) Fungus

कवक

Correct Answer Earthworms

Q127 Contraction of heart is also

known as

हदयकहॳ सकचनकह८ mdash- भीकहाजाताहहॴ 23-Jan-

2017

Options

1) Systole

लससटह८ि

2) Aristotle

अरसत

3) Diastole

िायसटह८ि

4) Lub

मयब

Correct Answer Systole

Diastole is the part of the cardiac cycle

when the heart refills with blood

following systole (contraction)

Ventricular diastole is the period during

which the ventricles are filling and

relaxing while atrial diastole is the

period during which the atria are

relaxing

Q128 Azadirachta indica is the

botanical name of which of the

following

अजाटदराचताइडिकानननननलिखितमसहॳककसकावानसपनतनामहहॴ

23-Jan-2017

Options

1) Rose plant

गिाबकापह९धा 2) Apple tree

सहॳबकापहॳड़

3) Neem

नीम

4)Mango

आम

Correct Answer Neem

Q129 Which of the following is the

main end product of carbohydrate

digestion

नननननलिखितमसहॳकह९नसाकाबोहाइडरहॳटकहॳ पाचनकापरमिअतउतपादकहह८ताहहॴ 23-Jan-2017

Options

1) Fats

वसा 2) Lipids

लिपपडस

3) Glucose

गिकह८ज

4) Cellulose

F A C E B O O K

P A G E h t t p w w w f a c e b o o k c o m s s c m e n t o r s o f f i c i a l P a g e | 31

FOR MORE UPDATES AND MORE MATERIAL DO LIKE OUR FACEBOOK PAGE httpwwwfacebookcomsscmentorsofficial

सहॳमयिह८ज

Correct Answer Glucose

Intestinal absorption of end products

from digestion of carbohydrates and

proteins in the pig hellip During absorption some sugars (fructose or

galactose) released from the

corresponding sucrose and lactose

respectively during digestion were

partly metabolized into glucose by the

enterocyte

Q130 Which of the following glands is a

source of the enzyme Ptyalin

नननननलिखितगरचियोमसहॳएजाइमटयालिनकासरह८तहहॴ 23-Jan-2017

Options

1) Pancreas

अगरािय

2) Thyroid Gland

िाइराइिगरिी 3) Pituitary Gland

पीयषगरिी 4) Salivary Glands

िारगरचियाा Correct Answer Salivary Glands

Q131 Which of the following is not true

about Pteridophyta

ननननमसहॳकह९नसीबातटहॳररिह८फाईटकहॳ बारहॳमसचनहीहहॴ 23-Jan-2017

Options

1) Dominant phase is saprophytes

परमिचरणसहॳपरह८फाईइटसहह८ताहहॴ 2) Main plant body is diploid

पह९दह८कामखयिरीरदपवगखणतहह८ताहहॴ 3) Seeds are present

बीजमह९जदहह८तहॳहहॴ 4)Flowers are absent

फिअनपनसतिहह८तहॳहहॴ

Correct Answer Seeds are present

Q132 The largest dolphin species is the

orca also called as

िॉिकफनकीसबसहॳबड़ीपरजानतकाकानामआकायहहॴनजसहॳ mdash- भीकहतहॳहहॴ 23-Jan-2017

Options

1) Bottle Nose

बाटिनह८ज

2) Baiji

बहॳजी 3) Killer whale

ककिरहहॳि

4)Tucuxi

टकवसी Correct Answer Killer whale

Q133 The fat digesting enzyme Lipase

is secreted by which of the following

वसाकापाचनकरनहॳवािाएजाइमिाइपहॳजनननननलिखितमसहॳककसकहॳ दवारासतरापवतहह८ताहहॴ

24-Jan-2017

Options

1) Kidneys

गद

2) Pancreas

अगनयािय

3) Large Intestine

बड़ीआत

4)Liver

नजगर

Correct Answer Pancreas

Lipase is an enzyme that splits fats so

the intestines can absorb them Lipase

hydrolyzes fats like triglycerides into

their component fatty acid and glycerol

molecules It is found in the blood

gastric juices pancreatic secretions

intestinal juices and adipose tissues

F A C E B O O K

P A G E h t t p w w w f a c e b o o k c o m s s c m e n t o r s o f f i c i a l P a g e | 32

FOR MORE UPDATES AND MORE MATERIAL DO LIKE OUR FACEBOOK PAGE httpwwwfacebookcomsscmentorsofficial

Q134 The arrangement of leaves on an

axis or stem is called

एकअकषयातनहॳपरपनततयोकीयवसिाकह८कयाकहाजाताहहॴ SSC CHSL Science (biology) 2016

Question Paper

24-Jan-2017

Options

1) Phyllotaxy

फाइिह८टहॴकसी 2) Vernation

वनिन

3) Venation

वहॳनहॳिन

4)Phytotaxy

फाइटह८टहॴकसी Correct Answer Phyllotaxy

In botany phyllotaxis or phyllotaxy is

the arrangement of leaves on a plant

stem (from Ancient Greek phyacutellon

ldquoleafrdquo and taacutexis ldquoarrangementrdquo)

Phyllotactic spirals form a distinctive

class of patterns in nature

Q135 The study of Cells is also known

as

कह८लिकाओकहॳ अधययनकह८ mdashmdashndash

भीकहाजाताहहॴ 24-Jan-2017

Options

1) Cytology

सायटह८िह८जी 2) Physiology

कफनजयह८िह८जी 3) Nucleology

नयककमयह८िह८जी 4)Cellology

सहॳिह८िह८जी Correct Answer Cytology

Q136 Which of the following scientists

is also known as the Father of Biology

नननननलिखितमसहॳककसवहॴजञाननककह८ ldquoजीवपवजञानकहॳ जनकrdquoकहॳ नामसहॳभीजानाजाताहहॴ 24-Jan-2017

Options

1) Herbert Spencer

हबयटयसपसर

2) Aristotle

अरसत 3) Lamarck

िहॳमाकय 4)Darwin

िापवयन

Correct Answer Aristotle

Q137 Which cells give rise to various

organs of the plant and keep the plant

growing

कह९नसीकह८लिकाएपह९धह८कहॳ लभननअगह८कह८जनमदहॳतीहहॴऔरपह९धह८कह८बढ़नहॳममददकरतीहहॴ

24-Jan-2017

Options

1) Permanent

सिायी 2) Dermal

तवचीय

3) Meristematic

मररसटहॳमटटक

4)Mature

परह८ढ़

Correct Answer Meristematic

A meristem is the tissue in most plants

containing undifferentiated cells

(meristematic cells) found in zones of

the plant where growth can take place

Q138 Rodentia Muridae is the scientific

name of

F A C E B O O K

P A G E h t t p w w w f a c e b o o k c o m s s c m e n t o r s o f f i c i a l P a g e | 33

FOR MORE UPDATES AND MORE MATERIAL DO LIKE OUR FACEBOOK PAGE httpwwwfacebookcomsscmentorsofficial

रह८िहॳलियानयररिी mdashmdash- कावहॴजञाननकनामहहॴ 24-

Jan-2017

Options

1) Mouse

चहा 2) Squirrel

चगिहरी 3) Monkey

बदर

4) Lizard

नछपकिी Correct Answer Mouse

Q139 Name the scientist who proposed

the cell theory

कह८लिकालसदातकापरसतावदहॳनहॳवािहॳवहॴजञाननककानामबताइए 24-Jan-2017

Options

1) Schleiden and Schwann

िीमिनऔरशरववान

2) Lamarck

िहॳमाकय 3) Treviranus

टरहॳवायरहॳनस

4)Whittaker and Stanley

हीटकरऔरसटहॳनिहॳ Correct Answer Schleiden and

Schwann

Q140 The flower with the worldrsquos

largest bloom is

दननयाकासबसहॳबड़ाफिखििनहॳवािा mdashmdashndash हहॴ 24-Jan-2017

Options

1) Pando

पािह८ 2) Posidonia

पह८सीिह८ननया 3) Rafflesia arnoldii

ररफिहॳलियाअनोमिी 4)Helianthus annuus

हहॳलिएनिसएनयअस

Correct Answer Rafflesia arnoldii

Rafflesia arnoldii is a species of

flowering plant in the parasitic genus

Rafflesia It is noted for producing the

largest individual flower on earth It has

a very strong and horrible odour of

decaying flesh earning it the nickname

ldquocorpse flower

Q141 Deficiency of which vitamin

causes night blindness

ककसपवटालमनकीकमीकहॳ कारणरतौधीहह८ताहहॴ 24-Jan-2017

Options

1) Vitamin K

पवटालमन K

2) Vitamin C

पवटालमन C

3) Vitamin B1

पवटालमन B1

4)Vitamin A

पवटालमन A

Correct Answer Vitamin A

Q142 Nongreen plants lack which of the

following

गहॴर-

हररतवनसपनतमनननननलिखितमसहॳककसकीकमीहह८तीहहॴ

24-Jan-2017

Options

1) Chlorophyll

किह८रह८कफि

2) Lycophyll

िायकह८कफि

3) Cyanophyll

F A C E B O O K

P A G E h t t p w w w f a c e b o o k c o m s s c m e n t o r s o f f i c i a l P a g e | 34

FOR MORE UPDATES AND MORE MATERIAL DO LIKE OUR FACEBOOK PAGE httpwwwfacebookcomsscmentorsofficial

सायनह८कफि

4)Phototropism

फह८टह८टरोपपजम

Correct Answer Chlorophyll

Q143 Organisms that use light to

prepare food are known as

जह८जीवपरकािकाउपयह८गकरभह८जनतहॴयारकरतहॳहहॴ उनह mdashmdash- कहॳ पमजानजाताहहॴ 24-Jan-2017

Options

1) Autotrophs

सवपह८षी 2) Heterotrophs

पवषमपह८षज

3) Omnivores

सवायहारी 4)Decomposers

पवघटनकरनहॳवािा Correct Answer Autotrophs

autotrophs often make their own food

by using sunlight carbon dioxide and

water to form sugars which they can use

for energy Some examples of

autotrophs include plants algae and

even some bacteria Autotrophs

(producer) are important because they

are a food source for heterotrophs

(consumers)

A heterotroph is an organism that

ingests or absorbs organic carbon

(rather than fix carbon from inorganic

sources such as carbon dioxide) in order

to be able to produce energy and

synthesize compounds to maintain its

life Ninety-five percent or more of all

types of living organisms are

heterotrophic including all animals and

fungi and some bacteria

Q144 Which of the following is a

primary function of haemoglobin

नननननलिखितमसहॳकह९नसाटहमह८गिह८बबनकाएकपरािलमककाययहहॴ

25-Jan-2017

Options

1) Utilization of energy

उजायकाउपयह८गकरना 2) Prevention of anaemia

रकतामपताहह८नहॳसहॳरह८कना 3) Destruction of bacteria

बहॴकटीररयाकापवनािकरना 4) To transport oxygen

ऑकसीजनकावहनकरना Correct Answer To transport oxygen

Q145 Vascular bundles are absent in

सवहनीबिि mdashmdash- मअनपनसतिरहतहॳहहॴ 25-Jan-2017

Options

1) Bryophyta

िायह८फाइटा 2) Pteridophyta

टहॳररिह८फाईटा 3) Gymnosperms

नजननह८सपमय 4) Angiosperms

एननजयह८सपहॳनसय Correct Answer Bryophyta

Q146 Sauria Lacertidae is the scientific

name of

सहॴररयािहॳसरटाईिी mdashmdashndash कावहॴजञाननकनामहहॴ 25-Jan-2017

Options

1) Crocodile

मगरमचछ

2) Hippopotamus

टहपपह८पह८टहॳमस

3) Lizard

नछपकिी 4) House fly

F A C E B O O K

P A G E h t t p w w w f a c e b o o k c o m s s c m e n t o r s o f f i c i a l P a g e | 35

FOR MORE UPDATES AND MORE MATERIAL DO LIKE OUR FACEBOOK PAGE httpwwwfacebookcomsscmentorsofficial

घरहॳिमकिी Correct Answer Lizard

Q147 Which type of pathogen causes

the water-borne disease SARS (Severe

Acute Respiratory Syndrome)

ककसपरकािकारह८गज़नकजिजननतबीमारीसासयकाकारणबनताहहॴ 25-Jan-2017

Options

1) Viral

वायरि

2) Parasitic

परजीवी 3) Protozoan

परह८टह८जअन

4) Bacterial

बहॴकटीररयि

Correct Answer Viral

Q148 Which of the following organs

produces the enzyme lipase

नननननलिखितमसहॳकह९नसाअगिायपहॳजएजाइमउतपननकरताहहॴ 25-Jan-2017

Options

1) Pancreas

अगनयािय

2) Large Intestine

बड़ीआत

3) Liver

नजगर

4) Small Intestine

छह८टीआत

Correct Answer Pancreas

Q149 A is a long internode forming the

basal part or the whole of a peduncle

एक mdashmdash- एकिबाइटरनह८िहहॴ जह८ननचिाटहससायासनपणयिठिबनताहहॴ 25-

Jan-2017

Options

1) Rhizome

परकद

2) Rachis

महॳ दि

3) floral axis

पषपअकष

4) Scape

भगदड़

Correct Answer scape

Q150 ndash Which of the following

organisms are considered to be both

Living and Non-living

नननननलिखितमसहॳकह९नसहॳजीवाणकह८जीपवतऔरअजीपवतमानाजाताहहॴ

25-Jan-2017

Options

1) Bacteria

बहॴकटीररया 2) Fungi

कवक

3) Algae

िहॴवाि

4)Virus

वायरस

Correct Answer Virus

They are considered to be living as they

possess a protein coat as a protective

covering DNA as the genetic material

etc

They are said to be non-living as they

can be crystallised and they survive for

billions of years They can tolerate high

temperatures freezing cold

temperatures ultra-violet radiations etc

Q151 Deficiency of fluorine causes

which of the following

फिह८ररनकीकमीकहॳ कारणनननननलिखितमसहॳकयाहह८ताहहॴ

F A C E B O O K

P A G E h t t p w w w f a c e b o o k c o m s s c m e n t o r s o f f i c i a l P a g e | 36

FOR MORE UPDATES AND MORE MATERIAL DO LIKE OUR FACEBOOK PAGE httpwwwfacebookcomsscmentorsofficial

27-Jan-2017

Options

1) Dental Caries

िटिकहॴ ररज

2) Scurvy

सकवरी 3) Anaemia

रकतामपता 4) Arthritis

गटठया Correct Answer Dental Caries

Q152 In a Punnett Square with the

cross AaBb x AaBb how many Aabb

genotypes would be created

पनहॳटसककायरमिह८स AaBb x AaBb कहॳ साि

ककतनहॳ Aabb जीनह८टाइपबनगहॳ 27-Jan-2017

Options

1) 1

2) 8

3) 2

4) 3

Correct Answer 2

Q153 Which of the following is the

Controlling Center of the Cell

नननननलिखित म सहॳ कह८लिकाका ननयतरण

क दर कह९न हहॴ

27-Jan-2017

Options

1) Nucleus

क दर

2) Plasma

पिाजमा 3) Lysosome

िायसह८सह८म

4) Chromosome

िह८मह८सह८म

Correct Answer Nucleus

The control centre of the cell is the

nucleus in eukaryotic cells The nucleus

contains genetic material in the form of

DNA

Q154 Myopia affects which of the

following organs

मायह८पपयानननननलिखितअगह८मसहॳककसहॳपरभापवतकरताहहॴ

25-Jan-2017

Options

1) Heart

हदय

2) Skin

तवचा 3) Eyes

आािहॳ 4)Mouth

मह

Correct Answer Eyes

Q155 Which of the following bears

flowers

नननननलिखितमसहॳकह९नफिधारणकरताहहॴ

25-Jan-2017

Options

1) Bryophyta

िायह८फाइटा 2) Pteridophyta

टहॳरीिह८फाईटा 3) Gymnosperms

नजननह८सपमय 4)Angiosperms

एननजयह८सपमय Correct Answer Angiosperms

Q156 Oxygenated blood flows out of the

heart through the

ऑकसीजनयकतरकत mdashmdashmdash

कहॳ माधयमसहॳहदयकहॳ बाहरबहताहहॴ 25-Jan-2017

F A C E B O O K

P A G E h t t p w w w f a c e b o o k c o m s s c m e n t o r s o f f i c i a l P a g e | 37

FOR MORE UPDATES AND MORE MATERIAL DO LIKE OUR FACEBOOK PAGE httpwwwfacebookcomsscmentorsofficial

Options

1) Aorta

महाधमनी 2) pulmonary artery

फहॳ फड़हॳकीधमनी 3) vena cava

वहॳनाकावा 4)Atrium

चह९क

Correct Answer aorta

Q157 Blood leaving the liver and

moving towards the

heart has a higher concentration of

नजगरसहॳननकिकरहदयकीतरफजानहॳवािहॳरकतम mdashmdashmdashmdash कीउचचसादरताहह८तीहहॴ 27-Jan-2017

Options

1) Lipids

लिपपडस

2) Urea

यररया 3) Bile Pigments

पपततकहॳ रगकरण

4) Carbon dioxide

काबयनिायऑकसाइि

Correct Answer Bile Pigments

Urea is nitrogen containing substance

which is produced in the liver in order

to deal with excess amino-acids in the

body As urea is produced it leaves the

liver in the blood stream and passes via

the circulatory system to all parts of the

body

Q158 Bulb is a modification of which

part of a plant

बमबएकपह९धहॳकहॳ ककसटहससहॳकाएक पातरणहह८ताहहॴ 27-Jan-2017

Options

1) The root

जड़

2) The stem

तना 3) The radicle

मिाकर

4)The fruit

फि

Correct Answer The stem

Q159 Which of the following carries

blood away from the heart to different

body parts

इनमहॳसहॳकह९नरकतकह८हदयसहॳिरीरकहॳ पवलभननअगह८तकिहॳजातीहहॴ

27-Jan-2017

Options

1) Arteries

धमननया 2) Nerves

तबतरहाए

3) Capillaries

कहॳ लिकाए

4)Veins

नसहॳ Correct Answer Arteries

Q160 The series of processes by which

nitrogen and its compounds are

interconverted in the environment and

in living organisms is called

27-Jan-2017

Options

1)Absorption of Nitrogen

2)Ammonification

3)Nitrogen Fixation

4)Nitrogen Cycle

Correct Answer Nitrogen Cycle

Ammonification or Mineralization is

performed by bacteria to convert

organic nitrogen to ammonia

F A C E B O O K

P A G E h t t p w w w f a c e b o o k c o m s s c m e n t o r s o f f i c i a l P a g e | 38

FOR MORE UPDATES AND MORE MATERIAL DO LIKE OUR FACEBOOK PAGE httpwwwfacebookcomsscmentorsofficial

Nitrification can then occur to convert

the ammonium to nitrite and nitrate

Nitrogen fixation is a process by which

nitrogen in the Earthrsquos atmosphere is

converted into ammonia (NH3) or other

molecules available to living organisms

Q161 BCG vaccine is given to protect

from which of the following

बीसीजीकाटटकानननननलिखितमसहॳककसकहॳ बचावकहॳ लिएटदयाजातहहॴ

27-Jan-2017

Options

1) Jaundice

पीलिया 2) Anaemia

रकतमपता 3) Tuberculosis

कषयरह८ग

4) Polio

पह८लियह८ Correct Answer Tuberculosis

Q162 Parallel venation is found in

समानतरवहॳनहॳिन mdashmdashmdash- मपायाजाताहहॴ 27-Jan-2017

Options

1) plants which are monocots

पह९धहॳजह८एकबीजपतरीहह८तहॳहहॴ 2) plants which have a dicot stem

वहॳपह९धहॳनजनकातनादपवदलियहह८ताहहॴ 3) plants with leaves similar to Tulsi

वहॳपह९धहॳनजनकीपनततयतिसीकीपनततयोकहॳ समानहह८तहॳहहॴ 4)plants with tap roots

टहॳप टवािहॳपह९धहॳ Correct Answer plants which are

monocots

Q163 The hardest part of the body is

िरीरकासबसहॳकठह८रभाग mdashndash हहॴ 27-Jan-2017

Options

1) Bones

हडडिय

2) Tooth Enamel

दातकहॳ इनहॳमि

3) Skull

िह८पड़ी 4) Spinal Cord

महॳ रजज

Correct Answer Tooth Enamel

Q164 Which type of pathogen causes

the waterborne disease E coli Infection

ककसपरकारकारह८गजननकजिजननतरह८गईकह८िाईसिमणकाकारणबनताहहॴ 27-Jan-2017

Options

1) Protozoan

परह८टह८जआ

2) Parasitic

परजीवी 3) Bacterial

बहॴकटीररयि

4)Viral

वायरि

Correct Answer Bacterial

Q165 The amount of blood filtered

together by both the kidneys in a 70 kg

adult male human in a minute is

70 की गरा वािहॳएकवयसकप षमएकलमनटमदह८नोगदकहॳदवाराएकसािचाबनीगयीरकतकीमातरहह८तीहहॴ 29-Jan-2017

Options

1) 1100 ml

1100 लमलि

2) 100 ml

F A C E B O O K

P A G E h t t p w w w f a c e b o o k c o m s s c m e n t o r s o f f i c i a l P a g e | 39

FOR MORE UPDATES AND MORE MATERIAL DO LIKE OUR FACEBOOK PAGE httpwwwfacebookcomsscmentorsofficial

100 लमलि

3) 1500 ml

1500 लमलि

4) 500 ml

500 लमलि

Correct Answer 1100 ml

Q166 Which feature of a plant helps to

distinguish a monocot from a dicot

पह९धहॳकीवहकह९नसीपविहॳषताहहॴजह८एकदपवदलियहॳऔरएकएकदिीयपह९धहॳसहॳभहॳदकरनहॳममददकरतीहहॴ 29-Jan-2017

Options

1) Pollination

परागम

2) Venation

वहॳनहॳिन

3) Vernation

वनिन

4) Aestivation

एसटीवहॳिहॳन

Correct Answer venation

Q167 The Mutation Theory was

proposed by

उतवररवतयनकालसदात mdashmdashndash

कहॳ दवरापरसतापवतककयाजाताहहॴ 29-Jan-2017

Options

1) Charles Lyell

चामसयलियहॳि

2) William Smith

पवलियमनसमि

3) Hugo De Vries

हयगह८िीराईस

4)Harrison Schmitt

हहॳरीसननसमट

Correct Answer Hugo De Vries

Q168 Which type of pathogen causes

the waterborne disease HepatitisA

ककसपरकारकहॳ रह८गजनकजिजननतरह८गहहॳपहॳटाइटटस-A काकारणबनताहहॴ

29-Jan-2017

Options

1) Parasitic

परजीवी 2) Viral

वायरि

3) Protozoan

परह८टह८जआ

4) Bacterial

बहॴकटीररयि

Correct Answer Viral

Q169 In a Punnett Square with the

cross AaBb x Aabb how many AaBb

genotypes would be created

पनहॳटसकवायरमिह८स AaBb x Aabb

कहॳ सािककतनहॳ AaBb जीनह८टाइपबनगहॳ 29-Jan-

2017

Options

1) 4

2) 1

3) 7

4) 6

Correct Answer 4

Q170 Arboreal Ateles is the scientific

name of

अिह८ररयिएटटलिस mdashmdashmdash कावहॴजञाननकनामहहॴ 29-Jan-2017

Options

1) Squirrel

चगिहरी 2) Sparrow

गह८रहॴया 3) Lizard

नछपकिी 4) Spider monkey

F A C E B O O K

P A G E h t t p w w w f a c e b o o k c o m s s c m e n t o r s o f f i c i a l P a g e | 40

FOR MORE UPDATES AND MORE MATERIAL DO LIKE OUR FACEBOOK PAGE httpwwwfacebookcomsscmentorsofficial

मकड़ीबदर

Correct Answer Spider monkey

Q171 Which type of pathogen causes

the waterborne disease Salmonellosis

ककसपरकारकारह८गाणजिजननतबीमारीसािमह८नहॳिह८लसज़काकारकहहॴ

29-Jan-2017

Options

1) Algal

िहॳवालियहॳ 2) Parasitic

परजीवी 3) Bacterial

बहॴकटीररयि

4)Viral

वायरि

Correct Answer Bacterial

An infection with salmonella bacteria

commonly caused by contaminated food

or water

Symptoms include diarrhoea fever

chills and abdominal pain

Q172 is a condition in which there is a

deficiency of red cells or of haemoglobin

in the blood

mdashmdash-

एकनसिनतहहॴनजसमहॳरकतमिािकह८लिकाओकीयाहीमह८गिह८बबनकीकमीहह८तीहहॴ 29-Jan-2017

Options

1) Albinism

एनमबननजम

2) Propyria

परह८पीररया 3) Anaemia

एनीलमया 4)Keloid disorder

कहॳ िह८इिडिसओिर

Correct Answer Anaemia

Q173 Ananas comosus is the scientific

name of

Options

अनानासकह८मह८सस mdashmdashmdashndash

कावहॴजञाननकनामहहॴ 29-Jan-2017

1) Custard Apple

सीताफि

2) Pineapple

पाइनएपपि

3) Bamboo

बास

4)Pomegranate

अनार

Correct Answer Pineapple

Q174 Which organ produces insulin

कह९नसाअगइनसलिनपहॴदाकरताहहॴ 29-Jan-

2017

Options

1) Liver

यकत

2) Thyroid gland

िायराइिगरिी 3) Spleen

पिीहा 4)Pancreas

अगरयिय

Correct Answer Pancreas

Q175 Which of the following disease is

not caused by water pollution

नननननलिखितमसहॳकह९नसारह८गपानीकहॳ परदषणकहॳकारणनहीहह८ता

29-Jan-2017

Options

1) Cholera

हहॴजा 2) Typhoid

F A C E B O O K

P A G E h t t p w w w f a c e b o o k c o m s s c m e n t o r s o f f i c i a l P a g e | 41

FOR MORE UPDATES AND MORE MATERIAL DO LIKE OUR FACEBOOK PAGE httpwwwfacebookcomsscmentorsofficial

टाइफाइि

3) Asthma

दमा 4)Diarrhoea

दसत

Correct Answer Asthma

Q176 Ocimum tenuiflorum is the

scientific name of

ओलिलममटहॳयईफिह८रमइसकावहॴजञाननकनाम mdash

ndash हहॴ 30-Jan-2017

Options

1) Neem

नीम

2) Mango

आम

3) Babul

बबि

4)Tulsi

तिसी Correct Answer Tulsi

Q177 Which gland secretes bile a

digestive fluid

कह९नसीगरिीपपतत एकपाचनतरिपरदािय सरापवतकरतीहहॴ 30-Jan-2017

Options

1) Pancreas

अगनयािय

2) Liver

यकत

3) Thyroid

िायराइि

4) Testes

टहॳनसटस

Correct Answer liver

Q178 In which of the following the

dominant phase is Gametophyte

नननननलिखितमसहॳककसकहॳ परमिचरणयगमकह८दपवधद (Gametophyte)हहॴ 30-Jan-2017

Options

1) Bryophyta

िायह८फाइटा 2) Pteridophyta

टहॳररिह८फाइटा 3) Gymnosperms

नजननह८सपमय 4) Angiosperms

एननजयह८सपमय Correct Answer Bryophyta

Q179 Anaerobic respiration refers to

which of the following

नननननलिखितमसहॳककसहॳअवायवीयशवसनकहाजाताहहॴ

30-Jan-2017

Options

1) Respiration without Oxygen

ऑकसीजनकहॳ बबनाशवसन

2) Respiration with Oxygen

ऑकसीजनकहॳ सािशवसन

3) Respiration without CO2

काबयनिायऑकसाइिकहॳ बबनाशवसन

4) Respiration with CO2

काबयनिायऑकसाइिकहॳ सािशविन

Correct Answer Respiration without

Oxygen

Q180 Which type of pathogen causes

the waterborne disease Cholera

ककसपरकारकारह८गजनकजिजननतरह८गहहॴजाकाकारणबनताहहॴ

30-Jan-2017

Options

1) Algal

िहॴवालियहॳ

F A C E B O O K

P A G E h t t p w w w f a c e b o o k c o m s s c m e n t o r s o f f i c i a l P a g e | 42

FOR MORE UPDATES AND MORE MATERIAL DO LIKE OUR FACEBOOK PAGE httpwwwfacebookcomsscmentorsofficial

2) Bacterial

बहॴकटीररयि

3) Protozoan

परह८टह८जआ

4) Viral

वायरि

Correct Answer Bacterial

Q181 To which class does

Oxyreductases transferases hydrolases

belong

ओकसीररिकटहॳसटरासफरहॳजहॳस

हाइडरह८िहॳसहॳसककसवगयमआतहॳहहॴ 30-Jan-2017

Options

1) Hormones

हारमोस

2) Enzymes

एजाइनस

3) Proteins

परह८टीनस

4) Vitamins

पवटालमनस

Correct Answer Enzymes

Q182 Which of the following is not true

about Gymnosperms

ननननमसहॳकह९नसीबातअनावतबीजीकहॳ बारहॳमसचनहीहहॴ 30-Jan-2017

Options

1) Dominant phase is saprophytes

परमिचरणसहॳपरह८फाइटसहह८ताहहॴ 2) Vascular bundles are absent

सवहनीबििअनपनसितहह८ताहहॴ 3) spores are heterospores

बीजाणहहॳटहॳरह८सपह८रसहह८तहॳहहॴ 4) Flowers are absent

फिअनपनसितहह८तहॳहहॴ

Correct Answer Vascular bundles are

absent

Q183 The name of first mammal clone sheep is

भहॳड़कीपरिमसतनपायीपरनत प (किह८न)

कानामहहॴ 30-Jan-2017

Options

1) Noori

नरी 2) Dolly

िॉिी 3) Louise

िसी 4)Durga

दगाय Correct Answer Dolly

Q184 Which type of pathogen causes

the water-borne disease Typhoid fever

ककसपरकारकारह८गजनकजिजननतरह८गटाइफाइिबिारकाकारणबनताहहॴ 30-Jan-2017

Options

1) Algal

िहॴवािीय

2) Parasitic

परजीवी 3) Protozoan

परह८टह८जनअन

4)Bacterial

बहॴकटीररयि

Correct Answer Bacterial

Q185 In which part of the cell are

proteins made

कह८लिकाकहॳ ककसटहससहॳमपरह८टीनबनायाजाताहहॴ

31-Jan-2017

Options

1) Reticulum

रहॳटटकिम

F A C E B O O K

P A G E h t t p w w w f a c e b o o k c o m s s c m e n t o r s o f f i c i a l P a g e | 43

FOR MORE UPDATES AND MORE MATERIAL DO LIKE OUR FACEBOOK PAGE httpwwwfacebookcomsscmentorsofficial

2) Golgi apparatus

गह८मजीएपहॳरहॳटस

3) Ribosomes

ररबह८सह८नस

4) Lysosome

िायसह८सह८नस

Correct Answer ribosomes

Proteins are produced by stringing

amino acids together in the order

specified by messenger RNA strands

that were transcribed from DNA in the

cell nucleus The process of synthesizing

a protein is called translation and it

occurs on ribosomes in the cytoplasm of

a cell

Q186 Polio is a disease caused by which

of the following

नननननलिखितमसहॳपह८लियह८कीबबमारह८हह८नहॳकाकारणकयाहहॴ

31-Jan-2017

Options

1) Bacteria

बहॴकटीररयि

2) Mosquito

मचछर

3) Virus

वायरस

4) Cockroach

नतिच हॳ Correct Answer Virus

Polio or poliomyelitis is a crippling and

potentially deadly infectious disease It

is caused by the poliovirus

Q187 ndash Hay fever is a sign of which of

the following

हहॳकफवरनननननलिखितमसहॳककसकाएकसकहॳ तहहॴ

31-Jan-2017

Options

1) Old Age

वदावसिा 2) Malnutrition

कपह८सण

3) Allergy

एिनजय 4) Over Work

अतयचधककाययकरना Correct Answer Allergy

Q188 How many chromosomes does a

human cell contain

एकमानवकह८लिकामककतनहॳगणसतरहह८तहॳहहॴ

29-Jan-2017

Options

1) 6

2) 26

3) 46

4) 66

Correct Answer 46

In humans each cell normally contains

23 pairs of chromosomes for a total of

46 Twenty-two of these pairs called

autosomes look the same in both males

and females The 23rd pair the sex

chromosomes differ between males and

females

Q189 Which of the following is not true

about Bryophyta

ननननमसहॳकह९नसीबातिायह८फाइटकहॳ बारहॳमसचनहीहहॴ 31-Jan-2017

Options

1) Dominant phase is gametophytes

परमिचरणगहॳलमतह८फाइटसहह८ताहहॴ 2) Main plant body is haploid

पह९धहॳकामखयिरीरअगखणतहह८ताहहॴ 3) Spores are homospores

बीजाणहह८मह८सफह८रसहह८तहॳहहॴ 4) Flowers are present

फिमह८जदहह८तहॳहहॴ Correct Answer Flowers are present

F A C E B O O K

P A G E h t t p w w w f a c e b o o k c o m s s c m e n t o r s o f f i c i a l P a g e | 44

FOR MORE UPDATES AND MORE MATERIAL DO LIKE OUR FACEBOOK PAGE httpwwwfacebookcomsscmentorsofficial

Q190 Which aquatic animal has

trailing tentacles

ककसजिीयजानवरकहॳ पीछहॳचिनहॳवािहॳटहॳटकिसहह८तहॳहहॴ

31-Jan-2017

Options

1) Sea horse

समदरीघह८िा 2) Corals

मगा 3) Jelly fish

जहॳिीमछिी 4) Star fish

तारामछिी Correct Answer Jelly fish

Jellyfish with its umbrella-shaped bell

and trailing tentacles

Q191 Which type of pathogen causes

the water-borne disease Poliomyelitis

(Polio)

ककसपरकारकारह८गजनकजिजननतरह८गपह८लियह८मायहॳटटस (पह८लियह८) काकारणहहॴ 31-Jan-

2017

Options

1) Parasitic

परजीवी 2) Algal

िहॴवालिय

3) Viral

वायरि

4) Bacterial

बहॴकटीररयि

Correct Answer Viral

Q192 The outer white part of the eye

that protects the inner structures is

आािकाबाहरीसफहॳ दटहससाजह८आतररकसरचनाओकीरकषाकरताहहॴ वह mdashmdashmdash हहॴ 31-Jan-

2017

Options

1) Iris

आयररस

2) Sclera

सकिहॳरा 3) Retina

रहॳटटना 4) Cornea

कह८ननयया Correct Answer Sclera

Q193 Proteins are made up of

परह८टीनकाननमायण mdashndash सहॳहह८ताहहॴ 31-Jan-2017

Options

1) Amino acids

एलमनह८अनि

2) Fatty acids

वसायकतअनि

3) Glucose

गिकह८ज

4)Nucleotides

नयनकियह८टाईिस

Correct Answer Amino acids

Q194 Moringa Oleifera is the scientific

name of

मह८ररगओलिफहॳ रा mdashmdashndash कावहॴजञाननकनामहहॴ 31-Jan-2017

Options

1) Banyan

बरगद

2) Gulmohar

गिमह८हर

3) Amla

आमिा

F A C E B O O K

P A G E h t t p w w w f a c e b o o k c o m s s c m e n t o r s o f f i c i a l P a g e | 45

FOR MORE UPDATES AND MORE MATERIAL DO LIKE OUR FACEBOOK PAGE httpwwwfacebookcomsscmentorsofficial

4) Drumstick

डरमनसटक

Correct Answer Drumstick

Q195 Kidney stones are composed of

गदकीपिरी mdashndash सहॳबनीहह८तीहहॴ 1-Feb-2017

Options

1) Calcium Oxalate

कहॴ नमसयमओकजहॳिहॳट

2) Sodium Chloride

सह८डियमकिह८राइि

3) Magnesium Nitrate

महॳनगनलियमनाइतटरहॳट

4) Calcium Bicarbonate

कहॴ नमियमबायकबोनहॳट

Correct Answer Calcium Oxalate

Q196 ndash Which of the following is not

true about Angiosperms

ननननमसहॳकह९नसीबातआवतबीजीकहॳ बारहॳमसचनहीहहॴ 1-Feb-2017

Options

1) Dominant phase is gametophytes

परमिचरणगहॳलमतह८फाइटहह८ताहहॴ 2) Vascular bundles are present

सवहनीबििमह९जदहह८ताहहॴ 3) Spores are heterospores

बीजाणहहॳटहॳरह८सपह८रसहह८तहॳहहॴ 4) Seeds are covered

बीजढकहॳ हह८तहॳहहॴ Correct Answer Dominant phase is

gametophytes

Q197 All of the following are excretory

(waste) products of animals except

नननननलिखितमसहॳककसएककह८छह८ड़करअनयसभीपराखणयोदवाराउतसनजयतपदाियहहॴ 1-Feb-

2017

Options

1) Uric Acid

यररकएलसि

2) Ammonia

अमह८ननया 3) Carbohydrates

काबोहाइडरहॳट

4) Urea

यररया Correct Answer Carbohydrates

In animals the main excretory products

are carbon dioxide ammonia (in

ammoniotelics) urea (in ureotelics) uric

acid (in uricotelics) guanine (in

Arachnida) and creatine

Q198 RNA is a polymeric molecule

What does RNA stand for

आरएनइएएकबहिकआणहहॴ इसकाकापवय पकयाहहॴ 1-Feb-2017

Options

1) Rado Nuclear Acid

रािह८नयनकियरएलसि

2) Ribo Nucleic Acid

राइबह८नयनकिकएलसि

3) Rhino Nuclear Acid

हाइनह८नयनकियरएलसि

4) Resto Nucleus Acid

रहॳसटह८नयकिीयसएलसि

Correct Answer Ribo Nucleic Acid

Q199 Which organ does detoxification

and produces chemicals needed for

digestion

कह९नसाअगपवषहरणकरताहहॴऔरपाचनकहॳ लिएआवशयकरसायनोकह८पहॴदाकरताहहॴ 1-Feb-

2017

Options

1) Salivary glands

िारगरचिया 2) Pancreas

अगनयािय

F A C E B O O K

P A G E h t t p w w w f a c e b o o k c o m s s c m e n t o r s o f f i c i a l P a g e | 46

FOR MORE UPDATES AND MORE MATERIAL DO LIKE OUR FACEBOOK PAGE httpwwwfacebookcomsscmentorsofficial

3) Thyroid gland

िायराइिगरिी 4) Liver

यकत

Correct Answer Liver

Q200 Psidium guajava is the scientific

name of

लसडियमगआजावा mdashmdash कावहॴजञाननकनामहहॴ 1-

Feb-2017

Options

1) Guava

अम द

2) Mango

आम

3) Bamboo

बास

4) Jack fruit

कटहि

Correct Answer Guava

Q201 Which drug is used as a Blood

Thinner

चधरकह८पतिाकरनहॳकहॳ पमककसदवाकापरयह८गककयाजाताहहॴ

1-Feb-2017

Options

1) Warfarin

वाफर न

2) Tramadol

टरहॳमािह८ि

3) Azithromycin

एनजरह८मायलसन

4) Hydralazine

हाइडरह८िहॳनजन

Correct Answer Warfarin

Q202 Which of the following disease is

caused due to the deficiency of protein

परह८टीनकीकमीकहॳ कारणनननननलिखितमसहॳकह९नसारह८गहह८ताहहॴ 1-Feb-2017

Options

1) Arthritis

गटठया 2) Kwashiorkor

कािीओकय र

3) Goitre

गाइटर

4) Night Blindness

रतह९चध

Correct Answer Kwashiorkor

Q203 A is species of plant that has

adapted to survive in an environment

with little liquid water

mdashmdashndashपह९धहॳकीएकऐसहॳऐसहॳपरजानतहहॴ नजसनहॳकमपानीवािहॳवातावरणमजीपवतरहनहॳकहॳलिएअनकिनहहॴ 1-Feb-2017

Options

1) Xerophyte

म दपवद

2) Hydrophyte

जिीयपादप

3) Mesophyte

समह८दपवद

4) Thallophyte

िहॴिह८फाइटा Correct Answer xerophyte

xerophyte is a species of plant that has

adapted to survive in an environment

with little liquid water such as a desert

or an ice- or snow-covered region in the

Alps or the Arctic

Mesophytes are terrestrial plants which

are adapted to neither a particularly

dry nor particularly wet environment

An example of a mesophytic habitat

would be a rural temperate meadow

F A C E B O O K

P A G E h t t p w w w f a c e b o o k c o m s s c m e n t o r s o f f i c i a l P a g e | 47

FOR MORE UPDATES AND MORE MATERIAL DO LIKE OUR FACEBOOK PAGE httpwwwfacebookcomsscmentorsofficial

which might contain goldenrod clover

oxeye daisy and Rosa multiflora

thallophyte any of a group of plants or

plantlike organisms (such as algae and

fungi) that lack differentiated stems

leaves and roots and that were formerly

classified as a primary division

(Thallophyta) of the plant kingdom

Q204 How many types of teeth are

there in humans

मनषयोमककतनहॳपरकारकहॳ दातहह८तहॳहहॴ

1-Feb-2017

Options

1) 4

2) 5

3) 2

4) 3

Correct Answer 4

teeth -Humans have four types of

teethincisors canines premolars and

molars each with a specific function

The incisors cut the food the canines

tear the food and the molars and

premolars crush the food

Q205 Carica papaya is the scientific name of

कहॴ ररकापपाया mdashmdashndash कावहॴजञाननकनामहहॴ 2-

Feb-2017

Options

1) Peepal

पीपि

2) Papaya

पपीता 3) Tamarind

इमिी 4) Drumstick

ढह८िकाछड़ी Correct Answer Papaya

Q206 Muscles get tired when there is

shortfall of

जब mdashndash कीकमीहह८तीहहॴतबपहॳिीयिकजातीहहॴ 2-Feb-2017

Options

1) Lactic acid

िहॴनकटकएलसि

2) Na+ ions

Na+ आयन

3) ATP

एटीपी 4) Sulphates

समफहॳ टस

Correct Answer ATP

ATP is the energy source muscle fibers

use to make muscles contract

muscle tissuersquos main source of energy

called adenosine triphosphate or ATP

As your muscles use up this energy

source they become tired and fatigued

Oxygen is the key ingredient that helps

create new ATP to replenish the burned

up ATP in your muscles

Q207 Artocarpus integra is the

scientific name of आटह८कापयसइटीगरा mdashmdashmdash कावहॴजञाननकनामहहॴ 2-Feb-2017

Options

1) Guava

अम द

2) Pineapple

अनानास

3) Silver Oak

लसमवरओक

4) Jack fruit

कटहि

Correct Answer Jack fruit

Q208 Which organ stores fat soluble

vitamins

कह९नसाअगवसामघिनिीिपवटालमनह८काभिाराकरताहहॴ

2-Feb-2017

F A C E B O O K

P A G E h t t p w w w f a c e b o o k c o m s s c m e n t o r s o f f i c i a l P a g e | 48

FOR MORE UPDATES AND MORE MATERIAL DO LIKE OUR FACEBOOK PAGE httpwwwfacebookcomsscmentorsofficial

Options

1) Blood

रकत

2) Skin

तवचा 3) Liver

यकत

4) Pancreas

अगनयािय

Correct Answer Liver

Q209 Which disease is caused due to

deficiency of Iodine

आयह८िीनकहॳ कारणकह९नसारह८गहह८ताहहॴ 2-Feb-2017

Options

1) Rickets

ररकहॳ टस

2) Scurvy

सकवी 3) Goitre

गणमािा 4) Growth retardation

पवकासका कना Correct Answer Goitre

rickets A softening and weakening of

bones in children usually due to

inadequate vitamin D

Q210 Grevillea Robusta is the scientific name of

गरहॳपवलियारह८बसटा mdashmdashmdash- कापवजञाननकनामहहॴ 2-Feb-2017

Options

1) Peepal

पीपि

2) Teak

सागह९न

3) Silver Oak

लसमवरओक

4) Jack fruit

कटहि

Correct Answer Silver Oak

Q211 When a Cuttlefish is described as a Molluscs it is at which level of

classification

जबएककटिकफिकह८एकमह८िसकाकहॳ पमवखणयतककयाजाताहहॴतबयहॳवगीकरणकहॳ ककससतरपहॳनसितहहॴ 2-Feb-2017

Options

1) Class

वगय 2) Order

िम

3) Family

पररवार

4) Phylum

सघ

Correct Answer Phylum

Q212 Bambusa dendrocalmus is the

scientific name of बानबसािहॳडराकामस mdashmdashmdash कावहॴजञाननकनामहहॴ 3-Feb-2017

Options

1) Banyan

बरगद

2) Papaya

पपीता 3) Bamboo

बास

4) Pomegranate

अनार

Correct Answer Bamboo

Q213 Acinonyx Jubatus is the scientific name of

एलसनह८ननकसजयबहॳटस mdashmdashmdash

कावहॴजञाननकनामहहॴ 3-Feb-2017

F A C E B O O K

P A G E h t t p w w w f a c e b o o k c o m s s c m e n t o r s o f f i c i a l P a g e | 49

FOR MORE UPDATES AND MORE MATERIAL DO LIKE OUR FACEBOOK PAGE httpwwwfacebookcomsscmentorsofficial

Options

1) Bear

भाि 2) Horse

घह८िा 3) Cheetah

चीता 4) Zebra

जहॳिा Correct Answer Cheetah

Q214 The pale yellow colour of urine is

due to the presence of which pigment

मतरकाफीकापीिारगरगदरयकहॳ उपनसिनतकहॳ कारणहह८ताहहॴ

3-Feb-2017

Options

1) Urochrome

यरह८िह८म

2) Urophyll

यरह८कफि

3) Chlorophyll

किह८रह८कफि

4) Chloroplast

किह८रह८पिासट

Correct Answer Urochrome

Q215 Which of the following constitute

to form a gene

नननननलिखितमसहॳकह९नसीचीज़एकजीनकागठनकरतीहहॴ

3-Feb-2017

Options

1) Polynucleotides

पह८िीनयनकियह८टाईडस

2) Hydrocarbons

हाइडरह८काबोस

3) Lipoproteins

िाईपह८परह८टीनस

4) Lipids

लिपपडस

Correct Answer Polynucleotides

Polynucleotide molecule is a biopolymer

composed of 13 or more nucleotide

monomers covalently bonded in a chain

DNA (deoxyribonucleic acid) and RNA

(ribonucleic acid) are examples of

polynucleotides with distinct biological

function

Q216 Vertebrates belongs to the

phylum

रीढ़कीहडिीवािहॳपराणी mdashmdashmdash

परजानतकहॳ अतगायतआतहॳहहॴ 3-Feb-2017

Options

1) Arthropoda

आरह८पह८ड़ा 2) Annelida

एननलििा 3) Cnidaria

ननिहॳररया 4) Chordata

कह८िटा Correct Answer Chordata

Q217 Punica granatum is the scientific name of

पननकगरहॳनहॳटस mdashmdashmdash कावहॴजञाननकनामहहॴ 3-Feb-2017

Options

1) Custard Apple

सीताफि

2) Gulmohar

गिमह८हर

3) Silver Oak

लसमवरओक

4) Pomegranate

अनार

Correct Answer Pomegranate

F A C E B O O K

P A G E h t t p w w w f a c e b o o k c o m s s c m e n t o r s o f f i c i a l P a g e | 50

FOR MORE UPDATES AND MORE MATERIAL DO LIKE OUR FACEBOOK PAGE httpwwwfacebookcomsscmentorsofficial

Q218 Between a tiger and an monkey

which of the following is different

एकबाघऔरबदरकहॳ बीचनननननलिखितमसहॳकह९नसीबातअिगहहॴ 3-Feb-2017

Options

1) Kingdom

राजय

2) Phylum

जानत

3) Order

िम

4) Class

वगय Correct Answer order

Q219 The artificial heart was invented by

कबतरमहदयका mdashmdashmdash

दवाराअपवषकारककयागयािा 3-Feb-2017

Options

1) Muhammad Yunus

महनमदयनस

2) Linus Yale Jr

िाइनसयहॳिजय

3) Gazi Yasargil

गाजीयासचगयि

4) Paul Winchell

पह९िपवमकि Correct Answer Paul Winchell

Q220 Tamarindus indica is the

scientific name of

टहॳमररनडसइडिका mdashmdash कावहॴजञाननकनामहहॴ 7-

Feb-2017

Options

1) Neem

नीम

2) Pineapple

अनानास

3) Tamarind

इमिी 4)Chiku

चीक

Correct Answer Tamarind

Q221 In eukaryotic cells synthesis of

RNA takes place in the

यकहॳ योटटककह८लिकाओमआरएनएकासशिहॳषण

mdashndash महह८ताहहॴ 7-Feb-2017

Options

1) Mitochondria

माईटह८कोडडरया 2) Centrioles

सटरीयह८मस

3) Ribosomes

ररबह८सह८नस

4) Nucleus

नयनकियस

Correct Answer nucleus

eukaryotic cell -Transcription is the

process of synthesizing ribonucleic acid

(RNA)Synthesis takes place within the

nucleus of eukaryotic cells or in the

cytoplasm of prokaryotes and converts

the genetic code from a gene in

deoxyribonucleic acid ( DNA ) to a

strand of RNA that then directs

proteinsynthesis

Q222 _________is caused by parasites

of the Plasmodium genus

पिाजमह८डियमजातीकहॳ परजीवी mdash- कहॳ कारणहहॴ 7-Feb-2017

Options

1) Dysentery

पहॳचचि

2) Malaria

मिहॳररया 3) Chickenpox

F A C E B O O K

P A G E h t t p w w w f a c e b o o k c o m s s c m e n t o r s o f f i c i a l P a g e | 51

FOR MORE UPDATES AND MORE MATERIAL DO LIKE OUR FACEBOOK PAGE httpwwwfacebookcomsscmentorsofficial

चहॳचक

4) Herpes

हहॳपपयस

Correct Answer Malaria

Q223 Carotene in fruits and vegetables

gives it which color

फिह८औरसनलजयोमनसितकहॳ रह८टीनउनहकह९नसारगपरदानकरताहहॴ 7-Feb-2017

Options

1) Green

हरा 2) Pink

गिाबी 3) Orange

नारगी 4) Blue

नीिा Correct Answer Orange

Q224 Equus Caballus is the scientific

name of

एकवसकहॴ बहॳिस mdashmdashndash कापवजञाननकनामहहॴ 7-Feb-2017

Options

1) Horse

घह८िा 2) Zebra

ज़हॳिा 3) Donkey

गधा 4) Buffalo

भस

Correct Answer Horse

Q225 Elapidae Naja is the scientific name of

एिीपीिीनाजा mdashmdash- कावहॴजञाननकनामहहॴ 8-Feb-2017

Options

1) Cobra

कह८बरा 2) Elephant

हािी 3) Eagle

ग ि

4) Owl

उमि Correct Answer Cobra

Q226 Which disease is caused due to

deficiency of Iron

िह८हकीकमीकहॳ कारणकह९नसारह८गहह८ताहहॴ 8-Feb-

2017

Options

1) Beriberi

बहॳरीबहॳरी 2) Tetany

टहॳटनी 3) Kwashiorkor

कवािीऔरकर

4) Anaemia

रकतामपता Correct Answer Anaemia

Beriberi is a disease caused by a vitamin

B-1 deficiency also known as thiamine

deficiency

Tetany can be the result of an

electrolyte imbalance Most often itrsquos a

dramatically low calcium level also

known as hypocalcemia Tetany can also

be caused by magnesium deficiency or

too little potassium Having too much

acid (acidosis) or too much alkali

(alkalosis) in the body can also result in

tetany

Kwashiorkor also known as

ldquoedematous malnutrition It is a form of

malnutrition caused by a lack of protein

in the diet

Anaemia means that you have fewer red

blood cells than normal or you have less

F A C E B O O K

P A G E h t t p w w w f a c e b o o k c o m s s c m e n t o r s o f f i c i a l P a g e | 52

FOR MORE UPDATES AND MORE MATERIAL DO LIKE OUR FACEBOOK PAGE httpwwwfacebookcomsscmentorsofficial

haemoglobin than normal in each red

blood cell

Q227 is a leaf where the leaflets are

arranged along the middle vein

mdashndashएकपततीहहॴजहापतरकह८कीरचनाक ररयालिराकहॳ आसपासहह८तीहहॴ 8-Feb-2017

Options

1) Pinnately compound leaf

पपनहॳटिीसयकतपतती 2) Palmately compound leaf

पामहॳटिीसयकतपतती 3) Compound leaf

सयकतपतती 4) Simple leaf

साधारणपतती Correct Answer Pinnately compound

leaf

Q228 Haustoria or sucking roots are

found in which of the following

हह८सटह८ररयायाचसनहॳवािीजड़हॳनननननलिखितमसहॳककसमपाईजातीहहॴ 8-Feb-2017

Options

1) Wheat

गहॳह

2) Mango

आम

3) Chestnut

चहॳसटनट

4) Cuscuta

कसकयटा Correct Answer Cuscuta

Haustorial roots -The roots of parasitic

plants which penetrate into the host

tissues to absorb nourishment are

called haustorial roots hellip Also known as suckingor parasitic roots

Q229 Equs Asinus is the scientific name

of

एकवसएलसनस mdashmdashndash कावहॴजञाननकनामहहॴ 8-

Feb-2017

Options

1) Donkey

गधा 2) Cow

गाय

3) Deer

टहरन

4) Kangaroo

कगा

Correct Answer Donkey

Q230 Ficus benghalensis is the scientific name of

फाईकसबहॳनगहॳिहॳलसस mdashndash कापवजञाननकनामहहॴ 8-Feb-2017

Options

1) Banyan

बरगद

2) Pineapple

अनानास

3) Babul

बबि

4) Tulsi

तिसी Correct Answer Banyan

Q231 Equus burchellii is the scientific name of

एकवसबचिी mdashmdash- कापवजञाननकनामहहॴ 8-Feb-2017

Options

1) Horse

घह८िा 2) Zebra

जहॳिा 3) Buffalo

F A C E B O O K

P A G E h t t p w w w f a c e b o o k c o m s s c m e n t o r s o f f i c i a l P a g e | 53

FOR MORE UPDATES AND MORE MATERIAL DO LIKE OUR FACEBOOK PAGE httpwwwfacebookcomsscmentorsofficial

भस

4) Ass

गधा Correct Answer Zebra

Page 7: COMPILATION OF ALL 72 SETS OF BIOLOGY SSC CHSL-2016 · OF BIOLOGY SSC CHSL-2016 PREPARED BY : SSC MENTORS BIOLOGY SPECIAL . F A C E B O O K P A G E : h t t p : / / w w w . f a c e

F A C E B O O K

P A G E h t t p w w w f a c e b o o k c o m s s c m e n t o r s o f f i c i a l P a g e | 6

FOR MORE UPDATES AND MORE MATERIAL DO LIKE OUR FACEBOOK PAGE httpwwwfacebookcomsscmentorsofficial

िािचीटटन

4) Lal Shamak

िाििामक

Correct Answer Bhut Jolokia

Q19 Brain fever is a disease spread

through which of the following

मनसतषकजवरनमकरह८गनननननलिखितमसहॳककसकहॳ कारणहह८ताहहॴ 9-Jan -2017

Options

1) Flies

मनकियो 2) Mosquito

मचछर

3) Virus

वायरस

4) Cockroach

नतिच हॳ Correct Answer Mosquito

Q20 Mangroves are plants that have

मगरह८ववहॳपहॳिहहॴनजनमहॳहह८ताहहॴ 9-Jan -2017

Options

1) Modified Roots

पातररतजड़हॳ 2) Modified Stems

पातररततनहॳ 3) Respiratory Roots

शरवसनकरनहॳवािीजड़हॳ 4) Respiratory Stems

शरवसनकरनहॳवािीतनहॳ Correct Answer Respiratory Roots

A mangrove is a shrub or small tree that

grows in coastal saline or brackish

water

Q21 Rodentia Sciurus is the scientific

name of

रह८िहॳलियासकीयरस mdashmdash कावयजजञाननकनामहहॴ

9-Jan -2017

Options

1) Rat

चहा 2) Platypus

पिहॳटीपस

3) Squirrel

चगिहरी 4) Beaver

बीवर

Correct Answer Squirrel

Q22 Which of the following is induced

by Oncogene

नननननलिखितमसहॳकह९नओकह८जीनदवारापरहॳररयतहह८ताहहॴ

10-Jan -2017

Options

1) Polio

पह८लियह८ 2) Cancer

क सर

3) Diarrhoea

दसत

4) Dengue

िग Correct Answer Cancer

An oncogene is a gene that has the

potential to cause cancer In tumor

cells they are often mutated andor

expressed at high levels

Q23 Azadirachata indica is the

scientific name of

अजाटदराकटाइडिका mdashmdashवहॴजञाननकनामहहॴ SSC CHSL Science (biology) 2016

Question Paper

10-Jan -2017

Options

1) Neem

नीम

F A C E B O O K

P A G E h t t p w w w f a c e b o o k c o m s s c m e n t o r s o f f i c i a l P a g e | 7

FOR MORE UPDATES AND MORE MATERIAL DO LIKE OUR FACEBOOK PAGE httpwwwfacebookcomsscmentorsofficial

2) Teak

सागह९न

3) Silver Oak

लसमवरओक

4) Tulsi

तिसी Correct Answer Neem

Q24 Octopus belongs to the phylum

ऑकटह८पसककसपरजानतकहॳ अतगयतआताहहॴ 10-

Jan -2017

Options

1) Mollusca

मह८िसका 2) Cnidaria

ननिहॳररया 3) Echinodermata

इकाइनह८ड़हॳमता 4) Chordata

कह८िता Correct Answer Mollusca

Q25 A living part of the organisms

environment is known as

जीवाणकहॳ वातावरणकहॳ जीपवतभागकह८ mdash-

कहतहॳहहॴ 10-Jan -2017

Options

1) Abiotic Factor

अजहॴपवककारक

2) Habitat

आवास

3) Biotic Factor

जहॴपवककारक

4) Nonliving factor

अ-जीपवतकारक

Correct Answer Biotic Factor

Abiotic factors are nonndash living chemical

and physical parts of the environment

that affect living organisms and the

functioning of ecosystems like rain

wind temperature altitude soil

pollution nutrients pH types of soil

and sunlight

Q26 Medulla oblongata is a part of

which of the following

महॳडयिाऑबिॉनगहॳटानननननलिखितमसहॳककसअगकाटहससाहहॴ

10-Jan -2017

Options

1) Heart

हदय

2) Brain

मनसतषक

3) Lungs

फहॳ फड़हॳ 4) Stomach

पहॳट

Correct Answer Brain

The medulla oblongata helps regulate

breathing heart and blood vessel

function digestion sneezing and

swallowing This part of the brain is a

center for respiration and circulation

Sensory and motor neurons (nerve cells)

from the forebrain and midbrain travel

through the medulla

Q27 ___________ is a typically

onecelled reproductive unit capable of

giving rise to a new individual without

sexual fusion

mdashmdash एकआमतह९रपरएककह८लिकीयहॳ परजननममसमकषइकाईहहॴजह८यह९नसियनकहॳ बबनाएकनयीइकाईकह८जनमदहॳतीहहॴ 10-Jan -2017

Options

1) Egg

अिाण

2) Spore

बीजाण

F A C E B O O K

P A G E h t t p w w w f a c e b o o k c o m s s c m e n t o r s o f f i c i a l P a g e | 8

FOR MORE UPDATES AND MORE MATERIAL DO LIKE OUR FACEBOOK PAGE httpwwwfacebookcomsscmentorsofficial

3) Sperm

ििाण

4) Seed

बीज

Correct Answer Spore

Q28 Bacteria was discovered by

बहॴकटीररयाकीिह८जककसकहॳ दवाराकीगयीिी

10-Jan -2017

1) Antonie van Leeuwenhoek

एटह८नीवहॳनलिबहॳनहक

2) Belarus

बहॳिा स

3) Hugo de Vries

हयगह८दीराईस

4)Robert Brown

रॉबटयिाउन

Correct Answer Antonie van

Leeuwenhoek

Q29 Which of the following is

responsible for Vermicomposting

नननननलिखितमसहॳकह९नकलमिादकहॳ लिएनजनमहॳदारहहॴ

10-Jan -2017

Options

1) Fungus

कवक

2) Worms

कलम

3) Bacteria

बहॴकटीररया 4) Birds

पकषी Correct Answer Worms

Vermicompost (or vermi-compost) is the

product of the composting process using

various species of worms usually red

wigglers white worms and other

earthworms to create a heterogeneous

mixture of decomposing vegetable or

food waste bedding materials and

vermicast

Q30 Scurvy (bleeding of gums) is

caused by the deficiency of which

vitamin

सकवी (मसढह८सहॳिनआना) ककसपवटालमनकीकमीकहॳ कारणहह८ताहहॴ

10-Jan-2017

Options

1) Vitamin K

पवटालमन K

2) Vitamin BZ

पवटालमन BZ

3) Vitamin C

पवटालमन C

4) Vitamin A

पवटालमन A

Correct Answer Vitamin C

Q31 Achras sapota is the scientific

name of

एिाससपह८ताइसकावहॴजञाननकनामहहॴ 10-Jan-2017

Options

1) Custard Apple

सीताफि

2) Gulmohar

गिमह८हर

3) Tamarind

इमिी 4) Chiku

चचक

Correct Answer Chiku

Q32 Prawn belongs to the phylum

झीगा mdashmdash- परजानतकहॳ अतगयतआताहहॴ 10-Jan-2017

Options

1) Arthropoda

F A C E B O O K

P A G E h t t p w w w f a c e b o o k c o m s s c m e n t o r s o f f i c i a l P a g e | 9

FOR MORE UPDATES AND MORE MATERIAL DO LIKE OUR FACEBOOK PAGE httpwwwfacebookcomsscmentorsofficial

अरोपह८िा 2) Cnidaria

नीिहॳररया 3) Echinodermata

इकाईनह८िमटा 4) Chordata

कह८िटा Correct Answer Arthropoda

Q33 Pulses are a rich source of which of

the following

दािहॳनननननलिखितमसहॳककसकीपरचरसह८तरहहॴ

11-Jan-2017

Options

1) Carbohydrates

काबोहाइडराईट

2) Proteins

परह८टीनस

3) Minerals

िननज

4) Vitamin A

पवटालमन A

Correct Answer Proteins

Q34 Plant cell wall is made up of

वनसपनतकह८लिकालभनततइससहॳबनीहह८तीहहॴ

11-Jan-2017

Options

1) Cellulose

सहॳमयिह८ज

2) Glucose

गिकह८ज

3) Sucrose

सिह८ज

4) Fructose

फरकटह८ज

Correct Answer Cellulose

Plant cell wall the major carbohydrates

are cellulose hemicellulose and pectin

The cellulose microfibrils are linked via

hemicellulosic tethers to form the

cellulose-hemicellulose network which

is embedded in the pectin matrix

Q35 The study of Fungi is also known

as कवकह८कहॳ अधययनकह८कहाजाताहहॴ

11-Jan-2017

Options

1) Cytology

सायटह८िह८जी 2) Myology

मायह८िह८जी 3) Mycology

मायकह८िह८जी 4) Neurology

नयरह८िह८जी Correct Answer Mycology

Cytology - structure and function of

plant and animal cells

Myology is the study of the muscular

system

Neurology is the branch of medicine

concerned with the study and treatment

of disorders of the nervous system

Q36 The outermost layer of skin is

तवचाकीसबसहॳबाहरीपरतकयाहह८तीहहॴ 11-Jan-

2017

Options

1) Epidermis

इपपिलमयस

2) Dermis

िलमयस

3) Tissues

ऊतक

4) Hypodermis

हायपह८िलमयस

Correct Answer Epidermis

Q37 Which of the following plants have

root nodules

F A C E B O O K

P A G E h t t p w w w f a c e b o o k c o m s s c m e n t o r s o f f i c i a l P a g e | 10

FOR MORE UPDATES AND MORE MATERIAL DO LIKE OUR FACEBOOK PAGE httpwwwfacebookcomsscmentorsofficial

नननननलिखितपह९धह८मसहॳककसकीजड़ह८मगाठहह८तीहहॴ

11-Jan-2017

Options

1) Leguminous plants

िहॳगयलमनसपह९धहॳ 2) Parasitic plants

परजीवीपह९धहॳ 3) Epiphytic Plants

एपीफाइटटकपह९धहॳ 4) Aquatic Plants

जिीयपह९धहॳ Correct Answer Leguminous plants

Q38 Earth-worms belongs to the

phylum

कहॳ चएmdashmdash- परजानतकहॳ अतगयतआतहॳहहॴ 11-Jan-2017

Options

1) Protozoa

परह८टह८जआ

2) Cnidaria

नीिहॳररया 3) Annelida

एनीलििा 4) Mollusca

मह८िसका Correct Answer Annelida

Q39 Ringworm is a disease caused by

ररगवमयनामकबीमारी mdashmdash- कहॳ कारणहह८तीहहॴ 11-Jan-2017

Options

1) Fungi

कवक

2) Bacteria

बहॴकटीररया 3) Virus

वायरस

4) Flies

मनकियाा Correct Answer Fungi

Q40 Mangifera indica is the scientific

name of

मननगफहॳ राइडिकाककसकावहॴजञाननकनामहहॴ 11-

Jan-2017

Options

1) Guava

अम द

2) Mango

आम

3) Amla

आविा 4) Jack fruit

कटहि

Correct Answer Mango

Q41 Crabs belongs to the phylum

कहॳ कड़हॳmdashmdash- परजानतकहॳ अतगयतआतहॳहहॴ 11-Jan-2017

Options

1) Mollusca

मह८िसका 2) Cnidaria

नीिहॳररया 3) Arthropoda

अरोपह८ड़ा 4) Platyhelminthes

पिहॳटटहहॳनममननिस

Correct Answer Arthropoda

Q42 Myopia is a defect of eyes which is

also known as

मायह८पपयाआिोकादह८षहहॴ नजसहॳ mdashmdashndash

भीकहाजाताहहॴ

12-Jan-2017

Options

1) Far Sightedness

F A C E B O O K

P A G E h t t p w w w f a c e b o o k c o m s s c m e n t o r s o f f i c i a l P a g e | 11

FOR MORE UPDATES AND MORE MATERIAL DO LIKE OUR FACEBOOK PAGE httpwwwfacebookcomsscmentorsofficial

दरदनषटदह८ष

2) Near Sightedness

ननकटदनषटदह८ष

3) Astigmatism

एसटीगमहॳटटजम

4) Night Blindness

रतोधी Correct Answer Near Sightedness

Myopia occurs when the eyeball is too

long relative to the focusing power of

the cornea and lens of the eye This

causes light rays to focus at a point in

front of the retina rather than directly

on its surface

Hyperopia Hypermetropia (

Farsightedness )- when light rays

entering the eye focus behind the retina

rather than directly on it The eyeball of

a farsighted person is shorter than

normal

Astigmatism usually is caused by an

irregularly shaped cornea Instead of

the cornea having a symmetrically

round shape (like a baseball) it is

shaped more like an American football

Nyctalopia also called night ndash blindness

is a condition making it difficult or

impossible to see in relatively low light

Q43 Who is known as the father of

Green Revolution

हररतिानतकहॳ जनककहॳ पमककसहॳजानाजाताहहॴ

12-Jan-2017

1) Dr Robert Nucleus

िॉ रॉबटयनयनकियस

2) Dr Ian Wilmut

िॉ इयानपविमट

3) Dr NE Borlaug

िॉ एनईबह८रिॉग

4) Dr JC Bose

िॉ जहॳसीबह८स

Correct Answer Dr NE Borlaug

Q44 Panthera Tigris is the scientific

name of

पिहॳराटटगरीस mdashmdashmdash कावहॴजञाननकनामहहॴ 12-Jan-2017

Options

1) Panther

तदआ

2) Tiger

बाघ

3) Whale

हहॳि

4)Goat

बकरी Correct Answer Tiger

Q45 How many facial bones are there

हमारहॳचहॳहरहॳमककतनीहडडियााहह८तीहहॴ 13-Jan-2017

Options

1)34

2)24

3)14

4)4

Correct Answer 14

Q46 ndash Halophytes are plants that grow

in

हहॴिह८फाईटसवहॳपह९धहॳहह८तीहहॴजह८ mdash- मउगतहॳहहॴ SSC CHSL Science (biology) 2016

Question Paper

13-Jan-2017

Options

1) Fresh Water

ताजापानी 2) Cold Water

ठिापानी 3) Ponds

तािाब

4) Salt Water

िारापानी Correct Answer Salt Water

F A C E B O O K

P A G E h t t p w w w f a c e b o o k c o m s s c m e n t o r s o f f i c i a l P a g e | 12

FOR MORE UPDATES AND MORE MATERIAL DO LIKE OUR FACEBOOK PAGE httpwwwfacebookcomsscmentorsofficial

Q47 Felis Catus is the scientific name of

फहॳ लिसकहॴ टस mdashndash कावहॴजञाननकनामहहॴ 13-Jan-2017

Options

1) Cat

बबमिी 2) Dog

कतता 3) Mouse

चहा 4) Porcupine

साही Correct Answer Cat

Q48 Which of the following induces

nitrogen fixation in soil

नननननलिखितमसहॳकह९नलम ीमनाइटरह८जनननयतनकह८परहॳररतकरताहहॴ

15-Jan-2017

Options

1) Protozoa

परह८टह८जआ

2) Bacteria

बहॴकटीररया 3) Fungi

कवक

4)Algae

िहॴवाि

Correct Answer Bacteria

Bacteria that change nitrogen gas from

the atmosphere into solid nitrogen

usable by plants are called nitrogen-

fixing bacteria These bacteria are

found both in the soil and in symbiotic

relationships with plants

They contain symbiotic bacteria called

rhizobia within nodules in their root

systems producing nitrogen compounds

that help the plant to grow and compete

with other plants When the plant dies

the fixed nitrogen is released making it

available to other plant

Q49 Which of the following is the

largest known cell

नननननलिखितमसहॳकह९नसीसबसहॳबड़ीजञातकह८लिकाहहॴ

SSC CHSL Science (biology) 2016

Question Paper

15-Jan-2017

1) Eukaryotic Cell

यकहॳ ररयह८टटककह८लिका 2) Prokaryotic Cell

परह८कहॳ ररयह८टटककह८लिका 3) Mycoplasma

मायकह८पिासम

4) Ostrich Eggs

ितरमगयकाअिा Correct Answer Ostrich Eggs

Q50 The association of animals in

which both the partners are benefitted

is known as

जानवरोकावहसहयह८गनजसमहॳदह८नोभागीदारिाभापवनतहह८तहॳहहॴ उसहॳ mdashmdashndash कहॳ पमजानाजाताहहॴ SSC CHSL Science (biology) 2016

Question Paper

15-Jan-2017

Options

1) Amensalism

सहजीपवत

2) Commensalism

परजीपवत

3) Colony

कॉिनी 4) Mutualism

अनयह८नयाशरयवाद

Correct Answer Mutualism

Amensalism is any relationship between

organisms of different species in which

F A C E B O O K

P A G E h t t p w w w f a c e b o o k c o m s s c m e n t o r s o f f i c i a l P a g e | 13

FOR MORE UPDATES AND MORE MATERIAL DO LIKE OUR FACEBOOK PAGE httpwwwfacebookcomsscmentorsofficial

one organism is inhibited or destroyed

while the other organism remains

unaffected

Commensalism an association between

two organisms in which one benefits and

the other derives neither benefit nor

harm

Q51 Pneumonia affects which of the

following organs of human body

ननमह८ननयामानविरीरकहॳ नननननलिखितमसहॳककसअगकह८परभापवतकरताहहॴ

15-Jan-2017

Options

1)Kidneys

गद

2)Lungs

फहॳ फड़हॳ 3) Throat

गिहॳ 4) Liver

यकत

Correct Answer Lungs

When the germs that cause pneumonia

reach your lungs the lungsrsquo air sacs

(alveoli) become inflamed and fill up

with fluid This causes the symptoms of

pneumonia such as a cough fever

chills and trouble breathing When you

have pneumonia oxygen may have

trouble reaching your blood

Q52 Mendel is known as

मििकह८ mdashmdash- कहॳ पमजानाजाताहहॴ 15-Jan-2017

Options

1) Father of Physiology

िरीरकियािासतरकहॳ जनक

2) Father of Geology

भगभयिासतरकहॳ जनक

3) Father of Genetics

जहॳनहॳटटकसकहॳ जनक

4) Father of Biology

जीविासतरकहॳ जनक

Correct Answer Father of Genetics

Q53 Which of the following are also

known as Suicidal bag of Cells

ननननलिखितमसहॳककसहॳआतमहतयाकरनहॳवािीकह८लिकाओकाबहॴगकहाजाताहहॴ

15-Jan-2017

Options

1) Lysosomes

िायसोसह८म

2) Lycosome

िायकह८सह८म

3) Nucleus

नालभक

4) Chromosome

िह८मह८सह८म

Correct Answer Lysosomes

Q54 Mesothelioma is a type of cancer

The most common area affected in it is

the lining of the ________

लमज़ह८िहॳिहॳलमयाक सरकाएकपरकारहहॴ इससहॳपरभापवतहह८नहॳवािासबसहॳसामानयकषहॳतर mdash

mdashmdash काअसतरहहॴ 15-Jan-2017

Options

1)Heart

हदय

2)Brain

मनसतषक

3)Stomach

आमािय

4)Lungs

फहॳ फड़हॳ Correct Answer lungs

Asbestos exposure is the main cause of

pleural mesothelioma When asbestos

fibers are breathed in they travel to the

F A C E B O O K

P A G E h t t p w w w f a c e b o o k c o m s s c m e n t o r s o f f i c i a l P a g e | 14

FOR MORE UPDATES AND MORE MATERIAL DO LIKE OUR FACEBOOK PAGE httpwwwfacebookcomsscmentorsofficial

ends of small air passages and reach the

pleura where they can cause

inflammation and

scarring

Q55 Which one of the following is an

insectivorous plant

नननननलिखितमसहॳकह९नसाएकककटाहरीवनसपनतहहॴ

15-Jan-2017

Options

1) Utricularia

यटरीकिहॳररया 2) Sequoia

सहॳकयओइया 3) Nostoc

नॉसटह८क

4) Bryophyta

िायह८फाईटा Correct Answer Utricularia

Q56 ______________ is a

multibranched polysaccharide of

glucose that serves as a form of energy

storage in animals and fungi

mdashmdashगिकह८जकाएकबहिािायकतपह८िीसहॳकहॳ राइिहहॴ जह८जानवरोऔरकवकमउजायभणिारणकहॳ एक पमकाययकरताहहॴ 15-Jan-2017

Options

1) Cellulose

सहॳमयिह८ज

2) Glycogen

गिायकह८जन

3) Pectin

पहॳनकटन

4) Chitin

चीटटन

Correct Answer Glycogen

Q57 The largest gland of the human

body is

mdashmdashmdashमानविरीरकीसबसहॳबड़ीगरिीहहॴ 16-Jan-2017

Options

1) Pancreas

अगयािय

2) Thyroid

िायरॉइि

3) Large Intestine

बड़ीआत

4) Liver

यकत

Correct Answer Liver

Q58 Photosynthesis in plants takes

place in

वनसपनतयोमपरकािसशिहॳषणकीकियाहह८तीहहॴ

16-Jan-2017

Options

1) Stem

तना 2) Leaves

पनततयाा 3) Roots

जड़हॳ 4) Flower

फि

Correct Answer Leaves

During this reaction carbon dioxide

and water are converted into glucose

and oxygen The reaction requires light

energy which is absorbed by a green

substance called

chlorophyll Photosynthesis takes place

in leaf

cells These contain chloroplasts which

are tiny objects containing chlorophyll

F A C E B O O K

P A G E h t t p w w w f a c e b o o k c o m s s c m e n t o r s o f f i c i a l P a g e | 15

FOR MORE UPDATES AND MORE MATERIAL DO LIKE OUR FACEBOOK PAGE httpwwwfacebookcomsscmentorsofficial

Q59 Insects that transmit diseases are

known as

जह८कीड़हॳरह८गसचाररतकरतहॳहहॴ उनह mdashmdash-

कहॳ नामसहॳजानाजाताहहॴ 16-Jan-2017

1)Pathogens

रह८गज़नक

2) Vectors

वहॳकटर

3) Drones

परजीवी 4)Scalars

अटदषट

Correct Answer Vectors

A vector is an organism that does not

cause disease itself but which spreads

infection by conveying pathogens from

one host to another Species of mosquito

for example serve as vectors for the

deadly disease Malaria

Q60 Which is the second largest gland

of Human body

मानविरीरकीदसरीसबसहॳबड़ीगरिीकह९नसीहहॴ

SSC CHSL Science (biology)

2016 Question Paper

16-Jan-2017

Options

1) Liver

यकत

2) Large Intestine

बड़ीआत

3) Thorax

छाती 4) Pancreas

अगनयािय

Correct Answer Pancreas

Q61 Annona squamosa is the scientific

name of

एनह८नासकवामह८सा (Annona squamosa) mdash

mdashmdash कावहॴजञाननकनामहहॴ 16-Jan-2017

Options

1) Custard Apple

सीताफि

2) Papaya

पपीता 3) Babhul

बबि

4) Drumstick

सहजन

Correct Answer Custard Apple

Q62 The disease Beri Beri is caused due

to the deficiency of which of the

following

बहॳरीबहॳरीरह८गनननननलिखितमसहॳककसकीकमीकहॳकारणहह८ताहहॴ

16-Jan-2017

Options

1) Vitamin B2

पवटालमन B2

2) Vitamin B1

पवटालमन B1

3) Vitamin B12

पवटालमन B12

4) Vitamin E

पवटालमन E

Correct Answer Vitamin B1

Beriberi is a disease caused by a vitamin

B-1 deficiency also known as thiamine

deficiency

Q63 Chlorophyll was first isolated and

named by

किह८रह८कफिकह८ mdash-

दवारापहिहॳपिकऔरनालमतककयागया 16-Jan-2017

F A C E B O O K

P A G E h t t p w w w f a c e b o o k c o m s s c m e n t o r s o f f i c i a l P a g e | 16

FOR MORE UPDATES AND MORE MATERIAL DO LIKE OUR FACEBOOK PAGE httpwwwfacebookcomsscmentorsofficial

Options

1) Caventou

कहॳ वहॳत 2) Pelletier

पहॳिहॳटटयर

3) Chlorophyll

किह८रह८कफि

4) Caventou and Pelletier

कहॳ वहॳतऔरपहॳिहॳटटयर

Correct Answer Caventou and Pelletier

Chlorophyll was first isolated and

named by

Joseph Bienaimeacute Caventou and Pierre

Joseph Pelletier in 1817 The presence of

magnesium in chlorophyll was

discovered in 1906 and was the first

time that magnesium had been detected

in living tissue

Q64 Which of the following organisms

does not fit into the Cell Theory

नननननलिखितमसहॳकह९नसाजीवकह८लिकालसदातअन पनहीहहॴ

16-Jan-2017

Options

1) Bacteria

बहॴकटीररया 2) Virus

वायरस

3) Fungi

कवक

4) Plants

पह९धहॳ Correct Answer Virus

The bottom line is that viruses are not

alive and not related to cells in any way

The cell theory states that all living

things are made of cells cells are the

basic units of structure and function of

living things and that all cells come

from other cells Since viruses are not

made of cells and do not use cells in any

of their processes they are not related to

the cell theory

Q65 Which of these is not a

macronutrient for Plants

नननननलिखितमसहॳकह९नसापह९धह८कहॳ लिएमिह८नयटरीएटनहीहहॴ

SSC CHSL Science (biology) 2016

Question Paper

17-Jan-2017

Options

1) Nitrogen

नाइटरह८जन

2) Phosphorus

फासफह८रस

3) Potassium

पह८टालसयम

4) Chlorine

किह८रीन

Correct Answer Chlorine

In relatively large amounts the soil

supplies nitrogen phosphorus

potassium calcium magnesium and

sulfur these are often called the

macronutrients In relatively small

amounts the soil supplies iron

manganese boron molybdenum

copper zinc chlorine and cobalt the

so-called micronutrients

Q66 Name the respiratory organs of

insects

कीटह८मनसतिशरवसनअगनामकानामहहॴ

17-Jan-2017

Options

1) Skin

तवचा 2) Body Surface

िरीरकीसतह

F A C E B O O K

P A G E h t t p w w w f a c e b o o k c o m s s c m e n t o r s o f f i c i a l P a g e | 17

FOR MORE UPDATES AND MORE MATERIAL DO LIKE OUR FACEBOOK PAGE httpwwwfacebookcomsscmentorsofficial

3) Gills

गिफड़हॳ 4) Tracheae

शरावस- निी Correct Answer Tracheae

Air enters the respiratory systems of

insects through a series of external

openings called

spiracles These external openings

which act as muscular valves in some

insects lead to the internal respiratory

system a densely networked array of

tubes called tracheae

Q67 The poisonous gas accidentally

released in Bhopal Gas Tragedy is

भह८पािगहॴसतरासदीमगितीसहॳमकतहईजहरीिीगहॴसिी

17-Jan-2017

1) Methane

मीिहॳन

2) Nitrous Oxide

नाइटरसऑकसाइि

3) Methyl Isocyanate

महॴचििआयसोसायनहॳट

4) Cyanogen

सायनह८जहॳन

Correct Answer Methyl Isocyanate

Q68 What does Trypsin do

टटरनपसनकयाकरताहहॴ

SSC CHSL Science (biology) 2016

Question Paper

17-Jan-2017

Options

1) Breaks down Carbohydrates

काबोहाइडरहॳटकापवघटनकरताहहॴ 2) Synthesizes proteins

परह८टीनकासििहॳषणकरताहहॴ 3) Breaks down fats

वसाकापवघटनकरताहहॴ 4) Breaks down proteins

परह८टीनकापवघटनकरताहहॴ Correct Answer Breaks down proteins

Trypsin is one of the three principal

digestive

proteinases the other two being pepsin

and

chymotrypsin In the digestive process

trypsin acts with the other proteinases

to break down dietary protein molecules

to their component

peptides and amino acids

A protease is any enzyme that performs

proteolysis protein catabolism by

hydrolysis of peptide bonds

Q69 Name the source from which

Aspirin is produced

उससरह८तकानामबताइए

नजससहॳएनसपररनकाउतपादनककयाजाताहहॴ

17-Jan-2017

Options

1) Willow bark

पविह८कीछाि

2) Oak Tree

ओककावकष

3) Acacia

बबि

4) Eucalyptus

नीिचगरी Correct Answer Willow bark

The compound from which the active

ingredient in aspirin was first derived

salicylic acid was found in the bark of a

willow tree in 1763 by Reverend

Edmund Stone of Chipping-Norton

Q70 Cannis Familiaris is the scientific

name of

कहॴ ननसफहॳ लमलियहॳररस mdash- कावहॴजञाननकनामहहॴ

17-Jan-2017

F A C E B O O K

P A G E h t t p w w w f a c e b o o k c o m s s c m e n t o r s o f f i c i a l P a g e | 18

FOR MORE UPDATES AND MORE MATERIAL DO LIKE OUR FACEBOOK PAGE httpwwwfacebookcomsscmentorsofficial

Options

1) Cat

बबमिी 2)Dog

कतता 3) Fox

िह८मड़ी 4) Wolf

भहॳडड़या Correct Answer Dog

Q71 Harmful bacteria in potable water

make the water

पीनहॳकहॳ पानीमनसतिघातकबहॴकटीररयाउसपानीकह८बनातहॳहहॴ 17-Jan-2017

Options

1) unfit to drink

पीनहॳकहॳ लिएअयह८गय

2) smelly

दगयनधयकत

3) Colored

रगीन

4) Turbid

मटमहॴिा Correct Answer unfit to drink

Q72 Musa paradisiaca is the scientific

name of which plant

मसापहॴराडिलसयाकाककसपह९धहॳकावहॴजञाननकनामहहॴ

17-Jan-2017

Options

1) Mango

आम

2) Wheat

गहॳह

3) Corn

भ ा 4) banana

कहॳ िा Correct Answer banana

Q73 Prawns belong to which family

झीगहॳककसपररवारकहॳ हह८तहॳहहॴ 17-Jan-2017

Options

1) Crustaceans

िसटहॳलियन

2)Fish

मछिी 3) Amphibians

अननफबबयस

4) Reptiles

रहॳपटाइमस

Correct Answer Crustaceans

Q74 Name the drug that is yielded from

Cinchona tree and is used to cure

malaria

उसऔषचधकानामबताइएनजसहॳलसगकह८नापहॳड़सहॳपरापतककयाजाताहहॴऔरनजसकाउपयह८गमिहॳररयाकहॳ उपचारमककयाजाताहहॴ 17-Jan-2017

Options

1) Camptothea

कहॴ नटह८चिया 2) Acuminata

एकयलमनहॳटा 3) Quinine

कनहॴन

4) Cinchonia

लसकह८ननया Correct Answer Quinine

Q75 Blood Circulation was discovered

by

रकतपररसचरणकी mdashmdashndash दवारािह८जकीिी 17-Jan-2017

Options

1) Mary Anderson

F A C E B O O K

P A G E h t t p w w w f a c e b o o k c o m s s c m e n t o r s o f f i c i a l P a g e | 19

FOR MORE UPDATES AND MORE MATERIAL DO LIKE OUR FACEBOOK PAGE httpwwwfacebookcomsscmentorsofficial

महॴरीएिरसन

2) Virginia Apgar

वनजयननयाएपगार

3) William Harvey

पवलियमहाव

4) Robert Feulgen

रॉबटयफ़यिजहॳन Correct Answer William Harvey

Q76 Vitamin A is also known as

पवटालमन A कह८ mdashmdash- कहॳ नामसहॳभीजानाजाताहहॴ SSC CHSL Science (biology) 2016

Question Paper

18Jan2017

Options

1) Thiamine

िायलमन

2) Riboflavin

ररबह८फिहॳपवन

3) Retinol

रहॳटटनॉि

4) Calciferol

कहॴ नमसफहॳ रह८ि

Correct Answer Retinol

Q77 Some roots called arise from an

organ other than the radicle

कछजड़हॳनजनह mdashmdashmdash कहाजाताहहॴ वहमिकहॳ अिावाककसीअनयअगसहॳउतपननहह८तीहहॴ 18Jan2017

Options

1) tap roots

मखयजड़

2) stilt roots

ि ाजड़

3) fibrous roots

रहॳिहॳदारजड़

4) adventitious roots

आकनसमकजड़

Correct Answer adventitious roots

Q78 Spiders belong to which class of

animals

मकडड़यापराणीवगीकरणकहॳ ककसवगयमआतीहहॴ 18Jan2017

Options

1) Arachnids

एरहॳकननडस

2) Aves

एपवस

3) Gastropods

गहॴसटरोपह८िस

4) Anthozoa

एिह८जआ

Correct Answer Arachnids

Q79 How many layers does Human

Skin have

मानवतवचामककतनीपरतहॳहह८तीहहॴ

18Jan2017

Options

1) 5

2) 7

3) 11

4) 3

Correct Answer 3

Skin has three layers The epidermis

the outermost layer of skin provides a

waterproof barrier and creates our skin

tone The dermis beneath the

epidermis contains tough connective

tissue hair follicles and sweat glands

The deeper subcutaneous tissue (

hypodermis ) is made of fat and

connective tissue

Q80 Allium Cepa is the scientific name

of

एलियमलसपपा mdashmdashndash कावहॴजञाननकनामहहॴ 18Jan2017

F A C E B O O K

P A G E h t t p w w w f a c e b o o k c o m s s c m e n t o r s o f f i c i a l P a g e | 20

FOR MORE UPDATES AND MORE MATERIAL DO LIKE OUR FACEBOOK PAGE httpwwwfacebookcomsscmentorsofficial

Options

1) Carrot

गाजर

2) Tomato

टमाटर

3) Potato

आि 4) Onion

पयाज़

Correct Answer Onion

Q81 DNA stands for

िीएनएकापणय प mdashmdash- हहॴ 18Jan2017

Options

1) Di Nucleic Acid

िाईनयनकिकएलसि

2) Deoxy Nucleic Acid

िीओकसीनयनकिकएलसि

3) Diribonucleic Acid

िाईराइबह८नयनकिकएलसि

4) Deoxyribonucleic Acid

िीऑकसीराइबह८नयनकिकएलसि

Correct Answer Deoxyribonucleic Acid

Q82 Organisms that generate energy

using light are known as

जह८जीवाणपरकािकाउपयह८गकरउजायउतपननकरतीहहॴ उनह mdashmdash कहॳ पमजानाजाताहहॴ

18Jan2017

Options

1) Chaemolithotrophs

ककमह८लििह८टरह८पस

2) Oligotrophs

ओलिगह८टरह८पस

3) Bacteria

बहॴकटीररया 4)Photoautotrophs

फह८टह८ओटह८टरह८पस

Correct Answer Photoautotrophs

An oligotroph is an organism that can

live in an environment that offers very

low levels of nutrients

Q83 Which drug is used as an

Antidepressant

ककसदवाएकहतािारह८धीकहॳ पमपयोगककयाजाताहहॴ Options

1) Oxybutynin

ओकसीलयटीनन

2)Tramadol

टरहॳमहॳिह८ि

3 ) Sumatriptan

समहॳटरीपटहॳन

4) Bupropion

लयपरह८पपयह८न

Correct Answer Bupropion

लयपरह८पपयह८न

Q84 The orange colour of carrot is

because of

गाजरकानारगीरगनननननलिखितमसहॳककसीएककीवजहसहॳहह८ताहहॴ 18Jan2017

Options

1) it grows in the soil

यहलम ीमउगतीहहॴ 2) Carotene

कहॴ रह८टीन

3) it is not exposed to sunlight

यहसययपरकािकहॳ सपकय मनहीआती 4) the entire plant is oranqe in colour

सनपणयपह९धानारगीरगकाहह८ताहहॴ Correct Answer Carotene

Q85 Snake venom is highly modified

saliva containing

F A C E B O O K

P A G E h t t p w w w f a c e b o o k c o m s s c m e n t o r s o f f i c i a l P a g e | 21

FOR MORE UPDATES AND MORE MATERIAL DO LIKE OUR FACEBOOK PAGE httpwwwfacebookcomsscmentorsofficial

सापकाजहरअततयाचधकसिह८चधतिारहह८तीहहॴनजसमहॳ mdashmdash- हह८ताहहॴ Options

l)Prototoxins

परह८टह८टॉनकसस

2)Neutrotoxins

नयटरोटॉनकसस

3)Zootoxins

जटॉनकसस

4)Electrotoxins

इिहॳकटरह८टॉनकसस

Correct Answer Zootoxins

जटॉनकसस

Q86 Which type of pathogen causes the

water-borne disease Schistosomiasis

ककसपरकारकारह८गज़नकजिजननतरह८गलससटह८सह८लमलससकाकारणबनताहहॴ

18Jan2017

Option

1) Parasitic

परजीवी 2)Protozoan

परह८टह८जआ

3) Bacterial

बहॴकटीररयि

4) Viral

वायरि

Correct Answer Parasitic

Schistosomiasis also known as snail

fever and bilharzia is a disease caused

by parasitic

flatworms called schistosomes

Q87 Prothrombin responsible for

clotting of blood is released by

परह८िह८ननबन

जह८रकतकािककाजमनहॳकहॳ लिएनजनमहॳदारहहॴ mdashndash

कहॳ दवारासतरापवतककयाजाताहहॴ

19Jan2017

Options

1) Small Intestine

छह८टीआत

2) Blood Platelets

रकतपिहॳटिहॳटस

3) Large Intestine

बड़ीआत

4Heart

हदय

Correct Answer Blood Platelets

Q88 Acacia arabica is the scientific

name of

अकहॳ लियाअरहॳबबका mdashmdashndash कावहॴजञाननकनामहहॴ 19-Jan-2017

Options

1) Neem

नीम

2) Teak

सागह९न

3) Babhul

बबि

4) Pomegranate

अनार

Correct Answer Babhul

Q89 Cannis Vulpes is the scientific

name of

कहॴ ननसवनमपस mdashmdash- कावहॴजञाननकनामहहॴ 19-Jan-2017

Options

1) Dog

कतता 2) Wolf

भहॳडड़या 3) Fox

िह८मड़ी 4) Hyena

िाकिबगघा

F A C E B O O K

P A G E h t t p w w w f a c e b o o k c o m s s c m e n t o r s o f f i c i a l P a g e | 22

FOR MORE UPDATES AND MORE MATERIAL DO LIKE OUR FACEBOOK PAGE httpwwwfacebookcomsscmentorsofficial

Correct Answer Fox

Q90 The beetroot is the portion of the

beet plant

चकदरपह९धहॳका mdashmdashndash भागहहॴ 19-Jan-2017

Options

1) tap root

मखयजड़

2) Adventitious

आकनसमक

3) bulb of the stem

तनहॳकाकद

4) Rhizome

परकद

Correct Answer tap root

Q91 What is the basic unit of heredity

आनवलिकताकीबननयादीइकाईकयाहहॴ 19-Jan-2017

Options

1) DNA

िीएनए

2) RNA

आरएनए

3) Chromosome

िह८मह८सह८म

4) Gene

जीन

Correct Answer gene

Genes are the units of heredity and are

the instructions that make up the bodyrsquos

blueprint They code for the proteins

that determine virtually all of a personrsquos

characteristics Most genes come in

pairs and are made of strands of genetic

material called deoxyribonucleic acid

or DNA

Q92 Lungs are the primary organs of

फहॳ फड़हॳmdashndashकहॳ परािलमकअगहहॴ

19-Jan-2017

Options

1) Digestion

पाचन

2) Constipation

कलज

3) Perspiration

पसीना 4)Respiration

शवसन

Correct Answer Respiration

Q93 Sugarcane is a type of

गननाएकपरकारका mdash- हहॴ 20-Jan-2017

Options

1)creeper

िता 2)tree

पहॳड़

3)shrub

झाड़ी 4)grass

घास

Correct Answer grass

Q94 Who is commonly known as ldquothe

Father of Microbiologyrdquo

सामानयत ldquo सकषमजीवपवजञानकहॳ जनक lsquo

कहॳ नामसहॳककसहॳजानाजातहहॴ 20-Jan-2017

Options

1) Robert Hooke

रॉबटयहक

2) Antonie Philips van Leeuwenhoek

एटह८नीकफलिपवानमयएनहह८क

3) Carl Linnaeus

काियिीनाईयस

4) Charles Darwin

चामसयिापवयन

F A C E B O O K

P A G E h t t p w w w f a c e b o o k c o m s s c m e n t o r s o f f i c i a l P a g e | 23

FOR MORE UPDATES AND MORE MATERIAL DO LIKE OUR FACEBOOK PAGE httpwwwfacebookcomsscmentorsofficial

Correct Answer Antonie Philips van

Leeuwenhoek

Q95 For the aquatic organisms the

source of food is

जिीयजीवाणकािाघसरह८तहहॴ 20-Jan-2017

Options

1) Phytoplankton

फायटह८पिहॳकटन

2) Sea Weed

समदरीिहॴवाि

3)Aqua plankton

एकवापिहॳकटन

4) Zooplankton

जपिहॳकटन

Correct Answer Phytoplankton

Q96 Haemoglobin has the highest

affinity with which of the following

हीमह८गिह८बबनकीननननमसहॳककसकहॳ सािउततमसमानताहहॴ

20-Jan-2017

Options

1)SO2

2)CO2

3)CO

4)NO2

Correct Answer CO

It has a greater affinity for hemoglobin

than oxygen does It displaces oxygen

and quickly binds so very little oxygen

is transported through the body cells

Q97 Who developed the theory of

Evolution

उदपवकासकालसदातककसनहॳपवकलसतककया

20-Jan-2017

Options

1) Charles Darwin

चामसयिापवयन

2) Isaac Newton

आयजहॳकनयटन

3) Pranav Mistry

परणवलमसतरी 4) Galileo Galilei

गहॳलिलियह८गहॳिीिी Correct Answer Charles Darwin

Q98 The primary function of RNA is

RNA कापरािलमककाययहह८ताहहॴ 20-Jan-2017

Options

1) Photosynthesis

परकािसशिहॳषण

2) Protein Synthesis

परह८टीनसशिहॳषण

3) Replication

परनतकनतबनाना 4) Translation

अनवादकरना Correct Answer Protein Synthesis

There are two main functions of RNA

It assists DNA by serving as a messenger

to relay the proper genetic information

to countless numbers of ribosomes in

your body The other main function of

RNA is to select the correct amino acid

needed by each ribosome to build new

proteins for your body

Q99 ______is the movement of

molecules across a cell membrane from

a region of their lower concentration to

a region of their higher concertration

उचचसादरताकहॳ कषहॳतरसहॳउसकीकमसादरतावािहॳकषहॳतरकीतरफएककह८लिकाखझमिीकहॳ माधयमसहॳहह८नहॳवािाअणओकहॳ सचिनकह८ mdash- कहतहॳहहॴ Options

1) Diffusion

पवसरण

2) Osmosis

ऑसमह८लसस

F A C E B O O K

P A G E h t t p w w w f a c e b o o k c o m s s c m e n t o r s o f f i c i a l P a g e | 24

FOR MORE UPDATES AND MORE MATERIAL DO LIKE OUR FACEBOOK PAGE httpwwwfacebookcomsscmentorsofficial

3) Active Transport

सकियआवागमन

4) Passive Transport

नननषियआवागमन

Correct Answer Active Transport

Q100 Study of classification of

organisms is known as 20-Jan-2017

जीवाणओकहॳ वगीकरणकहॳ अधययनकह८ mdash-

कहाजाताहहॴ Options

1) Serpentology

सपरहॳटह८िह८जी 2) Virology

वायरह८िह८जी 3) Taxonomy

टहॴकसोनह८मी 4) Physiology

कफनज़यह८िह८जी Correct Answer Taxonomy

Q101 Photosynthesis takes place inside

plant cells in

परकािसशिहॳषणवनसपनतकह८लिकामनसति mdash

mdashmdash महह८ताहहॴ 20-Jan-2017

Options

1) Ribosomes

राइबह८सह८नस

2) Chloroplasts

किह८रह८पिासट

3) Nucleus

नयकलियम

4) Mitochondria

माईटह८कोडडरया Correct Answer Chloroplasts

Q102 ______ is the cell organelle in

which the biochemical processes of

respiration and energy production

occur

mdashmdash- वहकह८लिकाअगहहॴ नजसमहॳशवसनऔरउजायउतपादनकहॳ जहॴसीजहॴवरासायननकपरकियायहह८तीहहॴ 20-Jan-2017

Options

1) Mitochondria

माइटह८कोडडरया 2) Chloroplast

किह८रह८पिासट

3) Ribosomes

राइबह८सह८नस

4) Nucleus

नयकिीयस

Correct Answer Mitochondria

Q103 Which non-flowering spore

bearing plants have roots

ककसफिनिगनहॳवािहॳऔरबीजाणधारकपह९धह८कीजड़हॳहह८तीहहॴ 21-Jan-2017

Options

1) Mosses

मह८सहॳस

2) Angiosperms

एननजयह८सपनसय 3) Ferns

फनसय 4) Gymnosperms

नजननह८सपनसय Correct Answer ferns

Q104 Which of the following is an

excretory organ of cockroach

नननननलिखितमसहॳकह९नसानतिच हॳकाउतसजयनअगहहॴ

21-Jan-2017

Options

F A C E B O O K

P A G E h t t p w w w f a c e b o o k c o m s s c m e n t o r s o f f i c i a l P a g e | 25

FOR MORE UPDATES AND MORE MATERIAL DO LIKE OUR FACEBOOK PAGE httpwwwfacebookcomsscmentorsofficial

1) Malphigian Tubules

मनमफनजयनटयबमस

2) Nephridia

नहॳकफरडिया 3) Coxal Gland

कह८कसिगरचिया 4) Green Gland

गरीनगरचिया Correct Answer Malphigian Tubules

Q105 Evaporation of water takes place

in which part of plants

पानीकहॳ वाषपीकरणकीकियापह९धोकहॳ ककसभागसहॳहह८तीहहॴ 21-Jan-2017

Options

1) Stem

तना 2) Stomata

सटह८मटा 3) Branch

िािाए

4) Fruit

फि

Correct Answer Stomata

Evaporation accounts for the movement

of water to the air from sources such as

the soil canopy interception and

waterbodies Transpiration accounts for

the movement of water within a plant

and the subsequent loss of water as

vapour through stomata in its leaves

Q106 A is the fleshy spore-bearing

fruiting body of a fungus

mdashmdashndashकवककामासि

बीजाणधारणकरनहॳवािाफिनहॳवािाअगहहॴ 21-

Jan-2017

Options

1) aloe vera

एिह८वहॳरा 2) Coral

मगा 3) Cactus

कहॴ कटस

4) Mushroom

ककरमतता Correct Answer mushroom

Q107 Which of the following is a fungal

disease

नननननलिखितमसहॳकह९नसाफफदसहॳहह८नहॳवािाएकरह८ग हहॴ

21-Jan-2017

Options

1) Dermatitis

तवचािह८ध

2) Cholera

हहॴजा 3) Jaundice

पीलिया 4) Indigofera

इननिगह८फहॳ रा Correct Answer Dermatitis

Dermatitis also known as eczema is a

group of diseases that results in

inflammation of the skin These diseases

are characterized by itchiness red skin

and a rash In cases of short duration

there may be small blisters while in

long-term cases the skin may become

thickened

Q108 In which form is glucose stored in

our body

हमारहॳिरीरमगिकह८जकासचयककस पमककयाजाताहहॴ

21-Jan-2017

Options

1) Insulin

F A C E B O O K

P A G E h t t p w w w f a c e b o o k c o m s s c m e n t o r s o f f i c i a l P a g e | 26

FOR MORE UPDATES AND MORE MATERIAL DO LIKE OUR FACEBOOK PAGE httpwwwfacebookcomsscmentorsofficial

इसलिन

2) Glucose

गिकह८ज

3) Glycogen

गिायकह८जहॳन

4) Fat

वसा Correct Answer Glycogen

Excess glucose is stored in the liver as

the large compound called glycogen

Glycogen is a polysaccharide of glucose

but its structure allows it to pack

compactly so more of it can be stored in

cells for later use

Q109 Where do plants synthesize

protein from

पह९धहॳपरह८टीनसशिहॳषणकहासहॳकरतहॳहहॴ

Options

1) Fatty Acids

वसाऐलसि

2) Sugar

िकर

3) Amino Acids

एलमनह८ऐलसि

4) Starch

सटाचय Correct Answer Amino Acids

Q110 Which part of the brain is

responsible for triggering actions like

thinking intelligence memory and

ability to learn

मनसतषककाकह९नसाटहससासह८चनहॳ बनधदमानी याददाशतऔरसीिनहॳकीकषमताजहॴसीकियाओकह८परहॳररतकरताहहॴ 21-Jan-2017

Options

1) Diencephalon

िायएनसहॳफहॳ िह८न

2) Hypothalamus

हयपह८िहॳिहॳमस

3) Cerebrum

सहॳरहॳिम

4) Control

कटरह८ि

Correct Answer Cerebrum

Q111 Which of the following is also

known as the Biochemical Laboratory

of the Human Body

नननननलिखितमसहॳककसहॳमानविरीरकीजहॴवरसायनपरयह८गिािाभीकहाजाताहहॴ 21-Jan-2017

Options

1) Small Intestine

छह८टीआत

2)Brain

मनसतषक

3) Pancreas

अगनयािय

4) Liver

नजगर

Correct Answer Liver

The liver makes bile that will help

emulsify and digest the fats we eat

The liver takes toxic substances and

convert them using enzymes the liver

cells makes into a non toxic form so the

body can dispose of them

The liver also converts fats protein and

carbohydrates into glucose which is the

energy source for our cells to use

The liver takes amino acids and makes

proteins by combining them

Q112 The yellow colour of human urine

is due to

मानवमतरकापीिारग mdashndash कीवजहसहॳहह८ताहहॴ 22-

Jan-2017

Options

1) Bile Salts

F A C E B O O K

P A G E h t t p w w w f a c e b o o k c o m s s c m e n t o r s o f f i c i a l P a g e | 27

FOR MORE UPDATES AND MORE MATERIAL DO LIKE OUR FACEBOOK PAGE httpwwwfacebookcomsscmentorsofficial

पपततनमक

2) Cholesterol

कह८िहॳसटरह८ि

3) Lymph

लिनफ

4) Urochrome

यरह८िह८म

Correct Answer Urochrome

Urobilin or urochrome is the chemical

primarily responsible for the yellow

color of urine

Q113 The wilting of plants takes place

due to

पह९धह८कालिचििहह८नाकी mdashmdash- कीवजहसहॳहह८ताहहॴ 22-Jan-2017

Options

1)Photosynthesis

परकािसशिहॳषण

2) Transpiration

वाषपह८तसजयन

3) Absorption

अविह८षण

4) Respiration

शरवसन

Correct Answer Transpiration

Wilting is the loss of rigidity of non-

woody parts of plants This occurs when

the turgor pressure in non-lignified

plant cells falls towards zero as a result

of diminished water in the cells

Q114 Bovidae Ovis is the scientific name of

बह८पविीओपवस mdashndash कावहॴजञाननकनामहहॴ 22-Jan-2017

Options

1) Goat

बकरी 2) Cow

गाय

3) Buffalo

भहॳस

4) Sheep

भहॳड़

Correct Answer Sheep

Q115 Plants get their energy to produce

food from which of the following

पह८धहॳभह८जनकाननमायणकरनहॳकहॳ लिएनननननलिखितमसहॳककससहॳउजायपरापतकरतहॳहहॴ

22-Jan-2017

Options

1) Photosynthesis

परकािसशिहॳषण

2)Bacteria

बहॴकटीररया 3)Fungi

कवक

4)Sun

सयय Correct Answer Sun

Q116 Which of the following is secreted

by the liver

नननननलिखितमसहॳककसकासरावनजगरसहॳहह८ताहहॴ

22-Jan-2017

Options

1) Glucose

गिकह८ज

2) Iodine

आयह८िीन

3) Cortisol

काटटरयसह८ि

4) Bile

पपतत

Correct Answer Bile

The liver makes bile that will help

emulsify and

digest the fats we eat

F A C E B O O K

P A G E h t t p w w w f a c e b o o k c o m s s c m e n t o r s o f f i c i a l P a g e | 28

FOR MORE UPDATES AND MORE MATERIAL DO LIKE OUR FACEBOOK PAGE httpwwwfacebookcomsscmentorsofficial

Q117 Ferns belong to which division of

plants

फनसयपह९धह८कहॳ ककसभागमआतहॳहहॴ

22-Jan-2017

Options

1) Gymnosperms

नजननह८सपनसय 2) Angiosperms

एनजयह८सपनसय 3) Thallophyta

िहॴिह८फाईटा 4)Pteridophyta

टहॳररिह८फाईटा Correct Answer Pteridophyta

Q118 Who invented Antibiotics

एटीबायह८टटककाअपवषकारककसनहॳककयािा

22-Jan-2017

Options

1) Joseph Lister

जह८सहॳफलिसटर

2) William Harvey

पवलियमहाव

3) Robert Knock

रॉबटयनॉक

4)Alexander Fleming

अिहॳकज़िरफिहॳलमग

Correct Answer Alexander Fleming

Q119 Milbecycin is used in the

eradication of

लममबहॳसायलसनका mdashndash

मउनमिनमपरयह८गककयाजाताहहॴ 22-Jan-2017

Options

1) Agricultural Fungus

कपषकवक

2) Agricultural Pests

कपषकीटक

3) Agricultural Herbs

कपषिाक

4)Agricultural Weeds

कपषननराना Correct Answer Agricultural Pests

Milbemycin oxime is a veterinary drug

from the group of milbemycins used as

a broad spectrum antiparasitic It is

active against worms and mites(insects

Q120 Intestinal bacteria synthesizes

which of the following in the human

body

मानविरीरमआतोकहॳ बहॴकटीररयानननननलिखितमसहॳककसकासशिहॳषणकरतहॳहहॴ 22-Jan-2017

Options

1) Vitamin K

पवटालमन K

2) Proteins

परह८टीन

3) Fats

वसा 4) Vitamin D

पवटालमन D

Correct Answer Vitamin K

Q121 is the study of the physical form

and external structure of plants

mdashmdash-

मपह९धह८काभहॴनतक पऔरबाहरीसरचनाकाआदयाककयाजाताहहॴ 22-Jan-2017

Options

1) Physiology

कफनजयह८िह८जी 2) Anatomy

िरीररचनापवजञान

3) Phytomorphology

फाईटह८मह८फह८िह८जी 4)Cytology

कह८लिकापवजञान

Correct Answer Phytomorphology

F A C E B O O K

P A G E h t t p w w w f a c e b o o k c o m s s c m e n t o r s o f f i c i a l P a g e | 29

FOR MORE UPDATES AND MORE MATERIAL DO LIKE OUR FACEBOOK PAGE httpwwwfacebookcomsscmentorsofficial

Q122 Which of the following is a

structural and functional unit of

kidneys

नननननलिखितमसहॳकह९नसीगदोकीसरचनातमकऔरकाययकरीईकाईहहॴ

22-Jan-2017

Options

1) Renette Cells

रहॳनहॳटकह८लिकाए

2) Flame Cells

फिहॳमकह८लिकाए

3) Nephrites

नहॳफ़राइटस

4)Nephrons

नहॳफरोस

Correct Answer Nephrons

Nephron functional unit of the kidney

the structure that actually produces

urine in the process of removing waste

and excess substances from the blood

There are about 1000000 nephrons in

each human kidney

Q123 Which of the following is the

largest part of the human brain

नननननलिखितमसहॳकह९नसामानवमनसतषककासबसहॳबड़ाटहससाहहॴ

23-Jan-2017

Options

1) Ribs

पसलियाा 2) Cerebrum

सहॳरहॳिम

3) Pons

पोस

4)Thalamus

िहॴिहॳमस

Correct Answer Cerebrum

The cerebrum is the largest part of the

human brain making up about two-

thirds of the brainrsquos mass It has two

hemispheres each of which has four

lobes frontal parietal temporal and

occipital

Q124 The auxiliary buds

सहायककालियाmdashndash 23-Jan-2017

Options

1) grow endogenously from the pericycle

पहॳरीसाईककिसहॳअनतजातयपवकलसतहह८ताहहॴ 2) arise endogenously from the main

growing point

मिवपदसहॳअनतजातयउठताहहॴ 3) is an embryonic shoot located in the

axil of a leaf

एकभरणिटहहॴजह८एकपततीकहॳ अकषपरनसतिहह८ताहहॴ 4)arise exogenously from the epidermis

एपपिलमयससहॳबटहजातयतरीकहॳ सहॳउठताहहॴ Correct Answer is an embryonic shoot

located in the axil of a leaf

Q125 Which of the following is a viral

disease

इनमहॳसहॳकह९सीएकवायरिबीमारीहहॴ

23-Jan-2017

Options

1) Polio

पह८लियह८ 2) Tetanus

धनसतनभ

3) Leprosy

कषठरह८ग

4) Plague

पिहॳग

Correct Answer Polio

A viral disease (or viral infection)

occurs when an organismrsquos body is

invaded by pathogenic viruses and

infectious virus particles (virions) attach

to and enter susceptible cells

F A C E B O O K

P A G E h t t p w w w f a c e b o o k c o m s s c m e n t o r s o f f i c i a l P a g e | 30

FOR MORE UPDATES AND MORE MATERIAL DO LIKE OUR FACEBOOK PAGE httpwwwfacebookcomsscmentorsofficial

Poliomyelitis often called polio or

infantile paralysis is an infectious

disease caused by the poliovirus

Tetanusmdash A serious bacterial infection

that causes painful muscle spasms and

can lead to death

Leprosy also known as Hansenrsquos

disease (HD) is a long-term infection by

the bacterium Mycobacterium leprae or

Mycobacterium lepromatosis

Plague is an infectious disease caused by

the bacterium Yersinia pestis

Symptoms include fever weakness and

headache

Q126 Which organisms can help to

carry out Vermicomposting

कह९नसाजीववमीकनपह८नसटगममददकरताहहॴ

23-Jan-2017

Options

1) Nitrifying Bacteria

नाईटरीफाईगबहॴकटीररया 2) Earthworms

कहॴ चऐ

3) Algae

िहॴवि

4) Fungus

कवक

Correct Answer Earthworms

Q127 Contraction of heart is also

known as

हदयकहॳ सकचनकह८ mdash- भीकहाजाताहहॴ 23-Jan-

2017

Options

1) Systole

लससटह८ि

2) Aristotle

अरसत

3) Diastole

िायसटह८ि

4) Lub

मयब

Correct Answer Systole

Diastole is the part of the cardiac cycle

when the heart refills with blood

following systole (contraction)

Ventricular diastole is the period during

which the ventricles are filling and

relaxing while atrial diastole is the

period during which the atria are

relaxing

Q128 Azadirachta indica is the

botanical name of which of the

following

अजाटदराचताइडिकानननननलिखितमसहॳककसकावानसपनतनामहहॴ

23-Jan-2017

Options

1) Rose plant

गिाबकापह९धा 2) Apple tree

सहॳबकापहॳड़

3) Neem

नीम

4)Mango

आम

Correct Answer Neem

Q129 Which of the following is the

main end product of carbohydrate

digestion

नननननलिखितमसहॳकह९नसाकाबोहाइडरहॳटकहॳ पाचनकापरमिअतउतपादकहह८ताहहॴ 23-Jan-2017

Options

1) Fats

वसा 2) Lipids

लिपपडस

3) Glucose

गिकह८ज

4) Cellulose

F A C E B O O K

P A G E h t t p w w w f a c e b o o k c o m s s c m e n t o r s o f f i c i a l P a g e | 31

FOR MORE UPDATES AND MORE MATERIAL DO LIKE OUR FACEBOOK PAGE httpwwwfacebookcomsscmentorsofficial

सहॳमयिह८ज

Correct Answer Glucose

Intestinal absorption of end products

from digestion of carbohydrates and

proteins in the pig hellip During absorption some sugars (fructose or

galactose) released from the

corresponding sucrose and lactose

respectively during digestion were

partly metabolized into glucose by the

enterocyte

Q130 Which of the following glands is a

source of the enzyme Ptyalin

नननननलिखितगरचियोमसहॳएजाइमटयालिनकासरह८तहहॴ 23-Jan-2017

Options

1) Pancreas

अगरािय

2) Thyroid Gland

िाइराइिगरिी 3) Pituitary Gland

पीयषगरिी 4) Salivary Glands

िारगरचियाा Correct Answer Salivary Glands

Q131 Which of the following is not true

about Pteridophyta

ननननमसहॳकह९नसीबातटहॳररिह८फाईटकहॳ बारहॳमसचनहीहहॴ 23-Jan-2017

Options

1) Dominant phase is saprophytes

परमिचरणसहॳपरह८फाईइटसहह८ताहहॴ 2) Main plant body is diploid

पह९दह८कामखयिरीरदपवगखणतहह८ताहहॴ 3) Seeds are present

बीजमह९जदहह८तहॳहहॴ 4)Flowers are absent

फिअनपनसतिहह८तहॳहहॴ

Correct Answer Seeds are present

Q132 The largest dolphin species is the

orca also called as

िॉिकफनकीसबसहॳबड़ीपरजानतकाकानामआकायहहॴनजसहॳ mdash- भीकहतहॳहहॴ 23-Jan-2017

Options

1) Bottle Nose

बाटिनह८ज

2) Baiji

बहॳजी 3) Killer whale

ककिरहहॳि

4)Tucuxi

टकवसी Correct Answer Killer whale

Q133 The fat digesting enzyme Lipase

is secreted by which of the following

वसाकापाचनकरनहॳवािाएजाइमिाइपहॳजनननननलिखितमसहॳककसकहॳ दवारासतरापवतहह८ताहहॴ

24-Jan-2017

Options

1) Kidneys

गद

2) Pancreas

अगनयािय

3) Large Intestine

बड़ीआत

4)Liver

नजगर

Correct Answer Pancreas

Lipase is an enzyme that splits fats so

the intestines can absorb them Lipase

hydrolyzes fats like triglycerides into

their component fatty acid and glycerol

molecules It is found in the blood

gastric juices pancreatic secretions

intestinal juices and adipose tissues

F A C E B O O K

P A G E h t t p w w w f a c e b o o k c o m s s c m e n t o r s o f f i c i a l P a g e | 32

FOR MORE UPDATES AND MORE MATERIAL DO LIKE OUR FACEBOOK PAGE httpwwwfacebookcomsscmentorsofficial

Q134 The arrangement of leaves on an

axis or stem is called

एकअकषयातनहॳपरपनततयोकीयवसिाकह८कयाकहाजाताहहॴ SSC CHSL Science (biology) 2016

Question Paper

24-Jan-2017

Options

1) Phyllotaxy

फाइिह८टहॴकसी 2) Vernation

वनिन

3) Venation

वहॳनहॳिन

4)Phytotaxy

फाइटह८टहॴकसी Correct Answer Phyllotaxy

In botany phyllotaxis or phyllotaxy is

the arrangement of leaves on a plant

stem (from Ancient Greek phyacutellon

ldquoleafrdquo and taacutexis ldquoarrangementrdquo)

Phyllotactic spirals form a distinctive

class of patterns in nature

Q135 The study of Cells is also known

as

कह८लिकाओकहॳ अधययनकह८ mdashmdashndash

भीकहाजाताहहॴ 24-Jan-2017

Options

1) Cytology

सायटह८िह८जी 2) Physiology

कफनजयह८िह८जी 3) Nucleology

नयककमयह८िह८जी 4)Cellology

सहॳिह८िह८जी Correct Answer Cytology

Q136 Which of the following scientists

is also known as the Father of Biology

नननननलिखितमसहॳककसवहॴजञाननककह८ ldquoजीवपवजञानकहॳ जनकrdquoकहॳ नामसहॳभीजानाजाताहहॴ 24-Jan-2017

Options

1) Herbert Spencer

हबयटयसपसर

2) Aristotle

अरसत 3) Lamarck

िहॳमाकय 4)Darwin

िापवयन

Correct Answer Aristotle

Q137 Which cells give rise to various

organs of the plant and keep the plant

growing

कह९नसीकह८लिकाएपह९धह८कहॳ लभननअगह८कह८जनमदहॳतीहहॴऔरपह९धह८कह८बढ़नहॳममददकरतीहहॴ

24-Jan-2017

Options

1) Permanent

सिायी 2) Dermal

तवचीय

3) Meristematic

मररसटहॳमटटक

4)Mature

परह८ढ़

Correct Answer Meristematic

A meristem is the tissue in most plants

containing undifferentiated cells

(meristematic cells) found in zones of

the plant where growth can take place

Q138 Rodentia Muridae is the scientific

name of

F A C E B O O K

P A G E h t t p w w w f a c e b o o k c o m s s c m e n t o r s o f f i c i a l P a g e | 33

FOR MORE UPDATES AND MORE MATERIAL DO LIKE OUR FACEBOOK PAGE httpwwwfacebookcomsscmentorsofficial

रह८िहॳलियानयररिी mdashmdash- कावहॴजञाननकनामहहॴ 24-

Jan-2017

Options

1) Mouse

चहा 2) Squirrel

चगिहरी 3) Monkey

बदर

4) Lizard

नछपकिी Correct Answer Mouse

Q139 Name the scientist who proposed

the cell theory

कह८लिकालसदातकापरसतावदहॳनहॳवािहॳवहॴजञाननककानामबताइए 24-Jan-2017

Options

1) Schleiden and Schwann

िीमिनऔरशरववान

2) Lamarck

िहॳमाकय 3) Treviranus

टरहॳवायरहॳनस

4)Whittaker and Stanley

हीटकरऔरसटहॳनिहॳ Correct Answer Schleiden and

Schwann

Q140 The flower with the worldrsquos

largest bloom is

दननयाकासबसहॳबड़ाफिखििनहॳवािा mdashmdashndash हहॴ 24-Jan-2017

Options

1) Pando

पािह८ 2) Posidonia

पह८सीिह८ननया 3) Rafflesia arnoldii

ररफिहॳलियाअनोमिी 4)Helianthus annuus

हहॳलिएनिसएनयअस

Correct Answer Rafflesia arnoldii

Rafflesia arnoldii is a species of

flowering plant in the parasitic genus

Rafflesia It is noted for producing the

largest individual flower on earth It has

a very strong and horrible odour of

decaying flesh earning it the nickname

ldquocorpse flower

Q141 Deficiency of which vitamin

causes night blindness

ककसपवटालमनकीकमीकहॳ कारणरतौधीहह८ताहहॴ 24-Jan-2017

Options

1) Vitamin K

पवटालमन K

2) Vitamin C

पवटालमन C

3) Vitamin B1

पवटालमन B1

4)Vitamin A

पवटालमन A

Correct Answer Vitamin A

Q142 Nongreen plants lack which of the

following

गहॴर-

हररतवनसपनतमनननननलिखितमसहॳककसकीकमीहह८तीहहॴ

24-Jan-2017

Options

1) Chlorophyll

किह८रह८कफि

2) Lycophyll

िायकह८कफि

3) Cyanophyll

F A C E B O O K

P A G E h t t p w w w f a c e b o o k c o m s s c m e n t o r s o f f i c i a l P a g e | 34

FOR MORE UPDATES AND MORE MATERIAL DO LIKE OUR FACEBOOK PAGE httpwwwfacebookcomsscmentorsofficial

सायनह८कफि

4)Phototropism

फह८टह८टरोपपजम

Correct Answer Chlorophyll

Q143 Organisms that use light to

prepare food are known as

जह८जीवपरकािकाउपयह८गकरभह८जनतहॴयारकरतहॳहहॴ उनह mdashmdash- कहॳ पमजानजाताहहॴ 24-Jan-2017

Options

1) Autotrophs

सवपह८षी 2) Heterotrophs

पवषमपह८षज

3) Omnivores

सवायहारी 4)Decomposers

पवघटनकरनहॳवािा Correct Answer Autotrophs

autotrophs often make their own food

by using sunlight carbon dioxide and

water to form sugars which they can use

for energy Some examples of

autotrophs include plants algae and

even some bacteria Autotrophs

(producer) are important because they

are a food source for heterotrophs

(consumers)

A heterotroph is an organism that

ingests or absorbs organic carbon

(rather than fix carbon from inorganic

sources such as carbon dioxide) in order

to be able to produce energy and

synthesize compounds to maintain its

life Ninety-five percent or more of all

types of living organisms are

heterotrophic including all animals and

fungi and some bacteria

Q144 Which of the following is a

primary function of haemoglobin

नननननलिखितमसहॳकह९नसाटहमह८गिह८बबनकाएकपरािलमककाययहहॴ

25-Jan-2017

Options

1) Utilization of energy

उजायकाउपयह८गकरना 2) Prevention of anaemia

रकतामपताहह८नहॳसहॳरह८कना 3) Destruction of bacteria

बहॴकटीररयाकापवनािकरना 4) To transport oxygen

ऑकसीजनकावहनकरना Correct Answer To transport oxygen

Q145 Vascular bundles are absent in

सवहनीबिि mdashmdash- मअनपनसतिरहतहॳहहॴ 25-Jan-2017

Options

1) Bryophyta

िायह८फाइटा 2) Pteridophyta

टहॳररिह८फाईटा 3) Gymnosperms

नजननह८सपमय 4) Angiosperms

एननजयह८सपहॳनसय Correct Answer Bryophyta

Q146 Sauria Lacertidae is the scientific

name of

सहॴररयािहॳसरटाईिी mdashmdashndash कावहॴजञाननकनामहहॴ 25-Jan-2017

Options

1) Crocodile

मगरमचछ

2) Hippopotamus

टहपपह८पह८टहॳमस

3) Lizard

नछपकिी 4) House fly

F A C E B O O K

P A G E h t t p w w w f a c e b o o k c o m s s c m e n t o r s o f f i c i a l P a g e | 35

FOR MORE UPDATES AND MORE MATERIAL DO LIKE OUR FACEBOOK PAGE httpwwwfacebookcomsscmentorsofficial

घरहॳिमकिी Correct Answer Lizard

Q147 Which type of pathogen causes

the water-borne disease SARS (Severe

Acute Respiratory Syndrome)

ककसपरकािकारह८गज़नकजिजननतबीमारीसासयकाकारणबनताहहॴ 25-Jan-2017

Options

1) Viral

वायरि

2) Parasitic

परजीवी 3) Protozoan

परह८टह८जअन

4) Bacterial

बहॴकटीररयि

Correct Answer Viral

Q148 Which of the following organs

produces the enzyme lipase

नननननलिखितमसहॳकह९नसाअगिायपहॳजएजाइमउतपननकरताहहॴ 25-Jan-2017

Options

1) Pancreas

अगनयािय

2) Large Intestine

बड़ीआत

3) Liver

नजगर

4) Small Intestine

छह८टीआत

Correct Answer Pancreas

Q149 A is a long internode forming the

basal part or the whole of a peduncle

एक mdashmdash- एकिबाइटरनह८िहहॴ जह८ननचिाटहससायासनपणयिठिबनताहहॴ 25-

Jan-2017

Options

1) Rhizome

परकद

2) Rachis

महॳ दि

3) floral axis

पषपअकष

4) Scape

भगदड़

Correct Answer scape

Q150 ndash Which of the following

organisms are considered to be both

Living and Non-living

नननननलिखितमसहॳकह९नसहॳजीवाणकह८जीपवतऔरअजीपवतमानाजाताहहॴ

25-Jan-2017

Options

1) Bacteria

बहॴकटीररया 2) Fungi

कवक

3) Algae

िहॴवाि

4)Virus

वायरस

Correct Answer Virus

They are considered to be living as they

possess a protein coat as a protective

covering DNA as the genetic material

etc

They are said to be non-living as they

can be crystallised and they survive for

billions of years They can tolerate high

temperatures freezing cold

temperatures ultra-violet radiations etc

Q151 Deficiency of fluorine causes

which of the following

फिह८ररनकीकमीकहॳ कारणनननननलिखितमसहॳकयाहह८ताहहॴ

F A C E B O O K

P A G E h t t p w w w f a c e b o o k c o m s s c m e n t o r s o f f i c i a l P a g e | 36

FOR MORE UPDATES AND MORE MATERIAL DO LIKE OUR FACEBOOK PAGE httpwwwfacebookcomsscmentorsofficial

27-Jan-2017

Options

1) Dental Caries

िटिकहॴ ररज

2) Scurvy

सकवरी 3) Anaemia

रकतामपता 4) Arthritis

गटठया Correct Answer Dental Caries

Q152 In a Punnett Square with the

cross AaBb x AaBb how many Aabb

genotypes would be created

पनहॳटसककायरमिह८स AaBb x AaBb कहॳ साि

ककतनहॳ Aabb जीनह८टाइपबनगहॳ 27-Jan-2017

Options

1) 1

2) 8

3) 2

4) 3

Correct Answer 2

Q153 Which of the following is the

Controlling Center of the Cell

नननननलिखित म सहॳ कह८लिकाका ननयतरण

क दर कह९न हहॴ

27-Jan-2017

Options

1) Nucleus

क दर

2) Plasma

पिाजमा 3) Lysosome

िायसह८सह८म

4) Chromosome

िह८मह८सह८म

Correct Answer Nucleus

The control centre of the cell is the

nucleus in eukaryotic cells The nucleus

contains genetic material in the form of

DNA

Q154 Myopia affects which of the

following organs

मायह८पपयानननननलिखितअगह८मसहॳककसहॳपरभापवतकरताहहॴ

25-Jan-2017

Options

1) Heart

हदय

2) Skin

तवचा 3) Eyes

आािहॳ 4)Mouth

मह

Correct Answer Eyes

Q155 Which of the following bears

flowers

नननननलिखितमसहॳकह९नफिधारणकरताहहॴ

25-Jan-2017

Options

1) Bryophyta

िायह८फाइटा 2) Pteridophyta

टहॳरीिह८फाईटा 3) Gymnosperms

नजननह८सपमय 4)Angiosperms

एननजयह८सपमय Correct Answer Angiosperms

Q156 Oxygenated blood flows out of the

heart through the

ऑकसीजनयकतरकत mdashmdashmdash

कहॳ माधयमसहॳहदयकहॳ बाहरबहताहहॴ 25-Jan-2017

F A C E B O O K

P A G E h t t p w w w f a c e b o o k c o m s s c m e n t o r s o f f i c i a l P a g e | 37

FOR MORE UPDATES AND MORE MATERIAL DO LIKE OUR FACEBOOK PAGE httpwwwfacebookcomsscmentorsofficial

Options

1) Aorta

महाधमनी 2) pulmonary artery

फहॳ फड़हॳकीधमनी 3) vena cava

वहॳनाकावा 4)Atrium

चह९क

Correct Answer aorta

Q157 Blood leaving the liver and

moving towards the

heart has a higher concentration of

नजगरसहॳननकिकरहदयकीतरफजानहॳवािहॳरकतम mdashmdashmdashmdash कीउचचसादरताहह८तीहहॴ 27-Jan-2017

Options

1) Lipids

लिपपडस

2) Urea

यररया 3) Bile Pigments

पपततकहॳ रगकरण

4) Carbon dioxide

काबयनिायऑकसाइि

Correct Answer Bile Pigments

Urea is nitrogen containing substance

which is produced in the liver in order

to deal with excess amino-acids in the

body As urea is produced it leaves the

liver in the blood stream and passes via

the circulatory system to all parts of the

body

Q158 Bulb is a modification of which

part of a plant

बमबएकपह९धहॳकहॳ ककसटहससहॳकाएक पातरणहह८ताहहॴ 27-Jan-2017

Options

1) The root

जड़

2) The stem

तना 3) The radicle

मिाकर

4)The fruit

फि

Correct Answer The stem

Q159 Which of the following carries

blood away from the heart to different

body parts

इनमहॳसहॳकह९नरकतकह८हदयसहॳिरीरकहॳ पवलभननअगह८तकिहॳजातीहहॴ

27-Jan-2017

Options

1) Arteries

धमननया 2) Nerves

तबतरहाए

3) Capillaries

कहॳ लिकाए

4)Veins

नसहॳ Correct Answer Arteries

Q160 The series of processes by which

nitrogen and its compounds are

interconverted in the environment and

in living organisms is called

27-Jan-2017

Options

1)Absorption of Nitrogen

2)Ammonification

3)Nitrogen Fixation

4)Nitrogen Cycle

Correct Answer Nitrogen Cycle

Ammonification or Mineralization is

performed by bacteria to convert

organic nitrogen to ammonia

F A C E B O O K

P A G E h t t p w w w f a c e b o o k c o m s s c m e n t o r s o f f i c i a l P a g e | 38

FOR MORE UPDATES AND MORE MATERIAL DO LIKE OUR FACEBOOK PAGE httpwwwfacebookcomsscmentorsofficial

Nitrification can then occur to convert

the ammonium to nitrite and nitrate

Nitrogen fixation is a process by which

nitrogen in the Earthrsquos atmosphere is

converted into ammonia (NH3) or other

molecules available to living organisms

Q161 BCG vaccine is given to protect

from which of the following

बीसीजीकाटटकानननननलिखितमसहॳककसकहॳ बचावकहॳ लिएटदयाजातहहॴ

27-Jan-2017

Options

1) Jaundice

पीलिया 2) Anaemia

रकतमपता 3) Tuberculosis

कषयरह८ग

4) Polio

पह८लियह८ Correct Answer Tuberculosis

Q162 Parallel venation is found in

समानतरवहॳनहॳिन mdashmdashmdash- मपायाजाताहहॴ 27-Jan-2017

Options

1) plants which are monocots

पह९धहॳजह८एकबीजपतरीहह८तहॳहहॴ 2) plants which have a dicot stem

वहॳपह९धहॳनजनकातनादपवदलियहह८ताहहॴ 3) plants with leaves similar to Tulsi

वहॳपह९धहॳनजनकीपनततयतिसीकीपनततयोकहॳ समानहह८तहॳहहॴ 4)plants with tap roots

टहॳप टवािहॳपह९धहॳ Correct Answer plants which are

monocots

Q163 The hardest part of the body is

िरीरकासबसहॳकठह८रभाग mdashndash हहॴ 27-Jan-2017

Options

1) Bones

हडडिय

2) Tooth Enamel

दातकहॳ इनहॳमि

3) Skull

िह८पड़ी 4) Spinal Cord

महॳ रजज

Correct Answer Tooth Enamel

Q164 Which type of pathogen causes

the waterborne disease E coli Infection

ककसपरकारकारह८गजननकजिजननतरह८गईकह८िाईसिमणकाकारणबनताहहॴ 27-Jan-2017

Options

1) Protozoan

परह८टह८जआ

2) Parasitic

परजीवी 3) Bacterial

बहॴकटीररयि

4)Viral

वायरि

Correct Answer Bacterial

Q165 The amount of blood filtered

together by both the kidneys in a 70 kg

adult male human in a minute is

70 की गरा वािहॳएकवयसकप षमएकलमनटमदह८नोगदकहॳदवाराएकसािचाबनीगयीरकतकीमातरहह८तीहहॴ 29-Jan-2017

Options

1) 1100 ml

1100 लमलि

2) 100 ml

F A C E B O O K

P A G E h t t p w w w f a c e b o o k c o m s s c m e n t o r s o f f i c i a l P a g e | 39

FOR MORE UPDATES AND MORE MATERIAL DO LIKE OUR FACEBOOK PAGE httpwwwfacebookcomsscmentorsofficial

100 लमलि

3) 1500 ml

1500 लमलि

4) 500 ml

500 लमलि

Correct Answer 1100 ml

Q166 Which feature of a plant helps to

distinguish a monocot from a dicot

पह९धहॳकीवहकह९नसीपविहॳषताहहॴजह८एकदपवदलियहॳऔरएकएकदिीयपह९धहॳसहॳभहॳदकरनहॳममददकरतीहहॴ 29-Jan-2017

Options

1) Pollination

परागम

2) Venation

वहॳनहॳिन

3) Vernation

वनिन

4) Aestivation

एसटीवहॳिहॳन

Correct Answer venation

Q167 The Mutation Theory was

proposed by

उतवररवतयनकालसदात mdashmdashndash

कहॳ दवरापरसतापवतककयाजाताहहॴ 29-Jan-2017

Options

1) Charles Lyell

चामसयलियहॳि

2) William Smith

पवलियमनसमि

3) Hugo De Vries

हयगह८िीराईस

4)Harrison Schmitt

हहॳरीसननसमट

Correct Answer Hugo De Vries

Q168 Which type of pathogen causes

the waterborne disease HepatitisA

ककसपरकारकहॳ रह८गजनकजिजननतरह८गहहॳपहॳटाइटटस-A काकारणबनताहहॴ

29-Jan-2017

Options

1) Parasitic

परजीवी 2) Viral

वायरि

3) Protozoan

परह८टह८जआ

4) Bacterial

बहॴकटीररयि

Correct Answer Viral

Q169 In a Punnett Square with the

cross AaBb x Aabb how many AaBb

genotypes would be created

पनहॳटसकवायरमिह८स AaBb x Aabb

कहॳ सािककतनहॳ AaBb जीनह८टाइपबनगहॳ 29-Jan-

2017

Options

1) 4

2) 1

3) 7

4) 6

Correct Answer 4

Q170 Arboreal Ateles is the scientific

name of

अिह८ररयिएटटलिस mdashmdashmdash कावहॴजञाननकनामहहॴ 29-Jan-2017

Options

1) Squirrel

चगिहरी 2) Sparrow

गह८रहॴया 3) Lizard

नछपकिी 4) Spider monkey

F A C E B O O K

P A G E h t t p w w w f a c e b o o k c o m s s c m e n t o r s o f f i c i a l P a g e | 40

FOR MORE UPDATES AND MORE MATERIAL DO LIKE OUR FACEBOOK PAGE httpwwwfacebookcomsscmentorsofficial

मकड़ीबदर

Correct Answer Spider monkey

Q171 Which type of pathogen causes

the waterborne disease Salmonellosis

ककसपरकारकारह८गाणजिजननतबीमारीसािमह८नहॳिह८लसज़काकारकहहॴ

29-Jan-2017

Options

1) Algal

िहॳवालियहॳ 2) Parasitic

परजीवी 3) Bacterial

बहॴकटीररयि

4)Viral

वायरि

Correct Answer Bacterial

An infection with salmonella bacteria

commonly caused by contaminated food

or water

Symptoms include diarrhoea fever

chills and abdominal pain

Q172 is a condition in which there is a

deficiency of red cells or of haemoglobin

in the blood

mdashmdash-

एकनसिनतहहॴनजसमहॳरकतमिािकह८लिकाओकीयाहीमह८गिह८बबनकीकमीहह८तीहहॴ 29-Jan-2017

Options

1) Albinism

एनमबननजम

2) Propyria

परह८पीररया 3) Anaemia

एनीलमया 4)Keloid disorder

कहॳ िह८इिडिसओिर

Correct Answer Anaemia

Q173 Ananas comosus is the scientific

name of

Options

अनानासकह८मह८सस mdashmdashmdashndash

कावहॴजञाननकनामहहॴ 29-Jan-2017

1) Custard Apple

सीताफि

2) Pineapple

पाइनएपपि

3) Bamboo

बास

4)Pomegranate

अनार

Correct Answer Pineapple

Q174 Which organ produces insulin

कह९नसाअगइनसलिनपहॴदाकरताहहॴ 29-Jan-

2017

Options

1) Liver

यकत

2) Thyroid gland

िायराइिगरिी 3) Spleen

पिीहा 4)Pancreas

अगरयिय

Correct Answer Pancreas

Q175 Which of the following disease is

not caused by water pollution

नननननलिखितमसहॳकह९नसारह८गपानीकहॳ परदषणकहॳकारणनहीहह८ता

29-Jan-2017

Options

1) Cholera

हहॴजा 2) Typhoid

F A C E B O O K

P A G E h t t p w w w f a c e b o o k c o m s s c m e n t o r s o f f i c i a l P a g e | 41

FOR MORE UPDATES AND MORE MATERIAL DO LIKE OUR FACEBOOK PAGE httpwwwfacebookcomsscmentorsofficial

टाइफाइि

3) Asthma

दमा 4)Diarrhoea

दसत

Correct Answer Asthma

Q176 Ocimum tenuiflorum is the

scientific name of

ओलिलममटहॳयईफिह८रमइसकावहॴजञाननकनाम mdash

ndash हहॴ 30-Jan-2017

Options

1) Neem

नीम

2) Mango

आम

3) Babul

बबि

4)Tulsi

तिसी Correct Answer Tulsi

Q177 Which gland secretes bile a

digestive fluid

कह९नसीगरिीपपतत एकपाचनतरिपरदािय सरापवतकरतीहहॴ 30-Jan-2017

Options

1) Pancreas

अगनयािय

2) Liver

यकत

3) Thyroid

िायराइि

4) Testes

टहॳनसटस

Correct Answer liver

Q178 In which of the following the

dominant phase is Gametophyte

नननननलिखितमसहॳककसकहॳ परमिचरणयगमकह८दपवधद (Gametophyte)हहॴ 30-Jan-2017

Options

1) Bryophyta

िायह८फाइटा 2) Pteridophyta

टहॳररिह८फाइटा 3) Gymnosperms

नजननह८सपमय 4) Angiosperms

एननजयह८सपमय Correct Answer Bryophyta

Q179 Anaerobic respiration refers to

which of the following

नननननलिखितमसहॳककसहॳअवायवीयशवसनकहाजाताहहॴ

30-Jan-2017

Options

1) Respiration without Oxygen

ऑकसीजनकहॳ बबनाशवसन

2) Respiration with Oxygen

ऑकसीजनकहॳ सािशवसन

3) Respiration without CO2

काबयनिायऑकसाइिकहॳ बबनाशवसन

4) Respiration with CO2

काबयनिायऑकसाइिकहॳ सािशविन

Correct Answer Respiration without

Oxygen

Q180 Which type of pathogen causes

the waterborne disease Cholera

ककसपरकारकारह८गजनकजिजननतरह८गहहॴजाकाकारणबनताहहॴ

30-Jan-2017

Options

1) Algal

िहॴवालियहॳ

F A C E B O O K

P A G E h t t p w w w f a c e b o o k c o m s s c m e n t o r s o f f i c i a l P a g e | 42

FOR MORE UPDATES AND MORE MATERIAL DO LIKE OUR FACEBOOK PAGE httpwwwfacebookcomsscmentorsofficial

2) Bacterial

बहॴकटीररयि

3) Protozoan

परह८टह८जआ

4) Viral

वायरि

Correct Answer Bacterial

Q181 To which class does

Oxyreductases transferases hydrolases

belong

ओकसीररिकटहॳसटरासफरहॳजहॳस

हाइडरह८िहॳसहॳसककसवगयमआतहॳहहॴ 30-Jan-2017

Options

1) Hormones

हारमोस

2) Enzymes

एजाइनस

3) Proteins

परह८टीनस

4) Vitamins

पवटालमनस

Correct Answer Enzymes

Q182 Which of the following is not true

about Gymnosperms

ननननमसहॳकह९नसीबातअनावतबीजीकहॳ बारहॳमसचनहीहहॴ 30-Jan-2017

Options

1) Dominant phase is saprophytes

परमिचरणसहॳपरह८फाइटसहह८ताहहॴ 2) Vascular bundles are absent

सवहनीबििअनपनसितहह८ताहहॴ 3) spores are heterospores

बीजाणहहॳटहॳरह८सपह८रसहह८तहॳहहॴ 4) Flowers are absent

फिअनपनसितहह८तहॳहहॴ

Correct Answer Vascular bundles are

absent

Q183 The name of first mammal clone sheep is

भहॳड़कीपरिमसतनपायीपरनत प (किह८न)

कानामहहॴ 30-Jan-2017

Options

1) Noori

नरी 2) Dolly

िॉिी 3) Louise

िसी 4)Durga

दगाय Correct Answer Dolly

Q184 Which type of pathogen causes

the water-borne disease Typhoid fever

ककसपरकारकारह८गजनकजिजननतरह८गटाइफाइिबिारकाकारणबनताहहॴ 30-Jan-2017

Options

1) Algal

िहॴवािीय

2) Parasitic

परजीवी 3) Protozoan

परह८टह८जनअन

4)Bacterial

बहॴकटीररयि

Correct Answer Bacterial

Q185 In which part of the cell are

proteins made

कह८लिकाकहॳ ककसटहससहॳमपरह८टीनबनायाजाताहहॴ

31-Jan-2017

Options

1) Reticulum

रहॳटटकिम

F A C E B O O K

P A G E h t t p w w w f a c e b o o k c o m s s c m e n t o r s o f f i c i a l P a g e | 43

FOR MORE UPDATES AND MORE MATERIAL DO LIKE OUR FACEBOOK PAGE httpwwwfacebookcomsscmentorsofficial

2) Golgi apparatus

गह८मजीएपहॳरहॳटस

3) Ribosomes

ररबह८सह८नस

4) Lysosome

िायसह८सह८नस

Correct Answer ribosomes

Proteins are produced by stringing

amino acids together in the order

specified by messenger RNA strands

that were transcribed from DNA in the

cell nucleus The process of synthesizing

a protein is called translation and it

occurs on ribosomes in the cytoplasm of

a cell

Q186 Polio is a disease caused by which

of the following

नननननलिखितमसहॳपह८लियह८कीबबमारह८हह८नहॳकाकारणकयाहहॴ

31-Jan-2017

Options

1) Bacteria

बहॴकटीररयि

2) Mosquito

मचछर

3) Virus

वायरस

4) Cockroach

नतिच हॳ Correct Answer Virus

Polio or poliomyelitis is a crippling and

potentially deadly infectious disease It

is caused by the poliovirus

Q187 ndash Hay fever is a sign of which of

the following

हहॳकफवरनननननलिखितमसहॳककसकाएकसकहॳ तहहॴ

31-Jan-2017

Options

1) Old Age

वदावसिा 2) Malnutrition

कपह८सण

3) Allergy

एिनजय 4) Over Work

अतयचधककाययकरना Correct Answer Allergy

Q188 How many chromosomes does a

human cell contain

एकमानवकह८लिकामककतनहॳगणसतरहह८तहॳहहॴ

29-Jan-2017

Options

1) 6

2) 26

3) 46

4) 66

Correct Answer 46

In humans each cell normally contains

23 pairs of chromosomes for a total of

46 Twenty-two of these pairs called

autosomes look the same in both males

and females The 23rd pair the sex

chromosomes differ between males and

females

Q189 Which of the following is not true

about Bryophyta

ननननमसहॳकह९नसीबातिायह८फाइटकहॳ बारहॳमसचनहीहहॴ 31-Jan-2017

Options

1) Dominant phase is gametophytes

परमिचरणगहॳलमतह८फाइटसहह८ताहहॴ 2) Main plant body is haploid

पह९धहॳकामखयिरीरअगखणतहह८ताहहॴ 3) Spores are homospores

बीजाणहह८मह८सफह८रसहह८तहॳहहॴ 4) Flowers are present

फिमह८जदहह८तहॳहहॴ Correct Answer Flowers are present

F A C E B O O K

P A G E h t t p w w w f a c e b o o k c o m s s c m e n t o r s o f f i c i a l P a g e | 44

FOR MORE UPDATES AND MORE MATERIAL DO LIKE OUR FACEBOOK PAGE httpwwwfacebookcomsscmentorsofficial

Q190 Which aquatic animal has

trailing tentacles

ककसजिीयजानवरकहॳ पीछहॳचिनहॳवािहॳटहॳटकिसहह८तहॳहहॴ

31-Jan-2017

Options

1) Sea horse

समदरीघह८िा 2) Corals

मगा 3) Jelly fish

जहॳिीमछिी 4) Star fish

तारामछिी Correct Answer Jelly fish

Jellyfish with its umbrella-shaped bell

and trailing tentacles

Q191 Which type of pathogen causes

the water-borne disease Poliomyelitis

(Polio)

ककसपरकारकारह८गजनकजिजननतरह८गपह८लियह८मायहॳटटस (पह८लियह८) काकारणहहॴ 31-Jan-

2017

Options

1) Parasitic

परजीवी 2) Algal

िहॴवालिय

3) Viral

वायरि

4) Bacterial

बहॴकटीररयि

Correct Answer Viral

Q192 The outer white part of the eye

that protects the inner structures is

आािकाबाहरीसफहॳ दटहससाजह८आतररकसरचनाओकीरकषाकरताहहॴ वह mdashmdashmdash हहॴ 31-Jan-

2017

Options

1) Iris

आयररस

2) Sclera

सकिहॳरा 3) Retina

रहॳटटना 4) Cornea

कह८ननयया Correct Answer Sclera

Q193 Proteins are made up of

परह८टीनकाननमायण mdashndash सहॳहह८ताहहॴ 31-Jan-2017

Options

1) Amino acids

एलमनह८अनि

2) Fatty acids

वसायकतअनि

3) Glucose

गिकह८ज

4)Nucleotides

नयनकियह८टाईिस

Correct Answer Amino acids

Q194 Moringa Oleifera is the scientific

name of

मह८ररगओलिफहॳ रा mdashmdashndash कावहॴजञाननकनामहहॴ 31-Jan-2017

Options

1) Banyan

बरगद

2) Gulmohar

गिमह८हर

3) Amla

आमिा

F A C E B O O K

P A G E h t t p w w w f a c e b o o k c o m s s c m e n t o r s o f f i c i a l P a g e | 45

FOR MORE UPDATES AND MORE MATERIAL DO LIKE OUR FACEBOOK PAGE httpwwwfacebookcomsscmentorsofficial

4) Drumstick

डरमनसटक

Correct Answer Drumstick

Q195 Kidney stones are composed of

गदकीपिरी mdashndash सहॳबनीहह८तीहहॴ 1-Feb-2017

Options

1) Calcium Oxalate

कहॴ नमसयमओकजहॳिहॳट

2) Sodium Chloride

सह८डियमकिह८राइि

3) Magnesium Nitrate

महॳनगनलियमनाइतटरहॳट

4) Calcium Bicarbonate

कहॴ नमियमबायकबोनहॳट

Correct Answer Calcium Oxalate

Q196 ndash Which of the following is not

true about Angiosperms

ननननमसहॳकह९नसीबातआवतबीजीकहॳ बारहॳमसचनहीहहॴ 1-Feb-2017

Options

1) Dominant phase is gametophytes

परमिचरणगहॳलमतह८फाइटहह८ताहहॴ 2) Vascular bundles are present

सवहनीबििमह९जदहह८ताहहॴ 3) Spores are heterospores

बीजाणहहॳटहॳरह८सपह८रसहह८तहॳहहॴ 4) Seeds are covered

बीजढकहॳ हह८तहॳहहॴ Correct Answer Dominant phase is

gametophytes

Q197 All of the following are excretory

(waste) products of animals except

नननननलिखितमसहॳककसएककह८छह८ड़करअनयसभीपराखणयोदवाराउतसनजयतपदाियहहॴ 1-Feb-

2017

Options

1) Uric Acid

यररकएलसि

2) Ammonia

अमह८ननया 3) Carbohydrates

काबोहाइडरहॳट

4) Urea

यररया Correct Answer Carbohydrates

In animals the main excretory products

are carbon dioxide ammonia (in

ammoniotelics) urea (in ureotelics) uric

acid (in uricotelics) guanine (in

Arachnida) and creatine

Q198 RNA is a polymeric molecule

What does RNA stand for

आरएनइएएकबहिकआणहहॴ इसकाकापवय पकयाहहॴ 1-Feb-2017

Options

1) Rado Nuclear Acid

रािह८नयनकियरएलसि

2) Ribo Nucleic Acid

राइबह८नयनकिकएलसि

3) Rhino Nuclear Acid

हाइनह८नयनकियरएलसि

4) Resto Nucleus Acid

रहॳसटह८नयकिीयसएलसि

Correct Answer Ribo Nucleic Acid

Q199 Which organ does detoxification

and produces chemicals needed for

digestion

कह९नसाअगपवषहरणकरताहहॴऔरपाचनकहॳ लिएआवशयकरसायनोकह८पहॴदाकरताहहॴ 1-Feb-

2017

Options

1) Salivary glands

िारगरचिया 2) Pancreas

अगनयािय

F A C E B O O K

P A G E h t t p w w w f a c e b o o k c o m s s c m e n t o r s o f f i c i a l P a g e | 46

FOR MORE UPDATES AND MORE MATERIAL DO LIKE OUR FACEBOOK PAGE httpwwwfacebookcomsscmentorsofficial

3) Thyroid gland

िायराइिगरिी 4) Liver

यकत

Correct Answer Liver

Q200 Psidium guajava is the scientific

name of

लसडियमगआजावा mdashmdash कावहॴजञाननकनामहहॴ 1-

Feb-2017

Options

1) Guava

अम द

2) Mango

आम

3) Bamboo

बास

4) Jack fruit

कटहि

Correct Answer Guava

Q201 Which drug is used as a Blood

Thinner

चधरकह८पतिाकरनहॳकहॳ पमककसदवाकापरयह८गककयाजाताहहॴ

1-Feb-2017

Options

1) Warfarin

वाफर न

2) Tramadol

टरहॳमािह८ि

3) Azithromycin

एनजरह८मायलसन

4) Hydralazine

हाइडरह८िहॳनजन

Correct Answer Warfarin

Q202 Which of the following disease is

caused due to the deficiency of protein

परह८टीनकीकमीकहॳ कारणनननननलिखितमसहॳकह९नसारह८गहह८ताहहॴ 1-Feb-2017

Options

1) Arthritis

गटठया 2) Kwashiorkor

कािीओकय र

3) Goitre

गाइटर

4) Night Blindness

रतह९चध

Correct Answer Kwashiorkor

Q203 A is species of plant that has

adapted to survive in an environment

with little liquid water

mdashmdashndashपह९धहॳकीएकऐसहॳऐसहॳपरजानतहहॴ नजसनहॳकमपानीवािहॳवातावरणमजीपवतरहनहॳकहॳलिएअनकिनहहॴ 1-Feb-2017

Options

1) Xerophyte

म दपवद

2) Hydrophyte

जिीयपादप

3) Mesophyte

समह८दपवद

4) Thallophyte

िहॴिह८फाइटा Correct Answer xerophyte

xerophyte is a species of plant that has

adapted to survive in an environment

with little liquid water such as a desert

or an ice- or snow-covered region in the

Alps or the Arctic

Mesophytes are terrestrial plants which

are adapted to neither a particularly

dry nor particularly wet environment

An example of a mesophytic habitat

would be a rural temperate meadow

F A C E B O O K

P A G E h t t p w w w f a c e b o o k c o m s s c m e n t o r s o f f i c i a l P a g e | 47

FOR MORE UPDATES AND MORE MATERIAL DO LIKE OUR FACEBOOK PAGE httpwwwfacebookcomsscmentorsofficial

which might contain goldenrod clover

oxeye daisy and Rosa multiflora

thallophyte any of a group of plants or

plantlike organisms (such as algae and

fungi) that lack differentiated stems

leaves and roots and that were formerly

classified as a primary division

(Thallophyta) of the plant kingdom

Q204 How many types of teeth are

there in humans

मनषयोमककतनहॳपरकारकहॳ दातहह८तहॳहहॴ

1-Feb-2017

Options

1) 4

2) 5

3) 2

4) 3

Correct Answer 4

teeth -Humans have four types of

teethincisors canines premolars and

molars each with a specific function

The incisors cut the food the canines

tear the food and the molars and

premolars crush the food

Q205 Carica papaya is the scientific name of

कहॴ ररकापपाया mdashmdashndash कावहॴजञाननकनामहहॴ 2-

Feb-2017

Options

1) Peepal

पीपि

2) Papaya

पपीता 3) Tamarind

इमिी 4) Drumstick

ढह८िकाछड़ी Correct Answer Papaya

Q206 Muscles get tired when there is

shortfall of

जब mdashndash कीकमीहह८तीहहॴतबपहॳिीयिकजातीहहॴ 2-Feb-2017

Options

1) Lactic acid

िहॴनकटकएलसि

2) Na+ ions

Na+ आयन

3) ATP

एटीपी 4) Sulphates

समफहॳ टस

Correct Answer ATP

ATP is the energy source muscle fibers

use to make muscles contract

muscle tissuersquos main source of energy

called adenosine triphosphate or ATP

As your muscles use up this energy

source they become tired and fatigued

Oxygen is the key ingredient that helps

create new ATP to replenish the burned

up ATP in your muscles

Q207 Artocarpus integra is the

scientific name of आटह८कापयसइटीगरा mdashmdashmdash कावहॴजञाननकनामहहॴ 2-Feb-2017

Options

1) Guava

अम द

2) Pineapple

अनानास

3) Silver Oak

लसमवरओक

4) Jack fruit

कटहि

Correct Answer Jack fruit

Q208 Which organ stores fat soluble

vitamins

कह९नसाअगवसामघिनिीिपवटालमनह८काभिाराकरताहहॴ

2-Feb-2017

F A C E B O O K

P A G E h t t p w w w f a c e b o o k c o m s s c m e n t o r s o f f i c i a l P a g e | 48

FOR MORE UPDATES AND MORE MATERIAL DO LIKE OUR FACEBOOK PAGE httpwwwfacebookcomsscmentorsofficial

Options

1) Blood

रकत

2) Skin

तवचा 3) Liver

यकत

4) Pancreas

अगनयािय

Correct Answer Liver

Q209 Which disease is caused due to

deficiency of Iodine

आयह८िीनकहॳ कारणकह९नसारह८गहह८ताहहॴ 2-Feb-2017

Options

1) Rickets

ररकहॳ टस

2) Scurvy

सकवी 3) Goitre

गणमािा 4) Growth retardation

पवकासका कना Correct Answer Goitre

rickets A softening and weakening of

bones in children usually due to

inadequate vitamin D

Q210 Grevillea Robusta is the scientific name of

गरहॳपवलियारह८बसटा mdashmdashmdash- कापवजञाननकनामहहॴ 2-Feb-2017

Options

1) Peepal

पीपि

2) Teak

सागह९न

3) Silver Oak

लसमवरओक

4) Jack fruit

कटहि

Correct Answer Silver Oak

Q211 When a Cuttlefish is described as a Molluscs it is at which level of

classification

जबएककटिकफिकह८एकमह८िसकाकहॳ पमवखणयतककयाजाताहहॴतबयहॳवगीकरणकहॳ ककससतरपहॳनसितहहॴ 2-Feb-2017

Options

1) Class

वगय 2) Order

िम

3) Family

पररवार

4) Phylum

सघ

Correct Answer Phylum

Q212 Bambusa dendrocalmus is the

scientific name of बानबसािहॳडराकामस mdashmdashmdash कावहॴजञाननकनामहहॴ 3-Feb-2017

Options

1) Banyan

बरगद

2) Papaya

पपीता 3) Bamboo

बास

4) Pomegranate

अनार

Correct Answer Bamboo

Q213 Acinonyx Jubatus is the scientific name of

एलसनह८ननकसजयबहॳटस mdashmdashmdash

कावहॴजञाननकनामहहॴ 3-Feb-2017

F A C E B O O K

P A G E h t t p w w w f a c e b o o k c o m s s c m e n t o r s o f f i c i a l P a g e | 49

FOR MORE UPDATES AND MORE MATERIAL DO LIKE OUR FACEBOOK PAGE httpwwwfacebookcomsscmentorsofficial

Options

1) Bear

भाि 2) Horse

घह८िा 3) Cheetah

चीता 4) Zebra

जहॳिा Correct Answer Cheetah

Q214 The pale yellow colour of urine is

due to the presence of which pigment

मतरकाफीकापीिारगरगदरयकहॳ उपनसिनतकहॳ कारणहह८ताहहॴ

3-Feb-2017

Options

1) Urochrome

यरह८िह८म

2) Urophyll

यरह८कफि

3) Chlorophyll

किह८रह८कफि

4) Chloroplast

किह८रह८पिासट

Correct Answer Urochrome

Q215 Which of the following constitute

to form a gene

नननननलिखितमसहॳकह९नसीचीज़एकजीनकागठनकरतीहहॴ

3-Feb-2017

Options

1) Polynucleotides

पह८िीनयनकियह८टाईडस

2) Hydrocarbons

हाइडरह८काबोस

3) Lipoproteins

िाईपह८परह८टीनस

4) Lipids

लिपपडस

Correct Answer Polynucleotides

Polynucleotide molecule is a biopolymer

composed of 13 or more nucleotide

monomers covalently bonded in a chain

DNA (deoxyribonucleic acid) and RNA

(ribonucleic acid) are examples of

polynucleotides with distinct biological

function

Q216 Vertebrates belongs to the

phylum

रीढ़कीहडिीवािहॳपराणी mdashmdashmdash

परजानतकहॳ अतगायतआतहॳहहॴ 3-Feb-2017

Options

1) Arthropoda

आरह८पह८ड़ा 2) Annelida

एननलििा 3) Cnidaria

ननिहॳररया 4) Chordata

कह८िटा Correct Answer Chordata

Q217 Punica granatum is the scientific name of

पननकगरहॳनहॳटस mdashmdashmdash कावहॴजञाननकनामहहॴ 3-Feb-2017

Options

1) Custard Apple

सीताफि

2) Gulmohar

गिमह८हर

3) Silver Oak

लसमवरओक

4) Pomegranate

अनार

Correct Answer Pomegranate

F A C E B O O K

P A G E h t t p w w w f a c e b o o k c o m s s c m e n t o r s o f f i c i a l P a g e | 50

FOR MORE UPDATES AND MORE MATERIAL DO LIKE OUR FACEBOOK PAGE httpwwwfacebookcomsscmentorsofficial

Q218 Between a tiger and an monkey

which of the following is different

एकबाघऔरबदरकहॳ बीचनननननलिखितमसहॳकह९नसीबातअिगहहॴ 3-Feb-2017

Options

1) Kingdom

राजय

2) Phylum

जानत

3) Order

िम

4) Class

वगय Correct Answer order

Q219 The artificial heart was invented by

कबतरमहदयका mdashmdashmdash

दवाराअपवषकारककयागयािा 3-Feb-2017

Options

1) Muhammad Yunus

महनमदयनस

2) Linus Yale Jr

िाइनसयहॳिजय

3) Gazi Yasargil

गाजीयासचगयि

4) Paul Winchell

पह९िपवमकि Correct Answer Paul Winchell

Q220 Tamarindus indica is the

scientific name of

टहॳमररनडसइडिका mdashmdash कावहॴजञाननकनामहहॴ 7-

Feb-2017

Options

1) Neem

नीम

2) Pineapple

अनानास

3) Tamarind

इमिी 4)Chiku

चीक

Correct Answer Tamarind

Q221 In eukaryotic cells synthesis of

RNA takes place in the

यकहॳ योटटककह८लिकाओमआरएनएकासशिहॳषण

mdashndash महह८ताहहॴ 7-Feb-2017

Options

1) Mitochondria

माईटह८कोडडरया 2) Centrioles

सटरीयह८मस

3) Ribosomes

ररबह८सह८नस

4) Nucleus

नयनकियस

Correct Answer nucleus

eukaryotic cell -Transcription is the

process of synthesizing ribonucleic acid

(RNA)Synthesis takes place within the

nucleus of eukaryotic cells or in the

cytoplasm of prokaryotes and converts

the genetic code from a gene in

deoxyribonucleic acid ( DNA ) to a

strand of RNA that then directs

proteinsynthesis

Q222 _________is caused by parasites

of the Plasmodium genus

पिाजमह८डियमजातीकहॳ परजीवी mdash- कहॳ कारणहहॴ 7-Feb-2017

Options

1) Dysentery

पहॳचचि

2) Malaria

मिहॳररया 3) Chickenpox

F A C E B O O K

P A G E h t t p w w w f a c e b o o k c o m s s c m e n t o r s o f f i c i a l P a g e | 51

FOR MORE UPDATES AND MORE MATERIAL DO LIKE OUR FACEBOOK PAGE httpwwwfacebookcomsscmentorsofficial

चहॳचक

4) Herpes

हहॳपपयस

Correct Answer Malaria

Q223 Carotene in fruits and vegetables

gives it which color

फिह८औरसनलजयोमनसितकहॳ रह८टीनउनहकह९नसारगपरदानकरताहहॴ 7-Feb-2017

Options

1) Green

हरा 2) Pink

गिाबी 3) Orange

नारगी 4) Blue

नीिा Correct Answer Orange

Q224 Equus Caballus is the scientific

name of

एकवसकहॴ बहॳिस mdashmdashndash कापवजञाननकनामहहॴ 7-Feb-2017

Options

1) Horse

घह८िा 2) Zebra

ज़हॳिा 3) Donkey

गधा 4) Buffalo

भस

Correct Answer Horse

Q225 Elapidae Naja is the scientific name of

एिीपीिीनाजा mdashmdash- कावहॴजञाननकनामहहॴ 8-Feb-2017

Options

1) Cobra

कह८बरा 2) Elephant

हािी 3) Eagle

ग ि

4) Owl

उमि Correct Answer Cobra

Q226 Which disease is caused due to

deficiency of Iron

िह८हकीकमीकहॳ कारणकह९नसारह८गहह८ताहहॴ 8-Feb-

2017

Options

1) Beriberi

बहॳरीबहॳरी 2) Tetany

टहॳटनी 3) Kwashiorkor

कवािीऔरकर

4) Anaemia

रकतामपता Correct Answer Anaemia

Beriberi is a disease caused by a vitamin

B-1 deficiency also known as thiamine

deficiency

Tetany can be the result of an

electrolyte imbalance Most often itrsquos a

dramatically low calcium level also

known as hypocalcemia Tetany can also

be caused by magnesium deficiency or

too little potassium Having too much

acid (acidosis) or too much alkali

(alkalosis) in the body can also result in

tetany

Kwashiorkor also known as

ldquoedematous malnutrition It is a form of

malnutrition caused by a lack of protein

in the diet

Anaemia means that you have fewer red

blood cells than normal or you have less

F A C E B O O K

P A G E h t t p w w w f a c e b o o k c o m s s c m e n t o r s o f f i c i a l P a g e | 52

FOR MORE UPDATES AND MORE MATERIAL DO LIKE OUR FACEBOOK PAGE httpwwwfacebookcomsscmentorsofficial

haemoglobin than normal in each red

blood cell

Q227 is a leaf where the leaflets are

arranged along the middle vein

mdashndashएकपततीहहॴजहापतरकह८कीरचनाक ररयालिराकहॳ आसपासहह८तीहहॴ 8-Feb-2017

Options

1) Pinnately compound leaf

पपनहॳटिीसयकतपतती 2) Palmately compound leaf

पामहॳटिीसयकतपतती 3) Compound leaf

सयकतपतती 4) Simple leaf

साधारणपतती Correct Answer Pinnately compound

leaf

Q228 Haustoria or sucking roots are

found in which of the following

हह८सटह८ररयायाचसनहॳवािीजड़हॳनननननलिखितमसहॳककसमपाईजातीहहॴ 8-Feb-2017

Options

1) Wheat

गहॳह

2) Mango

आम

3) Chestnut

चहॳसटनट

4) Cuscuta

कसकयटा Correct Answer Cuscuta

Haustorial roots -The roots of parasitic

plants which penetrate into the host

tissues to absorb nourishment are

called haustorial roots hellip Also known as suckingor parasitic roots

Q229 Equs Asinus is the scientific name

of

एकवसएलसनस mdashmdashndash कावहॴजञाननकनामहहॴ 8-

Feb-2017

Options

1) Donkey

गधा 2) Cow

गाय

3) Deer

टहरन

4) Kangaroo

कगा

Correct Answer Donkey

Q230 Ficus benghalensis is the scientific name of

फाईकसबहॳनगहॳिहॳलसस mdashndash कापवजञाननकनामहहॴ 8-Feb-2017

Options

1) Banyan

बरगद

2) Pineapple

अनानास

3) Babul

बबि

4) Tulsi

तिसी Correct Answer Banyan

Q231 Equus burchellii is the scientific name of

एकवसबचिी mdashmdash- कापवजञाननकनामहहॴ 8-Feb-2017

Options

1) Horse

घह८िा 2) Zebra

जहॳिा 3) Buffalo

F A C E B O O K

P A G E h t t p w w w f a c e b o o k c o m s s c m e n t o r s o f f i c i a l P a g e | 53

FOR MORE UPDATES AND MORE MATERIAL DO LIKE OUR FACEBOOK PAGE httpwwwfacebookcomsscmentorsofficial

भस

4) Ass

गधा Correct Answer Zebra

Page 8: COMPILATION OF ALL 72 SETS OF BIOLOGY SSC CHSL-2016 · OF BIOLOGY SSC CHSL-2016 PREPARED BY : SSC MENTORS BIOLOGY SPECIAL . F A C E B O O K P A G E : h t t p : / / w w w . f a c e

F A C E B O O K

P A G E h t t p w w w f a c e b o o k c o m s s c m e n t o r s o f f i c i a l P a g e | 7

FOR MORE UPDATES AND MORE MATERIAL DO LIKE OUR FACEBOOK PAGE httpwwwfacebookcomsscmentorsofficial

2) Teak

सागह९न

3) Silver Oak

लसमवरओक

4) Tulsi

तिसी Correct Answer Neem

Q24 Octopus belongs to the phylum

ऑकटह८पसककसपरजानतकहॳ अतगयतआताहहॴ 10-

Jan -2017

Options

1) Mollusca

मह८िसका 2) Cnidaria

ननिहॳररया 3) Echinodermata

इकाइनह८ड़हॳमता 4) Chordata

कह८िता Correct Answer Mollusca

Q25 A living part of the organisms

environment is known as

जीवाणकहॳ वातावरणकहॳ जीपवतभागकह८ mdash-

कहतहॳहहॴ 10-Jan -2017

Options

1) Abiotic Factor

अजहॴपवककारक

2) Habitat

आवास

3) Biotic Factor

जहॴपवककारक

4) Nonliving factor

अ-जीपवतकारक

Correct Answer Biotic Factor

Abiotic factors are nonndash living chemical

and physical parts of the environment

that affect living organisms and the

functioning of ecosystems like rain

wind temperature altitude soil

pollution nutrients pH types of soil

and sunlight

Q26 Medulla oblongata is a part of

which of the following

महॳडयिाऑबिॉनगहॳटानननननलिखितमसहॳककसअगकाटहससाहहॴ

10-Jan -2017

Options

1) Heart

हदय

2) Brain

मनसतषक

3) Lungs

फहॳ फड़हॳ 4) Stomach

पहॳट

Correct Answer Brain

The medulla oblongata helps regulate

breathing heart and blood vessel

function digestion sneezing and

swallowing This part of the brain is a

center for respiration and circulation

Sensory and motor neurons (nerve cells)

from the forebrain and midbrain travel

through the medulla

Q27 ___________ is a typically

onecelled reproductive unit capable of

giving rise to a new individual without

sexual fusion

mdashmdash एकआमतह९रपरएककह८लिकीयहॳ परजननममसमकषइकाईहहॴजह८यह९नसियनकहॳ बबनाएकनयीइकाईकह८जनमदहॳतीहहॴ 10-Jan -2017

Options

1) Egg

अिाण

2) Spore

बीजाण

F A C E B O O K

P A G E h t t p w w w f a c e b o o k c o m s s c m e n t o r s o f f i c i a l P a g e | 8

FOR MORE UPDATES AND MORE MATERIAL DO LIKE OUR FACEBOOK PAGE httpwwwfacebookcomsscmentorsofficial

3) Sperm

ििाण

4) Seed

बीज

Correct Answer Spore

Q28 Bacteria was discovered by

बहॴकटीररयाकीिह८जककसकहॳ दवाराकीगयीिी

10-Jan -2017

1) Antonie van Leeuwenhoek

एटह८नीवहॳनलिबहॳनहक

2) Belarus

बहॳिा स

3) Hugo de Vries

हयगह८दीराईस

4)Robert Brown

रॉबटयिाउन

Correct Answer Antonie van

Leeuwenhoek

Q29 Which of the following is

responsible for Vermicomposting

नननननलिखितमसहॳकह९नकलमिादकहॳ लिएनजनमहॳदारहहॴ

10-Jan -2017

Options

1) Fungus

कवक

2) Worms

कलम

3) Bacteria

बहॴकटीररया 4) Birds

पकषी Correct Answer Worms

Vermicompost (or vermi-compost) is the

product of the composting process using

various species of worms usually red

wigglers white worms and other

earthworms to create a heterogeneous

mixture of decomposing vegetable or

food waste bedding materials and

vermicast

Q30 Scurvy (bleeding of gums) is

caused by the deficiency of which

vitamin

सकवी (मसढह८सहॳिनआना) ककसपवटालमनकीकमीकहॳ कारणहह८ताहहॴ

10-Jan-2017

Options

1) Vitamin K

पवटालमन K

2) Vitamin BZ

पवटालमन BZ

3) Vitamin C

पवटालमन C

4) Vitamin A

पवटालमन A

Correct Answer Vitamin C

Q31 Achras sapota is the scientific

name of

एिाससपह८ताइसकावहॴजञाननकनामहहॴ 10-Jan-2017

Options

1) Custard Apple

सीताफि

2) Gulmohar

गिमह८हर

3) Tamarind

इमिी 4) Chiku

चचक

Correct Answer Chiku

Q32 Prawn belongs to the phylum

झीगा mdashmdash- परजानतकहॳ अतगयतआताहहॴ 10-Jan-2017

Options

1) Arthropoda

F A C E B O O K

P A G E h t t p w w w f a c e b o o k c o m s s c m e n t o r s o f f i c i a l P a g e | 9

FOR MORE UPDATES AND MORE MATERIAL DO LIKE OUR FACEBOOK PAGE httpwwwfacebookcomsscmentorsofficial

अरोपह८िा 2) Cnidaria

नीिहॳररया 3) Echinodermata

इकाईनह८िमटा 4) Chordata

कह८िटा Correct Answer Arthropoda

Q33 Pulses are a rich source of which of

the following

दािहॳनननननलिखितमसहॳककसकीपरचरसह८तरहहॴ

11-Jan-2017

Options

1) Carbohydrates

काबोहाइडराईट

2) Proteins

परह८टीनस

3) Minerals

िननज

4) Vitamin A

पवटालमन A

Correct Answer Proteins

Q34 Plant cell wall is made up of

वनसपनतकह८लिकालभनततइससहॳबनीहह८तीहहॴ

11-Jan-2017

Options

1) Cellulose

सहॳमयिह८ज

2) Glucose

गिकह८ज

3) Sucrose

सिह८ज

4) Fructose

फरकटह८ज

Correct Answer Cellulose

Plant cell wall the major carbohydrates

are cellulose hemicellulose and pectin

The cellulose microfibrils are linked via

hemicellulosic tethers to form the

cellulose-hemicellulose network which

is embedded in the pectin matrix

Q35 The study of Fungi is also known

as कवकह८कहॳ अधययनकह८कहाजाताहहॴ

11-Jan-2017

Options

1) Cytology

सायटह८िह८जी 2) Myology

मायह८िह८जी 3) Mycology

मायकह८िह८जी 4) Neurology

नयरह८िह८जी Correct Answer Mycology

Cytology - structure and function of

plant and animal cells

Myology is the study of the muscular

system

Neurology is the branch of medicine

concerned with the study and treatment

of disorders of the nervous system

Q36 The outermost layer of skin is

तवचाकीसबसहॳबाहरीपरतकयाहह८तीहहॴ 11-Jan-

2017

Options

1) Epidermis

इपपिलमयस

2) Dermis

िलमयस

3) Tissues

ऊतक

4) Hypodermis

हायपह८िलमयस

Correct Answer Epidermis

Q37 Which of the following plants have

root nodules

F A C E B O O K

P A G E h t t p w w w f a c e b o o k c o m s s c m e n t o r s o f f i c i a l P a g e | 10

FOR MORE UPDATES AND MORE MATERIAL DO LIKE OUR FACEBOOK PAGE httpwwwfacebookcomsscmentorsofficial

नननननलिखितपह९धह८मसहॳककसकीजड़ह८मगाठहह८तीहहॴ

11-Jan-2017

Options

1) Leguminous plants

िहॳगयलमनसपह९धहॳ 2) Parasitic plants

परजीवीपह९धहॳ 3) Epiphytic Plants

एपीफाइटटकपह९धहॳ 4) Aquatic Plants

जिीयपह९धहॳ Correct Answer Leguminous plants

Q38 Earth-worms belongs to the

phylum

कहॳ चएmdashmdash- परजानतकहॳ अतगयतआतहॳहहॴ 11-Jan-2017

Options

1) Protozoa

परह८टह८जआ

2) Cnidaria

नीिहॳररया 3) Annelida

एनीलििा 4) Mollusca

मह८िसका Correct Answer Annelida

Q39 Ringworm is a disease caused by

ररगवमयनामकबीमारी mdashmdash- कहॳ कारणहह८तीहहॴ 11-Jan-2017

Options

1) Fungi

कवक

2) Bacteria

बहॴकटीररया 3) Virus

वायरस

4) Flies

मनकियाा Correct Answer Fungi

Q40 Mangifera indica is the scientific

name of

मननगफहॳ राइडिकाककसकावहॴजञाननकनामहहॴ 11-

Jan-2017

Options

1) Guava

अम द

2) Mango

आम

3) Amla

आविा 4) Jack fruit

कटहि

Correct Answer Mango

Q41 Crabs belongs to the phylum

कहॳ कड़हॳmdashmdash- परजानतकहॳ अतगयतआतहॳहहॴ 11-Jan-2017

Options

1) Mollusca

मह८िसका 2) Cnidaria

नीिहॳररया 3) Arthropoda

अरोपह८ड़ा 4) Platyhelminthes

पिहॳटटहहॳनममननिस

Correct Answer Arthropoda

Q42 Myopia is a defect of eyes which is

also known as

मायह८पपयाआिोकादह८षहहॴ नजसहॳ mdashmdashndash

भीकहाजाताहहॴ

12-Jan-2017

Options

1) Far Sightedness

F A C E B O O K

P A G E h t t p w w w f a c e b o o k c o m s s c m e n t o r s o f f i c i a l P a g e | 11

FOR MORE UPDATES AND MORE MATERIAL DO LIKE OUR FACEBOOK PAGE httpwwwfacebookcomsscmentorsofficial

दरदनषटदह८ष

2) Near Sightedness

ननकटदनषटदह८ष

3) Astigmatism

एसटीगमहॳटटजम

4) Night Blindness

रतोधी Correct Answer Near Sightedness

Myopia occurs when the eyeball is too

long relative to the focusing power of

the cornea and lens of the eye This

causes light rays to focus at a point in

front of the retina rather than directly

on its surface

Hyperopia Hypermetropia (

Farsightedness )- when light rays

entering the eye focus behind the retina

rather than directly on it The eyeball of

a farsighted person is shorter than

normal

Astigmatism usually is caused by an

irregularly shaped cornea Instead of

the cornea having a symmetrically

round shape (like a baseball) it is

shaped more like an American football

Nyctalopia also called night ndash blindness

is a condition making it difficult or

impossible to see in relatively low light

Q43 Who is known as the father of

Green Revolution

हररतिानतकहॳ जनककहॳ पमककसहॳजानाजाताहहॴ

12-Jan-2017

1) Dr Robert Nucleus

िॉ रॉबटयनयनकियस

2) Dr Ian Wilmut

िॉ इयानपविमट

3) Dr NE Borlaug

िॉ एनईबह८रिॉग

4) Dr JC Bose

िॉ जहॳसीबह८स

Correct Answer Dr NE Borlaug

Q44 Panthera Tigris is the scientific

name of

पिहॳराटटगरीस mdashmdashmdash कावहॴजञाननकनामहहॴ 12-Jan-2017

Options

1) Panther

तदआ

2) Tiger

बाघ

3) Whale

हहॳि

4)Goat

बकरी Correct Answer Tiger

Q45 How many facial bones are there

हमारहॳचहॳहरहॳमककतनीहडडियााहह८तीहहॴ 13-Jan-2017

Options

1)34

2)24

3)14

4)4

Correct Answer 14

Q46 ndash Halophytes are plants that grow

in

हहॴिह८फाईटसवहॳपह९धहॳहह८तीहहॴजह८ mdash- मउगतहॳहहॴ SSC CHSL Science (biology) 2016

Question Paper

13-Jan-2017

Options

1) Fresh Water

ताजापानी 2) Cold Water

ठिापानी 3) Ponds

तािाब

4) Salt Water

िारापानी Correct Answer Salt Water

F A C E B O O K

P A G E h t t p w w w f a c e b o o k c o m s s c m e n t o r s o f f i c i a l P a g e | 12

FOR MORE UPDATES AND MORE MATERIAL DO LIKE OUR FACEBOOK PAGE httpwwwfacebookcomsscmentorsofficial

Q47 Felis Catus is the scientific name of

फहॳ लिसकहॴ टस mdashndash कावहॴजञाननकनामहहॴ 13-Jan-2017

Options

1) Cat

बबमिी 2) Dog

कतता 3) Mouse

चहा 4) Porcupine

साही Correct Answer Cat

Q48 Which of the following induces

nitrogen fixation in soil

नननननलिखितमसहॳकह९नलम ीमनाइटरह८जनननयतनकह८परहॳररतकरताहहॴ

15-Jan-2017

Options

1) Protozoa

परह८टह८जआ

2) Bacteria

बहॴकटीररया 3) Fungi

कवक

4)Algae

िहॴवाि

Correct Answer Bacteria

Bacteria that change nitrogen gas from

the atmosphere into solid nitrogen

usable by plants are called nitrogen-

fixing bacteria These bacteria are

found both in the soil and in symbiotic

relationships with plants

They contain symbiotic bacteria called

rhizobia within nodules in their root

systems producing nitrogen compounds

that help the plant to grow and compete

with other plants When the plant dies

the fixed nitrogen is released making it

available to other plant

Q49 Which of the following is the

largest known cell

नननननलिखितमसहॳकह९नसीसबसहॳबड़ीजञातकह८लिकाहहॴ

SSC CHSL Science (biology) 2016

Question Paper

15-Jan-2017

1) Eukaryotic Cell

यकहॳ ररयह८टटककह८लिका 2) Prokaryotic Cell

परह८कहॳ ररयह८टटककह८लिका 3) Mycoplasma

मायकह८पिासम

4) Ostrich Eggs

ितरमगयकाअिा Correct Answer Ostrich Eggs

Q50 The association of animals in

which both the partners are benefitted

is known as

जानवरोकावहसहयह८गनजसमहॳदह८नोभागीदारिाभापवनतहह८तहॳहहॴ उसहॳ mdashmdashndash कहॳ पमजानाजाताहहॴ SSC CHSL Science (biology) 2016

Question Paper

15-Jan-2017

Options

1) Amensalism

सहजीपवत

2) Commensalism

परजीपवत

3) Colony

कॉिनी 4) Mutualism

अनयह८नयाशरयवाद

Correct Answer Mutualism

Amensalism is any relationship between

organisms of different species in which

F A C E B O O K

P A G E h t t p w w w f a c e b o o k c o m s s c m e n t o r s o f f i c i a l P a g e | 13

FOR MORE UPDATES AND MORE MATERIAL DO LIKE OUR FACEBOOK PAGE httpwwwfacebookcomsscmentorsofficial

one organism is inhibited or destroyed

while the other organism remains

unaffected

Commensalism an association between

two organisms in which one benefits and

the other derives neither benefit nor

harm

Q51 Pneumonia affects which of the

following organs of human body

ननमह८ननयामानविरीरकहॳ नननननलिखितमसहॳककसअगकह८परभापवतकरताहहॴ

15-Jan-2017

Options

1)Kidneys

गद

2)Lungs

फहॳ फड़हॳ 3) Throat

गिहॳ 4) Liver

यकत

Correct Answer Lungs

When the germs that cause pneumonia

reach your lungs the lungsrsquo air sacs

(alveoli) become inflamed and fill up

with fluid This causes the symptoms of

pneumonia such as a cough fever

chills and trouble breathing When you

have pneumonia oxygen may have

trouble reaching your blood

Q52 Mendel is known as

मििकह८ mdashmdash- कहॳ पमजानाजाताहहॴ 15-Jan-2017

Options

1) Father of Physiology

िरीरकियािासतरकहॳ जनक

2) Father of Geology

भगभयिासतरकहॳ जनक

3) Father of Genetics

जहॳनहॳटटकसकहॳ जनक

4) Father of Biology

जीविासतरकहॳ जनक

Correct Answer Father of Genetics

Q53 Which of the following are also

known as Suicidal bag of Cells

ननननलिखितमसहॳककसहॳआतमहतयाकरनहॳवािीकह८लिकाओकाबहॴगकहाजाताहहॴ

15-Jan-2017

Options

1) Lysosomes

िायसोसह८म

2) Lycosome

िायकह८सह८म

3) Nucleus

नालभक

4) Chromosome

िह८मह८सह८म

Correct Answer Lysosomes

Q54 Mesothelioma is a type of cancer

The most common area affected in it is

the lining of the ________

लमज़ह८िहॳिहॳलमयाक सरकाएकपरकारहहॴ इससहॳपरभापवतहह८नहॳवािासबसहॳसामानयकषहॳतर mdash

mdashmdash काअसतरहहॴ 15-Jan-2017

Options

1)Heart

हदय

2)Brain

मनसतषक

3)Stomach

आमािय

4)Lungs

फहॳ फड़हॳ Correct Answer lungs

Asbestos exposure is the main cause of

pleural mesothelioma When asbestos

fibers are breathed in they travel to the

F A C E B O O K

P A G E h t t p w w w f a c e b o o k c o m s s c m e n t o r s o f f i c i a l P a g e | 14

FOR MORE UPDATES AND MORE MATERIAL DO LIKE OUR FACEBOOK PAGE httpwwwfacebookcomsscmentorsofficial

ends of small air passages and reach the

pleura where they can cause

inflammation and

scarring

Q55 Which one of the following is an

insectivorous plant

नननननलिखितमसहॳकह९नसाएकककटाहरीवनसपनतहहॴ

15-Jan-2017

Options

1) Utricularia

यटरीकिहॳररया 2) Sequoia

सहॳकयओइया 3) Nostoc

नॉसटह८क

4) Bryophyta

िायह८फाईटा Correct Answer Utricularia

Q56 ______________ is a

multibranched polysaccharide of

glucose that serves as a form of energy

storage in animals and fungi

mdashmdashगिकह८जकाएकबहिािायकतपह८िीसहॳकहॳ राइिहहॴ जह८जानवरोऔरकवकमउजायभणिारणकहॳ एक पमकाययकरताहहॴ 15-Jan-2017

Options

1) Cellulose

सहॳमयिह८ज

2) Glycogen

गिायकह८जन

3) Pectin

पहॳनकटन

4) Chitin

चीटटन

Correct Answer Glycogen

Q57 The largest gland of the human

body is

mdashmdashmdashमानविरीरकीसबसहॳबड़ीगरिीहहॴ 16-Jan-2017

Options

1) Pancreas

अगयािय

2) Thyroid

िायरॉइि

3) Large Intestine

बड़ीआत

4) Liver

यकत

Correct Answer Liver

Q58 Photosynthesis in plants takes

place in

वनसपनतयोमपरकािसशिहॳषणकीकियाहह८तीहहॴ

16-Jan-2017

Options

1) Stem

तना 2) Leaves

पनततयाा 3) Roots

जड़हॳ 4) Flower

फि

Correct Answer Leaves

During this reaction carbon dioxide

and water are converted into glucose

and oxygen The reaction requires light

energy which is absorbed by a green

substance called

chlorophyll Photosynthesis takes place

in leaf

cells These contain chloroplasts which

are tiny objects containing chlorophyll

F A C E B O O K

P A G E h t t p w w w f a c e b o o k c o m s s c m e n t o r s o f f i c i a l P a g e | 15

FOR MORE UPDATES AND MORE MATERIAL DO LIKE OUR FACEBOOK PAGE httpwwwfacebookcomsscmentorsofficial

Q59 Insects that transmit diseases are

known as

जह८कीड़हॳरह८गसचाररतकरतहॳहहॴ उनह mdashmdash-

कहॳ नामसहॳजानाजाताहहॴ 16-Jan-2017

1)Pathogens

रह८गज़नक

2) Vectors

वहॳकटर

3) Drones

परजीवी 4)Scalars

अटदषट

Correct Answer Vectors

A vector is an organism that does not

cause disease itself but which spreads

infection by conveying pathogens from

one host to another Species of mosquito

for example serve as vectors for the

deadly disease Malaria

Q60 Which is the second largest gland

of Human body

मानविरीरकीदसरीसबसहॳबड़ीगरिीकह९नसीहहॴ

SSC CHSL Science (biology)

2016 Question Paper

16-Jan-2017

Options

1) Liver

यकत

2) Large Intestine

बड़ीआत

3) Thorax

छाती 4) Pancreas

अगनयािय

Correct Answer Pancreas

Q61 Annona squamosa is the scientific

name of

एनह८नासकवामह८सा (Annona squamosa) mdash

mdashmdash कावहॴजञाननकनामहहॴ 16-Jan-2017

Options

1) Custard Apple

सीताफि

2) Papaya

पपीता 3) Babhul

बबि

4) Drumstick

सहजन

Correct Answer Custard Apple

Q62 The disease Beri Beri is caused due

to the deficiency of which of the

following

बहॳरीबहॳरीरह८गनननननलिखितमसहॳककसकीकमीकहॳकारणहह८ताहहॴ

16-Jan-2017

Options

1) Vitamin B2

पवटालमन B2

2) Vitamin B1

पवटालमन B1

3) Vitamin B12

पवटालमन B12

4) Vitamin E

पवटालमन E

Correct Answer Vitamin B1

Beriberi is a disease caused by a vitamin

B-1 deficiency also known as thiamine

deficiency

Q63 Chlorophyll was first isolated and

named by

किह८रह८कफिकह८ mdash-

दवारापहिहॳपिकऔरनालमतककयागया 16-Jan-2017

F A C E B O O K

P A G E h t t p w w w f a c e b o o k c o m s s c m e n t o r s o f f i c i a l P a g e | 16

FOR MORE UPDATES AND MORE MATERIAL DO LIKE OUR FACEBOOK PAGE httpwwwfacebookcomsscmentorsofficial

Options

1) Caventou

कहॳ वहॳत 2) Pelletier

पहॳिहॳटटयर

3) Chlorophyll

किह८रह८कफि

4) Caventou and Pelletier

कहॳ वहॳतऔरपहॳिहॳटटयर

Correct Answer Caventou and Pelletier

Chlorophyll was first isolated and

named by

Joseph Bienaimeacute Caventou and Pierre

Joseph Pelletier in 1817 The presence of

magnesium in chlorophyll was

discovered in 1906 and was the first

time that magnesium had been detected

in living tissue

Q64 Which of the following organisms

does not fit into the Cell Theory

नननननलिखितमसहॳकह९नसाजीवकह८लिकालसदातअन पनहीहहॴ

16-Jan-2017

Options

1) Bacteria

बहॴकटीररया 2) Virus

वायरस

3) Fungi

कवक

4) Plants

पह९धहॳ Correct Answer Virus

The bottom line is that viruses are not

alive and not related to cells in any way

The cell theory states that all living

things are made of cells cells are the

basic units of structure and function of

living things and that all cells come

from other cells Since viruses are not

made of cells and do not use cells in any

of their processes they are not related to

the cell theory

Q65 Which of these is not a

macronutrient for Plants

नननननलिखितमसहॳकह९नसापह९धह८कहॳ लिएमिह८नयटरीएटनहीहहॴ

SSC CHSL Science (biology) 2016

Question Paper

17-Jan-2017

Options

1) Nitrogen

नाइटरह८जन

2) Phosphorus

फासफह८रस

3) Potassium

पह८टालसयम

4) Chlorine

किह८रीन

Correct Answer Chlorine

In relatively large amounts the soil

supplies nitrogen phosphorus

potassium calcium magnesium and

sulfur these are often called the

macronutrients In relatively small

amounts the soil supplies iron

manganese boron molybdenum

copper zinc chlorine and cobalt the

so-called micronutrients

Q66 Name the respiratory organs of

insects

कीटह८मनसतिशरवसनअगनामकानामहहॴ

17-Jan-2017

Options

1) Skin

तवचा 2) Body Surface

िरीरकीसतह

F A C E B O O K

P A G E h t t p w w w f a c e b o o k c o m s s c m e n t o r s o f f i c i a l P a g e | 17

FOR MORE UPDATES AND MORE MATERIAL DO LIKE OUR FACEBOOK PAGE httpwwwfacebookcomsscmentorsofficial

3) Gills

गिफड़हॳ 4) Tracheae

शरावस- निी Correct Answer Tracheae

Air enters the respiratory systems of

insects through a series of external

openings called

spiracles These external openings

which act as muscular valves in some

insects lead to the internal respiratory

system a densely networked array of

tubes called tracheae

Q67 The poisonous gas accidentally

released in Bhopal Gas Tragedy is

भह८पािगहॴसतरासदीमगितीसहॳमकतहईजहरीिीगहॴसिी

17-Jan-2017

1) Methane

मीिहॳन

2) Nitrous Oxide

नाइटरसऑकसाइि

3) Methyl Isocyanate

महॴचििआयसोसायनहॳट

4) Cyanogen

सायनह८जहॳन

Correct Answer Methyl Isocyanate

Q68 What does Trypsin do

टटरनपसनकयाकरताहहॴ

SSC CHSL Science (biology) 2016

Question Paper

17-Jan-2017

Options

1) Breaks down Carbohydrates

काबोहाइडरहॳटकापवघटनकरताहहॴ 2) Synthesizes proteins

परह८टीनकासििहॳषणकरताहहॴ 3) Breaks down fats

वसाकापवघटनकरताहहॴ 4) Breaks down proteins

परह८टीनकापवघटनकरताहहॴ Correct Answer Breaks down proteins

Trypsin is one of the three principal

digestive

proteinases the other two being pepsin

and

chymotrypsin In the digestive process

trypsin acts with the other proteinases

to break down dietary protein molecules

to their component

peptides and amino acids

A protease is any enzyme that performs

proteolysis protein catabolism by

hydrolysis of peptide bonds

Q69 Name the source from which

Aspirin is produced

उससरह८तकानामबताइए

नजससहॳएनसपररनकाउतपादनककयाजाताहहॴ

17-Jan-2017

Options

1) Willow bark

पविह८कीछाि

2) Oak Tree

ओककावकष

3) Acacia

बबि

4) Eucalyptus

नीिचगरी Correct Answer Willow bark

The compound from which the active

ingredient in aspirin was first derived

salicylic acid was found in the bark of a

willow tree in 1763 by Reverend

Edmund Stone of Chipping-Norton

Q70 Cannis Familiaris is the scientific

name of

कहॴ ननसफहॳ लमलियहॳररस mdash- कावहॴजञाननकनामहहॴ

17-Jan-2017

F A C E B O O K

P A G E h t t p w w w f a c e b o o k c o m s s c m e n t o r s o f f i c i a l P a g e | 18

FOR MORE UPDATES AND MORE MATERIAL DO LIKE OUR FACEBOOK PAGE httpwwwfacebookcomsscmentorsofficial

Options

1) Cat

बबमिी 2)Dog

कतता 3) Fox

िह८मड़ी 4) Wolf

भहॳडड़या Correct Answer Dog

Q71 Harmful bacteria in potable water

make the water

पीनहॳकहॳ पानीमनसतिघातकबहॴकटीररयाउसपानीकह८बनातहॳहहॴ 17-Jan-2017

Options

1) unfit to drink

पीनहॳकहॳ लिएअयह८गय

2) smelly

दगयनधयकत

3) Colored

रगीन

4) Turbid

मटमहॴिा Correct Answer unfit to drink

Q72 Musa paradisiaca is the scientific

name of which plant

मसापहॴराडिलसयाकाककसपह९धहॳकावहॴजञाननकनामहहॴ

17-Jan-2017

Options

1) Mango

आम

2) Wheat

गहॳह

3) Corn

भ ा 4) banana

कहॳ िा Correct Answer banana

Q73 Prawns belong to which family

झीगहॳककसपररवारकहॳ हह८तहॳहहॴ 17-Jan-2017

Options

1) Crustaceans

िसटहॳलियन

2)Fish

मछिी 3) Amphibians

अननफबबयस

4) Reptiles

रहॳपटाइमस

Correct Answer Crustaceans

Q74 Name the drug that is yielded from

Cinchona tree and is used to cure

malaria

उसऔषचधकानामबताइएनजसहॳलसगकह८नापहॳड़सहॳपरापतककयाजाताहहॴऔरनजसकाउपयह८गमिहॳररयाकहॳ उपचारमककयाजाताहहॴ 17-Jan-2017

Options

1) Camptothea

कहॴ नटह८चिया 2) Acuminata

एकयलमनहॳटा 3) Quinine

कनहॴन

4) Cinchonia

लसकह८ननया Correct Answer Quinine

Q75 Blood Circulation was discovered

by

रकतपररसचरणकी mdashmdashndash दवारािह८जकीिी 17-Jan-2017

Options

1) Mary Anderson

F A C E B O O K

P A G E h t t p w w w f a c e b o o k c o m s s c m e n t o r s o f f i c i a l P a g e | 19

FOR MORE UPDATES AND MORE MATERIAL DO LIKE OUR FACEBOOK PAGE httpwwwfacebookcomsscmentorsofficial

महॴरीएिरसन

2) Virginia Apgar

वनजयननयाएपगार

3) William Harvey

पवलियमहाव

4) Robert Feulgen

रॉबटयफ़यिजहॳन Correct Answer William Harvey

Q76 Vitamin A is also known as

पवटालमन A कह८ mdashmdash- कहॳ नामसहॳभीजानाजाताहहॴ SSC CHSL Science (biology) 2016

Question Paper

18Jan2017

Options

1) Thiamine

िायलमन

2) Riboflavin

ररबह८फिहॳपवन

3) Retinol

रहॳटटनॉि

4) Calciferol

कहॴ नमसफहॳ रह८ि

Correct Answer Retinol

Q77 Some roots called arise from an

organ other than the radicle

कछजड़हॳनजनह mdashmdashmdash कहाजाताहहॴ वहमिकहॳ अिावाककसीअनयअगसहॳउतपननहह८तीहहॴ 18Jan2017

Options

1) tap roots

मखयजड़

2) stilt roots

ि ाजड़

3) fibrous roots

रहॳिहॳदारजड़

4) adventitious roots

आकनसमकजड़

Correct Answer adventitious roots

Q78 Spiders belong to which class of

animals

मकडड़यापराणीवगीकरणकहॳ ककसवगयमआतीहहॴ 18Jan2017

Options

1) Arachnids

एरहॳकननडस

2) Aves

एपवस

3) Gastropods

गहॴसटरोपह८िस

4) Anthozoa

एिह८जआ

Correct Answer Arachnids

Q79 How many layers does Human

Skin have

मानवतवचामककतनीपरतहॳहह८तीहहॴ

18Jan2017

Options

1) 5

2) 7

3) 11

4) 3

Correct Answer 3

Skin has three layers The epidermis

the outermost layer of skin provides a

waterproof barrier and creates our skin

tone The dermis beneath the

epidermis contains tough connective

tissue hair follicles and sweat glands

The deeper subcutaneous tissue (

hypodermis ) is made of fat and

connective tissue

Q80 Allium Cepa is the scientific name

of

एलियमलसपपा mdashmdashndash कावहॴजञाननकनामहहॴ 18Jan2017

F A C E B O O K

P A G E h t t p w w w f a c e b o o k c o m s s c m e n t o r s o f f i c i a l P a g e | 20

FOR MORE UPDATES AND MORE MATERIAL DO LIKE OUR FACEBOOK PAGE httpwwwfacebookcomsscmentorsofficial

Options

1) Carrot

गाजर

2) Tomato

टमाटर

3) Potato

आि 4) Onion

पयाज़

Correct Answer Onion

Q81 DNA stands for

िीएनएकापणय प mdashmdash- हहॴ 18Jan2017

Options

1) Di Nucleic Acid

िाईनयनकिकएलसि

2) Deoxy Nucleic Acid

िीओकसीनयनकिकएलसि

3) Diribonucleic Acid

िाईराइबह८नयनकिकएलसि

4) Deoxyribonucleic Acid

िीऑकसीराइबह८नयनकिकएलसि

Correct Answer Deoxyribonucleic Acid

Q82 Organisms that generate energy

using light are known as

जह८जीवाणपरकािकाउपयह८गकरउजायउतपननकरतीहहॴ उनह mdashmdash कहॳ पमजानाजाताहहॴ

18Jan2017

Options

1) Chaemolithotrophs

ककमह८लििह८टरह८पस

2) Oligotrophs

ओलिगह८टरह८पस

3) Bacteria

बहॴकटीररया 4)Photoautotrophs

फह८टह८ओटह८टरह८पस

Correct Answer Photoautotrophs

An oligotroph is an organism that can

live in an environment that offers very

low levels of nutrients

Q83 Which drug is used as an

Antidepressant

ककसदवाएकहतािारह८धीकहॳ पमपयोगककयाजाताहहॴ Options

1) Oxybutynin

ओकसीलयटीनन

2)Tramadol

टरहॳमहॳिह८ि

3 ) Sumatriptan

समहॳटरीपटहॳन

4) Bupropion

लयपरह८पपयह८न

Correct Answer Bupropion

लयपरह८पपयह८न

Q84 The orange colour of carrot is

because of

गाजरकानारगीरगनननननलिखितमसहॳककसीएककीवजहसहॳहह८ताहहॴ 18Jan2017

Options

1) it grows in the soil

यहलम ीमउगतीहहॴ 2) Carotene

कहॴ रह८टीन

3) it is not exposed to sunlight

यहसययपरकािकहॳ सपकय मनहीआती 4) the entire plant is oranqe in colour

सनपणयपह९धानारगीरगकाहह८ताहहॴ Correct Answer Carotene

Q85 Snake venom is highly modified

saliva containing

F A C E B O O K

P A G E h t t p w w w f a c e b o o k c o m s s c m e n t o r s o f f i c i a l P a g e | 21

FOR MORE UPDATES AND MORE MATERIAL DO LIKE OUR FACEBOOK PAGE httpwwwfacebookcomsscmentorsofficial

सापकाजहरअततयाचधकसिह८चधतिारहह८तीहहॴनजसमहॳ mdashmdash- हह८ताहहॴ Options

l)Prototoxins

परह८टह८टॉनकसस

2)Neutrotoxins

नयटरोटॉनकसस

3)Zootoxins

जटॉनकसस

4)Electrotoxins

इिहॳकटरह८टॉनकसस

Correct Answer Zootoxins

जटॉनकसस

Q86 Which type of pathogen causes the

water-borne disease Schistosomiasis

ककसपरकारकारह८गज़नकजिजननतरह८गलससटह८सह८लमलससकाकारणबनताहहॴ

18Jan2017

Option

1) Parasitic

परजीवी 2)Protozoan

परह८टह८जआ

3) Bacterial

बहॴकटीररयि

4) Viral

वायरि

Correct Answer Parasitic

Schistosomiasis also known as snail

fever and bilharzia is a disease caused

by parasitic

flatworms called schistosomes

Q87 Prothrombin responsible for

clotting of blood is released by

परह८िह८ननबन

जह८रकतकािककाजमनहॳकहॳ लिएनजनमहॳदारहहॴ mdashndash

कहॳ दवारासतरापवतककयाजाताहहॴ

19Jan2017

Options

1) Small Intestine

छह८टीआत

2) Blood Platelets

रकतपिहॳटिहॳटस

3) Large Intestine

बड़ीआत

4Heart

हदय

Correct Answer Blood Platelets

Q88 Acacia arabica is the scientific

name of

अकहॳ लियाअरहॳबबका mdashmdashndash कावहॴजञाननकनामहहॴ 19-Jan-2017

Options

1) Neem

नीम

2) Teak

सागह९न

3) Babhul

बबि

4) Pomegranate

अनार

Correct Answer Babhul

Q89 Cannis Vulpes is the scientific

name of

कहॴ ननसवनमपस mdashmdash- कावहॴजञाननकनामहहॴ 19-Jan-2017

Options

1) Dog

कतता 2) Wolf

भहॳडड़या 3) Fox

िह८मड़ी 4) Hyena

िाकिबगघा

F A C E B O O K

P A G E h t t p w w w f a c e b o o k c o m s s c m e n t o r s o f f i c i a l P a g e | 22

FOR MORE UPDATES AND MORE MATERIAL DO LIKE OUR FACEBOOK PAGE httpwwwfacebookcomsscmentorsofficial

Correct Answer Fox

Q90 The beetroot is the portion of the

beet plant

चकदरपह९धहॳका mdashmdashndash भागहहॴ 19-Jan-2017

Options

1) tap root

मखयजड़

2) Adventitious

आकनसमक

3) bulb of the stem

तनहॳकाकद

4) Rhizome

परकद

Correct Answer tap root

Q91 What is the basic unit of heredity

आनवलिकताकीबननयादीइकाईकयाहहॴ 19-Jan-2017

Options

1) DNA

िीएनए

2) RNA

आरएनए

3) Chromosome

िह८मह८सह८म

4) Gene

जीन

Correct Answer gene

Genes are the units of heredity and are

the instructions that make up the bodyrsquos

blueprint They code for the proteins

that determine virtually all of a personrsquos

characteristics Most genes come in

pairs and are made of strands of genetic

material called deoxyribonucleic acid

or DNA

Q92 Lungs are the primary organs of

फहॳ फड़हॳmdashndashकहॳ परािलमकअगहहॴ

19-Jan-2017

Options

1) Digestion

पाचन

2) Constipation

कलज

3) Perspiration

पसीना 4)Respiration

शवसन

Correct Answer Respiration

Q93 Sugarcane is a type of

गननाएकपरकारका mdash- हहॴ 20-Jan-2017

Options

1)creeper

िता 2)tree

पहॳड़

3)shrub

झाड़ी 4)grass

घास

Correct Answer grass

Q94 Who is commonly known as ldquothe

Father of Microbiologyrdquo

सामानयत ldquo सकषमजीवपवजञानकहॳ जनक lsquo

कहॳ नामसहॳककसहॳजानाजातहहॴ 20-Jan-2017

Options

1) Robert Hooke

रॉबटयहक

2) Antonie Philips van Leeuwenhoek

एटह८नीकफलिपवानमयएनहह८क

3) Carl Linnaeus

काियिीनाईयस

4) Charles Darwin

चामसयिापवयन

F A C E B O O K

P A G E h t t p w w w f a c e b o o k c o m s s c m e n t o r s o f f i c i a l P a g e | 23

FOR MORE UPDATES AND MORE MATERIAL DO LIKE OUR FACEBOOK PAGE httpwwwfacebookcomsscmentorsofficial

Correct Answer Antonie Philips van

Leeuwenhoek

Q95 For the aquatic organisms the

source of food is

जिीयजीवाणकािाघसरह८तहहॴ 20-Jan-2017

Options

1) Phytoplankton

फायटह८पिहॳकटन

2) Sea Weed

समदरीिहॴवाि

3)Aqua plankton

एकवापिहॳकटन

4) Zooplankton

जपिहॳकटन

Correct Answer Phytoplankton

Q96 Haemoglobin has the highest

affinity with which of the following

हीमह८गिह८बबनकीननननमसहॳककसकहॳ सािउततमसमानताहहॴ

20-Jan-2017

Options

1)SO2

2)CO2

3)CO

4)NO2

Correct Answer CO

It has a greater affinity for hemoglobin

than oxygen does It displaces oxygen

and quickly binds so very little oxygen

is transported through the body cells

Q97 Who developed the theory of

Evolution

उदपवकासकालसदातककसनहॳपवकलसतककया

20-Jan-2017

Options

1) Charles Darwin

चामसयिापवयन

2) Isaac Newton

आयजहॳकनयटन

3) Pranav Mistry

परणवलमसतरी 4) Galileo Galilei

गहॳलिलियह८गहॳिीिी Correct Answer Charles Darwin

Q98 The primary function of RNA is

RNA कापरािलमककाययहह८ताहहॴ 20-Jan-2017

Options

1) Photosynthesis

परकािसशिहॳषण

2) Protein Synthesis

परह८टीनसशिहॳषण

3) Replication

परनतकनतबनाना 4) Translation

अनवादकरना Correct Answer Protein Synthesis

There are two main functions of RNA

It assists DNA by serving as a messenger

to relay the proper genetic information

to countless numbers of ribosomes in

your body The other main function of

RNA is to select the correct amino acid

needed by each ribosome to build new

proteins for your body

Q99 ______is the movement of

molecules across a cell membrane from

a region of their lower concentration to

a region of their higher concertration

उचचसादरताकहॳ कषहॳतरसहॳउसकीकमसादरतावािहॳकषहॳतरकीतरफएककह८लिकाखझमिीकहॳ माधयमसहॳहह८नहॳवािाअणओकहॳ सचिनकह८ mdash- कहतहॳहहॴ Options

1) Diffusion

पवसरण

2) Osmosis

ऑसमह८लसस

F A C E B O O K

P A G E h t t p w w w f a c e b o o k c o m s s c m e n t o r s o f f i c i a l P a g e | 24

FOR MORE UPDATES AND MORE MATERIAL DO LIKE OUR FACEBOOK PAGE httpwwwfacebookcomsscmentorsofficial

3) Active Transport

सकियआवागमन

4) Passive Transport

नननषियआवागमन

Correct Answer Active Transport

Q100 Study of classification of

organisms is known as 20-Jan-2017

जीवाणओकहॳ वगीकरणकहॳ अधययनकह८ mdash-

कहाजाताहहॴ Options

1) Serpentology

सपरहॳटह८िह८जी 2) Virology

वायरह८िह८जी 3) Taxonomy

टहॴकसोनह८मी 4) Physiology

कफनज़यह८िह८जी Correct Answer Taxonomy

Q101 Photosynthesis takes place inside

plant cells in

परकािसशिहॳषणवनसपनतकह८लिकामनसति mdash

mdashmdash महह८ताहहॴ 20-Jan-2017

Options

1) Ribosomes

राइबह८सह८नस

2) Chloroplasts

किह८रह८पिासट

3) Nucleus

नयकलियम

4) Mitochondria

माईटह८कोडडरया Correct Answer Chloroplasts

Q102 ______ is the cell organelle in

which the biochemical processes of

respiration and energy production

occur

mdashmdash- वहकह८लिकाअगहहॴ नजसमहॳशवसनऔरउजायउतपादनकहॳ जहॴसीजहॴवरासायननकपरकियायहह८तीहहॴ 20-Jan-2017

Options

1) Mitochondria

माइटह८कोडडरया 2) Chloroplast

किह८रह८पिासट

3) Ribosomes

राइबह८सह८नस

4) Nucleus

नयकिीयस

Correct Answer Mitochondria

Q103 Which non-flowering spore

bearing plants have roots

ककसफिनिगनहॳवािहॳऔरबीजाणधारकपह९धह८कीजड़हॳहह८तीहहॴ 21-Jan-2017

Options

1) Mosses

मह८सहॳस

2) Angiosperms

एननजयह८सपनसय 3) Ferns

फनसय 4) Gymnosperms

नजननह८सपनसय Correct Answer ferns

Q104 Which of the following is an

excretory organ of cockroach

नननननलिखितमसहॳकह९नसानतिच हॳकाउतसजयनअगहहॴ

21-Jan-2017

Options

F A C E B O O K

P A G E h t t p w w w f a c e b o o k c o m s s c m e n t o r s o f f i c i a l P a g e | 25

FOR MORE UPDATES AND MORE MATERIAL DO LIKE OUR FACEBOOK PAGE httpwwwfacebookcomsscmentorsofficial

1) Malphigian Tubules

मनमफनजयनटयबमस

2) Nephridia

नहॳकफरडिया 3) Coxal Gland

कह८कसिगरचिया 4) Green Gland

गरीनगरचिया Correct Answer Malphigian Tubules

Q105 Evaporation of water takes place

in which part of plants

पानीकहॳ वाषपीकरणकीकियापह९धोकहॳ ककसभागसहॳहह८तीहहॴ 21-Jan-2017

Options

1) Stem

तना 2) Stomata

सटह८मटा 3) Branch

िािाए

4) Fruit

फि

Correct Answer Stomata

Evaporation accounts for the movement

of water to the air from sources such as

the soil canopy interception and

waterbodies Transpiration accounts for

the movement of water within a plant

and the subsequent loss of water as

vapour through stomata in its leaves

Q106 A is the fleshy spore-bearing

fruiting body of a fungus

mdashmdashndashकवककामासि

बीजाणधारणकरनहॳवािाफिनहॳवािाअगहहॴ 21-

Jan-2017

Options

1) aloe vera

एिह८वहॳरा 2) Coral

मगा 3) Cactus

कहॴ कटस

4) Mushroom

ककरमतता Correct Answer mushroom

Q107 Which of the following is a fungal

disease

नननननलिखितमसहॳकह९नसाफफदसहॳहह८नहॳवािाएकरह८ग हहॴ

21-Jan-2017

Options

1) Dermatitis

तवचािह८ध

2) Cholera

हहॴजा 3) Jaundice

पीलिया 4) Indigofera

इननिगह८फहॳ रा Correct Answer Dermatitis

Dermatitis also known as eczema is a

group of diseases that results in

inflammation of the skin These diseases

are characterized by itchiness red skin

and a rash In cases of short duration

there may be small blisters while in

long-term cases the skin may become

thickened

Q108 In which form is glucose stored in

our body

हमारहॳिरीरमगिकह८जकासचयककस पमककयाजाताहहॴ

21-Jan-2017

Options

1) Insulin

F A C E B O O K

P A G E h t t p w w w f a c e b o o k c o m s s c m e n t o r s o f f i c i a l P a g e | 26

FOR MORE UPDATES AND MORE MATERIAL DO LIKE OUR FACEBOOK PAGE httpwwwfacebookcomsscmentorsofficial

इसलिन

2) Glucose

गिकह८ज

3) Glycogen

गिायकह८जहॳन

4) Fat

वसा Correct Answer Glycogen

Excess glucose is stored in the liver as

the large compound called glycogen

Glycogen is a polysaccharide of glucose

but its structure allows it to pack

compactly so more of it can be stored in

cells for later use

Q109 Where do plants synthesize

protein from

पह९धहॳपरह८टीनसशिहॳषणकहासहॳकरतहॳहहॴ

Options

1) Fatty Acids

वसाऐलसि

2) Sugar

िकर

3) Amino Acids

एलमनह८ऐलसि

4) Starch

सटाचय Correct Answer Amino Acids

Q110 Which part of the brain is

responsible for triggering actions like

thinking intelligence memory and

ability to learn

मनसतषककाकह९नसाटहससासह८चनहॳ बनधदमानी याददाशतऔरसीिनहॳकीकषमताजहॴसीकियाओकह८परहॳररतकरताहहॴ 21-Jan-2017

Options

1) Diencephalon

िायएनसहॳफहॳ िह८न

2) Hypothalamus

हयपह८िहॳिहॳमस

3) Cerebrum

सहॳरहॳिम

4) Control

कटरह८ि

Correct Answer Cerebrum

Q111 Which of the following is also

known as the Biochemical Laboratory

of the Human Body

नननननलिखितमसहॳककसहॳमानविरीरकीजहॴवरसायनपरयह८गिािाभीकहाजाताहहॴ 21-Jan-2017

Options

1) Small Intestine

छह८टीआत

2)Brain

मनसतषक

3) Pancreas

अगनयािय

4) Liver

नजगर

Correct Answer Liver

The liver makes bile that will help

emulsify and digest the fats we eat

The liver takes toxic substances and

convert them using enzymes the liver

cells makes into a non toxic form so the

body can dispose of them

The liver also converts fats protein and

carbohydrates into glucose which is the

energy source for our cells to use

The liver takes amino acids and makes

proteins by combining them

Q112 The yellow colour of human urine

is due to

मानवमतरकापीिारग mdashndash कीवजहसहॳहह८ताहहॴ 22-

Jan-2017

Options

1) Bile Salts

F A C E B O O K

P A G E h t t p w w w f a c e b o o k c o m s s c m e n t o r s o f f i c i a l P a g e | 27

FOR MORE UPDATES AND MORE MATERIAL DO LIKE OUR FACEBOOK PAGE httpwwwfacebookcomsscmentorsofficial

पपततनमक

2) Cholesterol

कह८िहॳसटरह८ि

3) Lymph

लिनफ

4) Urochrome

यरह८िह८म

Correct Answer Urochrome

Urobilin or urochrome is the chemical

primarily responsible for the yellow

color of urine

Q113 The wilting of plants takes place

due to

पह९धह८कालिचििहह८नाकी mdashmdash- कीवजहसहॳहह८ताहहॴ 22-Jan-2017

Options

1)Photosynthesis

परकािसशिहॳषण

2) Transpiration

वाषपह८तसजयन

3) Absorption

अविह८षण

4) Respiration

शरवसन

Correct Answer Transpiration

Wilting is the loss of rigidity of non-

woody parts of plants This occurs when

the turgor pressure in non-lignified

plant cells falls towards zero as a result

of diminished water in the cells

Q114 Bovidae Ovis is the scientific name of

बह८पविीओपवस mdashndash कावहॴजञाननकनामहहॴ 22-Jan-2017

Options

1) Goat

बकरी 2) Cow

गाय

3) Buffalo

भहॳस

4) Sheep

भहॳड़

Correct Answer Sheep

Q115 Plants get their energy to produce

food from which of the following

पह८धहॳभह८जनकाननमायणकरनहॳकहॳ लिएनननननलिखितमसहॳककससहॳउजायपरापतकरतहॳहहॴ

22-Jan-2017

Options

1) Photosynthesis

परकािसशिहॳषण

2)Bacteria

बहॴकटीररया 3)Fungi

कवक

4)Sun

सयय Correct Answer Sun

Q116 Which of the following is secreted

by the liver

नननननलिखितमसहॳककसकासरावनजगरसहॳहह८ताहहॴ

22-Jan-2017

Options

1) Glucose

गिकह८ज

2) Iodine

आयह८िीन

3) Cortisol

काटटरयसह८ि

4) Bile

पपतत

Correct Answer Bile

The liver makes bile that will help

emulsify and

digest the fats we eat

F A C E B O O K

P A G E h t t p w w w f a c e b o o k c o m s s c m e n t o r s o f f i c i a l P a g e | 28

FOR MORE UPDATES AND MORE MATERIAL DO LIKE OUR FACEBOOK PAGE httpwwwfacebookcomsscmentorsofficial

Q117 Ferns belong to which division of

plants

फनसयपह९धह८कहॳ ककसभागमआतहॳहहॴ

22-Jan-2017

Options

1) Gymnosperms

नजननह८सपनसय 2) Angiosperms

एनजयह८सपनसय 3) Thallophyta

िहॴिह८फाईटा 4)Pteridophyta

टहॳररिह८फाईटा Correct Answer Pteridophyta

Q118 Who invented Antibiotics

एटीबायह८टटककाअपवषकारककसनहॳककयािा

22-Jan-2017

Options

1) Joseph Lister

जह८सहॳफलिसटर

2) William Harvey

पवलियमहाव

3) Robert Knock

रॉबटयनॉक

4)Alexander Fleming

अिहॳकज़िरफिहॳलमग

Correct Answer Alexander Fleming

Q119 Milbecycin is used in the

eradication of

लममबहॳसायलसनका mdashndash

मउनमिनमपरयह८गककयाजाताहहॴ 22-Jan-2017

Options

1) Agricultural Fungus

कपषकवक

2) Agricultural Pests

कपषकीटक

3) Agricultural Herbs

कपषिाक

4)Agricultural Weeds

कपषननराना Correct Answer Agricultural Pests

Milbemycin oxime is a veterinary drug

from the group of milbemycins used as

a broad spectrum antiparasitic It is

active against worms and mites(insects

Q120 Intestinal bacteria synthesizes

which of the following in the human

body

मानविरीरमआतोकहॳ बहॴकटीररयानननननलिखितमसहॳककसकासशिहॳषणकरतहॳहहॴ 22-Jan-2017

Options

1) Vitamin K

पवटालमन K

2) Proteins

परह८टीन

3) Fats

वसा 4) Vitamin D

पवटालमन D

Correct Answer Vitamin K

Q121 is the study of the physical form

and external structure of plants

mdashmdash-

मपह९धह८काभहॴनतक पऔरबाहरीसरचनाकाआदयाककयाजाताहहॴ 22-Jan-2017

Options

1) Physiology

कफनजयह८िह८जी 2) Anatomy

िरीररचनापवजञान

3) Phytomorphology

फाईटह८मह८फह८िह८जी 4)Cytology

कह८लिकापवजञान

Correct Answer Phytomorphology

F A C E B O O K

P A G E h t t p w w w f a c e b o o k c o m s s c m e n t o r s o f f i c i a l P a g e | 29

FOR MORE UPDATES AND MORE MATERIAL DO LIKE OUR FACEBOOK PAGE httpwwwfacebookcomsscmentorsofficial

Q122 Which of the following is a

structural and functional unit of

kidneys

नननननलिखितमसहॳकह९नसीगदोकीसरचनातमकऔरकाययकरीईकाईहहॴ

22-Jan-2017

Options

1) Renette Cells

रहॳनहॳटकह८लिकाए

2) Flame Cells

फिहॳमकह८लिकाए

3) Nephrites

नहॳफ़राइटस

4)Nephrons

नहॳफरोस

Correct Answer Nephrons

Nephron functional unit of the kidney

the structure that actually produces

urine in the process of removing waste

and excess substances from the blood

There are about 1000000 nephrons in

each human kidney

Q123 Which of the following is the

largest part of the human brain

नननननलिखितमसहॳकह९नसामानवमनसतषककासबसहॳबड़ाटहससाहहॴ

23-Jan-2017

Options

1) Ribs

पसलियाा 2) Cerebrum

सहॳरहॳिम

3) Pons

पोस

4)Thalamus

िहॴिहॳमस

Correct Answer Cerebrum

The cerebrum is the largest part of the

human brain making up about two-

thirds of the brainrsquos mass It has two

hemispheres each of which has four

lobes frontal parietal temporal and

occipital

Q124 The auxiliary buds

सहायककालियाmdashndash 23-Jan-2017

Options

1) grow endogenously from the pericycle

पहॳरीसाईककिसहॳअनतजातयपवकलसतहह८ताहहॴ 2) arise endogenously from the main

growing point

मिवपदसहॳअनतजातयउठताहहॴ 3) is an embryonic shoot located in the

axil of a leaf

एकभरणिटहहॴजह८एकपततीकहॳ अकषपरनसतिहह८ताहहॴ 4)arise exogenously from the epidermis

एपपिलमयससहॳबटहजातयतरीकहॳ सहॳउठताहहॴ Correct Answer is an embryonic shoot

located in the axil of a leaf

Q125 Which of the following is a viral

disease

इनमहॳसहॳकह९सीएकवायरिबीमारीहहॴ

23-Jan-2017

Options

1) Polio

पह८लियह८ 2) Tetanus

धनसतनभ

3) Leprosy

कषठरह८ग

4) Plague

पिहॳग

Correct Answer Polio

A viral disease (or viral infection)

occurs when an organismrsquos body is

invaded by pathogenic viruses and

infectious virus particles (virions) attach

to and enter susceptible cells

F A C E B O O K

P A G E h t t p w w w f a c e b o o k c o m s s c m e n t o r s o f f i c i a l P a g e | 30

FOR MORE UPDATES AND MORE MATERIAL DO LIKE OUR FACEBOOK PAGE httpwwwfacebookcomsscmentorsofficial

Poliomyelitis often called polio or

infantile paralysis is an infectious

disease caused by the poliovirus

Tetanusmdash A serious bacterial infection

that causes painful muscle spasms and

can lead to death

Leprosy also known as Hansenrsquos

disease (HD) is a long-term infection by

the bacterium Mycobacterium leprae or

Mycobacterium lepromatosis

Plague is an infectious disease caused by

the bacterium Yersinia pestis

Symptoms include fever weakness and

headache

Q126 Which organisms can help to

carry out Vermicomposting

कह९नसाजीववमीकनपह८नसटगममददकरताहहॴ

23-Jan-2017

Options

1) Nitrifying Bacteria

नाईटरीफाईगबहॴकटीररया 2) Earthworms

कहॴ चऐ

3) Algae

िहॴवि

4) Fungus

कवक

Correct Answer Earthworms

Q127 Contraction of heart is also

known as

हदयकहॳ सकचनकह८ mdash- भीकहाजाताहहॴ 23-Jan-

2017

Options

1) Systole

लससटह८ि

2) Aristotle

अरसत

3) Diastole

िायसटह८ि

4) Lub

मयब

Correct Answer Systole

Diastole is the part of the cardiac cycle

when the heart refills with blood

following systole (contraction)

Ventricular diastole is the period during

which the ventricles are filling and

relaxing while atrial diastole is the

period during which the atria are

relaxing

Q128 Azadirachta indica is the

botanical name of which of the

following

अजाटदराचताइडिकानननननलिखितमसहॳककसकावानसपनतनामहहॴ

23-Jan-2017

Options

1) Rose plant

गिाबकापह९धा 2) Apple tree

सहॳबकापहॳड़

3) Neem

नीम

4)Mango

आम

Correct Answer Neem

Q129 Which of the following is the

main end product of carbohydrate

digestion

नननननलिखितमसहॳकह९नसाकाबोहाइडरहॳटकहॳ पाचनकापरमिअतउतपादकहह८ताहहॴ 23-Jan-2017

Options

1) Fats

वसा 2) Lipids

लिपपडस

3) Glucose

गिकह८ज

4) Cellulose

F A C E B O O K

P A G E h t t p w w w f a c e b o o k c o m s s c m e n t o r s o f f i c i a l P a g e | 31

FOR MORE UPDATES AND MORE MATERIAL DO LIKE OUR FACEBOOK PAGE httpwwwfacebookcomsscmentorsofficial

सहॳमयिह८ज

Correct Answer Glucose

Intestinal absorption of end products

from digestion of carbohydrates and

proteins in the pig hellip During absorption some sugars (fructose or

galactose) released from the

corresponding sucrose and lactose

respectively during digestion were

partly metabolized into glucose by the

enterocyte

Q130 Which of the following glands is a

source of the enzyme Ptyalin

नननननलिखितगरचियोमसहॳएजाइमटयालिनकासरह८तहहॴ 23-Jan-2017

Options

1) Pancreas

अगरािय

2) Thyroid Gland

िाइराइिगरिी 3) Pituitary Gland

पीयषगरिी 4) Salivary Glands

िारगरचियाा Correct Answer Salivary Glands

Q131 Which of the following is not true

about Pteridophyta

ननननमसहॳकह९नसीबातटहॳररिह८फाईटकहॳ बारहॳमसचनहीहहॴ 23-Jan-2017

Options

1) Dominant phase is saprophytes

परमिचरणसहॳपरह८फाईइटसहह८ताहहॴ 2) Main plant body is diploid

पह९दह८कामखयिरीरदपवगखणतहह८ताहहॴ 3) Seeds are present

बीजमह९जदहह८तहॳहहॴ 4)Flowers are absent

फिअनपनसतिहह८तहॳहहॴ

Correct Answer Seeds are present

Q132 The largest dolphin species is the

orca also called as

िॉिकफनकीसबसहॳबड़ीपरजानतकाकानामआकायहहॴनजसहॳ mdash- भीकहतहॳहहॴ 23-Jan-2017

Options

1) Bottle Nose

बाटिनह८ज

2) Baiji

बहॳजी 3) Killer whale

ककिरहहॳि

4)Tucuxi

टकवसी Correct Answer Killer whale

Q133 The fat digesting enzyme Lipase

is secreted by which of the following

वसाकापाचनकरनहॳवािाएजाइमिाइपहॳजनननननलिखितमसहॳककसकहॳ दवारासतरापवतहह८ताहहॴ

24-Jan-2017

Options

1) Kidneys

गद

2) Pancreas

अगनयािय

3) Large Intestine

बड़ीआत

4)Liver

नजगर

Correct Answer Pancreas

Lipase is an enzyme that splits fats so

the intestines can absorb them Lipase

hydrolyzes fats like triglycerides into

their component fatty acid and glycerol

molecules It is found in the blood

gastric juices pancreatic secretions

intestinal juices and adipose tissues

F A C E B O O K

P A G E h t t p w w w f a c e b o o k c o m s s c m e n t o r s o f f i c i a l P a g e | 32

FOR MORE UPDATES AND MORE MATERIAL DO LIKE OUR FACEBOOK PAGE httpwwwfacebookcomsscmentorsofficial

Q134 The arrangement of leaves on an

axis or stem is called

एकअकषयातनहॳपरपनततयोकीयवसिाकह८कयाकहाजाताहहॴ SSC CHSL Science (biology) 2016

Question Paper

24-Jan-2017

Options

1) Phyllotaxy

फाइिह८टहॴकसी 2) Vernation

वनिन

3) Venation

वहॳनहॳिन

4)Phytotaxy

फाइटह८टहॴकसी Correct Answer Phyllotaxy

In botany phyllotaxis or phyllotaxy is

the arrangement of leaves on a plant

stem (from Ancient Greek phyacutellon

ldquoleafrdquo and taacutexis ldquoarrangementrdquo)

Phyllotactic spirals form a distinctive

class of patterns in nature

Q135 The study of Cells is also known

as

कह८लिकाओकहॳ अधययनकह८ mdashmdashndash

भीकहाजाताहहॴ 24-Jan-2017

Options

1) Cytology

सायटह८िह८जी 2) Physiology

कफनजयह८िह८जी 3) Nucleology

नयककमयह८िह८जी 4)Cellology

सहॳिह८िह८जी Correct Answer Cytology

Q136 Which of the following scientists

is also known as the Father of Biology

नननननलिखितमसहॳककसवहॴजञाननककह८ ldquoजीवपवजञानकहॳ जनकrdquoकहॳ नामसहॳभीजानाजाताहहॴ 24-Jan-2017

Options

1) Herbert Spencer

हबयटयसपसर

2) Aristotle

अरसत 3) Lamarck

िहॳमाकय 4)Darwin

िापवयन

Correct Answer Aristotle

Q137 Which cells give rise to various

organs of the plant and keep the plant

growing

कह९नसीकह८लिकाएपह९धह८कहॳ लभननअगह८कह८जनमदहॳतीहहॴऔरपह९धह८कह८बढ़नहॳममददकरतीहहॴ

24-Jan-2017

Options

1) Permanent

सिायी 2) Dermal

तवचीय

3) Meristematic

मररसटहॳमटटक

4)Mature

परह८ढ़

Correct Answer Meristematic

A meristem is the tissue in most plants

containing undifferentiated cells

(meristematic cells) found in zones of

the plant where growth can take place

Q138 Rodentia Muridae is the scientific

name of

F A C E B O O K

P A G E h t t p w w w f a c e b o o k c o m s s c m e n t o r s o f f i c i a l P a g e | 33

FOR MORE UPDATES AND MORE MATERIAL DO LIKE OUR FACEBOOK PAGE httpwwwfacebookcomsscmentorsofficial

रह८िहॳलियानयररिी mdashmdash- कावहॴजञाननकनामहहॴ 24-

Jan-2017

Options

1) Mouse

चहा 2) Squirrel

चगिहरी 3) Monkey

बदर

4) Lizard

नछपकिी Correct Answer Mouse

Q139 Name the scientist who proposed

the cell theory

कह८लिकालसदातकापरसतावदहॳनहॳवािहॳवहॴजञाननककानामबताइए 24-Jan-2017

Options

1) Schleiden and Schwann

िीमिनऔरशरववान

2) Lamarck

िहॳमाकय 3) Treviranus

टरहॳवायरहॳनस

4)Whittaker and Stanley

हीटकरऔरसटहॳनिहॳ Correct Answer Schleiden and

Schwann

Q140 The flower with the worldrsquos

largest bloom is

दननयाकासबसहॳबड़ाफिखििनहॳवािा mdashmdashndash हहॴ 24-Jan-2017

Options

1) Pando

पािह८ 2) Posidonia

पह८सीिह८ननया 3) Rafflesia arnoldii

ररफिहॳलियाअनोमिी 4)Helianthus annuus

हहॳलिएनिसएनयअस

Correct Answer Rafflesia arnoldii

Rafflesia arnoldii is a species of

flowering plant in the parasitic genus

Rafflesia It is noted for producing the

largest individual flower on earth It has

a very strong and horrible odour of

decaying flesh earning it the nickname

ldquocorpse flower

Q141 Deficiency of which vitamin

causes night blindness

ककसपवटालमनकीकमीकहॳ कारणरतौधीहह८ताहहॴ 24-Jan-2017

Options

1) Vitamin K

पवटालमन K

2) Vitamin C

पवटालमन C

3) Vitamin B1

पवटालमन B1

4)Vitamin A

पवटालमन A

Correct Answer Vitamin A

Q142 Nongreen plants lack which of the

following

गहॴर-

हररतवनसपनतमनननननलिखितमसहॳककसकीकमीहह८तीहहॴ

24-Jan-2017

Options

1) Chlorophyll

किह८रह८कफि

2) Lycophyll

िायकह८कफि

3) Cyanophyll

F A C E B O O K

P A G E h t t p w w w f a c e b o o k c o m s s c m e n t o r s o f f i c i a l P a g e | 34

FOR MORE UPDATES AND MORE MATERIAL DO LIKE OUR FACEBOOK PAGE httpwwwfacebookcomsscmentorsofficial

सायनह८कफि

4)Phototropism

फह८टह८टरोपपजम

Correct Answer Chlorophyll

Q143 Organisms that use light to

prepare food are known as

जह८जीवपरकािकाउपयह८गकरभह८जनतहॴयारकरतहॳहहॴ उनह mdashmdash- कहॳ पमजानजाताहहॴ 24-Jan-2017

Options

1) Autotrophs

सवपह८षी 2) Heterotrophs

पवषमपह८षज

3) Omnivores

सवायहारी 4)Decomposers

पवघटनकरनहॳवािा Correct Answer Autotrophs

autotrophs often make their own food

by using sunlight carbon dioxide and

water to form sugars which they can use

for energy Some examples of

autotrophs include plants algae and

even some bacteria Autotrophs

(producer) are important because they

are a food source for heterotrophs

(consumers)

A heterotroph is an organism that

ingests or absorbs organic carbon

(rather than fix carbon from inorganic

sources such as carbon dioxide) in order

to be able to produce energy and

synthesize compounds to maintain its

life Ninety-five percent or more of all

types of living organisms are

heterotrophic including all animals and

fungi and some bacteria

Q144 Which of the following is a

primary function of haemoglobin

नननननलिखितमसहॳकह९नसाटहमह८गिह८बबनकाएकपरािलमककाययहहॴ

25-Jan-2017

Options

1) Utilization of energy

उजायकाउपयह८गकरना 2) Prevention of anaemia

रकतामपताहह८नहॳसहॳरह८कना 3) Destruction of bacteria

बहॴकटीररयाकापवनािकरना 4) To transport oxygen

ऑकसीजनकावहनकरना Correct Answer To transport oxygen

Q145 Vascular bundles are absent in

सवहनीबिि mdashmdash- मअनपनसतिरहतहॳहहॴ 25-Jan-2017

Options

1) Bryophyta

िायह८फाइटा 2) Pteridophyta

टहॳररिह८फाईटा 3) Gymnosperms

नजननह८सपमय 4) Angiosperms

एननजयह८सपहॳनसय Correct Answer Bryophyta

Q146 Sauria Lacertidae is the scientific

name of

सहॴररयािहॳसरटाईिी mdashmdashndash कावहॴजञाननकनामहहॴ 25-Jan-2017

Options

1) Crocodile

मगरमचछ

2) Hippopotamus

टहपपह८पह८टहॳमस

3) Lizard

नछपकिी 4) House fly

F A C E B O O K

P A G E h t t p w w w f a c e b o o k c o m s s c m e n t o r s o f f i c i a l P a g e | 35

FOR MORE UPDATES AND MORE MATERIAL DO LIKE OUR FACEBOOK PAGE httpwwwfacebookcomsscmentorsofficial

घरहॳिमकिी Correct Answer Lizard

Q147 Which type of pathogen causes

the water-borne disease SARS (Severe

Acute Respiratory Syndrome)

ककसपरकािकारह८गज़नकजिजननतबीमारीसासयकाकारणबनताहहॴ 25-Jan-2017

Options

1) Viral

वायरि

2) Parasitic

परजीवी 3) Protozoan

परह८टह८जअन

4) Bacterial

बहॴकटीररयि

Correct Answer Viral

Q148 Which of the following organs

produces the enzyme lipase

नननननलिखितमसहॳकह९नसाअगिायपहॳजएजाइमउतपननकरताहहॴ 25-Jan-2017

Options

1) Pancreas

अगनयािय

2) Large Intestine

बड़ीआत

3) Liver

नजगर

4) Small Intestine

छह८टीआत

Correct Answer Pancreas

Q149 A is a long internode forming the

basal part or the whole of a peduncle

एक mdashmdash- एकिबाइटरनह८िहहॴ जह८ननचिाटहससायासनपणयिठिबनताहहॴ 25-

Jan-2017

Options

1) Rhizome

परकद

2) Rachis

महॳ दि

3) floral axis

पषपअकष

4) Scape

भगदड़

Correct Answer scape

Q150 ndash Which of the following

organisms are considered to be both

Living and Non-living

नननननलिखितमसहॳकह९नसहॳजीवाणकह८जीपवतऔरअजीपवतमानाजाताहहॴ

25-Jan-2017

Options

1) Bacteria

बहॴकटीररया 2) Fungi

कवक

3) Algae

िहॴवाि

4)Virus

वायरस

Correct Answer Virus

They are considered to be living as they

possess a protein coat as a protective

covering DNA as the genetic material

etc

They are said to be non-living as they

can be crystallised and they survive for

billions of years They can tolerate high

temperatures freezing cold

temperatures ultra-violet radiations etc

Q151 Deficiency of fluorine causes

which of the following

फिह८ररनकीकमीकहॳ कारणनननननलिखितमसहॳकयाहह८ताहहॴ

F A C E B O O K

P A G E h t t p w w w f a c e b o o k c o m s s c m e n t o r s o f f i c i a l P a g e | 36

FOR MORE UPDATES AND MORE MATERIAL DO LIKE OUR FACEBOOK PAGE httpwwwfacebookcomsscmentorsofficial

27-Jan-2017

Options

1) Dental Caries

िटिकहॴ ररज

2) Scurvy

सकवरी 3) Anaemia

रकतामपता 4) Arthritis

गटठया Correct Answer Dental Caries

Q152 In a Punnett Square with the

cross AaBb x AaBb how many Aabb

genotypes would be created

पनहॳटसककायरमिह८स AaBb x AaBb कहॳ साि

ककतनहॳ Aabb जीनह८टाइपबनगहॳ 27-Jan-2017

Options

1) 1

2) 8

3) 2

4) 3

Correct Answer 2

Q153 Which of the following is the

Controlling Center of the Cell

नननननलिखित म सहॳ कह८लिकाका ननयतरण

क दर कह९न हहॴ

27-Jan-2017

Options

1) Nucleus

क दर

2) Plasma

पिाजमा 3) Lysosome

िायसह८सह८म

4) Chromosome

िह८मह८सह८म

Correct Answer Nucleus

The control centre of the cell is the

nucleus in eukaryotic cells The nucleus

contains genetic material in the form of

DNA

Q154 Myopia affects which of the

following organs

मायह८पपयानननननलिखितअगह८मसहॳककसहॳपरभापवतकरताहहॴ

25-Jan-2017

Options

1) Heart

हदय

2) Skin

तवचा 3) Eyes

आािहॳ 4)Mouth

मह

Correct Answer Eyes

Q155 Which of the following bears

flowers

नननननलिखितमसहॳकह९नफिधारणकरताहहॴ

25-Jan-2017

Options

1) Bryophyta

िायह८फाइटा 2) Pteridophyta

टहॳरीिह८फाईटा 3) Gymnosperms

नजननह८सपमय 4)Angiosperms

एननजयह८सपमय Correct Answer Angiosperms

Q156 Oxygenated blood flows out of the

heart through the

ऑकसीजनयकतरकत mdashmdashmdash

कहॳ माधयमसहॳहदयकहॳ बाहरबहताहहॴ 25-Jan-2017

F A C E B O O K

P A G E h t t p w w w f a c e b o o k c o m s s c m e n t o r s o f f i c i a l P a g e | 37

FOR MORE UPDATES AND MORE MATERIAL DO LIKE OUR FACEBOOK PAGE httpwwwfacebookcomsscmentorsofficial

Options

1) Aorta

महाधमनी 2) pulmonary artery

फहॳ फड़हॳकीधमनी 3) vena cava

वहॳनाकावा 4)Atrium

चह९क

Correct Answer aorta

Q157 Blood leaving the liver and

moving towards the

heart has a higher concentration of

नजगरसहॳननकिकरहदयकीतरफजानहॳवािहॳरकतम mdashmdashmdashmdash कीउचचसादरताहह८तीहहॴ 27-Jan-2017

Options

1) Lipids

लिपपडस

2) Urea

यररया 3) Bile Pigments

पपततकहॳ रगकरण

4) Carbon dioxide

काबयनिायऑकसाइि

Correct Answer Bile Pigments

Urea is nitrogen containing substance

which is produced in the liver in order

to deal with excess amino-acids in the

body As urea is produced it leaves the

liver in the blood stream and passes via

the circulatory system to all parts of the

body

Q158 Bulb is a modification of which

part of a plant

बमबएकपह९धहॳकहॳ ककसटहससहॳकाएक पातरणहह८ताहहॴ 27-Jan-2017

Options

1) The root

जड़

2) The stem

तना 3) The radicle

मिाकर

4)The fruit

फि

Correct Answer The stem

Q159 Which of the following carries

blood away from the heart to different

body parts

इनमहॳसहॳकह९नरकतकह८हदयसहॳिरीरकहॳ पवलभननअगह८तकिहॳजातीहहॴ

27-Jan-2017

Options

1) Arteries

धमननया 2) Nerves

तबतरहाए

3) Capillaries

कहॳ लिकाए

4)Veins

नसहॳ Correct Answer Arteries

Q160 The series of processes by which

nitrogen and its compounds are

interconverted in the environment and

in living organisms is called

27-Jan-2017

Options

1)Absorption of Nitrogen

2)Ammonification

3)Nitrogen Fixation

4)Nitrogen Cycle

Correct Answer Nitrogen Cycle

Ammonification or Mineralization is

performed by bacteria to convert

organic nitrogen to ammonia

F A C E B O O K

P A G E h t t p w w w f a c e b o o k c o m s s c m e n t o r s o f f i c i a l P a g e | 38

FOR MORE UPDATES AND MORE MATERIAL DO LIKE OUR FACEBOOK PAGE httpwwwfacebookcomsscmentorsofficial

Nitrification can then occur to convert

the ammonium to nitrite and nitrate

Nitrogen fixation is a process by which

nitrogen in the Earthrsquos atmosphere is

converted into ammonia (NH3) or other

molecules available to living organisms

Q161 BCG vaccine is given to protect

from which of the following

बीसीजीकाटटकानननननलिखितमसहॳककसकहॳ बचावकहॳ लिएटदयाजातहहॴ

27-Jan-2017

Options

1) Jaundice

पीलिया 2) Anaemia

रकतमपता 3) Tuberculosis

कषयरह८ग

4) Polio

पह८लियह८ Correct Answer Tuberculosis

Q162 Parallel venation is found in

समानतरवहॳनहॳिन mdashmdashmdash- मपायाजाताहहॴ 27-Jan-2017

Options

1) plants which are monocots

पह९धहॳजह८एकबीजपतरीहह८तहॳहहॴ 2) plants which have a dicot stem

वहॳपह९धहॳनजनकातनादपवदलियहह८ताहहॴ 3) plants with leaves similar to Tulsi

वहॳपह९धहॳनजनकीपनततयतिसीकीपनततयोकहॳ समानहह८तहॳहहॴ 4)plants with tap roots

टहॳप टवािहॳपह९धहॳ Correct Answer plants which are

monocots

Q163 The hardest part of the body is

िरीरकासबसहॳकठह८रभाग mdashndash हहॴ 27-Jan-2017

Options

1) Bones

हडडिय

2) Tooth Enamel

दातकहॳ इनहॳमि

3) Skull

िह८पड़ी 4) Spinal Cord

महॳ रजज

Correct Answer Tooth Enamel

Q164 Which type of pathogen causes

the waterborne disease E coli Infection

ककसपरकारकारह८गजननकजिजननतरह८गईकह८िाईसिमणकाकारणबनताहहॴ 27-Jan-2017

Options

1) Protozoan

परह८टह८जआ

2) Parasitic

परजीवी 3) Bacterial

बहॴकटीररयि

4)Viral

वायरि

Correct Answer Bacterial

Q165 The amount of blood filtered

together by both the kidneys in a 70 kg

adult male human in a minute is

70 की गरा वािहॳएकवयसकप षमएकलमनटमदह८नोगदकहॳदवाराएकसािचाबनीगयीरकतकीमातरहह८तीहहॴ 29-Jan-2017

Options

1) 1100 ml

1100 लमलि

2) 100 ml

F A C E B O O K

P A G E h t t p w w w f a c e b o o k c o m s s c m e n t o r s o f f i c i a l P a g e | 39

FOR MORE UPDATES AND MORE MATERIAL DO LIKE OUR FACEBOOK PAGE httpwwwfacebookcomsscmentorsofficial

100 लमलि

3) 1500 ml

1500 लमलि

4) 500 ml

500 लमलि

Correct Answer 1100 ml

Q166 Which feature of a plant helps to

distinguish a monocot from a dicot

पह९धहॳकीवहकह९नसीपविहॳषताहहॴजह८एकदपवदलियहॳऔरएकएकदिीयपह९धहॳसहॳभहॳदकरनहॳममददकरतीहहॴ 29-Jan-2017

Options

1) Pollination

परागम

2) Venation

वहॳनहॳिन

3) Vernation

वनिन

4) Aestivation

एसटीवहॳिहॳन

Correct Answer venation

Q167 The Mutation Theory was

proposed by

उतवररवतयनकालसदात mdashmdashndash

कहॳ दवरापरसतापवतककयाजाताहहॴ 29-Jan-2017

Options

1) Charles Lyell

चामसयलियहॳि

2) William Smith

पवलियमनसमि

3) Hugo De Vries

हयगह८िीराईस

4)Harrison Schmitt

हहॳरीसननसमट

Correct Answer Hugo De Vries

Q168 Which type of pathogen causes

the waterborne disease HepatitisA

ककसपरकारकहॳ रह८गजनकजिजननतरह८गहहॳपहॳटाइटटस-A काकारणबनताहहॴ

29-Jan-2017

Options

1) Parasitic

परजीवी 2) Viral

वायरि

3) Protozoan

परह८टह८जआ

4) Bacterial

बहॴकटीररयि

Correct Answer Viral

Q169 In a Punnett Square with the

cross AaBb x Aabb how many AaBb

genotypes would be created

पनहॳटसकवायरमिह८स AaBb x Aabb

कहॳ सािककतनहॳ AaBb जीनह८टाइपबनगहॳ 29-Jan-

2017

Options

1) 4

2) 1

3) 7

4) 6

Correct Answer 4

Q170 Arboreal Ateles is the scientific

name of

अिह८ररयिएटटलिस mdashmdashmdash कावहॴजञाननकनामहहॴ 29-Jan-2017

Options

1) Squirrel

चगिहरी 2) Sparrow

गह८रहॴया 3) Lizard

नछपकिी 4) Spider monkey

F A C E B O O K

P A G E h t t p w w w f a c e b o o k c o m s s c m e n t o r s o f f i c i a l P a g e | 40

FOR MORE UPDATES AND MORE MATERIAL DO LIKE OUR FACEBOOK PAGE httpwwwfacebookcomsscmentorsofficial

मकड़ीबदर

Correct Answer Spider monkey

Q171 Which type of pathogen causes

the waterborne disease Salmonellosis

ककसपरकारकारह८गाणजिजननतबीमारीसािमह८नहॳिह८लसज़काकारकहहॴ

29-Jan-2017

Options

1) Algal

िहॳवालियहॳ 2) Parasitic

परजीवी 3) Bacterial

बहॴकटीररयि

4)Viral

वायरि

Correct Answer Bacterial

An infection with salmonella bacteria

commonly caused by contaminated food

or water

Symptoms include diarrhoea fever

chills and abdominal pain

Q172 is a condition in which there is a

deficiency of red cells or of haemoglobin

in the blood

mdashmdash-

एकनसिनतहहॴनजसमहॳरकतमिािकह८लिकाओकीयाहीमह८गिह८बबनकीकमीहह८तीहहॴ 29-Jan-2017

Options

1) Albinism

एनमबननजम

2) Propyria

परह८पीररया 3) Anaemia

एनीलमया 4)Keloid disorder

कहॳ िह८इिडिसओिर

Correct Answer Anaemia

Q173 Ananas comosus is the scientific

name of

Options

अनानासकह८मह८सस mdashmdashmdashndash

कावहॴजञाननकनामहहॴ 29-Jan-2017

1) Custard Apple

सीताफि

2) Pineapple

पाइनएपपि

3) Bamboo

बास

4)Pomegranate

अनार

Correct Answer Pineapple

Q174 Which organ produces insulin

कह९नसाअगइनसलिनपहॴदाकरताहहॴ 29-Jan-

2017

Options

1) Liver

यकत

2) Thyroid gland

िायराइिगरिी 3) Spleen

पिीहा 4)Pancreas

अगरयिय

Correct Answer Pancreas

Q175 Which of the following disease is

not caused by water pollution

नननननलिखितमसहॳकह९नसारह८गपानीकहॳ परदषणकहॳकारणनहीहह८ता

29-Jan-2017

Options

1) Cholera

हहॴजा 2) Typhoid

F A C E B O O K

P A G E h t t p w w w f a c e b o o k c o m s s c m e n t o r s o f f i c i a l P a g e | 41

FOR MORE UPDATES AND MORE MATERIAL DO LIKE OUR FACEBOOK PAGE httpwwwfacebookcomsscmentorsofficial

टाइफाइि

3) Asthma

दमा 4)Diarrhoea

दसत

Correct Answer Asthma

Q176 Ocimum tenuiflorum is the

scientific name of

ओलिलममटहॳयईफिह८रमइसकावहॴजञाननकनाम mdash

ndash हहॴ 30-Jan-2017

Options

1) Neem

नीम

2) Mango

आम

3) Babul

बबि

4)Tulsi

तिसी Correct Answer Tulsi

Q177 Which gland secretes bile a

digestive fluid

कह९नसीगरिीपपतत एकपाचनतरिपरदािय सरापवतकरतीहहॴ 30-Jan-2017

Options

1) Pancreas

अगनयािय

2) Liver

यकत

3) Thyroid

िायराइि

4) Testes

टहॳनसटस

Correct Answer liver

Q178 In which of the following the

dominant phase is Gametophyte

नननननलिखितमसहॳककसकहॳ परमिचरणयगमकह८दपवधद (Gametophyte)हहॴ 30-Jan-2017

Options

1) Bryophyta

िायह८फाइटा 2) Pteridophyta

टहॳररिह८फाइटा 3) Gymnosperms

नजननह८सपमय 4) Angiosperms

एननजयह८सपमय Correct Answer Bryophyta

Q179 Anaerobic respiration refers to

which of the following

नननननलिखितमसहॳककसहॳअवायवीयशवसनकहाजाताहहॴ

30-Jan-2017

Options

1) Respiration without Oxygen

ऑकसीजनकहॳ बबनाशवसन

2) Respiration with Oxygen

ऑकसीजनकहॳ सािशवसन

3) Respiration without CO2

काबयनिायऑकसाइिकहॳ बबनाशवसन

4) Respiration with CO2

काबयनिायऑकसाइिकहॳ सािशविन

Correct Answer Respiration without

Oxygen

Q180 Which type of pathogen causes

the waterborne disease Cholera

ककसपरकारकारह८गजनकजिजननतरह८गहहॴजाकाकारणबनताहहॴ

30-Jan-2017

Options

1) Algal

िहॴवालियहॳ

F A C E B O O K

P A G E h t t p w w w f a c e b o o k c o m s s c m e n t o r s o f f i c i a l P a g e | 42

FOR MORE UPDATES AND MORE MATERIAL DO LIKE OUR FACEBOOK PAGE httpwwwfacebookcomsscmentorsofficial

2) Bacterial

बहॴकटीररयि

3) Protozoan

परह८टह८जआ

4) Viral

वायरि

Correct Answer Bacterial

Q181 To which class does

Oxyreductases transferases hydrolases

belong

ओकसीररिकटहॳसटरासफरहॳजहॳस

हाइडरह८िहॳसहॳसककसवगयमआतहॳहहॴ 30-Jan-2017

Options

1) Hormones

हारमोस

2) Enzymes

एजाइनस

3) Proteins

परह८टीनस

4) Vitamins

पवटालमनस

Correct Answer Enzymes

Q182 Which of the following is not true

about Gymnosperms

ननननमसहॳकह९नसीबातअनावतबीजीकहॳ बारहॳमसचनहीहहॴ 30-Jan-2017

Options

1) Dominant phase is saprophytes

परमिचरणसहॳपरह८फाइटसहह८ताहहॴ 2) Vascular bundles are absent

सवहनीबििअनपनसितहह८ताहहॴ 3) spores are heterospores

बीजाणहहॳटहॳरह८सपह८रसहह८तहॳहहॴ 4) Flowers are absent

फिअनपनसितहह८तहॳहहॴ

Correct Answer Vascular bundles are

absent

Q183 The name of first mammal clone sheep is

भहॳड़कीपरिमसतनपायीपरनत प (किह८न)

कानामहहॴ 30-Jan-2017

Options

1) Noori

नरी 2) Dolly

िॉिी 3) Louise

िसी 4)Durga

दगाय Correct Answer Dolly

Q184 Which type of pathogen causes

the water-borne disease Typhoid fever

ककसपरकारकारह८गजनकजिजननतरह८गटाइफाइिबिारकाकारणबनताहहॴ 30-Jan-2017

Options

1) Algal

िहॴवािीय

2) Parasitic

परजीवी 3) Protozoan

परह८टह८जनअन

4)Bacterial

बहॴकटीररयि

Correct Answer Bacterial

Q185 In which part of the cell are

proteins made

कह८लिकाकहॳ ककसटहससहॳमपरह८टीनबनायाजाताहहॴ

31-Jan-2017

Options

1) Reticulum

रहॳटटकिम

F A C E B O O K

P A G E h t t p w w w f a c e b o o k c o m s s c m e n t o r s o f f i c i a l P a g e | 43

FOR MORE UPDATES AND MORE MATERIAL DO LIKE OUR FACEBOOK PAGE httpwwwfacebookcomsscmentorsofficial

2) Golgi apparatus

गह८मजीएपहॳरहॳटस

3) Ribosomes

ररबह८सह८नस

4) Lysosome

िायसह८सह८नस

Correct Answer ribosomes

Proteins are produced by stringing

amino acids together in the order

specified by messenger RNA strands

that were transcribed from DNA in the

cell nucleus The process of synthesizing

a protein is called translation and it

occurs on ribosomes in the cytoplasm of

a cell

Q186 Polio is a disease caused by which

of the following

नननननलिखितमसहॳपह८लियह८कीबबमारह८हह८नहॳकाकारणकयाहहॴ

31-Jan-2017

Options

1) Bacteria

बहॴकटीररयि

2) Mosquito

मचछर

3) Virus

वायरस

4) Cockroach

नतिच हॳ Correct Answer Virus

Polio or poliomyelitis is a crippling and

potentially deadly infectious disease It

is caused by the poliovirus

Q187 ndash Hay fever is a sign of which of

the following

हहॳकफवरनननननलिखितमसहॳककसकाएकसकहॳ तहहॴ

31-Jan-2017

Options

1) Old Age

वदावसिा 2) Malnutrition

कपह८सण

3) Allergy

एिनजय 4) Over Work

अतयचधककाययकरना Correct Answer Allergy

Q188 How many chromosomes does a

human cell contain

एकमानवकह८लिकामककतनहॳगणसतरहह८तहॳहहॴ

29-Jan-2017

Options

1) 6

2) 26

3) 46

4) 66

Correct Answer 46

In humans each cell normally contains

23 pairs of chromosomes for a total of

46 Twenty-two of these pairs called

autosomes look the same in both males

and females The 23rd pair the sex

chromosomes differ between males and

females

Q189 Which of the following is not true

about Bryophyta

ननननमसहॳकह९नसीबातिायह८फाइटकहॳ बारहॳमसचनहीहहॴ 31-Jan-2017

Options

1) Dominant phase is gametophytes

परमिचरणगहॳलमतह८फाइटसहह८ताहहॴ 2) Main plant body is haploid

पह९धहॳकामखयिरीरअगखणतहह८ताहहॴ 3) Spores are homospores

बीजाणहह८मह८सफह८रसहह८तहॳहहॴ 4) Flowers are present

फिमह८जदहह८तहॳहहॴ Correct Answer Flowers are present

F A C E B O O K

P A G E h t t p w w w f a c e b o o k c o m s s c m e n t o r s o f f i c i a l P a g e | 44

FOR MORE UPDATES AND MORE MATERIAL DO LIKE OUR FACEBOOK PAGE httpwwwfacebookcomsscmentorsofficial

Q190 Which aquatic animal has

trailing tentacles

ककसजिीयजानवरकहॳ पीछहॳचिनहॳवािहॳटहॳटकिसहह८तहॳहहॴ

31-Jan-2017

Options

1) Sea horse

समदरीघह८िा 2) Corals

मगा 3) Jelly fish

जहॳिीमछिी 4) Star fish

तारामछिी Correct Answer Jelly fish

Jellyfish with its umbrella-shaped bell

and trailing tentacles

Q191 Which type of pathogen causes

the water-borne disease Poliomyelitis

(Polio)

ककसपरकारकारह८गजनकजिजननतरह८गपह८लियह८मायहॳटटस (पह८लियह८) काकारणहहॴ 31-Jan-

2017

Options

1) Parasitic

परजीवी 2) Algal

िहॴवालिय

3) Viral

वायरि

4) Bacterial

बहॴकटीररयि

Correct Answer Viral

Q192 The outer white part of the eye

that protects the inner structures is

आािकाबाहरीसफहॳ दटहससाजह८आतररकसरचनाओकीरकषाकरताहहॴ वह mdashmdashmdash हहॴ 31-Jan-

2017

Options

1) Iris

आयररस

2) Sclera

सकिहॳरा 3) Retina

रहॳटटना 4) Cornea

कह८ननयया Correct Answer Sclera

Q193 Proteins are made up of

परह८टीनकाननमायण mdashndash सहॳहह८ताहहॴ 31-Jan-2017

Options

1) Amino acids

एलमनह८अनि

2) Fatty acids

वसायकतअनि

3) Glucose

गिकह८ज

4)Nucleotides

नयनकियह८टाईिस

Correct Answer Amino acids

Q194 Moringa Oleifera is the scientific

name of

मह८ररगओलिफहॳ रा mdashmdashndash कावहॴजञाननकनामहहॴ 31-Jan-2017

Options

1) Banyan

बरगद

2) Gulmohar

गिमह८हर

3) Amla

आमिा

F A C E B O O K

P A G E h t t p w w w f a c e b o o k c o m s s c m e n t o r s o f f i c i a l P a g e | 45

FOR MORE UPDATES AND MORE MATERIAL DO LIKE OUR FACEBOOK PAGE httpwwwfacebookcomsscmentorsofficial

4) Drumstick

डरमनसटक

Correct Answer Drumstick

Q195 Kidney stones are composed of

गदकीपिरी mdashndash सहॳबनीहह८तीहहॴ 1-Feb-2017

Options

1) Calcium Oxalate

कहॴ नमसयमओकजहॳिहॳट

2) Sodium Chloride

सह८डियमकिह८राइि

3) Magnesium Nitrate

महॳनगनलियमनाइतटरहॳट

4) Calcium Bicarbonate

कहॴ नमियमबायकबोनहॳट

Correct Answer Calcium Oxalate

Q196 ndash Which of the following is not

true about Angiosperms

ननननमसहॳकह९नसीबातआवतबीजीकहॳ बारहॳमसचनहीहहॴ 1-Feb-2017

Options

1) Dominant phase is gametophytes

परमिचरणगहॳलमतह८फाइटहह८ताहहॴ 2) Vascular bundles are present

सवहनीबििमह९जदहह८ताहहॴ 3) Spores are heterospores

बीजाणहहॳटहॳरह८सपह८रसहह८तहॳहहॴ 4) Seeds are covered

बीजढकहॳ हह८तहॳहहॴ Correct Answer Dominant phase is

gametophytes

Q197 All of the following are excretory

(waste) products of animals except

नननननलिखितमसहॳककसएककह८छह८ड़करअनयसभीपराखणयोदवाराउतसनजयतपदाियहहॴ 1-Feb-

2017

Options

1) Uric Acid

यररकएलसि

2) Ammonia

अमह८ननया 3) Carbohydrates

काबोहाइडरहॳट

4) Urea

यररया Correct Answer Carbohydrates

In animals the main excretory products

are carbon dioxide ammonia (in

ammoniotelics) urea (in ureotelics) uric

acid (in uricotelics) guanine (in

Arachnida) and creatine

Q198 RNA is a polymeric molecule

What does RNA stand for

आरएनइएएकबहिकआणहहॴ इसकाकापवय पकयाहहॴ 1-Feb-2017

Options

1) Rado Nuclear Acid

रािह८नयनकियरएलसि

2) Ribo Nucleic Acid

राइबह८नयनकिकएलसि

3) Rhino Nuclear Acid

हाइनह८नयनकियरएलसि

4) Resto Nucleus Acid

रहॳसटह८नयकिीयसएलसि

Correct Answer Ribo Nucleic Acid

Q199 Which organ does detoxification

and produces chemicals needed for

digestion

कह९नसाअगपवषहरणकरताहहॴऔरपाचनकहॳ लिएआवशयकरसायनोकह८पहॴदाकरताहहॴ 1-Feb-

2017

Options

1) Salivary glands

िारगरचिया 2) Pancreas

अगनयािय

F A C E B O O K

P A G E h t t p w w w f a c e b o o k c o m s s c m e n t o r s o f f i c i a l P a g e | 46

FOR MORE UPDATES AND MORE MATERIAL DO LIKE OUR FACEBOOK PAGE httpwwwfacebookcomsscmentorsofficial

3) Thyroid gland

िायराइिगरिी 4) Liver

यकत

Correct Answer Liver

Q200 Psidium guajava is the scientific

name of

लसडियमगआजावा mdashmdash कावहॴजञाननकनामहहॴ 1-

Feb-2017

Options

1) Guava

अम द

2) Mango

आम

3) Bamboo

बास

4) Jack fruit

कटहि

Correct Answer Guava

Q201 Which drug is used as a Blood

Thinner

चधरकह८पतिाकरनहॳकहॳ पमककसदवाकापरयह८गककयाजाताहहॴ

1-Feb-2017

Options

1) Warfarin

वाफर न

2) Tramadol

टरहॳमािह८ि

3) Azithromycin

एनजरह८मायलसन

4) Hydralazine

हाइडरह८िहॳनजन

Correct Answer Warfarin

Q202 Which of the following disease is

caused due to the deficiency of protein

परह८टीनकीकमीकहॳ कारणनननननलिखितमसहॳकह९नसारह८गहह८ताहहॴ 1-Feb-2017

Options

1) Arthritis

गटठया 2) Kwashiorkor

कािीओकय र

3) Goitre

गाइटर

4) Night Blindness

रतह९चध

Correct Answer Kwashiorkor

Q203 A is species of plant that has

adapted to survive in an environment

with little liquid water

mdashmdashndashपह९धहॳकीएकऐसहॳऐसहॳपरजानतहहॴ नजसनहॳकमपानीवािहॳवातावरणमजीपवतरहनहॳकहॳलिएअनकिनहहॴ 1-Feb-2017

Options

1) Xerophyte

म दपवद

2) Hydrophyte

जिीयपादप

3) Mesophyte

समह८दपवद

4) Thallophyte

िहॴिह८फाइटा Correct Answer xerophyte

xerophyte is a species of plant that has

adapted to survive in an environment

with little liquid water such as a desert

or an ice- or snow-covered region in the

Alps or the Arctic

Mesophytes are terrestrial plants which

are adapted to neither a particularly

dry nor particularly wet environment

An example of a mesophytic habitat

would be a rural temperate meadow

F A C E B O O K

P A G E h t t p w w w f a c e b o o k c o m s s c m e n t o r s o f f i c i a l P a g e | 47

FOR MORE UPDATES AND MORE MATERIAL DO LIKE OUR FACEBOOK PAGE httpwwwfacebookcomsscmentorsofficial

which might contain goldenrod clover

oxeye daisy and Rosa multiflora

thallophyte any of a group of plants or

plantlike organisms (such as algae and

fungi) that lack differentiated stems

leaves and roots and that were formerly

classified as a primary division

(Thallophyta) of the plant kingdom

Q204 How many types of teeth are

there in humans

मनषयोमककतनहॳपरकारकहॳ दातहह८तहॳहहॴ

1-Feb-2017

Options

1) 4

2) 5

3) 2

4) 3

Correct Answer 4

teeth -Humans have four types of

teethincisors canines premolars and

molars each with a specific function

The incisors cut the food the canines

tear the food and the molars and

premolars crush the food

Q205 Carica papaya is the scientific name of

कहॴ ररकापपाया mdashmdashndash कावहॴजञाननकनामहहॴ 2-

Feb-2017

Options

1) Peepal

पीपि

2) Papaya

पपीता 3) Tamarind

इमिी 4) Drumstick

ढह८िकाछड़ी Correct Answer Papaya

Q206 Muscles get tired when there is

shortfall of

जब mdashndash कीकमीहह८तीहहॴतबपहॳिीयिकजातीहहॴ 2-Feb-2017

Options

1) Lactic acid

िहॴनकटकएलसि

2) Na+ ions

Na+ आयन

3) ATP

एटीपी 4) Sulphates

समफहॳ टस

Correct Answer ATP

ATP is the energy source muscle fibers

use to make muscles contract

muscle tissuersquos main source of energy

called adenosine triphosphate or ATP

As your muscles use up this energy

source they become tired and fatigued

Oxygen is the key ingredient that helps

create new ATP to replenish the burned

up ATP in your muscles

Q207 Artocarpus integra is the

scientific name of आटह८कापयसइटीगरा mdashmdashmdash कावहॴजञाननकनामहहॴ 2-Feb-2017

Options

1) Guava

अम द

2) Pineapple

अनानास

3) Silver Oak

लसमवरओक

4) Jack fruit

कटहि

Correct Answer Jack fruit

Q208 Which organ stores fat soluble

vitamins

कह९नसाअगवसामघिनिीिपवटालमनह८काभिाराकरताहहॴ

2-Feb-2017

F A C E B O O K

P A G E h t t p w w w f a c e b o o k c o m s s c m e n t o r s o f f i c i a l P a g e | 48

FOR MORE UPDATES AND MORE MATERIAL DO LIKE OUR FACEBOOK PAGE httpwwwfacebookcomsscmentorsofficial

Options

1) Blood

रकत

2) Skin

तवचा 3) Liver

यकत

4) Pancreas

अगनयािय

Correct Answer Liver

Q209 Which disease is caused due to

deficiency of Iodine

आयह८िीनकहॳ कारणकह९नसारह८गहह८ताहहॴ 2-Feb-2017

Options

1) Rickets

ररकहॳ टस

2) Scurvy

सकवी 3) Goitre

गणमािा 4) Growth retardation

पवकासका कना Correct Answer Goitre

rickets A softening and weakening of

bones in children usually due to

inadequate vitamin D

Q210 Grevillea Robusta is the scientific name of

गरहॳपवलियारह८बसटा mdashmdashmdash- कापवजञाननकनामहहॴ 2-Feb-2017

Options

1) Peepal

पीपि

2) Teak

सागह९न

3) Silver Oak

लसमवरओक

4) Jack fruit

कटहि

Correct Answer Silver Oak

Q211 When a Cuttlefish is described as a Molluscs it is at which level of

classification

जबएककटिकफिकह८एकमह८िसकाकहॳ पमवखणयतककयाजाताहहॴतबयहॳवगीकरणकहॳ ककससतरपहॳनसितहहॴ 2-Feb-2017

Options

1) Class

वगय 2) Order

िम

3) Family

पररवार

4) Phylum

सघ

Correct Answer Phylum

Q212 Bambusa dendrocalmus is the

scientific name of बानबसािहॳडराकामस mdashmdashmdash कावहॴजञाननकनामहहॴ 3-Feb-2017

Options

1) Banyan

बरगद

2) Papaya

पपीता 3) Bamboo

बास

4) Pomegranate

अनार

Correct Answer Bamboo

Q213 Acinonyx Jubatus is the scientific name of

एलसनह८ननकसजयबहॳटस mdashmdashmdash

कावहॴजञाननकनामहहॴ 3-Feb-2017

F A C E B O O K

P A G E h t t p w w w f a c e b o o k c o m s s c m e n t o r s o f f i c i a l P a g e | 49

FOR MORE UPDATES AND MORE MATERIAL DO LIKE OUR FACEBOOK PAGE httpwwwfacebookcomsscmentorsofficial

Options

1) Bear

भाि 2) Horse

घह८िा 3) Cheetah

चीता 4) Zebra

जहॳिा Correct Answer Cheetah

Q214 The pale yellow colour of urine is

due to the presence of which pigment

मतरकाफीकापीिारगरगदरयकहॳ उपनसिनतकहॳ कारणहह८ताहहॴ

3-Feb-2017

Options

1) Urochrome

यरह८िह८म

2) Urophyll

यरह८कफि

3) Chlorophyll

किह८रह८कफि

4) Chloroplast

किह८रह८पिासट

Correct Answer Urochrome

Q215 Which of the following constitute

to form a gene

नननननलिखितमसहॳकह९नसीचीज़एकजीनकागठनकरतीहहॴ

3-Feb-2017

Options

1) Polynucleotides

पह८िीनयनकियह८टाईडस

2) Hydrocarbons

हाइडरह८काबोस

3) Lipoproteins

िाईपह८परह८टीनस

4) Lipids

लिपपडस

Correct Answer Polynucleotides

Polynucleotide molecule is a biopolymer

composed of 13 or more nucleotide

monomers covalently bonded in a chain

DNA (deoxyribonucleic acid) and RNA

(ribonucleic acid) are examples of

polynucleotides with distinct biological

function

Q216 Vertebrates belongs to the

phylum

रीढ़कीहडिीवािहॳपराणी mdashmdashmdash

परजानतकहॳ अतगायतआतहॳहहॴ 3-Feb-2017

Options

1) Arthropoda

आरह८पह८ड़ा 2) Annelida

एननलििा 3) Cnidaria

ननिहॳररया 4) Chordata

कह८िटा Correct Answer Chordata

Q217 Punica granatum is the scientific name of

पननकगरहॳनहॳटस mdashmdashmdash कावहॴजञाननकनामहहॴ 3-Feb-2017

Options

1) Custard Apple

सीताफि

2) Gulmohar

गिमह८हर

3) Silver Oak

लसमवरओक

4) Pomegranate

अनार

Correct Answer Pomegranate

F A C E B O O K

P A G E h t t p w w w f a c e b o o k c o m s s c m e n t o r s o f f i c i a l P a g e | 50

FOR MORE UPDATES AND MORE MATERIAL DO LIKE OUR FACEBOOK PAGE httpwwwfacebookcomsscmentorsofficial

Q218 Between a tiger and an monkey

which of the following is different

एकबाघऔरबदरकहॳ बीचनननननलिखितमसहॳकह९नसीबातअिगहहॴ 3-Feb-2017

Options

1) Kingdom

राजय

2) Phylum

जानत

3) Order

िम

4) Class

वगय Correct Answer order

Q219 The artificial heart was invented by

कबतरमहदयका mdashmdashmdash

दवाराअपवषकारककयागयािा 3-Feb-2017

Options

1) Muhammad Yunus

महनमदयनस

2) Linus Yale Jr

िाइनसयहॳिजय

3) Gazi Yasargil

गाजीयासचगयि

4) Paul Winchell

पह९िपवमकि Correct Answer Paul Winchell

Q220 Tamarindus indica is the

scientific name of

टहॳमररनडसइडिका mdashmdash कावहॴजञाननकनामहहॴ 7-

Feb-2017

Options

1) Neem

नीम

2) Pineapple

अनानास

3) Tamarind

इमिी 4)Chiku

चीक

Correct Answer Tamarind

Q221 In eukaryotic cells synthesis of

RNA takes place in the

यकहॳ योटटककह८लिकाओमआरएनएकासशिहॳषण

mdashndash महह८ताहहॴ 7-Feb-2017

Options

1) Mitochondria

माईटह८कोडडरया 2) Centrioles

सटरीयह८मस

3) Ribosomes

ररबह८सह८नस

4) Nucleus

नयनकियस

Correct Answer nucleus

eukaryotic cell -Transcription is the

process of synthesizing ribonucleic acid

(RNA)Synthesis takes place within the

nucleus of eukaryotic cells or in the

cytoplasm of prokaryotes and converts

the genetic code from a gene in

deoxyribonucleic acid ( DNA ) to a

strand of RNA that then directs

proteinsynthesis

Q222 _________is caused by parasites

of the Plasmodium genus

पिाजमह८डियमजातीकहॳ परजीवी mdash- कहॳ कारणहहॴ 7-Feb-2017

Options

1) Dysentery

पहॳचचि

2) Malaria

मिहॳररया 3) Chickenpox

F A C E B O O K

P A G E h t t p w w w f a c e b o o k c o m s s c m e n t o r s o f f i c i a l P a g e | 51

FOR MORE UPDATES AND MORE MATERIAL DO LIKE OUR FACEBOOK PAGE httpwwwfacebookcomsscmentorsofficial

चहॳचक

4) Herpes

हहॳपपयस

Correct Answer Malaria

Q223 Carotene in fruits and vegetables

gives it which color

फिह८औरसनलजयोमनसितकहॳ रह८टीनउनहकह९नसारगपरदानकरताहहॴ 7-Feb-2017

Options

1) Green

हरा 2) Pink

गिाबी 3) Orange

नारगी 4) Blue

नीिा Correct Answer Orange

Q224 Equus Caballus is the scientific

name of

एकवसकहॴ बहॳिस mdashmdashndash कापवजञाननकनामहहॴ 7-Feb-2017

Options

1) Horse

घह८िा 2) Zebra

ज़हॳिा 3) Donkey

गधा 4) Buffalo

भस

Correct Answer Horse

Q225 Elapidae Naja is the scientific name of

एिीपीिीनाजा mdashmdash- कावहॴजञाननकनामहहॴ 8-Feb-2017

Options

1) Cobra

कह८बरा 2) Elephant

हािी 3) Eagle

ग ि

4) Owl

उमि Correct Answer Cobra

Q226 Which disease is caused due to

deficiency of Iron

िह८हकीकमीकहॳ कारणकह९नसारह८गहह८ताहहॴ 8-Feb-

2017

Options

1) Beriberi

बहॳरीबहॳरी 2) Tetany

टहॳटनी 3) Kwashiorkor

कवािीऔरकर

4) Anaemia

रकतामपता Correct Answer Anaemia

Beriberi is a disease caused by a vitamin

B-1 deficiency also known as thiamine

deficiency

Tetany can be the result of an

electrolyte imbalance Most often itrsquos a

dramatically low calcium level also

known as hypocalcemia Tetany can also

be caused by magnesium deficiency or

too little potassium Having too much

acid (acidosis) or too much alkali

(alkalosis) in the body can also result in

tetany

Kwashiorkor also known as

ldquoedematous malnutrition It is a form of

malnutrition caused by a lack of protein

in the diet

Anaemia means that you have fewer red

blood cells than normal or you have less

F A C E B O O K

P A G E h t t p w w w f a c e b o o k c o m s s c m e n t o r s o f f i c i a l P a g e | 52

FOR MORE UPDATES AND MORE MATERIAL DO LIKE OUR FACEBOOK PAGE httpwwwfacebookcomsscmentorsofficial

haemoglobin than normal in each red

blood cell

Q227 is a leaf where the leaflets are

arranged along the middle vein

mdashndashएकपततीहहॴजहापतरकह८कीरचनाक ररयालिराकहॳ आसपासहह८तीहहॴ 8-Feb-2017

Options

1) Pinnately compound leaf

पपनहॳटिीसयकतपतती 2) Palmately compound leaf

पामहॳटिीसयकतपतती 3) Compound leaf

सयकतपतती 4) Simple leaf

साधारणपतती Correct Answer Pinnately compound

leaf

Q228 Haustoria or sucking roots are

found in which of the following

हह८सटह८ररयायाचसनहॳवािीजड़हॳनननननलिखितमसहॳककसमपाईजातीहहॴ 8-Feb-2017

Options

1) Wheat

गहॳह

2) Mango

आम

3) Chestnut

चहॳसटनट

4) Cuscuta

कसकयटा Correct Answer Cuscuta

Haustorial roots -The roots of parasitic

plants which penetrate into the host

tissues to absorb nourishment are

called haustorial roots hellip Also known as suckingor parasitic roots

Q229 Equs Asinus is the scientific name

of

एकवसएलसनस mdashmdashndash कावहॴजञाननकनामहहॴ 8-

Feb-2017

Options

1) Donkey

गधा 2) Cow

गाय

3) Deer

टहरन

4) Kangaroo

कगा

Correct Answer Donkey

Q230 Ficus benghalensis is the scientific name of

फाईकसबहॳनगहॳिहॳलसस mdashndash कापवजञाननकनामहहॴ 8-Feb-2017

Options

1) Banyan

बरगद

2) Pineapple

अनानास

3) Babul

बबि

4) Tulsi

तिसी Correct Answer Banyan

Q231 Equus burchellii is the scientific name of

एकवसबचिी mdashmdash- कापवजञाननकनामहहॴ 8-Feb-2017

Options

1) Horse

घह८िा 2) Zebra

जहॳिा 3) Buffalo

F A C E B O O K

P A G E h t t p w w w f a c e b o o k c o m s s c m e n t o r s o f f i c i a l P a g e | 53

FOR MORE UPDATES AND MORE MATERIAL DO LIKE OUR FACEBOOK PAGE httpwwwfacebookcomsscmentorsofficial

भस

4) Ass

गधा Correct Answer Zebra

Page 9: COMPILATION OF ALL 72 SETS OF BIOLOGY SSC CHSL-2016 · OF BIOLOGY SSC CHSL-2016 PREPARED BY : SSC MENTORS BIOLOGY SPECIAL . F A C E B O O K P A G E : h t t p : / / w w w . f a c e

F A C E B O O K

P A G E h t t p w w w f a c e b o o k c o m s s c m e n t o r s o f f i c i a l P a g e | 8

FOR MORE UPDATES AND MORE MATERIAL DO LIKE OUR FACEBOOK PAGE httpwwwfacebookcomsscmentorsofficial

3) Sperm

ििाण

4) Seed

बीज

Correct Answer Spore

Q28 Bacteria was discovered by

बहॴकटीररयाकीिह८जककसकहॳ दवाराकीगयीिी

10-Jan -2017

1) Antonie van Leeuwenhoek

एटह८नीवहॳनलिबहॳनहक

2) Belarus

बहॳिा स

3) Hugo de Vries

हयगह८दीराईस

4)Robert Brown

रॉबटयिाउन

Correct Answer Antonie van

Leeuwenhoek

Q29 Which of the following is

responsible for Vermicomposting

नननननलिखितमसहॳकह९नकलमिादकहॳ लिएनजनमहॳदारहहॴ

10-Jan -2017

Options

1) Fungus

कवक

2) Worms

कलम

3) Bacteria

बहॴकटीररया 4) Birds

पकषी Correct Answer Worms

Vermicompost (or vermi-compost) is the

product of the composting process using

various species of worms usually red

wigglers white worms and other

earthworms to create a heterogeneous

mixture of decomposing vegetable or

food waste bedding materials and

vermicast

Q30 Scurvy (bleeding of gums) is

caused by the deficiency of which

vitamin

सकवी (मसढह८सहॳिनआना) ककसपवटालमनकीकमीकहॳ कारणहह८ताहहॴ

10-Jan-2017

Options

1) Vitamin K

पवटालमन K

2) Vitamin BZ

पवटालमन BZ

3) Vitamin C

पवटालमन C

4) Vitamin A

पवटालमन A

Correct Answer Vitamin C

Q31 Achras sapota is the scientific

name of

एिाससपह८ताइसकावहॴजञाननकनामहहॴ 10-Jan-2017

Options

1) Custard Apple

सीताफि

2) Gulmohar

गिमह८हर

3) Tamarind

इमिी 4) Chiku

चचक

Correct Answer Chiku

Q32 Prawn belongs to the phylum

झीगा mdashmdash- परजानतकहॳ अतगयतआताहहॴ 10-Jan-2017

Options

1) Arthropoda

F A C E B O O K

P A G E h t t p w w w f a c e b o o k c o m s s c m e n t o r s o f f i c i a l P a g e | 9

FOR MORE UPDATES AND MORE MATERIAL DO LIKE OUR FACEBOOK PAGE httpwwwfacebookcomsscmentorsofficial

अरोपह८िा 2) Cnidaria

नीिहॳररया 3) Echinodermata

इकाईनह८िमटा 4) Chordata

कह८िटा Correct Answer Arthropoda

Q33 Pulses are a rich source of which of

the following

दािहॳनननननलिखितमसहॳककसकीपरचरसह८तरहहॴ

11-Jan-2017

Options

1) Carbohydrates

काबोहाइडराईट

2) Proteins

परह८टीनस

3) Minerals

िननज

4) Vitamin A

पवटालमन A

Correct Answer Proteins

Q34 Plant cell wall is made up of

वनसपनतकह८लिकालभनततइससहॳबनीहह८तीहहॴ

11-Jan-2017

Options

1) Cellulose

सहॳमयिह८ज

2) Glucose

गिकह८ज

3) Sucrose

सिह८ज

4) Fructose

फरकटह८ज

Correct Answer Cellulose

Plant cell wall the major carbohydrates

are cellulose hemicellulose and pectin

The cellulose microfibrils are linked via

hemicellulosic tethers to form the

cellulose-hemicellulose network which

is embedded in the pectin matrix

Q35 The study of Fungi is also known

as कवकह८कहॳ अधययनकह८कहाजाताहहॴ

11-Jan-2017

Options

1) Cytology

सायटह८िह८जी 2) Myology

मायह८िह८जी 3) Mycology

मायकह८िह८जी 4) Neurology

नयरह८िह८जी Correct Answer Mycology

Cytology - structure and function of

plant and animal cells

Myology is the study of the muscular

system

Neurology is the branch of medicine

concerned with the study and treatment

of disorders of the nervous system

Q36 The outermost layer of skin is

तवचाकीसबसहॳबाहरीपरतकयाहह८तीहहॴ 11-Jan-

2017

Options

1) Epidermis

इपपिलमयस

2) Dermis

िलमयस

3) Tissues

ऊतक

4) Hypodermis

हायपह८िलमयस

Correct Answer Epidermis

Q37 Which of the following plants have

root nodules

F A C E B O O K

P A G E h t t p w w w f a c e b o o k c o m s s c m e n t o r s o f f i c i a l P a g e | 10

FOR MORE UPDATES AND MORE MATERIAL DO LIKE OUR FACEBOOK PAGE httpwwwfacebookcomsscmentorsofficial

नननननलिखितपह९धह८मसहॳककसकीजड़ह८मगाठहह८तीहहॴ

11-Jan-2017

Options

1) Leguminous plants

िहॳगयलमनसपह९धहॳ 2) Parasitic plants

परजीवीपह९धहॳ 3) Epiphytic Plants

एपीफाइटटकपह९धहॳ 4) Aquatic Plants

जिीयपह९धहॳ Correct Answer Leguminous plants

Q38 Earth-worms belongs to the

phylum

कहॳ चएmdashmdash- परजानतकहॳ अतगयतआतहॳहहॴ 11-Jan-2017

Options

1) Protozoa

परह८टह८जआ

2) Cnidaria

नीिहॳररया 3) Annelida

एनीलििा 4) Mollusca

मह८िसका Correct Answer Annelida

Q39 Ringworm is a disease caused by

ररगवमयनामकबीमारी mdashmdash- कहॳ कारणहह८तीहहॴ 11-Jan-2017

Options

1) Fungi

कवक

2) Bacteria

बहॴकटीररया 3) Virus

वायरस

4) Flies

मनकियाा Correct Answer Fungi

Q40 Mangifera indica is the scientific

name of

मननगफहॳ राइडिकाककसकावहॴजञाननकनामहहॴ 11-

Jan-2017

Options

1) Guava

अम द

2) Mango

आम

3) Amla

आविा 4) Jack fruit

कटहि

Correct Answer Mango

Q41 Crabs belongs to the phylum

कहॳ कड़हॳmdashmdash- परजानतकहॳ अतगयतआतहॳहहॴ 11-Jan-2017

Options

1) Mollusca

मह८िसका 2) Cnidaria

नीिहॳररया 3) Arthropoda

अरोपह८ड़ा 4) Platyhelminthes

पिहॳटटहहॳनममननिस

Correct Answer Arthropoda

Q42 Myopia is a defect of eyes which is

also known as

मायह८पपयाआिोकादह८षहहॴ नजसहॳ mdashmdashndash

भीकहाजाताहहॴ

12-Jan-2017

Options

1) Far Sightedness

F A C E B O O K

P A G E h t t p w w w f a c e b o o k c o m s s c m e n t o r s o f f i c i a l P a g e | 11

FOR MORE UPDATES AND MORE MATERIAL DO LIKE OUR FACEBOOK PAGE httpwwwfacebookcomsscmentorsofficial

दरदनषटदह८ष

2) Near Sightedness

ननकटदनषटदह८ष

3) Astigmatism

एसटीगमहॳटटजम

4) Night Blindness

रतोधी Correct Answer Near Sightedness

Myopia occurs when the eyeball is too

long relative to the focusing power of

the cornea and lens of the eye This

causes light rays to focus at a point in

front of the retina rather than directly

on its surface

Hyperopia Hypermetropia (

Farsightedness )- when light rays

entering the eye focus behind the retina

rather than directly on it The eyeball of

a farsighted person is shorter than

normal

Astigmatism usually is caused by an

irregularly shaped cornea Instead of

the cornea having a symmetrically

round shape (like a baseball) it is

shaped more like an American football

Nyctalopia also called night ndash blindness

is a condition making it difficult or

impossible to see in relatively low light

Q43 Who is known as the father of

Green Revolution

हररतिानतकहॳ जनककहॳ पमककसहॳजानाजाताहहॴ

12-Jan-2017

1) Dr Robert Nucleus

िॉ रॉबटयनयनकियस

2) Dr Ian Wilmut

िॉ इयानपविमट

3) Dr NE Borlaug

िॉ एनईबह८रिॉग

4) Dr JC Bose

िॉ जहॳसीबह८स

Correct Answer Dr NE Borlaug

Q44 Panthera Tigris is the scientific

name of

पिहॳराटटगरीस mdashmdashmdash कावहॴजञाननकनामहहॴ 12-Jan-2017

Options

1) Panther

तदआ

2) Tiger

बाघ

3) Whale

हहॳि

4)Goat

बकरी Correct Answer Tiger

Q45 How many facial bones are there

हमारहॳचहॳहरहॳमककतनीहडडियााहह८तीहहॴ 13-Jan-2017

Options

1)34

2)24

3)14

4)4

Correct Answer 14

Q46 ndash Halophytes are plants that grow

in

हहॴिह८फाईटसवहॳपह९धहॳहह८तीहहॴजह८ mdash- मउगतहॳहहॴ SSC CHSL Science (biology) 2016

Question Paper

13-Jan-2017

Options

1) Fresh Water

ताजापानी 2) Cold Water

ठिापानी 3) Ponds

तािाब

4) Salt Water

िारापानी Correct Answer Salt Water

F A C E B O O K

P A G E h t t p w w w f a c e b o o k c o m s s c m e n t o r s o f f i c i a l P a g e | 12

FOR MORE UPDATES AND MORE MATERIAL DO LIKE OUR FACEBOOK PAGE httpwwwfacebookcomsscmentorsofficial

Q47 Felis Catus is the scientific name of

फहॳ लिसकहॴ टस mdashndash कावहॴजञाननकनामहहॴ 13-Jan-2017

Options

1) Cat

बबमिी 2) Dog

कतता 3) Mouse

चहा 4) Porcupine

साही Correct Answer Cat

Q48 Which of the following induces

nitrogen fixation in soil

नननननलिखितमसहॳकह९नलम ीमनाइटरह८जनननयतनकह८परहॳररतकरताहहॴ

15-Jan-2017

Options

1) Protozoa

परह८टह८जआ

2) Bacteria

बहॴकटीररया 3) Fungi

कवक

4)Algae

िहॴवाि

Correct Answer Bacteria

Bacteria that change nitrogen gas from

the atmosphere into solid nitrogen

usable by plants are called nitrogen-

fixing bacteria These bacteria are

found both in the soil and in symbiotic

relationships with plants

They contain symbiotic bacteria called

rhizobia within nodules in their root

systems producing nitrogen compounds

that help the plant to grow and compete

with other plants When the plant dies

the fixed nitrogen is released making it

available to other plant

Q49 Which of the following is the

largest known cell

नननननलिखितमसहॳकह९नसीसबसहॳबड़ीजञातकह८लिकाहहॴ

SSC CHSL Science (biology) 2016

Question Paper

15-Jan-2017

1) Eukaryotic Cell

यकहॳ ररयह८टटककह८लिका 2) Prokaryotic Cell

परह८कहॳ ररयह८टटककह८लिका 3) Mycoplasma

मायकह८पिासम

4) Ostrich Eggs

ितरमगयकाअिा Correct Answer Ostrich Eggs

Q50 The association of animals in

which both the partners are benefitted

is known as

जानवरोकावहसहयह८गनजसमहॳदह८नोभागीदारिाभापवनतहह८तहॳहहॴ उसहॳ mdashmdashndash कहॳ पमजानाजाताहहॴ SSC CHSL Science (biology) 2016

Question Paper

15-Jan-2017

Options

1) Amensalism

सहजीपवत

2) Commensalism

परजीपवत

3) Colony

कॉिनी 4) Mutualism

अनयह८नयाशरयवाद

Correct Answer Mutualism

Amensalism is any relationship between

organisms of different species in which

F A C E B O O K

P A G E h t t p w w w f a c e b o o k c o m s s c m e n t o r s o f f i c i a l P a g e | 13

FOR MORE UPDATES AND MORE MATERIAL DO LIKE OUR FACEBOOK PAGE httpwwwfacebookcomsscmentorsofficial

one organism is inhibited or destroyed

while the other organism remains

unaffected

Commensalism an association between

two organisms in which one benefits and

the other derives neither benefit nor

harm

Q51 Pneumonia affects which of the

following organs of human body

ननमह८ननयामानविरीरकहॳ नननननलिखितमसहॳककसअगकह८परभापवतकरताहहॴ

15-Jan-2017

Options

1)Kidneys

गद

2)Lungs

फहॳ फड़हॳ 3) Throat

गिहॳ 4) Liver

यकत

Correct Answer Lungs

When the germs that cause pneumonia

reach your lungs the lungsrsquo air sacs

(alveoli) become inflamed and fill up

with fluid This causes the symptoms of

pneumonia such as a cough fever

chills and trouble breathing When you

have pneumonia oxygen may have

trouble reaching your blood

Q52 Mendel is known as

मििकह८ mdashmdash- कहॳ पमजानाजाताहहॴ 15-Jan-2017

Options

1) Father of Physiology

िरीरकियािासतरकहॳ जनक

2) Father of Geology

भगभयिासतरकहॳ जनक

3) Father of Genetics

जहॳनहॳटटकसकहॳ जनक

4) Father of Biology

जीविासतरकहॳ जनक

Correct Answer Father of Genetics

Q53 Which of the following are also

known as Suicidal bag of Cells

ननननलिखितमसहॳककसहॳआतमहतयाकरनहॳवािीकह८लिकाओकाबहॴगकहाजाताहहॴ

15-Jan-2017

Options

1) Lysosomes

िायसोसह८म

2) Lycosome

िायकह८सह८म

3) Nucleus

नालभक

4) Chromosome

िह८मह८सह८म

Correct Answer Lysosomes

Q54 Mesothelioma is a type of cancer

The most common area affected in it is

the lining of the ________

लमज़ह८िहॳिहॳलमयाक सरकाएकपरकारहहॴ इससहॳपरभापवतहह८नहॳवािासबसहॳसामानयकषहॳतर mdash

mdashmdash काअसतरहहॴ 15-Jan-2017

Options

1)Heart

हदय

2)Brain

मनसतषक

3)Stomach

आमािय

4)Lungs

फहॳ फड़हॳ Correct Answer lungs

Asbestos exposure is the main cause of

pleural mesothelioma When asbestos

fibers are breathed in they travel to the

F A C E B O O K

P A G E h t t p w w w f a c e b o o k c o m s s c m e n t o r s o f f i c i a l P a g e | 14

FOR MORE UPDATES AND MORE MATERIAL DO LIKE OUR FACEBOOK PAGE httpwwwfacebookcomsscmentorsofficial

ends of small air passages and reach the

pleura where they can cause

inflammation and

scarring

Q55 Which one of the following is an

insectivorous plant

नननननलिखितमसहॳकह९नसाएकककटाहरीवनसपनतहहॴ

15-Jan-2017

Options

1) Utricularia

यटरीकिहॳररया 2) Sequoia

सहॳकयओइया 3) Nostoc

नॉसटह८क

4) Bryophyta

िायह८फाईटा Correct Answer Utricularia

Q56 ______________ is a

multibranched polysaccharide of

glucose that serves as a form of energy

storage in animals and fungi

mdashmdashगिकह८जकाएकबहिािायकतपह८िीसहॳकहॳ राइिहहॴ जह८जानवरोऔरकवकमउजायभणिारणकहॳ एक पमकाययकरताहहॴ 15-Jan-2017

Options

1) Cellulose

सहॳमयिह८ज

2) Glycogen

गिायकह८जन

3) Pectin

पहॳनकटन

4) Chitin

चीटटन

Correct Answer Glycogen

Q57 The largest gland of the human

body is

mdashmdashmdashमानविरीरकीसबसहॳबड़ीगरिीहहॴ 16-Jan-2017

Options

1) Pancreas

अगयािय

2) Thyroid

िायरॉइि

3) Large Intestine

बड़ीआत

4) Liver

यकत

Correct Answer Liver

Q58 Photosynthesis in plants takes

place in

वनसपनतयोमपरकािसशिहॳषणकीकियाहह८तीहहॴ

16-Jan-2017

Options

1) Stem

तना 2) Leaves

पनततयाा 3) Roots

जड़हॳ 4) Flower

फि

Correct Answer Leaves

During this reaction carbon dioxide

and water are converted into glucose

and oxygen The reaction requires light

energy which is absorbed by a green

substance called

chlorophyll Photosynthesis takes place

in leaf

cells These contain chloroplasts which

are tiny objects containing chlorophyll

F A C E B O O K

P A G E h t t p w w w f a c e b o o k c o m s s c m e n t o r s o f f i c i a l P a g e | 15

FOR MORE UPDATES AND MORE MATERIAL DO LIKE OUR FACEBOOK PAGE httpwwwfacebookcomsscmentorsofficial

Q59 Insects that transmit diseases are

known as

जह८कीड़हॳरह८गसचाररतकरतहॳहहॴ उनह mdashmdash-

कहॳ नामसहॳजानाजाताहहॴ 16-Jan-2017

1)Pathogens

रह८गज़नक

2) Vectors

वहॳकटर

3) Drones

परजीवी 4)Scalars

अटदषट

Correct Answer Vectors

A vector is an organism that does not

cause disease itself but which spreads

infection by conveying pathogens from

one host to another Species of mosquito

for example serve as vectors for the

deadly disease Malaria

Q60 Which is the second largest gland

of Human body

मानविरीरकीदसरीसबसहॳबड़ीगरिीकह९नसीहहॴ

SSC CHSL Science (biology)

2016 Question Paper

16-Jan-2017

Options

1) Liver

यकत

2) Large Intestine

बड़ीआत

3) Thorax

छाती 4) Pancreas

अगनयािय

Correct Answer Pancreas

Q61 Annona squamosa is the scientific

name of

एनह८नासकवामह८सा (Annona squamosa) mdash

mdashmdash कावहॴजञाननकनामहहॴ 16-Jan-2017

Options

1) Custard Apple

सीताफि

2) Papaya

पपीता 3) Babhul

बबि

4) Drumstick

सहजन

Correct Answer Custard Apple

Q62 The disease Beri Beri is caused due

to the deficiency of which of the

following

बहॳरीबहॳरीरह८गनननननलिखितमसहॳककसकीकमीकहॳकारणहह८ताहहॴ

16-Jan-2017

Options

1) Vitamin B2

पवटालमन B2

2) Vitamin B1

पवटालमन B1

3) Vitamin B12

पवटालमन B12

4) Vitamin E

पवटालमन E

Correct Answer Vitamin B1

Beriberi is a disease caused by a vitamin

B-1 deficiency also known as thiamine

deficiency

Q63 Chlorophyll was first isolated and

named by

किह८रह८कफिकह८ mdash-

दवारापहिहॳपिकऔरनालमतककयागया 16-Jan-2017

F A C E B O O K

P A G E h t t p w w w f a c e b o o k c o m s s c m e n t o r s o f f i c i a l P a g e | 16

FOR MORE UPDATES AND MORE MATERIAL DO LIKE OUR FACEBOOK PAGE httpwwwfacebookcomsscmentorsofficial

Options

1) Caventou

कहॳ वहॳत 2) Pelletier

पहॳिहॳटटयर

3) Chlorophyll

किह८रह८कफि

4) Caventou and Pelletier

कहॳ वहॳतऔरपहॳिहॳटटयर

Correct Answer Caventou and Pelletier

Chlorophyll was first isolated and

named by

Joseph Bienaimeacute Caventou and Pierre

Joseph Pelletier in 1817 The presence of

magnesium in chlorophyll was

discovered in 1906 and was the first

time that magnesium had been detected

in living tissue

Q64 Which of the following organisms

does not fit into the Cell Theory

नननननलिखितमसहॳकह९नसाजीवकह८लिकालसदातअन पनहीहहॴ

16-Jan-2017

Options

1) Bacteria

बहॴकटीररया 2) Virus

वायरस

3) Fungi

कवक

4) Plants

पह९धहॳ Correct Answer Virus

The bottom line is that viruses are not

alive and not related to cells in any way

The cell theory states that all living

things are made of cells cells are the

basic units of structure and function of

living things and that all cells come

from other cells Since viruses are not

made of cells and do not use cells in any

of their processes they are not related to

the cell theory

Q65 Which of these is not a

macronutrient for Plants

नननननलिखितमसहॳकह९नसापह९धह८कहॳ लिएमिह८नयटरीएटनहीहहॴ

SSC CHSL Science (biology) 2016

Question Paper

17-Jan-2017

Options

1) Nitrogen

नाइटरह८जन

2) Phosphorus

फासफह८रस

3) Potassium

पह८टालसयम

4) Chlorine

किह८रीन

Correct Answer Chlorine

In relatively large amounts the soil

supplies nitrogen phosphorus

potassium calcium magnesium and

sulfur these are often called the

macronutrients In relatively small

amounts the soil supplies iron

manganese boron molybdenum

copper zinc chlorine and cobalt the

so-called micronutrients

Q66 Name the respiratory organs of

insects

कीटह८मनसतिशरवसनअगनामकानामहहॴ

17-Jan-2017

Options

1) Skin

तवचा 2) Body Surface

िरीरकीसतह

F A C E B O O K

P A G E h t t p w w w f a c e b o o k c o m s s c m e n t o r s o f f i c i a l P a g e | 17

FOR MORE UPDATES AND MORE MATERIAL DO LIKE OUR FACEBOOK PAGE httpwwwfacebookcomsscmentorsofficial

3) Gills

गिफड़हॳ 4) Tracheae

शरावस- निी Correct Answer Tracheae

Air enters the respiratory systems of

insects through a series of external

openings called

spiracles These external openings

which act as muscular valves in some

insects lead to the internal respiratory

system a densely networked array of

tubes called tracheae

Q67 The poisonous gas accidentally

released in Bhopal Gas Tragedy is

भह८पािगहॴसतरासदीमगितीसहॳमकतहईजहरीिीगहॴसिी

17-Jan-2017

1) Methane

मीिहॳन

2) Nitrous Oxide

नाइटरसऑकसाइि

3) Methyl Isocyanate

महॴचििआयसोसायनहॳट

4) Cyanogen

सायनह८जहॳन

Correct Answer Methyl Isocyanate

Q68 What does Trypsin do

टटरनपसनकयाकरताहहॴ

SSC CHSL Science (biology) 2016

Question Paper

17-Jan-2017

Options

1) Breaks down Carbohydrates

काबोहाइडरहॳटकापवघटनकरताहहॴ 2) Synthesizes proteins

परह८टीनकासििहॳषणकरताहहॴ 3) Breaks down fats

वसाकापवघटनकरताहहॴ 4) Breaks down proteins

परह८टीनकापवघटनकरताहहॴ Correct Answer Breaks down proteins

Trypsin is one of the three principal

digestive

proteinases the other two being pepsin

and

chymotrypsin In the digestive process

trypsin acts with the other proteinases

to break down dietary protein molecules

to their component

peptides and amino acids

A protease is any enzyme that performs

proteolysis protein catabolism by

hydrolysis of peptide bonds

Q69 Name the source from which

Aspirin is produced

उससरह८तकानामबताइए

नजससहॳएनसपररनकाउतपादनककयाजाताहहॴ

17-Jan-2017

Options

1) Willow bark

पविह८कीछाि

2) Oak Tree

ओककावकष

3) Acacia

बबि

4) Eucalyptus

नीिचगरी Correct Answer Willow bark

The compound from which the active

ingredient in aspirin was first derived

salicylic acid was found in the bark of a

willow tree in 1763 by Reverend

Edmund Stone of Chipping-Norton

Q70 Cannis Familiaris is the scientific

name of

कहॴ ननसफहॳ लमलियहॳररस mdash- कावहॴजञाननकनामहहॴ

17-Jan-2017

F A C E B O O K

P A G E h t t p w w w f a c e b o o k c o m s s c m e n t o r s o f f i c i a l P a g e | 18

FOR MORE UPDATES AND MORE MATERIAL DO LIKE OUR FACEBOOK PAGE httpwwwfacebookcomsscmentorsofficial

Options

1) Cat

बबमिी 2)Dog

कतता 3) Fox

िह८मड़ी 4) Wolf

भहॳडड़या Correct Answer Dog

Q71 Harmful bacteria in potable water

make the water

पीनहॳकहॳ पानीमनसतिघातकबहॴकटीररयाउसपानीकह८बनातहॳहहॴ 17-Jan-2017

Options

1) unfit to drink

पीनहॳकहॳ लिएअयह८गय

2) smelly

दगयनधयकत

3) Colored

रगीन

4) Turbid

मटमहॴिा Correct Answer unfit to drink

Q72 Musa paradisiaca is the scientific

name of which plant

मसापहॴराडिलसयाकाककसपह९धहॳकावहॴजञाननकनामहहॴ

17-Jan-2017

Options

1) Mango

आम

2) Wheat

गहॳह

3) Corn

भ ा 4) banana

कहॳ िा Correct Answer banana

Q73 Prawns belong to which family

झीगहॳककसपररवारकहॳ हह८तहॳहहॴ 17-Jan-2017

Options

1) Crustaceans

िसटहॳलियन

2)Fish

मछिी 3) Amphibians

अननफबबयस

4) Reptiles

रहॳपटाइमस

Correct Answer Crustaceans

Q74 Name the drug that is yielded from

Cinchona tree and is used to cure

malaria

उसऔषचधकानामबताइएनजसहॳलसगकह८नापहॳड़सहॳपरापतककयाजाताहहॴऔरनजसकाउपयह८गमिहॳररयाकहॳ उपचारमककयाजाताहहॴ 17-Jan-2017

Options

1) Camptothea

कहॴ नटह८चिया 2) Acuminata

एकयलमनहॳटा 3) Quinine

कनहॴन

4) Cinchonia

लसकह८ननया Correct Answer Quinine

Q75 Blood Circulation was discovered

by

रकतपररसचरणकी mdashmdashndash दवारािह८जकीिी 17-Jan-2017

Options

1) Mary Anderson

F A C E B O O K

P A G E h t t p w w w f a c e b o o k c o m s s c m e n t o r s o f f i c i a l P a g e | 19

FOR MORE UPDATES AND MORE MATERIAL DO LIKE OUR FACEBOOK PAGE httpwwwfacebookcomsscmentorsofficial

महॴरीएिरसन

2) Virginia Apgar

वनजयननयाएपगार

3) William Harvey

पवलियमहाव

4) Robert Feulgen

रॉबटयफ़यिजहॳन Correct Answer William Harvey

Q76 Vitamin A is also known as

पवटालमन A कह८ mdashmdash- कहॳ नामसहॳभीजानाजाताहहॴ SSC CHSL Science (biology) 2016

Question Paper

18Jan2017

Options

1) Thiamine

िायलमन

2) Riboflavin

ररबह८फिहॳपवन

3) Retinol

रहॳटटनॉि

4) Calciferol

कहॴ नमसफहॳ रह८ि

Correct Answer Retinol

Q77 Some roots called arise from an

organ other than the radicle

कछजड़हॳनजनह mdashmdashmdash कहाजाताहहॴ वहमिकहॳ अिावाककसीअनयअगसहॳउतपननहह८तीहहॴ 18Jan2017

Options

1) tap roots

मखयजड़

2) stilt roots

ि ाजड़

3) fibrous roots

रहॳिहॳदारजड़

4) adventitious roots

आकनसमकजड़

Correct Answer adventitious roots

Q78 Spiders belong to which class of

animals

मकडड़यापराणीवगीकरणकहॳ ककसवगयमआतीहहॴ 18Jan2017

Options

1) Arachnids

एरहॳकननडस

2) Aves

एपवस

3) Gastropods

गहॴसटरोपह८िस

4) Anthozoa

एिह८जआ

Correct Answer Arachnids

Q79 How many layers does Human

Skin have

मानवतवचामककतनीपरतहॳहह८तीहहॴ

18Jan2017

Options

1) 5

2) 7

3) 11

4) 3

Correct Answer 3

Skin has three layers The epidermis

the outermost layer of skin provides a

waterproof barrier and creates our skin

tone The dermis beneath the

epidermis contains tough connective

tissue hair follicles and sweat glands

The deeper subcutaneous tissue (

hypodermis ) is made of fat and

connective tissue

Q80 Allium Cepa is the scientific name

of

एलियमलसपपा mdashmdashndash कावहॴजञाननकनामहहॴ 18Jan2017

F A C E B O O K

P A G E h t t p w w w f a c e b o o k c o m s s c m e n t o r s o f f i c i a l P a g e | 20

FOR MORE UPDATES AND MORE MATERIAL DO LIKE OUR FACEBOOK PAGE httpwwwfacebookcomsscmentorsofficial

Options

1) Carrot

गाजर

2) Tomato

टमाटर

3) Potato

आि 4) Onion

पयाज़

Correct Answer Onion

Q81 DNA stands for

िीएनएकापणय प mdashmdash- हहॴ 18Jan2017

Options

1) Di Nucleic Acid

िाईनयनकिकएलसि

2) Deoxy Nucleic Acid

िीओकसीनयनकिकएलसि

3) Diribonucleic Acid

िाईराइबह८नयनकिकएलसि

4) Deoxyribonucleic Acid

िीऑकसीराइबह८नयनकिकएलसि

Correct Answer Deoxyribonucleic Acid

Q82 Organisms that generate energy

using light are known as

जह८जीवाणपरकािकाउपयह८गकरउजायउतपननकरतीहहॴ उनह mdashmdash कहॳ पमजानाजाताहहॴ

18Jan2017

Options

1) Chaemolithotrophs

ककमह८लििह८टरह८पस

2) Oligotrophs

ओलिगह८टरह८पस

3) Bacteria

बहॴकटीररया 4)Photoautotrophs

फह८टह८ओटह८टरह८पस

Correct Answer Photoautotrophs

An oligotroph is an organism that can

live in an environment that offers very

low levels of nutrients

Q83 Which drug is used as an

Antidepressant

ककसदवाएकहतािारह८धीकहॳ पमपयोगककयाजाताहहॴ Options

1) Oxybutynin

ओकसीलयटीनन

2)Tramadol

टरहॳमहॳिह८ि

3 ) Sumatriptan

समहॳटरीपटहॳन

4) Bupropion

लयपरह८पपयह८न

Correct Answer Bupropion

लयपरह८पपयह८न

Q84 The orange colour of carrot is

because of

गाजरकानारगीरगनननननलिखितमसहॳककसीएककीवजहसहॳहह८ताहहॴ 18Jan2017

Options

1) it grows in the soil

यहलम ीमउगतीहहॴ 2) Carotene

कहॴ रह८टीन

3) it is not exposed to sunlight

यहसययपरकािकहॳ सपकय मनहीआती 4) the entire plant is oranqe in colour

सनपणयपह९धानारगीरगकाहह८ताहहॴ Correct Answer Carotene

Q85 Snake venom is highly modified

saliva containing

F A C E B O O K

P A G E h t t p w w w f a c e b o o k c o m s s c m e n t o r s o f f i c i a l P a g e | 21

FOR MORE UPDATES AND MORE MATERIAL DO LIKE OUR FACEBOOK PAGE httpwwwfacebookcomsscmentorsofficial

सापकाजहरअततयाचधकसिह८चधतिारहह८तीहहॴनजसमहॳ mdashmdash- हह८ताहहॴ Options

l)Prototoxins

परह८टह८टॉनकसस

2)Neutrotoxins

नयटरोटॉनकसस

3)Zootoxins

जटॉनकसस

4)Electrotoxins

इिहॳकटरह८टॉनकसस

Correct Answer Zootoxins

जटॉनकसस

Q86 Which type of pathogen causes the

water-borne disease Schistosomiasis

ककसपरकारकारह८गज़नकजिजननतरह८गलससटह८सह८लमलससकाकारणबनताहहॴ

18Jan2017

Option

1) Parasitic

परजीवी 2)Protozoan

परह८टह८जआ

3) Bacterial

बहॴकटीररयि

4) Viral

वायरि

Correct Answer Parasitic

Schistosomiasis also known as snail

fever and bilharzia is a disease caused

by parasitic

flatworms called schistosomes

Q87 Prothrombin responsible for

clotting of blood is released by

परह८िह८ननबन

जह८रकतकािककाजमनहॳकहॳ लिएनजनमहॳदारहहॴ mdashndash

कहॳ दवारासतरापवतककयाजाताहहॴ

19Jan2017

Options

1) Small Intestine

छह८टीआत

2) Blood Platelets

रकतपिहॳटिहॳटस

3) Large Intestine

बड़ीआत

4Heart

हदय

Correct Answer Blood Platelets

Q88 Acacia arabica is the scientific

name of

अकहॳ लियाअरहॳबबका mdashmdashndash कावहॴजञाननकनामहहॴ 19-Jan-2017

Options

1) Neem

नीम

2) Teak

सागह९न

3) Babhul

बबि

4) Pomegranate

अनार

Correct Answer Babhul

Q89 Cannis Vulpes is the scientific

name of

कहॴ ननसवनमपस mdashmdash- कावहॴजञाननकनामहहॴ 19-Jan-2017

Options

1) Dog

कतता 2) Wolf

भहॳडड़या 3) Fox

िह८मड़ी 4) Hyena

िाकिबगघा

F A C E B O O K

P A G E h t t p w w w f a c e b o o k c o m s s c m e n t o r s o f f i c i a l P a g e | 22

FOR MORE UPDATES AND MORE MATERIAL DO LIKE OUR FACEBOOK PAGE httpwwwfacebookcomsscmentorsofficial

Correct Answer Fox

Q90 The beetroot is the portion of the

beet plant

चकदरपह९धहॳका mdashmdashndash भागहहॴ 19-Jan-2017

Options

1) tap root

मखयजड़

2) Adventitious

आकनसमक

3) bulb of the stem

तनहॳकाकद

4) Rhizome

परकद

Correct Answer tap root

Q91 What is the basic unit of heredity

आनवलिकताकीबननयादीइकाईकयाहहॴ 19-Jan-2017

Options

1) DNA

िीएनए

2) RNA

आरएनए

3) Chromosome

िह८मह८सह८म

4) Gene

जीन

Correct Answer gene

Genes are the units of heredity and are

the instructions that make up the bodyrsquos

blueprint They code for the proteins

that determine virtually all of a personrsquos

characteristics Most genes come in

pairs and are made of strands of genetic

material called deoxyribonucleic acid

or DNA

Q92 Lungs are the primary organs of

फहॳ फड़हॳmdashndashकहॳ परािलमकअगहहॴ

19-Jan-2017

Options

1) Digestion

पाचन

2) Constipation

कलज

3) Perspiration

पसीना 4)Respiration

शवसन

Correct Answer Respiration

Q93 Sugarcane is a type of

गननाएकपरकारका mdash- हहॴ 20-Jan-2017

Options

1)creeper

िता 2)tree

पहॳड़

3)shrub

झाड़ी 4)grass

घास

Correct Answer grass

Q94 Who is commonly known as ldquothe

Father of Microbiologyrdquo

सामानयत ldquo सकषमजीवपवजञानकहॳ जनक lsquo

कहॳ नामसहॳककसहॳजानाजातहहॴ 20-Jan-2017

Options

1) Robert Hooke

रॉबटयहक

2) Antonie Philips van Leeuwenhoek

एटह८नीकफलिपवानमयएनहह८क

3) Carl Linnaeus

काियिीनाईयस

4) Charles Darwin

चामसयिापवयन

F A C E B O O K

P A G E h t t p w w w f a c e b o o k c o m s s c m e n t o r s o f f i c i a l P a g e | 23

FOR MORE UPDATES AND MORE MATERIAL DO LIKE OUR FACEBOOK PAGE httpwwwfacebookcomsscmentorsofficial

Correct Answer Antonie Philips van

Leeuwenhoek

Q95 For the aquatic organisms the

source of food is

जिीयजीवाणकािाघसरह८तहहॴ 20-Jan-2017

Options

1) Phytoplankton

फायटह८पिहॳकटन

2) Sea Weed

समदरीिहॴवाि

3)Aqua plankton

एकवापिहॳकटन

4) Zooplankton

जपिहॳकटन

Correct Answer Phytoplankton

Q96 Haemoglobin has the highest

affinity with which of the following

हीमह८गिह८बबनकीननननमसहॳककसकहॳ सािउततमसमानताहहॴ

20-Jan-2017

Options

1)SO2

2)CO2

3)CO

4)NO2

Correct Answer CO

It has a greater affinity for hemoglobin

than oxygen does It displaces oxygen

and quickly binds so very little oxygen

is transported through the body cells

Q97 Who developed the theory of

Evolution

उदपवकासकालसदातककसनहॳपवकलसतककया

20-Jan-2017

Options

1) Charles Darwin

चामसयिापवयन

2) Isaac Newton

आयजहॳकनयटन

3) Pranav Mistry

परणवलमसतरी 4) Galileo Galilei

गहॳलिलियह८गहॳिीिी Correct Answer Charles Darwin

Q98 The primary function of RNA is

RNA कापरािलमककाययहह८ताहहॴ 20-Jan-2017

Options

1) Photosynthesis

परकािसशिहॳषण

2) Protein Synthesis

परह८टीनसशिहॳषण

3) Replication

परनतकनतबनाना 4) Translation

अनवादकरना Correct Answer Protein Synthesis

There are two main functions of RNA

It assists DNA by serving as a messenger

to relay the proper genetic information

to countless numbers of ribosomes in

your body The other main function of

RNA is to select the correct amino acid

needed by each ribosome to build new

proteins for your body

Q99 ______is the movement of

molecules across a cell membrane from

a region of their lower concentration to

a region of their higher concertration

उचचसादरताकहॳ कषहॳतरसहॳउसकीकमसादरतावािहॳकषहॳतरकीतरफएककह८लिकाखझमिीकहॳ माधयमसहॳहह८नहॳवािाअणओकहॳ सचिनकह८ mdash- कहतहॳहहॴ Options

1) Diffusion

पवसरण

2) Osmosis

ऑसमह८लसस

F A C E B O O K

P A G E h t t p w w w f a c e b o o k c o m s s c m e n t o r s o f f i c i a l P a g e | 24

FOR MORE UPDATES AND MORE MATERIAL DO LIKE OUR FACEBOOK PAGE httpwwwfacebookcomsscmentorsofficial

3) Active Transport

सकियआवागमन

4) Passive Transport

नननषियआवागमन

Correct Answer Active Transport

Q100 Study of classification of

organisms is known as 20-Jan-2017

जीवाणओकहॳ वगीकरणकहॳ अधययनकह८ mdash-

कहाजाताहहॴ Options

1) Serpentology

सपरहॳटह८िह८जी 2) Virology

वायरह८िह८जी 3) Taxonomy

टहॴकसोनह८मी 4) Physiology

कफनज़यह८िह८जी Correct Answer Taxonomy

Q101 Photosynthesis takes place inside

plant cells in

परकािसशिहॳषणवनसपनतकह८लिकामनसति mdash

mdashmdash महह८ताहहॴ 20-Jan-2017

Options

1) Ribosomes

राइबह८सह८नस

2) Chloroplasts

किह८रह८पिासट

3) Nucleus

नयकलियम

4) Mitochondria

माईटह८कोडडरया Correct Answer Chloroplasts

Q102 ______ is the cell organelle in

which the biochemical processes of

respiration and energy production

occur

mdashmdash- वहकह८लिकाअगहहॴ नजसमहॳशवसनऔरउजायउतपादनकहॳ जहॴसीजहॴवरासायननकपरकियायहह८तीहहॴ 20-Jan-2017

Options

1) Mitochondria

माइटह८कोडडरया 2) Chloroplast

किह८रह८पिासट

3) Ribosomes

राइबह८सह८नस

4) Nucleus

नयकिीयस

Correct Answer Mitochondria

Q103 Which non-flowering spore

bearing plants have roots

ककसफिनिगनहॳवािहॳऔरबीजाणधारकपह९धह८कीजड़हॳहह८तीहहॴ 21-Jan-2017

Options

1) Mosses

मह८सहॳस

2) Angiosperms

एननजयह८सपनसय 3) Ferns

फनसय 4) Gymnosperms

नजननह८सपनसय Correct Answer ferns

Q104 Which of the following is an

excretory organ of cockroach

नननननलिखितमसहॳकह९नसानतिच हॳकाउतसजयनअगहहॴ

21-Jan-2017

Options

F A C E B O O K

P A G E h t t p w w w f a c e b o o k c o m s s c m e n t o r s o f f i c i a l P a g e | 25

FOR MORE UPDATES AND MORE MATERIAL DO LIKE OUR FACEBOOK PAGE httpwwwfacebookcomsscmentorsofficial

1) Malphigian Tubules

मनमफनजयनटयबमस

2) Nephridia

नहॳकफरडिया 3) Coxal Gland

कह८कसिगरचिया 4) Green Gland

गरीनगरचिया Correct Answer Malphigian Tubules

Q105 Evaporation of water takes place

in which part of plants

पानीकहॳ वाषपीकरणकीकियापह९धोकहॳ ककसभागसहॳहह८तीहहॴ 21-Jan-2017

Options

1) Stem

तना 2) Stomata

सटह८मटा 3) Branch

िािाए

4) Fruit

फि

Correct Answer Stomata

Evaporation accounts for the movement

of water to the air from sources such as

the soil canopy interception and

waterbodies Transpiration accounts for

the movement of water within a plant

and the subsequent loss of water as

vapour through stomata in its leaves

Q106 A is the fleshy spore-bearing

fruiting body of a fungus

mdashmdashndashकवककामासि

बीजाणधारणकरनहॳवािाफिनहॳवािाअगहहॴ 21-

Jan-2017

Options

1) aloe vera

एिह८वहॳरा 2) Coral

मगा 3) Cactus

कहॴ कटस

4) Mushroom

ककरमतता Correct Answer mushroom

Q107 Which of the following is a fungal

disease

नननननलिखितमसहॳकह९नसाफफदसहॳहह८नहॳवािाएकरह८ग हहॴ

21-Jan-2017

Options

1) Dermatitis

तवचािह८ध

2) Cholera

हहॴजा 3) Jaundice

पीलिया 4) Indigofera

इननिगह८फहॳ रा Correct Answer Dermatitis

Dermatitis also known as eczema is a

group of diseases that results in

inflammation of the skin These diseases

are characterized by itchiness red skin

and a rash In cases of short duration

there may be small blisters while in

long-term cases the skin may become

thickened

Q108 In which form is glucose stored in

our body

हमारहॳिरीरमगिकह८जकासचयककस पमककयाजाताहहॴ

21-Jan-2017

Options

1) Insulin

F A C E B O O K

P A G E h t t p w w w f a c e b o o k c o m s s c m e n t o r s o f f i c i a l P a g e | 26

FOR MORE UPDATES AND MORE MATERIAL DO LIKE OUR FACEBOOK PAGE httpwwwfacebookcomsscmentorsofficial

इसलिन

2) Glucose

गिकह८ज

3) Glycogen

गिायकह८जहॳन

4) Fat

वसा Correct Answer Glycogen

Excess glucose is stored in the liver as

the large compound called glycogen

Glycogen is a polysaccharide of glucose

but its structure allows it to pack

compactly so more of it can be stored in

cells for later use

Q109 Where do plants synthesize

protein from

पह९धहॳपरह८टीनसशिहॳषणकहासहॳकरतहॳहहॴ

Options

1) Fatty Acids

वसाऐलसि

2) Sugar

िकर

3) Amino Acids

एलमनह८ऐलसि

4) Starch

सटाचय Correct Answer Amino Acids

Q110 Which part of the brain is

responsible for triggering actions like

thinking intelligence memory and

ability to learn

मनसतषककाकह९नसाटहससासह८चनहॳ बनधदमानी याददाशतऔरसीिनहॳकीकषमताजहॴसीकियाओकह८परहॳररतकरताहहॴ 21-Jan-2017

Options

1) Diencephalon

िायएनसहॳफहॳ िह८न

2) Hypothalamus

हयपह८िहॳिहॳमस

3) Cerebrum

सहॳरहॳिम

4) Control

कटरह८ि

Correct Answer Cerebrum

Q111 Which of the following is also

known as the Biochemical Laboratory

of the Human Body

नननननलिखितमसहॳककसहॳमानविरीरकीजहॴवरसायनपरयह८गिािाभीकहाजाताहहॴ 21-Jan-2017

Options

1) Small Intestine

छह८टीआत

2)Brain

मनसतषक

3) Pancreas

अगनयािय

4) Liver

नजगर

Correct Answer Liver

The liver makes bile that will help

emulsify and digest the fats we eat

The liver takes toxic substances and

convert them using enzymes the liver

cells makes into a non toxic form so the

body can dispose of them

The liver also converts fats protein and

carbohydrates into glucose which is the

energy source for our cells to use

The liver takes amino acids and makes

proteins by combining them

Q112 The yellow colour of human urine

is due to

मानवमतरकापीिारग mdashndash कीवजहसहॳहह८ताहहॴ 22-

Jan-2017

Options

1) Bile Salts

F A C E B O O K

P A G E h t t p w w w f a c e b o o k c o m s s c m e n t o r s o f f i c i a l P a g e | 27

FOR MORE UPDATES AND MORE MATERIAL DO LIKE OUR FACEBOOK PAGE httpwwwfacebookcomsscmentorsofficial

पपततनमक

2) Cholesterol

कह८िहॳसटरह८ि

3) Lymph

लिनफ

4) Urochrome

यरह८िह८म

Correct Answer Urochrome

Urobilin or urochrome is the chemical

primarily responsible for the yellow

color of urine

Q113 The wilting of plants takes place

due to

पह९धह८कालिचििहह८नाकी mdashmdash- कीवजहसहॳहह८ताहहॴ 22-Jan-2017

Options

1)Photosynthesis

परकािसशिहॳषण

2) Transpiration

वाषपह८तसजयन

3) Absorption

अविह८षण

4) Respiration

शरवसन

Correct Answer Transpiration

Wilting is the loss of rigidity of non-

woody parts of plants This occurs when

the turgor pressure in non-lignified

plant cells falls towards zero as a result

of diminished water in the cells

Q114 Bovidae Ovis is the scientific name of

बह८पविीओपवस mdashndash कावहॴजञाननकनामहहॴ 22-Jan-2017

Options

1) Goat

बकरी 2) Cow

गाय

3) Buffalo

भहॳस

4) Sheep

भहॳड़

Correct Answer Sheep

Q115 Plants get their energy to produce

food from which of the following

पह८धहॳभह८जनकाननमायणकरनहॳकहॳ लिएनननननलिखितमसहॳककससहॳउजायपरापतकरतहॳहहॴ

22-Jan-2017

Options

1) Photosynthesis

परकािसशिहॳषण

2)Bacteria

बहॴकटीररया 3)Fungi

कवक

4)Sun

सयय Correct Answer Sun

Q116 Which of the following is secreted

by the liver

नननननलिखितमसहॳककसकासरावनजगरसहॳहह८ताहहॴ

22-Jan-2017

Options

1) Glucose

गिकह८ज

2) Iodine

आयह८िीन

3) Cortisol

काटटरयसह८ि

4) Bile

पपतत

Correct Answer Bile

The liver makes bile that will help

emulsify and

digest the fats we eat

F A C E B O O K

P A G E h t t p w w w f a c e b o o k c o m s s c m e n t o r s o f f i c i a l P a g e | 28

FOR MORE UPDATES AND MORE MATERIAL DO LIKE OUR FACEBOOK PAGE httpwwwfacebookcomsscmentorsofficial

Q117 Ferns belong to which division of

plants

फनसयपह९धह८कहॳ ककसभागमआतहॳहहॴ

22-Jan-2017

Options

1) Gymnosperms

नजननह८सपनसय 2) Angiosperms

एनजयह८सपनसय 3) Thallophyta

िहॴिह८फाईटा 4)Pteridophyta

टहॳररिह८फाईटा Correct Answer Pteridophyta

Q118 Who invented Antibiotics

एटीबायह८टटककाअपवषकारककसनहॳककयािा

22-Jan-2017

Options

1) Joseph Lister

जह८सहॳफलिसटर

2) William Harvey

पवलियमहाव

3) Robert Knock

रॉबटयनॉक

4)Alexander Fleming

अिहॳकज़िरफिहॳलमग

Correct Answer Alexander Fleming

Q119 Milbecycin is used in the

eradication of

लममबहॳसायलसनका mdashndash

मउनमिनमपरयह८गककयाजाताहहॴ 22-Jan-2017

Options

1) Agricultural Fungus

कपषकवक

2) Agricultural Pests

कपषकीटक

3) Agricultural Herbs

कपषिाक

4)Agricultural Weeds

कपषननराना Correct Answer Agricultural Pests

Milbemycin oxime is a veterinary drug

from the group of milbemycins used as

a broad spectrum antiparasitic It is

active against worms and mites(insects

Q120 Intestinal bacteria synthesizes

which of the following in the human

body

मानविरीरमआतोकहॳ बहॴकटीररयानननननलिखितमसहॳककसकासशिहॳषणकरतहॳहहॴ 22-Jan-2017

Options

1) Vitamin K

पवटालमन K

2) Proteins

परह८टीन

3) Fats

वसा 4) Vitamin D

पवटालमन D

Correct Answer Vitamin K

Q121 is the study of the physical form

and external structure of plants

mdashmdash-

मपह९धह८काभहॴनतक पऔरबाहरीसरचनाकाआदयाककयाजाताहहॴ 22-Jan-2017

Options

1) Physiology

कफनजयह८िह८जी 2) Anatomy

िरीररचनापवजञान

3) Phytomorphology

फाईटह८मह८फह८िह८जी 4)Cytology

कह८लिकापवजञान

Correct Answer Phytomorphology

F A C E B O O K

P A G E h t t p w w w f a c e b o o k c o m s s c m e n t o r s o f f i c i a l P a g e | 29

FOR MORE UPDATES AND MORE MATERIAL DO LIKE OUR FACEBOOK PAGE httpwwwfacebookcomsscmentorsofficial

Q122 Which of the following is a

structural and functional unit of

kidneys

नननननलिखितमसहॳकह९नसीगदोकीसरचनातमकऔरकाययकरीईकाईहहॴ

22-Jan-2017

Options

1) Renette Cells

रहॳनहॳटकह८लिकाए

2) Flame Cells

फिहॳमकह८लिकाए

3) Nephrites

नहॳफ़राइटस

4)Nephrons

नहॳफरोस

Correct Answer Nephrons

Nephron functional unit of the kidney

the structure that actually produces

urine in the process of removing waste

and excess substances from the blood

There are about 1000000 nephrons in

each human kidney

Q123 Which of the following is the

largest part of the human brain

नननननलिखितमसहॳकह९नसामानवमनसतषककासबसहॳबड़ाटहससाहहॴ

23-Jan-2017

Options

1) Ribs

पसलियाा 2) Cerebrum

सहॳरहॳिम

3) Pons

पोस

4)Thalamus

िहॴिहॳमस

Correct Answer Cerebrum

The cerebrum is the largest part of the

human brain making up about two-

thirds of the brainrsquos mass It has two

hemispheres each of which has four

lobes frontal parietal temporal and

occipital

Q124 The auxiliary buds

सहायककालियाmdashndash 23-Jan-2017

Options

1) grow endogenously from the pericycle

पहॳरीसाईककिसहॳअनतजातयपवकलसतहह८ताहहॴ 2) arise endogenously from the main

growing point

मिवपदसहॳअनतजातयउठताहहॴ 3) is an embryonic shoot located in the

axil of a leaf

एकभरणिटहहॴजह८एकपततीकहॳ अकषपरनसतिहह८ताहहॴ 4)arise exogenously from the epidermis

एपपिलमयससहॳबटहजातयतरीकहॳ सहॳउठताहहॴ Correct Answer is an embryonic shoot

located in the axil of a leaf

Q125 Which of the following is a viral

disease

इनमहॳसहॳकह९सीएकवायरिबीमारीहहॴ

23-Jan-2017

Options

1) Polio

पह८लियह८ 2) Tetanus

धनसतनभ

3) Leprosy

कषठरह८ग

4) Plague

पिहॳग

Correct Answer Polio

A viral disease (or viral infection)

occurs when an organismrsquos body is

invaded by pathogenic viruses and

infectious virus particles (virions) attach

to and enter susceptible cells

F A C E B O O K

P A G E h t t p w w w f a c e b o o k c o m s s c m e n t o r s o f f i c i a l P a g e | 30

FOR MORE UPDATES AND MORE MATERIAL DO LIKE OUR FACEBOOK PAGE httpwwwfacebookcomsscmentorsofficial

Poliomyelitis often called polio or

infantile paralysis is an infectious

disease caused by the poliovirus

Tetanusmdash A serious bacterial infection

that causes painful muscle spasms and

can lead to death

Leprosy also known as Hansenrsquos

disease (HD) is a long-term infection by

the bacterium Mycobacterium leprae or

Mycobacterium lepromatosis

Plague is an infectious disease caused by

the bacterium Yersinia pestis

Symptoms include fever weakness and

headache

Q126 Which organisms can help to

carry out Vermicomposting

कह९नसाजीववमीकनपह८नसटगममददकरताहहॴ

23-Jan-2017

Options

1) Nitrifying Bacteria

नाईटरीफाईगबहॴकटीररया 2) Earthworms

कहॴ चऐ

3) Algae

िहॴवि

4) Fungus

कवक

Correct Answer Earthworms

Q127 Contraction of heart is also

known as

हदयकहॳ सकचनकह८ mdash- भीकहाजाताहहॴ 23-Jan-

2017

Options

1) Systole

लससटह८ि

2) Aristotle

अरसत

3) Diastole

िायसटह८ि

4) Lub

मयब

Correct Answer Systole

Diastole is the part of the cardiac cycle

when the heart refills with blood

following systole (contraction)

Ventricular diastole is the period during

which the ventricles are filling and

relaxing while atrial diastole is the

period during which the atria are

relaxing

Q128 Azadirachta indica is the

botanical name of which of the

following

अजाटदराचताइडिकानननननलिखितमसहॳककसकावानसपनतनामहहॴ

23-Jan-2017

Options

1) Rose plant

गिाबकापह९धा 2) Apple tree

सहॳबकापहॳड़

3) Neem

नीम

4)Mango

आम

Correct Answer Neem

Q129 Which of the following is the

main end product of carbohydrate

digestion

नननननलिखितमसहॳकह९नसाकाबोहाइडरहॳटकहॳ पाचनकापरमिअतउतपादकहह८ताहहॴ 23-Jan-2017

Options

1) Fats

वसा 2) Lipids

लिपपडस

3) Glucose

गिकह८ज

4) Cellulose

F A C E B O O K

P A G E h t t p w w w f a c e b o o k c o m s s c m e n t o r s o f f i c i a l P a g e | 31

FOR MORE UPDATES AND MORE MATERIAL DO LIKE OUR FACEBOOK PAGE httpwwwfacebookcomsscmentorsofficial

सहॳमयिह८ज

Correct Answer Glucose

Intestinal absorption of end products

from digestion of carbohydrates and

proteins in the pig hellip During absorption some sugars (fructose or

galactose) released from the

corresponding sucrose and lactose

respectively during digestion were

partly metabolized into glucose by the

enterocyte

Q130 Which of the following glands is a

source of the enzyme Ptyalin

नननननलिखितगरचियोमसहॳएजाइमटयालिनकासरह८तहहॴ 23-Jan-2017

Options

1) Pancreas

अगरािय

2) Thyroid Gland

िाइराइिगरिी 3) Pituitary Gland

पीयषगरिी 4) Salivary Glands

िारगरचियाा Correct Answer Salivary Glands

Q131 Which of the following is not true

about Pteridophyta

ननननमसहॳकह९नसीबातटहॳररिह८फाईटकहॳ बारहॳमसचनहीहहॴ 23-Jan-2017

Options

1) Dominant phase is saprophytes

परमिचरणसहॳपरह८फाईइटसहह८ताहहॴ 2) Main plant body is diploid

पह९दह८कामखयिरीरदपवगखणतहह८ताहहॴ 3) Seeds are present

बीजमह९जदहह८तहॳहहॴ 4)Flowers are absent

फिअनपनसतिहह८तहॳहहॴ

Correct Answer Seeds are present

Q132 The largest dolphin species is the

orca also called as

िॉिकफनकीसबसहॳबड़ीपरजानतकाकानामआकायहहॴनजसहॳ mdash- भीकहतहॳहहॴ 23-Jan-2017

Options

1) Bottle Nose

बाटिनह८ज

2) Baiji

बहॳजी 3) Killer whale

ककिरहहॳि

4)Tucuxi

टकवसी Correct Answer Killer whale

Q133 The fat digesting enzyme Lipase

is secreted by which of the following

वसाकापाचनकरनहॳवािाएजाइमिाइपहॳजनननननलिखितमसहॳककसकहॳ दवारासतरापवतहह८ताहहॴ

24-Jan-2017

Options

1) Kidneys

गद

2) Pancreas

अगनयािय

3) Large Intestine

बड़ीआत

4)Liver

नजगर

Correct Answer Pancreas

Lipase is an enzyme that splits fats so

the intestines can absorb them Lipase

hydrolyzes fats like triglycerides into

their component fatty acid and glycerol

molecules It is found in the blood

gastric juices pancreatic secretions

intestinal juices and adipose tissues

F A C E B O O K

P A G E h t t p w w w f a c e b o o k c o m s s c m e n t o r s o f f i c i a l P a g e | 32

FOR MORE UPDATES AND MORE MATERIAL DO LIKE OUR FACEBOOK PAGE httpwwwfacebookcomsscmentorsofficial

Q134 The arrangement of leaves on an

axis or stem is called

एकअकषयातनहॳपरपनततयोकीयवसिाकह८कयाकहाजाताहहॴ SSC CHSL Science (biology) 2016

Question Paper

24-Jan-2017

Options

1) Phyllotaxy

फाइिह८टहॴकसी 2) Vernation

वनिन

3) Venation

वहॳनहॳिन

4)Phytotaxy

फाइटह८टहॴकसी Correct Answer Phyllotaxy

In botany phyllotaxis or phyllotaxy is

the arrangement of leaves on a plant

stem (from Ancient Greek phyacutellon

ldquoleafrdquo and taacutexis ldquoarrangementrdquo)

Phyllotactic spirals form a distinctive

class of patterns in nature

Q135 The study of Cells is also known

as

कह८लिकाओकहॳ अधययनकह८ mdashmdashndash

भीकहाजाताहहॴ 24-Jan-2017

Options

1) Cytology

सायटह८िह८जी 2) Physiology

कफनजयह८िह८जी 3) Nucleology

नयककमयह८िह८जी 4)Cellology

सहॳिह८िह८जी Correct Answer Cytology

Q136 Which of the following scientists

is also known as the Father of Biology

नननननलिखितमसहॳककसवहॴजञाननककह८ ldquoजीवपवजञानकहॳ जनकrdquoकहॳ नामसहॳभीजानाजाताहहॴ 24-Jan-2017

Options

1) Herbert Spencer

हबयटयसपसर

2) Aristotle

अरसत 3) Lamarck

िहॳमाकय 4)Darwin

िापवयन

Correct Answer Aristotle

Q137 Which cells give rise to various

organs of the plant and keep the plant

growing

कह९नसीकह८लिकाएपह९धह८कहॳ लभननअगह८कह८जनमदहॳतीहहॴऔरपह९धह८कह८बढ़नहॳममददकरतीहहॴ

24-Jan-2017

Options

1) Permanent

सिायी 2) Dermal

तवचीय

3) Meristematic

मररसटहॳमटटक

4)Mature

परह८ढ़

Correct Answer Meristematic

A meristem is the tissue in most plants

containing undifferentiated cells

(meristematic cells) found in zones of

the plant where growth can take place

Q138 Rodentia Muridae is the scientific

name of

F A C E B O O K

P A G E h t t p w w w f a c e b o o k c o m s s c m e n t o r s o f f i c i a l P a g e | 33

FOR MORE UPDATES AND MORE MATERIAL DO LIKE OUR FACEBOOK PAGE httpwwwfacebookcomsscmentorsofficial

रह८िहॳलियानयररिी mdashmdash- कावहॴजञाननकनामहहॴ 24-

Jan-2017

Options

1) Mouse

चहा 2) Squirrel

चगिहरी 3) Monkey

बदर

4) Lizard

नछपकिी Correct Answer Mouse

Q139 Name the scientist who proposed

the cell theory

कह८लिकालसदातकापरसतावदहॳनहॳवािहॳवहॴजञाननककानामबताइए 24-Jan-2017

Options

1) Schleiden and Schwann

िीमिनऔरशरववान

2) Lamarck

िहॳमाकय 3) Treviranus

टरहॳवायरहॳनस

4)Whittaker and Stanley

हीटकरऔरसटहॳनिहॳ Correct Answer Schleiden and

Schwann

Q140 The flower with the worldrsquos

largest bloom is

दननयाकासबसहॳबड़ाफिखििनहॳवािा mdashmdashndash हहॴ 24-Jan-2017

Options

1) Pando

पािह८ 2) Posidonia

पह८सीिह८ननया 3) Rafflesia arnoldii

ररफिहॳलियाअनोमिी 4)Helianthus annuus

हहॳलिएनिसएनयअस

Correct Answer Rafflesia arnoldii

Rafflesia arnoldii is a species of

flowering plant in the parasitic genus

Rafflesia It is noted for producing the

largest individual flower on earth It has

a very strong and horrible odour of

decaying flesh earning it the nickname

ldquocorpse flower

Q141 Deficiency of which vitamin

causes night blindness

ककसपवटालमनकीकमीकहॳ कारणरतौधीहह८ताहहॴ 24-Jan-2017

Options

1) Vitamin K

पवटालमन K

2) Vitamin C

पवटालमन C

3) Vitamin B1

पवटालमन B1

4)Vitamin A

पवटालमन A

Correct Answer Vitamin A

Q142 Nongreen plants lack which of the

following

गहॴर-

हररतवनसपनतमनननननलिखितमसहॳककसकीकमीहह८तीहहॴ

24-Jan-2017

Options

1) Chlorophyll

किह८रह८कफि

2) Lycophyll

िायकह८कफि

3) Cyanophyll

F A C E B O O K

P A G E h t t p w w w f a c e b o o k c o m s s c m e n t o r s o f f i c i a l P a g e | 34

FOR MORE UPDATES AND MORE MATERIAL DO LIKE OUR FACEBOOK PAGE httpwwwfacebookcomsscmentorsofficial

सायनह८कफि

4)Phototropism

फह८टह८टरोपपजम

Correct Answer Chlorophyll

Q143 Organisms that use light to

prepare food are known as

जह८जीवपरकािकाउपयह८गकरभह८जनतहॴयारकरतहॳहहॴ उनह mdashmdash- कहॳ पमजानजाताहहॴ 24-Jan-2017

Options

1) Autotrophs

सवपह८षी 2) Heterotrophs

पवषमपह८षज

3) Omnivores

सवायहारी 4)Decomposers

पवघटनकरनहॳवािा Correct Answer Autotrophs

autotrophs often make their own food

by using sunlight carbon dioxide and

water to form sugars which they can use

for energy Some examples of

autotrophs include plants algae and

even some bacteria Autotrophs

(producer) are important because they

are a food source for heterotrophs

(consumers)

A heterotroph is an organism that

ingests or absorbs organic carbon

(rather than fix carbon from inorganic

sources such as carbon dioxide) in order

to be able to produce energy and

synthesize compounds to maintain its

life Ninety-five percent or more of all

types of living organisms are

heterotrophic including all animals and

fungi and some bacteria

Q144 Which of the following is a

primary function of haemoglobin

नननननलिखितमसहॳकह९नसाटहमह८गिह८बबनकाएकपरािलमककाययहहॴ

25-Jan-2017

Options

1) Utilization of energy

उजायकाउपयह८गकरना 2) Prevention of anaemia

रकतामपताहह८नहॳसहॳरह८कना 3) Destruction of bacteria

बहॴकटीररयाकापवनािकरना 4) To transport oxygen

ऑकसीजनकावहनकरना Correct Answer To transport oxygen

Q145 Vascular bundles are absent in

सवहनीबिि mdashmdash- मअनपनसतिरहतहॳहहॴ 25-Jan-2017

Options

1) Bryophyta

िायह८फाइटा 2) Pteridophyta

टहॳररिह८फाईटा 3) Gymnosperms

नजननह८सपमय 4) Angiosperms

एननजयह८सपहॳनसय Correct Answer Bryophyta

Q146 Sauria Lacertidae is the scientific

name of

सहॴररयािहॳसरटाईिी mdashmdashndash कावहॴजञाननकनामहहॴ 25-Jan-2017

Options

1) Crocodile

मगरमचछ

2) Hippopotamus

टहपपह८पह८टहॳमस

3) Lizard

नछपकिी 4) House fly

F A C E B O O K

P A G E h t t p w w w f a c e b o o k c o m s s c m e n t o r s o f f i c i a l P a g e | 35

FOR MORE UPDATES AND MORE MATERIAL DO LIKE OUR FACEBOOK PAGE httpwwwfacebookcomsscmentorsofficial

घरहॳिमकिी Correct Answer Lizard

Q147 Which type of pathogen causes

the water-borne disease SARS (Severe

Acute Respiratory Syndrome)

ककसपरकािकारह८गज़नकजिजननतबीमारीसासयकाकारणबनताहहॴ 25-Jan-2017

Options

1) Viral

वायरि

2) Parasitic

परजीवी 3) Protozoan

परह८टह८जअन

4) Bacterial

बहॴकटीररयि

Correct Answer Viral

Q148 Which of the following organs

produces the enzyme lipase

नननननलिखितमसहॳकह९नसाअगिायपहॳजएजाइमउतपननकरताहहॴ 25-Jan-2017

Options

1) Pancreas

अगनयािय

2) Large Intestine

बड़ीआत

3) Liver

नजगर

4) Small Intestine

छह८टीआत

Correct Answer Pancreas

Q149 A is a long internode forming the

basal part or the whole of a peduncle

एक mdashmdash- एकिबाइटरनह८िहहॴ जह८ननचिाटहससायासनपणयिठिबनताहहॴ 25-

Jan-2017

Options

1) Rhizome

परकद

2) Rachis

महॳ दि

3) floral axis

पषपअकष

4) Scape

भगदड़

Correct Answer scape

Q150 ndash Which of the following

organisms are considered to be both

Living and Non-living

नननननलिखितमसहॳकह९नसहॳजीवाणकह८जीपवतऔरअजीपवतमानाजाताहहॴ

25-Jan-2017

Options

1) Bacteria

बहॴकटीररया 2) Fungi

कवक

3) Algae

िहॴवाि

4)Virus

वायरस

Correct Answer Virus

They are considered to be living as they

possess a protein coat as a protective

covering DNA as the genetic material

etc

They are said to be non-living as they

can be crystallised and they survive for

billions of years They can tolerate high

temperatures freezing cold

temperatures ultra-violet radiations etc

Q151 Deficiency of fluorine causes

which of the following

फिह८ररनकीकमीकहॳ कारणनननननलिखितमसहॳकयाहह८ताहहॴ

F A C E B O O K

P A G E h t t p w w w f a c e b o o k c o m s s c m e n t o r s o f f i c i a l P a g e | 36

FOR MORE UPDATES AND MORE MATERIAL DO LIKE OUR FACEBOOK PAGE httpwwwfacebookcomsscmentorsofficial

27-Jan-2017

Options

1) Dental Caries

िटिकहॴ ररज

2) Scurvy

सकवरी 3) Anaemia

रकतामपता 4) Arthritis

गटठया Correct Answer Dental Caries

Q152 In a Punnett Square with the

cross AaBb x AaBb how many Aabb

genotypes would be created

पनहॳटसककायरमिह८स AaBb x AaBb कहॳ साि

ककतनहॳ Aabb जीनह८टाइपबनगहॳ 27-Jan-2017

Options

1) 1

2) 8

3) 2

4) 3

Correct Answer 2

Q153 Which of the following is the

Controlling Center of the Cell

नननननलिखित म सहॳ कह८लिकाका ननयतरण

क दर कह९न हहॴ

27-Jan-2017

Options

1) Nucleus

क दर

2) Plasma

पिाजमा 3) Lysosome

िायसह८सह८म

4) Chromosome

िह८मह८सह८म

Correct Answer Nucleus

The control centre of the cell is the

nucleus in eukaryotic cells The nucleus

contains genetic material in the form of

DNA

Q154 Myopia affects which of the

following organs

मायह८पपयानननननलिखितअगह८मसहॳककसहॳपरभापवतकरताहहॴ

25-Jan-2017

Options

1) Heart

हदय

2) Skin

तवचा 3) Eyes

आािहॳ 4)Mouth

मह

Correct Answer Eyes

Q155 Which of the following bears

flowers

नननननलिखितमसहॳकह९नफिधारणकरताहहॴ

25-Jan-2017

Options

1) Bryophyta

िायह८फाइटा 2) Pteridophyta

टहॳरीिह८फाईटा 3) Gymnosperms

नजननह८सपमय 4)Angiosperms

एननजयह८सपमय Correct Answer Angiosperms

Q156 Oxygenated blood flows out of the

heart through the

ऑकसीजनयकतरकत mdashmdashmdash

कहॳ माधयमसहॳहदयकहॳ बाहरबहताहहॴ 25-Jan-2017

F A C E B O O K

P A G E h t t p w w w f a c e b o o k c o m s s c m e n t o r s o f f i c i a l P a g e | 37

FOR MORE UPDATES AND MORE MATERIAL DO LIKE OUR FACEBOOK PAGE httpwwwfacebookcomsscmentorsofficial

Options

1) Aorta

महाधमनी 2) pulmonary artery

फहॳ फड़हॳकीधमनी 3) vena cava

वहॳनाकावा 4)Atrium

चह९क

Correct Answer aorta

Q157 Blood leaving the liver and

moving towards the

heart has a higher concentration of

नजगरसहॳननकिकरहदयकीतरफजानहॳवािहॳरकतम mdashmdashmdashmdash कीउचचसादरताहह८तीहहॴ 27-Jan-2017

Options

1) Lipids

लिपपडस

2) Urea

यररया 3) Bile Pigments

पपततकहॳ रगकरण

4) Carbon dioxide

काबयनिायऑकसाइि

Correct Answer Bile Pigments

Urea is nitrogen containing substance

which is produced in the liver in order

to deal with excess amino-acids in the

body As urea is produced it leaves the

liver in the blood stream and passes via

the circulatory system to all parts of the

body

Q158 Bulb is a modification of which

part of a plant

बमबएकपह९धहॳकहॳ ककसटहससहॳकाएक पातरणहह८ताहहॴ 27-Jan-2017

Options

1) The root

जड़

2) The stem

तना 3) The radicle

मिाकर

4)The fruit

फि

Correct Answer The stem

Q159 Which of the following carries

blood away from the heart to different

body parts

इनमहॳसहॳकह९नरकतकह८हदयसहॳिरीरकहॳ पवलभननअगह८तकिहॳजातीहहॴ

27-Jan-2017

Options

1) Arteries

धमननया 2) Nerves

तबतरहाए

3) Capillaries

कहॳ लिकाए

4)Veins

नसहॳ Correct Answer Arteries

Q160 The series of processes by which

nitrogen and its compounds are

interconverted in the environment and

in living organisms is called

27-Jan-2017

Options

1)Absorption of Nitrogen

2)Ammonification

3)Nitrogen Fixation

4)Nitrogen Cycle

Correct Answer Nitrogen Cycle

Ammonification or Mineralization is

performed by bacteria to convert

organic nitrogen to ammonia

F A C E B O O K

P A G E h t t p w w w f a c e b o o k c o m s s c m e n t o r s o f f i c i a l P a g e | 38

FOR MORE UPDATES AND MORE MATERIAL DO LIKE OUR FACEBOOK PAGE httpwwwfacebookcomsscmentorsofficial

Nitrification can then occur to convert

the ammonium to nitrite and nitrate

Nitrogen fixation is a process by which

nitrogen in the Earthrsquos atmosphere is

converted into ammonia (NH3) or other

molecules available to living organisms

Q161 BCG vaccine is given to protect

from which of the following

बीसीजीकाटटकानननननलिखितमसहॳककसकहॳ बचावकहॳ लिएटदयाजातहहॴ

27-Jan-2017

Options

1) Jaundice

पीलिया 2) Anaemia

रकतमपता 3) Tuberculosis

कषयरह८ग

4) Polio

पह८लियह८ Correct Answer Tuberculosis

Q162 Parallel venation is found in

समानतरवहॳनहॳिन mdashmdashmdash- मपायाजाताहहॴ 27-Jan-2017

Options

1) plants which are monocots

पह९धहॳजह८एकबीजपतरीहह८तहॳहहॴ 2) plants which have a dicot stem

वहॳपह९धहॳनजनकातनादपवदलियहह८ताहहॴ 3) plants with leaves similar to Tulsi

वहॳपह९धहॳनजनकीपनततयतिसीकीपनततयोकहॳ समानहह८तहॳहहॴ 4)plants with tap roots

टहॳप टवािहॳपह९धहॳ Correct Answer plants which are

monocots

Q163 The hardest part of the body is

िरीरकासबसहॳकठह८रभाग mdashndash हहॴ 27-Jan-2017

Options

1) Bones

हडडिय

2) Tooth Enamel

दातकहॳ इनहॳमि

3) Skull

िह८पड़ी 4) Spinal Cord

महॳ रजज

Correct Answer Tooth Enamel

Q164 Which type of pathogen causes

the waterborne disease E coli Infection

ककसपरकारकारह८गजननकजिजननतरह८गईकह८िाईसिमणकाकारणबनताहहॴ 27-Jan-2017

Options

1) Protozoan

परह८टह८जआ

2) Parasitic

परजीवी 3) Bacterial

बहॴकटीररयि

4)Viral

वायरि

Correct Answer Bacterial

Q165 The amount of blood filtered

together by both the kidneys in a 70 kg

adult male human in a minute is

70 की गरा वािहॳएकवयसकप षमएकलमनटमदह८नोगदकहॳदवाराएकसािचाबनीगयीरकतकीमातरहह८तीहहॴ 29-Jan-2017

Options

1) 1100 ml

1100 लमलि

2) 100 ml

F A C E B O O K

P A G E h t t p w w w f a c e b o o k c o m s s c m e n t o r s o f f i c i a l P a g e | 39

FOR MORE UPDATES AND MORE MATERIAL DO LIKE OUR FACEBOOK PAGE httpwwwfacebookcomsscmentorsofficial

100 लमलि

3) 1500 ml

1500 लमलि

4) 500 ml

500 लमलि

Correct Answer 1100 ml

Q166 Which feature of a plant helps to

distinguish a monocot from a dicot

पह९धहॳकीवहकह९नसीपविहॳषताहहॴजह८एकदपवदलियहॳऔरएकएकदिीयपह९धहॳसहॳभहॳदकरनहॳममददकरतीहहॴ 29-Jan-2017

Options

1) Pollination

परागम

2) Venation

वहॳनहॳिन

3) Vernation

वनिन

4) Aestivation

एसटीवहॳिहॳन

Correct Answer venation

Q167 The Mutation Theory was

proposed by

उतवररवतयनकालसदात mdashmdashndash

कहॳ दवरापरसतापवतककयाजाताहहॴ 29-Jan-2017

Options

1) Charles Lyell

चामसयलियहॳि

2) William Smith

पवलियमनसमि

3) Hugo De Vries

हयगह८िीराईस

4)Harrison Schmitt

हहॳरीसननसमट

Correct Answer Hugo De Vries

Q168 Which type of pathogen causes

the waterborne disease HepatitisA

ककसपरकारकहॳ रह८गजनकजिजननतरह८गहहॳपहॳटाइटटस-A काकारणबनताहहॴ

29-Jan-2017

Options

1) Parasitic

परजीवी 2) Viral

वायरि

3) Protozoan

परह८टह८जआ

4) Bacterial

बहॴकटीररयि

Correct Answer Viral

Q169 In a Punnett Square with the

cross AaBb x Aabb how many AaBb

genotypes would be created

पनहॳटसकवायरमिह८स AaBb x Aabb

कहॳ सािककतनहॳ AaBb जीनह८टाइपबनगहॳ 29-Jan-

2017

Options

1) 4

2) 1

3) 7

4) 6

Correct Answer 4

Q170 Arboreal Ateles is the scientific

name of

अिह८ररयिएटटलिस mdashmdashmdash कावहॴजञाननकनामहहॴ 29-Jan-2017

Options

1) Squirrel

चगिहरी 2) Sparrow

गह८रहॴया 3) Lizard

नछपकिी 4) Spider monkey

F A C E B O O K

P A G E h t t p w w w f a c e b o o k c o m s s c m e n t o r s o f f i c i a l P a g e | 40

FOR MORE UPDATES AND MORE MATERIAL DO LIKE OUR FACEBOOK PAGE httpwwwfacebookcomsscmentorsofficial

मकड़ीबदर

Correct Answer Spider monkey

Q171 Which type of pathogen causes

the waterborne disease Salmonellosis

ककसपरकारकारह८गाणजिजननतबीमारीसािमह८नहॳिह८लसज़काकारकहहॴ

29-Jan-2017

Options

1) Algal

िहॳवालियहॳ 2) Parasitic

परजीवी 3) Bacterial

बहॴकटीररयि

4)Viral

वायरि

Correct Answer Bacterial

An infection with salmonella bacteria

commonly caused by contaminated food

or water

Symptoms include diarrhoea fever

chills and abdominal pain

Q172 is a condition in which there is a

deficiency of red cells or of haemoglobin

in the blood

mdashmdash-

एकनसिनतहहॴनजसमहॳरकतमिािकह८लिकाओकीयाहीमह८गिह८बबनकीकमीहह८तीहहॴ 29-Jan-2017

Options

1) Albinism

एनमबननजम

2) Propyria

परह८पीररया 3) Anaemia

एनीलमया 4)Keloid disorder

कहॳ िह८इिडिसओिर

Correct Answer Anaemia

Q173 Ananas comosus is the scientific

name of

Options

अनानासकह८मह८सस mdashmdashmdashndash

कावहॴजञाननकनामहहॴ 29-Jan-2017

1) Custard Apple

सीताफि

2) Pineapple

पाइनएपपि

3) Bamboo

बास

4)Pomegranate

अनार

Correct Answer Pineapple

Q174 Which organ produces insulin

कह९नसाअगइनसलिनपहॴदाकरताहहॴ 29-Jan-

2017

Options

1) Liver

यकत

2) Thyroid gland

िायराइिगरिी 3) Spleen

पिीहा 4)Pancreas

अगरयिय

Correct Answer Pancreas

Q175 Which of the following disease is

not caused by water pollution

नननननलिखितमसहॳकह९नसारह८गपानीकहॳ परदषणकहॳकारणनहीहह८ता

29-Jan-2017

Options

1) Cholera

हहॴजा 2) Typhoid

F A C E B O O K

P A G E h t t p w w w f a c e b o o k c o m s s c m e n t o r s o f f i c i a l P a g e | 41

FOR MORE UPDATES AND MORE MATERIAL DO LIKE OUR FACEBOOK PAGE httpwwwfacebookcomsscmentorsofficial

टाइफाइि

3) Asthma

दमा 4)Diarrhoea

दसत

Correct Answer Asthma

Q176 Ocimum tenuiflorum is the

scientific name of

ओलिलममटहॳयईफिह८रमइसकावहॴजञाननकनाम mdash

ndash हहॴ 30-Jan-2017

Options

1) Neem

नीम

2) Mango

आम

3) Babul

बबि

4)Tulsi

तिसी Correct Answer Tulsi

Q177 Which gland secretes bile a

digestive fluid

कह९नसीगरिीपपतत एकपाचनतरिपरदािय सरापवतकरतीहहॴ 30-Jan-2017

Options

1) Pancreas

अगनयािय

2) Liver

यकत

3) Thyroid

िायराइि

4) Testes

टहॳनसटस

Correct Answer liver

Q178 In which of the following the

dominant phase is Gametophyte

नननननलिखितमसहॳककसकहॳ परमिचरणयगमकह८दपवधद (Gametophyte)हहॴ 30-Jan-2017

Options

1) Bryophyta

िायह८फाइटा 2) Pteridophyta

टहॳररिह८फाइटा 3) Gymnosperms

नजननह८सपमय 4) Angiosperms

एननजयह८सपमय Correct Answer Bryophyta

Q179 Anaerobic respiration refers to

which of the following

नननननलिखितमसहॳककसहॳअवायवीयशवसनकहाजाताहहॴ

30-Jan-2017

Options

1) Respiration without Oxygen

ऑकसीजनकहॳ बबनाशवसन

2) Respiration with Oxygen

ऑकसीजनकहॳ सािशवसन

3) Respiration without CO2

काबयनिायऑकसाइिकहॳ बबनाशवसन

4) Respiration with CO2

काबयनिायऑकसाइिकहॳ सािशविन

Correct Answer Respiration without

Oxygen

Q180 Which type of pathogen causes

the waterborne disease Cholera

ककसपरकारकारह८गजनकजिजननतरह८गहहॴजाकाकारणबनताहहॴ

30-Jan-2017

Options

1) Algal

िहॴवालियहॳ

F A C E B O O K

P A G E h t t p w w w f a c e b o o k c o m s s c m e n t o r s o f f i c i a l P a g e | 42

FOR MORE UPDATES AND MORE MATERIAL DO LIKE OUR FACEBOOK PAGE httpwwwfacebookcomsscmentorsofficial

2) Bacterial

बहॴकटीररयि

3) Protozoan

परह८टह८जआ

4) Viral

वायरि

Correct Answer Bacterial

Q181 To which class does

Oxyreductases transferases hydrolases

belong

ओकसीररिकटहॳसटरासफरहॳजहॳस

हाइडरह८िहॳसहॳसककसवगयमआतहॳहहॴ 30-Jan-2017

Options

1) Hormones

हारमोस

2) Enzymes

एजाइनस

3) Proteins

परह८टीनस

4) Vitamins

पवटालमनस

Correct Answer Enzymes

Q182 Which of the following is not true

about Gymnosperms

ननननमसहॳकह९नसीबातअनावतबीजीकहॳ बारहॳमसचनहीहहॴ 30-Jan-2017

Options

1) Dominant phase is saprophytes

परमिचरणसहॳपरह८फाइटसहह८ताहहॴ 2) Vascular bundles are absent

सवहनीबििअनपनसितहह८ताहहॴ 3) spores are heterospores

बीजाणहहॳटहॳरह८सपह८रसहह८तहॳहहॴ 4) Flowers are absent

फिअनपनसितहह८तहॳहहॴ

Correct Answer Vascular bundles are

absent

Q183 The name of first mammal clone sheep is

भहॳड़कीपरिमसतनपायीपरनत प (किह८न)

कानामहहॴ 30-Jan-2017

Options

1) Noori

नरी 2) Dolly

िॉिी 3) Louise

िसी 4)Durga

दगाय Correct Answer Dolly

Q184 Which type of pathogen causes

the water-borne disease Typhoid fever

ककसपरकारकारह८गजनकजिजननतरह८गटाइफाइिबिारकाकारणबनताहहॴ 30-Jan-2017

Options

1) Algal

िहॴवािीय

2) Parasitic

परजीवी 3) Protozoan

परह८टह८जनअन

4)Bacterial

बहॴकटीररयि

Correct Answer Bacterial

Q185 In which part of the cell are

proteins made

कह८लिकाकहॳ ककसटहससहॳमपरह८टीनबनायाजाताहहॴ

31-Jan-2017

Options

1) Reticulum

रहॳटटकिम

F A C E B O O K

P A G E h t t p w w w f a c e b o o k c o m s s c m e n t o r s o f f i c i a l P a g e | 43

FOR MORE UPDATES AND MORE MATERIAL DO LIKE OUR FACEBOOK PAGE httpwwwfacebookcomsscmentorsofficial

2) Golgi apparatus

गह८मजीएपहॳरहॳटस

3) Ribosomes

ररबह८सह८नस

4) Lysosome

िायसह८सह८नस

Correct Answer ribosomes

Proteins are produced by stringing

amino acids together in the order

specified by messenger RNA strands

that were transcribed from DNA in the

cell nucleus The process of synthesizing

a protein is called translation and it

occurs on ribosomes in the cytoplasm of

a cell

Q186 Polio is a disease caused by which

of the following

नननननलिखितमसहॳपह८लियह८कीबबमारह८हह८नहॳकाकारणकयाहहॴ

31-Jan-2017

Options

1) Bacteria

बहॴकटीररयि

2) Mosquito

मचछर

3) Virus

वायरस

4) Cockroach

नतिच हॳ Correct Answer Virus

Polio or poliomyelitis is a crippling and

potentially deadly infectious disease It

is caused by the poliovirus

Q187 ndash Hay fever is a sign of which of

the following

हहॳकफवरनननननलिखितमसहॳककसकाएकसकहॳ तहहॴ

31-Jan-2017

Options

1) Old Age

वदावसिा 2) Malnutrition

कपह८सण

3) Allergy

एिनजय 4) Over Work

अतयचधककाययकरना Correct Answer Allergy

Q188 How many chromosomes does a

human cell contain

एकमानवकह८लिकामककतनहॳगणसतरहह८तहॳहहॴ

29-Jan-2017

Options

1) 6

2) 26

3) 46

4) 66

Correct Answer 46

In humans each cell normally contains

23 pairs of chromosomes for a total of

46 Twenty-two of these pairs called

autosomes look the same in both males

and females The 23rd pair the sex

chromosomes differ between males and

females

Q189 Which of the following is not true

about Bryophyta

ननननमसहॳकह९नसीबातिायह८फाइटकहॳ बारहॳमसचनहीहहॴ 31-Jan-2017

Options

1) Dominant phase is gametophytes

परमिचरणगहॳलमतह८फाइटसहह८ताहहॴ 2) Main plant body is haploid

पह९धहॳकामखयिरीरअगखणतहह८ताहहॴ 3) Spores are homospores

बीजाणहह८मह८सफह८रसहह८तहॳहहॴ 4) Flowers are present

फिमह८जदहह८तहॳहहॴ Correct Answer Flowers are present

F A C E B O O K

P A G E h t t p w w w f a c e b o o k c o m s s c m e n t o r s o f f i c i a l P a g e | 44

FOR MORE UPDATES AND MORE MATERIAL DO LIKE OUR FACEBOOK PAGE httpwwwfacebookcomsscmentorsofficial

Q190 Which aquatic animal has

trailing tentacles

ककसजिीयजानवरकहॳ पीछहॳचिनहॳवािहॳटहॳटकिसहह८तहॳहहॴ

31-Jan-2017

Options

1) Sea horse

समदरीघह८िा 2) Corals

मगा 3) Jelly fish

जहॳिीमछिी 4) Star fish

तारामछिी Correct Answer Jelly fish

Jellyfish with its umbrella-shaped bell

and trailing tentacles

Q191 Which type of pathogen causes

the water-borne disease Poliomyelitis

(Polio)

ककसपरकारकारह८गजनकजिजननतरह८गपह८लियह८मायहॳटटस (पह८लियह८) काकारणहहॴ 31-Jan-

2017

Options

1) Parasitic

परजीवी 2) Algal

िहॴवालिय

3) Viral

वायरि

4) Bacterial

बहॴकटीररयि

Correct Answer Viral

Q192 The outer white part of the eye

that protects the inner structures is

आािकाबाहरीसफहॳ दटहससाजह८आतररकसरचनाओकीरकषाकरताहहॴ वह mdashmdashmdash हहॴ 31-Jan-

2017

Options

1) Iris

आयररस

2) Sclera

सकिहॳरा 3) Retina

रहॳटटना 4) Cornea

कह८ननयया Correct Answer Sclera

Q193 Proteins are made up of

परह८टीनकाननमायण mdashndash सहॳहह८ताहहॴ 31-Jan-2017

Options

1) Amino acids

एलमनह८अनि

2) Fatty acids

वसायकतअनि

3) Glucose

गिकह८ज

4)Nucleotides

नयनकियह८टाईिस

Correct Answer Amino acids

Q194 Moringa Oleifera is the scientific

name of

मह८ररगओलिफहॳ रा mdashmdashndash कावहॴजञाननकनामहहॴ 31-Jan-2017

Options

1) Banyan

बरगद

2) Gulmohar

गिमह८हर

3) Amla

आमिा

F A C E B O O K

P A G E h t t p w w w f a c e b o o k c o m s s c m e n t o r s o f f i c i a l P a g e | 45

FOR MORE UPDATES AND MORE MATERIAL DO LIKE OUR FACEBOOK PAGE httpwwwfacebookcomsscmentorsofficial

4) Drumstick

डरमनसटक

Correct Answer Drumstick

Q195 Kidney stones are composed of

गदकीपिरी mdashndash सहॳबनीहह८तीहहॴ 1-Feb-2017

Options

1) Calcium Oxalate

कहॴ नमसयमओकजहॳिहॳट

2) Sodium Chloride

सह८डियमकिह८राइि

3) Magnesium Nitrate

महॳनगनलियमनाइतटरहॳट

4) Calcium Bicarbonate

कहॴ नमियमबायकबोनहॳट

Correct Answer Calcium Oxalate

Q196 ndash Which of the following is not

true about Angiosperms

ननननमसहॳकह९नसीबातआवतबीजीकहॳ बारहॳमसचनहीहहॴ 1-Feb-2017

Options

1) Dominant phase is gametophytes

परमिचरणगहॳलमतह८फाइटहह८ताहहॴ 2) Vascular bundles are present

सवहनीबििमह९जदहह८ताहहॴ 3) Spores are heterospores

बीजाणहहॳटहॳरह८सपह८रसहह८तहॳहहॴ 4) Seeds are covered

बीजढकहॳ हह८तहॳहहॴ Correct Answer Dominant phase is

gametophytes

Q197 All of the following are excretory

(waste) products of animals except

नननननलिखितमसहॳककसएककह८छह८ड़करअनयसभीपराखणयोदवाराउतसनजयतपदाियहहॴ 1-Feb-

2017

Options

1) Uric Acid

यररकएलसि

2) Ammonia

अमह८ननया 3) Carbohydrates

काबोहाइडरहॳट

4) Urea

यररया Correct Answer Carbohydrates

In animals the main excretory products

are carbon dioxide ammonia (in

ammoniotelics) urea (in ureotelics) uric

acid (in uricotelics) guanine (in

Arachnida) and creatine

Q198 RNA is a polymeric molecule

What does RNA stand for

आरएनइएएकबहिकआणहहॴ इसकाकापवय पकयाहहॴ 1-Feb-2017

Options

1) Rado Nuclear Acid

रािह८नयनकियरएलसि

2) Ribo Nucleic Acid

राइबह८नयनकिकएलसि

3) Rhino Nuclear Acid

हाइनह८नयनकियरएलसि

4) Resto Nucleus Acid

रहॳसटह८नयकिीयसएलसि

Correct Answer Ribo Nucleic Acid

Q199 Which organ does detoxification

and produces chemicals needed for

digestion

कह९नसाअगपवषहरणकरताहहॴऔरपाचनकहॳ लिएआवशयकरसायनोकह८पहॴदाकरताहहॴ 1-Feb-

2017

Options

1) Salivary glands

िारगरचिया 2) Pancreas

अगनयािय

F A C E B O O K

P A G E h t t p w w w f a c e b o o k c o m s s c m e n t o r s o f f i c i a l P a g e | 46

FOR MORE UPDATES AND MORE MATERIAL DO LIKE OUR FACEBOOK PAGE httpwwwfacebookcomsscmentorsofficial

3) Thyroid gland

िायराइिगरिी 4) Liver

यकत

Correct Answer Liver

Q200 Psidium guajava is the scientific

name of

लसडियमगआजावा mdashmdash कावहॴजञाननकनामहहॴ 1-

Feb-2017

Options

1) Guava

अम द

2) Mango

आम

3) Bamboo

बास

4) Jack fruit

कटहि

Correct Answer Guava

Q201 Which drug is used as a Blood

Thinner

चधरकह८पतिाकरनहॳकहॳ पमककसदवाकापरयह८गककयाजाताहहॴ

1-Feb-2017

Options

1) Warfarin

वाफर न

2) Tramadol

टरहॳमािह८ि

3) Azithromycin

एनजरह८मायलसन

4) Hydralazine

हाइडरह८िहॳनजन

Correct Answer Warfarin

Q202 Which of the following disease is

caused due to the deficiency of protein

परह८टीनकीकमीकहॳ कारणनननननलिखितमसहॳकह९नसारह८गहह८ताहहॴ 1-Feb-2017

Options

1) Arthritis

गटठया 2) Kwashiorkor

कािीओकय र

3) Goitre

गाइटर

4) Night Blindness

रतह९चध

Correct Answer Kwashiorkor

Q203 A is species of plant that has

adapted to survive in an environment

with little liquid water

mdashmdashndashपह९धहॳकीएकऐसहॳऐसहॳपरजानतहहॴ नजसनहॳकमपानीवािहॳवातावरणमजीपवतरहनहॳकहॳलिएअनकिनहहॴ 1-Feb-2017

Options

1) Xerophyte

म दपवद

2) Hydrophyte

जिीयपादप

3) Mesophyte

समह८दपवद

4) Thallophyte

िहॴिह८फाइटा Correct Answer xerophyte

xerophyte is a species of plant that has

adapted to survive in an environment

with little liquid water such as a desert

or an ice- or snow-covered region in the

Alps or the Arctic

Mesophytes are terrestrial plants which

are adapted to neither a particularly

dry nor particularly wet environment

An example of a mesophytic habitat

would be a rural temperate meadow

F A C E B O O K

P A G E h t t p w w w f a c e b o o k c o m s s c m e n t o r s o f f i c i a l P a g e | 47

FOR MORE UPDATES AND MORE MATERIAL DO LIKE OUR FACEBOOK PAGE httpwwwfacebookcomsscmentorsofficial

which might contain goldenrod clover

oxeye daisy and Rosa multiflora

thallophyte any of a group of plants or

plantlike organisms (such as algae and

fungi) that lack differentiated stems

leaves and roots and that were formerly

classified as a primary division

(Thallophyta) of the plant kingdom

Q204 How many types of teeth are

there in humans

मनषयोमककतनहॳपरकारकहॳ दातहह८तहॳहहॴ

1-Feb-2017

Options

1) 4

2) 5

3) 2

4) 3

Correct Answer 4

teeth -Humans have four types of

teethincisors canines premolars and

molars each with a specific function

The incisors cut the food the canines

tear the food and the molars and

premolars crush the food

Q205 Carica papaya is the scientific name of

कहॴ ररकापपाया mdashmdashndash कावहॴजञाननकनामहहॴ 2-

Feb-2017

Options

1) Peepal

पीपि

2) Papaya

पपीता 3) Tamarind

इमिी 4) Drumstick

ढह८िकाछड़ी Correct Answer Papaya

Q206 Muscles get tired when there is

shortfall of

जब mdashndash कीकमीहह८तीहहॴतबपहॳिीयिकजातीहहॴ 2-Feb-2017

Options

1) Lactic acid

िहॴनकटकएलसि

2) Na+ ions

Na+ आयन

3) ATP

एटीपी 4) Sulphates

समफहॳ टस

Correct Answer ATP

ATP is the energy source muscle fibers

use to make muscles contract

muscle tissuersquos main source of energy

called adenosine triphosphate or ATP

As your muscles use up this energy

source they become tired and fatigued

Oxygen is the key ingredient that helps

create new ATP to replenish the burned

up ATP in your muscles

Q207 Artocarpus integra is the

scientific name of आटह८कापयसइटीगरा mdashmdashmdash कावहॴजञाननकनामहहॴ 2-Feb-2017

Options

1) Guava

अम द

2) Pineapple

अनानास

3) Silver Oak

लसमवरओक

4) Jack fruit

कटहि

Correct Answer Jack fruit

Q208 Which organ stores fat soluble

vitamins

कह९नसाअगवसामघिनिीिपवटालमनह८काभिाराकरताहहॴ

2-Feb-2017

F A C E B O O K

P A G E h t t p w w w f a c e b o o k c o m s s c m e n t o r s o f f i c i a l P a g e | 48

FOR MORE UPDATES AND MORE MATERIAL DO LIKE OUR FACEBOOK PAGE httpwwwfacebookcomsscmentorsofficial

Options

1) Blood

रकत

2) Skin

तवचा 3) Liver

यकत

4) Pancreas

अगनयािय

Correct Answer Liver

Q209 Which disease is caused due to

deficiency of Iodine

आयह८िीनकहॳ कारणकह९नसारह८गहह८ताहहॴ 2-Feb-2017

Options

1) Rickets

ररकहॳ टस

2) Scurvy

सकवी 3) Goitre

गणमािा 4) Growth retardation

पवकासका कना Correct Answer Goitre

rickets A softening and weakening of

bones in children usually due to

inadequate vitamin D

Q210 Grevillea Robusta is the scientific name of

गरहॳपवलियारह८बसटा mdashmdashmdash- कापवजञाननकनामहहॴ 2-Feb-2017

Options

1) Peepal

पीपि

2) Teak

सागह९न

3) Silver Oak

लसमवरओक

4) Jack fruit

कटहि

Correct Answer Silver Oak

Q211 When a Cuttlefish is described as a Molluscs it is at which level of

classification

जबएककटिकफिकह८एकमह८िसकाकहॳ पमवखणयतककयाजाताहहॴतबयहॳवगीकरणकहॳ ककससतरपहॳनसितहहॴ 2-Feb-2017

Options

1) Class

वगय 2) Order

िम

3) Family

पररवार

4) Phylum

सघ

Correct Answer Phylum

Q212 Bambusa dendrocalmus is the

scientific name of बानबसािहॳडराकामस mdashmdashmdash कावहॴजञाननकनामहहॴ 3-Feb-2017

Options

1) Banyan

बरगद

2) Papaya

पपीता 3) Bamboo

बास

4) Pomegranate

अनार

Correct Answer Bamboo

Q213 Acinonyx Jubatus is the scientific name of

एलसनह८ननकसजयबहॳटस mdashmdashmdash

कावहॴजञाननकनामहहॴ 3-Feb-2017

F A C E B O O K

P A G E h t t p w w w f a c e b o o k c o m s s c m e n t o r s o f f i c i a l P a g e | 49

FOR MORE UPDATES AND MORE MATERIAL DO LIKE OUR FACEBOOK PAGE httpwwwfacebookcomsscmentorsofficial

Options

1) Bear

भाि 2) Horse

घह८िा 3) Cheetah

चीता 4) Zebra

जहॳिा Correct Answer Cheetah

Q214 The pale yellow colour of urine is

due to the presence of which pigment

मतरकाफीकापीिारगरगदरयकहॳ उपनसिनतकहॳ कारणहह८ताहहॴ

3-Feb-2017

Options

1) Urochrome

यरह८िह८म

2) Urophyll

यरह८कफि

3) Chlorophyll

किह८रह८कफि

4) Chloroplast

किह८रह८पिासट

Correct Answer Urochrome

Q215 Which of the following constitute

to form a gene

नननननलिखितमसहॳकह९नसीचीज़एकजीनकागठनकरतीहहॴ

3-Feb-2017

Options

1) Polynucleotides

पह८िीनयनकियह८टाईडस

2) Hydrocarbons

हाइडरह८काबोस

3) Lipoproteins

िाईपह८परह८टीनस

4) Lipids

लिपपडस

Correct Answer Polynucleotides

Polynucleotide molecule is a biopolymer

composed of 13 or more nucleotide

monomers covalently bonded in a chain

DNA (deoxyribonucleic acid) and RNA

(ribonucleic acid) are examples of

polynucleotides with distinct biological

function

Q216 Vertebrates belongs to the

phylum

रीढ़कीहडिीवािहॳपराणी mdashmdashmdash

परजानतकहॳ अतगायतआतहॳहहॴ 3-Feb-2017

Options

1) Arthropoda

आरह८पह८ड़ा 2) Annelida

एननलििा 3) Cnidaria

ननिहॳररया 4) Chordata

कह८िटा Correct Answer Chordata

Q217 Punica granatum is the scientific name of

पननकगरहॳनहॳटस mdashmdashmdash कावहॴजञाननकनामहहॴ 3-Feb-2017

Options

1) Custard Apple

सीताफि

2) Gulmohar

गिमह८हर

3) Silver Oak

लसमवरओक

4) Pomegranate

अनार

Correct Answer Pomegranate

F A C E B O O K

P A G E h t t p w w w f a c e b o o k c o m s s c m e n t o r s o f f i c i a l P a g e | 50

FOR MORE UPDATES AND MORE MATERIAL DO LIKE OUR FACEBOOK PAGE httpwwwfacebookcomsscmentorsofficial

Q218 Between a tiger and an monkey

which of the following is different

एकबाघऔरबदरकहॳ बीचनननननलिखितमसहॳकह९नसीबातअिगहहॴ 3-Feb-2017

Options

1) Kingdom

राजय

2) Phylum

जानत

3) Order

िम

4) Class

वगय Correct Answer order

Q219 The artificial heart was invented by

कबतरमहदयका mdashmdashmdash

दवाराअपवषकारककयागयािा 3-Feb-2017

Options

1) Muhammad Yunus

महनमदयनस

2) Linus Yale Jr

िाइनसयहॳिजय

3) Gazi Yasargil

गाजीयासचगयि

4) Paul Winchell

पह९िपवमकि Correct Answer Paul Winchell

Q220 Tamarindus indica is the

scientific name of

टहॳमररनडसइडिका mdashmdash कावहॴजञाननकनामहहॴ 7-

Feb-2017

Options

1) Neem

नीम

2) Pineapple

अनानास

3) Tamarind

इमिी 4)Chiku

चीक

Correct Answer Tamarind

Q221 In eukaryotic cells synthesis of

RNA takes place in the

यकहॳ योटटककह८लिकाओमआरएनएकासशिहॳषण

mdashndash महह८ताहहॴ 7-Feb-2017

Options

1) Mitochondria

माईटह८कोडडरया 2) Centrioles

सटरीयह८मस

3) Ribosomes

ररबह८सह८नस

4) Nucleus

नयनकियस

Correct Answer nucleus

eukaryotic cell -Transcription is the

process of synthesizing ribonucleic acid

(RNA)Synthesis takes place within the

nucleus of eukaryotic cells or in the

cytoplasm of prokaryotes and converts

the genetic code from a gene in

deoxyribonucleic acid ( DNA ) to a

strand of RNA that then directs

proteinsynthesis

Q222 _________is caused by parasites

of the Plasmodium genus

पिाजमह८डियमजातीकहॳ परजीवी mdash- कहॳ कारणहहॴ 7-Feb-2017

Options

1) Dysentery

पहॳचचि

2) Malaria

मिहॳररया 3) Chickenpox

F A C E B O O K

P A G E h t t p w w w f a c e b o o k c o m s s c m e n t o r s o f f i c i a l P a g e | 51

FOR MORE UPDATES AND MORE MATERIAL DO LIKE OUR FACEBOOK PAGE httpwwwfacebookcomsscmentorsofficial

चहॳचक

4) Herpes

हहॳपपयस

Correct Answer Malaria

Q223 Carotene in fruits and vegetables

gives it which color

फिह८औरसनलजयोमनसितकहॳ रह८टीनउनहकह९नसारगपरदानकरताहहॴ 7-Feb-2017

Options

1) Green

हरा 2) Pink

गिाबी 3) Orange

नारगी 4) Blue

नीिा Correct Answer Orange

Q224 Equus Caballus is the scientific

name of

एकवसकहॴ बहॳिस mdashmdashndash कापवजञाननकनामहहॴ 7-Feb-2017

Options

1) Horse

घह८िा 2) Zebra

ज़हॳिा 3) Donkey

गधा 4) Buffalo

भस

Correct Answer Horse

Q225 Elapidae Naja is the scientific name of

एिीपीिीनाजा mdashmdash- कावहॴजञाननकनामहहॴ 8-Feb-2017

Options

1) Cobra

कह८बरा 2) Elephant

हािी 3) Eagle

ग ि

4) Owl

उमि Correct Answer Cobra

Q226 Which disease is caused due to

deficiency of Iron

िह८हकीकमीकहॳ कारणकह९नसारह८गहह८ताहहॴ 8-Feb-

2017

Options

1) Beriberi

बहॳरीबहॳरी 2) Tetany

टहॳटनी 3) Kwashiorkor

कवािीऔरकर

4) Anaemia

रकतामपता Correct Answer Anaemia

Beriberi is a disease caused by a vitamin

B-1 deficiency also known as thiamine

deficiency

Tetany can be the result of an

electrolyte imbalance Most often itrsquos a

dramatically low calcium level also

known as hypocalcemia Tetany can also

be caused by magnesium deficiency or

too little potassium Having too much

acid (acidosis) or too much alkali

(alkalosis) in the body can also result in

tetany

Kwashiorkor also known as

ldquoedematous malnutrition It is a form of

malnutrition caused by a lack of protein

in the diet

Anaemia means that you have fewer red

blood cells than normal or you have less

F A C E B O O K

P A G E h t t p w w w f a c e b o o k c o m s s c m e n t o r s o f f i c i a l P a g e | 52

FOR MORE UPDATES AND MORE MATERIAL DO LIKE OUR FACEBOOK PAGE httpwwwfacebookcomsscmentorsofficial

haemoglobin than normal in each red

blood cell

Q227 is a leaf where the leaflets are

arranged along the middle vein

mdashndashएकपततीहहॴजहापतरकह८कीरचनाक ररयालिराकहॳ आसपासहह८तीहहॴ 8-Feb-2017

Options

1) Pinnately compound leaf

पपनहॳटिीसयकतपतती 2) Palmately compound leaf

पामहॳटिीसयकतपतती 3) Compound leaf

सयकतपतती 4) Simple leaf

साधारणपतती Correct Answer Pinnately compound

leaf

Q228 Haustoria or sucking roots are

found in which of the following

हह८सटह८ररयायाचसनहॳवािीजड़हॳनननननलिखितमसहॳककसमपाईजातीहहॴ 8-Feb-2017

Options

1) Wheat

गहॳह

2) Mango

आम

3) Chestnut

चहॳसटनट

4) Cuscuta

कसकयटा Correct Answer Cuscuta

Haustorial roots -The roots of parasitic

plants which penetrate into the host

tissues to absorb nourishment are

called haustorial roots hellip Also known as suckingor parasitic roots

Q229 Equs Asinus is the scientific name

of

एकवसएलसनस mdashmdashndash कावहॴजञाननकनामहहॴ 8-

Feb-2017

Options

1) Donkey

गधा 2) Cow

गाय

3) Deer

टहरन

4) Kangaroo

कगा

Correct Answer Donkey

Q230 Ficus benghalensis is the scientific name of

फाईकसबहॳनगहॳिहॳलसस mdashndash कापवजञाननकनामहहॴ 8-Feb-2017

Options

1) Banyan

बरगद

2) Pineapple

अनानास

3) Babul

बबि

4) Tulsi

तिसी Correct Answer Banyan

Q231 Equus burchellii is the scientific name of

एकवसबचिी mdashmdash- कापवजञाननकनामहहॴ 8-Feb-2017

Options

1) Horse

घह८िा 2) Zebra

जहॳिा 3) Buffalo

F A C E B O O K

P A G E h t t p w w w f a c e b o o k c o m s s c m e n t o r s o f f i c i a l P a g e | 53

FOR MORE UPDATES AND MORE MATERIAL DO LIKE OUR FACEBOOK PAGE httpwwwfacebookcomsscmentorsofficial

भस

4) Ass

गधा Correct Answer Zebra

Page 10: COMPILATION OF ALL 72 SETS OF BIOLOGY SSC CHSL-2016 · OF BIOLOGY SSC CHSL-2016 PREPARED BY : SSC MENTORS BIOLOGY SPECIAL . F A C E B O O K P A G E : h t t p : / / w w w . f a c e

F A C E B O O K

P A G E h t t p w w w f a c e b o o k c o m s s c m e n t o r s o f f i c i a l P a g e | 9

FOR MORE UPDATES AND MORE MATERIAL DO LIKE OUR FACEBOOK PAGE httpwwwfacebookcomsscmentorsofficial

अरोपह८िा 2) Cnidaria

नीिहॳररया 3) Echinodermata

इकाईनह८िमटा 4) Chordata

कह८िटा Correct Answer Arthropoda

Q33 Pulses are a rich source of which of

the following

दािहॳनननननलिखितमसहॳककसकीपरचरसह८तरहहॴ

11-Jan-2017

Options

1) Carbohydrates

काबोहाइडराईट

2) Proteins

परह८टीनस

3) Minerals

िननज

4) Vitamin A

पवटालमन A

Correct Answer Proteins

Q34 Plant cell wall is made up of

वनसपनतकह८लिकालभनततइससहॳबनीहह८तीहहॴ

11-Jan-2017

Options

1) Cellulose

सहॳमयिह८ज

2) Glucose

गिकह८ज

3) Sucrose

सिह८ज

4) Fructose

फरकटह८ज

Correct Answer Cellulose

Plant cell wall the major carbohydrates

are cellulose hemicellulose and pectin

The cellulose microfibrils are linked via

hemicellulosic tethers to form the

cellulose-hemicellulose network which

is embedded in the pectin matrix

Q35 The study of Fungi is also known

as कवकह८कहॳ अधययनकह८कहाजाताहहॴ

11-Jan-2017

Options

1) Cytology

सायटह८िह८जी 2) Myology

मायह८िह८जी 3) Mycology

मायकह८िह८जी 4) Neurology

नयरह८िह८जी Correct Answer Mycology

Cytology - structure and function of

plant and animal cells

Myology is the study of the muscular

system

Neurology is the branch of medicine

concerned with the study and treatment

of disorders of the nervous system

Q36 The outermost layer of skin is

तवचाकीसबसहॳबाहरीपरतकयाहह८तीहहॴ 11-Jan-

2017

Options

1) Epidermis

इपपिलमयस

2) Dermis

िलमयस

3) Tissues

ऊतक

4) Hypodermis

हायपह८िलमयस

Correct Answer Epidermis

Q37 Which of the following plants have

root nodules

F A C E B O O K

P A G E h t t p w w w f a c e b o o k c o m s s c m e n t o r s o f f i c i a l P a g e | 10

FOR MORE UPDATES AND MORE MATERIAL DO LIKE OUR FACEBOOK PAGE httpwwwfacebookcomsscmentorsofficial

नननननलिखितपह९धह८मसहॳककसकीजड़ह८मगाठहह८तीहहॴ

11-Jan-2017

Options

1) Leguminous plants

िहॳगयलमनसपह९धहॳ 2) Parasitic plants

परजीवीपह९धहॳ 3) Epiphytic Plants

एपीफाइटटकपह९धहॳ 4) Aquatic Plants

जिीयपह९धहॳ Correct Answer Leguminous plants

Q38 Earth-worms belongs to the

phylum

कहॳ चएmdashmdash- परजानतकहॳ अतगयतआतहॳहहॴ 11-Jan-2017

Options

1) Protozoa

परह८टह८जआ

2) Cnidaria

नीिहॳररया 3) Annelida

एनीलििा 4) Mollusca

मह८िसका Correct Answer Annelida

Q39 Ringworm is a disease caused by

ररगवमयनामकबीमारी mdashmdash- कहॳ कारणहह८तीहहॴ 11-Jan-2017

Options

1) Fungi

कवक

2) Bacteria

बहॴकटीररया 3) Virus

वायरस

4) Flies

मनकियाा Correct Answer Fungi

Q40 Mangifera indica is the scientific

name of

मननगफहॳ राइडिकाककसकावहॴजञाननकनामहहॴ 11-

Jan-2017

Options

1) Guava

अम द

2) Mango

आम

3) Amla

आविा 4) Jack fruit

कटहि

Correct Answer Mango

Q41 Crabs belongs to the phylum

कहॳ कड़हॳmdashmdash- परजानतकहॳ अतगयतआतहॳहहॴ 11-Jan-2017

Options

1) Mollusca

मह८िसका 2) Cnidaria

नीिहॳररया 3) Arthropoda

अरोपह८ड़ा 4) Platyhelminthes

पिहॳटटहहॳनममननिस

Correct Answer Arthropoda

Q42 Myopia is a defect of eyes which is

also known as

मायह८पपयाआिोकादह८षहहॴ नजसहॳ mdashmdashndash

भीकहाजाताहहॴ

12-Jan-2017

Options

1) Far Sightedness

F A C E B O O K

P A G E h t t p w w w f a c e b o o k c o m s s c m e n t o r s o f f i c i a l P a g e | 11

FOR MORE UPDATES AND MORE MATERIAL DO LIKE OUR FACEBOOK PAGE httpwwwfacebookcomsscmentorsofficial

दरदनषटदह८ष

2) Near Sightedness

ननकटदनषटदह८ष

3) Astigmatism

एसटीगमहॳटटजम

4) Night Blindness

रतोधी Correct Answer Near Sightedness

Myopia occurs when the eyeball is too

long relative to the focusing power of

the cornea and lens of the eye This

causes light rays to focus at a point in

front of the retina rather than directly

on its surface

Hyperopia Hypermetropia (

Farsightedness )- when light rays

entering the eye focus behind the retina

rather than directly on it The eyeball of

a farsighted person is shorter than

normal

Astigmatism usually is caused by an

irregularly shaped cornea Instead of

the cornea having a symmetrically

round shape (like a baseball) it is

shaped more like an American football

Nyctalopia also called night ndash blindness

is a condition making it difficult or

impossible to see in relatively low light

Q43 Who is known as the father of

Green Revolution

हररतिानतकहॳ जनककहॳ पमककसहॳजानाजाताहहॴ

12-Jan-2017

1) Dr Robert Nucleus

िॉ रॉबटयनयनकियस

2) Dr Ian Wilmut

िॉ इयानपविमट

3) Dr NE Borlaug

िॉ एनईबह८रिॉग

4) Dr JC Bose

िॉ जहॳसीबह८स

Correct Answer Dr NE Borlaug

Q44 Panthera Tigris is the scientific

name of

पिहॳराटटगरीस mdashmdashmdash कावहॴजञाननकनामहहॴ 12-Jan-2017

Options

1) Panther

तदआ

2) Tiger

बाघ

3) Whale

हहॳि

4)Goat

बकरी Correct Answer Tiger

Q45 How many facial bones are there

हमारहॳचहॳहरहॳमककतनीहडडियााहह८तीहहॴ 13-Jan-2017

Options

1)34

2)24

3)14

4)4

Correct Answer 14

Q46 ndash Halophytes are plants that grow

in

हहॴिह८फाईटसवहॳपह९धहॳहह८तीहहॴजह८ mdash- मउगतहॳहहॴ SSC CHSL Science (biology) 2016

Question Paper

13-Jan-2017

Options

1) Fresh Water

ताजापानी 2) Cold Water

ठिापानी 3) Ponds

तािाब

4) Salt Water

िारापानी Correct Answer Salt Water

F A C E B O O K

P A G E h t t p w w w f a c e b o o k c o m s s c m e n t o r s o f f i c i a l P a g e | 12

FOR MORE UPDATES AND MORE MATERIAL DO LIKE OUR FACEBOOK PAGE httpwwwfacebookcomsscmentorsofficial

Q47 Felis Catus is the scientific name of

फहॳ लिसकहॴ टस mdashndash कावहॴजञाननकनामहहॴ 13-Jan-2017

Options

1) Cat

बबमिी 2) Dog

कतता 3) Mouse

चहा 4) Porcupine

साही Correct Answer Cat

Q48 Which of the following induces

nitrogen fixation in soil

नननननलिखितमसहॳकह९नलम ीमनाइटरह८जनननयतनकह८परहॳररतकरताहहॴ

15-Jan-2017

Options

1) Protozoa

परह८टह८जआ

2) Bacteria

बहॴकटीररया 3) Fungi

कवक

4)Algae

िहॴवाि

Correct Answer Bacteria

Bacteria that change nitrogen gas from

the atmosphere into solid nitrogen

usable by plants are called nitrogen-

fixing bacteria These bacteria are

found both in the soil and in symbiotic

relationships with plants

They contain symbiotic bacteria called

rhizobia within nodules in their root

systems producing nitrogen compounds

that help the plant to grow and compete

with other plants When the plant dies

the fixed nitrogen is released making it

available to other plant

Q49 Which of the following is the

largest known cell

नननननलिखितमसहॳकह९नसीसबसहॳबड़ीजञातकह८लिकाहहॴ

SSC CHSL Science (biology) 2016

Question Paper

15-Jan-2017

1) Eukaryotic Cell

यकहॳ ररयह८टटककह८लिका 2) Prokaryotic Cell

परह८कहॳ ररयह८टटककह८लिका 3) Mycoplasma

मायकह८पिासम

4) Ostrich Eggs

ितरमगयकाअिा Correct Answer Ostrich Eggs

Q50 The association of animals in

which both the partners are benefitted

is known as

जानवरोकावहसहयह८गनजसमहॳदह८नोभागीदारिाभापवनतहह८तहॳहहॴ उसहॳ mdashmdashndash कहॳ पमजानाजाताहहॴ SSC CHSL Science (biology) 2016

Question Paper

15-Jan-2017

Options

1) Amensalism

सहजीपवत

2) Commensalism

परजीपवत

3) Colony

कॉिनी 4) Mutualism

अनयह८नयाशरयवाद

Correct Answer Mutualism

Amensalism is any relationship between

organisms of different species in which

F A C E B O O K

P A G E h t t p w w w f a c e b o o k c o m s s c m e n t o r s o f f i c i a l P a g e | 13

FOR MORE UPDATES AND MORE MATERIAL DO LIKE OUR FACEBOOK PAGE httpwwwfacebookcomsscmentorsofficial

one organism is inhibited or destroyed

while the other organism remains

unaffected

Commensalism an association between

two organisms in which one benefits and

the other derives neither benefit nor

harm

Q51 Pneumonia affects which of the

following organs of human body

ननमह८ननयामानविरीरकहॳ नननननलिखितमसहॳककसअगकह८परभापवतकरताहहॴ

15-Jan-2017

Options

1)Kidneys

गद

2)Lungs

फहॳ फड़हॳ 3) Throat

गिहॳ 4) Liver

यकत

Correct Answer Lungs

When the germs that cause pneumonia

reach your lungs the lungsrsquo air sacs

(alveoli) become inflamed and fill up

with fluid This causes the symptoms of

pneumonia such as a cough fever

chills and trouble breathing When you

have pneumonia oxygen may have

trouble reaching your blood

Q52 Mendel is known as

मििकह८ mdashmdash- कहॳ पमजानाजाताहहॴ 15-Jan-2017

Options

1) Father of Physiology

िरीरकियािासतरकहॳ जनक

2) Father of Geology

भगभयिासतरकहॳ जनक

3) Father of Genetics

जहॳनहॳटटकसकहॳ जनक

4) Father of Biology

जीविासतरकहॳ जनक

Correct Answer Father of Genetics

Q53 Which of the following are also

known as Suicidal bag of Cells

ननननलिखितमसहॳककसहॳआतमहतयाकरनहॳवािीकह८लिकाओकाबहॴगकहाजाताहहॴ

15-Jan-2017

Options

1) Lysosomes

िायसोसह८म

2) Lycosome

िायकह८सह८म

3) Nucleus

नालभक

4) Chromosome

िह८मह८सह८म

Correct Answer Lysosomes

Q54 Mesothelioma is a type of cancer

The most common area affected in it is

the lining of the ________

लमज़ह८िहॳिहॳलमयाक सरकाएकपरकारहहॴ इससहॳपरभापवतहह८नहॳवािासबसहॳसामानयकषहॳतर mdash

mdashmdash काअसतरहहॴ 15-Jan-2017

Options

1)Heart

हदय

2)Brain

मनसतषक

3)Stomach

आमािय

4)Lungs

फहॳ फड़हॳ Correct Answer lungs

Asbestos exposure is the main cause of

pleural mesothelioma When asbestos

fibers are breathed in they travel to the

F A C E B O O K

P A G E h t t p w w w f a c e b o o k c o m s s c m e n t o r s o f f i c i a l P a g e | 14

FOR MORE UPDATES AND MORE MATERIAL DO LIKE OUR FACEBOOK PAGE httpwwwfacebookcomsscmentorsofficial

ends of small air passages and reach the

pleura where they can cause

inflammation and

scarring

Q55 Which one of the following is an

insectivorous plant

नननननलिखितमसहॳकह९नसाएकककटाहरीवनसपनतहहॴ

15-Jan-2017

Options

1) Utricularia

यटरीकिहॳररया 2) Sequoia

सहॳकयओइया 3) Nostoc

नॉसटह८क

4) Bryophyta

िायह८फाईटा Correct Answer Utricularia

Q56 ______________ is a

multibranched polysaccharide of

glucose that serves as a form of energy

storage in animals and fungi

mdashmdashगिकह८जकाएकबहिािायकतपह८िीसहॳकहॳ राइिहहॴ जह८जानवरोऔरकवकमउजायभणिारणकहॳ एक पमकाययकरताहहॴ 15-Jan-2017

Options

1) Cellulose

सहॳमयिह८ज

2) Glycogen

गिायकह८जन

3) Pectin

पहॳनकटन

4) Chitin

चीटटन

Correct Answer Glycogen

Q57 The largest gland of the human

body is

mdashmdashmdashमानविरीरकीसबसहॳबड़ीगरिीहहॴ 16-Jan-2017

Options

1) Pancreas

अगयािय

2) Thyroid

िायरॉइि

3) Large Intestine

बड़ीआत

4) Liver

यकत

Correct Answer Liver

Q58 Photosynthesis in plants takes

place in

वनसपनतयोमपरकािसशिहॳषणकीकियाहह८तीहहॴ

16-Jan-2017

Options

1) Stem

तना 2) Leaves

पनततयाा 3) Roots

जड़हॳ 4) Flower

फि

Correct Answer Leaves

During this reaction carbon dioxide

and water are converted into glucose

and oxygen The reaction requires light

energy which is absorbed by a green

substance called

chlorophyll Photosynthesis takes place

in leaf

cells These contain chloroplasts which

are tiny objects containing chlorophyll

F A C E B O O K

P A G E h t t p w w w f a c e b o o k c o m s s c m e n t o r s o f f i c i a l P a g e | 15

FOR MORE UPDATES AND MORE MATERIAL DO LIKE OUR FACEBOOK PAGE httpwwwfacebookcomsscmentorsofficial

Q59 Insects that transmit diseases are

known as

जह८कीड़हॳरह८गसचाररतकरतहॳहहॴ उनह mdashmdash-

कहॳ नामसहॳजानाजाताहहॴ 16-Jan-2017

1)Pathogens

रह८गज़नक

2) Vectors

वहॳकटर

3) Drones

परजीवी 4)Scalars

अटदषट

Correct Answer Vectors

A vector is an organism that does not

cause disease itself but which spreads

infection by conveying pathogens from

one host to another Species of mosquito

for example serve as vectors for the

deadly disease Malaria

Q60 Which is the second largest gland

of Human body

मानविरीरकीदसरीसबसहॳबड़ीगरिीकह९नसीहहॴ

SSC CHSL Science (biology)

2016 Question Paper

16-Jan-2017

Options

1) Liver

यकत

2) Large Intestine

बड़ीआत

3) Thorax

छाती 4) Pancreas

अगनयािय

Correct Answer Pancreas

Q61 Annona squamosa is the scientific

name of

एनह८नासकवामह८सा (Annona squamosa) mdash

mdashmdash कावहॴजञाननकनामहहॴ 16-Jan-2017

Options

1) Custard Apple

सीताफि

2) Papaya

पपीता 3) Babhul

बबि

4) Drumstick

सहजन

Correct Answer Custard Apple

Q62 The disease Beri Beri is caused due

to the deficiency of which of the

following

बहॳरीबहॳरीरह८गनननननलिखितमसहॳककसकीकमीकहॳकारणहह८ताहहॴ

16-Jan-2017

Options

1) Vitamin B2

पवटालमन B2

2) Vitamin B1

पवटालमन B1

3) Vitamin B12

पवटालमन B12

4) Vitamin E

पवटालमन E

Correct Answer Vitamin B1

Beriberi is a disease caused by a vitamin

B-1 deficiency also known as thiamine

deficiency

Q63 Chlorophyll was first isolated and

named by

किह८रह८कफिकह८ mdash-

दवारापहिहॳपिकऔरनालमतककयागया 16-Jan-2017

F A C E B O O K

P A G E h t t p w w w f a c e b o o k c o m s s c m e n t o r s o f f i c i a l P a g e | 16

FOR MORE UPDATES AND MORE MATERIAL DO LIKE OUR FACEBOOK PAGE httpwwwfacebookcomsscmentorsofficial

Options

1) Caventou

कहॳ वहॳत 2) Pelletier

पहॳिहॳटटयर

3) Chlorophyll

किह८रह८कफि

4) Caventou and Pelletier

कहॳ वहॳतऔरपहॳिहॳटटयर

Correct Answer Caventou and Pelletier

Chlorophyll was first isolated and

named by

Joseph Bienaimeacute Caventou and Pierre

Joseph Pelletier in 1817 The presence of

magnesium in chlorophyll was

discovered in 1906 and was the first

time that magnesium had been detected

in living tissue

Q64 Which of the following organisms

does not fit into the Cell Theory

नननननलिखितमसहॳकह९नसाजीवकह८लिकालसदातअन पनहीहहॴ

16-Jan-2017

Options

1) Bacteria

बहॴकटीररया 2) Virus

वायरस

3) Fungi

कवक

4) Plants

पह९धहॳ Correct Answer Virus

The bottom line is that viruses are not

alive and not related to cells in any way

The cell theory states that all living

things are made of cells cells are the

basic units of structure and function of

living things and that all cells come

from other cells Since viruses are not

made of cells and do not use cells in any

of their processes they are not related to

the cell theory

Q65 Which of these is not a

macronutrient for Plants

नननननलिखितमसहॳकह९नसापह९धह८कहॳ लिएमिह८नयटरीएटनहीहहॴ

SSC CHSL Science (biology) 2016

Question Paper

17-Jan-2017

Options

1) Nitrogen

नाइटरह८जन

2) Phosphorus

फासफह८रस

3) Potassium

पह८टालसयम

4) Chlorine

किह८रीन

Correct Answer Chlorine

In relatively large amounts the soil

supplies nitrogen phosphorus

potassium calcium magnesium and

sulfur these are often called the

macronutrients In relatively small

amounts the soil supplies iron

manganese boron molybdenum

copper zinc chlorine and cobalt the

so-called micronutrients

Q66 Name the respiratory organs of

insects

कीटह८मनसतिशरवसनअगनामकानामहहॴ

17-Jan-2017

Options

1) Skin

तवचा 2) Body Surface

िरीरकीसतह

F A C E B O O K

P A G E h t t p w w w f a c e b o o k c o m s s c m e n t o r s o f f i c i a l P a g e | 17

FOR MORE UPDATES AND MORE MATERIAL DO LIKE OUR FACEBOOK PAGE httpwwwfacebookcomsscmentorsofficial

3) Gills

गिफड़हॳ 4) Tracheae

शरावस- निी Correct Answer Tracheae

Air enters the respiratory systems of

insects through a series of external

openings called

spiracles These external openings

which act as muscular valves in some

insects lead to the internal respiratory

system a densely networked array of

tubes called tracheae

Q67 The poisonous gas accidentally

released in Bhopal Gas Tragedy is

भह८पािगहॴसतरासदीमगितीसहॳमकतहईजहरीिीगहॴसिी

17-Jan-2017

1) Methane

मीिहॳन

2) Nitrous Oxide

नाइटरसऑकसाइि

3) Methyl Isocyanate

महॴचििआयसोसायनहॳट

4) Cyanogen

सायनह८जहॳन

Correct Answer Methyl Isocyanate

Q68 What does Trypsin do

टटरनपसनकयाकरताहहॴ

SSC CHSL Science (biology) 2016

Question Paper

17-Jan-2017

Options

1) Breaks down Carbohydrates

काबोहाइडरहॳटकापवघटनकरताहहॴ 2) Synthesizes proteins

परह८टीनकासििहॳषणकरताहहॴ 3) Breaks down fats

वसाकापवघटनकरताहहॴ 4) Breaks down proteins

परह८टीनकापवघटनकरताहहॴ Correct Answer Breaks down proteins

Trypsin is one of the three principal

digestive

proteinases the other two being pepsin

and

chymotrypsin In the digestive process

trypsin acts with the other proteinases

to break down dietary protein molecules

to their component

peptides and amino acids

A protease is any enzyme that performs

proteolysis protein catabolism by

hydrolysis of peptide bonds

Q69 Name the source from which

Aspirin is produced

उससरह८तकानामबताइए

नजससहॳएनसपररनकाउतपादनककयाजाताहहॴ

17-Jan-2017

Options

1) Willow bark

पविह८कीछाि

2) Oak Tree

ओककावकष

3) Acacia

बबि

4) Eucalyptus

नीिचगरी Correct Answer Willow bark

The compound from which the active

ingredient in aspirin was first derived

salicylic acid was found in the bark of a

willow tree in 1763 by Reverend

Edmund Stone of Chipping-Norton

Q70 Cannis Familiaris is the scientific

name of

कहॴ ननसफहॳ लमलियहॳररस mdash- कावहॴजञाननकनामहहॴ

17-Jan-2017

F A C E B O O K

P A G E h t t p w w w f a c e b o o k c o m s s c m e n t o r s o f f i c i a l P a g e | 18

FOR MORE UPDATES AND MORE MATERIAL DO LIKE OUR FACEBOOK PAGE httpwwwfacebookcomsscmentorsofficial

Options

1) Cat

बबमिी 2)Dog

कतता 3) Fox

िह८मड़ी 4) Wolf

भहॳडड़या Correct Answer Dog

Q71 Harmful bacteria in potable water

make the water

पीनहॳकहॳ पानीमनसतिघातकबहॴकटीररयाउसपानीकह८बनातहॳहहॴ 17-Jan-2017

Options

1) unfit to drink

पीनहॳकहॳ लिएअयह८गय

2) smelly

दगयनधयकत

3) Colored

रगीन

4) Turbid

मटमहॴिा Correct Answer unfit to drink

Q72 Musa paradisiaca is the scientific

name of which plant

मसापहॴराडिलसयाकाककसपह९धहॳकावहॴजञाननकनामहहॴ

17-Jan-2017

Options

1) Mango

आम

2) Wheat

गहॳह

3) Corn

भ ा 4) banana

कहॳ िा Correct Answer banana

Q73 Prawns belong to which family

झीगहॳककसपररवारकहॳ हह८तहॳहहॴ 17-Jan-2017

Options

1) Crustaceans

िसटहॳलियन

2)Fish

मछिी 3) Amphibians

अननफबबयस

4) Reptiles

रहॳपटाइमस

Correct Answer Crustaceans

Q74 Name the drug that is yielded from

Cinchona tree and is used to cure

malaria

उसऔषचधकानामबताइएनजसहॳलसगकह८नापहॳड़सहॳपरापतककयाजाताहहॴऔरनजसकाउपयह८गमिहॳररयाकहॳ उपचारमककयाजाताहहॴ 17-Jan-2017

Options

1) Camptothea

कहॴ नटह८चिया 2) Acuminata

एकयलमनहॳटा 3) Quinine

कनहॴन

4) Cinchonia

लसकह८ननया Correct Answer Quinine

Q75 Blood Circulation was discovered

by

रकतपररसचरणकी mdashmdashndash दवारािह८जकीिी 17-Jan-2017

Options

1) Mary Anderson

F A C E B O O K

P A G E h t t p w w w f a c e b o o k c o m s s c m e n t o r s o f f i c i a l P a g e | 19

FOR MORE UPDATES AND MORE MATERIAL DO LIKE OUR FACEBOOK PAGE httpwwwfacebookcomsscmentorsofficial

महॴरीएिरसन

2) Virginia Apgar

वनजयननयाएपगार

3) William Harvey

पवलियमहाव

4) Robert Feulgen

रॉबटयफ़यिजहॳन Correct Answer William Harvey

Q76 Vitamin A is also known as

पवटालमन A कह८ mdashmdash- कहॳ नामसहॳभीजानाजाताहहॴ SSC CHSL Science (biology) 2016

Question Paper

18Jan2017

Options

1) Thiamine

िायलमन

2) Riboflavin

ररबह८फिहॳपवन

3) Retinol

रहॳटटनॉि

4) Calciferol

कहॴ नमसफहॳ रह८ि

Correct Answer Retinol

Q77 Some roots called arise from an

organ other than the radicle

कछजड़हॳनजनह mdashmdashmdash कहाजाताहहॴ वहमिकहॳ अिावाककसीअनयअगसहॳउतपननहह८तीहहॴ 18Jan2017

Options

1) tap roots

मखयजड़

2) stilt roots

ि ाजड़

3) fibrous roots

रहॳिहॳदारजड़

4) adventitious roots

आकनसमकजड़

Correct Answer adventitious roots

Q78 Spiders belong to which class of

animals

मकडड़यापराणीवगीकरणकहॳ ककसवगयमआतीहहॴ 18Jan2017

Options

1) Arachnids

एरहॳकननडस

2) Aves

एपवस

3) Gastropods

गहॴसटरोपह८िस

4) Anthozoa

एिह८जआ

Correct Answer Arachnids

Q79 How many layers does Human

Skin have

मानवतवचामककतनीपरतहॳहह८तीहहॴ

18Jan2017

Options

1) 5

2) 7

3) 11

4) 3

Correct Answer 3

Skin has three layers The epidermis

the outermost layer of skin provides a

waterproof barrier and creates our skin

tone The dermis beneath the

epidermis contains tough connective

tissue hair follicles and sweat glands

The deeper subcutaneous tissue (

hypodermis ) is made of fat and

connective tissue

Q80 Allium Cepa is the scientific name

of

एलियमलसपपा mdashmdashndash कावहॴजञाननकनामहहॴ 18Jan2017

F A C E B O O K

P A G E h t t p w w w f a c e b o o k c o m s s c m e n t o r s o f f i c i a l P a g e | 20

FOR MORE UPDATES AND MORE MATERIAL DO LIKE OUR FACEBOOK PAGE httpwwwfacebookcomsscmentorsofficial

Options

1) Carrot

गाजर

2) Tomato

टमाटर

3) Potato

आि 4) Onion

पयाज़

Correct Answer Onion

Q81 DNA stands for

िीएनएकापणय प mdashmdash- हहॴ 18Jan2017

Options

1) Di Nucleic Acid

िाईनयनकिकएलसि

2) Deoxy Nucleic Acid

िीओकसीनयनकिकएलसि

3) Diribonucleic Acid

िाईराइबह८नयनकिकएलसि

4) Deoxyribonucleic Acid

िीऑकसीराइबह८नयनकिकएलसि

Correct Answer Deoxyribonucleic Acid

Q82 Organisms that generate energy

using light are known as

जह८जीवाणपरकािकाउपयह८गकरउजायउतपननकरतीहहॴ उनह mdashmdash कहॳ पमजानाजाताहहॴ

18Jan2017

Options

1) Chaemolithotrophs

ककमह८लििह८टरह८पस

2) Oligotrophs

ओलिगह८टरह८पस

3) Bacteria

बहॴकटीररया 4)Photoautotrophs

फह८टह८ओटह८टरह८पस

Correct Answer Photoautotrophs

An oligotroph is an organism that can

live in an environment that offers very

low levels of nutrients

Q83 Which drug is used as an

Antidepressant

ककसदवाएकहतािारह८धीकहॳ पमपयोगककयाजाताहहॴ Options

1) Oxybutynin

ओकसीलयटीनन

2)Tramadol

टरहॳमहॳिह८ि

3 ) Sumatriptan

समहॳटरीपटहॳन

4) Bupropion

लयपरह८पपयह८न

Correct Answer Bupropion

लयपरह८पपयह८न

Q84 The orange colour of carrot is

because of

गाजरकानारगीरगनननननलिखितमसहॳककसीएककीवजहसहॳहह८ताहहॴ 18Jan2017

Options

1) it grows in the soil

यहलम ीमउगतीहहॴ 2) Carotene

कहॴ रह८टीन

3) it is not exposed to sunlight

यहसययपरकािकहॳ सपकय मनहीआती 4) the entire plant is oranqe in colour

सनपणयपह९धानारगीरगकाहह८ताहहॴ Correct Answer Carotene

Q85 Snake venom is highly modified

saliva containing

F A C E B O O K

P A G E h t t p w w w f a c e b o o k c o m s s c m e n t o r s o f f i c i a l P a g e | 21

FOR MORE UPDATES AND MORE MATERIAL DO LIKE OUR FACEBOOK PAGE httpwwwfacebookcomsscmentorsofficial

सापकाजहरअततयाचधकसिह८चधतिारहह८तीहहॴनजसमहॳ mdashmdash- हह८ताहहॴ Options

l)Prototoxins

परह८टह८टॉनकसस

2)Neutrotoxins

नयटरोटॉनकसस

3)Zootoxins

जटॉनकसस

4)Electrotoxins

इिहॳकटरह८टॉनकसस

Correct Answer Zootoxins

जटॉनकसस

Q86 Which type of pathogen causes the

water-borne disease Schistosomiasis

ककसपरकारकारह८गज़नकजिजननतरह८गलससटह८सह८लमलससकाकारणबनताहहॴ

18Jan2017

Option

1) Parasitic

परजीवी 2)Protozoan

परह८टह८जआ

3) Bacterial

बहॴकटीररयि

4) Viral

वायरि

Correct Answer Parasitic

Schistosomiasis also known as snail

fever and bilharzia is a disease caused

by parasitic

flatworms called schistosomes

Q87 Prothrombin responsible for

clotting of blood is released by

परह८िह८ननबन

जह८रकतकािककाजमनहॳकहॳ लिएनजनमहॳदारहहॴ mdashndash

कहॳ दवारासतरापवतककयाजाताहहॴ

19Jan2017

Options

1) Small Intestine

छह८टीआत

2) Blood Platelets

रकतपिहॳटिहॳटस

3) Large Intestine

बड़ीआत

4Heart

हदय

Correct Answer Blood Platelets

Q88 Acacia arabica is the scientific

name of

अकहॳ लियाअरहॳबबका mdashmdashndash कावहॴजञाननकनामहहॴ 19-Jan-2017

Options

1) Neem

नीम

2) Teak

सागह९न

3) Babhul

बबि

4) Pomegranate

अनार

Correct Answer Babhul

Q89 Cannis Vulpes is the scientific

name of

कहॴ ननसवनमपस mdashmdash- कावहॴजञाननकनामहहॴ 19-Jan-2017

Options

1) Dog

कतता 2) Wolf

भहॳडड़या 3) Fox

िह८मड़ी 4) Hyena

िाकिबगघा

F A C E B O O K

P A G E h t t p w w w f a c e b o o k c o m s s c m e n t o r s o f f i c i a l P a g e | 22

FOR MORE UPDATES AND MORE MATERIAL DO LIKE OUR FACEBOOK PAGE httpwwwfacebookcomsscmentorsofficial

Correct Answer Fox

Q90 The beetroot is the portion of the

beet plant

चकदरपह९धहॳका mdashmdashndash भागहहॴ 19-Jan-2017

Options

1) tap root

मखयजड़

2) Adventitious

आकनसमक

3) bulb of the stem

तनहॳकाकद

4) Rhizome

परकद

Correct Answer tap root

Q91 What is the basic unit of heredity

आनवलिकताकीबननयादीइकाईकयाहहॴ 19-Jan-2017

Options

1) DNA

िीएनए

2) RNA

आरएनए

3) Chromosome

िह८मह८सह८म

4) Gene

जीन

Correct Answer gene

Genes are the units of heredity and are

the instructions that make up the bodyrsquos

blueprint They code for the proteins

that determine virtually all of a personrsquos

characteristics Most genes come in

pairs and are made of strands of genetic

material called deoxyribonucleic acid

or DNA

Q92 Lungs are the primary organs of

फहॳ फड़हॳmdashndashकहॳ परािलमकअगहहॴ

19-Jan-2017

Options

1) Digestion

पाचन

2) Constipation

कलज

3) Perspiration

पसीना 4)Respiration

शवसन

Correct Answer Respiration

Q93 Sugarcane is a type of

गननाएकपरकारका mdash- हहॴ 20-Jan-2017

Options

1)creeper

िता 2)tree

पहॳड़

3)shrub

झाड़ी 4)grass

घास

Correct Answer grass

Q94 Who is commonly known as ldquothe

Father of Microbiologyrdquo

सामानयत ldquo सकषमजीवपवजञानकहॳ जनक lsquo

कहॳ नामसहॳककसहॳजानाजातहहॴ 20-Jan-2017

Options

1) Robert Hooke

रॉबटयहक

2) Antonie Philips van Leeuwenhoek

एटह८नीकफलिपवानमयएनहह८क

3) Carl Linnaeus

काियिीनाईयस

4) Charles Darwin

चामसयिापवयन

F A C E B O O K

P A G E h t t p w w w f a c e b o o k c o m s s c m e n t o r s o f f i c i a l P a g e | 23

FOR MORE UPDATES AND MORE MATERIAL DO LIKE OUR FACEBOOK PAGE httpwwwfacebookcomsscmentorsofficial

Correct Answer Antonie Philips van

Leeuwenhoek

Q95 For the aquatic organisms the

source of food is

जिीयजीवाणकािाघसरह८तहहॴ 20-Jan-2017

Options

1) Phytoplankton

फायटह८पिहॳकटन

2) Sea Weed

समदरीिहॴवाि

3)Aqua plankton

एकवापिहॳकटन

4) Zooplankton

जपिहॳकटन

Correct Answer Phytoplankton

Q96 Haemoglobin has the highest

affinity with which of the following

हीमह८गिह८बबनकीननननमसहॳककसकहॳ सािउततमसमानताहहॴ

20-Jan-2017

Options

1)SO2

2)CO2

3)CO

4)NO2

Correct Answer CO

It has a greater affinity for hemoglobin

than oxygen does It displaces oxygen

and quickly binds so very little oxygen

is transported through the body cells

Q97 Who developed the theory of

Evolution

उदपवकासकालसदातककसनहॳपवकलसतककया

20-Jan-2017

Options

1) Charles Darwin

चामसयिापवयन

2) Isaac Newton

आयजहॳकनयटन

3) Pranav Mistry

परणवलमसतरी 4) Galileo Galilei

गहॳलिलियह८गहॳिीिी Correct Answer Charles Darwin

Q98 The primary function of RNA is

RNA कापरािलमककाययहह८ताहहॴ 20-Jan-2017

Options

1) Photosynthesis

परकािसशिहॳषण

2) Protein Synthesis

परह८टीनसशिहॳषण

3) Replication

परनतकनतबनाना 4) Translation

अनवादकरना Correct Answer Protein Synthesis

There are two main functions of RNA

It assists DNA by serving as a messenger

to relay the proper genetic information

to countless numbers of ribosomes in

your body The other main function of

RNA is to select the correct amino acid

needed by each ribosome to build new

proteins for your body

Q99 ______is the movement of

molecules across a cell membrane from

a region of their lower concentration to

a region of their higher concertration

उचचसादरताकहॳ कषहॳतरसहॳउसकीकमसादरतावािहॳकषहॳतरकीतरफएककह८लिकाखझमिीकहॳ माधयमसहॳहह८नहॳवािाअणओकहॳ सचिनकह८ mdash- कहतहॳहहॴ Options

1) Diffusion

पवसरण

2) Osmosis

ऑसमह८लसस

F A C E B O O K

P A G E h t t p w w w f a c e b o o k c o m s s c m e n t o r s o f f i c i a l P a g e | 24

FOR MORE UPDATES AND MORE MATERIAL DO LIKE OUR FACEBOOK PAGE httpwwwfacebookcomsscmentorsofficial

3) Active Transport

सकियआवागमन

4) Passive Transport

नननषियआवागमन

Correct Answer Active Transport

Q100 Study of classification of

organisms is known as 20-Jan-2017

जीवाणओकहॳ वगीकरणकहॳ अधययनकह८ mdash-

कहाजाताहहॴ Options

1) Serpentology

सपरहॳटह८िह८जी 2) Virology

वायरह८िह८जी 3) Taxonomy

टहॴकसोनह८मी 4) Physiology

कफनज़यह८िह८जी Correct Answer Taxonomy

Q101 Photosynthesis takes place inside

plant cells in

परकािसशिहॳषणवनसपनतकह८लिकामनसति mdash

mdashmdash महह८ताहहॴ 20-Jan-2017

Options

1) Ribosomes

राइबह८सह८नस

2) Chloroplasts

किह८रह८पिासट

3) Nucleus

नयकलियम

4) Mitochondria

माईटह८कोडडरया Correct Answer Chloroplasts

Q102 ______ is the cell organelle in

which the biochemical processes of

respiration and energy production

occur

mdashmdash- वहकह८लिकाअगहहॴ नजसमहॳशवसनऔरउजायउतपादनकहॳ जहॴसीजहॴवरासायननकपरकियायहह८तीहहॴ 20-Jan-2017

Options

1) Mitochondria

माइटह८कोडडरया 2) Chloroplast

किह८रह८पिासट

3) Ribosomes

राइबह८सह८नस

4) Nucleus

नयकिीयस

Correct Answer Mitochondria

Q103 Which non-flowering spore

bearing plants have roots

ककसफिनिगनहॳवािहॳऔरबीजाणधारकपह९धह८कीजड़हॳहह८तीहहॴ 21-Jan-2017

Options

1) Mosses

मह८सहॳस

2) Angiosperms

एननजयह८सपनसय 3) Ferns

फनसय 4) Gymnosperms

नजननह८सपनसय Correct Answer ferns

Q104 Which of the following is an

excretory organ of cockroach

नननननलिखितमसहॳकह९नसानतिच हॳकाउतसजयनअगहहॴ

21-Jan-2017

Options

F A C E B O O K

P A G E h t t p w w w f a c e b o o k c o m s s c m e n t o r s o f f i c i a l P a g e | 25

FOR MORE UPDATES AND MORE MATERIAL DO LIKE OUR FACEBOOK PAGE httpwwwfacebookcomsscmentorsofficial

1) Malphigian Tubules

मनमफनजयनटयबमस

2) Nephridia

नहॳकफरडिया 3) Coxal Gland

कह८कसिगरचिया 4) Green Gland

गरीनगरचिया Correct Answer Malphigian Tubules

Q105 Evaporation of water takes place

in which part of plants

पानीकहॳ वाषपीकरणकीकियापह९धोकहॳ ककसभागसहॳहह८तीहहॴ 21-Jan-2017

Options

1) Stem

तना 2) Stomata

सटह८मटा 3) Branch

िािाए

4) Fruit

फि

Correct Answer Stomata

Evaporation accounts for the movement

of water to the air from sources such as

the soil canopy interception and

waterbodies Transpiration accounts for

the movement of water within a plant

and the subsequent loss of water as

vapour through stomata in its leaves

Q106 A is the fleshy spore-bearing

fruiting body of a fungus

mdashmdashndashकवककामासि

बीजाणधारणकरनहॳवािाफिनहॳवािाअगहहॴ 21-

Jan-2017

Options

1) aloe vera

एिह८वहॳरा 2) Coral

मगा 3) Cactus

कहॴ कटस

4) Mushroom

ककरमतता Correct Answer mushroom

Q107 Which of the following is a fungal

disease

नननननलिखितमसहॳकह९नसाफफदसहॳहह८नहॳवािाएकरह८ग हहॴ

21-Jan-2017

Options

1) Dermatitis

तवचािह८ध

2) Cholera

हहॴजा 3) Jaundice

पीलिया 4) Indigofera

इननिगह८फहॳ रा Correct Answer Dermatitis

Dermatitis also known as eczema is a

group of diseases that results in

inflammation of the skin These diseases

are characterized by itchiness red skin

and a rash In cases of short duration

there may be small blisters while in

long-term cases the skin may become

thickened

Q108 In which form is glucose stored in

our body

हमारहॳिरीरमगिकह८जकासचयककस पमककयाजाताहहॴ

21-Jan-2017

Options

1) Insulin

F A C E B O O K

P A G E h t t p w w w f a c e b o o k c o m s s c m e n t o r s o f f i c i a l P a g e | 26

FOR MORE UPDATES AND MORE MATERIAL DO LIKE OUR FACEBOOK PAGE httpwwwfacebookcomsscmentorsofficial

इसलिन

2) Glucose

गिकह८ज

3) Glycogen

गिायकह८जहॳन

4) Fat

वसा Correct Answer Glycogen

Excess glucose is stored in the liver as

the large compound called glycogen

Glycogen is a polysaccharide of glucose

but its structure allows it to pack

compactly so more of it can be stored in

cells for later use

Q109 Where do plants synthesize

protein from

पह९धहॳपरह८टीनसशिहॳषणकहासहॳकरतहॳहहॴ

Options

1) Fatty Acids

वसाऐलसि

2) Sugar

िकर

3) Amino Acids

एलमनह८ऐलसि

4) Starch

सटाचय Correct Answer Amino Acids

Q110 Which part of the brain is

responsible for triggering actions like

thinking intelligence memory and

ability to learn

मनसतषककाकह९नसाटहससासह८चनहॳ बनधदमानी याददाशतऔरसीिनहॳकीकषमताजहॴसीकियाओकह८परहॳररतकरताहहॴ 21-Jan-2017

Options

1) Diencephalon

िायएनसहॳफहॳ िह८न

2) Hypothalamus

हयपह८िहॳिहॳमस

3) Cerebrum

सहॳरहॳिम

4) Control

कटरह८ि

Correct Answer Cerebrum

Q111 Which of the following is also

known as the Biochemical Laboratory

of the Human Body

नननननलिखितमसहॳककसहॳमानविरीरकीजहॴवरसायनपरयह८गिािाभीकहाजाताहहॴ 21-Jan-2017

Options

1) Small Intestine

छह८टीआत

2)Brain

मनसतषक

3) Pancreas

अगनयािय

4) Liver

नजगर

Correct Answer Liver

The liver makes bile that will help

emulsify and digest the fats we eat

The liver takes toxic substances and

convert them using enzymes the liver

cells makes into a non toxic form so the

body can dispose of them

The liver also converts fats protein and

carbohydrates into glucose which is the

energy source for our cells to use

The liver takes amino acids and makes

proteins by combining them

Q112 The yellow colour of human urine

is due to

मानवमतरकापीिारग mdashndash कीवजहसहॳहह८ताहहॴ 22-

Jan-2017

Options

1) Bile Salts

F A C E B O O K

P A G E h t t p w w w f a c e b o o k c o m s s c m e n t o r s o f f i c i a l P a g e | 27

FOR MORE UPDATES AND MORE MATERIAL DO LIKE OUR FACEBOOK PAGE httpwwwfacebookcomsscmentorsofficial

पपततनमक

2) Cholesterol

कह८िहॳसटरह८ि

3) Lymph

लिनफ

4) Urochrome

यरह८िह८म

Correct Answer Urochrome

Urobilin or urochrome is the chemical

primarily responsible for the yellow

color of urine

Q113 The wilting of plants takes place

due to

पह९धह८कालिचििहह८नाकी mdashmdash- कीवजहसहॳहह८ताहहॴ 22-Jan-2017

Options

1)Photosynthesis

परकािसशिहॳषण

2) Transpiration

वाषपह८तसजयन

3) Absorption

अविह८षण

4) Respiration

शरवसन

Correct Answer Transpiration

Wilting is the loss of rigidity of non-

woody parts of plants This occurs when

the turgor pressure in non-lignified

plant cells falls towards zero as a result

of diminished water in the cells

Q114 Bovidae Ovis is the scientific name of

बह८पविीओपवस mdashndash कावहॴजञाननकनामहहॴ 22-Jan-2017

Options

1) Goat

बकरी 2) Cow

गाय

3) Buffalo

भहॳस

4) Sheep

भहॳड़

Correct Answer Sheep

Q115 Plants get their energy to produce

food from which of the following

पह८धहॳभह८जनकाननमायणकरनहॳकहॳ लिएनननननलिखितमसहॳककससहॳउजायपरापतकरतहॳहहॴ

22-Jan-2017

Options

1) Photosynthesis

परकािसशिहॳषण

2)Bacteria

बहॴकटीररया 3)Fungi

कवक

4)Sun

सयय Correct Answer Sun

Q116 Which of the following is secreted

by the liver

नननननलिखितमसहॳककसकासरावनजगरसहॳहह८ताहहॴ

22-Jan-2017

Options

1) Glucose

गिकह८ज

2) Iodine

आयह८िीन

3) Cortisol

काटटरयसह८ि

4) Bile

पपतत

Correct Answer Bile

The liver makes bile that will help

emulsify and

digest the fats we eat

F A C E B O O K

P A G E h t t p w w w f a c e b o o k c o m s s c m e n t o r s o f f i c i a l P a g e | 28

FOR MORE UPDATES AND MORE MATERIAL DO LIKE OUR FACEBOOK PAGE httpwwwfacebookcomsscmentorsofficial

Q117 Ferns belong to which division of

plants

फनसयपह९धह८कहॳ ककसभागमआतहॳहहॴ

22-Jan-2017

Options

1) Gymnosperms

नजननह८सपनसय 2) Angiosperms

एनजयह८सपनसय 3) Thallophyta

िहॴिह८फाईटा 4)Pteridophyta

टहॳररिह८फाईटा Correct Answer Pteridophyta

Q118 Who invented Antibiotics

एटीबायह८टटककाअपवषकारककसनहॳककयािा

22-Jan-2017

Options

1) Joseph Lister

जह८सहॳफलिसटर

2) William Harvey

पवलियमहाव

3) Robert Knock

रॉबटयनॉक

4)Alexander Fleming

अिहॳकज़िरफिहॳलमग

Correct Answer Alexander Fleming

Q119 Milbecycin is used in the

eradication of

लममबहॳसायलसनका mdashndash

मउनमिनमपरयह८गककयाजाताहहॴ 22-Jan-2017

Options

1) Agricultural Fungus

कपषकवक

2) Agricultural Pests

कपषकीटक

3) Agricultural Herbs

कपषिाक

4)Agricultural Weeds

कपषननराना Correct Answer Agricultural Pests

Milbemycin oxime is a veterinary drug

from the group of milbemycins used as

a broad spectrum antiparasitic It is

active against worms and mites(insects

Q120 Intestinal bacteria synthesizes

which of the following in the human

body

मानविरीरमआतोकहॳ बहॴकटीररयानननननलिखितमसहॳककसकासशिहॳषणकरतहॳहहॴ 22-Jan-2017

Options

1) Vitamin K

पवटालमन K

2) Proteins

परह८टीन

3) Fats

वसा 4) Vitamin D

पवटालमन D

Correct Answer Vitamin K

Q121 is the study of the physical form

and external structure of plants

mdashmdash-

मपह९धह८काभहॴनतक पऔरबाहरीसरचनाकाआदयाककयाजाताहहॴ 22-Jan-2017

Options

1) Physiology

कफनजयह८िह८जी 2) Anatomy

िरीररचनापवजञान

3) Phytomorphology

फाईटह८मह८फह८िह८जी 4)Cytology

कह८लिकापवजञान

Correct Answer Phytomorphology

F A C E B O O K

P A G E h t t p w w w f a c e b o o k c o m s s c m e n t o r s o f f i c i a l P a g e | 29

FOR MORE UPDATES AND MORE MATERIAL DO LIKE OUR FACEBOOK PAGE httpwwwfacebookcomsscmentorsofficial

Q122 Which of the following is a

structural and functional unit of

kidneys

नननननलिखितमसहॳकह९नसीगदोकीसरचनातमकऔरकाययकरीईकाईहहॴ

22-Jan-2017

Options

1) Renette Cells

रहॳनहॳटकह८लिकाए

2) Flame Cells

फिहॳमकह८लिकाए

3) Nephrites

नहॳफ़राइटस

4)Nephrons

नहॳफरोस

Correct Answer Nephrons

Nephron functional unit of the kidney

the structure that actually produces

urine in the process of removing waste

and excess substances from the blood

There are about 1000000 nephrons in

each human kidney

Q123 Which of the following is the

largest part of the human brain

नननननलिखितमसहॳकह९नसामानवमनसतषककासबसहॳबड़ाटहससाहहॴ

23-Jan-2017

Options

1) Ribs

पसलियाा 2) Cerebrum

सहॳरहॳिम

3) Pons

पोस

4)Thalamus

िहॴिहॳमस

Correct Answer Cerebrum

The cerebrum is the largest part of the

human brain making up about two-

thirds of the brainrsquos mass It has two

hemispheres each of which has four

lobes frontal parietal temporal and

occipital

Q124 The auxiliary buds

सहायककालियाmdashndash 23-Jan-2017

Options

1) grow endogenously from the pericycle

पहॳरीसाईककिसहॳअनतजातयपवकलसतहह८ताहहॴ 2) arise endogenously from the main

growing point

मिवपदसहॳअनतजातयउठताहहॴ 3) is an embryonic shoot located in the

axil of a leaf

एकभरणिटहहॴजह८एकपततीकहॳ अकषपरनसतिहह८ताहहॴ 4)arise exogenously from the epidermis

एपपिलमयससहॳबटहजातयतरीकहॳ सहॳउठताहहॴ Correct Answer is an embryonic shoot

located in the axil of a leaf

Q125 Which of the following is a viral

disease

इनमहॳसहॳकह९सीएकवायरिबीमारीहहॴ

23-Jan-2017

Options

1) Polio

पह८लियह८ 2) Tetanus

धनसतनभ

3) Leprosy

कषठरह८ग

4) Plague

पिहॳग

Correct Answer Polio

A viral disease (or viral infection)

occurs when an organismrsquos body is

invaded by pathogenic viruses and

infectious virus particles (virions) attach

to and enter susceptible cells

F A C E B O O K

P A G E h t t p w w w f a c e b o o k c o m s s c m e n t o r s o f f i c i a l P a g e | 30

FOR MORE UPDATES AND MORE MATERIAL DO LIKE OUR FACEBOOK PAGE httpwwwfacebookcomsscmentorsofficial

Poliomyelitis often called polio or

infantile paralysis is an infectious

disease caused by the poliovirus

Tetanusmdash A serious bacterial infection

that causes painful muscle spasms and

can lead to death

Leprosy also known as Hansenrsquos

disease (HD) is a long-term infection by

the bacterium Mycobacterium leprae or

Mycobacterium lepromatosis

Plague is an infectious disease caused by

the bacterium Yersinia pestis

Symptoms include fever weakness and

headache

Q126 Which organisms can help to

carry out Vermicomposting

कह९नसाजीववमीकनपह८नसटगममददकरताहहॴ

23-Jan-2017

Options

1) Nitrifying Bacteria

नाईटरीफाईगबहॴकटीररया 2) Earthworms

कहॴ चऐ

3) Algae

िहॴवि

4) Fungus

कवक

Correct Answer Earthworms

Q127 Contraction of heart is also

known as

हदयकहॳ सकचनकह८ mdash- भीकहाजाताहहॴ 23-Jan-

2017

Options

1) Systole

लससटह८ि

2) Aristotle

अरसत

3) Diastole

िायसटह८ि

4) Lub

मयब

Correct Answer Systole

Diastole is the part of the cardiac cycle

when the heart refills with blood

following systole (contraction)

Ventricular diastole is the period during

which the ventricles are filling and

relaxing while atrial diastole is the

period during which the atria are

relaxing

Q128 Azadirachta indica is the

botanical name of which of the

following

अजाटदराचताइडिकानननननलिखितमसहॳककसकावानसपनतनामहहॴ

23-Jan-2017

Options

1) Rose plant

गिाबकापह९धा 2) Apple tree

सहॳबकापहॳड़

3) Neem

नीम

4)Mango

आम

Correct Answer Neem

Q129 Which of the following is the

main end product of carbohydrate

digestion

नननननलिखितमसहॳकह९नसाकाबोहाइडरहॳटकहॳ पाचनकापरमिअतउतपादकहह८ताहहॴ 23-Jan-2017

Options

1) Fats

वसा 2) Lipids

लिपपडस

3) Glucose

गिकह८ज

4) Cellulose

F A C E B O O K

P A G E h t t p w w w f a c e b o o k c o m s s c m e n t o r s o f f i c i a l P a g e | 31

FOR MORE UPDATES AND MORE MATERIAL DO LIKE OUR FACEBOOK PAGE httpwwwfacebookcomsscmentorsofficial

सहॳमयिह८ज

Correct Answer Glucose

Intestinal absorption of end products

from digestion of carbohydrates and

proteins in the pig hellip During absorption some sugars (fructose or

galactose) released from the

corresponding sucrose and lactose

respectively during digestion were

partly metabolized into glucose by the

enterocyte

Q130 Which of the following glands is a

source of the enzyme Ptyalin

नननननलिखितगरचियोमसहॳएजाइमटयालिनकासरह८तहहॴ 23-Jan-2017

Options

1) Pancreas

अगरािय

2) Thyroid Gland

िाइराइिगरिी 3) Pituitary Gland

पीयषगरिी 4) Salivary Glands

िारगरचियाा Correct Answer Salivary Glands

Q131 Which of the following is not true

about Pteridophyta

ननननमसहॳकह९नसीबातटहॳररिह८फाईटकहॳ बारहॳमसचनहीहहॴ 23-Jan-2017

Options

1) Dominant phase is saprophytes

परमिचरणसहॳपरह८फाईइटसहह८ताहहॴ 2) Main plant body is diploid

पह९दह८कामखयिरीरदपवगखणतहह८ताहहॴ 3) Seeds are present

बीजमह९जदहह८तहॳहहॴ 4)Flowers are absent

फिअनपनसतिहह८तहॳहहॴ

Correct Answer Seeds are present

Q132 The largest dolphin species is the

orca also called as

िॉिकफनकीसबसहॳबड़ीपरजानतकाकानामआकायहहॴनजसहॳ mdash- भीकहतहॳहहॴ 23-Jan-2017

Options

1) Bottle Nose

बाटिनह८ज

2) Baiji

बहॳजी 3) Killer whale

ककिरहहॳि

4)Tucuxi

टकवसी Correct Answer Killer whale

Q133 The fat digesting enzyme Lipase

is secreted by which of the following

वसाकापाचनकरनहॳवािाएजाइमिाइपहॳजनननननलिखितमसहॳककसकहॳ दवारासतरापवतहह८ताहहॴ

24-Jan-2017

Options

1) Kidneys

गद

2) Pancreas

अगनयािय

3) Large Intestine

बड़ीआत

4)Liver

नजगर

Correct Answer Pancreas

Lipase is an enzyme that splits fats so

the intestines can absorb them Lipase

hydrolyzes fats like triglycerides into

their component fatty acid and glycerol

molecules It is found in the blood

gastric juices pancreatic secretions

intestinal juices and adipose tissues

F A C E B O O K

P A G E h t t p w w w f a c e b o o k c o m s s c m e n t o r s o f f i c i a l P a g e | 32

FOR MORE UPDATES AND MORE MATERIAL DO LIKE OUR FACEBOOK PAGE httpwwwfacebookcomsscmentorsofficial

Q134 The arrangement of leaves on an

axis or stem is called

एकअकषयातनहॳपरपनततयोकीयवसिाकह८कयाकहाजाताहहॴ SSC CHSL Science (biology) 2016

Question Paper

24-Jan-2017

Options

1) Phyllotaxy

फाइिह८टहॴकसी 2) Vernation

वनिन

3) Venation

वहॳनहॳिन

4)Phytotaxy

फाइटह८टहॴकसी Correct Answer Phyllotaxy

In botany phyllotaxis or phyllotaxy is

the arrangement of leaves on a plant

stem (from Ancient Greek phyacutellon

ldquoleafrdquo and taacutexis ldquoarrangementrdquo)

Phyllotactic spirals form a distinctive

class of patterns in nature

Q135 The study of Cells is also known

as

कह८लिकाओकहॳ अधययनकह८ mdashmdashndash

भीकहाजाताहहॴ 24-Jan-2017

Options

1) Cytology

सायटह८िह८जी 2) Physiology

कफनजयह८िह८जी 3) Nucleology

नयककमयह८िह८जी 4)Cellology

सहॳिह८िह८जी Correct Answer Cytology

Q136 Which of the following scientists

is also known as the Father of Biology

नननननलिखितमसहॳककसवहॴजञाननककह८ ldquoजीवपवजञानकहॳ जनकrdquoकहॳ नामसहॳभीजानाजाताहहॴ 24-Jan-2017

Options

1) Herbert Spencer

हबयटयसपसर

2) Aristotle

अरसत 3) Lamarck

िहॳमाकय 4)Darwin

िापवयन

Correct Answer Aristotle

Q137 Which cells give rise to various

organs of the plant and keep the plant

growing

कह९नसीकह८लिकाएपह९धह८कहॳ लभननअगह८कह८जनमदहॳतीहहॴऔरपह९धह८कह८बढ़नहॳममददकरतीहहॴ

24-Jan-2017

Options

1) Permanent

सिायी 2) Dermal

तवचीय

3) Meristematic

मररसटहॳमटटक

4)Mature

परह८ढ़

Correct Answer Meristematic

A meristem is the tissue in most plants

containing undifferentiated cells

(meristematic cells) found in zones of

the plant where growth can take place

Q138 Rodentia Muridae is the scientific

name of

F A C E B O O K

P A G E h t t p w w w f a c e b o o k c o m s s c m e n t o r s o f f i c i a l P a g e | 33

FOR MORE UPDATES AND MORE MATERIAL DO LIKE OUR FACEBOOK PAGE httpwwwfacebookcomsscmentorsofficial

रह८िहॳलियानयररिी mdashmdash- कावहॴजञाननकनामहहॴ 24-

Jan-2017

Options

1) Mouse

चहा 2) Squirrel

चगिहरी 3) Monkey

बदर

4) Lizard

नछपकिी Correct Answer Mouse

Q139 Name the scientist who proposed

the cell theory

कह८लिकालसदातकापरसतावदहॳनहॳवािहॳवहॴजञाननककानामबताइए 24-Jan-2017

Options

1) Schleiden and Schwann

िीमिनऔरशरववान

2) Lamarck

िहॳमाकय 3) Treviranus

टरहॳवायरहॳनस

4)Whittaker and Stanley

हीटकरऔरसटहॳनिहॳ Correct Answer Schleiden and

Schwann

Q140 The flower with the worldrsquos

largest bloom is

दननयाकासबसहॳबड़ाफिखििनहॳवािा mdashmdashndash हहॴ 24-Jan-2017

Options

1) Pando

पािह८ 2) Posidonia

पह८सीिह८ननया 3) Rafflesia arnoldii

ररफिहॳलियाअनोमिी 4)Helianthus annuus

हहॳलिएनिसएनयअस

Correct Answer Rafflesia arnoldii

Rafflesia arnoldii is a species of

flowering plant in the parasitic genus

Rafflesia It is noted for producing the

largest individual flower on earth It has

a very strong and horrible odour of

decaying flesh earning it the nickname

ldquocorpse flower

Q141 Deficiency of which vitamin

causes night blindness

ककसपवटालमनकीकमीकहॳ कारणरतौधीहह८ताहहॴ 24-Jan-2017

Options

1) Vitamin K

पवटालमन K

2) Vitamin C

पवटालमन C

3) Vitamin B1

पवटालमन B1

4)Vitamin A

पवटालमन A

Correct Answer Vitamin A

Q142 Nongreen plants lack which of the

following

गहॴर-

हररतवनसपनतमनननननलिखितमसहॳककसकीकमीहह८तीहहॴ

24-Jan-2017

Options

1) Chlorophyll

किह८रह८कफि

2) Lycophyll

िायकह८कफि

3) Cyanophyll

F A C E B O O K

P A G E h t t p w w w f a c e b o o k c o m s s c m e n t o r s o f f i c i a l P a g e | 34

FOR MORE UPDATES AND MORE MATERIAL DO LIKE OUR FACEBOOK PAGE httpwwwfacebookcomsscmentorsofficial

सायनह८कफि

4)Phototropism

फह८टह८टरोपपजम

Correct Answer Chlorophyll

Q143 Organisms that use light to

prepare food are known as

जह८जीवपरकािकाउपयह८गकरभह८जनतहॴयारकरतहॳहहॴ उनह mdashmdash- कहॳ पमजानजाताहहॴ 24-Jan-2017

Options

1) Autotrophs

सवपह८षी 2) Heterotrophs

पवषमपह८षज

3) Omnivores

सवायहारी 4)Decomposers

पवघटनकरनहॳवािा Correct Answer Autotrophs

autotrophs often make their own food

by using sunlight carbon dioxide and

water to form sugars which they can use

for energy Some examples of

autotrophs include plants algae and

even some bacteria Autotrophs

(producer) are important because they

are a food source for heterotrophs

(consumers)

A heterotroph is an organism that

ingests or absorbs organic carbon

(rather than fix carbon from inorganic

sources such as carbon dioxide) in order

to be able to produce energy and

synthesize compounds to maintain its

life Ninety-five percent or more of all

types of living organisms are

heterotrophic including all animals and

fungi and some bacteria

Q144 Which of the following is a

primary function of haemoglobin

नननननलिखितमसहॳकह९नसाटहमह८गिह८बबनकाएकपरािलमककाययहहॴ

25-Jan-2017

Options

1) Utilization of energy

उजायकाउपयह८गकरना 2) Prevention of anaemia

रकतामपताहह८नहॳसहॳरह८कना 3) Destruction of bacteria

बहॴकटीररयाकापवनािकरना 4) To transport oxygen

ऑकसीजनकावहनकरना Correct Answer To transport oxygen

Q145 Vascular bundles are absent in

सवहनीबिि mdashmdash- मअनपनसतिरहतहॳहहॴ 25-Jan-2017

Options

1) Bryophyta

िायह८फाइटा 2) Pteridophyta

टहॳररिह८फाईटा 3) Gymnosperms

नजननह८सपमय 4) Angiosperms

एननजयह८सपहॳनसय Correct Answer Bryophyta

Q146 Sauria Lacertidae is the scientific

name of

सहॴररयािहॳसरटाईिी mdashmdashndash कावहॴजञाननकनामहहॴ 25-Jan-2017

Options

1) Crocodile

मगरमचछ

2) Hippopotamus

टहपपह८पह८टहॳमस

3) Lizard

नछपकिी 4) House fly

F A C E B O O K

P A G E h t t p w w w f a c e b o o k c o m s s c m e n t o r s o f f i c i a l P a g e | 35

FOR MORE UPDATES AND MORE MATERIAL DO LIKE OUR FACEBOOK PAGE httpwwwfacebookcomsscmentorsofficial

घरहॳिमकिी Correct Answer Lizard

Q147 Which type of pathogen causes

the water-borne disease SARS (Severe

Acute Respiratory Syndrome)

ककसपरकािकारह८गज़नकजिजननतबीमारीसासयकाकारणबनताहहॴ 25-Jan-2017

Options

1) Viral

वायरि

2) Parasitic

परजीवी 3) Protozoan

परह८टह८जअन

4) Bacterial

बहॴकटीररयि

Correct Answer Viral

Q148 Which of the following organs

produces the enzyme lipase

नननननलिखितमसहॳकह९नसाअगिायपहॳजएजाइमउतपननकरताहहॴ 25-Jan-2017

Options

1) Pancreas

अगनयािय

2) Large Intestine

बड़ीआत

3) Liver

नजगर

4) Small Intestine

छह८टीआत

Correct Answer Pancreas

Q149 A is a long internode forming the

basal part or the whole of a peduncle

एक mdashmdash- एकिबाइटरनह८िहहॴ जह८ननचिाटहससायासनपणयिठिबनताहहॴ 25-

Jan-2017

Options

1) Rhizome

परकद

2) Rachis

महॳ दि

3) floral axis

पषपअकष

4) Scape

भगदड़

Correct Answer scape

Q150 ndash Which of the following

organisms are considered to be both

Living and Non-living

नननननलिखितमसहॳकह९नसहॳजीवाणकह८जीपवतऔरअजीपवतमानाजाताहहॴ

25-Jan-2017

Options

1) Bacteria

बहॴकटीररया 2) Fungi

कवक

3) Algae

िहॴवाि

4)Virus

वायरस

Correct Answer Virus

They are considered to be living as they

possess a protein coat as a protective

covering DNA as the genetic material

etc

They are said to be non-living as they

can be crystallised and they survive for

billions of years They can tolerate high

temperatures freezing cold

temperatures ultra-violet radiations etc

Q151 Deficiency of fluorine causes

which of the following

फिह८ररनकीकमीकहॳ कारणनननननलिखितमसहॳकयाहह८ताहहॴ

F A C E B O O K

P A G E h t t p w w w f a c e b o o k c o m s s c m e n t o r s o f f i c i a l P a g e | 36

FOR MORE UPDATES AND MORE MATERIAL DO LIKE OUR FACEBOOK PAGE httpwwwfacebookcomsscmentorsofficial

27-Jan-2017

Options

1) Dental Caries

िटिकहॴ ररज

2) Scurvy

सकवरी 3) Anaemia

रकतामपता 4) Arthritis

गटठया Correct Answer Dental Caries

Q152 In a Punnett Square with the

cross AaBb x AaBb how many Aabb

genotypes would be created

पनहॳटसककायरमिह८स AaBb x AaBb कहॳ साि

ककतनहॳ Aabb जीनह८टाइपबनगहॳ 27-Jan-2017

Options

1) 1

2) 8

3) 2

4) 3

Correct Answer 2

Q153 Which of the following is the

Controlling Center of the Cell

नननननलिखित म सहॳ कह८लिकाका ननयतरण

क दर कह९न हहॴ

27-Jan-2017

Options

1) Nucleus

क दर

2) Plasma

पिाजमा 3) Lysosome

िायसह८सह८म

4) Chromosome

िह८मह८सह८म

Correct Answer Nucleus

The control centre of the cell is the

nucleus in eukaryotic cells The nucleus

contains genetic material in the form of

DNA

Q154 Myopia affects which of the

following organs

मायह८पपयानननननलिखितअगह८मसहॳककसहॳपरभापवतकरताहहॴ

25-Jan-2017

Options

1) Heart

हदय

2) Skin

तवचा 3) Eyes

आािहॳ 4)Mouth

मह

Correct Answer Eyes

Q155 Which of the following bears

flowers

नननननलिखितमसहॳकह९नफिधारणकरताहहॴ

25-Jan-2017

Options

1) Bryophyta

िायह८फाइटा 2) Pteridophyta

टहॳरीिह८फाईटा 3) Gymnosperms

नजननह८सपमय 4)Angiosperms

एननजयह८सपमय Correct Answer Angiosperms

Q156 Oxygenated blood flows out of the

heart through the

ऑकसीजनयकतरकत mdashmdashmdash

कहॳ माधयमसहॳहदयकहॳ बाहरबहताहहॴ 25-Jan-2017

F A C E B O O K

P A G E h t t p w w w f a c e b o o k c o m s s c m e n t o r s o f f i c i a l P a g e | 37

FOR MORE UPDATES AND MORE MATERIAL DO LIKE OUR FACEBOOK PAGE httpwwwfacebookcomsscmentorsofficial

Options

1) Aorta

महाधमनी 2) pulmonary artery

फहॳ फड़हॳकीधमनी 3) vena cava

वहॳनाकावा 4)Atrium

चह९क

Correct Answer aorta

Q157 Blood leaving the liver and

moving towards the

heart has a higher concentration of

नजगरसहॳननकिकरहदयकीतरफजानहॳवािहॳरकतम mdashmdashmdashmdash कीउचचसादरताहह८तीहहॴ 27-Jan-2017

Options

1) Lipids

लिपपडस

2) Urea

यररया 3) Bile Pigments

पपततकहॳ रगकरण

4) Carbon dioxide

काबयनिायऑकसाइि

Correct Answer Bile Pigments

Urea is nitrogen containing substance

which is produced in the liver in order

to deal with excess amino-acids in the

body As urea is produced it leaves the

liver in the blood stream and passes via

the circulatory system to all parts of the

body

Q158 Bulb is a modification of which

part of a plant

बमबएकपह९धहॳकहॳ ककसटहससहॳकाएक पातरणहह८ताहहॴ 27-Jan-2017

Options

1) The root

जड़

2) The stem

तना 3) The radicle

मिाकर

4)The fruit

फि

Correct Answer The stem

Q159 Which of the following carries

blood away from the heart to different

body parts

इनमहॳसहॳकह९नरकतकह८हदयसहॳिरीरकहॳ पवलभननअगह८तकिहॳजातीहहॴ

27-Jan-2017

Options

1) Arteries

धमननया 2) Nerves

तबतरहाए

3) Capillaries

कहॳ लिकाए

4)Veins

नसहॳ Correct Answer Arteries

Q160 The series of processes by which

nitrogen and its compounds are

interconverted in the environment and

in living organisms is called

27-Jan-2017

Options

1)Absorption of Nitrogen

2)Ammonification

3)Nitrogen Fixation

4)Nitrogen Cycle

Correct Answer Nitrogen Cycle

Ammonification or Mineralization is

performed by bacteria to convert

organic nitrogen to ammonia

F A C E B O O K

P A G E h t t p w w w f a c e b o o k c o m s s c m e n t o r s o f f i c i a l P a g e | 38

FOR MORE UPDATES AND MORE MATERIAL DO LIKE OUR FACEBOOK PAGE httpwwwfacebookcomsscmentorsofficial

Nitrification can then occur to convert

the ammonium to nitrite and nitrate

Nitrogen fixation is a process by which

nitrogen in the Earthrsquos atmosphere is

converted into ammonia (NH3) or other

molecules available to living organisms

Q161 BCG vaccine is given to protect

from which of the following

बीसीजीकाटटकानननननलिखितमसहॳककसकहॳ बचावकहॳ लिएटदयाजातहहॴ

27-Jan-2017

Options

1) Jaundice

पीलिया 2) Anaemia

रकतमपता 3) Tuberculosis

कषयरह८ग

4) Polio

पह८लियह८ Correct Answer Tuberculosis

Q162 Parallel venation is found in

समानतरवहॳनहॳिन mdashmdashmdash- मपायाजाताहहॴ 27-Jan-2017

Options

1) plants which are monocots

पह९धहॳजह८एकबीजपतरीहह८तहॳहहॴ 2) plants which have a dicot stem

वहॳपह९धहॳनजनकातनादपवदलियहह८ताहहॴ 3) plants with leaves similar to Tulsi

वहॳपह९धहॳनजनकीपनततयतिसीकीपनततयोकहॳ समानहह८तहॳहहॴ 4)plants with tap roots

टहॳप टवािहॳपह९धहॳ Correct Answer plants which are

monocots

Q163 The hardest part of the body is

िरीरकासबसहॳकठह८रभाग mdashndash हहॴ 27-Jan-2017

Options

1) Bones

हडडिय

2) Tooth Enamel

दातकहॳ इनहॳमि

3) Skull

िह८पड़ी 4) Spinal Cord

महॳ रजज

Correct Answer Tooth Enamel

Q164 Which type of pathogen causes

the waterborne disease E coli Infection

ककसपरकारकारह८गजननकजिजननतरह८गईकह८िाईसिमणकाकारणबनताहहॴ 27-Jan-2017

Options

1) Protozoan

परह८टह८जआ

2) Parasitic

परजीवी 3) Bacterial

बहॴकटीररयि

4)Viral

वायरि

Correct Answer Bacterial

Q165 The amount of blood filtered

together by both the kidneys in a 70 kg

adult male human in a minute is

70 की गरा वािहॳएकवयसकप षमएकलमनटमदह८नोगदकहॳदवाराएकसािचाबनीगयीरकतकीमातरहह८तीहहॴ 29-Jan-2017

Options

1) 1100 ml

1100 लमलि

2) 100 ml

F A C E B O O K

P A G E h t t p w w w f a c e b o o k c o m s s c m e n t o r s o f f i c i a l P a g e | 39

FOR MORE UPDATES AND MORE MATERIAL DO LIKE OUR FACEBOOK PAGE httpwwwfacebookcomsscmentorsofficial

100 लमलि

3) 1500 ml

1500 लमलि

4) 500 ml

500 लमलि

Correct Answer 1100 ml

Q166 Which feature of a plant helps to

distinguish a monocot from a dicot

पह९धहॳकीवहकह९नसीपविहॳषताहहॴजह८एकदपवदलियहॳऔरएकएकदिीयपह९धहॳसहॳभहॳदकरनहॳममददकरतीहहॴ 29-Jan-2017

Options

1) Pollination

परागम

2) Venation

वहॳनहॳिन

3) Vernation

वनिन

4) Aestivation

एसटीवहॳिहॳन

Correct Answer venation

Q167 The Mutation Theory was

proposed by

उतवररवतयनकालसदात mdashmdashndash

कहॳ दवरापरसतापवतककयाजाताहहॴ 29-Jan-2017

Options

1) Charles Lyell

चामसयलियहॳि

2) William Smith

पवलियमनसमि

3) Hugo De Vries

हयगह८िीराईस

4)Harrison Schmitt

हहॳरीसननसमट

Correct Answer Hugo De Vries

Q168 Which type of pathogen causes

the waterborne disease HepatitisA

ककसपरकारकहॳ रह८गजनकजिजननतरह८गहहॳपहॳटाइटटस-A काकारणबनताहहॴ

29-Jan-2017

Options

1) Parasitic

परजीवी 2) Viral

वायरि

3) Protozoan

परह८टह८जआ

4) Bacterial

बहॴकटीररयि

Correct Answer Viral

Q169 In a Punnett Square with the

cross AaBb x Aabb how many AaBb

genotypes would be created

पनहॳटसकवायरमिह८स AaBb x Aabb

कहॳ सािककतनहॳ AaBb जीनह८टाइपबनगहॳ 29-Jan-

2017

Options

1) 4

2) 1

3) 7

4) 6

Correct Answer 4

Q170 Arboreal Ateles is the scientific

name of

अिह८ररयिएटटलिस mdashmdashmdash कावहॴजञाननकनामहहॴ 29-Jan-2017

Options

1) Squirrel

चगिहरी 2) Sparrow

गह८रहॴया 3) Lizard

नछपकिी 4) Spider monkey

F A C E B O O K

P A G E h t t p w w w f a c e b o o k c o m s s c m e n t o r s o f f i c i a l P a g e | 40

FOR MORE UPDATES AND MORE MATERIAL DO LIKE OUR FACEBOOK PAGE httpwwwfacebookcomsscmentorsofficial

मकड़ीबदर

Correct Answer Spider monkey

Q171 Which type of pathogen causes

the waterborne disease Salmonellosis

ककसपरकारकारह८गाणजिजननतबीमारीसािमह८नहॳिह८लसज़काकारकहहॴ

29-Jan-2017

Options

1) Algal

िहॳवालियहॳ 2) Parasitic

परजीवी 3) Bacterial

बहॴकटीररयि

4)Viral

वायरि

Correct Answer Bacterial

An infection with salmonella bacteria

commonly caused by contaminated food

or water

Symptoms include diarrhoea fever

chills and abdominal pain

Q172 is a condition in which there is a

deficiency of red cells or of haemoglobin

in the blood

mdashmdash-

एकनसिनतहहॴनजसमहॳरकतमिािकह८लिकाओकीयाहीमह८गिह८बबनकीकमीहह८तीहहॴ 29-Jan-2017

Options

1) Albinism

एनमबननजम

2) Propyria

परह८पीररया 3) Anaemia

एनीलमया 4)Keloid disorder

कहॳ िह८इिडिसओिर

Correct Answer Anaemia

Q173 Ananas comosus is the scientific

name of

Options

अनानासकह८मह८सस mdashmdashmdashndash

कावहॴजञाननकनामहहॴ 29-Jan-2017

1) Custard Apple

सीताफि

2) Pineapple

पाइनएपपि

3) Bamboo

बास

4)Pomegranate

अनार

Correct Answer Pineapple

Q174 Which organ produces insulin

कह९नसाअगइनसलिनपहॴदाकरताहहॴ 29-Jan-

2017

Options

1) Liver

यकत

2) Thyroid gland

िायराइिगरिी 3) Spleen

पिीहा 4)Pancreas

अगरयिय

Correct Answer Pancreas

Q175 Which of the following disease is

not caused by water pollution

नननननलिखितमसहॳकह९नसारह८गपानीकहॳ परदषणकहॳकारणनहीहह८ता

29-Jan-2017

Options

1) Cholera

हहॴजा 2) Typhoid

F A C E B O O K

P A G E h t t p w w w f a c e b o o k c o m s s c m e n t o r s o f f i c i a l P a g e | 41

FOR MORE UPDATES AND MORE MATERIAL DO LIKE OUR FACEBOOK PAGE httpwwwfacebookcomsscmentorsofficial

टाइफाइि

3) Asthma

दमा 4)Diarrhoea

दसत

Correct Answer Asthma

Q176 Ocimum tenuiflorum is the

scientific name of

ओलिलममटहॳयईफिह८रमइसकावहॴजञाननकनाम mdash

ndash हहॴ 30-Jan-2017

Options

1) Neem

नीम

2) Mango

आम

3) Babul

बबि

4)Tulsi

तिसी Correct Answer Tulsi

Q177 Which gland secretes bile a

digestive fluid

कह९नसीगरिीपपतत एकपाचनतरिपरदािय सरापवतकरतीहहॴ 30-Jan-2017

Options

1) Pancreas

अगनयािय

2) Liver

यकत

3) Thyroid

िायराइि

4) Testes

टहॳनसटस

Correct Answer liver

Q178 In which of the following the

dominant phase is Gametophyte

नननननलिखितमसहॳककसकहॳ परमिचरणयगमकह८दपवधद (Gametophyte)हहॴ 30-Jan-2017

Options

1) Bryophyta

िायह८फाइटा 2) Pteridophyta

टहॳररिह८फाइटा 3) Gymnosperms

नजननह८सपमय 4) Angiosperms

एननजयह८सपमय Correct Answer Bryophyta

Q179 Anaerobic respiration refers to

which of the following

नननननलिखितमसहॳककसहॳअवायवीयशवसनकहाजाताहहॴ

30-Jan-2017

Options

1) Respiration without Oxygen

ऑकसीजनकहॳ बबनाशवसन

2) Respiration with Oxygen

ऑकसीजनकहॳ सािशवसन

3) Respiration without CO2

काबयनिायऑकसाइिकहॳ बबनाशवसन

4) Respiration with CO2

काबयनिायऑकसाइिकहॳ सािशविन

Correct Answer Respiration without

Oxygen

Q180 Which type of pathogen causes

the waterborne disease Cholera

ककसपरकारकारह८गजनकजिजननतरह८गहहॴजाकाकारणबनताहहॴ

30-Jan-2017

Options

1) Algal

िहॴवालियहॳ

F A C E B O O K

P A G E h t t p w w w f a c e b o o k c o m s s c m e n t o r s o f f i c i a l P a g e | 42

FOR MORE UPDATES AND MORE MATERIAL DO LIKE OUR FACEBOOK PAGE httpwwwfacebookcomsscmentorsofficial

2) Bacterial

बहॴकटीररयि

3) Protozoan

परह८टह८जआ

4) Viral

वायरि

Correct Answer Bacterial

Q181 To which class does

Oxyreductases transferases hydrolases

belong

ओकसीररिकटहॳसटरासफरहॳजहॳस

हाइडरह८िहॳसहॳसककसवगयमआतहॳहहॴ 30-Jan-2017

Options

1) Hormones

हारमोस

2) Enzymes

एजाइनस

3) Proteins

परह८टीनस

4) Vitamins

पवटालमनस

Correct Answer Enzymes

Q182 Which of the following is not true

about Gymnosperms

ननननमसहॳकह९नसीबातअनावतबीजीकहॳ बारहॳमसचनहीहहॴ 30-Jan-2017

Options

1) Dominant phase is saprophytes

परमिचरणसहॳपरह८फाइटसहह८ताहहॴ 2) Vascular bundles are absent

सवहनीबििअनपनसितहह८ताहहॴ 3) spores are heterospores

बीजाणहहॳटहॳरह८सपह८रसहह८तहॳहहॴ 4) Flowers are absent

फिअनपनसितहह८तहॳहहॴ

Correct Answer Vascular bundles are

absent

Q183 The name of first mammal clone sheep is

भहॳड़कीपरिमसतनपायीपरनत प (किह८न)

कानामहहॴ 30-Jan-2017

Options

1) Noori

नरी 2) Dolly

िॉिी 3) Louise

िसी 4)Durga

दगाय Correct Answer Dolly

Q184 Which type of pathogen causes

the water-borne disease Typhoid fever

ककसपरकारकारह८गजनकजिजननतरह८गटाइफाइिबिारकाकारणबनताहहॴ 30-Jan-2017

Options

1) Algal

िहॴवािीय

2) Parasitic

परजीवी 3) Protozoan

परह८टह८जनअन

4)Bacterial

बहॴकटीररयि

Correct Answer Bacterial

Q185 In which part of the cell are

proteins made

कह८लिकाकहॳ ककसटहससहॳमपरह८टीनबनायाजाताहहॴ

31-Jan-2017

Options

1) Reticulum

रहॳटटकिम

F A C E B O O K

P A G E h t t p w w w f a c e b o o k c o m s s c m e n t o r s o f f i c i a l P a g e | 43

FOR MORE UPDATES AND MORE MATERIAL DO LIKE OUR FACEBOOK PAGE httpwwwfacebookcomsscmentorsofficial

2) Golgi apparatus

गह८मजीएपहॳरहॳटस

3) Ribosomes

ररबह८सह८नस

4) Lysosome

िायसह८सह८नस

Correct Answer ribosomes

Proteins are produced by stringing

amino acids together in the order

specified by messenger RNA strands

that were transcribed from DNA in the

cell nucleus The process of synthesizing

a protein is called translation and it

occurs on ribosomes in the cytoplasm of

a cell

Q186 Polio is a disease caused by which

of the following

नननननलिखितमसहॳपह८लियह८कीबबमारह८हह८नहॳकाकारणकयाहहॴ

31-Jan-2017

Options

1) Bacteria

बहॴकटीररयि

2) Mosquito

मचछर

3) Virus

वायरस

4) Cockroach

नतिच हॳ Correct Answer Virus

Polio or poliomyelitis is a crippling and

potentially deadly infectious disease It

is caused by the poliovirus

Q187 ndash Hay fever is a sign of which of

the following

हहॳकफवरनननननलिखितमसहॳककसकाएकसकहॳ तहहॴ

31-Jan-2017

Options

1) Old Age

वदावसिा 2) Malnutrition

कपह८सण

3) Allergy

एिनजय 4) Over Work

अतयचधककाययकरना Correct Answer Allergy

Q188 How many chromosomes does a

human cell contain

एकमानवकह८लिकामककतनहॳगणसतरहह८तहॳहहॴ

29-Jan-2017

Options

1) 6

2) 26

3) 46

4) 66

Correct Answer 46

In humans each cell normally contains

23 pairs of chromosomes for a total of

46 Twenty-two of these pairs called

autosomes look the same in both males

and females The 23rd pair the sex

chromosomes differ between males and

females

Q189 Which of the following is not true

about Bryophyta

ननननमसहॳकह९नसीबातिायह८फाइटकहॳ बारहॳमसचनहीहहॴ 31-Jan-2017

Options

1) Dominant phase is gametophytes

परमिचरणगहॳलमतह८फाइटसहह८ताहहॴ 2) Main plant body is haploid

पह९धहॳकामखयिरीरअगखणतहह८ताहहॴ 3) Spores are homospores

बीजाणहह८मह८सफह८रसहह८तहॳहहॴ 4) Flowers are present

फिमह८जदहह८तहॳहहॴ Correct Answer Flowers are present

F A C E B O O K

P A G E h t t p w w w f a c e b o o k c o m s s c m e n t o r s o f f i c i a l P a g e | 44

FOR MORE UPDATES AND MORE MATERIAL DO LIKE OUR FACEBOOK PAGE httpwwwfacebookcomsscmentorsofficial

Q190 Which aquatic animal has

trailing tentacles

ककसजिीयजानवरकहॳ पीछहॳचिनहॳवािहॳटहॳटकिसहह८तहॳहहॴ

31-Jan-2017

Options

1) Sea horse

समदरीघह८िा 2) Corals

मगा 3) Jelly fish

जहॳिीमछिी 4) Star fish

तारामछिी Correct Answer Jelly fish

Jellyfish with its umbrella-shaped bell

and trailing tentacles

Q191 Which type of pathogen causes

the water-borne disease Poliomyelitis

(Polio)

ककसपरकारकारह८गजनकजिजननतरह८गपह८लियह८मायहॳटटस (पह८लियह८) काकारणहहॴ 31-Jan-

2017

Options

1) Parasitic

परजीवी 2) Algal

िहॴवालिय

3) Viral

वायरि

4) Bacterial

बहॴकटीररयि

Correct Answer Viral

Q192 The outer white part of the eye

that protects the inner structures is

आािकाबाहरीसफहॳ दटहससाजह८आतररकसरचनाओकीरकषाकरताहहॴ वह mdashmdashmdash हहॴ 31-Jan-

2017

Options

1) Iris

आयररस

2) Sclera

सकिहॳरा 3) Retina

रहॳटटना 4) Cornea

कह८ननयया Correct Answer Sclera

Q193 Proteins are made up of

परह८टीनकाननमायण mdashndash सहॳहह८ताहहॴ 31-Jan-2017

Options

1) Amino acids

एलमनह८अनि

2) Fatty acids

वसायकतअनि

3) Glucose

गिकह८ज

4)Nucleotides

नयनकियह८टाईिस

Correct Answer Amino acids

Q194 Moringa Oleifera is the scientific

name of

मह८ररगओलिफहॳ रा mdashmdashndash कावहॴजञाननकनामहहॴ 31-Jan-2017

Options

1) Banyan

बरगद

2) Gulmohar

गिमह८हर

3) Amla

आमिा

F A C E B O O K

P A G E h t t p w w w f a c e b o o k c o m s s c m e n t o r s o f f i c i a l P a g e | 45

FOR MORE UPDATES AND MORE MATERIAL DO LIKE OUR FACEBOOK PAGE httpwwwfacebookcomsscmentorsofficial

4) Drumstick

डरमनसटक

Correct Answer Drumstick

Q195 Kidney stones are composed of

गदकीपिरी mdashndash सहॳबनीहह८तीहहॴ 1-Feb-2017

Options

1) Calcium Oxalate

कहॴ नमसयमओकजहॳिहॳट

2) Sodium Chloride

सह८डियमकिह८राइि

3) Magnesium Nitrate

महॳनगनलियमनाइतटरहॳट

4) Calcium Bicarbonate

कहॴ नमियमबायकबोनहॳट

Correct Answer Calcium Oxalate

Q196 ndash Which of the following is not

true about Angiosperms

ननननमसहॳकह९नसीबातआवतबीजीकहॳ बारहॳमसचनहीहहॴ 1-Feb-2017

Options

1) Dominant phase is gametophytes

परमिचरणगहॳलमतह८फाइटहह८ताहहॴ 2) Vascular bundles are present

सवहनीबििमह९जदहह८ताहहॴ 3) Spores are heterospores

बीजाणहहॳटहॳरह८सपह८रसहह८तहॳहहॴ 4) Seeds are covered

बीजढकहॳ हह८तहॳहहॴ Correct Answer Dominant phase is

gametophytes

Q197 All of the following are excretory

(waste) products of animals except

नननननलिखितमसहॳककसएककह८छह८ड़करअनयसभीपराखणयोदवाराउतसनजयतपदाियहहॴ 1-Feb-

2017

Options

1) Uric Acid

यररकएलसि

2) Ammonia

अमह८ननया 3) Carbohydrates

काबोहाइडरहॳट

4) Urea

यररया Correct Answer Carbohydrates

In animals the main excretory products

are carbon dioxide ammonia (in

ammoniotelics) urea (in ureotelics) uric

acid (in uricotelics) guanine (in

Arachnida) and creatine

Q198 RNA is a polymeric molecule

What does RNA stand for

आरएनइएएकबहिकआणहहॴ इसकाकापवय पकयाहहॴ 1-Feb-2017

Options

1) Rado Nuclear Acid

रािह८नयनकियरएलसि

2) Ribo Nucleic Acid

राइबह८नयनकिकएलसि

3) Rhino Nuclear Acid

हाइनह८नयनकियरएलसि

4) Resto Nucleus Acid

रहॳसटह८नयकिीयसएलसि

Correct Answer Ribo Nucleic Acid

Q199 Which organ does detoxification

and produces chemicals needed for

digestion

कह९नसाअगपवषहरणकरताहहॴऔरपाचनकहॳ लिएआवशयकरसायनोकह८पहॴदाकरताहहॴ 1-Feb-

2017

Options

1) Salivary glands

िारगरचिया 2) Pancreas

अगनयािय

F A C E B O O K

P A G E h t t p w w w f a c e b o o k c o m s s c m e n t o r s o f f i c i a l P a g e | 46

FOR MORE UPDATES AND MORE MATERIAL DO LIKE OUR FACEBOOK PAGE httpwwwfacebookcomsscmentorsofficial

3) Thyroid gland

िायराइिगरिी 4) Liver

यकत

Correct Answer Liver

Q200 Psidium guajava is the scientific

name of

लसडियमगआजावा mdashmdash कावहॴजञाननकनामहहॴ 1-

Feb-2017

Options

1) Guava

अम द

2) Mango

आम

3) Bamboo

बास

4) Jack fruit

कटहि

Correct Answer Guava

Q201 Which drug is used as a Blood

Thinner

चधरकह८पतिाकरनहॳकहॳ पमककसदवाकापरयह८गककयाजाताहहॴ

1-Feb-2017

Options

1) Warfarin

वाफर न

2) Tramadol

टरहॳमािह८ि

3) Azithromycin

एनजरह८मायलसन

4) Hydralazine

हाइडरह८िहॳनजन

Correct Answer Warfarin

Q202 Which of the following disease is

caused due to the deficiency of protein

परह८टीनकीकमीकहॳ कारणनननननलिखितमसहॳकह९नसारह८गहह८ताहहॴ 1-Feb-2017

Options

1) Arthritis

गटठया 2) Kwashiorkor

कािीओकय र

3) Goitre

गाइटर

4) Night Blindness

रतह९चध

Correct Answer Kwashiorkor

Q203 A is species of plant that has

adapted to survive in an environment

with little liquid water

mdashmdashndashपह९धहॳकीएकऐसहॳऐसहॳपरजानतहहॴ नजसनहॳकमपानीवािहॳवातावरणमजीपवतरहनहॳकहॳलिएअनकिनहहॴ 1-Feb-2017

Options

1) Xerophyte

म दपवद

2) Hydrophyte

जिीयपादप

3) Mesophyte

समह८दपवद

4) Thallophyte

िहॴिह८फाइटा Correct Answer xerophyte

xerophyte is a species of plant that has

adapted to survive in an environment

with little liquid water such as a desert

or an ice- or snow-covered region in the

Alps or the Arctic

Mesophytes are terrestrial plants which

are adapted to neither a particularly

dry nor particularly wet environment

An example of a mesophytic habitat

would be a rural temperate meadow

F A C E B O O K

P A G E h t t p w w w f a c e b o o k c o m s s c m e n t o r s o f f i c i a l P a g e | 47

FOR MORE UPDATES AND MORE MATERIAL DO LIKE OUR FACEBOOK PAGE httpwwwfacebookcomsscmentorsofficial

which might contain goldenrod clover

oxeye daisy and Rosa multiflora

thallophyte any of a group of plants or

plantlike organisms (such as algae and

fungi) that lack differentiated stems

leaves and roots and that were formerly

classified as a primary division

(Thallophyta) of the plant kingdom

Q204 How many types of teeth are

there in humans

मनषयोमककतनहॳपरकारकहॳ दातहह८तहॳहहॴ

1-Feb-2017

Options

1) 4

2) 5

3) 2

4) 3

Correct Answer 4

teeth -Humans have four types of

teethincisors canines premolars and

molars each with a specific function

The incisors cut the food the canines

tear the food and the molars and

premolars crush the food

Q205 Carica papaya is the scientific name of

कहॴ ररकापपाया mdashmdashndash कावहॴजञाननकनामहहॴ 2-

Feb-2017

Options

1) Peepal

पीपि

2) Papaya

पपीता 3) Tamarind

इमिी 4) Drumstick

ढह८िकाछड़ी Correct Answer Papaya

Q206 Muscles get tired when there is

shortfall of

जब mdashndash कीकमीहह८तीहहॴतबपहॳिीयिकजातीहहॴ 2-Feb-2017

Options

1) Lactic acid

िहॴनकटकएलसि

2) Na+ ions

Na+ आयन

3) ATP

एटीपी 4) Sulphates

समफहॳ टस

Correct Answer ATP

ATP is the energy source muscle fibers

use to make muscles contract

muscle tissuersquos main source of energy

called adenosine triphosphate or ATP

As your muscles use up this energy

source they become tired and fatigued

Oxygen is the key ingredient that helps

create new ATP to replenish the burned

up ATP in your muscles

Q207 Artocarpus integra is the

scientific name of आटह८कापयसइटीगरा mdashmdashmdash कावहॴजञाननकनामहहॴ 2-Feb-2017

Options

1) Guava

अम द

2) Pineapple

अनानास

3) Silver Oak

लसमवरओक

4) Jack fruit

कटहि

Correct Answer Jack fruit

Q208 Which organ stores fat soluble

vitamins

कह९नसाअगवसामघिनिीिपवटालमनह८काभिाराकरताहहॴ

2-Feb-2017

F A C E B O O K

P A G E h t t p w w w f a c e b o o k c o m s s c m e n t o r s o f f i c i a l P a g e | 48

FOR MORE UPDATES AND MORE MATERIAL DO LIKE OUR FACEBOOK PAGE httpwwwfacebookcomsscmentorsofficial

Options

1) Blood

रकत

2) Skin

तवचा 3) Liver

यकत

4) Pancreas

अगनयािय

Correct Answer Liver

Q209 Which disease is caused due to

deficiency of Iodine

आयह८िीनकहॳ कारणकह९नसारह८गहह८ताहहॴ 2-Feb-2017

Options

1) Rickets

ररकहॳ टस

2) Scurvy

सकवी 3) Goitre

गणमािा 4) Growth retardation

पवकासका कना Correct Answer Goitre

rickets A softening and weakening of

bones in children usually due to

inadequate vitamin D

Q210 Grevillea Robusta is the scientific name of

गरहॳपवलियारह८बसटा mdashmdashmdash- कापवजञाननकनामहहॴ 2-Feb-2017

Options

1) Peepal

पीपि

2) Teak

सागह९न

3) Silver Oak

लसमवरओक

4) Jack fruit

कटहि

Correct Answer Silver Oak

Q211 When a Cuttlefish is described as a Molluscs it is at which level of

classification

जबएककटिकफिकह८एकमह८िसकाकहॳ पमवखणयतककयाजाताहहॴतबयहॳवगीकरणकहॳ ककससतरपहॳनसितहहॴ 2-Feb-2017

Options

1) Class

वगय 2) Order

िम

3) Family

पररवार

4) Phylum

सघ

Correct Answer Phylum

Q212 Bambusa dendrocalmus is the

scientific name of बानबसािहॳडराकामस mdashmdashmdash कावहॴजञाननकनामहहॴ 3-Feb-2017

Options

1) Banyan

बरगद

2) Papaya

पपीता 3) Bamboo

बास

4) Pomegranate

अनार

Correct Answer Bamboo

Q213 Acinonyx Jubatus is the scientific name of

एलसनह८ननकसजयबहॳटस mdashmdashmdash

कावहॴजञाननकनामहहॴ 3-Feb-2017

F A C E B O O K

P A G E h t t p w w w f a c e b o o k c o m s s c m e n t o r s o f f i c i a l P a g e | 49

FOR MORE UPDATES AND MORE MATERIAL DO LIKE OUR FACEBOOK PAGE httpwwwfacebookcomsscmentorsofficial

Options

1) Bear

भाि 2) Horse

घह८िा 3) Cheetah

चीता 4) Zebra

जहॳिा Correct Answer Cheetah

Q214 The pale yellow colour of urine is

due to the presence of which pigment

मतरकाफीकापीिारगरगदरयकहॳ उपनसिनतकहॳ कारणहह८ताहहॴ

3-Feb-2017

Options

1) Urochrome

यरह८िह८म

2) Urophyll

यरह८कफि

3) Chlorophyll

किह८रह८कफि

4) Chloroplast

किह८रह८पिासट

Correct Answer Urochrome

Q215 Which of the following constitute

to form a gene

नननननलिखितमसहॳकह९नसीचीज़एकजीनकागठनकरतीहहॴ

3-Feb-2017

Options

1) Polynucleotides

पह८िीनयनकियह८टाईडस

2) Hydrocarbons

हाइडरह८काबोस

3) Lipoproteins

िाईपह८परह८टीनस

4) Lipids

लिपपडस

Correct Answer Polynucleotides

Polynucleotide molecule is a biopolymer

composed of 13 or more nucleotide

monomers covalently bonded in a chain

DNA (deoxyribonucleic acid) and RNA

(ribonucleic acid) are examples of

polynucleotides with distinct biological

function

Q216 Vertebrates belongs to the

phylum

रीढ़कीहडिीवािहॳपराणी mdashmdashmdash

परजानतकहॳ अतगायतआतहॳहहॴ 3-Feb-2017

Options

1) Arthropoda

आरह८पह८ड़ा 2) Annelida

एननलििा 3) Cnidaria

ननिहॳररया 4) Chordata

कह८िटा Correct Answer Chordata

Q217 Punica granatum is the scientific name of

पननकगरहॳनहॳटस mdashmdashmdash कावहॴजञाननकनामहहॴ 3-Feb-2017

Options

1) Custard Apple

सीताफि

2) Gulmohar

गिमह८हर

3) Silver Oak

लसमवरओक

4) Pomegranate

अनार

Correct Answer Pomegranate

F A C E B O O K

P A G E h t t p w w w f a c e b o o k c o m s s c m e n t o r s o f f i c i a l P a g e | 50

FOR MORE UPDATES AND MORE MATERIAL DO LIKE OUR FACEBOOK PAGE httpwwwfacebookcomsscmentorsofficial

Q218 Between a tiger and an monkey

which of the following is different

एकबाघऔरबदरकहॳ बीचनननननलिखितमसहॳकह९नसीबातअिगहहॴ 3-Feb-2017

Options

1) Kingdom

राजय

2) Phylum

जानत

3) Order

िम

4) Class

वगय Correct Answer order

Q219 The artificial heart was invented by

कबतरमहदयका mdashmdashmdash

दवाराअपवषकारककयागयािा 3-Feb-2017

Options

1) Muhammad Yunus

महनमदयनस

2) Linus Yale Jr

िाइनसयहॳिजय

3) Gazi Yasargil

गाजीयासचगयि

4) Paul Winchell

पह९िपवमकि Correct Answer Paul Winchell

Q220 Tamarindus indica is the

scientific name of

टहॳमररनडसइडिका mdashmdash कावहॴजञाननकनामहहॴ 7-

Feb-2017

Options

1) Neem

नीम

2) Pineapple

अनानास

3) Tamarind

इमिी 4)Chiku

चीक

Correct Answer Tamarind

Q221 In eukaryotic cells synthesis of

RNA takes place in the

यकहॳ योटटककह८लिकाओमआरएनएकासशिहॳषण

mdashndash महह८ताहहॴ 7-Feb-2017

Options

1) Mitochondria

माईटह८कोडडरया 2) Centrioles

सटरीयह८मस

3) Ribosomes

ररबह८सह८नस

4) Nucleus

नयनकियस

Correct Answer nucleus

eukaryotic cell -Transcription is the

process of synthesizing ribonucleic acid

(RNA)Synthesis takes place within the

nucleus of eukaryotic cells or in the

cytoplasm of prokaryotes and converts

the genetic code from a gene in

deoxyribonucleic acid ( DNA ) to a

strand of RNA that then directs

proteinsynthesis

Q222 _________is caused by parasites

of the Plasmodium genus

पिाजमह८डियमजातीकहॳ परजीवी mdash- कहॳ कारणहहॴ 7-Feb-2017

Options

1) Dysentery

पहॳचचि

2) Malaria

मिहॳररया 3) Chickenpox

F A C E B O O K

P A G E h t t p w w w f a c e b o o k c o m s s c m e n t o r s o f f i c i a l P a g e | 51

FOR MORE UPDATES AND MORE MATERIAL DO LIKE OUR FACEBOOK PAGE httpwwwfacebookcomsscmentorsofficial

चहॳचक

4) Herpes

हहॳपपयस

Correct Answer Malaria

Q223 Carotene in fruits and vegetables

gives it which color

फिह८औरसनलजयोमनसितकहॳ रह८टीनउनहकह९नसारगपरदानकरताहहॴ 7-Feb-2017

Options

1) Green

हरा 2) Pink

गिाबी 3) Orange

नारगी 4) Blue

नीिा Correct Answer Orange

Q224 Equus Caballus is the scientific

name of

एकवसकहॴ बहॳिस mdashmdashndash कापवजञाननकनामहहॴ 7-Feb-2017

Options

1) Horse

घह८िा 2) Zebra

ज़हॳिा 3) Donkey

गधा 4) Buffalo

भस

Correct Answer Horse

Q225 Elapidae Naja is the scientific name of

एिीपीिीनाजा mdashmdash- कावहॴजञाननकनामहहॴ 8-Feb-2017

Options

1) Cobra

कह८बरा 2) Elephant

हािी 3) Eagle

ग ि

4) Owl

उमि Correct Answer Cobra

Q226 Which disease is caused due to

deficiency of Iron

िह८हकीकमीकहॳ कारणकह९नसारह८गहह८ताहहॴ 8-Feb-

2017

Options

1) Beriberi

बहॳरीबहॳरी 2) Tetany

टहॳटनी 3) Kwashiorkor

कवािीऔरकर

4) Anaemia

रकतामपता Correct Answer Anaemia

Beriberi is a disease caused by a vitamin

B-1 deficiency also known as thiamine

deficiency

Tetany can be the result of an

electrolyte imbalance Most often itrsquos a

dramatically low calcium level also

known as hypocalcemia Tetany can also

be caused by magnesium deficiency or

too little potassium Having too much

acid (acidosis) or too much alkali

(alkalosis) in the body can also result in

tetany

Kwashiorkor also known as

ldquoedematous malnutrition It is a form of

malnutrition caused by a lack of protein

in the diet

Anaemia means that you have fewer red

blood cells than normal or you have less

F A C E B O O K

P A G E h t t p w w w f a c e b o o k c o m s s c m e n t o r s o f f i c i a l P a g e | 52

FOR MORE UPDATES AND MORE MATERIAL DO LIKE OUR FACEBOOK PAGE httpwwwfacebookcomsscmentorsofficial

haemoglobin than normal in each red

blood cell

Q227 is a leaf where the leaflets are

arranged along the middle vein

mdashndashएकपततीहहॴजहापतरकह८कीरचनाक ररयालिराकहॳ आसपासहह८तीहहॴ 8-Feb-2017

Options

1) Pinnately compound leaf

पपनहॳटिीसयकतपतती 2) Palmately compound leaf

पामहॳटिीसयकतपतती 3) Compound leaf

सयकतपतती 4) Simple leaf

साधारणपतती Correct Answer Pinnately compound

leaf

Q228 Haustoria or sucking roots are

found in which of the following

हह८सटह८ररयायाचसनहॳवािीजड़हॳनननननलिखितमसहॳककसमपाईजातीहहॴ 8-Feb-2017

Options

1) Wheat

गहॳह

2) Mango

आम

3) Chestnut

चहॳसटनट

4) Cuscuta

कसकयटा Correct Answer Cuscuta

Haustorial roots -The roots of parasitic

plants which penetrate into the host

tissues to absorb nourishment are

called haustorial roots hellip Also known as suckingor parasitic roots

Q229 Equs Asinus is the scientific name

of

एकवसएलसनस mdashmdashndash कावहॴजञाननकनामहहॴ 8-

Feb-2017

Options

1) Donkey

गधा 2) Cow

गाय

3) Deer

टहरन

4) Kangaroo

कगा

Correct Answer Donkey

Q230 Ficus benghalensis is the scientific name of

फाईकसबहॳनगहॳिहॳलसस mdashndash कापवजञाननकनामहहॴ 8-Feb-2017

Options

1) Banyan

बरगद

2) Pineapple

अनानास

3) Babul

बबि

4) Tulsi

तिसी Correct Answer Banyan

Q231 Equus burchellii is the scientific name of

एकवसबचिी mdashmdash- कापवजञाननकनामहहॴ 8-Feb-2017

Options

1) Horse

घह८िा 2) Zebra

जहॳिा 3) Buffalo

F A C E B O O K

P A G E h t t p w w w f a c e b o o k c o m s s c m e n t o r s o f f i c i a l P a g e | 53

FOR MORE UPDATES AND MORE MATERIAL DO LIKE OUR FACEBOOK PAGE httpwwwfacebookcomsscmentorsofficial

भस

4) Ass

गधा Correct Answer Zebra

Page 11: COMPILATION OF ALL 72 SETS OF BIOLOGY SSC CHSL-2016 · OF BIOLOGY SSC CHSL-2016 PREPARED BY : SSC MENTORS BIOLOGY SPECIAL . F A C E B O O K P A G E : h t t p : / / w w w . f a c e

F A C E B O O K

P A G E h t t p w w w f a c e b o o k c o m s s c m e n t o r s o f f i c i a l P a g e | 10

FOR MORE UPDATES AND MORE MATERIAL DO LIKE OUR FACEBOOK PAGE httpwwwfacebookcomsscmentorsofficial

नननननलिखितपह९धह८मसहॳककसकीजड़ह८मगाठहह८तीहहॴ

11-Jan-2017

Options

1) Leguminous plants

िहॳगयलमनसपह९धहॳ 2) Parasitic plants

परजीवीपह९धहॳ 3) Epiphytic Plants

एपीफाइटटकपह९धहॳ 4) Aquatic Plants

जिीयपह९धहॳ Correct Answer Leguminous plants

Q38 Earth-worms belongs to the

phylum

कहॳ चएmdashmdash- परजानतकहॳ अतगयतआतहॳहहॴ 11-Jan-2017

Options

1) Protozoa

परह८टह८जआ

2) Cnidaria

नीिहॳररया 3) Annelida

एनीलििा 4) Mollusca

मह८िसका Correct Answer Annelida

Q39 Ringworm is a disease caused by

ररगवमयनामकबीमारी mdashmdash- कहॳ कारणहह८तीहहॴ 11-Jan-2017

Options

1) Fungi

कवक

2) Bacteria

बहॴकटीररया 3) Virus

वायरस

4) Flies

मनकियाा Correct Answer Fungi

Q40 Mangifera indica is the scientific

name of

मननगफहॳ राइडिकाककसकावहॴजञाननकनामहहॴ 11-

Jan-2017

Options

1) Guava

अम द

2) Mango

आम

3) Amla

आविा 4) Jack fruit

कटहि

Correct Answer Mango

Q41 Crabs belongs to the phylum

कहॳ कड़हॳmdashmdash- परजानतकहॳ अतगयतआतहॳहहॴ 11-Jan-2017

Options

1) Mollusca

मह८िसका 2) Cnidaria

नीिहॳररया 3) Arthropoda

अरोपह८ड़ा 4) Platyhelminthes

पिहॳटटहहॳनममननिस

Correct Answer Arthropoda

Q42 Myopia is a defect of eyes which is

also known as

मायह८पपयाआिोकादह८षहहॴ नजसहॳ mdashmdashndash

भीकहाजाताहहॴ

12-Jan-2017

Options

1) Far Sightedness

F A C E B O O K

P A G E h t t p w w w f a c e b o o k c o m s s c m e n t o r s o f f i c i a l P a g e | 11

FOR MORE UPDATES AND MORE MATERIAL DO LIKE OUR FACEBOOK PAGE httpwwwfacebookcomsscmentorsofficial

दरदनषटदह८ष

2) Near Sightedness

ननकटदनषटदह८ष

3) Astigmatism

एसटीगमहॳटटजम

4) Night Blindness

रतोधी Correct Answer Near Sightedness

Myopia occurs when the eyeball is too

long relative to the focusing power of

the cornea and lens of the eye This

causes light rays to focus at a point in

front of the retina rather than directly

on its surface

Hyperopia Hypermetropia (

Farsightedness )- when light rays

entering the eye focus behind the retina

rather than directly on it The eyeball of

a farsighted person is shorter than

normal

Astigmatism usually is caused by an

irregularly shaped cornea Instead of

the cornea having a symmetrically

round shape (like a baseball) it is

shaped more like an American football

Nyctalopia also called night ndash blindness

is a condition making it difficult or

impossible to see in relatively low light

Q43 Who is known as the father of

Green Revolution

हररतिानतकहॳ जनककहॳ पमककसहॳजानाजाताहहॴ

12-Jan-2017

1) Dr Robert Nucleus

िॉ रॉबटयनयनकियस

2) Dr Ian Wilmut

िॉ इयानपविमट

3) Dr NE Borlaug

िॉ एनईबह८रिॉग

4) Dr JC Bose

िॉ जहॳसीबह८स

Correct Answer Dr NE Borlaug

Q44 Panthera Tigris is the scientific

name of

पिहॳराटटगरीस mdashmdashmdash कावहॴजञाननकनामहहॴ 12-Jan-2017

Options

1) Panther

तदआ

2) Tiger

बाघ

3) Whale

हहॳि

4)Goat

बकरी Correct Answer Tiger

Q45 How many facial bones are there

हमारहॳचहॳहरहॳमककतनीहडडियााहह८तीहहॴ 13-Jan-2017

Options

1)34

2)24

3)14

4)4

Correct Answer 14

Q46 ndash Halophytes are plants that grow

in

हहॴिह८फाईटसवहॳपह९धहॳहह८तीहहॴजह८ mdash- मउगतहॳहहॴ SSC CHSL Science (biology) 2016

Question Paper

13-Jan-2017

Options

1) Fresh Water

ताजापानी 2) Cold Water

ठिापानी 3) Ponds

तािाब

4) Salt Water

िारापानी Correct Answer Salt Water

F A C E B O O K

P A G E h t t p w w w f a c e b o o k c o m s s c m e n t o r s o f f i c i a l P a g e | 12

FOR MORE UPDATES AND MORE MATERIAL DO LIKE OUR FACEBOOK PAGE httpwwwfacebookcomsscmentorsofficial

Q47 Felis Catus is the scientific name of

फहॳ लिसकहॴ टस mdashndash कावहॴजञाननकनामहहॴ 13-Jan-2017

Options

1) Cat

बबमिी 2) Dog

कतता 3) Mouse

चहा 4) Porcupine

साही Correct Answer Cat

Q48 Which of the following induces

nitrogen fixation in soil

नननननलिखितमसहॳकह९नलम ीमनाइटरह८जनननयतनकह८परहॳररतकरताहहॴ

15-Jan-2017

Options

1) Protozoa

परह८टह८जआ

2) Bacteria

बहॴकटीररया 3) Fungi

कवक

4)Algae

िहॴवाि

Correct Answer Bacteria

Bacteria that change nitrogen gas from

the atmosphere into solid nitrogen

usable by plants are called nitrogen-

fixing bacteria These bacteria are

found both in the soil and in symbiotic

relationships with plants

They contain symbiotic bacteria called

rhizobia within nodules in their root

systems producing nitrogen compounds

that help the plant to grow and compete

with other plants When the plant dies

the fixed nitrogen is released making it

available to other plant

Q49 Which of the following is the

largest known cell

नननननलिखितमसहॳकह९नसीसबसहॳबड़ीजञातकह८लिकाहहॴ

SSC CHSL Science (biology) 2016

Question Paper

15-Jan-2017

1) Eukaryotic Cell

यकहॳ ररयह८टटककह८लिका 2) Prokaryotic Cell

परह८कहॳ ररयह८टटककह८लिका 3) Mycoplasma

मायकह८पिासम

4) Ostrich Eggs

ितरमगयकाअिा Correct Answer Ostrich Eggs

Q50 The association of animals in

which both the partners are benefitted

is known as

जानवरोकावहसहयह८गनजसमहॳदह८नोभागीदारिाभापवनतहह८तहॳहहॴ उसहॳ mdashmdashndash कहॳ पमजानाजाताहहॴ SSC CHSL Science (biology) 2016

Question Paper

15-Jan-2017

Options

1) Amensalism

सहजीपवत

2) Commensalism

परजीपवत

3) Colony

कॉिनी 4) Mutualism

अनयह८नयाशरयवाद

Correct Answer Mutualism

Amensalism is any relationship between

organisms of different species in which

F A C E B O O K

P A G E h t t p w w w f a c e b o o k c o m s s c m e n t o r s o f f i c i a l P a g e | 13

FOR MORE UPDATES AND MORE MATERIAL DO LIKE OUR FACEBOOK PAGE httpwwwfacebookcomsscmentorsofficial

one organism is inhibited or destroyed

while the other organism remains

unaffected

Commensalism an association between

two organisms in which one benefits and

the other derives neither benefit nor

harm

Q51 Pneumonia affects which of the

following organs of human body

ननमह८ननयामानविरीरकहॳ नननननलिखितमसहॳककसअगकह८परभापवतकरताहहॴ

15-Jan-2017

Options

1)Kidneys

गद

2)Lungs

फहॳ फड़हॳ 3) Throat

गिहॳ 4) Liver

यकत

Correct Answer Lungs

When the germs that cause pneumonia

reach your lungs the lungsrsquo air sacs

(alveoli) become inflamed and fill up

with fluid This causes the symptoms of

pneumonia such as a cough fever

chills and trouble breathing When you

have pneumonia oxygen may have

trouble reaching your blood

Q52 Mendel is known as

मििकह८ mdashmdash- कहॳ पमजानाजाताहहॴ 15-Jan-2017

Options

1) Father of Physiology

िरीरकियािासतरकहॳ जनक

2) Father of Geology

भगभयिासतरकहॳ जनक

3) Father of Genetics

जहॳनहॳटटकसकहॳ जनक

4) Father of Biology

जीविासतरकहॳ जनक

Correct Answer Father of Genetics

Q53 Which of the following are also

known as Suicidal bag of Cells

ननननलिखितमसहॳककसहॳआतमहतयाकरनहॳवािीकह८लिकाओकाबहॴगकहाजाताहहॴ

15-Jan-2017

Options

1) Lysosomes

िायसोसह८म

2) Lycosome

िायकह८सह८म

3) Nucleus

नालभक

4) Chromosome

िह८मह८सह८म

Correct Answer Lysosomes

Q54 Mesothelioma is a type of cancer

The most common area affected in it is

the lining of the ________

लमज़ह८िहॳिहॳलमयाक सरकाएकपरकारहहॴ इससहॳपरभापवतहह८नहॳवािासबसहॳसामानयकषहॳतर mdash

mdashmdash काअसतरहहॴ 15-Jan-2017

Options

1)Heart

हदय

2)Brain

मनसतषक

3)Stomach

आमािय

4)Lungs

फहॳ फड़हॳ Correct Answer lungs

Asbestos exposure is the main cause of

pleural mesothelioma When asbestos

fibers are breathed in they travel to the

F A C E B O O K

P A G E h t t p w w w f a c e b o o k c o m s s c m e n t o r s o f f i c i a l P a g e | 14

FOR MORE UPDATES AND MORE MATERIAL DO LIKE OUR FACEBOOK PAGE httpwwwfacebookcomsscmentorsofficial

ends of small air passages and reach the

pleura where they can cause

inflammation and

scarring

Q55 Which one of the following is an

insectivorous plant

नननननलिखितमसहॳकह९नसाएकककटाहरीवनसपनतहहॴ

15-Jan-2017

Options

1) Utricularia

यटरीकिहॳररया 2) Sequoia

सहॳकयओइया 3) Nostoc

नॉसटह८क

4) Bryophyta

िायह८फाईटा Correct Answer Utricularia

Q56 ______________ is a

multibranched polysaccharide of

glucose that serves as a form of energy

storage in animals and fungi

mdashmdashगिकह८जकाएकबहिािायकतपह८िीसहॳकहॳ राइिहहॴ जह८जानवरोऔरकवकमउजायभणिारणकहॳ एक पमकाययकरताहहॴ 15-Jan-2017

Options

1) Cellulose

सहॳमयिह८ज

2) Glycogen

गिायकह८जन

3) Pectin

पहॳनकटन

4) Chitin

चीटटन

Correct Answer Glycogen

Q57 The largest gland of the human

body is

mdashmdashmdashमानविरीरकीसबसहॳबड़ीगरिीहहॴ 16-Jan-2017

Options

1) Pancreas

अगयािय

2) Thyroid

िायरॉइि

3) Large Intestine

बड़ीआत

4) Liver

यकत

Correct Answer Liver

Q58 Photosynthesis in plants takes

place in

वनसपनतयोमपरकािसशिहॳषणकीकियाहह८तीहहॴ

16-Jan-2017

Options

1) Stem

तना 2) Leaves

पनततयाा 3) Roots

जड़हॳ 4) Flower

फि

Correct Answer Leaves

During this reaction carbon dioxide

and water are converted into glucose

and oxygen The reaction requires light

energy which is absorbed by a green

substance called

chlorophyll Photosynthesis takes place

in leaf

cells These contain chloroplasts which

are tiny objects containing chlorophyll

F A C E B O O K

P A G E h t t p w w w f a c e b o o k c o m s s c m e n t o r s o f f i c i a l P a g e | 15

FOR MORE UPDATES AND MORE MATERIAL DO LIKE OUR FACEBOOK PAGE httpwwwfacebookcomsscmentorsofficial

Q59 Insects that transmit diseases are

known as

जह८कीड़हॳरह८गसचाररतकरतहॳहहॴ उनह mdashmdash-

कहॳ नामसहॳजानाजाताहहॴ 16-Jan-2017

1)Pathogens

रह८गज़नक

2) Vectors

वहॳकटर

3) Drones

परजीवी 4)Scalars

अटदषट

Correct Answer Vectors

A vector is an organism that does not

cause disease itself but which spreads

infection by conveying pathogens from

one host to another Species of mosquito

for example serve as vectors for the

deadly disease Malaria

Q60 Which is the second largest gland

of Human body

मानविरीरकीदसरीसबसहॳबड़ीगरिीकह९नसीहहॴ

SSC CHSL Science (biology)

2016 Question Paper

16-Jan-2017

Options

1) Liver

यकत

2) Large Intestine

बड़ीआत

3) Thorax

छाती 4) Pancreas

अगनयािय

Correct Answer Pancreas

Q61 Annona squamosa is the scientific

name of

एनह८नासकवामह८सा (Annona squamosa) mdash

mdashmdash कावहॴजञाननकनामहहॴ 16-Jan-2017

Options

1) Custard Apple

सीताफि

2) Papaya

पपीता 3) Babhul

बबि

4) Drumstick

सहजन

Correct Answer Custard Apple

Q62 The disease Beri Beri is caused due

to the deficiency of which of the

following

बहॳरीबहॳरीरह८गनननननलिखितमसहॳककसकीकमीकहॳकारणहह८ताहहॴ

16-Jan-2017

Options

1) Vitamin B2

पवटालमन B2

2) Vitamin B1

पवटालमन B1

3) Vitamin B12

पवटालमन B12

4) Vitamin E

पवटालमन E

Correct Answer Vitamin B1

Beriberi is a disease caused by a vitamin

B-1 deficiency also known as thiamine

deficiency

Q63 Chlorophyll was first isolated and

named by

किह८रह८कफिकह८ mdash-

दवारापहिहॳपिकऔरनालमतककयागया 16-Jan-2017

F A C E B O O K

P A G E h t t p w w w f a c e b o o k c o m s s c m e n t o r s o f f i c i a l P a g e | 16

FOR MORE UPDATES AND MORE MATERIAL DO LIKE OUR FACEBOOK PAGE httpwwwfacebookcomsscmentorsofficial

Options

1) Caventou

कहॳ वहॳत 2) Pelletier

पहॳिहॳटटयर

3) Chlorophyll

किह८रह८कफि

4) Caventou and Pelletier

कहॳ वहॳतऔरपहॳिहॳटटयर

Correct Answer Caventou and Pelletier

Chlorophyll was first isolated and

named by

Joseph Bienaimeacute Caventou and Pierre

Joseph Pelletier in 1817 The presence of

magnesium in chlorophyll was

discovered in 1906 and was the first

time that magnesium had been detected

in living tissue

Q64 Which of the following organisms

does not fit into the Cell Theory

नननननलिखितमसहॳकह९नसाजीवकह८लिकालसदातअन पनहीहहॴ

16-Jan-2017

Options

1) Bacteria

बहॴकटीररया 2) Virus

वायरस

3) Fungi

कवक

4) Plants

पह९धहॳ Correct Answer Virus

The bottom line is that viruses are not

alive and not related to cells in any way

The cell theory states that all living

things are made of cells cells are the

basic units of structure and function of

living things and that all cells come

from other cells Since viruses are not

made of cells and do not use cells in any

of their processes they are not related to

the cell theory

Q65 Which of these is not a

macronutrient for Plants

नननननलिखितमसहॳकह९नसापह९धह८कहॳ लिएमिह८नयटरीएटनहीहहॴ

SSC CHSL Science (biology) 2016

Question Paper

17-Jan-2017

Options

1) Nitrogen

नाइटरह८जन

2) Phosphorus

फासफह८रस

3) Potassium

पह८टालसयम

4) Chlorine

किह८रीन

Correct Answer Chlorine

In relatively large amounts the soil

supplies nitrogen phosphorus

potassium calcium magnesium and

sulfur these are often called the

macronutrients In relatively small

amounts the soil supplies iron

manganese boron molybdenum

copper zinc chlorine and cobalt the

so-called micronutrients

Q66 Name the respiratory organs of

insects

कीटह८मनसतिशरवसनअगनामकानामहहॴ

17-Jan-2017

Options

1) Skin

तवचा 2) Body Surface

िरीरकीसतह

F A C E B O O K

P A G E h t t p w w w f a c e b o o k c o m s s c m e n t o r s o f f i c i a l P a g e | 17

FOR MORE UPDATES AND MORE MATERIAL DO LIKE OUR FACEBOOK PAGE httpwwwfacebookcomsscmentorsofficial

3) Gills

गिफड़हॳ 4) Tracheae

शरावस- निी Correct Answer Tracheae

Air enters the respiratory systems of

insects through a series of external

openings called

spiracles These external openings

which act as muscular valves in some

insects lead to the internal respiratory

system a densely networked array of

tubes called tracheae

Q67 The poisonous gas accidentally

released in Bhopal Gas Tragedy is

भह८पािगहॴसतरासदीमगितीसहॳमकतहईजहरीिीगहॴसिी

17-Jan-2017

1) Methane

मीिहॳन

2) Nitrous Oxide

नाइटरसऑकसाइि

3) Methyl Isocyanate

महॴचििआयसोसायनहॳट

4) Cyanogen

सायनह८जहॳन

Correct Answer Methyl Isocyanate

Q68 What does Trypsin do

टटरनपसनकयाकरताहहॴ

SSC CHSL Science (biology) 2016

Question Paper

17-Jan-2017

Options

1) Breaks down Carbohydrates

काबोहाइडरहॳटकापवघटनकरताहहॴ 2) Synthesizes proteins

परह८टीनकासििहॳषणकरताहहॴ 3) Breaks down fats

वसाकापवघटनकरताहहॴ 4) Breaks down proteins

परह८टीनकापवघटनकरताहहॴ Correct Answer Breaks down proteins

Trypsin is one of the three principal

digestive

proteinases the other two being pepsin

and

chymotrypsin In the digestive process

trypsin acts with the other proteinases

to break down dietary protein molecules

to their component

peptides and amino acids

A protease is any enzyme that performs

proteolysis protein catabolism by

hydrolysis of peptide bonds

Q69 Name the source from which

Aspirin is produced

उससरह८तकानामबताइए

नजससहॳएनसपररनकाउतपादनककयाजाताहहॴ

17-Jan-2017

Options

1) Willow bark

पविह८कीछाि

2) Oak Tree

ओककावकष

3) Acacia

बबि

4) Eucalyptus

नीिचगरी Correct Answer Willow bark

The compound from which the active

ingredient in aspirin was first derived

salicylic acid was found in the bark of a

willow tree in 1763 by Reverend

Edmund Stone of Chipping-Norton

Q70 Cannis Familiaris is the scientific

name of

कहॴ ननसफहॳ लमलियहॳररस mdash- कावहॴजञाननकनामहहॴ

17-Jan-2017

F A C E B O O K

P A G E h t t p w w w f a c e b o o k c o m s s c m e n t o r s o f f i c i a l P a g e | 18

FOR MORE UPDATES AND MORE MATERIAL DO LIKE OUR FACEBOOK PAGE httpwwwfacebookcomsscmentorsofficial

Options

1) Cat

बबमिी 2)Dog

कतता 3) Fox

िह८मड़ी 4) Wolf

भहॳडड़या Correct Answer Dog

Q71 Harmful bacteria in potable water

make the water

पीनहॳकहॳ पानीमनसतिघातकबहॴकटीररयाउसपानीकह८बनातहॳहहॴ 17-Jan-2017

Options

1) unfit to drink

पीनहॳकहॳ लिएअयह८गय

2) smelly

दगयनधयकत

3) Colored

रगीन

4) Turbid

मटमहॴिा Correct Answer unfit to drink

Q72 Musa paradisiaca is the scientific

name of which plant

मसापहॴराडिलसयाकाककसपह९धहॳकावहॴजञाननकनामहहॴ

17-Jan-2017

Options

1) Mango

आम

2) Wheat

गहॳह

3) Corn

भ ा 4) banana

कहॳ िा Correct Answer banana

Q73 Prawns belong to which family

झीगहॳककसपररवारकहॳ हह८तहॳहहॴ 17-Jan-2017

Options

1) Crustaceans

िसटहॳलियन

2)Fish

मछिी 3) Amphibians

अननफबबयस

4) Reptiles

रहॳपटाइमस

Correct Answer Crustaceans

Q74 Name the drug that is yielded from

Cinchona tree and is used to cure

malaria

उसऔषचधकानामबताइएनजसहॳलसगकह८नापहॳड़सहॳपरापतककयाजाताहहॴऔरनजसकाउपयह८गमिहॳररयाकहॳ उपचारमककयाजाताहहॴ 17-Jan-2017

Options

1) Camptothea

कहॴ नटह८चिया 2) Acuminata

एकयलमनहॳटा 3) Quinine

कनहॴन

4) Cinchonia

लसकह८ननया Correct Answer Quinine

Q75 Blood Circulation was discovered

by

रकतपररसचरणकी mdashmdashndash दवारािह८जकीिी 17-Jan-2017

Options

1) Mary Anderson

F A C E B O O K

P A G E h t t p w w w f a c e b o o k c o m s s c m e n t o r s o f f i c i a l P a g e | 19

FOR MORE UPDATES AND MORE MATERIAL DO LIKE OUR FACEBOOK PAGE httpwwwfacebookcomsscmentorsofficial

महॴरीएिरसन

2) Virginia Apgar

वनजयननयाएपगार

3) William Harvey

पवलियमहाव

4) Robert Feulgen

रॉबटयफ़यिजहॳन Correct Answer William Harvey

Q76 Vitamin A is also known as

पवटालमन A कह८ mdashmdash- कहॳ नामसहॳभीजानाजाताहहॴ SSC CHSL Science (biology) 2016

Question Paper

18Jan2017

Options

1) Thiamine

िायलमन

2) Riboflavin

ररबह८फिहॳपवन

3) Retinol

रहॳटटनॉि

4) Calciferol

कहॴ नमसफहॳ रह८ि

Correct Answer Retinol

Q77 Some roots called arise from an

organ other than the radicle

कछजड़हॳनजनह mdashmdashmdash कहाजाताहहॴ वहमिकहॳ अिावाककसीअनयअगसहॳउतपननहह८तीहहॴ 18Jan2017

Options

1) tap roots

मखयजड़

2) stilt roots

ि ाजड़

3) fibrous roots

रहॳिहॳदारजड़

4) adventitious roots

आकनसमकजड़

Correct Answer adventitious roots

Q78 Spiders belong to which class of

animals

मकडड़यापराणीवगीकरणकहॳ ककसवगयमआतीहहॴ 18Jan2017

Options

1) Arachnids

एरहॳकननडस

2) Aves

एपवस

3) Gastropods

गहॴसटरोपह८िस

4) Anthozoa

एिह८जआ

Correct Answer Arachnids

Q79 How many layers does Human

Skin have

मानवतवचामककतनीपरतहॳहह८तीहहॴ

18Jan2017

Options

1) 5

2) 7

3) 11

4) 3

Correct Answer 3

Skin has three layers The epidermis

the outermost layer of skin provides a

waterproof barrier and creates our skin

tone The dermis beneath the

epidermis contains tough connective

tissue hair follicles and sweat glands

The deeper subcutaneous tissue (

hypodermis ) is made of fat and

connective tissue

Q80 Allium Cepa is the scientific name

of

एलियमलसपपा mdashmdashndash कावहॴजञाननकनामहहॴ 18Jan2017

F A C E B O O K

P A G E h t t p w w w f a c e b o o k c o m s s c m e n t o r s o f f i c i a l P a g e | 20

FOR MORE UPDATES AND MORE MATERIAL DO LIKE OUR FACEBOOK PAGE httpwwwfacebookcomsscmentorsofficial

Options

1) Carrot

गाजर

2) Tomato

टमाटर

3) Potato

आि 4) Onion

पयाज़

Correct Answer Onion

Q81 DNA stands for

िीएनएकापणय प mdashmdash- हहॴ 18Jan2017

Options

1) Di Nucleic Acid

िाईनयनकिकएलसि

2) Deoxy Nucleic Acid

िीओकसीनयनकिकएलसि

3) Diribonucleic Acid

िाईराइबह८नयनकिकएलसि

4) Deoxyribonucleic Acid

िीऑकसीराइबह८नयनकिकएलसि

Correct Answer Deoxyribonucleic Acid

Q82 Organisms that generate energy

using light are known as

जह८जीवाणपरकािकाउपयह८गकरउजायउतपननकरतीहहॴ उनह mdashmdash कहॳ पमजानाजाताहहॴ

18Jan2017

Options

1) Chaemolithotrophs

ककमह८लििह८टरह८पस

2) Oligotrophs

ओलिगह८टरह८पस

3) Bacteria

बहॴकटीररया 4)Photoautotrophs

फह८टह८ओटह८टरह८पस

Correct Answer Photoautotrophs

An oligotroph is an organism that can

live in an environment that offers very

low levels of nutrients

Q83 Which drug is used as an

Antidepressant

ककसदवाएकहतािारह८धीकहॳ पमपयोगककयाजाताहहॴ Options

1) Oxybutynin

ओकसीलयटीनन

2)Tramadol

टरहॳमहॳिह८ि

3 ) Sumatriptan

समहॳटरीपटहॳन

4) Bupropion

लयपरह८पपयह८न

Correct Answer Bupropion

लयपरह८पपयह८न

Q84 The orange colour of carrot is

because of

गाजरकानारगीरगनननननलिखितमसहॳककसीएककीवजहसहॳहह८ताहहॴ 18Jan2017

Options

1) it grows in the soil

यहलम ीमउगतीहहॴ 2) Carotene

कहॴ रह८टीन

3) it is not exposed to sunlight

यहसययपरकािकहॳ सपकय मनहीआती 4) the entire plant is oranqe in colour

सनपणयपह९धानारगीरगकाहह८ताहहॴ Correct Answer Carotene

Q85 Snake venom is highly modified

saliva containing

F A C E B O O K

P A G E h t t p w w w f a c e b o o k c o m s s c m e n t o r s o f f i c i a l P a g e | 21

FOR MORE UPDATES AND MORE MATERIAL DO LIKE OUR FACEBOOK PAGE httpwwwfacebookcomsscmentorsofficial

सापकाजहरअततयाचधकसिह८चधतिारहह८तीहहॴनजसमहॳ mdashmdash- हह८ताहहॴ Options

l)Prototoxins

परह८टह८टॉनकसस

2)Neutrotoxins

नयटरोटॉनकसस

3)Zootoxins

जटॉनकसस

4)Electrotoxins

इिहॳकटरह८टॉनकसस

Correct Answer Zootoxins

जटॉनकसस

Q86 Which type of pathogen causes the

water-borne disease Schistosomiasis

ककसपरकारकारह८गज़नकजिजननतरह८गलससटह८सह८लमलससकाकारणबनताहहॴ

18Jan2017

Option

1) Parasitic

परजीवी 2)Protozoan

परह८टह८जआ

3) Bacterial

बहॴकटीररयि

4) Viral

वायरि

Correct Answer Parasitic

Schistosomiasis also known as snail

fever and bilharzia is a disease caused

by parasitic

flatworms called schistosomes

Q87 Prothrombin responsible for

clotting of blood is released by

परह८िह८ननबन

जह८रकतकािककाजमनहॳकहॳ लिएनजनमहॳदारहहॴ mdashndash

कहॳ दवारासतरापवतककयाजाताहहॴ

19Jan2017

Options

1) Small Intestine

छह८टीआत

2) Blood Platelets

रकतपिहॳटिहॳटस

3) Large Intestine

बड़ीआत

4Heart

हदय

Correct Answer Blood Platelets

Q88 Acacia arabica is the scientific

name of

अकहॳ लियाअरहॳबबका mdashmdashndash कावहॴजञाननकनामहहॴ 19-Jan-2017

Options

1) Neem

नीम

2) Teak

सागह९न

3) Babhul

बबि

4) Pomegranate

अनार

Correct Answer Babhul

Q89 Cannis Vulpes is the scientific

name of

कहॴ ननसवनमपस mdashmdash- कावहॴजञाननकनामहहॴ 19-Jan-2017

Options

1) Dog

कतता 2) Wolf

भहॳडड़या 3) Fox

िह८मड़ी 4) Hyena

िाकिबगघा

F A C E B O O K

P A G E h t t p w w w f a c e b o o k c o m s s c m e n t o r s o f f i c i a l P a g e | 22

FOR MORE UPDATES AND MORE MATERIAL DO LIKE OUR FACEBOOK PAGE httpwwwfacebookcomsscmentorsofficial

Correct Answer Fox

Q90 The beetroot is the portion of the

beet plant

चकदरपह९धहॳका mdashmdashndash भागहहॴ 19-Jan-2017

Options

1) tap root

मखयजड़

2) Adventitious

आकनसमक

3) bulb of the stem

तनहॳकाकद

4) Rhizome

परकद

Correct Answer tap root

Q91 What is the basic unit of heredity

आनवलिकताकीबननयादीइकाईकयाहहॴ 19-Jan-2017

Options

1) DNA

िीएनए

2) RNA

आरएनए

3) Chromosome

िह८मह८सह८म

4) Gene

जीन

Correct Answer gene

Genes are the units of heredity and are

the instructions that make up the bodyrsquos

blueprint They code for the proteins

that determine virtually all of a personrsquos

characteristics Most genes come in

pairs and are made of strands of genetic

material called deoxyribonucleic acid

or DNA

Q92 Lungs are the primary organs of

फहॳ फड़हॳmdashndashकहॳ परािलमकअगहहॴ

19-Jan-2017

Options

1) Digestion

पाचन

2) Constipation

कलज

3) Perspiration

पसीना 4)Respiration

शवसन

Correct Answer Respiration

Q93 Sugarcane is a type of

गननाएकपरकारका mdash- हहॴ 20-Jan-2017

Options

1)creeper

िता 2)tree

पहॳड़

3)shrub

झाड़ी 4)grass

घास

Correct Answer grass

Q94 Who is commonly known as ldquothe

Father of Microbiologyrdquo

सामानयत ldquo सकषमजीवपवजञानकहॳ जनक lsquo

कहॳ नामसहॳककसहॳजानाजातहहॴ 20-Jan-2017

Options

1) Robert Hooke

रॉबटयहक

2) Antonie Philips van Leeuwenhoek

एटह८नीकफलिपवानमयएनहह८क

3) Carl Linnaeus

काियिीनाईयस

4) Charles Darwin

चामसयिापवयन

F A C E B O O K

P A G E h t t p w w w f a c e b o o k c o m s s c m e n t o r s o f f i c i a l P a g e | 23

FOR MORE UPDATES AND MORE MATERIAL DO LIKE OUR FACEBOOK PAGE httpwwwfacebookcomsscmentorsofficial

Correct Answer Antonie Philips van

Leeuwenhoek

Q95 For the aquatic organisms the

source of food is

जिीयजीवाणकािाघसरह८तहहॴ 20-Jan-2017

Options

1) Phytoplankton

फायटह८पिहॳकटन

2) Sea Weed

समदरीिहॴवाि

3)Aqua plankton

एकवापिहॳकटन

4) Zooplankton

जपिहॳकटन

Correct Answer Phytoplankton

Q96 Haemoglobin has the highest

affinity with which of the following

हीमह८गिह८बबनकीननननमसहॳककसकहॳ सािउततमसमानताहहॴ

20-Jan-2017

Options

1)SO2

2)CO2

3)CO

4)NO2

Correct Answer CO

It has a greater affinity for hemoglobin

than oxygen does It displaces oxygen

and quickly binds so very little oxygen

is transported through the body cells

Q97 Who developed the theory of

Evolution

उदपवकासकालसदातककसनहॳपवकलसतककया

20-Jan-2017

Options

1) Charles Darwin

चामसयिापवयन

2) Isaac Newton

आयजहॳकनयटन

3) Pranav Mistry

परणवलमसतरी 4) Galileo Galilei

गहॳलिलियह८गहॳिीिी Correct Answer Charles Darwin

Q98 The primary function of RNA is

RNA कापरािलमककाययहह८ताहहॴ 20-Jan-2017

Options

1) Photosynthesis

परकािसशिहॳषण

2) Protein Synthesis

परह८टीनसशिहॳषण

3) Replication

परनतकनतबनाना 4) Translation

अनवादकरना Correct Answer Protein Synthesis

There are two main functions of RNA

It assists DNA by serving as a messenger

to relay the proper genetic information

to countless numbers of ribosomes in

your body The other main function of

RNA is to select the correct amino acid

needed by each ribosome to build new

proteins for your body

Q99 ______is the movement of

molecules across a cell membrane from

a region of their lower concentration to

a region of their higher concertration

उचचसादरताकहॳ कषहॳतरसहॳउसकीकमसादरतावािहॳकषहॳतरकीतरफएककह८लिकाखझमिीकहॳ माधयमसहॳहह८नहॳवािाअणओकहॳ सचिनकह८ mdash- कहतहॳहहॴ Options

1) Diffusion

पवसरण

2) Osmosis

ऑसमह८लसस

F A C E B O O K

P A G E h t t p w w w f a c e b o o k c o m s s c m e n t o r s o f f i c i a l P a g e | 24

FOR MORE UPDATES AND MORE MATERIAL DO LIKE OUR FACEBOOK PAGE httpwwwfacebookcomsscmentorsofficial

3) Active Transport

सकियआवागमन

4) Passive Transport

नननषियआवागमन

Correct Answer Active Transport

Q100 Study of classification of

organisms is known as 20-Jan-2017

जीवाणओकहॳ वगीकरणकहॳ अधययनकह८ mdash-

कहाजाताहहॴ Options

1) Serpentology

सपरहॳटह८िह८जी 2) Virology

वायरह८िह८जी 3) Taxonomy

टहॴकसोनह८मी 4) Physiology

कफनज़यह८िह८जी Correct Answer Taxonomy

Q101 Photosynthesis takes place inside

plant cells in

परकािसशिहॳषणवनसपनतकह८लिकामनसति mdash

mdashmdash महह८ताहहॴ 20-Jan-2017

Options

1) Ribosomes

राइबह८सह८नस

2) Chloroplasts

किह८रह८पिासट

3) Nucleus

नयकलियम

4) Mitochondria

माईटह८कोडडरया Correct Answer Chloroplasts

Q102 ______ is the cell organelle in

which the biochemical processes of

respiration and energy production

occur

mdashmdash- वहकह८लिकाअगहहॴ नजसमहॳशवसनऔरउजायउतपादनकहॳ जहॴसीजहॴवरासायननकपरकियायहह८तीहहॴ 20-Jan-2017

Options

1) Mitochondria

माइटह८कोडडरया 2) Chloroplast

किह८रह८पिासट

3) Ribosomes

राइबह८सह८नस

4) Nucleus

नयकिीयस

Correct Answer Mitochondria

Q103 Which non-flowering spore

bearing plants have roots

ककसफिनिगनहॳवािहॳऔरबीजाणधारकपह९धह८कीजड़हॳहह८तीहहॴ 21-Jan-2017

Options

1) Mosses

मह८सहॳस

2) Angiosperms

एननजयह८सपनसय 3) Ferns

फनसय 4) Gymnosperms

नजननह८सपनसय Correct Answer ferns

Q104 Which of the following is an

excretory organ of cockroach

नननननलिखितमसहॳकह९नसानतिच हॳकाउतसजयनअगहहॴ

21-Jan-2017

Options

F A C E B O O K

P A G E h t t p w w w f a c e b o o k c o m s s c m e n t o r s o f f i c i a l P a g e | 25

FOR MORE UPDATES AND MORE MATERIAL DO LIKE OUR FACEBOOK PAGE httpwwwfacebookcomsscmentorsofficial

1) Malphigian Tubules

मनमफनजयनटयबमस

2) Nephridia

नहॳकफरडिया 3) Coxal Gland

कह८कसिगरचिया 4) Green Gland

गरीनगरचिया Correct Answer Malphigian Tubules

Q105 Evaporation of water takes place

in which part of plants

पानीकहॳ वाषपीकरणकीकियापह९धोकहॳ ककसभागसहॳहह८तीहहॴ 21-Jan-2017

Options

1) Stem

तना 2) Stomata

सटह८मटा 3) Branch

िािाए

4) Fruit

फि

Correct Answer Stomata

Evaporation accounts for the movement

of water to the air from sources such as

the soil canopy interception and

waterbodies Transpiration accounts for

the movement of water within a plant

and the subsequent loss of water as

vapour through stomata in its leaves

Q106 A is the fleshy spore-bearing

fruiting body of a fungus

mdashmdashndashकवककामासि

बीजाणधारणकरनहॳवािाफिनहॳवािाअगहहॴ 21-

Jan-2017

Options

1) aloe vera

एिह८वहॳरा 2) Coral

मगा 3) Cactus

कहॴ कटस

4) Mushroom

ककरमतता Correct Answer mushroom

Q107 Which of the following is a fungal

disease

नननननलिखितमसहॳकह९नसाफफदसहॳहह८नहॳवािाएकरह८ग हहॴ

21-Jan-2017

Options

1) Dermatitis

तवचािह८ध

2) Cholera

हहॴजा 3) Jaundice

पीलिया 4) Indigofera

इननिगह८फहॳ रा Correct Answer Dermatitis

Dermatitis also known as eczema is a

group of diseases that results in

inflammation of the skin These diseases

are characterized by itchiness red skin

and a rash In cases of short duration

there may be small blisters while in

long-term cases the skin may become

thickened

Q108 In which form is glucose stored in

our body

हमारहॳिरीरमगिकह८जकासचयककस पमककयाजाताहहॴ

21-Jan-2017

Options

1) Insulin

F A C E B O O K

P A G E h t t p w w w f a c e b o o k c o m s s c m e n t o r s o f f i c i a l P a g e | 26

FOR MORE UPDATES AND MORE MATERIAL DO LIKE OUR FACEBOOK PAGE httpwwwfacebookcomsscmentorsofficial

इसलिन

2) Glucose

गिकह८ज

3) Glycogen

गिायकह८जहॳन

4) Fat

वसा Correct Answer Glycogen

Excess glucose is stored in the liver as

the large compound called glycogen

Glycogen is a polysaccharide of glucose

but its structure allows it to pack

compactly so more of it can be stored in

cells for later use

Q109 Where do plants synthesize

protein from

पह९धहॳपरह८टीनसशिहॳषणकहासहॳकरतहॳहहॴ

Options

1) Fatty Acids

वसाऐलसि

2) Sugar

िकर

3) Amino Acids

एलमनह८ऐलसि

4) Starch

सटाचय Correct Answer Amino Acids

Q110 Which part of the brain is

responsible for triggering actions like

thinking intelligence memory and

ability to learn

मनसतषककाकह९नसाटहससासह८चनहॳ बनधदमानी याददाशतऔरसीिनहॳकीकषमताजहॴसीकियाओकह८परहॳररतकरताहहॴ 21-Jan-2017

Options

1) Diencephalon

िायएनसहॳफहॳ िह८न

2) Hypothalamus

हयपह८िहॳिहॳमस

3) Cerebrum

सहॳरहॳिम

4) Control

कटरह८ि

Correct Answer Cerebrum

Q111 Which of the following is also

known as the Biochemical Laboratory

of the Human Body

नननननलिखितमसहॳककसहॳमानविरीरकीजहॴवरसायनपरयह८गिािाभीकहाजाताहहॴ 21-Jan-2017

Options

1) Small Intestine

छह८टीआत

2)Brain

मनसतषक

3) Pancreas

अगनयािय

4) Liver

नजगर

Correct Answer Liver

The liver makes bile that will help

emulsify and digest the fats we eat

The liver takes toxic substances and

convert them using enzymes the liver

cells makes into a non toxic form so the

body can dispose of them

The liver also converts fats protein and

carbohydrates into glucose which is the

energy source for our cells to use

The liver takes amino acids and makes

proteins by combining them

Q112 The yellow colour of human urine

is due to

मानवमतरकापीिारग mdashndash कीवजहसहॳहह८ताहहॴ 22-

Jan-2017

Options

1) Bile Salts

F A C E B O O K

P A G E h t t p w w w f a c e b o o k c o m s s c m e n t o r s o f f i c i a l P a g e | 27

FOR MORE UPDATES AND MORE MATERIAL DO LIKE OUR FACEBOOK PAGE httpwwwfacebookcomsscmentorsofficial

पपततनमक

2) Cholesterol

कह८िहॳसटरह८ि

3) Lymph

लिनफ

4) Urochrome

यरह८िह८म

Correct Answer Urochrome

Urobilin or urochrome is the chemical

primarily responsible for the yellow

color of urine

Q113 The wilting of plants takes place

due to

पह९धह८कालिचििहह८नाकी mdashmdash- कीवजहसहॳहह८ताहहॴ 22-Jan-2017

Options

1)Photosynthesis

परकािसशिहॳषण

2) Transpiration

वाषपह८तसजयन

3) Absorption

अविह८षण

4) Respiration

शरवसन

Correct Answer Transpiration

Wilting is the loss of rigidity of non-

woody parts of plants This occurs when

the turgor pressure in non-lignified

plant cells falls towards zero as a result

of diminished water in the cells

Q114 Bovidae Ovis is the scientific name of

बह८पविीओपवस mdashndash कावहॴजञाननकनामहहॴ 22-Jan-2017

Options

1) Goat

बकरी 2) Cow

गाय

3) Buffalo

भहॳस

4) Sheep

भहॳड़

Correct Answer Sheep

Q115 Plants get their energy to produce

food from which of the following

पह८धहॳभह८जनकाननमायणकरनहॳकहॳ लिएनननननलिखितमसहॳककससहॳउजायपरापतकरतहॳहहॴ

22-Jan-2017

Options

1) Photosynthesis

परकािसशिहॳषण

2)Bacteria

बहॴकटीररया 3)Fungi

कवक

4)Sun

सयय Correct Answer Sun

Q116 Which of the following is secreted

by the liver

नननननलिखितमसहॳककसकासरावनजगरसहॳहह८ताहहॴ

22-Jan-2017

Options

1) Glucose

गिकह८ज

2) Iodine

आयह८िीन

3) Cortisol

काटटरयसह८ि

4) Bile

पपतत

Correct Answer Bile

The liver makes bile that will help

emulsify and

digest the fats we eat

F A C E B O O K

P A G E h t t p w w w f a c e b o o k c o m s s c m e n t o r s o f f i c i a l P a g e | 28

FOR MORE UPDATES AND MORE MATERIAL DO LIKE OUR FACEBOOK PAGE httpwwwfacebookcomsscmentorsofficial

Q117 Ferns belong to which division of

plants

फनसयपह९धह८कहॳ ककसभागमआतहॳहहॴ

22-Jan-2017

Options

1) Gymnosperms

नजननह८सपनसय 2) Angiosperms

एनजयह८सपनसय 3) Thallophyta

िहॴिह८फाईटा 4)Pteridophyta

टहॳररिह८फाईटा Correct Answer Pteridophyta

Q118 Who invented Antibiotics

एटीबायह८टटककाअपवषकारककसनहॳककयािा

22-Jan-2017

Options

1) Joseph Lister

जह८सहॳफलिसटर

2) William Harvey

पवलियमहाव

3) Robert Knock

रॉबटयनॉक

4)Alexander Fleming

अिहॳकज़िरफिहॳलमग

Correct Answer Alexander Fleming

Q119 Milbecycin is used in the

eradication of

लममबहॳसायलसनका mdashndash

मउनमिनमपरयह८गककयाजाताहहॴ 22-Jan-2017

Options

1) Agricultural Fungus

कपषकवक

2) Agricultural Pests

कपषकीटक

3) Agricultural Herbs

कपषिाक

4)Agricultural Weeds

कपषननराना Correct Answer Agricultural Pests

Milbemycin oxime is a veterinary drug

from the group of milbemycins used as

a broad spectrum antiparasitic It is

active against worms and mites(insects

Q120 Intestinal bacteria synthesizes

which of the following in the human

body

मानविरीरमआतोकहॳ बहॴकटीररयानननननलिखितमसहॳककसकासशिहॳषणकरतहॳहहॴ 22-Jan-2017

Options

1) Vitamin K

पवटालमन K

2) Proteins

परह८टीन

3) Fats

वसा 4) Vitamin D

पवटालमन D

Correct Answer Vitamin K

Q121 is the study of the physical form

and external structure of plants

mdashmdash-

मपह९धह८काभहॴनतक पऔरबाहरीसरचनाकाआदयाककयाजाताहहॴ 22-Jan-2017

Options

1) Physiology

कफनजयह८िह८जी 2) Anatomy

िरीररचनापवजञान

3) Phytomorphology

फाईटह८मह८फह८िह८जी 4)Cytology

कह८लिकापवजञान

Correct Answer Phytomorphology

F A C E B O O K

P A G E h t t p w w w f a c e b o o k c o m s s c m e n t o r s o f f i c i a l P a g e | 29

FOR MORE UPDATES AND MORE MATERIAL DO LIKE OUR FACEBOOK PAGE httpwwwfacebookcomsscmentorsofficial

Q122 Which of the following is a

structural and functional unit of

kidneys

नननननलिखितमसहॳकह९नसीगदोकीसरचनातमकऔरकाययकरीईकाईहहॴ

22-Jan-2017

Options

1) Renette Cells

रहॳनहॳटकह८लिकाए

2) Flame Cells

फिहॳमकह८लिकाए

3) Nephrites

नहॳफ़राइटस

4)Nephrons

नहॳफरोस

Correct Answer Nephrons

Nephron functional unit of the kidney

the structure that actually produces

urine in the process of removing waste

and excess substances from the blood

There are about 1000000 nephrons in

each human kidney

Q123 Which of the following is the

largest part of the human brain

नननननलिखितमसहॳकह९नसामानवमनसतषककासबसहॳबड़ाटहससाहहॴ

23-Jan-2017

Options

1) Ribs

पसलियाा 2) Cerebrum

सहॳरहॳिम

3) Pons

पोस

4)Thalamus

िहॴिहॳमस

Correct Answer Cerebrum

The cerebrum is the largest part of the

human brain making up about two-

thirds of the brainrsquos mass It has two

hemispheres each of which has four

lobes frontal parietal temporal and

occipital

Q124 The auxiliary buds

सहायककालियाmdashndash 23-Jan-2017

Options

1) grow endogenously from the pericycle

पहॳरीसाईककिसहॳअनतजातयपवकलसतहह८ताहहॴ 2) arise endogenously from the main

growing point

मिवपदसहॳअनतजातयउठताहहॴ 3) is an embryonic shoot located in the

axil of a leaf

एकभरणिटहहॴजह८एकपततीकहॳ अकषपरनसतिहह८ताहहॴ 4)arise exogenously from the epidermis

एपपिलमयससहॳबटहजातयतरीकहॳ सहॳउठताहहॴ Correct Answer is an embryonic shoot

located in the axil of a leaf

Q125 Which of the following is a viral

disease

इनमहॳसहॳकह९सीएकवायरिबीमारीहहॴ

23-Jan-2017

Options

1) Polio

पह८लियह८ 2) Tetanus

धनसतनभ

3) Leprosy

कषठरह८ग

4) Plague

पिहॳग

Correct Answer Polio

A viral disease (or viral infection)

occurs when an organismrsquos body is

invaded by pathogenic viruses and

infectious virus particles (virions) attach

to and enter susceptible cells

F A C E B O O K

P A G E h t t p w w w f a c e b o o k c o m s s c m e n t o r s o f f i c i a l P a g e | 30

FOR MORE UPDATES AND MORE MATERIAL DO LIKE OUR FACEBOOK PAGE httpwwwfacebookcomsscmentorsofficial

Poliomyelitis often called polio or

infantile paralysis is an infectious

disease caused by the poliovirus

Tetanusmdash A serious bacterial infection

that causes painful muscle spasms and

can lead to death

Leprosy also known as Hansenrsquos

disease (HD) is a long-term infection by

the bacterium Mycobacterium leprae or

Mycobacterium lepromatosis

Plague is an infectious disease caused by

the bacterium Yersinia pestis

Symptoms include fever weakness and

headache

Q126 Which organisms can help to

carry out Vermicomposting

कह९नसाजीववमीकनपह८नसटगममददकरताहहॴ

23-Jan-2017

Options

1) Nitrifying Bacteria

नाईटरीफाईगबहॴकटीररया 2) Earthworms

कहॴ चऐ

3) Algae

िहॴवि

4) Fungus

कवक

Correct Answer Earthworms

Q127 Contraction of heart is also

known as

हदयकहॳ सकचनकह८ mdash- भीकहाजाताहहॴ 23-Jan-

2017

Options

1) Systole

लससटह८ि

2) Aristotle

अरसत

3) Diastole

िायसटह८ि

4) Lub

मयब

Correct Answer Systole

Diastole is the part of the cardiac cycle

when the heart refills with blood

following systole (contraction)

Ventricular diastole is the period during

which the ventricles are filling and

relaxing while atrial diastole is the

period during which the atria are

relaxing

Q128 Azadirachta indica is the

botanical name of which of the

following

अजाटदराचताइडिकानननननलिखितमसहॳककसकावानसपनतनामहहॴ

23-Jan-2017

Options

1) Rose plant

गिाबकापह९धा 2) Apple tree

सहॳबकापहॳड़

3) Neem

नीम

4)Mango

आम

Correct Answer Neem

Q129 Which of the following is the

main end product of carbohydrate

digestion

नननननलिखितमसहॳकह९नसाकाबोहाइडरहॳटकहॳ पाचनकापरमिअतउतपादकहह८ताहहॴ 23-Jan-2017

Options

1) Fats

वसा 2) Lipids

लिपपडस

3) Glucose

गिकह८ज

4) Cellulose

F A C E B O O K

P A G E h t t p w w w f a c e b o o k c o m s s c m e n t o r s o f f i c i a l P a g e | 31

FOR MORE UPDATES AND MORE MATERIAL DO LIKE OUR FACEBOOK PAGE httpwwwfacebookcomsscmentorsofficial

सहॳमयिह८ज

Correct Answer Glucose

Intestinal absorption of end products

from digestion of carbohydrates and

proteins in the pig hellip During absorption some sugars (fructose or

galactose) released from the

corresponding sucrose and lactose

respectively during digestion were

partly metabolized into glucose by the

enterocyte

Q130 Which of the following glands is a

source of the enzyme Ptyalin

नननननलिखितगरचियोमसहॳएजाइमटयालिनकासरह८तहहॴ 23-Jan-2017

Options

1) Pancreas

अगरािय

2) Thyroid Gland

िाइराइिगरिी 3) Pituitary Gland

पीयषगरिी 4) Salivary Glands

िारगरचियाा Correct Answer Salivary Glands

Q131 Which of the following is not true

about Pteridophyta

ननननमसहॳकह९नसीबातटहॳररिह८फाईटकहॳ बारहॳमसचनहीहहॴ 23-Jan-2017

Options

1) Dominant phase is saprophytes

परमिचरणसहॳपरह८फाईइटसहह८ताहहॴ 2) Main plant body is diploid

पह९दह८कामखयिरीरदपवगखणतहह८ताहहॴ 3) Seeds are present

बीजमह९जदहह८तहॳहहॴ 4)Flowers are absent

फिअनपनसतिहह८तहॳहहॴ

Correct Answer Seeds are present

Q132 The largest dolphin species is the

orca also called as

िॉिकफनकीसबसहॳबड़ीपरजानतकाकानामआकायहहॴनजसहॳ mdash- भीकहतहॳहहॴ 23-Jan-2017

Options

1) Bottle Nose

बाटिनह८ज

2) Baiji

बहॳजी 3) Killer whale

ककिरहहॳि

4)Tucuxi

टकवसी Correct Answer Killer whale

Q133 The fat digesting enzyme Lipase

is secreted by which of the following

वसाकापाचनकरनहॳवािाएजाइमिाइपहॳजनननननलिखितमसहॳककसकहॳ दवारासतरापवतहह८ताहहॴ

24-Jan-2017

Options

1) Kidneys

गद

2) Pancreas

अगनयािय

3) Large Intestine

बड़ीआत

4)Liver

नजगर

Correct Answer Pancreas

Lipase is an enzyme that splits fats so

the intestines can absorb them Lipase

hydrolyzes fats like triglycerides into

their component fatty acid and glycerol

molecules It is found in the blood

gastric juices pancreatic secretions

intestinal juices and adipose tissues

F A C E B O O K

P A G E h t t p w w w f a c e b o o k c o m s s c m e n t o r s o f f i c i a l P a g e | 32

FOR MORE UPDATES AND MORE MATERIAL DO LIKE OUR FACEBOOK PAGE httpwwwfacebookcomsscmentorsofficial

Q134 The arrangement of leaves on an

axis or stem is called

एकअकषयातनहॳपरपनततयोकीयवसिाकह८कयाकहाजाताहहॴ SSC CHSL Science (biology) 2016

Question Paper

24-Jan-2017

Options

1) Phyllotaxy

फाइिह८टहॴकसी 2) Vernation

वनिन

3) Venation

वहॳनहॳिन

4)Phytotaxy

फाइटह८टहॴकसी Correct Answer Phyllotaxy

In botany phyllotaxis or phyllotaxy is

the arrangement of leaves on a plant

stem (from Ancient Greek phyacutellon

ldquoleafrdquo and taacutexis ldquoarrangementrdquo)

Phyllotactic spirals form a distinctive

class of patterns in nature

Q135 The study of Cells is also known

as

कह८लिकाओकहॳ अधययनकह८ mdashmdashndash

भीकहाजाताहहॴ 24-Jan-2017

Options

1) Cytology

सायटह८िह८जी 2) Physiology

कफनजयह८िह८जी 3) Nucleology

नयककमयह८िह८जी 4)Cellology

सहॳिह८िह८जी Correct Answer Cytology

Q136 Which of the following scientists

is also known as the Father of Biology

नननननलिखितमसहॳककसवहॴजञाननककह८ ldquoजीवपवजञानकहॳ जनकrdquoकहॳ नामसहॳभीजानाजाताहहॴ 24-Jan-2017

Options

1) Herbert Spencer

हबयटयसपसर

2) Aristotle

अरसत 3) Lamarck

िहॳमाकय 4)Darwin

िापवयन

Correct Answer Aristotle

Q137 Which cells give rise to various

organs of the plant and keep the plant

growing

कह९नसीकह८लिकाएपह९धह८कहॳ लभननअगह८कह८जनमदहॳतीहहॴऔरपह९धह८कह८बढ़नहॳममददकरतीहहॴ

24-Jan-2017

Options

1) Permanent

सिायी 2) Dermal

तवचीय

3) Meristematic

मररसटहॳमटटक

4)Mature

परह८ढ़

Correct Answer Meristematic

A meristem is the tissue in most plants

containing undifferentiated cells

(meristematic cells) found in zones of

the plant where growth can take place

Q138 Rodentia Muridae is the scientific

name of

F A C E B O O K

P A G E h t t p w w w f a c e b o o k c o m s s c m e n t o r s o f f i c i a l P a g e | 33

FOR MORE UPDATES AND MORE MATERIAL DO LIKE OUR FACEBOOK PAGE httpwwwfacebookcomsscmentorsofficial

रह८िहॳलियानयररिी mdashmdash- कावहॴजञाननकनामहहॴ 24-

Jan-2017

Options

1) Mouse

चहा 2) Squirrel

चगिहरी 3) Monkey

बदर

4) Lizard

नछपकिी Correct Answer Mouse

Q139 Name the scientist who proposed

the cell theory

कह८लिकालसदातकापरसतावदहॳनहॳवािहॳवहॴजञाननककानामबताइए 24-Jan-2017

Options

1) Schleiden and Schwann

िीमिनऔरशरववान

2) Lamarck

िहॳमाकय 3) Treviranus

टरहॳवायरहॳनस

4)Whittaker and Stanley

हीटकरऔरसटहॳनिहॳ Correct Answer Schleiden and

Schwann

Q140 The flower with the worldrsquos

largest bloom is

दननयाकासबसहॳबड़ाफिखििनहॳवािा mdashmdashndash हहॴ 24-Jan-2017

Options

1) Pando

पािह८ 2) Posidonia

पह८सीिह८ननया 3) Rafflesia arnoldii

ररफिहॳलियाअनोमिी 4)Helianthus annuus

हहॳलिएनिसएनयअस

Correct Answer Rafflesia arnoldii

Rafflesia arnoldii is a species of

flowering plant in the parasitic genus

Rafflesia It is noted for producing the

largest individual flower on earth It has

a very strong and horrible odour of

decaying flesh earning it the nickname

ldquocorpse flower

Q141 Deficiency of which vitamin

causes night blindness

ककसपवटालमनकीकमीकहॳ कारणरतौधीहह८ताहहॴ 24-Jan-2017

Options

1) Vitamin K

पवटालमन K

2) Vitamin C

पवटालमन C

3) Vitamin B1

पवटालमन B1

4)Vitamin A

पवटालमन A

Correct Answer Vitamin A

Q142 Nongreen plants lack which of the

following

गहॴर-

हररतवनसपनतमनननननलिखितमसहॳककसकीकमीहह८तीहहॴ

24-Jan-2017

Options

1) Chlorophyll

किह८रह८कफि

2) Lycophyll

िायकह८कफि

3) Cyanophyll

F A C E B O O K

P A G E h t t p w w w f a c e b o o k c o m s s c m e n t o r s o f f i c i a l P a g e | 34

FOR MORE UPDATES AND MORE MATERIAL DO LIKE OUR FACEBOOK PAGE httpwwwfacebookcomsscmentorsofficial

सायनह८कफि

4)Phototropism

फह८टह८टरोपपजम

Correct Answer Chlorophyll

Q143 Organisms that use light to

prepare food are known as

जह८जीवपरकािकाउपयह८गकरभह८जनतहॴयारकरतहॳहहॴ उनह mdashmdash- कहॳ पमजानजाताहहॴ 24-Jan-2017

Options

1) Autotrophs

सवपह८षी 2) Heterotrophs

पवषमपह८षज

3) Omnivores

सवायहारी 4)Decomposers

पवघटनकरनहॳवािा Correct Answer Autotrophs

autotrophs often make their own food

by using sunlight carbon dioxide and

water to form sugars which they can use

for energy Some examples of

autotrophs include plants algae and

even some bacteria Autotrophs

(producer) are important because they

are a food source for heterotrophs

(consumers)

A heterotroph is an organism that

ingests or absorbs organic carbon

(rather than fix carbon from inorganic

sources such as carbon dioxide) in order

to be able to produce energy and

synthesize compounds to maintain its

life Ninety-five percent or more of all

types of living organisms are

heterotrophic including all animals and

fungi and some bacteria

Q144 Which of the following is a

primary function of haemoglobin

नननननलिखितमसहॳकह९नसाटहमह८गिह८बबनकाएकपरािलमककाययहहॴ

25-Jan-2017

Options

1) Utilization of energy

उजायकाउपयह८गकरना 2) Prevention of anaemia

रकतामपताहह८नहॳसहॳरह८कना 3) Destruction of bacteria

बहॴकटीररयाकापवनािकरना 4) To transport oxygen

ऑकसीजनकावहनकरना Correct Answer To transport oxygen

Q145 Vascular bundles are absent in

सवहनीबिि mdashmdash- मअनपनसतिरहतहॳहहॴ 25-Jan-2017

Options

1) Bryophyta

िायह८फाइटा 2) Pteridophyta

टहॳररिह८फाईटा 3) Gymnosperms

नजननह८सपमय 4) Angiosperms

एननजयह८सपहॳनसय Correct Answer Bryophyta

Q146 Sauria Lacertidae is the scientific

name of

सहॴररयािहॳसरटाईिी mdashmdashndash कावहॴजञाननकनामहहॴ 25-Jan-2017

Options

1) Crocodile

मगरमचछ

2) Hippopotamus

टहपपह८पह८टहॳमस

3) Lizard

नछपकिी 4) House fly

F A C E B O O K

P A G E h t t p w w w f a c e b o o k c o m s s c m e n t o r s o f f i c i a l P a g e | 35

FOR MORE UPDATES AND MORE MATERIAL DO LIKE OUR FACEBOOK PAGE httpwwwfacebookcomsscmentorsofficial

घरहॳिमकिी Correct Answer Lizard

Q147 Which type of pathogen causes

the water-borne disease SARS (Severe

Acute Respiratory Syndrome)

ककसपरकािकारह८गज़नकजिजननतबीमारीसासयकाकारणबनताहहॴ 25-Jan-2017

Options

1) Viral

वायरि

2) Parasitic

परजीवी 3) Protozoan

परह८टह८जअन

4) Bacterial

बहॴकटीररयि

Correct Answer Viral

Q148 Which of the following organs

produces the enzyme lipase

नननननलिखितमसहॳकह९नसाअगिायपहॳजएजाइमउतपननकरताहहॴ 25-Jan-2017

Options

1) Pancreas

अगनयािय

2) Large Intestine

बड़ीआत

3) Liver

नजगर

4) Small Intestine

छह८टीआत

Correct Answer Pancreas

Q149 A is a long internode forming the

basal part or the whole of a peduncle

एक mdashmdash- एकिबाइटरनह८िहहॴ जह८ननचिाटहससायासनपणयिठिबनताहहॴ 25-

Jan-2017

Options

1) Rhizome

परकद

2) Rachis

महॳ दि

3) floral axis

पषपअकष

4) Scape

भगदड़

Correct Answer scape

Q150 ndash Which of the following

organisms are considered to be both

Living and Non-living

नननननलिखितमसहॳकह९नसहॳजीवाणकह८जीपवतऔरअजीपवतमानाजाताहहॴ

25-Jan-2017

Options

1) Bacteria

बहॴकटीररया 2) Fungi

कवक

3) Algae

िहॴवाि

4)Virus

वायरस

Correct Answer Virus

They are considered to be living as they

possess a protein coat as a protective

covering DNA as the genetic material

etc

They are said to be non-living as they

can be crystallised and they survive for

billions of years They can tolerate high

temperatures freezing cold

temperatures ultra-violet radiations etc

Q151 Deficiency of fluorine causes

which of the following

फिह८ररनकीकमीकहॳ कारणनननननलिखितमसहॳकयाहह८ताहहॴ

F A C E B O O K

P A G E h t t p w w w f a c e b o o k c o m s s c m e n t o r s o f f i c i a l P a g e | 36

FOR MORE UPDATES AND MORE MATERIAL DO LIKE OUR FACEBOOK PAGE httpwwwfacebookcomsscmentorsofficial

27-Jan-2017

Options

1) Dental Caries

िटिकहॴ ररज

2) Scurvy

सकवरी 3) Anaemia

रकतामपता 4) Arthritis

गटठया Correct Answer Dental Caries

Q152 In a Punnett Square with the

cross AaBb x AaBb how many Aabb

genotypes would be created

पनहॳटसककायरमिह८स AaBb x AaBb कहॳ साि

ककतनहॳ Aabb जीनह८टाइपबनगहॳ 27-Jan-2017

Options

1) 1

2) 8

3) 2

4) 3

Correct Answer 2

Q153 Which of the following is the

Controlling Center of the Cell

नननननलिखित म सहॳ कह८लिकाका ननयतरण

क दर कह९न हहॴ

27-Jan-2017

Options

1) Nucleus

क दर

2) Plasma

पिाजमा 3) Lysosome

िायसह८सह८म

4) Chromosome

िह८मह८सह८म

Correct Answer Nucleus

The control centre of the cell is the

nucleus in eukaryotic cells The nucleus

contains genetic material in the form of

DNA

Q154 Myopia affects which of the

following organs

मायह८पपयानननननलिखितअगह८मसहॳककसहॳपरभापवतकरताहहॴ

25-Jan-2017

Options

1) Heart

हदय

2) Skin

तवचा 3) Eyes

आािहॳ 4)Mouth

मह

Correct Answer Eyes

Q155 Which of the following bears

flowers

नननननलिखितमसहॳकह९नफिधारणकरताहहॴ

25-Jan-2017

Options

1) Bryophyta

िायह८फाइटा 2) Pteridophyta

टहॳरीिह८फाईटा 3) Gymnosperms

नजननह८सपमय 4)Angiosperms

एननजयह८सपमय Correct Answer Angiosperms

Q156 Oxygenated blood flows out of the

heart through the

ऑकसीजनयकतरकत mdashmdashmdash

कहॳ माधयमसहॳहदयकहॳ बाहरबहताहहॴ 25-Jan-2017

F A C E B O O K

P A G E h t t p w w w f a c e b o o k c o m s s c m e n t o r s o f f i c i a l P a g e | 37

FOR MORE UPDATES AND MORE MATERIAL DO LIKE OUR FACEBOOK PAGE httpwwwfacebookcomsscmentorsofficial

Options

1) Aorta

महाधमनी 2) pulmonary artery

फहॳ फड़हॳकीधमनी 3) vena cava

वहॳनाकावा 4)Atrium

चह९क

Correct Answer aorta

Q157 Blood leaving the liver and

moving towards the

heart has a higher concentration of

नजगरसहॳननकिकरहदयकीतरफजानहॳवािहॳरकतम mdashmdashmdashmdash कीउचचसादरताहह८तीहहॴ 27-Jan-2017

Options

1) Lipids

लिपपडस

2) Urea

यररया 3) Bile Pigments

पपततकहॳ रगकरण

4) Carbon dioxide

काबयनिायऑकसाइि

Correct Answer Bile Pigments

Urea is nitrogen containing substance

which is produced in the liver in order

to deal with excess amino-acids in the

body As urea is produced it leaves the

liver in the blood stream and passes via

the circulatory system to all parts of the

body

Q158 Bulb is a modification of which

part of a plant

बमबएकपह९धहॳकहॳ ककसटहससहॳकाएक पातरणहह८ताहहॴ 27-Jan-2017

Options

1) The root

जड़

2) The stem

तना 3) The radicle

मिाकर

4)The fruit

फि

Correct Answer The stem

Q159 Which of the following carries

blood away from the heart to different

body parts

इनमहॳसहॳकह९नरकतकह८हदयसहॳिरीरकहॳ पवलभननअगह८तकिहॳजातीहहॴ

27-Jan-2017

Options

1) Arteries

धमननया 2) Nerves

तबतरहाए

3) Capillaries

कहॳ लिकाए

4)Veins

नसहॳ Correct Answer Arteries

Q160 The series of processes by which

nitrogen and its compounds are

interconverted in the environment and

in living organisms is called

27-Jan-2017

Options

1)Absorption of Nitrogen

2)Ammonification

3)Nitrogen Fixation

4)Nitrogen Cycle

Correct Answer Nitrogen Cycle

Ammonification or Mineralization is

performed by bacteria to convert

organic nitrogen to ammonia

F A C E B O O K

P A G E h t t p w w w f a c e b o o k c o m s s c m e n t o r s o f f i c i a l P a g e | 38

FOR MORE UPDATES AND MORE MATERIAL DO LIKE OUR FACEBOOK PAGE httpwwwfacebookcomsscmentorsofficial

Nitrification can then occur to convert

the ammonium to nitrite and nitrate

Nitrogen fixation is a process by which

nitrogen in the Earthrsquos atmosphere is

converted into ammonia (NH3) or other

molecules available to living organisms

Q161 BCG vaccine is given to protect

from which of the following

बीसीजीकाटटकानननननलिखितमसहॳककसकहॳ बचावकहॳ लिएटदयाजातहहॴ

27-Jan-2017

Options

1) Jaundice

पीलिया 2) Anaemia

रकतमपता 3) Tuberculosis

कषयरह८ग

4) Polio

पह८लियह८ Correct Answer Tuberculosis

Q162 Parallel venation is found in

समानतरवहॳनहॳिन mdashmdashmdash- मपायाजाताहहॴ 27-Jan-2017

Options

1) plants which are monocots

पह९धहॳजह८एकबीजपतरीहह८तहॳहहॴ 2) plants which have a dicot stem

वहॳपह९धहॳनजनकातनादपवदलियहह८ताहहॴ 3) plants with leaves similar to Tulsi

वहॳपह९धहॳनजनकीपनततयतिसीकीपनततयोकहॳ समानहह८तहॳहहॴ 4)plants with tap roots

टहॳप टवािहॳपह९धहॳ Correct Answer plants which are

monocots

Q163 The hardest part of the body is

िरीरकासबसहॳकठह८रभाग mdashndash हहॴ 27-Jan-2017

Options

1) Bones

हडडिय

2) Tooth Enamel

दातकहॳ इनहॳमि

3) Skull

िह८पड़ी 4) Spinal Cord

महॳ रजज

Correct Answer Tooth Enamel

Q164 Which type of pathogen causes

the waterborne disease E coli Infection

ककसपरकारकारह८गजननकजिजननतरह८गईकह८िाईसिमणकाकारणबनताहहॴ 27-Jan-2017

Options

1) Protozoan

परह८टह८जआ

2) Parasitic

परजीवी 3) Bacterial

बहॴकटीररयि

4)Viral

वायरि

Correct Answer Bacterial

Q165 The amount of blood filtered

together by both the kidneys in a 70 kg

adult male human in a minute is

70 की गरा वािहॳएकवयसकप षमएकलमनटमदह८नोगदकहॳदवाराएकसािचाबनीगयीरकतकीमातरहह८तीहहॴ 29-Jan-2017

Options

1) 1100 ml

1100 लमलि

2) 100 ml

F A C E B O O K

P A G E h t t p w w w f a c e b o o k c o m s s c m e n t o r s o f f i c i a l P a g e | 39

FOR MORE UPDATES AND MORE MATERIAL DO LIKE OUR FACEBOOK PAGE httpwwwfacebookcomsscmentorsofficial

100 लमलि

3) 1500 ml

1500 लमलि

4) 500 ml

500 लमलि

Correct Answer 1100 ml

Q166 Which feature of a plant helps to

distinguish a monocot from a dicot

पह९धहॳकीवहकह९नसीपविहॳषताहहॴजह८एकदपवदलियहॳऔरएकएकदिीयपह९धहॳसहॳभहॳदकरनहॳममददकरतीहहॴ 29-Jan-2017

Options

1) Pollination

परागम

2) Venation

वहॳनहॳिन

3) Vernation

वनिन

4) Aestivation

एसटीवहॳिहॳन

Correct Answer venation

Q167 The Mutation Theory was

proposed by

उतवररवतयनकालसदात mdashmdashndash

कहॳ दवरापरसतापवतककयाजाताहहॴ 29-Jan-2017

Options

1) Charles Lyell

चामसयलियहॳि

2) William Smith

पवलियमनसमि

3) Hugo De Vries

हयगह८िीराईस

4)Harrison Schmitt

हहॳरीसननसमट

Correct Answer Hugo De Vries

Q168 Which type of pathogen causes

the waterborne disease HepatitisA

ककसपरकारकहॳ रह८गजनकजिजननतरह८गहहॳपहॳटाइटटस-A काकारणबनताहहॴ

29-Jan-2017

Options

1) Parasitic

परजीवी 2) Viral

वायरि

3) Protozoan

परह८टह८जआ

4) Bacterial

बहॴकटीररयि

Correct Answer Viral

Q169 In a Punnett Square with the

cross AaBb x Aabb how many AaBb

genotypes would be created

पनहॳटसकवायरमिह८स AaBb x Aabb

कहॳ सािककतनहॳ AaBb जीनह८टाइपबनगहॳ 29-Jan-

2017

Options

1) 4

2) 1

3) 7

4) 6

Correct Answer 4

Q170 Arboreal Ateles is the scientific

name of

अिह८ररयिएटटलिस mdashmdashmdash कावहॴजञाननकनामहहॴ 29-Jan-2017

Options

1) Squirrel

चगिहरी 2) Sparrow

गह८रहॴया 3) Lizard

नछपकिी 4) Spider monkey

F A C E B O O K

P A G E h t t p w w w f a c e b o o k c o m s s c m e n t o r s o f f i c i a l P a g e | 40

FOR MORE UPDATES AND MORE MATERIAL DO LIKE OUR FACEBOOK PAGE httpwwwfacebookcomsscmentorsofficial

मकड़ीबदर

Correct Answer Spider monkey

Q171 Which type of pathogen causes

the waterborne disease Salmonellosis

ककसपरकारकारह८गाणजिजननतबीमारीसािमह८नहॳिह८लसज़काकारकहहॴ

29-Jan-2017

Options

1) Algal

िहॳवालियहॳ 2) Parasitic

परजीवी 3) Bacterial

बहॴकटीररयि

4)Viral

वायरि

Correct Answer Bacterial

An infection with salmonella bacteria

commonly caused by contaminated food

or water

Symptoms include diarrhoea fever

chills and abdominal pain

Q172 is a condition in which there is a

deficiency of red cells or of haemoglobin

in the blood

mdashmdash-

एकनसिनतहहॴनजसमहॳरकतमिािकह८लिकाओकीयाहीमह८गिह८बबनकीकमीहह८तीहहॴ 29-Jan-2017

Options

1) Albinism

एनमबननजम

2) Propyria

परह८पीररया 3) Anaemia

एनीलमया 4)Keloid disorder

कहॳ िह८इिडिसओिर

Correct Answer Anaemia

Q173 Ananas comosus is the scientific

name of

Options

अनानासकह८मह८सस mdashmdashmdashndash

कावहॴजञाननकनामहहॴ 29-Jan-2017

1) Custard Apple

सीताफि

2) Pineapple

पाइनएपपि

3) Bamboo

बास

4)Pomegranate

अनार

Correct Answer Pineapple

Q174 Which organ produces insulin

कह९नसाअगइनसलिनपहॴदाकरताहहॴ 29-Jan-

2017

Options

1) Liver

यकत

2) Thyroid gland

िायराइिगरिी 3) Spleen

पिीहा 4)Pancreas

अगरयिय

Correct Answer Pancreas

Q175 Which of the following disease is

not caused by water pollution

नननननलिखितमसहॳकह९नसारह८गपानीकहॳ परदषणकहॳकारणनहीहह८ता

29-Jan-2017

Options

1) Cholera

हहॴजा 2) Typhoid

F A C E B O O K

P A G E h t t p w w w f a c e b o o k c o m s s c m e n t o r s o f f i c i a l P a g e | 41

FOR MORE UPDATES AND MORE MATERIAL DO LIKE OUR FACEBOOK PAGE httpwwwfacebookcomsscmentorsofficial

टाइफाइि

3) Asthma

दमा 4)Diarrhoea

दसत

Correct Answer Asthma

Q176 Ocimum tenuiflorum is the

scientific name of

ओलिलममटहॳयईफिह८रमइसकावहॴजञाननकनाम mdash

ndash हहॴ 30-Jan-2017

Options

1) Neem

नीम

2) Mango

आम

3) Babul

बबि

4)Tulsi

तिसी Correct Answer Tulsi

Q177 Which gland secretes bile a

digestive fluid

कह९नसीगरिीपपतत एकपाचनतरिपरदािय सरापवतकरतीहहॴ 30-Jan-2017

Options

1) Pancreas

अगनयािय

2) Liver

यकत

3) Thyroid

िायराइि

4) Testes

टहॳनसटस

Correct Answer liver

Q178 In which of the following the

dominant phase is Gametophyte

नननननलिखितमसहॳककसकहॳ परमिचरणयगमकह८दपवधद (Gametophyte)हहॴ 30-Jan-2017

Options

1) Bryophyta

िायह८फाइटा 2) Pteridophyta

टहॳररिह८फाइटा 3) Gymnosperms

नजननह८सपमय 4) Angiosperms

एननजयह८सपमय Correct Answer Bryophyta

Q179 Anaerobic respiration refers to

which of the following

नननननलिखितमसहॳककसहॳअवायवीयशवसनकहाजाताहहॴ

30-Jan-2017

Options

1) Respiration without Oxygen

ऑकसीजनकहॳ बबनाशवसन

2) Respiration with Oxygen

ऑकसीजनकहॳ सािशवसन

3) Respiration without CO2

काबयनिायऑकसाइिकहॳ बबनाशवसन

4) Respiration with CO2

काबयनिायऑकसाइिकहॳ सािशविन

Correct Answer Respiration without

Oxygen

Q180 Which type of pathogen causes

the waterborne disease Cholera

ककसपरकारकारह८गजनकजिजननतरह८गहहॴजाकाकारणबनताहहॴ

30-Jan-2017

Options

1) Algal

िहॴवालियहॳ

F A C E B O O K

P A G E h t t p w w w f a c e b o o k c o m s s c m e n t o r s o f f i c i a l P a g e | 42

FOR MORE UPDATES AND MORE MATERIAL DO LIKE OUR FACEBOOK PAGE httpwwwfacebookcomsscmentorsofficial

2) Bacterial

बहॴकटीररयि

3) Protozoan

परह८टह८जआ

4) Viral

वायरि

Correct Answer Bacterial

Q181 To which class does

Oxyreductases transferases hydrolases

belong

ओकसीररिकटहॳसटरासफरहॳजहॳस

हाइडरह८िहॳसहॳसककसवगयमआतहॳहहॴ 30-Jan-2017

Options

1) Hormones

हारमोस

2) Enzymes

एजाइनस

3) Proteins

परह८टीनस

4) Vitamins

पवटालमनस

Correct Answer Enzymes

Q182 Which of the following is not true

about Gymnosperms

ननननमसहॳकह९नसीबातअनावतबीजीकहॳ बारहॳमसचनहीहहॴ 30-Jan-2017

Options

1) Dominant phase is saprophytes

परमिचरणसहॳपरह८फाइटसहह८ताहहॴ 2) Vascular bundles are absent

सवहनीबििअनपनसितहह८ताहहॴ 3) spores are heterospores

बीजाणहहॳटहॳरह८सपह८रसहह८तहॳहहॴ 4) Flowers are absent

फिअनपनसितहह८तहॳहहॴ

Correct Answer Vascular bundles are

absent

Q183 The name of first mammal clone sheep is

भहॳड़कीपरिमसतनपायीपरनत प (किह८न)

कानामहहॴ 30-Jan-2017

Options

1) Noori

नरी 2) Dolly

िॉिी 3) Louise

िसी 4)Durga

दगाय Correct Answer Dolly

Q184 Which type of pathogen causes

the water-borne disease Typhoid fever

ककसपरकारकारह८गजनकजिजननतरह८गटाइफाइिबिारकाकारणबनताहहॴ 30-Jan-2017

Options

1) Algal

िहॴवािीय

2) Parasitic

परजीवी 3) Protozoan

परह८टह८जनअन

4)Bacterial

बहॴकटीररयि

Correct Answer Bacterial

Q185 In which part of the cell are

proteins made

कह८लिकाकहॳ ककसटहससहॳमपरह८टीनबनायाजाताहहॴ

31-Jan-2017

Options

1) Reticulum

रहॳटटकिम

F A C E B O O K

P A G E h t t p w w w f a c e b o o k c o m s s c m e n t o r s o f f i c i a l P a g e | 43

FOR MORE UPDATES AND MORE MATERIAL DO LIKE OUR FACEBOOK PAGE httpwwwfacebookcomsscmentorsofficial

2) Golgi apparatus

गह८मजीएपहॳरहॳटस

3) Ribosomes

ररबह८सह८नस

4) Lysosome

िायसह८सह८नस

Correct Answer ribosomes

Proteins are produced by stringing

amino acids together in the order

specified by messenger RNA strands

that were transcribed from DNA in the

cell nucleus The process of synthesizing

a protein is called translation and it

occurs on ribosomes in the cytoplasm of

a cell

Q186 Polio is a disease caused by which

of the following

नननननलिखितमसहॳपह८लियह८कीबबमारह८हह८नहॳकाकारणकयाहहॴ

31-Jan-2017

Options

1) Bacteria

बहॴकटीररयि

2) Mosquito

मचछर

3) Virus

वायरस

4) Cockroach

नतिच हॳ Correct Answer Virus

Polio or poliomyelitis is a crippling and

potentially deadly infectious disease It

is caused by the poliovirus

Q187 ndash Hay fever is a sign of which of

the following

हहॳकफवरनननननलिखितमसहॳककसकाएकसकहॳ तहहॴ

31-Jan-2017

Options

1) Old Age

वदावसिा 2) Malnutrition

कपह८सण

3) Allergy

एिनजय 4) Over Work

अतयचधककाययकरना Correct Answer Allergy

Q188 How many chromosomes does a

human cell contain

एकमानवकह८लिकामककतनहॳगणसतरहह८तहॳहहॴ

29-Jan-2017

Options

1) 6

2) 26

3) 46

4) 66

Correct Answer 46

In humans each cell normally contains

23 pairs of chromosomes for a total of

46 Twenty-two of these pairs called

autosomes look the same in both males

and females The 23rd pair the sex

chromosomes differ between males and

females

Q189 Which of the following is not true

about Bryophyta

ननननमसहॳकह९नसीबातिायह८फाइटकहॳ बारहॳमसचनहीहहॴ 31-Jan-2017

Options

1) Dominant phase is gametophytes

परमिचरणगहॳलमतह८फाइटसहह८ताहहॴ 2) Main plant body is haploid

पह९धहॳकामखयिरीरअगखणतहह८ताहहॴ 3) Spores are homospores

बीजाणहह८मह८सफह८रसहह८तहॳहहॴ 4) Flowers are present

फिमह८जदहह८तहॳहहॴ Correct Answer Flowers are present

F A C E B O O K

P A G E h t t p w w w f a c e b o o k c o m s s c m e n t o r s o f f i c i a l P a g e | 44

FOR MORE UPDATES AND MORE MATERIAL DO LIKE OUR FACEBOOK PAGE httpwwwfacebookcomsscmentorsofficial

Q190 Which aquatic animal has

trailing tentacles

ककसजिीयजानवरकहॳ पीछहॳचिनहॳवािहॳटहॳटकिसहह८तहॳहहॴ

31-Jan-2017

Options

1) Sea horse

समदरीघह८िा 2) Corals

मगा 3) Jelly fish

जहॳिीमछिी 4) Star fish

तारामछिी Correct Answer Jelly fish

Jellyfish with its umbrella-shaped bell

and trailing tentacles

Q191 Which type of pathogen causes

the water-borne disease Poliomyelitis

(Polio)

ककसपरकारकारह८गजनकजिजननतरह८गपह८लियह८मायहॳटटस (पह८लियह८) काकारणहहॴ 31-Jan-

2017

Options

1) Parasitic

परजीवी 2) Algal

िहॴवालिय

3) Viral

वायरि

4) Bacterial

बहॴकटीररयि

Correct Answer Viral

Q192 The outer white part of the eye

that protects the inner structures is

आािकाबाहरीसफहॳ दटहससाजह८आतररकसरचनाओकीरकषाकरताहहॴ वह mdashmdashmdash हहॴ 31-Jan-

2017

Options

1) Iris

आयररस

2) Sclera

सकिहॳरा 3) Retina

रहॳटटना 4) Cornea

कह८ननयया Correct Answer Sclera

Q193 Proteins are made up of

परह८टीनकाननमायण mdashndash सहॳहह८ताहहॴ 31-Jan-2017

Options

1) Amino acids

एलमनह८अनि

2) Fatty acids

वसायकतअनि

3) Glucose

गिकह८ज

4)Nucleotides

नयनकियह८टाईिस

Correct Answer Amino acids

Q194 Moringa Oleifera is the scientific

name of

मह८ररगओलिफहॳ रा mdashmdashndash कावहॴजञाननकनामहहॴ 31-Jan-2017

Options

1) Banyan

बरगद

2) Gulmohar

गिमह८हर

3) Amla

आमिा

F A C E B O O K

P A G E h t t p w w w f a c e b o o k c o m s s c m e n t o r s o f f i c i a l P a g e | 45

FOR MORE UPDATES AND MORE MATERIAL DO LIKE OUR FACEBOOK PAGE httpwwwfacebookcomsscmentorsofficial

4) Drumstick

डरमनसटक

Correct Answer Drumstick

Q195 Kidney stones are composed of

गदकीपिरी mdashndash सहॳबनीहह८तीहहॴ 1-Feb-2017

Options

1) Calcium Oxalate

कहॴ नमसयमओकजहॳिहॳट

2) Sodium Chloride

सह८डियमकिह८राइि

3) Magnesium Nitrate

महॳनगनलियमनाइतटरहॳट

4) Calcium Bicarbonate

कहॴ नमियमबायकबोनहॳट

Correct Answer Calcium Oxalate

Q196 ndash Which of the following is not

true about Angiosperms

ननननमसहॳकह९नसीबातआवतबीजीकहॳ बारहॳमसचनहीहहॴ 1-Feb-2017

Options

1) Dominant phase is gametophytes

परमिचरणगहॳलमतह८फाइटहह८ताहहॴ 2) Vascular bundles are present

सवहनीबििमह९जदहह८ताहहॴ 3) Spores are heterospores

बीजाणहहॳटहॳरह८सपह८रसहह८तहॳहहॴ 4) Seeds are covered

बीजढकहॳ हह८तहॳहहॴ Correct Answer Dominant phase is

gametophytes

Q197 All of the following are excretory

(waste) products of animals except

नननननलिखितमसहॳककसएककह८छह८ड़करअनयसभीपराखणयोदवाराउतसनजयतपदाियहहॴ 1-Feb-

2017

Options

1) Uric Acid

यररकएलसि

2) Ammonia

अमह८ननया 3) Carbohydrates

काबोहाइडरहॳट

4) Urea

यररया Correct Answer Carbohydrates

In animals the main excretory products

are carbon dioxide ammonia (in

ammoniotelics) urea (in ureotelics) uric

acid (in uricotelics) guanine (in

Arachnida) and creatine

Q198 RNA is a polymeric molecule

What does RNA stand for

आरएनइएएकबहिकआणहहॴ इसकाकापवय पकयाहहॴ 1-Feb-2017

Options

1) Rado Nuclear Acid

रािह८नयनकियरएलसि

2) Ribo Nucleic Acid

राइबह८नयनकिकएलसि

3) Rhino Nuclear Acid

हाइनह८नयनकियरएलसि

4) Resto Nucleus Acid

रहॳसटह८नयकिीयसएलसि

Correct Answer Ribo Nucleic Acid

Q199 Which organ does detoxification

and produces chemicals needed for

digestion

कह९नसाअगपवषहरणकरताहहॴऔरपाचनकहॳ लिएआवशयकरसायनोकह८पहॴदाकरताहहॴ 1-Feb-

2017

Options

1) Salivary glands

िारगरचिया 2) Pancreas

अगनयािय

F A C E B O O K

P A G E h t t p w w w f a c e b o o k c o m s s c m e n t o r s o f f i c i a l P a g e | 46

FOR MORE UPDATES AND MORE MATERIAL DO LIKE OUR FACEBOOK PAGE httpwwwfacebookcomsscmentorsofficial

3) Thyroid gland

िायराइिगरिी 4) Liver

यकत

Correct Answer Liver

Q200 Psidium guajava is the scientific

name of

लसडियमगआजावा mdashmdash कावहॴजञाननकनामहहॴ 1-

Feb-2017

Options

1) Guava

अम द

2) Mango

आम

3) Bamboo

बास

4) Jack fruit

कटहि

Correct Answer Guava

Q201 Which drug is used as a Blood

Thinner

चधरकह८पतिाकरनहॳकहॳ पमककसदवाकापरयह८गककयाजाताहहॴ

1-Feb-2017

Options

1) Warfarin

वाफर न

2) Tramadol

टरहॳमािह८ि

3) Azithromycin

एनजरह८मायलसन

4) Hydralazine

हाइडरह८िहॳनजन

Correct Answer Warfarin

Q202 Which of the following disease is

caused due to the deficiency of protein

परह८टीनकीकमीकहॳ कारणनननननलिखितमसहॳकह९नसारह८गहह८ताहहॴ 1-Feb-2017

Options

1) Arthritis

गटठया 2) Kwashiorkor

कािीओकय र

3) Goitre

गाइटर

4) Night Blindness

रतह९चध

Correct Answer Kwashiorkor

Q203 A is species of plant that has

adapted to survive in an environment

with little liquid water

mdashmdashndashपह९धहॳकीएकऐसहॳऐसहॳपरजानतहहॴ नजसनहॳकमपानीवािहॳवातावरणमजीपवतरहनहॳकहॳलिएअनकिनहहॴ 1-Feb-2017

Options

1) Xerophyte

म दपवद

2) Hydrophyte

जिीयपादप

3) Mesophyte

समह८दपवद

4) Thallophyte

िहॴिह८फाइटा Correct Answer xerophyte

xerophyte is a species of plant that has

adapted to survive in an environment

with little liquid water such as a desert

or an ice- or snow-covered region in the

Alps or the Arctic

Mesophytes are terrestrial plants which

are adapted to neither a particularly

dry nor particularly wet environment

An example of a mesophytic habitat

would be a rural temperate meadow

F A C E B O O K

P A G E h t t p w w w f a c e b o o k c o m s s c m e n t o r s o f f i c i a l P a g e | 47

FOR MORE UPDATES AND MORE MATERIAL DO LIKE OUR FACEBOOK PAGE httpwwwfacebookcomsscmentorsofficial

which might contain goldenrod clover

oxeye daisy and Rosa multiflora

thallophyte any of a group of plants or

plantlike organisms (such as algae and

fungi) that lack differentiated stems

leaves and roots and that were formerly

classified as a primary division

(Thallophyta) of the plant kingdom

Q204 How many types of teeth are

there in humans

मनषयोमककतनहॳपरकारकहॳ दातहह८तहॳहहॴ

1-Feb-2017

Options

1) 4

2) 5

3) 2

4) 3

Correct Answer 4

teeth -Humans have four types of

teethincisors canines premolars and

molars each with a specific function

The incisors cut the food the canines

tear the food and the molars and

premolars crush the food

Q205 Carica papaya is the scientific name of

कहॴ ररकापपाया mdashmdashndash कावहॴजञाननकनामहहॴ 2-

Feb-2017

Options

1) Peepal

पीपि

2) Papaya

पपीता 3) Tamarind

इमिी 4) Drumstick

ढह८िकाछड़ी Correct Answer Papaya

Q206 Muscles get tired when there is

shortfall of

जब mdashndash कीकमीहह८तीहहॴतबपहॳिीयिकजातीहहॴ 2-Feb-2017

Options

1) Lactic acid

िहॴनकटकएलसि

2) Na+ ions

Na+ आयन

3) ATP

एटीपी 4) Sulphates

समफहॳ टस

Correct Answer ATP

ATP is the energy source muscle fibers

use to make muscles contract

muscle tissuersquos main source of energy

called adenosine triphosphate or ATP

As your muscles use up this energy

source they become tired and fatigued

Oxygen is the key ingredient that helps

create new ATP to replenish the burned

up ATP in your muscles

Q207 Artocarpus integra is the

scientific name of आटह८कापयसइटीगरा mdashmdashmdash कावहॴजञाननकनामहहॴ 2-Feb-2017

Options

1) Guava

अम द

2) Pineapple

अनानास

3) Silver Oak

लसमवरओक

4) Jack fruit

कटहि

Correct Answer Jack fruit

Q208 Which organ stores fat soluble

vitamins

कह९नसाअगवसामघिनिीिपवटालमनह८काभिाराकरताहहॴ

2-Feb-2017

F A C E B O O K

P A G E h t t p w w w f a c e b o o k c o m s s c m e n t o r s o f f i c i a l P a g e | 48

FOR MORE UPDATES AND MORE MATERIAL DO LIKE OUR FACEBOOK PAGE httpwwwfacebookcomsscmentorsofficial

Options

1) Blood

रकत

2) Skin

तवचा 3) Liver

यकत

4) Pancreas

अगनयािय

Correct Answer Liver

Q209 Which disease is caused due to

deficiency of Iodine

आयह८िीनकहॳ कारणकह९नसारह८गहह८ताहहॴ 2-Feb-2017

Options

1) Rickets

ररकहॳ टस

2) Scurvy

सकवी 3) Goitre

गणमािा 4) Growth retardation

पवकासका कना Correct Answer Goitre

rickets A softening and weakening of

bones in children usually due to

inadequate vitamin D

Q210 Grevillea Robusta is the scientific name of

गरहॳपवलियारह८बसटा mdashmdashmdash- कापवजञाननकनामहहॴ 2-Feb-2017

Options

1) Peepal

पीपि

2) Teak

सागह९न

3) Silver Oak

लसमवरओक

4) Jack fruit

कटहि

Correct Answer Silver Oak

Q211 When a Cuttlefish is described as a Molluscs it is at which level of

classification

जबएककटिकफिकह८एकमह८िसकाकहॳ पमवखणयतककयाजाताहहॴतबयहॳवगीकरणकहॳ ककससतरपहॳनसितहहॴ 2-Feb-2017

Options

1) Class

वगय 2) Order

िम

3) Family

पररवार

4) Phylum

सघ

Correct Answer Phylum

Q212 Bambusa dendrocalmus is the

scientific name of बानबसािहॳडराकामस mdashmdashmdash कावहॴजञाननकनामहहॴ 3-Feb-2017

Options

1) Banyan

बरगद

2) Papaya

पपीता 3) Bamboo

बास

4) Pomegranate

अनार

Correct Answer Bamboo

Q213 Acinonyx Jubatus is the scientific name of

एलसनह८ननकसजयबहॳटस mdashmdashmdash

कावहॴजञाननकनामहहॴ 3-Feb-2017

F A C E B O O K

P A G E h t t p w w w f a c e b o o k c o m s s c m e n t o r s o f f i c i a l P a g e | 49

FOR MORE UPDATES AND MORE MATERIAL DO LIKE OUR FACEBOOK PAGE httpwwwfacebookcomsscmentorsofficial

Options

1) Bear

भाि 2) Horse

घह८िा 3) Cheetah

चीता 4) Zebra

जहॳिा Correct Answer Cheetah

Q214 The pale yellow colour of urine is

due to the presence of which pigment

मतरकाफीकापीिारगरगदरयकहॳ उपनसिनतकहॳ कारणहह८ताहहॴ

3-Feb-2017

Options

1) Urochrome

यरह८िह८म

2) Urophyll

यरह८कफि

3) Chlorophyll

किह८रह८कफि

4) Chloroplast

किह८रह८पिासट

Correct Answer Urochrome

Q215 Which of the following constitute

to form a gene

नननननलिखितमसहॳकह९नसीचीज़एकजीनकागठनकरतीहहॴ

3-Feb-2017

Options

1) Polynucleotides

पह८िीनयनकियह८टाईडस

2) Hydrocarbons

हाइडरह८काबोस

3) Lipoproteins

िाईपह८परह८टीनस

4) Lipids

लिपपडस

Correct Answer Polynucleotides

Polynucleotide molecule is a biopolymer

composed of 13 or more nucleotide

monomers covalently bonded in a chain

DNA (deoxyribonucleic acid) and RNA

(ribonucleic acid) are examples of

polynucleotides with distinct biological

function

Q216 Vertebrates belongs to the

phylum

रीढ़कीहडिीवािहॳपराणी mdashmdashmdash

परजानतकहॳ अतगायतआतहॳहहॴ 3-Feb-2017

Options

1) Arthropoda

आरह८पह८ड़ा 2) Annelida

एननलििा 3) Cnidaria

ननिहॳररया 4) Chordata

कह८िटा Correct Answer Chordata

Q217 Punica granatum is the scientific name of

पननकगरहॳनहॳटस mdashmdashmdash कावहॴजञाननकनामहहॴ 3-Feb-2017

Options

1) Custard Apple

सीताफि

2) Gulmohar

गिमह८हर

3) Silver Oak

लसमवरओक

4) Pomegranate

अनार

Correct Answer Pomegranate

F A C E B O O K

P A G E h t t p w w w f a c e b o o k c o m s s c m e n t o r s o f f i c i a l P a g e | 50

FOR MORE UPDATES AND MORE MATERIAL DO LIKE OUR FACEBOOK PAGE httpwwwfacebookcomsscmentorsofficial

Q218 Between a tiger and an monkey

which of the following is different

एकबाघऔरबदरकहॳ बीचनननननलिखितमसहॳकह९नसीबातअिगहहॴ 3-Feb-2017

Options

1) Kingdom

राजय

2) Phylum

जानत

3) Order

िम

4) Class

वगय Correct Answer order

Q219 The artificial heart was invented by

कबतरमहदयका mdashmdashmdash

दवाराअपवषकारककयागयािा 3-Feb-2017

Options

1) Muhammad Yunus

महनमदयनस

2) Linus Yale Jr

िाइनसयहॳिजय

3) Gazi Yasargil

गाजीयासचगयि

4) Paul Winchell

पह९िपवमकि Correct Answer Paul Winchell

Q220 Tamarindus indica is the

scientific name of

टहॳमररनडसइडिका mdashmdash कावहॴजञाननकनामहहॴ 7-

Feb-2017

Options

1) Neem

नीम

2) Pineapple

अनानास

3) Tamarind

इमिी 4)Chiku

चीक

Correct Answer Tamarind

Q221 In eukaryotic cells synthesis of

RNA takes place in the

यकहॳ योटटककह८लिकाओमआरएनएकासशिहॳषण

mdashndash महह८ताहहॴ 7-Feb-2017

Options

1) Mitochondria

माईटह८कोडडरया 2) Centrioles

सटरीयह८मस

3) Ribosomes

ररबह८सह८नस

4) Nucleus

नयनकियस

Correct Answer nucleus

eukaryotic cell -Transcription is the

process of synthesizing ribonucleic acid

(RNA)Synthesis takes place within the

nucleus of eukaryotic cells or in the

cytoplasm of prokaryotes and converts

the genetic code from a gene in

deoxyribonucleic acid ( DNA ) to a

strand of RNA that then directs

proteinsynthesis

Q222 _________is caused by parasites

of the Plasmodium genus

पिाजमह८डियमजातीकहॳ परजीवी mdash- कहॳ कारणहहॴ 7-Feb-2017

Options

1) Dysentery

पहॳचचि

2) Malaria

मिहॳररया 3) Chickenpox

F A C E B O O K

P A G E h t t p w w w f a c e b o o k c o m s s c m e n t o r s o f f i c i a l P a g e | 51

FOR MORE UPDATES AND MORE MATERIAL DO LIKE OUR FACEBOOK PAGE httpwwwfacebookcomsscmentorsofficial

चहॳचक

4) Herpes

हहॳपपयस

Correct Answer Malaria

Q223 Carotene in fruits and vegetables

gives it which color

फिह८औरसनलजयोमनसितकहॳ रह८टीनउनहकह९नसारगपरदानकरताहहॴ 7-Feb-2017

Options

1) Green

हरा 2) Pink

गिाबी 3) Orange

नारगी 4) Blue

नीिा Correct Answer Orange

Q224 Equus Caballus is the scientific

name of

एकवसकहॴ बहॳिस mdashmdashndash कापवजञाननकनामहहॴ 7-Feb-2017

Options

1) Horse

घह८िा 2) Zebra

ज़हॳिा 3) Donkey

गधा 4) Buffalo

भस

Correct Answer Horse

Q225 Elapidae Naja is the scientific name of

एिीपीिीनाजा mdashmdash- कावहॴजञाननकनामहहॴ 8-Feb-2017

Options

1) Cobra

कह८बरा 2) Elephant

हािी 3) Eagle

ग ि

4) Owl

उमि Correct Answer Cobra

Q226 Which disease is caused due to

deficiency of Iron

िह८हकीकमीकहॳ कारणकह९नसारह८गहह८ताहहॴ 8-Feb-

2017

Options

1) Beriberi

बहॳरीबहॳरी 2) Tetany

टहॳटनी 3) Kwashiorkor

कवािीऔरकर

4) Anaemia

रकतामपता Correct Answer Anaemia

Beriberi is a disease caused by a vitamin

B-1 deficiency also known as thiamine

deficiency

Tetany can be the result of an

electrolyte imbalance Most often itrsquos a

dramatically low calcium level also

known as hypocalcemia Tetany can also

be caused by magnesium deficiency or

too little potassium Having too much

acid (acidosis) or too much alkali

(alkalosis) in the body can also result in

tetany

Kwashiorkor also known as

ldquoedematous malnutrition It is a form of

malnutrition caused by a lack of protein

in the diet

Anaemia means that you have fewer red

blood cells than normal or you have less

F A C E B O O K

P A G E h t t p w w w f a c e b o o k c o m s s c m e n t o r s o f f i c i a l P a g e | 52

FOR MORE UPDATES AND MORE MATERIAL DO LIKE OUR FACEBOOK PAGE httpwwwfacebookcomsscmentorsofficial

haemoglobin than normal in each red

blood cell

Q227 is a leaf where the leaflets are

arranged along the middle vein

mdashndashएकपततीहहॴजहापतरकह८कीरचनाक ररयालिराकहॳ आसपासहह८तीहहॴ 8-Feb-2017

Options

1) Pinnately compound leaf

पपनहॳटिीसयकतपतती 2) Palmately compound leaf

पामहॳटिीसयकतपतती 3) Compound leaf

सयकतपतती 4) Simple leaf

साधारणपतती Correct Answer Pinnately compound

leaf

Q228 Haustoria or sucking roots are

found in which of the following

हह८सटह८ररयायाचसनहॳवािीजड़हॳनननननलिखितमसहॳककसमपाईजातीहहॴ 8-Feb-2017

Options

1) Wheat

गहॳह

2) Mango

आम

3) Chestnut

चहॳसटनट

4) Cuscuta

कसकयटा Correct Answer Cuscuta

Haustorial roots -The roots of parasitic

plants which penetrate into the host

tissues to absorb nourishment are

called haustorial roots hellip Also known as suckingor parasitic roots

Q229 Equs Asinus is the scientific name

of

एकवसएलसनस mdashmdashndash कावहॴजञाननकनामहहॴ 8-

Feb-2017

Options

1) Donkey

गधा 2) Cow

गाय

3) Deer

टहरन

4) Kangaroo

कगा

Correct Answer Donkey

Q230 Ficus benghalensis is the scientific name of

फाईकसबहॳनगहॳिहॳलसस mdashndash कापवजञाननकनामहहॴ 8-Feb-2017

Options

1) Banyan

बरगद

2) Pineapple

अनानास

3) Babul

बबि

4) Tulsi

तिसी Correct Answer Banyan

Q231 Equus burchellii is the scientific name of

एकवसबचिी mdashmdash- कापवजञाननकनामहहॴ 8-Feb-2017

Options

1) Horse

घह८िा 2) Zebra

जहॳिा 3) Buffalo

F A C E B O O K

P A G E h t t p w w w f a c e b o o k c o m s s c m e n t o r s o f f i c i a l P a g e | 53

FOR MORE UPDATES AND MORE MATERIAL DO LIKE OUR FACEBOOK PAGE httpwwwfacebookcomsscmentorsofficial

भस

4) Ass

गधा Correct Answer Zebra

Page 12: COMPILATION OF ALL 72 SETS OF BIOLOGY SSC CHSL-2016 · OF BIOLOGY SSC CHSL-2016 PREPARED BY : SSC MENTORS BIOLOGY SPECIAL . F A C E B O O K P A G E : h t t p : / / w w w . f a c e

F A C E B O O K

P A G E h t t p w w w f a c e b o o k c o m s s c m e n t o r s o f f i c i a l P a g e | 11

FOR MORE UPDATES AND MORE MATERIAL DO LIKE OUR FACEBOOK PAGE httpwwwfacebookcomsscmentorsofficial

दरदनषटदह८ष

2) Near Sightedness

ननकटदनषटदह८ष

3) Astigmatism

एसटीगमहॳटटजम

4) Night Blindness

रतोधी Correct Answer Near Sightedness

Myopia occurs when the eyeball is too

long relative to the focusing power of

the cornea and lens of the eye This

causes light rays to focus at a point in

front of the retina rather than directly

on its surface

Hyperopia Hypermetropia (

Farsightedness )- when light rays

entering the eye focus behind the retina

rather than directly on it The eyeball of

a farsighted person is shorter than

normal

Astigmatism usually is caused by an

irregularly shaped cornea Instead of

the cornea having a symmetrically

round shape (like a baseball) it is

shaped more like an American football

Nyctalopia also called night ndash blindness

is a condition making it difficult or

impossible to see in relatively low light

Q43 Who is known as the father of

Green Revolution

हररतिानतकहॳ जनककहॳ पमककसहॳजानाजाताहहॴ

12-Jan-2017

1) Dr Robert Nucleus

िॉ रॉबटयनयनकियस

2) Dr Ian Wilmut

िॉ इयानपविमट

3) Dr NE Borlaug

िॉ एनईबह८रिॉग

4) Dr JC Bose

िॉ जहॳसीबह८स

Correct Answer Dr NE Borlaug

Q44 Panthera Tigris is the scientific

name of

पिहॳराटटगरीस mdashmdashmdash कावहॴजञाननकनामहहॴ 12-Jan-2017

Options

1) Panther

तदआ

2) Tiger

बाघ

3) Whale

हहॳि

4)Goat

बकरी Correct Answer Tiger

Q45 How many facial bones are there

हमारहॳचहॳहरहॳमककतनीहडडियााहह८तीहहॴ 13-Jan-2017

Options

1)34

2)24

3)14

4)4

Correct Answer 14

Q46 ndash Halophytes are plants that grow

in

हहॴिह८फाईटसवहॳपह९धहॳहह८तीहहॴजह८ mdash- मउगतहॳहहॴ SSC CHSL Science (biology) 2016

Question Paper

13-Jan-2017

Options

1) Fresh Water

ताजापानी 2) Cold Water

ठिापानी 3) Ponds

तािाब

4) Salt Water

िारापानी Correct Answer Salt Water

F A C E B O O K

P A G E h t t p w w w f a c e b o o k c o m s s c m e n t o r s o f f i c i a l P a g e | 12

FOR MORE UPDATES AND MORE MATERIAL DO LIKE OUR FACEBOOK PAGE httpwwwfacebookcomsscmentorsofficial

Q47 Felis Catus is the scientific name of

फहॳ लिसकहॴ टस mdashndash कावहॴजञाननकनामहहॴ 13-Jan-2017

Options

1) Cat

बबमिी 2) Dog

कतता 3) Mouse

चहा 4) Porcupine

साही Correct Answer Cat

Q48 Which of the following induces

nitrogen fixation in soil

नननननलिखितमसहॳकह९नलम ीमनाइटरह८जनननयतनकह८परहॳररतकरताहहॴ

15-Jan-2017

Options

1) Protozoa

परह८टह८जआ

2) Bacteria

बहॴकटीररया 3) Fungi

कवक

4)Algae

िहॴवाि

Correct Answer Bacteria

Bacteria that change nitrogen gas from

the atmosphere into solid nitrogen

usable by plants are called nitrogen-

fixing bacteria These bacteria are

found both in the soil and in symbiotic

relationships with plants

They contain symbiotic bacteria called

rhizobia within nodules in their root

systems producing nitrogen compounds

that help the plant to grow and compete

with other plants When the plant dies

the fixed nitrogen is released making it

available to other plant

Q49 Which of the following is the

largest known cell

नननननलिखितमसहॳकह९नसीसबसहॳबड़ीजञातकह८लिकाहहॴ

SSC CHSL Science (biology) 2016

Question Paper

15-Jan-2017

1) Eukaryotic Cell

यकहॳ ररयह८टटककह८लिका 2) Prokaryotic Cell

परह८कहॳ ररयह८टटककह८लिका 3) Mycoplasma

मायकह८पिासम

4) Ostrich Eggs

ितरमगयकाअिा Correct Answer Ostrich Eggs

Q50 The association of animals in

which both the partners are benefitted

is known as

जानवरोकावहसहयह८गनजसमहॳदह८नोभागीदारिाभापवनतहह८तहॳहहॴ उसहॳ mdashmdashndash कहॳ पमजानाजाताहहॴ SSC CHSL Science (biology) 2016

Question Paper

15-Jan-2017

Options

1) Amensalism

सहजीपवत

2) Commensalism

परजीपवत

3) Colony

कॉिनी 4) Mutualism

अनयह८नयाशरयवाद

Correct Answer Mutualism

Amensalism is any relationship between

organisms of different species in which

F A C E B O O K

P A G E h t t p w w w f a c e b o o k c o m s s c m e n t o r s o f f i c i a l P a g e | 13

FOR MORE UPDATES AND MORE MATERIAL DO LIKE OUR FACEBOOK PAGE httpwwwfacebookcomsscmentorsofficial

one organism is inhibited or destroyed

while the other organism remains

unaffected

Commensalism an association between

two organisms in which one benefits and

the other derives neither benefit nor

harm

Q51 Pneumonia affects which of the

following organs of human body

ननमह८ननयामानविरीरकहॳ नननननलिखितमसहॳककसअगकह८परभापवतकरताहहॴ

15-Jan-2017

Options

1)Kidneys

गद

2)Lungs

फहॳ फड़हॳ 3) Throat

गिहॳ 4) Liver

यकत

Correct Answer Lungs

When the germs that cause pneumonia

reach your lungs the lungsrsquo air sacs

(alveoli) become inflamed and fill up

with fluid This causes the symptoms of

pneumonia such as a cough fever

chills and trouble breathing When you

have pneumonia oxygen may have

trouble reaching your blood

Q52 Mendel is known as

मििकह८ mdashmdash- कहॳ पमजानाजाताहहॴ 15-Jan-2017

Options

1) Father of Physiology

िरीरकियािासतरकहॳ जनक

2) Father of Geology

भगभयिासतरकहॳ जनक

3) Father of Genetics

जहॳनहॳटटकसकहॳ जनक

4) Father of Biology

जीविासतरकहॳ जनक

Correct Answer Father of Genetics

Q53 Which of the following are also

known as Suicidal bag of Cells

ननननलिखितमसहॳककसहॳआतमहतयाकरनहॳवािीकह८लिकाओकाबहॴगकहाजाताहहॴ

15-Jan-2017

Options

1) Lysosomes

िायसोसह८म

2) Lycosome

िायकह८सह८म

3) Nucleus

नालभक

4) Chromosome

िह८मह८सह८म

Correct Answer Lysosomes

Q54 Mesothelioma is a type of cancer

The most common area affected in it is

the lining of the ________

लमज़ह८िहॳिहॳलमयाक सरकाएकपरकारहहॴ इससहॳपरभापवतहह८नहॳवािासबसहॳसामानयकषहॳतर mdash

mdashmdash काअसतरहहॴ 15-Jan-2017

Options

1)Heart

हदय

2)Brain

मनसतषक

3)Stomach

आमािय

4)Lungs

फहॳ फड़हॳ Correct Answer lungs

Asbestos exposure is the main cause of

pleural mesothelioma When asbestos

fibers are breathed in they travel to the

F A C E B O O K

P A G E h t t p w w w f a c e b o o k c o m s s c m e n t o r s o f f i c i a l P a g e | 14

FOR MORE UPDATES AND MORE MATERIAL DO LIKE OUR FACEBOOK PAGE httpwwwfacebookcomsscmentorsofficial

ends of small air passages and reach the

pleura where they can cause

inflammation and

scarring

Q55 Which one of the following is an

insectivorous plant

नननननलिखितमसहॳकह९नसाएकककटाहरीवनसपनतहहॴ

15-Jan-2017

Options

1) Utricularia

यटरीकिहॳररया 2) Sequoia

सहॳकयओइया 3) Nostoc

नॉसटह८क

4) Bryophyta

िायह८फाईटा Correct Answer Utricularia

Q56 ______________ is a

multibranched polysaccharide of

glucose that serves as a form of energy

storage in animals and fungi

mdashmdashगिकह८जकाएकबहिािायकतपह८िीसहॳकहॳ राइिहहॴ जह८जानवरोऔरकवकमउजायभणिारणकहॳ एक पमकाययकरताहहॴ 15-Jan-2017

Options

1) Cellulose

सहॳमयिह८ज

2) Glycogen

गिायकह८जन

3) Pectin

पहॳनकटन

4) Chitin

चीटटन

Correct Answer Glycogen

Q57 The largest gland of the human

body is

mdashmdashmdashमानविरीरकीसबसहॳबड़ीगरिीहहॴ 16-Jan-2017

Options

1) Pancreas

अगयािय

2) Thyroid

िायरॉइि

3) Large Intestine

बड़ीआत

4) Liver

यकत

Correct Answer Liver

Q58 Photosynthesis in plants takes

place in

वनसपनतयोमपरकािसशिहॳषणकीकियाहह८तीहहॴ

16-Jan-2017

Options

1) Stem

तना 2) Leaves

पनततयाा 3) Roots

जड़हॳ 4) Flower

फि

Correct Answer Leaves

During this reaction carbon dioxide

and water are converted into glucose

and oxygen The reaction requires light

energy which is absorbed by a green

substance called

chlorophyll Photosynthesis takes place

in leaf

cells These contain chloroplasts which

are tiny objects containing chlorophyll

F A C E B O O K

P A G E h t t p w w w f a c e b o o k c o m s s c m e n t o r s o f f i c i a l P a g e | 15

FOR MORE UPDATES AND MORE MATERIAL DO LIKE OUR FACEBOOK PAGE httpwwwfacebookcomsscmentorsofficial

Q59 Insects that transmit diseases are

known as

जह८कीड़हॳरह८गसचाररतकरतहॳहहॴ उनह mdashmdash-

कहॳ नामसहॳजानाजाताहहॴ 16-Jan-2017

1)Pathogens

रह८गज़नक

2) Vectors

वहॳकटर

3) Drones

परजीवी 4)Scalars

अटदषट

Correct Answer Vectors

A vector is an organism that does not

cause disease itself but which spreads

infection by conveying pathogens from

one host to another Species of mosquito

for example serve as vectors for the

deadly disease Malaria

Q60 Which is the second largest gland

of Human body

मानविरीरकीदसरीसबसहॳबड़ीगरिीकह९नसीहहॴ

SSC CHSL Science (biology)

2016 Question Paper

16-Jan-2017

Options

1) Liver

यकत

2) Large Intestine

बड़ीआत

3) Thorax

छाती 4) Pancreas

अगनयािय

Correct Answer Pancreas

Q61 Annona squamosa is the scientific

name of

एनह८नासकवामह८सा (Annona squamosa) mdash

mdashmdash कावहॴजञाननकनामहहॴ 16-Jan-2017

Options

1) Custard Apple

सीताफि

2) Papaya

पपीता 3) Babhul

बबि

4) Drumstick

सहजन

Correct Answer Custard Apple

Q62 The disease Beri Beri is caused due

to the deficiency of which of the

following

बहॳरीबहॳरीरह८गनननननलिखितमसहॳककसकीकमीकहॳकारणहह८ताहहॴ

16-Jan-2017

Options

1) Vitamin B2

पवटालमन B2

2) Vitamin B1

पवटालमन B1

3) Vitamin B12

पवटालमन B12

4) Vitamin E

पवटालमन E

Correct Answer Vitamin B1

Beriberi is a disease caused by a vitamin

B-1 deficiency also known as thiamine

deficiency

Q63 Chlorophyll was first isolated and

named by

किह८रह८कफिकह८ mdash-

दवारापहिहॳपिकऔरनालमतककयागया 16-Jan-2017

F A C E B O O K

P A G E h t t p w w w f a c e b o o k c o m s s c m e n t o r s o f f i c i a l P a g e | 16

FOR MORE UPDATES AND MORE MATERIAL DO LIKE OUR FACEBOOK PAGE httpwwwfacebookcomsscmentorsofficial

Options

1) Caventou

कहॳ वहॳत 2) Pelletier

पहॳिहॳटटयर

3) Chlorophyll

किह८रह८कफि

4) Caventou and Pelletier

कहॳ वहॳतऔरपहॳिहॳटटयर

Correct Answer Caventou and Pelletier

Chlorophyll was first isolated and

named by

Joseph Bienaimeacute Caventou and Pierre

Joseph Pelletier in 1817 The presence of

magnesium in chlorophyll was

discovered in 1906 and was the first

time that magnesium had been detected

in living tissue

Q64 Which of the following organisms

does not fit into the Cell Theory

नननननलिखितमसहॳकह९नसाजीवकह८लिकालसदातअन पनहीहहॴ

16-Jan-2017

Options

1) Bacteria

बहॴकटीररया 2) Virus

वायरस

3) Fungi

कवक

4) Plants

पह९धहॳ Correct Answer Virus

The bottom line is that viruses are not

alive and not related to cells in any way

The cell theory states that all living

things are made of cells cells are the

basic units of structure and function of

living things and that all cells come

from other cells Since viruses are not

made of cells and do not use cells in any

of their processes they are not related to

the cell theory

Q65 Which of these is not a

macronutrient for Plants

नननननलिखितमसहॳकह९नसापह९धह८कहॳ लिएमिह८नयटरीएटनहीहहॴ

SSC CHSL Science (biology) 2016

Question Paper

17-Jan-2017

Options

1) Nitrogen

नाइटरह८जन

2) Phosphorus

फासफह८रस

3) Potassium

पह८टालसयम

4) Chlorine

किह८रीन

Correct Answer Chlorine

In relatively large amounts the soil

supplies nitrogen phosphorus

potassium calcium magnesium and

sulfur these are often called the

macronutrients In relatively small

amounts the soil supplies iron

manganese boron molybdenum

copper zinc chlorine and cobalt the

so-called micronutrients

Q66 Name the respiratory organs of

insects

कीटह८मनसतिशरवसनअगनामकानामहहॴ

17-Jan-2017

Options

1) Skin

तवचा 2) Body Surface

िरीरकीसतह

F A C E B O O K

P A G E h t t p w w w f a c e b o o k c o m s s c m e n t o r s o f f i c i a l P a g e | 17

FOR MORE UPDATES AND MORE MATERIAL DO LIKE OUR FACEBOOK PAGE httpwwwfacebookcomsscmentorsofficial

3) Gills

गिफड़हॳ 4) Tracheae

शरावस- निी Correct Answer Tracheae

Air enters the respiratory systems of

insects through a series of external

openings called

spiracles These external openings

which act as muscular valves in some

insects lead to the internal respiratory

system a densely networked array of

tubes called tracheae

Q67 The poisonous gas accidentally

released in Bhopal Gas Tragedy is

भह८पािगहॴसतरासदीमगितीसहॳमकतहईजहरीिीगहॴसिी

17-Jan-2017

1) Methane

मीिहॳन

2) Nitrous Oxide

नाइटरसऑकसाइि

3) Methyl Isocyanate

महॴचििआयसोसायनहॳट

4) Cyanogen

सायनह८जहॳन

Correct Answer Methyl Isocyanate

Q68 What does Trypsin do

टटरनपसनकयाकरताहहॴ

SSC CHSL Science (biology) 2016

Question Paper

17-Jan-2017

Options

1) Breaks down Carbohydrates

काबोहाइडरहॳटकापवघटनकरताहहॴ 2) Synthesizes proteins

परह८टीनकासििहॳषणकरताहहॴ 3) Breaks down fats

वसाकापवघटनकरताहहॴ 4) Breaks down proteins

परह८टीनकापवघटनकरताहहॴ Correct Answer Breaks down proteins

Trypsin is one of the three principal

digestive

proteinases the other two being pepsin

and

chymotrypsin In the digestive process

trypsin acts with the other proteinases

to break down dietary protein molecules

to their component

peptides and amino acids

A protease is any enzyme that performs

proteolysis protein catabolism by

hydrolysis of peptide bonds

Q69 Name the source from which

Aspirin is produced

उससरह८तकानामबताइए

नजससहॳएनसपररनकाउतपादनककयाजाताहहॴ

17-Jan-2017

Options

1) Willow bark

पविह८कीछाि

2) Oak Tree

ओककावकष

3) Acacia

बबि

4) Eucalyptus

नीिचगरी Correct Answer Willow bark

The compound from which the active

ingredient in aspirin was first derived

salicylic acid was found in the bark of a

willow tree in 1763 by Reverend

Edmund Stone of Chipping-Norton

Q70 Cannis Familiaris is the scientific

name of

कहॴ ननसफहॳ लमलियहॳररस mdash- कावहॴजञाननकनामहहॴ

17-Jan-2017

F A C E B O O K

P A G E h t t p w w w f a c e b o o k c o m s s c m e n t o r s o f f i c i a l P a g e | 18

FOR MORE UPDATES AND MORE MATERIAL DO LIKE OUR FACEBOOK PAGE httpwwwfacebookcomsscmentorsofficial

Options

1) Cat

बबमिी 2)Dog

कतता 3) Fox

िह८मड़ी 4) Wolf

भहॳडड़या Correct Answer Dog

Q71 Harmful bacteria in potable water

make the water

पीनहॳकहॳ पानीमनसतिघातकबहॴकटीररयाउसपानीकह८बनातहॳहहॴ 17-Jan-2017

Options

1) unfit to drink

पीनहॳकहॳ लिएअयह८गय

2) smelly

दगयनधयकत

3) Colored

रगीन

4) Turbid

मटमहॴिा Correct Answer unfit to drink

Q72 Musa paradisiaca is the scientific

name of which plant

मसापहॴराडिलसयाकाककसपह९धहॳकावहॴजञाननकनामहहॴ

17-Jan-2017

Options

1) Mango

आम

2) Wheat

गहॳह

3) Corn

भ ा 4) banana

कहॳ िा Correct Answer banana

Q73 Prawns belong to which family

झीगहॳककसपररवारकहॳ हह८तहॳहहॴ 17-Jan-2017

Options

1) Crustaceans

िसटहॳलियन

2)Fish

मछिी 3) Amphibians

अननफबबयस

4) Reptiles

रहॳपटाइमस

Correct Answer Crustaceans

Q74 Name the drug that is yielded from

Cinchona tree and is used to cure

malaria

उसऔषचधकानामबताइएनजसहॳलसगकह८नापहॳड़सहॳपरापतककयाजाताहहॴऔरनजसकाउपयह८गमिहॳररयाकहॳ उपचारमककयाजाताहहॴ 17-Jan-2017

Options

1) Camptothea

कहॴ नटह८चिया 2) Acuminata

एकयलमनहॳटा 3) Quinine

कनहॴन

4) Cinchonia

लसकह८ननया Correct Answer Quinine

Q75 Blood Circulation was discovered

by

रकतपररसचरणकी mdashmdashndash दवारािह८जकीिी 17-Jan-2017

Options

1) Mary Anderson

F A C E B O O K

P A G E h t t p w w w f a c e b o o k c o m s s c m e n t o r s o f f i c i a l P a g e | 19

FOR MORE UPDATES AND MORE MATERIAL DO LIKE OUR FACEBOOK PAGE httpwwwfacebookcomsscmentorsofficial

महॴरीएिरसन

2) Virginia Apgar

वनजयननयाएपगार

3) William Harvey

पवलियमहाव

4) Robert Feulgen

रॉबटयफ़यिजहॳन Correct Answer William Harvey

Q76 Vitamin A is also known as

पवटालमन A कह८ mdashmdash- कहॳ नामसहॳभीजानाजाताहहॴ SSC CHSL Science (biology) 2016

Question Paper

18Jan2017

Options

1) Thiamine

िायलमन

2) Riboflavin

ररबह८फिहॳपवन

3) Retinol

रहॳटटनॉि

4) Calciferol

कहॴ नमसफहॳ रह८ि

Correct Answer Retinol

Q77 Some roots called arise from an

organ other than the radicle

कछजड़हॳनजनह mdashmdashmdash कहाजाताहहॴ वहमिकहॳ अिावाककसीअनयअगसहॳउतपननहह८तीहहॴ 18Jan2017

Options

1) tap roots

मखयजड़

2) stilt roots

ि ाजड़

3) fibrous roots

रहॳिहॳदारजड़

4) adventitious roots

आकनसमकजड़

Correct Answer adventitious roots

Q78 Spiders belong to which class of

animals

मकडड़यापराणीवगीकरणकहॳ ककसवगयमआतीहहॴ 18Jan2017

Options

1) Arachnids

एरहॳकननडस

2) Aves

एपवस

3) Gastropods

गहॴसटरोपह८िस

4) Anthozoa

एिह८जआ

Correct Answer Arachnids

Q79 How many layers does Human

Skin have

मानवतवचामककतनीपरतहॳहह८तीहहॴ

18Jan2017

Options

1) 5

2) 7

3) 11

4) 3

Correct Answer 3

Skin has three layers The epidermis

the outermost layer of skin provides a

waterproof barrier and creates our skin

tone The dermis beneath the

epidermis contains tough connective

tissue hair follicles and sweat glands

The deeper subcutaneous tissue (

hypodermis ) is made of fat and

connective tissue

Q80 Allium Cepa is the scientific name

of

एलियमलसपपा mdashmdashndash कावहॴजञाननकनामहहॴ 18Jan2017

F A C E B O O K

P A G E h t t p w w w f a c e b o o k c o m s s c m e n t o r s o f f i c i a l P a g e | 20

FOR MORE UPDATES AND MORE MATERIAL DO LIKE OUR FACEBOOK PAGE httpwwwfacebookcomsscmentorsofficial

Options

1) Carrot

गाजर

2) Tomato

टमाटर

3) Potato

आि 4) Onion

पयाज़

Correct Answer Onion

Q81 DNA stands for

िीएनएकापणय प mdashmdash- हहॴ 18Jan2017

Options

1) Di Nucleic Acid

िाईनयनकिकएलसि

2) Deoxy Nucleic Acid

िीओकसीनयनकिकएलसि

3) Diribonucleic Acid

िाईराइबह८नयनकिकएलसि

4) Deoxyribonucleic Acid

िीऑकसीराइबह८नयनकिकएलसि

Correct Answer Deoxyribonucleic Acid

Q82 Organisms that generate energy

using light are known as

जह८जीवाणपरकािकाउपयह८गकरउजायउतपननकरतीहहॴ उनह mdashmdash कहॳ पमजानाजाताहहॴ

18Jan2017

Options

1) Chaemolithotrophs

ककमह८लििह८टरह८पस

2) Oligotrophs

ओलिगह८टरह८पस

3) Bacteria

बहॴकटीररया 4)Photoautotrophs

फह८टह८ओटह८टरह८पस

Correct Answer Photoautotrophs

An oligotroph is an organism that can

live in an environment that offers very

low levels of nutrients

Q83 Which drug is used as an

Antidepressant

ककसदवाएकहतािारह८धीकहॳ पमपयोगककयाजाताहहॴ Options

1) Oxybutynin

ओकसीलयटीनन

2)Tramadol

टरहॳमहॳिह८ि

3 ) Sumatriptan

समहॳटरीपटहॳन

4) Bupropion

लयपरह८पपयह८न

Correct Answer Bupropion

लयपरह८पपयह८न

Q84 The orange colour of carrot is

because of

गाजरकानारगीरगनननननलिखितमसहॳककसीएककीवजहसहॳहह८ताहहॴ 18Jan2017

Options

1) it grows in the soil

यहलम ीमउगतीहहॴ 2) Carotene

कहॴ रह८टीन

3) it is not exposed to sunlight

यहसययपरकािकहॳ सपकय मनहीआती 4) the entire plant is oranqe in colour

सनपणयपह९धानारगीरगकाहह८ताहहॴ Correct Answer Carotene

Q85 Snake venom is highly modified

saliva containing

F A C E B O O K

P A G E h t t p w w w f a c e b o o k c o m s s c m e n t o r s o f f i c i a l P a g e | 21

FOR MORE UPDATES AND MORE MATERIAL DO LIKE OUR FACEBOOK PAGE httpwwwfacebookcomsscmentorsofficial

सापकाजहरअततयाचधकसिह८चधतिारहह८तीहहॴनजसमहॳ mdashmdash- हह८ताहहॴ Options

l)Prototoxins

परह८टह८टॉनकसस

2)Neutrotoxins

नयटरोटॉनकसस

3)Zootoxins

जटॉनकसस

4)Electrotoxins

इिहॳकटरह८टॉनकसस

Correct Answer Zootoxins

जटॉनकसस

Q86 Which type of pathogen causes the

water-borne disease Schistosomiasis

ककसपरकारकारह८गज़नकजिजननतरह८गलससटह८सह८लमलससकाकारणबनताहहॴ

18Jan2017

Option

1) Parasitic

परजीवी 2)Protozoan

परह८टह८जआ

3) Bacterial

बहॴकटीररयि

4) Viral

वायरि

Correct Answer Parasitic

Schistosomiasis also known as snail

fever and bilharzia is a disease caused

by parasitic

flatworms called schistosomes

Q87 Prothrombin responsible for

clotting of blood is released by

परह८िह८ननबन

जह८रकतकािककाजमनहॳकहॳ लिएनजनमहॳदारहहॴ mdashndash

कहॳ दवारासतरापवतककयाजाताहहॴ

19Jan2017

Options

1) Small Intestine

छह८टीआत

2) Blood Platelets

रकतपिहॳटिहॳटस

3) Large Intestine

बड़ीआत

4Heart

हदय

Correct Answer Blood Platelets

Q88 Acacia arabica is the scientific

name of

अकहॳ लियाअरहॳबबका mdashmdashndash कावहॴजञाननकनामहहॴ 19-Jan-2017

Options

1) Neem

नीम

2) Teak

सागह९न

3) Babhul

बबि

4) Pomegranate

अनार

Correct Answer Babhul

Q89 Cannis Vulpes is the scientific

name of

कहॴ ननसवनमपस mdashmdash- कावहॴजञाननकनामहहॴ 19-Jan-2017

Options

1) Dog

कतता 2) Wolf

भहॳडड़या 3) Fox

िह८मड़ी 4) Hyena

िाकिबगघा

F A C E B O O K

P A G E h t t p w w w f a c e b o o k c o m s s c m e n t o r s o f f i c i a l P a g e | 22

FOR MORE UPDATES AND MORE MATERIAL DO LIKE OUR FACEBOOK PAGE httpwwwfacebookcomsscmentorsofficial

Correct Answer Fox

Q90 The beetroot is the portion of the

beet plant

चकदरपह९धहॳका mdashmdashndash भागहहॴ 19-Jan-2017

Options

1) tap root

मखयजड़

2) Adventitious

आकनसमक

3) bulb of the stem

तनहॳकाकद

4) Rhizome

परकद

Correct Answer tap root

Q91 What is the basic unit of heredity

आनवलिकताकीबननयादीइकाईकयाहहॴ 19-Jan-2017

Options

1) DNA

िीएनए

2) RNA

आरएनए

3) Chromosome

िह८मह८सह८म

4) Gene

जीन

Correct Answer gene

Genes are the units of heredity and are

the instructions that make up the bodyrsquos

blueprint They code for the proteins

that determine virtually all of a personrsquos

characteristics Most genes come in

pairs and are made of strands of genetic

material called deoxyribonucleic acid

or DNA

Q92 Lungs are the primary organs of

फहॳ फड़हॳmdashndashकहॳ परािलमकअगहहॴ

19-Jan-2017

Options

1) Digestion

पाचन

2) Constipation

कलज

3) Perspiration

पसीना 4)Respiration

शवसन

Correct Answer Respiration

Q93 Sugarcane is a type of

गननाएकपरकारका mdash- हहॴ 20-Jan-2017

Options

1)creeper

िता 2)tree

पहॳड़

3)shrub

झाड़ी 4)grass

घास

Correct Answer grass

Q94 Who is commonly known as ldquothe

Father of Microbiologyrdquo

सामानयत ldquo सकषमजीवपवजञानकहॳ जनक lsquo

कहॳ नामसहॳककसहॳजानाजातहहॴ 20-Jan-2017

Options

1) Robert Hooke

रॉबटयहक

2) Antonie Philips van Leeuwenhoek

एटह८नीकफलिपवानमयएनहह८क

3) Carl Linnaeus

काियिीनाईयस

4) Charles Darwin

चामसयिापवयन

F A C E B O O K

P A G E h t t p w w w f a c e b o o k c o m s s c m e n t o r s o f f i c i a l P a g e | 23

FOR MORE UPDATES AND MORE MATERIAL DO LIKE OUR FACEBOOK PAGE httpwwwfacebookcomsscmentorsofficial

Correct Answer Antonie Philips van

Leeuwenhoek

Q95 For the aquatic organisms the

source of food is

जिीयजीवाणकािाघसरह८तहहॴ 20-Jan-2017

Options

1) Phytoplankton

फायटह८पिहॳकटन

2) Sea Weed

समदरीिहॴवाि

3)Aqua plankton

एकवापिहॳकटन

4) Zooplankton

जपिहॳकटन

Correct Answer Phytoplankton

Q96 Haemoglobin has the highest

affinity with which of the following

हीमह८गिह८बबनकीननननमसहॳककसकहॳ सािउततमसमानताहहॴ

20-Jan-2017

Options

1)SO2

2)CO2

3)CO

4)NO2

Correct Answer CO

It has a greater affinity for hemoglobin

than oxygen does It displaces oxygen

and quickly binds so very little oxygen

is transported through the body cells

Q97 Who developed the theory of

Evolution

उदपवकासकालसदातककसनहॳपवकलसतककया

20-Jan-2017

Options

1) Charles Darwin

चामसयिापवयन

2) Isaac Newton

आयजहॳकनयटन

3) Pranav Mistry

परणवलमसतरी 4) Galileo Galilei

गहॳलिलियह८गहॳिीिी Correct Answer Charles Darwin

Q98 The primary function of RNA is

RNA कापरािलमककाययहह८ताहहॴ 20-Jan-2017

Options

1) Photosynthesis

परकािसशिहॳषण

2) Protein Synthesis

परह८टीनसशिहॳषण

3) Replication

परनतकनतबनाना 4) Translation

अनवादकरना Correct Answer Protein Synthesis

There are two main functions of RNA

It assists DNA by serving as a messenger

to relay the proper genetic information

to countless numbers of ribosomes in

your body The other main function of

RNA is to select the correct amino acid

needed by each ribosome to build new

proteins for your body

Q99 ______is the movement of

molecules across a cell membrane from

a region of their lower concentration to

a region of their higher concertration

उचचसादरताकहॳ कषहॳतरसहॳउसकीकमसादरतावािहॳकषहॳतरकीतरफएककह८लिकाखझमिीकहॳ माधयमसहॳहह८नहॳवािाअणओकहॳ सचिनकह८ mdash- कहतहॳहहॴ Options

1) Diffusion

पवसरण

2) Osmosis

ऑसमह८लसस

F A C E B O O K

P A G E h t t p w w w f a c e b o o k c o m s s c m e n t o r s o f f i c i a l P a g e | 24

FOR MORE UPDATES AND MORE MATERIAL DO LIKE OUR FACEBOOK PAGE httpwwwfacebookcomsscmentorsofficial

3) Active Transport

सकियआवागमन

4) Passive Transport

नननषियआवागमन

Correct Answer Active Transport

Q100 Study of classification of

organisms is known as 20-Jan-2017

जीवाणओकहॳ वगीकरणकहॳ अधययनकह८ mdash-

कहाजाताहहॴ Options

1) Serpentology

सपरहॳटह८िह८जी 2) Virology

वायरह८िह८जी 3) Taxonomy

टहॴकसोनह८मी 4) Physiology

कफनज़यह८िह८जी Correct Answer Taxonomy

Q101 Photosynthesis takes place inside

plant cells in

परकािसशिहॳषणवनसपनतकह८लिकामनसति mdash

mdashmdash महह८ताहहॴ 20-Jan-2017

Options

1) Ribosomes

राइबह८सह८नस

2) Chloroplasts

किह८रह८पिासट

3) Nucleus

नयकलियम

4) Mitochondria

माईटह८कोडडरया Correct Answer Chloroplasts

Q102 ______ is the cell organelle in

which the biochemical processes of

respiration and energy production

occur

mdashmdash- वहकह८लिकाअगहहॴ नजसमहॳशवसनऔरउजायउतपादनकहॳ जहॴसीजहॴवरासायननकपरकियायहह८तीहहॴ 20-Jan-2017

Options

1) Mitochondria

माइटह८कोडडरया 2) Chloroplast

किह८रह८पिासट

3) Ribosomes

राइबह८सह८नस

4) Nucleus

नयकिीयस

Correct Answer Mitochondria

Q103 Which non-flowering spore

bearing plants have roots

ककसफिनिगनहॳवािहॳऔरबीजाणधारकपह९धह८कीजड़हॳहह८तीहहॴ 21-Jan-2017

Options

1) Mosses

मह८सहॳस

2) Angiosperms

एननजयह८सपनसय 3) Ferns

फनसय 4) Gymnosperms

नजननह८सपनसय Correct Answer ferns

Q104 Which of the following is an

excretory organ of cockroach

नननननलिखितमसहॳकह९नसानतिच हॳकाउतसजयनअगहहॴ

21-Jan-2017

Options

F A C E B O O K

P A G E h t t p w w w f a c e b o o k c o m s s c m e n t o r s o f f i c i a l P a g e | 25

FOR MORE UPDATES AND MORE MATERIAL DO LIKE OUR FACEBOOK PAGE httpwwwfacebookcomsscmentorsofficial

1) Malphigian Tubules

मनमफनजयनटयबमस

2) Nephridia

नहॳकफरडिया 3) Coxal Gland

कह८कसिगरचिया 4) Green Gland

गरीनगरचिया Correct Answer Malphigian Tubules

Q105 Evaporation of water takes place

in which part of plants

पानीकहॳ वाषपीकरणकीकियापह९धोकहॳ ककसभागसहॳहह८तीहहॴ 21-Jan-2017

Options

1) Stem

तना 2) Stomata

सटह८मटा 3) Branch

िािाए

4) Fruit

फि

Correct Answer Stomata

Evaporation accounts for the movement

of water to the air from sources such as

the soil canopy interception and

waterbodies Transpiration accounts for

the movement of water within a plant

and the subsequent loss of water as

vapour through stomata in its leaves

Q106 A is the fleshy spore-bearing

fruiting body of a fungus

mdashmdashndashकवककामासि

बीजाणधारणकरनहॳवािाफिनहॳवािाअगहहॴ 21-

Jan-2017

Options

1) aloe vera

एिह८वहॳरा 2) Coral

मगा 3) Cactus

कहॴ कटस

4) Mushroom

ककरमतता Correct Answer mushroom

Q107 Which of the following is a fungal

disease

नननननलिखितमसहॳकह९नसाफफदसहॳहह८नहॳवािाएकरह८ग हहॴ

21-Jan-2017

Options

1) Dermatitis

तवचािह८ध

2) Cholera

हहॴजा 3) Jaundice

पीलिया 4) Indigofera

इननिगह८फहॳ रा Correct Answer Dermatitis

Dermatitis also known as eczema is a

group of diseases that results in

inflammation of the skin These diseases

are characterized by itchiness red skin

and a rash In cases of short duration

there may be small blisters while in

long-term cases the skin may become

thickened

Q108 In which form is glucose stored in

our body

हमारहॳिरीरमगिकह८जकासचयककस पमककयाजाताहहॴ

21-Jan-2017

Options

1) Insulin

F A C E B O O K

P A G E h t t p w w w f a c e b o o k c o m s s c m e n t o r s o f f i c i a l P a g e | 26

FOR MORE UPDATES AND MORE MATERIAL DO LIKE OUR FACEBOOK PAGE httpwwwfacebookcomsscmentorsofficial

इसलिन

2) Glucose

गिकह८ज

3) Glycogen

गिायकह८जहॳन

4) Fat

वसा Correct Answer Glycogen

Excess glucose is stored in the liver as

the large compound called glycogen

Glycogen is a polysaccharide of glucose

but its structure allows it to pack

compactly so more of it can be stored in

cells for later use

Q109 Where do plants synthesize

protein from

पह९धहॳपरह८टीनसशिहॳषणकहासहॳकरतहॳहहॴ

Options

1) Fatty Acids

वसाऐलसि

2) Sugar

िकर

3) Amino Acids

एलमनह८ऐलसि

4) Starch

सटाचय Correct Answer Amino Acids

Q110 Which part of the brain is

responsible for triggering actions like

thinking intelligence memory and

ability to learn

मनसतषककाकह९नसाटहससासह८चनहॳ बनधदमानी याददाशतऔरसीिनहॳकीकषमताजहॴसीकियाओकह८परहॳररतकरताहहॴ 21-Jan-2017

Options

1) Diencephalon

िायएनसहॳफहॳ िह८न

2) Hypothalamus

हयपह८िहॳिहॳमस

3) Cerebrum

सहॳरहॳिम

4) Control

कटरह८ि

Correct Answer Cerebrum

Q111 Which of the following is also

known as the Biochemical Laboratory

of the Human Body

नननननलिखितमसहॳककसहॳमानविरीरकीजहॴवरसायनपरयह८गिािाभीकहाजाताहहॴ 21-Jan-2017

Options

1) Small Intestine

छह८टीआत

2)Brain

मनसतषक

3) Pancreas

अगनयािय

4) Liver

नजगर

Correct Answer Liver

The liver makes bile that will help

emulsify and digest the fats we eat

The liver takes toxic substances and

convert them using enzymes the liver

cells makes into a non toxic form so the

body can dispose of them

The liver also converts fats protein and

carbohydrates into glucose which is the

energy source for our cells to use

The liver takes amino acids and makes

proteins by combining them

Q112 The yellow colour of human urine

is due to

मानवमतरकापीिारग mdashndash कीवजहसहॳहह८ताहहॴ 22-

Jan-2017

Options

1) Bile Salts

F A C E B O O K

P A G E h t t p w w w f a c e b o o k c o m s s c m e n t o r s o f f i c i a l P a g e | 27

FOR MORE UPDATES AND MORE MATERIAL DO LIKE OUR FACEBOOK PAGE httpwwwfacebookcomsscmentorsofficial

पपततनमक

2) Cholesterol

कह८िहॳसटरह८ि

3) Lymph

लिनफ

4) Urochrome

यरह८िह८म

Correct Answer Urochrome

Urobilin or urochrome is the chemical

primarily responsible for the yellow

color of urine

Q113 The wilting of plants takes place

due to

पह९धह८कालिचििहह८नाकी mdashmdash- कीवजहसहॳहह८ताहहॴ 22-Jan-2017

Options

1)Photosynthesis

परकािसशिहॳषण

2) Transpiration

वाषपह८तसजयन

3) Absorption

अविह८षण

4) Respiration

शरवसन

Correct Answer Transpiration

Wilting is the loss of rigidity of non-

woody parts of plants This occurs when

the turgor pressure in non-lignified

plant cells falls towards zero as a result

of diminished water in the cells

Q114 Bovidae Ovis is the scientific name of

बह८पविीओपवस mdashndash कावहॴजञाननकनामहहॴ 22-Jan-2017

Options

1) Goat

बकरी 2) Cow

गाय

3) Buffalo

भहॳस

4) Sheep

भहॳड़

Correct Answer Sheep

Q115 Plants get their energy to produce

food from which of the following

पह८धहॳभह८जनकाननमायणकरनहॳकहॳ लिएनननननलिखितमसहॳककससहॳउजायपरापतकरतहॳहहॴ

22-Jan-2017

Options

1) Photosynthesis

परकािसशिहॳषण

2)Bacteria

बहॴकटीररया 3)Fungi

कवक

4)Sun

सयय Correct Answer Sun

Q116 Which of the following is secreted

by the liver

नननननलिखितमसहॳककसकासरावनजगरसहॳहह८ताहहॴ

22-Jan-2017

Options

1) Glucose

गिकह८ज

2) Iodine

आयह८िीन

3) Cortisol

काटटरयसह८ि

4) Bile

पपतत

Correct Answer Bile

The liver makes bile that will help

emulsify and

digest the fats we eat

F A C E B O O K

P A G E h t t p w w w f a c e b o o k c o m s s c m e n t o r s o f f i c i a l P a g e | 28

FOR MORE UPDATES AND MORE MATERIAL DO LIKE OUR FACEBOOK PAGE httpwwwfacebookcomsscmentorsofficial

Q117 Ferns belong to which division of

plants

फनसयपह९धह८कहॳ ककसभागमआतहॳहहॴ

22-Jan-2017

Options

1) Gymnosperms

नजननह८सपनसय 2) Angiosperms

एनजयह८सपनसय 3) Thallophyta

िहॴिह८फाईटा 4)Pteridophyta

टहॳररिह८फाईटा Correct Answer Pteridophyta

Q118 Who invented Antibiotics

एटीबायह८टटककाअपवषकारककसनहॳककयािा

22-Jan-2017

Options

1) Joseph Lister

जह८सहॳफलिसटर

2) William Harvey

पवलियमहाव

3) Robert Knock

रॉबटयनॉक

4)Alexander Fleming

अिहॳकज़िरफिहॳलमग

Correct Answer Alexander Fleming

Q119 Milbecycin is used in the

eradication of

लममबहॳसायलसनका mdashndash

मउनमिनमपरयह८गककयाजाताहहॴ 22-Jan-2017

Options

1) Agricultural Fungus

कपषकवक

2) Agricultural Pests

कपषकीटक

3) Agricultural Herbs

कपषिाक

4)Agricultural Weeds

कपषननराना Correct Answer Agricultural Pests

Milbemycin oxime is a veterinary drug

from the group of milbemycins used as

a broad spectrum antiparasitic It is

active against worms and mites(insects

Q120 Intestinal bacteria synthesizes

which of the following in the human

body

मानविरीरमआतोकहॳ बहॴकटीररयानननननलिखितमसहॳककसकासशिहॳषणकरतहॳहहॴ 22-Jan-2017

Options

1) Vitamin K

पवटालमन K

2) Proteins

परह८टीन

3) Fats

वसा 4) Vitamin D

पवटालमन D

Correct Answer Vitamin K

Q121 is the study of the physical form

and external structure of plants

mdashmdash-

मपह९धह८काभहॴनतक पऔरबाहरीसरचनाकाआदयाककयाजाताहहॴ 22-Jan-2017

Options

1) Physiology

कफनजयह८िह८जी 2) Anatomy

िरीररचनापवजञान

3) Phytomorphology

फाईटह८मह८फह८िह८जी 4)Cytology

कह८लिकापवजञान

Correct Answer Phytomorphology

F A C E B O O K

P A G E h t t p w w w f a c e b o o k c o m s s c m e n t o r s o f f i c i a l P a g e | 29

FOR MORE UPDATES AND MORE MATERIAL DO LIKE OUR FACEBOOK PAGE httpwwwfacebookcomsscmentorsofficial

Q122 Which of the following is a

structural and functional unit of

kidneys

नननननलिखितमसहॳकह९नसीगदोकीसरचनातमकऔरकाययकरीईकाईहहॴ

22-Jan-2017

Options

1) Renette Cells

रहॳनहॳटकह८लिकाए

2) Flame Cells

फिहॳमकह८लिकाए

3) Nephrites

नहॳफ़राइटस

4)Nephrons

नहॳफरोस

Correct Answer Nephrons

Nephron functional unit of the kidney

the structure that actually produces

urine in the process of removing waste

and excess substances from the blood

There are about 1000000 nephrons in

each human kidney

Q123 Which of the following is the

largest part of the human brain

नननननलिखितमसहॳकह९नसामानवमनसतषककासबसहॳबड़ाटहससाहहॴ

23-Jan-2017

Options

1) Ribs

पसलियाा 2) Cerebrum

सहॳरहॳिम

3) Pons

पोस

4)Thalamus

िहॴिहॳमस

Correct Answer Cerebrum

The cerebrum is the largest part of the

human brain making up about two-

thirds of the brainrsquos mass It has two

hemispheres each of which has four

lobes frontal parietal temporal and

occipital

Q124 The auxiliary buds

सहायककालियाmdashndash 23-Jan-2017

Options

1) grow endogenously from the pericycle

पहॳरीसाईककिसहॳअनतजातयपवकलसतहह८ताहहॴ 2) arise endogenously from the main

growing point

मिवपदसहॳअनतजातयउठताहहॴ 3) is an embryonic shoot located in the

axil of a leaf

एकभरणिटहहॴजह८एकपततीकहॳ अकषपरनसतिहह८ताहहॴ 4)arise exogenously from the epidermis

एपपिलमयससहॳबटहजातयतरीकहॳ सहॳउठताहहॴ Correct Answer is an embryonic shoot

located in the axil of a leaf

Q125 Which of the following is a viral

disease

इनमहॳसहॳकह९सीएकवायरिबीमारीहहॴ

23-Jan-2017

Options

1) Polio

पह८लियह८ 2) Tetanus

धनसतनभ

3) Leprosy

कषठरह८ग

4) Plague

पिहॳग

Correct Answer Polio

A viral disease (or viral infection)

occurs when an organismrsquos body is

invaded by pathogenic viruses and

infectious virus particles (virions) attach

to and enter susceptible cells

F A C E B O O K

P A G E h t t p w w w f a c e b o o k c o m s s c m e n t o r s o f f i c i a l P a g e | 30

FOR MORE UPDATES AND MORE MATERIAL DO LIKE OUR FACEBOOK PAGE httpwwwfacebookcomsscmentorsofficial

Poliomyelitis often called polio or

infantile paralysis is an infectious

disease caused by the poliovirus

Tetanusmdash A serious bacterial infection

that causes painful muscle spasms and

can lead to death

Leprosy also known as Hansenrsquos

disease (HD) is a long-term infection by

the bacterium Mycobacterium leprae or

Mycobacterium lepromatosis

Plague is an infectious disease caused by

the bacterium Yersinia pestis

Symptoms include fever weakness and

headache

Q126 Which organisms can help to

carry out Vermicomposting

कह९नसाजीववमीकनपह८नसटगममददकरताहहॴ

23-Jan-2017

Options

1) Nitrifying Bacteria

नाईटरीफाईगबहॴकटीररया 2) Earthworms

कहॴ चऐ

3) Algae

िहॴवि

4) Fungus

कवक

Correct Answer Earthworms

Q127 Contraction of heart is also

known as

हदयकहॳ सकचनकह८ mdash- भीकहाजाताहहॴ 23-Jan-

2017

Options

1) Systole

लससटह८ि

2) Aristotle

अरसत

3) Diastole

िायसटह८ि

4) Lub

मयब

Correct Answer Systole

Diastole is the part of the cardiac cycle

when the heart refills with blood

following systole (contraction)

Ventricular diastole is the period during

which the ventricles are filling and

relaxing while atrial diastole is the

period during which the atria are

relaxing

Q128 Azadirachta indica is the

botanical name of which of the

following

अजाटदराचताइडिकानननननलिखितमसहॳककसकावानसपनतनामहहॴ

23-Jan-2017

Options

1) Rose plant

गिाबकापह९धा 2) Apple tree

सहॳबकापहॳड़

3) Neem

नीम

4)Mango

आम

Correct Answer Neem

Q129 Which of the following is the

main end product of carbohydrate

digestion

नननननलिखितमसहॳकह९नसाकाबोहाइडरहॳटकहॳ पाचनकापरमिअतउतपादकहह८ताहहॴ 23-Jan-2017

Options

1) Fats

वसा 2) Lipids

लिपपडस

3) Glucose

गिकह८ज

4) Cellulose

F A C E B O O K

P A G E h t t p w w w f a c e b o o k c o m s s c m e n t o r s o f f i c i a l P a g e | 31

FOR MORE UPDATES AND MORE MATERIAL DO LIKE OUR FACEBOOK PAGE httpwwwfacebookcomsscmentorsofficial

सहॳमयिह८ज

Correct Answer Glucose

Intestinal absorption of end products

from digestion of carbohydrates and

proteins in the pig hellip During absorption some sugars (fructose or

galactose) released from the

corresponding sucrose and lactose

respectively during digestion were

partly metabolized into glucose by the

enterocyte

Q130 Which of the following glands is a

source of the enzyme Ptyalin

नननननलिखितगरचियोमसहॳएजाइमटयालिनकासरह८तहहॴ 23-Jan-2017

Options

1) Pancreas

अगरािय

2) Thyroid Gland

िाइराइिगरिी 3) Pituitary Gland

पीयषगरिी 4) Salivary Glands

िारगरचियाा Correct Answer Salivary Glands

Q131 Which of the following is not true

about Pteridophyta

ननननमसहॳकह९नसीबातटहॳररिह८फाईटकहॳ बारहॳमसचनहीहहॴ 23-Jan-2017

Options

1) Dominant phase is saprophytes

परमिचरणसहॳपरह८फाईइटसहह८ताहहॴ 2) Main plant body is diploid

पह९दह८कामखयिरीरदपवगखणतहह८ताहहॴ 3) Seeds are present

बीजमह९जदहह८तहॳहहॴ 4)Flowers are absent

फिअनपनसतिहह८तहॳहहॴ

Correct Answer Seeds are present

Q132 The largest dolphin species is the

orca also called as

िॉिकफनकीसबसहॳबड़ीपरजानतकाकानामआकायहहॴनजसहॳ mdash- भीकहतहॳहहॴ 23-Jan-2017

Options

1) Bottle Nose

बाटिनह८ज

2) Baiji

बहॳजी 3) Killer whale

ककिरहहॳि

4)Tucuxi

टकवसी Correct Answer Killer whale

Q133 The fat digesting enzyme Lipase

is secreted by which of the following

वसाकापाचनकरनहॳवािाएजाइमिाइपहॳजनननननलिखितमसहॳककसकहॳ दवारासतरापवतहह८ताहहॴ

24-Jan-2017

Options

1) Kidneys

गद

2) Pancreas

अगनयािय

3) Large Intestine

बड़ीआत

4)Liver

नजगर

Correct Answer Pancreas

Lipase is an enzyme that splits fats so

the intestines can absorb them Lipase

hydrolyzes fats like triglycerides into

their component fatty acid and glycerol

molecules It is found in the blood

gastric juices pancreatic secretions

intestinal juices and adipose tissues

F A C E B O O K

P A G E h t t p w w w f a c e b o o k c o m s s c m e n t o r s o f f i c i a l P a g e | 32

FOR MORE UPDATES AND MORE MATERIAL DO LIKE OUR FACEBOOK PAGE httpwwwfacebookcomsscmentorsofficial

Q134 The arrangement of leaves on an

axis or stem is called

एकअकषयातनहॳपरपनततयोकीयवसिाकह८कयाकहाजाताहहॴ SSC CHSL Science (biology) 2016

Question Paper

24-Jan-2017

Options

1) Phyllotaxy

फाइिह८टहॴकसी 2) Vernation

वनिन

3) Venation

वहॳनहॳिन

4)Phytotaxy

फाइटह८टहॴकसी Correct Answer Phyllotaxy

In botany phyllotaxis or phyllotaxy is

the arrangement of leaves on a plant

stem (from Ancient Greek phyacutellon

ldquoleafrdquo and taacutexis ldquoarrangementrdquo)

Phyllotactic spirals form a distinctive

class of patterns in nature

Q135 The study of Cells is also known

as

कह८लिकाओकहॳ अधययनकह८ mdashmdashndash

भीकहाजाताहहॴ 24-Jan-2017

Options

1) Cytology

सायटह८िह८जी 2) Physiology

कफनजयह८िह८जी 3) Nucleology

नयककमयह८िह८जी 4)Cellology

सहॳिह८िह८जी Correct Answer Cytology

Q136 Which of the following scientists

is also known as the Father of Biology

नननननलिखितमसहॳककसवहॴजञाननककह८ ldquoजीवपवजञानकहॳ जनकrdquoकहॳ नामसहॳभीजानाजाताहहॴ 24-Jan-2017

Options

1) Herbert Spencer

हबयटयसपसर

2) Aristotle

अरसत 3) Lamarck

िहॳमाकय 4)Darwin

िापवयन

Correct Answer Aristotle

Q137 Which cells give rise to various

organs of the plant and keep the plant

growing

कह९नसीकह८लिकाएपह९धह८कहॳ लभननअगह८कह८जनमदहॳतीहहॴऔरपह९धह८कह८बढ़नहॳममददकरतीहहॴ

24-Jan-2017

Options

1) Permanent

सिायी 2) Dermal

तवचीय

3) Meristematic

मररसटहॳमटटक

4)Mature

परह८ढ़

Correct Answer Meristematic

A meristem is the tissue in most plants

containing undifferentiated cells

(meristematic cells) found in zones of

the plant where growth can take place

Q138 Rodentia Muridae is the scientific

name of

F A C E B O O K

P A G E h t t p w w w f a c e b o o k c o m s s c m e n t o r s o f f i c i a l P a g e | 33

FOR MORE UPDATES AND MORE MATERIAL DO LIKE OUR FACEBOOK PAGE httpwwwfacebookcomsscmentorsofficial

रह८िहॳलियानयररिी mdashmdash- कावहॴजञाननकनामहहॴ 24-

Jan-2017

Options

1) Mouse

चहा 2) Squirrel

चगिहरी 3) Monkey

बदर

4) Lizard

नछपकिी Correct Answer Mouse

Q139 Name the scientist who proposed

the cell theory

कह८लिकालसदातकापरसतावदहॳनहॳवािहॳवहॴजञाननककानामबताइए 24-Jan-2017

Options

1) Schleiden and Schwann

िीमिनऔरशरववान

2) Lamarck

िहॳमाकय 3) Treviranus

टरहॳवायरहॳनस

4)Whittaker and Stanley

हीटकरऔरसटहॳनिहॳ Correct Answer Schleiden and

Schwann

Q140 The flower with the worldrsquos

largest bloom is

दननयाकासबसहॳबड़ाफिखििनहॳवािा mdashmdashndash हहॴ 24-Jan-2017

Options

1) Pando

पािह८ 2) Posidonia

पह८सीिह८ननया 3) Rafflesia arnoldii

ररफिहॳलियाअनोमिी 4)Helianthus annuus

हहॳलिएनिसएनयअस

Correct Answer Rafflesia arnoldii

Rafflesia arnoldii is a species of

flowering plant in the parasitic genus

Rafflesia It is noted for producing the

largest individual flower on earth It has

a very strong and horrible odour of

decaying flesh earning it the nickname

ldquocorpse flower

Q141 Deficiency of which vitamin

causes night blindness

ककसपवटालमनकीकमीकहॳ कारणरतौधीहह८ताहहॴ 24-Jan-2017

Options

1) Vitamin K

पवटालमन K

2) Vitamin C

पवटालमन C

3) Vitamin B1

पवटालमन B1

4)Vitamin A

पवटालमन A

Correct Answer Vitamin A

Q142 Nongreen plants lack which of the

following

गहॴर-

हररतवनसपनतमनननननलिखितमसहॳककसकीकमीहह८तीहहॴ

24-Jan-2017

Options

1) Chlorophyll

किह८रह८कफि

2) Lycophyll

िायकह८कफि

3) Cyanophyll

F A C E B O O K

P A G E h t t p w w w f a c e b o o k c o m s s c m e n t o r s o f f i c i a l P a g e | 34

FOR MORE UPDATES AND MORE MATERIAL DO LIKE OUR FACEBOOK PAGE httpwwwfacebookcomsscmentorsofficial

सायनह८कफि

4)Phototropism

फह८टह८टरोपपजम

Correct Answer Chlorophyll

Q143 Organisms that use light to

prepare food are known as

जह८जीवपरकािकाउपयह८गकरभह८जनतहॴयारकरतहॳहहॴ उनह mdashmdash- कहॳ पमजानजाताहहॴ 24-Jan-2017

Options

1) Autotrophs

सवपह८षी 2) Heterotrophs

पवषमपह८षज

3) Omnivores

सवायहारी 4)Decomposers

पवघटनकरनहॳवािा Correct Answer Autotrophs

autotrophs often make their own food

by using sunlight carbon dioxide and

water to form sugars which they can use

for energy Some examples of

autotrophs include plants algae and

even some bacteria Autotrophs

(producer) are important because they

are a food source for heterotrophs

(consumers)

A heterotroph is an organism that

ingests or absorbs organic carbon

(rather than fix carbon from inorganic

sources such as carbon dioxide) in order

to be able to produce energy and

synthesize compounds to maintain its

life Ninety-five percent or more of all

types of living organisms are

heterotrophic including all animals and

fungi and some bacteria

Q144 Which of the following is a

primary function of haemoglobin

नननननलिखितमसहॳकह९नसाटहमह८गिह८बबनकाएकपरािलमककाययहहॴ

25-Jan-2017

Options

1) Utilization of energy

उजायकाउपयह८गकरना 2) Prevention of anaemia

रकतामपताहह८नहॳसहॳरह८कना 3) Destruction of bacteria

बहॴकटीररयाकापवनािकरना 4) To transport oxygen

ऑकसीजनकावहनकरना Correct Answer To transport oxygen

Q145 Vascular bundles are absent in

सवहनीबिि mdashmdash- मअनपनसतिरहतहॳहहॴ 25-Jan-2017

Options

1) Bryophyta

िायह८फाइटा 2) Pteridophyta

टहॳररिह८फाईटा 3) Gymnosperms

नजननह८सपमय 4) Angiosperms

एननजयह८सपहॳनसय Correct Answer Bryophyta

Q146 Sauria Lacertidae is the scientific

name of

सहॴररयािहॳसरटाईिी mdashmdashndash कावहॴजञाननकनामहहॴ 25-Jan-2017

Options

1) Crocodile

मगरमचछ

2) Hippopotamus

टहपपह८पह८टहॳमस

3) Lizard

नछपकिी 4) House fly

F A C E B O O K

P A G E h t t p w w w f a c e b o o k c o m s s c m e n t o r s o f f i c i a l P a g e | 35

FOR MORE UPDATES AND MORE MATERIAL DO LIKE OUR FACEBOOK PAGE httpwwwfacebookcomsscmentorsofficial

घरहॳिमकिी Correct Answer Lizard

Q147 Which type of pathogen causes

the water-borne disease SARS (Severe

Acute Respiratory Syndrome)

ककसपरकािकारह८गज़नकजिजननतबीमारीसासयकाकारणबनताहहॴ 25-Jan-2017

Options

1) Viral

वायरि

2) Parasitic

परजीवी 3) Protozoan

परह८टह८जअन

4) Bacterial

बहॴकटीररयि

Correct Answer Viral

Q148 Which of the following organs

produces the enzyme lipase

नननननलिखितमसहॳकह९नसाअगिायपहॳजएजाइमउतपननकरताहहॴ 25-Jan-2017

Options

1) Pancreas

अगनयािय

2) Large Intestine

बड़ीआत

3) Liver

नजगर

4) Small Intestine

छह८टीआत

Correct Answer Pancreas

Q149 A is a long internode forming the

basal part or the whole of a peduncle

एक mdashmdash- एकिबाइटरनह८िहहॴ जह८ननचिाटहससायासनपणयिठिबनताहहॴ 25-

Jan-2017

Options

1) Rhizome

परकद

2) Rachis

महॳ दि

3) floral axis

पषपअकष

4) Scape

भगदड़

Correct Answer scape

Q150 ndash Which of the following

organisms are considered to be both

Living and Non-living

नननननलिखितमसहॳकह९नसहॳजीवाणकह८जीपवतऔरअजीपवतमानाजाताहहॴ

25-Jan-2017

Options

1) Bacteria

बहॴकटीररया 2) Fungi

कवक

3) Algae

िहॴवाि

4)Virus

वायरस

Correct Answer Virus

They are considered to be living as they

possess a protein coat as a protective

covering DNA as the genetic material

etc

They are said to be non-living as they

can be crystallised and they survive for

billions of years They can tolerate high

temperatures freezing cold

temperatures ultra-violet radiations etc

Q151 Deficiency of fluorine causes

which of the following

फिह८ररनकीकमीकहॳ कारणनननननलिखितमसहॳकयाहह८ताहहॴ

F A C E B O O K

P A G E h t t p w w w f a c e b o o k c o m s s c m e n t o r s o f f i c i a l P a g e | 36

FOR MORE UPDATES AND MORE MATERIAL DO LIKE OUR FACEBOOK PAGE httpwwwfacebookcomsscmentorsofficial

27-Jan-2017

Options

1) Dental Caries

िटिकहॴ ररज

2) Scurvy

सकवरी 3) Anaemia

रकतामपता 4) Arthritis

गटठया Correct Answer Dental Caries

Q152 In a Punnett Square with the

cross AaBb x AaBb how many Aabb

genotypes would be created

पनहॳटसककायरमिह८स AaBb x AaBb कहॳ साि

ककतनहॳ Aabb जीनह८टाइपबनगहॳ 27-Jan-2017

Options

1) 1

2) 8

3) 2

4) 3

Correct Answer 2

Q153 Which of the following is the

Controlling Center of the Cell

नननननलिखित म सहॳ कह८लिकाका ननयतरण

क दर कह९न हहॴ

27-Jan-2017

Options

1) Nucleus

क दर

2) Plasma

पिाजमा 3) Lysosome

िायसह८सह८म

4) Chromosome

िह८मह८सह८म

Correct Answer Nucleus

The control centre of the cell is the

nucleus in eukaryotic cells The nucleus

contains genetic material in the form of

DNA

Q154 Myopia affects which of the

following organs

मायह८पपयानननननलिखितअगह८मसहॳककसहॳपरभापवतकरताहहॴ

25-Jan-2017

Options

1) Heart

हदय

2) Skin

तवचा 3) Eyes

आािहॳ 4)Mouth

मह

Correct Answer Eyes

Q155 Which of the following bears

flowers

नननननलिखितमसहॳकह९नफिधारणकरताहहॴ

25-Jan-2017

Options

1) Bryophyta

िायह८फाइटा 2) Pteridophyta

टहॳरीिह८फाईटा 3) Gymnosperms

नजननह८सपमय 4)Angiosperms

एननजयह८सपमय Correct Answer Angiosperms

Q156 Oxygenated blood flows out of the

heart through the

ऑकसीजनयकतरकत mdashmdashmdash

कहॳ माधयमसहॳहदयकहॳ बाहरबहताहहॴ 25-Jan-2017

F A C E B O O K

P A G E h t t p w w w f a c e b o o k c o m s s c m e n t o r s o f f i c i a l P a g e | 37

FOR MORE UPDATES AND MORE MATERIAL DO LIKE OUR FACEBOOK PAGE httpwwwfacebookcomsscmentorsofficial

Options

1) Aorta

महाधमनी 2) pulmonary artery

फहॳ फड़हॳकीधमनी 3) vena cava

वहॳनाकावा 4)Atrium

चह९क

Correct Answer aorta

Q157 Blood leaving the liver and

moving towards the

heart has a higher concentration of

नजगरसहॳननकिकरहदयकीतरफजानहॳवािहॳरकतम mdashmdashmdashmdash कीउचचसादरताहह८तीहहॴ 27-Jan-2017

Options

1) Lipids

लिपपडस

2) Urea

यररया 3) Bile Pigments

पपततकहॳ रगकरण

4) Carbon dioxide

काबयनिायऑकसाइि

Correct Answer Bile Pigments

Urea is nitrogen containing substance

which is produced in the liver in order

to deal with excess amino-acids in the

body As urea is produced it leaves the

liver in the blood stream and passes via

the circulatory system to all parts of the

body

Q158 Bulb is a modification of which

part of a plant

बमबएकपह९धहॳकहॳ ककसटहससहॳकाएक पातरणहह८ताहहॴ 27-Jan-2017

Options

1) The root

जड़

2) The stem

तना 3) The radicle

मिाकर

4)The fruit

फि

Correct Answer The stem

Q159 Which of the following carries

blood away from the heart to different

body parts

इनमहॳसहॳकह९नरकतकह८हदयसहॳिरीरकहॳ पवलभननअगह८तकिहॳजातीहहॴ

27-Jan-2017

Options

1) Arteries

धमननया 2) Nerves

तबतरहाए

3) Capillaries

कहॳ लिकाए

4)Veins

नसहॳ Correct Answer Arteries

Q160 The series of processes by which

nitrogen and its compounds are

interconverted in the environment and

in living organisms is called

27-Jan-2017

Options

1)Absorption of Nitrogen

2)Ammonification

3)Nitrogen Fixation

4)Nitrogen Cycle

Correct Answer Nitrogen Cycle

Ammonification or Mineralization is

performed by bacteria to convert

organic nitrogen to ammonia

F A C E B O O K

P A G E h t t p w w w f a c e b o o k c o m s s c m e n t o r s o f f i c i a l P a g e | 38

FOR MORE UPDATES AND MORE MATERIAL DO LIKE OUR FACEBOOK PAGE httpwwwfacebookcomsscmentorsofficial

Nitrification can then occur to convert

the ammonium to nitrite and nitrate

Nitrogen fixation is a process by which

nitrogen in the Earthrsquos atmosphere is

converted into ammonia (NH3) or other

molecules available to living organisms

Q161 BCG vaccine is given to protect

from which of the following

बीसीजीकाटटकानननननलिखितमसहॳककसकहॳ बचावकहॳ लिएटदयाजातहहॴ

27-Jan-2017

Options

1) Jaundice

पीलिया 2) Anaemia

रकतमपता 3) Tuberculosis

कषयरह८ग

4) Polio

पह८लियह८ Correct Answer Tuberculosis

Q162 Parallel venation is found in

समानतरवहॳनहॳिन mdashmdashmdash- मपायाजाताहहॴ 27-Jan-2017

Options

1) plants which are monocots

पह९धहॳजह८एकबीजपतरीहह८तहॳहहॴ 2) plants which have a dicot stem

वहॳपह९धहॳनजनकातनादपवदलियहह८ताहहॴ 3) plants with leaves similar to Tulsi

वहॳपह९धहॳनजनकीपनततयतिसीकीपनततयोकहॳ समानहह८तहॳहहॴ 4)plants with tap roots

टहॳप टवािहॳपह९धहॳ Correct Answer plants which are

monocots

Q163 The hardest part of the body is

िरीरकासबसहॳकठह८रभाग mdashndash हहॴ 27-Jan-2017

Options

1) Bones

हडडिय

2) Tooth Enamel

दातकहॳ इनहॳमि

3) Skull

िह८पड़ी 4) Spinal Cord

महॳ रजज

Correct Answer Tooth Enamel

Q164 Which type of pathogen causes

the waterborne disease E coli Infection

ककसपरकारकारह८गजननकजिजननतरह८गईकह८िाईसिमणकाकारणबनताहहॴ 27-Jan-2017

Options

1) Protozoan

परह८टह८जआ

2) Parasitic

परजीवी 3) Bacterial

बहॴकटीररयि

4)Viral

वायरि

Correct Answer Bacterial

Q165 The amount of blood filtered

together by both the kidneys in a 70 kg

adult male human in a minute is

70 की गरा वािहॳएकवयसकप षमएकलमनटमदह८नोगदकहॳदवाराएकसािचाबनीगयीरकतकीमातरहह८तीहहॴ 29-Jan-2017

Options

1) 1100 ml

1100 लमलि

2) 100 ml

F A C E B O O K

P A G E h t t p w w w f a c e b o o k c o m s s c m e n t o r s o f f i c i a l P a g e | 39

FOR MORE UPDATES AND MORE MATERIAL DO LIKE OUR FACEBOOK PAGE httpwwwfacebookcomsscmentorsofficial

100 लमलि

3) 1500 ml

1500 लमलि

4) 500 ml

500 लमलि

Correct Answer 1100 ml

Q166 Which feature of a plant helps to

distinguish a monocot from a dicot

पह९धहॳकीवहकह९नसीपविहॳषताहहॴजह८एकदपवदलियहॳऔरएकएकदिीयपह९धहॳसहॳभहॳदकरनहॳममददकरतीहहॴ 29-Jan-2017

Options

1) Pollination

परागम

2) Venation

वहॳनहॳिन

3) Vernation

वनिन

4) Aestivation

एसटीवहॳिहॳन

Correct Answer venation

Q167 The Mutation Theory was

proposed by

उतवररवतयनकालसदात mdashmdashndash

कहॳ दवरापरसतापवतककयाजाताहहॴ 29-Jan-2017

Options

1) Charles Lyell

चामसयलियहॳि

2) William Smith

पवलियमनसमि

3) Hugo De Vries

हयगह८िीराईस

4)Harrison Schmitt

हहॳरीसननसमट

Correct Answer Hugo De Vries

Q168 Which type of pathogen causes

the waterborne disease HepatitisA

ककसपरकारकहॳ रह८गजनकजिजननतरह८गहहॳपहॳटाइटटस-A काकारणबनताहहॴ

29-Jan-2017

Options

1) Parasitic

परजीवी 2) Viral

वायरि

3) Protozoan

परह८टह८जआ

4) Bacterial

बहॴकटीररयि

Correct Answer Viral

Q169 In a Punnett Square with the

cross AaBb x Aabb how many AaBb

genotypes would be created

पनहॳटसकवायरमिह८स AaBb x Aabb

कहॳ सािककतनहॳ AaBb जीनह८टाइपबनगहॳ 29-Jan-

2017

Options

1) 4

2) 1

3) 7

4) 6

Correct Answer 4

Q170 Arboreal Ateles is the scientific

name of

अिह८ररयिएटटलिस mdashmdashmdash कावहॴजञाननकनामहहॴ 29-Jan-2017

Options

1) Squirrel

चगिहरी 2) Sparrow

गह८रहॴया 3) Lizard

नछपकिी 4) Spider monkey

F A C E B O O K

P A G E h t t p w w w f a c e b o o k c o m s s c m e n t o r s o f f i c i a l P a g e | 40

FOR MORE UPDATES AND MORE MATERIAL DO LIKE OUR FACEBOOK PAGE httpwwwfacebookcomsscmentorsofficial

मकड़ीबदर

Correct Answer Spider monkey

Q171 Which type of pathogen causes

the waterborne disease Salmonellosis

ककसपरकारकारह८गाणजिजननतबीमारीसािमह८नहॳिह८लसज़काकारकहहॴ

29-Jan-2017

Options

1) Algal

िहॳवालियहॳ 2) Parasitic

परजीवी 3) Bacterial

बहॴकटीररयि

4)Viral

वायरि

Correct Answer Bacterial

An infection with salmonella bacteria

commonly caused by contaminated food

or water

Symptoms include diarrhoea fever

chills and abdominal pain

Q172 is a condition in which there is a

deficiency of red cells or of haemoglobin

in the blood

mdashmdash-

एकनसिनतहहॴनजसमहॳरकतमिािकह८लिकाओकीयाहीमह८गिह८बबनकीकमीहह८तीहहॴ 29-Jan-2017

Options

1) Albinism

एनमबननजम

2) Propyria

परह८पीररया 3) Anaemia

एनीलमया 4)Keloid disorder

कहॳ िह८इिडिसओिर

Correct Answer Anaemia

Q173 Ananas comosus is the scientific

name of

Options

अनानासकह८मह८सस mdashmdashmdashndash

कावहॴजञाननकनामहहॴ 29-Jan-2017

1) Custard Apple

सीताफि

2) Pineapple

पाइनएपपि

3) Bamboo

बास

4)Pomegranate

अनार

Correct Answer Pineapple

Q174 Which organ produces insulin

कह९नसाअगइनसलिनपहॴदाकरताहहॴ 29-Jan-

2017

Options

1) Liver

यकत

2) Thyroid gland

िायराइिगरिी 3) Spleen

पिीहा 4)Pancreas

अगरयिय

Correct Answer Pancreas

Q175 Which of the following disease is

not caused by water pollution

नननननलिखितमसहॳकह९नसारह८गपानीकहॳ परदषणकहॳकारणनहीहह८ता

29-Jan-2017

Options

1) Cholera

हहॴजा 2) Typhoid

F A C E B O O K

P A G E h t t p w w w f a c e b o o k c o m s s c m e n t o r s o f f i c i a l P a g e | 41

FOR MORE UPDATES AND MORE MATERIAL DO LIKE OUR FACEBOOK PAGE httpwwwfacebookcomsscmentorsofficial

टाइफाइि

3) Asthma

दमा 4)Diarrhoea

दसत

Correct Answer Asthma

Q176 Ocimum tenuiflorum is the

scientific name of

ओलिलममटहॳयईफिह८रमइसकावहॴजञाननकनाम mdash

ndash हहॴ 30-Jan-2017

Options

1) Neem

नीम

2) Mango

आम

3) Babul

बबि

4)Tulsi

तिसी Correct Answer Tulsi

Q177 Which gland secretes bile a

digestive fluid

कह९नसीगरिीपपतत एकपाचनतरिपरदािय सरापवतकरतीहहॴ 30-Jan-2017

Options

1) Pancreas

अगनयािय

2) Liver

यकत

3) Thyroid

िायराइि

4) Testes

टहॳनसटस

Correct Answer liver

Q178 In which of the following the

dominant phase is Gametophyte

नननननलिखितमसहॳककसकहॳ परमिचरणयगमकह८दपवधद (Gametophyte)हहॴ 30-Jan-2017

Options

1) Bryophyta

िायह८फाइटा 2) Pteridophyta

टहॳररिह८फाइटा 3) Gymnosperms

नजननह८सपमय 4) Angiosperms

एननजयह८सपमय Correct Answer Bryophyta

Q179 Anaerobic respiration refers to

which of the following

नननननलिखितमसहॳककसहॳअवायवीयशवसनकहाजाताहहॴ

30-Jan-2017

Options

1) Respiration without Oxygen

ऑकसीजनकहॳ बबनाशवसन

2) Respiration with Oxygen

ऑकसीजनकहॳ सािशवसन

3) Respiration without CO2

काबयनिायऑकसाइिकहॳ बबनाशवसन

4) Respiration with CO2

काबयनिायऑकसाइिकहॳ सािशविन

Correct Answer Respiration without

Oxygen

Q180 Which type of pathogen causes

the waterborne disease Cholera

ककसपरकारकारह८गजनकजिजननतरह८गहहॴजाकाकारणबनताहहॴ

30-Jan-2017

Options

1) Algal

िहॴवालियहॳ

F A C E B O O K

P A G E h t t p w w w f a c e b o o k c o m s s c m e n t o r s o f f i c i a l P a g e | 42

FOR MORE UPDATES AND MORE MATERIAL DO LIKE OUR FACEBOOK PAGE httpwwwfacebookcomsscmentorsofficial

2) Bacterial

बहॴकटीररयि

3) Protozoan

परह८टह८जआ

4) Viral

वायरि

Correct Answer Bacterial

Q181 To which class does

Oxyreductases transferases hydrolases

belong

ओकसीररिकटहॳसटरासफरहॳजहॳस

हाइडरह८िहॳसहॳसककसवगयमआतहॳहहॴ 30-Jan-2017

Options

1) Hormones

हारमोस

2) Enzymes

एजाइनस

3) Proteins

परह८टीनस

4) Vitamins

पवटालमनस

Correct Answer Enzymes

Q182 Which of the following is not true

about Gymnosperms

ननननमसहॳकह९नसीबातअनावतबीजीकहॳ बारहॳमसचनहीहहॴ 30-Jan-2017

Options

1) Dominant phase is saprophytes

परमिचरणसहॳपरह८फाइटसहह८ताहहॴ 2) Vascular bundles are absent

सवहनीबििअनपनसितहह८ताहहॴ 3) spores are heterospores

बीजाणहहॳटहॳरह८सपह८रसहह८तहॳहहॴ 4) Flowers are absent

फिअनपनसितहह८तहॳहहॴ

Correct Answer Vascular bundles are

absent

Q183 The name of first mammal clone sheep is

भहॳड़कीपरिमसतनपायीपरनत प (किह८न)

कानामहहॴ 30-Jan-2017

Options

1) Noori

नरी 2) Dolly

िॉिी 3) Louise

िसी 4)Durga

दगाय Correct Answer Dolly

Q184 Which type of pathogen causes

the water-borne disease Typhoid fever

ककसपरकारकारह८गजनकजिजननतरह८गटाइफाइिबिारकाकारणबनताहहॴ 30-Jan-2017

Options

1) Algal

िहॴवािीय

2) Parasitic

परजीवी 3) Protozoan

परह८टह८जनअन

4)Bacterial

बहॴकटीररयि

Correct Answer Bacterial

Q185 In which part of the cell are

proteins made

कह८लिकाकहॳ ककसटहससहॳमपरह८टीनबनायाजाताहहॴ

31-Jan-2017

Options

1) Reticulum

रहॳटटकिम

F A C E B O O K

P A G E h t t p w w w f a c e b o o k c o m s s c m e n t o r s o f f i c i a l P a g e | 43

FOR MORE UPDATES AND MORE MATERIAL DO LIKE OUR FACEBOOK PAGE httpwwwfacebookcomsscmentorsofficial

2) Golgi apparatus

गह८मजीएपहॳरहॳटस

3) Ribosomes

ररबह८सह८नस

4) Lysosome

िायसह८सह८नस

Correct Answer ribosomes

Proteins are produced by stringing

amino acids together in the order

specified by messenger RNA strands

that were transcribed from DNA in the

cell nucleus The process of synthesizing

a protein is called translation and it

occurs on ribosomes in the cytoplasm of

a cell

Q186 Polio is a disease caused by which

of the following

नननननलिखितमसहॳपह८लियह८कीबबमारह८हह८नहॳकाकारणकयाहहॴ

31-Jan-2017

Options

1) Bacteria

बहॴकटीररयि

2) Mosquito

मचछर

3) Virus

वायरस

4) Cockroach

नतिच हॳ Correct Answer Virus

Polio or poliomyelitis is a crippling and

potentially deadly infectious disease It

is caused by the poliovirus

Q187 ndash Hay fever is a sign of which of

the following

हहॳकफवरनननननलिखितमसहॳककसकाएकसकहॳ तहहॴ

31-Jan-2017

Options

1) Old Age

वदावसिा 2) Malnutrition

कपह८सण

3) Allergy

एिनजय 4) Over Work

अतयचधककाययकरना Correct Answer Allergy

Q188 How many chromosomes does a

human cell contain

एकमानवकह८लिकामककतनहॳगणसतरहह८तहॳहहॴ

29-Jan-2017

Options

1) 6

2) 26

3) 46

4) 66

Correct Answer 46

In humans each cell normally contains

23 pairs of chromosomes for a total of

46 Twenty-two of these pairs called

autosomes look the same in both males

and females The 23rd pair the sex

chromosomes differ between males and

females

Q189 Which of the following is not true

about Bryophyta

ननननमसहॳकह९नसीबातिायह८फाइटकहॳ बारहॳमसचनहीहहॴ 31-Jan-2017

Options

1) Dominant phase is gametophytes

परमिचरणगहॳलमतह८फाइटसहह८ताहहॴ 2) Main plant body is haploid

पह९धहॳकामखयिरीरअगखणतहह८ताहहॴ 3) Spores are homospores

बीजाणहह८मह८सफह८रसहह८तहॳहहॴ 4) Flowers are present

फिमह८जदहह८तहॳहहॴ Correct Answer Flowers are present

F A C E B O O K

P A G E h t t p w w w f a c e b o o k c o m s s c m e n t o r s o f f i c i a l P a g e | 44

FOR MORE UPDATES AND MORE MATERIAL DO LIKE OUR FACEBOOK PAGE httpwwwfacebookcomsscmentorsofficial

Q190 Which aquatic animal has

trailing tentacles

ककसजिीयजानवरकहॳ पीछहॳचिनहॳवािहॳटहॳटकिसहह८तहॳहहॴ

31-Jan-2017

Options

1) Sea horse

समदरीघह८िा 2) Corals

मगा 3) Jelly fish

जहॳिीमछिी 4) Star fish

तारामछिी Correct Answer Jelly fish

Jellyfish with its umbrella-shaped bell

and trailing tentacles

Q191 Which type of pathogen causes

the water-borne disease Poliomyelitis

(Polio)

ककसपरकारकारह८गजनकजिजननतरह८गपह८लियह८मायहॳटटस (पह८लियह८) काकारणहहॴ 31-Jan-

2017

Options

1) Parasitic

परजीवी 2) Algal

िहॴवालिय

3) Viral

वायरि

4) Bacterial

बहॴकटीररयि

Correct Answer Viral

Q192 The outer white part of the eye

that protects the inner structures is

आािकाबाहरीसफहॳ दटहससाजह८आतररकसरचनाओकीरकषाकरताहहॴ वह mdashmdashmdash हहॴ 31-Jan-

2017

Options

1) Iris

आयररस

2) Sclera

सकिहॳरा 3) Retina

रहॳटटना 4) Cornea

कह८ननयया Correct Answer Sclera

Q193 Proteins are made up of

परह८टीनकाननमायण mdashndash सहॳहह८ताहहॴ 31-Jan-2017

Options

1) Amino acids

एलमनह८अनि

2) Fatty acids

वसायकतअनि

3) Glucose

गिकह८ज

4)Nucleotides

नयनकियह८टाईिस

Correct Answer Amino acids

Q194 Moringa Oleifera is the scientific

name of

मह८ररगओलिफहॳ रा mdashmdashndash कावहॴजञाननकनामहहॴ 31-Jan-2017

Options

1) Banyan

बरगद

2) Gulmohar

गिमह८हर

3) Amla

आमिा

F A C E B O O K

P A G E h t t p w w w f a c e b o o k c o m s s c m e n t o r s o f f i c i a l P a g e | 45

FOR MORE UPDATES AND MORE MATERIAL DO LIKE OUR FACEBOOK PAGE httpwwwfacebookcomsscmentorsofficial

4) Drumstick

डरमनसटक

Correct Answer Drumstick

Q195 Kidney stones are composed of

गदकीपिरी mdashndash सहॳबनीहह८तीहहॴ 1-Feb-2017

Options

1) Calcium Oxalate

कहॴ नमसयमओकजहॳिहॳट

2) Sodium Chloride

सह८डियमकिह८राइि

3) Magnesium Nitrate

महॳनगनलियमनाइतटरहॳट

4) Calcium Bicarbonate

कहॴ नमियमबायकबोनहॳट

Correct Answer Calcium Oxalate

Q196 ndash Which of the following is not

true about Angiosperms

ननननमसहॳकह९नसीबातआवतबीजीकहॳ बारहॳमसचनहीहहॴ 1-Feb-2017

Options

1) Dominant phase is gametophytes

परमिचरणगहॳलमतह८फाइटहह८ताहहॴ 2) Vascular bundles are present

सवहनीबििमह९जदहह८ताहहॴ 3) Spores are heterospores

बीजाणहहॳटहॳरह८सपह८रसहह८तहॳहहॴ 4) Seeds are covered

बीजढकहॳ हह८तहॳहहॴ Correct Answer Dominant phase is

gametophytes

Q197 All of the following are excretory

(waste) products of animals except

नननननलिखितमसहॳककसएककह८छह८ड़करअनयसभीपराखणयोदवाराउतसनजयतपदाियहहॴ 1-Feb-

2017

Options

1) Uric Acid

यररकएलसि

2) Ammonia

अमह८ननया 3) Carbohydrates

काबोहाइडरहॳट

4) Urea

यररया Correct Answer Carbohydrates

In animals the main excretory products

are carbon dioxide ammonia (in

ammoniotelics) urea (in ureotelics) uric

acid (in uricotelics) guanine (in

Arachnida) and creatine

Q198 RNA is a polymeric molecule

What does RNA stand for

आरएनइएएकबहिकआणहहॴ इसकाकापवय पकयाहहॴ 1-Feb-2017

Options

1) Rado Nuclear Acid

रािह८नयनकियरएलसि

2) Ribo Nucleic Acid

राइबह८नयनकिकएलसि

3) Rhino Nuclear Acid

हाइनह८नयनकियरएलसि

4) Resto Nucleus Acid

रहॳसटह८नयकिीयसएलसि

Correct Answer Ribo Nucleic Acid

Q199 Which organ does detoxification

and produces chemicals needed for

digestion

कह९नसाअगपवषहरणकरताहहॴऔरपाचनकहॳ लिएआवशयकरसायनोकह८पहॴदाकरताहहॴ 1-Feb-

2017

Options

1) Salivary glands

िारगरचिया 2) Pancreas

अगनयािय

F A C E B O O K

P A G E h t t p w w w f a c e b o o k c o m s s c m e n t o r s o f f i c i a l P a g e | 46

FOR MORE UPDATES AND MORE MATERIAL DO LIKE OUR FACEBOOK PAGE httpwwwfacebookcomsscmentorsofficial

3) Thyroid gland

िायराइिगरिी 4) Liver

यकत

Correct Answer Liver

Q200 Psidium guajava is the scientific

name of

लसडियमगआजावा mdashmdash कावहॴजञाननकनामहहॴ 1-

Feb-2017

Options

1) Guava

अम द

2) Mango

आम

3) Bamboo

बास

4) Jack fruit

कटहि

Correct Answer Guava

Q201 Which drug is used as a Blood

Thinner

चधरकह८पतिाकरनहॳकहॳ पमककसदवाकापरयह८गककयाजाताहहॴ

1-Feb-2017

Options

1) Warfarin

वाफर न

2) Tramadol

टरहॳमािह८ि

3) Azithromycin

एनजरह८मायलसन

4) Hydralazine

हाइडरह८िहॳनजन

Correct Answer Warfarin

Q202 Which of the following disease is

caused due to the deficiency of protein

परह८टीनकीकमीकहॳ कारणनननननलिखितमसहॳकह९नसारह८गहह८ताहहॴ 1-Feb-2017

Options

1) Arthritis

गटठया 2) Kwashiorkor

कािीओकय र

3) Goitre

गाइटर

4) Night Blindness

रतह९चध

Correct Answer Kwashiorkor

Q203 A is species of plant that has

adapted to survive in an environment

with little liquid water

mdashmdashndashपह९धहॳकीएकऐसहॳऐसहॳपरजानतहहॴ नजसनहॳकमपानीवािहॳवातावरणमजीपवतरहनहॳकहॳलिएअनकिनहहॴ 1-Feb-2017

Options

1) Xerophyte

म दपवद

2) Hydrophyte

जिीयपादप

3) Mesophyte

समह८दपवद

4) Thallophyte

िहॴिह८फाइटा Correct Answer xerophyte

xerophyte is a species of plant that has

adapted to survive in an environment

with little liquid water such as a desert

or an ice- or snow-covered region in the

Alps or the Arctic

Mesophytes are terrestrial plants which

are adapted to neither a particularly

dry nor particularly wet environment

An example of a mesophytic habitat

would be a rural temperate meadow

F A C E B O O K

P A G E h t t p w w w f a c e b o o k c o m s s c m e n t o r s o f f i c i a l P a g e | 47

FOR MORE UPDATES AND MORE MATERIAL DO LIKE OUR FACEBOOK PAGE httpwwwfacebookcomsscmentorsofficial

which might contain goldenrod clover

oxeye daisy and Rosa multiflora

thallophyte any of a group of plants or

plantlike organisms (such as algae and

fungi) that lack differentiated stems

leaves and roots and that were formerly

classified as a primary division

(Thallophyta) of the plant kingdom

Q204 How many types of teeth are

there in humans

मनषयोमककतनहॳपरकारकहॳ दातहह८तहॳहहॴ

1-Feb-2017

Options

1) 4

2) 5

3) 2

4) 3

Correct Answer 4

teeth -Humans have four types of

teethincisors canines premolars and

molars each with a specific function

The incisors cut the food the canines

tear the food and the molars and

premolars crush the food

Q205 Carica papaya is the scientific name of

कहॴ ररकापपाया mdashmdashndash कावहॴजञाननकनामहहॴ 2-

Feb-2017

Options

1) Peepal

पीपि

2) Papaya

पपीता 3) Tamarind

इमिी 4) Drumstick

ढह८िकाछड़ी Correct Answer Papaya

Q206 Muscles get tired when there is

shortfall of

जब mdashndash कीकमीहह८तीहहॴतबपहॳिीयिकजातीहहॴ 2-Feb-2017

Options

1) Lactic acid

िहॴनकटकएलसि

2) Na+ ions

Na+ आयन

3) ATP

एटीपी 4) Sulphates

समफहॳ टस

Correct Answer ATP

ATP is the energy source muscle fibers

use to make muscles contract

muscle tissuersquos main source of energy

called adenosine triphosphate or ATP

As your muscles use up this energy

source they become tired and fatigued

Oxygen is the key ingredient that helps

create new ATP to replenish the burned

up ATP in your muscles

Q207 Artocarpus integra is the

scientific name of आटह८कापयसइटीगरा mdashmdashmdash कावहॴजञाननकनामहहॴ 2-Feb-2017

Options

1) Guava

अम द

2) Pineapple

अनानास

3) Silver Oak

लसमवरओक

4) Jack fruit

कटहि

Correct Answer Jack fruit

Q208 Which organ stores fat soluble

vitamins

कह९नसाअगवसामघिनिीिपवटालमनह८काभिाराकरताहहॴ

2-Feb-2017

F A C E B O O K

P A G E h t t p w w w f a c e b o o k c o m s s c m e n t o r s o f f i c i a l P a g e | 48

FOR MORE UPDATES AND MORE MATERIAL DO LIKE OUR FACEBOOK PAGE httpwwwfacebookcomsscmentorsofficial

Options

1) Blood

रकत

2) Skin

तवचा 3) Liver

यकत

4) Pancreas

अगनयािय

Correct Answer Liver

Q209 Which disease is caused due to

deficiency of Iodine

आयह८िीनकहॳ कारणकह९नसारह८गहह८ताहहॴ 2-Feb-2017

Options

1) Rickets

ररकहॳ टस

2) Scurvy

सकवी 3) Goitre

गणमािा 4) Growth retardation

पवकासका कना Correct Answer Goitre

rickets A softening and weakening of

bones in children usually due to

inadequate vitamin D

Q210 Grevillea Robusta is the scientific name of

गरहॳपवलियारह८बसटा mdashmdashmdash- कापवजञाननकनामहहॴ 2-Feb-2017

Options

1) Peepal

पीपि

2) Teak

सागह९न

3) Silver Oak

लसमवरओक

4) Jack fruit

कटहि

Correct Answer Silver Oak

Q211 When a Cuttlefish is described as a Molluscs it is at which level of

classification

जबएककटिकफिकह८एकमह८िसकाकहॳ पमवखणयतककयाजाताहहॴतबयहॳवगीकरणकहॳ ककससतरपहॳनसितहहॴ 2-Feb-2017

Options

1) Class

वगय 2) Order

िम

3) Family

पररवार

4) Phylum

सघ

Correct Answer Phylum

Q212 Bambusa dendrocalmus is the

scientific name of बानबसािहॳडराकामस mdashmdashmdash कावहॴजञाननकनामहहॴ 3-Feb-2017

Options

1) Banyan

बरगद

2) Papaya

पपीता 3) Bamboo

बास

4) Pomegranate

अनार

Correct Answer Bamboo

Q213 Acinonyx Jubatus is the scientific name of

एलसनह८ननकसजयबहॳटस mdashmdashmdash

कावहॴजञाननकनामहहॴ 3-Feb-2017

F A C E B O O K

P A G E h t t p w w w f a c e b o o k c o m s s c m e n t o r s o f f i c i a l P a g e | 49

FOR MORE UPDATES AND MORE MATERIAL DO LIKE OUR FACEBOOK PAGE httpwwwfacebookcomsscmentorsofficial

Options

1) Bear

भाि 2) Horse

घह८िा 3) Cheetah

चीता 4) Zebra

जहॳिा Correct Answer Cheetah

Q214 The pale yellow colour of urine is

due to the presence of which pigment

मतरकाफीकापीिारगरगदरयकहॳ उपनसिनतकहॳ कारणहह८ताहहॴ

3-Feb-2017

Options

1) Urochrome

यरह८िह८म

2) Urophyll

यरह८कफि

3) Chlorophyll

किह८रह८कफि

4) Chloroplast

किह८रह८पिासट

Correct Answer Urochrome

Q215 Which of the following constitute

to form a gene

नननननलिखितमसहॳकह९नसीचीज़एकजीनकागठनकरतीहहॴ

3-Feb-2017

Options

1) Polynucleotides

पह८िीनयनकियह८टाईडस

2) Hydrocarbons

हाइडरह८काबोस

3) Lipoproteins

िाईपह८परह८टीनस

4) Lipids

लिपपडस

Correct Answer Polynucleotides

Polynucleotide molecule is a biopolymer

composed of 13 or more nucleotide

monomers covalently bonded in a chain

DNA (deoxyribonucleic acid) and RNA

(ribonucleic acid) are examples of

polynucleotides with distinct biological

function

Q216 Vertebrates belongs to the

phylum

रीढ़कीहडिीवािहॳपराणी mdashmdashmdash

परजानतकहॳ अतगायतआतहॳहहॴ 3-Feb-2017

Options

1) Arthropoda

आरह८पह८ड़ा 2) Annelida

एननलििा 3) Cnidaria

ननिहॳररया 4) Chordata

कह८िटा Correct Answer Chordata

Q217 Punica granatum is the scientific name of

पननकगरहॳनहॳटस mdashmdashmdash कावहॴजञाननकनामहहॴ 3-Feb-2017

Options

1) Custard Apple

सीताफि

2) Gulmohar

गिमह८हर

3) Silver Oak

लसमवरओक

4) Pomegranate

अनार

Correct Answer Pomegranate

F A C E B O O K

P A G E h t t p w w w f a c e b o o k c o m s s c m e n t o r s o f f i c i a l P a g e | 50

FOR MORE UPDATES AND MORE MATERIAL DO LIKE OUR FACEBOOK PAGE httpwwwfacebookcomsscmentorsofficial

Q218 Between a tiger and an monkey

which of the following is different

एकबाघऔरबदरकहॳ बीचनननननलिखितमसहॳकह९नसीबातअिगहहॴ 3-Feb-2017

Options

1) Kingdom

राजय

2) Phylum

जानत

3) Order

िम

4) Class

वगय Correct Answer order

Q219 The artificial heart was invented by

कबतरमहदयका mdashmdashmdash

दवाराअपवषकारककयागयािा 3-Feb-2017

Options

1) Muhammad Yunus

महनमदयनस

2) Linus Yale Jr

िाइनसयहॳिजय

3) Gazi Yasargil

गाजीयासचगयि

4) Paul Winchell

पह९िपवमकि Correct Answer Paul Winchell

Q220 Tamarindus indica is the

scientific name of

टहॳमररनडसइडिका mdashmdash कावहॴजञाननकनामहहॴ 7-

Feb-2017

Options

1) Neem

नीम

2) Pineapple

अनानास

3) Tamarind

इमिी 4)Chiku

चीक

Correct Answer Tamarind

Q221 In eukaryotic cells synthesis of

RNA takes place in the

यकहॳ योटटककह८लिकाओमआरएनएकासशिहॳषण

mdashndash महह८ताहहॴ 7-Feb-2017

Options

1) Mitochondria

माईटह८कोडडरया 2) Centrioles

सटरीयह८मस

3) Ribosomes

ररबह८सह८नस

4) Nucleus

नयनकियस

Correct Answer nucleus

eukaryotic cell -Transcription is the

process of synthesizing ribonucleic acid

(RNA)Synthesis takes place within the

nucleus of eukaryotic cells or in the

cytoplasm of prokaryotes and converts

the genetic code from a gene in

deoxyribonucleic acid ( DNA ) to a

strand of RNA that then directs

proteinsynthesis

Q222 _________is caused by parasites

of the Plasmodium genus

पिाजमह८डियमजातीकहॳ परजीवी mdash- कहॳ कारणहहॴ 7-Feb-2017

Options

1) Dysentery

पहॳचचि

2) Malaria

मिहॳररया 3) Chickenpox

F A C E B O O K

P A G E h t t p w w w f a c e b o o k c o m s s c m e n t o r s o f f i c i a l P a g e | 51

FOR MORE UPDATES AND MORE MATERIAL DO LIKE OUR FACEBOOK PAGE httpwwwfacebookcomsscmentorsofficial

चहॳचक

4) Herpes

हहॳपपयस

Correct Answer Malaria

Q223 Carotene in fruits and vegetables

gives it which color

फिह८औरसनलजयोमनसितकहॳ रह८टीनउनहकह९नसारगपरदानकरताहहॴ 7-Feb-2017

Options

1) Green

हरा 2) Pink

गिाबी 3) Orange

नारगी 4) Blue

नीिा Correct Answer Orange

Q224 Equus Caballus is the scientific

name of

एकवसकहॴ बहॳिस mdashmdashndash कापवजञाननकनामहहॴ 7-Feb-2017

Options

1) Horse

घह८िा 2) Zebra

ज़हॳिा 3) Donkey

गधा 4) Buffalo

भस

Correct Answer Horse

Q225 Elapidae Naja is the scientific name of

एिीपीिीनाजा mdashmdash- कावहॴजञाननकनामहहॴ 8-Feb-2017

Options

1) Cobra

कह८बरा 2) Elephant

हािी 3) Eagle

ग ि

4) Owl

उमि Correct Answer Cobra

Q226 Which disease is caused due to

deficiency of Iron

िह८हकीकमीकहॳ कारणकह९नसारह८गहह८ताहहॴ 8-Feb-

2017

Options

1) Beriberi

बहॳरीबहॳरी 2) Tetany

टहॳटनी 3) Kwashiorkor

कवािीऔरकर

4) Anaemia

रकतामपता Correct Answer Anaemia

Beriberi is a disease caused by a vitamin

B-1 deficiency also known as thiamine

deficiency

Tetany can be the result of an

electrolyte imbalance Most often itrsquos a

dramatically low calcium level also

known as hypocalcemia Tetany can also

be caused by magnesium deficiency or

too little potassium Having too much

acid (acidosis) or too much alkali

(alkalosis) in the body can also result in

tetany

Kwashiorkor also known as

ldquoedematous malnutrition It is a form of

malnutrition caused by a lack of protein

in the diet

Anaemia means that you have fewer red

blood cells than normal or you have less

F A C E B O O K

P A G E h t t p w w w f a c e b o o k c o m s s c m e n t o r s o f f i c i a l P a g e | 52

FOR MORE UPDATES AND MORE MATERIAL DO LIKE OUR FACEBOOK PAGE httpwwwfacebookcomsscmentorsofficial

haemoglobin than normal in each red

blood cell

Q227 is a leaf where the leaflets are

arranged along the middle vein

mdashndashएकपततीहहॴजहापतरकह८कीरचनाक ररयालिराकहॳ आसपासहह८तीहहॴ 8-Feb-2017

Options

1) Pinnately compound leaf

पपनहॳटिीसयकतपतती 2) Palmately compound leaf

पामहॳटिीसयकतपतती 3) Compound leaf

सयकतपतती 4) Simple leaf

साधारणपतती Correct Answer Pinnately compound

leaf

Q228 Haustoria or sucking roots are

found in which of the following

हह८सटह८ररयायाचसनहॳवािीजड़हॳनननननलिखितमसहॳककसमपाईजातीहहॴ 8-Feb-2017

Options

1) Wheat

गहॳह

2) Mango

आम

3) Chestnut

चहॳसटनट

4) Cuscuta

कसकयटा Correct Answer Cuscuta

Haustorial roots -The roots of parasitic

plants which penetrate into the host

tissues to absorb nourishment are

called haustorial roots hellip Also known as suckingor parasitic roots

Q229 Equs Asinus is the scientific name

of

एकवसएलसनस mdashmdashndash कावहॴजञाननकनामहहॴ 8-

Feb-2017

Options

1) Donkey

गधा 2) Cow

गाय

3) Deer

टहरन

4) Kangaroo

कगा

Correct Answer Donkey

Q230 Ficus benghalensis is the scientific name of

फाईकसबहॳनगहॳिहॳलसस mdashndash कापवजञाननकनामहहॴ 8-Feb-2017

Options

1) Banyan

बरगद

2) Pineapple

अनानास

3) Babul

बबि

4) Tulsi

तिसी Correct Answer Banyan

Q231 Equus burchellii is the scientific name of

एकवसबचिी mdashmdash- कापवजञाननकनामहहॴ 8-Feb-2017

Options

1) Horse

घह८िा 2) Zebra

जहॳिा 3) Buffalo

F A C E B O O K

P A G E h t t p w w w f a c e b o o k c o m s s c m e n t o r s o f f i c i a l P a g e | 53

FOR MORE UPDATES AND MORE MATERIAL DO LIKE OUR FACEBOOK PAGE httpwwwfacebookcomsscmentorsofficial

भस

4) Ass

गधा Correct Answer Zebra

Page 13: COMPILATION OF ALL 72 SETS OF BIOLOGY SSC CHSL-2016 · OF BIOLOGY SSC CHSL-2016 PREPARED BY : SSC MENTORS BIOLOGY SPECIAL . F A C E B O O K P A G E : h t t p : / / w w w . f a c e

F A C E B O O K

P A G E h t t p w w w f a c e b o o k c o m s s c m e n t o r s o f f i c i a l P a g e | 12

FOR MORE UPDATES AND MORE MATERIAL DO LIKE OUR FACEBOOK PAGE httpwwwfacebookcomsscmentorsofficial

Q47 Felis Catus is the scientific name of

फहॳ लिसकहॴ टस mdashndash कावहॴजञाननकनामहहॴ 13-Jan-2017

Options

1) Cat

बबमिी 2) Dog

कतता 3) Mouse

चहा 4) Porcupine

साही Correct Answer Cat

Q48 Which of the following induces

nitrogen fixation in soil

नननननलिखितमसहॳकह९नलम ीमनाइटरह८जनननयतनकह८परहॳररतकरताहहॴ

15-Jan-2017

Options

1) Protozoa

परह८टह८जआ

2) Bacteria

बहॴकटीररया 3) Fungi

कवक

4)Algae

िहॴवाि

Correct Answer Bacteria

Bacteria that change nitrogen gas from

the atmosphere into solid nitrogen

usable by plants are called nitrogen-

fixing bacteria These bacteria are

found both in the soil and in symbiotic

relationships with plants

They contain symbiotic bacteria called

rhizobia within nodules in their root

systems producing nitrogen compounds

that help the plant to grow and compete

with other plants When the plant dies

the fixed nitrogen is released making it

available to other plant

Q49 Which of the following is the

largest known cell

नननननलिखितमसहॳकह९नसीसबसहॳबड़ीजञातकह८लिकाहहॴ

SSC CHSL Science (biology) 2016

Question Paper

15-Jan-2017

1) Eukaryotic Cell

यकहॳ ररयह८टटककह८लिका 2) Prokaryotic Cell

परह८कहॳ ररयह८टटककह८लिका 3) Mycoplasma

मायकह८पिासम

4) Ostrich Eggs

ितरमगयकाअिा Correct Answer Ostrich Eggs

Q50 The association of animals in

which both the partners are benefitted

is known as

जानवरोकावहसहयह८गनजसमहॳदह८नोभागीदारिाभापवनतहह८तहॳहहॴ उसहॳ mdashmdashndash कहॳ पमजानाजाताहहॴ SSC CHSL Science (biology) 2016

Question Paper

15-Jan-2017

Options

1) Amensalism

सहजीपवत

2) Commensalism

परजीपवत

3) Colony

कॉिनी 4) Mutualism

अनयह८नयाशरयवाद

Correct Answer Mutualism

Amensalism is any relationship between

organisms of different species in which

F A C E B O O K

P A G E h t t p w w w f a c e b o o k c o m s s c m e n t o r s o f f i c i a l P a g e | 13

FOR MORE UPDATES AND MORE MATERIAL DO LIKE OUR FACEBOOK PAGE httpwwwfacebookcomsscmentorsofficial

one organism is inhibited or destroyed

while the other organism remains

unaffected

Commensalism an association between

two organisms in which one benefits and

the other derives neither benefit nor

harm

Q51 Pneumonia affects which of the

following organs of human body

ननमह८ननयामानविरीरकहॳ नननननलिखितमसहॳककसअगकह८परभापवतकरताहहॴ

15-Jan-2017

Options

1)Kidneys

गद

2)Lungs

फहॳ फड़हॳ 3) Throat

गिहॳ 4) Liver

यकत

Correct Answer Lungs

When the germs that cause pneumonia

reach your lungs the lungsrsquo air sacs

(alveoli) become inflamed and fill up

with fluid This causes the symptoms of

pneumonia such as a cough fever

chills and trouble breathing When you

have pneumonia oxygen may have

trouble reaching your blood

Q52 Mendel is known as

मििकह८ mdashmdash- कहॳ पमजानाजाताहहॴ 15-Jan-2017

Options

1) Father of Physiology

िरीरकियािासतरकहॳ जनक

2) Father of Geology

भगभयिासतरकहॳ जनक

3) Father of Genetics

जहॳनहॳटटकसकहॳ जनक

4) Father of Biology

जीविासतरकहॳ जनक

Correct Answer Father of Genetics

Q53 Which of the following are also

known as Suicidal bag of Cells

ननननलिखितमसहॳककसहॳआतमहतयाकरनहॳवािीकह८लिकाओकाबहॴगकहाजाताहहॴ

15-Jan-2017

Options

1) Lysosomes

िायसोसह८म

2) Lycosome

िायकह८सह८म

3) Nucleus

नालभक

4) Chromosome

िह८मह८सह८म

Correct Answer Lysosomes

Q54 Mesothelioma is a type of cancer

The most common area affected in it is

the lining of the ________

लमज़ह८िहॳिहॳलमयाक सरकाएकपरकारहहॴ इससहॳपरभापवतहह८नहॳवािासबसहॳसामानयकषहॳतर mdash

mdashmdash काअसतरहहॴ 15-Jan-2017

Options

1)Heart

हदय

2)Brain

मनसतषक

3)Stomach

आमािय

4)Lungs

फहॳ फड़हॳ Correct Answer lungs

Asbestos exposure is the main cause of

pleural mesothelioma When asbestos

fibers are breathed in they travel to the

F A C E B O O K

P A G E h t t p w w w f a c e b o o k c o m s s c m e n t o r s o f f i c i a l P a g e | 14

FOR MORE UPDATES AND MORE MATERIAL DO LIKE OUR FACEBOOK PAGE httpwwwfacebookcomsscmentorsofficial

ends of small air passages and reach the

pleura where they can cause

inflammation and

scarring

Q55 Which one of the following is an

insectivorous plant

नननननलिखितमसहॳकह९नसाएकककटाहरीवनसपनतहहॴ

15-Jan-2017

Options

1) Utricularia

यटरीकिहॳररया 2) Sequoia

सहॳकयओइया 3) Nostoc

नॉसटह८क

4) Bryophyta

िायह८फाईटा Correct Answer Utricularia

Q56 ______________ is a

multibranched polysaccharide of

glucose that serves as a form of energy

storage in animals and fungi

mdashmdashगिकह८जकाएकबहिािायकतपह८िीसहॳकहॳ राइिहहॴ जह८जानवरोऔरकवकमउजायभणिारणकहॳ एक पमकाययकरताहहॴ 15-Jan-2017

Options

1) Cellulose

सहॳमयिह८ज

2) Glycogen

गिायकह८जन

3) Pectin

पहॳनकटन

4) Chitin

चीटटन

Correct Answer Glycogen

Q57 The largest gland of the human

body is

mdashmdashmdashमानविरीरकीसबसहॳबड़ीगरिीहहॴ 16-Jan-2017

Options

1) Pancreas

अगयािय

2) Thyroid

िायरॉइि

3) Large Intestine

बड़ीआत

4) Liver

यकत

Correct Answer Liver

Q58 Photosynthesis in plants takes

place in

वनसपनतयोमपरकािसशिहॳषणकीकियाहह८तीहहॴ

16-Jan-2017

Options

1) Stem

तना 2) Leaves

पनततयाा 3) Roots

जड़हॳ 4) Flower

फि

Correct Answer Leaves

During this reaction carbon dioxide

and water are converted into glucose

and oxygen The reaction requires light

energy which is absorbed by a green

substance called

chlorophyll Photosynthesis takes place

in leaf

cells These contain chloroplasts which

are tiny objects containing chlorophyll

F A C E B O O K

P A G E h t t p w w w f a c e b o o k c o m s s c m e n t o r s o f f i c i a l P a g e | 15

FOR MORE UPDATES AND MORE MATERIAL DO LIKE OUR FACEBOOK PAGE httpwwwfacebookcomsscmentorsofficial

Q59 Insects that transmit diseases are

known as

जह८कीड़हॳरह८गसचाररतकरतहॳहहॴ उनह mdashmdash-

कहॳ नामसहॳजानाजाताहहॴ 16-Jan-2017

1)Pathogens

रह८गज़नक

2) Vectors

वहॳकटर

3) Drones

परजीवी 4)Scalars

अटदषट

Correct Answer Vectors

A vector is an organism that does not

cause disease itself but which spreads

infection by conveying pathogens from

one host to another Species of mosquito

for example serve as vectors for the

deadly disease Malaria

Q60 Which is the second largest gland

of Human body

मानविरीरकीदसरीसबसहॳबड़ीगरिीकह९नसीहहॴ

SSC CHSL Science (biology)

2016 Question Paper

16-Jan-2017

Options

1) Liver

यकत

2) Large Intestine

बड़ीआत

3) Thorax

छाती 4) Pancreas

अगनयािय

Correct Answer Pancreas

Q61 Annona squamosa is the scientific

name of

एनह८नासकवामह८सा (Annona squamosa) mdash

mdashmdash कावहॴजञाननकनामहहॴ 16-Jan-2017

Options

1) Custard Apple

सीताफि

2) Papaya

पपीता 3) Babhul

बबि

4) Drumstick

सहजन

Correct Answer Custard Apple

Q62 The disease Beri Beri is caused due

to the deficiency of which of the

following

बहॳरीबहॳरीरह८गनननननलिखितमसहॳककसकीकमीकहॳकारणहह८ताहहॴ

16-Jan-2017

Options

1) Vitamin B2

पवटालमन B2

2) Vitamin B1

पवटालमन B1

3) Vitamin B12

पवटालमन B12

4) Vitamin E

पवटालमन E

Correct Answer Vitamin B1

Beriberi is a disease caused by a vitamin

B-1 deficiency also known as thiamine

deficiency

Q63 Chlorophyll was first isolated and

named by

किह८रह८कफिकह८ mdash-

दवारापहिहॳपिकऔरनालमतककयागया 16-Jan-2017

F A C E B O O K

P A G E h t t p w w w f a c e b o o k c o m s s c m e n t o r s o f f i c i a l P a g e | 16

FOR MORE UPDATES AND MORE MATERIAL DO LIKE OUR FACEBOOK PAGE httpwwwfacebookcomsscmentorsofficial

Options

1) Caventou

कहॳ वहॳत 2) Pelletier

पहॳिहॳटटयर

3) Chlorophyll

किह८रह८कफि

4) Caventou and Pelletier

कहॳ वहॳतऔरपहॳिहॳटटयर

Correct Answer Caventou and Pelletier

Chlorophyll was first isolated and

named by

Joseph Bienaimeacute Caventou and Pierre

Joseph Pelletier in 1817 The presence of

magnesium in chlorophyll was

discovered in 1906 and was the first

time that magnesium had been detected

in living tissue

Q64 Which of the following organisms

does not fit into the Cell Theory

नननननलिखितमसहॳकह९नसाजीवकह८लिकालसदातअन पनहीहहॴ

16-Jan-2017

Options

1) Bacteria

बहॴकटीररया 2) Virus

वायरस

3) Fungi

कवक

4) Plants

पह९धहॳ Correct Answer Virus

The bottom line is that viruses are not

alive and not related to cells in any way

The cell theory states that all living

things are made of cells cells are the

basic units of structure and function of

living things and that all cells come

from other cells Since viruses are not

made of cells and do not use cells in any

of their processes they are not related to

the cell theory

Q65 Which of these is not a

macronutrient for Plants

नननननलिखितमसहॳकह९नसापह९धह८कहॳ लिएमिह८नयटरीएटनहीहहॴ

SSC CHSL Science (biology) 2016

Question Paper

17-Jan-2017

Options

1) Nitrogen

नाइटरह८जन

2) Phosphorus

फासफह८रस

3) Potassium

पह८टालसयम

4) Chlorine

किह८रीन

Correct Answer Chlorine

In relatively large amounts the soil

supplies nitrogen phosphorus

potassium calcium magnesium and

sulfur these are often called the

macronutrients In relatively small

amounts the soil supplies iron

manganese boron molybdenum

copper zinc chlorine and cobalt the

so-called micronutrients

Q66 Name the respiratory organs of

insects

कीटह८मनसतिशरवसनअगनामकानामहहॴ

17-Jan-2017

Options

1) Skin

तवचा 2) Body Surface

िरीरकीसतह

F A C E B O O K

P A G E h t t p w w w f a c e b o o k c o m s s c m e n t o r s o f f i c i a l P a g e | 17

FOR MORE UPDATES AND MORE MATERIAL DO LIKE OUR FACEBOOK PAGE httpwwwfacebookcomsscmentorsofficial

3) Gills

गिफड़हॳ 4) Tracheae

शरावस- निी Correct Answer Tracheae

Air enters the respiratory systems of

insects through a series of external

openings called

spiracles These external openings

which act as muscular valves in some

insects lead to the internal respiratory

system a densely networked array of

tubes called tracheae

Q67 The poisonous gas accidentally

released in Bhopal Gas Tragedy is

भह८पािगहॴसतरासदीमगितीसहॳमकतहईजहरीिीगहॴसिी

17-Jan-2017

1) Methane

मीिहॳन

2) Nitrous Oxide

नाइटरसऑकसाइि

3) Methyl Isocyanate

महॴचििआयसोसायनहॳट

4) Cyanogen

सायनह८जहॳन

Correct Answer Methyl Isocyanate

Q68 What does Trypsin do

टटरनपसनकयाकरताहहॴ

SSC CHSL Science (biology) 2016

Question Paper

17-Jan-2017

Options

1) Breaks down Carbohydrates

काबोहाइडरहॳटकापवघटनकरताहहॴ 2) Synthesizes proteins

परह८टीनकासििहॳषणकरताहहॴ 3) Breaks down fats

वसाकापवघटनकरताहहॴ 4) Breaks down proteins

परह८टीनकापवघटनकरताहहॴ Correct Answer Breaks down proteins

Trypsin is one of the three principal

digestive

proteinases the other two being pepsin

and

chymotrypsin In the digestive process

trypsin acts with the other proteinases

to break down dietary protein molecules

to their component

peptides and amino acids

A protease is any enzyme that performs

proteolysis protein catabolism by

hydrolysis of peptide bonds

Q69 Name the source from which

Aspirin is produced

उससरह८तकानामबताइए

नजससहॳएनसपररनकाउतपादनककयाजाताहहॴ

17-Jan-2017

Options

1) Willow bark

पविह८कीछाि

2) Oak Tree

ओककावकष

3) Acacia

बबि

4) Eucalyptus

नीिचगरी Correct Answer Willow bark

The compound from which the active

ingredient in aspirin was first derived

salicylic acid was found in the bark of a

willow tree in 1763 by Reverend

Edmund Stone of Chipping-Norton

Q70 Cannis Familiaris is the scientific

name of

कहॴ ननसफहॳ लमलियहॳररस mdash- कावहॴजञाननकनामहहॴ

17-Jan-2017

F A C E B O O K

P A G E h t t p w w w f a c e b o o k c o m s s c m e n t o r s o f f i c i a l P a g e | 18

FOR MORE UPDATES AND MORE MATERIAL DO LIKE OUR FACEBOOK PAGE httpwwwfacebookcomsscmentorsofficial

Options

1) Cat

बबमिी 2)Dog

कतता 3) Fox

िह८मड़ी 4) Wolf

भहॳडड़या Correct Answer Dog

Q71 Harmful bacteria in potable water

make the water

पीनहॳकहॳ पानीमनसतिघातकबहॴकटीररयाउसपानीकह८बनातहॳहहॴ 17-Jan-2017

Options

1) unfit to drink

पीनहॳकहॳ लिएअयह८गय

2) smelly

दगयनधयकत

3) Colored

रगीन

4) Turbid

मटमहॴिा Correct Answer unfit to drink

Q72 Musa paradisiaca is the scientific

name of which plant

मसापहॴराडिलसयाकाककसपह९धहॳकावहॴजञाननकनामहहॴ

17-Jan-2017

Options

1) Mango

आम

2) Wheat

गहॳह

3) Corn

भ ा 4) banana

कहॳ िा Correct Answer banana

Q73 Prawns belong to which family

झीगहॳककसपररवारकहॳ हह८तहॳहहॴ 17-Jan-2017

Options

1) Crustaceans

िसटहॳलियन

2)Fish

मछिी 3) Amphibians

अननफबबयस

4) Reptiles

रहॳपटाइमस

Correct Answer Crustaceans

Q74 Name the drug that is yielded from

Cinchona tree and is used to cure

malaria

उसऔषचधकानामबताइएनजसहॳलसगकह८नापहॳड़सहॳपरापतककयाजाताहहॴऔरनजसकाउपयह८गमिहॳररयाकहॳ उपचारमककयाजाताहहॴ 17-Jan-2017

Options

1) Camptothea

कहॴ नटह८चिया 2) Acuminata

एकयलमनहॳटा 3) Quinine

कनहॴन

4) Cinchonia

लसकह८ननया Correct Answer Quinine

Q75 Blood Circulation was discovered

by

रकतपररसचरणकी mdashmdashndash दवारािह८जकीिी 17-Jan-2017

Options

1) Mary Anderson

F A C E B O O K

P A G E h t t p w w w f a c e b o o k c o m s s c m e n t o r s o f f i c i a l P a g e | 19

FOR MORE UPDATES AND MORE MATERIAL DO LIKE OUR FACEBOOK PAGE httpwwwfacebookcomsscmentorsofficial

महॴरीएिरसन

2) Virginia Apgar

वनजयननयाएपगार

3) William Harvey

पवलियमहाव

4) Robert Feulgen

रॉबटयफ़यिजहॳन Correct Answer William Harvey

Q76 Vitamin A is also known as

पवटालमन A कह८ mdashmdash- कहॳ नामसहॳभीजानाजाताहहॴ SSC CHSL Science (biology) 2016

Question Paper

18Jan2017

Options

1) Thiamine

िायलमन

2) Riboflavin

ररबह८फिहॳपवन

3) Retinol

रहॳटटनॉि

4) Calciferol

कहॴ नमसफहॳ रह८ि

Correct Answer Retinol

Q77 Some roots called arise from an

organ other than the radicle

कछजड़हॳनजनह mdashmdashmdash कहाजाताहहॴ वहमिकहॳ अिावाककसीअनयअगसहॳउतपननहह८तीहहॴ 18Jan2017

Options

1) tap roots

मखयजड़

2) stilt roots

ि ाजड़

3) fibrous roots

रहॳिहॳदारजड़

4) adventitious roots

आकनसमकजड़

Correct Answer adventitious roots

Q78 Spiders belong to which class of

animals

मकडड़यापराणीवगीकरणकहॳ ककसवगयमआतीहहॴ 18Jan2017

Options

1) Arachnids

एरहॳकननडस

2) Aves

एपवस

3) Gastropods

गहॴसटरोपह८िस

4) Anthozoa

एिह८जआ

Correct Answer Arachnids

Q79 How many layers does Human

Skin have

मानवतवचामककतनीपरतहॳहह८तीहहॴ

18Jan2017

Options

1) 5

2) 7

3) 11

4) 3

Correct Answer 3

Skin has three layers The epidermis

the outermost layer of skin provides a

waterproof barrier and creates our skin

tone The dermis beneath the

epidermis contains tough connective

tissue hair follicles and sweat glands

The deeper subcutaneous tissue (

hypodermis ) is made of fat and

connective tissue

Q80 Allium Cepa is the scientific name

of

एलियमलसपपा mdashmdashndash कावहॴजञाननकनामहहॴ 18Jan2017

F A C E B O O K

P A G E h t t p w w w f a c e b o o k c o m s s c m e n t o r s o f f i c i a l P a g e | 20

FOR MORE UPDATES AND MORE MATERIAL DO LIKE OUR FACEBOOK PAGE httpwwwfacebookcomsscmentorsofficial

Options

1) Carrot

गाजर

2) Tomato

टमाटर

3) Potato

आि 4) Onion

पयाज़

Correct Answer Onion

Q81 DNA stands for

िीएनएकापणय प mdashmdash- हहॴ 18Jan2017

Options

1) Di Nucleic Acid

िाईनयनकिकएलसि

2) Deoxy Nucleic Acid

िीओकसीनयनकिकएलसि

3) Diribonucleic Acid

िाईराइबह८नयनकिकएलसि

4) Deoxyribonucleic Acid

िीऑकसीराइबह८नयनकिकएलसि

Correct Answer Deoxyribonucleic Acid

Q82 Organisms that generate energy

using light are known as

जह८जीवाणपरकािकाउपयह८गकरउजायउतपननकरतीहहॴ उनह mdashmdash कहॳ पमजानाजाताहहॴ

18Jan2017

Options

1) Chaemolithotrophs

ककमह८लििह८टरह८पस

2) Oligotrophs

ओलिगह८टरह८पस

3) Bacteria

बहॴकटीररया 4)Photoautotrophs

फह८टह८ओटह८टरह८पस

Correct Answer Photoautotrophs

An oligotroph is an organism that can

live in an environment that offers very

low levels of nutrients

Q83 Which drug is used as an

Antidepressant

ककसदवाएकहतािारह८धीकहॳ पमपयोगककयाजाताहहॴ Options

1) Oxybutynin

ओकसीलयटीनन

2)Tramadol

टरहॳमहॳिह८ि

3 ) Sumatriptan

समहॳटरीपटहॳन

4) Bupropion

लयपरह८पपयह८न

Correct Answer Bupropion

लयपरह८पपयह८न

Q84 The orange colour of carrot is

because of

गाजरकानारगीरगनननननलिखितमसहॳककसीएककीवजहसहॳहह८ताहहॴ 18Jan2017

Options

1) it grows in the soil

यहलम ीमउगतीहहॴ 2) Carotene

कहॴ रह८टीन

3) it is not exposed to sunlight

यहसययपरकािकहॳ सपकय मनहीआती 4) the entire plant is oranqe in colour

सनपणयपह९धानारगीरगकाहह८ताहहॴ Correct Answer Carotene

Q85 Snake venom is highly modified

saliva containing

F A C E B O O K

P A G E h t t p w w w f a c e b o o k c o m s s c m e n t o r s o f f i c i a l P a g e | 21

FOR MORE UPDATES AND MORE MATERIAL DO LIKE OUR FACEBOOK PAGE httpwwwfacebookcomsscmentorsofficial

सापकाजहरअततयाचधकसिह८चधतिारहह८तीहहॴनजसमहॳ mdashmdash- हह८ताहहॴ Options

l)Prototoxins

परह८टह८टॉनकसस

2)Neutrotoxins

नयटरोटॉनकसस

3)Zootoxins

जटॉनकसस

4)Electrotoxins

इिहॳकटरह८टॉनकसस

Correct Answer Zootoxins

जटॉनकसस

Q86 Which type of pathogen causes the

water-borne disease Schistosomiasis

ककसपरकारकारह८गज़नकजिजननतरह८गलससटह८सह८लमलससकाकारणबनताहहॴ

18Jan2017

Option

1) Parasitic

परजीवी 2)Protozoan

परह८टह८जआ

3) Bacterial

बहॴकटीररयि

4) Viral

वायरि

Correct Answer Parasitic

Schistosomiasis also known as snail

fever and bilharzia is a disease caused

by parasitic

flatworms called schistosomes

Q87 Prothrombin responsible for

clotting of blood is released by

परह८िह८ननबन

जह८रकतकािककाजमनहॳकहॳ लिएनजनमहॳदारहहॴ mdashndash

कहॳ दवारासतरापवतककयाजाताहहॴ

19Jan2017

Options

1) Small Intestine

छह८टीआत

2) Blood Platelets

रकतपिहॳटिहॳटस

3) Large Intestine

बड़ीआत

4Heart

हदय

Correct Answer Blood Platelets

Q88 Acacia arabica is the scientific

name of

अकहॳ लियाअरहॳबबका mdashmdashndash कावहॴजञाननकनामहहॴ 19-Jan-2017

Options

1) Neem

नीम

2) Teak

सागह९न

3) Babhul

बबि

4) Pomegranate

अनार

Correct Answer Babhul

Q89 Cannis Vulpes is the scientific

name of

कहॴ ननसवनमपस mdashmdash- कावहॴजञाननकनामहहॴ 19-Jan-2017

Options

1) Dog

कतता 2) Wolf

भहॳडड़या 3) Fox

िह८मड़ी 4) Hyena

िाकिबगघा

F A C E B O O K

P A G E h t t p w w w f a c e b o o k c o m s s c m e n t o r s o f f i c i a l P a g e | 22

FOR MORE UPDATES AND MORE MATERIAL DO LIKE OUR FACEBOOK PAGE httpwwwfacebookcomsscmentorsofficial

Correct Answer Fox

Q90 The beetroot is the portion of the

beet plant

चकदरपह९धहॳका mdashmdashndash भागहहॴ 19-Jan-2017

Options

1) tap root

मखयजड़

2) Adventitious

आकनसमक

3) bulb of the stem

तनहॳकाकद

4) Rhizome

परकद

Correct Answer tap root

Q91 What is the basic unit of heredity

आनवलिकताकीबननयादीइकाईकयाहहॴ 19-Jan-2017

Options

1) DNA

िीएनए

2) RNA

आरएनए

3) Chromosome

िह८मह८सह८म

4) Gene

जीन

Correct Answer gene

Genes are the units of heredity and are

the instructions that make up the bodyrsquos

blueprint They code for the proteins

that determine virtually all of a personrsquos

characteristics Most genes come in

pairs and are made of strands of genetic

material called deoxyribonucleic acid

or DNA

Q92 Lungs are the primary organs of

फहॳ फड़हॳmdashndashकहॳ परािलमकअगहहॴ

19-Jan-2017

Options

1) Digestion

पाचन

2) Constipation

कलज

3) Perspiration

पसीना 4)Respiration

शवसन

Correct Answer Respiration

Q93 Sugarcane is a type of

गननाएकपरकारका mdash- हहॴ 20-Jan-2017

Options

1)creeper

िता 2)tree

पहॳड़

3)shrub

झाड़ी 4)grass

घास

Correct Answer grass

Q94 Who is commonly known as ldquothe

Father of Microbiologyrdquo

सामानयत ldquo सकषमजीवपवजञानकहॳ जनक lsquo

कहॳ नामसहॳककसहॳजानाजातहहॴ 20-Jan-2017

Options

1) Robert Hooke

रॉबटयहक

2) Antonie Philips van Leeuwenhoek

एटह८नीकफलिपवानमयएनहह८क

3) Carl Linnaeus

काियिीनाईयस

4) Charles Darwin

चामसयिापवयन

F A C E B O O K

P A G E h t t p w w w f a c e b o o k c o m s s c m e n t o r s o f f i c i a l P a g e | 23

FOR MORE UPDATES AND MORE MATERIAL DO LIKE OUR FACEBOOK PAGE httpwwwfacebookcomsscmentorsofficial

Correct Answer Antonie Philips van

Leeuwenhoek

Q95 For the aquatic organisms the

source of food is

जिीयजीवाणकािाघसरह८तहहॴ 20-Jan-2017

Options

1) Phytoplankton

फायटह८पिहॳकटन

2) Sea Weed

समदरीिहॴवाि

3)Aqua plankton

एकवापिहॳकटन

4) Zooplankton

जपिहॳकटन

Correct Answer Phytoplankton

Q96 Haemoglobin has the highest

affinity with which of the following

हीमह८गिह८बबनकीननननमसहॳककसकहॳ सािउततमसमानताहहॴ

20-Jan-2017

Options

1)SO2

2)CO2

3)CO

4)NO2

Correct Answer CO

It has a greater affinity for hemoglobin

than oxygen does It displaces oxygen

and quickly binds so very little oxygen

is transported through the body cells

Q97 Who developed the theory of

Evolution

उदपवकासकालसदातककसनहॳपवकलसतककया

20-Jan-2017

Options

1) Charles Darwin

चामसयिापवयन

2) Isaac Newton

आयजहॳकनयटन

3) Pranav Mistry

परणवलमसतरी 4) Galileo Galilei

गहॳलिलियह८गहॳिीिी Correct Answer Charles Darwin

Q98 The primary function of RNA is

RNA कापरािलमककाययहह८ताहहॴ 20-Jan-2017

Options

1) Photosynthesis

परकािसशिहॳषण

2) Protein Synthesis

परह८टीनसशिहॳषण

3) Replication

परनतकनतबनाना 4) Translation

अनवादकरना Correct Answer Protein Synthesis

There are two main functions of RNA

It assists DNA by serving as a messenger

to relay the proper genetic information

to countless numbers of ribosomes in

your body The other main function of

RNA is to select the correct amino acid

needed by each ribosome to build new

proteins for your body

Q99 ______is the movement of

molecules across a cell membrane from

a region of their lower concentration to

a region of their higher concertration

उचचसादरताकहॳ कषहॳतरसहॳउसकीकमसादरतावािहॳकषहॳतरकीतरफएककह८लिकाखझमिीकहॳ माधयमसहॳहह८नहॳवािाअणओकहॳ सचिनकह८ mdash- कहतहॳहहॴ Options

1) Diffusion

पवसरण

2) Osmosis

ऑसमह८लसस

F A C E B O O K

P A G E h t t p w w w f a c e b o o k c o m s s c m e n t o r s o f f i c i a l P a g e | 24

FOR MORE UPDATES AND MORE MATERIAL DO LIKE OUR FACEBOOK PAGE httpwwwfacebookcomsscmentorsofficial

3) Active Transport

सकियआवागमन

4) Passive Transport

नननषियआवागमन

Correct Answer Active Transport

Q100 Study of classification of

organisms is known as 20-Jan-2017

जीवाणओकहॳ वगीकरणकहॳ अधययनकह८ mdash-

कहाजाताहहॴ Options

1) Serpentology

सपरहॳटह८िह८जी 2) Virology

वायरह८िह८जी 3) Taxonomy

टहॴकसोनह८मी 4) Physiology

कफनज़यह८िह८जी Correct Answer Taxonomy

Q101 Photosynthesis takes place inside

plant cells in

परकािसशिहॳषणवनसपनतकह८लिकामनसति mdash

mdashmdash महह८ताहहॴ 20-Jan-2017

Options

1) Ribosomes

राइबह८सह८नस

2) Chloroplasts

किह८रह८पिासट

3) Nucleus

नयकलियम

4) Mitochondria

माईटह८कोडडरया Correct Answer Chloroplasts

Q102 ______ is the cell organelle in

which the biochemical processes of

respiration and energy production

occur

mdashmdash- वहकह८लिकाअगहहॴ नजसमहॳशवसनऔरउजायउतपादनकहॳ जहॴसीजहॴवरासायननकपरकियायहह८तीहहॴ 20-Jan-2017

Options

1) Mitochondria

माइटह८कोडडरया 2) Chloroplast

किह८रह८पिासट

3) Ribosomes

राइबह८सह८नस

4) Nucleus

नयकिीयस

Correct Answer Mitochondria

Q103 Which non-flowering spore

bearing plants have roots

ककसफिनिगनहॳवािहॳऔरबीजाणधारकपह९धह८कीजड़हॳहह८तीहहॴ 21-Jan-2017

Options

1) Mosses

मह८सहॳस

2) Angiosperms

एननजयह८सपनसय 3) Ferns

फनसय 4) Gymnosperms

नजननह८सपनसय Correct Answer ferns

Q104 Which of the following is an

excretory organ of cockroach

नननननलिखितमसहॳकह९नसानतिच हॳकाउतसजयनअगहहॴ

21-Jan-2017

Options

F A C E B O O K

P A G E h t t p w w w f a c e b o o k c o m s s c m e n t o r s o f f i c i a l P a g e | 25

FOR MORE UPDATES AND MORE MATERIAL DO LIKE OUR FACEBOOK PAGE httpwwwfacebookcomsscmentorsofficial

1) Malphigian Tubules

मनमफनजयनटयबमस

2) Nephridia

नहॳकफरडिया 3) Coxal Gland

कह८कसिगरचिया 4) Green Gland

गरीनगरचिया Correct Answer Malphigian Tubules

Q105 Evaporation of water takes place

in which part of plants

पानीकहॳ वाषपीकरणकीकियापह९धोकहॳ ककसभागसहॳहह८तीहहॴ 21-Jan-2017

Options

1) Stem

तना 2) Stomata

सटह८मटा 3) Branch

िािाए

4) Fruit

फि

Correct Answer Stomata

Evaporation accounts for the movement

of water to the air from sources such as

the soil canopy interception and

waterbodies Transpiration accounts for

the movement of water within a plant

and the subsequent loss of water as

vapour through stomata in its leaves

Q106 A is the fleshy spore-bearing

fruiting body of a fungus

mdashmdashndashकवककामासि

बीजाणधारणकरनहॳवािाफिनहॳवािाअगहहॴ 21-

Jan-2017

Options

1) aloe vera

एिह८वहॳरा 2) Coral

मगा 3) Cactus

कहॴ कटस

4) Mushroom

ककरमतता Correct Answer mushroom

Q107 Which of the following is a fungal

disease

नननननलिखितमसहॳकह९नसाफफदसहॳहह८नहॳवािाएकरह८ग हहॴ

21-Jan-2017

Options

1) Dermatitis

तवचािह८ध

2) Cholera

हहॴजा 3) Jaundice

पीलिया 4) Indigofera

इननिगह८फहॳ रा Correct Answer Dermatitis

Dermatitis also known as eczema is a

group of diseases that results in

inflammation of the skin These diseases

are characterized by itchiness red skin

and a rash In cases of short duration

there may be small blisters while in

long-term cases the skin may become

thickened

Q108 In which form is glucose stored in

our body

हमारहॳिरीरमगिकह८जकासचयककस पमककयाजाताहहॴ

21-Jan-2017

Options

1) Insulin

F A C E B O O K

P A G E h t t p w w w f a c e b o o k c o m s s c m e n t o r s o f f i c i a l P a g e | 26

FOR MORE UPDATES AND MORE MATERIAL DO LIKE OUR FACEBOOK PAGE httpwwwfacebookcomsscmentorsofficial

इसलिन

2) Glucose

गिकह८ज

3) Glycogen

गिायकह८जहॳन

4) Fat

वसा Correct Answer Glycogen

Excess glucose is stored in the liver as

the large compound called glycogen

Glycogen is a polysaccharide of glucose

but its structure allows it to pack

compactly so more of it can be stored in

cells for later use

Q109 Where do plants synthesize

protein from

पह९धहॳपरह८टीनसशिहॳषणकहासहॳकरतहॳहहॴ

Options

1) Fatty Acids

वसाऐलसि

2) Sugar

िकर

3) Amino Acids

एलमनह८ऐलसि

4) Starch

सटाचय Correct Answer Amino Acids

Q110 Which part of the brain is

responsible for triggering actions like

thinking intelligence memory and

ability to learn

मनसतषककाकह९नसाटहससासह८चनहॳ बनधदमानी याददाशतऔरसीिनहॳकीकषमताजहॴसीकियाओकह८परहॳररतकरताहहॴ 21-Jan-2017

Options

1) Diencephalon

िायएनसहॳफहॳ िह८न

2) Hypothalamus

हयपह८िहॳिहॳमस

3) Cerebrum

सहॳरहॳिम

4) Control

कटरह८ि

Correct Answer Cerebrum

Q111 Which of the following is also

known as the Biochemical Laboratory

of the Human Body

नननननलिखितमसहॳककसहॳमानविरीरकीजहॴवरसायनपरयह८गिािाभीकहाजाताहहॴ 21-Jan-2017

Options

1) Small Intestine

छह८टीआत

2)Brain

मनसतषक

3) Pancreas

अगनयािय

4) Liver

नजगर

Correct Answer Liver

The liver makes bile that will help

emulsify and digest the fats we eat

The liver takes toxic substances and

convert them using enzymes the liver

cells makes into a non toxic form so the

body can dispose of them

The liver also converts fats protein and

carbohydrates into glucose which is the

energy source for our cells to use

The liver takes amino acids and makes

proteins by combining them

Q112 The yellow colour of human urine

is due to

मानवमतरकापीिारग mdashndash कीवजहसहॳहह८ताहहॴ 22-

Jan-2017

Options

1) Bile Salts

F A C E B O O K

P A G E h t t p w w w f a c e b o o k c o m s s c m e n t o r s o f f i c i a l P a g e | 27

FOR MORE UPDATES AND MORE MATERIAL DO LIKE OUR FACEBOOK PAGE httpwwwfacebookcomsscmentorsofficial

पपततनमक

2) Cholesterol

कह८िहॳसटरह८ि

3) Lymph

लिनफ

4) Urochrome

यरह८िह८म

Correct Answer Urochrome

Urobilin or urochrome is the chemical

primarily responsible for the yellow

color of urine

Q113 The wilting of plants takes place

due to

पह९धह८कालिचििहह८नाकी mdashmdash- कीवजहसहॳहह८ताहहॴ 22-Jan-2017

Options

1)Photosynthesis

परकािसशिहॳषण

2) Transpiration

वाषपह८तसजयन

3) Absorption

अविह८षण

4) Respiration

शरवसन

Correct Answer Transpiration

Wilting is the loss of rigidity of non-

woody parts of plants This occurs when

the turgor pressure in non-lignified

plant cells falls towards zero as a result

of diminished water in the cells

Q114 Bovidae Ovis is the scientific name of

बह८पविीओपवस mdashndash कावहॴजञाननकनामहहॴ 22-Jan-2017

Options

1) Goat

बकरी 2) Cow

गाय

3) Buffalo

भहॳस

4) Sheep

भहॳड़

Correct Answer Sheep

Q115 Plants get their energy to produce

food from which of the following

पह८धहॳभह८जनकाननमायणकरनहॳकहॳ लिएनननननलिखितमसहॳककससहॳउजायपरापतकरतहॳहहॴ

22-Jan-2017

Options

1) Photosynthesis

परकािसशिहॳषण

2)Bacteria

बहॴकटीररया 3)Fungi

कवक

4)Sun

सयय Correct Answer Sun

Q116 Which of the following is secreted

by the liver

नननननलिखितमसहॳककसकासरावनजगरसहॳहह८ताहहॴ

22-Jan-2017

Options

1) Glucose

गिकह८ज

2) Iodine

आयह८िीन

3) Cortisol

काटटरयसह८ि

4) Bile

पपतत

Correct Answer Bile

The liver makes bile that will help

emulsify and

digest the fats we eat

F A C E B O O K

P A G E h t t p w w w f a c e b o o k c o m s s c m e n t o r s o f f i c i a l P a g e | 28

FOR MORE UPDATES AND MORE MATERIAL DO LIKE OUR FACEBOOK PAGE httpwwwfacebookcomsscmentorsofficial

Q117 Ferns belong to which division of

plants

फनसयपह९धह८कहॳ ककसभागमआतहॳहहॴ

22-Jan-2017

Options

1) Gymnosperms

नजननह८सपनसय 2) Angiosperms

एनजयह८सपनसय 3) Thallophyta

िहॴिह८फाईटा 4)Pteridophyta

टहॳररिह८फाईटा Correct Answer Pteridophyta

Q118 Who invented Antibiotics

एटीबायह८टटककाअपवषकारककसनहॳककयािा

22-Jan-2017

Options

1) Joseph Lister

जह८सहॳफलिसटर

2) William Harvey

पवलियमहाव

3) Robert Knock

रॉबटयनॉक

4)Alexander Fleming

अिहॳकज़िरफिहॳलमग

Correct Answer Alexander Fleming

Q119 Milbecycin is used in the

eradication of

लममबहॳसायलसनका mdashndash

मउनमिनमपरयह८गककयाजाताहहॴ 22-Jan-2017

Options

1) Agricultural Fungus

कपषकवक

2) Agricultural Pests

कपषकीटक

3) Agricultural Herbs

कपषिाक

4)Agricultural Weeds

कपषननराना Correct Answer Agricultural Pests

Milbemycin oxime is a veterinary drug

from the group of milbemycins used as

a broad spectrum antiparasitic It is

active against worms and mites(insects

Q120 Intestinal bacteria synthesizes

which of the following in the human

body

मानविरीरमआतोकहॳ बहॴकटीररयानननननलिखितमसहॳककसकासशिहॳषणकरतहॳहहॴ 22-Jan-2017

Options

1) Vitamin K

पवटालमन K

2) Proteins

परह८टीन

3) Fats

वसा 4) Vitamin D

पवटालमन D

Correct Answer Vitamin K

Q121 is the study of the physical form

and external structure of plants

mdashmdash-

मपह९धह८काभहॴनतक पऔरबाहरीसरचनाकाआदयाककयाजाताहहॴ 22-Jan-2017

Options

1) Physiology

कफनजयह८िह८जी 2) Anatomy

िरीररचनापवजञान

3) Phytomorphology

फाईटह८मह८फह८िह८जी 4)Cytology

कह८लिकापवजञान

Correct Answer Phytomorphology

F A C E B O O K

P A G E h t t p w w w f a c e b o o k c o m s s c m e n t o r s o f f i c i a l P a g e | 29

FOR MORE UPDATES AND MORE MATERIAL DO LIKE OUR FACEBOOK PAGE httpwwwfacebookcomsscmentorsofficial

Q122 Which of the following is a

structural and functional unit of

kidneys

नननननलिखितमसहॳकह९नसीगदोकीसरचनातमकऔरकाययकरीईकाईहहॴ

22-Jan-2017

Options

1) Renette Cells

रहॳनहॳटकह८लिकाए

2) Flame Cells

फिहॳमकह८लिकाए

3) Nephrites

नहॳफ़राइटस

4)Nephrons

नहॳफरोस

Correct Answer Nephrons

Nephron functional unit of the kidney

the structure that actually produces

urine in the process of removing waste

and excess substances from the blood

There are about 1000000 nephrons in

each human kidney

Q123 Which of the following is the

largest part of the human brain

नननननलिखितमसहॳकह९नसामानवमनसतषककासबसहॳबड़ाटहससाहहॴ

23-Jan-2017

Options

1) Ribs

पसलियाा 2) Cerebrum

सहॳरहॳिम

3) Pons

पोस

4)Thalamus

िहॴिहॳमस

Correct Answer Cerebrum

The cerebrum is the largest part of the

human brain making up about two-

thirds of the brainrsquos mass It has two

hemispheres each of which has four

lobes frontal parietal temporal and

occipital

Q124 The auxiliary buds

सहायककालियाmdashndash 23-Jan-2017

Options

1) grow endogenously from the pericycle

पहॳरीसाईककिसहॳअनतजातयपवकलसतहह८ताहहॴ 2) arise endogenously from the main

growing point

मिवपदसहॳअनतजातयउठताहहॴ 3) is an embryonic shoot located in the

axil of a leaf

एकभरणिटहहॴजह८एकपततीकहॳ अकषपरनसतिहह८ताहहॴ 4)arise exogenously from the epidermis

एपपिलमयससहॳबटहजातयतरीकहॳ सहॳउठताहहॴ Correct Answer is an embryonic shoot

located in the axil of a leaf

Q125 Which of the following is a viral

disease

इनमहॳसहॳकह९सीएकवायरिबीमारीहहॴ

23-Jan-2017

Options

1) Polio

पह८लियह८ 2) Tetanus

धनसतनभ

3) Leprosy

कषठरह८ग

4) Plague

पिहॳग

Correct Answer Polio

A viral disease (or viral infection)

occurs when an organismrsquos body is

invaded by pathogenic viruses and

infectious virus particles (virions) attach

to and enter susceptible cells

F A C E B O O K

P A G E h t t p w w w f a c e b o o k c o m s s c m e n t o r s o f f i c i a l P a g e | 30

FOR MORE UPDATES AND MORE MATERIAL DO LIKE OUR FACEBOOK PAGE httpwwwfacebookcomsscmentorsofficial

Poliomyelitis often called polio or

infantile paralysis is an infectious

disease caused by the poliovirus

Tetanusmdash A serious bacterial infection

that causes painful muscle spasms and

can lead to death

Leprosy also known as Hansenrsquos

disease (HD) is a long-term infection by

the bacterium Mycobacterium leprae or

Mycobacterium lepromatosis

Plague is an infectious disease caused by

the bacterium Yersinia pestis

Symptoms include fever weakness and

headache

Q126 Which organisms can help to

carry out Vermicomposting

कह९नसाजीववमीकनपह८नसटगममददकरताहहॴ

23-Jan-2017

Options

1) Nitrifying Bacteria

नाईटरीफाईगबहॴकटीररया 2) Earthworms

कहॴ चऐ

3) Algae

िहॴवि

4) Fungus

कवक

Correct Answer Earthworms

Q127 Contraction of heart is also

known as

हदयकहॳ सकचनकह८ mdash- भीकहाजाताहहॴ 23-Jan-

2017

Options

1) Systole

लससटह८ि

2) Aristotle

अरसत

3) Diastole

िायसटह८ि

4) Lub

मयब

Correct Answer Systole

Diastole is the part of the cardiac cycle

when the heart refills with blood

following systole (contraction)

Ventricular diastole is the period during

which the ventricles are filling and

relaxing while atrial diastole is the

period during which the atria are

relaxing

Q128 Azadirachta indica is the

botanical name of which of the

following

अजाटदराचताइडिकानननननलिखितमसहॳककसकावानसपनतनामहहॴ

23-Jan-2017

Options

1) Rose plant

गिाबकापह९धा 2) Apple tree

सहॳबकापहॳड़

3) Neem

नीम

4)Mango

आम

Correct Answer Neem

Q129 Which of the following is the

main end product of carbohydrate

digestion

नननननलिखितमसहॳकह९नसाकाबोहाइडरहॳटकहॳ पाचनकापरमिअतउतपादकहह८ताहहॴ 23-Jan-2017

Options

1) Fats

वसा 2) Lipids

लिपपडस

3) Glucose

गिकह८ज

4) Cellulose

F A C E B O O K

P A G E h t t p w w w f a c e b o o k c o m s s c m e n t o r s o f f i c i a l P a g e | 31

FOR MORE UPDATES AND MORE MATERIAL DO LIKE OUR FACEBOOK PAGE httpwwwfacebookcomsscmentorsofficial

सहॳमयिह८ज

Correct Answer Glucose

Intestinal absorption of end products

from digestion of carbohydrates and

proteins in the pig hellip During absorption some sugars (fructose or

galactose) released from the

corresponding sucrose and lactose

respectively during digestion were

partly metabolized into glucose by the

enterocyte

Q130 Which of the following glands is a

source of the enzyme Ptyalin

नननननलिखितगरचियोमसहॳएजाइमटयालिनकासरह८तहहॴ 23-Jan-2017

Options

1) Pancreas

अगरािय

2) Thyroid Gland

िाइराइिगरिी 3) Pituitary Gland

पीयषगरिी 4) Salivary Glands

िारगरचियाा Correct Answer Salivary Glands

Q131 Which of the following is not true

about Pteridophyta

ननननमसहॳकह९नसीबातटहॳररिह८फाईटकहॳ बारहॳमसचनहीहहॴ 23-Jan-2017

Options

1) Dominant phase is saprophytes

परमिचरणसहॳपरह८फाईइटसहह८ताहहॴ 2) Main plant body is diploid

पह९दह८कामखयिरीरदपवगखणतहह८ताहहॴ 3) Seeds are present

बीजमह९जदहह८तहॳहहॴ 4)Flowers are absent

फिअनपनसतिहह८तहॳहहॴ

Correct Answer Seeds are present

Q132 The largest dolphin species is the

orca also called as

िॉिकफनकीसबसहॳबड़ीपरजानतकाकानामआकायहहॴनजसहॳ mdash- भीकहतहॳहहॴ 23-Jan-2017

Options

1) Bottle Nose

बाटिनह८ज

2) Baiji

बहॳजी 3) Killer whale

ककिरहहॳि

4)Tucuxi

टकवसी Correct Answer Killer whale

Q133 The fat digesting enzyme Lipase

is secreted by which of the following

वसाकापाचनकरनहॳवािाएजाइमिाइपहॳजनननननलिखितमसहॳककसकहॳ दवारासतरापवतहह८ताहहॴ

24-Jan-2017

Options

1) Kidneys

गद

2) Pancreas

अगनयािय

3) Large Intestine

बड़ीआत

4)Liver

नजगर

Correct Answer Pancreas

Lipase is an enzyme that splits fats so

the intestines can absorb them Lipase

hydrolyzes fats like triglycerides into

their component fatty acid and glycerol

molecules It is found in the blood

gastric juices pancreatic secretions

intestinal juices and adipose tissues

F A C E B O O K

P A G E h t t p w w w f a c e b o o k c o m s s c m e n t o r s o f f i c i a l P a g e | 32

FOR MORE UPDATES AND MORE MATERIAL DO LIKE OUR FACEBOOK PAGE httpwwwfacebookcomsscmentorsofficial

Q134 The arrangement of leaves on an

axis or stem is called

एकअकषयातनहॳपरपनततयोकीयवसिाकह८कयाकहाजाताहहॴ SSC CHSL Science (biology) 2016

Question Paper

24-Jan-2017

Options

1) Phyllotaxy

फाइिह८टहॴकसी 2) Vernation

वनिन

3) Venation

वहॳनहॳिन

4)Phytotaxy

फाइटह८टहॴकसी Correct Answer Phyllotaxy

In botany phyllotaxis or phyllotaxy is

the arrangement of leaves on a plant

stem (from Ancient Greek phyacutellon

ldquoleafrdquo and taacutexis ldquoarrangementrdquo)

Phyllotactic spirals form a distinctive

class of patterns in nature

Q135 The study of Cells is also known

as

कह८लिकाओकहॳ अधययनकह८ mdashmdashndash

भीकहाजाताहहॴ 24-Jan-2017

Options

1) Cytology

सायटह८िह८जी 2) Physiology

कफनजयह८िह८जी 3) Nucleology

नयककमयह८िह८जी 4)Cellology

सहॳिह८िह८जी Correct Answer Cytology

Q136 Which of the following scientists

is also known as the Father of Biology

नननननलिखितमसहॳककसवहॴजञाननककह८ ldquoजीवपवजञानकहॳ जनकrdquoकहॳ नामसहॳभीजानाजाताहहॴ 24-Jan-2017

Options

1) Herbert Spencer

हबयटयसपसर

2) Aristotle

अरसत 3) Lamarck

िहॳमाकय 4)Darwin

िापवयन

Correct Answer Aristotle

Q137 Which cells give rise to various

organs of the plant and keep the plant

growing

कह९नसीकह८लिकाएपह९धह८कहॳ लभननअगह८कह८जनमदहॳतीहहॴऔरपह९धह८कह८बढ़नहॳममददकरतीहहॴ

24-Jan-2017

Options

1) Permanent

सिायी 2) Dermal

तवचीय

3) Meristematic

मररसटहॳमटटक

4)Mature

परह८ढ़

Correct Answer Meristematic

A meristem is the tissue in most plants

containing undifferentiated cells

(meristematic cells) found in zones of

the plant where growth can take place

Q138 Rodentia Muridae is the scientific

name of

F A C E B O O K

P A G E h t t p w w w f a c e b o o k c o m s s c m e n t o r s o f f i c i a l P a g e | 33

FOR MORE UPDATES AND MORE MATERIAL DO LIKE OUR FACEBOOK PAGE httpwwwfacebookcomsscmentorsofficial

रह८िहॳलियानयररिी mdashmdash- कावहॴजञाननकनामहहॴ 24-

Jan-2017

Options

1) Mouse

चहा 2) Squirrel

चगिहरी 3) Monkey

बदर

4) Lizard

नछपकिी Correct Answer Mouse

Q139 Name the scientist who proposed

the cell theory

कह८लिकालसदातकापरसतावदहॳनहॳवािहॳवहॴजञाननककानामबताइए 24-Jan-2017

Options

1) Schleiden and Schwann

िीमिनऔरशरववान

2) Lamarck

िहॳमाकय 3) Treviranus

टरहॳवायरहॳनस

4)Whittaker and Stanley

हीटकरऔरसटहॳनिहॳ Correct Answer Schleiden and

Schwann

Q140 The flower with the worldrsquos

largest bloom is

दननयाकासबसहॳबड़ाफिखििनहॳवािा mdashmdashndash हहॴ 24-Jan-2017

Options

1) Pando

पािह८ 2) Posidonia

पह८सीिह८ननया 3) Rafflesia arnoldii

ररफिहॳलियाअनोमिी 4)Helianthus annuus

हहॳलिएनिसएनयअस

Correct Answer Rafflesia arnoldii

Rafflesia arnoldii is a species of

flowering plant in the parasitic genus

Rafflesia It is noted for producing the

largest individual flower on earth It has

a very strong and horrible odour of

decaying flesh earning it the nickname

ldquocorpse flower

Q141 Deficiency of which vitamin

causes night blindness

ककसपवटालमनकीकमीकहॳ कारणरतौधीहह८ताहहॴ 24-Jan-2017

Options

1) Vitamin K

पवटालमन K

2) Vitamin C

पवटालमन C

3) Vitamin B1

पवटालमन B1

4)Vitamin A

पवटालमन A

Correct Answer Vitamin A

Q142 Nongreen plants lack which of the

following

गहॴर-

हररतवनसपनतमनननननलिखितमसहॳककसकीकमीहह८तीहहॴ

24-Jan-2017

Options

1) Chlorophyll

किह८रह८कफि

2) Lycophyll

िायकह८कफि

3) Cyanophyll

F A C E B O O K

P A G E h t t p w w w f a c e b o o k c o m s s c m e n t o r s o f f i c i a l P a g e | 34

FOR MORE UPDATES AND MORE MATERIAL DO LIKE OUR FACEBOOK PAGE httpwwwfacebookcomsscmentorsofficial

सायनह८कफि

4)Phototropism

फह८टह८टरोपपजम

Correct Answer Chlorophyll

Q143 Organisms that use light to

prepare food are known as

जह८जीवपरकािकाउपयह८गकरभह८जनतहॴयारकरतहॳहहॴ उनह mdashmdash- कहॳ पमजानजाताहहॴ 24-Jan-2017

Options

1) Autotrophs

सवपह८षी 2) Heterotrophs

पवषमपह८षज

3) Omnivores

सवायहारी 4)Decomposers

पवघटनकरनहॳवािा Correct Answer Autotrophs

autotrophs often make their own food

by using sunlight carbon dioxide and

water to form sugars which they can use

for energy Some examples of

autotrophs include plants algae and

even some bacteria Autotrophs

(producer) are important because they

are a food source for heterotrophs

(consumers)

A heterotroph is an organism that

ingests or absorbs organic carbon

(rather than fix carbon from inorganic

sources such as carbon dioxide) in order

to be able to produce energy and

synthesize compounds to maintain its

life Ninety-five percent or more of all

types of living organisms are

heterotrophic including all animals and

fungi and some bacteria

Q144 Which of the following is a

primary function of haemoglobin

नननननलिखितमसहॳकह९नसाटहमह८गिह८बबनकाएकपरािलमककाययहहॴ

25-Jan-2017

Options

1) Utilization of energy

उजायकाउपयह८गकरना 2) Prevention of anaemia

रकतामपताहह८नहॳसहॳरह८कना 3) Destruction of bacteria

बहॴकटीररयाकापवनािकरना 4) To transport oxygen

ऑकसीजनकावहनकरना Correct Answer To transport oxygen

Q145 Vascular bundles are absent in

सवहनीबिि mdashmdash- मअनपनसतिरहतहॳहहॴ 25-Jan-2017

Options

1) Bryophyta

िायह८फाइटा 2) Pteridophyta

टहॳररिह८फाईटा 3) Gymnosperms

नजननह८सपमय 4) Angiosperms

एननजयह८सपहॳनसय Correct Answer Bryophyta

Q146 Sauria Lacertidae is the scientific

name of

सहॴररयािहॳसरटाईिी mdashmdashndash कावहॴजञाननकनामहहॴ 25-Jan-2017

Options

1) Crocodile

मगरमचछ

2) Hippopotamus

टहपपह८पह८टहॳमस

3) Lizard

नछपकिी 4) House fly

F A C E B O O K

P A G E h t t p w w w f a c e b o o k c o m s s c m e n t o r s o f f i c i a l P a g e | 35

FOR MORE UPDATES AND MORE MATERIAL DO LIKE OUR FACEBOOK PAGE httpwwwfacebookcomsscmentorsofficial

घरहॳिमकिी Correct Answer Lizard

Q147 Which type of pathogen causes

the water-borne disease SARS (Severe

Acute Respiratory Syndrome)

ककसपरकािकारह८गज़नकजिजननतबीमारीसासयकाकारणबनताहहॴ 25-Jan-2017

Options

1) Viral

वायरि

2) Parasitic

परजीवी 3) Protozoan

परह८टह८जअन

4) Bacterial

बहॴकटीररयि

Correct Answer Viral

Q148 Which of the following organs

produces the enzyme lipase

नननननलिखितमसहॳकह९नसाअगिायपहॳजएजाइमउतपननकरताहहॴ 25-Jan-2017

Options

1) Pancreas

अगनयािय

2) Large Intestine

बड़ीआत

3) Liver

नजगर

4) Small Intestine

छह८टीआत

Correct Answer Pancreas

Q149 A is a long internode forming the

basal part or the whole of a peduncle

एक mdashmdash- एकिबाइटरनह८िहहॴ जह८ननचिाटहससायासनपणयिठिबनताहहॴ 25-

Jan-2017

Options

1) Rhizome

परकद

2) Rachis

महॳ दि

3) floral axis

पषपअकष

4) Scape

भगदड़

Correct Answer scape

Q150 ndash Which of the following

organisms are considered to be both

Living and Non-living

नननननलिखितमसहॳकह९नसहॳजीवाणकह८जीपवतऔरअजीपवतमानाजाताहहॴ

25-Jan-2017

Options

1) Bacteria

बहॴकटीररया 2) Fungi

कवक

3) Algae

िहॴवाि

4)Virus

वायरस

Correct Answer Virus

They are considered to be living as they

possess a protein coat as a protective

covering DNA as the genetic material

etc

They are said to be non-living as they

can be crystallised and they survive for

billions of years They can tolerate high

temperatures freezing cold

temperatures ultra-violet radiations etc

Q151 Deficiency of fluorine causes

which of the following

फिह८ररनकीकमीकहॳ कारणनननननलिखितमसहॳकयाहह८ताहहॴ

F A C E B O O K

P A G E h t t p w w w f a c e b o o k c o m s s c m e n t o r s o f f i c i a l P a g e | 36

FOR MORE UPDATES AND MORE MATERIAL DO LIKE OUR FACEBOOK PAGE httpwwwfacebookcomsscmentorsofficial

27-Jan-2017

Options

1) Dental Caries

िटिकहॴ ररज

2) Scurvy

सकवरी 3) Anaemia

रकतामपता 4) Arthritis

गटठया Correct Answer Dental Caries

Q152 In a Punnett Square with the

cross AaBb x AaBb how many Aabb

genotypes would be created

पनहॳटसककायरमिह८स AaBb x AaBb कहॳ साि

ककतनहॳ Aabb जीनह८टाइपबनगहॳ 27-Jan-2017

Options

1) 1

2) 8

3) 2

4) 3

Correct Answer 2

Q153 Which of the following is the

Controlling Center of the Cell

नननननलिखित म सहॳ कह८लिकाका ननयतरण

क दर कह९न हहॴ

27-Jan-2017

Options

1) Nucleus

क दर

2) Plasma

पिाजमा 3) Lysosome

िायसह८सह८म

4) Chromosome

िह८मह८सह८म

Correct Answer Nucleus

The control centre of the cell is the

nucleus in eukaryotic cells The nucleus

contains genetic material in the form of

DNA

Q154 Myopia affects which of the

following organs

मायह८पपयानननननलिखितअगह८मसहॳककसहॳपरभापवतकरताहहॴ

25-Jan-2017

Options

1) Heart

हदय

2) Skin

तवचा 3) Eyes

आािहॳ 4)Mouth

मह

Correct Answer Eyes

Q155 Which of the following bears

flowers

नननननलिखितमसहॳकह९नफिधारणकरताहहॴ

25-Jan-2017

Options

1) Bryophyta

िायह८फाइटा 2) Pteridophyta

टहॳरीिह८फाईटा 3) Gymnosperms

नजननह८सपमय 4)Angiosperms

एननजयह८सपमय Correct Answer Angiosperms

Q156 Oxygenated blood flows out of the

heart through the

ऑकसीजनयकतरकत mdashmdashmdash

कहॳ माधयमसहॳहदयकहॳ बाहरबहताहहॴ 25-Jan-2017

F A C E B O O K

P A G E h t t p w w w f a c e b o o k c o m s s c m e n t o r s o f f i c i a l P a g e | 37

FOR MORE UPDATES AND MORE MATERIAL DO LIKE OUR FACEBOOK PAGE httpwwwfacebookcomsscmentorsofficial

Options

1) Aorta

महाधमनी 2) pulmonary artery

फहॳ फड़हॳकीधमनी 3) vena cava

वहॳनाकावा 4)Atrium

चह९क

Correct Answer aorta

Q157 Blood leaving the liver and

moving towards the

heart has a higher concentration of

नजगरसहॳननकिकरहदयकीतरफजानहॳवािहॳरकतम mdashmdashmdashmdash कीउचचसादरताहह८तीहहॴ 27-Jan-2017

Options

1) Lipids

लिपपडस

2) Urea

यररया 3) Bile Pigments

पपततकहॳ रगकरण

4) Carbon dioxide

काबयनिायऑकसाइि

Correct Answer Bile Pigments

Urea is nitrogen containing substance

which is produced in the liver in order

to deal with excess amino-acids in the

body As urea is produced it leaves the

liver in the blood stream and passes via

the circulatory system to all parts of the

body

Q158 Bulb is a modification of which

part of a plant

बमबएकपह९धहॳकहॳ ककसटहससहॳकाएक पातरणहह८ताहहॴ 27-Jan-2017

Options

1) The root

जड़

2) The stem

तना 3) The radicle

मिाकर

4)The fruit

फि

Correct Answer The stem

Q159 Which of the following carries

blood away from the heart to different

body parts

इनमहॳसहॳकह९नरकतकह८हदयसहॳिरीरकहॳ पवलभननअगह८तकिहॳजातीहहॴ

27-Jan-2017

Options

1) Arteries

धमननया 2) Nerves

तबतरहाए

3) Capillaries

कहॳ लिकाए

4)Veins

नसहॳ Correct Answer Arteries

Q160 The series of processes by which

nitrogen and its compounds are

interconverted in the environment and

in living organisms is called

27-Jan-2017

Options

1)Absorption of Nitrogen

2)Ammonification

3)Nitrogen Fixation

4)Nitrogen Cycle

Correct Answer Nitrogen Cycle

Ammonification or Mineralization is

performed by bacteria to convert

organic nitrogen to ammonia

F A C E B O O K

P A G E h t t p w w w f a c e b o o k c o m s s c m e n t o r s o f f i c i a l P a g e | 38

FOR MORE UPDATES AND MORE MATERIAL DO LIKE OUR FACEBOOK PAGE httpwwwfacebookcomsscmentorsofficial

Nitrification can then occur to convert

the ammonium to nitrite and nitrate

Nitrogen fixation is a process by which

nitrogen in the Earthrsquos atmosphere is

converted into ammonia (NH3) or other

molecules available to living organisms

Q161 BCG vaccine is given to protect

from which of the following

बीसीजीकाटटकानननननलिखितमसहॳककसकहॳ बचावकहॳ लिएटदयाजातहहॴ

27-Jan-2017

Options

1) Jaundice

पीलिया 2) Anaemia

रकतमपता 3) Tuberculosis

कषयरह८ग

4) Polio

पह८लियह८ Correct Answer Tuberculosis

Q162 Parallel venation is found in

समानतरवहॳनहॳिन mdashmdashmdash- मपायाजाताहहॴ 27-Jan-2017

Options

1) plants which are monocots

पह९धहॳजह८एकबीजपतरीहह८तहॳहहॴ 2) plants which have a dicot stem

वहॳपह९धहॳनजनकातनादपवदलियहह८ताहहॴ 3) plants with leaves similar to Tulsi

वहॳपह९धहॳनजनकीपनततयतिसीकीपनततयोकहॳ समानहह८तहॳहहॴ 4)plants with tap roots

टहॳप टवािहॳपह९धहॳ Correct Answer plants which are

monocots

Q163 The hardest part of the body is

िरीरकासबसहॳकठह८रभाग mdashndash हहॴ 27-Jan-2017

Options

1) Bones

हडडिय

2) Tooth Enamel

दातकहॳ इनहॳमि

3) Skull

िह८पड़ी 4) Spinal Cord

महॳ रजज

Correct Answer Tooth Enamel

Q164 Which type of pathogen causes

the waterborne disease E coli Infection

ककसपरकारकारह८गजननकजिजननतरह८गईकह८िाईसिमणकाकारणबनताहहॴ 27-Jan-2017

Options

1) Protozoan

परह८टह८जआ

2) Parasitic

परजीवी 3) Bacterial

बहॴकटीररयि

4)Viral

वायरि

Correct Answer Bacterial

Q165 The amount of blood filtered

together by both the kidneys in a 70 kg

adult male human in a minute is

70 की गरा वािहॳएकवयसकप षमएकलमनटमदह८नोगदकहॳदवाराएकसािचाबनीगयीरकतकीमातरहह८तीहहॴ 29-Jan-2017

Options

1) 1100 ml

1100 लमलि

2) 100 ml

F A C E B O O K

P A G E h t t p w w w f a c e b o o k c o m s s c m e n t o r s o f f i c i a l P a g e | 39

FOR MORE UPDATES AND MORE MATERIAL DO LIKE OUR FACEBOOK PAGE httpwwwfacebookcomsscmentorsofficial

100 लमलि

3) 1500 ml

1500 लमलि

4) 500 ml

500 लमलि

Correct Answer 1100 ml

Q166 Which feature of a plant helps to

distinguish a monocot from a dicot

पह९धहॳकीवहकह९नसीपविहॳषताहहॴजह८एकदपवदलियहॳऔरएकएकदिीयपह९धहॳसहॳभहॳदकरनहॳममददकरतीहहॴ 29-Jan-2017

Options

1) Pollination

परागम

2) Venation

वहॳनहॳिन

3) Vernation

वनिन

4) Aestivation

एसटीवहॳिहॳन

Correct Answer venation

Q167 The Mutation Theory was

proposed by

उतवररवतयनकालसदात mdashmdashndash

कहॳ दवरापरसतापवतककयाजाताहहॴ 29-Jan-2017

Options

1) Charles Lyell

चामसयलियहॳि

2) William Smith

पवलियमनसमि

3) Hugo De Vries

हयगह८िीराईस

4)Harrison Schmitt

हहॳरीसननसमट

Correct Answer Hugo De Vries

Q168 Which type of pathogen causes

the waterborne disease HepatitisA

ककसपरकारकहॳ रह८गजनकजिजननतरह८गहहॳपहॳटाइटटस-A काकारणबनताहहॴ

29-Jan-2017

Options

1) Parasitic

परजीवी 2) Viral

वायरि

3) Protozoan

परह८टह८जआ

4) Bacterial

बहॴकटीररयि

Correct Answer Viral

Q169 In a Punnett Square with the

cross AaBb x Aabb how many AaBb

genotypes would be created

पनहॳटसकवायरमिह८स AaBb x Aabb

कहॳ सािककतनहॳ AaBb जीनह८टाइपबनगहॳ 29-Jan-

2017

Options

1) 4

2) 1

3) 7

4) 6

Correct Answer 4

Q170 Arboreal Ateles is the scientific

name of

अिह८ररयिएटटलिस mdashmdashmdash कावहॴजञाननकनामहहॴ 29-Jan-2017

Options

1) Squirrel

चगिहरी 2) Sparrow

गह८रहॴया 3) Lizard

नछपकिी 4) Spider monkey

F A C E B O O K

P A G E h t t p w w w f a c e b o o k c o m s s c m e n t o r s o f f i c i a l P a g e | 40

FOR MORE UPDATES AND MORE MATERIAL DO LIKE OUR FACEBOOK PAGE httpwwwfacebookcomsscmentorsofficial

मकड़ीबदर

Correct Answer Spider monkey

Q171 Which type of pathogen causes

the waterborne disease Salmonellosis

ककसपरकारकारह८गाणजिजननतबीमारीसािमह८नहॳिह८लसज़काकारकहहॴ

29-Jan-2017

Options

1) Algal

िहॳवालियहॳ 2) Parasitic

परजीवी 3) Bacterial

बहॴकटीररयि

4)Viral

वायरि

Correct Answer Bacterial

An infection with salmonella bacteria

commonly caused by contaminated food

or water

Symptoms include diarrhoea fever

chills and abdominal pain

Q172 is a condition in which there is a

deficiency of red cells or of haemoglobin

in the blood

mdashmdash-

एकनसिनतहहॴनजसमहॳरकतमिािकह८लिकाओकीयाहीमह८गिह८बबनकीकमीहह८तीहहॴ 29-Jan-2017

Options

1) Albinism

एनमबननजम

2) Propyria

परह८पीररया 3) Anaemia

एनीलमया 4)Keloid disorder

कहॳ िह८इिडिसओिर

Correct Answer Anaemia

Q173 Ananas comosus is the scientific

name of

Options

अनानासकह८मह८सस mdashmdashmdashndash

कावहॴजञाननकनामहहॴ 29-Jan-2017

1) Custard Apple

सीताफि

2) Pineapple

पाइनएपपि

3) Bamboo

बास

4)Pomegranate

अनार

Correct Answer Pineapple

Q174 Which organ produces insulin

कह९नसाअगइनसलिनपहॴदाकरताहहॴ 29-Jan-

2017

Options

1) Liver

यकत

2) Thyroid gland

िायराइिगरिी 3) Spleen

पिीहा 4)Pancreas

अगरयिय

Correct Answer Pancreas

Q175 Which of the following disease is

not caused by water pollution

नननननलिखितमसहॳकह९नसारह८गपानीकहॳ परदषणकहॳकारणनहीहह८ता

29-Jan-2017

Options

1) Cholera

हहॴजा 2) Typhoid

F A C E B O O K

P A G E h t t p w w w f a c e b o o k c o m s s c m e n t o r s o f f i c i a l P a g e | 41

FOR MORE UPDATES AND MORE MATERIAL DO LIKE OUR FACEBOOK PAGE httpwwwfacebookcomsscmentorsofficial

टाइफाइि

3) Asthma

दमा 4)Diarrhoea

दसत

Correct Answer Asthma

Q176 Ocimum tenuiflorum is the

scientific name of

ओलिलममटहॳयईफिह८रमइसकावहॴजञाननकनाम mdash

ndash हहॴ 30-Jan-2017

Options

1) Neem

नीम

2) Mango

आम

3) Babul

बबि

4)Tulsi

तिसी Correct Answer Tulsi

Q177 Which gland secretes bile a

digestive fluid

कह९नसीगरिीपपतत एकपाचनतरिपरदािय सरापवतकरतीहहॴ 30-Jan-2017

Options

1) Pancreas

अगनयािय

2) Liver

यकत

3) Thyroid

िायराइि

4) Testes

टहॳनसटस

Correct Answer liver

Q178 In which of the following the

dominant phase is Gametophyte

नननननलिखितमसहॳककसकहॳ परमिचरणयगमकह८दपवधद (Gametophyte)हहॴ 30-Jan-2017

Options

1) Bryophyta

िायह८फाइटा 2) Pteridophyta

टहॳररिह८फाइटा 3) Gymnosperms

नजननह८सपमय 4) Angiosperms

एननजयह८सपमय Correct Answer Bryophyta

Q179 Anaerobic respiration refers to

which of the following

नननननलिखितमसहॳककसहॳअवायवीयशवसनकहाजाताहहॴ

30-Jan-2017

Options

1) Respiration without Oxygen

ऑकसीजनकहॳ बबनाशवसन

2) Respiration with Oxygen

ऑकसीजनकहॳ सािशवसन

3) Respiration without CO2

काबयनिायऑकसाइिकहॳ बबनाशवसन

4) Respiration with CO2

काबयनिायऑकसाइिकहॳ सािशविन

Correct Answer Respiration without

Oxygen

Q180 Which type of pathogen causes

the waterborne disease Cholera

ककसपरकारकारह८गजनकजिजननतरह८गहहॴजाकाकारणबनताहहॴ

30-Jan-2017

Options

1) Algal

िहॴवालियहॳ

F A C E B O O K

P A G E h t t p w w w f a c e b o o k c o m s s c m e n t o r s o f f i c i a l P a g e | 42

FOR MORE UPDATES AND MORE MATERIAL DO LIKE OUR FACEBOOK PAGE httpwwwfacebookcomsscmentorsofficial

2) Bacterial

बहॴकटीररयि

3) Protozoan

परह८टह८जआ

4) Viral

वायरि

Correct Answer Bacterial

Q181 To which class does

Oxyreductases transferases hydrolases

belong

ओकसीररिकटहॳसटरासफरहॳजहॳस

हाइडरह८िहॳसहॳसककसवगयमआतहॳहहॴ 30-Jan-2017

Options

1) Hormones

हारमोस

2) Enzymes

एजाइनस

3) Proteins

परह८टीनस

4) Vitamins

पवटालमनस

Correct Answer Enzymes

Q182 Which of the following is not true

about Gymnosperms

ननननमसहॳकह९नसीबातअनावतबीजीकहॳ बारहॳमसचनहीहहॴ 30-Jan-2017

Options

1) Dominant phase is saprophytes

परमिचरणसहॳपरह८फाइटसहह८ताहहॴ 2) Vascular bundles are absent

सवहनीबििअनपनसितहह८ताहहॴ 3) spores are heterospores

बीजाणहहॳटहॳरह८सपह८रसहह८तहॳहहॴ 4) Flowers are absent

फिअनपनसितहह८तहॳहहॴ

Correct Answer Vascular bundles are

absent

Q183 The name of first mammal clone sheep is

भहॳड़कीपरिमसतनपायीपरनत प (किह८न)

कानामहहॴ 30-Jan-2017

Options

1) Noori

नरी 2) Dolly

िॉिी 3) Louise

िसी 4)Durga

दगाय Correct Answer Dolly

Q184 Which type of pathogen causes

the water-borne disease Typhoid fever

ककसपरकारकारह८गजनकजिजननतरह८गटाइफाइिबिारकाकारणबनताहहॴ 30-Jan-2017

Options

1) Algal

िहॴवािीय

2) Parasitic

परजीवी 3) Protozoan

परह८टह८जनअन

4)Bacterial

बहॴकटीररयि

Correct Answer Bacterial

Q185 In which part of the cell are

proteins made

कह८लिकाकहॳ ककसटहससहॳमपरह८टीनबनायाजाताहहॴ

31-Jan-2017

Options

1) Reticulum

रहॳटटकिम

F A C E B O O K

P A G E h t t p w w w f a c e b o o k c o m s s c m e n t o r s o f f i c i a l P a g e | 43

FOR MORE UPDATES AND MORE MATERIAL DO LIKE OUR FACEBOOK PAGE httpwwwfacebookcomsscmentorsofficial

2) Golgi apparatus

गह८मजीएपहॳरहॳटस

3) Ribosomes

ररबह८सह८नस

4) Lysosome

िायसह८सह८नस

Correct Answer ribosomes

Proteins are produced by stringing

amino acids together in the order

specified by messenger RNA strands

that were transcribed from DNA in the

cell nucleus The process of synthesizing

a protein is called translation and it

occurs on ribosomes in the cytoplasm of

a cell

Q186 Polio is a disease caused by which

of the following

नननननलिखितमसहॳपह८लियह८कीबबमारह८हह८नहॳकाकारणकयाहहॴ

31-Jan-2017

Options

1) Bacteria

बहॴकटीररयि

2) Mosquito

मचछर

3) Virus

वायरस

4) Cockroach

नतिच हॳ Correct Answer Virus

Polio or poliomyelitis is a crippling and

potentially deadly infectious disease It

is caused by the poliovirus

Q187 ndash Hay fever is a sign of which of

the following

हहॳकफवरनननननलिखितमसहॳककसकाएकसकहॳ तहहॴ

31-Jan-2017

Options

1) Old Age

वदावसिा 2) Malnutrition

कपह८सण

3) Allergy

एिनजय 4) Over Work

अतयचधककाययकरना Correct Answer Allergy

Q188 How many chromosomes does a

human cell contain

एकमानवकह८लिकामककतनहॳगणसतरहह८तहॳहहॴ

29-Jan-2017

Options

1) 6

2) 26

3) 46

4) 66

Correct Answer 46

In humans each cell normally contains

23 pairs of chromosomes for a total of

46 Twenty-two of these pairs called

autosomes look the same in both males

and females The 23rd pair the sex

chromosomes differ between males and

females

Q189 Which of the following is not true

about Bryophyta

ननननमसहॳकह९नसीबातिायह८फाइटकहॳ बारहॳमसचनहीहहॴ 31-Jan-2017

Options

1) Dominant phase is gametophytes

परमिचरणगहॳलमतह८फाइटसहह८ताहहॴ 2) Main plant body is haploid

पह९धहॳकामखयिरीरअगखणतहह८ताहहॴ 3) Spores are homospores

बीजाणहह८मह८सफह८रसहह८तहॳहहॴ 4) Flowers are present

फिमह८जदहह८तहॳहहॴ Correct Answer Flowers are present

F A C E B O O K

P A G E h t t p w w w f a c e b o o k c o m s s c m e n t o r s o f f i c i a l P a g e | 44

FOR MORE UPDATES AND MORE MATERIAL DO LIKE OUR FACEBOOK PAGE httpwwwfacebookcomsscmentorsofficial

Q190 Which aquatic animal has

trailing tentacles

ककसजिीयजानवरकहॳ पीछहॳचिनहॳवािहॳटहॳटकिसहह८तहॳहहॴ

31-Jan-2017

Options

1) Sea horse

समदरीघह८िा 2) Corals

मगा 3) Jelly fish

जहॳिीमछिी 4) Star fish

तारामछिी Correct Answer Jelly fish

Jellyfish with its umbrella-shaped bell

and trailing tentacles

Q191 Which type of pathogen causes

the water-borne disease Poliomyelitis

(Polio)

ककसपरकारकारह८गजनकजिजननतरह८गपह८लियह८मायहॳटटस (पह८लियह८) काकारणहहॴ 31-Jan-

2017

Options

1) Parasitic

परजीवी 2) Algal

िहॴवालिय

3) Viral

वायरि

4) Bacterial

बहॴकटीररयि

Correct Answer Viral

Q192 The outer white part of the eye

that protects the inner structures is

आािकाबाहरीसफहॳ दटहससाजह८आतररकसरचनाओकीरकषाकरताहहॴ वह mdashmdashmdash हहॴ 31-Jan-

2017

Options

1) Iris

आयररस

2) Sclera

सकिहॳरा 3) Retina

रहॳटटना 4) Cornea

कह८ननयया Correct Answer Sclera

Q193 Proteins are made up of

परह८टीनकाननमायण mdashndash सहॳहह८ताहहॴ 31-Jan-2017

Options

1) Amino acids

एलमनह८अनि

2) Fatty acids

वसायकतअनि

3) Glucose

गिकह८ज

4)Nucleotides

नयनकियह८टाईिस

Correct Answer Amino acids

Q194 Moringa Oleifera is the scientific

name of

मह८ररगओलिफहॳ रा mdashmdashndash कावहॴजञाननकनामहहॴ 31-Jan-2017

Options

1) Banyan

बरगद

2) Gulmohar

गिमह८हर

3) Amla

आमिा

F A C E B O O K

P A G E h t t p w w w f a c e b o o k c o m s s c m e n t o r s o f f i c i a l P a g e | 45

FOR MORE UPDATES AND MORE MATERIAL DO LIKE OUR FACEBOOK PAGE httpwwwfacebookcomsscmentorsofficial

4) Drumstick

डरमनसटक

Correct Answer Drumstick

Q195 Kidney stones are composed of

गदकीपिरी mdashndash सहॳबनीहह८तीहहॴ 1-Feb-2017

Options

1) Calcium Oxalate

कहॴ नमसयमओकजहॳिहॳट

2) Sodium Chloride

सह८डियमकिह८राइि

3) Magnesium Nitrate

महॳनगनलियमनाइतटरहॳट

4) Calcium Bicarbonate

कहॴ नमियमबायकबोनहॳट

Correct Answer Calcium Oxalate

Q196 ndash Which of the following is not

true about Angiosperms

ननननमसहॳकह९नसीबातआवतबीजीकहॳ बारहॳमसचनहीहहॴ 1-Feb-2017

Options

1) Dominant phase is gametophytes

परमिचरणगहॳलमतह८फाइटहह८ताहहॴ 2) Vascular bundles are present

सवहनीबििमह९जदहह८ताहहॴ 3) Spores are heterospores

बीजाणहहॳटहॳरह८सपह८रसहह८तहॳहहॴ 4) Seeds are covered

बीजढकहॳ हह८तहॳहहॴ Correct Answer Dominant phase is

gametophytes

Q197 All of the following are excretory

(waste) products of animals except

नननननलिखितमसहॳककसएककह८छह८ड़करअनयसभीपराखणयोदवाराउतसनजयतपदाियहहॴ 1-Feb-

2017

Options

1) Uric Acid

यररकएलसि

2) Ammonia

अमह८ननया 3) Carbohydrates

काबोहाइडरहॳट

4) Urea

यररया Correct Answer Carbohydrates

In animals the main excretory products

are carbon dioxide ammonia (in

ammoniotelics) urea (in ureotelics) uric

acid (in uricotelics) guanine (in

Arachnida) and creatine

Q198 RNA is a polymeric molecule

What does RNA stand for

आरएनइएएकबहिकआणहहॴ इसकाकापवय पकयाहहॴ 1-Feb-2017

Options

1) Rado Nuclear Acid

रािह८नयनकियरएलसि

2) Ribo Nucleic Acid

राइबह८नयनकिकएलसि

3) Rhino Nuclear Acid

हाइनह८नयनकियरएलसि

4) Resto Nucleus Acid

रहॳसटह८नयकिीयसएलसि

Correct Answer Ribo Nucleic Acid

Q199 Which organ does detoxification

and produces chemicals needed for

digestion

कह९नसाअगपवषहरणकरताहहॴऔरपाचनकहॳ लिएआवशयकरसायनोकह८पहॴदाकरताहहॴ 1-Feb-

2017

Options

1) Salivary glands

िारगरचिया 2) Pancreas

अगनयािय

F A C E B O O K

P A G E h t t p w w w f a c e b o o k c o m s s c m e n t o r s o f f i c i a l P a g e | 46

FOR MORE UPDATES AND MORE MATERIAL DO LIKE OUR FACEBOOK PAGE httpwwwfacebookcomsscmentorsofficial

3) Thyroid gland

िायराइिगरिी 4) Liver

यकत

Correct Answer Liver

Q200 Psidium guajava is the scientific

name of

लसडियमगआजावा mdashmdash कावहॴजञाननकनामहहॴ 1-

Feb-2017

Options

1) Guava

अम द

2) Mango

आम

3) Bamboo

बास

4) Jack fruit

कटहि

Correct Answer Guava

Q201 Which drug is used as a Blood

Thinner

चधरकह८पतिाकरनहॳकहॳ पमककसदवाकापरयह८गककयाजाताहहॴ

1-Feb-2017

Options

1) Warfarin

वाफर न

2) Tramadol

टरहॳमािह८ि

3) Azithromycin

एनजरह८मायलसन

4) Hydralazine

हाइडरह८िहॳनजन

Correct Answer Warfarin

Q202 Which of the following disease is

caused due to the deficiency of protein

परह८टीनकीकमीकहॳ कारणनननननलिखितमसहॳकह९नसारह८गहह८ताहहॴ 1-Feb-2017

Options

1) Arthritis

गटठया 2) Kwashiorkor

कािीओकय र

3) Goitre

गाइटर

4) Night Blindness

रतह९चध

Correct Answer Kwashiorkor

Q203 A is species of plant that has

adapted to survive in an environment

with little liquid water

mdashmdashndashपह९धहॳकीएकऐसहॳऐसहॳपरजानतहहॴ नजसनहॳकमपानीवािहॳवातावरणमजीपवतरहनहॳकहॳलिएअनकिनहहॴ 1-Feb-2017

Options

1) Xerophyte

म दपवद

2) Hydrophyte

जिीयपादप

3) Mesophyte

समह८दपवद

4) Thallophyte

िहॴिह८फाइटा Correct Answer xerophyte

xerophyte is a species of plant that has

adapted to survive in an environment

with little liquid water such as a desert

or an ice- or snow-covered region in the

Alps or the Arctic

Mesophytes are terrestrial plants which

are adapted to neither a particularly

dry nor particularly wet environment

An example of a mesophytic habitat

would be a rural temperate meadow

F A C E B O O K

P A G E h t t p w w w f a c e b o o k c o m s s c m e n t o r s o f f i c i a l P a g e | 47

FOR MORE UPDATES AND MORE MATERIAL DO LIKE OUR FACEBOOK PAGE httpwwwfacebookcomsscmentorsofficial

which might contain goldenrod clover

oxeye daisy and Rosa multiflora

thallophyte any of a group of plants or

plantlike organisms (such as algae and

fungi) that lack differentiated stems

leaves and roots and that were formerly

classified as a primary division

(Thallophyta) of the plant kingdom

Q204 How many types of teeth are

there in humans

मनषयोमककतनहॳपरकारकहॳ दातहह८तहॳहहॴ

1-Feb-2017

Options

1) 4

2) 5

3) 2

4) 3

Correct Answer 4

teeth -Humans have four types of

teethincisors canines premolars and

molars each with a specific function

The incisors cut the food the canines

tear the food and the molars and

premolars crush the food

Q205 Carica papaya is the scientific name of

कहॴ ररकापपाया mdashmdashndash कावहॴजञाननकनामहहॴ 2-

Feb-2017

Options

1) Peepal

पीपि

2) Papaya

पपीता 3) Tamarind

इमिी 4) Drumstick

ढह८िकाछड़ी Correct Answer Papaya

Q206 Muscles get tired when there is

shortfall of

जब mdashndash कीकमीहह८तीहहॴतबपहॳिीयिकजातीहहॴ 2-Feb-2017

Options

1) Lactic acid

िहॴनकटकएलसि

2) Na+ ions

Na+ आयन

3) ATP

एटीपी 4) Sulphates

समफहॳ टस

Correct Answer ATP

ATP is the energy source muscle fibers

use to make muscles contract

muscle tissuersquos main source of energy

called adenosine triphosphate or ATP

As your muscles use up this energy

source they become tired and fatigued

Oxygen is the key ingredient that helps

create new ATP to replenish the burned

up ATP in your muscles

Q207 Artocarpus integra is the

scientific name of आटह८कापयसइटीगरा mdashmdashmdash कावहॴजञाननकनामहहॴ 2-Feb-2017

Options

1) Guava

अम द

2) Pineapple

अनानास

3) Silver Oak

लसमवरओक

4) Jack fruit

कटहि

Correct Answer Jack fruit

Q208 Which organ stores fat soluble

vitamins

कह९नसाअगवसामघिनिीिपवटालमनह८काभिाराकरताहहॴ

2-Feb-2017

F A C E B O O K

P A G E h t t p w w w f a c e b o o k c o m s s c m e n t o r s o f f i c i a l P a g e | 48

FOR MORE UPDATES AND MORE MATERIAL DO LIKE OUR FACEBOOK PAGE httpwwwfacebookcomsscmentorsofficial

Options

1) Blood

रकत

2) Skin

तवचा 3) Liver

यकत

4) Pancreas

अगनयािय

Correct Answer Liver

Q209 Which disease is caused due to

deficiency of Iodine

आयह८िीनकहॳ कारणकह९नसारह८गहह८ताहहॴ 2-Feb-2017

Options

1) Rickets

ररकहॳ टस

2) Scurvy

सकवी 3) Goitre

गणमािा 4) Growth retardation

पवकासका कना Correct Answer Goitre

rickets A softening and weakening of

bones in children usually due to

inadequate vitamin D

Q210 Grevillea Robusta is the scientific name of

गरहॳपवलियारह८बसटा mdashmdashmdash- कापवजञाननकनामहहॴ 2-Feb-2017

Options

1) Peepal

पीपि

2) Teak

सागह९न

3) Silver Oak

लसमवरओक

4) Jack fruit

कटहि

Correct Answer Silver Oak

Q211 When a Cuttlefish is described as a Molluscs it is at which level of

classification

जबएककटिकफिकह८एकमह८िसकाकहॳ पमवखणयतककयाजाताहहॴतबयहॳवगीकरणकहॳ ककससतरपहॳनसितहहॴ 2-Feb-2017

Options

1) Class

वगय 2) Order

िम

3) Family

पररवार

4) Phylum

सघ

Correct Answer Phylum

Q212 Bambusa dendrocalmus is the

scientific name of बानबसािहॳडराकामस mdashmdashmdash कावहॴजञाननकनामहहॴ 3-Feb-2017

Options

1) Banyan

बरगद

2) Papaya

पपीता 3) Bamboo

बास

4) Pomegranate

अनार

Correct Answer Bamboo

Q213 Acinonyx Jubatus is the scientific name of

एलसनह८ननकसजयबहॳटस mdashmdashmdash

कावहॴजञाननकनामहहॴ 3-Feb-2017

F A C E B O O K

P A G E h t t p w w w f a c e b o o k c o m s s c m e n t o r s o f f i c i a l P a g e | 49

FOR MORE UPDATES AND MORE MATERIAL DO LIKE OUR FACEBOOK PAGE httpwwwfacebookcomsscmentorsofficial

Options

1) Bear

भाि 2) Horse

घह८िा 3) Cheetah

चीता 4) Zebra

जहॳिा Correct Answer Cheetah

Q214 The pale yellow colour of urine is

due to the presence of which pigment

मतरकाफीकापीिारगरगदरयकहॳ उपनसिनतकहॳ कारणहह८ताहहॴ

3-Feb-2017

Options

1) Urochrome

यरह८िह८म

2) Urophyll

यरह८कफि

3) Chlorophyll

किह८रह८कफि

4) Chloroplast

किह८रह८पिासट

Correct Answer Urochrome

Q215 Which of the following constitute

to form a gene

नननननलिखितमसहॳकह९नसीचीज़एकजीनकागठनकरतीहहॴ

3-Feb-2017

Options

1) Polynucleotides

पह८िीनयनकियह८टाईडस

2) Hydrocarbons

हाइडरह८काबोस

3) Lipoproteins

िाईपह८परह८टीनस

4) Lipids

लिपपडस

Correct Answer Polynucleotides

Polynucleotide molecule is a biopolymer

composed of 13 or more nucleotide

monomers covalently bonded in a chain

DNA (deoxyribonucleic acid) and RNA

(ribonucleic acid) are examples of

polynucleotides with distinct biological

function

Q216 Vertebrates belongs to the

phylum

रीढ़कीहडिीवािहॳपराणी mdashmdashmdash

परजानतकहॳ अतगायतआतहॳहहॴ 3-Feb-2017

Options

1) Arthropoda

आरह८पह८ड़ा 2) Annelida

एननलििा 3) Cnidaria

ननिहॳररया 4) Chordata

कह८िटा Correct Answer Chordata

Q217 Punica granatum is the scientific name of

पननकगरहॳनहॳटस mdashmdashmdash कावहॴजञाननकनामहहॴ 3-Feb-2017

Options

1) Custard Apple

सीताफि

2) Gulmohar

गिमह८हर

3) Silver Oak

लसमवरओक

4) Pomegranate

अनार

Correct Answer Pomegranate

F A C E B O O K

P A G E h t t p w w w f a c e b o o k c o m s s c m e n t o r s o f f i c i a l P a g e | 50

FOR MORE UPDATES AND MORE MATERIAL DO LIKE OUR FACEBOOK PAGE httpwwwfacebookcomsscmentorsofficial

Q218 Between a tiger and an monkey

which of the following is different

एकबाघऔरबदरकहॳ बीचनननननलिखितमसहॳकह९नसीबातअिगहहॴ 3-Feb-2017

Options

1) Kingdom

राजय

2) Phylum

जानत

3) Order

िम

4) Class

वगय Correct Answer order

Q219 The artificial heart was invented by

कबतरमहदयका mdashmdashmdash

दवाराअपवषकारककयागयािा 3-Feb-2017

Options

1) Muhammad Yunus

महनमदयनस

2) Linus Yale Jr

िाइनसयहॳिजय

3) Gazi Yasargil

गाजीयासचगयि

4) Paul Winchell

पह९िपवमकि Correct Answer Paul Winchell

Q220 Tamarindus indica is the

scientific name of

टहॳमररनडसइडिका mdashmdash कावहॴजञाननकनामहहॴ 7-

Feb-2017

Options

1) Neem

नीम

2) Pineapple

अनानास

3) Tamarind

इमिी 4)Chiku

चीक

Correct Answer Tamarind

Q221 In eukaryotic cells synthesis of

RNA takes place in the

यकहॳ योटटककह८लिकाओमआरएनएकासशिहॳषण

mdashndash महह८ताहहॴ 7-Feb-2017

Options

1) Mitochondria

माईटह८कोडडरया 2) Centrioles

सटरीयह८मस

3) Ribosomes

ररबह८सह८नस

4) Nucleus

नयनकियस

Correct Answer nucleus

eukaryotic cell -Transcription is the

process of synthesizing ribonucleic acid

(RNA)Synthesis takes place within the

nucleus of eukaryotic cells or in the

cytoplasm of prokaryotes and converts

the genetic code from a gene in

deoxyribonucleic acid ( DNA ) to a

strand of RNA that then directs

proteinsynthesis

Q222 _________is caused by parasites

of the Plasmodium genus

पिाजमह८डियमजातीकहॳ परजीवी mdash- कहॳ कारणहहॴ 7-Feb-2017

Options

1) Dysentery

पहॳचचि

2) Malaria

मिहॳररया 3) Chickenpox

F A C E B O O K

P A G E h t t p w w w f a c e b o o k c o m s s c m e n t o r s o f f i c i a l P a g e | 51

FOR MORE UPDATES AND MORE MATERIAL DO LIKE OUR FACEBOOK PAGE httpwwwfacebookcomsscmentorsofficial

चहॳचक

4) Herpes

हहॳपपयस

Correct Answer Malaria

Q223 Carotene in fruits and vegetables

gives it which color

फिह८औरसनलजयोमनसितकहॳ रह८टीनउनहकह९नसारगपरदानकरताहहॴ 7-Feb-2017

Options

1) Green

हरा 2) Pink

गिाबी 3) Orange

नारगी 4) Blue

नीिा Correct Answer Orange

Q224 Equus Caballus is the scientific

name of

एकवसकहॴ बहॳिस mdashmdashndash कापवजञाननकनामहहॴ 7-Feb-2017

Options

1) Horse

घह८िा 2) Zebra

ज़हॳिा 3) Donkey

गधा 4) Buffalo

भस

Correct Answer Horse

Q225 Elapidae Naja is the scientific name of

एिीपीिीनाजा mdashmdash- कावहॴजञाननकनामहहॴ 8-Feb-2017

Options

1) Cobra

कह८बरा 2) Elephant

हािी 3) Eagle

ग ि

4) Owl

उमि Correct Answer Cobra

Q226 Which disease is caused due to

deficiency of Iron

िह८हकीकमीकहॳ कारणकह९नसारह८गहह८ताहहॴ 8-Feb-

2017

Options

1) Beriberi

बहॳरीबहॳरी 2) Tetany

टहॳटनी 3) Kwashiorkor

कवािीऔरकर

4) Anaemia

रकतामपता Correct Answer Anaemia

Beriberi is a disease caused by a vitamin

B-1 deficiency also known as thiamine

deficiency

Tetany can be the result of an

electrolyte imbalance Most often itrsquos a

dramatically low calcium level also

known as hypocalcemia Tetany can also

be caused by magnesium deficiency or

too little potassium Having too much

acid (acidosis) or too much alkali

(alkalosis) in the body can also result in

tetany

Kwashiorkor also known as

ldquoedematous malnutrition It is a form of

malnutrition caused by a lack of protein

in the diet

Anaemia means that you have fewer red

blood cells than normal or you have less

F A C E B O O K

P A G E h t t p w w w f a c e b o o k c o m s s c m e n t o r s o f f i c i a l P a g e | 52

FOR MORE UPDATES AND MORE MATERIAL DO LIKE OUR FACEBOOK PAGE httpwwwfacebookcomsscmentorsofficial

haemoglobin than normal in each red

blood cell

Q227 is a leaf where the leaflets are

arranged along the middle vein

mdashndashएकपततीहहॴजहापतरकह८कीरचनाक ररयालिराकहॳ आसपासहह८तीहहॴ 8-Feb-2017

Options

1) Pinnately compound leaf

पपनहॳटिीसयकतपतती 2) Palmately compound leaf

पामहॳटिीसयकतपतती 3) Compound leaf

सयकतपतती 4) Simple leaf

साधारणपतती Correct Answer Pinnately compound

leaf

Q228 Haustoria or sucking roots are

found in which of the following

हह८सटह८ररयायाचसनहॳवािीजड़हॳनननननलिखितमसहॳककसमपाईजातीहहॴ 8-Feb-2017

Options

1) Wheat

गहॳह

2) Mango

आम

3) Chestnut

चहॳसटनट

4) Cuscuta

कसकयटा Correct Answer Cuscuta

Haustorial roots -The roots of parasitic

plants which penetrate into the host

tissues to absorb nourishment are

called haustorial roots hellip Also known as suckingor parasitic roots

Q229 Equs Asinus is the scientific name

of

एकवसएलसनस mdashmdashndash कावहॴजञाननकनामहहॴ 8-

Feb-2017

Options

1) Donkey

गधा 2) Cow

गाय

3) Deer

टहरन

4) Kangaroo

कगा

Correct Answer Donkey

Q230 Ficus benghalensis is the scientific name of

फाईकसबहॳनगहॳिहॳलसस mdashndash कापवजञाननकनामहहॴ 8-Feb-2017

Options

1) Banyan

बरगद

2) Pineapple

अनानास

3) Babul

बबि

4) Tulsi

तिसी Correct Answer Banyan

Q231 Equus burchellii is the scientific name of

एकवसबचिी mdashmdash- कापवजञाननकनामहहॴ 8-Feb-2017

Options

1) Horse

घह८िा 2) Zebra

जहॳिा 3) Buffalo

F A C E B O O K

P A G E h t t p w w w f a c e b o o k c o m s s c m e n t o r s o f f i c i a l P a g e | 53

FOR MORE UPDATES AND MORE MATERIAL DO LIKE OUR FACEBOOK PAGE httpwwwfacebookcomsscmentorsofficial

भस

4) Ass

गधा Correct Answer Zebra

Page 14: COMPILATION OF ALL 72 SETS OF BIOLOGY SSC CHSL-2016 · OF BIOLOGY SSC CHSL-2016 PREPARED BY : SSC MENTORS BIOLOGY SPECIAL . F A C E B O O K P A G E : h t t p : / / w w w . f a c e

F A C E B O O K

P A G E h t t p w w w f a c e b o o k c o m s s c m e n t o r s o f f i c i a l P a g e | 13

FOR MORE UPDATES AND MORE MATERIAL DO LIKE OUR FACEBOOK PAGE httpwwwfacebookcomsscmentorsofficial

one organism is inhibited or destroyed

while the other organism remains

unaffected

Commensalism an association between

two organisms in which one benefits and

the other derives neither benefit nor

harm

Q51 Pneumonia affects which of the

following organs of human body

ननमह८ननयामानविरीरकहॳ नननननलिखितमसहॳककसअगकह८परभापवतकरताहहॴ

15-Jan-2017

Options

1)Kidneys

गद

2)Lungs

फहॳ फड़हॳ 3) Throat

गिहॳ 4) Liver

यकत

Correct Answer Lungs

When the germs that cause pneumonia

reach your lungs the lungsrsquo air sacs

(alveoli) become inflamed and fill up

with fluid This causes the symptoms of

pneumonia such as a cough fever

chills and trouble breathing When you

have pneumonia oxygen may have

trouble reaching your blood

Q52 Mendel is known as

मििकह८ mdashmdash- कहॳ पमजानाजाताहहॴ 15-Jan-2017

Options

1) Father of Physiology

िरीरकियािासतरकहॳ जनक

2) Father of Geology

भगभयिासतरकहॳ जनक

3) Father of Genetics

जहॳनहॳटटकसकहॳ जनक

4) Father of Biology

जीविासतरकहॳ जनक

Correct Answer Father of Genetics

Q53 Which of the following are also

known as Suicidal bag of Cells

ननननलिखितमसहॳककसहॳआतमहतयाकरनहॳवािीकह८लिकाओकाबहॴगकहाजाताहहॴ

15-Jan-2017

Options

1) Lysosomes

िायसोसह८म

2) Lycosome

िायकह८सह८म

3) Nucleus

नालभक

4) Chromosome

िह८मह८सह८म

Correct Answer Lysosomes

Q54 Mesothelioma is a type of cancer

The most common area affected in it is

the lining of the ________

लमज़ह८िहॳिहॳलमयाक सरकाएकपरकारहहॴ इससहॳपरभापवतहह८नहॳवािासबसहॳसामानयकषहॳतर mdash

mdashmdash काअसतरहहॴ 15-Jan-2017

Options

1)Heart

हदय

2)Brain

मनसतषक

3)Stomach

आमािय

4)Lungs

फहॳ फड़हॳ Correct Answer lungs

Asbestos exposure is the main cause of

pleural mesothelioma When asbestos

fibers are breathed in they travel to the

F A C E B O O K

P A G E h t t p w w w f a c e b o o k c o m s s c m e n t o r s o f f i c i a l P a g e | 14

FOR MORE UPDATES AND MORE MATERIAL DO LIKE OUR FACEBOOK PAGE httpwwwfacebookcomsscmentorsofficial

ends of small air passages and reach the

pleura where they can cause

inflammation and

scarring

Q55 Which one of the following is an

insectivorous plant

नननननलिखितमसहॳकह९नसाएकककटाहरीवनसपनतहहॴ

15-Jan-2017

Options

1) Utricularia

यटरीकिहॳररया 2) Sequoia

सहॳकयओइया 3) Nostoc

नॉसटह८क

4) Bryophyta

िायह८फाईटा Correct Answer Utricularia

Q56 ______________ is a

multibranched polysaccharide of

glucose that serves as a form of energy

storage in animals and fungi

mdashmdashगिकह८जकाएकबहिािायकतपह८िीसहॳकहॳ राइिहहॴ जह८जानवरोऔरकवकमउजायभणिारणकहॳ एक पमकाययकरताहहॴ 15-Jan-2017

Options

1) Cellulose

सहॳमयिह८ज

2) Glycogen

गिायकह८जन

3) Pectin

पहॳनकटन

4) Chitin

चीटटन

Correct Answer Glycogen

Q57 The largest gland of the human

body is

mdashmdashmdashमानविरीरकीसबसहॳबड़ीगरिीहहॴ 16-Jan-2017

Options

1) Pancreas

अगयािय

2) Thyroid

िायरॉइि

3) Large Intestine

बड़ीआत

4) Liver

यकत

Correct Answer Liver

Q58 Photosynthesis in plants takes

place in

वनसपनतयोमपरकािसशिहॳषणकीकियाहह८तीहहॴ

16-Jan-2017

Options

1) Stem

तना 2) Leaves

पनततयाा 3) Roots

जड़हॳ 4) Flower

फि

Correct Answer Leaves

During this reaction carbon dioxide

and water are converted into glucose

and oxygen The reaction requires light

energy which is absorbed by a green

substance called

chlorophyll Photosynthesis takes place

in leaf

cells These contain chloroplasts which

are tiny objects containing chlorophyll

F A C E B O O K

P A G E h t t p w w w f a c e b o o k c o m s s c m e n t o r s o f f i c i a l P a g e | 15

FOR MORE UPDATES AND MORE MATERIAL DO LIKE OUR FACEBOOK PAGE httpwwwfacebookcomsscmentorsofficial

Q59 Insects that transmit diseases are

known as

जह८कीड़हॳरह८गसचाररतकरतहॳहहॴ उनह mdashmdash-

कहॳ नामसहॳजानाजाताहहॴ 16-Jan-2017

1)Pathogens

रह८गज़नक

2) Vectors

वहॳकटर

3) Drones

परजीवी 4)Scalars

अटदषट

Correct Answer Vectors

A vector is an organism that does not

cause disease itself but which spreads

infection by conveying pathogens from

one host to another Species of mosquito

for example serve as vectors for the

deadly disease Malaria

Q60 Which is the second largest gland

of Human body

मानविरीरकीदसरीसबसहॳबड़ीगरिीकह९नसीहहॴ

SSC CHSL Science (biology)

2016 Question Paper

16-Jan-2017

Options

1) Liver

यकत

2) Large Intestine

बड़ीआत

3) Thorax

छाती 4) Pancreas

अगनयािय

Correct Answer Pancreas

Q61 Annona squamosa is the scientific

name of

एनह८नासकवामह८सा (Annona squamosa) mdash

mdashmdash कावहॴजञाननकनामहहॴ 16-Jan-2017

Options

1) Custard Apple

सीताफि

2) Papaya

पपीता 3) Babhul

बबि

4) Drumstick

सहजन

Correct Answer Custard Apple

Q62 The disease Beri Beri is caused due

to the deficiency of which of the

following

बहॳरीबहॳरीरह८गनननननलिखितमसहॳककसकीकमीकहॳकारणहह८ताहहॴ

16-Jan-2017

Options

1) Vitamin B2

पवटालमन B2

2) Vitamin B1

पवटालमन B1

3) Vitamin B12

पवटालमन B12

4) Vitamin E

पवटालमन E

Correct Answer Vitamin B1

Beriberi is a disease caused by a vitamin

B-1 deficiency also known as thiamine

deficiency

Q63 Chlorophyll was first isolated and

named by

किह८रह८कफिकह८ mdash-

दवारापहिहॳपिकऔरनालमतककयागया 16-Jan-2017

F A C E B O O K

P A G E h t t p w w w f a c e b o o k c o m s s c m e n t o r s o f f i c i a l P a g e | 16

FOR MORE UPDATES AND MORE MATERIAL DO LIKE OUR FACEBOOK PAGE httpwwwfacebookcomsscmentorsofficial

Options

1) Caventou

कहॳ वहॳत 2) Pelletier

पहॳिहॳटटयर

3) Chlorophyll

किह८रह८कफि

4) Caventou and Pelletier

कहॳ वहॳतऔरपहॳिहॳटटयर

Correct Answer Caventou and Pelletier

Chlorophyll was first isolated and

named by

Joseph Bienaimeacute Caventou and Pierre

Joseph Pelletier in 1817 The presence of

magnesium in chlorophyll was

discovered in 1906 and was the first

time that magnesium had been detected

in living tissue

Q64 Which of the following organisms

does not fit into the Cell Theory

नननननलिखितमसहॳकह९नसाजीवकह८लिकालसदातअन पनहीहहॴ

16-Jan-2017

Options

1) Bacteria

बहॴकटीररया 2) Virus

वायरस

3) Fungi

कवक

4) Plants

पह९धहॳ Correct Answer Virus

The bottom line is that viruses are not

alive and not related to cells in any way

The cell theory states that all living

things are made of cells cells are the

basic units of structure and function of

living things and that all cells come

from other cells Since viruses are not

made of cells and do not use cells in any

of their processes they are not related to

the cell theory

Q65 Which of these is not a

macronutrient for Plants

नननननलिखितमसहॳकह९नसापह९धह८कहॳ लिएमिह८नयटरीएटनहीहहॴ

SSC CHSL Science (biology) 2016

Question Paper

17-Jan-2017

Options

1) Nitrogen

नाइटरह८जन

2) Phosphorus

फासफह८रस

3) Potassium

पह८टालसयम

4) Chlorine

किह८रीन

Correct Answer Chlorine

In relatively large amounts the soil

supplies nitrogen phosphorus

potassium calcium magnesium and

sulfur these are often called the

macronutrients In relatively small

amounts the soil supplies iron

manganese boron molybdenum

copper zinc chlorine and cobalt the

so-called micronutrients

Q66 Name the respiratory organs of

insects

कीटह८मनसतिशरवसनअगनामकानामहहॴ

17-Jan-2017

Options

1) Skin

तवचा 2) Body Surface

िरीरकीसतह

F A C E B O O K

P A G E h t t p w w w f a c e b o o k c o m s s c m e n t o r s o f f i c i a l P a g e | 17

FOR MORE UPDATES AND MORE MATERIAL DO LIKE OUR FACEBOOK PAGE httpwwwfacebookcomsscmentorsofficial

3) Gills

गिफड़हॳ 4) Tracheae

शरावस- निी Correct Answer Tracheae

Air enters the respiratory systems of

insects through a series of external

openings called

spiracles These external openings

which act as muscular valves in some

insects lead to the internal respiratory

system a densely networked array of

tubes called tracheae

Q67 The poisonous gas accidentally

released in Bhopal Gas Tragedy is

भह८पािगहॴसतरासदीमगितीसहॳमकतहईजहरीिीगहॴसिी

17-Jan-2017

1) Methane

मीिहॳन

2) Nitrous Oxide

नाइटरसऑकसाइि

3) Methyl Isocyanate

महॴचििआयसोसायनहॳट

4) Cyanogen

सायनह८जहॳन

Correct Answer Methyl Isocyanate

Q68 What does Trypsin do

टटरनपसनकयाकरताहहॴ

SSC CHSL Science (biology) 2016

Question Paper

17-Jan-2017

Options

1) Breaks down Carbohydrates

काबोहाइडरहॳटकापवघटनकरताहहॴ 2) Synthesizes proteins

परह८टीनकासििहॳषणकरताहहॴ 3) Breaks down fats

वसाकापवघटनकरताहहॴ 4) Breaks down proteins

परह८टीनकापवघटनकरताहहॴ Correct Answer Breaks down proteins

Trypsin is one of the three principal

digestive

proteinases the other two being pepsin

and

chymotrypsin In the digestive process

trypsin acts with the other proteinases

to break down dietary protein molecules

to their component

peptides and amino acids

A protease is any enzyme that performs

proteolysis protein catabolism by

hydrolysis of peptide bonds

Q69 Name the source from which

Aspirin is produced

उससरह८तकानामबताइए

नजससहॳएनसपररनकाउतपादनककयाजाताहहॴ

17-Jan-2017

Options

1) Willow bark

पविह८कीछाि

2) Oak Tree

ओककावकष

3) Acacia

बबि

4) Eucalyptus

नीिचगरी Correct Answer Willow bark

The compound from which the active

ingredient in aspirin was first derived

salicylic acid was found in the bark of a

willow tree in 1763 by Reverend

Edmund Stone of Chipping-Norton

Q70 Cannis Familiaris is the scientific

name of

कहॴ ननसफहॳ लमलियहॳररस mdash- कावहॴजञाननकनामहहॴ

17-Jan-2017

F A C E B O O K

P A G E h t t p w w w f a c e b o o k c o m s s c m e n t o r s o f f i c i a l P a g e | 18

FOR MORE UPDATES AND MORE MATERIAL DO LIKE OUR FACEBOOK PAGE httpwwwfacebookcomsscmentorsofficial

Options

1) Cat

बबमिी 2)Dog

कतता 3) Fox

िह८मड़ी 4) Wolf

भहॳडड़या Correct Answer Dog

Q71 Harmful bacteria in potable water

make the water

पीनहॳकहॳ पानीमनसतिघातकबहॴकटीररयाउसपानीकह८बनातहॳहहॴ 17-Jan-2017

Options

1) unfit to drink

पीनहॳकहॳ लिएअयह८गय

2) smelly

दगयनधयकत

3) Colored

रगीन

4) Turbid

मटमहॴिा Correct Answer unfit to drink

Q72 Musa paradisiaca is the scientific

name of which plant

मसापहॴराडिलसयाकाककसपह९धहॳकावहॴजञाननकनामहहॴ

17-Jan-2017

Options

1) Mango

आम

2) Wheat

गहॳह

3) Corn

भ ा 4) banana

कहॳ िा Correct Answer banana

Q73 Prawns belong to which family

झीगहॳककसपररवारकहॳ हह८तहॳहहॴ 17-Jan-2017

Options

1) Crustaceans

िसटहॳलियन

2)Fish

मछिी 3) Amphibians

अननफबबयस

4) Reptiles

रहॳपटाइमस

Correct Answer Crustaceans

Q74 Name the drug that is yielded from

Cinchona tree and is used to cure

malaria

उसऔषचधकानामबताइएनजसहॳलसगकह८नापहॳड़सहॳपरापतककयाजाताहहॴऔरनजसकाउपयह८गमिहॳररयाकहॳ उपचारमककयाजाताहहॴ 17-Jan-2017

Options

1) Camptothea

कहॴ नटह८चिया 2) Acuminata

एकयलमनहॳटा 3) Quinine

कनहॴन

4) Cinchonia

लसकह८ननया Correct Answer Quinine

Q75 Blood Circulation was discovered

by

रकतपररसचरणकी mdashmdashndash दवारािह८जकीिी 17-Jan-2017

Options

1) Mary Anderson

F A C E B O O K

P A G E h t t p w w w f a c e b o o k c o m s s c m e n t o r s o f f i c i a l P a g e | 19

FOR MORE UPDATES AND MORE MATERIAL DO LIKE OUR FACEBOOK PAGE httpwwwfacebookcomsscmentorsofficial

महॴरीएिरसन

2) Virginia Apgar

वनजयननयाएपगार

3) William Harvey

पवलियमहाव

4) Robert Feulgen

रॉबटयफ़यिजहॳन Correct Answer William Harvey

Q76 Vitamin A is also known as

पवटालमन A कह८ mdashmdash- कहॳ नामसहॳभीजानाजाताहहॴ SSC CHSL Science (biology) 2016

Question Paper

18Jan2017

Options

1) Thiamine

िायलमन

2) Riboflavin

ररबह८फिहॳपवन

3) Retinol

रहॳटटनॉि

4) Calciferol

कहॴ नमसफहॳ रह८ि

Correct Answer Retinol

Q77 Some roots called arise from an

organ other than the radicle

कछजड़हॳनजनह mdashmdashmdash कहाजाताहहॴ वहमिकहॳ अिावाककसीअनयअगसहॳउतपननहह८तीहहॴ 18Jan2017

Options

1) tap roots

मखयजड़

2) stilt roots

ि ाजड़

3) fibrous roots

रहॳिहॳदारजड़

4) adventitious roots

आकनसमकजड़

Correct Answer adventitious roots

Q78 Spiders belong to which class of

animals

मकडड़यापराणीवगीकरणकहॳ ककसवगयमआतीहहॴ 18Jan2017

Options

1) Arachnids

एरहॳकननडस

2) Aves

एपवस

3) Gastropods

गहॴसटरोपह८िस

4) Anthozoa

एिह८जआ

Correct Answer Arachnids

Q79 How many layers does Human

Skin have

मानवतवचामककतनीपरतहॳहह८तीहहॴ

18Jan2017

Options

1) 5

2) 7

3) 11

4) 3

Correct Answer 3

Skin has three layers The epidermis

the outermost layer of skin provides a

waterproof barrier and creates our skin

tone The dermis beneath the

epidermis contains tough connective

tissue hair follicles and sweat glands

The deeper subcutaneous tissue (

hypodermis ) is made of fat and

connective tissue

Q80 Allium Cepa is the scientific name

of

एलियमलसपपा mdashmdashndash कावहॴजञाननकनामहहॴ 18Jan2017

F A C E B O O K

P A G E h t t p w w w f a c e b o o k c o m s s c m e n t o r s o f f i c i a l P a g e | 20

FOR MORE UPDATES AND MORE MATERIAL DO LIKE OUR FACEBOOK PAGE httpwwwfacebookcomsscmentorsofficial

Options

1) Carrot

गाजर

2) Tomato

टमाटर

3) Potato

आि 4) Onion

पयाज़

Correct Answer Onion

Q81 DNA stands for

िीएनएकापणय प mdashmdash- हहॴ 18Jan2017

Options

1) Di Nucleic Acid

िाईनयनकिकएलसि

2) Deoxy Nucleic Acid

िीओकसीनयनकिकएलसि

3) Diribonucleic Acid

िाईराइबह८नयनकिकएलसि

4) Deoxyribonucleic Acid

िीऑकसीराइबह८नयनकिकएलसि

Correct Answer Deoxyribonucleic Acid

Q82 Organisms that generate energy

using light are known as

जह८जीवाणपरकािकाउपयह८गकरउजायउतपननकरतीहहॴ उनह mdashmdash कहॳ पमजानाजाताहहॴ

18Jan2017

Options

1) Chaemolithotrophs

ककमह८लििह८टरह८पस

2) Oligotrophs

ओलिगह८टरह८पस

3) Bacteria

बहॴकटीररया 4)Photoautotrophs

फह८टह८ओटह८टरह८पस

Correct Answer Photoautotrophs

An oligotroph is an organism that can

live in an environment that offers very

low levels of nutrients

Q83 Which drug is used as an

Antidepressant

ककसदवाएकहतािारह८धीकहॳ पमपयोगककयाजाताहहॴ Options

1) Oxybutynin

ओकसीलयटीनन

2)Tramadol

टरहॳमहॳिह८ि

3 ) Sumatriptan

समहॳटरीपटहॳन

4) Bupropion

लयपरह८पपयह८न

Correct Answer Bupropion

लयपरह८पपयह८न

Q84 The orange colour of carrot is

because of

गाजरकानारगीरगनननननलिखितमसहॳककसीएककीवजहसहॳहह८ताहहॴ 18Jan2017

Options

1) it grows in the soil

यहलम ीमउगतीहहॴ 2) Carotene

कहॴ रह८टीन

3) it is not exposed to sunlight

यहसययपरकािकहॳ सपकय मनहीआती 4) the entire plant is oranqe in colour

सनपणयपह९धानारगीरगकाहह८ताहहॴ Correct Answer Carotene

Q85 Snake venom is highly modified

saliva containing

F A C E B O O K

P A G E h t t p w w w f a c e b o o k c o m s s c m e n t o r s o f f i c i a l P a g e | 21

FOR MORE UPDATES AND MORE MATERIAL DO LIKE OUR FACEBOOK PAGE httpwwwfacebookcomsscmentorsofficial

सापकाजहरअततयाचधकसिह८चधतिारहह८तीहहॴनजसमहॳ mdashmdash- हह८ताहहॴ Options

l)Prototoxins

परह८टह८टॉनकसस

2)Neutrotoxins

नयटरोटॉनकसस

3)Zootoxins

जटॉनकसस

4)Electrotoxins

इिहॳकटरह८टॉनकसस

Correct Answer Zootoxins

जटॉनकसस

Q86 Which type of pathogen causes the

water-borne disease Schistosomiasis

ककसपरकारकारह८गज़नकजिजननतरह८गलससटह८सह८लमलससकाकारणबनताहहॴ

18Jan2017

Option

1) Parasitic

परजीवी 2)Protozoan

परह८टह८जआ

3) Bacterial

बहॴकटीररयि

4) Viral

वायरि

Correct Answer Parasitic

Schistosomiasis also known as snail

fever and bilharzia is a disease caused

by parasitic

flatworms called schistosomes

Q87 Prothrombin responsible for

clotting of blood is released by

परह८िह८ननबन

जह८रकतकािककाजमनहॳकहॳ लिएनजनमहॳदारहहॴ mdashndash

कहॳ दवारासतरापवतककयाजाताहहॴ

19Jan2017

Options

1) Small Intestine

छह८टीआत

2) Blood Platelets

रकतपिहॳटिहॳटस

3) Large Intestine

बड़ीआत

4Heart

हदय

Correct Answer Blood Platelets

Q88 Acacia arabica is the scientific

name of

अकहॳ लियाअरहॳबबका mdashmdashndash कावहॴजञाननकनामहहॴ 19-Jan-2017

Options

1) Neem

नीम

2) Teak

सागह९न

3) Babhul

बबि

4) Pomegranate

अनार

Correct Answer Babhul

Q89 Cannis Vulpes is the scientific

name of

कहॴ ननसवनमपस mdashmdash- कावहॴजञाननकनामहहॴ 19-Jan-2017

Options

1) Dog

कतता 2) Wolf

भहॳडड़या 3) Fox

िह८मड़ी 4) Hyena

िाकिबगघा

F A C E B O O K

P A G E h t t p w w w f a c e b o o k c o m s s c m e n t o r s o f f i c i a l P a g e | 22

FOR MORE UPDATES AND MORE MATERIAL DO LIKE OUR FACEBOOK PAGE httpwwwfacebookcomsscmentorsofficial

Correct Answer Fox

Q90 The beetroot is the portion of the

beet plant

चकदरपह९धहॳका mdashmdashndash भागहहॴ 19-Jan-2017

Options

1) tap root

मखयजड़

2) Adventitious

आकनसमक

3) bulb of the stem

तनहॳकाकद

4) Rhizome

परकद

Correct Answer tap root

Q91 What is the basic unit of heredity

आनवलिकताकीबननयादीइकाईकयाहहॴ 19-Jan-2017

Options

1) DNA

िीएनए

2) RNA

आरएनए

3) Chromosome

िह८मह८सह८म

4) Gene

जीन

Correct Answer gene

Genes are the units of heredity and are

the instructions that make up the bodyrsquos

blueprint They code for the proteins

that determine virtually all of a personrsquos

characteristics Most genes come in

pairs and are made of strands of genetic

material called deoxyribonucleic acid

or DNA

Q92 Lungs are the primary organs of

फहॳ फड़हॳmdashndashकहॳ परािलमकअगहहॴ

19-Jan-2017

Options

1) Digestion

पाचन

2) Constipation

कलज

3) Perspiration

पसीना 4)Respiration

शवसन

Correct Answer Respiration

Q93 Sugarcane is a type of

गननाएकपरकारका mdash- हहॴ 20-Jan-2017

Options

1)creeper

िता 2)tree

पहॳड़

3)shrub

झाड़ी 4)grass

घास

Correct Answer grass

Q94 Who is commonly known as ldquothe

Father of Microbiologyrdquo

सामानयत ldquo सकषमजीवपवजञानकहॳ जनक lsquo

कहॳ नामसहॳककसहॳजानाजातहहॴ 20-Jan-2017

Options

1) Robert Hooke

रॉबटयहक

2) Antonie Philips van Leeuwenhoek

एटह८नीकफलिपवानमयएनहह८क

3) Carl Linnaeus

काियिीनाईयस

4) Charles Darwin

चामसयिापवयन

F A C E B O O K

P A G E h t t p w w w f a c e b o o k c o m s s c m e n t o r s o f f i c i a l P a g e | 23

FOR MORE UPDATES AND MORE MATERIAL DO LIKE OUR FACEBOOK PAGE httpwwwfacebookcomsscmentorsofficial

Correct Answer Antonie Philips van

Leeuwenhoek

Q95 For the aquatic organisms the

source of food is

जिीयजीवाणकािाघसरह८तहहॴ 20-Jan-2017

Options

1) Phytoplankton

फायटह८पिहॳकटन

2) Sea Weed

समदरीिहॴवाि

3)Aqua plankton

एकवापिहॳकटन

4) Zooplankton

जपिहॳकटन

Correct Answer Phytoplankton

Q96 Haemoglobin has the highest

affinity with which of the following

हीमह८गिह८बबनकीननननमसहॳककसकहॳ सािउततमसमानताहहॴ

20-Jan-2017

Options

1)SO2

2)CO2

3)CO

4)NO2

Correct Answer CO

It has a greater affinity for hemoglobin

than oxygen does It displaces oxygen

and quickly binds so very little oxygen

is transported through the body cells

Q97 Who developed the theory of

Evolution

उदपवकासकालसदातककसनहॳपवकलसतककया

20-Jan-2017

Options

1) Charles Darwin

चामसयिापवयन

2) Isaac Newton

आयजहॳकनयटन

3) Pranav Mistry

परणवलमसतरी 4) Galileo Galilei

गहॳलिलियह८गहॳिीिी Correct Answer Charles Darwin

Q98 The primary function of RNA is

RNA कापरािलमककाययहह८ताहहॴ 20-Jan-2017

Options

1) Photosynthesis

परकािसशिहॳषण

2) Protein Synthesis

परह८टीनसशिहॳषण

3) Replication

परनतकनतबनाना 4) Translation

अनवादकरना Correct Answer Protein Synthesis

There are two main functions of RNA

It assists DNA by serving as a messenger

to relay the proper genetic information

to countless numbers of ribosomes in

your body The other main function of

RNA is to select the correct amino acid

needed by each ribosome to build new

proteins for your body

Q99 ______is the movement of

molecules across a cell membrane from

a region of their lower concentration to

a region of their higher concertration

उचचसादरताकहॳ कषहॳतरसहॳउसकीकमसादरतावािहॳकषहॳतरकीतरफएककह८लिकाखझमिीकहॳ माधयमसहॳहह८नहॳवािाअणओकहॳ सचिनकह८ mdash- कहतहॳहहॴ Options

1) Diffusion

पवसरण

2) Osmosis

ऑसमह८लसस

F A C E B O O K

P A G E h t t p w w w f a c e b o o k c o m s s c m e n t o r s o f f i c i a l P a g e | 24

FOR MORE UPDATES AND MORE MATERIAL DO LIKE OUR FACEBOOK PAGE httpwwwfacebookcomsscmentorsofficial

3) Active Transport

सकियआवागमन

4) Passive Transport

नननषियआवागमन

Correct Answer Active Transport

Q100 Study of classification of

organisms is known as 20-Jan-2017

जीवाणओकहॳ वगीकरणकहॳ अधययनकह८ mdash-

कहाजाताहहॴ Options

1) Serpentology

सपरहॳटह८िह८जी 2) Virology

वायरह८िह८जी 3) Taxonomy

टहॴकसोनह८मी 4) Physiology

कफनज़यह८िह८जी Correct Answer Taxonomy

Q101 Photosynthesis takes place inside

plant cells in

परकािसशिहॳषणवनसपनतकह८लिकामनसति mdash

mdashmdash महह८ताहहॴ 20-Jan-2017

Options

1) Ribosomes

राइबह८सह८नस

2) Chloroplasts

किह८रह८पिासट

3) Nucleus

नयकलियम

4) Mitochondria

माईटह८कोडडरया Correct Answer Chloroplasts

Q102 ______ is the cell organelle in

which the biochemical processes of

respiration and energy production

occur

mdashmdash- वहकह८लिकाअगहहॴ नजसमहॳशवसनऔरउजायउतपादनकहॳ जहॴसीजहॴवरासायननकपरकियायहह८तीहहॴ 20-Jan-2017

Options

1) Mitochondria

माइटह८कोडडरया 2) Chloroplast

किह८रह८पिासट

3) Ribosomes

राइबह८सह८नस

4) Nucleus

नयकिीयस

Correct Answer Mitochondria

Q103 Which non-flowering spore

bearing plants have roots

ककसफिनिगनहॳवािहॳऔरबीजाणधारकपह९धह८कीजड़हॳहह८तीहहॴ 21-Jan-2017

Options

1) Mosses

मह८सहॳस

2) Angiosperms

एननजयह८सपनसय 3) Ferns

फनसय 4) Gymnosperms

नजननह८सपनसय Correct Answer ferns

Q104 Which of the following is an

excretory organ of cockroach

नननननलिखितमसहॳकह९नसानतिच हॳकाउतसजयनअगहहॴ

21-Jan-2017

Options

F A C E B O O K

P A G E h t t p w w w f a c e b o o k c o m s s c m e n t o r s o f f i c i a l P a g e | 25

FOR MORE UPDATES AND MORE MATERIAL DO LIKE OUR FACEBOOK PAGE httpwwwfacebookcomsscmentorsofficial

1) Malphigian Tubules

मनमफनजयनटयबमस

2) Nephridia

नहॳकफरडिया 3) Coxal Gland

कह८कसिगरचिया 4) Green Gland

गरीनगरचिया Correct Answer Malphigian Tubules

Q105 Evaporation of water takes place

in which part of plants

पानीकहॳ वाषपीकरणकीकियापह९धोकहॳ ककसभागसहॳहह८तीहहॴ 21-Jan-2017

Options

1) Stem

तना 2) Stomata

सटह८मटा 3) Branch

िािाए

4) Fruit

फि

Correct Answer Stomata

Evaporation accounts for the movement

of water to the air from sources such as

the soil canopy interception and

waterbodies Transpiration accounts for

the movement of water within a plant

and the subsequent loss of water as

vapour through stomata in its leaves

Q106 A is the fleshy spore-bearing

fruiting body of a fungus

mdashmdashndashकवककामासि

बीजाणधारणकरनहॳवािाफिनहॳवािाअगहहॴ 21-

Jan-2017

Options

1) aloe vera

एिह८वहॳरा 2) Coral

मगा 3) Cactus

कहॴ कटस

4) Mushroom

ककरमतता Correct Answer mushroom

Q107 Which of the following is a fungal

disease

नननननलिखितमसहॳकह९नसाफफदसहॳहह८नहॳवािाएकरह८ग हहॴ

21-Jan-2017

Options

1) Dermatitis

तवचािह८ध

2) Cholera

हहॴजा 3) Jaundice

पीलिया 4) Indigofera

इननिगह८फहॳ रा Correct Answer Dermatitis

Dermatitis also known as eczema is a

group of diseases that results in

inflammation of the skin These diseases

are characterized by itchiness red skin

and a rash In cases of short duration

there may be small blisters while in

long-term cases the skin may become

thickened

Q108 In which form is glucose stored in

our body

हमारहॳिरीरमगिकह८जकासचयककस पमककयाजाताहहॴ

21-Jan-2017

Options

1) Insulin

F A C E B O O K

P A G E h t t p w w w f a c e b o o k c o m s s c m e n t o r s o f f i c i a l P a g e | 26

FOR MORE UPDATES AND MORE MATERIAL DO LIKE OUR FACEBOOK PAGE httpwwwfacebookcomsscmentorsofficial

इसलिन

2) Glucose

गिकह८ज

3) Glycogen

गिायकह८जहॳन

4) Fat

वसा Correct Answer Glycogen

Excess glucose is stored in the liver as

the large compound called glycogen

Glycogen is a polysaccharide of glucose

but its structure allows it to pack

compactly so more of it can be stored in

cells for later use

Q109 Where do plants synthesize

protein from

पह९धहॳपरह८टीनसशिहॳषणकहासहॳकरतहॳहहॴ

Options

1) Fatty Acids

वसाऐलसि

2) Sugar

िकर

3) Amino Acids

एलमनह८ऐलसि

4) Starch

सटाचय Correct Answer Amino Acids

Q110 Which part of the brain is

responsible for triggering actions like

thinking intelligence memory and

ability to learn

मनसतषककाकह९नसाटहससासह८चनहॳ बनधदमानी याददाशतऔरसीिनहॳकीकषमताजहॴसीकियाओकह८परहॳररतकरताहहॴ 21-Jan-2017

Options

1) Diencephalon

िायएनसहॳफहॳ िह८न

2) Hypothalamus

हयपह८िहॳिहॳमस

3) Cerebrum

सहॳरहॳिम

4) Control

कटरह८ि

Correct Answer Cerebrum

Q111 Which of the following is also

known as the Biochemical Laboratory

of the Human Body

नननननलिखितमसहॳककसहॳमानविरीरकीजहॴवरसायनपरयह८गिािाभीकहाजाताहहॴ 21-Jan-2017

Options

1) Small Intestine

छह८टीआत

2)Brain

मनसतषक

3) Pancreas

अगनयािय

4) Liver

नजगर

Correct Answer Liver

The liver makes bile that will help

emulsify and digest the fats we eat

The liver takes toxic substances and

convert them using enzymes the liver

cells makes into a non toxic form so the

body can dispose of them

The liver also converts fats protein and

carbohydrates into glucose which is the

energy source for our cells to use

The liver takes amino acids and makes

proteins by combining them

Q112 The yellow colour of human urine

is due to

मानवमतरकापीिारग mdashndash कीवजहसहॳहह८ताहहॴ 22-

Jan-2017

Options

1) Bile Salts

F A C E B O O K

P A G E h t t p w w w f a c e b o o k c o m s s c m e n t o r s o f f i c i a l P a g e | 27

FOR MORE UPDATES AND MORE MATERIAL DO LIKE OUR FACEBOOK PAGE httpwwwfacebookcomsscmentorsofficial

पपततनमक

2) Cholesterol

कह८िहॳसटरह८ि

3) Lymph

लिनफ

4) Urochrome

यरह८िह८म

Correct Answer Urochrome

Urobilin or urochrome is the chemical

primarily responsible for the yellow

color of urine

Q113 The wilting of plants takes place

due to

पह९धह८कालिचििहह८नाकी mdashmdash- कीवजहसहॳहह८ताहहॴ 22-Jan-2017

Options

1)Photosynthesis

परकािसशिहॳषण

2) Transpiration

वाषपह८तसजयन

3) Absorption

अविह८षण

4) Respiration

शरवसन

Correct Answer Transpiration

Wilting is the loss of rigidity of non-

woody parts of plants This occurs when

the turgor pressure in non-lignified

plant cells falls towards zero as a result

of diminished water in the cells

Q114 Bovidae Ovis is the scientific name of

बह८पविीओपवस mdashndash कावहॴजञाननकनामहहॴ 22-Jan-2017

Options

1) Goat

बकरी 2) Cow

गाय

3) Buffalo

भहॳस

4) Sheep

भहॳड़

Correct Answer Sheep

Q115 Plants get their energy to produce

food from which of the following

पह८धहॳभह८जनकाननमायणकरनहॳकहॳ लिएनननननलिखितमसहॳककससहॳउजायपरापतकरतहॳहहॴ

22-Jan-2017

Options

1) Photosynthesis

परकािसशिहॳषण

2)Bacteria

बहॴकटीररया 3)Fungi

कवक

4)Sun

सयय Correct Answer Sun

Q116 Which of the following is secreted

by the liver

नननननलिखितमसहॳककसकासरावनजगरसहॳहह८ताहहॴ

22-Jan-2017

Options

1) Glucose

गिकह८ज

2) Iodine

आयह८िीन

3) Cortisol

काटटरयसह८ि

4) Bile

पपतत

Correct Answer Bile

The liver makes bile that will help

emulsify and

digest the fats we eat

F A C E B O O K

P A G E h t t p w w w f a c e b o o k c o m s s c m e n t o r s o f f i c i a l P a g e | 28

FOR MORE UPDATES AND MORE MATERIAL DO LIKE OUR FACEBOOK PAGE httpwwwfacebookcomsscmentorsofficial

Q117 Ferns belong to which division of

plants

फनसयपह९धह८कहॳ ककसभागमआतहॳहहॴ

22-Jan-2017

Options

1) Gymnosperms

नजननह८सपनसय 2) Angiosperms

एनजयह८सपनसय 3) Thallophyta

िहॴिह८फाईटा 4)Pteridophyta

टहॳररिह८फाईटा Correct Answer Pteridophyta

Q118 Who invented Antibiotics

एटीबायह८टटककाअपवषकारककसनहॳककयािा

22-Jan-2017

Options

1) Joseph Lister

जह८सहॳफलिसटर

2) William Harvey

पवलियमहाव

3) Robert Knock

रॉबटयनॉक

4)Alexander Fleming

अिहॳकज़िरफिहॳलमग

Correct Answer Alexander Fleming

Q119 Milbecycin is used in the

eradication of

लममबहॳसायलसनका mdashndash

मउनमिनमपरयह८गककयाजाताहहॴ 22-Jan-2017

Options

1) Agricultural Fungus

कपषकवक

2) Agricultural Pests

कपषकीटक

3) Agricultural Herbs

कपषिाक

4)Agricultural Weeds

कपषननराना Correct Answer Agricultural Pests

Milbemycin oxime is a veterinary drug

from the group of milbemycins used as

a broad spectrum antiparasitic It is

active against worms and mites(insects

Q120 Intestinal bacteria synthesizes

which of the following in the human

body

मानविरीरमआतोकहॳ बहॴकटीररयानननननलिखितमसहॳककसकासशिहॳषणकरतहॳहहॴ 22-Jan-2017

Options

1) Vitamin K

पवटालमन K

2) Proteins

परह८टीन

3) Fats

वसा 4) Vitamin D

पवटालमन D

Correct Answer Vitamin K

Q121 is the study of the physical form

and external structure of plants

mdashmdash-

मपह९धह८काभहॴनतक पऔरबाहरीसरचनाकाआदयाककयाजाताहहॴ 22-Jan-2017

Options

1) Physiology

कफनजयह८िह८जी 2) Anatomy

िरीररचनापवजञान

3) Phytomorphology

फाईटह८मह८फह८िह८जी 4)Cytology

कह८लिकापवजञान

Correct Answer Phytomorphology

F A C E B O O K

P A G E h t t p w w w f a c e b o o k c o m s s c m e n t o r s o f f i c i a l P a g e | 29

FOR MORE UPDATES AND MORE MATERIAL DO LIKE OUR FACEBOOK PAGE httpwwwfacebookcomsscmentorsofficial

Q122 Which of the following is a

structural and functional unit of

kidneys

नननननलिखितमसहॳकह९नसीगदोकीसरचनातमकऔरकाययकरीईकाईहहॴ

22-Jan-2017

Options

1) Renette Cells

रहॳनहॳटकह८लिकाए

2) Flame Cells

फिहॳमकह८लिकाए

3) Nephrites

नहॳफ़राइटस

4)Nephrons

नहॳफरोस

Correct Answer Nephrons

Nephron functional unit of the kidney

the structure that actually produces

urine in the process of removing waste

and excess substances from the blood

There are about 1000000 nephrons in

each human kidney

Q123 Which of the following is the

largest part of the human brain

नननननलिखितमसहॳकह९नसामानवमनसतषककासबसहॳबड़ाटहससाहहॴ

23-Jan-2017

Options

1) Ribs

पसलियाा 2) Cerebrum

सहॳरहॳिम

3) Pons

पोस

4)Thalamus

िहॴिहॳमस

Correct Answer Cerebrum

The cerebrum is the largest part of the

human brain making up about two-

thirds of the brainrsquos mass It has two

hemispheres each of which has four

lobes frontal parietal temporal and

occipital

Q124 The auxiliary buds

सहायककालियाmdashndash 23-Jan-2017

Options

1) grow endogenously from the pericycle

पहॳरीसाईककिसहॳअनतजातयपवकलसतहह८ताहहॴ 2) arise endogenously from the main

growing point

मिवपदसहॳअनतजातयउठताहहॴ 3) is an embryonic shoot located in the

axil of a leaf

एकभरणिटहहॴजह८एकपततीकहॳ अकषपरनसतिहह८ताहहॴ 4)arise exogenously from the epidermis

एपपिलमयससहॳबटहजातयतरीकहॳ सहॳउठताहहॴ Correct Answer is an embryonic shoot

located in the axil of a leaf

Q125 Which of the following is a viral

disease

इनमहॳसहॳकह९सीएकवायरिबीमारीहहॴ

23-Jan-2017

Options

1) Polio

पह८लियह८ 2) Tetanus

धनसतनभ

3) Leprosy

कषठरह८ग

4) Plague

पिहॳग

Correct Answer Polio

A viral disease (or viral infection)

occurs when an organismrsquos body is

invaded by pathogenic viruses and

infectious virus particles (virions) attach

to and enter susceptible cells

F A C E B O O K

P A G E h t t p w w w f a c e b o o k c o m s s c m e n t o r s o f f i c i a l P a g e | 30

FOR MORE UPDATES AND MORE MATERIAL DO LIKE OUR FACEBOOK PAGE httpwwwfacebookcomsscmentorsofficial

Poliomyelitis often called polio or

infantile paralysis is an infectious

disease caused by the poliovirus

Tetanusmdash A serious bacterial infection

that causes painful muscle spasms and

can lead to death

Leprosy also known as Hansenrsquos

disease (HD) is a long-term infection by

the bacterium Mycobacterium leprae or

Mycobacterium lepromatosis

Plague is an infectious disease caused by

the bacterium Yersinia pestis

Symptoms include fever weakness and

headache

Q126 Which organisms can help to

carry out Vermicomposting

कह९नसाजीववमीकनपह८नसटगममददकरताहहॴ

23-Jan-2017

Options

1) Nitrifying Bacteria

नाईटरीफाईगबहॴकटीररया 2) Earthworms

कहॴ चऐ

3) Algae

िहॴवि

4) Fungus

कवक

Correct Answer Earthworms

Q127 Contraction of heart is also

known as

हदयकहॳ सकचनकह८ mdash- भीकहाजाताहहॴ 23-Jan-

2017

Options

1) Systole

लससटह८ि

2) Aristotle

अरसत

3) Diastole

िायसटह८ि

4) Lub

मयब

Correct Answer Systole

Diastole is the part of the cardiac cycle

when the heart refills with blood

following systole (contraction)

Ventricular diastole is the period during

which the ventricles are filling and

relaxing while atrial diastole is the

period during which the atria are

relaxing

Q128 Azadirachta indica is the

botanical name of which of the

following

अजाटदराचताइडिकानननननलिखितमसहॳककसकावानसपनतनामहहॴ

23-Jan-2017

Options

1) Rose plant

गिाबकापह९धा 2) Apple tree

सहॳबकापहॳड़

3) Neem

नीम

4)Mango

आम

Correct Answer Neem

Q129 Which of the following is the

main end product of carbohydrate

digestion

नननननलिखितमसहॳकह९नसाकाबोहाइडरहॳटकहॳ पाचनकापरमिअतउतपादकहह८ताहहॴ 23-Jan-2017

Options

1) Fats

वसा 2) Lipids

लिपपडस

3) Glucose

गिकह८ज

4) Cellulose

F A C E B O O K

P A G E h t t p w w w f a c e b o o k c o m s s c m e n t o r s o f f i c i a l P a g e | 31

FOR MORE UPDATES AND MORE MATERIAL DO LIKE OUR FACEBOOK PAGE httpwwwfacebookcomsscmentorsofficial

सहॳमयिह८ज

Correct Answer Glucose

Intestinal absorption of end products

from digestion of carbohydrates and

proteins in the pig hellip During absorption some sugars (fructose or

galactose) released from the

corresponding sucrose and lactose

respectively during digestion were

partly metabolized into glucose by the

enterocyte

Q130 Which of the following glands is a

source of the enzyme Ptyalin

नननननलिखितगरचियोमसहॳएजाइमटयालिनकासरह८तहहॴ 23-Jan-2017

Options

1) Pancreas

अगरािय

2) Thyroid Gland

िाइराइिगरिी 3) Pituitary Gland

पीयषगरिी 4) Salivary Glands

िारगरचियाा Correct Answer Salivary Glands

Q131 Which of the following is not true

about Pteridophyta

ननननमसहॳकह९नसीबातटहॳररिह८फाईटकहॳ बारहॳमसचनहीहहॴ 23-Jan-2017

Options

1) Dominant phase is saprophytes

परमिचरणसहॳपरह८फाईइटसहह८ताहहॴ 2) Main plant body is diploid

पह९दह८कामखयिरीरदपवगखणतहह८ताहहॴ 3) Seeds are present

बीजमह९जदहह८तहॳहहॴ 4)Flowers are absent

फिअनपनसतिहह८तहॳहहॴ

Correct Answer Seeds are present

Q132 The largest dolphin species is the

orca also called as

िॉिकफनकीसबसहॳबड़ीपरजानतकाकानामआकायहहॴनजसहॳ mdash- भीकहतहॳहहॴ 23-Jan-2017

Options

1) Bottle Nose

बाटिनह८ज

2) Baiji

बहॳजी 3) Killer whale

ककिरहहॳि

4)Tucuxi

टकवसी Correct Answer Killer whale

Q133 The fat digesting enzyme Lipase

is secreted by which of the following

वसाकापाचनकरनहॳवािाएजाइमिाइपहॳजनननननलिखितमसहॳककसकहॳ दवारासतरापवतहह८ताहहॴ

24-Jan-2017

Options

1) Kidneys

गद

2) Pancreas

अगनयािय

3) Large Intestine

बड़ीआत

4)Liver

नजगर

Correct Answer Pancreas

Lipase is an enzyme that splits fats so

the intestines can absorb them Lipase

hydrolyzes fats like triglycerides into

their component fatty acid and glycerol

molecules It is found in the blood

gastric juices pancreatic secretions

intestinal juices and adipose tissues

F A C E B O O K

P A G E h t t p w w w f a c e b o o k c o m s s c m e n t o r s o f f i c i a l P a g e | 32

FOR MORE UPDATES AND MORE MATERIAL DO LIKE OUR FACEBOOK PAGE httpwwwfacebookcomsscmentorsofficial

Q134 The arrangement of leaves on an

axis or stem is called

एकअकषयातनहॳपरपनततयोकीयवसिाकह८कयाकहाजाताहहॴ SSC CHSL Science (biology) 2016

Question Paper

24-Jan-2017

Options

1) Phyllotaxy

फाइिह८टहॴकसी 2) Vernation

वनिन

3) Venation

वहॳनहॳिन

4)Phytotaxy

फाइटह८टहॴकसी Correct Answer Phyllotaxy

In botany phyllotaxis or phyllotaxy is

the arrangement of leaves on a plant

stem (from Ancient Greek phyacutellon

ldquoleafrdquo and taacutexis ldquoarrangementrdquo)

Phyllotactic spirals form a distinctive

class of patterns in nature

Q135 The study of Cells is also known

as

कह८लिकाओकहॳ अधययनकह८ mdashmdashndash

भीकहाजाताहहॴ 24-Jan-2017

Options

1) Cytology

सायटह८िह८जी 2) Physiology

कफनजयह८िह८जी 3) Nucleology

नयककमयह८िह८जी 4)Cellology

सहॳिह८िह८जी Correct Answer Cytology

Q136 Which of the following scientists

is also known as the Father of Biology

नननननलिखितमसहॳककसवहॴजञाननककह८ ldquoजीवपवजञानकहॳ जनकrdquoकहॳ नामसहॳभीजानाजाताहहॴ 24-Jan-2017

Options

1) Herbert Spencer

हबयटयसपसर

2) Aristotle

अरसत 3) Lamarck

िहॳमाकय 4)Darwin

िापवयन

Correct Answer Aristotle

Q137 Which cells give rise to various

organs of the plant and keep the plant

growing

कह९नसीकह८लिकाएपह९धह८कहॳ लभननअगह८कह८जनमदहॳतीहहॴऔरपह९धह८कह८बढ़नहॳममददकरतीहहॴ

24-Jan-2017

Options

1) Permanent

सिायी 2) Dermal

तवचीय

3) Meristematic

मररसटहॳमटटक

4)Mature

परह८ढ़

Correct Answer Meristematic

A meristem is the tissue in most plants

containing undifferentiated cells

(meristematic cells) found in zones of

the plant where growth can take place

Q138 Rodentia Muridae is the scientific

name of

F A C E B O O K

P A G E h t t p w w w f a c e b o o k c o m s s c m e n t o r s o f f i c i a l P a g e | 33

FOR MORE UPDATES AND MORE MATERIAL DO LIKE OUR FACEBOOK PAGE httpwwwfacebookcomsscmentorsofficial

रह८िहॳलियानयररिी mdashmdash- कावहॴजञाननकनामहहॴ 24-

Jan-2017

Options

1) Mouse

चहा 2) Squirrel

चगिहरी 3) Monkey

बदर

4) Lizard

नछपकिी Correct Answer Mouse

Q139 Name the scientist who proposed

the cell theory

कह८लिकालसदातकापरसतावदहॳनहॳवािहॳवहॴजञाननककानामबताइए 24-Jan-2017

Options

1) Schleiden and Schwann

िीमिनऔरशरववान

2) Lamarck

िहॳमाकय 3) Treviranus

टरहॳवायरहॳनस

4)Whittaker and Stanley

हीटकरऔरसटहॳनिहॳ Correct Answer Schleiden and

Schwann

Q140 The flower with the worldrsquos

largest bloom is

दननयाकासबसहॳबड़ाफिखििनहॳवािा mdashmdashndash हहॴ 24-Jan-2017

Options

1) Pando

पािह८ 2) Posidonia

पह८सीिह८ननया 3) Rafflesia arnoldii

ररफिहॳलियाअनोमिी 4)Helianthus annuus

हहॳलिएनिसएनयअस

Correct Answer Rafflesia arnoldii

Rafflesia arnoldii is a species of

flowering plant in the parasitic genus

Rafflesia It is noted for producing the

largest individual flower on earth It has

a very strong and horrible odour of

decaying flesh earning it the nickname

ldquocorpse flower

Q141 Deficiency of which vitamin

causes night blindness

ककसपवटालमनकीकमीकहॳ कारणरतौधीहह८ताहहॴ 24-Jan-2017

Options

1) Vitamin K

पवटालमन K

2) Vitamin C

पवटालमन C

3) Vitamin B1

पवटालमन B1

4)Vitamin A

पवटालमन A

Correct Answer Vitamin A

Q142 Nongreen plants lack which of the

following

गहॴर-

हररतवनसपनतमनननननलिखितमसहॳककसकीकमीहह८तीहहॴ

24-Jan-2017

Options

1) Chlorophyll

किह८रह८कफि

2) Lycophyll

िायकह८कफि

3) Cyanophyll

F A C E B O O K

P A G E h t t p w w w f a c e b o o k c o m s s c m e n t o r s o f f i c i a l P a g e | 34

FOR MORE UPDATES AND MORE MATERIAL DO LIKE OUR FACEBOOK PAGE httpwwwfacebookcomsscmentorsofficial

सायनह८कफि

4)Phototropism

फह८टह८टरोपपजम

Correct Answer Chlorophyll

Q143 Organisms that use light to

prepare food are known as

जह८जीवपरकािकाउपयह८गकरभह८जनतहॴयारकरतहॳहहॴ उनह mdashmdash- कहॳ पमजानजाताहहॴ 24-Jan-2017

Options

1) Autotrophs

सवपह८षी 2) Heterotrophs

पवषमपह८षज

3) Omnivores

सवायहारी 4)Decomposers

पवघटनकरनहॳवािा Correct Answer Autotrophs

autotrophs often make their own food

by using sunlight carbon dioxide and

water to form sugars which they can use

for energy Some examples of

autotrophs include plants algae and

even some bacteria Autotrophs

(producer) are important because they

are a food source for heterotrophs

(consumers)

A heterotroph is an organism that

ingests or absorbs organic carbon

(rather than fix carbon from inorganic

sources such as carbon dioxide) in order

to be able to produce energy and

synthesize compounds to maintain its

life Ninety-five percent or more of all

types of living organisms are

heterotrophic including all animals and

fungi and some bacteria

Q144 Which of the following is a

primary function of haemoglobin

नननननलिखितमसहॳकह९नसाटहमह८गिह८बबनकाएकपरािलमककाययहहॴ

25-Jan-2017

Options

1) Utilization of energy

उजायकाउपयह८गकरना 2) Prevention of anaemia

रकतामपताहह८नहॳसहॳरह८कना 3) Destruction of bacteria

बहॴकटीररयाकापवनािकरना 4) To transport oxygen

ऑकसीजनकावहनकरना Correct Answer To transport oxygen

Q145 Vascular bundles are absent in

सवहनीबिि mdashmdash- मअनपनसतिरहतहॳहहॴ 25-Jan-2017

Options

1) Bryophyta

िायह८फाइटा 2) Pteridophyta

टहॳररिह८फाईटा 3) Gymnosperms

नजननह८सपमय 4) Angiosperms

एननजयह८सपहॳनसय Correct Answer Bryophyta

Q146 Sauria Lacertidae is the scientific

name of

सहॴररयािहॳसरटाईिी mdashmdashndash कावहॴजञाननकनामहहॴ 25-Jan-2017

Options

1) Crocodile

मगरमचछ

2) Hippopotamus

टहपपह८पह८टहॳमस

3) Lizard

नछपकिी 4) House fly

F A C E B O O K

P A G E h t t p w w w f a c e b o o k c o m s s c m e n t o r s o f f i c i a l P a g e | 35

FOR MORE UPDATES AND MORE MATERIAL DO LIKE OUR FACEBOOK PAGE httpwwwfacebookcomsscmentorsofficial

घरहॳिमकिी Correct Answer Lizard

Q147 Which type of pathogen causes

the water-borne disease SARS (Severe

Acute Respiratory Syndrome)

ककसपरकािकारह८गज़नकजिजननतबीमारीसासयकाकारणबनताहहॴ 25-Jan-2017

Options

1) Viral

वायरि

2) Parasitic

परजीवी 3) Protozoan

परह८टह८जअन

4) Bacterial

बहॴकटीररयि

Correct Answer Viral

Q148 Which of the following organs

produces the enzyme lipase

नननननलिखितमसहॳकह९नसाअगिायपहॳजएजाइमउतपननकरताहहॴ 25-Jan-2017

Options

1) Pancreas

अगनयािय

2) Large Intestine

बड़ीआत

3) Liver

नजगर

4) Small Intestine

छह८टीआत

Correct Answer Pancreas

Q149 A is a long internode forming the

basal part or the whole of a peduncle

एक mdashmdash- एकिबाइटरनह८िहहॴ जह८ननचिाटहससायासनपणयिठिबनताहहॴ 25-

Jan-2017

Options

1) Rhizome

परकद

2) Rachis

महॳ दि

3) floral axis

पषपअकष

4) Scape

भगदड़

Correct Answer scape

Q150 ndash Which of the following

organisms are considered to be both

Living and Non-living

नननननलिखितमसहॳकह९नसहॳजीवाणकह८जीपवतऔरअजीपवतमानाजाताहहॴ

25-Jan-2017

Options

1) Bacteria

बहॴकटीररया 2) Fungi

कवक

3) Algae

िहॴवाि

4)Virus

वायरस

Correct Answer Virus

They are considered to be living as they

possess a protein coat as a protective

covering DNA as the genetic material

etc

They are said to be non-living as they

can be crystallised and they survive for

billions of years They can tolerate high

temperatures freezing cold

temperatures ultra-violet radiations etc

Q151 Deficiency of fluorine causes

which of the following

फिह८ररनकीकमीकहॳ कारणनननननलिखितमसहॳकयाहह८ताहहॴ

F A C E B O O K

P A G E h t t p w w w f a c e b o o k c o m s s c m e n t o r s o f f i c i a l P a g e | 36

FOR MORE UPDATES AND MORE MATERIAL DO LIKE OUR FACEBOOK PAGE httpwwwfacebookcomsscmentorsofficial

27-Jan-2017

Options

1) Dental Caries

िटिकहॴ ररज

2) Scurvy

सकवरी 3) Anaemia

रकतामपता 4) Arthritis

गटठया Correct Answer Dental Caries

Q152 In a Punnett Square with the

cross AaBb x AaBb how many Aabb

genotypes would be created

पनहॳटसककायरमिह८स AaBb x AaBb कहॳ साि

ककतनहॳ Aabb जीनह८टाइपबनगहॳ 27-Jan-2017

Options

1) 1

2) 8

3) 2

4) 3

Correct Answer 2

Q153 Which of the following is the

Controlling Center of the Cell

नननननलिखित म सहॳ कह८लिकाका ननयतरण

क दर कह९न हहॴ

27-Jan-2017

Options

1) Nucleus

क दर

2) Plasma

पिाजमा 3) Lysosome

िायसह८सह८म

4) Chromosome

िह८मह८सह८म

Correct Answer Nucleus

The control centre of the cell is the

nucleus in eukaryotic cells The nucleus

contains genetic material in the form of

DNA

Q154 Myopia affects which of the

following organs

मायह८पपयानननननलिखितअगह८मसहॳककसहॳपरभापवतकरताहहॴ

25-Jan-2017

Options

1) Heart

हदय

2) Skin

तवचा 3) Eyes

आािहॳ 4)Mouth

मह

Correct Answer Eyes

Q155 Which of the following bears

flowers

नननननलिखितमसहॳकह९नफिधारणकरताहहॴ

25-Jan-2017

Options

1) Bryophyta

िायह८फाइटा 2) Pteridophyta

टहॳरीिह८फाईटा 3) Gymnosperms

नजननह८सपमय 4)Angiosperms

एननजयह८सपमय Correct Answer Angiosperms

Q156 Oxygenated blood flows out of the

heart through the

ऑकसीजनयकतरकत mdashmdashmdash

कहॳ माधयमसहॳहदयकहॳ बाहरबहताहहॴ 25-Jan-2017

F A C E B O O K

P A G E h t t p w w w f a c e b o o k c o m s s c m e n t o r s o f f i c i a l P a g e | 37

FOR MORE UPDATES AND MORE MATERIAL DO LIKE OUR FACEBOOK PAGE httpwwwfacebookcomsscmentorsofficial

Options

1) Aorta

महाधमनी 2) pulmonary artery

फहॳ फड़हॳकीधमनी 3) vena cava

वहॳनाकावा 4)Atrium

चह९क

Correct Answer aorta

Q157 Blood leaving the liver and

moving towards the

heart has a higher concentration of

नजगरसहॳननकिकरहदयकीतरफजानहॳवािहॳरकतम mdashmdashmdashmdash कीउचचसादरताहह८तीहहॴ 27-Jan-2017

Options

1) Lipids

लिपपडस

2) Urea

यररया 3) Bile Pigments

पपततकहॳ रगकरण

4) Carbon dioxide

काबयनिायऑकसाइि

Correct Answer Bile Pigments

Urea is nitrogen containing substance

which is produced in the liver in order

to deal with excess amino-acids in the

body As urea is produced it leaves the

liver in the blood stream and passes via

the circulatory system to all parts of the

body

Q158 Bulb is a modification of which

part of a plant

बमबएकपह९धहॳकहॳ ककसटहससहॳकाएक पातरणहह८ताहहॴ 27-Jan-2017

Options

1) The root

जड़

2) The stem

तना 3) The radicle

मिाकर

4)The fruit

फि

Correct Answer The stem

Q159 Which of the following carries

blood away from the heart to different

body parts

इनमहॳसहॳकह९नरकतकह८हदयसहॳिरीरकहॳ पवलभननअगह८तकिहॳजातीहहॴ

27-Jan-2017

Options

1) Arteries

धमननया 2) Nerves

तबतरहाए

3) Capillaries

कहॳ लिकाए

4)Veins

नसहॳ Correct Answer Arteries

Q160 The series of processes by which

nitrogen and its compounds are

interconverted in the environment and

in living organisms is called

27-Jan-2017

Options

1)Absorption of Nitrogen

2)Ammonification

3)Nitrogen Fixation

4)Nitrogen Cycle

Correct Answer Nitrogen Cycle

Ammonification or Mineralization is

performed by bacteria to convert

organic nitrogen to ammonia

F A C E B O O K

P A G E h t t p w w w f a c e b o o k c o m s s c m e n t o r s o f f i c i a l P a g e | 38

FOR MORE UPDATES AND MORE MATERIAL DO LIKE OUR FACEBOOK PAGE httpwwwfacebookcomsscmentorsofficial

Nitrification can then occur to convert

the ammonium to nitrite and nitrate

Nitrogen fixation is a process by which

nitrogen in the Earthrsquos atmosphere is

converted into ammonia (NH3) or other

molecules available to living organisms

Q161 BCG vaccine is given to protect

from which of the following

बीसीजीकाटटकानननननलिखितमसहॳककसकहॳ बचावकहॳ लिएटदयाजातहहॴ

27-Jan-2017

Options

1) Jaundice

पीलिया 2) Anaemia

रकतमपता 3) Tuberculosis

कषयरह८ग

4) Polio

पह८लियह८ Correct Answer Tuberculosis

Q162 Parallel venation is found in

समानतरवहॳनहॳिन mdashmdashmdash- मपायाजाताहहॴ 27-Jan-2017

Options

1) plants which are monocots

पह९धहॳजह८एकबीजपतरीहह८तहॳहहॴ 2) plants which have a dicot stem

वहॳपह९धहॳनजनकातनादपवदलियहह८ताहहॴ 3) plants with leaves similar to Tulsi

वहॳपह९धहॳनजनकीपनततयतिसीकीपनततयोकहॳ समानहह८तहॳहहॴ 4)plants with tap roots

टहॳप टवािहॳपह९धहॳ Correct Answer plants which are

monocots

Q163 The hardest part of the body is

िरीरकासबसहॳकठह८रभाग mdashndash हहॴ 27-Jan-2017

Options

1) Bones

हडडिय

2) Tooth Enamel

दातकहॳ इनहॳमि

3) Skull

िह८पड़ी 4) Spinal Cord

महॳ रजज

Correct Answer Tooth Enamel

Q164 Which type of pathogen causes

the waterborne disease E coli Infection

ककसपरकारकारह८गजननकजिजननतरह८गईकह८िाईसिमणकाकारणबनताहहॴ 27-Jan-2017

Options

1) Protozoan

परह८टह८जआ

2) Parasitic

परजीवी 3) Bacterial

बहॴकटीररयि

4)Viral

वायरि

Correct Answer Bacterial

Q165 The amount of blood filtered

together by both the kidneys in a 70 kg

adult male human in a minute is

70 की गरा वािहॳएकवयसकप षमएकलमनटमदह८नोगदकहॳदवाराएकसािचाबनीगयीरकतकीमातरहह८तीहहॴ 29-Jan-2017

Options

1) 1100 ml

1100 लमलि

2) 100 ml

F A C E B O O K

P A G E h t t p w w w f a c e b o o k c o m s s c m e n t o r s o f f i c i a l P a g e | 39

FOR MORE UPDATES AND MORE MATERIAL DO LIKE OUR FACEBOOK PAGE httpwwwfacebookcomsscmentorsofficial

100 लमलि

3) 1500 ml

1500 लमलि

4) 500 ml

500 लमलि

Correct Answer 1100 ml

Q166 Which feature of a plant helps to

distinguish a monocot from a dicot

पह९धहॳकीवहकह९नसीपविहॳषताहहॴजह८एकदपवदलियहॳऔरएकएकदिीयपह९धहॳसहॳभहॳदकरनहॳममददकरतीहहॴ 29-Jan-2017

Options

1) Pollination

परागम

2) Venation

वहॳनहॳिन

3) Vernation

वनिन

4) Aestivation

एसटीवहॳिहॳन

Correct Answer venation

Q167 The Mutation Theory was

proposed by

उतवररवतयनकालसदात mdashmdashndash

कहॳ दवरापरसतापवतककयाजाताहहॴ 29-Jan-2017

Options

1) Charles Lyell

चामसयलियहॳि

2) William Smith

पवलियमनसमि

3) Hugo De Vries

हयगह८िीराईस

4)Harrison Schmitt

हहॳरीसननसमट

Correct Answer Hugo De Vries

Q168 Which type of pathogen causes

the waterborne disease HepatitisA

ककसपरकारकहॳ रह८गजनकजिजननतरह८गहहॳपहॳटाइटटस-A काकारणबनताहहॴ

29-Jan-2017

Options

1) Parasitic

परजीवी 2) Viral

वायरि

3) Protozoan

परह८टह८जआ

4) Bacterial

बहॴकटीररयि

Correct Answer Viral

Q169 In a Punnett Square with the

cross AaBb x Aabb how many AaBb

genotypes would be created

पनहॳटसकवायरमिह८स AaBb x Aabb

कहॳ सािककतनहॳ AaBb जीनह८टाइपबनगहॳ 29-Jan-

2017

Options

1) 4

2) 1

3) 7

4) 6

Correct Answer 4

Q170 Arboreal Ateles is the scientific

name of

अिह८ररयिएटटलिस mdashmdashmdash कावहॴजञाननकनामहहॴ 29-Jan-2017

Options

1) Squirrel

चगिहरी 2) Sparrow

गह८रहॴया 3) Lizard

नछपकिी 4) Spider monkey

F A C E B O O K

P A G E h t t p w w w f a c e b o o k c o m s s c m e n t o r s o f f i c i a l P a g e | 40

FOR MORE UPDATES AND MORE MATERIAL DO LIKE OUR FACEBOOK PAGE httpwwwfacebookcomsscmentorsofficial

मकड़ीबदर

Correct Answer Spider monkey

Q171 Which type of pathogen causes

the waterborne disease Salmonellosis

ककसपरकारकारह८गाणजिजननतबीमारीसािमह८नहॳिह८लसज़काकारकहहॴ

29-Jan-2017

Options

1) Algal

िहॳवालियहॳ 2) Parasitic

परजीवी 3) Bacterial

बहॴकटीररयि

4)Viral

वायरि

Correct Answer Bacterial

An infection with salmonella bacteria

commonly caused by contaminated food

or water

Symptoms include diarrhoea fever

chills and abdominal pain

Q172 is a condition in which there is a

deficiency of red cells or of haemoglobin

in the blood

mdashmdash-

एकनसिनतहहॴनजसमहॳरकतमिािकह८लिकाओकीयाहीमह८गिह८बबनकीकमीहह८तीहहॴ 29-Jan-2017

Options

1) Albinism

एनमबननजम

2) Propyria

परह८पीररया 3) Anaemia

एनीलमया 4)Keloid disorder

कहॳ िह८इिडिसओिर

Correct Answer Anaemia

Q173 Ananas comosus is the scientific

name of

Options

अनानासकह८मह८सस mdashmdashmdashndash

कावहॴजञाननकनामहहॴ 29-Jan-2017

1) Custard Apple

सीताफि

2) Pineapple

पाइनएपपि

3) Bamboo

बास

4)Pomegranate

अनार

Correct Answer Pineapple

Q174 Which organ produces insulin

कह९नसाअगइनसलिनपहॴदाकरताहहॴ 29-Jan-

2017

Options

1) Liver

यकत

2) Thyroid gland

िायराइिगरिी 3) Spleen

पिीहा 4)Pancreas

अगरयिय

Correct Answer Pancreas

Q175 Which of the following disease is

not caused by water pollution

नननननलिखितमसहॳकह९नसारह८गपानीकहॳ परदषणकहॳकारणनहीहह८ता

29-Jan-2017

Options

1) Cholera

हहॴजा 2) Typhoid

F A C E B O O K

P A G E h t t p w w w f a c e b o o k c o m s s c m e n t o r s o f f i c i a l P a g e | 41

FOR MORE UPDATES AND MORE MATERIAL DO LIKE OUR FACEBOOK PAGE httpwwwfacebookcomsscmentorsofficial

टाइफाइि

3) Asthma

दमा 4)Diarrhoea

दसत

Correct Answer Asthma

Q176 Ocimum tenuiflorum is the

scientific name of

ओलिलममटहॳयईफिह८रमइसकावहॴजञाननकनाम mdash

ndash हहॴ 30-Jan-2017

Options

1) Neem

नीम

2) Mango

आम

3) Babul

बबि

4)Tulsi

तिसी Correct Answer Tulsi

Q177 Which gland secretes bile a

digestive fluid

कह९नसीगरिीपपतत एकपाचनतरिपरदािय सरापवतकरतीहहॴ 30-Jan-2017

Options

1) Pancreas

अगनयािय

2) Liver

यकत

3) Thyroid

िायराइि

4) Testes

टहॳनसटस

Correct Answer liver

Q178 In which of the following the

dominant phase is Gametophyte

नननननलिखितमसहॳककसकहॳ परमिचरणयगमकह८दपवधद (Gametophyte)हहॴ 30-Jan-2017

Options

1) Bryophyta

िायह८फाइटा 2) Pteridophyta

टहॳररिह८फाइटा 3) Gymnosperms

नजननह८सपमय 4) Angiosperms

एननजयह८सपमय Correct Answer Bryophyta

Q179 Anaerobic respiration refers to

which of the following

नननननलिखितमसहॳककसहॳअवायवीयशवसनकहाजाताहहॴ

30-Jan-2017

Options

1) Respiration without Oxygen

ऑकसीजनकहॳ बबनाशवसन

2) Respiration with Oxygen

ऑकसीजनकहॳ सािशवसन

3) Respiration without CO2

काबयनिायऑकसाइिकहॳ बबनाशवसन

4) Respiration with CO2

काबयनिायऑकसाइिकहॳ सािशविन

Correct Answer Respiration without

Oxygen

Q180 Which type of pathogen causes

the waterborne disease Cholera

ककसपरकारकारह८गजनकजिजननतरह८गहहॴजाकाकारणबनताहहॴ

30-Jan-2017

Options

1) Algal

िहॴवालियहॳ

F A C E B O O K

P A G E h t t p w w w f a c e b o o k c o m s s c m e n t o r s o f f i c i a l P a g e | 42

FOR MORE UPDATES AND MORE MATERIAL DO LIKE OUR FACEBOOK PAGE httpwwwfacebookcomsscmentorsofficial

2) Bacterial

बहॴकटीररयि

3) Protozoan

परह८टह८जआ

4) Viral

वायरि

Correct Answer Bacterial

Q181 To which class does

Oxyreductases transferases hydrolases

belong

ओकसीररिकटहॳसटरासफरहॳजहॳस

हाइडरह८िहॳसहॳसककसवगयमआतहॳहहॴ 30-Jan-2017

Options

1) Hormones

हारमोस

2) Enzymes

एजाइनस

3) Proteins

परह८टीनस

4) Vitamins

पवटालमनस

Correct Answer Enzymes

Q182 Which of the following is not true

about Gymnosperms

ननननमसहॳकह९नसीबातअनावतबीजीकहॳ बारहॳमसचनहीहहॴ 30-Jan-2017

Options

1) Dominant phase is saprophytes

परमिचरणसहॳपरह८फाइटसहह८ताहहॴ 2) Vascular bundles are absent

सवहनीबििअनपनसितहह८ताहहॴ 3) spores are heterospores

बीजाणहहॳटहॳरह८सपह८रसहह८तहॳहहॴ 4) Flowers are absent

फिअनपनसितहह८तहॳहहॴ

Correct Answer Vascular bundles are

absent

Q183 The name of first mammal clone sheep is

भहॳड़कीपरिमसतनपायीपरनत प (किह८न)

कानामहहॴ 30-Jan-2017

Options

1) Noori

नरी 2) Dolly

िॉिी 3) Louise

िसी 4)Durga

दगाय Correct Answer Dolly

Q184 Which type of pathogen causes

the water-borne disease Typhoid fever

ककसपरकारकारह८गजनकजिजननतरह८गटाइफाइिबिारकाकारणबनताहहॴ 30-Jan-2017

Options

1) Algal

िहॴवािीय

2) Parasitic

परजीवी 3) Protozoan

परह८टह८जनअन

4)Bacterial

बहॴकटीररयि

Correct Answer Bacterial

Q185 In which part of the cell are

proteins made

कह८लिकाकहॳ ककसटहससहॳमपरह८टीनबनायाजाताहहॴ

31-Jan-2017

Options

1) Reticulum

रहॳटटकिम

F A C E B O O K

P A G E h t t p w w w f a c e b o o k c o m s s c m e n t o r s o f f i c i a l P a g e | 43

FOR MORE UPDATES AND MORE MATERIAL DO LIKE OUR FACEBOOK PAGE httpwwwfacebookcomsscmentorsofficial

2) Golgi apparatus

गह८मजीएपहॳरहॳटस

3) Ribosomes

ररबह८सह८नस

4) Lysosome

िायसह८सह८नस

Correct Answer ribosomes

Proteins are produced by stringing

amino acids together in the order

specified by messenger RNA strands

that were transcribed from DNA in the

cell nucleus The process of synthesizing

a protein is called translation and it

occurs on ribosomes in the cytoplasm of

a cell

Q186 Polio is a disease caused by which

of the following

नननननलिखितमसहॳपह८लियह८कीबबमारह८हह८नहॳकाकारणकयाहहॴ

31-Jan-2017

Options

1) Bacteria

बहॴकटीररयि

2) Mosquito

मचछर

3) Virus

वायरस

4) Cockroach

नतिच हॳ Correct Answer Virus

Polio or poliomyelitis is a crippling and

potentially deadly infectious disease It

is caused by the poliovirus

Q187 ndash Hay fever is a sign of which of

the following

हहॳकफवरनननननलिखितमसहॳककसकाएकसकहॳ तहहॴ

31-Jan-2017

Options

1) Old Age

वदावसिा 2) Malnutrition

कपह८सण

3) Allergy

एिनजय 4) Over Work

अतयचधककाययकरना Correct Answer Allergy

Q188 How many chromosomes does a

human cell contain

एकमानवकह८लिकामककतनहॳगणसतरहह८तहॳहहॴ

29-Jan-2017

Options

1) 6

2) 26

3) 46

4) 66

Correct Answer 46

In humans each cell normally contains

23 pairs of chromosomes for a total of

46 Twenty-two of these pairs called

autosomes look the same in both males

and females The 23rd pair the sex

chromosomes differ between males and

females

Q189 Which of the following is not true

about Bryophyta

ननननमसहॳकह९नसीबातिायह८फाइटकहॳ बारहॳमसचनहीहहॴ 31-Jan-2017

Options

1) Dominant phase is gametophytes

परमिचरणगहॳलमतह८फाइटसहह८ताहहॴ 2) Main plant body is haploid

पह९धहॳकामखयिरीरअगखणतहह८ताहहॴ 3) Spores are homospores

बीजाणहह८मह८सफह८रसहह८तहॳहहॴ 4) Flowers are present

फिमह८जदहह८तहॳहहॴ Correct Answer Flowers are present

F A C E B O O K

P A G E h t t p w w w f a c e b o o k c o m s s c m e n t o r s o f f i c i a l P a g e | 44

FOR MORE UPDATES AND MORE MATERIAL DO LIKE OUR FACEBOOK PAGE httpwwwfacebookcomsscmentorsofficial

Q190 Which aquatic animal has

trailing tentacles

ककसजिीयजानवरकहॳ पीछहॳचिनहॳवािहॳटहॳटकिसहह८तहॳहहॴ

31-Jan-2017

Options

1) Sea horse

समदरीघह८िा 2) Corals

मगा 3) Jelly fish

जहॳिीमछिी 4) Star fish

तारामछिी Correct Answer Jelly fish

Jellyfish with its umbrella-shaped bell

and trailing tentacles

Q191 Which type of pathogen causes

the water-borne disease Poliomyelitis

(Polio)

ककसपरकारकारह८गजनकजिजननतरह८गपह८लियह८मायहॳटटस (पह८लियह८) काकारणहहॴ 31-Jan-

2017

Options

1) Parasitic

परजीवी 2) Algal

िहॴवालिय

3) Viral

वायरि

4) Bacterial

बहॴकटीररयि

Correct Answer Viral

Q192 The outer white part of the eye

that protects the inner structures is

आािकाबाहरीसफहॳ दटहससाजह८आतररकसरचनाओकीरकषाकरताहहॴ वह mdashmdashmdash हहॴ 31-Jan-

2017

Options

1) Iris

आयररस

2) Sclera

सकिहॳरा 3) Retina

रहॳटटना 4) Cornea

कह८ननयया Correct Answer Sclera

Q193 Proteins are made up of

परह८टीनकाननमायण mdashndash सहॳहह८ताहहॴ 31-Jan-2017

Options

1) Amino acids

एलमनह८अनि

2) Fatty acids

वसायकतअनि

3) Glucose

गिकह८ज

4)Nucleotides

नयनकियह८टाईिस

Correct Answer Amino acids

Q194 Moringa Oleifera is the scientific

name of

मह८ररगओलिफहॳ रा mdashmdashndash कावहॴजञाननकनामहहॴ 31-Jan-2017

Options

1) Banyan

बरगद

2) Gulmohar

गिमह८हर

3) Amla

आमिा

F A C E B O O K

P A G E h t t p w w w f a c e b o o k c o m s s c m e n t o r s o f f i c i a l P a g e | 45

FOR MORE UPDATES AND MORE MATERIAL DO LIKE OUR FACEBOOK PAGE httpwwwfacebookcomsscmentorsofficial

4) Drumstick

डरमनसटक

Correct Answer Drumstick

Q195 Kidney stones are composed of

गदकीपिरी mdashndash सहॳबनीहह८तीहहॴ 1-Feb-2017

Options

1) Calcium Oxalate

कहॴ नमसयमओकजहॳिहॳट

2) Sodium Chloride

सह८डियमकिह८राइि

3) Magnesium Nitrate

महॳनगनलियमनाइतटरहॳट

4) Calcium Bicarbonate

कहॴ नमियमबायकबोनहॳट

Correct Answer Calcium Oxalate

Q196 ndash Which of the following is not

true about Angiosperms

ननननमसहॳकह९नसीबातआवतबीजीकहॳ बारहॳमसचनहीहहॴ 1-Feb-2017

Options

1) Dominant phase is gametophytes

परमिचरणगहॳलमतह८फाइटहह८ताहहॴ 2) Vascular bundles are present

सवहनीबििमह९जदहह८ताहहॴ 3) Spores are heterospores

बीजाणहहॳटहॳरह८सपह८रसहह८तहॳहहॴ 4) Seeds are covered

बीजढकहॳ हह८तहॳहहॴ Correct Answer Dominant phase is

gametophytes

Q197 All of the following are excretory

(waste) products of animals except

नननननलिखितमसहॳककसएककह८छह८ड़करअनयसभीपराखणयोदवाराउतसनजयतपदाियहहॴ 1-Feb-

2017

Options

1) Uric Acid

यररकएलसि

2) Ammonia

अमह८ननया 3) Carbohydrates

काबोहाइडरहॳट

4) Urea

यररया Correct Answer Carbohydrates

In animals the main excretory products

are carbon dioxide ammonia (in

ammoniotelics) urea (in ureotelics) uric

acid (in uricotelics) guanine (in

Arachnida) and creatine

Q198 RNA is a polymeric molecule

What does RNA stand for

आरएनइएएकबहिकआणहहॴ इसकाकापवय पकयाहहॴ 1-Feb-2017

Options

1) Rado Nuclear Acid

रािह८नयनकियरएलसि

2) Ribo Nucleic Acid

राइबह८नयनकिकएलसि

3) Rhino Nuclear Acid

हाइनह८नयनकियरएलसि

4) Resto Nucleus Acid

रहॳसटह८नयकिीयसएलसि

Correct Answer Ribo Nucleic Acid

Q199 Which organ does detoxification

and produces chemicals needed for

digestion

कह९नसाअगपवषहरणकरताहहॴऔरपाचनकहॳ लिएआवशयकरसायनोकह८पहॴदाकरताहहॴ 1-Feb-

2017

Options

1) Salivary glands

िारगरचिया 2) Pancreas

अगनयािय

F A C E B O O K

P A G E h t t p w w w f a c e b o o k c o m s s c m e n t o r s o f f i c i a l P a g e | 46

FOR MORE UPDATES AND MORE MATERIAL DO LIKE OUR FACEBOOK PAGE httpwwwfacebookcomsscmentorsofficial

3) Thyroid gland

िायराइिगरिी 4) Liver

यकत

Correct Answer Liver

Q200 Psidium guajava is the scientific

name of

लसडियमगआजावा mdashmdash कावहॴजञाननकनामहहॴ 1-

Feb-2017

Options

1) Guava

अम द

2) Mango

आम

3) Bamboo

बास

4) Jack fruit

कटहि

Correct Answer Guava

Q201 Which drug is used as a Blood

Thinner

चधरकह८पतिाकरनहॳकहॳ पमककसदवाकापरयह८गककयाजाताहहॴ

1-Feb-2017

Options

1) Warfarin

वाफर न

2) Tramadol

टरहॳमािह८ि

3) Azithromycin

एनजरह८मायलसन

4) Hydralazine

हाइडरह८िहॳनजन

Correct Answer Warfarin

Q202 Which of the following disease is

caused due to the deficiency of protein

परह८टीनकीकमीकहॳ कारणनननननलिखितमसहॳकह९नसारह८गहह८ताहहॴ 1-Feb-2017

Options

1) Arthritis

गटठया 2) Kwashiorkor

कािीओकय र

3) Goitre

गाइटर

4) Night Blindness

रतह९चध

Correct Answer Kwashiorkor

Q203 A is species of plant that has

adapted to survive in an environment

with little liquid water

mdashmdashndashपह९धहॳकीएकऐसहॳऐसहॳपरजानतहहॴ नजसनहॳकमपानीवािहॳवातावरणमजीपवतरहनहॳकहॳलिएअनकिनहहॴ 1-Feb-2017

Options

1) Xerophyte

म दपवद

2) Hydrophyte

जिीयपादप

3) Mesophyte

समह८दपवद

4) Thallophyte

िहॴिह८फाइटा Correct Answer xerophyte

xerophyte is a species of plant that has

adapted to survive in an environment

with little liquid water such as a desert

or an ice- or snow-covered region in the

Alps or the Arctic

Mesophytes are terrestrial plants which

are adapted to neither a particularly

dry nor particularly wet environment

An example of a mesophytic habitat

would be a rural temperate meadow

F A C E B O O K

P A G E h t t p w w w f a c e b o o k c o m s s c m e n t o r s o f f i c i a l P a g e | 47

FOR MORE UPDATES AND MORE MATERIAL DO LIKE OUR FACEBOOK PAGE httpwwwfacebookcomsscmentorsofficial

which might contain goldenrod clover

oxeye daisy and Rosa multiflora

thallophyte any of a group of plants or

plantlike organisms (such as algae and

fungi) that lack differentiated stems

leaves and roots and that were formerly

classified as a primary division

(Thallophyta) of the plant kingdom

Q204 How many types of teeth are

there in humans

मनषयोमककतनहॳपरकारकहॳ दातहह८तहॳहहॴ

1-Feb-2017

Options

1) 4

2) 5

3) 2

4) 3

Correct Answer 4

teeth -Humans have four types of

teethincisors canines premolars and

molars each with a specific function

The incisors cut the food the canines

tear the food and the molars and

premolars crush the food

Q205 Carica papaya is the scientific name of

कहॴ ररकापपाया mdashmdashndash कावहॴजञाननकनामहहॴ 2-

Feb-2017

Options

1) Peepal

पीपि

2) Papaya

पपीता 3) Tamarind

इमिी 4) Drumstick

ढह८िकाछड़ी Correct Answer Papaya

Q206 Muscles get tired when there is

shortfall of

जब mdashndash कीकमीहह८तीहहॴतबपहॳिीयिकजातीहहॴ 2-Feb-2017

Options

1) Lactic acid

िहॴनकटकएलसि

2) Na+ ions

Na+ आयन

3) ATP

एटीपी 4) Sulphates

समफहॳ टस

Correct Answer ATP

ATP is the energy source muscle fibers

use to make muscles contract

muscle tissuersquos main source of energy

called adenosine triphosphate or ATP

As your muscles use up this energy

source they become tired and fatigued

Oxygen is the key ingredient that helps

create new ATP to replenish the burned

up ATP in your muscles

Q207 Artocarpus integra is the

scientific name of आटह८कापयसइटीगरा mdashmdashmdash कावहॴजञाननकनामहहॴ 2-Feb-2017

Options

1) Guava

अम द

2) Pineapple

अनानास

3) Silver Oak

लसमवरओक

4) Jack fruit

कटहि

Correct Answer Jack fruit

Q208 Which organ stores fat soluble

vitamins

कह९नसाअगवसामघिनिीिपवटालमनह८काभिाराकरताहहॴ

2-Feb-2017

F A C E B O O K

P A G E h t t p w w w f a c e b o o k c o m s s c m e n t o r s o f f i c i a l P a g e | 48

FOR MORE UPDATES AND MORE MATERIAL DO LIKE OUR FACEBOOK PAGE httpwwwfacebookcomsscmentorsofficial

Options

1) Blood

रकत

2) Skin

तवचा 3) Liver

यकत

4) Pancreas

अगनयािय

Correct Answer Liver

Q209 Which disease is caused due to

deficiency of Iodine

आयह८िीनकहॳ कारणकह९नसारह८गहह८ताहहॴ 2-Feb-2017

Options

1) Rickets

ररकहॳ टस

2) Scurvy

सकवी 3) Goitre

गणमािा 4) Growth retardation

पवकासका कना Correct Answer Goitre

rickets A softening and weakening of

bones in children usually due to

inadequate vitamin D

Q210 Grevillea Robusta is the scientific name of

गरहॳपवलियारह८बसटा mdashmdashmdash- कापवजञाननकनामहहॴ 2-Feb-2017

Options

1) Peepal

पीपि

2) Teak

सागह९न

3) Silver Oak

लसमवरओक

4) Jack fruit

कटहि

Correct Answer Silver Oak

Q211 When a Cuttlefish is described as a Molluscs it is at which level of

classification

जबएककटिकफिकह८एकमह८िसकाकहॳ पमवखणयतककयाजाताहहॴतबयहॳवगीकरणकहॳ ककससतरपहॳनसितहहॴ 2-Feb-2017

Options

1) Class

वगय 2) Order

िम

3) Family

पररवार

4) Phylum

सघ

Correct Answer Phylum

Q212 Bambusa dendrocalmus is the

scientific name of बानबसािहॳडराकामस mdashmdashmdash कावहॴजञाननकनामहहॴ 3-Feb-2017

Options

1) Banyan

बरगद

2) Papaya

पपीता 3) Bamboo

बास

4) Pomegranate

अनार

Correct Answer Bamboo

Q213 Acinonyx Jubatus is the scientific name of

एलसनह८ननकसजयबहॳटस mdashmdashmdash

कावहॴजञाननकनामहहॴ 3-Feb-2017

F A C E B O O K

P A G E h t t p w w w f a c e b o o k c o m s s c m e n t o r s o f f i c i a l P a g e | 49

FOR MORE UPDATES AND MORE MATERIAL DO LIKE OUR FACEBOOK PAGE httpwwwfacebookcomsscmentorsofficial

Options

1) Bear

भाि 2) Horse

घह८िा 3) Cheetah

चीता 4) Zebra

जहॳिा Correct Answer Cheetah

Q214 The pale yellow colour of urine is

due to the presence of which pigment

मतरकाफीकापीिारगरगदरयकहॳ उपनसिनतकहॳ कारणहह८ताहहॴ

3-Feb-2017

Options

1) Urochrome

यरह८िह८म

2) Urophyll

यरह८कफि

3) Chlorophyll

किह८रह८कफि

4) Chloroplast

किह८रह८पिासट

Correct Answer Urochrome

Q215 Which of the following constitute

to form a gene

नननननलिखितमसहॳकह९नसीचीज़एकजीनकागठनकरतीहहॴ

3-Feb-2017

Options

1) Polynucleotides

पह८िीनयनकियह८टाईडस

2) Hydrocarbons

हाइडरह८काबोस

3) Lipoproteins

िाईपह८परह८टीनस

4) Lipids

लिपपडस

Correct Answer Polynucleotides

Polynucleotide molecule is a biopolymer

composed of 13 or more nucleotide

monomers covalently bonded in a chain

DNA (deoxyribonucleic acid) and RNA

(ribonucleic acid) are examples of

polynucleotides with distinct biological

function

Q216 Vertebrates belongs to the

phylum

रीढ़कीहडिीवािहॳपराणी mdashmdashmdash

परजानतकहॳ अतगायतआतहॳहहॴ 3-Feb-2017

Options

1) Arthropoda

आरह८पह८ड़ा 2) Annelida

एननलििा 3) Cnidaria

ननिहॳररया 4) Chordata

कह८िटा Correct Answer Chordata

Q217 Punica granatum is the scientific name of

पननकगरहॳनहॳटस mdashmdashmdash कावहॴजञाननकनामहहॴ 3-Feb-2017

Options

1) Custard Apple

सीताफि

2) Gulmohar

गिमह८हर

3) Silver Oak

लसमवरओक

4) Pomegranate

अनार

Correct Answer Pomegranate

F A C E B O O K

P A G E h t t p w w w f a c e b o o k c o m s s c m e n t o r s o f f i c i a l P a g e | 50

FOR MORE UPDATES AND MORE MATERIAL DO LIKE OUR FACEBOOK PAGE httpwwwfacebookcomsscmentorsofficial

Q218 Between a tiger and an monkey

which of the following is different

एकबाघऔरबदरकहॳ बीचनननननलिखितमसहॳकह९नसीबातअिगहहॴ 3-Feb-2017

Options

1) Kingdom

राजय

2) Phylum

जानत

3) Order

िम

4) Class

वगय Correct Answer order

Q219 The artificial heart was invented by

कबतरमहदयका mdashmdashmdash

दवाराअपवषकारककयागयािा 3-Feb-2017

Options

1) Muhammad Yunus

महनमदयनस

2) Linus Yale Jr

िाइनसयहॳिजय

3) Gazi Yasargil

गाजीयासचगयि

4) Paul Winchell

पह९िपवमकि Correct Answer Paul Winchell

Q220 Tamarindus indica is the

scientific name of

टहॳमररनडसइडिका mdashmdash कावहॴजञाननकनामहहॴ 7-

Feb-2017

Options

1) Neem

नीम

2) Pineapple

अनानास

3) Tamarind

इमिी 4)Chiku

चीक

Correct Answer Tamarind

Q221 In eukaryotic cells synthesis of

RNA takes place in the

यकहॳ योटटककह८लिकाओमआरएनएकासशिहॳषण

mdashndash महह८ताहहॴ 7-Feb-2017

Options

1) Mitochondria

माईटह८कोडडरया 2) Centrioles

सटरीयह८मस

3) Ribosomes

ररबह८सह८नस

4) Nucleus

नयनकियस

Correct Answer nucleus

eukaryotic cell -Transcription is the

process of synthesizing ribonucleic acid

(RNA)Synthesis takes place within the

nucleus of eukaryotic cells or in the

cytoplasm of prokaryotes and converts

the genetic code from a gene in

deoxyribonucleic acid ( DNA ) to a

strand of RNA that then directs

proteinsynthesis

Q222 _________is caused by parasites

of the Plasmodium genus

पिाजमह८डियमजातीकहॳ परजीवी mdash- कहॳ कारणहहॴ 7-Feb-2017

Options

1) Dysentery

पहॳचचि

2) Malaria

मिहॳररया 3) Chickenpox

F A C E B O O K

P A G E h t t p w w w f a c e b o o k c o m s s c m e n t o r s o f f i c i a l P a g e | 51

FOR MORE UPDATES AND MORE MATERIAL DO LIKE OUR FACEBOOK PAGE httpwwwfacebookcomsscmentorsofficial

चहॳचक

4) Herpes

हहॳपपयस

Correct Answer Malaria

Q223 Carotene in fruits and vegetables

gives it which color

फिह८औरसनलजयोमनसितकहॳ रह८टीनउनहकह९नसारगपरदानकरताहहॴ 7-Feb-2017

Options

1) Green

हरा 2) Pink

गिाबी 3) Orange

नारगी 4) Blue

नीिा Correct Answer Orange

Q224 Equus Caballus is the scientific

name of

एकवसकहॴ बहॳिस mdashmdashndash कापवजञाननकनामहहॴ 7-Feb-2017

Options

1) Horse

घह८िा 2) Zebra

ज़हॳिा 3) Donkey

गधा 4) Buffalo

भस

Correct Answer Horse

Q225 Elapidae Naja is the scientific name of

एिीपीिीनाजा mdashmdash- कावहॴजञाननकनामहहॴ 8-Feb-2017

Options

1) Cobra

कह८बरा 2) Elephant

हािी 3) Eagle

ग ि

4) Owl

उमि Correct Answer Cobra

Q226 Which disease is caused due to

deficiency of Iron

िह८हकीकमीकहॳ कारणकह९नसारह८गहह८ताहहॴ 8-Feb-

2017

Options

1) Beriberi

बहॳरीबहॳरी 2) Tetany

टहॳटनी 3) Kwashiorkor

कवािीऔरकर

4) Anaemia

रकतामपता Correct Answer Anaemia

Beriberi is a disease caused by a vitamin

B-1 deficiency also known as thiamine

deficiency

Tetany can be the result of an

electrolyte imbalance Most often itrsquos a

dramatically low calcium level also

known as hypocalcemia Tetany can also

be caused by magnesium deficiency or

too little potassium Having too much

acid (acidosis) or too much alkali

(alkalosis) in the body can also result in

tetany

Kwashiorkor also known as

ldquoedematous malnutrition It is a form of

malnutrition caused by a lack of protein

in the diet

Anaemia means that you have fewer red

blood cells than normal or you have less

F A C E B O O K

P A G E h t t p w w w f a c e b o o k c o m s s c m e n t o r s o f f i c i a l P a g e | 52

FOR MORE UPDATES AND MORE MATERIAL DO LIKE OUR FACEBOOK PAGE httpwwwfacebookcomsscmentorsofficial

haemoglobin than normal in each red

blood cell

Q227 is a leaf where the leaflets are

arranged along the middle vein

mdashndashएकपततीहहॴजहापतरकह८कीरचनाक ररयालिराकहॳ आसपासहह८तीहहॴ 8-Feb-2017

Options

1) Pinnately compound leaf

पपनहॳटिीसयकतपतती 2) Palmately compound leaf

पामहॳटिीसयकतपतती 3) Compound leaf

सयकतपतती 4) Simple leaf

साधारणपतती Correct Answer Pinnately compound

leaf

Q228 Haustoria or sucking roots are

found in which of the following

हह८सटह८ररयायाचसनहॳवािीजड़हॳनननननलिखितमसहॳककसमपाईजातीहहॴ 8-Feb-2017

Options

1) Wheat

गहॳह

2) Mango

आम

3) Chestnut

चहॳसटनट

4) Cuscuta

कसकयटा Correct Answer Cuscuta

Haustorial roots -The roots of parasitic

plants which penetrate into the host

tissues to absorb nourishment are

called haustorial roots hellip Also known as suckingor parasitic roots

Q229 Equs Asinus is the scientific name

of

एकवसएलसनस mdashmdashndash कावहॴजञाननकनामहहॴ 8-

Feb-2017

Options

1) Donkey

गधा 2) Cow

गाय

3) Deer

टहरन

4) Kangaroo

कगा

Correct Answer Donkey

Q230 Ficus benghalensis is the scientific name of

फाईकसबहॳनगहॳिहॳलसस mdashndash कापवजञाननकनामहहॴ 8-Feb-2017

Options

1) Banyan

बरगद

2) Pineapple

अनानास

3) Babul

बबि

4) Tulsi

तिसी Correct Answer Banyan

Q231 Equus burchellii is the scientific name of

एकवसबचिी mdashmdash- कापवजञाननकनामहहॴ 8-Feb-2017

Options

1) Horse

घह८िा 2) Zebra

जहॳिा 3) Buffalo

F A C E B O O K

P A G E h t t p w w w f a c e b o o k c o m s s c m e n t o r s o f f i c i a l P a g e | 53

FOR MORE UPDATES AND MORE MATERIAL DO LIKE OUR FACEBOOK PAGE httpwwwfacebookcomsscmentorsofficial

भस

4) Ass

गधा Correct Answer Zebra

Page 15: COMPILATION OF ALL 72 SETS OF BIOLOGY SSC CHSL-2016 · OF BIOLOGY SSC CHSL-2016 PREPARED BY : SSC MENTORS BIOLOGY SPECIAL . F A C E B O O K P A G E : h t t p : / / w w w . f a c e

F A C E B O O K

P A G E h t t p w w w f a c e b o o k c o m s s c m e n t o r s o f f i c i a l P a g e | 14

FOR MORE UPDATES AND MORE MATERIAL DO LIKE OUR FACEBOOK PAGE httpwwwfacebookcomsscmentorsofficial

ends of small air passages and reach the

pleura where they can cause

inflammation and

scarring

Q55 Which one of the following is an

insectivorous plant

नननननलिखितमसहॳकह९नसाएकककटाहरीवनसपनतहहॴ

15-Jan-2017

Options

1) Utricularia

यटरीकिहॳररया 2) Sequoia

सहॳकयओइया 3) Nostoc

नॉसटह८क

4) Bryophyta

िायह८फाईटा Correct Answer Utricularia

Q56 ______________ is a

multibranched polysaccharide of

glucose that serves as a form of energy

storage in animals and fungi

mdashmdashगिकह८जकाएकबहिािायकतपह८िीसहॳकहॳ राइिहहॴ जह८जानवरोऔरकवकमउजायभणिारणकहॳ एक पमकाययकरताहहॴ 15-Jan-2017

Options

1) Cellulose

सहॳमयिह८ज

2) Glycogen

गिायकह८जन

3) Pectin

पहॳनकटन

4) Chitin

चीटटन

Correct Answer Glycogen

Q57 The largest gland of the human

body is

mdashmdashmdashमानविरीरकीसबसहॳबड़ीगरिीहहॴ 16-Jan-2017

Options

1) Pancreas

अगयािय

2) Thyroid

िायरॉइि

3) Large Intestine

बड़ीआत

4) Liver

यकत

Correct Answer Liver

Q58 Photosynthesis in plants takes

place in

वनसपनतयोमपरकािसशिहॳषणकीकियाहह८तीहहॴ

16-Jan-2017

Options

1) Stem

तना 2) Leaves

पनततयाा 3) Roots

जड़हॳ 4) Flower

फि

Correct Answer Leaves

During this reaction carbon dioxide

and water are converted into glucose

and oxygen The reaction requires light

energy which is absorbed by a green

substance called

chlorophyll Photosynthesis takes place

in leaf

cells These contain chloroplasts which

are tiny objects containing chlorophyll

F A C E B O O K

P A G E h t t p w w w f a c e b o o k c o m s s c m e n t o r s o f f i c i a l P a g e | 15

FOR MORE UPDATES AND MORE MATERIAL DO LIKE OUR FACEBOOK PAGE httpwwwfacebookcomsscmentorsofficial

Q59 Insects that transmit diseases are

known as

जह८कीड़हॳरह८गसचाररतकरतहॳहहॴ उनह mdashmdash-

कहॳ नामसहॳजानाजाताहहॴ 16-Jan-2017

1)Pathogens

रह८गज़नक

2) Vectors

वहॳकटर

3) Drones

परजीवी 4)Scalars

अटदषट

Correct Answer Vectors

A vector is an organism that does not

cause disease itself but which spreads

infection by conveying pathogens from

one host to another Species of mosquito

for example serve as vectors for the

deadly disease Malaria

Q60 Which is the second largest gland

of Human body

मानविरीरकीदसरीसबसहॳबड़ीगरिीकह९नसीहहॴ

SSC CHSL Science (biology)

2016 Question Paper

16-Jan-2017

Options

1) Liver

यकत

2) Large Intestine

बड़ीआत

3) Thorax

छाती 4) Pancreas

अगनयािय

Correct Answer Pancreas

Q61 Annona squamosa is the scientific

name of

एनह८नासकवामह८सा (Annona squamosa) mdash

mdashmdash कावहॴजञाननकनामहहॴ 16-Jan-2017

Options

1) Custard Apple

सीताफि

2) Papaya

पपीता 3) Babhul

बबि

4) Drumstick

सहजन

Correct Answer Custard Apple

Q62 The disease Beri Beri is caused due

to the deficiency of which of the

following

बहॳरीबहॳरीरह८गनननननलिखितमसहॳककसकीकमीकहॳकारणहह८ताहहॴ

16-Jan-2017

Options

1) Vitamin B2

पवटालमन B2

2) Vitamin B1

पवटालमन B1

3) Vitamin B12

पवटालमन B12

4) Vitamin E

पवटालमन E

Correct Answer Vitamin B1

Beriberi is a disease caused by a vitamin

B-1 deficiency also known as thiamine

deficiency

Q63 Chlorophyll was first isolated and

named by

किह८रह८कफिकह८ mdash-

दवारापहिहॳपिकऔरनालमतककयागया 16-Jan-2017

F A C E B O O K

P A G E h t t p w w w f a c e b o o k c o m s s c m e n t o r s o f f i c i a l P a g e | 16

FOR MORE UPDATES AND MORE MATERIAL DO LIKE OUR FACEBOOK PAGE httpwwwfacebookcomsscmentorsofficial

Options

1) Caventou

कहॳ वहॳत 2) Pelletier

पहॳिहॳटटयर

3) Chlorophyll

किह८रह८कफि

4) Caventou and Pelletier

कहॳ वहॳतऔरपहॳिहॳटटयर

Correct Answer Caventou and Pelletier

Chlorophyll was first isolated and

named by

Joseph Bienaimeacute Caventou and Pierre

Joseph Pelletier in 1817 The presence of

magnesium in chlorophyll was

discovered in 1906 and was the first

time that magnesium had been detected

in living tissue

Q64 Which of the following organisms

does not fit into the Cell Theory

नननननलिखितमसहॳकह९नसाजीवकह८लिकालसदातअन पनहीहहॴ

16-Jan-2017

Options

1) Bacteria

बहॴकटीररया 2) Virus

वायरस

3) Fungi

कवक

4) Plants

पह९धहॳ Correct Answer Virus

The bottom line is that viruses are not

alive and not related to cells in any way

The cell theory states that all living

things are made of cells cells are the

basic units of structure and function of

living things and that all cells come

from other cells Since viruses are not

made of cells and do not use cells in any

of their processes they are not related to

the cell theory

Q65 Which of these is not a

macronutrient for Plants

नननननलिखितमसहॳकह९नसापह९धह८कहॳ लिएमिह८नयटरीएटनहीहहॴ

SSC CHSL Science (biology) 2016

Question Paper

17-Jan-2017

Options

1) Nitrogen

नाइटरह८जन

2) Phosphorus

फासफह८रस

3) Potassium

पह८टालसयम

4) Chlorine

किह८रीन

Correct Answer Chlorine

In relatively large amounts the soil

supplies nitrogen phosphorus

potassium calcium magnesium and

sulfur these are often called the

macronutrients In relatively small

amounts the soil supplies iron

manganese boron molybdenum

copper zinc chlorine and cobalt the

so-called micronutrients

Q66 Name the respiratory organs of

insects

कीटह८मनसतिशरवसनअगनामकानामहहॴ

17-Jan-2017

Options

1) Skin

तवचा 2) Body Surface

िरीरकीसतह

F A C E B O O K

P A G E h t t p w w w f a c e b o o k c o m s s c m e n t o r s o f f i c i a l P a g e | 17

FOR MORE UPDATES AND MORE MATERIAL DO LIKE OUR FACEBOOK PAGE httpwwwfacebookcomsscmentorsofficial

3) Gills

गिफड़हॳ 4) Tracheae

शरावस- निी Correct Answer Tracheae

Air enters the respiratory systems of

insects through a series of external

openings called

spiracles These external openings

which act as muscular valves in some

insects lead to the internal respiratory

system a densely networked array of

tubes called tracheae

Q67 The poisonous gas accidentally

released in Bhopal Gas Tragedy is

भह८पािगहॴसतरासदीमगितीसहॳमकतहईजहरीिीगहॴसिी

17-Jan-2017

1) Methane

मीिहॳन

2) Nitrous Oxide

नाइटरसऑकसाइि

3) Methyl Isocyanate

महॴचििआयसोसायनहॳट

4) Cyanogen

सायनह८जहॳन

Correct Answer Methyl Isocyanate

Q68 What does Trypsin do

टटरनपसनकयाकरताहहॴ

SSC CHSL Science (biology) 2016

Question Paper

17-Jan-2017

Options

1) Breaks down Carbohydrates

काबोहाइडरहॳटकापवघटनकरताहहॴ 2) Synthesizes proteins

परह८टीनकासििहॳषणकरताहहॴ 3) Breaks down fats

वसाकापवघटनकरताहहॴ 4) Breaks down proteins

परह८टीनकापवघटनकरताहहॴ Correct Answer Breaks down proteins

Trypsin is one of the three principal

digestive

proteinases the other two being pepsin

and

chymotrypsin In the digestive process

trypsin acts with the other proteinases

to break down dietary protein molecules

to their component

peptides and amino acids

A protease is any enzyme that performs

proteolysis protein catabolism by

hydrolysis of peptide bonds

Q69 Name the source from which

Aspirin is produced

उससरह८तकानामबताइए

नजससहॳएनसपररनकाउतपादनककयाजाताहहॴ

17-Jan-2017

Options

1) Willow bark

पविह८कीछाि

2) Oak Tree

ओककावकष

3) Acacia

बबि

4) Eucalyptus

नीिचगरी Correct Answer Willow bark

The compound from which the active

ingredient in aspirin was first derived

salicylic acid was found in the bark of a

willow tree in 1763 by Reverend

Edmund Stone of Chipping-Norton

Q70 Cannis Familiaris is the scientific

name of

कहॴ ननसफहॳ लमलियहॳररस mdash- कावहॴजञाननकनामहहॴ

17-Jan-2017

F A C E B O O K

P A G E h t t p w w w f a c e b o o k c o m s s c m e n t o r s o f f i c i a l P a g e | 18

FOR MORE UPDATES AND MORE MATERIAL DO LIKE OUR FACEBOOK PAGE httpwwwfacebookcomsscmentorsofficial

Options

1) Cat

बबमिी 2)Dog

कतता 3) Fox

िह८मड़ी 4) Wolf

भहॳडड़या Correct Answer Dog

Q71 Harmful bacteria in potable water

make the water

पीनहॳकहॳ पानीमनसतिघातकबहॴकटीररयाउसपानीकह८बनातहॳहहॴ 17-Jan-2017

Options

1) unfit to drink

पीनहॳकहॳ लिएअयह८गय

2) smelly

दगयनधयकत

3) Colored

रगीन

4) Turbid

मटमहॴिा Correct Answer unfit to drink

Q72 Musa paradisiaca is the scientific

name of which plant

मसापहॴराडिलसयाकाककसपह९धहॳकावहॴजञाननकनामहहॴ

17-Jan-2017

Options

1) Mango

आम

2) Wheat

गहॳह

3) Corn

भ ा 4) banana

कहॳ िा Correct Answer banana

Q73 Prawns belong to which family

झीगहॳककसपररवारकहॳ हह८तहॳहहॴ 17-Jan-2017

Options

1) Crustaceans

िसटहॳलियन

2)Fish

मछिी 3) Amphibians

अननफबबयस

4) Reptiles

रहॳपटाइमस

Correct Answer Crustaceans

Q74 Name the drug that is yielded from

Cinchona tree and is used to cure

malaria

उसऔषचधकानामबताइएनजसहॳलसगकह८नापहॳड़सहॳपरापतककयाजाताहहॴऔरनजसकाउपयह८गमिहॳररयाकहॳ उपचारमककयाजाताहहॴ 17-Jan-2017

Options

1) Camptothea

कहॴ नटह८चिया 2) Acuminata

एकयलमनहॳटा 3) Quinine

कनहॴन

4) Cinchonia

लसकह८ननया Correct Answer Quinine

Q75 Blood Circulation was discovered

by

रकतपररसचरणकी mdashmdashndash दवारािह८जकीिी 17-Jan-2017

Options

1) Mary Anderson

F A C E B O O K

P A G E h t t p w w w f a c e b o o k c o m s s c m e n t o r s o f f i c i a l P a g e | 19

FOR MORE UPDATES AND MORE MATERIAL DO LIKE OUR FACEBOOK PAGE httpwwwfacebookcomsscmentorsofficial

महॴरीएिरसन

2) Virginia Apgar

वनजयननयाएपगार

3) William Harvey

पवलियमहाव

4) Robert Feulgen

रॉबटयफ़यिजहॳन Correct Answer William Harvey

Q76 Vitamin A is also known as

पवटालमन A कह८ mdashmdash- कहॳ नामसहॳभीजानाजाताहहॴ SSC CHSL Science (biology) 2016

Question Paper

18Jan2017

Options

1) Thiamine

िायलमन

2) Riboflavin

ररबह८फिहॳपवन

3) Retinol

रहॳटटनॉि

4) Calciferol

कहॴ नमसफहॳ रह८ि

Correct Answer Retinol

Q77 Some roots called arise from an

organ other than the radicle

कछजड़हॳनजनह mdashmdashmdash कहाजाताहहॴ वहमिकहॳ अिावाककसीअनयअगसहॳउतपननहह८तीहहॴ 18Jan2017

Options

1) tap roots

मखयजड़

2) stilt roots

ि ाजड़

3) fibrous roots

रहॳिहॳदारजड़

4) adventitious roots

आकनसमकजड़

Correct Answer adventitious roots

Q78 Spiders belong to which class of

animals

मकडड़यापराणीवगीकरणकहॳ ककसवगयमआतीहहॴ 18Jan2017

Options

1) Arachnids

एरहॳकननडस

2) Aves

एपवस

3) Gastropods

गहॴसटरोपह८िस

4) Anthozoa

एिह८जआ

Correct Answer Arachnids

Q79 How many layers does Human

Skin have

मानवतवचामककतनीपरतहॳहह८तीहहॴ

18Jan2017

Options

1) 5

2) 7

3) 11

4) 3

Correct Answer 3

Skin has three layers The epidermis

the outermost layer of skin provides a

waterproof barrier and creates our skin

tone The dermis beneath the

epidermis contains tough connective

tissue hair follicles and sweat glands

The deeper subcutaneous tissue (

hypodermis ) is made of fat and

connective tissue

Q80 Allium Cepa is the scientific name

of

एलियमलसपपा mdashmdashndash कावहॴजञाननकनामहहॴ 18Jan2017

F A C E B O O K

P A G E h t t p w w w f a c e b o o k c o m s s c m e n t o r s o f f i c i a l P a g e | 20

FOR MORE UPDATES AND MORE MATERIAL DO LIKE OUR FACEBOOK PAGE httpwwwfacebookcomsscmentorsofficial

Options

1) Carrot

गाजर

2) Tomato

टमाटर

3) Potato

आि 4) Onion

पयाज़

Correct Answer Onion

Q81 DNA stands for

िीएनएकापणय प mdashmdash- हहॴ 18Jan2017

Options

1) Di Nucleic Acid

िाईनयनकिकएलसि

2) Deoxy Nucleic Acid

िीओकसीनयनकिकएलसि

3) Diribonucleic Acid

िाईराइबह८नयनकिकएलसि

4) Deoxyribonucleic Acid

िीऑकसीराइबह८नयनकिकएलसि

Correct Answer Deoxyribonucleic Acid

Q82 Organisms that generate energy

using light are known as

जह८जीवाणपरकािकाउपयह८गकरउजायउतपननकरतीहहॴ उनह mdashmdash कहॳ पमजानाजाताहहॴ

18Jan2017

Options

1) Chaemolithotrophs

ककमह८लििह८टरह८पस

2) Oligotrophs

ओलिगह८टरह८पस

3) Bacteria

बहॴकटीररया 4)Photoautotrophs

फह८टह८ओटह८टरह८पस

Correct Answer Photoautotrophs

An oligotroph is an organism that can

live in an environment that offers very

low levels of nutrients

Q83 Which drug is used as an

Antidepressant

ककसदवाएकहतािारह८धीकहॳ पमपयोगककयाजाताहहॴ Options

1) Oxybutynin

ओकसीलयटीनन

2)Tramadol

टरहॳमहॳिह८ि

3 ) Sumatriptan

समहॳटरीपटहॳन

4) Bupropion

लयपरह८पपयह८न

Correct Answer Bupropion

लयपरह८पपयह८न

Q84 The orange colour of carrot is

because of

गाजरकानारगीरगनननननलिखितमसहॳककसीएककीवजहसहॳहह८ताहहॴ 18Jan2017

Options

1) it grows in the soil

यहलम ीमउगतीहहॴ 2) Carotene

कहॴ रह८टीन

3) it is not exposed to sunlight

यहसययपरकािकहॳ सपकय मनहीआती 4) the entire plant is oranqe in colour

सनपणयपह९धानारगीरगकाहह८ताहहॴ Correct Answer Carotene

Q85 Snake venom is highly modified

saliva containing

F A C E B O O K

P A G E h t t p w w w f a c e b o o k c o m s s c m e n t o r s o f f i c i a l P a g e | 21

FOR MORE UPDATES AND MORE MATERIAL DO LIKE OUR FACEBOOK PAGE httpwwwfacebookcomsscmentorsofficial

सापकाजहरअततयाचधकसिह८चधतिारहह८तीहहॴनजसमहॳ mdashmdash- हह८ताहहॴ Options

l)Prototoxins

परह८टह८टॉनकसस

2)Neutrotoxins

नयटरोटॉनकसस

3)Zootoxins

जटॉनकसस

4)Electrotoxins

इिहॳकटरह८टॉनकसस

Correct Answer Zootoxins

जटॉनकसस

Q86 Which type of pathogen causes the

water-borne disease Schistosomiasis

ककसपरकारकारह८गज़नकजिजननतरह८गलससटह८सह८लमलससकाकारणबनताहहॴ

18Jan2017

Option

1) Parasitic

परजीवी 2)Protozoan

परह८टह८जआ

3) Bacterial

बहॴकटीररयि

4) Viral

वायरि

Correct Answer Parasitic

Schistosomiasis also known as snail

fever and bilharzia is a disease caused

by parasitic

flatworms called schistosomes

Q87 Prothrombin responsible for

clotting of blood is released by

परह८िह८ननबन

जह८रकतकािककाजमनहॳकहॳ लिएनजनमहॳदारहहॴ mdashndash

कहॳ दवारासतरापवतककयाजाताहहॴ

19Jan2017

Options

1) Small Intestine

छह८टीआत

2) Blood Platelets

रकतपिहॳटिहॳटस

3) Large Intestine

बड़ीआत

4Heart

हदय

Correct Answer Blood Platelets

Q88 Acacia arabica is the scientific

name of

अकहॳ लियाअरहॳबबका mdashmdashndash कावहॴजञाननकनामहहॴ 19-Jan-2017

Options

1) Neem

नीम

2) Teak

सागह९न

3) Babhul

बबि

4) Pomegranate

अनार

Correct Answer Babhul

Q89 Cannis Vulpes is the scientific

name of

कहॴ ननसवनमपस mdashmdash- कावहॴजञाननकनामहहॴ 19-Jan-2017

Options

1) Dog

कतता 2) Wolf

भहॳडड़या 3) Fox

िह८मड़ी 4) Hyena

िाकिबगघा

F A C E B O O K

P A G E h t t p w w w f a c e b o o k c o m s s c m e n t o r s o f f i c i a l P a g e | 22

FOR MORE UPDATES AND MORE MATERIAL DO LIKE OUR FACEBOOK PAGE httpwwwfacebookcomsscmentorsofficial

Correct Answer Fox

Q90 The beetroot is the portion of the

beet plant

चकदरपह९धहॳका mdashmdashndash भागहहॴ 19-Jan-2017

Options

1) tap root

मखयजड़

2) Adventitious

आकनसमक

3) bulb of the stem

तनहॳकाकद

4) Rhizome

परकद

Correct Answer tap root

Q91 What is the basic unit of heredity

आनवलिकताकीबननयादीइकाईकयाहहॴ 19-Jan-2017

Options

1) DNA

िीएनए

2) RNA

आरएनए

3) Chromosome

िह८मह८सह८म

4) Gene

जीन

Correct Answer gene

Genes are the units of heredity and are

the instructions that make up the bodyrsquos

blueprint They code for the proteins

that determine virtually all of a personrsquos

characteristics Most genes come in

pairs and are made of strands of genetic

material called deoxyribonucleic acid

or DNA

Q92 Lungs are the primary organs of

फहॳ फड़हॳmdashndashकहॳ परािलमकअगहहॴ

19-Jan-2017

Options

1) Digestion

पाचन

2) Constipation

कलज

3) Perspiration

पसीना 4)Respiration

शवसन

Correct Answer Respiration

Q93 Sugarcane is a type of

गननाएकपरकारका mdash- हहॴ 20-Jan-2017

Options

1)creeper

िता 2)tree

पहॳड़

3)shrub

झाड़ी 4)grass

घास

Correct Answer grass

Q94 Who is commonly known as ldquothe

Father of Microbiologyrdquo

सामानयत ldquo सकषमजीवपवजञानकहॳ जनक lsquo

कहॳ नामसहॳककसहॳजानाजातहहॴ 20-Jan-2017

Options

1) Robert Hooke

रॉबटयहक

2) Antonie Philips van Leeuwenhoek

एटह८नीकफलिपवानमयएनहह८क

3) Carl Linnaeus

काियिीनाईयस

4) Charles Darwin

चामसयिापवयन

F A C E B O O K

P A G E h t t p w w w f a c e b o o k c o m s s c m e n t o r s o f f i c i a l P a g e | 23

FOR MORE UPDATES AND MORE MATERIAL DO LIKE OUR FACEBOOK PAGE httpwwwfacebookcomsscmentorsofficial

Correct Answer Antonie Philips van

Leeuwenhoek

Q95 For the aquatic organisms the

source of food is

जिीयजीवाणकािाघसरह८तहहॴ 20-Jan-2017

Options

1) Phytoplankton

फायटह८पिहॳकटन

2) Sea Weed

समदरीिहॴवाि

3)Aqua plankton

एकवापिहॳकटन

4) Zooplankton

जपिहॳकटन

Correct Answer Phytoplankton

Q96 Haemoglobin has the highest

affinity with which of the following

हीमह८गिह८बबनकीननननमसहॳककसकहॳ सािउततमसमानताहहॴ

20-Jan-2017

Options

1)SO2

2)CO2

3)CO

4)NO2

Correct Answer CO

It has a greater affinity for hemoglobin

than oxygen does It displaces oxygen

and quickly binds so very little oxygen

is transported through the body cells

Q97 Who developed the theory of

Evolution

उदपवकासकालसदातककसनहॳपवकलसतककया

20-Jan-2017

Options

1) Charles Darwin

चामसयिापवयन

2) Isaac Newton

आयजहॳकनयटन

3) Pranav Mistry

परणवलमसतरी 4) Galileo Galilei

गहॳलिलियह८गहॳिीिी Correct Answer Charles Darwin

Q98 The primary function of RNA is

RNA कापरािलमककाययहह८ताहहॴ 20-Jan-2017

Options

1) Photosynthesis

परकािसशिहॳषण

2) Protein Synthesis

परह८टीनसशिहॳषण

3) Replication

परनतकनतबनाना 4) Translation

अनवादकरना Correct Answer Protein Synthesis

There are two main functions of RNA

It assists DNA by serving as a messenger

to relay the proper genetic information

to countless numbers of ribosomes in

your body The other main function of

RNA is to select the correct amino acid

needed by each ribosome to build new

proteins for your body

Q99 ______is the movement of

molecules across a cell membrane from

a region of their lower concentration to

a region of their higher concertration

उचचसादरताकहॳ कषहॳतरसहॳउसकीकमसादरतावािहॳकषहॳतरकीतरफएककह८लिकाखझमिीकहॳ माधयमसहॳहह८नहॳवािाअणओकहॳ सचिनकह८ mdash- कहतहॳहहॴ Options

1) Diffusion

पवसरण

2) Osmosis

ऑसमह८लसस

F A C E B O O K

P A G E h t t p w w w f a c e b o o k c o m s s c m e n t o r s o f f i c i a l P a g e | 24

FOR MORE UPDATES AND MORE MATERIAL DO LIKE OUR FACEBOOK PAGE httpwwwfacebookcomsscmentorsofficial

3) Active Transport

सकियआवागमन

4) Passive Transport

नननषियआवागमन

Correct Answer Active Transport

Q100 Study of classification of

organisms is known as 20-Jan-2017

जीवाणओकहॳ वगीकरणकहॳ अधययनकह८ mdash-

कहाजाताहहॴ Options

1) Serpentology

सपरहॳटह८िह८जी 2) Virology

वायरह८िह८जी 3) Taxonomy

टहॴकसोनह८मी 4) Physiology

कफनज़यह८िह८जी Correct Answer Taxonomy

Q101 Photosynthesis takes place inside

plant cells in

परकािसशिहॳषणवनसपनतकह८लिकामनसति mdash

mdashmdash महह८ताहहॴ 20-Jan-2017

Options

1) Ribosomes

राइबह८सह८नस

2) Chloroplasts

किह८रह८पिासट

3) Nucleus

नयकलियम

4) Mitochondria

माईटह८कोडडरया Correct Answer Chloroplasts

Q102 ______ is the cell organelle in

which the biochemical processes of

respiration and energy production

occur

mdashmdash- वहकह८लिकाअगहहॴ नजसमहॳशवसनऔरउजायउतपादनकहॳ जहॴसीजहॴवरासायननकपरकियायहह८तीहहॴ 20-Jan-2017

Options

1) Mitochondria

माइटह८कोडडरया 2) Chloroplast

किह८रह८पिासट

3) Ribosomes

राइबह८सह८नस

4) Nucleus

नयकिीयस

Correct Answer Mitochondria

Q103 Which non-flowering spore

bearing plants have roots

ककसफिनिगनहॳवािहॳऔरबीजाणधारकपह९धह८कीजड़हॳहह८तीहहॴ 21-Jan-2017

Options

1) Mosses

मह८सहॳस

2) Angiosperms

एननजयह८सपनसय 3) Ferns

फनसय 4) Gymnosperms

नजननह८सपनसय Correct Answer ferns

Q104 Which of the following is an

excretory organ of cockroach

नननननलिखितमसहॳकह९नसानतिच हॳकाउतसजयनअगहहॴ

21-Jan-2017

Options

F A C E B O O K

P A G E h t t p w w w f a c e b o o k c o m s s c m e n t o r s o f f i c i a l P a g e | 25

FOR MORE UPDATES AND MORE MATERIAL DO LIKE OUR FACEBOOK PAGE httpwwwfacebookcomsscmentorsofficial

1) Malphigian Tubules

मनमफनजयनटयबमस

2) Nephridia

नहॳकफरडिया 3) Coxal Gland

कह८कसिगरचिया 4) Green Gland

गरीनगरचिया Correct Answer Malphigian Tubules

Q105 Evaporation of water takes place

in which part of plants

पानीकहॳ वाषपीकरणकीकियापह९धोकहॳ ककसभागसहॳहह८तीहहॴ 21-Jan-2017

Options

1) Stem

तना 2) Stomata

सटह८मटा 3) Branch

िािाए

4) Fruit

फि

Correct Answer Stomata

Evaporation accounts for the movement

of water to the air from sources such as

the soil canopy interception and

waterbodies Transpiration accounts for

the movement of water within a plant

and the subsequent loss of water as

vapour through stomata in its leaves

Q106 A is the fleshy spore-bearing

fruiting body of a fungus

mdashmdashndashकवककामासि

बीजाणधारणकरनहॳवािाफिनहॳवािाअगहहॴ 21-

Jan-2017

Options

1) aloe vera

एिह८वहॳरा 2) Coral

मगा 3) Cactus

कहॴ कटस

4) Mushroom

ककरमतता Correct Answer mushroom

Q107 Which of the following is a fungal

disease

नननननलिखितमसहॳकह९नसाफफदसहॳहह८नहॳवािाएकरह८ग हहॴ

21-Jan-2017

Options

1) Dermatitis

तवचािह८ध

2) Cholera

हहॴजा 3) Jaundice

पीलिया 4) Indigofera

इननिगह८फहॳ रा Correct Answer Dermatitis

Dermatitis also known as eczema is a

group of diseases that results in

inflammation of the skin These diseases

are characterized by itchiness red skin

and a rash In cases of short duration

there may be small blisters while in

long-term cases the skin may become

thickened

Q108 In which form is glucose stored in

our body

हमारहॳिरीरमगिकह८जकासचयककस पमककयाजाताहहॴ

21-Jan-2017

Options

1) Insulin

F A C E B O O K

P A G E h t t p w w w f a c e b o o k c o m s s c m e n t o r s o f f i c i a l P a g e | 26

FOR MORE UPDATES AND MORE MATERIAL DO LIKE OUR FACEBOOK PAGE httpwwwfacebookcomsscmentorsofficial

इसलिन

2) Glucose

गिकह८ज

3) Glycogen

गिायकह८जहॳन

4) Fat

वसा Correct Answer Glycogen

Excess glucose is stored in the liver as

the large compound called glycogen

Glycogen is a polysaccharide of glucose

but its structure allows it to pack

compactly so more of it can be stored in

cells for later use

Q109 Where do plants synthesize

protein from

पह९धहॳपरह८टीनसशिहॳषणकहासहॳकरतहॳहहॴ

Options

1) Fatty Acids

वसाऐलसि

2) Sugar

िकर

3) Amino Acids

एलमनह८ऐलसि

4) Starch

सटाचय Correct Answer Amino Acids

Q110 Which part of the brain is

responsible for triggering actions like

thinking intelligence memory and

ability to learn

मनसतषककाकह९नसाटहससासह८चनहॳ बनधदमानी याददाशतऔरसीिनहॳकीकषमताजहॴसीकियाओकह८परहॳररतकरताहहॴ 21-Jan-2017

Options

1) Diencephalon

िायएनसहॳफहॳ िह८न

2) Hypothalamus

हयपह८िहॳिहॳमस

3) Cerebrum

सहॳरहॳिम

4) Control

कटरह८ि

Correct Answer Cerebrum

Q111 Which of the following is also

known as the Biochemical Laboratory

of the Human Body

नननननलिखितमसहॳककसहॳमानविरीरकीजहॴवरसायनपरयह८गिािाभीकहाजाताहहॴ 21-Jan-2017

Options

1) Small Intestine

छह८टीआत

2)Brain

मनसतषक

3) Pancreas

अगनयािय

4) Liver

नजगर

Correct Answer Liver

The liver makes bile that will help

emulsify and digest the fats we eat

The liver takes toxic substances and

convert them using enzymes the liver

cells makes into a non toxic form so the

body can dispose of them

The liver also converts fats protein and

carbohydrates into glucose which is the

energy source for our cells to use

The liver takes amino acids and makes

proteins by combining them

Q112 The yellow colour of human urine

is due to

मानवमतरकापीिारग mdashndash कीवजहसहॳहह८ताहहॴ 22-

Jan-2017

Options

1) Bile Salts

F A C E B O O K

P A G E h t t p w w w f a c e b o o k c o m s s c m e n t o r s o f f i c i a l P a g e | 27

FOR MORE UPDATES AND MORE MATERIAL DO LIKE OUR FACEBOOK PAGE httpwwwfacebookcomsscmentorsofficial

पपततनमक

2) Cholesterol

कह८िहॳसटरह८ि

3) Lymph

लिनफ

4) Urochrome

यरह८िह८म

Correct Answer Urochrome

Urobilin or urochrome is the chemical

primarily responsible for the yellow

color of urine

Q113 The wilting of plants takes place

due to

पह९धह८कालिचििहह८नाकी mdashmdash- कीवजहसहॳहह८ताहहॴ 22-Jan-2017

Options

1)Photosynthesis

परकािसशिहॳषण

2) Transpiration

वाषपह८तसजयन

3) Absorption

अविह८षण

4) Respiration

शरवसन

Correct Answer Transpiration

Wilting is the loss of rigidity of non-

woody parts of plants This occurs when

the turgor pressure in non-lignified

plant cells falls towards zero as a result

of diminished water in the cells

Q114 Bovidae Ovis is the scientific name of

बह८पविीओपवस mdashndash कावहॴजञाननकनामहहॴ 22-Jan-2017

Options

1) Goat

बकरी 2) Cow

गाय

3) Buffalo

भहॳस

4) Sheep

भहॳड़

Correct Answer Sheep

Q115 Plants get their energy to produce

food from which of the following

पह८धहॳभह८जनकाननमायणकरनहॳकहॳ लिएनननननलिखितमसहॳककससहॳउजायपरापतकरतहॳहहॴ

22-Jan-2017

Options

1) Photosynthesis

परकािसशिहॳषण

2)Bacteria

बहॴकटीररया 3)Fungi

कवक

4)Sun

सयय Correct Answer Sun

Q116 Which of the following is secreted

by the liver

नननननलिखितमसहॳककसकासरावनजगरसहॳहह८ताहहॴ

22-Jan-2017

Options

1) Glucose

गिकह८ज

2) Iodine

आयह८िीन

3) Cortisol

काटटरयसह८ि

4) Bile

पपतत

Correct Answer Bile

The liver makes bile that will help

emulsify and

digest the fats we eat

F A C E B O O K

P A G E h t t p w w w f a c e b o o k c o m s s c m e n t o r s o f f i c i a l P a g e | 28

FOR MORE UPDATES AND MORE MATERIAL DO LIKE OUR FACEBOOK PAGE httpwwwfacebookcomsscmentorsofficial

Q117 Ferns belong to which division of

plants

फनसयपह९धह८कहॳ ककसभागमआतहॳहहॴ

22-Jan-2017

Options

1) Gymnosperms

नजननह८सपनसय 2) Angiosperms

एनजयह८सपनसय 3) Thallophyta

िहॴिह८फाईटा 4)Pteridophyta

टहॳररिह८फाईटा Correct Answer Pteridophyta

Q118 Who invented Antibiotics

एटीबायह८टटककाअपवषकारककसनहॳककयािा

22-Jan-2017

Options

1) Joseph Lister

जह८सहॳफलिसटर

2) William Harvey

पवलियमहाव

3) Robert Knock

रॉबटयनॉक

4)Alexander Fleming

अिहॳकज़िरफिहॳलमग

Correct Answer Alexander Fleming

Q119 Milbecycin is used in the

eradication of

लममबहॳसायलसनका mdashndash

मउनमिनमपरयह८गककयाजाताहहॴ 22-Jan-2017

Options

1) Agricultural Fungus

कपषकवक

2) Agricultural Pests

कपषकीटक

3) Agricultural Herbs

कपषिाक

4)Agricultural Weeds

कपषननराना Correct Answer Agricultural Pests

Milbemycin oxime is a veterinary drug

from the group of milbemycins used as

a broad spectrum antiparasitic It is

active against worms and mites(insects

Q120 Intestinal bacteria synthesizes

which of the following in the human

body

मानविरीरमआतोकहॳ बहॴकटीररयानननननलिखितमसहॳककसकासशिहॳषणकरतहॳहहॴ 22-Jan-2017

Options

1) Vitamin K

पवटालमन K

2) Proteins

परह८टीन

3) Fats

वसा 4) Vitamin D

पवटालमन D

Correct Answer Vitamin K

Q121 is the study of the physical form

and external structure of plants

mdashmdash-

मपह९धह८काभहॴनतक पऔरबाहरीसरचनाकाआदयाककयाजाताहहॴ 22-Jan-2017

Options

1) Physiology

कफनजयह८िह८जी 2) Anatomy

िरीररचनापवजञान

3) Phytomorphology

फाईटह८मह८फह८िह८जी 4)Cytology

कह८लिकापवजञान

Correct Answer Phytomorphology

F A C E B O O K

P A G E h t t p w w w f a c e b o o k c o m s s c m e n t o r s o f f i c i a l P a g e | 29

FOR MORE UPDATES AND MORE MATERIAL DO LIKE OUR FACEBOOK PAGE httpwwwfacebookcomsscmentorsofficial

Q122 Which of the following is a

structural and functional unit of

kidneys

नननननलिखितमसहॳकह९नसीगदोकीसरचनातमकऔरकाययकरीईकाईहहॴ

22-Jan-2017

Options

1) Renette Cells

रहॳनहॳटकह८लिकाए

2) Flame Cells

फिहॳमकह८लिकाए

3) Nephrites

नहॳफ़राइटस

4)Nephrons

नहॳफरोस

Correct Answer Nephrons

Nephron functional unit of the kidney

the structure that actually produces

urine in the process of removing waste

and excess substances from the blood

There are about 1000000 nephrons in

each human kidney

Q123 Which of the following is the

largest part of the human brain

नननननलिखितमसहॳकह९नसामानवमनसतषककासबसहॳबड़ाटहससाहहॴ

23-Jan-2017

Options

1) Ribs

पसलियाा 2) Cerebrum

सहॳरहॳिम

3) Pons

पोस

4)Thalamus

िहॴिहॳमस

Correct Answer Cerebrum

The cerebrum is the largest part of the

human brain making up about two-

thirds of the brainrsquos mass It has two

hemispheres each of which has four

lobes frontal parietal temporal and

occipital

Q124 The auxiliary buds

सहायककालियाmdashndash 23-Jan-2017

Options

1) grow endogenously from the pericycle

पहॳरीसाईककिसहॳअनतजातयपवकलसतहह८ताहहॴ 2) arise endogenously from the main

growing point

मिवपदसहॳअनतजातयउठताहहॴ 3) is an embryonic shoot located in the

axil of a leaf

एकभरणिटहहॴजह८एकपततीकहॳ अकषपरनसतिहह८ताहहॴ 4)arise exogenously from the epidermis

एपपिलमयससहॳबटहजातयतरीकहॳ सहॳउठताहहॴ Correct Answer is an embryonic shoot

located in the axil of a leaf

Q125 Which of the following is a viral

disease

इनमहॳसहॳकह९सीएकवायरिबीमारीहहॴ

23-Jan-2017

Options

1) Polio

पह८लियह८ 2) Tetanus

धनसतनभ

3) Leprosy

कषठरह८ग

4) Plague

पिहॳग

Correct Answer Polio

A viral disease (or viral infection)

occurs when an organismrsquos body is

invaded by pathogenic viruses and

infectious virus particles (virions) attach

to and enter susceptible cells

F A C E B O O K

P A G E h t t p w w w f a c e b o o k c o m s s c m e n t o r s o f f i c i a l P a g e | 30

FOR MORE UPDATES AND MORE MATERIAL DO LIKE OUR FACEBOOK PAGE httpwwwfacebookcomsscmentorsofficial

Poliomyelitis often called polio or

infantile paralysis is an infectious

disease caused by the poliovirus

Tetanusmdash A serious bacterial infection

that causes painful muscle spasms and

can lead to death

Leprosy also known as Hansenrsquos

disease (HD) is a long-term infection by

the bacterium Mycobacterium leprae or

Mycobacterium lepromatosis

Plague is an infectious disease caused by

the bacterium Yersinia pestis

Symptoms include fever weakness and

headache

Q126 Which organisms can help to

carry out Vermicomposting

कह९नसाजीववमीकनपह८नसटगममददकरताहहॴ

23-Jan-2017

Options

1) Nitrifying Bacteria

नाईटरीफाईगबहॴकटीररया 2) Earthworms

कहॴ चऐ

3) Algae

िहॴवि

4) Fungus

कवक

Correct Answer Earthworms

Q127 Contraction of heart is also

known as

हदयकहॳ सकचनकह८ mdash- भीकहाजाताहहॴ 23-Jan-

2017

Options

1) Systole

लससटह८ि

2) Aristotle

अरसत

3) Diastole

िायसटह८ि

4) Lub

मयब

Correct Answer Systole

Diastole is the part of the cardiac cycle

when the heart refills with blood

following systole (contraction)

Ventricular diastole is the period during

which the ventricles are filling and

relaxing while atrial diastole is the

period during which the atria are

relaxing

Q128 Azadirachta indica is the

botanical name of which of the

following

अजाटदराचताइडिकानननननलिखितमसहॳककसकावानसपनतनामहहॴ

23-Jan-2017

Options

1) Rose plant

गिाबकापह९धा 2) Apple tree

सहॳबकापहॳड़

3) Neem

नीम

4)Mango

आम

Correct Answer Neem

Q129 Which of the following is the

main end product of carbohydrate

digestion

नननननलिखितमसहॳकह९नसाकाबोहाइडरहॳटकहॳ पाचनकापरमिअतउतपादकहह८ताहहॴ 23-Jan-2017

Options

1) Fats

वसा 2) Lipids

लिपपडस

3) Glucose

गिकह८ज

4) Cellulose

F A C E B O O K

P A G E h t t p w w w f a c e b o o k c o m s s c m e n t o r s o f f i c i a l P a g e | 31

FOR MORE UPDATES AND MORE MATERIAL DO LIKE OUR FACEBOOK PAGE httpwwwfacebookcomsscmentorsofficial

सहॳमयिह८ज

Correct Answer Glucose

Intestinal absorption of end products

from digestion of carbohydrates and

proteins in the pig hellip During absorption some sugars (fructose or

galactose) released from the

corresponding sucrose and lactose

respectively during digestion were

partly metabolized into glucose by the

enterocyte

Q130 Which of the following glands is a

source of the enzyme Ptyalin

नननननलिखितगरचियोमसहॳएजाइमटयालिनकासरह८तहहॴ 23-Jan-2017

Options

1) Pancreas

अगरािय

2) Thyroid Gland

िाइराइिगरिी 3) Pituitary Gland

पीयषगरिी 4) Salivary Glands

िारगरचियाा Correct Answer Salivary Glands

Q131 Which of the following is not true

about Pteridophyta

ननननमसहॳकह९नसीबातटहॳररिह८फाईटकहॳ बारहॳमसचनहीहहॴ 23-Jan-2017

Options

1) Dominant phase is saprophytes

परमिचरणसहॳपरह८फाईइटसहह८ताहहॴ 2) Main plant body is diploid

पह९दह८कामखयिरीरदपवगखणतहह८ताहहॴ 3) Seeds are present

बीजमह९जदहह८तहॳहहॴ 4)Flowers are absent

फिअनपनसतिहह८तहॳहहॴ

Correct Answer Seeds are present

Q132 The largest dolphin species is the

orca also called as

िॉिकफनकीसबसहॳबड़ीपरजानतकाकानामआकायहहॴनजसहॳ mdash- भीकहतहॳहहॴ 23-Jan-2017

Options

1) Bottle Nose

बाटिनह८ज

2) Baiji

बहॳजी 3) Killer whale

ककिरहहॳि

4)Tucuxi

टकवसी Correct Answer Killer whale

Q133 The fat digesting enzyme Lipase

is secreted by which of the following

वसाकापाचनकरनहॳवािाएजाइमिाइपहॳजनननननलिखितमसहॳककसकहॳ दवारासतरापवतहह८ताहहॴ

24-Jan-2017

Options

1) Kidneys

गद

2) Pancreas

अगनयािय

3) Large Intestine

बड़ीआत

4)Liver

नजगर

Correct Answer Pancreas

Lipase is an enzyme that splits fats so

the intestines can absorb them Lipase

hydrolyzes fats like triglycerides into

their component fatty acid and glycerol

molecules It is found in the blood

gastric juices pancreatic secretions

intestinal juices and adipose tissues

F A C E B O O K

P A G E h t t p w w w f a c e b o o k c o m s s c m e n t o r s o f f i c i a l P a g e | 32

FOR MORE UPDATES AND MORE MATERIAL DO LIKE OUR FACEBOOK PAGE httpwwwfacebookcomsscmentorsofficial

Q134 The arrangement of leaves on an

axis or stem is called

एकअकषयातनहॳपरपनततयोकीयवसिाकह८कयाकहाजाताहहॴ SSC CHSL Science (biology) 2016

Question Paper

24-Jan-2017

Options

1) Phyllotaxy

फाइिह८टहॴकसी 2) Vernation

वनिन

3) Venation

वहॳनहॳिन

4)Phytotaxy

फाइटह८टहॴकसी Correct Answer Phyllotaxy

In botany phyllotaxis or phyllotaxy is

the arrangement of leaves on a plant

stem (from Ancient Greek phyacutellon

ldquoleafrdquo and taacutexis ldquoarrangementrdquo)

Phyllotactic spirals form a distinctive

class of patterns in nature

Q135 The study of Cells is also known

as

कह८लिकाओकहॳ अधययनकह८ mdashmdashndash

भीकहाजाताहहॴ 24-Jan-2017

Options

1) Cytology

सायटह८िह८जी 2) Physiology

कफनजयह८िह८जी 3) Nucleology

नयककमयह८िह८जी 4)Cellology

सहॳिह८िह८जी Correct Answer Cytology

Q136 Which of the following scientists

is also known as the Father of Biology

नननननलिखितमसहॳककसवहॴजञाननककह८ ldquoजीवपवजञानकहॳ जनकrdquoकहॳ नामसहॳभीजानाजाताहहॴ 24-Jan-2017

Options

1) Herbert Spencer

हबयटयसपसर

2) Aristotle

अरसत 3) Lamarck

िहॳमाकय 4)Darwin

िापवयन

Correct Answer Aristotle

Q137 Which cells give rise to various

organs of the plant and keep the plant

growing

कह९नसीकह८लिकाएपह९धह८कहॳ लभननअगह८कह८जनमदहॳतीहहॴऔरपह९धह८कह८बढ़नहॳममददकरतीहहॴ

24-Jan-2017

Options

1) Permanent

सिायी 2) Dermal

तवचीय

3) Meristematic

मररसटहॳमटटक

4)Mature

परह८ढ़

Correct Answer Meristematic

A meristem is the tissue in most plants

containing undifferentiated cells

(meristematic cells) found in zones of

the plant where growth can take place

Q138 Rodentia Muridae is the scientific

name of

F A C E B O O K

P A G E h t t p w w w f a c e b o o k c o m s s c m e n t o r s o f f i c i a l P a g e | 33

FOR MORE UPDATES AND MORE MATERIAL DO LIKE OUR FACEBOOK PAGE httpwwwfacebookcomsscmentorsofficial

रह८िहॳलियानयररिी mdashmdash- कावहॴजञाननकनामहहॴ 24-

Jan-2017

Options

1) Mouse

चहा 2) Squirrel

चगिहरी 3) Monkey

बदर

4) Lizard

नछपकिी Correct Answer Mouse

Q139 Name the scientist who proposed

the cell theory

कह८लिकालसदातकापरसतावदहॳनहॳवािहॳवहॴजञाननककानामबताइए 24-Jan-2017

Options

1) Schleiden and Schwann

िीमिनऔरशरववान

2) Lamarck

िहॳमाकय 3) Treviranus

टरहॳवायरहॳनस

4)Whittaker and Stanley

हीटकरऔरसटहॳनिहॳ Correct Answer Schleiden and

Schwann

Q140 The flower with the worldrsquos

largest bloom is

दननयाकासबसहॳबड़ाफिखििनहॳवािा mdashmdashndash हहॴ 24-Jan-2017

Options

1) Pando

पािह८ 2) Posidonia

पह८सीिह८ननया 3) Rafflesia arnoldii

ररफिहॳलियाअनोमिी 4)Helianthus annuus

हहॳलिएनिसएनयअस

Correct Answer Rafflesia arnoldii

Rafflesia arnoldii is a species of

flowering plant in the parasitic genus

Rafflesia It is noted for producing the

largest individual flower on earth It has

a very strong and horrible odour of

decaying flesh earning it the nickname

ldquocorpse flower

Q141 Deficiency of which vitamin

causes night blindness

ककसपवटालमनकीकमीकहॳ कारणरतौधीहह८ताहहॴ 24-Jan-2017

Options

1) Vitamin K

पवटालमन K

2) Vitamin C

पवटालमन C

3) Vitamin B1

पवटालमन B1

4)Vitamin A

पवटालमन A

Correct Answer Vitamin A

Q142 Nongreen plants lack which of the

following

गहॴर-

हररतवनसपनतमनननननलिखितमसहॳककसकीकमीहह८तीहहॴ

24-Jan-2017

Options

1) Chlorophyll

किह८रह८कफि

2) Lycophyll

िायकह८कफि

3) Cyanophyll

F A C E B O O K

P A G E h t t p w w w f a c e b o o k c o m s s c m e n t o r s o f f i c i a l P a g e | 34

FOR MORE UPDATES AND MORE MATERIAL DO LIKE OUR FACEBOOK PAGE httpwwwfacebookcomsscmentorsofficial

सायनह८कफि

4)Phototropism

फह८टह८टरोपपजम

Correct Answer Chlorophyll

Q143 Organisms that use light to

prepare food are known as

जह८जीवपरकािकाउपयह८गकरभह८जनतहॴयारकरतहॳहहॴ उनह mdashmdash- कहॳ पमजानजाताहहॴ 24-Jan-2017

Options

1) Autotrophs

सवपह८षी 2) Heterotrophs

पवषमपह८षज

3) Omnivores

सवायहारी 4)Decomposers

पवघटनकरनहॳवािा Correct Answer Autotrophs

autotrophs often make their own food

by using sunlight carbon dioxide and

water to form sugars which they can use

for energy Some examples of

autotrophs include plants algae and

even some bacteria Autotrophs

(producer) are important because they

are a food source for heterotrophs

(consumers)

A heterotroph is an organism that

ingests or absorbs organic carbon

(rather than fix carbon from inorganic

sources such as carbon dioxide) in order

to be able to produce energy and

synthesize compounds to maintain its

life Ninety-five percent or more of all

types of living organisms are

heterotrophic including all animals and

fungi and some bacteria

Q144 Which of the following is a

primary function of haemoglobin

नननननलिखितमसहॳकह९नसाटहमह८गिह८बबनकाएकपरािलमककाययहहॴ

25-Jan-2017

Options

1) Utilization of energy

उजायकाउपयह८गकरना 2) Prevention of anaemia

रकतामपताहह८नहॳसहॳरह८कना 3) Destruction of bacteria

बहॴकटीररयाकापवनािकरना 4) To transport oxygen

ऑकसीजनकावहनकरना Correct Answer To transport oxygen

Q145 Vascular bundles are absent in

सवहनीबिि mdashmdash- मअनपनसतिरहतहॳहहॴ 25-Jan-2017

Options

1) Bryophyta

िायह८फाइटा 2) Pteridophyta

टहॳररिह८फाईटा 3) Gymnosperms

नजननह८सपमय 4) Angiosperms

एननजयह८सपहॳनसय Correct Answer Bryophyta

Q146 Sauria Lacertidae is the scientific

name of

सहॴररयािहॳसरटाईिी mdashmdashndash कावहॴजञाननकनामहहॴ 25-Jan-2017

Options

1) Crocodile

मगरमचछ

2) Hippopotamus

टहपपह८पह८टहॳमस

3) Lizard

नछपकिी 4) House fly

F A C E B O O K

P A G E h t t p w w w f a c e b o o k c o m s s c m e n t o r s o f f i c i a l P a g e | 35

FOR MORE UPDATES AND MORE MATERIAL DO LIKE OUR FACEBOOK PAGE httpwwwfacebookcomsscmentorsofficial

घरहॳिमकिी Correct Answer Lizard

Q147 Which type of pathogen causes

the water-borne disease SARS (Severe

Acute Respiratory Syndrome)

ककसपरकािकारह८गज़नकजिजननतबीमारीसासयकाकारणबनताहहॴ 25-Jan-2017

Options

1) Viral

वायरि

2) Parasitic

परजीवी 3) Protozoan

परह८टह८जअन

4) Bacterial

बहॴकटीररयि

Correct Answer Viral

Q148 Which of the following organs

produces the enzyme lipase

नननननलिखितमसहॳकह९नसाअगिायपहॳजएजाइमउतपननकरताहहॴ 25-Jan-2017

Options

1) Pancreas

अगनयािय

2) Large Intestine

बड़ीआत

3) Liver

नजगर

4) Small Intestine

छह८टीआत

Correct Answer Pancreas

Q149 A is a long internode forming the

basal part or the whole of a peduncle

एक mdashmdash- एकिबाइटरनह८िहहॴ जह८ननचिाटहससायासनपणयिठिबनताहहॴ 25-

Jan-2017

Options

1) Rhizome

परकद

2) Rachis

महॳ दि

3) floral axis

पषपअकष

4) Scape

भगदड़

Correct Answer scape

Q150 ndash Which of the following

organisms are considered to be both

Living and Non-living

नननननलिखितमसहॳकह९नसहॳजीवाणकह८जीपवतऔरअजीपवतमानाजाताहहॴ

25-Jan-2017

Options

1) Bacteria

बहॴकटीररया 2) Fungi

कवक

3) Algae

िहॴवाि

4)Virus

वायरस

Correct Answer Virus

They are considered to be living as they

possess a protein coat as a protective

covering DNA as the genetic material

etc

They are said to be non-living as they

can be crystallised and they survive for

billions of years They can tolerate high

temperatures freezing cold

temperatures ultra-violet radiations etc

Q151 Deficiency of fluorine causes

which of the following

फिह८ररनकीकमीकहॳ कारणनननननलिखितमसहॳकयाहह८ताहहॴ

F A C E B O O K

P A G E h t t p w w w f a c e b o o k c o m s s c m e n t o r s o f f i c i a l P a g e | 36

FOR MORE UPDATES AND MORE MATERIAL DO LIKE OUR FACEBOOK PAGE httpwwwfacebookcomsscmentorsofficial

27-Jan-2017

Options

1) Dental Caries

िटिकहॴ ररज

2) Scurvy

सकवरी 3) Anaemia

रकतामपता 4) Arthritis

गटठया Correct Answer Dental Caries

Q152 In a Punnett Square with the

cross AaBb x AaBb how many Aabb

genotypes would be created

पनहॳटसककायरमिह८स AaBb x AaBb कहॳ साि

ककतनहॳ Aabb जीनह८टाइपबनगहॳ 27-Jan-2017

Options

1) 1

2) 8

3) 2

4) 3

Correct Answer 2

Q153 Which of the following is the

Controlling Center of the Cell

नननननलिखित म सहॳ कह८लिकाका ननयतरण

क दर कह९न हहॴ

27-Jan-2017

Options

1) Nucleus

क दर

2) Plasma

पिाजमा 3) Lysosome

िायसह८सह८म

4) Chromosome

िह८मह८सह८म

Correct Answer Nucleus

The control centre of the cell is the

nucleus in eukaryotic cells The nucleus

contains genetic material in the form of

DNA

Q154 Myopia affects which of the

following organs

मायह८पपयानननननलिखितअगह८मसहॳककसहॳपरभापवतकरताहहॴ

25-Jan-2017

Options

1) Heart

हदय

2) Skin

तवचा 3) Eyes

आािहॳ 4)Mouth

मह

Correct Answer Eyes

Q155 Which of the following bears

flowers

नननननलिखितमसहॳकह९नफिधारणकरताहहॴ

25-Jan-2017

Options

1) Bryophyta

िायह८फाइटा 2) Pteridophyta

टहॳरीिह८फाईटा 3) Gymnosperms

नजननह८सपमय 4)Angiosperms

एननजयह८सपमय Correct Answer Angiosperms

Q156 Oxygenated blood flows out of the

heart through the

ऑकसीजनयकतरकत mdashmdashmdash

कहॳ माधयमसहॳहदयकहॳ बाहरबहताहहॴ 25-Jan-2017

F A C E B O O K

P A G E h t t p w w w f a c e b o o k c o m s s c m e n t o r s o f f i c i a l P a g e | 37

FOR MORE UPDATES AND MORE MATERIAL DO LIKE OUR FACEBOOK PAGE httpwwwfacebookcomsscmentorsofficial

Options

1) Aorta

महाधमनी 2) pulmonary artery

फहॳ फड़हॳकीधमनी 3) vena cava

वहॳनाकावा 4)Atrium

चह९क

Correct Answer aorta

Q157 Blood leaving the liver and

moving towards the

heart has a higher concentration of

नजगरसहॳननकिकरहदयकीतरफजानहॳवािहॳरकतम mdashmdashmdashmdash कीउचचसादरताहह८तीहहॴ 27-Jan-2017

Options

1) Lipids

लिपपडस

2) Urea

यररया 3) Bile Pigments

पपततकहॳ रगकरण

4) Carbon dioxide

काबयनिायऑकसाइि

Correct Answer Bile Pigments

Urea is nitrogen containing substance

which is produced in the liver in order

to deal with excess amino-acids in the

body As urea is produced it leaves the

liver in the blood stream and passes via

the circulatory system to all parts of the

body

Q158 Bulb is a modification of which

part of a plant

बमबएकपह९धहॳकहॳ ककसटहससहॳकाएक पातरणहह८ताहहॴ 27-Jan-2017

Options

1) The root

जड़

2) The stem

तना 3) The radicle

मिाकर

4)The fruit

फि

Correct Answer The stem

Q159 Which of the following carries

blood away from the heart to different

body parts

इनमहॳसहॳकह९नरकतकह८हदयसहॳिरीरकहॳ पवलभननअगह८तकिहॳजातीहहॴ

27-Jan-2017

Options

1) Arteries

धमननया 2) Nerves

तबतरहाए

3) Capillaries

कहॳ लिकाए

4)Veins

नसहॳ Correct Answer Arteries

Q160 The series of processes by which

nitrogen and its compounds are

interconverted in the environment and

in living organisms is called

27-Jan-2017

Options

1)Absorption of Nitrogen

2)Ammonification

3)Nitrogen Fixation

4)Nitrogen Cycle

Correct Answer Nitrogen Cycle

Ammonification or Mineralization is

performed by bacteria to convert

organic nitrogen to ammonia

F A C E B O O K

P A G E h t t p w w w f a c e b o o k c o m s s c m e n t o r s o f f i c i a l P a g e | 38

FOR MORE UPDATES AND MORE MATERIAL DO LIKE OUR FACEBOOK PAGE httpwwwfacebookcomsscmentorsofficial

Nitrification can then occur to convert

the ammonium to nitrite and nitrate

Nitrogen fixation is a process by which

nitrogen in the Earthrsquos atmosphere is

converted into ammonia (NH3) or other

molecules available to living organisms

Q161 BCG vaccine is given to protect

from which of the following

बीसीजीकाटटकानननननलिखितमसहॳककसकहॳ बचावकहॳ लिएटदयाजातहहॴ

27-Jan-2017

Options

1) Jaundice

पीलिया 2) Anaemia

रकतमपता 3) Tuberculosis

कषयरह८ग

4) Polio

पह८लियह८ Correct Answer Tuberculosis

Q162 Parallel venation is found in

समानतरवहॳनहॳिन mdashmdashmdash- मपायाजाताहहॴ 27-Jan-2017

Options

1) plants which are monocots

पह९धहॳजह८एकबीजपतरीहह८तहॳहहॴ 2) plants which have a dicot stem

वहॳपह९धहॳनजनकातनादपवदलियहह८ताहहॴ 3) plants with leaves similar to Tulsi

वहॳपह९धहॳनजनकीपनततयतिसीकीपनततयोकहॳ समानहह८तहॳहहॴ 4)plants with tap roots

टहॳप टवािहॳपह९धहॳ Correct Answer plants which are

monocots

Q163 The hardest part of the body is

िरीरकासबसहॳकठह८रभाग mdashndash हहॴ 27-Jan-2017

Options

1) Bones

हडडिय

2) Tooth Enamel

दातकहॳ इनहॳमि

3) Skull

िह८पड़ी 4) Spinal Cord

महॳ रजज

Correct Answer Tooth Enamel

Q164 Which type of pathogen causes

the waterborne disease E coli Infection

ककसपरकारकारह८गजननकजिजननतरह८गईकह८िाईसिमणकाकारणबनताहहॴ 27-Jan-2017

Options

1) Protozoan

परह८टह८जआ

2) Parasitic

परजीवी 3) Bacterial

बहॴकटीररयि

4)Viral

वायरि

Correct Answer Bacterial

Q165 The amount of blood filtered

together by both the kidneys in a 70 kg

adult male human in a minute is

70 की गरा वािहॳएकवयसकप षमएकलमनटमदह८नोगदकहॳदवाराएकसािचाबनीगयीरकतकीमातरहह८तीहहॴ 29-Jan-2017

Options

1) 1100 ml

1100 लमलि

2) 100 ml

F A C E B O O K

P A G E h t t p w w w f a c e b o o k c o m s s c m e n t o r s o f f i c i a l P a g e | 39

FOR MORE UPDATES AND MORE MATERIAL DO LIKE OUR FACEBOOK PAGE httpwwwfacebookcomsscmentorsofficial

100 लमलि

3) 1500 ml

1500 लमलि

4) 500 ml

500 लमलि

Correct Answer 1100 ml

Q166 Which feature of a plant helps to

distinguish a monocot from a dicot

पह९धहॳकीवहकह९नसीपविहॳषताहहॴजह८एकदपवदलियहॳऔरएकएकदिीयपह९धहॳसहॳभहॳदकरनहॳममददकरतीहहॴ 29-Jan-2017

Options

1) Pollination

परागम

2) Venation

वहॳनहॳिन

3) Vernation

वनिन

4) Aestivation

एसटीवहॳिहॳन

Correct Answer venation

Q167 The Mutation Theory was

proposed by

उतवररवतयनकालसदात mdashmdashndash

कहॳ दवरापरसतापवतककयाजाताहहॴ 29-Jan-2017

Options

1) Charles Lyell

चामसयलियहॳि

2) William Smith

पवलियमनसमि

3) Hugo De Vries

हयगह८िीराईस

4)Harrison Schmitt

हहॳरीसननसमट

Correct Answer Hugo De Vries

Q168 Which type of pathogen causes

the waterborne disease HepatitisA

ककसपरकारकहॳ रह८गजनकजिजननतरह८गहहॳपहॳटाइटटस-A काकारणबनताहहॴ

29-Jan-2017

Options

1) Parasitic

परजीवी 2) Viral

वायरि

3) Protozoan

परह८टह८जआ

4) Bacterial

बहॴकटीररयि

Correct Answer Viral

Q169 In a Punnett Square with the

cross AaBb x Aabb how many AaBb

genotypes would be created

पनहॳटसकवायरमिह८स AaBb x Aabb

कहॳ सािककतनहॳ AaBb जीनह८टाइपबनगहॳ 29-Jan-

2017

Options

1) 4

2) 1

3) 7

4) 6

Correct Answer 4

Q170 Arboreal Ateles is the scientific

name of

अिह८ररयिएटटलिस mdashmdashmdash कावहॴजञाननकनामहहॴ 29-Jan-2017

Options

1) Squirrel

चगिहरी 2) Sparrow

गह८रहॴया 3) Lizard

नछपकिी 4) Spider monkey

F A C E B O O K

P A G E h t t p w w w f a c e b o o k c o m s s c m e n t o r s o f f i c i a l P a g e | 40

FOR MORE UPDATES AND MORE MATERIAL DO LIKE OUR FACEBOOK PAGE httpwwwfacebookcomsscmentorsofficial

मकड़ीबदर

Correct Answer Spider monkey

Q171 Which type of pathogen causes

the waterborne disease Salmonellosis

ककसपरकारकारह८गाणजिजननतबीमारीसािमह८नहॳिह८लसज़काकारकहहॴ

29-Jan-2017

Options

1) Algal

िहॳवालियहॳ 2) Parasitic

परजीवी 3) Bacterial

बहॴकटीररयि

4)Viral

वायरि

Correct Answer Bacterial

An infection with salmonella bacteria

commonly caused by contaminated food

or water

Symptoms include diarrhoea fever

chills and abdominal pain

Q172 is a condition in which there is a

deficiency of red cells or of haemoglobin

in the blood

mdashmdash-

एकनसिनतहहॴनजसमहॳरकतमिािकह८लिकाओकीयाहीमह८गिह८बबनकीकमीहह८तीहहॴ 29-Jan-2017

Options

1) Albinism

एनमबननजम

2) Propyria

परह८पीररया 3) Anaemia

एनीलमया 4)Keloid disorder

कहॳ िह८इिडिसओिर

Correct Answer Anaemia

Q173 Ananas comosus is the scientific

name of

Options

अनानासकह८मह८सस mdashmdashmdashndash

कावहॴजञाननकनामहहॴ 29-Jan-2017

1) Custard Apple

सीताफि

2) Pineapple

पाइनएपपि

3) Bamboo

बास

4)Pomegranate

अनार

Correct Answer Pineapple

Q174 Which organ produces insulin

कह९नसाअगइनसलिनपहॴदाकरताहहॴ 29-Jan-

2017

Options

1) Liver

यकत

2) Thyroid gland

िायराइिगरिी 3) Spleen

पिीहा 4)Pancreas

अगरयिय

Correct Answer Pancreas

Q175 Which of the following disease is

not caused by water pollution

नननननलिखितमसहॳकह९नसारह८गपानीकहॳ परदषणकहॳकारणनहीहह८ता

29-Jan-2017

Options

1) Cholera

हहॴजा 2) Typhoid

F A C E B O O K

P A G E h t t p w w w f a c e b o o k c o m s s c m e n t o r s o f f i c i a l P a g e | 41

FOR MORE UPDATES AND MORE MATERIAL DO LIKE OUR FACEBOOK PAGE httpwwwfacebookcomsscmentorsofficial

टाइफाइि

3) Asthma

दमा 4)Diarrhoea

दसत

Correct Answer Asthma

Q176 Ocimum tenuiflorum is the

scientific name of

ओलिलममटहॳयईफिह८रमइसकावहॴजञाननकनाम mdash

ndash हहॴ 30-Jan-2017

Options

1) Neem

नीम

2) Mango

आम

3) Babul

बबि

4)Tulsi

तिसी Correct Answer Tulsi

Q177 Which gland secretes bile a

digestive fluid

कह९नसीगरिीपपतत एकपाचनतरिपरदािय सरापवतकरतीहहॴ 30-Jan-2017

Options

1) Pancreas

अगनयािय

2) Liver

यकत

3) Thyroid

िायराइि

4) Testes

टहॳनसटस

Correct Answer liver

Q178 In which of the following the

dominant phase is Gametophyte

नननननलिखितमसहॳककसकहॳ परमिचरणयगमकह८दपवधद (Gametophyte)हहॴ 30-Jan-2017

Options

1) Bryophyta

िायह८फाइटा 2) Pteridophyta

टहॳररिह८फाइटा 3) Gymnosperms

नजननह८सपमय 4) Angiosperms

एननजयह८सपमय Correct Answer Bryophyta

Q179 Anaerobic respiration refers to

which of the following

नननननलिखितमसहॳककसहॳअवायवीयशवसनकहाजाताहहॴ

30-Jan-2017

Options

1) Respiration without Oxygen

ऑकसीजनकहॳ बबनाशवसन

2) Respiration with Oxygen

ऑकसीजनकहॳ सािशवसन

3) Respiration without CO2

काबयनिायऑकसाइिकहॳ बबनाशवसन

4) Respiration with CO2

काबयनिायऑकसाइिकहॳ सािशविन

Correct Answer Respiration without

Oxygen

Q180 Which type of pathogen causes

the waterborne disease Cholera

ककसपरकारकारह८गजनकजिजननतरह८गहहॴजाकाकारणबनताहहॴ

30-Jan-2017

Options

1) Algal

िहॴवालियहॳ

F A C E B O O K

P A G E h t t p w w w f a c e b o o k c o m s s c m e n t o r s o f f i c i a l P a g e | 42

FOR MORE UPDATES AND MORE MATERIAL DO LIKE OUR FACEBOOK PAGE httpwwwfacebookcomsscmentorsofficial

2) Bacterial

बहॴकटीररयि

3) Protozoan

परह८टह८जआ

4) Viral

वायरि

Correct Answer Bacterial

Q181 To which class does

Oxyreductases transferases hydrolases

belong

ओकसीररिकटहॳसटरासफरहॳजहॳस

हाइडरह८िहॳसहॳसककसवगयमआतहॳहहॴ 30-Jan-2017

Options

1) Hormones

हारमोस

2) Enzymes

एजाइनस

3) Proteins

परह८टीनस

4) Vitamins

पवटालमनस

Correct Answer Enzymes

Q182 Which of the following is not true

about Gymnosperms

ननननमसहॳकह९नसीबातअनावतबीजीकहॳ बारहॳमसचनहीहहॴ 30-Jan-2017

Options

1) Dominant phase is saprophytes

परमिचरणसहॳपरह८फाइटसहह८ताहहॴ 2) Vascular bundles are absent

सवहनीबििअनपनसितहह८ताहहॴ 3) spores are heterospores

बीजाणहहॳटहॳरह८सपह८रसहह८तहॳहहॴ 4) Flowers are absent

फिअनपनसितहह८तहॳहहॴ

Correct Answer Vascular bundles are

absent

Q183 The name of first mammal clone sheep is

भहॳड़कीपरिमसतनपायीपरनत प (किह८न)

कानामहहॴ 30-Jan-2017

Options

1) Noori

नरी 2) Dolly

िॉिी 3) Louise

िसी 4)Durga

दगाय Correct Answer Dolly

Q184 Which type of pathogen causes

the water-borne disease Typhoid fever

ककसपरकारकारह८गजनकजिजननतरह८गटाइफाइिबिारकाकारणबनताहहॴ 30-Jan-2017

Options

1) Algal

िहॴवािीय

2) Parasitic

परजीवी 3) Protozoan

परह८टह८जनअन

4)Bacterial

बहॴकटीररयि

Correct Answer Bacterial

Q185 In which part of the cell are

proteins made

कह८लिकाकहॳ ककसटहससहॳमपरह८टीनबनायाजाताहहॴ

31-Jan-2017

Options

1) Reticulum

रहॳटटकिम

F A C E B O O K

P A G E h t t p w w w f a c e b o o k c o m s s c m e n t o r s o f f i c i a l P a g e | 43

FOR MORE UPDATES AND MORE MATERIAL DO LIKE OUR FACEBOOK PAGE httpwwwfacebookcomsscmentorsofficial

2) Golgi apparatus

गह८मजीएपहॳरहॳटस

3) Ribosomes

ररबह८सह८नस

4) Lysosome

िायसह८सह८नस

Correct Answer ribosomes

Proteins are produced by stringing

amino acids together in the order

specified by messenger RNA strands

that were transcribed from DNA in the

cell nucleus The process of synthesizing

a protein is called translation and it

occurs on ribosomes in the cytoplasm of

a cell

Q186 Polio is a disease caused by which

of the following

नननननलिखितमसहॳपह८लियह८कीबबमारह८हह८नहॳकाकारणकयाहहॴ

31-Jan-2017

Options

1) Bacteria

बहॴकटीररयि

2) Mosquito

मचछर

3) Virus

वायरस

4) Cockroach

नतिच हॳ Correct Answer Virus

Polio or poliomyelitis is a crippling and

potentially deadly infectious disease It

is caused by the poliovirus

Q187 ndash Hay fever is a sign of which of

the following

हहॳकफवरनननननलिखितमसहॳककसकाएकसकहॳ तहहॴ

31-Jan-2017

Options

1) Old Age

वदावसिा 2) Malnutrition

कपह८सण

3) Allergy

एिनजय 4) Over Work

अतयचधककाययकरना Correct Answer Allergy

Q188 How many chromosomes does a

human cell contain

एकमानवकह८लिकामककतनहॳगणसतरहह८तहॳहहॴ

29-Jan-2017

Options

1) 6

2) 26

3) 46

4) 66

Correct Answer 46

In humans each cell normally contains

23 pairs of chromosomes for a total of

46 Twenty-two of these pairs called

autosomes look the same in both males

and females The 23rd pair the sex

chromosomes differ between males and

females

Q189 Which of the following is not true

about Bryophyta

ननननमसहॳकह९नसीबातिायह८फाइटकहॳ बारहॳमसचनहीहहॴ 31-Jan-2017

Options

1) Dominant phase is gametophytes

परमिचरणगहॳलमतह८फाइटसहह८ताहहॴ 2) Main plant body is haploid

पह९धहॳकामखयिरीरअगखणतहह८ताहहॴ 3) Spores are homospores

बीजाणहह८मह८सफह८रसहह८तहॳहहॴ 4) Flowers are present

फिमह८जदहह८तहॳहहॴ Correct Answer Flowers are present

F A C E B O O K

P A G E h t t p w w w f a c e b o o k c o m s s c m e n t o r s o f f i c i a l P a g e | 44

FOR MORE UPDATES AND MORE MATERIAL DO LIKE OUR FACEBOOK PAGE httpwwwfacebookcomsscmentorsofficial

Q190 Which aquatic animal has

trailing tentacles

ककसजिीयजानवरकहॳ पीछहॳचिनहॳवािहॳटहॳटकिसहह८तहॳहहॴ

31-Jan-2017

Options

1) Sea horse

समदरीघह८िा 2) Corals

मगा 3) Jelly fish

जहॳिीमछिी 4) Star fish

तारामछिी Correct Answer Jelly fish

Jellyfish with its umbrella-shaped bell

and trailing tentacles

Q191 Which type of pathogen causes

the water-borne disease Poliomyelitis

(Polio)

ककसपरकारकारह८गजनकजिजननतरह८गपह८लियह८मायहॳटटस (पह८लियह८) काकारणहहॴ 31-Jan-

2017

Options

1) Parasitic

परजीवी 2) Algal

िहॴवालिय

3) Viral

वायरि

4) Bacterial

बहॴकटीररयि

Correct Answer Viral

Q192 The outer white part of the eye

that protects the inner structures is

आािकाबाहरीसफहॳ दटहससाजह८आतररकसरचनाओकीरकषाकरताहहॴ वह mdashmdashmdash हहॴ 31-Jan-

2017

Options

1) Iris

आयररस

2) Sclera

सकिहॳरा 3) Retina

रहॳटटना 4) Cornea

कह८ननयया Correct Answer Sclera

Q193 Proteins are made up of

परह८टीनकाननमायण mdashndash सहॳहह८ताहहॴ 31-Jan-2017

Options

1) Amino acids

एलमनह८अनि

2) Fatty acids

वसायकतअनि

3) Glucose

गिकह८ज

4)Nucleotides

नयनकियह८टाईिस

Correct Answer Amino acids

Q194 Moringa Oleifera is the scientific

name of

मह८ररगओलिफहॳ रा mdashmdashndash कावहॴजञाननकनामहहॴ 31-Jan-2017

Options

1) Banyan

बरगद

2) Gulmohar

गिमह८हर

3) Amla

आमिा

F A C E B O O K

P A G E h t t p w w w f a c e b o o k c o m s s c m e n t o r s o f f i c i a l P a g e | 45

FOR MORE UPDATES AND MORE MATERIAL DO LIKE OUR FACEBOOK PAGE httpwwwfacebookcomsscmentorsofficial

4) Drumstick

डरमनसटक

Correct Answer Drumstick

Q195 Kidney stones are composed of

गदकीपिरी mdashndash सहॳबनीहह८तीहहॴ 1-Feb-2017

Options

1) Calcium Oxalate

कहॴ नमसयमओकजहॳिहॳट

2) Sodium Chloride

सह८डियमकिह८राइि

3) Magnesium Nitrate

महॳनगनलियमनाइतटरहॳट

4) Calcium Bicarbonate

कहॴ नमियमबायकबोनहॳट

Correct Answer Calcium Oxalate

Q196 ndash Which of the following is not

true about Angiosperms

ननननमसहॳकह९नसीबातआवतबीजीकहॳ बारहॳमसचनहीहहॴ 1-Feb-2017

Options

1) Dominant phase is gametophytes

परमिचरणगहॳलमतह८फाइटहह८ताहहॴ 2) Vascular bundles are present

सवहनीबििमह९जदहह८ताहहॴ 3) Spores are heterospores

बीजाणहहॳटहॳरह८सपह८रसहह८तहॳहहॴ 4) Seeds are covered

बीजढकहॳ हह८तहॳहहॴ Correct Answer Dominant phase is

gametophytes

Q197 All of the following are excretory

(waste) products of animals except

नननननलिखितमसहॳककसएककह८छह८ड़करअनयसभीपराखणयोदवाराउतसनजयतपदाियहहॴ 1-Feb-

2017

Options

1) Uric Acid

यररकएलसि

2) Ammonia

अमह८ननया 3) Carbohydrates

काबोहाइडरहॳट

4) Urea

यररया Correct Answer Carbohydrates

In animals the main excretory products

are carbon dioxide ammonia (in

ammoniotelics) urea (in ureotelics) uric

acid (in uricotelics) guanine (in

Arachnida) and creatine

Q198 RNA is a polymeric molecule

What does RNA stand for

आरएनइएएकबहिकआणहहॴ इसकाकापवय पकयाहहॴ 1-Feb-2017

Options

1) Rado Nuclear Acid

रािह८नयनकियरएलसि

2) Ribo Nucleic Acid

राइबह८नयनकिकएलसि

3) Rhino Nuclear Acid

हाइनह८नयनकियरएलसि

4) Resto Nucleus Acid

रहॳसटह८नयकिीयसएलसि

Correct Answer Ribo Nucleic Acid

Q199 Which organ does detoxification

and produces chemicals needed for

digestion

कह९नसाअगपवषहरणकरताहहॴऔरपाचनकहॳ लिएआवशयकरसायनोकह८पहॴदाकरताहहॴ 1-Feb-

2017

Options

1) Salivary glands

िारगरचिया 2) Pancreas

अगनयािय

F A C E B O O K

P A G E h t t p w w w f a c e b o o k c o m s s c m e n t o r s o f f i c i a l P a g e | 46

FOR MORE UPDATES AND MORE MATERIAL DO LIKE OUR FACEBOOK PAGE httpwwwfacebookcomsscmentorsofficial

3) Thyroid gland

िायराइिगरिी 4) Liver

यकत

Correct Answer Liver

Q200 Psidium guajava is the scientific

name of

लसडियमगआजावा mdashmdash कावहॴजञाननकनामहहॴ 1-

Feb-2017

Options

1) Guava

अम द

2) Mango

आम

3) Bamboo

बास

4) Jack fruit

कटहि

Correct Answer Guava

Q201 Which drug is used as a Blood

Thinner

चधरकह८पतिाकरनहॳकहॳ पमककसदवाकापरयह८गककयाजाताहहॴ

1-Feb-2017

Options

1) Warfarin

वाफर न

2) Tramadol

टरहॳमािह८ि

3) Azithromycin

एनजरह८मायलसन

4) Hydralazine

हाइडरह८िहॳनजन

Correct Answer Warfarin

Q202 Which of the following disease is

caused due to the deficiency of protein

परह८टीनकीकमीकहॳ कारणनननननलिखितमसहॳकह९नसारह८गहह८ताहहॴ 1-Feb-2017

Options

1) Arthritis

गटठया 2) Kwashiorkor

कािीओकय र

3) Goitre

गाइटर

4) Night Blindness

रतह९चध

Correct Answer Kwashiorkor

Q203 A is species of plant that has

adapted to survive in an environment

with little liquid water

mdashmdashndashपह९धहॳकीएकऐसहॳऐसहॳपरजानतहहॴ नजसनहॳकमपानीवािहॳवातावरणमजीपवतरहनहॳकहॳलिएअनकिनहहॴ 1-Feb-2017

Options

1) Xerophyte

म दपवद

2) Hydrophyte

जिीयपादप

3) Mesophyte

समह८दपवद

4) Thallophyte

िहॴिह८फाइटा Correct Answer xerophyte

xerophyte is a species of plant that has

adapted to survive in an environment

with little liquid water such as a desert

or an ice- or snow-covered region in the

Alps or the Arctic

Mesophytes are terrestrial plants which

are adapted to neither a particularly

dry nor particularly wet environment

An example of a mesophytic habitat

would be a rural temperate meadow

F A C E B O O K

P A G E h t t p w w w f a c e b o o k c o m s s c m e n t o r s o f f i c i a l P a g e | 47

FOR MORE UPDATES AND MORE MATERIAL DO LIKE OUR FACEBOOK PAGE httpwwwfacebookcomsscmentorsofficial

which might contain goldenrod clover

oxeye daisy and Rosa multiflora

thallophyte any of a group of plants or

plantlike organisms (such as algae and

fungi) that lack differentiated stems

leaves and roots and that were formerly

classified as a primary division

(Thallophyta) of the plant kingdom

Q204 How many types of teeth are

there in humans

मनषयोमककतनहॳपरकारकहॳ दातहह८तहॳहहॴ

1-Feb-2017

Options

1) 4

2) 5

3) 2

4) 3

Correct Answer 4

teeth -Humans have four types of

teethincisors canines premolars and

molars each with a specific function

The incisors cut the food the canines

tear the food and the molars and

premolars crush the food

Q205 Carica papaya is the scientific name of

कहॴ ररकापपाया mdashmdashndash कावहॴजञाननकनामहहॴ 2-

Feb-2017

Options

1) Peepal

पीपि

2) Papaya

पपीता 3) Tamarind

इमिी 4) Drumstick

ढह८िकाछड़ी Correct Answer Papaya

Q206 Muscles get tired when there is

shortfall of

जब mdashndash कीकमीहह८तीहहॴतबपहॳिीयिकजातीहहॴ 2-Feb-2017

Options

1) Lactic acid

िहॴनकटकएलसि

2) Na+ ions

Na+ आयन

3) ATP

एटीपी 4) Sulphates

समफहॳ टस

Correct Answer ATP

ATP is the energy source muscle fibers

use to make muscles contract

muscle tissuersquos main source of energy

called adenosine triphosphate or ATP

As your muscles use up this energy

source they become tired and fatigued

Oxygen is the key ingredient that helps

create new ATP to replenish the burned

up ATP in your muscles

Q207 Artocarpus integra is the

scientific name of आटह८कापयसइटीगरा mdashmdashmdash कावहॴजञाननकनामहहॴ 2-Feb-2017

Options

1) Guava

अम द

2) Pineapple

अनानास

3) Silver Oak

लसमवरओक

4) Jack fruit

कटहि

Correct Answer Jack fruit

Q208 Which organ stores fat soluble

vitamins

कह९नसाअगवसामघिनिीिपवटालमनह८काभिाराकरताहहॴ

2-Feb-2017

F A C E B O O K

P A G E h t t p w w w f a c e b o o k c o m s s c m e n t o r s o f f i c i a l P a g e | 48

FOR MORE UPDATES AND MORE MATERIAL DO LIKE OUR FACEBOOK PAGE httpwwwfacebookcomsscmentorsofficial

Options

1) Blood

रकत

2) Skin

तवचा 3) Liver

यकत

4) Pancreas

अगनयािय

Correct Answer Liver

Q209 Which disease is caused due to

deficiency of Iodine

आयह८िीनकहॳ कारणकह९नसारह८गहह८ताहहॴ 2-Feb-2017

Options

1) Rickets

ररकहॳ टस

2) Scurvy

सकवी 3) Goitre

गणमािा 4) Growth retardation

पवकासका कना Correct Answer Goitre

rickets A softening and weakening of

bones in children usually due to

inadequate vitamin D

Q210 Grevillea Robusta is the scientific name of

गरहॳपवलियारह८बसटा mdashmdashmdash- कापवजञाननकनामहहॴ 2-Feb-2017

Options

1) Peepal

पीपि

2) Teak

सागह९न

3) Silver Oak

लसमवरओक

4) Jack fruit

कटहि

Correct Answer Silver Oak

Q211 When a Cuttlefish is described as a Molluscs it is at which level of

classification

जबएककटिकफिकह८एकमह८िसकाकहॳ पमवखणयतककयाजाताहहॴतबयहॳवगीकरणकहॳ ककससतरपहॳनसितहहॴ 2-Feb-2017

Options

1) Class

वगय 2) Order

िम

3) Family

पररवार

4) Phylum

सघ

Correct Answer Phylum

Q212 Bambusa dendrocalmus is the

scientific name of बानबसािहॳडराकामस mdashmdashmdash कावहॴजञाननकनामहहॴ 3-Feb-2017

Options

1) Banyan

बरगद

2) Papaya

पपीता 3) Bamboo

बास

4) Pomegranate

अनार

Correct Answer Bamboo

Q213 Acinonyx Jubatus is the scientific name of

एलसनह८ननकसजयबहॳटस mdashmdashmdash

कावहॴजञाननकनामहहॴ 3-Feb-2017

F A C E B O O K

P A G E h t t p w w w f a c e b o o k c o m s s c m e n t o r s o f f i c i a l P a g e | 49

FOR MORE UPDATES AND MORE MATERIAL DO LIKE OUR FACEBOOK PAGE httpwwwfacebookcomsscmentorsofficial

Options

1) Bear

भाि 2) Horse

घह८िा 3) Cheetah

चीता 4) Zebra

जहॳिा Correct Answer Cheetah

Q214 The pale yellow colour of urine is

due to the presence of which pigment

मतरकाफीकापीिारगरगदरयकहॳ उपनसिनतकहॳ कारणहह८ताहहॴ

3-Feb-2017

Options

1) Urochrome

यरह८िह८म

2) Urophyll

यरह८कफि

3) Chlorophyll

किह८रह८कफि

4) Chloroplast

किह८रह८पिासट

Correct Answer Urochrome

Q215 Which of the following constitute

to form a gene

नननननलिखितमसहॳकह९नसीचीज़एकजीनकागठनकरतीहहॴ

3-Feb-2017

Options

1) Polynucleotides

पह८िीनयनकियह८टाईडस

2) Hydrocarbons

हाइडरह८काबोस

3) Lipoproteins

िाईपह८परह८टीनस

4) Lipids

लिपपडस

Correct Answer Polynucleotides

Polynucleotide molecule is a biopolymer

composed of 13 or more nucleotide

monomers covalently bonded in a chain

DNA (deoxyribonucleic acid) and RNA

(ribonucleic acid) are examples of

polynucleotides with distinct biological

function

Q216 Vertebrates belongs to the

phylum

रीढ़कीहडिीवािहॳपराणी mdashmdashmdash

परजानतकहॳ अतगायतआतहॳहहॴ 3-Feb-2017

Options

1) Arthropoda

आरह८पह८ड़ा 2) Annelida

एननलििा 3) Cnidaria

ननिहॳररया 4) Chordata

कह८िटा Correct Answer Chordata

Q217 Punica granatum is the scientific name of

पननकगरहॳनहॳटस mdashmdashmdash कावहॴजञाननकनामहहॴ 3-Feb-2017

Options

1) Custard Apple

सीताफि

2) Gulmohar

गिमह८हर

3) Silver Oak

लसमवरओक

4) Pomegranate

अनार

Correct Answer Pomegranate

F A C E B O O K

P A G E h t t p w w w f a c e b o o k c o m s s c m e n t o r s o f f i c i a l P a g e | 50

FOR MORE UPDATES AND MORE MATERIAL DO LIKE OUR FACEBOOK PAGE httpwwwfacebookcomsscmentorsofficial

Q218 Between a tiger and an monkey

which of the following is different

एकबाघऔरबदरकहॳ बीचनननननलिखितमसहॳकह९नसीबातअिगहहॴ 3-Feb-2017

Options

1) Kingdom

राजय

2) Phylum

जानत

3) Order

िम

4) Class

वगय Correct Answer order

Q219 The artificial heart was invented by

कबतरमहदयका mdashmdashmdash

दवाराअपवषकारककयागयािा 3-Feb-2017

Options

1) Muhammad Yunus

महनमदयनस

2) Linus Yale Jr

िाइनसयहॳिजय

3) Gazi Yasargil

गाजीयासचगयि

4) Paul Winchell

पह९िपवमकि Correct Answer Paul Winchell

Q220 Tamarindus indica is the

scientific name of

टहॳमररनडसइडिका mdashmdash कावहॴजञाननकनामहहॴ 7-

Feb-2017

Options

1) Neem

नीम

2) Pineapple

अनानास

3) Tamarind

इमिी 4)Chiku

चीक

Correct Answer Tamarind

Q221 In eukaryotic cells synthesis of

RNA takes place in the

यकहॳ योटटककह८लिकाओमआरएनएकासशिहॳषण

mdashndash महह८ताहहॴ 7-Feb-2017

Options

1) Mitochondria

माईटह८कोडडरया 2) Centrioles

सटरीयह८मस

3) Ribosomes

ररबह८सह८नस

4) Nucleus

नयनकियस

Correct Answer nucleus

eukaryotic cell -Transcription is the

process of synthesizing ribonucleic acid

(RNA)Synthesis takes place within the

nucleus of eukaryotic cells or in the

cytoplasm of prokaryotes and converts

the genetic code from a gene in

deoxyribonucleic acid ( DNA ) to a

strand of RNA that then directs

proteinsynthesis

Q222 _________is caused by parasites

of the Plasmodium genus

पिाजमह८डियमजातीकहॳ परजीवी mdash- कहॳ कारणहहॴ 7-Feb-2017

Options

1) Dysentery

पहॳचचि

2) Malaria

मिहॳररया 3) Chickenpox

F A C E B O O K

P A G E h t t p w w w f a c e b o o k c o m s s c m e n t o r s o f f i c i a l P a g e | 51

FOR MORE UPDATES AND MORE MATERIAL DO LIKE OUR FACEBOOK PAGE httpwwwfacebookcomsscmentorsofficial

चहॳचक

4) Herpes

हहॳपपयस

Correct Answer Malaria

Q223 Carotene in fruits and vegetables

gives it which color

फिह८औरसनलजयोमनसितकहॳ रह८टीनउनहकह९नसारगपरदानकरताहहॴ 7-Feb-2017

Options

1) Green

हरा 2) Pink

गिाबी 3) Orange

नारगी 4) Blue

नीिा Correct Answer Orange

Q224 Equus Caballus is the scientific

name of

एकवसकहॴ बहॳिस mdashmdashndash कापवजञाननकनामहहॴ 7-Feb-2017

Options

1) Horse

घह८िा 2) Zebra

ज़हॳिा 3) Donkey

गधा 4) Buffalo

भस

Correct Answer Horse

Q225 Elapidae Naja is the scientific name of

एिीपीिीनाजा mdashmdash- कावहॴजञाननकनामहहॴ 8-Feb-2017

Options

1) Cobra

कह८बरा 2) Elephant

हािी 3) Eagle

ग ि

4) Owl

उमि Correct Answer Cobra

Q226 Which disease is caused due to

deficiency of Iron

िह८हकीकमीकहॳ कारणकह९नसारह८गहह८ताहहॴ 8-Feb-

2017

Options

1) Beriberi

बहॳरीबहॳरी 2) Tetany

टहॳटनी 3) Kwashiorkor

कवािीऔरकर

4) Anaemia

रकतामपता Correct Answer Anaemia

Beriberi is a disease caused by a vitamin

B-1 deficiency also known as thiamine

deficiency

Tetany can be the result of an

electrolyte imbalance Most often itrsquos a

dramatically low calcium level also

known as hypocalcemia Tetany can also

be caused by magnesium deficiency or

too little potassium Having too much

acid (acidosis) or too much alkali

(alkalosis) in the body can also result in

tetany

Kwashiorkor also known as

ldquoedematous malnutrition It is a form of

malnutrition caused by a lack of protein

in the diet

Anaemia means that you have fewer red

blood cells than normal or you have less

F A C E B O O K

P A G E h t t p w w w f a c e b o o k c o m s s c m e n t o r s o f f i c i a l P a g e | 52

FOR MORE UPDATES AND MORE MATERIAL DO LIKE OUR FACEBOOK PAGE httpwwwfacebookcomsscmentorsofficial

haemoglobin than normal in each red

blood cell

Q227 is a leaf where the leaflets are

arranged along the middle vein

mdashndashएकपततीहहॴजहापतरकह८कीरचनाक ररयालिराकहॳ आसपासहह८तीहहॴ 8-Feb-2017

Options

1) Pinnately compound leaf

पपनहॳटिीसयकतपतती 2) Palmately compound leaf

पामहॳटिीसयकतपतती 3) Compound leaf

सयकतपतती 4) Simple leaf

साधारणपतती Correct Answer Pinnately compound

leaf

Q228 Haustoria or sucking roots are

found in which of the following

हह८सटह८ररयायाचसनहॳवािीजड़हॳनननननलिखितमसहॳककसमपाईजातीहहॴ 8-Feb-2017

Options

1) Wheat

गहॳह

2) Mango

आम

3) Chestnut

चहॳसटनट

4) Cuscuta

कसकयटा Correct Answer Cuscuta

Haustorial roots -The roots of parasitic

plants which penetrate into the host

tissues to absorb nourishment are

called haustorial roots hellip Also known as suckingor parasitic roots

Q229 Equs Asinus is the scientific name

of

एकवसएलसनस mdashmdashndash कावहॴजञाननकनामहहॴ 8-

Feb-2017

Options

1) Donkey

गधा 2) Cow

गाय

3) Deer

टहरन

4) Kangaroo

कगा

Correct Answer Donkey

Q230 Ficus benghalensis is the scientific name of

फाईकसबहॳनगहॳिहॳलसस mdashndash कापवजञाननकनामहहॴ 8-Feb-2017

Options

1) Banyan

बरगद

2) Pineapple

अनानास

3) Babul

बबि

4) Tulsi

तिसी Correct Answer Banyan

Q231 Equus burchellii is the scientific name of

एकवसबचिी mdashmdash- कापवजञाननकनामहहॴ 8-Feb-2017

Options

1) Horse

घह८िा 2) Zebra

जहॳिा 3) Buffalo

F A C E B O O K

P A G E h t t p w w w f a c e b o o k c o m s s c m e n t o r s o f f i c i a l P a g e | 53

FOR MORE UPDATES AND MORE MATERIAL DO LIKE OUR FACEBOOK PAGE httpwwwfacebookcomsscmentorsofficial

भस

4) Ass

गधा Correct Answer Zebra

Page 16: COMPILATION OF ALL 72 SETS OF BIOLOGY SSC CHSL-2016 · OF BIOLOGY SSC CHSL-2016 PREPARED BY : SSC MENTORS BIOLOGY SPECIAL . F A C E B O O K P A G E : h t t p : / / w w w . f a c e

F A C E B O O K

P A G E h t t p w w w f a c e b o o k c o m s s c m e n t o r s o f f i c i a l P a g e | 15

FOR MORE UPDATES AND MORE MATERIAL DO LIKE OUR FACEBOOK PAGE httpwwwfacebookcomsscmentorsofficial

Q59 Insects that transmit diseases are

known as

जह८कीड़हॳरह८गसचाररतकरतहॳहहॴ उनह mdashmdash-

कहॳ नामसहॳजानाजाताहहॴ 16-Jan-2017

1)Pathogens

रह८गज़नक

2) Vectors

वहॳकटर

3) Drones

परजीवी 4)Scalars

अटदषट

Correct Answer Vectors

A vector is an organism that does not

cause disease itself but which spreads

infection by conveying pathogens from

one host to another Species of mosquito

for example serve as vectors for the

deadly disease Malaria

Q60 Which is the second largest gland

of Human body

मानविरीरकीदसरीसबसहॳबड़ीगरिीकह९नसीहहॴ

SSC CHSL Science (biology)

2016 Question Paper

16-Jan-2017

Options

1) Liver

यकत

2) Large Intestine

बड़ीआत

3) Thorax

छाती 4) Pancreas

अगनयािय

Correct Answer Pancreas

Q61 Annona squamosa is the scientific

name of

एनह८नासकवामह८सा (Annona squamosa) mdash

mdashmdash कावहॴजञाननकनामहहॴ 16-Jan-2017

Options

1) Custard Apple

सीताफि

2) Papaya

पपीता 3) Babhul

बबि

4) Drumstick

सहजन

Correct Answer Custard Apple

Q62 The disease Beri Beri is caused due

to the deficiency of which of the

following

बहॳरीबहॳरीरह८गनननननलिखितमसहॳककसकीकमीकहॳकारणहह८ताहहॴ

16-Jan-2017

Options

1) Vitamin B2

पवटालमन B2

2) Vitamin B1

पवटालमन B1

3) Vitamin B12

पवटालमन B12

4) Vitamin E

पवटालमन E

Correct Answer Vitamin B1

Beriberi is a disease caused by a vitamin

B-1 deficiency also known as thiamine

deficiency

Q63 Chlorophyll was first isolated and

named by

किह८रह८कफिकह८ mdash-

दवारापहिहॳपिकऔरनालमतककयागया 16-Jan-2017

F A C E B O O K

P A G E h t t p w w w f a c e b o o k c o m s s c m e n t o r s o f f i c i a l P a g e | 16

FOR MORE UPDATES AND MORE MATERIAL DO LIKE OUR FACEBOOK PAGE httpwwwfacebookcomsscmentorsofficial

Options

1) Caventou

कहॳ वहॳत 2) Pelletier

पहॳिहॳटटयर

3) Chlorophyll

किह८रह८कफि

4) Caventou and Pelletier

कहॳ वहॳतऔरपहॳिहॳटटयर

Correct Answer Caventou and Pelletier

Chlorophyll was first isolated and

named by

Joseph Bienaimeacute Caventou and Pierre

Joseph Pelletier in 1817 The presence of

magnesium in chlorophyll was

discovered in 1906 and was the first

time that magnesium had been detected

in living tissue

Q64 Which of the following organisms

does not fit into the Cell Theory

नननननलिखितमसहॳकह९नसाजीवकह८लिकालसदातअन पनहीहहॴ

16-Jan-2017

Options

1) Bacteria

बहॴकटीररया 2) Virus

वायरस

3) Fungi

कवक

4) Plants

पह९धहॳ Correct Answer Virus

The bottom line is that viruses are not

alive and not related to cells in any way

The cell theory states that all living

things are made of cells cells are the

basic units of structure and function of

living things and that all cells come

from other cells Since viruses are not

made of cells and do not use cells in any

of their processes they are not related to

the cell theory

Q65 Which of these is not a

macronutrient for Plants

नननननलिखितमसहॳकह९नसापह९धह८कहॳ लिएमिह८नयटरीएटनहीहहॴ

SSC CHSL Science (biology) 2016

Question Paper

17-Jan-2017

Options

1) Nitrogen

नाइटरह८जन

2) Phosphorus

फासफह८रस

3) Potassium

पह८टालसयम

4) Chlorine

किह८रीन

Correct Answer Chlorine

In relatively large amounts the soil

supplies nitrogen phosphorus

potassium calcium magnesium and

sulfur these are often called the

macronutrients In relatively small

amounts the soil supplies iron

manganese boron molybdenum

copper zinc chlorine and cobalt the

so-called micronutrients

Q66 Name the respiratory organs of

insects

कीटह८मनसतिशरवसनअगनामकानामहहॴ

17-Jan-2017

Options

1) Skin

तवचा 2) Body Surface

िरीरकीसतह

F A C E B O O K

P A G E h t t p w w w f a c e b o o k c o m s s c m e n t o r s o f f i c i a l P a g e | 17

FOR MORE UPDATES AND MORE MATERIAL DO LIKE OUR FACEBOOK PAGE httpwwwfacebookcomsscmentorsofficial

3) Gills

गिफड़हॳ 4) Tracheae

शरावस- निी Correct Answer Tracheae

Air enters the respiratory systems of

insects through a series of external

openings called

spiracles These external openings

which act as muscular valves in some

insects lead to the internal respiratory

system a densely networked array of

tubes called tracheae

Q67 The poisonous gas accidentally

released in Bhopal Gas Tragedy is

भह८पािगहॴसतरासदीमगितीसहॳमकतहईजहरीिीगहॴसिी

17-Jan-2017

1) Methane

मीिहॳन

2) Nitrous Oxide

नाइटरसऑकसाइि

3) Methyl Isocyanate

महॴचििआयसोसायनहॳट

4) Cyanogen

सायनह८जहॳन

Correct Answer Methyl Isocyanate

Q68 What does Trypsin do

टटरनपसनकयाकरताहहॴ

SSC CHSL Science (biology) 2016

Question Paper

17-Jan-2017

Options

1) Breaks down Carbohydrates

काबोहाइडरहॳटकापवघटनकरताहहॴ 2) Synthesizes proteins

परह८टीनकासििहॳषणकरताहहॴ 3) Breaks down fats

वसाकापवघटनकरताहहॴ 4) Breaks down proteins

परह८टीनकापवघटनकरताहहॴ Correct Answer Breaks down proteins

Trypsin is one of the three principal

digestive

proteinases the other two being pepsin

and

chymotrypsin In the digestive process

trypsin acts with the other proteinases

to break down dietary protein molecules

to their component

peptides and amino acids

A protease is any enzyme that performs

proteolysis protein catabolism by

hydrolysis of peptide bonds

Q69 Name the source from which

Aspirin is produced

उससरह८तकानामबताइए

नजससहॳएनसपररनकाउतपादनककयाजाताहहॴ

17-Jan-2017

Options

1) Willow bark

पविह८कीछाि

2) Oak Tree

ओककावकष

3) Acacia

बबि

4) Eucalyptus

नीिचगरी Correct Answer Willow bark

The compound from which the active

ingredient in aspirin was first derived

salicylic acid was found in the bark of a

willow tree in 1763 by Reverend

Edmund Stone of Chipping-Norton

Q70 Cannis Familiaris is the scientific

name of

कहॴ ननसफहॳ लमलियहॳररस mdash- कावहॴजञाननकनामहहॴ

17-Jan-2017

F A C E B O O K

P A G E h t t p w w w f a c e b o o k c o m s s c m e n t o r s o f f i c i a l P a g e | 18

FOR MORE UPDATES AND MORE MATERIAL DO LIKE OUR FACEBOOK PAGE httpwwwfacebookcomsscmentorsofficial

Options

1) Cat

बबमिी 2)Dog

कतता 3) Fox

िह८मड़ी 4) Wolf

भहॳडड़या Correct Answer Dog

Q71 Harmful bacteria in potable water

make the water

पीनहॳकहॳ पानीमनसतिघातकबहॴकटीररयाउसपानीकह८बनातहॳहहॴ 17-Jan-2017

Options

1) unfit to drink

पीनहॳकहॳ लिएअयह८गय

2) smelly

दगयनधयकत

3) Colored

रगीन

4) Turbid

मटमहॴिा Correct Answer unfit to drink

Q72 Musa paradisiaca is the scientific

name of which plant

मसापहॴराडिलसयाकाककसपह९धहॳकावहॴजञाननकनामहहॴ

17-Jan-2017

Options

1) Mango

आम

2) Wheat

गहॳह

3) Corn

भ ा 4) banana

कहॳ िा Correct Answer banana

Q73 Prawns belong to which family

झीगहॳककसपररवारकहॳ हह८तहॳहहॴ 17-Jan-2017

Options

1) Crustaceans

िसटहॳलियन

2)Fish

मछिी 3) Amphibians

अननफबबयस

4) Reptiles

रहॳपटाइमस

Correct Answer Crustaceans

Q74 Name the drug that is yielded from

Cinchona tree and is used to cure

malaria

उसऔषचधकानामबताइएनजसहॳलसगकह८नापहॳड़सहॳपरापतककयाजाताहहॴऔरनजसकाउपयह८गमिहॳररयाकहॳ उपचारमककयाजाताहहॴ 17-Jan-2017

Options

1) Camptothea

कहॴ नटह८चिया 2) Acuminata

एकयलमनहॳटा 3) Quinine

कनहॴन

4) Cinchonia

लसकह८ननया Correct Answer Quinine

Q75 Blood Circulation was discovered

by

रकतपररसचरणकी mdashmdashndash दवारािह८जकीिी 17-Jan-2017

Options

1) Mary Anderson

F A C E B O O K

P A G E h t t p w w w f a c e b o o k c o m s s c m e n t o r s o f f i c i a l P a g e | 19

FOR MORE UPDATES AND MORE MATERIAL DO LIKE OUR FACEBOOK PAGE httpwwwfacebookcomsscmentorsofficial

महॴरीएिरसन

2) Virginia Apgar

वनजयननयाएपगार

3) William Harvey

पवलियमहाव

4) Robert Feulgen

रॉबटयफ़यिजहॳन Correct Answer William Harvey

Q76 Vitamin A is also known as

पवटालमन A कह८ mdashmdash- कहॳ नामसहॳभीजानाजाताहहॴ SSC CHSL Science (biology) 2016

Question Paper

18Jan2017

Options

1) Thiamine

िायलमन

2) Riboflavin

ररबह८फिहॳपवन

3) Retinol

रहॳटटनॉि

4) Calciferol

कहॴ नमसफहॳ रह८ि

Correct Answer Retinol

Q77 Some roots called arise from an

organ other than the radicle

कछजड़हॳनजनह mdashmdashmdash कहाजाताहहॴ वहमिकहॳ अिावाककसीअनयअगसहॳउतपननहह८तीहहॴ 18Jan2017

Options

1) tap roots

मखयजड़

2) stilt roots

ि ाजड़

3) fibrous roots

रहॳिहॳदारजड़

4) adventitious roots

आकनसमकजड़

Correct Answer adventitious roots

Q78 Spiders belong to which class of

animals

मकडड़यापराणीवगीकरणकहॳ ककसवगयमआतीहहॴ 18Jan2017

Options

1) Arachnids

एरहॳकननडस

2) Aves

एपवस

3) Gastropods

गहॴसटरोपह८िस

4) Anthozoa

एिह८जआ

Correct Answer Arachnids

Q79 How many layers does Human

Skin have

मानवतवचामककतनीपरतहॳहह८तीहहॴ

18Jan2017

Options

1) 5

2) 7

3) 11

4) 3

Correct Answer 3

Skin has three layers The epidermis

the outermost layer of skin provides a

waterproof barrier and creates our skin

tone The dermis beneath the

epidermis contains tough connective

tissue hair follicles and sweat glands

The deeper subcutaneous tissue (

hypodermis ) is made of fat and

connective tissue

Q80 Allium Cepa is the scientific name

of

एलियमलसपपा mdashmdashndash कावहॴजञाननकनामहहॴ 18Jan2017

F A C E B O O K

P A G E h t t p w w w f a c e b o o k c o m s s c m e n t o r s o f f i c i a l P a g e | 20

FOR MORE UPDATES AND MORE MATERIAL DO LIKE OUR FACEBOOK PAGE httpwwwfacebookcomsscmentorsofficial

Options

1) Carrot

गाजर

2) Tomato

टमाटर

3) Potato

आि 4) Onion

पयाज़

Correct Answer Onion

Q81 DNA stands for

िीएनएकापणय प mdashmdash- हहॴ 18Jan2017

Options

1) Di Nucleic Acid

िाईनयनकिकएलसि

2) Deoxy Nucleic Acid

िीओकसीनयनकिकएलसि

3) Diribonucleic Acid

िाईराइबह८नयनकिकएलसि

4) Deoxyribonucleic Acid

िीऑकसीराइबह८नयनकिकएलसि

Correct Answer Deoxyribonucleic Acid

Q82 Organisms that generate energy

using light are known as

जह८जीवाणपरकािकाउपयह८गकरउजायउतपननकरतीहहॴ उनह mdashmdash कहॳ पमजानाजाताहहॴ

18Jan2017

Options

1) Chaemolithotrophs

ककमह८लििह८टरह८पस

2) Oligotrophs

ओलिगह८टरह८पस

3) Bacteria

बहॴकटीररया 4)Photoautotrophs

फह८टह८ओटह८टरह८पस

Correct Answer Photoautotrophs

An oligotroph is an organism that can

live in an environment that offers very

low levels of nutrients

Q83 Which drug is used as an

Antidepressant

ककसदवाएकहतािारह८धीकहॳ पमपयोगककयाजाताहहॴ Options

1) Oxybutynin

ओकसीलयटीनन

2)Tramadol

टरहॳमहॳिह८ि

3 ) Sumatriptan

समहॳटरीपटहॳन

4) Bupropion

लयपरह८पपयह८न

Correct Answer Bupropion

लयपरह८पपयह८न

Q84 The orange colour of carrot is

because of

गाजरकानारगीरगनननननलिखितमसहॳककसीएककीवजहसहॳहह८ताहहॴ 18Jan2017

Options

1) it grows in the soil

यहलम ीमउगतीहहॴ 2) Carotene

कहॴ रह८टीन

3) it is not exposed to sunlight

यहसययपरकािकहॳ सपकय मनहीआती 4) the entire plant is oranqe in colour

सनपणयपह९धानारगीरगकाहह८ताहहॴ Correct Answer Carotene

Q85 Snake venom is highly modified

saliva containing

F A C E B O O K

P A G E h t t p w w w f a c e b o o k c o m s s c m e n t o r s o f f i c i a l P a g e | 21

FOR MORE UPDATES AND MORE MATERIAL DO LIKE OUR FACEBOOK PAGE httpwwwfacebookcomsscmentorsofficial

सापकाजहरअततयाचधकसिह८चधतिारहह८तीहहॴनजसमहॳ mdashmdash- हह८ताहहॴ Options

l)Prototoxins

परह८टह८टॉनकसस

2)Neutrotoxins

नयटरोटॉनकसस

3)Zootoxins

जटॉनकसस

4)Electrotoxins

इिहॳकटरह८टॉनकसस

Correct Answer Zootoxins

जटॉनकसस

Q86 Which type of pathogen causes the

water-borne disease Schistosomiasis

ककसपरकारकारह८गज़नकजिजननतरह८गलससटह८सह८लमलससकाकारणबनताहहॴ

18Jan2017

Option

1) Parasitic

परजीवी 2)Protozoan

परह८टह८जआ

3) Bacterial

बहॴकटीररयि

4) Viral

वायरि

Correct Answer Parasitic

Schistosomiasis also known as snail

fever and bilharzia is a disease caused

by parasitic

flatworms called schistosomes

Q87 Prothrombin responsible for

clotting of blood is released by

परह८िह८ननबन

जह८रकतकािककाजमनहॳकहॳ लिएनजनमहॳदारहहॴ mdashndash

कहॳ दवारासतरापवतककयाजाताहहॴ

19Jan2017

Options

1) Small Intestine

छह८टीआत

2) Blood Platelets

रकतपिहॳटिहॳटस

3) Large Intestine

बड़ीआत

4Heart

हदय

Correct Answer Blood Platelets

Q88 Acacia arabica is the scientific

name of

अकहॳ लियाअरहॳबबका mdashmdashndash कावहॴजञाननकनामहहॴ 19-Jan-2017

Options

1) Neem

नीम

2) Teak

सागह९न

3) Babhul

बबि

4) Pomegranate

अनार

Correct Answer Babhul

Q89 Cannis Vulpes is the scientific

name of

कहॴ ननसवनमपस mdashmdash- कावहॴजञाननकनामहहॴ 19-Jan-2017

Options

1) Dog

कतता 2) Wolf

भहॳडड़या 3) Fox

िह८मड़ी 4) Hyena

िाकिबगघा

F A C E B O O K

P A G E h t t p w w w f a c e b o o k c o m s s c m e n t o r s o f f i c i a l P a g e | 22

FOR MORE UPDATES AND MORE MATERIAL DO LIKE OUR FACEBOOK PAGE httpwwwfacebookcomsscmentorsofficial

Correct Answer Fox

Q90 The beetroot is the portion of the

beet plant

चकदरपह९धहॳका mdashmdashndash भागहहॴ 19-Jan-2017

Options

1) tap root

मखयजड़

2) Adventitious

आकनसमक

3) bulb of the stem

तनहॳकाकद

4) Rhizome

परकद

Correct Answer tap root

Q91 What is the basic unit of heredity

आनवलिकताकीबननयादीइकाईकयाहहॴ 19-Jan-2017

Options

1) DNA

िीएनए

2) RNA

आरएनए

3) Chromosome

िह८मह८सह८म

4) Gene

जीन

Correct Answer gene

Genes are the units of heredity and are

the instructions that make up the bodyrsquos

blueprint They code for the proteins

that determine virtually all of a personrsquos

characteristics Most genes come in

pairs and are made of strands of genetic

material called deoxyribonucleic acid

or DNA

Q92 Lungs are the primary organs of

फहॳ फड़हॳmdashndashकहॳ परािलमकअगहहॴ

19-Jan-2017

Options

1) Digestion

पाचन

2) Constipation

कलज

3) Perspiration

पसीना 4)Respiration

शवसन

Correct Answer Respiration

Q93 Sugarcane is a type of

गननाएकपरकारका mdash- हहॴ 20-Jan-2017

Options

1)creeper

िता 2)tree

पहॳड़

3)shrub

झाड़ी 4)grass

घास

Correct Answer grass

Q94 Who is commonly known as ldquothe

Father of Microbiologyrdquo

सामानयत ldquo सकषमजीवपवजञानकहॳ जनक lsquo

कहॳ नामसहॳककसहॳजानाजातहहॴ 20-Jan-2017

Options

1) Robert Hooke

रॉबटयहक

2) Antonie Philips van Leeuwenhoek

एटह८नीकफलिपवानमयएनहह८क

3) Carl Linnaeus

काियिीनाईयस

4) Charles Darwin

चामसयिापवयन

F A C E B O O K

P A G E h t t p w w w f a c e b o o k c o m s s c m e n t o r s o f f i c i a l P a g e | 23

FOR MORE UPDATES AND MORE MATERIAL DO LIKE OUR FACEBOOK PAGE httpwwwfacebookcomsscmentorsofficial

Correct Answer Antonie Philips van

Leeuwenhoek

Q95 For the aquatic organisms the

source of food is

जिीयजीवाणकािाघसरह८तहहॴ 20-Jan-2017

Options

1) Phytoplankton

फायटह८पिहॳकटन

2) Sea Weed

समदरीिहॴवाि

3)Aqua plankton

एकवापिहॳकटन

4) Zooplankton

जपिहॳकटन

Correct Answer Phytoplankton

Q96 Haemoglobin has the highest

affinity with which of the following

हीमह८गिह८बबनकीननननमसहॳककसकहॳ सािउततमसमानताहहॴ

20-Jan-2017

Options

1)SO2

2)CO2

3)CO

4)NO2

Correct Answer CO

It has a greater affinity for hemoglobin

than oxygen does It displaces oxygen

and quickly binds so very little oxygen

is transported through the body cells

Q97 Who developed the theory of

Evolution

उदपवकासकालसदातककसनहॳपवकलसतककया

20-Jan-2017

Options

1) Charles Darwin

चामसयिापवयन

2) Isaac Newton

आयजहॳकनयटन

3) Pranav Mistry

परणवलमसतरी 4) Galileo Galilei

गहॳलिलियह८गहॳिीिी Correct Answer Charles Darwin

Q98 The primary function of RNA is

RNA कापरािलमककाययहह८ताहहॴ 20-Jan-2017

Options

1) Photosynthesis

परकािसशिहॳषण

2) Protein Synthesis

परह८टीनसशिहॳषण

3) Replication

परनतकनतबनाना 4) Translation

अनवादकरना Correct Answer Protein Synthesis

There are two main functions of RNA

It assists DNA by serving as a messenger

to relay the proper genetic information

to countless numbers of ribosomes in

your body The other main function of

RNA is to select the correct amino acid

needed by each ribosome to build new

proteins for your body

Q99 ______is the movement of

molecules across a cell membrane from

a region of their lower concentration to

a region of their higher concertration

उचचसादरताकहॳ कषहॳतरसहॳउसकीकमसादरतावािहॳकषहॳतरकीतरफएककह८लिकाखझमिीकहॳ माधयमसहॳहह८नहॳवािाअणओकहॳ सचिनकह८ mdash- कहतहॳहहॴ Options

1) Diffusion

पवसरण

2) Osmosis

ऑसमह८लसस

F A C E B O O K

P A G E h t t p w w w f a c e b o o k c o m s s c m e n t o r s o f f i c i a l P a g e | 24

FOR MORE UPDATES AND MORE MATERIAL DO LIKE OUR FACEBOOK PAGE httpwwwfacebookcomsscmentorsofficial

3) Active Transport

सकियआवागमन

4) Passive Transport

नननषियआवागमन

Correct Answer Active Transport

Q100 Study of classification of

organisms is known as 20-Jan-2017

जीवाणओकहॳ वगीकरणकहॳ अधययनकह८ mdash-

कहाजाताहहॴ Options

1) Serpentology

सपरहॳटह८िह८जी 2) Virology

वायरह८िह८जी 3) Taxonomy

टहॴकसोनह८मी 4) Physiology

कफनज़यह८िह८जी Correct Answer Taxonomy

Q101 Photosynthesis takes place inside

plant cells in

परकािसशिहॳषणवनसपनतकह८लिकामनसति mdash

mdashmdash महह८ताहहॴ 20-Jan-2017

Options

1) Ribosomes

राइबह८सह८नस

2) Chloroplasts

किह८रह८पिासट

3) Nucleus

नयकलियम

4) Mitochondria

माईटह८कोडडरया Correct Answer Chloroplasts

Q102 ______ is the cell organelle in

which the biochemical processes of

respiration and energy production

occur

mdashmdash- वहकह८लिकाअगहहॴ नजसमहॳशवसनऔरउजायउतपादनकहॳ जहॴसीजहॴवरासायननकपरकियायहह८तीहहॴ 20-Jan-2017

Options

1) Mitochondria

माइटह८कोडडरया 2) Chloroplast

किह८रह८पिासट

3) Ribosomes

राइबह८सह८नस

4) Nucleus

नयकिीयस

Correct Answer Mitochondria

Q103 Which non-flowering spore

bearing plants have roots

ककसफिनिगनहॳवािहॳऔरबीजाणधारकपह९धह८कीजड़हॳहह८तीहहॴ 21-Jan-2017

Options

1) Mosses

मह८सहॳस

2) Angiosperms

एननजयह८सपनसय 3) Ferns

फनसय 4) Gymnosperms

नजननह८सपनसय Correct Answer ferns

Q104 Which of the following is an

excretory organ of cockroach

नननननलिखितमसहॳकह९नसानतिच हॳकाउतसजयनअगहहॴ

21-Jan-2017

Options

F A C E B O O K

P A G E h t t p w w w f a c e b o o k c o m s s c m e n t o r s o f f i c i a l P a g e | 25

FOR MORE UPDATES AND MORE MATERIAL DO LIKE OUR FACEBOOK PAGE httpwwwfacebookcomsscmentorsofficial

1) Malphigian Tubules

मनमफनजयनटयबमस

2) Nephridia

नहॳकफरडिया 3) Coxal Gland

कह८कसिगरचिया 4) Green Gland

गरीनगरचिया Correct Answer Malphigian Tubules

Q105 Evaporation of water takes place

in which part of plants

पानीकहॳ वाषपीकरणकीकियापह९धोकहॳ ककसभागसहॳहह८तीहहॴ 21-Jan-2017

Options

1) Stem

तना 2) Stomata

सटह८मटा 3) Branch

िािाए

4) Fruit

फि

Correct Answer Stomata

Evaporation accounts for the movement

of water to the air from sources such as

the soil canopy interception and

waterbodies Transpiration accounts for

the movement of water within a plant

and the subsequent loss of water as

vapour through stomata in its leaves

Q106 A is the fleshy spore-bearing

fruiting body of a fungus

mdashmdashndashकवककामासि

बीजाणधारणकरनहॳवािाफिनहॳवािाअगहहॴ 21-

Jan-2017

Options

1) aloe vera

एिह८वहॳरा 2) Coral

मगा 3) Cactus

कहॴ कटस

4) Mushroom

ककरमतता Correct Answer mushroom

Q107 Which of the following is a fungal

disease

नननननलिखितमसहॳकह९नसाफफदसहॳहह८नहॳवािाएकरह८ग हहॴ

21-Jan-2017

Options

1) Dermatitis

तवचािह८ध

2) Cholera

हहॴजा 3) Jaundice

पीलिया 4) Indigofera

इननिगह८फहॳ रा Correct Answer Dermatitis

Dermatitis also known as eczema is a

group of diseases that results in

inflammation of the skin These diseases

are characterized by itchiness red skin

and a rash In cases of short duration

there may be small blisters while in

long-term cases the skin may become

thickened

Q108 In which form is glucose stored in

our body

हमारहॳिरीरमगिकह८जकासचयककस पमककयाजाताहहॴ

21-Jan-2017

Options

1) Insulin

F A C E B O O K

P A G E h t t p w w w f a c e b o o k c o m s s c m e n t o r s o f f i c i a l P a g e | 26

FOR MORE UPDATES AND MORE MATERIAL DO LIKE OUR FACEBOOK PAGE httpwwwfacebookcomsscmentorsofficial

इसलिन

2) Glucose

गिकह८ज

3) Glycogen

गिायकह८जहॳन

4) Fat

वसा Correct Answer Glycogen

Excess glucose is stored in the liver as

the large compound called glycogen

Glycogen is a polysaccharide of glucose

but its structure allows it to pack

compactly so more of it can be stored in

cells for later use

Q109 Where do plants synthesize

protein from

पह९धहॳपरह८टीनसशिहॳषणकहासहॳकरतहॳहहॴ

Options

1) Fatty Acids

वसाऐलसि

2) Sugar

िकर

3) Amino Acids

एलमनह८ऐलसि

4) Starch

सटाचय Correct Answer Amino Acids

Q110 Which part of the brain is

responsible for triggering actions like

thinking intelligence memory and

ability to learn

मनसतषककाकह९नसाटहससासह८चनहॳ बनधदमानी याददाशतऔरसीिनहॳकीकषमताजहॴसीकियाओकह८परहॳररतकरताहहॴ 21-Jan-2017

Options

1) Diencephalon

िायएनसहॳफहॳ िह८न

2) Hypothalamus

हयपह८िहॳिहॳमस

3) Cerebrum

सहॳरहॳिम

4) Control

कटरह८ि

Correct Answer Cerebrum

Q111 Which of the following is also

known as the Biochemical Laboratory

of the Human Body

नननननलिखितमसहॳककसहॳमानविरीरकीजहॴवरसायनपरयह८गिािाभीकहाजाताहहॴ 21-Jan-2017

Options

1) Small Intestine

छह८टीआत

2)Brain

मनसतषक

3) Pancreas

अगनयािय

4) Liver

नजगर

Correct Answer Liver

The liver makes bile that will help

emulsify and digest the fats we eat

The liver takes toxic substances and

convert them using enzymes the liver

cells makes into a non toxic form so the

body can dispose of them

The liver also converts fats protein and

carbohydrates into glucose which is the

energy source for our cells to use

The liver takes amino acids and makes

proteins by combining them

Q112 The yellow colour of human urine

is due to

मानवमतरकापीिारग mdashndash कीवजहसहॳहह८ताहहॴ 22-

Jan-2017

Options

1) Bile Salts

F A C E B O O K

P A G E h t t p w w w f a c e b o o k c o m s s c m e n t o r s o f f i c i a l P a g e | 27

FOR MORE UPDATES AND MORE MATERIAL DO LIKE OUR FACEBOOK PAGE httpwwwfacebookcomsscmentorsofficial

पपततनमक

2) Cholesterol

कह८िहॳसटरह८ि

3) Lymph

लिनफ

4) Urochrome

यरह८िह८म

Correct Answer Urochrome

Urobilin or urochrome is the chemical

primarily responsible for the yellow

color of urine

Q113 The wilting of plants takes place

due to

पह९धह८कालिचििहह८नाकी mdashmdash- कीवजहसहॳहह८ताहहॴ 22-Jan-2017

Options

1)Photosynthesis

परकािसशिहॳषण

2) Transpiration

वाषपह८तसजयन

3) Absorption

अविह८षण

4) Respiration

शरवसन

Correct Answer Transpiration

Wilting is the loss of rigidity of non-

woody parts of plants This occurs when

the turgor pressure in non-lignified

plant cells falls towards zero as a result

of diminished water in the cells

Q114 Bovidae Ovis is the scientific name of

बह८पविीओपवस mdashndash कावहॴजञाननकनामहहॴ 22-Jan-2017

Options

1) Goat

बकरी 2) Cow

गाय

3) Buffalo

भहॳस

4) Sheep

भहॳड़

Correct Answer Sheep

Q115 Plants get their energy to produce

food from which of the following

पह८धहॳभह८जनकाननमायणकरनहॳकहॳ लिएनननननलिखितमसहॳककससहॳउजायपरापतकरतहॳहहॴ

22-Jan-2017

Options

1) Photosynthesis

परकािसशिहॳषण

2)Bacteria

बहॴकटीररया 3)Fungi

कवक

4)Sun

सयय Correct Answer Sun

Q116 Which of the following is secreted

by the liver

नननननलिखितमसहॳककसकासरावनजगरसहॳहह८ताहहॴ

22-Jan-2017

Options

1) Glucose

गिकह८ज

2) Iodine

आयह८िीन

3) Cortisol

काटटरयसह८ि

4) Bile

पपतत

Correct Answer Bile

The liver makes bile that will help

emulsify and

digest the fats we eat

F A C E B O O K

P A G E h t t p w w w f a c e b o o k c o m s s c m e n t o r s o f f i c i a l P a g e | 28

FOR MORE UPDATES AND MORE MATERIAL DO LIKE OUR FACEBOOK PAGE httpwwwfacebookcomsscmentorsofficial

Q117 Ferns belong to which division of

plants

फनसयपह९धह८कहॳ ककसभागमआतहॳहहॴ

22-Jan-2017

Options

1) Gymnosperms

नजननह८सपनसय 2) Angiosperms

एनजयह८सपनसय 3) Thallophyta

िहॴिह८फाईटा 4)Pteridophyta

टहॳररिह८फाईटा Correct Answer Pteridophyta

Q118 Who invented Antibiotics

एटीबायह८टटककाअपवषकारककसनहॳककयािा

22-Jan-2017

Options

1) Joseph Lister

जह८सहॳफलिसटर

2) William Harvey

पवलियमहाव

3) Robert Knock

रॉबटयनॉक

4)Alexander Fleming

अिहॳकज़िरफिहॳलमग

Correct Answer Alexander Fleming

Q119 Milbecycin is used in the

eradication of

लममबहॳसायलसनका mdashndash

मउनमिनमपरयह८गककयाजाताहहॴ 22-Jan-2017

Options

1) Agricultural Fungus

कपषकवक

2) Agricultural Pests

कपषकीटक

3) Agricultural Herbs

कपषिाक

4)Agricultural Weeds

कपषननराना Correct Answer Agricultural Pests

Milbemycin oxime is a veterinary drug

from the group of milbemycins used as

a broad spectrum antiparasitic It is

active against worms and mites(insects

Q120 Intestinal bacteria synthesizes

which of the following in the human

body

मानविरीरमआतोकहॳ बहॴकटीररयानननननलिखितमसहॳककसकासशिहॳषणकरतहॳहहॴ 22-Jan-2017

Options

1) Vitamin K

पवटालमन K

2) Proteins

परह८टीन

3) Fats

वसा 4) Vitamin D

पवटालमन D

Correct Answer Vitamin K

Q121 is the study of the physical form

and external structure of plants

mdashmdash-

मपह९धह८काभहॴनतक पऔरबाहरीसरचनाकाआदयाककयाजाताहहॴ 22-Jan-2017

Options

1) Physiology

कफनजयह८िह८जी 2) Anatomy

िरीररचनापवजञान

3) Phytomorphology

फाईटह८मह८फह८िह८जी 4)Cytology

कह८लिकापवजञान

Correct Answer Phytomorphology

F A C E B O O K

P A G E h t t p w w w f a c e b o o k c o m s s c m e n t o r s o f f i c i a l P a g e | 29

FOR MORE UPDATES AND MORE MATERIAL DO LIKE OUR FACEBOOK PAGE httpwwwfacebookcomsscmentorsofficial

Q122 Which of the following is a

structural and functional unit of

kidneys

नननननलिखितमसहॳकह९नसीगदोकीसरचनातमकऔरकाययकरीईकाईहहॴ

22-Jan-2017

Options

1) Renette Cells

रहॳनहॳटकह८लिकाए

2) Flame Cells

फिहॳमकह८लिकाए

3) Nephrites

नहॳफ़राइटस

4)Nephrons

नहॳफरोस

Correct Answer Nephrons

Nephron functional unit of the kidney

the structure that actually produces

urine in the process of removing waste

and excess substances from the blood

There are about 1000000 nephrons in

each human kidney

Q123 Which of the following is the

largest part of the human brain

नननननलिखितमसहॳकह९नसामानवमनसतषककासबसहॳबड़ाटहससाहहॴ

23-Jan-2017

Options

1) Ribs

पसलियाा 2) Cerebrum

सहॳरहॳिम

3) Pons

पोस

4)Thalamus

िहॴिहॳमस

Correct Answer Cerebrum

The cerebrum is the largest part of the

human brain making up about two-

thirds of the brainrsquos mass It has two

hemispheres each of which has four

lobes frontal parietal temporal and

occipital

Q124 The auxiliary buds

सहायककालियाmdashndash 23-Jan-2017

Options

1) grow endogenously from the pericycle

पहॳरीसाईककिसहॳअनतजातयपवकलसतहह८ताहहॴ 2) arise endogenously from the main

growing point

मिवपदसहॳअनतजातयउठताहहॴ 3) is an embryonic shoot located in the

axil of a leaf

एकभरणिटहहॴजह८एकपततीकहॳ अकषपरनसतिहह८ताहहॴ 4)arise exogenously from the epidermis

एपपिलमयससहॳबटहजातयतरीकहॳ सहॳउठताहहॴ Correct Answer is an embryonic shoot

located in the axil of a leaf

Q125 Which of the following is a viral

disease

इनमहॳसहॳकह९सीएकवायरिबीमारीहहॴ

23-Jan-2017

Options

1) Polio

पह८लियह८ 2) Tetanus

धनसतनभ

3) Leprosy

कषठरह८ग

4) Plague

पिहॳग

Correct Answer Polio

A viral disease (or viral infection)

occurs when an organismrsquos body is

invaded by pathogenic viruses and

infectious virus particles (virions) attach

to and enter susceptible cells

F A C E B O O K

P A G E h t t p w w w f a c e b o o k c o m s s c m e n t o r s o f f i c i a l P a g e | 30

FOR MORE UPDATES AND MORE MATERIAL DO LIKE OUR FACEBOOK PAGE httpwwwfacebookcomsscmentorsofficial

Poliomyelitis often called polio or

infantile paralysis is an infectious

disease caused by the poliovirus

Tetanusmdash A serious bacterial infection

that causes painful muscle spasms and

can lead to death

Leprosy also known as Hansenrsquos

disease (HD) is a long-term infection by

the bacterium Mycobacterium leprae or

Mycobacterium lepromatosis

Plague is an infectious disease caused by

the bacterium Yersinia pestis

Symptoms include fever weakness and

headache

Q126 Which organisms can help to

carry out Vermicomposting

कह९नसाजीववमीकनपह८नसटगममददकरताहहॴ

23-Jan-2017

Options

1) Nitrifying Bacteria

नाईटरीफाईगबहॴकटीररया 2) Earthworms

कहॴ चऐ

3) Algae

िहॴवि

4) Fungus

कवक

Correct Answer Earthworms

Q127 Contraction of heart is also

known as

हदयकहॳ सकचनकह८ mdash- भीकहाजाताहहॴ 23-Jan-

2017

Options

1) Systole

लससटह८ि

2) Aristotle

अरसत

3) Diastole

िायसटह८ि

4) Lub

मयब

Correct Answer Systole

Diastole is the part of the cardiac cycle

when the heart refills with blood

following systole (contraction)

Ventricular diastole is the period during

which the ventricles are filling and

relaxing while atrial diastole is the

period during which the atria are

relaxing

Q128 Azadirachta indica is the

botanical name of which of the

following

अजाटदराचताइडिकानननननलिखितमसहॳककसकावानसपनतनामहहॴ

23-Jan-2017

Options

1) Rose plant

गिाबकापह९धा 2) Apple tree

सहॳबकापहॳड़

3) Neem

नीम

4)Mango

आम

Correct Answer Neem

Q129 Which of the following is the

main end product of carbohydrate

digestion

नननननलिखितमसहॳकह९नसाकाबोहाइडरहॳटकहॳ पाचनकापरमिअतउतपादकहह८ताहहॴ 23-Jan-2017

Options

1) Fats

वसा 2) Lipids

लिपपडस

3) Glucose

गिकह८ज

4) Cellulose

F A C E B O O K

P A G E h t t p w w w f a c e b o o k c o m s s c m e n t o r s o f f i c i a l P a g e | 31

FOR MORE UPDATES AND MORE MATERIAL DO LIKE OUR FACEBOOK PAGE httpwwwfacebookcomsscmentorsofficial

सहॳमयिह८ज

Correct Answer Glucose

Intestinal absorption of end products

from digestion of carbohydrates and

proteins in the pig hellip During absorption some sugars (fructose or

galactose) released from the

corresponding sucrose and lactose

respectively during digestion were

partly metabolized into glucose by the

enterocyte

Q130 Which of the following glands is a

source of the enzyme Ptyalin

नननननलिखितगरचियोमसहॳएजाइमटयालिनकासरह८तहहॴ 23-Jan-2017

Options

1) Pancreas

अगरािय

2) Thyroid Gland

िाइराइिगरिी 3) Pituitary Gland

पीयषगरिी 4) Salivary Glands

िारगरचियाा Correct Answer Salivary Glands

Q131 Which of the following is not true

about Pteridophyta

ननननमसहॳकह९नसीबातटहॳररिह८फाईटकहॳ बारहॳमसचनहीहहॴ 23-Jan-2017

Options

1) Dominant phase is saprophytes

परमिचरणसहॳपरह८फाईइटसहह८ताहहॴ 2) Main plant body is diploid

पह९दह८कामखयिरीरदपवगखणतहह८ताहहॴ 3) Seeds are present

बीजमह९जदहह८तहॳहहॴ 4)Flowers are absent

फिअनपनसतिहह८तहॳहहॴ

Correct Answer Seeds are present

Q132 The largest dolphin species is the

orca also called as

िॉिकफनकीसबसहॳबड़ीपरजानतकाकानामआकायहहॴनजसहॳ mdash- भीकहतहॳहहॴ 23-Jan-2017

Options

1) Bottle Nose

बाटिनह८ज

2) Baiji

बहॳजी 3) Killer whale

ककिरहहॳि

4)Tucuxi

टकवसी Correct Answer Killer whale

Q133 The fat digesting enzyme Lipase

is secreted by which of the following

वसाकापाचनकरनहॳवािाएजाइमिाइपहॳजनननननलिखितमसहॳककसकहॳ दवारासतरापवतहह८ताहहॴ

24-Jan-2017

Options

1) Kidneys

गद

2) Pancreas

अगनयािय

3) Large Intestine

बड़ीआत

4)Liver

नजगर

Correct Answer Pancreas

Lipase is an enzyme that splits fats so

the intestines can absorb them Lipase

hydrolyzes fats like triglycerides into

their component fatty acid and glycerol

molecules It is found in the blood

gastric juices pancreatic secretions

intestinal juices and adipose tissues

F A C E B O O K

P A G E h t t p w w w f a c e b o o k c o m s s c m e n t o r s o f f i c i a l P a g e | 32

FOR MORE UPDATES AND MORE MATERIAL DO LIKE OUR FACEBOOK PAGE httpwwwfacebookcomsscmentorsofficial

Q134 The arrangement of leaves on an

axis or stem is called

एकअकषयातनहॳपरपनततयोकीयवसिाकह८कयाकहाजाताहहॴ SSC CHSL Science (biology) 2016

Question Paper

24-Jan-2017

Options

1) Phyllotaxy

फाइिह८टहॴकसी 2) Vernation

वनिन

3) Venation

वहॳनहॳिन

4)Phytotaxy

फाइटह८टहॴकसी Correct Answer Phyllotaxy

In botany phyllotaxis or phyllotaxy is

the arrangement of leaves on a plant

stem (from Ancient Greek phyacutellon

ldquoleafrdquo and taacutexis ldquoarrangementrdquo)

Phyllotactic spirals form a distinctive

class of patterns in nature

Q135 The study of Cells is also known

as

कह८लिकाओकहॳ अधययनकह८ mdashmdashndash

भीकहाजाताहहॴ 24-Jan-2017

Options

1) Cytology

सायटह८िह८जी 2) Physiology

कफनजयह८िह८जी 3) Nucleology

नयककमयह८िह८जी 4)Cellology

सहॳिह८िह८जी Correct Answer Cytology

Q136 Which of the following scientists

is also known as the Father of Biology

नननननलिखितमसहॳककसवहॴजञाननककह८ ldquoजीवपवजञानकहॳ जनकrdquoकहॳ नामसहॳभीजानाजाताहहॴ 24-Jan-2017

Options

1) Herbert Spencer

हबयटयसपसर

2) Aristotle

अरसत 3) Lamarck

िहॳमाकय 4)Darwin

िापवयन

Correct Answer Aristotle

Q137 Which cells give rise to various

organs of the plant and keep the plant

growing

कह९नसीकह८लिकाएपह९धह८कहॳ लभननअगह८कह८जनमदहॳतीहहॴऔरपह९धह८कह८बढ़नहॳममददकरतीहहॴ

24-Jan-2017

Options

1) Permanent

सिायी 2) Dermal

तवचीय

3) Meristematic

मररसटहॳमटटक

4)Mature

परह८ढ़

Correct Answer Meristematic

A meristem is the tissue in most plants

containing undifferentiated cells

(meristematic cells) found in zones of

the plant where growth can take place

Q138 Rodentia Muridae is the scientific

name of

F A C E B O O K

P A G E h t t p w w w f a c e b o o k c o m s s c m e n t o r s o f f i c i a l P a g e | 33

FOR MORE UPDATES AND MORE MATERIAL DO LIKE OUR FACEBOOK PAGE httpwwwfacebookcomsscmentorsofficial

रह८िहॳलियानयररिी mdashmdash- कावहॴजञाननकनामहहॴ 24-

Jan-2017

Options

1) Mouse

चहा 2) Squirrel

चगिहरी 3) Monkey

बदर

4) Lizard

नछपकिी Correct Answer Mouse

Q139 Name the scientist who proposed

the cell theory

कह८लिकालसदातकापरसतावदहॳनहॳवािहॳवहॴजञाननककानामबताइए 24-Jan-2017

Options

1) Schleiden and Schwann

िीमिनऔरशरववान

2) Lamarck

िहॳमाकय 3) Treviranus

टरहॳवायरहॳनस

4)Whittaker and Stanley

हीटकरऔरसटहॳनिहॳ Correct Answer Schleiden and

Schwann

Q140 The flower with the worldrsquos

largest bloom is

दननयाकासबसहॳबड़ाफिखििनहॳवािा mdashmdashndash हहॴ 24-Jan-2017

Options

1) Pando

पािह८ 2) Posidonia

पह८सीिह८ननया 3) Rafflesia arnoldii

ररफिहॳलियाअनोमिी 4)Helianthus annuus

हहॳलिएनिसएनयअस

Correct Answer Rafflesia arnoldii

Rafflesia arnoldii is a species of

flowering plant in the parasitic genus

Rafflesia It is noted for producing the

largest individual flower on earth It has

a very strong and horrible odour of

decaying flesh earning it the nickname

ldquocorpse flower

Q141 Deficiency of which vitamin

causes night blindness

ककसपवटालमनकीकमीकहॳ कारणरतौधीहह८ताहहॴ 24-Jan-2017

Options

1) Vitamin K

पवटालमन K

2) Vitamin C

पवटालमन C

3) Vitamin B1

पवटालमन B1

4)Vitamin A

पवटालमन A

Correct Answer Vitamin A

Q142 Nongreen plants lack which of the

following

गहॴर-

हररतवनसपनतमनननननलिखितमसहॳककसकीकमीहह८तीहहॴ

24-Jan-2017

Options

1) Chlorophyll

किह८रह८कफि

2) Lycophyll

िायकह८कफि

3) Cyanophyll

F A C E B O O K

P A G E h t t p w w w f a c e b o o k c o m s s c m e n t o r s o f f i c i a l P a g e | 34

FOR MORE UPDATES AND MORE MATERIAL DO LIKE OUR FACEBOOK PAGE httpwwwfacebookcomsscmentorsofficial

सायनह८कफि

4)Phototropism

फह८टह८टरोपपजम

Correct Answer Chlorophyll

Q143 Organisms that use light to

prepare food are known as

जह८जीवपरकािकाउपयह८गकरभह८जनतहॴयारकरतहॳहहॴ उनह mdashmdash- कहॳ पमजानजाताहहॴ 24-Jan-2017

Options

1) Autotrophs

सवपह८षी 2) Heterotrophs

पवषमपह८षज

3) Omnivores

सवायहारी 4)Decomposers

पवघटनकरनहॳवािा Correct Answer Autotrophs

autotrophs often make their own food

by using sunlight carbon dioxide and

water to form sugars which they can use

for energy Some examples of

autotrophs include plants algae and

even some bacteria Autotrophs

(producer) are important because they

are a food source for heterotrophs

(consumers)

A heterotroph is an organism that

ingests or absorbs organic carbon

(rather than fix carbon from inorganic

sources such as carbon dioxide) in order

to be able to produce energy and

synthesize compounds to maintain its

life Ninety-five percent or more of all

types of living organisms are

heterotrophic including all animals and

fungi and some bacteria

Q144 Which of the following is a

primary function of haemoglobin

नननननलिखितमसहॳकह९नसाटहमह८गिह८बबनकाएकपरािलमककाययहहॴ

25-Jan-2017

Options

1) Utilization of energy

उजायकाउपयह८गकरना 2) Prevention of anaemia

रकतामपताहह८नहॳसहॳरह८कना 3) Destruction of bacteria

बहॴकटीररयाकापवनािकरना 4) To transport oxygen

ऑकसीजनकावहनकरना Correct Answer To transport oxygen

Q145 Vascular bundles are absent in

सवहनीबिि mdashmdash- मअनपनसतिरहतहॳहहॴ 25-Jan-2017

Options

1) Bryophyta

िायह८फाइटा 2) Pteridophyta

टहॳररिह८फाईटा 3) Gymnosperms

नजननह८सपमय 4) Angiosperms

एननजयह८सपहॳनसय Correct Answer Bryophyta

Q146 Sauria Lacertidae is the scientific

name of

सहॴररयािहॳसरटाईिी mdashmdashndash कावहॴजञाननकनामहहॴ 25-Jan-2017

Options

1) Crocodile

मगरमचछ

2) Hippopotamus

टहपपह८पह८टहॳमस

3) Lizard

नछपकिी 4) House fly

F A C E B O O K

P A G E h t t p w w w f a c e b o o k c o m s s c m e n t o r s o f f i c i a l P a g e | 35

FOR MORE UPDATES AND MORE MATERIAL DO LIKE OUR FACEBOOK PAGE httpwwwfacebookcomsscmentorsofficial

घरहॳिमकिी Correct Answer Lizard

Q147 Which type of pathogen causes

the water-borne disease SARS (Severe

Acute Respiratory Syndrome)

ककसपरकािकारह८गज़नकजिजननतबीमारीसासयकाकारणबनताहहॴ 25-Jan-2017

Options

1) Viral

वायरि

2) Parasitic

परजीवी 3) Protozoan

परह८टह८जअन

4) Bacterial

बहॴकटीररयि

Correct Answer Viral

Q148 Which of the following organs

produces the enzyme lipase

नननननलिखितमसहॳकह९नसाअगिायपहॳजएजाइमउतपननकरताहहॴ 25-Jan-2017

Options

1) Pancreas

अगनयािय

2) Large Intestine

बड़ीआत

3) Liver

नजगर

4) Small Intestine

छह८टीआत

Correct Answer Pancreas

Q149 A is a long internode forming the

basal part or the whole of a peduncle

एक mdashmdash- एकिबाइटरनह८िहहॴ जह८ननचिाटहससायासनपणयिठिबनताहहॴ 25-

Jan-2017

Options

1) Rhizome

परकद

2) Rachis

महॳ दि

3) floral axis

पषपअकष

4) Scape

भगदड़

Correct Answer scape

Q150 ndash Which of the following

organisms are considered to be both

Living and Non-living

नननननलिखितमसहॳकह९नसहॳजीवाणकह८जीपवतऔरअजीपवतमानाजाताहहॴ

25-Jan-2017

Options

1) Bacteria

बहॴकटीररया 2) Fungi

कवक

3) Algae

िहॴवाि

4)Virus

वायरस

Correct Answer Virus

They are considered to be living as they

possess a protein coat as a protective

covering DNA as the genetic material

etc

They are said to be non-living as they

can be crystallised and they survive for

billions of years They can tolerate high

temperatures freezing cold

temperatures ultra-violet radiations etc

Q151 Deficiency of fluorine causes

which of the following

फिह८ररनकीकमीकहॳ कारणनननननलिखितमसहॳकयाहह८ताहहॴ

F A C E B O O K

P A G E h t t p w w w f a c e b o o k c o m s s c m e n t o r s o f f i c i a l P a g e | 36

FOR MORE UPDATES AND MORE MATERIAL DO LIKE OUR FACEBOOK PAGE httpwwwfacebookcomsscmentorsofficial

27-Jan-2017

Options

1) Dental Caries

िटिकहॴ ररज

2) Scurvy

सकवरी 3) Anaemia

रकतामपता 4) Arthritis

गटठया Correct Answer Dental Caries

Q152 In a Punnett Square with the

cross AaBb x AaBb how many Aabb

genotypes would be created

पनहॳटसककायरमिह८स AaBb x AaBb कहॳ साि

ककतनहॳ Aabb जीनह८टाइपबनगहॳ 27-Jan-2017

Options

1) 1

2) 8

3) 2

4) 3

Correct Answer 2

Q153 Which of the following is the

Controlling Center of the Cell

नननननलिखित म सहॳ कह८लिकाका ननयतरण

क दर कह९न हहॴ

27-Jan-2017

Options

1) Nucleus

क दर

2) Plasma

पिाजमा 3) Lysosome

िायसह८सह८म

4) Chromosome

िह८मह८सह८म

Correct Answer Nucleus

The control centre of the cell is the

nucleus in eukaryotic cells The nucleus

contains genetic material in the form of

DNA

Q154 Myopia affects which of the

following organs

मायह८पपयानननननलिखितअगह८मसहॳककसहॳपरभापवतकरताहहॴ

25-Jan-2017

Options

1) Heart

हदय

2) Skin

तवचा 3) Eyes

आािहॳ 4)Mouth

मह

Correct Answer Eyes

Q155 Which of the following bears

flowers

नननननलिखितमसहॳकह९नफिधारणकरताहहॴ

25-Jan-2017

Options

1) Bryophyta

िायह८फाइटा 2) Pteridophyta

टहॳरीिह८फाईटा 3) Gymnosperms

नजननह८सपमय 4)Angiosperms

एननजयह८सपमय Correct Answer Angiosperms

Q156 Oxygenated blood flows out of the

heart through the

ऑकसीजनयकतरकत mdashmdashmdash

कहॳ माधयमसहॳहदयकहॳ बाहरबहताहहॴ 25-Jan-2017

F A C E B O O K

P A G E h t t p w w w f a c e b o o k c o m s s c m e n t o r s o f f i c i a l P a g e | 37

FOR MORE UPDATES AND MORE MATERIAL DO LIKE OUR FACEBOOK PAGE httpwwwfacebookcomsscmentorsofficial

Options

1) Aorta

महाधमनी 2) pulmonary artery

फहॳ फड़हॳकीधमनी 3) vena cava

वहॳनाकावा 4)Atrium

चह९क

Correct Answer aorta

Q157 Blood leaving the liver and

moving towards the

heart has a higher concentration of

नजगरसहॳननकिकरहदयकीतरफजानहॳवािहॳरकतम mdashmdashmdashmdash कीउचचसादरताहह८तीहहॴ 27-Jan-2017

Options

1) Lipids

लिपपडस

2) Urea

यररया 3) Bile Pigments

पपततकहॳ रगकरण

4) Carbon dioxide

काबयनिायऑकसाइि

Correct Answer Bile Pigments

Urea is nitrogen containing substance

which is produced in the liver in order

to deal with excess amino-acids in the

body As urea is produced it leaves the

liver in the blood stream and passes via

the circulatory system to all parts of the

body

Q158 Bulb is a modification of which

part of a plant

बमबएकपह९धहॳकहॳ ककसटहससहॳकाएक पातरणहह८ताहहॴ 27-Jan-2017

Options

1) The root

जड़

2) The stem

तना 3) The radicle

मिाकर

4)The fruit

फि

Correct Answer The stem

Q159 Which of the following carries

blood away from the heart to different

body parts

इनमहॳसहॳकह९नरकतकह८हदयसहॳिरीरकहॳ पवलभननअगह८तकिहॳजातीहहॴ

27-Jan-2017

Options

1) Arteries

धमननया 2) Nerves

तबतरहाए

3) Capillaries

कहॳ लिकाए

4)Veins

नसहॳ Correct Answer Arteries

Q160 The series of processes by which

nitrogen and its compounds are

interconverted in the environment and

in living organisms is called

27-Jan-2017

Options

1)Absorption of Nitrogen

2)Ammonification

3)Nitrogen Fixation

4)Nitrogen Cycle

Correct Answer Nitrogen Cycle

Ammonification or Mineralization is

performed by bacteria to convert

organic nitrogen to ammonia

F A C E B O O K

P A G E h t t p w w w f a c e b o o k c o m s s c m e n t o r s o f f i c i a l P a g e | 38

FOR MORE UPDATES AND MORE MATERIAL DO LIKE OUR FACEBOOK PAGE httpwwwfacebookcomsscmentorsofficial

Nitrification can then occur to convert

the ammonium to nitrite and nitrate

Nitrogen fixation is a process by which

nitrogen in the Earthrsquos atmosphere is

converted into ammonia (NH3) or other

molecules available to living organisms

Q161 BCG vaccine is given to protect

from which of the following

बीसीजीकाटटकानननननलिखितमसहॳककसकहॳ बचावकहॳ लिएटदयाजातहहॴ

27-Jan-2017

Options

1) Jaundice

पीलिया 2) Anaemia

रकतमपता 3) Tuberculosis

कषयरह८ग

4) Polio

पह८लियह८ Correct Answer Tuberculosis

Q162 Parallel venation is found in

समानतरवहॳनहॳिन mdashmdashmdash- मपायाजाताहहॴ 27-Jan-2017

Options

1) plants which are monocots

पह९धहॳजह८एकबीजपतरीहह८तहॳहहॴ 2) plants which have a dicot stem

वहॳपह९धहॳनजनकातनादपवदलियहह८ताहहॴ 3) plants with leaves similar to Tulsi

वहॳपह९धहॳनजनकीपनततयतिसीकीपनततयोकहॳ समानहह८तहॳहहॴ 4)plants with tap roots

टहॳप टवािहॳपह९धहॳ Correct Answer plants which are

monocots

Q163 The hardest part of the body is

िरीरकासबसहॳकठह८रभाग mdashndash हहॴ 27-Jan-2017

Options

1) Bones

हडडिय

2) Tooth Enamel

दातकहॳ इनहॳमि

3) Skull

िह८पड़ी 4) Spinal Cord

महॳ रजज

Correct Answer Tooth Enamel

Q164 Which type of pathogen causes

the waterborne disease E coli Infection

ककसपरकारकारह८गजननकजिजननतरह८गईकह८िाईसिमणकाकारणबनताहहॴ 27-Jan-2017

Options

1) Protozoan

परह८टह८जआ

2) Parasitic

परजीवी 3) Bacterial

बहॴकटीररयि

4)Viral

वायरि

Correct Answer Bacterial

Q165 The amount of blood filtered

together by both the kidneys in a 70 kg

adult male human in a minute is

70 की गरा वािहॳएकवयसकप षमएकलमनटमदह८नोगदकहॳदवाराएकसािचाबनीगयीरकतकीमातरहह८तीहहॴ 29-Jan-2017

Options

1) 1100 ml

1100 लमलि

2) 100 ml

F A C E B O O K

P A G E h t t p w w w f a c e b o o k c o m s s c m e n t o r s o f f i c i a l P a g e | 39

FOR MORE UPDATES AND MORE MATERIAL DO LIKE OUR FACEBOOK PAGE httpwwwfacebookcomsscmentorsofficial

100 लमलि

3) 1500 ml

1500 लमलि

4) 500 ml

500 लमलि

Correct Answer 1100 ml

Q166 Which feature of a plant helps to

distinguish a monocot from a dicot

पह९धहॳकीवहकह९नसीपविहॳषताहहॴजह८एकदपवदलियहॳऔरएकएकदिीयपह९धहॳसहॳभहॳदकरनहॳममददकरतीहहॴ 29-Jan-2017

Options

1) Pollination

परागम

2) Venation

वहॳनहॳिन

3) Vernation

वनिन

4) Aestivation

एसटीवहॳिहॳन

Correct Answer venation

Q167 The Mutation Theory was

proposed by

उतवररवतयनकालसदात mdashmdashndash

कहॳ दवरापरसतापवतककयाजाताहहॴ 29-Jan-2017

Options

1) Charles Lyell

चामसयलियहॳि

2) William Smith

पवलियमनसमि

3) Hugo De Vries

हयगह८िीराईस

4)Harrison Schmitt

हहॳरीसननसमट

Correct Answer Hugo De Vries

Q168 Which type of pathogen causes

the waterborne disease HepatitisA

ककसपरकारकहॳ रह८गजनकजिजननतरह८गहहॳपहॳटाइटटस-A काकारणबनताहहॴ

29-Jan-2017

Options

1) Parasitic

परजीवी 2) Viral

वायरि

3) Protozoan

परह८टह८जआ

4) Bacterial

बहॴकटीररयि

Correct Answer Viral

Q169 In a Punnett Square with the

cross AaBb x Aabb how many AaBb

genotypes would be created

पनहॳटसकवायरमिह८स AaBb x Aabb

कहॳ सािककतनहॳ AaBb जीनह८टाइपबनगहॳ 29-Jan-

2017

Options

1) 4

2) 1

3) 7

4) 6

Correct Answer 4

Q170 Arboreal Ateles is the scientific

name of

अिह८ररयिएटटलिस mdashmdashmdash कावहॴजञाननकनामहहॴ 29-Jan-2017

Options

1) Squirrel

चगिहरी 2) Sparrow

गह८रहॴया 3) Lizard

नछपकिी 4) Spider monkey

F A C E B O O K

P A G E h t t p w w w f a c e b o o k c o m s s c m e n t o r s o f f i c i a l P a g e | 40

FOR MORE UPDATES AND MORE MATERIAL DO LIKE OUR FACEBOOK PAGE httpwwwfacebookcomsscmentorsofficial

मकड़ीबदर

Correct Answer Spider monkey

Q171 Which type of pathogen causes

the waterborne disease Salmonellosis

ककसपरकारकारह८गाणजिजननतबीमारीसािमह८नहॳिह८लसज़काकारकहहॴ

29-Jan-2017

Options

1) Algal

िहॳवालियहॳ 2) Parasitic

परजीवी 3) Bacterial

बहॴकटीररयि

4)Viral

वायरि

Correct Answer Bacterial

An infection with salmonella bacteria

commonly caused by contaminated food

or water

Symptoms include diarrhoea fever

chills and abdominal pain

Q172 is a condition in which there is a

deficiency of red cells or of haemoglobin

in the blood

mdashmdash-

एकनसिनतहहॴनजसमहॳरकतमिािकह८लिकाओकीयाहीमह८गिह८बबनकीकमीहह८तीहहॴ 29-Jan-2017

Options

1) Albinism

एनमबननजम

2) Propyria

परह८पीररया 3) Anaemia

एनीलमया 4)Keloid disorder

कहॳ िह८इिडिसओिर

Correct Answer Anaemia

Q173 Ananas comosus is the scientific

name of

Options

अनानासकह८मह८सस mdashmdashmdashndash

कावहॴजञाननकनामहहॴ 29-Jan-2017

1) Custard Apple

सीताफि

2) Pineapple

पाइनएपपि

3) Bamboo

बास

4)Pomegranate

अनार

Correct Answer Pineapple

Q174 Which organ produces insulin

कह९नसाअगइनसलिनपहॴदाकरताहहॴ 29-Jan-

2017

Options

1) Liver

यकत

2) Thyroid gland

िायराइिगरिी 3) Spleen

पिीहा 4)Pancreas

अगरयिय

Correct Answer Pancreas

Q175 Which of the following disease is

not caused by water pollution

नननननलिखितमसहॳकह९नसारह८गपानीकहॳ परदषणकहॳकारणनहीहह८ता

29-Jan-2017

Options

1) Cholera

हहॴजा 2) Typhoid

F A C E B O O K

P A G E h t t p w w w f a c e b o o k c o m s s c m e n t o r s o f f i c i a l P a g e | 41

FOR MORE UPDATES AND MORE MATERIAL DO LIKE OUR FACEBOOK PAGE httpwwwfacebookcomsscmentorsofficial

टाइफाइि

3) Asthma

दमा 4)Diarrhoea

दसत

Correct Answer Asthma

Q176 Ocimum tenuiflorum is the

scientific name of

ओलिलममटहॳयईफिह८रमइसकावहॴजञाननकनाम mdash

ndash हहॴ 30-Jan-2017

Options

1) Neem

नीम

2) Mango

आम

3) Babul

बबि

4)Tulsi

तिसी Correct Answer Tulsi

Q177 Which gland secretes bile a

digestive fluid

कह९नसीगरिीपपतत एकपाचनतरिपरदािय सरापवतकरतीहहॴ 30-Jan-2017

Options

1) Pancreas

अगनयािय

2) Liver

यकत

3) Thyroid

िायराइि

4) Testes

टहॳनसटस

Correct Answer liver

Q178 In which of the following the

dominant phase is Gametophyte

नननननलिखितमसहॳककसकहॳ परमिचरणयगमकह८दपवधद (Gametophyte)हहॴ 30-Jan-2017

Options

1) Bryophyta

िायह८फाइटा 2) Pteridophyta

टहॳररिह८फाइटा 3) Gymnosperms

नजननह८सपमय 4) Angiosperms

एननजयह८सपमय Correct Answer Bryophyta

Q179 Anaerobic respiration refers to

which of the following

नननननलिखितमसहॳककसहॳअवायवीयशवसनकहाजाताहहॴ

30-Jan-2017

Options

1) Respiration without Oxygen

ऑकसीजनकहॳ बबनाशवसन

2) Respiration with Oxygen

ऑकसीजनकहॳ सािशवसन

3) Respiration without CO2

काबयनिायऑकसाइिकहॳ बबनाशवसन

4) Respiration with CO2

काबयनिायऑकसाइिकहॳ सािशविन

Correct Answer Respiration without

Oxygen

Q180 Which type of pathogen causes

the waterborne disease Cholera

ककसपरकारकारह८गजनकजिजननतरह८गहहॴजाकाकारणबनताहहॴ

30-Jan-2017

Options

1) Algal

िहॴवालियहॳ

F A C E B O O K

P A G E h t t p w w w f a c e b o o k c o m s s c m e n t o r s o f f i c i a l P a g e | 42

FOR MORE UPDATES AND MORE MATERIAL DO LIKE OUR FACEBOOK PAGE httpwwwfacebookcomsscmentorsofficial

2) Bacterial

बहॴकटीररयि

3) Protozoan

परह८टह८जआ

4) Viral

वायरि

Correct Answer Bacterial

Q181 To which class does

Oxyreductases transferases hydrolases

belong

ओकसीररिकटहॳसटरासफरहॳजहॳस

हाइडरह८िहॳसहॳसककसवगयमआतहॳहहॴ 30-Jan-2017

Options

1) Hormones

हारमोस

2) Enzymes

एजाइनस

3) Proteins

परह८टीनस

4) Vitamins

पवटालमनस

Correct Answer Enzymes

Q182 Which of the following is not true

about Gymnosperms

ननननमसहॳकह९नसीबातअनावतबीजीकहॳ बारहॳमसचनहीहहॴ 30-Jan-2017

Options

1) Dominant phase is saprophytes

परमिचरणसहॳपरह८फाइटसहह८ताहहॴ 2) Vascular bundles are absent

सवहनीबििअनपनसितहह८ताहहॴ 3) spores are heterospores

बीजाणहहॳटहॳरह८सपह८रसहह८तहॳहहॴ 4) Flowers are absent

फिअनपनसितहह८तहॳहहॴ

Correct Answer Vascular bundles are

absent

Q183 The name of first mammal clone sheep is

भहॳड़कीपरिमसतनपायीपरनत प (किह८न)

कानामहहॴ 30-Jan-2017

Options

1) Noori

नरी 2) Dolly

िॉिी 3) Louise

िसी 4)Durga

दगाय Correct Answer Dolly

Q184 Which type of pathogen causes

the water-borne disease Typhoid fever

ककसपरकारकारह८गजनकजिजननतरह८गटाइफाइिबिारकाकारणबनताहहॴ 30-Jan-2017

Options

1) Algal

िहॴवािीय

2) Parasitic

परजीवी 3) Protozoan

परह८टह८जनअन

4)Bacterial

बहॴकटीररयि

Correct Answer Bacterial

Q185 In which part of the cell are

proteins made

कह८लिकाकहॳ ककसटहससहॳमपरह८टीनबनायाजाताहहॴ

31-Jan-2017

Options

1) Reticulum

रहॳटटकिम

F A C E B O O K

P A G E h t t p w w w f a c e b o o k c o m s s c m e n t o r s o f f i c i a l P a g e | 43

FOR MORE UPDATES AND MORE MATERIAL DO LIKE OUR FACEBOOK PAGE httpwwwfacebookcomsscmentorsofficial

2) Golgi apparatus

गह८मजीएपहॳरहॳटस

3) Ribosomes

ररबह८सह८नस

4) Lysosome

िायसह८सह८नस

Correct Answer ribosomes

Proteins are produced by stringing

amino acids together in the order

specified by messenger RNA strands

that were transcribed from DNA in the

cell nucleus The process of synthesizing

a protein is called translation and it

occurs on ribosomes in the cytoplasm of

a cell

Q186 Polio is a disease caused by which

of the following

नननननलिखितमसहॳपह८लियह८कीबबमारह८हह८नहॳकाकारणकयाहहॴ

31-Jan-2017

Options

1) Bacteria

बहॴकटीररयि

2) Mosquito

मचछर

3) Virus

वायरस

4) Cockroach

नतिच हॳ Correct Answer Virus

Polio or poliomyelitis is a crippling and

potentially deadly infectious disease It

is caused by the poliovirus

Q187 ndash Hay fever is a sign of which of

the following

हहॳकफवरनननननलिखितमसहॳककसकाएकसकहॳ तहहॴ

31-Jan-2017

Options

1) Old Age

वदावसिा 2) Malnutrition

कपह८सण

3) Allergy

एिनजय 4) Over Work

अतयचधककाययकरना Correct Answer Allergy

Q188 How many chromosomes does a

human cell contain

एकमानवकह८लिकामककतनहॳगणसतरहह८तहॳहहॴ

29-Jan-2017

Options

1) 6

2) 26

3) 46

4) 66

Correct Answer 46

In humans each cell normally contains

23 pairs of chromosomes for a total of

46 Twenty-two of these pairs called

autosomes look the same in both males

and females The 23rd pair the sex

chromosomes differ between males and

females

Q189 Which of the following is not true

about Bryophyta

ननननमसहॳकह९नसीबातिायह८फाइटकहॳ बारहॳमसचनहीहहॴ 31-Jan-2017

Options

1) Dominant phase is gametophytes

परमिचरणगहॳलमतह८फाइटसहह८ताहहॴ 2) Main plant body is haploid

पह९धहॳकामखयिरीरअगखणतहह८ताहहॴ 3) Spores are homospores

बीजाणहह८मह८सफह८रसहह८तहॳहहॴ 4) Flowers are present

फिमह८जदहह८तहॳहहॴ Correct Answer Flowers are present

F A C E B O O K

P A G E h t t p w w w f a c e b o o k c o m s s c m e n t o r s o f f i c i a l P a g e | 44

FOR MORE UPDATES AND MORE MATERIAL DO LIKE OUR FACEBOOK PAGE httpwwwfacebookcomsscmentorsofficial

Q190 Which aquatic animal has

trailing tentacles

ककसजिीयजानवरकहॳ पीछहॳचिनहॳवािहॳटहॳटकिसहह८तहॳहहॴ

31-Jan-2017

Options

1) Sea horse

समदरीघह८िा 2) Corals

मगा 3) Jelly fish

जहॳिीमछिी 4) Star fish

तारामछिी Correct Answer Jelly fish

Jellyfish with its umbrella-shaped bell

and trailing tentacles

Q191 Which type of pathogen causes

the water-borne disease Poliomyelitis

(Polio)

ककसपरकारकारह८गजनकजिजननतरह८गपह८लियह८मायहॳटटस (पह८लियह८) काकारणहहॴ 31-Jan-

2017

Options

1) Parasitic

परजीवी 2) Algal

िहॴवालिय

3) Viral

वायरि

4) Bacterial

बहॴकटीररयि

Correct Answer Viral

Q192 The outer white part of the eye

that protects the inner structures is

आािकाबाहरीसफहॳ दटहससाजह८आतररकसरचनाओकीरकषाकरताहहॴ वह mdashmdashmdash हहॴ 31-Jan-

2017

Options

1) Iris

आयररस

2) Sclera

सकिहॳरा 3) Retina

रहॳटटना 4) Cornea

कह८ननयया Correct Answer Sclera

Q193 Proteins are made up of

परह८टीनकाननमायण mdashndash सहॳहह८ताहहॴ 31-Jan-2017

Options

1) Amino acids

एलमनह८अनि

2) Fatty acids

वसायकतअनि

3) Glucose

गिकह८ज

4)Nucleotides

नयनकियह८टाईिस

Correct Answer Amino acids

Q194 Moringa Oleifera is the scientific

name of

मह८ररगओलिफहॳ रा mdashmdashndash कावहॴजञाननकनामहहॴ 31-Jan-2017

Options

1) Banyan

बरगद

2) Gulmohar

गिमह८हर

3) Amla

आमिा

F A C E B O O K

P A G E h t t p w w w f a c e b o o k c o m s s c m e n t o r s o f f i c i a l P a g e | 45

FOR MORE UPDATES AND MORE MATERIAL DO LIKE OUR FACEBOOK PAGE httpwwwfacebookcomsscmentorsofficial

4) Drumstick

डरमनसटक

Correct Answer Drumstick

Q195 Kidney stones are composed of

गदकीपिरी mdashndash सहॳबनीहह८तीहहॴ 1-Feb-2017

Options

1) Calcium Oxalate

कहॴ नमसयमओकजहॳिहॳट

2) Sodium Chloride

सह८डियमकिह८राइि

3) Magnesium Nitrate

महॳनगनलियमनाइतटरहॳट

4) Calcium Bicarbonate

कहॴ नमियमबायकबोनहॳट

Correct Answer Calcium Oxalate

Q196 ndash Which of the following is not

true about Angiosperms

ननननमसहॳकह९नसीबातआवतबीजीकहॳ बारहॳमसचनहीहहॴ 1-Feb-2017

Options

1) Dominant phase is gametophytes

परमिचरणगहॳलमतह८फाइटहह८ताहहॴ 2) Vascular bundles are present

सवहनीबििमह९जदहह८ताहहॴ 3) Spores are heterospores

बीजाणहहॳटहॳरह८सपह८रसहह८तहॳहहॴ 4) Seeds are covered

बीजढकहॳ हह८तहॳहहॴ Correct Answer Dominant phase is

gametophytes

Q197 All of the following are excretory

(waste) products of animals except

नननननलिखितमसहॳककसएककह८छह८ड़करअनयसभीपराखणयोदवाराउतसनजयतपदाियहहॴ 1-Feb-

2017

Options

1) Uric Acid

यररकएलसि

2) Ammonia

अमह८ननया 3) Carbohydrates

काबोहाइडरहॳट

4) Urea

यररया Correct Answer Carbohydrates

In animals the main excretory products

are carbon dioxide ammonia (in

ammoniotelics) urea (in ureotelics) uric

acid (in uricotelics) guanine (in

Arachnida) and creatine

Q198 RNA is a polymeric molecule

What does RNA stand for

आरएनइएएकबहिकआणहहॴ इसकाकापवय पकयाहहॴ 1-Feb-2017

Options

1) Rado Nuclear Acid

रािह८नयनकियरएलसि

2) Ribo Nucleic Acid

राइबह८नयनकिकएलसि

3) Rhino Nuclear Acid

हाइनह८नयनकियरएलसि

4) Resto Nucleus Acid

रहॳसटह८नयकिीयसएलसि

Correct Answer Ribo Nucleic Acid

Q199 Which organ does detoxification

and produces chemicals needed for

digestion

कह९नसाअगपवषहरणकरताहहॴऔरपाचनकहॳ लिएआवशयकरसायनोकह८पहॴदाकरताहहॴ 1-Feb-

2017

Options

1) Salivary glands

िारगरचिया 2) Pancreas

अगनयािय

F A C E B O O K

P A G E h t t p w w w f a c e b o o k c o m s s c m e n t o r s o f f i c i a l P a g e | 46

FOR MORE UPDATES AND MORE MATERIAL DO LIKE OUR FACEBOOK PAGE httpwwwfacebookcomsscmentorsofficial

3) Thyroid gland

िायराइिगरिी 4) Liver

यकत

Correct Answer Liver

Q200 Psidium guajava is the scientific

name of

लसडियमगआजावा mdashmdash कावहॴजञाननकनामहहॴ 1-

Feb-2017

Options

1) Guava

अम द

2) Mango

आम

3) Bamboo

बास

4) Jack fruit

कटहि

Correct Answer Guava

Q201 Which drug is used as a Blood

Thinner

चधरकह८पतिाकरनहॳकहॳ पमककसदवाकापरयह८गककयाजाताहहॴ

1-Feb-2017

Options

1) Warfarin

वाफर न

2) Tramadol

टरहॳमािह८ि

3) Azithromycin

एनजरह८मायलसन

4) Hydralazine

हाइडरह८िहॳनजन

Correct Answer Warfarin

Q202 Which of the following disease is

caused due to the deficiency of protein

परह८टीनकीकमीकहॳ कारणनननननलिखितमसहॳकह९नसारह८गहह८ताहहॴ 1-Feb-2017

Options

1) Arthritis

गटठया 2) Kwashiorkor

कािीओकय र

3) Goitre

गाइटर

4) Night Blindness

रतह९चध

Correct Answer Kwashiorkor

Q203 A is species of plant that has

adapted to survive in an environment

with little liquid water

mdashmdashndashपह९धहॳकीएकऐसहॳऐसहॳपरजानतहहॴ नजसनहॳकमपानीवािहॳवातावरणमजीपवतरहनहॳकहॳलिएअनकिनहहॴ 1-Feb-2017

Options

1) Xerophyte

म दपवद

2) Hydrophyte

जिीयपादप

3) Mesophyte

समह८दपवद

4) Thallophyte

िहॴिह८फाइटा Correct Answer xerophyte

xerophyte is a species of plant that has

adapted to survive in an environment

with little liquid water such as a desert

or an ice- or snow-covered region in the

Alps or the Arctic

Mesophytes are terrestrial plants which

are adapted to neither a particularly

dry nor particularly wet environment

An example of a mesophytic habitat

would be a rural temperate meadow

F A C E B O O K

P A G E h t t p w w w f a c e b o o k c o m s s c m e n t o r s o f f i c i a l P a g e | 47

FOR MORE UPDATES AND MORE MATERIAL DO LIKE OUR FACEBOOK PAGE httpwwwfacebookcomsscmentorsofficial

which might contain goldenrod clover

oxeye daisy and Rosa multiflora

thallophyte any of a group of plants or

plantlike organisms (such as algae and

fungi) that lack differentiated stems

leaves and roots and that were formerly

classified as a primary division

(Thallophyta) of the plant kingdom

Q204 How many types of teeth are

there in humans

मनषयोमककतनहॳपरकारकहॳ दातहह८तहॳहहॴ

1-Feb-2017

Options

1) 4

2) 5

3) 2

4) 3

Correct Answer 4

teeth -Humans have four types of

teethincisors canines premolars and

molars each with a specific function

The incisors cut the food the canines

tear the food and the molars and

premolars crush the food

Q205 Carica papaya is the scientific name of

कहॴ ररकापपाया mdashmdashndash कावहॴजञाननकनामहहॴ 2-

Feb-2017

Options

1) Peepal

पीपि

2) Papaya

पपीता 3) Tamarind

इमिी 4) Drumstick

ढह८िकाछड़ी Correct Answer Papaya

Q206 Muscles get tired when there is

shortfall of

जब mdashndash कीकमीहह८तीहहॴतबपहॳिीयिकजातीहहॴ 2-Feb-2017

Options

1) Lactic acid

िहॴनकटकएलसि

2) Na+ ions

Na+ आयन

3) ATP

एटीपी 4) Sulphates

समफहॳ टस

Correct Answer ATP

ATP is the energy source muscle fibers

use to make muscles contract

muscle tissuersquos main source of energy

called adenosine triphosphate or ATP

As your muscles use up this energy

source they become tired and fatigued

Oxygen is the key ingredient that helps

create new ATP to replenish the burned

up ATP in your muscles

Q207 Artocarpus integra is the

scientific name of आटह८कापयसइटीगरा mdashmdashmdash कावहॴजञाननकनामहहॴ 2-Feb-2017

Options

1) Guava

अम द

2) Pineapple

अनानास

3) Silver Oak

लसमवरओक

4) Jack fruit

कटहि

Correct Answer Jack fruit

Q208 Which organ stores fat soluble

vitamins

कह९नसाअगवसामघिनिीिपवटालमनह८काभिाराकरताहहॴ

2-Feb-2017

F A C E B O O K

P A G E h t t p w w w f a c e b o o k c o m s s c m e n t o r s o f f i c i a l P a g e | 48

FOR MORE UPDATES AND MORE MATERIAL DO LIKE OUR FACEBOOK PAGE httpwwwfacebookcomsscmentorsofficial

Options

1) Blood

रकत

2) Skin

तवचा 3) Liver

यकत

4) Pancreas

अगनयािय

Correct Answer Liver

Q209 Which disease is caused due to

deficiency of Iodine

आयह८िीनकहॳ कारणकह९नसारह८गहह८ताहहॴ 2-Feb-2017

Options

1) Rickets

ररकहॳ टस

2) Scurvy

सकवी 3) Goitre

गणमािा 4) Growth retardation

पवकासका कना Correct Answer Goitre

rickets A softening and weakening of

bones in children usually due to

inadequate vitamin D

Q210 Grevillea Robusta is the scientific name of

गरहॳपवलियारह८बसटा mdashmdashmdash- कापवजञाननकनामहहॴ 2-Feb-2017

Options

1) Peepal

पीपि

2) Teak

सागह९न

3) Silver Oak

लसमवरओक

4) Jack fruit

कटहि

Correct Answer Silver Oak

Q211 When a Cuttlefish is described as a Molluscs it is at which level of

classification

जबएककटिकफिकह८एकमह८िसकाकहॳ पमवखणयतककयाजाताहहॴतबयहॳवगीकरणकहॳ ककससतरपहॳनसितहहॴ 2-Feb-2017

Options

1) Class

वगय 2) Order

िम

3) Family

पररवार

4) Phylum

सघ

Correct Answer Phylum

Q212 Bambusa dendrocalmus is the

scientific name of बानबसािहॳडराकामस mdashmdashmdash कावहॴजञाननकनामहहॴ 3-Feb-2017

Options

1) Banyan

बरगद

2) Papaya

पपीता 3) Bamboo

बास

4) Pomegranate

अनार

Correct Answer Bamboo

Q213 Acinonyx Jubatus is the scientific name of

एलसनह८ननकसजयबहॳटस mdashmdashmdash

कावहॴजञाननकनामहहॴ 3-Feb-2017

F A C E B O O K

P A G E h t t p w w w f a c e b o o k c o m s s c m e n t o r s o f f i c i a l P a g e | 49

FOR MORE UPDATES AND MORE MATERIAL DO LIKE OUR FACEBOOK PAGE httpwwwfacebookcomsscmentorsofficial

Options

1) Bear

भाि 2) Horse

घह८िा 3) Cheetah

चीता 4) Zebra

जहॳिा Correct Answer Cheetah

Q214 The pale yellow colour of urine is

due to the presence of which pigment

मतरकाफीकापीिारगरगदरयकहॳ उपनसिनतकहॳ कारणहह८ताहहॴ

3-Feb-2017

Options

1) Urochrome

यरह८िह८म

2) Urophyll

यरह८कफि

3) Chlorophyll

किह८रह८कफि

4) Chloroplast

किह८रह८पिासट

Correct Answer Urochrome

Q215 Which of the following constitute

to form a gene

नननननलिखितमसहॳकह९नसीचीज़एकजीनकागठनकरतीहहॴ

3-Feb-2017

Options

1) Polynucleotides

पह८िीनयनकियह८टाईडस

2) Hydrocarbons

हाइडरह८काबोस

3) Lipoproteins

िाईपह८परह८टीनस

4) Lipids

लिपपडस

Correct Answer Polynucleotides

Polynucleotide molecule is a biopolymer

composed of 13 or more nucleotide

monomers covalently bonded in a chain

DNA (deoxyribonucleic acid) and RNA

(ribonucleic acid) are examples of

polynucleotides with distinct biological

function

Q216 Vertebrates belongs to the

phylum

रीढ़कीहडिीवािहॳपराणी mdashmdashmdash

परजानतकहॳ अतगायतआतहॳहहॴ 3-Feb-2017

Options

1) Arthropoda

आरह८पह८ड़ा 2) Annelida

एननलििा 3) Cnidaria

ननिहॳररया 4) Chordata

कह८िटा Correct Answer Chordata

Q217 Punica granatum is the scientific name of

पननकगरहॳनहॳटस mdashmdashmdash कावहॴजञाननकनामहहॴ 3-Feb-2017

Options

1) Custard Apple

सीताफि

2) Gulmohar

गिमह८हर

3) Silver Oak

लसमवरओक

4) Pomegranate

अनार

Correct Answer Pomegranate

F A C E B O O K

P A G E h t t p w w w f a c e b o o k c o m s s c m e n t o r s o f f i c i a l P a g e | 50

FOR MORE UPDATES AND MORE MATERIAL DO LIKE OUR FACEBOOK PAGE httpwwwfacebookcomsscmentorsofficial

Q218 Between a tiger and an monkey

which of the following is different

एकबाघऔरबदरकहॳ बीचनननननलिखितमसहॳकह९नसीबातअिगहहॴ 3-Feb-2017

Options

1) Kingdom

राजय

2) Phylum

जानत

3) Order

िम

4) Class

वगय Correct Answer order

Q219 The artificial heart was invented by

कबतरमहदयका mdashmdashmdash

दवाराअपवषकारककयागयािा 3-Feb-2017

Options

1) Muhammad Yunus

महनमदयनस

2) Linus Yale Jr

िाइनसयहॳिजय

3) Gazi Yasargil

गाजीयासचगयि

4) Paul Winchell

पह९िपवमकि Correct Answer Paul Winchell

Q220 Tamarindus indica is the

scientific name of

टहॳमररनडसइडिका mdashmdash कावहॴजञाननकनामहहॴ 7-

Feb-2017

Options

1) Neem

नीम

2) Pineapple

अनानास

3) Tamarind

इमिी 4)Chiku

चीक

Correct Answer Tamarind

Q221 In eukaryotic cells synthesis of

RNA takes place in the

यकहॳ योटटककह८लिकाओमआरएनएकासशिहॳषण

mdashndash महह८ताहहॴ 7-Feb-2017

Options

1) Mitochondria

माईटह८कोडडरया 2) Centrioles

सटरीयह८मस

3) Ribosomes

ररबह८सह८नस

4) Nucleus

नयनकियस

Correct Answer nucleus

eukaryotic cell -Transcription is the

process of synthesizing ribonucleic acid

(RNA)Synthesis takes place within the

nucleus of eukaryotic cells or in the

cytoplasm of prokaryotes and converts

the genetic code from a gene in

deoxyribonucleic acid ( DNA ) to a

strand of RNA that then directs

proteinsynthesis

Q222 _________is caused by parasites

of the Plasmodium genus

पिाजमह८डियमजातीकहॳ परजीवी mdash- कहॳ कारणहहॴ 7-Feb-2017

Options

1) Dysentery

पहॳचचि

2) Malaria

मिहॳररया 3) Chickenpox

F A C E B O O K

P A G E h t t p w w w f a c e b o o k c o m s s c m e n t o r s o f f i c i a l P a g e | 51

FOR MORE UPDATES AND MORE MATERIAL DO LIKE OUR FACEBOOK PAGE httpwwwfacebookcomsscmentorsofficial

चहॳचक

4) Herpes

हहॳपपयस

Correct Answer Malaria

Q223 Carotene in fruits and vegetables

gives it which color

फिह८औरसनलजयोमनसितकहॳ रह८टीनउनहकह९नसारगपरदानकरताहहॴ 7-Feb-2017

Options

1) Green

हरा 2) Pink

गिाबी 3) Orange

नारगी 4) Blue

नीिा Correct Answer Orange

Q224 Equus Caballus is the scientific

name of

एकवसकहॴ बहॳिस mdashmdashndash कापवजञाननकनामहहॴ 7-Feb-2017

Options

1) Horse

घह८िा 2) Zebra

ज़हॳिा 3) Donkey

गधा 4) Buffalo

भस

Correct Answer Horse

Q225 Elapidae Naja is the scientific name of

एिीपीिीनाजा mdashmdash- कावहॴजञाननकनामहहॴ 8-Feb-2017

Options

1) Cobra

कह८बरा 2) Elephant

हािी 3) Eagle

ग ि

4) Owl

उमि Correct Answer Cobra

Q226 Which disease is caused due to

deficiency of Iron

िह८हकीकमीकहॳ कारणकह९नसारह८गहह८ताहहॴ 8-Feb-

2017

Options

1) Beriberi

बहॳरीबहॳरी 2) Tetany

टहॳटनी 3) Kwashiorkor

कवािीऔरकर

4) Anaemia

रकतामपता Correct Answer Anaemia

Beriberi is a disease caused by a vitamin

B-1 deficiency also known as thiamine

deficiency

Tetany can be the result of an

electrolyte imbalance Most often itrsquos a

dramatically low calcium level also

known as hypocalcemia Tetany can also

be caused by magnesium deficiency or

too little potassium Having too much

acid (acidosis) or too much alkali

(alkalosis) in the body can also result in

tetany

Kwashiorkor also known as

ldquoedematous malnutrition It is a form of

malnutrition caused by a lack of protein

in the diet

Anaemia means that you have fewer red

blood cells than normal or you have less

F A C E B O O K

P A G E h t t p w w w f a c e b o o k c o m s s c m e n t o r s o f f i c i a l P a g e | 52

FOR MORE UPDATES AND MORE MATERIAL DO LIKE OUR FACEBOOK PAGE httpwwwfacebookcomsscmentorsofficial

haemoglobin than normal in each red

blood cell

Q227 is a leaf where the leaflets are

arranged along the middle vein

mdashndashएकपततीहहॴजहापतरकह८कीरचनाक ररयालिराकहॳ आसपासहह८तीहहॴ 8-Feb-2017

Options

1) Pinnately compound leaf

पपनहॳटिीसयकतपतती 2) Palmately compound leaf

पामहॳटिीसयकतपतती 3) Compound leaf

सयकतपतती 4) Simple leaf

साधारणपतती Correct Answer Pinnately compound

leaf

Q228 Haustoria or sucking roots are

found in which of the following

हह८सटह८ररयायाचसनहॳवािीजड़हॳनननननलिखितमसहॳककसमपाईजातीहहॴ 8-Feb-2017

Options

1) Wheat

गहॳह

2) Mango

आम

3) Chestnut

चहॳसटनट

4) Cuscuta

कसकयटा Correct Answer Cuscuta

Haustorial roots -The roots of parasitic

plants which penetrate into the host

tissues to absorb nourishment are

called haustorial roots hellip Also known as suckingor parasitic roots

Q229 Equs Asinus is the scientific name

of

एकवसएलसनस mdashmdashndash कावहॴजञाननकनामहहॴ 8-

Feb-2017

Options

1) Donkey

गधा 2) Cow

गाय

3) Deer

टहरन

4) Kangaroo

कगा

Correct Answer Donkey

Q230 Ficus benghalensis is the scientific name of

फाईकसबहॳनगहॳिहॳलसस mdashndash कापवजञाननकनामहहॴ 8-Feb-2017

Options

1) Banyan

बरगद

2) Pineapple

अनानास

3) Babul

बबि

4) Tulsi

तिसी Correct Answer Banyan

Q231 Equus burchellii is the scientific name of

एकवसबचिी mdashmdash- कापवजञाननकनामहहॴ 8-Feb-2017

Options

1) Horse

घह८िा 2) Zebra

जहॳिा 3) Buffalo

F A C E B O O K

P A G E h t t p w w w f a c e b o o k c o m s s c m e n t o r s o f f i c i a l P a g e | 53

FOR MORE UPDATES AND MORE MATERIAL DO LIKE OUR FACEBOOK PAGE httpwwwfacebookcomsscmentorsofficial

भस

4) Ass

गधा Correct Answer Zebra

Page 17: COMPILATION OF ALL 72 SETS OF BIOLOGY SSC CHSL-2016 · OF BIOLOGY SSC CHSL-2016 PREPARED BY : SSC MENTORS BIOLOGY SPECIAL . F A C E B O O K P A G E : h t t p : / / w w w . f a c e

F A C E B O O K

P A G E h t t p w w w f a c e b o o k c o m s s c m e n t o r s o f f i c i a l P a g e | 16

FOR MORE UPDATES AND MORE MATERIAL DO LIKE OUR FACEBOOK PAGE httpwwwfacebookcomsscmentorsofficial

Options

1) Caventou

कहॳ वहॳत 2) Pelletier

पहॳिहॳटटयर

3) Chlorophyll

किह८रह८कफि

4) Caventou and Pelletier

कहॳ वहॳतऔरपहॳिहॳटटयर

Correct Answer Caventou and Pelletier

Chlorophyll was first isolated and

named by

Joseph Bienaimeacute Caventou and Pierre

Joseph Pelletier in 1817 The presence of

magnesium in chlorophyll was

discovered in 1906 and was the first

time that magnesium had been detected

in living tissue

Q64 Which of the following organisms

does not fit into the Cell Theory

नननननलिखितमसहॳकह९नसाजीवकह८लिकालसदातअन पनहीहहॴ

16-Jan-2017

Options

1) Bacteria

बहॴकटीररया 2) Virus

वायरस

3) Fungi

कवक

4) Plants

पह९धहॳ Correct Answer Virus

The bottom line is that viruses are not

alive and not related to cells in any way

The cell theory states that all living

things are made of cells cells are the

basic units of structure and function of

living things and that all cells come

from other cells Since viruses are not

made of cells and do not use cells in any

of their processes they are not related to

the cell theory

Q65 Which of these is not a

macronutrient for Plants

नननननलिखितमसहॳकह९नसापह९धह८कहॳ लिएमिह८नयटरीएटनहीहहॴ

SSC CHSL Science (biology) 2016

Question Paper

17-Jan-2017

Options

1) Nitrogen

नाइटरह८जन

2) Phosphorus

फासफह८रस

3) Potassium

पह८टालसयम

4) Chlorine

किह८रीन

Correct Answer Chlorine

In relatively large amounts the soil

supplies nitrogen phosphorus

potassium calcium magnesium and

sulfur these are often called the

macronutrients In relatively small

amounts the soil supplies iron

manganese boron molybdenum

copper zinc chlorine and cobalt the

so-called micronutrients

Q66 Name the respiratory organs of

insects

कीटह८मनसतिशरवसनअगनामकानामहहॴ

17-Jan-2017

Options

1) Skin

तवचा 2) Body Surface

िरीरकीसतह

F A C E B O O K

P A G E h t t p w w w f a c e b o o k c o m s s c m e n t o r s o f f i c i a l P a g e | 17

FOR MORE UPDATES AND MORE MATERIAL DO LIKE OUR FACEBOOK PAGE httpwwwfacebookcomsscmentorsofficial

3) Gills

गिफड़हॳ 4) Tracheae

शरावस- निी Correct Answer Tracheae

Air enters the respiratory systems of

insects through a series of external

openings called

spiracles These external openings

which act as muscular valves in some

insects lead to the internal respiratory

system a densely networked array of

tubes called tracheae

Q67 The poisonous gas accidentally

released in Bhopal Gas Tragedy is

भह८पािगहॴसतरासदीमगितीसहॳमकतहईजहरीिीगहॴसिी

17-Jan-2017

1) Methane

मीिहॳन

2) Nitrous Oxide

नाइटरसऑकसाइि

3) Methyl Isocyanate

महॴचििआयसोसायनहॳट

4) Cyanogen

सायनह८जहॳन

Correct Answer Methyl Isocyanate

Q68 What does Trypsin do

टटरनपसनकयाकरताहहॴ

SSC CHSL Science (biology) 2016

Question Paper

17-Jan-2017

Options

1) Breaks down Carbohydrates

काबोहाइडरहॳटकापवघटनकरताहहॴ 2) Synthesizes proteins

परह८टीनकासििहॳषणकरताहहॴ 3) Breaks down fats

वसाकापवघटनकरताहहॴ 4) Breaks down proteins

परह८टीनकापवघटनकरताहहॴ Correct Answer Breaks down proteins

Trypsin is one of the three principal

digestive

proteinases the other two being pepsin

and

chymotrypsin In the digestive process

trypsin acts with the other proteinases

to break down dietary protein molecules

to their component

peptides and amino acids

A protease is any enzyme that performs

proteolysis protein catabolism by

hydrolysis of peptide bonds

Q69 Name the source from which

Aspirin is produced

उससरह८तकानामबताइए

नजससहॳएनसपररनकाउतपादनककयाजाताहहॴ

17-Jan-2017

Options

1) Willow bark

पविह८कीछाि

2) Oak Tree

ओककावकष

3) Acacia

बबि

4) Eucalyptus

नीिचगरी Correct Answer Willow bark

The compound from which the active

ingredient in aspirin was first derived

salicylic acid was found in the bark of a

willow tree in 1763 by Reverend

Edmund Stone of Chipping-Norton

Q70 Cannis Familiaris is the scientific

name of

कहॴ ननसफहॳ लमलियहॳररस mdash- कावहॴजञाननकनामहहॴ

17-Jan-2017

F A C E B O O K

P A G E h t t p w w w f a c e b o o k c o m s s c m e n t o r s o f f i c i a l P a g e | 18

FOR MORE UPDATES AND MORE MATERIAL DO LIKE OUR FACEBOOK PAGE httpwwwfacebookcomsscmentorsofficial

Options

1) Cat

बबमिी 2)Dog

कतता 3) Fox

िह८मड़ी 4) Wolf

भहॳडड़या Correct Answer Dog

Q71 Harmful bacteria in potable water

make the water

पीनहॳकहॳ पानीमनसतिघातकबहॴकटीररयाउसपानीकह८बनातहॳहहॴ 17-Jan-2017

Options

1) unfit to drink

पीनहॳकहॳ लिएअयह८गय

2) smelly

दगयनधयकत

3) Colored

रगीन

4) Turbid

मटमहॴिा Correct Answer unfit to drink

Q72 Musa paradisiaca is the scientific

name of which plant

मसापहॴराडिलसयाकाककसपह९धहॳकावहॴजञाननकनामहहॴ

17-Jan-2017

Options

1) Mango

आम

2) Wheat

गहॳह

3) Corn

भ ा 4) banana

कहॳ िा Correct Answer banana

Q73 Prawns belong to which family

झीगहॳककसपररवारकहॳ हह८तहॳहहॴ 17-Jan-2017

Options

1) Crustaceans

िसटहॳलियन

2)Fish

मछिी 3) Amphibians

अननफबबयस

4) Reptiles

रहॳपटाइमस

Correct Answer Crustaceans

Q74 Name the drug that is yielded from

Cinchona tree and is used to cure

malaria

उसऔषचधकानामबताइएनजसहॳलसगकह८नापहॳड़सहॳपरापतककयाजाताहहॴऔरनजसकाउपयह८गमिहॳररयाकहॳ उपचारमककयाजाताहहॴ 17-Jan-2017

Options

1) Camptothea

कहॴ नटह८चिया 2) Acuminata

एकयलमनहॳटा 3) Quinine

कनहॴन

4) Cinchonia

लसकह८ननया Correct Answer Quinine

Q75 Blood Circulation was discovered

by

रकतपररसचरणकी mdashmdashndash दवारािह८जकीिी 17-Jan-2017

Options

1) Mary Anderson

F A C E B O O K

P A G E h t t p w w w f a c e b o o k c o m s s c m e n t o r s o f f i c i a l P a g e | 19

FOR MORE UPDATES AND MORE MATERIAL DO LIKE OUR FACEBOOK PAGE httpwwwfacebookcomsscmentorsofficial

महॴरीएिरसन

2) Virginia Apgar

वनजयननयाएपगार

3) William Harvey

पवलियमहाव

4) Robert Feulgen

रॉबटयफ़यिजहॳन Correct Answer William Harvey

Q76 Vitamin A is also known as

पवटालमन A कह८ mdashmdash- कहॳ नामसहॳभीजानाजाताहहॴ SSC CHSL Science (biology) 2016

Question Paper

18Jan2017

Options

1) Thiamine

िायलमन

2) Riboflavin

ररबह८फिहॳपवन

3) Retinol

रहॳटटनॉि

4) Calciferol

कहॴ नमसफहॳ रह८ि

Correct Answer Retinol

Q77 Some roots called arise from an

organ other than the radicle

कछजड़हॳनजनह mdashmdashmdash कहाजाताहहॴ वहमिकहॳ अिावाककसीअनयअगसहॳउतपननहह८तीहहॴ 18Jan2017

Options

1) tap roots

मखयजड़

2) stilt roots

ि ाजड़

3) fibrous roots

रहॳिहॳदारजड़

4) adventitious roots

आकनसमकजड़

Correct Answer adventitious roots

Q78 Spiders belong to which class of

animals

मकडड़यापराणीवगीकरणकहॳ ककसवगयमआतीहहॴ 18Jan2017

Options

1) Arachnids

एरहॳकननडस

2) Aves

एपवस

3) Gastropods

गहॴसटरोपह८िस

4) Anthozoa

एिह८जआ

Correct Answer Arachnids

Q79 How many layers does Human

Skin have

मानवतवचामककतनीपरतहॳहह८तीहहॴ

18Jan2017

Options

1) 5

2) 7

3) 11

4) 3

Correct Answer 3

Skin has three layers The epidermis

the outermost layer of skin provides a

waterproof barrier and creates our skin

tone The dermis beneath the

epidermis contains tough connective

tissue hair follicles and sweat glands

The deeper subcutaneous tissue (

hypodermis ) is made of fat and

connective tissue

Q80 Allium Cepa is the scientific name

of

एलियमलसपपा mdashmdashndash कावहॴजञाननकनामहहॴ 18Jan2017

F A C E B O O K

P A G E h t t p w w w f a c e b o o k c o m s s c m e n t o r s o f f i c i a l P a g e | 20

FOR MORE UPDATES AND MORE MATERIAL DO LIKE OUR FACEBOOK PAGE httpwwwfacebookcomsscmentorsofficial

Options

1) Carrot

गाजर

2) Tomato

टमाटर

3) Potato

आि 4) Onion

पयाज़

Correct Answer Onion

Q81 DNA stands for

िीएनएकापणय प mdashmdash- हहॴ 18Jan2017

Options

1) Di Nucleic Acid

िाईनयनकिकएलसि

2) Deoxy Nucleic Acid

िीओकसीनयनकिकएलसि

3) Diribonucleic Acid

िाईराइबह८नयनकिकएलसि

4) Deoxyribonucleic Acid

िीऑकसीराइबह८नयनकिकएलसि

Correct Answer Deoxyribonucleic Acid

Q82 Organisms that generate energy

using light are known as

जह८जीवाणपरकािकाउपयह८गकरउजायउतपननकरतीहहॴ उनह mdashmdash कहॳ पमजानाजाताहहॴ

18Jan2017

Options

1) Chaemolithotrophs

ककमह८लििह८टरह८पस

2) Oligotrophs

ओलिगह८टरह८पस

3) Bacteria

बहॴकटीररया 4)Photoautotrophs

फह८टह८ओटह८टरह८पस

Correct Answer Photoautotrophs

An oligotroph is an organism that can

live in an environment that offers very

low levels of nutrients

Q83 Which drug is used as an

Antidepressant

ककसदवाएकहतािारह८धीकहॳ पमपयोगककयाजाताहहॴ Options

1) Oxybutynin

ओकसीलयटीनन

2)Tramadol

टरहॳमहॳिह८ि

3 ) Sumatriptan

समहॳटरीपटहॳन

4) Bupropion

लयपरह८पपयह८न

Correct Answer Bupropion

लयपरह८पपयह८न

Q84 The orange colour of carrot is

because of

गाजरकानारगीरगनननननलिखितमसहॳककसीएककीवजहसहॳहह८ताहहॴ 18Jan2017

Options

1) it grows in the soil

यहलम ीमउगतीहहॴ 2) Carotene

कहॴ रह८टीन

3) it is not exposed to sunlight

यहसययपरकािकहॳ सपकय मनहीआती 4) the entire plant is oranqe in colour

सनपणयपह९धानारगीरगकाहह८ताहहॴ Correct Answer Carotene

Q85 Snake venom is highly modified

saliva containing

F A C E B O O K

P A G E h t t p w w w f a c e b o o k c o m s s c m e n t o r s o f f i c i a l P a g e | 21

FOR MORE UPDATES AND MORE MATERIAL DO LIKE OUR FACEBOOK PAGE httpwwwfacebookcomsscmentorsofficial

सापकाजहरअततयाचधकसिह८चधतिारहह८तीहहॴनजसमहॳ mdashmdash- हह८ताहहॴ Options

l)Prototoxins

परह८टह८टॉनकसस

2)Neutrotoxins

नयटरोटॉनकसस

3)Zootoxins

जटॉनकसस

4)Electrotoxins

इिहॳकटरह८टॉनकसस

Correct Answer Zootoxins

जटॉनकसस

Q86 Which type of pathogen causes the

water-borne disease Schistosomiasis

ककसपरकारकारह८गज़नकजिजननतरह८गलससटह८सह८लमलससकाकारणबनताहहॴ

18Jan2017

Option

1) Parasitic

परजीवी 2)Protozoan

परह८टह८जआ

3) Bacterial

बहॴकटीररयि

4) Viral

वायरि

Correct Answer Parasitic

Schistosomiasis also known as snail

fever and bilharzia is a disease caused

by parasitic

flatworms called schistosomes

Q87 Prothrombin responsible for

clotting of blood is released by

परह८िह८ननबन

जह८रकतकािककाजमनहॳकहॳ लिएनजनमहॳदारहहॴ mdashndash

कहॳ दवारासतरापवतककयाजाताहहॴ

19Jan2017

Options

1) Small Intestine

छह८टीआत

2) Blood Platelets

रकतपिहॳटिहॳटस

3) Large Intestine

बड़ीआत

4Heart

हदय

Correct Answer Blood Platelets

Q88 Acacia arabica is the scientific

name of

अकहॳ लियाअरहॳबबका mdashmdashndash कावहॴजञाननकनामहहॴ 19-Jan-2017

Options

1) Neem

नीम

2) Teak

सागह९न

3) Babhul

बबि

4) Pomegranate

अनार

Correct Answer Babhul

Q89 Cannis Vulpes is the scientific

name of

कहॴ ननसवनमपस mdashmdash- कावहॴजञाननकनामहहॴ 19-Jan-2017

Options

1) Dog

कतता 2) Wolf

भहॳडड़या 3) Fox

िह८मड़ी 4) Hyena

िाकिबगघा

F A C E B O O K

P A G E h t t p w w w f a c e b o o k c o m s s c m e n t o r s o f f i c i a l P a g e | 22

FOR MORE UPDATES AND MORE MATERIAL DO LIKE OUR FACEBOOK PAGE httpwwwfacebookcomsscmentorsofficial

Correct Answer Fox

Q90 The beetroot is the portion of the

beet plant

चकदरपह९धहॳका mdashmdashndash भागहहॴ 19-Jan-2017

Options

1) tap root

मखयजड़

2) Adventitious

आकनसमक

3) bulb of the stem

तनहॳकाकद

4) Rhizome

परकद

Correct Answer tap root

Q91 What is the basic unit of heredity

आनवलिकताकीबननयादीइकाईकयाहहॴ 19-Jan-2017

Options

1) DNA

िीएनए

2) RNA

आरएनए

3) Chromosome

िह८मह८सह८म

4) Gene

जीन

Correct Answer gene

Genes are the units of heredity and are

the instructions that make up the bodyrsquos

blueprint They code for the proteins

that determine virtually all of a personrsquos

characteristics Most genes come in

pairs and are made of strands of genetic

material called deoxyribonucleic acid

or DNA

Q92 Lungs are the primary organs of

फहॳ फड़हॳmdashndashकहॳ परािलमकअगहहॴ

19-Jan-2017

Options

1) Digestion

पाचन

2) Constipation

कलज

3) Perspiration

पसीना 4)Respiration

शवसन

Correct Answer Respiration

Q93 Sugarcane is a type of

गननाएकपरकारका mdash- हहॴ 20-Jan-2017

Options

1)creeper

िता 2)tree

पहॳड़

3)shrub

झाड़ी 4)grass

घास

Correct Answer grass

Q94 Who is commonly known as ldquothe

Father of Microbiologyrdquo

सामानयत ldquo सकषमजीवपवजञानकहॳ जनक lsquo

कहॳ नामसहॳककसहॳजानाजातहहॴ 20-Jan-2017

Options

1) Robert Hooke

रॉबटयहक

2) Antonie Philips van Leeuwenhoek

एटह८नीकफलिपवानमयएनहह८क

3) Carl Linnaeus

काियिीनाईयस

4) Charles Darwin

चामसयिापवयन

F A C E B O O K

P A G E h t t p w w w f a c e b o o k c o m s s c m e n t o r s o f f i c i a l P a g e | 23

FOR MORE UPDATES AND MORE MATERIAL DO LIKE OUR FACEBOOK PAGE httpwwwfacebookcomsscmentorsofficial

Correct Answer Antonie Philips van

Leeuwenhoek

Q95 For the aquatic organisms the

source of food is

जिीयजीवाणकािाघसरह८तहहॴ 20-Jan-2017

Options

1) Phytoplankton

फायटह८पिहॳकटन

2) Sea Weed

समदरीिहॴवाि

3)Aqua plankton

एकवापिहॳकटन

4) Zooplankton

जपिहॳकटन

Correct Answer Phytoplankton

Q96 Haemoglobin has the highest

affinity with which of the following

हीमह८गिह८बबनकीननननमसहॳककसकहॳ सािउततमसमानताहहॴ

20-Jan-2017

Options

1)SO2

2)CO2

3)CO

4)NO2

Correct Answer CO

It has a greater affinity for hemoglobin

than oxygen does It displaces oxygen

and quickly binds so very little oxygen

is transported through the body cells

Q97 Who developed the theory of

Evolution

उदपवकासकालसदातककसनहॳपवकलसतककया

20-Jan-2017

Options

1) Charles Darwin

चामसयिापवयन

2) Isaac Newton

आयजहॳकनयटन

3) Pranav Mistry

परणवलमसतरी 4) Galileo Galilei

गहॳलिलियह८गहॳिीिी Correct Answer Charles Darwin

Q98 The primary function of RNA is

RNA कापरािलमककाययहह८ताहहॴ 20-Jan-2017

Options

1) Photosynthesis

परकािसशिहॳषण

2) Protein Synthesis

परह८टीनसशिहॳषण

3) Replication

परनतकनतबनाना 4) Translation

अनवादकरना Correct Answer Protein Synthesis

There are two main functions of RNA

It assists DNA by serving as a messenger

to relay the proper genetic information

to countless numbers of ribosomes in

your body The other main function of

RNA is to select the correct amino acid

needed by each ribosome to build new

proteins for your body

Q99 ______is the movement of

molecules across a cell membrane from

a region of their lower concentration to

a region of their higher concertration

उचचसादरताकहॳ कषहॳतरसहॳउसकीकमसादरतावािहॳकषहॳतरकीतरफएककह८लिकाखझमिीकहॳ माधयमसहॳहह८नहॳवािाअणओकहॳ सचिनकह८ mdash- कहतहॳहहॴ Options

1) Diffusion

पवसरण

2) Osmosis

ऑसमह८लसस

F A C E B O O K

P A G E h t t p w w w f a c e b o o k c o m s s c m e n t o r s o f f i c i a l P a g e | 24

FOR MORE UPDATES AND MORE MATERIAL DO LIKE OUR FACEBOOK PAGE httpwwwfacebookcomsscmentorsofficial

3) Active Transport

सकियआवागमन

4) Passive Transport

नननषियआवागमन

Correct Answer Active Transport

Q100 Study of classification of

organisms is known as 20-Jan-2017

जीवाणओकहॳ वगीकरणकहॳ अधययनकह८ mdash-

कहाजाताहहॴ Options

1) Serpentology

सपरहॳटह८िह८जी 2) Virology

वायरह८िह८जी 3) Taxonomy

टहॴकसोनह८मी 4) Physiology

कफनज़यह८िह८जी Correct Answer Taxonomy

Q101 Photosynthesis takes place inside

plant cells in

परकािसशिहॳषणवनसपनतकह८लिकामनसति mdash

mdashmdash महह८ताहहॴ 20-Jan-2017

Options

1) Ribosomes

राइबह८सह८नस

2) Chloroplasts

किह८रह८पिासट

3) Nucleus

नयकलियम

4) Mitochondria

माईटह८कोडडरया Correct Answer Chloroplasts

Q102 ______ is the cell organelle in

which the biochemical processes of

respiration and energy production

occur

mdashmdash- वहकह८लिकाअगहहॴ नजसमहॳशवसनऔरउजायउतपादनकहॳ जहॴसीजहॴवरासायननकपरकियायहह८तीहहॴ 20-Jan-2017

Options

1) Mitochondria

माइटह८कोडडरया 2) Chloroplast

किह८रह८पिासट

3) Ribosomes

राइबह८सह८नस

4) Nucleus

नयकिीयस

Correct Answer Mitochondria

Q103 Which non-flowering spore

bearing plants have roots

ककसफिनिगनहॳवािहॳऔरबीजाणधारकपह९धह८कीजड़हॳहह८तीहहॴ 21-Jan-2017

Options

1) Mosses

मह८सहॳस

2) Angiosperms

एननजयह८सपनसय 3) Ferns

फनसय 4) Gymnosperms

नजननह८सपनसय Correct Answer ferns

Q104 Which of the following is an

excretory organ of cockroach

नननननलिखितमसहॳकह९नसानतिच हॳकाउतसजयनअगहहॴ

21-Jan-2017

Options

F A C E B O O K

P A G E h t t p w w w f a c e b o o k c o m s s c m e n t o r s o f f i c i a l P a g e | 25

FOR MORE UPDATES AND MORE MATERIAL DO LIKE OUR FACEBOOK PAGE httpwwwfacebookcomsscmentorsofficial

1) Malphigian Tubules

मनमफनजयनटयबमस

2) Nephridia

नहॳकफरडिया 3) Coxal Gland

कह८कसिगरचिया 4) Green Gland

गरीनगरचिया Correct Answer Malphigian Tubules

Q105 Evaporation of water takes place

in which part of plants

पानीकहॳ वाषपीकरणकीकियापह९धोकहॳ ककसभागसहॳहह८तीहहॴ 21-Jan-2017

Options

1) Stem

तना 2) Stomata

सटह८मटा 3) Branch

िािाए

4) Fruit

फि

Correct Answer Stomata

Evaporation accounts for the movement

of water to the air from sources such as

the soil canopy interception and

waterbodies Transpiration accounts for

the movement of water within a plant

and the subsequent loss of water as

vapour through stomata in its leaves

Q106 A is the fleshy spore-bearing

fruiting body of a fungus

mdashmdashndashकवककामासि

बीजाणधारणकरनहॳवािाफिनहॳवािाअगहहॴ 21-

Jan-2017

Options

1) aloe vera

एिह८वहॳरा 2) Coral

मगा 3) Cactus

कहॴ कटस

4) Mushroom

ककरमतता Correct Answer mushroom

Q107 Which of the following is a fungal

disease

नननननलिखितमसहॳकह९नसाफफदसहॳहह८नहॳवािाएकरह८ग हहॴ

21-Jan-2017

Options

1) Dermatitis

तवचािह८ध

2) Cholera

हहॴजा 3) Jaundice

पीलिया 4) Indigofera

इननिगह८फहॳ रा Correct Answer Dermatitis

Dermatitis also known as eczema is a

group of diseases that results in

inflammation of the skin These diseases

are characterized by itchiness red skin

and a rash In cases of short duration

there may be small blisters while in

long-term cases the skin may become

thickened

Q108 In which form is glucose stored in

our body

हमारहॳिरीरमगिकह८जकासचयककस पमककयाजाताहहॴ

21-Jan-2017

Options

1) Insulin

F A C E B O O K

P A G E h t t p w w w f a c e b o o k c o m s s c m e n t o r s o f f i c i a l P a g e | 26

FOR MORE UPDATES AND MORE MATERIAL DO LIKE OUR FACEBOOK PAGE httpwwwfacebookcomsscmentorsofficial

इसलिन

2) Glucose

गिकह८ज

3) Glycogen

गिायकह८जहॳन

4) Fat

वसा Correct Answer Glycogen

Excess glucose is stored in the liver as

the large compound called glycogen

Glycogen is a polysaccharide of glucose

but its structure allows it to pack

compactly so more of it can be stored in

cells for later use

Q109 Where do plants synthesize

protein from

पह९धहॳपरह८टीनसशिहॳषणकहासहॳकरतहॳहहॴ

Options

1) Fatty Acids

वसाऐलसि

2) Sugar

िकर

3) Amino Acids

एलमनह८ऐलसि

4) Starch

सटाचय Correct Answer Amino Acids

Q110 Which part of the brain is

responsible for triggering actions like

thinking intelligence memory and

ability to learn

मनसतषककाकह९नसाटहससासह८चनहॳ बनधदमानी याददाशतऔरसीिनहॳकीकषमताजहॴसीकियाओकह८परहॳररतकरताहहॴ 21-Jan-2017

Options

1) Diencephalon

िायएनसहॳफहॳ िह८न

2) Hypothalamus

हयपह८िहॳिहॳमस

3) Cerebrum

सहॳरहॳिम

4) Control

कटरह८ि

Correct Answer Cerebrum

Q111 Which of the following is also

known as the Biochemical Laboratory

of the Human Body

नननननलिखितमसहॳककसहॳमानविरीरकीजहॴवरसायनपरयह८गिािाभीकहाजाताहहॴ 21-Jan-2017

Options

1) Small Intestine

छह८टीआत

2)Brain

मनसतषक

3) Pancreas

अगनयािय

4) Liver

नजगर

Correct Answer Liver

The liver makes bile that will help

emulsify and digest the fats we eat

The liver takes toxic substances and

convert them using enzymes the liver

cells makes into a non toxic form so the

body can dispose of them

The liver also converts fats protein and

carbohydrates into glucose which is the

energy source for our cells to use

The liver takes amino acids and makes

proteins by combining them

Q112 The yellow colour of human urine

is due to

मानवमतरकापीिारग mdashndash कीवजहसहॳहह८ताहहॴ 22-

Jan-2017

Options

1) Bile Salts

F A C E B O O K

P A G E h t t p w w w f a c e b o o k c o m s s c m e n t o r s o f f i c i a l P a g e | 27

FOR MORE UPDATES AND MORE MATERIAL DO LIKE OUR FACEBOOK PAGE httpwwwfacebookcomsscmentorsofficial

पपततनमक

2) Cholesterol

कह८िहॳसटरह८ि

3) Lymph

लिनफ

4) Urochrome

यरह८िह८म

Correct Answer Urochrome

Urobilin or urochrome is the chemical

primarily responsible for the yellow

color of urine

Q113 The wilting of plants takes place

due to

पह९धह८कालिचििहह८नाकी mdashmdash- कीवजहसहॳहह८ताहहॴ 22-Jan-2017

Options

1)Photosynthesis

परकािसशिहॳषण

2) Transpiration

वाषपह८तसजयन

3) Absorption

अविह८षण

4) Respiration

शरवसन

Correct Answer Transpiration

Wilting is the loss of rigidity of non-

woody parts of plants This occurs when

the turgor pressure in non-lignified

plant cells falls towards zero as a result

of diminished water in the cells

Q114 Bovidae Ovis is the scientific name of

बह८पविीओपवस mdashndash कावहॴजञाननकनामहहॴ 22-Jan-2017

Options

1) Goat

बकरी 2) Cow

गाय

3) Buffalo

भहॳस

4) Sheep

भहॳड़

Correct Answer Sheep

Q115 Plants get their energy to produce

food from which of the following

पह८धहॳभह८जनकाननमायणकरनहॳकहॳ लिएनननननलिखितमसहॳककससहॳउजायपरापतकरतहॳहहॴ

22-Jan-2017

Options

1) Photosynthesis

परकािसशिहॳषण

2)Bacteria

बहॴकटीररया 3)Fungi

कवक

4)Sun

सयय Correct Answer Sun

Q116 Which of the following is secreted

by the liver

नननननलिखितमसहॳककसकासरावनजगरसहॳहह८ताहहॴ

22-Jan-2017

Options

1) Glucose

गिकह८ज

2) Iodine

आयह८िीन

3) Cortisol

काटटरयसह८ि

4) Bile

पपतत

Correct Answer Bile

The liver makes bile that will help

emulsify and

digest the fats we eat

F A C E B O O K

P A G E h t t p w w w f a c e b o o k c o m s s c m e n t o r s o f f i c i a l P a g e | 28

FOR MORE UPDATES AND MORE MATERIAL DO LIKE OUR FACEBOOK PAGE httpwwwfacebookcomsscmentorsofficial

Q117 Ferns belong to which division of

plants

फनसयपह९धह८कहॳ ककसभागमआतहॳहहॴ

22-Jan-2017

Options

1) Gymnosperms

नजननह८सपनसय 2) Angiosperms

एनजयह८सपनसय 3) Thallophyta

िहॴिह८फाईटा 4)Pteridophyta

टहॳररिह८फाईटा Correct Answer Pteridophyta

Q118 Who invented Antibiotics

एटीबायह८टटककाअपवषकारककसनहॳककयािा

22-Jan-2017

Options

1) Joseph Lister

जह८सहॳफलिसटर

2) William Harvey

पवलियमहाव

3) Robert Knock

रॉबटयनॉक

4)Alexander Fleming

अिहॳकज़िरफिहॳलमग

Correct Answer Alexander Fleming

Q119 Milbecycin is used in the

eradication of

लममबहॳसायलसनका mdashndash

मउनमिनमपरयह८गककयाजाताहहॴ 22-Jan-2017

Options

1) Agricultural Fungus

कपषकवक

2) Agricultural Pests

कपषकीटक

3) Agricultural Herbs

कपषिाक

4)Agricultural Weeds

कपषननराना Correct Answer Agricultural Pests

Milbemycin oxime is a veterinary drug

from the group of milbemycins used as

a broad spectrum antiparasitic It is

active against worms and mites(insects

Q120 Intestinal bacteria synthesizes

which of the following in the human

body

मानविरीरमआतोकहॳ बहॴकटीररयानननननलिखितमसहॳककसकासशिहॳषणकरतहॳहहॴ 22-Jan-2017

Options

1) Vitamin K

पवटालमन K

2) Proteins

परह८टीन

3) Fats

वसा 4) Vitamin D

पवटालमन D

Correct Answer Vitamin K

Q121 is the study of the physical form

and external structure of plants

mdashmdash-

मपह९धह८काभहॴनतक पऔरबाहरीसरचनाकाआदयाककयाजाताहहॴ 22-Jan-2017

Options

1) Physiology

कफनजयह८िह८जी 2) Anatomy

िरीररचनापवजञान

3) Phytomorphology

फाईटह८मह८फह८िह८जी 4)Cytology

कह८लिकापवजञान

Correct Answer Phytomorphology

F A C E B O O K

P A G E h t t p w w w f a c e b o o k c o m s s c m e n t o r s o f f i c i a l P a g e | 29

FOR MORE UPDATES AND MORE MATERIAL DO LIKE OUR FACEBOOK PAGE httpwwwfacebookcomsscmentorsofficial

Q122 Which of the following is a

structural and functional unit of

kidneys

नननननलिखितमसहॳकह९नसीगदोकीसरचनातमकऔरकाययकरीईकाईहहॴ

22-Jan-2017

Options

1) Renette Cells

रहॳनहॳटकह८लिकाए

2) Flame Cells

फिहॳमकह८लिकाए

3) Nephrites

नहॳफ़राइटस

4)Nephrons

नहॳफरोस

Correct Answer Nephrons

Nephron functional unit of the kidney

the structure that actually produces

urine in the process of removing waste

and excess substances from the blood

There are about 1000000 nephrons in

each human kidney

Q123 Which of the following is the

largest part of the human brain

नननननलिखितमसहॳकह९नसामानवमनसतषककासबसहॳबड़ाटहससाहहॴ

23-Jan-2017

Options

1) Ribs

पसलियाा 2) Cerebrum

सहॳरहॳिम

3) Pons

पोस

4)Thalamus

िहॴिहॳमस

Correct Answer Cerebrum

The cerebrum is the largest part of the

human brain making up about two-

thirds of the brainrsquos mass It has two

hemispheres each of which has four

lobes frontal parietal temporal and

occipital

Q124 The auxiliary buds

सहायककालियाmdashndash 23-Jan-2017

Options

1) grow endogenously from the pericycle

पहॳरीसाईककिसहॳअनतजातयपवकलसतहह८ताहहॴ 2) arise endogenously from the main

growing point

मिवपदसहॳअनतजातयउठताहहॴ 3) is an embryonic shoot located in the

axil of a leaf

एकभरणिटहहॴजह८एकपततीकहॳ अकषपरनसतिहह८ताहहॴ 4)arise exogenously from the epidermis

एपपिलमयससहॳबटहजातयतरीकहॳ सहॳउठताहहॴ Correct Answer is an embryonic shoot

located in the axil of a leaf

Q125 Which of the following is a viral

disease

इनमहॳसहॳकह९सीएकवायरिबीमारीहहॴ

23-Jan-2017

Options

1) Polio

पह८लियह८ 2) Tetanus

धनसतनभ

3) Leprosy

कषठरह८ग

4) Plague

पिहॳग

Correct Answer Polio

A viral disease (or viral infection)

occurs when an organismrsquos body is

invaded by pathogenic viruses and

infectious virus particles (virions) attach

to and enter susceptible cells

F A C E B O O K

P A G E h t t p w w w f a c e b o o k c o m s s c m e n t o r s o f f i c i a l P a g e | 30

FOR MORE UPDATES AND MORE MATERIAL DO LIKE OUR FACEBOOK PAGE httpwwwfacebookcomsscmentorsofficial

Poliomyelitis often called polio or

infantile paralysis is an infectious

disease caused by the poliovirus

Tetanusmdash A serious bacterial infection

that causes painful muscle spasms and

can lead to death

Leprosy also known as Hansenrsquos

disease (HD) is a long-term infection by

the bacterium Mycobacterium leprae or

Mycobacterium lepromatosis

Plague is an infectious disease caused by

the bacterium Yersinia pestis

Symptoms include fever weakness and

headache

Q126 Which organisms can help to

carry out Vermicomposting

कह९नसाजीववमीकनपह८नसटगममददकरताहहॴ

23-Jan-2017

Options

1) Nitrifying Bacteria

नाईटरीफाईगबहॴकटीररया 2) Earthworms

कहॴ चऐ

3) Algae

िहॴवि

4) Fungus

कवक

Correct Answer Earthworms

Q127 Contraction of heart is also

known as

हदयकहॳ सकचनकह८ mdash- भीकहाजाताहहॴ 23-Jan-

2017

Options

1) Systole

लससटह८ि

2) Aristotle

अरसत

3) Diastole

िायसटह८ि

4) Lub

मयब

Correct Answer Systole

Diastole is the part of the cardiac cycle

when the heart refills with blood

following systole (contraction)

Ventricular diastole is the period during

which the ventricles are filling and

relaxing while atrial diastole is the

period during which the atria are

relaxing

Q128 Azadirachta indica is the

botanical name of which of the

following

अजाटदराचताइडिकानननननलिखितमसहॳककसकावानसपनतनामहहॴ

23-Jan-2017

Options

1) Rose plant

गिाबकापह९धा 2) Apple tree

सहॳबकापहॳड़

3) Neem

नीम

4)Mango

आम

Correct Answer Neem

Q129 Which of the following is the

main end product of carbohydrate

digestion

नननननलिखितमसहॳकह९नसाकाबोहाइडरहॳटकहॳ पाचनकापरमिअतउतपादकहह८ताहहॴ 23-Jan-2017

Options

1) Fats

वसा 2) Lipids

लिपपडस

3) Glucose

गिकह८ज

4) Cellulose

F A C E B O O K

P A G E h t t p w w w f a c e b o o k c o m s s c m e n t o r s o f f i c i a l P a g e | 31

FOR MORE UPDATES AND MORE MATERIAL DO LIKE OUR FACEBOOK PAGE httpwwwfacebookcomsscmentorsofficial

सहॳमयिह८ज

Correct Answer Glucose

Intestinal absorption of end products

from digestion of carbohydrates and

proteins in the pig hellip During absorption some sugars (fructose or

galactose) released from the

corresponding sucrose and lactose

respectively during digestion were

partly metabolized into glucose by the

enterocyte

Q130 Which of the following glands is a

source of the enzyme Ptyalin

नननननलिखितगरचियोमसहॳएजाइमटयालिनकासरह८तहहॴ 23-Jan-2017

Options

1) Pancreas

अगरािय

2) Thyroid Gland

िाइराइिगरिी 3) Pituitary Gland

पीयषगरिी 4) Salivary Glands

िारगरचियाा Correct Answer Salivary Glands

Q131 Which of the following is not true

about Pteridophyta

ननननमसहॳकह९नसीबातटहॳररिह८फाईटकहॳ बारहॳमसचनहीहहॴ 23-Jan-2017

Options

1) Dominant phase is saprophytes

परमिचरणसहॳपरह८फाईइटसहह८ताहहॴ 2) Main plant body is diploid

पह९दह८कामखयिरीरदपवगखणतहह८ताहहॴ 3) Seeds are present

बीजमह९जदहह८तहॳहहॴ 4)Flowers are absent

फिअनपनसतिहह८तहॳहहॴ

Correct Answer Seeds are present

Q132 The largest dolphin species is the

orca also called as

िॉिकफनकीसबसहॳबड़ीपरजानतकाकानामआकायहहॴनजसहॳ mdash- भीकहतहॳहहॴ 23-Jan-2017

Options

1) Bottle Nose

बाटिनह८ज

2) Baiji

बहॳजी 3) Killer whale

ककिरहहॳि

4)Tucuxi

टकवसी Correct Answer Killer whale

Q133 The fat digesting enzyme Lipase

is secreted by which of the following

वसाकापाचनकरनहॳवािाएजाइमिाइपहॳजनननननलिखितमसहॳककसकहॳ दवारासतरापवतहह८ताहहॴ

24-Jan-2017

Options

1) Kidneys

गद

2) Pancreas

अगनयािय

3) Large Intestine

बड़ीआत

4)Liver

नजगर

Correct Answer Pancreas

Lipase is an enzyme that splits fats so

the intestines can absorb them Lipase

hydrolyzes fats like triglycerides into

their component fatty acid and glycerol

molecules It is found in the blood

gastric juices pancreatic secretions

intestinal juices and adipose tissues

F A C E B O O K

P A G E h t t p w w w f a c e b o o k c o m s s c m e n t o r s o f f i c i a l P a g e | 32

FOR MORE UPDATES AND MORE MATERIAL DO LIKE OUR FACEBOOK PAGE httpwwwfacebookcomsscmentorsofficial

Q134 The arrangement of leaves on an

axis or stem is called

एकअकषयातनहॳपरपनततयोकीयवसिाकह८कयाकहाजाताहहॴ SSC CHSL Science (biology) 2016

Question Paper

24-Jan-2017

Options

1) Phyllotaxy

फाइिह८टहॴकसी 2) Vernation

वनिन

3) Venation

वहॳनहॳिन

4)Phytotaxy

फाइटह८टहॴकसी Correct Answer Phyllotaxy

In botany phyllotaxis or phyllotaxy is

the arrangement of leaves on a plant

stem (from Ancient Greek phyacutellon

ldquoleafrdquo and taacutexis ldquoarrangementrdquo)

Phyllotactic spirals form a distinctive

class of patterns in nature

Q135 The study of Cells is also known

as

कह८लिकाओकहॳ अधययनकह८ mdashmdashndash

भीकहाजाताहहॴ 24-Jan-2017

Options

1) Cytology

सायटह८िह८जी 2) Physiology

कफनजयह८िह८जी 3) Nucleology

नयककमयह८िह८जी 4)Cellology

सहॳिह८िह८जी Correct Answer Cytology

Q136 Which of the following scientists

is also known as the Father of Biology

नननननलिखितमसहॳककसवहॴजञाननककह८ ldquoजीवपवजञानकहॳ जनकrdquoकहॳ नामसहॳभीजानाजाताहहॴ 24-Jan-2017

Options

1) Herbert Spencer

हबयटयसपसर

2) Aristotle

अरसत 3) Lamarck

िहॳमाकय 4)Darwin

िापवयन

Correct Answer Aristotle

Q137 Which cells give rise to various

organs of the plant and keep the plant

growing

कह९नसीकह८लिकाएपह९धह८कहॳ लभननअगह८कह८जनमदहॳतीहहॴऔरपह९धह८कह८बढ़नहॳममददकरतीहहॴ

24-Jan-2017

Options

1) Permanent

सिायी 2) Dermal

तवचीय

3) Meristematic

मररसटहॳमटटक

4)Mature

परह८ढ़

Correct Answer Meristematic

A meristem is the tissue in most plants

containing undifferentiated cells

(meristematic cells) found in zones of

the plant where growth can take place

Q138 Rodentia Muridae is the scientific

name of

F A C E B O O K

P A G E h t t p w w w f a c e b o o k c o m s s c m e n t o r s o f f i c i a l P a g e | 33

FOR MORE UPDATES AND MORE MATERIAL DO LIKE OUR FACEBOOK PAGE httpwwwfacebookcomsscmentorsofficial

रह८िहॳलियानयररिी mdashmdash- कावहॴजञाननकनामहहॴ 24-

Jan-2017

Options

1) Mouse

चहा 2) Squirrel

चगिहरी 3) Monkey

बदर

4) Lizard

नछपकिी Correct Answer Mouse

Q139 Name the scientist who proposed

the cell theory

कह८लिकालसदातकापरसतावदहॳनहॳवािहॳवहॴजञाननककानामबताइए 24-Jan-2017

Options

1) Schleiden and Schwann

िीमिनऔरशरववान

2) Lamarck

िहॳमाकय 3) Treviranus

टरहॳवायरहॳनस

4)Whittaker and Stanley

हीटकरऔरसटहॳनिहॳ Correct Answer Schleiden and

Schwann

Q140 The flower with the worldrsquos

largest bloom is

दननयाकासबसहॳबड़ाफिखििनहॳवािा mdashmdashndash हहॴ 24-Jan-2017

Options

1) Pando

पािह८ 2) Posidonia

पह८सीिह८ननया 3) Rafflesia arnoldii

ररफिहॳलियाअनोमिी 4)Helianthus annuus

हहॳलिएनिसएनयअस

Correct Answer Rafflesia arnoldii

Rafflesia arnoldii is a species of

flowering plant in the parasitic genus

Rafflesia It is noted for producing the

largest individual flower on earth It has

a very strong and horrible odour of

decaying flesh earning it the nickname

ldquocorpse flower

Q141 Deficiency of which vitamin

causes night blindness

ककसपवटालमनकीकमीकहॳ कारणरतौधीहह८ताहहॴ 24-Jan-2017

Options

1) Vitamin K

पवटालमन K

2) Vitamin C

पवटालमन C

3) Vitamin B1

पवटालमन B1

4)Vitamin A

पवटालमन A

Correct Answer Vitamin A

Q142 Nongreen plants lack which of the

following

गहॴर-

हररतवनसपनतमनननननलिखितमसहॳककसकीकमीहह८तीहहॴ

24-Jan-2017

Options

1) Chlorophyll

किह८रह८कफि

2) Lycophyll

िायकह८कफि

3) Cyanophyll

F A C E B O O K

P A G E h t t p w w w f a c e b o o k c o m s s c m e n t o r s o f f i c i a l P a g e | 34

FOR MORE UPDATES AND MORE MATERIAL DO LIKE OUR FACEBOOK PAGE httpwwwfacebookcomsscmentorsofficial

सायनह८कफि

4)Phototropism

फह८टह८टरोपपजम

Correct Answer Chlorophyll

Q143 Organisms that use light to

prepare food are known as

जह८जीवपरकािकाउपयह८गकरभह८जनतहॴयारकरतहॳहहॴ उनह mdashmdash- कहॳ पमजानजाताहहॴ 24-Jan-2017

Options

1) Autotrophs

सवपह८षी 2) Heterotrophs

पवषमपह८षज

3) Omnivores

सवायहारी 4)Decomposers

पवघटनकरनहॳवािा Correct Answer Autotrophs

autotrophs often make their own food

by using sunlight carbon dioxide and

water to form sugars which they can use

for energy Some examples of

autotrophs include plants algae and

even some bacteria Autotrophs

(producer) are important because they

are a food source for heterotrophs

(consumers)

A heterotroph is an organism that

ingests or absorbs organic carbon

(rather than fix carbon from inorganic

sources such as carbon dioxide) in order

to be able to produce energy and

synthesize compounds to maintain its

life Ninety-five percent or more of all

types of living organisms are

heterotrophic including all animals and

fungi and some bacteria

Q144 Which of the following is a

primary function of haemoglobin

नननननलिखितमसहॳकह९नसाटहमह८गिह८बबनकाएकपरािलमककाययहहॴ

25-Jan-2017

Options

1) Utilization of energy

उजायकाउपयह८गकरना 2) Prevention of anaemia

रकतामपताहह८नहॳसहॳरह८कना 3) Destruction of bacteria

बहॴकटीररयाकापवनािकरना 4) To transport oxygen

ऑकसीजनकावहनकरना Correct Answer To transport oxygen

Q145 Vascular bundles are absent in

सवहनीबिि mdashmdash- मअनपनसतिरहतहॳहहॴ 25-Jan-2017

Options

1) Bryophyta

िायह८फाइटा 2) Pteridophyta

टहॳररिह८फाईटा 3) Gymnosperms

नजननह८सपमय 4) Angiosperms

एननजयह८सपहॳनसय Correct Answer Bryophyta

Q146 Sauria Lacertidae is the scientific

name of

सहॴररयािहॳसरटाईिी mdashmdashndash कावहॴजञाननकनामहहॴ 25-Jan-2017

Options

1) Crocodile

मगरमचछ

2) Hippopotamus

टहपपह८पह८टहॳमस

3) Lizard

नछपकिी 4) House fly

F A C E B O O K

P A G E h t t p w w w f a c e b o o k c o m s s c m e n t o r s o f f i c i a l P a g e | 35

FOR MORE UPDATES AND MORE MATERIAL DO LIKE OUR FACEBOOK PAGE httpwwwfacebookcomsscmentorsofficial

घरहॳिमकिी Correct Answer Lizard

Q147 Which type of pathogen causes

the water-borne disease SARS (Severe

Acute Respiratory Syndrome)

ककसपरकािकारह८गज़नकजिजननतबीमारीसासयकाकारणबनताहहॴ 25-Jan-2017

Options

1) Viral

वायरि

2) Parasitic

परजीवी 3) Protozoan

परह८टह८जअन

4) Bacterial

बहॴकटीररयि

Correct Answer Viral

Q148 Which of the following organs

produces the enzyme lipase

नननननलिखितमसहॳकह९नसाअगिायपहॳजएजाइमउतपननकरताहहॴ 25-Jan-2017

Options

1) Pancreas

अगनयािय

2) Large Intestine

बड़ीआत

3) Liver

नजगर

4) Small Intestine

छह८टीआत

Correct Answer Pancreas

Q149 A is a long internode forming the

basal part or the whole of a peduncle

एक mdashmdash- एकिबाइटरनह८िहहॴ जह८ननचिाटहससायासनपणयिठिबनताहहॴ 25-

Jan-2017

Options

1) Rhizome

परकद

2) Rachis

महॳ दि

3) floral axis

पषपअकष

4) Scape

भगदड़

Correct Answer scape

Q150 ndash Which of the following

organisms are considered to be both

Living and Non-living

नननननलिखितमसहॳकह९नसहॳजीवाणकह८जीपवतऔरअजीपवतमानाजाताहहॴ

25-Jan-2017

Options

1) Bacteria

बहॴकटीररया 2) Fungi

कवक

3) Algae

िहॴवाि

4)Virus

वायरस

Correct Answer Virus

They are considered to be living as they

possess a protein coat as a protective

covering DNA as the genetic material

etc

They are said to be non-living as they

can be crystallised and they survive for

billions of years They can tolerate high

temperatures freezing cold

temperatures ultra-violet radiations etc

Q151 Deficiency of fluorine causes

which of the following

फिह८ररनकीकमीकहॳ कारणनननननलिखितमसहॳकयाहह८ताहहॴ

F A C E B O O K

P A G E h t t p w w w f a c e b o o k c o m s s c m e n t o r s o f f i c i a l P a g e | 36

FOR MORE UPDATES AND MORE MATERIAL DO LIKE OUR FACEBOOK PAGE httpwwwfacebookcomsscmentorsofficial

27-Jan-2017

Options

1) Dental Caries

िटिकहॴ ररज

2) Scurvy

सकवरी 3) Anaemia

रकतामपता 4) Arthritis

गटठया Correct Answer Dental Caries

Q152 In a Punnett Square with the

cross AaBb x AaBb how many Aabb

genotypes would be created

पनहॳटसककायरमिह८स AaBb x AaBb कहॳ साि

ककतनहॳ Aabb जीनह८टाइपबनगहॳ 27-Jan-2017

Options

1) 1

2) 8

3) 2

4) 3

Correct Answer 2

Q153 Which of the following is the

Controlling Center of the Cell

नननननलिखित म सहॳ कह८लिकाका ननयतरण

क दर कह९न हहॴ

27-Jan-2017

Options

1) Nucleus

क दर

2) Plasma

पिाजमा 3) Lysosome

िायसह८सह८म

4) Chromosome

िह८मह८सह८म

Correct Answer Nucleus

The control centre of the cell is the

nucleus in eukaryotic cells The nucleus

contains genetic material in the form of

DNA

Q154 Myopia affects which of the

following organs

मायह८पपयानननननलिखितअगह८मसहॳककसहॳपरभापवतकरताहहॴ

25-Jan-2017

Options

1) Heart

हदय

2) Skin

तवचा 3) Eyes

आािहॳ 4)Mouth

मह

Correct Answer Eyes

Q155 Which of the following bears

flowers

नननननलिखितमसहॳकह९नफिधारणकरताहहॴ

25-Jan-2017

Options

1) Bryophyta

िायह८फाइटा 2) Pteridophyta

टहॳरीिह८फाईटा 3) Gymnosperms

नजननह८सपमय 4)Angiosperms

एननजयह८सपमय Correct Answer Angiosperms

Q156 Oxygenated blood flows out of the

heart through the

ऑकसीजनयकतरकत mdashmdashmdash

कहॳ माधयमसहॳहदयकहॳ बाहरबहताहहॴ 25-Jan-2017

F A C E B O O K

P A G E h t t p w w w f a c e b o o k c o m s s c m e n t o r s o f f i c i a l P a g e | 37

FOR MORE UPDATES AND MORE MATERIAL DO LIKE OUR FACEBOOK PAGE httpwwwfacebookcomsscmentorsofficial

Options

1) Aorta

महाधमनी 2) pulmonary artery

फहॳ फड़हॳकीधमनी 3) vena cava

वहॳनाकावा 4)Atrium

चह९क

Correct Answer aorta

Q157 Blood leaving the liver and

moving towards the

heart has a higher concentration of

नजगरसहॳननकिकरहदयकीतरफजानहॳवािहॳरकतम mdashmdashmdashmdash कीउचचसादरताहह८तीहहॴ 27-Jan-2017

Options

1) Lipids

लिपपडस

2) Urea

यररया 3) Bile Pigments

पपततकहॳ रगकरण

4) Carbon dioxide

काबयनिायऑकसाइि

Correct Answer Bile Pigments

Urea is nitrogen containing substance

which is produced in the liver in order

to deal with excess amino-acids in the

body As urea is produced it leaves the

liver in the blood stream and passes via

the circulatory system to all parts of the

body

Q158 Bulb is a modification of which

part of a plant

बमबएकपह९धहॳकहॳ ककसटहससहॳकाएक पातरणहह८ताहहॴ 27-Jan-2017

Options

1) The root

जड़

2) The stem

तना 3) The radicle

मिाकर

4)The fruit

फि

Correct Answer The stem

Q159 Which of the following carries

blood away from the heart to different

body parts

इनमहॳसहॳकह९नरकतकह८हदयसहॳिरीरकहॳ पवलभननअगह८तकिहॳजातीहहॴ

27-Jan-2017

Options

1) Arteries

धमननया 2) Nerves

तबतरहाए

3) Capillaries

कहॳ लिकाए

4)Veins

नसहॳ Correct Answer Arteries

Q160 The series of processes by which

nitrogen and its compounds are

interconverted in the environment and

in living organisms is called

27-Jan-2017

Options

1)Absorption of Nitrogen

2)Ammonification

3)Nitrogen Fixation

4)Nitrogen Cycle

Correct Answer Nitrogen Cycle

Ammonification or Mineralization is

performed by bacteria to convert

organic nitrogen to ammonia

F A C E B O O K

P A G E h t t p w w w f a c e b o o k c o m s s c m e n t o r s o f f i c i a l P a g e | 38

FOR MORE UPDATES AND MORE MATERIAL DO LIKE OUR FACEBOOK PAGE httpwwwfacebookcomsscmentorsofficial

Nitrification can then occur to convert

the ammonium to nitrite and nitrate

Nitrogen fixation is a process by which

nitrogen in the Earthrsquos atmosphere is

converted into ammonia (NH3) or other

molecules available to living organisms

Q161 BCG vaccine is given to protect

from which of the following

बीसीजीकाटटकानननननलिखितमसहॳककसकहॳ बचावकहॳ लिएटदयाजातहहॴ

27-Jan-2017

Options

1) Jaundice

पीलिया 2) Anaemia

रकतमपता 3) Tuberculosis

कषयरह८ग

4) Polio

पह८लियह८ Correct Answer Tuberculosis

Q162 Parallel venation is found in

समानतरवहॳनहॳिन mdashmdashmdash- मपायाजाताहहॴ 27-Jan-2017

Options

1) plants which are monocots

पह९धहॳजह८एकबीजपतरीहह८तहॳहहॴ 2) plants which have a dicot stem

वहॳपह९धहॳनजनकातनादपवदलियहह८ताहहॴ 3) plants with leaves similar to Tulsi

वहॳपह९धहॳनजनकीपनततयतिसीकीपनततयोकहॳ समानहह८तहॳहहॴ 4)plants with tap roots

टहॳप टवािहॳपह९धहॳ Correct Answer plants which are

monocots

Q163 The hardest part of the body is

िरीरकासबसहॳकठह८रभाग mdashndash हहॴ 27-Jan-2017

Options

1) Bones

हडडिय

2) Tooth Enamel

दातकहॳ इनहॳमि

3) Skull

िह८पड़ी 4) Spinal Cord

महॳ रजज

Correct Answer Tooth Enamel

Q164 Which type of pathogen causes

the waterborne disease E coli Infection

ककसपरकारकारह८गजननकजिजननतरह८गईकह८िाईसिमणकाकारणबनताहहॴ 27-Jan-2017

Options

1) Protozoan

परह८टह८जआ

2) Parasitic

परजीवी 3) Bacterial

बहॴकटीररयि

4)Viral

वायरि

Correct Answer Bacterial

Q165 The amount of blood filtered

together by both the kidneys in a 70 kg

adult male human in a minute is

70 की गरा वािहॳएकवयसकप षमएकलमनटमदह८नोगदकहॳदवाराएकसािचाबनीगयीरकतकीमातरहह८तीहहॴ 29-Jan-2017

Options

1) 1100 ml

1100 लमलि

2) 100 ml

F A C E B O O K

P A G E h t t p w w w f a c e b o o k c o m s s c m e n t o r s o f f i c i a l P a g e | 39

FOR MORE UPDATES AND MORE MATERIAL DO LIKE OUR FACEBOOK PAGE httpwwwfacebookcomsscmentorsofficial

100 लमलि

3) 1500 ml

1500 लमलि

4) 500 ml

500 लमलि

Correct Answer 1100 ml

Q166 Which feature of a plant helps to

distinguish a monocot from a dicot

पह९धहॳकीवहकह९नसीपविहॳषताहहॴजह८एकदपवदलियहॳऔरएकएकदिीयपह९धहॳसहॳभहॳदकरनहॳममददकरतीहहॴ 29-Jan-2017

Options

1) Pollination

परागम

2) Venation

वहॳनहॳिन

3) Vernation

वनिन

4) Aestivation

एसटीवहॳिहॳन

Correct Answer venation

Q167 The Mutation Theory was

proposed by

उतवररवतयनकालसदात mdashmdashndash

कहॳ दवरापरसतापवतककयाजाताहहॴ 29-Jan-2017

Options

1) Charles Lyell

चामसयलियहॳि

2) William Smith

पवलियमनसमि

3) Hugo De Vries

हयगह८िीराईस

4)Harrison Schmitt

हहॳरीसननसमट

Correct Answer Hugo De Vries

Q168 Which type of pathogen causes

the waterborne disease HepatitisA

ककसपरकारकहॳ रह८गजनकजिजननतरह८गहहॳपहॳटाइटटस-A काकारणबनताहहॴ

29-Jan-2017

Options

1) Parasitic

परजीवी 2) Viral

वायरि

3) Protozoan

परह८टह८जआ

4) Bacterial

बहॴकटीररयि

Correct Answer Viral

Q169 In a Punnett Square with the

cross AaBb x Aabb how many AaBb

genotypes would be created

पनहॳटसकवायरमिह८स AaBb x Aabb

कहॳ सािककतनहॳ AaBb जीनह८टाइपबनगहॳ 29-Jan-

2017

Options

1) 4

2) 1

3) 7

4) 6

Correct Answer 4

Q170 Arboreal Ateles is the scientific

name of

अिह८ररयिएटटलिस mdashmdashmdash कावहॴजञाननकनामहहॴ 29-Jan-2017

Options

1) Squirrel

चगिहरी 2) Sparrow

गह८रहॴया 3) Lizard

नछपकिी 4) Spider monkey

F A C E B O O K

P A G E h t t p w w w f a c e b o o k c o m s s c m e n t o r s o f f i c i a l P a g e | 40

FOR MORE UPDATES AND MORE MATERIAL DO LIKE OUR FACEBOOK PAGE httpwwwfacebookcomsscmentorsofficial

मकड़ीबदर

Correct Answer Spider monkey

Q171 Which type of pathogen causes

the waterborne disease Salmonellosis

ककसपरकारकारह८गाणजिजननतबीमारीसािमह८नहॳिह८लसज़काकारकहहॴ

29-Jan-2017

Options

1) Algal

िहॳवालियहॳ 2) Parasitic

परजीवी 3) Bacterial

बहॴकटीररयि

4)Viral

वायरि

Correct Answer Bacterial

An infection with salmonella bacteria

commonly caused by contaminated food

or water

Symptoms include diarrhoea fever

chills and abdominal pain

Q172 is a condition in which there is a

deficiency of red cells or of haemoglobin

in the blood

mdashmdash-

एकनसिनतहहॴनजसमहॳरकतमिािकह८लिकाओकीयाहीमह८गिह८बबनकीकमीहह८तीहहॴ 29-Jan-2017

Options

1) Albinism

एनमबननजम

2) Propyria

परह८पीररया 3) Anaemia

एनीलमया 4)Keloid disorder

कहॳ िह८इिडिसओिर

Correct Answer Anaemia

Q173 Ananas comosus is the scientific

name of

Options

अनानासकह८मह८सस mdashmdashmdashndash

कावहॴजञाननकनामहहॴ 29-Jan-2017

1) Custard Apple

सीताफि

2) Pineapple

पाइनएपपि

3) Bamboo

बास

4)Pomegranate

अनार

Correct Answer Pineapple

Q174 Which organ produces insulin

कह९नसाअगइनसलिनपहॴदाकरताहहॴ 29-Jan-

2017

Options

1) Liver

यकत

2) Thyroid gland

िायराइिगरिी 3) Spleen

पिीहा 4)Pancreas

अगरयिय

Correct Answer Pancreas

Q175 Which of the following disease is

not caused by water pollution

नननननलिखितमसहॳकह९नसारह८गपानीकहॳ परदषणकहॳकारणनहीहह८ता

29-Jan-2017

Options

1) Cholera

हहॴजा 2) Typhoid

F A C E B O O K

P A G E h t t p w w w f a c e b o o k c o m s s c m e n t o r s o f f i c i a l P a g e | 41

FOR MORE UPDATES AND MORE MATERIAL DO LIKE OUR FACEBOOK PAGE httpwwwfacebookcomsscmentorsofficial

टाइफाइि

3) Asthma

दमा 4)Diarrhoea

दसत

Correct Answer Asthma

Q176 Ocimum tenuiflorum is the

scientific name of

ओलिलममटहॳयईफिह८रमइसकावहॴजञाननकनाम mdash

ndash हहॴ 30-Jan-2017

Options

1) Neem

नीम

2) Mango

आम

3) Babul

बबि

4)Tulsi

तिसी Correct Answer Tulsi

Q177 Which gland secretes bile a

digestive fluid

कह९नसीगरिीपपतत एकपाचनतरिपरदािय सरापवतकरतीहहॴ 30-Jan-2017

Options

1) Pancreas

अगनयािय

2) Liver

यकत

3) Thyroid

िायराइि

4) Testes

टहॳनसटस

Correct Answer liver

Q178 In which of the following the

dominant phase is Gametophyte

नननननलिखितमसहॳककसकहॳ परमिचरणयगमकह८दपवधद (Gametophyte)हहॴ 30-Jan-2017

Options

1) Bryophyta

िायह८फाइटा 2) Pteridophyta

टहॳररिह८फाइटा 3) Gymnosperms

नजननह८सपमय 4) Angiosperms

एननजयह८सपमय Correct Answer Bryophyta

Q179 Anaerobic respiration refers to

which of the following

नननननलिखितमसहॳककसहॳअवायवीयशवसनकहाजाताहहॴ

30-Jan-2017

Options

1) Respiration without Oxygen

ऑकसीजनकहॳ बबनाशवसन

2) Respiration with Oxygen

ऑकसीजनकहॳ सािशवसन

3) Respiration without CO2

काबयनिायऑकसाइिकहॳ बबनाशवसन

4) Respiration with CO2

काबयनिायऑकसाइिकहॳ सािशविन

Correct Answer Respiration without

Oxygen

Q180 Which type of pathogen causes

the waterborne disease Cholera

ककसपरकारकारह८गजनकजिजननतरह८गहहॴजाकाकारणबनताहहॴ

30-Jan-2017

Options

1) Algal

िहॴवालियहॳ

F A C E B O O K

P A G E h t t p w w w f a c e b o o k c o m s s c m e n t o r s o f f i c i a l P a g e | 42

FOR MORE UPDATES AND MORE MATERIAL DO LIKE OUR FACEBOOK PAGE httpwwwfacebookcomsscmentorsofficial

2) Bacterial

बहॴकटीररयि

3) Protozoan

परह८टह८जआ

4) Viral

वायरि

Correct Answer Bacterial

Q181 To which class does

Oxyreductases transferases hydrolases

belong

ओकसीररिकटहॳसटरासफरहॳजहॳस

हाइडरह८िहॳसहॳसककसवगयमआतहॳहहॴ 30-Jan-2017

Options

1) Hormones

हारमोस

2) Enzymes

एजाइनस

3) Proteins

परह८टीनस

4) Vitamins

पवटालमनस

Correct Answer Enzymes

Q182 Which of the following is not true

about Gymnosperms

ननननमसहॳकह९नसीबातअनावतबीजीकहॳ बारहॳमसचनहीहहॴ 30-Jan-2017

Options

1) Dominant phase is saprophytes

परमिचरणसहॳपरह८फाइटसहह८ताहहॴ 2) Vascular bundles are absent

सवहनीबििअनपनसितहह८ताहहॴ 3) spores are heterospores

बीजाणहहॳटहॳरह८सपह८रसहह८तहॳहहॴ 4) Flowers are absent

फिअनपनसितहह८तहॳहहॴ

Correct Answer Vascular bundles are

absent

Q183 The name of first mammal clone sheep is

भहॳड़कीपरिमसतनपायीपरनत प (किह८न)

कानामहहॴ 30-Jan-2017

Options

1) Noori

नरी 2) Dolly

िॉिी 3) Louise

िसी 4)Durga

दगाय Correct Answer Dolly

Q184 Which type of pathogen causes

the water-borne disease Typhoid fever

ककसपरकारकारह८गजनकजिजननतरह८गटाइफाइिबिारकाकारणबनताहहॴ 30-Jan-2017

Options

1) Algal

िहॴवािीय

2) Parasitic

परजीवी 3) Protozoan

परह८टह८जनअन

4)Bacterial

बहॴकटीररयि

Correct Answer Bacterial

Q185 In which part of the cell are

proteins made

कह८लिकाकहॳ ककसटहससहॳमपरह८टीनबनायाजाताहहॴ

31-Jan-2017

Options

1) Reticulum

रहॳटटकिम

F A C E B O O K

P A G E h t t p w w w f a c e b o o k c o m s s c m e n t o r s o f f i c i a l P a g e | 43

FOR MORE UPDATES AND MORE MATERIAL DO LIKE OUR FACEBOOK PAGE httpwwwfacebookcomsscmentorsofficial

2) Golgi apparatus

गह८मजीएपहॳरहॳटस

3) Ribosomes

ररबह८सह८नस

4) Lysosome

िायसह८सह८नस

Correct Answer ribosomes

Proteins are produced by stringing

amino acids together in the order

specified by messenger RNA strands

that were transcribed from DNA in the

cell nucleus The process of synthesizing

a protein is called translation and it

occurs on ribosomes in the cytoplasm of

a cell

Q186 Polio is a disease caused by which

of the following

नननननलिखितमसहॳपह८लियह८कीबबमारह८हह८नहॳकाकारणकयाहहॴ

31-Jan-2017

Options

1) Bacteria

बहॴकटीररयि

2) Mosquito

मचछर

3) Virus

वायरस

4) Cockroach

नतिच हॳ Correct Answer Virus

Polio or poliomyelitis is a crippling and

potentially deadly infectious disease It

is caused by the poliovirus

Q187 ndash Hay fever is a sign of which of

the following

हहॳकफवरनननननलिखितमसहॳककसकाएकसकहॳ तहहॴ

31-Jan-2017

Options

1) Old Age

वदावसिा 2) Malnutrition

कपह८सण

3) Allergy

एिनजय 4) Over Work

अतयचधककाययकरना Correct Answer Allergy

Q188 How many chromosomes does a

human cell contain

एकमानवकह८लिकामककतनहॳगणसतरहह८तहॳहहॴ

29-Jan-2017

Options

1) 6

2) 26

3) 46

4) 66

Correct Answer 46

In humans each cell normally contains

23 pairs of chromosomes for a total of

46 Twenty-two of these pairs called

autosomes look the same in both males

and females The 23rd pair the sex

chromosomes differ between males and

females

Q189 Which of the following is not true

about Bryophyta

ननननमसहॳकह९नसीबातिायह८फाइटकहॳ बारहॳमसचनहीहहॴ 31-Jan-2017

Options

1) Dominant phase is gametophytes

परमिचरणगहॳलमतह८फाइटसहह८ताहहॴ 2) Main plant body is haploid

पह९धहॳकामखयिरीरअगखणतहह८ताहहॴ 3) Spores are homospores

बीजाणहह८मह८सफह८रसहह८तहॳहहॴ 4) Flowers are present

फिमह८जदहह८तहॳहहॴ Correct Answer Flowers are present

F A C E B O O K

P A G E h t t p w w w f a c e b o o k c o m s s c m e n t o r s o f f i c i a l P a g e | 44

FOR MORE UPDATES AND MORE MATERIAL DO LIKE OUR FACEBOOK PAGE httpwwwfacebookcomsscmentorsofficial

Q190 Which aquatic animal has

trailing tentacles

ककसजिीयजानवरकहॳ पीछहॳचिनहॳवािहॳटहॳटकिसहह८तहॳहहॴ

31-Jan-2017

Options

1) Sea horse

समदरीघह८िा 2) Corals

मगा 3) Jelly fish

जहॳिीमछिी 4) Star fish

तारामछिी Correct Answer Jelly fish

Jellyfish with its umbrella-shaped bell

and trailing tentacles

Q191 Which type of pathogen causes

the water-borne disease Poliomyelitis

(Polio)

ककसपरकारकारह८गजनकजिजननतरह८गपह८लियह८मायहॳटटस (पह८लियह८) काकारणहहॴ 31-Jan-

2017

Options

1) Parasitic

परजीवी 2) Algal

िहॴवालिय

3) Viral

वायरि

4) Bacterial

बहॴकटीररयि

Correct Answer Viral

Q192 The outer white part of the eye

that protects the inner structures is

आािकाबाहरीसफहॳ दटहससाजह८आतररकसरचनाओकीरकषाकरताहहॴ वह mdashmdashmdash हहॴ 31-Jan-

2017

Options

1) Iris

आयररस

2) Sclera

सकिहॳरा 3) Retina

रहॳटटना 4) Cornea

कह८ननयया Correct Answer Sclera

Q193 Proteins are made up of

परह८टीनकाननमायण mdashndash सहॳहह८ताहहॴ 31-Jan-2017

Options

1) Amino acids

एलमनह८अनि

2) Fatty acids

वसायकतअनि

3) Glucose

गिकह८ज

4)Nucleotides

नयनकियह८टाईिस

Correct Answer Amino acids

Q194 Moringa Oleifera is the scientific

name of

मह८ररगओलिफहॳ रा mdashmdashndash कावहॴजञाननकनामहहॴ 31-Jan-2017

Options

1) Banyan

बरगद

2) Gulmohar

गिमह८हर

3) Amla

आमिा

F A C E B O O K

P A G E h t t p w w w f a c e b o o k c o m s s c m e n t o r s o f f i c i a l P a g e | 45

FOR MORE UPDATES AND MORE MATERIAL DO LIKE OUR FACEBOOK PAGE httpwwwfacebookcomsscmentorsofficial

4) Drumstick

डरमनसटक

Correct Answer Drumstick

Q195 Kidney stones are composed of

गदकीपिरी mdashndash सहॳबनीहह८तीहहॴ 1-Feb-2017

Options

1) Calcium Oxalate

कहॴ नमसयमओकजहॳिहॳट

2) Sodium Chloride

सह८डियमकिह८राइि

3) Magnesium Nitrate

महॳनगनलियमनाइतटरहॳट

4) Calcium Bicarbonate

कहॴ नमियमबायकबोनहॳट

Correct Answer Calcium Oxalate

Q196 ndash Which of the following is not

true about Angiosperms

ननननमसहॳकह९नसीबातआवतबीजीकहॳ बारहॳमसचनहीहहॴ 1-Feb-2017

Options

1) Dominant phase is gametophytes

परमिचरणगहॳलमतह८फाइटहह८ताहहॴ 2) Vascular bundles are present

सवहनीबििमह९जदहह८ताहहॴ 3) Spores are heterospores

बीजाणहहॳटहॳरह८सपह८रसहह८तहॳहहॴ 4) Seeds are covered

बीजढकहॳ हह८तहॳहहॴ Correct Answer Dominant phase is

gametophytes

Q197 All of the following are excretory

(waste) products of animals except

नननननलिखितमसहॳककसएककह८छह८ड़करअनयसभीपराखणयोदवाराउतसनजयतपदाियहहॴ 1-Feb-

2017

Options

1) Uric Acid

यररकएलसि

2) Ammonia

अमह८ननया 3) Carbohydrates

काबोहाइडरहॳट

4) Urea

यररया Correct Answer Carbohydrates

In animals the main excretory products

are carbon dioxide ammonia (in

ammoniotelics) urea (in ureotelics) uric

acid (in uricotelics) guanine (in

Arachnida) and creatine

Q198 RNA is a polymeric molecule

What does RNA stand for

आरएनइएएकबहिकआणहहॴ इसकाकापवय पकयाहहॴ 1-Feb-2017

Options

1) Rado Nuclear Acid

रािह८नयनकियरएलसि

2) Ribo Nucleic Acid

राइबह८नयनकिकएलसि

3) Rhino Nuclear Acid

हाइनह८नयनकियरएलसि

4) Resto Nucleus Acid

रहॳसटह८नयकिीयसएलसि

Correct Answer Ribo Nucleic Acid

Q199 Which organ does detoxification

and produces chemicals needed for

digestion

कह९नसाअगपवषहरणकरताहहॴऔरपाचनकहॳ लिएआवशयकरसायनोकह८पहॴदाकरताहहॴ 1-Feb-

2017

Options

1) Salivary glands

िारगरचिया 2) Pancreas

अगनयािय

F A C E B O O K

P A G E h t t p w w w f a c e b o o k c o m s s c m e n t o r s o f f i c i a l P a g e | 46

FOR MORE UPDATES AND MORE MATERIAL DO LIKE OUR FACEBOOK PAGE httpwwwfacebookcomsscmentorsofficial

3) Thyroid gland

िायराइिगरिी 4) Liver

यकत

Correct Answer Liver

Q200 Psidium guajava is the scientific

name of

लसडियमगआजावा mdashmdash कावहॴजञाननकनामहहॴ 1-

Feb-2017

Options

1) Guava

अम द

2) Mango

आम

3) Bamboo

बास

4) Jack fruit

कटहि

Correct Answer Guava

Q201 Which drug is used as a Blood

Thinner

चधरकह८पतिाकरनहॳकहॳ पमककसदवाकापरयह८गककयाजाताहहॴ

1-Feb-2017

Options

1) Warfarin

वाफर न

2) Tramadol

टरहॳमािह८ि

3) Azithromycin

एनजरह८मायलसन

4) Hydralazine

हाइडरह८िहॳनजन

Correct Answer Warfarin

Q202 Which of the following disease is

caused due to the deficiency of protein

परह८टीनकीकमीकहॳ कारणनननननलिखितमसहॳकह९नसारह८गहह८ताहहॴ 1-Feb-2017

Options

1) Arthritis

गटठया 2) Kwashiorkor

कािीओकय र

3) Goitre

गाइटर

4) Night Blindness

रतह९चध

Correct Answer Kwashiorkor

Q203 A is species of plant that has

adapted to survive in an environment

with little liquid water

mdashmdashndashपह९धहॳकीएकऐसहॳऐसहॳपरजानतहहॴ नजसनहॳकमपानीवािहॳवातावरणमजीपवतरहनहॳकहॳलिएअनकिनहहॴ 1-Feb-2017

Options

1) Xerophyte

म दपवद

2) Hydrophyte

जिीयपादप

3) Mesophyte

समह८दपवद

4) Thallophyte

िहॴिह८फाइटा Correct Answer xerophyte

xerophyte is a species of plant that has

adapted to survive in an environment

with little liquid water such as a desert

or an ice- or snow-covered region in the

Alps or the Arctic

Mesophytes are terrestrial plants which

are adapted to neither a particularly

dry nor particularly wet environment

An example of a mesophytic habitat

would be a rural temperate meadow

F A C E B O O K

P A G E h t t p w w w f a c e b o o k c o m s s c m e n t o r s o f f i c i a l P a g e | 47

FOR MORE UPDATES AND MORE MATERIAL DO LIKE OUR FACEBOOK PAGE httpwwwfacebookcomsscmentorsofficial

which might contain goldenrod clover

oxeye daisy and Rosa multiflora

thallophyte any of a group of plants or

plantlike organisms (such as algae and

fungi) that lack differentiated stems

leaves and roots and that were formerly

classified as a primary division

(Thallophyta) of the plant kingdom

Q204 How many types of teeth are

there in humans

मनषयोमककतनहॳपरकारकहॳ दातहह८तहॳहहॴ

1-Feb-2017

Options

1) 4

2) 5

3) 2

4) 3

Correct Answer 4

teeth -Humans have four types of

teethincisors canines premolars and

molars each with a specific function

The incisors cut the food the canines

tear the food and the molars and

premolars crush the food

Q205 Carica papaya is the scientific name of

कहॴ ररकापपाया mdashmdashndash कावहॴजञाननकनामहहॴ 2-

Feb-2017

Options

1) Peepal

पीपि

2) Papaya

पपीता 3) Tamarind

इमिी 4) Drumstick

ढह८िकाछड़ी Correct Answer Papaya

Q206 Muscles get tired when there is

shortfall of

जब mdashndash कीकमीहह८तीहहॴतबपहॳिीयिकजातीहहॴ 2-Feb-2017

Options

1) Lactic acid

िहॴनकटकएलसि

2) Na+ ions

Na+ आयन

3) ATP

एटीपी 4) Sulphates

समफहॳ टस

Correct Answer ATP

ATP is the energy source muscle fibers

use to make muscles contract

muscle tissuersquos main source of energy

called adenosine triphosphate or ATP

As your muscles use up this energy

source they become tired and fatigued

Oxygen is the key ingredient that helps

create new ATP to replenish the burned

up ATP in your muscles

Q207 Artocarpus integra is the

scientific name of आटह८कापयसइटीगरा mdashmdashmdash कावहॴजञाननकनामहहॴ 2-Feb-2017

Options

1) Guava

अम द

2) Pineapple

अनानास

3) Silver Oak

लसमवरओक

4) Jack fruit

कटहि

Correct Answer Jack fruit

Q208 Which organ stores fat soluble

vitamins

कह९नसाअगवसामघिनिीिपवटालमनह८काभिाराकरताहहॴ

2-Feb-2017

F A C E B O O K

P A G E h t t p w w w f a c e b o o k c o m s s c m e n t o r s o f f i c i a l P a g e | 48

FOR MORE UPDATES AND MORE MATERIAL DO LIKE OUR FACEBOOK PAGE httpwwwfacebookcomsscmentorsofficial

Options

1) Blood

रकत

2) Skin

तवचा 3) Liver

यकत

4) Pancreas

अगनयािय

Correct Answer Liver

Q209 Which disease is caused due to

deficiency of Iodine

आयह८िीनकहॳ कारणकह९नसारह८गहह८ताहहॴ 2-Feb-2017

Options

1) Rickets

ररकहॳ टस

2) Scurvy

सकवी 3) Goitre

गणमािा 4) Growth retardation

पवकासका कना Correct Answer Goitre

rickets A softening and weakening of

bones in children usually due to

inadequate vitamin D

Q210 Grevillea Robusta is the scientific name of

गरहॳपवलियारह८बसटा mdashmdashmdash- कापवजञाननकनामहहॴ 2-Feb-2017

Options

1) Peepal

पीपि

2) Teak

सागह९न

3) Silver Oak

लसमवरओक

4) Jack fruit

कटहि

Correct Answer Silver Oak

Q211 When a Cuttlefish is described as a Molluscs it is at which level of

classification

जबएककटिकफिकह८एकमह८िसकाकहॳ पमवखणयतककयाजाताहहॴतबयहॳवगीकरणकहॳ ककससतरपहॳनसितहहॴ 2-Feb-2017

Options

1) Class

वगय 2) Order

िम

3) Family

पररवार

4) Phylum

सघ

Correct Answer Phylum

Q212 Bambusa dendrocalmus is the

scientific name of बानबसािहॳडराकामस mdashmdashmdash कावहॴजञाननकनामहहॴ 3-Feb-2017

Options

1) Banyan

बरगद

2) Papaya

पपीता 3) Bamboo

बास

4) Pomegranate

अनार

Correct Answer Bamboo

Q213 Acinonyx Jubatus is the scientific name of

एलसनह८ननकसजयबहॳटस mdashmdashmdash

कावहॴजञाननकनामहहॴ 3-Feb-2017

F A C E B O O K

P A G E h t t p w w w f a c e b o o k c o m s s c m e n t o r s o f f i c i a l P a g e | 49

FOR MORE UPDATES AND MORE MATERIAL DO LIKE OUR FACEBOOK PAGE httpwwwfacebookcomsscmentorsofficial

Options

1) Bear

भाि 2) Horse

घह८िा 3) Cheetah

चीता 4) Zebra

जहॳिा Correct Answer Cheetah

Q214 The pale yellow colour of urine is

due to the presence of which pigment

मतरकाफीकापीिारगरगदरयकहॳ उपनसिनतकहॳ कारणहह८ताहहॴ

3-Feb-2017

Options

1) Urochrome

यरह८िह८म

2) Urophyll

यरह८कफि

3) Chlorophyll

किह८रह८कफि

4) Chloroplast

किह८रह८पिासट

Correct Answer Urochrome

Q215 Which of the following constitute

to form a gene

नननननलिखितमसहॳकह९नसीचीज़एकजीनकागठनकरतीहहॴ

3-Feb-2017

Options

1) Polynucleotides

पह८िीनयनकियह८टाईडस

2) Hydrocarbons

हाइडरह८काबोस

3) Lipoproteins

िाईपह८परह८टीनस

4) Lipids

लिपपडस

Correct Answer Polynucleotides

Polynucleotide molecule is a biopolymer

composed of 13 or more nucleotide

monomers covalently bonded in a chain

DNA (deoxyribonucleic acid) and RNA

(ribonucleic acid) are examples of

polynucleotides with distinct biological

function

Q216 Vertebrates belongs to the

phylum

रीढ़कीहडिीवािहॳपराणी mdashmdashmdash

परजानतकहॳ अतगायतआतहॳहहॴ 3-Feb-2017

Options

1) Arthropoda

आरह८पह८ड़ा 2) Annelida

एननलििा 3) Cnidaria

ननिहॳररया 4) Chordata

कह८िटा Correct Answer Chordata

Q217 Punica granatum is the scientific name of

पननकगरहॳनहॳटस mdashmdashmdash कावहॴजञाननकनामहहॴ 3-Feb-2017

Options

1) Custard Apple

सीताफि

2) Gulmohar

गिमह८हर

3) Silver Oak

लसमवरओक

4) Pomegranate

अनार

Correct Answer Pomegranate

F A C E B O O K

P A G E h t t p w w w f a c e b o o k c o m s s c m e n t o r s o f f i c i a l P a g e | 50

FOR MORE UPDATES AND MORE MATERIAL DO LIKE OUR FACEBOOK PAGE httpwwwfacebookcomsscmentorsofficial

Q218 Between a tiger and an monkey

which of the following is different

एकबाघऔरबदरकहॳ बीचनननननलिखितमसहॳकह९नसीबातअिगहहॴ 3-Feb-2017

Options

1) Kingdom

राजय

2) Phylum

जानत

3) Order

िम

4) Class

वगय Correct Answer order

Q219 The artificial heart was invented by

कबतरमहदयका mdashmdashmdash

दवाराअपवषकारककयागयािा 3-Feb-2017

Options

1) Muhammad Yunus

महनमदयनस

2) Linus Yale Jr

िाइनसयहॳिजय

3) Gazi Yasargil

गाजीयासचगयि

4) Paul Winchell

पह९िपवमकि Correct Answer Paul Winchell

Q220 Tamarindus indica is the

scientific name of

टहॳमररनडसइडिका mdashmdash कावहॴजञाननकनामहहॴ 7-

Feb-2017

Options

1) Neem

नीम

2) Pineapple

अनानास

3) Tamarind

इमिी 4)Chiku

चीक

Correct Answer Tamarind

Q221 In eukaryotic cells synthesis of

RNA takes place in the

यकहॳ योटटककह८लिकाओमआरएनएकासशिहॳषण

mdashndash महह८ताहहॴ 7-Feb-2017

Options

1) Mitochondria

माईटह८कोडडरया 2) Centrioles

सटरीयह८मस

3) Ribosomes

ररबह८सह८नस

4) Nucleus

नयनकियस

Correct Answer nucleus

eukaryotic cell -Transcription is the

process of synthesizing ribonucleic acid

(RNA)Synthesis takes place within the

nucleus of eukaryotic cells or in the

cytoplasm of prokaryotes and converts

the genetic code from a gene in

deoxyribonucleic acid ( DNA ) to a

strand of RNA that then directs

proteinsynthesis

Q222 _________is caused by parasites

of the Plasmodium genus

पिाजमह८डियमजातीकहॳ परजीवी mdash- कहॳ कारणहहॴ 7-Feb-2017

Options

1) Dysentery

पहॳचचि

2) Malaria

मिहॳररया 3) Chickenpox

F A C E B O O K

P A G E h t t p w w w f a c e b o o k c o m s s c m e n t o r s o f f i c i a l P a g e | 51

FOR MORE UPDATES AND MORE MATERIAL DO LIKE OUR FACEBOOK PAGE httpwwwfacebookcomsscmentorsofficial

चहॳचक

4) Herpes

हहॳपपयस

Correct Answer Malaria

Q223 Carotene in fruits and vegetables

gives it which color

फिह८औरसनलजयोमनसितकहॳ रह८टीनउनहकह९नसारगपरदानकरताहहॴ 7-Feb-2017

Options

1) Green

हरा 2) Pink

गिाबी 3) Orange

नारगी 4) Blue

नीिा Correct Answer Orange

Q224 Equus Caballus is the scientific

name of

एकवसकहॴ बहॳिस mdashmdashndash कापवजञाननकनामहहॴ 7-Feb-2017

Options

1) Horse

घह८िा 2) Zebra

ज़हॳिा 3) Donkey

गधा 4) Buffalo

भस

Correct Answer Horse

Q225 Elapidae Naja is the scientific name of

एिीपीिीनाजा mdashmdash- कावहॴजञाननकनामहहॴ 8-Feb-2017

Options

1) Cobra

कह८बरा 2) Elephant

हािी 3) Eagle

ग ि

4) Owl

उमि Correct Answer Cobra

Q226 Which disease is caused due to

deficiency of Iron

िह८हकीकमीकहॳ कारणकह९नसारह८गहह८ताहहॴ 8-Feb-

2017

Options

1) Beriberi

बहॳरीबहॳरी 2) Tetany

टहॳटनी 3) Kwashiorkor

कवािीऔरकर

4) Anaemia

रकतामपता Correct Answer Anaemia

Beriberi is a disease caused by a vitamin

B-1 deficiency also known as thiamine

deficiency

Tetany can be the result of an

electrolyte imbalance Most often itrsquos a

dramatically low calcium level also

known as hypocalcemia Tetany can also

be caused by magnesium deficiency or

too little potassium Having too much

acid (acidosis) or too much alkali

(alkalosis) in the body can also result in

tetany

Kwashiorkor also known as

ldquoedematous malnutrition It is a form of

malnutrition caused by a lack of protein

in the diet

Anaemia means that you have fewer red

blood cells than normal or you have less

F A C E B O O K

P A G E h t t p w w w f a c e b o o k c o m s s c m e n t o r s o f f i c i a l P a g e | 52

FOR MORE UPDATES AND MORE MATERIAL DO LIKE OUR FACEBOOK PAGE httpwwwfacebookcomsscmentorsofficial

haemoglobin than normal in each red

blood cell

Q227 is a leaf where the leaflets are

arranged along the middle vein

mdashndashएकपततीहहॴजहापतरकह८कीरचनाक ररयालिराकहॳ आसपासहह८तीहहॴ 8-Feb-2017

Options

1) Pinnately compound leaf

पपनहॳटिीसयकतपतती 2) Palmately compound leaf

पामहॳटिीसयकतपतती 3) Compound leaf

सयकतपतती 4) Simple leaf

साधारणपतती Correct Answer Pinnately compound

leaf

Q228 Haustoria or sucking roots are

found in which of the following

हह८सटह८ररयायाचसनहॳवािीजड़हॳनननननलिखितमसहॳककसमपाईजातीहहॴ 8-Feb-2017

Options

1) Wheat

गहॳह

2) Mango

आम

3) Chestnut

चहॳसटनट

4) Cuscuta

कसकयटा Correct Answer Cuscuta

Haustorial roots -The roots of parasitic

plants which penetrate into the host

tissues to absorb nourishment are

called haustorial roots hellip Also known as suckingor parasitic roots

Q229 Equs Asinus is the scientific name

of

एकवसएलसनस mdashmdashndash कावहॴजञाननकनामहहॴ 8-

Feb-2017

Options

1) Donkey

गधा 2) Cow

गाय

3) Deer

टहरन

4) Kangaroo

कगा

Correct Answer Donkey

Q230 Ficus benghalensis is the scientific name of

फाईकसबहॳनगहॳिहॳलसस mdashndash कापवजञाननकनामहहॴ 8-Feb-2017

Options

1) Banyan

बरगद

2) Pineapple

अनानास

3) Babul

बबि

4) Tulsi

तिसी Correct Answer Banyan

Q231 Equus burchellii is the scientific name of

एकवसबचिी mdashmdash- कापवजञाननकनामहहॴ 8-Feb-2017

Options

1) Horse

घह८िा 2) Zebra

जहॳिा 3) Buffalo

F A C E B O O K

P A G E h t t p w w w f a c e b o o k c o m s s c m e n t o r s o f f i c i a l P a g e | 53

FOR MORE UPDATES AND MORE MATERIAL DO LIKE OUR FACEBOOK PAGE httpwwwfacebookcomsscmentorsofficial

भस

4) Ass

गधा Correct Answer Zebra

Page 18: COMPILATION OF ALL 72 SETS OF BIOLOGY SSC CHSL-2016 · OF BIOLOGY SSC CHSL-2016 PREPARED BY : SSC MENTORS BIOLOGY SPECIAL . F A C E B O O K P A G E : h t t p : / / w w w . f a c e

F A C E B O O K

P A G E h t t p w w w f a c e b o o k c o m s s c m e n t o r s o f f i c i a l P a g e | 17

FOR MORE UPDATES AND MORE MATERIAL DO LIKE OUR FACEBOOK PAGE httpwwwfacebookcomsscmentorsofficial

3) Gills

गिफड़हॳ 4) Tracheae

शरावस- निी Correct Answer Tracheae

Air enters the respiratory systems of

insects through a series of external

openings called

spiracles These external openings

which act as muscular valves in some

insects lead to the internal respiratory

system a densely networked array of

tubes called tracheae

Q67 The poisonous gas accidentally

released in Bhopal Gas Tragedy is

भह८पािगहॴसतरासदीमगितीसहॳमकतहईजहरीिीगहॴसिी

17-Jan-2017

1) Methane

मीिहॳन

2) Nitrous Oxide

नाइटरसऑकसाइि

3) Methyl Isocyanate

महॴचििआयसोसायनहॳट

4) Cyanogen

सायनह८जहॳन

Correct Answer Methyl Isocyanate

Q68 What does Trypsin do

टटरनपसनकयाकरताहहॴ

SSC CHSL Science (biology) 2016

Question Paper

17-Jan-2017

Options

1) Breaks down Carbohydrates

काबोहाइडरहॳटकापवघटनकरताहहॴ 2) Synthesizes proteins

परह८टीनकासििहॳषणकरताहहॴ 3) Breaks down fats

वसाकापवघटनकरताहहॴ 4) Breaks down proteins

परह८टीनकापवघटनकरताहहॴ Correct Answer Breaks down proteins

Trypsin is one of the three principal

digestive

proteinases the other two being pepsin

and

chymotrypsin In the digestive process

trypsin acts with the other proteinases

to break down dietary protein molecules

to their component

peptides and amino acids

A protease is any enzyme that performs

proteolysis protein catabolism by

hydrolysis of peptide bonds

Q69 Name the source from which

Aspirin is produced

उससरह८तकानामबताइए

नजससहॳएनसपररनकाउतपादनककयाजाताहहॴ

17-Jan-2017

Options

1) Willow bark

पविह८कीछाि

2) Oak Tree

ओककावकष

3) Acacia

बबि

4) Eucalyptus

नीिचगरी Correct Answer Willow bark

The compound from which the active

ingredient in aspirin was first derived

salicylic acid was found in the bark of a

willow tree in 1763 by Reverend

Edmund Stone of Chipping-Norton

Q70 Cannis Familiaris is the scientific

name of

कहॴ ननसफहॳ लमलियहॳररस mdash- कावहॴजञाननकनामहहॴ

17-Jan-2017

F A C E B O O K

P A G E h t t p w w w f a c e b o o k c o m s s c m e n t o r s o f f i c i a l P a g e | 18

FOR MORE UPDATES AND MORE MATERIAL DO LIKE OUR FACEBOOK PAGE httpwwwfacebookcomsscmentorsofficial

Options

1) Cat

बबमिी 2)Dog

कतता 3) Fox

िह८मड़ी 4) Wolf

भहॳडड़या Correct Answer Dog

Q71 Harmful bacteria in potable water

make the water

पीनहॳकहॳ पानीमनसतिघातकबहॴकटीररयाउसपानीकह८बनातहॳहहॴ 17-Jan-2017

Options

1) unfit to drink

पीनहॳकहॳ लिएअयह८गय

2) smelly

दगयनधयकत

3) Colored

रगीन

4) Turbid

मटमहॴिा Correct Answer unfit to drink

Q72 Musa paradisiaca is the scientific

name of which plant

मसापहॴराडिलसयाकाककसपह९धहॳकावहॴजञाननकनामहहॴ

17-Jan-2017

Options

1) Mango

आम

2) Wheat

गहॳह

3) Corn

भ ा 4) banana

कहॳ िा Correct Answer banana

Q73 Prawns belong to which family

झीगहॳककसपररवारकहॳ हह८तहॳहहॴ 17-Jan-2017

Options

1) Crustaceans

िसटहॳलियन

2)Fish

मछिी 3) Amphibians

अननफबबयस

4) Reptiles

रहॳपटाइमस

Correct Answer Crustaceans

Q74 Name the drug that is yielded from

Cinchona tree and is used to cure

malaria

उसऔषचधकानामबताइएनजसहॳलसगकह८नापहॳड़सहॳपरापतककयाजाताहहॴऔरनजसकाउपयह८गमिहॳररयाकहॳ उपचारमककयाजाताहहॴ 17-Jan-2017

Options

1) Camptothea

कहॴ नटह८चिया 2) Acuminata

एकयलमनहॳटा 3) Quinine

कनहॴन

4) Cinchonia

लसकह८ननया Correct Answer Quinine

Q75 Blood Circulation was discovered

by

रकतपररसचरणकी mdashmdashndash दवारािह८जकीिी 17-Jan-2017

Options

1) Mary Anderson

F A C E B O O K

P A G E h t t p w w w f a c e b o o k c o m s s c m e n t o r s o f f i c i a l P a g e | 19

FOR MORE UPDATES AND MORE MATERIAL DO LIKE OUR FACEBOOK PAGE httpwwwfacebookcomsscmentorsofficial

महॴरीएिरसन

2) Virginia Apgar

वनजयननयाएपगार

3) William Harvey

पवलियमहाव

4) Robert Feulgen

रॉबटयफ़यिजहॳन Correct Answer William Harvey

Q76 Vitamin A is also known as

पवटालमन A कह८ mdashmdash- कहॳ नामसहॳभीजानाजाताहहॴ SSC CHSL Science (biology) 2016

Question Paper

18Jan2017

Options

1) Thiamine

िायलमन

2) Riboflavin

ररबह८फिहॳपवन

3) Retinol

रहॳटटनॉि

4) Calciferol

कहॴ नमसफहॳ रह८ि

Correct Answer Retinol

Q77 Some roots called arise from an

organ other than the radicle

कछजड़हॳनजनह mdashmdashmdash कहाजाताहहॴ वहमिकहॳ अिावाककसीअनयअगसहॳउतपननहह८तीहहॴ 18Jan2017

Options

1) tap roots

मखयजड़

2) stilt roots

ि ाजड़

3) fibrous roots

रहॳिहॳदारजड़

4) adventitious roots

आकनसमकजड़

Correct Answer adventitious roots

Q78 Spiders belong to which class of

animals

मकडड़यापराणीवगीकरणकहॳ ककसवगयमआतीहहॴ 18Jan2017

Options

1) Arachnids

एरहॳकननडस

2) Aves

एपवस

3) Gastropods

गहॴसटरोपह८िस

4) Anthozoa

एिह८जआ

Correct Answer Arachnids

Q79 How many layers does Human

Skin have

मानवतवचामककतनीपरतहॳहह८तीहहॴ

18Jan2017

Options

1) 5

2) 7

3) 11

4) 3

Correct Answer 3

Skin has three layers The epidermis

the outermost layer of skin provides a

waterproof barrier and creates our skin

tone The dermis beneath the

epidermis contains tough connective

tissue hair follicles and sweat glands

The deeper subcutaneous tissue (

hypodermis ) is made of fat and

connective tissue

Q80 Allium Cepa is the scientific name

of

एलियमलसपपा mdashmdashndash कावहॴजञाननकनामहहॴ 18Jan2017

F A C E B O O K

P A G E h t t p w w w f a c e b o o k c o m s s c m e n t o r s o f f i c i a l P a g e | 20

FOR MORE UPDATES AND MORE MATERIAL DO LIKE OUR FACEBOOK PAGE httpwwwfacebookcomsscmentorsofficial

Options

1) Carrot

गाजर

2) Tomato

टमाटर

3) Potato

आि 4) Onion

पयाज़

Correct Answer Onion

Q81 DNA stands for

िीएनएकापणय प mdashmdash- हहॴ 18Jan2017

Options

1) Di Nucleic Acid

िाईनयनकिकएलसि

2) Deoxy Nucleic Acid

िीओकसीनयनकिकएलसि

3) Diribonucleic Acid

िाईराइबह८नयनकिकएलसि

4) Deoxyribonucleic Acid

िीऑकसीराइबह८नयनकिकएलसि

Correct Answer Deoxyribonucleic Acid

Q82 Organisms that generate energy

using light are known as

जह८जीवाणपरकािकाउपयह८गकरउजायउतपननकरतीहहॴ उनह mdashmdash कहॳ पमजानाजाताहहॴ

18Jan2017

Options

1) Chaemolithotrophs

ककमह८लििह८टरह८पस

2) Oligotrophs

ओलिगह८टरह८पस

3) Bacteria

बहॴकटीररया 4)Photoautotrophs

फह८टह८ओटह८टरह८पस

Correct Answer Photoautotrophs

An oligotroph is an organism that can

live in an environment that offers very

low levels of nutrients

Q83 Which drug is used as an

Antidepressant

ककसदवाएकहतािारह८धीकहॳ पमपयोगककयाजाताहहॴ Options

1) Oxybutynin

ओकसीलयटीनन

2)Tramadol

टरहॳमहॳिह८ि

3 ) Sumatriptan

समहॳटरीपटहॳन

4) Bupropion

लयपरह८पपयह८न

Correct Answer Bupropion

लयपरह८पपयह८न

Q84 The orange colour of carrot is

because of

गाजरकानारगीरगनननननलिखितमसहॳककसीएककीवजहसहॳहह८ताहहॴ 18Jan2017

Options

1) it grows in the soil

यहलम ीमउगतीहहॴ 2) Carotene

कहॴ रह८टीन

3) it is not exposed to sunlight

यहसययपरकािकहॳ सपकय मनहीआती 4) the entire plant is oranqe in colour

सनपणयपह९धानारगीरगकाहह८ताहहॴ Correct Answer Carotene

Q85 Snake venom is highly modified

saliva containing

F A C E B O O K

P A G E h t t p w w w f a c e b o o k c o m s s c m e n t o r s o f f i c i a l P a g e | 21

FOR MORE UPDATES AND MORE MATERIAL DO LIKE OUR FACEBOOK PAGE httpwwwfacebookcomsscmentorsofficial

सापकाजहरअततयाचधकसिह८चधतिारहह८तीहहॴनजसमहॳ mdashmdash- हह८ताहहॴ Options

l)Prototoxins

परह८टह८टॉनकसस

2)Neutrotoxins

नयटरोटॉनकसस

3)Zootoxins

जटॉनकसस

4)Electrotoxins

इिहॳकटरह८टॉनकसस

Correct Answer Zootoxins

जटॉनकसस

Q86 Which type of pathogen causes the

water-borne disease Schistosomiasis

ककसपरकारकारह८गज़नकजिजननतरह८गलससटह८सह८लमलससकाकारणबनताहहॴ

18Jan2017

Option

1) Parasitic

परजीवी 2)Protozoan

परह८टह८जआ

3) Bacterial

बहॴकटीररयि

4) Viral

वायरि

Correct Answer Parasitic

Schistosomiasis also known as snail

fever and bilharzia is a disease caused

by parasitic

flatworms called schistosomes

Q87 Prothrombin responsible for

clotting of blood is released by

परह८िह८ननबन

जह८रकतकािककाजमनहॳकहॳ लिएनजनमहॳदारहहॴ mdashndash

कहॳ दवारासतरापवतककयाजाताहहॴ

19Jan2017

Options

1) Small Intestine

छह८टीआत

2) Blood Platelets

रकतपिहॳटिहॳटस

3) Large Intestine

बड़ीआत

4Heart

हदय

Correct Answer Blood Platelets

Q88 Acacia arabica is the scientific

name of

अकहॳ लियाअरहॳबबका mdashmdashndash कावहॴजञाननकनामहहॴ 19-Jan-2017

Options

1) Neem

नीम

2) Teak

सागह९न

3) Babhul

बबि

4) Pomegranate

अनार

Correct Answer Babhul

Q89 Cannis Vulpes is the scientific

name of

कहॴ ननसवनमपस mdashmdash- कावहॴजञाननकनामहहॴ 19-Jan-2017

Options

1) Dog

कतता 2) Wolf

भहॳडड़या 3) Fox

िह८मड़ी 4) Hyena

िाकिबगघा

F A C E B O O K

P A G E h t t p w w w f a c e b o o k c o m s s c m e n t o r s o f f i c i a l P a g e | 22

FOR MORE UPDATES AND MORE MATERIAL DO LIKE OUR FACEBOOK PAGE httpwwwfacebookcomsscmentorsofficial

Correct Answer Fox

Q90 The beetroot is the portion of the

beet plant

चकदरपह९धहॳका mdashmdashndash भागहहॴ 19-Jan-2017

Options

1) tap root

मखयजड़

2) Adventitious

आकनसमक

3) bulb of the stem

तनहॳकाकद

4) Rhizome

परकद

Correct Answer tap root

Q91 What is the basic unit of heredity

आनवलिकताकीबननयादीइकाईकयाहहॴ 19-Jan-2017

Options

1) DNA

िीएनए

2) RNA

आरएनए

3) Chromosome

िह८मह८सह८म

4) Gene

जीन

Correct Answer gene

Genes are the units of heredity and are

the instructions that make up the bodyrsquos

blueprint They code for the proteins

that determine virtually all of a personrsquos

characteristics Most genes come in

pairs and are made of strands of genetic

material called deoxyribonucleic acid

or DNA

Q92 Lungs are the primary organs of

फहॳ फड़हॳmdashndashकहॳ परािलमकअगहहॴ

19-Jan-2017

Options

1) Digestion

पाचन

2) Constipation

कलज

3) Perspiration

पसीना 4)Respiration

शवसन

Correct Answer Respiration

Q93 Sugarcane is a type of

गननाएकपरकारका mdash- हहॴ 20-Jan-2017

Options

1)creeper

िता 2)tree

पहॳड़

3)shrub

झाड़ी 4)grass

घास

Correct Answer grass

Q94 Who is commonly known as ldquothe

Father of Microbiologyrdquo

सामानयत ldquo सकषमजीवपवजञानकहॳ जनक lsquo

कहॳ नामसहॳककसहॳजानाजातहहॴ 20-Jan-2017

Options

1) Robert Hooke

रॉबटयहक

2) Antonie Philips van Leeuwenhoek

एटह८नीकफलिपवानमयएनहह८क

3) Carl Linnaeus

काियिीनाईयस

4) Charles Darwin

चामसयिापवयन

F A C E B O O K

P A G E h t t p w w w f a c e b o o k c o m s s c m e n t o r s o f f i c i a l P a g e | 23

FOR MORE UPDATES AND MORE MATERIAL DO LIKE OUR FACEBOOK PAGE httpwwwfacebookcomsscmentorsofficial

Correct Answer Antonie Philips van

Leeuwenhoek

Q95 For the aquatic organisms the

source of food is

जिीयजीवाणकािाघसरह८तहहॴ 20-Jan-2017

Options

1) Phytoplankton

फायटह८पिहॳकटन

2) Sea Weed

समदरीिहॴवाि

3)Aqua plankton

एकवापिहॳकटन

4) Zooplankton

जपिहॳकटन

Correct Answer Phytoplankton

Q96 Haemoglobin has the highest

affinity with which of the following

हीमह८गिह८बबनकीननननमसहॳककसकहॳ सािउततमसमानताहहॴ

20-Jan-2017

Options

1)SO2

2)CO2

3)CO

4)NO2

Correct Answer CO

It has a greater affinity for hemoglobin

than oxygen does It displaces oxygen

and quickly binds so very little oxygen

is transported through the body cells

Q97 Who developed the theory of

Evolution

उदपवकासकालसदातककसनहॳपवकलसतककया

20-Jan-2017

Options

1) Charles Darwin

चामसयिापवयन

2) Isaac Newton

आयजहॳकनयटन

3) Pranav Mistry

परणवलमसतरी 4) Galileo Galilei

गहॳलिलियह८गहॳिीिी Correct Answer Charles Darwin

Q98 The primary function of RNA is

RNA कापरािलमककाययहह८ताहहॴ 20-Jan-2017

Options

1) Photosynthesis

परकािसशिहॳषण

2) Protein Synthesis

परह८टीनसशिहॳषण

3) Replication

परनतकनतबनाना 4) Translation

अनवादकरना Correct Answer Protein Synthesis

There are two main functions of RNA

It assists DNA by serving as a messenger

to relay the proper genetic information

to countless numbers of ribosomes in

your body The other main function of

RNA is to select the correct amino acid

needed by each ribosome to build new

proteins for your body

Q99 ______is the movement of

molecules across a cell membrane from

a region of their lower concentration to

a region of their higher concertration

उचचसादरताकहॳ कषहॳतरसहॳउसकीकमसादरतावािहॳकषहॳतरकीतरफएककह८लिकाखझमिीकहॳ माधयमसहॳहह८नहॳवािाअणओकहॳ सचिनकह८ mdash- कहतहॳहहॴ Options

1) Diffusion

पवसरण

2) Osmosis

ऑसमह८लसस

F A C E B O O K

P A G E h t t p w w w f a c e b o o k c o m s s c m e n t o r s o f f i c i a l P a g e | 24

FOR MORE UPDATES AND MORE MATERIAL DO LIKE OUR FACEBOOK PAGE httpwwwfacebookcomsscmentorsofficial

3) Active Transport

सकियआवागमन

4) Passive Transport

नननषियआवागमन

Correct Answer Active Transport

Q100 Study of classification of

organisms is known as 20-Jan-2017

जीवाणओकहॳ वगीकरणकहॳ अधययनकह८ mdash-

कहाजाताहहॴ Options

1) Serpentology

सपरहॳटह८िह८जी 2) Virology

वायरह८िह८जी 3) Taxonomy

टहॴकसोनह८मी 4) Physiology

कफनज़यह८िह८जी Correct Answer Taxonomy

Q101 Photosynthesis takes place inside

plant cells in

परकािसशिहॳषणवनसपनतकह८लिकामनसति mdash

mdashmdash महह८ताहहॴ 20-Jan-2017

Options

1) Ribosomes

राइबह८सह८नस

2) Chloroplasts

किह८रह८पिासट

3) Nucleus

नयकलियम

4) Mitochondria

माईटह८कोडडरया Correct Answer Chloroplasts

Q102 ______ is the cell organelle in

which the biochemical processes of

respiration and energy production

occur

mdashmdash- वहकह८लिकाअगहहॴ नजसमहॳशवसनऔरउजायउतपादनकहॳ जहॴसीजहॴवरासायननकपरकियायहह८तीहहॴ 20-Jan-2017

Options

1) Mitochondria

माइटह८कोडडरया 2) Chloroplast

किह८रह८पिासट

3) Ribosomes

राइबह८सह८नस

4) Nucleus

नयकिीयस

Correct Answer Mitochondria

Q103 Which non-flowering spore

bearing plants have roots

ककसफिनिगनहॳवािहॳऔरबीजाणधारकपह९धह८कीजड़हॳहह८तीहहॴ 21-Jan-2017

Options

1) Mosses

मह८सहॳस

2) Angiosperms

एननजयह८सपनसय 3) Ferns

फनसय 4) Gymnosperms

नजननह८सपनसय Correct Answer ferns

Q104 Which of the following is an

excretory organ of cockroach

नननननलिखितमसहॳकह९नसानतिच हॳकाउतसजयनअगहहॴ

21-Jan-2017

Options

F A C E B O O K

P A G E h t t p w w w f a c e b o o k c o m s s c m e n t o r s o f f i c i a l P a g e | 25

FOR MORE UPDATES AND MORE MATERIAL DO LIKE OUR FACEBOOK PAGE httpwwwfacebookcomsscmentorsofficial

1) Malphigian Tubules

मनमफनजयनटयबमस

2) Nephridia

नहॳकफरडिया 3) Coxal Gland

कह८कसिगरचिया 4) Green Gland

गरीनगरचिया Correct Answer Malphigian Tubules

Q105 Evaporation of water takes place

in which part of plants

पानीकहॳ वाषपीकरणकीकियापह९धोकहॳ ककसभागसहॳहह८तीहहॴ 21-Jan-2017

Options

1) Stem

तना 2) Stomata

सटह८मटा 3) Branch

िािाए

4) Fruit

फि

Correct Answer Stomata

Evaporation accounts for the movement

of water to the air from sources such as

the soil canopy interception and

waterbodies Transpiration accounts for

the movement of water within a plant

and the subsequent loss of water as

vapour through stomata in its leaves

Q106 A is the fleshy spore-bearing

fruiting body of a fungus

mdashmdashndashकवककामासि

बीजाणधारणकरनहॳवािाफिनहॳवािाअगहहॴ 21-

Jan-2017

Options

1) aloe vera

एिह८वहॳरा 2) Coral

मगा 3) Cactus

कहॴ कटस

4) Mushroom

ककरमतता Correct Answer mushroom

Q107 Which of the following is a fungal

disease

नननननलिखितमसहॳकह९नसाफफदसहॳहह८नहॳवािाएकरह८ग हहॴ

21-Jan-2017

Options

1) Dermatitis

तवचािह८ध

2) Cholera

हहॴजा 3) Jaundice

पीलिया 4) Indigofera

इननिगह८फहॳ रा Correct Answer Dermatitis

Dermatitis also known as eczema is a

group of diseases that results in

inflammation of the skin These diseases

are characterized by itchiness red skin

and a rash In cases of short duration

there may be small blisters while in

long-term cases the skin may become

thickened

Q108 In which form is glucose stored in

our body

हमारहॳिरीरमगिकह८जकासचयककस पमककयाजाताहहॴ

21-Jan-2017

Options

1) Insulin

F A C E B O O K

P A G E h t t p w w w f a c e b o o k c o m s s c m e n t o r s o f f i c i a l P a g e | 26

FOR MORE UPDATES AND MORE MATERIAL DO LIKE OUR FACEBOOK PAGE httpwwwfacebookcomsscmentorsofficial

इसलिन

2) Glucose

गिकह८ज

3) Glycogen

गिायकह८जहॳन

4) Fat

वसा Correct Answer Glycogen

Excess glucose is stored in the liver as

the large compound called glycogen

Glycogen is a polysaccharide of glucose

but its structure allows it to pack

compactly so more of it can be stored in

cells for later use

Q109 Where do plants synthesize

protein from

पह९धहॳपरह८टीनसशिहॳषणकहासहॳकरतहॳहहॴ

Options

1) Fatty Acids

वसाऐलसि

2) Sugar

िकर

3) Amino Acids

एलमनह८ऐलसि

4) Starch

सटाचय Correct Answer Amino Acids

Q110 Which part of the brain is

responsible for triggering actions like

thinking intelligence memory and

ability to learn

मनसतषककाकह९नसाटहससासह८चनहॳ बनधदमानी याददाशतऔरसीिनहॳकीकषमताजहॴसीकियाओकह८परहॳररतकरताहहॴ 21-Jan-2017

Options

1) Diencephalon

िायएनसहॳफहॳ िह८न

2) Hypothalamus

हयपह८िहॳिहॳमस

3) Cerebrum

सहॳरहॳिम

4) Control

कटरह८ि

Correct Answer Cerebrum

Q111 Which of the following is also

known as the Biochemical Laboratory

of the Human Body

नननननलिखितमसहॳककसहॳमानविरीरकीजहॴवरसायनपरयह८गिािाभीकहाजाताहहॴ 21-Jan-2017

Options

1) Small Intestine

छह८टीआत

2)Brain

मनसतषक

3) Pancreas

अगनयािय

4) Liver

नजगर

Correct Answer Liver

The liver makes bile that will help

emulsify and digest the fats we eat

The liver takes toxic substances and

convert them using enzymes the liver

cells makes into a non toxic form so the

body can dispose of them

The liver also converts fats protein and

carbohydrates into glucose which is the

energy source for our cells to use

The liver takes amino acids and makes

proteins by combining them

Q112 The yellow colour of human urine

is due to

मानवमतरकापीिारग mdashndash कीवजहसहॳहह८ताहहॴ 22-

Jan-2017

Options

1) Bile Salts

F A C E B O O K

P A G E h t t p w w w f a c e b o o k c o m s s c m e n t o r s o f f i c i a l P a g e | 27

FOR MORE UPDATES AND MORE MATERIAL DO LIKE OUR FACEBOOK PAGE httpwwwfacebookcomsscmentorsofficial

पपततनमक

2) Cholesterol

कह८िहॳसटरह८ि

3) Lymph

लिनफ

4) Urochrome

यरह८िह८म

Correct Answer Urochrome

Urobilin or urochrome is the chemical

primarily responsible for the yellow

color of urine

Q113 The wilting of plants takes place

due to

पह९धह८कालिचििहह८नाकी mdashmdash- कीवजहसहॳहह८ताहहॴ 22-Jan-2017

Options

1)Photosynthesis

परकािसशिहॳषण

2) Transpiration

वाषपह८तसजयन

3) Absorption

अविह८षण

4) Respiration

शरवसन

Correct Answer Transpiration

Wilting is the loss of rigidity of non-

woody parts of plants This occurs when

the turgor pressure in non-lignified

plant cells falls towards zero as a result

of diminished water in the cells

Q114 Bovidae Ovis is the scientific name of

बह८पविीओपवस mdashndash कावहॴजञाननकनामहहॴ 22-Jan-2017

Options

1) Goat

बकरी 2) Cow

गाय

3) Buffalo

भहॳस

4) Sheep

भहॳड़

Correct Answer Sheep

Q115 Plants get their energy to produce

food from which of the following

पह८धहॳभह८जनकाननमायणकरनहॳकहॳ लिएनननननलिखितमसहॳककससहॳउजायपरापतकरतहॳहहॴ

22-Jan-2017

Options

1) Photosynthesis

परकािसशिहॳषण

2)Bacteria

बहॴकटीररया 3)Fungi

कवक

4)Sun

सयय Correct Answer Sun

Q116 Which of the following is secreted

by the liver

नननननलिखितमसहॳककसकासरावनजगरसहॳहह८ताहहॴ

22-Jan-2017

Options

1) Glucose

गिकह८ज

2) Iodine

आयह८िीन

3) Cortisol

काटटरयसह८ि

4) Bile

पपतत

Correct Answer Bile

The liver makes bile that will help

emulsify and

digest the fats we eat

F A C E B O O K

P A G E h t t p w w w f a c e b o o k c o m s s c m e n t o r s o f f i c i a l P a g e | 28

FOR MORE UPDATES AND MORE MATERIAL DO LIKE OUR FACEBOOK PAGE httpwwwfacebookcomsscmentorsofficial

Q117 Ferns belong to which division of

plants

फनसयपह९धह८कहॳ ककसभागमआतहॳहहॴ

22-Jan-2017

Options

1) Gymnosperms

नजननह८सपनसय 2) Angiosperms

एनजयह८सपनसय 3) Thallophyta

िहॴिह८फाईटा 4)Pteridophyta

टहॳररिह८फाईटा Correct Answer Pteridophyta

Q118 Who invented Antibiotics

एटीबायह८टटककाअपवषकारककसनहॳककयािा

22-Jan-2017

Options

1) Joseph Lister

जह८सहॳफलिसटर

2) William Harvey

पवलियमहाव

3) Robert Knock

रॉबटयनॉक

4)Alexander Fleming

अिहॳकज़िरफिहॳलमग

Correct Answer Alexander Fleming

Q119 Milbecycin is used in the

eradication of

लममबहॳसायलसनका mdashndash

मउनमिनमपरयह८गककयाजाताहहॴ 22-Jan-2017

Options

1) Agricultural Fungus

कपषकवक

2) Agricultural Pests

कपषकीटक

3) Agricultural Herbs

कपषिाक

4)Agricultural Weeds

कपषननराना Correct Answer Agricultural Pests

Milbemycin oxime is a veterinary drug

from the group of milbemycins used as

a broad spectrum antiparasitic It is

active against worms and mites(insects

Q120 Intestinal bacteria synthesizes

which of the following in the human

body

मानविरीरमआतोकहॳ बहॴकटीररयानननननलिखितमसहॳककसकासशिहॳषणकरतहॳहहॴ 22-Jan-2017

Options

1) Vitamin K

पवटालमन K

2) Proteins

परह८टीन

3) Fats

वसा 4) Vitamin D

पवटालमन D

Correct Answer Vitamin K

Q121 is the study of the physical form

and external structure of plants

mdashmdash-

मपह९धह८काभहॴनतक पऔरबाहरीसरचनाकाआदयाककयाजाताहहॴ 22-Jan-2017

Options

1) Physiology

कफनजयह८िह८जी 2) Anatomy

िरीररचनापवजञान

3) Phytomorphology

फाईटह८मह८फह८िह८जी 4)Cytology

कह८लिकापवजञान

Correct Answer Phytomorphology

F A C E B O O K

P A G E h t t p w w w f a c e b o o k c o m s s c m e n t o r s o f f i c i a l P a g e | 29

FOR MORE UPDATES AND MORE MATERIAL DO LIKE OUR FACEBOOK PAGE httpwwwfacebookcomsscmentorsofficial

Q122 Which of the following is a

structural and functional unit of

kidneys

नननननलिखितमसहॳकह९नसीगदोकीसरचनातमकऔरकाययकरीईकाईहहॴ

22-Jan-2017

Options

1) Renette Cells

रहॳनहॳटकह८लिकाए

2) Flame Cells

फिहॳमकह८लिकाए

3) Nephrites

नहॳफ़राइटस

4)Nephrons

नहॳफरोस

Correct Answer Nephrons

Nephron functional unit of the kidney

the structure that actually produces

urine in the process of removing waste

and excess substances from the blood

There are about 1000000 nephrons in

each human kidney

Q123 Which of the following is the

largest part of the human brain

नननननलिखितमसहॳकह९नसामानवमनसतषककासबसहॳबड़ाटहससाहहॴ

23-Jan-2017

Options

1) Ribs

पसलियाा 2) Cerebrum

सहॳरहॳिम

3) Pons

पोस

4)Thalamus

िहॴिहॳमस

Correct Answer Cerebrum

The cerebrum is the largest part of the

human brain making up about two-

thirds of the brainrsquos mass It has two

hemispheres each of which has four

lobes frontal parietal temporal and

occipital

Q124 The auxiliary buds

सहायककालियाmdashndash 23-Jan-2017

Options

1) grow endogenously from the pericycle

पहॳरीसाईककिसहॳअनतजातयपवकलसतहह८ताहहॴ 2) arise endogenously from the main

growing point

मिवपदसहॳअनतजातयउठताहहॴ 3) is an embryonic shoot located in the

axil of a leaf

एकभरणिटहहॴजह८एकपततीकहॳ अकषपरनसतिहह८ताहहॴ 4)arise exogenously from the epidermis

एपपिलमयससहॳबटहजातयतरीकहॳ सहॳउठताहहॴ Correct Answer is an embryonic shoot

located in the axil of a leaf

Q125 Which of the following is a viral

disease

इनमहॳसहॳकह९सीएकवायरिबीमारीहहॴ

23-Jan-2017

Options

1) Polio

पह८लियह८ 2) Tetanus

धनसतनभ

3) Leprosy

कषठरह८ग

4) Plague

पिहॳग

Correct Answer Polio

A viral disease (or viral infection)

occurs when an organismrsquos body is

invaded by pathogenic viruses and

infectious virus particles (virions) attach

to and enter susceptible cells

F A C E B O O K

P A G E h t t p w w w f a c e b o o k c o m s s c m e n t o r s o f f i c i a l P a g e | 30

FOR MORE UPDATES AND MORE MATERIAL DO LIKE OUR FACEBOOK PAGE httpwwwfacebookcomsscmentorsofficial

Poliomyelitis often called polio or

infantile paralysis is an infectious

disease caused by the poliovirus

Tetanusmdash A serious bacterial infection

that causes painful muscle spasms and

can lead to death

Leprosy also known as Hansenrsquos

disease (HD) is a long-term infection by

the bacterium Mycobacterium leprae or

Mycobacterium lepromatosis

Plague is an infectious disease caused by

the bacterium Yersinia pestis

Symptoms include fever weakness and

headache

Q126 Which organisms can help to

carry out Vermicomposting

कह९नसाजीववमीकनपह८नसटगममददकरताहहॴ

23-Jan-2017

Options

1) Nitrifying Bacteria

नाईटरीफाईगबहॴकटीररया 2) Earthworms

कहॴ चऐ

3) Algae

िहॴवि

4) Fungus

कवक

Correct Answer Earthworms

Q127 Contraction of heart is also

known as

हदयकहॳ सकचनकह८ mdash- भीकहाजाताहहॴ 23-Jan-

2017

Options

1) Systole

लससटह८ि

2) Aristotle

अरसत

3) Diastole

िायसटह८ि

4) Lub

मयब

Correct Answer Systole

Diastole is the part of the cardiac cycle

when the heart refills with blood

following systole (contraction)

Ventricular diastole is the period during

which the ventricles are filling and

relaxing while atrial diastole is the

period during which the atria are

relaxing

Q128 Azadirachta indica is the

botanical name of which of the

following

अजाटदराचताइडिकानननननलिखितमसहॳककसकावानसपनतनामहहॴ

23-Jan-2017

Options

1) Rose plant

गिाबकापह९धा 2) Apple tree

सहॳबकापहॳड़

3) Neem

नीम

4)Mango

आम

Correct Answer Neem

Q129 Which of the following is the

main end product of carbohydrate

digestion

नननननलिखितमसहॳकह९नसाकाबोहाइडरहॳटकहॳ पाचनकापरमिअतउतपादकहह८ताहहॴ 23-Jan-2017

Options

1) Fats

वसा 2) Lipids

लिपपडस

3) Glucose

गिकह८ज

4) Cellulose

F A C E B O O K

P A G E h t t p w w w f a c e b o o k c o m s s c m e n t o r s o f f i c i a l P a g e | 31

FOR MORE UPDATES AND MORE MATERIAL DO LIKE OUR FACEBOOK PAGE httpwwwfacebookcomsscmentorsofficial

सहॳमयिह८ज

Correct Answer Glucose

Intestinal absorption of end products

from digestion of carbohydrates and

proteins in the pig hellip During absorption some sugars (fructose or

galactose) released from the

corresponding sucrose and lactose

respectively during digestion were

partly metabolized into glucose by the

enterocyte

Q130 Which of the following glands is a

source of the enzyme Ptyalin

नननननलिखितगरचियोमसहॳएजाइमटयालिनकासरह८तहहॴ 23-Jan-2017

Options

1) Pancreas

अगरािय

2) Thyroid Gland

िाइराइिगरिी 3) Pituitary Gland

पीयषगरिी 4) Salivary Glands

िारगरचियाा Correct Answer Salivary Glands

Q131 Which of the following is not true

about Pteridophyta

ननननमसहॳकह९नसीबातटहॳररिह८फाईटकहॳ बारहॳमसचनहीहहॴ 23-Jan-2017

Options

1) Dominant phase is saprophytes

परमिचरणसहॳपरह८फाईइटसहह८ताहहॴ 2) Main plant body is diploid

पह९दह८कामखयिरीरदपवगखणतहह८ताहहॴ 3) Seeds are present

बीजमह९जदहह८तहॳहहॴ 4)Flowers are absent

फिअनपनसतिहह८तहॳहहॴ

Correct Answer Seeds are present

Q132 The largest dolphin species is the

orca also called as

िॉिकफनकीसबसहॳबड़ीपरजानतकाकानामआकायहहॴनजसहॳ mdash- भीकहतहॳहहॴ 23-Jan-2017

Options

1) Bottle Nose

बाटिनह८ज

2) Baiji

बहॳजी 3) Killer whale

ककिरहहॳि

4)Tucuxi

टकवसी Correct Answer Killer whale

Q133 The fat digesting enzyme Lipase

is secreted by which of the following

वसाकापाचनकरनहॳवािाएजाइमिाइपहॳजनननननलिखितमसहॳककसकहॳ दवारासतरापवतहह८ताहहॴ

24-Jan-2017

Options

1) Kidneys

गद

2) Pancreas

अगनयािय

3) Large Intestine

बड़ीआत

4)Liver

नजगर

Correct Answer Pancreas

Lipase is an enzyme that splits fats so

the intestines can absorb them Lipase

hydrolyzes fats like triglycerides into

their component fatty acid and glycerol

molecules It is found in the blood

gastric juices pancreatic secretions

intestinal juices and adipose tissues

F A C E B O O K

P A G E h t t p w w w f a c e b o o k c o m s s c m e n t o r s o f f i c i a l P a g e | 32

FOR MORE UPDATES AND MORE MATERIAL DO LIKE OUR FACEBOOK PAGE httpwwwfacebookcomsscmentorsofficial

Q134 The arrangement of leaves on an

axis or stem is called

एकअकषयातनहॳपरपनततयोकीयवसिाकह८कयाकहाजाताहहॴ SSC CHSL Science (biology) 2016

Question Paper

24-Jan-2017

Options

1) Phyllotaxy

फाइिह८टहॴकसी 2) Vernation

वनिन

3) Venation

वहॳनहॳिन

4)Phytotaxy

फाइटह८टहॴकसी Correct Answer Phyllotaxy

In botany phyllotaxis or phyllotaxy is

the arrangement of leaves on a plant

stem (from Ancient Greek phyacutellon

ldquoleafrdquo and taacutexis ldquoarrangementrdquo)

Phyllotactic spirals form a distinctive

class of patterns in nature

Q135 The study of Cells is also known

as

कह८लिकाओकहॳ अधययनकह८ mdashmdashndash

भीकहाजाताहहॴ 24-Jan-2017

Options

1) Cytology

सायटह८िह८जी 2) Physiology

कफनजयह८िह८जी 3) Nucleology

नयककमयह८िह८जी 4)Cellology

सहॳिह८िह८जी Correct Answer Cytology

Q136 Which of the following scientists

is also known as the Father of Biology

नननननलिखितमसहॳककसवहॴजञाननककह८ ldquoजीवपवजञानकहॳ जनकrdquoकहॳ नामसहॳभीजानाजाताहहॴ 24-Jan-2017

Options

1) Herbert Spencer

हबयटयसपसर

2) Aristotle

अरसत 3) Lamarck

िहॳमाकय 4)Darwin

िापवयन

Correct Answer Aristotle

Q137 Which cells give rise to various

organs of the plant and keep the plant

growing

कह९नसीकह८लिकाएपह९धह८कहॳ लभननअगह८कह८जनमदहॳतीहहॴऔरपह९धह८कह८बढ़नहॳममददकरतीहहॴ

24-Jan-2017

Options

1) Permanent

सिायी 2) Dermal

तवचीय

3) Meristematic

मररसटहॳमटटक

4)Mature

परह८ढ़

Correct Answer Meristematic

A meristem is the tissue in most plants

containing undifferentiated cells

(meristematic cells) found in zones of

the plant where growth can take place

Q138 Rodentia Muridae is the scientific

name of

F A C E B O O K

P A G E h t t p w w w f a c e b o o k c o m s s c m e n t o r s o f f i c i a l P a g e | 33

FOR MORE UPDATES AND MORE MATERIAL DO LIKE OUR FACEBOOK PAGE httpwwwfacebookcomsscmentorsofficial

रह८िहॳलियानयररिी mdashmdash- कावहॴजञाननकनामहहॴ 24-

Jan-2017

Options

1) Mouse

चहा 2) Squirrel

चगिहरी 3) Monkey

बदर

4) Lizard

नछपकिी Correct Answer Mouse

Q139 Name the scientist who proposed

the cell theory

कह८लिकालसदातकापरसतावदहॳनहॳवािहॳवहॴजञाननककानामबताइए 24-Jan-2017

Options

1) Schleiden and Schwann

िीमिनऔरशरववान

2) Lamarck

िहॳमाकय 3) Treviranus

टरहॳवायरहॳनस

4)Whittaker and Stanley

हीटकरऔरसटहॳनिहॳ Correct Answer Schleiden and

Schwann

Q140 The flower with the worldrsquos

largest bloom is

दननयाकासबसहॳबड़ाफिखििनहॳवािा mdashmdashndash हहॴ 24-Jan-2017

Options

1) Pando

पािह८ 2) Posidonia

पह८सीिह८ननया 3) Rafflesia arnoldii

ररफिहॳलियाअनोमिी 4)Helianthus annuus

हहॳलिएनिसएनयअस

Correct Answer Rafflesia arnoldii

Rafflesia arnoldii is a species of

flowering plant in the parasitic genus

Rafflesia It is noted for producing the

largest individual flower on earth It has

a very strong and horrible odour of

decaying flesh earning it the nickname

ldquocorpse flower

Q141 Deficiency of which vitamin

causes night blindness

ककसपवटालमनकीकमीकहॳ कारणरतौधीहह८ताहहॴ 24-Jan-2017

Options

1) Vitamin K

पवटालमन K

2) Vitamin C

पवटालमन C

3) Vitamin B1

पवटालमन B1

4)Vitamin A

पवटालमन A

Correct Answer Vitamin A

Q142 Nongreen plants lack which of the

following

गहॴर-

हररतवनसपनतमनननननलिखितमसहॳककसकीकमीहह८तीहहॴ

24-Jan-2017

Options

1) Chlorophyll

किह८रह८कफि

2) Lycophyll

िायकह८कफि

3) Cyanophyll

F A C E B O O K

P A G E h t t p w w w f a c e b o o k c o m s s c m e n t o r s o f f i c i a l P a g e | 34

FOR MORE UPDATES AND MORE MATERIAL DO LIKE OUR FACEBOOK PAGE httpwwwfacebookcomsscmentorsofficial

सायनह८कफि

4)Phototropism

फह८टह८टरोपपजम

Correct Answer Chlorophyll

Q143 Organisms that use light to

prepare food are known as

जह८जीवपरकािकाउपयह८गकरभह८जनतहॴयारकरतहॳहहॴ उनह mdashmdash- कहॳ पमजानजाताहहॴ 24-Jan-2017

Options

1) Autotrophs

सवपह८षी 2) Heterotrophs

पवषमपह८षज

3) Omnivores

सवायहारी 4)Decomposers

पवघटनकरनहॳवािा Correct Answer Autotrophs

autotrophs often make their own food

by using sunlight carbon dioxide and

water to form sugars which they can use

for energy Some examples of

autotrophs include plants algae and

even some bacteria Autotrophs

(producer) are important because they

are a food source for heterotrophs

(consumers)

A heterotroph is an organism that

ingests or absorbs organic carbon

(rather than fix carbon from inorganic

sources such as carbon dioxide) in order

to be able to produce energy and

synthesize compounds to maintain its

life Ninety-five percent or more of all

types of living organisms are

heterotrophic including all animals and

fungi and some bacteria

Q144 Which of the following is a

primary function of haemoglobin

नननननलिखितमसहॳकह९नसाटहमह८गिह८बबनकाएकपरािलमककाययहहॴ

25-Jan-2017

Options

1) Utilization of energy

उजायकाउपयह८गकरना 2) Prevention of anaemia

रकतामपताहह८नहॳसहॳरह८कना 3) Destruction of bacteria

बहॴकटीररयाकापवनािकरना 4) To transport oxygen

ऑकसीजनकावहनकरना Correct Answer To transport oxygen

Q145 Vascular bundles are absent in

सवहनीबिि mdashmdash- मअनपनसतिरहतहॳहहॴ 25-Jan-2017

Options

1) Bryophyta

िायह८फाइटा 2) Pteridophyta

टहॳररिह८फाईटा 3) Gymnosperms

नजननह८सपमय 4) Angiosperms

एननजयह८सपहॳनसय Correct Answer Bryophyta

Q146 Sauria Lacertidae is the scientific

name of

सहॴररयािहॳसरटाईिी mdashmdashndash कावहॴजञाननकनामहहॴ 25-Jan-2017

Options

1) Crocodile

मगरमचछ

2) Hippopotamus

टहपपह८पह८टहॳमस

3) Lizard

नछपकिी 4) House fly

F A C E B O O K

P A G E h t t p w w w f a c e b o o k c o m s s c m e n t o r s o f f i c i a l P a g e | 35

FOR MORE UPDATES AND MORE MATERIAL DO LIKE OUR FACEBOOK PAGE httpwwwfacebookcomsscmentorsofficial

घरहॳिमकिी Correct Answer Lizard

Q147 Which type of pathogen causes

the water-borne disease SARS (Severe

Acute Respiratory Syndrome)

ककसपरकािकारह८गज़नकजिजननतबीमारीसासयकाकारणबनताहहॴ 25-Jan-2017

Options

1) Viral

वायरि

2) Parasitic

परजीवी 3) Protozoan

परह८टह८जअन

4) Bacterial

बहॴकटीररयि

Correct Answer Viral

Q148 Which of the following organs

produces the enzyme lipase

नननननलिखितमसहॳकह९नसाअगिायपहॳजएजाइमउतपननकरताहहॴ 25-Jan-2017

Options

1) Pancreas

अगनयािय

2) Large Intestine

बड़ीआत

3) Liver

नजगर

4) Small Intestine

छह८टीआत

Correct Answer Pancreas

Q149 A is a long internode forming the

basal part or the whole of a peduncle

एक mdashmdash- एकिबाइटरनह८िहहॴ जह८ननचिाटहससायासनपणयिठिबनताहहॴ 25-

Jan-2017

Options

1) Rhizome

परकद

2) Rachis

महॳ दि

3) floral axis

पषपअकष

4) Scape

भगदड़

Correct Answer scape

Q150 ndash Which of the following

organisms are considered to be both

Living and Non-living

नननननलिखितमसहॳकह९नसहॳजीवाणकह८जीपवतऔरअजीपवतमानाजाताहहॴ

25-Jan-2017

Options

1) Bacteria

बहॴकटीररया 2) Fungi

कवक

3) Algae

िहॴवाि

4)Virus

वायरस

Correct Answer Virus

They are considered to be living as they

possess a protein coat as a protective

covering DNA as the genetic material

etc

They are said to be non-living as they

can be crystallised and they survive for

billions of years They can tolerate high

temperatures freezing cold

temperatures ultra-violet radiations etc

Q151 Deficiency of fluorine causes

which of the following

फिह८ररनकीकमीकहॳ कारणनननननलिखितमसहॳकयाहह८ताहहॴ

F A C E B O O K

P A G E h t t p w w w f a c e b o o k c o m s s c m e n t o r s o f f i c i a l P a g e | 36

FOR MORE UPDATES AND MORE MATERIAL DO LIKE OUR FACEBOOK PAGE httpwwwfacebookcomsscmentorsofficial

27-Jan-2017

Options

1) Dental Caries

िटिकहॴ ररज

2) Scurvy

सकवरी 3) Anaemia

रकतामपता 4) Arthritis

गटठया Correct Answer Dental Caries

Q152 In a Punnett Square with the

cross AaBb x AaBb how many Aabb

genotypes would be created

पनहॳटसककायरमिह८स AaBb x AaBb कहॳ साि

ककतनहॳ Aabb जीनह८टाइपबनगहॳ 27-Jan-2017

Options

1) 1

2) 8

3) 2

4) 3

Correct Answer 2

Q153 Which of the following is the

Controlling Center of the Cell

नननननलिखित म सहॳ कह८लिकाका ननयतरण

क दर कह९न हहॴ

27-Jan-2017

Options

1) Nucleus

क दर

2) Plasma

पिाजमा 3) Lysosome

िायसह८सह८म

4) Chromosome

िह८मह८सह८म

Correct Answer Nucleus

The control centre of the cell is the

nucleus in eukaryotic cells The nucleus

contains genetic material in the form of

DNA

Q154 Myopia affects which of the

following organs

मायह८पपयानननननलिखितअगह८मसहॳककसहॳपरभापवतकरताहहॴ

25-Jan-2017

Options

1) Heart

हदय

2) Skin

तवचा 3) Eyes

आािहॳ 4)Mouth

मह

Correct Answer Eyes

Q155 Which of the following bears

flowers

नननननलिखितमसहॳकह९नफिधारणकरताहहॴ

25-Jan-2017

Options

1) Bryophyta

िायह८फाइटा 2) Pteridophyta

टहॳरीिह८फाईटा 3) Gymnosperms

नजननह८सपमय 4)Angiosperms

एननजयह८सपमय Correct Answer Angiosperms

Q156 Oxygenated blood flows out of the

heart through the

ऑकसीजनयकतरकत mdashmdashmdash

कहॳ माधयमसहॳहदयकहॳ बाहरबहताहहॴ 25-Jan-2017

F A C E B O O K

P A G E h t t p w w w f a c e b o o k c o m s s c m e n t o r s o f f i c i a l P a g e | 37

FOR MORE UPDATES AND MORE MATERIAL DO LIKE OUR FACEBOOK PAGE httpwwwfacebookcomsscmentorsofficial

Options

1) Aorta

महाधमनी 2) pulmonary artery

फहॳ फड़हॳकीधमनी 3) vena cava

वहॳनाकावा 4)Atrium

चह९क

Correct Answer aorta

Q157 Blood leaving the liver and

moving towards the

heart has a higher concentration of

नजगरसहॳननकिकरहदयकीतरफजानहॳवािहॳरकतम mdashmdashmdashmdash कीउचचसादरताहह८तीहहॴ 27-Jan-2017

Options

1) Lipids

लिपपडस

2) Urea

यररया 3) Bile Pigments

पपततकहॳ रगकरण

4) Carbon dioxide

काबयनिायऑकसाइि

Correct Answer Bile Pigments

Urea is nitrogen containing substance

which is produced in the liver in order

to deal with excess amino-acids in the

body As urea is produced it leaves the

liver in the blood stream and passes via

the circulatory system to all parts of the

body

Q158 Bulb is a modification of which

part of a plant

बमबएकपह९धहॳकहॳ ककसटहससहॳकाएक पातरणहह८ताहहॴ 27-Jan-2017

Options

1) The root

जड़

2) The stem

तना 3) The radicle

मिाकर

4)The fruit

फि

Correct Answer The stem

Q159 Which of the following carries

blood away from the heart to different

body parts

इनमहॳसहॳकह९नरकतकह८हदयसहॳिरीरकहॳ पवलभननअगह८तकिहॳजातीहहॴ

27-Jan-2017

Options

1) Arteries

धमननया 2) Nerves

तबतरहाए

3) Capillaries

कहॳ लिकाए

4)Veins

नसहॳ Correct Answer Arteries

Q160 The series of processes by which

nitrogen and its compounds are

interconverted in the environment and

in living organisms is called

27-Jan-2017

Options

1)Absorption of Nitrogen

2)Ammonification

3)Nitrogen Fixation

4)Nitrogen Cycle

Correct Answer Nitrogen Cycle

Ammonification or Mineralization is

performed by bacteria to convert

organic nitrogen to ammonia

F A C E B O O K

P A G E h t t p w w w f a c e b o o k c o m s s c m e n t o r s o f f i c i a l P a g e | 38

FOR MORE UPDATES AND MORE MATERIAL DO LIKE OUR FACEBOOK PAGE httpwwwfacebookcomsscmentorsofficial

Nitrification can then occur to convert

the ammonium to nitrite and nitrate

Nitrogen fixation is a process by which

nitrogen in the Earthrsquos atmosphere is

converted into ammonia (NH3) or other

molecules available to living organisms

Q161 BCG vaccine is given to protect

from which of the following

बीसीजीकाटटकानननननलिखितमसहॳककसकहॳ बचावकहॳ लिएटदयाजातहहॴ

27-Jan-2017

Options

1) Jaundice

पीलिया 2) Anaemia

रकतमपता 3) Tuberculosis

कषयरह८ग

4) Polio

पह८लियह८ Correct Answer Tuberculosis

Q162 Parallel venation is found in

समानतरवहॳनहॳिन mdashmdashmdash- मपायाजाताहहॴ 27-Jan-2017

Options

1) plants which are monocots

पह९धहॳजह८एकबीजपतरीहह८तहॳहहॴ 2) plants which have a dicot stem

वहॳपह९धहॳनजनकातनादपवदलियहह८ताहहॴ 3) plants with leaves similar to Tulsi

वहॳपह९धहॳनजनकीपनततयतिसीकीपनततयोकहॳ समानहह८तहॳहहॴ 4)plants with tap roots

टहॳप टवािहॳपह९धहॳ Correct Answer plants which are

monocots

Q163 The hardest part of the body is

िरीरकासबसहॳकठह८रभाग mdashndash हहॴ 27-Jan-2017

Options

1) Bones

हडडिय

2) Tooth Enamel

दातकहॳ इनहॳमि

3) Skull

िह८पड़ी 4) Spinal Cord

महॳ रजज

Correct Answer Tooth Enamel

Q164 Which type of pathogen causes

the waterborne disease E coli Infection

ककसपरकारकारह८गजननकजिजननतरह८गईकह८िाईसिमणकाकारणबनताहहॴ 27-Jan-2017

Options

1) Protozoan

परह८टह८जआ

2) Parasitic

परजीवी 3) Bacterial

बहॴकटीररयि

4)Viral

वायरि

Correct Answer Bacterial

Q165 The amount of blood filtered

together by both the kidneys in a 70 kg

adult male human in a minute is

70 की गरा वािहॳएकवयसकप षमएकलमनटमदह८नोगदकहॳदवाराएकसािचाबनीगयीरकतकीमातरहह८तीहहॴ 29-Jan-2017

Options

1) 1100 ml

1100 लमलि

2) 100 ml

F A C E B O O K

P A G E h t t p w w w f a c e b o o k c o m s s c m e n t o r s o f f i c i a l P a g e | 39

FOR MORE UPDATES AND MORE MATERIAL DO LIKE OUR FACEBOOK PAGE httpwwwfacebookcomsscmentorsofficial

100 लमलि

3) 1500 ml

1500 लमलि

4) 500 ml

500 लमलि

Correct Answer 1100 ml

Q166 Which feature of a plant helps to

distinguish a monocot from a dicot

पह९धहॳकीवहकह९नसीपविहॳषताहहॴजह८एकदपवदलियहॳऔरएकएकदिीयपह९धहॳसहॳभहॳदकरनहॳममददकरतीहहॴ 29-Jan-2017

Options

1) Pollination

परागम

2) Venation

वहॳनहॳिन

3) Vernation

वनिन

4) Aestivation

एसटीवहॳिहॳन

Correct Answer venation

Q167 The Mutation Theory was

proposed by

उतवररवतयनकालसदात mdashmdashndash

कहॳ दवरापरसतापवतककयाजाताहहॴ 29-Jan-2017

Options

1) Charles Lyell

चामसयलियहॳि

2) William Smith

पवलियमनसमि

3) Hugo De Vries

हयगह८िीराईस

4)Harrison Schmitt

हहॳरीसननसमट

Correct Answer Hugo De Vries

Q168 Which type of pathogen causes

the waterborne disease HepatitisA

ककसपरकारकहॳ रह८गजनकजिजननतरह८गहहॳपहॳटाइटटस-A काकारणबनताहहॴ

29-Jan-2017

Options

1) Parasitic

परजीवी 2) Viral

वायरि

3) Protozoan

परह८टह८जआ

4) Bacterial

बहॴकटीररयि

Correct Answer Viral

Q169 In a Punnett Square with the

cross AaBb x Aabb how many AaBb

genotypes would be created

पनहॳटसकवायरमिह८स AaBb x Aabb

कहॳ सािककतनहॳ AaBb जीनह८टाइपबनगहॳ 29-Jan-

2017

Options

1) 4

2) 1

3) 7

4) 6

Correct Answer 4

Q170 Arboreal Ateles is the scientific

name of

अिह८ररयिएटटलिस mdashmdashmdash कावहॴजञाननकनामहहॴ 29-Jan-2017

Options

1) Squirrel

चगिहरी 2) Sparrow

गह८रहॴया 3) Lizard

नछपकिी 4) Spider monkey

F A C E B O O K

P A G E h t t p w w w f a c e b o o k c o m s s c m e n t o r s o f f i c i a l P a g e | 40

FOR MORE UPDATES AND MORE MATERIAL DO LIKE OUR FACEBOOK PAGE httpwwwfacebookcomsscmentorsofficial

मकड़ीबदर

Correct Answer Spider monkey

Q171 Which type of pathogen causes

the waterborne disease Salmonellosis

ककसपरकारकारह८गाणजिजननतबीमारीसािमह८नहॳिह८लसज़काकारकहहॴ

29-Jan-2017

Options

1) Algal

िहॳवालियहॳ 2) Parasitic

परजीवी 3) Bacterial

बहॴकटीररयि

4)Viral

वायरि

Correct Answer Bacterial

An infection with salmonella bacteria

commonly caused by contaminated food

or water

Symptoms include diarrhoea fever

chills and abdominal pain

Q172 is a condition in which there is a

deficiency of red cells or of haemoglobin

in the blood

mdashmdash-

एकनसिनतहहॴनजसमहॳरकतमिािकह८लिकाओकीयाहीमह८गिह८बबनकीकमीहह८तीहहॴ 29-Jan-2017

Options

1) Albinism

एनमबननजम

2) Propyria

परह८पीररया 3) Anaemia

एनीलमया 4)Keloid disorder

कहॳ िह८इिडिसओिर

Correct Answer Anaemia

Q173 Ananas comosus is the scientific

name of

Options

अनानासकह८मह८सस mdashmdashmdashndash

कावहॴजञाननकनामहहॴ 29-Jan-2017

1) Custard Apple

सीताफि

2) Pineapple

पाइनएपपि

3) Bamboo

बास

4)Pomegranate

अनार

Correct Answer Pineapple

Q174 Which organ produces insulin

कह९नसाअगइनसलिनपहॴदाकरताहहॴ 29-Jan-

2017

Options

1) Liver

यकत

2) Thyroid gland

िायराइिगरिी 3) Spleen

पिीहा 4)Pancreas

अगरयिय

Correct Answer Pancreas

Q175 Which of the following disease is

not caused by water pollution

नननननलिखितमसहॳकह९नसारह८गपानीकहॳ परदषणकहॳकारणनहीहह८ता

29-Jan-2017

Options

1) Cholera

हहॴजा 2) Typhoid

F A C E B O O K

P A G E h t t p w w w f a c e b o o k c o m s s c m e n t o r s o f f i c i a l P a g e | 41

FOR MORE UPDATES AND MORE MATERIAL DO LIKE OUR FACEBOOK PAGE httpwwwfacebookcomsscmentorsofficial

टाइफाइि

3) Asthma

दमा 4)Diarrhoea

दसत

Correct Answer Asthma

Q176 Ocimum tenuiflorum is the

scientific name of

ओलिलममटहॳयईफिह८रमइसकावहॴजञाननकनाम mdash

ndash हहॴ 30-Jan-2017

Options

1) Neem

नीम

2) Mango

आम

3) Babul

बबि

4)Tulsi

तिसी Correct Answer Tulsi

Q177 Which gland secretes bile a

digestive fluid

कह९नसीगरिीपपतत एकपाचनतरिपरदािय सरापवतकरतीहहॴ 30-Jan-2017

Options

1) Pancreas

अगनयािय

2) Liver

यकत

3) Thyroid

िायराइि

4) Testes

टहॳनसटस

Correct Answer liver

Q178 In which of the following the

dominant phase is Gametophyte

नननननलिखितमसहॳककसकहॳ परमिचरणयगमकह८दपवधद (Gametophyte)हहॴ 30-Jan-2017

Options

1) Bryophyta

िायह८फाइटा 2) Pteridophyta

टहॳररिह८फाइटा 3) Gymnosperms

नजननह८सपमय 4) Angiosperms

एननजयह८सपमय Correct Answer Bryophyta

Q179 Anaerobic respiration refers to

which of the following

नननननलिखितमसहॳककसहॳअवायवीयशवसनकहाजाताहहॴ

30-Jan-2017

Options

1) Respiration without Oxygen

ऑकसीजनकहॳ बबनाशवसन

2) Respiration with Oxygen

ऑकसीजनकहॳ सािशवसन

3) Respiration without CO2

काबयनिायऑकसाइिकहॳ बबनाशवसन

4) Respiration with CO2

काबयनिायऑकसाइिकहॳ सािशविन

Correct Answer Respiration without

Oxygen

Q180 Which type of pathogen causes

the waterborne disease Cholera

ककसपरकारकारह८गजनकजिजननतरह८गहहॴजाकाकारणबनताहहॴ

30-Jan-2017

Options

1) Algal

िहॴवालियहॳ

F A C E B O O K

P A G E h t t p w w w f a c e b o o k c o m s s c m e n t o r s o f f i c i a l P a g e | 42

FOR MORE UPDATES AND MORE MATERIAL DO LIKE OUR FACEBOOK PAGE httpwwwfacebookcomsscmentorsofficial

2) Bacterial

बहॴकटीररयि

3) Protozoan

परह८टह८जआ

4) Viral

वायरि

Correct Answer Bacterial

Q181 To which class does

Oxyreductases transferases hydrolases

belong

ओकसीररिकटहॳसटरासफरहॳजहॳस

हाइडरह८िहॳसहॳसककसवगयमआतहॳहहॴ 30-Jan-2017

Options

1) Hormones

हारमोस

2) Enzymes

एजाइनस

3) Proteins

परह८टीनस

4) Vitamins

पवटालमनस

Correct Answer Enzymes

Q182 Which of the following is not true

about Gymnosperms

ननननमसहॳकह९नसीबातअनावतबीजीकहॳ बारहॳमसचनहीहहॴ 30-Jan-2017

Options

1) Dominant phase is saprophytes

परमिचरणसहॳपरह८फाइटसहह८ताहहॴ 2) Vascular bundles are absent

सवहनीबििअनपनसितहह८ताहहॴ 3) spores are heterospores

बीजाणहहॳटहॳरह८सपह८रसहह८तहॳहहॴ 4) Flowers are absent

फिअनपनसितहह८तहॳहहॴ

Correct Answer Vascular bundles are

absent

Q183 The name of first mammal clone sheep is

भहॳड़कीपरिमसतनपायीपरनत प (किह८न)

कानामहहॴ 30-Jan-2017

Options

1) Noori

नरी 2) Dolly

िॉिी 3) Louise

िसी 4)Durga

दगाय Correct Answer Dolly

Q184 Which type of pathogen causes

the water-borne disease Typhoid fever

ककसपरकारकारह८गजनकजिजननतरह८गटाइफाइिबिारकाकारणबनताहहॴ 30-Jan-2017

Options

1) Algal

िहॴवािीय

2) Parasitic

परजीवी 3) Protozoan

परह८टह८जनअन

4)Bacterial

बहॴकटीररयि

Correct Answer Bacterial

Q185 In which part of the cell are

proteins made

कह८लिकाकहॳ ककसटहससहॳमपरह८टीनबनायाजाताहहॴ

31-Jan-2017

Options

1) Reticulum

रहॳटटकिम

F A C E B O O K

P A G E h t t p w w w f a c e b o o k c o m s s c m e n t o r s o f f i c i a l P a g e | 43

FOR MORE UPDATES AND MORE MATERIAL DO LIKE OUR FACEBOOK PAGE httpwwwfacebookcomsscmentorsofficial

2) Golgi apparatus

गह८मजीएपहॳरहॳटस

3) Ribosomes

ररबह८सह८नस

4) Lysosome

िायसह८सह८नस

Correct Answer ribosomes

Proteins are produced by stringing

amino acids together in the order

specified by messenger RNA strands

that were transcribed from DNA in the

cell nucleus The process of synthesizing

a protein is called translation and it

occurs on ribosomes in the cytoplasm of

a cell

Q186 Polio is a disease caused by which

of the following

नननननलिखितमसहॳपह८लियह८कीबबमारह८हह८नहॳकाकारणकयाहहॴ

31-Jan-2017

Options

1) Bacteria

बहॴकटीररयि

2) Mosquito

मचछर

3) Virus

वायरस

4) Cockroach

नतिच हॳ Correct Answer Virus

Polio or poliomyelitis is a crippling and

potentially deadly infectious disease It

is caused by the poliovirus

Q187 ndash Hay fever is a sign of which of

the following

हहॳकफवरनननननलिखितमसहॳककसकाएकसकहॳ तहहॴ

31-Jan-2017

Options

1) Old Age

वदावसिा 2) Malnutrition

कपह८सण

3) Allergy

एिनजय 4) Over Work

अतयचधककाययकरना Correct Answer Allergy

Q188 How many chromosomes does a

human cell contain

एकमानवकह८लिकामककतनहॳगणसतरहह८तहॳहहॴ

29-Jan-2017

Options

1) 6

2) 26

3) 46

4) 66

Correct Answer 46

In humans each cell normally contains

23 pairs of chromosomes for a total of

46 Twenty-two of these pairs called

autosomes look the same in both males

and females The 23rd pair the sex

chromosomes differ between males and

females

Q189 Which of the following is not true

about Bryophyta

ननननमसहॳकह९नसीबातिायह८फाइटकहॳ बारहॳमसचनहीहहॴ 31-Jan-2017

Options

1) Dominant phase is gametophytes

परमिचरणगहॳलमतह८फाइटसहह८ताहहॴ 2) Main plant body is haploid

पह९धहॳकामखयिरीरअगखणतहह८ताहहॴ 3) Spores are homospores

बीजाणहह८मह८सफह८रसहह८तहॳहहॴ 4) Flowers are present

फिमह८जदहह८तहॳहहॴ Correct Answer Flowers are present

F A C E B O O K

P A G E h t t p w w w f a c e b o o k c o m s s c m e n t o r s o f f i c i a l P a g e | 44

FOR MORE UPDATES AND MORE MATERIAL DO LIKE OUR FACEBOOK PAGE httpwwwfacebookcomsscmentorsofficial

Q190 Which aquatic animal has

trailing tentacles

ककसजिीयजानवरकहॳ पीछहॳचिनहॳवािहॳटहॳटकिसहह८तहॳहहॴ

31-Jan-2017

Options

1) Sea horse

समदरीघह८िा 2) Corals

मगा 3) Jelly fish

जहॳिीमछिी 4) Star fish

तारामछिी Correct Answer Jelly fish

Jellyfish with its umbrella-shaped bell

and trailing tentacles

Q191 Which type of pathogen causes

the water-borne disease Poliomyelitis

(Polio)

ककसपरकारकारह८गजनकजिजननतरह८गपह८लियह८मायहॳटटस (पह८लियह८) काकारणहहॴ 31-Jan-

2017

Options

1) Parasitic

परजीवी 2) Algal

िहॴवालिय

3) Viral

वायरि

4) Bacterial

बहॴकटीररयि

Correct Answer Viral

Q192 The outer white part of the eye

that protects the inner structures is

आािकाबाहरीसफहॳ दटहससाजह८आतररकसरचनाओकीरकषाकरताहहॴ वह mdashmdashmdash हहॴ 31-Jan-

2017

Options

1) Iris

आयररस

2) Sclera

सकिहॳरा 3) Retina

रहॳटटना 4) Cornea

कह८ननयया Correct Answer Sclera

Q193 Proteins are made up of

परह८टीनकाननमायण mdashndash सहॳहह८ताहहॴ 31-Jan-2017

Options

1) Amino acids

एलमनह८अनि

2) Fatty acids

वसायकतअनि

3) Glucose

गिकह८ज

4)Nucleotides

नयनकियह८टाईिस

Correct Answer Amino acids

Q194 Moringa Oleifera is the scientific

name of

मह८ररगओलिफहॳ रा mdashmdashndash कावहॴजञाननकनामहहॴ 31-Jan-2017

Options

1) Banyan

बरगद

2) Gulmohar

गिमह८हर

3) Amla

आमिा

F A C E B O O K

P A G E h t t p w w w f a c e b o o k c o m s s c m e n t o r s o f f i c i a l P a g e | 45

FOR MORE UPDATES AND MORE MATERIAL DO LIKE OUR FACEBOOK PAGE httpwwwfacebookcomsscmentorsofficial

4) Drumstick

डरमनसटक

Correct Answer Drumstick

Q195 Kidney stones are composed of

गदकीपिरी mdashndash सहॳबनीहह८तीहहॴ 1-Feb-2017

Options

1) Calcium Oxalate

कहॴ नमसयमओकजहॳिहॳट

2) Sodium Chloride

सह८डियमकिह८राइि

3) Magnesium Nitrate

महॳनगनलियमनाइतटरहॳट

4) Calcium Bicarbonate

कहॴ नमियमबायकबोनहॳट

Correct Answer Calcium Oxalate

Q196 ndash Which of the following is not

true about Angiosperms

ननननमसहॳकह९नसीबातआवतबीजीकहॳ बारहॳमसचनहीहहॴ 1-Feb-2017

Options

1) Dominant phase is gametophytes

परमिचरणगहॳलमतह८फाइटहह८ताहहॴ 2) Vascular bundles are present

सवहनीबििमह९जदहह८ताहहॴ 3) Spores are heterospores

बीजाणहहॳटहॳरह८सपह८रसहह८तहॳहहॴ 4) Seeds are covered

बीजढकहॳ हह८तहॳहहॴ Correct Answer Dominant phase is

gametophytes

Q197 All of the following are excretory

(waste) products of animals except

नननननलिखितमसहॳककसएककह८छह८ड़करअनयसभीपराखणयोदवाराउतसनजयतपदाियहहॴ 1-Feb-

2017

Options

1) Uric Acid

यररकएलसि

2) Ammonia

अमह८ननया 3) Carbohydrates

काबोहाइडरहॳट

4) Urea

यररया Correct Answer Carbohydrates

In animals the main excretory products

are carbon dioxide ammonia (in

ammoniotelics) urea (in ureotelics) uric

acid (in uricotelics) guanine (in

Arachnida) and creatine

Q198 RNA is a polymeric molecule

What does RNA stand for

आरएनइएएकबहिकआणहहॴ इसकाकापवय पकयाहहॴ 1-Feb-2017

Options

1) Rado Nuclear Acid

रािह८नयनकियरएलसि

2) Ribo Nucleic Acid

राइबह८नयनकिकएलसि

3) Rhino Nuclear Acid

हाइनह८नयनकियरएलसि

4) Resto Nucleus Acid

रहॳसटह८नयकिीयसएलसि

Correct Answer Ribo Nucleic Acid

Q199 Which organ does detoxification

and produces chemicals needed for

digestion

कह९नसाअगपवषहरणकरताहहॴऔरपाचनकहॳ लिएआवशयकरसायनोकह८पहॴदाकरताहहॴ 1-Feb-

2017

Options

1) Salivary glands

िारगरचिया 2) Pancreas

अगनयािय

F A C E B O O K

P A G E h t t p w w w f a c e b o o k c o m s s c m e n t o r s o f f i c i a l P a g e | 46

FOR MORE UPDATES AND MORE MATERIAL DO LIKE OUR FACEBOOK PAGE httpwwwfacebookcomsscmentorsofficial

3) Thyroid gland

िायराइिगरिी 4) Liver

यकत

Correct Answer Liver

Q200 Psidium guajava is the scientific

name of

लसडियमगआजावा mdashmdash कावहॴजञाननकनामहहॴ 1-

Feb-2017

Options

1) Guava

अम द

2) Mango

आम

3) Bamboo

बास

4) Jack fruit

कटहि

Correct Answer Guava

Q201 Which drug is used as a Blood

Thinner

चधरकह८पतिाकरनहॳकहॳ पमककसदवाकापरयह८गककयाजाताहहॴ

1-Feb-2017

Options

1) Warfarin

वाफर न

2) Tramadol

टरहॳमािह८ि

3) Azithromycin

एनजरह८मायलसन

4) Hydralazine

हाइडरह८िहॳनजन

Correct Answer Warfarin

Q202 Which of the following disease is

caused due to the deficiency of protein

परह८टीनकीकमीकहॳ कारणनननननलिखितमसहॳकह९नसारह८गहह८ताहहॴ 1-Feb-2017

Options

1) Arthritis

गटठया 2) Kwashiorkor

कािीओकय र

3) Goitre

गाइटर

4) Night Blindness

रतह९चध

Correct Answer Kwashiorkor

Q203 A is species of plant that has

adapted to survive in an environment

with little liquid water

mdashmdashndashपह९धहॳकीएकऐसहॳऐसहॳपरजानतहहॴ नजसनहॳकमपानीवािहॳवातावरणमजीपवतरहनहॳकहॳलिएअनकिनहहॴ 1-Feb-2017

Options

1) Xerophyte

म दपवद

2) Hydrophyte

जिीयपादप

3) Mesophyte

समह८दपवद

4) Thallophyte

िहॴिह८फाइटा Correct Answer xerophyte

xerophyte is a species of plant that has

adapted to survive in an environment

with little liquid water such as a desert

or an ice- or snow-covered region in the

Alps or the Arctic

Mesophytes are terrestrial plants which

are adapted to neither a particularly

dry nor particularly wet environment

An example of a mesophytic habitat

would be a rural temperate meadow

F A C E B O O K

P A G E h t t p w w w f a c e b o o k c o m s s c m e n t o r s o f f i c i a l P a g e | 47

FOR MORE UPDATES AND MORE MATERIAL DO LIKE OUR FACEBOOK PAGE httpwwwfacebookcomsscmentorsofficial

which might contain goldenrod clover

oxeye daisy and Rosa multiflora

thallophyte any of a group of plants or

plantlike organisms (such as algae and

fungi) that lack differentiated stems

leaves and roots and that were formerly

classified as a primary division

(Thallophyta) of the plant kingdom

Q204 How many types of teeth are

there in humans

मनषयोमककतनहॳपरकारकहॳ दातहह८तहॳहहॴ

1-Feb-2017

Options

1) 4

2) 5

3) 2

4) 3

Correct Answer 4

teeth -Humans have four types of

teethincisors canines premolars and

molars each with a specific function

The incisors cut the food the canines

tear the food and the molars and

premolars crush the food

Q205 Carica papaya is the scientific name of

कहॴ ररकापपाया mdashmdashndash कावहॴजञाननकनामहहॴ 2-

Feb-2017

Options

1) Peepal

पीपि

2) Papaya

पपीता 3) Tamarind

इमिी 4) Drumstick

ढह८िकाछड़ी Correct Answer Papaya

Q206 Muscles get tired when there is

shortfall of

जब mdashndash कीकमीहह८तीहहॴतबपहॳिीयिकजातीहहॴ 2-Feb-2017

Options

1) Lactic acid

िहॴनकटकएलसि

2) Na+ ions

Na+ आयन

3) ATP

एटीपी 4) Sulphates

समफहॳ टस

Correct Answer ATP

ATP is the energy source muscle fibers

use to make muscles contract

muscle tissuersquos main source of energy

called adenosine triphosphate or ATP

As your muscles use up this energy

source they become tired and fatigued

Oxygen is the key ingredient that helps

create new ATP to replenish the burned

up ATP in your muscles

Q207 Artocarpus integra is the

scientific name of आटह८कापयसइटीगरा mdashmdashmdash कावहॴजञाननकनामहहॴ 2-Feb-2017

Options

1) Guava

अम द

2) Pineapple

अनानास

3) Silver Oak

लसमवरओक

4) Jack fruit

कटहि

Correct Answer Jack fruit

Q208 Which organ stores fat soluble

vitamins

कह९नसाअगवसामघिनिीिपवटालमनह८काभिाराकरताहहॴ

2-Feb-2017

F A C E B O O K

P A G E h t t p w w w f a c e b o o k c o m s s c m e n t o r s o f f i c i a l P a g e | 48

FOR MORE UPDATES AND MORE MATERIAL DO LIKE OUR FACEBOOK PAGE httpwwwfacebookcomsscmentorsofficial

Options

1) Blood

रकत

2) Skin

तवचा 3) Liver

यकत

4) Pancreas

अगनयािय

Correct Answer Liver

Q209 Which disease is caused due to

deficiency of Iodine

आयह८िीनकहॳ कारणकह९नसारह८गहह८ताहहॴ 2-Feb-2017

Options

1) Rickets

ररकहॳ टस

2) Scurvy

सकवी 3) Goitre

गणमािा 4) Growth retardation

पवकासका कना Correct Answer Goitre

rickets A softening and weakening of

bones in children usually due to

inadequate vitamin D

Q210 Grevillea Robusta is the scientific name of

गरहॳपवलियारह८बसटा mdashmdashmdash- कापवजञाननकनामहहॴ 2-Feb-2017

Options

1) Peepal

पीपि

2) Teak

सागह९न

3) Silver Oak

लसमवरओक

4) Jack fruit

कटहि

Correct Answer Silver Oak

Q211 When a Cuttlefish is described as a Molluscs it is at which level of

classification

जबएककटिकफिकह८एकमह८िसकाकहॳ पमवखणयतककयाजाताहहॴतबयहॳवगीकरणकहॳ ककससतरपहॳनसितहहॴ 2-Feb-2017

Options

1) Class

वगय 2) Order

िम

3) Family

पररवार

4) Phylum

सघ

Correct Answer Phylum

Q212 Bambusa dendrocalmus is the

scientific name of बानबसािहॳडराकामस mdashmdashmdash कावहॴजञाननकनामहहॴ 3-Feb-2017

Options

1) Banyan

बरगद

2) Papaya

पपीता 3) Bamboo

बास

4) Pomegranate

अनार

Correct Answer Bamboo

Q213 Acinonyx Jubatus is the scientific name of

एलसनह८ननकसजयबहॳटस mdashmdashmdash

कावहॴजञाननकनामहहॴ 3-Feb-2017

F A C E B O O K

P A G E h t t p w w w f a c e b o o k c o m s s c m e n t o r s o f f i c i a l P a g e | 49

FOR MORE UPDATES AND MORE MATERIAL DO LIKE OUR FACEBOOK PAGE httpwwwfacebookcomsscmentorsofficial

Options

1) Bear

भाि 2) Horse

घह८िा 3) Cheetah

चीता 4) Zebra

जहॳिा Correct Answer Cheetah

Q214 The pale yellow colour of urine is

due to the presence of which pigment

मतरकाफीकापीिारगरगदरयकहॳ उपनसिनतकहॳ कारणहह८ताहहॴ

3-Feb-2017

Options

1) Urochrome

यरह८िह८म

2) Urophyll

यरह८कफि

3) Chlorophyll

किह८रह८कफि

4) Chloroplast

किह८रह८पिासट

Correct Answer Urochrome

Q215 Which of the following constitute

to form a gene

नननननलिखितमसहॳकह९नसीचीज़एकजीनकागठनकरतीहहॴ

3-Feb-2017

Options

1) Polynucleotides

पह८िीनयनकियह८टाईडस

2) Hydrocarbons

हाइडरह८काबोस

3) Lipoproteins

िाईपह८परह८टीनस

4) Lipids

लिपपडस

Correct Answer Polynucleotides

Polynucleotide molecule is a biopolymer

composed of 13 or more nucleotide

monomers covalently bonded in a chain

DNA (deoxyribonucleic acid) and RNA

(ribonucleic acid) are examples of

polynucleotides with distinct biological

function

Q216 Vertebrates belongs to the

phylum

रीढ़कीहडिीवािहॳपराणी mdashmdashmdash

परजानतकहॳ अतगायतआतहॳहहॴ 3-Feb-2017

Options

1) Arthropoda

आरह८पह८ड़ा 2) Annelida

एननलििा 3) Cnidaria

ननिहॳररया 4) Chordata

कह८िटा Correct Answer Chordata

Q217 Punica granatum is the scientific name of

पननकगरहॳनहॳटस mdashmdashmdash कावहॴजञाननकनामहहॴ 3-Feb-2017

Options

1) Custard Apple

सीताफि

2) Gulmohar

गिमह८हर

3) Silver Oak

लसमवरओक

4) Pomegranate

अनार

Correct Answer Pomegranate

F A C E B O O K

P A G E h t t p w w w f a c e b o o k c o m s s c m e n t o r s o f f i c i a l P a g e | 50

FOR MORE UPDATES AND MORE MATERIAL DO LIKE OUR FACEBOOK PAGE httpwwwfacebookcomsscmentorsofficial

Q218 Between a tiger and an monkey

which of the following is different

एकबाघऔरबदरकहॳ बीचनननननलिखितमसहॳकह९नसीबातअिगहहॴ 3-Feb-2017

Options

1) Kingdom

राजय

2) Phylum

जानत

3) Order

िम

4) Class

वगय Correct Answer order

Q219 The artificial heart was invented by

कबतरमहदयका mdashmdashmdash

दवाराअपवषकारककयागयािा 3-Feb-2017

Options

1) Muhammad Yunus

महनमदयनस

2) Linus Yale Jr

िाइनसयहॳिजय

3) Gazi Yasargil

गाजीयासचगयि

4) Paul Winchell

पह९िपवमकि Correct Answer Paul Winchell

Q220 Tamarindus indica is the

scientific name of

टहॳमररनडसइडिका mdashmdash कावहॴजञाननकनामहहॴ 7-

Feb-2017

Options

1) Neem

नीम

2) Pineapple

अनानास

3) Tamarind

इमिी 4)Chiku

चीक

Correct Answer Tamarind

Q221 In eukaryotic cells synthesis of

RNA takes place in the

यकहॳ योटटककह८लिकाओमआरएनएकासशिहॳषण

mdashndash महह८ताहहॴ 7-Feb-2017

Options

1) Mitochondria

माईटह८कोडडरया 2) Centrioles

सटरीयह८मस

3) Ribosomes

ररबह८सह८नस

4) Nucleus

नयनकियस

Correct Answer nucleus

eukaryotic cell -Transcription is the

process of synthesizing ribonucleic acid

(RNA)Synthesis takes place within the

nucleus of eukaryotic cells or in the

cytoplasm of prokaryotes and converts

the genetic code from a gene in

deoxyribonucleic acid ( DNA ) to a

strand of RNA that then directs

proteinsynthesis

Q222 _________is caused by parasites

of the Plasmodium genus

पिाजमह८डियमजातीकहॳ परजीवी mdash- कहॳ कारणहहॴ 7-Feb-2017

Options

1) Dysentery

पहॳचचि

2) Malaria

मिहॳररया 3) Chickenpox

F A C E B O O K

P A G E h t t p w w w f a c e b o o k c o m s s c m e n t o r s o f f i c i a l P a g e | 51

FOR MORE UPDATES AND MORE MATERIAL DO LIKE OUR FACEBOOK PAGE httpwwwfacebookcomsscmentorsofficial

चहॳचक

4) Herpes

हहॳपपयस

Correct Answer Malaria

Q223 Carotene in fruits and vegetables

gives it which color

फिह८औरसनलजयोमनसितकहॳ रह८टीनउनहकह९नसारगपरदानकरताहहॴ 7-Feb-2017

Options

1) Green

हरा 2) Pink

गिाबी 3) Orange

नारगी 4) Blue

नीिा Correct Answer Orange

Q224 Equus Caballus is the scientific

name of

एकवसकहॴ बहॳिस mdashmdashndash कापवजञाननकनामहहॴ 7-Feb-2017

Options

1) Horse

घह८िा 2) Zebra

ज़हॳिा 3) Donkey

गधा 4) Buffalo

भस

Correct Answer Horse

Q225 Elapidae Naja is the scientific name of

एिीपीिीनाजा mdashmdash- कावहॴजञाननकनामहहॴ 8-Feb-2017

Options

1) Cobra

कह८बरा 2) Elephant

हािी 3) Eagle

ग ि

4) Owl

उमि Correct Answer Cobra

Q226 Which disease is caused due to

deficiency of Iron

िह८हकीकमीकहॳ कारणकह९नसारह८गहह८ताहहॴ 8-Feb-

2017

Options

1) Beriberi

बहॳरीबहॳरी 2) Tetany

टहॳटनी 3) Kwashiorkor

कवािीऔरकर

4) Anaemia

रकतामपता Correct Answer Anaemia

Beriberi is a disease caused by a vitamin

B-1 deficiency also known as thiamine

deficiency

Tetany can be the result of an

electrolyte imbalance Most often itrsquos a

dramatically low calcium level also

known as hypocalcemia Tetany can also

be caused by magnesium deficiency or

too little potassium Having too much

acid (acidosis) or too much alkali

(alkalosis) in the body can also result in

tetany

Kwashiorkor also known as

ldquoedematous malnutrition It is a form of

malnutrition caused by a lack of protein

in the diet

Anaemia means that you have fewer red

blood cells than normal or you have less

F A C E B O O K

P A G E h t t p w w w f a c e b o o k c o m s s c m e n t o r s o f f i c i a l P a g e | 52

FOR MORE UPDATES AND MORE MATERIAL DO LIKE OUR FACEBOOK PAGE httpwwwfacebookcomsscmentorsofficial

haemoglobin than normal in each red

blood cell

Q227 is a leaf where the leaflets are

arranged along the middle vein

mdashndashएकपततीहहॴजहापतरकह८कीरचनाक ररयालिराकहॳ आसपासहह८तीहहॴ 8-Feb-2017

Options

1) Pinnately compound leaf

पपनहॳटिीसयकतपतती 2) Palmately compound leaf

पामहॳटिीसयकतपतती 3) Compound leaf

सयकतपतती 4) Simple leaf

साधारणपतती Correct Answer Pinnately compound

leaf

Q228 Haustoria or sucking roots are

found in which of the following

हह८सटह८ररयायाचसनहॳवािीजड़हॳनननननलिखितमसहॳककसमपाईजातीहहॴ 8-Feb-2017

Options

1) Wheat

गहॳह

2) Mango

आम

3) Chestnut

चहॳसटनट

4) Cuscuta

कसकयटा Correct Answer Cuscuta

Haustorial roots -The roots of parasitic

plants which penetrate into the host

tissues to absorb nourishment are

called haustorial roots hellip Also known as suckingor parasitic roots

Q229 Equs Asinus is the scientific name

of

एकवसएलसनस mdashmdashndash कावहॴजञाननकनामहहॴ 8-

Feb-2017

Options

1) Donkey

गधा 2) Cow

गाय

3) Deer

टहरन

4) Kangaroo

कगा

Correct Answer Donkey

Q230 Ficus benghalensis is the scientific name of

फाईकसबहॳनगहॳिहॳलसस mdashndash कापवजञाननकनामहहॴ 8-Feb-2017

Options

1) Banyan

बरगद

2) Pineapple

अनानास

3) Babul

बबि

4) Tulsi

तिसी Correct Answer Banyan

Q231 Equus burchellii is the scientific name of

एकवसबचिी mdashmdash- कापवजञाननकनामहहॴ 8-Feb-2017

Options

1) Horse

घह८िा 2) Zebra

जहॳिा 3) Buffalo

F A C E B O O K

P A G E h t t p w w w f a c e b o o k c o m s s c m e n t o r s o f f i c i a l P a g e | 53

FOR MORE UPDATES AND MORE MATERIAL DO LIKE OUR FACEBOOK PAGE httpwwwfacebookcomsscmentorsofficial

भस

4) Ass

गधा Correct Answer Zebra

Page 19: COMPILATION OF ALL 72 SETS OF BIOLOGY SSC CHSL-2016 · OF BIOLOGY SSC CHSL-2016 PREPARED BY : SSC MENTORS BIOLOGY SPECIAL . F A C E B O O K P A G E : h t t p : / / w w w . f a c e

F A C E B O O K

P A G E h t t p w w w f a c e b o o k c o m s s c m e n t o r s o f f i c i a l P a g e | 18

FOR MORE UPDATES AND MORE MATERIAL DO LIKE OUR FACEBOOK PAGE httpwwwfacebookcomsscmentorsofficial

Options

1) Cat

बबमिी 2)Dog

कतता 3) Fox

िह८मड़ी 4) Wolf

भहॳडड़या Correct Answer Dog

Q71 Harmful bacteria in potable water

make the water

पीनहॳकहॳ पानीमनसतिघातकबहॴकटीररयाउसपानीकह८बनातहॳहहॴ 17-Jan-2017

Options

1) unfit to drink

पीनहॳकहॳ लिएअयह८गय

2) smelly

दगयनधयकत

3) Colored

रगीन

4) Turbid

मटमहॴिा Correct Answer unfit to drink

Q72 Musa paradisiaca is the scientific

name of which plant

मसापहॴराडिलसयाकाककसपह९धहॳकावहॴजञाननकनामहहॴ

17-Jan-2017

Options

1) Mango

आम

2) Wheat

गहॳह

3) Corn

भ ा 4) banana

कहॳ िा Correct Answer banana

Q73 Prawns belong to which family

झीगहॳककसपररवारकहॳ हह८तहॳहहॴ 17-Jan-2017

Options

1) Crustaceans

िसटहॳलियन

2)Fish

मछिी 3) Amphibians

अननफबबयस

4) Reptiles

रहॳपटाइमस

Correct Answer Crustaceans

Q74 Name the drug that is yielded from

Cinchona tree and is used to cure

malaria

उसऔषचधकानामबताइएनजसहॳलसगकह८नापहॳड़सहॳपरापतककयाजाताहहॴऔरनजसकाउपयह८गमिहॳररयाकहॳ उपचारमककयाजाताहहॴ 17-Jan-2017

Options

1) Camptothea

कहॴ नटह८चिया 2) Acuminata

एकयलमनहॳटा 3) Quinine

कनहॴन

4) Cinchonia

लसकह८ननया Correct Answer Quinine

Q75 Blood Circulation was discovered

by

रकतपररसचरणकी mdashmdashndash दवारािह८जकीिी 17-Jan-2017

Options

1) Mary Anderson

F A C E B O O K

P A G E h t t p w w w f a c e b o o k c o m s s c m e n t o r s o f f i c i a l P a g e | 19

FOR MORE UPDATES AND MORE MATERIAL DO LIKE OUR FACEBOOK PAGE httpwwwfacebookcomsscmentorsofficial

महॴरीएिरसन

2) Virginia Apgar

वनजयननयाएपगार

3) William Harvey

पवलियमहाव

4) Robert Feulgen

रॉबटयफ़यिजहॳन Correct Answer William Harvey

Q76 Vitamin A is also known as

पवटालमन A कह८ mdashmdash- कहॳ नामसहॳभीजानाजाताहहॴ SSC CHSL Science (biology) 2016

Question Paper

18Jan2017

Options

1) Thiamine

िायलमन

2) Riboflavin

ररबह८फिहॳपवन

3) Retinol

रहॳटटनॉि

4) Calciferol

कहॴ नमसफहॳ रह८ि

Correct Answer Retinol

Q77 Some roots called arise from an

organ other than the radicle

कछजड़हॳनजनह mdashmdashmdash कहाजाताहहॴ वहमिकहॳ अिावाककसीअनयअगसहॳउतपननहह८तीहहॴ 18Jan2017

Options

1) tap roots

मखयजड़

2) stilt roots

ि ाजड़

3) fibrous roots

रहॳिहॳदारजड़

4) adventitious roots

आकनसमकजड़

Correct Answer adventitious roots

Q78 Spiders belong to which class of

animals

मकडड़यापराणीवगीकरणकहॳ ककसवगयमआतीहहॴ 18Jan2017

Options

1) Arachnids

एरहॳकननडस

2) Aves

एपवस

3) Gastropods

गहॴसटरोपह८िस

4) Anthozoa

एिह८जआ

Correct Answer Arachnids

Q79 How many layers does Human

Skin have

मानवतवचामककतनीपरतहॳहह८तीहहॴ

18Jan2017

Options

1) 5

2) 7

3) 11

4) 3

Correct Answer 3

Skin has three layers The epidermis

the outermost layer of skin provides a

waterproof barrier and creates our skin

tone The dermis beneath the

epidermis contains tough connective

tissue hair follicles and sweat glands

The deeper subcutaneous tissue (

hypodermis ) is made of fat and

connective tissue

Q80 Allium Cepa is the scientific name

of

एलियमलसपपा mdashmdashndash कावहॴजञाननकनामहहॴ 18Jan2017

F A C E B O O K

P A G E h t t p w w w f a c e b o o k c o m s s c m e n t o r s o f f i c i a l P a g e | 20

FOR MORE UPDATES AND MORE MATERIAL DO LIKE OUR FACEBOOK PAGE httpwwwfacebookcomsscmentorsofficial

Options

1) Carrot

गाजर

2) Tomato

टमाटर

3) Potato

आि 4) Onion

पयाज़

Correct Answer Onion

Q81 DNA stands for

िीएनएकापणय प mdashmdash- हहॴ 18Jan2017

Options

1) Di Nucleic Acid

िाईनयनकिकएलसि

2) Deoxy Nucleic Acid

िीओकसीनयनकिकएलसि

3) Diribonucleic Acid

िाईराइबह८नयनकिकएलसि

4) Deoxyribonucleic Acid

िीऑकसीराइबह८नयनकिकएलसि

Correct Answer Deoxyribonucleic Acid

Q82 Organisms that generate energy

using light are known as

जह८जीवाणपरकािकाउपयह८गकरउजायउतपननकरतीहहॴ उनह mdashmdash कहॳ पमजानाजाताहहॴ

18Jan2017

Options

1) Chaemolithotrophs

ककमह८लििह८टरह८पस

2) Oligotrophs

ओलिगह८टरह८पस

3) Bacteria

बहॴकटीररया 4)Photoautotrophs

फह८टह८ओटह८टरह८पस

Correct Answer Photoautotrophs

An oligotroph is an organism that can

live in an environment that offers very

low levels of nutrients

Q83 Which drug is used as an

Antidepressant

ककसदवाएकहतािारह८धीकहॳ पमपयोगककयाजाताहहॴ Options

1) Oxybutynin

ओकसीलयटीनन

2)Tramadol

टरहॳमहॳिह८ि

3 ) Sumatriptan

समहॳटरीपटहॳन

4) Bupropion

लयपरह८पपयह८न

Correct Answer Bupropion

लयपरह८पपयह८न

Q84 The orange colour of carrot is

because of

गाजरकानारगीरगनननननलिखितमसहॳककसीएककीवजहसहॳहह८ताहहॴ 18Jan2017

Options

1) it grows in the soil

यहलम ीमउगतीहहॴ 2) Carotene

कहॴ रह८टीन

3) it is not exposed to sunlight

यहसययपरकािकहॳ सपकय मनहीआती 4) the entire plant is oranqe in colour

सनपणयपह९धानारगीरगकाहह८ताहहॴ Correct Answer Carotene

Q85 Snake venom is highly modified

saliva containing

F A C E B O O K

P A G E h t t p w w w f a c e b o o k c o m s s c m e n t o r s o f f i c i a l P a g e | 21

FOR MORE UPDATES AND MORE MATERIAL DO LIKE OUR FACEBOOK PAGE httpwwwfacebookcomsscmentorsofficial

सापकाजहरअततयाचधकसिह८चधतिारहह८तीहहॴनजसमहॳ mdashmdash- हह८ताहहॴ Options

l)Prototoxins

परह८टह८टॉनकसस

2)Neutrotoxins

नयटरोटॉनकसस

3)Zootoxins

जटॉनकसस

4)Electrotoxins

इिहॳकटरह८टॉनकसस

Correct Answer Zootoxins

जटॉनकसस

Q86 Which type of pathogen causes the

water-borne disease Schistosomiasis

ककसपरकारकारह८गज़नकजिजननतरह८गलससटह८सह८लमलससकाकारणबनताहहॴ

18Jan2017

Option

1) Parasitic

परजीवी 2)Protozoan

परह८टह८जआ

3) Bacterial

बहॴकटीररयि

4) Viral

वायरि

Correct Answer Parasitic

Schistosomiasis also known as snail

fever and bilharzia is a disease caused

by parasitic

flatworms called schistosomes

Q87 Prothrombin responsible for

clotting of blood is released by

परह८िह८ननबन

जह८रकतकािककाजमनहॳकहॳ लिएनजनमहॳदारहहॴ mdashndash

कहॳ दवारासतरापवतककयाजाताहहॴ

19Jan2017

Options

1) Small Intestine

छह८टीआत

2) Blood Platelets

रकतपिहॳटिहॳटस

3) Large Intestine

बड़ीआत

4Heart

हदय

Correct Answer Blood Platelets

Q88 Acacia arabica is the scientific

name of

अकहॳ लियाअरहॳबबका mdashmdashndash कावहॴजञाननकनामहहॴ 19-Jan-2017

Options

1) Neem

नीम

2) Teak

सागह९न

3) Babhul

बबि

4) Pomegranate

अनार

Correct Answer Babhul

Q89 Cannis Vulpes is the scientific

name of

कहॴ ननसवनमपस mdashmdash- कावहॴजञाननकनामहहॴ 19-Jan-2017

Options

1) Dog

कतता 2) Wolf

भहॳडड़या 3) Fox

िह८मड़ी 4) Hyena

िाकिबगघा

F A C E B O O K

P A G E h t t p w w w f a c e b o o k c o m s s c m e n t o r s o f f i c i a l P a g e | 22

FOR MORE UPDATES AND MORE MATERIAL DO LIKE OUR FACEBOOK PAGE httpwwwfacebookcomsscmentorsofficial

Correct Answer Fox

Q90 The beetroot is the portion of the

beet plant

चकदरपह९धहॳका mdashmdashndash भागहहॴ 19-Jan-2017

Options

1) tap root

मखयजड़

2) Adventitious

आकनसमक

3) bulb of the stem

तनहॳकाकद

4) Rhizome

परकद

Correct Answer tap root

Q91 What is the basic unit of heredity

आनवलिकताकीबननयादीइकाईकयाहहॴ 19-Jan-2017

Options

1) DNA

िीएनए

2) RNA

आरएनए

3) Chromosome

िह८मह८सह८म

4) Gene

जीन

Correct Answer gene

Genes are the units of heredity and are

the instructions that make up the bodyrsquos

blueprint They code for the proteins

that determine virtually all of a personrsquos

characteristics Most genes come in

pairs and are made of strands of genetic

material called deoxyribonucleic acid

or DNA

Q92 Lungs are the primary organs of

फहॳ फड़हॳmdashndashकहॳ परािलमकअगहहॴ

19-Jan-2017

Options

1) Digestion

पाचन

2) Constipation

कलज

3) Perspiration

पसीना 4)Respiration

शवसन

Correct Answer Respiration

Q93 Sugarcane is a type of

गननाएकपरकारका mdash- हहॴ 20-Jan-2017

Options

1)creeper

िता 2)tree

पहॳड़

3)shrub

झाड़ी 4)grass

घास

Correct Answer grass

Q94 Who is commonly known as ldquothe

Father of Microbiologyrdquo

सामानयत ldquo सकषमजीवपवजञानकहॳ जनक lsquo

कहॳ नामसहॳककसहॳजानाजातहहॴ 20-Jan-2017

Options

1) Robert Hooke

रॉबटयहक

2) Antonie Philips van Leeuwenhoek

एटह८नीकफलिपवानमयएनहह८क

3) Carl Linnaeus

काियिीनाईयस

4) Charles Darwin

चामसयिापवयन

F A C E B O O K

P A G E h t t p w w w f a c e b o o k c o m s s c m e n t o r s o f f i c i a l P a g e | 23

FOR MORE UPDATES AND MORE MATERIAL DO LIKE OUR FACEBOOK PAGE httpwwwfacebookcomsscmentorsofficial

Correct Answer Antonie Philips van

Leeuwenhoek

Q95 For the aquatic organisms the

source of food is

जिीयजीवाणकािाघसरह८तहहॴ 20-Jan-2017

Options

1) Phytoplankton

फायटह८पिहॳकटन

2) Sea Weed

समदरीिहॴवाि

3)Aqua plankton

एकवापिहॳकटन

4) Zooplankton

जपिहॳकटन

Correct Answer Phytoplankton

Q96 Haemoglobin has the highest

affinity with which of the following

हीमह८गिह८बबनकीननननमसहॳककसकहॳ सािउततमसमानताहहॴ

20-Jan-2017

Options

1)SO2

2)CO2

3)CO

4)NO2

Correct Answer CO

It has a greater affinity for hemoglobin

than oxygen does It displaces oxygen

and quickly binds so very little oxygen

is transported through the body cells

Q97 Who developed the theory of

Evolution

उदपवकासकालसदातककसनहॳपवकलसतककया

20-Jan-2017

Options

1) Charles Darwin

चामसयिापवयन

2) Isaac Newton

आयजहॳकनयटन

3) Pranav Mistry

परणवलमसतरी 4) Galileo Galilei

गहॳलिलियह८गहॳिीिी Correct Answer Charles Darwin

Q98 The primary function of RNA is

RNA कापरािलमककाययहह८ताहहॴ 20-Jan-2017

Options

1) Photosynthesis

परकािसशिहॳषण

2) Protein Synthesis

परह८टीनसशिहॳषण

3) Replication

परनतकनतबनाना 4) Translation

अनवादकरना Correct Answer Protein Synthesis

There are two main functions of RNA

It assists DNA by serving as a messenger

to relay the proper genetic information

to countless numbers of ribosomes in

your body The other main function of

RNA is to select the correct amino acid

needed by each ribosome to build new

proteins for your body

Q99 ______is the movement of

molecules across a cell membrane from

a region of their lower concentration to

a region of their higher concertration

उचचसादरताकहॳ कषहॳतरसहॳउसकीकमसादरतावािहॳकषहॳतरकीतरफएककह८लिकाखझमिीकहॳ माधयमसहॳहह८नहॳवािाअणओकहॳ सचिनकह८ mdash- कहतहॳहहॴ Options

1) Diffusion

पवसरण

2) Osmosis

ऑसमह८लसस

F A C E B O O K

P A G E h t t p w w w f a c e b o o k c o m s s c m e n t o r s o f f i c i a l P a g e | 24

FOR MORE UPDATES AND MORE MATERIAL DO LIKE OUR FACEBOOK PAGE httpwwwfacebookcomsscmentorsofficial

3) Active Transport

सकियआवागमन

4) Passive Transport

नननषियआवागमन

Correct Answer Active Transport

Q100 Study of classification of

organisms is known as 20-Jan-2017

जीवाणओकहॳ वगीकरणकहॳ अधययनकह८ mdash-

कहाजाताहहॴ Options

1) Serpentology

सपरहॳटह८िह८जी 2) Virology

वायरह८िह८जी 3) Taxonomy

टहॴकसोनह८मी 4) Physiology

कफनज़यह८िह८जी Correct Answer Taxonomy

Q101 Photosynthesis takes place inside

plant cells in

परकािसशिहॳषणवनसपनतकह८लिकामनसति mdash

mdashmdash महह८ताहहॴ 20-Jan-2017

Options

1) Ribosomes

राइबह८सह८नस

2) Chloroplasts

किह८रह८पिासट

3) Nucleus

नयकलियम

4) Mitochondria

माईटह८कोडडरया Correct Answer Chloroplasts

Q102 ______ is the cell organelle in

which the biochemical processes of

respiration and energy production

occur

mdashmdash- वहकह८लिकाअगहहॴ नजसमहॳशवसनऔरउजायउतपादनकहॳ जहॴसीजहॴवरासायननकपरकियायहह८तीहहॴ 20-Jan-2017

Options

1) Mitochondria

माइटह८कोडडरया 2) Chloroplast

किह८रह८पिासट

3) Ribosomes

राइबह८सह८नस

4) Nucleus

नयकिीयस

Correct Answer Mitochondria

Q103 Which non-flowering spore

bearing plants have roots

ककसफिनिगनहॳवािहॳऔरबीजाणधारकपह९धह८कीजड़हॳहह८तीहहॴ 21-Jan-2017

Options

1) Mosses

मह८सहॳस

2) Angiosperms

एननजयह८सपनसय 3) Ferns

फनसय 4) Gymnosperms

नजननह८सपनसय Correct Answer ferns

Q104 Which of the following is an

excretory organ of cockroach

नननननलिखितमसहॳकह९नसानतिच हॳकाउतसजयनअगहहॴ

21-Jan-2017

Options

F A C E B O O K

P A G E h t t p w w w f a c e b o o k c o m s s c m e n t o r s o f f i c i a l P a g e | 25

FOR MORE UPDATES AND MORE MATERIAL DO LIKE OUR FACEBOOK PAGE httpwwwfacebookcomsscmentorsofficial

1) Malphigian Tubules

मनमफनजयनटयबमस

2) Nephridia

नहॳकफरडिया 3) Coxal Gland

कह८कसिगरचिया 4) Green Gland

गरीनगरचिया Correct Answer Malphigian Tubules

Q105 Evaporation of water takes place

in which part of plants

पानीकहॳ वाषपीकरणकीकियापह९धोकहॳ ककसभागसहॳहह८तीहहॴ 21-Jan-2017

Options

1) Stem

तना 2) Stomata

सटह८मटा 3) Branch

िािाए

4) Fruit

फि

Correct Answer Stomata

Evaporation accounts for the movement

of water to the air from sources such as

the soil canopy interception and

waterbodies Transpiration accounts for

the movement of water within a plant

and the subsequent loss of water as

vapour through stomata in its leaves

Q106 A is the fleshy spore-bearing

fruiting body of a fungus

mdashmdashndashकवककामासि

बीजाणधारणकरनहॳवािाफिनहॳवािाअगहहॴ 21-

Jan-2017

Options

1) aloe vera

एिह८वहॳरा 2) Coral

मगा 3) Cactus

कहॴ कटस

4) Mushroom

ककरमतता Correct Answer mushroom

Q107 Which of the following is a fungal

disease

नननननलिखितमसहॳकह९नसाफफदसहॳहह८नहॳवािाएकरह८ग हहॴ

21-Jan-2017

Options

1) Dermatitis

तवचािह८ध

2) Cholera

हहॴजा 3) Jaundice

पीलिया 4) Indigofera

इननिगह८फहॳ रा Correct Answer Dermatitis

Dermatitis also known as eczema is a

group of diseases that results in

inflammation of the skin These diseases

are characterized by itchiness red skin

and a rash In cases of short duration

there may be small blisters while in

long-term cases the skin may become

thickened

Q108 In which form is glucose stored in

our body

हमारहॳिरीरमगिकह८जकासचयककस पमककयाजाताहहॴ

21-Jan-2017

Options

1) Insulin

F A C E B O O K

P A G E h t t p w w w f a c e b o o k c o m s s c m e n t o r s o f f i c i a l P a g e | 26

FOR MORE UPDATES AND MORE MATERIAL DO LIKE OUR FACEBOOK PAGE httpwwwfacebookcomsscmentorsofficial

इसलिन

2) Glucose

गिकह८ज

3) Glycogen

गिायकह८जहॳन

4) Fat

वसा Correct Answer Glycogen

Excess glucose is stored in the liver as

the large compound called glycogen

Glycogen is a polysaccharide of glucose

but its structure allows it to pack

compactly so more of it can be stored in

cells for later use

Q109 Where do plants synthesize

protein from

पह९धहॳपरह८टीनसशिहॳषणकहासहॳकरतहॳहहॴ

Options

1) Fatty Acids

वसाऐलसि

2) Sugar

िकर

3) Amino Acids

एलमनह८ऐलसि

4) Starch

सटाचय Correct Answer Amino Acids

Q110 Which part of the brain is

responsible for triggering actions like

thinking intelligence memory and

ability to learn

मनसतषककाकह९नसाटहससासह८चनहॳ बनधदमानी याददाशतऔरसीिनहॳकीकषमताजहॴसीकियाओकह८परहॳररतकरताहहॴ 21-Jan-2017

Options

1) Diencephalon

िायएनसहॳफहॳ िह८न

2) Hypothalamus

हयपह८िहॳिहॳमस

3) Cerebrum

सहॳरहॳिम

4) Control

कटरह८ि

Correct Answer Cerebrum

Q111 Which of the following is also

known as the Biochemical Laboratory

of the Human Body

नननननलिखितमसहॳककसहॳमानविरीरकीजहॴवरसायनपरयह८गिािाभीकहाजाताहहॴ 21-Jan-2017

Options

1) Small Intestine

छह८टीआत

2)Brain

मनसतषक

3) Pancreas

अगनयािय

4) Liver

नजगर

Correct Answer Liver

The liver makes bile that will help

emulsify and digest the fats we eat

The liver takes toxic substances and

convert them using enzymes the liver

cells makes into a non toxic form so the

body can dispose of them

The liver also converts fats protein and

carbohydrates into glucose which is the

energy source for our cells to use

The liver takes amino acids and makes

proteins by combining them

Q112 The yellow colour of human urine

is due to

मानवमतरकापीिारग mdashndash कीवजहसहॳहह८ताहहॴ 22-

Jan-2017

Options

1) Bile Salts

F A C E B O O K

P A G E h t t p w w w f a c e b o o k c o m s s c m e n t o r s o f f i c i a l P a g e | 27

FOR MORE UPDATES AND MORE MATERIAL DO LIKE OUR FACEBOOK PAGE httpwwwfacebookcomsscmentorsofficial

पपततनमक

2) Cholesterol

कह८िहॳसटरह८ि

3) Lymph

लिनफ

4) Urochrome

यरह८िह८म

Correct Answer Urochrome

Urobilin or urochrome is the chemical

primarily responsible for the yellow

color of urine

Q113 The wilting of plants takes place

due to

पह९धह८कालिचििहह८नाकी mdashmdash- कीवजहसहॳहह८ताहहॴ 22-Jan-2017

Options

1)Photosynthesis

परकािसशिहॳषण

2) Transpiration

वाषपह८तसजयन

3) Absorption

अविह८षण

4) Respiration

शरवसन

Correct Answer Transpiration

Wilting is the loss of rigidity of non-

woody parts of plants This occurs when

the turgor pressure in non-lignified

plant cells falls towards zero as a result

of diminished water in the cells

Q114 Bovidae Ovis is the scientific name of

बह८पविीओपवस mdashndash कावहॴजञाननकनामहहॴ 22-Jan-2017

Options

1) Goat

बकरी 2) Cow

गाय

3) Buffalo

भहॳस

4) Sheep

भहॳड़

Correct Answer Sheep

Q115 Plants get their energy to produce

food from which of the following

पह८धहॳभह८जनकाननमायणकरनहॳकहॳ लिएनननननलिखितमसहॳककससहॳउजायपरापतकरतहॳहहॴ

22-Jan-2017

Options

1) Photosynthesis

परकािसशिहॳषण

2)Bacteria

बहॴकटीररया 3)Fungi

कवक

4)Sun

सयय Correct Answer Sun

Q116 Which of the following is secreted

by the liver

नननननलिखितमसहॳककसकासरावनजगरसहॳहह८ताहहॴ

22-Jan-2017

Options

1) Glucose

गिकह८ज

2) Iodine

आयह८िीन

3) Cortisol

काटटरयसह८ि

4) Bile

पपतत

Correct Answer Bile

The liver makes bile that will help

emulsify and

digest the fats we eat

F A C E B O O K

P A G E h t t p w w w f a c e b o o k c o m s s c m e n t o r s o f f i c i a l P a g e | 28

FOR MORE UPDATES AND MORE MATERIAL DO LIKE OUR FACEBOOK PAGE httpwwwfacebookcomsscmentorsofficial

Q117 Ferns belong to which division of

plants

फनसयपह९धह८कहॳ ककसभागमआतहॳहहॴ

22-Jan-2017

Options

1) Gymnosperms

नजननह८सपनसय 2) Angiosperms

एनजयह८सपनसय 3) Thallophyta

िहॴिह८फाईटा 4)Pteridophyta

टहॳररिह८फाईटा Correct Answer Pteridophyta

Q118 Who invented Antibiotics

एटीबायह८टटककाअपवषकारककसनहॳककयािा

22-Jan-2017

Options

1) Joseph Lister

जह८सहॳफलिसटर

2) William Harvey

पवलियमहाव

3) Robert Knock

रॉबटयनॉक

4)Alexander Fleming

अिहॳकज़िरफिहॳलमग

Correct Answer Alexander Fleming

Q119 Milbecycin is used in the

eradication of

लममबहॳसायलसनका mdashndash

मउनमिनमपरयह८गककयाजाताहहॴ 22-Jan-2017

Options

1) Agricultural Fungus

कपषकवक

2) Agricultural Pests

कपषकीटक

3) Agricultural Herbs

कपषिाक

4)Agricultural Weeds

कपषननराना Correct Answer Agricultural Pests

Milbemycin oxime is a veterinary drug

from the group of milbemycins used as

a broad spectrum antiparasitic It is

active against worms and mites(insects

Q120 Intestinal bacteria synthesizes

which of the following in the human

body

मानविरीरमआतोकहॳ बहॴकटीररयानननननलिखितमसहॳककसकासशिहॳषणकरतहॳहहॴ 22-Jan-2017

Options

1) Vitamin K

पवटालमन K

2) Proteins

परह८टीन

3) Fats

वसा 4) Vitamin D

पवटालमन D

Correct Answer Vitamin K

Q121 is the study of the physical form

and external structure of plants

mdashmdash-

मपह९धह८काभहॴनतक पऔरबाहरीसरचनाकाआदयाककयाजाताहहॴ 22-Jan-2017

Options

1) Physiology

कफनजयह८िह८जी 2) Anatomy

िरीररचनापवजञान

3) Phytomorphology

फाईटह८मह८फह८िह८जी 4)Cytology

कह८लिकापवजञान

Correct Answer Phytomorphology

F A C E B O O K

P A G E h t t p w w w f a c e b o o k c o m s s c m e n t o r s o f f i c i a l P a g e | 29

FOR MORE UPDATES AND MORE MATERIAL DO LIKE OUR FACEBOOK PAGE httpwwwfacebookcomsscmentorsofficial

Q122 Which of the following is a

structural and functional unit of

kidneys

नननननलिखितमसहॳकह९नसीगदोकीसरचनातमकऔरकाययकरीईकाईहहॴ

22-Jan-2017

Options

1) Renette Cells

रहॳनहॳटकह८लिकाए

2) Flame Cells

फिहॳमकह८लिकाए

3) Nephrites

नहॳफ़राइटस

4)Nephrons

नहॳफरोस

Correct Answer Nephrons

Nephron functional unit of the kidney

the structure that actually produces

urine in the process of removing waste

and excess substances from the blood

There are about 1000000 nephrons in

each human kidney

Q123 Which of the following is the

largest part of the human brain

नननननलिखितमसहॳकह९नसामानवमनसतषककासबसहॳबड़ाटहससाहहॴ

23-Jan-2017

Options

1) Ribs

पसलियाा 2) Cerebrum

सहॳरहॳिम

3) Pons

पोस

4)Thalamus

िहॴिहॳमस

Correct Answer Cerebrum

The cerebrum is the largest part of the

human brain making up about two-

thirds of the brainrsquos mass It has two

hemispheres each of which has four

lobes frontal parietal temporal and

occipital

Q124 The auxiliary buds

सहायककालियाmdashndash 23-Jan-2017

Options

1) grow endogenously from the pericycle

पहॳरीसाईककिसहॳअनतजातयपवकलसतहह८ताहहॴ 2) arise endogenously from the main

growing point

मिवपदसहॳअनतजातयउठताहहॴ 3) is an embryonic shoot located in the

axil of a leaf

एकभरणिटहहॴजह८एकपततीकहॳ अकषपरनसतिहह८ताहहॴ 4)arise exogenously from the epidermis

एपपिलमयससहॳबटहजातयतरीकहॳ सहॳउठताहहॴ Correct Answer is an embryonic shoot

located in the axil of a leaf

Q125 Which of the following is a viral

disease

इनमहॳसहॳकह९सीएकवायरिबीमारीहहॴ

23-Jan-2017

Options

1) Polio

पह८लियह८ 2) Tetanus

धनसतनभ

3) Leprosy

कषठरह८ग

4) Plague

पिहॳग

Correct Answer Polio

A viral disease (or viral infection)

occurs when an organismrsquos body is

invaded by pathogenic viruses and

infectious virus particles (virions) attach

to and enter susceptible cells

F A C E B O O K

P A G E h t t p w w w f a c e b o o k c o m s s c m e n t o r s o f f i c i a l P a g e | 30

FOR MORE UPDATES AND MORE MATERIAL DO LIKE OUR FACEBOOK PAGE httpwwwfacebookcomsscmentorsofficial

Poliomyelitis often called polio or

infantile paralysis is an infectious

disease caused by the poliovirus

Tetanusmdash A serious bacterial infection

that causes painful muscle spasms and

can lead to death

Leprosy also known as Hansenrsquos

disease (HD) is a long-term infection by

the bacterium Mycobacterium leprae or

Mycobacterium lepromatosis

Plague is an infectious disease caused by

the bacterium Yersinia pestis

Symptoms include fever weakness and

headache

Q126 Which organisms can help to

carry out Vermicomposting

कह९नसाजीववमीकनपह८नसटगममददकरताहहॴ

23-Jan-2017

Options

1) Nitrifying Bacteria

नाईटरीफाईगबहॴकटीररया 2) Earthworms

कहॴ चऐ

3) Algae

िहॴवि

4) Fungus

कवक

Correct Answer Earthworms

Q127 Contraction of heart is also

known as

हदयकहॳ सकचनकह८ mdash- भीकहाजाताहहॴ 23-Jan-

2017

Options

1) Systole

लससटह८ि

2) Aristotle

अरसत

3) Diastole

िायसटह८ि

4) Lub

मयब

Correct Answer Systole

Diastole is the part of the cardiac cycle

when the heart refills with blood

following systole (contraction)

Ventricular diastole is the period during

which the ventricles are filling and

relaxing while atrial diastole is the

period during which the atria are

relaxing

Q128 Azadirachta indica is the

botanical name of which of the

following

अजाटदराचताइडिकानननननलिखितमसहॳककसकावानसपनतनामहहॴ

23-Jan-2017

Options

1) Rose plant

गिाबकापह९धा 2) Apple tree

सहॳबकापहॳड़

3) Neem

नीम

4)Mango

आम

Correct Answer Neem

Q129 Which of the following is the

main end product of carbohydrate

digestion

नननननलिखितमसहॳकह९नसाकाबोहाइडरहॳटकहॳ पाचनकापरमिअतउतपादकहह८ताहहॴ 23-Jan-2017

Options

1) Fats

वसा 2) Lipids

लिपपडस

3) Glucose

गिकह८ज

4) Cellulose

F A C E B O O K

P A G E h t t p w w w f a c e b o o k c o m s s c m e n t o r s o f f i c i a l P a g e | 31

FOR MORE UPDATES AND MORE MATERIAL DO LIKE OUR FACEBOOK PAGE httpwwwfacebookcomsscmentorsofficial

सहॳमयिह८ज

Correct Answer Glucose

Intestinal absorption of end products

from digestion of carbohydrates and

proteins in the pig hellip During absorption some sugars (fructose or

galactose) released from the

corresponding sucrose and lactose

respectively during digestion were

partly metabolized into glucose by the

enterocyte

Q130 Which of the following glands is a

source of the enzyme Ptyalin

नननननलिखितगरचियोमसहॳएजाइमटयालिनकासरह८तहहॴ 23-Jan-2017

Options

1) Pancreas

अगरािय

2) Thyroid Gland

िाइराइिगरिी 3) Pituitary Gland

पीयषगरिी 4) Salivary Glands

िारगरचियाा Correct Answer Salivary Glands

Q131 Which of the following is not true

about Pteridophyta

ननननमसहॳकह९नसीबातटहॳररिह८फाईटकहॳ बारहॳमसचनहीहहॴ 23-Jan-2017

Options

1) Dominant phase is saprophytes

परमिचरणसहॳपरह८फाईइटसहह८ताहहॴ 2) Main plant body is diploid

पह९दह८कामखयिरीरदपवगखणतहह८ताहहॴ 3) Seeds are present

बीजमह९जदहह८तहॳहहॴ 4)Flowers are absent

फिअनपनसतिहह८तहॳहहॴ

Correct Answer Seeds are present

Q132 The largest dolphin species is the

orca also called as

िॉिकफनकीसबसहॳबड़ीपरजानतकाकानामआकायहहॴनजसहॳ mdash- भीकहतहॳहहॴ 23-Jan-2017

Options

1) Bottle Nose

बाटिनह८ज

2) Baiji

बहॳजी 3) Killer whale

ककिरहहॳि

4)Tucuxi

टकवसी Correct Answer Killer whale

Q133 The fat digesting enzyme Lipase

is secreted by which of the following

वसाकापाचनकरनहॳवािाएजाइमिाइपहॳजनननननलिखितमसहॳककसकहॳ दवारासतरापवतहह८ताहहॴ

24-Jan-2017

Options

1) Kidneys

गद

2) Pancreas

अगनयािय

3) Large Intestine

बड़ीआत

4)Liver

नजगर

Correct Answer Pancreas

Lipase is an enzyme that splits fats so

the intestines can absorb them Lipase

hydrolyzes fats like triglycerides into

their component fatty acid and glycerol

molecules It is found in the blood

gastric juices pancreatic secretions

intestinal juices and adipose tissues

F A C E B O O K

P A G E h t t p w w w f a c e b o o k c o m s s c m e n t o r s o f f i c i a l P a g e | 32

FOR MORE UPDATES AND MORE MATERIAL DO LIKE OUR FACEBOOK PAGE httpwwwfacebookcomsscmentorsofficial

Q134 The arrangement of leaves on an

axis or stem is called

एकअकषयातनहॳपरपनततयोकीयवसिाकह८कयाकहाजाताहहॴ SSC CHSL Science (biology) 2016

Question Paper

24-Jan-2017

Options

1) Phyllotaxy

फाइिह८टहॴकसी 2) Vernation

वनिन

3) Venation

वहॳनहॳिन

4)Phytotaxy

फाइटह८टहॴकसी Correct Answer Phyllotaxy

In botany phyllotaxis or phyllotaxy is

the arrangement of leaves on a plant

stem (from Ancient Greek phyacutellon

ldquoleafrdquo and taacutexis ldquoarrangementrdquo)

Phyllotactic spirals form a distinctive

class of patterns in nature

Q135 The study of Cells is also known

as

कह८लिकाओकहॳ अधययनकह८ mdashmdashndash

भीकहाजाताहहॴ 24-Jan-2017

Options

1) Cytology

सायटह८िह८जी 2) Physiology

कफनजयह८िह८जी 3) Nucleology

नयककमयह८िह८जी 4)Cellology

सहॳिह८िह८जी Correct Answer Cytology

Q136 Which of the following scientists

is also known as the Father of Biology

नननननलिखितमसहॳककसवहॴजञाननककह८ ldquoजीवपवजञानकहॳ जनकrdquoकहॳ नामसहॳभीजानाजाताहहॴ 24-Jan-2017

Options

1) Herbert Spencer

हबयटयसपसर

2) Aristotle

अरसत 3) Lamarck

िहॳमाकय 4)Darwin

िापवयन

Correct Answer Aristotle

Q137 Which cells give rise to various

organs of the plant and keep the plant

growing

कह९नसीकह८लिकाएपह९धह८कहॳ लभननअगह८कह८जनमदहॳतीहहॴऔरपह९धह८कह८बढ़नहॳममददकरतीहहॴ

24-Jan-2017

Options

1) Permanent

सिायी 2) Dermal

तवचीय

3) Meristematic

मररसटहॳमटटक

4)Mature

परह८ढ़

Correct Answer Meristematic

A meristem is the tissue in most plants

containing undifferentiated cells

(meristematic cells) found in zones of

the plant where growth can take place

Q138 Rodentia Muridae is the scientific

name of

F A C E B O O K

P A G E h t t p w w w f a c e b o o k c o m s s c m e n t o r s o f f i c i a l P a g e | 33

FOR MORE UPDATES AND MORE MATERIAL DO LIKE OUR FACEBOOK PAGE httpwwwfacebookcomsscmentorsofficial

रह८िहॳलियानयररिी mdashmdash- कावहॴजञाननकनामहहॴ 24-

Jan-2017

Options

1) Mouse

चहा 2) Squirrel

चगिहरी 3) Monkey

बदर

4) Lizard

नछपकिी Correct Answer Mouse

Q139 Name the scientist who proposed

the cell theory

कह८लिकालसदातकापरसतावदहॳनहॳवािहॳवहॴजञाननककानामबताइए 24-Jan-2017

Options

1) Schleiden and Schwann

िीमिनऔरशरववान

2) Lamarck

िहॳमाकय 3) Treviranus

टरहॳवायरहॳनस

4)Whittaker and Stanley

हीटकरऔरसटहॳनिहॳ Correct Answer Schleiden and

Schwann

Q140 The flower with the worldrsquos

largest bloom is

दननयाकासबसहॳबड़ाफिखििनहॳवािा mdashmdashndash हहॴ 24-Jan-2017

Options

1) Pando

पािह८ 2) Posidonia

पह८सीिह८ननया 3) Rafflesia arnoldii

ररफिहॳलियाअनोमिी 4)Helianthus annuus

हहॳलिएनिसएनयअस

Correct Answer Rafflesia arnoldii

Rafflesia arnoldii is a species of

flowering plant in the parasitic genus

Rafflesia It is noted for producing the

largest individual flower on earth It has

a very strong and horrible odour of

decaying flesh earning it the nickname

ldquocorpse flower

Q141 Deficiency of which vitamin

causes night blindness

ककसपवटालमनकीकमीकहॳ कारणरतौधीहह८ताहहॴ 24-Jan-2017

Options

1) Vitamin K

पवटालमन K

2) Vitamin C

पवटालमन C

3) Vitamin B1

पवटालमन B1

4)Vitamin A

पवटालमन A

Correct Answer Vitamin A

Q142 Nongreen plants lack which of the

following

गहॴर-

हररतवनसपनतमनननननलिखितमसहॳककसकीकमीहह८तीहहॴ

24-Jan-2017

Options

1) Chlorophyll

किह८रह८कफि

2) Lycophyll

िायकह८कफि

3) Cyanophyll

F A C E B O O K

P A G E h t t p w w w f a c e b o o k c o m s s c m e n t o r s o f f i c i a l P a g e | 34

FOR MORE UPDATES AND MORE MATERIAL DO LIKE OUR FACEBOOK PAGE httpwwwfacebookcomsscmentorsofficial

सायनह८कफि

4)Phototropism

फह८टह८टरोपपजम

Correct Answer Chlorophyll

Q143 Organisms that use light to

prepare food are known as

जह८जीवपरकािकाउपयह८गकरभह८जनतहॴयारकरतहॳहहॴ उनह mdashmdash- कहॳ पमजानजाताहहॴ 24-Jan-2017

Options

1) Autotrophs

सवपह८षी 2) Heterotrophs

पवषमपह८षज

3) Omnivores

सवायहारी 4)Decomposers

पवघटनकरनहॳवािा Correct Answer Autotrophs

autotrophs often make their own food

by using sunlight carbon dioxide and

water to form sugars which they can use

for energy Some examples of

autotrophs include plants algae and

even some bacteria Autotrophs

(producer) are important because they

are a food source for heterotrophs

(consumers)

A heterotroph is an organism that

ingests or absorbs organic carbon

(rather than fix carbon from inorganic

sources such as carbon dioxide) in order

to be able to produce energy and

synthesize compounds to maintain its

life Ninety-five percent or more of all

types of living organisms are

heterotrophic including all animals and

fungi and some bacteria

Q144 Which of the following is a

primary function of haemoglobin

नननननलिखितमसहॳकह९नसाटहमह८गिह८बबनकाएकपरािलमककाययहहॴ

25-Jan-2017

Options

1) Utilization of energy

उजायकाउपयह८गकरना 2) Prevention of anaemia

रकतामपताहह८नहॳसहॳरह८कना 3) Destruction of bacteria

बहॴकटीररयाकापवनािकरना 4) To transport oxygen

ऑकसीजनकावहनकरना Correct Answer To transport oxygen

Q145 Vascular bundles are absent in

सवहनीबिि mdashmdash- मअनपनसतिरहतहॳहहॴ 25-Jan-2017

Options

1) Bryophyta

िायह८फाइटा 2) Pteridophyta

टहॳररिह८फाईटा 3) Gymnosperms

नजननह८सपमय 4) Angiosperms

एननजयह८सपहॳनसय Correct Answer Bryophyta

Q146 Sauria Lacertidae is the scientific

name of

सहॴररयािहॳसरटाईिी mdashmdashndash कावहॴजञाननकनामहहॴ 25-Jan-2017

Options

1) Crocodile

मगरमचछ

2) Hippopotamus

टहपपह८पह८टहॳमस

3) Lizard

नछपकिी 4) House fly

F A C E B O O K

P A G E h t t p w w w f a c e b o o k c o m s s c m e n t o r s o f f i c i a l P a g e | 35

FOR MORE UPDATES AND MORE MATERIAL DO LIKE OUR FACEBOOK PAGE httpwwwfacebookcomsscmentorsofficial

घरहॳिमकिी Correct Answer Lizard

Q147 Which type of pathogen causes

the water-borne disease SARS (Severe

Acute Respiratory Syndrome)

ककसपरकािकारह८गज़नकजिजननतबीमारीसासयकाकारणबनताहहॴ 25-Jan-2017

Options

1) Viral

वायरि

2) Parasitic

परजीवी 3) Protozoan

परह८टह८जअन

4) Bacterial

बहॴकटीररयि

Correct Answer Viral

Q148 Which of the following organs

produces the enzyme lipase

नननननलिखितमसहॳकह९नसाअगिायपहॳजएजाइमउतपननकरताहहॴ 25-Jan-2017

Options

1) Pancreas

अगनयािय

2) Large Intestine

बड़ीआत

3) Liver

नजगर

4) Small Intestine

छह८टीआत

Correct Answer Pancreas

Q149 A is a long internode forming the

basal part or the whole of a peduncle

एक mdashmdash- एकिबाइटरनह८िहहॴ जह८ननचिाटहससायासनपणयिठिबनताहहॴ 25-

Jan-2017

Options

1) Rhizome

परकद

2) Rachis

महॳ दि

3) floral axis

पषपअकष

4) Scape

भगदड़

Correct Answer scape

Q150 ndash Which of the following

organisms are considered to be both

Living and Non-living

नननननलिखितमसहॳकह९नसहॳजीवाणकह८जीपवतऔरअजीपवतमानाजाताहहॴ

25-Jan-2017

Options

1) Bacteria

बहॴकटीररया 2) Fungi

कवक

3) Algae

िहॴवाि

4)Virus

वायरस

Correct Answer Virus

They are considered to be living as they

possess a protein coat as a protective

covering DNA as the genetic material

etc

They are said to be non-living as they

can be crystallised and they survive for

billions of years They can tolerate high

temperatures freezing cold

temperatures ultra-violet radiations etc

Q151 Deficiency of fluorine causes

which of the following

फिह८ररनकीकमीकहॳ कारणनननननलिखितमसहॳकयाहह८ताहहॴ

F A C E B O O K

P A G E h t t p w w w f a c e b o o k c o m s s c m e n t o r s o f f i c i a l P a g e | 36

FOR MORE UPDATES AND MORE MATERIAL DO LIKE OUR FACEBOOK PAGE httpwwwfacebookcomsscmentorsofficial

27-Jan-2017

Options

1) Dental Caries

िटिकहॴ ररज

2) Scurvy

सकवरी 3) Anaemia

रकतामपता 4) Arthritis

गटठया Correct Answer Dental Caries

Q152 In a Punnett Square with the

cross AaBb x AaBb how many Aabb

genotypes would be created

पनहॳटसककायरमिह८स AaBb x AaBb कहॳ साि

ककतनहॳ Aabb जीनह८टाइपबनगहॳ 27-Jan-2017

Options

1) 1

2) 8

3) 2

4) 3

Correct Answer 2

Q153 Which of the following is the

Controlling Center of the Cell

नननननलिखित म सहॳ कह८लिकाका ननयतरण

क दर कह९न हहॴ

27-Jan-2017

Options

1) Nucleus

क दर

2) Plasma

पिाजमा 3) Lysosome

िायसह८सह८म

4) Chromosome

िह८मह८सह८म

Correct Answer Nucleus

The control centre of the cell is the

nucleus in eukaryotic cells The nucleus

contains genetic material in the form of

DNA

Q154 Myopia affects which of the

following organs

मायह८पपयानननननलिखितअगह८मसहॳककसहॳपरभापवतकरताहहॴ

25-Jan-2017

Options

1) Heart

हदय

2) Skin

तवचा 3) Eyes

आािहॳ 4)Mouth

मह

Correct Answer Eyes

Q155 Which of the following bears

flowers

नननननलिखितमसहॳकह९नफिधारणकरताहहॴ

25-Jan-2017

Options

1) Bryophyta

िायह८फाइटा 2) Pteridophyta

टहॳरीिह८फाईटा 3) Gymnosperms

नजननह८सपमय 4)Angiosperms

एननजयह८सपमय Correct Answer Angiosperms

Q156 Oxygenated blood flows out of the

heart through the

ऑकसीजनयकतरकत mdashmdashmdash

कहॳ माधयमसहॳहदयकहॳ बाहरबहताहहॴ 25-Jan-2017

F A C E B O O K

P A G E h t t p w w w f a c e b o o k c o m s s c m e n t o r s o f f i c i a l P a g e | 37

FOR MORE UPDATES AND MORE MATERIAL DO LIKE OUR FACEBOOK PAGE httpwwwfacebookcomsscmentorsofficial

Options

1) Aorta

महाधमनी 2) pulmonary artery

फहॳ फड़हॳकीधमनी 3) vena cava

वहॳनाकावा 4)Atrium

चह९क

Correct Answer aorta

Q157 Blood leaving the liver and

moving towards the

heart has a higher concentration of

नजगरसहॳननकिकरहदयकीतरफजानहॳवािहॳरकतम mdashmdashmdashmdash कीउचचसादरताहह८तीहहॴ 27-Jan-2017

Options

1) Lipids

लिपपडस

2) Urea

यररया 3) Bile Pigments

पपततकहॳ रगकरण

4) Carbon dioxide

काबयनिायऑकसाइि

Correct Answer Bile Pigments

Urea is nitrogen containing substance

which is produced in the liver in order

to deal with excess amino-acids in the

body As urea is produced it leaves the

liver in the blood stream and passes via

the circulatory system to all parts of the

body

Q158 Bulb is a modification of which

part of a plant

बमबएकपह९धहॳकहॳ ककसटहससहॳकाएक पातरणहह८ताहहॴ 27-Jan-2017

Options

1) The root

जड़

2) The stem

तना 3) The radicle

मिाकर

4)The fruit

फि

Correct Answer The stem

Q159 Which of the following carries

blood away from the heart to different

body parts

इनमहॳसहॳकह९नरकतकह८हदयसहॳिरीरकहॳ पवलभननअगह८तकिहॳजातीहहॴ

27-Jan-2017

Options

1) Arteries

धमननया 2) Nerves

तबतरहाए

3) Capillaries

कहॳ लिकाए

4)Veins

नसहॳ Correct Answer Arteries

Q160 The series of processes by which

nitrogen and its compounds are

interconverted in the environment and

in living organisms is called

27-Jan-2017

Options

1)Absorption of Nitrogen

2)Ammonification

3)Nitrogen Fixation

4)Nitrogen Cycle

Correct Answer Nitrogen Cycle

Ammonification or Mineralization is

performed by bacteria to convert

organic nitrogen to ammonia

F A C E B O O K

P A G E h t t p w w w f a c e b o o k c o m s s c m e n t o r s o f f i c i a l P a g e | 38

FOR MORE UPDATES AND MORE MATERIAL DO LIKE OUR FACEBOOK PAGE httpwwwfacebookcomsscmentorsofficial

Nitrification can then occur to convert

the ammonium to nitrite and nitrate

Nitrogen fixation is a process by which

nitrogen in the Earthrsquos atmosphere is

converted into ammonia (NH3) or other

molecules available to living organisms

Q161 BCG vaccine is given to protect

from which of the following

बीसीजीकाटटकानननननलिखितमसहॳककसकहॳ बचावकहॳ लिएटदयाजातहहॴ

27-Jan-2017

Options

1) Jaundice

पीलिया 2) Anaemia

रकतमपता 3) Tuberculosis

कषयरह८ग

4) Polio

पह८लियह८ Correct Answer Tuberculosis

Q162 Parallel venation is found in

समानतरवहॳनहॳिन mdashmdashmdash- मपायाजाताहहॴ 27-Jan-2017

Options

1) plants which are monocots

पह९धहॳजह८एकबीजपतरीहह८तहॳहहॴ 2) plants which have a dicot stem

वहॳपह९धहॳनजनकातनादपवदलियहह८ताहहॴ 3) plants with leaves similar to Tulsi

वहॳपह९धहॳनजनकीपनततयतिसीकीपनततयोकहॳ समानहह८तहॳहहॴ 4)plants with tap roots

टहॳप टवािहॳपह९धहॳ Correct Answer plants which are

monocots

Q163 The hardest part of the body is

िरीरकासबसहॳकठह८रभाग mdashndash हहॴ 27-Jan-2017

Options

1) Bones

हडडिय

2) Tooth Enamel

दातकहॳ इनहॳमि

3) Skull

िह८पड़ी 4) Spinal Cord

महॳ रजज

Correct Answer Tooth Enamel

Q164 Which type of pathogen causes

the waterborne disease E coli Infection

ककसपरकारकारह८गजननकजिजननतरह८गईकह८िाईसिमणकाकारणबनताहहॴ 27-Jan-2017

Options

1) Protozoan

परह८टह८जआ

2) Parasitic

परजीवी 3) Bacterial

बहॴकटीररयि

4)Viral

वायरि

Correct Answer Bacterial

Q165 The amount of blood filtered

together by both the kidneys in a 70 kg

adult male human in a minute is

70 की गरा वािहॳएकवयसकप षमएकलमनटमदह८नोगदकहॳदवाराएकसािचाबनीगयीरकतकीमातरहह८तीहहॴ 29-Jan-2017

Options

1) 1100 ml

1100 लमलि

2) 100 ml

F A C E B O O K

P A G E h t t p w w w f a c e b o o k c o m s s c m e n t o r s o f f i c i a l P a g e | 39

FOR MORE UPDATES AND MORE MATERIAL DO LIKE OUR FACEBOOK PAGE httpwwwfacebookcomsscmentorsofficial

100 लमलि

3) 1500 ml

1500 लमलि

4) 500 ml

500 लमलि

Correct Answer 1100 ml

Q166 Which feature of a plant helps to

distinguish a monocot from a dicot

पह९धहॳकीवहकह९नसीपविहॳषताहहॴजह८एकदपवदलियहॳऔरएकएकदिीयपह९धहॳसहॳभहॳदकरनहॳममददकरतीहहॴ 29-Jan-2017

Options

1) Pollination

परागम

2) Venation

वहॳनहॳिन

3) Vernation

वनिन

4) Aestivation

एसटीवहॳिहॳन

Correct Answer venation

Q167 The Mutation Theory was

proposed by

उतवररवतयनकालसदात mdashmdashndash

कहॳ दवरापरसतापवतककयाजाताहहॴ 29-Jan-2017

Options

1) Charles Lyell

चामसयलियहॳि

2) William Smith

पवलियमनसमि

3) Hugo De Vries

हयगह८िीराईस

4)Harrison Schmitt

हहॳरीसननसमट

Correct Answer Hugo De Vries

Q168 Which type of pathogen causes

the waterborne disease HepatitisA

ककसपरकारकहॳ रह८गजनकजिजननतरह८गहहॳपहॳटाइटटस-A काकारणबनताहहॴ

29-Jan-2017

Options

1) Parasitic

परजीवी 2) Viral

वायरि

3) Protozoan

परह८टह८जआ

4) Bacterial

बहॴकटीररयि

Correct Answer Viral

Q169 In a Punnett Square with the

cross AaBb x Aabb how many AaBb

genotypes would be created

पनहॳटसकवायरमिह८स AaBb x Aabb

कहॳ सािककतनहॳ AaBb जीनह८टाइपबनगहॳ 29-Jan-

2017

Options

1) 4

2) 1

3) 7

4) 6

Correct Answer 4

Q170 Arboreal Ateles is the scientific

name of

अिह८ररयिएटटलिस mdashmdashmdash कावहॴजञाननकनामहहॴ 29-Jan-2017

Options

1) Squirrel

चगिहरी 2) Sparrow

गह८रहॴया 3) Lizard

नछपकिी 4) Spider monkey

F A C E B O O K

P A G E h t t p w w w f a c e b o o k c o m s s c m e n t o r s o f f i c i a l P a g e | 40

FOR MORE UPDATES AND MORE MATERIAL DO LIKE OUR FACEBOOK PAGE httpwwwfacebookcomsscmentorsofficial

मकड़ीबदर

Correct Answer Spider monkey

Q171 Which type of pathogen causes

the waterborne disease Salmonellosis

ककसपरकारकारह८गाणजिजननतबीमारीसािमह८नहॳिह८लसज़काकारकहहॴ

29-Jan-2017

Options

1) Algal

िहॳवालियहॳ 2) Parasitic

परजीवी 3) Bacterial

बहॴकटीररयि

4)Viral

वायरि

Correct Answer Bacterial

An infection with salmonella bacteria

commonly caused by contaminated food

or water

Symptoms include diarrhoea fever

chills and abdominal pain

Q172 is a condition in which there is a

deficiency of red cells or of haemoglobin

in the blood

mdashmdash-

एकनसिनतहहॴनजसमहॳरकतमिािकह८लिकाओकीयाहीमह८गिह८बबनकीकमीहह८तीहहॴ 29-Jan-2017

Options

1) Albinism

एनमबननजम

2) Propyria

परह८पीररया 3) Anaemia

एनीलमया 4)Keloid disorder

कहॳ िह८इिडिसओिर

Correct Answer Anaemia

Q173 Ananas comosus is the scientific

name of

Options

अनानासकह८मह८सस mdashmdashmdashndash

कावहॴजञाननकनामहहॴ 29-Jan-2017

1) Custard Apple

सीताफि

2) Pineapple

पाइनएपपि

3) Bamboo

बास

4)Pomegranate

अनार

Correct Answer Pineapple

Q174 Which organ produces insulin

कह९नसाअगइनसलिनपहॴदाकरताहहॴ 29-Jan-

2017

Options

1) Liver

यकत

2) Thyroid gland

िायराइिगरिी 3) Spleen

पिीहा 4)Pancreas

अगरयिय

Correct Answer Pancreas

Q175 Which of the following disease is

not caused by water pollution

नननननलिखितमसहॳकह९नसारह८गपानीकहॳ परदषणकहॳकारणनहीहह८ता

29-Jan-2017

Options

1) Cholera

हहॴजा 2) Typhoid

F A C E B O O K

P A G E h t t p w w w f a c e b o o k c o m s s c m e n t o r s o f f i c i a l P a g e | 41

FOR MORE UPDATES AND MORE MATERIAL DO LIKE OUR FACEBOOK PAGE httpwwwfacebookcomsscmentorsofficial

टाइफाइि

3) Asthma

दमा 4)Diarrhoea

दसत

Correct Answer Asthma

Q176 Ocimum tenuiflorum is the

scientific name of

ओलिलममटहॳयईफिह८रमइसकावहॴजञाननकनाम mdash

ndash हहॴ 30-Jan-2017

Options

1) Neem

नीम

2) Mango

आम

3) Babul

बबि

4)Tulsi

तिसी Correct Answer Tulsi

Q177 Which gland secretes bile a

digestive fluid

कह९नसीगरिीपपतत एकपाचनतरिपरदािय सरापवतकरतीहहॴ 30-Jan-2017

Options

1) Pancreas

अगनयािय

2) Liver

यकत

3) Thyroid

िायराइि

4) Testes

टहॳनसटस

Correct Answer liver

Q178 In which of the following the

dominant phase is Gametophyte

नननननलिखितमसहॳककसकहॳ परमिचरणयगमकह८दपवधद (Gametophyte)हहॴ 30-Jan-2017

Options

1) Bryophyta

िायह८फाइटा 2) Pteridophyta

टहॳररिह८फाइटा 3) Gymnosperms

नजननह८सपमय 4) Angiosperms

एननजयह८सपमय Correct Answer Bryophyta

Q179 Anaerobic respiration refers to

which of the following

नननननलिखितमसहॳककसहॳअवायवीयशवसनकहाजाताहहॴ

30-Jan-2017

Options

1) Respiration without Oxygen

ऑकसीजनकहॳ बबनाशवसन

2) Respiration with Oxygen

ऑकसीजनकहॳ सािशवसन

3) Respiration without CO2

काबयनिायऑकसाइिकहॳ बबनाशवसन

4) Respiration with CO2

काबयनिायऑकसाइिकहॳ सािशविन

Correct Answer Respiration without

Oxygen

Q180 Which type of pathogen causes

the waterborne disease Cholera

ककसपरकारकारह८गजनकजिजननतरह८गहहॴजाकाकारणबनताहहॴ

30-Jan-2017

Options

1) Algal

िहॴवालियहॳ

F A C E B O O K

P A G E h t t p w w w f a c e b o o k c o m s s c m e n t o r s o f f i c i a l P a g e | 42

FOR MORE UPDATES AND MORE MATERIAL DO LIKE OUR FACEBOOK PAGE httpwwwfacebookcomsscmentorsofficial

2) Bacterial

बहॴकटीररयि

3) Protozoan

परह८टह८जआ

4) Viral

वायरि

Correct Answer Bacterial

Q181 To which class does

Oxyreductases transferases hydrolases

belong

ओकसीररिकटहॳसटरासफरहॳजहॳस

हाइडरह८िहॳसहॳसककसवगयमआतहॳहहॴ 30-Jan-2017

Options

1) Hormones

हारमोस

2) Enzymes

एजाइनस

3) Proteins

परह८टीनस

4) Vitamins

पवटालमनस

Correct Answer Enzymes

Q182 Which of the following is not true

about Gymnosperms

ननननमसहॳकह९नसीबातअनावतबीजीकहॳ बारहॳमसचनहीहहॴ 30-Jan-2017

Options

1) Dominant phase is saprophytes

परमिचरणसहॳपरह८फाइटसहह८ताहहॴ 2) Vascular bundles are absent

सवहनीबििअनपनसितहह८ताहहॴ 3) spores are heterospores

बीजाणहहॳटहॳरह८सपह८रसहह८तहॳहहॴ 4) Flowers are absent

फिअनपनसितहह८तहॳहहॴ

Correct Answer Vascular bundles are

absent

Q183 The name of first mammal clone sheep is

भहॳड़कीपरिमसतनपायीपरनत प (किह८न)

कानामहहॴ 30-Jan-2017

Options

1) Noori

नरी 2) Dolly

िॉिी 3) Louise

िसी 4)Durga

दगाय Correct Answer Dolly

Q184 Which type of pathogen causes

the water-borne disease Typhoid fever

ककसपरकारकारह८गजनकजिजननतरह८गटाइफाइिबिारकाकारणबनताहहॴ 30-Jan-2017

Options

1) Algal

िहॴवािीय

2) Parasitic

परजीवी 3) Protozoan

परह८टह८जनअन

4)Bacterial

बहॴकटीररयि

Correct Answer Bacterial

Q185 In which part of the cell are

proteins made

कह८लिकाकहॳ ककसटहससहॳमपरह८टीनबनायाजाताहहॴ

31-Jan-2017

Options

1) Reticulum

रहॳटटकिम

F A C E B O O K

P A G E h t t p w w w f a c e b o o k c o m s s c m e n t o r s o f f i c i a l P a g e | 43

FOR MORE UPDATES AND MORE MATERIAL DO LIKE OUR FACEBOOK PAGE httpwwwfacebookcomsscmentorsofficial

2) Golgi apparatus

गह८मजीएपहॳरहॳटस

3) Ribosomes

ररबह८सह८नस

4) Lysosome

िायसह८सह८नस

Correct Answer ribosomes

Proteins are produced by stringing

amino acids together in the order

specified by messenger RNA strands

that were transcribed from DNA in the

cell nucleus The process of synthesizing

a protein is called translation and it

occurs on ribosomes in the cytoplasm of

a cell

Q186 Polio is a disease caused by which

of the following

नननननलिखितमसहॳपह८लियह८कीबबमारह८हह८नहॳकाकारणकयाहहॴ

31-Jan-2017

Options

1) Bacteria

बहॴकटीररयि

2) Mosquito

मचछर

3) Virus

वायरस

4) Cockroach

नतिच हॳ Correct Answer Virus

Polio or poliomyelitis is a crippling and

potentially deadly infectious disease It

is caused by the poliovirus

Q187 ndash Hay fever is a sign of which of

the following

हहॳकफवरनननननलिखितमसहॳककसकाएकसकहॳ तहहॴ

31-Jan-2017

Options

1) Old Age

वदावसिा 2) Malnutrition

कपह८सण

3) Allergy

एिनजय 4) Over Work

अतयचधककाययकरना Correct Answer Allergy

Q188 How many chromosomes does a

human cell contain

एकमानवकह८लिकामककतनहॳगणसतरहह८तहॳहहॴ

29-Jan-2017

Options

1) 6

2) 26

3) 46

4) 66

Correct Answer 46

In humans each cell normally contains

23 pairs of chromosomes for a total of

46 Twenty-two of these pairs called

autosomes look the same in both males

and females The 23rd pair the sex

chromosomes differ between males and

females

Q189 Which of the following is not true

about Bryophyta

ननननमसहॳकह९नसीबातिायह८फाइटकहॳ बारहॳमसचनहीहहॴ 31-Jan-2017

Options

1) Dominant phase is gametophytes

परमिचरणगहॳलमतह८फाइटसहह८ताहहॴ 2) Main plant body is haploid

पह९धहॳकामखयिरीरअगखणतहह८ताहहॴ 3) Spores are homospores

बीजाणहह८मह८सफह८रसहह८तहॳहहॴ 4) Flowers are present

फिमह८जदहह८तहॳहहॴ Correct Answer Flowers are present

F A C E B O O K

P A G E h t t p w w w f a c e b o o k c o m s s c m e n t o r s o f f i c i a l P a g e | 44

FOR MORE UPDATES AND MORE MATERIAL DO LIKE OUR FACEBOOK PAGE httpwwwfacebookcomsscmentorsofficial

Q190 Which aquatic animal has

trailing tentacles

ककसजिीयजानवरकहॳ पीछहॳचिनहॳवािहॳटहॳटकिसहह८तहॳहहॴ

31-Jan-2017

Options

1) Sea horse

समदरीघह८िा 2) Corals

मगा 3) Jelly fish

जहॳिीमछिी 4) Star fish

तारामछिी Correct Answer Jelly fish

Jellyfish with its umbrella-shaped bell

and trailing tentacles

Q191 Which type of pathogen causes

the water-borne disease Poliomyelitis

(Polio)

ककसपरकारकारह८गजनकजिजननतरह८गपह८लियह८मायहॳटटस (पह८लियह८) काकारणहहॴ 31-Jan-

2017

Options

1) Parasitic

परजीवी 2) Algal

िहॴवालिय

3) Viral

वायरि

4) Bacterial

बहॴकटीररयि

Correct Answer Viral

Q192 The outer white part of the eye

that protects the inner structures is

आािकाबाहरीसफहॳ दटहससाजह८आतररकसरचनाओकीरकषाकरताहहॴ वह mdashmdashmdash हहॴ 31-Jan-

2017

Options

1) Iris

आयररस

2) Sclera

सकिहॳरा 3) Retina

रहॳटटना 4) Cornea

कह८ननयया Correct Answer Sclera

Q193 Proteins are made up of

परह८टीनकाननमायण mdashndash सहॳहह८ताहहॴ 31-Jan-2017

Options

1) Amino acids

एलमनह८अनि

2) Fatty acids

वसायकतअनि

3) Glucose

गिकह८ज

4)Nucleotides

नयनकियह८टाईिस

Correct Answer Amino acids

Q194 Moringa Oleifera is the scientific

name of

मह८ररगओलिफहॳ रा mdashmdashndash कावहॴजञाननकनामहहॴ 31-Jan-2017

Options

1) Banyan

बरगद

2) Gulmohar

गिमह८हर

3) Amla

आमिा

F A C E B O O K

P A G E h t t p w w w f a c e b o o k c o m s s c m e n t o r s o f f i c i a l P a g e | 45

FOR MORE UPDATES AND MORE MATERIAL DO LIKE OUR FACEBOOK PAGE httpwwwfacebookcomsscmentorsofficial

4) Drumstick

डरमनसटक

Correct Answer Drumstick

Q195 Kidney stones are composed of

गदकीपिरी mdashndash सहॳबनीहह८तीहहॴ 1-Feb-2017

Options

1) Calcium Oxalate

कहॴ नमसयमओकजहॳिहॳट

2) Sodium Chloride

सह८डियमकिह८राइि

3) Magnesium Nitrate

महॳनगनलियमनाइतटरहॳट

4) Calcium Bicarbonate

कहॴ नमियमबायकबोनहॳट

Correct Answer Calcium Oxalate

Q196 ndash Which of the following is not

true about Angiosperms

ननननमसहॳकह९नसीबातआवतबीजीकहॳ बारहॳमसचनहीहहॴ 1-Feb-2017

Options

1) Dominant phase is gametophytes

परमिचरणगहॳलमतह८फाइटहह८ताहहॴ 2) Vascular bundles are present

सवहनीबििमह९जदहह८ताहहॴ 3) Spores are heterospores

बीजाणहहॳटहॳरह८सपह८रसहह८तहॳहहॴ 4) Seeds are covered

बीजढकहॳ हह८तहॳहहॴ Correct Answer Dominant phase is

gametophytes

Q197 All of the following are excretory

(waste) products of animals except

नननननलिखितमसहॳककसएककह८छह८ड़करअनयसभीपराखणयोदवाराउतसनजयतपदाियहहॴ 1-Feb-

2017

Options

1) Uric Acid

यररकएलसि

2) Ammonia

अमह८ननया 3) Carbohydrates

काबोहाइडरहॳट

4) Urea

यररया Correct Answer Carbohydrates

In animals the main excretory products

are carbon dioxide ammonia (in

ammoniotelics) urea (in ureotelics) uric

acid (in uricotelics) guanine (in

Arachnida) and creatine

Q198 RNA is a polymeric molecule

What does RNA stand for

आरएनइएएकबहिकआणहहॴ इसकाकापवय पकयाहहॴ 1-Feb-2017

Options

1) Rado Nuclear Acid

रािह८नयनकियरएलसि

2) Ribo Nucleic Acid

राइबह८नयनकिकएलसि

3) Rhino Nuclear Acid

हाइनह८नयनकियरएलसि

4) Resto Nucleus Acid

रहॳसटह८नयकिीयसएलसि

Correct Answer Ribo Nucleic Acid

Q199 Which organ does detoxification

and produces chemicals needed for

digestion

कह९नसाअगपवषहरणकरताहहॴऔरपाचनकहॳ लिएआवशयकरसायनोकह८पहॴदाकरताहहॴ 1-Feb-

2017

Options

1) Salivary glands

िारगरचिया 2) Pancreas

अगनयािय

F A C E B O O K

P A G E h t t p w w w f a c e b o o k c o m s s c m e n t o r s o f f i c i a l P a g e | 46

FOR MORE UPDATES AND MORE MATERIAL DO LIKE OUR FACEBOOK PAGE httpwwwfacebookcomsscmentorsofficial

3) Thyroid gland

िायराइिगरिी 4) Liver

यकत

Correct Answer Liver

Q200 Psidium guajava is the scientific

name of

लसडियमगआजावा mdashmdash कावहॴजञाननकनामहहॴ 1-

Feb-2017

Options

1) Guava

अम द

2) Mango

आम

3) Bamboo

बास

4) Jack fruit

कटहि

Correct Answer Guava

Q201 Which drug is used as a Blood

Thinner

चधरकह८पतिाकरनहॳकहॳ पमककसदवाकापरयह८गककयाजाताहहॴ

1-Feb-2017

Options

1) Warfarin

वाफर न

2) Tramadol

टरहॳमािह८ि

3) Azithromycin

एनजरह८मायलसन

4) Hydralazine

हाइडरह८िहॳनजन

Correct Answer Warfarin

Q202 Which of the following disease is

caused due to the deficiency of protein

परह८टीनकीकमीकहॳ कारणनननननलिखितमसहॳकह९नसारह८गहह८ताहहॴ 1-Feb-2017

Options

1) Arthritis

गटठया 2) Kwashiorkor

कािीओकय र

3) Goitre

गाइटर

4) Night Blindness

रतह९चध

Correct Answer Kwashiorkor

Q203 A is species of plant that has

adapted to survive in an environment

with little liquid water

mdashmdashndashपह९धहॳकीएकऐसहॳऐसहॳपरजानतहहॴ नजसनहॳकमपानीवािहॳवातावरणमजीपवतरहनहॳकहॳलिएअनकिनहहॴ 1-Feb-2017

Options

1) Xerophyte

म दपवद

2) Hydrophyte

जिीयपादप

3) Mesophyte

समह८दपवद

4) Thallophyte

िहॴिह८फाइटा Correct Answer xerophyte

xerophyte is a species of plant that has

adapted to survive in an environment

with little liquid water such as a desert

or an ice- or snow-covered region in the

Alps or the Arctic

Mesophytes are terrestrial plants which

are adapted to neither a particularly

dry nor particularly wet environment

An example of a mesophytic habitat

would be a rural temperate meadow

F A C E B O O K

P A G E h t t p w w w f a c e b o o k c o m s s c m e n t o r s o f f i c i a l P a g e | 47

FOR MORE UPDATES AND MORE MATERIAL DO LIKE OUR FACEBOOK PAGE httpwwwfacebookcomsscmentorsofficial

which might contain goldenrod clover

oxeye daisy and Rosa multiflora

thallophyte any of a group of plants or

plantlike organisms (such as algae and

fungi) that lack differentiated stems

leaves and roots and that were formerly

classified as a primary division

(Thallophyta) of the plant kingdom

Q204 How many types of teeth are

there in humans

मनषयोमककतनहॳपरकारकहॳ दातहह८तहॳहहॴ

1-Feb-2017

Options

1) 4

2) 5

3) 2

4) 3

Correct Answer 4

teeth -Humans have four types of

teethincisors canines premolars and

molars each with a specific function

The incisors cut the food the canines

tear the food and the molars and

premolars crush the food

Q205 Carica papaya is the scientific name of

कहॴ ररकापपाया mdashmdashndash कावहॴजञाननकनामहहॴ 2-

Feb-2017

Options

1) Peepal

पीपि

2) Papaya

पपीता 3) Tamarind

इमिी 4) Drumstick

ढह८िकाछड़ी Correct Answer Papaya

Q206 Muscles get tired when there is

shortfall of

जब mdashndash कीकमीहह८तीहहॴतबपहॳिीयिकजातीहहॴ 2-Feb-2017

Options

1) Lactic acid

िहॴनकटकएलसि

2) Na+ ions

Na+ आयन

3) ATP

एटीपी 4) Sulphates

समफहॳ टस

Correct Answer ATP

ATP is the energy source muscle fibers

use to make muscles contract

muscle tissuersquos main source of energy

called adenosine triphosphate or ATP

As your muscles use up this energy

source they become tired and fatigued

Oxygen is the key ingredient that helps

create new ATP to replenish the burned

up ATP in your muscles

Q207 Artocarpus integra is the

scientific name of आटह८कापयसइटीगरा mdashmdashmdash कावहॴजञाननकनामहहॴ 2-Feb-2017

Options

1) Guava

अम द

2) Pineapple

अनानास

3) Silver Oak

लसमवरओक

4) Jack fruit

कटहि

Correct Answer Jack fruit

Q208 Which organ stores fat soluble

vitamins

कह९नसाअगवसामघिनिीिपवटालमनह८काभिाराकरताहहॴ

2-Feb-2017

F A C E B O O K

P A G E h t t p w w w f a c e b o o k c o m s s c m e n t o r s o f f i c i a l P a g e | 48

FOR MORE UPDATES AND MORE MATERIAL DO LIKE OUR FACEBOOK PAGE httpwwwfacebookcomsscmentorsofficial

Options

1) Blood

रकत

2) Skin

तवचा 3) Liver

यकत

4) Pancreas

अगनयािय

Correct Answer Liver

Q209 Which disease is caused due to

deficiency of Iodine

आयह८िीनकहॳ कारणकह९नसारह८गहह८ताहहॴ 2-Feb-2017

Options

1) Rickets

ररकहॳ टस

2) Scurvy

सकवी 3) Goitre

गणमािा 4) Growth retardation

पवकासका कना Correct Answer Goitre

rickets A softening and weakening of

bones in children usually due to

inadequate vitamin D

Q210 Grevillea Robusta is the scientific name of

गरहॳपवलियारह८बसटा mdashmdashmdash- कापवजञाननकनामहहॴ 2-Feb-2017

Options

1) Peepal

पीपि

2) Teak

सागह९न

3) Silver Oak

लसमवरओक

4) Jack fruit

कटहि

Correct Answer Silver Oak

Q211 When a Cuttlefish is described as a Molluscs it is at which level of

classification

जबएककटिकफिकह८एकमह८िसकाकहॳ पमवखणयतककयाजाताहहॴतबयहॳवगीकरणकहॳ ककससतरपहॳनसितहहॴ 2-Feb-2017

Options

1) Class

वगय 2) Order

िम

3) Family

पररवार

4) Phylum

सघ

Correct Answer Phylum

Q212 Bambusa dendrocalmus is the

scientific name of बानबसािहॳडराकामस mdashmdashmdash कावहॴजञाननकनामहहॴ 3-Feb-2017

Options

1) Banyan

बरगद

2) Papaya

पपीता 3) Bamboo

बास

4) Pomegranate

अनार

Correct Answer Bamboo

Q213 Acinonyx Jubatus is the scientific name of

एलसनह८ननकसजयबहॳटस mdashmdashmdash

कावहॴजञाननकनामहहॴ 3-Feb-2017

F A C E B O O K

P A G E h t t p w w w f a c e b o o k c o m s s c m e n t o r s o f f i c i a l P a g e | 49

FOR MORE UPDATES AND MORE MATERIAL DO LIKE OUR FACEBOOK PAGE httpwwwfacebookcomsscmentorsofficial

Options

1) Bear

भाि 2) Horse

घह८िा 3) Cheetah

चीता 4) Zebra

जहॳिा Correct Answer Cheetah

Q214 The pale yellow colour of urine is

due to the presence of which pigment

मतरकाफीकापीिारगरगदरयकहॳ उपनसिनतकहॳ कारणहह८ताहहॴ

3-Feb-2017

Options

1) Urochrome

यरह८िह८म

2) Urophyll

यरह८कफि

3) Chlorophyll

किह८रह८कफि

4) Chloroplast

किह८रह८पिासट

Correct Answer Urochrome

Q215 Which of the following constitute

to form a gene

नननननलिखितमसहॳकह९नसीचीज़एकजीनकागठनकरतीहहॴ

3-Feb-2017

Options

1) Polynucleotides

पह८िीनयनकियह८टाईडस

2) Hydrocarbons

हाइडरह८काबोस

3) Lipoproteins

िाईपह८परह८टीनस

4) Lipids

लिपपडस

Correct Answer Polynucleotides

Polynucleotide molecule is a biopolymer

composed of 13 or more nucleotide

monomers covalently bonded in a chain

DNA (deoxyribonucleic acid) and RNA

(ribonucleic acid) are examples of

polynucleotides with distinct biological

function

Q216 Vertebrates belongs to the

phylum

रीढ़कीहडिीवािहॳपराणी mdashmdashmdash

परजानतकहॳ अतगायतआतहॳहहॴ 3-Feb-2017

Options

1) Arthropoda

आरह८पह८ड़ा 2) Annelida

एननलििा 3) Cnidaria

ननिहॳररया 4) Chordata

कह८िटा Correct Answer Chordata

Q217 Punica granatum is the scientific name of

पननकगरहॳनहॳटस mdashmdashmdash कावहॴजञाननकनामहहॴ 3-Feb-2017

Options

1) Custard Apple

सीताफि

2) Gulmohar

गिमह८हर

3) Silver Oak

लसमवरओक

4) Pomegranate

अनार

Correct Answer Pomegranate

F A C E B O O K

P A G E h t t p w w w f a c e b o o k c o m s s c m e n t o r s o f f i c i a l P a g e | 50

FOR MORE UPDATES AND MORE MATERIAL DO LIKE OUR FACEBOOK PAGE httpwwwfacebookcomsscmentorsofficial

Q218 Between a tiger and an monkey

which of the following is different

एकबाघऔरबदरकहॳ बीचनननननलिखितमसहॳकह९नसीबातअिगहहॴ 3-Feb-2017

Options

1) Kingdom

राजय

2) Phylum

जानत

3) Order

िम

4) Class

वगय Correct Answer order

Q219 The artificial heart was invented by

कबतरमहदयका mdashmdashmdash

दवाराअपवषकारककयागयािा 3-Feb-2017

Options

1) Muhammad Yunus

महनमदयनस

2) Linus Yale Jr

िाइनसयहॳिजय

3) Gazi Yasargil

गाजीयासचगयि

4) Paul Winchell

पह९िपवमकि Correct Answer Paul Winchell

Q220 Tamarindus indica is the

scientific name of

टहॳमररनडसइडिका mdashmdash कावहॴजञाननकनामहहॴ 7-

Feb-2017

Options

1) Neem

नीम

2) Pineapple

अनानास

3) Tamarind

इमिी 4)Chiku

चीक

Correct Answer Tamarind

Q221 In eukaryotic cells synthesis of

RNA takes place in the

यकहॳ योटटककह८लिकाओमआरएनएकासशिहॳषण

mdashndash महह८ताहहॴ 7-Feb-2017

Options

1) Mitochondria

माईटह८कोडडरया 2) Centrioles

सटरीयह८मस

3) Ribosomes

ररबह८सह८नस

4) Nucleus

नयनकियस

Correct Answer nucleus

eukaryotic cell -Transcription is the

process of synthesizing ribonucleic acid

(RNA)Synthesis takes place within the

nucleus of eukaryotic cells or in the

cytoplasm of prokaryotes and converts

the genetic code from a gene in

deoxyribonucleic acid ( DNA ) to a

strand of RNA that then directs

proteinsynthesis

Q222 _________is caused by parasites

of the Plasmodium genus

पिाजमह८डियमजातीकहॳ परजीवी mdash- कहॳ कारणहहॴ 7-Feb-2017

Options

1) Dysentery

पहॳचचि

2) Malaria

मिहॳररया 3) Chickenpox

F A C E B O O K

P A G E h t t p w w w f a c e b o o k c o m s s c m e n t o r s o f f i c i a l P a g e | 51

FOR MORE UPDATES AND MORE MATERIAL DO LIKE OUR FACEBOOK PAGE httpwwwfacebookcomsscmentorsofficial

चहॳचक

4) Herpes

हहॳपपयस

Correct Answer Malaria

Q223 Carotene in fruits and vegetables

gives it which color

फिह८औरसनलजयोमनसितकहॳ रह८टीनउनहकह९नसारगपरदानकरताहहॴ 7-Feb-2017

Options

1) Green

हरा 2) Pink

गिाबी 3) Orange

नारगी 4) Blue

नीिा Correct Answer Orange

Q224 Equus Caballus is the scientific

name of

एकवसकहॴ बहॳिस mdashmdashndash कापवजञाननकनामहहॴ 7-Feb-2017

Options

1) Horse

घह८िा 2) Zebra

ज़हॳिा 3) Donkey

गधा 4) Buffalo

भस

Correct Answer Horse

Q225 Elapidae Naja is the scientific name of

एिीपीिीनाजा mdashmdash- कावहॴजञाननकनामहहॴ 8-Feb-2017

Options

1) Cobra

कह८बरा 2) Elephant

हािी 3) Eagle

ग ि

4) Owl

उमि Correct Answer Cobra

Q226 Which disease is caused due to

deficiency of Iron

िह८हकीकमीकहॳ कारणकह९नसारह८गहह८ताहहॴ 8-Feb-

2017

Options

1) Beriberi

बहॳरीबहॳरी 2) Tetany

टहॳटनी 3) Kwashiorkor

कवािीऔरकर

4) Anaemia

रकतामपता Correct Answer Anaemia

Beriberi is a disease caused by a vitamin

B-1 deficiency also known as thiamine

deficiency

Tetany can be the result of an

electrolyte imbalance Most often itrsquos a

dramatically low calcium level also

known as hypocalcemia Tetany can also

be caused by magnesium deficiency or

too little potassium Having too much

acid (acidosis) or too much alkali

(alkalosis) in the body can also result in

tetany

Kwashiorkor also known as

ldquoedematous malnutrition It is a form of

malnutrition caused by a lack of protein

in the diet

Anaemia means that you have fewer red

blood cells than normal or you have less

F A C E B O O K

P A G E h t t p w w w f a c e b o o k c o m s s c m e n t o r s o f f i c i a l P a g e | 52

FOR MORE UPDATES AND MORE MATERIAL DO LIKE OUR FACEBOOK PAGE httpwwwfacebookcomsscmentorsofficial

haemoglobin than normal in each red

blood cell

Q227 is a leaf where the leaflets are

arranged along the middle vein

mdashndashएकपततीहहॴजहापतरकह८कीरचनाक ररयालिराकहॳ आसपासहह८तीहहॴ 8-Feb-2017

Options

1) Pinnately compound leaf

पपनहॳटिीसयकतपतती 2) Palmately compound leaf

पामहॳटिीसयकतपतती 3) Compound leaf

सयकतपतती 4) Simple leaf

साधारणपतती Correct Answer Pinnately compound

leaf

Q228 Haustoria or sucking roots are

found in which of the following

हह८सटह८ररयायाचसनहॳवािीजड़हॳनननननलिखितमसहॳककसमपाईजातीहहॴ 8-Feb-2017

Options

1) Wheat

गहॳह

2) Mango

आम

3) Chestnut

चहॳसटनट

4) Cuscuta

कसकयटा Correct Answer Cuscuta

Haustorial roots -The roots of parasitic

plants which penetrate into the host

tissues to absorb nourishment are

called haustorial roots hellip Also known as suckingor parasitic roots

Q229 Equs Asinus is the scientific name

of

एकवसएलसनस mdashmdashndash कावहॴजञाननकनामहहॴ 8-

Feb-2017

Options

1) Donkey

गधा 2) Cow

गाय

3) Deer

टहरन

4) Kangaroo

कगा

Correct Answer Donkey

Q230 Ficus benghalensis is the scientific name of

फाईकसबहॳनगहॳिहॳलसस mdashndash कापवजञाननकनामहहॴ 8-Feb-2017

Options

1) Banyan

बरगद

2) Pineapple

अनानास

3) Babul

बबि

4) Tulsi

तिसी Correct Answer Banyan

Q231 Equus burchellii is the scientific name of

एकवसबचिी mdashmdash- कापवजञाननकनामहहॴ 8-Feb-2017

Options

1) Horse

घह८िा 2) Zebra

जहॳिा 3) Buffalo

F A C E B O O K

P A G E h t t p w w w f a c e b o o k c o m s s c m e n t o r s o f f i c i a l P a g e | 53

FOR MORE UPDATES AND MORE MATERIAL DO LIKE OUR FACEBOOK PAGE httpwwwfacebookcomsscmentorsofficial

भस

4) Ass

गधा Correct Answer Zebra

Page 20: COMPILATION OF ALL 72 SETS OF BIOLOGY SSC CHSL-2016 · OF BIOLOGY SSC CHSL-2016 PREPARED BY : SSC MENTORS BIOLOGY SPECIAL . F A C E B O O K P A G E : h t t p : / / w w w . f a c e

F A C E B O O K

P A G E h t t p w w w f a c e b o o k c o m s s c m e n t o r s o f f i c i a l P a g e | 19

FOR MORE UPDATES AND MORE MATERIAL DO LIKE OUR FACEBOOK PAGE httpwwwfacebookcomsscmentorsofficial

महॴरीएिरसन

2) Virginia Apgar

वनजयननयाएपगार

3) William Harvey

पवलियमहाव

4) Robert Feulgen

रॉबटयफ़यिजहॳन Correct Answer William Harvey

Q76 Vitamin A is also known as

पवटालमन A कह८ mdashmdash- कहॳ नामसहॳभीजानाजाताहहॴ SSC CHSL Science (biology) 2016

Question Paper

18Jan2017

Options

1) Thiamine

िायलमन

2) Riboflavin

ररबह८फिहॳपवन

3) Retinol

रहॳटटनॉि

4) Calciferol

कहॴ नमसफहॳ रह८ि

Correct Answer Retinol

Q77 Some roots called arise from an

organ other than the radicle

कछजड़हॳनजनह mdashmdashmdash कहाजाताहहॴ वहमिकहॳ अिावाककसीअनयअगसहॳउतपननहह८तीहहॴ 18Jan2017

Options

1) tap roots

मखयजड़

2) stilt roots

ि ाजड़

3) fibrous roots

रहॳिहॳदारजड़

4) adventitious roots

आकनसमकजड़

Correct Answer adventitious roots

Q78 Spiders belong to which class of

animals

मकडड़यापराणीवगीकरणकहॳ ककसवगयमआतीहहॴ 18Jan2017

Options

1) Arachnids

एरहॳकननडस

2) Aves

एपवस

3) Gastropods

गहॴसटरोपह८िस

4) Anthozoa

एिह८जआ

Correct Answer Arachnids

Q79 How many layers does Human

Skin have

मानवतवचामककतनीपरतहॳहह८तीहहॴ

18Jan2017

Options

1) 5

2) 7

3) 11

4) 3

Correct Answer 3

Skin has three layers The epidermis

the outermost layer of skin provides a

waterproof barrier and creates our skin

tone The dermis beneath the

epidermis contains tough connective

tissue hair follicles and sweat glands

The deeper subcutaneous tissue (

hypodermis ) is made of fat and

connective tissue

Q80 Allium Cepa is the scientific name

of

एलियमलसपपा mdashmdashndash कावहॴजञाननकनामहहॴ 18Jan2017

F A C E B O O K

P A G E h t t p w w w f a c e b o o k c o m s s c m e n t o r s o f f i c i a l P a g e | 20

FOR MORE UPDATES AND MORE MATERIAL DO LIKE OUR FACEBOOK PAGE httpwwwfacebookcomsscmentorsofficial

Options

1) Carrot

गाजर

2) Tomato

टमाटर

3) Potato

आि 4) Onion

पयाज़

Correct Answer Onion

Q81 DNA stands for

िीएनएकापणय प mdashmdash- हहॴ 18Jan2017

Options

1) Di Nucleic Acid

िाईनयनकिकएलसि

2) Deoxy Nucleic Acid

िीओकसीनयनकिकएलसि

3) Diribonucleic Acid

िाईराइबह८नयनकिकएलसि

4) Deoxyribonucleic Acid

िीऑकसीराइबह८नयनकिकएलसि

Correct Answer Deoxyribonucleic Acid

Q82 Organisms that generate energy

using light are known as

जह८जीवाणपरकािकाउपयह८गकरउजायउतपननकरतीहहॴ उनह mdashmdash कहॳ पमजानाजाताहहॴ

18Jan2017

Options

1) Chaemolithotrophs

ककमह८लििह८टरह८पस

2) Oligotrophs

ओलिगह८टरह८पस

3) Bacteria

बहॴकटीररया 4)Photoautotrophs

फह८टह८ओटह८टरह८पस

Correct Answer Photoautotrophs

An oligotroph is an organism that can

live in an environment that offers very

low levels of nutrients

Q83 Which drug is used as an

Antidepressant

ककसदवाएकहतािारह८धीकहॳ पमपयोगककयाजाताहहॴ Options

1) Oxybutynin

ओकसीलयटीनन

2)Tramadol

टरहॳमहॳिह८ि

3 ) Sumatriptan

समहॳटरीपटहॳन

4) Bupropion

लयपरह८पपयह८न

Correct Answer Bupropion

लयपरह८पपयह८न

Q84 The orange colour of carrot is

because of

गाजरकानारगीरगनननननलिखितमसहॳककसीएककीवजहसहॳहह८ताहहॴ 18Jan2017

Options

1) it grows in the soil

यहलम ीमउगतीहहॴ 2) Carotene

कहॴ रह८टीन

3) it is not exposed to sunlight

यहसययपरकािकहॳ सपकय मनहीआती 4) the entire plant is oranqe in colour

सनपणयपह९धानारगीरगकाहह८ताहहॴ Correct Answer Carotene

Q85 Snake venom is highly modified

saliva containing

F A C E B O O K

P A G E h t t p w w w f a c e b o o k c o m s s c m e n t o r s o f f i c i a l P a g e | 21

FOR MORE UPDATES AND MORE MATERIAL DO LIKE OUR FACEBOOK PAGE httpwwwfacebookcomsscmentorsofficial

सापकाजहरअततयाचधकसिह८चधतिारहह८तीहहॴनजसमहॳ mdashmdash- हह८ताहहॴ Options

l)Prototoxins

परह८टह८टॉनकसस

2)Neutrotoxins

नयटरोटॉनकसस

3)Zootoxins

जटॉनकसस

4)Electrotoxins

इिहॳकटरह८टॉनकसस

Correct Answer Zootoxins

जटॉनकसस

Q86 Which type of pathogen causes the

water-borne disease Schistosomiasis

ककसपरकारकारह८गज़नकजिजननतरह८गलससटह८सह८लमलससकाकारणबनताहहॴ

18Jan2017

Option

1) Parasitic

परजीवी 2)Protozoan

परह८टह८जआ

3) Bacterial

बहॴकटीररयि

4) Viral

वायरि

Correct Answer Parasitic

Schistosomiasis also known as snail

fever and bilharzia is a disease caused

by parasitic

flatworms called schistosomes

Q87 Prothrombin responsible for

clotting of blood is released by

परह८िह८ननबन

जह८रकतकािककाजमनहॳकहॳ लिएनजनमहॳदारहहॴ mdashndash

कहॳ दवारासतरापवतककयाजाताहहॴ

19Jan2017

Options

1) Small Intestine

छह८टीआत

2) Blood Platelets

रकतपिहॳटिहॳटस

3) Large Intestine

बड़ीआत

4Heart

हदय

Correct Answer Blood Platelets

Q88 Acacia arabica is the scientific

name of

अकहॳ लियाअरहॳबबका mdashmdashndash कावहॴजञाननकनामहहॴ 19-Jan-2017

Options

1) Neem

नीम

2) Teak

सागह९न

3) Babhul

बबि

4) Pomegranate

अनार

Correct Answer Babhul

Q89 Cannis Vulpes is the scientific

name of

कहॴ ननसवनमपस mdashmdash- कावहॴजञाननकनामहहॴ 19-Jan-2017

Options

1) Dog

कतता 2) Wolf

भहॳडड़या 3) Fox

िह८मड़ी 4) Hyena

िाकिबगघा

F A C E B O O K

P A G E h t t p w w w f a c e b o o k c o m s s c m e n t o r s o f f i c i a l P a g e | 22

FOR MORE UPDATES AND MORE MATERIAL DO LIKE OUR FACEBOOK PAGE httpwwwfacebookcomsscmentorsofficial

Correct Answer Fox

Q90 The beetroot is the portion of the

beet plant

चकदरपह९धहॳका mdashmdashndash भागहहॴ 19-Jan-2017

Options

1) tap root

मखयजड़

2) Adventitious

आकनसमक

3) bulb of the stem

तनहॳकाकद

4) Rhizome

परकद

Correct Answer tap root

Q91 What is the basic unit of heredity

आनवलिकताकीबननयादीइकाईकयाहहॴ 19-Jan-2017

Options

1) DNA

िीएनए

2) RNA

आरएनए

3) Chromosome

िह८मह८सह८म

4) Gene

जीन

Correct Answer gene

Genes are the units of heredity and are

the instructions that make up the bodyrsquos

blueprint They code for the proteins

that determine virtually all of a personrsquos

characteristics Most genes come in

pairs and are made of strands of genetic

material called deoxyribonucleic acid

or DNA

Q92 Lungs are the primary organs of

फहॳ फड़हॳmdashndashकहॳ परािलमकअगहहॴ

19-Jan-2017

Options

1) Digestion

पाचन

2) Constipation

कलज

3) Perspiration

पसीना 4)Respiration

शवसन

Correct Answer Respiration

Q93 Sugarcane is a type of

गननाएकपरकारका mdash- हहॴ 20-Jan-2017

Options

1)creeper

िता 2)tree

पहॳड़

3)shrub

झाड़ी 4)grass

घास

Correct Answer grass

Q94 Who is commonly known as ldquothe

Father of Microbiologyrdquo

सामानयत ldquo सकषमजीवपवजञानकहॳ जनक lsquo

कहॳ नामसहॳककसहॳजानाजातहहॴ 20-Jan-2017

Options

1) Robert Hooke

रॉबटयहक

2) Antonie Philips van Leeuwenhoek

एटह८नीकफलिपवानमयएनहह८क

3) Carl Linnaeus

काियिीनाईयस

4) Charles Darwin

चामसयिापवयन

F A C E B O O K

P A G E h t t p w w w f a c e b o o k c o m s s c m e n t o r s o f f i c i a l P a g e | 23

FOR MORE UPDATES AND MORE MATERIAL DO LIKE OUR FACEBOOK PAGE httpwwwfacebookcomsscmentorsofficial

Correct Answer Antonie Philips van

Leeuwenhoek

Q95 For the aquatic organisms the

source of food is

जिीयजीवाणकािाघसरह८तहहॴ 20-Jan-2017

Options

1) Phytoplankton

फायटह८पिहॳकटन

2) Sea Weed

समदरीिहॴवाि

3)Aqua plankton

एकवापिहॳकटन

4) Zooplankton

जपिहॳकटन

Correct Answer Phytoplankton

Q96 Haemoglobin has the highest

affinity with which of the following

हीमह८गिह८बबनकीननननमसहॳककसकहॳ सािउततमसमानताहहॴ

20-Jan-2017

Options

1)SO2

2)CO2

3)CO

4)NO2

Correct Answer CO

It has a greater affinity for hemoglobin

than oxygen does It displaces oxygen

and quickly binds so very little oxygen

is transported through the body cells

Q97 Who developed the theory of

Evolution

उदपवकासकालसदातककसनहॳपवकलसतककया

20-Jan-2017

Options

1) Charles Darwin

चामसयिापवयन

2) Isaac Newton

आयजहॳकनयटन

3) Pranav Mistry

परणवलमसतरी 4) Galileo Galilei

गहॳलिलियह८गहॳिीिी Correct Answer Charles Darwin

Q98 The primary function of RNA is

RNA कापरािलमककाययहह८ताहहॴ 20-Jan-2017

Options

1) Photosynthesis

परकािसशिहॳषण

2) Protein Synthesis

परह८टीनसशिहॳषण

3) Replication

परनतकनतबनाना 4) Translation

अनवादकरना Correct Answer Protein Synthesis

There are two main functions of RNA

It assists DNA by serving as a messenger

to relay the proper genetic information

to countless numbers of ribosomes in

your body The other main function of

RNA is to select the correct amino acid

needed by each ribosome to build new

proteins for your body

Q99 ______is the movement of

molecules across a cell membrane from

a region of their lower concentration to

a region of their higher concertration

उचचसादरताकहॳ कषहॳतरसहॳउसकीकमसादरतावािहॳकषहॳतरकीतरफएककह८लिकाखझमिीकहॳ माधयमसहॳहह८नहॳवािाअणओकहॳ सचिनकह८ mdash- कहतहॳहहॴ Options

1) Diffusion

पवसरण

2) Osmosis

ऑसमह८लसस

F A C E B O O K

P A G E h t t p w w w f a c e b o o k c o m s s c m e n t o r s o f f i c i a l P a g e | 24

FOR MORE UPDATES AND MORE MATERIAL DO LIKE OUR FACEBOOK PAGE httpwwwfacebookcomsscmentorsofficial

3) Active Transport

सकियआवागमन

4) Passive Transport

नननषियआवागमन

Correct Answer Active Transport

Q100 Study of classification of

organisms is known as 20-Jan-2017

जीवाणओकहॳ वगीकरणकहॳ अधययनकह८ mdash-

कहाजाताहहॴ Options

1) Serpentology

सपरहॳटह८िह८जी 2) Virology

वायरह८िह८जी 3) Taxonomy

टहॴकसोनह८मी 4) Physiology

कफनज़यह८िह८जी Correct Answer Taxonomy

Q101 Photosynthesis takes place inside

plant cells in

परकािसशिहॳषणवनसपनतकह८लिकामनसति mdash

mdashmdash महह८ताहहॴ 20-Jan-2017

Options

1) Ribosomes

राइबह८सह८नस

2) Chloroplasts

किह८रह८पिासट

3) Nucleus

नयकलियम

4) Mitochondria

माईटह८कोडडरया Correct Answer Chloroplasts

Q102 ______ is the cell organelle in

which the biochemical processes of

respiration and energy production

occur

mdashmdash- वहकह८लिकाअगहहॴ नजसमहॳशवसनऔरउजायउतपादनकहॳ जहॴसीजहॴवरासायननकपरकियायहह८तीहहॴ 20-Jan-2017

Options

1) Mitochondria

माइटह८कोडडरया 2) Chloroplast

किह८रह८पिासट

3) Ribosomes

राइबह८सह८नस

4) Nucleus

नयकिीयस

Correct Answer Mitochondria

Q103 Which non-flowering spore

bearing plants have roots

ककसफिनिगनहॳवािहॳऔरबीजाणधारकपह९धह८कीजड़हॳहह८तीहहॴ 21-Jan-2017

Options

1) Mosses

मह८सहॳस

2) Angiosperms

एननजयह८सपनसय 3) Ferns

फनसय 4) Gymnosperms

नजननह८सपनसय Correct Answer ferns

Q104 Which of the following is an

excretory organ of cockroach

नननननलिखितमसहॳकह९नसानतिच हॳकाउतसजयनअगहहॴ

21-Jan-2017

Options

F A C E B O O K

P A G E h t t p w w w f a c e b o o k c o m s s c m e n t o r s o f f i c i a l P a g e | 25

FOR MORE UPDATES AND MORE MATERIAL DO LIKE OUR FACEBOOK PAGE httpwwwfacebookcomsscmentorsofficial

1) Malphigian Tubules

मनमफनजयनटयबमस

2) Nephridia

नहॳकफरडिया 3) Coxal Gland

कह८कसिगरचिया 4) Green Gland

गरीनगरचिया Correct Answer Malphigian Tubules

Q105 Evaporation of water takes place

in which part of plants

पानीकहॳ वाषपीकरणकीकियापह९धोकहॳ ककसभागसहॳहह८तीहहॴ 21-Jan-2017

Options

1) Stem

तना 2) Stomata

सटह८मटा 3) Branch

िािाए

4) Fruit

फि

Correct Answer Stomata

Evaporation accounts for the movement

of water to the air from sources such as

the soil canopy interception and

waterbodies Transpiration accounts for

the movement of water within a plant

and the subsequent loss of water as

vapour through stomata in its leaves

Q106 A is the fleshy spore-bearing

fruiting body of a fungus

mdashmdashndashकवककामासि

बीजाणधारणकरनहॳवािाफिनहॳवािाअगहहॴ 21-

Jan-2017

Options

1) aloe vera

एिह८वहॳरा 2) Coral

मगा 3) Cactus

कहॴ कटस

4) Mushroom

ककरमतता Correct Answer mushroom

Q107 Which of the following is a fungal

disease

नननननलिखितमसहॳकह९नसाफफदसहॳहह८नहॳवािाएकरह८ग हहॴ

21-Jan-2017

Options

1) Dermatitis

तवचािह८ध

2) Cholera

हहॴजा 3) Jaundice

पीलिया 4) Indigofera

इननिगह८फहॳ रा Correct Answer Dermatitis

Dermatitis also known as eczema is a

group of diseases that results in

inflammation of the skin These diseases

are characterized by itchiness red skin

and a rash In cases of short duration

there may be small blisters while in

long-term cases the skin may become

thickened

Q108 In which form is glucose stored in

our body

हमारहॳिरीरमगिकह८जकासचयककस पमककयाजाताहहॴ

21-Jan-2017

Options

1) Insulin

F A C E B O O K

P A G E h t t p w w w f a c e b o o k c o m s s c m e n t o r s o f f i c i a l P a g e | 26

FOR MORE UPDATES AND MORE MATERIAL DO LIKE OUR FACEBOOK PAGE httpwwwfacebookcomsscmentorsofficial

इसलिन

2) Glucose

गिकह८ज

3) Glycogen

गिायकह८जहॳन

4) Fat

वसा Correct Answer Glycogen

Excess glucose is stored in the liver as

the large compound called glycogen

Glycogen is a polysaccharide of glucose

but its structure allows it to pack

compactly so more of it can be stored in

cells for later use

Q109 Where do plants synthesize

protein from

पह९धहॳपरह८टीनसशिहॳषणकहासहॳकरतहॳहहॴ

Options

1) Fatty Acids

वसाऐलसि

2) Sugar

िकर

3) Amino Acids

एलमनह८ऐलसि

4) Starch

सटाचय Correct Answer Amino Acids

Q110 Which part of the brain is

responsible for triggering actions like

thinking intelligence memory and

ability to learn

मनसतषककाकह९नसाटहससासह८चनहॳ बनधदमानी याददाशतऔरसीिनहॳकीकषमताजहॴसीकियाओकह८परहॳररतकरताहहॴ 21-Jan-2017

Options

1) Diencephalon

िायएनसहॳफहॳ िह८न

2) Hypothalamus

हयपह८िहॳिहॳमस

3) Cerebrum

सहॳरहॳिम

4) Control

कटरह८ि

Correct Answer Cerebrum

Q111 Which of the following is also

known as the Biochemical Laboratory

of the Human Body

नननननलिखितमसहॳककसहॳमानविरीरकीजहॴवरसायनपरयह८गिािाभीकहाजाताहहॴ 21-Jan-2017

Options

1) Small Intestine

छह८टीआत

2)Brain

मनसतषक

3) Pancreas

अगनयािय

4) Liver

नजगर

Correct Answer Liver

The liver makes bile that will help

emulsify and digest the fats we eat

The liver takes toxic substances and

convert them using enzymes the liver

cells makes into a non toxic form so the

body can dispose of them

The liver also converts fats protein and

carbohydrates into glucose which is the

energy source for our cells to use

The liver takes amino acids and makes

proteins by combining them

Q112 The yellow colour of human urine

is due to

मानवमतरकापीिारग mdashndash कीवजहसहॳहह८ताहहॴ 22-

Jan-2017

Options

1) Bile Salts

F A C E B O O K

P A G E h t t p w w w f a c e b o o k c o m s s c m e n t o r s o f f i c i a l P a g e | 27

FOR MORE UPDATES AND MORE MATERIAL DO LIKE OUR FACEBOOK PAGE httpwwwfacebookcomsscmentorsofficial

पपततनमक

2) Cholesterol

कह८िहॳसटरह८ि

3) Lymph

लिनफ

4) Urochrome

यरह८िह८म

Correct Answer Urochrome

Urobilin or urochrome is the chemical

primarily responsible for the yellow

color of urine

Q113 The wilting of plants takes place

due to

पह९धह८कालिचििहह८नाकी mdashmdash- कीवजहसहॳहह८ताहहॴ 22-Jan-2017

Options

1)Photosynthesis

परकािसशिहॳषण

2) Transpiration

वाषपह८तसजयन

3) Absorption

अविह८षण

4) Respiration

शरवसन

Correct Answer Transpiration

Wilting is the loss of rigidity of non-

woody parts of plants This occurs when

the turgor pressure in non-lignified

plant cells falls towards zero as a result

of diminished water in the cells

Q114 Bovidae Ovis is the scientific name of

बह८पविीओपवस mdashndash कावहॴजञाननकनामहहॴ 22-Jan-2017

Options

1) Goat

बकरी 2) Cow

गाय

3) Buffalo

भहॳस

4) Sheep

भहॳड़

Correct Answer Sheep

Q115 Plants get their energy to produce

food from which of the following

पह८धहॳभह८जनकाननमायणकरनहॳकहॳ लिएनननननलिखितमसहॳककससहॳउजायपरापतकरतहॳहहॴ

22-Jan-2017

Options

1) Photosynthesis

परकािसशिहॳषण

2)Bacteria

बहॴकटीररया 3)Fungi

कवक

4)Sun

सयय Correct Answer Sun

Q116 Which of the following is secreted

by the liver

नननननलिखितमसहॳककसकासरावनजगरसहॳहह८ताहहॴ

22-Jan-2017

Options

1) Glucose

गिकह८ज

2) Iodine

आयह८िीन

3) Cortisol

काटटरयसह८ि

4) Bile

पपतत

Correct Answer Bile

The liver makes bile that will help

emulsify and

digest the fats we eat

F A C E B O O K

P A G E h t t p w w w f a c e b o o k c o m s s c m e n t o r s o f f i c i a l P a g e | 28

FOR MORE UPDATES AND MORE MATERIAL DO LIKE OUR FACEBOOK PAGE httpwwwfacebookcomsscmentorsofficial

Q117 Ferns belong to which division of

plants

फनसयपह९धह८कहॳ ककसभागमआतहॳहहॴ

22-Jan-2017

Options

1) Gymnosperms

नजननह८सपनसय 2) Angiosperms

एनजयह८सपनसय 3) Thallophyta

िहॴिह८फाईटा 4)Pteridophyta

टहॳररिह८फाईटा Correct Answer Pteridophyta

Q118 Who invented Antibiotics

एटीबायह८टटककाअपवषकारककसनहॳककयािा

22-Jan-2017

Options

1) Joseph Lister

जह८सहॳफलिसटर

2) William Harvey

पवलियमहाव

3) Robert Knock

रॉबटयनॉक

4)Alexander Fleming

अिहॳकज़िरफिहॳलमग

Correct Answer Alexander Fleming

Q119 Milbecycin is used in the

eradication of

लममबहॳसायलसनका mdashndash

मउनमिनमपरयह८गककयाजाताहहॴ 22-Jan-2017

Options

1) Agricultural Fungus

कपषकवक

2) Agricultural Pests

कपषकीटक

3) Agricultural Herbs

कपषिाक

4)Agricultural Weeds

कपषननराना Correct Answer Agricultural Pests

Milbemycin oxime is a veterinary drug

from the group of milbemycins used as

a broad spectrum antiparasitic It is

active against worms and mites(insects

Q120 Intestinal bacteria synthesizes

which of the following in the human

body

मानविरीरमआतोकहॳ बहॴकटीररयानननननलिखितमसहॳककसकासशिहॳषणकरतहॳहहॴ 22-Jan-2017

Options

1) Vitamin K

पवटालमन K

2) Proteins

परह८टीन

3) Fats

वसा 4) Vitamin D

पवटालमन D

Correct Answer Vitamin K

Q121 is the study of the physical form

and external structure of plants

mdashmdash-

मपह९धह८काभहॴनतक पऔरबाहरीसरचनाकाआदयाककयाजाताहहॴ 22-Jan-2017

Options

1) Physiology

कफनजयह८िह८जी 2) Anatomy

िरीररचनापवजञान

3) Phytomorphology

फाईटह८मह८फह८िह८जी 4)Cytology

कह८लिकापवजञान

Correct Answer Phytomorphology

F A C E B O O K

P A G E h t t p w w w f a c e b o o k c o m s s c m e n t o r s o f f i c i a l P a g e | 29

FOR MORE UPDATES AND MORE MATERIAL DO LIKE OUR FACEBOOK PAGE httpwwwfacebookcomsscmentorsofficial

Q122 Which of the following is a

structural and functional unit of

kidneys

नननननलिखितमसहॳकह९नसीगदोकीसरचनातमकऔरकाययकरीईकाईहहॴ

22-Jan-2017

Options

1) Renette Cells

रहॳनहॳटकह८लिकाए

2) Flame Cells

फिहॳमकह८लिकाए

3) Nephrites

नहॳफ़राइटस

4)Nephrons

नहॳफरोस

Correct Answer Nephrons

Nephron functional unit of the kidney

the structure that actually produces

urine in the process of removing waste

and excess substances from the blood

There are about 1000000 nephrons in

each human kidney

Q123 Which of the following is the

largest part of the human brain

नननननलिखितमसहॳकह९नसामानवमनसतषककासबसहॳबड़ाटहससाहहॴ

23-Jan-2017

Options

1) Ribs

पसलियाा 2) Cerebrum

सहॳरहॳिम

3) Pons

पोस

4)Thalamus

िहॴिहॳमस

Correct Answer Cerebrum

The cerebrum is the largest part of the

human brain making up about two-

thirds of the brainrsquos mass It has two

hemispheres each of which has four

lobes frontal parietal temporal and

occipital

Q124 The auxiliary buds

सहायककालियाmdashndash 23-Jan-2017

Options

1) grow endogenously from the pericycle

पहॳरीसाईककिसहॳअनतजातयपवकलसतहह८ताहहॴ 2) arise endogenously from the main

growing point

मिवपदसहॳअनतजातयउठताहहॴ 3) is an embryonic shoot located in the

axil of a leaf

एकभरणिटहहॴजह८एकपततीकहॳ अकषपरनसतिहह८ताहहॴ 4)arise exogenously from the epidermis

एपपिलमयससहॳबटहजातयतरीकहॳ सहॳउठताहहॴ Correct Answer is an embryonic shoot

located in the axil of a leaf

Q125 Which of the following is a viral

disease

इनमहॳसहॳकह९सीएकवायरिबीमारीहहॴ

23-Jan-2017

Options

1) Polio

पह८लियह८ 2) Tetanus

धनसतनभ

3) Leprosy

कषठरह८ग

4) Plague

पिहॳग

Correct Answer Polio

A viral disease (or viral infection)

occurs when an organismrsquos body is

invaded by pathogenic viruses and

infectious virus particles (virions) attach

to and enter susceptible cells

F A C E B O O K

P A G E h t t p w w w f a c e b o o k c o m s s c m e n t o r s o f f i c i a l P a g e | 30

FOR MORE UPDATES AND MORE MATERIAL DO LIKE OUR FACEBOOK PAGE httpwwwfacebookcomsscmentorsofficial

Poliomyelitis often called polio or

infantile paralysis is an infectious

disease caused by the poliovirus

Tetanusmdash A serious bacterial infection

that causes painful muscle spasms and

can lead to death

Leprosy also known as Hansenrsquos

disease (HD) is a long-term infection by

the bacterium Mycobacterium leprae or

Mycobacterium lepromatosis

Plague is an infectious disease caused by

the bacterium Yersinia pestis

Symptoms include fever weakness and

headache

Q126 Which organisms can help to

carry out Vermicomposting

कह९नसाजीववमीकनपह८नसटगममददकरताहहॴ

23-Jan-2017

Options

1) Nitrifying Bacteria

नाईटरीफाईगबहॴकटीररया 2) Earthworms

कहॴ चऐ

3) Algae

िहॴवि

4) Fungus

कवक

Correct Answer Earthworms

Q127 Contraction of heart is also

known as

हदयकहॳ सकचनकह८ mdash- भीकहाजाताहहॴ 23-Jan-

2017

Options

1) Systole

लससटह८ि

2) Aristotle

अरसत

3) Diastole

िायसटह८ि

4) Lub

मयब

Correct Answer Systole

Diastole is the part of the cardiac cycle

when the heart refills with blood

following systole (contraction)

Ventricular diastole is the period during

which the ventricles are filling and

relaxing while atrial diastole is the

period during which the atria are

relaxing

Q128 Azadirachta indica is the

botanical name of which of the

following

अजाटदराचताइडिकानननननलिखितमसहॳककसकावानसपनतनामहहॴ

23-Jan-2017

Options

1) Rose plant

गिाबकापह९धा 2) Apple tree

सहॳबकापहॳड़

3) Neem

नीम

4)Mango

आम

Correct Answer Neem

Q129 Which of the following is the

main end product of carbohydrate

digestion

नननननलिखितमसहॳकह९नसाकाबोहाइडरहॳटकहॳ पाचनकापरमिअतउतपादकहह८ताहहॴ 23-Jan-2017

Options

1) Fats

वसा 2) Lipids

लिपपडस

3) Glucose

गिकह८ज

4) Cellulose

F A C E B O O K

P A G E h t t p w w w f a c e b o o k c o m s s c m e n t o r s o f f i c i a l P a g e | 31

FOR MORE UPDATES AND MORE MATERIAL DO LIKE OUR FACEBOOK PAGE httpwwwfacebookcomsscmentorsofficial

सहॳमयिह८ज

Correct Answer Glucose

Intestinal absorption of end products

from digestion of carbohydrates and

proteins in the pig hellip During absorption some sugars (fructose or

galactose) released from the

corresponding sucrose and lactose

respectively during digestion were

partly metabolized into glucose by the

enterocyte

Q130 Which of the following glands is a

source of the enzyme Ptyalin

नननननलिखितगरचियोमसहॳएजाइमटयालिनकासरह८तहहॴ 23-Jan-2017

Options

1) Pancreas

अगरािय

2) Thyroid Gland

िाइराइिगरिी 3) Pituitary Gland

पीयषगरिी 4) Salivary Glands

िारगरचियाा Correct Answer Salivary Glands

Q131 Which of the following is not true

about Pteridophyta

ननननमसहॳकह९नसीबातटहॳररिह८फाईटकहॳ बारहॳमसचनहीहहॴ 23-Jan-2017

Options

1) Dominant phase is saprophytes

परमिचरणसहॳपरह८फाईइटसहह८ताहहॴ 2) Main plant body is diploid

पह९दह८कामखयिरीरदपवगखणतहह८ताहहॴ 3) Seeds are present

बीजमह९जदहह८तहॳहहॴ 4)Flowers are absent

फिअनपनसतिहह८तहॳहहॴ

Correct Answer Seeds are present

Q132 The largest dolphin species is the

orca also called as

िॉिकफनकीसबसहॳबड़ीपरजानतकाकानामआकायहहॴनजसहॳ mdash- भीकहतहॳहहॴ 23-Jan-2017

Options

1) Bottle Nose

बाटिनह८ज

2) Baiji

बहॳजी 3) Killer whale

ककिरहहॳि

4)Tucuxi

टकवसी Correct Answer Killer whale

Q133 The fat digesting enzyme Lipase

is secreted by which of the following

वसाकापाचनकरनहॳवािाएजाइमिाइपहॳजनननननलिखितमसहॳककसकहॳ दवारासतरापवतहह८ताहहॴ

24-Jan-2017

Options

1) Kidneys

गद

2) Pancreas

अगनयािय

3) Large Intestine

बड़ीआत

4)Liver

नजगर

Correct Answer Pancreas

Lipase is an enzyme that splits fats so

the intestines can absorb them Lipase

hydrolyzes fats like triglycerides into

their component fatty acid and glycerol

molecules It is found in the blood

gastric juices pancreatic secretions

intestinal juices and adipose tissues

F A C E B O O K

P A G E h t t p w w w f a c e b o o k c o m s s c m e n t o r s o f f i c i a l P a g e | 32

FOR MORE UPDATES AND MORE MATERIAL DO LIKE OUR FACEBOOK PAGE httpwwwfacebookcomsscmentorsofficial

Q134 The arrangement of leaves on an

axis or stem is called

एकअकषयातनहॳपरपनततयोकीयवसिाकह८कयाकहाजाताहहॴ SSC CHSL Science (biology) 2016

Question Paper

24-Jan-2017

Options

1) Phyllotaxy

फाइिह८टहॴकसी 2) Vernation

वनिन

3) Venation

वहॳनहॳिन

4)Phytotaxy

फाइटह८टहॴकसी Correct Answer Phyllotaxy

In botany phyllotaxis or phyllotaxy is

the arrangement of leaves on a plant

stem (from Ancient Greek phyacutellon

ldquoleafrdquo and taacutexis ldquoarrangementrdquo)

Phyllotactic spirals form a distinctive

class of patterns in nature

Q135 The study of Cells is also known

as

कह८लिकाओकहॳ अधययनकह८ mdashmdashndash

भीकहाजाताहहॴ 24-Jan-2017

Options

1) Cytology

सायटह८िह८जी 2) Physiology

कफनजयह८िह८जी 3) Nucleology

नयककमयह८िह८जी 4)Cellology

सहॳिह८िह८जी Correct Answer Cytology

Q136 Which of the following scientists

is also known as the Father of Biology

नननननलिखितमसहॳककसवहॴजञाननककह८ ldquoजीवपवजञानकहॳ जनकrdquoकहॳ नामसहॳभीजानाजाताहहॴ 24-Jan-2017

Options

1) Herbert Spencer

हबयटयसपसर

2) Aristotle

अरसत 3) Lamarck

िहॳमाकय 4)Darwin

िापवयन

Correct Answer Aristotle

Q137 Which cells give rise to various

organs of the plant and keep the plant

growing

कह९नसीकह८लिकाएपह९धह८कहॳ लभननअगह८कह८जनमदहॳतीहहॴऔरपह९धह८कह८बढ़नहॳममददकरतीहहॴ

24-Jan-2017

Options

1) Permanent

सिायी 2) Dermal

तवचीय

3) Meristematic

मररसटहॳमटटक

4)Mature

परह८ढ़

Correct Answer Meristematic

A meristem is the tissue in most plants

containing undifferentiated cells

(meristematic cells) found in zones of

the plant where growth can take place

Q138 Rodentia Muridae is the scientific

name of

F A C E B O O K

P A G E h t t p w w w f a c e b o o k c o m s s c m e n t o r s o f f i c i a l P a g e | 33

FOR MORE UPDATES AND MORE MATERIAL DO LIKE OUR FACEBOOK PAGE httpwwwfacebookcomsscmentorsofficial

रह८िहॳलियानयररिी mdashmdash- कावहॴजञाननकनामहहॴ 24-

Jan-2017

Options

1) Mouse

चहा 2) Squirrel

चगिहरी 3) Monkey

बदर

4) Lizard

नछपकिी Correct Answer Mouse

Q139 Name the scientist who proposed

the cell theory

कह८लिकालसदातकापरसतावदहॳनहॳवािहॳवहॴजञाननककानामबताइए 24-Jan-2017

Options

1) Schleiden and Schwann

िीमिनऔरशरववान

2) Lamarck

िहॳमाकय 3) Treviranus

टरहॳवायरहॳनस

4)Whittaker and Stanley

हीटकरऔरसटहॳनिहॳ Correct Answer Schleiden and

Schwann

Q140 The flower with the worldrsquos

largest bloom is

दननयाकासबसहॳबड़ाफिखििनहॳवािा mdashmdashndash हहॴ 24-Jan-2017

Options

1) Pando

पािह८ 2) Posidonia

पह८सीिह८ननया 3) Rafflesia arnoldii

ररफिहॳलियाअनोमिी 4)Helianthus annuus

हहॳलिएनिसएनयअस

Correct Answer Rafflesia arnoldii

Rafflesia arnoldii is a species of

flowering plant in the parasitic genus

Rafflesia It is noted for producing the

largest individual flower on earth It has

a very strong and horrible odour of

decaying flesh earning it the nickname

ldquocorpse flower

Q141 Deficiency of which vitamin

causes night blindness

ककसपवटालमनकीकमीकहॳ कारणरतौधीहह८ताहहॴ 24-Jan-2017

Options

1) Vitamin K

पवटालमन K

2) Vitamin C

पवटालमन C

3) Vitamin B1

पवटालमन B1

4)Vitamin A

पवटालमन A

Correct Answer Vitamin A

Q142 Nongreen plants lack which of the

following

गहॴर-

हररतवनसपनतमनननननलिखितमसहॳककसकीकमीहह८तीहहॴ

24-Jan-2017

Options

1) Chlorophyll

किह८रह८कफि

2) Lycophyll

िायकह८कफि

3) Cyanophyll

F A C E B O O K

P A G E h t t p w w w f a c e b o o k c o m s s c m e n t o r s o f f i c i a l P a g e | 34

FOR MORE UPDATES AND MORE MATERIAL DO LIKE OUR FACEBOOK PAGE httpwwwfacebookcomsscmentorsofficial

सायनह८कफि

4)Phototropism

फह८टह८टरोपपजम

Correct Answer Chlorophyll

Q143 Organisms that use light to

prepare food are known as

जह८जीवपरकािकाउपयह८गकरभह८जनतहॴयारकरतहॳहहॴ उनह mdashmdash- कहॳ पमजानजाताहहॴ 24-Jan-2017

Options

1) Autotrophs

सवपह८षी 2) Heterotrophs

पवषमपह८षज

3) Omnivores

सवायहारी 4)Decomposers

पवघटनकरनहॳवािा Correct Answer Autotrophs

autotrophs often make their own food

by using sunlight carbon dioxide and

water to form sugars which they can use

for energy Some examples of

autotrophs include plants algae and

even some bacteria Autotrophs

(producer) are important because they

are a food source for heterotrophs

(consumers)

A heterotroph is an organism that

ingests or absorbs organic carbon

(rather than fix carbon from inorganic

sources such as carbon dioxide) in order

to be able to produce energy and

synthesize compounds to maintain its

life Ninety-five percent or more of all

types of living organisms are

heterotrophic including all animals and

fungi and some bacteria

Q144 Which of the following is a

primary function of haemoglobin

नननननलिखितमसहॳकह९नसाटहमह८गिह८बबनकाएकपरािलमककाययहहॴ

25-Jan-2017

Options

1) Utilization of energy

उजायकाउपयह८गकरना 2) Prevention of anaemia

रकतामपताहह८नहॳसहॳरह८कना 3) Destruction of bacteria

बहॴकटीररयाकापवनािकरना 4) To transport oxygen

ऑकसीजनकावहनकरना Correct Answer To transport oxygen

Q145 Vascular bundles are absent in

सवहनीबिि mdashmdash- मअनपनसतिरहतहॳहहॴ 25-Jan-2017

Options

1) Bryophyta

िायह८फाइटा 2) Pteridophyta

टहॳररिह८फाईटा 3) Gymnosperms

नजननह८सपमय 4) Angiosperms

एननजयह८सपहॳनसय Correct Answer Bryophyta

Q146 Sauria Lacertidae is the scientific

name of

सहॴररयािहॳसरटाईिी mdashmdashndash कावहॴजञाननकनामहहॴ 25-Jan-2017

Options

1) Crocodile

मगरमचछ

2) Hippopotamus

टहपपह८पह८टहॳमस

3) Lizard

नछपकिी 4) House fly

F A C E B O O K

P A G E h t t p w w w f a c e b o o k c o m s s c m e n t o r s o f f i c i a l P a g e | 35

FOR MORE UPDATES AND MORE MATERIAL DO LIKE OUR FACEBOOK PAGE httpwwwfacebookcomsscmentorsofficial

घरहॳिमकिी Correct Answer Lizard

Q147 Which type of pathogen causes

the water-borne disease SARS (Severe

Acute Respiratory Syndrome)

ककसपरकािकारह८गज़नकजिजननतबीमारीसासयकाकारणबनताहहॴ 25-Jan-2017

Options

1) Viral

वायरि

2) Parasitic

परजीवी 3) Protozoan

परह८टह८जअन

4) Bacterial

बहॴकटीररयि

Correct Answer Viral

Q148 Which of the following organs

produces the enzyme lipase

नननननलिखितमसहॳकह९नसाअगिायपहॳजएजाइमउतपननकरताहहॴ 25-Jan-2017

Options

1) Pancreas

अगनयािय

2) Large Intestine

बड़ीआत

3) Liver

नजगर

4) Small Intestine

छह८टीआत

Correct Answer Pancreas

Q149 A is a long internode forming the

basal part or the whole of a peduncle

एक mdashmdash- एकिबाइटरनह८िहहॴ जह८ननचिाटहससायासनपणयिठिबनताहहॴ 25-

Jan-2017

Options

1) Rhizome

परकद

2) Rachis

महॳ दि

3) floral axis

पषपअकष

4) Scape

भगदड़

Correct Answer scape

Q150 ndash Which of the following

organisms are considered to be both

Living and Non-living

नननननलिखितमसहॳकह९नसहॳजीवाणकह८जीपवतऔरअजीपवतमानाजाताहहॴ

25-Jan-2017

Options

1) Bacteria

बहॴकटीररया 2) Fungi

कवक

3) Algae

िहॴवाि

4)Virus

वायरस

Correct Answer Virus

They are considered to be living as they

possess a protein coat as a protective

covering DNA as the genetic material

etc

They are said to be non-living as they

can be crystallised and they survive for

billions of years They can tolerate high

temperatures freezing cold

temperatures ultra-violet radiations etc

Q151 Deficiency of fluorine causes

which of the following

फिह८ररनकीकमीकहॳ कारणनननननलिखितमसहॳकयाहह८ताहहॴ

F A C E B O O K

P A G E h t t p w w w f a c e b o o k c o m s s c m e n t o r s o f f i c i a l P a g e | 36

FOR MORE UPDATES AND MORE MATERIAL DO LIKE OUR FACEBOOK PAGE httpwwwfacebookcomsscmentorsofficial

27-Jan-2017

Options

1) Dental Caries

िटिकहॴ ररज

2) Scurvy

सकवरी 3) Anaemia

रकतामपता 4) Arthritis

गटठया Correct Answer Dental Caries

Q152 In a Punnett Square with the

cross AaBb x AaBb how many Aabb

genotypes would be created

पनहॳटसककायरमिह८स AaBb x AaBb कहॳ साि

ककतनहॳ Aabb जीनह८टाइपबनगहॳ 27-Jan-2017

Options

1) 1

2) 8

3) 2

4) 3

Correct Answer 2

Q153 Which of the following is the

Controlling Center of the Cell

नननननलिखित म सहॳ कह८लिकाका ननयतरण

क दर कह९न हहॴ

27-Jan-2017

Options

1) Nucleus

क दर

2) Plasma

पिाजमा 3) Lysosome

िायसह८सह८म

4) Chromosome

िह८मह८सह८म

Correct Answer Nucleus

The control centre of the cell is the

nucleus in eukaryotic cells The nucleus

contains genetic material in the form of

DNA

Q154 Myopia affects which of the

following organs

मायह८पपयानननननलिखितअगह८मसहॳककसहॳपरभापवतकरताहहॴ

25-Jan-2017

Options

1) Heart

हदय

2) Skin

तवचा 3) Eyes

आािहॳ 4)Mouth

मह

Correct Answer Eyes

Q155 Which of the following bears

flowers

नननननलिखितमसहॳकह९नफिधारणकरताहहॴ

25-Jan-2017

Options

1) Bryophyta

िायह८फाइटा 2) Pteridophyta

टहॳरीिह८फाईटा 3) Gymnosperms

नजननह८सपमय 4)Angiosperms

एननजयह८सपमय Correct Answer Angiosperms

Q156 Oxygenated blood flows out of the

heart through the

ऑकसीजनयकतरकत mdashmdashmdash

कहॳ माधयमसहॳहदयकहॳ बाहरबहताहहॴ 25-Jan-2017

F A C E B O O K

P A G E h t t p w w w f a c e b o o k c o m s s c m e n t o r s o f f i c i a l P a g e | 37

FOR MORE UPDATES AND MORE MATERIAL DO LIKE OUR FACEBOOK PAGE httpwwwfacebookcomsscmentorsofficial

Options

1) Aorta

महाधमनी 2) pulmonary artery

फहॳ फड़हॳकीधमनी 3) vena cava

वहॳनाकावा 4)Atrium

चह९क

Correct Answer aorta

Q157 Blood leaving the liver and

moving towards the

heart has a higher concentration of

नजगरसहॳननकिकरहदयकीतरफजानहॳवािहॳरकतम mdashmdashmdashmdash कीउचचसादरताहह८तीहहॴ 27-Jan-2017

Options

1) Lipids

लिपपडस

2) Urea

यररया 3) Bile Pigments

पपततकहॳ रगकरण

4) Carbon dioxide

काबयनिायऑकसाइि

Correct Answer Bile Pigments

Urea is nitrogen containing substance

which is produced in the liver in order

to deal with excess amino-acids in the

body As urea is produced it leaves the

liver in the blood stream and passes via

the circulatory system to all parts of the

body

Q158 Bulb is a modification of which

part of a plant

बमबएकपह९धहॳकहॳ ककसटहससहॳकाएक पातरणहह८ताहहॴ 27-Jan-2017

Options

1) The root

जड़

2) The stem

तना 3) The radicle

मिाकर

4)The fruit

फि

Correct Answer The stem

Q159 Which of the following carries

blood away from the heart to different

body parts

इनमहॳसहॳकह९नरकतकह८हदयसहॳिरीरकहॳ पवलभननअगह८तकिहॳजातीहहॴ

27-Jan-2017

Options

1) Arteries

धमननया 2) Nerves

तबतरहाए

3) Capillaries

कहॳ लिकाए

4)Veins

नसहॳ Correct Answer Arteries

Q160 The series of processes by which

nitrogen and its compounds are

interconverted in the environment and

in living organisms is called

27-Jan-2017

Options

1)Absorption of Nitrogen

2)Ammonification

3)Nitrogen Fixation

4)Nitrogen Cycle

Correct Answer Nitrogen Cycle

Ammonification or Mineralization is

performed by bacteria to convert

organic nitrogen to ammonia

F A C E B O O K

P A G E h t t p w w w f a c e b o o k c o m s s c m e n t o r s o f f i c i a l P a g e | 38

FOR MORE UPDATES AND MORE MATERIAL DO LIKE OUR FACEBOOK PAGE httpwwwfacebookcomsscmentorsofficial

Nitrification can then occur to convert

the ammonium to nitrite and nitrate

Nitrogen fixation is a process by which

nitrogen in the Earthrsquos atmosphere is

converted into ammonia (NH3) or other

molecules available to living organisms

Q161 BCG vaccine is given to protect

from which of the following

बीसीजीकाटटकानननननलिखितमसहॳककसकहॳ बचावकहॳ लिएटदयाजातहहॴ

27-Jan-2017

Options

1) Jaundice

पीलिया 2) Anaemia

रकतमपता 3) Tuberculosis

कषयरह८ग

4) Polio

पह८लियह८ Correct Answer Tuberculosis

Q162 Parallel venation is found in

समानतरवहॳनहॳिन mdashmdashmdash- मपायाजाताहहॴ 27-Jan-2017

Options

1) plants which are monocots

पह९धहॳजह८एकबीजपतरीहह८तहॳहहॴ 2) plants which have a dicot stem

वहॳपह९धहॳनजनकातनादपवदलियहह८ताहहॴ 3) plants with leaves similar to Tulsi

वहॳपह९धहॳनजनकीपनततयतिसीकीपनततयोकहॳ समानहह८तहॳहहॴ 4)plants with tap roots

टहॳप टवािहॳपह९धहॳ Correct Answer plants which are

monocots

Q163 The hardest part of the body is

िरीरकासबसहॳकठह८रभाग mdashndash हहॴ 27-Jan-2017

Options

1) Bones

हडडिय

2) Tooth Enamel

दातकहॳ इनहॳमि

3) Skull

िह८पड़ी 4) Spinal Cord

महॳ रजज

Correct Answer Tooth Enamel

Q164 Which type of pathogen causes

the waterborne disease E coli Infection

ककसपरकारकारह८गजननकजिजननतरह८गईकह८िाईसिमणकाकारणबनताहहॴ 27-Jan-2017

Options

1) Protozoan

परह८टह८जआ

2) Parasitic

परजीवी 3) Bacterial

बहॴकटीररयि

4)Viral

वायरि

Correct Answer Bacterial

Q165 The amount of blood filtered

together by both the kidneys in a 70 kg

adult male human in a minute is

70 की गरा वािहॳएकवयसकप षमएकलमनटमदह८नोगदकहॳदवाराएकसािचाबनीगयीरकतकीमातरहह८तीहहॴ 29-Jan-2017

Options

1) 1100 ml

1100 लमलि

2) 100 ml

F A C E B O O K

P A G E h t t p w w w f a c e b o o k c o m s s c m e n t o r s o f f i c i a l P a g e | 39

FOR MORE UPDATES AND MORE MATERIAL DO LIKE OUR FACEBOOK PAGE httpwwwfacebookcomsscmentorsofficial

100 लमलि

3) 1500 ml

1500 लमलि

4) 500 ml

500 लमलि

Correct Answer 1100 ml

Q166 Which feature of a plant helps to

distinguish a monocot from a dicot

पह९धहॳकीवहकह९नसीपविहॳषताहहॴजह८एकदपवदलियहॳऔरएकएकदिीयपह९धहॳसहॳभहॳदकरनहॳममददकरतीहहॴ 29-Jan-2017

Options

1) Pollination

परागम

2) Venation

वहॳनहॳिन

3) Vernation

वनिन

4) Aestivation

एसटीवहॳिहॳन

Correct Answer venation

Q167 The Mutation Theory was

proposed by

उतवररवतयनकालसदात mdashmdashndash

कहॳ दवरापरसतापवतककयाजाताहहॴ 29-Jan-2017

Options

1) Charles Lyell

चामसयलियहॳि

2) William Smith

पवलियमनसमि

3) Hugo De Vries

हयगह८िीराईस

4)Harrison Schmitt

हहॳरीसननसमट

Correct Answer Hugo De Vries

Q168 Which type of pathogen causes

the waterborne disease HepatitisA

ककसपरकारकहॳ रह८गजनकजिजननतरह८गहहॳपहॳटाइटटस-A काकारणबनताहहॴ

29-Jan-2017

Options

1) Parasitic

परजीवी 2) Viral

वायरि

3) Protozoan

परह८टह८जआ

4) Bacterial

बहॴकटीररयि

Correct Answer Viral

Q169 In a Punnett Square with the

cross AaBb x Aabb how many AaBb

genotypes would be created

पनहॳटसकवायरमिह८स AaBb x Aabb

कहॳ सािककतनहॳ AaBb जीनह८टाइपबनगहॳ 29-Jan-

2017

Options

1) 4

2) 1

3) 7

4) 6

Correct Answer 4

Q170 Arboreal Ateles is the scientific

name of

अिह८ररयिएटटलिस mdashmdashmdash कावहॴजञाननकनामहहॴ 29-Jan-2017

Options

1) Squirrel

चगिहरी 2) Sparrow

गह८रहॴया 3) Lizard

नछपकिी 4) Spider monkey

F A C E B O O K

P A G E h t t p w w w f a c e b o o k c o m s s c m e n t o r s o f f i c i a l P a g e | 40

FOR MORE UPDATES AND MORE MATERIAL DO LIKE OUR FACEBOOK PAGE httpwwwfacebookcomsscmentorsofficial

मकड़ीबदर

Correct Answer Spider monkey

Q171 Which type of pathogen causes

the waterborne disease Salmonellosis

ककसपरकारकारह८गाणजिजननतबीमारीसािमह८नहॳिह८लसज़काकारकहहॴ

29-Jan-2017

Options

1) Algal

िहॳवालियहॳ 2) Parasitic

परजीवी 3) Bacterial

बहॴकटीररयि

4)Viral

वायरि

Correct Answer Bacterial

An infection with salmonella bacteria

commonly caused by contaminated food

or water

Symptoms include diarrhoea fever

chills and abdominal pain

Q172 is a condition in which there is a

deficiency of red cells or of haemoglobin

in the blood

mdashmdash-

एकनसिनतहहॴनजसमहॳरकतमिािकह८लिकाओकीयाहीमह८गिह८बबनकीकमीहह८तीहहॴ 29-Jan-2017

Options

1) Albinism

एनमबननजम

2) Propyria

परह८पीररया 3) Anaemia

एनीलमया 4)Keloid disorder

कहॳ िह८इिडिसओिर

Correct Answer Anaemia

Q173 Ananas comosus is the scientific

name of

Options

अनानासकह८मह८सस mdashmdashmdashndash

कावहॴजञाननकनामहहॴ 29-Jan-2017

1) Custard Apple

सीताफि

2) Pineapple

पाइनएपपि

3) Bamboo

बास

4)Pomegranate

अनार

Correct Answer Pineapple

Q174 Which organ produces insulin

कह९नसाअगइनसलिनपहॴदाकरताहहॴ 29-Jan-

2017

Options

1) Liver

यकत

2) Thyroid gland

िायराइिगरिी 3) Spleen

पिीहा 4)Pancreas

अगरयिय

Correct Answer Pancreas

Q175 Which of the following disease is

not caused by water pollution

नननननलिखितमसहॳकह९नसारह८गपानीकहॳ परदषणकहॳकारणनहीहह८ता

29-Jan-2017

Options

1) Cholera

हहॴजा 2) Typhoid

F A C E B O O K

P A G E h t t p w w w f a c e b o o k c o m s s c m e n t o r s o f f i c i a l P a g e | 41

FOR MORE UPDATES AND MORE MATERIAL DO LIKE OUR FACEBOOK PAGE httpwwwfacebookcomsscmentorsofficial

टाइफाइि

3) Asthma

दमा 4)Diarrhoea

दसत

Correct Answer Asthma

Q176 Ocimum tenuiflorum is the

scientific name of

ओलिलममटहॳयईफिह८रमइसकावहॴजञाननकनाम mdash

ndash हहॴ 30-Jan-2017

Options

1) Neem

नीम

2) Mango

आम

3) Babul

बबि

4)Tulsi

तिसी Correct Answer Tulsi

Q177 Which gland secretes bile a

digestive fluid

कह९नसीगरिीपपतत एकपाचनतरिपरदािय सरापवतकरतीहहॴ 30-Jan-2017

Options

1) Pancreas

अगनयािय

2) Liver

यकत

3) Thyroid

िायराइि

4) Testes

टहॳनसटस

Correct Answer liver

Q178 In which of the following the

dominant phase is Gametophyte

नननननलिखितमसहॳककसकहॳ परमिचरणयगमकह८दपवधद (Gametophyte)हहॴ 30-Jan-2017

Options

1) Bryophyta

िायह८फाइटा 2) Pteridophyta

टहॳररिह८फाइटा 3) Gymnosperms

नजननह८सपमय 4) Angiosperms

एननजयह८सपमय Correct Answer Bryophyta

Q179 Anaerobic respiration refers to

which of the following

नननननलिखितमसहॳककसहॳअवायवीयशवसनकहाजाताहहॴ

30-Jan-2017

Options

1) Respiration without Oxygen

ऑकसीजनकहॳ बबनाशवसन

2) Respiration with Oxygen

ऑकसीजनकहॳ सािशवसन

3) Respiration without CO2

काबयनिायऑकसाइिकहॳ बबनाशवसन

4) Respiration with CO2

काबयनिायऑकसाइिकहॳ सािशविन

Correct Answer Respiration without

Oxygen

Q180 Which type of pathogen causes

the waterborne disease Cholera

ककसपरकारकारह८गजनकजिजननतरह८गहहॴजाकाकारणबनताहहॴ

30-Jan-2017

Options

1) Algal

िहॴवालियहॳ

F A C E B O O K

P A G E h t t p w w w f a c e b o o k c o m s s c m e n t o r s o f f i c i a l P a g e | 42

FOR MORE UPDATES AND MORE MATERIAL DO LIKE OUR FACEBOOK PAGE httpwwwfacebookcomsscmentorsofficial

2) Bacterial

बहॴकटीररयि

3) Protozoan

परह८टह८जआ

4) Viral

वायरि

Correct Answer Bacterial

Q181 To which class does

Oxyreductases transferases hydrolases

belong

ओकसीररिकटहॳसटरासफरहॳजहॳस

हाइडरह८िहॳसहॳसककसवगयमआतहॳहहॴ 30-Jan-2017

Options

1) Hormones

हारमोस

2) Enzymes

एजाइनस

3) Proteins

परह८टीनस

4) Vitamins

पवटालमनस

Correct Answer Enzymes

Q182 Which of the following is not true

about Gymnosperms

ननननमसहॳकह९नसीबातअनावतबीजीकहॳ बारहॳमसचनहीहहॴ 30-Jan-2017

Options

1) Dominant phase is saprophytes

परमिचरणसहॳपरह८फाइटसहह८ताहहॴ 2) Vascular bundles are absent

सवहनीबििअनपनसितहह८ताहहॴ 3) spores are heterospores

बीजाणहहॳटहॳरह८सपह८रसहह८तहॳहहॴ 4) Flowers are absent

फिअनपनसितहह८तहॳहहॴ

Correct Answer Vascular bundles are

absent

Q183 The name of first mammal clone sheep is

भहॳड़कीपरिमसतनपायीपरनत प (किह८न)

कानामहहॴ 30-Jan-2017

Options

1) Noori

नरी 2) Dolly

िॉिी 3) Louise

िसी 4)Durga

दगाय Correct Answer Dolly

Q184 Which type of pathogen causes

the water-borne disease Typhoid fever

ककसपरकारकारह८गजनकजिजननतरह८गटाइफाइिबिारकाकारणबनताहहॴ 30-Jan-2017

Options

1) Algal

िहॴवािीय

2) Parasitic

परजीवी 3) Protozoan

परह८टह८जनअन

4)Bacterial

बहॴकटीररयि

Correct Answer Bacterial

Q185 In which part of the cell are

proteins made

कह८लिकाकहॳ ककसटहससहॳमपरह८टीनबनायाजाताहहॴ

31-Jan-2017

Options

1) Reticulum

रहॳटटकिम

F A C E B O O K

P A G E h t t p w w w f a c e b o o k c o m s s c m e n t o r s o f f i c i a l P a g e | 43

FOR MORE UPDATES AND MORE MATERIAL DO LIKE OUR FACEBOOK PAGE httpwwwfacebookcomsscmentorsofficial

2) Golgi apparatus

गह८मजीएपहॳरहॳटस

3) Ribosomes

ररबह८सह८नस

4) Lysosome

िायसह८सह८नस

Correct Answer ribosomes

Proteins are produced by stringing

amino acids together in the order

specified by messenger RNA strands

that were transcribed from DNA in the

cell nucleus The process of synthesizing

a protein is called translation and it

occurs on ribosomes in the cytoplasm of

a cell

Q186 Polio is a disease caused by which

of the following

नननननलिखितमसहॳपह८लियह८कीबबमारह८हह८नहॳकाकारणकयाहहॴ

31-Jan-2017

Options

1) Bacteria

बहॴकटीररयि

2) Mosquito

मचछर

3) Virus

वायरस

4) Cockroach

नतिच हॳ Correct Answer Virus

Polio or poliomyelitis is a crippling and

potentially deadly infectious disease It

is caused by the poliovirus

Q187 ndash Hay fever is a sign of which of

the following

हहॳकफवरनननननलिखितमसहॳककसकाएकसकहॳ तहहॴ

31-Jan-2017

Options

1) Old Age

वदावसिा 2) Malnutrition

कपह८सण

3) Allergy

एिनजय 4) Over Work

अतयचधककाययकरना Correct Answer Allergy

Q188 How many chromosomes does a

human cell contain

एकमानवकह८लिकामककतनहॳगणसतरहह८तहॳहहॴ

29-Jan-2017

Options

1) 6

2) 26

3) 46

4) 66

Correct Answer 46

In humans each cell normally contains

23 pairs of chromosomes for a total of

46 Twenty-two of these pairs called

autosomes look the same in both males

and females The 23rd pair the sex

chromosomes differ between males and

females

Q189 Which of the following is not true

about Bryophyta

ननननमसहॳकह९नसीबातिायह८फाइटकहॳ बारहॳमसचनहीहहॴ 31-Jan-2017

Options

1) Dominant phase is gametophytes

परमिचरणगहॳलमतह८फाइटसहह८ताहहॴ 2) Main plant body is haploid

पह९धहॳकामखयिरीरअगखणतहह८ताहहॴ 3) Spores are homospores

बीजाणहह८मह८सफह८रसहह८तहॳहहॴ 4) Flowers are present

फिमह८जदहह८तहॳहहॴ Correct Answer Flowers are present

F A C E B O O K

P A G E h t t p w w w f a c e b o o k c o m s s c m e n t o r s o f f i c i a l P a g e | 44

FOR MORE UPDATES AND MORE MATERIAL DO LIKE OUR FACEBOOK PAGE httpwwwfacebookcomsscmentorsofficial

Q190 Which aquatic animal has

trailing tentacles

ककसजिीयजानवरकहॳ पीछहॳचिनहॳवािहॳटहॳटकिसहह८तहॳहहॴ

31-Jan-2017

Options

1) Sea horse

समदरीघह८िा 2) Corals

मगा 3) Jelly fish

जहॳिीमछिी 4) Star fish

तारामछिी Correct Answer Jelly fish

Jellyfish with its umbrella-shaped bell

and trailing tentacles

Q191 Which type of pathogen causes

the water-borne disease Poliomyelitis

(Polio)

ककसपरकारकारह८गजनकजिजननतरह८गपह८लियह८मायहॳटटस (पह८लियह८) काकारणहहॴ 31-Jan-

2017

Options

1) Parasitic

परजीवी 2) Algal

िहॴवालिय

3) Viral

वायरि

4) Bacterial

बहॴकटीररयि

Correct Answer Viral

Q192 The outer white part of the eye

that protects the inner structures is

आािकाबाहरीसफहॳ दटहससाजह८आतररकसरचनाओकीरकषाकरताहहॴ वह mdashmdashmdash हहॴ 31-Jan-

2017

Options

1) Iris

आयररस

2) Sclera

सकिहॳरा 3) Retina

रहॳटटना 4) Cornea

कह८ननयया Correct Answer Sclera

Q193 Proteins are made up of

परह८टीनकाननमायण mdashndash सहॳहह८ताहहॴ 31-Jan-2017

Options

1) Amino acids

एलमनह८अनि

2) Fatty acids

वसायकतअनि

3) Glucose

गिकह८ज

4)Nucleotides

नयनकियह८टाईिस

Correct Answer Amino acids

Q194 Moringa Oleifera is the scientific

name of

मह८ररगओलिफहॳ रा mdashmdashndash कावहॴजञाननकनामहहॴ 31-Jan-2017

Options

1) Banyan

बरगद

2) Gulmohar

गिमह८हर

3) Amla

आमिा

F A C E B O O K

P A G E h t t p w w w f a c e b o o k c o m s s c m e n t o r s o f f i c i a l P a g e | 45

FOR MORE UPDATES AND MORE MATERIAL DO LIKE OUR FACEBOOK PAGE httpwwwfacebookcomsscmentorsofficial

4) Drumstick

डरमनसटक

Correct Answer Drumstick

Q195 Kidney stones are composed of

गदकीपिरी mdashndash सहॳबनीहह८तीहहॴ 1-Feb-2017

Options

1) Calcium Oxalate

कहॴ नमसयमओकजहॳिहॳट

2) Sodium Chloride

सह८डियमकिह८राइि

3) Magnesium Nitrate

महॳनगनलियमनाइतटरहॳट

4) Calcium Bicarbonate

कहॴ नमियमबायकबोनहॳट

Correct Answer Calcium Oxalate

Q196 ndash Which of the following is not

true about Angiosperms

ननननमसहॳकह९नसीबातआवतबीजीकहॳ बारहॳमसचनहीहहॴ 1-Feb-2017

Options

1) Dominant phase is gametophytes

परमिचरणगहॳलमतह८फाइटहह८ताहहॴ 2) Vascular bundles are present

सवहनीबििमह९जदहह८ताहहॴ 3) Spores are heterospores

बीजाणहहॳटहॳरह८सपह८रसहह८तहॳहहॴ 4) Seeds are covered

बीजढकहॳ हह८तहॳहहॴ Correct Answer Dominant phase is

gametophytes

Q197 All of the following are excretory

(waste) products of animals except

नननननलिखितमसहॳककसएककह८छह८ड़करअनयसभीपराखणयोदवाराउतसनजयतपदाियहहॴ 1-Feb-

2017

Options

1) Uric Acid

यररकएलसि

2) Ammonia

अमह८ननया 3) Carbohydrates

काबोहाइडरहॳट

4) Urea

यररया Correct Answer Carbohydrates

In animals the main excretory products

are carbon dioxide ammonia (in

ammoniotelics) urea (in ureotelics) uric

acid (in uricotelics) guanine (in

Arachnida) and creatine

Q198 RNA is a polymeric molecule

What does RNA stand for

आरएनइएएकबहिकआणहहॴ इसकाकापवय पकयाहहॴ 1-Feb-2017

Options

1) Rado Nuclear Acid

रािह८नयनकियरएलसि

2) Ribo Nucleic Acid

राइबह८नयनकिकएलसि

3) Rhino Nuclear Acid

हाइनह८नयनकियरएलसि

4) Resto Nucleus Acid

रहॳसटह८नयकिीयसएलसि

Correct Answer Ribo Nucleic Acid

Q199 Which organ does detoxification

and produces chemicals needed for

digestion

कह९नसाअगपवषहरणकरताहहॴऔरपाचनकहॳ लिएआवशयकरसायनोकह८पहॴदाकरताहहॴ 1-Feb-

2017

Options

1) Salivary glands

िारगरचिया 2) Pancreas

अगनयािय

F A C E B O O K

P A G E h t t p w w w f a c e b o o k c o m s s c m e n t o r s o f f i c i a l P a g e | 46

FOR MORE UPDATES AND MORE MATERIAL DO LIKE OUR FACEBOOK PAGE httpwwwfacebookcomsscmentorsofficial

3) Thyroid gland

िायराइिगरिी 4) Liver

यकत

Correct Answer Liver

Q200 Psidium guajava is the scientific

name of

लसडियमगआजावा mdashmdash कावहॴजञाननकनामहहॴ 1-

Feb-2017

Options

1) Guava

अम द

2) Mango

आम

3) Bamboo

बास

4) Jack fruit

कटहि

Correct Answer Guava

Q201 Which drug is used as a Blood

Thinner

चधरकह८पतिाकरनहॳकहॳ पमककसदवाकापरयह८गककयाजाताहहॴ

1-Feb-2017

Options

1) Warfarin

वाफर न

2) Tramadol

टरहॳमािह८ि

3) Azithromycin

एनजरह८मायलसन

4) Hydralazine

हाइडरह८िहॳनजन

Correct Answer Warfarin

Q202 Which of the following disease is

caused due to the deficiency of protein

परह८टीनकीकमीकहॳ कारणनननननलिखितमसहॳकह९नसारह८गहह८ताहहॴ 1-Feb-2017

Options

1) Arthritis

गटठया 2) Kwashiorkor

कािीओकय र

3) Goitre

गाइटर

4) Night Blindness

रतह९चध

Correct Answer Kwashiorkor

Q203 A is species of plant that has

adapted to survive in an environment

with little liquid water

mdashmdashndashपह९धहॳकीएकऐसहॳऐसहॳपरजानतहहॴ नजसनहॳकमपानीवािहॳवातावरणमजीपवतरहनहॳकहॳलिएअनकिनहहॴ 1-Feb-2017

Options

1) Xerophyte

म दपवद

2) Hydrophyte

जिीयपादप

3) Mesophyte

समह८दपवद

4) Thallophyte

िहॴिह८फाइटा Correct Answer xerophyte

xerophyte is a species of plant that has

adapted to survive in an environment

with little liquid water such as a desert

or an ice- or snow-covered region in the

Alps or the Arctic

Mesophytes are terrestrial plants which

are adapted to neither a particularly

dry nor particularly wet environment

An example of a mesophytic habitat

would be a rural temperate meadow

F A C E B O O K

P A G E h t t p w w w f a c e b o o k c o m s s c m e n t o r s o f f i c i a l P a g e | 47

FOR MORE UPDATES AND MORE MATERIAL DO LIKE OUR FACEBOOK PAGE httpwwwfacebookcomsscmentorsofficial

which might contain goldenrod clover

oxeye daisy and Rosa multiflora

thallophyte any of a group of plants or

plantlike organisms (such as algae and

fungi) that lack differentiated stems

leaves and roots and that were formerly

classified as a primary division

(Thallophyta) of the plant kingdom

Q204 How many types of teeth are

there in humans

मनषयोमककतनहॳपरकारकहॳ दातहह८तहॳहहॴ

1-Feb-2017

Options

1) 4

2) 5

3) 2

4) 3

Correct Answer 4

teeth -Humans have four types of

teethincisors canines premolars and

molars each with a specific function

The incisors cut the food the canines

tear the food and the molars and

premolars crush the food

Q205 Carica papaya is the scientific name of

कहॴ ररकापपाया mdashmdashndash कावहॴजञाननकनामहहॴ 2-

Feb-2017

Options

1) Peepal

पीपि

2) Papaya

पपीता 3) Tamarind

इमिी 4) Drumstick

ढह८िकाछड़ी Correct Answer Papaya

Q206 Muscles get tired when there is

shortfall of

जब mdashndash कीकमीहह८तीहहॴतबपहॳिीयिकजातीहहॴ 2-Feb-2017

Options

1) Lactic acid

िहॴनकटकएलसि

2) Na+ ions

Na+ आयन

3) ATP

एटीपी 4) Sulphates

समफहॳ टस

Correct Answer ATP

ATP is the energy source muscle fibers

use to make muscles contract

muscle tissuersquos main source of energy

called adenosine triphosphate or ATP

As your muscles use up this energy

source they become tired and fatigued

Oxygen is the key ingredient that helps

create new ATP to replenish the burned

up ATP in your muscles

Q207 Artocarpus integra is the

scientific name of आटह८कापयसइटीगरा mdashmdashmdash कावहॴजञाननकनामहहॴ 2-Feb-2017

Options

1) Guava

अम द

2) Pineapple

अनानास

3) Silver Oak

लसमवरओक

4) Jack fruit

कटहि

Correct Answer Jack fruit

Q208 Which organ stores fat soluble

vitamins

कह९नसाअगवसामघिनिीिपवटालमनह८काभिाराकरताहहॴ

2-Feb-2017

F A C E B O O K

P A G E h t t p w w w f a c e b o o k c o m s s c m e n t o r s o f f i c i a l P a g e | 48

FOR MORE UPDATES AND MORE MATERIAL DO LIKE OUR FACEBOOK PAGE httpwwwfacebookcomsscmentorsofficial

Options

1) Blood

रकत

2) Skin

तवचा 3) Liver

यकत

4) Pancreas

अगनयािय

Correct Answer Liver

Q209 Which disease is caused due to

deficiency of Iodine

आयह८िीनकहॳ कारणकह९नसारह८गहह८ताहहॴ 2-Feb-2017

Options

1) Rickets

ररकहॳ टस

2) Scurvy

सकवी 3) Goitre

गणमािा 4) Growth retardation

पवकासका कना Correct Answer Goitre

rickets A softening and weakening of

bones in children usually due to

inadequate vitamin D

Q210 Grevillea Robusta is the scientific name of

गरहॳपवलियारह८बसटा mdashmdashmdash- कापवजञाननकनामहहॴ 2-Feb-2017

Options

1) Peepal

पीपि

2) Teak

सागह९न

3) Silver Oak

लसमवरओक

4) Jack fruit

कटहि

Correct Answer Silver Oak

Q211 When a Cuttlefish is described as a Molluscs it is at which level of

classification

जबएककटिकफिकह८एकमह८िसकाकहॳ पमवखणयतककयाजाताहहॴतबयहॳवगीकरणकहॳ ककससतरपहॳनसितहहॴ 2-Feb-2017

Options

1) Class

वगय 2) Order

िम

3) Family

पररवार

4) Phylum

सघ

Correct Answer Phylum

Q212 Bambusa dendrocalmus is the

scientific name of बानबसािहॳडराकामस mdashmdashmdash कावहॴजञाननकनामहहॴ 3-Feb-2017

Options

1) Banyan

बरगद

2) Papaya

पपीता 3) Bamboo

बास

4) Pomegranate

अनार

Correct Answer Bamboo

Q213 Acinonyx Jubatus is the scientific name of

एलसनह८ननकसजयबहॳटस mdashmdashmdash

कावहॴजञाननकनामहहॴ 3-Feb-2017

F A C E B O O K

P A G E h t t p w w w f a c e b o o k c o m s s c m e n t o r s o f f i c i a l P a g e | 49

FOR MORE UPDATES AND MORE MATERIAL DO LIKE OUR FACEBOOK PAGE httpwwwfacebookcomsscmentorsofficial

Options

1) Bear

भाि 2) Horse

घह८िा 3) Cheetah

चीता 4) Zebra

जहॳिा Correct Answer Cheetah

Q214 The pale yellow colour of urine is

due to the presence of which pigment

मतरकाफीकापीिारगरगदरयकहॳ उपनसिनतकहॳ कारणहह८ताहहॴ

3-Feb-2017

Options

1) Urochrome

यरह८िह८म

2) Urophyll

यरह८कफि

3) Chlorophyll

किह८रह८कफि

4) Chloroplast

किह८रह८पिासट

Correct Answer Urochrome

Q215 Which of the following constitute

to form a gene

नननननलिखितमसहॳकह९नसीचीज़एकजीनकागठनकरतीहहॴ

3-Feb-2017

Options

1) Polynucleotides

पह८िीनयनकियह८टाईडस

2) Hydrocarbons

हाइडरह८काबोस

3) Lipoproteins

िाईपह८परह८टीनस

4) Lipids

लिपपडस

Correct Answer Polynucleotides

Polynucleotide molecule is a biopolymer

composed of 13 or more nucleotide

monomers covalently bonded in a chain

DNA (deoxyribonucleic acid) and RNA

(ribonucleic acid) are examples of

polynucleotides with distinct biological

function

Q216 Vertebrates belongs to the

phylum

रीढ़कीहडिीवािहॳपराणी mdashmdashmdash

परजानतकहॳ अतगायतआतहॳहहॴ 3-Feb-2017

Options

1) Arthropoda

आरह८पह८ड़ा 2) Annelida

एननलििा 3) Cnidaria

ननिहॳररया 4) Chordata

कह८िटा Correct Answer Chordata

Q217 Punica granatum is the scientific name of

पननकगरहॳनहॳटस mdashmdashmdash कावहॴजञाननकनामहहॴ 3-Feb-2017

Options

1) Custard Apple

सीताफि

2) Gulmohar

गिमह८हर

3) Silver Oak

लसमवरओक

4) Pomegranate

अनार

Correct Answer Pomegranate

F A C E B O O K

P A G E h t t p w w w f a c e b o o k c o m s s c m e n t o r s o f f i c i a l P a g e | 50

FOR MORE UPDATES AND MORE MATERIAL DO LIKE OUR FACEBOOK PAGE httpwwwfacebookcomsscmentorsofficial

Q218 Between a tiger and an monkey

which of the following is different

एकबाघऔरबदरकहॳ बीचनननननलिखितमसहॳकह९नसीबातअिगहहॴ 3-Feb-2017

Options

1) Kingdom

राजय

2) Phylum

जानत

3) Order

िम

4) Class

वगय Correct Answer order

Q219 The artificial heart was invented by

कबतरमहदयका mdashmdashmdash

दवाराअपवषकारककयागयािा 3-Feb-2017

Options

1) Muhammad Yunus

महनमदयनस

2) Linus Yale Jr

िाइनसयहॳिजय

3) Gazi Yasargil

गाजीयासचगयि

4) Paul Winchell

पह९िपवमकि Correct Answer Paul Winchell

Q220 Tamarindus indica is the

scientific name of

टहॳमररनडसइडिका mdashmdash कावहॴजञाननकनामहहॴ 7-

Feb-2017

Options

1) Neem

नीम

2) Pineapple

अनानास

3) Tamarind

इमिी 4)Chiku

चीक

Correct Answer Tamarind

Q221 In eukaryotic cells synthesis of

RNA takes place in the

यकहॳ योटटककह८लिकाओमआरएनएकासशिहॳषण

mdashndash महह८ताहहॴ 7-Feb-2017

Options

1) Mitochondria

माईटह८कोडडरया 2) Centrioles

सटरीयह८मस

3) Ribosomes

ररबह८सह८नस

4) Nucleus

नयनकियस

Correct Answer nucleus

eukaryotic cell -Transcription is the

process of synthesizing ribonucleic acid

(RNA)Synthesis takes place within the

nucleus of eukaryotic cells or in the

cytoplasm of prokaryotes and converts

the genetic code from a gene in

deoxyribonucleic acid ( DNA ) to a

strand of RNA that then directs

proteinsynthesis

Q222 _________is caused by parasites

of the Plasmodium genus

पिाजमह८डियमजातीकहॳ परजीवी mdash- कहॳ कारणहहॴ 7-Feb-2017

Options

1) Dysentery

पहॳचचि

2) Malaria

मिहॳररया 3) Chickenpox

F A C E B O O K

P A G E h t t p w w w f a c e b o o k c o m s s c m e n t o r s o f f i c i a l P a g e | 51

FOR MORE UPDATES AND MORE MATERIAL DO LIKE OUR FACEBOOK PAGE httpwwwfacebookcomsscmentorsofficial

चहॳचक

4) Herpes

हहॳपपयस

Correct Answer Malaria

Q223 Carotene in fruits and vegetables

gives it which color

फिह८औरसनलजयोमनसितकहॳ रह८टीनउनहकह९नसारगपरदानकरताहहॴ 7-Feb-2017

Options

1) Green

हरा 2) Pink

गिाबी 3) Orange

नारगी 4) Blue

नीिा Correct Answer Orange

Q224 Equus Caballus is the scientific

name of

एकवसकहॴ बहॳिस mdashmdashndash कापवजञाननकनामहहॴ 7-Feb-2017

Options

1) Horse

घह८िा 2) Zebra

ज़हॳिा 3) Donkey

गधा 4) Buffalo

भस

Correct Answer Horse

Q225 Elapidae Naja is the scientific name of

एिीपीिीनाजा mdashmdash- कावहॴजञाननकनामहहॴ 8-Feb-2017

Options

1) Cobra

कह८बरा 2) Elephant

हािी 3) Eagle

ग ि

4) Owl

उमि Correct Answer Cobra

Q226 Which disease is caused due to

deficiency of Iron

िह८हकीकमीकहॳ कारणकह९नसारह८गहह८ताहहॴ 8-Feb-

2017

Options

1) Beriberi

बहॳरीबहॳरी 2) Tetany

टहॳटनी 3) Kwashiorkor

कवािीऔरकर

4) Anaemia

रकतामपता Correct Answer Anaemia

Beriberi is a disease caused by a vitamin

B-1 deficiency also known as thiamine

deficiency

Tetany can be the result of an

electrolyte imbalance Most often itrsquos a

dramatically low calcium level also

known as hypocalcemia Tetany can also

be caused by magnesium deficiency or

too little potassium Having too much

acid (acidosis) or too much alkali

(alkalosis) in the body can also result in

tetany

Kwashiorkor also known as

ldquoedematous malnutrition It is a form of

malnutrition caused by a lack of protein

in the diet

Anaemia means that you have fewer red

blood cells than normal or you have less

F A C E B O O K

P A G E h t t p w w w f a c e b o o k c o m s s c m e n t o r s o f f i c i a l P a g e | 52

FOR MORE UPDATES AND MORE MATERIAL DO LIKE OUR FACEBOOK PAGE httpwwwfacebookcomsscmentorsofficial

haemoglobin than normal in each red

blood cell

Q227 is a leaf where the leaflets are

arranged along the middle vein

mdashndashएकपततीहहॴजहापतरकह८कीरचनाक ररयालिराकहॳ आसपासहह८तीहहॴ 8-Feb-2017

Options

1) Pinnately compound leaf

पपनहॳटिीसयकतपतती 2) Palmately compound leaf

पामहॳटिीसयकतपतती 3) Compound leaf

सयकतपतती 4) Simple leaf

साधारणपतती Correct Answer Pinnately compound

leaf

Q228 Haustoria or sucking roots are

found in which of the following

हह८सटह८ररयायाचसनहॳवािीजड़हॳनननननलिखितमसहॳककसमपाईजातीहहॴ 8-Feb-2017

Options

1) Wheat

गहॳह

2) Mango

आम

3) Chestnut

चहॳसटनट

4) Cuscuta

कसकयटा Correct Answer Cuscuta

Haustorial roots -The roots of parasitic

plants which penetrate into the host

tissues to absorb nourishment are

called haustorial roots hellip Also known as suckingor parasitic roots

Q229 Equs Asinus is the scientific name

of

एकवसएलसनस mdashmdashndash कावहॴजञाननकनामहहॴ 8-

Feb-2017

Options

1) Donkey

गधा 2) Cow

गाय

3) Deer

टहरन

4) Kangaroo

कगा

Correct Answer Donkey

Q230 Ficus benghalensis is the scientific name of

फाईकसबहॳनगहॳिहॳलसस mdashndash कापवजञाननकनामहहॴ 8-Feb-2017

Options

1) Banyan

बरगद

2) Pineapple

अनानास

3) Babul

बबि

4) Tulsi

तिसी Correct Answer Banyan

Q231 Equus burchellii is the scientific name of

एकवसबचिी mdashmdash- कापवजञाननकनामहहॴ 8-Feb-2017

Options

1) Horse

घह८िा 2) Zebra

जहॳिा 3) Buffalo

F A C E B O O K

P A G E h t t p w w w f a c e b o o k c o m s s c m e n t o r s o f f i c i a l P a g e | 53

FOR MORE UPDATES AND MORE MATERIAL DO LIKE OUR FACEBOOK PAGE httpwwwfacebookcomsscmentorsofficial

भस

4) Ass

गधा Correct Answer Zebra

Page 21: COMPILATION OF ALL 72 SETS OF BIOLOGY SSC CHSL-2016 · OF BIOLOGY SSC CHSL-2016 PREPARED BY : SSC MENTORS BIOLOGY SPECIAL . F A C E B O O K P A G E : h t t p : / / w w w . f a c e

F A C E B O O K

P A G E h t t p w w w f a c e b o o k c o m s s c m e n t o r s o f f i c i a l P a g e | 20

FOR MORE UPDATES AND MORE MATERIAL DO LIKE OUR FACEBOOK PAGE httpwwwfacebookcomsscmentorsofficial

Options

1) Carrot

गाजर

2) Tomato

टमाटर

3) Potato

आि 4) Onion

पयाज़

Correct Answer Onion

Q81 DNA stands for

िीएनएकापणय प mdashmdash- हहॴ 18Jan2017

Options

1) Di Nucleic Acid

िाईनयनकिकएलसि

2) Deoxy Nucleic Acid

िीओकसीनयनकिकएलसि

3) Diribonucleic Acid

िाईराइबह८नयनकिकएलसि

4) Deoxyribonucleic Acid

िीऑकसीराइबह८नयनकिकएलसि

Correct Answer Deoxyribonucleic Acid

Q82 Organisms that generate energy

using light are known as

जह८जीवाणपरकािकाउपयह८गकरउजायउतपननकरतीहहॴ उनह mdashmdash कहॳ पमजानाजाताहहॴ

18Jan2017

Options

1) Chaemolithotrophs

ककमह८लििह८टरह८पस

2) Oligotrophs

ओलिगह८टरह८पस

3) Bacteria

बहॴकटीररया 4)Photoautotrophs

फह८टह८ओटह८टरह८पस

Correct Answer Photoautotrophs

An oligotroph is an organism that can

live in an environment that offers very

low levels of nutrients

Q83 Which drug is used as an

Antidepressant

ककसदवाएकहतािारह८धीकहॳ पमपयोगककयाजाताहहॴ Options

1) Oxybutynin

ओकसीलयटीनन

2)Tramadol

टरहॳमहॳिह८ि

3 ) Sumatriptan

समहॳटरीपटहॳन

4) Bupropion

लयपरह८पपयह८न

Correct Answer Bupropion

लयपरह८पपयह८न

Q84 The orange colour of carrot is

because of

गाजरकानारगीरगनननननलिखितमसहॳककसीएककीवजहसहॳहह८ताहहॴ 18Jan2017

Options

1) it grows in the soil

यहलम ीमउगतीहहॴ 2) Carotene

कहॴ रह८टीन

3) it is not exposed to sunlight

यहसययपरकािकहॳ सपकय मनहीआती 4) the entire plant is oranqe in colour

सनपणयपह९धानारगीरगकाहह८ताहहॴ Correct Answer Carotene

Q85 Snake venom is highly modified

saliva containing

F A C E B O O K

P A G E h t t p w w w f a c e b o o k c o m s s c m e n t o r s o f f i c i a l P a g e | 21

FOR MORE UPDATES AND MORE MATERIAL DO LIKE OUR FACEBOOK PAGE httpwwwfacebookcomsscmentorsofficial

सापकाजहरअततयाचधकसिह८चधतिारहह८तीहहॴनजसमहॳ mdashmdash- हह८ताहहॴ Options

l)Prototoxins

परह८टह८टॉनकसस

2)Neutrotoxins

नयटरोटॉनकसस

3)Zootoxins

जटॉनकसस

4)Electrotoxins

इिहॳकटरह८टॉनकसस

Correct Answer Zootoxins

जटॉनकसस

Q86 Which type of pathogen causes the

water-borne disease Schistosomiasis

ककसपरकारकारह८गज़नकजिजननतरह८गलससटह८सह८लमलससकाकारणबनताहहॴ

18Jan2017

Option

1) Parasitic

परजीवी 2)Protozoan

परह८टह८जआ

3) Bacterial

बहॴकटीररयि

4) Viral

वायरि

Correct Answer Parasitic

Schistosomiasis also known as snail

fever and bilharzia is a disease caused

by parasitic

flatworms called schistosomes

Q87 Prothrombin responsible for

clotting of blood is released by

परह८िह८ननबन

जह८रकतकािककाजमनहॳकहॳ लिएनजनमहॳदारहहॴ mdashndash

कहॳ दवारासतरापवतककयाजाताहहॴ

19Jan2017

Options

1) Small Intestine

छह८टीआत

2) Blood Platelets

रकतपिहॳटिहॳटस

3) Large Intestine

बड़ीआत

4Heart

हदय

Correct Answer Blood Platelets

Q88 Acacia arabica is the scientific

name of

अकहॳ लियाअरहॳबबका mdashmdashndash कावहॴजञाननकनामहहॴ 19-Jan-2017

Options

1) Neem

नीम

2) Teak

सागह९न

3) Babhul

बबि

4) Pomegranate

अनार

Correct Answer Babhul

Q89 Cannis Vulpes is the scientific

name of

कहॴ ननसवनमपस mdashmdash- कावहॴजञाननकनामहहॴ 19-Jan-2017

Options

1) Dog

कतता 2) Wolf

भहॳडड़या 3) Fox

िह८मड़ी 4) Hyena

िाकिबगघा

F A C E B O O K

P A G E h t t p w w w f a c e b o o k c o m s s c m e n t o r s o f f i c i a l P a g e | 22

FOR MORE UPDATES AND MORE MATERIAL DO LIKE OUR FACEBOOK PAGE httpwwwfacebookcomsscmentorsofficial

Correct Answer Fox

Q90 The beetroot is the portion of the

beet plant

चकदरपह९धहॳका mdashmdashndash भागहहॴ 19-Jan-2017

Options

1) tap root

मखयजड़

2) Adventitious

आकनसमक

3) bulb of the stem

तनहॳकाकद

4) Rhizome

परकद

Correct Answer tap root

Q91 What is the basic unit of heredity

आनवलिकताकीबननयादीइकाईकयाहहॴ 19-Jan-2017

Options

1) DNA

िीएनए

2) RNA

आरएनए

3) Chromosome

िह८मह८सह८म

4) Gene

जीन

Correct Answer gene

Genes are the units of heredity and are

the instructions that make up the bodyrsquos

blueprint They code for the proteins

that determine virtually all of a personrsquos

characteristics Most genes come in

pairs and are made of strands of genetic

material called deoxyribonucleic acid

or DNA

Q92 Lungs are the primary organs of

फहॳ फड़हॳmdashndashकहॳ परािलमकअगहहॴ

19-Jan-2017

Options

1) Digestion

पाचन

2) Constipation

कलज

3) Perspiration

पसीना 4)Respiration

शवसन

Correct Answer Respiration

Q93 Sugarcane is a type of

गननाएकपरकारका mdash- हहॴ 20-Jan-2017

Options

1)creeper

िता 2)tree

पहॳड़

3)shrub

झाड़ी 4)grass

घास

Correct Answer grass

Q94 Who is commonly known as ldquothe

Father of Microbiologyrdquo

सामानयत ldquo सकषमजीवपवजञानकहॳ जनक lsquo

कहॳ नामसहॳककसहॳजानाजातहहॴ 20-Jan-2017

Options

1) Robert Hooke

रॉबटयहक

2) Antonie Philips van Leeuwenhoek

एटह८नीकफलिपवानमयएनहह८क

3) Carl Linnaeus

काियिीनाईयस

4) Charles Darwin

चामसयिापवयन

F A C E B O O K

P A G E h t t p w w w f a c e b o o k c o m s s c m e n t o r s o f f i c i a l P a g e | 23

FOR MORE UPDATES AND MORE MATERIAL DO LIKE OUR FACEBOOK PAGE httpwwwfacebookcomsscmentorsofficial

Correct Answer Antonie Philips van

Leeuwenhoek

Q95 For the aquatic organisms the

source of food is

जिीयजीवाणकािाघसरह८तहहॴ 20-Jan-2017

Options

1) Phytoplankton

फायटह८पिहॳकटन

2) Sea Weed

समदरीिहॴवाि

3)Aqua plankton

एकवापिहॳकटन

4) Zooplankton

जपिहॳकटन

Correct Answer Phytoplankton

Q96 Haemoglobin has the highest

affinity with which of the following

हीमह८गिह८बबनकीननननमसहॳककसकहॳ सािउततमसमानताहहॴ

20-Jan-2017

Options

1)SO2

2)CO2

3)CO

4)NO2

Correct Answer CO

It has a greater affinity for hemoglobin

than oxygen does It displaces oxygen

and quickly binds so very little oxygen

is transported through the body cells

Q97 Who developed the theory of

Evolution

उदपवकासकालसदातककसनहॳपवकलसतककया

20-Jan-2017

Options

1) Charles Darwin

चामसयिापवयन

2) Isaac Newton

आयजहॳकनयटन

3) Pranav Mistry

परणवलमसतरी 4) Galileo Galilei

गहॳलिलियह८गहॳिीिी Correct Answer Charles Darwin

Q98 The primary function of RNA is

RNA कापरािलमककाययहह८ताहहॴ 20-Jan-2017

Options

1) Photosynthesis

परकािसशिहॳषण

2) Protein Synthesis

परह८टीनसशिहॳषण

3) Replication

परनतकनतबनाना 4) Translation

अनवादकरना Correct Answer Protein Synthesis

There are two main functions of RNA

It assists DNA by serving as a messenger

to relay the proper genetic information

to countless numbers of ribosomes in

your body The other main function of

RNA is to select the correct amino acid

needed by each ribosome to build new

proteins for your body

Q99 ______is the movement of

molecules across a cell membrane from

a region of their lower concentration to

a region of their higher concertration

उचचसादरताकहॳ कषहॳतरसहॳउसकीकमसादरतावािहॳकषहॳतरकीतरफएककह८लिकाखझमिीकहॳ माधयमसहॳहह८नहॳवािाअणओकहॳ सचिनकह८ mdash- कहतहॳहहॴ Options

1) Diffusion

पवसरण

2) Osmosis

ऑसमह८लसस

F A C E B O O K

P A G E h t t p w w w f a c e b o o k c o m s s c m e n t o r s o f f i c i a l P a g e | 24

FOR MORE UPDATES AND MORE MATERIAL DO LIKE OUR FACEBOOK PAGE httpwwwfacebookcomsscmentorsofficial

3) Active Transport

सकियआवागमन

4) Passive Transport

नननषियआवागमन

Correct Answer Active Transport

Q100 Study of classification of

organisms is known as 20-Jan-2017

जीवाणओकहॳ वगीकरणकहॳ अधययनकह८ mdash-

कहाजाताहहॴ Options

1) Serpentology

सपरहॳटह८िह८जी 2) Virology

वायरह८िह८जी 3) Taxonomy

टहॴकसोनह८मी 4) Physiology

कफनज़यह८िह८जी Correct Answer Taxonomy

Q101 Photosynthesis takes place inside

plant cells in

परकािसशिहॳषणवनसपनतकह८लिकामनसति mdash

mdashmdash महह८ताहहॴ 20-Jan-2017

Options

1) Ribosomes

राइबह८सह८नस

2) Chloroplasts

किह८रह८पिासट

3) Nucleus

नयकलियम

4) Mitochondria

माईटह८कोडडरया Correct Answer Chloroplasts

Q102 ______ is the cell organelle in

which the biochemical processes of

respiration and energy production

occur

mdashmdash- वहकह८लिकाअगहहॴ नजसमहॳशवसनऔरउजायउतपादनकहॳ जहॴसीजहॴवरासायननकपरकियायहह८तीहहॴ 20-Jan-2017

Options

1) Mitochondria

माइटह८कोडडरया 2) Chloroplast

किह८रह८पिासट

3) Ribosomes

राइबह८सह८नस

4) Nucleus

नयकिीयस

Correct Answer Mitochondria

Q103 Which non-flowering spore

bearing plants have roots

ककसफिनिगनहॳवािहॳऔरबीजाणधारकपह९धह८कीजड़हॳहह८तीहहॴ 21-Jan-2017

Options

1) Mosses

मह८सहॳस

2) Angiosperms

एननजयह८सपनसय 3) Ferns

फनसय 4) Gymnosperms

नजननह८सपनसय Correct Answer ferns

Q104 Which of the following is an

excretory organ of cockroach

नननननलिखितमसहॳकह९नसानतिच हॳकाउतसजयनअगहहॴ

21-Jan-2017

Options

F A C E B O O K

P A G E h t t p w w w f a c e b o o k c o m s s c m e n t o r s o f f i c i a l P a g e | 25

FOR MORE UPDATES AND MORE MATERIAL DO LIKE OUR FACEBOOK PAGE httpwwwfacebookcomsscmentorsofficial

1) Malphigian Tubules

मनमफनजयनटयबमस

2) Nephridia

नहॳकफरडिया 3) Coxal Gland

कह८कसिगरचिया 4) Green Gland

गरीनगरचिया Correct Answer Malphigian Tubules

Q105 Evaporation of water takes place

in which part of plants

पानीकहॳ वाषपीकरणकीकियापह९धोकहॳ ककसभागसहॳहह८तीहहॴ 21-Jan-2017

Options

1) Stem

तना 2) Stomata

सटह८मटा 3) Branch

िािाए

4) Fruit

फि

Correct Answer Stomata

Evaporation accounts for the movement

of water to the air from sources such as

the soil canopy interception and

waterbodies Transpiration accounts for

the movement of water within a plant

and the subsequent loss of water as

vapour through stomata in its leaves

Q106 A is the fleshy spore-bearing

fruiting body of a fungus

mdashmdashndashकवककामासि

बीजाणधारणकरनहॳवािाफिनहॳवािाअगहहॴ 21-

Jan-2017

Options

1) aloe vera

एिह८वहॳरा 2) Coral

मगा 3) Cactus

कहॴ कटस

4) Mushroom

ककरमतता Correct Answer mushroom

Q107 Which of the following is a fungal

disease

नननननलिखितमसहॳकह९नसाफफदसहॳहह८नहॳवािाएकरह८ग हहॴ

21-Jan-2017

Options

1) Dermatitis

तवचािह८ध

2) Cholera

हहॴजा 3) Jaundice

पीलिया 4) Indigofera

इननिगह८फहॳ रा Correct Answer Dermatitis

Dermatitis also known as eczema is a

group of diseases that results in

inflammation of the skin These diseases

are characterized by itchiness red skin

and a rash In cases of short duration

there may be small blisters while in

long-term cases the skin may become

thickened

Q108 In which form is glucose stored in

our body

हमारहॳिरीरमगिकह८जकासचयककस पमककयाजाताहहॴ

21-Jan-2017

Options

1) Insulin

F A C E B O O K

P A G E h t t p w w w f a c e b o o k c o m s s c m e n t o r s o f f i c i a l P a g e | 26

FOR MORE UPDATES AND MORE MATERIAL DO LIKE OUR FACEBOOK PAGE httpwwwfacebookcomsscmentorsofficial

इसलिन

2) Glucose

गिकह८ज

3) Glycogen

गिायकह८जहॳन

4) Fat

वसा Correct Answer Glycogen

Excess glucose is stored in the liver as

the large compound called glycogen

Glycogen is a polysaccharide of glucose

but its structure allows it to pack

compactly so more of it can be stored in

cells for later use

Q109 Where do plants synthesize

protein from

पह९धहॳपरह८टीनसशिहॳषणकहासहॳकरतहॳहहॴ

Options

1) Fatty Acids

वसाऐलसि

2) Sugar

िकर

3) Amino Acids

एलमनह८ऐलसि

4) Starch

सटाचय Correct Answer Amino Acids

Q110 Which part of the brain is

responsible for triggering actions like

thinking intelligence memory and

ability to learn

मनसतषककाकह९नसाटहससासह८चनहॳ बनधदमानी याददाशतऔरसीिनहॳकीकषमताजहॴसीकियाओकह८परहॳररतकरताहहॴ 21-Jan-2017

Options

1) Diencephalon

िायएनसहॳफहॳ िह८न

2) Hypothalamus

हयपह८िहॳिहॳमस

3) Cerebrum

सहॳरहॳिम

4) Control

कटरह८ि

Correct Answer Cerebrum

Q111 Which of the following is also

known as the Biochemical Laboratory

of the Human Body

नननननलिखितमसहॳककसहॳमानविरीरकीजहॴवरसायनपरयह८गिािाभीकहाजाताहहॴ 21-Jan-2017

Options

1) Small Intestine

छह८टीआत

2)Brain

मनसतषक

3) Pancreas

अगनयािय

4) Liver

नजगर

Correct Answer Liver

The liver makes bile that will help

emulsify and digest the fats we eat

The liver takes toxic substances and

convert them using enzymes the liver

cells makes into a non toxic form so the

body can dispose of them

The liver also converts fats protein and

carbohydrates into glucose which is the

energy source for our cells to use

The liver takes amino acids and makes

proteins by combining them

Q112 The yellow colour of human urine

is due to

मानवमतरकापीिारग mdashndash कीवजहसहॳहह८ताहहॴ 22-

Jan-2017

Options

1) Bile Salts

F A C E B O O K

P A G E h t t p w w w f a c e b o o k c o m s s c m e n t o r s o f f i c i a l P a g e | 27

FOR MORE UPDATES AND MORE MATERIAL DO LIKE OUR FACEBOOK PAGE httpwwwfacebookcomsscmentorsofficial

पपततनमक

2) Cholesterol

कह८िहॳसटरह८ि

3) Lymph

लिनफ

4) Urochrome

यरह८िह८म

Correct Answer Urochrome

Urobilin or urochrome is the chemical

primarily responsible for the yellow

color of urine

Q113 The wilting of plants takes place

due to

पह९धह८कालिचििहह८नाकी mdashmdash- कीवजहसहॳहह८ताहहॴ 22-Jan-2017

Options

1)Photosynthesis

परकािसशिहॳषण

2) Transpiration

वाषपह८तसजयन

3) Absorption

अविह८षण

4) Respiration

शरवसन

Correct Answer Transpiration

Wilting is the loss of rigidity of non-

woody parts of plants This occurs when

the turgor pressure in non-lignified

plant cells falls towards zero as a result

of diminished water in the cells

Q114 Bovidae Ovis is the scientific name of

बह८पविीओपवस mdashndash कावहॴजञाननकनामहहॴ 22-Jan-2017

Options

1) Goat

बकरी 2) Cow

गाय

3) Buffalo

भहॳस

4) Sheep

भहॳड़

Correct Answer Sheep

Q115 Plants get their energy to produce

food from which of the following

पह८धहॳभह८जनकाननमायणकरनहॳकहॳ लिएनननननलिखितमसहॳककससहॳउजायपरापतकरतहॳहहॴ

22-Jan-2017

Options

1) Photosynthesis

परकािसशिहॳषण

2)Bacteria

बहॴकटीररया 3)Fungi

कवक

4)Sun

सयय Correct Answer Sun

Q116 Which of the following is secreted

by the liver

नननननलिखितमसहॳककसकासरावनजगरसहॳहह८ताहहॴ

22-Jan-2017

Options

1) Glucose

गिकह८ज

2) Iodine

आयह८िीन

3) Cortisol

काटटरयसह८ि

4) Bile

पपतत

Correct Answer Bile

The liver makes bile that will help

emulsify and

digest the fats we eat

F A C E B O O K

P A G E h t t p w w w f a c e b o o k c o m s s c m e n t o r s o f f i c i a l P a g e | 28

FOR MORE UPDATES AND MORE MATERIAL DO LIKE OUR FACEBOOK PAGE httpwwwfacebookcomsscmentorsofficial

Q117 Ferns belong to which division of

plants

फनसयपह९धह८कहॳ ककसभागमआतहॳहहॴ

22-Jan-2017

Options

1) Gymnosperms

नजननह८सपनसय 2) Angiosperms

एनजयह८सपनसय 3) Thallophyta

िहॴिह८फाईटा 4)Pteridophyta

टहॳररिह८फाईटा Correct Answer Pteridophyta

Q118 Who invented Antibiotics

एटीबायह८टटककाअपवषकारककसनहॳककयािा

22-Jan-2017

Options

1) Joseph Lister

जह८सहॳफलिसटर

2) William Harvey

पवलियमहाव

3) Robert Knock

रॉबटयनॉक

4)Alexander Fleming

अिहॳकज़िरफिहॳलमग

Correct Answer Alexander Fleming

Q119 Milbecycin is used in the

eradication of

लममबहॳसायलसनका mdashndash

मउनमिनमपरयह८गककयाजाताहहॴ 22-Jan-2017

Options

1) Agricultural Fungus

कपषकवक

2) Agricultural Pests

कपषकीटक

3) Agricultural Herbs

कपषिाक

4)Agricultural Weeds

कपषननराना Correct Answer Agricultural Pests

Milbemycin oxime is a veterinary drug

from the group of milbemycins used as

a broad spectrum antiparasitic It is

active against worms and mites(insects

Q120 Intestinal bacteria synthesizes

which of the following in the human

body

मानविरीरमआतोकहॳ बहॴकटीररयानननननलिखितमसहॳककसकासशिहॳषणकरतहॳहहॴ 22-Jan-2017

Options

1) Vitamin K

पवटालमन K

2) Proteins

परह८टीन

3) Fats

वसा 4) Vitamin D

पवटालमन D

Correct Answer Vitamin K

Q121 is the study of the physical form

and external structure of plants

mdashmdash-

मपह९धह८काभहॴनतक पऔरबाहरीसरचनाकाआदयाककयाजाताहहॴ 22-Jan-2017

Options

1) Physiology

कफनजयह८िह८जी 2) Anatomy

िरीररचनापवजञान

3) Phytomorphology

फाईटह८मह८फह८िह८जी 4)Cytology

कह८लिकापवजञान

Correct Answer Phytomorphology

F A C E B O O K

P A G E h t t p w w w f a c e b o o k c o m s s c m e n t o r s o f f i c i a l P a g e | 29

FOR MORE UPDATES AND MORE MATERIAL DO LIKE OUR FACEBOOK PAGE httpwwwfacebookcomsscmentorsofficial

Q122 Which of the following is a

structural and functional unit of

kidneys

नननननलिखितमसहॳकह९नसीगदोकीसरचनातमकऔरकाययकरीईकाईहहॴ

22-Jan-2017

Options

1) Renette Cells

रहॳनहॳटकह८लिकाए

2) Flame Cells

फिहॳमकह८लिकाए

3) Nephrites

नहॳफ़राइटस

4)Nephrons

नहॳफरोस

Correct Answer Nephrons

Nephron functional unit of the kidney

the structure that actually produces

urine in the process of removing waste

and excess substances from the blood

There are about 1000000 nephrons in

each human kidney

Q123 Which of the following is the

largest part of the human brain

नननननलिखितमसहॳकह९नसामानवमनसतषककासबसहॳबड़ाटहससाहहॴ

23-Jan-2017

Options

1) Ribs

पसलियाा 2) Cerebrum

सहॳरहॳिम

3) Pons

पोस

4)Thalamus

िहॴिहॳमस

Correct Answer Cerebrum

The cerebrum is the largest part of the

human brain making up about two-

thirds of the brainrsquos mass It has two

hemispheres each of which has four

lobes frontal parietal temporal and

occipital

Q124 The auxiliary buds

सहायककालियाmdashndash 23-Jan-2017

Options

1) grow endogenously from the pericycle

पहॳरीसाईककिसहॳअनतजातयपवकलसतहह८ताहहॴ 2) arise endogenously from the main

growing point

मिवपदसहॳअनतजातयउठताहहॴ 3) is an embryonic shoot located in the

axil of a leaf

एकभरणिटहहॴजह८एकपततीकहॳ अकषपरनसतिहह८ताहहॴ 4)arise exogenously from the epidermis

एपपिलमयससहॳबटहजातयतरीकहॳ सहॳउठताहहॴ Correct Answer is an embryonic shoot

located in the axil of a leaf

Q125 Which of the following is a viral

disease

इनमहॳसहॳकह९सीएकवायरिबीमारीहहॴ

23-Jan-2017

Options

1) Polio

पह८लियह८ 2) Tetanus

धनसतनभ

3) Leprosy

कषठरह८ग

4) Plague

पिहॳग

Correct Answer Polio

A viral disease (or viral infection)

occurs when an organismrsquos body is

invaded by pathogenic viruses and

infectious virus particles (virions) attach

to and enter susceptible cells

F A C E B O O K

P A G E h t t p w w w f a c e b o o k c o m s s c m e n t o r s o f f i c i a l P a g e | 30

FOR MORE UPDATES AND MORE MATERIAL DO LIKE OUR FACEBOOK PAGE httpwwwfacebookcomsscmentorsofficial

Poliomyelitis often called polio or

infantile paralysis is an infectious

disease caused by the poliovirus

Tetanusmdash A serious bacterial infection

that causes painful muscle spasms and

can lead to death

Leprosy also known as Hansenrsquos

disease (HD) is a long-term infection by

the bacterium Mycobacterium leprae or

Mycobacterium lepromatosis

Plague is an infectious disease caused by

the bacterium Yersinia pestis

Symptoms include fever weakness and

headache

Q126 Which organisms can help to

carry out Vermicomposting

कह९नसाजीववमीकनपह८नसटगममददकरताहहॴ

23-Jan-2017

Options

1) Nitrifying Bacteria

नाईटरीफाईगबहॴकटीररया 2) Earthworms

कहॴ चऐ

3) Algae

िहॴवि

4) Fungus

कवक

Correct Answer Earthworms

Q127 Contraction of heart is also

known as

हदयकहॳ सकचनकह८ mdash- भीकहाजाताहहॴ 23-Jan-

2017

Options

1) Systole

लससटह८ि

2) Aristotle

अरसत

3) Diastole

िायसटह८ि

4) Lub

मयब

Correct Answer Systole

Diastole is the part of the cardiac cycle

when the heart refills with blood

following systole (contraction)

Ventricular diastole is the period during

which the ventricles are filling and

relaxing while atrial diastole is the

period during which the atria are

relaxing

Q128 Azadirachta indica is the

botanical name of which of the

following

अजाटदराचताइडिकानननननलिखितमसहॳककसकावानसपनतनामहहॴ

23-Jan-2017

Options

1) Rose plant

गिाबकापह९धा 2) Apple tree

सहॳबकापहॳड़

3) Neem

नीम

4)Mango

आम

Correct Answer Neem

Q129 Which of the following is the

main end product of carbohydrate

digestion

नननननलिखितमसहॳकह९नसाकाबोहाइडरहॳटकहॳ पाचनकापरमिअतउतपादकहह८ताहहॴ 23-Jan-2017

Options

1) Fats

वसा 2) Lipids

लिपपडस

3) Glucose

गिकह८ज

4) Cellulose

F A C E B O O K

P A G E h t t p w w w f a c e b o o k c o m s s c m e n t o r s o f f i c i a l P a g e | 31

FOR MORE UPDATES AND MORE MATERIAL DO LIKE OUR FACEBOOK PAGE httpwwwfacebookcomsscmentorsofficial

सहॳमयिह८ज

Correct Answer Glucose

Intestinal absorption of end products

from digestion of carbohydrates and

proteins in the pig hellip During absorption some sugars (fructose or

galactose) released from the

corresponding sucrose and lactose

respectively during digestion were

partly metabolized into glucose by the

enterocyte

Q130 Which of the following glands is a

source of the enzyme Ptyalin

नननननलिखितगरचियोमसहॳएजाइमटयालिनकासरह८तहहॴ 23-Jan-2017

Options

1) Pancreas

अगरािय

2) Thyroid Gland

िाइराइिगरिी 3) Pituitary Gland

पीयषगरिी 4) Salivary Glands

िारगरचियाा Correct Answer Salivary Glands

Q131 Which of the following is not true

about Pteridophyta

ननननमसहॳकह९नसीबातटहॳररिह८फाईटकहॳ बारहॳमसचनहीहहॴ 23-Jan-2017

Options

1) Dominant phase is saprophytes

परमिचरणसहॳपरह८फाईइटसहह८ताहहॴ 2) Main plant body is diploid

पह९दह८कामखयिरीरदपवगखणतहह८ताहहॴ 3) Seeds are present

बीजमह९जदहह८तहॳहहॴ 4)Flowers are absent

फिअनपनसतिहह८तहॳहहॴ

Correct Answer Seeds are present

Q132 The largest dolphin species is the

orca also called as

िॉिकफनकीसबसहॳबड़ीपरजानतकाकानामआकायहहॴनजसहॳ mdash- भीकहतहॳहहॴ 23-Jan-2017

Options

1) Bottle Nose

बाटिनह८ज

2) Baiji

बहॳजी 3) Killer whale

ककिरहहॳि

4)Tucuxi

टकवसी Correct Answer Killer whale

Q133 The fat digesting enzyme Lipase

is secreted by which of the following

वसाकापाचनकरनहॳवािाएजाइमिाइपहॳजनननननलिखितमसहॳककसकहॳ दवारासतरापवतहह८ताहहॴ

24-Jan-2017

Options

1) Kidneys

गद

2) Pancreas

अगनयािय

3) Large Intestine

बड़ीआत

4)Liver

नजगर

Correct Answer Pancreas

Lipase is an enzyme that splits fats so

the intestines can absorb them Lipase

hydrolyzes fats like triglycerides into

their component fatty acid and glycerol

molecules It is found in the blood

gastric juices pancreatic secretions

intestinal juices and adipose tissues

F A C E B O O K

P A G E h t t p w w w f a c e b o o k c o m s s c m e n t o r s o f f i c i a l P a g e | 32

FOR MORE UPDATES AND MORE MATERIAL DO LIKE OUR FACEBOOK PAGE httpwwwfacebookcomsscmentorsofficial

Q134 The arrangement of leaves on an

axis or stem is called

एकअकषयातनहॳपरपनततयोकीयवसिाकह८कयाकहाजाताहहॴ SSC CHSL Science (biology) 2016

Question Paper

24-Jan-2017

Options

1) Phyllotaxy

फाइिह८टहॴकसी 2) Vernation

वनिन

3) Venation

वहॳनहॳिन

4)Phytotaxy

फाइटह८टहॴकसी Correct Answer Phyllotaxy

In botany phyllotaxis or phyllotaxy is

the arrangement of leaves on a plant

stem (from Ancient Greek phyacutellon

ldquoleafrdquo and taacutexis ldquoarrangementrdquo)

Phyllotactic spirals form a distinctive

class of patterns in nature

Q135 The study of Cells is also known

as

कह८लिकाओकहॳ अधययनकह८ mdashmdashndash

भीकहाजाताहहॴ 24-Jan-2017

Options

1) Cytology

सायटह८िह८जी 2) Physiology

कफनजयह८िह८जी 3) Nucleology

नयककमयह८िह८जी 4)Cellology

सहॳिह८िह८जी Correct Answer Cytology

Q136 Which of the following scientists

is also known as the Father of Biology

नननननलिखितमसहॳककसवहॴजञाननककह८ ldquoजीवपवजञानकहॳ जनकrdquoकहॳ नामसहॳभीजानाजाताहहॴ 24-Jan-2017

Options

1) Herbert Spencer

हबयटयसपसर

2) Aristotle

अरसत 3) Lamarck

िहॳमाकय 4)Darwin

िापवयन

Correct Answer Aristotle

Q137 Which cells give rise to various

organs of the plant and keep the plant

growing

कह९नसीकह८लिकाएपह९धह८कहॳ लभननअगह८कह८जनमदहॳतीहहॴऔरपह९धह८कह८बढ़नहॳममददकरतीहहॴ

24-Jan-2017

Options

1) Permanent

सिायी 2) Dermal

तवचीय

3) Meristematic

मररसटहॳमटटक

4)Mature

परह८ढ़

Correct Answer Meristematic

A meristem is the tissue in most plants

containing undifferentiated cells

(meristematic cells) found in zones of

the plant where growth can take place

Q138 Rodentia Muridae is the scientific

name of

F A C E B O O K

P A G E h t t p w w w f a c e b o o k c o m s s c m e n t o r s o f f i c i a l P a g e | 33

FOR MORE UPDATES AND MORE MATERIAL DO LIKE OUR FACEBOOK PAGE httpwwwfacebookcomsscmentorsofficial

रह८िहॳलियानयररिी mdashmdash- कावहॴजञाननकनामहहॴ 24-

Jan-2017

Options

1) Mouse

चहा 2) Squirrel

चगिहरी 3) Monkey

बदर

4) Lizard

नछपकिी Correct Answer Mouse

Q139 Name the scientist who proposed

the cell theory

कह८लिकालसदातकापरसतावदहॳनहॳवािहॳवहॴजञाननककानामबताइए 24-Jan-2017

Options

1) Schleiden and Schwann

िीमिनऔरशरववान

2) Lamarck

िहॳमाकय 3) Treviranus

टरहॳवायरहॳनस

4)Whittaker and Stanley

हीटकरऔरसटहॳनिहॳ Correct Answer Schleiden and

Schwann

Q140 The flower with the worldrsquos

largest bloom is

दननयाकासबसहॳबड़ाफिखििनहॳवािा mdashmdashndash हहॴ 24-Jan-2017

Options

1) Pando

पािह८ 2) Posidonia

पह८सीिह८ननया 3) Rafflesia arnoldii

ररफिहॳलियाअनोमिी 4)Helianthus annuus

हहॳलिएनिसएनयअस

Correct Answer Rafflesia arnoldii

Rafflesia arnoldii is a species of

flowering plant in the parasitic genus

Rafflesia It is noted for producing the

largest individual flower on earth It has

a very strong and horrible odour of

decaying flesh earning it the nickname

ldquocorpse flower

Q141 Deficiency of which vitamin

causes night blindness

ककसपवटालमनकीकमीकहॳ कारणरतौधीहह८ताहहॴ 24-Jan-2017

Options

1) Vitamin K

पवटालमन K

2) Vitamin C

पवटालमन C

3) Vitamin B1

पवटालमन B1

4)Vitamin A

पवटालमन A

Correct Answer Vitamin A

Q142 Nongreen plants lack which of the

following

गहॴर-

हररतवनसपनतमनननननलिखितमसहॳककसकीकमीहह८तीहहॴ

24-Jan-2017

Options

1) Chlorophyll

किह८रह८कफि

2) Lycophyll

िायकह८कफि

3) Cyanophyll

F A C E B O O K

P A G E h t t p w w w f a c e b o o k c o m s s c m e n t o r s o f f i c i a l P a g e | 34

FOR MORE UPDATES AND MORE MATERIAL DO LIKE OUR FACEBOOK PAGE httpwwwfacebookcomsscmentorsofficial

सायनह८कफि

4)Phototropism

फह८टह८टरोपपजम

Correct Answer Chlorophyll

Q143 Organisms that use light to

prepare food are known as

जह८जीवपरकािकाउपयह८गकरभह८जनतहॴयारकरतहॳहहॴ उनह mdashmdash- कहॳ पमजानजाताहहॴ 24-Jan-2017

Options

1) Autotrophs

सवपह८षी 2) Heterotrophs

पवषमपह८षज

3) Omnivores

सवायहारी 4)Decomposers

पवघटनकरनहॳवािा Correct Answer Autotrophs

autotrophs often make their own food

by using sunlight carbon dioxide and

water to form sugars which they can use

for energy Some examples of

autotrophs include plants algae and

even some bacteria Autotrophs

(producer) are important because they

are a food source for heterotrophs

(consumers)

A heterotroph is an organism that

ingests or absorbs organic carbon

(rather than fix carbon from inorganic

sources such as carbon dioxide) in order

to be able to produce energy and

synthesize compounds to maintain its

life Ninety-five percent or more of all

types of living organisms are

heterotrophic including all animals and

fungi and some bacteria

Q144 Which of the following is a

primary function of haemoglobin

नननननलिखितमसहॳकह९नसाटहमह८गिह८बबनकाएकपरािलमककाययहहॴ

25-Jan-2017

Options

1) Utilization of energy

उजायकाउपयह८गकरना 2) Prevention of anaemia

रकतामपताहह८नहॳसहॳरह८कना 3) Destruction of bacteria

बहॴकटीररयाकापवनािकरना 4) To transport oxygen

ऑकसीजनकावहनकरना Correct Answer To transport oxygen

Q145 Vascular bundles are absent in

सवहनीबिि mdashmdash- मअनपनसतिरहतहॳहहॴ 25-Jan-2017

Options

1) Bryophyta

िायह८फाइटा 2) Pteridophyta

टहॳररिह८फाईटा 3) Gymnosperms

नजननह८सपमय 4) Angiosperms

एननजयह८सपहॳनसय Correct Answer Bryophyta

Q146 Sauria Lacertidae is the scientific

name of

सहॴररयािहॳसरटाईिी mdashmdashndash कावहॴजञाननकनामहहॴ 25-Jan-2017

Options

1) Crocodile

मगरमचछ

2) Hippopotamus

टहपपह८पह८टहॳमस

3) Lizard

नछपकिी 4) House fly

F A C E B O O K

P A G E h t t p w w w f a c e b o o k c o m s s c m e n t o r s o f f i c i a l P a g e | 35

FOR MORE UPDATES AND MORE MATERIAL DO LIKE OUR FACEBOOK PAGE httpwwwfacebookcomsscmentorsofficial

घरहॳिमकिी Correct Answer Lizard

Q147 Which type of pathogen causes

the water-borne disease SARS (Severe

Acute Respiratory Syndrome)

ककसपरकािकारह८गज़नकजिजननतबीमारीसासयकाकारणबनताहहॴ 25-Jan-2017

Options

1) Viral

वायरि

2) Parasitic

परजीवी 3) Protozoan

परह८टह८जअन

4) Bacterial

बहॴकटीररयि

Correct Answer Viral

Q148 Which of the following organs

produces the enzyme lipase

नननननलिखितमसहॳकह९नसाअगिायपहॳजएजाइमउतपननकरताहहॴ 25-Jan-2017

Options

1) Pancreas

अगनयािय

2) Large Intestine

बड़ीआत

3) Liver

नजगर

4) Small Intestine

छह८टीआत

Correct Answer Pancreas

Q149 A is a long internode forming the

basal part or the whole of a peduncle

एक mdashmdash- एकिबाइटरनह८िहहॴ जह८ननचिाटहससायासनपणयिठिबनताहहॴ 25-

Jan-2017

Options

1) Rhizome

परकद

2) Rachis

महॳ दि

3) floral axis

पषपअकष

4) Scape

भगदड़

Correct Answer scape

Q150 ndash Which of the following

organisms are considered to be both

Living and Non-living

नननननलिखितमसहॳकह९नसहॳजीवाणकह८जीपवतऔरअजीपवतमानाजाताहहॴ

25-Jan-2017

Options

1) Bacteria

बहॴकटीररया 2) Fungi

कवक

3) Algae

िहॴवाि

4)Virus

वायरस

Correct Answer Virus

They are considered to be living as they

possess a protein coat as a protective

covering DNA as the genetic material

etc

They are said to be non-living as they

can be crystallised and they survive for

billions of years They can tolerate high

temperatures freezing cold

temperatures ultra-violet radiations etc

Q151 Deficiency of fluorine causes

which of the following

फिह८ररनकीकमीकहॳ कारणनननननलिखितमसहॳकयाहह८ताहहॴ

F A C E B O O K

P A G E h t t p w w w f a c e b o o k c o m s s c m e n t o r s o f f i c i a l P a g e | 36

FOR MORE UPDATES AND MORE MATERIAL DO LIKE OUR FACEBOOK PAGE httpwwwfacebookcomsscmentorsofficial

27-Jan-2017

Options

1) Dental Caries

िटिकहॴ ररज

2) Scurvy

सकवरी 3) Anaemia

रकतामपता 4) Arthritis

गटठया Correct Answer Dental Caries

Q152 In a Punnett Square with the

cross AaBb x AaBb how many Aabb

genotypes would be created

पनहॳटसककायरमिह८स AaBb x AaBb कहॳ साि

ककतनहॳ Aabb जीनह८टाइपबनगहॳ 27-Jan-2017

Options

1) 1

2) 8

3) 2

4) 3

Correct Answer 2

Q153 Which of the following is the

Controlling Center of the Cell

नननननलिखित म सहॳ कह८लिकाका ननयतरण

क दर कह९न हहॴ

27-Jan-2017

Options

1) Nucleus

क दर

2) Plasma

पिाजमा 3) Lysosome

िायसह८सह८म

4) Chromosome

िह८मह८सह८म

Correct Answer Nucleus

The control centre of the cell is the

nucleus in eukaryotic cells The nucleus

contains genetic material in the form of

DNA

Q154 Myopia affects which of the

following organs

मायह८पपयानननननलिखितअगह८मसहॳककसहॳपरभापवतकरताहहॴ

25-Jan-2017

Options

1) Heart

हदय

2) Skin

तवचा 3) Eyes

आािहॳ 4)Mouth

मह

Correct Answer Eyes

Q155 Which of the following bears

flowers

नननननलिखितमसहॳकह९नफिधारणकरताहहॴ

25-Jan-2017

Options

1) Bryophyta

िायह८फाइटा 2) Pteridophyta

टहॳरीिह८फाईटा 3) Gymnosperms

नजननह८सपमय 4)Angiosperms

एननजयह८सपमय Correct Answer Angiosperms

Q156 Oxygenated blood flows out of the

heart through the

ऑकसीजनयकतरकत mdashmdashmdash

कहॳ माधयमसहॳहदयकहॳ बाहरबहताहहॴ 25-Jan-2017

F A C E B O O K

P A G E h t t p w w w f a c e b o o k c o m s s c m e n t o r s o f f i c i a l P a g e | 37

FOR MORE UPDATES AND MORE MATERIAL DO LIKE OUR FACEBOOK PAGE httpwwwfacebookcomsscmentorsofficial

Options

1) Aorta

महाधमनी 2) pulmonary artery

फहॳ फड़हॳकीधमनी 3) vena cava

वहॳनाकावा 4)Atrium

चह९क

Correct Answer aorta

Q157 Blood leaving the liver and

moving towards the

heart has a higher concentration of

नजगरसहॳननकिकरहदयकीतरफजानहॳवािहॳरकतम mdashmdashmdashmdash कीउचचसादरताहह८तीहहॴ 27-Jan-2017

Options

1) Lipids

लिपपडस

2) Urea

यररया 3) Bile Pigments

पपततकहॳ रगकरण

4) Carbon dioxide

काबयनिायऑकसाइि

Correct Answer Bile Pigments

Urea is nitrogen containing substance

which is produced in the liver in order

to deal with excess amino-acids in the

body As urea is produced it leaves the

liver in the blood stream and passes via

the circulatory system to all parts of the

body

Q158 Bulb is a modification of which

part of a plant

बमबएकपह९धहॳकहॳ ककसटहससहॳकाएक पातरणहह८ताहहॴ 27-Jan-2017

Options

1) The root

जड़

2) The stem

तना 3) The radicle

मिाकर

4)The fruit

फि

Correct Answer The stem

Q159 Which of the following carries

blood away from the heart to different

body parts

इनमहॳसहॳकह९नरकतकह८हदयसहॳिरीरकहॳ पवलभननअगह८तकिहॳजातीहहॴ

27-Jan-2017

Options

1) Arteries

धमननया 2) Nerves

तबतरहाए

3) Capillaries

कहॳ लिकाए

4)Veins

नसहॳ Correct Answer Arteries

Q160 The series of processes by which

nitrogen and its compounds are

interconverted in the environment and

in living organisms is called

27-Jan-2017

Options

1)Absorption of Nitrogen

2)Ammonification

3)Nitrogen Fixation

4)Nitrogen Cycle

Correct Answer Nitrogen Cycle

Ammonification or Mineralization is

performed by bacteria to convert

organic nitrogen to ammonia

F A C E B O O K

P A G E h t t p w w w f a c e b o o k c o m s s c m e n t o r s o f f i c i a l P a g e | 38

FOR MORE UPDATES AND MORE MATERIAL DO LIKE OUR FACEBOOK PAGE httpwwwfacebookcomsscmentorsofficial

Nitrification can then occur to convert

the ammonium to nitrite and nitrate

Nitrogen fixation is a process by which

nitrogen in the Earthrsquos atmosphere is

converted into ammonia (NH3) or other

molecules available to living organisms

Q161 BCG vaccine is given to protect

from which of the following

बीसीजीकाटटकानननननलिखितमसहॳककसकहॳ बचावकहॳ लिएटदयाजातहहॴ

27-Jan-2017

Options

1) Jaundice

पीलिया 2) Anaemia

रकतमपता 3) Tuberculosis

कषयरह८ग

4) Polio

पह८लियह८ Correct Answer Tuberculosis

Q162 Parallel venation is found in

समानतरवहॳनहॳिन mdashmdashmdash- मपायाजाताहहॴ 27-Jan-2017

Options

1) plants which are monocots

पह९धहॳजह८एकबीजपतरीहह८तहॳहहॴ 2) plants which have a dicot stem

वहॳपह९धहॳनजनकातनादपवदलियहह८ताहहॴ 3) plants with leaves similar to Tulsi

वहॳपह९धहॳनजनकीपनततयतिसीकीपनततयोकहॳ समानहह८तहॳहहॴ 4)plants with tap roots

टहॳप टवािहॳपह९धहॳ Correct Answer plants which are

monocots

Q163 The hardest part of the body is

िरीरकासबसहॳकठह८रभाग mdashndash हहॴ 27-Jan-2017

Options

1) Bones

हडडिय

2) Tooth Enamel

दातकहॳ इनहॳमि

3) Skull

िह८पड़ी 4) Spinal Cord

महॳ रजज

Correct Answer Tooth Enamel

Q164 Which type of pathogen causes

the waterborne disease E coli Infection

ककसपरकारकारह८गजननकजिजननतरह८गईकह८िाईसिमणकाकारणबनताहहॴ 27-Jan-2017

Options

1) Protozoan

परह८टह८जआ

2) Parasitic

परजीवी 3) Bacterial

बहॴकटीररयि

4)Viral

वायरि

Correct Answer Bacterial

Q165 The amount of blood filtered

together by both the kidneys in a 70 kg

adult male human in a minute is

70 की गरा वािहॳएकवयसकप षमएकलमनटमदह८नोगदकहॳदवाराएकसािचाबनीगयीरकतकीमातरहह८तीहहॴ 29-Jan-2017

Options

1) 1100 ml

1100 लमलि

2) 100 ml

F A C E B O O K

P A G E h t t p w w w f a c e b o o k c o m s s c m e n t o r s o f f i c i a l P a g e | 39

FOR MORE UPDATES AND MORE MATERIAL DO LIKE OUR FACEBOOK PAGE httpwwwfacebookcomsscmentorsofficial

100 लमलि

3) 1500 ml

1500 लमलि

4) 500 ml

500 लमलि

Correct Answer 1100 ml

Q166 Which feature of a plant helps to

distinguish a monocot from a dicot

पह९धहॳकीवहकह९नसीपविहॳषताहहॴजह८एकदपवदलियहॳऔरएकएकदिीयपह९धहॳसहॳभहॳदकरनहॳममददकरतीहहॴ 29-Jan-2017

Options

1) Pollination

परागम

2) Venation

वहॳनहॳिन

3) Vernation

वनिन

4) Aestivation

एसटीवहॳिहॳन

Correct Answer venation

Q167 The Mutation Theory was

proposed by

उतवररवतयनकालसदात mdashmdashndash

कहॳ दवरापरसतापवतककयाजाताहहॴ 29-Jan-2017

Options

1) Charles Lyell

चामसयलियहॳि

2) William Smith

पवलियमनसमि

3) Hugo De Vries

हयगह८िीराईस

4)Harrison Schmitt

हहॳरीसननसमट

Correct Answer Hugo De Vries

Q168 Which type of pathogen causes

the waterborne disease HepatitisA

ककसपरकारकहॳ रह८गजनकजिजननतरह८गहहॳपहॳटाइटटस-A काकारणबनताहहॴ

29-Jan-2017

Options

1) Parasitic

परजीवी 2) Viral

वायरि

3) Protozoan

परह८टह८जआ

4) Bacterial

बहॴकटीररयि

Correct Answer Viral

Q169 In a Punnett Square with the

cross AaBb x Aabb how many AaBb

genotypes would be created

पनहॳटसकवायरमिह८स AaBb x Aabb

कहॳ सािककतनहॳ AaBb जीनह८टाइपबनगहॳ 29-Jan-

2017

Options

1) 4

2) 1

3) 7

4) 6

Correct Answer 4

Q170 Arboreal Ateles is the scientific

name of

अिह८ररयिएटटलिस mdashmdashmdash कावहॴजञाननकनामहहॴ 29-Jan-2017

Options

1) Squirrel

चगिहरी 2) Sparrow

गह८रहॴया 3) Lizard

नछपकिी 4) Spider monkey

F A C E B O O K

P A G E h t t p w w w f a c e b o o k c o m s s c m e n t o r s o f f i c i a l P a g e | 40

FOR MORE UPDATES AND MORE MATERIAL DO LIKE OUR FACEBOOK PAGE httpwwwfacebookcomsscmentorsofficial

मकड़ीबदर

Correct Answer Spider monkey

Q171 Which type of pathogen causes

the waterborne disease Salmonellosis

ककसपरकारकारह८गाणजिजननतबीमारीसािमह८नहॳिह८लसज़काकारकहहॴ

29-Jan-2017

Options

1) Algal

िहॳवालियहॳ 2) Parasitic

परजीवी 3) Bacterial

बहॴकटीररयि

4)Viral

वायरि

Correct Answer Bacterial

An infection with salmonella bacteria

commonly caused by contaminated food

or water

Symptoms include diarrhoea fever

chills and abdominal pain

Q172 is a condition in which there is a

deficiency of red cells or of haemoglobin

in the blood

mdashmdash-

एकनसिनतहहॴनजसमहॳरकतमिािकह८लिकाओकीयाहीमह८गिह८बबनकीकमीहह८तीहहॴ 29-Jan-2017

Options

1) Albinism

एनमबननजम

2) Propyria

परह८पीररया 3) Anaemia

एनीलमया 4)Keloid disorder

कहॳ िह८इिडिसओिर

Correct Answer Anaemia

Q173 Ananas comosus is the scientific

name of

Options

अनानासकह८मह८सस mdashmdashmdashndash

कावहॴजञाननकनामहहॴ 29-Jan-2017

1) Custard Apple

सीताफि

2) Pineapple

पाइनएपपि

3) Bamboo

बास

4)Pomegranate

अनार

Correct Answer Pineapple

Q174 Which organ produces insulin

कह९नसाअगइनसलिनपहॴदाकरताहहॴ 29-Jan-

2017

Options

1) Liver

यकत

2) Thyroid gland

िायराइिगरिी 3) Spleen

पिीहा 4)Pancreas

अगरयिय

Correct Answer Pancreas

Q175 Which of the following disease is

not caused by water pollution

नननननलिखितमसहॳकह९नसारह८गपानीकहॳ परदषणकहॳकारणनहीहह८ता

29-Jan-2017

Options

1) Cholera

हहॴजा 2) Typhoid

F A C E B O O K

P A G E h t t p w w w f a c e b o o k c o m s s c m e n t o r s o f f i c i a l P a g e | 41

FOR MORE UPDATES AND MORE MATERIAL DO LIKE OUR FACEBOOK PAGE httpwwwfacebookcomsscmentorsofficial

टाइफाइि

3) Asthma

दमा 4)Diarrhoea

दसत

Correct Answer Asthma

Q176 Ocimum tenuiflorum is the

scientific name of

ओलिलममटहॳयईफिह८रमइसकावहॴजञाननकनाम mdash

ndash हहॴ 30-Jan-2017

Options

1) Neem

नीम

2) Mango

आम

3) Babul

बबि

4)Tulsi

तिसी Correct Answer Tulsi

Q177 Which gland secretes bile a

digestive fluid

कह९नसीगरिीपपतत एकपाचनतरिपरदािय सरापवतकरतीहहॴ 30-Jan-2017

Options

1) Pancreas

अगनयािय

2) Liver

यकत

3) Thyroid

िायराइि

4) Testes

टहॳनसटस

Correct Answer liver

Q178 In which of the following the

dominant phase is Gametophyte

नननननलिखितमसहॳककसकहॳ परमिचरणयगमकह८दपवधद (Gametophyte)हहॴ 30-Jan-2017

Options

1) Bryophyta

िायह८फाइटा 2) Pteridophyta

टहॳररिह८फाइटा 3) Gymnosperms

नजननह८सपमय 4) Angiosperms

एननजयह८सपमय Correct Answer Bryophyta

Q179 Anaerobic respiration refers to

which of the following

नननननलिखितमसहॳककसहॳअवायवीयशवसनकहाजाताहहॴ

30-Jan-2017

Options

1) Respiration without Oxygen

ऑकसीजनकहॳ बबनाशवसन

2) Respiration with Oxygen

ऑकसीजनकहॳ सािशवसन

3) Respiration without CO2

काबयनिायऑकसाइिकहॳ बबनाशवसन

4) Respiration with CO2

काबयनिायऑकसाइिकहॳ सािशविन

Correct Answer Respiration without

Oxygen

Q180 Which type of pathogen causes

the waterborne disease Cholera

ककसपरकारकारह८गजनकजिजननतरह८गहहॴजाकाकारणबनताहहॴ

30-Jan-2017

Options

1) Algal

िहॴवालियहॳ

F A C E B O O K

P A G E h t t p w w w f a c e b o o k c o m s s c m e n t o r s o f f i c i a l P a g e | 42

FOR MORE UPDATES AND MORE MATERIAL DO LIKE OUR FACEBOOK PAGE httpwwwfacebookcomsscmentorsofficial

2) Bacterial

बहॴकटीररयि

3) Protozoan

परह८टह८जआ

4) Viral

वायरि

Correct Answer Bacterial

Q181 To which class does

Oxyreductases transferases hydrolases

belong

ओकसीररिकटहॳसटरासफरहॳजहॳस

हाइडरह८िहॳसहॳसककसवगयमआतहॳहहॴ 30-Jan-2017

Options

1) Hormones

हारमोस

2) Enzymes

एजाइनस

3) Proteins

परह८टीनस

4) Vitamins

पवटालमनस

Correct Answer Enzymes

Q182 Which of the following is not true

about Gymnosperms

ननननमसहॳकह९नसीबातअनावतबीजीकहॳ बारहॳमसचनहीहहॴ 30-Jan-2017

Options

1) Dominant phase is saprophytes

परमिचरणसहॳपरह८फाइटसहह८ताहहॴ 2) Vascular bundles are absent

सवहनीबििअनपनसितहह८ताहहॴ 3) spores are heterospores

बीजाणहहॳटहॳरह८सपह८रसहह८तहॳहहॴ 4) Flowers are absent

फिअनपनसितहह८तहॳहहॴ

Correct Answer Vascular bundles are

absent

Q183 The name of first mammal clone sheep is

भहॳड़कीपरिमसतनपायीपरनत प (किह८न)

कानामहहॴ 30-Jan-2017

Options

1) Noori

नरी 2) Dolly

िॉिी 3) Louise

िसी 4)Durga

दगाय Correct Answer Dolly

Q184 Which type of pathogen causes

the water-borne disease Typhoid fever

ककसपरकारकारह८गजनकजिजननतरह८गटाइफाइिबिारकाकारणबनताहहॴ 30-Jan-2017

Options

1) Algal

िहॴवािीय

2) Parasitic

परजीवी 3) Protozoan

परह८टह८जनअन

4)Bacterial

बहॴकटीररयि

Correct Answer Bacterial

Q185 In which part of the cell are

proteins made

कह८लिकाकहॳ ककसटहससहॳमपरह८टीनबनायाजाताहहॴ

31-Jan-2017

Options

1) Reticulum

रहॳटटकिम

F A C E B O O K

P A G E h t t p w w w f a c e b o o k c o m s s c m e n t o r s o f f i c i a l P a g e | 43

FOR MORE UPDATES AND MORE MATERIAL DO LIKE OUR FACEBOOK PAGE httpwwwfacebookcomsscmentorsofficial

2) Golgi apparatus

गह८मजीएपहॳरहॳटस

3) Ribosomes

ररबह८सह८नस

4) Lysosome

िायसह८सह८नस

Correct Answer ribosomes

Proteins are produced by stringing

amino acids together in the order

specified by messenger RNA strands

that were transcribed from DNA in the

cell nucleus The process of synthesizing

a protein is called translation and it

occurs on ribosomes in the cytoplasm of

a cell

Q186 Polio is a disease caused by which

of the following

नननननलिखितमसहॳपह८लियह८कीबबमारह८हह८नहॳकाकारणकयाहहॴ

31-Jan-2017

Options

1) Bacteria

बहॴकटीररयि

2) Mosquito

मचछर

3) Virus

वायरस

4) Cockroach

नतिच हॳ Correct Answer Virus

Polio or poliomyelitis is a crippling and

potentially deadly infectious disease It

is caused by the poliovirus

Q187 ndash Hay fever is a sign of which of

the following

हहॳकफवरनननननलिखितमसहॳककसकाएकसकहॳ तहहॴ

31-Jan-2017

Options

1) Old Age

वदावसिा 2) Malnutrition

कपह८सण

3) Allergy

एिनजय 4) Over Work

अतयचधककाययकरना Correct Answer Allergy

Q188 How many chromosomes does a

human cell contain

एकमानवकह८लिकामककतनहॳगणसतरहह८तहॳहहॴ

29-Jan-2017

Options

1) 6

2) 26

3) 46

4) 66

Correct Answer 46

In humans each cell normally contains

23 pairs of chromosomes for a total of

46 Twenty-two of these pairs called

autosomes look the same in both males

and females The 23rd pair the sex

chromosomes differ between males and

females

Q189 Which of the following is not true

about Bryophyta

ननननमसहॳकह९नसीबातिायह८फाइटकहॳ बारहॳमसचनहीहहॴ 31-Jan-2017

Options

1) Dominant phase is gametophytes

परमिचरणगहॳलमतह८फाइटसहह८ताहहॴ 2) Main plant body is haploid

पह९धहॳकामखयिरीरअगखणतहह८ताहहॴ 3) Spores are homospores

बीजाणहह८मह८सफह८रसहह८तहॳहहॴ 4) Flowers are present

फिमह८जदहह८तहॳहहॴ Correct Answer Flowers are present

F A C E B O O K

P A G E h t t p w w w f a c e b o o k c o m s s c m e n t o r s o f f i c i a l P a g e | 44

FOR MORE UPDATES AND MORE MATERIAL DO LIKE OUR FACEBOOK PAGE httpwwwfacebookcomsscmentorsofficial

Q190 Which aquatic animal has

trailing tentacles

ककसजिीयजानवरकहॳ पीछहॳचिनहॳवािहॳटहॳटकिसहह८तहॳहहॴ

31-Jan-2017

Options

1) Sea horse

समदरीघह८िा 2) Corals

मगा 3) Jelly fish

जहॳिीमछिी 4) Star fish

तारामछिी Correct Answer Jelly fish

Jellyfish with its umbrella-shaped bell

and trailing tentacles

Q191 Which type of pathogen causes

the water-borne disease Poliomyelitis

(Polio)

ककसपरकारकारह८गजनकजिजननतरह८गपह८लियह८मायहॳटटस (पह८लियह८) काकारणहहॴ 31-Jan-

2017

Options

1) Parasitic

परजीवी 2) Algal

िहॴवालिय

3) Viral

वायरि

4) Bacterial

बहॴकटीररयि

Correct Answer Viral

Q192 The outer white part of the eye

that protects the inner structures is

आािकाबाहरीसफहॳ दटहससाजह८आतररकसरचनाओकीरकषाकरताहहॴ वह mdashmdashmdash हहॴ 31-Jan-

2017

Options

1) Iris

आयररस

2) Sclera

सकिहॳरा 3) Retina

रहॳटटना 4) Cornea

कह८ननयया Correct Answer Sclera

Q193 Proteins are made up of

परह८टीनकाननमायण mdashndash सहॳहह८ताहहॴ 31-Jan-2017

Options

1) Amino acids

एलमनह८अनि

2) Fatty acids

वसायकतअनि

3) Glucose

गिकह८ज

4)Nucleotides

नयनकियह८टाईिस

Correct Answer Amino acids

Q194 Moringa Oleifera is the scientific

name of

मह८ररगओलिफहॳ रा mdashmdashndash कावहॴजञाननकनामहहॴ 31-Jan-2017

Options

1) Banyan

बरगद

2) Gulmohar

गिमह८हर

3) Amla

आमिा

F A C E B O O K

P A G E h t t p w w w f a c e b o o k c o m s s c m e n t o r s o f f i c i a l P a g e | 45

FOR MORE UPDATES AND MORE MATERIAL DO LIKE OUR FACEBOOK PAGE httpwwwfacebookcomsscmentorsofficial

4) Drumstick

डरमनसटक

Correct Answer Drumstick

Q195 Kidney stones are composed of

गदकीपिरी mdashndash सहॳबनीहह८तीहहॴ 1-Feb-2017

Options

1) Calcium Oxalate

कहॴ नमसयमओकजहॳिहॳट

2) Sodium Chloride

सह८डियमकिह८राइि

3) Magnesium Nitrate

महॳनगनलियमनाइतटरहॳट

4) Calcium Bicarbonate

कहॴ नमियमबायकबोनहॳट

Correct Answer Calcium Oxalate

Q196 ndash Which of the following is not

true about Angiosperms

ननननमसहॳकह९नसीबातआवतबीजीकहॳ बारहॳमसचनहीहहॴ 1-Feb-2017

Options

1) Dominant phase is gametophytes

परमिचरणगहॳलमतह८फाइटहह८ताहहॴ 2) Vascular bundles are present

सवहनीबििमह९जदहह८ताहहॴ 3) Spores are heterospores

बीजाणहहॳटहॳरह८सपह८रसहह८तहॳहहॴ 4) Seeds are covered

बीजढकहॳ हह८तहॳहहॴ Correct Answer Dominant phase is

gametophytes

Q197 All of the following are excretory

(waste) products of animals except

नननननलिखितमसहॳककसएककह८छह८ड़करअनयसभीपराखणयोदवाराउतसनजयतपदाियहहॴ 1-Feb-

2017

Options

1) Uric Acid

यररकएलसि

2) Ammonia

अमह८ननया 3) Carbohydrates

काबोहाइडरहॳट

4) Urea

यररया Correct Answer Carbohydrates

In animals the main excretory products

are carbon dioxide ammonia (in

ammoniotelics) urea (in ureotelics) uric

acid (in uricotelics) guanine (in

Arachnida) and creatine

Q198 RNA is a polymeric molecule

What does RNA stand for

आरएनइएएकबहिकआणहहॴ इसकाकापवय पकयाहहॴ 1-Feb-2017

Options

1) Rado Nuclear Acid

रािह८नयनकियरएलसि

2) Ribo Nucleic Acid

राइबह८नयनकिकएलसि

3) Rhino Nuclear Acid

हाइनह८नयनकियरएलसि

4) Resto Nucleus Acid

रहॳसटह८नयकिीयसएलसि

Correct Answer Ribo Nucleic Acid

Q199 Which organ does detoxification

and produces chemicals needed for

digestion

कह९नसाअगपवषहरणकरताहहॴऔरपाचनकहॳ लिएआवशयकरसायनोकह८पहॴदाकरताहहॴ 1-Feb-

2017

Options

1) Salivary glands

िारगरचिया 2) Pancreas

अगनयािय

F A C E B O O K

P A G E h t t p w w w f a c e b o o k c o m s s c m e n t o r s o f f i c i a l P a g e | 46

FOR MORE UPDATES AND MORE MATERIAL DO LIKE OUR FACEBOOK PAGE httpwwwfacebookcomsscmentorsofficial

3) Thyroid gland

िायराइिगरिी 4) Liver

यकत

Correct Answer Liver

Q200 Psidium guajava is the scientific

name of

लसडियमगआजावा mdashmdash कावहॴजञाननकनामहहॴ 1-

Feb-2017

Options

1) Guava

अम द

2) Mango

आम

3) Bamboo

बास

4) Jack fruit

कटहि

Correct Answer Guava

Q201 Which drug is used as a Blood

Thinner

चधरकह८पतिाकरनहॳकहॳ पमककसदवाकापरयह८गककयाजाताहहॴ

1-Feb-2017

Options

1) Warfarin

वाफर न

2) Tramadol

टरहॳमािह८ि

3) Azithromycin

एनजरह८मायलसन

4) Hydralazine

हाइडरह८िहॳनजन

Correct Answer Warfarin

Q202 Which of the following disease is

caused due to the deficiency of protein

परह८टीनकीकमीकहॳ कारणनननननलिखितमसहॳकह९नसारह८गहह८ताहहॴ 1-Feb-2017

Options

1) Arthritis

गटठया 2) Kwashiorkor

कािीओकय र

3) Goitre

गाइटर

4) Night Blindness

रतह९चध

Correct Answer Kwashiorkor

Q203 A is species of plant that has

adapted to survive in an environment

with little liquid water

mdashmdashndashपह९धहॳकीएकऐसहॳऐसहॳपरजानतहहॴ नजसनहॳकमपानीवािहॳवातावरणमजीपवतरहनहॳकहॳलिएअनकिनहहॴ 1-Feb-2017

Options

1) Xerophyte

म दपवद

2) Hydrophyte

जिीयपादप

3) Mesophyte

समह८दपवद

4) Thallophyte

िहॴिह८फाइटा Correct Answer xerophyte

xerophyte is a species of plant that has

adapted to survive in an environment

with little liquid water such as a desert

or an ice- or snow-covered region in the

Alps or the Arctic

Mesophytes are terrestrial plants which

are adapted to neither a particularly

dry nor particularly wet environment

An example of a mesophytic habitat

would be a rural temperate meadow

F A C E B O O K

P A G E h t t p w w w f a c e b o o k c o m s s c m e n t o r s o f f i c i a l P a g e | 47

FOR MORE UPDATES AND MORE MATERIAL DO LIKE OUR FACEBOOK PAGE httpwwwfacebookcomsscmentorsofficial

which might contain goldenrod clover

oxeye daisy and Rosa multiflora

thallophyte any of a group of plants or

plantlike organisms (such as algae and

fungi) that lack differentiated stems

leaves and roots and that were formerly

classified as a primary division

(Thallophyta) of the plant kingdom

Q204 How many types of teeth are

there in humans

मनषयोमककतनहॳपरकारकहॳ दातहह८तहॳहहॴ

1-Feb-2017

Options

1) 4

2) 5

3) 2

4) 3

Correct Answer 4

teeth -Humans have four types of

teethincisors canines premolars and

molars each with a specific function

The incisors cut the food the canines

tear the food and the molars and

premolars crush the food

Q205 Carica papaya is the scientific name of

कहॴ ररकापपाया mdashmdashndash कावहॴजञाननकनामहहॴ 2-

Feb-2017

Options

1) Peepal

पीपि

2) Papaya

पपीता 3) Tamarind

इमिी 4) Drumstick

ढह८िकाछड़ी Correct Answer Papaya

Q206 Muscles get tired when there is

shortfall of

जब mdashndash कीकमीहह८तीहहॴतबपहॳिीयिकजातीहहॴ 2-Feb-2017

Options

1) Lactic acid

िहॴनकटकएलसि

2) Na+ ions

Na+ आयन

3) ATP

एटीपी 4) Sulphates

समफहॳ टस

Correct Answer ATP

ATP is the energy source muscle fibers

use to make muscles contract

muscle tissuersquos main source of energy

called adenosine triphosphate or ATP

As your muscles use up this energy

source they become tired and fatigued

Oxygen is the key ingredient that helps

create new ATP to replenish the burned

up ATP in your muscles

Q207 Artocarpus integra is the

scientific name of आटह८कापयसइटीगरा mdashmdashmdash कावहॴजञाननकनामहहॴ 2-Feb-2017

Options

1) Guava

अम द

2) Pineapple

अनानास

3) Silver Oak

लसमवरओक

4) Jack fruit

कटहि

Correct Answer Jack fruit

Q208 Which organ stores fat soluble

vitamins

कह९नसाअगवसामघिनिीिपवटालमनह८काभिाराकरताहहॴ

2-Feb-2017

F A C E B O O K

P A G E h t t p w w w f a c e b o o k c o m s s c m e n t o r s o f f i c i a l P a g e | 48

FOR MORE UPDATES AND MORE MATERIAL DO LIKE OUR FACEBOOK PAGE httpwwwfacebookcomsscmentorsofficial

Options

1) Blood

रकत

2) Skin

तवचा 3) Liver

यकत

4) Pancreas

अगनयािय

Correct Answer Liver

Q209 Which disease is caused due to

deficiency of Iodine

आयह८िीनकहॳ कारणकह९नसारह८गहह८ताहहॴ 2-Feb-2017

Options

1) Rickets

ररकहॳ टस

2) Scurvy

सकवी 3) Goitre

गणमािा 4) Growth retardation

पवकासका कना Correct Answer Goitre

rickets A softening and weakening of

bones in children usually due to

inadequate vitamin D

Q210 Grevillea Robusta is the scientific name of

गरहॳपवलियारह८बसटा mdashmdashmdash- कापवजञाननकनामहहॴ 2-Feb-2017

Options

1) Peepal

पीपि

2) Teak

सागह९न

3) Silver Oak

लसमवरओक

4) Jack fruit

कटहि

Correct Answer Silver Oak

Q211 When a Cuttlefish is described as a Molluscs it is at which level of

classification

जबएककटिकफिकह८एकमह८िसकाकहॳ पमवखणयतककयाजाताहहॴतबयहॳवगीकरणकहॳ ककससतरपहॳनसितहहॴ 2-Feb-2017

Options

1) Class

वगय 2) Order

िम

3) Family

पररवार

4) Phylum

सघ

Correct Answer Phylum

Q212 Bambusa dendrocalmus is the

scientific name of बानबसािहॳडराकामस mdashmdashmdash कावहॴजञाननकनामहहॴ 3-Feb-2017

Options

1) Banyan

बरगद

2) Papaya

पपीता 3) Bamboo

बास

4) Pomegranate

अनार

Correct Answer Bamboo

Q213 Acinonyx Jubatus is the scientific name of

एलसनह८ननकसजयबहॳटस mdashmdashmdash

कावहॴजञाननकनामहहॴ 3-Feb-2017

F A C E B O O K

P A G E h t t p w w w f a c e b o o k c o m s s c m e n t o r s o f f i c i a l P a g e | 49

FOR MORE UPDATES AND MORE MATERIAL DO LIKE OUR FACEBOOK PAGE httpwwwfacebookcomsscmentorsofficial

Options

1) Bear

भाि 2) Horse

घह८िा 3) Cheetah

चीता 4) Zebra

जहॳिा Correct Answer Cheetah

Q214 The pale yellow colour of urine is

due to the presence of which pigment

मतरकाफीकापीिारगरगदरयकहॳ उपनसिनतकहॳ कारणहह८ताहहॴ

3-Feb-2017

Options

1) Urochrome

यरह८िह८म

2) Urophyll

यरह८कफि

3) Chlorophyll

किह८रह८कफि

4) Chloroplast

किह८रह८पिासट

Correct Answer Urochrome

Q215 Which of the following constitute

to form a gene

नननननलिखितमसहॳकह९नसीचीज़एकजीनकागठनकरतीहहॴ

3-Feb-2017

Options

1) Polynucleotides

पह८िीनयनकियह८टाईडस

2) Hydrocarbons

हाइडरह८काबोस

3) Lipoproteins

िाईपह८परह८टीनस

4) Lipids

लिपपडस

Correct Answer Polynucleotides

Polynucleotide molecule is a biopolymer

composed of 13 or more nucleotide

monomers covalently bonded in a chain

DNA (deoxyribonucleic acid) and RNA

(ribonucleic acid) are examples of

polynucleotides with distinct biological

function

Q216 Vertebrates belongs to the

phylum

रीढ़कीहडिीवािहॳपराणी mdashmdashmdash

परजानतकहॳ अतगायतआतहॳहहॴ 3-Feb-2017

Options

1) Arthropoda

आरह८पह८ड़ा 2) Annelida

एननलििा 3) Cnidaria

ननिहॳररया 4) Chordata

कह८िटा Correct Answer Chordata

Q217 Punica granatum is the scientific name of

पननकगरहॳनहॳटस mdashmdashmdash कावहॴजञाननकनामहहॴ 3-Feb-2017

Options

1) Custard Apple

सीताफि

2) Gulmohar

गिमह८हर

3) Silver Oak

लसमवरओक

4) Pomegranate

अनार

Correct Answer Pomegranate

F A C E B O O K

P A G E h t t p w w w f a c e b o o k c o m s s c m e n t o r s o f f i c i a l P a g e | 50

FOR MORE UPDATES AND MORE MATERIAL DO LIKE OUR FACEBOOK PAGE httpwwwfacebookcomsscmentorsofficial

Q218 Between a tiger and an monkey

which of the following is different

एकबाघऔरबदरकहॳ बीचनननननलिखितमसहॳकह९नसीबातअिगहहॴ 3-Feb-2017

Options

1) Kingdom

राजय

2) Phylum

जानत

3) Order

िम

4) Class

वगय Correct Answer order

Q219 The artificial heart was invented by

कबतरमहदयका mdashmdashmdash

दवाराअपवषकारककयागयािा 3-Feb-2017

Options

1) Muhammad Yunus

महनमदयनस

2) Linus Yale Jr

िाइनसयहॳिजय

3) Gazi Yasargil

गाजीयासचगयि

4) Paul Winchell

पह९िपवमकि Correct Answer Paul Winchell

Q220 Tamarindus indica is the

scientific name of

टहॳमररनडसइडिका mdashmdash कावहॴजञाननकनामहहॴ 7-

Feb-2017

Options

1) Neem

नीम

2) Pineapple

अनानास

3) Tamarind

इमिी 4)Chiku

चीक

Correct Answer Tamarind

Q221 In eukaryotic cells synthesis of

RNA takes place in the

यकहॳ योटटककह८लिकाओमआरएनएकासशिहॳषण

mdashndash महह८ताहहॴ 7-Feb-2017

Options

1) Mitochondria

माईटह८कोडडरया 2) Centrioles

सटरीयह८मस

3) Ribosomes

ररबह८सह८नस

4) Nucleus

नयनकियस

Correct Answer nucleus

eukaryotic cell -Transcription is the

process of synthesizing ribonucleic acid

(RNA)Synthesis takes place within the

nucleus of eukaryotic cells or in the

cytoplasm of prokaryotes and converts

the genetic code from a gene in

deoxyribonucleic acid ( DNA ) to a

strand of RNA that then directs

proteinsynthesis

Q222 _________is caused by parasites

of the Plasmodium genus

पिाजमह८डियमजातीकहॳ परजीवी mdash- कहॳ कारणहहॴ 7-Feb-2017

Options

1) Dysentery

पहॳचचि

2) Malaria

मिहॳररया 3) Chickenpox

F A C E B O O K

P A G E h t t p w w w f a c e b o o k c o m s s c m e n t o r s o f f i c i a l P a g e | 51

FOR MORE UPDATES AND MORE MATERIAL DO LIKE OUR FACEBOOK PAGE httpwwwfacebookcomsscmentorsofficial

चहॳचक

4) Herpes

हहॳपपयस

Correct Answer Malaria

Q223 Carotene in fruits and vegetables

gives it which color

फिह८औरसनलजयोमनसितकहॳ रह८टीनउनहकह९नसारगपरदानकरताहहॴ 7-Feb-2017

Options

1) Green

हरा 2) Pink

गिाबी 3) Orange

नारगी 4) Blue

नीिा Correct Answer Orange

Q224 Equus Caballus is the scientific

name of

एकवसकहॴ बहॳिस mdashmdashndash कापवजञाननकनामहहॴ 7-Feb-2017

Options

1) Horse

घह८िा 2) Zebra

ज़हॳिा 3) Donkey

गधा 4) Buffalo

भस

Correct Answer Horse

Q225 Elapidae Naja is the scientific name of

एिीपीिीनाजा mdashmdash- कावहॴजञाननकनामहहॴ 8-Feb-2017

Options

1) Cobra

कह८बरा 2) Elephant

हािी 3) Eagle

ग ि

4) Owl

उमि Correct Answer Cobra

Q226 Which disease is caused due to

deficiency of Iron

िह८हकीकमीकहॳ कारणकह९नसारह८गहह८ताहहॴ 8-Feb-

2017

Options

1) Beriberi

बहॳरीबहॳरी 2) Tetany

टहॳटनी 3) Kwashiorkor

कवािीऔरकर

4) Anaemia

रकतामपता Correct Answer Anaemia

Beriberi is a disease caused by a vitamin

B-1 deficiency also known as thiamine

deficiency

Tetany can be the result of an

electrolyte imbalance Most often itrsquos a

dramatically low calcium level also

known as hypocalcemia Tetany can also

be caused by magnesium deficiency or

too little potassium Having too much

acid (acidosis) or too much alkali

(alkalosis) in the body can also result in

tetany

Kwashiorkor also known as

ldquoedematous malnutrition It is a form of

malnutrition caused by a lack of protein

in the diet

Anaemia means that you have fewer red

blood cells than normal or you have less

F A C E B O O K

P A G E h t t p w w w f a c e b o o k c o m s s c m e n t o r s o f f i c i a l P a g e | 52

FOR MORE UPDATES AND MORE MATERIAL DO LIKE OUR FACEBOOK PAGE httpwwwfacebookcomsscmentorsofficial

haemoglobin than normal in each red

blood cell

Q227 is a leaf where the leaflets are

arranged along the middle vein

mdashndashएकपततीहहॴजहापतरकह८कीरचनाक ररयालिराकहॳ आसपासहह८तीहहॴ 8-Feb-2017

Options

1) Pinnately compound leaf

पपनहॳटिीसयकतपतती 2) Palmately compound leaf

पामहॳटिीसयकतपतती 3) Compound leaf

सयकतपतती 4) Simple leaf

साधारणपतती Correct Answer Pinnately compound

leaf

Q228 Haustoria or sucking roots are

found in which of the following

हह८सटह८ररयायाचसनहॳवािीजड़हॳनननननलिखितमसहॳककसमपाईजातीहहॴ 8-Feb-2017

Options

1) Wheat

गहॳह

2) Mango

आम

3) Chestnut

चहॳसटनट

4) Cuscuta

कसकयटा Correct Answer Cuscuta

Haustorial roots -The roots of parasitic

plants which penetrate into the host

tissues to absorb nourishment are

called haustorial roots hellip Also known as suckingor parasitic roots

Q229 Equs Asinus is the scientific name

of

एकवसएलसनस mdashmdashndash कावहॴजञाननकनामहहॴ 8-

Feb-2017

Options

1) Donkey

गधा 2) Cow

गाय

3) Deer

टहरन

4) Kangaroo

कगा

Correct Answer Donkey

Q230 Ficus benghalensis is the scientific name of

फाईकसबहॳनगहॳिहॳलसस mdashndash कापवजञाननकनामहहॴ 8-Feb-2017

Options

1) Banyan

बरगद

2) Pineapple

अनानास

3) Babul

बबि

4) Tulsi

तिसी Correct Answer Banyan

Q231 Equus burchellii is the scientific name of

एकवसबचिी mdashmdash- कापवजञाननकनामहहॴ 8-Feb-2017

Options

1) Horse

घह८िा 2) Zebra

जहॳिा 3) Buffalo

F A C E B O O K

P A G E h t t p w w w f a c e b o o k c o m s s c m e n t o r s o f f i c i a l P a g e | 53

FOR MORE UPDATES AND MORE MATERIAL DO LIKE OUR FACEBOOK PAGE httpwwwfacebookcomsscmentorsofficial

भस

4) Ass

गधा Correct Answer Zebra

Page 22: COMPILATION OF ALL 72 SETS OF BIOLOGY SSC CHSL-2016 · OF BIOLOGY SSC CHSL-2016 PREPARED BY : SSC MENTORS BIOLOGY SPECIAL . F A C E B O O K P A G E : h t t p : / / w w w . f a c e

F A C E B O O K

P A G E h t t p w w w f a c e b o o k c o m s s c m e n t o r s o f f i c i a l P a g e | 21

FOR MORE UPDATES AND MORE MATERIAL DO LIKE OUR FACEBOOK PAGE httpwwwfacebookcomsscmentorsofficial

सापकाजहरअततयाचधकसिह८चधतिारहह८तीहहॴनजसमहॳ mdashmdash- हह८ताहहॴ Options

l)Prototoxins

परह८टह८टॉनकसस

2)Neutrotoxins

नयटरोटॉनकसस

3)Zootoxins

जटॉनकसस

4)Electrotoxins

इिहॳकटरह८टॉनकसस

Correct Answer Zootoxins

जटॉनकसस

Q86 Which type of pathogen causes the

water-borne disease Schistosomiasis

ककसपरकारकारह८गज़नकजिजननतरह८गलससटह८सह८लमलससकाकारणबनताहहॴ

18Jan2017

Option

1) Parasitic

परजीवी 2)Protozoan

परह८टह८जआ

3) Bacterial

बहॴकटीररयि

4) Viral

वायरि

Correct Answer Parasitic

Schistosomiasis also known as snail

fever and bilharzia is a disease caused

by parasitic

flatworms called schistosomes

Q87 Prothrombin responsible for

clotting of blood is released by

परह८िह८ननबन

जह८रकतकािककाजमनहॳकहॳ लिएनजनमहॳदारहहॴ mdashndash

कहॳ दवारासतरापवतककयाजाताहहॴ

19Jan2017

Options

1) Small Intestine

छह८टीआत

2) Blood Platelets

रकतपिहॳटिहॳटस

3) Large Intestine

बड़ीआत

4Heart

हदय

Correct Answer Blood Platelets

Q88 Acacia arabica is the scientific

name of

अकहॳ लियाअरहॳबबका mdashmdashndash कावहॴजञाननकनामहहॴ 19-Jan-2017

Options

1) Neem

नीम

2) Teak

सागह९न

3) Babhul

बबि

4) Pomegranate

अनार

Correct Answer Babhul

Q89 Cannis Vulpes is the scientific

name of

कहॴ ननसवनमपस mdashmdash- कावहॴजञाननकनामहहॴ 19-Jan-2017

Options

1) Dog

कतता 2) Wolf

भहॳडड़या 3) Fox

िह८मड़ी 4) Hyena

िाकिबगघा

F A C E B O O K

P A G E h t t p w w w f a c e b o o k c o m s s c m e n t o r s o f f i c i a l P a g e | 22

FOR MORE UPDATES AND MORE MATERIAL DO LIKE OUR FACEBOOK PAGE httpwwwfacebookcomsscmentorsofficial

Correct Answer Fox

Q90 The beetroot is the portion of the

beet plant

चकदरपह९धहॳका mdashmdashndash भागहहॴ 19-Jan-2017

Options

1) tap root

मखयजड़

2) Adventitious

आकनसमक

3) bulb of the stem

तनहॳकाकद

4) Rhizome

परकद

Correct Answer tap root

Q91 What is the basic unit of heredity

आनवलिकताकीबननयादीइकाईकयाहहॴ 19-Jan-2017

Options

1) DNA

िीएनए

2) RNA

आरएनए

3) Chromosome

िह८मह८सह८म

4) Gene

जीन

Correct Answer gene

Genes are the units of heredity and are

the instructions that make up the bodyrsquos

blueprint They code for the proteins

that determine virtually all of a personrsquos

characteristics Most genes come in

pairs and are made of strands of genetic

material called deoxyribonucleic acid

or DNA

Q92 Lungs are the primary organs of

फहॳ फड़हॳmdashndashकहॳ परािलमकअगहहॴ

19-Jan-2017

Options

1) Digestion

पाचन

2) Constipation

कलज

3) Perspiration

पसीना 4)Respiration

शवसन

Correct Answer Respiration

Q93 Sugarcane is a type of

गननाएकपरकारका mdash- हहॴ 20-Jan-2017

Options

1)creeper

िता 2)tree

पहॳड़

3)shrub

झाड़ी 4)grass

घास

Correct Answer grass

Q94 Who is commonly known as ldquothe

Father of Microbiologyrdquo

सामानयत ldquo सकषमजीवपवजञानकहॳ जनक lsquo

कहॳ नामसहॳककसहॳजानाजातहहॴ 20-Jan-2017

Options

1) Robert Hooke

रॉबटयहक

2) Antonie Philips van Leeuwenhoek

एटह८नीकफलिपवानमयएनहह८क

3) Carl Linnaeus

काियिीनाईयस

4) Charles Darwin

चामसयिापवयन

F A C E B O O K

P A G E h t t p w w w f a c e b o o k c o m s s c m e n t o r s o f f i c i a l P a g e | 23

FOR MORE UPDATES AND MORE MATERIAL DO LIKE OUR FACEBOOK PAGE httpwwwfacebookcomsscmentorsofficial

Correct Answer Antonie Philips van

Leeuwenhoek

Q95 For the aquatic organisms the

source of food is

जिीयजीवाणकािाघसरह८तहहॴ 20-Jan-2017

Options

1) Phytoplankton

फायटह८पिहॳकटन

2) Sea Weed

समदरीिहॴवाि

3)Aqua plankton

एकवापिहॳकटन

4) Zooplankton

जपिहॳकटन

Correct Answer Phytoplankton

Q96 Haemoglobin has the highest

affinity with which of the following

हीमह८गिह८बबनकीननननमसहॳककसकहॳ सािउततमसमानताहहॴ

20-Jan-2017

Options

1)SO2

2)CO2

3)CO

4)NO2

Correct Answer CO

It has a greater affinity for hemoglobin

than oxygen does It displaces oxygen

and quickly binds so very little oxygen

is transported through the body cells

Q97 Who developed the theory of

Evolution

उदपवकासकालसदातककसनहॳपवकलसतककया

20-Jan-2017

Options

1) Charles Darwin

चामसयिापवयन

2) Isaac Newton

आयजहॳकनयटन

3) Pranav Mistry

परणवलमसतरी 4) Galileo Galilei

गहॳलिलियह८गहॳिीिी Correct Answer Charles Darwin

Q98 The primary function of RNA is

RNA कापरािलमककाययहह८ताहहॴ 20-Jan-2017

Options

1) Photosynthesis

परकािसशिहॳषण

2) Protein Synthesis

परह८टीनसशिहॳषण

3) Replication

परनतकनतबनाना 4) Translation

अनवादकरना Correct Answer Protein Synthesis

There are two main functions of RNA

It assists DNA by serving as a messenger

to relay the proper genetic information

to countless numbers of ribosomes in

your body The other main function of

RNA is to select the correct amino acid

needed by each ribosome to build new

proteins for your body

Q99 ______is the movement of

molecules across a cell membrane from

a region of their lower concentration to

a region of their higher concertration

उचचसादरताकहॳ कषहॳतरसहॳउसकीकमसादरतावािहॳकषहॳतरकीतरफएककह८लिकाखझमिीकहॳ माधयमसहॳहह८नहॳवािाअणओकहॳ सचिनकह८ mdash- कहतहॳहहॴ Options

1) Diffusion

पवसरण

2) Osmosis

ऑसमह८लसस

F A C E B O O K

P A G E h t t p w w w f a c e b o o k c o m s s c m e n t o r s o f f i c i a l P a g e | 24

FOR MORE UPDATES AND MORE MATERIAL DO LIKE OUR FACEBOOK PAGE httpwwwfacebookcomsscmentorsofficial

3) Active Transport

सकियआवागमन

4) Passive Transport

नननषियआवागमन

Correct Answer Active Transport

Q100 Study of classification of

organisms is known as 20-Jan-2017

जीवाणओकहॳ वगीकरणकहॳ अधययनकह८ mdash-

कहाजाताहहॴ Options

1) Serpentology

सपरहॳटह८िह८जी 2) Virology

वायरह८िह८जी 3) Taxonomy

टहॴकसोनह८मी 4) Physiology

कफनज़यह८िह८जी Correct Answer Taxonomy

Q101 Photosynthesis takes place inside

plant cells in

परकािसशिहॳषणवनसपनतकह८लिकामनसति mdash

mdashmdash महह८ताहहॴ 20-Jan-2017

Options

1) Ribosomes

राइबह८सह८नस

2) Chloroplasts

किह८रह८पिासट

3) Nucleus

नयकलियम

4) Mitochondria

माईटह८कोडडरया Correct Answer Chloroplasts

Q102 ______ is the cell organelle in

which the biochemical processes of

respiration and energy production

occur

mdashmdash- वहकह८लिकाअगहहॴ नजसमहॳशवसनऔरउजायउतपादनकहॳ जहॴसीजहॴवरासायननकपरकियायहह८तीहहॴ 20-Jan-2017

Options

1) Mitochondria

माइटह८कोडडरया 2) Chloroplast

किह८रह८पिासट

3) Ribosomes

राइबह८सह८नस

4) Nucleus

नयकिीयस

Correct Answer Mitochondria

Q103 Which non-flowering spore

bearing plants have roots

ककसफिनिगनहॳवािहॳऔरबीजाणधारकपह९धह८कीजड़हॳहह८तीहहॴ 21-Jan-2017

Options

1) Mosses

मह८सहॳस

2) Angiosperms

एननजयह८सपनसय 3) Ferns

फनसय 4) Gymnosperms

नजननह८सपनसय Correct Answer ferns

Q104 Which of the following is an

excretory organ of cockroach

नननननलिखितमसहॳकह९नसानतिच हॳकाउतसजयनअगहहॴ

21-Jan-2017

Options

F A C E B O O K

P A G E h t t p w w w f a c e b o o k c o m s s c m e n t o r s o f f i c i a l P a g e | 25

FOR MORE UPDATES AND MORE MATERIAL DO LIKE OUR FACEBOOK PAGE httpwwwfacebookcomsscmentorsofficial

1) Malphigian Tubules

मनमफनजयनटयबमस

2) Nephridia

नहॳकफरडिया 3) Coxal Gland

कह८कसिगरचिया 4) Green Gland

गरीनगरचिया Correct Answer Malphigian Tubules

Q105 Evaporation of water takes place

in which part of plants

पानीकहॳ वाषपीकरणकीकियापह९धोकहॳ ककसभागसहॳहह८तीहहॴ 21-Jan-2017

Options

1) Stem

तना 2) Stomata

सटह८मटा 3) Branch

िािाए

4) Fruit

फि

Correct Answer Stomata

Evaporation accounts for the movement

of water to the air from sources such as

the soil canopy interception and

waterbodies Transpiration accounts for

the movement of water within a plant

and the subsequent loss of water as

vapour through stomata in its leaves

Q106 A is the fleshy spore-bearing

fruiting body of a fungus

mdashmdashndashकवककामासि

बीजाणधारणकरनहॳवािाफिनहॳवािाअगहहॴ 21-

Jan-2017

Options

1) aloe vera

एिह८वहॳरा 2) Coral

मगा 3) Cactus

कहॴ कटस

4) Mushroom

ककरमतता Correct Answer mushroom

Q107 Which of the following is a fungal

disease

नननननलिखितमसहॳकह९नसाफफदसहॳहह८नहॳवािाएकरह८ग हहॴ

21-Jan-2017

Options

1) Dermatitis

तवचािह८ध

2) Cholera

हहॴजा 3) Jaundice

पीलिया 4) Indigofera

इननिगह८फहॳ रा Correct Answer Dermatitis

Dermatitis also known as eczema is a

group of diseases that results in

inflammation of the skin These diseases

are characterized by itchiness red skin

and a rash In cases of short duration

there may be small blisters while in

long-term cases the skin may become

thickened

Q108 In which form is glucose stored in

our body

हमारहॳिरीरमगिकह८जकासचयककस पमककयाजाताहहॴ

21-Jan-2017

Options

1) Insulin

F A C E B O O K

P A G E h t t p w w w f a c e b o o k c o m s s c m e n t o r s o f f i c i a l P a g e | 26

FOR MORE UPDATES AND MORE MATERIAL DO LIKE OUR FACEBOOK PAGE httpwwwfacebookcomsscmentorsofficial

इसलिन

2) Glucose

गिकह८ज

3) Glycogen

गिायकह८जहॳन

4) Fat

वसा Correct Answer Glycogen

Excess glucose is stored in the liver as

the large compound called glycogen

Glycogen is a polysaccharide of glucose

but its structure allows it to pack

compactly so more of it can be stored in

cells for later use

Q109 Where do plants synthesize

protein from

पह९धहॳपरह८टीनसशिहॳषणकहासहॳकरतहॳहहॴ

Options

1) Fatty Acids

वसाऐलसि

2) Sugar

िकर

3) Amino Acids

एलमनह८ऐलसि

4) Starch

सटाचय Correct Answer Amino Acids

Q110 Which part of the brain is

responsible for triggering actions like

thinking intelligence memory and

ability to learn

मनसतषककाकह९नसाटहससासह८चनहॳ बनधदमानी याददाशतऔरसीिनहॳकीकषमताजहॴसीकियाओकह८परहॳररतकरताहहॴ 21-Jan-2017

Options

1) Diencephalon

िायएनसहॳफहॳ िह८न

2) Hypothalamus

हयपह८िहॳिहॳमस

3) Cerebrum

सहॳरहॳिम

4) Control

कटरह८ि

Correct Answer Cerebrum

Q111 Which of the following is also

known as the Biochemical Laboratory

of the Human Body

नननननलिखितमसहॳककसहॳमानविरीरकीजहॴवरसायनपरयह८गिािाभीकहाजाताहहॴ 21-Jan-2017

Options

1) Small Intestine

छह८टीआत

2)Brain

मनसतषक

3) Pancreas

अगनयािय

4) Liver

नजगर

Correct Answer Liver

The liver makes bile that will help

emulsify and digest the fats we eat

The liver takes toxic substances and

convert them using enzymes the liver

cells makes into a non toxic form so the

body can dispose of them

The liver also converts fats protein and

carbohydrates into glucose which is the

energy source for our cells to use

The liver takes amino acids and makes

proteins by combining them

Q112 The yellow colour of human urine

is due to

मानवमतरकापीिारग mdashndash कीवजहसहॳहह८ताहहॴ 22-

Jan-2017

Options

1) Bile Salts

F A C E B O O K

P A G E h t t p w w w f a c e b o o k c o m s s c m e n t o r s o f f i c i a l P a g e | 27

FOR MORE UPDATES AND MORE MATERIAL DO LIKE OUR FACEBOOK PAGE httpwwwfacebookcomsscmentorsofficial

पपततनमक

2) Cholesterol

कह८िहॳसटरह८ि

3) Lymph

लिनफ

4) Urochrome

यरह८िह८म

Correct Answer Urochrome

Urobilin or urochrome is the chemical

primarily responsible for the yellow

color of urine

Q113 The wilting of plants takes place

due to

पह९धह८कालिचििहह८नाकी mdashmdash- कीवजहसहॳहह८ताहहॴ 22-Jan-2017

Options

1)Photosynthesis

परकािसशिहॳषण

2) Transpiration

वाषपह८तसजयन

3) Absorption

अविह८षण

4) Respiration

शरवसन

Correct Answer Transpiration

Wilting is the loss of rigidity of non-

woody parts of plants This occurs when

the turgor pressure in non-lignified

plant cells falls towards zero as a result

of diminished water in the cells

Q114 Bovidae Ovis is the scientific name of

बह८पविीओपवस mdashndash कावहॴजञाननकनामहहॴ 22-Jan-2017

Options

1) Goat

बकरी 2) Cow

गाय

3) Buffalo

भहॳस

4) Sheep

भहॳड़

Correct Answer Sheep

Q115 Plants get their energy to produce

food from which of the following

पह८धहॳभह८जनकाननमायणकरनहॳकहॳ लिएनननननलिखितमसहॳककससहॳउजायपरापतकरतहॳहहॴ

22-Jan-2017

Options

1) Photosynthesis

परकािसशिहॳषण

2)Bacteria

बहॴकटीररया 3)Fungi

कवक

4)Sun

सयय Correct Answer Sun

Q116 Which of the following is secreted

by the liver

नननननलिखितमसहॳककसकासरावनजगरसहॳहह८ताहहॴ

22-Jan-2017

Options

1) Glucose

गिकह८ज

2) Iodine

आयह८िीन

3) Cortisol

काटटरयसह८ि

4) Bile

पपतत

Correct Answer Bile

The liver makes bile that will help

emulsify and

digest the fats we eat

F A C E B O O K

P A G E h t t p w w w f a c e b o o k c o m s s c m e n t o r s o f f i c i a l P a g e | 28

FOR MORE UPDATES AND MORE MATERIAL DO LIKE OUR FACEBOOK PAGE httpwwwfacebookcomsscmentorsofficial

Q117 Ferns belong to which division of

plants

फनसयपह९धह८कहॳ ककसभागमआतहॳहहॴ

22-Jan-2017

Options

1) Gymnosperms

नजननह८सपनसय 2) Angiosperms

एनजयह८सपनसय 3) Thallophyta

िहॴिह८फाईटा 4)Pteridophyta

टहॳररिह८फाईटा Correct Answer Pteridophyta

Q118 Who invented Antibiotics

एटीबायह८टटककाअपवषकारककसनहॳककयािा

22-Jan-2017

Options

1) Joseph Lister

जह८सहॳफलिसटर

2) William Harvey

पवलियमहाव

3) Robert Knock

रॉबटयनॉक

4)Alexander Fleming

अिहॳकज़िरफिहॳलमग

Correct Answer Alexander Fleming

Q119 Milbecycin is used in the

eradication of

लममबहॳसायलसनका mdashndash

मउनमिनमपरयह८गककयाजाताहहॴ 22-Jan-2017

Options

1) Agricultural Fungus

कपषकवक

2) Agricultural Pests

कपषकीटक

3) Agricultural Herbs

कपषिाक

4)Agricultural Weeds

कपषननराना Correct Answer Agricultural Pests

Milbemycin oxime is a veterinary drug

from the group of milbemycins used as

a broad spectrum antiparasitic It is

active against worms and mites(insects

Q120 Intestinal bacteria synthesizes

which of the following in the human

body

मानविरीरमआतोकहॳ बहॴकटीररयानननननलिखितमसहॳककसकासशिहॳषणकरतहॳहहॴ 22-Jan-2017

Options

1) Vitamin K

पवटालमन K

2) Proteins

परह८टीन

3) Fats

वसा 4) Vitamin D

पवटालमन D

Correct Answer Vitamin K

Q121 is the study of the physical form

and external structure of plants

mdashmdash-

मपह९धह८काभहॴनतक पऔरबाहरीसरचनाकाआदयाककयाजाताहहॴ 22-Jan-2017

Options

1) Physiology

कफनजयह८िह८जी 2) Anatomy

िरीररचनापवजञान

3) Phytomorphology

फाईटह८मह८फह८िह८जी 4)Cytology

कह८लिकापवजञान

Correct Answer Phytomorphology

F A C E B O O K

P A G E h t t p w w w f a c e b o o k c o m s s c m e n t o r s o f f i c i a l P a g e | 29

FOR MORE UPDATES AND MORE MATERIAL DO LIKE OUR FACEBOOK PAGE httpwwwfacebookcomsscmentorsofficial

Q122 Which of the following is a

structural and functional unit of

kidneys

नननननलिखितमसहॳकह९नसीगदोकीसरचनातमकऔरकाययकरीईकाईहहॴ

22-Jan-2017

Options

1) Renette Cells

रहॳनहॳटकह८लिकाए

2) Flame Cells

फिहॳमकह८लिकाए

3) Nephrites

नहॳफ़राइटस

4)Nephrons

नहॳफरोस

Correct Answer Nephrons

Nephron functional unit of the kidney

the structure that actually produces

urine in the process of removing waste

and excess substances from the blood

There are about 1000000 nephrons in

each human kidney

Q123 Which of the following is the

largest part of the human brain

नननननलिखितमसहॳकह९नसामानवमनसतषककासबसहॳबड़ाटहससाहहॴ

23-Jan-2017

Options

1) Ribs

पसलियाा 2) Cerebrum

सहॳरहॳिम

3) Pons

पोस

4)Thalamus

िहॴिहॳमस

Correct Answer Cerebrum

The cerebrum is the largest part of the

human brain making up about two-

thirds of the brainrsquos mass It has two

hemispheres each of which has four

lobes frontal parietal temporal and

occipital

Q124 The auxiliary buds

सहायककालियाmdashndash 23-Jan-2017

Options

1) grow endogenously from the pericycle

पहॳरीसाईककिसहॳअनतजातयपवकलसतहह८ताहहॴ 2) arise endogenously from the main

growing point

मिवपदसहॳअनतजातयउठताहहॴ 3) is an embryonic shoot located in the

axil of a leaf

एकभरणिटहहॴजह८एकपततीकहॳ अकषपरनसतिहह८ताहहॴ 4)arise exogenously from the epidermis

एपपिलमयससहॳबटहजातयतरीकहॳ सहॳउठताहहॴ Correct Answer is an embryonic shoot

located in the axil of a leaf

Q125 Which of the following is a viral

disease

इनमहॳसहॳकह९सीएकवायरिबीमारीहहॴ

23-Jan-2017

Options

1) Polio

पह८लियह८ 2) Tetanus

धनसतनभ

3) Leprosy

कषठरह८ग

4) Plague

पिहॳग

Correct Answer Polio

A viral disease (or viral infection)

occurs when an organismrsquos body is

invaded by pathogenic viruses and

infectious virus particles (virions) attach

to and enter susceptible cells

F A C E B O O K

P A G E h t t p w w w f a c e b o o k c o m s s c m e n t o r s o f f i c i a l P a g e | 30

FOR MORE UPDATES AND MORE MATERIAL DO LIKE OUR FACEBOOK PAGE httpwwwfacebookcomsscmentorsofficial

Poliomyelitis often called polio or

infantile paralysis is an infectious

disease caused by the poliovirus

Tetanusmdash A serious bacterial infection

that causes painful muscle spasms and

can lead to death

Leprosy also known as Hansenrsquos

disease (HD) is a long-term infection by

the bacterium Mycobacterium leprae or

Mycobacterium lepromatosis

Plague is an infectious disease caused by

the bacterium Yersinia pestis

Symptoms include fever weakness and

headache

Q126 Which organisms can help to

carry out Vermicomposting

कह९नसाजीववमीकनपह८नसटगममददकरताहहॴ

23-Jan-2017

Options

1) Nitrifying Bacteria

नाईटरीफाईगबहॴकटीररया 2) Earthworms

कहॴ चऐ

3) Algae

िहॴवि

4) Fungus

कवक

Correct Answer Earthworms

Q127 Contraction of heart is also

known as

हदयकहॳ सकचनकह८ mdash- भीकहाजाताहहॴ 23-Jan-

2017

Options

1) Systole

लससटह८ि

2) Aristotle

अरसत

3) Diastole

िायसटह८ि

4) Lub

मयब

Correct Answer Systole

Diastole is the part of the cardiac cycle

when the heart refills with blood

following systole (contraction)

Ventricular diastole is the period during

which the ventricles are filling and

relaxing while atrial diastole is the

period during which the atria are

relaxing

Q128 Azadirachta indica is the

botanical name of which of the

following

अजाटदराचताइडिकानननननलिखितमसहॳककसकावानसपनतनामहहॴ

23-Jan-2017

Options

1) Rose plant

गिाबकापह९धा 2) Apple tree

सहॳबकापहॳड़

3) Neem

नीम

4)Mango

आम

Correct Answer Neem

Q129 Which of the following is the

main end product of carbohydrate

digestion

नननननलिखितमसहॳकह९नसाकाबोहाइडरहॳटकहॳ पाचनकापरमिअतउतपादकहह८ताहहॴ 23-Jan-2017

Options

1) Fats

वसा 2) Lipids

लिपपडस

3) Glucose

गिकह८ज

4) Cellulose

F A C E B O O K

P A G E h t t p w w w f a c e b o o k c o m s s c m e n t o r s o f f i c i a l P a g e | 31

FOR MORE UPDATES AND MORE MATERIAL DO LIKE OUR FACEBOOK PAGE httpwwwfacebookcomsscmentorsofficial

सहॳमयिह८ज

Correct Answer Glucose

Intestinal absorption of end products

from digestion of carbohydrates and

proteins in the pig hellip During absorption some sugars (fructose or

galactose) released from the

corresponding sucrose and lactose

respectively during digestion were

partly metabolized into glucose by the

enterocyte

Q130 Which of the following glands is a

source of the enzyme Ptyalin

नननननलिखितगरचियोमसहॳएजाइमटयालिनकासरह८तहहॴ 23-Jan-2017

Options

1) Pancreas

अगरािय

2) Thyroid Gland

िाइराइिगरिी 3) Pituitary Gland

पीयषगरिी 4) Salivary Glands

िारगरचियाा Correct Answer Salivary Glands

Q131 Which of the following is not true

about Pteridophyta

ननननमसहॳकह९नसीबातटहॳररिह८फाईटकहॳ बारहॳमसचनहीहहॴ 23-Jan-2017

Options

1) Dominant phase is saprophytes

परमिचरणसहॳपरह८फाईइटसहह८ताहहॴ 2) Main plant body is diploid

पह९दह८कामखयिरीरदपवगखणतहह८ताहहॴ 3) Seeds are present

बीजमह९जदहह८तहॳहहॴ 4)Flowers are absent

फिअनपनसतिहह८तहॳहहॴ

Correct Answer Seeds are present

Q132 The largest dolphin species is the

orca also called as

िॉिकफनकीसबसहॳबड़ीपरजानतकाकानामआकायहहॴनजसहॳ mdash- भीकहतहॳहहॴ 23-Jan-2017

Options

1) Bottle Nose

बाटिनह८ज

2) Baiji

बहॳजी 3) Killer whale

ककिरहहॳि

4)Tucuxi

टकवसी Correct Answer Killer whale

Q133 The fat digesting enzyme Lipase

is secreted by which of the following

वसाकापाचनकरनहॳवािाएजाइमिाइपहॳजनननननलिखितमसहॳककसकहॳ दवारासतरापवतहह८ताहहॴ

24-Jan-2017

Options

1) Kidneys

गद

2) Pancreas

अगनयािय

3) Large Intestine

बड़ीआत

4)Liver

नजगर

Correct Answer Pancreas

Lipase is an enzyme that splits fats so

the intestines can absorb them Lipase

hydrolyzes fats like triglycerides into

their component fatty acid and glycerol

molecules It is found in the blood

gastric juices pancreatic secretions

intestinal juices and adipose tissues

F A C E B O O K

P A G E h t t p w w w f a c e b o o k c o m s s c m e n t o r s o f f i c i a l P a g e | 32

FOR MORE UPDATES AND MORE MATERIAL DO LIKE OUR FACEBOOK PAGE httpwwwfacebookcomsscmentorsofficial

Q134 The arrangement of leaves on an

axis or stem is called

एकअकषयातनहॳपरपनततयोकीयवसिाकह८कयाकहाजाताहहॴ SSC CHSL Science (biology) 2016

Question Paper

24-Jan-2017

Options

1) Phyllotaxy

फाइिह८टहॴकसी 2) Vernation

वनिन

3) Venation

वहॳनहॳिन

4)Phytotaxy

फाइटह८टहॴकसी Correct Answer Phyllotaxy

In botany phyllotaxis or phyllotaxy is

the arrangement of leaves on a plant

stem (from Ancient Greek phyacutellon

ldquoleafrdquo and taacutexis ldquoarrangementrdquo)

Phyllotactic spirals form a distinctive

class of patterns in nature

Q135 The study of Cells is also known

as

कह८लिकाओकहॳ अधययनकह८ mdashmdashndash

भीकहाजाताहहॴ 24-Jan-2017

Options

1) Cytology

सायटह८िह८जी 2) Physiology

कफनजयह८िह८जी 3) Nucleology

नयककमयह८िह८जी 4)Cellology

सहॳिह८िह८जी Correct Answer Cytology

Q136 Which of the following scientists

is also known as the Father of Biology

नननननलिखितमसहॳककसवहॴजञाननककह८ ldquoजीवपवजञानकहॳ जनकrdquoकहॳ नामसहॳभीजानाजाताहहॴ 24-Jan-2017

Options

1) Herbert Spencer

हबयटयसपसर

2) Aristotle

अरसत 3) Lamarck

िहॳमाकय 4)Darwin

िापवयन

Correct Answer Aristotle

Q137 Which cells give rise to various

organs of the plant and keep the plant

growing

कह९नसीकह८लिकाएपह९धह८कहॳ लभननअगह८कह८जनमदहॳतीहहॴऔरपह९धह८कह८बढ़नहॳममददकरतीहहॴ

24-Jan-2017

Options

1) Permanent

सिायी 2) Dermal

तवचीय

3) Meristematic

मररसटहॳमटटक

4)Mature

परह८ढ़

Correct Answer Meristematic

A meristem is the tissue in most plants

containing undifferentiated cells

(meristematic cells) found in zones of

the plant where growth can take place

Q138 Rodentia Muridae is the scientific

name of

F A C E B O O K

P A G E h t t p w w w f a c e b o o k c o m s s c m e n t o r s o f f i c i a l P a g e | 33

FOR MORE UPDATES AND MORE MATERIAL DO LIKE OUR FACEBOOK PAGE httpwwwfacebookcomsscmentorsofficial

रह८िहॳलियानयररिी mdashmdash- कावहॴजञाननकनामहहॴ 24-

Jan-2017

Options

1) Mouse

चहा 2) Squirrel

चगिहरी 3) Monkey

बदर

4) Lizard

नछपकिी Correct Answer Mouse

Q139 Name the scientist who proposed

the cell theory

कह८लिकालसदातकापरसतावदहॳनहॳवािहॳवहॴजञाननककानामबताइए 24-Jan-2017

Options

1) Schleiden and Schwann

िीमिनऔरशरववान

2) Lamarck

िहॳमाकय 3) Treviranus

टरहॳवायरहॳनस

4)Whittaker and Stanley

हीटकरऔरसटहॳनिहॳ Correct Answer Schleiden and

Schwann

Q140 The flower with the worldrsquos

largest bloom is

दननयाकासबसहॳबड़ाफिखििनहॳवािा mdashmdashndash हहॴ 24-Jan-2017

Options

1) Pando

पािह८ 2) Posidonia

पह८सीिह८ननया 3) Rafflesia arnoldii

ररफिहॳलियाअनोमिी 4)Helianthus annuus

हहॳलिएनिसएनयअस

Correct Answer Rafflesia arnoldii

Rafflesia arnoldii is a species of

flowering plant in the parasitic genus

Rafflesia It is noted for producing the

largest individual flower on earth It has

a very strong and horrible odour of

decaying flesh earning it the nickname

ldquocorpse flower

Q141 Deficiency of which vitamin

causes night blindness

ककसपवटालमनकीकमीकहॳ कारणरतौधीहह८ताहहॴ 24-Jan-2017

Options

1) Vitamin K

पवटालमन K

2) Vitamin C

पवटालमन C

3) Vitamin B1

पवटालमन B1

4)Vitamin A

पवटालमन A

Correct Answer Vitamin A

Q142 Nongreen plants lack which of the

following

गहॴर-

हररतवनसपनतमनननननलिखितमसहॳककसकीकमीहह८तीहहॴ

24-Jan-2017

Options

1) Chlorophyll

किह८रह८कफि

2) Lycophyll

िायकह८कफि

3) Cyanophyll

F A C E B O O K

P A G E h t t p w w w f a c e b o o k c o m s s c m e n t o r s o f f i c i a l P a g e | 34

FOR MORE UPDATES AND MORE MATERIAL DO LIKE OUR FACEBOOK PAGE httpwwwfacebookcomsscmentorsofficial

सायनह८कफि

4)Phototropism

फह८टह८टरोपपजम

Correct Answer Chlorophyll

Q143 Organisms that use light to

prepare food are known as

जह८जीवपरकािकाउपयह८गकरभह८जनतहॴयारकरतहॳहहॴ उनह mdashmdash- कहॳ पमजानजाताहहॴ 24-Jan-2017

Options

1) Autotrophs

सवपह८षी 2) Heterotrophs

पवषमपह८षज

3) Omnivores

सवायहारी 4)Decomposers

पवघटनकरनहॳवािा Correct Answer Autotrophs

autotrophs often make their own food

by using sunlight carbon dioxide and

water to form sugars which they can use

for energy Some examples of

autotrophs include plants algae and

even some bacteria Autotrophs

(producer) are important because they

are a food source for heterotrophs

(consumers)

A heterotroph is an organism that

ingests or absorbs organic carbon

(rather than fix carbon from inorganic

sources such as carbon dioxide) in order

to be able to produce energy and

synthesize compounds to maintain its

life Ninety-five percent or more of all

types of living organisms are

heterotrophic including all animals and

fungi and some bacteria

Q144 Which of the following is a

primary function of haemoglobin

नननननलिखितमसहॳकह९नसाटहमह८गिह८बबनकाएकपरािलमककाययहहॴ

25-Jan-2017

Options

1) Utilization of energy

उजायकाउपयह८गकरना 2) Prevention of anaemia

रकतामपताहह८नहॳसहॳरह८कना 3) Destruction of bacteria

बहॴकटीररयाकापवनािकरना 4) To transport oxygen

ऑकसीजनकावहनकरना Correct Answer To transport oxygen

Q145 Vascular bundles are absent in

सवहनीबिि mdashmdash- मअनपनसतिरहतहॳहहॴ 25-Jan-2017

Options

1) Bryophyta

िायह८फाइटा 2) Pteridophyta

टहॳररिह८फाईटा 3) Gymnosperms

नजननह८सपमय 4) Angiosperms

एननजयह८सपहॳनसय Correct Answer Bryophyta

Q146 Sauria Lacertidae is the scientific

name of

सहॴररयािहॳसरटाईिी mdashmdashndash कावहॴजञाननकनामहहॴ 25-Jan-2017

Options

1) Crocodile

मगरमचछ

2) Hippopotamus

टहपपह८पह८टहॳमस

3) Lizard

नछपकिी 4) House fly

F A C E B O O K

P A G E h t t p w w w f a c e b o o k c o m s s c m e n t o r s o f f i c i a l P a g e | 35

FOR MORE UPDATES AND MORE MATERIAL DO LIKE OUR FACEBOOK PAGE httpwwwfacebookcomsscmentorsofficial

घरहॳिमकिी Correct Answer Lizard

Q147 Which type of pathogen causes

the water-borne disease SARS (Severe

Acute Respiratory Syndrome)

ककसपरकािकारह८गज़नकजिजननतबीमारीसासयकाकारणबनताहहॴ 25-Jan-2017

Options

1) Viral

वायरि

2) Parasitic

परजीवी 3) Protozoan

परह८टह८जअन

4) Bacterial

बहॴकटीररयि

Correct Answer Viral

Q148 Which of the following organs

produces the enzyme lipase

नननननलिखितमसहॳकह९नसाअगिायपहॳजएजाइमउतपननकरताहहॴ 25-Jan-2017

Options

1) Pancreas

अगनयािय

2) Large Intestine

बड़ीआत

3) Liver

नजगर

4) Small Intestine

छह८टीआत

Correct Answer Pancreas

Q149 A is a long internode forming the

basal part or the whole of a peduncle

एक mdashmdash- एकिबाइटरनह८िहहॴ जह८ननचिाटहससायासनपणयिठिबनताहहॴ 25-

Jan-2017

Options

1) Rhizome

परकद

2) Rachis

महॳ दि

3) floral axis

पषपअकष

4) Scape

भगदड़

Correct Answer scape

Q150 ndash Which of the following

organisms are considered to be both

Living and Non-living

नननननलिखितमसहॳकह९नसहॳजीवाणकह८जीपवतऔरअजीपवतमानाजाताहहॴ

25-Jan-2017

Options

1) Bacteria

बहॴकटीररया 2) Fungi

कवक

3) Algae

िहॴवाि

4)Virus

वायरस

Correct Answer Virus

They are considered to be living as they

possess a protein coat as a protective

covering DNA as the genetic material

etc

They are said to be non-living as they

can be crystallised and they survive for

billions of years They can tolerate high

temperatures freezing cold

temperatures ultra-violet radiations etc

Q151 Deficiency of fluorine causes

which of the following

फिह८ररनकीकमीकहॳ कारणनननननलिखितमसहॳकयाहह८ताहहॴ

F A C E B O O K

P A G E h t t p w w w f a c e b o o k c o m s s c m e n t o r s o f f i c i a l P a g e | 36

FOR MORE UPDATES AND MORE MATERIAL DO LIKE OUR FACEBOOK PAGE httpwwwfacebookcomsscmentorsofficial

27-Jan-2017

Options

1) Dental Caries

िटिकहॴ ररज

2) Scurvy

सकवरी 3) Anaemia

रकतामपता 4) Arthritis

गटठया Correct Answer Dental Caries

Q152 In a Punnett Square with the

cross AaBb x AaBb how many Aabb

genotypes would be created

पनहॳटसककायरमिह८स AaBb x AaBb कहॳ साि

ककतनहॳ Aabb जीनह८टाइपबनगहॳ 27-Jan-2017

Options

1) 1

2) 8

3) 2

4) 3

Correct Answer 2

Q153 Which of the following is the

Controlling Center of the Cell

नननननलिखित म सहॳ कह८लिकाका ननयतरण

क दर कह९न हहॴ

27-Jan-2017

Options

1) Nucleus

क दर

2) Plasma

पिाजमा 3) Lysosome

िायसह८सह८म

4) Chromosome

िह८मह८सह८म

Correct Answer Nucleus

The control centre of the cell is the

nucleus in eukaryotic cells The nucleus

contains genetic material in the form of

DNA

Q154 Myopia affects which of the

following organs

मायह८पपयानननननलिखितअगह८मसहॳककसहॳपरभापवतकरताहहॴ

25-Jan-2017

Options

1) Heart

हदय

2) Skin

तवचा 3) Eyes

आािहॳ 4)Mouth

मह

Correct Answer Eyes

Q155 Which of the following bears

flowers

नननननलिखितमसहॳकह९नफिधारणकरताहहॴ

25-Jan-2017

Options

1) Bryophyta

िायह८फाइटा 2) Pteridophyta

टहॳरीिह८फाईटा 3) Gymnosperms

नजननह८सपमय 4)Angiosperms

एननजयह८सपमय Correct Answer Angiosperms

Q156 Oxygenated blood flows out of the

heart through the

ऑकसीजनयकतरकत mdashmdashmdash

कहॳ माधयमसहॳहदयकहॳ बाहरबहताहहॴ 25-Jan-2017

F A C E B O O K

P A G E h t t p w w w f a c e b o o k c o m s s c m e n t o r s o f f i c i a l P a g e | 37

FOR MORE UPDATES AND MORE MATERIAL DO LIKE OUR FACEBOOK PAGE httpwwwfacebookcomsscmentorsofficial

Options

1) Aorta

महाधमनी 2) pulmonary artery

फहॳ फड़हॳकीधमनी 3) vena cava

वहॳनाकावा 4)Atrium

चह९क

Correct Answer aorta

Q157 Blood leaving the liver and

moving towards the

heart has a higher concentration of

नजगरसहॳननकिकरहदयकीतरफजानहॳवािहॳरकतम mdashmdashmdashmdash कीउचचसादरताहह८तीहहॴ 27-Jan-2017

Options

1) Lipids

लिपपडस

2) Urea

यररया 3) Bile Pigments

पपततकहॳ रगकरण

4) Carbon dioxide

काबयनिायऑकसाइि

Correct Answer Bile Pigments

Urea is nitrogen containing substance

which is produced in the liver in order

to deal with excess amino-acids in the

body As urea is produced it leaves the

liver in the blood stream and passes via

the circulatory system to all parts of the

body

Q158 Bulb is a modification of which

part of a plant

बमबएकपह९धहॳकहॳ ककसटहससहॳकाएक पातरणहह८ताहहॴ 27-Jan-2017

Options

1) The root

जड़

2) The stem

तना 3) The radicle

मिाकर

4)The fruit

फि

Correct Answer The stem

Q159 Which of the following carries

blood away from the heart to different

body parts

इनमहॳसहॳकह९नरकतकह८हदयसहॳिरीरकहॳ पवलभननअगह८तकिहॳजातीहहॴ

27-Jan-2017

Options

1) Arteries

धमननया 2) Nerves

तबतरहाए

3) Capillaries

कहॳ लिकाए

4)Veins

नसहॳ Correct Answer Arteries

Q160 The series of processes by which

nitrogen and its compounds are

interconverted in the environment and

in living organisms is called

27-Jan-2017

Options

1)Absorption of Nitrogen

2)Ammonification

3)Nitrogen Fixation

4)Nitrogen Cycle

Correct Answer Nitrogen Cycle

Ammonification or Mineralization is

performed by bacteria to convert

organic nitrogen to ammonia

F A C E B O O K

P A G E h t t p w w w f a c e b o o k c o m s s c m e n t o r s o f f i c i a l P a g e | 38

FOR MORE UPDATES AND MORE MATERIAL DO LIKE OUR FACEBOOK PAGE httpwwwfacebookcomsscmentorsofficial

Nitrification can then occur to convert

the ammonium to nitrite and nitrate

Nitrogen fixation is a process by which

nitrogen in the Earthrsquos atmosphere is

converted into ammonia (NH3) or other

molecules available to living organisms

Q161 BCG vaccine is given to protect

from which of the following

बीसीजीकाटटकानननननलिखितमसहॳककसकहॳ बचावकहॳ लिएटदयाजातहहॴ

27-Jan-2017

Options

1) Jaundice

पीलिया 2) Anaemia

रकतमपता 3) Tuberculosis

कषयरह८ग

4) Polio

पह८लियह८ Correct Answer Tuberculosis

Q162 Parallel venation is found in

समानतरवहॳनहॳिन mdashmdashmdash- मपायाजाताहहॴ 27-Jan-2017

Options

1) plants which are monocots

पह९धहॳजह८एकबीजपतरीहह८तहॳहहॴ 2) plants which have a dicot stem

वहॳपह९धहॳनजनकातनादपवदलियहह८ताहहॴ 3) plants with leaves similar to Tulsi

वहॳपह९धहॳनजनकीपनततयतिसीकीपनततयोकहॳ समानहह८तहॳहहॴ 4)plants with tap roots

टहॳप टवािहॳपह९धहॳ Correct Answer plants which are

monocots

Q163 The hardest part of the body is

िरीरकासबसहॳकठह८रभाग mdashndash हहॴ 27-Jan-2017

Options

1) Bones

हडडिय

2) Tooth Enamel

दातकहॳ इनहॳमि

3) Skull

िह८पड़ी 4) Spinal Cord

महॳ रजज

Correct Answer Tooth Enamel

Q164 Which type of pathogen causes

the waterborne disease E coli Infection

ककसपरकारकारह८गजननकजिजननतरह८गईकह८िाईसिमणकाकारणबनताहहॴ 27-Jan-2017

Options

1) Protozoan

परह८टह८जआ

2) Parasitic

परजीवी 3) Bacterial

बहॴकटीररयि

4)Viral

वायरि

Correct Answer Bacterial

Q165 The amount of blood filtered

together by both the kidneys in a 70 kg

adult male human in a minute is

70 की गरा वािहॳएकवयसकप षमएकलमनटमदह८नोगदकहॳदवाराएकसािचाबनीगयीरकतकीमातरहह८तीहहॴ 29-Jan-2017

Options

1) 1100 ml

1100 लमलि

2) 100 ml

F A C E B O O K

P A G E h t t p w w w f a c e b o o k c o m s s c m e n t o r s o f f i c i a l P a g e | 39

FOR MORE UPDATES AND MORE MATERIAL DO LIKE OUR FACEBOOK PAGE httpwwwfacebookcomsscmentorsofficial

100 लमलि

3) 1500 ml

1500 लमलि

4) 500 ml

500 लमलि

Correct Answer 1100 ml

Q166 Which feature of a plant helps to

distinguish a monocot from a dicot

पह९धहॳकीवहकह९नसीपविहॳषताहहॴजह८एकदपवदलियहॳऔरएकएकदिीयपह९धहॳसहॳभहॳदकरनहॳममददकरतीहहॴ 29-Jan-2017

Options

1) Pollination

परागम

2) Venation

वहॳनहॳिन

3) Vernation

वनिन

4) Aestivation

एसटीवहॳिहॳन

Correct Answer venation

Q167 The Mutation Theory was

proposed by

उतवररवतयनकालसदात mdashmdashndash

कहॳ दवरापरसतापवतककयाजाताहहॴ 29-Jan-2017

Options

1) Charles Lyell

चामसयलियहॳि

2) William Smith

पवलियमनसमि

3) Hugo De Vries

हयगह८िीराईस

4)Harrison Schmitt

हहॳरीसननसमट

Correct Answer Hugo De Vries

Q168 Which type of pathogen causes

the waterborne disease HepatitisA

ककसपरकारकहॳ रह८गजनकजिजननतरह८गहहॳपहॳटाइटटस-A काकारणबनताहहॴ

29-Jan-2017

Options

1) Parasitic

परजीवी 2) Viral

वायरि

3) Protozoan

परह८टह८जआ

4) Bacterial

बहॴकटीररयि

Correct Answer Viral

Q169 In a Punnett Square with the

cross AaBb x Aabb how many AaBb

genotypes would be created

पनहॳटसकवायरमिह८स AaBb x Aabb

कहॳ सािककतनहॳ AaBb जीनह८टाइपबनगहॳ 29-Jan-

2017

Options

1) 4

2) 1

3) 7

4) 6

Correct Answer 4

Q170 Arboreal Ateles is the scientific

name of

अिह८ररयिएटटलिस mdashmdashmdash कावहॴजञाननकनामहहॴ 29-Jan-2017

Options

1) Squirrel

चगिहरी 2) Sparrow

गह८रहॴया 3) Lizard

नछपकिी 4) Spider monkey

F A C E B O O K

P A G E h t t p w w w f a c e b o o k c o m s s c m e n t o r s o f f i c i a l P a g e | 40

FOR MORE UPDATES AND MORE MATERIAL DO LIKE OUR FACEBOOK PAGE httpwwwfacebookcomsscmentorsofficial

मकड़ीबदर

Correct Answer Spider monkey

Q171 Which type of pathogen causes

the waterborne disease Salmonellosis

ककसपरकारकारह८गाणजिजननतबीमारीसािमह८नहॳिह८लसज़काकारकहहॴ

29-Jan-2017

Options

1) Algal

िहॳवालियहॳ 2) Parasitic

परजीवी 3) Bacterial

बहॴकटीररयि

4)Viral

वायरि

Correct Answer Bacterial

An infection with salmonella bacteria

commonly caused by contaminated food

or water

Symptoms include diarrhoea fever

chills and abdominal pain

Q172 is a condition in which there is a

deficiency of red cells or of haemoglobin

in the blood

mdashmdash-

एकनसिनतहहॴनजसमहॳरकतमिािकह८लिकाओकीयाहीमह८गिह८बबनकीकमीहह८तीहहॴ 29-Jan-2017

Options

1) Albinism

एनमबननजम

2) Propyria

परह८पीररया 3) Anaemia

एनीलमया 4)Keloid disorder

कहॳ िह८इिडिसओिर

Correct Answer Anaemia

Q173 Ananas comosus is the scientific

name of

Options

अनानासकह८मह८सस mdashmdashmdashndash

कावहॴजञाननकनामहहॴ 29-Jan-2017

1) Custard Apple

सीताफि

2) Pineapple

पाइनएपपि

3) Bamboo

बास

4)Pomegranate

अनार

Correct Answer Pineapple

Q174 Which organ produces insulin

कह९नसाअगइनसलिनपहॴदाकरताहहॴ 29-Jan-

2017

Options

1) Liver

यकत

2) Thyroid gland

िायराइिगरिी 3) Spleen

पिीहा 4)Pancreas

अगरयिय

Correct Answer Pancreas

Q175 Which of the following disease is

not caused by water pollution

नननननलिखितमसहॳकह९नसारह८गपानीकहॳ परदषणकहॳकारणनहीहह८ता

29-Jan-2017

Options

1) Cholera

हहॴजा 2) Typhoid

F A C E B O O K

P A G E h t t p w w w f a c e b o o k c o m s s c m e n t o r s o f f i c i a l P a g e | 41

FOR MORE UPDATES AND MORE MATERIAL DO LIKE OUR FACEBOOK PAGE httpwwwfacebookcomsscmentorsofficial

टाइफाइि

3) Asthma

दमा 4)Diarrhoea

दसत

Correct Answer Asthma

Q176 Ocimum tenuiflorum is the

scientific name of

ओलिलममटहॳयईफिह८रमइसकावहॴजञाननकनाम mdash

ndash हहॴ 30-Jan-2017

Options

1) Neem

नीम

2) Mango

आम

3) Babul

बबि

4)Tulsi

तिसी Correct Answer Tulsi

Q177 Which gland secretes bile a

digestive fluid

कह९नसीगरिीपपतत एकपाचनतरिपरदािय सरापवतकरतीहहॴ 30-Jan-2017

Options

1) Pancreas

अगनयािय

2) Liver

यकत

3) Thyroid

िायराइि

4) Testes

टहॳनसटस

Correct Answer liver

Q178 In which of the following the

dominant phase is Gametophyte

नननननलिखितमसहॳककसकहॳ परमिचरणयगमकह८दपवधद (Gametophyte)हहॴ 30-Jan-2017

Options

1) Bryophyta

िायह८फाइटा 2) Pteridophyta

टहॳररिह८फाइटा 3) Gymnosperms

नजननह८सपमय 4) Angiosperms

एननजयह८सपमय Correct Answer Bryophyta

Q179 Anaerobic respiration refers to

which of the following

नननननलिखितमसहॳककसहॳअवायवीयशवसनकहाजाताहहॴ

30-Jan-2017

Options

1) Respiration without Oxygen

ऑकसीजनकहॳ बबनाशवसन

2) Respiration with Oxygen

ऑकसीजनकहॳ सािशवसन

3) Respiration without CO2

काबयनिायऑकसाइिकहॳ बबनाशवसन

4) Respiration with CO2

काबयनिायऑकसाइिकहॳ सािशविन

Correct Answer Respiration without

Oxygen

Q180 Which type of pathogen causes

the waterborne disease Cholera

ककसपरकारकारह८गजनकजिजननतरह८गहहॴजाकाकारणबनताहहॴ

30-Jan-2017

Options

1) Algal

िहॴवालियहॳ

F A C E B O O K

P A G E h t t p w w w f a c e b o o k c o m s s c m e n t o r s o f f i c i a l P a g e | 42

FOR MORE UPDATES AND MORE MATERIAL DO LIKE OUR FACEBOOK PAGE httpwwwfacebookcomsscmentorsofficial

2) Bacterial

बहॴकटीररयि

3) Protozoan

परह८टह८जआ

4) Viral

वायरि

Correct Answer Bacterial

Q181 To which class does

Oxyreductases transferases hydrolases

belong

ओकसीररिकटहॳसटरासफरहॳजहॳस

हाइडरह८िहॳसहॳसककसवगयमआतहॳहहॴ 30-Jan-2017

Options

1) Hormones

हारमोस

2) Enzymes

एजाइनस

3) Proteins

परह८टीनस

4) Vitamins

पवटालमनस

Correct Answer Enzymes

Q182 Which of the following is not true

about Gymnosperms

ननननमसहॳकह९नसीबातअनावतबीजीकहॳ बारहॳमसचनहीहहॴ 30-Jan-2017

Options

1) Dominant phase is saprophytes

परमिचरणसहॳपरह८फाइटसहह८ताहहॴ 2) Vascular bundles are absent

सवहनीबििअनपनसितहह८ताहहॴ 3) spores are heterospores

बीजाणहहॳटहॳरह८सपह८रसहह८तहॳहहॴ 4) Flowers are absent

फिअनपनसितहह८तहॳहहॴ

Correct Answer Vascular bundles are

absent

Q183 The name of first mammal clone sheep is

भहॳड़कीपरिमसतनपायीपरनत प (किह८न)

कानामहहॴ 30-Jan-2017

Options

1) Noori

नरी 2) Dolly

िॉिी 3) Louise

िसी 4)Durga

दगाय Correct Answer Dolly

Q184 Which type of pathogen causes

the water-borne disease Typhoid fever

ककसपरकारकारह८गजनकजिजननतरह८गटाइफाइिबिारकाकारणबनताहहॴ 30-Jan-2017

Options

1) Algal

िहॴवािीय

2) Parasitic

परजीवी 3) Protozoan

परह८टह८जनअन

4)Bacterial

बहॴकटीररयि

Correct Answer Bacterial

Q185 In which part of the cell are

proteins made

कह८लिकाकहॳ ककसटहससहॳमपरह८टीनबनायाजाताहहॴ

31-Jan-2017

Options

1) Reticulum

रहॳटटकिम

F A C E B O O K

P A G E h t t p w w w f a c e b o o k c o m s s c m e n t o r s o f f i c i a l P a g e | 43

FOR MORE UPDATES AND MORE MATERIAL DO LIKE OUR FACEBOOK PAGE httpwwwfacebookcomsscmentorsofficial

2) Golgi apparatus

गह८मजीएपहॳरहॳटस

3) Ribosomes

ररबह८सह८नस

4) Lysosome

िायसह८सह८नस

Correct Answer ribosomes

Proteins are produced by stringing

amino acids together in the order

specified by messenger RNA strands

that were transcribed from DNA in the

cell nucleus The process of synthesizing

a protein is called translation and it

occurs on ribosomes in the cytoplasm of

a cell

Q186 Polio is a disease caused by which

of the following

नननननलिखितमसहॳपह८लियह८कीबबमारह८हह८नहॳकाकारणकयाहहॴ

31-Jan-2017

Options

1) Bacteria

बहॴकटीररयि

2) Mosquito

मचछर

3) Virus

वायरस

4) Cockroach

नतिच हॳ Correct Answer Virus

Polio or poliomyelitis is a crippling and

potentially deadly infectious disease It

is caused by the poliovirus

Q187 ndash Hay fever is a sign of which of

the following

हहॳकफवरनननननलिखितमसहॳककसकाएकसकहॳ तहहॴ

31-Jan-2017

Options

1) Old Age

वदावसिा 2) Malnutrition

कपह८सण

3) Allergy

एिनजय 4) Over Work

अतयचधककाययकरना Correct Answer Allergy

Q188 How many chromosomes does a

human cell contain

एकमानवकह८लिकामककतनहॳगणसतरहह८तहॳहहॴ

29-Jan-2017

Options

1) 6

2) 26

3) 46

4) 66

Correct Answer 46

In humans each cell normally contains

23 pairs of chromosomes for a total of

46 Twenty-two of these pairs called

autosomes look the same in both males

and females The 23rd pair the sex

chromosomes differ between males and

females

Q189 Which of the following is not true

about Bryophyta

ननननमसहॳकह९नसीबातिायह८फाइटकहॳ बारहॳमसचनहीहहॴ 31-Jan-2017

Options

1) Dominant phase is gametophytes

परमिचरणगहॳलमतह८फाइटसहह८ताहहॴ 2) Main plant body is haploid

पह९धहॳकामखयिरीरअगखणतहह८ताहहॴ 3) Spores are homospores

बीजाणहह८मह८सफह८रसहह८तहॳहहॴ 4) Flowers are present

फिमह८जदहह८तहॳहहॴ Correct Answer Flowers are present

F A C E B O O K

P A G E h t t p w w w f a c e b o o k c o m s s c m e n t o r s o f f i c i a l P a g e | 44

FOR MORE UPDATES AND MORE MATERIAL DO LIKE OUR FACEBOOK PAGE httpwwwfacebookcomsscmentorsofficial

Q190 Which aquatic animal has

trailing tentacles

ककसजिीयजानवरकहॳ पीछहॳचिनहॳवािहॳटहॳटकिसहह८तहॳहहॴ

31-Jan-2017

Options

1) Sea horse

समदरीघह८िा 2) Corals

मगा 3) Jelly fish

जहॳिीमछिी 4) Star fish

तारामछिी Correct Answer Jelly fish

Jellyfish with its umbrella-shaped bell

and trailing tentacles

Q191 Which type of pathogen causes

the water-borne disease Poliomyelitis

(Polio)

ककसपरकारकारह८गजनकजिजननतरह८गपह८लियह८मायहॳटटस (पह८लियह८) काकारणहहॴ 31-Jan-

2017

Options

1) Parasitic

परजीवी 2) Algal

िहॴवालिय

3) Viral

वायरि

4) Bacterial

बहॴकटीररयि

Correct Answer Viral

Q192 The outer white part of the eye

that protects the inner structures is

आािकाबाहरीसफहॳ दटहससाजह८आतररकसरचनाओकीरकषाकरताहहॴ वह mdashmdashmdash हहॴ 31-Jan-

2017

Options

1) Iris

आयररस

2) Sclera

सकिहॳरा 3) Retina

रहॳटटना 4) Cornea

कह८ननयया Correct Answer Sclera

Q193 Proteins are made up of

परह८टीनकाननमायण mdashndash सहॳहह८ताहहॴ 31-Jan-2017

Options

1) Amino acids

एलमनह८अनि

2) Fatty acids

वसायकतअनि

3) Glucose

गिकह८ज

4)Nucleotides

नयनकियह८टाईिस

Correct Answer Amino acids

Q194 Moringa Oleifera is the scientific

name of

मह८ररगओलिफहॳ रा mdashmdashndash कावहॴजञाननकनामहहॴ 31-Jan-2017

Options

1) Banyan

बरगद

2) Gulmohar

गिमह८हर

3) Amla

आमिा

F A C E B O O K

P A G E h t t p w w w f a c e b o o k c o m s s c m e n t o r s o f f i c i a l P a g e | 45

FOR MORE UPDATES AND MORE MATERIAL DO LIKE OUR FACEBOOK PAGE httpwwwfacebookcomsscmentorsofficial

4) Drumstick

डरमनसटक

Correct Answer Drumstick

Q195 Kidney stones are composed of

गदकीपिरी mdashndash सहॳबनीहह८तीहहॴ 1-Feb-2017

Options

1) Calcium Oxalate

कहॴ नमसयमओकजहॳिहॳट

2) Sodium Chloride

सह८डियमकिह८राइि

3) Magnesium Nitrate

महॳनगनलियमनाइतटरहॳट

4) Calcium Bicarbonate

कहॴ नमियमबायकबोनहॳट

Correct Answer Calcium Oxalate

Q196 ndash Which of the following is not

true about Angiosperms

ननननमसहॳकह९नसीबातआवतबीजीकहॳ बारहॳमसचनहीहहॴ 1-Feb-2017

Options

1) Dominant phase is gametophytes

परमिचरणगहॳलमतह८फाइटहह८ताहहॴ 2) Vascular bundles are present

सवहनीबििमह९जदहह८ताहहॴ 3) Spores are heterospores

बीजाणहहॳटहॳरह८सपह८रसहह८तहॳहहॴ 4) Seeds are covered

बीजढकहॳ हह८तहॳहहॴ Correct Answer Dominant phase is

gametophytes

Q197 All of the following are excretory

(waste) products of animals except

नननननलिखितमसहॳककसएककह८छह८ड़करअनयसभीपराखणयोदवाराउतसनजयतपदाियहहॴ 1-Feb-

2017

Options

1) Uric Acid

यररकएलसि

2) Ammonia

अमह८ननया 3) Carbohydrates

काबोहाइडरहॳट

4) Urea

यररया Correct Answer Carbohydrates

In animals the main excretory products

are carbon dioxide ammonia (in

ammoniotelics) urea (in ureotelics) uric

acid (in uricotelics) guanine (in

Arachnida) and creatine

Q198 RNA is a polymeric molecule

What does RNA stand for

आरएनइएएकबहिकआणहहॴ इसकाकापवय पकयाहहॴ 1-Feb-2017

Options

1) Rado Nuclear Acid

रािह८नयनकियरएलसि

2) Ribo Nucleic Acid

राइबह८नयनकिकएलसि

3) Rhino Nuclear Acid

हाइनह८नयनकियरएलसि

4) Resto Nucleus Acid

रहॳसटह८नयकिीयसएलसि

Correct Answer Ribo Nucleic Acid

Q199 Which organ does detoxification

and produces chemicals needed for

digestion

कह९नसाअगपवषहरणकरताहहॴऔरपाचनकहॳ लिएआवशयकरसायनोकह८पहॴदाकरताहहॴ 1-Feb-

2017

Options

1) Salivary glands

िारगरचिया 2) Pancreas

अगनयािय

F A C E B O O K

P A G E h t t p w w w f a c e b o o k c o m s s c m e n t o r s o f f i c i a l P a g e | 46

FOR MORE UPDATES AND MORE MATERIAL DO LIKE OUR FACEBOOK PAGE httpwwwfacebookcomsscmentorsofficial

3) Thyroid gland

िायराइिगरिी 4) Liver

यकत

Correct Answer Liver

Q200 Psidium guajava is the scientific

name of

लसडियमगआजावा mdashmdash कावहॴजञाननकनामहहॴ 1-

Feb-2017

Options

1) Guava

अम द

2) Mango

आम

3) Bamboo

बास

4) Jack fruit

कटहि

Correct Answer Guava

Q201 Which drug is used as a Blood

Thinner

चधरकह८पतिाकरनहॳकहॳ पमककसदवाकापरयह८गककयाजाताहहॴ

1-Feb-2017

Options

1) Warfarin

वाफर न

2) Tramadol

टरहॳमािह८ि

3) Azithromycin

एनजरह८मायलसन

4) Hydralazine

हाइडरह८िहॳनजन

Correct Answer Warfarin

Q202 Which of the following disease is

caused due to the deficiency of protein

परह८टीनकीकमीकहॳ कारणनननननलिखितमसहॳकह९नसारह८गहह८ताहहॴ 1-Feb-2017

Options

1) Arthritis

गटठया 2) Kwashiorkor

कािीओकय र

3) Goitre

गाइटर

4) Night Blindness

रतह९चध

Correct Answer Kwashiorkor

Q203 A is species of plant that has

adapted to survive in an environment

with little liquid water

mdashmdashndashपह९धहॳकीएकऐसहॳऐसहॳपरजानतहहॴ नजसनहॳकमपानीवािहॳवातावरणमजीपवतरहनहॳकहॳलिएअनकिनहहॴ 1-Feb-2017

Options

1) Xerophyte

म दपवद

2) Hydrophyte

जिीयपादप

3) Mesophyte

समह८दपवद

4) Thallophyte

िहॴिह८फाइटा Correct Answer xerophyte

xerophyte is a species of plant that has

adapted to survive in an environment

with little liquid water such as a desert

or an ice- or snow-covered region in the

Alps or the Arctic

Mesophytes are terrestrial plants which

are adapted to neither a particularly

dry nor particularly wet environment

An example of a mesophytic habitat

would be a rural temperate meadow

F A C E B O O K

P A G E h t t p w w w f a c e b o o k c o m s s c m e n t o r s o f f i c i a l P a g e | 47

FOR MORE UPDATES AND MORE MATERIAL DO LIKE OUR FACEBOOK PAGE httpwwwfacebookcomsscmentorsofficial

which might contain goldenrod clover

oxeye daisy and Rosa multiflora

thallophyte any of a group of plants or

plantlike organisms (such as algae and

fungi) that lack differentiated stems

leaves and roots and that were formerly

classified as a primary division

(Thallophyta) of the plant kingdom

Q204 How many types of teeth are

there in humans

मनषयोमककतनहॳपरकारकहॳ दातहह८तहॳहहॴ

1-Feb-2017

Options

1) 4

2) 5

3) 2

4) 3

Correct Answer 4

teeth -Humans have four types of

teethincisors canines premolars and

molars each with a specific function

The incisors cut the food the canines

tear the food and the molars and

premolars crush the food

Q205 Carica papaya is the scientific name of

कहॴ ररकापपाया mdashmdashndash कावहॴजञाननकनामहहॴ 2-

Feb-2017

Options

1) Peepal

पीपि

2) Papaya

पपीता 3) Tamarind

इमिी 4) Drumstick

ढह८िकाछड़ी Correct Answer Papaya

Q206 Muscles get tired when there is

shortfall of

जब mdashndash कीकमीहह८तीहहॴतबपहॳिीयिकजातीहहॴ 2-Feb-2017

Options

1) Lactic acid

िहॴनकटकएलसि

2) Na+ ions

Na+ आयन

3) ATP

एटीपी 4) Sulphates

समफहॳ टस

Correct Answer ATP

ATP is the energy source muscle fibers

use to make muscles contract

muscle tissuersquos main source of energy

called adenosine triphosphate or ATP

As your muscles use up this energy

source they become tired and fatigued

Oxygen is the key ingredient that helps

create new ATP to replenish the burned

up ATP in your muscles

Q207 Artocarpus integra is the

scientific name of आटह८कापयसइटीगरा mdashmdashmdash कावहॴजञाननकनामहहॴ 2-Feb-2017

Options

1) Guava

अम द

2) Pineapple

अनानास

3) Silver Oak

लसमवरओक

4) Jack fruit

कटहि

Correct Answer Jack fruit

Q208 Which organ stores fat soluble

vitamins

कह९नसाअगवसामघिनिीिपवटालमनह८काभिाराकरताहहॴ

2-Feb-2017

F A C E B O O K

P A G E h t t p w w w f a c e b o o k c o m s s c m e n t o r s o f f i c i a l P a g e | 48

FOR MORE UPDATES AND MORE MATERIAL DO LIKE OUR FACEBOOK PAGE httpwwwfacebookcomsscmentorsofficial

Options

1) Blood

रकत

2) Skin

तवचा 3) Liver

यकत

4) Pancreas

अगनयािय

Correct Answer Liver

Q209 Which disease is caused due to

deficiency of Iodine

आयह८िीनकहॳ कारणकह९नसारह८गहह८ताहहॴ 2-Feb-2017

Options

1) Rickets

ररकहॳ टस

2) Scurvy

सकवी 3) Goitre

गणमािा 4) Growth retardation

पवकासका कना Correct Answer Goitre

rickets A softening and weakening of

bones in children usually due to

inadequate vitamin D

Q210 Grevillea Robusta is the scientific name of

गरहॳपवलियारह८बसटा mdashmdashmdash- कापवजञाननकनामहहॴ 2-Feb-2017

Options

1) Peepal

पीपि

2) Teak

सागह९न

3) Silver Oak

लसमवरओक

4) Jack fruit

कटहि

Correct Answer Silver Oak

Q211 When a Cuttlefish is described as a Molluscs it is at which level of

classification

जबएककटिकफिकह८एकमह८िसकाकहॳ पमवखणयतककयाजाताहहॴतबयहॳवगीकरणकहॳ ककससतरपहॳनसितहहॴ 2-Feb-2017

Options

1) Class

वगय 2) Order

िम

3) Family

पररवार

4) Phylum

सघ

Correct Answer Phylum

Q212 Bambusa dendrocalmus is the

scientific name of बानबसािहॳडराकामस mdashmdashmdash कावहॴजञाननकनामहहॴ 3-Feb-2017

Options

1) Banyan

बरगद

2) Papaya

पपीता 3) Bamboo

बास

4) Pomegranate

अनार

Correct Answer Bamboo

Q213 Acinonyx Jubatus is the scientific name of

एलसनह८ननकसजयबहॳटस mdashmdashmdash

कावहॴजञाननकनामहहॴ 3-Feb-2017

F A C E B O O K

P A G E h t t p w w w f a c e b o o k c o m s s c m e n t o r s o f f i c i a l P a g e | 49

FOR MORE UPDATES AND MORE MATERIAL DO LIKE OUR FACEBOOK PAGE httpwwwfacebookcomsscmentorsofficial

Options

1) Bear

भाि 2) Horse

घह८िा 3) Cheetah

चीता 4) Zebra

जहॳिा Correct Answer Cheetah

Q214 The pale yellow colour of urine is

due to the presence of which pigment

मतरकाफीकापीिारगरगदरयकहॳ उपनसिनतकहॳ कारणहह८ताहहॴ

3-Feb-2017

Options

1) Urochrome

यरह८िह८म

2) Urophyll

यरह८कफि

3) Chlorophyll

किह८रह८कफि

4) Chloroplast

किह८रह८पिासट

Correct Answer Urochrome

Q215 Which of the following constitute

to form a gene

नननननलिखितमसहॳकह९नसीचीज़एकजीनकागठनकरतीहहॴ

3-Feb-2017

Options

1) Polynucleotides

पह८िीनयनकियह८टाईडस

2) Hydrocarbons

हाइडरह८काबोस

3) Lipoproteins

िाईपह८परह८टीनस

4) Lipids

लिपपडस

Correct Answer Polynucleotides

Polynucleotide molecule is a biopolymer

composed of 13 or more nucleotide

monomers covalently bonded in a chain

DNA (deoxyribonucleic acid) and RNA

(ribonucleic acid) are examples of

polynucleotides with distinct biological

function

Q216 Vertebrates belongs to the

phylum

रीढ़कीहडिीवािहॳपराणी mdashmdashmdash

परजानतकहॳ अतगायतआतहॳहहॴ 3-Feb-2017

Options

1) Arthropoda

आरह८पह८ड़ा 2) Annelida

एननलििा 3) Cnidaria

ननिहॳररया 4) Chordata

कह८िटा Correct Answer Chordata

Q217 Punica granatum is the scientific name of

पननकगरहॳनहॳटस mdashmdashmdash कावहॴजञाननकनामहहॴ 3-Feb-2017

Options

1) Custard Apple

सीताफि

2) Gulmohar

गिमह८हर

3) Silver Oak

लसमवरओक

4) Pomegranate

अनार

Correct Answer Pomegranate

F A C E B O O K

P A G E h t t p w w w f a c e b o o k c o m s s c m e n t o r s o f f i c i a l P a g e | 50

FOR MORE UPDATES AND MORE MATERIAL DO LIKE OUR FACEBOOK PAGE httpwwwfacebookcomsscmentorsofficial

Q218 Between a tiger and an monkey

which of the following is different

एकबाघऔरबदरकहॳ बीचनननननलिखितमसहॳकह९नसीबातअिगहहॴ 3-Feb-2017

Options

1) Kingdom

राजय

2) Phylum

जानत

3) Order

िम

4) Class

वगय Correct Answer order

Q219 The artificial heart was invented by

कबतरमहदयका mdashmdashmdash

दवाराअपवषकारककयागयािा 3-Feb-2017

Options

1) Muhammad Yunus

महनमदयनस

2) Linus Yale Jr

िाइनसयहॳिजय

3) Gazi Yasargil

गाजीयासचगयि

4) Paul Winchell

पह९िपवमकि Correct Answer Paul Winchell

Q220 Tamarindus indica is the

scientific name of

टहॳमररनडसइडिका mdashmdash कावहॴजञाननकनामहहॴ 7-

Feb-2017

Options

1) Neem

नीम

2) Pineapple

अनानास

3) Tamarind

इमिी 4)Chiku

चीक

Correct Answer Tamarind

Q221 In eukaryotic cells synthesis of

RNA takes place in the

यकहॳ योटटककह८लिकाओमआरएनएकासशिहॳषण

mdashndash महह८ताहहॴ 7-Feb-2017

Options

1) Mitochondria

माईटह८कोडडरया 2) Centrioles

सटरीयह८मस

3) Ribosomes

ररबह८सह८नस

4) Nucleus

नयनकियस

Correct Answer nucleus

eukaryotic cell -Transcription is the

process of synthesizing ribonucleic acid

(RNA)Synthesis takes place within the

nucleus of eukaryotic cells or in the

cytoplasm of prokaryotes and converts

the genetic code from a gene in

deoxyribonucleic acid ( DNA ) to a

strand of RNA that then directs

proteinsynthesis

Q222 _________is caused by parasites

of the Plasmodium genus

पिाजमह८डियमजातीकहॳ परजीवी mdash- कहॳ कारणहहॴ 7-Feb-2017

Options

1) Dysentery

पहॳचचि

2) Malaria

मिहॳररया 3) Chickenpox

F A C E B O O K

P A G E h t t p w w w f a c e b o o k c o m s s c m e n t o r s o f f i c i a l P a g e | 51

FOR MORE UPDATES AND MORE MATERIAL DO LIKE OUR FACEBOOK PAGE httpwwwfacebookcomsscmentorsofficial

चहॳचक

4) Herpes

हहॳपपयस

Correct Answer Malaria

Q223 Carotene in fruits and vegetables

gives it which color

फिह८औरसनलजयोमनसितकहॳ रह८टीनउनहकह९नसारगपरदानकरताहहॴ 7-Feb-2017

Options

1) Green

हरा 2) Pink

गिाबी 3) Orange

नारगी 4) Blue

नीिा Correct Answer Orange

Q224 Equus Caballus is the scientific

name of

एकवसकहॴ बहॳिस mdashmdashndash कापवजञाननकनामहहॴ 7-Feb-2017

Options

1) Horse

घह८िा 2) Zebra

ज़हॳिा 3) Donkey

गधा 4) Buffalo

भस

Correct Answer Horse

Q225 Elapidae Naja is the scientific name of

एिीपीिीनाजा mdashmdash- कावहॴजञाननकनामहहॴ 8-Feb-2017

Options

1) Cobra

कह८बरा 2) Elephant

हािी 3) Eagle

ग ि

4) Owl

उमि Correct Answer Cobra

Q226 Which disease is caused due to

deficiency of Iron

िह८हकीकमीकहॳ कारणकह९नसारह८गहह८ताहहॴ 8-Feb-

2017

Options

1) Beriberi

बहॳरीबहॳरी 2) Tetany

टहॳटनी 3) Kwashiorkor

कवािीऔरकर

4) Anaemia

रकतामपता Correct Answer Anaemia

Beriberi is a disease caused by a vitamin

B-1 deficiency also known as thiamine

deficiency

Tetany can be the result of an

electrolyte imbalance Most often itrsquos a

dramatically low calcium level also

known as hypocalcemia Tetany can also

be caused by magnesium deficiency or

too little potassium Having too much

acid (acidosis) or too much alkali

(alkalosis) in the body can also result in

tetany

Kwashiorkor also known as

ldquoedematous malnutrition It is a form of

malnutrition caused by a lack of protein

in the diet

Anaemia means that you have fewer red

blood cells than normal or you have less

F A C E B O O K

P A G E h t t p w w w f a c e b o o k c o m s s c m e n t o r s o f f i c i a l P a g e | 52

FOR MORE UPDATES AND MORE MATERIAL DO LIKE OUR FACEBOOK PAGE httpwwwfacebookcomsscmentorsofficial

haemoglobin than normal in each red

blood cell

Q227 is a leaf where the leaflets are

arranged along the middle vein

mdashndashएकपततीहहॴजहापतरकह८कीरचनाक ररयालिराकहॳ आसपासहह८तीहहॴ 8-Feb-2017

Options

1) Pinnately compound leaf

पपनहॳटिीसयकतपतती 2) Palmately compound leaf

पामहॳटिीसयकतपतती 3) Compound leaf

सयकतपतती 4) Simple leaf

साधारणपतती Correct Answer Pinnately compound

leaf

Q228 Haustoria or sucking roots are

found in which of the following

हह८सटह८ररयायाचसनहॳवािीजड़हॳनननननलिखितमसहॳककसमपाईजातीहहॴ 8-Feb-2017

Options

1) Wheat

गहॳह

2) Mango

आम

3) Chestnut

चहॳसटनट

4) Cuscuta

कसकयटा Correct Answer Cuscuta

Haustorial roots -The roots of parasitic

plants which penetrate into the host

tissues to absorb nourishment are

called haustorial roots hellip Also known as suckingor parasitic roots

Q229 Equs Asinus is the scientific name

of

एकवसएलसनस mdashmdashndash कावहॴजञाननकनामहहॴ 8-

Feb-2017

Options

1) Donkey

गधा 2) Cow

गाय

3) Deer

टहरन

4) Kangaroo

कगा

Correct Answer Donkey

Q230 Ficus benghalensis is the scientific name of

फाईकसबहॳनगहॳिहॳलसस mdashndash कापवजञाननकनामहहॴ 8-Feb-2017

Options

1) Banyan

बरगद

2) Pineapple

अनानास

3) Babul

बबि

4) Tulsi

तिसी Correct Answer Banyan

Q231 Equus burchellii is the scientific name of

एकवसबचिी mdashmdash- कापवजञाननकनामहहॴ 8-Feb-2017

Options

1) Horse

घह८िा 2) Zebra

जहॳिा 3) Buffalo

F A C E B O O K

P A G E h t t p w w w f a c e b o o k c o m s s c m e n t o r s o f f i c i a l P a g e | 53

FOR MORE UPDATES AND MORE MATERIAL DO LIKE OUR FACEBOOK PAGE httpwwwfacebookcomsscmentorsofficial

भस

4) Ass

गधा Correct Answer Zebra

Page 23: COMPILATION OF ALL 72 SETS OF BIOLOGY SSC CHSL-2016 · OF BIOLOGY SSC CHSL-2016 PREPARED BY : SSC MENTORS BIOLOGY SPECIAL . F A C E B O O K P A G E : h t t p : / / w w w . f a c e

F A C E B O O K

P A G E h t t p w w w f a c e b o o k c o m s s c m e n t o r s o f f i c i a l P a g e | 22

FOR MORE UPDATES AND MORE MATERIAL DO LIKE OUR FACEBOOK PAGE httpwwwfacebookcomsscmentorsofficial

Correct Answer Fox

Q90 The beetroot is the portion of the

beet plant

चकदरपह९धहॳका mdashmdashndash भागहहॴ 19-Jan-2017

Options

1) tap root

मखयजड़

2) Adventitious

आकनसमक

3) bulb of the stem

तनहॳकाकद

4) Rhizome

परकद

Correct Answer tap root

Q91 What is the basic unit of heredity

आनवलिकताकीबननयादीइकाईकयाहहॴ 19-Jan-2017

Options

1) DNA

िीएनए

2) RNA

आरएनए

3) Chromosome

िह८मह८सह८म

4) Gene

जीन

Correct Answer gene

Genes are the units of heredity and are

the instructions that make up the bodyrsquos

blueprint They code for the proteins

that determine virtually all of a personrsquos

characteristics Most genes come in

pairs and are made of strands of genetic

material called deoxyribonucleic acid

or DNA

Q92 Lungs are the primary organs of

फहॳ फड़हॳmdashndashकहॳ परािलमकअगहहॴ

19-Jan-2017

Options

1) Digestion

पाचन

2) Constipation

कलज

3) Perspiration

पसीना 4)Respiration

शवसन

Correct Answer Respiration

Q93 Sugarcane is a type of

गननाएकपरकारका mdash- हहॴ 20-Jan-2017

Options

1)creeper

िता 2)tree

पहॳड़

3)shrub

झाड़ी 4)grass

घास

Correct Answer grass

Q94 Who is commonly known as ldquothe

Father of Microbiologyrdquo

सामानयत ldquo सकषमजीवपवजञानकहॳ जनक lsquo

कहॳ नामसहॳककसहॳजानाजातहहॴ 20-Jan-2017

Options

1) Robert Hooke

रॉबटयहक

2) Antonie Philips van Leeuwenhoek

एटह८नीकफलिपवानमयएनहह८क

3) Carl Linnaeus

काियिीनाईयस

4) Charles Darwin

चामसयिापवयन

F A C E B O O K

P A G E h t t p w w w f a c e b o o k c o m s s c m e n t o r s o f f i c i a l P a g e | 23

FOR MORE UPDATES AND MORE MATERIAL DO LIKE OUR FACEBOOK PAGE httpwwwfacebookcomsscmentorsofficial

Correct Answer Antonie Philips van

Leeuwenhoek

Q95 For the aquatic organisms the

source of food is

जिीयजीवाणकािाघसरह८तहहॴ 20-Jan-2017

Options

1) Phytoplankton

फायटह८पिहॳकटन

2) Sea Weed

समदरीिहॴवाि

3)Aqua plankton

एकवापिहॳकटन

4) Zooplankton

जपिहॳकटन

Correct Answer Phytoplankton

Q96 Haemoglobin has the highest

affinity with which of the following

हीमह८गिह८बबनकीननननमसहॳककसकहॳ सािउततमसमानताहहॴ

20-Jan-2017

Options

1)SO2

2)CO2

3)CO

4)NO2

Correct Answer CO

It has a greater affinity for hemoglobin

than oxygen does It displaces oxygen

and quickly binds so very little oxygen

is transported through the body cells

Q97 Who developed the theory of

Evolution

उदपवकासकालसदातककसनहॳपवकलसतककया

20-Jan-2017

Options

1) Charles Darwin

चामसयिापवयन

2) Isaac Newton

आयजहॳकनयटन

3) Pranav Mistry

परणवलमसतरी 4) Galileo Galilei

गहॳलिलियह८गहॳिीिी Correct Answer Charles Darwin

Q98 The primary function of RNA is

RNA कापरािलमककाययहह८ताहहॴ 20-Jan-2017

Options

1) Photosynthesis

परकािसशिहॳषण

2) Protein Synthesis

परह८टीनसशिहॳषण

3) Replication

परनतकनतबनाना 4) Translation

अनवादकरना Correct Answer Protein Synthesis

There are two main functions of RNA

It assists DNA by serving as a messenger

to relay the proper genetic information

to countless numbers of ribosomes in

your body The other main function of

RNA is to select the correct amino acid

needed by each ribosome to build new

proteins for your body

Q99 ______is the movement of

molecules across a cell membrane from

a region of their lower concentration to

a region of their higher concertration

उचचसादरताकहॳ कषहॳतरसहॳउसकीकमसादरतावािहॳकषहॳतरकीतरफएककह८लिकाखझमिीकहॳ माधयमसहॳहह८नहॳवािाअणओकहॳ सचिनकह८ mdash- कहतहॳहहॴ Options

1) Diffusion

पवसरण

2) Osmosis

ऑसमह८लसस

F A C E B O O K

P A G E h t t p w w w f a c e b o o k c o m s s c m e n t o r s o f f i c i a l P a g e | 24

FOR MORE UPDATES AND MORE MATERIAL DO LIKE OUR FACEBOOK PAGE httpwwwfacebookcomsscmentorsofficial

3) Active Transport

सकियआवागमन

4) Passive Transport

नननषियआवागमन

Correct Answer Active Transport

Q100 Study of classification of

organisms is known as 20-Jan-2017

जीवाणओकहॳ वगीकरणकहॳ अधययनकह८ mdash-

कहाजाताहहॴ Options

1) Serpentology

सपरहॳटह८िह८जी 2) Virology

वायरह८िह८जी 3) Taxonomy

टहॴकसोनह८मी 4) Physiology

कफनज़यह८िह८जी Correct Answer Taxonomy

Q101 Photosynthesis takes place inside

plant cells in

परकािसशिहॳषणवनसपनतकह८लिकामनसति mdash

mdashmdash महह८ताहहॴ 20-Jan-2017

Options

1) Ribosomes

राइबह८सह८नस

2) Chloroplasts

किह८रह८पिासट

3) Nucleus

नयकलियम

4) Mitochondria

माईटह८कोडडरया Correct Answer Chloroplasts

Q102 ______ is the cell organelle in

which the biochemical processes of

respiration and energy production

occur

mdashmdash- वहकह८लिकाअगहहॴ नजसमहॳशवसनऔरउजायउतपादनकहॳ जहॴसीजहॴवरासायननकपरकियायहह८तीहहॴ 20-Jan-2017

Options

1) Mitochondria

माइटह८कोडडरया 2) Chloroplast

किह८रह८पिासट

3) Ribosomes

राइबह८सह८नस

4) Nucleus

नयकिीयस

Correct Answer Mitochondria

Q103 Which non-flowering spore

bearing plants have roots

ककसफिनिगनहॳवािहॳऔरबीजाणधारकपह९धह८कीजड़हॳहह८तीहहॴ 21-Jan-2017

Options

1) Mosses

मह८सहॳस

2) Angiosperms

एननजयह८सपनसय 3) Ferns

फनसय 4) Gymnosperms

नजननह८सपनसय Correct Answer ferns

Q104 Which of the following is an

excretory organ of cockroach

नननननलिखितमसहॳकह९नसानतिच हॳकाउतसजयनअगहहॴ

21-Jan-2017

Options

F A C E B O O K

P A G E h t t p w w w f a c e b o o k c o m s s c m e n t o r s o f f i c i a l P a g e | 25

FOR MORE UPDATES AND MORE MATERIAL DO LIKE OUR FACEBOOK PAGE httpwwwfacebookcomsscmentorsofficial

1) Malphigian Tubules

मनमफनजयनटयबमस

2) Nephridia

नहॳकफरडिया 3) Coxal Gland

कह८कसिगरचिया 4) Green Gland

गरीनगरचिया Correct Answer Malphigian Tubules

Q105 Evaporation of water takes place

in which part of plants

पानीकहॳ वाषपीकरणकीकियापह९धोकहॳ ककसभागसहॳहह८तीहहॴ 21-Jan-2017

Options

1) Stem

तना 2) Stomata

सटह८मटा 3) Branch

िािाए

4) Fruit

फि

Correct Answer Stomata

Evaporation accounts for the movement

of water to the air from sources such as

the soil canopy interception and

waterbodies Transpiration accounts for

the movement of water within a plant

and the subsequent loss of water as

vapour through stomata in its leaves

Q106 A is the fleshy spore-bearing

fruiting body of a fungus

mdashmdashndashकवककामासि

बीजाणधारणकरनहॳवािाफिनहॳवािाअगहहॴ 21-

Jan-2017

Options

1) aloe vera

एिह८वहॳरा 2) Coral

मगा 3) Cactus

कहॴ कटस

4) Mushroom

ककरमतता Correct Answer mushroom

Q107 Which of the following is a fungal

disease

नननननलिखितमसहॳकह९नसाफफदसहॳहह८नहॳवािाएकरह८ग हहॴ

21-Jan-2017

Options

1) Dermatitis

तवचािह८ध

2) Cholera

हहॴजा 3) Jaundice

पीलिया 4) Indigofera

इननिगह८फहॳ रा Correct Answer Dermatitis

Dermatitis also known as eczema is a

group of diseases that results in

inflammation of the skin These diseases

are characterized by itchiness red skin

and a rash In cases of short duration

there may be small blisters while in

long-term cases the skin may become

thickened

Q108 In which form is glucose stored in

our body

हमारहॳिरीरमगिकह८जकासचयककस पमककयाजाताहहॴ

21-Jan-2017

Options

1) Insulin

F A C E B O O K

P A G E h t t p w w w f a c e b o o k c o m s s c m e n t o r s o f f i c i a l P a g e | 26

FOR MORE UPDATES AND MORE MATERIAL DO LIKE OUR FACEBOOK PAGE httpwwwfacebookcomsscmentorsofficial

इसलिन

2) Glucose

गिकह८ज

3) Glycogen

गिायकह८जहॳन

4) Fat

वसा Correct Answer Glycogen

Excess glucose is stored in the liver as

the large compound called glycogen

Glycogen is a polysaccharide of glucose

but its structure allows it to pack

compactly so more of it can be stored in

cells for later use

Q109 Where do plants synthesize

protein from

पह९धहॳपरह८टीनसशिहॳषणकहासहॳकरतहॳहहॴ

Options

1) Fatty Acids

वसाऐलसि

2) Sugar

िकर

3) Amino Acids

एलमनह८ऐलसि

4) Starch

सटाचय Correct Answer Amino Acids

Q110 Which part of the brain is

responsible for triggering actions like

thinking intelligence memory and

ability to learn

मनसतषककाकह९नसाटहससासह८चनहॳ बनधदमानी याददाशतऔरसीिनहॳकीकषमताजहॴसीकियाओकह८परहॳररतकरताहहॴ 21-Jan-2017

Options

1) Diencephalon

िायएनसहॳफहॳ िह८न

2) Hypothalamus

हयपह८िहॳिहॳमस

3) Cerebrum

सहॳरहॳिम

4) Control

कटरह८ि

Correct Answer Cerebrum

Q111 Which of the following is also

known as the Biochemical Laboratory

of the Human Body

नननननलिखितमसहॳककसहॳमानविरीरकीजहॴवरसायनपरयह८गिािाभीकहाजाताहहॴ 21-Jan-2017

Options

1) Small Intestine

छह८टीआत

2)Brain

मनसतषक

3) Pancreas

अगनयािय

4) Liver

नजगर

Correct Answer Liver

The liver makes bile that will help

emulsify and digest the fats we eat

The liver takes toxic substances and

convert them using enzymes the liver

cells makes into a non toxic form so the

body can dispose of them

The liver also converts fats protein and

carbohydrates into glucose which is the

energy source for our cells to use

The liver takes amino acids and makes

proteins by combining them

Q112 The yellow colour of human urine

is due to

मानवमतरकापीिारग mdashndash कीवजहसहॳहह८ताहहॴ 22-

Jan-2017

Options

1) Bile Salts

F A C E B O O K

P A G E h t t p w w w f a c e b o o k c o m s s c m e n t o r s o f f i c i a l P a g e | 27

FOR MORE UPDATES AND MORE MATERIAL DO LIKE OUR FACEBOOK PAGE httpwwwfacebookcomsscmentorsofficial

पपततनमक

2) Cholesterol

कह८िहॳसटरह८ि

3) Lymph

लिनफ

4) Urochrome

यरह८िह८म

Correct Answer Urochrome

Urobilin or urochrome is the chemical

primarily responsible for the yellow

color of urine

Q113 The wilting of plants takes place

due to

पह९धह८कालिचििहह८नाकी mdashmdash- कीवजहसहॳहह८ताहहॴ 22-Jan-2017

Options

1)Photosynthesis

परकािसशिहॳषण

2) Transpiration

वाषपह८तसजयन

3) Absorption

अविह८षण

4) Respiration

शरवसन

Correct Answer Transpiration

Wilting is the loss of rigidity of non-

woody parts of plants This occurs when

the turgor pressure in non-lignified

plant cells falls towards zero as a result

of diminished water in the cells

Q114 Bovidae Ovis is the scientific name of

बह८पविीओपवस mdashndash कावहॴजञाननकनामहहॴ 22-Jan-2017

Options

1) Goat

बकरी 2) Cow

गाय

3) Buffalo

भहॳस

4) Sheep

भहॳड़

Correct Answer Sheep

Q115 Plants get their energy to produce

food from which of the following

पह८धहॳभह८जनकाननमायणकरनहॳकहॳ लिएनननननलिखितमसहॳककससहॳउजायपरापतकरतहॳहहॴ

22-Jan-2017

Options

1) Photosynthesis

परकािसशिहॳषण

2)Bacteria

बहॴकटीररया 3)Fungi

कवक

4)Sun

सयय Correct Answer Sun

Q116 Which of the following is secreted

by the liver

नननननलिखितमसहॳककसकासरावनजगरसहॳहह८ताहहॴ

22-Jan-2017

Options

1) Glucose

गिकह८ज

2) Iodine

आयह८िीन

3) Cortisol

काटटरयसह८ि

4) Bile

पपतत

Correct Answer Bile

The liver makes bile that will help

emulsify and

digest the fats we eat

F A C E B O O K

P A G E h t t p w w w f a c e b o o k c o m s s c m e n t o r s o f f i c i a l P a g e | 28

FOR MORE UPDATES AND MORE MATERIAL DO LIKE OUR FACEBOOK PAGE httpwwwfacebookcomsscmentorsofficial

Q117 Ferns belong to which division of

plants

फनसयपह९धह८कहॳ ककसभागमआतहॳहहॴ

22-Jan-2017

Options

1) Gymnosperms

नजननह८सपनसय 2) Angiosperms

एनजयह८सपनसय 3) Thallophyta

िहॴिह८फाईटा 4)Pteridophyta

टहॳररिह८फाईटा Correct Answer Pteridophyta

Q118 Who invented Antibiotics

एटीबायह८टटककाअपवषकारककसनहॳककयािा

22-Jan-2017

Options

1) Joseph Lister

जह८सहॳफलिसटर

2) William Harvey

पवलियमहाव

3) Robert Knock

रॉबटयनॉक

4)Alexander Fleming

अिहॳकज़िरफिहॳलमग

Correct Answer Alexander Fleming

Q119 Milbecycin is used in the

eradication of

लममबहॳसायलसनका mdashndash

मउनमिनमपरयह८गककयाजाताहहॴ 22-Jan-2017

Options

1) Agricultural Fungus

कपषकवक

2) Agricultural Pests

कपषकीटक

3) Agricultural Herbs

कपषिाक

4)Agricultural Weeds

कपषननराना Correct Answer Agricultural Pests

Milbemycin oxime is a veterinary drug

from the group of milbemycins used as

a broad spectrum antiparasitic It is

active against worms and mites(insects

Q120 Intestinal bacteria synthesizes

which of the following in the human

body

मानविरीरमआतोकहॳ बहॴकटीररयानननननलिखितमसहॳककसकासशिहॳषणकरतहॳहहॴ 22-Jan-2017

Options

1) Vitamin K

पवटालमन K

2) Proteins

परह८टीन

3) Fats

वसा 4) Vitamin D

पवटालमन D

Correct Answer Vitamin K

Q121 is the study of the physical form

and external structure of plants

mdashmdash-

मपह९धह८काभहॴनतक पऔरबाहरीसरचनाकाआदयाककयाजाताहहॴ 22-Jan-2017

Options

1) Physiology

कफनजयह८िह८जी 2) Anatomy

िरीररचनापवजञान

3) Phytomorphology

फाईटह८मह८फह८िह८जी 4)Cytology

कह८लिकापवजञान

Correct Answer Phytomorphology

F A C E B O O K

P A G E h t t p w w w f a c e b o o k c o m s s c m e n t o r s o f f i c i a l P a g e | 29

FOR MORE UPDATES AND MORE MATERIAL DO LIKE OUR FACEBOOK PAGE httpwwwfacebookcomsscmentorsofficial

Q122 Which of the following is a

structural and functional unit of

kidneys

नननननलिखितमसहॳकह९नसीगदोकीसरचनातमकऔरकाययकरीईकाईहहॴ

22-Jan-2017

Options

1) Renette Cells

रहॳनहॳटकह८लिकाए

2) Flame Cells

फिहॳमकह८लिकाए

3) Nephrites

नहॳफ़राइटस

4)Nephrons

नहॳफरोस

Correct Answer Nephrons

Nephron functional unit of the kidney

the structure that actually produces

urine in the process of removing waste

and excess substances from the blood

There are about 1000000 nephrons in

each human kidney

Q123 Which of the following is the

largest part of the human brain

नननननलिखितमसहॳकह९नसामानवमनसतषककासबसहॳबड़ाटहससाहहॴ

23-Jan-2017

Options

1) Ribs

पसलियाा 2) Cerebrum

सहॳरहॳिम

3) Pons

पोस

4)Thalamus

िहॴिहॳमस

Correct Answer Cerebrum

The cerebrum is the largest part of the

human brain making up about two-

thirds of the brainrsquos mass It has two

hemispheres each of which has four

lobes frontal parietal temporal and

occipital

Q124 The auxiliary buds

सहायककालियाmdashndash 23-Jan-2017

Options

1) grow endogenously from the pericycle

पहॳरीसाईककिसहॳअनतजातयपवकलसतहह८ताहहॴ 2) arise endogenously from the main

growing point

मिवपदसहॳअनतजातयउठताहहॴ 3) is an embryonic shoot located in the

axil of a leaf

एकभरणिटहहॴजह८एकपततीकहॳ अकषपरनसतिहह८ताहहॴ 4)arise exogenously from the epidermis

एपपिलमयससहॳबटहजातयतरीकहॳ सहॳउठताहहॴ Correct Answer is an embryonic shoot

located in the axil of a leaf

Q125 Which of the following is a viral

disease

इनमहॳसहॳकह९सीएकवायरिबीमारीहहॴ

23-Jan-2017

Options

1) Polio

पह८लियह८ 2) Tetanus

धनसतनभ

3) Leprosy

कषठरह८ग

4) Plague

पिहॳग

Correct Answer Polio

A viral disease (or viral infection)

occurs when an organismrsquos body is

invaded by pathogenic viruses and

infectious virus particles (virions) attach

to and enter susceptible cells

F A C E B O O K

P A G E h t t p w w w f a c e b o o k c o m s s c m e n t o r s o f f i c i a l P a g e | 30

FOR MORE UPDATES AND MORE MATERIAL DO LIKE OUR FACEBOOK PAGE httpwwwfacebookcomsscmentorsofficial

Poliomyelitis often called polio or

infantile paralysis is an infectious

disease caused by the poliovirus

Tetanusmdash A serious bacterial infection

that causes painful muscle spasms and

can lead to death

Leprosy also known as Hansenrsquos

disease (HD) is a long-term infection by

the bacterium Mycobacterium leprae or

Mycobacterium lepromatosis

Plague is an infectious disease caused by

the bacterium Yersinia pestis

Symptoms include fever weakness and

headache

Q126 Which organisms can help to

carry out Vermicomposting

कह९नसाजीववमीकनपह८नसटगममददकरताहहॴ

23-Jan-2017

Options

1) Nitrifying Bacteria

नाईटरीफाईगबहॴकटीररया 2) Earthworms

कहॴ चऐ

3) Algae

िहॴवि

4) Fungus

कवक

Correct Answer Earthworms

Q127 Contraction of heart is also

known as

हदयकहॳ सकचनकह८ mdash- भीकहाजाताहहॴ 23-Jan-

2017

Options

1) Systole

लससटह८ि

2) Aristotle

अरसत

3) Diastole

िायसटह८ि

4) Lub

मयब

Correct Answer Systole

Diastole is the part of the cardiac cycle

when the heart refills with blood

following systole (contraction)

Ventricular diastole is the period during

which the ventricles are filling and

relaxing while atrial diastole is the

period during which the atria are

relaxing

Q128 Azadirachta indica is the

botanical name of which of the

following

अजाटदराचताइडिकानननननलिखितमसहॳककसकावानसपनतनामहहॴ

23-Jan-2017

Options

1) Rose plant

गिाबकापह९धा 2) Apple tree

सहॳबकापहॳड़

3) Neem

नीम

4)Mango

आम

Correct Answer Neem

Q129 Which of the following is the

main end product of carbohydrate

digestion

नननननलिखितमसहॳकह९नसाकाबोहाइडरहॳटकहॳ पाचनकापरमिअतउतपादकहह८ताहहॴ 23-Jan-2017

Options

1) Fats

वसा 2) Lipids

लिपपडस

3) Glucose

गिकह८ज

4) Cellulose

F A C E B O O K

P A G E h t t p w w w f a c e b o o k c o m s s c m e n t o r s o f f i c i a l P a g e | 31

FOR MORE UPDATES AND MORE MATERIAL DO LIKE OUR FACEBOOK PAGE httpwwwfacebookcomsscmentorsofficial

सहॳमयिह८ज

Correct Answer Glucose

Intestinal absorption of end products

from digestion of carbohydrates and

proteins in the pig hellip During absorption some sugars (fructose or

galactose) released from the

corresponding sucrose and lactose

respectively during digestion were

partly metabolized into glucose by the

enterocyte

Q130 Which of the following glands is a

source of the enzyme Ptyalin

नननननलिखितगरचियोमसहॳएजाइमटयालिनकासरह८तहहॴ 23-Jan-2017

Options

1) Pancreas

अगरािय

2) Thyroid Gland

िाइराइिगरिी 3) Pituitary Gland

पीयषगरिी 4) Salivary Glands

िारगरचियाा Correct Answer Salivary Glands

Q131 Which of the following is not true

about Pteridophyta

ननननमसहॳकह९नसीबातटहॳररिह८फाईटकहॳ बारहॳमसचनहीहहॴ 23-Jan-2017

Options

1) Dominant phase is saprophytes

परमिचरणसहॳपरह८फाईइटसहह८ताहहॴ 2) Main plant body is diploid

पह९दह८कामखयिरीरदपवगखणतहह८ताहहॴ 3) Seeds are present

बीजमह९जदहह८तहॳहहॴ 4)Flowers are absent

फिअनपनसतिहह८तहॳहहॴ

Correct Answer Seeds are present

Q132 The largest dolphin species is the

orca also called as

िॉिकफनकीसबसहॳबड़ीपरजानतकाकानामआकायहहॴनजसहॳ mdash- भीकहतहॳहहॴ 23-Jan-2017

Options

1) Bottle Nose

बाटिनह८ज

2) Baiji

बहॳजी 3) Killer whale

ककिरहहॳि

4)Tucuxi

टकवसी Correct Answer Killer whale

Q133 The fat digesting enzyme Lipase

is secreted by which of the following

वसाकापाचनकरनहॳवािाएजाइमिाइपहॳजनननननलिखितमसहॳककसकहॳ दवारासतरापवतहह८ताहहॴ

24-Jan-2017

Options

1) Kidneys

गद

2) Pancreas

अगनयािय

3) Large Intestine

बड़ीआत

4)Liver

नजगर

Correct Answer Pancreas

Lipase is an enzyme that splits fats so

the intestines can absorb them Lipase

hydrolyzes fats like triglycerides into

their component fatty acid and glycerol

molecules It is found in the blood

gastric juices pancreatic secretions

intestinal juices and adipose tissues

F A C E B O O K

P A G E h t t p w w w f a c e b o o k c o m s s c m e n t o r s o f f i c i a l P a g e | 32

FOR MORE UPDATES AND MORE MATERIAL DO LIKE OUR FACEBOOK PAGE httpwwwfacebookcomsscmentorsofficial

Q134 The arrangement of leaves on an

axis or stem is called

एकअकषयातनहॳपरपनततयोकीयवसिाकह८कयाकहाजाताहहॴ SSC CHSL Science (biology) 2016

Question Paper

24-Jan-2017

Options

1) Phyllotaxy

फाइिह८टहॴकसी 2) Vernation

वनिन

3) Venation

वहॳनहॳिन

4)Phytotaxy

फाइटह८टहॴकसी Correct Answer Phyllotaxy

In botany phyllotaxis or phyllotaxy is

the arrangement of leaves on a plant

stem (from Ancient Greek phyacutellon

ldquoleafrdquo and taacutexis ldquoarrangementrdquo)

Phyllotactic spirals form a distinctive

class of patterns in nature

Q135 The study of Cells is also known

as

कह८लिकाओकहॳ अधययनकह८ mdashmdashndash

भीकहाजाताहहॴ 24-Jan-2017

Options

1) Cytology

सायटह८िह८जी 2) Physiology

कफनजयह८िह८जी 3) Nucleology

नयककमयह८िह८जी 4)Cellology

सहॳिह८िह८जी Correct Answer Cytology

Q136 Which of the following scientists

is also known as the Father of Biology

नननननलिखितमसहॳककसवहॴजञाननककह८ ldquoजीवपवजञानकहॳ जनकrdquoकहॳ नामसहॳभीजानाजाताहहॴ 24-Jan-2017

Options

1) Herbert Spencer

हबयटयसपसर

2) Aristotle

अरसत 3) Lamarck

िहॳमाकय 4)Darwin

िापवयन

Correct Answer Aristotle

Q137 Which cells give rise to various

organs of the plant and keep the plant

growing

कह९नसीकह८लिकाएपह९धह८कहॳ लभननअगह८कह८जनमदहॳतीहहॴऔरपह९धह८कह८बढ़नहॳममददकरतीहहॴ

24-Jan-2017

Options

1) Permanent

सिायी 2) Dermal

तवचीय

3) Meristematic

मररसटहॳमटटक

4)Mature

परह८ढ़

Correct Answer Meristematic

A meristem is the tissue in most plants

containing undifferentiated cells

(meristematic cells) found in zones of

the plant where growth can take place

Q138 Rodentia Muridae is the scientific

name of

F A C E B O O K

P A G E h t t p w w w f a c e b o o k c o m s s c m e n t o r s o f f i c i a l P a g e | 33

FOR MORE UPDATES AND MORE MATERIAL DO LIKE OUR FACEBOOK PAGE httpwwwfacebookcomsscmentorsofficial

रह८िहॳलियानयररिी mdashmdash- कावहॴजञाननकनामहहॴ 24-

Jan-2017

Options

1) Mouse

चहा 2) Squirrel

चगिहरी 3) Monkey

बदर

4) Lizard

नछपकिी Correct Answer Mouse

Q139 Name the scientist who proposed

the cell theory

कह८लिकालसदातकापरसतावदहॳनहॳवािहॳवहॴजञाननककानामबताइए 24-Jan-2017

Options

1) Schleiden and Schwann

िीमिनऔरशरववान

2) Lamarck

िहॳमाकय 3) Treviranus

टरहॳवायरहॳनस

4)Whittaker and Stanley

हीटकरऔरसटहॳनिहॳ Correct Answer Schleiden and

Schwann

Q140 The flower with the worldrsquos

largest bloom is

दननयाकासबसहॳबड़ाफिखििनहॳवािा mdashmdashndash हहॴ 24-Jan-2017

Options

1) Pando

पािह८ 2) Posidonia

पह८सीिह८ननया 3) Rafflesia arnoldii

ररफिहॳलियाअनोमिी 4)Helianthus annuus

हहॳलिएनिसएनयअस

Correct Answer Rafflesia arnoldii

Rafflesia arnoldii is a species of

flowering plant in the parasitic genus

Rafflesia It is noted for producing the

largest individual flower on earth It has

a very strong and horrible odour of

decaying flesh earning it the nickname

ldquocorpse flower

Q141 Deficiency of which vitamin

causes night blindness

ककसपवटालमनकीकमीकहॳ कारणरतौधीहह८ताहहॴ 24-Jan-2017

Options

1) Vitamin K

पवटालमन K

2) Vitamin C

पवटालमन C

3) Vitamin B1

पवटालमन B1

4)Vitamin A

पवटालमन A

Correct Answer Vitamin A

Q142 Nongreen plants lack which of the

following

गहॴर-

हररतवनसपनतमनननननलिखितमसहॳककसकीकमीहह८तीहहॴ

24-Jan-2017

Options

1) Chlorophyll

किह८रह८कफि

2) Lycophyll

िायकह८कफि

3) Cyanophyll

F A C E B O O K

P A G E h t t p w w w f a c e b o o k c o m s s c m e n t o r s o f f i c i a l P a g e | 34

FOR MORE UPDATES AND MORE MATERIAL DO LIKE OUR FACEBOOK PAGE httpwwwfacebookcomsscmentorsofficial

सायनह८कफि

4)Phototropism

फह८टह८टरोपपजम

Correct Answer Chlorophyll

Q143 Organisms that use light to

prepare food are known as

जह८जीवपरकािकाउपयह८गकरभह८जनतहॴयारकरतहॳहहॴ उनह mdashmdash- कहॳ पमजानजाताहहॴ 24-Jan-2017

Options

1) Autotrophs

सवपह८षी 2) Heterotrophs

पवषमपह८षज

3) Omnivores

सवायहारी 4)Decomposers

पवघटनकरनहॳवािा Correct Answer Autotrophs

autotrophs often make their own food

by using sunlight carbon dioxide and

water to form sugars which they can use

for energy Some examples of

autotrophs include plants algae and

even some bacteria Autotrophs

(producer) are important because they

are a food source for heterotrophs

(consumers)

A heterotroph is an organism that

ingests or absorbs organic carbon

(rather than fix carbon from inorganic

sources such as carbon dioxide) in order

to be able to produce energy and

synthesize compounds to maintain its

life Ninety-five percent or more of all

types of living organisms are

heterotrophic including all animals and

fungi and some bacteria

Q144 Which of the following is a

primary function of haemoglobin

नननननलिखितमसहॳकह९नसाटहमह८गिह८बबनकाएकपरािलमककाययहहॴ

25-Jan-2017

Options

1) Utilization of energy

उजायकाउपयह८गकरना 2) Prevention of anaemia

रकतामपताहह८नहॳसहॳरह८कना 3) Destruction of bacteria

बहॴकटीररयाकापवनािकरना 4) To transport oxygen

ऑकसीजनकावहनकरना Correct Answer To transport oxygen

Q145 Vascular bundles are absent in

सवहनीबिि mdashmdash- मअनपनसतिरहतहॳहहॴ 25-Jan-2017

Options

1) Bryophyta

िायह८फाइटा 2) Pteridophyta

टहॳररिह८फाईटा 3) Gymnosperms

नजननह८सपमय 4) Angiosperms

एननजयह८सपहॳनसय Correct Answer Bryophyta

Q146 Sauria Lacertidae is the scientific

name of

सहॴररयािहॳसरटाईिी mdashmdashndash कावहॴजञाननकनामहहॴ 25-Jan-2017

Options

1) Crocodile

मगरमचछ

2) Hippopotamus

टहपपह८पह८टहॳमस

3) Lizard

नछपकिी 4) House fly

F A C E B O O K

P A G E h t t p w w w f a c e b o o k c o m s s c m e n t o r s o f f i c i a l P a g e | 35

FOR MORE UPDATES AND MORE MATERIAL DO LIKE OUR FACEBOOK PAGE httpwwwfacebookcomsscmentorsofficial

घरहॳिमकिी Correct Answer Lizard

Q147 Which type of pathogen causes

the water-borne disease SARS (Severe

Acute Respiratory Syndrome)

ककसपरकािकारह८गज़नकजिजननतबीमारीसासयकाकारणबनताहहॴ 25-Jan-2017

Options

1) Viral

वायरि

2) Parasitic

परजीवी 3) Protozoan

परह८टह८जअन

4) Bacterial

बहॴकटीररयि

Correct Answer Viral

Q148 Which of the following organs

produces the enzyme lipase

नननननलिखितमसहॳकह९नसाअगिायपहॳजएजाइमउतपननकरताहहॴ 25-Jan-2017

Options

1) Pancreas

अगनयािय

2) Large Intestine

बड़ीआत

3) Liver

नजगर

4) Small Intestine

छह८टीआत

Correct Answer Pancreas

Q149 A is a long internode forming the

basal part or the whole of a peduncle

एक mdashmdash- एकिबाइटरनह८िहहॴ जह८ननचिाटहससायासनपणयिठिबनताहहॴ 25-

Jan-2017

Options

1) Rhizome

परकद

2) Rachis

महॳ दि

3) floral axis

पषपअकष

4) Scape

भगदड़

Correct Answer scape

Q150 ndash Which of the following

organisms are considered to be both

Living and Non-living

नननननलिखितमसहॳकह९नसहॳजीवाणकह८जीपवतऔरअजीपवतमानाजाताहहॴ

25-Jan-2017

Options

1) Bacteria

बहॴकटीररया 2) Fungi

कवक

3) Algae

िहॴवाि

4)Virus

वायरस

Correct Answer Virus

They are considered to be living as they

possess a protein coat as a protective

covering DNA as the genetic material

etc

They are said to be non-living as they

can be crystallised and they survive for

billions of years They can tolerate high

temperatures freezing cold

temperatures ultra-violet radiations etc

Q151 Deficiency of fluorine causes

which of the following

फिह८ररनकीकमीकहॳ कारणनननननलिखितमसहॳकयाहह८ताहहॴ

F A C E B O O K

P A G E h t t p w w w f a c e b o o k c o m s s c m e n t o r s o f f i c i a l P a g e | 36

FOR MORE UPDATES AND MORE MATERIAL DO LIKE OUR FACEBOOK PAGE httpwwwfacebookcomsscmentorsofficial

27-Jan-2017

Options

1) Dental Caries

िटिकहॴ ररज

2) Scurvy

सकवरी 3) Anaemia

रकतामपता 4) Arthritis

गटठया Correct Answer Dental Caries

Q152 In a Punnett Square with the

cross AaBb x AaBb how many Aabb

genotypes would be created

पनहॳटसककायरमिह८स AaBb x AaBb कहॳ साि

ककतनहॳ Aabb जीनह८टाइपबनगहॳ 27-Jan-2017

Options

1) 1

2) 8

3) 2

4) 3

Correct Answer 2

Q153 Which of the following is the

Controlling Center of the Cell

नननननलिखित म सहॳ कह८लिकाका ननयतरण

क दर कह९न हहॴ

27-Jan-2017

Options

1) Nucleus

क दर

2) Plasma

पिाजमा 3) Lysosome

िायसह८सह८म

4) Chromosome

िह८मह८सह८म

Correct Answer Nucleus

The control centre of the cell is the

nucleus in eukaryotic cells The nucleus

contains genetic material in the form of

DNA

Q154 Myopia affects which of the

following organs

मायह८पपयानननननलिखितअगह८मसहॳककसहॳपरभापवतकरताहहॴ

25-Jan-2017

Options

1) Heart

हदय

2) Skin

तवचा 3) Eyes

आािहॳ 4)Mouth

मह

Correct Answer Eyes

Q155 Which of the following bears

flowers

नननननलिखितमसहॳकह९नफिधारणकरताहहॴ

25-Jan-2017

Options

1) Bryophyta

िायह८फाइटा 2) Pteridophyta

टहॳरीिह८फाईटा 3) Gymnosperms

नजननह८सपमय 4)Angiosperms

एननजयह८सपमय Correct Answer Angiosperms

Q156 Oxygenated blood flows out of the

heart through the

ऑकसीजनयकतरकत mdashmdashmdash

कहॳ माधयमसहॳहदयकहॳ बाहरबहताहहॴ 25-Jan-2017

F A C E B O O K

P A G E h t t p w w w f a c e b o o k c o m s s c m e n t o r s o f f i c i a l P a g e | 37

FOR MORE UPDATES AND MORE MATERIAL DO LIKE OUR FACEBOOK PAGE httpwwwfacebookcomsscmentorsofficial

Options

1) Aorta

महाधमनी 2) pulmonary artery

फहॳ फड़हॳकीधमनी 3) vena cava

वहॳनाकावा 4)Atrium

चह९क

Correct Answer aorta

Q157 Blood leaving the liver and

moving towards the

heart has a higher concentration of

नजगरसहॳननकिकरहदयकीतरफजानहॳवािहॳरकतम mdashmdashmdashmdash कीउचचसादरताहह८तीहहॴ 27-Jan-2017

Options

1) Lipids

लिपपडस

2) Urea

यररया 3) Bile Pigments

पपततकहॳ रगकरण

4) Carbon dioxide

काबयनिायऑकसाइि

Correct Answer Bile Pigments

Urea is nitrogen containing substance

which is produced in the liver in order

to deal with excess amino-acids in the

body As urea is produced it leaves the

liver in the blood stream and passes via

the circulatory system to all parts of the

body

Q158 Bulb is a modification of which

part of a plant

बमबएकपह९धहॳकहॳ ककसटहससहॳकाएक पातरणहह८ताहहॴ 27-Jan-2017

Options

1) The root

जड़

2) The stem

तना 3) The radicle

मिाकर

4)The fruit

फि

Correct Answer The stem

Q159 Which of the following carries

blood away from the heart to different

body parts

इनमहॳसहॳकह९नरकतकह८हदयसहॳिरीरकहॳ पवलभननअगह८तकिहॳजातीहहॴ

27-Jan-2017

Options

1) Arteries

धमननया 2) Nerves

तबतरहाए

3) Capillaries

कहॳ लिकाए

4)Veins

नसहॳ Correct Answer Arteries

Q160 The series of processes by which

nitrogen and its compounds are

interconverted in the environment and

in living organisms is called

27-Jan-2017

Options

1)Absorption of Nitrogen

2)Ammonification

3)Nitrogen Fixation

4)Nitrogen Cycle

Correct Answer Nitrogen Cycle

Ammonification or Mineralization is

performed by bacteria to convert

organic nitrogen to ammonia

F A C E B O O K

P A G E h t t p w w w f a c e b o o k c o m s s c m e n t o r s o f f i c i a l P a g e | 38

FOR MORE UPDATES AND MORE MATERIAL DO LIKE OUR FACEBOOK PAGE httpwwwfacebookcomsscmentorsofficial

Nitrification can then occur to convert

the ammonium to nitrite and nitrate

Nitrogen fixation is a process by which

nitrogen in the Earthrsquos atmosphere is

converted into ammonia (NH3) or other

molecules available to living organisms

Q161 BCG vaccine is given to protect

from which of the following

बीसीजीकाटटकानननननलिखितमसहॳककसकहॳ बचावकहॳ लिएटदयाजातहहॴ

27-Jan-2017

Options

1) Jaundice

पीलिया 2) Anaemia

रकतमपता 3) Tuberculosis

कषयरह८ग

4) Polio

पह८लियह८ Correct Answer Tuberculosis

Q162 Parallel venation is found in

समानतरवहॳनहॳिन mdashmdashmdash- मपायाजाताहहॴ 27-Jan-2017

Options

1) plants which are monocots

पह९धहॳजह८एकबीजपतरीहह८तहॳहहॴ 2) plants which have a dicot stem

वहॳपह९धहॳनजनकातनादपवदलियहह८ताहहॴ 3) plants with leaves similar to Tulsi

वहॳपह९धहॳनजनकीपनततयतिसीकीपनततयोकहॳ समानहह८तहॳहहॴ 4)plants with tap roots

टहॳप टवािहॳपह९धहॳ Correct Answer plants which are

monocots

Q163 The hardest part of the body is

िरीरकासबसहॳकठह८रभाग mdashndash हहॴ 27-Jan-2017

Options

1) Bones

हडडिय

2) Tooth Enamel

दातकहॳ इनहॳमि

3) Skull

िह८पड़ी 4) Spinal Cord

महॳ रजज

Correct Answer Tooth Enamel

Q164 Which type of pathogen causes

the waterborne disease E coli Infection

ककसपरकारकारह८गजननकजिजननतरह८गईकह८िाईसिमणकाकारणबनताहहॴ 27-Jan-2017

Options

1) Protozoan

परह८टह८जआ

2) Parasitic

परजीवी 3) Bacterial

बहॴकटीररयि

4)Viral

वायरि

Correct Answer Bacterial

Q165 The amount of blood filtered

together by both the kidneys in a 70 kg

adult male human in a minute is

70 की गरा वािहॳएकवयसकप षमएकलमनटमदह८नोगदकहॳदवाराएकसािचाबनीगयीरकतकीमातरहह८तीहहॴ 29-Jan-2017

Options

1) 1100 ml

1100 लमलि

2) 100 ml

F A C E B O O K

P A G E h t t p w w w f a c e b o o k c o m s s c m e n t o r s o f f i c i a l P a g e | 39

FOR MORE UPDATES AND MORE MATERIAL DO LIKE OUR FACEBOOK PAGE httpwwwfacebookcomsscmentorsofficial

100 लमलि

3) 1500 ml

1500 लमलि

4) 500 ml

500 लमलि

Correct Answer 1100 ml

Q166 Which feature of a plant helps to

distinguish a monocot from a dicot

पह९धहॳकीवहकह९नसीपविहॳषताहहॴजह८एकदपवदलियहॳऔरएकएकदिीयपह९धहॳसहॳभहॳदकरनहॳममददकरतीहहॴ 29-Jan-2017

Options

1) Pollination

परागम

2) Venation

वहॳनहॳिन

3) Vernation

वनिन

4) Aestivation

एसटीवहॳिहॳन

Correct Answer venation

Q167 The Mutation Theory was

proposed by

उतवररवतयनकालसदात mdashmdashndash

कहॳ दवरापरसतापवतककयाजाताहहॴ 29-Jan-2017

Options

1) Charles Lyell

चामसयलियहॳि

2) William Smith

पवलियमनसमि

3) Hugo De Vries

हयगह८िीराईस

4)Harrison Schmitt

हहॳरीसननसमट

Correct Answer Hugo De Vries

Q168 Which type of pathogen causes

the waterborne disease HepatitisA

ककसपरकारकहॳ रह८गजनकजिजननतरह८गहहॳपहॳटाइटटस-A काकारणबनताहहॴ

29-Jan-2017

Options

1) Parasitic

परजीवी 2) Viral

वायरि

3) Protozoan

परह८टह८जआ

4) Bacterial

बहॴकटीररयि

Correct Answer Viral

Q169 In a Punnett Square with the

cross AaBb x Aabb how many AaBb

genotypes would be created

पनहॳटसकवायरमिह८स AaBb x Aabb

कहॳ सािककतनहॳ AaBb जीनह८टाइपबनगहॳ 29-Jan-

2017

Options

1) 4

2) 1

3) 7

4) 6

Correct Answer 4

Q170 Arboreal Ateles is the scientific

name of

अिह८ररयिएटटलिस mdashmdashmdash कावहॴजञाननकनामहहॴ 29-Jan-2017

Options

1) Squirrel

चगिहरी 2) Sparrow

गह८रहॴया 3) Lizard

नछपकिी 4) Spider monkey

F A C E B O O K

P A G E h t t p w w w f a c e b o o k c o m s s c m e n t o r s o f f i c i a l P a g e | 40

FOR MORE UPDATES AND MORE MATERIAL DO LIKE OUR FACEBOOK PAGE httpwwwfacebookcomsscmentorsofficial

मकड़ीबदर

Correct Answer Spider monkey

Q171 Which type of pathogen causes

the waterborne disease Salmonellosis

ककसपरकारकारह८गाणजिजननतबीमारीसािमह८नहॳिह८लसज़काकारकहहॴ

29-Jan-2017

Options

1) Algal

िहॳवालियहॳ 2) Parasitic

परजीवी 3) Bacterial

बहॴकटीररयि

4)Viral

वायरि

Correct Answer Bacterial

An infection with salmonella bacteria

commonly caused by contaminated food

or water

Symptoms include diarrhoea fever

chills and abdominal pain

Q172 is a condition in which there is a

deficiency of red cells or of haemoglobin

in the blood

mdashmdash-

एकनसिनतहहॴनजसमहॳरकतमिािकह८लिकाओकीयाहीमह८गिह८बबनकीकमीहह८तीहहॴ 29-Jan-2017

Options

1) Albinism

एनमबननजम

2) Propyria

परह८पीररया 3) Anaemia

एनीलमया 4)Keloid disorder

कहॳ िह८इिडिसओिर

Correct Answer Anaemia

Q173 Ananas comosus is the scientific

name of

Options

अनानासकह८मह८सस mdashmdashmdashndash

कावहॴजञाननकनामहहॴ 29-Jan-2017

1) Custard Apple

सीताफि

2) Pineapple

पाइनएपपि

3) Bamboo

बास

4)Pomegranate

अनार

Correct Answer Pineapple

Q174 Which organ produces insulin

कह९नसाअगइनसलिनपहॴदाकरताहहॴ 29-Jan-

2017

Options

1) Liver

यकत

2) Thyroid gland

िायराइिगरिी 3) Spleen

पिीहा 4)Pancreas

अगरयिय

Correct Answer Pancreas

Q175 Which of the following disease is

not caused by water pollution

नननननलिखितमसहॳकह९नसारह८गपानीकहॳ परदषणकहॳकारणनहीहह८ता

29-Jan-2017

Options

1) Cholera

हहॴजा 2) Typhoid

F A C E B O O K

P A G E h t t p w w w f a c e b o o k c o m s s c m e n t o r s o f f i c i a l P a g e | 41

FOR MORE UPDATES AND MORE MATERIAL DO LIKE OUR FACEBOOK PAGE httpwwwfacebookcomsscmentorsofficial

टाइफाइि

3) Asthma

दमा 4)Diarrhoea

दसत

Correct Answer Asthma

Q176 Ocimum tenuiflorum is the

scientific name of

ओलिलममटहॳयईफिह८रमइसकावहॴजञाननकनाम mdash

ndash हहॴ 30-Jan-2017

Options

1) Neem

नीम

2) Mango

आम

3) Babul

बबि

4)Tulsi

तिसी Correct Answer Tulsi

Q177 Which gland secretes bile a

digestive fluid

कह९नसीगरिीपपतत एकपाचनतरिपरदािय सरापवतकरतीहहॴ 30-Jan-2017

Options

1) Pancreas

अगनयािय

2) Liver

यकत

3) Thyroid

िायराइि

4) Testes

टहॳनसटस

Correct Answer liver

Q178 In which of the following the

dominant phase is Gametophyte

नननननलिखितमसहॳककसकहॳ परमिचरणयगमकह८दपवधद (Gametophyte)हहॴ 30-Jan-2017

Options

1) Bryophyta

िायह८फाइटा 2) Pteridophyta

टहॳररिह८फाइटा 3) Gymnosperms

नजननह८सपमय 4) Angiosperms

एननजयह८सपमय Correct Answer Bryophyta

Q179 Anaerobic respiration refers to

which of the following

नननननलिखितमसहॳककसहॳअवायवीयशवसनकहाजाताहहॴ

30-Jan-2017

Options

1) Respiration without Oxygen

ऑकसीजनकहॳ बबनाशवसन

2) Respiration with Oxygen

ऑकसीजनकहॳ सािशवसन

3) Respiration without CO2

काबयनिायऑकसाइिकहॳ बबनाशवसन

4) Respiration with CO2

काबयनिायऑकसाइिकहॳ सािशविन

Correct Answer Respiration without

Oxygen

Q180 Which type of pathogen causes

the waterborne disease Cholera

ककसपरकारकारह८गजनकजिजननतरह८गहहॴजाकाकारणबनताहहॴ

30-Jan-2017

Options

1) Algal

िहॴवालियहॳ

F A C E B O O K

P A G E h t t p w w w f a c e b o o k c o m s s c m e n t o r s o f f i c i a l P a g e | 42

FOR MORE UPDATES AND MORE MATERIAL DO LIKE OUR FACEBOOK PAGE httpwwwfacebookcomsscmentorsofficial

2) Bacterial

बहॴकटीररयि

3) Protozoan

परह८टह८जआ

4) Viral

वायरि

Correct Answer Bacterial

Q181 To which class does

Oxyreductases transferases hydrolases

belong

ओकसीररिकटहॳसटरासफरहॳजहॳस

हाइडरह८िहॳसहॳसककसवगयमआतहॳहहॴ 30-Jan-2017

Options

1) Hormones

हारमोस

2) Enzymes

एजाइनस

3) Proteins

परह८टीनस

4) Vitamins

पवटालमनस

Correct Answer Enzymes

Q182 Which of the following is not true

about Gymnosperms

ननननमसहॳकह९नसीबातअनावतबीजीकहॳ बारहॳमसचनहीहहॴ 30-Jan-2017

Options

1) Dominant phase is saprophytes

परमिचरणसहॳपरह८फाइटसहह८ताहहॴ 2) Vascular bundles are absent

सवहनीबििअनपनसितहह८ताहहॴ 3) spores are heterospores

बीजाणहहॳटहॳरह८सपह८रसहह८तहॳहहॴ 4) Flowers are absent

फिअनपनसितहह८तहॳहहॴ

Correct Answer Vascular bundles are

absent

Q183 The name of first mammal clone sheep is

भहॳड़कीपरिमसतनपायीपरनत प (किह८न)

कानामहहॴ 30-Jan-2017

Options

1) Noori

नरी 2) Dolly

िॉिी 3) Louise

िसी 4)Durga

दगाय Correct Answer Dolly

Q184 Which type of pathogen causes

the water-borne disease Typhoid fever

ककसपरकारकारह८गजनकजिजननतरह८गटाइफाइिबिारकाकारणबनताहहॴ 30-Jan-2017

Options

1) Algal

िहॴवािीय

2) Parasitic

परजीवी 3) Protozoan

परह८टह८जनअन

4)Bacterial

बहॴकटीररयि

Correct Answer Bacterial

Q185 In which part of the cell are

proteins made

कह८लिकाकहॳ ककसटहससहॳमपरह८टीनबनायाजाताहहॴ

31-Jan-2017

Options

1) Reticulum

रहॳटटकिम

F A C E B O O K

P A G E h t t p w w w f a c e b o o k c o m s s c m e n t o r s o f f i c i a l P a g e | 43

FOR MORE UPDATES AND MORE MATERIAL DO LIKE OUR FACEBOOK PAGE httpwwwfacebookcomsscmentorsofficial

2) Golgi apparatus

गह८मजीएपहॳरहॳटस

3) Ribosomes

ररबह८सह८नस

4) Lysosome

िायसह८सह८नस

Correct Answer ribosomes

Proteins are produced by stringing

amino acids together in the order

specified by messenger RNA strands

that were transcribed from DNA in the

cell nucleus The process of synthesizing

a protein is called translation and it

occurs on ribosomes in the cytoplasm of

a cell

Q186 Polio is a disease caused by which

of the following

नननननलिखितमसहॳपह८लियह८कीबबमारह८हह८नहॳकाकारणकयाहहॴ

31-Jan-2017

Options

1) Bacteria

बहॴकटीररयि

2) Mosquito

मचछर

3) Virus

वायरस

4) Cockroach

नतिच हॳ Correct Answer Virus

Polio or poliomyelitis is a crippling and

potentially deadly infectious disease It

is caused by the poliovirus

Q187 ndash Hay fever is a sign of which of

the following

हहॳकफवरनननननलिखितमसहॳककसकाएकसकहॳ तहहॴ

31-Jan-2017

Options

1) Old Age

वदावसिा 2) Malnutrition

कपह८सण

3) Allergy

एिनजय 4) Over Work

अतयचधककाययकरना Correct Answer Allergy

Q188 How many chromosomes does a

human cell contain

एकमानवकह८लिकामककतनहॳगणसतरहह८तहॳहहॴ

29-Jan-2017

Options

1) 6

2) 26

3) 46

4) 66

Correct Answer 46

In humans each cell normally contains

23 pairs of chromosomes for a total of

46 Twenty-two of these pairs called

autosomes look the same in both males

and females The 23rd pair the sex

chromosomes differ between males and

females

Q189 Which of the following is not true

about Bryophyta

ननननमसहॳकह९नसीबातिायह८फाइटकहॳ बारहॳमसचनहीहहॴ 31-Jan-2017

Options

1) Dominant phase is gametophytes

परमिचरणगहॳलमतह८फाइटसहह८ताहहॴ 2) Main plant body is haploid

पह९धहॳकामखयिरीरअगखणतहह८ताहहॴ 3) Spores are homospores

बीजाणहह८मह८सफह८रसहह८तहॳहहॴ 4) Flowers are present

फिमह८जदहह८तहॳहहॴ Correct Answer Flowers are present

F A C E B O O K

P A G E h t t p w w w f a c e b o o k c o m s s c m e n t o r s o f f i c i a l P a g e | 44

FOR MORE UPDATES AND MORE MATERIAL DO LIKE OUR FACEBOOK PAGE httpwwwfacebookcomsscmentorsofficial

Q190 Which aquatic animal has

trailing tentacles

ककसजिीयजानवरकहॳ पीछहॳचिनहॳवािहॳटहॳटकिसहह८तहॳहहॴ

31-Jan-2017

Options

1) Sea horse

समदरीघह८िा 2) Corals

मगा 3) Jelly fish

जहॳिीमछिी 4) Star fish

तारामछिी Correct Answer Jelly fish

Jellyfish with its umbrella-shaped bell

and trailing tentacles

Q191 Which type of pathogen causes

the water-borne disease Poliomyelitis

(Polio)

ककसपरकारकारह८गजनकजिजननतरह८गपह८लियह८मायहॳटटस (पह८लियह८) काकारणहहॴ 31-Jan-

2017

Options

1) Parasitic

परजीवी 2) Algal

िहॴवालिय

3) Viral

वायरि

4) Bacterial

बहॴकटीररयि

Correct Answer Viral

Q192 The outer white part of the eye

that protects the inner structures is

आािकाबाहरीसफहॳ दटहससाजह८आतररकसरचनाओकीरकषाकरताहहॴ वह mdashmdashmdash हहॴ 31-Jan-

2017

Options

1) Iris

आयररस

2) Sclera

सकिहॳरा 3) Retina

रहॳटटना 4) Cornea

कह८ननयया Correct Answer Sclera

Q193 Proteins are made up of

परह८टीनकाननमायण mdashndash सहॳहह८ताहहॴ 31-Jan-2017

Options

1) Amino acids

एलमनह८अनि

2) Fatty acids

वसायकतअनि

3) Glucose

गिकह८ज

4)Nucleotides

नयनकियह८टाईिस

Correct Answer Amino acids

Q194 Moringa Oleifera is the scientific

name of

मह८ररगओलिफहॳ रा mdashmdashndash कावहॴजञाननकनामहहॴ 31-Jan-2017

Options

1) Banyan

बरगद

2) Gulmohar

गिमह८हर

3) Amla

आमिा

F A C E B O O K

P A G E h t t p w w w f a c e b o o k c o m s s c m e n t o r s o f f i c i a l P a g e | 45

FOR MORE UPDATES AND MORE MATERIAL DO LIKE OUR FACEBOOK PAGE httpwwwfacebookcomsscmentorsofficial

4) Drumstick

डरमनसटक

Correct Answer Drumstick

Q195 Kidney stones are composed of

गदकीपिरी mdashndash सहॳबनीहह८तीहहॴ 1-Feb-2017

Options

1) Calcium Oxalate

कहॴ नमसयमओकजहॳिहॳट

2) Sodium Chloride

सह८डियमकिह८राइि

3) Magnesium Nitrate

महॳनगनलियमनाइतटरहॳट

4) Calcium Bicarbonate

कहॴ नमियमबायकबोनहॳट

Correct Answer Calcium Oxalate

Q196 ndash Which of the following is not

true about Angiosperms

ननननमसहॳकह९नसीबातआवतबीजीकहॳ बारहॳमसचनहीहहॴ 1-Feb-2017

Options

1) Dominant phase is gametophytes

परमिचरणगहॳलमतह८फाइटहह८ताहहॴ 2) Vascular bundles are present

सवहनीबििमह९जदहह८ताहहॴ 3) Spores are heterospores

बीजाणहहॳटहॳरह८सपह८रसहह८तहॳहहॴ 4) Seeds are covered

बीजढकहॳ हह८तहॳहहॴ Correct Answer Dominant phase is

gametophytes

Q197 All of the following are excretory

(waste) products of animals except

नननननलिखितमसहॳककसएककह८छह८ड़करअनयसभीपराखणयोदवाराउतसनजयतपदाियहहॴ 1-Feb-

2017

Options

1) Uric Acid

यररकएलसि

2) Ammonia

अमह८ननया 3) Carbohydrates

काबोहाइडरहॳट

4) Urea

यररया Correct Answer Carbohydrates

In animals the main excretory products

are carbon dioxide ammonia (in

ammoniotelics) urea (in ureotelics) uric

acid (in uricotelics) guanine (in

Arachnida) and creatine

Q198 RNA is a polymeric molecule

What does RNA stand for

आरएनइएएकबहिकआणहहॴ इसकाकापवय पकयाहहॴ 1-Feb-2017

Options

1) Rado Nuclear Acid

रािह८नयनकियरएलसि

2) Ribo Nucleic Acid

राइबह८नयनकिकएलसि

3) Rhino Nuclear Acid

हाइनह८नयनकियरएलसि

4) Resto Nucleus Acid

रहॳसटह८नयकिीयसएलसि

Correct Answer Ribo Nucleic Acid

Q199 Which organ does detoxification

and produces chemicals needed for

digestion

कह९नसाअगपवषहरणकरताहहॴऔरपाचनकहॳ लिएआवशयकरसायनोकह८पहॴदाकरताहहॴ 1-Feb-

2017

Options

1) Salivary glands

िारगरचिया 2) Pancreas

अगनयािय

F A C E B O O K

P A G E h t t p w w w f a c e b o o k c o m s s c m e n t o r s o f f i c i a l P a g e | 46

FOR MORE UPDATES AND MORE MATERIAL DO LIKE OUR FACEBOOK PAGE httpwwwfacebookcomsscmentorsofficial

3) Thyroid gland

िायराइिगरिी 4) Liver

यकत

Correct Answer Liver

Q200 Psidium guajava is the scientific

name of

लसडियमगआजावा mdashmdash कावहॴजञाननकनामहहॴ 1-

Feb-2017

Options

1) Guava

अम द

2) Mango

आम

3) Bamboo

बास

4) Jack fruit

कटहि

Correct Answer Guava

Q201 Which drug is used as a Blood

Thinner

चधरकह८पतिाकरनहॳकहॳ पमककसदवाकापरयह८गककयाजाताहहॴ

1-Feb-2017

Options

1) Warfarin

वाफर न

2) Tramadol

टरहॳमािह८ि

3) Azithromycin

एनजरह८मायलसन

4) Hydralazine

हाइडरह८िहॳनजन

Correct Answer Warfarin

Q202 Which of the following disease is

caused due to the deficiency of protein

परह८टीनकीकमीकहॳ कारणनननननलिखितमसहॳकह९नसारह८गहह८ताहहॴ 1-Feb-2017

Options

1) Arthritis

गटठया 2) Kwashiorkor

कािीओकय र

3) Goitre

गाइटर

4) Night Blindness

रतह९चध

Correct Answer Kwashiorkor

Q203 A is species of plant that has

adapted to survive in an environment

with little liquid water

mdashmdashndashपह९धहॳकीएकऐसहॳऐसहॳपरजानतहहॴ नजसनहॳकमपानीवािहॳवातावरणमजीपवतरहनहॳकहॳलिएअनकिनहहॴ 1-Feb-2017

Options

1) Xerophyte

म दपवद

2) Hydrophyte

जिीयपादप

3) Mesophyte

समह८दपवद

4) Thallophyte

िहॴिह८फाइटा Correct Answer xerophyte

xerophyte is a species of plant that has

adapted to survive in an environment

with little liquid water such as a desert

or an ice- or snow-covered region in the

Alps or the Arctic

Mesophytes are terrestrial plants which

are adapted to neither a particularly

dry nor particularly wet environment

An example of a mesophytic habitat

would be a rural temperate meadow

F A C E B O O K

P A G E h t t p w w w f a c e b o o k c o m s s c m e n t o r s o f f i c i a l P a g e | 47

FOR MORE UPDATES AND MORE MATERIAL DO LIKE OUR FACEBOOK PAGE httpwwwfacebookcomsscmentorsofficial

which might contain goldenrod clover

oxeye daisy and Rosa multiflora

thallophyte any of a group of plants or

plantlike organisms (such as algae and

fungi) that lack differentiated stems

leaves and roots and that were formerly

classified as a primary division

(Thallophyta) of the plant kingdom

Q204 How many types of teeth are

there in humans

मनषयोमककतनहॳपरकारकहॳ दातहह८तहॳहहॴ

1-Feb-2017

Options

1) 4

2) 5

3) 2

4) 3

Correct Answer 4

teeth -Humans have four types of

teethincisors canines premolars and

molars each with a specific function

The incisors cut the food the canines

tear the food and the molars and

premolars crush the food

Q205 Carica papaya is the scientific name of

कहॴ ररकापपाया mdashmdashndash कावहॴजञाननकनामहहॴ 2-

Feb-2017

Options

1) Peepal

पीपि

2) Papaya

पपीता 3) Tamarind

इमिी 4) Drumstick

ढह८िकाछड़ी Correct Answer Papaya

Q206 Muscles get tired when there is

shortfall of

जब mdashndash कीकमीहह८तीहहॴतबपहॳिीयिकजातीहहॴ 2-Feb-2017

Options

1) Lactic acid

िहॴनकटकएलसि

2) Na+ ions

Na+ आयन

3) ATP

एटीपी 4) Sulphates

समफहॳ टस

Correct Answer ATP

ATP is the energy source muscle fibers

use to make muscles contract

muscle tissuersquos main source of energy

called adenosine triphosphate or ATP

As your muscles use up this energy

source they become tired and fatigued

Oxygen is the key ingredient that helps

create new ATP to replenish the burned

up ATP in your muscles

Q207 Artocarpus integra is the

scientific name of आटह८कापयसइटीगरा mdashmdashmdash कावहॴजञाननकनामहहॴ 2-Feb-2017

Options

1) Guava

अम द

2) Pineapple

अनानास

3) Silver Oak

लसमवरओक

4) Jack fruit

कटहि

Correct Answer Jack fruit

Q208 Which organ stores fat soluble

vitamins

कह९नसाअगवसामघिनिीिपवटालमनह८काभिाराकरताहहॴ

2-Feb-2017

F A C E B O O K

P A G E h t t p w w w f a c e b o o k c o m s s c m e n t o r s o f f i c i a l P a g e | 48

FOR MORE UPDATES AND MORE MATERIAL DO LIKE OUR FACEBOOK PAGE httpwwwfacebookcomsscmentorsofficial

Options

1) Blood

रकत

2) Skin

तवचा 3) Liver

यकत

4) Pancreas

अगनयािय

Correct Answer Liver

Q209 Which disease is caused due to

deficiency of Iodine

आयह८िीनकहॳ कारणकह९नसारह८गहह८ताहहॴ 2-Feb-2017

Options

1) Rickets

ररकहॳ टस

2) Scurvy

सकवी 3) Goitre

गणमािा 4) Growth retardation

पवकासका कना Correct Answer Goitre

rickets A softening and weakening of

bones in children usually due to

inadequate vitamin D

Q210 Grevillea Robusta is the scientific name of

गरहॳपवलियारह८बसटा mdashmdashmdash- कापवजञाननकनामहहॴ 2-Feb-2017

Options

1) Peepal

पीपि

2) Teak

सागह९न

3) Silver Oak

लसमवरओक

4) Jack fruit

कटहि

Correct Answer Silver Oak

Q211 When a Cuttlefish is described as a Molluscs it is at which level of

classification

जबएककटिकफिकह८एकमह८िसकाकहॳ पमवखणयतककयाजाताहहॴतबयहॳवगीकरणकहॳ ककससतरपहॳनसितहहॴ 2-Feb-2017

Options

1) Class

वगय 2) Order

िम

3) Family

पररवार

4) Phylum

सघ

Correct Answer Phylum

Q212 Bambusa dendrocalmus is the

scientific name of बानबसािहॳडराकामस mdashmdashmdash कावहॴजञाननकनामहहॴ 3-Feb-2017

Options

1) Banyan

बरगद

2) Papaya

पपीता 3) Bamboo

बास

4) Pomegranate

अनार

Correct Answer Bamboo

Q213 Acinonyx Jubatus is the scientific name of

एलसनह८ननकसजयबहॳटस mdashmdashmdash

कावहॴजञाननकनामहहॴ 3-Feb-2017

F A C E B O O K

P A G E h t t p w w w f a c e b o o k c o m s s c m e n t o r s o f f i c i a l P a g e | 49

FOR MORE UPDATES AND MORE MATERIAL DO LIKE OUR FACEBOOK PAGE httpwwwfacebookcomsscmentorsofficial

Options

1) Bear

भाि 2) Horse

घह८िा 3) Cheetah

चीता 4) Zebra

जहॳिा Correct Answer Cheetah

Q214 The pale yellow colour of urine is

due to the presence of which pigment

मतरकाफीकापीिारगरगदरयकहॳ उपनसिनतकहॳ कारणहह८ताहहॴ

3-Feb-2017

Options

1) Urochrome

यरह८िह८म

2) Urophyll

यरह८कफि

3) Chlorophyll

किह८रह८कफि

4) Chloroplast

किह८रह८पिासट

Correct Answer Urochrome

Q215 Which of the following constitute

to form a gene

नननननलिखितमसहॳकह९नसीचीज़एकजीनकागठनकरतीहहॴ

3-Feb-2017

Options

1) Polynucleotides

पह८िीनयनकियह८टाईडस

2) Hydrocarbons

हाइडरह८काबोस

3) Lipoproteins

िाईपह८परह८टीनस

4) Lipids

लिपपडस

Correct Answer Polynucleotides

Polynucleotide molecule is a biopolymer

composed of 13 or more nucleotide

monomers covalently bonded in a chain

DNA (deoxyribonucleic acid) and RNA

(ribonucleic acid) are examples of

polynucleotides with distinct biological

function

Q216 Vertebrates belongs to the

phylum

रीढ़कीहडिीवािहॳपराणी mdashmdashmdash

परजानतकहॳ अतगायतआतहॳहहॴ 3-Feb-2017

Options

1) Arthropoda

आरह८पह८ड़ा 2) Annelida

एननलििा 3) Cnidaria

ननिहॳररया 4) Chordata

कह८िटा Correct Answer Chordata

Q217 Punica granatum is the scientific name of

पननकगरहॳनहॳटस mdashmdashmdash कावहॴजञाननकनामहहॴ 3-Feb-2017

Options

1) Custard Apple

सीताफि

2) Gulmohar

गिमह८हर

3) Silver Oak

लसमवरओक

4) Pomegranate

अनार

Correct Answer Pomegranate

F A C E B O O K

P A G E h t t p w w w f a c e b o o k c o m s s c m e n t o r s o f f i c i a l P a g e | 50

FOR MORE UPDATES AND MORE MATERIAL DO LIKE OUR FACEBOOK PAGE httpwwwfacebookcomsscmentorsofficial

Q218 Between a tiger and an monkey

which of the following is different

एकबाघऔरबदरकहॳ बीचनननननलिखितमसहॳकह९नसीबातअिगहहॴ 3-Feb-2017

Options

1) Kingdom

राजय

2) Phylum

जानत

3) Order

िम

4) Class

वगय Correct Answer order

Q219 The artificial heart was invented by

कबतरमहदयका mdashmdashmdash

दवाराअपवषकारककयागयािा 3-Feb-2017

Options

1) Muhammad Yunus

महनमदयनस

2) Linus Yale Jr

िाइनसयहॳिजय

3) Gazi Yasargil

गाजीयासचगयि

4) Paul Winchell

पह९िपवमकि Correct Answer Paul Winchell

Q220 Tamarindus indica is the

scientific name of

टहॳमररनडसइडिका mdashmdash कावहॴजञाननकनामहहॴ 7-

Feb-2017

Options

1) Neem

नीम

2) Pineapple

अनानास

3) Tamarind

इमिी 4)Chiku

चीक

Correct Answer Tamarind

Q221 In eukaryotic cells synthesis of

RNA takes place in the

यकहॳ योटटककह८लिकाओमआरएनएकासशिहॳषण

mdashndash महह८ताहहॴ 7-Feb-2017

Options

1) Mitochondria

माईटह८कोडडरया 2) Centrioles

सटरीयह८मस

3) Ribosomes

ररबह८सह८नस

4) Nucleus

नयनकियस

Correct Answer nucleus

eukaryotic cell -Transcription is the

process of synthesizing ribonucleic acid

(RNA)Synthesis takes place within the

nucleus of eukaryotic cells or in the

cytoplasm of prokaryotes and converts

the genetic code from a gene in

deoxyribonucleic acid ( DNA ) to a

strand of RNA that then directs

proteinsynthesis

Q222 _________is caused by parasites

of the Plasmodium genus

पिाजमह८डियमजातीकहॳ परजीवी mdash- कहॳ कारणहहॴ 7-Feb-2017

Options

1) Dysentery

पहॳचचि

2) Malaria

मिहॳररया 3) Chickenpox

F A C E B O O K

P A G E h t t p w w w f a c e b o o k c o m s s c m e n t o r s o f f i c i a l P a g e | 51

FOR MORE UPDATES AND MORE MATERIAL DO LIKE OUR FACEBOOK PAGE httpwwwfacebookcomsscmentorsofficial

चहॳचक

4) Herpes

हहॳपपयस

Correct Answer Malaria

Q223 Carotene in fruits and vegetables

gives it which color

फिह८औरसनलजयोमनसितकहॳ रह८टीनउनहकह९नसारगपरदानकरताहहॴ 7-Feb-2017

Options

1) Green

हरा 2) Pink

गिाबी 3) Orange

नारगी 4) Blue

नीिा Correct Answer Orange

Q224 Equus Caballus is the scientific

name of

एकवसकहॴ बहॳिस mdashmdashndash कापवजञाननकनामहहॴ 7-Feb-2017

Options

1) Horse

घह८िा 2) Zebra

ज़हॳिा 3) Donkey

गधा 4) Buffalo

भस

Correct Answer Horse

Q225 Elapidae Naja is the scientific name of

एिीपीिीनाजा mdashmdash- कावहॴजञाननकनामहहॴ 8-Feb-2017

Options

1) Cobra

कह८बरा 2) Elephant

हािी 3) Eagle

ग ि

4) Owl

उमि Correct Answer Cobra

Q226 Which disease is caused due to

deficiency of Iron

िह८हकीकमीकहॳ कारणकह९नसारह८गहह८ताहहॴ 8-Feb-

2017

Options

1) Beriberi

बहॳरीबहॳरी 2) Tetany

टहॳटनी 3) Kwashiorkor

कवािीऔरकर

4) Anaemia

रकतामपता Correct Answer Anaemia

Beriberi is a disease caused by a vitamin

B-1 deficiency also known as thiamine

deficiency

Tetany can be the result of an

electrolyte imbalance Most often itrsquos a

dramatically low calcium level also

known as hypocalcemia Tetany can also

be caused by magnesium deficiency or

too little potassium Having too much

acid (acidosis) or too much alkali

(alkalosis) in the body can also result in

tetany

Kwashiorkor also known as

ldquoedematous malnutrition It is a form of

malnutrition caused by a lack of protein

in the diet

Anaemia means that you have fewer red

blood cells than normal or you have less

F A C E B O O K

P A G E h t t p w w w f a c e b o o k c o m s s c m e n t o r s o f f i c i a l P a g e | 52

FOR MORE UPDATES AND MORE MATERIAL DO LIKE OUR FACEBOOK PAGE httpwwwfacebookcomsscmentorsofficial

haemoglobin than normal in each red

blood cell

Q227 is a leaf where the leaflets are

arranged along the middle vein

mdashndashएकपततीहहॴजहापतरकह८कीरचनाक ररयालिराकहॳ आसपासहह८तीहहॴ 8-Feb-2017

Options

1) Pinnately compound leaf

पपनहॳटिीसयकतपतती 2) Palmately compound leaf

पामहॳटिीसयकतपतती 3) Compound leaf

सयकतपतती 4) Simple leaf

साधारणपतती Correct Answer Pinnately compound

leaf

Q228 Haustoria or sucking roots are

found in which of the following

हह८सटह८ररयायाचसनहॳवािीजड़हॳनननननलिखितमसहॳककसमपाईजातीहहॴ 8-Feb-2017

Options

1) Wheat

गहॳह

2) Mango

आम

3) Chestnut

चहॳसटनट

4) Cuscuta

कसकयटा Correct Answer Cuscuta

Haustorial roots -The roots of parasitic

plants which penetrate into the host

tissues to absorb nourishment are

called haustorial roots hellip Also known as suckingor parasitic roots

Q229 Equs Asinus is the scientific name

of

एकवसएलसनस mdashmdashndash कावहॴजञाननकनामहहॴ 8-

Feb-2017

Options

1) Donkey

गधा 2) Cow

गाय

3) Deer

टहरन

4) Kangaroo

कगा

Correct Answer Donkey

Q230 Ficus benghalensis is the scientific name of

फाईकसबहॳनगहॳिहॳलसस mdashndash कापवजञाननकनामहहॴ 8-Feb-2017

Options

1) Banyan

बरगद

2) Pineapple

अनानास

3) Babul

बबि

4) Tulsi

तिसी Correct Answer Banyan

Q231 Equus burchellii is the scientific name of

एकवसबचिी mdashmdash- कापवजञाननकनामहहॴ 8-Feb-2017

Options

1) Horse

घह८िा 2) Zebra

जहॳिा 3) Buffalo

F A C E B O O K

P A G E h t t p w w w f a c e b o o k c o m s s c m e n t o r s o f f i c i a l P a g e | 53

FOR MORE UPDATES AND MORE MATERIAL DO LIKE OUR FACEBOOK PAGE httpwwwfacebookcomsscmentorsofficial

भस

4) Ass

गधा Correct Answer Zebra

Page 24: COMPILATION OF ALL 72 SETS OF BIOLOGY SSC CHSL-2016 · OF BIOLOGY SSC CHSL-2016 PREPARED BY : SSC MENTORS BIOLOGY SPECIAL . F A C E B O O K P A G E : h t t p : / / w w w . f a c e

F A C E B O O K

P A G E h t t p w w w f a c e b o o k c o m s s c m e n t o r s o f f i c i a l P a g e | 23

FOR MORE UPDATES AND MORE MATERIAL DO LIKE OUR FACEBOOK PAGE httpwwwfacebookcomsscmentorsofficial

Correct Answer Antonie Philips van

Leeuwenhoek

Q95 For the aquatic organisms the

source of food is

जिीयजीवाणकािाघसरह८तहहॴ 20-Jan-2017

Options

1) Phytoplankton

फायटह८पिहॳकटन

2) Sea Weed

समदरीिहॴवाि

3)Aqua plankton

एकवापिहॳकटन

4) Zooplankton

जपिहॳकटन

Correct Answer Phytoplankton

Q96 Haemoglobin has the highest

affinity with which of the following

हीमह८गिह८बबनकीननननमसहॳककसकहॳ सािउततमसमानताहहॴ

20-Jan-2017

Options

1)SO2

2)CO2

3)CO

4)NO2

Correct Answer CO

It has a greater affinity for hemoglobin

than oxygen does It displaces oxygen

and quickly binds so very little oxygen

is transported through the body cells

Q97 Who developed the theory of

Evolution

उदपवकासकालसदातककसनहॳपवकलसतककया

20-Jan-2017

Options

1) Charles Darwin

चामसयिापवयन

2) Isaac Newton

आयजहॳकनयटन

3) Pranav Mistry

परणवलमसतरी 4) Galileo Galilei

गहॳलिलियह८गहॳिीिी Correct Answer Charles Darwin

Q98 The primary function of RNA is

RNA कापरािलमककाययहह८ताहहॴ 20-Jan-2017

Options

1) Photosynthesis

परकािसशिहॳषण

2) Protein Synthesis

परह८टीनसशिहॳषण

3) Replication

परनतकनतबनाना 4) Translation

अनवादकरना Correct Answer Protein Synthesis

There are two main functions of RNA

It assists DNA by serving as a messenger

to relay the proper genetic information

to countless numbers of ribosomes in

your body The other main function of

RNA is to select the correct amino acid

needed by each ribosome to build new

proteins for your body

Q99 ______is the movement of

molecules across a cell membrane from

a region of their lower concentration to

a region of their higher concertration

उचचसादरताकहॳ कषहॳतरसहॳउसकीकमसादरतावािहॳकषहॳतरकीतरफएककह८लिकाखझमिीकहॳ माधयमसहॳहह८नहॳवािाअणओकहॳ सचिनकह८ mdash- कहतहॳहहॴ Options

1) Diffusion

पवसरण

2) Osmosis

ऑसमह८लसस

F A C E B O O K

P A G E h t t p w w w f a c e b o o k c o m s s c m e n t o r s o f f i c i a l P a g e | 24

FOR MORE UPDATES AND MORE MATERIAL DO LIKE OUR FACEBOOK PAGE httpwwwfacebookcomsscmentorsofficial

3) Active Transport

सकियआवागमन

4) Passive Transport

नननषियआवागमन

Correct Answer Active Transport

Q100 Study of classification of

organisms is known as 20-Jan-2017

जीवाणओकहॳ वगीकरणकहॳ अधययनकह८ mdash-

कहाजाताहहॴ Options

1) Serpentology

सपरहॳटह८िह८जी 2) Virology

वायरह८िह८जी 3) Taxonomy

टहॴकसोनह८मी 4) Physiology

कफनज़यह८िह८जी Correct Answer Taxonomy

Q101 Photosynthesis takes place inside

plant cells in

परकािसशिहॳषणवनसपनतकह८लिकामनसति mdash

mdashmdash महह८ताहहॴ 20-Jan-2017

Options

1) Ribosomes

राइबह८सह८नस

2) Chloroplasts

किह८रह८पिासट

3) Nucleus

नयकलियम

4) Mitochondria

माईटह८कोडडरया Correct Answer Chloroplasts

Q102 ______ is the cell organelle in

which the biochemical processes of

respiration and energy production

occur

mdashmdash- वहकह८लिकाअगहहॴ नजसमहॳशवसनऔरउजायउतपादनकहॳ जहॴसीजहॴवरासायननकपरकियायहह८तीहहॴ 20-Jan-2017

Options

1) Mitochondria

माइटह८कोडडरया 2) Chloroplast

किह८रह८पिासट

3) Ribosomes

राइबह८सह८नस

4) Nucleus

नयकिीयस

Correct Answer Mitochondria

Q103 Which non-flowering spore

bearing plants have roots

ककसफिनिगनहॳवािहॳऔरबीजाणधारकपह९धह८कीजड़हॳहह८तीहहॴ 21-Jan-2017

Options

1) Mosses

मह८सहॳस

2) Angiosperms

एननजयह८सपनसय 3) Ferns

फनसय 4) Gymnosperms

नजननह८सपनसय Correct Answer ferns

Q104 Which of the following is an

excretory organ of cockroach

नननननलिखितमसहॳकह९नसानतिच हॳकाउतसजयनअगहहॴ

21-Jan-2017

Options

F A C E B O O K

P A G E h t t p w w w f a c e b o o k c o m s s c m e n t o r s o f f i c i a l P a g e | 25

FOR MORE UPDATES AND MORE MATERIAL DO LIKE OUR FACEBOOK PAGE httpwwwfacebookcomsscmentorsofficial

1) Malphigian Tubules

मनमफनजयनटयबमस

2) Nephridia

नहॳकफरडिया 3) Coxal Gland

कह८कसिगरचिया 4) Green Gland

गरीनगरचिया Correct Answer Malphigian Tubules

Q105 Evaporation of water takes place

in which part of plants

पानीकहॳ वाषपीकरणकीकियापह९धोकहॳ ककसभागसहॳहह८तीहहॴ 21-Jan-2017

Options

1) Stem

तना 2) Stomata

सटह८मटा 3) Branch

िािाए

4) Fruit

फि

Correct Answer Stomata

Evaporation accounts for the movement

of water to the air from sources such as

the soil canopy interception and

waterbodies Transpiration accounts for

the movement of water within a plant

and the subsequent loss of water as

vapour through stomata in its leaves

Q106 A is the fleshy spore-bearing

fruiting body of a fungus

mdashmdashndashकवककामासि

बीजाणधारणकरनहॳवािाफिनहॳवािाअगहहॴ 21-

Jan-2017

Options

1) aloe vera

एिह८वहॳरा 2) Coral

मगा 3) Cactus

कहॴ कटस

4) Mushroom

ककरमतता Correct Answer mushroom

Q107 Which of the following is a fungal

disease

नननननलिखितमसहॳकह९नसाफफदसहॳहह८नहॳवािाएकरह८ग हहॴ

21-Jan-2017

Options

1) Dermatitis

तवचािह८ध

2) Cholera

हहॴजा 3) Jaundice

पीलिया 4) Indigofera

इननिगह८फहॳ रा Correct Answer Dermatitis

Dermatitis also known as eczema is a

group of diseases that results in

inflammation of the skin These diseases

are characterized by itchiness red skin

and a rash In cases of short duration

there may be small blisters while in

long-term cases the skin may become

thickened

Q108 In which form is glucose stored in

our body

हमारहॳिरीरमगिकह८जकासचयककस पमककयाजाताहहॴ

21-Jan-2017

Options

1) Insulin

F A C E B O O K

P A G E h t t p w w w f a c e b o o k c o m s s c m e n t o r s o f f i c i a l P a g e | 26

FOR MORE UPDATES AND MORE MATERIAL DO LIKE OUR FACEBOOK PAGE httpwwwfacebookcomsscmentorsofficial

इसलिन

2) Glucose

गिकह८ज

3) Glycogen

गिायकह८जहॳन

4) Fat

वसा Correct Answer Glycogen

Excess glucose is stored in the liver as

the large compound called glycogen

Glycogen is a polysaccharide of glucose

but its structure allows it to pack

compactly so more of it can be stored in

cells for later use

Q109 Where do plants synthesize

protein from

पह९धहॳपरह८टीनसशिहॳषणकहासहॳकरतहॳहहॴ

Options

1) Fatty Acids

वसाऐलसि

2) Sugar

िकर

3) Amino Acids

एलमनह८ऐलसि

4) Starch

सटाचय Correct Answer Amino Acids

Q110 Which part of the brain is

responsible for triggering actions like

thinking intelligence memory and

ability to learn

मनसतषककाकह९नसाटहससासह८चनहॳ बनधदमानी याददाशतऔरसीिनहॳकीकषमताजहॴसीकियाओकह८परहॳररतकरताहहॴ 21-Jan-2017

Options

1) Diencephalon

िायएनसहॳफहॳ िह८न

2) Hypothalamus

हयपह८िहॳिहॳमस

3) Cerebrum

सहॳरहॳिम

4) Control

कटरह८ि

Correct Answer Cerebrum

Q111 Which of the following is also

known as the Biochemical Laboratory

of the Human Body

नननननलिखितमसहॳककसहॳमानविरीरकीजहॴवरसायनपरयह८गिािाभीकहाजाताहहॴ 21-Jan-2017

Options

1) Small Intestine

छह८टीआत

2)Brain

मनसतषक

3) Pancreas

अगनयािय

4) Liver

नजगर

Correct Answer Liver

The liver makes bile that will help

emulsify and digest the fats we eat

The liver takes toxic substances and

convert them using enzymes the liver

cells makes into a non toxic form so the

body can dispose of them

The liver also converts fats protein and

carbohydrates into glucose which is the

energy source for our cells to use

The liver takes amino acids and makes

proteins by combining them

Q112 The yellow colour of human urine

is due to

मानवमतरकापीिारग mdashndash कीवजहसहॳहह८ताहहॴ 22-

Jan-2017

Options

1) Bile Salts

F A C E B O O K

P A G E h t t p w w w f a c e b o o k c o m s s c m e n t o r s o f f i c i a l P a g e | 27

FOR MORE UPDATES AND MORE MATERIAL DO LIKE OUR FACEBOOK PAGE httpwwwfacebookcomsscmentorsofficial

पपततनमक

2) Cholesterol

कह८िहॳसटरह८ि

3) Lymph

लिनफ

4) Urochrome

यरह८िह८म

Correct Answer Urochrome

Urobilin or urochrome is the chemical

primarily responsible for the yellow

color of urine

Q113 The wilting of plants takes place

due to

पह९धह८कालिचििहह८नाकी mdashmdash- कीवजहसहॳहह८ताहहॴ 22-Jan-2017

Options

1)Photosynthesis

परकािसशिहॳषण

2) Transpiration

वाषपह८तसजयन

3) Absorption

अविह८षण

4) Respiration

शरवसन

Correct Answer Transpiration

Wilting is the loss of rigidity of non-

woody parts of plants This occurs when

the turgor pressure in non-lignified

plant cells falls towards zero as a result

of diminished water in the cells

Q114 Bovidae Ovis is the scientific name of

बह८पविीओपवस mdashndash कावहॴजञाननकनामहहॴ 22-Jan-2017

Options

1) Goat

बकरी 2) Cow

गाय

3) Buffalo

भहॳस

4) Sheep

भहॳड़

Correct Answer Sheep

Q115 Plants get their energy to produce

food from which of the following

पह८धहॳभह८जनकाननमायणकरनहॳकहॳ लिएनननननलिखितमसहॳककससहॳउजायपरापतकरतहॳहहॴ

22-Jan-2017

Options

1) Photosynthesis

परकािसशिहॳषण

2)Bacteria

बहॴकटीररया 3)Fungi

कवक

4)Sun

सयय Correct Answer Sun

Q116 Which of the following is secreted

by the liver

नननननलिखितमसहॳककसकासरावनजगरसहॳहह८ताहहॴ

22-Jan-2017

Options

1) Glucose

गिकह८ज

2) Iodine

आयह८िीन

3) Cortisol

काटटरयसह८ि

4) Bile

पपतत

Correct Answer Bile

The liver makes bile that will help

emulsify and

digest the fats we eat

F A C E B O O K

P A G E h t t p w w w f a c e b o o k c o m s s c m e n t o r s o f f i c i a l P a g e | 28

FOR MORE UPDATES AND MORE MATERIAL DO LIKE OUR FACEBOOK PAGE httpwwwfacebookcomsscmentorsofficial

Q117 Ferns belong to which division of

plants

फनसयपह९धह८कहॳ ककसभागमआतहॳहहॴ

22-Jan-2017

Options

1) Gymnosperms

नजननह८सपनसय 2) Angiosperms

एनजयह८सपनसय 3) Thallophyta

िहॴिह८फाईटा 4)Pteridophyta

टहॳररिह८फाईटा Correct Answer Pteridophyta

Q118 Who invented Antibiotics

एटीबायह८टटककाअपवषकारककसनहॳककयािा

22-Jan-2017

Options

1) Joseph Lister

जह८सहॳफलिसटर

2) William Harvey

पवलियमहाव

3) Robert Knock

रॉबटयनॉक

4)Alexander Fleming

अिहॳकज़िरफिहॳलमग

Correct Answer Alexander Fleming

Q119 Milbecycin is used in the

eradication of

लममबहॳसायलसनका mdashndash

मउनमिनमपरयह८गककयाजाताहहॴ 22-Jan-2017

Options

1) Agricultural Fungus

कपषकवक

2) Agricultural Pests

कपषकीटक

3) Agricultural Herbs

कपषिाक

4)Agricultural Weeds

कपषननराना Correct Answer Agricultural Pests

Milbemycin oxime is a veterinary drug

from the group of milbemycins used as

a broad spectrum antiparasitic It is

active against worms and mites(insects

Q120 Intestinal bacteria synthesizes

which of the following in the human

body

मानविरीरमआतोकहॳ बहॴकटीररयानननननलिखितमसहॳककसकासशिहॳषणकरतहॳहहॴ 22-Jan-2017

Options

1) Vitamin K

पवटालमन K

2) Proteins

परह८टीन

3) Fats

वसा 4) Vitamin D

पवटालमन D

Correct Answer Vitamin K

Q121 is the study of the physical form

and external structure of plants

mdashmdash-

मपह९धह८काभहॴनतक पऔरबाहरीसरचनाकाआदयाककयाजाताहहॴ 22-Jan-2017

Options

1) Physiology

कफनजयह८िह८जी 2) Anatomy

िरीररचनापवजञान

3) Phytomorphology

फाईटह८मह८फह८िह८जी 4)Cytology

कह८लिकापवजञान

Correct Answer Phytomorphology

F A C E B O O K

P A G E h t t p w w w f a c e b o o k c o m s s c m e n t o r s o f f i c i a l P a g e | 29

FOR MORE UPDATES AND MORE MATERIAL DO LIKE OUR FACEBOOK PAGE httpwwwfacebookcomsscmentorsofficial

Q122 Which of the following is a

structural and functional unit of

kidneys

नननननलिखितमसहॳकह९नसीगदोकीसरचनातमकऔरकाययकरीईकाईहहॴ

22-Jan-2017

Options

1) Renette Cells

रहॳनहॳटकह८लिकाए

2) Flame Cells

फिहॳमकह८लिकाए

3) Nephrites

नहॳफ़राइटस

4)Nephrons

नहॳफरोस

Correct Answer Nephrons

Nephron functional unit of the kidney

the structure that actually produces

urine in the process of removing waste

and excess substances from the blood

There are about 1000000 nephrons in

each human kidney

Q123 Which of the following is the

largest part of the human brain

नननननलिखितमसहॳकह९नसामानवमनसतषककासबसहॳबड़ाटहससाहहॴ

23-Jan-2017

Options

1) Ribs

पसलियाा 2) Cerebrum

सहॳरहॳिम

3) Pons

पोस

4)Thalamus

िहॴिहॳमस

Correct Answer Cerebrum

The cerebrum is the largest part of the

human brain making up about two-

thirds of the brainrsquos mass It has two

hemispheres each of which has four

lobes frontal parietal temporal and

occipital

Q124 The auxiliary buds

सहायककालियाmdashndash 23-Jan-2017

Options

1) grow endogenously from the pericycle

पहॳरीसाईककिसहॳअनतजातयपवकलसतहह८ताहहॴ 2) arise endogenously from the main

growing point

मिवपदसहॳअनतजातयउठताहहॴ 3) is an embryonic shoot located in the

axil of a leaf

एकभरणिटहहॴजह८एकपततीकहॳ अकषपरनसतिहह८ताहहॴ 4)arise exogenously from the epidermis

एपपिलमयससहॳबटहजातयतरीकहॳ सहॳउठताहहॴ Correct Answer is an embryonic shoot

located in the axil of a leaf

Q125 Which of the following is a viral

disease

इनमहॳसहॳकह९सीएकवायरिबीमारीहहॴ

23-Jan-2017

Options

1) Polio

पह८लियह८ 2) Tetanus

धनसतनभ

3) Leprosy

कषठरह८ग

4) Plague

पिहॳग

Correct Answer Polio

A viral disease (or viral infection)

occurs when an organismrsquos body is

invaded by pathogenic viruses and

infectious virus particles (virions) attach

to and enter susceptible cells

F A C E B O O K

P A G E h t t p w w w f a c e b o o k c o m s s c m e n t o r s o f f i c i a l P a g e | 30

FOR MORE UPDATES AND MORE MATERIAL DO LIKE OUR FACEBOOK PAGE httpwwwfacebookcomsscmentorsofficial

Poliomyelitis often called polio or

infantile paralysis is an infectious

disease caused by the poliovirus

Tetanusmdash A serious bacterial infection

that causes painful muscle spasms and

can lead to death

Leprosy also known as Hansenrsquos

disease (HD) is a long-term infection by

the bacterium Mycobacterium leprae or

Mycobacterium lepromatosis

Plague is an infectious disease caused by

the bacterium Yersinia pestis

Symptoms include fever weakness and

headache

Q126 Which organisms can help to

carry out Vermicomposting

कह९नसाजीववमीकनपह८नसटगममददकरताहहॴ

23-Jan-2017

Options

1) Nitrifying Bacteria

नाईटरीफाईगबहॴकटीररया 2) Earthworms

कहॴ चऐ

3) Algae

िहॴवि

4) Fungus

कवक

Correct Answer Earthworms

Q127 Contraction of heart is also

known as

हदयकहॳ सकचनकह८ mdash- भीकहाजाताहहॴ 23-Jan-

2017

Options

1) Systole

लससटह८ि

2) Aristotle

अरसत

3) Diastole

िायसटह८ि

4) Lub

मयब

Correct Answer Systole

Diastole is the part of the cardiac cycle

when the heart refills with blood

following systole (contraction)

Ventricular diastole is the period during

which the ventricles are filling and

relaxing while atrial diastole is the

period during which the atria are

relaxing

Q128 Azadirachta indica is the

botanical name of which of the

following

अजाटदराचताइडिकानननननलिखितमसहॳककसकावानसपनतनामहहॴ

23-Jan-2017

Options

1) Rose plant

गिाबकापह९धा 2) Apple tree

सहॳबकापहॳड़

3) Neem

नीम

4)Mango

आम

Correct Answer Neem

Q129 Which of the following is the

main end product of carbohydrate

digestion

नननननलिखितमसहॳकह९नसाकाबोहाइडरहॳटकहॳ पाचनकापरमिअतउतपादकहह८ताहहॴ 23-Jan-2017

Options

1) Fats

वसा 2) Lipids

लिपपडस

3) Glucose

गिकह८ज

4) Cellulose

F A C E B O O K

P A G E h t t p w w w f a c e b o o k c o m s s c m e n t o r s o f f i c i a l P a g e | 31

FOR MORE UPDATES AND MORE MATERIAL DO LIKE OUR FACEBOOK PAGE httpwwwfacebookcomsscmentorsofficial

सहॳमयिह८ज

Correct Answer Glucose

Intestinal absorption of end products

from digestion of carbohydrates and

proteins in the pig hellip During absorption some sugars (fructose or

galactose) released from the

corresponding sucrose and lactose

respectively during digestion were

partly metabolized into glucose by the

enterocyte

Q130 Which of the following glands is a

source of the enzyme Ptyalin

नननननलिखितगरचियोमसहॳएजाइमटयालिनकासरह८तहहॴ 23-Jan-2017

Options

1) Pancreas

अगरािय

2) Thyroid Gland

िाइराइिगरिी 3) Pituitary Gland

पीयषगरिी 4) Salivary Glands

िारगरचियाा Correct Answer Salivary Glands

Q131 Which of the following is not true

about Pteridophyta

ननननमसहॳकह९नसीबातटहॳररिह८फाईटकहॳ बारहॳमसचनहीहहॴ 23-Jan-2017

Options

1) Dominant phase is saprophytes

परमिचरणसहॳपरह८फाईइटसहह८ताहहॴ 2) Main plant body is diploid

पह९दह८कामखयिरीरदपवगखणतहह८ताहहॴ 3) Seeds are present

बीजमह९जदहह८तहॳहहॴ 4)Flowers are absent

फिअनपनसतिहह८तहॳहहॴ

Correct Answer Seeds are present

Q132 The largest dolphin species is the

orca also called as

िॉिकफनकीसबसहॳबड़ीपरजानतकाकानामआकायहहॴनजसहॳ mdash- भीकहतहॳहहॴ 23-Jan-2017

Options

1) Bottle Nose

बाटिनह८ज

2) Baiji

बहॳजी 3) Killer whale

ककिरहहॳि

4)Tucuxi

टकवसी Correct Answer Killer whale

Q133 The fat digesting enzyme Lipase

is secreted by which of the following

वसाकापाचनकरनहॳवािाएजाइमिाइपहॳजनननननलिखितमसहॳककसकहॳ दवारासतरापवतहह८ताहहॴ

24-Jan-2017

Options

1) Kidneys

गद

2) Pancreas

अगनयािय

3) Large Intestine

बड़ीआत

4)Liver

नजगर

Correct Answer Pancreas

Lipase is an enzyme that splits fats so

the intestines can absorb them Lipase

hydrolyzes fats like triglycerides into

their component fatty acid and glycerol

molecules It is found in the blood

gastric juices pancreatic secretions

intestinal juices and adipose tissues

F A C E B O O K

P A G E h t t p w w w f a c e b o o k c o m s s c m e n t o r s o f f i c i a l P a g e | 32

FOR MORE UPDATES AND MORE MATERIAL DO LIKE OUR FACEBOOK PAGE httpwwwfacebookcomsscmentorsofficial

Q134 The arrangement of leaves on an

axis or stem is called

एकअकषयातनहॳपरपनततयोकीयवसिाकह८कयाकहाजाताहहॴ SSC CHSL Science (biology) 2016

Question Paper

24-Jan-2017

Options

1) Phyllotaxy

फाइिह८टहॴकसी 2) Vernation

वनिन

3) Venation

वहॳनहॳिन

4)Phytotaxy

फाइटह८टहॴकसी Correct Answer Phyllotaxy

In botany phyllotaxis or phyllotaxy is

the arrangement of leaves on a plant

stem (from Ancient Greek phyacutellon

ldquoleafrdquo and taacutexis ldquoarrangementrdquo)

Phyllotactic spirals form a distinctive

class of patterns in nature

Q135 The study of Cells is also known

as

कह८लिकाओकहॳ अधययनकह८ mdashmdashndash

भीकहाजाताहहॴ 24-Jan-2017

Options

1) Cytology

सायटह८िह८जी 2) Physiology

कफनजयह८िह८जी 3) Nucleology

नयककमयह८िह८जी 4)Cellology

सहॳिह८िह८जी Correct Answer Cytology

Q136 Which of the following scientists

is also known as the Father of Biology

नननननलिखितमसहॳककसवहॴजञाननककह८ ldquoजीवपवजञानकहॳ जनकrdquoकहॳ नामसहॳभीजानाजाताहहॴ 24-Jan-2017

Options

1) Herbert Spencer

हबयटयसपसर

2) Aristotle

अरसत 3) Lamarck

िहॳमाकय 4)Darwin

िापवयन

Correct Answer Aristotle

Q137 Which cells give rise to various

organs of the plant and keep the plant

growing

कह९नसीकह८लिकाएपह९धह८कहॳ लभननअगह८कह८जनमदहॳतीहहॴऔरपह९धह८कह८बढ़नहॳममददकरतीहहॴ

24-Jan-2017

Options

1) Permanent

सिायी 2) Dermal

तवचीय

3) Meristematic

मररसटहॳमटटक

4)Mature

परह८ढ़

Correct Answer Meristematic

A meristem is the tissue in most plants

containing undifferentiated cells

(meristematic cells) found in zones of

the plant where growth can take place

Q138 Rodentia Muridae is the scientific

name of

F A C E B O O K

P A G E h t t p w w w f a c e b o o k c o m s s c m e n t o r s o f f i c i a l P a g e | 33

FOR MORE UPDATES AND MORE MATERIAL DO LIKE OUR FACEBOOK PAGE httpwwwfacebookcomsscmentorsofficial

रह८िहॳलियानयररिी mdashmdash- कावहॴजञाननकनामहहॴ 24-

Jan-2017

Options

1) Mouse

चहा 2) Squirrel

चगिहरी 3) Monkey

बदर

4) Lizard

नछपकिी Correct Answer Mouse

Q139 Name the scientist who proposed

the cell theory

कह८लिकालसदातकापरसतावदहॳनहॳवािहॳवहॴजञाननककानामबताइए 24-Jan-2017

Options

1) Schleiden and Schwann

िीमिनऔरशरववान

2) Lamarck

िहॳमाकय 3) Treviranus

टरहॳवायरहॳनस

4)Whittaker and Stanley

हीटकरऔरसटहॳनिहॳ Correct Answer Schleiden and

Schwann

Q140 The flower with the worldrsquos

largest bloom is

दननयाकासबसहॳबड़ाफिखििनहॳवािा mdashmdashndash हहॴ 24-Jan-2017

Options

1) Pando

पािह८ 2) Posidonia

पह८सीिह८ननया 3) Rafflesia arnoldii

ररफिहॳलियाअनोमिी 4)Helianthus annuus

हहॳलिएनिसएनयअस

Correct Answer Rafflesia arnoldii

Rafflesia arnoldii is a species of

flowering plant in the parasitic genus

Rafflesia It is noted for producing the

largest individual flower on earth It has

a very strong and horrible odour of

decaying flesh earning it the nickname

ldquocorpse flower

Q141 Deficiency of which vitamin

causes night blindness

ककसपवटालमनकीकमीकहॳ कारणरतौधीहह८ताहहॴ 24-Jan-2017

Options

1) Vitamin K

पवटालमन K

2) Vitamin C

पवटालमन C

3) Vitamin B1

पवटालमन B1

4)Vitamin A

पवटालमन A

Correct Answer Vitamin A

Q142 Nongreen plants lack which of the

following

गहॴर-

हररतवनसपनतमनननननलिखितमसहॳककसकीकमीहह८तीहहॴ

24-Jan-2017

Options

1) Chlorophyll

किह८रह८कफि

2) Lycophyll

िायकह८कफि

3) Cyanophyll

F A C E B O O K

P A G E h t t p w w w f a c e b o o k c o m s s c m e n t o r s o f f i c i a l P a g e | 34

FOR MORE UPDATES AND MORE MATERIAL DO LIKE OUR FACEBOOK PAGE httpwwwfacebookcomsscmentorsofficial

सायनह८कफि

4)Phototropism

फह८टह८टरोपपजम

Correct Answer Chlorophyll

Q143 Organisms that use light to

prepare food are known as

जह८जीवपरकािकाउपयह८गकरभह८जनतहॴयारकरतहॳहहॴ उनह mdashmdash- कहॳ पमजानजाताहहॴ 24-Jan-2017

Options

1) Autotrophs

सवपह८षी 2) Heterotrophs

पवषमपह८षज

3) Omnivores

सवायहारी 4)Decomposers

पवघटनकरनहॳवािा Correct Answer Autotrophs

autotrophs often make their own food

by using sunlight carbon dioxide and

water to form sugars which they can use

for energy Some examples of

autotrophs include plants algae and

even some bacteria Autotrophs

(producer) are important because they

are a food source for heterotrophs

(consumers)

A heterotroph is an organism that

ingests or absorbs organic carbon

(rather than fix carbon from inorganic

sources such as carbon dioxide) in order

to be able to produce energy and

synthesize compounds to maintain its

life Ninety-five percent or more of all

types of living organisms are

heterotrophic including all animals and

fungi and some bacteria

Q144 Which of the following is a

primary function of haemoglobin

नननननलिखितमसहॳकह९नसाटहमह८गिह८बबनकाएकपरािलमककाययहहॴ

25-Jan-2017

Options

1) Utilization of energy

उजायकाउपयह८गकरना 2) Prevention of anaemia

रकतामपताहह८नहॳसहॳरह८कना 3) Destruction of bacteria

बहॴकटीररयाकापवनािकरना 4) To transport oxygen

ऑकसीजनकावहनकरना Correct Answer To transport oxygen

Q145 Vascular bundles are absent in

सवहनीबिि mdashmdash- मअनपनसतिरहतहॳहहॴ 25-Jan-2017

Options

1) Bryophyta

िायह८फाइटा 2) Pteridophyta

टहॳररिह८फाईटा 3) Gymnosperms

नजननह८सपमय 4) Angiosperms

एननजयह८सपहॳनसय Correct Answer Bryophyta

Q146 Sauria Lacertidae is the scientific

name of

सहॴररयािहॳसरटाईिी mdashmdashndash कावहॴजञाननकनामहहॴ 25-Jan-2017

Options

1) Crocodile

मगरमचछ

2) Hippopotamus

टहपपह८पह८टहॳमस

3) Lizard

नछपकिी 4) House fly

F A C E B O O K

P A G E h t t p w w w f a c e b o o k c o m s s c m e n t o r s o f f i c i a l P a g e | 35

FOR MORE UPDATES AND MORE MATERIAL DO LIKE OUR FACEBOOK PAGE httpwwwfacebookcomsscmentorsofficial

घरहॳिमकिी Correct Answer Lizard

Q147 Which type of pathogen causes

the water-borne disease SARS (Severe

Acute Respiratory Syndrome)

ककसपरकािकारह८गज़नकजिजननतबीमारीसासयकाकारणबनताहहॴ 25-Jan-2017

Options

1) Viral

वायरि

2) Parasitic

परजीवी 3) Protozoan

परह८टह८जअन

4) Bacterial

बहॴकटीररयि

Correct Answer Viral

Q148 Which of the following organs

produces the enzyme lipase

नननननलिखितमसहॳकह९नसाअगिायपहॳजएजाइमउतपननकरताहहॴ 25-Jan-2017

Options

1) Pancreas

अगनयािय

2) Large Intestine

बड़ीआत

3) Liver

नजगर

4) Small Intestine

छह८टीआत

Correct Answer Pancreas

Q149 A is a long internode forming the

basal part or the whole of a peduncle

एक mdashmdash- एकिबाइटरनह८िहहॴ जह८ननचिाटहससायासनपणयिठिबनताहहॴ 25-

Jan-2017

Options

1) Rhizome

परकद

2) Rachis

महॳ दि

3) floral axis

पषपअकष

4) Scape

भगदड़

Correct Answer scape

Q150 ndash Which of the following

organisms are considered to be both

Living and Non-living

नननननलिखितमसहॳकह९नसहॳजीवाणकह८जीपवतऔरअजीपवतमानाजाताहहॴ

25-Jan-2017

Options

1) Bacteria

बहॴकटीररया 2) Fungi

कवक

3) Algae

िहॴवाि

4)Virus

वायरस

Correct Answer Virus

They are considered to be living as they

possess a protein coat as a protective

covering DNA as the genetic material

etc

They are said to be non-living as they

can be crystallised and they survive for

billions of years They can tolerate high

temperatures freezing cold

temperatures ultra-violet radiations etc

Q151 Deficiency of fluorine causes

which of the following

फिह८ररनकीकमीकहॳ कारणनननननलिखितमसहॳकयाहह८ताहहॴ

F A C E B O O K

P A G E h t t p w w w f a c e b o o k c o m s s c m e n t o r s o f f i c i a l P a g e | 36

FOR MORE UPDATES AND MORE MATERIAL DO LIKE OUR FACEBOOK PAGE httpwwwfacebookcomsscmentorsofficial

27-Jan-2017

Options

1) Dental Caries

िटिकहॴ ररज

2) Scurvy

सकवरी 3) Anaemia

रकतामपता 4) Arthritis

गटठया Correct Answer Dental Caries

Q152 In a Punnett Square with the

cross AaBb x AaBb how many Aabb

genotypes would be created

पनहॳटसककायरमिह८स AaBb x AaBb कहॳ साि

ककतनहॳ Aabb जीनह८टाइपबनगहॳ 27-Jan-2017

Options

1) 1

2) 8

3) 2

4) 3

Correct Answer 2

Q153 Which of the following is the

Controlling Center of the Cell

नननननलिखित म सहॳ कह८लिकाका ननयतरण

क दर कह९न हहॴ

27-Jan-2017

Options

1) Nucleus

क दर

2) Plasma

पिाजमा 3) Lysosome

िायसह८सह८म

4) Chromosome

िह८मह८सह८म

Correct Answer Nucleus

The control centre of the cell is the

nucleus in eukaryotic cells The nucleus

contains genetic material in the form of

DNA

Q154 Myopia affects which of the

following organs

मायह८पपयानननननलिखितअगह८मसहॳककसहॳपरभापवतकरताहहॴ

25-Jan-2017

Options

1) Heart

हदय

2) Skin

तवचा 3) Eyes

आािहॳ 4)Mouth

मह

Correct Answer Eyes

Q155 Which of the following bears

flowers

नननननलिखितमसहॳकह९नफिधारणकरताहहॴ

25-Jan-2017

Options

1) Bryophyta

िायह८फाइटा 2) Pteridophyta

टहॳरीिह८फाईटा 3) Gymnosperms

नजननह८सपमय 4)Angiosperms

एननजयह८सपमय Correct Answer Angiosperms

Q156 Oxygenated blood flows out of the

heart through the

ऑकसीजनयकतरकत mdashmdashmdash

कहॳ माधयमसहॳहदयकहॳ बाहरबहताहहॴ 25-Jan-2017

F A C E B O O K

P A G E h t t p w w w f a c e b o o k c o m s s c m e n t o r s o f f i c i a l P a g e | 37

FOR MORE UPDATES AND MORE MATERIAL DO LIKE OUR FACEBOOK PAGE httpwwwfacebookcomsscmentorsofficial

Options

1) Aorta

महाधमनी 2) pulmonary artery

फहॳ फड़हॳकीधमनी 3) vena cava

वहॳनाकावा 4)Atrium

चह९क

Correct Answer aorta

Q157 Blood leaving the liver and

moving towards the

heart has a higher concentration of

नजगरसहॳननकिकरहदयकीतरफजानहॳवािहॳरकतम mdashmdashmdashmdash कीउचचसादरताहह८तीहहॴ 27-Jan-2017

Options

1) Lipids

लिपपडस

2) Urea

यररया 3) Bile Pigments

पपततकहॳ रगकरण

4) Carbon dioxide

काबयनिायऑकसाइि

Correct Answer Bile Pigments

Urea is nitrogen containing substance

which is produced in the liver in order

to deal with excess amino-acids in the

body As urea is produced it leaves the

liver in the blood stream and passes via

the circulatory system to all parts of the

body

Q158 Bulb is a modification of which

part of a plant

बमबएकपह९धहॳकहॳ ककसटहससहॳकाएक पातरणहह८ताहहॴ 27-Jan-2017

Options

1) The root

जड़

2) The stem

तना 3) The radicle

मिाकर

4)The fruit

फि

Correct Answer The stem

Q159 Which of the following carries

blood away from the heart to different

body parts

इनमहॳसहॳकह९नरकतकह८हदयसहॳिरीरकहॳ पवलभननअगह८तकिहॳजातीहहॴ

27-Jan-2017

Options

1) Arteries

धमननया 2) Nerves

तबतरहाए

3) Capillaries

कहॳ लिकाए

4)Veins

नसहॳ Correct Answer Arteries

Q160 The series of processes by which

nitrogen and its compounds are

interconverted in the environment and

in living organisms is called

27-Jan-2017

Options

1)Absorption of Nitrogen

2)Ammonification

3)Nitrogen Fixation

4)Nitrogen Cycle

Correct Answer Nitrogen Cycle

Ammonification or Mineralization is

performed by bacteria to convert

organic nitrogen to ammonia

F A C E B O O K

P A G E h t t p w w w f a c e b o o k c o m s s c m e n t o r s o f f i c i a l P a g e | 38

FOR MORE UPDATES AND MORE MATERIAL DO LIKE OUR FACEBOOK PAGE httpwwwfacebookcomsscmentorsofficial

Nitrification can then occur to convert

the ammonium to nitrite and nitrate

Nitrogen fixation is a process by which

nitrogen in the Earthrsquos atmosphere is

converted into ammonia (NH3) or other

molecules available to living organisms

Q161 BCG vaccine is given to protect

from which of the following

बीसीजीकाटटकानननननलिखितमसहॳककसकहॳ बचावकहॳ लिएटदयाजातहहॴ

27-Jan-2017

Options

1) Jaundice

पीलिया 2) Anaemia

रकतमपता 3) Tuberculosis

कषयरह८ग

4) Polio

पह८लियह८ Correct Answer Tuberculosis

Q162 Parallel venation is found in

समानतरवहॳनहॳिन mdashmdashmdash- मपायाजाताहहॴ 27-Jan-2017

Options

1) plants which are monocots

पह९धहॳजह८एकबीजपतरीहह८तहॳहहॴ 2) plants which have a dicot stem

वहॳपह९धहॳनजनकातनादपवदलियहह८ताहहॴ 3) plants with leaves similar to Tulsi

वहॳपह९धहॳनजनकीपनततयतिसीकीपनततयोकहॳ समानहह८तहॳहहॴ 4)plants with tap roots

टहॳप टवािहॳपह९धहॳ Correct Answer plants which are

monocots

Q163 The hardest part of the body is

िरीरकासबसहॳकठह८रभाग mdashndash हहॴ 27-Jan-2017

Options

1) Bones

हडडिय

2) Tooth Enamel

दातकहॳ इनहॳमि

3) Skull

िह८पड़ी 4) Spinal Cord

महॳ रजज

Correct Answer Tooth Enamel

Q164 Which type of pathogen causes

the waterborne disease E coli Infection

ककसपरकारकारह८गजननकजिजननतरह८गईकह८िाईसिमणकाकारणबनताहहॴ 27-Jan-2017

Options

1) Protozoan

परह८टह८जआ

2) Parasitic

परजीवी 3) Bacterial

बहॴकटीररयि

4)Viral

वायरि

Correct Answer Bacterial

Q165 The amount of blood filtered

together by both the kidneys in a 70 kg

adult male human in a minute is

70 की गरा वािहॳएकवयसकप षमएकलमनटमदह८नोगदकहॳदवाराएकसािचाबनीगयीरकतकीमातरहह८तीहहॴ 29-Jan-2017

Options

1) 1100 ml

1100 लमलि

2) 100 ml

F A C E B O O K

P A G E h t t p w w w f a c e b o o k c o m s s c m e n t o r s o f f i c i a l P a g e | 39

FOR MORE UPDATES AND MORE MATERIAL DO LIKE OUR FACEBOOK PAGE httpwwwfacebookcomsscmentorsofficial

100 लमलि

3) 1500 ml

1500 लमलि

4) 500 ml

500 लमलि

Correct Answer 1100 ml

Q166 Which feature of a plant helps to

distinguish a monocot from a dicot

पह९धहॳकीवहकह९नसीपविहॳषताहहॴजह८एकदपवदलियहॳऔरएकएकदिीयपह९धहॳसहॳभहॳदकरनहॳममददकरतीहहॴ 29-Jan-2017

Options

1) Pollination

परागम

2) Venation

वहॳनहॳिन

3) Vernation

वनिन

4) Aestivation

एसटीवहॳिहॳन

Correct Answer venation

Q167 The Mutation Theory was

proposed by

उतवररवतयनकालसदात mdashmdashndash

कहॳ दवरापरसतापवतककयाजाताहहॴ 29-Jan-2017

Options

1) Charles Lyell

चामसयलियहॳि

2) William Smith

पवलियमनसमि

3) Hugo De Vries

हयगह८िीराईस

4)Harrison Schmitt

हहॳरीसननसमट

Correct Answer Hugo De Vries

Q168 Which type of pathogen causes

the waterborne disease HepatitisA

ककसपरकारकहॳ रह८गजनकजिजननतरह८गहहॳपहॳटाइटटस-A काकारणबनताहहॴ

29-Jan-2017

Options

1) Parasitic

परजीवी 2) Viral

वायरि

3) Protozoan

परह८टह८जआ

4) Bacterial

बहॴकटीररयि

Correct Answer Viral

Q169 In a Punnett Square with the

cross AaBb x Aabb how many AaBb

genotypes would be created

पनहॳटसकवायरमिह८स AaBb x Aabb

कहॳ सािककतनहॳ AaBb जीनह८टाइपबनगहॳ 29-Jan-

2017

Options

1) 4

2) 1

3) 7

4) 6

Correct Answer 4

Q170 Arboreal Ateles is the scientific

name of

अिह८ररयिएटटलिस mdashmdashmdash कावहॴजञाननकनामहहॴ 29-Jan-2017

Options

1) Squirrel

चगिहरी 2) Sparrow

गह८रहॴया 3) Lizard

नछपकिी 4) Spider monkey

F A C E B O O K

P A G E h t t p w w w f a c e b o o k c o m s s c m e n t o r s o f f i c i a l P a g e | 40

FOR MORE UPDATES AND MORE MATERIAL DO LIKE OUR FACEBOOK PAGE httpwwwfacebookcomsscmentorsofficial

मकड़ीबदर

Correct Answer Spider monkey

Q171 Which type of pathogen causes

the waterborne disease Salmonellosis

ककसपरकारकारह८गाणजिजननतबीमारीसािमह८नहॳिह८लसज़काकारकहहॴ

29-Jan-2017

Options

1) Algal

िहॳवालियहॳ 2) Parasitic

परजीवी 3) Bacterial

बहॴकटीररयि

4)Viral

वायरि

Correct Answer Bacterial

An infection with salmonella bacteria

commonly caused by contaminated food

or water

Symptoms include diarrhoea fever

chills and abdominal pain

Q172 is a condition in which there is a

deficiency of red cells or of haemoglobin

in the blood

mdashmdash-

एकनसिनतहहॴनजसमहॳरकतमिािकह८लिकाओकीयाहीमह८गिह८बबनकीकमीहह८तीहहॴ 29-Jan-2017

Options

1) Albinism

एनमबननजम

2) Propyria

परह८पीररया 3) Anaemia

एनीलमया 4)Keloid disorder

कहॳ िह८इिडिसओिर

Correct Answer Anaemia

Q173 Ananas comosus is the scientific

name of

Options

अनानासकह८मह८सस mdashmdashmdashndash

कावहॴजञाननकनामहहॴ 29-Jan-2017

1) Custard Apple

सीताफि

2) Pineapple

पाइनएपपि

3) Bamboo

बास

4)Pomegranate

अनार

Correct Answer Pineapple

Q174 Which organ produces insulin

कह९नसाअगइनसलिनपहॴदाकरताहहॴ 29-Jan-

2017

Options

1) Liver

यकत

2) Thyroid gland

िायराइिगरिी 3) Spleen

पिीहा 4)Pancreas

अगरयिय

Correct Answer Pancreas

Q175 Which of the following disease is

not caused by water pollution

नननननलिखितमसहॳकह९नसारह८गपानीकहॳ परदषणकहॳकारणनहीहह८ता

29-Jan-2017

Options

1) Cholera

हहॴजा 2) Typhoid

F A C E B O O K

P A G E h t t p w w w f a c e b o o k c o m s s c m e n t o r s o f f i c i a l P a g e | 41

FOR MORE UPDATES AND MORE MATERIAL DO LIKE OUR FACEBOOK PAGE httpwwwfacebookcomsscmentorsofficial

टाइफाइि

3) Asthma

दमा 4)Diarrhoea

दसत

Correct Answer Asthma

Q176 Ocimum tenuiflorum is the

scientific name of

ओलिलममटहॳयईफिह८रमइसकावहॴजञाननकनाम mdash

ndash हहॴ 30-Jan-2017

Options

1) Neem

नीम

2) Mango

आम

3) Babul

बबि

4)Tulsi

तिसी Correct Answer Tulsi

Q177 Which gland secretes bile a

digestive fluid

कह९नसीगरिीपपतत एकपाचनतरिपरदािय सरापवतकरतीहहॴ 30-Jan-2017

Options

1) Pancreas

अगनयािय

2) Liver

यकत

3) Thyroid

िायराइि

4) Testes

टहॳनसटस

Correct Answer liver

Q178 In which of the following the

dominant phase is Gametophyte

नननननलिखितमसहॳककसकहॳ परमिचरणयगमकह८दपवधद (Gametophyte)हहॴ 30-Jan-2017

Options

1) Bryophyta

िायह८फाइटा 2) Pteridophyta

टहॳररिह८फाइटा 3) Gymnosperms

नजननह८सपमय 4) Angiosperms

एननजयह८सपमय Correct Answer Bryophyta

Q179 Anaerobic respiration refers to

which of the following

नननननलिखितमसहॳककसहॳअवायवीयशवसनकहाजाताहहॴ

30-Jan-2017

Options

1) Respiration without Oxygen

ऑकसीजनकहॳ बबनाशवसन

2) Respiration with Oxygen

ऑकसीजनकहॳ सािशवसन

3) Respiration without CO2

काबयनिायऑकसाइिकहॳ बबनाशवसन

4) Respiration with CO2

काबयनिायऑकसाइिकहॳ सािशविन

Correct Answer Respiration without

Oxygen

Q180 Which type of pathogen causes

the waterborne disease Cholera

ककसपरकारकारह८गजनकजिजननतरह८गहहॴजाकाकारणबनताहहॴ

30-Jan-2017

Options

1) Algal

िहॴवालियहॳ

F A C E B O O K

P A G E h t t p w w w f a c e b o o k c o m s s c m e n t o r s o f f i c i a l P a g e | 42

FOR MORE UPDATES AND MORE MATERIAL DO LIKE OUR FACEBOOK PAGE httpwwwfacebookcomsscmentorsofficial

2) Bacterial

बहॴकटीररयि

3) Protozoan

परह८टह८जआ

4) Viral

वायरि

Correct Answer Bacterial

Q181 To which class does

Oxyreductases transferases hydrolases

belong

ओकसीररिकटहॳसटरासफरहॳजहॳस

हाइडरह८िहॳसहॳसककसवगयमआतहॳहहॴ 30-Jan-2017

Options

1) Hormones

हारमोस

2) Enzymes

एजाइनस

3) Proteins

परह८टीनस

4) Vitamins

पवटालमनस

Correct Answer Enzymes

Q182 Which of the following is not true

about Gymnosperms

ननननमसहॳकह९नसीबातअनावतबीजीकहॳ बारहॳमसचनहीहहॴ 30-Jan-2017

Options

1) Dominant phase is saprophytes

परमिचरणसहॳपरह८फाइटसहह८ताहहॴ 2) Vascular bundles are absent

सवहनीबििअनपनसितहह८ताहहॴ 3) spores are heterospores

बीजाणहहॳटहॳरह८सपह८रसहह८तहॳहहॴ 4) Flowers are absent

फिअनपनसितहह८तहॳहहॴ

Correct Answer Vascular bundles are

absent

Q183 The name of first mammal clone sheep is

भहॳड़कीपरिमसतनपायीपरनत प (किह८न)

कानामहहॴ 30-Jan-2017

Options

1) Noori

नरी 2) Dolly

िॉिी 3) Louise

िसी 4)Durga

दगाय Correct Answer Dolly

Q184 Which type of pathogen causes

the water-borne disease Typhoid fever

ककसपरकारकारह८गजनकजिजननतरह८गटाइफाइिबिारकाकारणबनताहहॴ 30-Jan-2017

Options

1) Algal

िहॴवािीय

2) Parasitic

परजीवी 3) Protozoan

परह८टह८जनअन

4)Bacterial

बहॴकटीररयि

Correct Answer Bacterial

Q185 In which part of the cell are

proteins made

कह८लिकाकहॳ ककसटहससहॳमपरह८टीनबनायाजाताहहॴ

31-Jan-2017

Options

1) Reticulum

रहॳटटकिम

F A C E B O O K

P A G E h t t p w w w f a c e b o o k c o m s s c m e n t o r s o f f i c i a l P a g e | 43

FOR MORE UPDATES AND MORE MATERIAL DO LIKE OUR FACEBOOK PAGE httpwwwfacebookcomsscmentorsofficial

2) Golgi apparatus

गह८मजीएपहॳरहॳटस

3) Ribosomes

ररबह८सह८नस

4) Lysosome

िायसह८सह८नस

Correct Answer ribosomes

Proteins are produced by stringing

amino acids together in the order

specified by messenger RNA strands

that were transcribed from DNA in the

cell nucleus The process of synthesizing

a protein is called translation and it

occurs on ribosomes in the cytoplasm of

a cell

Q186 Polio is a disease caused by which

of the following

नननननलिखितमसहॳपह८लियह८कीबबमारह८हह८नहॳकाकारणकयाहहॴ

31-Jan-2017

Options

1) Bacteria

बहॴकटीररयि

2) Mosquito

मचछर

3) Virus

वायरस

4) Cockroach

नतिच हॳ Correct Answer Virus

Polio or poliomyelitis is a crippling and

potentially deadly infectious disease It

is caused by the poliovirus

Q187 ndash Hay fever is a sign of which of

the following

हहॳकफवरनननननलिखितमसहॳककसकाएकसकहॳ तहहॴ

31-Jan-2017

Options

1) Old Age

वदावसिा 2) Malnutrition

कपह८सण

3) Allergy

एिनजय 4) Over Work

अतयचधककाययकरना Correct Answer Allergy

Q188 How many chromosomes does a

human cell contain

एकमानवकह८लिकामककतनहॳगणसतरहह८तहॳहहॴ

29-Jan-2017

Options

1) 6

2) 26

3) 46

4) 66

Correct Answer 46

In humans each cell normally contains

23 pairs of chromosomes for a total of

46 Twenty-two of these pairs called

autosomes look the same in both males

and females The 23rd pair the sex

chromosomes differ between males and

females

Q189 Which of the following is not true

about Bryophyta

ननननमसहॳकह९नसीबातिायह८फाइटकहॳ बारहॳमसचनहीहहॴ 31-Jan-2017

Options

1) Dominant phase is gametophytes

परमिचरणगहॳलमतह८फाइटसहह८ताहहॴ 2) Main plant body is haploid

पह९धहॳकामखयिरीरअगखणतहह८ताहहॴ 3) Spores are homospores

बीजाणहह८मह८सफह८रसहह८तहॳहहॴ 4) Flowers are present

फिमह८जदहह८तहॳहहॴ Correct Answer Flowers are present

F A C E B O O K

P A G E h t t p w w w f a c e b o o k c o m s s c m e n t o r s o f f i c i a l P a g e | 44

FOR MORE UPDATES AND MORE MATERIAL DO LIKE OUR FACEBOOK PAGE httpwwwfacebookcomsscmentorsofficial

Q190 Which aquatic animal has

trailing tentacles

ककसजिीयजानवरकहॳ पीछहॳचिनहॳवािहॳटहॳटकिसहह८तहॳहहॴ

31-Jan-2017

Options

1) Sea horse

समदरीघह८िा 2) Corals

मगा 3) Jelly fish

जहॳिीमछिी 4) Star fish

तारामछिी Correct Answer Jelly fish

Jellyfish with its umbrella-shaped bell

and trailing tentacles

Q191 Which type of pathogen causes

the water-borne disease Poliomyelitis

(Polio)

ककसपरकारकारह८गजनकजिजननतरह८गपह८लियह८मायहॳटटस (पह८लियह८) काकारणहहॴ 31-Jan-

2017

Options

1) Parasitic

परजीवी 2) Algal

िहॴवालिय

3) Viral

वायरि

4) Bacterial

बहॴकटीररयि

Correct Answer Viral

Q192 The outer white part of the eye

that protects the inner structures is

आािकाबाहरीसफहॳ दटहससाजह८आतररकसरचनाओकीरकषाकरताहहॴ वह mdashmdashmdash हहॴ 31-Jan-

2017

Options

1) Iris

आयररस

2) Sclera

सकिहॳरा 3) Retina

रहॳटटना 4) Cornea

कह८ननयया Correct Answer Sclera

Q193 Proteins are made up of

परह८टीनकाननमायण mdashndash सहॳहह८ताहहॴ 31-Jan-2017

Options

1) Amino acids

एलमनह८अनि

2) Fatty acids

वसायकतअनि

3) Glucose

गिकह८ज

4)Nucleotides

नयनकियह८टाईिस

Correct Answer Amino acids

Q194 Moringa Oleifera is the scientific

name of

मह८ररगओलिफहॳ रा mdashmdashndash कावहॴजञाननकनामहहॴ 31-Jan-2017

Options

1) Banyan

बरगद

2) Gulmohar

गिमह८हर

3) Amla

आमिा

F A C E B O O K

P A G E h t t p w w w f a c e b o o k c o m s s c m e n t o r s o f f i c i a l P a g e | 45

FOR MORE UPDATES AND MORE MATERIAL DO LIKE OUR FACEBOOK PAGE httpwwwfacebookcomsscmentorsofficial

4) Drumstick

डरमनसटक

Correct Answer Drumstick

Q195 Kidney stones are composed of

गदकीपिरी mdashndash सहॳबनीहह८तीहहॴ 1-Feb-2017

Options

1) Calcium Oxalate

कहॴ नमसयमओकजहॳिहॳट

2) Sodium Chloride

सह८डियमकिह८राइि

3) Magnesium Nitrate

महॳनगनलियमनाइतटरहॳट

4) Calcium Bicarbonate

कहॴ नमियमबायकबोनहॳट

Correct Answer Calcium Oxalate

Q196 ndash Which of the following is not

true about Angiosperms

ननननमसहॳकह९नसीबातआवतबीजीकहॳ बारहॳमसचनहीहहॴ 1-Feb-2017

Options

1) Dominant phase is gametophytes

परमिचरणगहॳलमतह८फाइटहह८ताहहॴ 2) Vascular bundles are present

सवहनीबििमह९जदहह८ताहहॴ 3) Spores are heterospores

बीजाणहहॳटहॳरह८सपह८रसहह८तहॳहहॴ 4) Seeds are covered

बीजढकहॳ हह८तहॳहहॴ Correct Answer Dominant phase is

gametophytes

Q197 All of the following are excretory

(waste) products of animals except

नननननलिखितमसहॳककसएककह८छह८ड़करअनयसभीपराखणयोदवाराउतसनजयतपदाियहहॴ 1-Feb-

2017

Options

1) Uric Acid

यररकएलसि

2) Ammonia

अमह८ननया 3) Carbohydrates

काबोहाइडरहॳट

4) Urea

यररया Correct Answer Carbohydrates

In animals the main excretory products

are carbon dioxide ammonia (in

ammoniotelics) urea (in ureotelics) uric

acid (in uricotelics) guanine (in

Arachnida) and creatine

Q198 RNA is a polymeric molecule

What does RNA stand for

आरएनइएएकबहिकआणहहॴ इसकाकापवय पकयाहहॴ 1-Feb-2017

Options

1) Rado Nuclear Acid

रािह८नयनकियरएलसि

2) Ribo Nucleic Acid

राइबह८नयनकिकएलसि

3) Rhino Nuclear Acid

हाइनह८नयनकियरएलसि

4) Resto Nucleus Acid

रहॳसटह८नयकिीयसएलसि

Correct Answer Ribo Nucleic Acid

Q199 Which organ does detoxification

and produces chemicals needed for

digestion

कह९नसाअगपवषहरणकरताहहॴऔरपाचनकहॳ लिएआवशयकरसायनोकह८पहॴदाकरताहहॴ 1-Feb-

2017

Options

1) Salivary glands

िारगरचिया 2) Pancreas

अगनयािय

F A C E B O O K

P A G E h t t p w w w f a c e b o o k c o m s s c m e n t o r s o f f i c i a l P a g e | 46

FOR MORE UPDATES AND MORE MATERIAL DO LIKE OUR FACEBOOK PAGE httpwwwfacebookcomsscmentorsofficial

3) Thyroid gland

िायराइिगरिी 4) Liver

यकत

Correct Answer Liver

Q200 Psidium guajava is the scientific

name of

लसडियमगआजावा mdashmdash कावहॴजञाननकनामहहॴ 1-

Feb-2017

Options

1) Guava

अम द

2) Mango

आम

3) Bamboo

बास

4) Jack fruit

कटहि

Correct Answer Guava

Q201 Which drug is used as a Blood

Thinner

चधरकह८पतिाकरनहॳकहॳ पमककसदवाकापरयह८गककयाजाताहहॴ

1-Feb-2017

Options

1) Warfarin

वाफर न

2) Tramadol

टरहॳमािह८ि

3) Azithromycin

एनजरह८मायलसन

4) Hydralazine

हाइडरह८िहॳनजन

Correct Answer Warfarin

Q202 Which of the following disease is

caused due to the deficiency of protein

परह८टीनकीकमीकहॳ कारणनननननलिखितमसहॳकह९नसारह८गहह८ताहहॴ 1-Feb-2017

Options

1) Arthritis

गटठया 2) Kwashiorkor

कािीओकय र

3) Goitre

गाइटर

4) Night Blindness

रतह९चध

Correct Answer Kwashiorkor

Q203 A is species of plant that has

adapted to survive in an environment

with little liquid water

mdashmdashndashपह९धहॳकीएकऐसहॳऐसहॳपरजानतहहॴ नजसनहॳकमपानीवािहॳवातावरणमजीपवतरहनहॳकहॳलिएअनकिनहहॴ 1-Feb-2017

Options

1) Xerophyte

म दपवद

2) Hydrophyte

जिीयपादप

3) Mesophyte

समह८दपवद

4) Thallophyte

िहॴिह८फाइटा Correct Answer xerophyte

xerophyte is a species of plant that has

adapted to survive in an environment

with little liquid water such as a desert

or an ice- or snow-covered region in the

Alps or the Arctic

Mesophytes are terrestrial plants which

are adapted to neither a particularly

dry nor particularly wet environment

An example of a mesophytic habitat

would be a rural temperate meadow

F A C E B O O K

P A G E h t t p w w w f a c e b o o k c o m s s c m e n t o r s o f f i c i a l P a g e | 47

FOR MORE UPDATES AND MORE MATERIAL DO LIKE OUR FACEBOOK PAGE httpwwwfacebookcomsscmentorsofficial

which might contain goldenrod clover

oxeye daisy and Rosa multiflora

thallophyte any of a group of plants or

plantlike organisms (such as algae and

fungi) that lack differentiated stems

leaves and roots and that were formerly

classified as a primary division

(Thallophyta) of the plant kingdom

Q204 How many types of teeth are

there in humans

मनषयोमककतनहॳपरकारकहॳ दातहह८तहॳहहॴ

1-Feb-2017

Options

1) 4

2) 5

3) 2

4) 3

Correct Answer 4

teeth -Humans have four types of

teethincisors canines premolars and

molars each with a specific function

The incisors cut the food the canines

tear the food and the molars and

premolars crush the food

Q205 Carica papaya is the scientific name of

कहॴ ररकापपाया mdashmdashndash कावहॴजञाननकनामहहॴ 2-

Feb-2017

Options

1) Peepal

पीपि

2) Papaya

पपीता 3) Tamarind

इमिी 4) Drumstick

ढह८िकाछड़ी Correct Answer Papaya

Q206 Muscles get tired when there is

shortfall of

जब mdashndash कीकमीहह८तीहहॴतबपहॳिीयिकजातीहहॴ 2-Feb-2017

Options

1) Lactic acid

िहॴनकटकएलसि

2) Na+ ions

Na+ आयन

3) ATP

एटीपी 4) Sulphates

समफहॳ टस

Correct Answer ATP

ATP is the energy source muscle fibers

use to make muscles contract

muscle tissuersquos main source of energy

called adenosine triphosphate or ATP

As your muscles use up this energy

source they become tired and fatigued

Oxygen is the key ingredient that helps

create new ATP to replenish the burned

up ATP in your muscles

Q207 Artocarpus integra is the

scientific name of आटह८कापयसइटीगरा mdashmdashmdash कावहॴजञाननकनामहहॴ 2-Feb-2017

Options

1) Guava

अम द

2) Pineapple

अनानास

3) Silver Oak

लसमवरओक

4) Jack fruit

कटहि

Correct Answer Jack fruit

Q208 Which organ stores fat soluble

vitamins

कह९नसाअगवसामघिनिीिपवटालमनह८काभिाराकरताहहॴ

2-Feb-2017

F A C E B O O K

P A G E h t t p w w w f a c e b o o k c o m s s c m e n t o r s o f f i c i a l P a g e | 48

FOR MORE UPDATES AND MORE MATERIAL DO LIKE OUR FACEBOOK PAGE httpwwwfacebookcomsscmentorsofficial

Options

1) Blood

रकत

2) Skin

तवचा 3) Liver

यकत

4) Pancreas

अगनयािय

Correct Answer Liver

Q209 Which disease is caused due to

deficiency of Iodine

आयह८िीनकहॳ कारणकह९नसारह८गहह८ताहहॴ 2-Feb-2017

Options

1) Rickets

ररकहॳ टस

2) Scurvy

सकवी 3) Goitre

गणमािा 4) Growth retardation

पवकासका कना Correct Answer Goitre

rickets A softening and weakening of

bones in children usually due to

inadequate vitamin D

Q210 Grevillea Robusta is the scientific name of

गरहॳपवलियारह८बसटा mdashmdashmdash- कापवजञाननकनामहहॴ 2-Feb-2017

Options

1) Peepal

पीपि

2) Teak

सागह९न

3) Silver Oak

लसमवरओक

4) Jack fruit

कटहि

Correct Answer Silver Oak

Q211 When a Cuttlefish is described as a Molluscs it is at which level of

classification

जबएककटिकफिकह८एकमह८िसकाकहॳ पमवखणयतककयाजाताहहॴतबयहॳवगीकरणकहॳ ककससतरपहॳनसितहहॴ 2-Feb-2017

Options

1) Class

वगय 2) Order

िम

3) Family

पररवार

4) Phylum

सघ

Correct Answer Phylum

Q212 Bambusa dendrocalmus is the

scientific name of बानबसािहॳडराकामस mdashmdashmdash कावहॴजञाननकनामहहॴ 3-Feb-2017

Options

1) Banyan

बरगद

2) Papaya

पपीता 3) Bamboo

बास

4) Pomegranate

अनार

Correct Answer Bamboo

Q213 Acinonyx Jubatus is the scientific name of

एलसनह८ननकसजयबहॳटस mdashmdashmdash

कावहॴजञाननकनामहहॴ 3-Feb-2017

F A C E B O O K

P A G E h t t p w w w f a c e b o o k c o m s s c m e n t o r s o f f i c i a l P a g e | 49

FOR MORE UPDATES AND MORE MATERIAL DO LIKE OUR FACEBOOK PAGE httpwwwfacebookcomsscmentorsofficial

Options

1) Bear

भाि 2) Horse

घह८िा 3) Cheetah

चीता 4) Zebra

जहॳिा Correct Answer Cheetah

Q214 The pale yellow colour of urine is

due to the presence of which pigment

मतरकाफीकापीिारगरगदरयकहॳ उपनसिनतकहॳ कारणहह८ताहहॴ

3-Feb-2017

Options

1) Urochrome

यरह८िह८म

2) Urophyll

यरह८कफि

3) Chlorophyll

किह८रह८कफि

4) Chloroplast

किह८रह८पिासट

Correct Answer Urochrome

Q215 Which of the following constitute

to form a gene

नननननलिखितमसहॳकह९नसीचीज़एकजीनकागठनकरतीहहॴ

3-Feb-2017

Options

1) Polynucleotides

पह८िीनयनकियह८टाईडस

2) Hydrocarbons

हाइडरह८काबोस

3) Lipoproteins

िाईपह८परह८टीनस

4) Lipids

लिपपडस

Correct Answer Polynucleotides

Polynucleotide molecule is a biopolymer

composed of 13 or more nucleotide

monomers covalently bonded in a chain

DNA (deoxyribonucleic acid) and RNA

(ribonucleic acid) are examples of

polynucleotides with distinct biological

function

Q216 Vertebrates belongs to the

phylum

रीढ़कीहडिीवािहॳपराणी mdashmdashmdash

परजानतकहॳ अतगायतआतहॳहहॴ 3-Feb-2017

Options

1) Arthropoda

आरह८पह८ड़ा 2) Annelida

एननलििा 3) Cnidaria

ननिहॳररया 4) Chordata

कह८िटा Correct Answer Chordata

Q217 Punica granatum is the scientific name of

पननकगरहॳनहॳटस mdashmdashmdash कावहॴजञाननकनामहहॴ 3-Feb-2017

Options

1) Custard Apple

सीताफि

2) Gulmohar

गिमह८हर

3) Silver Oak

लसमवरओक

4) Pomegranate

अनार

Correct Answer Pomegranate

F A C E B O O K

P A G E h t t p w w w f a c e b o o k c o m s s c m e n t o r s o f f i c i a l P a g e | 50

FOR MORE UPDATES AND MORE MATERIAL DO LIKE OUR FACEBOOK PAGE httpwwwfacebookcomsscmentorsofficial

Q218 Between a tiger and an monkey

which of the following is different

एकबाघऔरबदरकहॳ बीचनननननलिखितमसहॳकह९नसीबातअिगहहॴ 3-Feb-2017

Options

1) Kingdom

राजय

2) Phylum

जानत

3) Order

िम

4) Class

वगय Correct Answer order

Q219 The artificial heart was invented by

कबतरमहदयका mdashmdashmdash

दवाराअपवषकारककयागयािा 3-Feb-2017

Options

1) Muhammad Yunus

महनमदयनस

2) Linus Yale Jr

िाइनसयहॳिजय

3) Gazi Yasargil

गाजीयासचगयि

4) Paul Winchell

पह९िपवमकि Correct Answer Paul Winchell

Q220 Tamarindus indica is the

scientific name of

टहॳमररनडसइडिका mdashmdash कावहॴजञाननकनामहहॴ 7-

Feb-2017

Options

1) Neem

नीम

2) Pineapple

अनानास

3) Tamarind

इमिी 4)Chiku

चीक

Correct Answer Tamarind

Q221 In eukaryotic cells synthesis of

RNA takes place in the

यकहॳ योटटककह८लिकाओमआरएनएकासशिहॳषण

mdashndash महह८ताहहॴ 7-Feb-2017

Options

1) Mitochondria

माईटह८कोडडरया 2) Centrioles

सटरीयह८मस

3) Ribosomes

ररबह८सह८नस

4) Nucleus

नयनकियस

Correct Answer nucleus

eukaryotic cell -Transcription is the

process of synthesizing ribonucleic acid

(RNA)Synthesis takes place within the

nucleus of eukaryotic cells or in the

cytoplasm of prokaryotes and converts

the genetic code from a gene in

deoxyribonucleic acid ( DNA ) to a

strand of RNA that then directs

proteinsynthesis

Q222 _________is caused by parasites

of the Plasmodium genus

पिाजमह८डियमजातीकहॳ परजीवी mdash- कहॳ कारणहहॴ 7-Feb-2017

Options

1) Dysentery

पहॳचचि

2) Malaria

मिहॳररया 3) Chickenpox

F A C E B O O K

P A G E h t t p w w w f a c e b o o k c o m s s c m e n t o r s o f f i c i a l P a g e | 51

FOR MORE UPDATES AND MORE MATERIAL DO LIKE OUR FACEBOOK PAGE httpwwwfacebookcomsscmentorsofficial

चहॳचक

4) Herpes

हहॳपपयस

Correct Answer Malaria

Q223 Carotene in fruits and vegetables

gives it which color

फिह८औरसनलजयोमनसितकहॳ रह८टीनउनहकह९नसारगपरदानकरताहहॴ 7-Feb-2017

Options

1) Green

हरा 2) Pink

गिाबी 3) Orange

नारगी 4) Blue

नीिा Correct Answer Orange

Q224 Equus Caballus is the scientific

name of

एकवसकहॴ बहॳिस mdashmdashndash कापवजञाननकनामहहॴ 7-Feb-2017

Options

1) Horse

घह८िा 2) Zebra

ज़हॳिा 3) Donkey

गधा 4) Buffalo

भस

Correct Answer Horse

Q225 Elapidae Naja is the scientific name of

एिीपीिीनाजा mdashmdash- कावहॴजञाननकनामहहॴ 8-Feb-2017

Options

1) Cobra

कह८बरा 2) Elephant

हािी 3) Eagle

ग ि

4) Owl

उमि Correct Answer Cobra

Q226 Which disease is caused due to

deficiency of Iron

िह८हकीकमीकहॳ कारणकह९नसारह८गहह८ताहहॴ 8-Feb-

2017

Options

1) Beriberi

बहॳरीबहॳरी 2) Tetany

टहॳटनी 3) Kwashiorkor

कवािीऔरकर

4) Anaemia

रकतामपता Correct Answer Anaemia

Beriberi is a disease caused by a vitamin

B-1 deficiency also known as thiamine

deficiency

Tetany can be the result of an

electrolyte imbalance Most often itrsquos a

dramatically low calcium level also

known as hypocalcemia Tetany can also

be caused by magnesium deficiency or

too little potassium Having too much

acid (acidosis) or too much alkali

(alkalosis) in the body can also result in

tetany

Kwashiorkor also known as

ldquoedematous malnutrition It is a form of

malnutrition caused by a lack of protein

in the diet

Anaemia means that you have fewer red

blood cells than normal or you have less

F A C E B O O K

P A G E h t t p w w w f a c e b o o k c o m s s c m e n t o r s o f f i c i a l P a g e | 52

FOR MORE UPDATES AND MORE MATERIAL DO LIKE OUR FACEBOOK PAGE httpwwwfacebookcomsscmentorsofficial

haemoglobin than normal in each red

blood cell

Q227 is a leaf where the leaflets are

arranged along the middle vein

mdashndashएकपततीहहॴजहापतरकह८कीरचनाक ररयालिराकहॳ आसपासहह८तीहहॴ 8-Feb-2017

Options

1) Pinnately compound leaf

पपनहॳटिीसयकतपतती 2) Palmately compound leaf

पामहॳटिीसयकतपतती 3) Compound leaf

सयकतपतती 4) Simple leaf

साधारणपतती Correct Answer Pinnately compound

leaf

Q228 Haustoria or sucking roots are

found in which of the following

हह८सटह८ररयायाचसनहॳवािीजड़हॳनननननलिखितमसहॳककसमपाईजातीहहॴ 8-Feb-2017

Options

1) Wheat

गहॳह

2) Mango

आम

3) Chestnut

चहॳसटनट

4) Cuscuta

कसकयटा Correct Answer Cuscuta

Haustorial roots -The roots of parasitic

plants which penetrate into the host

tissues to absorb nourishment are

called haustorial roots hellip Also known as suckingor parasitic roots

Q229 Equs Asinus is the scientific name

of

एकवसएलसनस mdashmdashndash कावहॴजञाननकनामहहॴ 8-

Feb-2017

Options

1) Donkey

गधा 2) Cow

गाय

3) Deer

टहरन

4) Kangaroo

कगा

Correct Answer Donkey

Q230 Ficus benghalensis is the scientific name of

फाईकसबहॳनगहॳिहॳलसस mdashndash कापवजञाननकनामहहॴ 8-Feb-2017

Options

1) Banyan

बरगद

2) Pineapple

अनानास

3) Babul

बबि

4) Tulsi

तिसी Correct Answer Banyan

Q231 Equus burchellii is the scientific name of

एकवसबचिी mdashmdash- कापवजञाननकनामहहॴ 8-Feb-2017

Options

1) Horse

घह८िा 2) Zebra

जहॳिा 3) Buffalo

F A C E B O O K

P A G E h t t p w w w f a c e b o o k c o m s s c m e n t o r s o f f i c i a l P a g e | 53

FOR MORE UPDATES AND MORE MATERIAL DO LIKE OUR FACEBOOK PAGE httpwwwfacebookcomsscmentorsofficial

भस

4) Ass

गधा Correct Answer Zebra

Page 25: COMPILATION OF ALL 72 SETS OF BIOLOGY SSC CHSL-2016 · OF BIOLOGY SSC CHSL-2016 PREPARED BY : SSC MENTORS BIOLOGY SPECIAL . F A C E B O O K P A G E : h t t p : / / w w w . f a c e

F A C E B O O K

P A G E h t t p w w w f a c e b o o k c o m s s c m e n t o r s o f f i c i a l P a g e | 24

FOR MORE UPDATES AND MORE MATERIAL DO LIKE OUR FACEBOOK PAGE httpwwwfacebookcomsscmentorsofficial

3) Active Transport

सकियआवागमन

4) Passive Transport

नननषियआवागमन

Correct Answer Active Transport

Q100 Study of classification of

organisms is known as 20-Jan-2017

जीवाणओकहॳ वगीकरणकहॳ अधययनकह८ mdash-

कहाजाताहहॴ Options

1) Serpentology

सपरहॳटह८िह८जी 2) Virology

वायरह८िह८जी 3) Taxonomy

टहॴकसोनह८मी 4) Physiology

कफनज़यह८िह८जी Correct Answer Taxonomy

Q101 Photosynthesis takes place inside

plant cells in

परकािसशिहॳषणवनसपनतकह८लिकामनसति mdash

mdashmdash महह८ताहहॴ 20-Jan-2017

Options

1) Ribosomes

राइबह८सह८नस

2) Chloroplasts

किह८रह८पिासट

3) Nucleus

नयकलियम

4) Mitochondria

माईटह८कोडडरया Correct Answer Chloroplasts

Q102 ______ is the cell organelle in

which the biochemical processes of

respiration and energy production

occur

mdashmdash- वहकह८लिकाअगहहॴ नजसमहॳशवसनऔरउजायउतपादनकहॳ जहॴसीजहॴवरासायननकपरकियायहह८तीहहॴ 20-Jan-2017

Options

1) Mitochondria

माइटह८कोडडरया 2) Chloroplast

किह८रह८पिासट

3) Ribosomes

राइबह८सह८नस

4) Nucleus

नयकिीयस

Correct Answer Mitochondria

Q103 Which non-flowering spore

bearing plants have roots

ककसफिनिगनहॳवािहॳऔरबीजाणधारकपह९धह८कीजड़हॳहह८तीहहॴ 21-Jan-2017

Options

1) Mosses

मह८सहॳस

2) Angiosperms

एननजयह८सपनसय 3) Ferns

फनसय 4) Gymnosperms

नजननह८सपनसय Correct Answer ferns

Q104 Which of the following is an

excretory organ of cockroach

नननननलिखितमसहॳकह९नसानतिच हॳकाउतसजयनअगहहॴ

21-Jan-2017

Options

F A C E B O O K

P A G E h t t p w w w f a c e b o o k c o m s s c m e n t o r s o f f i c i a l P a g e | 25

FOR MORE UPDATES AND MORE MATERIAL DO LIKE OUR FACEBOOK PAGE httpwwwfacebookcomsscmentorsofficial

1) Malphigian Tubules

मनमफनजयनटयबमस

2) Nephridia

नहॳकफरडिया 3) Coxal Gland

कह८कसिगरचिया 4) Green Gland

गरीनगरचिया Correct Answer Malphigian Tubules

Q105 Evaporation of water takes place

in which part of plants

पानीकहॳ वाषपीकरणकीकियापह९धोकहॳ ककसभागसहॳहह८तीहहॴ 21-Jan-2017

Options

1) Stem

तना 2) Stomata

सटह८मटा 3) Branch

िािाए

4) Fruit

फि

Correct Answer Stomata

Evaporation accounts for the movement

of water to the air from sources such as

the soil canopy interception and

waterbodies Transpiration accounts for

the movement of water within a plant

and the subsequent loss of water as

vapour through stomata in its leaves

Q106 A is the fleshy spore-bearing

fruiting body of a fungus

mdashmdashndashकवककामासि

बीजाणधारणकरनहॳवािाफिनहॳवािाअगहहॴ 21-

Jan-2017

Options

1) aloe vera

एिह८वहॳरा 2) Coral

मगा 3) Cactus

कहॴ कटस

4) Mushroom

ककरमतता Correct Answer mushroom

Q107 Which of the following is a fungal

disease

नननननलिखितमसहॳकह९नसाफफदसहॳहह८नहॳवािाएकरह८ग हहॴ

21-Jan-2017

Options

1) Dermatitis

तवचािह८ध

2) Cholera

हहॴजा 3) Jaundice

पीलिया 4) Indigofera

इननिगह८फहॳ रा Correct Answer Dermatitis

Dermatitis also known as eczema is a

group of diseases that results in

inflammation of the skin These diseases

are characterized by itchiness red skin

and a rash In cases of short duration

there may be small blisters while in

long-term cases the skin may become

thickened

Q108 In which form is glucose stored in

our body

हमारहॳिरीरमगिकह८जकासचयककस पमककयाजाताहहॴ

21-Jan-2017

Options

1) Insulin

F A C E B O O K

P A G E h t t p w w w f a c e b o o k c o m s s c m e n t o r s o f f i c i a l P a g e | 26

FOR MORE UPDATES AND MORE MATERIAL DO LIKE OUR FACEBOOK PAGE httpwwwfacebookcomsscmentorsofficial

इसलिन

2) Glucose

गिकह८ज

3) Glycogen

गिायकह८जहॳन

4) Fat

वसा Correct Answer Glycogen

Excess glucose is stored in the liver as

the large compound called glycogen

Glycogen is a polysaccharide of glucose

but its structure allows it to pack

compactly so more of it can be stored in

cells for later use

Q109 Where do plants synthesize

protein from

पह९धहॳपरह८टीनसशिहॳषणकहासहॳकरतहॳहहॴ

Options

1) Fatty Acids

वसाऐलसि

2) Sugar

िकर

3) Amino Acids

एलमनह८ऐलसि

4) Starch

सटाचय Correct Answer Amino Acids

Q110 Which part of the brain is

responsible for triggering actions like

thinking intelligence memory and

ability to learn

मनसतषककाकह९नसाटहससासह८चनहॳ बनधदमानी याददाशतऔरसीिनहॳकीकषमताजहॴसीकियाओकह८परहॳररतकरताहहॴ 21-Jan-2017

Options

1) Diencephalon

िायएनसहॳफहॳ िह८न

2) Hypothalamus

हयपह८िहॳिहॳमस

3) Cerebrum

सहॳरहॳिम

4) Control

कटरह८ि

Correct Answer Cerebrum

Q111 Which of the following is also

known as the Biochemical Laboratory

of the Human Body

नननननलिखितमसहॳककसहॳमानविरीरकीजहॴवरसायनपरयह८गिािाभीकहाजाताहहॴ 21-Jan-2017

Options

1) Small Intestine

छह८टीआत

2)Brain

मनसतषक

3) Pancreas

अगनयािय

4) Liver

नजगर

Correct Answer Liver

The liver makes bile that will help

emulsify and digest the fats we eat

The liver takes toxic substances and

convert them using enzymes the liver

cells makes into a non toxic form so the

body can dispose of them

The liver also converts fats protein and

carbohydrates into glucose which is the

energy source for our cells to use

The liver takes amino acids and makes

proteins by combining them

Q112 The yellow colour of human urine

is due to

मानवमतरकापीिारग mdashndash कीवजहसहॳहह८ताहहॴ 22-

Jan-2017

Options

1) Bile Salts

F A C E B O O K

P A G E h t t p w w w f a c e b o o k c o m s s c m e n t o r s o f f i c i a l P a g e | 27

FOR MORE UPDATES AND MORE MATERIAL DO LIKE OUR FACEBOOK PAGE httpwwwfacebookcomsscmentorsofficial

पपततनमक

2) Cholesterol

कह८िहॳसटरह८ि

3) Lymph

लिनफ

4) Urochrome

यरह८िह८म

Correct Answer Urochrome

Urobilin or urochrome is the chemical

primarily responsible for the yellow

color of urine

Q113 The wilting of plants takes place

due to

पह९धह८कालिचििहह८नाकी mdashmdash- कीवजहसहॳहह८ताहहॴ 22-Jan-2017

Options

1)Photosynthesis

परकािसशिहॳषण

2) Transpiration

वाषपह८तसजयन

3) Absorption

अविह८षण

4) Respiration

शरवसन

Correct Answer Transpiration

Wilting is the loss of rigidity of non-

woody parts of plants This occurs when

the turgor pressure in non-lignified

plant cells falls towards zero as a result

of diminished water in the cells

Q114 Bovidae Ovis is the scientific name of

बह८पविीओपवस mdashndash कावहॴजञाननकनामहहॴ 22-Jan-2017

Options

1) Goat

बकरी 2) Cow

गाय

3) Buffalo

भहॳस

4) Sheep

भहॳड़

Correct Answer Sheep

Q115 Plants get their energy to produce

food from which of the following

पह८धहॳभह८जनकाननमायणकरनहॳकहॳ लिएनननननलिखितमसहॳककससहॳउजायपरापतकरतहॳहहॴ

22-Jan-2017

Options

1) Photosynthesis

परकािसशिहॳषण

2)Bacteria

बहॴकटीररया 3)Fungi

कवक

4)Sun

सयय Correct Answer Sun

Q116 Which of the following is secreted

by the liver

नननननलिखितमसहॳककसकासरावनजगरसहॳहह८ताहहॴ

22-Jan-2017

Options

1) Glucose

गिकह८ज

2) Iodine

आयह८िीन

3) Cortisol

काटटरयसह८ि

4) Bile

पपतत

Correct Answer Bile

The liver makes bile that will help

emulsify and

digest the fats we eat

F A C E B O O K

P A G E h t t p w w w f a c e b o o k c o m s s c m e n t o r s o f f i c i a l P a g e | 28

FOR MORE UPDATES AND MORE MATERIAL DO LIKE OUR FACEBOOK PAGE httpwwwfacebookcomsscmentorsofficial

Q117 Ferns belong to which division of

plants

फनसयपह९धह८कहॳ ककसभागमआतहॳहहॴ

22-Jan-2017

Options

1) Gymnosperms

नजननह८सपनसय 2) Angiosperms

एनजयह८सपनसय 3) Thallophyta

िहॴिह८फाईटा 4)Pteridophyta

टहॳररिह८फाईटा Correct Answer Pteridophyta

Q118 Who invented Antibiotics

एटीबायह८टटककाअपवषकारककसनहॳककयािा

22-Jan-2017

Options

1) Joseph Lister

जह८सहॳफलिसटर

2) William Harvey

पवलियमहाव

3) Robert Knock

रॉबटयनॉक

4)Alexander Fleming

अिहॳकज़िरफिहॳलमग

Correct Answer Alexander Fleming

Q119 Milbecycin is used in the

eradication of

लममबहॳसायलसनका mdashndash

मउनमिनमपरयह८गककयाजाताहहॴ 22-Jan-2017

Options

1) Agricultural Fungus

कपषकवक

2) Agricultural Pests

कपषकीटक

3) Agricultural Herbs

कपषिाक

4)Agricultural Weeds

कपषननराना Correct Answer Agricultural Pests

Milbemycin oxime is a veterinary drug

from the group of milbemycins used as

a broad spectrum antiparasitic It is

active against worms and mites(insects

Q120 Intestinal bacteria synthesizes

which of the following in the human

body

मानविरीरमआतोकहॳ बहॴकटीररयानननननलिखितमसहॳककसकासशिहॳषणकरतहॳहहॴ 22-Jan-2017

Options

1) Vitamin K

पवटालमन K

2) Proteins

परह८टीन

3) Fats

वसा 4) Vitamin D

पवटालमन D

Correct Answer Vitamin K

Q121 is the study of the physical form

and external structure of plants

mdashmdash-

मपह९धह८काभहॴनतक पऔरबाहरीसरचनाकाआदयाककयाजाताहहॴ 22-Jan-2017

Options

1) Physiology

कफनजयह८िह८जी 2) Anatomy

िरीररचनापवजञान

3) Phytomorphology

फाईटह८मह८फह८िह८जी 4)Cytology

कह८लिकापवजञान

Correct Answer Phytomorphology

F A C E B O O K

P A G E h t t p w w w f a c e b o o k c o m s s c m e n t o r s o f f i c i a l P a g e | 29

FOR MORE UPDATES AND MORE MATERIAL DO LIKE OUR FACEBOOK PAGE httpwwwfacebookcomsscmentorsofficial

Q122 Which of the following is a

structural and functional unit of

kidneys

नननननलिखितमसहॳकह९नसीगदोकीसरचनातमकऔरकाययकरीईकाईहहॴ

22-Jan-2017

Options

1) Renette Cells

रहॳनहॳटकह८लिकाए

2) Flame Cells

फिहॳमकह८लिकाए

3) Nephrites

नहॳफ़राइटस

4)Nephrons

नहॳफरोस

Correct Answer Nephrons

Nephron functional unit of the kidney

the structure that actually produces

urine in the process of removing waste

and excess substances from the blood

There are about 1000000 nephrons in

each human kidney

Q123 Which of the following is the

largest part of the human brain

नननननलिखितमसहॳकह९नसामानवमनसतषककासबसहॳबड़ाटहससाहहॴ

23-Jan-2017

Options

1) Ribs

पसलियाा 2) Cerebrum

सहॳरहॳिम

3) Pons

पोस

4)Thalamus

िहॴिहॳमस

Correct Answer Cerebrum

The cerebrum is the largest part of the

human brain making up about two-

thirds of the brainrsquos mass It has two

hemispheres each of which has four

lobes frontal parietal temporal and

occipital

Q124 The auxiliary buds

सहायककालियाmdashndash 23-Jan-2017

Options

1) grow endogenously from the pericycle

पहॳरीसाईककिसहॳअनतजातयपवकलसतहह८ताहहॴ 2) arise endogenously from the main

growing point

मिवपदसहॳअनतजातयउठताहहॴ 3) is an embryonic shoot located in the

axil of a leaf

एकभरणिटहहॴजह८एकपततीकहॳ अकषपरनसतिहह८ताहहॴ 4)arise exogenously from the epidermis

एपपिलमयससहॳबटहजातयतरीकहॳ सहॳउठताहहॴ Correct Answer is an embryonic shoot

located in the axil of a leaf

Q125 Which of the following is a viral

disease

इनमहॳसहॳकह९सीएकवायरिबीमारीहहॴ

23-Jan-2017

Options

1) Polio

पह८लियह८ 2) Tetanus

धनसतनभ

3) Leprosy

कषठरह८ग

4) Plague

पिहॳग

Correct Answer Polio

A viral disease (or viral infection)

occurs when an organismrsquos body is

invaded by pathogenic viruses and

infectious virus particles (virions) attach

to and enter susceptible cells

F A C E B O O K

P A G E h t t p w w w f a c e b o o k c o m s s c m e n t o r s o f f i c i a l P a g e | 30

FOR MORE UPDATES AND MORE MATERIAL DO LIKE OUR FACEBOOK PAGE httpwwwfacebookcomsscmentorsofficial

Poliomyelitis often called polio or

infantile paralysis is an infectious

disease caused by the poliovirus

Tetanusmdash A serious bacterial infection

that causes painful muscle spasms and

can lead to death

Leprosy also known as Hansenrsquos

disease (HD) is a long-term infection by

the bacterium Mycobacterium leprae or

Mycobacterium lepromatosis

Plague is an infectious disease caused by

the bacterium Yersinia pestis

Symptoms include fever weakness and

headache

Q126 Which organisms can help to

carry out Vermicomposting

कह९नसाजीववमीकनपह८नसटगममददकरताहहॴ

23-Jan-2017

Options

1) Nitrifying Bacteria

नाईटरीफाईगबहॴकटीररया 2) Earthworms

कहॴ चऐ

3) Algae

िहॴवि

4) Fungus

कवक

Correct Answer Earthworms

Q127 Contraction of heart is also

known as

हदयकहॳ सकचनकह८ mdash- भीकहाजाताहहॴ 23-Jan-

2017

Options

1) Systole

लससटह८ि

2) Aristotle

अरसत

3) Diastole

िायसटह८ि

4) Lub

मयब

Correct Answer Systole

Diastole is the part of the cardiac cycle

when the heart refills with blood

following systole (contraction)

Ventricular diastole is the period during

which the ventricles are filling and

relaxing while atrial diastole is the

period during which the atria are

relaxing

Q128 Azadirachta indica is the

botanical name of which of the

following

अजाटदराचताइडिकानननननलिखितमसहॳककसकावानसपनतनामहहॴ

23-Jan-2017

Options

1) Rose plant

गिाबकापह९धा 2) Apple tree

सहॳबकापहॳड़

3) Neem

नीम

4)Mango

आम

Correct Answer Neem

Q129 Which of the following is the

main end product of carbohydrate

digestion

नननननलिखितमसहॳकह९नसाकाबोहाइडरहॳटकहॳ पाचनकापरमिअतउतपादकहह८ताहहॴ 23-Jan-2017

Options

1) Fats

वसा 2) Lipids

लिपपडस

3) Glucose

गिकह८ज

4) Cellulose

F A C E B O O K

P A G E h t t p w w w f a c e b o o k c o m s s c m e n t o r s o f f i c i a l P a g e | 31

FOR MORE UPDATES AND MORE MATERIAL DO LIKE OUR FACEBOOK PAGE httpwwwfacebookcomsscmentorsofficial

सहॳमयिह८ज

Correct Answer Glucose

Intestinal absorption of end products

from digestion of carbohydrates and

proteins in the pig hellip During absorption some sugars (fructose or

galactose) released from the

corresponding sucrose and lactose

respectively during digestion were

partly metabolized into glucose by the

enterocyte

Q130 Which of the following glands is a

source of the enzyme Ptyalin

नननननलिखितगरचियोमसहॳएजाइमटयालिनकासरह८तहहॴ 23-Jan-2017

Options

1) Pancreas

अगरािय

2) Thyroid Gland

िाइराइिगरिी 3) Pituitary Gland

पीयषगरिी 4) Salivary Glands

िारगरचियाा Correct Answer Salivary Glands

Q131 Which of the following is not true

about Pteridophyta

ननननमसहॳकह९नसीबातटहॳररिह८फाईटकहॳ बारहॳमसचनहीहहॴ 23-Jan-2017

Options

1) Dominant phase is saprophytes

परमिचरणसहॳपरह८फाईइटसहह८ताहहॴ 2) Main plant body is diploid

पह९दह८कामखयिरीरदपवगखणतहह८ताहहॴ 3) Seeds are present

बीजमह९जदहह८तहॳहहॴ 4)Flowers are absent

फिअनपनसतिहह८तहॳहहॴ

Correct Answer Seeds are present

Q132 The largest dolphin species is the

orca also called as

िॉिकफनकीसबसहॳबड़ीपरजानतकाकानामआकायहहॴनजसहॳ mdash- भीकहतहॳहहॴ 23-Jan-2017

Options

1) Bottle Nose

बाटिनह८ज

2) Baiji

बहॳजी 3) Killer whale

ककिरहहॳि

4)Tucuxi

टकवसी Correct Answer Killer whale

Q133 The fat digesting enzyme Lipase

is secreted by which of the following

वसाकापाचनकरनहॳवािाएजाइमिाइपहॳजनननननलिखितमसहॳककसकहॳ दवारासतरापवतहह८ताहहॴ

24-Jan-2017

Options

1) Kidneys

गद

2) Pancreas

अगनयािय

3) Large Intestine

बड़ीआत

4)Liver

नजगर

Correct Answer Pancreas

Lipase is an enzyme that splits fats so

the intestines can absorb them Lipase

hydrolyzes fats like triglycerides into

their component fatty acid and glycerol

molecules It is found in the blood

gastric juices pancreatic secretions

intestinal juices and adipose tissues

F A C E B O O K

P A G E h t t p w w w f a c e b o o k c o m s s c m e n t o r s o f f i c i a l P a g e | 32

FOR MORE UPDATES AND MORE MATERIAL DO LIKE OUR FACEBOOK PAGE httpwwwfacebookcomsscmentorsofficial

Q134 The arrangement of leaves on an

axis or stem is called

एकअकषयातनहॳपरपनततयोकीयवसिाकह८कयाकहाजाताहहॴ SSC CHSL Science (biology) 2016

Question Paper

24-Jan-2017

Options

1) Phyllotaxy

फाइिह८टहॴकसी 2) Vernation

वनिन

3) Venation

वहॳनहॳिन

4)Phytotaxy

फाइटह८टहॴकसी Correct Answer Phyllotaxy

In botany phyllotaxis or phyllotaxy is

the arrangement of leaves on a plant

stem (from Ancient Greek phyacutellon

ldquoleafrdquo and taacutexis ldquoarrangementrdquo)

Phyllotactic spirals form a distinctive

class of patterns in nature

Q135 The study of Cells is also known

as

कह८लिकाओकहॳ अधययनकह८ mdashmdashndash

भीकहाजाताहहॴ 24-Jan-2017

Options

1) Cytology

सायटह८िह८जी 2) Physiology

कफनजयह८िह८जी 3) Nucleology

नयककमयह८िह८जी 4)Cellology

सहॳिह८िह८जी Correct Answer Cytology

Q136 Which of the following scientists

is also known as the Father of Biology

नननननलिखितमसहॳककसवहॴजञाननककह८ ldquoजीवपवजञानकहॳ जनकrdquoकहॳ नामसहॳभीजानाजाताहहॴ 24-Jan-2017

Options

1) Herbert Spencer

हबयटयसपसर

2) Aristotle

अरसत 3) Lamarck

िहॳमाकय 4)Darwin

िापवयन

Correct Answer Aristotle

Q137 Which cells give rise to various

organs of the plant and keep the plant

growing

कह९नसीकह८लिकाएपह९धह८कहॳ लभननअगह८कह८जनमदहॳतीहहॴऔरपह९धह८कह८बढ़नहॳममददकरतीहहॴ

24-Jan-2017

Options

1) Permanent

सिायी 2) Dermal

तवचीय

3) Meristematic

मररसटहॳमटटक

4)Mature

परह८ढ़

Correct Answer Meristematic

A meristem is the tissue in most plants

containing undifferentiated cells

(meristematic cells) found in zones of

the plant where growth can take place

Q138 Rodentia Muridae is the scientific

name of

F A C E B O O K

P A G E h t t p w w w f a c e b o o k c o m s s c m e n t o r s o f f i c i a l P a g e | 33

FOR MORE UPDATES AND MORE MATERIAL DO LIKE OUR FACEBOOK PAGE httpwwwfacebookcomsscmentorsofficial

रह८िहॳलियानयररिी mdashmdash- कावहॴजञाननकनामहहॴ 24-

Jan-2017

Options

1) Mouse

चहा 2) Squirrel

चगिहरी 3) Monkey

बदर

4) Lizard

नछपकिी Correct Answer Mouse

Q139 Name the scientist who proposed

the cell theory

कह८लिकालसदातकापरसतावदहॳनहॳवािहॳवहॴजञाननककानामबताइए 24-Jan-2017

Options

1) Schleiden and Schwann

िीमिनऔरशरववान

2) Lamarck

िहॳमाकय 3) Treviranus

टरहॳवायरहॳनस

4)Whittaker and Stanley

हीटकरऔरसटहॳनिहॳ Correct Answer Schleiden and

Schwann

Q140 The flower with the worldrsquos

largest bloom is

दननयाकासबसहॳबड़ाफिखििनहॳवािा mdashmdashndash हहॴ 24-Jan-2017

Options

1) Pando

पािह८ 2) Posidonia

पह८सीिह८ननया 3) Rafflesia arnoldii

ररफिहॳलियाअनोमिी 4)Helianthus annuus

हहॳलिएनिसएनयअस

Correct Answer Rafflesia arnoldii

Rafflesia arnoldii is a species of

flowering plant in the parasitic genus

Rafflesia It is noted for producing the

largest individual flower on earth It has

a very strong and horrible odour of

decaying flesh earning it the nickname

ldquocorpse flower

Q141 Deficiency of which vitamin

causes night blindness

ककसपवटालमनकीकमीकहॳ कारणरतौधीहह८ताहहॴ 24-Jan-2017

Options

1) Vitamin K

पवटालमन K

2) Vitamin C

पवटालमन C

3) Vitamin B1

पवटालमन B1

4)Vitamin A

पवटालमन A

Correct Answer Vitamin A

Q142 Nongreen plants lack which of the

following

गहॴर-

हररतवनसपनतमनननननलिखितमसहॳककसकीकमीहह८तीहहॴ

24-Jan-2017

Options

1) Chlorophyll

किह८रह८कफि

2) Lycophyll

िायकह८कफि

3) Cyanophyll

F A C E B O O K

P A G E h t t p w w w f a c e b o o k c o m s s c m e n t o r s o f f i c i a l P a g e | 34

FOR MORE UPDATES AND MORE MATERIAL DO LIKE OUR FACEBOOK PAGE httpwwwfacebookcomsscmentorsofficial

सायनह८कफि

4)Phototropism

फह८टह८टरोपपजम

Correct Answer Chlorophyll

Q143 Organisms that use light to

prepare food are known as

जह८जीवपरकािकाउपयह८गकरभह८जनतहॴयारकरतहॳहहॴ उनह mdashmdash- कहॳ पमजानजाताहहॴ 24-Jan-2017

Options

1) Autotrophs

सवपह८षी 2) Heterotrophs

पवषमपह८षज

3) Omnivores

सवायहारी 4)Decomposers

पवघटनकरनहॳवािा Correct Answer Autotrophs

autotrophs often make their own food

by using sunlight carbon dioxide and

water to form sugars which they can use

for energy Some examples of

autotrophs include plants algae and

even some bacteria Autotrophs

(producer) are important because they

are a food source for heterotrophs

(consumers)

A heterotroph is an organism that

ingests or absorbs organic carbon

(rather than fix carbon from inorganic

sources such as carbon dioxide) in order

to be able to produce energy and

synthesize compounds to maintain its

life Ninety-five percent or more of all

types of living organisms are

heterotrophic including all animals and

fungi and some bacteria

Q144 Which of the following is a

primary function of haemoglobin

नननननलिखितमसहॳकह९नसाटहमह८गिह८बबनकाएकपरािलमककाययहहॴ

25-Jan-2017

Options

1) Utilization of energy

उजायकाउपयह८गकरना 2) Prevention of anaemia

रकतामपताहह८नहॳसहॳरह८कना 3) Destruction of bacteria

बहॴकटीररयाकापवनािकरना 4) To transport oxygen

ऑकसीजनकावहनकरना Correct Answer To transport oxygen

Q145 Vascular bundles are absent in

सवहनीबिि mdashmdash- मअनपनसतिरहतहॳहहॴ 25-Jan-2017

Options

1) Bryophyta

िायह८फाइटा 2) Pteridophyta

टहॳररिह८फाईटा 3) Gymnosperms

नजननह८सपमय 4) Angiosperms

एननजयह८सपहॳनसय Correct Answer Bryophyta

Q146 Sauria Lacertidae is the scientific

name of

सहॴररयािहॳसरटाईिी mdashmdashndash कावहॴजञाननकनामहहॴ 25-Jan-2017

Options

1) Crocodile

मगरमचछ

2) Hippopotamus

टहपपह८पह८टहॳमस

3) Lizard

नछपकिी 4) House fly

F A C E B O O K

P A G E h t t p w w w f a c e b o o k c o m s s c m e n t o r s o f f i c i a l P a g e | 35

FOR MORE UPDATES AND MORE MATERIAL DO LIKE OUR FACEBOOK PAGE httpwwwfacebookcomsscmentorsofficial

घरहॳिमकिी Correct Answer Lizard

Q147 Which type of pathogen causes

the water-borne disease SARS (Severe

Acute Respiratory Syndrome)

ककसपरकािकारह८गज़नकजिजननतबीमारीसासयकाकारणबनताहहॴ 25-Jan-2017

Options

1) Viral

वायरि

2) Parasitic

परजीवी 3) Protozoan

परह८टह८जअन

4) Bacterial

बहॴकटीररयि

Correct Answer Viral

Q148 Which of the following organs

produces the enzyme lipase

नननननलिखितमसहॳकह९नसाअगिायपहॳजएजाइमउतपननकरताहहॴ 25-Jan-2017

Options

1) Pancreas

अगनयािय

2) Large Intestine

बड़ीआत

3) Liver

नजगर

4) Small Intestine

छह८टीआत

Correct Answer Pancreas

Q149 A is a long internode forming the

basal part or the whole of a peduncle

एक mdashmdash- एकिबाइटरनह८िहहॴ जह८ननचिाटहससायासनपणयिठिबनताहहॴ 25-

Jan-2017

Options

1) Rhizome

परकद

2) Rachis

महॳ दि

3) floral axis

पषपअकष

4) Scape

भगदड़

Correct Answer scape

Q150 ndash Which of the following

organisms are considered to be both

Living and Non-living

नननननलिखितमसहॳकह९नसहॳजीवाणकह८जीपवतऔरअजीपवतमानाजाताहहॴ

25-Jan-2017

Options

1) Bacteria

बहॴकटीररया 2) Fungi

कवक

3) Algae

िहॴवाि

4)Virus

वायरस

Correct Answer Virus

They are considered to be living as they

possess a protein coat as a protective

covering DNA as the genetic material

etc

They are said to be non-living as they

can be crystallised and they survive for

billions of years They can tolerate high

temperatures freezing cold

temperatures ultra-violet radiations etc

Q151 Deficiency of fluorine causes

which of the following

फिह८ररनकीकमीकहॳ कारणनननननलिखितमसहॳकयाहह८ताहहॴ

F A C E B O O K

P A G E h t t p w w w f a c e b o o k c o m s s c m e n t o r s o f f i c i a l P a g e | 36

FOR MORE UPDATES AND MORE MATERIAL DO LIKE OUR FACEBOOK PAGE httpwwwfacebookcomsscmentorsofficial

27-Jan-2017

Options

1) Dental Caries

िटिकहॴ ररज

2) Scurvy

सकवरी 3) Anaemia

रकतामपता 4) Arthritis

गटठया Correct Answer Dental Caries

Q152 In a Punnett Square with the

cross AaBb x AaBb how many Aabb

genotypes would be created

पनहॳटसककायरमिह८स AaBb x AaBb कहॳ साि

ककतनहॳ Aabb जीनह८टाइपबनगहॳ 27-Jan-2017

Options

1) 1

2) 8

3) 2

4) 3

Correct Answer 2

Q153 Which of the following is the

Controlling Center of the Cell

नननननलिखित म सहॳ कह८लिकाका ननयतरण

क दर कह९न हहॴ

27-Jan-2017

Options

1) Nucleus

क दर

2) Plasma

पिाजमा 3) Lysosome

िायसह८सह८म

4) Chromosome

िह८मह८सह८म

Correct Answer Nucleus

The control centre of the cell is the

nucleus in eukaryotic cells The nucleus

contains genetic material in the form of

DNA

Q154 Myopia affects which of the

following organs

मायह८पपयानननननलिखितअगह८मसहॳककसहॳपरभापवतकरताहहॴ

25-Jan-2017

Options

1) Heart

हदय

2) Skin

तवचा 3) Eyes

आािहॳ 4)Mouth

मह

Correct Answer Eyes

Q155 Which of the following bears

flowers

नननननलिखितमसहॳकह९नफिधारणकरताहहॴ

25-Jan-2017

Options

1) Bryophyta

िायह८फाइटा 2) Pteridophyta

टहॳरीिह८फाईटा 3) Gymnosperms

नजननह८सपमय 4)Angiosperms

एननजयह८सपमय Correct Answer Angiosperms

Q156 Oxygenated blood flows out of the

heart through the

ऑकसीजनयकतरकत mdashmdashmdash

कहॳ माधयमसहॳहदयकहॳ बाहरबहताहहॴ 25-Jan-2017

F A C E B O O K

P A G E h t t p w w w f a c e b o o k c o m s s c m e n t o r s o f f i c i a l P a g e | 37

FOR MORE UPDATES AND MORE MATERIAL DO LIKE OUR FACEBOOK PAGE httpwwwfacebookcomsscmentorsofficial

Options

1) Aorta

महाधमनी 2) pulmonary artery

फहॳ फड़हॳकीधमनी 3) vena cava

वहॳनाकावा 4)Atrium

चह९क

Correct Answer aorta

Q157 Blood leaving the liver and

moving towards the

heart has a higher concentration of

नजगरसहॳननकिकरहदयकीतरफजानहॳवािहॳरकतम mdashmdashmdashmdash कीउचचसादरताहह८तीहहॴ 27-Jan-2017

Options

1) Lipids

लिपपडस

2) Urea

यररया 3) Bile Pigments

पपततकहॳ रगकरण

4) Carbon dioxide

काबयनिायऑकसाइि

Correct Answer Bile Pigments

Urea is nitrogen containing substance

which is produced in the liver in order

to deal with excess amino-acids in the

body As urea is produced it leaves the

liver in the blood stream and passes via

the circulatory system to all parts of the

body

Q158 Bulb is a modification of which

part of a plant

बमबएकपह९धहॳकहॳ ककसटहससहॳकाएक पातरणहह८ताहहॴ 27-Jan-2017

Options

1) The root

जड़

2) The stem

तना 3) The radicle

मिाकर

4)The fruit

फि

Correct Answer The stem

Q159 Which of the following carries

blood away from the heart to different

body parts

इनमहॳसहॳकह९नरकतकह८हदयसहॳिरीरकहॳ पवलभननअगह८तकिहॳजातीहहॴ

27-Jan-2017

Options

1) Arteries

धमननया 2) Nerves

तबतरहाए

3) Capillaries

कहॳ लिकाए

4)Veins

नसहॳ Correct Answer Arteries

Q160 The series of processes by which

nitrogen and its compounds are

interconverted in the environment and

in living organisms is called

27-Jan-2017

Options

1)Absorption of Nitrogen

2)Ammonification

3)Nitrogen Fixation

4)Nitrogen Cycle

Correct Answer Nitrogen Cycle

Ammonification or Mineralization is

performed by bacteria to convert

organic nitrogen to ammonia

F A C E B O O K

P A G E h t t p w w w f a c e b o o k c o m s s c m e n t o r s o f f i c i a l P a g e | 38

FOR MORE UPDATES AND MORE MATERIAL DO LIKE OUR FACEBOOK PAGE httpwwwfacebookcomsscmentorsofficial

Nitrification can then occur to convert

the ammonium to nitrite and nitrate

Nitrogen fixation is a process by which

nitrogen in the Earthrsquos atmosphere is

converted into ammonia (NH3) or other

molecules available to living organisms

Q161 BCG vaccine is given to protect

from which of the following

बीसीजीकाटटकानननननलिखितमसहॳककसकहॳ बचावकहॳ लिएटदयाजातहहॴ

27-Jan-2017

Options

1) Jaundice

पीलिया 2) Anaemia

रकतमपता 3) Tuberculosis

कषयरह८ग

4) Polio

पह८लियह८ Correct Answer Tuberculosis

Q162 Parallel venation is found in

समानतरवहॳनहॳिन mdashmdashmdash- मपायाजाताहहॴ 27-Jan-2017

Options

1) plants which are monocots

पह९धहॳजह८एकबीजपतरीहह८तहॳहहॴ 2) plants which have a dicot stem

वहॳपह९धहॳनजनकातनादपवदलियहह८ताहहॴ 3) plants with leaves similar to Tulsi

वहॳपह९धहॳनजनकीपनततयतिसीकीपनततयोकहॳ समानहह८तहॳहहॴ 4)plants with tap roots

टहॳप टवािहॳपह९धहॳ Correct Answer plants which are

monocots

Q163 The hardest part of the body is

िरीरकासबसहॳकठह८रभाग mdashndash हहॴ 27-Jan-2017

Options

1) Bones

हडडिय

2) Tooth Enamel

दातकहॳ इनहॳमि

3) Skull

िह८पड़ी 4) Spinal Cord

महॳ रजज

Correct Answer Tooth Enamel

Q164 Which type of pathogen causes

the waterborne disease E coli Infection

ककसपरकारकारह८गजननकजिजननतरह८गईकह८िाईसिमणकाकारणबनताहहॴ 27-Jan-2017

Options

1) Protozoan

परह८टह८जआ

2) Parasitic

परजीवी 3) Bacterial

बहॴकटीररयि

4)Viral

वायरि

Correct Answer Bacterial

Q165 The amount of blood filtered

together by both the kidneys in a 70 kg

adult male human in a minute is

70 की गरा वािहॳएकवयसकप षमएकलमनटमदह८नोगदकहॳदवाराएकसािचाबनीगयीरकतकीमातरहह८तीहहॴ 29-Jan-2017

Options

1) 1100 ml

1100 लमलि

2) 100 ml

F A C E B O O K

P A G E h t t p w w w f a c e b o o k c o m s s c m e n t o r s o f f i c i a l P a g e | 39

FOR MORE UPDATES AND MORE MATERIAL DO LIKE OUR FACEBOOK PAGE httpwwwfacebookcomsscmentorsofficial

100 लमलि

3) 1500 ml

1500 लमलि

4) 500 ml

500 लमलि

Correct Answer 1100 ml

Q166 Which feature of a plant helps to

distinguish a monocot from a dicot

पह९धहॳकीवहकह९नसीपविहॳषताहहॴजह८एकदपवदलियहॳऔरएकएकदिीयपह९धहॳसहॳभहॳदकरनहॳममददकरतीहहॴ 29-Jan-2017

Options

1) Pollination

परागम

2) Venation

वहॳनहॳिन

3) Vernation

वनिन

4) Aestivation

एसटीवहॳिहॳन

Correct Answer venation

Q167 The Mutation Theory was

proposed by

उतवररवतयनकालसदात mdashmdashndash

कहॳ दवरापरसतापवतककयाजाताहहॴ 29-Jan-2017

Options

1) Charles Lyell

चामसयलियहॳि

2) William Smith

पवलियमनसमि

3) Hugo De Vries

हयगह८िीराईस

4)Harrison Schmitt

हहॳरीसननसमट

Correct Answer Hugo De Vries

Q168 Which type of pathogen causes

the waterborne disease HepatitisA

ककसपरकारकहॳ रह८गजनकजिजननतरह८गहहॳपहॳटाइटटस-A काकारणबनताहहॴ

29-Jan-2017

Options

1) Parasitic

परजीवी 2) Viral

वायरि

3) Protozoan

परह८टह८जआ

4) Bacterial

बहॴकटीररयि

Correct Answer Viral

Q169 In a Punnett Square with the

cross AaBb x Aabb how many AaBb

genotypes would be created

पनहॳटसकवायरमिह८स AaBb x Aabb

कहॳ सािककतनहॳ AaBb जीनह८टाइपबनगहॳ 29-Jan-

2017

Options

1) 4

2) 1

3) 7

4) 6

Correct Answer 4

Q170 Arboreal Ateles is the scientific

name of

अिह८ररयिएटटलिस mdashmdashmdash कावहॴजञाननकनामहहॴ 29-Jan-2017

Options

1) Squirrel

चगिहरी 2) Sparrow

गह८रहॴया 3) Lizard

नछपकिी 4) Spider monkey

F A C E B O O K

P A G E h t t p w w w f a c e b o o k c o m s s c m e n t o r s o f f i c i a l P a g e | 40

FOR MORE UPDATES AND MORE MATERIAL DO LIKE OUR FACEBOOK PAGE httpwwwfacebookcomsscmentorsofficial

मकड़ीबदर

Correct Answer Spider monkey

Q171 Which type of pathogen causes

the waterborne disease Salmonellosis

ककसपरकारकारह८गाणजिजननतबीमारीसािमह८नहॳिह८लसज़काकारकहहॴ

29-Jan-2017

Options

1) Algal

िहॳवालियहॳ 2) Parasitic

परजीवी 3) Bacterial

बहॴकटीररयि

4)Viral

वायरि

Correct Answer Bacterial

An infection with salmonella bacteria

commonly caused by contaminated food

or water

Symptoms include diarrhoea fever

chills and abdominal pain

Q172 is a condition in which there is a

deficiency of red cells or of haemoglobin

in the blood

mdashmdash-

एकनसिनतहहॴनजसमहॳरकतमिािकह८लिकाओकीयाहीमह८गिह८बबनकीकमीहह८तीहहॴ 29-Jan-2017

Options

1) Albinism

एनमबननजम

2) Propyria

परह८पीररया 3) Anaemia

एनीलमया 4)Keloid disorder

कहॳ िह८इिडिसओिर

Correct Answer Anaemia

Q173 Ananas comosus is the scientific

name of

Options

अनानासकह८मह८सस mdashmdashmdashndash

कावहॴजञाननकनामहहॴ 29-Jan-2017

1) Custard Apple

सीताफि

2) Pineapple

पाइनएपपि

3) Bamboo

बास

4)Pomegranate

अनार

Correct Answer Pineapple

Q174 Which organ produces insulin

कह९नसाअगइनसलिनपहॴदाकरताहहॴ 29-Jan-

2017

Options

1) Liver

यकत

2) Thyroid gland

िायराइिगरिी 3) Spleen

पिीहा 4)Pancreas

अगरयिय

Correct Answer Pancreas

Q175 Which of the following disease is

not caused by water pollution

नननननलिखितमसहॳकह९नसारह८गपानीकहॳ परदषणकहॳकारणनहीहह८ता

29-Jan-2017

Options

1) Cholera

हहॴजा 2) Typhoid

F A C E B O O K

P A G E h t t p w w w f a c e b o o k c o m s s c m e n t o r s o f f i c i a l P a g e | 41

FOR MORE UPDATES AND MORE MATERIAL DO LIKE OUR FACEBOOK PAGE httpwwwfacebookcomsscmentorsofficial

टाइफाइि

3) Asthma

दमा 4)Diarrhoea

दसत

Correct Answer Asthma

Q176 Ocimum tenuiflorum is the

scientific name of

ओलिलममटहॳयईफिह८रमइसकावहॴजञाननकनाम mdash

ndash हहॴ 30-Jan-2017

Options

1) Neem

नीम

2) Mango

आम

3) Babul

बबि

4)Tulsi

तिसी Correct Answer Tulsi

Q177 Which gland secretes bile a

digestive fluid

कह९नसीगरिीपपतत एकपाचनतरिपरदािय सरापवतकरतीहहॴ 30-Jan-2017

Options

1) Pancreas

अगनयािय

2) Liver

यकत

3) Thyroid

िायराइि

4) Testes

टहॳनसटस

Correct Answer liver

Q178 In which of the following the

dominant phase is Gametophyte

नननननलिखितमसहॳककसकहॳ परमिचरणयगमकह८दपवधद (Gametophyte)हहॴ 30-Jan-2017

Options

1) Bryophyta

िायह८फाइटा 2) Pteridophyta

टहॳररिह८फाइटा 3) Gymnosperms

नजननह८सपमय 4) Angiosperms

एननजयह८सपमय Correct Answer Bryophyta

Q179 Anaerobic respiration refers to

which of the following

नननननलिखितमसहॳककसहॳअवायवीयशवसनकहाजाताहहॴ

30-Jan-2017

Options

1) Respiration without Oxygen

ऑकसीजनकहॳ बबनाशवसन

2) Respiration with Oxygen

ऑकसीजनकहॳ सािशवसन

3) Respiration without CO2

काबयनिायऑकसाइिकहॳ बबनाशवसन

4) Respiration with CO2

काबयनिायऑकसाइिकहॳ सािशविन

Correct Answer Respiration without

Oxygen

Q180 Which type of pathogen causes

the waterborne disease Cholera

ककसपरकारकारह८गजनकजिजननतरह८गहहॴजाकाकारणबनताहहॴ

30-Jan-2017

Options

1) Algal

िहॴवालियहॳ

F A C E B O O K

P A G E h t t p w w w f a c e b o o k c o m s s c m e n t o r s o f f i c i a l P a g e | 42

FOR MORE UPDATES AND MORE MATERIAL DO LIKE OUR FACEBOOK PAGE httpwwwfacebookcomsscmentorsofficial

2) Bacterial

बहॴकटीररयि

3) Protozoan

परह८टह८जआ

4) Viral

वायरि

Correct Answer Bacterial

Q181 To which class does

Oxyreductases transferases hydrolases

belong

ओकसीररिकटहॳसटरासफरहॳजहॳस

हाइडरह८िहॳसहॳसककसवगयमआतहॳहहॴ 30-Jan-2017

Options

1) Hormones

हारमोस

2) Enzymes

एजाइनस

3) Proteins

परह८टीनस

4) Vitamins

पवटालमनस

Correct Answer Enzymes

Q182 Which of the following is not true

about Gymnosperms

ननननमसहॳकह९नसीबातअनावतबीजीकहॳ बारहॳमसचनहीहहॴ 30-Jan-2017

Options

1) Dominant phase is saprophytes

परमिचरणसहॳपरह८फाइटसहह८ताहहॴ 2) Vascular bundles are absent

सवहनीबििअनपनसितहह८ताहहॴ 3) spores are heterospores

बीजाणहहॳटहॳरह८सपह८रसहह८तहॳहहॴ 4) Flowers are absent

फिअनपनसितहह८तहॳहहॴ

Correct Answer Vascular bundles are

absent

Q183 The name of first mammal clone sheep is

भहॳड़कीपरिमसतनपायीपरनत प (किह८न)

कानामहहॴ 30-Jan-2017

Options

1) Noori

नरी 2) Dolly

िॉिी 3) Louise

िसी 4)Durga

दगाय Correct Answer Dolly

Q184 Which type of pathogen causes

the water-borne disease Typhoid fever

ककसपरकारकारह८गजनकजिजननतरह८गटाइफाइिबिारकाकारणबनताहहॴ 30-Jan-2017

Options

1) Algal

िहॴवािीय

2) Parasitic

परजीवी 3) Protozoan

परह८टह८जनअन

4)Bacterial

बहॴकटीररयि

Correct Answer Bacterial

Q185 In which part of the cell are

proteins made

कह८लिकाकहॳ ककसटहससहॳमपरह८टीनबनायाजाताहहॴ

31-Jan-2017

Options

1) Reticulum

रहॳटटकिम

F A C E B O O K

P A G E h t t p w w w f a c e b o o k c o m s s c m e n t o r s o f f i c i a l P a g e | 43

FOR MORE UPDATES AND MORE MATERIAL DO LIKE OUR FACEBOOK PAGE httpwwwfacebookcomsscmentorsofficial

2) Golgi apparatus

गह८मजीएपहॳरहॳटस

3) Ribosomes

ररबह८सह८नस

4) Lysosome

िायसह८सह८नस

Correct Answer ribosomes

Proteins are produced by stringing

amino acids together in the order

specified by messenger RNA strands

that were transcribed from DNA in the

cell nucleus The process of synthesizing

a protein is called translation and it

occurs on ribosomes in the cytoplasm of

a cell

Q186 Polio is a disease caused by which

of the following

नननननलिखितमसहॳपह८लियह८कीबबमारह८हह८नहॳकाकारणकयाहहॴ

31-Jan-2017

Options

1) Bacteria

बहॴकटीररयि

2) Mosquito

मचछर

3) Virus

वायरस

4) Cockroach

नतिच हॳ Correct Answer Virus

Polio or poliomyelitis is a crippling and

potentially deadly infectious disease It

is caused by the poliovirus

Q187 ndash Hay fever is a sign of which of

the following

हहॳकफवरनननननलिखितमसहॳककसकाएकसकहॳ तहहॴ

31-Jan-2017

Options

1) Old Age

वदावसिा 2) Malnutrition

कपह८सण

3) Allergy

एिनजय 4) Over Work

अतयचधककाययकरना Correct Answer Allergy

Q188 How many chromosomes does a

human cell contain

एकमानवकह८लिकामककतनहॳगणसतरहह८तहॳहहॴ

29-Jan-2017

Options

1) 6

2) 26

3) 46

4) 66

Correct Answer 46

In humans each cell normally contains

23 pairs of chromosomes for a total of

46 Twenty-two of these pairs called

autosomes look the same in both males

and females The 23rd pair the sex

chromosomes differ between males and

females

Q189 Which of the following is not true

about Bryophyta

ननननमसहॳकह९नसीबातिायह८फाइटकहॳ बारहॳमसचनहीहहॴ 31-Jan-2017

Options

1) Dominant phase is gametophytes

परमिचरणगहॳलमतह८फाइटसहह८ताहहॴ 2) Main plant body is haploid

पह९धहॳकामखयिरीरअगखणतहह८ताहहॴ 3) Spores are homospores

बीजाणहह८मह८सफह८रसहह८तहॳहहॴ 4) Flowers are present

फिमह८जदहह८तहॳहहॴ Correct Answer Flowers are present

F A C E B O O K

P A G E h t t p w w w f a c e b o o k c o m s s c m e n t o r s o f f i c i a l P a g e | 44

FOR MORE UPDATES AND MORE MATERIAL DO LIKE OUR FACEBOOK PAGE httpwwwfacebookcomsscmentorsofficial

Q190 Which aquatic animal has

trailing tentacles

ककसजिीयजानवरकहॳ पीछहॳचिनहॳवािहॳटहॳटकिसहह८तहॳहहॴ

31-Jan-2017

Options

1) Sea horse

समदरीघह८िा 2) Corals

मगा 3) Jelly fish

जहॳिीमछिी 4) Star fish

तारामछिी Correct Answer Jelly fish

Jellyfish with its umbrella-shaped bell

and trailing tentacles

Q191 Which type of pathogen causes

the water-borne disease Poliomyelitis

(Polio)

ककसपरकारकारह८गजनकजिजननतरह८गपह८लियह८मायहॳटटस (पह८लियह८) काकारणहहॴ 31-Jan-

2017

Options

1) Parasitic

परजीवी 2) Algal

िहॴवालिय

3) Viral

वायरि

4) Bacterial

बहॴकटीररयि

Correct Answer Viral

Q192 The outer white part of the eye

that protects the inner structures is

आािकाबाहरीसफहॳ दटहससाजह८आतररकसरचनाओकीरकषाकरताहहॴ वह mdashmdashmdash हहॴ 31-Jan-

2017

Options

1) Iris

आयररस

2) Sclera

सकिहॳरा 3) Retina

रहॳटटना 4) Cornea

कह८ननयया Correct Answer Sclera

Q193 Proteins are made up of

परह८टीनकाननमायण mdashndash सहॳहह८ताहहॴ 31-Jan-2017

Options

1) Amino acids

एलमनह८अनि

2) Fatty acids

वसायकतअनि

3) Glucose

गिकह८ज

4)Nucleotides

नयनकियह८टाईिस

Correct Answer Amino acids

Q194 Moringa Oleifera is the scientific

name of

मह८ररगओलिफहॳ रा mdashmdashndash कावहॴजञाननकनामहहॴ 31-Jan-2017

Options

1) Banyan

बरगद

2) Gulmohar

गिमह८हर

3) Amla

आमिा

F A C E B O O K

P A G E h t t p w w w f a c e b o o k c o m s s c m e n t o r s o f f i c i a l P a g e | 45

FOR MORE UPDATES AND MORE MATERIAL DO LIKE OUR FACEBOOK PAGE httpwwwfacebookcomsscmentorsofficial

4) Drumstick

डरमनसटक

Correct Answer Drumstick

Q195 Kidney stones are composed of

गदकीपिरी mdashndash सहॳबनीहह८तीहहॴ 1-Feb-2017

Options

1) Calcium Oxalate

कहॴ नमसयमओकजहॳिहॳट

2) Sodium Chloride

सह८डियमकिह८राइि

3) Magnesium Nitrate

महॳनगनलियमनाइतटरहॳट

4) Calcium Bicarbonate

कहॴ नमियमबायकबोनहॳट

Correct Answer Calcium Oxalate

Q196 ndash Which of the following is not

true about Angiosperms

ननननमसहॳकह९नसीबातआवतबीजीकहॳ बारहॳमसचनहीहहॴ 1-Feb-2017

Options

1) Dominant phase is gametophytes

परमिचरणगहॳलमतह८फाइटहह८ताहहॴ 2) Vascular bundles are present

सवहनीबििमह९जदहह८ताहहॴ 3) Spores are heterospores

बीजाणहहॳटहॳरह८सपह८रसहह८तहॳहहॴ 4) Seeds are covered

बीजढकहॳ हह८तहॳहहॴ Correct Answer Dominant phase is

gametophytes

Q197 All of the following are excretory

(waste) products of animals except

नननननलिखितमसहॳककसएककह८छह८ड़करअनयसभीपराखणयोदवाराउतसनजयतपदाियहहॴ 1-Feb-

2017

Options

1) Uric Acid

यररकएलसि

2) Ammonia

अमह८ननया 3) Carbohydrates

काबोहाइडरहॳट

4) Urea

यररया Correct Answer Carbohydrates

In animals the main excretory products

are carbon dioxide ammonia (in

ammoniotelics) urea (in ureotelics) uric

acid (in uricotelics) guanine (in

Arachnida) and creatine

Q198 RNA is a polymeric molecule

What does RNA stand for

आरएनइएएकबहिकआणहहॴ इसकाकापवय पकयाहहॴ 1-Feb-2017

Options

1) Rado Nuclear Acid

रािह८नयनकियरएलसि

2) Ribo Nucleic Acid

राइबह८नयनकिकएलसि

3) Rhino Nuclear Acid

हाइनह८नयनकियरएलसि

4) Resto Nucleus Acid

रहॳसटह८नयकिीयसएलसि

Correct Answer Ribo Nucleic Acid

Q199 Which organ does detoxification

and produces chemicals needed for

digestion

कह९नसाअगपवषहरणकरताहहॴऔरपाचनकहॳ लिएआवशयकरसायनोकह८पहॴदाकरताहहॴ 1-Feb-

2017

Options

1) Salivary glands

िारगरचिया 2) Pancreas

अगनयािय

F A C E B O O K

P A G E h t t p w w w f a c e b o o k c o m s s c m e n t o r s o f f i c i a l P a g e | 46

FOR MORE UPDATES AND MORE MATERIAL DO LIKE OUR FACEBOOK PAGE httpwwwfacebookcomsscmentorsofficial

3) Thyroid gland

िायराइिगरिी 4) Liver

यकत

Correct Answer Liver

Q200 Psidium guajava is the scientific

name of

लसडियमगआजावा mdashmdash कावहॴजञाननकनामहहॴ 1-

Feb-2017

Options

1) Guava

अम द

2) Mango

आम

3) Bamboo

बास

4) Jack fruit

कटहि

Correct Answer Guava

Q201 Which drug is used as a Blood

Thinner

चधरकह८पतिाकरनहॳकहॳ पमककसदवाकापरयह८गककयाजाताहहॴ

1-Feb-2017

Options

1) Warfarin

वाफर न

2) Tramadol

टरहॳमािह८ि

3) Azithromycin

एनजरह८मायलसन

4) Hydralazine

हाइडरह८िहॳनजन

Correct Answer Warfarin

Q202 Which of the following disease is

caused due to the deficiency of protein

परह८टीनकीकमीकहॳ कारणनननननलिखितमसहॳकह९नसारह८गहह८ताहहॴ 1-Feb-2017

Options

1) Arthritis

गटठया 2) Kwashiorkor

कािीओकय र

3) Goitre

गाइटर

4) Night Blindness

रतह९चध

Correct Answer Kwashiorkor

Q203 A is species of plant that has

adapted to survive in an environment

with little liquid water

mdashmdashndashपह९धहॳकीएकऐसहॳऐसहॳपरजानतहहॴ नजसनहॳकमपानीवािहॳवातावरणमजीपवतरहनहॳकहॳलिएअनकिनहहॴ 1-Feb-2017

Options

1) Xerophyte

म दपवद

2) Hydrophyte

जिीयपादप

3) Mesophyte

समह८दपवद

4) Thallophyte

िहॴिह८फाइटा Correct Answer xerophyte

xerophyte is a species of plant that has

adapted to survive in an environment

with little liquid water such as a desert

or an ice- or snow-covered region in the

Alps or the Arctic

Mesophytes are terrestrial plants which

are adapted to neither a particularly

dry nor particularly wet environment

An example of a mesophytic habitat

would be a rural temperate meadow

F A C E B O O K

P A G E h t t p w w w f a c e b o o k c o m s s c m e n t o r s o f f i c i a l P a g e | 47

FOR MORE UPDATES AND MORE MATERIAL DO LIKE OUR FACEBOOK PAGE httpwwwfacebookcomsscmentorsofficial

which might contain goldenrod clover

oxeye daisy and Rosa multiflora

thallophyte any of a group of plants or

plantlike organisms (such as algae and

fungi) that lack differentiated stems

leaves and roots and that were formerly

classified as a primary division

(Thallophyta) of the plant kingdom

Q204 How many types of teeth are

there in humans

मनषयोमककतनहॳपरकारकहॳ दातहह८तहॳहहॴ

1-Feb-2017

Options

1) 4

2) 5

3) 2

4) 3

Correct Answer 4

teeth -Humans have four types of

teethincisors canines premolars and

molars each with a specific function

The incisors cut the food the canines

tear the food and the molars and

premolars crush the food

Q205 Carica papaya is the scientific name of

कहॴ ररकापपाया mdashmdashndash कावहॴजञाननकनामहहॴ 2-

Feb-2017

Options

1) Peepal

पीपि

2) Papaya

पपीता 3) Tamarind

इमिी 4) Drumstick

ढह८िकाछड़ी Correct Answer Papaya

Q206 Muscles get tired when there is

shortfall of

जब mdashndash कीकमीहह८तीहहॴतबपहॳिीयिकजातीहहॴ 2-Feb-2017

Options

1) Lactic acid

िहॴनकटकएलसि

2) Na+ ions

Na+ आयन

3) ATP

एटीपी 4) Sulphates

समफहॳ टस

Correct Answer ATP

ATP is the energy source muscle fibers

use to make muscles contract

muscle tissuersquos main source of energy

called adenosine triphosphate or ATP

As your muscles use up this energy

source they become tired and fatigued

Oxygen is the key ingredient that helps

create new ATP to replenish the burned

up ATP in your muscles

Q207 Artocarpus integra is the

scientific name of आटह८कापयसइटीगरा mdashmdashmdash कावहॴजञाननकनामहहॴ 2-Feb-2017

Options

1) Guava

अम द

2) Pineapple

अनानास

3) Silver Oak

लसमवरओक

4) Jack fruit

कटहि

Correct Answer Jack fruit

Q208 Which organ stores fat soluble

vitamins

कह९नसाअगवसामघिनिीिपवटालमनह८काभिाराकरताहहॴ

2-Feb-2017

F A C E B O O K

P A G E h t t p w w w f a c e b o o k c o m s s c m e n t o r s o f f i c i a l P a g e | 48

FOR MORE UPDATES AND MORE MATERIAL DO LIKE OUR FACEBOOK PAGE httpwwwfacebookcomsscmentorsofficial

Options

1) Blood

रकत

2) Skin

तवचा 3) Liver

यकत

4) Pancreas

अगनयािय

Correct Answer Liver

Q209 Which disease is caused due to

deficiency of Iodine

आयह८िीनकहॳ कारणकह९नसारह८गहह८ताहहॴ 2-Feb-2017

Options

1) Rickets

ररकहॳ टस

2) Scurvy

सकवी 3) Goitre

गणमािा 4) Growth retardation

पवकासका कना Correct Answer Goitre

rickets A softening and weakening of

bones in children usually due to

inadequate vitamin D

Q210 Grevillea Robusta is the scientific name of

गरहॳपवलियारह८बसटा mdashmdashmdash- कापवजञाननकनामहहॴ 2-Feb-2017

Options

1) Peepal

पीपि

2) Teak

सागह९न

3) Silver Oak

लसमवरओक

4) Jack fruit

कटहि

Correct Answer Silver Oak

Q211 When a Cuttlefish is described as a Molluscs it is at which level of

classification

जबएककटिकफिकह८एकमह८िसकाकहॳ पमवखणयतककयाजाताहहॴतबयहॳवगीकरणकहॳ ककससतरपहॳनसितहहॴ 2-Feb-2017

Options

1) Class

वगय 2) Order

िम

3) Family

पररवार

4) Phylum

सघ

Correct Answer Phylum

Q212 Bambusa dendrocalmus is the

scientific name of बानबसािहॳडराकामस mdashmdashmdash कावहॴजञाननकनामहहॴ 3-Feb-2017

Options

1) Banyan

बरगद

2) Papaya

पपीता 3) Bamboo

बास

4) Pomegranate

अनार

Correct Answer Bamboo

Q213 Acinonyx Jubatus is the scientific name of

एलसनह८ननकसजयबहॳटस mdashmdashmdash

कावहॴजञाननकनामहहॴ 3-Feb-2017

F A C E B O O K

P A G E h t t p w w w f a c e b o o k c o m s s c m e n t o r s o f f i c i a l P a g e | 49

FOR MORE UPDATES AND MORE MATERIAL DO LIKE OUR FACEBOOK PAGE httpwwwfacebookcomsscmentorsofficial

Options

1) Bear

भाि 2) Horse

घह८िा 3) Cheetah

चीता 4) Zebra

जहॳिा Correct Answer Cheetah

Q214 The pale yellow colour of urine is

due to the presence of which pigment

मतरकाफीकापीिारगरगदरयकहॳ उपनसिनतकहॳ कारणहह८ताहहॴ

3-Feb-2017

Options

1) Urochrome

यरह८िह८म

2) Urophyll

यरह८कफि

3) Chlorophyll

किह८रह८कफि

4) Chloroplast

किह८रह८पिासट

Correct Answer Urochrome

Q215 Which of the following constitute

to form a gene

नननननलिखितमसहॳकह९नसीचीज़एकजीनकागठनकरतीहहॴ

3-Feb-2017

Options

1) Polynucleotides

पह८िीनयनकियह८टाईडस

2) Hydrocarbons

हाइडरह८काबोस

3) Lipoproteins

िाईपह८परह८टीनस

4) Lipids

लिपपडस

Correct Answer Polynucleotides

Polynucleotide molecule is a biopolymer

composed of 13 or more nucleotide

monomers covalently bonded in a chain

DNA (deoxyribonucleic acid) and RNA

(ribonucleic acid) are examples of

polynucleotides with distinct biological

function

Q216 Vertebrates belongs to the

phylum

रीढ़कीहडिीवािहॳपराणी mdashmdashmdash

परजानतकहॳ अतगायतआतहॳहहॴ 3-Feb-2017

Options

1) Arthropoda

आरह८पह८ड़ा 2) Annelida

एननलििा 3) Cnidaria

ननिहॳररया 4) Chordata

कह८िटा Correct Answer Chordata

Q217 Punica granatum is the scientific name of

पननकगरहॳनहॳटस mdashmdashmdash कावहॴजञाननकनामहहॴ 3-Feb-2017

Options

1) Custard Apple

सीताफि

2) Gulmohar

गिमह८हर

3) Silver Oak

लसमवरओक

4) Pomegranate

अनार

Correct Answer Pomegranate

F A C E B O O K

P A G E h t t p w w w f a c e b o o k c o m s s c m e n t o r s o f f i c i a l P a g e | 50

FOR MORE UPDATES AND MORE MATERIAL DO LIKE OUR FACEBOOK PAGE httpwwwfacebookcomsscmentorsofficial

Q218 Between a tiger and an monkey

which of the following is different

एकबाघऔरबदरकहॳ बीचनननननलिखितमसहॳकह९नसीबातअिगहहॴ 3-Feb-2017

Options

1) Kingdom

राजय

2) Phylum

जानत

3) Order

िम

4) Class

वगय Correct Answer order

Q219 The artificial heart was invented by

कबतरमहदयका mdashmdashmdash

दवाराअपवषकारककयागयािा 3-Feb-2017

Options

1) Muhammad Yunus

महनमदयनस

2) Linus Yale Jr

िाइनसयहॳिजय

3) Gazi Yasargil

गाजीयासचगयि

4) Paul Winchell

पह९िपवमकि Correct Answer Paul Winchell

Q220 Tamarindus indica is the

scientific name of

टहॳमररनडसइडिका mdashmdash कावहॴजञाननकनामहहॴ 7-

Feb-2017

Options

1) Neem

नीम

2) Pineapple

अनानास

3) Tamarind

इमिी 4)Chiku

चीक

Correct Answer Tamarind

Q221 In eukaryotic cells synthesis of

RNA takes place in the

यकहॳ योटटककह८लिकाओमआरएनएकासशिहॳषण

mdashndash महह८ताहहॴ 7-Feb-2017

Options

1) Mitochondria

माईटह८कोडडरया 2) Centrioles

सटरीयह८मस

3) Ribosomes

ररबह८सह८नस

4) Nucleus

नयनकियस

Correct Answer nucleus

eukaryotic cell -Transcription is the

process of synthesizing ribonucleic acid

(RNA)Synthesis takes place within the

nucleus of eukaryotic cells or in the

cytoplasm of prokaryotes and converts

the genetic code from a gene in

deoxyribonucleic acid ( DNA ) to a

strand of RNA that then directs

proteinsynthesis

Q222 _________is caused by parasites

of the Plasmodium genus

पिाजमह८डियमजातीकहॳ परजीवी mdash- कहॳ कारणहहॴ 7-Feb-2017

Options

1) Dysentery

पहॳचचि

2) Malaria

मिहॳररया 3) Chickenpox

F A C E B O O K

P A G E h t t p w w w f a c e b o o k c o m s s c m e n t o r s o f f i c i a l P a g e | 51

FOR MORE UPDATES AND MORE MATERIAL DO LIKE OUR FACEBOOK PAGE httpwwwfacebookcomsscmentorsofficial

चहॳचक

4) Herpes

हहॳपपयस

Correct Answer Malaria

Q223 Carotene in fruits and vegetables

gives it which color

फिह८औरसनलजयोमनसितकहॳ रह८टीनउनहकह९नसारगपरदानकरताहहॴ 7-Feb-2017

Options

1) Green

हरा 2) Pink

गिाबी 3) Orange

नारगी 4) Blue

नीिा Correct Answer Orange

Q224 Equus Caballus is the scientific

name of

एकवसकहॴ बहॳिस mdashmdashndash कापवजञाननकनामहहॴ 7-Feb-2017

Options

1) Horse

घह८िा 2) Zebra

ज़हॳिा 3) Donkey

गधा 4) Buffalo

भस

Correct Answer Horse

Q225 Elapidae Naja is the scientific name of

एिीपीिीनाजा mdashmdash- कावहॴजञाननकनामहहॴ 8-Feb-2017

Options

1) Cobra

कह८बरा 2) Elephant

हािी 3) Eagle

ग ि

4) Owl

उमि Correct Answer Cobra

Q226 Which disease is caused due to

deficiency of Iron

िह८हकीकमीकहॳ कारणकह९नसारह८गहह८ताहहॴ 8-Feb-

2017

Options

1) Beriberi

बहॳरीबहॳरी 2) Tetany

टहॳटनी 3) Kwashiorkor

कवािीऔरकर

4) Anaemia

रकतामपता Correct Answer Anaemia

Beriberi is a disease caused by a vitamin

B-1 deficiency also known as thiamine

deficiency

Tetany can be the result of an

electrolyte imbalance Most often itrsquos a

dramatically low calcium level also

known as hypocalcemia Tetany can also

be caused by magnesium deficiency or

too little potassium Having too much

acid (acidosis) or too much alkali

(alkalosis) in the body can also result in

tetany

Kwashiorkor also known as

ldquoedematous malnutrition It is a form of

malnutrition caused by a lack of protein

in the diet

Anaemia means that you have fewer red

blood cells than normal or you have less

F A C E B O O K

P A G E h t t p w w w f a c e b o o k c o m s s c m e n t o r s o f f i c i a l P a g e | 52

FOR MORE UPDATES AND MORE MATERIAL DO LIKE OUR FACEBOOK PAGE httpwwwfacebookcomsscmentorsofficial

haemoglobin than normal in each red

blood cell

Q227 is a leaf where the leaflets are

arranged along the middle vein

mdashndashएकपततीहहॴजहापतरकह८कीरचनाक ररयालिराकहॳ आसपासहह८तीहहॴ 8-Feb-2017

Options

1) Pinnately compound leaf

पपनहॳटिीसयकतपतती 2) Palmately compound leaf

पामहॳटिीसयकतपतती 3) Compound leaf

सयकतपतती 4) Simple leaf

साधारणपतती Correct Answer Pinnately compound

leaf

Q228 Haustoria or sucking roots are

found in which of the following

हह८सटह८ररयायाचसनहॳवािीजड़हॳनननननलिखितमसहॳककसमपाईजातीहहॴ 8-Feb-2017

Options

1) Wheat

गहॳह

2) Mango

आम

3) Chestnut

चहॳसटनट

4) Cuscuta

कसकयटा Correct Answer Cuscuta

Haustorial roots -The roots of parasitic

plants which penetrate into the host

tissues to absorb nourishment are

called haustorial roots hellip Also known as suckingor parasitic roots

Q229 Equs Asinus is the scientific name

of

एकवसएलसनस mdashmdashndash कावहॴजञाननकनामहहॴ 8-

Feb-2017

Options

1) Donkey

गधा 2) Cow

गाय

3) Deer

टहरन

4) Kangaroo

कगा

Correct Answer Donkey

Q230 Ficus benghalensis is the scientific name of

फाईकसबहॳनगहॳिहॳलसस mdashndash कापवजञाननकनामहहॴ 8-Feb-2017

Options

1) Banyan

बरगद

2) Pineapple

अनानास

3) Babul

बबि

4) Tulsi

तिसी Correct Answer Banyan

Q231 Equus burchellii is the scientific name of

एकवसबचिी mdashmdash- कापवजञाननकनामहहॴ 8-Feb-2017

Options

1) Horse

घह८िा 2) Zebra

जहॳिा 3) Buffalo

F A C E B O O K

P A G E h t t p w w w f a c e b o o k c o m s s c m e n t o r s o f f i c i a l P a g e | 53

FOR MORE UPDATES AND MORE MATERIAL DO LIKE OUR FACEBOOK PAGE httpwwwfacebookcomsscmentorsofficial

भस

4) Ass

गधा Correct Answer Zebra

Page 26: COMPILATION OF ALL 72 SETS OF BIOLOGY SSC CHSL-2016 · OF BIOLOGY SSC CHSL-2016 PREPARED BY : SSC MENTORS BIOLOGY SPECIAL . F A C E B O O K P A G E : h t t p : / / w w w . f a c e

F A C E B O O K

P A G E h t t p w w w f a c e b o o k c o m s s c m e n t o r s o f f i c i a l P a g e | 25

FOR MORE UPDATES AND MORE MATERIAL DO LIKE OUR FACEBOOK PAGE httpwwwfacebookcomsscmentorsofficial

1) Malphigian Tubules

मनमफनजयनटयबमस

2) Nephridia

नहॳकफरडिया 3) Coxal Gland

कह८कसिगरचिया 4) Green Gland

गरीनगरचिया Correct Answer Malphigian Tubules

Q105 Evaporation of water takes place

in which part of plants

पानीकहॳ वाषपीकरणकीकियापह९धोकहॳ ककसभागसहॳहह८तीहहॴ 21-Jan-2017

Options

1) Stem

तना 2) Stomata

सटह८मटा 3) Branch

िािाए

4) Fruit

फि

Correct Answer Stomata

Evaporation accounts for the movement

of water to the air from sources such as

the soil canopy interception and

waterbodies Transpiration accounts for

the movement of water within a plant

and the subsequent loss of water as

vapour through stomata in its leaves

Q106 A is the fleshy spore-bearing

fruiting body of a fungus

mdashmdashndashकवककामासि

बीजाणधारणकरनहॳवािाफिनहॳवािाअगहहॴ 21-

Jan-2017

Options

1) aloe vera

एिह८वहॳरा 2) Coral

मगा 3) Cactus

कहॴ कटस

4) Mushroom

ककरमतता Correct Answer mushroom

Q107 Which of the following is a fungal

disease

नननननलिखितमसहॳकह९नसाफफदसहॳहह८नहॳवािाएकरह८ग हहॴ

21-Jan-2017

Options

1) Dermatitis

तवचािह८ध

2) Cholera

हहॴजा 3) Jaundice

पीलिया 4) Indigofera

इननिगह८फहॳ रा Correct Answer Dermatitis

Dermatitis also known as eczema is a

group of diseases that results in

inflammation of the skin These diseases

are characterized by itchiness red skin

and a rash In cases of short duration

there may be small blisters while in

long-term cases the skin may become

thickened

Q108 In which form is glucose stored in

our body

हमारहॳिरीरमगिकह८जकासचयककस पमककयाजाताहहॴ

21-Jan-2017

Options

1) Insulin

F A C E B O O K

P A G E h t t p w w w f a c e b o o k c o m s s c m e n t o r s o f f i c i a l P a g e | 26

FOR MORE UPDATES AND MORE MATERIAL DO LIKE OUR FACEBOOK PAGE httpwwwfacebookcomsscmentorsofficial

इसलिन

2) Glucose

गिकह८ज

3) Glycogen

गिायकह८जहॳन

4) Fat

वसा Correct Answer Glycogen

Excess glucose is stored in the liver as

the large compound called glycogen

Glycogen is a polysaccharide of glucose

but its structure allows it to pack

compactly so more of it can be stored in

cells for later use

Q109 Where do plants synthesize

protein from

पह९धहॳपरह८टीनसशिहॳषणकहासहॳकरतहॳहहॴ

Options

1) Fatty Acids

वसाऐलसि

2) Sugar

िकर

3) Amino Acids

एलमनह८ऐलसि

4) Starch

सटाचय Correct Answer Amino Acids

Q110 Which part of the brain is

responsible for triggering actions like

thinking intelligence memory and

ability to learn

मनसतषककाकह९नसाटहससासह८चनहॳ बनधदमानी याददाशतऔरसीिनहॳकीकषमताजहॴसीकियाओकह८परहॳररतकरताहहॴ 21-Jan-2017

Options

1) Diencephalon

िायएनसहॳफहॳ िह८न

2) Hypothalamus

हयपह८िहॳिहॳमस

3) Cerebrum

सहॳरहॳिम

4) Control

कटरह८ि

Correct Answer Cerebrum

Q111 Which of the following is also

known as the Biochemical Laboratory

of the Human Body

नननननलिखितमसहॳककसहॳमानविरीरकीजहॴवरसायनपरयह८गिािाभीकहाजाताहहॴ 21-Jan-2017

Options

1) Small Intestine

छह८टीआत

2)Brain

मनसतषक

3) Pancreas

अगनयािय

4) Liver

नजगर

Correct Answer Liver

The liver makes bile that will help

emulsify and digest the fats we eat

The liver takes toxic substances and

convert them using enzymes the liver

cells makes into a non toxic form so the

body can dispose of them

The liver also converts fats protein and

carbohydrates into glucose which is the

energy source for our cells to use

The liver takes amino acids and makes

proteins by combining them

Q112 The yellow colour of human urine

is due to

मानवमतरकापीिारग mdashndash कीवजहसहॳहह८ताहहॴ 22-

Jan-2017

Options

1) Bile Salts

F A C E B O O K

P A G E h t t p w w w f a c e b o o k c o m s s c m e n t o r s o f f i c i a l P a g e | 27

FOR MORE UPDATES AND MORE MATERIAL DO LIKE OUR FACEBOOK PAGE httpwwwfacebookcomsscmentorsofficial

पपततनमक

2) Cholesterol

कह८िहॳसटरह८ि

3) Lymph

लिनफ

4) Urochrome

यरह८िह८म

Correct Answer Urochrome

Urobilin or urochrome is the chemical

primarily responsible for the yellow

color of urine

Q113 The wilting of plants takes place

due to

पह९धह८कालिचििहह८नाकी mdashmdash- कीवजहसहॳहह८ताहहॴ 22-Jan-2017

Options

1)Photosynthesis

परकािसशिहॳषण

2) Transpiration

वाषपह८तसजयन

3) Absorption

अविह८षण

4) Respiration

शरवसन

Correct Answer Transpiration

Wilting is the loss of rigidity of non-

woody parts of plants This occurs when

the turgor pressure in non-lignified

plant cells falls towards zero as a result

of diminished water in the cells

Q114 Bovidae Ovis is the scientific name of

बह८पविीओपवस mdashndash कावहॴजञाननकनामहहॴ 22-Jan-2017

Options

1) Goat

बकरी 2) Cow

गाय

3) Buffalo

भहॳस

4) Sheep

भहॳड़

Correct Answer Sheep

Q115 Plants get their energy to produce

food from which of the following

पह८धहॳभह८जनकाननमायणकरनहॳकहॳ लिएनननननलिखितमसहॳककससहॳउजायपरापतकरतहॳहहॴ

22-Jan-2017

Options

1) Photosynthesis

परकािसशिहॳषण

2)Bacteria

बहॴकटीररया 3)Fungi

कवक

4)Sun

सयय Correct Answer Sun

Q116 Which of the following is secreted

by the liver

नननननलिखितमसहॳककसकासरावनजगरसहॳहह८ताहहॴ

22-Jan-2017

Options

1) Glucose

गिकह८ज

2) Iodine

आयह८िीन

3) Cortisol

काटटरयसह८ि

4) Bile

पपतत

Correct Answer Bile

The liver makes bile that will help

emulsify and

digest the fats we eat

F A C E B O O K

P A G E h t t p w w w f a c e b o o k c o m s s c m e n t o r s o f f i c i a l P a g e | 28

FOR MORE UPDATES AND MORE MATERIAL DO LIKE OUR FACEBOOK PAGE httpwwwfacebookcomsscmentorsofficial

Q117 Ferns belong to which division of

plants

फनसयपह९धह८कहॳ ककसभागमआतहॳहहॴ

22-Jan-2017

Options

1) Gymnosperms

नजननह८सपनसय 2) Angiosperms

एनजयह८सपनसय 3) Thallophyta

िहॴिह८फाईटा 4)Pteridophyta

टहॳररिह८फाईटा Correct Answer Pteridophyta

Q118 Who invented Antibiotics

एटीबायह८टटककाअपवषकारककसनहॳककयािा

22-Jan-2017

Options

1) Joseph Lister

जह८सहॳफलिसटर

2) William Harvey

पवलियमहाव

3) Robert Knock

रॉबटयनॉक

4)Alexander Fleming

अिहॳकज़िरफिहॳलमग

Correct Answer Alexander Fleming

Q119 Milbecycin is used in the

eradication of

लममबहॳसायलसनका mdashndash

मउनमिनमपरयह८गककयाजाताहहॴ 22-Jan-2017

Options

1) Agricultural Fungus

कपषकवक

2) Agricultural Pests

कपषकीटक

3) Agricultural Herbs

कपषिाक

4)Agricultural Weeds

कपषननराना Correct Answer Agricultural Pests

Milbemycin oxime is a veterinary drug

from the group of milbemycins used as

a broad spectrum antiparasitic It is

active against worms and mites(insects

Q120 Intestinal bacteria synthesizes

which of the following in the human

body

मानविरीरमआतोकहॳ बहॴकटीररयानननननलिखितमसहॳककसकासशिहॳषणकरतहॳहहॴ 22-Jan-2017

Options

1) Vitamin K

पवटालमन K

2) Proteins

परह८टीन

3) Fats

वसा 4) Vitamin D

पवटालमन D

Correct Answer Vitamin K

Q121 is the study of the physical form

and external structure of plants

mdashmdash-

मपह९धह८काभहॴनतक पऔरबाहरीसरचनाकाआदयाककयाजाताहहॴ 22-Jan-2017

Options

1) Physiology

कफनजयह८िह८जी 2) Anatomy

िरीररचनापवजञान

3) Phytomorphology

फाईटह८मह८फह८िह८जी 4)Cytology

कह८लिकापवजञान

Correct Answer Phytomorphology

F A C E B O O K

P A G E h t t p w w w f a c e b o o k c o m s s c m e n t o r s o f f i c i a l P a g e | 29

FOR MORE UPDATES AND MORE MATERIAL DO LIKE OUR FACEBOOK PAGE httpwwwfacebookcomsscmentorsofficial

Q122 Which of the following is a

structural and functional unit of

kidneys

नननननलिखितमसहॳकह९नसीगदोकीसरचनातमकऔरकाययकरीईकाईहहॴ

22-Jan-2017

Options

1) Renette Cells

रहॳनहॳटकह८लिकाए

2) Flame Cells

फिहॳमकह८लिकाए

3) Nephrites

नहॳफ़राइटस

4)Nephrons

नहॳफरोस

Correct Answer Nephrons

Nephron functional unit of the kidney

the structure that actually produces

urine in the process of removing waste

and excess substances from the blood

There are about 1000000 nephrons in

each human kidney

Q123 Which of the following is the

largest part of the human brain

नननननलिखितमसहॳकह९नसामानवमनसतषककासबसहॳबड़ाटहससाहहॴ

23-Jan-2017

Options

1) Ribs

पसलियाा 2) Cerebrum

सहॳरहॳिम

3) Pons

पोस

4)Thalamus

िहॴिहॳमस

Correct Answer Cerebrum

The cerebrum is the largest part of the

human brain making up about two-

thirds of the brainrsquos mass It has two

hemispheres each of which has four

lobes frontal parietal temporal and

occipital

Q124 The auxiliary buds

सहायककालियाmdashndash 23-Jan-2017

Options

1) grow endogenously from the pericycle

पहॳरीसाईककिसहॳअनतजातयपवकलसतहह८ताहहॴ 2) arise endogenously from the main

growing point

मिवपदसहॳअनतजातयउठताहहॴ 3) is an embryonic shoot located in the

axil of a leaf

एकभरणिटहहॴजह८एकपततीकहॳ अकषपरनसतिहह८ताहहॴ 4)arise exogenously from the epidermis

एपपिलमयससहॳबटहजातयतरीकहॳ सहॳउठताहहॴ Correct Answer is an embryonic shoot

located in the axil of a leaf

Q125 Which of the following is a viral

disease

इनमहॳसहॳकह९सीएकवायरिबीमारीहहॴ

23-Jan-2017

Options

1) Polio

पह८लियह८ 2) Tetanus

धनसतनभ

3) Leprosy

कषठरह८ग

4) Plague

पिहॳग

Correct Answer Polio

A viral disease (or viral infection)

occurs when an organismrsquos body is

invaded by pathogenic viruses and

infectious virus particles (virions) attach

to and enter susceptible cells

F A C E B O O K

P A G E h t t p w w w f a c e b o o k c o m s s c m e n t o r s o f f i c i a l P a g e | 30

FOR MORE UPDATES AND MORE MATERIAL DO LIKE OUR FACEBOOK PAGE httpwwwfacebookcomsscmentorsofficial

Poliomyelitis often called polio or

infantile paralysis is an infectious

disease caused by the poliovirus

Tetanusmdash A serious bacterial infection

that causes painful muscle spasms and

can lead to death

Leprosy also known as Hansenrsquos

disease (HD) is a long-term infection by

the bacterium Mycobacterium leprae or

Mycobacterium lepromatosis

Plague is an infectious disease caused by

the bacterium Yersinia pestis

Symptoms include fever weakness and

headache

Q126 Which organisms can help to

carry out Vermicomposting

कह९नसाजीववमीकनपह८नसटगममददकरताहहॴ

23-Jan-2017

Options

1) Nitrifying Bacteria

नाईटरीफाईगबहॴकटीररया 2) Earthworms

कहॴ चऐ

3) Algae

िहॴवि

4) Fungus

कवक

Correct Answer Earthworms

Q127 Contraction of heart is also

known as

हदयकहॳ सकचनकह८ mdash- भीकहाजाताहहॴ 23-Jan-

2017

Options

1) Systole

लससटह८ि

2) Aristotle

अरसत

3) Diastole

िायसटह८ि

4) Lub

मयब

Correct Answer Systole

Diastole is the part of the cardiac cycle

when the heart refills with blood

following systole (contraction)

Ventricular diastole is the period during

which the ventricles are filling and

relaxing while atrial diastole is the

period during which the atria are

relaxing

Q128 Azadirachta indica is the

botanical name of which of the

following

अजाटदराचताइडिकानननननलिखितमसहॳककसकावानसपनतनामहहॴ

23-Jan-2017

Options

1) Rose plant

गिाबकापह९धा 2) Apple tree

सहॳबकापहॳड़

3) Neem

नीम

4)Mango

आम

Correct Answer Neem

Q129 Which of the following is the

main end product of carbohydrate

digestion

नननननलिखितमसहॳकह९नसाकाबोहाइडरहॳटकहॳ पाचनकापरमिअतउतपादकहह८ताहहॴ 23-Jan-2017

Options

1) Fats

वसा 2) Lipids

लिपपडस

3) Glucose

गिकह८ज

4) Cellulose

F A C E B O O K

P A G E h t t p w w w f a c e b o o k c o m s s c m e n t o r s o f f i c i a l P a g e | 31

FOR MORE UPDATES AND MORE MATERIAL DO LIKE OUR FACEBOOK PAGE httpwwwfacebookcomsscmentorsofficial

सहॳमयिह८ज

Correct Answer Glucose

Intestinal absorption of end products

from digestion of carbohydrates and

proteins in the pig hellip During absorption some sugars (fructose or

galactose) released from the

corresponding sucrose and lactose

respectively during digestion were

partly metabolized into glucose by the

enterocyte

Q130 Which of the following glands is a

source of the enzyme Ptyalin

नननननलिखितगरचियोमसहॳएजाइमटयालिनकासरह८तहहॴ 23-Jan-2017

Options

1) Pancreas

अगरािय

2) Thyroid Gland

िाइराइिगरिी 3) Pituitary Gland

पीयषगरिी 4) Salivary Glands

िारगरचियाा Correct Answer Salivary Glands

Q131 Which of the following is not true

about Pteridophyta

ननननमसहॳकह९नसीबातटहॳररिह८फाईटकहॳ बारहॳमसचनहीहहॴ 23-Jan-2017

Options

1) Dominant phase is saprophytes

परमिचरणसहॳपरह८फाईइटसहह८ताहहॴ 2) Main plant body is diploid

पह९दह८कामखयिरीरदपवगखणतहह८ताहहॴ 3) Seeds are present

बीजमह९जदहह८तहॳहहॴ 4)Flowers are absent

फिअनपनसतिहह८तहॳहहॴ

Correct Answer Seeds are present

Q132 The largest dolphin species is the

orca also called as

िॉिकफनकीसबसहॳबड़ीपरजानतकाकानामआकायहहॴनजसहॳ mdash- भीकहतहॳहहॴ 23-Jan-2017

Options

1) Bottle Nose

बाटिनह८ज

2) Baiji

बहॳजी 3) Killer whale

ककिरहहॳि

4)Tucuxi

टकवसी Correct Answer Killer whale

Q133 The fat digesting enzyme Lipase

is secreted by which of the following

वसाकापाचनकरनहॳवािाएजाइमिाइपहॳजनननननलिखितमसहॳककसकहॳ दवारासतरापवतहह८ताहहॴ

24-Jan-2017

Options

1) Kidneys

गद

2) Pancreas

अगनयािय

3) Large Intestine

बड़ीआत

4)Liver

नजगर

Correct Answer Pancreas

Lipase is an enzyme that splits fats so

the intestines can absorb them Lipase

hydrolyzes fats like triglycerides into

their component fatty acid and glycerol

molecules It is found in the blood

gastric juices pancreatic secretions

intestinal juices and adipose tissues

F A C E B O O K

P A G E h t t p w w w f a c e b o o k c o m s s c m e n t o r s o f f i c i a l P a g e | 32

FOR MORE UPDATES AND MORE MATERIAL DO LIKE OUR FACEBOOK PAGE httpwwwfacebookcomsscmentorsofficial

Q134 The arrangement of leaves on an

axis or stem is called

एकअकषयातनहॳपरपनततयोकीयवसिाकह८कयाकहाजाताहहॴ SSC CHSL Science (biology) 2016

Question Paper

24-Jan-2017

Options

1) Phyllotaxy

फाइिह८टहॴकसी 2) Vernation

वनिन

3) Venation

वहॳनहॳिन

4)Phytotaxy

फाइटह८टहॴकसी Correct Answer Phyllotaxy

In botany phyllotaxis or phyllotaxy is

the arrangement of leaves on a plant

stem (from Ancient Greek phyacutellon

ldquoleafrdquo and taacutexis ldquoarrangementrdquo)

Phyllotactic spirals form a distinctive

class of patterns in nature

Q135 The study of Cells is also known

as

कह८लिकाओकहॳ अधययनकह८ mdashmdashndash

भीकहाजाताहहॴ 24-Jan-2017

Options

1) Cytology

सायटह८िह८जी 2) Physiology

कफनजयह८िह८जी 3) Nucleology

नयककमयह८िह८जी 4)Cellology

सहॳिह८िह८जी Correct Answer Cytology

Q136 Which of the following scientists

is also known as the Father of Biology

नननननलिखितमसहॳककसवहॴजञाननककह८ ldquoजीवपवजञानकहॳ जनकrdquoकहॳ नामसहॳभीजानाजाताहहॴ 24-Jan-2017

Options

1) Herbert Spencer

हबयटयसपसर

2) Aristotle

अरसत 3) Lamarck

िहॳमाकय 4)Darwin

िापवयन

Correct Answer Aristotle

Q137 Which cells give rise to various

organs of the plant and keep the plant

growing

कह९नसीकह८लिकाएपह९धह८कहॳ लभननअगह८कह८जनमदहॳतीहहॴऔरपह९धह८कह८बढ़नहॳममददकरतीहहॴ

24-Jan-2017

Options

1) Permanent

सिायी 2) Dermal

तवचीय

3) Meristematic

मररसटहॳमटटक

4)Mature

परह८ढ़

Correct Answer Meristematic

A meristem is the tissue in most plants

containing undifferentiated cells

(meristematic cells) found in zones of

the plant where growth can take place

Q138 Rodentia Muridae is the scientific

name of

F A C E B O O K

P A G E h t t p w w w f a c e b o o k c o m s s c m e n t o r s o f f i c i a l P a g e | 33

FOR MORE UPDATES AND MORE MATERIAL DO LIKE OUR FACEBOOK PAGE httpwwwfacebookcomsscmentorsofficial

रह८िहॳलियानयररिी mdashmdash- कावहॴजञाननकनामहहॴ 24-

Jan-2017

Options

1) Mouse

चहा 2) Squirrel

चगिहरी 3) Monkey

बदर

4) Lizard

नछपकिी Correct Answer Mouse

Q139 Name the scientist who proposed

the cell theory

कह८लिकालसदातकापरसतावदहॳनहॳवािहॳवहॴजञाननककानामबताइए 24-Jan-2017

Options

1) Schleiden and Schwann

िीमिनऔरशरववान

2) Lamarck

िहॳमाकय 3) Treviranus

टरहॳवायरहॳनस

4)Whittaker and Stanley

हीटकरऔरसटहॳनिहॳ Correct Answer Schleiden and

Schwann

Q140 The flower with the worldrsquos

largest bloom is

दननयाकासबसहॳबड़ाफिखििनहॳवािा mdashmdashndash हहॴ 24-Jan-2017

Options

1) Pando

पािह८ 2) Posidonia

पह८सीिह८ननया 3) Rafflesia arnoldii

ररफिहॳलियाअनोमिी 4)Helianthus annuus

हहॳलिएनिसएनयअस

Correct Answer Rafflesia arnoldii

Rafflesia arnoldii is a species of

flowering plant in the parasitic genus

Rafflesia It is noted for producing the

largest individual flower on earth It has

a very strong and horrible odour of

decaying flesh earning it the nickname

ldquocorpse flower

Q141 Deficiency of which vitamin

causes night blindness

ककसपवटालमनकीकमीकहॳ कारणरतौधीहह८ताहहॴ 24-Jan-2017

Options

1) Vitamin K

पवटालमन K

2) Vitamin C

पवटालमन C

3) Vitamin B1

पवटालमन B1

4)Vitamin A

पवटालमन A

Correct Answer Vitamin A

Q142 Nongreen plants lack which of the

following

गहॴर-

हररतवनसपनतमनननननलिखितमसहॳककसकीकमीहह८तीहहॴ

24-Jan-2017

Options

1) Chlorophyll

किह८रह८कफि

2) Lycophyll

िायकह८कफि

3) Cyanophyll

F A C E B O O K

P A G E h t t p w w w f a c e b o o k c o m s s c m e n t o r s o f f i c i a l P a g e | 34

FOR MORE UPDATES AND MORE MATERIAL DO LIKE OUR FACEBOOK PAGE httpwwwfacebookcomsscmentorsofficial

सायनह८कफि

4)Phototropism

फह८टह८टरोपपजम

Correct Answer Chlorophyll

Q143 Organisms that use light to

prepare food are known as

जह८जीवपरकािकाउपयह८गकरभह८जनतहॴयारकरतहॳहहॴ उनह mdashmdash- कहॳ पमजानजाताहहॴ 24-Jan-2017

Options

1) Autotrophs

सवपह८षी 2) Heterotrophs

पवषमपह८षज

3) Omnivores

सवायहारी 4)Decomposers

पवघटनकरनहॳवािा Correct Answer Autotrophs

autotrophs often make their own food

by using sunlight carbon dioxide and

water to form sugars which they can use

for energy Some examples of

autotrophs include plants algae and

even some bacteria Autotrophs

(producer) are important because they

are a food source for heterotrophs

(consumers)

A heterotroph is an organism that

ingests or absorbs organic carbon

(rather than fix carbon from inorganic

sources such as carbon dioxide) in order

to be able to produce energy and

synthesize compounds to maintain its

life Ninety-five percent or more of all

types of living organisms are

heterotrophic including all animals and

fungi and some bacteria

Q144 Which of the following is a

primary function of haemoglobin

नननननलिखितमसहॳकह९नसाटहमह८गिह८बबनकाएकपरािलमककाययहहॴ

25-Jan-2017

Options

1) Utilization of energy

उजायकाउपयह८गकरना 2) Prevention of anaemia

रकतामपताहह८नहॳसहॳरह८कना 3) Destruction of bacteria

बहॴकटीररयाकापवनािकरना 4) To transport oxygen

ऑकसीजनकावहनकरना Correct Answer To transport oxygen

Q145 Vascular bundles are absent in

सवहनीबिि mdashmdash- मअनपनसतिरहतहॳहहॴ 25-Jan-2017

Options

1) Bryophyta

िायह८फाइटा 2) Pteridophyta

टहॳररिह८फाईटा 3) Gymnosperms

नजननह८सपमय 4) Angiosperms

एननजयह८सपहॳनसय Correct Answer Bryophyta

Q146 Sauria Lacertidae is the scientific

name of

सहॴररयािहॳसरटाईिी mdashmdashndash कावहॴजञाननकनामहहॴ 25-Jan-2017

Options

1) Crocodile

मगरमचछ

2) Hippopotamus

टहपपह८पह८टहॳमस

3) Lizard

नछपकिी 4) House fly

F A C E B O O K

P A G E h t t p w w w f a c e b o o k c o m s s c m e n t o r s o f f i c i a l P a g e | 35

FOR MORE UPDATES AND MORE MATERIAL DO LIKE OUR FACEBOOK PAGE httpwwwfacebookcomsscmentorsofficial

घरहॳिमकिी Correct Answer Lizard

Q147 Which type of pathogen causes

the water-borne disease SARS (Severe

Acute Respiratory Syndrome)

ककसपरकािकारह८गज़नकजिजननतबीमारीसासयकाकारणबनताहहॴ 25-Jan-2017

Options

1) Viral

वायरि

2) Parasitic

परजीवी 3) Protozoan

परह८टह८जअन

4) Bacterial

बहॴकटीररयि

Correct Answer Viral

Q148 Which of the following organs

produces the enzyme lipase

नननननलिखितमसहॳकह९नसाअगिायपहॳजएजाइमउतपननकरताहहॴ 25-Jan-2017

Options

1) Pancreas

अगनयािय

2) Large Intestine

बड़ीआत

3) Liver

नजगर

4) Small Intestine

छह८टीआत

Correct Answer Pancreas

Q149 A is a long internode forming the

basal part or the whole of a peduncle

एक mdashmdash- एकिबाइटरनह८िहहॴ जह८ननचिाटहससायासनपणयिठिबनताहहॴ 25-

Jan-2017

Options

1) Rhizome

परकद

2) Rachis

महॳ दि

3) floral axis

पषपअकष

4) Scape

भगदड़

Correct Answer scape

Q150 ndash Which of the following

organisms are considered to be both

Living and Non-living

नननननलिखितमसहॳकह९नसहॳजीवाणकह८जीपवतऔरअजीपवतमानाजाताहहॴ

25-Jan-2017

Options

1) Bacteria

बहॴकटीररया 2) Fungi

कवक

3) Algae

िहॴवाि

4)Virus

वायरस

Correct Answer Virus

They are considered to be living as they

possess a protein coat as a protective

covering DNA as the genetic material

etc

They are said to be non-living as they

can be crystallised and they survive for

billions of years They can tolerate high

temperatures freezing cold

temperatures ultra-violet radiations etc

Q151 Deficiency of fluorine causes

which of the following

फिह८ररनकीकमीकहॳ कारणनननननलिखितमसहॳकयाहह८ताहहॴ

F A C E B O O K

P A G E h t t p w w w f a c e b o o k c o m s s c m e n t o r s o f f i c i a l P a g e | 36

FOR MORE UPDATES AND MORE MATERIAL DO LIKE OUR FACEBOOK PAGE httpwwwfacebookcomsscmentorsofficial

27-Jan-2017

Options

1) Dental Caries

िटिकहॴ ररज

2) Scurvy

सकवरी 3) Anaemia

रकतामपता 4) Arthritis

गटठया Correct Answer Dental Caries

Q152 In a Punnett Square with the

cross AaBb x AaBb how many Aabb

genotypes would be created

पनहॳटसककायरमिह८स AaBb x AaBb कहॳ साि

ककतनहॳ Aabb जीनह८टाइपबनगहॳ 27-Jan-2017

Options

1) 1

2) 8

3) 2

4) 3

Correct Answer 2

Q153 Which of the following is the

Controlling Center of the Cell

नननननलिखित म सहॳ कह८लिकाका ननयतरण

क दर कह९न हहॴ

27-Jan-2017

Options

1) Nucleus

क दर

2) Plasma

पिाजमा 3) Lysosome

िायसह८सह८म

4) Chromosome

िह८मह८सह८म

Correct Answer Nucleus

The control centre of the cell is the

nucleus in eukaryotic cells The nucleus

contains genetic material in the form of

DNA

Q154 Myopia affects which of the

following organs

मायह८पपयानननननलिखितअगह८मसहॳककसहॳपरभापवतकरताहहॴ

25-Jan-2017

Options

1) Heart

हदय

2) Skin

तवचा 3) Eyes

आािहॳ 4)Mouth

मह

Correct Answer Eyes

Q155 Which of the following bears

flowers

नननननलिखितमसहॳकह९नफिधारणकरताहहॴ

25-Jan-2017

Options

1) Bryophyta

िायह८फाइटा 2) Pteridophyta

टहॳरीिह८फाईटा 3) Gymnosperms

नजननह८सपमय 4)Angiosperms

एननजयह८सपमय Correct Answer Angiosperms

Q156 Oxygenated blood flows out of the

heart through the

ऑकसीजनयकतरकत mdashmdashmdash

कहॳ माधयमसहॳहदयकहॳ बाहरबहताहहॴ 25-Jan-2017

F A C E B O O K

P A G E h t t p w w w f a c e b o o k c o m s s c m e n t o r s o f f i c i a l P a g e | 37

FOR MORE UPDATES AND MORE MATERIAL DO LIKE OUR FACEBOOK PAGE httpwwwfacebookcomsscmentorsofficial

Options

1) Aorta

महाधमनी 2) pulmonary artery

फहॳ फड़हॳकीधमनी 3) vena cava

वहॳनाकावा 4)Atrium

चह९क

Correct Answer aorta

Q157 Blood leaving the liver and

moving towards the

heart has a higher concentration of

नजगरसहॳननकिकरहदयकीतरफजानहॳवािहॳरकतम mdashmdashmdashmdash कीउचचसादरताहह८तीहहॴ 27-Jan-2017

Options

1) Lipids

लिपपडस

2) Urea

यररया 3) Bile Pigments

पपततकहॳ रगकरण

4) Carbon dioxide

काबयनिायऑकसाइि

Correct Answer Bile Pigments

Urea is nitrogen containing substance

which is produced in the liver in order

to deal with excess amino-acids in the

body As urea is produced it leaves the

liver in the blood stream and passes via

the circulatory system to all parts of the

body

Q158 Bulb is a modification of which

part of a plant

बमबएकपह९धहॳकहॳ ककसटहससहॳकाएक पातरणहह८ताहहॴ 27-Jan-2017

Options

1) The root

जड़

2) The stem

तना 3) The radicle

मिाकर

4)The fruit

फि

Correct Answer The stem

Q159 Which of the following carries

blood away from the heart to different

body parts

इनमहॳसहॳकह९नरकतकह८हदयसहॳिरीरकहॳ पवलभननअगह८तकिहॳजातीहहॴ

27-Jan-2017

Options

1) Arteries

धमननया 2) Nerves

तबतरहाए

3) Capillaries

कहॳ लिकाए

4)Veins

नसहॳ Correct Answer Arteries

Q160 The series of processes by which

nitrogen and its compounds are

interconverted in the environment and

in living organisms is called

27-Jan-2017

Options

1)Absorption of Nitrogen

2)Ammonification

3)Nitrogen Fixation

4)Nitrogen Cycle

Correct Answer Nitrogen Cycle

Ammonification or Mineralization is

performed by bacteria to convert

organic nitrogen to ammonia

F A C E B O O K

P A G E h t t p w w w f a c e b o o k c o m s s c m e n t o r s o f f i c i a l P a g e | 38

FOR MORE UPDATES AND MORE MATERIAL DO LIKE OUR FACEBOOK PAGE httpwwwfacebookcomsscmentorsofficial

Nitrification can then occur to convert

the ammonium to nitrite and nitrate

Nitrogen fixation is a process by which

nitrogen in the Earthrsquos atmosphere is

converted into ammonia (NH3) or other

molecules available to living organisms

Q161 BCG vaccine is given to protect

from which of the following

बीसीजीकाटटकानननननलिखितमसहॳककसकहॳ बचावकहॳ लिएटदयाजातहहॴ

27-Jan-2017

Options

1) Jaundice

पीलिया 2) Anaemia

रकतमपता 3) Tuberculosis

कषयरह८ग

4) Polio

पह८लियह८ Correct Answer Tuberculosis

Q162 Parallel venation is found in

समानतरवहॳनहॳिन mdashmdashmdash- मपायाजाताहहॴ 27-Jan-2017

Options

1) plants which are monocots

पह९धहॳजह८एकबीजपतरीहह८तहॳहहॴ 2) plants which have a dicot stem

वहॳपह९धहॳनजनकातनादपवदलियहह८ताहहॴ 3) plants with leaves similar to Tulsi

वहॳपह९धहॳनजनकीपनततयतिसीकीपनततयोकहॳ समानहह८तहॳहहॴ 4)plants with tap roots

टहॳप टवािहॳपह९धहॳ Correct Answer plants which are

monocots

Q163 The hardest part of the body is

िरीरकासबसहॳकठह८रभाग mdashndash हहॴ 27-Jan-2017

Options

1) Bones

हडडिय

2) Tooth Enamel

दातकहॳ इनहॳमि

3) Skull

िह८पड़ी 4) Spinal Cord

महॳ रजज

Correct Answer Tooth Enamel

Q164 Which type of pathogen causes

the waterborne disease E coli Infection

ककसपरकारकारह८गजननकजिजननतरह८गईकह८िाईसिमणकाकारणबनताहहॴ 27-Jan-2017

Options

1) Protozoan

परह८टह८जआ

2) Parasitic

परजीवी 3) Bacterial

बहॴकटीररयि

4)Viral

वायरि

Correct Answer Bacterial

Q165 The amount of blood filtered

together by both the kidneys in a 70 kg

adult male human in a minute is

70 की गरा वािहॳएकवयसकप षमएकलमनटमदह८नोगदकहॳदवाराएकसािचाबनीगयीरकतकीमातरहह८तीहहॴ 29-Jan-2017

Options

1) 1100 ml

1100 लमलि

2) 100 ml

F A C E B O O K

P A G E h t t p w w w f a c e b o o k c o m s s c m e n t o r s o f f i c i a l P a g e | 39

FOR MORE UPDATES AND MORE MATERIAL DO LIKE OUR FACEBOOK PAGE httpwwwfacebookcomsscmentorsofficial

100 लमलि

3) 1500 ml

1500 लमलि

4) 500 ml

500 लमलि

Correct Answer 1100 ml

Q166 Which feature of a plant helps to

distinguish a monocot from a dicot

पह९धहॳकीवहकह९नसीपविहॳषताहहॴजह८एकदपवदलियहॳऔरएकएकदिीयपह९धहॳसहॳभहॳदकरनहॳममददकरतीहहॴ 29-Jan-2017

Options

1) Pollination

परागम

2) Venation

वहॳनहॳिन

3) Vernation

वनिन

4) Aestivation

एसटीवहॳिहॳन

Correct Answer venation

Q167 The Mutation Theory was

proposed by

उतवररवतयनकालसदात mdashmdashndash

कहॳ दवरापरसतापवतककयाजाताहहॴ 29-Jan-2017

Options

1) Charles Lyell

चामसयलियहॳि

2) William Smith

पवलियमनसमि

3) Hugo De Vries

हयगह८िीराईस

4)Harrison Schmitt

हहॳरीसननसमट

Correct Answer Hugo De Vries

Q168 Which type of pathogen causes

the waterborne disease HepatitisA

ककसपरकारकहॳ रह८गजनकजिजननतरह८गहहॳपहॳटाइटटस-A काकारणबनताहहॴ

29-Jan-2017

Options

1) Parasitic

परजीवी 2) Viral

वायरि

3) Protozoan

परह८टह८जआ

4) Bacterial

बहॴकटीररयि

Correct Answer Viral

Q169 In a Punnett Square with the

cross AaBb x Aabb how many AaBb

genotypes would be created

पनहॳटसकवायरमिह८स AaBb x Aabb

कहॳ सािककतनहॳ AaBb जीनह८टाइपबनगहॳ 29-Jan-

2017

Options

1) 4

2) 1

3) 7

4) 6

Correct Answer 4

Q170 Arboreal Ateles is the scientific

name of

अिह८ररयिएटटलिस mdashmdashmdash कावहॴजञाननकनामहहॴ 29-Jan-2017

Options

1) Squirrel

चगिहरी 2) Sparrow

गह८रहॴया 3) Lizard

नछपकिी 4) Spider monkey

F A C E B O O K

P A G E h t t p w w w f a c e b o o k c o m s s c m e n t o r s o f f i c i a l P a g e | 40

FOR MORE UPDATES AND MORE MATERIAL DO LIKE OUR FACEBOOK PAGE httpwwwfacebookcomsscmentorsofficial

मकड़ीबदर

Correct Answer Spider monkey

Q171 Which type of pathogen causes

the waterborne disease Salmonellosis

ककसपरकारकारह८गाणजिजननतबीमारीसािमह८नहॳिह८लसज़काकारकहहॴ

29-Jan-2017

Options

1) Algal

िहॳवालियहॳ 2) Parasitic

परजीवी 3) Bacterial

बहॴकटीररयि

4)Viral

वायरि

Correct Answer Bacterial

An infection with salmonella bacteria

commonly caused by contaminated food

or water

Symptoms include diarrhoea fever

chills and abdominal pain

Q172 is a condition in which there is a

deficiency of red cells or of haemoglobin

in the blood

mdashmdash-

एकनसिनतहहॴनजसमहॳरकतमिािकह८लिकाओकीयाहीमह८गिह८बबनकीकमीहह८तीहहॴ 29-Jan-2017

Options

1) Albinism

एनमबननजम

2) Propyria

परह८पीररया 3) Anaemia

एनीलमया 4)Keloid disorder

कहॳ िह८इिडिसओिर

Correct Answer Anaemia

Q173 Ananas comosus is the scientific

name of

Options

अनानासकह८मह८सस mdashmdashmdashndash

कावहॴजञाननकनामहहॴ 29-Jan-2017

1) Custard Apple

सीताफि

2) Pineapple

पाइनएपपि

3) Bamboo

बास

4)Pomegranate

अनार

Correct Answer Pineapple

Q174 Which organ produces insulin

कह९नसाअगइनसलिनपहॴदाकरताहहॴ 29-Jan-

2017

Options

1) Liver

यकत

2) Thyroid gland

िायराइिगरिी 3) Spleen

पिीहा 4)Pancreas

अगरयिय

Correct Answer Pancreas

Q175 Which of the following disease is

not caused by water pollution

नननननलिखितमसहॳकह९नसारह८गपानीकहॳ परदषणकहॳकारणनहीहह८ता

29-Jan-2017

Options

1) Cholera

हहॴजा 2) Typhoid

F A C E B O O K

P A G E h t t p w w w f a c e b o o k c o m s s c m e n t o r s o f f i c i a l P a g e | 41

FOR MORE UPDATES AND MORE MATERIAL DO LIKE OUR FACEBOOK PAGE httpwwwfacebookcomsscmentorsofficial

टाइफाइि

3) Asthma

दमा 4)Diarrhoea

दसत

Correct Answer Asthma

Q176 Ocimum tenuiflorum is the

scientific name of

ओलिलममटहॳयईफिह८रमइसकावहॴजञाननकनाम mdash

ndash हहॴ 30-Jan-2017

Options

1) Neem

नीम

2) Mango

आम

3) Babul

बबि

4)Tulsi

तिसी Correct Answer Tulsi

Q177 Which gland secretes bile a

digestive fluid

कह९नसीगरिीपपतत एकपाचनतरिपरदािय सरापवतकरतीहहॴ 30-Jan-2017

Options

1) Pancreas

अगनयािय

2) Liver

यकत

3) Thyroid

िायराइि

4) Testes

टहॳनसटस

Correct Answer liver

Q178 In which of the following the

dominant phase is Gametophyte

नननननलिखितमसहॳककसकहॳ परमिचरणयगमकह८दपवधद (Gametophyte)हहॴ 30-Jan-2017

Options

1) Bryophyta

िायह८फाइटा 2) Pteridophyta

टहॳररिह८फाइटा 3) Gymnosperms

नजननह८सपमय 4) Angiosperms

एननजयह८सपमय Correct Answer Bryophyta

Q179 Anaerobic respiration refers to

which of the following

नननननलिखितमसहॳककसहॳअवायवीयशवसनकहाजाताहहॴ

30-Jan-2017

Options

1) Respiration without Oxygen

ऑकसीजनकहॳ बबनाशवसन

2) Respiration with Oxygen

ऑकसीजनकहॳ सािशवसन

3) Respiration without CO2

काबयनिायऑकसाइिकहॳ बबनाशवसन

4) Respiration with CO2

काबयनिायऑकसाइिकहॳ सािशविन

Correct Answer Respiration without

Oxygen

Q180 Which type of pathogen causes

the waterborne disease Cholera

ककसपरकारकारह८गजनकजिजननतरह८गहहॴजाकाकारणबनताहहॴ

30-Jan-2017

Options

1) Algal

िहॴवालियहॳ

F A C E B O O K

P A G E h t t p w w w f a c e b o o k c o m s s c m e n t o r s o f f i c i a l P a g e | 42

FOR MORE UPDATES AND MORE MATERIAL DO LIKE OUR FACEBOOK PAGE httpwwwfacebookcomsscmentorsofficial

2) Bacterial

बहॴकटीररयि

3) Protozoan

परह८टह८जआ

4) Viral

वायरि

Correct Answer Bacterial

Q181 To which class does

Oxyreductases transferases hydrolases

belong

ओकसीररिकटहॳसटरासफरहॳजहॳस

हाइडरह८िहॳसहॳसककसवगयमआतहॳहहॴ 30-Jan-2017

Options

1) Hormones

हारमोस

2) Enzymes

एजाइनस

3) Proteins

परह८टीनस

4) Vitamins

पवटालमनस

Correct Answer Enzymes

Q182 Which of the following is not true

about Gymnosperms

ननननमसहॳकह९नसीबातअनावतबीजीकहॳ बारहॳमसचनहीहहॴ 30-Jan-2017

Options

1) Dominant phase is saprophytes

परमिचरणसहॳपरह८फाइटसहह८ताहहॴ 2) Vascular bundles are absent

सवहनीबििअनपनसितहह८ताहहॴ 3) spores are heterospores

बीजाणहहॳटहॳरह८सपह८रसहह८तहॳहहॴ 4) Flowers are absent

फिअनपनसितहह८तहॳहहॴ

Correct Answer Vascular bundles are

absent

Q183 The name of first mammal clone sheep is

भहॳड़कीपरिमसतनपायीपरनत प (किह८न)

कानामहहॴ 30-Jan-2017

Options

1) Noori

नरी 2) Dolly

िॉिी 3) Louise

िसी 4)Durga

दगाय Correct Answer Dolly

Q184 Which type of pathogen causes

the water-borne disease Typhoid fever

ककसपरकारकारह८गजनकजिजननतरह८गटाइफाइिबिारकाकारणबनताहहॴ 30-Jan-2017

Options

1) Algal

िहॴवािीय

2) Parasitic

परजीवी 3) Protozoan

परह८टह८जनअन

4)Bacterial

बहॴकटीररयि

Correct Answer Bacterial

Q185 In which part of the cell are

proteins made

कह८लिकाकहॳ ककसटहससहॳमपरह८टीनबनायाजाताहहॴ

31-Jan-2017

Options

1) Reticulum

रहॳटटकिम

F A C E B O O K

P A G E h t t p w w w f a c e b o o k c o m s s c m e n t o r s o f f i c i a l P a g e | 43

FOR MORE UPDATES AND MORE MATERIAL DO LIKE OUR FACEBOOK PAGE httpwwwfacebookcomsscmentorsofficial

2) Golgi apparatus

गह८मजीएपहॳरहॳटस

3) Ribosomes

ररबह८सह८नस

4) Lysosome

िायसह८सह८नस

Correct Answer ribosomes

Proteins are produced by stringing

amino acids together in the order

specified by messenger RNA strands

that were transcribed from DNA in the

cell nucleus The process of synthesizing

a protein is called translation and it

occurs on ribosomes in the cytoplasm of

a cell

Q186 Polio is a disease caused by which

of the following

नननननलिखितमसहॳपह८लियह८कीबबमारह८हह८नहॳकाकारणकयाहहॴ

31-Jan-2017

Options

1) Bacteria

बहॴकटीररयि

2) Mosquito

मचछर

3) Virus

वायरस

4) Cockroach

नतिच हॳ Correct Answer Virus

Polio or poliomyelitis is a crippling and

potentially deadly infectious disease It

is caused by the poliovirus

Q187 ndash Hay fever is a sign of which of

the following

हहॳकफवरनननननलिखितमसहॳककसकाएकसकहॳ तहहॴ

31-Jan-2017

Options

1) Old Age

वदावसिा 2) Malnutrition

कपह८सण

3) Allergy

एिनजय 4) Over Work

अतयचधककाययकरना Correct Answer Allergy

Q188 How many chromosomes does a

human cell contain

एकमानवकह८लिकामककतनहॳगणसतरहह८तहॳहहॴ

29-Jan-2017

Options

1) 6

2) 26

3) 46

4) 66

Correct Answer 46

In humans each cell normally contains

23 pairs of chromosomes for a total of

46 Twenty-two of these pairs called

autosomes look the same in both males

and females The 23rd pair the sex

chromosomes differ between males and

females

Q189 Which of the following is not true

about Bryophyta

ननननमसहॳकह९नसीबातिायह८फाइटकहॳ बारहॳमसचनहीहहॴ 31-Jan-2017

Options

1) Dominant phase is gametophytes

परमिचरणगहॳलमतह८फाइटसहह८ताहहॴ 2) Main plant body is haploid

पह९धहॳकामखयिरीरअगखणतहह८ताहहॴ 3) Spores are homospores

बीजाणहह८मह८सफह८रसहह८तहॳहहॴ 4) Flowers are present

फिमह८जदहह८तहॳहहॴ Correct Answer Flowers are present

F A C E B O O K

P A G E h t t p w w w f a c e b o o k c o m s s c m e n t o r s o f f i c i a l P a g e | 44

FOR MORE UPDATES AND MORE MATERIAL DO LIKE OUR FACEBOOK PAGE httpwwwfacebookcomsscmentorsofficial

Q190 Which aquatic animal has

trailing tentacles

ककसजिीयजानवरकहॳ पीछहॳचिनहॳवािहॳटहॳटकिसहह८तहॳहहॴ

31-Jan-2017

Options

1) Sea horse

समदरीघह८िा 2) Corals

मगा 3) Jelly fish

जहॳिीमछिी 4) Star fish

तारामछिी Correct Answer Jelly fish

Jellyfish with its umbrella-shaped bell

and trailing tentacles

Q191 Which type of pathogen causes

the water-borne disease Poliomyelitis

(Polio)

ककसपरकारकारह८गजनकजिजननतरह८गपह८लियह८मायहॳटटस (पह८लियह८) काकारणहहॴ 31-Jan-

2017

Options

1) Parasitic

परजीवी 2) Algal

िहॴवालिय

3) Viral

वायरि

4) Bacterial

बहॴकटीररयि

Correct Answer Viral

Q192 The outer white part of the eye

that protects the inner structures is

आािकाबाहरीसफहॳ दटहससाजह८आतररकसरचनाओकीरकषाकरताहहॴ वह mdashmdashmdash हहॴ 31-Jan-

2017

Options

1) Iris

आयररस

2) Sclera

सकिहॳरा 3) Retina

रहॳटटना 4) Cornea

कह८ननयया Correct Answer Sclera

Q193 Proteins are made up of

परह८टीनकाननमायण mdashndash सहॳहह८ताहहॴ 31-Jan-2017

Options

1) Amino acids

एलमनह८अनि

2) Fatty acids

वसायकतअनि

3) Glucose

गिकह८ज

4)Nucleotides

नयनकियह८टाईिस

Correct Answer Amino acids

Q194 Moringa Oleifera is the scientific

name of

मह८ररगओलिफहॳ रा mdashmdashndash कावहॴजञाननकनामहहॴ 31-Jan-2017

Options

1) Banyan

बरगद

2) Gulmohar

गिमह८हर

3) Amla

आमिा

F A C E B O O K

P A G E h t t p w w w f a c e b o o k c o m s s c m e n t o r s o f f i c i a l P a g e | 45

FOR MORE UPDATES AND MORE MATERIAL DO LIKE OUR FACEBOOK PAGE httpwwwfacebookcomsscmentorsofficial

4) Drumstick

डरमनसटक

Correct Answer Drumstick

Q195 Kidney stones are composed of

गदकीपिरी mdashndash सहॳबनीहह८तीहहॴ 1-Feb-2017

Options

1) Calcium Oxalate

कहॴ नमसयमओकजहॳिहॳट

2) Sodium Chloride

सह८डियमकिह८राइि

3) Magnesium Nitrate

महॳनगनलियमनाइतटरहॳट

4) Calcium Bicarbonate

कहॴ नमियमबायकबोनहॳट

Correct Answer Calcium Oxalate

Q196 ndash Which of the following is not

true about Angiosperms

ननननमसहॳकह९नसीबातआवतबीजीकहॳ बारहॳमसचनहीहहॴ 1-Feb-2017

Options

1) Dominant phase is gametophytes

परमिचरणगहॳलमतह८फाइटहह८ताहहॴ 2) Vascular bundles are present

सवहनीबििमह९जदहह८ताहहॴ 3) Spores are heterospores

बीजाणहहॳटहॳरह८सपह८रसहह८तहॳहहॴ 4) Seeds are covered

बीजढकहॳ हह८तहॳहहॴ Correct Answer Dominant phase is

gametophytes

Q197 All of the following are excretory

(waste) products of animals except

नननननलिखितमसहॳककसएककह८छह८ड़करअनयसभीपराखणयोदवाराउतसनजयतपदाियहहॴ 1-Feb-

2017

Options

1) Uric Acid

यररकएलसि

2) Ammonia

अमह८ननया 3) Carbohydrates

काबोहाइडरहॳट

4) Urea

यररया Correct Answer Carbohydrates

In animals the main excretory products

are carbon dioxide ammonia (in

ammoniotelics) urea (in ureotelics) uric

acid (in uricotelics) guanine (in

Arachnida) and creatine

Q198 RNA is a polymeric molecule

What does RNA stand for

आरएनइएएकबहिकआणहहॴ इसकाकापवय पकयाहहॴ 1-Feb-2017

Options

1) Rado Nuclear Acid

रािह८नयनकियरएलसि

2) Ribo Nucleic Acid

राइबह८नयनकिकएलसि

3) Rhino Nuclear Acid

हाइनह८नयनकियरएलसि

4) Resto Nucleus Acid

रहॳसटह८नयकिीयसएलसि

Correct Answer Ribo Nucleic Acid

Q199 Which organ does detoxification

and produces chemicals needed for

digestion

कह९नसाअगपवषहरणकरताहहॴऔरपाचनकहॳ लिएआवशयकरसायनोकह८पहॴदाकरताहहॴ 1-Feb-

2017

Options

1) Salivary glands

िारगरचिया 2) Pancreas

अगनयािय

F A C E B O O K

P A G E h t t p w w w f a c e b o o k c o m s s c m e n t o r s o f f i c i a l P a g e | 46

FOR MORE UPDATES AND MORE MATERIAL DO LIKE OUR FACEBOOK PAGE httpwwwfacebookcomsscmentorsofficial

3) Thyroid gland

िायराइिगरिी 4) Liver

यकत

Correct Answer Liver

Q200 Psidium guajava is the scientific

name of

लसडियमगआजावा mdashmdash कावहॴजञाननकनामहहॴ 1-

Feb-2017

Options

1) Guava

अम द

2) Mango

आम

3) Bamboo

बास

4) Jack fruit

कटहि

Correct Answer Guava

Q201 Which drug is used as a Blood

Thinner

चधरकह८पतिाकरनहॳकहॳ पमककसदवाकापरयह८गककयाजाताहहॴ

1-Feb-2017

Options

1) Warfarin

वाफर न

2) Tramadol

टरहॳमािह८ि

3) Azithromycin

एनजरह८मायलसन

4) Hydralazine

हाइडरह८िहॳनजन

Correct Answer Warfarin

Q202 Which of the following disease is

caused due to the deficiency of protein

परह८टीनकीकमीकहॳ कारणनननननलिखितमसहॳकह९नसारह८गहह८ताहहॴ 1-Feb-2017

Options

1) Arthritis

गटठया 2) Kwashiorkor

कािीओकय र

3) Goitre

गाइटर

4) Night Blindness

रतह९चध

Correct Answer Kwashiorkor

Q203 A is species of plant that has

adapted to survive in an environment

with little liquid water

mdashmdashndashपह९धहॳकीएकऐसहॳऐसहॳपरजानतहहॴ नजसनहॳकमपानीवािहॳवातावरणमजीपवतरहनहॳकहॳलिएअनकिनहहॴ 1-Feb-2017

Options

1) Xerophyte

म दपवद

2) Hydrophyte

जिीयपादप

3) Mesophyte

समह८दपवद

4) Thallophyte

िहॴिह८फाइटा Correct Answer xerophyte

xerophyte is a species of plant that has

adapted to survive in an environment

with little liquid water such as a desert

or an ice- or snow-covered region in the

Alps or the Arctic

Mesophytes are terrestrial plants which

are adapted to neither a particularly

dry nor particularly wet environment

An example of a mesophytic habitat

would be a rural temperate meadow

F A C E B O O K

P A G E h t t p w w w f a c e b o o k c o m s s c m e n t o r s o f f i c i a l P a g e | 47

FOR MORE UPDATES AND MORE MATERIAL DO LIKE OUR FACEBOOK PAGE httpwwwfacebookcomsscmentorsofficial

which might contain goldenrod clover

oxeye daisy and Rosa multiflora

thallophyte any of a group of plants or

plantlike organisms (such as algae and

fungi) that lack differentiated stems

leaves and roots and that were formerly

classified as a primary division

(Thallophyta) of the plant kingdom

Q204 How many types of teeth are

there in humans

मनषयोमककतनहॳपरकारकहॳ दातहह८तहॳहहॴ

1-Feb-2017

Options

1) 4

2) 5

3) 2

4) 3

Correct Answer 4

teeth -Humans have four types of

teethincisors canines premolars and

molars each with a specific function

The incisors cut the food the canines

tear the food and the molars and

premolars crush the food

Q205 Carica papaya is the scientific name of

कहॴ ररकापपाया mdashmdashndash कावहॴजञाननकनामहहॴ 2-

Feb-2017

Options

1) Peepal

पीपि

2) Papaya

पपीता 3) Tamarind

इमिी 4) Drumstick

ढह८िकाछड़ी Correct Answer Papaya

Q206 Muscles get tired when there is

shortfall of

जब mdashndash कीकमीहह८तीहहॴतबपहॳिीयिकजातीहहॴ 2-Feb-2017

Options

1) Lactic acid

िहॴनकटकएलसि

2) Na+ ions

Na+ आयन

3) ATP

एटीपी 4) Sulphates

समफहॳ टस

Correct Answer ATP

ATP is the energy source muscle fibers

use to make muscles contract

muscle tissuersquos main source of energy

called adenosine triphosphate or ATP

As your muscles use up this energy

source they become tired and fatigued

Oxygen is the key ingredient that helps

create new ATP to replenish the burned

up ATP in your muscles

Q207 Artocarpus integra is the

scientific name of आटह८कापयसइटीगरा mdashmdashmdash कावहॴजञाननकनामहहॴ 2-Feb-2017

Options

1) Guava

अम द

2) Pineapple

अनानास

3) Silver Oak

लसमवरओक

4) Jack fruit

कटहि

Correct Answer Jack fruit

Q208 Which organ stores fat soluble

vitamins

कह९नसाअगवसामघिनिीिपवटालमनह८काभिाराकरताहहॴ

2-Feb-2017

F A C E B O O K

P A G E h t t p w w w f a c e b o o k c o m s s c m e n t o r s o f f i c i a l P a g e | 48

FOR MORE UPDATES AND MORE MATERIAL DO LIKE OUR FACEBOOK PAGE httpwwwfacebookcomsscmentorsofficial

Options

1) Blood

रकत

2) Skin

तवचा 3) Liver

यकत

4) Pancreas

अगनयािय

Correct Answer Liver

Q209 Which disease is caused due to

deficiency of Iodine

आयह८िीनकहॳ कारणकह९नसारह८गहह८ताहहॴ 2-Feb-2017

Options

1) Rickets

ररकहॳ टस

2) Scurvy

सकवी 3) Goitre

गणमािा 4) Growth retardation

पवकासका कना Correct Answer Goitre

rickets A softening and weakening of

bones in children usually due to

inadequate vitamin D

Q210 Grevillea Robusta is the scientific name of

गरहॳपवलियारह८बसटा mdashmdashmdash- कापवजञाननकनामहहॴ 2-Feb-2017

Options

1) Peepal

पीपि

2) Teak

सागह९न

3) Silver Oak

लसमवरओक

4) Jack fruit

कटहि

Correct Answer Silver Oak

Q211 When a Cuttlefish is described as a Molluscs it is at which level of

classification

जबएककटिकफिकह८एकमह८िसकाकहॳ पमवखणयतककयाजाताहहॴतबयहॳवगीकरणकहॳ ककससतरपहॳनसितहहॴ 2-Feb-2017

Options

1) Class

वगय 2) Order

िम

3) Family

पररवार

4) Phylum

सघ

Correct Answer Phylum

Q212 Bambusa dendrocalmus is the

scientific name of बानबसािहॳडराकामस mdashmdashmdash कावहॴजञाननकनामहहॴ 3-Feb-2017

Options

1) Banyan

बरगद

2) Papaya

पपीता 3) Bamboo

बास

4) Pomegranate

अनार

Correct Answer Bamboo

Q213 Acinonyx Jubatus is the scientific name of

एलसनह८ननकसजयबहॳटस mdashmdashmdash

कावहॴजञाननकनामहहॴ 3-Feb-2017

F A C E B O O K

P A G E h t t p w w w f a c e b o o k c o m s s c m e n t o r s o f f i c i a l P a g e | 49

FOR MORE UPDATES AND MORE MATERIAL DO LIKE OUR FACEBOOK PAGE httpwwwfacebookcomsscmentorsofficial

Options

1) Bear

भाि 2) Horse

घह८िा 3) Cheetah

चीता 4) Zebra

जहॳिा Correct Answer Cheetah

Q214 The pale yellow colour of urine is

due to the presence of which pigment

मतरकाफीकापीिारगरगदरयकहॳ उपनसिनतकहॳ कारणहह८ताहहॴ

3-Feb-2017

Options

1) Urochrome

यरह८िह८म

2) Urophyll

यरह८कफि

3) Chlorophyll

किह८रह८कफि

4) Chloroplast

किह८रह८पिासट

Correct Answer Urochrome

Q215 Which of the following constitute

to form a gene

नननननलिखितमसहॳकह९नसीचीज़एकजीनकागठनकरतीहहॴ

3-Feb-2017

Options

1) Polynucleotides

पह८िीनयनकियह८टाईडस

2) Hydrocarbons

हाइडरह८काबोस

3) Lipoproteins

िाईपह८परह८टीनस

4) Lipids

लिपपडस

Correct Answer Polynucleotides

Polynucleotide molecule is a biopolymer

composed of 13 or more nucleotide

monomers covalently bonded in a chain

DNA (deoxyribonucleic acid) and RNA

(ribonucleic acid) are examples of

polynucleotides with distinct biological

function

Q216 Vertebrates belongs to the

phylum

रीढ़कीहडिीवािहॳपराणी mdashmdashmdash

परजानतकहॳ अतगायतआतहॳहहॴ 3-Feb-2017

Options

1) Arthropoda

आरह८पह८ड़ा 2) Annelida

एननलििा 3) Cnidaria

ननिहॳररया 4) Chordata

कह८िटा Correct Answer Chordata

Q217 Punica granatum is the scientific name of

पननकगरहॳनहॳटस mdashmdashmdash कावहॴजञाननकनामहहॴ 3-Feb-2017

Options

1) Custard Apple

सीताफि

2) Gulmohar

गिमह८हर

3) Silver Oak

लसमवरओक

4) Pomegranate

अनार

Correct Answer Pomegranate

F A C E B O O K

P A G E h t t p w w w f a c e b o o k c o m s s c m e n t o r s o f f i c i a l P a g e | 50

FOR MORE UPDATES AND MORE MATERIAL DO LIKE OUR FACEBOOK PAGE httpwwwfacebookcomsscmentorsofficial

Q218 Between a tiger and an monkey

which of the following is different

एकबाघऔरबदरकहॳ बीचनननननलिखितमसहॳकह९नसीबातअिगहहॴ 3-Feb-2017

Options

1) Kingdom

राजय

2) Phylum

जानत

3) Order

िम

4) Class

वगय Correct Answer order

Q219 The artificial heart was invented by

कबतरमहदयका mdashmdashmdash

दवाराअपवषकारककयागयािा 3-Feb-2017

Options

1) Muhammad Yunus

महनमदयनस

2) Linus Yale Jr

िाइनसयहॳिजय

3) Gazi Yasargil

गाजीयासचगयि

4) Paul Winchell

पह९िपवमकि Correct Answer Paul Winchell

Q220 Tamarindus indica is the

scientific name of

टहॳमररनडसइडिका mdashmdash कावहॴजञाननकनामहहॴ 7-

Feb-2017

Options

1) Neem

नीम

2) Pineapple

अनानास

3) Tamarind

इमिी 4)Chiku

चीक

Correct Answer Tamarind

Q221 In eukaryotic cells synthesis of

RNA takes place in the

यकहॳ योटटककह८लिकाओमआरएनएकासशिहॳषण

mdashndash महह८ताहहॴ 7-Feb-2017

Options

1) Mitochondria

माईटह८कोडडरया 2) Centrioles

सटरीयह८मस

3) Ribosomes

ररबह८सह८नस

4) Nucleus

नयनकियस

Correct Answer nucleus

eukaryotic cell -Transcription is the

process of synthesizing ribonucleic acid

(RNA)Synthesis takes place within the

nucleus of eukaryotic cells or in the

cytoplasm of prokaryotes and converts

the genetic code from a gene in

deoxyribonucleic acid ( DNA ) to a

strand of RNA that then directs

proteinsynthesis

Q222 _________is caused by parasites

of the Plasmodium genus

पिाजमह८डियमजातीकहॳ परजीवी mdash- कहॳ कारणहहॴ 7-Feb-2017

Options

1) Dysentery

पहॳचचि

2) Malaria

मिहॳररया 3) Chickenpox

F A C E B O O K

P A G E h t t p w w w f a c e b o o k c o m s s c m e n t o r s o f f i c i a l P a g e | 51

FOR MORE UPDATES AND MORE MATERIAL DO LIKE OUR FACEBOOK PAGE httpwwwfacebookcomsscmentorsofficial

चहॳचक

4) Herpes

हहॳपपयस

Correct Answer Malaria

Q223 Carotene in fruits and vegetables

gives it which color

फिह८औरसनलजयोमनसितकहॳ रह८टीनउनहकह९नसारगपरदानकरताहहॴ 7-Feb-2017

Options

1) Green

हरा 2) Pink

गिाबी 3) Orange

नारगी 4) Blue

नीिा Correct Answer Orange

Q224 Equus Caballus is the scientific

name of

एकवसकहॴ बहॳिस mdashmdashndash कापवजञाननकनामहहॴ 7-Feb-2017

Options

1) Horse

घह८िा 2) Zebra

ज़हॳिा 3) Donkey

गधा 4) Buffalo

भस

Correct Answer Horse

Q225 Elapidae Naja is the scientific name of

एिीपीिीनाजा mdashmdash- कावहॴजञाननकनामहहॴ 8-Feb-2017

Options

1) Cobra

कह८बरा 2) Elephant

हािी 3) Eagle

ग ि

4) Owl

उमि Correct Answer Cobra

Q226 Which disease is caused due to

deficiency of Iron

िह८हकीकमीकहॳ कारणकह९नसारह८गहह८ताहहॴ 8-Feb-

2017

Options

1) Beriberi

बहॳरीबहॳरी 2) Tetany

टहॳटनी 3) Kwashiorkor

कवािीऔरकर

4) Anaemia

रकतामपता Correct Answer Anaemia

Beriberi is a disease caused by a vitamin

B-1 deficiency also known as thiamine

deficiency

Tetany can be the result of an

electrolyte imbalance Most often itrsquos a

dramatically low calcium level also

known as hypocalcemia Tetany can also

be caused by magnesium deficiency or

too little potassium Having too much

acid (acidosis) or too much alkali

(alkalosis) in the body can also result in

tetany

Kwashiorkor also known as

ldquoedematous malnutrition It is a form of

malnutrition caused by a lack of protein

in the diet

Anaemia means that you have fewer red

blood cells than normal or you have less

F A C E B O O K

P A G E h t t p w w w f a c e b o o k c o m s s c m e n t o r s o f f i c i a l P a g e | 52

FOR MORE UPDATES AND MORE MATERIAL DO LIKE OUR FACEBOOK PAGE httpwwwfacebookcomsscmentorsofficial

haemoglobin than normal in each red

blood cell

Q227 is a leaf where the leaflets are

arranged along the middle vein

mdashndashएकपततीहहॴजहापतरकह८कीरचनाक ररयालिराकहॳ आसपासहह८तीहहॴ 8-Feb-2017

Options

1) Pinnately compound leaf

पपनहॳटिीसयकतपतती 2) Palmately compound leaf

पामहॳटिीसयकतपतती 3) Compound leaf

सयकतपतती 4) Simple leaf

साधारणपतती Correct Answer Pinnately compound

leaf

Q228 Haustoria or sucking roots are

found in which of the following

हह८सटह८ररयायाचसनहॳवािीजड़हॳनननननलिखितमसहॳककसमपाईजातीहहॴ 8-Feb-2017

Options

1) Wheat

गहॳह

2) Mango

आम

3) Chestnut

चहॳसटनट

4) Cuscuta

कसकयटा Correct Answer Cuscuta

Haustorial roots -The roots of parasitic

plants which penetrate into the host

tissues to absorb nourishment are

called haustorial roots hellip Also known as suckingor parasitic roots

Q229 Equs Asinus is the scientific name

of

एकवसएलसनस mdashmdashndash कावहॴजञाननकनामहहॴ 8-

Feb-2017

Options

1) Donkey

गधा 2) Cow

गाय

3) Deer

टहरन

4) Kangaroo

कगा

Correct Answer Donkey

Q230 Ficus benghalensis is the scientific name of

फाईकसबहॳनगहॳिहॳलसस mdashndash कापवजञाननकनामहहॴ 8-Feb-2017

Options

1) Banyan

बरगद

2) Pineapple

अनानास

3) Babul

बबि

4) Tulsi

तिसी Correct Answer Banyan

Q231 Equus burchellii is the scientific name of

एकवसबचिी mdashmdash- कापवजञाननकनामहहॴ 8-Feb-2017

Options

1) Horse

घह८िा 2) Zebra

जहॳिा 3) Buffalo

F A C E B O O K

P A G E h t t p w w w f a c e b o o k c o m s s c m e n t o r s o f f i c i a l P a g e | 53

FOR MORE UPDATES AND MORE MATERIAL DO LIKE OUR FACEBOOK PAGE httpwwwfacebookcomsscmentorsofficial

भस

4) Ass

गधा Correct Answer Zebra

Page 27: COMPILATION OF ALL 72 SETS OF BIOLOGY SSC CHSL-2016 · OF BIOLOGY SSC CHSL-2016 PREPARED BY : SSC MENTORS BIOLOGY SPECIAL . F A C E B O O K P A G E : h t t p : / / w w w . f a c e

F A C E B O O K

P A G E h t t p w w w f a c e b o o k c o m s s c m e n t o r s o f f i c i a l P a g e | 26

FOR MORE UPDATES AND MORE MATERIAL DO LIKE OUR FACEBOOK PAGE httpwwwfacebookcomsscmentorsofficial

इसलिन

2) Glucose

गिकह८ज

3) Glycogen

गिायकह८जहॳन

4) Fat

वसा Correct Answer Glycogen

Excess glucose is stored in the liver as

the large compound called glycogen

Glycogen is a polysaccharide of glucose

but its structure allows it to pack

compactly so more of it can be stored in

cells for later use

Q109 Where do plants synthesize

protein from

पह९धहॳपरह८टीनसशिहॳषणकहासहॳकरतहॳहहॴ

Options

1) Fatty Acids

वसाऐलसि

2) Sugar

िकर

3) Amino Acids

एलमनह८ऐलसि

4) Starch

सटाचय Correct Answer Amino Acids

Q110 Which part of the brain is

responsible for triggering actions like

thinking intelligence memory and

ability to learn

मनसतषककाकह९नसाटहससासह८चनहॳ बनधदमानी याददाशतऔरसीिनहॳकीकषमताजहॴसीकियाओकह८परहॳररतकरताहहॴ 21-Jan-2017

Options

1) Diencephalon

िायएनसहॳफहॳ िह८न

2) Hypothalamus

हयपह८िहॳिहॳमस

3) Cerebrum

सहॳरहॳिम

4) Control

कटरह८ि

Correct Answer Cerebrum

Q111 Which of the following is also

known as the Biochemical Laboratory

of the Human Body

नननननलिखितमसहॳककसहॳमानविरीरकीजहॴवरसायनपरयह८गिािाभीकहाजाताहहॴ 21-Jan-2017

Options

1) Small Intestine

छह८टीआत

2)Brain

मनसतषक

3) Pancreas

अगनयािय

4) Liver

नजगर

Correct Answer Liver

The liver makes bile that will help

emulsify and digest the fats we eat

The liver takes toxic substances and

convert them using enzymes the liver

cells makes into a non toxic form so the

body can dispose of them

The liver also converts fats protein and

carbohydrates into glucose which is the

energy source for our cells to use

The liver takes amino acids and makes

proteins by combining them

Q112 The yellow colour of human urine

is due to

मानवमतरकापीिारग mdashndash कीवजहसहॳहह८ताहहॴ 22-

Jan-2017

Options

1) Bile Salts

F A C E B O O K

P A G E h t t p w w w f a c e b o o k c o m s s c m e n t o r s o f f i c i a l P a g e | 27

FOR MORE UPDATES AND MORE MATERIAL DO LIKE OUR FACEBOOK PAGE httpwwwfacebookcomsscmentorsofficial

पपततनमक

2) Cholesterol

कह८िहॳसटरह८ि

3) Lymph

लिनफ

4) Urochrome

यरह८िह८म

Correct Answer Urochrome

Urobilin or urochrome is the chemical

primarily responsible for the yellow

color of urine

Q113 The wilting of plants takes place

due to

पह९धह८कालिचििहह८नाकी mdashmdash- कीवजहसहॳहह८ताहहॴ 22-Jan-2017

Options

1)Photosynthesis

परकािसशिहॳषण

2) Transpiration

वाषपह८तसजयन

3) Absorption

अविह८षण

4) Respiration

शरवसन

Correct Answer Transpiration

Wilting is the loss of rigidity of non-

woody parts of plants This occurs when

the turgor pressure in non-lignified

plant cells falls towards zero as a result

of diminished water in the cells

Q114 Bovidae Ovis is the scientific name of

बह८पविीओपवस mdashndash कावहॴजञाननकनामहहॴ 22-Jan-2017

Options

1) Goat

बकरी 2) Cow

गाय

3) Buffalo

भहॳस

4) Sheep

भहॳड़

Correct Answer Sheep

Q115 Plants get their energy to produce

food from which of the following

पह८धहॳभह८जनकाननमायणकरनहॳकहॳ लिएनननननलिखितमसहॳककससहॳउजायपरापतकरतहॳहहॴ

22-Jan-2017

Options

1) Photosynthesis

परकािसशिहॳषण

2)Bacteria

बहॴकटीररया 3)Fungi

कवक

4)Sun

सयय Correct Answer Sun

Q116 Which of the following is secreted

by the liver

नननननलिखितमसहॳककसकासरावनजगरसहॳहह८ताहहॴ

22-Jan-2017

Options

1) Glucose

गिकह८ज

2) Iodine

आयह८िीन

3) Cortisol

काटटरयसह८ि

4) Bile

पपतत

Correct Answer Bile

The liver makes bile that will help

emulsify and

digest the fats we eat

F A C E B O O K

P A G E h t t p w w w f a c e b o o k c o m s s c m e n t o r s o f f i c i a l P a g e | 28

FOR MORE UPDATES AND MORE MATERIAL DO LIKE OUR FACEBOOK PAGE httpwwwfacebookcomsscmentorsofficial

Q117 Ferns belong to which division of

plants

फनसयपह९धह८कहॳ ककसभागमआतहॳहहॴ

22-Jan-2017

Options

1) Gymnosperms

नजननह८सपनसय 2) Angiosperms

एनजयह८सपनसय 3) Thallophyta

िहॴिह८फाईटा 4)Pteridophyta

टहॳररिह८फाईटा Correct Answer Pteridophyta

Q118 Who invented Antibiotics

एटीबायह८टटककाअपवषकारककसनहॳककयािा

22-Jan-2017

Options

1) Joseph Lister

जह८सहॳफलिसटर

2) William Harvey

पवलियमहाव

3) Robert Knock

रॉबटयनॉक

4)Alexander Fleming

अिहॳकज़िरफिहॳलमग

Correct Answer Alexander Fleming

Q119 Milbecycin is used in the

eradication of

लममबहॳसायलसनका mdashndash

मउनमिनमपरयह८गककयाजाताहहॴ 22-Jan-2017

Options

1) Agricultural Fungus

कपषकवक

2) Agricultural Pests

कपषकीटक

3) Agricultural Herbs

कपषिाक

4)Agricultural Weeds

कपषननराना Correct Answer Agricultural Pests

Milbemycin oxime is a veterinary drug

from the group of milbemycins used as

a broad spectrum antiparasitic It is

active against worms and mites(insects

Q120 Intestinal bacteria synthesizes

which of the following in the human

body

मानविरीरमआतोकहॳ बहॴकटीररयानननननलिखितमसहॳककसकासशिहॳषणकरतहॳहहॴ 22-Jan-2017

Options

1) Vitamin K

पवटालमन K

2) Proteins

परह८टीन

3) Fats

वसा 4) Vitamin D

पवटालमन D

Correct Answer Vitamin K

Q121 is the study of the physical form

and external structure of plants

mdashmdash-

मपह९धह८काभहॴनतक पऔरबाहरीसरचनाकाआदयाककयाजाताहहॴ 22-Jan-2017

Options

1) Physiology

कफनजयह८िह८जी 2) Anatomy

िरीररचनापवजञान

3) Phytomorphology

फाईटह८मह८फह८िह८जी 4)Cytology

कह८लिकापवजञान

Correct Answer Phytomorphology

F A C E B O O K

P A G E h t t p w w w f a c e b o o k c o m s s c m e n t o r s o f f i c i a l P a g e | 29

FOR MORE UPDATES AND MORE MATERIAL DO LIKE OUR FACEBOOK PAGE httpwwwfacebookcomsscmentorsofficial

Q122 Which of the following is a

structural and functional unit of

kidneys

नननननलिखितमसहॳकह९नसीगदोकीसरचनातमकऔरकाययकरीईकाईहहॴ

22-Jan-2017

Options

1) Renette Cells

रहॳनहॳटकह८लिकाए

2) Flame Cells

फिहॳमकह८लिकाए

3) Nephrites

नहॳफ़राइटस

4)Nephrons

नहॳफरोस

Correct Answer Nephrons

Nephron functional unit of the kidney

the structure that actually produces

urine in the process of removing waste

and excess substances from the blood

There are about 1000000 nephrons in

each human kidney

Q123 Which of the following is the

largest part of the human brain

नननननलिखितमसहॳकह९नसामानवमनसतषककासबसहॳबड़ाटहससाहहॴ

23-Jan-2017

Options

1) Ribs

पसलियाा 2) Cerebrum

सहॳरहॳिम

3) Pons

पोस

4)Thalamus

िहॴिहॳमस

Correct Answer Cerebrum

The cerebrum is the largest part of the

human brain making up about two-

thirds of the brainrsquos mass It has two

hemispheres each of which has four

lobes frontal parietal temporal and

occipital

Q124 The auxiliary buds

सहायककालियाmdashndash 23-Jan-2017

Options

1) grow endogenously from the pericycle

पहॳरीसाईककिसहॳअनतजातयपवकलसतहह८ताहहॴ 2) arise endogenously from the main

growing point

मिवपदसहॳअनतजातयउठताहहॴ 3) is an embryonic shoot located in the

axil of a leaf

एकभरणिटहहॴजह८एकपततीकहॳ अकषपरनसतिहह८ताहहॴ 4)arise exogenously from the epidermis

एपपिलमयससहॳबटहजातयतरीकहॳ सहॳउठताहहॴ Correct Answer is an embryonic shoot

located in the axil of a leaf

Q125 Which of the following is a viral

disease

इनमहॳसहॳकह९सीएकवायरिबीमारीहहॴ

23-Jan-2017

Options

1) Polio

पह८लियह८ 2) Tetanus

धनसतनभ

3) Leprosy

कषठरह८ग

4) Plague

पिहॳग

Correct Answer Polio

A viral disease (or viral infection)

occurs when an organismrsquos body is

invaded by pathogenic viruses and

infectious virus particles (virions) attach

to and enter susceptible cells

F A C E B O O K

P A G E h t t p w w w f a c e b o o k c o m s s c m e n t o r s o f f i c i a l P a g e | 30

FOR MORE UPDATES AND MORE MATERIAL DO LIKE OUR FACEBOOK PAGE httpwwwfacebookcomsscmentorsofficial

Poliomyelitis often called polio or

infantile paralysis is an infectious

disease caused by the poliovirus

Tetanusmdash A serious bacterial infection

that causes painful muscle spasms and

can lead to death

Leprosy also known as Hansenrsquos

disease (HD) is a long-term infection by

the bacterium Mycobacterium leprae or

Mycobacterium lepromatosis

Plague is an infectious disease caused by

the bacterium Yersinia pestis

Symptoms include fever weakness and

headache

Q126 Which organisms can help to

carry out Vermicomposting

कह९नसाजीववमीकनपह८नसटगममददकरताहहॴ

23-Jan-2017

Options

1) Nitrifying Bacteria

नाईटरीफाईगबहॴकटीररया 2) Earthworms

कहॴ चऐ

3) Algae

िहॴवि

4) Fungus

कवक

Correct Answer Earthworms

Q127 Contraction of heart is also

known as

हदयकहॳ सकचनकह८ mdash- भीकहाजाताहहॴ 23-Jan-

2017

Options

1) Systole

लससटह८ि

2) Aristotle

अरसत

3) Diastole

िायसटह८ि

4) Lub

मयब

Correct Answer Systole

Diastole is the part of the cardiac cycle

when the heart refills with blood

following systole (contraction)

Ventricular diastole is the period during

which the ventricles are filling and

relaxing while atrial diastole is the

period during which the atria are

relaxing

Q128 Azadirachta indica is the

botanical name of which of the

following

अजाटदराचताइडिकानननननलिखितमसहॳककसकावानसपनतनामहहॴ

23-Jan-2017

Options

1) Rose plant

गिाबकापह९धा 2) Apple tree

सहॳबकापहॳड़

3) Neem

नीम

4)Mango

आम

Correct Answer Neem

Q129 Which of the following is the

main end product of carbohydrate

digestion

नननननलिखितमसहॳकह९नसाकाबोहाइडरहॳटकहॳ पाचनकापरमिअतउतपादकहह८ताहहॴ 23-Jan-2017

Options

1) Fats

वसा 2) Lipids

लिपपडस

3) Glucose

गिकह८ज

4) Cellulose

F A C E B O O K

P A G E h t t p w w w f a c e b o o k c o m s s c m e n t o r s o f f i c i a l P a g e | 31

FOR MORE UPDATES AND MORE MATERIAL DO LIKE OUR FACEBOOK PAGE httpwwwfacebookcomsscmentorsofficial

सहॳमयिह८ज

Correct Answer Glucose

Intestinal absorption of end products

from digestion of carbohydrates and

proteins in the pig hellip During absorption some sugars (fructose or

galactose) released from the

corresponding sucrose and lactose

respectively during digestion were

partly metabolized into glucose by the

enterocyte

Q130 Which of the following glands is a

source of the enzyme Ptyalin

नननननलिखितगरचियोमसहॳएजाइमटयालिनकासरह८तहहॴ 23-Jan-2017

Options

1) Pancreas

अगरािय

2) Thyroid Gland

िाइराइिगरिी 3) Pituitary Gland

पीयषगरिी 4) Salivary Glands

िारगरचियाा Correct Answer Salivary Glands

Q131 Which of the following is not true

about Pteridophyta

ननननमसहॳकह९नसीबातटहॳररिह८फाईटकहॳ बारहॳमसचनहीहहॴ 23-Jan-2017

Options

1) Dominant phase is saprophytes

परमिचरणसहॳपरह८फाईइटसहह८ताहहॴ 2) Main plant body is diploid

पह९दह८कामखयिरीरदपवगखणतहह८ताहहॴ 3) Seeds are present

बीजमह९जदहह८तहॳहहॴ 4)Flowers are absent

फिअनपनसतिहह८तहॳहहॴ

Correct Answer Seeds are present

Q132 The largest dolphin species is the

orca also called as

िॉिकफनकीसबसहॳबड़ीपरजानतकाकानामआकायहहॴनजसहॳ mdash- भीकहतहॳहहॴ 23-Jan-2017

Options

1) Bottle Nose

बाटिनह८ज

2) Baiji

बहॳजी 3) Killer whale

ककिरहहॳि

4)Tucuxi

टकवसी Correct Answer Killer whale

Q133 The fat digesting enzyme Lipase

is secreted by which of the following

वसाकापाचनकरनहॳवािाएजाइमिाइपहॳजनननननलिखितमसहॳककसकहॳ दवारासतरापवतहह८ताहहॴ

24-Jan-2017

Options

1) Kidneys

गद

2) Pancreas

अगनयािय

3) Large Intestine

बड़ीआत

4)Liver

नजगर

Correct Answer Pancreas

Lipase is an enzyme that splits fats so

the intestines can absorb them Lipase

hydrolyzes fats like triglycerides into

their component fatty acid and glycerol

molecules It is found in the blood

gastric juices pancreatic secretions

intestinal juices and adipose tissues

F A C E B O O K

P A G E h t t p w w w f a c e b o o k c o m s s c m e n t o r s o f f i c i a l P a g e | 32

FOR MORE UPDATES AND MORE MATERIAL DO LIKE OUR FACEBOOK PAGE httpwwwfacebookcomsscmentorsofficial

Q134 The arrangement of leaves on an

axis or stem is called

एकअकषयातनहॳपरपनततयोकीयवसिाकह८कयाकहाजाताहहॴ SSC CHSL Science (biology) 2016

Question Paper

24-Jan-2017

Options

1) Phyllotaxy

फाइिह८टहॴकसी 2) Vernation

वनिन

3) Venation

वहॳनहॳिन

4)Phytotaxy

फाइटह८टहॴकसी Correct Answer Phyllotaxy

In botany phyllotaxis or phyllotaxy is

the arrangement of leaves on a plant

stem (from Ancient Greek phyacutellon

ldquoleafrdquo and taacutexis ldquoarrangementrdquo)

Phyllotactic spirals form a distinctive

class of patterns in nature

Q135 The study of Cells is also known

as

कह८लिकाओकहॳ अधययनकह८ mdashmdashndash

भीकहाजाताहहॴ 24-Jan-2017

Options

1) Cytology

सायटह८िह८जी 2) Physiology

कफनजयह८िह८जी 3) Nucleology

नयककमयह८िह८जी 4)Cellology

सहॳिह८िह८जी Correct Answer Cytology

Q136 Which of the following scientists

is also known as the Father of Biology

नननननलिखितमसहॳककसवहॴजञाननककह८ ldquoजीवपवजञानकहॳ जनकrdquoकहॳ नामसहॳभीजानाजाताहहॴ 24-Jan-2017

Options

1) Herbert Spencer

हबयटयसपसर

2) Aristotle

अरसत 3) Lamarck

िहॳमाकय 4)Darwin

िापवयन

Correct Answer Aristotle

Q137 Which cells give rise to various

organs of the plant and keep the plant

growing

कह९नसीकह८लिकाएपह९धह८कहॳ लभननअगह८कह८जनमदहॳतीहहॴऔरपह९धह८कह८बढ़नहॳममददकरतीहहॴ

24-Jan-2017

Options

1) Permanent

सिायी 2) Dermal

तवचीय

3) Meristematic

मररसटहॳमटटक

4)Mature

परह८ढ़

Correct Answer Meristematic

A meristem is the tissue in most plants

containing undifferentiated cells

(meristematic cells) found in zones of

the plant where growth can take place

Q138 Rodentia Muridae is the scientific

name of

F A C E B O O K

P A G E h t t p w w w f a c e b o o k c o m s s c m e n t o r s o f f i c i a l P a g e | 33

FOR MORE UPDATES AND MORE MATERIAL DO LIKE OUR FACEBOOK PAGE httpwwwfacebookcomsscmentorsofficial

रह८िहॳलियानयररिी mdashmdash- कावहॴजञाननकनामहहॴ 24-

Jan-2017

Options

1) Mouse

चहा 2) Squirrel

चगिहरी 3) Monkey

बदर

4) Lizard

नछपकिी Correct Answer Mouse

Q139 Name the scientist who proposed

the cell theory

कह८लिकालसदातकापरसतावदहॳनहॳवािहॳवहॴजञाननककानामबताइए 24-Jan-2017

Options

1) Schleiden and Schwann

िीमिनऔरशरववान

2) Lamarck

िहॳमाकय 3) Treviranus

टरहॳवायरहॳनस

4)Whittaker and Stanley

हीटकरऔरसटहॳनिहॳ Correct Answer Schleiden and

Schwann

Q140 The flower with the worldrsquos

largest bloom is

दननयाकासबसहॳबड़ाफिखििनहॳवािा mdashmdashndash हहॴ 24-Jan-2017

Options

1) Pando

पािह८ 2) Posidonia

पह८सीिह८ननया 3) Rafflesia arnoldii

ररफिहॳलियाअनोमिी 4)Helianthus annuus

हहॳलिएनिसएनयअस

Correct Answer Rafflesia arnoldii

Rafflesia arnoldii is a species of

flowering plant in the parasitic genus

Rafflesia It is noted for producing the

largest individual flower on earth It has

a very strong and horrible odour of

decaying flesh earning it the nickname

ldquocorpse flower

Q141 Deficiency of which vitamin

causes night blindness

ककसपवटालमनकीकमीकहॳ कारणरतौधीहह८ताहहॴ 24-Jan-2017

Options

1) Vitamin K

पवटालमन K

2) Vitamin C

पवटालमन C

3) Vitamin B1

पवटालमन B1

4)Vitamin A

पवटालमन A

Correct Answer Vitamin A

Q142 Nongreen plants lack which of the

following

गहॴर-

हररतवनसपनतमनननननलिखितमसहॳककसकीकमीहह८तीहहॴ

24-Jan-2017

Options

1) Chlorophyll

किह८रह८कफि

2) Lycophyll

िायकह८कफि

3) Cyanophyll

F A C E B O O K

P A G E h t t p w w w f a c e b o o k c o m s s c m e n t o r s o f f i c i a l P a g e | 34

FOR MORE UPDATES AND MORE MATERIAL DO LIKE OUR FACEBOOK PAGE httpwwwfacebookcomsscmentorsofficial

सायनह८कफि

4)Phototropism

फह८टह८टरोपपजम

Correct Answer Chlorophyll

Q143 Organisms that use light to

prepare food are known as

जह८जीवपरकािकाउपयह८गकरभह८जनतहॴयारकरतहॳहहॴ उनह mdashmdash- कहॳ पमजानजाताहहॴ 24-Jan-2017

Options

1) Autotrophs

सवपह८षी 2) Heterotrophs

पवषमपह८षज

3) Omnivores

सवायहारी 4)Decomposers

पवघटनकरनहॳवािा Correct Answer Autotrophs

autotrophs often make their own food

by using sunlight carbon dioxide and

water to form sugars which they can use

for energy Some examples of

autotrophs include plants algae and

even some bacteria Autotrophs

(producer) are important because they

are a food source for heterotrophs

(consumers)

A heterotroph is an organism that

ingests or absorbs organic carbon

(rather than fix carbon from inorganic

sources such as carbon dioxide) in order

to be able to produce energy and

synthesize compounds to maintain its

life Ninety-five percent or more of all

types of living organisms are

heterotrophic including all animals and

fungi and some bacteria

Q144 Which of the following is a

primary function of haemoglobin

नननननलिखितमसहॳकह९नसाटहमह८गिह८बबनकाएकपरािलमककाययहहॴ

25-Jan-2017

Options

1) Utilization of energy

उजायकाउपयह८गकरना 2) Prevention of anaemia

रकतामपताहह८नहॳसहॳरह८कना 3) Destruction of bacteria

बहॴकटीररयाकापवनािकरना 4) To transport oxygen

ऑकसीजनकावहनकरना Correct Answer To transport oxygen

Q145 Vascular bundles are absent in

सवहनीबिि mdashmdash- मअनपनसतिरहतहॳहहॴ 25-Jan-2017

Options

1) Bryophyta

िायह८फाइटा 2) Pteridophyta

टहॳररिह८फाईटा 3) Gymnosperms

नजननह८सपमय 4) Angiosperms

एननजयह८सपहॳनसय Correct Answer Bryophyta

Q146 Sauria Lacertidae is the scientific

name of

सहॴररयािहॳसरटाईिी mdashmdashndash कावहॴजञाननकनामहहॴ 25-Jan-2017

Options

1) Crocodile

मगरमचछ

2) Hippopotamus

टहपपह८पह८टहॳमस

3) Lizard

नछपकिी 4) House fly

F A C E B O O K

P A G E h t t p w w w f a c e b o o k c o m s s c m e n t o r s o f f i c i a l P a g e | 35

FOR MORE UPDATES AND MORE MATERIAL DO LIKE OUR FACEBOOK PAGE httpwwwfacebookcomsscmentorsofficial

घरहॳिमकिी Correct Answer Lizard

Q147 Which type of pathogen causes

the water-borne disease SARS (Severe

Acute Respiratory Syndrome)

ककसपरकािकारह८गज़नकजिजननतबीमारीसासयकाकारणबनताहहॴ 25-Jan-2017

Options

1) Viral

वायरि

2) Parasitic

परजीवी 3) Protozoan

परह८टह८जअन

4) Bacterial

बहॴकटीररयि

Correct Answer Viral

Q148 Which of the following organs

produces the enzyme lipase

नननननलिखितमसहॳकह९नसाअगिायपहॳजएजाइमउतपननकरताहहॴ 25-Jan-2017

Options

1) Pancreas

अगनयािय

2) Large Intestine

बड़ीआत

3) Liver

नजगर

4) Small Intestine

छह८टीआत

Correct Answer Pancreas

Q149 A is a long internode forming the

basal part or the whole of a peduncle

एक mdashmdash- एकिबाइटरनह८िहहॴ जह८ननचिाटहससायासनपणयिठिबनताहहॴ 25-

Jan-2017

Options

1) Rhizome

परकद

2) Rachis

महॳ दि

3) floral axis

पषपअकष

4) Scape

भगदड़

Correct Answer scape

Q150 ndash Which of the following

organisms are considered to be both

Living and Non-living

नननननलिखितमसहॳकह९नसहॳजीवाणकह८जीपवतऔरअजीपवतमानाजाताहहॴ

25-Jan-2017

Options

1) Bacteria

बहॴकटीररया 2) Fungi

कवक

3) Algae

िहॴवाि

4)Virus

वायरस

Correct Answer Virus

They are considered to be living as they

possess a protein coat as a protective

covering DNA as the genetic material

etc

They are said to be non-living as they

can be crystallised and they survive for

billions of years They can tolerate high

temperatures freezing cold

temperatures ultra-violet radiations etc

Q151 Deficiency of fluorine causes

which of the following

फिह८ररनकीकमीकहॳ कारणनननननलिखितमसहॳकयाहह८ताहहॴ

F A C E B O O K

P A G E h t t p w w w f a c e b o o k c o m s s c m e n t o r s o f f i c i a l P a g e | 36

FOR MORE UPDATES AND MORE MATERIAL DO LIKE OUR FACEBOOK PAGE httpwwwfacebookcomsscmentorsofficial

27-Jan-2017

Options

1) Dental Caries

िटिकहॴ ररज

2) Scurvy

सकवरी 3) Anaemia

रकतामपता 4) Arthritis

गटठया Correct Answer Dental Caries

Q152 In a Punnett Square with the

cross AaBb x AaBb how many Aabb

genotypes would be created

पनहॳटसककायरमिह८स AaBb x AaBb कहॳ साि

ककतनहॳ Aabb जीनह८टाइपबनगहॳ 27-Jan-2017

Options

1) 1

2) 8

3) 2

4) 3

Correct Answer 2

Q153 Which of the following is the

Controlling Center of the Cell

नननननलिखित म सहॳ कह८लिकाका ननयतरण

क दर कह९न हहॴ

27-Jan-2017

Options

1) Nucleus

क दर

2) Plasma

पिाजमा 3) Lysosome

िायसह८सह८म

4) Chromosome

िह८मह८सह८म

Correct Answer Nucleus

The control centre of the cell is the

nucleus in eukaryotic cells The nucleus

contains genetic material in the form of

DNA

Q154 Myopia affects which of the

following organs

मायह८पपयानननननलिखितअगह८मसहॳककसहॳपरभापवतकरताहहॴ

25-Jan-2017

Options

1) Heart

हदय

2) Skin

तवचा 3) Eyes

आािहॳ 4)Mouth

मह

Correct Answer Eyes

Q155 Which of the following bears

flowers

नननननलिखितमसहॳकह९नफिधारणकरताहहॴ

25-Jan-2017

Options

1) Bryophyta

िायह८फाइटा 2) Pteridophyta

टहॳरीिह८फाईटा 3) Gymnosperms

नजननह८सपमय 4)Angiosperms

एननजयह८सपमय Correct Answer Angiosperms

Q156 Oxygenated blood flows out of the

heart through the

ऑकसीजनयकतरकत mdashmdashmdash

कहॳ माधयमसहॳहदयकहॳ बाहरबहताहहॴ 25-Jan-2017

F A C E B O O K

P A G E h t t p w w w f a c e b o o k c o m s s c m e n t o r s o f f i c i a l P a g e | 37

FOR MORE UPDATES AND MORE MATERIAL DO LIKE OUR FACEBOOK PAGE httpwwwfacebookcomsscmentorsofficial

Options

1) Aorta

महाधमनी 2) pulmonary artery

फहॳ फड़हॳकीधमनी 3) vena cava

वहॳनाकावा 4)Atrium

चह९क

Correct Answer aorta

Q157 Blood leaving the liver and

moving towards the

heart has a higher concentration of

नजगरसहॳननकिकरहदयकीतरफजानहॳवािहॳरकतम mdashmdashmdashmdash कीउचचसादरताहह८तीहहॴ 27-Jan-2017

Options

1) Lipids

लिपपडस

2) Urea

यररया 3) Bile Pigments

पपततकहॳ रगकरण

4) Carbon dioxide

काबयनिायऑकसाइि

Correct Answer Bile Pigments

Urea is nitrogen containing substance

which is produced in the liver in order

to deal with excess amino-acids in the

body As urea is produced it leaves the

liver in the blood stream and passes via

the circulatory system to all parts of the

body

Q158 Bulb is a modification of which

part of a plant

बमबएकपह९धहॳकहॳ ककसटहससहॳकाएक पातरणहह८ताहहॴ 27-Jan-2017

Options

1) The root

जड़

2) The stem

तना 3) The radicle

मिाकर

4)The fruit

फि

Correct Answer The stem

Q159 Which of the following carries

blood away from the heart to different

body parts

इनमहॳसहॳकह९नरकतकह८हदयसहॳिरीरकहॳ पवलभननअगह८तकिहॳजातीहहॴ

27-Jan-2017

Options

1) Arteries

धमननया 2) Nerves

तबतरहाए

3) Capillaries

कहॳ लिकाए

4)Veins

नसहॳ Correct Answer Arteries

Q160 The series of processes by which

nitrogen and its compounds are

interconverted in the environment and

in living organisms is called

27-Jan-2017

Options

1)Absorption of Nitrogen

2)Ammonification

3)Nitrogen Fixation

4)Nitrogen Cycle

Correct Answer Nitrogen Cycle

Ammonification or Mineralization is

performed by bacteria to convert

organic nitrogen to ammonia

F A C E B O O K

P A G E h t t p w w w f a c e b o o k c o m s s c m e n t o r s o f f i c i a l P a g e | 38

FOR MORE UPDATES AND MORE MATERIAL DO LIKE OUR FACEBOOK PAGE httpwwwfacebookcomsscmentorsofficial

Nitrification can then occur to convert

the ammonium to nitrite and nitrate

Nitrogen fixation is a process by which

nitrogen in the Earthrsquos atmosphere is

converted into ammonia (NH3) or other

molecules available to living organisms

Q161 BCG vaccine is given to protect

from which of the following

बीसीजीकाटटकानननननलिखितमसहॳककसकहॳ बचावकहॳ लिएटदयाजातहहॴ

27-Jan-2017

Options

1) Jaundice

पीलिया 2) Anaemia

रकतमपता 3) Tuberculosis

कषयरह८ग

4) Polio

पह८लियह८ Correct Answer Tuberculosis

Q162 Parallel venation is found in

समानतरवहॳनहॳिन mdashmdashmdash- मपायाजाताहहॴ 27-Jan-2017

Options

1) plants which are monocots

पह९धहॳजह८एकबीजपतरीहह८तहॳहहॴ 2) plants which have a dicot stem

वहॳपह९धहॳनजनकातनादपवदलियहह८ताहहॴ 3) plants with leaves similar to Tulsi

वहॳपह९धहॳनजनकीपनततयतिसीकीपनततयोकहॳ समानहह८तहॳहहॴ 4)plants with tap roots

टहॳप टवािहॳपह९धहॳ Correct Answer plants which are

monocots

Q163 The hardest part of the body is

िरीरकासबसहॳकठह८रभाग mdashndash हहॴ 27-Jan-2017

Options

1) Bones

हडडिय

2) Tooth Enamel

दातकहॳ इनहॳमि

3) Skull

िह८पड़ी 4) Spinal Cord

महॳ रजज

Correct Answer Tooth Enamel

Q164 Which type of pathogen causes

the waterborne disease E coli Infection

ककसपरकारकारह८गजननकजिजननतरह८गईकह८िाईसिमणकाकारणबनताहहॴ 27-Jan-2017

Options

1) Protozoan

परह८टह८जआ

2) Parasitic

परजीवी 3) Bacterial

बहॴकटीररयि

4)Viral

वायरि

Correct Answer Bacterial

Q165 The amount of blood filtered

together by both the kidneys in a 70 kg

adult male human in a minute is

70 की गरा वािहॳएकवयसकप षमएकलमनटमदह८नोगदकहॳदवाराएकसािचाबनीगयीरकतकीमातरहह८तीहहॴ 29-Jan-2017

Options

1) 1100 ml

1100 लमलि

2) 100 ml

F A C E B O O K

P A G E h t t p w w w f a c e b o o k c o m s s c m e n t o r s o f f i c i a l P a g e | 39

FOR MORE UPDATES AND MORE MATERIAL DO LIKE OUR FACEBOOK PAGE httpwwwfacebookcomsscmentorsofficial

100 लमलि

3) 1500 ml

1500 लमलि

4) 500 ml

500 लमलि

Correct Answer 1100 ml

Q166 Which feature of a plant helps to

distinguish a monocot from a dicot

पह९धहॳकीवहकह९नसीपविहॳषताहहॴजह८एकदपवदलियहॳऔरएकएकदिीयपह९धहॳसहॳभहॳदकरनहॳममददकरतीहहॴ 29-Jan-2017

Options

1) Pollination

परागम

2) Venation

वहॳनहॳिन

3) Vernation

वनिन

4) Aestivation

एसटीवहॳिहॳन

Correct Answer venation

Q167 The Mutation Theory was

proposed by

उतवररवतयनकालसदात mdashmdashndash

कहॳ दवरापरसतापवतककयाजाताहहॴ 29-Jan-2017

Options

1) Charles Lyell

चामसयलियहॳि

2) William Smith

पवलियमनसमि

3) Hugo De Vries

हयगह८िीराईस

4)Harrison Schmitt

हहॳरीसननसमट

Correct Answer Hugo De Vries

Q168 Which type of pathogen causes

the waterborne disease HepatitisA

ककसपरकारकहॳ रह८गजनकजिजननतरह८गहहॳपहॳटाइटटस-A काकारणबनताहहॴ

29-Jan-2017

Options

1) Parasitic

परजीवी 2) Viral

वायरि

3) Protozoan

परह८टह८जआ

4) Bacterial

बहॴकटीररयि

Correct Answer Viral

Q169 In a Punnett Square with the

cross AaBb x Aabb how many AaBb

genotypes would be created

पनहॳटसकवायरमिह८स AaBb x Aabb

कहॳ सािककतनहॳ AaBb जीनह८टाइपबनगहॳ 29-Jan-

2017

Options

1) 4

2) 1

3) 7

4) 6

Correct Answer 4

Q170 Arboreal Ateles is the scientific

name of

अिह८ररयिएटटलिस mdashmdashmdash कावहॴजञाननकनामहहॴ 29-Jan-2017

Options

1) Squirrel

चगिहरी 2) Sparrow

गह८रहॴया 3) Lizard

नछपकिी 4) Spider monkey

F A C E B O O K

P A G E h t t p w w w f a c e b o o k c o m s s c m e n t o r s o f f i c i a l P a g e | 40

FOR MORE UPDATES AND MORE MATERIAL DO LIKE OUR FACEBOOK PAGE httpwwwfacebookcomsscmentorsofficial

मकड़ीबदर

Correct Answer Spider monkey

Q171 Which type of pathogen causes

the waterborne disease Salmonellosis

ककसपरकारकारह८गाणजिजननतबीमारीसािमह८नहॳिह८लसज़काकारकहहॴ

29-Jan-2017

Options

1) Algal

िहॳवालियहॳ 2) Parasitic

परजीवी 3) Bacterial

बहॴकटीररयि

4)Viral

वायरि

Correct Answer Bacterial

An infection with salmonella bacteria

commonly caused by contaminated food

or water

Symptoms include diarrhoea fever

chills and abdominal pain

Q172 is a condition in which there is a

deficiency of red cells or of haemoglobin

in the blood

mdashmdash-

एकनसिनतहहॴनजसमहॳरकतमिािकह८लिकाओकीयाहीमह८गिह८बबनकीकमीहह८तीहहॴ 29-Jan-2017

Options

1) Albinism

एनमबननजम

2) Propyria

परह८पीररया 3) Anaemia

एनीलमया 4)Keloid disorder

कहॳ िह८इिडिसओिर

Correct Answer Anaemia

Q173 Ananas comosus is the scientific

name of

Options

अनानासकह८मह८सस mdashmdashmdashndash

कावहॴजञाननकनामहहॴ 29-Jan-2017

1) Custard Apple

सीताफि

2) Pineapple

पाइनएपपि

3) Bamboo

बास

4)Pomegranate

अनार

Correct Answer Pineapple

Q174 Which organ produces insulin

कह९नसाअगइनसलिनपहॴदाकरताहहॴ 29-Jan-

2017

Options

1) Liver

यकत

2) Thyroid gland

िायराइिगरिी 3) Spleen

पिीहा 4)Pancreas

अगरयिय

Correct Answer Pancreas

Q175 Which of the following disease is

not caused by water pollution

नननननलिखितमसहॳकह९नसारह८गपानीकहॳ परदषणकहॳकारणनहीहह८ता

29-Jan-2017

Options

1) Cholera

हहॴजा 2) Typhoid

F A C E B O O K

P A G E h t t p w w w f a c e b o o k c o m s s c m e n t o r s o f f i c i a l P a g e | 41

FOR MORE UPDATES AND MORE MATERIAL DO LIKE OUR FACEBOOK PAGE httpwwwfacebookcomsscmentorsofficial

टाइफाइि

3) Asthma

दमा 4)Diarrhoea

दसत

Correct Answer Asthma

Q176 Ocimum tenuiflorum is the

scientific name of

ओलिलममटहॳयईफिह८रमइसकावहॴजञाननकनाम mdash

ndash हहॴ 30-Jan-2017

Options

1) Neem

नीम

2) Mango

आम

3) Babul

बबि

4)Tulsi

तिसी Correct Answer Tulsi

Q177 Which gland secretes bile a

digestive fluid

कह९नसीगरिीपपतत एकपाचनतरिपरदािय सरापवतकरतीहहॴ 30-Jan-2017

Options

1) Pancreas

अगनयािय

2) Liver

यकत

3) Thyroid

िायराइि

4) Testes

टहॳनसटस

Correct Answer liver

Q178 In which of the following the

dominant phase is Gametophyte

नननननलिखितमसहॳककसकहॳ परमिचरणयगमकह८दपवधद (Gametophyte)हहॴ 30-Jan-2017

Options

1) Bryophyta

िायह८फाइटा 2) Pteridophyta

टहॳररिह८फाइटा 3) Gymnosperms

नजननह८सपमय 4) Angiosperms

एननजयह८सपमय Correct Answer Bryophyta

Q179 Anaerobic respiration refers to

which of the following

नननननलिखितमसहॳककसहॳअवायवीयशवसनकहाजाताहहॴ

30-Jan-2017

Options

1) Respiration without Oxygen

ऑकसीजनकहॳ बबनाशवसन

2) Respiration with Oxygen

ऑकसीजनकहॳ सािशवसन

3) Respiration without CO2

काबयनिायऑकसाइिकहॳ बबनाशवसन

4) Respiration with CO2

काबयनिायऑकसाइिकहॳ सािशविन

Correct Answer Respiration without

Oxygen

Q180 Which type of pathogen causes

the waterborne disease Cholera

ककसपरकारकारह८गजनकजिजननतरह८गहहॴजाकाकारणबनताहहॴ

30-Jan-2017

Options

1) Algal

िहॴवालियहॳ

F A C E B O O K

P A G E h t t p w w w f a c e b o o k c o m s s c m e n t o r s o f f i c i a l P a g e | 42

FOR MORE UPDATES AND MORE MATERIAL DO LIKE OUR FACEBOOK PAGE httpwwwfacebookcomsscmentorsofficial

2) Bacterial

बहॴकटीररयि

3) Protozoan

परह८टह८जआ

4) Viral

वायरि

Correct Answer Bacterial

Q181 To which class does

Oxyreductases transferases hydrolases

belong

ओकसीररिकटहॳसटरासफरहॳजहॳस

हाइडरह८िहॳसहॳसककसवगयमआतहॳहहॴ 30-Jan-2017

Options

1) Hormones

हारमोस

2) Enzymes

एजाइनस

3) Proteins

परह८टीनस

4) Vitamins

पवटालमनस

Correct Answer Enzymes

Q182 Which of the following is not true

about Gymnosperms

ननननमसहॳकह९नसीबातअनावतबीजीकहॳ बारहॳमसचनहीहहॴ 30-Jan-2017

Options

1) Dominant phase is saprophytes

परमिचरणसहॳपरह८फाइटसहह८ताहहॴ 2) Vascular bundles are absent

सवहनीबििअनपनसितहह८ताहहॴ 3) spores are heterospores

बीजाणहहॳटहॳरह८सपह८रसहह८तहॳहहॴ 4) Flowers are absent

फिअनपनसितहह८तहॳहहॴ

Correct Answer Vascular bundles are

absent

Q183 The name of first mammal clone sheep is

भहॳड़कीपरिमसतनपायीपरनत प (किह८न)

कानामहहॴ 30-Jan-2017

Options

1) Noori

नरी 2) Dolly

िॉिी 3) Louise

िसी 4)Durga

दगाय Correct Answer Dolly

Q184 Which type of pathogen causes

the water-borne disease Typhoid fever

ककसपरकारकारह८गजनकजिजननतरह८गटाइफाइिबिारकाकारणबनताहहॴ 30-Jan-2017

Options

1) Algal

िहॴवािीय

2) Parasitic

परजीवी 3) Protozoan

परह८टह८जनअन

4)Bacterial

बहॴकटीररयि

Correct Answer Bacterial

Q185 In which part of the cell are

proteins made

कह८लिकाकहॳ ककसटहससहॳमपरह८टीनबनायाजाताहहॴ

31-Jan-2017

Options

1) Reticulum

रहॳटटकिम

F A C E B O O K

P A G E h t t p w w w f a c e b o o k c o m s s c m e n t o r s o f f i c i a l P a g e | 43

FOR MORE UPDATES AND MORE MATERIAL DO LIKE OUR FACEBOOK PAGE httpwwwfacebookcomsscmentorsofficial

2) Golgi apparatus

गह८मजीएपहॳरहॳटस

3) Ribosomes

ररबह८सह८नस

4) Lysosome

िायसह८सह८नस

Correct Answer ribosomes

Proteins are produced by stringing

amino acids together in the order

specified by messenger RNA strands

that were transcribed from DNA in the

cell nucleus The process of synthesizing

a protein is called translation and it

occurs on ribosomes in the cytoplasm of

a cell

Q186 Polio is a disease caused by which

of the following

नननननलिखितमसहॳपह८लियह८कीबबमारह८हह८नहॳकाकारणकयाहहॴ

31-Jan-2017

Options

1) Bacteria

बहॴकटीररयि

2) Mosquito

मचछर

3) Virus

वायरस

4) Cockroach

नतिच हॳ Correct Answer Virus

Polio or poliomyelitis is a crippling and

potentially deadly infectious disease It

is caused by the poliovirus

Q187 ndash Hay fever is a sign of which of

the following

हहॳकफवरनननननलिखितमसहॳककसकाएकसकहॳ तहहॴ

31-Jan-2017

Options

1) Old Age

वदावसिा 2) Malnutrition

कपह८सण

3) Allergy

एिनजय 4) Over Work

अतयचधककाययकरना Correct Answer Allergy

Q188 How many chromosomes does a

human cell contain

एकमानवकह८लिकामककतनहॳगणसतरहह८तहॳहहॴ

29-Jan-2017

Options

1) 6

2) 26

3) 46

4) 66

Correct Answer 46

In humans each cell normally contains

23 pairs of chromosomes for a total of

46 Twenty-two of these pairs called

autosomes look the same in both males

and females The 23rd pair the sex

chromosomes differ between males and

females

Q189 Which of the following is not true

about Bryophyta

ननननमसहॳकह९नसीबातिायह८फाइटकहॳ बारहॳमसचनहीहहॴ 31-Jan-2017

Options

1) Dominant phase is gametophytes

परमिचरणगहॳलमतह८फाइटसहह८ताहहॴ 2) Main plant body is haploid

पह९धहॳकामखयिरीरअगखणतहह८ताहहॴ 3) Spores are homospores

बीजाणहह८मह८सफह८रसहह८तहॳहहॴ 4) Flowers are present

फिमह८जदहह८तहॳहहॴ Correct Answer Flowers are present

F A C E B O O K

P A G E h t t p w w w f a c e b o o k c o m s s c m e n t o r s o f f i c i a l P a g e | 44

FOR MORE UPDATES AND MORE MATERIAL DO LIKE OUR FACEBOOK PAGE httpwwwfacebookcomsscmentorsofficial

Q190 Which aquatic animal has

trailing tentacles

ककसजिीयजानवरकहॳ पीछहॳचिनहॳवािहॳटहॳटकिसहह८तहॳहहॴ

31-Jan-2017

Options

1) Sea horse

समदरीघह८िा 2) Corals

मगा 3) Jelly fish

जहॳिीमछिी 4) Star fish

तारामछिी Correct Answer Jelly fish

Jellyfish with its umbrella-shaped bell

and trailing tentacles

Q191 Which type of pathogen causes

the water-borne disease Poliomyelitis

(Polio)

ककसपरकारकारह८गजनकजिजननतरह८गपह८लियह८मायहॳटटस (पह८लियह८) काकारणहहॴ 31-Jan-

2017

Options

1) Parasitic

परजीवी 2) Algal

िहॴवालिय

3) Viral

वायरि

4) Bacterial

बहॴकटीररयि

Correct Answer Viral

Q192 The outer white part of the eye

that protects the inner structures is

आािकाबाहरीसफहॳ दटहससाजह८आतररकसरचनाओकीरकषाकरताहहॴ वह mdashmdashmdash हहॴ 31-Jan-

2017

Options

1) Iris

आयररस

2) Sclera

सकिहॳरा 3) Retina

रहॳटटना 4) Cornea

कह८ननयया Correct Answer Sclera

Q193 Proteins are made up of

परह८टीनकाननमायण mdashndash सहॳहह८ताहहॴ 31-Jan-2017

Options

1) Amino acids

एलमनह८अनि

2) Fatty acids

वसायकतअनि

3) Glucose

गिकह८ज

4)Nucleotides

नयनकियह८टाईिस

Correct Answer Amino acids

Q194 Moringa Oleifera is the scientific

name of

मह८ररगओलिफहॳ रा mdashmdashndash कावहॴजञाननकनामहहॴ 31-Jan-2017

Options

1) Banyan

बरगद

2) Gulmohar

गिमह८हर

3) Amla

आमिा

F A C E B O O K

P A G E h t t p w w w f a c e b o o k c o m s s c m e n t o r s o f f i c i a l P a g e | 45

FOR MORE UPDATES AND MORE MATERIAL DO LIKE OUR FACEBOOK PAGE httpwwwfacebookcomsscmentorsofficial

4) Drumstick

डरमनसटक

Correct Answer Drumstick

Q195 Kidney stones are composed of

गदकीपिरी mdashndash सहॳबनीहह८तीहहॴ 1-Feb-2017

Options

1) Calcium Oxalate

कहॴ नमसयमओकजहॳिहॳट

2) Sodium Chloride

सह८डियमकिह८राइि

3) Magnesium Nitrate

महॳनगनलियमनाइतटरहॳट

4) Calcium Bicarbonate

कहॴ नमियमबायकबोनहॳट

Correct Answer Calcium Oxalate

Q196 ndash Which of the following is not

true about Angiosperms

ननननमसहॳकह९नसीबातआवतबीजीकहॳ बारहॳमसचनहीहहॴ 1-Feb-2017

Options

1) Dominant phase is gametophytes

परमिचरणगहॳलमतह८फाइटहह८ताहहॴ 2) Vascular bundles are present

सवहनीबििमह९जदहह८ताहहॴ 3) Spores are heterospores

बीजाणहहॳटहॳरह८सपह८रसहह८तहॳहहॴ 4) Seeds are covered

बीजढकहॳ हह८तहॳहहॴ Correct Answer Dominant phase is

gametophytes

Q197 All of the following are excretory

(waste) products of animals except

नननननलिखितमसहॳककसएककह८छह८ड़करअनयसभीपराखणयोदवाराउतसनजयतपदाियहहॴ 1-Feb-

2017

Options

1) Uric Acid

यररकएलसि

2) Ammonia

अमह८ननया 3) Carbohydrates

काबोहाइडरहॳट

4) Urea

यररया Correct Answer Carbohydrates

In animals the main excretory products

are carbon dioxide ammonia (in

ammoniotelics) urea (in ureotelics) uric

acid (in uricotelics) guanine (in

Arachnida) and creatine

Q198 RNA is a polymeric molecule

What does RNA stand for

आरएनइएएकबहिकआणहहॴ इसकाकापवय पकयाहहॴ 1-Feb-2017

Options

1) Rado Nuclear Acid

रािह८नयनकियरएलसि

2) Ribo Nucleic Acid

राइबह८नयनकिकएलसि

3) Rhino Nuclear Acid

हाइनह८नयनकियरएलसि

4) Resto Nucleus Acid

रहॳसटह८नयकिीयसएलसि

Correct Answer Ribo Nucleic Acid

Q199 Which organ does detoxification

and produces chemicals needed for

digestion

कह९नसाअगपवषहरणकरताहहॴऔरपाचनकहॳ लिएआवशयकरसायनोकह८पहॴदाकरताहहॴ 1-Feb-

2017

Options

1) Salivary glands

िारगरचिया 2) Pancreas

अगनयािय

F A C E B O O K

P A G E h t t p w w w f a c e b o o k c o m s s c m e n t o r s o f f i c i a l P a g e | 46

FOR MORE UPDATES AND MORE MATERIAL DO LIKE OUR FACEBOOK PAGE httpwwwfacebookcomsscmentorsofficial

3) Thyroid gland

िायराइिगरिी 4) Liver

यकत

Correct Answer Liver

Q200 Psidium guajava is the scientific

name of

लसडियमगआजावा mdashmdash कावहॴजञाननकनामहहॴ 1-

Feb-2017

Options

1) Guava

अम द

2) Mango

आम

3) Bamboo

बास

4) Jack fruit

कटहि

Correct Answer Guava

Q201 Which drug is used as a Blood

Thinner

चधरकह८पतिाकरनहॳकहॳ पमककसदवाकापरयह८गककयाजाताहहॴ

1-Feb-2017

Options

1) Warfarin

वाफर न

2) Tramadol

टरहॳमािह८ि

3) Azithromycin

एनजरह८मायलसन

4) Hydralazine

हाइडरह८िहॳनजन

Correct Answer Warfarin

Q202 Which of the following disease is

caused due to the deficiency of protein

परह८टीनकीकमीकहॳ कारणनननननलिखितमसहॳकह९नसारह८गहह८ताहहॴ 1-Feb-2017

Options

1) Arthritis

गटठया 2) Kwashiorkor

कािीओकय र

3) Goitre

गाइटर

4) Night Blindness

रतह९चध

Correct Answer Kwashiorkor

Q203 A is species of plant that has

adapted to survive in an environment

with little liquid water

mdashmdashndashपह९धहॳकीएकऐसहॳऐसहॳपरजानतहहॴ नजसनहॳकमपानीवािहॳवातावरणमजीपवतरहनहॳकहॳलिएअनकिनहहॴ 1-Feb-2017

Options

1) Xerophyte

म दपवद

2) Hydrophyte

जिीयपादप

3) Mesophyte

समह८दपवद

4) Thallophyte

िहॴिह८फाइटा Correct Answer xerophyte

xerophyte is a species of plant that has

adapted to survive in an environment

with little liquid water such as a desert

or an ice- or snow-covered region in the

Alps or the Arctic

Mesophytes are terrestrial plants which

are adapted to neither a particularly

dry nor particularly wet environment

An example of a mesophytic habitat

would be a rural temperate meadow

F A C E B O O K

P A G E h t t p w w w f a c e b o o k c o m s s c m e n t o r s o f f i c i a l P a g e | 47

FOR MORE UPDATES AND MORE MATERIAL DO LIKE OUR FACEBOOK PAGE httpwwwfacebookcomsscmentorsofficial

which might contain goldenrod clover

oxeye daisy and Rosa multiflora

thallophyte any of a group of plants or

plantlike organisms (such as algae and

fungi) that lack differentiated stems

leaves and roots and that were formerly

classified as a primary division

(Thallophyta) of the plant kingdom

Q204 How many types of teeth are

there in humans

मनषयोमककतनहॳपरकारकहॳ दातहह८तहॳहहॴ

1-Feb-2017

Options

1) 4

2) 5

3) 2

4) 3

Correct Answer 4

teeth -Humans have four types of

teethincisors canines premolars and

molars each with a specific function

The incisors cut the food the canines

tear the food and the molars and

premolars crush the food

Q205 Carica papaya is the scientific name of

कहॴ ररकापपाया mdashmdashndash कावहॴजञाननकनामहहॴ 2-

Feb-2017

Options

1) Peepal

पीपि

2) Papaya

पपीता 3) Tamarind

इमिी 4) Drumstick

ढह८िकाछड़ी Correct Answer Papaya

Q206 Muscles get tired when there is

shortfall of

जब mdashndash कीकमीहह८तीहहॴतबपहॳिीयिकजातीहहॴ 2-Feb-2017

Options

1) Lactic acid

िहॴनकटकएलसि

2) Na+ ions

Na+ आयन

3) ATP

एटीपी 4) Sulphates

समफहॳ टस

Correct Answer ATP

ATP is the energy source muscle fibers

use to make muscles contract

muscle tissuersquos main source of energy

called adenosine triphosphate or ATP

As your muscles use up this energy

source they become tired and fatigued

Oxygen is the key ingredient that helps

create new ATP to replenish the burned

up ATP in your muscles

Q207 Artocarpus integra is the

scientific name of आटह८कापयसइटीगरा mdashmdashmdash कावहॴजञाननकनामहहॴ 2-Feb-2017

Options

1) Guava

अम द

2) Pineapple

अनानास

3) Silver Oak

लसमवरओक

4) Jack fruit

कटहि

Correct Answer Jack fruit

Q208 Which organ stores fat soluble

vitamins

कह९नसाअगवसामघिनिीिपवटालमनह८काभिाराकरताहहॴ

2-Feb-2017

F A C E B O O K

P A G E h t t p w w w f a c e b o o k c o m s s c m e n t o r s o f f i c i a l P a g e | 48

FOR MORE UPDATES AND MORE MATERIAL DO LIKE OUR FACEBOOK PAGE httpwwwfacebookcomsscmentorsofficial

Options

1) Blood

रकत

2) Skin

तवचा 3) Liver

यकत

4) Pancreas

अगनयािय

Correct Answer Liver

Q209 Which disease is caused due to

deficiency of Iodine

आयह८िीनकहॳ कारणकह९नसारह८गहह८ताहहॴ 2-Feb-2017

Options

1) Rickets

ररकहॳ टस

2) Scurvy

सकवी 3) Goitre

गणमािा 4) Growth retardation

पवकासका कना Correct Answer Goitre

rickets A softening and weakening of

bones in children usually due to

inadequate vitamin D

Q210 Grevillea Robusta is the scientific name of

गरहॳपवलियारह८बसटा mdashmdashmdash- कापवजञाननकनामहहॴ 2-Feb-2017

Options

1) Peepal

पीपि

2) Teak

सागह९न

3) Silver Oak

लसमवरओक

4) Jack fruit

कटहि

Correct Answer Silver Oak

Q211 When a Cuttlefish is described as a Molluscs it is at which level of

classification

जबएककटिकफिकह८एकमह८िसकाकहॳ पमवखणयतककयाजाताहहॴतबयहॳवगीकरणकहॳ ककससतरपहॳनसितहहॴ 2-Feb-2017

Options

1) Class

वगय 2) Order

िम

3) Family

पररवार

4) Phylum

सघ

Correct Answer Phylum

Q212 Bambusa dendrocalmus is the

scientific name of बानबसािहॳडराकामस mdashmdashmdash कावहॴजञाननकनामहहॴ 3-Feb-2017

Options

1) Banyan

बरगद

2) Papaya

पपीता 3) Bamboo

बास

4) Pomegranate

अनार

Correct Answer Bamboo

Q213 Acinonyx Jubatus is the scientific name of

एलसनह८ननकसजयबहॳटस mdashmdashmdash

कावहॴजञाननकनामहहॴ 3-Feb-2017

F A C E B O O K

P A G E h t t p w w w f a c e b o o k c o m s s c m e n t o r s o f f i c i a l P a g e | 49

FOR MORE UPDATES AND MORE MATERIAL DO LIKE OUR FACEBOOK PAGE httpwwwfacebookcomsscmentorsofficial

Options

1) Bear

भाि 2) Horse

घह८िा 3) Cheetah

चीता 4) Zebra

जहॳिा Correct Answer Cheetah

Q214 The pale yellow colour of urine is

due to the presence of which pigment

मतरकाफीकापीिारगरगदरयकहॳ उपनसिनतकहॳ कारणहह८ताहहॴ

3-Feb-2017

Options

1) Urochrome

यरह८िह८म

2) Urophyll

यरह८कफि

3) Chlorophyll

किह८रह८कफि

4) Chloroplast

किह८रह८पिासट

Correct Answer Urochrome

Q215 Which of the following constitute

to form a gene

नननननलिखितमसहॳकह९नसीचीज़एकजीनकागठनकरतीहहॴ

3-Feb-2017

Options

1) Polynucleotides

पह८िीनयनकियह८टाईडस

2) Hydrocarbons

हाइडरह८काबोस

3) Lipoproteins

िाईपह८परह८टीनस

4) Lipids

लिपपडस

Correct Answer Polynucleotides

Polynucleotide molecule is a biopolymer

composed of 13 or more nucleotide

monomers covalently bonded in a chain

DNA (deoxyribonucleic acid) and RNA

(ribonucleic acid) are examples of

polynucleotides with distinct biological

function

Q216 Vertebrates belongs to the

phylum

रीढ़कीहडिीवािहॳपराणी mdashmdashmdash

परजानतकहॳ अतगायतआतहॳहहॴ 3-Feb-2017

Options

1) Arthropoda

आरह८पह८ड़ा 2) Annelida

एननलििा 3) Cnidaria

ननिहॳररया 4) Chordata

कह८िटा Correct Answer Chordata

Q217 Punica granatum is the scientific name of

पननकगरहॳनहॳटस mdashmdashmdash कावहॴजञाननकनामहहॴ 3-Feb-2017

Options

1) Custard Apple

सीताफि

2) Gulmohar

गिमह८हर

3) Silver Oak

लसमवरओक

4) Pomegranate

अनार

Correct Answer Pomegranate

F A C E B O O K

P A G E h t t p w w w f a c e b o o k c o m s s c m e n t o r s o f f i c i a l P a g e | 50

FOR MORE UPDATES AND MORE MATERIAL DO LIKE OUR FACEBOOK PAGE httpwwwfacebookcomsscmentorsofficial

Q218 Between a tiger and an monkey

which of the following is different

एकबाघऔरबदरकहॳ बीचनननननलिखितमसहॳकह९नसीबातअिगहहॴ 3-Feb-2017

Options

1) Kingdom

राजय

2) Phylum

जानत

3) Order

िम

4) Class

वगय Correct Answer order

Q219 The artificial heart was invented by

कबतरमहदयका mdashmdashmdash

दवाराअपवषकारककयागयािा 3-Feb-2017

Options

1) Muhammad Yunus

महनमदयनस

2) Linus Yale Jr

िाइनसयहॳिजय

3) Gazi Yasargil

गाजीयासचगयि

4) Paul Winchell

पह९िपवमकि Correct Answer Paul Winchell

Q220 Tamarindus indica is the

scientific name of

टहॳमररनडसइडिका mdashmdash कावहॴजञाननकनामहहॴ 7-

Feb-2017

Options

1) Neem

नीम

2) Pineapple

अनानास

3) Tamarind

इमिी 4)Chiku

चीक

Correct Answer Tamarind

Q221 In eukaryotic cells synthesis of

RNA takes place in the

यकहॳ योटटककह८लिकाओमआरएनएकासशिहॳषण

mdashndash महह८ताहहॴ 7-Feb-2017

Options

1) Mitochondria

माईटह८कोडडरया 2) Centrioles

सटरीयह८मस

3) Ribosomes

ररबह८सह८नस

4) Nucleus

नयनकियस

Correct Answer nucleus

eukaryotic cell -Transcription is the

process of synthesizing ribonucleic acid

(RNA)Synthesis takes place within the

nucleus of eukaryotic cells or in the

cytoplasm of prokaryotes and converts

the genetic code from a gene in

deoxyribonucleic acid ( DNA ) to a

strand of RNA that then directs

proteinsynthesis

Q222 _________is caused by parasites

of the Plasmodium genus

पिाजमह८डियमजातीकहॳ परजीवी mdash- कहॳ कारणहहॴ 7-Feb-2017

Options

1) Dysentery

पहॳचचि

2) Malaria

मिहॳररया 3) Chickenpox

F A C E B O O K

P A G E h t t p w w w f a c e b o o k c o m s s c m e n t o r s o f f i c i a l P a g e | 51

FOR MORE UPDATES AND MORE MATERIAL DO LIKE OUR FACEBOOK PAGE httpwwwfacebookcomsscmentorsofficial

चहॳचक

4) Herpes

हहॳपपयस

Correct Answer Malaria

Q223 Carotene in fruits and vegetables

gives it which color

फिह८औरसनलजयोमनसितकहॳ रह८टीनउनहकह९नसारगपरदानकरताहहॴ 7-Feb-2017

Options

1) Green

हरा 2) Pink

गिाबी 3) Orange

नारगी 4) Blue

नीिा Correct Answer Orange

Q224 Equus Caballus is the scientific

name of

एकवसकहॴ बहॳिस mdashmdashndash कापवजञाननकनामहहॴ 7-Feb-2017

Options

1) Horse

घह८िा 2) Zebra

ज़हॳिा 3) Donkey

गधा 4) Buffalo

भस

Correct Answer Horse

Q225 Elapidae Naja is the scientific name of

एिीपीिीनाजा mdashmdash- कावहॴजञाननकनामहहॴ 8-Feb-2017

Options

1) Cobra

कह८बरा 2) Elephant

हािी 3) Eagle

ग ि

4) Owl

उमि Correct Answer Cobra

Q226 Which disease is caused due to

deficiency of Iron

िह८हकीकमीकहॳ कारणकह९नसारह८गहह८ताहहॴ 8-Feb-

2017

Options

1) Beriberi

बहॳरीबहॳरी 2) Tetany

टहॳटनी 3) Kwashiorkor

कवािीऔरकर

4) Anaemia

रकतामपता Correct Answer Anaemia

Beriberi is a disease caused by a vitamin

B-1 deficiency also known as thiamine

deficiency

Tetany can be the result of an

electrolyte imbalance Most often itrsquos a

dramatically low calcium level also

known as hypocalcemia Tetany can also

be caused by magnesium deficiency or

too little potassium Having too much

acid (acidosis) or too much alkali

(alkalosis) in the body can also result in

tetany

Kwashiorkor also known as

ldquoedematous malnutrition It is a form of

malnutrition caused by a lack of protein

in the diet

Anaemia means that you have fewer red

blood cells than normal or you have less

F A C E B O O K

P A G E h t t p w w w f a c e b o o k c o m s s c m e n t o r s o f f i c i a l P a g e | 52

FOR MORE UPDATES AND MORE MATERIAL DO LIKE OUR FACEBOOK PAGE httpwwwfacebookcomsscmentorsofficial

haemoglobin than normal in each red

blood cell

Q227 is a leaf where the leaflets are

arranged along the middle vein

mdashndashएकपततीहहॴजहापतरकह८कीरचनाक ररयालिराकहॳ आसपासहह८तीहहॴ 8-Feb-2017

Options

1) Pinnately compound leaf

पपनहॳटिीसयकतपतती 2) Palmately compound leaf

पामहॳटिीसयकतपतती 3) Compound leaf

सयकतपतती 4) Simple leaf

साधारणपतती Correct Answer Pinnately compound

leaf

Q228 Haustoria or sucking roots are

found in which of the following

हह८सटह८ररयायाचसनहॳवािीजड़हॳनननननलिखितमसहॳककसमपाईजातीहहॴ 8-Feb-2017

Options

1) Wheat

गहॳह

2) Mango

आम

3) Chestnut

चहॳसटनट

4) Cuscuta

कसकयटा Correct Answer Cuscuta

Haustorial roots -The roots of parasitic

plants which penetrate into the host

tissues to absorb nourishment are

called haustorial roots hellip Also known as suckingor parasitic roots

Q229 Equs Asinus is the scientific name

of

एकवसएलसनस mdashmdashndash कावहॴजञाननकनामहहॴ 8-

Feb-2017

Options

1) Donkey

गधा 2) Cow

गाय

3) Deer

टहरन

4) Kangaroo

कगा

Correct Answer Donkey

Q230 Ficus benghalensis is the scientific name of

फाईकसबहॳनगहॳिहॳलसस mdashndash कापवजञाननकनामहहॴ 8-Feb-2017

Options

1) Banyan

बरगद

2) Pineapple

अनानास

3) Babul

बबि

4) Tulsi

तिसी Correct Answer Banyan

Q231 Equus burchellii is the scientific name of

एकवसबचिी mdashmdash- कापवजञाननकनामहहॴ 8-Feb-2017

Options

1) Horse

घह८िा 2) Zebra

जहॳिा 3) Buffalo

F A C E B O O K

P A G E h t t p w w w f a c e b o o k c o m s s c m e n t o r s o f f i c i a l P a g e | 53

FOR MORE UPDATES AND MORE MATERIAL DO LIKE OUR FACEBOOK PAGE httpwwwfacebookcomsscmentorsofficial

भस

4) Ass

गधा Correct Answer Zebra

Page 28: COMPILATION OF ALL 72 SETS OF BIOLOGY SSC CHSL-2016 · OF BIOLOGY SSC CHSL-2016 PREPARED BY : SSC MENTORS BIOLOGY SPECIAL . F A C E B O O K P A G E : h t t p : / / w w w . f a c e

F A C E B O O K

P A G E h t t p w w w f a c e b o o k c o m s s c m e n t o r s o f f i c i a l P a g e | 27

FOR MORE UPDATES AND MORE MATERIAL DO LIKE OUR FACEBOOK PAGE httpwwwfacebookcomsscmentorsofficial

पपततनमक

2) Cholesterol

कह८िहॳसटरह८ि

3) Lymph

लिनफ

4) Urochrome

यरह८िह८म

Correct Answer Urochrome

Urobilin or urochrome is the chemical

primarily responsible for the yellow

color of urine

Q113 The wilting of plants takes place

due to

पह९धह८कालिचििहह८नाकी mdashmdash- कीवजहसहॳहह८ताहहॴ 22-Jan-2017

Options

1)Photosynthesis

परकािसशिहॳषण

2) Transpiration

वाषपह८तसजयन

3) Absorption

अविह८षण

4) Respiration

शरवसन

Correct Answer Transpiration

Wilting is the loss of rigidity of non-

woody parts of plants This occurs when

the turgor pressure in non-lignified

plant cells falls towards zero as a result

of diminished water in the cells

Q114 Bovidae Ovis is the scientific name of

बह८पविीओपवस mdashndash कावहॴजञाननकनामहहॴ 22-Jan-2017

Options

1) Goat

बकरी 2) Cow

गाय

3) Buffalo

भहॳस

4) Sheep

भहॳड़

Correct Answer Sheep

Q115 Plants get their energy to produce

food from which of the following

पह८धहॳभह८जनकाननमायणकरनहॳकहॳ लिएनननननलिखितमसहॳककससहॳउजायपरापतकरतहॳहहॴ

22-Jan-2017

Options

1) Photosynthesis

परकािसशिहॳषण

2)Bacteria

बहॴकटीररया 3)Fungi

कवक

4)Sun

सयय Correct Answer Sun

Q116 Which of the following is secreted

by the liver

नननननलिखितमसहॳककसकासरावनजगरसहॳहह८ताहहॴ

22-Jan-2017

Options

1) Glucose

गिकह८ज

2) Iodine

आयह८िीन

3) Cortisol

काटटरयसह८ि

4) Bile

पपतत

Correct Answer Bile

The liver makes bile that will help

emulsify and

digest the fats we eat

F A C E B O O K

P A G E h t t p w w w f a c e b o o k c o m s s c m e n t o r s o f f i c i a l P a g e | 28

FOR MORE UPDATES AND MORE MATERIAL DO LIKE OUR FACEBOOK PAGE httpwwwfacebookcomsscmentorsofficial

Q117 Ferns belong to which division of

plants

फनसयपह९धह८कहॳ ककसभागमआतहॳहहॴ

22-Jan-2017

Options

1) Gymnosperms

नजननह८सपनसय 2) Angiosperms

एनजयह८सपनसय 3) Thallophyta

िहॴिह८फाईटा 4)Pteridophyta

टहॳररिह८फाईटा Correct Answer Pteridophyta

Q118 Who invented Antibiotics

एटीबायह८टटककाअपवषकारककसनहॳककयािा

22-Jan-2017

Options

1) Joseph Lister

जह८सहॳफलिसटर

2) William Harvey

पवलियमहाव

3) Robert Knock

रॉबटयनॉक

4)Alexander Fleming

अिहॳकज़िरफिहॳलमग

Correct Answer Alexander Fleming

Q119 Milbecycin is used in the

eradication of

लममबहॳसायलसनका mdashndash

मउनमिनमपरयह८गककयाजाताहहॴ 22-Jan-2017

Options

1) Agricultural Fungus

कपषकवक

2) Agricultural Pests

कपषकीटक

3) Agricultural Herbs

कपषिाक

4)Agricultural Weeds

कपषननराना Correct Answer Agricultural Pests

Milbemycin oxime is a veterinary drug

from the group of milbemycins used as

a broad spectrum antiparasitic It is

active against worms and mites(insects

Q120 Intestinal bacteria synthesizes

which of the following in the human

body

मानविरीरमआतोकहॳ बहॴकटीररयानननननलिखितमसहॳककसकासशिहॳषणकरतहॳहहॴ 22-Jan-2017

Options

1) Vitamin K

पवटालमन K

2) Proteins

परह८टीन

3) Fats

वसा 4) Vitamin D

पवटालमन D

Correct Answer Vitamin K

Q121 is the study of the physical form

and external structure of plants

mdashmdash-

मपह९धह८काभहॴनतक पऔरबाहरीसरचनाकाआदयाककयाजाताहहॴ 22-Jan-2017

Options

1) Physiology

कफनजयह८िह८जी 2) Anatomy

िरीररचनापवजञान

3) Phytomorphology

फाईटह८मह८फह८िह८जी 4)Cytology

कह८लिकापवजञान

Correct Answer Phytomorphology

F A C E B O O K

P A G E h t t p w w w f a c e b o o k c o m s s c m e n t o r s o f f i c i a l P a g e | 29

FOR MORE UPDATES AND MORE MATERIAL DO LIKE OUR FACEBOOK PAGE httpwwwfacebookcomsscmentorsofficial

Q122 Which of the following is a

structural and functional unit of

kidneys

नननननलिखितमसहॳकह९नसीगदोकीसरचनातमकऔरकाययकरीईकाईहहॴ

22-Jan-2017

Options

1) Renette Cells

रहॳनहॳटकह८लिकाए

2) Flame Cells

फिहॳमकह८लिकाए

3) Nephrites

नहॳफ़राइटस

4)Nephrons

नहॳफरोस

Correct Answer Nephrons

Nephron functional unit of the kidney

the structure that actually produces

urine in the process of removing waste

and excess substances from the blood

There are about 1000000 nephrons in

each human kidney

Q123 Which of the following is the

largest part of the human brain

नननननलिखितमसहॳकह९नसामानवमनसतषककासबसहॳबड़ाटहससाहहॴ

23-Jan-2017

Options

1) Ribs

पसलियाा 2) Cerebrum

सहॳरहॳिम

3) Pons

पोस

4)Thalamus

िहॴिहॳमस

Correct Answer Cerebrum

The cerebrum is the largest part of the

human brain making up about two-

thirds of the brainrsquos mass It has two

hemispheres each of which has four

lobes frontal parietal temporal and

occipital

Q124 The auxiliary buds

सहायककालियाmdashndash 23-Jan-2017

Options

1) grow endogenously from the pericycle

पहॳरीसाईककिसहॳअनतजातयपवकलसतहह८ताहहॴ 2) arise endogenously from the main

growing point

मिवपदसहॳअनतजातयउठताहहॴ 3) is an embryonic shoot located in the

axil of a leaf

एकभरणिटहहॴजह८एकपततीकहॳ अकषपरनसतिहह८ताहहॴ 4)arise exogenously from the epidermis

एपपिलमयससहॳबटहजातयतरीकहॳ सहॳउठताहहॴ Correct Answer is an embryonic shoot

located in the axil of a leaf

Q125 Which of the following is a viral

disease

इनमहॳसहॳकह९सीएकवायरिबीमारीहहॴ

23-Jan-2017

Options

1) Polio

पह८लियह८ 2) Tetanus

धनसतनभ

3) Leprosy

कषठरह८ग

4) Plague

पिहॳग

Correct Answer Polio

A viral disease (or viral infection)

occurs when an organismrsquos body is

invaded by pathogenic viruses and

infectious virus particles (virions) attach

to and enter susceptible cells

F A C E B O O K

P A G E h t t p w w w f a c e b o o k c o m s s c m e n t o r s o f f i c i a l P a g e | 30

FOR MORE UPDATES AND MORE MATERIAL DO LIKE OUR FACEBOOK PAGE httpwwwfacebookcomsscmentorsofficial

Poliomyelitis often called polio or

infantile paralysis is an infectious

disease caused by the poliovirus

Tetanusmdash A serious bacterial infection

that causes painful muscle spasms and

can lead to death

Leprosy also known as Hansenrsquos

disease (HD) is a long-term infection by

the bacterium Mycobacterium leprae or

Mycobacterium lepromatosis

Plague is an infectious disease caused by

the bacterium Yersinia pestis

Symptoms include fever weakness and

headache

Q126 Which organisms can help to

carry out Vermicomposting

कह९नसाजीववमीकनपह८नसटगममददकरताहहॴ

23-Jan-2017

Options

1) Nitrifying Bacteria

नाईटरीफाईगबहॴकटीररया 2) Earthworms

कहॴ चऐ

3) Algae

िहॴवि

4) Fungus

कवक

Correct Answer Earthworms

Q127 Contraction of heart is also

known as

हदयकहॳ सकचनकह८ mdash- भीकहाजाताहहॴ 23-Jan-

2017

Options

1) Systole

लससटह८ि

2) Aristotle

अरसत

3) Diastole

िायसटह८ि

4) Lub

मयब

Correct Answer Systole

Diastole is the part of the cardiac cycle

when the heart refills with blood

following systole (contraction)

Ventricular diastole is the period during

which the ventricles are filling and

relaxing while atrial diastole is the

period during which the atria are

relaxing

Q128 Azadirachta indica is the

botanical name of which of the

following

अजाटदराचताइडिकानननननलिखितमसहॳककसकावानसपनतनामहहॴ

23-Jan-2017

Options

1) Rose plant

गिाबकापह९धा 2) Apple tree

सहॳबकापहॳड़

3) Neem

नीम

4)Mango

आम

Correct Answer Neem

Q129 Which of the following is the

main end product of carbohydrate

digestion

नननननलिखितमसहॳकह९नसाकाबोहाइडरहॳटकहॳ पाचनकापरमिअतउतपादकहह८ताहहॴ 23-Jan-2017

Options

1) Fats

वसा 2) Lipids

लिपपडस

3) Glucose

गिकह८ज

4) Cellulose

F A C E B O O K

P A G E h t t p w w w f a c e b o o k c o m s s c m e n t o r s o f f i c i a l P a g e | 31

FOR MORE UPDATES AND MORE MATERIAL DO LIKE OUR FACEBOOK PAGE httpwwwfacebookcomsscmentorsofficial

सहॳमयिह८ज

Correct Answer Glucose

Intestinal absorption of end products

from digestion of carbohydrates and

proteins in the pig hellip During absorption some sugars (fructose or

galactose) released from the

corresponding sucrose and lactose

respectively during digestion were

partly metabolized into glucose by the

enterocyte

Q130 Which of the following glands is a

source of the enzyme Ptyalin

नननननलिखितगरचियोमसहॳएजाइमटयालिनकासरह८तहहॴ 23-Jan-2017

Options

1) Pancreas

अगरािय

2) Thyroid Gland

िाइराइिगरिी 3) Pituitary Gland

पीयषगरिी 4) Salivary Glands

िारगरचियाा Correct Answer Salivary Glands

Q131 Which of the following is not true

about Pteridophyta

ननननमसहॳकह९नसीबातटहॳररिह८फाईटकहॳ बारहॳमसचनहीहहॴ 23-Jan-2017

Options

1) Dominant phase is saprophytes

परमिचरणसहॳपरह८फाईइटसहह८ताहहॴ 2) Main plant body is diploid

पह९दह८कामखयिरीरदपवगखणतहह८ताहहॴ 3) Seeds are present

बीजमह९जदहह८तहॳहहॴ 4)Flowers are absent

फिअनपनसतिहह८तहॳहहॴ

Correct Answer Seeds are present

Q132 The largest dolphin species is the

orca also called as

िॉिकफनकीसबसहॳबड़ीपरजानतकाकानामआकायहहॴनजसहॳ mdash- भीकहतहॳहहॴ 23-Jan-2017

Options

1) Bottle Nose

बाटिनह८ज

2) Baiji

बहॳजी 3) Killer whale

ककिरहहॳि

4)Tucuxi

टकवसी Correct Answer Killer whale

Q133 The fat digesting enzyme Lipase

is secreted by which of the following

वसाकापाचनकरनहॳवािाएजाइमिाइपहॳजनननननलिखितमसहॳककसकहॳ दवारासतरापवतहह८ताहहॴ

24-Jan-2017

Options

1) Kidneys

गद

2) Pancreas

अगनयािय

3) Large Intestine

बड़ीआत

4)Liver

नजगर

Correct Answer Pancreas

Lipase is an enzyme that splits fats so

the intestines can absorb them Lipase

hydrolyzes fats like triglycerides into

their component fatty acid and glycerol

molecules It is found in the blood

gastric juices pancreatic secretions

intestinal juices and adipose tissues

F A C E B O O K

P A G E h t t p w w w f a c e b o o k c o m s s c m e n t o r s o f f i c i a l P a g e | 32

FOR MORE UPDATES AND MORE MATERIAL DO LIKE OUR FACEBOOK PAGE httpwwwfacebookcomsscmentorsofficial

Q134 The arrangement of leaves on an

axis or stem is called

एकअकषयातनहॳपरपनततयोकीयवसिाकह८कयाकहाजाताहहॴ SSC CHSL Science (biology) 2016

Question Paper

24-Jan-2017

Options

1) Phyllotaxy

फाइिह८टहॴकसी 2) Vernation

वनिन

3) Venation

वहॳनहॳिन

4)Phytotaxy

फाइटह८टहॴकसी Correct Answer Phyllotaxy

In botany phyllotaxis or phyllotaxy is

the arrangement of leaves on a plant

stem (from Ancient Greek phyacutellon

ldquoleafrdquo and taacutexis ldquoarrangementrdquo)

Phyllotactic spirals form a distinctive

class of patterns in nature

Q135 The study of Cells is also known

as

कह८लिकाओकहॳ अधययनकह८ mdashmdashndash

भीकहाजाताहहॴ 24-Jan-2017

Options

1) Cytology

सायटह८िह८जी 2) Physiology

कफनजयह८िह८जी 3) Nucleology

नयककमयह८िह८जी 4)Cellology

सहॳिह८िह८जी Correct Answer Cytology

Q136 Which of the following scientists

is also known as the Father of Biology

नननननलिखितमसहॳककसवहॴजञाननककह८ ldquoजीवपवजञानकहॳ जनकrdquoकहॳ नामसहॳभीजानाजाताहहॴ 24-Jan-2017

Options

1) Herbert Spencer

हबयटयसपसर

2) Aristotle

अरसत 3) Lamarck

िहॳमाकय 4)Darwin

िापवयन

Correct Answer Aristotle

Q137 Which cells give rise to various

organs of the plant and keep the plant

growing

कह९नसीकह८लिकाएपह९धह८कहॳ लभननअगह८कह८जनमदहॳतीहहॴऔरपह९धह८कह८बढ़नहॳममददकरतीहहॴ

24-Jan-2017

Options

1) Permanent

सिायी 2) Dermal

तवचीय

3) Meristematic

मररसटहॳमटटक

4)Mature

परह८ढ़

Correct Answer Meristematic

A meristem is the tissue in most plants

containing undifferentiated cells

(meristematic cells) found in zones of

the plant where growth can take place

Q138 Rodentia Muridae is the scientific

name of

F A C E B O O K

P A G E h t t p w w w f a c e b o o k c o m s s c m e n t o r s o f f i c i a l P a g e | 33

FOR MORE UPDATES AND MORE MATERIAL DO LIKE OUR FACEBOOK PAGE httpwwwfacebookcomsscmentorsofficial

रह८िहॳलियानयररिी mdashmdash- कावहॴजञाननकनामहहॴ 24-

Jan-2017

Options

1) Mouse

चहा 2) Squirrel

चगिहरी 3) Monkey

बदर

4) Lizard

नछपकिी Correct Answer Mouse

Q139 Name the scientist who proposed

the cell theory

कह८लिकालसदातकापरसतावदहॳनहॳवािहॳवहॴजञाननककानामबताइए 24-Jan-2017

Options

1) Schleiden and Schwann

िीमिनऔरशरववान

2) Lamarck

िहॳमाकय 3) Treviranus

टरहॳवायरहॳनस

4)Whittaker and Stanley

हीटकरऔरसटहॳनिहॳ Correct Answer Schleiden and

Schwann

Q140 The flower with the worldrsquos

largest bloom is

दननयाकासबसहॳबड़ाफिखििनहॳवािा mdashmdashndash हहॴ 24-Jan-2017

Options

1) Pando

पािह८ 2) Posidonia

पह८सीिह८ननया 3) Rafflesia arnoldii

ररफिहॳलियाअनोमिी 4)Helianthus annuus

हहॳलिएनिसएनयअस

Correct Answer Rafflesia arnoldii

Rafflesia arnoldii is a species of

flowering plant in the parasitic genus

Rafflesia It is noted for producing the

largest individual flower on earth It has

a very strong and horrible odour of

decaying flesh earning it the nickname

ldquocorpse flower

Q141 Deficiency of which vitamin

causes night blindness

ककसपवटालमनकीकमीकहॳ कारणरतौधीहह८ताहहॴ 24-Jan-2017

Options

1) Vitamin K

पवटालमन K

2) Vitamin C

पवटालमन C

3) Vitamin B1

पवटालमन B1

4)Vitamin A

पवटालमन A

Correct Answer Vitamin A

Q142 Nongreen plants lack which of the

following

गहॴर-

हररतवनसपनतमनननननलिखितमसहॳककसकीकमीहह८तीहहॴ

24-Jan-2017

Options

1) Chlorophyll

किह८रह८कफि

2) Lycophyll

िायकह८कफि

3) Cyanophyll

F A C E B O O K

P A G E h t t p w w w f a c e b o o k c o m s s c m e n t o r s o f f i c i a l P a g e | 34

FOR MORE UPDATES AND MORE MATERIAL DO LIKE OUR FACEBOOK PAGE httpwwwfacebookcomsscmentorsofficial

सायनह८कफि

4)Phototropism

फह८टह८टरोपपजम

Correct Answer Chlorophyll

Q143 Organisms that use light to

prepare food are known as

जह८जीवपरकािकाउपयह८गकरभह८जनतहॴयारकरतहॳहहॴ उनह mdashmdash- कहॳ पमजानजाताहहॴ 24-Jan-2017

Options

1) Autotrophs

सवपह८षी 2) Heterotrophs

पवषमपह८षज

3) Omnivores

सवायहारी 4)Decomposers

पवघटनकरनहॳवािा Correct Answer Autotrophs

autotrophs often make their own food

by using sunlight carbon dioxide and

water to form sugars which they can use

for energy Some examples of

autotrophs include plants algae and

even some bacteria Autotrophs

(producer) are important because they

are a food source for heterotrophs

(consumers)

A heterotroph is an organism that

ingests or absorbs organic carbon

(rather than fix carbon from inorganic

sources such as carbon dioxide) in order

to be able to produce energy and

synthesize compounds to maintain its

life Ninety-five percent or more of all

types of living organisms are

heterotrophic including all animals and

fungi and some bacteria

Q144 Which of the following is a

primary function of haemoglobin

नननननलिखितमसहॳकह९नसाटहमह८गिह८बबनकाएकपरािलमककाययहहॴ

25-Jan-2017

Options

1) Utilization of energy

उजायकाउपयह८गकरना 2) Prevention of anaemia

रकतामपताहह८नहॳसहॳरह८कना 3) Destruction of bacteria

बहॴकटीररयाकापवनािकरना 4) To transport oxygen

ऑकसीजनकावहनकरना Correct Answer To transport oxygen

Q145 Vascular bundles are absent in

सवहनीबिि mdashmdash- मअनपनसतिरहतहॳहहॴ 25-Jan-2017

Options

1) Bryophyta

िायह८फाइटा 2) Pteridophyta

टहॳररिह८फाईटा 3) Gymnosperms

नजननह८सपमय 4) Angiosperms

एननजयह८सपहॳनसय Correct Answer Bryophyta

Q146 Sauria Lacertidae is the scientific

name of

सहॴररयािहॳसरटाईिी mdashmdashndash कावहॴजञाननकनामहहॴ 25-Jan-2017

Options

1) Crocodile

मगरमचछ

2) Hippopotamus

टहपपह८पह८टहॳमस

3) Lizard

नछपकिी 4) House fly

F A C E B O O K

P A G E h t t p w w w f a c e b o o k c o m s s c m e n t o r s o f f i c i a l P a g e | 35

FOR MORE UPDATES AND MORE MATERIAL DO LIKE OUR FACEBOOK PAGE httpwwwfacebookcomsscmentorsofficial

घरहॳिमकिी Correct Answer Lizard

Q147 Which type of pathogen causes

the water-borne disease SARS (Severe

Acute Respiratory Syndrome)

ककसपरकािकारह८गज़नकजिजननतबीमारीसासयकाकारणबनताहहॴ 25-Jan-2017

Options

1) Viral

वायरि

2) Parasitic

परजीवी 3) Protozoan

परह८टह८जअन

4) Bacterial

बहॴकटीररयि

Correct Answer Viral

Q148 Which of the following organs

produces the enzyme lipase

नननननलिखितमसहॳकह९नसाअगिायपहॳजएजाइमउतपननकरताहहॴ 25-Jan-2017

Options

1) Pancreas

अगनयािय

2) Large Intestine

बड़ीआत

3) Liver

नजगर

4) Small Intestine

छह८टीआत

Correct Answer Pancreas

Q149 A is a long internode forming the

basal part or the whole of a peduncle

एक mdashmdash- एकिबाइटरनह८िहहॴ जह८ननचिाटहससायासनपणयिठिबनताहहॴ 25-

Jan-2017

Options

1) Rhizome

परकद

2) Rachis

महॳ दि

3) floral axis

पषपअकष

4) Scape

भगदड़

Correct Answer scape

Q150 ndash Which of the following

organisms are considered to be both

Living and Non-living

नननननलिखितमसहॳकह९नसहॳजीवाणकह८जीपवतऔरअजीपवतमानाजाताहहॴ

25-Jan-2017

Options

1) Bacteria

बहॴकटीररया 2) Fungi

कवक

3) Algae

िहॴवाि

4)Virus

वायरस

Correct Answer Virus

They are considered to be living as they

possess a protein coat as a protective

covering DNA as the genetic material

etc

They are said to be non-living as they

can be crystallised and they survive for

billions of years They can tolerate high

temperatures freezing cold

temperatures ultra-violet radiations etc

Q151 Deficiency of fluorine causes

which of the following

फिह८ररनकीकमीकहॳ कारणनननननलिखितमसहॳकयाहह८ताहहॴ

F A C E B O O K

P A G E h t t p w w w f a c e b o o k c o m s s c m e n t o r s o f f i c i a l P a g e | 36

FOR MORE UPDATES AND MORE MATERIAL DO LIKE OUR FACEBOOK PAGE httpwwwfacebookcomsscmentorsofficial

27-Jan-2017

Options

1) Dental Caries

िटिकहॴ ररज

2) Scurvy

सकवरी 3) Anaemia

रकतामपता 4) Arthritis

गटठया Correct Answer Dental Caries

Q152 In a Punnett Square with the

cross AaBb x AaBb how many Aabb

genotypes would be created

पनहॳटसककायरमिह८स AaBb x AaBb कहॳ साि

ककतनहॳ Aabb जीनह८टाइपबनगहॳ 27-Jan-2017

Options

1) 1

2) 8

3) 2

4) 3

Correct Answer 2

Q153 Which of the following is the

Controlling Center of the Cell

नननननलिखित म सहॳ कह८लिकाका ननयतरण

क दर कह९न हहॴ

27-Jan-2017

Options

1) Nucleus

क दर

2) Plasma

पिाजमा 3) Lysosome

िायसह८सह८म

4) Chromosome

िह८मह८सह८म

Correct Answer Nucleus

The control centre of the cell is the

nucleus in eukaryotic cells The nucleus

contains genetic material in the form of

DNA

Q154 Myopia affects which of the

following organs

मायह८पपयानननननलिखितअगह८मसहॳककसहॳपरभापवतकरताहहॴ

25-Jan-2017

Options

1) Heart

हदय

2) Skin

तवचा 3) Eyes

आािहॳ 4)Mouth

मह

Correct Answer Eyes

Q155 Which of the following bears

flowers

नननननलिखितमसहॳकह९नफिधारणकरताहहॴ

25-Jan-2017

Options

1) Bryophyta

िायह८फाइटा 2) Pteridophyta

टहॳरीिह८फाईटा 3) Gymnosperms

नजननह८सपमय 4)Angiosperms

एननजयह८सपमय Correct Answer Angiosperms

Q156 Oxygenated blood flows out of the

heart through the

ऑकसीजनयकतरकत mdashmdashmdash

कहॳ माधयमसहॳहदयकहॳ बाहरबहताहहॴ 25-Jan-2017

F A C E B O O K

P A G E h t t p w w w f a c e b o o k c o m s s c m e n t o r s o f f i c i a l P a g e | 37

FOR MORE UPDATES AND MORE MATERIAL DO LIKE OUR FACEBOOK PAGE httpwwwfacebookcomsscmentorsofficial

Options

1) Aorta

महाधमनी 2) pulmonary artery

फहॳ फड़हॳकीधमनी 3) vena cava

वहॳनाकावा 4)Atrium

चह९क

Correct Answer aorta

Q157 Blood leaving the liver and

moving towards the

heart has a higher concentration of

नजगरसहॳननकिकरहदयकीतरफजानहॳवािहॳरकतम mdashmdashmdashmdash कीउचचसादरताहह८तीहहॴ 27-Jan-2017

Options

1) Lipids

लिपपडस

2) Urea

यररया 3) Bile Pigments

पपततकहॳ रगकरण

4) Carbon dioxide

काबयनिायऑकसाइि

Correct Answer Bile Pigments

Urea is nitrogen containing substance

which is produced in the liver in order

to deal with excess amino-acids in the

body As urea is produced it leaves the

liver in the blood stream and passes via

the circulatory system to all parts of the

body

Q158 Bulb is a modification of which

part of a plant

बमबएकपह९धहॳकहॳ ककसटहससहॳकाएक पातरणहह८ताहहॴ 27-Jan-2017

Options

1) The root

जड़

2) The stem

तना 3) The radicle

मिाकर

4)The fruit

फि

Correct Answer The stem

Q159 Which of the following carries

blood away from the heart to different

body parts

इनमहॳसहॳकह९नरकतकह८हदयसहॳिरीरकहॳ पवलभननअगह८तकिहॳजातीहहॴ

27-Jan-2017

Options

1) Arteries

धमननया 2) Nerves

तबतरहाए

3) Capillaries

कहॳ लिकाए

4)Veins

नसहॳ Correct Answer Arteries

Q160 The series of processes by which

nitrogen and its compounds are

interconverted in the environment and

in living organisms is called

27-Jan-2017

Options

1)Absorption of Nitrogen

2)Ammonification

3)Nitrogen Fixation

4)Nitrogen Cycle

Correct Answer Nitrogen Cycle

Ammonification or Mineralization is

performed by bacteria to convert

organic nitrogen to ammonia

F A C E B O O K

P A G E h t t p w w w f a c e b o o k c o m s s c m e n t o r s o f f i c i a l P a g e | 38

FOR MORE UPDATES AND MORE MATERIAL DO LIKE OUR FACEBOOK PAGE httpwwwfacebookcomsscmentorsofficial

Nitrification can then occur to convert

the ammonium to nitrite and nitrate

Nitrogen fixation is a process by which

nitrogen in the Earthrsquos atmosphere is

converted into ammonia (NH3) or other

molecules available to living organisms

Q161 BCG vaccine is given to protect

from which of the following

बीसीजीकाटटकानननननलिखितमसहॳककसकहॳ बचावकहॳ लिएटदयाजातहहॴ

27-Jan-2017

Options

1) Jaundice

पीलिया 2) Anaemia

रकतमपता 3) Tuberculosis

कषयरह८ग

4) Polio

पह८लियह८ Correct Answer Tuberculosis

Q162 Parallel venation is found in

समानतरवहॳनहॳिन mdashmdashmdash- मपायाजाताहहॴ 27-Jan-2017

Options

1) plants which are monocots

पह९धहॳजह८एकबीजपतरीहह८तहॳहहॴ 2) plants which have a dicot stem

वहॳपह९धहॳनजनकातनादपवदलियहह८ताहहॴ 3) plants with leaves similar to Tulsi

वहॳपह९धहॳनजनकीपनततयतिसीकीपनततयोकहॳ समानहह८तहॳहहॴ 4)plants with tap roots

टहॳप टवािहॳपह९धहॳ Correct Answer plants which are

monocots

Q163 The hardest part of the body is

िरीरकासबसहॳकठह८रभाग mdashndash हहॴ 27-Jan-2017

Options

1) Bones

हडडिय

2) Tooth Enamel

दातकहॳ इनहॳमि

3) Skull

िह८पड़ी 4) Spinal Cord

महॳ रजज

Correct Answer Tooth Enamel

Q164 Which type of pathogen causes

the waterborne disease E coli Infection

ककसपरकारकारह८गजननकजिजननतरह८गईकह८िाईसिमणकाकारणबनताहहॴ 27-Jan-2017

Options

1) Protozoan

परह८टह८जआ

2) Parasitic

परजीवी 3) Bacterial

बहॴकटीररयि

4)Viral

वायरि

Correct Answer Bacterial

Q165 The amount of blood filtered

together by both the kidneys in a 70 kg

adult male human in a minute is

70 की गरा वािहॳएकवयसकप षमएकलमनटमदह८नोगदकहॳदवाराएकसािचाबनीगयीरकतकीमातरहह८तीहहॴ 29-Jan-2017

Options

1) 1100 ml

1100 लमलि

2) 100 ml

F A C E B O O K

P A G E h t t p w w w f a c e b o o k c o m s s c m e n t o r s o f f i c i a l P a g e | 39

FOR MORE UPDATES AND MORE MATERIAL DO LIKE OUR FACEBOOK PAGE httpwwwfacebookcomsscmentorsofficial

100 लमलि

3) 1500 ml

1500 लमलि

4) 500 ml

500 लमलि

Correct Answer 1100 ml

Q166 Which feature of a plant helps to

distinguish a monocot from a dicot

पह९धहॳकीवहकह९नसीपविहॳषताहहॴजह८एकदपवदलियहॳऔरएकएकदिीयपह९धहॳसहॳभहॳदकरनहॳममददकरतीहहॴ 29-Jan-2017

Options

1) Pollination

परागम

2) Venation

वहॳनहॳिन

3) Vernation

वनिन

4) Aestivation

एसटीवहॳिहॳन

Correct Answer venation

Q167 The Mutation Theory was

proposed by

उतवररवतयनकालसदात mdashmdashndash

कहॳ दवरापरसतापवतककयाजाताहहॴ 29-Jan-2017

Options

1) Charles Lyell

चामसयलियहॳि

2) William Smith

पवलियमनसमि

3) Hugo De Vries

हयगह८िीराईस

4)Harrison Schmitt

हहॳरीसननसमट

Correct Answer Hugo De Vries

Q168 Which type of pathogen causes

the waterborne disease HepatitisA

ककसपरकारकहॳ रह८गजनकजिजननतरह८गहहॳपहॳटाइटटस-A काकारणबनताहहॴ

29-Jan-2017

Options

1) Parasitic

परजीवी 2) Viral

वायरि

3) Protozoan

परह८टह८जआ

4) Bacterial

बहॴकटीररयि

Correct Answer Viral

Q169 In a Punnett Square with the

cross AaBb x Aabb how many AaBb

genotypes would be created

पनहॳटसकवायरमिह८स AaBb x Aabb

कहॳ सािककतनहॳ AaBb जीनह८टाइपबनगहॳ 29-Jan-

2017

Options

1) 4

2) 1

3) 7

4) 6

Correct Answer 4

Q170 Arboreal Ateles is the scientific

name of

अिह८ररयिएटटलिस mdashmdashmdash कावहॴजञाननकनामहहॴ 29-Jan-2017

Options

1) Squirrel

चगिहरी 2) Sparrow

गह८रहॴया 3) Lizard

नछपकिी 4) Spider monkey

F A C E B O O K

P A G E h t t p w w w f a c e b o o k c o m s s c m e n t o r s o f f i c i a l P a g e | 40

FOR MORE UPDATES AND MORE MATERIAL DO LIKE OUR FACEBOOK PAGE httpwwwfacebookcomsscmentorsofficial

मकड़ीबदर

Correct Answer Spider monkey

Q171 Which type of pathogen causes

the waterborne disease Salmonellosis

ककसपरकारकारह८गाणजिजननतबीमारीसािमह८नहॳिह८लसज़काकारकहहॴ

29-Jan-2017

Options

1) Algal

िहॳवालियहॳ 2) Parasitic

परजीवी 3) Bacterial

बहॴकटीररयि

4)Viral

वायरि

Correct Answer Bacterial

An infection with salmonella bacteria

commonly caused by contaminated food

or water

Symptoms include diarrhoea fever

chills and abdominal pain

Q172 is a condition in which there is a

deficiency of red cells or of haemoglobin

in the blood

mdashmdash-

एकनसिनतहहॴनजसमहॳरकतमिािकह८लिकाओकीयाहीमह८गिह८बबनकीकमीहह८तीहहॴ 29-Jan-2017

Options

1) Albinism

एनमबननजम

2) Propyria

परह८पीररया 3) Anaemia

एनीलमया 4)Keloid disorder

कहॳ िह८इिडिसओिर

Correct Answer Anaemia

Q173 Ananas comosus is the scientific

name of

Options

अनानासकह८मह८सस mdashmdashmdashndash

कावहॴजञाननकनामहहॴ 29-Jan-2017

1) Custard Apple

सीताफि

2) Pineapple

पाइनएपपि

3) Bamboo

बास

4)Pomegranate

अनार

Correct Answer Pineapple

Q174 Which organ produces insulin

कह९नसाअगइनसलिनपहॴदाकरताहहॴ 29-Jan-

2017

Options

1) Liver

यकत

2) Thyroid gland

िायराइिगरिी 3) Spleen

पिीहा 4)Pancreas

अगरयिय

Correct Answer Pancreas

Q175 Which of the following disease is

not caused by water pollution

नननननलिखितमसहॳकह९नसारह८गपानीकहॳ परदषणकहॳकारणनहीहह८ता

29-Jan-2017

Options

1) Cholera

हहॴजा 2) Typhoid

F A C E B O O K

P A G E h t t p w w w f a c e b o o k c o m s s c m e n t o r s o f f i c i a l P a g e | 41

FOR MORE UPDATES AND MORE MATERIAL DO LIKE OUR FACEBOOK PAGE httpwwwfacebookcomsscmentorsofficial

टाइफाइि

3) Asthma

दमा 4)Diarrhoea

दसत

Correct Answer Asthma

Q176 Ocimum tenuiflorum is the

scientific name of

ओलिलममटहॳयईफिह८रमइसकावहॴजञाननकनाम mdash

ndash हहॴ 30-Jan-2017

Options

1) Neem

नीम

2) Mango

आम

3) Babul

बबि

4)Tulsi

तिसी Correct Answer Tulsi

Q177 Which gland secretes bile a

digestive fluid

कह९नसीगरिीपपतत एकपाचनतरिपरदािय सरापवतकरतीहहॴ 30-Jan-2017

Options

1) Pancreas

अगनयािय

2) Liver

यकत

3) Thyroid

िायराइि

4) Testes

टहॳनसटस

Correct Answer liver

Q178 In which of the following the

dominant phase is Gametophyte

नननननलिखितमसहॳककसकहॳ परमिचरणयगमकह८दपवधद (Gametophyte)हहॴ 30-Jan-2017

Options

1) Bryophyta

िायह८फाइटा 2) Pteridophyta

टहॳररिह८फाइटा 3) Gymnosperms

नजननह८सपमय 4) Angiosperms

एननजयह८सपमय Correct Answer Bryophyta

Q179 Anaerobic respiration refers to

which of the following

नननननलिखितमसहॳककसहॳअवायवीयशवसनकहाजाताहहॴ

30-Jan-2017

Options

1) Respiration without Oxygen

ऑकसीजनकहॳ बबनाशवसन

2) Respiration with Oxygen

ऑकसीजनकहॳ सािशवसन

3) Respiration without CO2

काबयनिायऑकसाइिकहॳ बबनाशवसन

4) Respiration with CO2

काबयनिायऑकसाइिकहॳ सािशविन

Correct Answer Respiration without

Oxygen

Q180 Which type of pathogen causes

the waterborne disease Cholera

ककसपरकारकारह८गजनकजिजननतरह८गहहॴजाकाकारणबनताहहॴ

30-Jan-2017

Options

1) Algal

िहॴवालियहॳ

F A C E B O O K

P A G E h t t p w w w f a c e b o o k c o m s s c m e n t o r s o f f i c i a l P a g e | 42

FOR MORE UPDATES AND MORE MATERIAL DO LIKE OUR FACEBOOK PAGE httpwwwfacebookcomsscmentorsofficial

2) Bacterial

बहॴकटीररयि

3) Protozoan

परह८टह८जआ

4) Viral

वायरि

Correct Answer Bacterial

Q181 To which class does

Oxyreductases transferases hydrolases

belong

ओकसीररिकटहॳसटरासफरहॳजहॳस

हाइडरह८िहॳसहॳसककसवगयमआतहॳहहॴ 30-Jan-2017

Options

1) Hormones

हारमोस

2) Enzymes

एजाइनस

3) Proteins

परह८टीनस

4) Vitamins

पवटालमनस

Correct Answer Enzymes

Q182 Which of the following is not true

about Gymnosperms

ननननमसहॳकह९नसीबातअनावतबीजीकहॳ बारहॳमसचनहीहहॴ 30-Jan-2017

Options

1) Dominant phase is saprophytes

परमिचरणसहॳपरह८फाइटसहह८ताहहॴ 2) Vascular bundles are absent

सवहनीबििअनपनसितहह८ताहहॴ 3) spores are heterospores

बीजाणहहॳटहॳरह८सपह८रसहह८तहॳहहॴ 4) Flowers are absent

फिअनपनसितहह८तहॳहहॴ

Correct Answer Vascular bundles are

absent

Q183 The name of first mammal clone sheep is

भहॳड़कीपरिमसतनपायीपरनत प (किह८न)

कानामहहॴ 30-Jan-2017

Options

1) Noori

नरी 2) Dolly

िॉिी 3) Louise

िसी 4)Durga

दगाय Correct Answer Dolly

Q184 Which type of pathogen causes

the water-borne disease Typhoid fever

ककसपरकारकारह८गजनकजिजननतरह८गटाइफाइिबिारकाकारणबनताहहॴ 30-Jan-2017

Options

1) Algal

िहॴवािीय

2) Parasitic

परजीवी 3) Protozoan

परह८टह८जनअन

4)Bacterial

बहॴकटीररयि

Correct Answer Bacterial

Q185 In which part of the cell are

proteins made

कह८लिकाकहॳ ककसटहससहॳमपरह८टीनबनायाजाताहहॴ

31-Jan-2017

Options

1) Reticulum

रहॳटटकिम

F A C E B O O K

P A G E h t t p w w w f a c e b o o k c o m s s c m e n t o r s o f f i c i a l P a g e | 43

FOR MORE UPDATES AND MORE MATERIAL DO LIKE OUR FACEBOOK PAGE httpwwwfacebookcomsscmentorsofficial

2) Golgi apparatus

गह८मजीएपहॳरहॳटस

3) Ribosomes

ररबह८सह८नस

4) Lysosome

िायसह८सह८नस

Correct Answer ribosomes

Proteins are produced by stringing

amino acids together in the order

specified by messenger RNA strands

that were transcribed from DNA in the

cell nucleus The process of synthesizing

a protein is called translation and it

occurs on ribosomes in the cytoplasm of

a cell

Q186 Polio is a disease caused by which

of the following

नननननलिखितमसहॳपह८लियह८कीबबमारह८हह८नहॳकाकारणकयाहहॴ

31-Jan-2017

Options

1) Bacteria

बहॴकटीररयि

2) Mosquito

मचछर

3) Virus

वायरस

4) Cockroach

नतिच हॳ Correct Answer Virus

Polio or poliomyelitis is a crippling and

potentially deadly infectious disease It

is caused by the poliovirus

Q187 ndash Hay fever is a sign of which of

the following

हहॳकफवरनननननलिखितमसहॳककसकाएकसकहॳ तहहॴ

31-Jan-2017

Options

1) Old Age

वदावसिा 2) Malnutrition

कपह८सण

3) Allergy

एिनजय 4) Over Work

अतयचधककाययकरना Correct Answer Allergy

Q188 How many chromosomes does a

human cell contain

एकमानवकह८लिकामककतनहॳगणसतरहह८तहॳहहॴ

29-Jan-2017

Options

1) 6

2) 26

3) 46

4) 66

Correct Answer 46

In humans each cell normally contains

23 pairs of chromosomes for a total of

46 Twenty-two of these pairs called

autosomes look the same in both males

and females The 23rd pair the sex

chromosomes differ between males and

females

Q189 Which of the following is not true

about Bryophyta

ननननमसहॳकह९नसीबातिायह८फाइटकहॳ बारहॳमसचनहीहहॴ 31-Jan-2017

Options

1) Dominant phase is gametophytes

परमिचरणगहॳलमतह८फाइटसहह८ताहहॴ 2) Main plant body is haploid

पह९धहॳकामखयिरीरअगखणतहह८ताहहॴ 3) Spores are homospores

बीजाणहह८मह८सफह८रसहह८तहॳहहॴ 4) Flowers are present

फिमह८जदहह८तहॳहहॴ Correct Answer Flowers are present

F A C E B O O K

P A G E h t t p w w w f a c e b o o k c o m s s c m e n t o r s o f f i c i a l P a g e | 44

FOR MORE UPDATES AND MORE MATERIAL DO LIKE OUR FACEBOOK PAGE httpwwwfacebookcomsscmentorsofficial

Q190 Which aquatic animal has

trailing tentacles

ककसजिीयजानवरकहॳ पीछहॳचिनहॳवािहॳटहॳटकिसहह८तहॳहहॴ

31-Jan-2017

Options

1) Sea horse

समदरीघह८िा 2) Corals

मगा 3) Jelly fish

जहॳिीमछिी 4) Star fish

तारामछिी Correct Answer Jelly fish

Jellyfish with its umbrella-shaped bell

and trailing tentacles

Q191 Which type of pathogen causes

the water-borne disease Poliomyelitis

(Polio)

ककसपरकारकारह८गजनकजिजननतरह८गपह८लियह८मायहॳटटस (पह८लियह८) काकारणहहॴ 31-Jan-

2017

Options

1) Parasitic

परजीवी 2) Algal

िहॴवालिय

3) Viral

वायरि

4) Bacterial

बहॴकटीररयि

Correct Answer Viral

Q192 The outer white part of the eye

that protects the inner structures is

आािकाबाहरीसफहॳ दटहससाजह८आतररकसरचनाओकीरकषाकरताहहॴ वह mdashmdashmdash हहॴ 31-Jan-

2017

Options

1) Iris

आयररस

2) Sclera

सकिहॳरा 3) Retina

रहॳटटना 4) Cornea

कह८ननयया Correct Answer Sclera

Q193 Proteins are made up of

परह८टीनकाननमायण mdashndash सहॳहह८ताहहॴ 31-Jan-2017

Options

1) Amino acids

एलमनह८अनि

2) Fatty acids

वसायकतअनि

3) Glucose

गिकह८ज

4)Nucleotides

नयनकियह८टाईिस

Correct Answer Amino acids

Q194 Moringa Oleifera is the scientific

name of

मह८ररगओलिफहॳ रा mdashmdashndash कावहॴजञाननकनामहहॴ 31-Jan-2017

Options

1) Banyan

बरगद

2) Gulmohar

गिमह८हर

3) Amla

आमिा

F A C E B O O K

P A G E h t t p w w w f a c e b o o k c o m s s c m e n t o r s o f f i c i a l P a g e | 45

FOR MORE UPDATES AND MORE MATERIAL DO LIKE OUR FACEBOOK PAGE httpwwwfacebookcomsscmentorsofficial

4) Drumstick

डरमनसटक

Correct Answer Drumstick

Q195 Kidney stones are composed of

गदकीपिरी mdashndash सहॳबनीहह८तीहहॴ 1-Feb-2017

Options

1) Calcium Oxalate

कहॴ नमसयमओकजहॳिहॳट

2) Sodium Chloride

सह८डियमकिह८राइि

3) Magnesium Nitrate

महॳनगनलियमनाइतटरहॳट

4) Calcium Bicarbonate

कहॴ नमियमबायकबोनहॳट

Correct Answer Calcium Oxalate

Q196 ndash Which of the following is not

true about Angiosperms

ननननमसहॳकह९नसीबातआवतबीजीकहॳ बारहॳमसचनहीहहॴ 1-Feb-2017

Options

1) Dominant phase is gametophytes

परमिचरणगहॳलमतह८फाइटहह८ताहहॴ 2) Vascular bundles are present

सवहनीबििमह९जदहह८ताहहॴ 3) Spores are heterospores

बीजाणहहॳटहॳरह८सपह८रसहह८तहॳहहॴ 4) Seeds are covered

बीजढकहॳ हह८तहॳहहॴ Correct Answer Dominant phase is

gametophytes

Q197 All of the following are excretory

(waste) products of animals except

नननननलिखितमसहॳककसएककह८छह८ड़करअनयसभीपराखणयोदवाराउतसनजयतपदाियहहॴ 1-Feb-

2017

Options

1) Uric Acid

यररकएलसि

2) Ammonia

अमह८ननया 3) Carbohydrates

काबोहाइडरहॳट

4) Urea

यररया Correct Answer Carbohydrates

In animals the main excretory products

are carbon dioxide ammonia (in

ammoniotelics) urea (in ureotelics) uric

acid (in uricotelics) guanine (in

Arachnida) and creatine

Q198 RNA is a polymeric molecule

What does RNA stand for

आरएनइएएकबहिकआणहहॴ इसकाकापवय पकयाहहॴ 1-Feb-2017

Options

1) Rado Nuclear Acid

रािह८नयनकियरएलसि

2) Ribo Nucleic Acid

राइबह८नयनकिकएलसि

3) Rhino Nuclear Acid

हाइनह८नयनकियरएलसि

4) Resto Nucleus Acid

रहॳसटह८नयकिीयसएलसि

Correct Answer Ribo Nucleic Acid

Q199 Which organ does detoxification

and produces chemicals needed for

digestion

कह९नसाअगपवषहरणकरताहहॴऔरपाचनकहॳ लिएआवशयकरसायनोकह८पहॴदाकरताहहॴ 1-Feb-

2017

Options

1) Salivary glands

िारगरचिया 2) Pancreas

अगनयािय

F A C E B O O K

P A G E h t t p w w w f a c e b o o k c o m s s c m e n t o r s o f f i c i a l P a g e | 46

FOR MORE UPDATES AND MORE MATERIAL DO LIKE OUR FACEBOOK PAGE httpwwwfacebookcomsscmentorsofficial

3) Thyroid gland

िायराइिगरिी 4) Liver

यकत

Correct Answer Liver

Q200 Psidium guajava is the scientific

name of

लसडियमगआजावा mdashmdash कावहॴजञाननकनामहहॴ 1-

Feb-2017

Options

1) Guava

अम द

2) Mango

आम

3) Bamboo

बास

4) Jack fruit

कटहि

Correct Answer Guava

Q201 Which drug is used as a Blood

Thinner

चधरकह८पतिाकरनहॳकहॳ पमककसदवाकापरयह८गककयाजाताहहॴ

1-Feb-2017

Options

1) Warfarin

वाफर न

2) Tramadol

टरहॳमािह८ि

3) Azithromycin

एनजरह८मायलसन

4) Hydralazine

हाइडरह८िहॳनजन

Correct Answer Warfarin

Q202 Which of the following disease is

caused due to the deficiency of protein

परह८टीनकीकमीकहॳ कारणनननननलिखितमसहॳकह९नसारह८गहह८ताहहॴ 1-Feb-2017

Options

1) Arthritis

गटठया 2) Kwashiorkor

कािीओकय र

3) Goitre

गाइटर

4) Night Blindness

रतह९चध

Correct Answer Kwashiorkor

Q203 A is species of plant that has

adapted to survive in an environment

with little liquid water

mdashmdashndashपह९धहॳकीएकऐसहॳऐसहॳपरजानतहहॴ नजसनहॳकमपानीवािहॳवातावरणमजीपवतरहनहॳकहॳलिएअनकिनहहॴ 1-Feb-2017

Options

1) Xerophyte

म दपवद

2) Hydrophyte

जिीयपादप

3) Mesophyte

समह८दपवद

4) Thallophyte

िहॴिह८फाइटा Correct Answer xerophyte

xerophyte is a species of plant that has

adapted to survive in an environment

with little liquid water such as a desert

or an ice- or snow-covered region in the

Alps or the Arctic

Mesophytes are terrestrial plants which

are adapted to neither a particularly

dry nor particularly wet environment

An example of a mesophytic habitat

would be a rural temperate meadow

F A C E B O O K

P A G E h t t p w w w f a c e b o o k c o m s s c m e n t o r s o f f i c i a l P a g e | 47

FOR MORE UPDATES AND MORE MATERIAL DO LIKE OUR FACEBOOK PAGE httpwwwfacebookcomsscmentorsofficial

which might contain goldenrod clover

oxeye daisy and Rosa multiflora

thallophyte any of a group of plants or

plantlike organisms (such as algae and

fungi) that lack differentiated stems

leaves and roots and that were formerly

classified as a primary division

(Thallophyta) of the plant kingdom

Q204 How many types of teeth are

there in humans

मनषयोमककतनहॳपरकारकहॳ दातहह८तहॳहहॴ

1-Feb-2017

Options

1) 4

2) 5

3) 2

4) 3

Correct Answer 4

teeth -Humans have four types of

teethincisors canines premolars and

molars each with a specific function

The incisors cut the food the canines

tear the food and the molars and

premolars crush the food

Q205 Carica papaya is the scientific name of

कहॴ ररकापपाया mdashmdashndash कावहॴजञाननकनामहहॴ 2-

Feb-2017

Options

1) Peepal

पीपि

2) Papaya

पपीता 3) Tamarind

इमिी 4) Drumstick

ढह८िकाछड़ी Correct Answer Papaya

Q206 Muscles get tired when there is

shortfall of

जब mdashndash कीकमीहह८तीहहॴतबपहॳिीयिकजातीहहॴ 2-Feb-2017

Options

1) Lactic acid

िहॴनकटकएलसि

2) Na+ ions

Na+ आयन

3) ATP

एटीपी 4) Sulphates

समफहॳ टस

Correct Answer ATP

ATP is the energy source muscle fibers

use to make muscles contract

muscle tissuersquos main source of energy

called adenosine triphosphate or ATP

As your muscles use up this energy

source they become tired and fatigued

Oxygen is the key ingredient that helps

create new ATP to replenish the burned

up ATP in your muscles

Q207 Artocarpus integra is the

scientific name of आटह८कापयसइटीगरा mdashmdashmdash कावहॴजञाननकनामहहॴ 2-Feb-2017

Options

1) Guava

अम द

2) Pineapple

अनानास

3) Silver Oak

लसमवरओक

4) Jack fruit

कटहि

Correct Answer Jack fruit

Q208 Which organ stores fat soluble

vitamins

कह९नसाअगवसामघिनिीिपवटालमनह८काभिाराकरताहहॴ

2-Feb-2017

F A C E B O O K

P A G E h t t p w w w f a c e b o o k c o m s s c m e n t o r s o f f i c i a l P a g e | 48

FOR MORE UPDATES AND MORE MATERIAL DO LIKE OUR FACEBOOK PAGE httpwwwfacebookcomsscmentorsofficial

Options

1) Blood

रकत

2) Skin

तवचा 3) Liver

यकत

4) Pancreas

अगनयािय

Correct Answer Liver

Q209 Which disease is caused due to

deficiency of Iodine

आयह८िीनकहॳ कारणकह९नसारह८गहह८ताहहॴ 2-Feb-2017

Options

1) Rickets

ररकहॳ टस

2) Scurvy

सकवी 3) Goitre

गणमािा 4) Growth retardation

पवकासका कना Correct Answer Goitre

rickets A softening and weakening of

bones in children usually due to

inadequate vitamin D

Q210 Grevillea Robusta is the scientific name of

गरहॳपवलियारह८बसटा mdashmdashmdash- कापवजञाननकनामहहॴ 2-Feb-2017

Options

1) Peepal

पीपि

2) Teak

सागह९न

3) Silver Oak

लसमवरओक

4) Jack fruit

कटहि

Correct Answer Silver Oak

Q211 When a Cuttlefish is described as a Molluscs it is at which level of

classification

जबएककटिकफिकह८एकमह८िसकाकहॳ पमवखणयतककयाजाताहहॴतबयहॳवगीकरणकहॳ ककससतरपहॳनसितहहॴ 2-Feb-2017

Options

1) Class

वगय 2) Order

िम

3) Family

पररवार

4) Phylum

सघ

Correct Answer Phylum

Q212 Bambusa dendrocalmus is the

scientific name of बानबसािहॳडराकामस mdashmdashmdash कावहॴजञाननकनामहहॴ 3-Feb-2017

Options

1) Banyan

बरगद

2) Papaya

पपीता 3) Bamboo

बास

4) Pomegranate

अनार

Correct Answer Bamboo

Q213 Acinonyx Jubatus is the scientific name of

एलसनह८ननकसजयबहॳटस mdashmdashmdash

कावहॴजञाननकनामहहॴ 3-Feb-2017

F A C E B O O K

P A G E h t t p w w w f a c e b o o k c o m s s c m e n t o r s o f f i c i a l P a g e | 49

FOR MORE UPDATES AND MORE MATERIAL DO LIKE OUR FACEBOOK PAGE httpwwwfacebookcomsscmentorsofficial

Options

1) Bear

भाि 2) Horse

घह८िा 3) Cheetah

चीता 4) Zebra

जहॳिा Correct Answer Cheetah

Q214 The pale yellow colour of urine is

due to the presence of which pigment

मतरकाफीकापीिारगरगदरयकहॳ उपनसिनतकहॳ कारणहह८ताहहॴ

3-Feb-2017

Options

1) Urochrome

यरह८िह८म

2) Urophyll

यरह८कफि

3) Chlorophyll

किह८रह८कफि

4) Chloroplast

किह८रह८पिासट

Correct Answer Urochrome

Q215 Which of the following constitute

to form a gene

नननननलिखितमसहॳकह९नसीचीज़एकजीनकागठनकरतीहहॴ

3-Feb-2017

Options

1) Polynucleotides

पह८िीनयनकियह८टाईडस

2) Hydrocarbons

हाइडरह८काबोस

3) Lipoproteins

िाईपह८परह८टीनस

4) Lipids

लिपपडस

Correct Answer Polynucleotides

Polynucleotide molecule is a biopolymer

composed of 13 or more nucleotide

monomers covalently bonded in a chain

DNA (deoxyribonucleic acid) and RNA

(ribonucleic acid) are examples of

polynucleotides with distinct biological

function

Q216 Vertebrates belongs to the

phylum

रीढ़कीहडिीवािहॳपराणी mdashmdashmdash

परजानतकहॳ अतगायतआतहॳहहॴ 3-Feb-2017

Options

1) Arthropoda

आरह८पह८ड़ा 2) Annelida

एननलििा 3) Cnidaria

ननिहॳररया 4) Chordata

कह८िटा Correct Answer Chordata

Q217 Punica granatum is the scientific name of

पननकगरहॳनहॳटस mdashmdashmdash कावहॴजञाननकनामहहॴ 3-Feb-2017

Options

1) Custard Apple

सीताफि

2) Gulmohar

गिमह८हर

3) Silver Oak

लसमवरओक

4) Pomegranate

अनार

Correct Answer Pomegranate

F A C E B O O K

P A G E h t t p w w w f a c e b o o k c o m s s c m e n t o r s o f f i c i a l P a g e | 50

FOR MORE UPDATES AND MORE MATERIAL DO LIKE OUR FACEBOOK PAGE httpwwwfacebookcomsscmentorsofficial

Q218 Between a tiger and an monkey

which of the following is different

एकबाघऔरबदरकहॳ बीचनननननलिखितमसहॳकह९नसीबातअिगहहॴ 3-Feb-2017

Options

1) Kingdom

राजय

2) Phylum

जानत

3) Order

िम

4) Class

वगय Correct Answer order

Q219 The artificial heart was invented by

कबतरमहदयका mdashmdashmdash

दवाराअपवषकारककयागयािा 3-Feb-2017

Options

1) Muhammad Yunus

महनमदयनस

2) Linus Yale Jr

िाइनसयहॳिजय

3) Gazi Yasargil

गाजीयासचगयि

4) Paul Winchell

पह९िपवमकि Correct Answer Paul Winchell

Q220 Tamarindus indica is the

scientific name of

टहॳमररनडसइडिका mdashmdash कावहॴजञाननकनामहहॴ 7-

Feb-2017

Options

1) Neem

नीम

2) Pineapple

अनानास

3) Tamarind

इमिी 4)Chiku

चीक

Correct Answer Tamarind

Q221 In eukaryotic cells synthesis of

RNA takes place in the

यकहॳ योटटककह८लिकाओमआरएनएकासशिहॳषण

mdashndash महह८ताहहॴ 7-Feb-2017

Options

1) Mitochondria

माईटह८कोडडरया 2) Centrioles

सटरीयह८मस

3) Ribosomes

ररबह८सह८नस

4) Nucleus

नयनकियस

Correct Answer nucleus

eukaryotic cell -Transcription is the

process of synthesizing ribonucleic acid

(RNA)Synthesis takes place within the

nucleus of eukaryotic cells or in the

cytoplasm of prokaryotes and converts

the genetic code from a gene in

deoxyribonucleic acid ( DNA ) to a

strand of RNA that then directs

proteinsynthesis

Q222 _________is caused by parasites

of the Plasmodium genus

पिाजमह८डियमजातीकहॳ परजीवी mdash- कहॳ कारणहहॴ 7-Feb-2017

Options

1) Dysentery

पहॳचचि

2) Malaria

मिहॳररया 3) Chickenpox

F A C E B O O K

P A G E h t t p w w w f a c e b o o k c o m s s c m e n t o r s o f f i c i a l P a g e | 51

FOR MORE UPDATES AND MORE MATERIAL DO LIKE OUR FACEBOOK PAGE httpwwwfacebookcomsscmentorsofficial

चहॳचक

4) Herpes

हहॳपपयस

Correct Answer Malaria

Q223 Carotene in fruits and vegetables

gives it which color

फिह८औरसनलजयोमनसितकहॳ रह८टीनउनहकह९नसारगपरदानकरताहहॴ 7-Feb-2017

Options

1) Green

हरा 2) Pink

गिाबी 3) Orange

नारगी 4) Blue

नीिा Correct Answer Orange

Q224 Equus Caballus is the scientific

name of

एकवसकहॴ बहॳिस mdashmdashndash कापवजञाननकनामहहॴ 7-Feb-2017

Options

1) Horse

घह८िा 2) Zebra

ज़हॳिा 3) Donkey

गधा 4) Buffalo

भस

Correct Answer Horse

Q225 Elapidae Naja is the scientific name of

एिीपीिीनाजा mdashmdash- कावहॴजञाननकनामहहॴ 8-Feb-2017

Options

1) Cobra

कह८बरा 2) Elephant

हािी 3) Eagle

ग ि

4) Owl

उमि Correct Answer Cobra

Q226 Which disease is caused due to

deficiency of Iron

िह८हकीकमीकहॳ कारणकह९नसारह८गहह८ताहहॴ 8-Feb-

2017

Options

1) Beriberi

बहॳरीबहॳरी 2) Tetany

टहॳटनी 3) Kwashiorkor

कवािीऔरकर

4) Anaemia

रकतामपता Correct Answer Anaemia

Beriberi is a disease caused by a vitamin

B-1 deficiency also known as thiamine

deficiency

Tetany can be the result of an

electrolyte imbalance Most often itrsquos a

dramatically low calcium level also

known as hypocalcemia Tetany can also

be caused by magnesium deficiency or

too little potassium Having too much

acid (acidosis) or too much alkali

(alkalosis) in the body can also result in

tetany

Kwashiorkor also known as

ldquoedematous malnutrition It is a form of

malnutrition caused by a lack of protein

in the diet

Anaemia means that you have fewer red

blood cells than normal or you have less

F A C E B O O K

P A G E h t t p w w w f a c e b o o k c o m s s c m e n t o r s o f f i c i a l P a g e | 52

FOR MORE UPDATES AND MORE MATERIAL DO LIKE OUR FACEBOOK PAGE httpwwwfacebookcomsscmentorsofficial

haemoglobin than normal in each red

blood cell

Q227 is a leaf where the leaflets are

arranged along the middle vein

mdashndashएकपततीहहॴजहापतरकह८कीरचनाक ररयालिराकहॳ आसपासहह८तीहहॴ 8-Feb-2017

Options

1) Pinnately compound leaf

पपनहॳटिीसयकतपतती 2) Palmately compound leaf

पामहॳटिीसयकतपतती 3) Compound leaf

सयकतपतती 4) Simple leaf

साधारणपतती Correct Answer Pinnately compound

leaf

Q228 Haustoria or sucking roots are

found in which of the following

हह८सटह८ररयायाचसनहॳवािीजड़हॳनननननलिखितमसहॳककसमपाईजातीहहॴ 8-Feb-2017

Options

1) Wheat

गहॳह

2) Mango

आम

3) Chestnut

चहॳसटनट

4) Cuscuta

कसकयटा Correct Answer Cuscuta

Haustorial roots -The roots of parasitic

plants which penetrate into the host

tissues to absorb nourishment are

called haustorial roots hellip Also known as suckingor parasitic roots

Q229 Equs Asinus is the scientific name

of

एकवसएलसनस mdashmdashndash कावहॴजञाननकनामहहॴ 8-

Feb-2017

Options

1) Donkey

गधा 2) Cow

गाय

3) Deer

टहरन

4) Kangaroo

कगा

Correct Answer Donkey

Q230 Ficus benghalensis is the scientific name of

फाईकसबहॳनगहॳिहॳलसस mdashndash कापवजञाननकनामहहॴ 8-Feb-2017

Options

1) Banyan

बरगद

2) Pineapple

अनानास

3) Babul

बबि

4) Tulsi

तिसी Correct Answer Banyan

Q231 Equus burchellii is the scientific name of

एकवसबचिी mdashmdash- कापवजञाननकनामहहॴ 8-Feb-2017

Options

1) Horse

घह८िा 2) Zebra

जहॳिा 3) Buffalo

F A C E B O O K

P A G E h t t p w w w f a c e b o o k c o m s s c m e n t o r s o f f i c i a l P a g e | 53

FOR MORE UPDATES AND MORE MATERIAL DO LIKE OUR FACEBOOK PAGE httpwwwfacebookcomsscmentorsofficial

भस

4) Ass

गधा Correct Answer Zebra

Page 29: COMPILATION OF ALL 72 SETS OF BIOLOGY SSC CHSL-2016 · OF BIOLOGY SSC CHSL-2016 PREPARED BY : SSC MENTORS BIOLOGY SPECIAL . F A C E B O O K P A G E : h t t p : / / w w w . f a c e

F A C E B O O K

P A G E h t t p w w w f a c e b o o k c o m s s c m e n t o r s o f f i c i a l P a g e | 28

FOR MORE UPDATES AND MORE MATERIAL DO LIKE OUR FACEBOOK PAGE httpwwwfacebookcomsscmentorsofficial

Q117 Ferns belong to which division of

plants

फनसयपह९धह८कहॳ ककसभागमआतहॳहहॴ

22-Jan-2017

Options

1) Gymnosperms

नजननह८सपनसय 2) Angiosperms

एनजयह८सपनसय 3) Thallophyta

िहॴिह८फाईटा 4)Pteridophyta

टहॳररिह८फाईटा Correct Answer Pteridophyta

Q118 Who invented Antibiotics

एटीबायह८टटककाअपवषकारककसनहॳककयािा

22-Jan-2017

Options

1) Joseph Lister

जह८सहॳफलिसटर

2) William Harvey

पवलियमहाव

3) Robert Knock

रॉबटयनॉक

4)Alexander Fleming

अिहॳकज़िरफिहॳलमग

Correct Answer Alexander Fleming

Q119 Milbecycin is used in the

eradication of

लममबहॳसायलसनका mdashndash

मउनमिनमपरयह८गककयाजाताहहॴ 22-Jan-2017

Options

1) Agricultural Fungus

कपषकवक

2) Agricultural Pests

कपषकीटक

3) Agricultural Herbs

कपषिाक

4)Agricultural Weeds

कपषननराना Correct Answer Agricultural Pests

Milbemycin oxime is a veterinary drug

from the group of milbemycins used as

a broad spectrum antiparasitic It is

active against worms and mites(insects

Q120 Intestinal bacteria synthesizes

which of the following in the human

body

मानविरीरमआतोकहॳ बहॴकटीररयानननननलिखितमसहॳककसकासशिहॳषणकरतहॳहहॴ 22-Jan-2017

Options

1) Vitamin K

पवटालमन K

2) Proteins

परह८टीन

3) Fats

वसा 4) Vitamin D

पवटालमन D

Correct Answer Vitamin K

Q121 is the study of the physical form

and external structure of plants

mdashmdash-

मपह९धह८काभहॴनतक पऔरबाहरीसरचनाकाआदयाककयाजाताहहॴ 22-Jan-2017

Options

1) Physiology

कफनजयह८िह८जी 2) Anatomy

िरीररचनापवजञान

3) Phytomorphology

फाईटह८मह८फह८िह८जी 4)Cytology

कह८लिकापवजञान

Correct Answer Phytomorphology

F A C E B O O K

P A G E h t t p w w w f a c e b o o k c o m s s c m e n t o r s o f f i c i a l P a g e | 29

FOR MORE UPDATES AND MORE MATERIAL DO LIKE OUR FACEBOOK PAGE httpwwwfacebookcomsscmentorsofficial

Q122 Which of the following is a

structural and functional unit of

kidneys

नननननलिखितमसहॳकह९नसीगदोकीसरचनातमकऔरकाययकरीईकाईहहॴ

22-Jan-2017

Options

1) Renette Cells

रहॳनहॳटकह८लिकाए

2) Flame Cells

फिहॳमकह८लिकाए

3) Nephrites

नहॳफ़राइटस

4)Nephrons

नहॳफरोस

Correct Answer Nephrons

Nephron functional unit of the kidney

the structure that actually produces

urine in the process of removing waste

and excess substances from the blood

There are about 1000000 nephrons in

each human kidney

Q123 Which of the following is the

largest part of the human brain

नननननलिखितमसहॳकह९नसामानवमनसतषककासबसहॳबड़ाटहससाहहॴ

23-Jan-2017

Options

1) Ribs

पसलियाा 2) Cerebrum

सहॳरहॳिम

3) Pons

पोस

4)Thalamus

िहॴिहॳमस

Correct Answer Cerebrum

The cerebrum is the largest part of the

human brain making up about two-

thirds of the brainrsquos mass It has two

hemispheres each of which has four

lobes frontal parietal temporal and

occipital

Q124 The auxiliary buds

सहायककालियाmdashndash 23-Jan-2017

Options

1) grow endogenously from the pericycle

पहॳरीसाईककिसहॳअनतजातयपवकलसतहह८ताहहॴ 2) arise endogenously from the main

growing point

मिवपदसहॳअनतजातयउठताहहॴ 3) is an embryonic shoot located in the

axil of a leaf

एकभरणिटहहॴजह८एकपततीकहॳ अकषपरनसतिहह८ताहहॴ 4)arise exogenously from the epidermis

एपपिलमयससहॳबटहजातयतरीकहॳ सहॳउठताहहॴ Correct Answer is an embryonic shoot

located in the axil of a leaf

Q125 Which of the following is a viral

disease

इनमहॳसहॳकह९सीएकवायरिबीमारीहहॴ

23-Jan-2017

Options

1) Polio

पह८लियह८ 2) Tetanus

धनसतनभ

3) Leprosy

कषठरह८ग

4) Plague

पिहॳग

Correct Answer Polio

A viral disease (or viral infection)

occurs when an organismrsquos body is

invaded by pathogenic viruses and

infectious virus particles (virions) attach

to and enter susceptible cells

F A C E B O O K

P A G E h t t p w w w f a c e b o o k c o m s s c m e n t o r s o f f i c i a l P a g e | 30

FOR MORE UPDATES AND MORE MATERIAL DO LIKE OUR FACEBOOK PAGE httpwwwfacebookcomsscmentorsofficial

Poliomyelitis often called polio or

infantile paralysis is an infectious

disease caused by the poliovirus

Tetanusmdash A serious bacterial infection

that causes painful muscle spasms and

can lead to death

Leprosy also known as Hansenrsquos

disease (HD) is a long-term infection by

the bacterium Mycobacterium leprae or

Mycobacterium lepromatosis

Plague is an infectious disease caused by

the bacterium Yersinia pestis

Symptoms include fever weakness and

headache

Q126 Which organisms can help to

carry out Vermicomposting

कह९नसाजीववमीकनपह८नसटगममददकरताहहॴ

23-Jan-2017

Options

1) Nitrifying Bacteria

नाईटरीफाईगबहॴकटीररया 2) Earthworms

कहॴ चऐ

3) Algae

िहॴवि

4) Fungus

कवक

Correct Answer Earthworms

Q127 Contraction of heart is also

known as

हदयकहॳ सकचनकह८ mdash- भीकहाजाताहहॴ 23-Jan-

2017

Options

1) Systole

लससटह८ि

2) Aristotle

अरसत

3) Diastole

िायसटह८ि

4) Lub

मयब

Correct Answer Systole

Diastole is the part of the cardiac cycle

when the heart refills with blood

following systole (contraction)

Ventricular diastole is the period during

which the ventricles are filling and

relaxing while atrial diastole is the

period during which the atria are

relaxing

Q128 Azadirachta indica is the

botanical name of which of the

following

अजाटदराचताइडिकानननननलिखितमसहॳककसकावानसपनतनामहहॴ

23-Jan-2017

Options

1) Rose plant

गिाबकापह९धा 2) Apple tree

सहॳबकापहॳड़

3) Neem

नीम

4)Mango

आम

Correct Answer Neem

Q129 Which of the following is the

main end product of carbohydrate

digestion

नननननलिखितमसहॳकह९नसाकाबोहाइडरहॳटकहॳ पाचनकापरमिअतउतपादकहह८ताहहॴ 23-Jan-2017

Options

1) Fats

वसा 2) Lipids

लिपपडस

3) Glucose

गिकह८ज

4) Cellulose

F A C E B O O K

P A G E h t t p w w w f a c e b o o k c o m s s c m e n t o r s o f f i c i a l P a g e | 31

FOR MORE UPDATES AND MORE MATERIAL DO LIKE OUR FACEBOOK PAGE httpwwwfacebookcomsscmentorsofficial

सहॳमयिह८ज

Correct Answer Glucose

Intestinal absorption of end products

from digestion of carbohydrates and

proteins in the pig hellip During absorption some sugars (fructose or

galactose) released from the

corresponding sucrose and lactose

respectively during digestion were

partly metabolized into glucose by the

enterocyte

Q130 Which of the following glands is a

source of the enzyme Ptyalin

नननननलिखितगरचियोमसहॳएजाइमटयालिनकासरह८तहहॴ 23-Jan-2017

Options

1) Pancreas

अगरािय

2) Thyroid Gland

िाइराइिगरिी 3) Pituitary Gland

पीयषगरिी 4) Salivary Glands

िारगरचियाा Correct Answer Salivary Glands

Q131 Which of the following is not true

about Pteridophyta

ननननमसहॳकह९नसीबातटहॳररिह८फाईटकहॳ बारहॳमसचनहीहहॴ 23-Jan-2017

Options

1) Dominant phase is saprophytes

परमिचरणसहॳपरह८फाईइटसहह८ताहहॴ 2) Main plant body is diploid

पह९दह८कामखयिरीरदपवगखणतहह८ताहहॴ 3) Seeds are present

बीजमह९जदहह८तहॳहहॴ 4)Flowers are absent

फिअनपनसतिहह८तहॳहहॴ

Correct Answer Seeds are present

Q132 The largest dolphin species is the

orca also called as

िॉिकफनकीसबसहॳबड़ीपरजानतकाकानामआकायहहॴनजसहॳ mdash- भीकहतहॳहहॴ 23-Jan-2017

Options

1) Bottle Nose

बाटिनह८ज

2) Baiji

बहॳजी 3) Killer whale

ककिरहहॳि

4)Tucuxi

टकवसी Correct Answer Killer whale

Q133 The fat digesting enzyme Lipase

is secreted by which of the following

वसाकापाचनकरनहॳवािाएजाइमिाइपहॳजनननननलिखितमसहॳककसकहॳ दवारासतरापवतहह८ताहहॴ

24-Jan-2017

Options

1) Kidneys

गद

2) Pancreas

अगनयािय

3) Large Intestine

बड़ीआत

4)Liver

नजगर

Correct Answer Pancreas

Lipase is an enzyme that splits fats so

the intestines can absorb them Lipase

hydrolyzes fats like triglycerides into

their component fatty acid and glycerol

molecules It is found in the blood

gastric juices pancreatic secretions

intestinal juices and adipose tissues

F A C E B O O K

P A G E h t t p w w w f a c e b o o k c o m s s c m e n t o r s o f f i c i a l P a g e | 32

FOR MORE UPDATES AND MORE MATERIAL DO LIKE OUR FACEBOOK PAGE httpwwwfacebookcomsscmentorsofficial

Q134 The arrangement of leaves on an

axis or stem is called

एकअकषयातनहॳपरपनततयोकीयवसिाकह८कयाकहाजाताहहॴ SSC CHSL Science (biology) 2016

Question Paper

24-Jan-2017

Options

1) Phyllotaxy

फाइिह८टहॴकसी 2) Vernation

वनिन

3) Venation

वहॳनहॳिन

4)Phytotaxy

फाइटह८टहॴकसी Correct Answer Phyllotaxy

In botany phyllotaxis or phyllotaxy is

the arrangement of leaves on a plant

stem (from Ancient Greek phyacutellon

ldquoleafrdquo and taacutexis ldquoarrangementrdquo)

Phyllotactic spirals form a distinctive

class of patterns in nature

Q135 The study of Cells is also known

as

कह८लिकाओकहॳ अधययनकह८ mdashmdashndash

भीकहाजाताहहॴ 24-Jan-2017

Options

1) Cytology

सायटह८िह८जी 2) Physiology

कफनजयह८िह८जी 3) Nucleology

नयककमयह८िह८जी 4)Cellology

सहॳिह८िह८जी Correct Answer Cytology

Q136 Which of the following scientists

is also known as the Father of Biology

नननननलिखितमसहॳककसवहॴजञाननककह८ ldquoजीवपवजञानकहॳ जनकrdquoकहॳ नामसहॳभीजानाजाताहहॴ 24-Jan-2017

Options

1) Herbert Spencer

हबयटयसपसर

2) Aristotle

अरसत 3) Lamarck

िहॳमाकय 4)Darwin

िापवयन

Correct Answer Aristotle

Q137 Which cells give rise to various

organs of the plant and keep the plant

growing

कह९नसीकह८लिकाएपह९धह८कहॳ लभननअगह८कह८जनमदहॳतीहहॴऔरपह९धह८कह८बढ़नहॳममददकरतीहहॴ

24-Jan-2017

Options

1) Permanent

सिायी 2) Dermal

तवचीय

3) Meristematic

मररसटहॳमटटक

4)Mature

परह८ढ़

Correct Answer Meristematic

A meristem is the tissue in most plants

containing undifferentiated cells

(meristematic cells) found in zones of

the plant where growth can take place

Q138 Rodentia Muridae is the scientific

name of

F A C E B O O K

P A G E h t t p w w w f a c e b o o k c o m s s c m e n t o r s o f f i c i a l P a g e | 33

FOR MORE UPDATES AND MORE MATERIAL DO LIKE OUR FACEBOOK PAGE httpwwwfacebookcomsscmentorsofficial

रह८िहॳलियानयररिी mdashmdash- कावहॴजञाननकनामहहॴ 24-

Jan-2017

Options

1) Mouse

चहा 2) Squirrel

चगिहरी 3) Monkey

बदर

4) Lizard

नछपकिी Correct Answer Mouse

Q139 Name the scientist who proposed

the cell theory

कह८लिकालसदातकापरसतावदहॳनहॳवािहॳवहॴजञाननककानामबताइए 24-Jan-2017

Options

1) Schleiden and Schwann

िीमिनऔरशरववान

2) Lamarck

िहॳमाकय 3) Treviranus

टरहॳवायरहॳनस

4)Whittaker and Stanley

हीटकरऔरसटहॳनिहॳ Correct Answer Schleiden and

Schwann

Q140 The flower with the worldrsquos

largest bloom is

दननयाकासबसहॳबड़ाफिखििनहॳवािा mdashmdashndash हहॴ 24-Jan-2017

Options

1) Pando

पािह८ 2) Posidonia

पह८सीिह८ननया 3) Rafflesia arnoldii

ररफिहॳलियाअनोमिी 4)Helianthus annuus

हहॳलिएनिसएनयअस

Correct Answer Rafflesia arnoldii

Rafflesia arnoldii is a species of

flowering plant in the parasitic genus

Rafflesia It is noted for producing the

largest individual flower on earth It has

a very strong and horrible odour of

decaying flesh earning it the nickname

ldquocorpse flower

Q141 Deficiency of which vitamin

causes night blindness

ककसपवटालमनकीकमीकहॳ कारणरतौधीहह८ताहहॴ 24-Jan-2017

Options

1) Vitamin K

पवटालमन K

2) Vitamin C

पवटालमन C

3) Vitamin B1

पवटालमन B1

4)Vitamin A

पवटालमन A

Correct Answer Vitamin A

Q142 Nongreen plants lack which of the

following

गहॴर-

हररतवनसपनतमनननननलिखितमसहॳककसकीकमीहह८तीहहॴ

24-Jan-2017

Options

1) Chlorophyll

किह८रह८कफि

2) Lycophyll

िायकह८कफि

3) Cyanophyll

F A C E B O O K

P A G E h t t p w w w f a c e b o o k c o m s s c m e n t o r s o f f i c i a l P a g e | 34

FOR MORE UPDATES AND MORE MATERIAL DO LIKE OUR FACEBOOK PAGE httpwwwfacebookcomsscmentorsofficial

सायनह८कफि

4)Phototropism

फह८टह८टरोपपजम

Correct Answer Chlorophyll

Q143 Organisms that use light to

prepare food are known as

जह८जीवपरकािकाउपयह८गकरभह८जनतहॴयारकरतहॳहहॴ उनह mdashmdash- कहॳ पमजानजाताहहॴ 24-Jan-2017

Options

1) Autotrophs

सवपह८षी 2) Heterotrophs

पवषमपह८षज

3) Omnivores

सवायहारी 4)Decomposers

पवघटनकरनहॳवािा Correct Answer Autotrophs

autotrophs often make their own food

by using sunlight carbon dioxide and

water to form sugars which they can use

for energy Some examples of

autotrophs include plants algae and

even some bacteria Autotrophs

(producer) are important because they

are a food source for heterotrophs

(consumers)

A heterotroph is an organism that

ingests or absorbs organic carbon

(rather than fix carbon from inorganic

sources such as carbon dioxide) in order

to be able to produce energy and

synthesize compounds to maintain its

life Ninety-five percent or more of all

types of living organisms are

heterotrophic including all animals and

fungi and some bacteria

Q144 Which of the following is a

primary function of haemoglobin

नननननलिखितमसहॳकह९नसाटहमह८गिह८बबनकाएकपरािलमककाययहहॴ

25-Jan-2017

Options

1) Utilization of energy

उजायकाउपयह८गकरना 2) Prevention of anaemia

रकतामपताहह८नहॳसहॳरह८कना 3) Destruction of bacteria

बहॴकटीररयाकापवनािकरना 4) To transport oxygen

ऑकसीजनकावहनकरना Correct Answer To transport oxygen

Q145 Vascular bundles are absent in

सवहनीबिि mdashmdash- मअनपनसतिरहतहॳहहॴ 25-Jan-2017

Options

1) Bryophyta

िायह८फाइटा 2) Pteridophyta

टहॳररिह८फाईटा 3) Gymnosperms

नजननह८सपमय 4) Angiosperms

एननजयह८सपहॳनसय Correct Answer Bryophyta

Q146 Sauria Lacertidae is the scientific

name of

सहॴररयािहॳसरटाईिी mdashmdashndash कावहॴजञाननकनामहहॴ 25-Jan-2017

Options

1) Crocodile

मगरमचछ

2) Hippopotamus

टहपपह८पह८टहॳमस

3) Lizard

नछपकिी 4) House fly

F A C E B O O K

P A G E h t t p w w w f a c e b o o k c o m s s c m e n t o r s o f f i c i a l P a g e | 35

FOR MORE UPDATES AND MORE MATERIAL DO LIKE OUR FACEBOOK PAGE httpwwwfacebookcomsscmentorsofficial

घरहॳिमकिी Correct Answer Lizard

Q147 Which type of pathogen causes

the water-borne disease SARS (Severe

Acute Respiratory Syndrome)

ककसपरकािकारह८गज़नकजिजननतबीमारीसासयकाकारणबनताहहॴ 25-Jan-2017

Options

1) Viral

वायरि

2) Parasitic

परजीवी 3) Protozoan

परह८टह८जअन

4) Bacterial

बहॴकटीररयि

Correct Answer Viral

Q148 Which of the following organs

produces the enzyme lipase

नननननलिखितमसहॳकह९नसाअगिायपहॳजएजाइमउतपननकरताहहॴ 25-Jan-2017

Options

1) Pancreas

अगनयािय

2) Large Intestine

बड़ीआत

3) Liver

नजगर

4) Small Intestine

छह८टीआत

Correct Answer Pancreas

Q149 A is a long internode forming the

basal part or the whole of a peduncle

एक mdashmdash- एकिबाइटरनह८िहहॴ जह८ननचिाटहससायासनपणयिठिबनताहहॴ 25-

Jan-2017

Options

1) Rhizome

परकद

2) Rachis

महॳ दि

3) floral axis

पषपअकष

4) Scape

भगदड़

Correct Answer scape

Q150 ndash Which of the following

organisms are considered to be both

Living and Non-living

नननननलिखितमसहॳकह९नसहॳजीवाणकह८जीपवतऔरअजीपवतमानाजाताहहॴ

25-Jan-2017

Options

1) Bacteria

बहॴकटीररया 2) Fungi

कवक

3) Algae

िहॴवाि

4)Virus

वायरस

Correct Answer Virus

They are considered to be living as they

possess a protein coat as a protective

covering DNA as the genetic material

etc

They are said to be non-living as they

can be crystallised and they survive for

billions of years They can tolerate high

temperatures freezing cold

temperatures ultra-violet radiations etc

Q151 Deficiency of fluorine causes

which of the following

फिह८ररनकीकमीकहॳ कारणनननननलिखितमसहॳकयाहह८ताहहॴ

F A C E B O O K

P A G E h t t p w w w f a c e b o o k c o m s s c m e n t o r s o f f i c i a l P a g e | 36

FOR MORE UPDATES AND MORE MATERIAL DO LIKE OUR FACEBOOK PAGE httpwwwfacebookcomsscmentorsofficial

27-Jan-2017

Options

1) Dental Caries

िटिकहॴ ररज

2) Scurvy

सकवरी 3) Anaemia

रकतामपता 4) Arthritis

गटठया Correct Answer Dental Caries

Q152 In a Punnett Square with the

cross AaBb x AaBb how many Aabb

genotypes would be created

पनहॳटसककायरमिह८स AaBb x AaBb कहॳ साि

ककतनहॳ Aabb जीनह८टाइपबनगहॳ 27-Jan-2017

Options

1) 1

2) 8

3) 2

4) 3

Correct Answer 2

Q153 Which of the following is the

Controlling Center of the Cell

नननननलिखित म सहॳ कह८लिकाका ननयतरण

क दर कह९न हहॴ

27-Jan-2017

Options

1) Nucleus

क दर

2) Plasma

पिाजमा 3) Lysosome

िायसह८सह८म

4) Chromosome

िह८मह८सह८म

Correct Answer Nucleus

The control centre of the cell is the

nucleus in eukaryotic cells The nucleus

contains genetic material in the form of

DNA

Q154 Myopia affects which of the

following organs

मायह८पपयानननननलिखितअगह८मसहॳककसहॳपरभापवतकरताहहॴ

25-Jan-2017

Options

1) Heart

हदय

2) Skin

तवचा 3) Eyes

आािहॳ 4)Mouth

मह

Correct Answer Eyes

Q155 Which of the following bears

flowers

नननननलिखितमसहॳकह९नफिधारणकरताहहॴ

25-Jan-2017

Options

1) Bryophyta

िायह८फाइटा 2) Pteridophyta

टहॳरीिह८फाईटा 3) Gymnosperms

नजननह८सपमय 4)Angiosperms

एननजयह८सपमय Correct Answer Angiosperms

Q156 Oxygenated blood flows out of the

heart through the

ऑकसीजनयकतरकत mdashmdashmdash

कहॳ माधयमसहॳहदयकहॳ बाहरबहताहहॴ 25-Jan-2017

F A C E B O O K

P A G E h t t p w w w f a c e b o o k c o m s s c m e n t o r s o f f i c i a l P a g e | 37

FOR MORE UPDATES AND MORE MATERIAL DO LIKE OUR FACEBOOK PAGE httpwwwfacebookcomsscmentorsofficial

Options

1) Aorta

महाधमनी 2) pulmonary artery

फहॳ फड़हॳकीधमनी 3) vena cava

वहॳनाकावा 4)Atrium

चह९क

Correct Answer aorta

Q157 Blood leaving the liver and

moving towards the

heart has a higher concentration of

नजगरसहॳननकिकरहदयकीतरफजानहॳवािहॳरकतम mdashmdashmdashmdash कीउचचसादरताहह८तीहहॴ 27-Jan-2017

Options

1) Lipids

लिपपडस

2) Urea

यररया 3) Bile Pigments

पपततकहॳ रगकरण

4) Carbon dioxide

काबयनिायऑकसाइि

Correct Answer Bile Pigments

Urea is nitrogen containing substance

which is produced in the liver in order

to deal with excess amino-acids in the

body As urea is produced it leaves the

liver in the blood stream and passes via

the circulatory system to all parts of the

body

Q158 Bulb is a modification of which

part of a plant

बमबएकपह९धहॳकहॳ ककसटहससहॳकाएक पातरणहह८ताहहॴ 27-Jan-2017

Options

1) The root

जड़

2) The stem

तना 3) The radicle

मिाकर

4)The fruit

फि

Correct Answer The stem

Q159 Which of the following carries

blood away from the heart to different

body parts

इनमहॳसहॳकह९नरकतकह८हदयसहॳिरीरकहॳ पवलभननअगह८तकिहॳजातीहहॴ

27-Jan-2017

Options

1) Arteries

धमननया 2) Nerves

तबतरहाए

3) Capillaries

कहॳ लिकाए

4)Veins

नसहॳ Correct Answer Arteries

Q160 The series of processes by which

nitrogen and its compounds are

interconverted in the environment and

in living organisms is called

27-Jan-2017

Options

1)Absorption of Nitrogen

2)Ammonification

3)Nitrogen Fixation

4)Nitrogen Cycle

Correct Answer Nitrogen Cycle

Ammonification or Mineralization is

performed by bacteria to convert

organic nitrogen to ammonia

F A C E B O O K

P A G E h t t p w w w f a c e b o o k c o m s s c m e n t o r s o f f i c i a l P a g e | 38

FOR MORE UPDATES AND MORE MATERIAL DO LIKE OUR FACEBOOK PAGE httpwwwfacebookcomsscmentorsofficial

Nitrification can then occur to convert

the ammonium to nitrite and nitrate

Nitrogen fixation is a process by which

nitrogen in the Earthrsquos atmosphere is

converted into ammonia (NH3) or other

molecules available to living organisms

Q161 BCG vaccine is given to protect

from which of the following

बीसीजीकाटटकानननननलिखितमसहॳककसकहॳ बचावकहॳ लिएटदयाजातहहॴ

27-Jan-2017

Options

1) Jaundice

पीलिया 2) Anaemia

रकतमपता 3) Tuberculosis

कषयरह८ग

4) Polio

पह८लियह८ Correct Answer Tuberculosis

Q162 Parallel venation is found in

समानतरवहॳनहॳिन mdashmdashmdash- मपायाजाताहहॴ 27-Jan-2017

Options

1) plants which are monocots

पह९धहॳजह८एकबीजपतरीहह८तहॳहहॴ 2) plants which have a dicot stem

वहॳपह९धहॳनजनकातनादपवदलियहह८ताहहॴ 3) plants with leaves similar to Tulsi

वहॳपह९धहॳनजनकीपनततयतिसीकीपनततयोकहॳ समानहह८तहॳहहॴ 4)plants with tap roots

टहॳप टवािहॳपह९धहॳ Correct Answer plants which are

monocots

Q163 The hardest part of the body is

िरीरकासबसहॳकठह८रभाग mdashndash हहॴ 27-Jan-2017

Options

1) Bones

हडडिय

2) Tooth Enamel

दातकहॳ इनहॳमि

3) Skull

िह८पड़ी 4) Spinal Cord

महॳ रजज

Correct Answer Tooth Enamel

Q164 Which type of pathogen causes

the waterborne disease E coli Infection

ककसपरकारकारह८गजननकजिजननतरह८गईकह८िाईसिमणकाकारणबनताहहॴ 27-Jan-2017

Options

1) Protozoan

परह८टह८जआ

2) Parasitic

परजीवी 3) Bacterial

बहॴकटीररयि

4)Viral

वायरि

Correct Answer Bacterial

Q165 The amount of blood filtered

together by both the kidneys in a 70 kg

adult male human in a minute is

70 की गरा वािहॳएकवयसकप षमएकलमनटमदह८नोगदकहॳदवाराएकसािचाबनीगयीरकतकीमातरहह८तीहहॴ 29-Jan-2017

Options

1) 1100 ml

1100 लमलि

2) 100 ml

F A C E B O O K

P A G E h t t p w w w f a c e b o o k c o m s s c m e n t o r s o f f i c i a l P a g e | 39

FOR MORE UPDATES AND MORE MATERIAL DO LIKE OUR FACEBOOK PAGE httpwwwfacebookcomsscmentorsofficial

100 लमलि

3) 1500 ml

1500 लमलि

4) 500 ml

500 लमलि

Correct Answer 1100 ml

Q166 Which feature of a plant helps to

distinguish a monocot from a dicot

पह९धहॳकीवहकह९नसीपविहॳषताहहॴजह८एकदपवदलियहॳऔरएकएकदिीयपह९धहॳसहॳभहॳदकरनहॳममददकरतीहहॴ 29-Jan-2017

Options

1) Pollination

परागम

2) Venation

वहॳनहॳिन

3) Vernation

वनिन

4) Aestivation

एसटीवहॳिहॳन

Correct Answer venation

Q167 The Mutation Theory was

proposed by

उतवररवतयनकालसदात mdashmdashndash

कहॳ दवरापरसतापवतककयाजाताहहॴ 29-Jan-2017

Options

1) Charles Lyell

चामसयलियहॳि

2) William Smith

पवलियमनसमि

3) Hugo De Vries

हयगह८िीराईस

4)Harrison Schmitt

हहॳरीसननसमट

Correct Answer Hugo De Vries

Q168 Which type of pathogen causes

the waterborne disease HepatitisA

ककसपरकारकहॳ रह८गजनकजिजननतरह८गहहॳपहॳटाइटटस-A काकारणबनताहहॴ

29-Jan-2017

Options

1) Parasitic

परजीवी 2) Viral

वायरि

3) Protozoan

परह८टह८जआ

4) Bacterial

बहॴकटीररयि

Correct Answer Viral

Q169 In a Punnett Square with the

cross AaBb x Aabb how many AaBb

genotypes would be created

पनहॳटसकवायरमिह८स AaBb x Aabb

कहॳ सािककतनहॳ AaBb जीनह८टाइपबनगहॳ 29-Jan-

2017

Options

1) 4

2) 1

3) 7

4) 6

Correct Answer 4

Q170 Arboreal Ateles is the scientific

name of

अिह८ररयिएटटलिस mdashmdashmdash कावहॴजञाननकनामहहॴ 29-Jan-2017

Options

1) Squirrel

चगिहरी 2) Sparrow

गह८रहॴया 3) Lizard

नछपकिी 4) Spider monkey

F A C E B O O K

P A G E h t t p w w w f a c e b o o k c o m s s c m e n t o r s o f f i c i a l P a g e | 40

FOR MORE UPDATES AND MORE MATERIAL DO LIKE OUR FACEBOOK PAGE httpwwwfacebookcomsscmentorsofficial

मकड़ीबदर

Correct Answer Spider monkey

Q171 Which type of pathogen causes

the waterborne disease Salmonellosis

ककसपरकारकारह८गाणजिजननतबीमारीसािमह८नहॳिह८लसज़काकारकहहॴ

29-Jan-2017

Options

1) Algal

िहॳवालियहॳ 2) Parasitic

परजीवी 3) Bacterial

बहॴकटीररयि

4)Viral

वायरि

Correct Answer Bacterial

An infection with salmonella bacteria

commonly caused by contaminated food

or water

Symptoms include diarrhoea fever

chills and abdominal pain

Q172 is a condition in which there is a

deficiency of red cells or of haemoglobin

in the blood

mdashmdash-

एकनसिनतहहॴनजसमहॳरकतमिािकह८लिकाओकीयाहीमह८गिह८बबनकीकमीहह८तीहहॴ 29-Jan-2017

Options

1) Albinism

एनमबननजम

2) Propyria

परह८पीररया 3) Anaemia

एनीलमया 4)Keloid disorder

कहॳ िह८इिडिसओिर

Correct Answer Anaemia

Q173 Ananas comosus is the scientific

name of

Options

अनानासकह८मह८सस mdashmdashmdashndash

कावहॴजञाननकनामहहॴ 29-Jan-2017

1) Custard Apple

सीताफि

2) Pineapple

पाइनएपपि

3) Bamboo

बास

4)Pomegranate

अनार

Correct Answer Pineapple

Q174 Which organ produces insulin

कह९नसाअगइनसलिनपहॴदाकरताहहॴ 29-Jan-

2017

Options

1) Liver

यकत

2) Thyroid gland

िायराइिगरिी 3) Spleen

पिीहा 4)Pancreas

अगरयिय

Correct Answer Pancreas

Q175 Which of the following disease is

not caused by water pollution

नननननलिखितमसहॳकह९नसारह८गपानीकहॳ परदषणकहॳकारणनहीहह८ता

29-Jan-2017

Options

1) Cholera

हहॴजा 2) Typhoid

F A C E B O O K

P A G E h t t p w w w f a c e b o o k c o m s s c m e n t o r s o f f i c i a l P a g e | 41

FOR MORE UPDATES AND MORE MATERIAL DO LIKE OUR FACEBOOK PAGE httpwwwfacebookcomsscmentorsofficial

टाइफाइि

3) Asthma

दमा 4)Diarrhoea

दसत

Correct Answer Asthma

Q176 Ocimum tenuiflorum is the

scientific name of

ओलिलममटहॳयईफिह८रमइसकावहॴजञाननकनाम mdash

ndash हहॴ 30-Jan-2017

Options

1) Neem

नीम

2) Mango

आम

3) Babul

बबि

4)Tulsi

तिसी Correct Answer Tulsi

Q177 Which gland secretes bile a

digestive fluid

कह९नसीगरिीपपतत एकपाचनतरिपरदािय सरापवतकरतीहहॴ 30-Jan-2017

Options

1) Pancreas

अगनयािय

2) Liver

यकत

3) Thyroid

िायराइि

4) Testes

टहॳनसटस

Correct Answer liver

Q178 In which of the following the

dominant phase is Gametophyte

नननननलिखितमसहॳककसकहॳ परमिचरणयगमकह८दपवधद (Gametophyte)हहॴ 30-Jan-2017

Options

1) Bryophyta

िायह८फाइटा 2) Pteridophyta

टहॳररिह८फाइटा 3) Gymnosperms

नजननह८सपमय 4) Angiosperms

एननजयह८सपमय Correct Answer Bryophyta

Q179 Anaerobic respiration refers to

which of the following

नननननलिखितमसहॳककसहॳअवायवीयशवसनकहाजाताहहॴ

30-Jan-2017

Options

1) Respiration without Oxygen

ऑकसीजनकहॳ बबनाशवसन

2) Respiration with Oxygen

ऑकसीजनकहॳ सािशवसन

3) Respiration without CO2

काबयनिायऑकसाइिकहॳ बबनाशवसन

4) Respiration with CO2

काबयनिायऑकसाइिकहॳ सािशविन

Correct Answer Respiration without

Oxygen

Q180 Which type of pathogen causes

the waterborne disease Cholera

ककसपरकारकारह८गजनकजिजननतरह८गहहॴजाकाकारणबनताहहॴ

30-Jan-2017

Options

1) Algal

िहॴवालियहॳ

F A C E B O O K

P A G E h t t p w w w f a c e b o o k c o m s s c m e n t o r s o f f i c i a l P a g e | 42

FOR MORE UPDATES AND MORE MATERIAL DO LIKE OUR FACEBOOK PAGE httpwwwfacebookcomsscmentorsofficial

2) Bacterial

बहॴकटीररयि

3) Protozoan

परह८टह८जआ

4) Viral

वायरि

Correct Answer Bacterial

Q181 To which class does

Oxyreductases transferases hydrolases

belong

ओकसीररिकटहॳसटरासफरहॳजहॳस

हाइडरह८िहॳसहॳसककसवगयमआतहॳहहॴ 30-Jan-2017

Options

1) Hormones

हारमोस

2) Enzymes

एजाइनस

3) Proteins

परह८टीनस

4) Vitamins

पवटालमनस

Correct Answer Enzymes

Q182 Which of the following is not true

about Gymnosperms

ननननमसहॳकह९नसीबातअनावतबीजीकहॳ बारहॳमसचनहीहहॴ 30-Jan-2017

Options

1) Dominant phase is saprophytes

परमिचरणसहॳपरह८फाइटसहह८ताहहॴ 2) Vascular bundles are absent

सवहनीबििअनपनसितहह८ताहहॴ 3) spores are heterospores

बीजाणहहॳटहॳरह८सपह८रसहह८तहॳहहॴ 4) Flowers are absent

फिअनपनसितहह८तहॳहहॴ

Correct Answer Vascular bundles are

absent

Q183 The name of first mammal clone sheep is

भहॳड़कीपरिमसतनपायीपरनत प (किह८न)

कानामहहॴ 30-Jan-2017

Options

1) Noori

नरी 2) Dolly

िॉिी 3) Louise

िसी 4)Durga

दगाय Correct Answer Dolly

Q184 Which type of pathogen causes

the water-borne disease Typhoid fever

ककसपरकारकारह८गजनकजिजननतरह८गटाइफाइिबिारकाकारणबनताहहॴ 30-Jan-2017

Options

1) Algal

िहॴवािीय

2) Parasitic

परजीवी 3) Protozoan

परह८टह८जनअन

4)Bacterial

बहॴकटीररयि

Correct Answer Bacterial

Q185 In which part of the cell are

proteins made

कह८लिकाकहॳ ककसटहससहॳमपरह८टीनबनायाजाताहहॴ

31-Jan-2017

Options

1) Reticulum

रहॳटटकिम

F A C E B O O K

P A G E h t t p w w w f a c e b o o k c o m s s c m e n t o r s o f f i c i a l P a g e | 43

FOR MORE UPDATES AND MORE MATERIAL DO LIKE OUR FACEBOOK PAGE httpwwwfacebookcomsscmentorsofficial

2) Golgi apparatus

गह८मजीएपहॳरहॳटस

3) Ribosomes

ररबह८सह८नस

4) Lysosome

िायसह८सह८नस

Correct Answer ribosomes

Proteins are produced by stringing

amino acids together in the order

specified by messenger RNA strands

that were transcribed from DNA in the

cell nucleus The process of synthesizing

a protein is called translation and it

occurs on ribosomes in the cytoplasm of

a cell

Q186 Polio is a disease caused by which

of the following

नननननलिखितमसहॳपह८लियह८कीबबमारह८हह८नहॳकाकारणकयाहहॴ

31-Jan-2017

Options

1) Bacteria

बहॴकटीररयि

2) Mosquito

मचछर

3) Virus

वायरस

4) Cockroach

नतिच हॳ Correct Answer Virus

Polio or poliomyelitis is a crippling and

potentially deadly infectious disease It

is caused by the poliovirus

Q187 ndash Hay fever is a sign of which of

the following

हहॳकफवरनननननलिखितमसहॳककसकाएकसकहॳ तहहॴ

31-Jan-2017

Options

1) Old Age

वदावसिा 2) Malnutrition

कपह८सण

3) Allergy

एिनजय 4) Over Work

अतयचधककाययकरना Correct Answer Allergy

Q188 How many chromosomes does a

human cell contain

एकमानवकह८लिकामककतनहॳगणसतरहह८तहॳहहॴ

29-Jan-2017

Options

1) 6

2) 26

3) 46

4) 66

Correct Answer 46

In humans each cell normally contains

23 pairs of chromosomes for a total of

46 Twenty-two of these pairs called

autosomes look the same in both males

and females The 23rd pair the sex

chromosomes differ between males and

females

Q189 Which of the following is not true

about Bryophyta

ननननमसहॳकह९नसीबातिायह८फाइटकहॳ बारहॳमसचनहीहहॴ 31-Jan-2017

Options

1) Dominant phase is gametophytes

परमिचरणगहॳलमतह८फाइटसहह८ताहहॴ 2) Main plant body is haploid

पह९धहॳकामखयिरीरअगखणतहह८ताहहॴ 3) Spores are homospores

बीजाणहह८मह८सफह८रसहह८तहॳहहॴ 4) Flowers are present

फिमह८जदहह८तहॳहहॴ Correct Answer Flowers are present

F A C E B O O K

P A G E h t t p w w w f a c e b o o k c o m s s c m e n t o r s o f f i c i a l P a g e | 44

FOR MORE UPDATES AND MORE MATERIAL DO LIKE OUR FACEBOOK PAGE httpwwwfacebookcomsscmentorsofficial

Q190 Which aquatic animal has

trailing tentacles

ककसजिीयजानवरकहॳ पीछहॳचिनहॳवािहॳटहॳटकिसहह८तहॳहहॴ

31-Jan-2017

Options

1) Sea horse

समदरीघह८िा 2) Corals

मगा 3) Jelly fish

जहॳिीमछिी 4) Star fish

तारामछिी Correct Answer Jelly fish

Jellyfish with its umbrella-shaped bell

and trailing tentacles

Q191 Which type of pathogen causes

the water-borne disease Poliomyelitis

(Polio)

ककसपरकारकारह८गजनकजिजननतरह८गपह८लियह८मायहॳटटस (पह८लियह८) काकारणहहॴ 31-Jan-

2017

Options

1) Parasitic

परजीवी 2) Algal

िहॴवालिय

3) Viral

वायरि

4) Bacterial

बहॴकटीररयि

Correct Answer Viral

Q192 The outer white part of the eye

that protects the inner structures is

आािकाबाहरीसफहॳ दटहससाजह८आतररकसरचनाओकीरकषाकरताहहॴ वह mdashmdashmdash हहॴ 31-Jan-

2017

Options

1) Iris

आयररस

2) Sclera

सकिहॳरा 3) Retina

रहॳटटना 4) Cornea

कह८ननयया Correct Answer Sclera

Q193 Proteins are made up of

परह८टीनकाननमायण mdashndash सहॳहह८ताहहॴ 31-Jan-2017

Options

1) Amino acids

एलमनह८अनि

2) Fatty acids

वसायकतअनि

3) Glucose

गिकह८ज

4)Nucleotides

नयनकियह८टाईिस

Correct Answer Amino acids

Q194 Moringa Oleifera is the scientific

name of

मह८ररगओलिफहॳ रा mdashmdashndash कावहॴजञाननकनामहहॴ 31-Jan-2017

Options

1) Banyan

बरगद

2) Gulmohar

गिमह८हर

3) Amla

आमिा

F A C E B O O K

P A G E h t t p w w w f a c e b o o k c o m s s c m e n t o r s o f f i c i a l P a g e | 45

FOR MORE UPDATES AND MORE MATERIAL DO LIKE OUR FACEBOOK PAGE httpwwwfacebookcomsscmentorsofficial

4) Drumstick

डरमनसटक

Correct Answer Drumstick

Q195 Kidney stones are composed of

गदकीपिरी mdashndash सहॳबनीहह८तीहहॴ 1-Feb-2017

Options

1) Calcium Oxalate

कहॴ नमसयमओकजहॳिहॳट

2) Sodium Chloride

सह८डियमकिह८राइि

3) Magnesium Nitrate

महॳनगनलियमनाइतटरहॳट

4) Calcium Bicarbonate

कहॴ नमियमबायकबोनहॳट

Correct Answer Calcium Oxalate

Q196 ndash Which of the following is not

true about Angiosperms

ननननमसहॳकह९नसीबातआवतबीजीकहॳ बारहॳमसचनहीहहॴ 1-Feb-2017

Options

1) Dominant phase is gametophytes

परमिचरणगहॳलमतह८फाइटहह८ताहहॴ 2) Vascular bundles are present

सवहनीबििमह९जदहह८ताहहॴ 3) Spores are heterospores

बीजाणहहॳटहॳरह८सपह८रसहह८तहॳहहॴ 4) Seeds are covered

बीजढकहॳ हह८तहॳहहॴ Correct Answer Dominant phase is

gametophytes

Q197 All of the following are excretory

(waste) products of animals except

नननननलिखितमसहॳककसएककह८छह८ड़करअनयसभीपराखणयोदवाराउतसनजयतपदाियहहॴ 1-Feb-

2017

Options

1) Uric Acid

यररकएलसि

2) Ammonia

अमह८ननया 3) Carbohydrates

काबोहाइडरहॳट

4) Urea

यररया Correct Answer Carbohydrates

In animals the main excretory products

are carbon dioxide ammonia (in

ammoniotelics) urea (in ureotelics) uric

acid (in uricotelics) guanine (in

Arachnida) and creatine

Q198 RNA is a polymeric molecule

What does RNA stand for

आरएनइएएकबहिकआणहहॴ इसकाकापवय पकयाहहॴ 1-Feb-2017

Options

1) Rado Nuclear Acid

रािह८नयनकियरएलसि

2) Ribo Nucleic Acid

राइबह८नयनकिकएलसि

3) Rhino Nuclear Acid

हाइनह८नयनकियरएलसि

4) Resto Nucleus Acid

रहॳसटह८नयकिीयसएलसि

Correct Answer Ribo Nucleic Acid

Q199 Which organ does detoxification

and produces chemicals needed for

digestion

कह९नसाअगपवषहरणकरताहहॴऔरपाचनकहॳ लिएआवशयकरसायनोकह८पहॴदाकरताहहॴ 1-Feb-

2017

Options

1) Salivary glands

िारगरचिया 2) Pancreas

अगनयािय

F A C E B O O K

P A G E h t t p w w w f a c e b o o k c o m s s c m e n t o r s o f f i c i a l P a g e | 46

FOR MORE UPDATES AND MORE MATERIAL DO LIKE OUR FACEBOOK PAGE httpwwwfacebookcomsscmentorsofficial

3) Thyroid gland

िायराइिगरिी 4) Liver

यकत

Correct Answer Liver

Q200 Psidium guajava is the scientific

name of

लसडियमगआजावा mdashmdash कावहॴजञाननकनामहहॴ 1-

Feb-2017

Options

1) Guava

अम द

2) Mango

आम

3) Bamboo

बास

4) Jack fruit

कटहि

Correct Answer Guava

Q201 Which drug is used as a Blood

Thinner

चधरकह८पतिाकरनहॳकहॳ पमककसदवाकापरयह८गककयाजाताहहॴ

1-Feb-2017

Options

1) Warfarin

वाफर न

2) Tramadol

टरहॳमािह८ि

3) Azithromycin

एनजरह८मायलसन

4) Hydralazine

हाइडरह८िहॳनजन

Correct Answer Warfarin

Q202 Which of the following disease is

caused due to the deficiency of protein

परह८टीनकीकमीकहॳ कारणनननननलिखितमसहॳकह९नसारह८गहह८ताहहॴ 1-Feb-2017

Options

1) Arthritis

गटठया 2) Kwashiorkor

कािीओकय र

3) Goitre

गाइटर

4) Night Blindness

रतह९चध

Correct Answer Kwashiorkor

Q203 A is species of plant that has

adapted to survive in an environment

with little liquid water

mdashmdashndashपह९धहॳकीएकऐसहॳऐसहॳपरजानतहहॴ नजसनहॳकमपानीवािहॳवातावरणमजीपवतरहनहॳकहॳलिएअनकिनहहॴ 1-Feb-2017

Options

1) Xerophyte

म दपवद

2) Hydrophyte

जिीयपादप

3) Mesophyte

समह८दपवद

4) Thallophyte

िहॴिह८फाइटा Correct Answer xerophyte

xerophyte is a species of plant that has

adapted to survive in an environment

with little liquid water such as a desert

or an ice- or snow-covered region in the

Alps or the Arctic

Mesophytes are terrestrial plants which

are adapted to neither a particularly

dry nor particularly wet environment

An example of a mesophytic habitat

would be a rural temperate meadow

F A C E B O O K

P A G E h t t p w w w f a c e b o o k c o m s s c m e n t o r s o f f i c i a l P a g e | 47

FOR MORE UPDATES AND MORE MATERIAL DO LIKE OUR FACEBOOK PAGE httpwwwfacebookcomsscmentorsofficial

which might contain goldenrod clover

oxeye daisy and Rosa multiflora

thallophyte any of a group of plants or

plantlike organisms (such as algae and

fungi) that lack differentiated stems

leaves and roots and that were formerly

classified as a primary division

(Thallophyta) of the plant kingdom

Q204 How many types of teeth are

there in humans

मनषयोमककतनहॳपरकारकहॳ दातहह८तहॳहहॴ

1-Feb-2017

Options

1) 4

2) 5

3) 2

4) 3

Correct Answer 4

teeth -Humans have four types of

teethincisors canines premolars and

molars each with a specific function

The incisors cut the food the canines

tear the food and the molars and

premolars crush the food

Q205 Carica papaya is the scientific name of

कहॴ ररकापपाया mdashmdashndash कावहॴजञाननकनामहहॴ 2-

Feb-2017

Options

1) Peepal

पीपि

2) Papaya

पपीता 3) Tamarind

इमिी 4) Drumstick

ढह८िकाछड़ी Correct Answer Papaya

Q206 Muscles get tired when there is

shortfall of

जब mdashndash कीकमीहह८तीहहॴतबपहॳिीयिकजातीहहॴ 2-Feb-2017

Options

1) Lactic acid

िहॴनकटकएलसि

2) Na+ ions

Na+ आयन

3) ATP

एटीपी 4) Sulphates

समफहॳ टस

Correct Answer ATP

ATP is the energy source muscle fibers

use to make muscles contract

muscle tissuersquos main source of energy

called adenosine triphosphate or ATP

As your muscles use up this energy

source they become tired and fatigued

Oxygen is the key ingredient that helps

create new ATP to replenish the burned

up ATP in your muscles

Q207 Artocarpus integra is the

scientific name of आटह८कापयसइटीगरा mdashmdashmdash कावहॴजञाननकनामहहॴ 2-Feb-2017

Options

1) Guava

अम द

2) Pineapple

अनानास

3) Silver Oak

लसमवरओक

4) Jack fruit

कटहि

Correct Answer Jack fruit

Q208 Which organ stores fat soluble

vitamins

कह९नसाअगवसामघिनिीिपवटालमनह८काभिाराकरताहहॴ

2-Feb-2017

F A C E B O O K

P A G E h t t p w w w f a c e b o o k c o m s s c m e n t o r s o f f i c i a l P a g e | 48

FOR MORE UPDATES AND MORE MATERIAL DO LIKE OUR FACEBOOK PAGE httpwwwfacebookcomsscmentorsofficial

Options

1) Blood

रकत

2) Skin

तवचा 3) Liver

यकत

4) Pancreas

अगनयािय

Correct Answer Liver

Q209 Which disease is caused due to

deficiency of Iodine

आयह८िीनकहॳ कारणकह९नसारह८गहह८ताहहॴ 2-Feb-2017

Options

1) Rickets

ररकहॳ टस

2) Scurvy

सकवी 3) Goitre

गणमािा 4) Growth retardation

पवकासका कना Correct Answer Goitre

rickets A softening and weakening of

bones in children usually due to

inadequate vitamin D

Q210 Grevillea Robusta is the scientific name of

गरहॳपवलियारह८बसटा mdashmdashmdash- कापवजञाननकनामहहॴ 2-Feb-2017

Options

1) Peepal

पीपि

2) Teak

सागह९न

3) Silver Oak

लसमवरओक

4) Jack fruit

कटहि

Correct Answer Silver Oak

Q211 When a Cuttlefish is described as a Molluscs it is at which level of

classification

जबएककटिकफिकह८एकमह८िसकाकहॳ पमवखणयतककयाजाताहहॴतबयहॳवगीकरणकहॳ ककससतरपहॳनसितहहॴ 2-Feb-2017

Options

1) Class

वगय 2) Order

िम

3) Family

पररवार

4) Phylum

सघ

Correct Answer Phylum

Q212 Bambusa dendrocalmus is the

scientific name of बानबसािहॳडराकामस mdashmdashmdash कावहॴजञाननकनामहहॴ 3-Feb-2017

Options

1) Banyan

बरगद

2) Papaya

पपीता 3) Bamboo

बास

4) Pomegranate

अनार

Correct Answer Bamboo

Q213 Acinonyx Jubatus is the scientific name of

एलसनह८ननकसजयबहॳटस mdashmdashmdash

कावहॴजञाननकनामहहॴ 3-Feb-2017

F A C E B O O K

P A G E h t t p w w w f a c e b o o k c o m s s c m e n t o r s o f f i c i a l P a g e | 49

FOR MORE UPDATES AND MORE MATERIAL DO LIKE OUR FACEBOOK PAGE httpwwwfacebookcomsscmentorsofficial

Options

1) Bear

भाि 2) Horse

घह८िा 3) Cheetah

चीता 4) Zebra

जहॳिा Correct Answer Cheetah

Q214 The pale yellow colour of urine is

due to the presence of which pigment

मतरकाफीकापीिारगरगदरयकहॳ उपनसिनतकहॳ कारणहह८ताहहॴ

3-Feb-2017

Options

1) Urochrome

यरह८िह८म

2) Urophyll

यरह८कफि

3) Chlorophyll

किह८रह८कफि

4) Chloroplast

किह८रह८पिासट

Correct Answer Urochrome

Q215 Which of the following constitute

to form a gene

नननननलिखितमसहॳकह९नसीचीज़एकजीनकागठनकरतीहहॴ

3-Feb-2017

Options

1) Polynucleotides

पह८िीनयनकियह८टाईडस

2) Hydrocarbons

हाइडरह८काबोस

3) Lipoproteins

िाईपह८परह८टीनस

4) Lipids

लिपपडस

Correct Answer Polynucleotides

Polynucleotide molecule is a biopolymer

composed of 13 or more nucleotide

monomers covalently bonded in a chain

DNA (deoxyribonucleic acid) and RNA

(ribonucleic acid) are examples of

polynucleotides with distinct biological

function

Q216 Vertebrates belongs to the

phylum

रीढ़कीहडिीवािहॳपराणी mdashmdashmdash

परजानतकहॳ अतगायतआतहॳहहॴ 3-Feb-2017

Options

1) Arthropoda

आरह८पह८ड़ा 2) Annelida

एननलििा 3) Cnidaria

ननिहॳररया 4) Chordata

कह८िटा Correct Answer Chordata

Q217 Punica granatum is the scientific name of

पननकगरहॳनहॳटस mdashmdashmdash कावहॴजञाननकनामहहॴ 3-Feb-2017

Options

1) Custard Apple

सीताफि

2) Gulmohar

गिमह८हर

3) Silver Oak

लसमवरओक

4) Pomegranate

अनार

Correct Answer Pomegranate

F A C E B O O K

P A G E h t t p w w w f a c e b o o k c o m s s c m e n t o r s o f f i c i a l P a g e | 50

FOR MORE UPDATES AND MORE MATERIAL DO LIKE OUR FACEBOOK PAGE httpwwwfacebookcomsscmentorsofficial

Q218 Between a tiger and an monkey

which of the following is different

एकबाघऔरबदरकहॳ बीचनननननलिखितमसहॳकह९नसीबातअिगहहॴ 3-Feb-2017

Options

1) Kingdom

राजय

2) Phylum

जानत

3) Order

िम

4) Class

वगय Correct Answer order

Q219 The artificial heart was invented by

कबतरमहदयका mdashmdashmdash

दवाराअपवषकारककयागयािा 3-Feb-2017

Options

1) Muhammad Yunus

महनमदयनस

2) Linus Yale Jr

िाइनसयहॳिजय

3) Gazi Yasargil

गाजीयासचगयि

4) Paul Winchell

पह९िपवमकि Correct Answer Paul Winchell

Q220 Tamarindus indica is the

scientific name of

टहॳमररनडसइडिका mdashmdash कावहॴजञाननकनामहहॴ 7-

Feb-2017

Options

1) Neem

नीम

2) Pineapple

अनानास

3) Tamarind

इमिी 4)Chiku

चीक

Correct Answer Tamarind

Q221 In eukaryotic cells synthesis of

RNA takes place in the

यकहॳ योटटककह८लिकाओमआरएनएकासशिहॳषण

mdashndash महह८ताहहॴ 7-Feb-2017

Options

1) Mitochondria

माईटह८कोडडरया 2) Centrioles

सटरीयह८मस

3) Ribosomes

ररबह८सह८नस

4) Nucleus

नयनकियस

Correct Answer nucleus

eukaryotic cell -Transcription is the

process of synthesizing ribonucleic acid

(RNA)Synthesis takes place within the

nucleus of eukaryotic cells or in the

cytoplasm of prokaryotes and converts

the genetic code from a gene in

deoxyribonucleic acid ( DNA ) to a

strand of RNA that then directs

proteinsynthesis

Q222 _________is caused by parasites

of the Plasmodium genus

पिाजमह८डियमजातीकहॳ परजीवी mdash- कहॳ कारणहहॴ 7-Feb-2017

Options

1) Dysentery

पहॳचचि

2) Malaria

मिहॳररया 3) Chickenpox

F A C E B O O K

P A G E h t t p w w w f a c e b o o k c o m s s c m e n t o r s o f f i c i a l P a g e | 51

FOR MORE UPDATES AND MORE MATERIAL DO LIKE OUR FACEBOOK PAGE httpwwwfacebookcomsscmentorsofficial

चहॳचक

4) Herpes

हहॳपपयस

Correct Answer Malaria

Q223 Carotene in fruits and vegetables

gives it which color

फिह८औरसनलजयोमनसितकहॳ रह८टीनउनहकह९नसारगपरदानकरताहहॴ 7-Feb-2017

Options

1) Green

हरा 2) Pink

गिाबी 3) Orange

नारगी 4) Blue

नीिा Correct Answer Orange

Q224 Equus Caballus is the scientific

name of

एकवसकहॴ बहॳिस mdashmdashndash कापवजञाननकनामहहॴ 7-Feb-2017

Options

1) Horse

घह८िा 2) Zebra

ज़हॳिा 3) Donkey

गधा 4) Buffalo

भस

Correct Answer Horse

Q225 Elapidae Naja is the scientific name of

एिीपीिीनाजा mdashmdash- कावहॴजञाननकनामहहॴ 8-Feb-2017

Options

1) Cobra

कह८बरा 2) Elephant

हािी 3) Eagle

ग ि

4) Owl

उमि Correct Answer Cobra

Q226 Which disease is caused due to

deficiency of Iron

िह८हकीकमीकहॳ कारणकह९नसारह८गहह८ताहहॴ 8-Feb-

2017

Options

1) Beriberi

बहॳरीबहॳरी 2) Tetany

टहॳटनी 3) Kwashiorkor

कवािीऔरकर

4) Anaemia

रकतामपता Correct Answer Anaemia

Beriberi is a disease caused by a vitamin

B-1 deficiency also known as thiamine

deficiency

Tetany can be the result of an

electrolyte imbalance Most often itrsquos a

dramatically low calcium level also

known as hypocalcemia Tetany can also

be caused by magnesium deficiency or

too little potassium Having too much

acid (acidosis) or too much alkali

(alkalosis) in the body can also result in

tetany

Kwashiorkor also known as

ldquoedematous malnutrition It is a form of

malnutrition caused by a lack of protein

in the diet

Anaemia means that you have fewer red

blood cells than normal or you have less

F A C E B O O K

P A G E h t t p w w w f a c e b o o k c o m s s c m e n t o r s o f f i c i a l P a g e | 52

FOR MORE UPDATES AND MORE MATERIAL DO LIKE OUR FACEBOOK PAGE httpwwwfacebookcomsscmentorsofficial

haemoglobin than normal in each red

blood cell

Q227 is a leaf where the leaflets are

arranged along the middle vein

mdashndashएकपततीहहॴजहापतरकह८कीरचनाक ररयालिराकहॳ आसपासहह८तीहहॴ 8-Feb-2017

Options

1) Pinnately compound leaf

पपनहॳटिीसयकतपतती 2) Palmately compound leaf

पामहॳटिीसयकतपतती 3) Compound leaf

सयकतपतती 4) Simple leaf

साधारणपतती Correct Answer Pinnately compound

leaf

Q228 Haustoria or sucking roots are

found in which of the following

हह८सटह८ररयायाचसनहॳवािीजड़हॳनननननलिखितमसहॳककसमपाईजातीहहॴ 8-Feb-2017

Options

1) Wheat

गहॳह

2) Mango

आम

3) Chestnut

चहॳसटनट

4) Cuscuta

कसकयटा Correct Answer Cuscuta

Haustorial roots -The roots of parasitic

plants which penetrate into the host

tissues to absorb nourishment are

called haustorial roots hellip Also known as suckingor parasitic roots

Q229 Equs Asinus is the scientific name

of

एकवसएलसनस mdashmdashndash कावहॴजञाननकनामहहॴ 8-

Feb-2017

Options

1) Donkey

गधा 2) Cow

गाय

3) Deer

टहरन

4) Kangaroo

कगा

Correct Answer Donkey

Q230 Ficus benghalensis is the scientific name of

फाईकसबहॳनगहॳिहॳलसस mdashndash कापवजञाननकनामहहॴ 8-Feb-2017

Options

1) Banyan

बरगद

2) Pineapple

अनानास

3) Babul

बबि

4) Tulsi

तिसी Correct Answer Banyan

Q231 Equus burchellii is the scientific name of

एकवसबचिी mdashmdash- कापवजञाननकनामहहॴ 8-Feb-2017

Options

1) Horse

घह८िा 2) Zebra

जहॳिा 3) Buffalo

F A C E B O O K

P A G E h t t p w w w f a c e b o o k c o m s s c m e n t o r s o f f i c i a l P a g e | 53

FOR MORE UPDATES AND MORE MATERIAL DO LIKE OUR FACEBOOK PAGE httpwwwfacebookcomsscmentorsofficial

भस

4) Ass

गधा Correct Answer Zebra

Page 30: COMPILATION OF ALL 72 SETS OF BIOLOGY SSC CHSL-2016 · OF BIOLOGY SSC CHSL-2016 PREPARED BY : SSC MENTORS BIOLOGY SPECIAL . F A C E B O O K P A G E : h t t p : / / w w w . f a c e

F A C E B O O K

P A G E h t t p w w w f a c e b o o k c o m s s c m e n t o r s o f f i c i a l P a g e | 29

FOR MORE UPDATES AND MORE MATERIAL DO LIKE OUR FACEBOOK PAGE httpwwwfacebookcomsscmentorsofficial

Q122 Which of the following is a

structural and functional unit of

kidneys

नननननलिखितमसहॳकह९नसीगदोकीसरचनातमकऔरकाययकरीईकाईहहॴ

22-Jan-2017

Options

1) Renette Cells

रहॳनहॳटकह८लिकाए

2) Flame Cells

फिहॳमकह८लिकाए

3) Nephrites

नहॳफ़राइटस

4)Nephrons

नहॳफरोस

Correct Answer Nephrons

Nephron functional unit of the kidney

the structure that actually produces

urine in the process of removing waste

and excess substances from the blood

There are about 1000000 nephrons in

each human kidney

Q123 Which of the following is the

largest part of the human brain

नननननलिखितमसहॳकह९नसामानवमनसतषककासबसहॳबड़ाटहससाहहॴ

23-Jan-2017

Options

1) Ribs

पसलियाा 2) Cerebrum

सहॳरहॳिम

3) Pons

पोस

4)Thalamus

िहॴिहॳमस

Correct Answer Cerebrum

The cerebrum is the largest part of the

human brain making up about two-

thirds of the brainrsquos mass It has two

hemispheres each of which has four

lobes frontal parietal temporal and

occipital

Q124 The auxiliary buds

सहायककालियाmdashndash 23-Jan-2017

Options

1) grow endogenously from the pericycle

पहॳरीसाईककिसहॳअनतजातयपवकलसतहह८ताहहॴ 2) arise endogenously from the main

growing point

मिवपदसहॳअनतजातयउठताहहॴ 3) is an embryonic shoot located in the

axil of a leaf

एकभरणिटहहॴजह८एकपततीकहॳ अकषपरनसतिहह८ताहहॴ 4)arise exogenously from the epidermis

एपपिलमयससहॳबटहजातयतरीकहॳ सहॳउठताहहॴ Correct Answer is an embryonic shoot

located in the axil of a leaf

Q125 Which of the following is a viral

disease

इनमहॳसहॳकह९सीएकवायरिबीमारीहहॴ

23-Jan-2017

Options

1) Polio

पह८लियह८ 2) Tetanus

धनसतनभ

3) Leprosy

कषठरह८ग

4) Plague

पिहॳग

Correct Answer Polio

A viral disease (or viral infection)

occurs when an organismrsquos body is

invaded by pathogenic viruses and

infectious virus particles (virions) attach

to and enter susceptible cells

F A C E B O O K

P A G E h t t p w w w f a c e b o o k c o m s s c m e n t o r s o f f i c i a l P a g e | 30

FOR MORE UPDATES AND MORE MATERIAL DO LIKE OUR FACEBOOK PAGE httpwwwfacebookcomsscmentorsofficial

Poliomyelitis often called polio or

infantile paralysis is an infectious

disease caused by the poliovirus

Tetanusmdash A serious bacterial infection

that causes painful muscle spasms and

can lead to death

Leprosy also known as Hansenrsquos

disease (HD) is a long-term infection by

the bacterium Mycobacterium leprae or

Mycobacterium lepromatosis

Plague is an infectious disease caused by

the bacterium Yersinia pestis

Symptoms include fever weakness and

headache

Q126 Which organisms can help to

carry out Vermicomposting

कह९नसाजीववमीकनपह८नसटगममददकरताहहॴ

23-Jan-2017

Options

1) Nitrifying Bacteria

नाईटरीफाईगबहॴकटीररया 2) Earthworms

कहॴ चऐ

3) Algae

िहॴवि

4) Fungus

कवक

Correct Answer Earthworms

Q127 Contraction of heart is also

known as

हदयकहॳ सकचनकह८ mdash- भीकहाजाताहहॴ 23-Jan-

2017

Options

1) Systole

लससटह८ि

2) Aristotle

अरसत

3) Diastole

िायसटह८ि

4) Lub

मयब

Correct Answer Systole

Diastole is the part of the cardiac cycle

when the heart refills with blood

following systole (contraction)

Ventricular diastole is the period during

which the ventricles are filling and

relaxing while atrial diastole is the

period during which the atria are

relaxing

Q128 Azadirachta indica is the

botanical name of which of the

following

अजाटदराचताइडिकानननननलिखितमसहॳककसकावानसपनतनामहहॴ

23-Jan-2017

Options

1) Rose plant

गिाबकापह९धा 2) Apple tree

सहॳबकापहॳड़

3) Neem

नीम

4)Mango

आम

Correct Answer Neem

Q129 Which of the following is the

main end product of carbohydrate

digestion

नननननलिखितमसहॳकह९नसाकाबोहाइडरहॳटकहॳ पाचनकापरमिअतउतपादकहह८ताहहॴ 23-Jan-2017

Options

1) Fats

वसा 2) Lipids

लिपपडस

3) Glucose

गिकह८ज

4) Cellulose

F A C E B O O K

P A G E h t t p w w w f a c e b o o k c o m s s c m e n t o r s o f f i c i a l P a g e | 31

FOR MORE UPDATES AND MORE MATERIAL DO LIKE OUR FACEBOOK PAGE httpwwwfacebookcomsscmentorsofficial

सहॳमयिह८ज

Correct Answer Glucose

Intestinal absorption of end products

from digestion of carbohydrates and

proteins in the pig hellip During absorption some sugars (fructose or

galactose) released from the

corresponding sucrose and lactose

respectively during digestion were

partly metabolized into glucose by the

enterocyte

Q130 Which of the following glands is a

source of the enzyme Ptyalin

नननननलिखितगरचियोमसहॳएजाइमटयालिनकासरह८तहहॴ 23-Jan-2017

Options

1) Pancreas

अगरािय

2) Thyroid Gland

िाइराइिगरिी 3) Pituitary Gland

पीयषगरिी 4) Salivary Glands

िारगरचियाा Correct Answer Salivary Glands

Q131 Which of the following is not true

about Pteridophyta

ननननमसहॳकह९नसीबातटहॳररिह८फाईटकहॳ बारहॳमसचनहीहहॴ 23-Jan-2017

Options

1) Dominant phase is saprophytes

परमिचरणसहॳपरह८फाईइटसहह८ताहहॴ 2) Main plant body is diploid

पह९दह८कामखयिरीरदपवगखणतहह८ताहहॴ 3) Seeds are present

बीजमह९जदहह८तहॳहहॴ 4)Flowers are absent

फिअनपनसतिहह८तहॳहहॴ

Correct Answer Seeds are present

Q132 The largest dolphin species is the

orca also called as

िॉिकफनकीसबसहॳबड़ीपरजानतकाकानामआकायहहॴनजसहॳ mdash- भीकहतहॳहहॴ 23-Jan-2017

Options

1) Bottle Nose

बाटिनह८ज

2) Baiji

बहॳजी 3) Killer whale

ककिरहहॳि

4)Tucuxi

टकवसी Correct Answer Killer whale

Q133 The fat digesting enzyme Lipase

is secreted by which of the following

वसाकापाचनकरनहॳवािाएजाइमिाइपहॳजनननननलिखितमसहॳककसकहॳ दवारासतरापवतहह८ताहहॴ

24-Jan-2017

Options

1) Kidneys

गद

2) Pancreas

अगनयािय

3) Large Intestine

बड़ीआत

4)Liver

नजगर

Correct Answer Pancreas

Lipase is an enzyme that splits fats so

the intestines can absorb them Lipase

hydrolyzes fats like triglycerides into

their component fatty acid and glycerol

molecules It is found in the blood

gastric juices pancreatic secretions

intestinal juices and adipose tissues

F A C E B O O K

P A G E h t t p w w w f a c e b o o k c o m s s c m e n t o r s o f f i c i a l P a g e | 32

FOR MORE UPDATES AND MORE MATERIAL DO LIKE OUR FACEBOOK PAGE httpwwwfacebookcomsscmentorsofficial

Q134 The arrangement of leaves on an

axis or stem is called

एकअकषयातनहॳपरपनततयोकीयवसिाकह८कयाकहाजाताहहॴ SSC CHSL Science (biology) 2016

Question Paper

24-Jan-2017

Options

1) Phyllotaxy

फाइिह८टहॴकसी 2) Vernation

वनिन

3) Venation

वहॳनहॳिन

4)Phytotaxy

फाइटह८टहॴकसी Correct Answer Phyllotaxy

In botany phyllotaxis or phyllotaxy is

the arrangement of leaves on a plant

stem (from Ancient Greek phyacutellon

ldquoleafrdquo and taacutexis ldquoarrangementrdquo)

Phyllotactic spirals form a distinctive

class of patterns in nature

Q135 The study of Cells is also known

as

कह८लिकाओकहॳ अधययनकह८ mdashmdashndash

भीकहाजाताहहॴ 24-Jan-2017

Options

1) Cytology

सायटह८िह८जी 2) Physiology

कफनजयह८िह८जी 3) Nucleology

नयककमयह८िह८जी 4)Cellology

सहॳिह८िह८जी Correct Answer Cytology

Q136 Which of the following scientists

is also known as the Father of Biology

नननननलिखितमसहॳककसवहॴजञाननककह८ ldquoजीवपवजञानकहॳ जनकrdquoकहॳ नामसहॳभीजानाजाताहहॴ 24-Jan-2017

Options

1) Herbert Spencer

हबयटयसपसर

2) Aristotle

अरसत 3) Lamarck

िहॳमाकय 4)Darwin

िापवयन

Correct Answer Aristotle

Q137 Which cells give rise to various

organs of the plant and keep the plant

growing

कह९नसीकह८लिकाएपह९धह८कहॳ लभननअगह८कह८जनमदहॳतीहहॴऔरपह९धह८कह८बढ़नहॳममददकरतीहहॴ

24-Jan-2017

Options

1) Permanent

सिायी 2) Dermal

तवचीय

3) Meristematic

मररसटहॳमटटक

4)Mature

परह८ढ़

Correct Answer Meristematic

A meristem is the tissue in most plants

containing undifferentiated cells

(meristematic cells) found in zones of

the plant where growth can take place

Q138 Rodentia Muridae is the scientific

name of

F A C E B O O K

P A G E h t t p w w w f a c e b o o k c o m s s c m e n t o r s o f f i c i a l P a g e | 33

FOR MORE UPDATES AND MORE MATERIAL DO LIKE OUR FACEBOOK PAGE httpwwwfacebookcomsscmentorsofficial

रह८िहॳलियानयररिी mdashmdash- कावहॴजञाननकनामहहॴ 24-

Jan-2017

Options

1) Mouse

चहा 2) Squirrel

चगिहरी 3) Monkey

बदर

4) Lizard

नछपकिी Correct Answer Mouse

Q139 Name the scientist who proposed

the cell theory

कह८लिकालसदातकापरसतावदहॳनहॳवािहॳवहॴजञाननककानामबताइए 24-Jan-2017

Options

1) Schleiden and Schwann

िीमिनऔरशरववान

2) Lamarck

िहॳमाकय 3) Treviranus

टरहॳवायरहॳनस

4)Whittaker and Stanley

हीटकरऔरसटहॳनिहॳ Correct Answer Schleiden and

Schwann

Q140 The flower with the worldrsquos

largest bloom is

दननयाकासबसहॳबड़ाफिखििनहॳवािा mdashmdashndash हहॴ 24-Jan-2017

Options

1) Pando

पािह८ 2) Posidonia

पह८सीिह८ननया 3) Rafflesia arnoldii

ररफिहॳलियाअनोमिी 4)Helianthus annuus

हहॳलिएनिसएनयअस

Correct Answer Rafflesia arnoldii

Rafflesia arnoldii is a species of

flowering plant in the parasitic genus

Rafflesia It is noted for producing the

largest individual flower on earth It has

a very strong and horrible odour of

decaying flesh earning it the nickname

ldquocorpse flower

Q141 Deficiency of which vitamin

causes night blindness

ककसपवटालमनकीकमीकहॳ कारणरतौधीहह८ताहहॴ 24-Jan-2017

Options

1) Vitamin K

पवटालमन K

2) Vitamin C

पवटालमन C

3) Vitamin B1

पवटालमन B1

4)Vitamin A

पवटालमन A

Correct Answer Vitamin A

Q142 Nongreen plants lack which of the

following

गहॴर-

हररतवनसपनतमनननननलिखितमसहॳककसकीकमीहह८तीहहॴ

24-Jan-2017

Options

1) Chlorophyll

किह८रह८कफि

2) Lycophyll

िायकह८कफि

3) Cyanophyll

F A C E B O O K

P A G E h t t p w w w f a c e b o o k c o m s s c m e n t o r s o f f i c i a l P a g e | 34

FOR MORE UPDATES AND MORE MATERIAL DO LIKE OUR FACEBOOK PAGE httpwwwfacebookcomsscmentorsofficial

सायनह८कफि

4)Phototropism

फह८टह८टरोपपजम

Correct Answer Chlorophyll

Q143 Organisms that use light to

prepare food are known as

जह८जीवपरकािकाउपयह८गकरभह८जनतहॴयारकरतहॳहहॴ उनह mdashmdash- कहॳ पमजानजाताहहॴ 24-Jan-2017

Options

1) Autotrophs

सवपह८षी 2) Heterotrophs

पवषमपह८षज

3) Omnivores

सवायहारी 4)Decomposers

पवघटनकरनहॳवािा Correct Answer Autotrophs

autotrophs often make their own food

by using sunlight carbon dioxide and

water to form sugars which they can use

for energy Some examples of

autotrophs include plants algae and

even some bacteria Autotrophs

(producer) are important because they

are a food source for heterotrophs

(consumers)

A heterotroph is an organism that

ingests or absorbs organic carbon

(rather than fix carbon from inorganic

sources such as carbon dioxide) in order

to be able to produce energy and

synthesize compounds to maintain its

life Ninety-five percent or more of all

types of living organisms are

heterotrophic including all animals and

fungi and some bacteria

Q144 Which of the following is a

primary function of haemoglobin

नननननलिखितमसहॳकह९नसाटहमह८गिह८बबनकाएकपरािलमककाययहहॴ

25-Jan-2017

Options

1) Utilization of energy

उजायकाउपयह८गकरना 2) Prevention of anaemia

रकतामपताहह८नहॳसहॳरह८कना 3) Destruction of bacteria

बहॴकटीररयाकापवनािकरना 4) To transport oxygen

ऑकसीजनकावहनकरना Correct Answer To transport oxygen

Q145 Vascular bundles are absent in

सवहनीबिि mdashmdash- मअनपनसतिरहतहॳहहॴ 25-Jan-2017

Options

1) Bryophyta

िायह८फाइटा 2) Pteridophyta

टहॳररिह८फाईटा 3) Gymnosperms

नजननह८सपमय 4) Angiosperms

एननजयह८सपहॳनसय Correct Answer Bryophyta

Q146 Sauria Lacertidae is the scientific

name of

सहॴररयािहॳसरटाईिी mdashmdashndash कावहॴजञाननकनामहहॴ 25-Jan-2017

Options

1) Crocodile

मगरमचछ

2) Hippopotamus

टहपपह८पह८टहॳमस

3) Lizard

नछपकिी 4) House fly

F A C E B O O K

P A G E h t t p w w w f a c e b o o k c o m s s c m e n t o r s o f f i c i a l P a g e | 35

FOR MORE UPDATES AND MORE MATERIAL DO LIKE OUR FACEBOOK PAGE httpwwwfacebookcomsscmentorsofficial

घरहॳिमकिी Correct Answer Lizard

Q147 Which type of pathogen causes

the water-borne disease SARS (Severe

Acute Respiratory Syndrome)

ककसपरकािकारह८गज़नकजिजननतबीमारीसासयकाकारणबनताहहॴ 25-Jan-2017

Options

1) Viral

वायरि

2) Parasitic

परजीवी 3) Protozoan

परह८टह८जअन

4) Bacterial

बहॴकटीररयि

Correct Answer Viral

Q148 Which of the following organs

produces the enzyme lipase

नननननलिखितमसहॳकह९नसाअगिायपहॳजएजाइमउतपननकरताहहॴ 25-Jan-2017

Options

1) Pancreas

अगनयािय

2) Large Intestine

बड़ीआत

3) Liver

नजगर

4) Small Intestine

छह८टीआत

Correct Answer Pancreas

Q149 A is a long internode forming the

basal part or the whole of a peduncle

एक mdashmdash- एकिबाइटरनह८िहहॴ जह८ननचिाटहससायासनपणयिठिबनताहहॴ 25-

Jan-2017

Options

1) Rhizome

परकद

2) Rachis

महॳ दि

3) floral axis

पषपअकष

4) Scape

भगदड़

Correct Answer scape

Q150 ndash Which of the following

organisms are considered to be both

Living and Non-living

नननननलिखितमसहॳकह९नसहॳजीवाणकह८जीपवतऔरअजीपवतमानाजाताहहॴ

25-Jan-2017

Options

1) Bacteria

बहॴकटीररया 2) Fungi

कवक

3) Algae

िहॴवाि

4)Virus

वायरस

Correct Answer Virus

They are considered to be living as they

possess a protein coat as a protective

covering DNA as the genetic material

etc

They are said to be non-living as they

can be crystallised and they survive for

billions of years They can tolerate high

temperatures freezing cold

temperatures ultra-violet radiations etc

Q151 Deficiency of fluorine causes

which of the following

फिह८ररनकीकमीकहॳ कारणनननननलिखितमसहॳकयाहह८ताहहॴ

F A C E B O O K

P A G E h t t p w w w f a c e b o o k c o m s s c m e n t o r s o f f i c i a l P a g e | 36

FOR MORE UPDATES AND MORE MATERIAL DO LIKE OUR FACEBOOK PAGE httpwwwfacebookcomsscmentorsofficial

27-Jan-2017

Options

1) Dental Caries

िटिकहॴ ररज

2) Scurvy

सकवरी 3) Anaemia

रकतामपता 4) Arthritis

गटठया Correct Answer Dental Caries

Q152 In a Punnett Square with the

cross AaBb x AaBb how many Aabb

genotypes would be created

पनहॳटसककायरमिह८स AaBb x AaBb कहॳ साि

ककतनहॳ Aabb जीनह८टाइपबनगहॳ 27-Jan-2017

Options

1) 1

2) 8

3) 2

4) 3

Correct Answer 2

Q153 Which of the following is the

Controlling Center of the Cell

नननननलिखित म सहॳ कह८लिकाका ननयतरण

क दर कह९न हहॴ

27-Jan-2017

Options

1) Nucleus

क दर

2) Plasma

पिाजमा 3) Lysosome

िायसह८सह८म

4) Chromosome

िह८मह८सह८म

Correct Answer Nucleus

The control centre of the cell is the

nucleus in eukaryotic cells The nucleus

contains genetic material in the form of

DNA

Q154 Myopia affects which of the

following organs

मायह८पपयानननननलिखितअगह८मसहॳककसहॳपरभापवतकरताहहॴ

25-Jan-2017

Options

1) Heart

हदय

2) Skin

तवचा 3) Eyes

आािहॳ 4)Mouth

मह

Correct Answer Eyes

Q155 Which of the following bears

flowers

नननननलिखितमसहॳकह९नफिधारणकरताहहॴ

25-Jan-2017

Options

1) Bryophyta

िायह८फाइटा 2) Pteridophyta

टहॳरीिह८फाईटा 3) Gymnosperms

नजननह८सपमय 4)Angiosperms

एननजयह८सपमय Correct Answer Angiosperms

Q156 Oxygenated blood flows out of the

heart through the

ऑकसीजनयकतरकत mdashmdashmdash

कहॳ माधयमसहॳहदयकहॳ बाहरबहताहहॴ 25-Jan-2017

F A C E B O O K

P A G E h t t p w w w f a c e b o o k c o m s s c m e n t o r s o f f i c i a l P a g e | 37

FOR MORE UPDATES AND MORE MATERIAL DO LIKE OUR FACEBOOK PAGE httpwwwfacebookcomsscmentorsofficial

Options

1) Aorta

महाधमनी 2) pulmonary artery

फहॳ फड़हॳकीधमनी 3) vena cava

वहॳनाकावा 4)Atrium

चह९क

Correct Answer aorta

Q157 Blood leaving the liver and

moving towards the

heart has a higher concentration of

नजगरसहॳननकिकरहदयकीतरफजानहॳवािहॳरकतम mdashmdashmdashmdash कीउचचसादरताहह८तीहहॴ 27-Jan-2017

Options

1) Lipids

लिपपडस

2) Urea

यररया 3) Bile Pigments

पपततकहॳ रगकरण

4) Carbon dioxide

काबयनिायऑकसाइि

Correct Answer Bile Pigments

Urea is nitrogen containing substance

which is produced in the liver in order

to deal with excess amino-acids in the

body As urea is produced it leaves the

liver in the blood stream and passes via

the circulatory system to all parts of the

body

Q158 Bulb is a modification of which

part of a plant

बमबएकपह९धहॳकहॳ ककसटहससहॳकाएक पातरणहह८ताहहॴ 27-Jan-2017

Options

1) The root

जड़

2) The stem

तना 3) The radicle

मिाकर

4)The fruit

फि

Correct Answer The stem

Q159 Which of the following carries

blood away from the heart to different

body parts

इनमहॳसहॳकह९नरकतकह८हदयसहॳिरीरकहॳ पवलभननअगह८तकिहॳजातीहहॴ

27-Jan-2017

Options

1) Arteries

धमननया 2) Nerves

तबतरहाए

3) Capillaries

कहॳ लिकाए

4)Veins

नसहॳ Correct Answer Arteries

Q160 The series of processes by which

nitrogen and its compounds are

interconverted in the environment and

in living organisms is called

27-Jan-2017

Options

1)Absorption of Nitrogen

2)Ammonification

3)Nitrogen Fixation

4)Nitrogen Cycle

Correct Answer Nitrogen Cycle

Ammonification or Mineralization is

performed by bacteria to convert

organic nitrogen to ammonia

F A C E B O O K

P A G E h t t p w w w f a c e b o o k c o m s s c m e n t o r s o f f i c i a l P a g e | 38

FOR MORE UPDATES AND MORE MATERIAL DO LIKE OUR FACEBOOK PAGE httpwwwfacebookcomsscmentorsofficial

Nitrification can then occur to convert

the ammonium to nitrite and nitrate

Nitrogen fixation is a process by which

nitrogen in the Earthrsquos atmosphere is

converted into ammonia (NH3) or other

molecules available to living organisms

Q161 BCG vaccine is given to protect

from which of the following

बीसीजीकाटटकानननननलिखितमसहॳककसकहॳ बचावकहॳ लिएटदयाजातहहॴ

27-Jan-2017

Options

1) Jaundice

पीलिया 2) Anaemia

रकतमपता 3) Tuberculosis

कषयरह८ग

4) Polio

पह८लियह८ Correct Answer Tuberculosis

Q162 Parallel venation is found in

समानतरवहॳनहॳिन mdashmdashmdash- मपायाजाताहहॴ 27-Jan-2017

Options

1) plants which are monocots

पह९धहॳजह८एकबीजपतरीहह८तहॳहहॴ 2) plants which have a dicot stem

वहॳपह९धहॳनजनकातनादपवदलियहह८ताहहॴ 3) plants with leaves similar to Tulsi

वहॳपह९धहॳनजनकीपनततयतिसीकीपनततयोकहॳ समानहह८तहॳहहॴ 4)plants with tap roots

टहॳप टवािहॳपह९धहॳ Correct Answer plants which are

monocots

Q163 The hardest part of the body is

िरीरकासबसहॳकठह८रभाग mdashndash हहॴ 27-Jan-2017

Options

1) Bones

हडडिय

2) Tooth Enamel

दातकहॳ इनहॳमि

3) Skull

िह८पड़ी 4) Spinal Cord

महॳ रजज

Correct Answer Tooth Enamel

Q164 Which type of pathogen causes

the waterborne disease E coli Infection

ककसपरकारकारह८गजननकजिजननतरह८गईकह८िाईसिमणकाकारणबनताहहॴ 27-Jan-2017

Options

1) Protozoan

परह८टह८जआ

2) Parasitic

परजीवी 3) Bacterial

बहॴकटीररयि

4)Viral

वायरि

Correct Answer Bacterial

Q165 The amount of blood filtered

together by both the kidneys in a 70 kg

adult male human in a minute is

70 की गरा वािहॳएकवयसकप षमएकलमनटमदह८नोगदकहॳदवाराएकसािचाबनीगयीरकतकीमातरहह८तीहहॴ 29-Jan-2017

Options

1) 1100 ml

1100 लमलि

2) 100 ml

F A C E B O O K

P A G E h t t p w w w f a c e b o o k c o m s s c m e n t o r s o f f i c i a l P a g e | 39

FOR MORE UPDATES AND MORE MATERIAL DO LIKE OUR FACEBOOK PAGE httpwwwfacebookcomsscmentorsofficial

100 लमलि

3) 1500 ml

1500 लमलि

4) 500 ml

500 लमलि

Correct Answer 1100 ml

Q166 Which feature of a plant helps to

distinguish a monocot from a dicot

पह९धहॳकीवहकह९नसीपविहॳषताहहॴजह८एकदपवदलियहॳऔरएकएकदिीयपह९धहॳसहॳभहॳदकरनहॳममददकरतीहहॴ 29-Jan-2017

Options

1) Pollination

परागम

2) Venation

वहॳनहॳिन

3) Vernation

वनिन

4) Aestivation

एसटीवहॳिहॳन

Correct Answer venation

Q167 The Mutation Theory was

proposed by

उतवररवतयनकालसदात mdashmdashndash

कहॳ दवरापरसतापवतककयाजाताहहॴ 29-Jan-2017

Options

1) Charles Lyell

चामसयलियहॳि

2) William Smith

पवलियमनसमि

3) Hugo De Vries

हयगह८िीराईस

4)Harrison Schmitt

हहॳरीसननसमट

Correct Answer Hugo De Vries

Q168 Which type of pathogen causes

the waterborne disease HepatitisA

ककसपरकारकहॳ रह८गजनकजिजननतरह८गहहॳपहॳटाइटटस-A काकारणबनताहहॴ

29-Jan-2017

Options

1) Parasitic

परजीवी 2) Viral

वायरि

3) Protozoan

परह८टह८जआ

4) Bacterial

बहॴकटीररयि

Correct Answer Viral

Q169 In a Punnett Square with the

cross AaBb x Aabb how many AaBb

genotypes would be created

पनहॳटसकवायरमिह८स AaBb x Aabb

कहॳ सािककतनहॳ AaBb जीनह८टाइपबनगहॳ 29-Jan-

2017

Options

1) 4

2) 1

3) 7

4) 6

Correct Answer 4

Q170 Arboreal Ateles is the scientific

name of

अिह८ररयिएटटलिस mdashmdashmdash कावहॴजञाननकनामहहॴ 29-Jan-2017

Options

1) Squirrel

चगिहरी 2) Sparrow

गह८रहॴया 3) Lizard

नछपकिी 4) Spider monkey

F A C E B O O K

P A G E h t t p w w w f a c e b o o k c o m s s c m e n t o r s o f f i c i a l P a g e | 40

FOR MORE UPDATES AND MORE MATERIAL DO LIKE OUR FACEBOOK PAGE httpwwwfacebookcomsscmentorsofficial

मकड़ीबदर

Correct Answer Spider monkey

Q171 Which type of pathogen causes

the waterborne disease Salmonellosis

ककसपरकारकारह८गाणजिजननतबीमारीसािमह८नहॳिह८लसज़काकारकहहॴ

29-Jan-2017

Options

1) Algal

िहॳवालियहॳ 2) Parasitic

परजीवी 3) Bacterial

बहॴकटीररयि

4)Viral

वायरि

Correct Answer Bacterial

An infection with salmonella bacteria

commonly caused by contaminated food

or water

Symptoms include diarrhoea fever

chills and abdominal pain

Q172 is a condition in which there is a

deficiency of red cells or of haemoglobin

in the blood

mdashmdash-

एकनसिनतहहॴनजसमहॳरकतमिािकह८लिकाओकीयाहीमह८गिह८बबनकीकमीहह८तीहहॴ 29-Jan-2017

Options

1) Albinism

एनमबननजम

2) Propyria

परह८पीररया 3) Anaemia

एनीलमया 4)Keloid disorder

कहॳ िह८इिडिसओिर

Correct Answer Anaemia

Q173 Ananas comosus is the scientific

name of

Options

अनानासकह८मह८सस mdashmdashmdashndash

कावहॴजञाननकनामहहॴ 29-Jan-2017

1) Custard Apple

सीताफि

2) Pineapple

पाइनएपपि

3) Bamboo

बास

4)Pomegranate

अनार

Correct Answer Pineapple

Q174 Which organ produces insulin

कह९नसाअगइनसलिनपहॴदाकरताहहॴ 29-Jan-

2017

Options

1) Liver

यकत

2) Thyroid gland

िायराइिगरिी 3) Spleen

पिीहा 4)Pancreas

अगरयिय

Correct Answer Pancreas

Q175 Which of the following disease is

not caused by water pollution

नननननलिखितमसहॳकह९नसारह८गपानीकहॳ परदषणकहॳकारणनहीहह८ता

29-Jan-2017

Options

1) Cholera

हहॴजा 2) Typhoid

F A C E B O O K

P A G E h t t p w w w f a c e b o o k c o m s s c m e n t o r s o f f i c i a l P a g e | 41

FOR MORE UPDATES AND MORE MATERIAL DO LIKE OUR FACEBOOK PAGE httpwwwfacebookcomsscmentorsofficial

टाइफाइि

3) Asthma

दमा 4)Diarrhoea

दसत

Correct Answer Asthma

Q176 Ocimum tenuiflorum is the

scientific name of

ओलिलममटहॳयईफिह८रमइसकावहॴजञाननकनाम mdash

ndash हहॴ 30-Jan-2017

Options

1) Neem

नीम

2) Mango

आम

3) Babul

बबि

4)Tulsi

तिसी Correct Answer Tulsi

Q177 Which gland secretes bile a

digestive fluid

कह९नसीगरिीपपतत एकपाचनतरिपरदािय सरापवतकरतीहहॴ 30-Jan-2017

Options

1) Pancreas

अगनयािय

2) Liver

यकत

3) Thyroid

िायराइि

4) Testes

टहॳनसटस

Correct Answer liver

Q178 In which of the following the

dominant phase is Gametophyte

नननननलिखितमसहॳककसकहॳ परमिचरणयगमकह८दपवधद (Gametophyte)हहॴ 30-Jan-2017

Options

1) Bryophyta

िायह८फाइटा 2) Pteridophyta

टहॳररिह८फाइटा 3) Gymnosperms

नजननह८सपमय 4) Angiosperms

एननजयह८सपमय Correct Answer Bryophyta

Q179 Anaerobic respiration refers to

which of the following

नननननलिखितमसहॳककसहॳअवायवीयशवसनकहाजाताहहॴ

30-Jan-2017

Options

1) Respiration without Oxygen

ऑकसीजनकहॳ बबनाशवसन

2) Respiration with Oxygen

ऑकसीजनकहॳ सािशवसन

3) Respiration without CO2

काबयनिायऑकसाइिकहॳ बबनाशवसन

4) Respiration with CO2

काबयनिायऑकसाइिकहॳ सािशविन

Correct Answer Respiration without

Oxygen

Q180 Which type of pathogen causes

the waterborne disease Cholera

ककसपरकारकारह८गजनकजिजननतरह८गहहॴजाकाकारणबनताहहॴ

30-Jan-2017

Options

1) Algal

िहॴवालियहॳ

F A C E B O O K

P A G E h t t p w w w f a c e b o o k c o m s s c m e n t o r s o f f i c i a l P a g e | 42

FOR MORE UPDATES AND MORE MATERIAL DO LIKE OUR FACEBOOK PAGE httpwwwfacebookcomsscmentorsofficial

2) Bacterial

बहॴकटीररयि

3) Protozoan

परह८टह८जआ

4) Viral

वायरि

Correct Answer Bacterial

Q181 To which class does

Oxyreductases transferases hydrolases

belong

ओकसीररिकटहॳसटरासफरहॳजहॳस

हाइडरह८िहॳसहॳसककसवगयमआतहॳहहॴ 30-Jan-2017

Options

1) Hormones

हारमोस

2) Enzymes

एजाइनस

3) Proteins

परह८टीनस

4) Vitamins

पवटालमनस

Correct Answer Enzymes

Q182 Which of the following is not true

about Gymnosperms

ननननमसहॳकह९नसीबातअनावतबीजीकहॳ बारहॳमसचनहीहहॴ 30-Jan-2017

Options

1) Dominant phase is saprophytes

परमिचरणसहॳपरह८फाइटसहह८ताहहॴ 2) Vascular bundles are absent

सवहनीबििअनपनसितहह८ताहहॴ 3) spores are heterospores

बीजाणहहॳटहॳरह८सपह८रसहह८तहॳहहॴ 4) Flowers are absent

फिअनपनसितहह८तहॳहहॴ

Correct Answer Vascular bundles are

absent

Q183 The name of first mammal clone sheep is

भहॳड़कीपरिमसतनपायीपरनत प (किह८न)

कानामहहॴ 30-Jan-2017

Options

1) Noori

नरी 2) Dolly

िॉिी 3) Louise

िसी 4)Durga

दगाय Correct Answer Dolly

Q184 Which type of pathogen causes

the water-borne disease Typhoid fever

ककसपरकारकारह८गजनकजिजननतरह८गटाइफाइिबिारकाकारणबनताहहॴ 30-Jan-2017

Options

1) Algal

िहॴवािीय

2) Parasitic

परजीवी 3) Protozoan

परह८टह८जनअन

4)Bacterial

बहॴकटीररयि

Correct Answer Bacterial

Q185 In which part of the cell are

proteins made

कह८लिकाकहॳ ककसटहससहॳमपरह८टीनबनायाजाताहहॴ

31-Jan-2017

Options

1) Reticulum

रहॳटटकिम

F A C E B O O K

P A G E h t t p w w w f a c e b o o k c o m s s c m e n t o r s o f f i c i a l P a g e | 43

FOR MORE UPDATES AND MORE MATERIAL DO LIKE OUR FACEBOOK PAGE httpwwwfacebookcomsscmentorsofficial

2) Golgi apparatus

गह८मजीएपहॳरहॳटस

3) Ribosomes

ररबह८सह८नस

4) Lysosome

िायसह८सह८नस

Correct Answer ribosomes

Proteins are produced by stringing

amino acids together in the order

specified by messenger RNA strands

that were transcribed from DNA in the

cell nucleus The process of synthesizing

a protein is called translation and it

occurs on ribosomes in the cytoplasm of

a cell

Q186 Polio is a disease caused by which

of the following

नननननलिखितमसहॳपह८लियह८कीबबमारह८हह८नहॳकाकारणकयाहहॴ

31-Jan-2017

Options

1) Bacteria

बहॴकटीररयि

2) Mosquito

मचछर

3) Virus

वायरस

4) Cockroach

नतिच हॳ Correct Answer Virus

Polio or poliomyelitis is a crippling and

potentially deadly infectious disease It

is caused by the poliovirus

Q187 ndash Hay fever is a sign of which of

the following

हहॳकफवरनननननलिखितमसहॳककसकाएकसकहॳ तहहॴ

31-Jan-2017

Options

1) Old Age

वदावसिा 2) Malnutrition

कपह८सण

3) Allergy

एिनजय 4) Over Work

अतयचधककाययकरना Correct Answer Allergy

Q188 How many chromosomes does a

human cell contain

एकमानवकह८लिकामककतनहॳगणसतरहह८तहॳहहॴ

29-Jan-2017

Options

1) 6

2) 26

3) 46

4) 66

Correct Answer 46

In humans each cell normally contains

23 pairs of chromosomes for a total of

46 Twenty-two of these pairs called

autosomes look the same in both males

and females The 23rd pair the sex

chromosomes differ between males and

females

Q189 Which of the following is not true

about Bryophyta

ननननमसहॳकह९नसीबातिायह८फाइटकहॳ बारहॳमसचनहीहहॴ 31-Jan-2017

Options

1) Dominant phase is gametophytes

परमिचरणगहॳलमतह८फाइटसहह८ताहहॴ 2) Main plant body is haploid

पह९धहॳकामखयिरीरअगखणतहह८ताहहॴ 3) Spores are homospores

बीजाणहह८मह८सफह८रसहह८तहॳहहॴ 4) Flowers are present

फिमह८जदहह८तहॳहहॴ Correct Answer Flowers are present

F A C E B O O K

P A G E h t t p w w w f a c e b o o k c o m s s c m e n t o r s o f f i c i a l P a g e | 44

FOR MORE UPDATES AND MORE MATERIAL DO LIKE OUR FACEBOOK PAGE httpwwwfacebookcomsscmentorsofficial

Q190 Which aquatic animal has

trailing tentacles

ककसजिीयजानवरकहॳ पीछहॳचिनहॳवािहॳटहॳटकिसहह८तहॳहहॴ

31-Jan-2017

Options

1) Sea horse

समदरीघह८िा 2) Corals

मगा 3) Jelly fish

जहॳिीमछिी 4) Star fish

तारामछिी Correct Answer Jelly fish

Jellyfish with its umbrella-shaped bell

and trailing tentacles

Q191 Which type of pathogen causes

the water-borne disease Poliomyelitis

(Polio)

ककसपरकारकारह८गजनकजिजननतरह८गपह८लियह८मायहॳटटस (पह८लियह८) काकारणहहॴ 31-Jan-

2017

Options

1) Parasitic

परजीवी 2) Algal

िहॴवालिय

3) Viral

वायरि

4) Bacterial

बहॴकटीररयि

Correct Answer Viral

Q192 The outer white part of the eye

that protects the inner structures is

आािकाबाहरीसफहॳ दटहससाजह८आतररकसरचनाओकीरकषाकरताहहॴ वह mdashmdashmdash हहॴ 31-Jan-

2017

Options

1) Iris

आयररस

2) Sclera

सकिहॳरा 3) Retina

रहॳटटना 4) Cornea

कह८ननयया Correct Answer Sclera

Q193 Proteins are made up of

परह८टीनकाननमायण mdashndash सहॳहह८ताहहॴ 31-Jan-2017

Options

1) Amino acids

एलमनह८अनि

2) Fatty acids

वसायकतअनि

3) Glucose

गिकह८ज

4)Nucleotides

नयनकियह८टाईिस

Correct Answer Amino acids

Q194 Moringa Oleifera is the scientific

name of

मह८ररगओलिफहॳ रा mdashmdashndash कावहॴजञाननकनामहहॴ 31-Jan-2017

Options

1) Banyan

बरगद

2) Gulmohar

गिमह८हर

3) Amla

आमिा

F A C E B O O K

P A G E h t t p w w w f a c e b o o k c o m s s c m e n t o r s o f f i c i a l P a g e | 45

FOR MORE UPDATES AND MORE MATERIAL DO LIKE OUR FACEBOOK PAGE httpwwwfacebookcomsscmentorsofficial

4) Drumstick

डरमनसटक

Correct Answer Drumstick

Q195 Kidney stones are composed of

गदकीपिरी mdashndash सहॳबनीहह८तीहहॴ 1-Feb-2017

Options

1) Calcium Oxalate

कहॴ नमसयमओकजहॳिहॳट

2) Sodium Chloride

सह८डियमकिह८राइि

3) Magnesium Nitrate

महॳनगनलियमनाइतटरहॳट

4) Calcium Bicarbonate

कहॴ नमियमबायकबोनहॳट

Correct Answer Calcium Oxalate

Q196 ndash Which of the following is not

true about Angiosperms

ननननमसहॳकह९नसीबातआवतबीजीकहॳ बारहॳमसचनहीहहॴ 1-Feb-2017

Options

1) Dominant phase is gametophytes

परमिचरणगहॳलमतह८फाइटहह८ताहहॴ 2) Vascular bundles are present

सवहनीबििमह९जदहह८ताहहॴ 3) Spores are heterospores

बीजाणहहॳटहॳरह८सपह८रसहह८तहॳहहॴ 4) Seeds are covered

बीजढकहॳ हह८तहॳहहॴ Correct Answer Dominant phase is

gametophytes

Q197 All of the following are excretory

(waste) products of animals except

नननननलिखितमसहॳककसएककह८छह८ड़करअनयसभीपराखणयोदवाराउतसनजयतपदाियहहॴ 1-Feb-

2017

Options

1) Uric Acid

यररकएलसि

2) Ammonia

अमह८ननया 3) Carbohydrates

काबोहाइडरहॳट

4) Urea

यररया Correct Answer Carbohydrates

In animals the main excretory products

are carbon dioxide ammonia (in

ammoniotelics) urea (in ureotelics) uric

acid (in uricotelics) guanine (in

Arachnida) and creatine

Q198 RNA is a polymeric molecule

What does RNA stand for

आरएनइएएकबहिकआणहहॴ इसकाकापवय पकयाहहॴ 1-Feb-2017

Options

1) Rado Nuclear Acid

रािह८नयनकियरएलसि

2) Ribo Nucleic Acid

राइबह८नयनकिकएलसि

3) Rhino Nuclear Acid

हाइनह८नयनकियरएलसि

4) Resto Nucleus Acid

रहॳसटह८नयकिीयसएलसि

Correct Answer Ribo Nucleic Acid

Q199 Which organ does detoxification

and produces chemicals needed for

digestion

कह९नसाअगपवषहरणकरताहहॴऔरपाचनकहॳ लिएआवशयकरसायनोकह८पहॴदाकरताहहॴ 1-Feb-

2017

Options

1) Salivary glands

िारगरचिया 2) Pancreas

अगनयािय

F A C E B O O K

P A G E h t t p w w w f a c e b o o k c o m s s c m e n t o r s o f f i c i a l P a g e | 46

FOR MORE UPDATES AND MORE MATERIAL DO LIKE OUR FACEBOOK PAGE httpwwwfacebookcomsscmentorsofficial

3) Thyroid gland

िायराइिगरिी 4) Liver

यकत

Correct Answer Liver

Q200 Psidium guajava is the scientific

name of

लसडियमगआजावा mdashmdash कावहॴजञाननकनामहहॴ 1-

Feb-2017

Options

1) Guava

अम द

2) Mango

आम

3) Bamboo

बास

4) Jack fruit

कटहि

Correct Answer Guava

Q201 Which drug is used as a Blood

Thinner

चधरकह८पतिाकरनहॳकहॳ पमककसदवाकापरयह८गककयाजाताहहॴ

1-Feb-2017

Options

1) Warfarin

वाफर न

2) Tramadol

टरहॳमािह८ि

3) Azithromycin

एनजरह८मायलसन

4) Hydralazine

हाइडरह८िहॳनजन

Correct Answer Warfarin

Q202 Which of the following disease is

caused due to the deficiency of protein

परह८टीनकीकमीकहॳ कारणनननननलिखितमसहॳकह९नसारह८गहह८ताहहॴ 1-Feb-2017

Options

1) Arthritis

गटठया 2) Kwashiorkor

कािीओकय र

3) Goitre

गाइटर

4) Night Blindness

रतह९चध

Correct Answer Kwashiorkor

Q203 A is species of plant that has

adapted to survive in an environment

with little liquid water

mdashmdashndashपह९धहॳकीएकऐसहॳऐसहॳपरजानतहहॴ नजसनहॳकमपानीवािहॳवातावरणमजीपवतरहनहॳकहॳलिएअनकिनहहॴ 1-Feb-2017

Options

1) Xerophyte

म दपवद

2) Hydrophyte

जिीयपादप

3) Mesophyte

समह८दपवद

4) Thallophyte

िहॴिह८फाइटा Correct Answer xerophyte

xerophyte is a species of plant that has

adapted to survive in an environment

with little liquid water such as a desert

or an ice- or snow-covered region in the

Alps or the Arctic

Mesophytes are terrestrial plants which

are adapted to neither a particularly

dry nor particularly wet environment

An example of a mesophytic habitat

would be a rural temperate meadow

F A C E B O O K

P A G E h t t p w w w f a c e b o o k c o m s s c m e n t o r s o f f i c i a l P a g e | 47

FOR MORE UPDATES AND MORE MATERIAL DO LIKE OUR FACEBOOK PAGE httpwwwfacebookcomsscmentorsofficial

which might contain goldenrod clover

oxeye daisy and Rosa multiflora

thallophyte any of a group of plants or

plantlike organisms (such as algae and

fungi) that lack differentiated stems

leaves and roots and that were formerly

classified as a primary division

(Thallophyta) of the plant kingdom

Q204 How many types of teeth are

there in humans

मनषयोमककतनहॳपरकारकहॳ दातहह८तहॳहहॴ

1-Feb-2017

Options

1) 4

2) 5

3) 2

4) 3

Correct Answer 4

teeth -Humans have four types of

teethincisors canines premolars and

molars each with a specific function

The incisors cut the food the canines

tear the food and the molars and

premolars crush the food

Q205 Carica papaya is the scientific name of

कहॴ ररकापपाया mdashmdashndash कावहॴजञाननकनामहहॴ 2-

Feb-2017

Options

1) Peepal

पीपि

2) Papaya

पपीता 3) Tamarind

इमिी 4) Drumstick

ढह८िकाछड़ी Correct Answer Papaya

Q206 Muscles get tired when there is

shortfall of

जब mdashndash कीकमीहह८तीहहॴतबपहॳिीयिकजातीहहॴ 2-Feb-2017

Options

1) Lactic acid

िहॴनकटकएलसि

2) Na+ ions

Na+ आयन

3) ATP

एटीपी 4) Sulphates

समफहॳ टस

Correct Answer ATP

ATP is the energy source muscle fibers

use to make muscles contract

muscle tissuersquos main source of energy

called adenosine triphosphate or ATP

As your muscles use up this energy

source they become tired and fatigued

Oxygen is the key ingredient that helps

create new ATP to replenish the burned

up ATP in your muscles

Q207 Artocarpus integra is the

scientific name of आटह८कापयसइटीगरा mdashmdashmdash कावहॴजञाननकनामहहॴ 2-Feb-2017

Options

1) Guava

अम द

2) Pineapple

अनानास

3) Silver Oak

लसमवरओक

4) Jack fruit

कटहि

Correct Answer Jack fruit

Q208 Which organ stores fat soluble

vitamins

कह९नसाअगवसामघिनिीिपवटालमनह८काभिाराकरताहहॴ

2-Feb-2017

F A C E B O O K

P A G E h t t p w w w f a c e b o o k c o m s s c m e n t o r s o f f i c i a l P a g e | 48

FOR MORE UPDATES AND MORE MATERIAL DO LIKE OUR FACEBOOK PAGE httpwwwfacebookcomsscmentorsofficial

Options

1) Blood

रकत

2) Skin

तवचा 3) Liver

यकत

4) Pancreas

अगनयािय

Correct Answer Liver

Q209 Which disease is caused due to

deficiency of Iodine

आयह८िीनकहॳ कारणकह९नसारह८गहह८ताहहॴ 2-Feb-2017

Options

1) Rickets

ररकहॳ टस

2) Scurvy

सकवी 3) Goitre

गणमािा 4) Growth retardation

पवकासका कना Correct Answer Goitre

rickets A softening and weakening of

bones in children usually due to

inadequate vitamin D

Q210 Grevillea Robusta is the scientific name of

गरहॳपवलियारह८बसटा mdashmdashmdash- कापवजञाननकनामहहॴ 2-Feb-2017

Options

1) Peepal

पीपि

2) Teak

सागह९न

3) Silver Oak

लसमवरओक

4) Jack fruit

कटहि

Correct Answer Silver Oak

Q211 When a Cuttlefish is described as a Molluscs it is at which level of

classification

जबएककटिकफिकह८एकमह८िसकाकहॳ पमवखणयतककयाजाताहहॴतबयहॳवगीकरणकहॳ ककससतरपहॳनसितहहॴ 2-Feb-2017

Options

1) Class

वगय 2) Order

िम

3) Family

पररवार

4) Phylum

सघ

Correct Answer Phylum

Q212 Bambusa dendrocalmus is the

scientific name of बानबसािहॳडराकामस mdashmdashmdash कावहॴजञाननकनामहहॴ 3-Feb-2017

Options

1) Banyan

बरगद

2) Papaya

पपीता 3) Bamboo

बास

4) Pomegranate

अनार

Correct Answer Bamboo

Q213 Acinonyx Jubatus is the scientific name of

एलसनह८ननकसजयबहॳटस mdashmdashmdash

कावहॴजञाननकनामहहॴ 3-Feb-2017

F A C E B O O K

P A G E h t t p w w w f a c e b o o k c o m s s c m e n t o r s o f f i c i a l P a g e | 49

FOR MORE UPDATES AND MORE MATERIAL DO LIKE OUR FACEBOOK PAGE httpwwwfacebookcomsscmentorsofficial

Options

1) Bear

भाि 2) Horse

घह८िा 3) Cheetah

चीता 4) Zebra

जहॳिा Correct Answer Cheetah

Q214 The pale yellow colour of urine is

due to the presence of which pigment

मतरकाफीकापीिारगरगदरयकहॳ उपनसिनतकहॳ कारणहह८ताहहॴ

3-Feb-2017

Options

1) Urochrome

यरह८िह८म

2) Urophyll

यरह८कफि

3) Chlorophyll

किह८रह८कफि

4) Chloroplast

किह८रह८पिासट

Correct Answer Urochrome

Q215 Which of the following constitute

to form a gene

नननननलिखितमसहॳकह९नसीचीज़एकजीनकागठनकरतीहहॴ

3-Feb-2017

Options

1) Polynucleotides

पह८िीनयनकियह८टाईडस

2) Hydrocarbons

हाइडरह८काबोस

3) Lipoproteins

िाईपह८परह८टीनस

4) Lipids

लिपपडस

Correct Answer Polynucleotides

Polynucleotide molecule is a biopolymer

composed of 13 or more nucleotide

monomers covalently bonded in a chain

DNA (deoxyribonucleic acid) and RNA

(ribonucleic acid) are examples of

polynucleotides with distinct biological

function

Q216 Vertebrates belongs to the

phylum

रीढ़कीहडिीवािहॳपराणी mdashmdashmdash

परजानतकहॳ अतगायतआतहॳहहॴ 3-Feb-2017

Options

1) Arthropoda

आरह८पह८ड़ा 2) Annelida

एननलििा 3) Cnidaria

ननिहॳररया 4) Chordata

कह८िटा Correct Answer Chordata

Q217 Punica granatum is the scientific name of

पननकगरहॳनहॳटस mdashmdashmdash कावहॴजञाननकनामहहॴ 3-Feb-2017

Options

1) Custard Apple

सीताफि

2) Gulmohar

गिमह८हर

3) Silver Oak

लसमवरओक

4) Pomegranate

अनार

Correct Answer Pomegranate

F A C E B O O K

P A G E h t t p w w w f a c e b o o k c o m s s c m e n t o r s o f f i c i a l P a g e | 50

FOR MORE UPDATES AND MORE MATERIAL DO LIKE OUR FACEBOOK PAGE httpwwwfacebookcomsscmentorsofficial

Q218 Between a tiger and an monkey

which of the following is different

एकबाघऔरबदरकहॳ बीचनननननलिखितमसहॳकह९नसीबातअिगहहॴ 3-Feb-2017

Options

1) Kingdom

राजय

2) Phylum

जानत

3) Order

िम

4) Class

वगय Correct Answer order

Q219 The artificial heart was invented by

कबतरमहदयका mdashmdashmdash

दवाराअपवषकारककयागयािा 3-Feb-2017

Options

1) Muhammad Yunus

महनमदयनस

2) Linus Yale Jr

िाइनसयहॳिजय

3) Gazi Yasargil

गाजीयासचगयि

4) Paul Winchell

पह९िपवमकि Correct Answer Paul Winchell

Q220 Tamarindus indica is the

scientific name of

टहॳमररनडसइडिका mdashmdash कावहॴजञाननकनामहहॴ 7-

Feb-2017

Options

1) Neem

नीम

2) Pineapple

अनानास

3) Tamarind

इमिी 4)Chiku

चीक

Correct Answer Tamarind

Q221 In eukaryotic cells synthesis of

RNA takes place in the

यकहॳ योटटककह८लिकाओमआरएनएकासशिहॳषण

mdashndash महह८ताहहॴ 7-Feb-2017

Options

1) Mitochondria

माईटह८कोडडरया 2) Centrioles

सटरीयह८मस

3) Ribosomes

ररबह८सह८नस

4) Nucleus

नयनकियस

Correct Answer nucleus

eukaryotic cell -Transcription is the

process of synthesizing ribonucleic acid

(RNA)Synthesis takes place within the

nucleus of eukaryotic cells or in the

cytoplasm of prokaryotes and converts

the genetic code from a gene in

deoxyribonucleic acid ( DNA ) to a

strand of RNA that then directs

proteinsynthesis

Q222 _________is caused by parasites

of the Plasmodium genus

पिाजमह८डियमजातीकहॳ परजीवी mdash- कहॳ कारणहहॴ 7-Feb-2017

Options

1) Dysentery

पहॳचचि

2) Malaria

मिहॳररया 3) Chickenpox

F A C E B O O K

P A G E h t t p w w w f a c e b o o k c o m s s c m e n t o r s o f f i c i a l P a g e | 51

FOR MORE UPDATES AND MORE MATERIAL DO LIKE OUR FACEBOOK PAGE httpwwwfacebookcomsscmentorsofficial

चहॳचक

4) Herpes

हहॳपपयस

Correct Answer Malaria

Q223 Carotene in fruits and vegetables

gives it which color

फिह८औरसनलजयोमनसितकहॳ रह८टीनउनहकह९नसारगपरदानकरताहहॴ 7-Feb-2017

Options

1) Green

हरा 2) Pink

गिाबी 3) Orange

नारगी 4) Blue

नीिा Correct Answer Orange

Q224 Equus Caballus is the scientific

name of

एकवसकहॴ बहॳिस mdashmdashndash कापवजञाननकनामहहॴ 7-Feb-2017

Options

1) Horse

घह८िा 2) Zebra

ज़हॳिा 3) Donkey

गधा 4) Buffalo

भस

Correct Answer Horse

Q225 Elapidae Naja is the scientific name of

एिीपीिीनाजा mdashmdash- कावहॴजञाननकनामहहॴ 8-Feb-2017

Options

1) Cobra

कह८बरा 2) Elephant

हािी 3) Eagle

ग ि

4) Owl

उमि Correct Answer Cobra

Q226 Which disease is caused due to

deficiency of Iron

िह८हकीकमीकहॳ कारणकह९नसारह८गहह८ताहहॴ 8-Feb-

2017

Options

1) Beriberi

बहॳरीबहॳरी 2) Tetany

टहॳटनी 3) Kwashiorkor

कवािीऔरकर

4) Anaemia

रकतामपता Correct Answer Anaemia

Beriberi is a disease caused by a vitamin

B-1 deficiency also known as thiamine

deficiency

Tetany can be the result of an

electrolyte imbalance Most often itrsquos a

dramatically low calcium level also

known as hypocalcemia Tetany can also

be caused by magnesium deficiency or

too little potassium Having too much

acid (acidosis) or too much alkali

(alkalosis) in the body can also result in

tetany

Kwashiorkor also known as

ldquoedematous malnutrition It is a form of

malnutrition caused by a lack of protein

in the diet

Anaemia means that you have fewer red

blood cells than normal or you have less

F A C E B O O K

P A G E h t t p w w w f a c e b o o k c o m s s c m e n t o r s o f f i c i a l P a g e | 52

FOR MORE UPDATES AND MORE MATERIAL DO LIKE OUR FACEBOOK PAGE httpwwwfacebookcomsscmentorsofficial

haemoglobin than normal in each red

blood cell

Q227 is a leaf where the leaflets are

arranged along the middle vein

mdashndashएकपततीहहॴजहापतरकह८कीरचनाक ररयालिराकहॳ आसपासहह८तीहहॴ 8-Feb-2017

Options

1) Pinnately compound leaf

पपनहॳटिीसयकतपतती 2) Palmately compound leaf

पामहॳटिीसयकतपतती 3) Compound leaf

सयकतपतती 4) Simple leaf

साधारणपतती Correct Answer Pinnately compound

leaf

Q228 Haustoria or sucking roots are

found in which of the following

हह८सटह८ररयायाचसनहॳवािीजड़हॳनननननलिखितमसहॳककसमपाईजातीहहॴ 8-Feb-2017

Options

1) Wheat

गहॳह

2) Mango

आम

3) Chestnut

चहॳसटनट

4) Cuscuta

कसकयटा Correct Answer Cuscuta

Haustorial roots -The roots of parasitic

plants which penetrate into the host

tissues to absorb nourishment are

called haustorial roots hellip Also known as suckingor parasitic roots

Q229 Equs Asinus is the scientific name

of

एकवसएलसनस mdashmdashndash कावहॴजञाननकनामहहॴ 8-

Feb-2017

Options

1) Donkey

गधा 2) Cow

गाय

3) Deer

टहरन

4) Kangaroo

कगा

Correct Answer Donkey

Q230 Ficus benghalensis is the scientific name of

फाईकसबहॳनगहॳिहॳलसस mdashndash कापवजञाननकनामहहॴ 8-Feb-2017

Options

1) Banyan

बरगद

2) Pineapple

अनानास

3) Babul

बबि

4) Tulsi

तिसी Correct Answer Banyan

Q231 Equus burchellii is the scientific name of

एकवसबचिी mdashmdash- कापवजञाननकनामहहॴ 8-Feb-2017

Options

1) Horse

घह८िा 2) Zebra

जहॳिा 3) Buffalo

F A C E B O O K

P A G E h t t p w w w f a c e b o o k c o m s s c m e n t o r s o f f i c i a l P a g e | 53

FOR MORE UPDATES AND MORE MATERIAL DO LIKE OUR FACEBOOK PAGE httpwwwfacebookcomsscmentorsofficial

भस

4) Ass

गधा Correct Answer Zebra

Page 31: COMPILATION OF ALL 72 SETS OF BIOLOGY SSC CHSL-2016 · OF BIOLOGY SSC CHSL-2016 PREPARED BY : SSC MENTORS BIOLOGY SPECIAL . F A C E B O O K P A G E : h t t p : / / w w w . f a c e

F A C E B O O K

P A G E h t t p w w w f a c e b o o k c o m s s c m e n t o r s o f f i c i a l P a g e | 30

FOR MORE UPDATES AND MORE MATERIAL DO LIKE OUR FACEBOOK PAGE httpwwwfacebookcomsscmentorsofficial

Poliomyelitis often called polio or

infantile paralysis is an infectious

disease caused by the poliovirus

Tetanusmdash A serious bacterial infection

that causes painful muscle spasms and

can lead to death

Leprosy also known as Hansenrsquos

disease (HD) is a long-term infection by

the bacterium Mycobacterium leprae or

Mycobacterium lepromatosis

Plague is an infectious disease caused by

the bacterium Yersinia pestis

Symptoms include fever weakness and

headache

Q126 Which organisms can help to

carry out Vermicomposting

कह९नसाजीववमीकनपह८नसटगममददकरताहहॴ

23-Jan-2017

Options

1) Nitrifying Bacteria

नाईटरीफाईगबहॴकटीररया 2) Earthworms

कहॴ चऐ

3) Algae

िहॴवि

4) Fungus

कवक

Correct Answer Earthworms

Q127 Contraction of heart is also

known as

हदयकहॳ सकचनकह८ mdash- भीकहाजाताहहॴ 23-Jan-

2017

Options

1) Systole

लससटह८ि

2) Aristotle

अरसत

3) Diastole

िायसटह८ि

4) Lub

मयब

Correct Answer Systole

Diastole is the part of the cardiac cycle

when the heart refills with blood

following systole (contraction)

Ventricular diastole is the period during

which the ventricles are filling and

relaxing while atrial diastole is the

period during which the atria are

relaxing

Q128 Azadirachta indica is the

botanical name of which of the

following

अजाटदराचताइडिकानननननलिखितमसहॳककसकावानसपनतनामहहॴ

23-Jan-2017

Options

1) Rose plant

गिाबकापह९धा 2) Apple tree

सहॳबकापहॳड़

3) Neem

नीम

4)Mango

आम

Correct Answer Neem

Q129 Which of the following is the

main end product of carbohydrate

digestion

नननननलिखितमसहॳकह९नसाकाबोहाइडरहॳटकहॳ पाचनकापरमिअतउतपादकहह८ताहहॴ 23-Jan-2017

Options

1) Fats

वसा 2) Lipids

लिपपडस

3) Glucose

गिकह८ज

4) Cellulose

F A C E B O O K

P A G E h t t p w w w f a c e b o o k c o m s s c m e n t o r s o f f i c i a l P a g e | 31

FOR MORE UPDATES AND MORE MATERIAL DO LIKE OUR FACEBOOK PAGE httpwwwfacebookcomsscmentorsofficial

सहॳमयिह८ज

Correct Answer Glucose

Intestinal absorption of end products

from digestion of carbohydrates and

proteins in the pig hellip During absorption some sugars (fructose or

galactose) released from the

corresponding sucrose and lactose

respectively during digestion were

partly metabolized into glucose by the

enterocyte

Q130 Which of the following glands is a

source of the enzyme Ptyalin

नननननलिखितगरचियोमसहॳएजाइमटयालिनकासरह८तहहॴ 23-Jan-2017

Options

1) Pancreas

अगरािय

2) Thyroid Gland

िाइराइिगरिी 3) Pituitary Gland

पीयषगरिी 4) Salivary Glands

िारगरचियाा Correct Answer Salivary Glands

Q131 Which of the following is not true

about Pteridophyta

ननननमसहॳकह९नसीबातटहॳररिह८फाईटकहॳ बारहॳमसचनहीहहॴ 23-Jan-2017

Options

1) Dominant phase is saprophytes

परमिचरणसहॳपरह८फाईइटसहह८ताहहॴ 2) Main plant body is diploid

पह९दह८कामखयिरीरदपवगखणतहह८ताहहॴ 3) Seeds are present

बीजमह९जदहह८तहॳहहॴ 4)Flowers are absent

फिअनपनसतिहह८तहॳहहॴ

Correct Answer Seeds are present

Q132 The largest dolphin species is the

orca also called as

िॉिकफनकीसबसहॳबड़ीपरजानतकाकानामआकायहहॴनजसहॳ mdash- भीकहतहॳहहॴ 23-Jan-2017

Options

1) Bottle Nose

बाटिनह८ज

2) Baiji

बहॳजी 3) Killer whale

ककिरहहॳि

4)Tucuxi

टकवसी Correct Answer Killer whale

Q133 The fat digesting enzyme Lipase

is secreted by which of the following

वसाकापाचनकरनहॳवािाएजाइमिाइपहॳजनननननलिखितमसहॳककसकहॳ दवारासतरापवतहह८ताहहॴ

24-Jan-2017

Options

1) Kidneys

गद

2) Pancreas

अगनयािय

3) Large Intestine

बड़ीआत

4)Liver

नजगर

Correct Answer Pancreas

Lipase is an enzyme that splits fats so

the intestines can absorb them Lipase

hydrolyzes fats like triglycerides into

their component fatty acid and glycerol

molecules It is found in the blood

gastric juices pancreatic secretions

intestinal juices and adipose tissues

F A C E B O O K

P A G E h t t p w w w f a c e b o o k c o m s s c m e n t o r s o f f i c i a l P a g e | 32

FOR MORE UPDATES AND MORE MATERIAL DO LIKE OUR FACEBOOK PAGE httpwwwfacebookcomsscmentorsofficial

Q134 The arrangement of leaves on an

axis or stem is called

एकअकषयातनहॳपरपनततयोकीयवसिाकह८कयाकहाजाताहहॴ SSC CHSL Science (biology) 2016

Question Paper

24-Jan-2017

Options

1) Phyllotaxy

फाइिह८टहॴकसी 2) Vernation

वनिन

3) Venation

वहॳनहॳिन

4)Phytotaxy

फाइटह८टहॴकसी Correct Answer Phyllotaxy

In botany phyllotaxis or phyllotaxy is

the arrangement of leaves on a plant

stem (from Ancient Greek phyacutellon

ldquoleafrdquo and taacutexis ldquoarrangementrdquo)

Phyllotactic spirals form a distinctive

class of patterns in nature

Q135 The study of Cells is also known

as

कह८लिकाओकहॳ अधययनकह८ mdashmdashndash

भीकहाजाताहहॴ 24-Jan-2017

Options

1) Cytology

सायटह८िह८जी 2) Physiology

कफनजयह८िह८जी 3) Nucleology

नयककमयह८िह८जी 4)Cellology

सहॳिह८िह८जी Correct Answer Cytology

Q136 Which of the following scientists

is also known as the Father of Biology

नननननलिखितमसहॳककसवहॴजञाननककह८ ldquoजीवपवजञानकहॳ जनकrdquoकहॳ नामसहॳभीजानाजाताहहॴ 24-Jan-2017

Options

1) Herbert Spencer

हबयटयसपसर

2) Aristotle

अरसत 3) Lamarck

िहॳमाकय 4)Darwin

िापवयन

Correct Answer Aristotle

Q137 Which cells give rise to various

organs of the plant and keep the plant

growing

कह९नसीकह८लिकाएपह९धह८कहॳ लभननअगह८कह८जनमदहॳतीहहॴऔरपह९धह८कह८बढ़नहॳममददकरतीहहॴ

24-Jan-2017

Options

1) Permanent

सिायी 2) Dermal

तवचीय

3) Meristematic

मररसटहॳमटटक

4)Mature

परह८ढ़

Correct Answer Meristematic

A meristem is the tissue in most plants

containing undifferentiated cells

(meristematic cells) found in zones of

the plant where growth can take place

Q138 Rodentia Muridae is the scientific

name of

F A C E B O O K

P A G E h t t p w w w f a c e b o o k c o m s s c m e n t o r s o f f i c i a l P a g e | 33

FOR MORE UPDATES AND MORE MATERIAL DO LIKE OUR FACEBOOK PAGE httpwwwfacebookcomsscmentorsofficial

रह८िहॳलियानयररिी mdashmdash- कावहॴजञाननकनामहहॴ 24-

Jan-2017

Options

1) Mouse

चहा 2) Squirrel

चगिहरी 3) Monkey

बदर

4) Lizard

नछपकिी Correct Answer Mouse

Q139 Name the scientist who proposed

the cell theory

कह८लिकालसदातकापरसतावदहॳनहॳवािहॳवहॴजञाननककानामबताइए 24-Jan-2017

Options

1) Schleiden and Schwann

िीमिनऔरशरववान

2) Lamarck

िहॳमाकय 3) Treviranus

टरहॳवायरहॳनस

4)Whittaker and Stanley

हीटकरऔरसटहॳनिहॳ Correct Answer Schleiden and

Schwann

Q140 The flower with the worldrsquos

largest bloom is

दननयाकासबसहॳबड़ाफिखििनहॳवािा mdashmdashndash हहॴ 24-Jan-2017

Options

1) Pando

पािह८ 2) Posidonia

पह८सीिह८ननया 3) Rafflesia arnoldii

ररफिहॳलियाअनोमिी 4)Helianthus annuus

हहॳलिएनिसएनयअस

Correct Answer Rafflesia arnoldii

Rafflesia arnoldii is a species of

flowering plant in the parasitic genus

Rafflesia It is noted for producing the

largest individual flower on earth It has

a very strong and horrible odour of

decaying flesh earning it the nickname

ldquocorpse flower

Q141 Deficiency of which vitamin

causes night blindness

ककसपवटालमनकीकमीकहॳ कारणरतौधीहह८ताहहॴ 24-Jan-2017

Options

1) Vitamin K

पवटालमन K

2) Vitamin C

पवटालमन C

3) Vitamin B1

पवटालमन B1

4)Vitamin A

पवटालमन A

Correct Answer Vitamin A

Q142 Nongreen plants lack which of the

following

गहॴर-

हररतवनसपनतमनननननलिखितमसहॳककसकीकमीहह८तीहहॴ

24-Jan-2017

Options

1) Chlorophyll

किह८रह८कफि

2) Lycophyll

िायकह८कफि

3) Cyanophyll

F A C E B O O K

P A G E h t t p w w w f a c e b o o k c o m s s c m e n t o r s o f f i c i a l P a g e | 34

FOR MORE UPDATES AND MORE MATERIAL DO LIKE OUR FACEBOOK PAGE httpwwwfacebookcomsscmentorsofficial

सायनह८कफि

4)Phototropism

फह८टह८टरोपपजम

Correct Answer Chlorophyll

Q143 Organisms that use light to

prepare food are known as

जह८जीवपरकािकाउपयह८गकरभह८जनतहॴयारकरतहॳहहॴ उनह mdashmdash- कहॳ पमजानजाताहहॴ 24-Jan-2017

Options

1) Autotrophs

सवपह८षी 2) Heterotrophs

पवषमपह८षज

3) Omnivores

सवायहारी 4)Decomposers

पवघटनकरनहॳवािा Correct Answer Autotrophs

autotrophs often make their own food

by using sunlight carbon dioxide and

water to form sugars which they can use

for energy Some examples of

autotrophs include plants algae and

even some bacteria Autotrophs

(producer) are important because they

are a food source for heterotrophs

(consumers)

A heterotroph is an organism that

ingests or absorbs organic carbon

(rather than fix carbon from inorganic

sources such as carbon dioxide) in order

to be able to produce energy and

synthesize compounds to maintain its

life Ninety-five percent or more of all

types of living organisms are

heterotrophic including all animals and

fungi and some bacteria

Q144 Which of the following is a

primary function of haemoglobin

नननननलिखितमसहॳकह९नसाटहमह८गिह८बबनकाएकपरािलमककाययहहॴ

25-Jan-2017

Options

1) Utilization of energy

उजायकाउपयह८गकरना 2) Prevention of anaemia

रकतामपताहह८नहॳसहॳरह८कना 3) Destruction of bacteria

बहॴकटीररयाकापवनािकरना 4) To transport oxygen

ऑकसीजनकावहनकरना Correct Answer To transport oxygen

Q145 Vascular bundles are absent in

सवहनीबिि mdashmdash- मअनपनसतिरहतहॳहहॴ 25-Jan-2017

Options

1) Bryophyta

िायह८फाइटा 2) Pteridophyta

टहॳररिह८फाईटा 3) Gymnosperms

नजननह८सपमय 4) Angiosperms

एननजयह८सपहॳनसय Correct Answer Bryophyta

Q146 Sauria Lacertidae is the scientific

name of

सहॴररयािहॳसरटाईिी mdashmdashndash कावहॴजञाननकनामहहॴ 25-Jan-2017

Options

1) Crocodile

मगरमचछ

2) Hippopotamus

टहपपह८पह८टहॳमस

3) Lizard

नछपकिी 4) House fly

F A C E B O O K

P A G E h t t p w w w f a c e b o o k c o m s s c m e n t o r s o f f i c i a l P a g e | 35

FOR MORE UPDATES AND MORE MATERIAL DO LIKE OUR FACEBOOK PAGE httpwwwfacebookcomsscmentorsofficial

घरहॳिमकिी Correct Answer Lizard

Q147 Which type of pathogen causes

the water-borne disease SARS (Severe

Acute Respiratory Syndrome)

ककसपरकािकारह८गज़नकजिजननतबीमारीसासयकाकारणबनताहहॴ 25-Jan-2017

Options

1) Viral

वायरि

2) Parasitic

परजीवी 3) Protozoan

परह८टह८जअन

4) Bacterial

बहॴकटीररयि

Correct Answer Viral

Q148 Which of the following organs

produces the enzyme lipase

नननननलिखितमसहॳकह९नसाअगिायपहॳजएजाइमउतपननकरताहहॴ 25-Jan-2017

Options

1) Pancreas

अगनयािय

2) Large Intestine

बड़ीआत

3) Liver

नजगर

4) Small Intestine

छह८टीआत

Correct Answer Pancreas

Q149 A is a long internode forming the

basal part or the whole of a peduncle

एक mdashmdash- एकिबाइटरनह८िहहॴ जह८ननचिाटहससायासनपणयिठिबनताहहॴ 25-

Jan-2017

Options

1) Rhizome

परकद

2) Rachis

महॳ दि

3) floral axis

पषपअकष

4) Scape

भगदड़

Correct Answer scape

Q150 ndash Which of the following

organisms are considered to be both

Living and Non-living

नननननलिखितमसहॳकह९नसहॳजीवाणकह८जीपवतऔरअजीपवतमानाजाताहहॴ

25-Jan-2017

Options

1) Bacteria

बहॴकटीररया 2) Fungi

कवक

3) Algae

िहॴवाि

4)Virus

वायरस

Correct Answer Virus

They are considered to be living as they

possess a protein coat as a protective

covering DNA as the genetic material

etc

They are said to be non-living as they

can be crystallised and they survive for

billions of years They can tolerate high

temperatures freezing cold

temperatures ultra-violet radiations etc

Q151 Deficiency of fluorine causes

which of the following

फिह८ररनकीकमीकहॳ कारणनननननलिखितमसहॳकयाहह८ताहहॴ

F A C E B O O K

P A G E h t t p w w w f a c e b o o k c o m s s c m e n t o r s o f f i c i a l P a g e | 36

FOR MORE UPDATES AND MORE MATERIAL DO LIKE OUR FACEBOOK PAGE httpwwwfacebookcomsscmentorsofficial

27-Jan-2017

Options

1) Dental Caries

िटिकहॴ ररज

2) Scurvy

सकवरी 3) Anaemia

रकतामपता 4) Arthritis

गटठया Correct Answer Dental Caries

Q152 In a Punnett Square with the

cross AaBb x AaBb how many Aabb

genotypes would be created

पनहॳटसककायरमिह८स AaBb x AaBb कहॳ साि

ककतनहॳ Aabb जीनह८टाइपबनगहॳ 27-Jan-2017

Options

1) 1

2) 8

3) 2

4) 3

Correct Answer 2

Q153 Which of the following is the

Controlling Center of the Cell

नननननलिखित म सहॳ कह८लिकाका ननयतरण

क दर कह९न हहॴ

27-Jan-2017

Options

1) Nucleus

क दर

2) Plasma

पिाजमा 3) Lysosome

िायसह८सह८म

4) Chromosome

िह८मह८सह८म

Correct Answer Nucleus

The control centre of the cell is the

nucleus in eukaryotic cells The nucleus

contains genetic material in the form of

DNA

Q154 Myopia affects which of the

following organs

मायह८पपयानननननलिखितअगह८मसहॳककसहॳपरभापवतकरताहहॴ

25-Jan-2017

Options

1) Heart

हदय

2) Skin

तवचा 3) Eyes

आािहॳ 4)Mouth

मह

Correct Answer Eyes

Q155 Which of the following bears

flowers

नननननलिखितमसहॳकह९नफिधारणकरताहहॴ

25-Jan-2017

Options

1) Bryophyta

िायह८फाइटा 2) Pteridophyta

टहॳरीिह८फाईटा 3) Gymnosperms

नजननह८सपमय 4)Angiosperms

एननजयह८सपमय Correct Answer Angiosperms

Q156 Oxygenated blood flows out of the

heart through the

ऑकसीजनयकतरकत mdashmdashmdash

कहॳ माधयमसहॳहदयकहॳ बाहरबहताहहॴ 25-Jan-2017

F A C E B O O K

P A G E h t t p w w w f a c e b o o k c o m s s c m e n t o r s o f f i c i a l P a g e | 37

FOR MORE UPDATES AND MORE MATERIAL DO LIKE OUR FACEBOOK PAGE httpwwwfacebookcomsscmentorsofficial

Options

1) Aorta

महाधमनी 2) pulmonary artery

फहॳ फड़हॳकीधमनी 3) vena cava

वहॳनाकावा 4)Atrium

चह९क

Correct Answer aorta

Q157 Blood leaving the liver and

moving towards the

heart has a higher concentration of

नजगरसहॳननकिकरहदयकीतरफजानहॳवािहॳरकतम mdashmdashmdashmdash कीउचचसादरताहह८तीहहॴ 27-Jan-2017

Options

1) Lipids

लिपपडस

2) Urea

यररया 3) Bile Pigments

पपततकहॳ रगकरण

4) Carbon dioxide

काबयनिायऑकसाइि

Correct Answer Bile Pigments

Urea is nitrogen containing substance

which is produced in the liver in order

to deal with excess amino-acids in the

body As urea is produced it leaves the

liver in the blood stream and passes via

the circulatory system to all parts of the

body

Q158 Bulb is a modification of which

part of a plant

बमबएकपह९धहॳकहॳ ककसटहससहॳकाएक पातरणहह८ताहहॴ 27-Jan-2017

Options

1) The root

जड़

2) The stem

तना 3) The radicle

मिाकर

4)The fruit

फि

Correct Answer The stem

Q159 Which of the following carries

blood away from the heart to different

body parts

इनमहॳसहॳकह९नरकतकह८हदयसहॳिरीरकहॳ पवलभननअगह८तकिहॳजातीहहॴ

27-Jan-2017

Options

1) Arteries

धमननया 2) Nerves

तबतरहाए

3) Capillaries

कहॳ लिकाए

4)Veins

नसहॳ Correct Answer Arteries

Q160 The series of processes by which

nitrogen and its compounds are

interconverted in the environment and

in living organisms is called

27-Jan-2017

Options

1)Absorption of Nitrogen

2)Ammonification

3)Nitrogen Fixation

4)Nitrogen Cycle

Correct Answer Nitrogen Cycle

Ammonification or Mineralization is

performed by bacteria to convert

organic nitrogen to ammonia

F A C E B O O K

P A G E h t t p w w w f a c e b o o k c o m s s c m e n t o r s o f f i c i a l P a g e | 38

FOR MORE UPDATES AND MORE MATERIAL DO LIKE OUR FACEBOOK PAGE httpwwwfacebookcomsscmentorsofficial

Nitrification can then occur to convert

the ammonium to nitrite and nitrate

Nitrogen fixation is a process by which

nitrogen in the Earthrsquos atmosphere is

converted into ammonia (NH3) or other

molecules available to living organisms

Q161 BCG vaccine is given to protect

from which of the following

बीसीजीकाटटकानननननलिखितमसहॳककसकहॳ बचावकहॳ लिएटदयाजातहहॴ

27-Jan-2017

Options

1) Jaundice

पीलिया 2) Anaemia

रकतमपता 3) Tuberculosis

कषयरह८ग

4) Polio

पह८लियह८ Correct Answer Tuberculosis

Q162 Parallel venation is found in

समानतरवहॳनहॳिन mdashmdashmdash- मपायाजाताहहॴ 27-Jan-2017

Options

1) plants which are monocots

पह९धहॳजह८एकबीजपतरीहह८तहॳहहॴ 2) plants which have a dicot stem

वहॳपह९धहॳनजनकातनादपवदलियहह८ताहहॴ 3) plants with leaves similar to Tulsi

वहॳपह९धहॳनजनकीपनततयतिसीकीपनततयोकहॳ समानहह८तहॳहहॴ 4)plants with tap roots

टहॳप टवािहॳपह९धहॳ Correct Answer plants which are

monocots

Q163 The hardest part of the body is

िरीरकासबसहॳकठह८रभाग mdashndash हहॴ 27-Jan-2017

Options

1) Bones

हडडिय

2) Tooth Enamel

दातकहॳ इनहॳमि

3) Skull

िह८पड़ी 4) Spinal Cord

महॳ रजज

Correct Answer Tooth Enamel

Q164 Which type of pathogen causes

the waterborne disease E coli Infection

ककसपरकारकारह८गजननकजिजननतरह८गईकह८िाईसिमणकाकारणबनताहहॴ 27-Jan-2017

Options

1) Protozoan

परह८टह८जआ

2) Parasitic

परजीवी 3) Bacterial

बहॴकटीररयि

4)Viral

वायरि

Correct Answer Bacterial

Q165 The amount of blood filtered

together by both the kidneys in a 70 kg

adult male human in a minute is

70 की गरा वािहॳएकवयसकप षमएकलमनटमदह८नोगदकहॳदवाराएकसािचाबनीगयीरकतकीमातरहह८तीहहॴ 29-Jan-2017

Options

1) 1100 ml

1100 लमलि

2) 100 ml

F A C E B O O K

P A G E h t t p w w w f a c e b o o k c o m s s c m e n t o r s o f f i c i a l P a g e | 39

FOR MORE UPDATES AND MORE MATERIAL DO LIKE OUR FACEBOOK PAGE httpwwwfacebookcomsscmentorsofficial

100 लमलि

3) 1500 ml

1500 लमलि

4) 500 ml

500 लमलि

Correct Answer 1100 ml

Q166 Which feature of a plant helps to

distinguish a monocot from a dicot

पह९धहॳकीवहकह९नसीपविहॳषताहहॴजह८एकदपवदलियहॳऔरएकएकदिीयपह९धहॳसहॳभहॳदकरनहॳममददकरतीहहॴ 29-Jan-2017

Options

1) Pollination

परागम

2) Venation

वहॳनहॳिन

3) Vernation

वनिन

4) Aestivation

एसटीवहॳिहॳन

Correct Answer venation

Q167 The Mutation Theory was

proposed by

उतवररवतयनकालसदात mdashmdashndash

कहॳ दवरापरसतापवतककयाजाताहहॴ 29-Jan-2017

Options

1) Charles Lyell

चामसयलियहॳि

2) William Smith

पवलियमनसमि

3) Hugo De Vries

हयगह८िीराईस

4)Harrison Schmitt

हहॳरीसननसमट

Correct Answer Hugo De Vries

Q168 Which type of pathogen causes

the waterborne disease HepatitisA

ककसपरकारकहॳ रह८गजनकजिजननतरह८गहहॳपहॳटाइटटस-A काकारणबनताहहॴ

29-Jan-2017

Options

1) Parasitic

परजीवी 2) Viral

वायरि

3) Protozoan

परह८टह८जआ

4) Bacterial

बहॴकटीररयि

Correct Answer Viral

Q169 In a Punnett Square with the

cross AaBb x Aabb how many AaBb

genotypes would be created

पनहॳटसकवायरमिह८स AaBb x Aabb

कहॳ सािककतनहॳ AaBb जीनह८टाइपबनगहॳ 29-Jan-

2017

Options

1) 4

2) 1

3) 7

4) 6

Correct Answer 4

Q170 Arboreal Ateles is the scientific

name of

अिह८ररयिएटटलिस mdashmdashmdash कावहॴजञाननकनामहहॴ 29-Jan-2017

Options

1) Squirrel

चगिहरी 2) Sparrow

गह८रहॴया 3) Lizard

नछपकिी 4) Spider monkey

F A C E B O O K

P A G E h t t p w w w f a c e b o o k c o m s s c m e n t o r s o f f i c i a l P a g e | 40

FOR MORE UPDATES AND MORE MATERIAL DO LIKE OUR FACEBOOK PAGE httpwwwfacebookcomsscmentorsofficial

मकड़ीबदर

Correct Answer Spider monkey

Q171 Which type of pathogen causes

the waterborne disease Salmonellosis

ककसपरकारकारह८गाणजिजननतबीमारीसािमह८नहॳिह८लसज़काकारकहहॴ

29-Jan-2017

Options

1) Algal

िहॳवालियहॳ 2) Parasitic

परजीवी 3) Bacterial

बहॴकटीररयि

4)Viral

वायरि

Correct Answer Bacterial

An infection with salmonella bacteria

commonly caused by contaminated food

or water

Symptoms include diarrhoea fever

chills and abdominal pain

Q172 is a condition in which there is a

deficiency of red cells or of haemoglobin

in the blood

mdashmdash-

एकनसिनतहहॴनजसमहॳरकतमिािकह८लिकाओकीयाहीमह८गिह८बबनकीकमीहह८तीहहॴ 29-Jan-2017

Options

1) Albinism

एनमबननजम

2) Propyria

परह८पीररया 3) Anaemia

एनीलमया 4)Keloid disorder

कहॳ िह८इिडिसओिर

Correct Answer Anaemia

Q173 Ananas comosus is the scientific

name of

Options

अनानासकह८मह८सस mdashmdashmdashndash

कावहॴजञाननकनामहहॴ 29-Jan-2017

1) Custard Apple

सीताफि

2) Pineapple

पाइनएपपि

3) Bamboo

बास

4)Pomegranate

अनार

Correct Answer Pineapple

Q174 Which organ produces insulin

कह९नसाअगइनसलिनपहॴदाकरताहहॴ 29-Jan-

2017

Options

1) Liver

यकत

2) Thyroid gland

िायराइिगरिी 3) Spleen

पिीहा 4)Pancreas

अगरयिय

Correct Answer Pancreas

Q175 Which of the following disease is

not caused by water pollution

नननननलिखितमसहॳकह९नसारह८गपानीकहॳ परदषणकहॳकारणनहीहह८ता

29-Jan-2017

Options

1) Cholera

हहॴजा 2) Typhoid

F A C E B O O K

P A G E h t t p w w w f a c e b o o k c o m s s c m e n t o r s o f f i c i a l P a g e | 41

FOR MORE UPDATES AND MORE MATERIAL DO LIKE OUR FACEBOOK PAGE httpwwwfacebookcomsscmentorsofficial

टाइफाइि

3) Asthma

दमा 4)Diarrhoea

दसत

Correct Answer Asthma

Q176 Ocimum tenuiflorum is the

scientific name of

ओलिलममटहॳयईफिह८रमइसकावहॴजञाननकनाम mdash

ndash हहॴ 30-Jan-2017

Options

1) Neem

नीम

2) Mango

आम

3) Babul

बबि

4)Tulsi

तिसी Correct Answer Tulsi

Q177 Which gland secretes bile a

digestive fluid

कह९नसीगरिीपपतत एकपाचनतरिपरदािय सरापवतकरतीहहॴ 30-Jan-2017

Options

1) Pancreas

अगनयािय

2) Liver

यकत

3) Thyroid

िायराइि

4) Testes

टहॳनसटस

Correct Answer liver

Q178 In which of the following the

dominant phase is Gametophyte

नननननलिखितमसहॳककसकहॳ परमिचरणयगमकह८दपवधद (Gametophyte)हहॴ 30-Jan-2017

Options

1) Bryophyta

िायह८फाइटा 2) Pteridophyta

टहॳररिह८फाइटा 3) Gymnosperms

नजननह८सपमय 4) Angiosperms

एननजयह८सपमय Correct Answer Bryophyta

Q179 Anaerobic respiration refers to

which of the following

नननननलिखितमसहॳककसहॳअवायवीयशवसनकहाजाताहहॴ

30-Jan-2017

Options

1) Respiration without Oxygen

ऑकसीजनकहॳ बबनाशवसन

2) Respiration with Oxygen

ऑकसीजनकहॳ सािशवसन

3) Respiration without CO2

काबयनिायऑकसाइिकहॳ बबनाशवसन

4) Respiration with CO2

काबयनिायऑकसाइिकहॳ सािशविन

Correct Answer Respiration without

Oxygen

Q180 Which type of pathogen causes

the waterborne disease Cholera

ककसपरकारकारह८गजनकजिजननतरह८गहहॴजाकाकारणबनताहहॴ

30-Jan-2017

Options

1) Algal

िहॴवालियहॳ

F A C E B O O K

P A G E h t t p w w w f a c e b o o k c o m s s c m e n t o r s o f f i c i a l P a g e | 42

FOR MORE UPDATES AND MORE MATERIAL DO LIKE OUR FACEBOOK PAGE httpwwwfacebookcomsscmentorsofficial

2) Bacterial

बहॴकटीररयि

3) Protozoan

परह८टह८जआ

4) Viral

वायरि

Correct Answer Bacterial

Q181 To which class does

Oxyreductases transferases hydrolases

belong

ओकसीररिकटहॳसटरासफरहॳजहॳस

हाइडरह८िहॳसहॳसककसवगयमआतहॳहहॴ 30-Jan-2017

Options

1) Hormones

हारमोस

2) Enzymes

एजाइनस

3) Proteins

परह८टीनस

4) Vitamins

पवटालमनस

Correct Answer Enzymes

Q182 Which of the following is not true

about Gymnosperms

ननननमसहॳकह९नसीबातअनावतबीजीकहॳ बारहॳमसचनहीहहॴ 30-Jan-2017

Options

1) Dominant phase is saprophytes

परमिचरणसहॳपरह८फाइटसहह८ताहहॴ 2) Vascular bundles are absent

सवहनीबििअनपनसितहह८ताहहॴ 3) spores are heterospores

बीजाणहहॳटहॳरह८सपह८रसहह८तहॳहहॴ 4) Flowers are absent

फिअनपनसितहह८तहॳहहॴ

Correct Answer Vascular bundles are

absent

Q183 The name of first mammal clone sheep is

भहॳड़कीपरिमसतनपायीपरनत प (किह८न)

कानामहहॴ 30-Jan-2017

Options

1) Noori

नरी 2) Dolly

िॉिी 3) Louise

िसी 4)Durga

दगाय Correct Answer Dolly

Q184 Which type of pathogen causes

the water-borne disease Typhoid fever

ककसपरकारकारह८गजनकजिजननतरह८गटाइफाइिबिारकाकारणबनताहहॴ 30-Jan-2017

Options

1) Algal

िहॴवािीय

2) Parasitic

परजीवी 3) Protozoan

परह८टह८जनअन

4)Bacterial

बहॴकटीररयि

Correct Answer Bacterial

Q185 In which part of the cell are

proteins made

कह८लिकाकहॳ ककसटहससहॳमपरह८टीनबनायाजाताहहॴ

31-Jan-2017

Options

1) Reticulum

रहॳटटकिम

F A C E B O O K

P A G E h t t p w w w f a c e b o o k c o m s s c m e n t o r s o f f i c i a l P a g e | 43

FOR MORE UPDATES AND MORE MATERIAL DO LIKE OUR FACEBOOK PAGE httpwwwfacebookcomsscmentorsofficial

2) Golgi apparatus

गह८मजीएपहॳरहॳटस

3) Ribosomes

ररबह८सह८नस

4) Lysosome

िायसह८सह८नस

Correct Answer ribosomes

Proteins are produced by stringing

amino acids together in the order

specified by messenger RNA strands

that were transcribed from DNA in the

cell nucleus The process of synthesizing

a protein is called translation and it

occurs on ribosomes in the cytoplasm of

a cell

Q186 Polio is a disease caused by which

of the following

नननननलिखितमसहॳपह८लियह८कीबबमारह८हह८नहॳकाकारणकयाहहॴ

31-Jan-2017

Options

1) Bacteria

बहॴकटीररयि

2) Mosquito

मचछर

3) Virus

वायरस

4) Cockroach

नतिच हॳ Correct Answer Virus

Polio or poliomyelitis is a crippling and

potentially deadly infectious disease It

is caused by the poliovirus

Q187 ndash Hay fever is a sign of which of

the following

हहॳकफवरनननननलिखितमसहॳककसकाएकसकहॳ तहहॴ

31-Jan-2017

Options

1) Old Age

वदावसिा 2) Malnutrition

कपह८सण

3) Allergy

एिनजय 4) Over Work

अतयचधककाययकरना Correct Answer Allergy

Q188 How many chromosomes does a

human cell contain

एकमानवकह८लिकामककतनहॳगणसतरहह८तहॳहहॴ

29-Jan-2017

Options

1) 6

2) 26

3) 46

4) 66

Correct Answer 46

In humans each cell normally contains

23 pairs of chromosomes for a total of

46 Twenty-two of these pairs called

autosomes look the same in both males

and females The 23rd pair the sex

chromosomes differ between males and

females

Q189 Which of the following is not true

about Bryophyta

ननननमसहॳकह९नसीबातिायह८फाइटकहॳ बारहॳमसचनहीहहॴ 31-Jan-2017

Options

1) Dominant phase is gametophytes

परमिचरणगहॳलमतह८फाइटसहह८ताहहॴ 2) Main plant body is haploid

पह९धहॳकामखयिरीरअगखणतहह८ताहहॴ 3) Spores are homospores

बीजाणहह८मह८सफह८रसहह८तहॳहहॴ 4) Flowers are present

फिमह८जदहह८तहॳहहॴ Correct Answer Flowers are present

F A C E B O O K

P A G E h t t p w w w f a c e b o o k c o m s s c m e n t o r s o f f i c i a l P a g e | 44

FOR MORE UPDATES AND MORE MATERIAL DO LIKE OUR FACEBOOK PAGE httpwwwfacebookcomsscmentorsofficial

Q190 Which aquatic animal has

trailing tentacles

ककसजिीयजानवरकहॳ पीछहॳचिनहॳवािहॳटहॳटकिसहह८तहॳहहॴ

31-Jan-2017

Options

1) Sea horse

समदरीघह८िा 2) Corals

मगा 3) Jelly fish

जहॳिीमछिी 4) Star fish

तारामछिी Correct Answer Jelly fish

Jellyfish with its umbrella-shaped bell

and trailing tentacles

Q191 Which type of pathogen causes

the water-borne disease Poliomyelitis

(Polio)

ककसपरकारकारह८गजनकजिजननतरह८गपह८लियह८मायहॳटटस (पह८लियह८) काकारणहहॴ 31-Jan-

2017

Options

1) Parasitic

परजीवी 2) Algal

िहॴवालिय

3) Viral

वायरि

4) Bacterial

बहॴकटीररयि

Correct Answer Viral

Q192 The outer white part of the eye

that protects the inner structures is

आािकाबाहरीसफहॳ दटहससाजह८आतररकसरचनाओकीरकषाकरताहहॴ वह mdashmdashmdash हहॴ 31-Jan-

2017

Options

1) Iris

आयररस

2) Sclera

सकिहॳरा 3) Retina

रहॳटटना 4) Cornea

कह८ननयया Correct Answer Sclera

Q193 Proteins are made up of

परह८टीनकाननमायण mdashndash सहॳहह८ताहहॴ 31-Jan-2017

Options

1) Amino acids

एलमनह८अनि

2) Fatty acids

वसायकतअनि

3) Glucose

गिकह८ज

4)Nucleotides

नयनकियह८टाईिस

Correct Answer Amino acids

Q194 Moringa Oleifera is the scientific

name of

मह८ररगओलिफहॳ रा mdashmdashndash कावहॴजञाननकनामहहॴ 31-Jan-2017

Options

1) Banyan

बरगद

2) Gulmohar

गिमह८हर

3) Amla

आमिा

F A C E B O O K

P A G E h t t p w w w f a c e b o o k c o m s s c m e n t o r s o f f i c i a l P a g e | 45

FOR MORE UPDATES AND MORE MATERIAL DO LIKE OUR FACEBOOK PAGE httpwwwfacebookcomsscmentorsofficial

4) Drumstick

डरमनसटक

Correct Answer Drumstick

Q195 Kidney stones are composed of

गदकीपिरी mdashndash सहॳबनीहह८तीहहॴ 1-Feb-2017

Options

1) Calcium Oxalate

कहॴ नमसयमओकजहॳिहॳट

2) Sodium Chloride

सह८डियमकिह८राइि

3) Magnesium Nitrate

महॳनगनलियमनाइतटरहॳट

4) Calcium Bicarbonate

कहॴ नमियमबायकबोनहॳट

Correct Answer Calcium Oxalate

Q196 ndash Which of the following is not

true about Angiosperms

ननननमसहॳकह९नसीबातआवतबीजीकहॳ बारहॳमसचनहीहहॴ 1-Feb-2017

Options

1) Dominant phase is gametophytes

परमिचरणगहॳलमतह८फाइटहह८ताहहॴ 2) Vascular bundles are present

सवहनीबििमह९जदहह८ताहहॴ 3) Spores are heterospores

बीजाणहहॳटहॳरह८सपह८रसहह८तहॳहहॴ 4) Seeds are covered

बीजढकहॳ हह८तहॳहहॴ Correct Answer Dominant phase is

gametophytes

Q197 All of the following are excretory

(waste) products of animals except

नननननलिखितमसहॳककसएककह८छह८ड़करअनयसभीपराखणयोदवाराउतसनजयतपदाियहहॴ 1-Feb-

2017

Options

1) Uric Acid

यररकएलसि

2) Ammonia

अमह८ननया 3) Carbohydrates

काबोहाइडरहॳट

4) Urea

यररया Correct Answer Carbohydrates

In animals the main excretory products

are carbon dioxide ammonia (in

ammoniotelics) urea (in ureotelics) uric

acid (in uricotelics) guanine (in

Arachnida) and creatine

Q198 RNA is a polymeric molecule

What does RNA stand for

आरएनइएएकबहिकआणहहॴ इसकाकापवय पकयाहहॴ 1-Feb-2017

Options

1) Rado Nuclear Acid

रािह८नयनकियरएलसि

2) Ribo Nucleic Acid

राइबह८नयनकिकएलसि

3) Rhino Nuclear Acid

हाइनह८नयनकियरएलसि

4) Resto Nucleus Acid

रहॳसटह८नयकिीयसएलसि

Correct Answer Ribo Nucleic Acid

Q199 Which organ does detoxification

and produces chemicals needed for

digestion

कह९नसाअगपवषहरणकरताहहॴऔरपाचनकहॳ लिएआवशयकरसायनोकह८पहॴदाकरताहहॴ 1-Feb-

2017

Options

1) Salivary glands

िारगरचिया 2) Pancreas

अगनयािय

F A C E B O O K

P A G E h t t p w w w f a c e b o o k c o m s s c m e n t o r s o f f i c i a l P a g e | 46

FOR MORE UPDATES AND MORE MATERIAL DO LIKE OUR FACEBOOK PAGE httpwwwfacebookcomsscmentorsofficial

3) Thyroid gland

िायराइिगरिी 4) Liver

यकत

Correct Answer Liver

Q200 Psidium guajava is the scientific

name of

लसडियमगआजावा mdashmdash कावहॴजञाननकनामहहॴ 1-

Feb-2017

Options

1) Guava

अम द

2) Mango

आम

3) Bamboo

बास

4) Jack fruit

कटहि

Correct Answer Guava

Q201 Which drug is used as a Blood

Thinner

चधरकह८पतिाकरनहॳकहॳ पमककसदवाकापरयह८गककयाजाताहहॴ

1-Feb-2017

Options

1) Warfarin

वाफर न

2) Tramadol

टरहॳमािह८ि

3) Azithromycin

एनजरह८मायलसन

4) Hydralazine

हाइडरह८िहॳनजन

Correct Answer Warfarin

Q202 Which of the following disease is

caused due to the deficiency of protein

परह८टीनकीकमीकहॳ कारणनननननलिखितमसहॳकह९नसारह८गहह८ताहहॴ 1-Feb-2017

Options

1) Arthritis

गटठया 2) Kwashiorkor

कािीओकय र

3) Goitre

गाइटर

4) Night Blindness

रतह९चध

Correct Answer Kwashiorkor

Q203 A is species of plant that has

adapted to survive in an environment

with little liquid water

mdashmdashndashपह९धहॳकीएकऐसहॳऐसहॳपरजानतहहॴ नजसनहॳकमपानीवािहॳवातावरणमजीपवतरहनहॳकहॳलिएअनकिनहहॴ 1-Feb-2017

Options

1) Xerophyte

म दपवद

2) Hydrophyte

जिीयपादप

3) Mesophyte

समह८दपवद

4) Thallophyte

िहॴिह८फाइटा Correct Answer xerophyte

xerophyte is a species of plant that has

adapted to survive in an environment

with little liquid water such as a desert

or an ice- or snow-covered region in the

Alps or the Arctic

Mesophytes are terrestrial plants which

are adapted to neither a particularly

dry nor particularly wet environment

An example of a mesophytic habitat

would be a rural temperate meadow

F A C E B O O K

P A G E h t t p w w w f a c e b o o k c o m s s c m e n t o r s o f f i c i a l P a g e | 47

FOR MORE UPDATES AND MORE MATERIAL DO LIKE OUR FACEBOOK PAGE httpwwwfacebookcomsscmentorsofficial

which might contain goldenrod clover

oxeye daisy and Rosa multiflora

thallophyte any of a group of plants or

plantlike organisms (such as algae and

fungi) that lack differentiated stems

leaves and roots and that were formerly

classified as a primary division

(Thallophyta) of the plant kingdom

Q204 How many types of teeth are

there in humans

मनषयोमककतनहॳपरकारकहॳ दातहह८तहॳहहॴ

1-Feb-2017

Options

1) 4

2) 5

3) 2

4) 3

Correct Answer 4

teeth -Humans have four types of

teethincisors canines premolars and

molars each with a specific function

The incisors cut the food the canines

tear the food and the molars and

premolars crush the food

Q205 Carica papaya is the scientific name of

कहॴ ररकापपाया mdashmdashndash कावहॴजञाननकनामहहॴ 2-

Feb-2017

Options

1) Peepal

पीपि

2) Papaya

पपीता 3) Tamarind

इमिी 4) Drumstick

ढह८िकाछड़ी Correct Answer Papaya

Q206 Muscles get tired when there is

shortfall of

जब mdashndash कीकमीहह८तीहहॴतबपहॳिीयिकजातीहहॴ 2-Feb-2017

Options

1) Lactic acid

िहॴनकटकएलसि

2) Na+ ions

Na+ आयन

3) ATP

एटीपी 4) Sulphates

समफहॳ टस

Correct Answer ATP

ATP is the energy source muscle fibers

use to make muscles contract

muscle tissuersquos main source of energy

called adenosine triphosphate or ATP

As your muscles use up this energy

source they become tired and fatigued

Oxygen is the key ingredient that helps

create new ATP to replenish the burned

up ATP in your muscles

Q207 Artocarpus integra is the

scientific name of आटह८कापयसइटीगरा mdashmdashmdash कावहॴजञाननकनामहहॴ 2-Feb-2017

Options

1) Guava

अम द

2) Pineapple

अनानास

3) Silver Oak

लसमवरओक

4) Jack fruit

कटहि

Correct Answer Jack fruit

Q208 Which organ stores fat soluble

vitamins

कह९नसाअगवसामघिनिीिपवटालमनह८काभिाराकरताहहॴ

2-Feb-2017

F A C E B O O K

P A G E h t t p w w w f a c e b o o k c o m s s c m e n t o r s o f f i c i a l P a g e | 48

FOR MORE UPDATES AND MORE MATERIAL DO LIKE OUR FACEBOOK PAGE httpwwwfacebookcomsscmentorsofficial

Options

1) Blood

रकत

2) Skin

तवचा 3) Liver

यकत

4) Pancreas

अगनयािय

Correct Answer Liver

Q209 Which disease is caused due to

deficiency of Iodine

आयह८िीनकहॳ कारणकह९नसारह८गहह८ताहहॴ 2-Feb-2017

Options

1) Rickets

ररकहॳ टस

2) Scurvy

सकवी 3) Goitre

गणमािा 4) Growth retardation

पवकासका कना Correct Answer Goitre

rickets A softening and weakening of

bones in children usually due to

inadequate vitamin D

Q210 Grevillea Robusta is the scientific name of

गरहॳपवलियारह८बसटा mdashmdashmdash- कापवजञाननकनामहहॴ 2-Feb-2017

Options

1) Peepal

पीपि

2) Teak

सागह९न

3) Silver Oak

लसमवरओक

4) Jack fruit

कटहि

Correct Answer Silver Oak

Q211 When a Cuttlefish is described as a Molluscs it is at which level of

classification

जबएककटिकफिकह८एकमह८िसकाकहॳ पमवखणयतककयाजाताहहॴतबयहॳवगीकरणकहॳ ककससतरपहॳनसितहहॴ 2-Feb-2017

Options

1) Class

वगय 2) Order

िम

3) Family

पररवार

4) Phylum

सघ

Correct Answer Phylum

Q212 Bambusa dendrocalmus is the

scientific name of बानबसािहॳडराकामस mdashmdashmdash कावहॴजञाननकनामहहॴ 3-Feb-2017

Options

1) Banyan

बरगद

2) Papaya

पपीता 3) Bamboo

बास

4) Pomegranate

अनार

Correct Answer Bamboo

Q213 Acinonyx Jubatus is the scientific name of

एलसनह८ननकसजयबहॳटस mdashmdashmdash

कावहॴजञाननकनामहहॴ 3-Feb-2017

F A C E B O O K

P A G E h t t p w w w f a c e b o o k c o m s s c m e n t o r s o f f i c i a l P a g e | 49

FOR MORE UPDATES AND MORE MATERIAL DO LIKE OUR FACEBOOK PAGE httpwwwfacebookcomsscmentorsofficial

Options

1) Bear

भाि 2) Horse

घह८िा 3) Cheetah

चीता 4) Zebra

जहॳिा Correct Answer Cheetah

Q214 The pale yellow colour of urine is

due to the presence of which pigment

मतरकाफीकापीिारगरगदरयकहॳ उपनसिनतकहॳ कारणहह८ताहहॴ

3-Feb-2017

Options

1) Urochrome

यरह८िह८म

2) Urophyll

यरह८कफि

3) Chlorophyll

किह८रह८कफि

4) Chloroplast

किह८रह८पिासट

Correct Answer Urochrome

Q215 Which of the following constitute

to form a gene

नननननलिखितमसहॳकह९नसीचीज़एकजीनकागठनकरतीहहॴ

3-Feb-2017

Options

1) Polynucleotides

पह८िीनयनकियह८टाईडस

2) Hydrocarbons

हाइडरह८काबोस

3) Lipoproteins

िाईपह८परह८टीनस

4) Lipids

लिपपडस

Correct Answer Polynucleotides

Polynucleotide molecule is a biopolymer

composed of 13 or more nucleotide

monomers covalently bonded in a chain

DNA (deoxyribonucleic acid) and RNA

(ribonucleic acid) are examples of

polynucleotides with distinct biological

function

Q216 Vertebrates belongs to the

phylum

रीढ़कीहडिीवािहॳपराणी mdashmdashmdash

परजानतकहॳ अतगायतआतहॳहहॴ 3-Feb-2017

Options

1) Arthropoda

आरह८पह८ड़ा 2) Annelida

एननलििा 3) Cnidaria

ननिहॳररया 4) Chordata

कह८िटा Correct Answer Chordata

Q217 Punica granatum is the scientific name of

पननकगरहॳनहॳटस mdashmdashmdash कावहॴजञाननकनामहहॴ 3-Feb-2017

Options

1) Custard Apple

सीताफि

2) Gulmohar

गिमह८हर

3) Silver Oak

लसमवरओक

4) Pomegranate

अनार

Correct Answer Pomegranate

F A C E B O O K

P A G E h t t p w w w f a c e b o o k c o m s s c m e n t o r s o f f i c i a l P a g e | 50

FOR MORE UPDATES AND MORE MATERIAL DO LIKE OUR FACEBOOK PAGE httpwwwfacebookcomsscmentorsofficial

Q218 Between a tiger and an monkey

which of the following is different

एकबाघऔरबदरकहॳ बीचनननननलिखितमसहॳकह९नसीबातअिगहहॴ 3-Feb-2017

Options

1) Kingdom

राजय

2) Phylum

जानत

3) Order

िम

4) Class

वगय Correct Answer order

Q219 The artificial heart was invented by

कबतरमहदयका mdashmdashmdash

दवाराअपवषकारककयागयािा 3-Feb-2017

Options

1) Muhammad Yunus

महनमदयनस

2) Linus Yale Jr

िाइनसयहॳिजय

3) Gazi Yasargil

गाजीयासचगयि

4) Paul Winchell

पह९िपवमकि Correct Answer Paul Winchell

Q220 Tamarindus indica is the

scientific name of

टहॳमररनडसइडिका mdashmdash कावहॴजञाननकनामहहॴ 7-

Feb-2017

Options

1) Neem

नीम

2) Pineapple

अनानास

3) Tamarind

इमिी 4)Chiku

चीक

Correct Answer Tamarind

Q221 In eukaryotic cells synthesis of

RNA takes place in the

यकहॳ योटटककह८लिकाओमआरएनएकासशिहॳषण

mdashndash महह८ताहहॴ 7-Feb-2017

Options

1) Mitochondria

माईटह८कोडडरया 2) Centrioles

सटरीयह८मस

3) Ribosomes

ररबह८सह८नस

4) Nucleus

नयनकियस

Correct Answer nucleus

eukaryotic cell -Transcription is the

process of synthesizing ribonucleic acid

(RNA)Synthesis takes place within the

nucleus of eukaryotic cells or in the

cytoplasm of prokaryotes and converts

the genetic code from a gene in

deoxyribonucleic acid ( DNA ) to a

strand of RNA that then directs

proteinsynthesis

Q222 _________is caused by parasites

of the Plasmodium genus

पिाजमह८डियमजातीकहॳ परजीवी mdash- कहॳ कारणहहॴ 7-Feb-2017

Options

1) Dysentery

पहॳचचि

2) Malaria

मिहॳररया 3) Chickenpox

F A C E B O O K

P A G E h t t p w w w f a c e b o o k c o m s s c m e n t o r s o f f i c i a l P a g e | 51

FOR MORE UPDATES AND MORE MATERIAL DO LIKE OUR FACEBOOK PAGE httpwwwfacebookcomsscmentorsofficial

चहॳचक

4) Herpes

हहॳपपयस

Correct Answer Malaria

Q223 Carotene in fruits and vegetables

gives it which color

फिह८औरसनलजयोमनसितकहॳ रह८टीनउनहकह९नसारगपरदानकरताहहॴ 7-Feb-2017

Options

1) Green

हरा 2) Pink

गिाबी 3) Orange

नारगी 4) Blue

नीिा Correct Answer Orange

Q224 Equus Caballus is the scientific

name of

एकवसकहॴ बहॳिस mdashmdashndash कापवजञाननकनामहहॴ 7-Feb-2017

Options

1) Horse

घह८िा 2) Zebra

ज़हॳिा 3) Donkey

गधा 4) Buffalo

भस

Correct Answer Horse

Q225 Elapidae Naja is the scientific name of

एिीपीिीनाजा mdashmdash- कावहॴजञाननकनामहहॴ 8-Feb-2017

Options

1) Cobra

कह८बरा 2) Elephant

हािी 3) Eagle

ग ि

4) Owl

उमि Correct Answer Cobra

Q226 Which disease is caused due to

deficiency of Iron

िह८हकीकमीकहॳ कारणकह९नसारह८गहह८ताहहॴ 8-Feb-

2017

Options

1) Beriberi

बहॳरीबहॳरी 2) Tetany

टहॳटनी 3) Kwashiorkor

कवािीऔरकर

4) Anaemia

रकतामपता Correct Answer Anaemia

Beriberi is a disease caused by a vitamin

B-1 deficiency also known as thiamine

deficiency

Tetany can be the result of an

electrolyte imbalance Most often itrsquos a

dramatically low calcium level also

known as hypocalcemia Tetany can also

be caused by magnesium deficiency or

too little potassium Having too much

acid (acidosis) or too much alkali

(alkalosis) in the body can also result in

tetany

Kwashiorkor also known as

ldquoedematous malnutrition It is a form of

malnutrition caused by a lack of protein

in the diet

Anaemia means that you have fewer red

blood cells than normal or you have less

F A C E B O O K

P A G E h t t p w w w f a c e b o o k c o m s s c m e n t o r s o f f i c i a l P a g e | 52

FOR MORE UPDATES AND MORE MATERIAL DO LIKE OUR FACEBOOK PAGE httpwwwfacebookcomsscmentorsofficial

haemoglobin than normal in each red

blood cell

Q227 is a leaf where the leaflets are

arranged along the middle vein

mdashndashएकपततीहहॴजहापतरकह८कीरचनाक ररयालिराकहॳ आसपासहह८तीहहॴ 8-Feb-2017

Options

1) Pinnately compound leaf

पपनहॳटिीसयकतपतती 2) Palmately compound leaf

पामहॳटिीसयकतपतती 3) Compound leaf

सयकतपतती 4) Simple leaf

साधारणपतती Correct Answer Pinnately compound

leaf

Q228 Haustoria or sucking roots are

found in which of the following

हह८सटह८ररयायाचसनहॳवािीजड़हॳनननननलिखितमसहॳककसमपाईजातीहहॴ 8-Feb-2017

Options

1) Wheat

गहॳह

2) Mango

आम

3) Chestnut

चहॳसटनट

4) Cuscuta

कसकयटा Correct Answer Cuscuta

Haustorial roots -The roots of parasitic

plants which penetrate into the host

tissues to absorb nourishment are

called haustorial roots hellip Also known as suckingor parasitic roots

Q229 Equs Asinus is the scientific name

of

एकवसएलसनस mdashmdashndash कावहॴजञाननकनामहहॴ 8-

Feb-2017

Options

1) Donkey

गधा 2) Cow

गाय

3) Deer

टहरन

4) Kangaroo

कगा

Correct Answer Donkey

Q230 Ficus benghalensis is the scientific name of

फाईकसबहॳनगहॳिहॳलसस mdashndash कापवजञाननकनामहहॴ 8-Feb-2017

Options

1) Banyan

बरगद

2) Pineapple

अनानास

3) Babul

बबि

4) Tulsi

तिसी Correct Answer Banyan

Q231 Equus burchellii is the scientific name of

एकवसबचिी mdashmdash- कापवजञाननकनामहहॴ 8-Feb-2017

Options

1) Horse

घह८िा 2) Zebra

जहॳिा 3) Buffalo

F A C E B O O K

P A G E h t t p w w w f a c e b o o k c o m s s c m e n t o r s o f f i c i a l P a g e | 53

FOR MORE UPDATES AND MORE MATERIAL DO LIKE OUR FACEBOOK PAGE httpwwwfacebookcomsscmentorsofficial

भस

4) Ass

गधा Correct Answer Zebra

Page 32: COMPILATION OF ALL 72 SETS OF BIOLOGY SSC CHSL-2016 · OF BIOLOGY SSC CHSL-2016 PREPARED BY : SSC MENTORS BIOLOGY SPECIAL . F A C E B O O K P A G E : h t t p : / / w w w . f a c e

F A C E B O O K

P A G E h t t p w w w f a c e b o o k c o m s s c m e n t o r s o f f i c i a l P a g e | 31

FOR MORE UPDATES AND MORE MATERIAL DO LIKE OUR FACEBOOK PAGE httpwwwfacebookcomsscmentorsofficial

सहॳमयिह८ज

Correct Answer Glucose

Intestinal absorption of end products

from digestion of carbohydrates and

proteins in the pig hellip During absorption some sugars (fructose or

galactose) released from the

corresponding sucrose and lactose

respectively during digestion were

partly metabolized into glucose by the

enterocyte

Q130 Which of the following glands is a

source of the enzyme Ptyalin

नननननलिखितगरचियोमसहॳएजाइमटयालिनकासरह८तहहॴ 23-Jan-2017

Options

1) Pancreas

अगरािय

2) Thyroid Gland

िाइराइिगरिी 3) Pituitary Gland

पीयषगरिी 4) Salivary Glands

िारगरचियाा Correct Answer Salivary Glands

Q131 Which of the following is not true

about Pteridophyta

ननननमसहॳकह९नसीबातटहॳररिह८फाईटकहॳ बारहॳमसचनहीहहॴ 23-Jan-2017

Options

1) Dominant phase is saprophytes

परमिचरणसहॳपरह८फाईइटसहह८ताहहॴ 2) Main plant body is diploid

पह९दह८कामखयिरीरदपवगखणतहह८ताहहॴ 3) Seeds are present

बीजमह९जदहह८तहॳहहॴ 4)Flowers are absent

फिअनपनसतिहह८तहॳहहॴ

Correct Answer Seeds are present

Q132 The largest dolphin species is the

orca also called as

िॉिकफनकीसबसहॳबड़ीपरजानतकाकानामआकायहहॴनजसहॳ mdash- भीकहतहॳहहॴ 23-Jan-2017

Options

1) Bottle Nose

बाटिनह८ज

2) Baiji

बहॳजी 3) Killer whale

ककिरहहॳि

4)Tucuxi

टकवसी Correct Answer Killer whale

Q133 The fat digesting enzyme Lipase

is secreted by which of the following

वसाकापाचनकरनहॳवािाएजाइमिाइपहॳजनननननलिखितमसहॳककसकहॳ दवारासतरापवतहह८ताहहॴ

24-Jan-2017

Options

1) Kidneys

गद

2) Pancreas

अगनयािय

3) Large Intestine

बड़ीआत

4)Liver

नजगर

Correct Answer Pancreas

Lipase is an enzyme that splits fats so

the intestines can absorb them Lipase

hydrolyzes fats like triglycerides into

their component fatty acid and glycerol

molecules It is found in the blood

gastric juices pancreatic secretions

intestinal juices and adipose tissues

F A C E B O O K

P A G E h t t p w w w f a c e b o o k c o m s s c m e n t o r s o f f i c i a l P a g e | 32

FOR MORE UPDATES AND MORE MATERIAL DO LIKE OUR FACEBOOK PAGE httpwwwfacebookcomsscmentorsofficial

Q134 The arrangement of leaves on an

axis or stem is called

एकअकषयातनहॳपरपनततयोकीयवसिाकह८कयाकहाजाताहहॴ SSC CHSL Science (biology) 2016

Question Paper

24-Jan-2017

Options

1) Phyllotaxy

फाइिह८टहॴकसी 2) Vernation

वनिन

3) Venation

वहॳनहॳिन

4)Phytotaxy

फाइटह८टहॴकसी Correct Answer Phyllotaxy

In botany phyllotaxis or phyllotaxy is

the arrangement of leaves on a plant

stem (from Ancient Greek phyacutellon

ldquoleafrdquo and taacutexis ldquoarrangementrdquo)

Phyllotactic spirals form a distinctive

class of patterns in nature

Q135 The study of Cells is also known

as

कह८लिकाओकहॳ अधययनकह८ mdashmdashndash

भीकहाजाताहहॴ 24-Jan-2017

Options

1) Cytology

सायटह८िह८जी 2) Physiology

कफनजयह८िह८जी 3) Nucleology

नयककमयह८िह८जी 4)Cellology

सहॳिह८िह८जी Correct Answer Cytology

Q136 Which of the following scientists

is also known as the Father of Biology

नननननलिखितमसहॳककसवहॴजञाननककह८ ldquoजीवपवजञानकहॳ जनकrdquoकहॳ नामसहॳभीजानाजाताहहॴ 24-Jan-2017

Options

1) Herbert Spencer

हबयटयसपसर

2) Aristotle

अरसत 3) Lamarck

िहॳमाकय 4)Darwin

िापवयन

Correct Answer Aristotle

Q137 Which cells give rise to various

organs of the plant and keep the plant

growing

कह९नसीकह८लिकाएपह९धह८कहॳ लभननअगह८कह८जनमदहॳतीहहॴऔरपह९धह८कह८बढ़नहॳममददकरतीहहॴ

24-Jan-2017

Options

1) Permanent

सिायी 2) Dermal

तवचीय

3) Meristematic

मररसटहॳमटटक

4)Mature

परह८ढ़

Correct Answer Meristematic

A meristem is the tissue in most plants

containing undifferentiated cells

(meristematic cells) found in zones of

the plant where growth can take place

Q138 Rodentia Muridae is the scientific

name of

F A C E B O O K

P A G E h t t p w w w f a c e b o o k c o m s s c m e n t o r s o f f i c i a l P a g e | 33

FOR MORE UPDATES AND MORE MATERIAL DO LIKE OUR FACEBOOK PAGE httpwwwfacebookcomsscmentorsofficial

रह८िहॳलियानयररिी mdashmdash- कावहॴजञाननकनामहहॴ 24-

Jan-2017

Options

1) Mouse

चहा 2) Squirrel

चगिहरी 3) Monkey

बदर

4) Lizard

नछपकिी Correct Answer Mouse

Q139 Name the scientist who proposed

the cell theory

कह८लिकालसदातकापरसतावदहॳनहॳवािहॳवहॴजञाननककानामबताइए 24-Jan-2017

Options

1) Schleiden and Schwann

िीमिनऔरशरववान

2) Lamarck

िहॳमाकय 3) Treviranus

टरहॳवायरहॳनस

4)Whittaker and Stanley

हीटकरऔरसटहॳनिहॳ Correct Answer Schleiden and

Schwann

Q140 The flower with the worldrsquos

largest bloom is

दननयाकासबसहॳबड़ाफिखििनहॳवािा mdashmdashndash हहॴ 24-Jan-2017

Options

1) Pando

पािह८ 2) Posidonia

पह८सीिह८ननया 3) Rafflesia arnoldii

ररफिहॳलियाअनोमिी 4)Helianthus annuus

हहॳलिएनिसएनयअस

Correct Answer Rafflesia arnoldii

Rafflesia arnoldii is a species of

flowering plant in the parasitic genus

Rafflesia It is noted for producing the

largest individual flower on earth It has

a very strong and horrible odour of

decaying flesh earning it the nickname

ldquocorpse flower

Q141 Deficiency of which vitamin

causes night blindness

ककसपवटालमनकीकमीकहॳ कारणरतौधीहह८ताहहॴ 24-Jan-2017

Options

1) Vitamin K

पवटालमन K

2) Vitamin C

पवटालमन C

3) Vitamin B1

पवटालमन B1

4)Vitamin A

पवटालमन A

Correct Answer Vitamin A

Q142 Nongreen plants lack which of the

following

गहॴर-

हररतवनसपनतमनननननलिखितमसहॳककसकीकमीहह८तीहहॴ

24-Jan-2017

Options

1) Chlorophyll

किह८रह८कफि

2) Lycophyll

िायकह८कफि

3) Cyanophyll

F A C E B O O K

P A G E h t t p w w w f a c e b o o k c o m s s c m e n t o r s o f f i c i a l P a g e | 34

FOR MORE UPDATES AND MORE MATERIAL DO LIKE OUR FACEBOOK PAGE httpwwwfacebookcomsscmentorsofficial

सायनह८कफि

4)Phototropism

फह८टह८टरोपपजम

Correct Answer Chlorophyll

Q143 Organisms that use light to

prepare food are known as

जह८जीवपरकािकाउपयह८गकरभह८जनतहॴयारकरतहॳहहॴ उनह mdashmdash- कहॳ पमजानजाताहहॴ 24-Jan-2017

Options

1) Autotrophs

सवपह८षी 2) Heterotrophs

पवषमपह८षज

3) Omnivores

सवायहारी 4)Decomposers

पवघटनकरनहॳवािा Correct Answer Autotrophs

autotrophs often make their own food

by using sunlight carbon dioxide and

water to form sugars which they can use

for energy Some examples of

autotrophs include plants algae and

even some bacteria Autotrophs

(producer) are important because they

are a food source for heterotrophs

(consumers)

A heterotroph is an organism that

ingests or absorbs organic carbon

(rather than fix carbon from inorganic

sources such as carbon dioxide) in order

to be able to produce energy and

synthesize compounds to maintain its

life Ninety-five percent or more of all

types of living organisms are

heterotrophic including all animals and

fungi and some bacteria

Q144 Which of the following is a

primary function of haemoglobin

नननननलिखितमसहॳकह९नसाटहमह८गिह८बबनकाएकपरािलमककाययहहॴ

25-Jan-2017

Options

1) Utilization of energy

उजायकाउपयह८गकरना 2) Prevention of anaemia

रकतामपताहह८नहॳसहॳरह८कना 3) Destruction of bacteria

बहॴकटीररयाकापवनािकरना 4) To transport oxygen

ऑकसीजनकावहनकरना Correct Answer To transport oxygen

Q145 Vascular bundles are absent in

सवहनीबिि mdashmdash- मअनपनसतिरहतहॳहहॴ 25-Jan-2017

Options

1) Bryophyta

िायह८फाइटा 2) Pteridophyta

टहॳररिह८फाईटा 3) Gymnosperms

नजननह८सपमय 4) Angiosperms

एननजयह८सपहॳनसय Correct Answer Bryophyta

Q146 Sauria Lacertidae is the scientific

name of

सहॴररयािहॳसरटाईिी mdashmdashndash कावहॴजञाननकनामहहॴ 25-Jan-2017

Options

1) Crocodile

मगरमचछ

2) Hippopotamus

टहपपह८पह८टहॳमस

3) Lizard

नछपकिी 4) House fly

F A C E B O O K

P A G E h t t p w w w f a c e b o o k c o m s s c m e n t o r s o f f i c i a l P a g e | 35

FOR MORE UPDATES AND MORE MATERIAL DO LIKE OUR FACEBOOK PAGE httpwwwfacebookcomsscmentorsofficial

घरहॳिमकिी Correct Answer Lizard

Q147 Which type of pathogen causes

the water-borne disease SARS (Severe

Acute Respiratory Syndrome)

ककसपरकािकारह८गज़नकजिजननतबीमारीसासयकाकारणबनताहहॴ 25-Jan-2017

Options

1) Viral

वायरि

2) Parasitic

परजीवी 3) Protozoan

परह८टह८जअन

4) Bacterial

बहॴकटीररयि

Correct Answer Viral

Q148 Which of the following organs

produces the enzyme lipase

नननननलिखितमसहॳकह९नसाअगिायपहॳजएजाइमउतपननकरताहहॴ 25-Jan-2017

Options

1) Pancreas

अगनयािय

2) Large Intestine

बड़ीआत

3) Liver

नजगर

4) Small Intestine

छह८टीआत

Correct Answer Pancreas

Q149 A is a long internode forming the

basal part or the whole of a peduncle

एक mdashmdash- एकिबाइटरनह८िहहॴ जह८ननचिाटहससायासनपणयिठिबनताहहॴ 25-

Jan-2017

Options

1) Rhizome

परकद

2) Rachis

महॳ दि

3) floral axis

पषपअकष

4) Scape

भगदड़

Correct Answer scape

Q150 ndash Which of the following

organisms are considered to be both

Living and Non-living

नननननलिखितमसहॳकह९नसहॳजीवाणकह८जीपवतऔरअजीपवतमानाजाताहहॴ

25-Jan-2017

Options

1) Bacteria

बहॴकटीररया 2) Fungi

कवक

3) Algae

िहॴवाि

4)Virus

वायरस

Correct Answer Virus

They are considered to be living as they

possess a protein coat as a protective

covering DNA as the genetic material

etc

They are said to be non-living as they

can be crystallised and they survive for

billions of years They can tolerate high

temperatures freezing cold

temperatures ultra-violet radiations etc

Q151 Deficiency of fluorine causes

which of the following

फिह८ररनकीकमीकहॳ कारणनननननलिखितमसहॳकयाहह८ताहहॴ

F A C E B O O K

P A G E h t t p w w w f a c e b o o k c o m s s c m e n t o r s o f f i c i a l P a g e | 36

FOR MORE UPDATES AND MORE MATERIAL DO LIKE OUR FACEBOOK PAGE httpwwwfacebookcomsscmentorsofficial

27-Jan-2017

Options

1) Dental Caries

िटिकहॴ ररज

2) Scurvy

सकवरी 3) Anaemia

रकतामपता 4) Arthritis

गटठया Correct Answer Dental Caries

Q152 In a Punnett Square with the

cross AaBb x AaBb how many Aabb

genotypes would be created

पनहॳटसककायरमिह८स AaBb x AaBb कहॳ साि

ककतनहॳ Aabb जीनह८टाइपबनगहॳ 27-Jan-2017

Options

1) 1

2) 8

3) 2

4) 3

Correct Answer 2

Q153 Which of the following is the

Controlling Center of the Cell

नननननलिखित म सहॳ कह८लिकाका ननयतरण

क दर कह९न हहॴ

27-Jan-2017

Options

1) Nucleus

क दर

2) Plasma

पिाजमा 3) Lysosome

िायसह८सह८म

4) Chromosome

िह८मह८सह८म

Correct Answer Nucleus

The control centre of the cell is the

nucleus in eukaryotic cells The nucleus

contains genetic material in the form of

DNA

Q154 Myopia affects which of the

following organs

मायह८पपयानननननलिखितअगह८मसहॳककसहॳपरभापवतकरताहहॴ

25-Jan-2017

Options

1) Heart

हदय

2) Skin

तवचा 3) Eyes

आािहॳ 4)Mouth

मह

Correct Answer Eyes

Q155 Which of the following bears

flowers

नननननलिखितमसहॳकह९नफिधारणकरताहहॴ

25-Jan-2017

Options

1) Bryophyta

िायह८फाइटा 2) Pteridophyta

टहॳरीिह८फाईटा 3) Gymnosperms

नजननह८सपमय 4)Angiosperms

एननजयह८सपमय Correct Answer Angiosperms

Q156 Oxygenated blood flows out of the

heart through the

ऑकसीजनयकतरकत mdashmdashmdash

कहॳ माधयमसहॳहदयकहॳ बाहरबहताहहॴ 25-Jan-2017

F A C E B O O K

P A G E h t t p w w w f a c e b o o k c o m s s c m e n t o r s o f f i c i a l P a g e | 37

FOR MORE UPDATES AND MORE MATERIAL DO LIKE OUR FACEBOOK PAGE httpwwwfacebookcomsscmentorsofficial

Options

1) Aorta

महाधमनी 2) pulmonary artery

फहॳ फड़हॳकीधमनी 3) vena cava

वहॳनाकावा 4)Atrium

चह९क

Correct Answer aorta

Q157 Blood leaving the liver and

moving towards the

heart has a higher concentration of

नजगरसहॳननकिकरहदयकीतरफजानहॳवािहॳरकतम mdashmdashmdashmdash कीउचचसादरताहह८तीहहॴ 27-Jan-2017

Options

1) Lipids

लिपपडस

2) Urea

यररया 3) Bile Pigments

पपततकहॳ रगकरण

4) Carbon dioxide

काबयनिायऑकसाइि

Correct Answer Bile Pigments

Urea is nitrogen containing substance

which is produced in the liver in order

to deal with excess amino-acids in the

body As urea is produced it leaves the

liver in the blood stream and passes via

the circulatory system to all parts of the

body

Q158 Bulb is a modification of which

part of a plant

बमबएकपह९धहॳकहॳ ककसटहससहॳकाएक पातरणहह८ताहहॴ 27-Jan-2017

Options

1) The root

जड़

2) The stem

तना 3) The radicle

मिाकर

4)The fruit

फि

Correct Answer The stem

Q159 Which of the following carries

blood away from the heart to different

body parts

इनमहॳसहॳकह९नरकतकह८हदयसहॳिरीरकहॳ पवलभननअगह८तकिहॳजातीहहॴ

27-Jan-2017

Options

1) Arteries

धमननया 2) Nerves

तबतरहाए

3) Capillaries

कहॳ लिकाए

4)Veins

नसहॳ Correct Answer Arteries

Q160 The series of processes by which

nitrogen and its compounds are

interconverted in the environment and

in living organisms is called

27-Jan-2017

Options

1)Absorption of Nitrogen

2)Ammonification

3)Nitrogen Fixation

4)Nitrogen Cycle

Correct Answer Nitrogen Cycle

Ammonification or Mineralization is

performed by bacteria to convert

organic nitrogen to ammonia

F A C E B O O K

P A G E h t t p w w w f a c e b o o k c o m s s c m e n t o r s o f f i c i a l P a g e | 38

FOR MORE UPDATES AND MORE MATERIAL DO LIKE OUR FACEBOOK PAGE httpwwwfacebookcomsscmentorsofficial

Nitrification can then occur to convert

the ammonium to nitrite and nitrate

Nitrogen fixation is a process by which

nitrogen in the Earthrsquos atmosphere is

converted into ammonia (NH3) or other

molecules available to living organisms

Q161 BCG vaccine is given to protect

from which of the following

बीसीजीकाटटकानननननलिखितमसहॳककसकहॳ बचावकहॳ लिएटदयाजातहहॴ

27-Jan-2017

Options

1) Jaundice

पीलिया 2) Anaemia

रकतमपता 3) Tuberculosis

कषयरह८ग

4) Polio

पह८लियह८ Correct Answer Tuberculosis

Q162 Parallel venation is found in

समानतरवहॳनहॳिन mdashmdashmdash- मपायाजाताहहॴ 27-Jan-2017

Options

1) plants which are monocots

पह९धहॳजह८एकबीजपतरीहह८तहॳहहॴ 2) plants which have a dicot stem

वहॳपह९धहॳनजनकातनादपवदलियहह८ताहहॴ 3) plants with leaves similar to Tulsi

वहॳपह९धहॳनजनकीपनततयतिसीकीपनततयोकहॳ समानहह८तहॳहहॴ 4)plants with tap roots

टहॳप टवािहॳपह९धहॳ Correct Answer plants which are

monocots

Q163 The hardest part of the body is

िरीरकासबसहॳकठह८रभाग mdashndash हहॴ 27-Jan-2017

Options

1) Bones

हडडिय

2) Tooth Enamel

दातकहॳ इनहॳमि

3) Skull

िह८पड़ी 4) Spinal Cord

महॳ रजज

Correct Answer Tooth Enamel

Q164 Which type of pathogen causes

the waterborne disease E coli Infection

ककसपरकारकारह८गजननकजिजननतरह८गईकह८िाईसिमणकाकारणबनताहहॴ 27-Jan-2017

Options

1) Protozoan

परह८टह८जआ

2) Parasitic

परजीवी 3) Bacterial

बहॴकटीररयि

4)Viral

वायरि

Correct Answer Bacterial

Q165 The amount of blood filtered

together by both the kidneys in a 70 kg

adult male human in a minute is

70 की गरा वािहॳएकवयसकप षमएकलमनटमदह८नोगदकहॳदवाराएकसािचाबनीगयीरकतकीमातरहह८तीहहॴ 29-Jan-2017

Options

1) 1100 ml

1100 लमलि

2) 100 ml

F A C E B O O K

P A G E h t t p w w w f a c e b o o k c o m s s c m e n t o r s o f f i c i a l P a g e | 39

FOR MORE UPDATES AND MORE MATERIAL DO LIKE OUR FACEBOOK PAGE httpwwwfacebookcomsscmentorsofficial

100 लमलि

3) 1500 ml

1500 लमलि

4) 500 ml

500 लमलि

Correct Answer 1100 ml

Q166 Which feature of a plant helps to

distinguish a monocot from a dicot

पह९धहॳकीवहकह९नसीपविहॳषताहहॴजह८एकदपवदलियहॳऔरएकएकदिीयपह९धहॳसहॳभहॳदकरनहॳममददकरतीहहॴ 29-Jan-2017

Options

1) Pollination

परागम

2) Venation

वहॳनहॳिन

3) Vernation

वनिन

4) Aestivation

एसटीवहॳिहॳन

Correct Answer venation

Q167 The Mutation Theory was

proposed by

उतवररवतयनकालसदात mdashmdashndash

कहॳ दवरापरसतापवतककयाजाताहहॴ 29-Jan-2017

Options

1) Charles Lyell

चामसयलियहॳि

2) William Smith

पवलियमनसमि

3) Hugo De Vries

हयगह८िीराईस

4)Harrison Schmitt

हहॳरीसननसमट

Correct Answer Hugo De Vries

Q168 Which type of pathogen causes

the waterborne disease HepatitisA

ककसपरकारकहॳ रह८गजनकजिजननतरह८गहहॳपहॳटाइटटस-A काकारणबनताहहॴ

29-Jan-2017

Options

1) Parasitic

परजीवी 2) Viral

वायरि

3) Protozoan

परह८टह८जआ

4) Bacterial

बहॴकटीररयि

Correct Answer Viral

Q169 In a Punnett Square with the

cross AaBb x Aabb how many AaBb

genotypes would be created

पनहॳटसकवायरमिह८स AaBb x Aabb

कहॳ सािककतनहॳ AaBb जीनह८टाइपबनगहॳ 29-Jan-

2017

Options

1) 4

2) 1

3) 7

4) 6

Correct Answer 4

Q170 Arboreal Ateles is the scientific

name of

अिह८ररयिएटटलिस mdashmdashmdash कावहॴजञाननकनामहहॴ 29-Jan-2017

Options

1) Squirrel

चगिहरी 2) Sparrow

गह८रहॴया 3) Lizard

नछपकिी 4) Spider monkey

F A C E B O O K

P A G E h t t p w w w f a c e b o o k c o m s s c m e n t o r s o f f i c i a l P a g e | 40

FOR MORE UPDATES AND MORE MATERIAL DO LIKE OUR FACEBOOK PAGE httpwwwfacebookcomsscmentorsofficial

मकड़ीबदर

Correct Answer Spider monkey

Q171 Which type of pathogen causes

the waterborne disease Salmonellosis

ककसपरकारकारह८गाणजिजननतबीमारीसािमह८नहॳिह८लसज़काकारकहहॴ

29-Jan-2017

Options

1) Algal

िहॳवालियहॳ 2) Parasitic

परजीवी 3) Bacterial

बहॴकटीररयि

4)Viral

वायरि

Correct Answer Bacterial

An infection with salmonella bacteria

commonly caused by contaminated food

or water

Symptoms include diarrhoea fever

chills and abdominal pain

Q172 is a condition in which there is a

deficiency of red cells or of haemoglobin

in the blood

mdashmdash-

एकनसिनतहहॴनजसमहॳरकतमिािकह८लिकाओकीयाहीमह८गिह८बबनकीकमीहह८तीहहॴ 29-Jan-2017

Options

1) Albinism

एनमबननजम

2) Propyria

परह८पीररया 3) Anaemia

एनीलमया 4)Keloid disorder

कहॳ िह८इिडिसओिर

Correct Answer Anaemia

Q173 Ananas comosus is the scientific

name of

Options

अनानासकह८मह८सस mdashmdashmdashndash

कावहॴजञाननकनामहहॴ 29-Jan-2017

1) Custard Apple

सीताफि

2) Pineapple

पाइनएपपि

3) Bamboo

बास

4)Pomegranate

अनार

Correct Answer Pineapple

Q174 Which organ produces insulin

कह९नसाअगइनसलिनपहॴदाकरताहहॴ 29-Jan-

2017

Options

1) Liver

यकत

2) Thyroid gland

िायराइिगरिी 3) Spleen

पिीहा 4)Pancreas

अगरयिय

Correct Answer Pancreas

Q175 Which of the following disease is

not caused by water pollution

नननननलिखितमसहॳकह९नसारह८गपानीकहॳ परदषणकहॳकारणनहीहह८ता

29-Jan-2017

Options

1) Cholera

हहॴजा 2) Typhoid

F A C E B O O K

P A G E h t t p w w w f a c e b o o k c o m s s c m e n t o r s o f f i c i a l P a g e | 41

FOR MORE UPDATES AND MORE MATERIAL DO LIKE OUR FACEBOOK PAGE httpwwwfacebookcomsscmentorsofficial

टाइफाइि

3) Asthma

दमा 4)Diarrhoea

दसत

Correct Answer Asthma

Q176 Ocimum tenuiflorum is the

scientific name of

ओलिलममटहॳयईफिह८रमइसकावहॴजञाननकनाम mdash

ndash हहॴ 30-Jan-2017

Options

1) Neem

नीम

2) Mango

आम

3) Babul

बबि

4)Tulsi

तिसी Correct Answer Tulsi

Q177 Which gland secretes bile a

digestive fluid

कह९नसीगरिीपपतत एकपाचनतरिपरदािय सरापवतकरतीहहॴ 30-Jan-2017

Options

1) Pancreas

अगनयािय

2) Liver

यकत

3) Thyroid

िायराइि

4) Testes

टहॳनसटस

Correct Answer liver

Q178 In which of the following the

dominant phase is Gametophyte

नननननलिखितमसहॳककसकहॳ परमिचरणयगमकह८दपवधद (Gametophyte)हहॴ 30-Jan-2017

Options

1) Bryophyta

िायह८फाइटा 2) Pteridophyta

टहॳररिह८फाइटा 3) Gymnosperms

नजननह८सपमय 4) Angiosperms

एननजयह८सपमय Correct Answer Bryophyta

Q179 Anaerobic respiration refers to

which of the following

नननननलिखितमसहॳककसहॳअवायवीयशवसनकहाजाताहहॴ

30-Jan-2017

Options

1) Respiration without Oxygen

ऑकसीजनकहॳ बबनाशवसन

2) Respiration with Oxygen

ऑकसीजनकहॳ सािशवसन

3) Respiration without CO2

काबयनिायऑकसाइिकहॳ बबनाशवसन

4) Respiration with CO2

काबयनिायऑकसाइिकहॳ सािशविन

Correct Answer Respiration without

Oxygen

Q180 Which type of pathogen causes

the waterborne disease Cholera

ककसपरकारकारह८गजनकजिजननतरह८गहहॴजाकाकारणबनताहहॴ

30-Jan-2017

Options

1) Algal

िहॴवालियहॳ

F A C E B O O K

P A G E h t t p w w w f a c e b o o k c o m s s c m e n t o r s o f f i c i a l P a g e | 42

FOR MORE UPDATES AND MORE MATERIAL DO LIKE OUR FACEBOOK PAGE httpwwwfacebookcomsscmentorsofficial

2) Bacterial

बहॴकटीररयि

3) Protozoan

परह८टह८जआ

4) Viral

वायरि

Correct Answer Bacterial

Q181 To which class does

Oxyreductases transferases hydrolases

belong

ओकसीररिकटहॳसटरासफरहॳजहॳस

हाइडरह८िहॳसहॳसककसवगयमआतहॳहहॴ 30-Jan-2017

Options

1) Hormones

हारमोस

2) Enzymes

एजाइनस

3) Proteins

परह८टीनस

4) Vitamins

पवटालमनस

Correct Answer Enzymes

Q182 Which of the following is not true

about Gymnosperms

ननननमसहॳकह९नसीबातअनावतबीजीकहॳ बारहॳमसचनहीहहॴ 30-Jan-2017

Options

1) Dominant phase is saprophytes

परमिचरणसहॳपरह८फाइटसहह८ताहहॴ 2) Vascular bundles are absent

सवहनीबििअनपनसितहह८ताहहॴ 3) spores are heterospores

बीजाणहहॳटहॳरह८सपह८रसहह८तहॳहहॴ 4) Flowers are absent

फिअनपनसितहह८तहॳहहॴ

Correct Answer Vascular bundles are

absent

Q183 The name of first mammal clone sheep is

भहॳड़कीपरिमसतनपायीपरनत प (किह८न)

कानामहहॴ 30-Jan-2017

Options

1) Noori

नरी 2) Dolly

िॉिी 3) Louise

िसी 4)Durga

दगाय Correct Answer Dolly

Q184 Which type of pathogen causes

the water-borne disease Typhoid fever

ककसपरकारकारह८गजनकजिजननतरह८गटाइफाइिबिारकाकारणबनताहहॴ 30-Jan-2017

Options

1) Algal

िहॴवािीय

2) Parasitic

परजीवी 3) Protozoan

परह८टह८जनअन

4)Bacterial

बहॴकटीररयि

Correct Answer Bacterial

Q185 In which part of the cell are

proteins made

कह८लिकाकहॳ ककसटहससहॳमपरह८टीनबनायाजाताहहॴ

31-Jan-2017

Options

1) Reticulum

रहॳटटकिम

F A C E B O O K

P A G E h t t p w w w f a c e b o o k c o m s s c m e n t o r s o f f i c i a l P a g e | 43

FOR MORE UPDATES AND MORE MATERIAL DO LIKE OUR FACEBOOK PAGE httpwwwfacebookcomsscmentorsofficial

2) Golgi apparatus

गह८मजीएपहॳरहॳटस

3) Ribosomes

ररबह८सह८नस

4) Lysosome

िायसह८सह८नस

Correct Answer ribosomes

Proteins are produced by stringing

amino acids together in the order

specified by messenger RNA strands

that were transcribed from DNA in the

cell nucleus The process of synthesizing

a protein is called translation and it

occurs on ribosomes in the cytoplasm of

a cell

Q186 Polio is a disease caused by which

of the following

नननननलिखितमसहॳपह८लियह८कीबबमारह८हह८नहॳकाकारणकयाहहॴ

31-Jan-2017

Options

1) Bacteria

बहॴकटीररयि

2) Mosquito

मचछर

3) Virus

वायरस

4) Cockroach

नतिच हॳ Correct Answer Virus

Polio or poliomyelitis is a crippling and

potentially deadly infectious disease It

is caused by the poliovirus

Q187 ndash Hay fever is a sign of which of

the following

हहॳकफवरनननननलिखितमसहॳककसकाएकसकहॳ तहहॴ

31-Jan-2017

Options

1) Old Age

वदावसिा 2) Malnutrition

कपह८सण

3) Allergy

एिनजय 4) Over Work

अतयचधककाययकरना Correct Answer Allergy

Q188 How many chromosomes does a

human cell contain

एकमानवकह८लिकामककतनहॳगणसतरहह८तहॳहहॴ

29-Jan-2017

Options

1) 6

2) 26

3) 46

4) 66

Correct Answer 46

In humans each cell normally contains

23 pairs of chromosomes for a total of

46 Twenty-two of these pairs called

autosomes look the same in both males

and females The 23rd pair the sex

chromosomes differ between males and

females

Q189 Which of the following is not true

about Bryophyta

ननननमसहॳकह९नसीबातिायह८फाइटकहॳ बारहॳमसचनहीहहॴ 31-Jan-2017

Options

1) Dominant phase is gametophytes

परमिचरणगहॳलमतह८फाइटसहह८ताहहॴ 2) Main plant body is haploid

पह९धहॳकामखयिरीरअगखणतहह८ताहहॴ 3) Spores are homospores

बीजाणहह८मह८सफह८रसहह८तहॳहहॴ 4) Flowers are present

फिमह८जदहह८तहॳहहॴ Correct Answer Flowers are present

F A C E B O O K

P A G E h t t p w w w f a c e b o o k c o m s s c m e n t o r s o f f i c i a l P a g e | 44

FOR MORE UPDATES AND MORE MATERIAL DO LIKE OUR FACEBOOK PAGE httpwwwfacebookcomsscmentorsofficial

Q190 Which aquatic animal has

trailing tentacles

ककसजिीयजानवरकहॳ पीछहॳचिनहॳवािहॳटहॳटकिसहह८तहॳहहॴ

31-Jan-2017

Options

1) Sea horse

समदरीघह८िा 2) Corals

मगा 3) Jelly fish

जहॳिीमछिी 4) Star fish

तारामछिी Correct Answer Jelly fish

Jellyfish with its umbrella-shaped bell

and trailing tentacles

Q191 Which type of pathogen causes

the water-borne disease Poliomyelitis

(Polio)

ककसपरकारकारह८गजनकजिजननतरह८गपह८लियह८मायहॳटटस (पह८लियह८) काकारणहहॴ 31-Jan-

2017

Options

1) Parasitic

परजीवी 2) Algal

िहॴवालिय

3) Viral

वायरि

4) Bacterial

बहॴकटीररयि

Correct Answer Viral

Q192 The outer white part of the eye

that protects the inner structures is

आािकाबाहरीसफहॳ दटहससाजह८आतररकसरचनाओकीरकषाकरताहहॴ वह mdashmdashmdash हहॴ 31-Jan-

2017

Options

1) Iris

आयररस

2) Sclera

सकिहॳरा 3) Retina

रहॳटटना 4) Cornea

कह८ननयया Correct Answer Sclera

Q193 Proteins are made up of

परह८टीनकाननमायण mdashndash सहॳहह८ताहहॴ 31-Jan-2017

Options

1) Amino acids

एलमनह८अनि

2) Fatty acids

वसायकतअनि

3) Glucose

गिकह८ज

4)Nucleotides

नयनकियह८टाईिस

Correct Answer Amino acids

Q194 Moringa Oleifera is the scientific

name of

मह८ररगओलिफहॳ रा mdashmdashndash कावहॴजञाननकनामहहॴ 31-Jan-2017

Options

1) Banyan

बरगद

2) Gulmohar

गिमह८हर

3) Amla

आमिा

F A C E B O O K

P A G E h t t p w w w f a c e b o o k c o m s s c m e n t o r s o f f i c i a l P a g e | 45

FOR MORE UPDATES AND MORE MATERIAL DO LIKE OUR FACEBOOK PAGE httpwwwfacebookcomsscmentorsofficial

4) Drumstick

डरमनसटक

Correct Answer Drumstick

Q195 Kidney stones are composed of

गदकीपिरी mdashndash सहॳबनीहह८तीहहॴ 1-Feb-2017

Options

1) Calcium Oxalate

कहॴ नमसयमओकजहॳिहॳट

2) Sodium Chloride

सह८डियमकिह८राइि

3) Magnesium Nitrate

महॳनगनलियमनाइतटरहॳट

4) Calcium Bicarbonate

कहॴ नमियमबायकबोनहॳट

Correct Answer Calcium Oxalate

Q196 ndash Which of the following is not

true about Angiosperms

ननननमसहॳकह९नसीबातआवतबीजीकहॳ बारहॳमसचनहीहहॴ 1-Feb-2017

Options

1) Dominant phase is gametophytes

परमिचरणगहॳलमतह८फाइटहह८ताहहॴ 2) Vascular bundles are present

सवहनीबििमह९जदहह८ताहहॴ 3) Spores are heterospores

बीजाणहहॳटहॳरह८सपह८रसहह८तहॳहहॴ 4) Seeds are covered

बीजढकहॳ हह८तहॳहहॴ Correct Answer Dominant phase is

gametophytes

Q197 All of the following are excretory

(waste) products of animals except

नननननलिखितमसहॳककसएककह८छह८ड़करअनयसभीपराखणयोदवाराउतसनजयतपदाियहहॴ 1-Feb-

2017

Options

1) Uric Acid

यररकएलसि

2) Ammonia

अमह८ननया 3) Carbohydrates

काबोहाइडरहॳट

4) Urea

यररया Correct Answer Carbohydrates

In animals the main excretory products

are carbon dioxide ammonia (in

ammoniotelics) urea (in ureotelics) uric

acid (in uricotelics) guanine (in

Arachnida) and creatine

Q198 RNA is a polymeric molecule

What does RNA stand for

आरएनइएएकबहिकआणहहॴ इसकाकापवय पकयाहहॴ 1-Feb-2017

Options

1) Rado Nuclear Acid

रािह८नयनकियरएलसि

2) Ribo Nucleic Acid

राइबह८नयनकिकएलसि

3) Rhino Nuclear Acid

हाइनह८नयनकियरएलसि

4) Resto Nucleus Acid

रहॳसटह८नयकिीयसएलसि

Correct Answer Ribo Nucleic Acid

Q199 Which organ does detoxification

and produces chemicals needed for

digestion

कह९नसाअगपवषहरणकरताहहॴऔरपाचनकहॳ लिएआवशयकरसायनोकह८पहॴदाकरताहहॴ 1-Feb-

2017

Options

1) Salivary glands

िारगरचिया 2) Pancreas

अगनयािय

F A C E B O O K

P A G E h t t p w w w f a c e b o o k c o m s s c m e n t o r s o f f i c i a l P a g e | 46

FOR MORE UPDATES AND MORE MATERIAL DO LIKE OUR FACEBOOK PAGE httpwwwfacebookcomsscmentorsofficial

3) Thyroid gland

िायराइिगरिी 4) Liver

यकत

Correct Answer Liver

Q200 Psidium guajava is the scientific

name of

लसडियमगआजावा mdashmdash कावहॴजञाननकनामहहॴ 1-

Feb-2017

Options

1) Guava

अम द

2) Mango

आम

3) Bamboo

बास

4) Jack fruit

कटहि

Correct Answer Guava

Q201 Which drug is used as a Blood

Thinner

चधरकह८पतिाकरनहॳकहॳ पमककसदवाकापरयह८गककयाजाताहहॴ

1-Feb-2017

Options

1) Warfarin

वाफर न

2) Tramadol

टरहॳमािह८ि

3) Azithromycin

एनजरह८मायलसन

4) Hydralazine

हाइडरह८िहॳनजन

Correct Answer Warfarin

Q202 Which of the following disease is

caused due to the deficiency of protein

परह८टीनकीकमीकहॳ कारणनननननलिखितमसहॳकह९नसारह८गहह८ताहहॴ 1-Feb-2017

Options

1) Arthritis

गटठया 2) Kwashiorkor

कािीओकय र

3) Goitre

गाइटर

4) Night Blindness

रतह९चध

Correct Answer Kwashiorkor

Q203 A is species of plant that has

adapted to survive in an environment

with little liquid water

mdashmdashndashपह९धहॳकीएकऐसहॳऐसहॳपरजानतहहॴ नजसनहॳकमपानीवािहॳवातावरणमजीपवतरहनहॳकहॳलिएअनकिनहहॴ 1-Feb-2017

Options

1) Xerophyte

म दपवद

2) Hydrophyte

जिीयपादप

3) Mesophyte

समह८दपवद

4) Thallophyte

िहॴिह८फाइटा Correct Answer xerophyte

xerophyte is a species of plant that has

adapted to survive in an environment

with little liquid water such as a desert

or an ice- or snow-covered region in the

Alps or the Arctic

Mesophytes are terrestrial plants which

are adapted to neither a particularly

dry nor particularly wet environment

An example of a mesophytic habitat

would be a rural temperate meadow

F A C E B O O K

P A G E h t t p w w w f a c e b o o k c o m s s c m e n t o r s o f f i c i a l P a g e | 47

FOR MORE UPDATES AND MORE MATERIAL DO LIKE OUR FACEBOOK PAGE httpwwwfacebookcomsscmentorsofficial

which might contain goldenrod clover

oxeye daisy and Rosa multiflora

thallophyte any of a group of plants or

plantlike organisms (such as algae and

fungi) that lack differentiated stems

leaves and roots and that were formerly

classified as a primary division

(Thallophyta) of the plant kingdom

Q204 How many types of teeth are

there in humans

मनषयोमककतनहॳपरकारकहॳ दातहह८तहॳहहॴ

1-Feb-2017

Options

1) 4

2) 5

3) 2

4) 3

Correct Answer 4

teeth -Humans have four types of

teethincisors canines premolars and

molars each with a specific function

The incisors cut the food the canines

tear the food and the molars and

premolars crush the food

Q205 Carica papaya is the scientific name of

कहॴ ररकापपाया mdashmdashndash कावहॴजञाननकनामहहॴ 2-

Feb-2017

Options

1) Peepal

पीपि

2) Papaya

पपीता 3) Tamarind

इमिी 4) Drumstick

ढह८िकाछड़ी Correct Answer Papaya

Q206 Muscles get tired when there is

shortfall of

जब mdashndash कीकमीहह८तीहहॴतबपहॳिीयिकजातीहहॴ 2-Feb-2017

Options

1) Lactic acid

िहॴनकटकएलसि

2) Na+ ions

Na+ आयन

3) ATP

एटीपी 4) Sulphates

समफहॳ टस

Correct Answer ATP

ATP is the energy source muscle fibers

use to make muscles contract

muscle tissuersquos main source of energy

called adenosine triphosphate or ATP

As your muscles use up this energy

source they become tired and fatigued

Oxygen is the key ingredient that helps

create new ATP to replenish the burned

up ATP in your muscles

Q207 Artocarpus integra is the

scientific name of आटह८कापयसइटीगरा mdashmdashmdash कावहॴजञाननकनामहहॴ 2-Feb-2017

Options

1) Guava

अम द

2) Pineapple

अनानास

3) Silver Oak

लसमवरओक

4) Jack fruit

कटहि

Correct Answer Jack fruit

Q208 Which organ stores fat soluble

vitamins

कह९नसाअगवसामघिनिीिपवटालमनह८काभिाराकरताहहॴ

2-Feb-2017

F A C E B O O K

P A G E h t t p w w w f a c e b o o k c o m s s c m e n t o r s o f f i c i a l P a g e | 48

FOR MORE UPDATES AND MORE MATERIAL DO LIKE OUR FACEBOOK PAGE httpwwwfacebookcomsscmentorsofficial

Options

1) Blood

रकत

2) Skin

तवचा 3) Liver

यकत

4) Pancreas

अगनयािय

Correct Answer Liver

Q209 Which disease is caused due to

deficiency of Iodine

आयह८िीनकहॳ कारणकह९नसारह८गहह८ताहहॴ 2-Feb-2017

Options

1) Rickets

ररकहॳ टस

2) Scurvy

सकवी 3) Goitre

गणमािा 4) Growth retardation

पवकासका कना Correct Answer Goitre

rickets A softening and weakening of

bones in children usually due to

inadequate vitamin D

Q210 Grevillea Robusta is the scientific name of

गरहॳपवलियारह८बसटा mdashmdashmdash- कापवजञाननकनामहहॴ 2-Feb-2017

Options

1) Peepal

पीपि

2) Teak

सागह९न

3) Silver Oak

लसमवरओक

4) Jack fruit

कटहि

Correct Answer Silver Oak

Q211 When a Cuttlefish is described as a Molluscs it is at which level of

classification

जबएककटिकफिकह८एकमह८िसकाकहॳ पमवखणयतककयाजाताहहॴतबयहॳवगीकरणकहॳ ककससतरपहॳनसितहहॴ 2-Feb-2017

Options

1) Class

वगय 2) Order

िम

3) Family

पररवार

4) Phylum

सघ

Correct Answer Phylum

Q212 Bambusa dendrocalmus is the

scientific name of बानबसािहॳडराकामस mdashmdashmdash कावहॴजञाननकनामहहॴ 3-Feb-2017

Options

1) Banyan

बरगद

2) Papaya

पपीता 3) Bamboo

बास

4) Pomegranate

अनार

Correct Answer Bamboo

Q213 Acinonyx Jubatus is the scientific name of

एलसनह८ननकसजयबहॳटस mdashmdashmdash

कावहॴजञाननकनामहहॴ 3-Feb-2017

F A C E B O O K

P A G E h t t p w w w f a c e b o o k c o m s s c m e n t o r s o f f i c i a l P a g e | 49

FOR MORE UPDATES AND MORE MATERIAL DO LIKE OUR FACEBOOK PAGE httpwwwfacebookcomsscmentorsofficial

Options

1) Bear

भाि 2) Horse

घह८िा 3) Cheetah

चीता 4) Zebra

जहॳिा Correct Answer Cheetah

Q214 The pale yellow colour of urine is

due to the presence of which pigment

मतरकाफीकापीिारगरगदरयकहॳ उपनसिनतकहॳ कारणहह८ताहहॴ

3-Feb-2017

Options

1) Urochrome

यरह८िह८म

2) Urophyll

यरह८कफि

3) Chlorophyll

किह८रह८कफि

4) Chloroplast

किह८रह८पिासट

Correct Answer Urochrome

Q215 Which of the following constitute

to form a gene

नननननलिखितमसहॳकह९नसीचीज़एकजीनकागठनकरतीहहॴ

3-Feb-2017

Options

1) Polynucleotides

पह८िीनयनकियह८टाईडस

2) Hydrocarbons

हाइडरह८काबोस

3) Lipoproteins

िाईपह८परह८टीनस

4) Lipids

लिपपडस

Correct Answer Polynucleotides

Polynucleotide molecule is a biopolymer

composed of 13 or more nucleotide

monomers covalently bonded in a chain

DNA (deoxyribonucleic acid) and RNA

(ribonucleic acid) are examples of

polynucleotides with distinct biological

function

Q216 Vertebrates belongs to the

phylum

रीढ़कीहडिीवािहॳपराणी mdashmdashmdash

परजानतकहॳ अतगायतआतहॳहहॴ 3-Feb-2017

Options

1) Arthropoda

आरह८पह८ड़ा 2) Annelida

एननलििा 3) Cnidaria

ननिहॳररया 4) Chordata

कह८िटा Correct Answer Chordata

Q217 Punica granatum is the scientific name of

पननकगरहॳनहॳटस mdashmdashmdash कावहॴजञाननकनामहहॴ 3-Feb-2017

Options

1) Custard Apple

सीताफि

2) Gulmohar

गिमह८हर

3) Silver Oak

लसमवरओक

4) Pomegranate

अनार

Correct Answer Pomegranate

F A C E B O O K

P A G E h t t p w w w f a c e b o o k c o m s s c m e n t o r s o f f i c i a l P a g e | 50

FOR MORE UPDATES AND MORE MATERIAL DO LIKE OUR FACEBOOK PAGE httpwwwfacebookcomsscmentorsofficial

Q218 Between a tiger and an monkey

which of the following is different

एकबाघऔरबदरकहॳ बीचनननननलिखितमसहॳकह९नसीबातअिगहहॴ 3-Feb-2017

Options

1) Kingdom

राजय

2) Phylum

जानत

3) Order

िम

4) Class

वगय Correct Answer order

Q219 The artificial heart was invented by

कबतरमहदयका mdashmdashmdash

दवाराअपवषकारककयागयािा 3-Feb-2017

Options

1) Muhammad Yunus

महनमदयनस

2) Linus Yale Jr

िाइनसयहॳिजय

3) Gazi Yasargil

गाजीयासचगयि

4) Paul Winchell

पह९िपवमकि Correct Answer Paul Winchell

Q220 Tamarindus indica is the

scientific name of

टहॳमररनडसइडिका mdashmdash कावहॴजञाननकनामहहॴ 7-

Feb-2017

Options

1) Neem

नीम

2) Pineapple

अनानास

3) Tamarind

इमिी 4)Chiku

चीक

Correct Answer Tamarind

Q221 In eukaryotic cells synthesis of

RNA takes place in the

यकहॳ योटटककह८लिकाओमआरएनएकासशिहॳषण

mdashndash महह८ताहहॴ 7-Feb-2017

Options

1) Mitochondria

माईटह८कोडडरया 2) Centrioles

सटरीयह८मस

3) Ribosomes

ररबह८सह८नस

4) Nucleus

नयनकियस

Correct Answer nucleus

eukaryotic cell -Transcription is the

process of synthesizing ribonucleic acid

(RNA)Synthesis takes place within the

nucleus of eukaryotic cells or in the

cytoplasm of prokaryotes and converts

the genetic code from a gene in

deoxyribonucleic acid ( DNA ) to a

strand of RNA that then directs

proteinsynthesis

Q222 _________is caused by parasites

of the Plasmodium genus

पिाजमह८डियमजातीकहॳ परजीवी mdash- कहॳ कारणहहॴ 7-Feb-2017

Options

1) Dysentery

पहॳचचि

2) Malaria

मिहॳररया 3) Chickenpox

F A C E B O O K

P A G E h t t p w w w f a c e b o o k c o m s s c m e n t o r s o f f i c i a l P a g e | 51

FOR MORE UPDATES AND MORE MATERIAL DO LIKE OUR FACEBOOK PAGE httpwwwfacebookcomsscmentorsofficial

चहॳचक

4) Herpes

हहॳपपयस

Correct Answer Malaria

Q223 Carotene in fruits and vegetables

gives it which color

फिह८औरसनलजयोमनसितकहॳ रह८टीनउनहकह९नसारगपरदानकरताहहॴ 7-Feb-2017

Options

1) Green

हरा 2) Pink

गिाबी 3) Orange

नारगी 4) Blue

नीिा Correct Answer Orange

Q224 Equus Caballus is the scientific

name of

एकवसकहॴ बहॳिस mdashmdashndash कापवजञाननकनामहहॴ 7-Feb-2017

Options

1) Horse

घह८िा 2) Zebra

ज़हॳिा 3) Donkey

गधा 4) Buffalo

भस

Correct Answer Horse

Q225 Elapidae Naja is the scientific name of

एिीपीिीनाजा mdashmdash- कावहॴजञाननकनामहहॴ 8-Feb-2017

Options

1) Cobra

कह८बरा 2) Elephant

हािी 3) Eagle

ग ि

4) Owl

उमि Correct Answer Cobra

Q226 Which disease is caused due to

deficiency of Iron

िह८हकीकमीकहॳ कारणकह९नसारह८गहह८ताहहॴ 8-Feb-

2017

Options

1) Beriberi

बहॳरीबहॳरी 2) Tetany

टहॳटनी 3) Kwashiorkor

कवािीऔरकर

4) Anaemia

रकतामपता Correct Answer Anaemia

Beriberi is a disease caused by a vitamin

B-1 deficiency also known as thiamine

deficiency

Tetany can be the result of an

electrolyte imbalance Most often itrsquos a

dramatically low calcium level also

known as hypocalcemia Tetany can also

be caused by magnesium deficiency or

too little potassium Having too much

acid (acidosis) or too much alkali

(alkalosis) in the body can also result in

tetany

Kwashiorkor also known as

ldquoedematous malnutrition It is a form of

malnutrition caused by a lack of protein

in the diet

Anaemia means that you have fewer red

blood cells than normal or you have less

F A C E B O O K

P A G E h t t p w w w f a c e b o o k c o m s s c m e n t o r s o f f i c i a l P a g e | 52

FOR MORE UPDATES AND MORE MATERIAL DO LIKE OUR FACEBOOK PAGE httpwwwfacebookcomsscmentorsofficial

haemoglobin than normal in each red

blood cell

Q227 is a leaf where the leaflets are

arranged along the middle vein

mdashndashएकपततीहहॴजहापतरकह८कीरचनाक ररयालिराकहॳ आसपासहह८तीहहॴ 8-Feb-2017

Options

1) Pinnately compound leaf

पपनहॳटिीसयकतपतती 2) Palmately compound leaf

पामहॳटिीसयकतपतती 3) Compound leaf

सयकतपतती 4) Simple leaf

साधारणपतती Correct Answer Pinnately compound

leaf

Q228 Haustoria or sucking roots are

found in which of the following

हह८सटह८ररयायाचसनहॳवािीजड़हॳनननननलिखितमसहॳककसमपाईजातीहहॴ 8-Feb-2017

Options

1) Wheat

गहॳह

2) Mango

आम

3) Chestnut

चहॳसटनट

4) Cuscuta

कसकयटा Correct Answer Cuscuta

Haustorial roots -The roots of parasitic

plants which penetrate into the host

tissues to absorb nourishment are

called haustorial roots hellip Also known as suckingor parasitic roots

Q229 Equs Asinus is the scientific name

of

एकवसएलसनस mdashmdashndash कावहॴजञाननकनामहहॴ 8-

Feb-2017

Options

1) Donkey

गधा 2) Cow

गाय

3) Deer

टहरन

4) Kangaroo

कगा

Correct Answer Donkey

Q230 Ficus benghalensis is the scientific name of

फाईकसबहॳनगहॳिहॳलसस mdashndash कापवजञाननकनामहहॴ 8-Feb-2017

Options

1) Banyan

बरगद

2) Pineapple

अनानास

3) Babul

बबि

4) Tulsi

तिसी Correct Answer Banyan

Q231 Equus burchellii is the scientific name of

एकवसबचिी mdashmdash- कापवजञाननकनामहहॴ 8-Feb-2017

Options

1) Horse

घह८िा 2) Zebra

जहॳिा 3) Buffalo

F A C E B O O K

P A G E h t t p w w w f a c e b o o k c o m s s c m e n t o r s o f f i c i a l P a g e | 53

FOR MORE UPDATES AND MORE MATERIAL DO LIKE OUR FACEBOOK PAGE httpwwwfacebookcomsscmentorsofficial

भस

4) Ass

गधा Correct Answer Zebra

Page 33: COMPILATION OF ALL 72 SETS OF BIOLOGY SSC CHSL-2016 · OF BIOLOGY SSC CHSL-2016 PREPARED BY : SSC MENTORS BIOLOGY SPECIAL . F A C E B O O K P A G E : h t t p : / / w w w . f a c e

F A C E B O O K

P A G E h t t p w w w f a c e b o o k c o m s s c m e n t o r s o f f i c i a l P a g e | 32

FOR MORE UPDATES AND MORE MATERIAL DO LIKE OUR FACEBOOK PAGE httpwwwfacebookcomsscmentorsofficial

Q134 The arrangement of leaves on an

axis or stem is called

एकअकषयातनहॳपरपनततयोकीयवसिाकह८कयाकहाजाताहहॴ SSC CHSL Science (biology) 2016

Question Paper

24-Jan-2017

Options

1) Phyllotaxy

फाइिह८टहॴकसी 2) Vernation

वनिन

3) Venation

वहॳनहॳिन

4)Phytotaxy

फाइटह८टहॴकसी Correct Answer Phyllotaxy

In botany phyllotaxis or phyllotaxy is

the arrangement of leaves on a plant

stem (from Ancient Greek phyacutellon

ldquoleafrdquo and taacutexis ldquoarrangementrdquo)

Phyllotactic spirals form a distinctive

class of patterns in nature

Q135 The study of Cells is also known

as

कह८लिकाओकहॳ अधययनकह८ mdashmdashndash

भीकहाजाताहहॴ 24-Jan-2017

Options

1) Cytology

सायटह८िह८जी 2) Physiology

कफनजयह८िह८जी 3) Nucleology

नयककमयह८िह८जी 4)Cellology

सहॳिह८िह८जी Correct Answer Cytology

Q136 Which of the following scientists

is also known as the Father of Biology

नननननलिखितमसहॳककसवहॴजञाननककह८ ldquoजीवपवजञानकहॳ जनकrdquoकहॳ नामसहॳभीजानाजाताहहॴ 24-Jan-2017

Options

1) Herbert Spencer

हबयटयसपसर

2) Aristotle

अरसत 3) Lamarck

िहॳमाकय 4)Darwin

िापवयन

Correct Answer Aristotle

Q137 Which cells give rise to various

organs of the plant and keep the plant

growing

कह९नसीकह८लिकाएपह९धह८कहॳ लभननअगह८कह८जनमदहॳतीहहॴऔरपह९धह८कह८बढ़नहॳममददकरतीहहॴ

24-Jan-2017

Options

1) Permanent

सिायी 2) Dermal

तवचीय

3) Meristematic

मररसटहॳमटटक

4)Mature

परह८ढ़

Correct Answer Meristematic

A meristem is the tissue in most plants

containing undifferentiated cells

(meristematic cells) found in zones of

the plant where growth can take place

Q138 Rodentia Muridae is the scientific

name of

F A C E B O O K

P A G E h t t p w w w f a c e b o o k c o m s s c m e n t o r s o f f i c i a l P a g e | 33

FOR MORE UPDATES AND MORE MATERIAL DO LIKE OUR FACEBOOK PAGE httpwwwfacebookcomsscmentorsofficial

रह८िहॳलियानयररिी mdashmdash- कावहॴजञाननकनामहहॴ 24-

Jan-2017

Options

1) Mouse

चहा 2) Squirrel

चगिहरी 3) Monkey

बदर

4) Lizard

नछपकिी Correct Answer Mouse

Q139 Name the scientist who proposed

the cell theory

कह८लिकालसदातकापरसतावदहॳनहॳवािहॳवहॴजञाननककानामबताइए 24-Jan-2017

Options

1) Schleiden and Schwann

िीमिनऔरशरववान

2) Lamarck

िहॳमाकय 3) Treviranus

टरहॳवायरहॳनस

4)Whittaker and Stanley

हीटकरऔरसटहॳनिहॳ Correct Answer Schleiden and

Schwann

Q140 The flower with the worldrsquos

largest bloom is

दननयाकासबसहॳबड़ाफिखििनहॳवािा mdashmdashndash हहॴ 24-Jan-2017

Options

1) Pando

पािह८ 2) Posidonia

पह८सीिह८ननया 3) Rafflesia arnoldii

ररफिहॳलियाअनोमिी 4)Helianthus annuus

हहॳलिएनिसएनयअस

Correct Answer Rafflesia arnoldii

Rafflesia arnoldii is a species of

flowering plant in the parasitic genus

Rafflesia It is noted for producing the

largest individual flower on earth It has

a very strong and horrible odour of

decaying flesh earning it the nickname

ldquocorpse flower

Q141 Deficiency of which vitamin

causes night blindness

ककसपवटालमनकीकमीकहॳ कारणरतौधीहह८ताहहॴ 24-Jan-2017

Options

1) Vitamin K

पवटालमन K

2) Vitamin C

पवटालमन C

3) Vitamin B1

पवटालमन B1

4)Vitamin A

पवटालमन A

Correct Answer Vitamin A

Q142 Nongreen plants lack which of the

following

गहॴर-

हररतवनसपनतमनननननलिखितमसहॳककसकीकमीहह८तीहहॴ

24-Jan-2017

Options

1) Chlorophyll

किह८रह८कफि

2) Lycophyll

िायकह८कफि

3) Cyanophyll

F A C E B O O K

P A G E h t t p w w w f a c e b o o k c o m s s c m e n t o r s o f f i c i a l P a g e | 34

FOR MORE UPDATES AND MORE MATERIAL DO LIKE OUR FACEBOOK PAGE httpwwwfacebookcomsscmentorsofficial

सायनह८कफि

4)Phototropism

फह८टह८टरोपपजम

Correct Answer Chlorophyll

Q143 Organisms that use light to

prepare food are known as

जह८जीवपरकािकाउपयह८गकरभह८जनतहॴयारकरतहॳहहॴ उनह mdashmdash- कहॳ पमजानजाताहहॴ 24-Jan-2017

Options

1) Autotrophs

सवपह८षी 2) Heterotrophs

पवषमपह८षज

3) Omnivores

सवायहारी 4)Decomposers

पवघटनकरनहॳवािा Correct Answer Autotrophs

autotrophs often make their own food

by using sunlight carbon dioxide and

water to form sugars which they can use

for energy Some examples of

autotrophs include plants algae and

even some bacteria Autotrophs

(producer) are important because they

are a food source for heterotrophs

(consumers)

A heterotroph is an organism that

ingests or absorbs organic carbon

(rather than fix carbon from inorganic

sources such as carbon dioxide) in order

to be able to produce energy and

synthesize compounds to maintain its

life Ninety-five percent or more of all

types of living organisms are

heterotrophic including all animals and

fungi and some bacteria

Q144 Which of the following is a

primary function of haemoglobin

नननननलिखितमसहॳकह९नसाटहमह८गिह८बबनकाएकपरािलमककाययहहॴ

25-Jan-2017

Options

1) Utilization of energy

उजायकाउपयह८गकरना 2) Prevention of anaemia

रकतामपताहह८नहॳसहॳरह८कना 3) Destruction of bacteria

बहॴकटीररयाकापवनािकरना 4) To transport oxygen

ऑकसीजनकावहनकरना Correct Answer To transport oxygen

Q145 Vascular bundles are absent in

सवहनीबिि mdashmdash- मअनपनसतिरहतहॳहहॴ 25-Jan-2017

Options

1) Bryophyta

िायह८फाइटा 2) Pteridophyta

टहॳररिह८फाईटा 3) Gymnosperms

नजननह८सपमय 4) Angiosperms

एननजयह८सपहॳनसय Correct Answer Bryophyta

Q146 Sauria Lacertidae is the scientific

name of

सहॴररयािहॳसरटाईिी mdashmdashndash कावहॴजञाननकनामहहॴ 25-Jan-2017

Options

1) Crocodile

मगरमचछ

2) Hippopotamus

टहपपह८पह८टहॳमस

3) Lizard

नछपकिी 4) House fly

F A C E B O O K

P A G E h t t p w w w f a c e b o o k c o m s s c m e n t o r s o f f i c i a l P a g e | 35

FOR MORE UPDATES AND MORE MATERIAL DO LIKE OUR FACEBOOK PAGE httpwwwfacebookcomsscmentorsofficial

घरहॳिमकिी Correct Answer Lizard

Q147 Which type of pathogen causes

the water-borne disease SARS (Severe

Acute Respiratory Syndrome)

ककसपरकािकारह८गज़नकजिजननतबीमारीसासयकाकारणबनताहहॴ 25-Jan-2017

Options

1) Viral

वायरि

2) Parasitic

परजीवी 3) Protozoan

परह८टह८जअन

4) Bacterial

बहॴकटीररयि

Correct Answer Viral

Q148 Which of the following organs

produces the enzyme lipase

नननननलिखितमसहॳकह९नसाअगिायपहॳजएजाइमउतपननकरताहहॴ 25-Jan-2017

Options

1) Pancreas

अगनयािय

2) Large Intestine

बड़ीआत

3) Liver

नजगर

4) Small Intestine

छह८टीआत

Correct Answer Pancreas

Q149 A is a long internode forming the

basal part or the whole of a peduncle

एक mdashmdash- एकिबाइटरनह८िहहॴ जह८ननचिाटहससायासनपणयिठिबनताहहॴ 25-

Jan-2017

Options

1) Rhizome

परकद

2) Rachis

महॳ दि

3) floral axis

पषपअकष

4) Scape

भगदड़

Correct Answer scape

Q150 ndash Which of the following

organisms are considered to be both

Living and Non-living

नननननलिखितमसहॳकह९नसहॳजीवाणकह८जीपवतऔरअजीपवतमानाजाताहहॴ

25-Jan-2017

Options

1) Bacteria

बहॴकटीररया 2) Fungi

कवक

3) Algae

िहॴवाि

4)Virus

वायरस

Correct Answer Virus

They are considered to be living as they

possess a protein coat as a protective

covering DNA as the genetic material

etc

They are said to be non-living as they

can be crystallised and they survive for

billions of years They can tolerate high

temperatures freezing cold

temperatures ultra-violet radiations etc

Q151 Deficiency of fluorine causes

which of the following

फिह८ररनकीकमीकहॳ कारणनननननलिखितमसहॳकयाहह८ताहहॴ

F A C E B O O K

P A G E h t t p w w w f a c e b o o k c o m s s c m e n t o r s o f f i c i a l P a g e | 36

FOR MORE UPDATES AND MORE MATERIAL DO LIKE OUR FACEBOOK PAGE httpwwwfacebookcomsscmentorsofficial

27-Jan-2017

Options

1) Dental Caries

िटिकहॴ ररज

2) Scurvy

सकवरी 3) Anaemia

रकतामपता 4) Arthritis

गटठया Correct Answer Dental Caries

Q152 In a Punnett Square with the

cross AaBb x AaBb how many Aabb

genotypes would be created

पनहॳटसककायरमिह८स AaBb x AaBb कहॳ साि

ककतनहॳ Aabb जीनह८टाइपबनगहॳ 27-Jan-2017

Options

1) 1

2) 8

3) 2

4) 3

Correct Answer 2

Q153 Which of the following is the

Controlling Center of the Cell

नननननलिखित म सहॳ कह८लिकाका ननयतरण

क दर कह९न हहॴ

27-Jan-2017

Options

1) Nucleus

क दर

2) Plasma

पिाजमा 3) Lysosome

िायसह८सह८म

4) Chromosome

िह८मह८सह८म

Correct Answer Nucleus

The control centre of the cell is the

nucleus in eukaryotic cells The nucleus

contains genetic material in the form of

DNA

Q154 Myopia affects which of the

following organs

मायह८पपयानननननलिखितअगह८मसहॳककसहॳपरभापवतकरताहहॴ

25-Jan-2017

Options

1) Heart

हदय

2) Skin

तवचा 3) Eyes

आािहॳ 4)Mouth

मह

Correct Answer Eyes

Q155 Which of the following bears

flowers

नननननलिखितमसहॳकह९नफिधारणकरताहहॴ

25-Jan-2017

Options

1) Bryophyta

िायह८फाइटा 2) Pteridophyta

टहॳरीिह८फाईटा 3) Gymnosperms

नजननह८सपमय 4)Angiosperms

एननजयह८सपमय Correct Answer Angiosperms

Q156 Oxygenated blood flows out of the

heart through the

ऑकसीजनयकतरकत mdashmdashmdash

कहॳ माधयमसहॳहदयकहॳ बाहरबहताहहॴ 25-Jan-2017

F A C E B O O K

P A G E h t t p w w w f a c e b o o k c o m s s c m e n t o r s o f f i c i a l P a g e | 37

FOR MORE UPDATES AND MORE MATERIAL DO LIKE OUR FACEBOOK PAGE httpwwwfacebookcomsscmentorsofficial

Options

1) Aorta

महाधमनी 2) pulmonary artery

फहॳ फड़हॳकीधमनी 3) vena cava

वहॳनाकावा 4)Atrium

चह९क

Correct Answer aorta

Q157 Blood leaving the liver and

moving towards the

heart has a higher concentration of

नजगरसहॳननकिकरहदयकीतरफजानहॳवािहॳरकतम mdashmdashmdashmdash कीउचचसादरताहह८तीहहॴ 27-Jan-2017

Options

1) Lipids

लिपपडस

2) Urea

यररया 3) Bile Pigments

पपततकहॳ रगकरण

4) Carbon dioxide

काबयनिायऑकसाइि

Correct Answer Bile Pigments

Urea is nitrogen containing substance

which is produced in the liver in order

to deal with excess amino-acids in the

body As urea is produced it leaves the

liver in the blood stream and passes via

the circulatory system to all parts of the

body

Q158 Bulb is a modification of which

part of a plant

बमबएकपह९धहॳकहॳ ककसटहससहॳकाएक पातरणहह८ताहहॴ 27-Jan-2017

Options

1) The root

जड़

2) The stem

तना 3) The radicle

मिाकर

4)The fruit

फि

Correct Answer The stem

Q159 Which of the following carries

blood away from the heart to different

body parts

इनमहॳसहॳकह९नरकतकह८हदयसहॳिरीरकहॳ पवलभननअगह८तकिहॳजातीहहॴ

27-Jan-2017

Options

1) Arteries

धमननया 2) Nerves

तबतरहाए

3) Capillaries

कहॳ लिकाए

4)Veins

नसहॳ Correct Answer Arteries

Q160 The series of processes by which

nitrogen and its compounds are

interconverted in the environment and

in living organisms is called

27-Jan-2017

Options

1)Absorption of Nitrogen

2)Ammonification

3)Nitrogen Fixation

4)Nitrogen Cycle

Correct Answer Nitrogen Cycle

Ammonification or Mineralization is

performed by bacteria to convert

organic nitrogen to ammonia

F A C E B O O K

P A G E h t t p w w w f a c e b o o k c o m s s c m e n t o r s o f f i c i a l P a g e | 38

FOR MORE UPDATES AND MORE MATERIAL DO LIKE OUR FACEBOOK PAGE httpwwwfacebookcomsscmentorsofficial

Nitrification can then occur to convert

the ammonium to nitrite and nitrate

Nitrogen fixation is a process by which

nitrogen in the Earthrsquos atmosphere is

converted into ammonia (NH3) or other

molecules available to living organisms

Q161 BCG vaccine is given to protect

from which of the following

बीसीजीकाटटकानननननलिखितमसहॳककसकहॳ बचावकहॳ लिएटदयाजातहहॴ

27-Jan-2017

Options

1) Jaundice

पीलिया 2) Anaemia

रकतमपता 3) Tuberculosis

कषयरह८ग

4) Polio

पह८लियह८ Correct Answer Tuberculosis

Q162 Parallel venation is found in

समानतरवहॳनहॳिन mdashmdashmdash- मपायाजाताहहॴ 27-Jan-2017

Options

1) plants which are monocots

पह९धहॳजह८एकबीजपतरीहह८तहॳहहॴ 2) plants which have a dicot stem

वहॳपह९धहॳनजनकातनादपवदलियहह८ताहहॴ 3) plants with leaves similar to Tulsi

वहॳपह९धहॳनजनकीपनततयतिसीकीपनततयोकहॳ समानहह८तहॳहहॴ 4)plants with tap roots

टहॳप टवािहॳपह९धहॳ Correct Answer plants which are

monocots

Q163 The hardest part of the body is

िरीरकासबसहॳकठह८रभाग mdashndash हहॴ 27-Jan-2017

Options

1) Bones

हडडिय

2) Tooth Enamel

दातकहॳ इनहॳमि

3) Skull

िह८पड़ी 4) Spinal Cord

महॳ रजज

Correct Answer Tooth Enamel

Q164 Which type of pathogen causes

the waterborne disease E coli Infection

ककसपरकारकारह८गजननकजिजननतरह८गईकह८िाईसिमणकाकारणबनताहहॴ 27-Jan-2017

Options

1) Protozoan

परह८टह८जआ

2) Parasitic

परजीवी 3) Bacterial

बहॴकटीररयि

4)Viral

वायरि

Correct Answer Bacterial

Q165 The amount of blood filtered

together by both the kidneys in a 70 kg

adult male human in a minute is

70 की गरा वािहॳएकवयसकप षमएकलमनटमदह८नोगदकहॳदवाराएकसािचाबनीगयीरकतकीमातरहह८तीहहॴ 29-Jan-2017

Options

1) 1100 ml

1100 लमलि

2) 100 ml

F A C E B O O K

P A G E h t t p w w w f a c e b o o k c o m s s c m e n t o r s o f f i c i a l P a g e | 39

FOR MORE UPDATES AND MORE MATERIAL DO LIKE OUR FACEBOOK PAGE httpwwwfacebookcomsscmentorsofficial

100 लमलि

3) 1500 ml

1500 लमलि

4) 500 ml

500 लमलि

Correct Answer 1100 ml

Q166 Which feature of a plant helps to

distinguish a monocot from a dicot

पह९धहॳकीवहकह९नसीपविहॳषताहहॴजह८एकदपवदलियहॳऔरएकएकदिीयपह९धहॳसहॳभहॳदकरनहॳममददकरतीहहॴ 29-Jan-2017

Options

1) Pollination

परागम

2) Venation

वहॳनहॳिन

3) Vernation

वनिन

4) Aestivation

एसटीवहॳिहॳन

Correct Answer venation

Q167 The Mutation Theory was

proposed by

उतवररवतयनकालसदात mdashmdashndash

कहॳ दवरापरसतापवतककयाजाताहहॴ 29-Jan-2017

Options

1) Charles Lyell

चामसयलियहॳि

2) William Smith

पवलियमनसमि

3) Hugo De Vries

हयगह८िीराईस

4)Harrison Schmitt

हहॳरीसननसमट

Correct Answer Hugo De Vries

Q168 Which type of pathogen causes

the waterborne disease HepatitisA

ककसपरकारकहॳ रह८गजनकजिजननतरह८गहहॳपहॳटाइटटस-A काकारणबनताहहॴ

29-Jan-2017

Options

1) Parasitic

परजीवी 2) Viral

वायरि

3) Protozoan

परह८टह८जआ

4) Bacterial

बहॴकटीररयि

Correct Answer Viral

Q169 In a Punnett Square with the

cross AaBb x Aabb how many AaBb

genotypes would be created

पनहॳटसकवायरमिह८स AaBb x Aabb

कहॳ सािककतनहॳ AaBb जीनह८टाइपबनगहॳ 29-Jan-

2017

Options

1) 4

2) 1

3) 7

4) 6

Correct Answer 4

Q170 Arboreal Ateles is the scientific

name of

अिह८ररयिएटटलिस mdashmdashmdash कावहॴजञाननकनामहहॴ 29-Jan-2017

Options

1) Squirrel

चगिहरी 2) Sparrow

गह८रहॴया 3) Lizard

नछपकिी 4) Spider monkey

F A C E B O O K

P A G E h t t p w w w f a c e b o o k c o m s s c m e n t o r s o f f i c i a l P a g e | 40

FOR MORE UPDATES AND MORE MATERIAL DO LIKE OUR FACEBOOK PAGE httpwwwfacebookcomsscmentorsofficial

मकड़ीबदर

Correct Answer Spider monkey

Q171 Which type of pathogen causes

the waterborne disease Salmonellosis

ककसपरकारकारह८गाणजिजननतबीमारीसािमह८नहॳिह८लसज़काकारकहहॴ

29-Jan-2017

Options

1) Algal

िहॳवालियहॳ 2) Parasitic

परजीवी 3) Bacterial

बहॴकटीररयि

4)Viral

वायरि

Correct Answer Bacterial

An infection with salmonella bacteria

commonly caused by contaminated food

or water

Symptoms include diarrhoea fever

chills and abdominal pain

Q172 is a condition in which there is a

deficiency of red cells or of haemoglobin

in the blood

mdashmdash-

एकनसिनतहहॴनजसमहॳरकतमिािकह८लिकाओकीयाहीमह८गिह८बबनकीकमीहह८तीहहॴ 29-Jan-2017

Options

1) Albinism

एनमबननजम

2) Propyria

परह८पीररया 3) Anaemia

एनीलमया 4)Keloid disorder

कहॳ िह८इिडिसओिर

Correct Answer Anaemia

Q173 Ananas comosus is the scientific

name of

Options

अनानासकह८मह८सस mdashmdashmdashndash

कावहॴजञाननकनामहहॴ 29-Jan-2017

1) Custard Apple

सीताफि

2) Pineapple

पाइनएपपि

3) Bamboo

बास

4)Pomegranate

अनार

Correct Answer Pineapple

Q174 Which organ produces insulin

कह९नसाअगइनसलिनपहॴदाकरताहहॴ 29-Jan-

2017

Options

1) Liver

यकत

2) Thyroid gland

िायराइिगरिी 3) Spleen

पिीहा 4)Pancreas

अगरयिय

Correct Answer Pancreas

Q175 Which of the following disease is

not caused by water pollution

नननननलिखितमसहॳकह९नसारह८गपानीकहॳ परदषणकहॳकारणनहीहह८ता

29-Jan-2017

Options

1) Cholera

हहॴजा 2) Typhoid

F A C E B O O K

P A G E h t t p w w w f a c e b o o k c o m s s c m e n t o r s o f f i c i a l P a g e | 41

FOR MORE UPDATES AND MORE MATERIAL DO LIKE OUR FACEBOOK PAGE httpwwwfacebookcomsscmentorsofficial

टाइफाइि

3) Asthma

दमा 4)Diarrhoea

दसत

Correct Answer Asthma

Q176 Ocimum tenuiflorum is the

scientific name of

ओलिलममटहॳयईफिह८रमइसकावहॴजञाननकनाम mdash

ndash हहॴ 30-Jan-2017

Options

1) Neem

नीम

2) Mango

आम

3) Babul

बबि

4)Tulsi

तिसी Correct Answer Tulsi

Q177 Which gland secretes bile a

digestive fluid

कह९नसीगरिीपपतत एकपाचनतरिपरदािय सरापवतकरतीहहॴ 30-Jan-2017

Options

1) Pancreas

अगनयािय

2) Liver

यकत

3) Thyroid

िायराइि

4) Testes

टहॳनसटस

Correct Answer liver

Q178 In which of the following the

dominant phase is Gametophyte

नननननलिखितमसहॳककसकहॳ परमिचरणयगमकह८दपवधद (Gametophyte)हहॴ 30-Jan-2017

Options

1) Bryophyta

िायह८फाइटा 2) Pteridophyta

टहॳररिह८फाइटा 3) Gymnosperms

नजननह८सपमय 4) Angiosperms

एननजयह८सपमय Correct Answer Bryophyta

Q179 Anaerobic respiration refers to

which of the following

नननननलिखितमसहॳककसहॳअवायवीयशवसनकहाजाताहहॴ

30-Jan-2017

Options

1) Respiration without Oxygen

ऑकसीजनकहॳ बबनाशवसन

2) Respiration with Oxygen

ऑकसीजनकहॳ सािशवसन

3) Respiration without CO2

काबयनिायऑकसाइिकहॳ बबनाशवसन

4) Respiration with CO2

काबयनिायऑकसाइिकहॳ सािशविन

Correct Answer Respiration without

Oxygen

Q180 Which type of pathogen causes

the waterborne disease Cholera

ककसपरकारकारह८गजनकजिजननतरह८गहहॴजाकाकारणबनताहहॴ

30-Jan-2017

Options

1) Algal

िहॴवालियहॳ

F A C E B O O K

P A G E h t t p w w w f a c e b o o k c o m s s c m e n t o r s o f f i c i a l P a g e | 42

FOR MORE UPDATES AND MORE MATERIAL DO LIKE OUR FACEBOOK PAGE httpwwwfacebookcomsscmentorsofficial

2) Bacterial

बहॴकटीररयि

3) Protozoan

परह८टह८जआ

4) Viral

वायरि

Correct Answer Bacterial

Q181 To which class does

Oxyreductases transferases hydrolases

belong

ओकसीररिकटहॳसटरासफरहॳजहॳस

हाइडरह८िहॳसहॳसककसवगयमआतहॳहहॴ 30-Jan-2017

Options

1) Hormones

हारमोस

2) Enzymes

एजाइनस

3) Proteins

परह८टीनस

4) Vitamins

पवटालमनस

Correct Answer Enzymes

Q182 Which of the following is not true

about Gymnosperms

ननननमसहॳकह९नसीबातअनावतबीजीकहॳ बारहॳमसचनहीहहॴ 30-Jan-2017

Options

1) Dominant phase is saprophytes

परमिचरणसहॳपरह८फाइटसहह८ताहहॴ 2) Vascular bundles are absent

सवहनीबििअनपनसितहह८ताहहॴ 3) spores are heterospores

बीजाणहहॳटहॳरह८सपह८रसहह८तहॳहहॴ 4) Flowers are absent

फिअनपनसितहह८तहॳहहॴ

Correct Answer Vascular bundles are

absent

Q183 The name of first mammal clone sheep is

भहॳड़कीपरिमसतनपायीपरनत प (किह८न)

कानामहहॴ 30-Jan-2017

Options

1) Noori

नरी 2) Dolly

िॉिी 3) Louise

िसी 4)Durga

दगाय Correct Answer Dolly

Q184 Which type of pathogen causes

the water-borne disease Typhoid fever

ककसपरकारकारह८गजनकजिजननतरह८गटाइफाइिबिारकाकारणबनताहहॴ 30-Jan-2017

Options

1) Algal

िहॴवािीय

2) Parasitic

परजीवी 3) Protozoan

परह८टह८जनअन

4)Bacterial

बहॴकटीररयि

Correct Answer Bacterial

Q185 In which part of the cell are

proteins made

कह८लिकाकहॳ ककसटहससहॳमपरह८टीनबनायाजाताहहॴ

31-Jan-2017

Options

1) Reticulum

रहॳटटकिम

F A C E B O O K

P A G E h t t p w w w f a c e b o o k c o m s s c m e n t o r s o f f i c i a l P a g e | 43

FOR MORE UPDATES AND MORE MATERIAL DO LIKE OUR FACEBOOK PAGE httpwwwfacebookcomsscmentorsofficial

2) Golgi apparatus

गह८मजीएपहॳरहॳटस

3) Ribosomes

ररबह८सह८नस

4) Lysosome

िायसह८सह८नस

Correct Answer ribosomes

Proteins are produced by stringing

amino acids together in the order

specified by messenger RNA strands

that were transcribed from DNA in the

cell nucleus The process of synthesizing

a protein is called translation and it

occurs on ribosomes in the cytoplasm of

a cell

Q186 Polio is a disease caused by which

of the following

नननननलिखितमसहॳपह८लियह८कीबबमारह८हह८नहॳकाकारणकयाहहॴ

31-Jan-2017

Options

1) Bacteria

बहॴकटीररयि

2) Mosquito

मचछर

3) Virus

वायरस

4) Cockroach

नतिच हॳ Correct Answer Virus

Polio or poliomyelitis is a crippling and

potentially deadly infectious disease It

is caused by the poliovirus

Q187 ndash Hay fever is a sign of which of

the following

हहॳकफवरनननननलिखितमसहॳककसकाएकसकहॳ तहहॴ

31-Jan-2017

Options

1) Old Age

वदावसिा 2) Malnutrition

कपह८सण

3) Allergy

एिनजय 4) Over Work

अतयचधककाययकरना Correct Answer Allergy

Q188 How many chromosomes does a

human cell contain

एकमानवकह८लिकामककतनहॳगणसतरहह८तहॳहहॴ

29-Jan-2017

Options

1) 6

2) 26

3) 46

4) 66

Correct Answer 46

In humans each cell normally contains

23 pairs of chromosomes for a total of

46 Twenty-two of these pairs called

autosomes look the same in both males

and females The 23rd pair the sex

chromosomes differ between males and

females

Q189 Which of the following is not true

about Bryophyta

ननननमसहॳकह९नसीबातिायह८फाइटकहॳ बारहॳमसचनहीहहॴ 31-Jan-2017

Options

1) Dominant phase is gametophytes

परमिचरणगहॳलमतह८फाइटसहह८ताहहॴ 2) Main plant body is haploid

पह९धहॳकामखयिरीरअगखणतहह८ताहहॴ 3) Spores are homospores

बीजाणहह८मह८सफह८रसहह८तहॳहहॴ 4) Flowers are present

फिमह८जदहह८तहॳहहॴ Correct Answer Flowers are present

F A C E B O O K

P A G E h t t p w w w f a c e b o o k c o m s s c m e n t o r s o f f i c i a l P a g e | 44

FOR MORE UPDATES AND MORE MATERIAL DO LIKE OUR FACEBOOK PAGE httpwwwfacebookcomsscmentorsofficial

Q190 Which aquatic animal has

trailing tentacles

ककसजिीयजानवरकहॳ पीछहॳचिनहॳवािहॳटहॳटकिसहह८तहॳहहॴ

31-Jan-2017

Options

1) Sea horse

समदरीघह८िा 2) Corals

मगा 3) Jelly fish

जहॳिीमछिी 4) Star fish

तारामछिी Correct Answer Jelly fish

Jellyfish with its umbrella-shaped bell

and trailing tentacles

Q191 Which type of pathogen causes

the water-borne disease Poliomyelitis

(Polio)

ककसपरकारकारह८गजनकजिजननतरह८गपह८लियह८मायहॳटटस (पह८लियह८) काकारणहहॴ 31-Jan-

2017

Options

1) Parasitic

परजीवी 2) Algal

िहॴवालिय

3) Viral

वायरि

4) Bacterial

बहॴकटीररयि

Correct Answer Viral

Q192 The outer white part of the eye

that protects the inner structures is

आािकाबाहरीसफहॳ दटहससाजह८आतररकसरचनाओकीरकषाकरताहहॴ वह mdashmdashmdash हहॴ 31-Jan-

2017

Options

1) Iris

आयररस

2) Sclera

सकिहॳरा 3) Retina

रहॳटटना 4) Cornea

कह८ननयया Correct Answer Sclera

Q193 Proteins are made up of

परह८टीनकाननमायण mdashndash सहॳहह८ताहहॴ 31-Jan-2017

Options

1) Amino acids

एलमनह८अनि

2) Fatty acids

वसायकतअनि

3) Glucose

गिकह८ज

4)Nucleotides

नयनकियह८टाईिस

Correct Answer Amino acids

Q194 Moringa Oleifera is the scientific

name of

मह८ररगओलिफहॳ रा mdashmdashndash कावहॴजञाननकनामहहॴ 31-Jan-2017

Options

1) Banyan

बरगद

2) Gulmohar

गिमह८हर

3) Amla

आमिा

F A C E B O O K

P A G E h t t p w w w f a c e b o o k c o m s s c m e n t o r s o f f i c i a l P a g e | 45

FOR MORE UPDATES AND MORE MATERIAL DO LIKE OUR FACEBOOK PAGE httpwwwfacebookcomsscmentorsofficial

4) Drumstick

डरमनसटक

Correct Answer Drumstick

Q195 Kidney stones are composed of

गदकीपिरी mdashndash सहॳबनीहह८तीहहॴ 1-Feb-2017

Options

1) Calcium Oxalate

कहॴ नमसयमओकजहॳिहॳट

2) Sodium Chloride

सह८डियमकिह८राइि

3) Magnesium Nitrate

महॳनगनलियमनाइतटरहॳट

4) Calcium Bicarbonate

कहॴ नमियमबायकबोनहॳट

Correct Answer Calcium Oxalate

Q196 ndash Which of the following is not

true about Angiosperms

ननननमसहॳकह९नसीबातआवतबीजीकहॳ बारहॳमसचनहीहहॴ 1-Feb-2017

Options

1) Dominant phase is gametophytes

परमिचरणगहॳलमतह८फाइटहह८ताहहॴ 2) Vascular bundles are present

सवहनीबििमह९जदहह८ताहहॴ 3) Spores are heterospores

बीजाणहहॳटहॳरह८सपह८रसहह८तहॳहहॴ 4) Seeds are covered

बीजढकहॳ हह८तहॳहहॴ Correct Answer Dominant phase is

gametophytes

Q197 All of the following are excretory

(waste) products of animals except

नननननलिखितमसहॳककसएककह८छह८ड़करअनयसभीपराखणयोदवाराउतसनजयतपदाियहहॴ 1-Feb-

2017

Options

1) Uric Acid

यररकएलसि

2) Ammonia

अमह८ननया 3) Carbohydrates

काबोहाइडरहॳट

4) Urea

यररया Correct Answer Carbohydrates

In animals the main excretory products

are carbon dioxide ammonia (in

ammoniotelics) urea (in ureotelics) uric

acid (in uricotelics) guanine (in

Arachnida) and creatine

Q198 RNA is a polymeric molecule

What does RNA stand for

आरएनइएएकबहिकआणहहॴ इसकाकापवय पकयाहहॴ 1-Feb-2017

Options

1) Rado Nuclear Acid

रािह८नयनकियरएलसि

2) Ribo Nucleic Acid

राइबह८नयनकिकएलसि

3) Rhino Nuclear Acid

हाइनह८नयनकियरएलसि

4) Resto Nucleus Acid

रहॳसटह८नयकिीयसएलसि

Correct Answer Ribo Nucleic Acid

Q199 Which organ does detoxification

and produces chemicals needed for

digestion

कह९नसाअगपवषहरणकरताहहॴऔरपाचनकहॳ लिएआवशयकरसायनोकह८पहॴदाकरताहहॴ 1-Feb-

2017

Options

1) Salivary glands

िारगरचिया 2) Pancreas

अगनयािय

F A C E B O O K

P A G E h t t p w w w f a c e b o o k c o m s s c m e n t o r s o f f i c i a l P a g e | 46

FOR MORE UPDATES AND MORE MATERIAL DO LIKE OUR FACEBOOK PAGE httpwwwfacebookcomsscmentorsofficial

3) Thyroid gland

िायराइिगरिी 4) Liver

यकत

Correct Answer Liver

Q200 Psidium guajava is the scientific

name of

लसडियमगआजावा mdashmdash कावहॴजञाननकनामहहॴ 1-

Feb-2017

Options

1) Guava

अम द

2) Mango

आम

3) Bamboo

बास

4) Jack fruit

कटहि

Correct Answer Guava

Q201 Which drug is used as a Blood

Thinner

चधरकह८पतिाकरनहॳकहॳ पमककसदवाकापरयह८गककयाजाताहहॴ

1-Feb-2017

Options

1) Warfarin

वाफर न

2) Tramadol

टरहॳमािह८ि

3) Azithromycin

एनजरह८मायलसन

4) Hydralazine

हाइडरह८िहॳनजन

Correct Answer Warfarin

Q202 Which of the following disease is

caused due to the deficiency of protein

परह८टीनकीकमीकहॳ कारणनननननलिखितमसहॳकह९नसारह८गहह८ताहहॴ 1-Feb-2017

Options

1) Arthritis

गटठया 2) Kwashiorkor

कािीओकय र

3) Goitre

गाइटर

4) Night Blindness

रतह९चध

Correct Answer Kwashiorkor

Q203 A is species of plant that has

adapted to survive in an environment

with little liquid water

mdashmdashndashपह९धहॳकीएकऐसहॳऐसहॳपरजानतहहॴ नजसनहॳकमपानीवािहॳवातावरणमजीपवतरहनहॳकहॳलिएअनकिनहहॴ 1-Feb-2017

Options

1) Xerophyte

म दपवद

2) Hydrophyte

जिीयपादप

3) Mesophyte

समह८दपवद

4) Thallophyte

िहॴिह८फाइटा Correct Answer xerophyte

xerophyte is a species of plant that has

adapted to survive in an environment

with little liquid water such as a desert

or an ice- or snow-covered region in the

Alps or the Arctic

Mesophytes are terrestrial plants which

are adapted to neither a particularly

dry nor particularly wet environment

An example of a mesophytic habitat

would be a rural temperate meadow

F A C E B O O K

P A G E h t t p w w w f a c e b o o k c o m s s c m e n t o r s o f f i c i a l P a g e | 47

FOR MORE UPDATES AND MORE MATERIAL DO LIKE OUR FACEBOOK PAGE httpwwwfacebookcomsscmentorsofficial

which might contain goldenrod clover

oxeye daisy and Rosa multiflora

thallophyte any of a group of plants or

plantlike organisms (such as algae and

fungi) that lack differentiated stems

leaves and roots and that were formerly

classified as a primary division

(Thallophyta) of the plant kingdom

Q204 How many types of teeth are

there in humans

मनषयोमककतनहॳपरकारकहॳ दातहह८तहॳहहॴ

1-Feb-2017

Options

1) 4

2) 5

3) 2

4) 3

Correct Answer 4

teeth -Humans have four types of

teethincisors canines premolars and

molars each with a specific function

The incisors cut the food the canines

tear the food and the molars and

premolars crush the food

Q205 Carica papaya is the scientific name of

कहॴ ररकापपाया mdashmdashndash कावहॴजञाननकनामहहॴ 2-

Feb-2017

Options

1) Peepal

पीपि

2) Papaya

पपीता 3) Tamarind

इमिी 4) Drumstick

ढह८िकाछड़ी Correct Answer Papaya

Q206 Muscles get tired when there is

shortfall of

जब mdashndash कीकमीहह८तीहहॴतबपहॳिीयिकजातीहहॴ 2-Feb-2017

Options

1) Lactic acid

िहॴनकटकएलसि

2) Na+ ions

Na+ आयन

3) ATP

एटीपी 4) Sulphates

समफहॳ टस

Correct Answer ATP

ATP is the energy source muscle fibers

use to make muscles contract

muscle tissuersquos main source of energy

called adenosine triphosphate or ATP

As your muscles use up this energy

source they become tired and fatigued

Oxygen is the key ingredient that helps

create new ATP to replenish the burned

up ATP in your muscles

Q207 Artocarpus integra is the

scientific name of आटह८कापयसइटीगरा mdashmdashmdash कावहॴजञाननकनामहहॴ 2-Feb-2017

Options

1) Guava

अम द

2) Pineapple

अनानास

3) Silver Oak

लसमवरओक

4) Jack fruit

कटहि

Correct Answer Jack fruit

Q208 Which organ stores fat soluble

vitamins

कह९नसाअगवसामघिनिीिपवटालमनह८काभिाराकरताहहॴ

2-Feb-2017

F A C E B O O K

P A G E h t t p w w w f a c e b o o k c o m s s c m e n t o r s o f f i c i a l P a g e | 48

FOR MORE UPDATES AND MORE MATERIAL DO LIKE OUR FACEBOOK PAGE httpwwwfacebookcomsscmentorsofficial

Options

1) Blood

रकत

2) Skin

तवचा 3) Liver

यकत

4) Pancreas

अगनयािय

Correct Answer Liver

Q209 Which disease is caused due to

deficiency of Iodine

आयह८िीनकहॳ कारणकह९नसारह८गहह८ताहहॴ 2-Feb-2017

Options

1) Rickets

ररकहॳ टस

2) Scurvy

सकवी 3) Goitre

गणमािा 4) Growth retardation

पवकासका कना Correct Answer Goitre

rickets A softening and weakening of

bones in children usually due to

inadequate vitamin D

Q210 Grevillea Robusta is the scientific name of

गरहॳपवलियारह८बसटा mdashmdashmdash- कापवजञाननकनामहहॴ 2-Feb-2017

Options

1) Peepal

पीपि

2) Teak

सागह९न

3) Silver Oak

लसमवरओक

4) Jack fruit

कटहि

Correct Answer Silver Oak

Q211 When a Cuttlefish is described as a Molluscs it is at which level of

classification

जबएककटिकफिकह८एकमह८िसकाकहॳ पमवखणयतककयाजाताहहॴतबयहॳवगीकरणकहॳ ककससतरपहॳनसितहहॴ 2-Feb-2017

Options

1) Class

वगय 2) Order

िम

3) Family

पररवार

4) Phylum

सघ

Correct Answer Phylum

Q212 Bambusa dendrocalmus is the

scientific name of बानबसािहॳडराकामस mdashmdashmdash कावहॴजञाननकनामहहॴ 3-Feb-2017

Options

1) Banyan

बरगद

2) Papaya

पपीता 3) Bamboo

बास

4) Pomegranate

अनार

Correct Answer Bamboo

Q213 Acinonyx Jubatus is the scientific name of

एलसनह८ननकसजयबहॳटस mdashmdashmdash

कावहॴजञाननकनामहहॴ 3-Feb-2017

F A C E B O O K

P A G E h t t p w w w f a c e b o o k c o m s s c m e n t o r s o f f i c i a l P a g e | 49

FOR MORE UPDATES AND MORE MATERIAL DO LIKE OUR FACEBOOK PAGE httpwwwfacebookcomsscmentorsofficial

Options

1) Bear

भाि 2) Horse

घह८िा 3) Cheetah

चीता 4) Zebra

जहॳिा Correct Answer Cheetah

Q214 The pale yellow colour of urine is

due to the presence of which pigment

मतरकाफीकापीिारगरगदरयकहॳ उपनसिनतकहॳ कारणहह८ताहहॴ

3-Feb-2017

Options

1) Urochrome

यरह८िह८म

2) Urophyll

यरह८कफि

3) Chlorophyll

किह८रह८कफि

4) Chloroplast

किह८रह८पिासट

Correct Answer Urochrome

Q215 Which of the following constitute

to form a gene

नननननलिखितमसहॳकह९नसीचीज़एकजीनकागठनकरतीहहॴ

3-Feb-2017

Options

1) Polynucleotides

पह८िीनयनकियह८टाईडस

2) Hydrocarbons

हाइडरह८काबोस

3) Lipoproteins

िाईपह८परह८टीनस

4) Lipids

लिपपडस

Correct Answer Polynucleotides

Polynucleotide molecule is a biopolymer

composed of 13 or more nucleotide

monomers covalently bonded in a chain

DNA (deoxyribonucleic acid) and RNA

(ribonucleic acid) are examples of

polynucleotides with distinct biological

function

Q216 Vertebrates belongs to the

phylum

रीढ़कीहडिीवािहॳपराणी mdashmdashmdash

परजानतकहॳ अतगायतआतहॳहहॴ 3-Feb-2017

Options

1) Arthropoda

आरह८पह८ड़ा 2) Annelida

एननलििा 3) Cnidaria

ननिहॳररया 4) Chordata

कह८िटा Correct Answer Chordata

Q217 Punica granatum is the scientific name of

पननकगरहॳनहॳटस mdashmdashmdash कावहॴजञाननकनामहहॴ 3-Feb-2017

Options

1) Custard Apple

सीताफि

2) Gulmohar

गिमह८हर

3) Silver Oak

लसमवरओक

4) Pomegranate

अनार

Correct Answer Pomegranate

F A C E B O O K

P A G E h t t p w w w f a c e b o o k c o m s s c m e n t o r s o f f i c i a l P a g e | 50

FOR MORE UPDATES AND MORE MATERIAL DO LIKE OUR FACEBOOK PAGE httpwwwfacebookcomsscmentorsofficial

Q218 Between a tiger and an monkey

which of the following is different

एकबाघऔरबदरकहॳ बीचनननननलिखितमसहॳकह९नसीबातअिगहहॴ 3-Feb-2017

Options

1) Kingdom

राजय

2) Phylum

जानत

3) Order

िम

4) Class

वगय Correct Answer order

Q219 The artificial heart was invented by

कबतरमहदयका mdashmdashmdash

दवाराअपवषकारककयागयािा 3-Feb-2017

Options

1) Muhammad Yunus

महनमदयनस

2) Linus Yale Jr

िाइनसयहॳिजय

3) Gazi Yasargil

गाजीयासचगयि

4) Paul Winchell

पह९िपवमकि Correct Answer Paul Winchell

Q220 Tamarindus indica is the

scientific name of

टहॳमररनडसइडिका mdashmdash कावहॴजञाननकनामहहॴ 7-

Feb-2017

Options

1) Neem

नीम

2) Pineapple

अनानास

3) Tamarind

इमिी 4)Chiku

चीक

Correct Answer Tamarind

Q221 In eukaryotic cells synthesis of

RNA takes place in the

यकहॳ योटटककह८लिकाओमआरएनएकासशिहॳषण

mdashndash महह८ताहहॴ 7-Feb-2017

Options

1) Mitochondria

माईटह८कोडडरया 2) Centrioles

सटरीयह८मस

3) Ribosomes

ररबह८सह८नस

4) Nucleus

नयनकियस

Correct Answer nucleus

eukaryotic cell -Transcription is the

process of synthesizing ribonucleic acid

(RNA)Synthesis takes place within the

nucleus of eukaryotic cells or in the

cytoplasm of prokaryotes and converts

the genetic code from a gene in

deoxyribonucleic acid ( DNA ) to a

strand of RNA that then directs

proteinsynthesis

Q222 _________is caused by parasites

of the Plasmodium genus

पिाजमह८डियमजातीकहॳ परजीवी mdash- कहॳ कारणहहॴ 7-Feb-2017

Options

1) Dysentery

पहॳचचि

2) Malaria

मिहॳररया 3) Chickenpox

F A C E B O O K

P A G E h t t p w w w f a c e b o o k c o m s s c m e n t o r s o f f i c i a l P a g e | 51

FOR MORE UPDATES AND MORE MATERIAL DO LIKE OUR FACEBOOK PAGE httpwwwfacebookcomsscmentorsofficial

चहॳचक

4) Herpes

हहॳपपयस

Correct Answer Malaria

Q223 Carotene in fruits and vegetables

gives it which color

फिह८औरसनलजयोमनसितकहॳ रह८टीनउनहकह९नसारगपरदानकरताहहॴ 7-Feb-2017

Options

1) Green

हरा 2) Pink

गिाबी 3) Orange

नारगी 4) Blue

नीिा Correct Answer Orange

Q224 Equus Caballus is the scientific

name of

एकवसकहॴ बहॳिस mdashmdashndash कापवजञाननकनामहहॴ 7-Feb-2017

Options

1) Horse

घह८िा 2) Zebra

ज़हॳिा 3) Donkey

गधा 4) Buffalo

भस

Correct Answer Horse

Q225 Elapidae Naja is the scientific name of

एिीपीिीनाजा mdashmdash- कावहॴजञाननकनामहहॴ 8-Feb-2017

Options

1) Cobra

कह८बरा 2) Elephant

हािी 3) Eagle

ग ि

4) Owl

उमि Correct Answer Cobra

Q226 Which disease is caused due to

deficiency of Iron

िह८हकीकमीकहॳ कारणकह९नसारह८गहह८ताहहॴ 8-Feb-

2017

Options

1) Beriberi

बहॳरीबहॳरी 2) Tetany

टहॳटनी 3) Kwashiorkor

कवािीऔरकर

4) Anaemia

रकतामपता Correct Answer Anaemia

Beriberi is a disease caused by a vitamin

B-1 deficiency also known as thiamine

deficiency

Tetany can be the result of an

electrolyte imbalance Most often itrsquos a

dramatically low calcium level also

known as hypocalcemia Tetany can also

be caused by magnesium deficiency or

too little potassium Having too much

acid (acidosis) or too much alkali

(alkalosis) in the body can also result in

tetany

Kwashiorkor also known as

ldquoedematous malnutrition It is a form of

malnutrition caused by a lack of protein

in the diet

Anaemia means that you have fewer red

blood cells than normal or you have less

F A C E B O O K

P A G E h t t p w w w f a c e b o o k c o m s s c m e n t o r s o f f i c i a l P a g e | 52

FOR MORE UPDATES AND MORE MATERIAL DO LIKE OUR FACEBOOK PAGE httpwwwfacebookcomsscmentorsofficial

haemoglobin than normal in each red

blood cell

Q227 is a leaf where the leaflets are

arranged along the middle vein

mdashndashएकपततीहहॴजहापतरकह८कीरचनाक ररयालिराकहॳ आसपासहह८तीहहॴ 8-Feb-2017

Options

1) Pinnately compound leaf

पपनहॳटिीसयकतपतती 2) Palmately compound leaf

पामहॳटिीसयकतपतती 3) Compound leaf

सयकतपतती 4) Simple leaf

साधारणपतती Correct Answer Pinnately compound

leaf

Q228 Haustoria or sucking roots are

found in which of the following

हह८सटह८ररयायाचसनहॳवािीजड़हॳनननननलिखितमसहॳककसमपाईजातीहहॴ 8-Feb-2017

Options

1) Wheat

गहॳह

2) Mango

आम

3) Chestnut

चहॳसटनट

4) Cuscuta

कसकयटा Correct Answer Cuscuta

Haustorial roots -The roots of parasitic

plants which penetrate into the host

tissues to absorb nourishment are

called haustorial roots hellip Also known as suckingor parasitic roots

Q229 Equs Asinus is the scientific name

of

एकवसएलसनस mdashmdashndash कावहॴजञाननकनामहहॴ 8-

Feb-2017

Options

1) Donkey

गधा 2) Cow

गाय

3) Deer

टहरन

4) Kangaroo

कगा

Correct Answer Donkey

Q230 Ficus benghalensis is the scientific name of

फाईकसबहॳनगहॳिहॳलसस mdashndash कापवजञाननकनामहहॴ 8-Feb-2017

Options

1) Banyan

बरगद

2) Pineapple

अनानास

3) Babul

बबि

4) Tulsi

तिसी Correct Answer Banyan

Q231 Equus burchellii is the scientific name of

एकवसबचिी mdashmdash- कापवजञाननकनामहहॴ 8-Feb-2017

Options

1) Horse

घह८िा 2) Zebra

जहॳिा 3) Buffalo

F A C E B O O K

P A G E h t t p w w w f a c e b o o k c o m s s c m e n t o r s o f f i c i a l P a g e | 53

FOR MORE UPDATES AND MORE MATERIAL DO LIKE OUR FACEBOOK PAGE httpwwwfacebookcomsscmentorsofficial

भस

4) Ass

गधा Correct Answer Zebra

Page 34: COMPILATION OF ALL 72 SETS OF BIOLOGY SSC CHSL-2016 · OF BIOLOGY SSC CHSL-2016 PREPARED BY : SSC MENTORS BIOLOGY SPECIAL . F A C E B O O K P A G E : h t t p : / / w w w . f a c e

F A C E B O O K

P A G E h t t p w w w f a c e b o o k c o m s s c m e n t o r s o f f i c i a l P a g e | 33

FOR MORE UPDATES AND MORE MATERIAL DO LIKE OUR FACEBOOK PAGE httpwwwfacebookcomsscmentorsofficial

रह८िहॳलियानयररिी mdashmdash- कावहॴजञाननकनामहहॴ 24-

Jan-2017

Options

1) Mouse

चहा 2) Squirrel

चगिहरी 3) Monkey

बदर

4) Lizard

नछपकिी Correct Answer Mouse

Q139 Name the scientist who proposed

the cell theory

कह८लिकालसदातकापरसतावदहॳनहॳवािहॳवहॴजञाननककानामबताइए 24-Jan-2017

Options

1) Schleiden and Schwann

िीमिनऔरशरववान

2) Lamarck

िहॳमाकय 3) Treviranus

टरहॳवायरहॳनस

4)Whittaker and Stanley

हीटकरऔरसटहॳनिहॳ Correct Answer Schleiden and

Schwann

Q140 The flower with the worldrsquos

largest bloom is

दननयाकासबसहॳबड़ाफिखििनहॳवािा mdashmdashndash हहॴ 24-Jan-2017

Options

1) Pando

पािह८ 2) Posidonia

पह८सीिह८ननया 3) Rafflesia arnoldii

ररफिहॳलियाअनोमिी 4)Helianthus annuus

हहॳलिएनिसएनयअस

Correct Answer Rafflesia arnoldii

Rafflesia arnoldii is a species of

flowering plant in the parasitic genus

Rafflesia It is noted for producing the

largest individual flower on earth It has

a very strong and horrible odour of

decaying flesh earning it the nickname

ldquocorpse flower

Q141 Deficiency of which vitamin

causes night blindness

ककसपवटालमनकीकमीकहॳ कारणरतौधीहह८ताहहॴ 24-Jan-2017

Options

1) Vitamin K

पवटालमन K

2) Vitamin C

पवटालमन C

3) Vitamin B1

पवटालमन B1

4)Vitamin A

पवटालमन A

Correct Answer Vitamin A

Q142 Nongreen plants lack which of the

following

गहॴर-

हररतवनसपनतमनननननलिखितमसहॳककसकीकमीहह८तीहहॴ

24-Jan-2017

Options

1) Chlorophyll

किह८रह८कफि

2) Lycophyll

िायकह८कफि

3) Cyanophyll

F A C E B O O K

P A G E h t t p w w w f a c e b o o k c o m s s c m e n t o r s o f f i c i a l P a g e | 34

FOR MORE UPDATES AND MORE MATERIAL DO LIKE OUR FACEBOOK PAGE httpwwwfacebookcomsscmentorsofficial

सायनह८कफि

4)Phototropism

फह८टह८टरोपपजम

Correct Answer Chlorophyll

Q143 Organisms that use light to

prepare food are known as

जह८जीवपरकािकाउपयह८गकरभह८जनतहॴयारकरतहॳहहॴ उनह mdashmdash- कहॳ पमजानजाताहहॴ 24-Jan-2017

Options

1) Autotrophs

सवपह८षी 2) Heterotrophs

पवषमपह८षज

3) Omnivores

सवायहारी 4)Decomposers

पवघटनकरनहॳवािा Correct Answer Autotrophs

autotrophs often make their own food

by using sunlight carbon dioxide and

water to form sugars which they can use

for energy Some examples of

autotrophs include plants algae and

even some bacteria Autotrophs

(producer) are important because they

are a food source for heterotrophs

(consumers)

A heterotroph is an organism that

ingests or absorbs organic carbon

(rather than fix carbon from inorganic

sources such as carbon dioxide) in order

to be able to produce energy and

synthesize compounds to maintain its

life Ninety-five percent or more of all

types of living organisms are

heterotrophic including all animals and

fungi and some bacteria

Q144 Which of the following is a

primary function of haemoglobin

नननननलिखितमसहॳकह९नसाटहमह८गिह८बबनकाएकपरािलमककाययहहॴ

25-Jan-2017

Options

1) Utilization of energy

उजायकाउपयह८गकरना 2) Prevention of anaemia

रकतामपताहह८नहॳसहॳरह८कना 3) Destruction of bacteria

बहॴकटीररयाकापवनािकरना 4) To transport oxygen

ऑकसीजनकावहनकरना Correct Answer To transport oxygen

Q145 Vascular bundles are absent in

सवहनीबिि mdashmdash- मअनपनसतिरहतहॳहहॴ 25-Jan-2017

Options

1) Bryophyta

िायह८फाइटा 2) Pteridophyta

टहॳररिह८फाईटा 3) Gymnosperms

नजननह८सपमय 4) Angiosperms

एननजयह८सपहॳनसय Correct Answer Bryophyta

Q146 Sauria Lacertidae is the scientific

name of

सहॴररयािहॳसरटाईिी mdashmdashndash कावहॴजञाननकनामहहॴ 25-Jan-2017

Options

1) Crocodile

मगरमचछ

2) Hippopotamus

टहपपह८पह८टहॳमस

3) Lizard

नछपकिी 4) House fly

F A C E B O O K

P A G E h t t p w w w f a c e b o o k c o m s s c m e n t o r s o f f i c i a l P a g e | 35

FOR MORE UPDATES AND MORE MATERIAL DO LIKE OUR FACEBOOK PAGE httpwwwfacebookcomsscmentorsofficial

घरहॳिमकिी Correct Answer Lizard

Q147 Which type of pathogen causes

the water-borne disease SARS (Severe

Acute Respiratory Syndrome)

ककसपरकािकारह८गज़नकजिजननतबीमारीसासयकाकारणबनताहहॴ 25-Jan-2017

Options

1) Viral

वायरि

2) Parasitic

परजीवी 3) Protozoan

परह८टह८जअन

4) Bacterial

बहॴकटीररयि

Correct Answer Viral

Q148 Which of the following organs

produces the enzyme lipase

नननननलिखितमसहॳकह९नसाअगिायपहॳजएजाइमउतपननकरताहहॴ 25-Jan-2017

Options

1) Pancreas

अगनयािय

2) Large Intestine

बड़ीआत

3) Liver

नजगर

4) Small Intestine

छह८टीआत

Correct Answer Pancreas

Q149 A is a long internode forming the

basal part or the whole of a peduncle

एक mdashmdash- एकिबाइटरनह८िहहॴ जह८ननचिाटहससायासनपणयिठिबनताहहॴ 25-

Jan-2017

Options

1) Rhizome

परकद

2) Rachis

महॳ दि

3) floral axis

पषपअकष

4) Scape

भगदड़

Correct Answer scape

Q150 ndash Which of the following

organisms are considered to be both

Living and Non-living

नननननलिखितमसहॳकह९नसहॳजीवाणकह८जीपवतऔरअजीपवतमानाजाताहहॴ

25-Jan-2017

Options

1) Bacteria

बहॴकटीररया 2) Fungi

कवक

3) Algae

िहॴवाि

4)Virus

वायरस

Correct Answer Virus

They are considered to be living as they

possess a protein coat as a protective

covering DNA as the genetic material

etc

They are said to be non-living as they

can be crystallised and they survive for

billions of years They can tolerate high

temperatures freezing cold

temperatures ultra-violet radiations etc

Q151 Deficiency of fluorine causes

which of the following

फिह८ररनकीकमीकहॳ कारणनननननलिखितमसहॳकयाहह८ताहहॴ

F A C E B O O K

P A G E h t t p w w w f a c e b o o k c o m s s c m e n t o r s o f f i c i a l P a g e | 36

FOR MORE UPDATES AND MORE MATERIAL DO LIKE OUR FACEBOOK PAGE httpwwwfacebookcomsscmentorsofficial

27-Jan-2017

Options

1) Dental Caries

िटिकहॴ ररज

2) Scurvy

सकवरी 3) Anaemia

रकतामपता 4) Arthritis

गटठया Correct Answer Dental Caries

Q152 In a Punnett Square with the

cross AaBb x AaBb how many Aabb

genotypes would be created

पनहॳटसककायरमिह८स AaBb x AaBb कहॳ साि

ककतनहॳ Aabb जीनह८टाइपबनगहॳ 27-Jan-2017

Options

1) 1

2) 8

3) 2

4) 3

Correct Answer 2

Q153 Which of the following is the

Controlling Center of the Cell

नननननलिखित म सहॳ कह८लिकाका ननयतरण

क दर कह९न हहॴ

27-Jan-2017

Options

1) Nucleus

क दर

2) Plasma

पिाजमा 3) Lysosome

िायसह८सह८म

4) Chromosome

िह८मह८सह८म

Correct Answer Nucleus

The control centre of the cell is the

nucleus in eukaryotic cells The nucleus

contains genetic material in the form of

DNA

Q154 Myopia affects which of the

following organs

मायह८पपयानननननलिखितअगह८मसहॳककसहॳपरभापवतकरताहहॴ

25-Jan-2017

Options

1) Heart

हदय

2) Skin

तवचा 3) Eyes

आािहॳ 4)Mouth

मह

Correct Answer Eyes

Q155 Which of the following bears

flowers

नननननलिखितमसहॳकह९नफिधारणकरताहहॴ

25-Jan-2017

Options

1) Bryophyta

िायह८फाइटा 2) Pteridophyta

टहॳरीिह८फाईटा 3) Gymnosperms

नजननह८सपमय 4)Angiosperms

एननजयह८सपमय Correct Answer Angiosperms

Q156 Oxygenated blood flows out of the

heart through the

ऑकसीजनयकतरकत mdashmdashmdash

कहॳ माधयमसहॳहदयकहॳ बाहरबहताहहॴ 25-Jan-2017

F A C E B O O K

P A G E h t t p w w w f a c e b o o k c o m s s c m e n t o r s o f f i c i a l P a g e | 37

FOR MORE UPDATES AND MORE MATERIAL DO LIKE OUR FACEBOOK PAGE httpwwwfacebookcomsscmentorsofficial

Options

1) Aorta

महाधमनी 2) pulmonary artery

फहॳ फड़हॳकीधमनी 3) vena cava

वहॳनाकावा 4)Atrium

चह९क

Correct Answer aorta

Q157 Blood leaving the liver and

moving towards the

heart has a higher concentration of

नजगरसहॳननकिकरहदयकीतरफजानहॳवािहॳरकतम mdashmdashmdashmdash कीउचचसादरताहह८तीहहॴ 27-Jan-2017

Options

1) Lipids

लिपपडस

2) Urea

यररया 3) Bile Pigments

पपततकहॳ रगकरण

4) Carbon dioxide

काबयनिायऑकसाइि

Correct Answer Bile Pigments

Urea is nitrogen containing substance

which is produced in the liver in order

to deal with excess amino-acids in the

body As urea is produced it leaves the

liver in the blood stream and passes via

the circulatory system to all parts of the

body

Q158 Bulb is a modification of which

part of a plant

बमबएकपह९धहॳकहॳ ककसटहससहॳकाएक पातरणहह८ताहहॴ 27-Jan-2017

Options

1) The root

जड़

2) The stem

तना 3) The radicle

मिाकर

4)The fruit

फि

Correct Answer The stem

Q159 Which of the following carries

blood away from the heart to different

body parts

इनमहॳसहॳकह९नरकतकह८हदयसहॳिरीरकहॳ पवलभननअगह८तकिहॳजातीहहॴ

27-Jan-2017

Options

1) Arteries

धमननया 2) Nerves

तबतरहाए

3) Capillaries

कहॳ लिकाए

4)Veins

नसहॳ Correct Answer Arteries

Q160 The series of processes by which

nitrogen and its compounds are

interconverted in the environment and

in living organisms is called

27-Jan-2017

Options

1)Absorption of Nitrogen

2)Ammonification

3)Nitrogen Fixation

4)Nitrogen Cycle

Correct Answer Nitrogen Cycle

Ammonification or Mineralization is

performed by bacteria to convert

organic nitrogen to ammonia

F A C E B O O K

P A G E h t t p w w w f a c e b o o k c o m s s c m e n t o r s o f f i c i a l P a g e | 38

FOR MORE UPDATES AND MORE MATERIAL DO LIKE OUR FACEBOOK PAGE httpwwwfacebookcomsscmentorsofficial

Nitrification can then occur to convert

the ammonium to nitrite and nitrate

Nitrogen fixation is a process by which

nitrogen in the Earthrsquos atmosphere is

converted into ammonia (NH3) or other

molecules available to living organisms

Q161 BCG vaccine is given to protect

from which of the following

बीसीजीकाटटकानननननलिखितमसहॳककसकहॳ बचावकहॳ लिएटदयाजातहहॴ

27-Jan-2017

Options

1) Jaundice

पीलिया 2) Anaemia

रकतमपता 3) Tuberculosis

कषयरह८ग

4) Polio

पह८लियह८ Correct Answer Tuberculosis

Q162 Parallel venation is found in

समानतरवहॳनहॳिन mdashmdashmdash- मपायाजाताहहॴ 27-Jan-2017

Options

1) plants which are monocots

पह९धहॳजह८एकबीजपतरीहह८तहॳहहॴ 2) plants which have a dicot stem

वहॳपह९धहॳनजनकातनादपवदलियहह८ताहहॴ 3) plants with leaves similar to Tulsi

वहॳपह९धहॳनजनकीपनततयतिसीकीपनततयोकहॳ समानहह८तहॳहहॴ 4)plants with tap roots

टहॳप टवािहॳपह९धहॳ Correct Answer plants which are

monocots

Q163 The hardest part of the body is

िरीरकासबसहॳकठह८रभाग mdashndash हहॴ 27-Jan-2017

Options

1) Bones

हडडिय

2) Tooth Enamel

दातकहॳ इनहॳमि

3) Skull

िह८पड़ी 4) Spinal Cord

महॳ रजज

Correct Answer Tooth Enamel

Q164 Which type of pathogen causes

the waterborne disease E coli Infection

ककसपरकारकारह८गजननकजिजननतरह८गईकह८िाईसिमणकाकारणबनताहहॴ 27-Jan-2017

Options

1) Protozoan

परह८टह८जआ

2) Parasitic

परजीवी 3) Bacterial

बहॴकटीररयि

4)Viral

वायरि

Correct Answer Bacterial

Q165 The amount of blood filtered

together by both the kidneys in a 70 kg

adult male human in a minute is

70 की गरा वािहॳएकवयसकप षमएकलमनटमदह८नोगदकहॳदवाराएकसािचाबनीगयीरकतकीमातरहह८तीहहॴ 29-Jan-2017

Options

1) 1100 ml

1100 लमलि

2) 100 ml

F A C E B O O K

P A G E h t t p w w w f a c e b o o k c o m s s c m e n t o r s o f f i c i a l P a g e | 39

FOR MORE UPDATES AND MORE MATERIAL DO LIKE OUR FACEBOOK PAGE httpwwwfacebookcomsscmentorsofficial

100 लमलि

3) 1500 ml

1500 लमलि

4) 500 ml

500 लमलि

Correct Answer 1100 ml

Q166 Which feature of a plant helps to

distinguish a monocot from a dicot

पह९धहॳकीवहकह९नसीपविहॳषताहहॴजह८एकदपवदलियहॳऔरएकएकदिीयपह९धहॳसहॳभहॳदकरनहॳममददकरतीहहॴ 29-Jan-2017

Options

1) Pollination

परागम

2) Venation

वहॳनहॳिन

3) Vernation

वनिन

4) Aestivation

एसटीवहॳिहॳन

Correct Answer venation

Q167 The Mutation Theory was

proposed by

उतवररवतयनकालसदात mdashmdashndash

कहॳ दवरापरसतापवतककयाजाताहहॴ 29-Jan-2017

Options

1) Charles Lyell

चामसयलियहॳि

2) William Smith

पवलियमनसमि

3) Hugo De Vries

हयगह८िीराईस

4)Harrison Schmitt

हहॳरीसननसमट

Correct Answer Hugo De Vries

Q168 Which type of pathogen causes

the waterborne disease HepatitisA

ककसपरकारकहॳ रह८गजनकजिजननतरह८गहहॳपहॳटाइटटस-A काकारणबनताहहॴ

29-Jan-2017

Options

1) Parasitic

परजीवी 2) Viral

वायरि

3) Protozoan

परह८टह८जआ

4) Bacterial

बहॴकटीररयि

Correct Answer Viral

Q169 In a Punnett Square with the

cross AaBb x Aabb how many AaBb

genotypes would be created

पनहॳटसकवायरमिह८स AaBb x Aabb

कहॳ सािककतनहॳ AaBb जीनह८टाइपबनगहॳ 29-Jan-

2017

Options

1) 4

2) 1

3) 7

4) 6

Correct Answer 4

Q170 Arboreal Ateles is the scientific

name of

अिह८ररयिएटटलिस mdashmdashmdash कावहॴजञाननकनामहहॴ 29-Jan-2017

Options

1) Squirrel

चगिहरी 2) Sparrow

गह८रहॴया 3) Lizard

नछपकिी 4) Spider monkey

F A C E B O O K

P A G E h t t p w w w f a c e b o o k c o m s s c m e n t o r s o f f i c i a l P a g e | 40

FOR MORE UPDATES AND MORE MATERIAL DO LIKE OUR FACEBOOK PAGE httpwwwfacebookcomsscmentorsofficial

मकड़ीबदर

Correct Answer Spider monkey

Q171 Which type of pathogen causes

the waterborne disease Salmonellosis

ककसपरकारकारह८गाणजिजननतबीमारीसािमह८नहॳिह८लसज़काकारकहहॴ

29-Jan-2017

Options

1) Algal

िहॳवालियहॳ 2) Parasitic

परजीवी 3) Bacterial

बहॴकटीररयि

4)Viral

वायरि

Correct Answer Bacterial

An infection with salmonella bacteria

commonly caused by contaminated food

or water

Symptoms include diarrhoea fever

chills and abdominal pain

Q172 is a condition in which there is a

deficiency of red cells or of haemoglobin

in the blood

mdashmdash-

एकनसिनतहहॴनजसमहॳरकतमिािकह८लिकाओकीयाहीमह८गिह८बबनकीकमीहह८तीहहॴ 29-Jan-2017

Options

1) Albinism

एनमबननजम

2) Propyria

परह८पीररया 3) Anaemia

एनीलमया 4)Keloid disorder

कहॳ िह८इिडिसओिर

Correct Answer Anaemia

Q173 Ananas comosus is the scientific

name of

Options

अनानासकह८मह८सस mdashmdashmdashndash

कावहॴजञाननकनामहहॴ 29-Jan-2017

1) Custard Apple

सीताफि

2) Pineapple

पाइनएपपि

3) Bamboo

बास

4)Pomegranate

अनार

Correct Answer Pineapple

Q174 Which organ produces insulin

कह९नसाअगइनसलिनपहॴदाकरताहहॴ 29-Jan-

2017

Options

1) Liver

यकत

2) Thyroid gland

िायराइिगरिी 3) Spleen

पिीहा 4)Pancreas

अगरयिय

Correct Answer Pancreas

Q175 Which of the following disease is

not caused by water pollution

नननननलिखितमसहॳकह९नसारह८गपानीकहॳ परदषणकहॳकारणनहीहह८ता

29-Jan-2017

Options

1) Cholera

हहॴजा 2) Typhoid

F A C E B O O K

P A G E h t t p w w w f a c e b o o k c o m s s c m e n t o r s o f f i c i a l P a g e | 41

FOR MORE UPDATES AND MORE MATERIAL DO LIKE OUR FACEBOOK PAGE httpwwwfacebookcomsscmentorsofficial

टाइफाइि

3) Asthma

दमा 4)Diarrhoea

दसत

Correct Answer Asthma

Q176 Ocimum tenuiflorum is the

scientific name of

ओलिलममटहॳयईफिह८रमइसकावहॴजञाननकनाम mdash

ndash हहॴ 30-Jan-2017

Options

1) Neem

नीम

2) Mango

आम

3) Babul

बबि

4)Tulsi

तिसी Correct Answer Tulsi

Q177 Which gland secretes bile a

digestive fluid

कह९नसीगरिीपपतत एकपाचनतरिपरदािय सरापवतकरतीहहॴ 30-Jan-2017

Options

1) Pancreas

अगनयािय

2) Liver

यकत

3) Thyroid

िायराइि

4) Testes

टहॳनसटस

Correct Answer liver

Q178 In which of the following the

dominant phase is Gametophyte

नननननलिखितमसहॳककसकहॳ परमिचरणयगमकह८दपवधद (Gametophyte)हहॴ 30-Jan-2017

Options

1) Bryophyta

िायह८फाइटा 2) Pteridophyta

टहॳररिह८फाइटा 3) Gymnosperms

नजननह८सपमय 4) Angiosperms

एननजयह८सपमय Correct Answer Bryophyta

Q179 Anaerobic respiration refers to

which of the following

नननननलिखितमसहॳककसहॳअवायवीयशवसनकहाजाताहहॴ

30-Jan-2017

Options

1) Respiration without Oxygen

ऑकसीजनकहॳ बबनाशवसन

2) Respiration with Oxygen

ऑकसीजनकहॳ सािशवसन

3) Respiration without CO2

काबयनिायऑकसाइिकहॳ बबनाशवसन

4) Respiration with CO2

काबयनिायऑकसाइिकहॳ सािशविन

Correct Answer Respiration without

Oxygen

Q180 Which type of pathogen causes

the waterborne disease Cholera

ककसपरकारकारह८गजनकजिजननतरह८गहहॴजाकाकारणबनताहहॴ

30-Jan-2017

Options

1) Algal

िहॴवालियहॳ

F A C E B O O K

P A G E h t t p w w w f a c e b o o k c o m s s c m e n t o r s o f f i c i a l P a g e | 42

FOR MORE UPDATES AND MORE MATERIAL DO LIKE OUR FACEBOOK PAGE httpwwwfacebookcomsscmentorsofficial

2) Bacterial

बहॴकटीररयि

3) Protozoan

परह८टह८जआ

4) Viral

वायरि

Correct Answer Bacterial

Q181 To which class does

Oxyreductases transferases hydrolases

belong

ओकसीररिकटहॳसटरासफरहॳजहॳस

हाइडरह८िहॳसहॳसककसवगयमआतहॳहहॴ 30-Jan-2017

Options

1) Hormones

हारमोस

2) Enzymes

एजाइनस

3) Proteins

परह८टीनस

4) Vitamins

पवटालमनस

Correct Answer Enzymes

Q182 Which of the following is not true

about Gymnosperms

ननननमसहॳकह९नसीबातअनावतबीजीकहॳ बारहॳमसचनहीहहॴ 30-Jan-2017

Options

1) Dominant phase is saprophytes

परमिचरणसहॳपरह८फाइटसहह८ताहहॴ 2) Vascular bundles are absent

सवहनीबििअनपनसितहह८ताहहॴ 3) spores are heterospores

बीजाणहहॳटहॳरह८सपह८रसहह८तहॳहहॴ 4) Flowers are absent

फिअनपनसितहह८तहॳहहॴ

Correct Answer Vascular bundles are

absent

Q183 The name of first mammal clone sheep is

भहॳड़कीपरिमसतनपायीपरनत प (किह८न)

कानामहहॴ 30-Jan-2017

Options

1) Noori

नरी 2) Dolly

िॉिी 3) Louise

िसी 4)Durga

दगाय Correct Answer Dolly

Q184 Which type of pathogen causes

the water-borne disease Typhoid fever

ककसपरकारकारह८गजनकजिजननतरह८गटाइफाइिबिारकाकारणबनताहहॴ 30-Jan-2017

Options

1) Algal

िहॴवािीय

2) Parasitic

परजीवी 3) Protozoan

परह८टह८जनअन

4)Bacterial

बहॴकटीररयि

Correct Answer Bacterial

Q185 In which part of the cell are

proteins made

कह८लिकाकहॳ ककसटहससहॳमपरह८टीनबनायाजाताहहॴ

31-Jan-2017

Options

1) Reticulum

रहॳटटकिम

F A C E B O O K

P A G E h t t p w w w f a c e b o o k c o m s s c m e n t o r s o f f i c i a l P a g e | 43

FOR MORE UPDATES AND MORE MATERIAL DO LIKE OUR FACEBOOK PAGE httpwwwfacebookcomsscmentorsofficial

2) Golgi apparatus

गह८मजीएपहॳरहॳटस

3) Ribosomes

ररबह८सह८नस

4) Lysosome

िायसह८सह८नस

Correct Answer ribosomes

Proteins are produced by stringing

amino acids together in the order

specified by messenger RNA strands

that were transcribed from DNA in the

cell nucleus The process of synthesizing

a protein is called translation and it

occurs on ribosomes in the cytoplasm of

a cell

Q186 Polio is a disease caused by which

of the following

नननननलिखितमसहॳपह८लियह८कीबबमारह८हह८नहॳकाकारणकयाहहॴ

31-Jan-2017

Options

1) Bacteria

बहॴकटीररयि

2) Mosquito

मचछर

3) Virus

वायरस

4) Cockroach

नतिच हॳ Correct Answer Virus

Polio or poliomyelitis is a crippling and

potentially deadly infectious disease It

is caused by the poliovirus

Q187 ndash Hay fever is a sign of which of

the following

हहॳकफवरनननननलिखितमसहॳककसकाएकसकहॳ तहहॴ

31-Jan-2017

Options

1) Old Age

वदावसिा 2) Malnutrition

कपह८सण

3) Allergy

एिनजय 4) Over Work

अतयचधककाययकरना Correct Answer Allergy

Q188 How many chromosomes does a

human cell contain

एकमानवकह८लिकामककतनहॳगणसतरहह८तहॳहहॴ

29-Jan-2017

Options

1) 6

2) 26

3) 46

4) 66

Correct Answer 46

In humans each cell normally contains

23 pairs of chromosomes for a total of

46 Twenty-two of these pairs called

autosomes look the same in both males

and females The 23rd pair the sex

chromosomes differ between males and

females

Q189 Which of the following is not true

about Bryophyta

ननननमसहॳकह९नसीबातिायह८फाइटकहॳ बारहॳमसचनहीहहॴ 31-Jan-2017

Options

1) Dominant phase is gametophytes

परमिचरणगहॳलमतह८फाइटसहह८ताहहॴ 2) Main plant body is haploid

पह९धहॳकामखयिरीरअगखणतहह८ताहहॴ 3) Spores are homospores

बीजाणहह८मह८सफह८रसहह८तहॳहहॴ 4) Flowers are present

फिमह८जदहह८तहॳहहॴ Correct Answer Flowers are present

F A C E B O O K

P A G E h t t p w w w f a c e b o o k c o m s s c m e n t o r s o f f i c i a l P a g e | 44

FOR MORE UPDATES AND MORE MATERIAL DO LIKE OUR FACEBOOK PAGE httpwwwfacebookcomsscmentorsofficial

Q190 Which aquatic animal has

trailing tentacles

ककसजिीयजानवरकहॳ पीछहॳचिनहॳवािहॳटहॳटकिसहह८तहॳहहॴ

31-Jan-2017

Options

1) Sea horse

समदरीघह८िा 2) Corals

मगा 3) Jelly fish

जहॳिीमछिी 4) Star fish

तारामछिी Correct Answer Jelly fish

Jellyfish with its umbrella-shaped bell

and trailing tentacles

Q191 Which type of pathogen causes

the water-borne disease Poliomyelitis

(Polio)

ककसपरकारकारह८गजनकजिजननतरह८गपह८लियह८मायहॳटटस (पह८लियह८) काकारणहहॴ 31-Jan-

2017

Options

1) Parasitic

परजीवी 2) Algal

िहॴवालिय

3) Viral

वायरि

4) Bacterial

बहॴकटीररयि

Correct Answer Viral

Q192 The outer white part of the eye

that protects the inner structures is

आािकाबाहरीसफहॳ दटहससाजह८आतररकसरचनाओकीरकषाकरताहहॴ वह mdashmdashmdash हहॴ 31-Jan-

2017

Options

1) Iris

आयररस

2) Sclera

सकिहॳरा 3) Retina

रहॳटटना 4) Cornea

कह८ननयया Correct Answer Sclera

Q193 Proteins are made up of

परह८टीनकाननमायण mdashndash सहॳहह८ताहहॴ 31-Jan-2017

Options

1) Amino acids

एलमनह८अनि

2) Fatty acids

वसायकतअनि

3) Glucose

गिकह८ज

4)Nucleotides

नयनकियह८टाईिस

Correct Answer Amino acids

Q194 Moringa Oleifera is the scientific

name of

मह८ररगओलिफहॳ रा mdashmdashndash कावहॴजञाननकनामहहॴ 31-Jan-2017

Options

1) Banyan

बरगद

2) Gulmohar

गिमह८हर

3) Amla

आमिा

F A C E B O O K

P A G E h t t p w w w f a c e b o o k c o m s s c m e n t o r s o f f i c i a l P a g e | 45

FOR MORE UPDATES AND MORE MATERIAL DO LIKE OUR FACEBOOK PAGE httpwwwfacebookcomsscmentorsofficial

4) Drumstick

डरमनसटक

Correct Answer Drumstick

Q195 Kidney stones are composed of

गदकीपिरी mdashndash सहॳबनीहह८तीहहॴ 1-Feb-2017

Options

1) Calcium Oxalate

कहॴ नमसयमओकजहॳिहॳट

2) Sodium Chloride

सह८डियमकिह८राइि

3) Magnesium Nitrate

महॳनगनलियमनाइतटरहॳट

4) Calcium Bicarbonate

कहॴ नमियमबायकबोनहॳट

Correct Answer Calcium Oxalate

Q196 ndash Which of the following is not

true about Angiosperms

ननननमसहॳकह९नसीबातआवतबीजीकहॳ बारहॳमसचनहीहहॴ 1-Feb-2017

Options

1) Dominant phase is gametophytes

परमिचरणगहॳलमतह८फाइटहह८ताहहॴ 2) Vascular bundles are present

सवहनीबििमह९जदहह८ताहहॴ 3) Spores are heterospores

बीजाणहहॳटहॳरह८सपह८रसहह८तहॳहहॴ 4) Seeds are covered

बीजढकहॳ हह८तहॳहहॴ Correct Answer Dominant phase is

gametophytes

Q197 All of the following are excretory

(waste) products of animals except

नननननलिखितमसहॳककसएककह८छह८ड़करअनयसभीपराखणयोदवाराउतसनजयतपदाियहहॴ 1-Feb-

2017

Options

1) Uric Acid

यररकएलसि

2) Ammonia

अमह८ननया 3) Carbohydrates

काबोहाइडरहॳट

4) Urea

यररया Correct Answer Carbohydrates

In animals the main excretory products

are carbon dioxide ammonia (in

ammoniotelics) urea (in ureotelics) uric

acid (in uricotelics) guanine (in

Arachnida) and creatine

Q198 RNA is a polymeric molecule

What does RNA stand for

आरएनइएएकबहिकआणहहॴ इसकाकापवय पकयाहहॴ 1-Feb-2017

Options

1) Rado Nuclear Acid

रािह८नयनकियरएलसि

2) Ribo Nucleic Acid

राइबह८नयनकिकएलसि

3) Rhino Nuclear Acid

हाइनह८नयनकियरएलसि

4) Resto Nucleus Acid

रहॳसटह८नयकिीयसएलसि

Correct Answer Ribo Nucleic Acid

Q199 Which organ does detoxification

and produces chemicals needed for

digestion

कह९नसाअगपवषहरणकरताहहॴऔरपाचनकहॳ लिएआवशयकरसायनोकह८पहॴदाकरताहहॴ 1-Feb-

2017

Options

1) Salivary glands

िारगरचिया 2) Pancreas

अगनयािय

F A C E B O O K

P A G E h t t p w w w f a c e b o o k c o m s s c m e n t o r s o f f i c i a l P a g e | 46

FOR MORE UPDATES AND MORE MATERIAL DO LIKE OUR FACEBOOK PAGE httpwwwfacebookcomsscmentorsofficial

3) Thyroid gland

िायराइिगरिी 4) Liver

यकत

Correct Answer Liver

Q200 Psidium guajava is the scientific

name of

लसडियमगआजावा mdashmdash कावहॴजञाननकनामहहॴ 1-

Feb-2017

Options

1) Guava

अम द

2) Mango

आम

3) Bamboo

बास

4) Jack fruit

कटहि

Correct Answer Guava

Q201 Which drug is used as a Blood

Thinner

चधरकह८पतिाकरनहॳकहॳ पमककसदवाकापरयह८गककयाजाताहहॴ

1-Feb-2017

Options

1) Warfarin

वाफर न

2) Tramadol

टरहॳमािह८ि

3) Azithromycin

एनजरह८मायलसन

4) Hydralazine

हाइडरह८िहॳनजन

Correct Answer Warfarin

Q202 Which of the following disease is

caused due to the deficiency of protein

परह८टीनकीकमीकहॳ कारणनननननलिखितमसहॳकह९नसारह८गहह८ताहहॴ 1-Feb-2017

Options

1) Arthritis

गटठया 2) Kwashiorkor

कािीओकय र

3) Goitre

गाइटर

4) Night Blindness

रतह९चध

Correct Answer Kwashiorkor

Q203 A is species of plant that has

adapted to survive in an environment

with little liquid water

mdashmdashndashपह९धहॳकीएकऐसहॳऐसहॳपरजानतहहॴ नजसनहॳकमपानीवािहॳवातावरणमजीपवतरहनहॳकहॳलिएअनकिनहहॴ 1-Feb-2017

Options

1) Xerophyte

म दपवद

2) Hydrophyte

जिीयपादप

3) Mesophyte

समह८दपवद

4) Thallophyte

िहॴिह८फाइटा Correct Answer xerophyte

xerophyte is a species of plant that has

adapted to survive in an environment

with little liquid water such as a desert

or an ice- or snow-covered region in the

Alps or the Arctic

Mesophytes are terrestrial plants which

are adapted to neither a particularly

dry nor particularly wet environment

An example of a mesophytic habitat

would be a rural temperate meadow

F A C E B O O K

P A G E h t t p w w w f a c e b o o k c o m s s c m e n t o r s o f f i c i a l P a g e | 47

FOR MORE UPDATES AND MORE MATERIAL DO LIKE OUR FACEBOOK PAGE httpwwwfacebookcomsscmentorsofficial

which might contain goldenrod clover

oxeye daisy and Rosa multiflora

thallophyte any of a group of plants or

plantlike organisms (such as algae and

fungi) that lack differentiated stems

leaves and roots and that were formerly

classified as a primary division

(Thallophyta) of the plant kingdom

Q204 How many types of teeth are

there in humans

मनषयोमककतनहॳपरकारकहॳ दातहह८तहॳहहॴ

1-Feb-2017

Options

1) 4

2) 5

3) 2

4) 3

Correct Answer 4

teeth -Humans have four types of

teethincisors canines premolars and

molars each with a specific function

The incisors cut the food the canines

tear the food and the molars and

premolars crush the food

Q205 Carica papaya is the scientific name of

कहॴ ररकापपाया mdashmdashndash कावहॴजञाननकनामहहॴ 2-

Feb-2017

Options

1) Peepal

पीपि

2) Papaya

पपीता 3) Tamarind

इमिी 4) Drumstick

ढह८िकाछड़ी Correct Answer Papaya

Q206 Muscles get tired when there is

shortfall of

जब mdashndash कीकमीहह८तीहहॴतबपहॳिीयिकजातीहहॴ 2-Feb-2017

Options

1) Lactic acid

िहॴनकटकएलसि

2) Na+ ions

Na+ आयन

3) ATP

एटीपी 4) Sulphates

समफहॳ टस

Correct Answer ATP

ATP is the energy source muscle fibers

use to make muscles contract

muscle tissuersquos main source of energy

called adenosine triphosphate or ATP

As your muscles use up this energy

source they become tired and fatigued

Oxygen is the key ingredient that helps

create new ATP to replenish the burned

up ATP in your muscles

Q207 Artocarpus integra is the

scientific name of आटह८कापयसइटीगरा mdashmdashmdash कावहॴजञाननकनामहहॴ 2-Feb-2017

Options

1) Guava

अम द

2) Pineapple

अनानास

3) Silver Oak

लसमवरओक

4) Jack fruit

कटहि

Correct Answer Jack fruit

Q208 Which organ stores fat soluble

vitamins

कह९नसाअगवसामघिनिीिपवटालमनह८काभिाराकरताहहॴ

2-Feb-2017

F A C E B O O K

P A G E h t t p w w w f a c e b o o k c o m s s c m e n t o r s o f f i c i a l P a g e | 48

FOR MORE UPDATES AND MORE MATERIAL DO LIKE OUR FACEBOOK PAGE httpwwwfacebookcomsscmentorsofficial

Options

1) Blood

रकत

2) Skin

तवचा 3) Liver

यकत

4) Pancreas

अगनयािय

Correct Answer Liver

Q209 Which disease is caused due to

deficiency of Iodine

आयह८िीनकहॳ कारणकह९नसारह८गहह८ताहहॴ 2-Feb-2017

Options

1) Rickets

ररकहॳ टस

2) Scurvy

सकवी 3) Goitre

गणमािा 4) Growth retardation

पवकासका कना Correct Answer Goitre

rickets A softening and weakening of

bones in children usually due to

inadequate vitamin D

Q210 Grevillea Robusta is the scientific name of

गरहॳपवलियारह८बसटा mdashmdashmdash- कापवजञाननकनामहहॴ 2-Feb-2017

Options

1) Peepal

पीपि

2) Teak

सागह९न

3) Silver Oak

लसमवरओक

4) Jack fruit

कटहि

Correct Answer Silver Oak

Q211 When a Cuttlefish is described as a Molluscs it is at which level of

classification

जबएककटिकफिकह८एकमह८िसकाकहॳ पमवखणयतककयाजाताहहॴतबयहॳवगीकरणकहॳ ककससतरपहॳनसितहहॴ 2-Feb-2017

Options

1) Class

वगय 2) Order

िम

3) Family

पररवार

4) Phylum

सघ

Correct Answer Phylum

Q212 Bambusa dendrocalmus is the

scientific name of बानबसािहॳडराकामस mdashmdashmdash कावहॴजञाननकनामहहॴ 3-Feb-2017

Options

1) Banyan

बरगद

2) Papaya

पपीता 3) Bamboo

बास

4) Pomegranate

अनार

Correct Answer Bamboo

Q213 Acinonyx Jubatus is the scientific name of

एलसनह८ननकसजयबहॳटस mdashmdashmdash

कावहॴजञाननकनामहहॴ 3-Feb-2017

F A C E B O O K

P A G E h t t p w w w f a c e b o o k c o m s s c m e n t o r s o f f i c i a l P a g e | 49

FOR MORE UPDATES AND MORE MATERIAL DO LIKE OUR FACEBOOK PAGE httpwwwfacebookcomsscmentorsofficial

Options

1) Bear

भाि 2) Horse

घह८िा 3) Cheetah

चीता 4) Zebra

जहॳिा Correct Answer Cheetah

Q214 The pale yellow colour of urine is

due to the presence of which pigment

मतरकाफीकापीिारगरगदरयकहॳ उपनसिनतकहॳ कारणहह८ताहहॴ

3-Feb-2017

Options

1) Urochrome

यरह८िह८म

2) Urophyll

यरह८कफि

3) Chlorophyll

किह८रह८कफि

4) Chloroplast

किह८रह८पिासट

Correct Answer Urochrome

Q215 Which of the following constitute

to form a gene

नननननलिखितमसहॳकह९नसीचीज़एकजीनकागठनकरतीहहॴ

3-Feb-2017

Options

1) Polynucleotides

पह८िीनयनकियह८टाईडस

2) Hydrocarbons

हाइडरह८काबोस

3) Lipoproteins

िाईपह८परह८टीनस

4) Lipids

लिपपडस

Correct Answer Polynucleotides

Polynucleotide molecule is a biopolymer

composed of 13 or more nucleotide

monomers covalently bonded in a chain

DNA (deoxyribonucleic acid) and RNA

(ribonucleic acid) are examples of

polynucleotides with distinct biological

function

Q216 Vertebrates belongs to the

phylum

रीढ़कीहडिीवािहॳपराणी mdashmdashmdash

परजानतकहॳ अतगायतआतहॳहहॴ 3-Feb-2017

Options

1) Arthropoda

आरह८पह८ड़ा 2) Annelida

एननलििा 3) Cnidaria

ननिहॳररया 4) Chordata

कह८िटा Correct Answer Chordata

Q217 Punica granatum is the scientific name of

पननकगरहॳनहॳटस mdashmdashmdash कावहॴजञाननकनामहहॴ 3-Feb-2017

Options

1) Custard Apple

सीताफि

2) Gulmohar

गिमह८हर

3) Silver Oak

लसमवरओक

4) Pomegranate

अनार

Correct Answer Pomegranate

F A C E B O O K

P A G E h t t p w w w f a c e b o o k c o m s s c m e n t o r s o f f i c i a l P a g e | 50

FOR MORE UPDATES AND MORE MATERIAL DO LIKE OUR FACEBOOK PAGE httpwwwfacebookcomsscmentorsofficial

Q218 Between a tiger and an monkey

which of the following is different

एकबाघऔरबदरकहॳ बीचनननननलिखितमसहॳकह९नसीबातअिगहहॴ 3-Feb-2017

Options

1) Kingdom

राजय

2) Phylum

जानत

3) Order

िम

4) Class

वगय Correct Answer order

Q219 The artificial heart was invented by

कबतरमहदयका mdashmdashmdash

दवाराअपवषकारककयागयािा 3-Feb-2017

Options

1) Muhammad Yunus

महनमदयनस

2) Linus Yale Jr

िाइनसयहॳिजय

3) Gazi Yasargil

गाजीयासचगयि

4) Paul Winchell

पह९िपवमकि Correct Answer Paul Winchell

Q220 Tamarindus indica is the

scientific name of

टहॳमररनडसइडिका mdashmdash कावहॴजञाननकनामहहॴ 7-

Feb-2017

Options

1) Neem

नीम

2) Pineapple

अनानास

3) Tamarind

इमिी 4)Chiku

चीक

Correct Answer Tamarind

Q221 In eukaryotic cells synthesis of

RNA takes place in the

यकहॳ योटटककह८लिकाओमआरएनएकासशिहॳषण

mdashndash महह८ताहहॴ 7-Feb-2017

Options

1) Mitochondria

माईटह८कोडडरया 2) Centrioles

सटरीयह८मस

3) Ribosomes

ररबह८सह८नस

4) Nucleus

नयनकियस

Correct Answer nucleus

eukaryotic cell -Transcription is the

process of synthesizing ribonucleic acid

(RNA)Synthesis takes place within the

nucleus of eukaryotic cells or in the

cytoplasm of prokaryotes and converts

the genetic code from a gene in

deoxyribonucleic acid ( DNA ) to a

strand of RNA that then directs

proteinsynthesis

Q222 _________is caused by parasites

of the Plasmodium genus

पिाजमह८डियमजातीकहॳ परजीवी mdash- कहॳ कारणहहॴ 7-Feb-2017

Options

1) Dysentery

पहॳचचि

2) Malaria

मिहॳररया 3) Chickenpox

F A C E B O O K

P A G E h t t p w w w f a c e b o o k c o m s s c m e n t o r s o f f i c i a l P a g e | 51

FOR MORE UPDATES AND MORE MATERIAL DO LIKE OUR FACEBOOK PAGE httpwwwfacebookcomsscmentorsofficial

चहॳचक

4) Herpes

हहॳपपयस

Correct Answer Malaria

Q223 Carotene in fruits and vegetables

gives it which color

फिह८औरसनलजयोमनसितकहॳ रह८टीनउनहकह९नसारगपरदानकरताहहॴ 7-Feb-2017

Options

1) Green

हरा 2) Pink

गिाबी 3) Orange

नारगी 4) Blue

नीिा Correct Answer Orange

Q224 Equus Caballus is the scientific

name of

एकवसकहॴ बहॳिस mdashmdashndash कापवजञाननकनामहहॴ 7-Feb-2017

Options

1) Horse

घह८िा 2) Zebra

ज़हॳिा 3) Donkey

गधा 4) Buffalo

भस

Correct Answer Horse

Q225 Elapidae Naja is the scientific name of

एिीपीिीनाजा mdashmdash- कावहॴजञाननकनामहहॴ 8-Feb-2017

Options

1) Cobra

कह८बरा 2) Elephant

हािी 3) Eagle

ग ि

4) Owl

उमि Correct Answer Cobra

Q226 Which disease is caused due to

deficiency of Iron

िह८हकीकमीकहॳ कारणकह९नसारह८गहह८ताहहॴ 8-Feb-

2017

Options

1) Beriberi

बहॳरीबहॳरी 2) Tetany

टहॳटनी 3) Kwashiorkor

कवािीऔरकर

4) Anaemia

रकतामपता Correct Answer Anaemia

Beriberi is a disease caused by a vitamin

B-1 deficiency also known as thiamine

deficiency

Tetany can be the result of an

electrolyte imbalance Most often itrsquos a

dramatically low calcium level also

known as hypocalcemia Tetany can also

be caused by magnesium deficiency or

too little potassium Having too much

acid (acidosis) or too much alkali

(alkalosis) in the body can also result in

tetany

Kwashiorkor also known as

ldquoedematous malnutrition It is a form of

malnutrition caused by a lack of protein

in the diet

Anaemia means that you have fewer red

blood cells than normal or you have less

F A C E B O O K

P A G E h t t p w w w f a c e b o o k c o m s s c m e n t o r s o f f i c i a l P a g e | 52

FOR MORE UPDATES AND MORE MATERIAL DO LIKE OUR FACEBOOK PAGE httpwwwfacebookcomsscmentorsofficial

haemoglobin than normal in each red

blood cell

Q227 is a leaf where the leaflets are

arranged along the middle vein

mdashndashएकपततीहहॴजहापतरकह८कीरचनाक ररयालिराकहॳ आसपासहह८तीहहॴ 8-Feb-2017

Options

1) Pinnately compound leaf

पपनहॳटिीसयकतपतती 2) Palmately compound leaf

पामहॳटिीसयकतपतती 3) Compound leaf

सयकतपतती 4) Simple leaf

साधारणपतती Correct Answer Pinnately compound

leaf

Q228 Haustoria or sucking roots are

found in which of the following

हह८सटह८ररयायाचसनहॳवािीजड़हॳनननननलिखितमसहॳककसमपाईजातीहहॴ 8-Feb-2017

Options

1) Wheat

गहॳह

2) Mango

आम

3) Chestnut

चहॳसटनट

4) Cuscuta

कसकयटा Correct Answer Cuscuta

Haustorial roots -The roots of parasitic

plants which penetrate into the host

tissues to absorb nourishment are

called haustorial roots hellip Also known as suckingor parasitic roots

Q229 Equs Asinus is the scientific name

of

एकवसएलसनस mdashmdashndash कावहॴजञाननकनामहहॴ 8-

Feb-2017

Options

1) Donkey

गधा 2) Cow

गाय

3) Deer

टहरन

4) Kangaroo

कगा

Correct Answer Donkey

Q230 Ficus benghalensis is the scientific name of

फाईकसबहॳनगहॳिहॳलसस mdashndash कापवजञाननकनामहहॴ 8-Feb-2017

Options

1) Banyan

बरगद

2) Pineapple

अनानास

3) Babul

बबि

4) Tulsi

तिसी Correct Answer Banyan

Q231 Equus burchellii is the scientific name of

एकवसबचिी mdashmdash- कापवजञाननकनामहहॴ 8-Feb-2017

Options

1) Horse

घह८िा 2) Zebra

जहॳिा 3) Buffalo

F A C E B O O K

P A G E h t t p w w w f a c e b o o k c o m s s c m e n t o r s o f f i c i a l P a g e | 53

FOR MORE UPDATES AND MORE MATERIAL DO LIKE OUR FACEBOOK PAGE httpwwwfacebookcomsscmentorsofficial

भस

4) Ass

गधा Correct Answer Zebra

Page 35: COMPILATION OF ALL 72 SETS OF BIOLOGY SSC CHSL-2016 · OF BIOLOGY SSC CHSL-2016 PREPARED BY : SSC MENTORS BIOLOGY SPECIAL . F A C E B O O K P A G E : h t t p : / / w w w . f a c e

F A C E B O O K

P A G E h t t p w w w f a c e b o o k c o m s s c m e n t o r s o f f i c i a l P a g e | 34

FOR MORE UPDATES AND MORE MATERIAL DO LIKE OUR FACEBOOK PAGE httpwwwfacebookcomsscmentorsofficial

सायनह८कफि

4)Phototropism

फह८टह८टरोपपजम

Correct Answer Chlorophyll

Q143 Organisms that use light to

prepare food are known as

जह८जीवपरकािकाउपयह८गकरभह८जनतहॴयारकरतहॳहहॴ उनह mdashmdash- कहॳ पमजानजाताहहॴ 24-Jan-2017

Options

1) Autotrophs

सवपह८षी 2) Heterotrophs

पवषमपह८षज

3) Omnivores

सवायहारी 4)Decomposers

पवघटनकरनहॳवािा Correct Answer Autotrophs

autotrophs often make their own food

by using sunlight carbon dioxide and

water to form sugars which they can use

for energy Some examples of

autotrophs include plants algae and

even some bacteria Autotrophs

(producer) are important because they

are a food source for heterotrophs

(consumers)

A heterotroph is an organism that

ingests or absorbs organic carbon

(rather than fix carbon from inorganic

sources such as carbon dioxide) in order

to be able to produce energy and

synthesize compounds to maintain its

life Ninety-five percent or more of all

types of living organisms are

heterotrophic including all animals and

fungi and some bacteria

Q144 Which of the following is a

primary function of haemoglobin

नननननलिखितमसहॳकह९नसाटहमह८गिह८बबनकाएकपरािलमककाययहहॴ

25-Jan-2017

Options

1) Utilization of energy

उजायकाउपयह८गकरना 2) Prevention of anaemia

रकतामपताहह८नहॳसहॳरह८कना 3) Destruction of bacteria

बहॴकटीररयाकापवनािकरना 4) To transport oxygen

ऑकसीजनकावहनकरना Correct Answer To transport oxygen

Q145 Vascular bundles are absent in

सवहनीबिि mdashmdash- मअनपनसतिरहतहॳहहॴ 25-Jan-2017

Options

1) Bryophyta

िायह८फाइटा 2) Pteridophyta

टहॳररिह८फाईटा 3) Gymnosperms

नजननह८सपमय 4) Angiosperms

एननजयह८सपहॳनसय Correct Answer Bryophyta

Q146 Sauria Lacertidae is the scientific

name of

सहॴररयािहॳसरटाईिी mdashmdashndash कावहॴजञाननकनामहहॴ 25-Jan-2017

Options

1) Crocodile

मगरमचछ

2) Hippopotamus

टहपपह८पह८टहॳमस

3) Lizard

नछपकिी 4) House fly

F A C E B O O K

P A G E h t t p w w w f a c e b o o k c o m s s c m e n t o r s o f f i c i a l P a g e | 35

FOR MORE UPDATES AND MORE MATERIAL DO LIKE OUR FACEBOOK PAGE httpwwwfacebookcomsscmentorsofficial

घरहॳिमकिी Correct Answer Lizard

Q147 Which type of pathogen causes

the water-borne disease SARS (Severe

Acute Respiratory Syndrome)

ककसपरकािकारह८गज़नकजिजननतबीमारीसासयकाकारणबनताहहॴ 25-Jan-2017

Options

1) Viral

वायरि

2) Parasitic

परजीवी 3) Protozoan

परह८टह८जअन

4) Bacterial

बहॴकटीररयि

Correct Answer Viral

Q148 Which of the following organs

produces the enzyme lipase

नननननलिखितमसहॳकह९नसाअगिायपहॳजएजाइमउतपननकरताहहॴ 25-Jan-2017

Options

1) Pancreas

अगनयािय

2) Large Intestine

बड़ीआत

3) Liver

नजगर

4) Small Intestine

छह८टीआत

Correct Answer Pancreas

Q149 A is a long internode forming the

basal part or the whole of a peduncle

एक mdashmdash- एकिबाइटरनह८िहहॴ जह८ननचिाटहससायासनपणयिठिबनताहहॴ 25-

Jan-2017

Options

1) Rhizome

परकद

2) Rachis

महॳ दि

3) floral axis

पषपअकष

4) Scape

भगदड़

Correct Answer scape

Q150 ndash Which of the following

organisms are considered to be both

Living and Non-living

नननननलिखितमसहॳकह९नसहॳजीवाणकह८जीपवतऔरअजीपवतमानाजाताहहॴ

25-Jan-2017

Options

1) Bacteria

बहॴकटीररया 2) Fungi

कवक

3) Algae

िहॴवाि

4)Virus

वायरस

Correct Answer Virus

They are considered to be living as they

possess a protein coat as a protective

covering DNA as the genetic material

etc

They are said to be non-living as they

can be crystallised and they survive for

billions of years They can tolerate high

temperatures freezing cold

temperatures ultra-violet radiations etc

Q151 Deficiency of fluorine causes

which of the following

फिह८ररनकीकमीकहॳ कारणनननननलिखितमसहॳकयाहह८ताहहॴ

F A C E B O O K

P A G E h t t p w w w f a c e b o o k c o m s s c m e n t o r s o f f i c i a l P a g e | 36

FOR MORE UPDATES AND MORE MATERIAL DO LIKE OUR FACEBOOK PAGE httpwwwfacebookcomsscmentorsofficial

27-Jan-2017

Options

1) Dental Caries

िटिकहॴ ररज

2) Scurvy

सकवरी 3) Anaemia

रकतामपता 4) Arthritis

गटठया Correct Answer Dental Caries

Q152 In a Punnett Square with the

cross AaBb x AaBb how many Aabb

genotypes would be created

पनहॳटसककायरमिह८स AaBb x AaBb कहॳ साि

ककतनहॳ Aabb जीनह८टाइपबनगहॳ 27-Jan-2017

Options

1) 1

2) 8

3) 2

4) 3

Correct Answer 2

Q153 Which of the following is the

Controlling Center of the Cell

नननननलिखित म सहॳ कह८लिकाका ननयतरण

क दर कह९न हहॴ

27-Jan-2017

Options

1) Nucleus

क दर

2) Plasma

पिाजमा 3) Lysosome

िायसह८सह८म

4) Chromosome

िह८मह८सह८म

Correct Answer Nucleus

The control centre of the cell is the

nucleus in eukaryotic cells The nucleus

contains genetic material in the form of

DNA

Q154 Myopia affects which of the

following organs

मायह८पपयानननननलिखितअगह८मसहॳककसहॳपरभापवतकरताहहॴ

25-Jan-2017

Options

1) Heart

हदय

2) Skin

तवचा 3) Eyes

आािहॳ 4)Mouth

मह

Correct Answer Eyes

Q155 Which of the following bears

flowers

नननननलिखितमसहॳकह९नफिधारणकरताहहॴ

25-Jan-2017

Options

1) Bryophyta

िायह८फाइटा 2) Pteridophyta

टहॳरीिह८फाईटा 3) Gymnosperms

नजननह८सपमय 4)Angiosperms

एननजयह८सपमय Correct Answer Angiosperms

Q156 Oxygenated blood flows out of the

heart through the

ऑकसीजनयकतरकत mdashmdashmdash

कहॳ माधयमसहॳहदयकहॳ बाहरबहताहहॴ 25-Jan-2017

F A C E B O O K

P A G E h t t p w w w f a c e b o o k c o m s s c m e n t o r s o f f i c i a l P a g e | 37

FOR MORE UPDATES AND MORE MATERIAL DO LIKE OUR FACEBOOK PAGE httpwwwfacebookcomsscmentorsofficial

Options

1) Aorta

महाधमनी 2) pulmonary artery

फहॳ फड़हॳकीधमनी 3) vena cava

वहॳनाकावा 4)Atrium

चह९क

Correct Answer aorta

Q157 Blood leaving the liver and

moving towards the

heart has a higher concentration of

नजगरसहॳननकिकरहदयकीतरफजानहॳवािहॳरकतम mdashmdashmdashmdash कीउचचसादरताहह८तीहहॴ 27-Jan-2017

Options

1) Lipids

लिपपडस

2) Urea

यररया 3) Bile Pigments

पपततकहॳ रगकरण

4) Carbon dioxide

काबयनिायऑकसाइि

Correct Answer Bile Pigments

Urea is nitrogen containing substance

which is produced in the liver in order

to deal with excess amino-acids in the

body As urea is produced it leaves the

liver in the blood stream and passes via

the circulatory system to all parts of the

body

Q158 Bulb is a modification of which

part of a plant

बमबएकपह९धहॳकहॳ ककसटहससहॳकाएक पातरणहह८ताहहॴ 27-Jan-2017

Options

1) The root

जड़

2) The stem

तना 3) The radicle

मिाकर

4)The fruit

फि

Correct Answer The stem

Q159 Which of the following carries

blood away from the heart to different

body parts

इनमहॳसहॳकह९नरकतकह८हदयसहॳिरीरकहॳ पवलभननअगह८तकिहॳजातीहहॴ

27-Jan-2017

Options

1) Arteries

धमननया 2) Nerves

तबतरहाए

3) Capillaries

कहॳ लिकाए

4)Veins

नसहॳ Correct Answer Arteries

Q160 The series of processes by which

nitrogen and its compounds are

interconverted in the environment and

in living organisms is called

27-Jan-2017

Options

1)Absorption of Nitrogen

2)Ammonification

3)Nitrogen Fixation

4)Nitrogen Cycle

Correct Answer Nitrogen Cycle

Ammonification or Mineralization is

performed by bacteria to convert

organic nitrogen to ammonia

F A C E B O O K

P A G E h t t p w w w f a c e b o o k c o m s s c m e n t o r s o f f i c i a l P a g e | 38

FOR MORE UPDATES AND MORE MATERIAL DO LIKE OUR FACEBOOK PAGE httpwwwfacebookcomsscmentorsofficial

Nitrification can then occur to convert

the ammonium to nitrite and nitrate

Nitrogen fixation is a process by which

nitrogen in the Earthrsquos atmosphere is

converted into ammonia (NH3) or other

molecules available to living organisms

Q161 BCG vaccine is given to protect

from which of the following

बीसीजीकाटटकानननननलिखितमसहॳककसकहॳ बचावकहॳ लिएटदयाजातहहॴ

27-Jan-2017

Options

1) Jaundice

पीलिया 2) Anaemia

रकतमपता 3) Tuberculosis

कषयरह८ग

4) Polio

पह८लियह८ Correct Answer Tuberculosis

Q162 Parallel venation is found in

समानतरवहॳनहॳिन mdashmdashmdash- मपायाजाताहहॴ 27-Jan-2017

Options

1) plants which are monocots

पह९धहॳजह८एकबीजपतरीहह८तहॳहहॴ 2) plants which have a dicot stem

वहॳपह९धहॳनजनकातनादपवदलियहह८ताहहॴ 3) plants with leaves similar to Tulsi

वहॳपह९धहॳनजनकीपनततयतिसीकीपनततयोकहॳ समानहह८तहॳहहॴ 4)plants with tap roots

टहॳप टवािहॳपह९धहॳ Correct Answer plants which are

monocots

Q163 The hardest part of the body is

िरीरकासबसहॳकठह८रभाग mdashndash हहॴ 27-Jan-2017

Options

1) Bones

हडडिय

2) Tooth Enamel

दातकहॳ इनहॳमि

3) Skull

िह८पड़ी 4) Spinal Cord

महॳ रजज

Correct Answer Tooth Enamel

Q164 Which type of pathogen causes

the waterborne disease E coli Infection

ककसपरकारकारह८गजननकजिजननतरह८गईकह८िाईसिमणकाकारणबनताहहॴ 27-Jan-2017

Options

1) Protozoan

परह८टह८जआ

2) Parasitic

परजीवी 3) Bacterial

बहॴकटीररयि

4)Viral

वायरि

Correct Answer Bacterial

Q165 The amount of blood filtered

together by both the kidneys in a 70 kg

adult male human in a minute is

70 की गरा वािहॳएकवयसकप षमएकलमनटमदह८नोगदकहॳदवाराएकसािचाबनीगयीरकतकीमातरहह८तीहहॴ 29-Jan-2017

Options

1) 1100 ml

1100 लमलि

2) 100 ml

F A C E B O O K

P A G E h t t p w w w f a c e b o o k c o m s s c m e n t o r s o f f i c i a l P a g e | 39

FOR MORE UPDATES AND MORE MATERIAL DO LIKE OUR FACEBOOK PAGE httpwwwfacebookcomsscmentorsofficial

100 लमलि

3) 1500 ml

1500 लमलि

4) 500 ml

500 लमलि

Correct Answer 1100 ml

Q166 Which feature of a plant helps to

distinguish a monocot from a dicot

पह९धहॳकीवहकह९नसीपविहॳषताहहॴजह८एकदपवदलियहॳऔरएकएकदिीयपह९धहॳसहॳभहॳदकरनहॳममददकरतीहहॴ 29-Jan-2017

Options

1) Pollination

परागम

2) Venation

वहॳनहॳिन

3) Vernation

वनिन

4) Aestivation

एसटीवहॳिहॳन

Correct Answer venation

Q167 The Mutation Theory was

proposed by

उतवररवतयनकालसदात mdashmdashndash

कहॳ दवरापरसतापवतककयाजाताहहॴ 29-Jan-2017

Options

1) Charles Lyell

चामसयलियहॳि

2) William Smith

पवलियमनसमि

3) Hugo De Vries

हयगह८िीराईस

4)Harrison Schmitt

हहॳरीसननसमट

Correct Answer Hugo De Vries

Q168 Which type of pathogen causes

the waterborne disease HepatitisA

ककसपरकारकहॳ रह८गजनकजिजननतरह८गहहॳपहॳटाइटटस-A काकारणबनताहहॴ

29-Jan-2017

Options

1) Parasitic

परजीवी 2) Viral

वायरि

3) Protozoan

परह८टह८जआ

4) Bacterial

बहॴकटीररयि

Correct Answer Viral

Q169 In a Punnett Square with the

cross AaBb x Aabb how many AaBb

genotypes would be created

पनहॳटसकवायरमिह८स AaBb x Aabb

कहॳ सािककतनहॳ AaBb जीनह८टाइपबनगहॳ 29-Jan-

2017

Options

1) 4

2) 1

3) 7

4) 6

Correct Answer 4

Q170 Arboreal Ateles is the scientific

name of

अिह८ररयिएटटलिस mdashmdashmdash कावहॴजञाननकनामहहॴ 29-Jan-2017

Options

1) Squirrel

चगिहरी 2) Sparrow

गह८रहॴया 3) Lizard

नछपकिी 4) Spider monkey

F A C E B O O K

P A G E h t t p w w w f a c e b o o k c o m s s c m e n t o r s o f f i c i a l P a g e | 40

FOR MORE UPDATES AND MORE MATERIAL DO LIKE OUR FACEBOOK PAGE httpwwwfacebookcomsscmentorsofficial

मकड़ीबदर

Correct Answer Spider monkey

Q171 Which type of pathogen causes

the waterborne disease Salmonellosis

ककसपरकारकारह८गाणजिजननतबीमारीसािमह८नहॳिह८लसज़काकारकहहॴ

29-Jan-2017

Options

1) Algal

िहॳवालियहॳ 2) Parasitic

परजीवी 3) Bacterial

बहॴकटीररयि

4)Viral

वायरि

Correct Answer Bacterial

An infection with salmonella bacteria

commonly caused by contaminated food

or water

Symptoms include diarrhoea fever

chills and abdominal pain

Q172 is a condition in which there is a

deficiency of red cells or of haemoglobin

in the blood

mdashmdash-

एकनसिनतहहॴनजसमहॳरकतमिािकह८लिकाओकीयाहीमह८गिह८बबनकीकमीहह८तीहहॴ 29-Jan-2017

Options

1) Albinism

एनमबननजम

2) Propyria

परह८पीररया 3) Anaemia

एनीलमया 4)Keloid disorder

कहॳ िह८इिडिसओिर

Correct Answer Anaemia

Q173 Ananas comosus is the scientific

name of

Options

अनानासकह८मह८सस mdashmdashmdashndash

कावहॴजञाननकनामहहॴ 29-Jan-2017

1) Custard Apple

सीताफि

2) Pineapple

पाइनएपपि

3) Bamboo

बास

4)Pomegranate

अनार

Correct Answer Pineapple

Q174 Which organ produces insulin

कह९नसाअगइनसलिनपहॴदाकरताहहॴ 29-Jan-

2017

Options

1) Liver

यकत

2) Thyroid gland

िायराइिगरिी 3) Spleen

पिीहा 4)Pancreas

अगरयिय

Correct Answer Pancreas

Q175 Which of the following disease is

not caused by water pollution

नननननलिखितमसहॳकह९नसारह८गपानीकहॳ परदषणकहॳकारणनहीहह८ता

29-Jan-2017

Options

1) Cholera

हहॴजा 2) Typhoid

F A C E B O O K

P A G E h t t p w w w f a c e b o o k c o m s s c m e n t o r s o f f i c i a l P a g e | 41

FOR MORE UPDATES AND MORE MATERIAL DO LIKE OUR FACEBOOK PAGE httpwwwfacebookcomsscmentorsofficial

टाइफाइि

3) Asthma

दमा 4)Diarrhoea

दसत

Correct Answer Asthma

Q176 Ocimum tenuiflorum is the

scientific name of

ओलिलममटहॳयईफिह८रमइसकावहॴजञाननकनाम mdash

ndash हहॴ 30-Jan-2017

Options

1) Neem

नीम

2) Mango

आम

3) Babul

बबि

4)Tulsi

तिसी Correct Answer Tulsi

Q177 Which gland secretes bile a

digestive fluid

कह९नसीगरिीपपतत एकपाचनतरिपरदािय सरापवतकरतीहहॴ 30-Jan-2017

Options

1) Pancreas

अगनयािय

2) Liver

यकत

3) Thyroid

िायराइि

4) Testes

टहॳनसटस

Correct Answer liver

Q178 In which of the following the

dominant phase is Gametophyte

नननननलिखितमसहॳककसकहॳ परमिचरणयगमकह८दपवधद (Gametophyte)हहॴ 30-Jan-2017

Options

1) Bryophyta

िायह८फाइटा 2) Pteridophyta

टहॳररिह८फाइटा 3) Gymnosperms

नजननह८सपमय 4) Angiosperms

एननजयह८सपमय Correct Answer Bryophyta

Q179 Anaerobic respiration refers to

which of the following

नननननलिखितमसहॳककसहॳअवायवीयशवसनकहाजाताहहॴ

30-Jan-2017

Options

1) Respiration without Oxygen

ऑकसीजनकहॳ बबनाशवसन

2) Respiration with Oxygen

ऑकसीजनकहॳ सािशवसन

3) Respiration without CO2

काबयनिायऑकसाइिकहॳ बबनाशवसन

4) Respiration with CO2

काबयनिायऑकसाइिकहॳ सािशविन

Correct Answer Respiration without

Oxygen

Q180 Which type of pathogen causes

the waterborne disease Cholera

ककसपरकारकारह८गजनकजिजननतरह८गहहॴजाकाकारणबनताहहॴ

30-Jan-2017

Options

1) Algal

िहॴवालियहॳ

F A C E B O O K

P A G E h t t p w w w f a c e b o o k c o m s s c m e n t o r s o f f i c i a l P a g e | 42

FOR MORE UPDATES AND MORE MATERIAL DO LIKE OUR FACEBOOK PAGE httpwwwfacebookcomsscmentorsofficial

2) Bacterial

बहॴकटीररयि

3) Protozoan

परह८टह८जआ

4) Viral

वायरि

Correct Answer Bacterial

Q181 To which class does

Oxyreductases transferases hydrolases

belong

ओकसीररिकटहॳसटरासफरहॳजहॳस

हाइडरह८िहॳसहॳसककसवगयमआतहॳहहॴ 30-Jan-2017

Options

1) Hormones

हारमोस

2) Enzymes

एजाइनस

3) Proteins

परह८टीनस

4) Vitamins

पवटालमनस

Correct Answer Enzymes

Q182 Which of the following is not true

about Gymnosperms

ननननमसहॳकह९नसीबातअनावतबीजीकहॳ बारहॳमसचनहीहहॴ 30-Jan-2017

Options

1) Dominant phase is saprophytes

परमिचरणसहॳपरह८फाइटसहह८ताहहॴ 2) Vascular bundles are absent

सवहनीबििअनपनसितहह८ताहहॴ 3) spores are heterospores

बीजाणहहॳटहॳरह८सपह८रसहह८तहॳहहॴ 4) Flowers are absent

फिअनपनसितहह८तहॳहहॴ

Correct Answer Vascular bundles are

absent

Q183 The name of first mammal clone sheep is

भहॳड़कीपरिमसतनपायीपरनत प (किह८न)

कानामहहॴ 30-Jan-2017

Options

1) Noori

नरी 2) Dolly

िॉिी 3) Louise

िसी 4)Durga

दगाय Correct Answer Dolly

Q184 Which type of pathogen causes

the water-borne disease Typhoid fever

ककसपरकारकारह८गजनकजिजननतरह८गटाइफाइिबिारकाकारणबनताहहॴ 30-Jan-2017

Options

1) Algal

िहॴवािीय

2) Parasitic

परजीवी 3) Protozoan

परह८टह८जनअन

4)Bacterial

बहॴकटीररयि

Correct Answer Bacterial

Q185 In which part of the cell are

proteins made

कह८लिकाकहॳ ककसटहससहॳमपरह८टीनबनायाजाताहहॴ

31-Jan-2017

Options

1) Reticulum

रहॳटटकिम

F A C E B O O K

P A G E h t t p w w w f a c e b o o k c o m s s c m e n t o r s o f f i c i a l P a g e | 43

FOR MORE UPDATES AND MORE MATERIAL DO LIKE OUR FACEBOOK PAGE httpwwwfacebookcomsscmentorsofficial

2) Golgi apparatus

गह८मजीएपहॳरहॳटस

3) Ribosomes

ररबह८सह८नस

4) Lysosome

िायसह८सह८नस

Correct Answer ribosomes

Proteins are produced by stringing

amino acids together in the order

specified by messenger RNA strands

that were transcribed from DNA in the

cell nucleus The process of synthesizing

a protein is called translation and it

occurs on ribosomes in the cytoplasm of

a cell

Q186 Polio is a disease caused by which

of the following

नननननलिखितमसहॳपह८लियह८कीबबमारह८हह८नहॳकाकारणकयाहहॴ

31-Jan-2017

Options

1) Bacteria

बहॴकटीररयि

2) Mosquito

मचछर

3) Virus

वायरस

4) Cockroach

नतिच हॳ Correct Answer Virus

Polio or poliomyelitis is a crippling and

potentially deadly infectious disease It

is caused by the poliovirus

Q187 ndash Hay fever is a sign of which of

the following

हहॳकफवरनननननलिखितमसहॳककसकाएकसकहॳ तहहॴ

31-Jan-2017

Options

1) Old Age

वदावसिा 2) Malnutrition

कपह८सण

3) Allergy

एिनजय 4) Over Work

अतयचधककाययकरना Correct Answer Allergy

Q188 How many chromosomes does a

human cell contain

एकमानवकह८लिकामककतनहॳगणसतरहह८तहॳहहॴ

29-Jan-2017

Options

1) 6

2) 26

3) 46

4) 66

Correct Answer 46

In humans each cell normally contains

23 pairs of chromosomes for a total of

46 Twenty-two of these pairs called

autosomes look the same in both males

and females The 23rd pair the sex

chromosomes differ between males and

females

Q189 Which of the following is not true

about Bryophyta

ननननमसहॳकह९नसीबातिायह८फाइटकहॳ बारहॳमसचनहीहहॴ 31-Jan-2017

Options

1) Dominant phase is gametophytes

परमिचरणगहॳलमतह८फाइटसहह८ताहहॴ 2) Main plant body is haploid

पह९धहॳकामखयिरीरअगखणतहह८ताहहॴ 3) Spores are homospores

बीजाणहह८मह८सफह८रसहह८तहॳहहॴ 4) Flowers are present

फिमह८जदहह८तहॳहहॴ Correct Answer Flowers are present

F A C E B O O K

P A G E h t t p w w w f a c e b o o k c o m s s c m e n t o r s o f f i c i a l P a g e | 44

FOR MORE UPDATES AND MORE MATERIAL DO LIKE OUR FACEBOOK PAGE httpwwwfacebookcomsscmentorsofficial

Q190 Which aquatic animal has

trailing tentacles

ककसजिीयजानवरकहॳ पीछहॳचिनहॳवािहॳटहॳटकिसहह८तहॳहहॴ

31-Jan-2017

Options

1) Sea horse

समदरीघह८िा 2) Corals

मगा 3) Jelly fish

जहॳिीमछिी 4) Star fish

तारामछिी Correct Answer Jelly fish

Jellyfish with its umbrella-shaped bell

and trailing tentacles

Q191 Which type of pathogen causes

the water-borne disease Poliomyelitis

(Polio)

ककसपरकारकारह८गजनकजिजननतरह८गपह८लियह८मायहॳटटस (पह८लियह८) काकारणहहॴ 31-Jan-

2017

Options

1) Parasitic

परजीवी 2) Algal

िहॴवालिय

3) Viral

वायरि

4) Bacterial

बहॴकटीररयि

Correct Answer Viral

Q192 The outer white part of the eye

that protects the inner structures is

आािकाबाहरीसफहॳ दटहससाजह८आतररकसरचनाओकीरकषाकरताहहॴ वह mdashmdashmdash हहॴ 31-Jan-

2017

Options

1) Iris

आयररस

2) Sclera

सकिहॳरा 3) Retina

रहॳटटना 4) Cornea

कह८ननयया Correct Answer Sclera

Q193 Proteins are made up of

परह८टीनकाननमायण mdashndash सहॳहह८ताहहॴ 31-Jan-2017

Options

1) Amino acids

एलमनह८अनि

2) Fatty acids

वसायकतअनि

3) Glucose

गिकह८ज

4)Nucleotides

नयनकियह८टाईिस

Correct Answer Amino acids

Q194 Moringa Oleifera is the scientific

name of

मह८ररगओलिफहॳ रा mdashmdashndash कावहॴजञाननकनामहहॴ 31-Jan-2017

Options

1) Banyan

बरगद

2) Gulmohar

गिमह८हर

3) Amla

आमिा

F A C E B O O K

P A G E h t t p w w w f a c e b o o k c o m s s c m e n t o r s o f f i c i a l P a g e | 45

FOR MORE UPDATES AND MORE MATERIAL DO LIKE OUR FACEBOOK PAGE httpwwwfacebookcomsscmentorsofficial

4) Drumstick

डरमनसटक

Correct Answer Drumstick

Q195 Kidney stones are composed of

गदकीपिरी mdashndash सहॳबनीहह८तीहहॴ 1-Feb-2017

Options

1) Calcium Oxalate

कहॴ नमसयमओकजहॳिहॳट

2) Sodium Chloride

सह८डियमकिह८राइि

3) Magnesium Nitrate

महॳनगनलियमनाइतटरहॳट

4) Calcium Bicarbonate

कहॴ नमियमबायकबोनहॳट

Correct Answer Calcium Oxalate

Q196 ndash Which of the following is not

true about Angiosperms

ननननमसहॳकह९नसीबातआवतबीजीकहॳ बारहॳमसचनहीहहॴ 1-Feb-2017

Options

1) Dominant phase is gametophytes

परमिचरणगहॳलमतह८फाइटहह८ताहहॴ 2) Vascular bundles are present

सवहनीबििमह९जदहह८ताहहॴ 3) Spores are heterospores

बीजाणहहॳटहॳरह८सपह८रसहह८तहॳहहॴ 4) Seeds are covered

बीजढकहॳ हह८तहॳहहॴ Correct Answer Dominant phase is

gametophytes

Q197 All of the following are excretory

(waste) products of animals except

नननननलिखितमसहॳककसएककह८छह८ड़करअनयसभीपराखणयोदवाराउतसनजयतपदाियहहॴ 1-Feb-

2017

Options

1) Uric Acid

यररकएलसि

2) Ammonia

अमह८ननया 3) Carbohydrates

काबोहाइडरहॳट

4) Urea

यररया Correct Answer Carbohydrates

In animals the main excretory products

are carbon dioxide ammonia (in

ammoniotelics) urea (in ureotelics) uric

acid (in uricotelics) guanine (in

Arachnida) and creatine

Q198 RNA is a polymeric molecule

What does RNA stand for

आरएनइएएकबहिकआणहहॴ इसकाकापवय पकयाहहॴ 1-Feb-2017

Options

1) Rado Nuclear Acid

रािह८नयनकियरएलसि

2) Ribo Nucleic Acid

राइबह८नयनकिकएलसि

3) Rhino Nuclear Acid

हाइनह८नयनकियरएलसि

4) Resto Nucleus Acid

रहॳसटह८नयकिीयसएलसि

Correct Answer Ribo Nucleic Acid

Q199 Which organ does detoxification

and produces chemicals needed for

digestion

कह९नसाअगपवषहरणकरताहहॴऔरपाचनकहॳ लिएआवशयकरसायनोकह८पहॴदाकरताहहॴ 1-Feb-

2017

Options

1) Salivary glands

िारगरचिया 2) Pancreas

अगनयािय

F A C E B O O K

P A G E h t t p w w w f a c e b o o k c o m s s c m e n t o r s o f f i c i a l P a g e | 46

FOR MORE UPDATES AND MORE MATERIAL DO LIKE OUR FACEBOOK PAGE httpwwwfacebookcomsscmentorsofficial

3) Thyroid gland

िायराइिगरिी 4) Liver

यकत

Correct Answer Liver

Q200 Psidium guajava is the scientific

name of

लसडियमगआजावा mdashmdash कावहॴजञाननकनामहहॴ 1-

Feb-2017

Options

1) Guava

अम द

2) Mango

आम

3) Bamboo

बास

4) Jack fruit

कटहि

Correct Answer Guava

Q201 Which drug is used as a Blood

Thinner

चधरकह८पतिाकरनहॳकहॳ पमककसदवाकापरयह८गककयाजाताहहॴ

1-Feb-2017

Options

1) Warfarin

वाफर न

2) Tramadol

टरहॳमािह८ि

3) Azithromycin

एनजरह८मायलसन

4) Hydralazine

हाइडरह८िहॳनजन

Correct Answer Warfarin

Q202 Which of the following disease is

caused due to the deficiency of protein

परह८टीनकीकमीकहॳ कारणनननननलिखितमसहॳकह९नसारह८गहह८ताहहॴ 1-Feb-2017

Options

1) Arthritis

गटठया 2) Kwashiorkor

कािीओकय र

3) Goitre

गाइटर

4) Night Blindness

रतह९चध

Correct Answer Kwashiorkor

Q203 A is species of plant that has

adapted to survive in an environment

with little liquid water

mdashmdashndashपह९धहॳकीएकऐसहॳऐसहॳपरजानतहहॴ नजसनहॳकमपानीवािहॳवातावरणमजीपवतरहनहॳकहॳलिएअनकिनहहॴ 1-Feb-2017

Options

1) Xerophyte

म दपवद

2) Hydrophyte

जिीयपादप

3) Mesophyte

समह८दपवद

4) Thallophyte

िहॴिह८फाइटा Correct Answer xerophyte

xerophyte is a species of plant that has

adapted to survive in an environment

with little liquid water such as a desert

or an ice- or snow-covered region in the

Alps or the Arctic

Mesophytes are terrestrial plants which

are adapted to neither a particularly

dry nor particularly wet environment

An example of a mesophytic habitat

would be a rural temperate meadow

F A C E B O O K

P A G E h t t p w w w f a c e b o o k c o m s s c m e n t o r s o f f i c i a l P a g e | 47

FOR MORE UPDATES AND MORE MATERIAL DO LIKE OUR FACEBOOK PAGE httpwwwfacebookcomsscmentorsofficial

which might contain goldenrod clover

oxeye daisy and Rosa multiflora

thallophyte any of a group of plants or

plantlike organisms (such as algae and

fungi) that lack differentiated stems

leaves and roots and that were formerly

classified as a primary division

(Thallophyta) of the plant kingdom

Q204 How many types of teeth are

there in humans

मनषयोमककतनहॳपरकारकहॳ दातहह८तहॳहहॴ

1-Feb-2017

Options

1) 4

2) 5

3) 2

4) 3

Correct Answer 4

teeth -Humans have four types of

teethincisors canines premolars and

molars each with a specific function

The incisors cut the food the canines

tear the food and the molars and

premolars crush the food

Q205 Carica papaya is the scientific name of

कहॴ ररकापपाया mdashmdashndash कावहॴजञाननकनामहहॴ 2-

Feb-2017

Options

1) Peepal

पीपि

2) Papaya

पपीता 3) Tamarind

इमिी 4) Drumstick

ढह८िकाछड़ी Correct Answer Papaya

Q206 Muscles get tired when there is

shortfall of

जब mdashndash कीकमीहह८तीहहॴतबपहॳिीयिकजातीहहॴ 2-Feb-2017

Options

1) Lactic acid

िहॴनकटकएलसि

2) Na+ ions

Na+ आयन

3) ATP

एटीपी 4) Sulphates

समफहॳ टस

Correct Answer ATP

ATP is the energy source muscle fibers

use to make muscles contract

muscle tissuersquos main source of energy

called adenosine triphosphate or ATP

As your muscles use up this energy

source they become tired and fatigued

Oxygen is the key ingredient that helps

create new ATP to replenish the burned

up ATP in your muscles

Q207 Artocarpus integra is the

scientific name of आटह८कापयसइटीगरा mdashmdashmdash कावहॴजञाननकनामहहॴ 2-Feb-2017

Options

1) Guava

अम द

2) Pineapple

अनानास

3) Silver Oak

लसमवरओक

4) Jack fruit

कटहि

Correct Answer Jack fruit

Q208 Which organ stores fat soluble

vitamins

कह९नसाअगवसामघिनिीिपवटालमनह८काभिाराकरताहहॴ

2-Feb-2017

F A C E B O O K

P A G E h t t p w w w f a c e b o o k c o m s s c m e n t o r s o f f i c i a l P a g e | 48

FOR MORE UPDATES AND MORE MATERIAL DO LIKE OUR FACEBOOK PAGE httpwwwfacebookcomsscmentorsofficial

Options

1) Blood

रकत

2) Skin

तवचा 3) Liver

यकत

4) Pancreas

अगनयािय

Correct Answer Liver

Q209 Which disease is caused due to

deficiency of Iodine

आयह८िीनकहॳ कारणकह९नसारह८गहह८ताहहॴ 2-Feb-2017

Options

1) Rickets

ररकहॳ टस

2) Scurvy

सकवी 3) Goitre

गणमािा 4) Growth retardation

पवकासका कना Correct Answer Goitre

rickets A softening and weakening of

bones in children usually due to

inadequate vitamin D

Q210 Grevillea Robusta is the scientific name of

गरहॳपवलियारह८बसटा mdashmdashmdash- कापवजञाननकनामहहॴ 2-Feb-2017

Options

1) Peepal

पीपि

2) Teak

सागह९न

3) Silver Oak

लसमवरओक

4) Jack fruit

कटहि

Correct Answer Silver Oak

Q211 When a Cuttlefish is described as a Molluscs it is at which level of

classification

जबएककटिकफिकह८एकमह८िसकाकहॳ पमवखणयतककयाजाताहहॴतबयहॳवगीकरणकहॳ ककससतरपहॳनसितहहॴ 2-Feb-2017

Options

1) Class

वगय 2) Order

िम

3) Family

पररवार

4) Phylum

सघ

Correct Answer Phylum

Q212 Bambusa dendrocalmus is the

scientific name of बानबसािहॳडराकामस mdashmdashmdash कावहॴजञाननकनामहहॴ 3-Feb-2017

Options

1) Banyan

बरगद

2) Papaya

पपीता 3) Bamboo

बास

4) Pomegranate

अनार

Correct Answer Bamboo

Q213 Acinonyx Jubatus is the scientific name of

एलसनह८ननकसजयबहॳटस mdashmdashmdash

कावहॴजञाननकनामहहॴ 3-Feb-2017

F A C E B O O K

P A G E h t t p w w w f a c e b o o k c o m s s c m e n t o r s o f f i c i a l P a g e | 49

FOR MORE UPDATES AND MORE MATERIAL DO LIKE OUR FACEBOOK PAGE httpwwwfacebookcomsscmentorsofficial

Options

1) Bear

भाि 2) Horse

घह८िा 3) Cheetah

चीता 4) Zebra

जहॳिा Correct Answer Cheetah

Q214 The pale yellow colour of urine is

due to the presence of which pigment

मतरकाफीकापीिारगरगदरयकहॳ उपनसिनतकहॳ कारणहह८ताहहॴ

3-Feb-2017

Options

1) Urochrome

यरह८िह८म

2) Urophyll

यरह८कफि

3) Chlorophyll

किह८रह८कफि

4) Chloroplast

किह८रह८पिासट

Correct Answer Urochrome

Q215 Which of the following constitute

to form a gene

नननननलिखितमसहॳकह९नसीचीज़एकजीनकागठनकरतीहहॴ

3-Feb-2017

Options

1) Polynucleotides

पह८िीनयनकियह८टाईडस

2) Hydrocarbons

हाइडरह८काबोस

3) Lipoproteins

िाईपह८परह८टीनस

4) Lipids

लिपपडस

Correct Answer Polynucleotides

Polynucleotide molecule is a biopolymer

composed of 13 or more nucleotide

monomers covalently bonded in a chain

DNA (deoxyribonucleic acid) and RNA

(ribonucleic acid) are examples of

polynucleotides with distinct biological

function

Q216 Vertebrates belongs to the

phylum

रीढ़कीहडिीवािहॳपराणी mdashmdashmdash

परजानतकहॳ अतगायतआतहॳहहॴ 3-Feb-2017

Options

1) Arthropoda

आरह८पह८ड़ा 2) Annelida

एननलििा 3) Cnidaria

ननिहॳररया 4) Chordata

कह८िटा Correct Answer Chordata

Q217 Punica granatum is the scientific name of

पननकगरहॳनहॳटस mdashmdashmdash कावहॴजञाननकनामहहॴ 3-Feb-2017

Options

1) Custard Apple

सीताफि

2) Gulmohar

गिमह८हर

3) Silver Oak

लसमवरओक

4) Pomegranate

अनार

Correct Answer Pomegranate

F A C E B O O K

P A G E h t t p w w w f a c e b o o k c o m s s c m e n t o r s o f f i c i a l P a g e | 50

FOR MORE UPDATES AND MORE MATERIAL DO LIKE OUR FACEBOOK PAGE httpwwwfacebookcomsscmentorsofficial

Q218 Between a tiger and an monkey

which of the following is different

एकबाघऔरबदरकहॳ बीचनननननलिखितमसहॳकह९नसीबातअिगहहॴ 3-Feb-2017

Options

1) Kingdom

राजय

2) Phylum

जानत

3) Order

िम

4) Class

वगय Correct Answer order

Q219 The artificial heart was invented by

कबतरमहदयका mdashmdashmdash

दवाराअपवषकारककयागयािा 3-Feb-2017

Options

1) Muhammad Yunus

महनमदयनस

2) Linus Yale Jr

िाइनसयहॳिजय

3) Gazi Yasargil

गाजीयासचगयि

4) Paul Winchell

पह९िपवमकि Correct Answer Paul Winchell

Q220 Tamarindus indica is the

scientific name of

टहॳमररनडसइडिका mdashmdash कावहॴजञाननकनामहहॴ 7-

Feb-2017

Options

1) Neem

नीम

2) Pineapple

अनानास

3) Tamarind

इमिी 4)Chiku

चीक

Correct Answer Tamarind

Q221 In eukaryotic cells synthesis of

RNA takes place in the

यकहॳ योटटककह८लिकाओमआरएनएकासशिहॳषण

mdashndash महह८ताहहॴ 7-Feb-2017

Options

1) Mitochondria

माईटह८कोडडरया 2) Centrioles

सटरीयह८मस

3) Ribosomes

ररबह८सह८नस

4) Nucleus

नयनकियस

Correct Answer nucleus

eukaryotic cell -Transcription is the

process of synthesizing ribonucleic acid

(RNA)Synthesis takes place within the

nucleus of eukaryotic cells or in the

cytoplasm of prokaryotes and converts

the genetic code from a gene in

deoxyribonucleic acid ( DNA ) to a

strand of RNA that then directs

proteinsynthesis

Q222 _________is caused by parasites

of the Plasmodium genus

पिाजमह८डियमजातीकहॳ परजीवी mdash- कहॳ कारणहहॴ 7-Feb-2017

Options

1) Dysentery

पहॳचचि

2) Malaria

मिहॳररया 3) Chickenpox

F A C E B O O K

P A G E h t t p w w w f a c e b o o k c o m s s c m e n t o r s o f f i c i a l P a g e | 51

FOR MORE UPDATES AND MORE MATERIAL DO LIKE OUR FACEBOOK PAGE httpwwwfacebookcomsscmentorsofficial

चहॳचक

4) Herpes

हहॳपपयस

Correct Answer Malaria

Q223 Carotene in fruits and vegetables

gives it which color

फिह८औरसनलजयोमनसितकहॳ रह८टीनउनहकह९नसारगपरदानकरताहहॴ 7-Feb-2017

Options

1) Green

हरा 2) Pink

गिाबी 3) Orange

नारगी 4) Blue

नीिा Correct Answer Orange

Q224 Equus Caballus is the scientific

name of

एकवसकहॴ बहॳिस mdashmdashndash कापवजञाननकनामहहॴ 7-Feb-2017

Options

1) Horse

घह८िा 2) Zebra

ज़हॳिा 3) Donkey

गधा 4) Buffalo

भस

Correct Answer Horse

Q225 Elapidae Naja is the scientific name of

एिीपीिीनाजा mdashmdash- कावहॴजञाननकनामहहॴ 8-Feb-2017

Options

1) Cobra

कह८बरा 2) Elephant

हािी 3) Eagle

ग ि

4) Owl

उमि Correct Answer Cobra

Q226 Which disease is caused due to

deficiency of Iron

िह८हकीकमीकहॳ कारणकह९नसारह८गहह८ताहहॴ 8-Feb-

2017

Options

1) Beriberi

बहॳरीबहॳरी 2) Tetany

टहॳटनी 3) Kwashiorkor

कवािीऔरकर

4) Anaemia

रकतामपता Correct Answer Anaemia

Beriberi is a disease caused by a vitamin

B-1 deficiency also known as thiamine

deficiency

Tetany can be the result of an

electrolyte imbalance Most often itrsquos a

dramatically low calcium level also

known as hypocalcemia Tetany can also

be caused by magnesium deficiency or

too little potassium Having too much

acid (acidosis) or too much alkali

(alkalosis) in the body can also result in

tetany

Kwashiorkor also known as

ldquoedematous malnutrition It is a form of

malnutrition caused by a lack of protein

in the diet

Anaemia means that you have fewer red

blood cells than normal or you have less

F A C E B O O K

P A G E h t t p w w w f a c e b o o k c o m s s c m e n t o r s o f f i c i a l P a g e | 52

FOR MORE UPDATES AND MORE MATERIAL DO LIKE OUR FACEBOOK PAGE httpwwwfacebookcomsscmentorsofficial

haemoglobin than normal in each red

blood cell

Q227 is a leaf where the leaflets are

arranged along the middle vein

mdashndashएकपततीहहॴजहापतरकह८कीरचनाक ररयालिराकहॳ आसपासहह८तीहहॴ 8-Feb-2017

Options

1) Pinnately compound leaf

पपनहॳटिीसयकतपतती 2) Palmately compound leaf

पामहॳटिीसयकतपतती 3) Compound leaf

सयकतपतती 4) Simple leaf

साधारणपतती Correct Answer Pinnately compound

leaf

Q228 Haustoria or sucking roots are

found in which of the following

हह८सटह८ररयायाचसनहॳवािीजड़हॳनननननलिखितमसहॳककसमपाईजातीहहॴ 8-Feb-2017

Options

1) Wheat

गहॳह

2) Mango

आम

3) Chestnut

चहॳसटनट

4) Cuscuta

कसकयटा Correct Answer Cuscuta

Haustorial roots -The roots of parasitic

plants which penetrate into the host

tissues to absorb nourishment are

called haustorial roots hellip Also known as suckingor parasitic roots

Q229 Equs Asinus is the scientific name

of

एकवसएलसनस mdashmdashndash कावहॴजञाननकनामहहॴ 8-

Feb-2017

Options

1) Donkey

गधा 2) Cow

गाय

3) Deer

टहरन

4) Kangaroo

कगा

Correct Answer Donkey

Q230 Ficus benghalensis is the scientific name of

फाईकसबहॳनगहॳिहॳलसस mdashndash कापवजञाननकनामहहॴ 8-Feb-2017

Options

1) Banyan

बरगद

2) Pineapple

अनानास

3) Babul

बबि

4) Tulsi

तिसी Correct Answer Banyan

Q231 Equus burchellii is the scientific name of

एकवसबचिी mdashmdash- कापवजञाननकनामहहॴ 8-Feb-2017

Options

1) Horse

घह८िा 2) Zebra

जहॳिा 3) Buffalo

F A C E B O O K

P A G E h t t p w w w f a c e b o o k c o m s s c m e n t o r s o f f i c i a l P a g e | 53

FOR MORE UPDATES AND MORE MATERIAL DO LIKE OUR FACEBOOK PAGE httpwwwfacebookcomsscmentorsofficial

भस

4) Ass

गधा Correct Answer Zebra

Page 36: COMPILATION OF ALL 72 SETS OF BIOLOGY SSC CHSL-2016 · OF BIOLOGY SSC CHSL-2016 PREPARED BY : SSC MENTORS BIOLOGY SPECIAL . F A C E B O O K P A G E : h t t p : / / w w w . f a c e

F A C E B O O K

P A G E h t t p w w w f a c e b o o k c o m s s c m e n t o r s o f f i c i a l P a g e | 35

FOR MORE UPDATES AND MORE MATERIAL DO LIKE OUR FACEBOOK PAGE httpwwwfacebookcomsscmentorsofficial

घरहॳिमकिी Correct Answer Lizard

Q147 Which type of pathogen causes

the water-borne disease SARS (Severe

Acute Respiratory Syndrome)

ककसपरकािकारह८गज़नकजिजननतबीमारीसासयकाकारणबनताहहॴ 25-Jan-2017

Options

1) Viral

वायरि

2) Parasitic

परजीवी 3) Protozoan

परह८टह८जअन

4) Bacterial

बहॴकटीररयि

Correct Answer Viral

Q148 Which of the following organs

produces the enzyme lipase

नननननलिखितमसहॳकह९नसाअगिायपहॳजएजाइमउतपननकरताहहॴ 25-Jan-2017

Options

1) Pancreas

अगनयािय

2) Large Intestine

बड़ीआत

3) Liver

नजगर

4) Small Intestine

छह८टीआत

Correct Answer Pancreas

Q149 A is a long internode forming the

basal part or the whole of a peduncle

एक mdashmdash- एकिबाइटरनह८िहहॴ जह८ननचिाटहससायासनपणयिठिबनताहहॴ 25-

Jan-2017

Options

1) Rhizome

परकद

2) Rachis

महॳ दि

3) floral axis

पषपअकष

4) Scape

भगदड़

Correct Answer scape

Q150 ndash Which of the following

organisms are considered to be both

Living and Non-living

नननननलिखितमसहॳकह९नसहॳजीवाणकह८जीपवतऔरअजीपवतमानाजाताहहॴ

25-Jan-2017

Options

1) Bacteria

बहॴकटीररया 2) Fungi

कवक

3) Algae

िहॴवाि

4)Virus

वायरस

Correct Answer Virus

They are considered to be living as they

possess a protein coat as a protective

covering DNA as the genetic material

etc

They are said to be non-living as they

can be crystallised and they survive for

billions of years They can tolerate high

temperatures freezing cold

temperatures ultra-violet radiations etc

Q151 Deficiency of fluorine causes

which of the following

फिह८ररनकीकमीकहॳ कारणनननननलिखितमसहॳकयाहह८ताहहॴ

F A C E B O O K

P A G E h t t p w w w f a c e b o o k c o m s s c m e n t o r s o f f i c i a l P a g e | 36

FOR MORE UPDATES AND MORE MATERIAL DO LIKE OUR FACEBOOK PAGE httpwwwfacebookcomsscmentorsofficial

27-Jan-2017

Options

1) Dental Caries

िटिकहॴ ररज

2) Scurvy

सकवरी 3) Anaemia

रकतामपता 4) Arthritis

गटठया Correct Answer Dental Caries

Q152 In a Punnett Square with the

cross AaBb x AaBb how many Aabb

genotypes would be created

पनहॳटसककायरमिह८स AaBb x AaBb कहॳ साि

ककतनहॳ Aabb जीनह८टाइपबनगहॳ 27-Jan-2017

Options

1) 1

2) 8

3) 2

4) 3

Correct Answer 2

Q153 Which of the following is the

Controlling Center of the Cell

नननननलिखित म सहॳ कह८लिकाका ननयतरण

क दर कह९न हहॴ

27-Jan-2017

Options

1) Nucleus

क दर

2) Plasma

पिाजमा 3) Lysosome

िायसह८सह८म

4) Chromosome

िह८मह८सह८म

Correct Answer Nucleus

The control centre of the cell is the

nucleus in eukaryotic cells The nucleus

contains genetic material in the form of

DNA

Q154 Myopia affects which of the

following organs

मायह८पपयानननननलिखितअगह८मसहॳककसहॳपरभापवतकरताहहॴ

25-Jan-2017

Options

1) Heart

हदय

2) Skin

तवचा 3) Eyes

आािहॳ 4)Mouth

मह

Correct Answer Eyes

Q155 Which of the following bears

flowers

नननननलिखितमसहॳकह९नफिधारणकरताहहॴ

25-Jan-2017

Options

1) Bryophyta

िायह८फाइटा 2) Pteridophyta

टहॳरीिह८फाईटा 3) Gymnosperms

नजननह८सपमय 4)Angiosperms

एननजयह८सपमय Correct Answer Angiosperms

Q156 Oxygenated blood flows out of the

heart through the

ऑकसीजनयकतरकत mdashmdashmdash

कहॳ माधयमसहॳहदयकहॳ बाहरबहताहहॴ 25-Jan-2017

F A C E B O O K

P A G E h t t p w w w f a c e b o o k c o m s s c m e n t o r s o f f i c i a l P a g e | 37

FOR MORE UPDATES AND MORE MATERIAL DO LIKE OUR FACEBOOK PAGE httpwwwfacebookcomsscmentorsofficial

Options

1) Aorta

महाधमनी 2) pulmonary artery

फहॳ फड़हॳकीधमनी 3) vena cava

वहॳनाकावा 4)Atrium

चह९क

Correct Answer aorta

Q157 Blood leaving the liver and

moving towards the

heart has a higher concentration of

नजगरसहॳननकिकरहदयकीतरफजानहॳवािहॳरकतम mdashmdashmdashmdash कीउचचसादरताहह८तीहहॴ 27-Jan-2017

Options

1) Lipids

लिपपडस

2) Urea

यररया 3) Bile Pigments

पपततकहॳ रगकरण

4) Carbon dioxide

काबयनिायऑकसाइि

Correct Answer Bile Pigments

Urea is nitrogen containing substance

which is produced in the liver in order

to deal with excess amino-acids in the

body As urea is produced it leaves the

liver in the blood stream and passes via

the circulatory system to all parts of the

body

Q158 Bulb is a modification of which

part of a plant

बमबएकपह९धहॳकहॳ ककसटहससहॳकाएक पातरणहह८ताहहॴ 27-Jan-2017

Options

1) The root

जड़

2) The stem

तना 3) The radicle

मिाकर

4)The fruit

फि

Correct Answer The stem

Q159 Which of the following carries

blood away from the heart to different

body parts

इनमहॳसहॳकह९नरकतकह८हदयसहॳिरीरकहॳ पवलभननअगह८तकिहॳजातीहहॴ

27-Jan-2017

Options

1) Arteries

धमननया 2) Nerves

तबतरहाए

3) Capillaries

कहॳ लिकाए

4)Veins

नसहॳ Correct Answer Arteries

Q160 The series of processes by which

nitrogen and its compounds are

interconverted in the environment and

in living organisms is called

27-Jan-2017

Options

1)Absorption of Nitrogen

2)Ammonification

3)Nitrogen Fixation

4)Nitrogen Cycle

Correct Answer Nitrogen Cycle

Ammonification or Mineralization is

performed by bacteria to convert

organic nitrogen to ammonia

F A C E B O O K

P A G E h t t p w w w f a c e b o o k c o m s s c m e n t o r s o f f i c i a l P a g e | 38

FOR MORE UPDATES AND MORE MATERIAL DO LIKE OUR FACEBOOK PAGE httpwwwfacebookcomsscmentorsofficial

Nitrification can then occur to convert

the ammonium to nitrite and nitrate

Nitrogen fixation is a process by which

nitrogen in the Earthrsquos atmosphere is

converted into ammonia (NH3) or other

molecules available to living organisms

Q161 BCG vaccine is given to protect

from which of the following

बीसीजीकाटटकानननननलिखितमसहॳककसकहॳ बचावकहॳ लिएटदयाजातहहॴ

27-Jan-2017

Options

1) Jaundice

पीलिया 2) Anaemia

रकतमपता 3) Tuberculosis

कषयरह८ग

4) Polio

पह८लियह८ Correct Answer Tuberculosis

Q162 Parallel venation is found in

समानतरवहॳनहॳिन mdashmdashmdash- मपायाजाताहहॴ 27-Jan-2017

Options

1) plants which are monocots

पह९धहॳजह८एकबीजपतरीहह८तहॳहहॴ 2) plants which have a dicot stem

वहॳपह९धहॳनजनकातनादपवदलियहह८ताहहॴ 3) plants with leaves similar to Tulsi

वहॳपह९धहॳनजनकीपनततयतिसीकीपनततयोकहॳ समानहह८तहॳहहॴ 4)plants with tap roots

टहॳप टवािहॳपह९धहॳ Correct Answer plants which are

monocots

Q163 The hardest part of the body is

िरीरकासबसहॳकठह८रभाग mdashndash हहॴ 27-Jan-2017

Options

1) Bones

हडडिय

2) Tooth Enamel

दातकहॳ इनहॳमि

3) Skull

िह८पड़ी 4) Spinal Cord

महॳ रजज

Correct Answer Tooth Enamel

Q164 Which type of pathogen causes

the waterborne disease E coli Infection

ककसपरकारकारह८गजननकजिजननतरह८गईकह८िाईसिमणकाकारणबनताहहॴ 27-Jan-2017

Options

1) Protozoan

परह८टह८जआ

2) Parasitic

परजीवी 3) Bacterial

बहॴकटीररयि

4)Viral

वायरि

Correct Answer Bacterial

Q165 The amount of blood filtered

together by both the kidneys in a 70 kg

adult male human in a minute is

70 की गरा वािहॳएकवयसकप षमएकलमनटमदह८नोगदकहॳदवाराएकसािचाबनीगयीरकतकीमातरहह८तीहहॴ 29-Jan-2017

Options

1) 1100 ml

1100 लमलि

2) 100 ml

F A C E B O O K

P A G E h t t p w w w f a c e b o o k c o m s s c m e n t o r s o f f i c i a l P a g e | 39

FOR MORE UPDATES AND MORE MATERIAL DO LIKE OUR FACEBOOK PAGE httpwwwfacebookcomsscmentorsofficial

100 लमलि

3) 1500 ml

1500 लमलि

4) 500 ml

500 लमलि

Correct Answer 1100 ml

Q166 Which feature of a plant helps to

distinguish a monocot from a dicot

पह९धहॳकीवहकह९नसीपविहॳषताहहॴजह८एकदपवदलियहॳऔरएकएकदिीयपह९धहॳसहॳभहॳदकरनहॳममददकरतीहहॴ 29-Jan-2017

Options

1) Pollination

परागम

2) Venation

वहॳनहॳिन

3) Vernation

वनिन

4) Aestivation

एसटीवहॳिहॳन

Correct Answer venation

Q167 The Mutation Theory was

proposed by

उतवररवतयनकालसदात mdashmdashndash

कहॳ दवरापरसतापवतककयाजाताहहॴ 29-Jan-2017

Options

1) Charles Lyell

चामसयलियहॳि

2) William Smith

पवलियमनसमि

3) Hugo De Vries

हयगह८िीराईस

4)Harrison Schmitt

हहॳरीसननसमट

Correct Answer Hugo De Vries

Q168 Which type of pathogen causes

the waterborne disease HepatitisA

ककसपरकारकहॳ रह८गजनकजिजननतरह८गहहॳपहॳटाइटटस-A काकारणबनताहहॴ

29-Jan-2017

Options

1) Parasitic

परजीवी 2) Viral

वायरि

3) Protozoan

परह८टह८जआ

4) Bacterial

बहॴकटीररयि

Correct Answer Viral

Q169 In a Punnett Square with the

cross AaBb x Aabb how many AaBb

genotypes would be created

पनहॳटसकवायरमिह८स AaBb x Aabb

कहॳ सािककतनहॳ AaBb जीनह८टाइपबनगहॳ 29-Jan-

2017

Options

1) 4

2) 1

3) 7

4) 6

Correct Answer 4

Q170 Arboreal Ateles is the scientific

name of

अिह८ररयिएटटलिस mdashmdashmdash कावहॴजञाननकनामहहॴ 29-Jan-2017

Options

1) Squirrel

चगिहरी 2) Sparrow

गह८रहॴया 3) Lizard

नछपकिी 4) Spider monkey

F A C E B O O K

P A G E h t t p w w w f a c e b o o k c o m s s c m e n t o r s o f f i c i a l P a g e | 40

FOR MORE UPDATES AND MORE MATERIAL DO LIKE OUR FACEBOOK PAGE httpwwwfacebookcomsscmentorsofficial

मकड़ीबदर

Correct Answer Spider monkey

Q171 Which type of pathogen causes

the waterborne disease Salmonellosis

ककसपरकारकारह८गाणजिजननतबीमारीसािमह८नहॳिह८लसज़काकारकहहॴ

29-Jan-2017

Options

1) Algal

िहॳवालियहॳ 2) Parasitic

परजीवी 3) Bacterial

बहॴकटीररयि

4)Viral

वायरि

Correct Answer Bacterial

An infection with salmonella bacteria

commonly caused by contaminated food

or water

Symptoms include diarrhoea fever

chills and abdominal pain

Q172 is a condition in which there is a

deficiency of red cells or of haemoglobin

in the blood

mdashmdash-

एकनसिनतहहॴनजसमहॳरकतमिािकह८लिकाओकीयाहीमह८गिह८बबनकीकमीहह८तीहहॴ 29-Jan-2017

Options

1) Albinism

एनमबननजम

2) Propyria

परह८पीररया 3) Anaemia

एनीलमया 4)Keloid disorder

कहॳ िह८इिडिसओिर

Correct Answer Anaemia

Q173 Ananas comosus is the scientific

name of

Options

अनानासकह८मह८सस mdashmdashmdashndash

कावहॴजञाननकनामहहॴ 29-Jan-2017

1) Custard Apple

सीताफि

2) Pineapple

पाइनएपपि

3) Bamboo

बास

4)Pomegranate

अनार

Correct Answer Pineapple

Q174 Which organ produces insulin

कह९नसाअगइनसलिनपहॴदाकरताहहॴ 29-Jan-

2017

Options

1) Liver

यकत

2) Thyroid gland

िायराइिगरिी 3) Spleen

पिीहा 4)Pancreas

अगरयिय

Correct Answer Pancreas

Q175 Which of the following disease is

not caused by water pollution

नननननलिखितमसहॳकह९नसारह८गपानीकहॳ परदषणकहॳकारणनहीहह८ता

29-Jan-2017

Options

1) Cholera

हहॴजा 2) Typhoid

F A C E B O O K

P A G E h t t p w w w f a c e b o o k c o m s s c m e n t o r s o f f i c i a l P a g e | 41

FOR MORE UPDATES AND MORE MATERIAL DO LIKE OUR FACEBOOK PAGE httpwwwfacebookcomsscmentorsofficial

टाइफाइि

3) Asthma

दमा 4)Diarrhoea

दसत

Correct Answer Asthma

Q176 Ocimum tenuiflorum is the

scientific name of

ओलिलममटहॳयईफिह८रमइसकावहॴजञाननकनाम mdash

ndash हहॴ 30-Jan-2017

Options

1) Neem

नीम

2) Mango

आम

3) Babul

बबि

4)Tulsi

तिसी Correct Answer Tulsi

Q177 Which gland secretes bile a

digestive fluid

कह९नसीगरिीपपतत एकपाचनतरिपरदािय सरापवतकरतीहहॴ 30-Jan-2017

Options

1) Pancreas

अगनयािय

2) Liver

यकत

3) Thyroid

िायराइि

4) Testes

टहॳनसटस

Correct Answer liver

Q178 In which of the following the

dominant phase is Gametophyte

नननननलिखितमसहॳककसकहॳ परमिचरणयगमकह८दपवधद (Gametophyte)हहॴ 30-Jan-2017

Options

1) Bryophyta

िायह८फाइटा 2) Pteridophyta

टहॳररिह८फाइटा 3) Gymnosperms

नजननह८सपमय 4) Angiosperms

एननजयह८सपमय Correct Answer Bryophyta

Q179 Anaerobic respiration refers to

which of the following

नननननलिखितमसहॳककसहॳअवायवीयशवसनकहाजाताहहॴ

30-Jan-2017

Options

1) Respiration without Oxygen

ऑकसीजनकहॳ बबनाशवसन

2) Respiration with Oxygen

ऑकसीजनकहॳ सािशवसन

3) Respiration without CO2

काबयनिायऑकसाइिकहॳ बबनाशवसन

4) Respiration with CO2

काबयनिायऑकसाइिकहॳ सािशविन

Correct Answer Respiration without

Oxygen

Q180 Which type of pathogen causes

the waterborne disease Cholera

ककसपरकारकारह८गजनकजिजननतरह८गहहॴजाकाकारणबनताहहॴ

30-Jan-2017

Options

1) Algal

िहॴवालियहॳ

F A C E B O O K

P A G E h t t p w w w f a c e b o o k c o m s s c m e n t o r s o f f i c i a l P a g e | 42

FOR MORE UPDATES AND MORE MATERIAL DO LIKE OUR FACEBOOK PAGE httpwwwfacebookcomsscmentorsofficial

2) Bacterial

बहॴकटीररयि

3) Protozoan

परह८टह८जआ

4) Viral

वायरि

Correct Answer Bacterial

Q181 To which class does

Oxyreductases transferases hydrolases

belong

ओकसीररिकटहॳसटरासफरहॳजहॳस

हाइडरह८िहॳसहॳसककसवगयमआतहॳहहॴ 30-Jan-2017

Options

1) Hormones

हारमोस

2) Enzymes

एजाइनस

3) Proteins

परह८टीनस

4) Vitamins

पवटालमनस

Correct Answer Enzymes

Q182 Which of the following is not true

about Gymnosperms

ननननमसहॳकह९नसीबातअनावतबीजीकहॳ बारहॳमसचनहीहहॴ 30-Jan-2017

Options

1) Dominant phase is saprophytes

परमिचरणसहॳपरह८फाइटसहह८ताहहॴ 2) Vascular bundles are absent

सवहनीबििअनपनसितहह८ताहहॴ 3) spores are heterospores

बीजाणहहॳटहॳरह८सपह८रसहह८तहॳहहॴ 4) Flowers are absent

फिअनपनसितहह८तहॳहहॴ

Correct Answer Vascular bundles are

absent

Q183 The name of first mammal clone sheep is

भहॳड़कीपरिमसतनपायीपरनत प (किह८न)

कानामहहॴ 30-Jan-2017

Options

1) Noori

नरी 2) Dolly

िॉिी 3) Louise

िसी 4)Durga

दगाय Correct Answer Dolly

Q184 Which type of pathogen causes

the water-borne disease Typhoid fever

ककसपरकारकारह८गजनकजिजननतरह८गटाइफाइिबिारकाकारणबनताहहॴ 30-Jan-2017

Options

1) Algal

िहॴवािीय

2) Parasitic

परजीवी 3) Protozoan

परह८टह८जनअन

4)Bacterial

बहॴकटीररयि

Correct Answer Bacterial

Q185 In which part of the cell are

proteins made

कह८लिकाकहॳ ककसटहससहॳमपरह८टीनबनायाजाताहहॴ

31-Jan-2017

Options

1) Reticulum

रहॳटटकिम

F A C E B O O K

P A G E h t t p w w w f a c e b o o k c o m s s c m e n t o r s o f f i c i a l P a g e | 43

FOR MORE UPDATES AND MORE MATERIAL DO LIKE OUR FACEBOOK PAGE httpwwwfacebookcomsscmentorsofficial

2) Golgi apparatus

गह८मजीएपहॳरहॳटस

3) Ribosomes

ररबह८सह८नस

4) Lysosome

िायसह८सह८नस

Correct Answer ribosomes

Proteins are produced by stringing

amino acids together in the order

specified by messenger RNA strands

that were transcribed from DNA in the

cell nucleus The process of synthesizing

a protein is called translation and it

occurs on ribosomes in the cytoplasm of

a cell

Q186 Polio is a disease caused by which

of the following

नननननलिखितमसहॳपह८लियह८कीबबमारह८हह८नहॳकाकारणकयाहहॴ

31-Jan-2017

Options

1) Bacteria

बहॴकटीररयि

2) Mosquito

मचछर

3) Virus

वायरस

4) Cockroach

नतिच हॳ Correct Answer Virus

Polio or poliomyelitis is a crippling and

potentially deadly infectious disease It

is caused by the poliovirus

Q187 ndash Hay fever is a sign of which of

the following

हहॳकफवरनननननलिखितमसहॳककसकाएकसकहॳ तहहॴ

31-Jan-2017

Options

1) Old Age

वदावसिा 2) Malnutrition

कपह८सण

3) Allergy

एिनजय 4) Over Work

अतयचधककाययकरना Correct Answer Allergy

Q188 How many chromosomes does a

human cell contain

एकमानवकह८लिकामककतनहॳगणसतरहह८तहॳहहॴ

29-Jan-2017

Options

1) 6

2) 26

3) 46

4) 66

Correct Answer 46

In humans each cell normally contains

23 pairs of chromosomes for a total of

46 Twenty-two of these pairs called

autosomes look the same in both males

and females The 23rd pair the sex

chromosomes differ between males and

females

Q189 Which of the following is not true

about Bryophyta

ननननमसहॳकह९नसीबातिायह८फाइटकहॳ बारहॳमसचनहीहहॴ 31-Jan-2017

Options

1) Dominant phase is gametophytes

परमिचरणगहॳलमतह८फाइटसहह८ताहहॴ 2) Main plant body is haploid

पह९धहॳकामखयिरीरअगखणतहह८ताहहॴ 3) Spores are homospores

बीजाणहह८मह८सफह८रसहह८तहॳहहॴ 4) Flowers are present

फिमह८जदहह८तहॳहहॴ Correct Answer Flowers are present

F A C E B O O K

P A G E h t t p w w w f a c e b o o k c o m s s c m e n t o r s o f f i c i a l P a g e | 44

FOR MORE UPDATES AND MORE MATERIAL DO LIKE OUR FACEBOOK PAGE httpwwwfacebookcomsscmentorsofficial

Q190 Which aquatic animal has

trailing tentacles

ककसजिीयजानवरकहॳ पीछहॳचिनहॳवािहॳटहॳटकिसहह८तहॳहहॴ

31-Jan-2017

Options

1) Sea horse

समदरीघह८िा 2) Corals

मगा 3) Jelly fish

जहॳिीमछिी 4) Star fish

तारामछिी Correct Answer Jelly fish

Jellyfish with its umbrella-shaped bell

and trailing tentacles

Q191 Which type of pathogen causes

the water-borne disease Poliomyelitis

(Polio)

ककसपरकारकारह८गजनकजिजननतरह८गपह८लियह८मायहॳटटस (पह८लियह८) काकारणहहॴ 31-Jan-

2017

Options

1) Parasitic

परजीवी 2) Algal

िहॴवालिय

3) Viral

वायरि

4) Bacterial

बहॴकटीररयि

Correct Answer Viral

Q192 The outer white part of the eye

that protects the inner structures is

आािकाबाहरीसफहॳ दटहससाजह८आतररकसरचनाओकीरकषाकरताहहॴ वह mdashmdashmdash हहॴ 31-Jan-

2017

Options

1) Iris

आयररस

2) Sclera

सकिहॳरा 3) Retina

रहॳटटना 4) Cornea

कह८ननयया Correct Answer Sclera

Q193 Proteins are made up of

परह८टीनकाननमायण mdashndash सहॳहह८ताहहॴ 31-Jan-2017

Options

1) Amino acids

एलमनह८अनि

2) Fatty acids

वसायकतअनि

3) Glucose

गिकह८ज

4)Nucleotides

नयनकियह८टाईिस

Correct Answer Amino acids

Q194 Moringa Oleifera is the scientific

name of

मह८ररगओलिफहॳ रा mdashmdashndash कावहॴजञाननकनामहहॴ 31-Jan-2017

Options

1) Banyan

बरगद

2) Gulmohar

गिमह८हर

3) Amla

आमिा

F A C E B O O K

P A G E h t t p w w w f a c e b o o k c o m s s c m e n t o r s o f f i c i a l P a g e | 45

FOR MORE UPDATES AND MORE MATERIAL DO LIKE OUR FACEBOOK PAGE httpwwwfacebookcomsscmentorsofficial

4) Drumstick

डरमनसटक

Correct Answer Drumstick

Q195 Kidney stones are composed of

गदकीपिरी mdashndash सहॳबनीहह८तीहहॴ 1-Feb-2017

Options

1) Calcium Oxalate

कहॴ नमसयमओकजहॳिहॳट

2) Sodium Chloride

सह८डियमकिह८राइि

3) Magnesium Nitrate

महॳनगनलियमनाइतटरहॳट

4) Calcium Bicarbonate

कहॴ नमियमबायकबोनहॳट

Correct Answer Calcium Oxalate

Q196 ndash Which of the following is not

true about Angiosperms

ननननमसहॳकह९नसीबातआवतबीजीकहॳ बारहॳमसचनहीहहॴ 1-Feb-2017

Options

1) Dominant phase is gametophytes

परमिचरणगहॳलमतह८फाइटहह८ताहहॴ 2) Vascular bundles are present

सवहनीबििमह९जदहह८ताहहॴ 3) Spores are heterospores

बीजाणहहॳटहॳरह८सपह८रसहह८तहॳहहॴ 4) Seeds are covered

बीजढकहॳ हह८तहॳहहॴ Correct Answer Dominant phase is

gametophytes

Q197 All of the following are excretory

(waste) products of animals except

नननननलिखितमसहॳककसएककह८छह८ड़करअनयसभीपराखणयोदवाराउतसनजयतपदाियहहॴ 1-Feb-

2017

Options

1) Uric Acid

यररकएलसि

2) Ammonia

अमह८ननया 3) Carbohydrates

काबोहाइडरहॳट

4) Urea

यररया Correct Answer Carbohydrates

In animals the main excretory products

are carbon dioxide ammonia (in

ammoniotelics) urea (in ureotelics) uric

acid (in uricotelics) guanine (in

Arachnida) and creatine

Q198 RNA is a polymeric molecule

What does RNA stand for

आरएनइएएकबहिकआणहहॴ इसकाकापवय पकयाहहॴ 1-Feb-2017

Options

1) Rado Nuclear Acid

रािह८नयनकियरएलसि

2) Ribo Nucleic Acid

राइबह८नयनकिकएलसि

3) Rhino Nuclear Acid

हाइनह८नयनकियरएलसि

4) Resto Nucleus Acid

रहॳसटह८नयकिीयसएलसि

Correct Answer Ribo Nucleic Acid

Q199 Which organ does detoxification

and produces chemicals needed for

digestion

कह९नसाअगपवषहरणकरताहहॴऔरपाचनकहॳ लिएआवशयकरसायनोकह८पहॴदाकरताहहॴ 1-Feb-

2017

Options

1) Salivary glands

िारगरचिया 2) Pancreas

अगनयािय

F A C E B O O K

P A G E h t t p w w w f a c e b o o k c o m s s c m e n t o r s o f f i c i a l P a g e | 46

FOR MORE UPDATES AND MORE MATERIAL DO LIKE OUR FACEBOOK PAGE httpwwwfacebookcomsscmentorsofficial

3) Thyroid gland

िायराइिगरिी 4) Liver

यकत

Correct Answer Liver

Q200 Psidium guajava is the scientific

name of

लसडियमगआजावा mdashmdash कावहॴजञाननकनामहहॴ 1-

Feb-2017

Options

1) Guava

अम द

2) Mango

आम

3) Bamboo

बास

4) Jack fruit

कटहि

Correct Answer Guava

Q201 Which drug is used as a Blood

Thinner

चधरकह८पतिाकरनहॳकहॳ पमककसदवाकापरयह८गककयाजाताहहॴ

1-Feb-2017

Options

1) Warfarin

वाफर न

2) Tramadol

टरहॳमािह८ि

3) Azithromycin

एनजरह८मायलसन

4) Hydralazine

हाइडरह८िहॳनजन

Correct Answer Warfarin

Q202 Which of the following disease is

caused due to the deficiency of protein

परह८टीनकीकमीकहॳ कारणनननननलिखितमसहॳकह९नसारह८गहह८ताहहॴ 1-Feb-2017

Options

1) Arthritis

गटठया 2) Kwashiorkor

कािीओकय र

3) Goitre

गाइटर

4) Night Blindness

रतह९चध

Correct Answer Kwashiorkor

Q203 A is species of plant that has

adapted to survive in an environment

with little liquid water

mdashmdashndashपह९धहॳकीएकऐसहॳऐसहॳपरजानतहहॴ नजसनहॳकमपानीवािहॳवातावरणमजीपवतरहनहॳकहॳलिएअनकिनहहॴ 1-Feb-2017

Options

1) Xerophyte

म दपवद

2) Hydrophyte

जिीयपादप

3) Mesophyte

समह८दपवद

4) Thallophyte

िहॴिह८फाइटा Correct Answer xerophyte

xerophyte is a species of plant that has

adapted to survive in an environment

with little liquid water such as a desert

or an ice- or snow-covered region in the

Alps or the Arctic

Mesophytes are terrestrial plants which

are adapted to neither a particularly

dry nor particularly wet environment

An example of a mesophytic habitat

would be a rural temperate meadow

F A C E B O O K

P A G E h t t p w w w f a c e b o o k c o m s s c m e n t o r s o f f i c i a l P a g e | 47

FOR MORE UPDATES AND MORE MATERIAL DO LIKE OUR FACEBOOK PAGE httpwwwfacebookcomsscmentorsofficial

which might contain goldenrod clover

oxeye daisy and Rosa multiflora

thallophyte any of a group of plants or

plantlike organisms (such as algae and

fungi) that lack differentiated stems

leaves and roots and that were formerly

classified as a primary division

(Thallophyta) of the plant kingdom

Q204 How many types of teeth are

there in humans

मनषयोमककतनहॳपरकारकहॳ दातहह८तहॳहहॴ

1-Feb-2017

Options

1) 4

2) 5

3) 2

4) 3

Correct Answer 4

teeth -Humans have four types of

teethincisors canines premolars and

molars each with a specific function

The incisors cut the food the canines

tear the food and the molars and

premolars crush the food

Q205 Carica papaya is the scientific name of

कहॴ ररकापपाया mdashmdashndash कावहॴजञाननकनामहहॴ 2-

Feb-2017

Options

1) Peepal

पीपि

2) Papaya

पपीता 3) Tamarind

इमिी 4) Drumstick

ढह८िकाछड़ी Correct Answer Papaya

Q206 Muscles get tired when there is

shortfall of

जब mdashndash कीकमीहह८तीहहॴतबपहॳिीयिकजातीहहॴ 2-Feb-2017

Options

1) Lactic acid

िहॴनकटकएलसि

2) Na+ ions

Na+ आयन

3) ATP

एटीपी 4) Sulphates

समफहॳ टस

Correct Answer ATP

ATP is the energy source muscle fibers

use to make muscles contract

muscle tissuersquos main source of energy

called adenosine triphosphate or ATP

As your muscles use up this energy

source they become tired and fatigued

Oxygen is the key ingredient that helps

create new ATP to replenish the burned

up ATP in your muscles

Q207 Artocarpus integra is the

scientific name of आटह८कापयसइटीगरा mdashmdashmdash कावहॴजञाननकनामहहॴ 2-Feb-2017

Options

1) Guava

अम द

2) Pineapple

अनानास

3) Silver Oak

लसमवरओक

4) Jack fruit

कटहि

Correct Answer Jack fruit

Q208 Which organ stores fat soluble

vitamins

कह९नसाअगवसामघिनिीिपवटालमनह८काभिाराकरताहहॴ

2-Feb-2017

F A C E B O O K

P A G E h t t p w w w f a c e b o o k c o m s s c m e n t o r s o f f i c i a l P a g e | 48

FOR MORE UPDATES AND MORE MATERIAL DO LIKE OUR FACEBOOK PAGE httpwwwfacebookcomsscmentorsofficial

Options

1) Blood

रकत

2) Skin

तवचा 3) Liver

यकत

4) Pancreas

अगनयािय

Correct Answer Liver

Q209 Which disease is caused due to

deficiency of Iodine

आयह८िीनकहॳ कारणकह९नसारह८गहह८ताहहॴ 2-Feb-2017

Options

1) Rickets

ररकहॳ टस

2) Scurvy

सकवी 3) Goitre

गणमािा 4) Growth retardation

पवकासका कना Correct Answer Goitre

rickets A softening and weakening of

bones in children usually due to

inadequate vitamin D

Q210 Grevillea Robusta is the scientific name of

गरहॳपवलियारह८बसटा mdashmdashmdash- कापवजञाननकनामहहॴ 2-Feb-2017

Options

1) Peepal

पीपि

2) Teak

सागह९न

3) Silver Oak

लसमवरओक

4) Jack fruit

कटहि

Correct Answer Silver Oak

Q211 When a Cuttlefish is described as a Molluscs it is at which level of

classification

जबएककटिकफिकह८एकमह८िसकाकहॳ पमवखणयतककयाजाताहहॴतबयहॳवगीकरणकहॳ ककससतरपहॳनसितहहॴ 2-Feb-2017

Options

1) Class

वगय 2) Order

िम

3) Family

पररवार

4) Phylum

सघ

Correct Answer Phylum

Q212 Bambusa dendrocalmus is the

scientific name of बानबसािहॳडराकामस mdashmdashmdash कावहॴजञाननकनामहहॴ 3-Feb-2017

Options

1) Banyan

बरगद

2) Papaya

पपीता 3) Bamboo

बास

4) Pomegranate

अनार

Correct Answer Bamboo

Q213 Acinonyx Jubatus is the scientific name of

एलसनह८ननकसजयबहॳटस mdashmdashmdash

कावहॴजञाननकनामहहॴ 3-Feb-2017

F A C E B O O K

P A G E h t t p w w w f a c e b o o k c o m s s c m e n t o r s o f f i c i a l P a g e | 49

FOR MORE UPDATES AND MORE MATERIAL DO LIKE OUR FACEBOOK PAGE httpwwwfacebookcomsscmentorsofficial

Options

1) Bear

भाि 2) Horse

घह८िा 3) Cheetah

चीता 4) Zebra

जहॳिा Correct Answer Cheetah

Q214 The pale yellow colour of urine is

due to the presence of which pigment

मतरकाफीकापीिारगरगदरयकहॳ उपनसिनतकहॳ कारणहह८ताहहॴ

3-Feb-2017

Options

1) Urochrome

यरह८िह८म

2) Urophyll

यरह८कफि

3) Chlorophyll

किह८रह८कफि

4) Chloroplast

किह८रह८पिासट

Correct Answer Urochrome

Q215 Which of the following constitute

to form a gene

नननननलिखितमसहॳकह९नसीचीज़एकजीनकागठनकरतीहहॴ

3-Feb-2017

Options

1) Polynucleotides

पह८िीनयनकियह८टाईडस

2) Hydrocarbons

हाइडरह८काबोस

3) Lipoproteins

िाईपह८परह८टीनस

4) Lipids

लिपपडस

Correct Answer Polynucleotides

Polynucleotide molecule is a biopolymer

composed of 13 or more nucleotide

monomers covalently bonded in a chain

DNA (deoxyribonucleic acid) and RNA

(ribonucleic acid) are examples of

polynucleotides with distinct biological

function

Q216 Vertebrates belongs to the

phylum

रीढ़कीहडिीवािहॳपराणी mdashmdashmdash

परजानतकहॳ अतगायतआतहॳहहॴ 3-Feb-2017

Options

1) Arthropoda

आरह८पह८ड़ा 2) Annelida

एननलििा 3) Cnidaria

ननिहॳररया 4) Chordata

कह८िटा Correct Answer Chordata

Q217 Punica granatum is the scientific name of

पननकगरहॳनहॳटस mdashmdashmdash कावहॴजञाननकनामहहॴ 3-Feb-2017

Options

1) Custard Apple

सीताफि

2) Gulmohar

गिमह८हर

3) Silver Oak

लसमवरओक

4) Pomegranate

अनार

Correct Answer Pomegranate

F A C E B O O K

P A G E h t t p w w w f a c e b o o k c o m s s c m e n t o r s o f f i c i a l P a g e | 50

FOR MORE UPDATES AND MORE MATERIAL DO LIKE OUR FACEBOOK PAGE httpwwwfacebookcomsscmentorsofficial

Q218 Between a tiger and an monkey

which of the following is different

एकबाघऔरबदरकहॳ बीचनननननलिखितमसहॳकह९नसीबातअिगहहॴ 3-Feb-2017

Options

1) Kingdom

राजय

2) Phylum

जानत

3) Order

िम

4) Class

वगय Correct Answer order

Q219 The artificial heart was invented by

कबतरमहदयका mdashmdashmdash

दवाराअपवषकारककयागयािा 3-Feb-2017

Options

1) Muhammad Yunus

महनमदयनस

2) Linus Yale Jr

िाइनसयहॳिजय

3) Gazi Yasargil

गाजीयासचगयि

4) Paul Winchell

पह९िपवमकि Correct Answer Paul Winchell

Q220 Tamarindus indica is the

scientific name of

टहॳमररनडसइडिका mdashmdash कावहॴजञाननकनामहहॴ 7-

Feb-2017

Options

1) Neem

नीम

2) Pineapple

अनानास

3) Tamarind

इमिी 4)Chiku

चीक

Correct Answer Tamarind

Q221 In eukaryotic cells synthesis of

RNA takes place in the

यकहॳ योटटककह८लिकाओमआरएनएकासशिहॳषण

mdashndash महह८ताहहॴ 7-Feb-2017

Options

1) Mitochondria

माईटह८कोडडरया 2) Centrioles

सटरीयह८मस

3) Ribosomes

ररबह८सह८नस

4) Nucleus

नयनकियस

Correct Answer nucleus

eukaryotic cell -Transcription is the

process of synthesizing ribonucleic acid

(RNA)Synthesis takes place within the

nucleus of eukaryotic cells or in the

cytoplasm of prokaryotes and converts

the genetic code from a gene in

deoxyribonucleic acid ( DNA ) to a

strand of RNA that then directs

proteinsynthesis

Q222 _________is caused by parasites

of the Plasmodium genus

पिाजमह८डियमजातीकहॳ परजीवी mdash- कहॳ कारणहहॴ 7-Feb-2017

Options

1) Dysentery

पहॳचचि

2) Malaria

मिहॳररया 3) Chickenpox

F A C E B O O K

P A G E h t t p w w w f a c e b o o k c o m s s c m e n t o r s o f f i c i a l P a g e | 51

FOR MORE UPDATES AND MORE MATERIAL DO LIKE OUR FACEBOOK PAGE httpwwwfacebookcomsscmentorsofficial

चहॳचक

4) Herpes

हहॳपपयस

Correct Answer Malaria

Q223 Carotene in fruits and vegetables

gives it which color

फिह८औरसनलजयोमनसितकहॳ रह८टीनउनहकह९नसारगपरदानकरताहहॴ 7-Feb-2017

Options

1) Green

हरा 2) Pink

गिाबी 3) Orange

नारगी 4) Blue

नीिा Correct Answer Orange

Q224 Equus Caballus is the scientific

name of

एकवसकहॴ बहॳिस mdashmdashndash कापवजञाननकनामहहॴ 7-Feb-2017

Options

1) Horse

घह८िा 2) Zebra

ज़हॳिा 3) Donkey

गधा 4) Buffalo

भस

Correct Answer Horse

Q225 Elapidae Naja is the scientific name of

एिीपीिीनाजा mdashmdash- कावहॴजञाननकनामहहॴ 8-Feb-2017

Options

1) Cobra

कह८बरा 2) Elephant

हािी 3) Eagle

ग ि

4) Owl

उमि Correct Answer Cobra

Q226 Which disease is caused due to

deficiency of Iron

िह८हकीकमीकहॳ कारणकह९नसारह८गहह८ताहहॴ 8-Feb-

2017

Options

1) Beriberi

बहॳरीबहॳरी 2) Tetany

टहॳटनी 3) Kwashiorkor

कवािीऔरकर

4) Anaemia

रकतामपता Correct Answer Anaemia

Beriberi is a disease caused by a vitamin

B-1 deficiency also known as thiamine

deficiency

Tetany can be the result of an

electrolyte imbalance Most often itrsquos a

dramatically low calcium level also

known as hypocalcemia Tetany can also

be caused by magnesium deficiency or

too little potassium Having too much

acid (acidosis) or too much alkali

(alkalosis) in the body can also result in

tetany

Kwashiorkor also known as

ldquoedematous malnutrition It is a form of

malnutrition caused by a lack of protein

in the diet

Anaemia means that you have fewer red

blood cells than normal or you have less

F A C E B O O K

P A G E h t t p w w w f a c e b o o k c o m s s c m e n t o r s o f f i c i a l P a g e | 52

FOR MORE UPDATES AND MORE MATERIAL DO LIKE OUR FACEBOOK PAGE httpwwwfacebookcomsscmentorsofficial

haemoglobin than normal in each red

blood cell

Q227 is a leaf where the leaflets are

arranged along the middle vein

mdashndashएकपततीहहॴजहापतरकह८कीरचनाक ररयालिराकहॳ आसपासहह८तीहहॴ 8-Feb-2017

Options

1) Pinnately compound leaf

पपनहॳटिीसयकतपतती 2) Palmately compound leaf

पामहॳटिीसयकतपतती 3) Compound leaf

सयकतपतती 4) Simple leaf

साधारणपतती Correct Answer Pinnately compound

leaf

Q228 Haustoria or sucking roots are

found in which of the following

हह८सटह८ररयायाचसनहॳवािीजड़हॳनननननलिखितमसहॳककसमपाईजातीहहॴ 8-Feb-2017

Options

1) Wheat

गहॳह

2) Mango

आम

3) Chestnut

चहॳसटनट

4) Cuscuta

कसकयटा Correct Answer Cuscuta

Haustorial roots -The roots of parasitic

plants which penetrate into the host

tissues to absorb nourishment are

called haustorial roots hellip Also known as suckingor parasitic roots

Q229 Equs Asinus is the scientific name

of

एकवसएलसनस mdashmdashndash कावहॴजञाननकनामहहॴ 8-

Feb-2017

Options

1) Donkey

गधा 2) Cow

गाय

3) Deer

टहरन

4) Kangaroo

कगा

Correct Answer Donkey

Q230 Ficus benghalensis is the scientific name of

फाईकसबहॳनगहॳिहॳलसस mdashndash कापवजञाननकनामहहॴ 8-Feb-2017

Options

1) Banyan

बरगद

2) Pineapple

अनानास

3) Babul

बबि

4) Tulsi

तिसी Correct Answer Banyan

Q231 Equus burchellii is the scientific name of

एकवसबचिी mdashmdash- कापवजञाननकनामहहॴ 8-Feb-2017

Options

1) Horse

घह८िा 2) Zebra

जहॳिा 3) Buffalo

F A C E B O O K

P A G E h t t p w w w f a c e b o o k c o m s s c m e n t o r s o f f i c i a l P a g e | 53

FOR MORE UPDATES AND MORE MATERIAL DO LIKE OUR FACEBOOK PAGE httpwwwfacebookcomsscmentorsofficial

भस

4) Ass

गधा Correct Answer Zebra

Page 37: COMPILATION OF ALL 72 SETS OF BIOLOGY SSC CHSL-2016 · OF BIOLOGY SSC CHSL-2016 PREPARED BY : SSC MENTORS BIOLOGY SPECIAL . F A C E B O O K P A G E : h t t p : / / w w w . f a c e

F A C E B O O K

P A G E h t t p w w w f a c e b o o k c o m s s c m e n t o r s o f f i c i a l P a g e | 36

FOR MORE UPDATES AND MORE MATERIAL DO LIKE OUR FACEBOOK PAGE httpwwwfacebookcomsscmentorsofficial

27-Jan-2017

Options

1) Dental Caries

िटिकहॴ ररज

2) Scurvy

सकवरी 3) Anaemia

रकतामपता 4) Arthritis

गटठया Correct Answer Dental Caries

Q152 In a Punnett Square with the

cross AaBb x AaBb how many Aabb

genotypes would be created

पनहॳटसककायरमिह८स AaBb x AaBb कहॳ साि

ककतनहॳ Aabb जीनह८टाइपबनगहॳ 27-Jan-2017

Options

1) 1

2) 8

3) 2

4) 3

Correct Answer 2

Q153 Which of the following is the

Controlling Center of the Cell

नननननलिखित म सहॳ कह८लिकाका ननयतरण

क दर कह९न हहॴ

27-Jan-2017

Options

1) Nucleus

क दर

2) Plasma

पिाजमा 3) Lysosome

िायसह८सह८म

4) Chromosome

िह८मह८सह८म

Correct Answer Nucleus

The control centre of the cell is the

nucleus in eukaryotic cells The nucleus

contains genetic material in the form of

DNA

Q154 Myopia affects which of the

following organs

मायह८पपयानननननलिखितअगह८मसहॳककसहॳपरभापवतकरताहहॴ

25-Jan-2017

Options

1) Heart

हदय

2) Skin

तवचा 3) Eyes

आािहॳ 4)Mouth

मह

Correct Answer Eyes

Q155 Which of the following bears

flowers

नननननलिखितमसहॳकह९नफिधारणकरताहहॴ

25-Jan-2017

Options

1) Bryophyta

िायह८फाइटा 2) Pteridophyta

टहॳरीिह८फाईटा 3) Gymnosperms

नजननह८सपमय 4)Angiosperms

एननजयह८सपमय Correct Answer Angiosperms

Q156 Oxygenated blood flows out of the

heart through the

ऑकसीजनयकतरकत mdashmdashmdash

कहॳ माधयमसहॳहदयकहॳ बाहरबहताहहॴ 25-Jan-2017

F A C E B O O K

P A G E h t t p w w w f a c e b o o k c o m s s c m e n t o r s o f f i c i a l P a g e | 37

FOR MORE UPDATES AND MORE MATERIAL DO LIKE OUR FACEBOOK PAGE httpwwwfacebookcomsscmentorsofficial

Options

1) Aorta

महाधमनी 2) pulmonary artery

फहॳ फड़हॳकीधमनी 3) vena cava

वहॳनाकावा 4)Atrium

चह९क

Correct Answer aorta

Q157 Blood leaving the liver and

moving towards the

heart has a higher concentration of

नजगरसहॳननकिकरहदयकीतरफजानहॳवािहॳरकतम mdashmdashmdashmdash कीउचचसादरताहह८तीहहॴ 27-Jan-2017

Options

1) Lipids

लिपपडस

2) Urea

यररया 3) Bile Pigments

पपततकहॳ रगकरण

4) Carbon dioxide

काबयनिायऑकसाइि

Correct Answer Bile Pigments

Urea is nitrogen containing substance

which is produced in the liver in order

to deal with excess amino-acids in the

body As urea is produced it leaves the

liver in the blood stream and passes via

the circulatory system to all parts of the

body

Q158 Bulb is a modification of which

part of a plant

बमबएकपह९धहॳकहॳ ककसटहससहॳकाएक पातरणहह८ताहहॴ 27-Jan-2017

Options

1) The root

जड़

2) The stem

तना 3) The radicle

मिाकर

4)The fruit

फि

Correct Answer The stem

Q159 Which of the following carries

blood away from the heart to different

body parts

इनमहॳसहॳकह९नरकतकह८हदयसहॳिरीरकहॳ पवलभननअगह८तकिहॳजातीहहॴ

27-Jan-2017

Options

1) Arteries

धमननया 2) Nerves

तबतरहाए

3) Capillaries

कहॳ लिकाए

4)Veins

नसहॳ Correct Answer Arteries

Q160 The series of processes by which

nitrogen and its compounds are

interconverted in the environment and

in living organisms is called

27-Jan-2017

Options

1)Absorption of Nitrogen

2)Ammonification

3)Nitrogen Fixation

4)Nitrogen Cycle

Correct Answer Nitrogen Cycle

Ammonification or Mineralization is

performed by bacteria to convert

organic nitrogen to ammonia

F A C E B O O K

P A G E h t t p w w w f a c e b o o k c o m s s c m e n t o r s o f f i c i a l P a g e | 38

FOR MORE UPDATES AND MORE MATERIAL DO LIKE OUR FACEBOOK PAGE httpwwwfacebookcomsscmentorsofficial

Nitrification can then occur to convert

the ammonium to nitrite and nitrate

Nitrogen fixation is a process by which

nitrogen in the Earthrsquos atmosphere is

converted into ammonia (NH3) or other

molecules available to living organisms

Q161 BCG vaccine is given to protect

from which of the following

बीसीजीकाटटकानननननलिखितमसहॳककसकहॳ बचावकहॳ लिएटदयाजातहहॴ

27-Jan-2017

Options

1) Jaundice

पीलिया 2) Anaemia

रकतमपता 3) Tuberculosis

कषयरह८ग

4) Polio

पह८लियह८ Correct Answer Tuberculosis

Q162 Parallel venation is found in

समानतरवहॳनहॳिन mdashmdashmdash- मपायाजाताहहॴ 27-Jan-2017

Options

1) plants which are monocots

पह९धहॳजह८एकबीजपतरीहह८तहॳहहॴ 2) plants which have a dicot stem

वहॳपह९धहॳनजनकातनादपवदलियहह८ताहहॴ 3) plants with leaves similar to Tulsi

वहॳपह९धहॳनजनकीपनततयतिसीकीपनततयोकहॳ समानहह८तहॳहहॴ 4)plants with tap roots

टहॳप टवािहॳपह९धहॳ Correct Answer plants which are

monocots

Q163 The hardest part of the body is

िरीरकासबसहॳकठह८रभाग mdashndash हहॴ 27-Jan-2017

Options

1) Bones

हडडिय

2) Tooth Enamel

दातकहॳ इनहॳमि

3) Skull

िह८पड़ी 4) Spinal Cord

महॳ रजज

Correct Answer Tooth Enamel

Q164 Which type of pathogen causes

the waterborne disease E coli Infection

ककसपरकारकारह८गजननकजिजननतरह८गईकह८िाईसिमणकाकारणबनताहहॴ 27-Jan-2017

Options

1) Protozoan

परह८टह८जआ

2) Parasitic

परजीवी 3) Bacterial

बहॴकटीररयि

4)Viral

वायरि

Correct Answer Bacterial

Q165 The amount of blood filtered

together by both the kidneys in a 70 kg

adult male human in a minute is

70 की गरा वािहॳएकवयसकप षमएकलमनटमदह८नोगदकहॳदवाराएकसािचाबनीगयीरकतकीमातरहह८तीहहॴ 29-Jan-2017

Options

1) 1100 ml

1100 लमलि

2) 100 ml

F A C E B O O K

P A G E h t t p w w w f a c e b o o k c o m s s c m e n t o r s o f f i c i a l P a g e | 39

FOR MORE UPDATES AND MORE MATERIAL DO LIKE OUR FACEBOOK PAGE httpwwwfacebookcomsscmentorsofficial

100 लमलि

3) 1500 ml

1500 लमलि

4) 500 ml

500 लमलि

Correct Answer 1100 ml

Q166 Which feature of a plant helps to

distinguish a monocot from a dicot

पह९धहॳकीवहकह९नसीपविहॳषताहहॴजह८एकदपवदलियहॳऔरएकएकदिीयपह९धहॳसहॳभहॳदकरनहॳममददकरतीहहॴ 29-Jan-2017

Options

1) Pollination

परागम

2) Venation

वहॳनहॳिन

3) Vernation

वनिन

4) Aestivation

एसटीवहॳिहॳन

Correct Answer venation

Q167 The Mutation Theory was

proposed by

उतवररवतयनकालसदात mdashmdashndash

कहॳ दवरापरसतापवतककयाजाताहहॴ 29-Jan-2017

Options

1) Charles Lyell

चामसयलियहॳि

2) William Smith

पवलियमनसमि

3) Hugo De Vries

हयगह८िीराईस

4)Harrison Schmitt

हहॳरीसननसमट

Correct Answer Hugo De Vries

Q168 Which type of pathogen causes

the waterborne disease HepatitisA

ककसपरकारकहॳ रह८गजनकजिजननतरह८गहहॳपहॳटाइटटस-A काकारणबनताहहॴ

29-Jan-2017

Options

1) Parasitic

परजीवी 2) Viral

वायरि

3) Protozoan

परह८टह८जआ

4) Bacterial

बहॴकटीररयि

Correct Answer Viral

Q169 In a Punnett Square with the

cross AaBb x Aabb how many AaBb

genotypes would be created

पनहॳटसकवायरमिह८स AaBb x Aabb

कहॳ सािककतनहॳ AaBb जीनह८टाइपबनगहॳ 29-Jan-

2017

Options

1) 4

2) 1

3) 7

4) 6

Correct Answer 4

Q170 Arboreal Ateles is the scientific

name of

अिह८ररयिएटटलिस mdashmdashmdash कावहॴजञाननकनामहहॴ 29-Jan-2017

Options

1) Squirrel

चगिहरी 2) Sparrow

गह८रहॴया 3) Lizard

नछपकिी 4) Spider monkey

F A C E B O O K

P A G E h t t p w w w f a c e b o o k c o m s s c m e n t o r s o f f i c i a l P a g e | 40

FOR MORE UPDATES AND MORE MATERIAL DO LIKE OUR FACEBOOK PAGE httpwwwfacebookcomsscmentorsofficial

मकड़ीबदर

Correct Answer Spider monkey

Q171 Which type of pathogen causes

the waterborne disease Salmonellosis

ककसपरकारकारह८गाणजिजननतबीमारीसािमह८नहॳिह८लसज़काकारकहहॴ

29-Jan-2017

Options

1) Algal

िहॳवालियहॳ 2) Parasitic

परजीवी 3) Bacterial

बहॴकटीररयि

4)Viral

वायरि

Correct Answer Bacterial

An infection with salmonella bacteria

commonly caused by contaminated food

or water

Symptoms include diarrhoea fever

chills and abdominal pain

Q172 is a condition in which there is a

deficiency of red cells or of haemoglobin

in the blood

mdashmdash-

एकनसिनतहहॴनजसमहॳरकतमिािकह८लिकाओकीयाहीमह८गिह८बबनकीकमीहह८तीहहॴ 29-Jan-2017

Options

1) Albinism

एनमबननजम

2) Propyria

परह८पीररया 3) Anaemia

एनीलमया 4)Keloid disorder

कहॳ िह८इिडिसओिर

Correct Answer Anaemia

Q173 Ananas comosus is the scientific

name of

Options

अनानासकह८मह८सस mdashmdashmdashndash

कावहॴजञाननकनामहहॴ 29-Jan-2017

1) Custard Apple

सीताफि

2) Pineapple

पाइनएपपि

3) Bamboo

बास

4)Pomegranate

अनार

Correct Answer Pineapple

Q174 Which organ produces insulin

कह९नसाअगइनसलिनपहॴदाकरताहहॴ 29-Jan-

2017

Options

1) Liver

यकत

2) Thyroid gland

िायराइिगरिी 3) Spleen

पिीहा 4)Pancreas

अगरयिय

Correct Answer Pancreas

Q175 Which of the following disease is

not caused by water pollution

नननननलिखितमसहॳकह९नसारह८गपानीकहॳ परदषणकहॳकारणनहीहह८ता

29-Jan-2017

Options

1) Cholera

हहॴजा 2) Typhoid

F A C E B O O K

P A G E h t t p w w w f a c e b o o k c o m s s c m e n t o r s o f f i c i a l P a g e | 41

FOR MORE UPDATES AND MORE MATERIAL DO LIKE OUR FACEBOOK PAGE httpwwwfacebookcomsscmentorsofficial

टाइफाइि

3) Asthma

दमा 4)Diarrhoea

दसत

Correct Answer Asthma

Q176 Ocimum tenuiflorum is the

scientific name of

ओलिलममटहॳयईफिह८रमइसकावहॴजञाननकनाम mdash

ndash हहॴ 30-Jan-2017

Options

1) Neem

नीम

2) Mango

आम

3) Babul

बबि

4)Tulsi

तिसी Correct Answer Tulsi

Q177 Which gland secretes bile a

digestive fluid

कह९नसीगरिीपपतत एकपाचनतरिपरदािय सरापवतकरतीहहॴ 30-Jan-2017

Options

1) Pancreas

अगनयािय

2) Liver

यकत

3) Thyroid

िायराइि

4) Testes

टहॳनसटस

Correct Answer liver

Q178 In which of the following the

dominant phase is Gametophyte

नननननलिखितमसहॳककसकहॳ परमिचरणयगमकह८दपवधद (Gametophyte)हहॴ 30-Jan-2017

Options

1) Bryophyta

िायह८फाइटा 2) Pteridophyta

टहॳररिह८फाइटा 3) Gymnosperms

नजननह८सपमय 4) Angiosperms

एननजयह८सपमय Correct Answer Bryophyta

Q179 Anaerobic respiration refers to

which of the following

नननननलिखितमसहॳककसहॳअवायवीयशवसनकहाजाताहहॴ

30-Jan-2017

Options

1) Respiration without Oxygen

ऑकसीजनकहॳ बबनाशवसन

2) Respiration with Oxygen

ऑकसीजनकहॳ सािशवसन

3) Respiration without CO2

काबयनिायऑकसाइिकहॳ बबनाशवसन

4) Respiration with CO2

काबयनिायऑकसाइिकहॳ सािशविन

Correct Answer Respiration without

Oxygen

Q180 Which type of pathogen causes

the waterborne disease Cholera

ककसपरकारकारह८गजनकजिजननतरह८गहहॴजाकाकारणबनताहहॴ

30-Jan-2017

Options

1) Algal

िहॴवालियहॳ

F A C E B O O K

P A G E h t t p w w w f a c e b o o k c o m s s c m e n t o r s o f f i c i a l P a g e | 42

FOR MORE UPDATES AND MORE MATERIAL DO LIKE OUR FACEBOOK PAGE httpwwwfacebookcomsscmentorsofficial

2) Bacterial

बहॴकटीररयि

3) Protozoan

परह८टह८जआ

4) Viral

वायरि

Correct Answer Bacterial

Q181 To which class does

Oxyreductases transferases hydrolases

belong

ओकसीररिकटहॳसटरासफरहॳजहॳस

हाइडरह८िहॳसहॳसककसवगयमआतहॳहहॴ 30-Jan-2017

Options

1) Hormones

हारमोस

2) Enzymes

एजाइनस

3) Proteins

परह८टीनस

4) Vitamins

पवटालमनस

Correct Answer Enzymes

Q182 Which of the following is not true

about Gymnosperms

ननननमसहॳकह९नसीबातअनावतबीजीकहॳ बारहॳमसचनहीहहॴ 30-Jan-2017

Options

1) Dominant phase is saprophytes

परमिचरणसहॳपरह८फाइटसहह८ताहहॴ 2) Vascular bundles are absent

सवहनीबििअनपनसितहह८ताहहॴ 3) spores are heterospores

बीजाणहहॳटहॳरह८सपह८रसहह८तहॳहहॴ 4) Flowers are absent

फिअनपनसितहह८तहॳहहॴ

Correct Answer Vascular bundles are

absent

Q183 The name of first mammal clone sheep is

भहॳड़कीपरिमसतनपायीपरनत प (किह८न)

कानामहहॴ 30-Jan-2017

Options

1) Noori

नरी 2) Dolly

िॉिी 3) Louise

िसी 4)Durga

दगाय Correct Answer Dolly

Q184 Which type of pathogen causes

the water-borne disease Typhoid fever

ककसपरकारकारह८गजनकजिजननतरह८गटाइफाइिबिारकाकारणबनताहहॴ 30-Jan-2017

Options

1) Algal

िहॴवािीय

2) Parasitic

परजीवी 3) Protozoan

परह८टह८जनअन

4)Bacterial

बहॴकटीररयि

Correct Answer Bacterial

Q185 In which part of the cell are

proteins made

कह८लिकाकहॳ ककसटहससहॳमपरह८टीनबनायाजाताहहॴ

31-Jan-2017

Options

1) Reticulum

रहॳटटकिम

F A C E B O O K

P A G E h t t p w w w f a c e b o o k c o m s s c m e n t o r s o f f i c i a l P a g e | 43

FOR MORE UPDATES AND MORE MATERIAL DO LIKE OUR FACEBOOK PAGE httpwwwfacebookcomsscmentorsofficial

2) Golgi apparatus

गह८मजीएपहॳरहॳटस

3) Ribosomes

ररबह८सह८नस

4) Lysosome

िायसह८सह८नस

Correct Answer ribosomes

Proteins are produced by stringing

amino acids together in the order

specified by messenger RNA strands

that were transcribed from DNA in the

cell nucleus The process of synthesizing

a protein is called translation and it

occurs on ribosomes in the cytoplasm of

a cell

Q186 Polio is a disease caused by which

of the following

नननननलिखितमसहॳपह८लियह८कीबबमारह८हह८नहॳकाकारणकयाहहॴ

31-Jan-2017

Options

1) Bacteria

बहॴकटीररयि

2) Mosquito

मचछर

3) Virus

वायरस

4) Cockroach

नतिच हॳ Correct Answer Virus

Polio or poliomyelitis is a crippling and

potentially deadly infectious disease It

is caused by the poliovirus

Q187 ndash Hay fever is a sign of which of

the following

हहॳकफवरनननननलिखितमसहॳककसकाएकसकहॳ तहहॴ

31-Jan-2017

Options

1) Old Age

वदावसिा 2) Malnutrition

कपह८सण

3) Allergy

एिनजय 4) Over Work

अतयचधककाययकरना Correct Answer Allergy

Q188 How many chromosomes does a

human cell contain

एकमानवकह८लिकामककतनहॳगणसतरहह८तहॳहहॴ

29-Jan-2017

Options

1) 6

2) 26

3) 46

4) 66

Correct Answer 46

In humans each cell normally contains

23 pairs of chromosomes for a total of

46 Twenty-two of these pairs called

autosomes look the same in both males

and females The 23rd pair the sex

chromosomes differ between males and

females

Q189 Which of the following is not true

about Bryophyta

ननननमसहॳकह९नसीबातिायह८फाइटकहॳ बारहॳमसचनहीहहॴ 31-Jan-2017

Options

1) Dominant phase is gametophytes

परमिचरणगहॳलमतह८फाइटसहह८ताहहॴ 2) Main plant body is haploid

पह९धहॳकामखयिरीरअगखणतहह८ताहहॴ 3) Spores are homospores

बीजाणहह८मह८सफह८रसहह८तहॳहहॴ 4) Flowers are present

फिमह८जदहह८तहॳहहॴ Correct Answer Flowers are present

F A C E B O O K

P A G E h t t p w w w f a c e b o o k c o m s s c m e n t o r s o f f i c i a l P a g e | 44

FOR MORE UPDATES AND MORE MATERIAL DO LIKE OUR FACEBOOK PAGE httpwwwfacebookcomsscmentorsofficial

Q190 Which aquatic animal has

trailing tentacles

ककसजिीयजानवरकहॳ पीछहॳचिनहॳवािहॳटहॳटकिसहह८तहॳहहॴ

31-Jan-2017

Options

1) Sea horse

समदरीघह८िा 2) Corals

मगा 3) Jelly fish

जहॳिीमछिी 4) Star fish

तारामछिी Correct Answer Jelly fish

Jellyfish with its umbrella-shaped bell

and trailing tentacles

Q191 Which type of pathogen causes

the water-borne disease Poliomyelitis

(Polio)

ककसपरकारकारह८गजनकजिजननतरह८गपह८लियह८मायहॳटटस (पह८लियह८) काकारणहहॴ 31-Jan-

2017

Options

1) Parasitic

परजीवी 2) Algal

िहॴवालिय

3) Viral

वायरि

4) Bacterial

बहॴकटीररयि

Correct Answer Viral

Q192 The outer white part of the eye

that protects the inner structures is

आािकाबाहरीसफहॳ दटहससाजह८आतररकसरचनाओकीरकषाकरताहहॴ वह mdashmdashmdash हहॴ 31-Jan-

2017

Options

1) Iris

आयररस

2) Sclera

सकिहॳरा 3) Retina

रहॳटटना 4) Cornea

कह८ननयया Correct Answer Sclera

Q193 Proteins are made up of

परह८टीनकाननमायण mdashndash सहॳहह८ताहहॴ 31-Jan-2017

Options

1) Amino acids

एलमनह८अनि

2) Fatty acids

वसायकतअनि

3) Glucose

गिकह८ज

4)Nucleotides

नयनकियह८टाईिस

Correct Answer Amino acids

Q194 Moringa Oleifera is the scientific

name of

मह८ररगओलिफहॳ रा mdashmdashndash कावहॴजञाननकनामहहॴ 31-Jan-2017

Options

1) Banyan

बरगद

2) Gulmohar

गिमह८हर

3) Amla

आमिा

F A C E B O O K

P A G E h t t p w w w f a c e b o o k c o m s s c m e n t o r s o f f i c i a l P a g e | 45

FOR MORE UPDATES AND MORE MATERIAL DO LIKE OUR FACEBOOK PAGE httpwwwfacebookcomsscmentorsofficial

4) Drumstick

डरमनसटक

Correct Answer Drumstick

Q195 Kidney stones are composed of

गदकीपिरी mdashndash सहॳबनीहह८तीहहॴ 1-Feb-2017

Options

1) Calcium Oxalate

कहॴ नमसयमओकजहॳिहॳट

2) Sodium Chloride

सह८डियमकिह८राइि

3) Magnesium Nitrate

महॳनगनलियमनाइतटरहॳट

4) Calcium Bicarbonate

कहॴ नमियमबायकबोनहॳट

Correct Answer Calcium Oxalate

Q196 ndash Which of the following is not

true about Angiosperms

ननननमसहॳकह९नसीबातआवतबीजीकहॳ बारहॳमसचनहीहहॴ 1-Feb-2017

Options

1) Dominant phase is gametophytes

परमिचरणगहॳलमतह८फाइटहह८ताहहॴ 2) Vascular bundles are present

सवहनीबििमह९जदहह८ताहहॴ 3) Spores are heterospores

बीजाणहहॳटहॳरह८सपह८रसहह८तहॳहहॴ 4) Seeds are covered

बीजढकहॳ हह८तहॳहहॴ Correct Answer Dominant phase is

gametophytes

Q197 All of the following are excretory

(waste) products of animals except

नननननलिखितमसहॳककसएककह८छह८ड़करअनयसभीपराखणयोदवाराउतसनजयतपदाियहहॴ 1-Feb-

2017

Options

1) Uric Acid

यररकएलसि

2) Ammonia

अमह८ननया 3) Carbohydrates

काबोहाइडरहॳट

4) Urea

यररया Correct Answer Carbohydrates

In animals the main excretory products

are carbon dioxide ammonia (in

ammoniotelics) urea (in ureotelics) uric

acid (in uricotelics) guanine (in

Arachnida) and creatine

Q198 RNA is a polymeric molecule

What does RNA stand for

आरएनइएएकबहिकआणहहॴ इसकाकापवय पकयाहहॴ 1-Feb-2017

Options

1) Rado Nuclear Acid

रािह८नयनकियरएलसि

2) Ribo Nucleic Acid

राइबह८नयनकिकएलसि

3) Rhino Nuclear Acid

हाइनह८नयनकियरएलसि

4) Resto Nucleus Acid

रहॳसटह८नयकिीयसएलसि

Correct Answer Ribo Nucleic Acid

Q199 Which organ does detoxification

and produces chemicals needed for

digestion

कह९नसाअगपवषहरणकरताहहॴऔरपाचनकहॳ लिएआवशयकरसायनोकह८पहॴदाकरताहहॴ 1-Feb-

2017

Options

1) Salivary glands

िारगरचिया 2) Pancreas

अगनयािय

F A C E B O O K

P A G E h t t p w w w f a c e b o o k c o m s s c m e n t o r s o f f i c i a l P a g e | 46

FOR MORE UPDATES AND MORE MATERIAL DO LIKE OUR FACEBOOK PAGE httpwwwfacebookcomsscmentorsofficial

3) Thyroid gland

िायराइिगरिी 4) Liver

यकत

Correct Answer Liver

Q200 Psidium guajava is the scientific

name of

लसडियमगआजावा mdashmdash कावहॴजञाननकनामहहॴ 1-

Feb-2017

Options

1) Guava

अम द

2) Mango

आम

3) Bamboo

बास

4) Jack fruit

कटहि

Correct Answer Guava

Q201 Which drug is used as a Blood

Thinner

चधरकह८पतिाकरनहॳकहॳ पमककसदवाकापरयह८गककयाजाताहहॴ

1-Feb-2017

Options

1) Warfarin

वाफर न

2) Tramadol

टरहॳमािह८ि

3) Azithromycin

एनजरह८मायलसन

4) Hydralazine

हाइडरह८िहॳनजन

Correct Answer Warfarin

Q202 Which of the following disease is

caused due to the deficiency of protein

परह८टीनकीकमीकहॳ कारणनननननलिखितमसहॳकह९नसारह८गहह८ताहहॴ 1-Feb-2017

Options

1) Arthritis

गटठया 2) Kwashiorkor

कािीओकय र

3) Goitre

गाइटर

4) Night Blindness

रतह९चध

Correct Answer Kwashiorkor

Q203 A is species of plant that has

adapted to survive in an environment

with little liquid water

mdashmdashndashपह९धहॳकीएकऐसहॳऐसहॳपरजानतहहॴ नजसनहॳकमपानीवािहॳवातावरणमजीपवतरहनहॳकहॳलिएअनकिनहहॴ 1-Feb-2017

Options

1) Xerophyte

म दपवद

2) Hydrophyte

जिीयपादप

3) Mesophyte

समह८दपवद

4) Thallophyte

िहॴिह८फाइटा Correct Answer xerophyte

xerophyte is a species of plant that has

adapted to survive in an environment

with little liquid water such as a desert

or an ice- or snow-covered region in the

Alps or the Arctic

Mesophytes are terrestrial plants which

are adapted to neither a particularly

dry nor particularly wet environment

An example of a mesophytic habitat

would be a rural temperate meadow

F A C E B O O K

P A G E h t t p w w w f a c e b o o k c o m s s c m e n t o r s o f f i c i a l P a g e | 47

FOR MORE UPDATES AND MORE MATERIAL DO LIKE OUR FACEBOOK PAGE httpwwwfacebookcomsscmentorsofficial

which might contain goldenrod clover

oxeye daisy and Rosa multiflora

thallophyte any of a group of plants or

plantlike organisms (such as algae and

fungi) that lack differentiated stems

leaves and roots and that were formerly

classified as a primary division

(Thallophyta) of the plant kingdom

Q204 How many types of teeth are

there in humans

मनषयोमककतनहॳपरकारकहॳ दातहह८तहॳहहॴ

1-Feb-2017

Options

1) 4

2) 5

3) 2

4) 3

Correct Answer 4

teeth -Humans have four types of

teethincisors canines premolars and

molars each with a specific function

The incisors cut the food the canines

tear the food and the molars and

premolars crush the food

Q205 Carica papaya is the scientific name of

कहॴ ररकापपाया mdashmdashndash कावहॴजञाननकनामहहॴ 2-

Feb-2017

Options

1) Peepal

पीपि

2) Papaya

पपीता 3) Tamarind

इमिी 4) Drumstick

ढह८िकाछड़ी Correct Answer Papaya

Q206 Muscles get tired when there is

shortfall of

जब mdashndash कीकमीहह८तीहहॴतबपहॳिीयिकजातीहहॴ 2-Feb-2017

Options

1) Lactic acid

िहॴनकटकएलसि

2) Na+ ions

Na+ आयन

3) ATP

एटीपी 4) Sulphates

समफहॳ टस

Correct Answer ATP

ATP is the energy source muscle fibers

use to make muscles contract

muscle tissuersquos main source of energy

called adenosine triphosphate or ATP

As your muscles use up this energy

source they become tired and fatigued

Oxygen is the key ingredient that helps

create new ATP to replenish the burned

up ATP in your muscles

Q207 Artocarpus integra is the

scientific name of आटह८कापयसइटीगरा mdashmdashmdash कावहॴजञाननकनामहहॴ 2-Feb-2017

Options

1) Guava

अम द

2) Pineapple

अनानास

3) Silver Oak

लसमवरओक

4) Jack fruit

कटहि

Correct Answer Jack fruit

Q208 Which organ stores fat soluble

vitamins

कह९नसाअगवसामघिनिीिपवटालमनह८काभिाराकरताहहॴ

2-Feb-2017

F A C E B O O K

P A G E h t t p w w w f a c e b o o k c o m s s c m e n t o r s o f f i c i a l P a g e | 48

FOR MORE UPDATES AND MORE MATERIAL DO LIKE OUR FACEBOOK PAGE httpwwwfacebookcomsscmentorsofficial

Options

1) Blood

रकत

2) Skin

तवचा 3) Liver

यकत

4) Pancreas

अगनयािय

Correct Answer Liver

Q209 Which disease is caused due to

deficiency of Iodine

आयह८िीनकहॳ कारणकह९नसारह८गहह८ताहहॴ 2-Feb-2017

Options

1) Rickets

ररकहॳ टस

2) Scurvy

सकवी 3) Goitre

गणमािा 4) Growth retardation

पवकासका कना Correct Answer Goitre

rickets A softening and weakening of

bones in children usually due to

inadequate vitamin D

Q210 Grevillea Robusta is the scientific name of

गरहॳपवलियारह८बसटा mdashmdashmdash- कापवजञाननकनामहहॴ 2-Feb-2017

Options

1) Peepal

पीपि

2) Teak

सागह९न

3) Silver Oak

लसमवरओक

4) Jack fruit

कटहि

Correct Answer Silver Oak

Q211 When a Cuttlefish is described as a Molluscs it is at which level of

classification

जबएककटिकफिकह८एकमह८िसकाकहॳ पमवखणयतककयाजाताहहॴतबयहॳवगीकरणकहॳ ककससतरपहॳनसितहहॴ 2-Feb-2017

Options

1) Class

वगय 2) Order

िम

3) Family

पररवार

4) Phylum

सघ

Correct Answer Phylum

Q212 Bambusa dendrocalmus is the

scientific name of बानबसािहॳडराकामस mdashmdashmdash कावहॴजञाननकनामहहॴ 3-Feb-2017

Options

1) Banyan

बरगद

2) Papaya

पपीता 3) Bamboo

बास

4) Pomegranate

अनार

Correct Answer Bamboo

Q213 Acinonyx Jubatus is the scientific name of

एलसनह८ननकसजयबहॳटस mdashmdashmdash

कावहॴजञाननकनामहहॴ 3-Feb-2017

F A C E B O O K

P A G E h t t p w w w f a c e b o o k c o m s s c m e n t o r s o f f i c i a l P a g e | 49

FOR MORE UPDATES AND MORE MATERIAL DO LIKE OUR FACEBOOK PAGE httpwwwfacebookcomsscmentorsofficial

Options

1) Bear

भाि 2) Horse

घह८िा 3) Cheetah

चीता 4) Zebra

जहॳिा Correct Answer Cheetah

Q214 The pale yellow colour of urine is

due to the presence of which pigment

मतरकाफीकापीिारगरगदरयकहॳ उपनसिनतकहॳ कारणहह८ताहहॴ

3-Feb-2017

Options

1) Urochrome

यरह८िह८म

2) Urophyll

यरह८कफि

3) Chlorophyll

किह८रह८कफि

4) Chloroplast

किह८रह८पिासट

Correct Answer Urochrome

Q215 Which of the following constitute

to form a gene

नननननलिखितमसहॳकह९नसीचीज़एकजीनकागठनकरतीहहॴ

3-Feb-2017

Options

1) Polynucleotides

पह८िीनयनकियह८टाईडस

2) Hydrocarbons

हाइडरह८काबोस

3) Lipoproteins

िाईपह८परह८टीनस

4) Lipids

लिपपडस

Correct Answer Polynucleotides

Polynucleotide molecule is a biopolymer

composed of 13 or more nucleotide

monomers covalently bonded in a chain

DNA (deoxyribonucleic acid) and RNA

(ribonucleic acid) are examples of

polynucleotides with distinct biological

function

Q216 Vertebrates belongs to the

phylum

रीढ़कीहडिीवािहॳपराणी mdashmdashmdash

परजानतकहॳ अतगायतआतहॳहहॴ 3-Feb-2017

Options

1) Arthropoda

आरह८पह८ड़ा 2) Annelida

एननलििा 3) Cnidaria

ननिहॳररया 4) Chordata

कह८िटा Correct Answer Chordata

Q217 Punica granatum is the scientific name of

पननकगरहॳनहॳटस mdashmdashmdash कावहॴजञाननकनामहहॴ 3-Feb-2017

Options

1) Custard Apple

सीताफि

2) Gulmohar

गिमह८हर

3) Silver Oak

लसमवरओक

4) Pomegranate

अनार

Correct Answer Pomegranate

F A C E B O O K

P A G E h t t p w w w f a c e b o o k c o m s s c m e n t o r s o f f i c i a l P a g e | 50

FOR MORE UPDATES AND MORE MATERIAL DO LIKE OUR FACEBOOK PAGE httpwwwfacebookcomsscmentorsofficial

Q218 Between a tiger and an monkey

which of the following is different

एकबाघऔरबदरकहॳ बीचनननननलिखितमसहॳकह९नसीबातअिगहहॴ 3-Feb-2017

Options

1) Kingdom

राजय

2) Phylum

जानत

3) Order

िम

4) Class

वगय Correct Answer order

Q219 The artificial heart was invented by

कबतरमहदयका mdashmdashmdash

दवाराअपवषकारककयागयािा 3-Feb-2017

Options

1) Muhammad Yunus

महनमदयनस

2) Linus Yale Jr

िाइनसयहॳिजय

3) Gazi Yasargil

गाजीयासचगयि

4) Paul Winchell

पह९िपवमकि Correct Answer Paul Winchell

Q220 Tamarindus indica is the

scientific name of

टहॳमररनडसइडिका mdashmdash कावहॴजञाननकनामहहॴ 7-

Feb-2017

Options

1) Neem

नीम

2) Pineapple

अनानास

3) Tamarind

इमिी 4)Chiku

चीक

Correct Answer Tamarind

Q221 In eukaryotic cells synthesis of

RNA takes place in the

यकहॳ योटटककह८लिकाओमआरएनएकासशिहॳषण

mdashndash महह८ताहहॴ 7-Feb-2017

Options

1) Mitochondria

माईटह८कोडडरया 2) Centrioles

सटरीयह८मस

3) Ribosomes

ररबह८सह८नस

4) Nucleus

नयनकियस

Correct Answer nucleus

eukaryotic cell -Transcription is the

process of synthesizing ribonucleic acid

(RNA)Synthesis takes place within the

nucleus of eukaryotic cells or in the

cytoplasm of prokaryotes and converts

the genetic code from a gene in

deoxyribonucleic acid ( DNA ) to a

strand of RNA that then directs

proteinsynthesis

Q222 _________is caused by parasites

of the Plasmodium genus

पिाजमह८डियमजातीकहॳ परजीवी mdash- कहॳ कारणहहॴ 7-Feb-2017

Options

1) Dysentery

पहॳचचि

2) Malaria

मिहॳररया 3) Chickenpox

F A C E B O O K

P A G E h t t p w w w f a c e b o o k c o m s s c m e n t o r s o f f i c i a l P a g e | 51

FOR MORE UPDATES AND MORE MATERIAL DO LIKE OUR FACEBOOK PAGE httpwwwfacebookcomsscmentorsofficial

चहॳचक

4) Herpes

हहॳपपयस

Correct Answer Malaria

Q223 Carotene in fruits and vegetables

gives it which color

फिह८औरसनलजयोमनसितकहॳ रह८टीनउनहकह९नसारगपरदानकरताहहॴ 7-Feb-2017

Options

1) Green

हरा 2) Pink

गिाबी 3) Orange

नारगी 4) Blue

नीिा Correct Answer Orange

Q224 Equus Caballus is the scientific

name of

एकवसकहॴ बहॳिस mdashmdashndash कापवजञाननकनामहहॴ 7-Feb-2017

Options

1) Horse

घह८िा 2) Zebra

ज़हॳिा 3) Donkey

गधा 4) Buffalo

भस

Correct Answer Horse

Q225 Elapidae Naja is the scientific name of

एिीपीिीनाजा mdashmdash- कावहॴजञाननकनामहहॴ 8-Feb-2017

Options

1) Cobra

कह८बरा 2) Elephant

हािी 3) Eagle

ग ि

4) Owl

उमि Correct Answer Cobra

Q226 Which disease is caused due to

deficiency of Iron

िह८हकीकमीकहॳ कारणकह९नसारह८गहह८ताहहॴ 8-Feb-

2017

Options

1) Beriberi

बहॳरीबहॳरी 2) Tetany

टहॳटनी 3) Kwashiorkor

कवािीऔरकर

4) Anaemia

रकतामपता Correct Answer Anaemia

Beriberi is a disease caused by a vitamin

B-1 deficiency also known as thiamine

deficiency

Tetany can be the result of an

electrolyte imbalance Most often itrsquos a

dramatically low calcium level also

known as hypocalcemia Tetany can also

be caused by magnesium deficiency or

too little potassium Having too much

acid (acidosis) or too much alkali

(alkalosis) in the body can also result in

tetany

Kwashiorkor also known as

ldquoedematous malnutrition It is a form of

malnutrition caused by a lack of protein

in the diet

Anaemia means that you have fewer red

blood cells than normal or you have less

F A C E B O O K

P A G E h t t p w w w f a c e b o o k c o m s s c m e n t o r s o f f i c i a l P a g e | 52

FOR MORE UPDATES AND MORE MATERIAL DO LIKE OUR FACEBOOK PAGE httpwwwfacebookcomsscmentorsofficial

haemoglobin than normal in each red

blood cell

Q227 is a leaf where the leaflets are

arranged along the middle vein

mdashndashएकपततीहहॴजहापतरकह८कीरचनाक ररयालिराकहॳ आसपासहह८तीहहॴ 8-Feb-2017

Options

1) Pinnately compound leaf

पपनहॳटिीसयकतपतती 2) Palmately compound leaf

पामहॳटिीसयकतपतती 3) Compound leaf

सयकतपतती 4) Simple leaf

साधारणपतती Correct Answer Pinnately compound

leaf

Q228 Haustoria or sucking roots are

found in which of the following

हह८सटह८ररयायाचसनहॳवािीजड़हॳनननननलिखितमसहॳककसमपाईजातीहहॴ 8-Feb-2017

Options

1) Wheat

गहॳह

2) Mango

आम

3) Chestnut

चहॳसटनट

4) Cuscuta

कसकयटा Correct Answer Cuscuta

Haustorial roots -The roots of parasitic

plants which penetrate into the host

tissues to absorb nourishment are

called haustorial roots hellip Also known as suckingor parasitic roots

Q229 Equs Asinus is the scientific name

of

एकवसएलसनस mdashmdashndash कावहॴजञाननकनामहहॴ 8-

Feb-2017

Options

1) Donkey

गधा 2) Cow

गाय

3) Deer

टहरन

4) Kangaroo

कगा

Correct Answer Donkey

Q230 Ficus benghalensis is the scientific name of

फाईकसबहॳनगहॳिहॳलसस mdashndash कापवजञाननकनामहहॴ 8-Feb-2017

Options

1) Banyan

बरगद

2) Pineapple

अनानास

3) Babul

बबि

4) Tulsi

तिसी Correct Answer Banyan

Q231 Equus burchellii is the scientific name of

एकवसबचिी mdashmdash- कापवजञाननकनामहहॴ 8-Feb-2017

Options

1) Horse

घह८िा 2) Zebra

जहॳिा 3) Buffalo

F A C E B O O K

P A G E h t t p w w w f a c e b o o k c o m s s c m e n t o r s o f f i c i a l P a g e | 53

FOR MORE UPDATES AND MORE MATERIAL DO LIKE OUR FACEBOOK PAGE httpwwwfacebookcomsscmentorsofficial

भस

4) Ass

गधा Correct Answer Zebra

Page 38: COMPILATION OF ALL 72 SETS OF BIOLOGY SSC CHSL-2016 · OF BIOLOGY SSC CHSL-2016 PREPARED BY : SSC MENTORS BIOLOGY SPECIAL . F A C E B O O K P A G E : h t t p : / / w w w . f a c e

F A C E B O O K

P A G E h t t p w w w f a c e b o o k c o m s s c m e n t o r s o f f i c i a l P a g e | 37

FOR MORE UPDATES AND MORE MATERIAL DO LIKE OUR FACEBOOK PAGE httpwwwfacebookcomsscmentorsofficial

Options

1) Aorta

महाधमनी 2) pulmonary artery

फहॳ फड़हॳकीधमनी 3) vena cava

वहॳनाकावा 4)Atrium

चह९क

Correct Answer aorta

Q157 Blood leaving the liver and

moving towards the

heart has a higher concentration of

नजगरसहॳननकिकरहदयकीतरफजानहॳवािहॳरकतम mdashmdashmdashmdash कीउचचसादरताहह८तीहहॴ 27-Jan-2017

Options

1) Lipids

लिपपडस

2) Urea

यररया 3) Bile Pigments

पपततकहॳ रगकरण

4) Carbon dioxide

काबयनिायऑकसाइि

Correct Answer Bile Pigments

Urea is nitrogen containing substance

which is produced in the liver in order

to deal with excess amino-acids in the

body As urea is produced it leaves the

liver in the blood stream and passes via

the circulatory system to all parts of the

body

Q158 Bulb is a modification of which

part of a plant

बमबएकपह९धहॳकहॳ ककसटहससहॳकाएक पातरणहह८ताहहॴ 27-Jan-2017

Options

1) The root

जड़

2) The stem

तना 3) The radicle

मिाकर

4)The fruit

फि

Correct Answer The stem

Q159 Which of the following carries

blood away from the heart to different

body parts

इनमहॳसहॳकह९नरकतकह८हदयसहॳिरीरकहॳ पवलभननअगह८तकिहॳजातीहहॴ

27-Jan-2017

Options

1) Arteries

धमननया 2) Nerves

तबतरहाए

3) Capillaries

कहॳ लिकाए

4)Veins

नसहॳ Correct Answer Arteries

Q160 The series of processes by which

nitrogen and its compounds are

interconverted in the environment and

in living organisms is called

27-Jan-2017

Options

1)Absorption of Nitrogen

2)Ammonification

3)Nitrogen Fixation

4)Nitrogen Cycle

Correct Answer Nitrogen Cycle

Ammonification or Mineralization is

performed by bacteria to convert

organic nitrogen to ammonia

F A C E B O O K

P A G E h t t p w w w f a c e b o o k c o m s s c m e n t o r s o f f i c i a l P a g e | 38

FOR MORE UPDATES AND MORE MATERIAL DO LIKE OUR FACEBOOK PAGE httpwwwfacebookcomsscmentorsofficial

Nitrification can then occur to convert

the ammonium to nitrite and nitrate

Nitrogen fixation is a process by which

nitrogen in the Earthrsquos atmosphere is

converted into ammonia (NH3) or other

molecules available to living organisms

Q161 BCG vaccine is given to protect

from which of the following

बीसीजीकाटटकानननननलिखितमसहॳककसकहॳ बचावकहॳ लिएटदयाजातहहॴ

27-Jan-2017

Options

1) Jaundice

पीलिया 2) Anaemia

रकतमपता 3) Tuberculosis

कषयरह८ग

4) Polio

पह८लियह८ Correct Answer Tuberculosis

Q162 Parallel venation is found in

समानतरवहॳनहॳिन mdashmdashmdash- मपायाजाताहहॴ 27-Jan-2017

Options

1) plants which are monocots

पह९धहॳजह८एकबीजपतरीहह८तहॳहहॴ 2) plants which have a dicot stem

वहॳपह९धहॳनजनकातनादपवदलियहह८ताहहॴ 3) plants with leaves similar to Tulsi

वहॳपह९धहॳनजनकीपनततयतिसीकीपनततयोकहॳ समानहह८तहॳहहॴ 4)plants with tap roots

टहॳप टवािहॳपह९धहॳ Correct Answer plants which are

monocots

Q163 The hardest part of the body is

िरीरकासबसहॳकठह८रभाग mdashndash हहॴ 27-Jan-2017

Options

1) Bones

हडडिय

2) Tooth Enamel

दातकहॳ इनहॳमि

3) Skull

िह८पड़ी 4) Spinal Cord

महॳ रजज

Correct Answer Tooth Enamel

Q164 Which type of pathogen causes

the waterborne disease E coli Infection

ककसपरकारकारह८गजननकजिजननतरह८गईकह८िाईसिमणकाकारणबनताहहॴ 27-Jan-2017

Options

1) Protozoan

परह८टह८जआ

2) Parasitic

परजीवी 3) Bacterial

बहॴकटीररयि

4)Viral

वायरि

Correct Answer Bacterial

Q165 The amount of blood filtered

together by both the kidneys in a 70 kg

adult male human in a minute is

70 की गरा वािहॳएकवयसकप षमएकलमनटमदह८नोगदकहॳदवाराएकसािचाबनीगयीरकतकीमातरहह८तीहहॴ 29-Jan-2017

Options

1) 1100 ml

1100 लमलि

2) 100 ml

F A C E B O O K

P A G E h t t p w w w f a c e b o o k c o m s s c m e n t o r s o f f i c i a l P a g e | 39

FOR MORE UPDATES AND MORE MATERIAL DO LIKE OUR FACEBOOK PAGE httpwwwfacebookcomsscmentorsofficial

100 लमलि

3) 1500 ml

1500 लमलि

4) 500 ml

500 लमलि

Correct Answer 1100 ml

Q166 Which feature of a plant helps to

distinguish a monocot from a dicot

पह९धहॳकीवहकह९नसीपविहॳषताहहॴजह८एकदपवदलियहॳऔरएकएकदिीयपह९धहॳसहॳभहॳदकरनहॳममददकरतीहहॴ 29-Jan-2017

Options

1) Pollination

परागम

2) Venation

वहॳनहॳिन

3) Vernation

वनिन

4) Aestivation

एसटीवहॳिहॳन

Correct Answer venation

Q167 The Mutation Theory was

proposed by

उतवररवतयनकालसदात mdashmdashndash

कहॳ दवरापरसतापवतककयाजाताहहॴ 29-Jan-2017

Options

1) Charles Lyell

चामसयलियहॳि

2) William Smith

पवलियमनसमि

3) Hugo De Vries

हयगह८िीराईस

4)Harrison Schmitt

हहॳरीसननसमट

Correct Answer Hugo De Vries

Q168 Which type of pathogen causes

the waterborne disease HepatitisA

ककसपरकारकहॳ रह८गजनकजिजननतरह८गहहॳपहॳटाइटटस-A काकारणबनताहहॴ

29-Jan-2017

Options

1) Parasitic

परजीवी 2) Viral

वायरि

3) Protozoan

परह८टह८जआ

4) Bacterial

बहॴकटीररयि

Correct Answer Viral

Q169 In a Punnett Square with the

cross AaBb x Aabb how many AaBb

genotypes would be created

पनहॳटसकवायरमिह८स AaBb x Aabb

कहॳ सािककतनहॳ AaBb जीनह८टाइपबनगहॳ 29-Jan-

2017

Options

1) 4

2) 1

3) 7

4) 6

Correct Answer 4

Q170 Arboreal Ateles is the scientific

name of

अिह८ररयिएटटलिस mdashmdashmdash कावहॴजञाननकनामहहॴ 29-Jan-2017

Options

1) Squirrel

चगिहरी 2) Sparrow

गह८रहॴया 3) Lizard

नछपकिी 4) Spider monkey

F A C E B O O K

P A G E h t t p w w w f a c e b o o k c o m s s c m e n t o r s o f f i c i a l P a g e | 40

FOR MORE UPDATES AND MORE MATERIAL DO LIKE OUR FACEBOOK PAGE httpwwwfacebookcomsscmentorsofficial

मकड़ीबदर

Correct Answer Spider monkey

Q171 Which type of pathogen causes

the waterborne disease Salmonellosis

ककसपरकारकारह८गाणजिजननतबीमारीसािमह८नहॳिह८लसज़काकारकहहॴ

29-Jan-2017

Options

1) Algal

िहॳवालियहॳ 2) Parasitic

परजीवी 3) Bacterial

बहॴकटीररयि

4)Viral

वायरि

Correct Answer Bacterial

An infection with salmonella bacteria

commonly caused by contaminated food

or water

Symptoms include diarrhoea fever

chills and abdominal pain

Q172 is a condition in which there is a

deficiency of red cells or of haemoglobin

in the blood

mdashmdash-

एकनसिनतहहॴनजसमहॳरकतमिािकह८लिकाओकीयाहीमह८गिह८बबनकीकमीहह८तीहहॴ 29-Jan-2017

Options

1) Albinism

एनमबननजम

2) Propyria

परह८पीररया 3) Anaemia

एनीलमया 4)Keloid disorder

कहॳ िह८इिडिसओिर

Correct Answer Anaemia

Q173 Ananas comosus is the scientific

name of

Options

अनानासकह८मह८सस mdashmdashmdashndash

कावहॴजञाननकनामहहॴ 29-Jan-2017

1) Custard Apple

सीताफि

2) Pineapple

पाइनएपपि

3) Bamboo

बास

4)Pomegranate

अनार

Correct Answer Pineapple

Q174 Which organ produces insulin

कह९नसाअगइनसलिनपहॴदाकरताहहॴ 29-Jan-

2017

Options

1) Liver

यकत

2) Thyroid gland

िायराइिगरिी 3) Spleen

पिीहा 4)Pancreas

अगरयिय

Correct Answer Pancreas

Q175 Which of the following disease is

not caused by water pollution

नननननलिखितमसहॳकह९नसारह८गपानीकहॳ परदषणकहॳकारणनहीहह८ता

29-Jan-2017

Options

1) Cholera

हहॴजा 2) Typhoid

F A C E B O O K

P A G E h t t p w w w f a c e b o o k c o m s s c m e n t o r s o f f i c i a l P a g e | 41

FOR MORE UPDATES AND MORE MATERIAL DO LIKE OUR FACEBOOK PAGE httpwwwfacebookcomsscmentorsofficial

टाइफाइि

3) Asthma

दमा 4)Diarrhoea

दसत

Correct Answer Asthma

Q176 Ocimum tenuiflorum is the

scientific name of

ओलिलममटहॳयईफिह८रमइसकावहॴजञाननकनाम mdash

ndash हहॴ 30-Jan-2017

Options

1) Neem

नीम

2) Mango

आम

3) Babul

बबि

4)Tulsi

तिसी Correct Answer Tulsi

Q177 Which gland secretes bile a

digestive fluid

कह९नसीगरिीपपतत एकपाचनतरिपरदािय सरापवतकरतीहहॴ 30-Jan-2017

Options

1) Pancreas

अगनयािय

2) Liver

यकत

3) Thyroid

िायराइि

4) Testes

टहॳनसटस

Correct Answer liver

Q178 In which of the following the

dominant phase is Gametophyte

नननननलिखितमसहॳककसकहॳ परमिचरणयगमकह८दपवधद (Gametophyte)हहॴ 30-Jan-2017

Options

1) Bryophyta

िायह८फाइटा 2) Pteridophyta

टहॳररिह८फाइटा 3) Gymnosperms

नजननह८सपमय 4) Angiosperms

एननजयह८सपमय Correct Answer Bryophyta

Q179 Anaerobic respiration refers to

which of the following

नननननलिखितमसहॳककसहॳअवायवीयशवसनकहाजाताहहॴ

30-Jan-2017

Options

1) Respiration without Oxygen

ऑकसीजनकहॳ बबनाशवसन

2) Respiration with Oxygen

ऑकसीजनकहॳ सािशवसन

3) Respiration without CO2

काबयनिायऑकसाइिकहॳ बबनाशवसन

4) Respiration with CO2

काबयनिायऑकसाइिकहॳ सािशविन

Correct Answer Respiration without

Oxygen

Q180 Which type of pathogen causes

the waterborne disease Cholera

ककसपरकारकारह८गजनकजिजननतरह८गहहॴजाकाकारणबनताहहॴ

30-Jan-2017

Options

1) Algal

िहॴवालियहॳ

F A C E B O O K

P A G E h t t p w w w f a c e b o o k c o m s s c m e n t o r s o f f i c i a l P a g e | 42

FOR MORE UPDATES AND MORE MATERIAL DO LIKE OUR FACEBOOK PAGE httpwwwfacebookcomsscmentorsofficial

2) Bacterial

बहॴकटीररयि

3) Protozoan

परह८टह८जआ

4) Viral

वायरि

Correct Answer Bacterial

Q181 To which class does

Oxyreductases transferases hydrolases

belong

ओकसीररिकटहॳसटरासफरहॳजहॳस

हाइडरह८िहॳसहॳसककसवगयमआतहॳहहॴ 30-Jan-2017

Options

1) Hormones

हारमोस

2) Enzymes

एजाइनस

3) Proteins

परह८टीनस

4) Vitamins

पवटालमनस

Correct Answer Enzymes

Q182 Which of the following is not true

about Gymnosperms

ननननमसहॳकह९नसीबातअनावतबीजीकहॳ बारहॳमसचनहीहहॴ 30-Jan-2017

Options

1) Dominant phase is saprophytes

परमिचरणसहॳपरह८फाइटसहह८ताहहॴ 2) Vascular bundles are absent

सवहनीबििअनपनसितहह८ताहहॴ 3) spores are heterospores

बीजाणहहॳटहॳरह८सपह८रसहह८तहॳहहॴ 4) Flowers are absent

फिअनपनसितहह८तहॳहहॴ

Correct Answer Vascular bundles are

absent

Q183 The name of first mammal clone sheep is

भहॳड़कीपरिमसतनपायीपरनत प (किह८न)

कानामहहॴ 30-Jan-2017

Options

1) Noori

नरी 2) Dolly

िॉिी 3) Louise

िसी 4)Durga

दगाय Correct Answer Dolly

Q184 Which type of pathogen causes

the water-borne disease Typhoid fever

ककसपरकारकारह८गजनकजिजननतरह८गटाइफाइिबिारकाकारणबनताहहॴ 30-Jan-2017

Options

1) Algal

िहॴवािीय

2) Parasitic

परजीवी 3) Protozoan

परह८टह८जनअन

4)Bacterial

बहॴकटीररयि

Correct Answer Bacterial

Q185 In which part of the cell are

proteins made

कह८लिकाकहॳ ककसटहससहॳमपरह८टीनबनायाजाताहहॴ

31-Jan-2017

Options

1) Reticulum

रहॳटटकिम

F A C E B O O K

P A G E h t t p w w w f a c e b o o k c o m s s c m e n t o r s o f f i c i a l P a g e | 43

FOR MORE UPDATES AND MORE MATERIAL DO LIKE OUR FACEBOOK PAGE httpwwwfacebookcomsscmentorsofficial

2) Golgi apparatus

गह८मजीएपहॳरहॳटस

3) Ribosomes

ररबह८सह८नस

4) Lysosome

िायसह८सह८नस

Correct Answer ribosomes

Proteins are produced by stringing

amino acids together in the order

specified by messenger RNA strands

that were transcribed from DNA in the

cell nucleus The process of synthesizing

a protein is called translation and it

occurs on ribosomes in the cytoplasm of

a cell

Q186 Polio is a disease caused by which

of the following

नननननलिखितमसहॳपह८लियह८कीबबमारह८हह८नहॳकाकारणकयाहहॴ

31-Jan-2017

Options

1) Bacteria

बहॴकटीररयि

2) Mosquito

मचछर

3) Virus

वायरस

4) Cockroach

नतिच हॳ Correct Answer Virus

Polio or poliomyelitis is a crippling and

potentially deadly infectious disease It

is caused by the poliovirus

Q187 ndash Hay fever is a sign of which of

the following

हहॳकफवरनननननलिखितमसहॳककसकाएकसकहॳ तहहॴ

31-Jan-2017

Options

1) Old Age

वदावसिा 2) Malnutrition

कपह८सण

3) Allergy

एिनजय 4) Over Work

अतयचधककाययकरना Correct Answer Allergy

Q188 How many chromosomes does a

human cell contain

एकमानवकह८लिकामककतनहॳगणसतरहह८तहॳहहॴ

29-Jan-2017

Options

1) 6

2) 26

3) 46

4) 66

Correct Answer 46

In humans each cell normally contains

23 pairs of chromosomes for a total of

46 Twenty-two of these pairs called

autosomes look the same in both males

and females The 23rd pair the sex

chromosomes differ between males and

females

Q189 Which of the following is not true

about Bryophyta

ननननमसहॳकह९नसीबातिायह८फाइटकहॳ बारहॳमसचनहीहहॴ 31-Jan-2017

Options

1) Dominant phase is gametophytes

परमिचरणगहॳलमतह८फाइटसहह८ताहहॴ 2) Main plant body is haploid

पह९धहॳकामखयिरीरअगखणतहह८ताहहॴ 3) Spores are homospores

बीजाणहह८मह८सफह८रसहह८तहॳहहॴ 4) Flowers are present

फिमह८जदहह८तहॳहहॴ Correct Answer Flowers are present

F A C E B O O K

P A G E h t t p w w w f a c e b o o k c o m s s c m e n t o r s o f f i c i a l P a g e | 44

FOR MORE UPDATES AND MORE MATERIAL DO LIKE OUR FACEBOOK PAGE httpwwwfacebookcomsscmentorsofficial

Q190 Which aquatic animal has

trailing tentacles

ककसजिीयजानवरकहॳ पीछहॳचिनहॳवािहॳटहॳटकिसहह८तहॳहहॴ

31-Jan-2017

Options

1) Sea horse

समदरीघह८िा 2) Corals

मगा 3) Jelly fish

जहॳिीमछिी 4) Star fish

तारामछिी Correct Answer Jelly fish

Jellyfish with its umbrella-shaped bell

and trailing tentacles

Q191 Which type of pathogen causes

the water-borne disease Poliomyelitis

(Polio)

ककसपरकारकारह८गजनकजिजननतरह८गपह८लियह८मायहॳटटस (पह८लियह८) काकारणहहॴ 31-Jan-

2017

Options

1) Parasitic

परजीवी 2) Algal

िहॴवालिय

3) Viral

वायरि

4) Bacterial

बहॴकटीररयि

Correct Answer Viral

Q192 The outer white part of the eye

that protects the inner structures is

आािकाबाहरीसफहॳ दटहससाजह८आतररकसरचनाओकीरकषाकरताहहॴ वह mdashmdashmdash हहॴ 31-Jan-

2017

Options

1) Iris

आयररस

2) Sclera

सकिहॳरा 3) Retina

रहॳटटना 4) Cornea

कह८ननयया Correct Answer Sclera

Q193 Proteins are made up of

परह८टीनकाननमायण mdashndash सहॳहह८ताहहॴ 31-Jan-2017

Options

1) Amino acids

एलमनह८अनि

2) Fatty acids

वसायकतअनि

3) Glucose

गिकह८ज

4)Nucleotides

नयनकियह८टाईिस

Correct Answer Amino acids

Q194 Moringa Oleifera is the scientific

name of

मह८ररगओलिफहॳ रा mdashmdashndash कावहॴजञाननकनामहहॴ 31-Jan-2017

Options

1) Banyan

बरगद

2) Gulmohar

गिमह८हर

3) Amla

आमिा

F A C E B O O K

P A G E h t t p w w w f a c e b o o k c o m s s c m e n t o r s o f f i c i a l P a g e | 45

FOR MORE UPDATES AND MORE MATERIAL DO LIKE OUR FACEBOOK PAGE httpwwwfacebookcomsscmentorsofficial

4) Drumstick

डरमनसटक

Correct Answer Drumstick

Q195 Kidney stones are composed of

गदकीपिरी mdashndash सहॳबनीहह८तीहहॴ 1-Feb-2017

Options

1) Calcium Oxalate

कहॴ नमसयमओकजहॳिहॳट

2) Sodium Chloride

सह८डियमकिह८राइि

3) Magnesium Nitrate

महॳनगनलियमनाइतटरहॳट

4) Calcium Bicarbonate

कहॴ नमियमबायकबोनहॳट

Correct Answer Calcium Oxalate

Q196 ndash Which of the following is not

true about Angiosperms

ननननमसहॳकह९नसीबातआवतबीजीकहॳ बारहॳमसचनहीहहॴ 1-Feb-2017

Options

1) Dominant phase is gametophytes

परमिचरणगहॳलमतह८फाइटहह८ताहहॴ 2) Vascular bundles are present

सवहनीबििमह९जदहह८ताहहॴ 3) Spores are heterospores

बीजाणहहॳटहॳरह८सपह८रसहह८तहॳहहॴ 4) Seeds are covered

बीजढकहॳ हह८तहॳहहॴ Correct Answer Dominant phase is

gametophytes

Q197 All of the following are excretory

(waste) products of animals except

नननननलिखितमसहॳककसएककह८छह८ड़करअनयसभीपराखणयोदवाराउतसनजयतपदाियहहॴ 1-Feb-

2017

Options

1) Uric Acid

यररकएलसि

2) Ammonia

अमह८ननया 3) Carbohydrates

काबोहाइडरहॳट

4) Urea

यररया Correct Answer Carbohydrates

In animals the main excretory products

are carbon dioxide ammonia (in

ammoniotelics) urea (in ureotelics) uric

acid (in uricotelics) guanine (in

Arachnida) and creatine

Q198 RNA is a polymeric molecule

What does RNA stand for

आरएनइएएकबहिकआणहहॴ इसकाकापवय पकयाहहॴ 1-Feb-2017

Options

1) Rado Nuclear Acid

रािह८नयनकियरएलसि

2) Ribo Nucleic Acid

राइबह८नयनकिकएलसि

3) Rhino Nuclear Acid

हाइनह८नयनकियरएलसि

4) Resto Nucleus Acid

रहॳसटह८नयकिीयसएलसि

Correct Answer Ribo Nucleic Acid

Q199 Which organ does detoxification

and produces chemicals needed for

digestion

कह९नसाअगपवषहरणकरताहहॴऔरपाचनकहॳ लिएआवशयकरसायनोकह८पहॴदाकरताहहॴ 1-Feb-

2017

Options

1) Salivary glands

िारगरचिया 2) Pancreas

अगनयािय

F A C E B O O K

P A G E h t t p w w w f a c e b o o k c o m s s c m e n t o r s o f f i c i a l P a g e | 46

FOR MORE UPDATES AND MORE MATERIAL DO LIKE OUR FACEBOOK PAGE httpwwwfacebookcomsscmentorsofficial

3) Thyroid gland

िायराइिगरिी 4) Liver

यकत

Correct Answer Liver

Q200 Psidium guajava is the scientific

name of

लसडियमगआजावा mdashmdash कावहॴजञाननकनामहहॴ 1-

Feb-2017

Options

1) Guava

अम द

2) Mango

आम

3) Bamboo

बास

4) Jack fruit

कटहि

Correct Answer Guava

Q201 Which drug is used as a Blood

Thinner

चधरकह८पतिाकरनहॳकहॳ पमककसदवाकापरयह८गककयाजाताहहॴ

1-Feb-2017

Options

1) Warfarin

वाफर न

2) Tramadol

टरहॳमािह८ि

3) Azithromycin

एनजरह८मायलसन

4) Hydralazine

हाइडरह८िहॳनजन

Correct Answer Warfarin

Q202 Which of the following disease is

caused due to the deficiency of protein

परह८टीनकीकमीकहॳ कारणनननननलिखितमसहॳकह९नसारह८गहह८ताहहॴ 1-Feb-2017

Options

1) Arthritis

गटठया 2) Kwashiorkor

कािीओकय र

3) Goitre

गाइटर

4) Night Blindness

रतह९चध

Correct Answer Kwashiorkor

Q203 A is species of plant that has

adapted to survive in an environment

with little liquid water

mdashmdashndashपह९धहॳकीएकऐसहॳऐसहॳपरजानतहहॴ नजसनहॳकमपानीवािहॳवातावरणमजीपवतरहनहॳकहॳलिएअनकिनहहॴ 1-Feb-2017

Options

1) Xerophyte

म दपवद

2) Hydrophyte

जिीयपादप

3) Mesophyte

समह८दपवद

4) Thallophyte

िहॴिह८फाइटा Correct Answer xerophyte

xerophyte is a species of plant that has

adapted to survive in an environment

with little liquid water such as a desert

or an ice- or snow-covered region in the

Alps or the Arctic

Mesophytes are terrestrial plants which

are adapted to neither a particularly

dry nor particularly wet environment

An example of a mesophytic habitat

would be a rural temperate meadow

F A C E B O O K

P A G E h t t p w w w f a c e b o o k c o m s s c m e n t o r s o f f i c i a l P a g e | 47

FOR MORE UPDATES AND MORE MATERIAL DO LIKE OUR FACEBOOK PAGE httpwwwfacebookcomsscmentorsofficial

which might contain goldenrod clover

oxeye daisy and Rosa multiflora

thallophyte any of a group of plants or

plantlike organisms (such as algae and

fungi) that lack differentiated stems

leaves and roots and that were formerly

classified as a primary division

(Thallophyta) of the plant kingdom

Q204 How many types of teeth are

there in humans

मनषयोमककतनहॳपरकारकहॳ दातहह८तहॳहहॴ

1-Feb-2017

Options

1) 4

2) 5

3) 2

4) 3

Correct Answer 4

teeth -Humans have four types of

teethincisors canines premolars and

molars each with a specific function

The incisors cut the food the canines

tear the food and the molars and

premolars crush the food

Q205 Carica papaya is the scientific name of

कहॴ ररकापपाया mdashmdashndash कावहॴजञाननकनामहहॴ 2-

Feb-2017

Options

1) Peepal

पीपि

2) Papaya

पपीता 3) Tamarind

इमिी 4) Drumstick

ढह८िकाछड़ी Correct Answer Papaya

Q206 Muscles get tired when there is

shortfall of

जब mdashndash कीकमीहह८तीहहॴतबपहॳिीयिकजातीहहॴ 2-Feb-2017

Options

1) Lactic acid

िहॴनकटकएलसि

2) Na+ ions

Na+ आयन

3) ATP

एटीपी 4) Sulphates

समफहॳ टस

Correct Answer ATP

ATP is the energy source muscle fibers

use to make muscles contract

muscle tissuersquos main source of energy

called adenosine triphosphate or ATP

As your muscles use up this energy

source they become tired and fatigued

Oxygen is the key ingredient that helps

create new ATP to replenish the burned

up ATP in your muscles

Q207 Artocarpus integra is the

scientific name of आटह८कापयसइटीगरा mdashmdashmdash कावहॴजञाननकनामहहॴ 2-Feb-2017

Options

1) Guava

अम द

2) Pineapple

अनानास

3) Silver Oak

लसमवरओक

4) Jack fruit

कटहि

Correct Answer Jack fruit

Q208 Which organ stores fat soluble

vitamins

कह९नसाअगवसामघिनिीिपवटालमनह८काभिाराकरताहहॴ

2-Feb-2017

F A C E B O O K

P A G E h t t p w w w f a c e b o o k c o m s s c m e n t o r s o f f i c i a l P a g e | 48

FOR MORE UPDATES AND MORE MATERIAL DO LIKE OUR FACEBOOK PAGE httpwwwfacebookcomsscmentorsofficial

Options

1) Blood

रकत

2) Skin

तवचा 3) Liver

यकत

4) Pancreas

अगनयािय

Correct Answer Liver

Q209 Which disease is caused due to

deficiency of Iodine

आयह८िीनकहॳ कारणकह९नसारह८गहह८ताहहॴ 2-Feb-2017

Options

1) Rickets

ररकहॳ टस

2) Scurvy

सकवी 3) Goitre

गणमािा 4) Growth retardation

पवकासका कना Correct Answer Goitre

rickets A softening and weakening of

bones in children usually due to

inadequate vitamin D

Q210 Grevillea Robusta is the scientific name of

गरहॳपवलियारह८बसटा mdashmdashmdash- कापवजञाननकनामहहॴ 2-Feb-2017

Options

1) Peepal

पीपि

2) Teak

सागह९न

3) Silver Oak

लसमवरओक

4) Jack fruit

कटहि

Correct Answer Silver Oak

Q211 When a Cuttlefish is described as a Molluscs it is at which level of

classification

जबएककटिकफिकह८एकमह८िसकाकहॳ पमवखणयतककयाजाताहहॴतबयहॳवगीकरणकहॳ ककससतरपहॳनसितहहॴ 2-Feb-2017

Options

1) Class

वगय 2) Order

िम

3) Family

पररवार

4) Phylum

सघ

Correct Answer Phylum

Q212 Bambusa dendrocalmus is the

scientific name of बानबसािहॳडराकामस mdashmdashmdash कावहॴजञाननकनामहहॴ 3-Feb-2017

Options

1) Banyan

बरगद

2) Papaya

पपीता 3) Bamboo

बास

4) Pomegranate

अनार

Correct Answer Bamboo

Q213 Acinonyx Jubatus is the scientific name of

एलसनह८ननकसजयबहॳटस mdashmdashmdash

कावहॴजञाननकनामहहॴ 3-Feb-2017

F A C E B O O K

P A G E h t t p w w w f a c e b o o k c o m s s c m e n t o r s o f f i c i a l P a g e | 49

FOR MORE UPDATES AND MORE MATERIAL DO LIKE OUR FACEBOOK PAGE httpwwwfacebookcomsscmentorsofficial

Options

1) Bear

भाि 2) Horse

घह८िा 3) Cheetah

चीता 4) Zebra

जहॳिा Correct Answer Cheetah

Q214 The pale yellow colour of urine is

due to the presence of which pigment

मतरकाफीकापीिारगरगदरयकहॳ उपनसिनतकहॳ कारणहह८ताहहॴ

3-Feb-2017

Options

1) Urochrome

यरह८िह८म

2) Urophyll

यरह८कफि

3) Chlorophyll

किह८रह८कफि

4) Chloroplast

किह८रह८पिासट

Correct Answer Urochrome

Q215 Which of the following constitute

to form a gene

नननननलिखितमसहॳकह९नसीचीज़एकजीनकागठनकरतीहहॴ

3-Feb-2017

Options

1) Polynucleotides

पह८िीनयनकियह८टाईडस

2) Hydrocarbons

हाइडरह८काबोस

3) Lipoproteins

िाईपह८परह८टीनस

4) Lipids

लिपपडस

Correct Answer Polynucleotides

Polynucleotide molecule is a biopolymer

composed of 13 or more nucleotide

monomers covalently bonded in a chain

DNA (deoxyribonucleic acid) and RNA

(ribonucleic acid) are examples of

polynucleotides with distinct biological

function

Q216 Vertebrates belongs to the

phylum

रीढ़कीहडिीवािहॳपराणी mdashmdashmdash

परजानतकहॳ अतगायतआतहॳहहॴ 3-Feb-2017

Options

1) Arthropoda

आरह८पह८ड़ा 2) Annelida

एननलििा 3) Cnidaria

ननिहॳररया 4) Chordata

कह८िटा Correct Answer Chordata

Q217 Punica granatum is the scientific name of

पननकगरहॳनहॳटस mdashmdashmdash कावहॴजञाननकनामहहॴ 3-Feb-2017

Options

1) Custard Apple

सीताफि

2) Gulmohar

गिमह८हर

3) Silver Oak

लसमवरओक

4) Pomegranate

अनार

Correct Answer Pomegranate

F A C E B O O K

P A G E h t t p w w w f a c e b o o k c o m s s c m e n t o r s o f f i c i a l P a g e | 50

FOR MORE UPDATES AND MORE MATERIAL DO LIKE OUR FACEBOOK PAGE httpwwwfacebookcomsscmentorsofficial

Q218 Between a tiger and an monkey

which of the following is different

एकबाघऔरबदरकहॳ बीचनननननलिखितमसहॳकह९नसीबातअिगहहॴ 3-Feb-2017

Options

1) Kingdom

राजय

2) Phylum

जानत

3) Order

िम

4) Class

वगय Correct Answer order

Q219 The artificial heart was invented by

कबतरमहदयका mdashmdashmdash

दवाराअपवषकारककयागयािा 3-Feb-2017

Options

1) Muhammad Yunus

महनमदयनस

2) Linus Yale Jr

िाइनसयहॳिजय

3) Gazi Yasargil

गाजीयासचगयि

4) Paul Winchell

पह९िपवमकि Correct Answer Paul Winchell

Q220 Tamarindus indica is the

scientific name of

टहॳमररनडसइडिका mdashmdash कावहॴजञाननकनामहहॴ 7-

Feb-2017

Options

1) Neem

नीम

2) Pineapple

अनानास

3) Tamarind

इमिी 4)Chiku

चीक

Correct Answer Tamarind

Q221 In eukaryotic cells synthesis of

RNA takes place in the

यकहॳ योटटककह८लिकाओमआरएनएकासशिहॳषण

mdashndash महह८ताहहॴ 7-Feb-2017

Options

1) Mitochondria

माईटह८कोडडरया 2) Centrioles

सटरीयह८मस

3) Ribosomes

ररबह८सह८नस

4) Nucleus

नयनकियस

Correct Answer nucleus

eukaryotic cell -Transcription is the

process of synthesizing ribonucleic acid

(RNA)Synthesis takes place within the

nucleus of eukaryotic cells or in the

cytoplasm of prokaryotes and converts

the genetic code from a gene in

deoxyribonucleic acid ( DNA ) to a

strand of RNA that then directs

proteinsynthesis

Q222 _________is caused by parasites

of the Plasmodium genus

पिाजमह८डियमजातीकहॳ परजीवी mdash- कहॳ कारणहहॴ 7-Feb-2017

Options

1) Dysentery

पहॳचचि

2) Malaria

मिहॳररया 3) Chickenpox

F A C E B O O K

P A G E h t t p w w w f a c e b o o k c o m s s c m e n t o r s o f f i c i a l P a g e | 51

FOR MORE UPDATES AND MORE MATERIAL DO LIKE OUR FACEBOOK PAGE httpwwwfacebookcomsscmentorsofficial

चहॳचक

4) Herpes

हहॳपपयस

Correct Answer Malaria

Q223 Carotene in fruits and vegetables

gives it which color

फिह८औरसनलजयोमनसितकहॳ रह८टीनउनहकह९नसारगपरदानकरताहहॴ 7-Feb-2017

Options

1) Green

हरा 2) Pink

गिाबी 3) Orange

नारगी 4) Blue

नीिा Correct Answer Orange

Q224 Equus Caballus is the scientific

name of

एकवसकहॴ बहॳिस mdashmdashndash कापवजञाननकनामहहॴ 7-Feb-2017

Options

1) Horse

घह८िा 2) Zebra

ज़हॳिा 3) Donkey

गधा 4) Buffalo

भस

Correct Answer Horse

Q225 Elapidae Naja is the scientific name of

एिीपीिीनाजा mdashmdash- कावहॴजञाननकनामहहॴ 8-Feb-2017

Options

1) Cobra

कह८बरा 2) Elephant

हािी 3) Eagle

ग ि

4) Owl

उमि Correct Answer Cobra

Q226 Which disease is caused due to

deficiency of Iron

िह८हकीकमीकहॳ कारणकह९नसारह८गहह८ताहहॴ 8-Feb-

2017

Options

1) Beriberi

बहॳरीबहॳरी 2) Tetany

टहॳटनी 3) Kwashiorkor

कवािीऔरकर

4) Anaemia

रकतामपता Correct Answer Anaemia

Beriberi is a disease caused by a vitamin

B-1 deficiency also known as thiamine

deficiency

Tetany can be the result of an

electrolyte imbalance Most often itrsquos a

dramatically low calcium level also

known as hypocalcemia Tetany can also

be caused by magnesium deficiency or

too little potassium Having too much

acid (acidosis) or too much alkali

(alkalosis) in the body can also result in

tetany

Kwashiorkor also known as

ldquoedematous malnutrition It is a form of

malnutrition caused by a lack of protein

in the diet

Anaemia means that you have fewer red

blood cells than normal or you have less

F A C E B O O K

P A G E h t t p w w w f a c e b o o k c o m s s c m e n t o r s o f f i c i a l P a g e | 52

FOR MORE UPDATES AND MORE MATERIAL DO LIKE OUR FACEBOOK PAGE httpwwwfacebookcomsscmentorsofficial

haemoglobin than normal in each red

blood cell

Q227 is a leaf where the leaflets are

arranged along the middle vein

mdashndashएकपततीहहॴजहापतरकह८कीरचनाक ररयालिराकहॳ आसपासहह८तीहहॴ 8-Feb-2017

Options

1) Pinnately compound leaf

पपनहॳटिीसयकतपतती 2) Palmately compound leaf

पामहॳटिीसयकतपतती 3) Compound leaf

सयकतपतती 4) Simple leaf

साधारणपतती Correct Answer Pinnately compound

leaf

Q228 Haustoria or sucking roots are

found in which of the following

हह८सटह८ररयायाचसनहॳवािीजड़हॳनननननलिखितमसहॳककसमपाईजातीहहॴ 8-Feb-2017

Options

1) Wheat

गहॳह

2) Mango

आम

3) Chestnut

चहॳसटनट

4) Cuscuta

कसकयटा Correct Answer Cuscuta

Haustorial roots -The roots of parasitic

plants which penetrate into the host

tissues to absorb nourishment are

called haustorial roots hellip Also known as suckingor parasitic roots

Q229 Equs Asinus is the scientific name

of

एकवसएलसनस mdashmdashndash कावहॴजञाननकनामहहॴ 8-

Feb-2017

Options

1) Donkey

गधा 2) Cow

गाय

3) Deer

टहरन

4) Kangaroo

कगा

Correct Answer Donkey

Q230 Ficus benghalensis is the scientific name of

फाईकसबहॳनगहॳिहॳलसस mdashndash कापवजञाननकनामहहॴ 8-Feb-2017

Options

1) Banyan

बरगद

2) Pineapple

अनानास

3) Babul

बबि

4) Tulsi

तिसी Correct Answer Banyan

Q231 Equus burchellii is the scientific name of

एकवसबचिी mdashmdash- कापवजञाननकनामहहॴ 8-Feb-2017

Options

1) Horse

घह८िा 2) Zebra

जहॳिा 3) Buffalo

F A C E B O O K

P A G E h t t p w w w f a c e b o o k c o m s s c m e n t o r s o f f i c i a l P a g e | 53

FOR MORE UPDATES AND MORE MATERIAL DO LIKE OUR FACEBOOK PAGE httpwwwfacebookcomsscmentorsofficial

भस

4) Ass

गधा Correct Answer Zebra

Page 39: COMPILATION OF ALL 72 SETS OF BIOLOGY SSC CHSL-2016 · OF BIOLOGY SSC CHSL-2016 PREPARED BY : SSC MENTORS BIOLOGY SPECIAL . F A C E B O O K P A G E : h t t p : / / w w w . f a c e

F A C E B O O K

P A G E h t t p w w w f a c e b o o k c o m s s c m e n t o r s o f f i c i a l P a g e | 38

FOR MORE UPDATES AND MORE MATERIAL DO LIKE OUR FACEBOOK PAGE httpwwwfacebookcomsscmentorsofficial

Nitrification can then occur to convert

the ammonium to nitrite and nitrate

Nitrogen fixation is a process by which

nitrogen in the Earthrsquos atmosphere is

converted into ammonia (NH3) or other

molecules available to living organisms

Q161 BCG vaccine is given to protect

from which of the following

बीसीजीकाटटकानननननलिखितमसहॳककसकहॳ बचावकहॳ लिएटदयाजातहहॴ

27-Jan-2017

Options

1) Jaundice

पीलिया 2) Anaemia

रकतमपता 3) Tuberculosis

कषयरह८ग

4) Polio

पह८लियह८ Correct Answer Tuberculosis

Q162 Parallel venation is found in

समानतरवहॳनहॳिन mdashmdashmdash- मपायाजाताहहॴ 27-Jan-2017

Options

1) plants which are monocots

पह९धहॳजह८एकबीजपतरीहह८तहॳहहॴ 2) plants which have a dicot stem

वहॳपह९धहॳनजनकातनादपवदलियहह८ताहहॴ 3) plants with leaves similar to Tulsi

वहॳपह९धहॳनजनकीपनततयतिसीकीपनततयोकहॳ समानहह८तहॳहहॴ 4)plants with tap roots

टहॳप टवािहॳपह९धहॳ Correct Answer plants which are

monocots

Q163 The hardest part of the body is

िरीरकासबसहॳकठह८रभाग mdashndash हहॴ 27-Jan-2017

Options

1) Bones

हडडिय

2) Tooth Enamel

दातकहॳ इनहॳमि

3) Skull

िह८पड़ी 4) Spinal Cord

महॳ रजज

Correct Answer Tooth Enamel

Q164 Which type of pathogen causes

the waterborne disease E coli Infection

ककसपरकारकारह८गजननकजिजननतरह८गईकह८िाईसिमणकाकारणबनताहहॴ 27-Jan-2017

Options

1) Protozoan

परह८टह८जआ

2) Parasitic

परजीवी 3) Bacterial

बहॴकटीररयि

4)Viral

वायरि

Correct Answer Bacterial

Q165 The amount of blood filtered

together by both the kidneys in a 70 kg

adult male human in a minute is

70 की गरा वािहॳएकवयसकप षमएकलमनटमदह८नोगदकहॳदवाराएकसािचाबनीगयीरकतकीमातरहह८तीहहॴ 29-Jan-2017

Options

1) 1100 ml

1100 लमलि

2) 100 ml

F A C E B O O K

P A G E h t t p w w w f a c e b o o k c o m s s c m e n t o r s o f f i c i a l P a g e | 39

FOR MORE UPDATES AND MORE MATERIAL DO LIKE OUR FACEBOOK PAGE httpwwwfacebookcomsscmentorsofficial

100 लमलि

3) 1500 ml

1500 लमलि

4) 500 ml

500 लमलि

Correct Answer 1100 ml

Q166 Which feature of a plant helps to

distinguish a monocot from a dicot

पह९धहॳकीवहकह९नसीपविहॳषताहहॴजह८एकदपवदलियहॳऔरएकएकदिीयपह९धहॳसहॳभहॳदकरनहॳममददकरतीहहॴ 29-Jan-2017

Options

1) Pollination

परागम

2) Venation

वहॳनहॳिन

3) Vernation

वनिन

4) Aestivation

एसटीवहॳिहॳन

Correct Answer venation

Q167 The Mutation Theory was

proposed by

उतवररवतयनकालसदात mdashmdashndash

कहॳ दवरापरसतापवतककयाजाताहहॴ 29-Jan-2017

Options

1) Charles Lyell

चामसयलियहॳि

2) William Smith

पवलियमनसमि

3) Hugo De Vries

हयगह८िीराईस

4)Harrison Schmitt

हहॳरीसननसमट

Correct Answer Hugo De Vries

Q168 Which type of pathogen causes

the waterborne disease HepatitisA

ककसपरकारकहॳ रह८गजनकजिजननतरह८गहहॳपहॳटाइटटस-A काकारणबनताहहॴ

29-Jan-2017

Options

1) Parasitic

परजीवी 2) Viral

वायरि

3) Protozoan

परह८टह८जआ

4) Bacterial

बहॴकटीररयि

Correct Answer Viral

Q169 In a Punnett Square with the

cross AaBb x Aabb how many AaBb

genotypes would be created

पनहॳटसकवायरमिह८स AaBb x Aabb

कहॳ सािककतनहॳ AaBb जीनह८टाइपबनगहॳ 29-Jan-

2017

Options

1) 4

2) 1

3) 7

4) 6

Correct Answer 4

Q170 Arboreal Ateles is the scientific

name of

अिह८ररयिएटटलिस mdashmdashmdash कावहॴजञाननकनामहहॴ 29-Jan-2017

Options

1) Squirrel

चगिहरी 2) Sparrow

गह८रहॴया 3) Lizard

नछपकिी 4) Spider monkey

F A C E B O O K

P A G E h t t p w w w f a c e b o o k c o m s s c m e n t o r s o f f i c i a l P a g e | 40

FOR MORE UPDATES AND MORE MATERIAL DO LIKE OUR FACEBOOK PAGE httpwwwfacebookcomsscmentorsofficial

मकड़ीबदर

Correct Answer Spider monkey

Q171 Which type of pathogen causes

the waterborne disease Salmonellosis

ककसपरकारकारह८गाणजिजननतबीमारीसािमह८नहॳिह८लसज़काकारकहहॴ

29-Jan-2017

Options

1) Algal

िहॳवालियहॳ 2) Parasitic

परजीवी 3) Bacterial

बहॴकटीररयि

4)Viral

वायरि

Correct Answer Bacterial

An infection with salmonella bacteria

commonly caused by contaminated food

or water

Symptoms include diarrhoea fever

chills and abdominal pain

Q172 is a condition in which there is a

deficiency of red cells or of haemoglobin

in the blood

mdashmdash-

एकनसिनतहहॴनजसमहॳरकतमिािकह८लिकाओकीयाहीमह८गिह८बबनकीकमीहह८तीहहॴ 29-Jan-2017

Options

1) Albinism

एनमबननजम

2) Propyria

परह८पीररया 3) Anaemia

एनीलमया 4)Keloid disorder

कहॳ िह८इिडिसओिर

Correct Answer Anaemia

Q173 Ananas comosus is the scientific

name of

Options

अनानासकह८मह८सस mdashmdashmdashndash

कावहॴजञाननकनामहहॴ 29-Jan-2017

1) Custard Apple

सीताफि

2) Pineapple

पाइनएपपि

3) Bamboo

बास

4)Pomegranate

अनार

Correct Answer Pineapple

Q174 Which organ produces insulin

कह९नसाअगइनसलिनपहॴदाकरताहहॴ 29-Jan-

2017

Options

1) Liver

यकत

2) Thyroid gland

िायराइिगरिी 3) Spleen

पिीहा 4)Pancreas

अगरयिय

Correct Answer Pancreas

Q175 Which of the following disease is

not caused by water pollution

नननननलिखितमसहॳकह९नसारह८गपानीकहॳ परदषणकहॳकारणनहीहह८ता

29-Jan-2017

Options

1) Cholera

हहॴजा 2) Typhoid

F A C E B O O K

P A G E h t t p w w w f a c e b o o k c o m s s c m e n t o r s o f f i c i a l P a g e | 41

FOR MORE UPDATES AND MORE MATERIAL DO LIKE OUR FACEBOOK PAGE httpwwwfacebookcomsscmentorsofficial

टाइफाइि

3) Asthma

दमा 4)Diarrhoea

दसत

Correct Answer Asthma

Q176 Ocimum tenuiflorum is the

scientific name of

ओलिलममटहॳयईफिह८रमइसकावहॴजञाननकनाम mdash

ndash हहॴ 30-Jan-2017

Options

1) Neem

नीम

2) Mango

आम

3) Babul

बबि

4)Tulsi

तिसी Correct Answer Tulsi

Q177 Which gland secretes bile a

digestive fluid

कह९नसीगरिीपपतत एकपाचनतरिपरदािय सरापवतकरतीहहॴ 30-Jan-2017

Options

1) Pancreas

अगनयािय

2) Liver

यकत

3) Thyroid

िायराइि

4) Testes

टहॳनसटस

Correct Answer liver

Q178 In which of the following the

dominant phase is Gametophyte

नननननलिखितमसहॳककसकहॳ परमिचरणयगमकह८दपवधद (Gametophyte)हहॴ 30-Jan-2017

Options

1) Bryophyta

िायह८फाइटा 2) Pteridophyta

टहॳररिह८फाइटा 3) Gymnosperms

नजननह८सपमय 4) Angiosperms

एननजयह८सपमय Correct Answer Bryophyta

Q179 Anaerobic respiration refers to

which of the following

नननननलिखितमसहॳककसहॳअवायवीयशवसनकहाजाताहहॴ

30-Jan-2017

Options

1) Respiration without Oxygen

ऑकसीजनकहॳ बबनाशवसन

2) Respiration with Oxygen

ऑकसीजनकहॳ सािशवसन

3) Respiration without CO2

काबयनिायऑकसाइिकहॳ बबनाशवसन

4) Respiration with CO2

काबयनिायऑकसाइिकहॳ सािशविन

Correct Answer Respiration without

Oxygen

Q180 Which type of pathogen causes

the waterborne disease Cholera

ककसपरकारकारह८गजनकजिजननतरह८गहहॴजाकाकारणबनताहहॴ

30-Jan-2017

Options

1) Algal

िहॴवालियहॳ

F A C E B O O K

P A G E h t t p w w w f a c e b o o k c o m s s c m e n t o r s o f f i c i a l P a g e | 42

FOR MORE UPDATES AND MORE MATERIAL DO LIKE OUR FACEBOOK PAGE httpwwwfacebookcomsscmentorsofficial

2) Bacterial

बहॴकटीररयि

3) Protozoan

परह८टह८जआ

4) Viral

वायरि

Correct Answer Bacterial

Q181 To which class does

Oxyreductases transferases hydrolases

belong

ओकसीररिकटहॳसटरासफरहॳजहॳस

हाइडरह८िहॳसहॳसककसवगयमआतहॳहहॴ 30-Jan-2017

Options

1) Hormones

हारमोस

2) Enzymes

एजाइनस

3) Proteins

परह८टीनस

4) Vitamins

पवटालमनस

Correct Answer Enzymes

Q182 Which of the following is not true

about Gymnosperms

ननननमसहॳकह९नसीबातअनावतबीजीकहॳ बारहॳमसचनहीहहॴ 30-Jan-2017

Options

1) Dominant phase is saprophytes

परमिचरणसहॳपरह८फाइटसहह८ताहहॴ 2) Vascular bundles are absent

सवहनीबििअनपनसितहह८ताहहॴ 3) spores are heterospores

बीजाणहहॳटहॳरह८सपह८रसहह८तहॳहहॴ 4) Flowers are absent

फिअनपनसितहह८तहॳहहॴ

Correct Answer Vascular bundles are

absent

Q183 The name of first mammal clone sheep is

भहॳड़कीपरिमसतनपायीपरनत प (किह८न)

कानामहहॴ 30-Jan-2017

Options

1) Noori

नरी 2) Dolly

िॉिी 3) Louise

िसी 4)Durga

दगाय Correct Answer Dolly

Q184 Which type of pathogen causes

the water-borne disease Typhoid fever

ककसपरकारकारह८गजनकजिजननतरह८गटाइफाइिबिारकाकारणबनताहहॴ 30-Jan-2017

Options

1) Algal

िहॴवािीय

2) Parasitic

परजीवी 3) Protozoan

परह८टह८जनअन

4)Bacterial

बहॴकटीररयि

Correct Answer Bacterial

Q185 In which part of the cell are

proteins made

कह८लिकाकहॳ ककसटहससहॳमपरह८टीनबनायाजाताहहॴ

31-Jan-2017

Options

1) Reticulum

रहॳटटकिम

F A C E B O O K

P A G E h t t p w w w f a c e b o o k c o m s s c m e n t o r s o f f i c i a l P a g e | 43

FOR MORE UPDATES AND MORE MATERIAL DO LIKE OUR FACEBOOK PAGE httpwwwfacebookcomsscmentorsofficial

2) Golgi apparatus

गह८मजीएपहॳरहॳटस

3) Ribosomes

ररबह८सह८नस

4) Lysosome

िायसह८सह८नस

Correct Answer ribosomes

Proteins are produced by stringing

amino acids together in the order

specified by messenger RNA strands

that were transcribed from DNA in the

cell nucleus The process of synthesizing

a protein is called translation and it

occurs on ribosomes in the cytoplasm of

a cell

Q186 Polio is a disease caused by which

of the following

नननननलिखितमसहॳपह८लियह८कीबबमारह८हह८नहॳकाकारणकयाहहॴ

31-Jan-2017

Options

1) Bacteria

बहॴकटीररयि

2) Mosquito

मचछर

3) Virus

वायरस

4) Cockroach

नतिच हॳ Correct Answer Virus

Polio or poliomyelitis is a crippling and

potentially deadly infectious disease It

is caused by the poliovirus

Q187 ndash Hay fever is a sign of which of

the following

हहॳकफवरनननननलिखितमसहॳककसकाएकसकहॳ तहहॴ

31-Jan-2017

Options

1) Old Age

वदावसिा 2) Malnutrition

कपह८सण

3) Allergy

एिनजय 4) Over Work

अतयचधककाययकरना Correct Answer Allergy

Q188 How many chromosomes does a

human cell contain

एकमानवकह८लिकामककतनहॳगणसतरहह८तहॳहहॴ

29-Jan-2017

Options

1) 6

2) 26

3) 46

4) 66

Correct Answer 46

In humans each cell normally contains

23 pairs of chromosomes for a total of

46 Twenty-two of these pairs called

autosomes look the same in both males

and females The 23rd pair the sex

chromosomes differ between males and

females

Q189 Which of the following is not true

about Bryophyta

ननननमसहॳकह९नसीबातिायह८फाइटकहॳ बारहॳमसचनहीहहॴ 31-Jan-2017

Options

1) Dominant phase is gametophytes

परमिचरणगहॳलमतह८फाइटसहह८ताहहॴ 2) Main plant body is haploid

पह९धहॳकामखयिरीरअगखणतहह८ताहहॴ 3) Spores are homospores

बीजाणहह८मह८सफह८रसहह८तहॳहहॴ 4) Flowers are present

फिमह८जदहह८तहॳहहॴ Correct Answer Flowers are present

F A C E B O O K

P A G E h t t p w w w f a c e b o o k c o m s s c m e n t o r s o f f i c i a l P a g e | 44

FOR MORE UPDATES AND MORE MATERIAL DO LIKE OUR FACEBOOK PAGE httpwwwfacebookcomsscmentorsofficial

Q190 Which aquatic animal has

trailing tentacles

ककसजिीयजानवरकहॳ पीछहॳचिनहॳवािहॳटहॳटकिसहह८तहॳहहॴ

31-Jan-2017

Options

1) Sea horse

समदरीघह८िा 2) Corals

मगा 3) Jelly fish

जहॳिीमछिी 4) Star fish

तारामछिी Correct Answer Jelly fish

Jellyfish with its umbrella-shaped bell

and trailing tentacles

Q191 Which type of pathogen causes

the water-borne disease Poliomyelitis

(Polio)

ककसपरकारकारह८गजनकजिजननतरह८गपह८लियह८मायहॳटटस (पह८लियह८) काकारणहहॴ 31-Jan-

2017

Options

1) Parasitic

परजीवी 2) Algal

िहॴवालिय

3) Viral

वायरि

4) Bacterial

बहॴकटीररयि

Correct Answer Viral

Q192 The outer white part of the eye

that protects the inner structures is

आािकाबाहरीसफहॳ दटहससाजह८आतररकसरचनाओकीरकषाकरताहहॴ वह mdashmdashmdash हहॴ 31-Jan-

2017

Options

1) Iris

आयररस

2) Sclera

सकिहॳरा 3) Retina

रहॳटटना 4) Cornea

कह८ननयया Correct Answer Sclera

Q193 Proteins are made up of

परह८टीनकाननमायण mdashndash सहॳहह८ताहहॴ 31-Jan-2017

Options

1) Amino acids

एलमनह८अनि

2) Fatty acids

वसायकतअनि

3) Glucose

गिकह८ज

4)Nucleotides

नयनकियह८टाईिस

Correct Answer Amino acids

Q194 Moringa Oleifera is the scientific

name of

मह८ररगओलिफहॳ रा mdashmdashndash कावहॴजञाननकनामहहॴ 31-Jan-2017

Options

1) Banyan

बरगद

2) Gulmohar

गिमह८हर

3) Amla

आमिा

F A C E B O O K

P A G E h t t p w w w f a c e b o o k c o m s s c m e n t o r s o f f i c i a l P a g e | 45

FOR MORE UPDATES AND MORE MATERIAL DO LIKE OUR FACEBOOK PAGE httpwwwfacebookcomsscmentorsofficial

4) Drumstick

डरमनसटक

Correct Answer Drumstick

Q195 Kidney stones are composed of

गदकीपिरी mdashndash सहॳबनीहह८तीहहॴ 1-Feb-2017

Options

1) Calcium Oxalate

कहॴ नमसयमओकजहॳिहॳट

2) Sodium Chloride

सह८डियमकिह८राइि

3) Magnesium Nitrate

महॳनगनलियमनाइतटरहॳट

4) Calcium Bicarbonate

कहॴ नमियमबायकबोनहॳट

Correct Answer Calcium Oxalate

Q196 ndash Which of the following is not

true about Angiosperms

ननननमसहॳकह९नसीबातआवतबीजीकहॳ बारहॳमसचनहीहहॴ 1-Feb-2017

Options

1) Dominant phase is gametophytes

परमिचरणगहॳलमतह८फाइटहह८ताहहॴ 2) Vascular bundles are present

सवहनीबििमह९जदहह८ताहहॴ 3) Spores are heterospores

बीजाणहहॳटहॳरह८सपह८रसहह८तहॳहहॴ 4) Seeds are covered

बीजढकहॳ हह८तहॳहहॴ Correct Answer Dominant phase is

gametophytes

Q197 All of the following are excretory

(waste) products of animals except

नननननलिखितमसहॳककसएककह८छह८ड़करअनयसभीपराखणयोदवाराउतसनजयतपदाियहहॴ 1-Feb-

2017

Options

1) Uric Acid

यररकएलसि

2) Ammonia

अमह८ननया 3) Carbohydrates

काबोहाइडरहॳट

4) Urea

यररया Correct Answer Carbohydrates

In animals the main excretory products

are carbon dioxide ammonia (in

ammoniotelics) urea (in ureotelics) uric

acid (in uricotelics) guanine (in

Arachnida) and creatine

Q198 RNA is a polymeric molecule

What does RNA stand for

आरएनइएएकबहिकआणहहॴ इसकाकापवय पकयाहहॴ 1-Feb-2017

Options

1) Rado Nuclear Acid

रािह८नयनकियरएलसि

2) Ribo Nucleic Acid

राइबह८नयनकिकएलसि

3) Rhino Nuclear Acid

हाइनह८नयनकियरएलसि

4) Resto Nucleus Acid

रहॳसटह८नयकिीयसएलसि

Correct Answer Ribo Nucleic Acid

Q199 Which organ does detoxification

and produces chemicals needed for

digestion

कह९नसाअगपवषहरणकरताहहॴऔरपाचनकहॳ लिएआवशयकरसायनोकह८पहॴदाकरताहहॴ 1-Feb-

2017

Options

1) Salivary glands

िारगरचिया 2) Pancreas

अगनयािय

F A C E B O O K

P A G E h t t p w w w f a c e b o o k c o m s s c m e n t o r s o f f i c i a l P a g e | 46

FOR MORE UPDATES AND MORE MATERIAL DO LIKE OUR FACEBOOK PAGE httpwwwfacebookcomsscmentorsofficial

3) Thyroid gland

िायराइिगरिी 4) Liver

यकत

Correct Answer Liver

Q200 Psidium guajava is the scientific

name of

लसडियमगआजावा mdashmdash कावहॴजञाननकनामहहॴ 1-

Feb-2017

Options

1) Guava

अम द

2) Mango

आम

3) Bamboo

बास

4) Jack fruit

कटहि

Correct Answer Guava

Q201 Which drug is used as a Blood

Thinner

चधरकह८पतिाकरनहॳकहॳ पमककसदवाकापरयह८गककयाजाताहहॴ

1-Feb-2017

Options

1) Warfarin

वाफर न

2) Tramadol

टरहॳमािह८ि

3) Azithromycin

एनजरह८मायलसन

4) Hydralazine

हाइडरह८िहॳनजन

Correct Answer Warfarin

Q202 Which of the following disease is

caused due to the deficiency of protein

परह८टीनकीकमीकहॳ कारणनननननलिखितमसहॳकह९नसारह८गहह८ताहहॴ 1-Feb-2017

Options

1) Arthritis

गटठया 2) Kwashiorkor

कािीओकय र

3) Goitre

गाइटर

4) Night Blindness

रतह९चध

Correct Answer Kwashiorkor

Q203 A is species of plant that has

adapted to survive in an environment

with little liquid water

mdashmdashndashपह९धहॳकीएकऐसहॳऐसहॳपरजानतहहॴ नजसनहॳकमपानीवािहॳवातावरणमजीपवतरहनहॳकहॳलिएअनकिनहहॴ 1-Feb-2017

Options

1) Xerophyte

म दपवद

2) Hydrophyte

जिीयपादप

3) Mesophyte

समह८दपवद

4) Thallophyte

िहॴिह८फाइटा Correct Answer xerophyte

xerophyte is a species of plant that has

adapted to survive in an environment

with little liquid water such as a desert

or an ice- or snow-covered region in the

Alps or the Arctic

Mesophytes are terrestrial plants which

are adapted to neither a particularly

dry nor particularly wet environment

An example of a mesophytic habitat

would be a rural temperate meadow

F A C E B O O K

P A G E h t t p w w w f a c e b o o k c o m s s c m e n t o r s o f f i c i a l P a g e | 47

FOR MORE UPDATES AND MORE MATERIAL DO LIKE OUR FACEBOOK PAGE httpwwwfacebookcomsscmentorsofficial

which might contain goldenrod clover

oxeye daisy and Rosa multiflora

thallophyte any of a group of plants or

plantlike organisms (such as algae and

fungi) that lack differentiated stems

leaves and roots and that were formerly

classified as a primary division

(Thallophyta) of the plant kingdom

Q204 How many types of teeth are

there in humans

मनषयोमककतनहॳपरकारकहॳ दातहह८तहॳहहॴ

1-Feb-2017

Options

1) 4

2) 5

3) 2

4) 3

Correct Answer 4

teeth -Humans have four types of

teethincisors canines premolars and

molars each with a specific function

The incisors cut the food the canines

tear the food and the molars and

premolars crush the food

Q205 Carica papaya is the scientific name of

कहॴ ररकापपाया mdashmdashndash कावहॴजञाननकनामहहॴ 2-

Feb-2017

Options

1) Peepal

पीपि

2) Papaya

पपीता 3) Tamarind

इमिी 4) Drumstick

ढह८िकाछड़ी Correct Answer Papaya

Q206 Muscles get tired when there is

shortfall of

जब mdashndash कीकमीहह८तीहहॴतबपहॳिीयिकजातीहहॴ 2-Feb-2017

Options

1) Lactic acid

िहॴनकटकएलसि

2) Na+ ions

Na+ आयन

3) ATP

एटीपी 4) Sulphates

समफहॳ टस

Correct Answer ATP

ATP is the energy source muscle fibers

use to make muscles contract

muscle tissuersquos main source of energy

called adenosine triphosphate or ATP

As your muscles use up this energy

source they become tired and fatigued

Oxygen is the key ingredient that helps

create new ATP to replenish the burned

up ATP in your muscles

Q207 Artocarpus integra is the

scientific name of आटह८कापयसइटीगरा mdashmdashmdash कावहॴजञाननकनामहहॴ 2-Feb-2017

Options

1) Guava

अम द

2) Pineapple

अनानास

3) Silver Oak

लसमवरओक

4) Jack fruit

कटहि

Correct Answer Jack fruit

Q208 Which organ stores fat soluble

vitamins

कह९नसाअगवसामघिनिीिपवटालमनह८काभिाराकरताहहॴ

2-Feb-2017

F A C E B O O K

P A G E h t t p w w w f a c e b o o k c o m s s c m e n t o r s o f f i c i a l P a g e | 48

FOR MORE UPDATES AND MORE MATERIAL DO LIKE OUR FACEBOOK PAGE httpwwwfacebookcomsscmentorsofficial

Options

1) Blood

रकत

2) Skin

तवचा 3) Liver

यकत

4) Pancreas

अगनयािय

Correct Answer Liver

Q209 Which disease is caused due to

deficiency of Iodine

आयह८िीनकहॳ कारणकह९नसारह८गहह८ताहहॴ 2-Feb-2017

Options

1) Rickets

ररकहॳ टस

2) Scurvy

सकवी 3) Goitre

गणमािा 4) Growth retardation

पवकासका कना Correct Answer Goitre

rickets A softening and weakening of

bones in children usually due to

inadequate vitamin D

Q210 Grevillea Robusta is the scientific name of

गरहॳपवलियारह८बसटा mdashmdashmdash- कापवजञाननकनामहहॴ 2-Feb-2017

Options

1) Peepal

पीपि

2) Teak

सागह९न

3) Silver Oak

लसमवरओक

4) Jack fruit

कटहि

Correct Answer Silver Oak

Q211 When a Cuttlefish is described as a Molluscs it is at which level of

classification

जबएककटिकफिकह८एकमह८िसकाकहॳ पमवखणयतककयाजाताहहॴतबयहॳवगीकरणकहॳ ककससतरपहॳनसितहहॴ 2-Feb-2017

Options

1) Class

वगय 2) Order

िम

3) Family

पररवार

4) Phylum

सघ

Correct Answer Phylum

Q212 Bambusa dendrocalmus is the

scientific name of बानबसािहॳडराकामस mdashmdashmdash कावहॴजञाननकनामहहॴ 3-Feb-2017

Options

1) Banyan

बरगद

2) Papaya

पपीता 3) Bamboo

बास

4) Pomegranate

अनार

Correct Answer Bamboo

Q213 Acinonyx Jubatus is the scientific name of

एलसनह८ननकसजयबहॳटस mdashmdashmdash

कावहॴजञाननकनामहहॴ 3-Feb-2017

F A C E B O O K

P A G E h t t p w w w f a c e b o o k c o m s s c m e n t o r s o f f i c i a l P a g e | 49

FOR MORE UPDATES AND MORE MATERIAL DO LIKE OUR FACEBOOK PAGE httpwwwfacebookcomsscmentorsofficial

Options

1) Bear

भाि 2) Horse

घह८िा 3) Cheetah

चीता 4) Zebra

जहॳिा Correct Answer Cheetah

Q214 The pale yellow colour of urine is

due to the presence of which pigment

मतरकाफीकापीिारगरगदरयकहॳ उपनसिनतकहॳ कारणहह८ताहहॴ

3-Feb-2017

Options

1) Urochrome

यरह८िह८म

2) Urophyll

यरह८कफि

3) Chlorophyll

किह८रह८कफि

4) Chloroplast

किह८रह८पिासट

Correct Answer Urochrome

Q215 Which of the following constitute

to form a gene

नननननलिखितमसहॳकह९नसीचीज़एकजीनकागठनकरतीहहॴ

3-Feb-2017

Options

1) Polynucleotides

पह८िीनयनकियह८टाईडस

2) Hydrocarbons

हाइडरह८काबोस

3) Lipoproteins

िाईपह८परह८टीनस

4) Lipids

लिपपडस

Correct Answer Polynucleotides

Polynucleotide molecule is a biopolymer

composed of 13 or more nucleotide

monomers covalently bonded in a chain

DNA (deoxyribonucleic acid) and RNA

(ribonucleic acid) are examples of

polynucleotides with distinct biological

function

Q216 Vertebrates belongs to the

phylum

रीढ़कीहडिीवािहॳपराणी mdashmdashmdash

परजानतकहॳ अतगायतआतहॳहहॴ 3-Feb-2017

Options

1) Arthropoda

आरह८पह८ड़ा 2) Annelida

एननलििा 3) Cnidaria

ननिहॳररया 4) Chordata

कह८िटा Correct Answer Chordata

Q217 Punica granatum is the scientific name of

पननकगरहॳनहॳटस mdashmdashmdash कावहॴजञाननकनामहहॴ 3-Feb-2017

Options

1) Custard Apple

सीताफि

2) Gulmohar

गिमह८हर

3) Silver Oak

लसमवरओक

4) Pomegranate

अनार

Correct Answer Pomegranate

F A C E B O O K

P A G E h t t p w w w f a c e b o o k c o m s s c m e n t o r s o f f i c i a l P a g e | 50

FOR MORE UPDATES AND MORE MATERIAL DO LIKE OUR FACEBOOK PAGE httpwwwfacebookcomsscmentorsofficial

Q218 Between a tiger and an monkey

which of the following is different

एकबाघऔरबदरकहॳ बीचनननननलिखितमसहॳकह९नसीबातअिगहहॴ 3-Feb-2017

Options

1) Kingdom

राजय

2) Phylum

जानत

3) Order

िम

4) Class

वगय Correct Answer order

Q219 The artificial heart was invented by

कबतरमहदयका mdashmdashmdash

दवाराअपवषकारककयागयािा 3-Feb-2017

Options

1) Muhammad Yunus

महनमदयनस

2) Linus Yale Jr

िाइनसयहॳिजय

3) Gazi Yasargil

गाजीयासचगयि

4) Paul Winchell

पह९िपवमकि Correct Answer Paul Winchell

Q220 Tamarindus indica is the

scientific name of

टहॳमररनडसइडिका mdashmdash कावहॴजञाननकनामहहॴ 7-

Feb-2017

Options

1) Neem

नीम

2) Pineapple

अनानास

3) Tamarind

इमिी 4)Chiku

चीक

Correct Answer Tamarind

Q221 In eukaryotic cells synthesis of

RNA takes place in the

यकहॳ योटटककह८लिकाओमआरएनएकासशिहॳषण

mdashndash महह८ताहहॴ 7-Feb-2017

Options

1) Mitochondria

माईटह८कोडडरया 2) Centrioles

सटरीयह८मस

3) Ribosomes

ररबह८सह८नस

4) Nucleus

नयनकियस

Correct Answer nucleus

eukaryotic cell -Transcription is the

process of synthesizing ribonucleic acid

(RNA)Synthesis takes place within the

nucleus of eukaryotic cells or in the

cytoplasm of prokaryotes and converts

the genetic code from a gene in

deoxyribonucleic acid ( DNA ) to a

strand of RNA that then directs

proteinsynthesis

Q222 _________is caused by parasites

of the Plasmodium genus

पिाजमह८डियमजातीकहॳ परजीवी mdash- कहॳ कारणहहॴ 7-Feb-2017

Options

1) Dysentery

पहॳचचि

2) Malaria

मिहॳररया 3) Chickenpox

F A C E B O O K

P A G E h t t p w w w f a c e b o o k c o m s s c m e n t o r s o f f i c i a l P a g e | 51

FOR MORE UPDATES AND MORE MATERIAL DO LIKE OUR FACEBOOK PAGE httpwwwfacebookcomsscmentorsofficial

चहॳचक

4) Herpes

हहॳपपयस

Correct Answer Malaria

Q223 Carotene in fruits and vegetables

gives it which color

फिह८औरसनलजयोमनसितकहॳ रह८टीनउनहकह९नसारगपरदानकरताहहॴ 7-Feb-2017

Options

1) Green

हरा 2) Pink

गिाबी 3) Orange

नारगी 4) Blue

नीिा Correct Answer Orange

Q224 Equus Caballus is the scientific

name of

एकवसकहॴ बहॳिस mdashmdashndash कापवजञाननकनामहहॴ 7-Feb-2017

Options

1) Horse

घह८िा 2) Zebra

ज़हॳिा 3) Donkey

गधा 4) Buffalo

भस

Correct Answer Horse

Q225 Elapidae Naja is the scientific name of

एिीपीिीनाजा mdashmdash- कावहॴजञाननकनामहहॴ 8-Feb-2017

Options

1) Cobra

कह८बरा 2) Elephant

हािी 3) Eagle

ग ि

4) Owl

उमि Correct Answer Cobra

Q226 Which disease is caused due to

deficiency of Iron

िह८हकीकमीकहॳ कारणकह९नसारह८गहह८ताहहॴ 8-Feb-

2017

Options

1) Beriberi

बहॳरीबहॳरी 2) Tetany

टहॳटनी 3) Kwashiorkor

कवािीऔरकर

4) Anaemia

रकतामपता Correct Answer Anaemia

Beriberi is a disease caused by a vitamin

B-1 deficiency also known as thiamine

deficiency

Tetany can be the result of an

electrolyte imbalance Most often itrsquos a

dramatically low calcium level also

known as hypocalcemia Tetany can also

be caused by magnesium deficiency or

too little potassium Having too much

acid (acidosis) or too much alkali

(alkalosis) in the body can also result in

tetany

Kwashiorkor also known as

ldquoedematous malnutrition It is a form of

malnutrition caused by a lack of protein

in the diet

Anaemia means that you have fewer red

blood cells than normal or you have less

F A C E B O O K

P A G E h t t p w w w f a c e b o o k c o m s s c m e n t o r s o f f i c i a l P a g e | 52

FOR MORE UPDATES AND MORE MATERIAL DO LIKE OUR FACEBOOK PAGE httpwwwfacebookcomsscmentorsofficial

haemoglobin than normal in each red

blood cell

Q227 is a leaf where the leaflets are

arranged along the middle vein

mdashndashएकपततीहहॴजहापतरकह८कीरचनाक ररयालिराकहॳ आसपासहह८तीहहॴ 8-Feb-2017

Options

1) Pinnately compound leaf

पपनहॳटिीसयकतपतती 2) Palmately compound leaf

पामहॳटिीसयकतपतती 3) Compound leaf

सयकतपतती 4) Simple leaf

साधारणपतती Correct Answer Pinnately compound

leaf

Q228 Haustoria or sucking roots are

found in which of the following

हह८सटह८ररयायाचसनहॳवािीजड़हॳनननननलिखितमसहॳककसमपाईजातीहहॴ 8-Feb-2017

Options

1) Wheat

गहॳह

2) Mango

आम

3) Chestnut

चहॳसटनट

4) Cuscuta

कसकयटा Correct Answer Cuscuta

Haustorial roots -The roots of parasitic

plants which penetrate into the host

tissues to absorb nourishment are

called haustorial roots hellip Also known as suckingor parasitic roots

Q229 Equs Asinus is the scientific name

of

एकवसएलसनस mdashmdashndash कावहॴजञाननकनामहहॴ 8-

Feb-2017

Options

1) Donkey

गधा 2) Cow

गाय

3) Deer

टहरन

4) Kangaroo

कगा

Correct Answer Donkey

Q230 Ficus benghalensis is the scientific name of

फाईकसबहॳनगहॳिहॳलसस mdashndash कापवजञाननकनामहहॴ 8-Feb-2017

Options

1) Banyan

बरगद

2) Pineapple

अनानास

3) Babul

बबि

4) Tulsi

तिसी Correct Answer Banyan

Q231 Equus burchellii is the scientific name of

एकवसबचिी mdashmdash- कापवजञाननकनामहहॴ 8-Feb-2017

Options

1) Horse

घह८िा 2) Zebra

जहॳिा 3) Buffalo

F A C E B O O K

P A G E h t t p w w w f a c e b o o k c o m s s c m e n t o r s o f f i c i a l P a g e | 53

FOR MORE UPDATES AND MORE MATERIAL DO LIKE OUR FACEBOOK PAGE httpwwwfacebookcomsscmentorsofficial

भस

4) Ass

गधा Correct Answer Zebra

Page 40: COMPILATION OF ALL 72 SETS OF BIOLOGY SSC CHSL-2016 · OF BIOLOGY SSC CHSL-2016 PREPARED BY : SSC MENTORS BIOLOGY SPECIAL . F A C E B O O K P A G E : h t t p : / / w w w . f a c e

F A C E B O O K

P A G E h t t p w w w f a c e b o o k c o m s s c m e n t o r s o f f i c i a l P a g e | 39

FOR MORE UPDATES AND MORE MATERIAL DO LIKE OUR FACEBOOK PAGE httpwwwfacebookcomsscmentorsofficial

100 लमलि

3) 1500 ml

1500 लमलि

4) 500 ml

500 लमलि

Correct Answer 1100 ml

Q166 Which feature of a plant helps to

distinguish a monocot from a dicot

पह९धहॳकीवहकह९नसीपविहॳषताहहॴजह८एकदपवदलियहॳऔरएकएकदिीयपह९धहॳसहॳभहॳदकरनहॳममददकरतीहहॴ 29-Jan-2017

Options

1) Pollination

परागम

2) Venation

वहॳनहॳिन

3) Vernation

वनिन

4) Aestivation

एसटीवहॳिहॳन

Correct Answer venation

Q167 The Mutation Theory was

proposed by

उतवररवतयनकालसदात mdashmdashndash

कहॳ दवरापरसतापवतककयाजाताहहॴ 29-Jan-2017

Options

1) Charles Lyell

चामसयलियहॳि

2) William Smith

पवलियमनसमि

3) Hugo De Vries

हयगह८िीराईस

4)Harrison Schmitt

हहॳरीसननसमट

Correct Answer Hugo De Vries

Q168 Which type of pathogen causes

the waterborne disease HepatitisA

ककसपरकारकहॳ रह८गजनकजिजननतरह८गहहॳपहॳटाइटटस-A काकारणबनताहहॴ

29-Jan-2017

Options

1) Parasitic

परजीवी 2) Viral

वायरि

3) Protozoan

परह८टह८जआ

4) Bacterial

बहॴकटीररयि

Correct Answer Viral

Q169 In a Punnett Square with the

cross AaBb x Aabb how many AaBb

genotypes would be created

पनहॳटसकवायरमिह८स AaBb x Aabb

कहॳ सािककतनहॳ AaBb जीनह८टाइपबनगहॳ 29-Jan-

2017

Options

1) 4

2) 1

3) 7

4) 6

Correct Answer 4

Q170 Arboreal Ateles is the scientific

name of

अिह८ररयिएटटलिस mdashmdashmdash कावहॴजञाननकनामहहॴ 29-Jan-2017

Options

1) Squirrel

चगिहरी 2) Sparrow

गह८रहॴया 3) Lizard

नछपकिी 4) Spider monkey

F A C E B O O K

P A G E h t t p w w w f a c e b o o k c o m s s c m e n t o r s o f f i c i a l P a g e | 40

FOR MORE UPDATES AND MORE MATERIAL DO LIKE OUR FACEBOOK PAGE httpwwwfacebookcomsscmentorsofficial

मकड़ीबदर

Correct Answer Spider monkey

Q171 Which type of pathogen causes

the waterborne disease Salmonellosis

ककसपरकारकारह८गाणजिजननतबीमारीसािमह८नहॳिह८लसज़काकारकहहॴ

29-Jan-2017

Options

1) Algal

िहॳवालियहॳ 2) Parasitic

परजीवी 3) Bacterial

बहॴकटीररयि

4)Viral

वायरि

Correct Answer Bacterial

An infection with salmonella bacteria

commonly caused by contaminated food

or water

Symptoms include diarrhoea fever

chills and abdominal pain

Q172 is a condition in which there is a

deficiency of red cells or of haemoglobin

in the blood

mdashmdash-

एकनसिनतहहॴनजसमहॳरकतमिािकह८लिकाओकीयाहीमह८गिह८बबनकीकमीहह८तीहहॴ 29-Jan-2017

Options

1) Albinism

एनमबननजम

2) Propyria

परह८पीररया 3) Anaemia

एनीलमया 4)Keloid disorder

कहॳ िह८इिडिसओिर

Correct Answer Anaemia

Q173 Ananas comosus is the scientific

name of

Options

अनानासकह८मह८सस mdashmdashmdashndash

कावहॴजञाननकनामहहॴ 29-Jan-2017

1) Custard Apple

सीताफि

2) Pineapple

पाइनएपपि

3) Bamboo

बास

4)Pomegranate

अनार

Correct Answer Pineapple

Q174 Which organ produces insulin

कह९नसाअगइनसलिनपहॴदाकरताहहॴ 29-Jan-

2017

Options

1) Liver

यकत

2) Thyroid gland

िायराइिगरिी 3) Spleen

पिीहा 4)Pancreas

अगरयिय

Correct Answer Pancreas

Q175 Which of the following disease is

not caused by water pollution

नननननलिखितमसहॳकह९नसारह८गपानीकहॳ परदषणकहॳकारणनहीहह८ता

29-Jan-2017

Options

1) Cholera

हहॴजा 2) Typhoid

F A C E B O O K

P A G E h t t p w w w f a c e b o o k c o m s s c m e n t o r s o f f i c i a l P a g e | 41

FOR MORE UPDATES AND MORE MATERIAL DO LIKE OUR FACEBOOK PAGE httpwwwfacebookcomsscmentorsofficial

टाइफाइि

3) Asthma

दमा 4)Diarrhoea

दसत

Correct Answer Asthma

Q176 Ocimum tenuiflorum is the

scientific name of

ओलिलममटहॳयईफिह८रमइसकावहॴजञाननकनाम mdash

ndash हहॴ 30-Jan-2017

Options

1) Neem

नीम

2) Mango

आम

3) Babul

बबि

4)Tulsi

तिसी Correct Answer Tulsi

Q177 Which gland secretes bile a

digestive fluid

कह९नसीगरिीपपतत एकपाचनतरिपरदािय सरापवतकरतीहहॴ 30-Jan-2017

Options

1) Pancreas

अगनयािय

2) Liver

यकत

3) Thyroid

िायराइि

4) Testes

टहॳनसटस

Correct Answer liver

Q178 In which of the following the

dominant phase is Gametophyte

नननननलिखितमसहॳककसकहॳ परमिचरणयगमकह८दपवधद (Gametophyte)हहॴ 30-Jan-2017

Options

1) Bryophyta

िायह८फाइटा 2) Pteridophyta

टहॳररिह८फाइटा 3) Gymnosperms

नजननह८सपमय 4) Angiosperms

एननजयह८सपमय Correct Answer Bryophyta

Q179 Anaerobic respiration refers to

which of the following

नननननलिखितमसहॳककसहॳअवायवीयशवसनकहाजाताहहॴ

30-Jan-2017

Options

1) Respiration without Oxygen

ऑकसीजनकहॳ बबनाशवसन

2) Respiration with Oxygen

ऑकसीजनकहॳ सािशवसन

3) Respiration without CO2

काबयनिायऑकसाइिकहॳ बबनाशवसन

4) Respiration with CO2

काबयनिायऑकसाइिकहॳ सािशविन

Correct Answer Respiration without

Oxygen

Q180 Which type of pathogen causes

the waterborne disease Cholera

ककसपरकारकारह८गजनकजिजननतरह८गहहॴजाकाकारणबनताहहॴ

30-Jan-2017

Options

1) Algal

िहॴवालियहॳ

F A C E B O O K

P A G E h t t p w w w f a c e b o o k c o m s s c m e n t o r s o f f i c i a l P a g e | 42

FOR MORE UPDATES AND MORE MATERIAL DO LIKE OUR FACEBOOK PAGE httpwwwfacebookcomsscmentorsofficial

2) Bacterial

बहॴकटीररयि

3) Protozoan

परह८टह८जआ

4) Viral

वायरि

Correct Answer Bacterial

Q181 To which class does

Oxyreductases transferases hydrolases

belong

ओकसीररिकटहॳसटरासफरहॳजहॳस

हाइडरह८िहॳसहॳसककसवगयमआतहॳहहॴ 30-Jan-2017

Options

1) Hormones

हारमोस

2) Enzymes

एजाइनस

3) Proteins

परह८टीनस

4) Vitamins

पवटालमनस

Correct Answer Enzymes

Q182 Which of the following is not true

about Gymnosperms

ननननमसहॳकह९नसीबातअनावतबीजीकहॳ बारहॳमसचनहीहहॴ 30-Jan-2017

Options

1) Dominant phase is saprophytes

परमिचरणसहॳपरह८फाइटसहह८ताहहॴ 2) Vascular bundles are absent

सवहनीबििअनपनसितहह८ताहहॴ 3) spores are heterospores

बीजाणहहॳटहॳरह८सपह८रसहह८तहॳहहॴ 4) Flowers are absent

फिअनपनसितहह८तहॳहहॴ

Correct Answer Vascular bundles are

absent

Q183 The name of first mammal clone sheep is

भहॳड़कीपरिमसतनपायीपरनत प (किह८न)

कानामहहॴ 30-Jan-2017

Options

1) Noori

नरी 2) Dolly

िॉिी 3) Louise

िसी 4)Durga

दगाय Correct Answer Dolly

Q184 Which type of pathogen causes

the water-borne disease Typhoid fever

ककसपरकारकारह८गजनकजिजननतरह८गटाइफाइिबिारकाकारणबनताहहॴ 30-Jan-2017

Options

1) Algal

िहॴवािीय

2) Parasitic

परजीवी 3) Protozoan

परह८टह८जनअन

4)Bacterial

बहॴकटीररयि

Correct Answer Bacterial

Q185 In which part of the cell are

proteins made

कह८लिकाकहॳ ककसटहससहॳमपरह८टीनबनायाजाताहहॴ

31-Jan-2017

Options

1) Reticulum

रहॳटटकिम

F A C E B O O K

P A G E h t t p w w w f a c e b o o k c o m s s c m e n t o r s o f f i c i a l P a g e | 43

FOR MORE UPDATES AND MORE MATERIAL DO LIKE OUR FACEBOOK PAGE httpwwwfacebookcomsscmentorsofficial

2) Golgi apparatus

गह८मजीएपहॳरहॳटस

3) Ribosomes

ररबह८सह८नस

4) Lysosome

िायसह८सह८नस

Correct Answer ribosomes

Proteins are produced by stringing

amino acids together in the order

specified by messenger RNA strands

that were transcribed from DNA in the

cell nucleus The process of synthesizing

a protein is called translation and it

occurs on ribosomes in the cytoplasm of

a cell

Q186 Polio is a disease caused by which

of the following

नननननलिखितमसहॳपह८लियह८कीबबमारह८हह८नहॳकाकारणकयाहहॴ

31-Jan-2017

Options

1) Bacteria

बहॴकटीररयि

2) Mosquito

मचछर

3) Virus

वायरस

4) Cockroach

नतिच हॳ Correct Answer Virus

Polio or poliomyelitis is a crippling and

potentially deadly infectious disease It

is caused by the poliovirus

Q187 ndash Hay fever is a sign of which of

the following

हहॳकफवरनननननलिखितमसहॳककसकाएकसकहॳ तहहॴ

31-Jan-2017

Options

1) Old Age

वदावसिा 2) Malnutrition

कपह८सण

3) Allergy

एिनजय 4) Over Work

अतयचधककाययकरना Correct Answer Allergy

Q188 How many chromosomes does a

human cell contain

एकमानवकह८लिकामककतनहॳगणसतरहह८तहॳहहॴ

29-Jan-2017

Options

1) 6

2) 26

3) 46

4) 66

Correct Answer 46

In humans each cell normally contains

23 pairs of chromosomes for a total of

46 Twenty-two of these pairs called

autosomes look the same in both males

and females The 23rd pair the sex

chromosomes differ between males and

females

Q189 Which of the following is not true

about Bryophyta

ननननमसहॳकह९नसीबातिायह८फाइटकहॳ बारहॳमसचनहीहहॴ 31-Jan-2017

Options

1) Dominant phase is gametophytes

परमिचरणगहॳलमतह८फाइटसहह८ताहहॴ 2) Main plant body is haploid

पह९धहॳकामखयिरीरअगखणतहह८ताहहॴ 3) Spores are homospores

बीजाणहह८मह८सफह८रसहह८तहॳहहॴ 4) Flowers are present

फिमह८जदहह८तहॳहहॴ Correct Answer Flowers are present

F A C E B O O K

P A G E h t t p w w w f a c e b o o k c o m s s c m e n t o r s o f f i c i a l P a g e | 44

FOR MORE UPDATES AND MORE MATERIAL DO LIKE OUR FACEBOOK PAGE httpwwwfacebookcomsscmentorsofficial

Q190 Which aquatic animal has

trailing tentacles

ककसजिीयजानवरकहॳ पीछहॳचिनहॳवािहॳटहॳटकिसहह८तहॳहहॴ

31-Jan-2017

Options

1) Sea horse

समदरीघह८िा 2) Corals

मगा 3) Jelly fish

जहॳिीमछिी 4) Star fish

तारामछिी Correct Answer Jelly fish

Jellyfish with its umbrella-shaped bell

and trailing tentacles

Q191 Which type of pathogen causes

the water-borne disease Poliomyelitis

(Polio)

ककसपरकारकारह८गजनकजिजननतरह८गपह८लियह८मायहॳटटस (पह८लियह८) काकारणहहॴ 31-Jan-

2017

Options

1) Parasitic

परजीवी 2) Algal

िहॴवालिय

3) Viral

वायरि

4) Bacterial

बहॴकटीररयि

Correct Answer Viral

Q192 The outer white part of the eye

that protects the inner structures is

आािकाबाहरीसफहॳ दटहससाजह८आतररकसरचनाओकीरकषाकरताहहॴ वह mdashmdashmdash हहॴ 31-Jan-

2017

Options

1) Iris

आयररस

2) Sclera

सकिहॳरा 3) Retina

रहॳटटना 4) Cornea

कह८ननयया Correct Answer Sclera

Q193 Proteins are made up of

परह८टीनकाननमायण mdashndash सहॳहह८ताहहॴ 31-Jan-2017

Options

1) Amino acids

एलमनह८अनि

2) Fatty acids

वसायकतअनि

3) Glucose

गिकह८ज

4)Nucleotides

नयनकियह८टाईिस

Correct Answer Amino acids

Q194 Moringa Oleifera is the scientific

name of

मह८ररगओलिफहॳ रा mdashmdashndash कावहॴजञाननकनामहहॴ 31-Jan-2017

Options

1) Banyan

बरगद

2) Gulmohar

गिमह८हर

3) Amla

आमिा

F A C E B O O K

P A G E h t t p w w w f a c e b o o k c o m s s c m e n t o r s o f f i c i a l P a g e | 45

FOR MORE UPDATES AND MORE MATERIAL DO LIKE OUR FACEBOOK PAGE httpwwwfacebookcomsscmentorsofficial

4) Drumstick

डरमनसटक

Correct Answer Drumstick

Q195 Kidney stones are composed of

गदकीपिरी mdashndash सहॳबनीहह८तीहहॴ 1-Feb-2017

Options

1) Calcium Oxalate

कहॴ नमसयमओकजहॳिहॳट

2) Sodium Chloride

सह८डियमकिह८राइि

3) Magnesium Nitrate

महॳनगनलियमनाइतटरहॳट

4) Calcium Bicarbonate

कहॴ नमियमबायकबोनहॳट

Correct Answer Calcium Oxalate

Q196 ndash Which of the following is not

true about Angiosperms

ननननमसहॳकह९नसीबातआवतबीजीकहॳ बारहॳमसचनहीहहॴ 1-Feb-2017

Options

1) Dominant phase is gametophytes

परमिचरणगहॳलमतह८फाइटहह८ताहहॴ 2) Vascular bundles are present

सवहनीबििमह९जदहह८ताहहॴ 3) Spores are heterospores

बीजाणहहॳटहॳरह८सपह८रसहह८तहॳहहॴ 4) Seeds are covered

बीजढकहॳ हह८तहॳहहॴ Correct Answer Dominant phase is

gametophytes

Q197 All of the following are excretory

(waste) products of animals except

नननननलिखितमसहॳककसएककह८छह८ड़करअनयसभीपराखणयोदवाराउतसनजयतपदाियहहॴ 1-Feb-

2017

Options

1) Uric Acid

यररकएलसि

2) Ammonia

अमह८ननया 3) Carbohydrates

काबोहाइडरहॳट

4) Urea

यररया Correct Answer Carbohydrates

In animals the main excretory products

are carbon dioxide ammonia (in

ammoniotelics) urea (in ureotelics) uric

acid (in uricotelics) guanine (in

Arachnida) and creatine

Q198 RNA is a polymeric molecule

What does RNA stand for

आरएनइएएकबहिकआणहहॴ इसकाकापवय पकयाहहॴ 1-Feb-2017

Options

1) Rado Nuclear Acid

रािह८नयनकियरएलसि

2) Ribo Nucleic Acid

राइबह८नयनकिकएलसि

3) Rhino Nuclear Acid

हाइनह८नयनकियरएलसि

4) Resto Nucleus Acid

रहॳसटह८नयकिीयसएलसि

Correct Answer Ribo Nucleic Acid

Q199 Which organ does detoxification

and produces chemicals needed for

digestion

कह९नसाअगपवषहरणकरताहहॴऔरपाचनकहॳ लिएआवशयकरसायनोकह८पहॴदाकरताहहॴ 1-Feb-

2017

Options

1) Salivary glands

िारगरचिया 2) Pancreas

अगनयािय

F A C E B O O K

P A G E h t t p w w w f a c e b o o k c o m s s c m e n t o r s o f f i c i a l P a g e | 46

FOR MORE UPDATES AND MORE MATERIAL DO LIKE OUR FACEBOOK PAGE httpwwwfacebookcomsscmentorsofficial

3) Thyroid gland

िायराइिगरिी 4) Liver

यकत

Correct Answer Liver

Q200 Psidium guajava is the scientific

name of

लसडियमगआजावा mdashmdash कावहॴजञाननकनामहहॴ 1-

Feb-2017

Options

1) Guava

अम द

2) Mango

आम

3) Bamboo

बास

4) Jack fruit

कटहि

Correct Answer Guava

Q201 Which drug is used as a Blood

Thinner

चधरकह८पतिाकरनहॳकहॳ पमककसदवाकापरयह८गककयाजाताहहॴ

1-Feb-2017

Options

1) Warfarin

वाफर न

2) Tramadol

टरहॳमािह८ि

3) Azithromycin

एनजरह८मायलसन

4) Hydralazine

हाइडरह८िहॳनजन

Correct Answer Warfarin

Q202 Which of the following disease is

caused due to the deficiency of protein

परह८टीनकीकमीकहॳ कारणनननननलिखितमसहॳकह९नसारह८गहह८ताहहॴ 1-Feb-2017

Options

1) Arthritis

गटठया 2) Kwashiorkor

कािीओकय र

3) Goitre

गाइटर

4) Night Blindness

रतह९चध

Correct Answer Kwashiorkor

Q203 A is species of plant that has

adapted to survive in an environment

with little liquid water

mdashmdashndashपह९धहॳकीएकऐसहॳऐसहॳपरजानतहहॴ नजसनहॳकमपानीवािहॳवातावरणमजीपवतरहनहॳकहॳलिएअनकिनहहॴ 1-Feb-2017

Options

1) Xerophyte

म दपवद

2) Hydrophyte

जिीयपादप

3) Mesophyte

समह८दपवद

4) Thallophyte

िहॴिह८फाइटा Correct Answer xerophyte

xerophyte is a species of plant that has

adapted to survive in an environment

with little liquid water such as a desert

or an ice- or snow-covered region in the

Alps or the Arctic

Mesophytes are terrestrial plants which

are adapted to neither a particularly

dry nor particularly wet environment

An example of a mesophytic habitat

would be a rural temperate meadow

F A C E B O O K

P A G E h t t p w w w f a c e b o o k c o m s s c m e n t o r s o f f i c i a l P a g e | 47

FOR MORE UPDATES AND MORE MATERIAL DO LIKE OUR FACEBOOK PAGE httpwwwfacebookcomsscmentorsofficial

which might contain goldenrod clover

oxeye daisy and Rosa multiflora

thallophyte any of a group of plants or

plantlike organisms (such as algae and

fungi) that lack differentiated stems

leaves and roots and that were formerly

classified as a primary division

(Thallophyta) of the plant kingdom

Q204 How many types of teeth are

there in humans

मनषयोमककतनहॳपरकारकहॳ दातहह८तहॳहहॴ

1-Feb-2017

Options

1) 4

2) 5

3) 2

4) 3

Correct Answer 4

teeth -Humans have four types of

teethincisors canines premolars and

molars each with a specific function

The incisors cut the food the canines

tear the food and the molars and

premolars crush the food

Q205 Carica papaya is the scientific name of

कहॴ ररकापपाया mdashmdashndash कावहॴजञाननकनामहहॴ 2-

Feb-2017

Options

1) Peepal

पीपि

2) Papaya

पपीता 3) Tamarind

इमिी 4) Drumstick

ढह८िकाछड़ी Correct Answer Papaya

Q206 Muscles get tired when there is

shortfall of

जब mdashndash कीकमीहह८तीहहॴतबपहॳिीयिकजातीहहॴ 2-Feb-2017

Options

1) Lactic acid

िहॴनकटकएलसि

2) Na+ ions

Na+ आयन

3) ATP

एटीपी 4) Sulphates

समफहॳ टस

Correct Answer ATP

ATP is the energy source muscle fibers

use to make muscles contract

muscle tissuersquos main source of energy

called adenosine triphosphate or ATP

As your muscles use up this energy

source they become tired and fatigued

Oxygen is the key ingredient that helps

create new ATP to replenish the burned

up ATP in your muscles

Q207 Artocarpus integra is the

scientific name of आटह८कापयसइटीगरा mdashmdashmdash कावहॴजञाननकनामहहॴ 2-Feb-2017

Options

1) Guava

अम द

2) Pineapple

अनानास

3) Silver Oak

लसमवरओक

4) Jack fruit

कटहि

Correct Answer Jack fruit

Q208 Which organ stores fat soluble

vitamins

कह९नसाअगवसामघिनिीिपवटालमनह८काभिाराकरताहहॴ

2-Feb-2017

F A C E B O O K

P A G E h t t p w w w f a c e b o o k c o m s s c m e n t o r s o f f i c i a l P a g e | 48

FOR MORE UPDATES AND MORE MATERIAL DO LIKE OUR FACEBOOK PAGE httpwwwfacebookcomsscmentorsofficial

Options

1) Blood

रकत

2) Skin

तवचा 3) Liver

यकत

4) Pancreas

अगनयािय

Correct Answer Liver

Q209 Which disease is caused due to

deficiency of Iodine

आयह८िीनकहॳ कारणकह९नसारह८गहह८ताहहॴ 2-Feb-2017

Options

1) Rickets

ररकहॳ टस

2) Scurvy

सकवी 3) Goitre

गणमािा 4) Growth retardation

पवकासका कना Correct Answer Goitre

rickets A softening and weakening of

bones in children usually due to

inadequate vitamin D

Q210 Grevillea Robusta is the scientific name of

गरहॳपवलियारह८बसटा mdashmdashmdash- कापवजञाननकनामहहॴ 2-Feb-2017

Options

1) Peepal

पीपि

2) Teak

सागह९न

3) Silver Oak

लसमवरओक

4) Jack fruit

कटहि

Correct Answer Silver Oak

Q211 When a Cuttlefish is described as a Molluscs it is at which level of

classification

जबएककटिकफिकह८एकमह८िसकाकहॳ पमवखणयतककयाजाताहहॴतबयहॳवगीकरणकहॳ ककससतरपहॳनसितहहॴ 2-Feb-2017

Options

1) Class

वगय 2) Order

िम

3) Family

पररवार

4) Phylum

सघ

Correct Answer Phylum

Q212 Bambusa dendrocalmus is the

scientific name of बानबसािहॳडराकामस mdashmdashmdash कावहॴजञाननकनामहहॴ 3-Feb-2017

Options

1) Banyan

बरगद

2) Papaya

पपीता 3) Bamboo

बास

4) Pomegranate

अनार

Correct Answer Bamboo

Q213 Acinonyx Jubatus is the scientific name of

एलसनह८ननकसजयबहॳटस mdashmdashmdash

कावहॴजञाननकनामहहॴ 3-Feb-2017

F A C E B O O K

P A G E h t t p w w w f a c e b o o k c o m s s c m e n t o r s o f f i c i a l P a g e | 49

FOR MORE UPDATES AND MORE MATERIAL DO LIKE OUR FACEBOOK PAGE httpwwwfacebookcomsscmentorsofficial

Options

1) Bear

भाि 2) Horse

घह८िा 3) Cheetah

चीता 4) Zebra

जहॳिा Correct Answer Cheetah

Q214 The pale yellow colour of urine is

due to the presence of which pigment

मतरकाफीकापीिारगरगदरयकहॳ उपनसिनतकहॳ कारणहह८ताहहॴ

3-Feb-2017

Options

1) Urochrome

यरह८िह८म

2) Urophyll

यरह८कफि

3) Chlorophyll

किह८रह८कफि

4) Chloroplast

किह८रह८पिासट

Correct Answer Urochrome

Q215 Which of the following constitute

to form a gene

नननननलिखितमसहॳकह९नसीचीज़एकजीनकागठनकरतीहहॴ

3-Feb-2017

Options

1) Polynucleotides

पह८िीनयनकियह८टाईडस

2) Hydrocarbons

हाइडरह८काबोस

3) Lipoproteins

िाईपह८परह८टीनस

4) Lipids

लिपपडस

Correct Answer Polynucleotides

Polynucleotide molecule is a biopolymer

composed of 13 or more nucleotide

monomers covalently bonded in a chain

DNA (deoxyribonucleic acid) and RNA

(ribonucleic acid) are examples of

polynucleotides with distinct biological

function

Q216 Vertebrates belongs to the

phylum

रीढ़कीहडिीवािहॳपराणी mdashmdashmdash

परजानतकहॳ अतगायतआतहॳहहॴ 3-Feb-2017

Options

1) Arthropoda

आरह८पह८ड़ा 2) Annelida

एननलििा 3) Cnidaria

ननिहॳररया 4) Chordata

कह८िटा Correct Answer Chordata

Q217 Punica granatum is the scientific name of

पननकगरहॳनहॳटस mdashmdashmdash कावहॴजञाननकनामहहॴ 3-Feb-2017

Options

1) Custard Apple

सीताफि

2) Gulmohar

गिमह८हर

3) Silver Oak

लसमवरओक

4) Pomegranate

अनार

Correct Answer Pomegranate

F A C E B O O K

P A G E h t t p w w w f a c e b o o k c o m s s c m e n t o r s o f f i c i a l P a g e | 50

FOR MORE UPDATES AND MORE MATERIAL DO LIKE OUR FACEBOOK PAGE httpwwwfacebookcomsscmentorsofficial

Q218 Between a tiger and an monkey

which of the following is different

एकबाघऔरबदरकहॳ बीचनननननलिखितमसहॳकह९नसीबातअिगहहॴ 3-Feb-2017

Options

1) Kingdom

राजय

2) Phylum

जानत

3) Order

िम

4) Class

वगय Correct Answer order

Q219 The artificial heart was invented by

कबतरमहदयका mdashmdashmdash

दवाराअपवषकारककयागयािा 3-Feb-2017

Options

1) Muhammad Yunus

महनमदयनस

2) Linus Yale Jr

िाइनसयहॳिजय

3) Gazi Yasargil

गाजीयासचगयि

4) Paul Winchell

पह९िपवमकि Correct Answer Paul Winchell

Q220 Tamarindus indica is the

scientific name of

टहॳमररनडसइडिका mdashmdash कावहॴजञाननकनामहहॴ 7-

Feb-2017

Options

1) Neem

नीम

2) Pineapple

अनानास

3) Tamarind

इमिी 4)Chiku

चीक

Correct Answer Tamarind

Q221 In eukaryotic cells synthesis of

RNA takes place in the

यकहॳ योटटककह८लिकाओमआरएनएकासशिहॳषण

mdashndash महह८ताहहॴ 7-Feb-2017

Options

1) Mitochondria

माईटह८कोडडरया 2) Centrioles

सटरीयह८मस

3) Ribosomes

ररबह८सह८नस

4) Nucleus

नयनकियस

Correct Answer nucleus

eukaryotic cell -Transcription is the

process of synthesizing ribonucleic acid

(RNA)Synthesis takes place within the

nucleus of eukaryotic cells or in the

cytoplasm of prokaryotes and converts

the genetic code from a gene in

deoxyribonucleic acid ( DNA ) to a

strand of RNA that then directs

proteinsynthesis

Q222 _________is caused by parasites

of the Plasmodium genus

पिाजमह८डियमजातीकहॳ परजीवी mdash- कहॳ कारणहहॴ 7-Feb-2017

Options

1) Dysentery

पहॳचचि

2) Malaria

मिहॳररया 3) Chickenpox

F A C E B O O K

P A G E h t t p w w w f a c e b o o k c o m s s c m e n t o r s o f f i c i a l P a g e | 51

FOR MORE UPDATES AND MORE MATERIAL DO LIKE OUR FACEBOOK PAGE httpwwwfacebookcomsscmentorsofficial

चहॳचक

4) Herpes

हहॳपपयस

Correct Answer Malaria

Q223 Carotene in fruits and vegetables

gives it which color

फिह८औरसनलजयोमनसितकहॳ रह८टीनउनहकह९नसारगपरदानकरताहहॴ 7-Feb-2017

Options

1) Green

हरा 2) Pink

गिाबी 3) Orange

नारगी 4) Blue

नीिा Correct Answer Orange

Q224 Equus Caballus is the scientific

name of

एकवसकहॴ बहॳिस mdashmdashndash कापवजञाननकनामहहॴ 7-Feb-2017

Options

1) Horse

घह८िा 2) Zebra

ज़हॳिा 3) Donkey

गधा 4) Buffalo

भस

Correct Answer Horse

Q225 Elapidae Naja is the scientific name of

एिीपीिीनाजा mdashmdash- कावहॴजञाननकनामहहॴ 8-Feb-2017

Options

1) Cobra

कह८बरा 2) Elephant

हािी 3) Eagle

ग ि

4) Owl

उमि Correct Answer Cobra

Q226 Which disease is caused due to

deficiency of Iron

िह८हकीकमीकहॳ कारणकह९नसारह८गहह८ताहहॴ 8-Feb-

2017

Options

1) Beriberi

बहॳरीबहॳरी 2) Tetany

टहॳटनी 3) Kwashiorkor

कवािीऔरकर

4) Anaemia

रकतामपता Correct Answer Anaemia

Beriberi is a disease caused by a vitamin

B-1 deficiency also known as thiamine

deficiency

Tetany can be the result of an

electrolyte imbalance Most often itrsquos a

dramatically low calcium level also

known as hypocalcemia Tetany can also

be caused by magnesium deficiency or

too little potassium Having too much

acid (acidosis) or too much alkali

(alkalosis) in the body can also result in

tetany

Kwashiorkor also known as

ldquoedematous malnutrition It is a form of

malnutrition caused by a lack of protein

in the diet

Anaemia means that you have fewer red

blood cells than normal or you have less

F A C E B O O K

P A G E h t t p w w w f a c e b o o k c o m s s c m e n t o r s o f f i c i a l P a g e | 52

FOR MORE UPDATES AND MORE MATERIAL DO LIKE OUR FACEBOOK PAGE httpwwwfacebookcomsscmentorsofficial

haemoglobin than normal in each red

blood cell

Q227 is a leaf where the leaflets are

arranged along the middle vein

mdashndashएकपततीहहॴजहापतरकह८कीरचनाक ररयालिराकहॳ आसपासहह८तीहहॴ 8-Feb-2017

Options

1) Pinnately compound leaf

पपनहॳटिीसयकतपतती 2) Palmately compound leaf

पामहॳटिीसयकतपतती 3) Compound leaf

सयकतपतती 4) Simple leaf

साधारणपतती Correct Answer Pinnately compound

leaf

Q228 Haustoria or sucking roots are

found in which of the following

हह८सटह८ररयायाचसनहॳवािीजड़हॳनननननलिखितमसहॳककसमपाईजातीहहॴ 8-Feb-2017

Options

1) Wheat

गहॳह

2) Mango

आम

3) Chestnut

चहॳसटनट

4) Cuscuta

कसकयटा Correct Answer Cuscuta

Haustorial roots -The roots of parasitic

plants which penetrate into the host

tissues to absorb nourishment are

called haustorial roots hellip Also known as suckingor parasitic roots

Q229 Equs Asinus is the scientific name

of

एकवसएलसनस mdashmdashndash कावहॴजञाननकनामहहॴ 8-

Feb-2017

Options

1) Donkey

गधा 2) Cow

गाय

3) Deer

टहरन

4) Kangaroo

कगा

Correct Answer Donkey

Q230 Ficus benghalensis is the scientific name of

फाईकसबहॳनगहॳिहॳलसस mdashndash कापवजञाननकनामहहॴ 8-Feb-2017

Options

1) Banyan

बरगद

2) Pineapple

अनानास

3) Babul

बबि

4) Tulsi

तिसी Correct Answer Banyan

Q231 Equus burchellii is the scientific name of

एकवसबचिी mdashmdash- कापवजञाननकनामहहॴ 8-Feb-2017

Options

1) Horse

घह८िा 2) Zebra

जहॳिा 3) Buffalo

F A C E B O O K

P A G E h t t p w w w f a c e b o o k c o m s s c m e n t o r s o f f i c i a l P a g e | 53

FOR MORE UPDATES AND MORE MATERIAL DO LIKE OUR FACEBOOK PAGE httpwwwfacebookcomsscmentorsofficial

भस

4) Ass

गधा Correct Answer Zebra

Page 41: COMPILATION OF ALL 72 SETS OF BIOLOGY SSC CHSL-2016 · OF BIOLOGY SSC CHSL-2016 PREPARED BY : SSC MENTORS BIOLOGY SPECIAL . F A C E B O O K P A G E : h t t p : / / w w w . f a c e

F A C E B O O K

P A G E h t t p w w w f a c e b o o k c o m s s c m e n t o r s o f f i c i a l P a g e | 40

FOR MORE UPDATES AND MORE MATERIAL DO LIKE OUR FACEBOOK PAGE httpwwwfacebookcomsscmentorsofficial

मकड़ीबदर

Correct Answer Spider monkey

Q171 Which type of pathogen causes

the waterborne disease Salmonellosis

ककसपरकारकारह८गाणजिजननतबीमारीसािमह८नहॳिह८लसज़काकारकहहॴ

29-Jan-2017

Options

1) Algal

िहॳवालियहॳ 2) Parasitic

परजीवी 3) Bacterial

बहॴकटीररयि

4)Viral

वायरि

Correct Answer Bacterial

An infection with salmonella bacteria

commonly caused by contaminated food

or water

Symptoms include diarrhoea fever

chills and abdominal pain

Q172 is a condition in which there is a

deficiency of red cells or of haemoglobin

in the blood

mdashmdash-

एकनसिनतहहॴनजसमहॳरकतमिािकह८लिकाओकीयाहीमह८गिह८बबनकीकमीहह८तीहहॴ 29-Jan-2017

Options

1) Albinism

एनमबननजम

2) Propyria

परह८पीररया 3) Anaemia

एनीलमया 4)Keloid disorder

कहॳ िह८इिडिसओिर

Correct Answer Anaemia

Q173 Ananas comosus is the scientific

name of

Options

अनानासकह८मह८सस mdashmdashmdashndash

कावहॴजञाननकनामहहॴ 29-Jan-2017

1) Custard Apple

सीताफि

2) Pineapple

पाइनएपपि

3) Bamboo

बास

4)Pomegranate

अनार

Correct Answer Pineapple

Q174 Which organ produces insulin

कह९नसाअगइनसलिनपहॴदाकरताहहॴ 29-Jan-

2017

Options

1) Liver

यकत

2) Thyroid gland

िायराइिगरिी 3) Spleen

पिीहा 4)Pancreas

अगरयिय

Correct Answer Pancreas

Q175 Which of the following disease is

not caused by water pollution

नननननलिखितमसहॳकह९नसारह८गपानीकहॳ परदषणकहॳकारणनहीहह८ता

29-Jan-2017

Options

1) Cholera

हहॴजा 2) Typhoid

F A C E B O O K

P A G E h t t p w w w f a c e b o o k c o m s s c m e n t o r s o f f i c i a l P a g e | 41

FOR MORE UPDATES AND MORE MATERIAL DO LIKE OUR FACEBOOK PAGE httpwwwfacebookcomsscmentorsofficial

टाइफाइि

3) Asthma

दमा 4)Diarrhoea

दसत

Correct Answer Asthma

Q176 Ocimum tenuiflorum is the

scientific name of

ओलिलममटहॳयईफिह८रमइसकावहॴजञाननकनाम mdash

ndash हहॴ 30-Jan-2017

Options

1) Neem

नीम

2) Mango

आम

3) Babul

बबि

4)Tulsi

तिसी Correct Answer Tulsi

Q177 Which gland secretes bile a

digestive fluid

कह९नसीगरिीपपतत एकपाचनतरिपरदािय सरापवतकरतीहहॴ 30-Jan-2017

Options

1) Pancreas

अगनयािय

2) Liver

यकत

3) Thyroid

िायराइि

4) Testes

टहॳनसटस

Correct Answer liver

Q178 In which of the following the

dominant phase is Gametophyte

नननननलिखितमसहॳककसकहॳ परमिचरणयगमकह८दपवधद (Gametophyte)हहॴ 30-Jan-2017

Options

1) Bryophyta

िायह८फाइटा 2) Pteridophyta

टहॳररिह८फाइटा 3) Gymnosperms

नजननह८सपमय 4) Angiosperms

एननजयह८सपमय Correct Answer Bryophyta

Q179 Anaerobic respiration refers to

which of the following

नननननलिखितमसहॳककसहॳअवायवीयशवसनकहाजाताहहॴ

30-Jan-2017

Options

1) Respiration without Oxygen

ऑकसीजनकहॳ बबनाशवसन

2) Respiration with Oxygen

ऑकसीजनकहॳ सािशवसन

3) Respiration without CO2

काबयनिायऑकसाइिकहॳ बबनाशवसन

4) Respiration with CO2

काबयनिायऑकसाइिकहॳ सािशविन

Correct Answer Respiration without

Oxygen

Q180 Which type of pathogen causes

the waterborne disease Cholera

ककसपरकारकारह८गजनकजिजननतरह८गहहॴजाकाकारणबनताहहॴ

30-Jan-2017

Options

1) Algal

िहॴवालियहॳ

F A C E B O O K

P A G E h t t p w w w f a c e b o o k c o m s s c m e n t o r s o f f i c i a l P a g e | 42

FOR MORE UPDATES AND MORE MATERIAL DO LIKE OUR FACEBOOK PAGE httpwwwfacebookcomsscmentorsofficial

2) Bacterial

बहॴकटीररयि

3) Protozoan

परह८टह८जआ

4) Viral

वायरि

Correct Answer Bacterial

Q181 To which class does

Oxyreductases transferases hydrolases

belong

ओकसीररिकटहॳसटरासफरहॳजहॳस

हाइडरह८िहॳसहॳसककसवगयमआतहॳहहॴ 30-Jan-2017

Options

1) Hormones

हारमोस

2) Enzymes

एजाइनस

3) Proteins

परह८टीनस

4) Vitamins

पवटालमनस

Correct Answer Enzymes

Q182 Which of the following is not true

about Gymnosperms

ननननमसहॳकह९नसीबातअनावतबीजीकहॳ बारहॳमसचनहीहहॴ 30-Jan-2017

Options

1) Dominant phase is saprophytes

परमिचरणसहॳपरह८फाइटसहह८ताहहॴ 2) Vascular bundles are absent

सवहनीबििअनपनसितहह८ताहहॴ 3) spores are heterospores

बीजाणहहॳटहॳरह८सपह८रसहह८तहॳहहॴ 4) Flowers are absent

फिअनपनसितहह८तहॳहहॴ

Correct Answer Vascular bundles are

absent

Q183 The name of first mammal clone sheep is

भहॳड़कीपरिमसतनपायीपरनत प (किह८न)

कानामहहॴ 30-Jan-2017

Options

1) Noori

नरी 2) Dolly

िॉिी 3) Louise

िसी 4)Durga

दगाय Correct Answer Dolly

Q184 Which type of pathogen causes

the water-borne disease Typhoid fever

ककसपरकारकारह८गजनकजिजननतरह८गटाइफाइिबिारकाकारणबनताहहॴ 30-Jan-2017

Options

1) Algal

िहॴवािीय

2) Parasitic

परजीवी 3) Protozoan

परह८टह८जनअन

4)Bacterial

बहॴकटीररयि

Correct Answer Bacterial

Q185 In which part of the cell are

proteins made

कह८लिकाकहॳ ककसटहससहॳमपरह८टीनबनायाजाताहहॴ

31-Jan-2017

Options

1) Reticulum

रहॳटटकिम

F A C E B O O K

P A G E h t t p w w w f a c e b o o k c o m s s c m e n t o r s o f f i c i a l P a g e | 43

FOR MORE UPDATES AND MORE MATERIAL DO LIKE OUR FACEBOOK PAGE httpwwwfacebookcomsscmentorsofficial

2) Golgi apparatus

गह८मजीएपहॳरहॳटस

3) Ribosomes

ररबह८सह८नस

4) Lysosome

िायसह८सह८नस

Correct Answer ribosomes

Proteins are produced by stringing

amino acids together in the order

specified by messenger RNA strands

that were transcribed from DNA in the

cell nucleus The process of synthesizing

a protein is called translation and it

occurs on ribosomes in the cytoplasm of

a cell

Q186 Polio is a disease caused by which

of the following

नननननलिखितमसहॳपह८लियह८कीबबमारह८हह८नहॳकाकारणकयाहहॴ

31-Jan-2017

Options

1) Bacteria

बहॴकटीररयि

2) Mosquito

मचछर

3) Virus

वायरस

4) Cockroach

नतिच हॳ Correct Answer Virus

Polio or poliomyelitis is a crippling and

potentially deadly infectious disease It

is caused by the poliovirus

Q187 ndash Hay fever is a sign of which of

the following

हहॳकफवरनननननलिखितमसहॳककसकाएकसकहॳ तहहॴ

31-Jan-2017

Options

1) Old Age

वदावसिा 2) Malnutrition

कपह८सण

3) Allergy

एिनजय 4) Over Work

अतयचधककाययकरना Correct Answer Allergy

Q188 How many chromosomes does a

human cell contain

एकमानवकह८लिकामककतनहॳगणसतरहह८तहॳहहॴ

29-Jan-2017

Options

1) 6

2) 26

3) 46

4) 66

Correct Answer 46

In humans each cell normally contains

23 pairs of chromosomes for a total of

46 Twenty-two of these pairs called

autosomes look the same in both males

and females The 23rd pair the sex

chromosomes differ between males and

females

Q189 Which of the following is not true

about Bryophyta

ननननमसहॳकह९नसीबातिायह८फाइटकहॳ बारहॳमसचनहीहहॴ 31-Jan-2017

Options

1) Dominant phase is gametophytes

परमिचरणगहॳलमतह८फाइटसहह८ताहहॴ 2) Main plant body is haploid

पह९धहॳकामखयिरीरअगखणतहह८ताहहॴ 3) Spores are homospores

बीजाणहह८मह८सफह८रसहह८तहॳहहॴ 4) Flowers are present

फिमह८जदहह८तहॳहहॴ Correct Answer Flowers are present

F A C E B O O K

P A G E h t t p w w w f a c e b o o k c o m s s c m e n t o r s o f f i c i a l P a g e | 44

FOR MORE UPDATES AND MORE MATERIAL DO LIKE OUR FACEBOOK PAGE httpwwwfacebookcomsscmentorsofficial

Q190 Which aquatic animal has

trailing tentacles

ककसजिीयजानवरकहॳ पीछहॳचिनहॳवािहॳटहॳटकिसहह८तहॳहहॴ

31-Jan-2017

Options

1) Sea horse

समदरीघह८िा 2) Corals

मगा 3) Jelly fish

जहॳिीमछिी 4) Star fish

तारामछिी Correct Answer Jelly fish

Jellyfish with its umbrella-shaped bell

and trailing tentacles

Q191 Which type of pathogen causes

the water-borne disease Poliomyelitis

(Polio)

ककसपरकारकारह८गजनकजिजननतरह८गपह८लियह८मायहॳटटस (पह८लियह८) काकारणहहॴ 31-Jan-

2017

Options

1) Parasitic

परजीवी 2) Algal

िहॴवालिय

3) Viral

वायरि

4) Bacterial

बहॴकटीररयि

Correct Answer Viral

Q192 The outer white part of the eye

that protects the inner structures is

आािकाबाहरीसफहॳ दटहससाजह८आतररकसरचनाओकीरकषाकरताहहॴ वह mdashmdashmdash हहॴ 31-Jan-

2017

Options

1) Iris

आयररस

2) Sclera

सकिहॳरा 3) Retina

रहॳटटना 4) Cornea

कह८ननयया Correct Answer Sclera

Q193 Proteins are made up of

परह८टीनकाननमायण mdashndash सहॳहह८ताहहॴ 31-Jan-2017

Options

1) Amino acids

एलमनह८अनि

2) Fatty acids

वसायकतअनि

3) Glucose

गिकह८ज

4)Nucleotides

नयनकियह८टाईिस

Correct Answer Amino acids

Q194 Moringa Oleifera is the scientific

name of

मह८ररगओलिफहॳ रा mdashmdashndash कावहॴजञाननकनामहहॴ 31-Jan-2017

Options

1) Banyan

बरगद

2) Gulmohar

गिमह८हर

3) Amla

आमिा

F A C E B O O K

P A G E h t t p w w w f a c e b o o k c o m s s c m e n t o r s o f f i c i a l P a g e | 45

FOR MORE UPDATES AND MORE MATERIAL DO LIKE OUR FACEBOOK PAGE httpwwwfacebookcomsscmentorsofficial

4) Drumstick

डरमनसटक

Correct Answer Drumstick

Q195 Kidney stones are composed of

गदकीपिरी mdashndash सहॳबनीहह८तीहहॴ 1-Feb-2017

Options

1) Calcium Oxalate

कहॴ नमसयमओकजहॳिहॳट

2) Sodium Chloride

सह८डियमकिह८राइि

3) Magnesium Nitrate

महॳनगनलियमनाइतटरहॳट

4) Calcium Bicarbonate

कहॴ नमियमबायकबोनहॳट

Correct Answer Calcium Oxalate

Q196 ndash Which of the following is not

true about Angiosperms

ननननमसहॳकह९नसीबातआवतबीजीकहॳ बारहॳमसचनहीहहॴ 1-Feb-2017

Options

1) Dominant phase is gametophytes

परमिचरणगहॳलमतह८फाइटहह८ताहहॴ 2) Vascular bundles are present

सवहनीबििमह९जदहह८ताहहॴ 3) Spores are heterospores

बीजाणहहॳटहॳरह८सपह८रसहह८तहॳहहॴ 4) Seeds are covered

बीजढकहॳ हह८तहॳहहॴ Correct Answer Dominant phase is

gametophytes

Q197 All of the following are excretory

(waste) products of animals except

नननननलिखितमसहॳककसएककह८छह८ड़करअनयसभीपराखणयोदवाराउतसनजयतपदाियहहॴ 1-Feb-

2017

Options

1) Uric Acid

यररकएलसि

2) Ammonia

अमह८ननया 3) Carbohydrates

काबोहाइडरहॳट

4) Urea

यररया Correct Answer Carbohydrates

In animals the main excretory products

are carbon dioxide ammonia (in

ammoniotelics) urea (in ureotelics) uric

acid (in uricotelics) guanine (in

Arachnida) and creatine

Q198 RNA is a polymeric molecule

What does RNA stand for

आरएनइएएकबहिकआणहहॴ इसकाकापवय पकयाहहॴ 1-Feb-2017

Options

1) Rado Nuclear Acid

रािह८नयनकियरएलसि

2) Ribo Nucleic Acid

राइबह८नयनकिकएलसि

3) Rhino Nuclear Acid

हाइनह८नयनकियरएलसि

4) Resto Nucleus Acid

रहॳसटह८नयकिीयसएलसि

Correct Answer Ribo Nucleic Acid

Q199 Which organ does detoxification

and produces chemicals needed for

digestion

कह९नसाअगपवषहरणकरताहहॴऔरपाचनकहॳ लिएआवशयकरसायनोकह८पहॴदाकरताहहॴ 1-Feb-

2017

Options

1) Salivary glands

िारगरचिया 2) Pancreas

अगनयािय

F A C E B O O K

P A G E h t t p w w w f a c e b o o k c o m s s c m e n t o r s o f f i c i a l P a g e | 46

FOR MORE UPDATES AND MORE MATERIAL DO LIKE OUR FACEBOOK PAGE httpwwwfacebookcomsscmentorsofficial

3) Thyroid gland

िायराइिगरिी 4) Liver

यकत

Correct Answer Liver

Q200 Psidium guajava is the scientific

name of

लसडियमगआजावा mdashmdash कावहॴजञाननकनामहहॴ 1-

Feb-2017

Options

1) Guava

अम द

2) Mango

आम

3) Bamboo

बास

4) Jack fruit

कटहि

Correct Answer Guava

Q201 Which drug is used as a Blood

Thinner

चधरकह८पतिाकरनहॳकहॳ पमककसदवाकापरयह८गककयाजाताहहॴ

1-Feb-2017

Options

1) Warfarin

वाफर न

2) Tramadol

टरहॳमािह८ि

3) Azithromycin

एनजरह८मायलसन

4) Hydralazine

हाइडरह८िहॳनजन

Correct Answer Warfarin

Q202 Which of the following disease is

caused due to the deficiency of protein

परह८टीनकीकमीकहॳ कारणनननननलिखितमसहॳकह९नसारह८गहह८ताहहॴ 1-Feb-2017

Options

1) Arthritis

गटठया 2) Kwashiorkor

कािीओकय र

3) Goitre

गाइटर

4) Night Blindness

रतह९चध

Correct Answer Kwashiorkor

Q203 A is species of plant that has

adapted to survive in an environment

with little liquid water

mdashmdashndashपह९धहॳकीएकऐसहॳऐसहॳपरजानतहहॴ नजसनहॳकमपानीवािहॳवातावरणमजीपवतरहनहॳकहॳलिएअनकिनहहॴ 1-Feb-2017

Options

1) Xerophyte

म दपवद

2) Hydrophyte

जिीयपादप

3) Mesophyte

समह८दपवद

4) Thallophyte

िहॴिह८फाइटा Correct Answer xerophyte

xerophyte is a species of plant that has

adapted to survive in an environment

with little liquid water such as a desert

or an ice- or snow-covered region in the

Alps or the Arctic

Mesophytes are terrestrial plants which

are adapted to neither a particularly

dry nor particularly wet environment

An example of a mesophytic habitat

would be a rural temperate meadow

F A C E B O O K

P A G E h t t p w w w f a c e b o o k c o m s s c m e n t o r s o f f i c i a l P a g e | 47

FOR MORE UPDATES AND MORE MATERIAL DO LIKE OUR FACEBOOK PAGE httpwwwfacebookcomsscmentorsofficial

which might contain goldenrod clover

oxeye daisy and Rosa multiflora

thallophyte any of a group of plants or

plantlike organisms (such as algae and

fungi) that lack differentiated stems

leaves and roots and that were formerly

classified as a primary division

(Thallophyta) of the plant kingdom

Q204 How many types of teeth are

there in humans

मनषयोमककतनहॳपरकारकहॳ दातहह८तहॳहहॴ

1-Feb-2017

Options

1) 4

2) 5

3) 2

4) 3

Correct Answer 4

teeth -Humans have four types of

teethincisors canines premolars and

molars each with a specific function

The incisors cut the food the canines

tear the food and the molars and

premolars crush the food

Q205 Carica papaya is the scientific name of

कहॴ ररकापपाया mdashmdashndash कावहॴजञाननकनामहहॴ 2-

Feb-2017

Options

1) Peepal

पीपि

2) Papaya

पपीता 3) Tamarind

इमिी 4) Drumstick

ढह८िकाछड़ी Correct Answer Papaya

Q206 Muscles get tired when there is

shortfall of

जब mdashndash कीकमीहह८तीहहॴतबपहॳिीयिकजातीहहॴ 2-Feb-2017

Options

1) Lactic acid

िहॴनकटकएलसि

2) Na+ ions

Na+ आयन

3) ATP

एटीपी 4) Sulphates

समफहॳ टस

Correct Answer ATP

ATP is the energy source muscle fibers

use to make muscles contract

muscle tissuersquos main source of energy

called adenosine triphosphate or ATP

As your muscles use up this energy

source they become tired and fatigued

Oxygen is the key ingredient that helps

create new ATP to replenish the burned

up ATP in your muscles

Q207 Artocarpus integra is the

scientific name of आटह८कापयसइटीगरा mdashmdashmdash कावहॴजञाननकनामहहॴ 2-Feb-2017

Options

1) Guava

अम द

2) Pineapple

अनानास

3) Silver Oak

लसमवरओक

4) Jack fruit

कटहि

Correct Answer Jack fruit

Q208 Which organ stores fat soluble

vitamins

कह९नसाअगवसामघिनिीिपवटालमनह८काभिाराकरताहहॴ

2-Feb-2017

F A C E B O O K

P A G E h t t p w w w f a c e b o o k c o m s s c m e n t o r s o f f i c i a l P a g e | 48

FOR MORE UPDATES AND MORE MATERIAL DO LIKE OUR FACEBOOK PAGE httpwwwfacebookcomsscmentorsofficial

Options

1) Blood

रकत

2) Skin

तवचा 3) Liver

यकत

4) Pancreas

अगनयािय

Correct Answer Liver

Q209 Which disease is caused due to

deficiency of Iodine

आयह८िीनकहॳ कारणकह९नसारह८गहह८ताहहॴ 2-Feb-2017

Options

1) Rickets

ररकहॳ टस

2) Scurvy

सकवी 3) Goitre

गणमािा 4) Growth retardation

पवकासका कना Correct Answer Goitre

rickets A softening and weakening of

bones in children usually due to

inadequate vitamin D

Q210 Grevillea Robusta is the scientific name of

गरहॳपवलियारह८बसटा mdashmdashmdash- कापवजञाननकनामहहॴ 2-Feb-2017

Options

1) Peepal

पीपि

2) Teak

सागह९न

3) Silver Oak

लसमवरओक

4) Jack fruit

कटहि

Correct Answer Silver Oak

Q211 When a Cuttlefish is described as a Molluscs it is at which level of

classification

जबएककटिकफिकह८एकमह८िसकाकहॳ पमवखणयतककयाजाताहहॴतबयहॳवगीकरणकहॳ ककससतरपहॳनसितहहॴ 2-Feb-2017

Options

1) Class

वगय 2) Order

िम

3) Family

पररवार

4) Phylum

सघ

Correct Answer Phylum

Q212 Bambusa dendrocalmus is the

scientific name of बानबसािहॳडराकामस mdashmdashmdash कावहॴजञाननकनामहहॴ 3-Feb-2017

Options

1) Banyan

बरगद

2) Papaya

पपीता 3) Bamboo

बास

4) Pomegranate

अनार

Correct Answer Bamboo

Q213 Acinonyx Jubatus is the scientific name of

एलसनह८ननकसजयबहॳटस mdashmdashmdash

कावहॴजञाननकनामहहॴ 3-Feb-2017

F A C E B O O K

P A G E h t t p w w w f a c e b o o k c o m s s c m e n t o r s o f f i c i a l P a g e | 49

FOR MORE UPDATES AND MORE MATERIAL DO LIKE OUR FACEBOOK PAGE httpwwwfacebookcomsscmentorsofficial

Options

1) Bear

भाि 2) Horse

घह८िा 3) Cheetah

चीता 4) Zebra

जहॳिा Correct Answer Cheetah

Q214 The pale yellow colour of urine is

due to the presence of which pigment

मतरकाफीकापीिारगरगदरयकहॳ उपनसिनतकहॳ कारणहह८ताहहॴ

3-Feb-2017

Options

1) Urochrome

यरह८िह८म

2) Urophyll

यरह८कफि

3) Chlorophyll

किह८रह८कफि

4) Chloroplast

किह८रह८पिासट

Correct Answer Urochrome

Q215 Which of the following constitute

to form a gene

नननननलिखितमसहॳकह९नसीचीज़एकजीनकागठनकरतीहहॴ

3-Feb-2017

Options

1) Polynucleotides

पह८िीनयनकियह८टाईडस

2) Hydrocarbons

हाइडरह८काबोस

3) Lipoproteins

िाईपह८परह८टीनस

4) Lipids

लिपपडस

Correct Answer Polynucleotides

Polynucleotide molecule is a biopolymer

composed of 13 or more nucleotide

monomers covalently bonded in a chain

DNA (deoxyribonucleic acid) and RNA

(ribonucleic acid) are examples of

polynucleotides with distinct biological

function

Q216 Vertebrates belongs to the

phylum

रीढ़कीहडिीवािहॳपराणी mdashmdashmdash

परजानतकहॳ अतगायतआतहॳहहॴ 3-Feb-2017

Options

1) Arthropoda

आरह८पह८ड़ा 2) Annelida

एननलििा 3) Cnidaria

ननिहॳररया 4) Chordata

कह८िटा Correct Answer Chordata

Q217 Punica granatum is the scientific name of

पननकगरहॳनहॳटस mdashmdashmdash कावहॴजञाननकनामहहॴ 3-Feb-2017

Options

1) Custard Apple

सीताफि

2) Gulmohar

गिमह८हर

3) Silver Oak

लसमवरओक

4) Pomegranate

अनार

Correct Answer Pomegranate

F A C E B O O K

P A G E h t t p w w w f a c e b o o k c o m s s c m e n t o r s o f f i c i a l P a g e | 50

FOR MORE UPDATES AND MORE MATERIAL DO LIKE OUR FACEBOOK PAGE httpwwwfacebookcomsscmentorsofficial

Q218 Between a tiger and an monkey

which of the following is different

एकबाघऔरबदरकहॳ बीचनननननलिखितमसहॳकह९नसीबातअिगहहॴ 3-Feb-2017

Options

1) Kingdom

राजय

2) Phylum

जानत

3) Order

िम

4) Class

वगय Correct Answer order

Q219 The artificial heart was invented by

कबतरमहदयका mdashmdashmdash

दवाराअपवषकारककयागयािा 3-Feb-2017

Options

1) Muhammad Yunus

महनमदयनस

2) Linus Yale Jr

िाइनसयहॳिजय

3) Gazi Yasargil

गाजीयासचगयि

4) Paul Winchell

पह९िपवमकि Correct Answer Paul Winchell

Q220 Tamarindus indica is the

scientific name of

टहॳमररनडसइडिका mdashmdash कावहॴजञाननकनामहहॴ 7-

Feb-2017

Options

1) Neem

नीम

2) Pineapple

अनानास

3) Tamarind

इमिी 4)Chiku

चीक

Correct Answer Tamarind

Q221 In eukaryotic cells synthesis of

RNA takes place in the

यकहॳ योटटककह८लिकाओमआरएनएकासशिहॳषण

mdashndash महह८ताहहॴ 7-Feb-2017

Options

1) Mitochondria

माईटह८कोडडरया 2) Centrioles

सटरीयह८मस

3) Ribosomes

ररबह८सह८नस

4) Nucleus

नयनकियस

Correct Answer nucleus

eukaryotic cell -Transcription is the

process of synthesizing ribonucleic acid

(RNA)Synthesis takes place within the

nucleus of eukaryotic cells or in the

cytoplasm of prokaryotes and converts

the genetic code from a gene in

deoxyribonucleic acid ( DNA ) to a

strand of RNA that then directs

proteinsynthesis

Q222 _________is caused by parasites

of the Plasmodium genus

पिाजमह८डियमजातीकहॳ परजीवी mdash- कहॳ कारणहहॴ 7-Feb-2017

Options

1) Dysentery

पहॳचचि

2) Malaria

मिहॳररया 3) Chickenpox

F A C E B O O K

P A G E h t t p w w w f a c e b o o k c o m s s c m e n t o r s o f f i c i a l P a g e | 51

FOR MORE UPDATES AND MORE MATERIAL DO LIKE OUR FACEBOOK PAGE httpwwwfacebookcomsscmentorsofficial

चहॳचक

4) Herpes

हहॳपपयस

Correct Answer Malaria

Q223 Carotene in fruits and vegetables

gives it which color

फिह८औरसनलजयोमनसितकहॳ रह८टीनउनहकह९नसारगपरदानकरताहहॴ 7-Feb-2017

Options

1) Green

हरा 2) Pink

गिाबी 3) Orange

नारगी 4) Blue

नीिा Correct Answer Orange

Q224 Equus Caballus is the scientific

name of

एकवसकहॴ बहॳिस mdashmdashndash कापवजञाननकनामहहॴ 7-Feb-2017

Options

1) Horse

घह८िा 2) Zebra

ज़हॳिा 3) Donkey

गधा 4) Buffalo

भस

Correct Answer Horse

Q225 Elapidae Naja is the scientific name of

एिीपीिीनाजा mdashmdash- कावहॴजञाननकनामहहॴ 8-Feb-2017

Options

1) Cobra

कह८बरा 2) Elephant

हािी 3) Eagle

ग ि

4) Owl

उमि Correct Answer Cobra

Q226 Which disease is caused due to

deficiency of Iron

िह८हकीकमीकहॳ कारणकह९नसारह८गहह८ताहहॴ 8-Feb-

2017

Options

1) Beriberi

बहॳरीबहॳरी 2) Tetany

टहॳटनी 3) Kwashiorkor

कवािीऔरकर

4) Anaemia

रकतामपता Correct Answer Anaemia

Beriberi is a disease caused by a vitamin

B-1 deficiency also known as thiamine

deficiency

Tetany can be the result of an

electrolyte imbalance Most often itrsquos a

dramatically low calcium level also

known as hypocalcemia Tetany can also

be caused by magnesium deficiency or

too little potassium Having too much

acid (acidosis) or too much alkali

(alkalosis) in the body can also result in

tetany

Kwashiorkor also known as

ldquoedematous malnutrition It is a form of

malnutrition caused by a lack of protein

in the diet

Anaemia means that you have fewer red

blood cells than normal or you have less

F A C E B O O K

P A G E h t t p w w w f a c e b o o k c o m s s c m e n t o r s o f f i c i a l P a g e | 52

FOR MORE UPDATES AND MORE MATERIAL DO LIKE OUR FACEBOOK PAGE httpwwwfacebookcomsscmentorsofficial

haemoglobin than normal in each red

blood cell

Q227 is a leaf where the leaflets are

arranged along the middle vein

mdashndashएकपततीहहॴजहापतरकह८कीरचनाक ररयालिराकहॳ आसपासहह८तीहहॴ 8-Feb-2017

Options

1) Pinnately compound leaf

पपनहॳटिीसयकतपतती 2) Palmately compound leaf

पामहॳटिीसयकतपतती 3) Compound leaf

सयकतपतती 4) Simple leaf

साधारणपतती Correct Answer Pinnately compound

leaf

Q228 Haustoria or sucking roots are

found in which of the following

हह८सटह८ररयायाचसनहॳवािीजड़हॳनननननलिखितमसहॳककसमपाईजातीहहॴ 8-Feb-2017

Options

1) Wheat

गहॳह

2) Mango

आम

3) Chestnut

चहॳसटनट

4) Cuscuta

कसकयटा Correct Answer Cuscuta

Haustorial roots -The roots of parasitic

plants which penetrate into the host

tissues to absorb nourishment are

called haustorial roots hellip Also known as suckingor parasitic roots

Q229 Equs Asinus is the scientific name

of

एकवसएलसनस mdashmdashndash कावहॴजञाननकनामहहॴ 8-

Feb-2017

Options

1) Donkey

गधा 2) Cow

गाय

3) Deer

टहरन

4) Kangaroo

कगा

Correct Answer Donkey

Q230 Ficus benghalensis is the scientific name of

फाईकसबहॳनगहॳिहॳलसस mdashndash कापवजञाननकनामहहॴ 8-Feb-2017

Options

1) Banyan

बरगद

2) Pineapple

अनानास

3) Babul

बबि

4) Tulsi

तिसी Correct Answer Banyan

Q231 Equus burchellii is the scientific name of

एकवसबचिी mdashmdash- कापवजञाननकनामहहॴ 8-Feb-2017

Options

1) Horse

घह८िा 2) Zebra

जहॳिा 3) Buffalo

F A C E B O O K

P A G E h t t p w w w f a c e b o o k c o m s s c m e n t o r s o f f i c i a l P a g e | 53

FOR MORE UPDATES AND MORE MATERIAL DO LIKE OUR FACEBOOK PAGE httpwwwfacebookcomsscmentorsofficial

भस

4) Ass

गधा Correct Answer Zebra

Page 42: COMPILATION OF ALL 72 SETS OF BIOLOGY SSC CHSL-2016 · OF BIOLOGY SSC CHSL-2016 PREPARED BY : SSC MENTORS BIOLOGY SPECIAL . F A C E B O O K P A G E : h t t p : / / w w w . f a c e

F A C E B O O K

P A G E h t t p w w w f a c e b o o k c o m s s c m e n t o r s o f f i c i a l P a g e | 41

FOR MORE UPDATES AND MORE MATERIAL DO LIKE OUR FACEBOOK PAGE httpwwwfacebookcomsscmentorsofficial

टाइफाइि

3) Asthma

दमा 4)Diarrhoea

दसत

Correct Answer Asthma

Q176 Ocimum tenuiflorum is the

scientific name of

ओलिलममटहॳयईफिह८रमइसकावहॴजञाननकनाम mdash

ndash हहॴ 30-Jan-2017

Options

1) Neem

नीम

2) Mango

आम

3) Babul

बबि

4)Tulsi

तिसी Correct Answer Tulsi

Q177 Which gland secretes bile a

digestive fluid

कह९नसीगरिीपपतत एकपाचनतरिपरदािय सरापवतकरतीहहॴ 30-Jan-2017

Options

1) Pancreas

अगनयािय

2) Liver

यकत

3) Thyroid

िायराइि

4) Testes

टहॳनसटस

Correct Answer liver

Q178 In which of the following the

dominant phase is Gametophyte

नननननलिखितमसहॳककसकहॳ परमिचरणयगमकह८दपवधद (Gametophyte)हहॴ 30-Jan-2017

Options

1) Bryophyta

िायह८फाइटा 2) Pteridophyta

टहॳररिह८फाइटा 3) Gymnosperms

नजननह८सपमय 4) Angiosperms

एननजयह८सपमय Correct Answer Bryophyta

Q179 Anaerobic respiration refers to

which of the following

नननननलिखितमसहॳककसहॳअवायवीयशवसनकहाजाताहहॴ

30-Jan-2017

Options

1) Respiration without Oxygen

ऑकसीजनकहॳ बबनाशवसन

2) Respiration with Oxygen

ऑकसीजनकहॳ सािशवसन

3) Respiration without CO2

काबयनिायऑकसाइिकहॳ बबनाशवसन

4) Respiration with CO2

काबयनिायऑकसाइिकहॳ सािशविन

Correct Answer Respiration without

Oxygen

Q180 Which type of pathogen causes

the waterborne disease Cholera

ककसपरकारकारह८गजनकजिजननतरह८गहहॴजाकाकारणबनताहहॴ

30-Jan-2017

Options

1) Algal

िहॴवालियहॳ

F A C E B O O K

P A G E h t t p w w w f a c e b o o k c o m s s c m e n t o r s o f f i c i a l P a g e | 42

FOR MORE UPDATES AND MORE MATERIAL DO LIKE OUR FACEBOOK PAGE httpwwwfacebookcomsscmentorsofficial

2) Bacterial

बहॴकटीररयि

3) Protozoan

परह८टह८जआ

4) Viral

वायरि

Correct Answer Bacterial

Q181 To which class does

Oxyreductases transferases hydrolases

belong

ओकसीररिकटहॳसटरासफरहॳजहॳस

हाइडरह८िहॳसहॳसककसवगयमआतहॳहहॴ 30-Jan-2017

Options

1) Hormones

हारमोस

2) Enzymes

एजाइनस

3) Proteins

परह८टीनस

4) Vitamins

पवटालमनस

Correct Answer Enzymes

Q182 Which of the following is not true

about Gymnosperms

ननननमसहॳकह९नसीबातअनावतबीजीकहॳ बारहॳमसचनहीहहॴ 30-Jan-2017

Options

1) Dominant phase is saprophytes

परमिचरणसहॳपरह८फाइटसहह८ताहहॴ 2) Vascular bundles are absent

सवहनीबििअनपनसितहह८ताहहॴ 3) spores are heterospores

बीजाणहहॳटहॳरह८सपह८रसहह८तहॳहहॴ 4) Flowers are absent

फिअनपनसितहह८तहॳहहॴ

Correct Answer Vascular bundles are

absent

Q183 The name of first mammal clone sheep is

भहॳड़कीपरिमसतनपायीपरनत प (किह८न)

कानामहहॴ 30-Jan-2017

Options

1) Noori

नरी 2) Dolly

िॉिी 3) Louise

िसी 4)Durga

दगाय Correct Answer Dolly

Q184 Which type of pathogen causes

the water-borne disease Typhoid fever

ककसपरकारकारह८गजनकजिजननतरह८गटाइफाइिबिारकाकारणबनताहहॴ 30-Jan-2017

Options

1) Algal

िहॴवािीय

2) Parasitic

परजीवी 3) Protozoan

परह८टह८जनअन

4)Bacterial

बहॴकटीररयि

Correct Answer Bacterial

Q185 In which part of the cell are

proteins made

कह८लिकाकहॳ ककसटहससहॳमपरह८टीनबनायाजाताहहॴ

31-Jan-2017

Options

1) Reticulum

रहॳटटकिम

F A C E B O O K

P A G E h t t p w w w f a c e b o o k c o m s s c m e n t o r s o f f i c i a l P a g e | 43

FOR MORE UPDATES AND MORE MATERIAL DO LIKE OUR FACEBOOK PAGE httpwwwfacebookcomsscmentorsofficial

2) Golgi apparatus

गह८मजीएपहॳरहॳटस

3) Ribosomes

ररबह८सह८नस

4) Lysosome

िायसह८सह८नस

Correct Answer ribosomes

Proteins are produced by stringing

amino acids together in the order

specified by messenger RNA strands

that were transcribed from DNA in the

cell nucleus The process of synthesizing

a protein is called translation and it

occurs on ribosomes in the cytoplasm of

a cell

Q186 Polio is a disease caused by which

of the following

नननननलिखितमसहॳपह८लियह८कीबबमारह८हह८नहॳकाकारणकयाहहॴ

31-Jan-2017

Options

1) Bacteria

बहॴकटीररयि

2) Mosquito

मचछर

3) Virus

वायरस

4) Cockroach

नतिच हॳ Correct Answer Virus

Polio or poliomyelitis is a crippling and

potentially deadly infectious disease It

is caused by the poliovirus

Q187 ndash Hay fever is a sign of which of

the following

हहॳकफवरनननननलिखितमसहॳककसकाएकसकहॳ तहहॴ

31-Jan-2017

Options

1) Old Age

वदावसिा 2) Malnutrition

कपह८सण

3) Allergy

एिनजय 4) Over Work

अतयचधककाययकरना Correct Answer Allergy

Q188 How many chromosomes does a

human cell contain

एकमानवकह८लिकामककतनहॳगणसतरहह८तहॳहहॴ

29-Jan-2017

Options

1) 6

2) 26

3) 46

4) 66

Correct Answer 46

In humans each cell normally contains

23 pairs of chromosomes for a total of

46 Twenty-two of these pairs called

autosomes look the same in both males

and females The 23rd pair the sex

chromosomes differ between males and

females

Q189 Which of the following is not true

about Bryophyta

ननननमसहॳकह९नसीबातिायह८फाइटकहॳ बारहॳमसचनहीहहॴ 31-Jan-2017

Options

1) Dominant phase is gametophytes

परमिचरणगहॳलमतह८फाइटसहह८ताहहॴ 2) Main plant body is haploid

पह९धहॳकामखयिरीरअगखणतहह८ताहहॴ 3) Spores are homospores

बीजाणहह८मह८सफह८रसहह८तहॳहहॴ 4) Flowers are present

फिमह८जदहह८तहॳहहॴ Correct Answer Flowers are present

F A C E B O O K

P A G E h t t p w w w f a c e b o o k c o m s s c m e n t o r s o f f i c i a l P a g e | 44

FOR MORE UPDATES AND MORE MATERIAL DO LIKE OUR FACEBOOK PAGE httpwwwfacebookcomsscmentorsofficial

Q190 Which aquatic animal has

trailing tentacles

ककसजिीयजानवरकहॳ पीछहॳचिनहॳवािहॳटहॳटकिसहह८तहॳहहॴ

31-Jan-2017

Options

1) Sea horse

समदरीघह८िा 2) Corals

मगा 3) Jelly fish

जहॳिीमछिी 4) Star fish

तारामछिी Correct Answer Jelly fish

Jellyfish with its umbrella-shaped bell

and trailing tentacles

Q191 Which type of pathogen causes

the water-borne disease Poliomyelitis

(Polio)

ककसपरकारकारह८गजनकजिजननतरह८गपह८लियह८मायहॳटटस (पह८लियह८) काकारणहहॴ 31-Jan-

2017

Options

1) Parasitic

परजीवी 2) Algal

िहॴवालिय

3) Viral

वायरि

4) Bacterial

बहॴकटीररयि

Correct Answer Viral

Q192 The outer white part of the eye

that protects the inner structures is

आािकाबाहरीसफहॳ दटहससाजह८आतररकसरचनाओकीरकषाकरताहहॴ वह mdashmdashmdash हहॴ 31-Jan-

2017

Options

1) Iris

आयररस

2) Sclera

सकिहॳरा 3) Retina

रहॳटटना 4) Cornea

कह८ननयया Correct Answer Sclera

Q193 Proteins are made up of

परह८टीनकाननमायण mdashndash सहॳहह८ताहहॴ 31-Jan-2017

Options

1) Amino acids

एलमनह८अनि

2) Fatty acids

वसायकतअनि

3) Glucose

गिकह८ज

4)Nucleotides

नयनकियह८टाईिस

Correct Answer Amino acids

Q194 Moringa Oleifera is the scientific

name of

मह८ररगओलिफहॳ रा mdashmdashndash कावहॴजञाननकनामहहॴ 31-Jan-2017

Options

1) Banyan

बरगद

2) Gulmohar

गिमह८हर

3) Amla

आमिा

F A C E B O O K

P A G E h t t p w w w f a c e b o o k c o m s s c m e n t o r s o f f i c i a l P a g e | 45

FOR MORE UPDATES AND MORE MATERIAL DO LIKE OUR FACEBOOK PAGE httpwwwfacebookcomsscmentorsofficial

4) Drumstick

डरमनसटक

Correct Answer Drumstick

Q195 Kidney stones are composed of

गदकीपिरी mdashndash सहॳबनीहह८तीहहॴ 1-Feb-2017

Options

1) Calcium Oxalate

कहॴ नमसयमओकजहॳिहॳट

2) Sodium Chloride

सह८डियमकिह८राइि

3) Magnesium Nitrate

महॳनगनलियमनाइतटरहॳट

4) Calcium Bicarbonate

कहॴ नमियमबायकबोनहॳट

Correct Answer Calcium Oxalate

Q196 ndash Which of the following is not

true about Angiosperms

ननननमसहॳकह९नसीबातआवतबीजीकहॳ बारहॳमसचनहीहहॴ 1-Feb-2017

Options

1) Dominant phase is gametophytes

परमिचरणगहॳलमतह८फाइटहह८ताहहॴ 2) Vascular bundles are present

सवहनीबििमह९जदहह८ताहहॴ 3) Spores are heterospores

बीजाणहहॳटहॳरह८सपह८रसहह८तहॳहहॴ 4) Seeds are covered

बीजढकहॳ हह८तहॳहहॴ Correct Answer Dominant phase is

gametophytes

Q197 All of the following are excretory

(waste) products of animals except

नननननलिखितमसहॳककसएककह८छह८ड़करअनयसभीपराखणयोदवाराउतसनजयतपदाियहहॴ 1-Feb-

2017

Options

1) Uric Acid

यररकएलसि

2) Ammonia

अमह८ननया 3) Carbohydrates

काबोहाइडरहॳट

4) Urea

यररया Correct Answer Carbohydrates

In animals the main excretory products

are carbon dioxide ammonia (in

ammoniotelics) urea (in ureotelics) uric

acid (in uricotelics) guanine (in

Arachnida) and creatine

Q198 RNA is a polymeric molecule

What does RNA stand for

आरएनइएएकबहिकआणहहॴ इसकाकापवय पकयाहहॴ 1-Feb-2017

Options

1) Rado Nuclear Acid

रािह८नयनकियरएलसि

2) Ribo Nucleic Acid

राइबह८नयनकिकएलसि

3) Rhino Nuclear Acid

हाइनह८नयनकियरएलसि

4) Resto Nucleus Acid

रहॳसटह८नयकिीयसएलसि

Correct Answer Ribo Nucleic Acid

Q199 Which organ does detoxification

and produces chemicals needed for

digestion

कह९नसाअगपवषहरणकरताहहॴऔरपाचनकहॳ लिएआवशयकरसायनोकह८पहॴदाकरताहहॴ 1-Feb-

2017

Options

1) Salivary glands

िारगरचिया 2) Pancreas

अगनयािय

F A C E B O O K

P A G E h t t p w w w f a c e b o o k c o m s s c m e n t o r s o f f i c i a l P a g e | 46

FOR MORE UPDATES AND MORE MATERIAL DO LIKE OUR FACEBOOK PAGE httpwwwfacebookcomsscmentorsofficial

3) Thyroid gland

िायराइिगरिी 4) Liver

यकत

Correct Answer Liver

Q200 Psidium guajava is the scientific

name of

लसडियमगआजावा mdashmdash कावहॴजञाननकनामहहॴ 1-

Feb-2017

Options

1) Guava

अम द

2) Mango

आम

3) Bamboo

बास

4) Jack fruit

कटहि

Correct Answer Guava

Q201 Which drug is used as a Blood

Thinner

चधरकह८पतिाकरनहॳकहॳ पमककसदवाकापरयह८गककयाजाताहहॴ

1-Feb-2017

Options

1) Warfarin

वाफर न

2) Tramadol

टरहॳमािह८ि

3) Azithromycin

एनजरह८मायलसन

4) Hydralazine

हाइडरह८िहॳनजन

Correct Answer Warfarin

Q202 Which of the following disease is

caused due to the deficiency of protein

परह८टीनकीकमीकहॳ कारणनननननलिखितमसहॳकह९नसारह८गहह८ताहहॴ 1-Feb-2017

Options

1) Arthritis

गटठया 2) Kwashiorkor

कािीओकय र

3) Goitre

गाइटर

4) Night Blindness

रतह९चध

Correct Answer Kwashiorkor

Q203 A is species of plant that has

adapted to survive in an environment

with little liquid water

mdashmdashndashपह९धहॳकीएकऐसहॳऐसहॳपरजानतहहॴ नजसनहॳकमपानीवािहॳवातावरणमजीपवतरहनहॳकहॳलिएअनकिनहहॴ 1-Feb-2017

Options

1) Xerophyte

म दपवद

2) Hydrophyte

जिीयपादप

3) Mesophyte

समह८दपवद

4) Thallophyte

िहॴिह८फाइटा Correct Answer xerophyte

xerophyte is a species of plant that has

adapted to survive in an environment

with little liquid water such as a desert

or an ice- or snow-covered region in the

Alps or the Arctic

Mesophytes are terrestrial plants which

are adapted to neither a particularly

dry nor particularly wet environment

An example of a mesophytic habitat

would be a rural temperate meadow

F A C E B O O K

P A G E h t t p w w w f a c e b o o k c o m s s c m e n t o r s o f f i c i a l P a g e | 47

FOR MORE UPDATES AND MORE MATERIAL DO LIKE OUR FACEBOOK PAGE httpwwwfacebookcomsscmentorsofficial

which might contain goldenrod clover

oxeye daisy and Rosa multiflora

thallophyte any of a group of plants or

plantlike organisms (such as algae and

fungi) that lack differentiated stems

leaves and roots and that were formerly

classified as a primary division

(Thallophyta) of the plant kingdom

Q204 How many types of teeth are

there in humans

मनषयोमककतनहॳपरकारकहॳ दातहह८तहॳहहॴ

1-Feb-2017

Options

1) 4

2) 5

3) 2

4) 3

Correct Answer 4

teeth -Humans have four types of

teethincisors canines premolars and

molars each with a specific function

The incisors cut the food the canines

tear the food and the molars and

premolars crush the food

Q205 Carica papaya is the scientific name of

कहॴ ररकापपाया mdashmdashndash कावहॴजञाननकनामहहॴ 2-

Feb-2017

Options

1) Peepal

पीपि

2) Papaya

पपीता 3) Tamarind

इमिी 4) Drumstick

ढह८िकाछड़ी Correct Answer Papaya

Q206 Muscles get tired when there is

shortfall of

जब mdashndash कीकमीहह८तीहहॴतबपहॳिीयिकजातीहहॴ 2-Feb-2017

Options

1) Lactic acid

िहॴनकटकएलसि

2) Na+ ions

Na+ आयन

3) ATP

एटीपी 4) Sulphates

समफहॳ टस

Correct Answer ATP

ATP is the energy source muscle fibers

use to make muscles contract

muscle tissuersquos main source of energy

called adenosine triphosphate or ATP

As your muscles use up this energy

source they become tired and fatigued

Oxygen is the key ingredient that helps

create new ATP to replenish the burned

up ATP in your muscles

Q207 Artocarpus integra is the

scientific name of आटह८कापयसइटीगरा mdashmdashmdash कावहॴजञाननकनामहहॴ 2-Feb-2017

Options

1) Guava

अम द

2) Pineapple

अनानास

3) Silver Oak

लसमवरओक

4) Jack fruit

कटहि

Correct Answer Jack fruit

Q208 Which organ stores fat soluble

vitamins

कह९नसाअगवसामघिनिीिपवटालमनह८काभिाराकरताहहॴ

2-Feb-2017

F A C E B O O K

P A G E h t t p w w w f a c e b o o k c o m s s c m e n t o r s o f f i c i a l P a g e | 48

FOR MORE UPDATES AND MORE MATERIAL DO LIKE OUR FACEBOOK PAGE httpwwwfacebookcomsscmentorsofficial

Options

1) Blood

रकत

2) Skin

तवचा 3) Liver

यकत

4) Pancreas

अगनयािय

Correct Answer Liver

Q209 Which disease is caused due to

deficiency of Iodine

आयह८िीनकहॳ कारणकह९नसारह८गहह८ताहहॴ 2-Feb-2017

Options

1) Rickets

ररकहॳ टस

2) Scurvy

सकवी 3) Goitre

गणमािा 4) Growth retardation

पवकासका कना Correct Answer Goitre

rickets A softening and weakening of

bones in children usually due to

inadequate vitamin D

Q210 Grevillea Robusta is the scientific name of

गरहॳपवलियारह८बसटा mdashmdashmdash- कापवजञाननकनामहहॴ 2-Feb-2017

Options

1) Peepal

पीपि

2) Teak

सागह९न

3) Silver Oak

लसमवरओक

4) Jack fruit

कटहि

Correct Answer Silver Oak

Q211 When a Cuttlefish is described as a Molluscs it is at which level of

classification

जबएककटिकफिकह८एकमह८िसकाकहॳ पमवखणयतककयाजाताहहॴतबयहॳवगीकरणकहॳ ककससतरपहॳनसितहहॴ 2-Feb-2017

Options

1) Class

वगय 2) Order

िम

3) Family

पररवार

4) Phylum

सघ

Correct Answer Phylum

Q212 Bambusa dendrocalmus is the

scientific name of बानबसािहॳडराकामस mdashmdashmdash कावहॴजञाननकनामहहॴ 3-Feb-2017

Options

1) Banyan

बरगद

2) Papaya

पपीता 3) Bamboo

बास

4) Pomegranate

अनार

Correct Answer Bamboo

Q213 Acinonyx Jubatus is the scientific name of

एलसनह८ननकसजयबहॳटस mdashmdashmdash

कावहॴजञाननकनामहहॴ 3-Feb-2017

F A C E B O O K

P A G E h t t p w w w f a c e b o o k c o m s s c m e n t o r s o f f i c i a l P a g e | 49

FOR MORE UPDATES AND MORE MATERIAL DO LIKE OUR FACEBOOK PAGE httpwwwfacebookcomsscmentorsofficial

Options

1) Bear

भाि 2) Horse

घह८िा 3) Cheetah

चीता 4) Zebra

जहॳिा Correct Answer Cheetah

Q214 The pale yellow colour of urine is

due to the presence of which pigment

मतरकाफीकापीिारगरगदरयकहॳ उपनसिनतकहॳ कारणहह८ताहहॴ

3-Feb-2017

Options

1) Urochrome

यरह८िह८म

2) Urophyll

यरह८कफि

3) Chlorophyll

किह८रह८कफि

4) Chloroplast

किह८रह८पिासट

Correct Answer Urochrome

Q215 Which of the following constitute

to form a gene

नननननलिखितमसहॳकह९नसीचीज़एकजीनकागठनकरतीहहॴ

3-Feb-2017

Options

1) Polynucleotides

पह८िीनयनकियह८टाईडस

2) Hydrocarbons

हाइडरह८काबोस

3) Lipoproteins

िाईपह८परह८टीनस

4) Lipids

लिपपडस

Correct Answer Polynucleotides

Polynucleotide molecule is a biopolymer

composed of 13 or more nucleotide

monomers covalently bonded in a chain

DNA (deoxyribonucleic acid) and RNA

(ribonucleic acid) are examples of

polynucleotides with distinct biological

function

Q216 Vertebrates belongs to the

phylum

रीढ़कीहडिीवािहॳपराणी mdashmdashmdash

परजानतकहॳ अतगायतआतहॳहहॴ 3-Feb-2017

Options

1) Arthropoda

आरह८पह८ड़ा 2) Annelida

एननलििा 3) Cnidaria

ननिहॳररया 4) Chordata

कह८िटा Correct Answer Chordata

Q217 Punica granatum is the scientific name of

पननकगरहॳनहॳटस mdashmdashmdash कावहॴजञाननकनामहहॴ 3-Feb-2017

Options

1) Custard Apple

सीताफि

2) Gulmohar

गिमह८हर

3) Silver Oak

लसमवरओक

4) Pomegranate

अनार

Correct Answer Pomegranate

F A C E B O O K

P A G E h t t p w w w f a c e b o o k c o m s s c m e n t o r s o f f i c i a l P a g e | 50

FOR MORE UPDATES AND MORE MATERIAL DO LIKE OUR FACEBOOK PAGE httpwwwfacebookcomsscmentorsofficial

Q218 Between a tiger and an monkey

which of the following is different

एकबाघऔरबदरकहॳ बीचनननननलिखितमसहॳकह९नसीबातअिगहहॴ 3-Feb-2017

Options

1) Kingdom

राजय

2) Phylum

जानत

3) Order

िम

4) Class

वगय Correct Answer order

Q219 The artificial heart was invented by

कबतरमहदयका mdashmdashmdash

दवाराअपवषकारककयागयािा 3-Feb-2017

Options

1) Muhammad Yunus

महनमदयनस

2) Linus Yale Jr

िाइनसयहॳिजय

3) Gazi Yasargil

गाजीयासचगयि

4) Paul Winchell

पह९िपवमकि Correct Answer Paul Winchell

Q220 Tamarindus indica is the

scientific name of

टहॳमररनडसइडिका mdashmdash कावहॴजञाननकनामहहॴ 7-

Feb-2017

Options

1) Neem

नीम

2) Pineapple

अनानास

3) Tamarind

इमिी 4)Chiku

चीक

Correct Answer Tamarind

Q221 In eukaryotic cells synthesis of

RNA takes place in the

यकहॳ योटटककह८लिकाओमआरएनएकासशिहॳषण

mdashndash महह८ताहहॴ 7-Feb-2017

Options

1) Mitochondria

माईटह८कोडडरया 2) Centrioles

सटरीयह८मस

3) Ribosomes

ररबह८सह८नस

4) Nucleus

नयनकियस

Correct Answer nucleus

eukaryotic cell -Transcription is the

process of synthesizing ribonucleic acid

(RNA)Synthesis takes place within the

nucleus of eukaryotic cells or in the

cytoplasm of prokaryotes and converts

the genetic code from a gene in

deoxyribonucleic acid ( DNA ) to a

strand of RNA that then directs

proteinsynthesis

Q222 _________is caused by parasites

of the Plasmodium genus

पिाजमह८डियमजातीकहॳ परजीवी mdash- कहॳ कारणहहॴ 7-Feb-2017

Options

1) Dysentery

पहॳचचि

2) Malaria

मिहॳररया 3) Chickenpox

F A C E B O O K

P A G E h t t p w w w f a c e b o o k c o m s s c m e n t o r s o f f i c i a l P a g e | 51

FOR MORE UPDATES AND MORE MATERIAL DO LIKE OUR FACEBOOK PAGE httpwwwfacebookcomsscmentorsofficial

चहॳचक

4) Herpes

हहॳपपयस

Correct Answer Malaria

Q223 Carotene in fruits and vegetables

gives it which color

फिह८औरसनलजयोमनसितकहॳ रह८टीनउनहकह९नसारगपरदानकरताहहॴ 7-Feb-2017

Options

1) Green

हरा 2) Pink

गिाबी 3) Orange

नारगी 4) Blue

नीिा Correct Answer Orange

Q224 Equus Caballus is the scientific

name of

एकवसकहॴ बहॳिस mdashmdashndash कापवजञाननकनामहहॴ 7-Feb-2017

Options

1) Horse

घह८िा 2) Zebra

ज़हॳिा 3) Donkey

गधा 4) Buffalo

भस

Correct Answer Horse

Q225 Elapidae Naja is the scientific name of

एिीपीिीनाजा mdashmdash- कावहॴजञाननकनामहहॴ 8-Feb-2017

Options

1) Cobra

कह८बरा 2) Elephant

हािी 3) Eagle

ग ि

4) Owl

उमि Correct Answer Cobra

Q226 Which disease is caused due to

deficiency of Iron

िह८हकीकमीकहॳ कारणकह९नसारह८गहह८ताहहॴ 8-Feb-

2017

Options

1) Beriberi

बहॳरीबहॳरी 2) Tetany

टहॳटनी 3) Kwashiorkor

कवािीऔरकर

4) Anaemia

रकतामपता Correct Answer Anaemia

Beriberi is a disease caused by a vitamin

B-1 deficiency also known as thiamine

deficiency

Tetany can be the result of an

electrolyte imbalance Most often itrsquos a

dramatically low calcium level also

known as hypocalcemia Tetany can also

be caused by magnesium deficiency or

too little potassium Having too much

acid (acidosis) or too much alkali

(alkalosis) in the body can also result in

tetany

Kwashiorkor also known as

ldquoedematous malnutrition It is a form of

malnutrition caused by a lack of protein

in the diet

Anaemia means that you have fewer red

blood cells than normal or you have less

F A C E B O O K

P A G E h t t p w w w f a c e b o o k c o m s s c m e n t o r s o f f i c i a l P a g e | 52

FOR MORE UPDATES AND MORE MATERIAL DO LIKE OUR FACEBOOK PAGE httpwwwfacebookcomsscmentorsofficial

haemoglobin than normal in each red

blood cell

Q227 is a leaf where the leaflets are

arranged along the middle vein

mdashndashएकपततीहहॴजहापतरकह८कीरचनाक ररयालिराकहॳ आसपासहह८तीहहॴ 8-Feb-2017

Options

1) Pinnately compound leaf

पपनहॳटिीसयकतपतती 2) Palmately compound leaf

पामहॳटिीसयकतपतती 3) Compound leaf

सयकतपतती 4) Simple leaf

साधारणपतती Correct Answer Pinnately compound

leaf

Q228 Haustoria or sucking roots are

found in which of the following

हह८सटह८ररयायाचसनहॳवािीजड़हॳनननननलिखितमसहॳककसमपाईजातीहहॴ 8-Feb-2017

Options

1) Wheat

गहॳह

2) Mango

आम

3) Chestnut

चहॳसटनट

4) Cuscuta

कसकयटा Correct Answer Cuscuta

Haustorial roots -The roots of parasitic

plants which penetrate into the host

tissues to absorb nourishment are

called haustorial roots hellip Also known as suckingor parasitic roots

Q229 Equs Asinus is the scientific name

of

एकवसएलसनस mdashmdashndash कावहॴजञाननकनामहहॴ 8-

Feb-2017

Options

1) Donkey

गधा 2) Cow

गाय

3) Deer

टहरन

4) Kangaroo

कगा

Correct Answer Donkey

Q230 Ficus benghalensis is the scientific name of

फाईकसबहॳनगहॳिहॳलसस mdashndash कापवजञाननकनामहहॴ 8-Feb-2017

Options

1) Banyan

बरगद

2) Pineapple

अनानास

3) Babul

बबि

4) Tulsi

तिसी Correct Answer Banyan

Q231 Equus burchellii is the scientific name of

एकवसबचिी mdashmdash- कापवजञाननकनामहहॴ 8-Feb-2017

Options

1) Horse

घह८िा 2) Zebra

जहॳिा 3) Buffalo

F A C E B O O K

P A G E h t t p w w w f a c e b o o k c o m s s c m e n t o r s o f f i c i a l P a g e | 53

FOR MORE UPDATES AND MORE MATERIAL DO LIKE OUR FACEBOOK PAGE httpwwwfacebookcomsscmentorsofficial

भस

4) Ass

गधा Correct Answer Zebra

Page 43: COMPILATION OF ALL 72 SETS OF BIOLOGY SSC CHSL-2016 · OF BIOLOGY SSC CHSL-2016 PREPARED BY : SSC MENTORS BIOLOGY SPECIAL . F A C E B O O K P A G E : h t t p : / / w w w . f a c e

F A C E B O O K

P A G E h t t p w w w f a c e b o o k c o m s s c m e n t o r s o f f i c i a l P a g e | 42

FOR MORE UPDATES AND MORE MATERIAL DO LIKE OUR FACEBOOK PAGE httpwwwfacebookcomsscmentorsofficial

2) Bacterial

बहॴकटीररयि

3) Protozoan

परह८टह८जआ

4) Viral

वायरि

Correct Answer Bacterial

Q181 To which class does

Oxyreductases transferases hydrolases

belong

ओकसीररिकटहॳसटरासफरहॳजहॳस

हाइडरह८िहॳसहॳसककसवगयमआतहॳहहॴ 30-Jan-2017

Options

1) Hormones

हारमोस

2) Enzymes

एजाइनस

3) Proteins

परह८टीनस

4) Vitamins

पवटालमनस

Correct Answer Enzymes

Q182 Which of the following is not true

about Gymnosperms

ननननमसहॳकह९नसीबातअनावतबीजीकहॳ बारहॳमसचनहीहहॴ 30-Jan-2017

Options

1) Dominant phase is saprophytes

परमिचरणसहॳपरह८फाइटसहह८ताहहॴ 2) Vascular bundles are absent

सवहनीबििअनपनसितहह८ताहहॴ 3) spores are heterospores

बीजाणहहॳटहॳरह८सपह८रसहह८तहॳहहॴ 4) Flowers are absent

फिअनपनसितहह८तहॳहहॴ

Correct Answer Vascular bundles are

absent

Q183 The name of first mammal clone sheep is

भहॳड़कीपरिमसतनपायीपरनत प (किह८न)

कानामहहॴ 30-Jan-2017

Options

1) Noori

नरी 2) Dolly

िॉिी 3) Louise

िसी 4)Durga

दगाय Correct Answer Dolly

Q184 Which type of pathogen causes

the water-borne disease Typhoid fever

ककसपरकारकारह८गजनकजिजननतरह८गटाइफाइिबिारकाकारणबनताहहॴ 30-Jan-2017

Options

1) Algal

िहॴवािीय

2) Parasitic

परजीवी 3) Protozoan

परह८टह८जनअन

4)Bacterial

बहॴकटीररयि

Correct Answer Bacterial

Q185 In which part of the cell are

proteins made

कह८लिकाकहॳ ककसटहससहॳमपरह८टीनबनायाजाताहहॴ

31-Jan-2017

Options

1) Reticulum

रहॳटटकिम

F A C E B O O K

P A G E h t t p w w w f a c e b o o k c o m s s c m e n t o r s o f f i c i a l P a g e | 43

FOR MORE UPDATES AND MORE MATERIAL DO LIKE OUR FACEBOOK PAGE httpwwwfacebookcomsscmentorsofficial

2) Golgi apparatus

गह८मजीएपहॳरहॳटस

3) Ribosomes

ररबह८सह८नस

4) Lysosome

िायसह८सह८नस

Correct Answer ribosomes

Proteins are produced by stringing

amino acids together in the order

specified by messenger RNA strands

that were transcribed from DNA in the

cell nucleus The process of synthesizing

a protein is called translation and it

occurs on ribosomes in the cytoplasm of

a cell

Q186 Polio is a disease caused by which

of the following

नननननलिखितमसहॳपह८लियह८कीबबमारह८हह८नहॳकाकारणकयाहहॴ

31-Jan-2017

Options

1) Bacteria

बहॴकटीररयि

2) Mosquito

मचछर

3) Virus

वायरस

4) Cockroach

नतिच हॳ Correct Answer Virus

Polio or poliomyelitis is a crippling and

potentially deadly infectious disease It

is caused by the poliovirus

Q187 ndash Hay fever is a sign of which of

the following

हहॳकफवरनननननलिखितमसहॳककसकाएकसकहॳ तहहॴ

31-Jan-2017

Options

1) Old Age

वदावसिा 2) Malnutrition

कपह८सण

3) Allergy

एिनजय 4) Over Work

अतयचधककाययकरना Correct Answer Allergy

Q188 How many chromosomes does a

human cell contain

एकमानवकह८लिकामककतनहॳगणसतरहह८तहॳहहॴ

29-Jan-2017

Options

1) 6

2) 26

3) 46

4) 66

Correct Answer 46

In humans each cell normally contains

23 pairs of chromosomes for a total of

46 Twenty-two of these pairs called

autosomes look the same in both males

and females The 23rd pair the sex

chromosomes differ between males and

females

Q189 Which of the following is not true

about Bryophyta

ननननमसहॳकह९नसीबातिायह८फाइटकहॳ बारहॳमसचनहीहहॴ 31-Jan-2017

Options

1) Dominant phase is gametophytes

परमिचरणगहॳलमतह८फाइटसहह८ताहहॴ 2) Main plant body is haploid

पह९धहॳकामखयिरीरअगखणतहह८ताहहॴ 3) Spores are homospores

बीजाणहह८मह८सफह८रसहह८तहॳहहॴ 4) Flowers are present

फिमह८जदहह८तहॳहहॴ Correct Answer Flowers are present

F A C E B O O K

P A G E h t t p w w w f a c e b o o k c o m s s c m e n t o r s o f f i c i a l P a g e | 44

FOR MORE UPDATES AND MORE MATERIAL DO LIKE OUR FACEBOOK PAGE httpwwwfacebookcomsscmentorsofficial

Q190 Which aquatic animal has

trailing tentacles

ककसजिीयजानवरकहॳ पीछहॳचिनहॳवािहॳटहॳटकिसहह८तहॳहहॴ

31-Jan-2017

Options

1) Sea horse

समदरीघह८िा 2) Corals

मगा 3) Jelly fish

जहॳिीमछिी 4) Star fish

तारामछिी Correct Answer Jelly fish

Jellyfish with its umbrella-shaped bell

and trailing tentacles

Q191 Which type of pathogen causes

the water-borne disease Poliomyelitis

(Polio)

ककसपरकारकारह८गजनकजिजननतरह८गपह८लियह८मायहॳटटस (पह८लियह८) काकारणहहॴ 31-Jan-

2017

Options

1) Parasitic

परजीवी 2) Algal

िहॴवालिय

3) Viral

वायरि

4) Bacterial

बहॴकटीररयि

Correct Answer Viral

Q192 The outer white part of the eye

that protects the inner structures is

आािकाबाहरीसफहॳ दटहससाजह८आतररकसरचनाओकीरकषाकरताहहॴ वह mdashmdashmdash हहॴ 31-Jan-

2017

Options

1) Iris

आयररस

2) Sclera

सकिहॳरा 3) Retina

रहॳटटना 4) Cornea

कह८ननयया Correct Answer Sclera

Q193 Proteins are made up of

परह८टीनकाननमायण mdashndash सहॳहह८ताहहॴ 31-Jan-2017

Options

1) Amino acids

एलमनह८अनि

2) Fatty acids

वसायकतअनि

3) Glucose

गिकह८ज

4)Nucleotides

नयनकियह८टाईिस

Correct Answer Amino acids

Q194 Moringa Oleifera is the scientific

name of

मह८ररगओलिफहॳ रा mdashmdashndash कावहॴजञाननकनामहहॴ 31-Jan-2017

Options

1) Banyan

बरगद

2) Gulmohar

गिमह८हर

3) Amla

आमिा

F A C E B O O K

P A G E h t t p w w w f a c e b o o k c o m s s c m e n t o r s o f f i c i a l P a g e | 45

FOR MORE UPDATES AND MORE MATERIAL DO LIKE OUR FACEBOOK PAGE httpwwwfacebookcomsscmentorsofficial

4) Drumstick

डरमनसटक

Correct Answer Drumstick

Q195 Kidney stones are composed of

गदकीपिरी mdashndash सहॳबनीहह८तीहहॴ 1-Feb-2017

Options

1) Calcium Oxalate

कहॴ नमसयमओकजहॳिहॳट

2) Sodium Chloride

सह८डियमकिह८राइि

3) Magnesium Nitrate

महॳनगनलियमनाइतटरहॳट

4) Calcium Bicarbonate

कहॴ नमियमबायकबोनहॳट

Correct Answer Calcium Oxalate

Q196 ndash Which of the following is not

true about Angiosperms

ननननमसहॳकह९नसीबातआवतबीजीकहॳ बारहॳमसचनहीहहॴ 1-Feb-2017

Options

1) Dominant phase is gametophytes

परमिचरणगहॳलमतह८फाइटहह८ताहहॴ 2) Vascular bundles are present

सवहनीबििमह९जदहह८ताहहॴ 3) Spores are heterospores

बीजाणहहॳटहॳरह८सपह८रसहह८तहॳहहॴ 4) Seeds are covered

बीजढकहॳ हह८तहॳहहॴ Correct Answer Dominant phase is

gametophytes

Q197 All of the following are excretory

(waste) products of animals except

नननननलिखितमसहॳककसएककह८छह८ड़करअनयसभीपराखणयोदवाराउतसनजयतपदाियहहॴ 1-Feb-

2017

Options

1) Uric Acid

यररकएलसि

2) Ammonia

अमह८ननया 3) Carbohydrates

काबोहाइडरहॳट

4) Urea

यररया Correct Answer Carbohydrates

In animals the main excretory products

are carbon dioxide ammonia (in

ammoniotelics) urea (in ureotelics) uric

acid (in uricotelics) guanine (in

Arachnida) and creatine

Q198 RNA is a polymeric molecule

What does RNA stand for

आरएनइएएकबहिकआणहहॴ इसकाकापवय पकयाहहॴ 1-Feb-2017

Options

1) Rado Nuclear Acid

रािह८नयनकियरएलसि

2) Ribo Nucleic Acid

राइबह८नयनकिकएलसि

3) Rhino Nuclear Acid

हाइनह८नयनकियरएलसि

4) Resto Nucleus Acid

रहॳसटह८नयकिीयसएलसि

Correct Answer Ribo Nucleic Acid

Q199 Which organ does detoxification

and produces chemicals needed for

digestion

कह९नसाअगपवषहरणकरताहहॴऔरपाचनकहॳ लिएआवशयकरसायनोकह८पहॴदाकरताहहॴ 1-Feb-

2017

Options

1) Salivary glands

िारगरचिया 2) Pancreas

अगनयािय

F A C E B O O K

P A G E h t t p w w w f a c e b o o k c o m s s c m e n t o r s o f f i c i a l P a g e | 46

FOR MORE UPDATES AND MORE MATERIAL DO LIKE OUR FACEBOOK PAGE httpwwwfacebookcomsscmentorsofficial

3) Thyroid gland

िायराइिगरिी 4) Liver

यकत

Correct Answer Liver

Q200 Psidium guajava is the scientific

name of

लसडियमगआजावा mdashmdash कावहॴजञाननकनामहहॴ 1-

Feb-2017

Options

1) Guava

अम द

2) Mango

आम

3) Bamboo

बास

4) Jack fruit

कटहि

Correct Answer Guava

Q201 Which drug is used as a Blood

Thinner

चधरकह८पतिाकरनहॳकहॳ पमककसदवाकापरयह८गककयाजाताहहॴ

1-Feb-2017

Options

1) Warfarin

वाफर न

2) Tramadol

टरहॳमािह८ि

3) Azithromycin

एनजरह८मायलसन

4) Hydralazine

हाइडरह८िहॳनजन

Correct Answer Warfarin

Q202 Which of the following disease is

caused due to the deficiency of protein

परह८टीनकीकमीकहॳ कारणनननननलिखितमसहॳकह९नसारह८गहह८ताहहॴ 1-Feb-2017

Options

1) Arthritis

गटठया 2) Kwashiorkor

कािीओकय र

3) Goitre

गाइटर

4) Night Blindness

रतह९चध

Correct Answer Kwashiorkor

Q203 A is species of plant that has

adapted to survive in an environment

with little liquid water

mdashmdashndashपह९धहॳकीएकऐसहॳऐसहॳपरजानतहहॴ नजसनहॳकमपानीवािहॳवातावरणमजीपवतरहनहॳकहॳलिएअनकिनहहॴ 1-Feb-2017

Options

1) Xerophyte

म दपवद

2) Hydrophyte

जिीयपादप

3) Mesophyte

समह८दपवद

4) Thallophyte

िहॴिह८फाइटा Correct Answer xerophyte

xerophyte is a species of plant that has

adapted to survive in an environment

with little liquid water such as a desert

or an ice- or snow-covered region in the

Alps or the Arctic

Mesophytes are terrestrial plants which

are adapted to neither a particularly

dry nor particularly wet environment

An example of a mesophytic habitat

would be a rural temperate meadow

F A C E B O O K

P A G E h t t p w w w f a c e b o o k c o m s s c m e n t o r s o f f i c i a l P a g e | 47

FOR MORE UPDATES AND MORE MATERIAL DO LIKE OUR FACEBOOK PAGE httpwwwfacebookcomsscmentorsofficial

which might contain goldenrod clover

oxeye daisy and Rosa multiflora

thallophyte any of a group of plants or

plantlike organisms (such as algae and

fungi) that lack differentiated stems

leaves and roots and that were formerly

classified as a primary division

(Thallophyta) of the plant kingdom

Q204 How many types of teeth are

there in humans

मनषयोमककतनहॳपरकारकहॳ दातहह८तहॳहहॴ

1-Feb-2017

Options

1) 4

2) 5

3) 2

4) 3

Correct Answer 4

teeth -Humans have four types of

teethincisors canines premolars and

molars each with a specific function

The incisors cut the food the canines

tear the food and the molars and

premolars crush the food

Q205 Carica papaya is the scientific name of

कहॴ ररकापपाया mdashmdashndash कावहॴजञाननकनामहहॴ 2-

Feb-2017

Options

1) Peepal

पीपि

2) Papaya

पपीता 3) Tamarind

इमिी 4) Drumstick

ढह८िकाछड़ी Correct Answer Papaya

Q206 Muscles get tired when there is

shortfall of

जब mdashndash कीकमीहह८तीहहॴतबपहॳिीयिकजातीहहॴ 2-Feb-2017

Options

1) Lactic acid

िहॴनकटकएलसि

2) Na+ ions

Na+ आयन

3) ATP

एटीपी 4) Sulphates

समफहॳ टस

Correct Answer ATP

ATP is the energy source muscle fibers

use to make muscles contract

muscle tissuersquos main source of energy

called adenosine triphosphate or ATP

As your muscles use up this energy

source they become tired and fatigued

Oxygen is the key ingredient that helps

create new ATP to replenish the burned

up ATP in your muscles

Q207 Artocarpus integra is the

scientific name of आटह८कापयसइटीगरा mdashmdashmdash कावहॴजञाननकनामहहॴ 2-Feb-2017

Options

1) Guava

अम द

2) Pineapple

अनानास

3) Silver Oak

लसमवरओक

4) Jack fruit

कटहि

Correct Answer Jack fruit

Q208 Which organ stores fat soluble

vitamins

कह९नसाअगवसामघिनिीिपवटालमनह८काभिाराकरताहहॴ

2-Feb-2017

F A C E B O O K

P A G E h t t p w w w f a c e b o o k c o m s s c m e n t o r s o f f i c i a l P a g e | 48

FOR MORE UPDATES AND MORE MATERIAL DO LIKE OUR FACEBOOK PAGE httpwwwfacebookcomsscmentorsofficial

Options

1) Blood

रकत

2) Skin

तवचा 3) Liver

यकत

4) Pancreas

अगनयािय

Correct Answer Liver

Q209 Which disease is caused due to

deficiency of Iodine

आयह८िीनकहॳ कारणकह९नसारह८गहह८ताहहॴ 2-Feb-2017

Options

1) Rickets

ररकहॳ टस

2) Scurvy

सकवी 3) Goitre

गणमािा 4) Growth retardation

पवकासका कना Correct Answer Goitre

rickets A softening and weakening of

bones in children usually due to

inadequate vitamin D

Q210 Grevillea Robusta is the scientific name of

गरहॳपवलियारह८बसटा mdashmdashmdash- कापवजञाननकनामहहॴ 2-Feb-2017

Options

1) Peepal

पीपि

2) Teak

सागह९न

3) Silver Oak

लसमवरओक

4) Jack fruit

कटहि

Correct Answer Silver Oak

Q211 When a Cuttlefish is described as a Molluscs it is at which level of

classification

जबएककटिकफिकह८एकमह८िसकाकहॳ पमवखणयतककयाजाताहहॴतबयहॳवगीकरणकहॳ ककससतरपहॳनसितहहॴ 2-Feb-2017

Options

1) Class

वगय 2) Order

िम

3) Family

पररवार

4) Phylum

सघ

Correct Answer Phylum

Q212 Bambusa dendrocalmus is the

scientific name of बानबसािहॳडराकामस mdashmdashmdash कावहॴजञाननकनामहहॴ 3-Feb-2017

Options

1) Banyan

बरगद

2) Papaya

पपीता 3) Bamboo

बास

4) Pomegranate

अनार

Correct Answer Bamboo

Q213 Acinonyx Jubatus is the scientific name of

एलसनह८ननकसजयबहॳटस mdashmdashmdash

कावहॴजञाननकनामहहॴ 3-Feb-2017

F A C E B O O K

P A G E h t t p w w w f a c e b o o k c o m s s c m e n t o r s o f f i c i a l P a g e | 49

FOR MORE UPDATES AND MORE MATERIAL DO LIKE OUR FACEBOOK PAGE httpwwwfacebookcomsscmentorsofficial

Options

1) Bear

भाि 2) Horse

घह८िा 3) Cheetah

चीता 4) Zebra

जहॳिा Correct Answer Cheetah

Q214 The pale yellow colour of urine is

due to the presence of which pigment

मतरकाफीकापीिारगरगदरयकहॳ उपनसिनतकहॳ कारणहह८ताहहॴ

3-Feb-2017

Options

1) Urochrome

यरह८िह८म

2) Urophyll

यरह८कफि

3) Chlorophyll

किह८रह८कफि

4) Chloroplast

किह८रह८पिासट

Correct Answer Urochrome

Q215 Which of the following constitute

to form a gene

नननननलिखितमसहॳकह९नसीचीज़एकजीनकागठनकरतीहहॴ

3-Feb-2017

Options

1) Polynucleotides

पह८िीनयनकियह८टाईडस

2) Hydrocarbons

हाइडरह८काबोस

3) Lipoproteins

िाईपह८परह८टीनस

4) Lipids

लिपपडस

Correct Answer Polynucleotides

Polynucleotide molecule is a biopolymer

composed of 13 or more nucleotide

monomers covalently bonded in a chain

DNA (deoxyribonucleic acid) and RNA

(ribonucleic acid) are examples of

polynucleotides with distinct biological

function

Q216 Vertebrates belongs to the

phylum

रीढ़कीहडिीवािहॳपराणी mdashmdashmdash

परजानतकहॳ अतगायतआतहॳहहॴ 3-Feb-2017

Options

1) Arthropoda

आरह८पह८ड़ा 2) Annelida

एननलििा 3) Cnidaria

ननिहॳररया 4) Chordata

कह८िटा Correct Answer Chordata

Q217 Punica granatum is the scientific name of

पननकगरहॳनहॳटस mdashmdashmdash कावहॴजञाननकनामहहॴ 3-Feb-2017

Options

1) Custard Apple

सीताफि

2) Gulmohar

गिमह८हर

3) Silver Oak

लसमवरओक

4) Pomegranate

अनार

Correct Answer Pomegranate

F A C E B O O K

P A G E h t t p w w w f a c e b o o k c o m s s c m e n t o r s o f f i c i a l P a g e | 50

FOR MORE UPDATES AND MORE MATERIAL DO LIKE OUR FACEBOOK PAGE httpwwwfacebookcomsscmentorsofficial

Q218 Between a tiger and an monkey

which of the following is different

एकबाघऔरबदरकहॳ बीचनननननलिखितमसहॳकह९नसीबातअिगहहॴ 3-Feb-2017

Options

1) Kingdom

राजय

2) Phylum

जानत

3) Order

िम

4) Class

वगय Correct Answer order

Q219 The artificial heart was invented by

कबतरमहदयका mdashmdashmdash

दवाराअपवषकारककयागयािा 3-Feb-2017

Options

1) Muhammad Yunus

महनमदयनस

2) Linus Yale Jr

िाइनसयहॳिजय

3) Gazi Yasargil

गाजीयासचगयि

4) Paul Winchell

पह९िपवमकि Correct Answer Paul Winchell

Q220 Tamarindus indica is the

scientific name of

टहॳमररनडसइडिका mdashmdash कावहॴजञाननकनामहहॴ 7-

Feb-2017

Options

1) Neem

नीम

2) Pineapple

अनानास

3) Tamarind

इमिी 4)Chiku

चीक

Correct Answer Tamarind

Q221 In eukaryotic cells synthesis of

RNA takes place in the

यकहॳ योटटककह८लिकाओमआरएनएकासशिहॳषण

mdashndash महह८ताहहॴ 7-Feb-2017

Options

1) Mitochondria

माईटह८कोडडरया 2) Centrioles

सटरीयह८मस

3) Ribosomes

ररबह८सह८नस

4) Nucleus

नयनकियस

Correct Answer nucleus

eukaryotic cell -Transcription is the

process of synthesizing ribonucleic acid

(RNA)Synthesis takes place within the

nucleus of eukaryotic cells or in the

cytoplasm of prokaryotes and converts

the genetic code from a gene in

deoxyribonucleic acid ( DNA ) to a

strand of RNA that then directs

proteinsynthesis

Q222 _________is caused by parasites

of the Plasmodium genus

पिाजमह८डियमजातीकहॳ परजीवी mdash- कहॳ कारणहहॴ 7-Feb-2017

Options

1) Dysentery

पहॳचचि

2) Malaria

मिहॳररया 3) Chickenpox

F A C E B O O K

P A G E h t t p w w w f a c e b o o k c o m s s c m e n t o r s o f f i c i a l P a g e | 51

FOR MORE UPDATES AND MORE MATERIAL DO LIKE OUR FACEBOOK PAGE httpwwwfacebookcomsscmentorsofficial

चहॳचक

4) Herpes

हहॳपपयस

Correct Answer Malaria

Q223 Carotene in fruits and vegetables

gives it which color

फिह८औरसनलजयोमनसितकहॳ रह८टीनउनहकह९नसारगपरदानकरताहहॴ 7-Feb-2017

Options

1) Green

हरा 2) Pink

गिाबी 3) Orange

नारगी 4) Blue

नीिा Correct Answer Orange

Q224 Equus Caballus is the scientific

name of

एकवसकहॴ बहॳिस mdashmdashndash कापवजञाननकनामहहॴ 7-Feb-2017

Options

1) Horse

घह८िा 2) Zebra

ज़हॳिा 3) Donkey

गधा 4) Buffalo

भस

Correct Answer Horse

Q225 Elapidae Naja is the scientific name of

एिीपीिीनाजा mdashmdash- कावहॴजञाननकनामहहॴ 8-Feb-2017

Options

1) Cobra

कह८बरा 2) Elephant

हािी 3) Eagle

ग ि

4) Owl

उमि Correct Answer Cobra

Q226 Which disease is caused due to

deficiency of Iron

िह८हकीकमीकहॳ कारणकह९नसारह८गहह८ताहहॴ 8-Feb-

2017

Options

1) Beriberi

बहॳरीबहॳरी 2) Tetany

टहॳटनी 3) Kwashiorkor

कवािीऔरकर

4) Anaemia

रकतामपता Correct Answer Anaemia

Beriberi is a disease caused by a vitamin

B-1 deficiency also known as thiamine

deficiency

Tetany can be the result of an

electrolyte imbalance Most often itrsquos a

dramatically low calcium level also

known as hypocalcemia Tetany can also

be caused by magnesium deficiency or

too little potassium Having too much

acid (acidosis) or too much alkali

(alkalosis) in the body can also result in

tetany

Kwashiorkor also known as

ldquoedematous malnutrition It is a form of

malnutrition caused by a lack of protein

in the diet

Anaemia means that you have fewer red

blood cells than normal or you have less

F A C E B O O K

P A G E h t t p w w w f a c e b o o k c o m s s c m e n t o r s o f f i c i a l P a g e | 52

FOR MORE UPDATES AND MORE MATERIAL DO LIKE OUR FACEBOOK PAGE httpwwwfacebookcomsscmentorsofficial

haemoglobin than normal in each red

blood cell

Q227 is a leaf where the leaflets are

arranged along the middle vein

mdashndashएकपततीहहॴजहापतरकह८कीरचनाक ररयालिराकहॳ आसपासहह८तीहहॴ 8-Feb-2017

Options

1) Pinnately compound leaf

पपनहॳटिीसयकतपतती 2) Palmately compound leaf

पामहॳटिीसयकतपतती 3) Compound leaf

सयकतपतती 4) Simple leaf

साधारणपतती Correct Answer Pinnately compound

leaf

Q228 Haustoria or sucking roots are

found in which of the following

हह८सटह८ररयायाचसनहॳवािीजड़हॳनननननलिखितमसहॳककसमपाईजातीहहॴ 8-Feb-2017

Options

1) Wheat

गहॳह

2) Mango

आम

3) Chestnut

चहॳसटनट

4) Cuscuta

कसकयटा Correct Answer Cuscuta

Haustorial roots -The roots of parasitic

plants which penetrate into the host

tissues to absorb nourishment are

called haustorial roots hellip Also known as suckingor parasitic roots

Q229 Equs Asinus is the scientific name

of

एकवसएलसनस mdashmdashndash कावहॴजञाननकनामहहॴ 8-

Feb-2017

Options

1) Donkey

गधा 2) Cow

गाय

3) Deer

टहरन

4) Kangaroo

कगा

Correct Answer Donkey

Q230 Ficus benghalensis is the scientific name of

फाईकसबहॳनगहॳिहॳलसस mdashndash कापवजञाननकनामहहॴ 8-Feb-2017

Options

1) Banyan

बरगद

2) Pineapple

अनानास

3) Babul

बबि

4) Tulsi

तिसी Correct Answer Banyan

Q231 Equus burchellii is the scientific name of

एकवसबचिी mdashmdash- कापवजञाननकनामहहॴ 8-Feb-2017

Options

1) Horse

घह८िा 2) Zebra

जहॳिा 3) Buffalo

F A C E B O O K

P A G E h t t p w w w f a c e b o o k c o m s s c m e n t o r s o f f i c i a l P a g e | 53

FOR MORE UPDATES AND MORE MATERIAL DO LIKE OUR FACEBOOK PAGE httpwwwfacebookcomsscmentorsofficial

भस

4) Ass

गधा Correct Answer Zebra

Page 44: COMPILATION OF ALL 72 SETS OF BIOLOGY SSC CHSL-2016 · OF BIOLOGY SSC CHSL-2016 PREPARED BY : SSC MENTORS BIOLOGY SPECIAL . F A C E B O O K P A G E : h t t p : / / w w w . f a c e

F A C E B O O K

P A G E h t t p w w w f a c e b o o k c o m s s c m e n t o r s o f f i c i a l P a g e | 43

FOR MORE UPDATES AND MORE MATERIAL DO LIKE OUR FACEBOOK PAGE httpwwwfacebookcomsscmentorsofficial

2) Golgi apparatus

गह८मजीएपहॳरहॳटस

3) Ribosomes

ररबह८सह८नस

4) Lysosome

िायसह८सह८नस

Correct Answer ribosomes

Proteins are produced by stringing

amino acids together in the order

specified by messenger RNA strands

that were transcribed from DNA in the

cell nucleus The process of synthesizing

a protein is called translation and it

occurs on ribosomes in the cytoplasm of

a cell

Q186 Polio is a disease caused by which

of the following

नननननलिखितमसहॳपह८लियह८कीबबमारह८हह८नहॳकाकारणकयाहहॴ

31-Jan-2017

Options

1) Bacteria

बहॴकटीररयि

2) Mosquito

मचछर

3) Virus

वायरस

4) Cockroach

नतिच हॳ Correct Answer Virus

Polio or poliomyelitis is a crippling and

potentially deadly infectious disease It

is caused by the poliovirus

Q187 ndash Hay fever is a sign of which of

the following

हहॳकफवरनननननलिखितमसहॳककसकाएकसकहॳ तहहॴ

31-Jan-2017

Options

1) Old Age

वदावसिा 2) Malnutrition

कपह८सण

3) Allergy

एिनजय 4) Over Work

अतयचधककाययकरना Correct Answer Allergy

Q188 How many chromosomes does a

human cell contain

एकमानवकह८लिकामककतनहॳगणसतरहह८तहॳहहॴ

29-Jan-2017

Options

1) 6

2) 26

3) 46

4) 66

Correct Answer 46

In humans each cell normally contains

23 pairs of chromosomes for a total of

46 Twenty-two of these pairs called

autosomes look the same in both males

and females The 23rd pair the sex

chromosomes differ between males and

females

Q189 Which of the following is not true

about Bryophyta

ननननमसहॳकह९नसीबातिायह८फाइटकहॳ बारहॳमसचनहीहहॴ 31-Jan-2017

Options

1) Dominant phase is gametophytes

परमिचरणगहॳलमतह८फाइटसहह८ताहहॴ 2) Main plant body is haploid

पह९धहॳकामखयिरीरअगखणतहह८ताहहॴ 3) Spores are homospores

बीजाणहह८मह८सफह८रसहह८तहॳहहॴ 4) Flowers are present

फिमह८जदहह८तहॳहहॴ Correct Answer Flowers are present

F A C E B O O K

P A G E h t t p w w w f a c e b o o k c o m s s c m e n t o r s o f f i c i a l P a g e | 44

FOR MORE UPDATES AND MORE MATERIAL DO LIKE OUR FACEBOOK PAGE httpwwwfacebookcomsscmentorsofficial

Q190 Which aquatic animal has

trailing tentacles

ककसजिीयजानवरकहॳ पीछहॳचिनहॳवािहॳटहॳटकिसहह८तहॳहहॴ

31-Jan-2017

Options

1) Sea horse

समदरीघह८िा 2) Corals

मगा 3) Jelly fish

जहॳिीमछिी 4) Star fish

तारामछिी Correct Answer Jelly fish

Jellyfish with its umbrella-shaped bell

and trailing tentacles

Q191 Which type of pathogen causes

the water-borne disease Poliomyelitis

(Polio)

ककसपरकारकारह८गजनकजिजननतरह८गपह८लियह८मायहॳटटस (पह८लियह८) काकारणहहॴ 31-Jan-

2017

Options

1) Parasitic

परजीवी 2) Algal

िहॴवालिय

3) Viral

वायरि

4) Bacterial

बहॴकटीररयि

Correct Answer Viral

Q192 The outer white part of the eye

that protects the inner structures is

आािकाबाहरीसफहॳ दटहससाजह८आतररकसरचनाओकीरकषाकरताहहॴ वह mdashmdashmdash हहॴ 31-Jan-

2017

Options

1) Iris

आयररस

2) Sclera

सकिहॳरा 3) Retina

रहॳटटना 4) Cornea

कह८ननयया Correct Answer Sclera

Q193 Proteins are made up of

परह८टीनकाननमायण mdashndash सहॳहह८ताहहॴ 31-Jan-2017

Options

1) Amino acids

एलमनह८अनि

2) Fatty acids

वसायकतअनि

3) Glucose

गिकह८ज

4)Nucleotides

नयनकियह८टाईिस

Correct Answer Amino acids

Q194 Moringa Oleifera is the scientific

name of

मह८ररगओलिफहॳ रा mdashmdashndash कावहॴजञाननकनामहहॴ 31-Jan-2017

Options

1) Banyan

बरगद

2) Gulmohar

गिमह८हर

3) Amla

आमिा

F A C E B O O K

P A G E h t t p w w w f a c e b o o k c o m s s c m e n t o r s o f f i c i a l P a g e | 45

FOR MORE UPDATES AND MORE MATERIAL DO LIKE OUR FACEBOOK PAGE httpwwwfacebookcomsscmentorsofficial

4) Drumstick

डरमनसटक

Correct Answer Drumstick

Q195 Kidney stones are composed of

गदकीपिरी mdashndash सहॳबनीहह८तीहहॴ 1-Feb-2017

Options

1) Calcium Oxalate

कहॴ नमसयमओकजहॳिहॳट

2) Sodium Chloride

सह८डियमकिह८राइि

3) Magnesium Nitrate

महॳनगनलियमनाइतटरहॳट

4) Calcium Bicarbonate

कहॴ नमियमबायकबोनहॳट

Correct Answer Calcium Oxalate

Q196 ndash Which of the following is not

true about Angiosperms

ननननमसहॳकह९नसीबातआवतबीजीकहॳ बारहॳमसचनहीहहॴ 1-Feb-2017

Options

1) Dominant phase is gametophytes

परमिचरणगहॳलमतह८फाइटहह८ताहहॴ 2) Vascular bundles are present

सवहनीबििमह९जदहह८ताहहॴ 3) Spores are heterospores

बीजाणहहॳटहॳरह८सपह८रसहह८तहॳहहॴ 4) Seeds are covered

बीजढकहॳ हह८तहॳहहॴ Correct Answer Dominant phase is

gametophytes

Q197 All of the following are excretory

(waste) products of animals except

नननननलिखितमसहॳककसएककह८छह८ड़करअनयसभीपराखणयोदवाराउतसनजयतपदाियहहॴ 1-Feb-

2017

Options

1) Uric Acid

यररकएलसि

2) Ammonia

अमह८ननया 3) Carbohydrates

काबोहाइडरहॳट

4) Urea

यररया Correct Answer Carbohydrates

In animals the main excretory products

are carbon dioxide ammonia (in

ammoniotelics) urea (in ureotelics) uric

acid (in uricotelics) guanine (in

Arachnida) and creatine

Q198 RNA is a polymeric molecule

What does RNA stand for

आरएनइएएकबहिकआणहहॴ इसकाकापवय पकयाहहॴ 1-Feb-2017

Options

1) Rado Nuclear Acid

रािह८नयनकियरएलसि

2) Ribo Nucleic Acid

राइबह८नयनकिकएलसि

3) Rhino Nuclear Acid

हाइनह८नयनकियरएलसि

4) Resto Nucleus Acid

रहॳसटह८नयकिीयसएलसि

Correct Answer Ribo Nucleic Acid

Q199 Which organ does detoxification

and produces chemicals needed for

digestion

कह९नसाअगपवषहरणकरताहहॴऔरपाचनकहॳ लिएआवशयकरसायनोकह८पहॴदाकरताहहॴ 1-Feb-

2017

Options

1) Salivary glands

िारगरचिया 2) Pancreas

अगनयािय

F A C E B O O K

P A G E h t t p w w w f a c e b o o k c o m s s c m e n t o r s o f f i c i a l P a g e | 46

FOR MORE UPDATES AND MORE MATERIAL DO LIKE OUR FACEBOOK PAGE httpwwwfacebookcomsscmentorsofficial

3) Thyroid gland

िायराइिगरिी 4) Liver

यकत

Correct Answer Liver

Q200 Psidium guajava is the scientific

name of

लसडियमगआजावा mdashmdash कावहॴजञाननकनामहहॴ 1-

Feb-2017

Options

1) Guava

अम द

2) Mango

आम

3) Bamboo

बास

4) Jack fruit

कटहि

Correct Answer Guava

Q201 Which drug is used as a Blood

Thinner

चधरकह८पतिाकरनहॳकहॳ पमककसदवाकापरयह८गककयाजाताहहॴ

1-Feb-2017

Options

1) Warfarin

वाफर न

2) Tramadol

टरहॳमािह८ि

3) Azithromycin

एनजरह८मायलसन

4) Hydralazine

हाइडरह८िहॳनजन

Correct Answer Warfarin

Q202 Which of the following disease is

caused due to the deficiency of protein

परह८टीनकीकमीकहॳ कारणनननननलिखितमसहॳकह९नसारह८गहह८ताहहॴ 1-Feb-2017

Options

1) Arthritis

गटठया 2) Kwashiorkor

कािीओकय र

3) Goitre

गाइटर

4) Night Blindness

रतह९चध

Correct Answer Kwashiorkor

Q203 A is species of plant that has

adapted to survive in an environment

with little liquid water

mdashmdashndashपह९धहॳकीएकऐसहॳऐसहॳपरजानतहहॴ नजसनहॳकमपानीवािहॳवातावरणमजीपवतरहनहॳकहॳलिएअनकिनहहॴ 1-Feb-2017

Options

1) Xerophyte

म दपवद

2) Hydrophyte

जिीयपादप

3) Mesophyte

समह८दपवद

4) Thallophyte

िहॴिह८फाइटा Correct Answer xerophyte

xerophyte is a species of plant that has

adapted to survive in an environment

with little liquid water such as a desert

or an ice- or snow-covered region in the

Alps or the Arctic

Mesophytes are terrestrial plants which

are adapted to neither a particularly

dry nor particularly wet environment

An example of a mesophytic habitat

would be a rural temperate meadow

F A C E B O O K

P A G E h t t p w w w f a c e b o o k c o m s s c m e n t o r s o f f i c i a l P a g e | 47

FOR MORE UPDATES AND MORE MATERIAL DO LIKE OUR FACEBOOK PAGE httpwwwfacebookcomsscmentorsofficial

which might contain goldenrod clover

oxeye daisy and Rosa multiflora

thallophyte any of a group of plants or

plantlike organisms (such as algae and

fungi) that lack differentiated stems

leaves and roots and that were formerly

classified as a primary division

(Thallophyta) of the plant kingdom

Q204 How many types of teeth are

there in humans

मनषयोमककतनहॳपरकारकहॳ दातहह८तहॳहहॴ

1-Feb-2017

Options

1) 4

2) 5

3) 2

4) 3

Correct Answer 4

teeth -Humans have four types of

teethincisors canines premolars and

molars each with a specific function

The incisors cut the food the canines

tear the food and the molars and

premolars crush the food

Q205 Carica papaya is the scientific name of

कहॴ ररकापपाया mdashmdashndash कावहॴजञाननकनामहहॴ 2-

Feb-2017

Options

1) Peepal

पीपि

2) Papaya

पपीता 3) Tamarind

इमिी 4) Drumstick

ढह८िकाछड़ी Correct Answer Papaya

Q206 Muscles get tired when there is

shortfall of

जब mdashndash कीकमीहह८तीहहॴतबपहॳिीयिकजातीहहॴ 2-Feb-2017

Options

1) Lactic acid

िहॴनकटकएलसि

2) Na+ ions

Na+ आयन

3) ATP

एटीपी 4) Sulphates

समफहॳ टस

Correct Answer ATP

ATP is the energy source muscle fibers

use to make muscles contract

muscle tissuersquos main source of energy

called adenosine triphosphate or ATP

As your muscles use up this energy

source they become tired and fatigued

Oxygen is the key ingredient that helps

create new ATP to replenish the burned

up ATP in your muscles

Q207 Artocarpus integra is the

scientific name of आटह८कापयसइटीगरा mdashmdashmdash कावहॴजञाननकनामहहॴ 2-Feb-2017

Options

1) Guava

अम द

2) Pineapple

अनानास

3) Silver Oak

लसमवरओक

4) Jack fruit

कटहि

Correct Answer Jack fruit

Q208 Which organ stores fat soluble

vitamins

कह९नसाअगवसामघिनिीिपवटालमनह८काभिाराकरताहहॴ

2-Feb-2017

F A C E B O O K

P A G E h t t p w w w f a c e b o o k c o m s s c m e n t o r s o f f i c i a l P a g e | 48

FOR MORE UPDATES AND MORE MATERIAL DO LIKE OUR FACEBOOK PAGE httpwwwfacebookcomsscmentorsofficial

Options

1) Blood

रकत

2) Skin

तवचा 3) Liver

यकत

4) Pancreas

अगनयािय

Correct Answer Liver

Q209 Which disease is caused due to

deficiency of Iodine

आयह८िीनकहॳ कारणकह९नसारह८गहह८ताहहॴ 2-Feb-2017

Options

1) Rickets

ररकहॳ टस

2) Scurvy

सकवी 3) Goitre

गणमािा 4) Growth retardation

पवकासका कना Correct Answer Goitre

rickets A softening and weakening of

bones in children usually due to

inadequate vitamin D

Q210 Grevillea Robusta is the scientific name of

गरहॳपवलियारह८बसटा mdashmdashmdash- कापवजञाननकनामहहॴ 2-Feb-2017

Options

1) Peepal

पीपि

2) Teak

सागह९न

3) Silver Oak

लसमवरओक

4) Jack fruit

कटहि

Correct Answer Silver Oak

Q211 When a Cuttlefish is described as a Molluscs it is at which level of

classification

जबएककटिकफिकह८एकमह८िसकाकहॳ पमवखणयतककयाजाताहहॴतबयहॳवगीकरणकहॳ ककससतरपहॳनसितहहॴ 2-Feb-2017

Options

1) Class

वगय 2) Order

िम

3) Family

पररवार

4) Phylum

सघ

Correct Answer Phylum

Q212 Bambusa dendrocalmus is the

scientific name of बानबसािहॳडराकामस mdashmdashmdash कावहॴजञाननकनामहहॴ 3-Feb-2017

Options

1) Banyan

बरगद

2) Papaya

पपीता 3) Bamboo

बास

4) Pomegranate

अनार

Correct Answer Bamboo

Q213 Acinonyx Jubatus is the scientific name of

एलसनह८ननकसजयबहॳटस mdashmdashmdash

कावहॴजञाननकनामहहॴ 3-Feb-2017

F A C E B O O K

P A G E h t t p w w w f a c e b o o k c o m s s c m e n t o r s o f f i c i a l P a g e | 49

FOR MORE UPDATES AND MORE MATERIAL DO LIKE OUR FACEBOOK PAGE httpwwwfacebookcomsscmentorsofficial

Options

1) Bear

भाि 2) Horse

घह८िा 3) Cheetah

चीता 4) Zebra

जहॳिा Correct Answer Cheetah

Q214 The pale yellow colour of urine is

due to the presence of which pigment

मतरकाफीकापीिारगरगदरयकहॳ उपनसिनतकहॳ कारणहह८ताहहॴ

3-Feb-2017

Options

1) Urochrome

यरह८िह८म

2) Urophyll

यरह८कफि

3) Chlorophyll

किह८रह८कफि

4) Chloroplast

किह८रह८पिासट

Correct Answer Urochrome

Q215 Which of the following constitute

to form a gene

नननननलिखितमसहॳकह९नसीचीज़एकजीनकागठनकरतीहहॴ

3-Feb-2017

Options

1) Polynucleotides

पह८िीनयनकियह८टाईडस

2) Hydrocarbons

हाइडरह८काबोस

3) Lipoproteins

िाईपह८परह८टीनस

4) Lipids

लिपपडस

Correct Answer Polynucleotides

Polynucleotide molecule is a biopolymer

composed of 13 or more nucleotide

monomers covalently bonded in a chain

DNA (deoxyribonucleic acid) and RNA

(ribonucleic acid) are examples of

polynucleotides with distinct biological

function

Q216 Vertebrates belongs to the

phylum

रीढ़कीहडिीवािहॳपराणी mdashmdashmdash

परजानतकहॳ अतगायतआतहॳहहॴ 3-Feb-2017

Options

1) Arthropoda

आरह८पह८ड़ा 2) Annelida

एननलििा 3) Cnidaria

ननिहॳररया 4) Chordata

कह८िटा Correct Answer Chordata

Q217 Punica granatum is the scientific name of

पननकगरहॳनहॳटस mdashmdashmdash कावहॴजञाननकनामहहॴ 3-Feb-2017

Options

1) Custard Apple

सीताफि

2) Gulmohar

गिमह८हर

3) Silver Oak

लसमवरओक

4) Pomegranate

अनार

Correct Answer Pomegranate

F A C E B O O K

P A G E h t t p w w w f a c e b o o k c o m s s c m e n t o r s o f f i c i a l P a g e | 50

FOR MORE UPDATES AND MORE MATERIAL DO LIKE OUR FACEBOOK PAGE httpwwwfacebookcomsscmentorsofficial

Q218 Between a tiger and an monkey

which of the following is different

एकबाघऔरबदरकहॳ बीचनननननलिखितमसहॳकह९नसीबातअिगहहॴ 3-Feb-2017

Options

1) Kingdom

राजय

2) Phylum

जानत

3) Order

िम

4) Class

वगय Correct Answer order

Q219 The artificial heart was invented by

कबतरमहदयका mdashmdashmdash

दवाराअपवषकारककयागयािा 3-Feb-2017

Options

1) Muhammad Yunus

महनमदयनस

2) Linus Yale Jr

िाइनसयहॳिजय

3) Gazi Yasargil

गाजीयासचगयि

4) Paul Winchell

पह९िपवमकि Correct Answer Paul Winchell

Q220 Tamarindus indica is the

scientific name of

टहॳमररनडसइडिका mdashmdash कावहॴजञाननकनामहहॴ 7-

Feb-2017

Options

1) Neem

नीम

2) Pineapple

अनानास

3) Tamarind

इमिी 4)Chiku

चीक

Correct Answer Tamarind

Q221 In eukaryotic cells synthesis of

RNA takes place in the

यकहॳ योटटककह८लिकाओमआरएनएकासशिहॳषण

mdashndash महह८ताहहॴ 7-Feb-2017

Options

1) Mitochondria

माईटह८कोडडरया 2) Centrioles

सटरीयह८मस

3) Ribosomes

ररबह८सह८नस

4) Nucleus

नयनकियस

Correct Answer nucleus

eukaryotic cell -Transcription is the

process of synthesizing ribonucleic acid

(RNA)Synthesis takes place within the

nucleus of eukaryotic cells or in the

cytoplasm of prokaryotes and converts

the genetic code from a gene in

deoxyribonucleic acid ( DNA ) to a

strand of RNA that then directs

proteinsynthesis

Q222 _________is caused by parasites

of the Plasmodium genus

पिाजमह८डियमजातीकहॳ परजीवी mdash- कहॳ कारणहहॴ 7-Feb-2017

Options

1) Dysentery

पहॳचचि

2) Malaria

मिहॳररया 3) Chickenpox

F A C E B O O K

P A G E h t t p w w w f a c e b o o k c o m s s c m e n t o r s o f f i c i a l P a g e | 51

FOR MORE UPDATES AND MORE MATERIAL DO LIKE OUR FACEBOOK PAGE httpwwwfacebookcomsscmentorsofficial

चहॳचक

4) Herpes

हहॳपपयस

Correct Answer Malaria

Q223 Carotene in fruits and vegetables

gives it which color

फिह८औरसनलजयोमनसितकहॳ रह८टीनउनहकह९नसारगपरदानकरताहहॴ 7-Feb-2017

Options

1) Green

हरा 2) Pink

गिाबी 3) Orange

नारगी 4) Blue

नीिा Correct Answer Orange

Q224 Equus Caballus is the scientific

name of

एकवसकहॴ बहॳिस mdashmdashndash कापवजञाननकनामहहॴ 7-Feb-2017

Options

1) Horse

घह८िा 2) Zebra

ज़हॳिा 3) Donkey

गधा 4) Buffalo

भस

Correct Answer Horse

Q225 Elapidae Naja is the scientific name of

एिीपीिीनाजा mdashmdash- कावहॴजञाननकनामहहॴ 8-Feb-2017

Options

1) Cobra

कह८बरा 2) Elephant

हािी 3) Eagle

ग ि

4) Owl

उमि Correct Answer Cobra

Q226 Which disease is caused due to

deficiency of Iron

िह८हकीकमीकहॳ कारणकह९नसारह८गहह८ताहहॴ 8-Feb-

2017

Options

1) Beriberi

बहॳरीबहॳरी 2) Tetany

टहॳटनी 3) Kwashiorkor

कवािीऔरकर

4) Anaemia

रकतामपता Correct Answer Anaemia

Beriberi is a disease caused by a vitamin

B-1 deficiency also known as thiamine

deficiency

Tetany can be the result of an

electrolyte imbalance Most often itrsquos a

dramatically low calcium level also

known as hypocalcemia Tetany can also

be caused by magnesium deficiency or

too little potassium Having too much

acid (acidosis) or too much alkali

(alkalosis) in the body can also result in

tetany

Kwashiorkor also known as

ldquoedematous malnutrition It is a form of

malnutrition caused by a lack of protein

in the diet

Anaemia means that you have fewer red

blood cells than normal or you have less

F A C E B O O K

P A G E h t t p w w w f a c e b o o k c o m s s c m e n t o r s o f f i c i a l P a g e | 52

FOR MORE UPDATES AND MORE MATERIAL DO LIKE OUR FACEBOOK PAGE httpwwwfacebookcomsscmentorsofficial

haemoglobin than normal in each red

blood cell

Q227 is a leaf where the leaflets are

arranged along the middle vein

mdashndashएकपततीहहॴजहापतरकह८कीरचनाक ररयालिराकहॳ आसपासहह८तीहहॴ 8-Feb-2017

Options

1) Pinnately compound leaf

पपनहॳटिीसयकतपतती 2) Palmately compound leaf

पामहॳटिीसयकतपतती 3) Compound leaf

सयकतपतती 4) Simple leaf

साधारणपतती Correct Answer Pinnately compound

leaf

Q228 Haustoria or sucking roots are

found in which of the following

हह८सटह८ररयायाचसनहॳवािीजड़हॳनननननलिखितमसहॳककसमपाईजातीहहॴ 8-Feb-2017

Options

1) Wheat

गहॳह

2) Mango

आम

3) Chestnut

चहॳसटनट

4) Cuscuta

कसकयटा Correct Answer Cuscuta

Haustorial roots -The roots of parasitic

plants which penetrate into the host

tissues to absorb nourishment are

called haustorial roots hellip Also known as suckingor parasitic roots

Q229 Equs Asinus is the scientific name

of

एकवसएलसनस mdashmdashndash कावहॴजञाननकनामहहॴ 8-

Feb-2017

Options

1) Donkey

गधा 2) Cow

गाय

3) Deer

टहरन

4) Kangaroo

कगा

Correct Answer Donkey

Q230 Ficus benghalensis is the scientific name of

फाईकसबहॳनगहॳिहॳलसस mdashndash कापवजञाननकनामहहॴ 8-Feb-2017

Options

1) Banyan

बरगद

2) Pineapple

अनानास

3) Babul

बबि

4) Tulsi

तिसी Correct Answer Banyan

Q231 Equus burchellii is the scientific name of

एकवसबचिी mdashmdash- कापवजञाननकनामहहॴ 8-Feb-2017

Options

1) Horse

घह८िा 2) Zebra

जहॳिा 3) Buffalo

F A C E B O O K

P A G E h t t p w w w f a c e b o o k c o m s s c m e n t o r s o f f i c i a l P a g e | 53

FOR MORE UPDATES AND MORE MATERIAL DO LIKE OUR FACEBOOK PAGE httpwwwfacebookcomsscmentorsofficial

भस

4) Ass

गधा Correct Answer Zebra

Page 45: COMPILATION OF ALL 72 SETS OF BIOLOGY SSC CHSL-2016 · OF BIOLOGY SSC CHSL-2016 PREPARED BY : SSC MENTORS BIOLOGY SPECIAL . F A C E B O O K P A G E : h t t p : / / w w w . f a c e

F A C E B O O K

P A G E h t t p w w w f a c e b o o k c o m s s c m e n t o r s o f f i c i a l P a g e | 44

FOR MORE UPDATES AND MORE MATERIAL DO LIKE OUR FACEBOOK PAGE httpwwwfacebookcomsscmentorsofficial

Q190 Which aquatic animal has

trailing tentacles

ककसजिीयजानवरकहॳ पीछहॳचिनहॳवािहॳटहॳटकिसहह८तहॳहहॴ

31-Jan-2017

Options

1) Sea horse

समदरीघह८िा 2) Corals

मगा 3) Jelly fish

जहॳिीमछिी 4) Star fish

तारामछिी Correct Answer Jelly fish

Jellyfish with its umbrella-shaped bell

and trailing tentacles

Q191 Which type of pathogen causes

the water-borne disease Poliomyelitis

(Polio)

ककसपरकारकारह८गजनकजिजननतरह८गपह८लियह८मायहॳटटस (पह८लियह८) काकारणहहॴ 31-Jan-

2017

Options

1) Parasitic

परजीवी 2) Algal

िहॴवालिय

3) Viral

वायरि

4) Bacterial

बहॴकटीररयि

Correct Answer Viral

Q192 The outer white part of the eye

that protects the inner structures is

आािकाबाहरीसफहॳ दटहससाजह८आतररकसरचनाओकीरकषाकरताहहॴ वह mdashmdashmdash हहॴ 31-Jan-

2017

Options

1) Iris

आयररस

2) Sclera

सकिहॳरा 3) Retina

रहॳटटना 4) Cornea

कह८ननयया Correct Answer Sclera

Q193 Proteins are made up of

परह८टीनकाननमायण mdashndash सहॳहह८ताहहॴ 31-Jan-2017

Options

1) Amino acids

एलमनह८अनि

2) Fatty acids

वसायकतअनि

3) Glucose

गिकह८ज

4)Nucleotides

नयनकियह८टाईिस

Correct Answer Amino acids

Q194 Moringa Oleifera is the scientific

name of

मह८ररगओलिफहॳ रा mdashmdashndash कावहॴजञाननकनामहहॴ 31-Jan-2017

Options

1) Banyan

बरगद

2) Gulmohar

गिमह८हर

3) Amla

आमिा

F A C E B O O K

P A G E h t t p w w w f a c e b o o k c o m s s c m e n t o r s o f f i c i a l P a g e | 45

FOR MORE UPDATES AND MORE MATERIAL DO LIKE OUR FACEBOOK PAGE httpwwwfacebookcomsscmentorsofficial

4) Drumstick

डरमनसटक

Correct Answer Drumstick

Q195 Kidney stones are composed of

गदकीपिरी mdashndash सहॳबनीहह८तीहहॴ 1-Feb-2017

Options

1) Calcium Oxalate

कहॴ नमसयमओकजहॳिहॳट

2) Sodium Chloride

सह८डियमकिह८राइि

3) Magnesium Nitrate

महॳनगनलियमनाइतटरहॳट

4) Calcium Bicarbonate

कहॴ नमियमबायकबोनहॳट

Correct Answer Calcium Oxalate

Q196 ndash Which of the following is not

true about Angiosperms

ननननमसहॳकह९नसीबातआवतबीजीकहॳ बारहॳमसचनहीहहॴ 1-Feb-2017

Options

1) Dominant phase is gametophytes

परमिचरणगहॳलमतह८फाइटहह८ताहहॴ 2) Vascular bundles are present

सवहनीबििमह९जदहह८ताहहॴ 3) Spores are heterospores

बीजाणहहॳटहॳरह८सपह८रसहह८तहॳहहॴ 4) Seeds are covered

बीजढकहॳ हह८तहॳहहॴ Correct Answer Dominant phase is

gametophytes

Q197 All of the following are excretory

(waste) products of animals except

नननननलिखितमसहॳककसएककह८छह८ड़करअनयसभीपराखणयोदवाराउतसनजयतपदाियहहॴ 1-Feb-

2017

Options

1) Uric Acid

यररकएलसि

2) Ammonia

अमह८ननया 3) Carbohydrates

काबोहाइडरहॳट

4) Urea

यररया Correct Answer Carbohydrates

In animals the main excretory products

are carbon dioxide ammonia (in

ammoniotelics) urea (in ureotelics) uric

acid (in uricotelics) guanine (in

Arachnida) and creatine

Q198 RNA is a polymeric molecule

What does RNA stand for

आरएनइएएकबहिकआणहहॴ इसकाकापवय पकयाहहॴ 1-Feb-2017

Options

1) Rado Nuclear Acid

रािह८नयनकियरएलसि

2) Ribo Nucleic Acid

राइबह८नयनकिकएलसि

3) Rhino Nuclear Acid

हाइनह८नयनकियरएलसि

4) Resto Nucleus Acid

रहॳसटह८नयकिीयसएलसि

Correct Answer Ribo Nucleic Acid

Q199 Which organ does detoxification

and produces chemicals needed for

digestion

कह९नसाअगपवषहरणकरताहहॴऔरपाचनकहॳ लिएआवशयकरसायनोकह८पहॴदाकरताहहॴ 1-Feb-

2017

Options

1) Salivary glands

िारगरचिया 2) Pancreas

अगनयािय

F A C E B O O K

P A G E h t t p w w w f a c e b o o k c o m s s c m e n t o r s o f f i c i a l P a g e | 46

FOR MORE UPDATES AND MORE MATERIAL DO LIKE OUR FACEBOOK PAGE httpwwwfacebookcomsscmentorsofficial

3) Thyroid gland

िायराइिगरिी 4) Liver

यकत

Correct Answer Liver

Q200 Psidium guajava is the scientific

name of

लसडियमगआजावा mdashmdash कावहॴजञाननकनामहहॴ 1-

Feb-2017

Options

1) Guava

अम द

2) Mango

आम

3) Bamboo

बास

4) Jack fruit

कटहि

Correct Answer Guava

Q201 Which drug is used as a Blood

Thinner

चधरकह८पतिाकरनहॳकहॳ पमककसदवाकापरयह८गककयाजाताहहॴ

1-Feb-2017

Options

1) Warfarin

वाफर न

2) Tramadol

टरहॳमािह८ि

3) Azithromycin

एनजरह८मायलसन

4) Hydralazine

हाइडरह८िहॳनजन

Correct Answer Warfarin

Q202 Which of the following disease is

caused due to the deficiency of protein

परह८टीनकीकमीकहॳ कारणनननननलिखितमसहॳकह९नसारह८गहह८ताहहॴ 1-Feb-2017

Options

1) Arthritis

गटठया 2) Kwashiorkor

कािीओकय र

3) Goitre

गाइटर

4) Night Blindness

रतह९चध

Correct Answer Kwashiorkor

Q203 A is species of plant that has

adapted to survive in an environment

with little liquid water

mdashmdashndashपह९धहॳकीएकऐसहॳऐसहॳपरजानतहहॴ नजसनहॳकमपानीवािहॳवातावरणमजीपवतरहनहॳकहॳलिएअनकिनहहॴ 1-Feb-2017

Options

1) Xerophyte

म दपवद

2) Hydrophyte

जिीयपादप

3) Mesophyte

समह८दपवद

4) Thallophyte

िहॴिह८फाइटा Correct Answer xerophyte

xerophyte is a species of plant that has

adapted to survive in an environment

with little liquid water such as a desert

or an ice- or snow-covered region in the

Alps or the Arctic

Mesophytes are terrestrial plants which

are adapted to neither a particularly

dry nor particularly wet environment

An example of a mesophytic habitat

would be a rural temperate meadow

F A C E B O O K

P A G E h t t p w w w f a c e b o o k c o m s s c m e n t o r s o f f i c i a l P a g e | 47

FOR MORE UPDATES AND MORE MATERIAL DO LIKE OUR FACEBOOK PAGE httpwwwfacebookcomsscmentorsofficial

which might contain goldenrod clover

oxeye daisy and Rosa multiflora

thallophyte any of a group of plants or

plantlike organisms (such as algae and

fungi) that lack differentiated stems

leaves and roots and that were formerly

classified as a primary division

(Thallophyta) of the plant kingdom

Q204 How many types of teeth are

there in humans

मनषयोमककतनहॳपरकारकहॳ दातहह८तहॳहहॴ

1-Feb-2017

Options

1) 4

2) 5

3) 2

4) 3

Correct Answer 4

teeth -Humans have four types of

teethincisors canines premolars and

molars each with a specific function

The incisors cut the food the canines

tear the food and the molars and

premolars crush the food

Q205 Carica papaya is the scientific name of

कहॴ ररकापपाया mdashmdashndash कावहॴजञाननकनामहहॴ 2-

Feb-2017

Options

1) Peepal

पीपि

2) Papaya

पपीता 3) Tamarind

इमिी 4) Drumstick

ढह८िकाछड़ी Correct Answer Papaya

Q206 Muscles get tired when there is

shortfall of

जब mdashndash कीकमीहह८तीहहॴतबपहॳिीयिकजातीहहॴ 2-Feb-2017

Options

1) Lactic acid

िहॴनकटकएलसि

2) Na+ ions

Na+ आयन

3) ATP

एटीपी 4) Sulphates

समफहॳ टस

Correct Answer ATP

ATP is the energy source muscle fibers

use to make muscles contract

muscle tissuersquos main source of energy

called adenosine triphosphate or ATP

As your muscles use up this energy

source they become tired and fatigued

Oxygen is the key ingredient that helps

create new ATP to replenish the burned

up ATP in your muscles

Q207 Artocarpus integra is the

scientific name of आटह८कापयसइटीगरा mdashmdashmdash कावहॴजञाननकनामहहॴ 2-Feb-2017

Options

1) Guava

अम द

2) Pineapple

अनानास

3) Silver Oak

लसमवरओक

4) Jack fruit

कटहि

Correct Answer Jack fruit

Q208 Which organ stores fat soluble

vitamins

कह९नसाअगवसामघिनिीिपवटालमनह८काभिाराकरताहहॴ

2-Feb-2017

F A C E B O O K

P A G E h t t p w w w f a c e b o o k c o m s s c m e n t o r s o f f i c i a l P a g e | 48

FOR MORE UPDATES AND MORE MATERIAL DO LIKE OUR FACEBOOK PAGE httpwwwfacebookcomsscmentorsofficial

Options

1) Blood

रकत

2) Skin

तवचा 3) Liver

यकत

4) Pancreas

अगनयािय

Correct Answer Liver

Q209 Which disease is caused due to

deficiency of Iodine

आयह८िीनकहॳ कारणकह९नसारह८गहह८ताहहॴ 2-Feb-2017

Options

1) Rickets

ररकहॳ टस

2) Scurvy

सकवी 3) Goitre

गणमािा 4) Growth retardation

पवकासका कना Correct Answer Goitre

rickets A softening and weakening of

bones in children usually due to

inadequate vitamin D

Q210 Grevillea Robusta is the scientific name of

गरहॳपवलियारह८बसटा mdashmdashmdash- कापवजञाननकनामहहॴ 2-Feb-2017

Options

1) Peepal

पीपि

2) Teak

सागह९न

3) Silver Oak

लसमवरओक

4) Jack fruit

कटहि

Correct Answer Silver Oak

Q211 When a Cuttlefish is described as a Molluscs it is at which level of

classification

जबएककटिकफिकह८एकमह८िसकाकहॳ पमवखणयतककयाजाताहहॴतबयहॳवगीकरणकहॳ ककससतरपहॳनसितहहॴ 2-Feb-2017

Options

1) Class

वगय 2) Order

िम

3) Family

पररवार

4) Phylum

सघ

Correct Answer Phylum

Q212 Bambusa dendrocalmus is the

scientific name of बानबसािहॳडराकामस mdashmdashmdash कावहॴजञाननकनामहहॴ 3-Feb-2017

Options

1) Banyan

बरगद

2) Papaya

पपीता 3) Bamboo

बास

4) Pomegranate

अनार

Correct Answer Bamboo

Q213 Acinonyx Jubatus is the scientific name of

एलसनह८ननकसजयबहॳटस mdashmdashmdash

कावहॴजञाननकनामहहॴ 3-Feb-2017

F A C E B O O K

P A G E h t t p w w w f a c e b o o k c o m s s c m e n t o r s o f f i c i a l P a g e | 49

FOR MORE UPDATES AND MORE MATERIAL DO LIKE OUR FACEBOOK PAGE httpwwwfacebookcomsscmentorsofficial

Options

1) Bear

भाि 2) Horse

घह८िा 3) Cheetah

चीता 4) Zebra

जहॳिा Correct Answer Cheetah

Q214 The pale yellow colour of urine is

due to the presence of which pigment

मतरकाफीकापीिारगरगदरयकहॳ उपनसिनतकहॳ कारणहह८ताहहॴ

3-Feb-2017

Options

1) Urochrome

यरह८िह८म

2) Urophyll

यरह८कफि

3) Chlorophyll

किह८रह८कफि

4) Chloroplast

किह८रह८पिासट

Correct Answer Urochrome

Q215 Which of the following constitute

to form a gene

नननननलिखितमसहॳकह९नसीचीज़एकजीनकागठनकरतीहहॴ

3-Feb-2017

Options

1) Polynucleotides

पह८िीनयनकियह८टाईडस

2) Hydrocarbons

हाइडरह८काबोस

3) Lipoproteins

िाईपह८परह८टीनस

4) Lipids

लिपपडस

Correct Answer Polynucleotides

Polynucleotide molecule is a biopolymer

composed of 13 or more nucleotide

monomers covalently bonded in a chain

DNA (deoxyribonucleic acid) and RNA

(ribonucleic acid) are examples of

polynucleotides with distinct biological

function

Q216 Vertebrates belongs to the

phylum

रीढ़कीहडिीवािहॳपराणी mdashmdashmdash

परजानतकहॳ अतगायतआतहॳहहॴ 3-Feb-2017

Options

1) Arthropoda

आरह८पह८ड़ा 2) Annelida

एननलििा 3) Cnidaria

ननिहॳररया 4) Chordata

कह८िटा Correct Answer Chordata

Q217 Punica granatum is the scientific name of

पननकगरहॳनहॳटस mdashmdashmdash कावहॴजञाननकनामहहॴ 3-Feb-2017

Options

1) Custard Apple

सीताफि

2) Gulmohar

गिमह८हर

3) Silver Oak

लसमवरओक

4) Pomegranate

अनार

Correct Answer Pomegranate

F A C E B O O K

P A G E h t t p w w w f a c e b o o k c o m s s c m e n t o r s o f f i c i a l P a g e | 50

FOR MORE UPDATES AND MORE MATERIAL DO LIKE OUR FACEBOOK PAGE httpwwwfacebookcomsscmentorsofficial

Q218 Between a tiger and an monkey

which of the following is different

एकबाघऔरबदरकहॳ बीचनननननलिखितमसहॳकह९नसीबातअिगहहॴ 3-Feb-2017

Options

1) Kingdom

राजय

2) Phylum

जानत

3) Order

िम

4) Class

वगय Correct Answer order

Q219 The artificial heart was invented by

कबतरमहदयका mdashmdashmdash

दवाराअपवषकारककयागयािा 3-Feb-2017

Options

1) Muhammad Yunus

महनमदयनस

2) Linus Yale Jr

िाइनसयहॳिजय

3) Gazi Yasargil

गाजीयासचगयि

4) Paul Winchell

पह९िपवमकि Correct Answer Paul Winchell

Q220 Tamarindus indica is the

scientific name of

टहॳमररनडसइडिका mdashmdash कावहॴजञाननकनामहहॴ 7-

Feb-2017

Options

1) Neem

नीम

2) Pineapple

अनानास

3) Tamarind

इमिी 4)Chiku

चीक

Correct Answer Tamarind

Q221 In eukaryotic cells synthesis of

RNA takes place in the

यकहॳ योटटककह८लिकाओमआरएनएकासशिहॳषण

mdashndash महह८ताहहॴ 7-Feb-2017

Options

1) Mitochondria

माईटह८कोडडरया 2) Centrioles

सटरीयह८मस

3) Ribosomes

ररबह८सह८नस

4) Nucleus

नयनकियस

Correct Answer nucleus

eukaryotic cell -Transcription is the

process of synthesizing ribonucleic acid

(RNA)Synthesis takes place within the

nucleus of eukaryotic cells or in the

cytoplasm of prokaryotes and converts

the genetic code from a gene in

deoxyribonucleic acid ( DNA ) to a

strand of RNA that then directs

proteinsynthesis

Q222 _________is caused by parasites

of the Plasmodium genus

पिाजमह८डियमजातीकहॳ परजीवी mdash- कहॳ कारणहहॴ 7-Feb-2017

Options

1) Dysentery

पहॳचचि

2) Malaria

मिहॳररया 3) Chickenpox

F A C E B O O K

P A G E h t t p w w w f a c e b o o k c o m s s c m e n t o r s o f f i c i a l P a g e | 51

FOR MORE UPDATES AND MORE MATERIAL DO LIKE OUR FACEBOOK PAGE httpwwwfacebookcomsscmentorsofficial

चहॳचक

4) Herpes

हहॳपपयस

Correct Answer Malaria

Q223 Carotene in fruits and vegetables

gives it which color

फिह८औरसनलजयोमनसितकहॳ रह८टीनउनहकह९नसारगपरदानकरताहहॴ 7-Feb-2017

Options

1) Green

हरा 2) Pink

गिाबी 3) Orange

नारगी 4) Blue

नीिा Correct Answer Orange

Q224 Equus Caballus is the scientific

name of

एकवसकहॴ बहॳिस mdashmdashndash कापवजञाननकनामहहॴ 7-Feb-2017

Options

1) Horse

घह८िा 2) Zebra

ज़हॳिा 3) Donkey

गधा 4) Buffalo

भस

Correct Answer Horse

Q225 Elapidae Naja is the scientific name of

एिीपीिीनाजा mdashmdash- कावहॴजञाननकनामहहॴ 8-Feb-2017

Options

1) Cobra

कह८बरा 2) Elephant

हािी 3) Eagle

ग ि

4) Owl

उमि Correct Answer Cobra

Q226 Which disease is caused due to

deficiency of Iron

िह८हकीकमीकहॳ कारणकह९नसारह८गहह८ताहहॴ 8-Feb-

2017

Options

1) Beriberi

बहॳरीबहॳरी 2) Tetany

टहॳटनी 3) Kwashiorkor

कवािीऔरकर

4) Anaemia

रकतामपता Correct Answer Anaemia

Beriberi is a disease caused by a vitamin

B-1 deficiency also known as thiamine

deficiency

Tetany can be the result of an

electrolyte imbalance Most often itrsquos a

dramatically low calcium level also

known as hypocalcemia Tetany can also

be caused by magnesium deficiency or

too little potassium Having too much

acid (acidosis) or too much alkali

(alkalosis) in the body can also result in

tetany

Kwashiorkor also known as

ldquoedematous malnutrition It is a form of

malnutrition caused by a lack of protein

in the diet

Anaemia means that you have fewer red

blood cells than normal or you have less

F A C E B O O K

P A G E h t t p w w w f a c e b o o k c o m s s c m e n t o r s o f f i c i a l P a g e | 52

FOR MORE UPDATES AND MORE MATERIAL DO LIKE OUR FACEBOOK PAGE httpwwwfacebookcomsscmentorsofficial

haemoglobin than normal in each red

blood cell

Q227 is a leaf where the leaflets are

arranged along the middle vein

mdashndashएकपततीहहॴजहापतरकह८कीरचनाक ररयालिराकहॳ आसपासहह८तीहहॴ 8-Feb-2017

Options

1) Pinnately compound leaf

पपनहॳटिीसयकतपतती 2) Palmately compound leaf

पामहॳटिीसयकतपतती 3) Compound leaf

सयकतपतती 4) Simple leaf

साधारणपतती Correct Answer Pinnately compound

leaf

Q228 Haustoria or sucking roots are

found in which of the following

हह८सटह८ररयायाचसनहॳवािीजड़हॳनननननलिखितमसहॳककसमपाईजातीहहॴ 8-Feb-2017

Options

1) Wheat

गहॳह

2) Mango

आम

3) Chestnut

चहॳसटनट

4) Cuscuta

कसकयटा Correct Answer Cuscuta

Haustorial roots -The roots of parasitic

plants which penetrate into the host

tissues to absorb nourishment are

called haustorial roots hellip Also known as suckingor parasitic roots

Q229 Equs Asinus is the scientific name

of

एकवसएलसनस mdashmdashndash कावहॴजञाननकनामहहॴ 8-

Feb-2017

Options

1) Donkey

गधा 2) Cow

गाय

3) Deer

टहरन

4) Kangaroo

कगा

Correct Answer Donkey

Q230 Ficus benghalensis is the scientific name of

फाईकसबहॳनगहॳिहॳलसस mdashndash कापवजञाननकनामहहॴ 8-Feb-2017

Options

1) Banyan

बरगद

2) Pineapple

अनानास

3) Babul

बबि

4) Tulsi

तिसी Correct Answer Banyan

Q231 Equus burchellii is the scientific name of

एकवसबचिी mdashmdash- कापवजञाननकनामहहॴ 8-Feb-2017

Options

1) Horse

घह८िा 2) Zebra

जहॳिा 3) Buffalo

F A C E B O O K

P A G E h t t p w w w f a c e b o o k c o m s s c m e n t o r s o f f i c i a l P a g e | 53

FOR MORE UPDATES AND MORE MATERIAL DO LIKE OUR FACEBOOK PAGE httpwwwfacebookcomsscmentorsofficial

भस

4) Ass

गधा Correct Answer Zebra

Page 46: COMPILATION OF ALL 72 SETS OF BIOLOGY SSC CHSL-2016 · OF BIOLOGY SSC CHSL-2016 PREPARED BY : SSC MENTORS BIOLOGY SPECIAL . F A C E B O O K P A G E : h t t p : / / w w w . f a c e

F A C E B O O K

P A G E h t t p w w w f a c e b o o k c o m s s c m e n t o r s o f f i c i a l P a g e | 45

FOR MORE UPDATES AND MORE MATERIAL DO LIKE OUR FACEBOOK PAGE httpwwwfacebookcomsscmentorsofficial

4) Drumstick

डरमनसटक

Correct Answer Drumstick

Q195 Kidney stones are composed of

गदकीपिरी mdashndash सहॳबनीहह८तीहहॴ 1-Feb-2017

Options

1) Calcium Oxalate

कहॴ नमसयमओकजहॳिहॳट

2) Sodium Chloride

सह८डियमकिह८राइि

3) Magnesium Nitrate

महॳनगनलियमनाइतटरहॳट

4) Calcium Bicarbonate

कहॴ नमियमबायकबोनहॳट

Correct Answer Calcium Oxalate

Q196 ndash Which of the following is not

true about Angiosperms

ननननमसहॳकह९नसीबातआवतबीजीकहॳ बारहॳमसचनहीहहॴ 1-Feb-2017

Options

1) Dominant phase is gametophytes

परमिचरणगहॳलमतह८फाइटहह८ताहहॴ 2) Vascular bundles are present

सवहनीबििमह९जदहह८ताहहॴ 3) Spores are heterospores

बीजाणहहॳटहॳरह८सपह८रसहह८तहॳहहॴ 4) Seeds are covered

बीजढकहॳ हह८तहॳहहॴ Correct Answer Dominant phase is

gametophytes

Q197 All of the following are excretory

(waste) products of animals except

नननननलिखितमसहॳककसएककह८छह८ड़करअनयसभीपराखणयोदवाराउतसनजयतपदाियहहॴ 1-Feb-

2017

Options

1) Uric Acid

यररकएलसि

2) Ammonia

अमह८ननया 3) Carbohydrates

काबोहाइडरहॳट

4) Urea

यररया Correct Answer Carbohydrates

In animals the main excretory products

are carbon dioxide ammonia (in

ammoniotelics) urea (in ureotelics) uric

acid (in uricotelics) guanine (in

Arachnida) and creatine

Q198 RNA is a polymeric molecule

What does RNA stand for

आरएनइएएकबहिकआणहहॴ इसकाकापवय पकयाहहॴ 1-Feb-2017

Options

1) Rado Nuclear Acid

रािह८नयनकियरएलसि

2) Ribo Nucleic Acid

राइबह८नयनकिकएलसि

3) Rhino Nuclear Acid

हाइनह८नयनकियरएलसि

4) Resto Nucleus Acid

रहॳसटह८नयकिीयसएलसि

Correct Answer Ribo Nucleic Acid

Q199 Which organ does detoxification

and produces chemicals needed for

digestion

कह९नसाअगपवषहरणकरताहहॴऔरपाचनकहॳ लिएआवशयकरसायनोकह८पहॴदाकरताहहॴ 1-Feb-

2017

Options

1) Salivary glands

िारगरचिया 2) Pancreas

अगनयािय

F A C E B O O K

P A G E h t t p w w w f a c e b o o k c o m s s c m e n t o r s o f f i c i a l P a g e | 46

FOR MORE UPDATES AND MORE MATERIAL DO LIKE OUR FACEBOOK PAGE httpwwwfacebookcomsscmentorsofficial

3) Thyroid gland

िायराइिगरिी 4) Liver

यकत

Correct Answer Liver

Q200 Psidium guajava is the scientific

name of

लसडियमगआजावा mdashmdash कावहॴजञाननकनामहहॴ 1-

Feb-2017

Options

1) Guava

अम द

2) Mango

आम

3) Bamboo

बास

4) Jack fruit

कटहि

Correct Answer Guava

Q201 Which drug is used as a Blood

Thinner

चधरकह८पतिाकरनहॳकहॳ पमककसदवाकापरयह८गककयाजाताहहॴ

1-Feb-2017

Options

1) Warfarin

वाफर न

2) Tramadol

टरहॳमािह८ि

3) Azithromycin

एनजरह८मायलसन

4) Hydralazine

हाइडरह८िहॳनजन

Correct Answer Warfarin

Q202 Which of the following disease is

caused due to the deficiency of protein

परह८टीनकीकमीकहॳ कारणनननननलिखितमसहॳकह९नसारह८गहह८ताहहॴ 1-Feb-2017

Options

1) Arthritis

गटठया 2) Kwashiorkor

कािीओकय र

3) Goitre

गाइटर

4) Night Blindness

रतह९चध

Correct Answer Kwashiorkor

Q203 A is species of plant that has

adapted to survive in an environment

with little liquid water

mdashmdashndashपह९धहॳकीएकऐसहॳऐसहॳपरजानतहहॴ नजसनहॳकमपानीवािहॳवातावरणमजीपवतरहनहॳकहॳलिएअनकिनहहॴ 1-Feb-2017

Options

1) Xerophyte

म दपवद

2) Hydrophyte

जिीयपादप

3) Mesophyte

समह८दपवद

4) Thallophyte

िहॴिह८फाइटा Correct Answer xerophyte

xerophyte is a species of plant that has

adapted to survive in an environment

with little liquid water such as a desert

or an ice- or snow-covered region in the

Alps or the Arctic

Mesophytes are terrestrial plants which

are adapted to neither a particularly

dry nor particularly wet environment

An example of a mesophytic habitat

would be a rural temperate meadow

F A C E B O O K

P A G E h t t p w w w f a c e b o o k c o m s s c m e n t o r s o f f i c i a l P a g e | 47

FOR MORE UPDATES AND MORE MATERIAL DO LIKE OUR FACEBOOK PAGE httpwwwfacebookcomsscmentorsofficial

which might contain goldenrod clover

oxeye daisy and Rosa multiflora

thallophyte any of a group of plants or

plantlike organisms (such as algae and

fungi) that lack differentiated stems

leaves and roots and that were formerly

classified as a primary division

(Thallophyta) of the plant kingdom

Q204 How many types of teeth are

there in humans

मनषयोमककतनहॳपरकारकहॳ दातहह८तहॳहहॴ

1-Feb-2017

Options

1) 4

2) 5

3) 2

4) 3

Correct Answer 4

teeth -Humans have four types of

teethincisors canines premolars and

molars each with a specific function

The incisors cut the food the canines

tear the food and the molars and

premolars crush the food

Q205 Carica papaya is the scientific name of

कहॴ ररकापपाया mdashmdashndash कावहॴजञाननकनामहहॴ 2-

Feb-2017

Options

1) Peepal

पीपि

2) Papaya

पपीता 3) Tamarind

इमिी 4) Drumstick

ढह८िकाछड़ी Correct Answer Papaya

Q206 Muscles get tired when there is

shortfall of

जब mdashndash कीकमीहह८तीहहॴतबपहॳिीयिकजातीहहॴ 2-Feb-2017

Options

1) Lactic acid

िहॴनकटकएलसि

2) Na+ ions

Na+ आयन

3) ATP

एटीपी 4) Sulphates

समफहॳ टस

Correct Answer ATP

ATP is the energy source muscle fibers

use to make muscles contract

muscle tissuersquos main source of energy

called adenosine triphosphate or ATP

As your muscles use up this energy

source they become tired and fatigued

Oxygen is the key ingredient that helps

create new ATP to replenish the burned

up ATP in your muscles

Q207 Artocarpus integra is the

scientific name of आटह८कापयसइटीगरा mdashmdashmdash कावहॴजञाननकनामहहॴ 2-Feb-2017

Options

1) Guava

अम द

2) Pineapple

अनानास

3) Silver Oak

लसमवरओक

4) Jack fruit

कटहि

Correct Answer Jack fruit

Q208 Which organ stores fat soluble

vitamins

कह९नसाअगवसामघिनिीिपवटालमनह८काभिाराकरताहहॴ

2-Feb-2017

F A C E B O O K

P A G E h t t p w w w f a c e b o o k c o m s s c m e n t o r s o f f i c i a l P a g e | 48

FOR MORE UPDATES AND MORE MATERIAL DO LIKE OUR FACEBOOK PAGE httpwwwfacebookcomsscmentorsofficial

Options

1) Blood

रकत

2) Skin

तवचा 3) Liver

यकत

4) Pancreas

अगनयािय

Correct Answer Liver

Q209 Which disease is caused due to

deficiency of Iodine

आयह८िीनकहॳ कारणकह९नसारह८गहह८ताहहॴ 2-Feb-2017

Options

1) Rickets

ररकहॳ टस

2) Scurvy

सकवी 3) Goitre

गणमािा 4) Growth retardation

पवकासका कना Correct Answer Goitre

rickets A softening and weakening of

bones in children usually due to

inadequate vitamin D

Q210 Grevillea Robusta is the scientific name of

गरहॳपवलियारह८बसटा mdashmdashmdash- कापवजञाननकनामहहॴ 2-Feb-2017

Options

1) Peepal

पीपि

2) Teak

सागह९न

3) Silver Oak

लसमवरओक

4) Jack fruit

कटहि

Correct Answer Silver Oak

Q211 When a Cuttlefish is described as a Molluscs it is at which level of

classification

जबएककटिकफिकह८एकमह८िसकाकहॳ पमवखणयतककयाजाताहहॴतबयहॳवगीकरणकहॳ ककससतरपहॳनसितहहॴ 2-Feb-2017

Options

1) Class

वगय 2) Order

िम

3) Family

पररवार

4) Phylum

सघ

Correct Answer Phylum

Q212 Bambusa dendrocalmus is the

scientific name of बानबसािहॳडराकामस mdashmdashmdash कावहॴजञाननकनामहहॴ 3-Feb-2017

Options

1) Banyan

बरगद

2) Papaya

पपीता 3) Bamboo

बास

4) Pomegranate

अनार

Correct Answer Bamboo

Q213 Acinonyx Jubatus is the scientific name of

एलसनह८ननकसजयबहॳटस mdashmdashmdash

कावहॴजञाननकनामहहॴ 3-Feb-2017

F A C E B O O K

P A G E h t t p w w w f a c e b o o k c o m s s c m e n t o r s o f f i c i a l P a g e | 49

FOR MORE UPDATES AND MORE MATERIAL DO LIKE OUR FACEBOOK PAGE httpwwwfacebookcomsscmentorsofficial

Options

1) Bear

भाि 2) Horse

घह८िा 3) Cheetah

चीता 4) Zebra

जहॳिा Correct Answer Cheetah

Q214 The pale yellow colour of urine is

due to the presence of which pigment

मतरकाफीकापीिारगरगदरयकहॳ उपनसिनतकहॳ कारणहह८ताहहॴ

3-Feb-2017

Options

1) Urochrome

यरह८िह८म

2) Urophyll

यरह८कफि

3) Chlorophyll

किह८रह८कफि

4) Chloroplast

किह८रह८पिासट

Correct Answer Urochrome

Q215 Which of the following constitute

to form a gene

नननननलिखितमसहॳकह९नसीचीज़एकजीनकागठनकरतीहहॴ

3-Feb-2017

Options

1) Polynucleotides

पह८िीनयनकियह८टाईडस

2) Hydrocarbons

हाइडरह८काबोस

3) Lipoproteins

िाईपह८परह८टीनस

4) Lipids

लिपपडस

Correct Answer Polynucleotides

Polynucleotide molecule is a biopolymer

composed of 13 or more nucleotide

monomers covalently bonded in a chain

DNA (deoxyribonucleic acid) and RNA

(ribonucleic acid) are examples of

polynucleotides with distinct biological

function

Q216 Vertebrates belongs to the

phylum

रीढ़कीहडिीवािहॳपराणी mdashmdashmdash

परजानतकहॳ अतगायतआतहॳहहॴ 3-Feb-2017

Options

1) Arthropoda

आरह८पह८ड़ा 2) Annelida

एननलििा 3) Cnidaria

ननिहॳररया 4) Chordata

कह८िटा Correct Answer Chordata

Q217 Punica granatum is the scientific name of

पननकगरहॳनहॳटस mdashmdashmdash कावहॴजञाननकनामहहॴ 3-Feb-2017

Options

1) Custard Apple

सीताफि

2) Gulmohar

गिमह८हर

3) Silver Oak

लसमवरओक

4) Pomegranate

अनार

Correct Answer Pomegranate

F A C E B O O K

P A G E h t t p w w w f a c e b o o k c o m s s c m e n t o r s o f f i c i a l P a g e | 50

FOR MORE UPDATES AND MORE MATERIAL DO LIKE OUR FACEBOOK PAGE httpwwwfacebookcomsscmentorsofficial

Q218 Between a tiger and an monkey

which of the following is different

एकबाघऔरबदरकहॳ बीचनननननलिखितमसहॳकह९नसीबातअिगहहॴ 3-Feb-2017

Options

1) Kingdom

राजय

2) Phylum

जानत

3) Order

िम

4) Class

वगय Correct Answer order

Q219 The artificial heart was invented by

कबतरमहदयका mdashmdashmdash

दवाराअपवषकारककयागयािा 3-Feb-2017

Options

1) Muhammad Yunus

महनमदयनस

2) Linus Yale Jr

िाइनसयहॳिजय

3) Gazi Yasargil

गाजीयासचगयि

4) Paul Winchell

पह९िपवमकि Correct Answer Paul Winchell

Q220 Tamarindus indica is the

scientific name of

टहॳमररनडसइडिका mdashmdash कावहॴजञाननकनामहहॴ 7-

Feb-2017

Options

1) Neem

नीम

2) Pineapple

अनानास

3) Tamarind

इमिी 4)Chiku

चीक

Correct Answer Tamarind

Q221 In eukaryotic cells synthesis of

RNA takes place in the

यकहॳ योटटककह८लिकाओमआरएनएकासशिहॳषण

mdashndash महह८ताहहॴ 7-Feb-2017

Options

1) Mitochondria

माईटह८कोडडरया 2) Centrioles

सटरीयह८मस

3) Ribosomes

ररबह८सह८नस

4) Nucleus

नयनकियस

Correct Answer nucleus

eukaryotic cell -Transcription is the

process of synthesizing ribonucleic acid

(RNA)Synthesis takes place within the

nucleus of eukaryotic cells or in the

cytoplasm of prokaryotes and converts

the genetic code from a gene in

deoxyribonucleic acid ( DNA ) to a

strand of RNA that then directs

proteinsynthesis

Q222 _________is caused by parasites

of the Plasmodium genus

पिाजमह८डियमजातीकहॳ परजीवी mdash- कहॳ कारणहहॴ 7-Feb-2017

Options

1) Dysentery

पहॳचचि

2) Malaria

मिहॳररया 3) Chickenpox

F A C E B O O K

P A G E h t t p w w w f a c e b o o k c o m s s c m e n t o r s o f f i c i a l P a g e | 51

FOR MORE UPDATES AND MORE MATERIAL DO LIKE OUR FACEBOOK PAGE httpwwwfacebookcomsscmentorsofficial

चहॳचक

4) Herpes

हहॳपपयस

Correct Answer Malaria

Q223 Carotene in fruits and vegetables

gives it which color

फिह८औरसनलजयोमनसितकहॳ रह८टीनउनहकह९नसारगपरदानकरताहहॴ 7-Feb-2017

Options

1) Green

हरा 2) Pink

गिाबी 3) Orange

नारगी 4) Blue

नीिा Correct Answer Orange

Q224 Equus Caballus is the scientific

name of

एकवसकहॴ बहॳिस mdashmdashndash कापवजञाननकनामहहॴ 7-Feb-2017

Options

1) Horse

घह८िा 2) Zebra

ज़हॳिा 3) Donkey

गधा 4) Buffalo

भस

Correct Answer Horse

Q225 Elapidae Naja is the scientific name of

एिीपीिीनाजा mdashmdash- कावहॴजञाननकनामहहॴ 8-Feb-2017

Options

1) Cobra

कह८बरा 2) Elephant

हािी 3) Eagle

ग ि

4) Owl

उमि Correct Answer Cobra

Q226 Which disease is caused due to

deficiency of Iron

िह८हकीकमीकहॳ कारणकह९नसारह८गहह८ताहहॴ 8-Feb-

2017

Options

1) Beriberi

बहॳरीबहॳरी 2) Tetany

टहॳटनी 3) Kwashiorkor

कवािीऔरकर

4) Anaemia

रकतामपता Correct Answer Anaemia

Beriberi is a disease caused by a vitamin

B-1 deficiency also known as thiamine

deficiency

Tetany can be the result of an

electrolyte imbalance Most often itrsquos a

dramatically low calcium level also

known as hypocalcemia Tetany can also

be caused by magnesium deficiency or

too little potassium Having too much

acid (acidosis) or too much alkali

(alkalosis) in the body can also result in

tetany

Kwashiorkor also known as

ldquoedematous malnutrition It is a form of

malnutrition caused by a lack of protein

in the diet

Anaemia means that you have fewer red

blood cells than normal or you have less

F A C E B O O K

P A G E h t t p w w w f a c e b o o k c o m s s c m e n t o r s o f f i c i a l P a g e | 52

FOR MORE UPDATES AND MORE MATERIAL DO LIKE OUR FACEBOOK PAGE httpwwwfacebookcomsscmentorsofficial

haemoglobin than normal in each red

blood cell

Q227 is a leaf where the leaflets are

arranged along the middle vein

mdashndashएकपततीहहॴजहापतरकह८कीरचनाक ररयालिराकहॳ आसपासहह८तीहहॴ 8-Feb-2017

Options

1) Pinnately compound leaf

पपनहॳटिीसयकतपतती 2) Palmately compound leaf

पामहॳटिीसयकतपतती 3) Compound leaf

सयकतपतती 4) Simple leaf

साधारणपतती Correct Answer Pinnately compound

leaf

Q228 Haustoria or sucking roots are

found in which of the following

हह८सटह८ररयायाचसनहॳवािीजड़हॳनननननलिखितमसहॳककसमपाईजातीहहॴ 8-Feb-2017

Options

1) Wheat

गहॳह

2) Mango

आम

3) Chestnut

चहॳसटनट

4) Cuscuta

कसकयटा Correct Answer Cuscuta

Haustorial roots -The roots of parasitic

plants which penetrate into the host

tissues to absorb nourishment are

called haustorial roots hellip Also known as suckingor parasitic roots

Q229 Equs Asinus is the scientific name

of

एकवसएलसनस mdashmdashndash कावहॴजञाननकनामहहॴ 8-

Feb-2017

Options

1) Donkey

गधा 2) Cow

गाय

3) Deer

टहरन

4) Kangaroo

कगा

Correct Answer Donkey

Q230 Ficus benghalensis is the scientific name of

फाईकसबहॳनगहॳिहॳलसस mdashndash कापवजञाननकनामहहॴ 8-Feb-2017

Options

1) Banyan

बरगद

2) Pineapple

अनानास

3) Babul

बबि

4) Tulsi

तिसी Correct Answer Banyan

Q231 Equus burchellii is the scientific name of

एकवसबचिी mdashmdash- कापवजञाननकनामहहॴ 8-Feb-2017

Options

1) Horse

घह८िा 2) Zebra

जहॳिा 3) Buffalo

F A C E B O O K

P A G E h t t p w w w f a c e b o o k c o m s s c m e n t o r s o f f i c i a l P a g e | 53

FOR MORE UPDATES AND MORE MATERIAL DO LIKE OUR FACEBOOK PAGE httpwwwfacebookcomsscmentorsofficial

भस

4) Ass

गधा Correct Answer Zebra

Page 47: COMPILATION OF ALL 72 SETS OF BIOLOGY SSC CHSL-2016 · OF BIOLOGY SSC CHSL-2016 PREPARED BY : SSC MENTORS BIOLOGY SPECIAL . F A C E B O O K P A G E : h t t p : / / w w w . f a c e

F A C E B O O K

P A G E h t t p w w w f a c e b o o k c o m s s c m e n t o r s o f f i c i a l P a g e | 46

FOR MORE UPDATES AND MORE MATERIAL DO LIKE OUR FACEBOOK PAGE httpwwwfacebookcomsscmentorsofficial

3) Thyroid gland

िायराइिगरिी 4) Liver

यकत

Correct Answer Liver

Q200 Psidium guajava is the scientific

name of

लसडियमगआजावा mdashmdash कावहॴजञाननकनामहहॴ 1-

Feb-2017

Options

1) Guava

अम द

2) Mango

आम

3) Bamboo

बास

4) Jack fruit

कटहि

Correct Answer Guava

Q201 Which drug is used as a Blood

Thinner

चधरकह८पतिाकरनहॳकहॳ पमककसदवाकापरयह८गककयाजाताहहॴ

1-Feb-2017

Options

1) Warfarin

वाफर न

2) Tramadol

टरहॳमािह८ि

3) Azithromycin

एनजरह८मायलसन

4) Hydralazine

हाइडरह८िहॳनजन

Correct Answer Warfarin

Q202 Which of the following disease is

caused due to the deficiency of protein

परह८टीनकीकमीकहॳ कारणनननननलिखितमसहॳकह९नसारह८गहह८ताहहॴ 1-Feb-2017

Options

1) Arthritis

गटठया 2) Kwashiorkor

कािीओकय र

3) Goitre

गाइटर

4) Night Blindness

रतह९चध

Correct Answer Kwashiorkor

Q203 A is species of plant that has

adapted to survive in an environment

with little liquid water

mdashmdashndashपह९धहॳकीएकऐसहॳऐसहॳपरजानतहहॴ नजसनहॳकमपानीवािहॳवातावरणमजीपवतरहनहॳकहॳलिएअनकिनहहॴ 1-Feb-2017

Options

1) Xerophyte

म दपवद

2) Hydrophyte

जिीयपादप

3) Mesophyte

समह८दपवद

4) Thallophyte

िहॴिह८फाइटा Correct Answer xerophyte

xerophyte is a species of plant that has

adapted to survive in an environment

with little liquid water such as a desert

or an ice- or snow-covered region in the

Alps or the Arctic

Mesophytes are terrestrial plants which

are adapted to neither a particularly

dry nor particularly wet environment

An example of a mesophytic habitat

would be a rural temperate meadow

F A C E B O O K

P A G E h t t p w w w f a c e b o o k c o m s s c m e n t o r s o f f i c i a l P a g e | 47

FOR MORE UPDATES AND MORE MATERIAL DO LIKE OUR FACEBOOK PAGE httpwwwfacebookcomsscmentorsofficial

which might contain goldenrod clover

oxeye daisy and Rosa multiflora

thallophyte any of a group of plants or

plantlike organisms (such as algae and

fungi) that lack differentiated stems

leaves and roots and that were formerly

classified as a primary division

(Thallophyta) of the plant kingdom

Q204 How many types of teeth are

there in humans

मनषयोमककतनहॳपरकारकहॳ दातहह८तहॳहहॴ

1-Feb-2017

Options

1) 4

2) 5

3) 2

4) 3

Correct Answer 4

teeth -Humans have four types of

teethincisors canines premolars and

molars each with a specific function

The incisors cut the food the canines

tear the food and the molars and

premolars crush the food

Q205 Carica papaya is the scientific name of

कहॴ ररकापपाया mdashmdashndash कावहॴजञाननकनामहहॴ 2-

Feb-2017

Options

1) Peepal

पीपि

2) Papaya

पपीता 3) Tamarind

इमिी 4) Drumstick

ढह८िकाछड़ी Correct Answer Papaya

Q206 Muscles get tired when there is

shortfall of

जब mdashndash कीकमीहह८तीहहॴतबपहॳिीयिकजातीहहॴ 2-Feb-2017

Options

1) Lactic acid

िहॴनकटकएलसि

2) Na+ ions

Na+ आयन

3) ATP

एटीपी 4) Sulphates

समफहॳ टस

Correct Answer ATP

ATP is the energy source muscle fibers

use to make muscles contract

muscle tissuersquos main source of energy

called adenosine triphosphate or ATP

As your muscles use up this energy

source they become tired and fatigued

Oxygen is the key ingredient that helps

create new ATP to replenish the burned

up ATP in your muscles

Q207 Artocarpus integra is the

scientific name of आटह८कापयसइटीगरा mdashmdashmdash कावहॴजञाननकनामहहॴ 2-Feb-2017

Options

1) Guava

अम द

2) Pineapple

अनानास

3) Silver Oak

लसमवरओक

4) Jack fruit

कटहि

Correct Answer Jack fruit

Q208 Which organ stores fat soluble

vitamins

कह९नसाअगवसामघिनिीिपवटालमनह८काभिाराकरताहहॴ

2-Feb-2017

F A C E B O O K

P A G E h t t p w w w f a c e b o o k c o m s s c m e n t o r s o f f i c i a l P a g e | 48

FOR MORE UPDATES AND MORE MATERIAL DO LIKE OUR FACEBOOK PAGE httpwwwfacebookcomsscmentorsofficial

Options

1) Blood

रकत

2) Skin

तवचा 3) Liver

यकत

4) Pancreas

अगनयािय

Correct Answer Liver

Q209 Which disease is caused due to

deficiency of Iodine

आयह८िीनकहॳ कारणकह९नसारह८गहह८ताहहॴ 2-Feb-2017

Options

1) Rickets

ररकहॳ टस

2) Scurvy

सकवी 3) Goitre

गणमािा 4) Growth retardation

पवकासका कना Correct Answer Goitre

rickets A softening and weakening of

bones in children usually due to

inadequate vitamin D

Q210 Grevillea Robusta is the scientific name of

गरहॳपवलियारह८बसटा mdashmdashmdash- कापवजञाननकनामहहॴ 2-Feb-2017

Options

1) Peepal

पीपि

2) Teak

सागह९न

3) Silver Oak

लसमवरओक

4) Jack fruit

कटहि

Correct Answer Silver Oak

Q211 When a Cuttlefish is described as a Molluscs it is at which level of

classification

जबएककटिकफिकह८एकमह८िसकाकहॳ पमवखणयतककयाजाताहहॴतबयहॳवगीकरणकहॳ ककससतरपहॳनसितहहॴ 2-Feb-2017

Options

1) Class

वगय 2) Order

िम

3) Family

पररवार

4) Phylum

सघ

Correct Answer Phylum

Q212 Bambusa dendrocalmus is the

scientific name of बानबसािहॳडराकामस mdashmdashmdash कावहॴजञाननकनामहहॴ 3-Feb-2017

Options

1) Banyan

बरगद

2) Papaya

पपीता 3) Bamboo

बास

4) Pomegranate

अनार

Correct Answer Bamboo

Q213 Acinonyx Jubatus is the scientific name of

एलसनह८ननकसजयबहॳटस mdashmdashmdash

कावहॴजञाननकनामहहॴ 3-Feb-2017

F A C E B O O K

P A G E h t t p w w w f a c e b o o k c o m s s c m e n t o r s o f f i c i a l P a g e | 49

FOR MORE UPDATES AND MORE MATERIAL DO LIKE OUR FACEBOOK PAGE httpwwwfacebookcomsscmentorsofficial

Options

1) Bear

भाि 2) Horse

घह८िा 3) Cheetah

चीता 4) Zebra

जहॳिा Correct Answer Cheetah

Q214 The pale yellow colour of urine is

due to the presence of which pigment

मतरकाफीकापीिारगरगदरयकहॳ उपनसिनतकहॳ कारणहह८ताहहॴ

3-Feb-2017

Options

1) Urochrome

यरह८िह८म

2) Urophyll

यरह८कफि

3) Chlorophyll

किह८रह८कफि

4) Chloroplast

किह८रह८पिासट

Correct Answer Urochrome

Q215 Which of the following constitute

to form a gene

नननननलिखितमसहॳकह९नसीचीज़एकजीनकागठनकरतीहहॴ

3-Feb-2017

Options

1) Polynucleotides

पह८िीनयनकियह८टाईडस

2) Hydrocarbons

हाइडरह८काबोस

3) Lipoproteins

िाईपह८परह८टीनस

4) Lipids

लिपपडस

Correct Answer Polynucleotides

Polynucleotide molecule is a biopolymer

composed of 13 or more nucleotide

monomers covalently bonded in a chain

DNA (deoxyribonucleic acid) and RNA

(ribonucleic acid) are examples of

polynucleotides with distinct biological

function

Q216 Vertebrates belongs to the

phylum

रीढ़कीहडिीवािहॳपराणी mdashmdashmdash

परजानतकहॳ अतगायतआतहॳहहॴ 3-Feb-2017

Options

1) Arthropoda

आरह८पह८ड़ा 2) Annelida

एननलििा 3) Cnidaria

ननिहॳररया 4) Chordata

कह८िटा Correct Answer Chordata

Q217 Punica granatum is the scientific name of

पननकगरहॳनहॳटस mdashmdashmdash कावहॴजञाननकनामहहॴ 3-Feb-2017

Options

1) Custard Apple

सीताफि

2) Gulmohar

गिमह८हर

3) Silver Oak

लसमवरओक

4) Pomegranate

अनार

Correct Answer Pomegranate

F A C E B O O K

P A G E h t t p w w w f a c e b o o k c o m s s c m e n t o r s o f f i c i a l P a g e | 50

FOR MORE UPDATES AND MORE MATERIAL DO LIKE OUR FACEBOOK PAGE httpwwwfacebookcomsscmentorsofficial

Q218 Between a tiger and an monkey

which of the following is different

एकबाघऔरबदरकहॳ बीचनननननलिखितमसहॳकह९नसीबातअिगहहॴ 3-Feb-2017

Options

1) Kingdom

राजय

2) Phylum

जानत

3) Order

िम

4) Class

वगय Correct Answer order

Q219 The artificial heart was invented by

कबतरमहदयका mdashmdashmdash

दवाराअपवषकारककयागयािा 3-Feb-2017

Options

1) Muhammad Yunus

महनमदयनस

2) Linus Yale Jr

िाइनसयहॳिजय

3) Gazi Yasargil

गाजीयासचगयि

4) Paul Winchell

पह९िपवमकि Correct Answer Paul Winchell

Q220 Tamarindus indica is the

scientific name of

टहॳमररनडसइडिका mdashmdash कावहॴजञाननकनामहहॴ 7-

Feb-2017

Options

1) Neem

नीम

2) Pineapple

अनानास

3) Tamarind

इमिी 4)Chiku

चीक

Correct Answer Tamarind

Q221 In eukaryotic cells synthesis of

RNA takes place in the

यकहॳ योटटककह८लिकाओमआरएनएकासशिहॳषण

mdashndash महह८ताहहॴ 7-Feb-2017

Options

1) Mitochondria

माईटह८कोडडरया 2) Centrioles

सटरीयह८मस

3) Ribosomes

ररबह८सह८नस

4) Nucleus

नयनकियस

Correct Answer nucleus

eukaryotic cell -Transcription is the

process of synthesizing ribonucleic acid

(RNA)Synthesis takes place within the

nucleus of eukaryotic cells or in the

cytoplasm of prokaryotes and converts

the genetic code from a gene in

deoxyribonucleic acid ( DNA ) to a

strand of RNA that then directs

proteinsynthesis

Q222 _________is caused by parasites

of the Plasmodium genus

पिाजमह८डियमजातीकहॳ परजीवी mdash- कहॳ कारणहहॴ 7-Feb-2017

Options

1) Dysentery

पहॳचचि

2) Malaria

मिहॳररया 3) Chickenpox

F A C E B O O K

P A G E h t t p w w w f a c e b o o k c o m s s c m e n t o r s o f f i c i a l P a g e | 51

FOR MORE UPDATES AND MORE MATERIAL DO LIKE OUR FACEBOOK PAGE httpwwwfacebookcomsscmentorsofficial

चहॳचक

4) Herpes

हहॳपपयस

Correct Answer Malaria

Q223 Carotene in fruits and vegetables

gives it which color

फिह८औरसनलजयोमनसितकहॳ रह८टीनउनहकह९नसारगपरदानकरताहहॴ 7-Feb-2017

Options

1) Green

हरा 2) Pink

गिाबी 3) Orange

नारगी 4) Blue

नीिा Correct Answer Orange

Q224 Equus Caballus is the scientific

name of

एकवसकहॴ बहॳिस mdashmdashndash कापवजञाननकनामहहॴ 7-Feb-2017

Options

1) Horse

घह८िा 2) Zebra

ज़हॳिा 3) Donkey

गधा 4) Buffalo

भस

Correct Answer Horse

Q225 Elapidae Naja is the scientific name of

एिीपीिीनाजा mdashmdash- कावहॴजञाननकनामहहॴ 8-Feb-2017

Options

1) Cobra

कह८बरा 2) Elephant

हािी 3) Eagle

ग ि

4) Owl

उमि Correct Answer Cobra

Q226 Which disease is caused due to

deficiency of Iron

िह८हकीकमीकहॳ कारणकह९नसारह८गहह८ताहहॴ 8-Feb-

2017

Options

1) Beriberi

बहॳरीबहॳरी 2) Tetany

टहॳटनी 3) Kwashiorkor

कवािीऔरकर

4) Anaemia

रकतामपता Correct Answer Anaemia

Beriberi is a disease caused by a vitamin

B-1 deficiency also known as thiamine

deficiency

Tetany can be the result of an

electrolyte imbalance Most often itrsquos a

dramatically low calcium level also

known as hypocalcemia Tetany can also

be caused by magnesium deficiency or

too little potassium Having too much

acid (acidosis) or too much alkali

(alkalosis) in the body can also result in

tetany

Kwashiorkor also known as

ldquoedematous malnutrition It is a form of

malnutrition caused by a lack of protein

in the diet

Anaemia means that you have fewer red

blood cells than normal or you have less

F A C E B O O K

P A G E h t t p w w w f a c e b o o k c o m s s c m e n t o r s o f f i c i a l P a g e | 52

FOR MORE UPDATES AND MORE MATERIAL DO LIKE OUR FACEBOOK PAGE httpwwwfacebookcomsscmentorsofficial

haemoglobin than normal in each red

blood cell

Q227 is a leaf where the leaflets are

arranged along the middle vein

mdashndashएकपततीहहॴजहापतरकह८कीरचनाक ररयालिराकहॳ आसपासहह८तीहहॴ 8-Feb-2017

Options

1) Pinnately compound leaf

पपनहॳटिीसयकतपतती 2) Palmately compound leaf

पामहॳटिीसयकतपतती 3) Compound leaf

सयकतपतती 4) Simple leaf

साधारणपतती Correct Answer Pinnately compound

leaf

Q228 Haustoria or sucking roots are

found in which of the following

हह८सटह८ररयायाचसनहॳवािीजड़हॳनननननलिखितमसहॳककसमपाईजातीहहॴ 8-Feb-2017

Options

1) Wheat

गहॳह

2) Mango

आम

3) Chestnut

चहॳसटनट

4) Cuscuta

कसकयटा Correct Answer Cuscuta

Haustorial roots -The roots of parasitic

plants which penetrate into the host

tissues to absorb nourishment are

called haustorial roots hellip Also known as suckingor parasitic roots

Q229 Equs Asinus is the scientific name

of

एकवसएलसनस mdashmdashndash कावहॴजञाननकनामहहॴ 8-

Feb-2017

Options

1) Donkey

गधा 2) Cow

गाय

3) Deer

टहरन

4) Kangaroo

कगा

Correct Answer Donkey

Q230 Ficus benghalensis is the scientific name of

फाईकसबहॳनगहॳिहॳलसस mdashndash कापवजञाननकनामहहॴ 8-Feb-2017

Options

1) Banyan

बरगद

2) Pineapple

अनानास

3) Babul

बबि

4) Tulsi

तिसी Correct Answer Banyan

Q231 Equus burchellii is the scientific name of

एकवसबचिी mdashmdash- कापवजञाननकनामहहॴ 8-Feb-2017

Options

1) Horse

घह८िा 2) Zebra

जहॳिा 3) Buffalo

F A C E B O O K

P A G E h t t p w w w f a c e b o o k c o m s s c m e n t o r s o f f i c i a l P a g e | 53

FOR MORE UPDATES AND MORE MATERIAL DO LIKE OUR FACEBOOK PAGE httpwwwfacebookcomsscmentorsofficial

भस

4) Ass

गधा Correct Answer Zebra

Page 48: COMPILATION OF ALL 72 SETS OF BIOLOGY SSC CHSL-2016 · OF BIOLOGY SSC CHSL-2016 PREPARED BY : SSC MENTORS BIOLOGY SPECIAL . F A C E B O O K P A G E : h t t p : / / w w w . f a c e

F A C E B O O K

P A G E h t t p w w w f a c e b o o k c o m s s c m e n t o r s o f f i c i a l P a g e | 47

FOR MORE UPDATES AND MORE MATERIAL DO LIKE OUR FACEBOOK PAGE httpwwwfacebookcomsscmentorsofficial

which might contain goldenrod clover

oxeye daisy and Rosa multiflora

thallophyte any of a group of plants or

plantlike organisms (such as algae and

fungi) that lack differentiated stems

leaves and roots and that were formerly

classified as a primary division

(Thallophyta) of the plant kingdom

Q204 How many types of teeth are

there in humans

मनषयोमककतनहॳपरकारकहॳ दातहह८तहॳहहॴ

1-Feb-2017

Options

1) 4

2) 5

3) 2

4) 3

Correct Answer 4

teeth -Humans have four types of

teethincisors canines premolars and

molars each with a specific function

The incisors cut the food the canines

tear the food and the molars and

premolars crush the food

Q205 Carica papaya is the scientific name of

कहॴ ररकापपाया mdashmdashndash कावहॴजञाननकनामहहॴ 2-

Feb-2017

Options

1) Peepal

पीपि

2) Papaya

पपीता 3) Tamarind

इमिी 4) Drumstick

ढह८िकाछड़ी Correct Answer Papaya

Q206 Muscles get tired when there is

shortfall of

जब mdashndash कीकमीहह८तीहहॴतबपहॳिीयिकजातीहहॴ 2-Feb-2017

Options

1) Lactic acid

िहॴनकटकएलसि

2) Na+ ions

Na+ आयन

3) ATP

एटीपी 4) Sulphates

समफहॳ टस

Correct Answer ATP

ATP is the energy source muscle fibers

use to make muscles contract

muscle tissuersquos main source of energy

called adenosine triphosphate or ATP

As your muscles use up this energy

source they become tired and fatigued

Oxygen is the key ingredient that helps

create new ATP to replenish the burned

up ATP in your muscles

Q207 Artocarpus integra is the

scientific name of आटह८कापयसइटीगरा mdashmdashmdash कावहॴजञाननकनामहहॴ 2-Feb-2017

Options

1) Guava

अम द

2) Pineapple

अनानास

3) Silver Oak

लसमवरओक

4) Jack fruit

कटहि

Correct Answer Jack fruit

Q208 Which organ stores fat soluble

vitamins

कह९नसाअगवसामघिनिीिपवटालमनह८काभिाराकरताहहॴ

2-Feb-2017

F A C E B O O K

P A G E h t t p w w w f a c e b o o k c o m s s c m e n t o r s o f f i c i a l P a g e | 48

FOR MORE UPDATES AND MORE MATERIAL DO LIKE OUR FACEBOOK PAGE httpwwwfacebookcomsscmentorsofficial

Options

1) Blood

रकत

2) Skin

तवचा 3) Liver

यकत

4) Pancreas

अगनयािय

Correct Answer Liver

Q209 Which disease is caused due to

deficiency of Iodine

आयह८िीनकहॳ कारणकह९नसारह८गहह८ताहहॴ 2-Feb-2017

Options

1) Rickets

ररकहॳ टस

2) Scurvy

सकवी 3) Goitre

गणमािा 4) Growth retardation

पवकासका कना Correct Answer Goitre

rickets A softening and weakening of

bones in children usually due to

inadequate vitamin D

Q210 Grevillea Robusta is the scientific name of

गरहॳपवलियारह८बसटा mdashmdashmdash- कापवजञाननकनामहहॴ 2-Feb-2017

Options

1) Peepal

पीपि

2) Teak

सागह९न

3) Silver Oak

लसमवरओक

4) Jack fruit

कटहि

Correct Answer Silver Oak

Q211 When a Cuttlefish is described as a Molluscs it is at which level of

classification

जबएककटिकफिकह८एकमह८िसकाकहॳ पमवखणयतककयाजाताहहॴतबयहॳवगीकरणकहॳ ककससतरपहॳनसितहहॴ 2-Feb-2017

Options

1) Class

वगय 2) Order

िम

3) Family

पररवार

4) Phylum

सघ

Correct Answer Phylum

Q212 Bambusa dendrocalmus is the

scientific name of बानबसािहॳडराकामस mdashmdashmdash कावहॴजञाननकनामहहॴ 3-Feb-2017

Options

1) Banyan

बरगद

2) Papaya

पपीता 3) Bamboo

बास

4) Pomegranate

अनार

Correct Answer Bamboo

Q213 Acinonyx Jubatus is the scientific name of

एलसनह८ननकसजयबहॳटस mdashmdashmdash

कावहॴजञाननकनामहहॴ 3-Feb-2017

F A C E B O O K

P A G E h t t p w w w f a c e b o o k c o m s s c m e n t o r s o f f i c i a l P a g e | 49

FOR MORE UPDATES AND MORE MATERIAL DO LIKE OUR FACEBOOK PAGE httpwwwfacebookcomsscmentorsofficial

Options

1) Bear

भाि 2) Horse

घह८िा 3) Cheetah

चीता 4) Zebra

जहॳिा Correct Answer Cheetah

Q214 The pale yellow colour of urine is

due to the presence of which pigment

मतरकाफीकापीिारगरगदरयकहॳ उपनसिनतकहॳ कारणहह८ताहहॴ

3-Feb-2017

Options

1) Urochrome

यरह८िह८म

2) Urophyll

यरह८कफि

3) Chlorophyll

किह८रह८कफि

4) Chloroplast

किह८रह८पिासट

Correct Answer Urochrome

Q215 Which of the following constitute

to form a gene

नननननलिखितमसहॳकह९नसीचीज़एकजीनकागठनकरतीहहॴ

3-Feb-2017

Options

1) Polynucleotides

पह८िीनयनकियह८टाईडस

2) Hydrocarbons

हाइडरह८काबोस

3) Lipoproteins

िाईपह८परह८टीनस

4) Lipids

लिपपडस

Correct Answer Polynucleotides

Polynucleotide molecule is a biopolymer

composed of 13 or more nucleotide

monomers covalently bonded in a chain

DNA (deoxyribonucleic acid) and RNA

(ribonucleic acid) are examples of

polynucleotides with distinct biological

function

Q216 Vertebrates belongs to the

phylum

रीढ़कीहडिीवािहॳपराणी mdashmdashmdash

परजानतकहॳ अतगायतआतहॳहहॴ 3-Feb-2017

Options

1) Arthropoda

आरह८पह८ड़ा 2) Annelida

एननलििा 3) Cnidaria

ननिहॳररया 4) Chordata

कह८िटा Correct Answer Chordata

Q217 Punica granatum is the scientific name of

पननकगरहॳनहॳटस mdashmdashmdash कावहॴजञाननकनामहहॴ 3-Feb-2017

Options

1) Custard Apple

सीताफि

2) Gulmohar

गिमह८हर

3) Silver Oak

लसमवरओक

4) Pomegranate

अनार

Correct Answer Pomegranate

F A C E B O O K

P A G E h t t p w w w f a c e b o o k c o m s s c m e n t o r s o f f i c i a l P a g e | 50

FOR MORE UPDATES AND MORE MATERIAL DO LIKE OUR FACEBOOK PAGE httpwwwfacebookcomsscmentorsofficial

Q218 Between a tiger and an monkey

which of the following is different

एकबाघऔरबदरकहॳ बीचनननननलिखितमसहॳकह९नसीबातअिगहहॴ 3-Feb-2017

Options

1) Kingdom

राजय

2) Phylum

जानत

3) Order

िम

4) Class

वगय Correct Answer order

Q219 The artificial heart was invented by

कबतरमहदयका mdashmdashmdash

दवाराअपवषकारककयागयािा 3-Feb-2017

Options

1) Muhammad Yunus

महनमदयनस

2) Linus Yale Jr

िाइनसयहॳिजय

3) Gazi Yasargil

गाजीयासचगयि

4) Paul Winchell

पह९िपवमकि Correct Answer Paul Winchell

Q220 Tamarindus indica is the

scientific name of

टहॳमररनडसइडिका mdashmdash कावहॴजञाननकनामहहॴ 7-

Feb-2017

Options

1) Neem

नीम

2) Pineapple

अनानास

3) Tamarind

इमिी 4)Chiku

चीक

Correct Answer Tamarind

Q221 In eukaryotic cells synthesis of

RNA takes place in the

यकहॳ योटटककह८लिकाओमआरएनएकासशिहॳषण

mdashndash महह८ताहहॴ 7-Feb-2017

Options

1) Mitochondria

माईटह८कोडडरया 2) Centrioles

सटरीयह८मस

3) Ribosomes

ररबह८सह८नस

4) Nucleus

नयनकियस

Correct Answer nucleus

eukaryotic cell -Transcription is the

process of synthesizing ribonucleic acid

(RNA)Synthesis takes place within the

nucleus of eukaryotic cells or in the

cytoplasm of prokaryotes and converts

the genetic code from a gene in

deoxyribonucleic acid ( DNA ) to a

strand of RNA that then directs

proteinsynthesis

Q222 _________is caused by parasites

of the Plasmodium genus

पिाजमह८डियमजातीकहॳ परजीवी mdash- कहॳ कारणहहॴ 7-Feb-2017

Options

1) Dysentery

पहॳचचि

2) Malaria

मिहॳररया 3) Chickenpox

F A C E B O O K

P A G E h t t p w w w f a c e b o o k c o m s s c m e n t o r s o f f i c i a l P a g e | 51

FOR MORE UPDATES AND MORE MATERIAL DO LIKE OUR FACEBOOK PAGE httpwwwfacebookcomsscmentorsofficial

चहॳचक

4) Herpes

हहॳपपयस

Correct Answer Malaria

Q223 Carotene in fruits and vegetables

gives it which color

फिह८औरसनलजयोमनसितकहॳ रह८टीनउनहकह९नसारगपरदानकरताहहॴ 7-Feb-2017

Options

1) Green

हरा 2) Pink

गिाबी 3) Orange

नारगी 4) Blue

नीिा Correct Answer Orange

Q224 Equus Caballus is the scientific

name of

एकवसकहॴ बहॳिस mdashmdashndash कापवजञाननकनामहहॴ 7-Feb-2017

Options

1) Horse

घह८िा 2) Zebra

ज़हॳिा 3) Donkey

गधा 4) Buffalo

भस

Correct Answer Horse

Q225 Elapidae Naja is the scientific name of

एिीपीिीनाजा mdashmdash- कावहॴजञाननकनामहहॴ 8-Feb-2017

Options

1) Cobra

कह८बरा 2) Elephant

हािी 3) Eagle

ग ि

4) Owl

उमि Correct Answer Cobra

Q226 Which disease is caused due to

deficiency of Iron

िह८हकीकमीकहॳ कारणकह९नसारह८गहह८ताहहॴ 8-Feb-

2017

Options

1) Beriberi

बहॳरीबहॳरी 2) Tetany

टहॳटनी 3) Kwashiorkor

कवािीऔरकर

4) Anaemia

रकतामपता Correct Answer Anaemia

Beriberi is a disease caused by a vitamin

B-1 deficiency also known as thiamine

deficiency

Tetany can be the result of an

electrolyte imbalance Most often itrsquos a

dramatically low calcium level also

known as hypocalcemia Tetany can also

be caused by magnesium deficiency or

too little potassium Having too much

acid (acidosis) or too much alkali

(alkalosis) in the body can also result in

tetany

Kwashiorkor also known as

ldquoedematous malnutrition It is a form of

malnutrition caused by a lack of protein

in the diet

Anaemia means that you have fewer red

blood cells than normal or you have less

F A C E B O O K

P A G E h t t p w w w f a c e b o o k c o m s s c m e n t o r s o f f i c i a l P a g e | 52

FOR MORE UPDATES AND MORE MATERIAL DO LIKE OUR FACEBOOK PAGE httpwwwfacebookcomsscmentorsofficial

haemoglobin than normal in each red

blood cell

Q227 is a leaf where the leaflets are

arranged along the middle vein

mdashndashएकपततीहहॴजहापतरकह८कीरचनाक ररयालिराकहॳ आसपासहह८तीहहॴ 8-Feb-2017

Options

1) Pinnately compound leaf

पपनहॳटिीसयकतपतती 2) Palmately compound leaf

पामहॳटिीसयकतपतती 3) Compound leaf

सयकतपतती 4) Simple leaf

साधारणपतती Correct Answer Pinnately compound

leaf

Q228 Haustoria or sucking roots are

found in which of the following

हह८सटह८ररयायाचसनहॳवािीजड़हॳनननननलिखितमसहॳककसमपाईजातीहहॴ 8-Feb-2017

Options

1) Wheat

गहॳह

2) Mango

आम

3) Chestnut

चहॳसटनट

4) Cuscuta

कसकयटा Correct Answer Cuscuta

Haustorial roots -The roots of parasitic

plants which penetrate into the host

tissues to absorb nourishment are

called haustorial roots hellip Also known as suckingor parasitic roots

Q229 Equs Asinus is the scientific name

of

एकवसएलसनस mdashmdashndash कावहॴजञाननकनामहहॴ 8-

Feb-2017

Options

1) Donkey

गधा 2) Cow

गाय

3) Deer

टहरन

4) Kangaroo

कगा

Correct Answer Donkey

Q230 Ficus benghalensis is the scientific name of

फाईकसबहॳनगहॳिहॳलसस mdashndash कापवजञाननकनामहहॴ 8-Feb-2017

Options

1) Banyan

बरगद

2) Pineapple

अनानास

3) Babul

बबि

4) Tulsi

तिसी Correct Answer Banyan

Q231 Equus burchellii is the scientific name of

एकवसबचिी mdashmdash- कापवजञाननकनामहहॴ 8-Feb-2017

Options

1) Horse

घह८िा 2) Zebra

जहॳिा 3) Buffalo

F A C E B O O K

P A G E h t t p w w w f a c e b o o k c o m s s c m e n t o r s o f f i c i a l P a g e | 53

FOR MORE UPDATES AND MORE MATERIAL DO LIKE OUR FACEBOOK PAGE httpwwwfacebookcomsscmentorsofficial

भस

4) Ass

गधा Correct Answer Zebra

Page 49: COMPILATION OF ALL 72 SETS OF BIOLOGY SSC CHSL-2016 · OF BIOLOGY SSC CHSL-2016 PREPARED BY : SSC MENTORS BIOLOGY SPECIAL . F A C E B O O K P A G E : h t t p : / / w w w . f a c e

F A C E B O O K

P A G E h t t p w w w f a c e b o o k c o m s s c m e n t o r s o f f i c i a l P a g e | 48

FOR MORE UPDATES AND MORE MATERIAL DO LIKE OUR FACEBOOK PAGE httpwwwfacebookcomsscmentorsofficial

Options

1) Blood

रकत

2) Skin

तवचा 3) Liver

यकत

4) Pancreas

अगनयािय

Correct Answer Liver

Q209 Which disease is caused due to

deficiency of Iodine

आयह८िीनकहॳ कारणकह९नसारह८गहह८ताहहॴ 2-Feb-2017

Options

1) Rickets

ररकहॳ टस

2) Scurvy

सकवी 3) Goitre

गणमािा 4) Growth retardation

पवकासका कना Correct Answer Goitre

rickets A softening and weakening of

bones in children usually due to

inadequate vitamin D

Q210 Grevillea Robusta is the scientific name of

गरहॳपवलियारह८बसटा mdashmdashmdash- कापवजञाननकनामहहॴ 2-Feb-2017

Options

1) Peepal

पीपि

2) Teak

सागह९न

3) Silver Oak

लसमवरओक

4) Jack fruit

कटहि

Correct Answer Silver Oak

Q211 When a Cuttlefish is described as a Molluscs it is at which level of

classification

जबएककटिकफिकह८एकमह८िसकाकहॳ पमवखणयतककयाजाताहहॴतबयहॳवगीकरणकहॳ ककससतरपहॳनसितहहॴ 2-Feb-2017

Options

1) Class

वगय 2) Order

िम

3) Family

पररवार

4) Phylum

सघ

Correct Answer Phylum

Q212 Bambusa dendrocalmus is the

scientific name of बानबसािहॳडराकामस mdashmdashmdash कावहॴजञाननकनामहहॴ 3-Feb-2017

Options

1) Banyan

बरगद

2) Papaya

पपीता 3) Bamboo

बास

4) Pomegranate

अनार

Correct Answer Bamboo

Q213 Acinonyx Jubatus is the scientific name of

एलसनह८ननकसजयबहॳटस mdashmdashmdash

कावहॴजञाननकनामहहॴ 3-Feb-2017

F A C E B O O K

P A G E h t t p w w w f a c e b o o k c o m s s c m e n t o r s o f f i c i a l P a g e | 49

FOR MORE UPDATES AND MORE MATERIAL DO LIKE OUR FACEBOOK PAGE httpwwwfacebookcomsscmentorsofficial

Options

1) Bear

भाि 2) Horse

घह८िा 3) Cheetah

चीता 4) Zebra

जहॳिा Correct Answer Cheetah

Q214 The pale yellow colour of urine is

due to the presence of which pigment

मतरकाफीकापीिारगरगदरयकहॳ उपनसिनतकहॳ कारणहह८ताहहॴ

3-Feb-2017

Options

1) Urochrome

यरह८िह८म

2) Urophyll

यरह८कफि

3) Chlorophyll

किह८रह८कफि

4) Chloroplast

किह८रह८पिासट

Correct Answer Urochrome

Q215 Which of the following constitute

to form a gene

नननननलिखितमसहॳकह९नसीचीज़एकजीनकागठनकरतीहहॴ

3-Feb-2017

Options

1) Polynucleotides

पह८िीनयनकियह८टाईडस

2) Hydrocarbons

हाइडरह८काबोस

3) Lipoproteins

िाईपह८परह८टीनस

4) Lipids

लिपपडस

Correct Answer Polynucleotides

Polynucleotide molecule is a biopolymer

composed of 13 or more nucleotide

monomers covalently bonded in a chain

DNA (deoxyribonucleic acid) and RNA

(ribonucleic acid) are examples of

polynucleotides with distinct biological

function

Q216 Vertebrates belongs to the

phylum

रीढ़कीहडिीवािहॳपराणी mdashmdashmdash

परजानतकहॳ अतगायतआतहॳहहॴ 3-Feb-2017

Options

1) Arthropoda

आरह८पह८ड़ा 2) Annelida

एननलििा 3) Cnidaria

ननिहॳररया 4) Chordata

कह८िटा Correct Answer Chordata

Q217 Punica granatum is the scientific name of

पननकगरहॳनहॳटस mdashmdashmdash कावहॴजञाननकनामहहॴ 3-Feb-2017

Options

1) Custard Apple

सीताफि

2) Gulmohar

गिमह८हर

3) Silver Oak

लसमवरओक

4) Pomegranate

अनार

Correct Answer Pomegranate

F A C E B O O K

P A G E h t t p w w w f a c e b o o k c o m s s c m e n t o r s o f f i c i a l P a g e | 50

FOR MORE UPDATES AND MORE MATERIAL DO LIKE OUR FACEBOOK PAGE httpwwwfacebookcomsscmentorsofficial

Q218 Between a tiger and an monkey

which of the following is different

एकबाघऔरबदरकहॳ बीचनननननलिखितमसहॳकह९नसीबातअिगहहॴ 3-Feb-2017

Options

1) Kingdom

राजय

2) Phylum

जानत

3) Order

िम

4) Class

वगय Correct Answer order

Q219 The artificial heart was invented by

कबतरमहदयका mdashmdashmdash

दवाराअपवषकारककयागयािा 3-Feb-2017

Options

1) Muhammad Yunus

महनमदयनस

2) Linus Yale Jr

िाइनसयहॳिजय

3) Gazi Yasargil

गाजीयासचगयि

4) Paul Winchell

पह९िपवमकि Correct Answer Paul Winchell

Q220 Tamarindus indica is the

scientific name of

टहॳमररनडसइडिका mdashmdash कावहॴजञाननकनामहहॴ 7-

Feb-2017

Options

1) Neem

नीम

2) Pineapple

अनानास

3) Tamarind

इमिी 4)Chiku

चीक

Correct Answer Tamarind

Q221 In eukaryotic cells synthesis of

RNA takes place in the

यकहॳ योटटककह८लिकाओमआरएनएकासशिहॳषण

mdashndash महह८ताहहॴ 7-Feb-2017

Options

1) Mitochondria

माईटह८कोडडरया 2) Centrioles

सटरीयह८मस

3) Ribosomes

ररबह८सह८नस

4) Nucleus

नयनकियस

Correct Answer nucleus

eukaryotic cell -Transcription is the

process of synthesizing ribonucleic acid

(RNA)Synthesis takes place within the

nucleus of eukaryotic cells or in the

cytoplasm of prokaryotes and converts

the genetic code from a gene in

deoxyribonucleic acid ( DNA ) to a

strand of RNA that then directs

proteinsynthesis

Q222 _________is caused by parasites

of the Plasmodium genus

पिाजमह८डियमजातीकहॳ परजीवी mdash- कहॳ कारणहहॴ 7-Feb-2017

Options

1) Dysentery

पहॳचचि

2) Malaria

मिहॳररया 3) Chickenpox

F A C E B O O K

P A G E h t t p w w w f a c e b o o k c o m s s c m e n t o r s o f f i c i a l P a g e | 51

FOR MORE UPDATES AND MORE MATERIAL DO LIKE OUR FACEBOOK PAGE httpwwwfacebookcomsscmentorsofficial

चहॳचक

4) Herpes

हहॳपपयस

Correct Answer Malaria

Q223 Carotene in fruits and vegetables

gives it which color

फिह८औरसनलजयोमनसितकहॳ रह८टीनउनहकह९नसारगपरदानकरताहहॴ 7-Feb-2017

Options

1) Green

हरा 2) Pink

गिाबी 3) Orange

नारगी 4) Blue

नीिा Correct Answer Orange

Q224 Equus Caballus is the scientific

name of

एकवसकहॴ बहॳिस mdashmdashndash कापवजञाननकनामहहॴ 7-Feb-2017

Options

1) Horse

घह८िा 2) Zebra

ज़हॳिा 3) Donkey

गधा 4) Buffalo

भस

Correct Answer Horse

Q225 Elapidae Naja is the scientific name of

एिीपीिीनाजा mdashmdash- कावहॴजञाननकनामहहॴ 8-Feb-2017

Options

1) Cobra

कह८बरा 2) Elephant

हािी 3) Eagle

ग ि

4) Owl

उमि Correct Answer Cobra

Q226 Which disease is caused due to

deficiency of Iron

िह८हकीकमीकहॳ कारणकह९नसारह८गहह८ताहहॴ 8-Feb-

2017

Options

1) Beriberi

बहॳरीबहॳरी 2) Tetany

टहॳटनी 3) Kwashiorkor

कवािीऔरकर

4) Anaemia

रकतामपता Correct Answer Anaemia

Beriberi is a disease caused by a vitamin

B-1 deficiency also known as thiamine

deficiency

Tetany can be the result of an

electrolyte imbalance Most often itrsquos a

dramatically low calcium level also

known as hypocalcemia Tetany can also

be caused by magnesium deficiency or

too little potassium Having too much

acid (acidosis) or too much alkali

(alkalosis) in the body can also result in

tetany

Kwashiorkor also known as

ldquoedematous malnutrition It is a form of

malnutrition caused by a lack of protein

in the diet

Anaemia means that you have fewer red

blood cells than normal or you have less

F A C E B O O K

P A G E h t t p w w w f a c e b o o k c o m s s c m e n t o r s o f f i c i a l P a g e | 52

FOR MORE UPDATES AND MORE MATERIAL DO LIKE OUR FACEBOOK PAGE httpwwwfacebookcomsscmentorsofficial

haemoglobin than normal in each red

blood cell

Q227 is a leaf where the leaflets are

arranged along the middle vein

mdashndashएकपततीहहॴजहापतरकह८कीरचनाक ररयालिराकहॳ आसपासहह८तीहहॴ 8-Feb-2017

Options

1) Pinnately compound leaf

पपनहॳटिीसयकतपतती 2) Palmately compound leaf

पामहॳटिीसयकतपतती 3) Compound leaf

सयकतपतती 4) Simple leaf

साधारणपतती Correct Answer Pinnately compound

leaf

Q228 Haustoria or sucking roots are

found in which of the following

हह८सटह८ररयायाचसनहॳवािीजड़हॳनननननलिखितमसहॳककसमपाईजातीहहॴ 8-Feb-2017

Options

1) Wheat

गहॳह

2) Mango

आम

3) Chestnut

चहॳसटनट

4) Cuscuta

कसकयटा Correct Answer Cuscuta

Haustorial roots -The roots of parasitic

plants which penetrate into the host

tissues to absorb nourishment are

called haustorial roots hellip Also known as suckingor parasitic roots

Q229 Equs Asinus is the scientific name

of

एकवसएलसनस mdashmdashndash कावहॴजञाननकनामहहॴ 8-

Feb-2017

Options

1) Donkey

गधा 2) Cow

गाय

3) Deer

टहरन

4) Kangaroo

कगा

Correct Answer Donkey

Q230 Ficus benghalensis is the scientific name of

फाईकसबहॳनगहॳिहॳलसस mdashndash कापवजञाननकनामहहॴ 8-Feb-2017

Options

1) Banyan

बरगद

2) Pineapple

अनानास

3) Babul

बबि

4) Tulsi

तिसी Correct Answer Banyan

Q231 Equus burchellii is the scientific name of

एकवसबचिी mdashmdash- कापवजञाननकनामहहॴ 8-Feb-2017

Options

1) Horse

घह८िा 2) Zebra

जहॳिा 3) Buffalo

F A C E B O O K

P A G E h t t p w w w f a c e b o o k c o m s s c m e n t o r s o f f i c i a l P a g e | 53

FOR MORE UPDATES AND MORE MATERIAL DO LIKE OUR FACEBOOK PAGE httpwwwfacebookcomsscmentorsofficial

भस

4) Ass

गधा Correct Answer Zebra

Page 50: COMPILATION OF ALL 72 SETS OF BIOLOGY SSC CHSL-2016 · OF BIOLOGY SSC CHSL-2016 PREPARED BY : SSC MENTORS BIOLOGY SPECIAL . F A C E B O O K P A G E : h t t p : / / w w w . f a c e

F A C E B O O K

P A G E h t t p w w w f a c e b o o k c o m s s c m e n t o r s o f f i c i a l P a g e | 49

FOR MORE UPDATES AND MORE MATERIAL DO LIKE OUR FACEBOOK PAGE httpwwwfacebookcomsscmentorsofficial

Options

1) Bear

भाि 2) Horse

घह८िा 3) Cheetah

चीता 4) Zebra

जहॳिा Correct Answer Cheetah

Q214 The pale yellow colour of urine is

due to the presence of which pigment

मतरकाफीकापीिारगरगदरयकहॳ उपनसिनतकहॳ कारणहह८ताहहॴ

3-Feb-2017

Options

1) Urochrome

यरह८िह८म

2) Urophyll

यरह८कफि

3) Chlorophyll

किह८रह८कफि

4) Chloroplast

किह८रह८पिासट

Correct Answer Urochrome

Q215 Which of the following constitute

to form a gene

नननननलिखितमसहॳकह९नसीचीज़एकजीनकागठनकरतीहहॴ

3-Feb-2017

Options

1) Polynucleotides

पह८िीनयनकियह८टाईडस

2) Hydrocarbons

हाइडरह८काबोस

3) Lipoproteins

िाईपह८परह८टीनस

4) Lipids

लिपपडस

Correct Answer Polynucleotides

Polynucleotide molecule is a biopolymer

composed of 13 or more nucleotide

monomers covalently bonded in a chain

DNA (deoxyribonucleic acid) and RNA

(ribonucleic acid) are examples of

polynucleotides with distinct biological

function

Q216 Vertebrates belongs to the

phylum

रीढ़कीहडिीवािहॳपराणी mdashmdashmdash

परजानतकहॳ अतगायतआतहॳहहॴ 3-Feb-2017

Options

1) Arthropoda

आरह८पह८ड़ा 2) Annelida

एननलििा 3) Cnidaria

ननिहॳररया 4) Chordata

कह८िटा Correct Answer Chordata

Q217 Punica granatum is the scientific name of

पननकगरहॳनहॳटस mdashmdashmdash कावहॴजञाननकनामहहॴ 3-Feb-2017

Options

1) Custard Apple

सीताफि

2) Gulmohar

गिमह८हर

3) Silver Oak

लसमवरओक

4) Pomegranate

अनार

Correct Answer Pomegranate

F A C E B O O K

P A G E h t t p w w w f a c e b o o k c o m s s c m e n t o r s o f f i c i a l P a g e | 50

FOR MORE UPDATES AND MORE MATERIAL DO LIKE OUR FACEBOOK PAGE httpwwwfacebookcomsscmentorsofficial

Q218 Between a tiger and an monkey

which of the following is different

एकबाघऔरबदरकहॳ बीचनननननलिखितमसहॳकह९नसीबातअिगहहॴ 3-Feb-2017

Options

1) Kingdom

राजय

2) Phylum

जानत

3) Order

िम

4) Class

वगय Correct Answer order

Q219 The artificial heart was invented by

कबतरमहदयका mdashmdashmdash

दवाराअपवषकारककयागयािा 3-Feb-2017

Options

1) Muhammad Yunus

महनमदयनस

2) Linus Yale Jr

िाइनसयहॳिजय

3) Gazi Yasargil

गाजीयासचगयि

4) Paul Winchell

पह९िपवमकि Correct Answer Paul Winchell

Q220 Tamarindus indica is the

scientific name of

टहॳमररनडसइडिका mdashmdash कावहॴजञाननकनामहहॴ 7-

Feb-2017

Options

1) Neem

नीम

2) Pineapple

अनानास

3) Tamarind

इमिी 4)Chiku

चीक

Correct Answer Tamarind

Q221 In eukaryotic cells synthesis of

RNA takes place in the

यकहॳ योटटककह८लिकाओमआरएनएकासशिहॳषण

mdashndash महह८ताहहॴ 7-Feb-2017

Options

1) Mitochondria

माईटह८कोडडरया 2) Centrioles

सटरीयह८मस

3) Ribosomes

ररबह८सह८नस

4) Nucleus

नयनकियस

Correct Answer nucleus

eukaryotic cell -Transcription is the

process of synthesizing ribonucleic acid

(RNA)Synthesis takes place within the

nucleus of eukaryotic cells or in the

cytoplasm of prokaryotes and converts

the genetic code from a gene in

deoxyribonucleic acid ( DNA ) to a

strand of RNA that then directs

proteinsynthesis

Q222 _________is caused by parasites

of the Plasmodium genus

पिाजमह८डियमजातीकहॳ परजीवी mdash- कहॳ कारणहहॴ 7-Feb-2017

Options

1) Dysentery

पहॳचचि

2) Malaria

मिहॳररया 3) Chickenpox

F A C E B O O K

P A G E h t t p w w w f a c e b o o k c o m s s c m e n t o r s o f f i c i a l P a g e | 51

FOR MORE UPDATES AND MORE MATERIAL DO LIKE OUR FACEBOOK PAGE httpwwwfacebookcomsscmentorsofficial

चहॳचक

4) Herpes

हहॳपपयस

Correct Answer Malaria

Q223 Carotene in fruits and vegetables

gives it which color

फिह८औरसनलजयोमनसितकहॳ रह८टीनउनहकह९नसारगपरदानकरताहहॴ 7-Feb-2017

Options

1) Green

हरा 2) Pink

गिाबी 3) Orange

नारगी 4) Blue

नीिा Correct Answer Orange

Q224 Equus Caballus is the scientific

name of

एकवसकहॴ बहॳिस mdashmdashndash कापवजञाननकनामहहॴ 7-Feb-2017

Options

1) Horse

घह८िा 2) Zebra

ज़हॳिा 3) Donkey

गधा 4) Buffalo

भस

Correct Answer Horse

Q225 Elapidae Naja is the scientific name of

एिीपीिीनाजा mdashmdash- कावहॴजञाननकनामहहॴ 8-Feb-2017

Options

1) Cobra

कह८बरा 2) Elephant

हािी 3) Eagle

ग ि

4) Owl

उमि Correct Answer Cobra

Q226 Which disease is caused due to

deficiency of Iron

िह८हकीकमीकहॳ कारणकह९नसारह८गहह८ताहहॴ 8-Feb-

2017

Options

1) Beriberi

बहॳरीबहॳरी 2) Tetany

टहॳटनी 3) Kwashiorkor

कवािीऔरकर

4) Anaemia

रकतामपता Correct Answer Anaemia

Beriberi is a disease caused by a vitamin

B-1 deficiency also known as thiamine

deficiency

Tetany can be the result of an

electrolyte imbalance Most often itrsquos a

dramatically low calcium level also

known as hypocalcemia Tetany can also

be caused by magnesium deficiency or

too little potassium Having too much

acid (acidosis) or too much alkali

(alkalosis) in the body can also result in

tetany

Kwashiorkor also known as

ldquoedematous malnutrition It is a form of

malnutrition caused by a lack of protein

in the diet

Anaemia means that you have fewer red

blood cells than normal or you have less

F A C E B O O K

P A G E h t t p w w w f a c e b o o k c o m s s c m e n t o r s o f f i c i a l P a g e | 52

FOR MORE UPDATES AND MORE MATERIAL DO LIKE OUR FACEBOOK PAGE httpwwwfacebookcomsscmentorsofficial

haemoglobin than normal in each red

blood cell

Q227 is a leaf where the leaflets are

arranged along the middle vein

mdashndashएकपततीहहॴजहापतरकह८कीरचनाक ररयालिराकहॳ आसपासहह८तीहहॴ 8-Feb-2017

Options

1) Pinnately compound leaf

पपनहॳटिीसयकतपतती 2) Palmately compound leaf

पामहॳटिीसयकतपतती 3) Compound leaf

सयकतपतती 4) Simple leaf

साधारणपतती Correct Answer Pinnately compound

leaf

Q228 Haustoria or sucking roots are

found in which of the following

हह८सटह८ररयायाचसनहॳवािीजड़हॳनननननलिखितमसहॳककसमपाईजातीहहॴ 8-Feb-2017

Options

1) Wheat

गहॳह

2) Mango

आम

3) Chestnut

चहॳसटनट

4) Cuscuta

कसकयटा Correct Answer Cuscuta

Haustorial roots -The roots of parasitic

plants which penetrate into the host

tissues to absorb nourishment are

called haustorial roots hellip Also known as suckingor parasitic roots

Q229 Equs Asinus is the scientific name

of

एकवसएलसनस mdashmdashndash कावहॴजञाननकनामहहॴ 8-

Feb-2017

Options

1) Donkey

गधा 2) Cow

गाय

3) Deer

टहरन

4) Kangaroo

कगा

Correct Answer Donkey

Q230 Ficus benghalensis is the scientific name of

फाईकसबहॳनगहॳिहॳलसस mdashndash कापवजञाननकनामहहॴ 8-Feb-2017

Options

1) Banyan

बरगद

2) Pineapple

अनानास

3) Babul

बबि

4) Tulsi

तिसी Correct Answer Banyan

Q231 Equus burchellii is the scientific name of

एकवसबचिी mdashmdash- कापवजञाननकनामहहॴ 8-Feb-2017

Options

1) Horse

घह८िा 2) Zebra

जहॳिा 3) Buffalo

F A C E B O O K

P A G E h t t p w w w f a c e b o o k c o m s s c m e n t o r s o f f i c i a l P a g e | 53

FOR MORE UPDATES AND MORE MATERIAL DO LIKE OUR FACEBOOK PAGE httpwwwfacebookcomsscmentorsofficial

भस

4) Ass

गधा Correct Answer Zebra

Page 51: COMPILATION OF ALL 72 SETS OF BIOLOGY SSC CHSL-2016 · OF BIOLOGY SSC CHSL-2016 PREPARED BY : SSC MENTORS BIOLOGY SPECIAL . F A C E B O O K P A G E : h t t p : / / w w w . f a c e

F A C E B O O K

P A G E h t t p w w w f a c e b o o k c o m s s c m e n t o r s o f f i c i a l P a g e | 50

FOR MORE UPDATES AND MORE MATERIAL DO LIKE OUR FACEBOOK PAGE httpwwwfacebookcomsscmentorsofficial

Q218 Between a tiger and an monkey

which of the following is different

एकबाघऔरबदरकहॳ बीचनननननलिखितमसहॳकह९नसीबातअिगहहॴ 3-Feb-2017

Options

1) Kingdom

राजय

2) Phylum

जानत

3) Order

िम

4) Class

वगय Correct Answer order

Q219 The artificial heart was invented by

कबतरमहदयका mdashmdashmdash

दवाराअपवषकारककयागयािा 3-Feb-2017

Options

1) Muhammad Yunus

महनमदयनस

2) Linus Yale Jr

िाइनसयहॳिजय

3) Gazi Yasargil

गाजीयासचगयि

4) Paul Winchell

पह९िपवमकि Correct Answer Paul Winchell

Q220 Tamarindus indica is the

scientific name of

टहॳमररनडसइडिका mdashmdash कावहॴजञाननकनामहहॴ 7-

Feb-2017

Options

1) Neem

नीम

2) Pineapple

अनानास

3) Tamarind

इमिी 4)Chiku

चीक

Correct Answer Tamarind

Q221 In eukaryotic cells synthesis of

RNA takes place in the

यकहॳ योटटककह८लिकाओमआरएनएकासशिहॳषण

mdashndash महह८ताहहॴ 7-Feb-2017

Options

1) Mitochondria

माईटह८कोडडरया 2) Centrioles

सटरीयह८मस

3) Ribosomes

ररबह८सह८नस

4) Nucleus

नयनकियस

Correct Answer nucleus

eukaryotic cell -Transcription is the

process of synthesizing ribonucleic acid

(RNA)Synthesis takes place within the

nucleus of eukaryotic cells or in the

cytoplasm of prokaryotes and converts

the genetic code from a gene in

deoxyribonucleic acid ( DNA ) to a

strand of RNA that then directs

proteinsynthesis

Q222 _________is caused by parasites

of the Plasmodium genus

पिाजमह८डियमजातीकहॳ परजीवी mdash- कहॳ कारणहहॴ 7-Feb-2017

Options

1) Dysentery

पहॳचचि

2) Malaria

मिहॳररया 3) Chickenpox

F A C E B O O K

P A G E h t t p w w w f a c e b o o k c o m s s c m e n t o r s o f f i c i a l P a g e | 51

FOR MORE UPDATES AND MORE MATERIAL DO LIKE OUR FACEBOOK PAGE httpwwwfacebookcomsscmentorsofficial

चहॳचक

4) Herpes

हहॳपपयस

Correct Answer Malaria

Q223 Carotene in fruits and vegetables

gives it which color

फिह८औरसनलजयोमनसितकहॳ रह८टीनउनहकह९नसारगपरदानकरताहहॴ 7-Feb-2017

Options

1) Green

हरा 2) Pink

गिाबी 3) Orange

नारगी 4) Blue

नीिा Correct Answer Orange

Q224 Equus Caballus is the scientific

name of

एकवसकहॴ बहॳिस mdashmdashndash कापवजञाननकनामहहॴ 7-Feb-2017

Options

1) Horse

घह८िा 2) Zebra

ज़हॳिा 3) Donkey

गधा 4) Buffalo

भस

Correct Answer Horse

Q225 Elapidae Naja is the scientific name of

एिीपीिीनाजा mdashmdash- कावहॴजञाननकनामहहॴ 8-Feb-2017

Options

1) Cobra

कह८बरा 2) Elephant

हािी 3) Eagle

ग ि

4) Owl

उमि Correct Answer Cobra

Q226 Which disease is caused due to

deficiency of Iron

िह८हकीकमीकहॳ कारणकह९नसारह८गहह८ताहहॴ 8-Feb-

2017

Options

1) Beriberi

बहॳरीबहॳरी 2) Tetany

टहॳटनी 3) Kwashiorkor

कवािीऔरकर

4) Anaemia

रकतामपता Correct Answer Anaemia

Beriberi is a disease caused by a vitamin

B-1 deficiency also known as thiamine

deficiency

Tetany can be the result of an

electrolyte imbalance Most often itrsquos a

dramatically low calcium level also

known as hypocalcemia Tetany can also

be caused by magnesium deficiency or

too little potassium Having too much

acid (acidosis) or too much alkali

(alkalosis) in the body can also result in

tetany

Kwashiorkor also known as

ldquoedematous malnutrition It is a form of

malnutrition caused by a lack of protein

in the diet

Anaemia means that you have fewer red

blood cells than normal or you have less

F A C E B O O K

P A G E h t t p w w w f a c e b o o k c o m s s c m e n t o r s o f f i c i a l P a g e | 52

FOR MORE UPDATES AND MORE MATERIAL DO LIKE OUR FACEBOOK PAGE httpwwwfacebookcomsscmentorsofficial

haemoglobin than normal in each red

blood cell

Q227 is a leaf where the leaflets are

arranged along the middle vein

mdashndashएकपततीहहॴजहापतरकह८कीरचनाक ररयालिराकहॳ आसपासहह८तीहहॴ 8-Feb-2017

Options

1) Pinnately compound leaf

पपनहॳटिीसयकतपतती 2) Palmately compound leaf

पामहॳटिीसयकतपतती 3) Compound leaf

सयकतपतती 4) Simple leaf

साधारणपतती Correct Answer Pinnately compound

leaf

Q228 Haustoria or sucking roots are

found in which of the following

हह८सटह८ररयायाचसनहॳवािीजड़हॳनननननलिखितमसहॳककसमपाईजातीहहॴ 8-Feb-2017

Options

1) Wheat

गहॳह

2) Mango

आम

3) Chestnut

चहॳसटनट

4) Cuscuta

कसकयटा Correct Answer Cuscuta

Haustorial roots -The roots of parasitic

plants which penetrate into the host

tissues to absorb nourishment are

called haustorial roots hellip Also known as suckingor parasitic roots

Q229 Equs Asinus is the scientific name

of

एकवसएलसनस mdashmdashndash कावहॴजञाननकनामहहॴ 8-

Feb-2017

Options

1) Donkey

गधा 2) Cow

गाय

3) Deer

टहरन

4) Kangaroo

कगा

Correct Answer Donkey

Q230 Ficus benghalensis is the scientific name of

फाईकसबहॳनगहॳिहॳलसस mdashndash कापवजञाननकनामहहॴ 8-Feb-2017

Options

1) Banyan

बरगद

2) Pineapple

अनानास

3) Babul

बबि

4) Tulsi

तिसी Correct Answer Banyan

Q231 Equus burchellii is the scientific name of

एकवसबचिी mdashmdash- कापवजञाननकनामहहॴ 8-Feb-2017

Options

1) Horse

घह८िा 2) Zebra

जहॳिा 3) Buffalo

F A C E B O O K

P A G E h t t p w w w f a c e b o o k c o m s s c m e n t o r s o f f i c i a l P a g e | 53

FOR MORE UPDATES AND MORE MATERIAL DO LIKE OUR FACEBOOK PAGE httpwwwfacebookcomsscmentorsofficial

भस

4) Ass

गधा Correct Answer Zebra

Page 52: COMPILATION OF ALL 72 SETS OF BIOLOGY SSC CHSL-2016 · OF BIOLOGY SSC CHSL-2016 PREPARED BY : SSC MENTORS BIOLOGY SPECIAL . F A C E B O O K P A G E : h t t p : / / w w w . f a c e

F A C E B O O K

P A G E h t t p w w w f a c e b o o k c o m s s c m e n t o r s o f f i c i a l P a g e | 51

FOR MORE UPDATES AND MORE MATERIAL DO LIKE OUR FACEBOOK PAGE httpwwwfacebookcomsscmentorsofficial

चहॳचक

4) Herpes

हहॳपपयस

Correct Answer Malaria

Q223 Carotene in fruits and vegetables

gives it which color

फिह८औरसनलजयोमनसितकहॳ रह८टीनउनहकह९नसारगपरदानकरताहहॴ 7-Feb-2017

Options

1) Green

हरा 2) Pink

गिाबी 3) Orange

नारगी 4) Blue

नीिा Correct Answer Orange

Q224 Equus Caballus is the scientific

name of

एकवसकहॴ बहॳिस mdashmdashndash कापवजञाननकनामहहॴ 7-Feb-2017

Options

1) Horse

घह८िा 2) Zebra

ज़हॳिा 3) Donkey

गधा 4) Buffalo

भस

Correct Answer Horse

Q225 Elapidae Naja is the scientific name of

एिीपीिीनाजा mdashmdash- कावहॴजञाननकनामहहॴ 8-Feb-2017

Options

1) Cobra

कह८बरा 2) Elephant

हािी 3) Eagle

ग ि

4) Owl

उमि Correct Answer Cobra

Q226 Which disease is caused due to

deficiency of Iron

िह८हकीकमीकहॳ कारणकह९नसारह८गहह८ताहहॴ 8-Feb-

2017

Options

1) Beriberi

बहॳरीबहॳरी 2) Tetany

टहॳटनी 3) Kwashiorkor

कवािीऔरकर

4) Anaemia

रकतामपता Correct Answer Anaemia

Beriberi is a disease caused by a vitamin

B-1 deficiency also known as thiamine

deficiency

Tetany can be the result of an

electrolyte imbalance Most often itrsquos a

dramatically low calcium level also

known as hypocalcemia Tetany can also

be caused by magnesium deficiency or

too little potassium Having too much

acid (acidosis) or too much alkali

(alkalosis) in the body can also result in

tetany

Kwashiorkor also known as

ldquoedematous malnutrition It is a form of

malnutrition caused by a lack of protein

in the diet

Anaemia means that you have fewer red

blood cells than normal or you have less

F A C E B O O K

P A G E h t t p w w w f a c e b o o k c o m s s c m e n t o r s o f f i c i a l P a g e | 52

FOR MORE UPDATES AND MORE MATERIAL DO LIKE OUR FACEBOOK PAGE httpwwwfacebookcomsscmentorsofficial

haemoglobin than normal in each red

blood cell

Q227 is a leaf where the leaflets are

arranged along the middle vein

mdashndashएकपततीहहॴजहापतरकह८कीरचनाक ररयालिराकहॳ आसपासहह८तीहहॴ 8-Feb-2017

Options

1) Pinnately compound leaf

पपनहॳटिीसयकतपतती 2) Palmately compound leaf

पामहॳटिीसयकतपतती 3) Compound leaf

सयकतपतती 4) Simple leaf

साधारणपतती Correct Answer Pinnately compound

leaf

Q228 Haustoria or sucking roots are

found in which of the following

हह८सटह८ररयायाचसनहॳवािीजड़हॳनननननलिखितमसहॳककसमपाईजातीहहॴ 8-Feb-2017

Options

1) Wheat

गहॳह

2) Mango

आम

3) Chestnut

चहॳसटनट

4) Cuscuta

कसकयटा Correct Answer Cuscuta

Haustorial roots -The roots of parasitic

plants which penetrate into the host

tissues to absorb nourishment are

called haustorial roots hellip Also known as suckingor parasitic roots

Q229 Equs Asinus is the scientific name

of

एकवसएलसनस mdashmdashndash कावहॴजञाननकनामहहॴ 8-

Feb-2017

Options

1) Donkey

गधा 2) Cow

गाय

3) Deer

टहरन

4) Kangaroo

कगा

Correct Answer Donkey

Q230 Ficus benghalensis is the scientific name of

फाईकसबहॳनगहॳिहॳलसस mdashndash कापवजञाननकनामहहॴ 8-Feb-2017

Options

1) Banyan

बरगद

2) Pineapple

अनानास

3) Babul

बबि

4) Tulsi

तिसी Correct Answer Banyan

Q231 Equus burchellii is the scientific name of

एकवसबचिी mdashmdash- कापवजञाननकनामहहॴ 8-Feb-2017

Options

1) Horse

घह८िा 2) Zebra

जहॳिा 3) Buffalo

F A C E B O O K

P A G E h t t p w w w f a c e b o o k c o m s s c m e n t o r s o f f i c i a l P a g e | 53

FOR MORE UPDATES AND MORE MATERIAL DO LIKE OUR FACEBOOK PAGE httpwwwfacebookcomsscmentorsofficial

भस

4) Ass

गधा Correct Answer Zebra

Page 53: COMPILATION OF ALL 72 SETS OF BIOLOGY SSC CHSL-2016 · OF BIOLOGY SSC CHSL-2016 PREPARED BY : SSC MENTORS BIOLOGY SPECIAL . F A C E B O O K P A G E : h t t p : / / w w w . f a c e

F A C E B O O K

P A G E h t t p w w w f a c e b o o k c o m s s c m e n t o r s o f f i c i a l P a g e | 52

FOR MORE UPDATES AND MORE MATERIAL DO LIKE OUR FACEBOOK PAGE httpwwwfacebookcomsscmentorsofficial

haemoglobin than normal in each red

blood cell

Q227 is a leaf where the leaflets are

arranged along the middle vein

mdashndashएकपततीहहॴजहापतरकह८कीरचनाक ररयालिराकहॳ आसपासहह८तीहहॴ 8-Feb-2017

Options

1) Pinnately compound leaf

पपनहॳटिीसयकतपतती 2) Palmately compound leaf

पामहॳटिीसयकतपतती 3) Compound leaf

सयकतपतती 4) Simple leaf

साधारणपतती Correct Answer Pinnately compound

leaf

Q228 Haustoria or sucking roots are

found in which of the following

हह८सटह८ररयायाचसनहॳवािीजड़हॳनननननलिखितमसहॳककसमपाईजातीहहॴ 8-Feb-2017

Options

1) Wheat

गहॳह

2) Mango

आम

3) Chestnut

चहॳसटनट

4) Cuscuta

कसकयटा Correct Answer Cuscuta

Haustorial roots -The roots of parasitic

plants which penetrate into the host

tissues to absorb nourishment are

called haustorial roots hellip Also known as suckingor parasitic roots

Q229 Equs Asinus is the scientific name

of

एकवसएलसनस mdashmdashndash कावहॴजञाननकनामहहॴ 8-

Feb-2017

Options

1) Donkey

गधा 2) Cow

गाय

3) Deer

टहरन

4) Kangaroo

कगा

Correct Answer Donkey

Q230 Ficus benghalensis is the scientific name of

फाईकसबहॳनगहॳिहॳलसस mdashndash कापवजञाननकनामहहॴ 8-Feb-2017

Options

1) Banyan

बरगद

2) Pineapple

अनानास

3) Babul

बबि

4) Tulsi

तिसी Correct Answer Banyan

Q231 Equus burchellii is the scientific name of

एकवसबचिी mdashmdash- कापवजञाननकनामहहॴ 8-Feb-2017

Options

1) Horse

घह८िा 2) Zebra

जहॳिा 3) Buffalo

F A C E B O O K

P A G E h t t p w w w f a c e b o o k c o m s s c m e n t o r s o f f i c i a l P a g e | 53

FOR MORE UPDATES AND MORE MATERIAL DO LIKE OUR FACEBOOK PAGE httpwwwfacebookcomsscmentorsofficial

भस

4) Ass

गधा Correct Answer Zebra

Page 54: COMPILATION OF ALL 72 SETS OF BIOLOGY SSC CHSL-2016 · OF BIOLOGY SSC CHSL-2016 PREPARED BY : SSC MENTORS BIOLOGY SPECIAL . F A C E B O O K P A G E : h t t p : / / w w w . f a c e

F A C E B O O K

P A G E h t t p w w w f a c e b o o k c o m s s c m e n t o r s o f f i c i a l P a g e | 53

FOR MORE UPDATES AND MORE MATERIAL DO LIKE OUR FACEBOOK PAGE httpwwwfacebookcomsscmentorsofficial

भस

4) Ass

गधा Correct Answer Zebra